Sie sind auf Seite 1von 1081

All ER 1963 Volume 2

[1963] 2 All ER 1

Government of Ceylon v Chandris


ADMINISTRATION OF JUSTICE; Arbitration

QUEENS BENCH DIVISION


MEGAW J
14, 15, 18, 25 FEBRUARY 1963

Arbitration Costs Fees of umpire and arbitrators Duty of umpire to tax or settle fees Duty of umpire to consider whether
fees fair and reasonable Arbitration Act, 1950 (14 Geo 6 c 27), s 18(1).

The High Court will not normally intervene in a motion to set aside or remit an award of an umpire on the ground that the umpire
misconducted himself by settling his own and the arbitrators remuneration at a figure which was wholly excessive unless (i) it is
satisfied that the fees can properly be described as extravagant, or (ii) it is apparent that the umpire has seriously misunderstood
his duty as regards the assessment of fees which he demands by his award, whether for himself or, through himself, for the
arbitrators, or for both. Absence of proper assistance from the umpire, in the way of adequate information to the court when the
fees prima facie appear to be out of relation to the work involved and are challenged as being grossly excessive, may be treated as
leading to the conclusion that the umpire has misunderstood his duty (see p 4, letter c, post).
It is desirable that an umpire or arbitrator, in fixing his fees, should do so by reference to considerations which he can put
forward and expect to justify as being reasonable, should a taxation be called for under s 19(1) a of the Arbitration Act, 1950.
Those considerations would normally involve, as at least two major factors, the period of time, whether days or hours, which he
has in fact reasonably devoted to the work which he has done, and the scale of charges for his time so computed (see p 5, letter a,
post).
________________________________________
a Section 19(1), so far as material, provides: If in any case an arbitrator or umpire refuses to deliver his award except on payment of the fees
demanded by him, the High Court may, on an application for the purpose, order that the arbitrator or umpire shall deliver the award to the
applicant on payment into court by the applicant of the fees demanded, and further that the fees demanded shall be taxed by the taxing
officer and that out of the money paid into court there shall be paid out to the arbitrator or umpire by way of fees such sum as may be found
reasonable on taxation

In his award the umpire in a commercial arbitration included an award of the costs of the award (as distinct from costs of the
reference), the aggregate sum so awarded including fees of the two arbitrators and a sum by way of his own remuneration. He
did not attempt to assess the value of the arbitrators services or the remuneration which they could fairly claim to be paid for the
work done, but included in the award the fees which they 1 asked him to include, believing this to be in accordance with the usual
practice in such arbitrations.

Held The umpires jurisdiction to fix the costs of the award derived from s 18(1) of the Arbitration Act, 1950 b, which conferred
power to tax or settle the costs, including arbitrators fees, and both tax and settle involved the application by the umpire of
his own independent mind and judgment to the fees demanded and the work done in order to be satisfied that the fees were fair
and reasonable, bearing in mind the interests of the party who would have to pay them as well as the legitimate interests of the
arbitrators (see p 6, letter e, post); the umpire had not, however, so taxed or settled the arbitrators fees or his own, and
accordingly was guilty of technical misconduct, and the award would be remitted for reconsideration of the provision as to costs
of the award (see p 7, letter f, post).
________________________________________
b Section 18(1), so far as material, provides: Unless a contrary intention is expressed therein, every arbitration agreement shall be deemed to
include a provision that the costs of the reference and award shall be in the discretion of the arbitrator or umpire, who may direct to and by
whom and in what manner those costs or any part thereof shall be paid, and may tax or settle the amount of costs to be so paid or any part
thereof

Per Curiam: if an umpire taxes and settles the costs of arbitrator-advocates, he should distinguish between that part of their
fees which is referable to their judicial functions before they have disagreed and that part which is referable to their functions as
advocates thereafter (see p 7, letter f, post).

Notes
As to costs of an arbitration, see 2 Halsburys Laws (3rd Edn) 47, para 103; and for cases on the subject, see 2 Digest (Repl) 545,
546, 808822.
As to remedies where an umpires or arbitrators remuneration is excessive, see 2 Halsburys Laws (3rd Edn) 49, para 108;
and for cases on the subject, see 2 Digest (Repl) 549, 550, 847853.
For the Arbitration Act, 1950, s 18, see 29 Halsburys Statutes (2nd Edn) 104.

Case referred to in judgment


Appleton v Norwich Union Fire Insurance Society Ltd (1922), 13 Lloyd LR 345.

Motion to set aside award


This was a motion by the respondent shipowner, Mrs Evgenia J Chandris, to set aside or remit an award of an umpire made in the
form of a Special Case under s 21(1)(b) of the Arbitration Act, 1950, in a dispute between the owner and the claimant charterers,
the Government of Ceylon. On 31 May 1953, the owner entered into a charterparty with the charterers whereby she chartered a
vessel to the charterers for a voyage from Burma to Ceylon to carry a cargo of rice in bags. The charterparty incorporated the
Centrocon charterparty arbitration provisions under which, in the event of arbitration, there were to be two arbitrators carrying on
business in London who should be members of the Baltic and engaged in the shipping and/or grain trades. Each side was to
nominate an arbitrator. During or after discharge of the cargo at Colombo, disputes arose. The owner appointed Mr J O Struthers
and the charterers Mr J Chesterman as arbitrators. The arbitrators, being unable to agree after a meeting of about two hours,
appointed Mr F J Norman as umpire. The case is reported only in relation to para 2 of the notice of motion, which alleged that
the umpire misconducted himself by settling his own and the arbitrators fees at a figure which was wholly excessive. The facts
are set out in the judgment.

C S Staughton and A D Coleman for the owner.


E F N Gratiaen QC, B C Sheen and N A Phillips for the charterers.

Cur adv vult


2

25 February 1963. The following judgment was delivered.

MEGAW J. I now come to para (2) of the notice of motion, which reads:

The umpire misconducted himself by settling his own and the arbitrators remuneration at a figure which was wholly
excessive.

The relevant parts of the award are as follows. The second paragraph of para 14 reads:

And I award that the owner does pay the costs of this award amounting to 612 3s. (of which 8 8s. is the cost of
hiring a room for the hearing of the arbitration and 183 15s. and 157 10s. are the fees of the two arbitrators respectively).
I further award that the owner does pay to the charterers their costs of the reference (to be taxed unless agreed).

Paragraph 18 reads:

And I direct that if the charterers shall in the first instance pay the costs of this award then the owner shall repay to the
charterers the amount so paid.

The charterers took up the award, paying the umpire the sum of 612 3s. If the award stands, both on this motion and in the
Special Case which may fall to be heard hereafter, the charterers will be entitled to recover the whole of that sum from the owner,
as well as their taxed costs of the reference.
The first question is whether the court has jurisdiction to consider an allegation of excessive remuneration. Both parties
agreed, through their counsel, that there is jurisdiction. That view is amply supported by the judgment of the Divisional Court in
Appleton v Norwich Union Fire Insurance Society Ltd ((1922), 13 Lloyd LR 345 at p 347). Salter J said:

I do not doubt that where an excessive charge is shown the court would act. Arbitrators are bound to act
conscientiously and with a proper regard to the interests of the parties, and if they dishonestly subordinate the interests of
the parties to their own, and take money from the parties in excess of what is just, I cannot myself doubt that that would be
misconduct.

The Arbitration Act, 1950, itself gives power, in certain circumstances, to have the remuneration of umpires and arbitrators
taxed. Section 18(1) gives to the arbitrator or umpire a discretion as to the costs of the reference and the award. When he awards
costs, he may either leave them to be taxed by the taxing machinery of the court in whole or in part, or he may himself tax or
settle the amount of costs to be so paid or any part thereof. A common practice is for the umpire to separate the costs of the
award (the arbitrators and umpires fees and any expenses which they have incurred) from the costs of the reference, which are
the costs incurred by the party to whom costs are awarded, other than the costs of the award. The costs of the award are taxed or
settled by the umpire. The costs of the reference are left to be taxed by the taxing machinery of the court, if necessary. The
umpire stipulates that the payment to him of the costs of the award shall be a condition of the delivery of the award to the party
requiring delivery of it, with a provision that, if the award be taken up by the party to whom the costs have been given, the other
party shall reimburse him. That was the practice which the umpire at least purported to follow in this case. Section 19(1) of the
Act of 1950 provides that, if the delivery of the award is made conditional on payment to the umpire, a party desiring delivery of
the award may apply to the High Court, which may order the delivery of the award to the applicant on his payment into court of
the full amount of the fees demanded. There is then a taxation of the fees, and the amount in court is paid out according to the
result of the taxation. The umpire can, of course, appear and be heard on such taxation. That is, no doubt, a useful provision, but
it leaves a gap. If the party who turns out to be the successful party has 3 taken up the award and paid the fees, the section does
not apply. The other party, who in fact has to bear the burden of the fees, cannot apply to the court. Thus, under the section as it
now stands, it is largely a matter of chance whether or not a party can make use of this statutory protection in respect of fees
which he claims to be excessive.
As I have said, both sides here agree that, in the present circumstances, the court has jurisdiction. The charterers say,
however, that not only is the onus on the owner to show that the fees demanded are excessive, but also that it is a very heavy
onus. They emphasise the words of Salter J, if they dishonestly subordinate the interests of the parties to their own; and they
contend that, before the court acts, it should be satisfied on the evidence before it that there has been such a dishonest
subordination. In my judgment, the court would not normally intervene in a motion such as this unless (i) it is satisfied that the
fees can properly be described as extravagant; or (ii) it is apparent that the umpire has seriously misunderstood his duty as
regards the assessment of fees which he demands by his award, whether for himself or, through himself, for the arbitrators, or for
both. Absence of proper assistance from the umpire, in the way of adequate information to the court when the fees prima facie
appear to be out of relation to the work involved and are challenged as being grossly excessive, may be treated as leading to the
conclusion that the umpire has misunderstood his duty. It has to be realised that a serious responsibility rests on one who is put in
a position to assess his own reward; particularly since his self-allotted remuneration may fall to be paid by someone who had no
part in his appointment; for example, in the case of the arbitrator appointed by the opposite party, or the umpire appointed, not by
either party, but by the two arbitrators. Of course, if the fee, or the scale of fees, to be charged is made known to both parties and
agreed by them before the appointment is accepted, there would be much less chance of embarrassment and complaint. However
that may be, this much is clear: an umpire or arbitrator, in fixing his fees, must consider not only his own interest, but also the
interest of the party or parties who will ultimately be required to pay those fees.
I was invited by counsel for the owner to take the view that, at least for arbitrations of this nature, where the arbitration
clause expressly provides that the arbitrators shall be Members of the Baltic and engaged in the shipping and/or grain trades,
the court should lay down some sort of appropriate scale, which would take account of the fact that the arbitrators whom the
clause contemplates are gentlemen normally engaged in business and accepting duties as arbitrators or umpires as a kind of
public service for the general good of the trade in which they are normally engaged, rather than as persons who look to
arbitrations as a substantial part of their livelihood. I am not sure that this is an altogether realistic view in relation to arbitrations
of this nature in the circumstances which exist today. It was suggested, further, that, whether or not that view be correct, in the
absence of special circumstances, which would have to be proved by those who have assessed their fees as arbitrators or umpires,
twenty-five guineas per day for the time actually occupied in the hearing should be regarded as the maximum reasonable figure
for this type of arbitration, and that anything at least substantially above that figure should normally be regarded as excessive or
calling for special explanation. It was pointed out that, in this case, the total time actually occupied by the umpire in the hearing
of the arguments was not more than about four or five hours, and that the total fees awarded by the umpire to himself and the
arbitrators, excluding out of pocket disbursements, which were not challenged, were over 500. It was submitted that the
umpires fee should not have exceeded seventy-five guineas and the arbitrators fees fifty guineas each.
I think that it would be undesirable for me to express any general view as to the appropriate scale of fees in arbitrations such
as this. The fees which are fair and appropriate depend on many factors, and generalisations would be 4 unsatisfactory and
dangerous. Nevertheless, I think that I can properly say this: it would be desirable that an umpire or arbitrator, in fixing his fees,
should do so by reference to considerations which he can put forward and expect to justify as being reasonable, should a taxation
be called for under s 19(1) of the Arbitration Act, 1950. Those considerations would normally involve, I apprehend, as at least
major factors, the period of time, whether days or hours, which he has in fact reasonably devoted to the work which he has done,
and the scale of charges for his time so computed. If the fees are challenged, their basis can be clearly stated and examined.
The umpires fee has been fixed by him at 172 16s. That is the total of 612 3s, less the arbitrators fees totalling 341 5s
and less 8 8s for the hire of a room and 89 14s legal charges incurred for, inter alia, the settling of the Special Case. Out of this
172 16s, the umpire paid a further twenty-five guineas to one of the arbitrators, Mr Struthers. That arose in this odd and, I am
bound to say, unsatisfactory way. Mr Chesterman, the other of the two arbitrators, originally requested that the umpire should
include for him a fee of 150 guineas. So did Mr Struthers. When a Special Case was ordered, Mr Chesterman thought that that
entitled him to additional remuneration, and he asked the umpire to include a further twenty-five guineas. Mr Struthers made no
such request. In the award, the umpire stated the arbitrators fees as 183 15s and 157 10s. He sent a cheque for these sums to
Mr Chesterman and Mr Struthers respectively. Mr Struthers replied that, on the first page of the award, his name appeared before
Mr Chestermans; therefore, he claimed, the sum first mentioned in para 14 (the arbitrators names not being there mentioned)
applied to him. The umpire replied, telling Mr Struthers of Mr Chestermans subsequent request for an additional twenty-five
guineas; but the umpire at the same time sent Mr Struthers a cheque for a further twenty-five guineas. It is fair to make it clear
that the umpire did not seek to make this further twenty-five guineas an additional charge against the parties. He paid it,
presumably, out of his own fee of 172 16s. For the purposes of this motion, however, the umpires remuneration must be treated
as being 172 16s, and that of the arbitrators as totalling 341 5s. The owner contends that, prima facie, the umpires fee of 172
16s is excessive in relation to an arbitration where the whole hearing lasted not much more than four hours at most, and where,
moreover, the task of preparation of the draft award was handed over by the umpire to solicitors, whose remuneration the owner
does not challenge. The owner says that, if there are special factors which justify such a fee for so apparently limited an amount
of work, those factors are within the knowledge of the umpire, and it is for him to declare them with sufficient precision if the
prima facie case of excessive fees is to be rebutted. The umpire, in his affidavit put in evidence by the charterers, does indeed
mention various matters, such as that he had to read carefully through the documents put in evidence; and the transcript of the
arguments on two occasions; that he had to have many consultations with the solicitors whom he instructed to settle the draft
award; and that he had extensive correspondence with the parties representatives. It would have been better if the umpire had
seen fit to specify more fully the time which was reasonably occupied by him on the work involved in these aspects, together
with any other special factors. I should, nevertheless, have hesitated long before holding that the umpires fee was extravagant,
and I should have been disposed, had it not been for another matter, to have adjourned the motion in order to give the umpire a
further opportunity, if he saw fit, to supplement the information which he has given. I shall come back to that other matter after I
have considered the question of the fees awarded by the umpire to the arbitrators.
As regards those fees, the evidence before me indicates no more than this: that the two arbitrators, having received and no
doubt read the documents supplied by their respective clients, had a two hour meeting on 2 June 1961, when they discussed these
documents and agreed to disagree. Mr Struthers 5subsequent work seems to have involved preparing two written submissions,
attending to ask for an adjournment on 5 July and attending the hearing on 26 July when counsel presented the owners case. Mr
Chesterman, in addition, prepared and presented the charterers case on 26 July and six months later prepared suggested findings
of fact. The umpire has not supplemented that information, which I have derived from other affidavits. The reason for this lack
of further information is that the umpire, as he makes clear in his affidavit, did not regard it as in any way his business to consider
or assess the value of the services of the arbitrators or the remuneration which they could fairly claim to be paid. He makes it
clear in his affidavit that he, quite simply, included in the award the amounts which they asked him to include, because they asked
for them. This, says the umpire, is in accordance with the usual practice. He says:

It is not the practice for an umpire to settle or to tax the fees of the arbitrators, and I did not settle or tax the fees of Mr.
Chesterman or Mr. Struthers in this case.

I have no doubt that the umpire genuinely believed that this was an accepted and proper practice, and that that is why he followed
it. If, however, that is the usual practice, it is, in my judgment, a practice which is wrong and unjustifiable and which should not
be followed. It is contrary to the meaning and intent of s 18(1) of the Arbitration Act, 1950. It is by virtue of that subsection that
an umpire has power to fix the amount of any costs to be awarded, including the arbitrators fees. The power is to tax or settle.
The umpire, as he himself says, did not purport to tax or settle the arbitrators fees. Therefore, his inclusion of the 341 5s in
respect of them is ultra vires. Both tax and settle involve the application by the umpire of his own independent mind and
judgment to the fees demanded and the work done in order to be satisfied that the fees are fair and reasonable, bearing in mind
the interests of the party who will have to pay them, as well as the legitimate interests of the arbitrators. It is, of course, open to
the umpire, if he does not wish to tax or settle the fees, because he finds it embarrassing or because he feels that he has not
sufficient information, not to tax or settle them under s 18(1), but to leave them to be dealt with outside the costs of the award, by
way of taxation or otherwise.
There may be cases where the umpire can satisfy himself that the fees put forward by the arbitrators have in fact been fixed
and agreed between the parties, whether by the arbitrators having express authority from the respective parties so to agree, or
otherwise. In such a case, an umpire need not go behind the agreement or investigate the agreed fees. There is no suggestion, in
the umpiress affidavit or elsewhere, that there was any such agreement, or authority on the part of the arbitrators to agree so as to
bind the parties, in this case. Indeed, Mr Chestermans twenty-five guineas for the Special Case, not known to Mr Struthers until
after the award was published, would destroy the possibility of any such agreement having existed here. I mention this only
because it was argued for the charterers that the umpire here, though he does not himself say so, must have thought that the
arbitrators, with authority from their respective appointors, had agreed one anothers fees. That being so, it follows that the
umpire has seriously misunderstood his duty as regards the assessment of the arbitrators fees. This amounts to what is
technically misconduct and the inclusion of those fees cannot stand as a part of the award.
I now have to return to the umpires own fees. The umpires attitude shows that, with regard to the arbitrators fees, he did
not understand that his duty was to consider them from the point of view of the interest of the party who would have to pay them,
as well as of the arbitrators themselves. Therefore, as it seems to me, there is at least the possibility that he did not, in the proper
sense, tax or settle his own fees. He may or may not have taken into account the interest of the party who would have to pay the
fees, in carrying out this 6 invidious and delicate task of assessing his own remuneration. It is impossible to be confident that he
did so. In the circumstances, I do not think that it would be right to allow that assessment to stand. The owner submitted that, if
the provision as to the fees in the award should not be allowed to stand, the whole award should be set aside. I do not think that
that is the proper course. Although the umpire has taken a wrong, and unfortunate, view as to the assessment of fees, I see no
reason to think that the fair inference is that he did not approach the decision of the substantive issues in the case in a proper
judicial spirit. Whether he decided the questions of law rightly or wrongly is a matter which remains to be determined hereafter,
and nothing that I have said in my judgment on the motion has any bearing on that question. In my judgment, the award should
be remitted to the umpire for the reconsideration of the fees awarded in the light of my judgment on this motion.
I do not think that I have power to direct the umpire what he should find on such reconsideration. But I tink I may properly
say that, in my view, it would obviously be gravely embarrassing for the umpire now himself to settle or tax his own fees or those
of the arbitrators. I assume, however, that his only desire, on reconsideration, will be to act fairly and properly. If I had taken
any other view as to his attitude, I should have set aside the whole award. The umpire will, I am confident, realise that there is
now open to him a course which will avoid such embarrassment, and which will ensure that there is an impartial review, such as
would not now be possible for him himself to be seen to have applied. That course is to maintain, as costs of the award properly
taxed and settled by him, the two unchallenged items of his out of pocket expenses, namely, 8 8s for hire of a room and 89 14s
for his legal expenses; and to direct that all the remaining costs of the award, as well as the costs of the reference, shall be taxed
unless agreed. In any such taxation, the umpire and the arbitrators would be entitled, as the umpire would have been in a taxation
under s 19 of the Arbitration Act, 1950, to appear and to be heard, in respect of the umpires and the arbitrators fees, on any
taxation or review of taxation.
I, therefore, remit the award for reconsideration of the provisions as to costs of the award contained in the second paragraph
of para 14 of the award. I would only add this. In my view, if in future an umpire taxes and settles the costs of arbitrator-
advocates, he should distinguish between that part of their fees which is referable to their judicial functions before they have
disagreed and that part which is referable to their functions as advocates thereafter. Otherwise, the position on any taxation
which may be required of the costs of the reference may be difficult or impossible, since the taxing officer may have no means of
distinguishing between the two in his taxing of the costs of the reference.

Order accordingly.

Solicitors: Richards, Butler & Co (for the owner); T L Wilson & Co (for the charterers).

Mary Colton Barrister.


7
[1963] 2 All ER 8

Baldwin v Worsman
CRIMINAL; Road Traffic

QUEENS BENCH DIVISION


LORD PARKER CJ, ASHWORTH AND WINN JJ
6 MARCH 1963

Road Traffic Licence Driving licence Groups of vehicles for which driving licence granted Three-wheeled motor vehicle
constructed with means of reversing which not immediately usable by reason of reversing equipment being blanked off Whether
vehicle within group A or group G Motor Vehicles (Driving Licences) Regulations, 1950 (SI 1950 No 333), Sch 2.

The respondent held a driving licence to drive vehicles in group G of Sch 2 to the Motor Vehicles (Driving Licences)
Regulations, 1950. He also held a provisional driving licence to drive vehicles in group A of that schedule. He drove a three-
wheeled motor vehicle constructed to carry more than one person and which had a reversing gear, but the reversing gear could
not be used because it was blanked off, ie, a piece of metal had been put in such a position that the gear lever would not go into
the reverse position. Group G of Sch 2 to the 1950 Regulations covered motor-bicycles or tricycles not equipped with means for
reversing, while group A covered, inter alia, motor cars or motor tricycles equipped with means for reversing. On appeal from
dismissal of an information charging failure to comply with a condition of the respondents provisional licence, viz, a charge
based on the motor vehicles falling within group G,

Held A vehicle constructed with the means of reversing did not cease to be a motor vehicle equipped with means for reversing
within group A of Sch 2 to the 1950 Regulations merely because the means for reversing was not immediately usable by reason
of it being blanked off (see p 9, letters f and h, post); accordingly the case would be remitted to the justices with a direction to
convict.
Appeal allowed.

Notes
As to the groups of vehicles for which driving licences may be granted, see 33 Halsburys Laws (3rd Edn) 458, para 780, note (p).

Case Stated
This was a Case Stated by justices for the City of Bradford in respect of their adjudication as a magistrates court sitting at
Bradford on 21 September 1962. On 7 September 1962, the appellant, Cyril Baldwin, preferred two informations against the
respondent, Fred Worsman, that, on 18 August 1962, in the City of Bradford, being a person to whom a provisional licence had
been granted and not having passed the appropriate test, he failed to comply with the condition subject to which the provisional
licence was granted, in that he used a motor car constructed to carry more than one person on a road called Hammerton Street (a)
when not under the supervision of a person who was present in the motor car with him and who held a licence other than a
provisional licence authorising him to drive a vehicle of the same class as the motor car and who had been the holder of a licence
for at least two years or had passed a test under s 6 of the Road Traffic Act, 1960, and (b) when, not clearly displaying in a
conspicuous manner on the front and on the back of the car a distinguishing mark in the form set out in Sch 7 to the Motor
Vehicles (Driving Licences) Regulations, 1950, contrary to reg 16(3) of the regulations and s 102(3) of the Road Traffic Act,
1960.
The following facts were found. On 18 August 1962, the respondent was driving a Reliant three-wheeled motor vehicle on
Hammerton Street. The vehicle was constructed to carry more than one person, and was not at the material time displaying a
distinguishing mark in the form set out in Sch 7 to the 1950 Regulations. The respondent was in possession of a provisional
licence to drive vehicles of group A of the 1950 Regulations, including a motor tricycle equipped with means 8 for reversing. He
was also in possession of a licence to drive vehicles of group G of the regulations, including a motor tricycle not equipped with
means for reversing. The respondent had not passed the appropriate test for vehicles of group A, and was not under the
supervision of any person who was present in the vehicle with him. The vehicle was constructed with a means for reversing,
namely, a reverse gear, which was blanked off, in that a piece of metal had been inserted into a position by the manufacturer in
its construction which rendered the gear unusable. The gear could be made usable by removing the piece of metal, the time
which would be taken to remove the metal being at least half an hour.
It was contended by the appellant that the vehicle was equipped with means for reversing and, therefore, the vehicle was a
vehicle of group A of the 1950 Regulations. It was contended by the respondent that the vehicle was not equipped with means for
reversing and, therefore, the vehicle was a vehicle of group G of the regulations.
The justices dismissed both informations, and the appellant now appealed.
The case noted belowa was cited during the argument.
________________________________________
a Lawrence v Howlett [1952] 2 All ER 74, DC, WN 308, 1 TLR 1476

J A Cotton for the appellant.


The respondent did not appear and was not represented.

6 March 1963. The following judgments were delivered.

LORD PARKER CJ stated the facts, and continued. The respondent was in fact the holder of a licence to drive vehicles in
group G, and, if this vehicle that he was driving was within group G, he committed no offence. On the other hand, if it was
within group A, he was merely the holder of a provisional licence, and he was in breach in the two respects alleged. The sole
question, therefore, is into which group the vehicle fell. In fact, this was a Reliant three-wheeled motor vehicle which had a
reversing gear, but the reversing gear at the time could not be used, because a piece of metal had been put in such a position that
the gear lever would not go into the reverse position; it was, as it is called, blanked off.
The groups of vehicles are set out in Sch 2 to the Motor Vehicles (Driving Licences) Regulations, 1950, and group A covers,
so far as is material here, a motor car or motor tricycle equipped with means for reversing. Group G on the other hand covers a
Motor-bicycle (with or without side-car) or tricycle not equipped with means for reversing. The short point here on which we
are told that magistrates courts are coming to different conclusions, is whether a vehicle of this sort, with the reversing gear
blanked off so that it cannot be used, is or is not equipped with means for reversing. For my part, I have come to the clear
conclusion that if a vehicle is constructed with the means of reversing and is found on a road with that equipment intact, it does
not cease to be a motor vehicle equipped with means for reversing merely because it is not immediately usable by reason of its
being blanked off.
In my judgment, the magistrates came to a wrong decision; this appeal should be allowed and the Case sent back to them
with a direction to convict.

ASHWORTH J. I agree.

WINN J. I agree.

Appeal allowed. Case remitted.

Solicitors: Wilkinson, Howlett & Moorhouse agents for Town clerk, Bradford (for the appellant).

Shireen Irani Barrister.


9
[1963] 2 All ER 10

Dove v Dove
FAMILY; Divorce

COURT OF APPEAL
ORMEROD, DONOVAN AND RUSSELL LJJ
1, 4 MARCH 1963

Divorce Decree nisi Substitution of decree of judicial separation Discretion Factors to be considered Respondents right
to apply in due course to make decree nisi absolute Appropriate procedure Matrimonial Causes Act, 1950 (14 Geo 6 c 25), s
12(3).

In determining whether to substitute a decree of judicial separation for a decree nisi of divorce the court should take into
consideration, in a case where three months from the earliest time when application could be made for a decree absolute has not
yet elapsed, that the substitution of the order sought in place of the decree nisi may nullify the right of the party against whom the
decree nisi was obtained to apply under s 12(3) of the Matrimonial Causes Act, 1950, for a decree absolute (see p 11, letter h, and
p 12, letters e and i, post); and (per Donovan and Russell LJJ) the proper practice, if the respondent wishes to apply under s 12(3),
is to defer the hearing of the application to substitute a decree for judicial separation until the time has elapsed which will enable
the other spouse to apply for a decree absolute, and then to determine both applications together (see p 12, letters d and h, post).

Notes
As to the substitution of a decree of judicial separation for a decree nisi of divorce, see 12 Halsburys Laws (3rd Edn) 322, para
650; and a case on the subject, see 27 Digest (Repl) 687, 6566.
For the Matrimonial Causes Act, 1950, s 12(3), see 29 Halsburys Statutes (2nd Edn) 400.

Cases referred to in judgments


Davies v Davies [1955] 3 All ER 589, [1956] P 212, [1955] 3 WLR 84, 3rd Digest Supp.
Jeffrey v Jeffrey [1950] 2 All ER 449, [1951] P 32, 27 Digest (Repl) 687, 6566.

Appeal
This was an appeal by a husband, who was respondent to a petition by his wife for divorce, against an order of Payne J, made on
the wifes application, substituting a decree of judicial separation for the decree nisi of divorce already granted on her petition.
The facts are stated in the judgment of Ormerod LJ.

D Armstead Fairweather for the husband.


J A Kirpal for the wife.

4 March 1963. The following judgments were delivered.

ORMEROD LJ. This is a respondent husbands appeal from a decision of Payne J given on 28 June 1962, in these
circumstances: the parties were married and eventually separated, having had six children as the result of the marriage. On the
wifes petition a decree nisi of divorce was pronounced on 2 February 1962. That was on the ground of adultery. The wife in due
course made an application to the court for the decree nisi to be set aside and for an order for judicial separation to be substituted.
The learned judge, after giving the matter careful consideration, decided to make the order asked for. It is sufficient to say,
although this is in no way material to the matter before us on this appeal, that the grounds of the wifes application were, in the
first place, that, owing to the difficulty of finding further accommodation for herself and her six children, she felt it wiser in the
circumstances to remain in her husbands house and that it would be better if she had an order for judicial separation rather than
divorce. The second ground was that she felt it would be in the best interests of the children that their father and mother should
remain married at least until they were old enough to form their own views; and thirdly, although I think that she 10 has not put
this reason very high, that on moral and religious grounds she felt that she would not care to be a party to divorce proceedings.
Be that as it may, the application came before Payne J on 30 May 1962; he adjourned it for twenty-one days, and it came on
finally on 28 June when he considered the matter and made the order sought. Counsel on behalf of the husband first raises the
point that there was no jurisdiction in the court to make this order, having regard to s 12(3) of the Matrimonial Causes Act, 1950.
The effect of that subsection, which is a re-enactment of a similar provision a in the Matrimonial Causes Act, 1937, is that if the
spouse obtaining the decree nisi does not make an application for it to be made absolute three months after the trial, when he or
she could first apply for it to be made absolute, then the other spouse can apply after a further period of three months, and, if the
circumstances warrant, obtain a decree absolute. Counsel for the husbands submission in the first place is that, accepting that
there was jurisdiction in the court to substitute for an order of decree nisi an order for judicial separation, or vice versa, that had
been ousted by inference as an effect of s 12(3). The learned judge did not accept that contention; he expressed the view that if
the legislature had intended to revoke this jurisdiction, it would have been done in a clearer way than by inference from the
subsection; and with that I am fully in agreement. I see no reason, if this jurisdiction existed, for saying that s 12(3), although it
may confer a right on the spouse when an order for decree nisi is made, means that there is no jurisdiction in the court to rescind
that order and substitute another; but that jurisdiction is, I think, not in question. There is no doubt, and neither party has said
anything to the contrary, that this jurisdiction has existed and that the practice has grown up whereby the court should exercise it,
and indeed the principles on which it is based are set out and discussed in Rayden on Divorce b with reference to the helpful
authorities.
________________________________________
a Section 9 of the Act of 1937, which added a new sub-s (3) to s 183 of the Supreme Court of Judicature (Consolidation) Act, 1925
b Rayden on Divorce (8th Edn), 1960, pp 321, 322, para 54, text and notes (a)-(d)

However, there is a further submission which was made by counsel for the husband which does seem to be material, and that
is that the jurisdiction which was in the court prior to the passing of the Matrimonial Causes Act, 1937, c was, of course, the
exercise of discretiona discretion to be exercised on established judicial principles. The submission is that a new factor had
been introduced by s 12(3) which the learned judge in exercising his discretion should take into account. It may be, of course,
that, as the result of the decree nisi, the other spouse has entered into some form of arrangement with some third party. It may be,
as I believe is the case here, that a child is likely to be born in the comparatively near future which would be illegitimate if the
husband is not in a position to marry the prospective mother of the child. Those and many others are matters which the learned
judge may choose to take into consideration, and it seems to me material that he should take into consideration the fact that if
there is a substitution of the one order for the other, it may well be that the circumstances are such that s 12(3) will in effect be
nullified. That is a contention which I am bound to say appeals to me. I think that it is clear that the learned judge did not take
that matter into consideration in coming to the decision. In my view the effect of s 12(3) is to introduce a new factor for
consideration, that new factor being that the guilty spouse has rights which have accrued as a result of the decree nisi being made,
and it is a factor to be taken into account by the learned judge in exercising his discretion. In those circumstances I would allow
this appeal to the extent that I would order that the whole matter be referred back and that there be a new hearing, so that whoever
hears the case shall have an opportunity of taking into account in the exercise of his discretion the matter to which I have
referred.
11
________________________________________
c Ie, under s 183(2) of the Act of 1925; see footnote (1) supra

I should add this: counsel for the husbands further contention was that on the proper construction of s 12(3), in the
circumstances of this case, as the three months had gone by and the wife was in a position to ask for an order for the decree to be
made absolute and had not done so, the proper construction was that she could not ask for a new order because of the effect of s
12(3). I do not think that counsel for the husband urged that submission with any great sense of conviction and all that I need say
is that I reject it.

DONOVAN LJ. I agree that s 12(3) of the Matrimonial Causes Act, 1950, re-enacting the provision introduced by s 9 of the
Matrimonial Causes Act, 1937, does not deprive the court of the jurisdiction, which it has exercised for many years, of altering a
decree nisi into a decree of judicial separation on the application of the person who obtained the decree nisi; but the section does
introduce this new factor, namely, that the respondent may be prejudicially affected if such an application be granted. He or she
will be deprived, in effect, of the value of his or her right to apply after the necessary six months for the decree to be made
absolute. This may well be serious if, for example, children are expected from some new relationship into which the respondent
may have entered relying on his or her prospective divorce. The right way to deal with this situation is to defer such application
to alter the decree nisi to a date when the respondents right to apply for the decree to be made absolute will have accrued. If the
respondent makes such an application, then both applications can be dealt with together in the light of the current facts. This was
not done in the present case. The wifes application was allowed on 28 June 1962, whereas the husbands right to ask for the
decree to be made absolute did not accrue until 6 August 1962. The eventual result in this case may be no different; I do not
know; but the husband is, I think, entitled to be allowed to make his application, and I accordingly agree with the order proposed.

RUSSELL LJ. It is undoubtedly a lawful practice of long standing to permit the substitution of a decree of judicial separation
for a decree nisi of divorce; but when, by s 9 of the Matrimonial Causes Act, 1937, a right was conferred for the first time on the
respondent, after a period (which is now six months) has elapsed from the decree nisi, to apply for it to be made absolute, it
seems to me that a new factor was introduced relative to the decision whether or not to substitute judicial separation for decree
nisi of divorce. I consider that the practice requires to be regulated so as to permit proper weight to be given to that new factor
new at least in 1937. If on such an application by the petitioner the respondent raises no objection, as was the case in Davies v
Davies, then the order can be made in favour of the petitioner even if the six months period has not elapsed. If the six months
have elapsed, then the respondent is in a position to make his, so to speak, cross-application under s 12(3) and the two
applications can be heard together, which is what happened in Jeffrey v Jeffrey. If on the petitioners application, the respondent
indicates that he will wish to apply under s 12(3) when the six months have elapsed (as was the case here), in my view the
petitioners application should be adjourned for a period sufficiently long to enable that to be done. As I read the judgment in this
case, the learned judge took the view that in dealing with the wifes application, no consideration at all should be given to s 12(3),
there being no application before the court under that subsection. This in my view was a wrong approach and the judges order
ought to be set aside. On the merits of the case I say nothing at all. The question will be considered on up-to-date evidence when
the wifes renewed application and the husband respondents application which he proposes to make under s 12(3) come to be
heard together.
12

Appeal allowed; case sent back for re-hearing. Order for judicial separation set aside; decree nisi of divorce restored as from
the original date; respondent husband to have an opportunity to apply for decree nisi of divorce to be made absolute.

Solicitors: Johnson, Jecks & Landons agents for Landons, Brentwood (for the husband); Nehra, Emerson, Naylor & Co (for the
wife).

Henry Summerfield Esq Barrister.


[1963] 2 All ER 13

Lovelock v Margo
LANDLORD AND TENANT; Leases

COURT OF APPEAL
LORD DENNING MR, DANCKWERTS AND DAVIES LJJ
27, 28 FEBRUARY 1963

Landlord and Tenant Lease Forfeiture Relief Breach of covenant Peaceable re-entry by landlord.

Landlord and Tenant Lease Assignment Covenant against assignment without consent Landlord giving unjustified reason
for withholding consent Whether landlord can later justify withholding consent on other grounds.

In May, 1960, the landlord let to the tenant a lock-up shop under a lease of twenty-one years at a yearly rent of 260 payable
weekly. The lease contained a proviso for re-entry in case of non-payment of rent, and a covenant against assignment without the
consent of the landlord. In 1961 the tenant asked the landlord for her consent to assign the lease and produced satisfactory
references of the proposed assignee. The landlord refused consent on the ground that the area of the premises to be transferred
was uncertain. In August, 1961, when 25 rent was due, the landlord peaceably re-entered the premises and excluded the tenant
therefrom. The tenant claimed relief against forfeiture and a declaration that the landlord had unreasonably withheld her consent
to assignment. On appeal by the landlord from an order of the county court judge granting relief against forfeiture and the
declaration claimed, it was contended on behalf of the landlord that there were other points, not taken by the landlord herself,
why it might be reasonable to withhold consent.

Held The order would be upheld because


(i) the right to relief against forfeiture extended to a case where there had been re-entry without action (see p 14, letter f, and
p 15, letters f and g, post).
Howard v Fanshawe ([1895] 2 Ch 581) applied.
(ii) the question whether consent was unreasonably withheld was not an objective question and could not be considered
without regard to the state of mind of the landlord herself as to her reasons for refusing consent (see p 15, letter c, post).
Appeal dismissed.

Notes
As to relief from forfeiture for non-payment of rent, see 23 Halsburys Laws (3rd Edn) 681, 682, para 1409; and for cases on the
subject, see 31 Digest (Repl) 535, 536, 65926608.
As to unreasonable withholding of consent to assign, see 23 Halsburys Laws (3rd Edn) 633, 634, para 1338; and for cases
on the subject, see 31 Digest (Repl) 424427, 55155536.

Case referred to in judgments


Howard v Fanshawe [1895] 2 Ch 581, 64 LJCh 666, 73 LT 77, 31 Digest (Repl) 535, 6599.

Appeal
This was an appeal by the landlord from an order of His Honour Judge Ruttle at Lambeth County Court, dated 17 July 1962,
granting the tenant the declaration that the landlord had unreasonably refused her consent to an assignment of 13 premises at
113A, Kennington Road, Lambeth, of which the tenant was lessee under a lease dated 18 May 1960, and that the tenant be
granted relief against forfeiture of the lease for non-payment of rent on terms that the tenant paid the arrears of rent and half the
landlords costs. The facts are set out in the judgment of Lord Denning MR.

R Shulman for the landlord.


Ashley Bramall for the tenant.

28 February 1963. The following judgments were delivered.


LORD DENNING MR. On 18 May 1960, the landlord let to the tenant a lock-up shop, No 113A, Kennington Road, under a
lease of twenty-one years from 7 March 1960, at a rent of 260 a year payable by weekly payments. There was a proviso for re-
entry in case the rent was not paid. In 1961 25 rent was not paid. I will not go into the reasons for it, but the landlord, on 28
August 1961, made what was said to be a peaceable re-entry; she got in and changed the locks and excluded the tenant from the
premises, claiming that it was done under a right of re-entry for non-payment of rent. The tenant, after a time, brought an action
in the county court claiming, amongst other things, relief against forfeiture, and the judge has given it.
A point was raised before us as a point of law to the effect that the court has not got power to grant relief from forfeiture
when there has been peaceable re-entry. It was said that the court can only grant relief when there has been an action for
forfeiture or an action for ejectment or for possession. In answer to that argument, it is sufficient to say that Stirling J, in Howard
v Fanshawe, pointed out that the jurisdiction

is not confined to cases where the lessor has recovered possession by legal process, but extends to cases where the
lessor has recovered peaceable possession without the assistance of any court

that being a jurisdiction not dependent on statute but the ancient jurisdiction of the Court of Chancery. Indeed, that is borne out
by the County Courts Act, 1959, s 191(3), which is framed on the very basis that that is the law; that the right to give relief
extends to a case where there has been re-entry without action. In my judgment, that point is clearly bad. The judge granted
relief and there was clear jurisdiction to grant it.
The second point is quite different. In this case, the tenant wanted to assign the lease of this fried fish shop quite early in the
term. On 8 August 1961, a letter was written by his solicitors saying that he had exchanged contracts with the proposed
purchaser, a Mr Jacobs, and enclosing references and asking for a licence to assign. The lease contained the usual covenant not to
assign without licence, and, of course, there was imported in it that licence was not to be unreasonably withheld. The matter was
held up because the landlord was abroad. Meanwhile the solicitor for the landlord went into the matter and said that he was
satisfied about it and would recommend the licence to the landlord, when she came home that week-end. When the landlord did
come home, she, or rather her husband, Mr Margo, stated on the premises that he would not give leave to assign. I need only
read the solicitors letter on behalf of the landlord on 24 August:

I confirm that my client has instructed me not to grant the licence to assign the premises requested as she is not
satisfied with the area proposed to be used in connexion with the shop.

That was the only reason given for the refusal to assign. In the action, the tenant claimed a declaration that the landlord had
unreasonably refused her consent to the assignment. The judge went into the matter and considered this question of the extent of
the area. He held that the point taken by the landlord was not a bona fide one. The reason for refusing the licence, he said, was
invalid. Even if there was a bona fide dispute between the parties as to the extent of the 14 premises held by the tenant (which he
did not accept), the tenant was entitled to assign (subject to his references, which were in fact satisfactory), whatever his interest
was under the lease. It seems to me that that reasoning by the judge is correct and unimpeachable. The landlord took a wrong
point and gave an unjustified reason for refusing the licence to assign.
Counsel for the landlord has argued that this is an objective question; that he desires to raise before us other points, not taken
by the landlord herself, why it might be reasonable to withhold consent. It was said that a reasonable time was not given for
consideration. The landlord never said that she needed further time. Then he said that third parties were involved; it would have
an effect on the next door property and the financial position of the tenant and matters of that kindmatters which were not
hinted at by the landlord herself or by her solicitors, and, indeed, there was no hint of it in the evidence. I am quite clearly of
opinion that it is not right to say that this is an objective question, as counsel said. This matter cannot be considered without
regard to the state of mind of the landlord herself as to her reasons for refusing consent. How otherwise can a lessee hope to see
whether he can assign unless he knows the landlords reasons for objection? It seems to me that, when the tenant produced an
assignee, a responsible person with good references to which no objection could be taken, the landlord had no good ground for
objecting. She put forward a bad ground and she has not shown any other before the court, and, in those circumstances, the judge
was perfectly justified in holding that consent had been unreasonably withheld and making a declaration accordingly.
As to the terms of the relief granted, no point is taken before us. The tenant had to pay the rent in arrear, but he only had to
pay half the costs and, on that being paid, he got relief. It seems to me that the judge was entitled to do that. We were told that,
notwithstanding the existence of this action and almost in defiance of it, a few days before the action came on for hearing, the
landlord let these premises to someone else, having got into the premises, as I have said. So much the worse for her. She takes
the risk if she lets them to others in face of a pending law case. If that is what the landlord did, the burden must rest on her. In
the circumstances, I see no fault in the judges judgment and I would dismiss the appeal.

DANCKWERTS LJ. I agree. I only want to add that I am completely satisfied that the decision of Stirling J, in Howard v
Fanshawe correctly stated the law and that that law has not been altered by the effect of the Judicature Acts and, in particular, the
provisions of s 46 of the Supreme Court of Judicature (Consolidation) Act, 1925.

DAVIES LJ. I also agree. I think that this appeal is perfectly hopeless.

Appeal dismissed.

Solicitors: Geoffrey Summers (for the landlord); Henry I Sidney & Co (for the tenant).

F Guttman Esq Barrister.


15
[1963] 2 All ER 16

Francis Day & Hunter Ltd and another v Bron (trading as Delmar Publishing
Co) and another
INTELLECTUAL PROPERTY; Copyright

COURT OF APPEAL
WILLMER, UPJOHN AND DIPLOCK LJJ
20, 21, 22, 25 FEBRUARY 1963
Copyright Infringement Reproduction Essential elements similarity and causal connexion Objective and subjective
questions, but questions of fact No irrebuttable presumption of causal connexion Subconscious copying as possible
infringement Copyright Act, 1956 (4 & 5 Eliz 2 c 74), s 2(5).

The opening bar of a song, In a Little Spanish Town, composed in 1926, was a common-place series of quavers found in
previous musical compositions and was very similar to though not identical with the opening bar of a new song, Why,
composed in 1959. This opening phrase was developed over the remainder of the first eight bars by the use of the same devices
or tricks of composition in both the songs, producing a definite or considerable similarity between them. In a Little Spanish
Town had been extensively exploited in the United States and elsewhere by the publication of sheet music, by the distribution of
records and by broadcasting. The composer of Why was a man of thirty-three who had lived most of his life in the United
States and had played various instruments in dance bands; his evidence was accepted that he had not seen or studied or in his
recollection played the music of In a Little Spanish Town nor had ever to his knowledge heard it, although he admitted that he
might have heard it at a younger age. In an action for breach of copyright in In a Little Spanish Town brought against the
publishers of Why, the trial judge found that he had insufficient factual material in the similarity of the works and in the
inference that the composer of Why had at some time heard In a Little Spanish Town to draw an inference, in the absence of
direct evidence and in face of the composers denial, that he had had sufficient knowledge or memory of In a Little Spanish
Town to have copied it without knowing that he was doing so, rather than that the similarity arose from coincidence, and
accordingly the trial judge held infringement of copyright was not established. On appeal,

Held The decision of the trial judge should stand, for the following reasons
(i) in order to constitute reproduction within s 2(5) of the Copyright Act, 1956, there must be both sufficient objective
similarity between the two works and some causal connexion between the work infringed and the infringers work, viz, it must be
proper to infer derivation of the latter from the former (see p 22, letters a and b, p 24, letter d, p 25, letter b, and p 27, letter i, to p
28, letter a, post).
(ii) the existence of both these elements was a question of fact, the first element (similarity) being an objective issue, and the
second element (causal connexion) being a subjective question (see, eg, p 24, letter g, post) but not to be presumed as a matter of
law merely from proof of possible access by the alleged infringer to the copyright work (see p 29, letter g, and p 23, letter i, to p
24, letter a, post).
(iii) in the present case there was no ground for interfering with the trial judges conclusion of fact (see p 23, letter e, p 26,
letter f, and p 30, letter c, post).
Per Willmer and Diplock LJJ: subconscious copying can constitute an infringement of copyright, for, in order to constitute
infringement, intention or knowledge is not necessary (see p 21, letter g, and p 28, letters c and e, post; and cf p 26, letter i, to p
27, letter a, post); and (per Willmer LJ), if subconscious copying is to be established, there must be proof or strong inference of
de facto familiarity with the work alleged to be copied (see p 21, letter e, post).
16
Dictum of Luxmoore J, in G Ricordi & Co (London) Ltd v Clayton and Waller, Ltd (Macgillivrays Copyright Cases (1928
1935) at p 162); dictum of Judge Learned Hand in Fred Fisher, Inc v Dillingham, ((1924), 298 Fed Rep at p 145), and Edwards
and Deutsch Lithographing Co v Boorman ((1926), 15 Fed Rep (2nd series) 35) considered.
Per Upjohn LJ: where there is evidence from the music itself that there is real practical possibility of independent
composition by the defendant, it requires quite strong evidence to support the view that there has been unconscious copying (see
p 25, letter i, post).
Appeal dismissed.

Notes
As to the meaning of reproduction infringing copyright, see 8 Halsburys Laws (3rd Edn) 426, 427, para 776; and for cases on the
subject, see 13 Digest (Repl) 104121, 451616.
For the Copyright Act, 1956, s 2(5), see 36 Halsburys Statutes (2nd Edn) 73.

Cases referred to in judgments


Austin v Columbia Gramophone Co Ltd (1923), Macgillivrays Copyright Cases, 19171923, p 398, 13 Digest (Repl) 69, 146.
DAlamaine v Boosey (1835), 1 Y & C 288, 4 LJEx 21, 13 Digest (Repl) 115, 564.
Edwards and Deutsch Lithographing Co v Boorman and Others (1926), 15 Fed Rep (2nd series) 35.
Fisher (Fred), Inc v Dillingham and Others (1924), 298 Fed Rep 145.
Hanfstaengl v Empire Palace [1894] 2 Ch 1, 63 LJCh 417, 70 LT 459, subsequent proceedings, (1895), 11 TLR 368, 13 Digest
(Repl) 111, 525.
Hill (William) (Football) Ltd v Ladbroke (Football) Ltd 19th December, 1962, CA, not reported.
Purejoy Engineering Co Ltd v Sykes Boxall & Co Ltd (1955), 72 RPC 89, 13 Digest (Repl) 112, 537.
Ricordi (G) & Co (London) Ltd v Clayton and Waller Ltd (1930), Macgillivrays Copyright Cases 19281935, p 154.

Appeal
The plaintiffs appealed against an order of Wilberforce J, made on 27 July 1962, dismissing the plaintiffs action for alleged
infringement of their copyright in a song called In a Little Spanish Town. The plaintiffs sought an injunction restraining the
defendants from reproducing in any material form a musical work entitled Why or any work which reproduced or was an
adaptation of any substantial part of the plaintiffs song, and an inquiry as to damages for infringement of copyright and for
conversion, and alternatively an account of profits; and an order for delivery up of infringing material. The grounds of appeal
were: (i) that the similarity between the work In a Little Spanish Town and Why was such that the judge ought to have found
as a fact that the latter was derived from the former and was therefore a reproduction of a substantial part of the former; (ii) that
the similarity between the two works was such that the judge ought to have found as a fact that the composer of Why in
composing it must have used even though unconsciously his memory of In a Little Spanish Town; (iii) that, if a work in fact
contained a substantial part of an earlier work, and it was found as a fact that this was due to the composers memory of the
earlier work, the latter work constituted an infringement of the copyright in the earlier work notwithstanding that the composer of
the later work was unaware that, when composing it, he was relying on his memory of the earlier work; (iv) that, where it was
established that a work in fact contained a substantial part of an earlier work, the onus was on the defendant to show that the
similarity was due to some cause other than the conscious or unconscious recollection of the composer; (v) that the judge, having
been prepared to find as a fact that the composer of 17Why had at some time in some circumstances heard In a Little Spanish
Town, having regard to the similarities of the two works ought to have gone on to find as a fact that Why reproduced a
substantial part of the earlier work; and (vi) that the judge gave insufficient weight to the defendants admission that the musical
work in In a Little Spanish Town had been extensively exploited in the United States of America and elsewhere ever since 1926
by the publication of sheet music, by the distribution of gramophone records and by broadcasting.

John G Foster QC and F E Skone James for the plaintiffs.


J L Arnold QC and John E Williams for the defendants.
25 February 1963. The following judgments were delivered.

WILLMER LJ. This is an appeal against a judgment of Wilberforce J given on 27 July 1962, whereby he dismissed an action
brought by the plaintiffs for infringement of their copyright in a song called In a Little Spanish Town (to which I will refer as
Spanish Town). This was composed in 1926, and (as has been admitted by the defendants) was extensively exploited in the
United States of America and elsewhere by the publication of sheet music, by the distribution of gramophone records and by
broadcasting. Unlike many popular songs, Spanish Town appears to have retained its popularity over the years. Records
published in this country (some of them quite recently) were played to us during the course of the hearing, and I was readily able
to recognise the tune as a familiar one which I had heard on frequent previous occasions.
The defendants are the publishers of another song called Why, which was composed in 1959 by Mr Peter de Angelis.
Spanish Town is written in 3/4 time, and Why in 4/4 time. There are other differences between the two works, which were
the subject of a good deal of evidence by musical experts on both sides; but, when the two songs were played to us, it was
immediately apparent, to me at any rate, that the effect on the ear was one of noticeable similarity. This is a matter which is not
without importance, for, as was pointed out by Astbury J, in Austin v Columbia Gramophone Co Ltd ((1923), Macgillivrays
Copyright Cases, 19171923, at pp 415 and 409): Infringement of copyright in music is not a question of note for note
comparison but falls to be determined by the ear as well as by the eye.
The judge included in his judgment a detailed analysis of the musical structure of the two songs. I accept this as correct, and
it is, I think, unnecessary for me to repeat it except in summary form. In each case the essential feature of the song is contained
in the first eight bars, which constitute what has been described as a musical sentence, and in which the main theme is stated. It
is common ground that in the case of Spanish Town these first eight bars constitute a substantial part of the work within the
meaning of s 49 of the Copyright Act, 1956. In Spanish Town a subsidiary and contrasting theme is then introduced, after
which there is a return to the original theme, which is then re-stated with variations. By way of contrast, Why is described as a
thematic song; there is no subsidiary or contrasting theme, but, practically speaking, the whole song is devoted to the
development of the original theme.
Having given his analysis of the musical structure of each song, the judge stated his conclusions as to the points of
similariety or difference under nine headings, which I will summarise. (1) The structure of the two songs is different in the way
that I have already described. (2) The first eight bars being the essential part of Spanish Town, if the theme therein stated has
been borrowed in Why, the fact that it is developed by staying with it, rather than by way of contrast and return, would not
make a significant difference, but might even accentuate the likeness. (3) The theme of Spanish Town is built up of musical
common-places or cliches. The six notes of the first bar are a common-place series, found in other previous musical works, and
the manner in which this phrase is developed during the rest of the first eight bars is by way of some of the commonest tricks of
composition. The result, however, is a combination which gives character and charm to Spanish Town. (4) On a note for note
comparison between the 18 two songs there is a noticeable correspondence, though at no point do more than five consecutive
notes correspond. But the fact that in Why the descent of one-sixth from the first to the second note is immediately followed
by a leap back of a sixth to the original note instead of an arpeggio, as in Spanish Town, constitutes a not insignificant
difference. (5) The harmonic structure of the first eight bars is the same in both cases; but this is completely common-place and
insignificant. (6) Correspondence of notes is not of itself enough to create similarity; time and rhythm are equally important. (7)
In the present case there is a difference in time, but this is not a decisive factor, for in the case of Spanish Town, as with other
popular songs, a change of time from 3/4 to 4/4 does not destroy its substantial identity, or cause loss of recognition. (8) There is,
however, a significant difference in rhythm between the two songs; this is exemplified in the first bar, which in the case of
Spanish Town consists of an even sequence of six quavers, whereas in the case of Why the first note is held for half a bar,
and is followed by a descent to an accented note. (9) There is a significant difference between the two songs in the third and
seventh bars. In Spanish Town these consist of a single held note; in Why these bars each start with the same note as in
Spanish Town, but after an interval of a crotchet there is a drop of a fifth and a fourth respectively to a different held note.
Having stated these various points of similarity and difference (which I wholly accept) the judge expressed the view that, in
relation to the aural appeal of the sentence as a whole, there is an undoubted degree of similarity between the two songs, the only
question being what adjective to put before the word degree. He expressed his conclusion as follows:

On the whole, I think Mr. Palmers word definite or considerable is the right weight to put on the degree of
similarity; it is such that an ordinary reasonably experienced listener might think that perhaps one had come from the
other.

With that conclusion, I entirely agree.


If the matter stopped there, I do not think it could be doubted that there was material on which to base the inference that the
composer of Why deliberately copied from Spanish Town. Were that the right inference, I am satisfied that the degree of
similarity would be sufficient to constitute an infringement of the plaintiffs copyright. But the composer of Why was called as
a witness, and not only denied copying, but denied that he had ever seen the music of Spanish Town, or even consciously heard
it. He was a man of thirty-three years of age, and had lived most of his life in the United States. He stated that he had been
composing music since he was eleven, and had played various instruments in dance bands. In cross-examination he admitted that
at a younger age he might have heard Spanish Town, because he had heard a lot of music, but he adhered to his statement that
he had never consciously studied it, and said that he did not recall ever playing it. The judge accepted his evidence, and I do not
think that we in this court could properly interfere with that finding even if we were invited to do so, which we were not. But the
plaintiffs say that that is by no means the end of the case, for Mr de Angelis could well have copied from Spanish Town
subconsciously. The song having been extensively exploited in the United States, the overwhelming probability (it is said) is that
he must have heard it; and the degree of similarity between Spanish Town and Why is such that an inference of, at any rate,
subconscious copying should be drawn. That, it is contended, would be enough to constitute an infringement of the plaintiffs
copyright. The judge, however, decided that there was not sufficient material to justify the inference that Mr de Angelis copied
the plaintiffs work, even subconsciously; and he accordingly dismissed the action. It is to this point that the present appeal has
been mainly directed.
In approaching the suggestion of subconscious copying on the part of Mr de Angelis, it is to be observed that the Copyright
Act, 1956, nowhere uses the word copying. Section 2(5) provides:
19

The acts restricted by the copyright in a literary, dramatic or musical work are(a) reproducing the work in any
material form; (b) publishing the work; (c) performing the work in public; (d) broadcasting the work; (e) causing the work
to be transmitted to subscribers to a diffusion service; (f) making any adaptation of the work; (g) doing, in relation to an
adaptation of the work, any of the acts specified in relation to the work in paras. (a) to (e) of this subsection.

By sub-s (6), para (b), adaptation in relation to a musical work is defined as meaning an arrangement or transcription of the
work. By s 48(1) reproduction is defined as including reproduction in the form of a record. There is no further relevant
definition of the word, and it has been left to judicial decision to introduce the notion of copying.
Counsel, in presenting his argument on behalf of the defendants, drew attention to the fact that in relation to musical
copyright, under s 2 of the Act of 1956, there are only three forbidden processes, viz, reproduction, arrangement, and
transcription. Arrangement and transcription, he submitted, can be only the result of a conscious and deliberate process; a man
cannot arrange or transcribe without knowing that he is doing so. The judges acceptance of the evidence of Mr de Angelis,
therefore, precludes the possibility of finding any infringement of the plaintiffs copyright by arrangement or transcription. This
submission must, I think, be accepted.
Counsel for the defendants conceded that reproduction could possibly be the result of a subconscious process. But he went
on to submit that reproduction within the section could mean nothing short of identity. Reproduction, under s 49, may be of a
substantial part; but there is no suggestion in the Act of 1956 of any such thing as a substantial reproduction. In the present
case it cannot be said that there is anything approaching identity between the plaintiffs work and that of Mr de Angelis.
Consequently, counsel submitted, there could be no infringement of the plaintiffs copyright, whether conscious or unconscious,
by way of reproduction.
I find myself quite unable to accept this submission, for I can find no warrant for the suggestion that reproduction, within the
meaning of the section, occurs only when identity is achieved. This not only offends against common sense, but, I think, is
contrary to authority. In Austin v Columbia Gramophone Co Ltd ((1923), Macgillivrays Copyright Cases, 19171923, at p 398)
the headnote reads:

Infringement of copyright in music is not a question of note for note comparison, but of whether the substance of the
original copyright work is taken or not.

In that case, Astbury J, quoted from the earlier case of DAlamaine v Boosey ((1835), 1 Y & C at p 302) where it was laid down
that it must depend on whether the air taken is substantially the same with the original. I accept that as a correct statement of
the principle.
On the other side, counsel for the plaintiffs submitted in the first place that Mr de Angeliss denial of copying was wholly
irrelevant. For where, as was said to be the case here, a sufficient degree of similarity is shown, and it is further proved that the
composer of the second work had access to the earlier work in the sense that he must probably have heard it, an irrebuttable
presumption arises that the former has been copied from the latter. No authority was cited in support of this proposition, which,
if well-founded, would eliminate the necessity for any further evidence once similarity coupled with access had been proved. In
my judgment, the proposition contended for is quite untenable; the most that can be said, it seems to me, is that proof of
similarity, coupled with access, raises a prima facie case for the defendant to answer.
Counsel for the plaintiffs contended in the alternative that the degree of similarity found by the judge in the present case was
such as to compel an inference 20 of copying which, even if subconscious, was sufficient to give the plaintiffs a cause of action
for infringement. I confess that I have found the notion of subconscious copying one of some difficulty, for at first sight it would
seem to amount to a contradiction in terms, the word copying in its ordinary usage connoting what is essentially a conscious
process. The text-books on copyright make no reference to the subject, and English authority in relation to it is confined to a
single dictum of Luxmoore J, in G Ricordi & Co (London) Ltd v Clayton and Waller Ltd ((1930), Macgillivrays Copyright
Cases, 19281935, at p 154). Our attention, however, was called to a number of cases in the United States in which the subject
has been discussed, and in some of which a decision in favour of the plaintiff has been based on a finding of subconscious
copying. It appears to me that the question must be considered in two stages, viz, (i) whether subconscious copying is a
psychological possibility; and (ii), if so, whether in a given case it is capable of amounting to an infringement of the plaintiffs
copyright.
As to the first of these questions, it was suggested by counsel for the defendants that medical evidence should always be
required before a finding of subconscious copying could be justified. I cannot think that this is necessary; for the psychological
possibility of subconscious copying was clearly recognised by Luxmoore J and in the various American decisions, which must be
regarded as of high persuasive authority. What Luxmoore J said, in relation to the defendants before him in the Ricordi case
((1930), Macgillivrays Copyright Cases, 19281935, at p 162), was:

If there has been any infringement it must have been subconsciously, because the persons responsible knew the air
complained of so well that they have taken it because they knew it.

Similarly, in two American cases in which the plaintiff succeeded on the ground of subconscious copying, viz, Fred Fisher, Inc v
Dillingham and Edwards & Deutsch Lithographing Co v Boorman, the decision was based on the finding of a high degree of
familiarity with the plaintiffs work. From this emerges the conclusion, which seems to me to be consonant with good sense, that,
if subconscious copying is to be found, there must be proof (or at least a strong inference) of de facto familiarity with the work
alleged to be copied. In the present case, on the findings of the judge, this element is conspicuously lacking.
On the second question, viz, whether any subconscious copying proved could amount to an infringement of the plaintiffs
copyright, it seems to me that all that can be said is that at least the dictum of Luxmoore J envisages the possibility. On this point
I do not think that much help is to be derived from the American decisions which have been cited, since the American statute
under which they were decided is markedly different in its terms. No evidence of American law was adduced, and in its absence
it is not for us to construe the American statute. However (as was pointed out by junior counsel for the plaintiffs) it may be
observed that, in order to establish an infringement of copyright, it is not necessary to prove anything in the nature of mens rea.
The printer, for instance, may be held guilty of infringement though he has no conscious intent.
The conclusion at which I arrive on this part of the case is that subconscious copying is a possibility which, if it occurs, may
amount to an infringement of copyright. But in order to establish liability on this ground, it must be shown that the composer of
the offending work was in fact familiar with the work alleged to have been copied. This view, I think, is not inconsistent with the
submissions put forward by Mr Skone James. In the course of an argument which I found convincing, he submitted that, in
considering whether there has been reproduction, so as to constitute an infringement within the Copyright Act, 1956, it is wholly
irrelevant to inquire whether any copying has been conscious or subconscious. It is for this reason, he modestly suggested, that
the text-books are silent on the subject of subconscious copying. Mr Skone James presented his argument in four propositions
which, if I understood him correctly, may be summarised as 21 follows: (i) in order to constitute reproduction within the
meaning of the Act, there must be (a) a sufficient degree of objective similarity between the two works; and (b) some causal
connexion between the plaintiffs and the defendants work. (ii) it is quite irrelevant to inquire whether the defendant was or was
not consciously aware of such causal connexion. (iii) where there is a substantial degree of objective similarity, this of itself will
afford prima facie evidence to show that there is a causal connexion between the plaintiffs and the defendants work; at least, it
is a circumstance from which the inference may be drawn. (iv) the fact that the defendant denies that he consciously copied
affords some evidence to rebut the inference of causal connexion arising from the objective similarity, but is in no way
conclusive.
If this is the right approach (as I think it is), it becomes a simple question of fact to decide whether the degree of objective
similarity proved is sufficient, in all the circumstances of the particular case, to warrant the inference that there is a causal
connexion between the plaintiffs and the defendants work. This is the way in which, as it seems to me, the judge in the present
case approached the question which he had to decide. He directed himself as follows:

The final question to be resolved is whether the plaintiffs work has been copied or reproduced, and it seems to me that
the answer can only be reached by a judgment of fact on a number of composite elements: The degree of familiarity (if
proved at all, or properly inferred) with the plaintiffs work, the character of the work, particularly its qualities of
impressing the mind and memory, the objective similarity of the defendants work, the inherent probability that such
similarity as is found could be due to coincidence, the existence of other influences on the defendant composer, and not
least the quality of the defendant composers own evidence on the presence or otherwise in his mind of the plaintiffs
work.

In my judgment that was a proper direction, against which no criticism can fairly be brought.
Having so stated the question to be determined, and the matters to be considered, the judge stated his conclusion as follows:

In this case, after taking account of the respective character and similarities of the two works as previously discussed,
and relating this to the fact that there is no direct evidence that Mr. Peter de Angelis ever knew the work of Spanish Town
before he composed Why, I have come to the conclusion that I have not sufficient factual material from which to draw an
inference that he had sufficient knowledge or memory of Spanish Town at the date of composition to justify me in
finding, against his express denial, that in composing Why he copied, without knowing that he did so, Spanish Town or
a part of Spanish Town. Putting it in another way, it does not seem to me that the degree of similarity shown, coupled
with the fact, which I think is as far as it is possible to go by inference, that at some time and in some circumstances Mr. de
Angelis must have heard Spanish Town, is enough to make good the plaintiffs case.

The question, being one of fact, is eminently one for the determination of the trial judge, as I think is recognised in the American
cases which were cited to us. It is to be remembered that the judge not only had the advantage, denied to us, of himself seeing
and hearing the witnesses at first hand; he also had the advantage, which strikes me as being of great importance, of hearing how
the musical experts who were called as witnesses illustrated the technical evidence which they gave by demonstrations, both
vocally and on the piano. Bearing this in mind, and having regard to the judges acceptance of the evidence given by Mr de
Angelis, in my judgment it is impossible for us in this court to say that he reached a wrong conclusion on what was eminently a
question of fact for him.
I should perhaps mention one further consideration which appears to me to be of possible significance, and which was not
dealt with specifically by the judge. I 22 have already referred to the fact that the six quavers which form the opening bar of
Spanish Town are, as the judge observed, a common-place series to be found in other previous musical compositions. Our
attention was drawn, for instance, to an Austrian dance tune composed in the early nineteenth century by Von Lichnowsky, the
opening bar of which is identical with that of Spanish Town. The same sequence of notes is also to be found in a song entitled
Let Us Sing Merrily, although in this case there is a difference of tempo. In these circumstances, the fact that Why begins
with an opening bar containing a similar, though not identical, phrase is of no special significance. By itself it would not be
sufficient to warrant the inference that, if the phrase was copied, it was copied from the plaintiffs work rather than some other
composition. What is significant is the fact that, both in Spanish Town and in Why, the opening phrase enunciated in the
first bar is developed over the remainder of the first eight bars by the use of the same devices or tricks of composition, viz,
repetition followed by a pause, followed again by further repetition with a slight variation. It is this circumstance which produces
the degree of similarity between the two compositions. If it could be said that this method of development was so distinctive and
idiosyncratic as to preclude the possibility that its adoption by the two composers was the result of coincidence, this would be a
very strong argument in the plaintiffs favour. But, as pointed out by the judge, the devices used by the two composers for
developing the phrase stated in the first bar are among the commonest tricks of composition and, I would add, exactly the sort to
be expected from the composer of a popular song. I do not think, therefore, that in the circumstances of this case the fact that Mr
de Angelis developed the opening phrase stated in the first bar by way of the same devices as were employed by the composer of
Spanish Town can be taken as in any sense proof of copying. There is at least an equal probability that his choice of these
devices was the result of coincidence.
In my judgment, no sufficient reason has been shown for interfering with the judges decision, and I would accordingly
dismiss the appeal.

UPJOHN LJ. I agree with the judgment which has just been delivered. When counsel for the plaintiffs opened this appeal, he
invited us in the name of international comity to say that a right of property (ie, copyright) which is the subject of international
convention must be protected in a most special and unique way. We were invited to say that, if similarity in the alleged infringing
work to the original work was established as a fact, and if it was further established that the alleged infringer had had some access
to the original work, then, although a denial of conscious plagiarism was accepted, we were bound, as an irrebuttable
presumption of law, to say that the alleged infringer must have unconsciously copied the original work. The doctrine was said to
be necessary to protect the author of the original work, for otherwise (so it was argued) any infringer could escape the
consequences of plagiarism by denying that he had done so. Alternatively, it was said that, if some undefined higher degree of
similarity between the two works could be provedsomething higher than is necessary to prove similarity in factthen that
would be sufficient to establish a similarity from which we were bound to infer unconscious copying.
Apart from the appeal to international comity, no authority and no text-book has been cited in support of this remarkable
doctrine. Copyright is statutory, and depends on s 2 of the Copyright Act, 1956. No hint of this doctrine appears there. As to
international comity, while it is true that in the United States of America a number of authorities (to which I shall have occasion
to refer later in some detail) accept the doctrine that subconscious or unconscious copying may be inferred in a proper case and
operate as a breach of copyright, not one of those authorities gives any support to this alleged and startling doctrine. The
authorities in question in each case treated the question of unconscious copying as purely a question of inference of fact which
might be drawn in the circumstances of a 23 particular case, and not as a presumption of law. We were not referred to the laws of
any other convention country, and the relevant paragraph (No 4) of the Brussels Convention itself lends no support to the
doctrine. I therefore reject this submission.
The truth is that the plaintiff in a copyright action must show that a substantial part of the original work has been reproduced
(see s 2(5) and s 49 of the Copyright Act, 1956); and, although not expressed in the Act, it is common ground that such a
reproduction, in the words of junior counsel for the plaintiffs, must be causally connected with the work of the original author. If
it is an independent work, then, though identical in every way, there is no infringement. If a true infringer wrongly persuades the
court that it is his own unaided work, the plaintiff fails, as do other plaintiffs when fraudulent defendants unhappily succeed (as,
no doubt, they sometimes do) in persuading the court that they have not been fraudulent. The question in this case, therefore, is
whether there has been a breach of s 2(5); that subsection has been read by my Lord, and I will not read it again.
This is really a question of fact and nothing else, which depends on the circumstances of each case. But it is a question of
fact which must be taken in two stages. The first stage is objective and the second stage subjective. The first question is whether
in fact the alleged infringing workwhich for the sake of brevity I will inaccurately call the defendants work, for though the
composer was a witness, he was not a defendantis similar to the work of the original author, which again for the sake of brevity
I will (with equal inaccuracy) call the plaintiffs work. Is it then proper to draw the inference that the defendants work may have
been copied from the plaintiffs work? This is purely an objective question of fact, and depends in large degree on the aural
perception of the judge, but also on the expert evidence tendered to him; but it is essentially a jury question. A defendant might
in theory go into the witness-box and say that he had deliberately made use of the plaintiffs work, but that it is not an
infringement, either because he did not make use of a substantial part of the plaintiffs work, or because though the plaintiffs
work has been utilised, he has been able so to alter it that it cannot properly be described as a reproduction. The onus is on the
plaintiff to prove the contrary as a matter of purely objective fact, and if he cannot do so then the morally dishonest defendant
will escape the consequence of the allegation of infringement. No such question arises in this case. At this stage similarity has
been found by the judge, and that is not challenged before us. For myself, I think that perhaps I would have used rather stronger
adjectives than definite or considerable similarity, which were the words used by the judge; the adjective close would be
more appropriate, but nothing, I think, turns on that.
The next stage is the subjective stage, and is equally a question of fact, though of course the degree of similarity is most
important in reaching this subjective conclusion. The question at this stage, put bluntly, is: Has the defendant copied the
plaintiffs work, or is it an independent work of his own? Mr Skone James, in an attractive argument, agrees that the plaintiff in
order to succeed must prove a causal connexion between his work and the defendants work; but he submits that, providing that,
on a proper inference from the known facts it is right to assume that the alleged infringing work was derived from the plaintiffs
work, it matters not whether it was done consciously or unconsciously. There is, he submits, no difference in principle between a
conscious act of piracy and an unconscious act of piracy; all that must be established is a causal connexion.
While conscious acts of piracy may be established in the witness-box, unconscious acts of piracy must clearly be a matter of
inference from surrounding circumstances. The alleged infringing work may be an identical reproduction of the original work
with all its idiosyncracies and all the same mistakes. Theoretically and mathematically, that may be a complete coincidence, and
both works may be the product of entirely independent brains; but the judge 24 has to judge of these matters on the balance of
probabilities; and such an identical reproduction may lead him to reject the evidence of the defendant, who otherwise appears to
be an honest witness and to make a finding of conscious piracy. Much less than complete identity may properly lead the judge,
on the balance of probabilities, to reject the evidence of an apparently honest witness on this question. This is a question of pure
fact in every case. It does not arise in this case for the judge accepted the authors evidence that he did not consciously derive the
composition of Why from In a Little Spanish Town, and that has not been challenged before us.
At this stage, therefore, the question is whether, on the facts of the case, it is proper to infer that Mr de Angelis has derived
Why unconsciously from the plaintiffs work, which he must have heard at some earlier time. This again is purely a question
of the proper inference of fact to be drawn from all the relevant and admissible known facts. There may be cases where, if the
circumstances do not justify the conclusion that the defendant, in denying conscious plagiarism, is not telling the truth, yet justify
the conclusion that he must have heard the plaintiffs tune, and subconsciously reproduced it.
I do not pause to recapitulate the facts of this case in any detail, for they have been set out in such meticulous detail in the
judges judgment, and also by my Lord. I draw the conclusion that, though, as I have already stated, the resemblance is a close
one, that resemblance in the circumstances of this case is little evidence of unconscious copying, The judge said this.:

Thirdly, the theme of Spanish Town is made up of common-place elements or, as some witnesses have called them,
clichs. The first six notes are a common-place enough series; they are found in an Austrian country dance and in a song,
Let Us Sing Merrily. The device of repetition, of resting for two bars on a long note and of repetition in sequence, are
the commonest tricks of composition. But many writers of great music have used cliches to produce masterpieces; indeed,
some of them have found in the common-place character of their basic phrase their stimulus. Professor Newman gave
some interesting examples from the music of Mozart, and most writers of popular songs use, and can use, nothing else. No
example was given of precisely this combination having been used in other compositions, though it was apparent that the
musical dictionaries and the experience of the witnesses had been thoroughly combed.

Having heard the arguments of counsel, accompanied by very helpful demonstrations on the piano, I reach the conclusion of
fact that it is not a mere legal or mathematical possibility, but a real live practical possibility that the defendants composition of
Why was an independent composition, apart altogether from Mr de Angeliss denial of conscious plagiarism, which was
accepted. This practical possibility again does not conclude the matter, for the defendants composition may nevertheless be the
result of unconscious memory. But first it is necessary to establish the probability that the defendant has heard the plaintiffs
composition.
The judge had to deal with a difficult situation as to whether Mr de Angelis had heard, or even played, as a youth in a dance
band, the plaintiffs composition. I think that it is possible that, although in perfect good faith he stated the contrary, Mr de
Angelis did hear the music, and possibly played it in his early youth. Each case must depend on its own facts, and it is not
possible to lay down any criteria. But it does seem to me that where, for the reasons which I have given, there is evidence from
the music itself that there is a real practical possibility of independent composition by the defendant, it requires quite strong
evidence to support the view that there may have been unconscious copying. To my mind, the possibility that the defendant had
heard it, or even played it in his early youth, is a quite insufficient ground on which it would be proper to draw the inference of
unconscious copying. It may be that in the future medical evidence will be available to guide us on this point, but in the absence
25 of acceptable and probative medical evidence I think that it requires quite strong evidence, in a case such as thiswhere, as I
have already pointed out, independent composition is a real practical possibilityto establish, as a matter of probability, that Mr
de Angeliss subconscious ego guided his hand.
The cases in the United States to which we have been referred offer some instructive comparison on their facts, although I
do not lose sight of the fact that cases are only authorities for legal propositions; but nevertheless the cases cited are helpful. In
the first case, Fred Fisher, Inc v Dillingham, that great judge, Judge Learned Hand, in giving the famous composer, Mr Jerome
Kern, the benefit of the doubt, said this ((1924) 298 Fed Rep at p 147):

On the whole, my belief is that, in composing the accompaniment to the refrain of Kalua, Mr. Kern must have
followed, probably unconsciously, what he had certainly often heard only a short time before.

That is in marked contrast to the facts of this case. Then in a rather different case, Edwards and Deutsch Lithographing Co v
Boorman, the plaintiffs prepared, printed, published and distributed certain discount tables, the copy-right work. The defendants
published very similar tables. But there it was established that the defendants had sold and handled the plaintiffs publications for
years, and on that the inference was drawn of unconscious copying. Again the facts of that case are very different from the one
before us.
Wilberforce J, put the relevant points to himself quite accurately, and my Lord has read that part of his judgment; and his
summary, which I will venture to repeat, was this:

In this case, after taking account of the respective character and similarities of the two works as previously discussed
and relating this to the fact that there is no direct evidence that Mr. Peter de Angelis ever knew the work of Spanish Town
before he composed Why, I have come to the conclusion that I have not sufficient factual material from which to draw an
inference that he had sufficient knowledge or memory of Spanish Town at the date of composition to justify me in
finding, against his express denial, that in composing Why he copied, without knowing that he did so, Spanish Town, or
a part of Spanish Town.

I entirely agree with that conclusion of fact of the judge in this case. I am not prepared on the evidence in this case to draw any
inference of unconscious copying.
That makes it unnecessary to decide the really interesting question whether if unconscious copying had been properly
inferred on the facts that would have amounted to an infringement and whether junior counsel for the plaintiff is right when he
says there is no difference in law between conscious and unconscious copying. It seems to me that that is an interesting question
on which I would venture to express no opinion, for, as I have said, it does not arise. In support of his argument that the infringer
may be entirely ignorant of knowledge of plagiarism, counsel has pointed out that, normally, the printer and publisher will also be
guilty of infringement, though they have no reason even to suspect that any plagiarism can be suggested. However, this does not
meet my difficulty. Knowledge or suspicion of plagiarism does not necessarily have to be shown against every defendant, but the
plaintiff always has to prove that the alleged infringement is not the independent work of the alleged infringing author or
composer, but is causally connected with the plaintiffs work. The real question is this: Can it be said to be an act of
reproduction, for the purposes of s 2(5) of the Copyright Act, 1956, if the alleged infringing work is not the conscious act of the
infringer? It has been argued that Luxmoore J, in Ricordis case ((1930), Macgillivrays Copyright Cases, 19281935, at p 162)
expressed the view that subconscious copying could be an infringement of 26 copyright; but I do not think that he intended to
express any view on the law at all. For my part, I think that this question, therefore, remains entirely open. I agree that this
appeal must be dismissed.

DIPLOCK LJ. This appeal seems to me to turn entirely on a question of fact: Was the judge entitled, notwithstanding the
similarities between the melodies of the plaintiffs song In a Little Spanish Town and the defendants song Why, to refuse to
infer that the composer of the latter work copied it from the former work? It is conceded on the one hand (as is obvious to the
ear) that the two works show considerable similarities, and on the other hand that the composer of Why did not intentionally
copy it from In a Little Spanish Town; but it was found by the judge that the composer of Why must at some time and in
some circumstances have heard In a Little Spanish Town; and it is contended by the plaintiffs that the only proper inference of
fact is that he must have stored it in his memory and reproduced it without being aware that he was performing an exercise of
recollection and not an act of independent creation.
To this assumed mental feat there has been applied the conveniently ambiguous term subconscious copying; and we have
heard much argument as to whether, if it is established, it constitutes an infringement of the copyright in the work which has been
unconsciously copied. I think that the law is perfectly clear, and that such difficulties as there are in this appeal are solely due to
the absence of any factual information about the mental process involved in subconscious copying. We know not whether it is
rare or common, general or idiosyncratic, nor indeed whether it is possible to remember, not a mere isolated phrase, but a
substantial part of the remembered work without remembering that one is remembering.
First, as to the law; and for this purpose I will assume that it is established that the composer of Why did in fact use his
recollection of a substantial part of the melody of In a Little Spanish Town as the model for his own composition, although
he was unaware that he was doing so, and genuinely thought that Why was his own independent creation. The verb to copy
is not used at all in the Copyright Act, 1956, nor was it in the Copyright Act, 1911. Nevertheless, it is well established that to
constitute infringement of copyright in any literary, dramatic or musical work there must be present two elements: First, there
must be sufficient objective similarity between the infringing work and the copyright work, or a substantial part thereof, for the
former to be properly described, not necessarily as identical with, but as a reproduction or adaptation of the latter; secondly, the
copyright work must be the source from which the infringing work is derived. The necessity for the second element was
expressly laid down by the Court of Appeal in Purejoy Engineering Co Ltd v Sykes Boxall & Co Ltd ((1955), 72 RPC at p 99),
and is, indeed, implicit in all the compilation cases, including the recent case in this court of William Hill (Football) Ltd v
Ladbroke (Football) Ltd, where tables of betting odds were unanimously held not to infringe the copyright in substantially
identical tables because the authors of the later tables, although very familiar with the earlier tables, had in fact worked out the
odds for themselves. But, while the copyright work must be the source from which the infringing work is derived, it need not be
the direct source; see Hanfstaengl v Empire Palace. Mr Skone James, I think, put it with his usual accuracy when he said there
must be a causal connexion between the copyright work and the infringing work. To borrow an expression once fashionable in
the law of negligence, the copyright work must be shown to be a causa sine qua non of the infringing work.
The necessity for a causal connexion between the copyright work and the infringing work, although well established under
the Copyright Act, 1911, either 27 as being implicit in the legal concept of copyright, or in the word reproduce, is, I think,
more easily deduced wording of the current Copyright Act, 1956. Section 1(1) defines copyright in relation to a work as the
exclusive right to do and to authorise other persons to do certain acts in relation to that work; and sub-s (2) defines
infringement as the doing of any of those acts by a person who is not the owner of the copyright or his licensee. The acts,
which are defined in s 2, and include reproducing the work in any material form, if they are to constitute infringement must
thus be done in relation to the work, an expression which connotes a causal connexion between the copyright work and the act
relied on as an infringement. If the existence of the copyright work has no causal connexion with the production of the alleged
infringing work, even though the latter be identical with the former, there is no infringement of copyright.
In my view, however, it is equally clear law that neither intention to infringe, nor knowledge that he is infringing on the part
of the defendant, is a necessary ingredient in the cause of action for infringement of copyright. Once the two elements of
sufficient objective similarity and causal connexion are established, it is no defence that the defendant was unaware (and could
not have been aware) that what he was doing infringed the copyright in the plaintiffs work. This is expressly recognised by s 17
and s 18 of the Copyright Act, 1956, which restrict the remedies available against an innocent infringer, but recognise his liability,
Thus under s 18, which gives to the copyright owner remedies in conversion and detinue in respect of infringing copies of his
work, a defendant who believed and had reasonable grounds for believing that they were not infringing copies is relieved of
any liability in damages, but not of his liability to deliver up any infringing copies in his possession.
Unconscious copying in the sense in which it has been used in the argument postulates, first, such objective similarity
between the copyright work and the alleged infringing work that the latter may properly be said to reproduce the former;
secondly, that there is a causal connexion between the copyright work and the alleged infringing work; thirdly, that the composer
of the alleged infringing work believed (and may indeed have had reasonable grounds for believing) that there was no such causal
connexion. The first two, if established, in my view constitute breach of copyright; the third is irrelevant on liability, although it
may be relevant on remedy.
The real difficulty in this case is one not of law but of fact. It involves an inquiry into the working of the human mind. It
may well be that this is a matter on which expert evidence is admissible; but cases in English courts are normally conducted on
the tacit assumption that, where no question of disease of the mind is involved, the ordinary man, whether sitting in the jury-box
or on the bench, is capable of determining (where it is relevant) what went on in the defendants mind. The present case was so
conducted before Wilberforce J. No expert evidence was called as to how the human memory or musical creative faculties work;
no investigation was made into the mental idiosyncracies of the composer of Why or his methods of composition. The judge
was left to draw the inference of subconscious copying from the evidence (i) of the similarities between the melodies of In a
Little Spanish Town and Why as explained by the conflicting evidence of expert musicians; (ii) of the likelihood of the
composer of Why having at some time heard In a Little Spanish Town; and (iii) of his denial that he had consciously copied
In a Little Spanish Town.
On this state of the evidence, there were three possible explanations of the similarities: conscious copying, unconscious
copying, coincidence. The first the judge rejected. He accepted the denial of the composer of Why. This is a finding of
primary fact, and it depends ultimately on credibility. The plaintiffs do not seek to disturb it. This reduces the possible
explanations to two: unconscious copying, or coincidence. The judge did not reject the possibility that unconscious copying of
musical works can occur. He proceeded to consider, 28in the light of the conflicting expert evidence, which was the more
probable explanation of the similarities, unconscious copying, or coincidence. The relevant similarities were to be found in the
first eight bars of the melody of In a Little Spanish Town which, it is common ground, do constitute a substantial part of that
musical work. They are described clearly (and it is conceded accurately) in Wilberforce Js judgment.
The rival contentions, supported by expert evidence, may be summarised thus: The plaintiffs, conceding that the first bar by
itself was a musical clich in which there was no copyright, contended that the similarities in the use made of the clich in eight
successive bars in each of the two works were too great to be explained by coincidence. The defendants contended that, once a
composer of popular songs had decided to use, as a basis of the theme of a popular song, the musical clich contained in the first
bar, the use which was in fact made of it in both Why and In a Little Spanish Town in the succeeding bars was a device by no
means uncommon in musical composition, and the similarities were readily explicable by coincidence. The judge was not
satisfied that the similarities were due to unconscious copying. This, no doubt, was an inference of fact, but one which depends,
in part at least, on the degree of conviction which the evidence of the respective experts carried, and thus one with which an
appellate court should be slow to interfere. No attempt has been made to demonstrate that he has overlooked or misunderstood
any of the evidence.
How, then, is the case for the plaintiffs put? The procedure of the English courts, says counsel for the plaintiffs, is ill-
adapted to deal with such esoteric problems as subconscious copying. It places too heavy a burden on those who seek to
establish that it has occurred. Copyright is an international proprietary right, and English law should keep in step with foreign
law. They order these things better in more sophisticated (though unspecified) jurisdictions. But the only foreign law to which
we have actually been referred is to be found in the United States cases which Willmer and Upjohn LJJ have discussed; and there,
it seems, the matter is dealt with in the same unsophisticated way as that in which Wilberforce J dealt with this case, without
making it impossible for the courts to find (where the evidence so warrants) that unconscious copying has taken place.
Faced with the difficulty that unconscious copying is by definition not susceptible of direct proof in the present state of
psychological techniques, it must always be a matter of inference from other facts, the first bold submission of counsel for the
plaintiffs was that, if the plaintiff proves (i) the presence of the necessary element of objective similarity between the copyright
work and the alleged infringing work; and (ii) the mere possibility of access to the copyright work by the author of the alleged
infringing work, there is an irrebuttable presumption (ie, a presumption of law) that the author of the alleged infringing work
unconsciously copied the copyright work; or, put more briefly, what cannot be proved must be presumed. With all respect, this is
bad logic as well as bad law. For, unless the law is an asswhich I must ex officio irrebuttably presume that it is notthe
essential, though unexpressed, premise of this proposition is that the similarities cannot be due to coincidence; proof of
possibility of access is thus unnecessary; access as well as unconscious copying can be irrebuttably presumed. But this is merely
a roundabout way of saying that proof of a causal connexion between the copyright work and the alleged infringing work is not a
necessary element in infringement of copyright; and that is not the law.
The alternative submission of counsel for the plaintiffs (although I understood it to be presented as one of law) was, I think,
on analysis merely one as to the proper inferences of fact to be drawn from varying degrees of similarity between the copyright
work and the alleged infringing work. The degree of objective similarity is, of course, not merely important, indeed essential, in
proving the first element in infringement, viz, that the defendants work can properly be described as a reproduction or adaptation
of the copyright work; it is also very cogent material 29 from which to draw the inference that the defendant has in fact copied,
whether consciously or unconsciously, the copyright work. But it is not the only material. Even complete identity of the two
works may not be conclusive evidence of copying, for it may be proved that it was impossible for the author of the alleged
infringing work to have had access to the copyright work. And, once the impossible (viz, copying) has been eliminated, that
which remains (viz, coincidence) however improbable, is the truth; I quote inaccurately, but not unconsciously, from Sherlock
Holmes.
No useful purpose can thus be served by seeking to classify degrees of similarity into categories which must be taken to be
sufficient to prove unconscious copying where access to the copyright work by the author of the alleged infringing work is
proved (i) as a certainty; (ii) as a probability; (iii) as a possibility, and (iv) as an impossibility. That is not how questions of fact
are decided in courts of law, or anywhere else. The answer, as the judge said at the conclusion of an impeccable summary of the
evidence, can only be reached by a judgment of fact upon a number of composite elements. Those elements on which the
judge directed himself have already been read by my Lord, and I need not repeat them. I agree that it is impossible for this court,
which has not heard the evidence or seen the witnesses, to say that the judge came to a wrong conclusion of fact.

Appeal dismissed. Leave to appeal to the House of Lords refused.

Solicitors: Davenport, Lyons & Co (for the plaintiffs); Maltz, Mitchell & Co (for the defendants).

F A Amies Esq Barrister.


[1963] 2 All ER 30

Practice Note
(Notice of Appeal: Setting down: Lodgment of papers)
PRACTICE DIRECTIONS
15 MARCH 1963

Court of Appeal Notice of Appeal Setting down Preparation and lodgment of papers for use of Court of Appeal
Expedition.

On 15 March 1963, after judgments in an appeal from the Westminster County Court, dated 25 January 1963, had been delivered,

15 March 1963. The following judgment was delivered.

LORD DENNING MR made the following statement. Before parting with the case I would like to say it was tried at the county
court on 25 January 1963, and comes into our list for decision today. I am gratified that the solicitors for the appellants were able
to get the papers prepared for us in time for the case. I would like all solicitors to realise that as soon as a notice of appeal is set
down it is necessary for them at once to prepare all the papers and lodge them with the court. The days are long past when they
can say: We have many months before the case will come on. The solicitors in this case remedied the position. They did not
realise how soon the case would come on. I hope all solicitors will realise that cases in this court come on with the greatest
expedition and as soon as a case is entered, the papers must be prepared at once.

Solicitors: Kinch & Richardson agents for Albert G Berry & Co, New Malden (for the appellants); Brignall, White & Orchard,
Stevenage (for the respondent).
F Guttman Esq Barrister.
30
[1963] 2 All ER 31

Cooper v Firth Brown Ltd


CIVIL PROCEDURE; QUANTUM

SHEFFIELD ASSIZES
LAWTON J
19, 20 FEBRUARY 1963

Damages Measure of damages Loss of earnings National insurance contributions Whether assessment to be based on net
earnings after deducting employees contributions.

In assessing, in an action for negligence, the special damages of an employee for loss of earnings, where national insurance
contributions have been currently deducted by his employer, the earnings to be regarded as lost are the net amount after making
deduction for the employees national insurance contributions that would have been deductible by law by the employers from the
earnings, as well as (in accordance with the decision in British Transport Commission v Gourley, infra) income tax on the
earnings.
Dictum of Lord Tucker in British Transport Commission v Gourley ([1955] 3 All ER 796 at p 810) applied.

Notes
By virtue of s 3 of the National Health Service Contributions Act, 1957, it seems that the principle of the present decision must
apply to an employees national health service contributions just as it does to his national insurance contributions. Payment of
contributions for national insurance and for national insurance (industrial injuries) is normally made by affixing a single stamp
(see National Insurance and Industrial Injuries (Collection of Contributions) Regulations, 1948 (SI 1948 No 1274), reg 6(1); see,
generally, as to contributions and their collection 27 Halsburys Laws (3rd Edn) 690, paras 1248, 1249, and pp 699, 700, para
1270.
As to collateral benefits in assessing damages in tort, see 11 Halsburys Laws (3rd Edn) 240, para 408; and for cases on the
subject, see 17 Digest (Repl) 80, 81, 2738.
For s 6 of the National Insurance Act, 1946, see 16 Halsburys Statutes (2nd Edn) 679, 680; and for s 3 of the National
Health Service Contributions Act, 1957, see 37 Halsburys Statutes (2nd Edn) 807809.
For s 2 of the Law Reform (Personal Injuries) Act, 1948, see 25 Halsburys Statutes (2nd Edn) 365.

Cases referred to in judgment


British Transport Commission v Gourley [1955] 3 All ER 796, [1956] AC 185, [1956] 2 WLR 41, 3rd Digest Supp.
Browning v War Office [1962] 3 All ER 1089, [1963] 2 WLR 52.

Action
The plaintiff, Tom Cooper, claimed damages against his employers, the defendants, for their negligence. He was sixty-five years
of age and was injured in an accident at work on 15 November 1960, which was caused by a crane driver employed by the
defendants; the crane driver was young and inexperienced and did not really know how to operate the crane. The defendants
were held liable for their negligence in putting the crane driver in charge of the crane in the circumstances; there was no
contributory negligence on the part of the plaintiff. The case is reported solely on the question whether national insurance
employees contributions, which would have been deducted from the plaintiffs wages if he had earned them, should be taken into
account for the purpose of assessing the special damage recoverable by him for loss of earnings.

H Scott QC and A C Lauriston for the plaintiff.


R Castle-Miller for the defendants.

20 February 1963. The following judgment was delivered.

LAWTON J after stating his findings of fact summarised above, holding the defendants liable in damages for the plaintiffs
injuries, and finding that there was no contributory negligence on the part of the plaintiff, assessed general damages at 600, and
continued. Now arises the point, which I have been told by counsel has so far been undecided, as to how the special damages
should be 31 calculated in this case. As a result of the plaintiffs injuries he has been away from work from time to time, and it
was agreed between counsel that if his wages were calculated on one basis the special damages would be 450, and if they were
calculated on another basis the special damages would be 433. The difference is small. I have been told by counsel for the
defendants, however, that in industry the question raised in a very important one in which, when finance throughout the country
as a whole is considered, very substantial sums are involved. The difference in calculation arises in this way. The defendants
seek to persuade me that, when I am deciding how much the plaintiff has lost by way of wages, I should pay regard solely to what
he would have had in his pay packet if he had not been injured by the accident. In other words, that I should take into account
income tax deductions. That has been decided by the House of Lords in British Transport Commission v Gourley. The
defendants go on to say that I should also take into account the fact that the plaintiff had a statutory deduction from his wages, as
long as he was working, of a sum representing his proportion of the weekly contribution for national insurance. The plaintiff, on
the other hand, says that I should disregard the statutory deduction for national insurance as that has really got nothing to do with
the defendants at all. It was an insurance payment and it was that the plaintiff himself was bound to pay; he was not under a legal
duty to pay it out of his wages.
The basis of the defendants submission is founded on an observation of Lord Tucker in British Transport Commission v
Gourley. Lord Tucker said this ([1955] 3 All ER at p 810; [1956] AC at p 215):

I agree that the phrase res inter alios acta does not assist in the solution of the problem, but the difficulty is, I feel, in
deciding what items of expenditure following the earnings of profits are to be taken into consideration and which are to be
ignored. Such items are clearly distinguishable from those which are incurred in the process of earning the profits and
which have to be deducted in the computation thereof. I think the true answer is that expenditure whichalthough not
actually a charge on earningsis imposed by law as a necessary consequence of their receipt, is relevant to the
ascertainment of the loss suffered by the party injured.
The plaintiff as a person in employment was bound to pay national insurance contributions, and under the provisions of s 6 of the
National Insurance Act, 1946,a the defendants, being liable in the first instance to pay contributions on behalf of the plaintiff,
were entitled to deduct the amount of his contributions from his wages. It is then argued on behalf of the defendants that a
situation arises which comes squarely within Lord Tuckers observations in Gourleys case. My attention was also drawn to the
recent decision of the Court of Appeal in Browning v War Office. In that case the Court of Appeal made it clear that the
principles of British Transport Commission v Gourley are not to be confined solely to questions whether income tax should or
should not be taken into consideration, and that the ratio decidendi of British Transport Commission v Gourley is that damages
should be compensatory and not punitive.
________________________________________
a The right to deduct is conferred by the joint effect of s 6(1) and (3) of the Act of 1946 and the National Insurance and Industrial Injuries
(Collection of Contributions) Regulations, 1948, SI 1948 No 1274, reg 7(2). This provides, so far as relevant,Where the insured person
receives any wages the amount of any contribution paid or to be paid by the employer on behalf of the insured person shall be
recoverable by means of deductions from the wages See also s 5(1) of the National Insurance (Industrial Injuries) Act, 1946

It seems to me that the object of damages is to compensate the plaintiff for what he has lost, and what he has lost is what
would have been in his pay packet when he took it home. As against that, it was submitted on behalf of the plaintiff that I should
bear in mind the provisions of s 2 of the Law Reform 32(Personal Injuries) Act, 1948, which oblige courts to take into account
one half of certain benefits which have been obtained by an injured person; and counsel for the plaintiff argued that, if that were
the position, the defendants could not at the same time claim that they should also have the benefit of the contributions to national
insurance out of which those benefits arise. I feel impelled to take the view that what the plaintiff is entitled to is the loss of the
contents of his pay packet, and it seems to me that when special damages are being calculated there should be deducted the
amount of any national insurance contributions which the plaintiff would have had to make if he had remained in work. In those
circumstances the appropriate sum to be taken into account by way of special damages in this case is the sum of 433. It follows
that the total damages which I award the plaintiff is the sum of 1,033.

Judgment for the plaintiff.

Solicitors: Russell Jones and Walker (for the plaintiff); Harold Jackson & Co, Sheffield (for the defendants).

G M Smailes Esq Barrister.


[1963] 2 All ER 33

Astley Industrial Trust Ltd v Grimley


CONSUMER; Consumer credit

COURT OF APPEAL
ORMEROD, UPJOHN AND PEARSON LJJ
13, 14, 15 NOVEMBER 1962, 8 MARCH 1963

Hire-Purchase Condition Implied condition Fitness Fundamental term that goods must correspond with description
Tipping lorry six years old Lorry initially in state of some disrepair Latent defects Extent of obligation of finance company.

Hire-Purchase Warranty Fitness Implied stipulation that motor vehicle hired should be as fit as reasonable skill and care
can make it Whether a fundamental term of the contract entitling hire-purchaser to terminate contract Whether able to be
excluded by term of contract Necessity for hire-purchaser to have relied on implied obligation.

The extent of any obligation of fitness impliedly undertaken by an owner on the letting on hire or hire-purchase of a specific
chattel, such as a motor vehicle, depends on the contractual intention of the parties, which is to be ascertained from the provisions
of the agreement and the relevant facts; in general it is an implied fundamental term of an agreement between a hirer and a
finance company for the hire-purchase of a motor vehicle that the vehicle hired shall correspond with the description of the
vehicle contracted to be hired (see p 40, letter h, p 44, letter f, p 46, letter d, and p 48, letter d, post).
Dicta of Parker LJ, in Karsales (Harrow) Ltd v Wallis ([1956] 2 All ER at p 870) and of Holroyd Pearce LJ, in Yeoman
Credit Ltd v Apps ([1961] 2 All ER at p 287) considered.
G, who was a haulage contractor, told C, a representative of dealers to whose garage G had gone on a Saturday, that he (G)
wanted a tipping lorry capable of carrying ten tons of hard core or other filling materials at a price between 500 and 700, that
he was trying to expand his business and that he proposed to start work on the following Monday. C showed G some tipping
vehicles and said something to the effect that one of them, which was a Bedford tipping lorry six years old, would be the one
from which G would get the most satisfaction. Its cash price was 525, whereas the price of a new vehicle of the same type
would have been 1,600. G took the lorry for a run and noticed certain defects, in particular, defects of the clutch and the starter.
C said that they could be put right over the weekend and that the lorry would be ready on Monday. G paid a deposit, and signed a
hire-purchase agreement form and a delivery form. The plaintiffs, 33a finance company, accepted the transaction and entered
into the hire-purchase agreement. The hiring was for twenty-four months. The delivery form included an acknowledgment by G
that he had examined the lorry and had satisfied himself that it was in good order and condition. Clause 3 of the agreement
provided that the hirers acceptance of delivery of the lorry should be conclusive that he had examined the vehicle and found it to
be in good order and condition and in every way satisfactory to him; cl 3 further provided that the finance company gave no
warranty as to the state or quality of the vehicle and that any warranty as to description, repair, quality or fitness for any purposes
was excluded. On Monday delivery of the lorry was taken on behalf of G, who later found that the defects had not been
remedied. G returned the lorry to the dealers, and C told him that it would be checked and put right by Saturday. By the
Saturday G lost work and profits estimated at 30. On Saturday G took the lorry under protest, since he found that the repairs
had not been done, but he had arranged to start work with it the following Monday, and drove it about 150 miles to the site of the
work. When he tipped a load there oil gushed out. He took the lorry to a garage where the oil leak was mended, and
subsequently took it again to that garage where it was overhauled and defects were remedied, the total cost being 18 to which
the dealers contributed 9. Subsequently further repairs were required amounting to some 55, including the 18. G could not
find the money for a final expensive repair, and wrote to the plaintiffs terminating the hire-purchase agreement. This letter was
treated as a repudiation of the agreement. The plaintiffs sued G for arrears of rental and for 212 18s 2d damages for termination
of the agreement. G contended that there was an implied condition that the lorry should be as fit for the purpose for which it was
hired as reasonable care and skill could make it, that it was not, and that, therefore, he had been entitled to terminate the
agreement; he counterclaimed 55 odd damages for attempting to remedy defects, and he joined the dealers as third parties
claiming against them damages for breach of an alleged express warranty of fitness and damages for breaches of contract to
repair the lorry. On appeal from a decision of the county court in favour of the plaintiffs that, though there was an implied
condition that the lorry should be usable as a tipping lorry, there had been no breach of it, and in favour of the dealers to the
extent of finding that there had not been any express warranty,

Held (i) The plaintiffs (the finance company) were in the circumstances under an obligation as to the fitness of the tipping lorry,
this obligation being that it was a lorry of the make specified, and a tipper, and a motor vehicle capable of self-propulsion along a
road and of receiving and carrying loads of material (see p 44, letter f); on the facts, however, no breach of this fundamental term
or implied condition was proved, and the plaintiffs were entitled to recover under the hire-purchase agreement (see p 44, letter g,
p 47, letter f, and p 48, letter e, post), moreover Gs acknowledgment that he had examined the lorry and found it in good order
must at least exempt the plaintiffs from contractual responsibility for visible external defects such as the defects in the clutch or
starter or oil leakage on tipping (see p 44, letter i, to p 45, letter a, post).
(ii) no express warranty by the dealers was established; but the promises by the dealers regarding repair of the lorry were
contractual and had been broken, accordingly G was entitled to damages for breach of these contracts, such damages being 30
(in respect of the first (Saturday) repair promise) and 9 (viz, the remaining half of 18 in respect of the second (Monday) repair
promise) (see p 45, letters f, g and i, and p 48, letters c and f, post).
Per Upjohn LJ (Ormerod LJ concurring): in general on a hiring of an ordinary motor vehicle for normal use two stipulations
are to be implied, 34subject to any express terms, in the contract of hiring the second of these is that the vehicle must be as fit
for the purpose for which it was hired as reasonable care and skill can make it, but this stipulation is in the nature of a warranty,
not a condition going to the root of the contract, and the lender can lawfully exclude liability on it, though such exclusion would
not, in an appropriate case, prevent the hirer claiming that the breach frustrated the contract (see p 46, letters b and h, and p 47,
letters e and g, post); in the present case the hirer knew of the principal defects and, having taken the vehicle relying on a
collateral agreement with the dealer to remedy the defects, could not thereafter complain that the lender had failed in some
implied obligation to put right those defects (see p 47, letters h and i, post).
Appeal dismissed against the plaintiffs (finance company); allowed in part as against the dealers.

Notes
Liability for breach of the implied fundamental term cannot be excluded by an exception clause in the contract (see p 46, letter e,
post).
As to implied condition of fitness in hire and hire-purchase contracts, see 2 Halsburys Laws (3rd Edn) 123, 124, para 237;
19 ibid, 532, 533, para 858; and for cases on the subject, see 3 Digest (Repl) 9597, 231250; 26 Digest (Repl) 666, 35, 36. As
to exempting clauses in hire-purchase contracts, see 19 Halsburys Laws (3rd Edn) 531, para 855, text and note (s).

Cases referred to in judgments


Andrews Bros (Bournemouth) Ltd v Singer & Co Ltd [1934] 1 KB 17, 103 LJKB 90, 150 LT 172, Digest Supp.
Brown v Edgington (1841), 2 Man & G 279, Drinkwater 106, 2 Scott, NR 496, 10 LJCP 26, 5 JP 276, 133 ER 751, 39 Digest 470,
940.
Financings Ltd v Baldock [1963] 1 All ER 443, [1963] 2 WLR 359.
Hong Kong Fir Shipping Co Ltd v Kawasaki Kisen Kaisha Ltd [1962] 1 All ER 474, [1962] 2 QB 26, [1962] 2 WLR 474, 3rd
Digest Supp.
Hyman v Nye (1881), 6 QBD 685, 44 LT 919, 45 JP 554, 3 Digest (Repl) 96, 239.
Jones v Page (1867), 15 LT 619, 3 Digest (Repl) 95, 234.
Karsales (Harrow) Ltd v Wallis [1956] 2 All ER 866, [1956] 1 WLR 936, 26 Digest (Repl) 666, 35.
Parkinson v Lee (1802), 2 East, 314, 102 ER 389, 39 Digest 454, 811.
Pollock & Co v Macrae 1922 SC (HL) 192, [1922] SLT 510, 60 Sc LR 11, 39 Digest 468, q, 661, q.
Reed v Dean [1949] 1 KB 188, [1949] LJR 852, 12 Digest (Repl) 704, 5381.
Robertson v Amazon Tug and Lighterage Co (1881), 7 QBD 598, 51 LJQB 68, 46 LT 146, 4 Asp MLC 496, 3 Digest (Repl) 96,
242.
Smeaton, Hanscomb & Co Ltd v Sassoon I Setty Son & Co [1953] 2 All ER 1471, [1953] 1 WLR 1468, 3rd Digest Supp.
Sutton v Temple (1843), 12 M & W 52, 13 LJEx 17, 2 LTOS 102, 150, 152 ER 1108, 3 Digest 95, 231.
Yeoman Credit Ltd v Apps [1961] 2 All ER 281, [1962] 2 QB 508, [1961] 3 WLR 94, 3rd Digest Supp.

Appeal and Cross-appeal


This was an appeal by George Arthur Grimley, the first defendant, and a cross-appeal by the Astley Industrial Trust Ltd, the
plaintiffs, from a judgment dated 4 June 1962, given by His Honour Judge Steel in the Manchester County Court in an action
commenced by writ dated 21 April 1961, for 235 10s 7d alleged to be due under a hire-purchase agreement dated 25 April 1960,
made between the plaintiffs and the first defendant, in which action W 35 Jones (Manchester) Ltd (the dealers) were added, first
as third party and subsequently as second defendants.
The plaintiffs, a finance company, and the first defendant, who was a haulage contractor, entered into a hire-purchase
agreement dated 25 April 1960, relating to a vehicle described as a used Bedford tipper manufactured or first registered in 1954.
The cash price of the vehicle was 525, the initial instalment was 140 and the balance of hire was 450 9s, which was payable
by twenty-four monthly instalments of 18 5s 5d each. Paragraphs 3, 7 and 9 of the agreement, so far as material, were as
follows:

3. The hirers acceptance of delivery of the vehicle shall be conclusive that he has examined the vehicle and found the
same to be complete and in good order and condition and in every way satisfactory to him the [finance company] give
no warranty as to the state or quality of the vehicle, and any warranty as to description, repair, quality or fitness for any
purpose is hereby excluded
7. The hirer may at any time determine this agreement by notice in writing to the owners and thereupon the provisions
of cl. 9 shall apply.
9. If this agreement or the hiring be terminated for any reason before the hirer has become entitled to exercise his
option to purchase hereunder the hirer shall forthwith: (a) deliver up the vehicle ; and (b) pay to the [finance
company] all arrears of hire rentals due and unpaid to the date of return of the vehicle to the [finance company] or as
agreed compensation for depreciation of the goods such further sum as together with all rentals (including first instalment
of rent) previously paid hereunder shall equal two-thirds of the hire-purchase price as specified overleaf, whichever is the
greater

By an agreement of sale made in writing on 25 April 1960, the dealers sold the motor vehicle, the subject of the hire-purchase
agreement, to the finance company for 525, the agreement of sale containing an express term that the vehicle was in roadworthy
condition and had not been misrepresented in any way to the first defendant or to the finance company.
The first defendant paid the initial instalment under the hire-purchase agreement and signed a delivery form which stated:
The vehicle being second-hand is sold in the condition in which it now is, as inspected by the buyer and without warranty. He
paid the first monthly instalment but no other instalment, and by a letter dated 15 July 1960, he wrote to the plaintiffs: I am
obliged to inform you I shall be unable to continue payments on this vehicle Please consider this a termination of agreement.
He subsequently delivered the motor vehicle to the finance company. The finance company sold the vehicle for 150.
By their re-amended particulars of claim the finance company claimed against the first defendant 235 10s 7d as the balance
due under cl 9(b) of the hire-purchase agreement or alternatively 212 18s 2d, as damages for breach of contract and 18 15s 5d,
one monthly instalment due but unpaid, making in all 231 13s 7d. Alternatively, the finance company claimed against the
dealers the said sum of 231 13s 7d, as being the loss caused by the dealers to the finance company by the dealers breach of the
terms of the agreement of sale, and an indemnity against any liability of the finance company to the first defendant. The first
defendant by his defence and counterclaim pleaded (by para 4) that it was an implied condition and/or fundamental term of the
hire-purchase agreement that the vehicle was reasonably fit for the purpose for which it was hired and (by para 5) that in breach
of that condition or fundamental term, on the commencement of the said hiring the vehicle suffered from or shortly thereafter
developed the defects set out therein and thereby was not reasonably fit for the purpose of being driven on a road and loading and
tipping materials. The first defendant pleaded (para 6) that by reason of this breach of contract he repudiated the agreement by
letter dated 15 July 1960, and (para 7) he 36 denied any liability to the plaintiffs. The first defendant also pleaded, in the
alternative, that the claim under cl 9(b) of the agreement was a penalty and that if the finance company were not in breach of
contract as alleged, they were disentitled to the sums claimed by reason of their failure to mitigate their loss. The first defendant
further pleaded (para 10) that by reason of the aforesaid breach of condition or fundamental term he had incurred expenditure and
damage totalling 55 19s in attempting to put the vehicle in a roadworthy and usable condition, and he claimed to set off the said
sum against the finance companys claim and he counterclaimed the said sum.
By a third party notice served on the dealers by the first defendant, the first defendant claimed (i) an indemnity against any
liability which he might be found to owe to the plaintiffs; (ii) damages for breach of an express warranty as to the fitness of the
vehicle for the purpose for which the first defendant required it; and (iii) damages for breaches of contracts to repair the vehicle.
The damages so claimed were quantified as follows: (i) the amount of the deposit and instalment paid under the hire-purchase
agreement, viz, 158 15s 5d; (ii) special damages under para 9, namely, 55 19s (cost of repairs) and 30 (estimated loss of
profit), totalling 85 19s; and (iii) damages for inconvenience limited to a sum which would bring the total amount payable to
400.
By para 2 of their defence to counterclaim the finance company pleaded as follows:

When the said motor vehicle was sold to the [finance company] by [the dealers] it conformed in all respects with
statutory requirements and regulations, it was in roadworthy condition and was not misrepresented to the first defendant.
Further, the first defendant signed a delivery receipt annexed to the said agreement on delivery of the said motor vehicle
and the [finance company] will rely on the terms thereof.

The dealers pleaded a defence to the third party notice but did not deliver any defence to the finance companys claim against
them.
On 4 June 1962, His Honour Judge Steel, in the Manchester County Court, gave judgment for the finance company against
the first defendant for 231 15s 7d, and for the finance company on the first defendants counterclaim, and for the dealers against
the first defendant. By a notice of appeal delivered on 10 July 1962, and amended on 18 October 1962, the first defendant
appealed to the Court of Appeal for an order setting aside the said judgment and entering judgment for the first defendant on the
finance companys claim against him and on his counterclaim and on his claim for an indemnity and damages against the dealers,
or, alternatively for an order remitting the first defendants claim against the dealers to the county court judge to give the dealers
an opportunity of adducing evidence in support of their case. The finance company cross-appealed, with leave out of time, for an
order in favour of the finance company on their claim against the dealers for damages and an indemnity if the first defendants
appeal were successful.

Vaughan Davies for the first defendant.


J Hugill for the plaintiffs.
L A Cohen for the dealers, the third party and second defendants.

Cur adv vult

8 March 1963. The following judgments were delivered.

PEARSON LJ read the first judgment at the request of Ormerod LJ. This is a hire-purchase case relating to a used tipping lorry.
The plaintiffs are the finance company; the first defendant was the hirer; the third parties (also made second defendants at a late
stage of the pleadings) were the dealers. The finance company bought the lorry from the dealers and let it on hire-purchase to the
first defendant. In the action the finance company claimed, in addition to some arrears of hiring rent which were admittedly
owing, a sum of 212 18s 2d as damages for wrongful termination of the hire-purchase agreement by the first 37 defendant. In
his defence the first defendant pleaded that it was it was an implied condition and/or fundamental term that the said lorry was
reasonably fit for the purpose for which it was hired, and that in breach of the condition or fundamental term the lorry was not so
fit and consequently he was entitled to terminate the agreement. He also counterclaimed 55 19s damages for money spent in
attempting to rectify the defects, and sought to set-off that sum against the finance companys claim. The first defendant also
served a third party notice on the dealers, and his claim can be summarised shortly as being (1) for damages for breach of an
express warranty by the dealers as to the fitness of the lorry for the first defendants purpose and (2) for damages for breaches of
contracts to repair the lorry. The finance company pleaded a defence to the first defendants counterclaim and the dealers pleaded
a defence to the third party notice.
The learned county court judge held that as between the finance company and the first defendant there was an implied
condition or fundamental term to the effect that the lorry should be usable as a tipping lorry, but that there was no breach.
Accordingly he gave judgment for the finance company against the first defendant for 212 18s 2d damages as well as for the
admitted sum of 18 5s 5d arrears of hiring rent, and he dismissed the first defendants counterclaim. With regard to the first
defendants claim under the third party notice against the dealers he held that no warranty was given by the dealers, and he
dismissed the whole claim, not dealing specifically with the alleged breaches of contracts to repair the lorry. The first defendant
appeals both against the decision in favour of the finance company on the claim and counterclaim and against the decision in
favour of the dealers as third parties. I should add for the sake of completeness that the finance company joined the dealers as
second defendants, claiming to be indemnified in respect of their loss if the first defendants defence and counterclaim should
succeed, and now the finance company have given a notice of appeal involving the same contingent claim against the dealers.
The question, however, did not arise in the court below, and does not arise in this court unless the first defendants appeal against
the decision in favour of the finance company succeeds.
The main facts are as follows:On Saturday, 23 April 1960, the first defendant, having been attracted by one of the dealers
advertisements, went with his brother to the dealers garage and spoke to a man named Wilfred Callister, whose exact position
does not appear from the evidence but who evidently had authority to act on behalf of the dealers. The first defendant told
Callister that he was looking for a tipping vehicle, and wanted one capable of carrying ten tons of hard core or other filling
materials; that he was already a haulage contractor with a B licence and pretty well established, and wished to expand his
business and therefore wanted a tipping vehicle; and that he had arranged to start work on the Monday morning (25 April 1960)
in connexion with some operations being carried out at Barton Bridge near Manchester. The first defendant was shown several
tipping vehicles, and eventually, as he said in his evidence, he put his cards on the table, telling Callister that he had not been
tipping at all, and his price was between 500 and 700, and that he was asking what would be the best. Callister took him over
to a used Bedford tipping lorry, and said something to the effect that that was definitely the one he would get the most satisfaction
from. The vehicle was about six years old and the cash price was 525, whereas the price of a new lorry of the same type would
have been 1,600. The first defendant with one of the garage mechanics took the vehicle for a run and the first defendant noticed
certain defects, principally that when the vehicle was stopped and re-started the clutch was very fierce and the starter failed to
operate. He mentioned these defects to Callister, and Callister said That can be put right over the week-end and added that his
mechanics would work over the week-end and the vehicle would be ready for work on Monday. Thereupon the first defendant
paid a deposit of 140 and signed a hire-purchase agreement form, putting on it the date 25 April 1960, though he signed it on
Saturday, 23 April. 38That was his offer, and it was accepted by the finance company also signing it, probably on 25 April 1960,
as that date was not altered. Also on 23 April 1960, the first defendant signed a delivery form. I will refer later to the provisions
of the hire-purchase agreement and the delivery form.
On Monday, 25 April the first defendants brother brought the vehicle from the dealers garage, and the first defendant saw it
later in the day and found the same defects in the clutch and the starter and an oil leakage in the tipping gear. He took it back to
the dealers and eventually saw Callister and complained to him of the continued existence of the defects, saying that nothing had
been done. Callister said at first that he would need the vehicle for at least a week, but later said it would be ready by Saturday
(30 April). He said that he would give it a complete check over and that it would be right by Saturday. The first defendant
accordingly left the vehicle at the garage to be checked over and put right.
When he came to take the vehicle on Saturday, 30 April he found it in no better state. In his words the clutch was still
juddering, the starter was exactly the same, and they had not done the gear box. He had, however, arranged to start work on the
following Monday, 2 May on the Llanwern site near Newport in Monmouthshire. He took the vehicle under protest, telling the
dealers that they would hear more of it. He drove the vehicle on a journey of about 150 miles from Manchester to the Llanwern
site, and there is no evidence of the vehicle giving any trouble on that journey. Having arrived at the Llanwern site, he went,
presumably on Monday, 2 May with the vehicle to the quarry from which loads of hard-core were to be obtained. He took the
first load. When he tipped the load, oil gushed out. On the same day, 2 May he took the vehicle into Larcombes garage. A new
gasket was put in the tipping gear box, and this appears to have cured the oil leakage because there is no evidence of any oil
leakage afterwards. The first defendant arranged with Larcombe that the vehicle should be taken in for a full service and
thorough overhaul at the end of the week. During the week the first defendant carried ten loads, whereas he would have expected
to carry seven or eight loads a day, and he said that he was nursing the vehicle.
The work on which he was using the vehicle was very heavy work, such as might wear out even a new tipping lorry in a
short period. Presumably the lorry fetching hard core from a quarry would have to manoeuvre on an irregular and very hard
surface and thus would undergo severe strains and stresses. The learned judge said in his judgment I have evidence that this
type of work can smash a new lorry in six months. He may well have had such evidence, though it does not appear in the notes
of evidence. In the notes of the cross-examination of the first defendant there is this passage:

The work was rough, but I would not say a new lorry could be worn out by it in six months. I know that vehicles have
been abandoned at the Llanwern site, but they are not wrecks after six months. I did see two or three fairly new lorries left
abandoned but I made no inquiries myself for lorries lying about. I agree that I took a six year old lorry for rough work.

The vehicle was taken in to Larcombes garage at the weekend (6 or 7 May), and remained there till the following Thursday (12
May). Larcombe made a thorough examination and found numerous defects. Three of the road wheels were cracked and
unserviceable, the safety bracing on the tipping gear was cracked ninety per cent through, there was a broken main spring and a
broken helper spring, and the hand brake was defective. There was a crack in the main chassis in front of the main spring hanger,
and this was not a new crack, as grime had to be removed to see it. The starter ring had a number of teeth missing or badly worn.
There was a violent judder caused by a mechanical fault in the clutch mechanism. In Larcombes opinion the vehicle was
unroadworthy.
On a Monday, which was probably 9 May after the defects had been ascertained, the first defendant telephoned to the
dealers and mentioned the defects 39 that had been found, and asked them what they would do and whether they would agree to
pay the cost of repairs. The dealers offered to pay half the cost of materials, but were not willing to make any contribution to the
labour cost as they had their own mechanics who could have done the work. Larcombe repaired the vehicle, and the bill was
estimated at 18, and the first defendants brother collected 9 from the dealers. It was necessary to obtain main springs, a helper
spring, and there was a considerable amount of welding and there were four wheels to be replaced. These wheels were bought at
a price of 1 each. After the repairs the vehicle returned to the same very heavy work. It was in Monmouthshire for about seven
weeks, and it operated on the site for about five weeks, but not for any full week.
At intervals in June three disasters occurred. One evening, when the vehicle was operating from the quarry, the engine
mounting bracket collapsed, dropping one side of the engine on the front axle. The vehicle was towed to the nearest garage and
repaired by welding at a cost of 6. In the following week the cab supports collapsed, and the vehicle was again taken to the
nearest garage and repaired by welding and the cost was 10 10s. Other repairs were carried out to the hand brake, and to the
push rods for the clutch, as these push rods had broken by reason of the juddering and shaking. Tyres had to be renewed because
the juddering had caused the wheels to spin, and the total cost of tyre renewals was 22 10s. The final disaster occurred when the
clutch collapsed, and the repairing of it would be expensive. Even then the first defendant, if he had been able to raise the money,
would have got the vehicle out and continued operating. He had spent about 55 on the vehicle between 1 May and 15 July. The
figures which I have mentioned18, 6, 10 10s and 22 10smake a total of 57, which is sufficiently close to the
approximate figure of 55.
The first defendant was not able to raise the money for repairing the collapsed clutch, and he wrote a letter dated 15 July
1960, to the finance company, stating that he was unable to continue payments and terminating the agreement. It has been
assumed throughout these proceedings that this letter constitutes a repudiation of the agreement. The finance company replied on
26 July 1960, indicating that claims would have to be made. They were treating the first defendants letter as a repudiation of the
agreement. Manifestly the first defendant has been unfortunate, and deserves sympathy, but I have to consider only the
contractual rights and obligations of the parties and the alleged breaches of contract. First I will consider the contractual position
as between the first defendant and the finance company.
Counsel for the first defendant contended, as he had to contend in order to succeed against the finance company, that from
the mere fact of the finance company letting this tipping lorry on hire to the first defendant there was to be implied a condition in
his favour, binding on the finance company, that the vehicle should be as fit for his purpose, ie, for use as a tipping lorry, as
reasonable care and skill could make it. The decided cases show that normally in a hiring or hire-purchase agreement the person
who lets goods on hire assumes some contractual responsibility for the fitness of the goods for the purpose for which the hirer
requires them, but the existence and the extent of the obligation depend on the contractual intention of the parties, which is to be
ascertained from the provisions of the particular agreement and from the relevant facts of the situation in relation to which the
agreement was made. There could be a case in which there would be no such obligation at all. Suppose that a customer, proud of
his skill as a mechanic, hired or took on hire-purchase a dilapidated and immobile vehicle, hoping to be able by his own efforts to
put it into good repair and working order, and making it clear to the persons letting it out on hire to him that he would not hold
them responsible either for the existing state of disrepair or for the success of his attempts to rectify it. In such a case no rule of
law would compel the court to defeat the contractual intention of the parties by imposing on the persons letting the vehicle on
hire an obligation as to the fitness of the vehicle 40 for the hirers purpose. Therefore it is not possible to accept as an unqualified
proposition the contention advanced in this appeal on behalf of the first defendant.
I will cite passages from decided cases as showing the general principle involved and as illustrating the matters to be taken
into account in determining the existence and extent of the obligation in a particular case.
In Sutton v Temple it was held that in a demise of land there was no implied obligation that the land should be fit for the
purpose for which it was taken. There are, however, in the judgments some obiter dicta as to the letting of goods. Lord Abinger
said ((1843), 12 M & W at p 61):

The letting of the goods and chattels, as well as the house, implies that the party who lets it so furnished is under an
obligation to supply the other contracting party with whatever goods and chattels may be fit for the use and occupation of
such a house, according to its particular description, and suitable in every respect for his use. Where the party has had an
opportunity of personally inspecting a ready-furnished house by himself or his agent before entering on the occupation of
it, perhaps the objection would not arise; but if a person take a ready-furnished house upon the faith of its being suitably
furnished, surely the owner is under an obligation to let it in a habitable state. Common sense and common justice concur
in that conclusion.

Baron Parke said ((1843), 12 M & W at p 64):

One class of cases is, where the agreement is for a specific chattel in its then state, as in Parkinson v. Lee; there there
is no implied warranty of its fitness or merchantable quality: another class of cases is, where a person is employed to make
a specific chattelthere the law implies a contract on his part that it shall be fit for the purpose for which it is ordinarily
used: and there is also an intermediate class of cases, where goods are ordered for a specific purpose from a person in a
particular department of trade; in that case, also, Brown v. Edgington is an authority for saying, that the law implies an
undertaking that they shall be fit for that specific purpose.

Then Jones v Page, Hyman v Nye and Reed v Dean, are examples of cases where goods are ordered for a specific purpose from a
person in a particular department of trade. In Hyman v Nye, Lindley J, said ((1881), 6 QBD at p 687) that the duty of a jobmaster,
letting out horses and carriages for hire, was to supply a carriage as fit for the purpose for which it is hired as care and skill could
render it. He went on to say ((1881), 6 QBD at p 688):

Nor does it appear to me to be at all unreasonable to exact such vigilance from a person who makes it his business to
let out carriages for hire. As between him and the hirer the risk of defects in the carriage, so far as care and skill can avoid
them, ought to be thrown on the owner of the carriage. The hirer trusts him to supply a fit and proper carriage; the lender
has it in his power not only to see that it is in a proper state, and to keep it so, and thus protect himself from risk, but also to
charge his customers enough to cover his expenses.

Then Mathew J said ((1881), 6 QBD at pp 689, 690):

It appears to me that the question which the jury ought to have been asked was, whether the carriage was, in fact,
reasonably safe when it was hired by the plaintiff. The cases referred to by my brother LINDLEY, seem to show that there
is no distinction in this respect between contracts for the sale and for the hire of an article for a specific purpose, where
trust is reposed in the 41 person who, in the ordinary course of business, sells or lets to hire. The purpose and use, the time
for which the article is intended to be used, seem to me the essential part of the contract Here, the defendant let the
carriage for the purpose of carrying the plaintiff safely. The plaintiff trusted him to select the carriage, horses, and driver,
and there seems to me nothing unreasonable in charging the defendant with a duty which it was certainly in his power to
fulfil, and which from his business he would be presumed to have bound himself to take the proper steps to perform
strictly.

Then in Robertson v Amazon Tug and Lighterage Co, there was no hiring of a chattel but there are dicta on the subject. Bramwell
B, said ((1881), 7 QBD at p 603):

The case seems to me the same as a contract of hiring, and as all contracts when one man furnishes a specific thing to
another which that other is to use. The man so letting and furnishing the thing does not, except in some cases, undertake
for its goodness or fitness, but he does undertake for the condition being such that it can do what its means enable it to do.
Thus, if a man hired a specific horse and said he intended to hunt with it next day, there would be no undertaking by the
letter that it could leap or go fast; but there would be that it should have its shoes on, and that it should not have been
excessively worked or unfed the day before.

Brett LJ said ((1881), 7 QBD at p 606):

When there is a specific thing, there is no implied contract that it shall be reasonably fit for the purpose for which it is
hired or is to be used. That is the great distinction between a contract to supply a thing which is to be made and which is
not specific, and a contract with regard to a specific thing. In the one case you take the thing as it is, in the other the person
who undertakes to supply it is bound to supply a thing reasonably fit for the purpose for which it is made.

Cotton LJ said ((1881), 7 QBD at pp 608, 609):

It has been suggested that the plaintiff is in the same position as the hirer of an ascertained chattel, and the defendants
in the same position as the person who lets the chattel to hire. There is at least a doubt what warranty the law implies from
the relation of hirer and letter to hire of an ascertained chattel.

In Karsales (Harrow) Ltd v Wallis, Parker LJ said ([1956] 2 All ER 866 at p 870):

I think it is the duty of a hire-purchase finance company, which is letting out a chattel on hire-purchase, to ascertain
that the chattel is reasonably fit for the purpose for which it is expressly hired.

In my view that sentence should be understood as stating a general proposition not necessarily applying to every case. For
instance, it cannot apply if (as in the present case) the evident contractual intention of the parties is that the finance company shall
not have an opportunity of ascertaining the fitness of the chattel before the hiring begins. Parker LJ went on to refer to the
exceptions clause in that case, which was cl 3(g), and he said ([1956] 2 All ER at p 871):

But, in my judgment, however extensive the exception clause may be, it has no application if there has been a breach
of a fundamental term. We were referred to a number of cases, including Smeaton Hanscomb & Co., Ltd. v. Sassoon I.
Setty, Son & Co..

After citing from the judgment of Devlin J, in that case, Parker LJ, went on ([1956] 2 All ER at p 871):

Applying that to the facts of this case, it seems to me that the vehicle delivered in effect is not properly described (as
the agreement describes it) 42as a motor vehicle, Buick, giving the chassis and engine number. By that I am not saying
that every defect in a car which renders it for the moment unusable on the road amounts to a breach of a fundamental term;
but where, as here, a vehicle is delivered incapable of self-propulsion except after a complete overhaul and in the condition
referred to by my Lord, it seems to me that it is abundantly clear that there was a breach of a fundamental term and that
accordingly the exceptions in cl. 3(g) do not apply. I think the same result is achieved by saying, in effect, that what was
delivered was not what was contracted for; and I think that Andrews Bros. (Bournemouth), Ltd. v. Singer & Co. Ltd. might
well have been decided on the basis that there had been a breach of a fundamental term.

Then in Yeoman Credit Ltd v Apps, Holroyd Pearce LJ, having referred to a number of decided cases and finally to the decision of
Lewis J, in Reed v Dean that there was in that case an implied undertaking by the defendant that the vessel should be as safe as
care and skill could make it, said ([1961] 2 All ER at p 287):

Those cases clearly establish that such a warranty, condition or undertaking exists in the hiring of a specific chattel
except in the cases where defect is apparent to the hirer and he does not rely on the skill and judgment of the owner.

In my view that sentence should be regarded as directing attention to what are usually the main factors affecting the extent of the
implied obligation, and not as excluding other factors which may be relevant and important in a particular case. Holroyd Pearce
LJ, also said ([1961] 2 All ER at pp 289, 290):

Whether there has been a breach of a fundamental condition of the agreement is a question of degree depending on the
facts. Such a breach is different in weight and gravity from breaches of condition which would come within the exemption.
There may be, as in Pollock & Co. v. Macrae and in the present case, an accumulation of defects which, taken singly, might
well be within an exception clause, but which, taken en masse, constitute such a non-performance, or repudiation, or breach
going to the root of the contract, as disentitles a party to take refuge behind an exception clause intended to give protection
only in regard to those breaches which are not inconsistent with and not destructive of the whole essence of the contract.
One would not lightly come to the conclusion that mere defects in repair, even though numerous, amount to breach of a
fundamental condition of the contract. In the present case, however, I accept the finding of the learned judge on the facts
that: The plaintiffs were in breach of a fundamental term of the contract to hire a motor car suitable for use on the
highway and cannot rely on the exemption clause. There was, to use the words of LORD DUNEDIN a, such a congeries
of defects as to destroy the workable character of the machine.
________________________________________
a In Pollock & Co v Macrae, 1922 SC (HL) 192 at p 200

Now I come to the hire-purchase agreement between the first defendant and the finance company. There is no evidence of
the dealers being for any relevant purpose agents of the finance company, or of the arrangements made between the first
defendant and the dealers having been communicated to the finance company. The express terms of the contract between the first
defendant and the finance company have to be found in the agreement itself. The main points are as follows: (1) The date for
commencement of hiring was Monday, 25 April 1960. The first defendant had made his offer by filling in or having filled in for
him the details in the schedule and by putting his signature on the second page, on Saturday, 23 April. He was intending to drive
the vehicle away on the Monday. 43Presumably the finance company had never seen the vehicle and had no previous
knowledge of it. The first defendant obviously was not relying on the skill or judgment of the finance company to choose a
suitable vehicle for his purpose, or to give him any advice as to his choice, and he was not giving them any opportunity to
examine it or make inquiries about it before the commencement of the hiring. (2) The first defendant described himself as a
haulage contractor and referred to previous hire-purchase transactions. He could be expected to have some knowledge of
vehicles. (3) The vehicle was described as a Bedford tipper manufactured or first registered on 31 March 1954, so that it was six
years old. It is reasonable to assume that a tipping lorry is likely to be subjected to more arduous use, and to be in the common
phrase older for its age, than a motor car. It could not reasonably be expected that a six-year old tipping lorry, not stated to have
been recently overhauled or reconditioned, would be in anything like new condition or free from latent defects. (4) This was a
hire-purchase agreement with a hiring period of twenty-four months, so that immediate readiness for use would not appear to be
so important as in the case of a hiring for a short period. (5) The hirer acknowledged that before signing the agreement he had
examined the goods and satisfied himself that they were in good order and condition. It was true that he had examined them, but
not true that he was satisfied with them. However his dissatisfaction cannot prejudice the finance company, as he made no
complaint to them, and this acknowledgment must have its due effect on the extent of the finance companys obligations under
the agreement. (6) Clause 3 provides:

The hirers acceptance of delivery of the vehicle shall be conclusive that he has examined the vehicle and found the
same to be complete and in good order and condition and in every way satisfactory to him. Except where it is implied by
the Hire-Purchase Acts, 1938 and 1954 the [finance company] give no warranty as to the state or quality of the vehicle,
and, save as aforesaid, any warranty as to description, repair, quality or fitness for any purpose is hereby excluded.

In my view the finance company are not entitled to say that they had no obligation at all under the agreement as to the
fitness of the vehicle for the first defendants purpose. They were letting on hire to him a Bedford tipper, and it had to be a
Bedford tipper, ie a lorry of that make, and a tipper. It had to be an automobile, capable of self-propulsion along a road, and it
had to be capable of receiving and carrying and tipping loads of materials. The condition or fundamental term is to be implied in
this case, as it was in the Karsales case and in the Apps case. The learned county court judge, however, has found that there was
no breach of this condition or fundamental term, and on the facts of this case he was in my view clearly right. The vehicle was
capable of self-propulsion because it propelled itself from Manchester to the vicinity of Newport, Monmouthshire, and thereafter
to and from the Llanwern site, and it was receiving and carrying and tipping loads under very adverse conditions for five weeks.
There was some initial disrepair of a not trivial character, but even after some aggravation, probably, by the first week of
hard use on the Llanwern site it was put right at a cost of 18, of which the dealers paid half. The later disasters and grave
defects were probably caused by the continuing hard use of this six-year old lorry on work which put too great a strain on it. That
is really the explanation of the unfortunate history of this lorry, and that was the view of the learned county court judge and I
agree with him. The initial state of disrepair consisted, largely at any rate, of visible external defects, such as the fierce clutch
and the starter failing to operate and the oil leakage on tipping. The acknowledgment by the first defendant in the hire-purchase
agreement that he had examined the vehicle and found it in good order must at least exempt the finance company 44 from
contractual responsibility for visible external defects. Then what is left of the initial state of disrepair? The first defendant may
well have had good reasons, as against the dealers who had failed to do the repairs which they had promised, for driving the lorry
in its unrepaired state from Manchester to Llanwern and putting it into service there, but the result may have been to cause further
defects, for which the finance company could not in any case be held responsible. It was however not reasonably to be expected
that a six-year old tipping lorry, not stated to have been overhauled or reconditioned, would be wholly free from latent defects.
Larcombes bill at the end of the first week in May was only 18, of which the dealers paid half. If it be assumed that the finance
company had some obligation in respect of repair greater than the obligation to provide a lorry capable of self-propulsion and of
tipping, it is not clear that there was any breach of it. However, in my judgment, having regard to the provisions of the hire
purchase agreement and the facts as stated in it, there was no such greater obligation.
Now as against the dealer the first defendant has in my view failed to establish any express warranty as to the fitness of the
vehicle for his purpose. In the first place the oral evidence does not prove any words which prima facie have the appearance of a
warranty. The first defendant said he wanted a tipping lorry costing 500-700. Callister took the first defendant to this vehicle
and said It is definitely the one you will get the most satisfaction from or words to that effect. That seems to be saying This is
the best vehicle that we can provide at your price, without any suggestion that the dealers are assuming contractual
responsibility for its efficiency. Secondly, the first defendant signed at the request of the dealers a delivery form which contained
the words The vehicle being second-hand is sold in condition in which it now is, as inspected by the buyer and without
warranty. I agree with the county court judge that that is strong evidence of the absence of a warranty.
Finally however there are to be considered the dealers two broken promises with regard to the repair of the vehicle. The
learned county court judge did not deal with this aspect of the case, which was of course of less importance than the other issues.
In my judgment however these promises were contractual promises. The first promise was given on Saturday, 23 April 1960, in
consideration of the first defendant signing the hire-purchase agreement, and it was a promise to remedy the defects by Monday,
25 April so as to enable the first defendant to start his work on the Burton Bridge site. In consequence of the dealers failure to
carry out their promise the first defendant could not do any work on the Burton Bridge site, and lost profits which he estimated at
30, and he was not, so far as appears from the notes of evidence, cross-examined as to that estimate. In my view the first
defendant should have 30 damages against the dealers for breach of the Saturday contract. The figure seems on the generous
side, but there is no basis for fixing any other figure, and it would be unreasonable in the circumstances to order an inquiry as to
damages. Secondly, there was the Monday contract, made on Monday, 25 April to give the vehicle a complete check-over and
have it right by Saturday, 30 April. There was consideration, as the first defendant allowed the dealers to keep the vehicle and
work on it and seek to minimise the damage caused by their breach of the earlier contract. This promise also was unfulfilled, and
consequently the first defendant had to drive the lorry and start using it in its unrepaired condition. Exact assessment of the
damage is not possible, but Mr Larcombes bill was 18, and the dealers have already paid half of it, and the remaining 9 would
be a reasonable assessment of the damages for this breach.
I would dismiss the first defendants appeal against the judgment in favour of the finance company. I would allow in part
the first defendants appeal against the dismissal of his claim against the dealers in the third party proceedings, and I would give
him judgment against them for 39 damages for breaches of contracts.
45

UPJOHN LJ. I agree with the judgment which has just been delivered by Pearson LJ and only desire to add a few words of my
own. My Lord has very fully recited the facts and discussed the authorities and no further reference either to fact or to authority
is necessary.
The question whether there are any implied conditions, warranties or stipulations relating to the hiring (whether it be pure
hire or hire-purchase) of any vehicle, must depend on the circumstances of each case and, as my Lord has pointed out, it is not
possible to lay down a general rule applicable to every hiring. So much must depend on the facts of each case. This case, as I
shall show, is a very good illustration of it. In general on a hiring of an ordinary motor vehicle, whether a car, lorry or a van
which the parties are intending shall be let and hired for normal use on the road for ordinary purposes for which the vehicle
appears to be suited and capable, there are in my judgment, subject to any express terms as to state of repair or condition and so
forth, two stipulations to be implied in the contract of letting and hiring.
First, there is an implied stipulation that the vehicle hired corresponds with the description of the vehicle contracted to be
hired, or to put it in another way, the lender must lend that which he contracts to lend and not something which is essentially
different. Thus, as my Lord has pointed out, a tipping lorry plainly hired for use as such must be capable of self-propulsion to a
reasonable degree and must have a rear compartment capable of being mechanically tipped.
This implied stipulation is a fundamental implied term and breach of it at once gives the hirer the right, if he desires to do so,
to treat the contract as repudiated. Furthermore, being a fundamental term the lender cannot by clauses of exclusion or exception,
however widely phrased, exclude liability for this fundamental term for the simple reason that the law will not permit one of the
contracting parties to escape liability for failure to deliver that which he has contracted to lend by delivery of something which is
essentially different. The question whether or not the motor vehicle delivered complies with this fundamental obligation of the
lender is very largely a question of fact and degree and must depend on the circumstances of each case. Many factors will be
relevant; I attempt no exhaustive catalogue. On the one side its condition so far as apparent to the ordinary inspection by a
layman and its known defects will be relevant. On the other side, of course, most important of all are defects unknown to the
hirer at the date of delivery; then the impact of such unknown defects not merely on the roadworthiness of the car but on its
general condition and ability to perform the tasks for which it is hired; and in deciding whether a somewhat defective vehicle
complies with this fundamental term it is clearly relevant to have in mind the type and expense of repairs and the time required to
put it into reasonable condition. It must not be forgotten that persons resort to hire-purchase because they cannot afford to pay
cash down and may reasonably want the early use of the vehicle in order to earn the hire-purchase rent. Therefore if a large sum
or a long time is required to put a car into a reasonable state of repair that is a most material circumstance in assessing whether a
vehicle does or does not comply with this fundamental term.
The second implied stipulation is that the vehicle must be as fit for the purpose for which it is hired as reasonable skill and
care can make it. I would regard this not as a condition going to the root of the contract but as a stipulation in the nature of a
warranty, that is to say a stipulation breach of which does not thereupon automatically give the hirer the right to terminate the
contract if he desires to do so. It is akin to the stipulation of seaworthiness and I discussed this aspect of the matter in the recent
case of Hong Kong Fir Shipping Co Ltd v Kawasaki Kisen Kaisha Ltd ([1962] 1 All ER 474 at p 483; [1962] 2 QB 26 at p 63),
where I pointed out the danger of using the words condition and warranty except under the Sale of Goods Act, 1893.
Although in my judgment breach of this stipulation does not at once 46 entitle the hirer to treat the contract as at an end, it may
nevertheless in certain circumstances entitle the hirer to do so. Let me give one example: suppose that a vehicle which is to be
let on hire or hire-purchase over a period of two years is at the moment when it ought to be delivered minus four tyres and all its
sparking plugs and the carburettor is unserviceable; that would not entitle, I think, the hirer to treat the contract as repudiated; not
at all events if the lender offers to put on four tyres, the necessary number of sparking plugs and fit a new carburettor, all of
which could be done within a day; no doubt the hirer would be entitled to sue for damages for one days late delivery. On the
other hand if a car which is to be hired at 9 am sharp for the express purpose known to the lender of carrying the hirer 150 miles
to lunch in the country and back that same afternoon, is at 9 am in the defective condition which I have just mentioned, in my
judgment the defects would go to the root of the contract and would clearly frustrate it, for the defects could not be put right in
time to permit the hirer to reach his luncheon engagement in time. He would be entitled therefore to treat the contract as
repudiated if he so desired. It may be that it would also amount to a breach of the fundamental term mentioned earlier for the
lender has contracted to deliver a car capable of starting at 9 am sharp and time is of the essence. This shows that these two
implied stipulations may in some cases tend to merge. But essentially they are separate and distinct and a breach of the
stipulation of fitness alone may in proper circumstances give rise to a frustration of the commercial purpose of the contract which
entitles the hirer to treat it as repudiated by the lender and at an end. On the other hand liability for breach of this stipulation of
fitness when sounding in damages is one which may be excluded by appropriate clauses of exclusion or exception. Such a clause
of exclusion could not however prevent the hirer from claiming that the breach frustrated the contract.
I turn then to apply these propositions of law to the facts of this case. The vehicle was delivered to the hirer in a very
defective condition but Pearson LJ has very fully reviewed the facts and has shown how on those facts it is impossible to treat the
lender as being in breach of the fundamental condition which I have mentioned above. The hirer accepted and used the vehicle
for very heavy work for many weeks and would have gone on using it if he had not run out of money. On those facts I agree with
the lord justice and the county court judge entirely and that matter therefore passes from consideration
On the second implied stipulation (of fitness) it is possible that the hirer might originally have had some claim for damages
against the lender for the defective condition in which he found the vehicle but for the fact that by cl 3 of the agreement the
lender excluded liability as, in my judgment, he lawfully could. But another answer to this claim which depends on the facts of
the case, is, that the hirer never relied on any implied obligation of fitness by the lender when he took the vehicle. He knew of
the principal defects yet took the vehicle relying on the collateral agreement with the dealers to rectify those defects. He gave the
dealers two opportunities to repair the vehicle before he took it away and was then content to take it some 150 miles to the
Llanwern site in South Wales and even when he arrived there and then had trouble he was content to come to a further
arrangement with the dealers with regard to its repair and the latter in fact paid some 9 as a contribution to that repair. If a man
takes a vehicle with known defects and uses it for its ordinary purposes and relies on some collateral agreement with the dealer to
put the defects right he cannot in my view afterwards complain to the lender that the latter (who in fact had no knowledge of the
defects or opportunity of putting them right) has failed in some implied obligation to put it right. In law the answer must be that
the implied obligation is excluded for the hirer never relied on it. It is true that later on defects appeared in the motor vehicle but
that was only after the vehicle, itself a somewhat old vehicle, had been subjected to the most severe treatment at the hands of the
hirer. For these reasons I do not see how the 47 hirer can have any complaint against the lender for the breach of this implied
obligation (had it not been excluded) in the particular circumstances of this case.
I want to add one word on the lenders claim for damages in the light of the recent decision (to which I was a party) in
Financings Ltd v Baldock, which was decided since this case was argued. In this case before the learned county court judge
damages were assessed on the footing that the hirer had repudiated the hire-purchase agreement. An application was made to us
to amend the notice of appeal by raising some question on the measure of damages on the footing that there had been no such
repudiation but at that late stage we refused leave to amend. It follows therefore that this court and the learned county court
judge treated the case, whether rightly or wrongly, as one of repudiation by the hirer and therefore there is no conflict of opinion
between Financings Ltd v Baldock and this case.
As to the claim of the hirer against the dealers, I desire to say only that I concur entirely with the judgment of Pearson LJ. I
agree that this appeal must be dismissed against the finance company but allowed to the extent indicated by my Lord against the
dealers.

ORMEROD LJ. I agree with the judgments which have been delivered and would also dismiss the appeal against the plaintiff
finance company but allow the appeal against the third party (the dealers) to the extent indicated by Pearson LJ. In my judgment
it is a fundamental term of a contract of hiring of this kind that the goods hired must correspond with the description of the goods
which are contracted to be hired. In this case, as Pearson LJ has pointed out, the goods in question consisted of a tipping lorry,
that is, a vehicle capable of self-propulsion and of being mechanically tipped. There can be little doubt that the vehicle complied
with these requirements. Not only was it driven from Manchester to South Wales but was driven to and from the quarry there to
the place where the stone was tipped. It is important to remember also that the first defendant was given an opportunity of trying
out the lorry before he made the agreement and knew, of course, that it was far from being a new one. It is true that the tipping
gear was not as efficient as it might have been when it was first taken over, but the defect was one which was easily remedied and
could not be said to amount to a breach of a fundamental term.
I agree with my brethren, however, that the first defendant has a claim for damages against the dealers in respect of their
failure to comply with their agreement to carry out certain repairs to the vehicle. This aspect of the matter has already been fully
considered by them and no useful purpose will be served in discussing it further.

Appeal dismissed as against plaintiffs; allowed in part as against the third party; judgment for first defendant against the third
party for 39.

Solicitors: J E Lickfold & Sons agents for Taylor, Hindle & Rhodes, Manchester (for the plaintiffs); Kanter, Jules & Co agents for
Boote, Dutton & Whittaker, Manchester (for the first defendant); Gibson & Weldon agents for Grover, Smith & Moss, Manchester
(for the second defendant and third party).

Henry Summerfield Esq Barrister.


48
[1963] 2 All ER 49

Lepre v Lepre
FAMILY; Other Family

PROBATE, DIVORCE AND ADMIRALTY DIVISION


SIR JOCELYN SIMON P, JULY 13, 1961, BEFORE COLLINGWOOD J
3, 4, 5 DECEMBER 1962

Nullity Foreign decree Recognition Contrary to justice Maltese husband marrying English wife at register office in
England Husband a Roman Catholic Marriage valid by English law but voidable by Maltese law Decree of nullity granted
to husband in Malta Whether decree recognised by English court.
L was a Maltese by birth and a Roman Catholic by religion. He came to England and in April, 1955, went through a civil
ceremony of marriage at a register office in Portsmouth with the wife who was an Englishwoman with whom he had lived for
some years. In May, 1955, the parties, together with their child went to Malta, but in 1956 they separated and the wife returned to
England with the child. In 1957 the wife obtained in the magistrates court orders for the maintenance of herself and for the
custody and maintenance of the child. The wifes subsequent application in Malta for registration and confirmation of these
ordersa was adjourned by the Maltese court, L contending that he was not married to the wife since, being a domiciled Maltese of
the Roman Catholic faith, he could not contract a valid marriage in a register office. On 28 March 1960, the Maltese court
pronounced at the suit of L a decree of nullity on the ground that the marriage in England had failed to comply with the canon
law. The wife now petitioned for a declaration that the decree of nullity by the Maltese court was invalid and of no effect, and for
a divorce on the grounds of Ls cruelty and constructive desertion, herself seeking the exercise of the courts discretion in respect
of her own adultery.
________________________________________
a See Maintenance Orders (Facilities for Enforcement) Act, 1920, s 3; 11 Halsburys Statutes (2nd Edn) 857

Held (i) In determining whether or not there was a subsisting marriage to dissolve, the court must necessarily determine
whether the Maltese decree of nullity should be recognised as valid; accordingly there was jurisdiction under RSC, Ord 25, r 5, to
make the declaration sought (see p 53, letter g, post).
Har-Shefi v Har-Shefi ([1953] 1 All ER 783) considered.
(ii) unless the decree of the Maltese court were vitiated by fraud or as being contrary to natural justice, it should be accepted
by the English court as conclusive (see p 56, letter i, to p 57, letter a, post), for the following reasons
(a) because the defect of the marriage in England was by English law a matter of formalities and was thus referable to the
lex loci celebrationis, viz, to English law, and by that law the marriage was valid, with the consequence that at the date of the
commencement of the Maltese nullity suit (which was the relevant point of time) the wife was married and had acquired, and
retained, a Maltese domicil, and accordingly the Maltese court was the court of the common domicil and its decree was valid (see
p 55, letter c, post); moreover it appeared that in Maltese law the defect rendered the marriage voidable, not void, and indeed that
all relevant systems of law would conclude that the marriage was subsisting when the nullity suit in Malta began, so that the wife
was then domiciled with L in Malta (see p 55, letter g, post).
Dictum of Lord Greene MR, in De Reneville v De Reneville ([1948] 1 All ER at p 61) applied.
(b) and because, even if the English marriage were void ipso jure, recognition ought to be accorded to the Maltese decree,
since in like circumstances the English court would assume jurisdiction in nullity if the petitioner alone were domiciled in
England, and, therefore, should concede the like jurisdiction 49 to the Maltese court, L being domiciled in Malta at the
commencement of the Maltese suit (see p 56, letter b, post).
Travers v Holley and Holley ([1953] 2 All ER 794) and Corbett v Corbett ([1957] 1 All ER 621) followed.
Ogden v Ogden ([19047] All ER Rep 86) doubted.
(iii) nevertheless the English court would not recognise the nullity decree of the Maltese court in the present case, because it
was unjust that a system of law should seek to impose as a condition of marriage in another country that the marriage should take
place according to the tenets of a particular faith (Formosa v Formosa, [1962] 3 All ER 419, applied; see p 57, letter g, and p 58,
letter a, post); the marriage was, therefore, subsisting and, cruelty and constructive desertion on Ls part being established, the
court would, in the particular circumstances of the case, exercise its discretion in favour of the wife and grant her a decree nisi of
divorce.

Notes
As to the recognition of a foreign decree of nullity where the marriage was celebrated in England, see 7 Halsburys Laws (3rd
Edn) 117, para 206; as to domicil as the test of jurisdiction, see ibid, 118, para 208, note (r); and for cases on the subject, see 11
Digest (Repl) 485, 486, 10981107.

Cases referred to in judgment


Apt (orse Magnus) v Apt [1947] 2 All ER 677, [1948] P 83, 177 LT 620, 27 Digest (Repl) 67, 459.
Bank of Hindustan, China and Japan, Re, Campbells Case, Hippisleys Case, Alisons Case (1873), 9 Ch App 1, 43 LJCh 1, 29
LT 524, 21 Digest (Repl) 225, 228.
Blamey v Blamey [1902] WN 138, 22 Digest (Repl) 528, 5902.
Chapelle v Chapelle [1950] 1 All ER 236, [1950] P 134, 11 Digest (Repl) 485, 1101.
Chetti v Chetti [190810] All ER Rep 49, [1909] P 67, sub nom Venugopal Chetti v Venugopal Chetti 78 LJP 23, 99 LT 885, 11
Digest (Repl) 460, 941.
Corbett v Corbett [1957] 1 All ER 621, [1957] 1 WLR 486, 3rd Digest Supp.
De Bono v De Bono [1948] 2 SALR 802.
De Reneville v De Reneville [1948] 1 All ER 56, [1948] P 100, [1948] LJR 1761, 11 Digest (Repl) 479, 1075.
Estin v Estin (1947), 334 US 541.
Formosa v Formosa [1962] 3 All ER 419, [1962] 3 WLR 1246.
Harrison v Harrison (1910), 54 Sol Jo 619.
Har-Shefi v Har-Shefi [1953] 1 All ER 783, [1953] P 161, [1953] 2 WLR 690, 3rd Digest Supp.
Igra v Igra [1951] P 404, 11 Digest (Repl) 518, 1324.
Kenward v Kenward [1950] 2 All ER 297, [1951] P 124, 11 Digest (Repl) 460, 942.
Merker v Merker [1962] 3 All ER 928, [1962] 3 WLR 1389.
Ogden v Ogden [19047] All ER Rep 86, [1908] P 46, 77 LJP 34, 97 LT 827, 11 Digest (Repl) 357, 260.
R v Wilkes (1769), 4 Burr 2527, 19 State Tr 1075, sub nom Wilkes v R, 4 Bro Par Cas 360, Wilm 322, 2 ER 244, 14 Digest (Repl)
375, 3674.
Ross Smith v Ross Smith (orse Radford) [1962] 1 All ER 344, [1962] 2 WLR 388.
Salvesen (or Von Lorang) v Austrian Property Administrator [1927] All ER Rep 78, [1927] AC 641, 96 LJPC 105, 137 LT 571,
11 Digest (Repl) 478, 1069.
Travers v Holley and Holley [1953] 2 All ER 794, [1953] P 246, [1953] 3 WLR 507, 3rd Digest Supp.
50
Vassallo v Vassallo [1952] SASR 129.
Williams v North Carolina (1942), 317 US 287.

Petition
In this case the wife, Hilda Lepre, by petition presented on 22 December 1960, sought (i) a declaration that a decree of nullity
pronounced by the Civil Court of Malta on 28 March 1960, [in respect of her marriage in a register office in Portsmouth to the
respondent Joseph Lepre] was invalid and of no effect in the United Kingdom and (ii) a decree of divorce on the ground of the
husbands cruelty and desertion. No prayer for discretion was included in the petition at that time. The husband acknowledged
service and the suit came before Collingwood J on 13 July 1961, when the case was adjourned for argument by the Queens
Proctor. On 16 October 1961, the Queens Proctor was granted leave to intervene in the suit and by his plea dated 25 October
1961, the Queens Proctor alleged that the wife had attempted to obtain a decree contrary to the justice of the case by her failure
to disclose that since 1 January 1957, she had lived and cohabited and frequently committed adultery with one Albert Voitasik.
By her answer to the plea the wife admitted that on five or six occasions she had committed adultery with Albert Voitasik, but
otherwise she denied the allegations in the plea; and she asked for the discretion of the court to be exercised in her favour and
filed a discretion statement. Collingwood J was unable through illness to resume the hearing of the case which now came before
Sir Jocelyn Simon P, His Lordship having made an order under RSC, Ord 72, r 4, nominating himself to try the case. The facts
appear in the judgment.

W A L Raeburn QC and F R N H Massey for the wife.


Brian T Neill for the Queens Proctor.
The respondent did not appear and was not represented.

5 December 1962. The following judgment was delivered.

SIR JOCELYN SIMON P. Here is another of those distressing cases where a domiciled Maltese of the Roman Cathoic faith has
married in a register office. The petitioner, whom I will call the wife prays, first, for a declaration that a decree of nullity of her
marriage pronounced by the Civil Court of Malta on 28 March 1960, is invalid and of no effect in the United Kingdom; secondly,
that her marriage may be dissolved on the ground that the respondent, whom I will call the husband, has treated her with
cruelty and has deserted her; and, thirdly, for ancillary relief. By amendment she prays that the courts discretion to pronounce
the divorce may be exercised in her favour.
The wife was born in this country and has lived most of her life here. She was married first to a Mr Shaw, a domiciled
Englishman. The husband was born and brought up in Malta, and baptised according to the rites of the Roman Catholic Church.
Between 1925 and 1926 he was employed at the naval dockyard at Malta. At some time between 1926 and 1944 he came to the
United Kingdom, and had a cafe in Cardiff; but he got into trouble with the police and left the country. Between May, 1944 and
July, 1946, he was in Malta, and then in Alexandria, and then back in Malta, in each case being employed by the royal navy in
their dockyard establishments. In August, 1946, he came to this country again, and started working at the Portsmouth dockyard.
He there met the wife, who was by then living apart from Mr Shaw, and within a short time the husband and the wife had set up a
joint establishment. On 24 November 1947, a child was born, called David. Some time before 27 April 1951, the husband
abandoned the wife for a time, and she obtained an affiliation order against him: however, they soon came together again. On 17
March 1952, the wife obtained a decree absolute of divorce from Mr Shaw. She and the husband did not, however, get married at
once, though they continued to live together. In April, 1955, the parties decided to go to Malta. The wife had for years been
asking the husband to marry her, and he now agreed. On 16 April 1955, the wife and the husband went through a ceremony of
marriage at a register office at Portsmouth. If English matrimonial 51 law were the only matter to be considered, there could be
no question but that this was a valid marriage. However, even before the wife and the husband had left this country he was
telling her that they were not really married, since he was a Roman Catholic and his church did not recognise a marriage in a
register office.
The parties, together with David, arrived in Malta on 20 May 1955. The husband continued to taunt the wife that she was
not really married. Nor, according to her account, was that the sum of his ill-treatment of her. He repeatedly abused her, accused
her falsely of immoral conduct and struck her. In September, 1956, he told the wife that since she did not like life in Malta, he
would pay her fare and Davids to England, hand her 100 when they got to the airport to give her a start in England, and pay her
2 a week for David when she got there. The wife jumped, as she said, at the chance of returning to England. The husband did
pay the fares; but when the parties arrived at the airport he refused to give the wife any money. Its a good riddance to both of
you, he said. Thats the last you will see of me. The wife has never heard from him since, though he did send two letters to
David. He has provided no support for either of them.
On 2 August 1957, the wife complained to the Portsmouth magistrates that the husband had deserted her, and they made a
maintenance and custody order in her favour. That order, however, could not be enforced against the husband until it had been
registered and confirmed in the Malta courts. The wife, therefore, through local agents, started proceedings to procure this. On 2
December 1957, the Maltese court adjourned the wifes application, the husband having alleged that he was not married to her:
his ground for that assertion was that he was a Roman Catholic Maltese domiciled in Malta and by his personal law he could not
contract a valid marriage anywhere in the world except in the manner stipulated by the canon lawthat is, within the context of
the present case, before the local parish priest or his nominee in a Roman Catholic church; the register office marriage was
therefore invalid.
The Maltese court, as I have said, adjourned the wifes application and directed the husband to take proper nullity
proceedings before the appropriate court to have his marriage declared void. On 16 December 1957, the husband issued his writ
in the civil court of Malta praying for a decree of nullity on the ground that his marriage did not comply with the canon law. The
court appointed a procurator and an advocate to represent the wife, and her evidence on certain matters was taken on commission
in this country. On 29 October 1959, before the proceedings had concluded, the Legitimacy Act, 1959, came into force here. By
that Act the child David became legitimated, provided that the husband had an English domicil at the time that he married the
wife. On the evidence adduced before me I was not satisfied that he had ever acquired an English domicil; but it is unnecessary
and indeed inappropriate to come to a final conclusion on this, affecting as it may Davids status but having no materiality to my
mind as to whether the marriage is a subsisting one. Even if, contrary to my present judgment, the husband had acquired an
English domicil of choice, I am satisfied that he had reverted to his Maltese domicil of origin by the time that the proceedings
there had started.
On 28 March 1960, the Maltese court pronounced a decree of nullity on the ground that the marriage had failed to comply
with the canon law. It held that the husband was at all times domiciled in Malta, and that by the law of his Maltese domicil he
had an incapacity to contract a marriage otherwise than in accordance with canon law. In view of the emphasis placed by counsel
for the wife on the effect of the Maltese decree on the status of the child, I may say that even if David had been legitimated by the
Act of 1959, I am by no means convinced that he would be bastardised by the Maltese decree. The ceremony, though devoid in
the eye of Maltese law of permanent legal effect as between the husband and the wife, may yet by that law have amounted to a
putative marriage so as to legitimate their offspring. That appears to have been the view of the Maltese court in the comparable
case of Chapelle v Chapelle, the Maltese proceedings 52 relating to the child being referred to in a footnote to an article by Mr J
K Grodecki in (1958) 74 LQR at p 233, n 50.
[His Lordship then referred to the pleadings of the wife and of the Queens Proctor and continued:] A preliminary question
arises as to the jurisdiction. There is no difficulty about the prayer for divorce. The wife was resident in England at the
commencement of the proceedings and had then been continuously resident here for over three years; the husband was then and
is now domiciled in Malta. I therefore have jurisdiction by the Matrimonial Causes Act, 1950, s 18(1)(b). But that provision,
though embracing proceedings for nullity, does not extend to proceedings for a declaration under RSC, Ord 25, r 5, as to the
international efficacy of a foreign judgment of nullity. If I am to make such a declaration I must seek jurisdiction on some other
than the statutory ground. In Har-Shefi v Har-Shefi the Court of Appeal held that the court had jurisdiction to make a declaration
under RSC, Ord 25, r 5, where the party seeking such a declaration is domiciled in England. Jurisdiction is, however, tested on
the assumption that the suitor is entitled to the relief claimed. Thus in Har-Shefi v Har-Shefi the wife petitioner was praying for a
declaration that the marriage had been validly dissolved by a foreign court: if successful she would have been domiciled in
England. In Merker v Merker, the foreign decree of nullity for recognition of which the petitioner was praying would have had a
similar effect. But if the wife here is entitled to the declaration she seeks, namely, that the Maltese decree was inefficacious to
annul her marriage, she was not domiciled here at the commencement of this suit, nor is she so domiciled today: she was and is
dependently domiciled in Malta. It may be thought artificial that jurisdiction to declare whether a foreign judgment is to be
recognised should depend on whether the petitioner is putting the claim in the affirmative or the negative; and there seems much
to be said for the contention, persuasively argued by Mr J A C Thomas b and by Mr I M Sinclairc, that the court has jurisdiction to
make such a declaration where the party seeking it is no more than resident in England, though no doubt the discretion to do so
would be exercised with caution.
________________________________________
b (1953), Vol 2 International and Comparative Law Quarterly, p 444
c (1953), 30 British Year Book of International Law, p 524

It is, however, unnecessary to decide that in the present case, as I am satisfied that I have jurisdiction on another ground. I
cannot proceed to consider whether the wife should be granted a decree of divorce without ascertaining whether there is a
subsisting marriage to dissolve, and this involves determining whether the Maltese decree should be recognised as an effective
judgment of nullity. In my view, where determination of the validity of a foreign judgment is a necessary step in proceeding to
adjudication on a matter within the jurisdiction of the court, the court has jurisdiction under RSC, Ord 25, r 5, to make a
declaration on such validity. It is then doing no more than to give formality to what is inherent in and clearly deducible from a
judgment which the court has jurisdiction to pronounce, so as in any event to constitute res judicata: see Alisons Case, per
Mellish LJ ((1873), 9 Ch App at p 25).
I have, therefore, to consider initially whether the decree of the Maltese court of 28 March 1960, should be recognised in
England as annulling the marriage. That itself raises two questions: first, should such a decree in principle be recognised in the
absence of fraud or offence against our notions of justice; secondly, does the decree in the present case in fact offend against our
notions of justice?
On the first question differing views of great weight have been expressed. On the one side stand Willmer J, in Chapelle v
Chapelle and Lord Denning MR and Donovan LJ, in Formosa v Formosa: they would not recognise the decree as effective to
annul the marriage. On the other side are Pearson LJ, 53in Formosa v Formosa, Herbstein J, in De Bono v De Bono, Reed J, in
Vassallo v Vassallo and the distinguished academic figures who have commented on Chapelle v Chapelle: they hold that the
decree ought to be recognised. Those who hold the first view argue in this way: the Maltese court can make a binding and
conclusive decree annulling the marriage if both parties were domiciled in Malta at the commencement of the suit there (Salvesen
(or Von Lorang) v Austrian Property Administrator); the Maltese decree, however, declared the marriage void; the wife,
therefore, never acquired the husbands Maltese domicil by operation of the law, but retained her English domicil throughout; it
follows that the decree of the Maltese court was not a decree of the common domicil and was not binding and conclusive. Those
who take the contrary view, that we should recognise the decree, retort that such a refusal to recognise the Maltese decree as
effectively annulling the marriage involves that the parties remain married in the eye of English law; the wife had thus at the
commencement of the Maltese proceedings a domicil in law dependent on her husbands, which was then admittedly Maltese;
both parties were, therefore, domiciled within the jurisdiction of the Maltese court, which in consequence had jurisdiction over
their marriage. The views expressed by the Court of Appeal in Formosa v Formosa were obiter on this part of the case. I must,
therefore, proceed to give my own judgment guided but not governed by what has been said elsewhere.
The apparent dilemma is, in my view, to be resolved by, first, carefully isolating the legal system to which reference should
be made in order to ascertain whether the Maltese court had a conclusive jurisdiction, and, secondly, bearing constantly in mind
the crucial time for invoking such legal system. That time is unquestionably the commencement of the Maltese proceedings. The
husband being then domiciled in Malta, where was the wife domiciled? This depends on whether the marriage was void or
voidable or valid in the eye of the legal system which should be invoked: if valid or voidable, the wife remained married to the
husband until the pronouncement of the decree of nullity, and therefore took his domicil until that event; if void, she had no legal
tie with the husband and had her own domicil throughout: see De Reneville v De Reneville, per Lord Greene MR ([1948] 1 All
ER at p 60; [1948] P at p 111). In his judgment in that case, in which Somervell LJ concurred, Lord Greene MR also indicated,
first, the meaning of void and voidable in this connexion and, secondly, what legal system should be invoked to determine
whether the marriage was void or voidable. As to the former he said ([1948] 1 All ER at p 60; [1948] P at p 111):

A void marriage is one that will be regarded by every court in any case in which the existence of the marriage is in
issue as never having taken place and can be so treated by both parties to it without the necessity of any decree annulling it:
a voidable marriage is one that will be regarded by every court as a valid subsisting marriage until a decree annulling it has
been pronounced by a court of competent jurisdiction.

And again ([1948] 1 All ER at p 62; [1948] P at p 115):

it would be for the English court, after hearing evidence of French law, to decide whether in French law the
marriage was void or voidable, not merely in a verbal sense, but in the sense of the words as understood in this country,
that is, as indicating or not indicating, as the case might be, that the marriage would be regarded in France as a nullity
without the necessity of a decree annulling it.

Lord Greenes reference to French law in the latter passage results from his conclusion as to the legal system that should be
invoked; as to that he said ([1948] 1 All ER at p 61; [1948] P at p 114):
54

the question whether the marriage is void or merely voidable is for French law to answer. My reasons are as
follows. The validity of a marriage so far as regards the observance of formalities is a matter for the lex loci celebrationis.
But this is not a case of forms. It is a case of essential validity. By what law is that to be decided? In my opinion, by the
law of France, either because that is the law of the husbands domicil at the date of the marriage or (preferably, in my view)
because at that date it was the law of the matrimonial domicil in reference to which the parties may have been supposed to
enter into the bonds of marriage.

There is no question but that our law characterises the marital defect alleged in the present casethat the ceremony did not take
place in a Roman Catholic church in the presence of a priestas a matter of formalities. Its legal result, says Lord Greene, must
in these circumstances be referred to the lex loci celebrationis; that is, to English law. English law says without any doubt that
the marriage was neither void nor voidable but valid. The wife and the husband were, therefore, married at the commencement
of the Maltese proceedings; the wife had acquired and retained the husbands Maltese domicil; the decree was a judgment of the
court of common domicil and should, therefore, be recognised here as binding and conclusive.
De Reneville v De Reneville was concerned with the jurisdiction of the English court to pronounce a decree of nullity.
Where the jurisdiction of a foreign court is in question it may reasonably be argued that we should pay regard not solely to our
own characterisation of the nature of the defect in question, but also to how it is characterised by the foreign court. The same
result, however, ensues. I had no expert evidence of Maltese law; but it is clear from the terms of the Maltese judgment that they
characterised the defect not as one of formalities but as affecting the husbands capacity to marry, and thus referable to the law of
his Maltese domicil. I conclude, however, from the proceedings of the Maltese courts that in Maltese law the marriage was
voidable and not void, in the senses indicated by Lord Greene MR. The wifes maintenance proceedings in Malta were not
dismissed out of hand on the ground that she was not the wife of the husband; on the contrary, they were adjourned so that the
husband could question the validity of the marriage in separate and appropriate proceedings: that, indeed, seems to be a common
feature of these cases. The marriage thus being voidable and not void in Maltese law, the parties had a common domicil in Malta
at the commencement of the Maltese proceedings, and the decree of nullity is binding and conclusive. In truth, there is no real
difficulty in the present case: all the systems of law to which reference could conceivably be madethe lex loci contractus, the
lex domicilii of the husband, the leges domicilii of the wife, the lex causae and the lex foriconcur at the time when the Maltese
proceedings started that the wife was married to the husband and domiciled with him in Malta. If that is so, then, whatever the
words of the decree, it is inadmissible to relate it back so as to destroy the basis of the jurisdiction to make it. To cite again Lord
Greene MR, in De Reneville v De Reneville ([1948] 1 All ER at p 60; [1948] P at p 111):

The fact that a domicil has been acquired by reason of a voidable marriage is a fact the existence of which cannot be
undone by a declaration of nullity.

A fortiori if the marriage is valid by our choice of law rule. To allow otherwise involves a circular argument from which there is
no escape.
But even if this marriage were void ipso jure, so that the husband alone was domiciled in Malta at the start of the
proceedings there, in my judgment we should still accord recognition to the Maltese decree. In the case of a marriage void ipso
jure, such as a marriage fundamentally defective as to formalities, the English court assumes jurisdiction in nullity if the
petitioner alone is domiciled in England: 55De Reneville v De Reneville; Apt (orse Magnus) v Apt; Kenward v Kenward, to cite
only authorities in the Court of Appeal. Moreover, in such circumstances we purport to operate on the status not only of the
petitioner who is domiciled within the jurisdiction but also of the respondent who is not; it is for this reason that we insist that he
or she should be made a party to the proceedings, so as to be bound by our decree. If we ourselves claim a ground of jurisdiction
we must concede a similar ground of jurisdiction to foreign courts: Travers v Holley and Holley, Corbett v Corbett. Therefore
even if the wife were, contrary to my view, domiciled in England at the start of the Maltese proceedings by reason of the nullity
of the marriage, we should none the less concede recognition to the Maltese decree, because we would regard ourselves as
competent to pronounce a decree of nullity of a marriage void ipso jure were the husband domiciled in England and the wife in
Malta. In as far as Ogden v Ogden appears to be to the contrary, it is in my judgment in conflict with the reasoning of the later
authorities in the Court of Appeal to which I have referred and must be taken to be confined to its particular factsif, indeed, it
can today stand at all in the light of the House of Lords decisions in Salvesen (or Von Lorang) v Administrator of Austrian
Property and Ross Smith v Ross Smith (orse Radford). Furthermore such assumption and concession of a binding jurisdiction in
nullity based on the domicil of one party only seems to me to accord with principle. A judgment declaratory of the status of some
subject-matter legally situated within the national and jurisdiction of the court pronouncing the judgment constitutes a judgment
in rem which is universally conclusive. The husband was legally situated within the jurisdiction of the Maltese court because he
was domiciled in Malta. That court was, therefore, competent to declare his status by a decree of nullity; such a decree
constitutes a judgment in rem, and should be regarded universally as conclusive as to his status, that is to say, that he is
unmarried. Counsel for the wife conceded that for the purpose of criminal proceedings in this country we would be bound so to
regard him; but not, he said, for the purpose of matrimonial proceedings, because the wife (on the present hypothesis) was not
domiciled in Malta, so that her status was not within the competence of the Maltese court. The wife therefore, it is claimed,
remains married to the husband, even though he is not married to her: the concept is no more difficult of acceptance than a
finding that a respondent has committed adultery with a co-respondent, but not he with her. Such schizoid situations reflect little
credit on the law, though the latter one is reasonably based on differential admissibility of evidence. But I cannot conceive how
our courts could accept as conclusive the decree of a competent court of the husbands domicil that he is unmarried and, at the
same time, purport to dissolve a marriage to which he is the other party. Moreover, there is high persuasive authority to suggest
that this contention for the wife is not correct: see Williams v North Carolina, a decision of the Supreme Court of the United
States, where comparable recognition problems arise after divorce owing to their rule that a wife retains during marriage a
domicil independent of her husbands. Incidentally, further authority from the same eminent source indicates that recognition of
the nullity decree of the court of the husbands domicil only ought not necessarily to exclude our courts from enforcing in favour
of the wife in appropriate circumstances financial obligations arising out of the marriagefor example, under a prior
maintenance order made in this country: see Estin v Estin.
Therefore, in my judgment, we should accept the Maltese decree as binding and conclusiveprimarily as a decree of the
court of the common domicil at the commencement of the proceedings there, though alternatively as a decree of 56 the husbands
domicil alone at that timeprovided always that it is not vitiated by fraud or contrary to natural justice.
That brings me to the second main issue in this part of the case. I confess that I approach it with some misgiving. We are
concerned here with the decree of a superior court of a Commonwealth country. Its procedure was manifestly solicitous of the
forensic interests of the wife. The code of law applied was an ancient and honoured one. Moreover, limping marriages are
themselves inherently liable to cause hardship and injustice: suppose, for example, the wife had remarried and had offspring in
reliance on the Maltese decreeor, for that matter, the husband. The refusal to recognise an otherwise binding foreign judgment
or rule of law on the ground that it is manifestly unjust is nowadays put as a matter of discretion.

But discretion, when applied to a court of justice, means sound discretion guided by law. It must be governed by rule,
not by humour: it must not be arbitrary, vague, and fanciful; but legal and regular.

See R v Wilkes, per Lord Mansfield ((1769), 4 Burr at p 2539). In short, there must be a reasonable consistency in its exercise. In
Corbett v Corbett, Barnard J recognised a foreign decree of nullity based on two grounds, one apparently identical with that
which constituted the defect in the present case, the otherthat a Jewess was incapable of marrying out of her faithdifficult to
differentiate in principle: see also Igra v Igra. But in Formosa v Formosa, the Court of Appeal, though not expressly adverting
to Corbett v Corbett, was unanimous that a decree pronounced by a Maltese court on a ground identical with that in the present
case and in largely similar circumstances offended so grossly against our notions of justice that it should not be recognised. It is
true that the present case differs in certain details from Formosa v Formosa. The husband here was not proved to have acquired
an English domicil at the time of the marriage, and I am not satisfied that the child would be adversely affected by recognition of
the decree. Corbett v Corbett was similar to the present case in the latter respect, and possibly in the former. But to differentiate
the present case and Corbett v Corbett from Formosa v Formosa on these grounds would be, in my opinion, to introduce idle
distinctions into the law and throw it into confusion. Not least in matters relating to marriage is it incumbent on the law to speak
with a clear, consistent and unequivocal voice. In truth, I do not believe that it was mere cumulation of detail which impelled the
Court of Appeal to their conclusion. I think that the crux of their decision was that it was an intolerable injustice that a system of
law should seek to impose extra-territorially, as a condition of the validity of a marriage, that it should take place according to the
tenets of a particular faith. As Donovan LJ put it ([1962] 3 All ER at p 423):
It ill accords with present-day notions of tolerance and justice that a wife, validly married according to our law, should
be told by a foreign court that she is a mere concubine and her children bastards, simply on the ground that her husband did
not marry her in the church of a particular religious denomination.

I cannot believe that the effect on the status of the children of the union was a decisive factor in the mind of Donovan LJ nor the
further element which he took into account ([1962] 3 All ER at p 423), namely, that the decree was

pronounced in favour of a husband who deserted her and his children and has left them to be supported by the
taxpayer in this country.

In any event that is common to the present case. Just as in Chetti v Chetti, Sir Gorell Barnes P, refused to give effect to an
incapacity to marry outside his caste or religion imposed extra-territorially on the husband by the law of 57 his domicil, so, I
think, the Court of Appeal discerned in Formosa v Formosa an attempt by Maltese law to impose an analogous incapacity based
on creed: they would refuse to recognise the incapacity, so they refused to recognise the domiciliary decree founded on it. If that
is so, the present case cannot be distinguished; and I am bound to hold that the Maltese decree of nullity, although on general
jurisdictional grounds conclusive, should not be accorded recognition because it offends intolerably against the concept of justice
which prevails in our courts.
It follows that the marriage is valid and subsisting. The wife petitions to dissolve it on the ground of the husbands cruelty
and desertion. As for cruelty, I accept her evidence. It is corroborated by an affidavit sworn by Mrs Zarb, the husbands sister,
who lives in Malta: though not entitled in this suit, it bears the stamp of the Government of Malta, and I therefore admitted it on
the authority of Blamey v Blamey, and the cases there cited. Mrs Zarb says that the husband kept the wife so short of money in
Malta that she (Mrs Zarb) used to have to make clothes for her and lend her money even for her public transport fares. On one
occasion she saw the wife in tears and bruised. Just before the wife left for England Mrs Zarb was told by the husband: It is
better that she should leave because if she remains here I shall suffocate her, meaning, I suppose, strangle her. Mrs Zarb also
bears out that the husband sent the wife back to England without a penny. Having also in mind what happened after the parting, I
am satisfied that the husband treated the wife with cruelty.
As for desertion, I have no difficulty in drawing the inference that the husband intended to drive the wife from cohabitation.
In so far as there may have been a residual consent by the wife to the separation, in my view it was procured and nullified by the
fraudulent misrepresentation of the husband as to his intentions of financial support: compare Harrison v Harrison. I, therefore,
hold that the husband deserted the wife and has remained in desertion up to the date of the petition.
There remains the difficult problem of the exercise of the courts discretion in favour of the wife. When she originally filed
her petition, she did not pray for discretion at all. When she did file her discretion statement it was far from a full disclosure of
her adultery. Even in the witness-box she told untruths about it, and it was only as a result of close questioning that she came
finally to disclose the adultery which is now set out in the amended discretion statementwith another man as well as Mr
Voitasik, and a more extensive course of adultery with Mr Voitasik than she admitted at first. The wife is a stupid woman, and I
have also no doubt that one of the reasons for her non-disclosure was that she is bitterly ashamed of her misconduct. I think that
she has now disclosed its full extent. She is anxious to marry Mr Voitasik, and he wants to marry her. This marriage has been
unequivocably repudiated by the husband, and no useful purpose would be served by keeping it in further subsistence. I am,
therefore, though I confess with some hesitation, prepared to exercise the discretion of the court in favour of the wife and to grant
her a decree nisi of divorce.

Decree nisi.

Solicitors: H M Gammans, Portsmouth (for the wife); Queens Proctor.

A T Hoolahan Esq Barrister.


58
[1963] 2 All ER 59

Good v Parry
CIVIL PROCEDURE

COURT OF APPEAL
LORD DENNING MR, DANCKWERTS AND DAVIES LJJ
26, 27 FEBRUARY 1963

Limitation of Action Acknowledgment Debt Claim Limitation Act, 1939 (2 & 3 Geo 6 c 21), s 23(4).

For there to be an acknowledgment of a claim within s 23(4) of the Limitation Act, 1939 a, there must be an admission that
there is a debt or other liquidated amount outstanding and unpaid (see p 61, letter e, and p 62, letters c and e, post).
________________________________________
a Section 23(4) is set out at p 61, letter b, post

In 1949 a landlord let premises to a husband and wife. The husband died and the wife carried on the tenancy. In 1962 the
landlord brought an action claiming more than six years arrears of rent. In 1957 the tenants agent wrote a letter to the landlord
during negotiations for the proposed purchase of the premises by the tenant, in which it was stated that the question of
outstanding rent can be settled as a separate agreement as soon as you present your account. The landlord contended that this
acknowledged the claim for arrears of rent for the purposes of s 23(4) of the Limitation Act, 1939 b, with the consequence that
time would not begin to run until the date of the letter.
________________________________________
b Section 23(4) is set out at p 61, letter b, post

Held The letter was not an acknowledgment within s 23(4) of the Limitation Act, 1939, because there was in it no admission of
any defined amount of rent or of any amount that could be ascertained by calculation being due, nor, indeed, any admission that
there was such a debt in fact (see p 62, letters a, d and e, post).
Skeet v Lindsay ((1877), 2 Ex D 314), and Langrish v Watts ([1903]1 KB 636), considered.
Appeal dismissed.

Notes
As to an action to recover a debt, see 24 Halsburys Laws (3rd Edn) 298, para 590, and as to the form of acknowledgment of debt,
see ibid, pp 299, 300, para 593.
For the Limitation Act, 1939, s 23, see 13 Halsburys Statutes (2nd Edn) 1184.

Cases referred to in judgments


Jones v Bellgrove Properties Ltd [1949] 2 All ER 198, [1949] 2 KB 700, 65 TLR 451, 93 Sol Jo 512, 2nd Digest Supp.
Langrish v Watts [1903] 1 KB 636, 72 LJKB 435, 88 LT 443, 51 WR 503, 19 TLR 359, 32 Digest 370, 541.
Skeet v Lindsay (1877), 2 ExD 314, 36 LT 98, 41 JP 456, 25 WR 322, sub nom Skeat v Lindsay, 46 LJQB 249, 32 Digest 373,
566.
Whitcomb v Whiting (1781), 2 Doug KB 652, 1 Sm LC 635, 32 Digest 353, 362.

Appeal
This was an appeal by the landlord from a judgment of His Honour Judge Potter, given on 5 November 1962, at the Bow County
Court, awarding the landlord 156 0s 7d arrears of rent due from the tenant, but refusing to enter judgment for the landlord for
the amount of arrears claimed, 368 16s 9d on the ground that the difference was statute-barred. The facts are set out in the
judgment of Lord Denning MR.
The cases noted belowc were cited during the argument in addition to those referred to in the judgment.
59
________________________________________
c Tanner v Smart (1827), 6 B & C 603, Moodie v Bannister, (1859), 4 Drew 432, Green v Humphreys, (1884), 26 ChD 474

F Bresler for the landlord.


S O Olson for the tenant.

27 February 1963. The following judgments were delivered.

LORD DENNING MR. This is a claim for rent in arrear. The landlord, Mr Good, let premises to Mr and Mrs Parry as long ago
as 1949. The landlord says that he has not received any rent for very many years. Mr Parry died and his widow, Mrs Parry, who
was joint tenant with him, survived and carried on. Rent of 2 3s 8d per week, it is said, became payable and was never paid.
Eventually, on 6 March 1962, the landlord brought an action against the tenant claiming 604 14s 5d, the amount of rent in
arreard as at 12 February 1962. By way of defence, the tenant said that the cause of action for arrears of rent did not accrue
within six years before the action and was barred by s 2 of the Limitation Act, 1939. It is quite plain that that Act bars any rent
which accrued due more than six years before the action was brought, subject, however, to the question whether there has been an
acknowledgment within s 23 and s 24 of the Act of 1939. The landlord has put before the court a number of letters, of which
only one arises for consideration now, which, he says, was an acknowledgment such as to prevent the statute from running. It is a
letter of 6 September 1957, written by Lieutenant John Parry, RN, the son of the tenant and undoubtedly her authorised agent. I
need not read the whole of it. It says:
________________________________________
d The figures were substantially agreed figures, but they do not correspond with the period of letting and rent without some explanation. In
July, 1961, a sum of 535 8s had been paid in respect of six years rent ending in June, 1961, after deducting Sch A tax and a sum of 49 7s
10d in respect of repairs claimed by the defendant to be deductible. The Sch A tax deducted included 24 8s 3d for the tax year 195455,
which was statute-barred. In the county court the plaintiff recovered judgment for 156 7s made up of the 24 8s 3d and 131 12s 3d,
which latter sum consisted of 82 4s 5d arrears of rent and mesne profits since the date in June, 1961, to which rent had been paid, together
with the 49 7s 10d wrongly deducted for repairs. Thus the agreed sum of 368 16s 9d (see letter f, above), which was to be the amount
recoverable on the basis that there had been acknowledgment of the indebtedness for rent within s 23(4) of the Limitation Act, 1939,
included a substantial sum in respect of rent prior to June, 1955

Dear Mr. Good, Since I have not had the opportunity of seeing you this week to discuss 11 Scotland Road, I am
writing to try and arrive at some satisfactory arrangement. [He discusses the proposed purchase of the house and says:] I
have considered the surveyors letter and am prepared to offer 1,350 as recommended for this property, subject to
contract.

Then he goes on to use this one sentence about the rent: The question of out-standing rent can be settled as a separate agreement
as soon as you present your account. It is said on the landlords behalf that that is an acknowledgment such as to take the case
out of the statute, and that he can recover the amount in arrear. It is agreed that, if it is an acknowledgment, the judgment is to be
for the sum of 368 16s 9d.
The landlord relies on two cases before the Act of 1939, namely, the decision of Cleasby B, in Skeet v Lindsay, and the
observations of Vaughan Williams LJ, in Langrish v Watts ([1903] 1 KB 636 at p 641), where he said:

Looking at the common law cases on the subject, apart from any question of a claim in equity to an account, they
appear to show that, where a defendant in effect has said that he does not think that he owes so much as is claimed, but, if
vouchers are furnished, he will pay whatever is due, that is an acknowledgment of a debt subject to the business operation
of ascertaining the amount being gone through.

It is urged by counsel on behalf of the landlord that this case comes within the 60 words of Vaughan Williams LJ. The words
here mean, he says, that, when the account is presented, the tenant will pay whatever is due, subject to the business operation of
ascertaining the amount. He points out that, at a later stage in the correspondence, some similar operation was gone through.
The law, it seems to me, is now to be taken from the Limitation Act, 1939. Section 23(4) replaces all the old law on the
subject. It says simply this:

Where any right of action has accrued to recover any debt or other liquidated pecuniary claim and the person liable
or accountable therefor acknowledges the claim or makes any payment in respect thereof, the right shall be deemed to have
accrued on and not before the date of the acknowledgment or the last payment.

Section 24 goes on to say that an acknowledgment has to be in writing and signed by the party or his agent. Clearly here the
writing and the agency is satisfied. The whole question is whether, in this letter, there is an acknowledgment of the claim. The
words the claim are not, perhaps, very happy. A person may acknowledge that a claim has been made against him without
acknowledging any indebtedness. It is clear that what the Limitation Act, 1939, means is acknowledges the debt or other
liquidated pecuniary amount. The question is: Is there such an acknowledgment? This statute alters the old law. Previously, in
order to get a case out of the statute, a creditor had to show a writing from which there could be implied a promise to pay; then
his cause of action was on the promise and not on the previous debt, and, so long as the promise was within the six years, all well
and good: see the notes to Whitcomb v Whiting. Nowadays, as the result of this new Act, there is no necessity to look for a
promise, express or implied. There need only be an acknowledgment of a debt or other liquidated amount. That means, I think,
that there must be an admission that there is a debt or other liquidated amount outstanding and unpaid. Even though the debtor
says in the same writing that he will never pay it, nevertheless it is a good acknowledgment. In order to be an acknowledgment,
however, the debt must be quantified in figures or, at all events, it must be liquidated in this sense that it is capable of
ascertainment by calculation, or by extrinsic evidence, without further agreement of the parties. For instance: I admit I owe you
the sum shown in this rent book would be a perfectly good acknowledgment, for it only needs to be calculated. Again, in Jones
v Bellgrove Properties, Ltd, the balance sheet contained the acknowledgment: To sundry creditors 7,638 6s 10d. It was
possible by extrinsic evidence to sort out the various items in that lump sum, and it was held to be a sufficient acknowledgment.
But if the debt is not quantified and is not ascertainable without further agreement, then there is no acknowledgment sufficient to
satisfy the statute.
No doubt a promise in writing by a debtor to pay whatever sum is found due on taking an account is a good
acknowledgment today just as it was before the Act of 1939, provided always that the amount is a mere matter of calculation
from vouchers, or can be ascertained by extrinsic evidence, and is not dependent on the further agreement of the debtor. If Skeet
v Lindsay and Langrish v Watts are to be regarded as good law since the Act of 1939, it can only be on the ground that the amount
was a mere matter of calculation from vouchers. I must say for myself that I feel that Skeet v Lindsay stretched the law to the
uttermost. I doubt whether it was right there to hold, as Cleasby B, seems to have done, that there was an engagement absolutely
to pay.
I come back to the sentence in this case. I agree with the submission of counsel for the tenant, which was accepted by the
county court judge, that this sentence means there may be some rent outstanding and it can be made the subject of an 61
agreement as soon as you present your account. Such being the meaning of it, I am quite satisfied that there is no
acknowledgment, because there is no admission of any rent of a defined amount due, or of any amount that can be ascertained by
calculation. The amount is uncertain altogether. Nor can I regard it as a promise to pay whatever amount may be found due on
taking an account. The tenant clearly reserves the right to examine it and not to be bound except by separate agreement. I agree,
therefore, with the county court judge and would dismiss this appeal.

DANCKWERTS LJ. I agree. The question which the learned county court judge had to decide, and which we have to decide,
is an important one, and the matter has been very fully and carefully argued by counsel before us. I agree with the judgment of
Lord Denning MR and it is really only necessary for me to add a few words. The question depends on the application of only a
few words in s 23(4) of the Limitation Act, 1939, the person liable or accountable therefor acknowledges the claim. I agree
with Lord Denning MR in his interpretation that the claim means any debt or other liquidated pecuniary claim referred to in
the earlier part of the subsection.
The only question then is whether the final sentence in the letter of 6 September 1957, written by the tenants son and on her
behalf, satisfied those words in the statute. The sentence is: The question of outstanding rent can be settled as a separate
agreement as soon as you present your account. I find it impossible to come to the conclusion that that sentence is in any way
an acknowledgment of any debt. It is merely, as it seems to me, an admission that there may be some possible justified claim, but
no admission that there is such a debt in fact. Therefore, I have come to the same conclusion as Lord Denning MR and, in my
opinion, this appeal should be dismissed.

DAVIES LJ. I entirely agree with my Lords, and only refer to one sentence in the judgment of Judge Potter in the court below.
After a very careful review of the authorities, he said this: In my judgment the letter did not acknowledge the claim; it only
acknowledged that there might be a claim. I entirely agree with that part of the county court judges judgment, as well as what
has fallen from my Lords.
I agree that the appeal fails and should be dismissed.

Appeal dismissed.

29 March. Leave to appeal to the House of Lords refused.

Solicitors: Shepherd, Harris & Co (for the landlord); Attwater & Liell (for the tenant).

F Guttman Esq Barrister.


62
[1963] 2 All ER 63

Laurent v Sale & Co (a firm)


CIVIL PROCEDURE

QUEENS BENCH DIVISION


MEGAW J
24 JANUARY 1963

Champerty Defence Assignment of chose in action void for champerty Plaintiffs title to sue depending on the assignment
Whether champerty could properly be raised by way of defence.

Where an essential link in a plaintiffs alleged title to what he claims in an action is a champertous assignment, the illegality of
the assignment for champerty may properly be raised by way of defence (see p 65, letter e, post).
Martell v Consett Iron Co Ltd ([1955] 1 All ER 481) distinguished.
By documents of assignment in 1956, made in the French language but to which English law was declared to be applicable,
K and M purported to assign to L their rights under letters dated 3 July 1953, addressed to K and M by the defendants, in which
the defendants acknowledged irrevocable instructions to pay K and M respectively at stated figures against shipments of goods.
Each document of assignment stated that the consideration for the assignment was to be the payment, by L to K or M
respectively, of a fraction of the total amount which should be paid to L by virtue of the letters of 3 July 1953. The assignments
were made with the knowledge of both the parties to them that L would not be able to enforce claims under the letters unless he
took legal proceedings against the defendants, and with the intention that L should take such proceedings. In an action by L to
enforce the claims assigned to him,

Held The assignments were void for champerty and L had no title to sue.

Notes
As to the purchase of rights of action and the unenforceability of champertous agreements, see 1 Halsburys Laws (3rd Edn) 43,
para 85, and p 41, para 83; and for cases on the unenforceability of champertous agreements, see 1 Digest (Repl) 8083, 607
624, 8587, 644666.

Case referred to in judgment


Martell v Consett Iron Co Ltd [1955] 1 All ER 481, [1955] Ch 363, [1955] 2 WLR 463, 3rd Digest Supp.

Preliminary Issue
On 3 July 1953, the defendants, Sale & Co wrote two almost identical letters, one to Mr Marcelle in Belgium and one to Herr
Kraus in Hamburg. The letters read as follows:

We confirm that against shipment of 860 metric tons of copper wire from Antwerp in respect of [a named company],
we have received irrevocable instructions to pay you $10 [or, in the letter to Herr Kraus, $5] per metric ton shipped and
paid for.

It appeared that on the same date the defendants also wrote a similar letter to a Mr Zehnder in Geneva. On 23 April 1956, Mr
Zehnders solicitors (with whom the plaintiffs solicitor was at that time associated) wrote to the defendants solicitors stating that
they were instructed in connexion with a claim of Mr Zehnder under the defendants letter of 3 July 1953. On 24 April 1956, the
defendants solicitors wrote in reply denying liability. By a document, dated 25 June 1956, made in the French language but
stating that English law should apply thereto Herr Kraus purported to assign to the plaintiff, Ferdinand Laurent, the right to the
sum, namely $4,300, which he claimed to be due to him from the defendants by virtue of their letter dated 3 July 1953, in
consideration of a payment by the plaintiff to Herr Kraus of one quarter of the total amount which should be recovered. A similar
document, except that the amount of the claim was stated to be $8,600, was executed by Mr Marcelle on 8 November 1956, and
63 supplemented by a further document dated 12 June 1959. On 24 June 1959, the plaintiffs solicitor wrote enclosing notice of
the two documents of assignment to the defendants solicitors, and informed them that unless the claims were admitted and
settled proceedings would be instituted against the defendants forthwith. On 1 July 1959, the writ in the action was issued. By
his statement of claim indorsed on the writ the plaintiff claimed 6,450, being the equivalent of $8,600 and $4,300 due
respectively to Mr Marcelle and Herr Kraus and confirmed in writing by the defendants by letters dated 3 July 1953, addressed to
Mr Marcelle and to Herr Kraus. Paragraph 5 and para 6 of the particulars of the statement of claim were as follows:

5. On Nov. 8, 1956, the said Marcelle absolutely assigned to the plaintiff the said debt of $8,600 due to him from the
defendants by an assignment in writing and a deed of assignment expressed to be supplemental thereto dated June 12,
1959.
6. On June 25, 1956, the said Kraus absolutely assigned to the plaintiff the said debt of $4,300 due to him from the
defendants by an assignment in writing.

On 1 October 1959, the defendants delivered their defence, by para 5 of which they pleaded as follows:

5. The defendants do not admit the assignments referred to in paras. 5 and 6 of the statement of claim. If (which is
not admitted) there was any such assignment, the same was and is illegal and champertous, (a) as being an assignment of a
bare right of litigation; and further or in the alternative (b) in that it was an assignment in consideration of a promise by the
assignee to pay to the assignor a proportion of the amount recovered in proceedings to enforce the chose in action thereby
assigned.

By order of Master Clayton, for Master Grundy, dated 26 June 1962, the issue raised by para 5 of the defence was ordered to be
tried as a preliminary issue. His Lordship (Megaw J) found on the facts that the assignments were champertous. The case is
reported for his decision on the point whether the fact that the assignments were champertous could be raised as a defence to the
action.
N Inglis Jones for the plaintiff: First, an agreement is not champertous merely because it involves the transfer of a debt,
which is due to A, and provides that the consideration or part of the consideration of that transfer shall be that A will have a
part of the proceeds of the debt when these proceeds are received by B to whom the debt is transferred. Second, the doctrine of
champerty should not be extended, and, in order that there should be champerty, there must be a lawsuit in the offing; no lawsuit
was in the offing in the present case.
Ronald Bernstein and J S Colyer for the defendants: It is submitted (i) that the plaintiff has no cause of action independently
of the assignments, and consequently (ii) that, if the assignments are champertous, they are illegal or void and no cause of action
arises from them; (iii) that an agreement between a claimant and a stranger whereby the stranger agrees to finance the prosecution
of a claim in consideration of a share of the proceeds is champertous, and (iv) it is immaterial in the case of such an agreement
which of the two parties to it thereafter becomes a plaintiff in an action.

24 January 1963. The following judgment was delivered.

MEGAW J stated the substance of the letters of 3 July 1953, and the material provisions (in an English translation) of the French
document of assignment dated 25 June 1956, read the letter of 24 June 1959, giving notice of assignment, reviewed the course of
events relevant to the claim and referred to the writ and statement of claim. His Lordship then read para 5 of the defence,
mentioned the order dated 26 June 1962, directing the trial of the issue thereby raised as a preliminary issue and stated that he had
not heard argument on sub-para. (a) of para 5 as he was satisfied that sub-para. (b) concluded the issue in 64 favour of the
defendants. His Lordship stated the four propositions that were submitted by counsel for the defendants (as set out at p 64, letter
h, ante), and said that propositions (i) and (ii) were not disputed by the plaintiff and that he (His Lordship) agreed with
propositions (iii) and (iv). His Lordship referred to the submissions on behalf of the plaintiff, agreed with the first submission as
stated at p 64, letter f, ante, and accepted, as regards the second submission, that the doctrine of champerty should not be
extended. His Lordship found on the evidence, however, that the the proper inference to draw was that the assignments in June
and November, 1956, were made by the assignors with the knowledge and intention, shared by the plaintiff, that legal
proceedings would be necessary if anything were to be recovered from the defendants, and that the plaintiffs intention in taking
the assignments was to seek to enforce by litigation against the defendants the supposed rights under the letters of 3 July 1956.
His Lordship found that both the assignments were plainly champertous agreements, and continued:] The assignments being
champertous agreements, it is quite unnecessary for me to go into the various authorities which have been cited. It might have
been of importance had I reached a different conclusion on the facts as to what was in the minds of the parties in making these
assignments. Counsel for the defendants has fairly pointed out that in the case of Martell v Consett Iron Co Ltd, it was held that
the mere fact that an action was being maintained was not a defence. It might be a matter which would arise thereafter, but it
could not be raised by way of defence to the action. In my view, the position is entirely different where the plaintiff in the action
is one who does not have any original title in respect of the matter which he claims. He then has to show how he comes to the
title to be plaintiff at all. That is an essential part of his cause of action. Here the step necessary to establish his title before he
can begin to present the rest of his case is these two assignments from Marcelle and Kraus. If the point be taken by the
defendants, as it has been taken here, that those assignments are champertous agreements, then, in my view, that is a matter which
can properly be taken by way of defence and has to be considered by the court as part of the defence, in the same way as any
other defects, that might be alleged in the assignment on which a plaintiff had to rely to show his title, might be raised by way of
defence.
The mere fact that champerty, if it exists, is illegal and, indeed, can be criminal, is certainly not a reason for refusing to
consider it as part of the defence to an action where, if the agreement is champertous and illegal, it destroys a necessary step in
the plaintiffs title in the action. I am therefore satisfied that the decision in Martell v Consett Iron Co Ltd is not a bar to the
course, which has been taken here, of pleading in the defence that this is a champertous agreement. Accepting as I do the four
propositionsa put forward by counsel on behalf of the defendants, it follows that I decide this issue in favour of the defendants.
________________________________________
a See p 64, letter h, ante

Judgment for the defendants on the issue.

Solicitors: F de B Brisley, Purley (for the plaintiff); Kanter, Jules & Co (for the defendants).

Shireen Irani Barrister.

CORRIGENDUM
The foregoing case, commencing at p 63, ante, was heard before, and decided by, Mr Justice Megaw, to whom the case was
transferred for hearing from Mr Justice Gormans list. It is regretted that in error the case is reported as having been decided by
Mr Justice Gorman, and it is requested that the name of Mr Justice Megaw should be substituted for that of Mr Justice Gorman
on pp 63, 64, ante.
65
[1963] 2 All ER 66

Ridge v Baldwin and others


CRIMINAL; Police

HOUSE OF LORDS
LORD REID, LORD EVERSHED, LORD MORRIS OF BORTH-Y-GEST, LORD HODSON AND LORD DEVLIN
5, 6, 7, 8, 12, 13, 14, 15 NOVEMBER 1962, 14 MARCH 1963

Police County borough police force Chief constable Summary dismissal by watch committee Whether rules of natural
justice applicable to proceedings for dismissal Chief constable previously indicted for alleged criminal offences Acquitted,
but conduct severely criticised by trial judge Appeal against decision of watch committee dismissed by Home Secretary
Whether action by chief constable thereby barred Municipal Corporations Act, 1882 (45 & 46 Vict c 50), s 191(4) Police Act,
1919 (9 & 10 Geo 5 c 46), s 4(1) Police (Appeals) Act, 1927 (17 & 18 Geo 5 c 19), s 2(3) Police (Discipline) (Deputy Chief
Constables, Assistant Chief Constables and Chief Constables) Regulations, 1952 (SI 1952 No 1706), reg 1 and reg 18.

The appellant, who in March, 1958, was nearly fifty-nine years of age, joined Brighton Borough Police Force in 1925; thereafter
he rose in the service and was appointed chief constable in 1956, the appointment being expressed to be subject to the Police
Acts and regulations. In October, 1957, he was suspended from duty after he had been arrested, together with two other officers
of the same police force, on charges which were subsequently the subjects of two indictments, one for criminal conspiracy to
corrupt the course of justice and the other for corruption. At the end of the trial of the first indictment in February, 1958, at which
the appellant had given evidence himself but had called no other witnesses, he was acquitted, but the other two police officers
were convicted. In passing sentence on the other two police officers, the trial judge intimated that they had not had from the
appellant the professional and moral leadership which they should have had. At the trial of the second indictment on 6 March
1958, the prosecution offered no evidence against the appellant, and the judge directed the jury to acquit him, but again he made
certain observations about the appellant. On 7 March 1958, the watch committee held a meeting at which, after considering
matters relating to the appellant, they unanimously dismissed him from his office of chief constable under s 191(4) a of the
Municipal Corporations Act, 1882. The appellant was not present at this meeting, nor was he charged or given notice of the
proposal to dismiss him or particulars of the grounds on which it was based or an opportunity of putting his case. By notice of
appeal dated 12 March 1958, the appellant appealed, under the Police (Appeals) Act, 1927, to the Home Secretary against his
dismissal, the notice stating that it was without prejudice to the validity of the watch committees decision, and reserving right to
contend that the procedure was bad. On 18 March 1958, the watch committee held a special meeting at which the appellants
solicitor requested them to re-consider their decision, particularly with regard to its consequences in relation to the plaintiffs
pension, but by a majority the watch committee adhered to their previous decision. On 5 July the Home Secretary dismissed the
appellants appeal. In October, 1958, the appellant commenced an action against the watch committee, claiming that his
purported dismissal was void, and also claiming payment of salary and pension, or alternatively, damages. His action was
dismissed. On appeal,
________________________________________
a The terms of s 191(4) are set out at p 96, letter h, post

Held Lord Evershed dissenting): (i) the decision of the watch committee on 7 March 1958, to dismiss the appellant was null
and void for the following reasons
(a) In exercising the power of dismissal conferred by s 191(4) b of the Municipal Corporations Act, 1882 (at any rate where
that power was to be 66 exercised on the ground of negligence, which required to be proved c) the watch committee were bound
to observe the principles of natural justice, but in this instance the committee had not observed them, for the appellant had not
been charged nor informed of the grounds on which they proposed to proceed and had not been given a proper opportunity to
present his defence (see p 80, letter g, p 109, letter h, and p 116, letter d, post).
________________________________________
b The terms of s 191(4) are set out at p 96, letter h, post
c The power of dismissal was exercisable in relation to a borough constable whom the watch committee think negligent in the discharge of
his duty or otherwise unfit for the same; there are dicta that the inclusion of the words or otherwise unfit conferred a residual discretion
which might be unfettered, and for the exercise of which no charge was necessary (see p 111, letter c, and p 114, letter g, post)

Baggs case ((1615), 11 Co Rep 93b); Cooper v Wandsworth Board of Works ((1863), 14 CBNS 180); De Verteuil v Knaggs
([1918] AC 557) applied.
Dicta of Atkin LJ, in R v Electricity Commissioners ([1923] All ER Rep at p 161) and of Lord Hewart CJ, in R v Church
Assembly Legislative Committee ([1927] All ER Rep at p 699) considered and explained.
Nakkuda Ali v M F de S Jayaratne ([1951] AC 66) disapproved in part.
and (b) (per Lord Morris of Borth-y-Gest, Lord Reid and Lord Hodson concurring) once there was a report or allegation
from which it appeared that a chief constable might have committed an offence against the discipline code, established by
regulations under the Police Act, 1919, it became a condition precedent to any dismissal based on a finding of guilty of such an
offence that the regulations should in essentials have been put into operation, but the watch committee had not complied with the
regulations, for they preferred no charge against the appellant and gave him no notice and no opportunity to defend himself (see p
102, letters f and g, p 116, letter c, and p 81, letter b, post);
or (c) (per Lord Devlin) compliance with reg 11(1) of the Police (Discipline) (Deputy Chief Constables etc) Regulations,
1952, but not all other of those regulations, was a condition precedent to dismissal, and here there had been no report or inquiry
satisfying reg 11(1) (see p 118, letter h, post).
and (d) the proceedings at the meeting of 18 March 1958, were not a full re-hearing and did not make good the failure on 7
March to observe the rules of natural justice (see p 81, letter a, p 106, letter i, and p 112, letter g, post).
(e) (Lord Devlin dissenting) the consequence of the failure to observe the rules of natural justice was that the decision of 7
March 1958, was void, not merely voidable (see p 81, letter e, p 110, letter a, and p 116, letter h, post; cf p 120, letter e, post).
Wood v Woad ((1874), LR 9 Exch 190), approved.
(f) the same consequence flowed from disregard of the regulations (see p 104, letter h, p 105, letter e, p 81, letter b, and p
116, letters c and d, post).
Andrews v Mitchell ([19047] All ER Rep 599) and Annamunthodo v Oilfields Workers Trade Union ([1961] 3 All ER 621)
applied.
(ii) the decision of 7 March 1958, was a nullity, and the decision of the Secretary of State, although final and binding by
virtue of s 2(3) of the Police (Appeals) Act, 1927, could not make valid that which was a nullity (see p 81, letter i, p 106, letter i,
to p 107, letter a, p 116, letter f, and p 119, letter b, post).
Decision of the Court of Appeal ([1962] 1 All ER 834) reversed.

Notes
In considering the application of the principles of natural justice to cases of dismissal, Lord Reid distinguishes three classes of
cases (i) dismissal of a servant by a master, (ii) dismissal from offices held at pleasure, and (iii) dismissal from an office where
there must be something against a man to warrant his dismissal (see p 71, letter f, post). The present case fell within class (iii).
67
The question of waiver (cf holding (ii) at [1962] 1 All ER p 835), as distinct from the effect of s 2(3) of the Police (Appeals)
Act, 1927, was referred to in the opinions (cf eg, p 81, letter h, and p 116, letter e, post), but it was doubted whether it really
arose.
As to the statutory power to dismiss a member of a police force, see 30 Halsburys Laws (3rd Edn) 103, para 171.
For the Municipal Corporations Act, 1882, s 191, see 14 Halsburys Statutes (2nd Edn) 140.
For the Police Act, 1919, see 18 Halsburys Statutes (2nd Edn) 122.
For the Police (Appeals) Act, 1927, s 2(3), see ibid, 136.
For the Police (Discipline) Regulations, 1952, Sch 1, see 17 Halsburys Statutory Instruments 200.

Cases referred to in opinions


Andrews v Mitchell [19047] All ER Rep 599, [1905] AC 78, 74 LJKB 333, 91 LT 537, 25 Digest (Repl) 347, 287.
Annamunthodo v Oilfields Workers Trade Union [1961] 3 All ER 621, [1961] AC 945, [1961] 3 WLR 650, 3rd Digest Supp.
Baggs Case (1615), 11 Co Rep 93b, 1 Roll Rep 224, 77 ER 1271, 13 Digest (Repl) 217, 382.
Barnard v National Dock Labour Board [1953] 1 All ER 1113, [1953] 2 QB 18, [1953] 2 WLR 995, 3rd Digest Supp.
Blisset v Daniel (1853), 10 Hare, 493, 1 Eq Rep 484, 1 WR 529, 68 ER 1022, 37 Digest (Repl) 609, 1703.
Board of Education v Rice [191113] All ER Rep 36, [1911] AC 179, 80 LJKB 796, 104 LT 689, 75 JP 393, HL, affg, SC sub
nom R v Board of Education, [1910] 2 KB 165, 79 LJKB 692, 102 LT 578, 74 JP 259, CA, 19 Digest (Repl) 630, 206.
Capel v Child (1832), 2 C & J 558, 2 Tyr 689, 1 LJEx 205, 149 ER 235, 19 Digest (Repl) 447, 2644.
Cooper v Wandsworth Board of Works (1863), 14 CBNS 180, 2 New Rep 31, 32 LJCP 185, 8 LT 278, 143 ER 414, 26 Digest
(Repl) 585, 2450.
Cooper v Wilson [1937] 2 All ER 726, [1937] 2 KB 309, 106 LJKB 728, 157 LT 290, 101 JP 349, 30 Digest (Repl) 171, 219.
Dawkins v Antrobus (1881), 17 ChD 615, 44 LT 557, CA, affg, (1879), 41 LT 490, 8 Digest (Repl) 652, 21.
Daws, Re (1838), 8 Ad & El 936, 1 Per & Dav 146, 112 ER 1095, 16 Digest (Repl) 464, 2841.
De Verteuil v Knaggs [1918] AC 557, 87 LJPC 128, 8 Digest (Repl) 691, 38.
Dean v Bennett (1870), 6 Ch App 489, 40 LJCh 452, 24 LT 169, 13 Digest (Repl) 228, 512.
Fisher v Jackson [1891] 2 Ch 84, 60 LJCh 482, 64 LT 782, 19 Digest (Repl) 642, 278.
Fisher v Keane (1878), 11 ChD 353, 49 LJCh 11, 41 LT 335, 8 Digest (Repl) 656, 36.
Hogg v Scott [1947] 1 All ER 788, [1947] KB 759, [1948] LJR 666, 177 LT 32, 111 JP 282, 2nd Digest Supp.
Hopkins v Smethwick Local Board of Health (1890), 24 QBD 712, 59 LJQB 250, 62 LT 783, 54 JP 693, 26 Digest (Repl) 634,
2822.
Kanda v Government of The Federation of Malaya [1962] AC 322, [1962] 2 WLR 1153.
Lapointe v LAssociation de Bienfaisance et de Retraite de la Police de Montreal [1906] AC 535, 75 LJPC 73, 95 LT 479, 25
Digest (Repl) 323, *38.
Liversidge v Anderson [1941] 3 All ER 338, [1942] AC 206, 110 LJKB 724, 116 LT 1, 17 Digest (Repl) 422, 27.
68
Local Government Board v Arlidge [191415] All ER Rep 1, [1915] AC 120, 84 LJKB 72, 111 LT 905, 79 JP 97, HL revsg SC
sub nom R v Local Government Board, Ex p Arlidge, [1914] 1 KB 160, 83 LJKB 86, 109 LT 651, 78 JP 25, CA, 38 Digest
(Repl) 102, 733.
Nakkuda Ali v Jayaratne (M F de S), [1951] AC 66, 2nd Digest Supp.
Osgood v Nelson (1872), LR 5 HL 636, 41 LJQB 329, 13 Digest (Repl) 229, 513.
R v Church Assembly Legislative Committee, Ex p Haynes Smith [1927] All ER Rep 696, [1928] 1 KB 411, 97 LJKB 222, 138 LT
399, 44 TLR 68, 19 Digest (Repl) 240, 2.
R v Darlington School (Governors), (1844), 6 QB 682, 14 LJQB 67, 4 LTOS 175, 115 ER 257, 13 Digest (Repl) 229, 518.
R v Electricity Comrs, Ex p London Electricity Joint Committee Co (1920) Ltd [1923] All ER Rep 150, [1924] 1 KB 171, 93
LJKB 390, 130 LT 164, 20 Digest (Repl) 202, 3.
R v Gaskin (1799), 8 Term Rep 209, 101 ER 1349, 16 Digest (Repl) 382, 1685.
R v Metropolitan Police Comr, Ex p Parker [1953] 2 All ER 717, [1953] 1 WLR 1150, 117 JP 440, 16 Digest (Repl) 461, 2811.
R v Nat Bell Liquors Ltd [1922] All ER Rep 335, [1922] 2 AC 128, 91 LJPC 146, 127 LT 437, 27 Cox, CC 253, 16 Digest (Repl)
469, 2897.
R v Neal [1949] 2 All ER 438, [1949] 2 KB 590, 33 Cr App Rep 189, 14 Digest (Repl) 662, 6717.
R v North, Ex p Oakey [1927] 1 KB 491, 96 LJKB 77, 136 LT 387, 19 Digest (Repl) 345, 1302.
R v Smith (1844), 5 QB 614, 1 Dar & Mer 564, 13 LJQB 166, 21 LTOS 400, 9 JP 5, 114 ER 1381, 16 Digest (Repl) 382, 1686.
R v Stratford-upon-Avon Corpn (1670), 1 Lev 291, sub nom Dighton v Stratford-on-Avon Corpn, 1 Sid 461, 2 Keb 641, 82 ER
1217, 13 Digest (Repl) 232, 554.
R v University of Cambridge (1723), 1 Stra 557, Fortes Rep 202, 2 Ld Raym 1334, 8 Mod Rep 148, 93 ER 698, 8 Digest (Repl)
506, 2281.
Ramshay, Ex p (1852), 18 QB 173, Cox, M & H 589, 21 LJQB 238, 18 LTOS 273, 16 JP 135, 118 ER 65, 13 Digest (Repl) 371,
24.
Russell v Norfolk (Duke) [1949] 1 All ER 109, 12 Digest (Repl) 693, 5321.
Smith v R (1878), 3 App Cas 614, 47 LJPC 51, 38 LT 233.
Spackman v Plumstead Board of Works (1885), 10 App Cas 229, 54 LJMC 81, 53 LT 157, 40 JP 420, HL, affg SC sub nom
Plumstead Board of Works v Spackman, (1884), 13 QBD 878, 53 LJMC 142, 51 LT 757, 49 JP 132, CA, 26 Digest (Repl)
571, 2357.
Terrell v Secretary of State for the Colonies [1953] 2 All ER 490, [1953] 2 QB 482, 8 Digest (Repl) 793, 509.
Urban Houses Co Ltd v Oxford Corpn [1939] 4 All ER 211, [1940] Ch 70, 109 LJCh 38, 162 LT 29, 38 Digest (Repl) 230, 462.
Weinberger v Inglis (No 2) [1918] 1 Ch 517, 87 LJCh 345, 118 LT 769, 34 TLR 337, CA, affd [1919] AC 606, 88 LJCh 287, 121
LT 65, HL, 8 Digest (Repl) 651, 11.
Willis v Childe (1851), 13 Beav 117, 30 LJCh 113, 17 LTOS 12, 51 ER 46, 19 Digest (Repl) 640, 258.
Wood v Woad (1874), LR 9 Exch 190, 43 LJEx 153, 30 LT 815, 2 Asp MLC 289, 8 Digest (Repl) 656, 35.

Appeal
This was an appeal by Charles Field Williams Ridge (the appellant) from a decision of the Court of Appeal (Holroyd Pearce,
Harman and Davies 69LJJ), dated 30 January 1962, and reported [1962] 1 All ER 834, affirming the judgment of Streatfeild J
dated 19 April 1962, and reported [1961] 2 All ER 523, dismissing the appellants claim against the respondent watch committee,
the police authority of the county borough of Brighton, that their dismissal of him under s 191(4) of the Municipal Corporation
Act, 1882, was invalid.
The Court of Appeal held that the watch committee, in dismissing the appellant under s 191(4) of the Act of 1882, were not
bound to apply the Police (Discipline) Regulations, 1952, as those regulations required that, as a condition precedent to acting
thereunder, a report or allegation must be received. In this case, however, the appellant had been dismissed after his acquittal on
a criminal charge following which the trial judge had made certain observations on the fitness of the appellant for office as chief
constable. Such observations, the Court of Appeal held, were not a report or allegation. Further the Court of Appeal held that the
watch committee, exercising their power under s 191(4) of the Act of 1882, were acting in an executive or administrative
capacity, not in a judicial or quasi-judicial nature with the consequence that the rules of natural justice did not apply to their
proceedings for dismissal. The Court of Appeal also held that by appealing to the Home Secretary under the Police (Appeals)
Act, 1927, the appellant had waived his right to bring the action in the courts, notwithstanding the appellant purported to reserve
his right to contend that the decision of the watch committee was a nullity.
It was not contended before the House of Lords that the power of dismissal conferred by s 191(4) of Act of 1882 was
impliedly repealed, by the joint effect of the Police Act, 1919, s 4 and regulations thereunder (cf p 99, letter i, to p 100, letter b,
post); accordingly the decision of the Court of Appeal (see [1962] 1 All ER 834, letter i) on that point stands.

D J C Ackner QC and J L E Macmanus for the appellant.


Neville Faulks QC and P A Harmsworth for the respondents.

Their Lordships took time for consideration

14 March 1963. The following opinions were delivered.

LORD REID. My Lords, the appellant, Mr Ridge, became Chief Constable of the County Borough of Brighton in 1956, after
serving in the Brighton Police Force for some thirty-three years. At a meeting of the watch committee, the police authority, on 7
March 1958, it was resolved that he should be dismissed and he now maintains that that resolution was void and of no effect
because he had no notice of the grounds on which the committee proposed to act and no opportunity to be heard in his own
defence.
The appellant had been arrested on 25 October 1957, and subsequently tried on a charge of conspiring with the senior
members of his force and others to obstruct the course of justice, and had been suspended from duty on 26 October. He was
acquitted on 28 February but the other two members of the force were convicted and in sentencing them the trial judge, Donovan
J made a statement which included grave reflections on the appellants conduct. He was then indicted on a charge of corruption
and was on 6 March acquitted, no evidence having been offered against him. On this occasion Donovan J made a further
statement. On the day following that statement the watch committee met and summarily dismissed the appellant. I shall not deal
further with these matters because my noble and learned friend Lord Morris of Borth-y-Gest intends to do so.
The power of dismissal is contained in s 191(4) of the Municipal Corporations Act, 1882. So far as I am aware that
subsection is the only statutory provision regarding dismissal, and the respondents purported to act under it. It is in these terms:

The watch committee, or any two justices having jurisdiction in the borough, may at any time suspend, and the watch
committee may at any time dismiss, any borough constable whom they think negligent in the discharge of his duty, or
otherwise unfit for the same.
70

The appellant maintains that the watch committee ought to have proceeded in accordance with regulations made under the
Police Act, 1919, s 4(1), which authorised the Secretary of State to make regulations as to, inter alia, the conditions of service of
the members of all police forces in England and Wales. Regulations were duly made but the respondents maintain that they do
not apply to this case. For the moment I shall assume in their favour that that is so and consider whether the Act of 1882 taken by
itself authorised them to do as they did.
The appellants case is that in proceeding under the Act of 1882 the watch committee were bound to observe what are
commonly called the principles of natural justice, that before attempting to reach any decision they were bound to inform him of
the grounds on which they proposed to act and to give him a fair opportunity of being heard in his own defence. The authorities
on the applicability of the principles of natural justice are in some confusion and so I find it necessary to examine this matter in
some detail. The principle audi alteram partem goes back many centuries in our law and appears in a multitude of judgments of
judges of the highest authority. In modern times opinions have sometimes been expressed to the effect that natural justice is so
vague as to be practically meaningless. But I would regard these as tainted by the perennial fallacy that because something
cannot be cut and dried or nicely weighted or measured therefore it does not exist. The idea of negligence is equally
insusceptible of exact definition but what a reasonable man would regard as fair procedure in particular circumstances and what
he would regard as negligence in particular circumstances are equally capable of serving as tests in law, and natural justice as it
had been interpreted in the courts is much more definite than that. It appears to me that one reason why the authorities on natural
justice have been found difficult to reconcile is that insufficient attention has been paid to the great difference between various
kinds of cases in which it has been sought to apply the principle. What a minister ought to do in considering objections to a
scheme may be very different from what a watch committee ought to do in considering whether to dismiss a chief constable. So I
shall deal first with cases of dismissal. These appear to fall into three classes, dismissal of a servant by his master, dismissal from
an office held during pleasure, and dismissal from an office where there must be something against a man to warrant his
dismissal.
The law regarding master and servant is not in doubt. There cannot be specific performance of a contract of service and the
master can terminate the contract with his servant at any time and for any reason or for none. But if he does so in a manner not
warranted by the contract he must pay damages for breach of contract. So the question in a pure case of master and servant does
not at all depend on whether the master has heard the servant in his own defence: it depends on whether the facts emerging at the
trial prove breach of contract. But this kind of case can resemble dismissal from an office where the body employing the man is
under some statutory or other restriction as to the kind of contract which it can make with its servants, or the grounds on which it
can dismiss them. The present case does not fall within this class because a chief constable is not the servant of the watch
committee or indeed of anyone else.
Then there are many cases where a man holds an office at pleasure. Apart from judges and others whose tenure of office is
governed by statute, all servants and officers of the Crown hold office at pleasure and this has even been held to apply to a
colonial judge (Terrell v Secretary of State). It has always been held, I think rightly, that such an officer has no right to be heard
before he is dismissed and the reason is clear. As the person having the power of dismissal need not have anything against the
officer, he need not give any reason. That was stated as long ago as 1670 in R v Stratford-upon-Avon Corpn where the
corporation dismissed a town clerk who held office durante bene placito. The leading case on 71 this matter appears to be R v
Governors of Darlington School, although that decision was doubted by Lord Hatherley LC in Dean v Bennett ((1870), 6 Ch App
489 at p 496) and distinguished on narrow grounds in Willis v Childe. I fully accept that where an office is simply held at
pleasure the person having power of dismissal cannot be bound to disclose his reasons. No doubt he would in many cases tell the
officer and hear his explanation before deciding to dismiss him. But if he is not bound to disclose his reason and does not do so,
then, if the court cannot require him to do so, it cannot determine whether it would be fair to hear the officers case before taking
action. Again that is not this case. In this case the Act of 1882 permits the watch committee to take action only on the grounds of
negligence or unfitness. Let me illustrate the difference by supposing that a watch committee who had no complaint against their
present chief constable heard of a man with quite outstanding qualifications who would like to be appointed. They might think it
in the public interest to make the change but they would have no right to do it. But there could be no legal objection to dismissal
of an officer holding office at pleasure in order to put a better man in his place.
So I come to the third class which includes the present case. There I find an unbroken line of authority to the effect that an
officer cannot lawfully be dismissed without first telling him what is alleged against him and hearing his defence or explanation.
An early example is Baggs case though it is more properly deprivation of the privilege of being a burgess of Plymouth. R v
Gaskin arose out of the dismissal of a parish clerk and Lord Kenyon CJ ((1799), 8 Term Rep at p 210), referred to audi alteram
partem as one of the first principles of justice. R v Smith was another case of dismissal of a parish clerk and Lord Denman CJ
((1844), 5 QB at pp 622, 623), held that even personal knowledge of the offence was no substitute for hearing the officer: his
explanation might disprove criminal motive or intent and bring forward other facts in mitigation, and in any event delaying to
hear him would prevent yielding too hastily to first impressions. Ex parte Ramshay is important. It dealt with the removal from
office of a county court judge and the form of the legislation which authorised the Lord Chancellor to act is hardly
distinguishable from the form of s 191 which confers powers on the watch committee. The Lord Chancellor was empowered if
he should think fit to remove on the ground of inability or misbehaviour but Lord Campbell CJ ((1852), 18 QB at p 190), said
that this was only on the implied condition prescribed by the principles of eternal justice. In Osgood v Nelson objection was
taken to the way in which the Corporation of the City of London had removed the clerk to the Sheriffs Court and Lord Hatherley
LC said ((1872), LR 5 HL at p 649):

I apprehend my Lords that as has been stated by the learned Baron who has delivered in the name of the judges their
unanimous opinion, the court of Queens Bench has always considered that it has been open to that court, as in this case it
appears to have considered, to correct any court or tribunal or body of men who may have a power of this description, a
power of removing from office, if it should be found such persons have disregarded any of the essentials of justice in the
course of their inquiry before making that removal, or if it should be found that in the place of reasonable cause those
persons have acted obviously upon mere individual caprice.

That citation of authority might seem sufficient but I had better proceed further. In Fisher v Jackson, three vicars had power to
remove the master of an endowed school. But unlike the Darlington case the trust deed set out the grounds on which he could be
removedbriefly, inefficiency or failing to set a 72 good exampleand it was held that they could not remove him without
affording him an opportunity of being heard in his own defence. Only two other cases of this class were cited in argument,
Cooper v Wilson and Hogg v Scott. Both dealt with the dismissal of police officers and both were complicated by consideration
of regulations made under the Police Acts. In the former the majority at least recognised that the principles of natural justice
applied and in deciding the latter Cassels J ([1947] 1 All ER at p 792; [1947] KB at p 767), in deciding that a chief constable
could dismiss without hearing him an officer who had been convicted of felony, appears to have proceeded on a construction of
the regulations. Of course, if the regulations authorised him to do that and were intra vires in doing so there would be no more to
be said. I do not think it necessary to consider whether the learned judge rightly construed the regulations, for he did not
expressly or I think by implication question the general principle that a man is not to be dismissed for misconduct without being
heard.
Stopping there I would think that authority was wholly in favour of the appellant, but the watch committees argument was
mainly based on what has been said in a number of fairly recent cases dealing with different subject-matter. Those cases deal
with decisions by ministers, officials and bodies of various kinds which adversely affected property rights or privileges of persons
who had had no opportunity or no proper opportunity of presenting their cases before the decisions were given. And it is
necessary to examine those cases for another reason. The question which was or ought to have been considered by the watch
committee on 7 March 1958, was not a simple question whether or not the appellant should be dismissed. There were three
possible courses open to the watch committeereinstating the appellant as chief constable, dismissing him, or requiring him to
resign. The difference between the latter two is that dismissal involved forfeiture of pension rights whereas requiring him to
resign did not. Indeed, it is now clear that the appellants real interest in this appeal is to try to save his pension rights.
It may be convenient at this point to deal with an argument that, even if as a general rule a watch committee must hear a
constable in his own defence before dismissing him, this case was so clear that nothing that the appellant could have said could
have made any difference. It is at least very doubtful whether that could be accepted as an excuse. But even if it could the watch
committee would in my view fail on the facts. It may well be that no reasonable body of men could have reinstated the appellant.
But as between the other two courses open to the watch committee the case is not so clear. Certainly on the facts as we know
them the watch committee could reasonably have decided to forfeit the appellants pension rights, but I could not hold that they
would have acted wrongly or wholly unreasonably if they had in the exercise of their discretion decided to take a more lenient
course.
I would start an examination of the authorities dealing with property rights and privileges with Cooper v Wandsworth Board
of Works. Where an owner had failed to give proper notice to the board, they had under an Act of 1855 d authority to demolish
any building which he had erected and recover the cost from him. This action was brought against the board because they had
used that power without giving the owner an opportunity of being heard. The board maintained that their discretion to order
demolition was not a judicial discretion and that any appeal should have been to the Metropolitan Board of Works. But the court
decided unanimously in favour of the owner. Erle CJ ((1863), 14 CBNS at p 189), held that the power was subject to a
qualification repeatedly recognised that no man is to be deprived 73 of his property without his having an opportunity of being
heard and that this had been applied to many exercises of power which in common understanding would not be at all a more
judicial proceeding than would be the act of the district board in ordering a house to be pulled down. Willes J ((1863), 14 CBNS
at p 190) said that the rule was of universal application and founded on the plainest principles of justice and Byles J ((1863), 14
CBNS at p 194), said that:
________________________________________
d See the Metropolis Management Act, 1855, s 76

although there are no positive words in a statute requiring that the party shall be heard, yet the justice of the common
law will supply the omission of the legislature.

This was followed in Hopkins v Smethwick Local Board of Health. Wills J ((1890), 24 QBD at pp 714, 715), said:

In condemning a man to have his house pulled down a judicial act is as much implied as in fining him 5: and as the
local board is the only tribunal that can make such an order its act must be a judicial act and the party to be affected should
have a notice given him The judgment of WILLES, J. (in Coopers case [(1890), 14 CBNS at pp 190194.]) goes far
more upon the nature of the thing done by the board than on the phraseology of the Act itself. It deals with the case on
principle: from the nature of the thing done it must be a judicial act and justice requires that the man should be heard.

In the Court of Appeal ((1890), 24 QBD at pp 716, 717) Lord Esher MR in dismissing an appeal expressly approved the
principles laid down in Coopers case.
The principle was applied in different circumstances in Smith v R. That was an action of ejectment on the alleged forfeiture
of a Crown lease in Queensland. The governor was entitled to forfeit the lease if it had been proved to the satisfaction of a
commissioner that the lessee had abandoned or ceased to reside on the land. The commissioner did not disclose to the lessee the
case against him so that he had no opportunity to meet it, and therefore the decision could not stand. The Commissioner was not
bound by any rules as to procedure or evidence but he had to conduct his inquiry according to the requirements of substantial
justice. In De Verteuil v Knaggs the governor of Trinidad was entitled to remove immigrants from an estate on sufficient
ground shown to his satisfaction. Lord Parmoor ([1918] AC at p 560) said that

the acting governor was not called upon to give a decision on an appeal between parties and it is not suggested that he
holds the position of a judge or that the appellant is entitled to insist on the forms used in ordinary judicial procedure

but he had

a duty of giving to any person against whom the complaint is made a fair opportunity to make any relevant
statement which he may desire to bring forward and a fair opportunity to correct or controvert any relevant statement
brought forward to his prejudice.

The duty of an official architect in fixing a building line was stated in somewhat similar terms in Spackman v Plumstead Board of
Works.
I shall now turn to a different class of casedeprivation of membership of a professional or social body. In Wood v Woad,
the committee purported to expel a member of a mutual insurance society without hearing him and it was held that their action
was void and so he was still a member. Kelly CB said of audi alteram partem ((1874), LR 9 Exch at p 196)
74

this rule is not confined to the conduct of strictly legal tribunals but is applicable to every tribunal or body of persons
invested with authority to adjudicate upon matters involving civil consequences to individuals.

This was expressly approved by Lord Macnaghten giving the judgment of the Board in Lapointe v Lassociation de Bienfaisance
etc ([1906] AC 535 at p 540). In that case the board of directors of the association had to decide whether to give a pension to a
dismissed constablethe very point the watch committee had to decide in this caseand it was held that they had to observe
the elementary principles of justice. Then there are the club cases Fisher v Keane and Dawkins v Antrobus. In the former
Jessel MR said ((1878), 11 ChD at pp 362, 363) of the committee:

They ought not as I understand it according to the ordinary rules by which justice should be administered by
committees of clubs, or by any other body of persons who decide upon the conduct of others, to blast a mans reputation for
everperhaps to ruin his prospects for life without giving him an opportunity of either defending or palliating his
conduct.

In the latter case it was held that nothing had been done contrary to natural justice. In Weinberger v Inglis (No 2) a member of
enemy birth was excluded from the stock exchange and it was held that the committee had heard him before acting. Lord
Birkenhead LC said ([1919] AC at p 616):

if I took the view that the appellant was condemned upon grounds never brought to his notice I should not assent to
the legality of that course unless compelled by authority.
He said this although the rule under which the committee acted was in the widest possible termsthat the committee should each
year re-elect such members as they should deem eligible as members of the stock exchange.
I shall not at present advert to the various trade union cases because I am deliberately considering the state of the law before
difficulties were introduced by statements in various fairly recent cases. It appears to me that if the present case had arisen thirty
or forty years ago the courts would have had no difficulty in deciding this issue in favour of the appellant on the authorities which
I have cited. So far as I am aware none of these authorities has ever been disapproved or even doubted. Yet the Court of Appeal
have decided this issue against the appellant on more recent authorities which apparently justify that result. How has this come
about? At least three things appear to me to have contributed. In the first place there have been many cases where it has been
sought to apply the principles of natural justice to the wider duties imposed on ministers and other organs of government by
modern legislation. For reasons which I shall attempt to state in a moment it has been held that those principles have a limited
application in such case and those limitations have tended to be reflected in other decisions on matters to which in principle they
do not appear to me to apply. Secondly, again for reasons which I shall attempt to state, those principles have been held to have a
limited application in cases arising out of war-time legislation; and again such limitations have tended to be reflected in other
cases. And thirdly, there has I think been a misunderstanding of the judgment of Atkin LJ, in R v Electricity Comrs, Ex p London
Electricity Joint Committee Co ([1923] All ER Rep 150 at pp 158 et seq; [1924] 1 KB 171 at pp 198 et seq).
In cases of the kind with which I have been dealing the Board of Works or the governor or the club committee was dealing
with a single isolated case. It was not deciding, like a judge in a lawsuit, what were the rights of the person before it. But it was
deciding how he should be treatedsomething analogous to a judges duty in imposing a penalty. No doubt policy would play
some part in the decisionbut so it might when a judge is imposing a sentence. So it was easy to say that such a body is
performing a quasi judicial task in considering and 75 deciding such a matter and to require it to observe the essentials of all
proceedings of a judicial characterthe principles of natural justice. Sometimes the functions of a minister or department may
also be of that character and then the rules of natural justice can apply in much the same way. But more often their functions are
of a very different character. If a minister is considering whether to make a scheme for say an important new road, his primary
concern will not be with the damage which its construction will do to the rights of individual owners of land. He will have to
consider all manner of questions of public interest and, it may be, a number of alternative schemes. He cannot be prevented from
attaching more importance to the fulfilment of his policy than to the fate of individual objectors and it would be quite wrong for
the courts to say that the minister should or could act in the same kind of way as a board of works deciding whether a house
should be pulled down. And there is another important difference. As explained in Local Government Board v Arlidge a minister
cannot do everything himself. His officers will have to gather and sift all the facts including objections by individuals and no
individual can complain if the ordinary accepted methods of carrying on public business do not give him as good protection as
would be given by the principles of natural justice in a different kind of case.
We do not have a developed system of administrative lawperhaps because until fairly recently we did not need it. So it is
not surprising that in dealing with new types of cases the courts have had to grope for solutions, and have found that old powers,
rules and procedure are largely inapplicable to cases which they were never designed or intended to deal with. But I see nothing
in that to justify our thinking that our old methods are any less applicable today than ever they were to the older types of case.
And, if there are any dicta in modern authorities which point in that direction, then in my judgment they should not be followed.
And now I must say something regarding war-time legislation. The older authorities clearly show how the courts engrafted
the principles of natural justice on to a host of provisions authorising administrative interference with private rights. Parliament
knew quite well that the courts had an inveterate habit of doing that and must therefore be held to have authorised them to do it
unless a particular Act showed a contrary intention. And such an intention could appear as a reasonable inference as well as from
express words. It seems to me to be a reasonable and almost an inevitable inference from the circumstances in which defence
regulations were made and from their subject-matter that at least in many cases the intention must have been to exclude the
principles of natural justice. War-time secrecy alone would often require that and the need for speed and general pressure of
work were other factors. But it was not to be expected that anyone would state in so many words that a temporary abandonment
of the rules of natural justice was one of the sacrifices which war conditions requiredthat would have been almost calculated to
create the alarm and despondency against which one of the regulations was specifically directed. And I would draw the same
conclusion from another fact. In many regulations there was set out an alternative safeguard more practicable in war timethe
objective test that the officer must have reasonable cause to believe whatever was the crucial matter. (I leave out of account the
very peculiar decision of this House in Liversidge v Anderson.) So I would not think that any decision that the rules of natural
justice were excluded from war-time legislation should be regarded as of any great weight in dealing with a case such as this case
which is of the older type, and which involves the interpretation of an Act passed long before modern modifications of the
principles of natural justice became necessary, and at a time when, as Parliament was well aware, the courts habitually applied the
principles of natural justice to provisions like s 191(4) of the Municipal Corporations Act, 1882.
76
The matter has been further complicated by what I believe to be a misunderstanding of a much quoted passage in the
judgment of Atkin LJ, in R v Electricity Comrs. He said ([1923] All ER Rep at p 161; [1924] 1 KB at p 205):

The operation of the writs [of prohibition and certiorari] has extended to control the proceedings of bodies which do
not claim to be and would not be recognised as, courts of justice. Whenever any body of persons having legal authority to
determine questions affecting the rights of subjects, and having the duty to act judicially, act in excess of their legal
authority, they are subject to the controlling jurisdiction of the Kings Bench Division exercised in these writs.

A gloss was put on this by Lord Hewart CJ, in R v Legislative Committee of the Church Assembly ([1927] All ER Rep 696 at p
699; [1928] 1 KB 411 at pp 415, 416). There it was sought to prohibit the Assembly from proceeding further with the Prayer
Book Measure, 1927. That seems to me to have no resemblance to a question whether a person should be deprived of his rights
or privileges, and the case was decided on the ground that this was a deliberative or legislative body and not a judicial body.
Salter J put it in a few lines ([1927] All ER Rep at p 701; [1928] 1 KB at p 419):

The person or body to whom these writs are to go must be a judicial body in this sense, that it has power to determine
and decide, and the power carries with it, of necessity, the duty to act judicially. I think that the Church Assembly has no
such power and, therefore, no such duty.

But Lord Hewart CJ said, having quoted the passage from Lord Atkins judgment ([1928] 1 KB at p 415; [1927] All ER Rep at p
699):

The question therefore which we have to ask ourselves in this case is whether it is true to say in this matter, either of
the Church Assembly as a whole or of the Legislative Committee of the Church Assembly, that it is a body or persons
having legal authority to determine questions affecting the rights of subjects and having the duty to act judicially. It is to be
observed that in the last sentence which I have quoted from the judgment of ATKIN, L.J. [[1923] All ER Rep at p 161;
[1924] 1 KB at pp 204, 205.], the word is not or but and. In order that a body may satisfy the required test it is not
enough that it should have legal authority to determine questions affecting the rights of subjects; there must be superadded
to that characteristic the further characteristic that the body has the duty to act judicially. The duty to act judicially is an
ingredient which if the test is to be satisfied must be present. As these writs in the earlier days were issued only to bodies
which without any harshness of construction could be called and naturally would be called courts so also today these writs
do not issue except to bodies which act or are under the duty to act in a judicial capacity.

I have quoted the whole of this passage because it is typical of what has been said in several subsequent cases. If Lord Hewart
CJ, meant that it is never enough that a body simply has a duty to determine what the rights of an individual should be, but that
there must always be something more to impose on it a duty to act judicially before it can be found to observe the principles of
natural justice, then that appears to me impossible to reconcile with the earlier authorities. I could not reconcile it with what Lord
Denman CJ, said in R v Smith ((1844), 5 QB at pp 620624), or what Lord Campbell CJ, said in Ex parte Ramshay ((1852), 18
QB 173 at pp 188196), or what Lord 77 Hatherley LC said in Osgood v Nelson ((1872), LR 5 HL at pp 648650), or what was
decided in Cooper v Wandsworth Board of Works, or Hopkins v Smethwich Local Board, or what Lord Parmoor said in De
Verteuil v Knaggs ([1918] AC at pp 558563), or what Kelly CB ((1874), LR 9 Exch at pp 195199) said, with the subsequent
approval of Lord MacNaghtene, in Wood v Woad, or what Jessel MR, said in Fisher v Keane ((1878), 11 ChD at pp 357363), or
what Lord Birkenhead LC said in Weinberger v Inglis (No 2) ([1919] AC at pp 610617) and that is only a selection of the earlier
authorities. And, as I shall try to show, it cannot be what Lord Atkin ([1923] All ER Rep at pp 160, 161; [1924] 1 KB at p 204)
meant.
________________________________________
e In Lapointes case [1906] AC at p 540

In R v Electricity Comrs the commissioners had a statutory duty to make schemes with regard to electricity districts and to
hold local inquiries before making them. They made a draft scheme which in effect allocated duties to one body which the Act
required should be allocated to a different kind of body. This was held to be ultra vires and the question was whether prohibition
would lie. It was argued that the proceedings of the commissioners were purely executive and controllable by Parliament alone.
Bankes LJ said ([1923] All ER Rep at p 157; [1924] 1 KB at p 198):

On principle and on authority it is, in my opinion, open to this court to hold, and I consider that it should hold, that
powers so far-reaching, affecting as they do individuals as well as property, are powers to be exercised judicially and not
ministerially, or merely, to use the languagef of PALLES, C.B., as proceedings towards legislation.
________________________________________
f In R v Kingstown Comrs (1885), 16 LR Ir 150

So he inferred the judicial element from the nature of the power. I think that Atkin LJ ([1923] All ER Rep at pp 157 et seq.;
[1924] 1 KB at pp 198 et seq), did the same. Immediately after the passage which I said has been misunderstood, he cited a
variety of cases and in most of them I can see nothing superadded (to use Lord Hewarts word) to the duty itself. Certainly
Lord Atkin did not say that anything was superadded. A later passage in his judgment convinces me that he, like Bankes LJ (See
[1923] All ER Rep at p 157; [1924] 1 KB at p 198), inferred the judicial character of the duty from the nature of the duty itself.
Although it is long I am afraid I must quote it ([1923] All ER Rep at pp 161, 162; [1924] 1 KB at pp 206, 207):

In the present case the Electricity Commissioners have to decide whether they will constitute a joint authority in a
district in accordance with law, and with what power they will invest that body. The question necessarily involves the
withdrawal from existing bodies of undertakers of some of their existing rights, and imposing upon them of new duties
including their subjection to the control of the new body, and new financial obligations. It also provides in the new body a
person to whom may be transferred rights of purchase which at present are vested in another authority. The commissioners
are proposing to create such a new body in violation of the Act of Parliament and are proposing to hold a possibly long and
expensive inquiry into the expediency of such a scheme, in respect of which they have the power to compel representatives
of the prosecutors to attend and produce papers. I think that in deciding upon the scheme and in holding the inquiry they
are acting judicially in the sense of the authorities I have cited

There is not a word in Lord Atkins judgment to suggest disapproval of the earlier line of authority which I have cited. On the
contrary, he goes further than those authorities. I have already stated my view that it is more difficult 78 for the courts to control
an exercise of power on a large scale where the treatment to be meted out to a particular individual is only one of many matters to
be considered. This was a case of that kind, and, if Lord Atkin was prepared to infer a judicial element from the nature of the
power in this case, he could hardly disapprove such an inference when the power relates solely to the treatment of a particular
individual.
The authority chiefly relied on by the Court of Appeal in holding that the watch committee were not bound to observe the
principles of natural justice was Nakkuda Ali v M F de S Jayaratne. In that case the Controller of Textiles in Ceylon made an
order cancelling the appellants licence to act as a dealer, and the appellant sought to have that order quashed. The controller
acted under a defence regulation, which empowered him to cancel a licence where the controller has reasonable grounds to
believe that any dealer is unfit to be allowed to continue as a dealer. The Privy Council regarded that as ([1951] AC at p 77)

imposing a condition that there must in fact exist such reasonable grounds known to the controller before he can
validly exercise the power of cancellation.

But according to their judgment certiorari did not lie and no other means was suggested whereby the appellant or anyone else in
his position could obtain redress even if the controller acted without a shred of evidence. It is quite true that the judgment went
on, admittedly unnecessarily, to find that the controller had reasonable grounds and did observe the principles of natural justice,
but the result would have been just the same if he had not. This House is not bound by decisions of the Privy Council and for my
own part nothing short of a decision of this House directly in point would induce me to accept the position that, although an
enactment expressly requires an official to have reasonable grounds for his decision, our law is so defective that a subject cannot
bring up such a decision for review however seriously he may be affected and however obvious it may be that the official acted in
breach of his statutory obligation. The judgment proceeds ([1951] AC at p 77):

But it does not seem to follow necessarily from this that the controller must be acting judicially in exercising the
power. Can one not act reasonably without acting judicially? It is not difficult to think of circumstances in which the
controller might in any ordinary sense of the word have reasonable grounds of belief without having ever confronted the
licence holder with the information which is the source of his belief. It is a long step in the argument to say that because a
man is enjoined that he must not take action unless he has reasonable grounds for believing something he can only arrive at
that belief by a course of conduct analagous to the judicial process. And yet unless that proposition is valid there is really
no ground for holding that the controller is acting judicially or quasi-judicially when he acts under this regulation. If he is
not under a duty so to act then it would not be according to law that his decision should be amenable to review and if
necessary to avoidance by the procedure of certiorari.
I would agree that in this and other defence regulation cases the legislator has substituted an obligation not to act without
reasonable grounds for the ordinary obligation to afford to the person affected an opportunity to submit his defence. It is not
necessary in this case to consider whether by so doing he has deprived the courts of the power to intervene if the officer acts
contrary to his duty. The question in the present case is not whether Parliament substituted a different safeguard for that afforded
by natural justice, but whether in the Municipal Corporations Act, 1882, it excluded the safeguard of natural justice and put
nothing in its place.
So far there is nothing in the judgment of the Privy Council directly relevant 79 to the present case. It is the next paragraph
which causes the difficulty and I must quote the crucial passage ([1951] AC at p 78):

But the basis of the jurisdiction of the courts by way of certiorari has been so exhaustively analysed in recent years
that individual instances are now only of importance as illustrating a general principle that is beyond dispute. That
principle is most precisely stated in the words of ATKIN, L.J., in R. v. Electricity Comrs.

and then follows the passagepg 80 (See p 77, letter a, ante, [1923] All ER Rep at p 161; [1924] 1 KB at p 205) with which I have
already dealt at length. And then there follows the quotation from Lord Hewart CJ (See p 77, letter f, ante, [1928] 1 KB at p 415;
[1927] All ER Rep at p 699), which I have already commented on ending with the words ([1951] AC at p 78)there must be
superadded to that characteristic the further characteristic that the body has the duty to act judicially. And then it is pointed out:
It is that characteristic that the controller lacks in acting under reg 62. Of course if it were right to say that Lord Hewart CJs
gloss on Lord Atkin stated a general principle that is beyond dispute the rest would follow. But I have given my reasons for
holding that it does no such thing, and in my judgment the older cases certainly do not illustrate any such general principle
they contradict it. No case older than 1911 was cited in Nakkuda Ali v M F de S Jayaratne on this question, and this question was
only one of several difficult questions which were argued and decided. So I am forced to the conclusion that this part of the
judgment in Nakkudas case was given under a serious misapprehension of the effect of the older authorities and therefore cannot
be regarded as authoritative.
I would sum up my opinion in this way. Between 1882 and the making of police regulations in 1920, s 191(4) of the
Municipal Corporations Act, 1882, had to be applied to every kind of case. The respondents contention is that, even where there
was a doubtful question whether a constable was guilty of a particular act of misconduct, the watch committee were under no
obligation to hear his defence before dismissing him. In my judgment it is abundantly clear from the authorities that I have
quoted that at that time the courts would have rejected any such contention. In later cases dealing with different subject-matter
opinions have been expressed in wide terms so as to appear to conflict with those earlier authorities. But learned judges who
expressed those opinions generally had no power to overrule those authorities, and in any event it is a salutary rule that a judge is
not to be assumed to have intended to overrule or disapprove of an authority which has not been cited to him and which he does
not even mention. So I would hold that the power of dismissal in the Act of 1882 could not then have been exercised and cannot
now be exercised until the watch committee have informed the constable of the grounds on which they propose to proceed and
have given him a proper opportunity to present his case in defence.
Next comes the question whether the respondents failure to follow the rules of natural justice on 7 March was made good
by the meeting on 18 March. I do not doubt that if an officer or body realises that it has acted hastily and reconsiders the whole
matter afresh after affording to the person affected a proper opportunity to present his case then its later decision will be valid.
An example is De Verteuil v Knaggs. But here the appellants solicitor was not fully informed of the charges against the appellant
and the watch committee did not annul the decision which they had already published and proceed to make a new decision. In
my judgment what was done on that day was a very inadequate substitute for a full rehearing. Even so three members of the
committee changed their minds, and it is impossible to say what the 80 decision of the committee would have been if there had
been a full hearing after disclosure to the appellant of the whole case against him. I agree with those of your lordships who hold
that this meeting of 18 March cannot affect the result of this appeal.
The other ground on which some of your lordships prefer to proceed is the watch committees failure to act in accordance
with the police regulations. I have had an opportunity of reading the speech about to be delivered by my noble and learned friend
Lord Morris of Borth-y-Gest and I agree with his views about this. I will only add that the circumstances in which the Police Act,
1919, was passed, and the consequent regulations were made, show that the regulations must have been intended to have a very
wide application and I see nothing unreasonable in applying them to this case. Dismissing a chief constable who has not been
convicted of any criminal offence is not a thing to be done lightly. If the whole of the matters against him are disclosed to him
and he refuses to admit some or all of them, it seems to me perfectly proper that there should be such an inquiry as the regulations
require. In particular to exclude this case from the ambit of the regulations because the watch committee did not proceed on any
report or allegation is a very narrow interpretation of the regulations and it would lead to a strange result. Counsel for the
respondents was constrained to admithe could not reasonably have done otherwisethat, if some busybody had formally
reported to the watch committee the observations of Donovan J and required them to deal with these allegations, then the watch
committee would have been bound to apply the regulations. But it would be absurd if the substantive rights of the appellant were
to depend on whether or not someone happened to have made a formal report or allegation to the watch committee before they
proceeded to deal with the case.
Then there was considerable argument whether in the result the watch committees decision is void or merely voidable.
Time and again in the cases I have cited it has been stated that a decision given without regard to the principles of natural justice
is void and that was expressly decided in Wood v Woad. I see no reason to doubt these authorities. The body with the power to
decide cannot lawfully proceed to make a decision until it has afforded to the person affected a proper opportunity to state his
case.
Finally there is the question whether by appealing to the Secretary of State the appellant is in some way prevented from now
asserting the nullity of the respondents decision. A person may be prevented from asserting the truth by estoppel but it is not
seriously argued that that doctrine applies here. Then it is said that the appellant elected to go to the Secretary of State and
thereby waived his right to come to the court. That appears to me to be an attempt to set up what is in effect estoppel where the
essential elements for estoppel are not present. There are many cases where two remedies are open to an aggrieved person but
there is no general rule that by going to some other tribunal he puts it out of his power thereafter to assert his rights in court; and
there was no express waiver because in appealing to the Secretary of State the appellant reserved his right to maintain that the
decision was a nullity. But then it was argued that this case is special because by statute the decision of the Secretary of State is
made final and binding. I need not consider what the result would have been if the Secretary of State had heard the case for the
appellant and then had given his own independent decision that the appellant should be dismissed. But the Secretary of State did
not do that. He merely decided that there was sufficient material on which the watch committee could properly exercise their
power of dismissal under s 191(4) of the Municipal Corporations Act, 1882. So the only operative decision is that of the watch
committee, and if it was a nullity, I do not see how this statement by the Secretary of State can make it valid.
81
Accordingly in my judgment this appeal must be allowed. There appears to have been no discussion in the courts below as
to remedies which may now be open to the appellant, and I do not think that this House should do more than declare that the
dismissal of the appellant is null and void and remit the case to the Queens Bench Division for further procedure. But it is right
to put on record that the appellant does not seek to be reinstated as chief constable: his whole concern is to avoid the serious
financial consequences involved in dismissal as against being required or allowed to resign.
LORD EVERSHED. My Lords, on the difficult problem presented by this appeal I regret to find myself differing from your
lordships; but I have felt myself constrained to agree with the conclusions reached by Streatfeild J, at the trial and by all the
members of the Court of Appeal.
It will be logical for me to deal first with the question whether the watch committee of Brighton Corporation were bound to
observe the requirements of what I will compendiously call the Police (Discipline) Regulations of 1952, before purporting to
exercise, as regards the appellant, the jurisdiction now admittedly vested in them by s 191(4) of the Municipal Corporations Act,
1882; for if they were so bound then, in the absence of such observance, it may be saidand was so contended on the appellants
partthat the watch committee had in truth no jurisdiction to reach their decision for the appellants dismissal. By the Police
(Discipline) Regulations of 1952 I refer compendiously to two statutory instruments, namely, (1) the Police (Discipline)
Regulations 1952, SI 1952 No 1705, and (2) the Police (Discipline) (Deputy Chief Constables, Assistant Chief Constables and
Chief Constables) Regulations, 1952, SI 1952 No 1706; as respectively amended by the two regulations, SI 1954 Nos. 1687 and
1688, being regulations of the same respective titles as those of 1952 already mentioned. The two instruments of 1952 were both
made on 17 September 1952, and came into operation on 1 October 1952. Similarly the two amending instruments of 1954 were
both made on 17 December 1954, and came into operation on 1 January 1955. The several instruments clearly form together a
single code. I shall have to make some references to them hereafter and I shall then refer to them respectively as Statutory
Instrument 1705 and Statutory Instrument 1706, the references being in each case intended to comprehend the amendments
made in 1954.
I have been unable to accept the argument that every case of indiscipline or of incapacity of any police officer whether a
chief constable or any other member of a police force (save only cases of incapacity arising from mental or physical illness) falls
or was intended to fall within the scope of the regulations. For my part I accept the view propounded by counsel for the watch
committee which appealed to the learned judge at the trial, that discreditable conduct and neglect of duty which constitute the
first and fourth headings in the discipline code set out in Sch 1 to Statutory Instrument 1705, should be construed as limited to the
kinds of conduct specified in those headings, each of which, be it observed, begins with the words that is to say. In the present
case the substance and gravamen of the appellants incapacity as chief constable, on which the watch committee proceeded to act,
was that expressed by Donovan J after presiding at a trial, lasting nineteen days, of the appellant and others charged with
conspiring to obstruct the course of public justice, namely that the appellant had not, in that learned and experienced judges
view, the professional or moral qualities requisite for one holding the office of chief constable. Although therefore the
appellant was himself acquitted of the charge, the learned judge thought it right to treat the appellants limitations, which he had
expressed, as justifying remission of the sentences otherwise appropriate to be passed on his two subordinate officers whom the
jury had convicted. As I understand the language of Donovan J (and as, I doubt not, the watch 82 committee also understood it),
the appellant had been shown not to possess a sense of probity or of responsibility sufficient for the office which he held, and so
had been unable to provide the essential leadership and example to the police force under his control which his office properly
required.
The first of the relevant headings in the discipline code, Discreditable conduct is thus defined:

that is to say, if a member of a police force acts in a disorderly manner or any manner prejudicial to discipline or
reasonably likely to bring discredit on the reputation of the force or of the police service.

I will not take time by reciting the more detailed expansion of the second heading, Neglect of duty g but I cannot (as could not
the learned judge in the first court) hold that the shortcomings of the appellant as chief constable, described by Donovan J fall
within either of the respective headings as expanded by their definitions. I add only that the two headings in question, being part
of the code specified in Statutory Instrument 1705, should be construed with regard to the fact that they were originally intended
to apply only to members of a police force of lower rank than chief constables or deputy or assistant chief constables though it is
true to say that by Statutory Instrument 1706 they were made applicable also to chief constables and deputy and assistant chief
constables.
________________________________________
g For this heading, see 17 Halsburys Statutory Instruments 201

My lords, it follows, in my opinion, that the watch committee were entitled to exercise their residual powers under s 191(4)
of the Act of 1882 without observance of the Police (Discipline) Regulations. I do not forget the terms of para (1)(f) of the watch
committees resolution of 7 March 1958. It is said that the subject-matter of this paragraph was that mentioned in para (c) of the
watch committees answer to the appellants appeal to the Secretary of State, namely, the suggestion that the appellant had given
false evidence at the trial before Donovan J; and that such a charge was in terms within para (b) of the heading numbered 5 in the
discipline code, namely, Falsehood or prevarication, that is to say, if a member of a police force wilfully makes any false
statement. But, assuming the premise, it is nevertheless, in my opinion, still clear that the reference was but to an incident in
the trial on the conclusion of which the watch committee were manifestly founding themselves; and must have been so
understood. I cannot think that such an incidental reference can sensibly have the startling result of making the watch
committees jurisdiction dependent on a strict application of the Police (Discipline) Regulations. It follows, if I am right in
thinking that the case against the appellant did not fall under any of the provisions of the discipline code, that it was not brought
within the regulations by cl 11 of Statutory Instrument 1706.
I also find myself in agreement with all the learned judges below in thinking that in any event this was not a case of there
having been a report or allegation to the watch committee as contemplated by the regulations. I have, for my part, been unable
to accept counsel for the appellants argument that any deliberation by the watch committee necessarily supposes the presence of
a report or allegation by someone. I do not attempt any definition of the phrase; but, in my opinion, the context of the
regulations suggests necessarily something in the nature of an accusation as distinct from a conclusion reached after proper
inquiry, and cannot sensibly be said to include a judicial conclusion after the protracted investigation of a trial. If this view be
wrong I would ask your lordships to observe the consequences. If counsel for the appellants submission be accepted, it must
follow (as counsel for the appellant indeed conceded) that the regulations were or would be equally applicable to any disciplinary
action taken by the watch committee in regard to Detective Sergeant Heath and Detective Inspector Hammersley, each found
guilty at the trial and sentenced to terms of imprisonment. If the argument submitted be correct, 83it must follow that the watch
committees duty must be or have been (notwithstanding the conclusion of their trial) to refer the cases of these two officers to an
Investigating Officer under Statutory Instrument 1705 who would report to the chief constable (sic), the officers having a
right of appeal to the watch committee. And so, on this view, it was the watch committees duty, as regards the appellant, under
Statutory Instrument 1706 first to instruct a solicitor to formulate the case against him; and then to appoint a tribunal (which
might consist of five members of the watch committee itself) whose duty it would be to report to the watch committee. My
Lords, I cannot think it right to accept an argument involving results which appear to me so manifestly absurd. In the present
case the conduct of the appellant had been the subject of a public trial lasting nineteen days; and, if the observations of Donovan
Jh, can sensibly be called a report at all, they were equivalent to the report of the investigating tribunal to the watch
committee contemplated by Statutory Instrument 1706. In this respect the situation after the conclusion of the trial was to my
mind wholly different from that at the time when the appellant was first charged and when, therefore, as it seems to me, the watch
committee rightly felt itself bound to act in accordance with the regulations in ordering the appellants suspension. My lords, any
other view, in my opinion, makes the regulations gravely offend against common sense. I agree therefore with the view of the
Court of Appealand particularly with that of Holroyd Pearce LJ ([1962] 1 All ER at p 843), that this was a special and entirely
exceptional case, outside the scope of the regulations and, as a matter of public notoriety, requiring instant action by the watch
committee. The extent of the public notoriety can fairly be gauged from the letter written by the appellants solicitor explaining
the remarkable request for his clients reinstatement as chief constable by reference to the telephone calls and offers of rewards
by newspapers to which he had been incessantly subjected.
________________________________________
h These are quoted in [1962] 1 All ER at p 837, letter b

I turn accordingly to what have appeared to me to be the most difficult questions raised in this appeal; that is to say, first,
whether the exercise of the statutory jurisdiction by the watch committee, which in my opinion was vested in them without regard
to the regulations, required the observance by the watch committee of what are called the principles of natural justice; and,
second, if so, whether on the facts of this case such principles were in fact observed.
It has been said many times that the exact requirements in any case of the so-called principles of natural justice cannot be
precisely defined; that they depend in each case on the circumstances of that case. According to Sir Frederick Pollock the
meaning of the phrase natural justice is the ultimate principle of fitness with regard to the nature of man as a rational and
social being; and he went on to point out that the origin of the principles could be traced to Aristotle and the Roman jurists.
Your lordships were therefore not unnaturally referred to a great many cases, but as I believe that your lordships agree, it is by no
means easy to treat these decisions as entirely uniform and still less easy to be able to extract from them the means of
propounding a precise statement of the circumstances or of the cases in which the principles can be invoked before the courts. I
am, however, content to assume that the invocation should not be limited to cases where the body concerned, whether a domestic
committee or some body established by a statute, is one which is exercising judicial or quasi-judicial functions strictly so called;
but that such invocation may also be had in cases where the body concerned can properly be described as administrativeso long
as it can be said, in Sir Frederick Pollocks language, that the invocation is required in order to conform to the ultimate principle
84 of fitness with regard to the nature of man as a rational and social being. On the other hand, it is (as I venture to think) no less
plain now that Parliament may by appropriate language in a statute make it clear that the activity or discretion of the body
constituted by the statute is not to be subject to any control or interference by the courts.
At this stage I venture to make two points. First, since there is no question here of bias or any suggestion that the watch
committee acted otherwise than entirely in good faith, the only principle of natural justice here involved is that enshrined in the
Latin phrase Audi alteram partem. Second, I for my part conclude that if the principles of natural justice can properly be
invoked in this case and if it should be held that such principles were not observed, then the decision of the watch committee was
not void but voidable only.
On this second question (whether the decision afterwards impugned can be said to be void or voidable only) the cases
provide, as I think, no certain answer; nor have I found one in the text-books. Indeed in the vast majority of circumstances, it
does not in the end matter whether the decision challenged is void or only voidable; for, if the court does decide to quash a
decision or otherwise set it aside, then the effect is in general the same whether such decision be considered as void or only
voidable. For my part, however, I have come to the conclusion that in a case where a body is acting within its jurisdiction but of
which the courts will say that it has failed to do substantial justice in accordance with the principles of natural justice, then the
decision is only voidable and cannot properly be described as a nullity.
Though I am in this respect anticipating what later follows, I refer first to the extremely wide and general terms of the
relevant subsection of the Act of 1882:

The watch committee may at any time dismiss any borough constable whom they think negligent in the discharge of
his duty or otherwise unfit for the same.

My lords, in my opinion it is impossible to accept the suggestion put forward on the appellants part that the final words of the
subsection, otherwise unfit for the same, must be regarded as ejusdem generis with what has gone before; that is to say, that
they are intended only to refer to some kind of negligence. I know of no authority in a case of this kind, where the jurisdiction of
the body in question is expressed in two alternatives, for supposing that the second of the alternatives does not mean what it says
but is somehow limited by the terms of the first alternative. Put in other words, where, may I ask, is the genus to be found of
which the second part of the alternative is said to be but a part? As I interpret the language used, the second part of the power
conferred is, as the language inevitably imports, intended to cover the case of someone who is regarded as unfit for his position
for reasons other than negligence.
I observe again that there is, as I think, no question here of the watch committees jurisdiction. If I had taken a different
view on the first question, namely, whether the exercise of the jurisdiction had to be subject to compliance with the regulations,
then my answer might have been different. On that assumption it is unnecessary for me to express any concluded opinion and I
do not do so. But, if I am right in thinking that there was here no question of compliance with the regulations, the only question
for your lordships is whether, admitting the jurisdiction of the watch committee, it was properly exercised having regard to any
application of the principles of natural justice to which the exercise of the jurisdiction was subject. I observe further that the
appellant in his action seeks a declaration. There was some discussion before your lordships concerning the office of a
declaration, as contrasted with that of an order for certiorari. In my judgment it must be accepted as tolerably clear that (subject
to what follows) the granting of a declaration in a case of this kind must prima facie be discretionary: and if that is so it must
equally follow that the question whether the decision of the watch committee is such that the court can quash it or otherwise
interfere with it involves the conclusion that such decision was 85 voidable and not void. If the decision was a complete nullity
(for example, on the ground that the watch committee never had any jurisdiction) then no doubt it would follow that the court
would have to say so in some form or other. But this is not, in my judgment, such a case. My lords, I have in mind on this matter
decisions such as that of the Privy Council in the recent case of Annamunthodo v Oilfield Workers Trade Union, where the
plaintiff had been expelled from the union and the board thought that the expulsion was wholly invalid. In fact, however, in that
case the plaintiff had been charged under one rule but was later expelled under another. It is also to be noted that the case was
dealing not with powers conferred by Act of Parliament but with a domestic tribunal; the point submitted (but rejected) was that
the appeal which the plaintiff had taken under the rules of the union constituted an affirmance of the jurisdiction of the council,
which had dismissed him. I have also in mind the case of Wood v Woad. My noble friend Lord Reid has stated in his opinion i
that in this case the Court of Exchequer Chamber expressly decided that a decision by a body acting in a quasi-judicial capacity
which failed to have due regard to the principle of natural justice audi alteram partem, is void and not merely voidable. With all
respect to my noble friend, I am unable to agree with this conclusion.
________________________________________
i See p 81, letter e, ante

My lords, it is, I think, necessary to have in mind what was the nature of the plaintiffs claim in Wood v Woad. The plaintiff
alleged that he had been a member of a mutual marine insurance association and, as such member, having paid to the treasurer the
appropriate deposit, had therefore been entitled to recover from the association the amount of loss incurred by him in respect of a
particular ship; that in the committee of the association was vested by its rules the whole power of the management of its affairs
and also the power, if they deemed the conduct of any member to be suspicious or that he was for any other reason unworthy of
remaining in the association, to exclude such member by appropriate notice from further participation in it; but that the
committee had wrongfully, collusively and improperly expelled the plaintiff from the association without any just reason or
probable cause; so that the plaintiff had been deprived of his right to the sum of money in respect of the damage done to his ship
and that he was accordingly entitled to recover as damages from the members of the committee the amount of such loss. This
being the nature of the claim the court decided on demurrer that the plaintiff could have, on his allegations, no cause of action for
damages at law against the members of the committee. I emphasise the important fact that the claim formulated was for damages
at law against the members of the committee (not all of whom in fact were or need have been members of the association).
It is clear from the headnote to the case that the learned barons of the Court of Exchequer Chamber did not arrive for
entirely the same reasons at the conclusion that the plaintiff could not succeed in his action. It is true that certain language in the
judgment of Kelly CB appears to support the view that in his opinion the committees failure to give to the plaintiff any
opportunity of answering the charge made by the committee against him rendered the committees decision void and a nullity.
See, for example, the chief barons citation of the decision in Blisset v Daniel. But if so, it was, in my judgment, because in the
view of the chief baron there was enough to show a collusive and unlawful exercise of power on the part of the committee
((1874), LR 9 Exch at p 198)in other words not a true exercise of the power at all or, at best, an exercise of the power of the
exceptional kind to which I later refer in the case of Osgood v Nelson. It is essential to have in mind the nature of the plaintiffs
claim as formulated by him which the Court of Exchequer Chamber rejected. On the one hand (as Kelly CB 86 pointed out
((1874), LR 9 Exch at p 196) if the discretion of the committee was absolute and if the committee in fact exercised their power
under the rules, the plaintiff could not question it. On the other hand, if, as the plaintiff in his declaration alleged, the committees
act was collusive and unlawful and therefore ineffective then the plaintiff remained a member of the association and (whatever
might be his rights or remedies in a court of equity) he therefore could have no claim for damages in law against the committee
((1874), LR 9 Exch at p 198):

The claim in this action is for damages sustained by reason of the expulsion of the plaintiff from the association; but in
law the plaintiff has sustained no damage at all, for whatever rights he may have possessed before he possesses still, as if
no action had been done calculated to deprive him of them.

Cleasby B, put his conclusion on somewhat different grounds ((1874), LR 9 Exch at pp 198, 200):

Now, we may suppose either that the committee expelled the plaintiff without just cause and without giving him
notice, or that they expelled him without just cause but did give him notice; and the declaration is framed so as to
comprehend in the breach both modes of wrongful expulsion.

After pointing out that by the rules the committee had absolute discretion, the learned baron concluded his judgment by saying
that the allegation not having made fraud the basis of the claim the declaration sought could not be sustained. Pollock Bs
judgment ((1874), LR 9 Exch at pp 201203) was to the same effectparticularly in respect of the absence of any claim based in
terms of fraud. The learned baron went on to observe that the plaintiffs declaration having alleged that the committees actions
were a nullity it was not on this premise possible for him to formulate a cause of action at law against the committee members.
Finally, Amphlett B, posed the matter thus ((1874), LR 9 Exch at p 204):

Now according to the allegations in the declaration, the defendants never gave the plaintiff that opportunity, and I
cannot entertain a doubt that if this allegation were proved, the plaintiff would, by filing a bill in a court of equity, be
restored to the enjoyment of his rights. But if so, what is his damage? He has not ceased to be a member of the society; he
has not lost the rights of a member. He is to recover damages for what? For an attempt to expel.

I have attempted at some length to analyse the reasons for the judgments of the Court of Exchequer Chamber in Wood v Woad. It
is, as I have more than once observed, of the essence of the matter in that case that the plaintiff was claiming damages personally
against the members of the committee. In such circumstances it is, as I venture to think, clear that the question whether the
purported exclusion from the association by the committee was void or voidable was not essential nor indeed material to his
claim made in the action by the plaintiff for damages against the members of the committee. Certainly in my judgment it cannot
be asserted that the judgments in the case cited, or indeed any of them, support or involve the proposition that where a body, such
as the watch committee in the present case, is invested by the express terms of a statute with a power of expulsion of any member
of the police force and purport in good faith to exercise such power, a failure on their part to observe the principle of natural
justice audi alteram partem has the result that the decision is not merely voidable by the court but is wholly void and a nullity.
My lords, I have for my part on this question, derived the greatest assistance from the case of Osgood v Nelson in which
Baron Martin gave ((1872), LR 5 HL at pp 645648) to your Lordships House the opinion of the judges. The case was
concerned with the 87 removal of the chief clerk or registrar of the Sheriffs Court in the City of London. By the Act 15 and 16
Vict c lxxvii power had been given to

the mayor, aldermen and commons in common council assembled for inability or misbehaviour or for any other
cause which may appear reasonable to the mayor and council to remove

a person in the position of the chief clerk or registrar. It was the unanimous opinion of the judges expressed by Baron Martin
((1872) LR 5 HL at pp 646, 647) that there was no doubt that the courts of law in this country would take care that any
proceeding of this kind should be conducted in a proper mannerthat is by giving to the person whose removal was in question
every opportunity of defending himself. If continued the learned Baron ((1872), LR 5 HL at pp 646, 647) your lordships are
satisfied that there was a real substantial miscarriage of justice, your lordships would not permit this amotion to remain.
Again ((1872) LR 5 HL at pp 646, 647):

we also think that it is possiblethough there is no necessity for giving any judgment upon itthat if a man was
removed from an office of this kind for any frivolous or futile cause you would in all probability be inclined to think the
removal is a nullity.

Lord Hatherley C, in adopting the view so expressed of the judges said ((1872), LR 5 HL at p 649):

The Court of Queens Bench, has always considered that it has been open to that court, as in this case it appears to
have considered, to correct a court or tribunal or body of men who may have a power of this description if it should be
found that such persons have disregarded any of the essentials of justice.

From these citations I deduce the conclusion that, save in the case where the tribunal or body of men have acted on frivolous
or futile grounds (in which case the court may treat not merely the decision but the whole proceeding as a nullity) the power of
the court is to correct the decision if, in the courts view, there has been real substantial miscarriage of justice. In other words
I think that, save in the excepted cases (of which the present cannot be said to be one) the right or duty of the court is to correct,
that is to set aside or otherwise restrain, the impugned decision if satisfied that there has been a real substantial miscarriage of
justice; a view which, if well-founded, must mean that (save in the excepted cases) the decision is voidable and not void.
My lords it is perhaps useful and necessary to inquire what in truth is meant by saying that a decision such as that of the
watch committee in the present case, is void or a nullity. Is it thereby intended that, though the proceedings up to the
pronouncement of the decision were proper and effective, the decision itself was a nullity? Or is it intended that the whole
proceedings ab initio were irregular and ineffective so that the decision was similarly and of necessity also of no effect? My
lords, the latter must in my judgment be the true analysis. In the first place, it does not to my mind appear correct or indeed
sensible to say that the decision reached was a nullity although the proceedings leading up to the decision were in order.
Second, I observe as I have earlier stated, that in cases of this kind it is not the function of the court to impugn the decision as
suchstill less to substitute its ownbut to examine the steps taken in reaching the decision and to decide whether, in the course
of those steps, there was a real substantial miscarriage of justice.
In the vast majority of cases it matters not in the result whether the decision is said to be void or voidable but avoided. It is
sufficient for the court to say that the decision cannot stand. In truth, as Sir Frederick Pollock pointed out (see Pollock on
Contract, 13th Edn, p 48) the words void and voidable are imprecise and apt to mislead. And so it is, as I venture to think,
that language such as that used by Kelly CB in Wood v Woad ((1874), LR 9 Exch at p 198) ought not to be strictly 88 construed
it was, indeed, for reasons which I have attempted to give, in any case obiter having regard to the nature of the claim in that case.
I do not doubt that in some cases the proper conclusion will be that the entire proceedings of the body or tribunal in question
(including therefore its decision) will properly be found to be wholly irregular and ineffective from first to last. The obvious case
is where the body or tribunal is shown to have been acting in excess of its jurisdiction. In this category no doubt will fall the
class of case mentioned by Martin B, in Osgood v Nelson ((1872), LR 5 HL at p 647) where the body concerned has acted on a
futile or frivolous cause; for in such case it could truthfully be said that the invocation by the body of its power was a pretence
and its proceedings no more than a sham. It may indeed well be that Kelly CB so regarded the performance of the committee in
Wood v Woad. But save in those cases, as I think, on true analysis the function and duty of the court is to correct, that is to say,
to set aside or quash the decision where it is shown that there has been some real substantial miscarriage of justice in the steps
taken by the body or tribunal in question in arriving at its decision in exercise of the powers vested in it.
My lords, I do not wish unduly to prolong this opinion but on this highly important matter it seems to me that useful analogy
may be found in the practice of the criminal courts. Thus, the Court of Criminal Appeal in the exercise of its powers under the
Criminal Appeal Act, 1907, may quash a conviction and substitute a verdict of acquittal, and may do so where there has at the
trial been what is regarded in effect as a failure to observe the principles of natural justice, eg where the jury has been told that it
must return with its verdict in ten minutes or where the jury was allowed after retirement under the bailiffs control to depart from
the court for luncheon. In these cases it is essential that there should have been an effective trial at least up to the point where the
departure from the principles of natural justice occurred; for otherwise the appeal court could not have ordered the prisoners
acquittal. If in truth the prisoner had never been really tried at all, he would be liable to be tried again for the same offence and
the appeal court could have issued a venire de novo. The second of the examples above given (ie where the jury had been
allowed to leave the court for luncheon) was involved in the case before the Court of Criminal Appeal of R v Neal and the point
with which I am concerned was in terms dealt with by Lord Goddard CJ ([1949] 2 All ER at p 441; [1949] 2 KB at p 596). I
would also refer your Lordships to the judgment of the Privy Council delivered by Lord Sumner in the case of R v Nat Bell
Liquors Ltd.
I only add that, as I apprehend, the same principles apply to an order for certiorari which has been held to be available, but at
the direction of the court, by way of declaration and an injunction in cases of decisions by statutory tribunals where the persons
affected would otherwise be without remedy (see Barnard v National Dock Labour Board following Andrews v Mitchell). On the
other hand it has also been held that certiorari will not be granted where the proceedings in the inferior tribunal are not merely
voidable but altogether voideg where the person purporting to act in a judicial capacity had in truth no authority so to do (see
Re Daws).
Finally, I venture to pose to your lordships the question, what would have been the situation had the Secretary of State
allowed the appellants appeal and held that he should be reinstated as chief constable? Would it have been open to the
corporation to refuse to give effect to such decision on the ground that the proceedings or the decision before or by the watch
committee had been a nullity?
89
I return accordingly to the first of the points previously mentioned, namely, the question whether the watch committee in
exercising its powers under the relevant section of the Act of 1882 was in the present circumstances bound to give to the
appellant an opportunity of putting forward his case and arguments before the committee. I have already said that the terms of te
discretion vested in the watch committee by the Act of Parliament have seemed to me to be of the widest. They are, as I think,
much wider than the phrase appearing in the case, to which I shall later refer, of De Verteuil v Knaggs, where the relevant
language was If it appears to the governor on sufficient grounds shown to his satisfaction. I also think that the language in
the Act of 1882 was at least as wide if not wider than the relevant language in the case of Nakkuda Ali v M F de S Jayaratne on
which the Court of Appeal considerably relied, namely [where] the controller has reasonable grounds to believe that any dealer
is unfit to be allowed to continue as a dealer. I understand that some of your lordships feel disposed to say that the decision of
the Privy Council in that case ought not to be followed. I must respectfully dissent from that view. It seems to me that on the
language of the enactment there in question there was in truth conferred on the governor an unfettered discretion. I am aware that
it is sometimes said that a different result may be appropriate where there is in question the grant or withdrawal of a licence as
distinct from the taking away of some right or proprietary interest. There is no doubt force in this argument and it has been
supported by our Court of Criminal Appeal in the case of R v Metropolitan Police Commissioner, Ex p Parker. At the same time
I would observe that though the withdrawal of a licence, which can be described as the removal of a privilege, is in some respects
different in character from the taking away of vested rights or proprietary interests, nevertheless the withdrawal of a licence from
the person from whom it is withdrawn may in fact mean the destruction of his means of livelihood.
My lords, if the only question which was here involved was whether the appellant should have ceased in March, 1958, to be
chief constable and whether therefore the watch committee was entitled without more so to decide I would, for my part, say that
there was in the circumstances no call for the watch committee to observe the so-called rule audi alteram partem. As I have
already observed the appellant had been subjected to a trial lasting nineteen days and it was as a result of the evidence in that trial
that Donovan J expressed the view that the appellant was in fact no longer fitted to act as chief constable. It would seem to me
frankly somewhat absurd that the watch committee should invite the appellant to state his points again after he had put forward a
case before the trial judge for so long a period. Moreover, as Holroyd Pearce LJ ([1962] 1 All ER at p 843), pointed out (and as I
have earlier noted), there was here a case of extreme urgency. The trial had attracted the greatest possible notoriety as had also
the observations in regard to the appellant of the trial judge. In my judgment the watch committee had a dutya duty not only to
the corporation of which they were a committee but also to the citizens of Brightonto act and to act at once so as to give effect
to what the trial judge had after so long a hearing in effect determined.
But my difficulty in the present case arises over the question of the appellants pension. For assuming it to be right that the
appellant would have to cease to be chief constableand I add in regard to that matter the not unimportant fact that his learned
counsel has not before your lordships suggested that he should have been retained as chief constablethen there were two ways
in which his appointment might be determined. First, he might have been required to resign in which case, though he would have
had so to resign, he would under the terms of the Police Pension Regulations, 1955, j have been entitled to receive the pension
which by that date had accrued in his favour. The alternative was the 90 appellants summary dismissal which was the course
adopted; though I do observe that in the watch committees minute of 7 March 1958, it is recorded that they had paid regard to
the length of the appellants service.
________________________________________
j See SI 1955 No 480, reg 52

It is undoubtedly a striking fact that the appellant had at the date when he had been suspended from his office of chief
constable served some 33 1/2 years and had risen from the rank of police constable through the various intervening ranks to that
of chief constable. During this long period of service it does not appear that there had ever been any criticism of his work in the
police force. Moreover, in March, 1958, he had attained the age of fifty-eight years and ten monthsin other words he was
within fourteen months of the age on which he would have been entitled to retire voluntarily with full pension. In these
circumstances I cannot conceal from myself that (unless the words of the statute deny it) there is shown an obvious case for
giving to the appellant an opportunity to put forward his argument for the first of the two alternatives, namely, that he should be
required to resign and not be summarily dismissed.
As I have said, I feel very great difficulty on this matter. I do not wish at all to denigrate the principles of natural justice or
of their proper invocation in the courts. On the other hand, we have, as I have already many times pointed out, the very wide
terms of the Act of Parliament here in question and the body in which was invested this wide discretion was an entirely
responsible body. To insist, as I venture to think, on the invocation of these principles whenever anyone is discharged from some
office seems to me to involve a danger of usurpation of power on the part of the courts and under the pretext of having regard to
the principles of natural justice to invoke what may often be in truth little more than sentiment; and on occasions when the courts,
though having necessarily far less knowledge of all the relevant circumstances, may be inclined to think that, had the decision
rested with them, they would have decided differently from the body in question. Yet I do observe again that it is not the decision
as such which is liable to review; it is only the circumstances in which the decision was reached and particularly in such a case as
the present the need for giving to the party dismissed an opportunity for putting his case.
I should therefore, for my part have been inclined to accept the view of the learned lord justices in the Court of Appeal. But
since, as I understand, your lordships take a different view and having read the exhaustive opinion of my noble and learned friend
Lord Reid, I will on this matter, express no concluded opinion of my own. I am ready to assume that the appellant should have
been given an opportunity at any rate to put his case for being required to resign rather than being summarily dismissed. If this
assumption be made, then the second question ariseswas justice done in all the circumstances of this case? My Lords, having
considered the whole matter with the greatest care of which I am capable I conclude that justice was here doneor, at least, that
there was no real substantial miscarriage of justice. I have already observed that in their minutes of 7 March 1958, the watch
committee (against whose good faith no kind of charge is made) stated that they had paid due regard to the length of the
appellants service. But at once after the receipt of notice of that resolution the appellants solicitor, Mr Bosley, asked the watch
committee to reconsider the matter and to hear observations that might be put forward on it. In acceding to this request it is to my
mind plain that the watch committee (and the corporation) thereupon decided that, notwithstanding their resolution of 7 March
the matter of the apellants dismissal should be held in suspenseand indeed after they had been informed of the appellants
appeal to the Secretary of State they also clearly decided that the operation of their previous resolution should be suspended
pending the result of that appeal.
It is clear that on 12 March the town clerk informed the appellants solicitor that the watch committee would reconsider their
decision with regard to the appellants pension and further told him that a meeting would be held on 18 March at which the
committee would consider such representations as might then be 91 made by or on behalf of the appellant either orally or in
writing, or both as may be preferred. Thereupon Mr Bosley sent to the town clerk his written observations dated the same day in
which in fourteen numbered paragraphs he set out the heads of his clients complaints; and in para 15 he asked, first, that the
appellant should be allowed to retire on full pension forthwith. Mr Bosley also sent to the town clerk a copy of his notice of
appeal to the Secretary of State, a lengthy document in which every kind of complaint made on the appellants behalf was
enumerated, though I would observe that nowhere in that document was any specific claim made that the appellants case really
was that of his being summarily dismissed instead of being requested to resign with the consequent right to receive his pension.
As a result there was a further special meeting of the watch committee held on 18 March. An extract from the minutes of that
meeting was duly sent to Mr Bosley and from the minute it is clear that the watch committee had given to Mr Bosley the fullest
opportunity to make such representations as he should think fit; and it is also recorded that the committee having heard all that
Mr Bosley had to say and considered also his written representations and the notice of appeal to the Home Secretary had decided
to adhere to their previous decision; though it is noted that there were three dissentients on this occasion.
My lords, having regard to all the circumstances, I have formed the view that your lordships ought not now to say that a
sufficient opportunity was not given to the appellant by himself or through his adviser to put before the watch committee such
points as he had and in particular to put before the watch committee the request that he should be required to resign rather than be
summarily dismissed. I therefore respectfully agree on this matter with the conclusion of Streatfeild J ([1961] 2 All ER at p 536),
at the trial of the present proceedings, and also with what I understand and believe to have been the view of Harman LJ as
reported ([1962] 1 WLR at p 735; cf [1962] 1 All ER at p 850) (though the language as there recorded does not contain, as I think
from a reading it should have contained, a negative). In reaching this conclusion I have derived support from the case of De
Verteuil v Knaggs above referred to. In that case the governor of Trinidad had acted in emergency with promptitude but without
giving to the person concerned any opportunity for a hearing. In the circumstances it was pointed out by Lord Parmoor ([1918]
AC at pp 560, 561) that this might well be justified provided that there was opportunity given afterwards when the original
decision might be reviewed. Similarly, in my view the present case was indeed one of grave emergency calling for the greatest
promptitude of action. But for reasons which I have attempted to state I think that assuming in the first place there was any
failure to observe the principles of natural justice by giving to the appellant an opportunity of being heard, this defect was
remedied afterwards when the original decision was suspended and an opportunity given to the appellant or his adviser or both to
make to the watch committee such representations as they wished.
But if I were wrong on the point last mentioned still in my opinion the appellant fails in the end on another point, namely, by
reason of the consequences of his appeal to the Secretary of State. This matter was also dealt with by the learned judges of the
Court of Appeal who similarly concluded that in any event the appeal to the Secretary of State barred the appellant from claiming
relief now. The case appears then to have been put on the basis of estoppel or election. For my part I prefer to rest my
conclusion simply on the terms of the relevant section in the Act itself. The Act is the Police (Appeals) Act, 1927a date, be it
noted, eight years later than the date of the Police Act, 1919, under which were promulgated the SI 1952 Nos. 1705 and 1706.
By s 1(1) of the Police (Appeals) Act, 1927, k, a member of a police force who after the passing of this Act is punished by
dismissal was given the right to appeal to the 92 Secretary of State. It is also by the Act provided that the Secretary of State
is not bound to entertain the appeal by way of hearing oral evidence if it appears to him that the case is of such a nature that it can
properly be determined without such evidence. This was in fact the course adopted by the Secretary of State in the present
circumstances. By his order of 5 July 1958, after reciting that an appeal had been made against the watch committees decision
of the previous March it is recorded that the Secretary of State having decided that the case is of such a nature that it can
properly be determined without taking oral evidence hereby order as follows; and then, in para 1, occurs the language I dismiss
the appeal.
________________________________________
k The enactment is printed as amended by the Police (Appeals) Act, 1943, s 1

It was not, as I followed the argument, suggested that the Secretary of State was acting otherwise than within the jurisdiction
conferred on him by the Act of Parliament in deciding to dispose of the appeal as he did on the written material before him and
without hearing oral argument. Indeed s 2(2) of the Act of 1927 provides thus:

The Secretary of State after considering the notice of appeal and any other documents submitted to him by the
appellant and the respondent and the report (if any) of the person or persons holding the inquiry shall by order, either
(a) allow the appeal; or (b) dismiss the appeal; or (c) vary the punishment

As it seems to me the action taken by the Secretary of State was in strict compliance with his powers and duties under the Act
and, with all respect to those who may take a contrary view, I cannot see how it can be said that the order of the Secretary of State
is ex facie unsustainable. What then is the result? By s 2(3) of the Act of 1927 it is provided that the decision on such appeal by
the Secretary of State is to be final and binding upon all parties. I agree that if it had been made out that the proceedings of the
watch committee were a nullity, then the appeal and the result of the appeal might well be regarded equally as a nullity. But for
reasons which I have endeavoured to justify it is in my opinion not true to say that the decision of the watch committee was a
nullity even if there was a failure on their part to obey the rules of natural justice by their omission to give to the appellant proper
opportunity to be heard. Their decision was voidable only. This being so, then the appellant having invoked his right under the
statute to appeal to the Secretary of State must, as I conceive, be bound by the result which Parliament has enjoined: and that
result is that after such an appeal the Secretary of States decision shall be final and binding as between himself and the watch
committee. I cannot imagine any language more explicit. Nor does it seem to me that the result can be avoided because both in
his original letter to the Secretary of State and in the documents stating his grounds of appeal itself the appellants solicitor stated
that his invocation of the power to appeal was without prejudice to his right thereafter to maintain that the watch committees
decision was in some way wrong in law. In my judgment the appellant invoked his right to appeal to the Secretary of State
under the Act of 1927 and, having done so, cannot escape the consequences which, as it seems to me, Parliament has stated in the
plainest terms.
It follows therefore that, whatever might be the right answers to the difficult questions involved in regard to the application
of the rules of natural justice, the appellant by proceeding as he did under the Police (Appeals) Act, 1927, to appeal to the
Secretary of State cannot now say other than that the conclusion of the Secretary of State which was entirely in accordance with
his statutory powers was a final and binding conclusion which put an end to any right that the appellant might otherwise have had
to invite the court in the exercise of its discretion to set aside or otherwise interfere with the watch committees decision.
My conclusion, therefore, with all respect to your lordships who take a different view, is, first, that there was in the present
case no requirement that the watch committee should observe the terms of the Police (Discipline) Regulations of 931952 and
therefore that the jurisdiction lay under s 191(4) of the Municipal Corporations Act, 1882, with the watch committee; that
therefore (second) the most that could be said against the watch committees decision was that by failing to observe the rules of
natural justice it was liable to be challenged and impugned in the courts; but (third) for reasons given, that assuming that there
was a failure to comply with the rules of natural justice in the first instance by omitting to give to the appellant the right to be
heard (before the passing of the resolution of 7 March 1958), that failure was afterwards remedied; in other words having regard
to the entirely exceptional circumstances that it cannot now be said on the appellants part that there was any real or substantial
injustice in what was done by the watch committee. But, fourth, it is my opinion that if in all other respects I am wrong the result
of invoking the Act of 1927 by way of appeal to the Secretary of State involved necessarily the result that the Secretary of States
conclusion must be regarded by your lordships as having finally disposed of all questions between the appellant and the watch
committee.
There was also raised on the appeal before your lordships a question of the true interpretation of s 220 of the Act of 1882.
So far as relevant that section is as follows:

A conviction, order, warrant or other matter made or done or purporting to be made or done by virtue of this Act shall
not be quashed for want of form, and shall not, unless it is an order of the council for payment of money out of the borough
fund, be removed by certiorari or otherwise into the High Court.

It was contended on behalf of the appellant that the terms of the section were only applicable to cases in which the question was
as regards purely formal matters. For my part I am not persuaded that this is a right construction of the words which Parliament
has used. But I prefer on this matter not to express any concluded opinion. If the view which I have tried to express and justify
were right it would follow that this section would not be a relevant consideration. Further than that, since the point was never
taken by the watch committee until the course of the argument before your lordships, it would as it seems to me, in any event be
too late for the watch committee to rely on this section if in other respects they were wrong. I therefore say no more on this
matter.
If the matter rested with me, my lords, I would dismiss the appeal.

LORD MORRIS OF BORTH-Y-GEST. My Lords, the appellant who in March, 1958, was nearly fifty-nine years of age
became a constable in the Brighton Borough Police Force in 1925 after a short period of service in another police force.
Thereafter he received progressive promotions in the Brighton Police Force. In 1935 he became a detective sergeant and in 1948
a detective inspector. In 1949 he was made detective chief inspector and in 1950 detective superintendent. In 1954 he was
promoted to be deputy chief constable. In the early part of 1956 there was a vacancy in the office of chief constable. The
appellant was an applicant for the appointment. He was one of five candidates who were interviewed by the watch committee.
The committee who had the opportunities for judging of the competence of the appellant which his prior service afforded them
resolved that subject to the approval of the Secretary of State the appellant should be appointed. He was so appointed. Amongst
other terms and conditions the appointment was to be subject to the Police Acts and Regulations.
In October of the following year the appellant and two police officers and two others were arrested. The allegation was one
of conspiracy to obstruct the course of public justice. The watch committee (who are the police authority) then took action under
the provisions of the Police (Discipline) (Deputy Chief Constables, Assistant Chief Constables and Chief Constables)
Regulations, 1952 (SI 1952 No 1706), as amended by SI 1954 No 1688. They decided to suspend him from duty as notified in a
letter dated 29 October 1957; they resolved in accordance with 94 reg 15 to pay him certain suspension allowances. The opening
part of reg 15(1) as amended provides that:

Where a report or allegation is received from which it appears that a deputy chief constable or assistant chief constable
of a borough police force may have committed an offence against discipline or a criminal offence the police authority may
suspend him from duty until such time as either
(a) it is decided that he shall not be charged with an offence against discipline, or
(b) the disciplinary proceedings referred to in these regulations are concluded.

Though that regulation refers to a deputy or assistant chief constable it may under certain circumstances be invoked in the case of
a chief constable. This is as a result of reg 18 which provides:
Where a report or allegation is received from which it appears that a chief constable may have committed an offence,
these regulations shall apply with the following modifications, adaptations and exceptions:

The regulations provided that the expression offence had the same meaning as it has in the Police (Discipline) Regulations,
1952. Regulation 1(1) of those regulations provides:

A member of a police force commits an offence against discipline (hereafter in these regulations referred to as an
offence) if he commits one or more of the offences set out in Sch. 1 hereto (hereafter in these regulations referred to as the
discipline code) or such additions thereto as may be made by the police authority for the police force with the consent of
the Secretary of State.

The position in October, 1957, was therefore that the watch committee suspended the appellant under reg 15 which was
applicable on the basis that the watch committee had received a report or allegation from which it appeared that the chief
constable might have committed one or more of the offences set out in the discipline code contained in the Police (Discipline)
Regulations, 1952. It followed that the suspension would continue either until it was decided that he would not be charged with
an offence against discipline or until any disciplinary proceedings were concluded. In December, 1957, the appellant was
committed for trial. An indictment dated 7 January 1958, charged him with the offence of conspiracy to obstruct the course of
public justice. The particulars alleged that he conspired with the four other accused and with other persons unknown to obstruct
the course of public justice in that the appellant and the two police officers accused should act contrary to their public duty as
police officers in relation to the administration of the law. The conspiracy was alleged to have been between 1 January 1949, and
18 October 1957. The trial began on 3 February 1958, and after a hearing which lasted for some nineteen days the appellant was
acquitted. That was on 27 February 1958. On 28 February his solicitors by letter requested the watch committee to remove his
suspension and to reinstate him. On that day two police officers who had been convicted by the jury were sentenced, and in
passing sentence the learned judge made certain observations in regard to the appellant. A second indictment had been preferred
against the appellant. The charge was that being a person serving under the Crown he corruptly obtained a gift of 20 from a
named person as a reward for showing favour to such person in relation to the affairs of the Crown. The appellant stood his trial
on that indictment on 6 March 1958. He pleaded Not Guilty. The prosecution offered no evidence. On the direction of the
learned judge the appellant was found not guilty. After the appellant had left the dock the learned judge made certain
observations in regard to the appellant.
On the following day, 7 March 1958, there was a meeting of the watch committee. The appellant had not been invited to
attend and was not sent for. He received a letter the same afternoon telling him that he had been summarily 95 dismissed. He
was informed of certain resolutions which the watch committee had passed. Information as to those resolutions was given to the
press. The resolutions which were passed by the watch committee as recorded in their minutes were as follows:

Resolved (unanimously):
(1) The committee after carefully considering
(a) the request by [the appellants] solicitors that [the appellants] suspension be removed and [the appellant] be
reinstated in his office as chief constable,
(b) the length of [the appellants] period of service in the Brighton Police Force,
(c) the trial of [the appellant], Detective Inspector J. R. Hammersley and Detective Sergeant T.E. Heath, two senior
members of the criminal investigation department of the Brighton Police Force and others on a charge of conspiring to
obstruct the course of public justice and the conviction of Hammersley and Heath and another,
(d) the statements of DONOVAN, J., on Feb. 28, 1958 and Mar. 6, 1958,
(e) the statements made by [the appellant] in evidence at his trial, and
(f) certain statements made today by members of the committee and the town clerk,
decide that [the appellant] has in the opinion of the committee been negligent in the discharge of his duty and is unfit
for the same and the committee in exercise of the powers conferred upon them by s. 191 of the Municipal Corporations
Act, 1882, accordingly hereby dismiss him from his office as Chief Constable of Brighton forthwith.
(2) That in accordance with the provisions of reg. 7 of the Police Regulations, 1955 the amount of [the appellants]
aggregate pension contributions be paid to him.
(3) That the town clerk be requested to arrange for the foregoing resolutions to be conveyed to [the appellant] and to
Messrs. Bosley and Co.
(4) That resolutions (1) and (2) be made available to the press at a conference to be held at 2.45 p.m. this day and the
chairman and the town clerk be requested to inform the press that no other statement will be made or questions answered in
amplification thereof.
(5) That no statements or disclosures be made by members of the committee concerning the matter other than the
foregoing resolutions.

Your Lordships were informed that the transcript of the proceedings at the criminal trial were not available for the watch
committee on 7 March but that there was a transcript of the statements which had been made by the learned judge on 28 February
and 6 March.
Section 191(4) of the Municipal Corporations Act, 1882, is in the following terms:

The watch committee, or any two justices having jurisdiction in the borough, may at any time suspend, and the watch
committee may at any time dismiss, any borough constable whom they think negligent in the discharge of his duty, or
otherwise unfit for the same.

By s 4(1) of the Police Act, 1919, it was provided:

It shall be lawful for the Secretary of State to make regulations as to the government, mutual aid, pay, allowances,
pensions, clothing, expenses and conditions of service of the members of all police forces within England and Wales, and
every police authority shall comply with the regulations so made.

At the material times the following regulations made by the Secretary of State pursuant to that power were in force: (a) Police
Regulations, 1952, SI 1952 No 1704; (b) Police (Discipline) Regulations, 1952, SI 1952 No 1705, as amended by SI 1954 No
1687; and (c) Police (Discipline) (Deputy Chief Constables, 96Assistant Chief Constables and Chief Constables) Regulations,
1952, SI 1706, as amended by SI 1954 No 1688. The regulations, SI 1952 No 1705, show that a member of a police force
commits an offence against discipline if he commits any of the offences which are set out in the discipline code contained in Sch
1 to the regulations. The regulations contain detailed provisions as to the procedure which must be followed where a report or
allegation is received from which it appears that a member of a police force may have committed an offence. Chief constables,
deputy chief constables and assistant chief constables are governed by the regulations, SI 1952 No 1706, which were in fact made
on the same day as those in SI 1952 No 1705. These officers are also subject to the discipline code and the regulations, SI
1952 No 1706, contain detailed provisions as to the procedure which must be followed where a report or allegation is received
from which it appears that a chief constable may have committed an offence, ie an offence contained in the discipline code (see
reg 18).
Amongst the many offences included in the discipline code are the following:

1. Discreditable conduct, that is to say, if a member of a police force acts in a disorderly manner or any manner
prejudicial to discipline or reasonably likely to bring discredit on the reputation of the force or of the police service.
4. Neglect of duty, that is to say, if a member of a police force
(a) neglects, or without good and sufficient cause omits, promptly and diligently to attend to or carry out anything
which is his duty as a constable,
5. Falsehood or prevarication, that is to say, if a member of a police force (b) wilfully or negligently makes any
false, misleading or inaccurate statement,
17. Conviction for a criminal offence, that is to say, if a member of a police force has been found guilty by a court of
law of a criminal offence.

If there is a report or allegation from which it appears that a chief constable may have committed an offence against the
discipline code then the police authority mustl
________________________________________
l SI 1952 No 1706, reg 1, applied by reg 18

unless they are satisfied that he has not committed an offence inform him in writing of the report or allegation and ask
him whether or not he admits that he has committed an offence and give him an opportunity, if he so desires, of making to
the police authority any oral or written statement he may wish to make concerning the matter.

Regulation 2 (of SI 1952 No 1706) as applied to chief constables by reg 18 provides that if the chief constable

admits that he has committed an offence, the police authority may impose a punishment in accordance with reg. 11 of
these regulations without the case being heard in accordance with the following provisions of these regulations.

The power to punish for an offence without a hearing was therefore made conditional on there being an admission of an offence.
Thereafter if the police authority are not satisfied with the statement of a chief constable who does not admit that he has
committed an offence the police authority must instruct a solicitor to enter on a discipline form the offence with which the chief
constable is charged and must give such particulars as will leave the chief constable in no doubt as to the precise nature of the
alleged offence. That having been done a chief constable must be supplied with (a) a copy of the discipline form (b) a copy of
the report or allegation on which the charge is founded and any reports thereon notwithstanding that they may be confidential (c)
a copy of any statement relating to the charge made by any witness to be called in support of the charge together with the
witnesss name and address and (d) a copy of any statement relating to the charge made by any person other than a witness to be
called in support of the charge, to the police authority or to anybody on their behalf, 97together with the persons name and
address. Thereafter there must be a hearing by a tribunal (which could consist of a person selected from a list of nominated
persons and assisted on matters pertaining to the police by an assessor or could consist of not more than five members of the
police authority). The regulations lay down the procedure for the hearing and provide that after the hearing the tribunal must
submit a report to the police authority and send a copy of it to the accused chief constable. The report must contain (inter alia) a
statement as to the facts found or admitted, as to the charges found to be proved, and as to any recommendation as to any
punishment. Then the police authority must come to a decision. That will be only on receipt of the report of the tribunal. They
may decide to dismiss the case: alternatively they may decide to impose any one of the following punishments (i) dismissal (ii)
requirement to resign either forthwith or on such date as may be specified in the decision as an alternative to dismissal (iii)
reprimand (see reg 11).
By the Police Pensions Act, 1948, it was provided that regulations to be made by the Secretary of State were to make
provision as to the pensions to be paid whether as of right or otherwise and also as to the times at which and the circumstances
in which members of police forces are or may be required to retire otherwise than on the ground of misconduct. The Police
Pensions Regulations, 1955 (SI 1955 No 480), contain the following provisions:

1.(1) Subject to the provisions of these regulations, every man or woman who is a regular policeman, that is to say,
a member of a home police force who is not an auxiliary policeman and a member of an overseas corps who is a
reversionary member of a home police force, shall, on retiring from the force of which he is a member, be entitled to an
award under these regulations.
3.(1) Subject to the provisions of these regulations, where a regular policeman who is entitled to reckon twenty-five
years pensionable service retires from a police force, the award shall be an ordinary pension.
7.(2) Where a member of a police force is dismissed the police authority shall pay an amount equal to the amount of
his aggregate pension contributions to such one of those persons hereinafter described as, in their discretion, they may
think fit or, if in their discretion they think fit, shall distribute that amount among such of those persons in such shares and
in such manner as in their discretion they may think fit.
52. If a police authority determine that the retention in the force of a regular policeman who if required to retire is
entitled to receive a pension of an amount not less than two-thirds of his average pensionable pay would not be in the
general interests of efficiency, he may be required to retire on such date as the police authority determine.

It is to be observed that s 191(4) of the Municipal (Corporations) Act, 1882, gives to the watch committee a power of
suspension. As I have already mentioned, a power to suspend a chief constable is also given to the police authority by the
regulations, SI 1952 No 1706, as amended. The case has proceeded on the basis that the watch committee suspended the
appellant in October, 1957, under the powers given to them by those regulations. At the trial of the action the appellant gave
evidence that after his arrest on 25 October 1957, the town clerk came to see him on the same day and said that he had been
suspended from duty and the appellants recollection was that the town clerk added under police regulations or in accord with
police regulations. The appellants recollection as to this was not challenged in cross-examination. The suspension allowances
which thereafter were paid to the appellant were payable because on 28 October 1957, the watch committee resolved that in
accordance with reg 15 of the regulations such allowances should be paid. It is not suggested by the appellant that there was any
irregularity in his suspension. The power to suspend arose because the watch committee must have received a report or
allegation from which it appeared that 98 the appellant may have committed some offence against discipline (ie some offence
against the discipline code).
It seems very probable that the watch committee received an oral report that the appellant had been arrested on criminal
charges, and it is abundantly clear that, if he were guilty of criminal charges, he would have committed one or more of the
offences set out in the discipline code. Quite apart from various other offences he would have been guilty of the offence under
para 17 of the discipline code as previously set out.
After the appellant was acquitted of all criminal charges there were various courses which were open to the watch
committee. They could have decided not to charge the appellant with any offence against discipline. In that event his
suspension would have ceased. They could have decided to charge him with some offence or offences against discipline. In that
event the suspension would have continued. That would be on the basis that there was some report or allegation that the
appellant may have committed an offence against discipline. The appellant would then have had the right to be informed of, and
to make a personal explanation concerning, any such report or allegation and the procedure laid down in the regulations would
have to be followed. Had there been disciplinary proceedings, the appellant would have had all the opportunities to defend
himself which the regulations give. If any charges were found to be proved and if the case were not dismissed, then there might
have been dismissal or a requirement to resign in lieu of dismissal or a reprimand. Another course which was open to the watch
committee was to consider (pursuant to reg 52 of the Police Pensions Regulations, 1955, SI 1955 No 480) whether the appellant
was one who if required to retire was entitled to receive a pension of an amount not less than two-thirds of his average
pensionable pay, and, if so, to consider whether the retention of the appellant in the force would not be in the general interests of
efficiency, and to decide whether to require the appellant to retire.
The documents show what the watch committee did. The documents further show their reasons for doing what they did.
What they did was summarily to dismiss the appellant without any prior communication of any sort to him and without inviting
any submission from him. They purported to exercise powers given by s 191(4). It is beyond dispute that the procedure of the
regulations was in no way operated. The issue that is raised is therefore whether the powers given by s 191(4) may be invoked
without paying any regard to the provisions contained in regulations, SI 1952 No 1706. The further issue that is raised is whether
such powers may also be invoked without paying regard to those principles which are conveniently referred to as the principles of
natural justice. I propose to deal with these issues separately.
The powers given by s 191(4) are impressive. There is, first, a power to suspend. It may well be that different
considerations apply to suspensions as compared to dismissals. It may well be that a power to suspend if exercised by a watch
committee in good faith may have to be exercised without any hearing and without any procedural requirements. That does not
have to be decided in the present case. The power to dismiss (given by the section to a watch committee but not to justices)
relates to any borough constable (which term includes a chief constable) whom the watch committee think negligent in the
discharge of his duty or otherwise unfit for the same. My lords, I consider that in the context the word otherwise denotes that
there may be dismissal of a constable if the watch committee considers that he is unfit for the discharge of his duties even though
he may not have been negligent in their discharge. In the section it seems to me that the words unfit for the same were
designed to cover situations where even apart from any misconduct or lack of care and even apart from any physical or health
condition a constable was thought to be unfit for the discharge of his duty. It was not contended before your lordships that s
191(4) has been impliedly repealed. Having regard to s 1(4) of the Police Act, 1946, and para 3(2) in Sch 2 to that Act, that
would have been a difficult contention to advance. But 99 though the powers given by s 191(4) are still exercisable I consider, in
agreement with the Court of Appeal, that the effect of the Police Act, 1919, is that the powers given by s 191(4) must be exercised
in accordance with any regulations made under the Police Act, 1919, which are applicable.
Pearce LJ said ([1962] 1 All ER at p 842) that in cases coming within the regulations the statutory power of the watch
committee must be used in accordance with the regulations, and that in such cases the watch committee must act judicially or
quasi-judicially. Harman LJ ([1962] 1 All ER at p 848), said that in cases to which the regulations are applicable the power to
dismiss given by s 191(4) is controlled. Davies LJ ([1962] 1 All ER at p 851), likewise agreed that the power is controlled by the
regulations. These conclusions followed and were in accord with the judgment of Greer LJ, in Cooper v Wilson. In that case
Greer LJ while rejecting a suggestion that the power to make regulations under s 4(1) of the Police Act, 1919, had impliedly
repealed s 191(4) of the Act of 1882 said ([1937] All ER at p 730; [1937] 2 KB at p 316):

The regulations, in my judgment, must be read as applying to the way in which the watch committee are to exercise
their powers in a borough

It may well be that the various Police Regulations and Police Pensions Regulations were designed to cover all the circumstances
and situations with which police authorities are likely to be faced, and in practice I would think that police authorities would
invariably wish to follow the spirit as well as the letter of the carefully devised procedures which the regulations lay down. As a
matter of construction, however, I am not prepared to say every power is controlled by the regulations: they do not appear to
make provisions in regard to the power to dismiss a constable who is thought by the watch committee to be unfit for the discharge
of his duties. If, however, a constable is thought to have been negligent in the discharge of his duties, and so guilty of an offence
under s 4 of the discipline code, or if he is thought to have been guilty of some other offence, then the provisions of the
regulations would be applicable.
The action of the watch committee in summarily dismissing the appellant was stated to be for two reasons. In the first place
they decided that the appellant had been negligent in the discharge of his duty. In the second place they decided that he was unfit
for the discharge of his duty. Had they been merely of the opinion that the appellant had become unfit for the discharge of his
duty, but had not been in any way negligent in the discharge of his duty, it would seem to be inherently unlikely that they would
have exercised a power of summary dismissal. They had appointed the appellant in 1956, after he had been successively
promoted in the period of years after 1925, when he first joined the Brighton Borough Police Force. He was nearing the time
when he could have retired on a pension. If the watch committee thought that there were reasons why his retention in the force
would not be in the general interests of efficiency they could have required him to retire (see reg 52 of the Police Pensions
Regulations, 1955). If the watch committee had thought that there was no element of misconduct and no suggestion of
negligence in the discharge of duty, then, assuming that the power given by s 191(4) to dismiss, where there is unfitness for duty,
is a power which is not governed by and has not been affected by regulations which have been made, then on such hypothesis I
do not suppose that summary dismissal would have been contemplated. In fact there was a decision in the discharge of his duty.
There was therefore a finding of misconduct, although there had not been a charge and although there had not been a hearing. A
member of a police force is guilty of an offence if he neglects or without good and sufficient cause omits promptly and
diligently to attend to or carry out anything which is his duty as a constable. I do not think that it can be open to doubt that, if
someone is found to have been negligent in the discharge of his duty, 100he is found to have been guilty of the offence defined
by these words. The watch committee therefore found the appellant guilty of this offence ad summarily dismissed him for it.
They found him guilty without giving him particulars and without charging him and without giving him any opportunity to
defend himself. They made no attempt to pay heed to the regulations. The explanation of this that is advanced is that they were
not obliged to do so, because they had not received any report or allegation from which it appeared that the appellant might have
committed an offence. I find this a most surprising suggestion. If they had not received any report or allegation to the effect that
he might have committed an offence then why did they suddenly decide that he had committed an offence? How could they find
him guilty of being negligent in the discharge of his duty without some suggestion or some information which amounted to a
report or allegation to that effect? Indeed, it is difficult to understand how the committee could ever act in a disciplinary matter
without first having some report or some allegation that an offence may have been committed. The minutes of the meeting record
that they carefully considered the trial of the appellant and the other accused. The trial had lasted some nineteen days. The
transcript of the evidence was not before the committee on 7 March. If the committee had acquired knowledge that in the
proceedings at the trial there was some material which might lead to the view that the appellant had been negligent in the
discharge of his duty, then they were treating that material as a report or allegation. The minutes further record that the
committee considered the statements made by the learned judge on 28 February and 6 March. They had transcripts before them
of what the learned judge had said. The statements made by the learned judge manifestly called for the most careful
consideration by the watch committee who would obviously pay the greatest heed to them. One of the statements besides
commenting on the failure of the appellant to give proper leadership raised two specific matters, viz (1) that the appellant had
contrived to go to a suspected briber of the police in private (that was a reference to a man named Leach) and (2) that the
appellant had admitted a much convicted and hectoring bookmaker to his private room and had discussed with him the policy
of the police in certain matters (that was a reference to a man named Page). My lords, I find it impossible to say that in
considering those statements the committee had not received, and were not considering, a report or allegation to the effect that the
appellant might have been guilty of an offence. The learned judge on 6 March had said in terms that he realised that the matter of
the leadership of the Brighton Police Force was about to engage the attention of those persons whose responsibility it is. No
words used by the learned judge, however, could or did in any way suggest that the matter was to be dealt with in disregard of the
requirements of the law.
The minutes of the watch committee further record that the committee considered certain statements made today by
members of the committee and the town clerk. If those were statements that might lead members to conclude that the appellant
had been negligent in the discharge of his duty then they must have been reports or allegations from which it appeared that the
appellant might have committed either the offence of neglect of duty or some other offence. As no evidence was called on behalf
of the watch committee at the trial of the action, it was not possible to ask any member of the watch committee what these
statements were. There is, however, material from which their nature may be reasonably inferred. In circumstances to which
reference must later be made, the solicitors for the appellant gave notice of appeal to the Secretary of State against his dismissal,
while stating that the appeal was without prejudice to the appellants rights to contend that the purported notice of dismissal was
bad in law as being contrary to natural justice and not in accordance with the appropriate statutes and regulations. Thereafter in a
written statement submitted by the watch committee to the Secretary of State the facts and contentions on which reliance was
placed included the following:

(c) In the course of the said trial the appellant gave false evidence in 101 respect of two matters of material
importance, namely (i) that he had reported to the deputy town clerk and also to the chairman of the watch committee the
facts relating to an interview between Alderman Cullen and one Page, and (ii) that he had reported to the then chief
constable the facts relating to an interview which he (the appellant) had had with one Mrs. Cherryman.
(d) The appellant failed to investigate and to take any action whatsoever in respect of complaints of a serious nature
made by the said Mrs. Cherryman as to the manner in which the Astor Club was conducted and as to the trustworthiness of
the Brighton Criminal Investigation Department.
(e) Following a report of an attempt to bribe a police officer, the appellant went to the house of the man concerned,
namely Harry Leach, and there interviewed him privately and alone.
(f) The appellant permitted a man with a criminal record, namely the said Page, to interview him in his private room
and to discuss with him matters of police policy.
(i) The [watch committee] contend that, having regard to the facts and matters aforesaid, the appellant has both been
negligent in the discharge of his duty and is also unfit for the same and that, therefore, they were entitled to dismiss him
pursuant to the provisions of s. 191 of the Municipal Corporations Act, 1882.

It is in my judgment a reasonable inference that the statements made at the meeting of the watch committee covered the matters
set out under (c). If someone made the serious suggestion that the appellant had at his criminal trial given false evidence on
material matters, that was surely a report or allegation from which it appeared that the appellant might have committed an
offence. There would have been an offence under para 5 of the discipline code.
The watch committee were under a statutory obligation (see Police Act, 1919, s 4(1)) to comply with the regulations made
under the Act. They dismissed the appellant after finding that he had been negligent in the discharge of his duty. That was a
finding of guilt of the offence of neglecting or omitting diligently to attend to or to carry out his duty. Yet they had preferred no
charge against the appellant and gave him no notice. They gave him no opportunity to defend himself or to be heard. Though
their good faith is in no way impugned they completely disregarded the regulations and did not begin to comply with them. My
lords, I cannot think that any decision so reached can have any validity and unless later events have made it valid it ought not to
be allowed to stand. Had the regulations been applied but if there had been some minor procedural failure different
considerations might have applied. There was, however, no kind of compliance with them. In my judgment once there was a
report or allegation from which it appeared that a chief constable might have committed an offence it was a condition precedent
to any dismissal based on a finding of guilt of such offence that the regulations should in essentials have been put into operation.
They included and incorporated the principles of natural justice which, as Harman LJ ([1962] 1 All ER at p 850), said, is only fair
play in action. It is well established that the essential requirements of natural justice at least include that before someone is
condemned he is to have an opportunity of defending himself and in order that he may do so that he is to be made aware of the
charges or allegations or suggestions which he has to meet; see Kanda v Government of The Federation of Malaya. My lords,
here is something which is basic to our system: the importance of upholding it far transcends the significance of any particular
case.
The circumstances existing in March, 1958, made it, in my judgment, particularly necessary quite apart from its being a
matter of legal obligation that before considering whether the appellant had been negligent in the discharge of his duty the
procedure of the regulations should be observed. The appellant had after a 102 long trial been acquitted on criminal charges.
There had been much publicity. He had not been on trial on charges of neglect of duty or of unfitness for duty. If any facts had
emerged which, while insufficient to support the charges at the criminal trial, had seemed to the watch committee to suggest that
the appellant had been negligent in the discharge of his duty it would have been so easy to state them. If at the criminal trial any
admission had been made or evidence given which seemed to support the view that there had been negligencehow simple it
would have been to state it. If some facts were clear and plainso that denials would have been unlikely or explanations
difficultthen the opportunity to make a statement might have shown a course ahead. But if, on the other hand, facts could be
explained and if conduct could be defended and if charges of neglect or of unfitness could be repelled, was the appellant to be
denied a hearing? It is to be noted that whatever suggestion or charge might be formulated in regard to the Leach matter raised
questions as to the desirability or propriety of a visit that had taken place as far back as 1954, which was some time before the
appellant was appointed to be chief constable. The appellants case was that he had told his then chief constable in advance of his
proposed visit to Leach. His case further was that neither in respect of the Leach matter nor in respect of the Page matter was
there any impropriety in his conduct or actions. On the charges brought against him in a court of law and on which he was tried
he was found not guilty. If a new charge of neglect of duty was to be brought against him was he not even to be told about it or
asked about it? Was he to have no chance of dealing with matters which may have influenced the committee? Were the
safeguards of a criminal trial, of which the law is a jealous protector, to find no reflection in the days that followed an acquittal?
My lords, before further considering the result of disregarding the regulations it becomes necessary to mention certain
events that followed the dismissal. The solicitor for the appellant addressed a letter to the Secretary of State on 7 March 1958, in
which he contended that the dismissal was contrary to natural justice and bad in law and gave notice of appeal. The letter pointed
out that the notice of dismissal had merely recited a general finding of negligence and unfitness without specifying any details.
The Police (Appeals) Act, 1927, as amended by the Police (Appeals) Act, 1943, by s 1(1) provides:
A member of a police force who after the passing of this Act is punished by dismissal, by being required to resign as
an alternative to dismissal, by reduction in rank, or a reduction in rate of pay, may appeal to a Secretary of State in
accordance with this Act and the rules made thereunder, if he gives notice of appeal in the prescribed manner and within
the prescribed time.

The Police (Appeals) Rules, 1943 (SI 1943 No 473), which apply to all appeals by a member of a police force provide m that
notice of appeal must be sent within ten days from the date when the appellant received on the misconduct form the notification
of the decision against which he desires to appeal. The solicitor for the appellant followed his letter of 7 March with another
dated 10 March in which he stated that the appeal was without prejudice to the appellants rights to contend that the purported
dismissal was bad in law as being contrary to natural justice and not in accordance with the appropriate statutes and regulations.
In his notice of appeal which was dated 12 March he set out some thirty grounds of appeal. While denying any neglect or any
unfitness, he set out that he had been given no notice of what was alleged against him and no opportunity of being heard. He
further set out that by lodging his appeal he did not recognise the legality of the watch committees decision, and that his appeal
was without prejudice to his contentions that the watch committees decision was invalid, and he stated that his notice of appeal
was only given within the limited time in case it should be held that the watch committees procedure was valid. In due 103
course a written statement dated 18 April 1958, was submitted to the Secretary of State on behalf of the watch committee: it set
out the facts and contentions on which the watch committee relied in opposing the appeal.
________________________________________
m See reg 2(2)

Following on the dismissal of the appellant his solicitor made a request to appear before the watch committee. He wished to
be informed about the case against his client so as to be able to deal with it, and furthermore he wished to submit that the best
way of dealing with the situation would be to allow his client to resign and to have his pension. A copy of the appellants written
statement to the Secretary of State and in addition some written observations were sent to the watch committee. In those
observations it was submitted that the appellant should be allowed to retire on full pension forthwith. The committee decided to
meet on 19 March and stated that they would consider any representations which were then made by or on behalf of the appellant
either orally or in writing, and that such representations need not be limited to the matter of the pension. The appellants solicitor
attended and addressed the watch committee. In the course of his address he stated that before being dismissed the appellant had
been given no notice of what was charged against him or of being heard. The solicitor was received with courtesy, but in silence.
It seems, however, to be beyond dispute that he was given no further particulars of the case against the appellant than appeared in
the letter of 7 March. The watch committees later submissions to the Secretary of State which were dated 18 April were, of
course, not then available. The result of the meeting of 18 March was that the watch committee by a majority resolved to adhere
to their previous decision: nine members voted in favour of such resolution and three members voted against it.
The written statement (dated 18 April 1958) submitted to the Secretary of State by the watch committee set out their
contentions, and they included the paragraphs to which I have already referred. The Secretary of State decided that the case
could be determined without taking oral evidence (see Police (Appeals) Act, 1927, s 2) and on 5 July 1958, he dismissed the
appeal. He came to the conclusion that there was sufficient material on which the watch committee could properly exercise their
power of dismissal under s 191(4). He did not take into account, as no evidence in support of them was before him, certain
allegations on which the watch committee relied, viz (i) that the appellant did not report to the deputy town clerk and to the
chairman of the watch committee the facts relating to an interview between Alderman Cullen and one Page; (ii) that the appellant
did not report to the then chief constable the facts relating to an interview which the appellant had with one Mrs Cherryman; and
(iii) that the appellant in giving evidence at his trial that he had so reported those matters gave false evidence.
My lords, in my judgment, inasmuch as the decision of the watch committee was that the appellant had committed an
offence or offences against the discipline code and inasmuch as the decision was arrived at in complete disregard of the
regulations it must be regarded as void and of no effect. The power to dismiss for an offence was a power that could only be
exercised if the procedure of the regulations was set in motion. A purported dismissal in complete disregard of them cannot be
recognised as having any validity. In Andrews v Mitchell a member of a friendly society, who had been duly summoned before
the arbitration committee for a breach of the rules, was in his absence expelled from the society by a resolution of the committee
on a different charge, ie of fraud and disgraceful conduct of which no written notice had been given to him as required by the
rules. By one of the general laws of the friendly society any member proved guilty of fraud or any conduct or offence calculated
to bring disgrace on the order before any recognised arbitration committee, provided a charge had been preferred against him as
required by the general laws with regard to arbitrations, might be expelled or suffer some less penalty. It was held that the
decision of the committee 104 was null and void. In his speech ([19047] All ER Rep at p 600; [1905] AC at p 80) the Earl of
Halsbury LC said that there are some principles of justice which it is impossible to disregard. He pointed out that while there
was a rule which justified expulsion it justified expulsion on the express proviso that the charge had been made as provided by
the rules. He added ([19047] All ER Rep at p 600; [1905] AC at p 81):

In this case the charge never was made as provided by the rules; and if you have no power given under the rules to
expel a member except upon a charge made and tried according to the rules, you have no power to expel in a case like
this.

He described the summoning of a member pursuant to the rules and giving him time to consider what he had to do and giving
him the charge against him in writing as matters of substance and not mere matters of form. He concluded that the arbitration
committee had no jurisdiction to entertain the matter. Lord Davey said ([19047] All ER Rep at p 601; [1905] AC at pp 82,
83):

It is not contended that this charge was properly made according to the rules, but it is said that it may be regarded that
as a mere informality which might be set right. But it was an informality which went to the root of the jurisdiction, and the
omission to follow the directions of the rules for preferring charges has had the unfortunate effect of making the resolution
which was come to for the expulsion of the respondent, in my opinion altogether invalid, null and void.

My lords, if the regulations were applicable in this case, as in my judgment they were, reg 2 of SI 1952 No 1706, to which I have
referred above, only gives a power to impose punishment without a hearing if a condition is satisfied, viz, if there is an admission
of the commission of an offence. In the present case there was no such admission and the watch committee therefore lacked
power to impose punishment for an offence without a hearing: in purporting to dismiss the appellant they acted without
jurisdiction and their decision was a nullity.
In Lapointe v Lassociation de Bienfaisance et de Retraite de la Police de Montreal, the appellant, who was a member of the
respondent benevolent and pension society, had been obliged to resign from the police force. Under those circumstances he
became entitled according to the rules to have his case for a gratuity or pension considered by the board of directors and his right
to such gratuity or pension determined by a majority of the board. The board in fact acted in a most extraordinary manner. In
delivering the judgment of the Privy Council Lord MacNaghten said ([1906] AC at p 538):
They first appointed a committee of four from their own body to investigate the reason of Lapointes resignation.
There would have been no objection to this course if the committee had been deputed to consider and report whether or not
there was a prima facie case for inquiry. But what the committee did was to listen to all sorts of stories about Lapointes
past history, and rake up everything that was against him during his connexion with the force. Then, without telling
Lapointe what the charges against him were, or giving him any opportunity of defending himself, they advised the board
that the pension should be refused. Thereupon the board abnegated their judicial duties altogether. They summoned a
general meeting of the members and submitted a question, which they were bound to determine themselves, to a popular
vote. The meeting was held on Apr. 26, 1892, when by a large majority of the members present it was resolved that
Lapointes name should not be entered on the pension roll of the society.
The whole of these proceedings were irregular, contrary to the rules of 105 the society, and above all contrary to the
elementary principles of justice. And the position of the board was certainly not improved by a formal resolution stating
solemnly, what was contrary to the truth, that after having inquired into the facts and circumstances which brought about
Lapointes resignation, and having deliberated upon his claim, the board desires that the pension on which he claims be
refused, seeing that he was obliged to tender his resignation.

Lord MacNaghten said ([1906] AC at p 451) that it was obvious that the so-called determination of the board was void and of no
effect, and the order which they humbly advised included a declaration and determination as required by the rules and that the
proceedings were null and void.
In Annamunthodo v Oilfields Workers Trade Union, it was held in the Privy Council that a decision of the general council of
the trade union was vitiated because it convicted the appellant of an offence against the rules with which he had never been
charged and it was held that it should be declared that the purported expulsion of the appellant was invalid and that an order
should be made to set it aside. My lords, so here should it, in my judgment, be declared that the purported termination on 7
March 1958, of the appellants appointment was void unless it be that later events debar the appellant from obtaining this relief.
If they do not then the effect of such a declaration will be that the respondents will have to consider what action to take and in
any course that they follow they must act according to law.
The appellant, through his counsel, has stated that he has no intention of applying for reinstatement, but would be content to
retire (as from March, 1958), with his pension. I apprehend that in all the circumstances it would not be appropriate for your
lordships to do more than to declare that the purported termination on 7 March 1958, of the appellants appointment was void.
Included in the other claims of the appellant in his action is a claim for his salary as from 7 March 1958. It would not seem
appropriate at the present stage to deal with the appellants claim for salary, and it would not be for your lordships to decide any
question as to a pension.
The question next arises whether the events subsequent to 7 March form any bar to the appellants claim. I have already
referred to the meeting of 18 March. That occasion afforded an opportunity for the watch committee to tell the appellant and his
solicitor what were the allegations that he had to meet. The documents which the solicitor sent to the watch committee
emphasised the point that the appellant had been given no notice of them: mention was also made of the fact that the appellant
did not know what were the statements made by members of the watch committee referred to in the letter of 7 March. He most
certainly had no hint that it was being said of him that he had given some false evidence. The appellants case is that he never
had the chancewhich he would have welcomedof refuting that suggestion before the watch committee and the chance of
calling such evidence as he might desire to call to deal with the suggestion. The oral request for information made by the
solicitor met with no response. Even though the watch committee had communicated their previous decision to the press a full
inquiry might still have been possible but the watch committee neither took the opportunity then to begin compliance with the
regulations nor even, in default of that, to give information to the appellant as to the case that he had to meet. In the result, in my
judgment, nothing occurred on 18 March to give validity to what the watch committee had purported to do on 7 March. Nor in
my view did the action of the appellant in appealing to the Secretary of State have any such effect. If the decision of 7 March
was a nullity and void the fact that the appellant appealed made no difference. The decision of 7 March remained a nullity. The
appellant made it as plain as possible that he was adhering to and was in no way abandoning his submission that the decision of 7
March had no validity. In 106 these circumstances the provision in s 2(3) of the Police (Appeals) Act, 1927, that the decision of
the Secretary of State on an appeal is to be final and binding upon all parties cannot produce the result that validity is given to
that which is a nullity.
The watch committee referred to s 220 of the Municipal Corporations Act, 1882. That section was not pleaded and was not
mentioned in the watch committees case, but it was argued that it could be relied on in support of the contention that the court
could not declare against the validity of the decision of the watch committee. I deal with the point because, if it had validity, it
would go to jurisdiction. I would not regard the complaints of the appellant as covered by the words want of form, nor would I
regard the words removed by certiorari or otherwise as apt to exclude the claims made in this action. Furthermore it would
clearly be contrary to the intention of s 11(1) of the Tribunals and Inquiries Act, 1958, if its effect on s 220 could be construed as
having the result that the court could make an order of certiorari, but could not entertain an action for a declaration.
In view of the opinions which I have expressed as to the applicability of the regulations and as to the consequences of
disregarding them I propose only to deal briefly with the question whether, had there been no regulations, the police authority
would have been bound to have regard to the principles of natural justice. In my view the regulations incorporate those
principles, but had there not been any and had the police authority in the exercise of powers given them by s 191(4) contemplated
dismissing the appellant on the ground of neglect of duty, they would in my view have been under obligation to give him an
opportunity to be heard and would have had to consider anything that he might say. I cannot think that the dismissal of the
appellant should be regarded as an executive or administrative act if based on a suggestion of neglect of duty: before it could be
decided that there had been neglect of duty it would be a pre-requisite that the question should be considered in a judicial spirit.
In order to give the appellant an opportunity to defend himself against a charge of neglect of duty he would have to be told what
the alleged neglect of duty was.
In a case in which a consistory court had made an order requiring a vicar to pay certain expenses and costs, but had given
him no opportunity of being heard in his defence, a writ of prohibition directed to the chancellor was issued (see R v North, Ex p
Oakey) and Scrutton LJ said ([1927] 1 KB at p 502):

In my view an order that any one shall pay the cost of work that has been obliterated without a faculty is in the nature
of a penalty for an ecclesiastical offence, and one of the most fundamental principles of English law is that if you are going
to impose on a person a penalty for an offence you must first clearly inform him that an application to that effect is going to
be made against him, so that he may know what he is charged with and have an opportunity of attending to meet it.

The proceedings in the consistory court were therefore without jurisdiction and prohibition lay. The application of the
fundamental principle is, however, not limited to proceedings in courts or to cases where penalties for offences are being
imposed. The conduct of James Bagg (see Baggs case) was hardly commendable but it was held that it did not give good cause
for his disfranchisement. In any case he had not been heard and the court said ((1615), 11 Co Rep at pp 98b, 99a):

And although they have lawful authority either by charter or prescription to remove any one from the freedom and that
they have just cause to remove him: yet it appears by the return that they have proceeded against him without hearing him
answer to what was objected or that he was not reasonably warned, such removal is void and shall not bind the party. Such
a removal was against justice and right.
107

So also did the courts come to the aid of Dr Bentley and grant a peremptory mandamus to restore him to his degree ( R v
University of Cambridge). Though the court was roundly critical of Dr Bentleys behaviour they considered that even if he had
been guilty of a contempt to the Vice-Chancellors court that court had no power to deprive him of his degrees: but they held that
in any event he could not be deprived without notice. The words of Fortescue J ((1723), 1 Stra at p 566), were emphatic:

Besides, the objection for want of notice can never be got over. The laws of God and man both give the party an
opportunity to make his defence if he has any.

In Wood v Woad, Kelly CB in speaking of the rule expressed in the maxim audi alteram partem said ((1874), LR 9 Exch at p
196):

This rule is not confined to the conduct of strictly legal tribunals but is applicable to every tribunal or body of persons
invested with authority to adjudicate upon matters involving civil consequences to individuals.

The relationship between the watch committee and the appellant was not that of master and servant. Nor was the appellant
one who held an office at pleasure with the consequence that he could be required at pleasure to relinquish it. He was in a
different position from someone possessing a licence to do various acts. The appellant held an office from which the watch
committee could at any time dismiss him if they thought he had been negligent in the discharge of his duty. The watch committee
did not however have an unfettered or unrestricted discretion. If it be assumed that no regulation had been made, then the fact
that s 191(4) is silent as to any procedure for a hearing does not involve that there could be a dismissal without a hearing. The
justice of the common law would require it, for, as Byles J said in Cooper v Wandsworth Board of Works ((1863), 14 CBNS at
p 194):

a long course of decisions, beginning with Dr. Bentleys case and ending with some very recent cases, establish that
although there are no positive words in a statute requiring that the party shall be heard, yet the justice of the common law
will supply the omission of the legislature.

In that case it was held that, although s 76 of the Metropolis Management Act, 1855, empowered the district board to alter or
demolish a house where the builder had neglected to give notice of his intention to build seven days before proceeding to lay or
dig the foundation, yet the district board were not empowered to demolish the building without first giving the party guilty of the
omission an opportunity of being heard. Erle CJ said ((1863), 14 CBNS at p 189):

It has been said that the principle that no man shall be deprived of his property without an opportunity of being heard
is limited to a judicial proceeding and that a district board ordering a house to be pulled down cannot be said to be doing a
judicial act. I do not quite agree with that; neither do I undertake to rest my judgment solely upon the ground that the
district board is a court exercising judicial discretion upon the point: but the law, I think, has been applied to many
exercises of power which in common understanding would not be at all more a judicial proceeding than would be the act of
the district board in ordering a house to be pulled down.

So Willis J said ((1863), 14 CBNS at p 190):

I apprehend that a tribunal which is by law invested with power to affect the property of one of Her Majestys subjects
is bound to give such subject an opportunity of being heard before it proceeds: and that that rule 108 is of universal
application and founded upon the plainest principles of justice.

So also in Spackman v Plumstead Board of Works, the Earl of Selborne LC said ((1885), 10 App Cas at p 240):

No doubt in the absence of special provisions as to how the person who is to decide is to proceed, the law will imply
no more than that substantial requirements of justice shall not be violated. He is not a judge in the proper sense of the
word: but he must give the parties an opportunity of being heard before him and stating their case and their view. He must
give notice when he will proceed with the matter and he must act honestly and impartially and not under the dictation of
some other person or persons to whom the authority is not given by law. There must be no malversation of any kind.
There would be no decision within the meaning of the statute if there were anything of that sort done contrary to the
essence of justice.

Lord Selborne was there speaking of the decision of an architect (under s 75 of the Metropolis Management Act, 1862) as to the
general line of buildings in a road. If the principles, to which he adverts, apply where property rights are in issue, surely they
must at least apply with equal effect where the issue is whether there has been misconduct which merits dismissal from an office.
It is to be remembered also that in the case of the appellant his summary dismissal involved the loss of valuable pension rights.
Property rights were at stake in Local Government Board v Arlidge. Viscount Haldane LC (([191415] All ER Rep at p 7; [1915]
AC at pp 132, 133), there expressed his approval of the view indicated by Lord Loreburn in Board of Education v Rice ([1911
13] All ER Rep 36 at p 38; [1911] AC 179 at p 182) that an administrative body to which the decision of a question in dispute
between parties has been entrusted must act in good faith and listen fairly to both sides. Lord Parmoor said ([191415] All ER
Rep at p 15; [1915] AC at p 142) that, whether in that case the order of the Local Government Board was to be regarded as of an
administrative or of a quasi-judicial character, if the order affected the rights and property of the respondent he was entitled to
have the matter determined in a judicial spirit in accordance with the principles of substantial justice. A right to be heard before
property rights were affected was upheld in the circumstances applying in Cooper v Wandsworth Board of Works, in Hopkins v
Smethwick Local Board of Health, and in Urban Housing Co Ltd v Oxford Corpn. Similarly a right to be heard in regard to
removal from an office was recognised in Osgood v Nelson, in Ex p Ramshay and in R v Gaskin. So also it has been recognised
that expulsion from a club must not take place in disregard either of the rules of the club or of the rules of natural justice. (The
cases of Fisher v Keane and Dawkins v Antrobus may be mentioned as typical examples.)
Being of the view that even if there had been no applicable regulations a decision to dismiss the appellant for neglect of duty
ought only to have been taken in the exercise of a quasi-judicial function which demanded an observance of the rules of natural
justiceI entertain no doubt that such rules were not observed. Before 7 March there was neither notice of what was alleged nor
opportunity to deal with what was alleged. It was contended that the criminal trial had been the appellants opportunity. My
lords, I cannot think that such a contention is valid. The trial was concerned with specific charges. In respect of them the
appellant was found not guilty. If there were other charges or charges of a different nature which were not submitted to the jury
but which the watch committee proposed 109 to consider, then it was for the watch committee to formulate them and only to
reach decision in regard to them after hearing and considering what the appellant or any witnesses of his had to say. For the
reasons that I have already given the hearing of 18 March did not remedy the previous defects. The consequence, in my view, is
that there was an abnegation of the quasi-judicial duties involved in the function of the watch committee with the result that their
decision must be regarded as of no effect and invalid, and so can be declared by the court to be void (see Baggs case, R v
University of Cambridge, Wood v Woad, Fisher v Keane).
It was submitted that the decision of the watch committee was voidable but not void. But this involves the inquiry as to the
sense in which the word voidable, a word deriving from the law of contract, is in this connexion used. If the appellant had
bowed to the decision of the watch committee and had not asserted that it was void, then no occasion to use either word would
have arisen. When the appellant in fact at once repudiated and challenged the decision, so claiming that it was invalid, and when
in fact the watch committee adhered to their decision, so claiming that it was valid, only the court could decide who was right. If
in that situation it was said that the decision was voidable, that was only to say that the decision of the court was awaited. But if
and when the court decides that the appellant was right, the court is deciding that the decision of the watch committee was invalid
and of no effect and null and void. The word voidable is, therefore, apposite in the sense that it became necessary for the
appellant to take his stand: he was obliged to take action for unless he did the view of the watch committee, who were in
authority, would prevail. In that sense the decision of the watch committee could be said to be voidable. The appellant could, I
think, have applied for an order of certiorari: he was not saying that those who purported to dismiss him were not the watch
committee; he was recognising that they had a power and jurisdiction to dismiss but he was saying that whether the regulations
applied or whether they did not the committee could only exercise their power and jurisdiction after hearing his reply to what was
said against him. In these circumstances he could, I think, have applied for an order of certiorari (though considerations of
convenience would probably have pointed against pursuing such a course) or he could have asked for a declaration. In either
proceeding the question of acquiescence by him might be raised or the question whether by some binding election he had barred
himself from taking proceedings in court or whether in some way he was estopped. It seems to me that he made it abundantly
clear that by his appeal to the Secretary of State he was not in any way abandoning his right to contend that the decision of the
watch committee was invalid. An appeal to the Secretary of State raises a question whether a decision, which as a decision has
validity, should or should not on the facts and on the merits be upheld. The question raised and reserved by the appellant was the
fundamental point that the purported decision of the watch committee was no decision. It would not have been unreasonable if
the Secretary of State had asked that that point should first be adjudicated; but in the events which happened I cannot think that
the careful steps which were taken to protect the appellants position ought to be held to have in fact compromised it. Compare
Annamunthodo v Oilfields Workers Trade Union. The appellant never abandoned his point and in my view nothing done by him
or by the Secretary of State gave validity to a decision which is now shown to have had none.
My lords, it was submitted to your lordships that the decision of the watch committee should be upheld as having been the
only reasonable decision. I consider this to be an entirely erroneous submission. Since no charges have been formulated it is
impossible to assess their weight or the weight of the answering evidence of the appellant and others. When the appellant was in
the witness-box 110 in the present action he was questioned as to what witnesses he would have wished to call in order to deal
with the Leach and the Page matters. As charges in respect of those matters were not formulated, I cannot think that it was
appropriate to elicit the names of certain witnesses whom the appellant might have decided to call and then without hearing or
being able to hear such witnesses to seek to discount their value and effectiveness and then to seek to draw a vague and artificial
conclusion that if matters had been regularly done and if the appellant had been heard and if his witnesses had been heard a result
adverse to him would have followed. All the defects and all the unfairness of the original irregularity are inherent in any such
approach. The suggested conclusion must fail because it is based on a perpetuation of the very defects which vitiate the dismissal
of the appellant and also because the process involves endowing the court with a function that belongs elsewhere.
I do not find it necessary to express any concluded opinion whether, if there were no suggestion of having been negligent in
the discharge of duty, a decision to dismiss on the ground of being unfit for the discharge of duty could be taken without giving
an opportunity to be heard. Clearly it would be desirable and reasonable to give such an opportunity even though the alleged
unfitness did not involve misconduct.
For the reasons that I have given I would allow the appeal.

LORD HODSON. My Lords, I have reached the conclusion apart from the application of the Police Act, 1919, and the
regulations which followed, that this appeal should succeed on the ground that the appellant was entitled to and did not receive
natural justice at the hands of the watch committee of Brighton when he was dismissed on 7 March 1958. Streatfeild J ([1961] 2
All ER at p 534), who heard the appellants suit at first instance held that the power given to the watch committee by the
Municipal Corporations Act, 1882, s 191(4), at any time to dismiss any borough constable, whom they think negligent in the
discharge of his duty or otherwise unfit for the same, was a power which had to be exercised in accordance with the principles of
natural justice but that the watch committee had acted in that manner. The Court of Appeal took a different view and held that the
watch committee were not bound, in taking the executive action of dismissing their chief constable, to hold an inquiry of a
judicial or quasi-judicial nature (per Pearce LJ ([1962] 1 All ER at p 844)). Harman LJ ([1962] 1 All ER at p 849), was of
opinion that the watch committee were acting in exercise of their administrative functions just as they were when they made the
appointment under s 191(1) of the Act of 1882 and that the principles of natural justice did not come into the case. He pointed
out that the defendants were not deciding a question between two opposing parties and that there was no lis and nothing to
decide. Davies LJ ([1962] 1 All ER at p 852), said that the exercise by the watch committee of their powers under s 191(4) of the
Act of 1882 was not a quasi-judicial but an executive one, emphasising the words whom they think as being very strong indeed
and much stronger than the sort of words to be found in most of the authorities cited to the court such as on sufficient grounds
shown to his satisfaction (De Verteuil v Knaggs).
I should add that Streatfeild J although holding that the principles of natural justice should prima facie have been applied,
held ([1961] 2 All ER at p 536) that in this case the appellant had at the Old Bailey, for the purposes of his trial for all the world
as well as the watch committee to hear, convicted himself of unfitness to hold the office of chief constable. The judge concluded
that on the evidence which the appellant had himself given at the Old Bailey there was no need for the watch committee to do
other than they in fact did, whatever also they might have done to be on the safe side. It would be unrealistic to suppose that the
watch committee had not a good idea of what took place at the criminal trial, 111although they were not provided with a
transcript of the evidence until after they had reached their conclusion, but in my opinion it will not do to say that the case was so
plain there was no need for the appellant to be heard and that, therefore, the claims of natural justice were satisfied.
What the watch committee had before them was primarily the observations of Donovan J who in the course of sentencing
the two police officers who were convicted, the appellant having been acquitted, gave as a ground for the moderation of his
sentences on these two men the extenuating circumstances that in his opinion they had not had the leadership to which they were
entitled. I think that it is clear from the learned judges observations at that time, and at the conclusion of the trial, that he
intended that what he said should be brought to the notice of the watch committee to act on as they thought fit. The watch
committee no doubt felt it necessary to act promptly, but there was nothing in the learned judges observations which would
suggest that the appellant could be dealt with on the basis that any charges had been proved against him and that no further
hearing was required.
I do not find that the answer put by counsel for the watch committee to your lordships that the case was as plain as a pike-
staff is an answer to the demand for natural justice. The case on natural justice does not rest on the events of 7 March 1958, alone
for the appellant was given a further opportunity on 18 March 1958, to address the watch committee and of this he availed
himself by his solicitor who appeared and was allowed to address the committee without restriction. I agree with Pearce LJ that
at that stage the watch committee could have re-opened the matter, and indeed three out of the twelve were in favour of so doing
(cf De Verteuil v Knaggs) but the position was then that the watch committee had given their decision that the appellant be
dismissed not only on the ground of unfitness but also on other grounds which included not only negligence in discharge of his
duty but also unspecified matters, which were said to be certain statements made today by members of the committee and the
town clerk. It was not until 5 April when the watch committee communicated with the Home Secretary prior to the appellants
appeal to the minister that it emerged that these statements had reference to allegations of perjury against the appellant. On 18
March Mr Bosley was given not only a full but a courteous hearing by the watch committee, but received no indication of the
nature of the charges which his client had to answer notwithstanding his repeated statements that he did not know what they
were. It is plain therefore that, if there were a failure on 7 March to give justice to the appellant, this was not cured on 18 March
when the watch committee confirmed their previous decision. At this hearing it was made plain by Mr Bosley that his client was
not seeking reinstatement but only his pension rights of which he had been deprived by his dismissal. This position is maintained
by the appellant through his counsel before your lordships.
I should not delay further before referring to the terms of the Municipal Corporations Act, 1882, itself, for it is on the
construction of that statute that the answer to the question posed before your lordships depends. It is quite true that on its terms
there is a power to dismiss any borough constable (and this applies to the appellant), whom the watch committee think negligent
in the discharge of his duty or otherwise unfit for the same. I entirely accept the reasoning underlying the judgments of the lords
justices that, if a statute gives an unfettered power to dismiss at pleasure without more, that is an end of the matter.
The topic is, however, not as simple as would seem. A large number of authorities were cited to your lordships beginning
with Baggs case and extending to the present day. I will not travel over the field of the authorities, which I am bound to say are
not easy to reconcile with one another, for if I did I should surely omit some which might be thought to be of equal or greater
importance than those I mentioned, but certain matters seem to me clearly to 112 emerge. One is that the absence of a lis or
dispute between opposing parties is not a decisive feature, although no doubt the presence of a lis would involve the necessity for
the applications of the principles of natural justice. Secondly, the answer in a given case is not provided by the statement that the
giver of the decision is acting in an executive or administrative capacity, as if that was the antithesis of a judicial capacity. The
cases seem to me to show that persons acting in a capacity which is not on the face of it judicial, but rather executive or
administrative, have been held by the courts to be subject to the principles of natural justice. Perhaps the most striking example
is to be found in the old case of Capel v Child, which is referred to at length by North J in Fisher v Jackson ([1891] 2 Ch 84 at p
95). The facts were these. By s 50 of the Act, 57 Geo 3 c 99, it was provided that:

Whenever it shall appear to the satisfaction of any bishop, either of his own knowledge, or upon proof by affidavit laid
before him, that by reason of the number of churches or chapels belonging to any benefice locally situate within his
diocese, or the distance of such churches or chapels from each other, and the distance of the residence of the spiritual
person serving the same from such churches or chapels, or any or either of them, or the negligence of the spiritual person
holding the same, that the ecclesiastical duties of such benefice are inadequately performed,

then, to put it shortly, the bishop may appoint a curate to perform or assist in performing the duties and may throw the burden
of the stipend of that curate on the person the insufficiency of whose performance of the duties has led to the necessity of the
appointment. The Bishop of London on the 18 January 1880, served a requisition on the plaintiff by virtue of the Act of
Parliament above mentioned reciting that of his own knowledge the ecclesiastical duties of the vicarage and parish church of
Watford were inadequately performed by reason of the plaintiffs negligence and requiring him to nominate a fit person with a
stipend to assist in performing those duties. The plaintiff did not appoint a curate and the bishop did so, assigning to him a
stipend. The stipend remained unpaid and the plaintiff was accordingly summoned before the bishop. The plaintiff did not attend
and the plaintiff was monished to pay the stipend. He then appeared for the first time and alleged that he had not had a proper
opportunity of being heard on the original application. Lord Lyndhurst CB used this language ((1832), 2 C & J at p 577):

Here is a new jurisdiction givena new authority given: a power is given to the bishop to pronounce a judgment;
and, according to every principle of law and equity, such judgment could not be pronounced, or, if pronounced, could not
for a moment be sustained, unless the party in the first instance had the opportunity of being heard in his defence, which in
this case he had not; and not only no charge is made against him which he had an opportunity of meeting, but he had not
been summoned that he might meet any charge.

Baron Bayley said ((1832), 2 C & J at pp 578, 579):

Upon the general principles of law, it would have been essential, if the bishop had proceeded by way of affidavit, to
have given the opposite party an opportunity of being heard. When the bishop proceeds on his own knowledge, I am of
opinion that it cannot possibly and within the meaning of this Act, appear to the satisfaction of the bishop, and of his own
knowledge, unless he gives the party an opportunity of being heard, in answer to that which the bishop states on his own
knowledge to be the foundation on which he proceeds It would be quite sufficient if the bishop were to call the party
before him, and to state to him the grounds on which he thought the duties were inadequately performed, by reason of his
113 negligence; and he should have asked whether he had or had not any grounds on which he could answer that charge.
But, is it not a common principle in every case which has in itself the character of a judicial proceeding, that the party
against whom the judgment is to operate should have an opportunity of being heard?

It is true that emphasis is laid on the judicial character of the proceedings in the view of both learned judges but it is not clear to
me that it could not be said in that case that the bishop was acting administratively. The situation under the Act, under which the
bishop was exercising his powers, was not unlike that of the watch committee here exercising powers under another Act, and it so
happens that the charge involved, that of negligence, was the same in each case. The matter which, to my mind, is relevant in this
case is that where the power to be exercised involves a charge made against the person who is dismissed, by that I mean a charge
of misconduct, the principles of natural justice have to be observed before the power is exercised.
One of the difficulties felt in applying principles of natural justice is that there is a certain vagueness in the term and as
Tucker LJ, said in Russell v Duke of Norfolk ([1949] 1 All ER 109 at p 118):

There are no words which are of universal application to every kind of inquiry and every kind of domestic
tribunal. The requirements of natural justice must depend on the circumstances of the case, the nature of the inquiry, the
rules under which the tribunal is acting, the subject-matter that is being dealt with, and so forth.

If it be said that this makes natural justice so vague as to be inapplicable, I would not agree. No one, I think, disputes that three
features of natural justice stand out(1) the right to be heard by an unbiassed tribunal, (2) the right to have notice of charges of
misconduct, (3) the right to be heard in answer to those charges. The first does not arise in the case before your lordships, but the
two last most certainly do and the proceedings before the watch committee therefore, in my opinion, cannot be allowed to stand.
I have reached this conclusion on the construction of the statute with some hesitation, not only because of the different view
taken by the Court of Appeal but also because of that taken by my noble and learned friend Lord Evershed, who also feels that
notwithstanding their findings of fault made against the appellant the watch committee had a perfect right to act as they did. In
one respect no doubt the watch committee were given an absolute discretion to act as they might think, that is to say, I agree that
their residual power to dismiss for unfitness may well be unfettered. I do not accept the contention of the appellant that unfitness
is to be construed ejusdem generis with negligence: indeed I think that it is the antithesis of negligence, and covers cases where
there is no fault in the accepted sense of the word in the officer dismissed. A man may be unfit because he is stupid, vacillating,
unable to meet a crisis or generally to command others, but I do not see this as the subject-matter of a charge. As I have
indicated, it is not clear to me that Donovan J necessarily had anything more in mind than absence of the qualities necessary for
leadership, when he made the observations that he did, but the watch committee went outside unfitness and made findings of
negligence and inferentially of perjury without giving the appellant any notice or opportunity of being heard. Even if the residual
power to dismiss for unfitness remains unimpaired, one could not conceive any watch committee exercising this power and at the
same time leaving the dismissed officer without a pension. This would only be expected where charges as here were made
against him. I cannot see that the general words of the statute are in the light of the authorities as I understand them wide enough
to cover a case of this character where allegations of misconduct are involved invoking the loss of an office and an element of
punishment for 114 offences committed. There is imposed a clog on the discretion in that it cannot be exercised arbitrarily
without regard to natural justice. I am aware that what I have said may not be thought to be in line with those cases where wide
words have been held sufficient to cover the exercise of an arbitrary power as in the matter of issue and withdrawal of licences
where no question of punishment arises, cf Nakkuda Ali v M F de S Jayaratne and R v Metropolitan Police Commissioner. It
may be that I must retreat to the last refuge of one confronted with as difficult a problem as this, namely, that each case depends
on its own facts and that here the deprivation of a pension without a hearing is on the face of it a denial of justice which cannot be
justified on the language of the subsection under consideration
I have little to add to what has already been said about the application of the Police Regulations, 1952. It was not contended
before your lordships that the Act of 1882 had been repealed by the Police Act, 1919, or any regulations made thereunder but it
was contended, in my opinion rightly, that, where a report or allegation against a police officer has been made, the regulations
apply and govern the form of the inquiry which must follow. Here there were no formulated charges, no tribunal appointed for
the purpose of hearing the charges and reporting to the police authority a statement of the charges found to be proved. The
learned judge at the trial and all the members of the Court of Appeal were of opinion that the regulations did not apply, because
no report or allegation was received from which it appeared that the appellant had committed an offence. My noble and learned
friend Lord Evershed, indeed, is of the same opinion. With all respect, I cannot agree. It is plain that the action taken by the
watch committee followed directly from the observations of Donovan J after the trial at the Old Bailey, which were intended for
the ears of the appropriate authority and did in fact reach the watch committee before it dismissed the appellant on 7 March. The
appellant had been acquitted of the offence with which he had been charged at the criminal trial but on a fair reading of those
observations which were severely critical of the appellant it cannot, I think, be said that it did not appear from them that the
appellant had committed an offence under the regulations. I need only read two of the offences named in the discipline code set
out in Sch 1 to the regulationsn:
________________________________________
n Viz, Sch 1 to the Police (Discipline) Regulations, 1952, SI 1952 No 1705

1. Discreditable conduct, that is to say, if a member of a police force acts in any manner prejudicial to discipline
or reasonably likely to bring discredit on the reputation of the force or of the police service.
4. Neglect of duty, that is to say, if a member of a police force(a) neglects or without good and sufficient cause
omits, promptly and diligently to carry out anything which is his duty as a constable.

It is difficult to see how any action would be taken in any event in respect of breaches of the police discipline code without a
report or allegation of some kind being made and I am quite unable to accept the submission that something different perhaps of a
formal nature or some complaint from an extra judicial source is necessary before there can be said to be a report or allegation.
Streatfeild J ([1961] 2 All ER at p 533), accepted the submission of counsel for the watch committee that their action arose, not as
a result of any report or allegation, but from the knowledge which was common to them and the country as a whole that the
appellant was unfit for office.
I am unable to accept that this was the position. The watch committee did not act solely on the ground that the appellant was
unfit for office irrespective of any offence that he might have committed, as their finding shows. They found him guilty of
offences which were founded on a report or allegation which they had received from the learned judge who had presided at the
trial and certain statements made by members of the committee and the town clerk. 115I have not taken into account any other
reports or allegations, for, whatever the watch committee may have known personally about the trial, they did not have a
transcript of the evidence, it now appears, until after they had given their decision on 7 March. There is, I should add, no
substance in the point taken that the reference to a copy of the report or allegation on which the charge is based contained in reg 4
of the Police (Discipline) (Deputy Chief Constables etc) Regulations, 1952 (SI 1952 No 1706), shows that there must be a written
report or allegation ab initio. No doubt an oral allegation will have to be reduced to writing, but it may well originate as an oral
allegation will have to be reduced to writing, but it may well originate as an oral statement, as it did in this case before the
transcript of the observations of Donovan J was sent to the watch committee.
Once the position is reached that the Police Regulations apply as, in my opinion, they did, it is clear that no attempt was
made by the watch committee to follow the regulations. These have been set out in detail by my noble and learned friend Lord
Morris Of Borth-Y-Gest whose judgment I have had the opportunity of reading and with which I respectfully agree. As he says,
and the Court of Appeal would have taken the same view if they had regarded the Police Regulations as applicable, the watch
committee disregarded the regulations and did not begin to comply with them.
On both grounds therefore, failure to comply with the requirements of natural justice and failure to comply with the Police
Regulations, I would hold that the decision of the watch committee to dismiss the appellant taken on 7 March 1958, was invalid.
This is not an end of the matter for the appellant did not let matters rest but appealed to the Home Secretary as he was
entitled to do under the Police (Appeals) Act, 1927, from the dismissal under s 191 of the Municipal Corporations Act, 1882.
Thus, it is said, since the decision of the Home Secretary by virtue of the Police (Appeals) Act, 1927, was final, the appellant had
waived his right to bring an action in the courts alleging that invalidity. I doubt whether any question of waiver arises, but I
appreciate the force of the opinion expressed by my noble and learned friend Lord Evershed that if Parliament has stated that the
appeal is final, that is an end of the matter and the appellant cannot, as it were, start again and by an action for a declaration seek
to undermine the decision from which he has unsuccessfully appealed. The answer to this point is, I think, and here again I find
myself in disagreement with the Court of Appeal, as well as with my noble and learned friend Lord Evershed, that the decision of
7 March 1958, taken by the watch committee was at all times a nullity, and nothing that was done thereafter by way of appeal
could give it validity.
In all the cases where the courts have held that the principles of natural justice have been flouted I can find none where the
language does not indicate the opinion held that the decision impugned was void. It is true that the distinction between void and
voidable is not drawn explicitly in the cases, but the language used shows that where there is a want of jurisdiction, as opposed to
a failure to follow a procedural requirement, the result is a nullity. This was indeed decided by the Court of Exchequer in Wood v
Woad, where as here there was a failure to give a hearing.
In Spackman v Plumstead Board of Works, referring to another statute Lord Selborne said ((1885), 10 App Cas at p 240):
There would be no decision within the meaning of the statute if there was anything of that sort done contrary to the
essence of justice.

I would apply this language, whether the Municipal Corporations Act, 1882, or the Police Regulations are to be considered. In
either case the watch 116 committee in failing to give a hearing to the appellant acted without jurisdiction.
I would allow the appeal accordingly.

LORD DEVLIN. My Lords, I am satisfied that s 191(4) of the Municipal Corporations Act, 1882, is wide enough to permit the
dismissal of a constable on the grounds of unfitness in the sense of inadequacy as well as on the grounds of negligence or
misconduct. The way in which this power may be used has, since 1919, been controlled as to conditions of service by
regulations made by the Home Secretary under s 4(1) of the Police Act, 1919, which requires that every police authority shall
comply with the regulations so made. The Police (Discipline) Regulations, 1952, o create a number of disciplinary offences
contained in a disciplinary code, and provide in detail for the way in which a charge of such an offence is to be investigated and
determined before a decision to dismiss is taken. I do not find it necessary to determine whether, before 1919, the power to
dismiss for neglect of duty could be exercised administratively and without any sort of judicial inquiry. Nor do I need to decide
whether or not the power to dismiss for inadequacy is purely administrative. I am satisfied that in all matters to which the
regulations apply the power to dismiss must be exercised in accordance with them.
________________________________________
o 183 Viz, SI 1952 No 1705, as amended by SI 1953 No 636, and cf, the Police (Discipline) (Deputy Chief Constables, etc) Regulations, 1952
(SI 1952 No 1706)

It is argued that the regulations do not apply in the present case for two reasons. It is said in the first place that the
disciplinary code is expressed in phraseology unsuited to the activities of chief constables, and in particular does not cover the
gravamen of the charge against the appellant as indicated by Donovan J which was that he was revealed by his conduct as a bad
example to and a bad influence on the Brighton constabulary. Undoubtedly the discipline code appears to be drafted with the
lower ranks in mind. But by SI 1952 No 1706 it is expressly made applicable to chief constables p and it must be construed
accordingly. It contains a number of specific offences which a chief constable could hardly commit but also a number which he
certainly could. There are specific matters put against the appellant in this case which I think certainly fall under the head of
discreditable conduct, if not also of neglect of duty. In my judgment the disciplinary code should be regarded as a
compendium covering all misconduct and neglect of duty in the case of all ranks from chief constable downwards. I find it
impossible to believe that there was intended to be a residue of neglect to be dealt with at large and in relation to which the
offender is deprived of the protection afforded by the regulations. If a case of inefficiency or inadequacy can be made without
proof of misconduct or neglect, the regulations do not apply; but if the case involves an allegation (and I use that word as will be
seen hereafter in its widest sense) of a disciplinary offence the procedure laid down by the regulations must be followed.
________________________________________
p Police (Discipline) (Deputy Chief Constables, etc) Regulations, 1952, reg 18

This in my opinion is the result of the impact of the Act of 1919 on the earlier one of 1882. This division of the power under
the Act of 1882 is in practice less inconvenient than it might sound. In and before 1919 there was a power, such as is now
contained in the Police Pensions Act, 1948, s 1(1) (c), to provide by regulation for cases in which policemen may be required to
retire otherwise than on the ground of misconduct. It is difficult to believe that the power of summary dismissal would now be
exercised in any case in which no fault is alleged, so that in practice the power under s 191(4) of the Act of 1882 has become a
controlled one. Legally, however, the power remains and can be used. It is unnecessary to consider whether or not it could have
been used in this case because one of the grounds given for the appellants dismissal was neglect of duty. The watch
committee ought not to have reached a decision on this ground without following the regulations, unless it can be said (and this is
the second 117 point to be considered) that the regulations are by their own terms inapplicable on the facts of this case. It is
argued that reg 2 requiresq that before the procedure laid down can be instituted a report or allegation must be made; and that
where as in the present case a matter comes to the knowledge of the watch committee as one of public notoriety, the regulations
do not apply. I need not elaborate on the extraordinary resultsmy noble and learned friends Lord Reid and Lord Morris of
Borth-y-Gest have mentioned themwhich as it seems to me would follow if the protection against dismissal depends on
whether or not the supposed misdemeanours of a police officer have been reported in the press. Such a construction ought not to
be put on reg 2 unless the language compels it, and in my opinion the language of the regulation does not. I think that the word
allegation is to be given a wide meaning. The main object of reg 2, as is shown by the introductory words italicised r in SI 1952
No 1706, is not to provide for some formal initiation of proceedings, equivalent to a writ of summons or an information, but to
ensure that an officer is told of any allegation made against him so that nothing is done behind his back. I do not see how the
watch committee can deal with any disciplinary matter unless an allegation of some sort is made even if it be only by one of their
own number; and I think it fair to assume that the word is chosen as the widest one that could be thought of to comprehend every
way in which such proceedings could be started.
________________________________________
q Police (Discipline) Regulations, 1952, reg 2; cf, reg 1 of SI 1952 No 1706
r Regulation 1 of SI 1952 No 1706 is printed beneath a heading, which is in italics and reads Right of accused to be informed of, and to
make a personal explanation concerning, allegations. Regulation 2(1) of SI 1952 No 1705 and reg 1 of SI 1952 No 1706 both begin
Where a report or allegation is received from which it appears

It is not disputed that if the regulations are applicable, as I think that they are, they were not complied with. On this basis
two further questions arise. The first is whether it is open to the House to question the decision of the watch committee on this
ground, and here I agree entirely with the conclusion reached by my noble and learned friend Lord Morris of Borth-y-Gest that it
is. The second is what is the effect of non-compliance on the decision. It is argued for the appellant that the effect is to avoid ab
initio the decision of the committee. That must mean that the committee had no statutory authority to make any decision at all. If
they had, then, although the decision they made might be a bad one and one that could be quashed by the court by virtue of its
supervisory jurisdiction over the proceedings of inferior tribunals, it would not be void ab initio, but would be good until
quashed. To make it void ab initio there must be some condition precedent to the conferment of authority on the committee
which has not been fulfilled. It is argued that compliance with the regulations is a condition precedent. It is not expressly made
so and I am not prepared to make the implication. I am very reluctant to imply such a condition where none is expressed, for the
utter avoidance of a decision of this sort is a very grave matter. All that has been done on the face of it falls to the ground. Even
if the appellant were satisfied with it, it could be impugned by any third party. The court would have no discretion to quash or
not to quash. It can only declare to be a nullity that which in law has never been done at all.
I see no reason therefore why I should do more than read the regulations into the Act of 1882, not as a condition precedent to
the power to dismiss, but simply as rules that the committee is required to observe. I do not hold that compliance with all the
rules is by implication a condition precedent to the power to dismiss under s 191(4). But if one of the regulations itself imposes
expressly a condition precedent, it is another matter. I am driven to the conclusion that reg 11(1) does s. Regulation 5 providest
that the case shall be heard by a tribunal appointed by the police authority and reg 11(1) provides that u 118the decision of the
police authority on receipt of the report of the tribunal shall be either to dismiss the case or to impose various punishments,
including dismissal. I cannot regard the power of dismissal under reg 11(1) as something distinct from the power of dismissal
under s 191(4), and I think that the effect of reg 11(1) is to make the power to dismiss conditional on the receipt of the report. I
do not say that any defect in a report would invalidate a dismissal. But where, as here, there has been no report at all and no
inquiry to substantiate one, I think that the statutory authority to dismiss never was created, so that the act of dismissal was a
nullity. If it was a nullity, it is not seriously argued that any subsequent proceedings before the Secretary of State could bring it to
life. The result in my opinion is that your lordships should allow the appeal and declare the dismissal to be void.
________________________________________
s Viz, reg 5 and reg 11(1) of the Police (Discipline) (Deputy Chief Constables, etc) Regulations, 1952 (SI 1952 No 1706)
t Viz, reg 5 and reg 11(1) of the Police (Discipline) (Deputy Chief Constables, etc) Regulations, 1952 (SI 1952 No 1706)
u Viz, reg 5 and reg 11(1) of the Police (Discipline) (Deputy Chief Constables, etc) Regulations, 1952 (SI 1952 No 1706)

My lords, I cannot say that I regard this result as altogether satisfactory. It is not that I regard the watch committees
decision as inevitably right or as one that can be faulted only on the ground that justice has not appeared to be done. The
appellant has not seriously complained about being put out of office; and since he has told your lordships that he will not seek to
be reinstated, it is permissible for me to say that I think the decision on that point to have been inevitable. But he could, instead
of being dismissed, have been compelled to retire, and thus he would have been saved some or all of his pension rights. That is
an issue of substance deserving of careful consideration. What is unfortunate about the result is that it means that during the
whole time taken up in the elucidation of this difficult point of law, the appellant has legally been in office and entitled to the
appropriate emoluments. That would be so, I suppose, even if he had been in profitable employment elsewhere, for his claim
would be for salary and not for damages for wrongful dismissal. Whatever course is now taken, the appellant is likely to reap a
substantial benefit from the fact that the committee fell into the pardonable error (pardonable if only because their view of the
law was the same as that taken by all the lords justices in the Court of Appeal) that they were entitled to deal with this matter
administratively and in their unfettered discretion.
It can be said with much force that all this is the result of ousting the ordinary jurisdiction of the courts. If the statute was
drafted so as to make a dismissal, as the common law does in contracts of service, effective whether rightful or wrongful and to
give compensation for wrongful dismissal, the issue would have been tried by an ordinary court of law and the appellant would
have got no more and no less than his deserts. But the statute gives the judicial power to a committee or tribunal. If the object of
that were to make one side a judge in its own cause, I should not be sad to see it miscarry. But the object here is the creation of a
special code, stricter in some respects at least than the ordinary obligations of a contract of service, and of an independent tribunal
to aid in its administration. Such tribunals must always be subject to the supervisory jurisdiction of the High Court. But it does
not by any means follow that a defect of natural justice sufficiently grave to be a ground for quashing the resulting decision,
inevitably leads as in the present case to a declaration that the decision is void ab initio. It is necessary always to bear in mind the
distinction so clearly drawn by Lord Sumner in R v Nat Bell Liquors Ltd ([1922] All ER Rep 335 at p 348, letter h; [1922] 2 AC
128 at p 151) between a wrong exercise of a jurisdiction which a judge has and a usurpation of a jurisdiction which he has not. If
there is no jurisdiction, the decision is a nullity whether the court quashes or not. If there is jurisdiction but there has been a
miscarriage of natural justice, the decision stands good until quashed. The occurrence of a miscarriage does not require the court
to quash if it is satisfied that justice can be done in some other way. The court in a case like the present, for example, if the
decision had been voidable and not void, might have left the appellant to his remedy in damages, if any. Your lordships heard
some argument about 119 whether the court could, if it exercised its discretion to quash, do so on terms which would have put the
parties back into the position in which they would have been if the proper procedure had been followed from the outset. I need
not say more than that I should be prepared to listen to such a contention in an appropriate case and I should certainly be glad if
the court had the power to do justice in that sort of way when reviewing the decisions of inferior tribunals.
In the view that I take of this case there is not much that I can usefully say about the principles of natural justice and their
application to the procedure under s 191(4) of the Act of 1882. Whether or not they are to be applied to any statutory procedure
depends on an implication to be drawn from the statute itself; and the question whether such an implication should be drawn in
this case cannot be answered without a consideration of the Police Act, 1919, and the regulations made thereunder from which s
191(4) cannot be divorced. Since the regulations themselves prescribe the rules of justice that are to be followed, it seems to me
that there is nothing to be gained by seeking to ascertain what the position would be if the discipline code did not apply.
There are three points, however, on which I desire to comment. First, I express no dissent from the view that if s 191(4)
stood alone the decision to be made under it is not purely administrative. Secondly, I do most emphatically dissent from the view
that natural justice did not require the watch committee to hear the appellant because, as it was said, he had had a full opportunity
of putting his case before the trial judge. The appellant was not and could not have been compelled to put any case at all before
the trial judge; he was there to answer an indictment on trial by jury. It would be quite wrong if an accused was to be
embarrassed in the conduct on a criminal charge by the reflection that if he did not also satisfy the trial judge about the propriety
of his actions in other respects, it might thereafter be the worse for him. Thirdly, if there was apart from the regulations a
miscarriage of justice in this case (and I think on the whole that there was), I agree with the opinion of my noble and learned
friend Lord Evershed for the reasons which he has given that the miscarriage rendered the watch committees decision voidable
and not null and void ab initio.
I agree with the order proposed by my noble and learned friend on the woolsack.

Appeal allowed.

Solicitors: Haslewoods agents for Bosley & Co, Brighton (for the appellant); Sharpe, Pritchard & Co agents for Town clerk,
Brighton (for the defendants).

C G Leonard Esq Barrister.


120
[1963] 2 All ER 121

Gordon v Cradock
CIVIL PROCEDURE

COURT OF APPEAL
WILLMER AND UPJOHN LJJ
5 MARCH 1963
Court of Appeal Interlocutory appeal Leave to appeal Conditional leave to defend Whether leave to appeal required
Supreme Court of Judicature (Consolidation) Act, 1925 (15 & 16 Geo 5 c 49), s 31(1) (i), (2).

Practice Summary judgment Leave to defend Conditional leave to defend Appeal Grounds on which Court of Appeal
may interfere with conditions imposed by judge in exercise of his discretion under RSC, Ord 14, r 6.

No leave is required by either a plaintiff or a defendant to appeal against an interlocutory order made by a judge granting a
defendant leave to defend on conditions, since such an order is an order refusing unconditional leave to defend an action,
within those words in s 31(2) of the Supreme Court of Judicature (Consolidation) Act, 1925, and is deemed, by s 31(2), not to be
an interlocutory order, with the consequence that s 31(1)(i) (which imposes a general prohibition on appeal without leave from an
interlocutory order) does not apply (see p 122, letters b, e and h, post).
Where a judge has exercised his discretion under RSC, Ord 14, r 6, and imposed conditions as a term of giving a defendant
leave to defend, the Court of Appeal will not interfere with the exercise of his discretion unless there has been some error of
principle or misapprehension of fact on his part, or unless he has given undue weight to a particular aspect of the facts (see p 123,
letters b and d, post).

Notes
As to appeals to the Court of Appeal against interlocutory orders, see 30 Halsburys Laws (3rd Edn) 452, 459, 460, paras 855 and
868; and for cases on the subject, see 16 Digest 178, 835, and 3rd Digest Supp.
For the Supreme Court of Judicature (Consolidation) Act, 1925, s 31(1)(i), (2), see 5 Halsburys Statutes (2nd Edn) 360, 361.

Interlocutory appeal
The plaintiff appealed against an interlocutory order of Megaw J made on 7 February 1963, varying the conditions on which
leave to defend an action was given by Master Diamond from payment into court of 20,000, the whole sum claimed by the
plaintiff in the action, to payment into court of 5,000 having regard to certain special features of the case sworn by the plaintiff.
At the hearing of the appeal the plaintiff contended that leave to appeal was not required, and he gave notice in his notice of
appeal that, if leave to appeal were required, he would apply to the court for leave. He sought to have the judges order reversed
and the order of the master restored on the ground that the judge was wrong in law or in principle in varying the masters order
merely because the plaintiffs case contained sufficient peculiar features, so as not to warrant that the whole 20,000 be paid
into court; and on the ground that there was no evidence that the plaintiffs case had such peculiar features or alternatively, that
such features (if any) were not a sufficient reason for varying the masters order. Counsel for the defendant drew the attention of
the Court of Appeal to various matters which might be said to be peculiar features, eg that sums so large as 10,000 were
handed over in cash in a hotel, that the money was advanced in furtherance of a real estate scheme of which no particulars were
given, that there had been very considerable delay in instituting proceedings, and that it seemed doubtful whether the alleged
transaction could have been carried out without contravening exchange control legislation.

S Chapman QC and C W G Ross-Munro for the plaintiff.


M Littman QC and R Gavin Freeman for the defendant.
121

5 March 1963. The following judgments were delivered.

WILLMER LJ. A preliminary question has been raised on this appeal whether leave is required. Leave was sought from the
judge, but, as I understand it, was refused. There is no doubt that a plaintiff has no right of appeal against an order granting to a
defendant unconditional leave to defend; that is provided by s 31(1)(c) of the Supreme Court of Judicature (Consolidation) Act,
1925. There is equally no doubt that a defendant who is refused unconditional leave to defend has an absolute right of appeal
without the necessity of asking leave; that, I think, must be the effect of s 31(2) of the Act of 1925. Here the somewhat unusual
position arises that leave to defend on conditions has been granted, and the appeal is an appeal by the plaintiff. We have not
heard the appeal yet, but I understand that the plaintiff intends to complain of the conditions that have been imposed. In those
circumstances it is said that s 31(2) does not apply; that this is an ordinary interlocutory appeal, and that consequently it requires
the leave of the judge, or (on the judges refusal) the leave of this court in pursuance of s 31(1)(i). Curiously enough, we are told,
there is no direct authority on the point. In those circumstances it falls to us to make a decision one way or the other.
So far as I am concerned, I feel no doubt what the decision must be, viz, that this is a case in which no leave to appeal is
required. In my judgment the words of s 31(2) are much too plain to admit of any other construction. Section 31(2) is not
dealing in terms with the right of appeal; the object of the subsection is to define whether a particular order is or is not an
interlocutory one. The wording is:

An order refusing unconditional leave to defend an action shall not be deemed to be an interlocutory order within the
meaning of this section.

If one construes those words according to their ordinary meaning, it must follow that no leave is required when an appeal is put
forward by either party against an order such as that in the present case, ie, an order refusing unconditional leave.
I would only add this. It appears to me, with all respect to the argument to the contrary, that any other view would lead to
the strangest possible results. To take this case, for instance: supposing the defendant, as well as the plaintiff, had wished to
appeal against the order made by the judge (the order being one refusing him unconditional leave to defend) he would admittedly
have had an absolute right of appeal. On the defendants argument, the plaintiff would have had no right to cross-appeal unless
leave were obtained. That would seem to me to be a very strange result; and for my part I do not think that it is a result which
follows from the words of the statute if they are construed in accordance with their ordinary sense. In my judgment, therefore,
there is here an absolute right of appeal without leave, and the plaintiffs counsel may therefore proceed with the presentation of
the appeal without asking us for leave.

UPJOHN LJ. I agree entirely with the judgment which has just been delivered. The point is of the shortest. My Lord has
expressed exactly, and with his usual cogency, the reasons which impel me to the same conclusion; and I should only be repeating
in less felicitous language what he has already said if I added further words of my own. I therefore agree that no leave is required
for the plaintiff to prosecute this appeal.
[Their Lordships then heard the plaintiffs appeal on the merits.]

WILLMER LJ. This is an appeal from an order made by Megaw J, in chambers on 7 February of this year, whereby he varied a
previous order made by Master Diamond in proceedings under RSC, Ord 14. The effect of the order both of the master and of the
judge was that the defendant should have leave to 122 defend, but on conditions. It was in relation to the conditions that the
judge differed from the master. We are now invited on behalf of the plaintiff to restore the conditions imposed by the master, and
to that extent (and to that extent only) to allow the appeal from the judges order. [His Lordship referred to the facts of the case
and continued:] I find it impossible to say that the judge was wrong to take these matters into consideration. It appears to me that
the question what terms ought to be imposed on a defendant as a condition of giving him leave to defend is very much a matter of
discretion for the judge. That being so, this court is naturally reluctant to interfere with the exercise of discretion by the judge
below, particularly so experienced a judge, unless it can be shown that there has been some error of principle on his part, or that
he has in some way misapprehended the facts, or has given undue weight to this or that aspect of the facts. I do not say that if I
had been occupying his seat and the matter had come before me, I would necessarily have arrived at the same conclusion as he
did. It was clear at that stage that some condition ought to be imposed on the defendant before giving him leave to defend; and it
may very well be that, had I been sitting at first instance, I might have imposed some different condition. But, giving the matter
the best consideration that I can, I find it impossible to say that the judges exercise of his discretion was wrong, so as to justify
this court in interfering. On the whole, therefore, I come to the conclusion that this appeal should be dismissed.

UPJOHN LJ. I agree with the judgment which has just been delivered. Like my Lord, it might be, had I been sitting at first
instance, I would not have made the same order as the judge. That is not the matter that we have to consider. The judge, who has
a very wide discretion under RSC, Ord 14, r 6, decided on what conditions he would give leave to defend. That was the only
question that he had to consider; and the only matter that we have to consider is whether he has wrongly exercised his discretion.
For the reasons given fully by my Lord, I agree that we cannot disturb that conclusion, and I agree that the appeal must be
dismissed.

No leave to appeal to the Court of Appeal required. Appeal dismissed.

Solicitors: Cooper, Bake, Fettes, Roche & Wade (for the plaintiff); Basil Greenby & Co (for the defendant).

F A Amies Esq Barrister.


123
[1963] 2 All ER 124

St Pancras Borough Council v Frey


LANDLORD AND TENANT; Tenancies

QUEENS BENCH DIVISION


LORD PARKER CJ, ASHWORTH AND WINN JJ
8 MARCH 1963

Landlord and Tenant Small tenement Recovery of possession Housing purposes Evidence Whether necessary to prove
that possession was required for the purpose of exercising housing functions Small Tenements Recovery Act, 1838 (1 & 2 Vict c
74), s 1 Housing Act, 1957 (5 & 6 Eliz 2 c 56), s 158(2).

A tenancy of premises let at a rent exceeding 20 a year by the appellant local authority having been duly determined by notice to
quit, the local authority served noticea on the tenant of their intention to recover possession under the Small Tenements Recovery
Act, 1838. At the hearing of the complaint the local authority proved the tenancy, its determination and the service of the
statutory notice, but tendered no other evidence, in particular no evidence of the purpose for which possession was required.
Section 158(2)b of the Housing Act, 1957, empowered a local authority to obtain possession under the Act of 1838 of any
building, whatever its rent, where the authority required possession of it for the purpose of exercising their powers under any
enactment relating to housing. The justices dismissed the complaint.
________________________________________
a This notice is required by the Small Tenements Recovery Act, 1838, s 2; 13 Halsburys Statutes (2nd Edn) 858
b Section 158(2) is as follows: Where a local authority for the purpose of exercising their powers under any enactment relating to housing,
require possession of any building or any part of a building of which they are the owners, then, whatever may be the value or rent of the
building or part of a building, they may obtain possession thereof under the Small Tenements Recovery Act, 1838, as in the cases therein
provided for, at any time after the tenancy of the occupier has expired, or has been determined.

Held The complaint was rightly dismissed, because, in order to be entitled to take advantage of the rights conferred by s 158(2)
of the Act of 1957, a local authority must tender some evidence to show that the condition precedent to jurisdiction, viz, that
possession was required for the purpose of exercising the authoritys housing powers, was satisfied.
Shelley v London County Council ([1948] 2 All ER 898) distinguished.
Dictum of Cassels J, in R v Snell, Ex p Marylebone Borough Council ([1942] 1 All ER at pp 614, 615) and Jenkins v
Paddington Borough Council (12 May 1954, unreported) considered.
Appeal dismissed.

Notes
As to the recovery of possession of small tenements, see 23 Halsburys Laws (3rd Edn) 710, para 1454; and for cases on the
subject, see 31 Digest (Repl) 615618, 73057332.
For the Small Tenements Recovery Act, 1838, see 13 Halsburys Statutes (2nd Edn) 855; and for the Housing Act, 1957, s
158, see 37 Halsburys Statutes (2nd Edn) 441.

Cases referred to in judgments


Jenkins v Paddington Borough Council (12 May 1954), unreported.
R v Snell, Ex p Marylebone Borough Council [1942] 1 All ER 612, [1942] 2 KB 137, 111 LJKB 530, 167 LT 13, 106 JP 160, 31
Digest (Repl) 616, 7318.
Shelley v London County Council, Harcourt v London County Council [1948] 2 All ER 898, [1949] AC 56, [1949] LJR 57, 113
JP 1, 31 Digest (Repl) 616, 7317.

Case Stated
This was a Case Stated by justices for the county of London acting in and for the petty sessional division of New River, in respect
of their adjudication as a magistrates court sitting at St Pancras Town Hall, in the county of London.
124
On 8 August 1962, a complaint was preferred by the appellants, the Mayor, Aldermen and Councillors of the Metropolitan
Borough of St Pancras, against the respondent, William Edward Frey, alleging that the appellants let to the respondent a tenement
consisting of 191, Caversham Road, NW5, that the tenancy had been determined by notice to quit on 7 May 1962, and that on 1
August 1962, the appellants had served on the respondent a notice in writing of their intention to apply to recover possession of
the tenement by giving the notice, and reading over and explaining the purport and intent of it, to the respondent c. The appellants
further alleged that notwithstanding the notice, the respondent neglected to deliver up possession of the tenement and still
detained it. The complaint was preferred under the Small Tenements Recovery Act, 1838, as extended by the Housing Act, 1957,
s 158(2). The following facts were found. A verbal weekly tenancy existed between the appellants and the respondent and the
tenancy was determined by a notice to quit on 7 May 1962. The appellants served on the respondent on 1 August 1962, a notice
in writing in accordance with the requirements of the Small Tenements Recovery Act, 1838, but the respondent did not deliver up
possession of the premises.
________________________________________
c This is the procedure laid down by s 2 of the Small Tenements Recovery Act, 1838; 13 Halsburys Statutes (2nd Edn) 858

It was contended by the appellants that on the facts the justices must issue a warrant of ejectment pursuant to the Small
Tenements Recovery Act, 1838, as extended by s 158 of the Housing Act, 1957, and that the appellants need not give any reason
for their application. It was contended by the respondent that, in order to succeed on their application, the appellants must prove
that they required possession for the purpose of exercising their powers under an enactment relating to housing within the
meaning of s 158(2) of the Housing Act, 1957, and that they had failed to do this.
The justices were of the opinion that the contention of the respondent was correct and that the appellants had not sufficiently
satisfied them that they required possession for the purpose of exercising their powers under an enactment relating to housing.
Accordingly, they refused the application. The appellants now appealed.

H B Grant for the appellants.


R G Rougier for the respondent.

8 March 1963. The following judgments were delivered.

ASHWORTH J delivered the first judgment at the request of Lord Parker CJ. The respondent was a weekly tenant of the
appellants of a tenement dwelling at a rent exceeding 20 per annum and the tenancy was determined by notice to quit on 7 May
1962. On 1 August 1962, notice in writing in accordance with the Small Tenements Recovery Act, 1838, was duly served on the
respondent, who did not give up possession of the premises. The complaint was heard on 16 August when evidence was adduced
on behalf of the appellants to prove the tenancy and its determination, and also the statutory notice required under the Act of
1838. No other evidence was tendered on their behalf: in particular there was no evidence to the effect that the respondent was
an unsatisfactory tenant or had committed a breach of the tenancy agreement, nor was there any specific evidence that the
appellants required possession for the purpose of exercising their powers under any enactment relating to housing. The justices
upheld a submission put forward on behalf of the respondent that in the circumstances the appellants claim had not been made
out, and accordingly they dismissed the complaint.
The statutory basis for the appellants claim before the justices is to be found in s 158 of the Housing Act, 1957, which re-
enacted (subject to an alteration not material to the present case) the provisions of s 156 of the Housing Act, 1936. Speaking
quite generally, the effect of s 158 is to remove from a tenant in the position of the present respondent the protection of the Rent
Restriction Acts and also to widen the jurisdiction under the Small Tenements Recovery Act, 1251838, by removing the
limitationd that the rent must not exceed 20 per annum. The advantages conferred on local authorities by s 158 are however
expressly confined to cases in which possession of premises is required for the purpose of enabling a local authority to exercise
their powers under any enactment relating to housing. The issue in this appeal is whether it is incumbent on a local authority,
seeking to recover possession of premises in pursuance of the provisions of s 158, to adduce evidence that possession is required
for the purpose mentioned in the section. Reference was naturally made to the decision of the House of Lords in Shelley v
London County Council in which the corresponding provisions of the Housing Act, 1936, were under consideration. The main
point of that decision relates, in my view, to the scope of a local authoritys powers of management regulation and control of
houses provided by them under the Act, and the problem raised in the present appeal was not considered. Lord Porter said
([1948] 2 All ER at p 900; [1949] AC at p 66):
________________________________________
d See s 1 of the Act of 1838; 13 Halsburys Statutes (2nd Edn) 855

It is, to my mind, one of the important duties of management that the local body should be able to pick and choose
their tenants at their will.

In that case it was accepted that the local authority on recovering possession intended to re-let the premises to another tenant, and
the possibility that possession was or might be required for some purpose not connected with housing did not arise. For my part I
do not consider that the decision in Shelley v London County Council is conclusive of the present appeal. It is certainly no
authority for the proposition advanced by counsel on behalf of the appellants that the mere service of a notice to quit is evidence
that such notice was given for the purpose of exercising their powers relating to housing.
Although the decision in R v Snell, Ex p Marylebone Borough Council turned on a different point, there is a passage in the
judgment of Cassels J, which foreshadows the issue raised in this appeal ([1942] 1 All ER at pp 614, 615; [1942] 2 KB at pp 144,
145):

If it could have been contended in this case that this local authority did not require possession of these premises for the
purpose of letting to another member of the working classes, but for the purpose of erecting a school, or widening a road or
for some other purpose which was not included in the Act, the tenant would have been entitled to succeed, and, indeed, the
magistrate would have been entitled to say that the court of summary jurisdiction had no jurisdiction to hear that
application.

I respectfully agree with that comment but it does not, and did not purport to, decide whether a local authority is bound to tender
evidence that their purpose in seeking possession is the statutory purpose mentioned in s 158 of the Housing Act, 1957.
Apart from these two reported cases reference was also made to an unreported decision of this court: Jenkins v Paddington
Borough Councile. In that case the local authority had passed a resolution that the tenancy should be terminated as it was not in
all respects satisfactory and this resolution was in evidence before the magistrate. The local authority did not however tender
evidence of an intention to re-let. The tenants appeal to this court was dismissed on the ground that the removal of an
unsatisfactory tenant was an exercise of the statutory powers of management and control of houses vested in the local authority.
For my part I do not read the judgments in that case as indicating that no evidence need be tendered by a local authority showing
that possession is required for the purpose of exercising their powers under an enactment relating to housing. The decision was
that evidence actually tendered was enough.
126
________________________________________
e (12 May 1954), unreported; see The Times 13 May 1954

In my judgment the effect of s 158 of the Housing Act, 1957, is to confer rights on a local authority in regard to recovering
possession of premises if (but only if) possession is required for the purpose mentioned in the section. It seems to me that in
order to take advantage of those rights a local authority must tender some evidence to show that their claim falls within the
section by establishing that the condition precedent to jurisdiction is satisfied. In most cases it is probable that there will be no
dispute and it is to be emphasised that, as Shelley v London County Council shows, the local authority can pick and choose its
tenants. Its motive for getting rid of a particular tenant or putting in another tenant is quite irrelevant in proceedings such as the
present proceedings. I would dismiss the appeal.

WINN J. I agree.

LORD PARKER CJ. I also agree.

Appeal dismissed.

Solicitors: Town clerk, St Pancras (for the appellants); M S Marks & Co (for the respondent).

W Beckett Terrell Barrister.


[1963] 2 All ER 127

Harvey v Smith-Wood
CIVIL PROCEDURE

QUEENS BENCH DIVISION


LAWTON J
7 MARCH 1963

Document Admissibility in evidence Previous written statement of witness at variance with his evidence Statement adduced
in examination-in-chief as evidence of the facts Evidence Act, 1938 (1 & 2 Geo 6 c 28), s 1(1).

A witness on behalf of the plaintiff gave evidence which was apparently in conflict with the plaintiffs own evidence. Counsel
for the plaintiff then sought to put in evidence a statement in writing made by the witness, who was an elderly man, more than six
years before the hearing.

Held The written statement was admissible under the Evidence Act, 1938, s 1(1) a, although this use of the provision should be
adopted only in very special circumstances.
________________________________________
a Section 1(1) so far as relevant, provides: In any civil proceedings where direct oral evidence of a fact would be admissible, any statement
made by a person in a document and tending to establish that fact shall, on production of the original document, be admissible as evidence
of that fact if the following conditions are satisfied, that is to say(i) if the maker of the statement either(a) had personal knowledge of
the matters dealt with by the statement; and (ii) if the maker of the statement is called as a witness in the proceedings

Cartwright v W Richardson & Co Ltd ([1955] 1 All ER 742), and Bearmans, Ltd v Metropolitan Police District Receiver
([1961] 1 All ER 384) considered.

Notes
As to the admissibility of written statements made by witnesses, see 15 Halsburys Laws (3rd Edn) 315, para 573.
For the Evidence Act, 1938, s 1(1), see 9 Halsburys Statutes (2nd Edn) 626.

Cases referred to in judgment


Bearmans Ltd v Metropolitan Police District Receiver [1961] 1 All ER 384, [1961] 1 WLR 634, 125 JP 268, 3rd Digest Supp.
Cartwright v W Richardson & Co Ltd [1955] 1 All ER 742, [1955] 1 WLR 340, 3rd Digest Supp.

Action
In this action the plaintiff claimed damages for negligence and the defendant counter-claimed damages for defamation. The case
is reported only with regard 127 to the submission by counsel for the plaintiff that a written statement made more than six years
before the hearing by one of the plaintiffs witnesses was admissible in evidence. The facts and the argument are stated in the
judges ruling.

D J C Ackner QC and S Terrell for the plaintiff.


W L Mars-Jones QC and D C W Hirst for the defendant.

7 March 1963. The following judgment was delivered.

LAWTON J. Counsel for the plaintiff has asked me to admit in evidence under the Evidence Act, 1938, a statement made by a
witness, Mervyn Drummond, who is at the moment giving evidence before me. The circumstances in which counsel comes to
make his application are as follows. It has become evident in this case that an important question of fact, with which I shall have
to deal when I come to deliver judgment, is what happened at the plaintiffs house, Orchard Lea, on or about 8 January 1951,
when the defendant, as he admits, called at that house and took possession of two deed boxes. The plaintiff in her evidence,
which has already been given, asserted that she did not see the defendant on that occasion; and she asked me to infer that the
defendant had not looked into the deed boxes whilst he was at her house. It is manifest that the object of calling Mr Mervyn
Drummond was to support the plaintiffs evidence in that respect. But when Mr Drummond went into the witnessbox he seemed
to be giving evidence which was in conflict with that which the plaintiff had already given. Counsel for the plaintiff, quite
properly, made one or two attempts to call Mr Drummonds attention to the point of the questions which he was putting, because
counsel seemed to be under the impression that Mr Drummond had not got his mind on the questions. Mr Drummond, however,
stuck to the evidence which he had already given: namely, that the boxes had been opened in his presence, at least, by the
defendant, and possibly in the presence of the plaintiff, though that is not altogether clear from what Mr Drummond in fact said.
Thereupon counsel for the plaintiff asked me to look at a statement in writing which had been made by Mr Drummond in 1956.
Counsel for the plaintiff has submitted to me that that statement is admissible under s 1(1) of the Evidence Act, 1938 b.
Counsel for the defendant has resisted that application strongly, and he has called my attention to the judgment of Barry J, in
Cartwright v W Richardson & Co Ltd. An analogous situation arose in that case, but it is not on all fours with the present
situation, because in the case dealt with by Barry J counsel for the defendants sought in re-examination to get admitted into
evidence a document containing something which the witness had said in writing on an earlier occasion. It is clear, however, that
when Barry J was delivering his judgment in that case he dealt with it, not on a technical point of procedure, but on general lines
as to the policy of the Act, saying in effect that the Evidence Act, 1938, had not changed the law relating to the rules on
previously inconsistent statements.
________________________________________
b The relevant terms of s 1(1) are set out in the footnote at p 127, ante

Counsel for the plaintiff has replied to that by calling my attention to the decision of the Court of Appeal in Bearmans Ltd v
Metropolitan Police District Receiver. That case, although it was decided on its own facts, as Sellers LJ, was at pains to stress
([1961] 1 All ER at p 388), did, however, call the attention of judges to the general purposes for which the Evidence Act, 1938,
was passed and the policy of that Act; and, as Devlin LJ pointed out ([1961] 1 All ER at p 392), the Act draws a distinction
between the question of admissibility and the question of the weight which is to be attached to a document when it has been
admitted.
Counsel for the plaintiff also called my attention to a passage in the text-book 128 written by Dr Rupert Cross, who is a
Fellow of Magdalen College, Oxford. It is Cross on Evidence (1958 reprint), and the passage in question comes at p 214. Dr
Cross puts the problem which I am called on to consider in the following words:

If A. calls W. to testify on a number of points, and W.s testimony concerning one of them is less favourable to A. than
a statement contained in W.s proof, that document would appear to fall precisely within the terms of s. 1(1) of the Act of
1938, but, in Cartwright v. W. Richardson & Co., Ltd. BARRY, J., held that this was not so. He reached this conclusion
because the Act has not changed the ordinary rules of procedure applicable in the trial of civil actions, and, if such evidence
were admissible, it might have the result of shaking the faith of the court in the evidence given by the witness in the box.
With respect, the rule that the consistency of witnesses may not be proved by their previous statements and the rule that
prior contradictory statements of witnesses under cross-examination are not evidence of the facts stated related to the
procedure at trials of civil actions, but we have seen that it is difficult to escape the conclusion that they have been reversed
by the Act of 1938. Accordingly, it is submitted that this part of the decision in Cartwright v. W. Richardson & Co. Ltd.
may have to be reconsidered on some future occasion. The truth is that the full effect of the Evidence Act on the
admissibility and utilisation of the previous statements of witnesses has not yet fully been worked out in the reported
cases.

I have come to the conclusion, with some regret, that on the proper construction of s 1(1) of the Act of 1938, the document which
counsel for the plaintiff seeks to have admitted in evidence can be so admitted. I say that I have come to that conclusion with
some regret because it seems to me that it is an unfortunate situation if counsel can call a witness and, when that witness does not
come up to proof, counsel should be allowed to produce some earlier document which shows that on some other occasion the
witness made a different statement. I appreciateand I have borne it in mind in coming to my decision as to the proper
construction of the Actthat the object of the Act is to see that the relevant evidence is before the court; and it may well be in the
sort of situation which has arisen in this case, it is right, in case one side or the other seeks to rely on evidence given by a
particular witness, that the court should have full information as to what the witness has said on some previous occasion.
Nevertheless, it seems to me that the use of this provision should be one which counsel should hesitate to adopt except in very
special circumstances. Having myself observed Mr Mervyn Drummond in the witness-box, I will content myself by saying at
this stage that I can well understand counsel thinking that this is such a special case. Mr Mervyn Drummond is an elderly man,
and it may well be that he is not as fit now as he was at the time when he made the statement which is sought to be admitted in
evidence.

Ruling accordingly.

Solicitors: Victor Mishcon & Co (for the plaintiff); Gascoin & Co (for the defendant).

Mary Colton Barrister.


129
[1963] 2 All ER 130

Inland Revenue Commissioners v Olive Mill Ltd (in liquidation)


Inland Revenue Commissioners v Olive Mill Spinners Ltd (in liquidation)
TAXATION; Other Taxation

CHANCERY DIVISION
BUCKLEY J
4, 5 MARCH 1963

Profits Tax Chargeable accounting period Last period of company Investment companys voluntary winding-up No longer
investment company after winding-up resolution Finance Act, 1937 (1 Edw 8 & 1 Geo 6 c 54), s 19(4).
Profits Tax Subsidiary company Principal companys notice that subsidiarys profits be treated as principals Determination
by winding-up of principal company Finance Act, 1937 (1 Edw 8 & 1 Geo 6 c 54), s 22(1).

In 1956 an investment company which held all the shares in a cotton spinning company gave a grouping notice pursuant to s 22
of the Finance Act, 1937, as a result of which the profits of the spinning company fell to be treated as the profits of the
investment company for profits tax purposes. On 12 December 1957, both companies passed resolutions for voluntary winding-
up. The investment company was assessed to the profits tax in the sum of 2,142 18s and to a distribution charge of 30,644 for
the period 27 January 1957, to 12 December 1957, which period the Commissioners of Inland Inland Revenue determined to be
an accounting period in accordance with s 20(2) of the Act of 1937. The spinning company was assessed to the profits tax in the
sum of 193 16s and a distribution charge of 71 4s for the chargeable accounting period 13 December 1957, to 25 January 1958,
on the basis that the grouping notice ceased to be effective on the commencement of the winding-up of the companies on 12
December 1957. The Special Commissioners of Income Tax found that the functions of the investment company, viz, the holding
of investments, continued after 12 December 1957, to be what they had been before that date and that the period then ending was
therefore not the companys last chargeable accounting period, and they discharged the assessments.

Held (i) The assessment on the investment company would be upheld because
(a) the commissioners finding that the functions of the investment company continued to be the same before and after the
resolution for voluntary winding-up was one of mixed fact and law, for its propriety depended on the proper interpretation of s
19(4) of the Finance Act, 1937; and therefore the finding was open to review (see p 137, letter i, post); and
(b) on the commencement of the winding-up of the investment company, its functions, which meant its activities appropriate
to its business, viz, its objective, ceased to be the holding of investments for the purpose of obtaining a benefit by doing so and
became the realisation of its assets and distribution of the proceeds among creditors and members (see p 138, letters c and e,
post).
Dictum of Atkinson J, in Inland Revenue Comrs v Buxton Palace Hotel, Ltd ((1948), 29 Tax Cas at p 344), and dictum of
Lord Reid in Henry Briggs, Son & Co Ltd (in liquidation) v Inland Revenue Comrs ([1961] 1 All ER 223) applied.
(ii) the assessment on the spinning company would be upheld because on 12 December 1957 (the commencement of the
winding-up of the investment company) the grouping notice under s 22 of the Finance Act, 1937, ceased to be operative with the
consequence that the spinning company became assessable to the profits tax in respect of any subsequent chargeable accounting
period, for the following two reasons
(a) the spinning company ceased to be a subsidiary of the investment 130 company within the definition of subsidiary in
s 42(1) of the Finance Act, 1938, because on the commencement of the winding-up of the investment company the capital of the
spinning company then held by the investment company ceased to belong beneficially to the investment company and became
trust property to be applied in discharge of its liabilities and in due course of administration (see p 139, letters c and f, post).
Dicta of James and Mellish LJJ, in Re Oriental Inland Steam Co ((1874), 9 Ch App at pp 559, 561) applied.
(b) the last accounting period of the investment company ended on 12 December 1957, and the investment company
thereafter was not a company to which the relevant sections of the Finance Act, 1937, were applicable (see p 139, letter b, post).
Appeals allowed.

Notes
As to the meaning of chargeable accounting period for the purposes of the profits tax, see 20 Halsburys Laws (3rd Edn) 617,
618, para 1204.
As to the duration of grouping notices, see ibid, 641, para 1253.
For the Finance Act, 1937, s 19(4) and s 22(1) see 12 Halsburys Statutes (2nd Edn) 375, 377.

Cases referred to in judgment


Briggs (Henry) Son & Co Ltd (in voluntary liquidation) v Inland Revenue Comrs [1961] 1 All ER 220, 39 Tax Cas 410, [1961] 1
WLR 68, 3rd Digest Supp.
Carpet Agencies Ltd v Inland Revenue Comrs (1958), 38 Tax Cas 223, 3rd Digest Supp.
Inland Revenue Comrs v Buxton Palace Hotel Ltd (1948), 29 Tax Cas 329, 28 Digest (Repl) 375, 1638.
Oriental Inland Steam Co, Re, Ex p Scinde Ry Co (1874), LR 9 Ch App 557, 43 LJCh 699, 31 LT 5, 10 Digest (Repl) 903, 6137.

Cases stated
The first taxpayer company, the holding company, appealed to the Special Commissioners of Income Tax against an assessment
to the profits tax for the chargeable accounting period 27 January 1957, to 12 December 1957, in the sum of 2,142 18s, and a
distribution charge for the same period in the sum of 30,644. The question for determination was whether the accounting period
to 12 December 1957, was the last chargeable accounting period of the company, as the Crown alleged, in which case the
distribution charge was admitted to be justified by virtue of various statutory enactments and particularly s 35(1) of the Finance
Act, 1947, or whether, as the company contended, its functions continued after 12 December 1957, to be what they were before
that date, viz, the holding of investments, so that it continued to be chargeable to the profits tax after 12 December 1957. The
commissioners found that the functions of the company, viz, the holding of investments, continued to be the same before and
after the resolution to wind up which was passed on 12 December 1957. Accordingly, the chargeable accounting period ending
on 12 December 1957, was not the last chargeable accounting period of the company, and its appeal succeeded. The Crown
appealed by way of Case Stated to the High Court.
The second taxpayer company, the spinning company, appealed to the Special Commissioners of Income Tax against an
assessment to the profits tax for the chargeable accounting period 13 December 1957, to 25 January 1958, in the sum of 193
16s, and a distribution charge for the same period of 71 4s. It was agreed that the decision in the case depended on the decision
in that of the first taxpayer company, the holding company, which owned all the shares in the spinning company. If the
commissioners decided, as they did, that the period to 12 December 1957, was not the last chargeable accounting period of the
holding company, then by reason of the grouping notice given by the holding company under s 22 of the Finance Act, 1937, with
respect to the profits of the spinning company, 131the profits of that company continued to be treated as the profits of the
holding company and the assessment on the spinning company fell to be discharged. In view of their decision in the holding
companys case the commissioners discharged the assessment on the spinning company. The Crown appealed by way of Case
Stated to the High Court.
The cases noted belowa were cited during the argument in addition to those referred to in the judgment.
________________________________________
a Re Farrows Bank Ltd [1921] All ER Rep 511, [1921] 2 Ch 164, Re General Rolling Stock Co, Joint Stock Discount Cos Claim, (1872), 7
Ch App 646, Inland Revenue Comrs v Costa Rica Ry Co Ltd (1942), 29 Tax Cas 34

H Magnus QC, E Blanshard Stamp and A S Orr for the Crown.


R Borneman QC and P Rees for the taxpayer companies.
5 March 1963. The following judgment was delivered.

BUCKLEY J. These are two appeals from decisions of the Special Commissioners of Income Tax relating to assessments to the
profits tax made on two companies, Olive Mill Ltd, which I will call the holding company, and Olive Mill Spinners Ltd, which I
will call the spinning company. Before 1953 the holding company had carried on the trade of cotton spinning. In 1953 the
spinning company was incorporated and acquired from the holding company its business of spinning cotton in consideration of
an issue to the holding company of shares of the spinning company credited as fully paid. From that time forward the holding
company held shares in the capital of the spinning company and also owned and held certain other investments. In fact, it
became an investment company. The spinning company continued to carry on the trade of spinning cotton. That state of affairs
continued until 12 December 1957, when both companies passed appropriate resolutions to go into members voluntary winding-
up.
In the course of 1956 the holding company gave a grouping notice pursuant to s 22 of the Finance Act, 1937, as a result of
which, so long as that notice remained effective, the profits of the spinning company fell to be treated as the profits of the holding
company for profits tax purposes. It is not disputed that, until the date of the resolution to wind up, the holding company was
chargeable to the profits tax in respect both of its own profits and of the profits of the spinning company in consequence of the
grouping notice. The major question in this case is whether, as a result of the resolution to wind up the holding company, the
chargeable accounting period for profits tax purposes applicable to the holding company which was then current and which then
came to an end was the last chargeable accounting period of that company, or whether the company remained chargeable to
profits tax so that the period ended 12 December 1957, was not the last chargeable accounting period.
Section 19(1) of the Finance Act, 1937, creates the charge to the profits tax in respect of each chargeable accounting period
of the liable company. Subsection (2) provides:

(2) Subject as hereafter provided, the trades and businesses to which this section applies are all trades or businesses of
any description carried on in the United Kingdom, or carried on, whether personally or through an agent, by persons
ordinarily resident in the United Kingdom.

Subsection (4) is in the following terms:

(4) Where the functions of a company or society incorporated by or under any enactment consist wholly or mainly in
the holding of investments or other property, the holding of the investments or property shall be deemed for the purpose of
this section to be a business carried on by the company or society.

Whether in the absence of sub-s (4) the business of an investment company would be held to be a trade or business within sub-s
(2) is perhaps now not very relevant because the section does in fact include sub-s (4). Subsection (4) may perhaps have been
included in the section ex majore cautela to make 132 sure that the business of an investment company would be caught by the
section; but as the section stands it is, I think, clear that the business of an investment company is chargeable if it falls within sub-
s (4), but not otherwise; in other words, the business of an investment company is not within sub-s (2). That view was shared by
Harman J, who in Carpet Agencies Ltd v Inland Revenue Comrs ((1958), 38 Tax Cas at p 229) said this:

This clearly means that, in the view of the legislature, in the absence of the subsection [sub-s. (4)] the mere holding of
investments or property would not constitute the carrying on of a business.

Section 30 of the Finance Act, 1947, contains provisions which are relevant to this case, but which I need not read, relating to
reliefs in respect of profits which are not distributed and distribution charges which fall to be made when retained profits are
distributed in a later accounting period.
The holding company was assessed to the profits tax for the chargeable accounting period 27 January 1957, to 12 December
1957, in the sum of 2,142 18s and a distribution charge for the same period in the sum of 30,644. The company appealed to
the Special Commissioners against that assessment. The propriety of that assessment depends on the proper view of the question
whether the accounting period ended 12 December 1957, was or was not the last chargeable accounting period of the company or
whether, as the company contended before the Special Commissioners, its functions continued after 12 December 1957, to be
what they were before that date, viz, the holding of investments, so that it continued after that date to be chargeable to the profits
tax; ie, so that it continued after that date to fall within the terms of s 19(4) of the Act of 1937.
Among the facts found by the Special Commissioners in their Case stated in the holding companys appeal was that in 1957,
in consequence of the difficulties that had then arisen in the cotton spinning trade, the directors of both companies were minded
to dispose of their assets to the best advantage before losses, which they anticipated, might be incurred. Various courses of action
were considered and investigated, but eventually the two companies were put into liquidation by the resolutions of 12 December
1957. The liquidator who then came into the saddle continued the efforts, which the directors had theretofore been making, to
dispose of the assets of the spinning company to the best advantage or to dispose of the shares of members of the holding
company. The liquidator for a time continued to hold the shares of the spinning company, and also continued to hold the other
investments of the holding company. Those other investments were sold in January, 1958, and in the same month the liquidator
sold the mill and machinery which belonged to the spinning company, but by arrangement with the purchasers he continued to
process yarn which belonged to the spinning company at the mill and to have the use of the offices and warehouse house there.
The yarn so processed was sold as occasion offered. The last sale of yarn did not take place until October, 1958. The secretary of
the holding company was retained in the employment of that company until that date because, as the commissioners find, he had
a very good knowledge of the cotton industry which presumably was of value to the liquidator. During this period there were
negotiations for the sale of the shares of the holding company on terms which contemplated that the liquidation might be stayed,
but these came to nothing.
On 21 May 1958, the liquidator, as liquidator of the spinning company, made a first distribution to the holding company as a
shareholder in the spinning company of a sum of 124,000, and on 17 March 1959, in the same capacity he made a further
distribution to the holding company of 18,000. On those same dates as liquidator of the holding company he made a first
distribution of sums amounting to 180,764, and on 24 March 1959, a further distribution amounting to 18,076. The
Commissioners of Inland Revenue under their statutory 133 powers determined that the period from 27 January 1957, to 12
December 1957, should be an accounting period for the relevant purposes of the holding company.
The contention of the holding company before the Special Commissioners was that its function of holding investments
continued after 12 December 1957, in the same way as it had existed before that date, and that accordingly the period ending on
12 December 1957, was not its last chargeable accounting period, because it remained a company chargeable to profits tax under
s 19(4) of the Act of 1937. On the other hand, the Crown contended that the companys trade or business ceased for the relevant
purposes on 12 December 1957, at the latest, and that that date was the end of the last chargeable accounting period. In para 6 of
the Case Stated the Special Commissioners say this:

We the commissioners who heard the appeal, found that the functions of the company, namely, the holding of
investments, continued to be the same before and after the resolution to wind up which was passed on Dec. 12, 1957.
Accordingly, the chargeable accounting period ending on Dec. 12, 1957, was not the last chargeable accounting period of
the company and the appeal must succeed.

It is from that conclusion that the Crown now seek to appeal.


As regards the spinning company, an assessment was made for the chargeable accounting period from 13 December 1957, to
25 January 1958, in the sum of 193 16s (tax) and a distribution charge for the same period in the sum of 71 4s. That
assessment was made on the footing that on 12 December 1957, the grouping notice which had been given by the holding
company ceased to operate, so that from that date the spinning company was chargeable in its own right to the profits tax in
respect of the profits earned as the result of the carrying on of the spinning companys business by the liquidator in the manner
that I have already indicated. The spinning company appealed against that assessment, and before the Special Commissioners the
matter was dealt with on the footing that the result of that appeal depended on the decision in the appeal of the holding company;
ie, on the footing that, if it could be shown that the holding company remained chargeable to the profits tax and that the functions
of the holding company remained the same after 12 December 1957, as before, the grouping notice would continue in operation
so that the profits of the spinning company would be treated as the profits of the holding company and the spinning company
would not be separately assessable.
Before me a point of law has been taken which I will explain later, which was not taken before the Special Commissioners
or brought to their notice.
The position with regard to the holding company turns on the question whether on the true interpretation of s 19 of the Act
of 1937 the functions of the holding company continued after 12 December 1957, to be wholly or mainly the holding of
investments or other property, and in that connexion I have been referred to various authorities. There are only two to which I
shall need to refer in any detail, but there are earlier decisions which are relevant and are sufficiently cited in those authorities for
it to be unnecessary for me to refer to them separately.
In Carpet Agencies Ltd v Inland Revenue Comrs the relevant company carried on the trade of dealing in carpets until 31
March 1953. It then ceased to deal in carpets. At that time it held certain investments and other assets but had disposed of all its
trading stock. The question which arose was whether after 31 March 1953, the company was still chargeable to the profits tax. It
had ceased to carry on its trade of dealing in carpets and so had ceased to come within the terms of s 19(2). Had it brought itself,
however, within the terms of s 19(4)? Harman J referred amongst other authorities, to the decision of Atkinson J in Inland
Revenue Comrs v Buxton Palace Hotal Ltd. In that case Atkinson J considered the construction of s 19(4), in relation to a
company the business of which had been carrying on a hotel but that hotel 134 had been requisitioned so that at the relevant time
the company was merely in receipt of a compensation rent. Atkinson J had to consider whether in those circumstances the
company could be said to be a company the functions of which consisted wholly or mainly in holding investments or other
property. Atkinson J said this ((1948), 29 Tax Cas at p 334; 38 Tax Cas at p 229):

Now, what is the meaning of the word functions? The dictionary definition (I think it is a very good one for the
purposes of this case) is: the activities appropriate to any business; so that it ought to read: Where the activities
appropriate to any business consist wholly or mainly in the holding of investments or other property. So read, of
course, it is plain that the words would only apply to a company whose business could be so described. The activities
appropriate to this business of hotel proprietors certainly did not consist wholly or mainly or at all in the holding of
investments or other property, but it is in effect argued that the words should be read as equivalent to activities simpliciter
during the relevant periodwhat, in fact, it is asked, during the relevant chargeable period were the activities of the
company? I do not accept that interpretation of the word. I think that sub-s. (4) is aimed at companies of a particular
nature and quality, companies who set out to make profit in the way indicated or who have changed their business into one
of that character.

Harman J said that he entirely agreed with those observations and then he, for himself, said this ((1958), 38 Tax Cas at p 230):

It seems to me that in order to get within s. 19(4) you must prove not merely that the company is one that happens to
hold some income-bearing investments but that one of its functions, that is to say one of its purposes, has always been or
has been for a considerable time the making of money by the holding of investments.

In a later case he somewhat modified that statement because he said that he did not think he was justified in saying that one had
to find that it had always been or had been for a considerable time one of the purposes of the company. The question was
whether at the relevant time it was one of the purposes of the company b.
________________________________________
b Henry Briggs, Son & Co Ltd (in liquidation) v Inland Revenue Comrs, [1960] 1 All ER at p 804; 39 Tax Cas at p 427

After that reference to Inland Revenue Comrs v Buxton Palace Hotel Ltd. Harman J went on thus ((1958), 38 Tax Cas at p
230):

On the facts stated in this Case, which I need not repeat, the commissioners held that this was not a company within s.
19(4) of the 1937 Act. With that finding I find myself entirely in agreement. I am of the opinion that the appellants never
had but one business, the business of carpet merchants. It is true it held some investments, but those were wholly ancillary
to its merchanting business and were used in fact to finance that very business,

and accordingly he reached the conclusion that on the true interpretation of s 19(4) the company, Carpet Agencies Ltd did not
come within the terms of that subsection.
The other case to which I must refer is Henry Briggs, Son & Co Ltd (in voluntary liquidation) v Inland Revenue Comrs. In
this case the relevant company, Briggs Collieries Ltd had owned coal mines and had engaged in the business of coal mining. As a
result of the nationalisation of the coal industry those assets, ie, the companys mine and coal mining business, had been
compulsorily acquired and the company became entitled to receive compensation under the 135 Coal Industry (Nationalisation)
Act, 1946. On the nationalisation of its colliery interests the company considered various ways of employing its resources in
different fields, but after 1948 it devoted itself to securing its compensation under the nationalisation provisions and ceased to
seek for fresh business. Eventually it went into liquidation. The question there under consideration was whether, during the
period when the company was entitled to receive the compensation but not carrying on any other kind of business, it could be
properly described as a company the functions of which consisted wholly or mainly in the holding of property. Upjohn J after
referring to the passage from Atkinson J, which I have read, reached this conclusion at the end of his judgment ((1961), 39 Tax
Cas at p 423):

A company whose business is temporarily stopped by requisition does not satisfy that test [i.e., ATKINSON, J.s test];
nor does one which has given up tradingsee the cases I have mentioned. They do not bind me, because in this field every
case depends on its own facts once the true meaning of the word functions has been determined; and in this case it is true
to say that Briggs never could re-enter into its colliery activities. Nevertheless, on ceasing to trade as a colliery company, a
company might do one of several things. It might decide to engage in some other trading activity, in which case, in my
judgment, it would remain a trading company; it might decide to become an investment trust company, in which case it
seems to me clear that it would engage in a new business of holding property. If it does neither but resolves to wind up, it
does not seem to me to gain any new character in the sense of carrying on an activity of holding property as being
appropriate to its business. It really has no business. It seems to me it is carrying out a residual function as a colliery
company. It can no longer trade as such; it can only receive its compensation and die,

and on those grounds he came to the conclusion that the company was not within s 19(4).
I do not think that I need read any part of the judgment of the Court of Appeal. Once again Atkinson Js interpretation of the
word functions is referred to without disapprovalindeed, with approvaland Upjohn Js decision was upheld. The case went
to the House of Lords, and in the House of Lords Lord Reid said this ([1961] 1 All ER at p 223):

The Court of Appeal and UPJOHN, J., reached that result by giving to the word functions a meaning which is to my
mind narrower than its ordinary meaning. I do not at all dissent from their view, but I would prefer to lay more stress on
the word holding. It appears to me that, in this context, the phrase the holding of investments or other property does not
mean simply the owning of investments or other property. The word holding often involves the idea of retention
permanently or for an indefinite time, and I think that that is involved in the use of that word in the context. I have said
that the apparent purpose of sub-s. (4) is to bring within the scope of the charge companies conducting something at least
analogous to a trade or business. From that point of view, there appears to me to be an essential distinction between
companies whose function or purpose or objective is to make profit by continuing to hold their assets in the form of
investments or property and companies which are not trying to make profitable use of their assets but whose whole
activities are directed to realising their assets as soon as possible. In the former case, the companys activities are of a
business character. They will almost always involve some degree of active management. Circumstances may for a time
require the company to remain quiescent, but it still has the purpose of deriving profit from continuing to hold its assets in
the form of investments or property. But, in the latter case, the function or purpose or objective of the company 136 is not
to continue to hold investments or property but to cease holding its assets in that form as soon as they can be turned into
money.

On those grounds he agreed with the view which had been taken by the lower courts.
Lord Morris of Borth-y-Gest, at the end of his speech says this ([1961] 1 All ER at p 225):

[It is to be observed that the subsection (i.e., s. 19(4)] does not provide that the mere ownership by a company of
investments or other property brings about the result that such ownership is to be deemed to be a business carried on by the
company. There is only such a deeming if the functions of the company consist wholly or mainly in the holding of
investments or other property. The conjunction of the words functions and holding suggest, to my mind, something
more than a mere ownership of some property and certainly something more than an enforced or involuntary ownership of
rights to receive compensation money, which ownership it is hoped will be terminated at the earliest possible moment by
the receipt of the money. The company did not set out to acquire or to own investments or property, and, just as there was
nothing intentional or purposeful in its acquisition of its right to compensation, so there was no intention or wish to
continue to own such rights but, on the contrary, there was an eager wish for an early settlement followed by a dissolution.
In these circumstances I cannot think that the company was one of which it could aptly be said that its functions consisted
wholly or mainly in the holding of investments or other property.

On those authorities counsel for the Crown submits that, on the true construction of s 19(4), the word functions is
synonymous with the word purposes or objectives. Secondly, he submits that, for a company to be within s 19(4), it must be
shown not merely that it holds investments or other property but also that its purpose is to continue to hold them in order to
derive advantages therefrom. Thirdly, he says that it is impossible, as a matter of law, to attribute such a purpose to a company in
liquidation, for on liquidation it ceases to become the beneficial owner of its assets and the statutory duty of the liquidator is to
realise those assets as soon as is convenient for the benefit of creditors and members of the company. In that connexion I was
referred to the Companies Act, 1948, s 281, which provides:

In the case of a voluntary winding-up, the company shall, from the commencement of the winding-up, cease to carry
on its business, except so far as may be required for the beneficial winding-up thereof.

The commencement of the winding-up in the case of a voluntary winding-up is, of course, the date of the resolution.
On the other hand, counsel for the holding company has drawn attention to the fact that in none of the cases to which I have
referred was there any earlier history of the company being what I may call a s 19(4) company. The question in those cases was:
had the company in each case acquired the character of being a s 19(4) company? He says that it is common ground that, until
the date of the winding-up resolution, the functions of the holding company were wholly or mainly the holding of investments or
other property, and he says that there was no significant change in that state of affairs at the time of the resolution to wind up. He
has contended that the question what the functions of the company are is a question of fact, and he has submitted that the
commissioners in para 6 of the Case Stated have found the fact that the functions of the company continued to be the same before
and after the resolution to wind up. That finding, I think, must be at least a finding of mixed fact and law, for the propriety of the
finding must depend on the proper interpretation of s 19(4) and the application of that section to the facts of this particular case.
In my judgment it is not possible 137 to say that this court is precluded from considering whether the Special Commissioners
came to the right conclusion because they have framed that paragraph in the language that I have read c. Counsel for the holding
company submits that one must distinguish the functions of the liquidator relating to the realisation of the companys assets and
the application of the proceeds of such realisation from the functions of the company, and he submits, as I say, that the functions
of the company were unaffected by the resolution to wind up.
________________________________________
c See p 134, letter c, ante

Now, although the cases to which I have referred are certainly distinguishable from the present case on the facts, it appears
to me that the decision of the House of Lords in Henry Briggs, Son & Co Ltd v Inland Revenue Comrs provides the answer to the
present problem; for, from the moment of the passing of the resolution to wind up, it was no longer the objective of the company
to continue to hold investments for the purpose of obtaining a benefit by doing so. From that moment the objective of the
company became the realisation of its assets as soon as they could advantageously be realised and the distribution of the proceeds
amongst creditors and members in accordance with their rights in a winding up. To borrow the language of Lord Reid, the
function or purpose or objective of the company ceased to be to continue to hold investments or property and became the
realisation of those assets or turning them into money. No doubt, in order that the assets might be realised in an orderly and
advantageous manner it was right and proper that there should have been some period of time before the assets were in fact
realised, but the objective of the company, from 12 December 1957, onwards was not, in my judgment, that of holding
investments; its objective was to realise its investments.
In that connexion it is important to bear in mind, I think, what Atkinson J said. It is not merely the activities of the company
that are relevant but the activities appropriate to its business, ie, appropriate to its objective. As I read his judgment in the light of
the later decision of the House of Lords, that is really what he means when he uses the words activities appropriate to any
business. He emphasises that activities simpliciter are not the test; one has got to see what are the activities appropriate to a
business set on a particular course. From 12 December 1957, onwards, the course on which the company here was set was the
realisation of its assets. No doubt, it is quite true, that, if the liquidator had found that it would be advantageous for the members
of the company to sell their shares to purchasers who might have wished to have the liquidation stayed, that course might have
been adopted. That course was considered, although that idea came to no fruition. But that was not one of the liquidators duties.
His duties were those provided by the Companies Act, 1948, ie, to realise the assets and to distribute the proceeds amongst the
creditors and members in accordance with their rights. For these reasons I think that the Special Commissioners took a mistaken
view of the proper interpretation to be put on s 19(4) and misdirected themselves in that respect. I think that the conclusions at
which they arrived were wrong and that the appeal in the holding companys case succeeds.
Now, the appeal in the spinning companys case depends on the question whether or not the spinning company remained
grouped with the holding company for the purpose of the Finance Act, 1937. The relevant section for this purpose is s 22 of the
Act of 1937, which, by sub-s (1) enables a company of which another company is subsidiary to give a grouping notice in respect
of the chargeable accounting period in which the notice is given, and that notice continues during all subsequent chargeable
accounting periods throughout which the subsidiary company remains a subsidiary of the principal company. By s 42 of the
Finance Act, 1938, the term subsidiary company is defined. Subsection (1) provides:

For the purposes of s. 22 of the Finance Act, 1937 a body corporate shall be deemed to be a subsidiary of
another body corporate if and so long as 138 not less than three-quarters of its ordinary share capital is owned by that other
body corporate,

and sub-s (3) provides that references to ownership shall be construed as references to beneficial ownership. So that the test is
whether the parent company is the beneficial owner of not less than three-quarters of the ordinary share capital of the subsidiary
company. It seems to me that in fact the grouping notice ceased to be operative on 12 December 1957, on two grounds. First, on
the view that I have taken on the holding companys appeal, the last accounting period of the holding company ended on that date
and, therefore, that company was after that date no longer a company to which the relevant sections were applicable, and the
grouping notice would on that ground have then ceased to operate so that the spinning company would be assessable in its own
right in respect of profits earned after that date.
The grouping notice, I think, also falls to the ground for the reason that the holding company, on going into liquidation,
ceased to be the beneficial owner of the shares in the spinning company within the meaning of the section that I have just read.
In this connexion I refer to the decision of the Court of Appeal in Re Oriental Inland Steam Co Ex p Scinde Ry Co. In that case
James LJ said this ((1874), 9 Ch App at p 559):

The English Act of Parliament has enacted that in the case of a winding-up the assets of the company so wound up are
to be collected and applied in discharge of its liabilities. That makes the property of the company clearly trust property. It
is property affected by the Act of Parliament with an obligation to be dealt with by the proper officer in a particular way.
Then it has ceased to be beneficially the property of the company; and, being so, it has ceased to be liable to be seized by
the execution creditors of the company.

Mellish LJ expressed a similar view. He said ((1874), 9 Ch App at p 561): But, in my opinion, the beneficial interest is clearly
taken out of the company. It has not been suggested to me that there has been any subsequent decision indicating that the views
of the learned lords justices are not still good law, and it therefore appears to me to be established by the authority of that case
that, on the winding-up resolution being passed, the beneficial interest in the spinning companys shares held by the holding
company ceased to reside in the holding company. Consequently the spinning company ceased to be a subsidiary of the holding
company within the meaning of these statutory provisions. For these reasons I think that the appeal of the Crown in the spinning
companys case also succeeds and that the assessment on the spinning company should be restored.

Appeals allowed.

Solicitors: Solicitor of Inland Revenue; Field, Roscoe & Co (for the taxpayer companies).

F A Amies Esq Barrister.


139

[1963] 2 All ER 140

Hall v Hall
FAMILY; Children

COURT OF APPEAL
ORMEROD, DONOVAN AND RUSSELL LJJ
28 FEBRUARY, 1 MARCH 1963

Divorce Custody Jurisdiction Infants Wards of court Infants made wards of court in Chancery Division No order of
chancery judge as to custody or access Prior order of Divorce Division as to access Jurisdiction of Divorce Court to vary
order as to access Matrimonial Causes Act, 1950 (14 Geo 6 c 25), s 26(1) Law Reform (Miscellaneous Provisions) Act, 1949
(12, 13 & 14 Geo 6 c 100), s 9(2).

The fact that proceedings have been taken in the Chancery Division, by virtue of which infants become wards of court, does not
of itself (the Chancery Division having made no order as to custody) oust the jurisdiction of the Divorce Division under s 26(1) a
of the Matrimonial Causes Act, 1950, to make an order as to the custody of or access to the infants (see p 142, letter h, p 144,
letter g, and p 145, letter h, post); but it seems (per Russell and Ormerod LJJ), that in the event of any conflict between orders as
to custody of or access to infants made in the Chancery Division and the Divorce Division, the infants being wards of court in
proceedings in the Chancery Division, the orders of the Chancery Division would prevail (see p 146, letter f, and p 142, letter h,
to p 143, letter a, post).
________________________________________
a Section 26(1) is printed at p 141, letter h, post

Andrews v Andrews and Sullivan ([1958] 2 All ER 305) not followed.


Re Andrews (Infants) ([1958] 2 All ER 308) considered.
A husband and wife were married in 1942. There were three daughters, children of the marriage, aged sixteen, ten and eight
years. In July, 1961, the wife left the husband, and subsequently the magistrates made an order on her application giving her
custody of the children and allowing the husband access. In January, 1962, the wife petitioned for divorce on the ground of
cruelty. In May, 1962, an order was made in this suit allowing the husband access to the children. In July, 1962, the husband
issued an originating summons (under the Law Reform (Miscellaneous Provisions) Act, 1949, s 9) in the Chancery Division
making the children wards of court; the children continued to be wards of court, but no order was made in the Chancery Division
as to custody or access. In October, 1962, applications were made in the Divorce Division by both husband and wife to vary the
order as to access. On appeal from an order of the judge, the effect of which was to deny access to the husband pending the
hearing of the petition, which was expedited,

Held There was jurisdiction under s 26(1) of the Matrimonial Causes Act, 1950, to make the order and in the circumstances the
Court of Appeal would not interfere with the judges exercise of his discretion.
Appeal dismissed.

Notes
As to the power of the Divorce Court to make orders for the custody of and access to infants, see 12 Halsburys Laws (3rd Edn)
393, para 872.
For the Matrimonial Causes Act, 1950, s 26(1), see 29 Halsburys Statutes (2nd Edn) 413; and for the Law Reform
(Miscellaneous Provisions) Act, 1949, s 9(2), see 28 ibid, 777.

Cases referred to in judgments


Andrews v Andrews and Sullivan [1958] 2 All ER 305, [1958] P 217, [1958] 2 WLR 942, 28 Digest (Repl) 710, 2191.
Andrews (Infants), Re [1958] 2 All ER 308, [1958] Ch 665, [1958] 2 WLR 946, 28 Digest (Repl) 706, 2152.

Interlocutory appeal
This was an appeal by a husband, the respondent to a divorce petition by his 140 wife, from an order of Scarman J the effect of
which was to deprive him of access pending suit to the children of the marriage. The facts are stated in the judgment of Ormerod
LJ.

Peter Sheridan for the husband.


C W S Lubbock for the wife.

1 March 1963. The following judgments were delivered.

ORMEROD LJ. This appeal is from a decision of Scarman J given on 20 November 1962, on appeal by the wife from a
decision of the registrar dated 25 October 1962, on a summons for a variation of an order for custody in respect of the children of
the marriage. The parties were married in 1942. There are three children of the marriage: Ann Denise, now aged 16 and,
therefore, not the subject of any custody order; Marie Suzette, aged 10, and Yvonne Teresa, aged 8. The parties lived together
with their children until 18 July 1961, when the wife left the husband. She issued a summons against the husband on 16
November 1961, on the ground of the husbands desertion, persistent cruelty and neglect to maintain (we have not seen the
summons) and the magistrates made an order inter alia giving the custody of the three children to the wife, with access to the
husband. On 23 January 1962, the wife petitioned the High Court for a decree of dissolution on the grounds of the cruelty of her
husband towards her. On 30 May 1962, the husband applied to the Divorce Registry, when an order was made in his favour for
access. The part of the order with which we are concerned is that the husband should have access to the three children of the
marriage for

one weekend in each month on the Saturday from 2 p.m. to 7 p.m. and on the Sunday from 10 a.m. to 5.30 p.m.

For reasons into which I will go in a moment, the attempts on the part of the husband to have access to the children were not very
successful. On 25 October there was before the registrar a summons by both parties to vary the order as to access. So far as the
husband was concerned, it was an application that the children should come to him for Christmas; the wifes application was that
the husband should no longer have access, pending suit, to the children of the marriage. The learned registrar dismissed the
application and, therefore, the matter came before Scarman J who decided that he should allow the appeal. The effect of this was
that the husband should have no access to the children pending a decision in the suit. It is on appeal from that decision that the
matter comes before this court.
The first of the two points that have been argued by counsel for the husband is that, in the circumstances of the case, the
Divorce Division of the High Court has no jurisdiction to make an order. The matter was argued before the learned judge, and
the only authority on the point, Andrews v Andrews and Sullivan, was cited to him. He came to the conclusion that it was a case
in which he had jurisdiction and proceeded to hear the case on its merits. The order sought to be made in this case is an order
under s 26(1) of the Matrimonial Causes Act, 1950, which says:

In any proceedings for divorce or nullity of marriage or judicial separation, the court may from time to time, either
before or by or after the final decree, make such provision as appears just with respect to the custody, maintenance and
education of the children the marriage of whose parents is the subject of the proceedings or, if it thinks fit, direct proper
proceedings to be taken by placing the children under the protection of the court

and that would include the making of the children wards of court. On the face of it, provided that the question of access can be
properly included in the term custody, maintenance and education of the children, it would appear that the wording of the
section is sufficiently clear to include an order of this kind, and in the ordinary way I have not the slightest doubt that it would.
141
However, there is a complication which arises, because, on 3 July 1962, a date subsequent to the hearing of the summons by
the learned registrar, but prior to the hearing of the appeal by Scarman J an originating summons was taken out in the Chancery
Division the effect of which was that the children became wards of court b. So far as this court is aware, these children are still
wards of courtc, but no further steps appear to have been taken in regard to the wardship.
________________________________________
b By virtue of the Law Reform (Miscellaneous Provisions) Act, 1949, s 9(2) and RSC, Ord 54p., r 3, under which an infant becomes a ward of
court on the issuing of the originating summons but, unless within twenty-one days after its issue, an appointment is obtained for the
hearing of the summons, the infant ceases to be a ward of court at the end of the twenty-one days. On an appointment being obtained the
infant continues to be a ward of court until the determination of the application
c It is understood that an appointment was duly obtained before the Chancery Master and that the hearing of the summons was adjourned
generally

The submission made to the court today is that there can be no jurisdiction to make an order under s 26(1) of the
Matrimonial Causes Act if that order is in conflict, as it must be in this case, with the fact that the children have already become
wards of court. On the face of it, that contention would not seem to have substance, because Parliament has enacted that the
court (that is, the High Court) shall have power to make orders. It cannot be questioned, in the ordinary course of events, that the
order which is sought in this case is an order which comes within the terms of s 26(1). The simple answer to this submission
would be that, if the legislature had intended that there should be the restriction on the making of orders under s 26(1), then it
would have said so; whereas it has said in plain terms that this power would be given to the court. Clearly the effect of giving
such a power to the court (if it has in fact been given) can mean that in certain circumstances there could be a head-on clash
between, on the one hand, a judge exercising this jurisdiction in the Probate, Divorce and Admiralty Division and, on the other, a
judge of the Chancery Division exercising his jurisdiction over some infant who has properly been made a ward by originating
summons in the Chancery Division.
There has been, so far as we have been told, only one other case which might be said to deal with this point, and that is
Andrews v Andrews and Sullivan, which is a decision of Wrangham J, and a decision of Upjohn J, in Re Andrews (Infants) in the
Chancery Division of questions arising on the same matter. It is fair to say, after referring to the judgments in those two cases,
that Wrangham J was of the view that, in the circumstances of that case, as the children had been made wards of court, he had no
jurisdiction under s 26(1) to make the appropriate order or any order for custody or maintenance, and he declined to make such
an order; whereas Upjohn J in his judgment, would go no further than to say that it was probable that there was no jurisdiction
under s 26(1) to make an order of the kind referred to, but that the matter depended very largely on the comity which exists
between judges, and it would be necessary to rely on that comity to avoid a head-on clash if the two jurisdictions might be
attempted to be exercised.
How far those decisions are right is probably not a matter for the consideration of this court. I am quite satisfied that the
court in this case had jurisdiction to make the order asked for under s 26(1), because it appears to me to be clear, provided the
other conditions are fulfilled, that the words of the section put no limitation on orders which may be made by the court. I would
not, therefore, hesitate to say that the court has jurisdiction to make an order, even although in the meantime the infant has been
made a ward of court. It may be that the real position is that if, an infant having been made a ward of court, the judge of the
Chancery Division to whom that particular infant is assigned makes some order which clashes with an order which may be made
under s 26(1), the order of the chancery judge might well prevail; but that is a matter with which we are 142 not concerned. I go
no further than to say that, in the circumstances of that case, if it arose, as at present advised, such might be the position.
But that is not this case. This is a case where three infants have become wards of court in the Chancery Division and no
further step has been taken in regard to them in the Chancery Division. I see no reason why the jurisdiction given to the court
under s 26(1) should be ousted in this way. Therefore, I would say that the first ground on which counsel for the husband bases
his appeal is wrong, and I would dismiss that part of the appeal.
However, counsel for the husband goes further and says that, in the particular circumstances of the case, the decision was
wrong because the learned judge, who clearly had a wide discretion in this matter, exercised his discretion on wrong principles
and, therefore, this court has the power to interfere; and, more than that, should interfere by making a different order. The effect
of the learned judges order was this. As I have already said, proceedings for divorce have been started. The date of the petition
was 23 January 1962. In 1962 there were various attempts, set out in the affidavits, on the part of the father to achieve the access
awarded to him. The history of those attempts shows that, even on his own evidence, only on one afternoonthat is, a Saturday
afternoon in Junecould he say that there was any meeting between him and the children which could be in any way described
as successful. That is denied by the mother, presumably on information she had from the children. So far as the other attempts
were concerned, they were, unfortunately, a failure, because, according to the affidavits, of the reluctance of the children to meet
their father and spend time with him.
In those circumstances, the learned judge has come to the conclusion that it would not be wise to make an order for access
pending the hearing of the suit, and the ground on which he based his decision is this. While the suit is pending, clearly there are
bitter feelings between the husband and wife, and in the circumstances of this petition it is perhaps not unnatural that those bitter
feelings still exist. As the children live with the mother, it is almost impossible that they should not be to some extent influenced
by the feelings between their parents. In those circumstances, it would be unlikely that they would have a kindly feeling towards
their father. The learned judge has come to the conclusion that once the proceedings are over, and a decree is granted or refused,
the feeling between the parties may well be a good deal less bitter and access may be easier to achieve. It is not for me to
conclude one way or the other what is the real cause of the reluctance of the children to meet their father or to spend any time
with him. Whether this lies in the attitude of the father, or whether they have in some way or other been put against their father
by their mother, either directly or indirectly, I do not know. The learned judge has ordered that the proceedings should be
expedited in order that as short a time as possible should elapse before the matter is finally decided, and the question of custody
can then be considered afresh.
I see no ground for saying that the learned judge is wrong. Counsel for the husband has argued that on the decided cases the
real object of an order for custody pending suit is to maintain, so far as possible, the status quo which presumably existed at the
time of the separation. Of course, it is a forceful argument that if the husband cannot have access to the children now there is
little reason why he should have access when the decree nisi is made (if it is made) and the question of access is considered
again; but it appears to me that any guide which can be derived from the decided cases on this subject can only be a guide to the
exercise of discretion having regard to the general circumstances of the case. The learned judge has, I think, considered the
various matters which he should consider in deciding whether these children should be, as it were, compelled to see their father
on the occasions when access is ordered; and he has decided that for the time being, and I think for the time being only, it would
be unwise on the childrens behalf to make an order of that kind. In consequence, he has come to the conclusion that he should
allow the appeal from the learned registrar and make an 143 order which in effect denies access to the children by the father
pending the hearing of the divorce proceedings. All I can say, in conclusion, is that I am particularly anxious not to say anything
which might influence any other court when the question of custody has to be considered. The learned judge has made an order
that the proceedings should be expedited so that the matter can be dealt with as quickly as possible. I can only hope that both
parties will take every advantage of that and that at as early a date as possible the issue between the mother and father can be
decided, and an order can then be considered on the question of custody. It may be that in those circumstances some order for
access may be given to the father; but it is not for me to predict, and it is not for me to attempt to influence any court which may
at a later stage have to consider that question. All I say is that I would dismiss the appeal.

DONOVAN LJ stated the facts and continued. The proceedings in the High Court for divorce had begun, and an order for access
had been made in those proceedings, before the children became wards of court. It is in those circumstances that this question of
jurisdiction has to be considered.
Section 26(1) of the Matrimonial Causes Act, 1950, provides inter alia that once proceedings for divorce have begun the
court (which by s 33 of the Act means the High Court) may from time to time, either before or after final decree, make orders as
to the custody, maintenance and education of the children; or it may, if it thinks fit, direct that proper proceedings be taken to
place the children under the protection of the court. Once the Divorce Division of the High Court has begun to exercise
jurisdiction under the subsection, as, for example, by making interim orders for custody, which is the case here, I do not think that
that jurisdiction can as a matter of law be ousted by one or even both of the parties going to the Chancery Division and making
the children wards of court. I draw attention to the words of the subsection which give the High Court, and, therefore, the
Divorce Division of it, jurisdiction from time to time; and give it also jurisdiction, if it thinks fit, to direct other proceedings for
the protection of the children, which would include wardship proceedings. The husbands contention in the present case would
confer on a party to a matrimonial suit power to deprive the Divorce Division of the discretion which Parliament has conferred on
it; and for such a result, I think, clear words would be required. One recognises the risk that may arise in a case like the present,
of conflicting orders, and of some inconvenience if two divisions of the High Court are seized of the same matter; but the comity
between judges should be sufficient to take care of such a risk.
Upjohn J dealt with this aspect of the matter in Re Andrews in terms with which I respectfully agree. If Wrangham J in
Andrews v Andrews and Sullivan intended to say (which is not completely clear) that making a child a ward of court entirely
ousted the jurisdiction of the Divorce Division under s 26(1), I should be bound respectfully to disagree. The matter is purely one
of the construction of s 26(1) and I cannot spell such a result out of its language.
As to the merits of the case, I feel that we should not interfere with the learned judges decision, which, after all, is not a
permanent one; and, while I understand the fathers feelings, they are probably such as not to make him the wisest judge of what
to do for the best at this moment. Looking at the problem dispassionately, I should have thought that there was much to be said
for an interval for feelings on both sides to subside. I think it was short-sighted folly for the mother to insist on being present at
the first meeting; and for the future she should remember that as they grow up these children may well need the help and
affection of a father, and it would be a grievous wrong to the children deliberately to deprive them of that advantage. I agree that
the appeal should be dismissed.

RUSSELL LJ. On the question of the merits of the case, it seems to me that the learned judge considered on the evidence that
there was a considerable 144 adverse effect produced on these children by their meeting with their father. He also considered that
this adverse effect was in large measure due to the bitterness of the current dispute affecting both the parents. Against the harm
done to the children he balanced the claims of the father to access to his own children, and he decided, on balance, in effect, that
further access should be postponed until a time when the matter was decided, when it might be hoped the bitterness of the parents
would be less, and a time, therefore, when it was more likely that these two small children would benefit more from their
meetings with their father. I do not think it can be said that, in striking this balance, the learned judge, in dealing with a short-
term problem, has erred in principle.
On the question of jurisdiction, it appears to me that Wrangham J did decide as a matter of law that once a child became a
ward of court by the issue of an originating summons in the Chancery Division then the jurisdiction of the judge in the Divorce
Division over the custody of the infant under s 26(1) of the Matrimonial Causes Act, 1950, ended. I think the composer of the
headnote in Andrews v Andrews and Sullivan was right when he said:

Held, that since the powers of the Divorce Division were less extensive than those possessed by the Chancery
Division over wards of court, the former were superseded by the latter; the Chancery Division had, moreover, on the
mother making her application, become, in effect, the guardian of the wards of court, and it was that division only which
could give leave for them to go out of the jurisdiction or otherwise control their persons or property.

In Re Andrews (Infants) Upjohn J expressed the view that probably as a matter of law (though he did not put it in this way)
Wrangham J was right. I venture to differ from the Andrews v Andrews and Sullivan decision to a limited extent. A section in the
form of s 26(1), or substantially in that form, has been in the divorce legislation since the Matrimonial Causes Act, 1857. It must
be remembered that until the Law Reform (Miscellaneous Provisions) Act, 1949, which introduced the new procedure in
connexion with making infants wards of court (a procedure under which no child can become a ward of court unless an order is
made to that effect, subject only to the temporary effect of the launching of the originating summons), an infant could become a
ward of court unintentionally or accidentally, without the fact being appreciated, owing to the view that if a trust of any property
of an infant became subject to administration at the hands of the court, that infant automatically became a ward of court: indeed,
a system by which one deliberately made wards of court in former days was by producing an ad hoc 100 settlement on the infant
and asking the Chancery Division to administer the trust. It seems to follow from that that, since before 1949 there were many
instances of infants who were wards of court without anybody really appreciating it and without anybody taking any steps in the
Chancery Division in respect of the control of the infant and his custody, it would have been quite wrong to hold in those days
that automatically when an infant became a ward of court all custody jurisdiction under s 26(1), or rather its then equivalent d,
ceased. To that extent, and for that reason, I would disagree with what I take to be the decision of Wrangham J I do not think
that, either then or now, the mere fact of a child becoming a ward of court somehow ousts the custody jurisdiction of the judge in
matrimonial proceedings under s 26(1). On the other hand, to construe this section and its ancestor in 1857 e, as conferring a
jurisdiction on the judge in matrimonial proceedings to make an order setting at nought an order already made by a judge in the
Chancery Division exercising his jurisdiction over his ward would be, it seems to me, to attribute a very perverse intention to
Parliament. Before the Matrimonial 145 Causes Act, 1857,f the law undoubtedly was that in the case of a ward of court the Court
of Chancery had the absolute say in respect of matters such as custody, access, education and so on. I do not think that a
matrimonial statutory provision should be taken as authorising in effect, a conflict between an irresistible force and an immovable
object. I venture to agree with this comment by Upjohn J, in Re Andrews (Infants) ([1958] 2 All ER 308 at p 310; [1958] Ch 665
at p 668):
________________________________________
d Supreme Court of Judicature (Consolidation) Act, 1925, s 193(1), which had itself replaced the Matrimonial Causes Act, 1857, s 35, and the
like Act of 1859, s 4
e See Matrimonial Causes Act, 1857, s 35
f 20 & 21 Vict c 85; ie, before 1858

One thing is, I think, clear; an exercise by the judges of the Divorce Division of their statutory powers cannot in any
way fetter the powers of the Chancery Division exercising the jurisdiction of the Crown as parens patriae over wards of
court.

If that be right, then either s 26(1) of the Matrimonial Causes Act, 1950, must be construed in a limiting way or there is the
potential absolute conflict that I have indicated. In my judgment, as a matter of sensible approach to the statutory provision
against the background of the existing powers of the chancery judges, the appropriate approach is to construe the powers
conferred by that section on the divorce judge in matrimonial proceedings in a limited way, so as not to authorise the judge in the
Divorce Division to make an order contrary to an order of the judge in the Chancery Division.
The result of that, first of all, it seems to me, is that while the chancery guardian (if I may so describe the Chancery
Division) still slept, while he had not been stirred to come to a decision about the ward, there is no reason why the divorce judge
should not exercise his jurisdiction under the section. That was the position both when the registrar made his order and when
Scarman J, made his order; and it is the position in which we sitting in the Court of Appeal are making our decision. It may be
that in due course this question of access, amongst other questions, will be brought before a chancery judge in wardship
proceedings; it seems to me that that may well be done. He would then have to consider what course he should adopt. It might
be that the evidence before him would be different, in which case he would perhaps feel himself at liberty to say: Not only have I
as absolute guardian of this child the right to override the divorce judges view, but, having regard to various matters in this case
and this evidence, I think it right to do so. If that were to happen the decision of the judge of the Chancery Division would, in my
judgment, override the decision of the judge in the Divorce Division. If the matter comes before a judge in the Chancery
Division in this way, he will be faced with the sort of problem discussed by Upjohn J in Re Andrews and no doubt will be guided
by the considerations which are there set out whether the judge of the Chancery Division should leave it to the Divorce Division,
and probably de-ward; or should keep the child a ward, and on any particular matter content himself with the views, for example,
of Scarman J on the interim access; or should say, I will keep the child a ward, and I am going to take a different view about
that. I agree, for those reasons, perhaps differing slightly from my brothers, that both on the merits and on the question of
jurisdiction the appeal fails.

Appeal dismissed.

Solicitors: Edward F Iwi agent for Steed & Steed, Sudbury (for the wife); Dale, Parkinson & Co (for the husband).

Henry Summerfield Esq Barrister.


146
[1963] 2 All ER 147

Ministry of Agriculture, Fisheries and Food v Jenkins and Another


AGRICULTURE: LANDLORD AND TENANT; Tenancies: CONSTITUTIONAL; Crown

COURT OF APPEAL
LORD DENNING MR, DANCKWERTS AND DAVIES LJJ
12 MARCH 1963

Agriculture Agricultural holding Notice to quit Crown land Required for non-agricultural use No consent of
Agricultural Land Tribunal necessary Agricultural Holdings Act, 1948 (11 & 12 Geo 6 c 6), s 24(1), (2) (b), as amended by the
Agriculture Act, 1958 (6 & 7 Eliz 2 c 71), s 8, Sch 1, para 8 Town and Country Planning Act, 1947 (10 & 11 Geo 6 c 51), s
87(2) (b).

The Minister of Agriculture, Fisheries and Food gave notice to quit in respect of two agricultural holdings of which the minister
was the landlord. The stated purpose for which the notices to quit were given was afforestation by the Forestry Commission, a
non-agricultural use. The tenants served counter-notices. The consent of the Agricultural Land Tribunal to the operation of the
notices to quit was not obtained. Accordingly the notices would be invalidated by s 24(1) of the Agricultural Holdings Act, 1948,
unless they fell within s 24(2)(b)a of that Act, by which enactment the tribunals consent would not be needed if the notices to
quit were given for a non-agricultural use for which permission under the enactments relating to town and country planning was
not required otherwise than by virtue of any provision of those enactments.
________________________________________
a The relevant terms of s 24(2)(b) are printed at p 149, letter a, post

Held The Crown did not require planning permission in respect of its own interests in land, because the Town and Country
Planning Act, 1947, did not bind the Crown (s 87(2)(b) of that Act proceeding, on its true construction, on that assumption), and
this reason was one that was not by virtue of any provision of that Act; therefore the notices to quit fell within the exemption
enacted by s 24(2)(b) of the Agricultural Holdings Act, 1948, and the consent of the Agricultural Land Tribunal was not required.

Notes
As to the circumstances in which counter-notices do not restrict the operation of a notice to quit an agricultural holding, see 1
Halsburys Laws (3rd Edn) 284, 285, para 601; and for cases on the subject, see 2 Digest (Repl) 1618, 7176.
As to the presumption that the Crown is not bound by a statute unless the contrary is expressly stated or necessarily implied,
see 36 Halsburys Laws (3rd Edn) 430432, para 652; and for cases on the subject, see 42 Digest 689693, 10361082.
For the Agricultural Holdings Act, 1948, s 24, see 28 Halsburys Statutes (2nd Edn) 46, 47; and for the Agriculture Act,
1958, s 8 and Sch 1, para 8, see 38 ibid 76, 81.
For the Town and Country Planning Act, 1947, s 87(2), see 25 Halsburys Statutes (2nd Edn) 603.

Appeal
This was an appeal from the decision and order of His Honour Judge Trevor Morgan QC, given on 19 December 1962, on a
Special Case Stated for the opinion of the court by Frank Kynaston Ikin, the arbitrator appointed in an arbitration between the
Ministry of Agriculture, Fisheries and Food (the landlords) and Isaac R Jenkins (the first tenant) and Jenkins Hughes (the second
tenant) whereby differences were referred to arbitration in accordance with Sch 6 to the Agricultural Holdings Act, 1948. The
following were the material facts. Before 11 August 1955, the first tenant was the tenant on an annual Michaelmas tenancy of a
farm holding in the county of Cardigan, and the second tenant was the tenant of a similar holding in another parish in that county.
On 11 August 1955, the landlords, in exercise of powers conferred by the Forestry Act, 1945, purchased the reversion expectant
on both of these tenancies and became the landlords 147 of both tenants. By notice to quit dated 18 September 1961, the
landlords required the first tenant to quit his holding on 29 September 1962; the notice stated that it was given pursuant to s 24(2)
and s 31(1) and (2) of the Agricultural Holdings Act, 1948, with a view to the use of the land to which it related for the planting
of trees, and on the ground that the land was required for use other than agriculture for which (otherwise than by virtue of any
provision of the enactments relating to town and country planning) permission was not required, namely, the use of the land by
the Forestry Commission for the planting of trees, not being a use for woodlands where that use was ancillary to the farming of
land. On the same day a similar notice was similarly served by the landlords on the second tenant. On 10 October 1961, each of
the tenants served on the landlords a notice stating that he wished to contest the reason stated in the notice to quit served on him,
and that he required the question to be determined by arbitration under the Agricultural Holdings Act, 1948. The material facts
proved at the arbitration were (a) that the land to which the tenancies were subject was purchased in 1955 by the Minister of
Agriculture, Fisheries and Food under powers given by the Forestry Act, 1945; (b) that on 5 October 1955, the minister duly
made an order under the Act placing the land at the disposal of the Forestry Commission, and (c) that the commission required
the land for the purpose of afforestation, such afforestation not being ancillary to agriculture. At the hearing of the Special Case
the learned county court judge decided that the notices to quit were valid and effective. By notice of appeal dated 4 January
1963, the first tenant and the second tenant gave notice of appeal to the Court of Appeal on the grounds that the learned county
court judge was wrong in law in finding that the notices to quit were valid, and that he was wrong in law in finding that the
Crown was not bound by material provisions of the Town and Country Planning Act, 1947, namely, s 87 thereof, so far as the
interests of the first and second defendants were concerned in the said holdings.

Anthony Cripps QC and John Morris for the appellant tenants.


J C Leonard for the respondent ministry.

12 March 1963. The following judgments were delivered.

LORD DENNING MR. This raises a question of some importance under the Agricultural Holdings Act, 1948, and the Town
and Country Planning Act, 1947. The Ministry of Agriculture, Fisheries and Food are the landlords of extensive tracts of land in
Wales. On 18 September 1961, they gave notices to quit to two farmers, Mr Jenkins and Mr Hughes, in respect of considerable
areas of land there. The notices to quit were a years notice in each case to expire on 29 September 1962. No doubt the farmers
had used these tracts of land as sheep runs, but the ministry, having the interests of the Forestry Commission in their hands,
desired to use them for the planting of forests, and so they gave notice to quit for that purpose. The question which arises in this
case is whether those notices to quit have effect without more ado. The farmers say that they have no effect unless the
Agricultural Land Tribunal gives its consent to the operation of them.
Section 24(1) of the Agricultural Holdings Act, 1948, as amended b in consequence of s 3(1) of the Agriculture Act, 1958,
says that
________________________________________
b The amendment is effected by s 8 of and Sch 1, para 8, to the Agriculture Act, 1958

where notice to quit an agricultural holding is given to the tenant thereof [as was done here], and the tenant
serves on the landlord a counter-notice [as each of these tenants did here] the notice to quit shall not have effect unless
the Agricultural Land Tribunal consents to the operation thereof.

It is quite plain from s 87(2) of the Act of 1948, c that that provision applies to the areas of land here in question, notwithstanding
that they are Crown lands. So, stopping there, it would seem that the notices to quit would not have effect unless the Agricultural
Land Tribunal consented to the operation thereof 148and no such consent has been given. But sub-s (2)(b) of s 24, which is a
rather complicated section, goes on to say that sub-s (1) does not apply, that is to say, the consent of the tribunal is not required,
where
________________________________________
c 28 Halsburys Statutes (2nd Edn) 90

the notice to quit is given on the ground that the land is required for a use, other than for agriculture, for which
permission has been granted on an application made under the enactments relating to town and country planning, or for
which (otherwise than by virtue of any provision of those enactments) such permission is not required, and that fact is
stated in the notice.

That is a provision which I find difficult to follow. It means, I think, that a landlord does not have to get the consent of the
Agricultural Land Tribunal if (a) he requires the land for a non-agricultural use, and (b) in addition he shows either (i) that he has
got permission under the town and county planning Acts so to use it or (ii) that he does not require permission under those Acts.
In a case under (i) where he has already got planning permission, he must have made a specific application for non-agricultural
use and have been granted specific permission for it. The general permission under the Town and Country Planning General
Development Order, 1950,d will not do. And in a case under (ii), where he does not require permission for the non-agricultural
use, the reason must be by virtue of some extrinsic good reason and not by virtue of any provision in the town and country
planning Acts.
________________________________________
d SI 1950 No 728, art 3; see 21 Halsburys Statutory Instruments (1st Re-issue) 88

In order to illustrate the operation of s 24(2)(b) of the Agricultural Holdings Act, 1948, I take the case of a private landlord,
who desires to turn his land over to afforestation instead of agriculture. He does not have to get planning permission, for the
simple reason that the use of land for afforestation is not development of the land; see s 12(2)(e) of the Town and Country
Planning Act, 1947, which says that the use of any land for the purposes of agriculture or forestry (including afforestation) shall
not be deemed to involve development of the land. So planning permission is not required. But in that case it is by virtue of a
provision of the enactments relating to town and country planning that it is not required, namely, by virtue of s 12(2)(e) of the
Act of 1947. He is not therefore exempted under s 24(2)(b) of the Agricultural Holdings Act, 1948, from obtaining the consent of
the Agricultural Land Tribunal. It follows that a private landlord, who desires to turn his land over to afforestation, must get the
consent of the tribunal in order to make his notice to quit effective.
But what about the Crown? If the Crown desires to turn Crown lands over to afforestation, it does not have to get planning
permission. That is quite plain. But why does the Crown not have to get planning permission? Is it by virtue of any provision in
the town and country planning Acts? That is to say, is it by virtue of s 12(2)(e) of the Town and Country Planning Act, 1947, as it
would be in the case of a private landlord? Or is it by virtue of s 87(2)(b) of the Town and Country Planning Act, 1947? Or is it
by virtue of the fact that it is the Crown and, as such, it is not bound by an Act of Parliament except in so far as it is included
expressly or by reasonable implication?
Looking at the whole of the Town and Country Planning Act, 1947, I am satisfied that the Crown does not need to get
planning permission in respect of its own interest in Crown lands. The reason why it is exempt is, not by virtue of any provision
in the Act itself, but by reason of the general principle that the Crown is not bound by an Act unless it is expressly or impliedly
included. Section 87(2)(b) does not exempt the Crown. It proceeds on the assumption that the Crown is already exempt. It says
that,
Notwithstanding any interest of the Crown in land being Crown land any [planning] restrictions shall apply
and be exercisable in relation 149 to the land, to the extent of any interest therein held otherwise than by or on behalf of the
Crown.

That provision assumes that the Crown is already exempt in respect of its own interest in Crown land. All it does is to make sure
that other persons (eg its tenants) have to get planning permission in respect of their interests: and it preserves the Crown
exemption in respect of its own interests.
Coming back then to s 24(2)(b) of the Agricultural Holdings Act, 1948, I hold that planning permission is not required by
the Crown in order to use these lands for afforestation, and that the reason why it is not required is because it is the Crown, not by
virtue of any provision in the town and country planning Acts. The Crown is entitled therefore to the benefit of s 24(2)(b) and it
is unnecessary for it to get the consent of the Agricultural Land Tribunal. The notices to quit operate and are effective, even
without the consent of the Agricultural Land Tribunal.
I find myself, therefore, in agreement with the decision reached by the county court judge and I would dismiss this appeal.

DANCKWERTS LJ. I agree. At one period during the course of the arguments I was in considerable confusion, but I have
emerged from the wallow of statutory enactments into which counsel for the tenants dragged us with the clear idea that the
argument of counsel for the Ministry of Agriculture, Fisheries and Food is right. The basis of his argument is the indisputable
one that the Crown is not bound by statute, unless it is expressly mentioned or involved by necessary implication. I think that he
is right in saying that there is nothing which does involve the Crown in that way in the Town and Country Planning Act, 1947.
Accordingly, the claim of the Crown in the present case to be free from the question of consent is not based on any provision in
that Act. It does not have to rely on the exemption, or whatever it may be, of forestry from the definition of development in the
Town and Country Planning Act, 1947, and its claim could equally be put if the desired development or desired operation were
the provision of a camp for the military and not merely the planting of trees. In that case it seems to me that the Crown, in the
words of this rather peculiar subsection, has a case where such permission is not required otherwise than by virtue of any
provision of those enactmentsthose enactments for the present purpose being the Town and Country Planning Act, 1947.
Therefore, I think that the county court judge came to the right conclusion and that the appeal should be dismissed.

DAVIES LJ. I also have suffered on the same muddy journey as Danckwerts LJ but I feel that I have at last emerged on to firm
ground. I can express my conclusion in one sentence, I agree with counsel for the ministrys submission that counsel for the
tenants argument in effect comes to this, that the parenthesis in s 24(2)(b) of the Agricultural Holdings Act, 1948, otherwise
than by virtue of any provision of those enactments, really means by some Act of Parliament other than those enactments.
But, as my lord has said, the position of the ministry here does not arise out of any Act of Parliament; it arises out of the fact that
they represent for this purpose the Crown. I agree, therefore, that the appeal fails.

Appeal dismissed. Leave to appeal to the House of Lords granted.

Solicitors: Horace C Davies & Co agents for D Emrys Williams & Co, Aberystwyth (for the tenants); Solicitor, Ministry of
Agriculture, Fisheries and Food.

F Guttman Esq Barrister.


150
[1963] 2 All ER 151

Lewis and another v Daily Telegraph Ltd


Same v Associated Newspapers Ltd
TORTS; Defamation

HOUSE OF LORDS
LORD REID, LORD JENKINS, LORD MORRIS OF BORTH-Y-GEST, LORD HODSON AND LORD DEVLIN
3, 4, 5, 6, 12, 13, 17, 18, 19 DECEMBER 1962, 26 MARCH 1963

Libel Innuendo No extrinsic evidence to support innuendo Innuendo meaning allegedly implicit in ordinary meaning
Whether defendants entitled to ruling whether words capable of bearing innuendo meaning Newspaper report that police fraud
squad inquiring into companys affairs, and naming chairman Admission that words defamatory in ordinary meaning
Innuendo that chairman guilty of fraud Whether words capable of imputing guilt of fraud as distinct from suspicion Pleading
innuendoes RSC, Ord 19, r 6(2).

Libel Damages Assessment Jury Appeal Unreasonable award.

Libel Damages Income tax to be taken into account where loss of income results.

Libel Damages Two actions for similar libels in different newspapers Direction to jury to consider whether and how far
damage attributable solely to the libel in one action Defamation Act, 1952 (15 & 16 Geo 6 & 1 Eliz 2 c 66), s 12.

Where words are defamatory in their natural or ordinary meaning an innuendo does not constitute a separate cause of action
unless it is an innuendo that requires the support of extrinsic fact (see p 160, letter b, p 159, letter c, p 167, letter b, and p 170,
letter g, post; cf p 154, letter e, post).
Grubb v Bristol United Press Ltd ([1962] 2 All ER 380) approved.
Observations on the pleading of innuendoes (see p 171, letters b to d, post; cf p 166, letter h, post).
Two newspapers each published statements that officers of the City of London Fraud Squad were inquiring into the affairs
of the [R Co] and its subsidiary companies and that the chairman of the R Co was L L and the R Co brought actions for libel
against each newspaper. The two sets of actions were tried separately. L pleaded an innuendo to the effect that the statement
meant that he had been guilty of fraud or was suspected by the police of having been guilty of fraud or dishonesty in connexion
with R Cos affairs. R Co pleaded an analogous innuendo. The plaintiffs did not allege special damage. The defendants
admitted that the words were defamatory in their ordinary meaning, but pleaded justification in that the fraud squad were at the
time of publication inquiring into the affairs of R Co. The defendants did not seek to justify the extended meaning pleaded in the
innuendo. At the trial no extrinsic fact was proved in support of the innuendo, but the trial judge rejected the defendants
submission that the innuendo should not be left to the jury. The plaintiffs did not tender in chief any evidence of financial
damage. The defendants established that an inquiry by the fraud squad into R Cos affairs was in existence at the the material
time. The trial judge left to the jury two questions, namely, (1) whether they found for the plaintiffs or the defendants and, (2) if
for the plaintiffs, for what damages; but he did not leave separate questions to the jury as to the natural and ordinary meaning of
the words and as to the innuendo, and did not give any directions to the jury as to the innuendohe simply left to the jury the
innuendo meaning as a possible ordinary meaning. He did not caution the jury on the absence of any real evidence of loss of
business resulting from the libel. He referred to the fact that a similar action was about to be heard for a similar article published
at the same time, and intimated that the jury should take that into account giving to it such weight as they thought that it deserved
in assessing damages. In the first trial the jury awarded L 25,000 and the R Co 75,000 damages. Two 151 days later the jury
in the second trial awarded L 17,000 and R Co 100,000 damages against the other newspaper. On appeal,

Held (i) On the question of misdirection concerning the particular defamatory meaning of the words complained of
(a) where an extended defamatory meaning of words admittedly libellous in their ordinary meaning was put forward by a
plaintiff, the defendant was entitled to a ruling whether the words were capable of bearing that particular extended meaning; in
the present case the trial judge had not so ruled (viz had not ruled whether the words were capable of imputing guilt of fraud as
distinct from suspicion), and (Lord Morris of Borth-y-Gest dissenting) his failure to do so amounted to a misdirection, sufficient
to require a new trial, since the ordinary man would not infer guilt of fraud merely from the existence of an inquiry (see p 154,
letter h, p 155, letter f, p 160, letter g, p 168, letter a, and p 174, letter i, to p 175, letter a, post; cf p 164, letter b, post).
Capital and Counties Bank v Henty ((1880), 5 CPD 514) and Nevill v Fine Art and General Insurance Co ([1897] AC 68)
considered.
(ii) on the question of damages
(a) the damages awarded were in any event so excessive that they could not be allowed to stand (see p 155, letter i, p 164,
letter c, p 168, letter i, p 175, letter b, and p 157, letter b, post).
(b) (per Lord Reid; Lord Jenkins, Lord Morris of Borth-y-Gest and Lord Hodson, concurring) where there is more than one
action arising on similar libels, the jury in each action should be directed that, in considering the evidence submitted to them, they
should consider how far the damage suffered by the plaintiffs can reasonably be attributed solely to the libel with which they are
concerned, and how far it ought to be regarded as the joint result of the libels (see p 156, letter c, p 157, letter b, p 164, letter d,
and p 169, letter b, post).
(c) (per Lord Reid; Lord Jenkins, Lord Morris of Borth-y-Gest and Lord Hodson concurring) where a plaintiff proved loss of
income as a result of a libel, then, if the plaintiff were a company, income tax at the standard rate on the loss of profit should be
taken into account, or, if the plaintiff were an individual and proved such loss of income, income tax (including surtax, if
applicable) should be taken into account, and the jury should be directed accordingly (see p 156, letter i, to p 157, letter a, p 157,
letter b, p 164, letter d, and p 169, letter a, post).
British Transport Commission v Gourley ([1955] 3 All ER 796) applied. Decision of the Court of Appeal ([1962] 2 All ER
698) affirmed.

Notes
As to meaning and sufficiency of an innuendo, see 24 Halsburys Laws (3rd Edn) 86, 87 paras 154156; and for cases on the
subject, see 32 Digest 6468, 912963; and as to the duty of a judge in relation to an innuendo, see 24 Halsburys Laws (3rd Edn)
108, para 199.
As to justification of a libel, see 24 Halsburys Laws (3rd Edn) 4348, paras 7585; and for cases on the subject, see 32
Digest 9199, 12191305.
As to when a new trial is granted on the ground of excessive damages in a libel action, see 24 Halsburys Laws (3rd Edn)
121, para 225.

Cases referred to in opinions


Barhams Case (1590), 1 Sav 121, 23 Digest (Repl) 74, 681.
British Transport Commission v Gourley [1955] 3 All ER 796, [1956] AC 185, [1956] 2 WLR 41, 220 LT 354, 3rd Digest Supp.
Capital and Counties Bank v Henty (George) & Sons (1880), 5 CPD 514, CA, affd (1882), 7 App Cas 741, 52 LJQB 232, 47 LT
662, 47 JP 214, HL, 32 Digest 21, 121.
Cookson v Harewood [1931] All ER Rep 533, [1932] 2 KB 478, n, 101 LJKB 328, 145 LT 1, Digest Supp.
152
English and Scottish Co-operative Properties Mortgage and Investment Society Ltd v Odhams Press Ltd [1940] 1 All ER 1,
[1940] 1 KB 440, 109 LJKB 273, 162 LT 82, 2nd Digest Supp.
Grubb v Bristol United Press Ltd [1962] 2 All ER 380, [1963] 1 QB 309, [1962] 3 WLR 25.
Harvey v French (1832), 1 Cr & M 11, 2 Moo & S 591, 1 LJEx 231, 32 Digest 61, 893.
Loughans v Odhams Press Ltd [1962] 1 All ER 404, [1963] 1 QB 299, [1962] 2 WLR 692.
Nevill v Fine Art and General Insurance Co [1897] AC 68, 66 LJQB 195, 75 LT 606, 61 JP 500, 32 Digest 72, 1009.
R v Horne (1777), 2 Cowp 672, 98 ER 1300, 32 Digest 197, 2458.
Sim v Stretch [1936] 2 All ER 1237, Digest Supp.
Simmons v Mitchell (1880), 6 App Cas 156, 50 LJPC 11, 43 LT 710, 45 JP 237, 32 Digest 49, 577.
Stubbs Ltd v Russell [1913] AC 386, 82 LJPC 98, 108 LT 529, 32 Digest 34, 292.
Turner (orse Robertson) v Metro-Goldwyn-Mayer Pictures Ltd [1950] 1 All ER 449, 2nd Digest Supp.
Watkin v Hall (1868), LR 3 QB 396, 9 B & S 279, 37 LJQB 125, 18 LT 561, 32 JP 485, 32 Digest 96, 1272.
Youssoupoff v Metro-Goldwyn-Mayer Pictures Ltd (1934), 50 TLR 581, Digest Supp.

Appeals
This was an appeal from an order of the Court of Appeal (Holroyd Pearce, Davies LJJ and Havers J) dated 4 April 1962, and
reported in [1962] 2 All ER 698, by which the verdicts given and the judgments entered on the trial of two consolidated actions
brought by the appellants separately against the respondents Daily Telegraph Ltd before Salmon J and a jury on 19 July 1961,
were wholly set aside and a new trial was ordered. This appeal was heard together with another appeal by the same appellants
from a like order of the same Court of Appeal, dated 4 April 1962, reported in [1962] 2 All ER 698, by which the verdicts given
and the judgments entered on the trial of consolidated actions brought by the appellants against the respondents, Associated
Newspapers Ltd before Salmon J and a jury on 21 July 1961, were wholly set aside and a new trial was ordered.
The words of the libels and the relevant terms of the statements of claim are set out at p 164, letters g to i, and p 157, letters
f, g and i, post.

H P J Milmo QC and Colin Duncan for the appellants.


Neville Faulks QC and H M Davidson for the respondents, Daily Telegraph Ltd.
Neville Faulks QC and David Hirst for the respondents, Associated Newspapers Ltd.
Their Lordships took time for consideration

26 March 1963. The following opinions were delivered.

LORD REID. My Lords, these are appeals in two actions for libel brought by the appellants, Mr Lewis and a company of which
he is managing director, against the proprietors of the Daily Telegraph and the Daily Mail in respect of paragraphs referring
to them which appeared on the front pages of those newspapers on 23 December 1958. I have had an opportunity of reading the
speeches about to be delivered by my noble and learned friends who deal fully with the facts and I shall not set out the passages
of which complaint is made. On 18 July 1961, a jury awarded damages against the Daily Telegraph of 25,000 to Mr Lewis
and 75,000 to his company. On the next day a different jury awarded against the Daily Mail 17,000 to Mr Lewis and
100,000 to his company. The Court of Appeal ordered new trials on several grounds of which the two most 153 important are
that the trial judge misdirected the juries and that the damages are so excessive that the awards cannot be allowed to stand. On
the matter of misdirection there is no material difference between the two cases.
The essence of the controversy between the parties is that the appellants maintain that these passages are capable of meaning
that they were guilty of fraud. The respondents deny this: they admit that the paragraphs are libellous but maintain that the juries
ought to have been directed that they are not capable of the meaning which the appellants attribute to them. The learned judge
directed the juries in such a way as to leave it open to them to accept the appellants contention and it is obvious from the
amounts of damages awarded that the juries must have done this.
The gist of the two paragraphs is that the police, the City Fraud Squad, were inquiring into the appellants affairs. There is
no doubt that in actions for libel the question is what the words would convey to the ordinary man: it is not one of construction in
the legal sense. The ordinary man does not live in an ivory tower and he is not inhibited by a knowledge of the rules of
construction. So he can and does read between the lines in the light of his general knowledge and experience of worldly affairs.
I leave aside questions of innuendo where the reader has some special knowledge which might lead him to attribute to the words
a meaning not apparent to those who do not have that knowledge. That only arises indirectly in this case. There has been much
argument about innuendoes, true or false, and about proper methods of pleading. My noble and learned friends intend to deal
with those matters and I shall not add to their explanations. I shall only make some observations on the footing that in this case
there is no question of innuendo in the true sense.
What the ordinary man would infer without special knowledge has generally been called the natural and ordinary meaning
of the words. But that expression is rather misleading in that it conceals the fact that there are two elements in it. Sometimes it is
not necessary to go beyond the words themselves as where the plaintiff has been called a thief or a murderer. But more often the
sting is not so much in the words themselves as in what the ordinary man will infer from them and that is also regarded as part of
their natural and ordinary meaning. Here there would be nothing libellous in saying that an inquiry into the appellants affairs
was proceeding: the inquiry might be by a statistician or other expert. The sting is in inferences drawn from the fact that it is the
fraud squad which is making the inquiry. What those inferences should be is ultimately a question for the jury but the trial judge
has an important duty to perform.
Generally the controversy is whether the words are capable of having a libellous meaning at all and undoubtedly it is the
judges duty to rule on that. I shall have to deal later with the test which he must apply. Here the controversy is in a different
form. The respondents admit that their words were libellous, although I am still in some doubt as to what is the admitted
libellous meaning. But they sought and seek a ruling that these words are not capable of having the particular meaning which the
appellants attribute to them. I think that they are entitled to such a ruling and that the test must be the same as that applied in
deciding whether words are capable of having any libellous meaning. I say that because it appears that when a particular
meaning has been pleaded either as a true or a false innuendo it has not been doubted that the judge must rule on the
innuendo. And the case surely cannot be different where a part of the natural and ordinary meaning is, and where it is not,
expressly pleaded. The leading case is Capital and Counties Bank v George Henty & Sons ((1882), 7 App Cas 741 at p 745). In
that case Lord Selborne LC said ((1882), 7 App Cas 741 at p 745):

The test according to the authorities is whether, under the circumstances in which the writing was published,
reasonable men to whom the publication was made would be likely to understand it in a libellous sense.
154

Each of the four noble Lords who formed the majority stated the test, in a different way, and the speeches of Lord Blackburn
((1882), 7 App Cas at p 771) and Lord Watson ((1882), 7 App Cas at p 778) could be read as imposing a heavier burden on the
plaintiff. But I do not think that they should now be so read. In Nevill v Fine Art and General Insurance Co, Lord Halsbury said
([1897] AC 68 at p 72):

What is the sense in which any ordinary reasonable man would understand the words of this communication so as to
expose the plaintiff to hatred or contempt or ridicule.
It is not enough to say that by some person or another the words might be understood in a defamatory sense [[1897] AC
at p 73].

These statements of the law appear to have been generally accepted and I would not attempt to restate the general principle.
In this case it is, I think, sufficient to put the test in this way. Ordinary men and women have different temperaments and
outlooks. Some are unusually suspicious and some are unusually nave. One must try to envisage people between these two
extremes and see what is the most damaging meaning that they would put on the words in question. So let me suppose a number
of ordinary people discussing one of these paragraphs which they had read in the newspaper. No doubt one of them might say
Oh, if the fraud squad are after these people you can take it they are guilty. But I would expect the others to turn on him, if
he did say that, with such remarks asBe fair. This is not a police state. No doubt their affairs are in a mess or the police
would not be interested. But that could be because Lewis or the cashier has been very stupid or careless. We really must not
jump to conclusions. The police are fair and know their job and we shall know soon enough if there is anything in it. Wait till we
see if they charge him. I wouldnt trust him until this is cleared up, but it is another thing to condemn him unheard.
What the ordinary man, not avid for scandal, would read into the words complained of must be a matter of impression. I can
only say that I do not think that he would infer guilt of fraud merely because an inquiry is on foot. And if that is so then it is the
duty of the trial judge to direct the jury that it is for them to determine the meaning of the paragraph but that they must not hold it
to impute guilt of fraud because as a matter of law the paragraph is not capable of having that meaning. So there was here, in my
opinion, misdirection of the two juries sufficiently serious to require that there must be new trials.
Before leaving this part of the case I must notice an argument to the effect that you can only justify a libel that the plaintiffs
have so conducted their affairs as to give rise to suspicion of fraud, or as to give rise to an inquiry whether there has been fraud,
by proving that they have acted fraudulently. Then it is said that, if that is so, there can be no difference between an allegation of
suspicious conduct and an allegation of guilt. To my mind there is a great difference between saying that a man has behaved in a
suspicious manner and saying that he is guilty of an offence and I am not convinced that you can only justify the former
statement by proving guilt. I can well understand that if you say there is a rumour that X is guilty you can only justify by proving
that he is guilty because repeating someone elses libellous statement is just as bad as making the statement directly. But I do not
think that it is necessary to reach a decision on this matter of justification in order to decide that these paragraphs can mean
suspicion but cannot be held to infer guilt.
Even if the paragraphs were capable of meaning that the appellants were guilty of fraud I would think that the damages
awarded were far too high, and a fortiori the awards could not stand if the most that could be read into the words is that they had
conducted their affairs in such a way as to give rise to suspicion or to justify a police inquiry. I do not say that these amounts of
damages could never 155 be justified but at least there would have to be evidence of a very different kind from that adduced in
these cases. I do not intend to analyse the evidence already given because that might hamper the conduct of the new trials with
regard to both the plaintiffs case and the defendants pleas of justification. But two particular matters raised in argument will
probably arise at the new trials and they require some clarification.
Here there were similar libels published in two national newspapers on the same day and each has to be dealt with by a
different jury. If each jury were to award damages without regard to the fact that the plaintiffs are also entitled to damages
against the other newspaper, the aggregate of the damages in the two actions would almost certainly be too large. Section 12 of
the Defamation Act, 1952, is intended to deal with that. In effect it requires that each jury shall be told about the other action, but
the question is what each jury should be told. I do not think that it is sufficient merely to tell each jury to make such allowance as
they may think fit. They ought, in my view, to be directed that in considering the evidence submitted to them they should
consider how far the damage suffered by the plaintiffs can reasonably be attributed solely to the libel with which they are
concerned and how far it ought to be regarded as the joint result of the two libels. If they think that some part of the damage is
the joint result of the two libels they should bear in mind that the plaintiffs ought not to be compensated twice for the same loss.
They can only deal with this matter on very broad lines and they must take it that the other jury will be given a similar direction.
They must do the best they can to ensure that the sum which they award will fully compensate the plaintiffs for the damage
caused by the libel with which they are concerned but will not take into account that part of the total damage suffered by the
plaintiffs which ought to enter into the other jurys assessment.
The other question arises out of the decision of this House in British Transport Commission v Gourley which deals with
damages for loss of income caused by a tort. In that case Mr Gourley had been seriously injured in a railway accident. He had
been earning a large income and it was found that his loss of income due to his injuries was 37,720. But this was the gross
income which he would have received but for his injuries. Out of it he would have had to pay income tax and surtax. And it was
found that after paying tax he would only have retained 6,695. So his real loss was only 6,695 because he could never have
derived any advantage from the balance which he would have had to pay away in tax. As damages are not subject to tax he
would have recovered far more than his real loss if he had recovered the gross amount of 37,720 and accordingly it was held
that he was only entitled to receive 6,695 in respect of his loss of income as this was sufficient to compensate him fully for the
income which he had lost by the fault of the defendants.
There can be no difference in principle between loss of income caused by negligence and loss of income caused by a libel.
Let me take first the case of the plaintiff company. A company cannot be injured in its feelings, it can only be injured in its
pocket. Its reputation can be injured by a libel, but that injury must sound in money. The injury need not necessarily be confined
to loss of income. Its goodwill may be injured. But in so far as the company establishes that the libel has, or has probably,
diminished its profits, I think that Gourleys case is relevant. Damages for libel have to be assessed by a jury, and juries are not
expected to make mathematical calculations, so they can only deal with this matter on broad lines. I think that a jury ought to be
directed to the effect that if they think that the plaintiff company has proved that it has suffered or will suffer loss of profit as a
result of the libel they must bear in mind that the company would have had to pay income tax at the standard rate out of that
profit if it had been earned and would only have been entitled to keep the balance. So in assessing damages they ought not to
take into account the whole of that 156 profit but should make allowance for the obligation to pay income tax out of it. The
position with regard to an individual plaintiff is rather different. He may be entitled to very substantial damages although his
income has not been affected by the libel. But if he does attempt to prove loss of income as a result of the libel then I think that a
similar direction must be given to the jury and it may be necessary to mention surtax as well as income tax.
Accordingly I shall move in each case that the appeal should be dismissed, that a new trial be ordered, that costs of the
abortive trial should abide the result of the new trial, and that the appellants be ordered to pay the respondents costs in the Court
of Appeal and in this House.
My noble and learned friend LORD JENKINS is unable to be present this morning and he desires me to say that he
concurs.

LORD MORRIS OF BORTH-Y-GEST. My Lords, on 23 December 1958, on the front page of the Daily Telegraph there
appeared the following words:

INQUIRY ON FIRM BY CITY POLICE

Daily Telegraph Reporter


Officers of the City of London Fraud Squad are inquiring into the affairs of Rubber Improvement, Ltd. and its
subsidiary companies. The investigation was requested after criticisms of the Chairmans statement and the accounts by a
shareholder at the recent company meeting.
The Chairman of the Company, which has an authorised capital of 1 million, is Mr. John Lewis, a former Socialist
M.P. for Bolton.

The company and Mr Lewis issued writs on that date and claimed damages for libel. The two actions were later consolidated.
Included in the statement of claim of the company, in addition to a paragraph pleading that the words had been falsely and
maliciously printed and published, was a paragraph which I consider, having regard to the fact that particulars under RSC, Ord
19, r 6(2) were set out, must be regarded as pleading an innuendo using that word in the strict or legal sense. The paragraph was
as follows:

By the said words the defendants meant and were understood to mean that the affairs of the plaintiffs and/or its
subsidiaries were conducted fraudulently or dishonestly or in such a way that the police suspected that their affairs were so
conducted.

Particulars pursuant to R.S.C., Ord. 19, r. 6(2)


(1) The plaintiffs repeat para. 1 hereof.
(2) The plaintiffs will rely on the tone and heading of the said article.
(3) It is generally known that the City Fraud Squad investigate serious cases of company fraud.

Paragraph 1 of the statement of claim had stated that the company was a public company carrying on a large and extensive
business both on its own account and through subsidiaries mainly in plastics and rubber industries. The provisions of RSC, Ord
19, r 6(2) require a plaintiff who alleges that words have been used in a defamatory sense other than their ordinary meaning to
give particulars of the facts and matters on which he relies in support of that sense.
In the statement of claim of the personal plaintiff there was a comparable paragraph. Apart from its supporting particulars it
was as follows:

By the said words the defendants meant and were understood to mean that the plaintiff had been guilty or was
suspected by the police of having been guilty of fraud or dishonesty in connexion with the affairs of the said company
and/or its subsidiaries and/or that he had caused or permitted the affairs of the said company and/or its subsidiaries to be
conducted fraudulently or dishonestly or in such a way that the police suspected that the affairs of the said company and/or
its subsidiaries had been so conducted and/or that the plaintiff was unfit to hold either of his said offices.
157

In the defences of the Daily Telegraph there were two paragraphs (in addition to alternative paragraphs containing pleas in
mitigation of damages) of special importance. In one there was a plea that the words in their natural and ordinary meaning
were true in substance and in fact. In the other there was a plea that the words do not bear and were not understood to bear and
were incapable of bearing the meanings attributed to them in the paragraphs in the statements of claim which I have set out. It
was admitted that the personal plaintiff was the chairman and managing director of the company and that the company carried on
a large and extensive business. The plaintiffs made a request for particulars of the justification which had been pleaded. This
matter was dealt with before the master in chambers, then on appeal to the judge in chambers and then on appeal in the Court of
Appeal. The master made the limited order that particulars should be given of the request for an investigation stating by whom
such request was made and when and the nature of the investigation requested. The learned judge allowed an appeal from that
order which he varied by ordering particulars of the facts and matters relied on in support of the allegation that the words in their
natural and ordinary meaning were true in substance and in fact. It is agreed that in the Court of Appeal learned counsel for the
Daily Telegraph made it clear that justification was only being asserted in regard to the limited fact that officers of the City of
London Fraud Squad had (after being requested as stated) inquired into the affairs of the company and its subsidiary companies.
On that being made clear the Court of Appeal set aside the order of the learned judge. Accordingly the limited order made by the
master was restored.
Before your Lordships it was common ground that the fact that certain meanings were alleged by way of innuendo did not
debar the plaintiffs from contending that such meanings were in fact the direct or ordinary or primary meanings of the words.
After the evidence was concluded at the trial a submission was made in the absence of the jury that the innuendoes should in
each case be withdrawn from the jury. There is no note as to the terms of the submission that was made, but it was the
recollection of learned counsel who made it that it was not limited to a submission that the two paragraphs should be withdrawn
because the particulars did not support the innuendo or were not proved, but that it extended to a submission that apart from any
question of an innuendo the words themselves in their ordinary meaning were incapable of bearing the meanings set out by the
plaintiffs in those paragraphs. The conclusion of the matter can best be recorded by setting out what the learned judge actually
said and what learned counsel said:

SALMON, J. [See [1962] 2 All ER 698 at 700]: I am inclined to think that no innuendo here was necessary. I can
well understand, however, that where there is any doubt about the matter the learned pleader very properly puts in an
innuendo. As counsel for the defendants candidly admits, that cannot do the defendants any harm, because it forewarns
them as to what the plaintiff is going to submit the words mean. Where there is no innuendo, it may in some cases be
difficult for the defendant to know. Even although the innuendo may strictly be unnecessary, I do not think that in the
exercise of my discretion I ought to strike it out; nor need it be amended. I can only say that the practice was at one time
always to plead an innuendo. The practice has altered, fortunately, so that innuendoes now are rarely pleaded, but there
may be casesand I agree with Mr. Duncan that this is one of themwhere there may be a doubt as to whether it is
necessary to plead an innuendo. Where there is such a doubt, there can be no harm in pleading it. It certainly cannot hurt
the defendants. I do not accept the argument that by pleading an innuendo you are necessarily alleging affirmatively that
the words in their ordinary and natural meaning mean something other than that which is 158 pleaded in the innuendo. I do
not propose to make any order on the application of the defendants.

The learned judge then said, addressing counsel for the defendants and counsel for the plaintiffs:

I propose only to leave one question to the jury: do you find for the plaintiff or for the defendant, and, if for the
plaintiff, how much? I will hear either of you if you wish to urge me to leave a series of questions to the jury.
Counsel for the defendants: No, my Lord, I am quite in accord with what your Lordship says.
Counsel for the plaintiffs: My Lord, so am I.

It is clearly settled that an innuendo constitutes a cause of action separate from the libel itself and in respect of which a separate
verdict should be returned and separate damages (if to be awarded) should be assessed; (see Sim v Stretch and Watkin v Hall).
Unless the court otherwise permits, any payment into court referable to an innuendo must be a separate payment.
The words of the learned judge show that he fully appreciated the difficulty that faced the pleader and that he considered that
the pleading of innuendoes had really been unnecessary. That was because the innuendoes did not go beyond the meanings that
the plaintiffs said were conveyed by the words of the libel. The effect of what the learned judge did was that the case proceeded
on the footing that the paragraphs should be treated as being no more than paragraphs which recorded what the plaintiffs
submitted were the ordinary meanings of the words. The paragraphs were however in form and must be regarded as being in fact
paragraphs which pleaded innuendoes. That being so if the learned judge took the view that no extrinsic facts were proved which
could support an innuendo he should, I think, have said in direct terms that he was not leaving any innuendo in its true sense to
the jury and instead of refusing the application should have, at least to some extent, acceded to it. If he was prepared to allow the
paragraphs to remain in some form he should perhaps have required that they be amended so that they were no longer paragraphs
which pleaded innuendoes. But the case went on just as though he had done that. The paragraphs were treated as though they
did not contain innuendoes in a true sense. They were regarded as being of the style of paragraphs which, in pleadings before the
introduction of RSC, Ord 19, r 6(2), used the word innuendo in a more general way and not in its strict or technical sense. So
no harm to the respondents resulted from the ruling of the learned judge. The case continued in spite of the actual language of his
ruling, just as the respondents suggested that it should proceed. No innuendo (using that word in its strict sense) was in fact left
to the jury. The summing-up directed the jury to consider what the words themselves meant and conveyed. Had the learned
judge left an innuendo to the jury, he would have had to require the jury to deal with it as a separate issue. His words show that
he regarded the paragraphs as harmless paragraphs, which had not hurt the defendants, but had perhaps helped them by
forewarning them as to what the plaintiffs said that the words meant. He did, however, consider that the words complained of
were of and by themselves capable of bearing the meanings alleged by the plaintiffs, and he left it to the jury to say what they
thought that the words meant. Here I think (apart from the issues concerning the damages) is the real issue in the case. Was the
learned judge right in deciding (as he implicitly did) that the words were capable of bearing all the alleged meanings? Once the
learned judge had reached that conclusion then any question as to innuendoes seems to me to have dropped out of the case.
Where a plaintiff brings an action for libel he may sustain his case (where there is a trial with a jury) if the judge rules that
the words, in what has been called their natural and ordinary meaning (or their ordinary meaningsee RSC, 159Ord 19, r
6(2)) are capable of being defamatory and if the jury find that they are defamatory. A plaintiff may, however, sustain his case in a
different way. He may plead an innuendo. He may establish that because there were extrinsic facts which were known to readers
of the words, such readers would be reasonably induced to understand the words in a defamatory sense which went beyond or
which altered their natural and ordinary meaning and which could be regarded as a secondary or as an extended meaning. The
nature of an innuendo (using that word in its correct legal sense) has recently been reviewed in the valuable judgments delivered
in the Court of Appeal in Grubb v Bristol United Press Ltd. A defamatory meaning which derives no support from extrinsic facts,
but which is said to be implied from the words which are used, is not a true innuendo. If there are some special extrinsic facts the
result may be that to those who know them words may convey a meaning which the words taken by themselves do not convey.
In the present case I am disposed to agree with the Court of Appeal that no extrinsic facts were proved which yielded the
necessary support to sustain an innuendo. This, however, became a matter of no consequence in the case for if the meanings
alleged in the pleaded innuendo were no more than the meanings expressed or conveyed by or to be implied from the words
themselves, then there was no need to plead innuendoes. It was not really being alleged that the words were used in a defamatory
sense other than their ordinary meaning. Though the two paras 4 were undoubtedly pleaded as innuendoes, once it was clear that
the contention of the plaintiffs was that the words themselves would be understood by ordinary readers to be conveying and
expressing the meanings recorded in those paragraphs, then the case for the plaintiffs was direct and straightforward and was not
in any way advanced or assisted by any mention of an innuendo. Though the paragraphs were not struck out the only
significance of their remaining was that they usefully contained and recorded the ordinary meanings which the plaintiffs said
were conveyed by the words printed in the newspaper. It followed that it was quite unnecessary for the learned judge to tell the
jury what an innuendo was or even to use the word (and he did not) or to leave any issue or question to them concerning an
innuendo. The question left to the jurywithin the limits of the meanings which the learned judge regarded the words as being
capable of bearingwas what they thought ordinary people would consider that the words meant.
In a case where there is no innuendo pleaded, it is not essential for a plaintiff to record and define in his pleadings what he
says are the ordinary or direct or natural or implied meanings of the words. If, however, he does do so (as may often be helpful
provided it is made clear what is being done) and, if the judge considers that the words are not capable of bearing any one or
more of such meanings, he ought so to rule. If the plaintiff does not do so the various meanings suggested by the plaintiff will
almost invariably be canvassed during the trial and if the judge considers that the words are not capable of bearing any one or
more of them again he ought so to rule.
It is of some importance to consider how the issues in the case rested. Publication of the words complained of was admitted.
The separate plaintiffs (the company and Mr Lewis) claimed and the defendants denied that in their natural and ordinary meaning
the words meant what was set out in the two paragraphs. It was not denied by the defendants and it was therefore tacitly admitted
that the words in their natural and ordinary meaning were defamatory of the plaintiffs but the defendants contention was that in
their natural and ordinary meanings the words only meant that there was an inquiry by the City of London Fraud Squad. The
defendants plea of justification was accordingly and for that reason limited to that meaning which was the only defamatory
meaning that they said that the words bore. They said that there had been an inquiry. The plaintiffs said that there had not been
anything that could be called an inquiry or that the 160 defendants had not proved that there had been such an inquiry. The
defendants pleaded in mitigation of damages that on the 24 December 1958 they published a statement by Mr Lewis expressing
his view of the facts and they also pleaded that the plaintiffs had claimed damages for the same or a similar libel from Associated
Newspapers Ltd. If the learned judge was correct in holding that the words were capable of bearing the meanings that the affairs
of the company or its subsidiaries were conducted fraudulently or dishonestly or that the personal plaintiff had been guilty of
fraud or dishonesty in connexion with the affairs of the company or its subsidiaries, then I see no grounds for criticism of his
summing-up on the issues of liability. He invited the jury to decide what they thought ordinary reasonable people would consider
the words to mean. Having regard to the guidance given by Lord Selborne LC in his speech in Capital and Counties Bank v
George Henty & Sons that was, I think, an entirely correct approach. Lord Selborne said ((1882), 7 App Cas at p 745):

The test according to the authorities is whether under the circumstances in which the writing was published reasonable
men to whom the publication was made would be likely to understand it in a libellous sense.

See also the words of Lord Blackburn ((1882), 7 App Cas at p 772).
My lords, words are but the instruments which men use to express and convey their meanings. The learned judge asked the
jury to say what meanings the words in question would convey, not to people with some special or particular knowledge, but just
to ordinary men and women going about their ordinary affairs. It is in this sense that in defamation cases the phrase natural and
ordinary meaning (which may include an implied or indirect meaning) is used. Not resting on any technical process of analysis
or construction, nor on a process of critical reading, the inquiry is as to what meanings are conveyed to hearers or readers by the
medium of words. This is a matter for the jury though a jury must not be asked to consider a meaning which the words in
question are not reasonably capable of bearing.
It was said in the Court of Appeal that the learned judge had failed to remember that the defendants were admitting that the
words, in what they said was their only natural meaning, were defamatory. I do not think that there is any substance in this for
the learned judge asked the jury whether, if the words bore the very limited meaning contended for by the newspaper, they
considered that the words were justifiable as being true: the necessity to consider the defence of justification would only arise on
the basis that the words were defamatory and there is no reason to think that the judge was either under a misapprehension or that
he need have said more to the jury than he did. He put very fairly before the jury the rival contentions as to what the words
meant. We do not know exactly what the jury decided that the words meant because with the assent of both sides only the one
question set out above was left to them. As it is important to see how the matter was put to the jury, I venture to quote the words
of the learned judge (See [1962] 2 All ER 698 at p 702):

This case very largely depends on what in your view those words mean. The question is, what would they have meant
to the ordinary man and woman when he or she read them on the morning of Dec. 23? It has been said that the ordinary
man, with his cup of tea in one hand, reading this paper, does not read it with a suspicious tortuous and sinister mind. That
is very true, you may think.
On the other hand, it has been said that when the ordinary man spreads his paper out on the table and reads it with his
cup of tea in one hand, he does not necessarily hold the scales of justice in delicate equipoise in the other. You have got to
think of the ordinary man. How would the ordinary man understand this?
161
The two views which have been canvassed before you are these: [Counsel for the defendants] has said: Well, the
ordinary man is not very suspicious; he would just regard it as a piece of intelligence, the police are looking into it, and it
would not really produce any other effect on his mind. [Counsel for the plaintiffs] says: Well, the ordinary man seeing this
City Police. Officers of the City of London Fraud Squad are inquiring into the affairs of Rubber Improvement, Ltd.the
ordinary man, not being any more suspicious than his neighbour, would immediately say to himself, says [counsel for the
plaintiffs]either he would say to himself: There is a fraud here, or the police would not be looking into it; or, he would
say to himself: At any rate, there is enough in this for the police to suspect that there is fraud.
I cannot really help you. Those are the two rival contentions. It is for you to say what it means. When you read the
newspapers, what would you have thought when you read that? You see, the only way you can get at what the ordinary
man and woman think is by getting a jury of twelve people together, who are ordinary men and women, and asking them
what they would have thought. You may ask yourselves, what would people in the City think if they woke up one morning
and read that in the paper?
Members of the jury, anything is defamatory which tends to lower you in the esteem of right thinking people. But if
anyone reading this thoughtany ordinary reasonable man reading this thoughtthat it meant that Mr. Lewis had been
guilty of fraud, or that the police suspected that Mr. Lewis had been guilty of fraud; or that he had allowed the affairs of the
company to be conducted fraudulently or dishonestly; or the police suspected that he had, would that tend to lower him in
the esteem of right thinking people?
And as far as the company is concerned, it is suggested by the plaintiffs that these words mean to the ordinary man that
the affairs of the company and its subsidiaries were conducted fraudulently or dishonestly; or that the police suspected that
they were so conducted. That is what is said by the plaintiffs these words would convey to the ordinary man and woman,
and that the ordinary man and woman would not merely say to themselves: Oh well, it is a very interesting piece of
intelligence: the police are inquiring into it. There may be a routine examination. We do not draw any conclusions at all.
As I say, consider that you get up one morning in a perfectly reasonable frame of mind; you are not feeling suspicious
particularly, but you have a look at that: what would it mean to you?

The learned judge then went on to deal with the issue of justification and to direct the jury that if they thought that the meaning
conveyed was no more than that the police were making an inquiry, then they had to consider whether the defendants had proved
that an inquiry had been made. The learned judge reminded the jury of the evidence and said:Does that constitute an inquiry
into these matters by the police? You may think it does: you may think it does not.
My lords, I turn to consider the question whether the words were capable of bearing the meaning that the affairs of the
company and/or its subsidiaries were conducted fraudulently or dishonestly. I do not understand any of your lordships to be of
the view that the words were not capable of bearing the meaning that the police suspected that the affairs of the company or its
subsidiaries were conducted fraudulently or dishonestly: nor did I understand any submission to be made that the words were not
so capable. It is a grave thing to say that someone is fraudulent. It is a different thing to say that someone is suspected of being
fraudulent. How much less wounding and damaging this would be must be a matter of opinion depending on the circumstances.
Similarly in the case of the personal plaintiff the submission is made that the words while capable of bearing 162 some of the
alleged meanings were not capable of bearing the meanings that Mr Lewis had been guilty of fraud or dishonesty in connexion
with the affairs of the company or its subsidiaries or had caused or permitted the affairs to be conducted fraudulently or
dishonestly.
My lords, the only question that now arises is not whether the words did bear, but whether they were capable of bearing, the
meanings to which I have referred. What could ordinary reasonable readers think? Some I consider might reasonably take the
view that there was just an inquiry to find out whether or not there had been any fraud or dishonesty. Some I consider might
reasonably take the view that the words meant that there was an inquiry because there had been fraud or dishonesty which
occasioned or required inquiry by the police. Some I consider might reasonably take the view that the words meant that the
inquiry was either (a) because there had been fraud or dishonesty or (b) because of a suspicion that there had been.
My lords, it is not for me to say what I think was the meaning which the words conveyed to the ordinary reasonable reader
of a newspaper nor is it for me to express any opinion as to what conclusion a jury should reach as to this matter but I do not
consider that that meaning which involved that there had been fraud or dishonesty was a meaning which the jury should have
been prohibited from considering on the basis that it was a meaning of which the words were not capable. I do not think that it
can be said that twelve jurors could not reasonably have come to the conclusion that the words bore the meaning now being
considered. In using this language I am following the approach suggested by Lord Porter in his speech in Turner v Metro-
Goldwyn-Mayer Pictures Ltd ([1950] 1 All ER 449 at pp 452, 453). See also Nevill v Fine Art and General Insurance Co.
My lords, a reasonable reader will probably be a fair-minded reader. The fair-minded reader would assume that a
responsible newspaper would also be fair. If there was some private police inquiry in progress, the purpose of which was to
ascertain whether or not there had been fraud or dishonesty what possible justification could there be for proclaiming this far and
wide to all the readers of a newspaper? If confidential information was received to the effect that there was a police inquiry on
what basis could the publishing of such information be warranted? There is no suggestion that the police had asked that any
notice should be published. Under certain circumstances a newspaper may enjoy qualified privilege if it publishes a notice issued
for the information of the public by or on behalf of a chief officer of police (see Defamation Act, 1952, s 7). If there was a police
inquiry by a Fraud Squad which might result in the conclusion that any suspicion of fraud or dishonesty was wholly
unwarranted how manifestly unfair it would be to make public mention of the inquiry. What purpose could there be in doing so?
With these thoughts and questions in his mind, a reasonable reader might well consider that no responsible newspaper would dare
to publish, or would be so cruel as to publish, the words in question unless the confidential information, which in some manner
they had obtained, was not information merely to the effect that there was some kind of inquiry in progress but was information
to the effect that there was fraud or dishonesty. Some reasonable readers might therefore think that the words conveyed the
meaning that there must have been fraud or dishonesty. Furthermore, a reasonable reader might reflect that while the police may
be concerned with inquiries as to whether some crime has or has not been committed, they are probably more often only
concerned after a crime has been committed. They have to inquire whether they possess the necessary evidence for the launching
of a prosecution. Reasonable readers might also think that inquiries into the affairs of a company if such inquiries were not
concerned with fraud or dishonesty would not be conducted by the police at all. They would be conducted by persons 163 or
departments having no connexion with the City of London Police Fraud Squad. Some of such readers might therefore be led to
believe that if there was an inquiry by the City of London Fraud Squad, which a newspaper felt justified in mentioning, it must
have been an inquiry to collect and marshall evidence in order to launch a prosecution for some offences involving fraud or
dishonesty which had been committed.
My lords, it was for the jury to determine what they considered was the meaning that the words would convey to ordinary
men and women: we have only to decide as to the limits of the range of meanings of which the words were capable. For the
reasons that I have given I have the misfortune to differ from your lordships as to this very important part of the case. I consider
that the learned judge was fully entitled to leave the matter to the jury in the way in which he did and I consider that his directions
concerning liability were clear and correct and fair.
My lords, in the consolidated action against the Daily Mail a similar issue arises to that which I have been discussing.
On the difficult issue as to damages I do not differ from your lordships or from the Court of Appeal that the awards of
damages were excessive and cannot stand. This issue was fully debated and the relevant evidence was carefully examined. As
there must be new trials I do not think that there is need to say more in regard to this matter. I would wish to add that having had
the privilege of reading in advance the speech which has been delivered by my noble and learned friend, Lord Reid, I am in
agreement with his observations in regard to s 12 of the Defamation Ac,t 1952, and in regard to the extent of any relevance of the
case of British Transport Commission v Gourley.
Because I do not dissent on the issue as to damages I agree that there must be new trials and that therefore the appeals
should be dismissed.

LORD HODSON. My Lords, In these actions large damages were awarded to the plaintiff Mr Lewis and to the company
Rubber Improvement Ltd of which the first plaintiff is the managing director. In the first pair of actions, which were consolidated
with one another, Daily Telegraph Ltd was the defendant, in the second pair, likewise consolidated, Associated Newspapers Ltd
proprietors of the Daily Mail, were defendants. The pairs of actions were tried separately no step being taken to have them
consolidated although the language of the libels is similar and each defendant is a newspaper having a wide national circulation.
The words complained of in the first actions are as follows:

INQUIRY ON FIRM BY CITY POLICE

Daily Telegraph Reporter.


Officers of the City of London Fraud Squad are inquiring into the affairs of Rubber Improvement, Ltd. and its
subsidiary companies. The investigation was requested after criticisms of the chairmans statement and their accounts by a
shareholder at the recent company meeting.
The chairman of the company, which has an authorised capital of 1 million, is Mr. John Lewis, former Socialist M.P.
for Bolton.

In the second action the words were

FRAUD SQUAD PROBE FIRM


The City Fraud Squad, under Superintendent Francis Lea, are inquiring into the affairs of Rubber Improvement, Ltd.
Chairman of the 4,000,000 group, whose shares have dropped from 22s. last year to 7s. 4 1/2d. yesterday, is Mr. John
Lewis, former Socialist M.P.
The company specialises in flexible rubber conveyor belting designed for the National Coal Board.

Both articles appeared on the front page of the respective newspapers on 23 December 1958.
164
The facts leading up to and surrounding the publication differed to some extent in respect of the two publications but the
general effect was the same. In the Daily Telegraph actions the plaintiff Lewis obtained 25,000 damages and the plaintiff
company 75,000 and in the Daily Mail actions he obtained 17,000 and the company 100,000. The Court of Appeal ordered
a new trial holding that in any event the damages were so excessive that no reasonable jury could have awarded so large a figure
and that there was a misdirection on the part of the trial judge in respect of the meaning of the libels.
The defendants did not deny that the words complained of were defamatory of the plaintiffs. They justified the words as
true in their natural and ordinary meaning and denied that they bore any of the meanings which they were said to bear by
innuendo in effect that the plaintiffs were guilty or suspected by the police of fraud or dishonesty in connexion with the affairs of
the company or its subsidiaries.
The appellants recognise that in awarding such large damages on each trial the juries must have taken the view that the
words of which they complain meant that they had been actually guilty of fraud, a meaning which the defendants have
throughout disclaimed.
No one doubts that it is for the jury to decide the meaning of words, not as a question of pure construction but as a question
of fact, as Lord Tenterden CJ put it in Harvey v French ((1832), 1 Cr & M 11 at p 18),

a court must read these words in the sense in which ordinary persons or in which we ourselves out of court would
understand them.

Whether the words are capable of defamatory meaning is for the judge, and where the words, whether on the face of them they
are or are not innocent in themselves, bear a defamatory or more defamatory meaning because of extraneous facts known to those
to whom the libel has been published, it is the duty of the judge to rule whether there is evidence of such extraneous facts fit to be
left to the jury.
It is in conjunction with secondary meanings that much of the difficulty surrounding the law of libel exists. These secondary
meanings are covered by the word innuendo which signifies pointing out what and who is meant by the words complained of.
Who is meant raises no problem here but what is meant is of necessity divided into two parts much discussed in this case. Libels
are of infinite variety and the literal meaning of the words even of such simple phrases as X is a thief does not carry one very
far for they may have been spoken in play or other circumstances showing that they could not be taken by reasonable persons as
imputing an accusation of theft. Conversely to say that a man is a good advertiser only becomes capable of a defamatory
meaning if coupled with proof, for example, that he was a professional man whose reputation would suffer if such were believed
of him.
The first subdivision of the innuendo has lately been called the false innuendo as it is no more than an elaboration or
embroidering of the words used without proof of extraneous facts. The true innuendo is that which depends on extraneous facts
which the plaintiff has to prove in order to give the words the secondary meaning of which he complains.
The classic example is to be found in Barhams case referred to by De Grey LCJ, in R v Horne ((1777), 2 Cowp 672 at p
684) in the following passage relating to a charge of criminal libel:

But as an innuendo is only used as a word of explanation it cannot extend the sense of the expressions in the libel
beyond their own meaning unless something is put upon the record for it to explain. As in actions upon the case against a
man for saying of another: He has burnt my barn. [Barhams case] the plaintiff cannot there by way of innuendo say 165
meaning his barn full of corn, because that is not an explanation of what was said before but an addition to it. But if in
the introduction it had been averred that the defendant had a barn full of corn and that in a discussion about the barn the
defendant had spoken the words charged in the libel of the plaintiff; an innuendo of its being the barn full of corn would
have been good: for by coupling the innuendo in the libel with the introductory averment, his barn full of corn, would
have made it complete.

The innuendo in this case was set out in the statement of claim as a separate paragraph and was or purported to be a true
innuendo, for the plaintiff gave particulars pursuant to RSC, Ord 19, r 6(2), of the facts and matters which he relied on in support
of a sense other than the ordinary meaning. It is plain on the authorities that, since the Common Law Procedure Act, 1852 a,
which did away with the necessity for a prefatory averment showing the sense in which words were used and enacted that where
the words or matter set forth with or without the alleged meaning show a cause of action the declaration shall be sufficient the
true innuendo has been treated as a separate cause of action from that which arose from the words in their natural and ordinary
meaning (with or without inferential meanings commonly called false innuendoes): see per Blackburn J, in Watkin v Hall
((1868), LR 3 QB 396 at p 402) and per Lord Atkin in Sim v Stretch ([1936] 2 All ER 1237 at p 1239).
________________________________________
a See ibid, s 61; 18 Halsburys Statutes (2nd Edn) 388

After the passing of the Common Law Procedure Act, 1852, until the year 1949, when RSC, Ord 19, r 6(2) came into force,
there was in many cases no distinction to be found between the true and the false innuendo. No special facts had to be pleaded to
support the innuendo and the distinction became blurred between the true innuendo and that which was very often nothing but a
wordy explanation or attempted explanation of the words complained of in their natural and ordinary meaning. This blurring is
manifest in the pleadings of the plaintiff here, for it contains the plaintiffs contention as to the natural and ordinary meaning of
the words complained of, that is to say, that they mean that he and his company were fraudulent or suspected of fraud, and it is at
the same time supported by particulars given under RSC, Ord 19, r 6(2) as of a true innuendo. As the Court of Appeal found, and
I have no doubt that they were right, the particulars did not show, nor was any evidence given of, extraneous facts in support of
the innuendo, and accordingly the innuendo should not have been left to the jury. Although the innuendo was not pleaded in the
alternative, yet as the Court of Appeal held, in my opinion quite rightly, this did not prevent the plaintiffs seeking to show, if they
could, that the natural and ordinary meaning of the words complained of was to the same effect. To hold otherwise and not to
permit the jury to impute to the ordinary meaning of the words any part of the failed innuendo would, as Holroyd Pearce LJ
([1962] 2 All ER 698 at pp 709, 710; [1963] 1 QB 340 at p 368) pointed out, have the effect of removing the jurys decision on
whether the words are in their ordinary sense a libel into an unreal technical and artificial sphere.
I agree with the observations of Holroyd Pearce LJ in Grubb v Bristol United Press Ltd ([1962] 2 All ER 380 at p 389;
[1963] 1 QB 309 at p 326) to the effect that RSC, Ord 19, r 6(2) makes no alteration in the law except in cases where a true
innuendo is pleaded. A pleader is entitled to allege in his statement of claim what the words in their natural and ordinary meaning
convey, provided he makes it clear that he is not relying on a true innuendo, which gives a separate cause of action and requires a
separate verdict from the jury. It is desirable that he should do so, for, where there is no true innuendo, the judge should define
the limits of the natural and ordinary meaning of the libel and leave to the jury only those meanings which he rules are capable of
being defamatory. If the natural and ordinary meaning is pleaded the defence will know what the contentions of the plaintiff are,
and the judge will not have to analyse the submissions of counsel in his charge to the jury 166 without having the benefit of a
pleading setting out what those submissions are.
There is no conflict in my opinion between the decisions of the Court of Appeal in Grubb v Bristol United Press Ltd and in
Loughans v Odhams Press Ltd when properly understood, as indeed was pointed out by Upjohn LJ ([1962] 2 All ER at p 393;
[1963] 1 QB at p 331) in the former case. The difficulty arises from some words (perhaps unguarded) used by Diplock LJ in
Loughans case ([1962] 1 All ER at p 409; [1963] 1 QB at 306) and cited by Davies LJ in Grubbs case ([1962] 2 All ER at p 396;
[1963] 1 QB at p 338), to the effect that the plaintiff may require a verdict from a jury whether the words bear a special
defamatory meaning, even though he had led no evidence of facts and matters on which he relies other than the words themselves
as giving rise to the meaning alleged in the innuendo. This, I agree with Davies LJ, he cannot do. There is one cause of action
based on the words in their natural and ordinary meaning and another based on the words in such meaning as may be alleged in a
true innuendo but not a third cause of action based on the false innuendo.
The defendants, having admitted that the words are defamatory in their ordinary meaning, have always maintained that their
ordinary meaning does not go so far as to include actual guilt of fraud. They have sought to justify by proving that an inquiry
was in fact held, not by proving actual suspicion of fraud. This is the gist of the whole case. Salmon J, who tried both pairs of
actions, took the view that the words were capable of imputing guilt of fraud. Davies LJ ([1962] 2 All ER at p 725; [1963] 1 QB
at p 394) was inclined to the same opinion, and my noble and learned friend Lord Morris of Borth-y-Gest has expressed the same
opinion as Salmon J, Holroyd Pearce LJ and Havers J took the contrary view. In view of this difference of judicial opinion one
naturally hesitates before expressing a concluded opinion of ones own, but after listening to many days of argument I am myself
satisfied that the words cannot reasonably be understood to impute guilt. Suspicion no doubt can be inferred from the fact of the
inquiry being held, if such was the case, but to take the further step and infer guilt is in my view wholly unreasonable. This is to
draw an inference from an inference and to take two substantial steps at the same time.
The distinction between suspicion and guilt is illustrated by the case of Simmons v Mitchell which decided that spoken
words which convey a mere suspicion that the plaintiff has committed a crime punishable by imprisonment will not support an
action without proof of special damage. It has been argued before your lordships that suspicion cannot be justified without proof
of actual guilt on the analogy of the rumour cases such as Watkins v Hall. Rumour and suspicion do, however, essentially differ
from one another. To say that something is rumoured to be the fact is, if the words are defamatory, a republication of the libel.
One cannot defend an action for libel by saying that one has been told the libel by someone else, for this might be only to make
the libel worse. The principle, as stated by Blackburn J, in Watkins v Hall ((1868), LR 3 QB at p 401), is that a party is not the
less entitled to recover damages from a court of law for injurious matter published concerning him because another person
previously published it. It is wholly different with suspicion. It may be defamatory to say that someone is suspected of an
offence, but it does not carry with it that that person has committed the offence, for this must surely offend against the ideas of
justice, which reasonable persons are supposed to entertain. If one repeats a rumour one adds ones own authority to it, and
implies that it is well founded, that is to say, that it is true. It is otherwise when one says or implies that a person is under
suspicion of guilt. 167This does not imply that he is in fact guilty, but only that there are reasonable grounds for suspicion,
which is a different matter.
Having reached the conclusion that the innuendo should not have been left to the jury as a separate issue and that the natural
and ordinary meaning of the words does not convey actual guilt of fraud, I agree with the Court of Appeal that there must be a
new trial, for the learned judge left the question to the jury Did they find for plaintiffs or defendants? without a direction that
the words were incapable of the extreme meaning which I have rejected.
I would not, but for this misdirection as to the meaning of the words, have thought that a new trial should be ordered simply
because the innuendo was wrongly left to the jury, for no harm would have been done if there had been no misdirection as to the
meaning of the words. The vital misdirection was as to the meaning which the plaintiffs sought to ascribe to the words. As to
this, in a Scottish case Stubbs Ltd v Russell ([1913] AC 386 at p 393), Lord Kinnear said:

The law is perfectly well settled. Before a question of libel or slander is submitted to a jury the court must be satisfied
that the words complained of are capable of the defamatory meaning ascribed to them. That is a matter of law for the
court.

This is also the law of England; cf English and Scottish Co-operative Properties Mortgage and Investment Society Ltd v Odhams
Press Ltd, a case where there was a long paragraph of innuendoes suggesting various meanings to be attributed to the words
complained of but no true innuendo supported by extrinsic facts. All the innuendo meanings were left to the jury and both
Slesser LJ ([1940] 1 All ER at p 7; [1940] 1 KB at p 453), and Goddard LJ ([1940] 1 All ER at p 12; [1940] 1 KB at p 460),
referred to the duty of the judge to withdraw meanings from the jury if the words are incapable of bearing such meanings. I have
mentioned this last point because at one stage of the argument it seemed that it might be contended that, once the judge had ruled
the words were capable of any as opposed to the defamatory meaning ascribed to them, the jury were masters of the
situation, but the contention I have adumbrated was not advanced before your lordships and was expressly disclaimed by counsel
in the course of the hearing before the Court of Appeal.
The responsibility of the judge to exclude a particular meaning which the plaintiff seeks to ascribe to words in their natural
or ordinary meaning is, I think, clearly established by the decision of this House in Capital and Counties Bank v George Henty &
Sons. Henty & Sons had sent out a circular to a number of their customers giving notice that they would not receive in payment
cheques drawn on any of the vouchers of the bank. There was no evidence to support the innuendo that the words imputed
insolvency to the bank, and it was held that in their natural and ordinary meaning the words were not libellous. Lord Blackburn
said ((1882), 7 App Cas at p 776):

Since Foxs Act [32 Geo 3 c 60] at least, however the law may have been before, the prosecutor or plaintiff must also
satisfy a jury that the words are such and so published as to convey the libellous imputation. If the defendant can get either
the court or the jury to be in his favour he succeeds. The prosecutor, or plaintiff, cannot succeed unless he gets both the
court and the jury to decide for him.

Since in my judgment there must be a new trial in order that the jury in each pair of cases may be directed as to the natural
and ordinary meaning of the words published in the two newspapers, I need say nothing on the question of damages except that I
agree with the Court of Appeal that the damages were in each case so excessive that they cannot be allowed to stand. I also agree
that, as a result 168 of the decision of your Lordships House in British Transport Commission v Gourley, the jury in each case so
far as the plaintiff company is concerned should have been directed that, since a company can only suffer in its pocket by loss of
revenue attributable to a libel, so regard must be had to the fact that the profits of the company will in large measure be passed on
to the revenue and not retained for the benefit of the shareholders.
I am further of opinion that a direction should be given to the jury as to the effect of s 12 of the Defamation Act, 1952,
which enables other claims by the plaintiffs to be disclosed to the jury with the object of preventing compensation being given
twice over for the same libel, so that the jury should be directed to apply themselves to the injury inflicted in the particular case.
I would dismiss the appeal.

LORD DEVLIN. My Lords, the natural and ordinary meaning of words ought in theory to be the same for the lawyer as for the
layman because the lawyers first rule of construction is that words are to be given their natural and ordinary meaning as
popularly understood. The proposition that ordinary words are the same for the lawyer as for the layman is, as a matter of pure
construction, undoubtedly true. But it is very difficult to draw the line between pure construction and implication, and the
laymans capacity for implication is much greater than the lawyers. The lawyers rule is that the implication must be necessary
as well as reasonable. The layman reads in an implication much more freely; and, unfortunately, as the law of defamation has to
take into account, is especially prone to do so when it is derogatory.
In the law of defamation these wider sorts of implication are called innuendoes. The word explains itself and is very apt for
the purpose. In R v Horne ((1777) 2 Cowp at p 484) De Grey LCJ, said:

In the case of a libel which does not in itself contain the crime, without some extrinsic aid, it is necessary that it should
be put upon the record, by way of introduction, if it is new matter; or by way of innuendo, if it is only matter of
explanation. For an innuendo means nothing more than the words, id est, scilicet, or meaning, or aforesaid, as,
explanatory of a subject-matter sufficiently expressed before;

An innuendo had to be pleaded and the line between an ordinary meaning and an innuendo might not always be easy to draw. A
derogatory implication may be so near the surface that it is hardly hidden at all or it may be more difficult to detect. If it is said
of a man that he is a fornicator the statement cannot be enlarged by innuendo. If it is said of him that he was seen going into a
brothel, the same meaning would probably be conveyed to nine men out of ten. But the lawyer might say that in the latter case a
derogatory meaning was not a necessary one because a man might go to a brothel for an innocent purpose. An innuendo pleading
that the words were understood to mean that he went there for an immoral purpose would not therefore be ridiculous. To be on
the safe side, a pleader used an innuendo whenever the defamation was not absolutely explicit. That was very frequent since
scandalmongers are induced by the penalties for defamation to veil their meaning to some extent. Moreover, there were some
pleaders who got to think that a statement of claim was somehow made more forceful by an innuendo, however plain the words.
So rhetorical innuendoes were pleaded, such as to say of a man that he was a fornicator meant and was understood to mean that
he was not fit to associate with his wife and family and was a man who ought to be shunned by all decent persons and so forth.
Your lordships were told, and I have no doubt it is true, that before 1949 it was very rare indeed to find a statement of claim in
defamation without an innuendo paragraph.
I have said that a derogatory implication might be easy or difficult to detect; and of course it might not be detected at all
except by a person who was already in possession of some specific information. Thus, to say of a man that he was 169 seen to
enter a named house would contain a derogatory implication for anyone who knew that that house was a brothel but not for
anyone who did not. In the passage which I have quoted De Grey LCJ ((1777), 2 Cowp at p 484), distinguished between this sort
of implication and the implication that is to be derived from the words themselves without extrinsic aid, and he treats the term
innuendo as descriptive only of the latter. Since then the term has come to be used for both sorts of implication. Either sort
had to be put upon the record, as the chief justice said, and extrinsic facts had to be pleaded by way of introduction, as the
chief justice said, or as a prefatory averment, as it came to be called. The Common Law Procedure Act, 1852, s 61 did away
with the necessity of pleading the prefatory averment, while leaving it necessary to plead the innuendo: the section provided that

the plaintiff should be at liberty to aver that the words or matter complained of were used in a defamatory sense,
specifying such defamatory sense without any prefatory averment.

My lords, a system of pleading was built up on this basis which in 1949 was disconcerted by the introduction of a new rule
RSC, Ord 19, r 6(2). The object of the rule was to require that extrinsic facts must not only be proved but also be pleaded, thus
restoring the position before 1852. The object was simple enough. It is the language of the rule that has caused the difficulties
which have recently been brought to a head and have been the subject of three decisions, including the present one, by the Court
of Appeal. The rule reads:

(2) In an action for libel or slander if the plaintiff alleges that the words or matter complained of were used in a
defamatory sense other than their ordinary meaning, he shall give particulars of the facts and matters on which he relies in
support of such sense.

The word innuendo is not used. But the effect of the language is that any meaning that does not require the support of extrinsic
fact is assumed to be part of the ordinary meaning of the words. Accordingly, an innuendo, however well concealed, that is
capable of being detected in the language used is deemed to be part of the ordinary meaning.
This might be an academic matter if it were not for the principle that the ordinary meaning of words and the meaning
enlarged by innuendo give rise to separate causes of action. This principle, which originated out of the old forms of pleading,
seems to me in modern times to be of dubious value. But it is now firmly settled on the authority of Sim v Stretch, and the House
was not asked to qualify it. How is this principle affected by the new rule? Are there now three causes of action? If there are
only two, to which of them does the innuendo that is inherent in the words belong? In Grubb v Bristol United Press Ltd the
Court of Appeal, disagreeing with some observations made by Diplock LJ in Loughans v Odhams Press Ltd ([1962] 1 All ER at p
409; [1963] 1 QB at p 306), decided in effect that there were only two causes of action and that the innuendo cause of action
comprised only the innuendo that was supported by extrinsic facts.
My lords, I think on the whole that this is the better solution though it brings with it a consequence that I dislike, namely,
that at two points there is a divergence between the popular and the legal meaning of words. Just as the popular and legal
meanings of malice have drifted apart, so the popular and legal meanings of innuendo must now be separated. I shall in the
rest of my speech describe as a legal innuendo the innuendo that is the subject-matter of a separate cause of action. I suppose that
it does not matter what terminology is used so long as it is agreed. But I do not care for the description of the popular innuendo
as a false innuendo; it is the law and not popular usage that gives a false and restricted meaning to the word. The other respect is
that the natural and ordinary 170 meaning of words for the purposes of defamation is not their natural and ordinary meaning for
other purposes of the law. There must be added to the implications which a court is prepared to make as a matter of construction
all such insinuations and innuendoes as could reasonably be read into them by the ordinary man.
The consequence of all this is, I think, that there will have to be three paragraphs in a statement of claim where previously
two have served. In the first paragraph the defamatory words will be set out as hitherto. It may be that they will speak for
themselves. If not, a second paragraph will set out those innuendoes or indirect meanings that go beyond the literal meaning of
the words but which the pleader claims to be inherent in them. Thirdly, if the pleader has the necessary material, he can plead a
secondary meaning or legal innuendo supported by particulars under RSC, Ord 19, r 6(2). Hitherto it has been customary to put
the whole innuendo into one paragraph, but now this may easily result in the confusion of two causes of action and in consequent
embarrassment. The essential distinction between the second and third paragraph will lie in the fact that particulars under the rule
must be appended to the third. That is, so to speak, the hallmark of the legal innuendo. The pleader can if he chooses emphasise
the character of the second paragraph by including in it some such words as were used in Loughans v Odhams Press Ltd. That
case was, in my opinion, rightly decided and rightly distinguished from Grubb v Bristol United Press Ltd by Upjohn LJ ([1962] 2
All ER at p 393; [1963] 1 QB at p 331), in the latter case. Or the pleader can, as was suggested by Holroyd Pearce LJ in Grubbs
case ([1962] 2 All ER at p 391; [1963] 1 QB at p 329), plead in the second paragraph that the words in their natural and ordinary
meaning were understood to mean one thing; and then he could plead in the third paragraph that, by reason of the facts
thereinafter particularised, they were understood to mean another. The meanings alleged in the third paragraph can be the same
as those alleged in the second paragraph if the pleader is relying on the legal innuendo only as an alternative; or they can be
different. But the essential thing is that, if a paragraph is unaccompanied by particulars, it cannot be a legal innuendo, since for a
legal innuendo particulars are mandatory and the innuendo cannot be proved without them.
It was suggested in argument that the division of the innuendo into two paragraphs would be awkward for the pleader. It is
said that it may not always be easy to decide whether an extrinsic factor relied on is a matter of special knowledge, or whether it
is just general knowledge in the light of which the ordinary though indirect meaning of the words has to be ascertained. I do not
think that this should present any difficulty in practice. The pleader must ask himself whether he contemplates that evidence will
be called in support of the allegation: if he does, it is a legal innuendo and if he does not, it is not. If he is in doubt, he can plead
in two paragraphs; and then if at the trial his opponent agrees or the judge rules that it is a matter of general knowledge, the legal
innuendo can be dropped.
It was also suggested to your lordships that the pleading of a middle paragraph was unnecessary and even improper and your
lordships were told that, since 1949, some judges have discouraged the pleading of all innuendoes that are not legal innuendoes.
I should certainly like to see what I have called rhetorical innuendoes discouraged; but I am satisfied that the pleading of an
innuendo in every case where the defamatory meaning is not quite explicit is at the least highly desirable, and I am glad to
observe that in Loughans case the attempt to strike out the innuendo failed. An attempt of this sort is no doubt inspired by the
thought that it is unnecessary to plead the ordinary meaning of words and that that is all that the popular innuendo is. I think that
that thought is fallacious. It does not take into account the difference 171 which I have pointed out between the meaning of
words in the law of defamation and their meaning for the general purposes of the law. In general the meaning of words is a
matter of law and therefore need not be pleaded, though, where there is a difficult question of construction in issue, it is usual and
convenient to do so. But in defamation the meaning of words is a question of fact, that is, there is libel or no libel according to
the impression the words convey to the jury and not according to the construction put on them by the judge. I do not mean that
ingenuity should be expended in devising and setting out different shades of meaning. Distinct meanings are what should be
pleaded: and a reasonable test of distinctness would be whether the justification would be substantially different. In the present
case, for example, there could have been three distinct categories of justificationproof of the fact of an inquiry, proof of
reasonable grounds for it and proof of guilt. If no innuendo had been pleaded and there had been full proof of grounds for
inquiry, I cannot think that in a closing speech the plaintiff could without any previous notice invite the jury to say that the words
meant guilt and to reject the justification as insufficient. Moreover, where distinct meanings are possible and the judge is invited
to rule separately on one or more, it is desirable that the meanings put to the jury should be on the record. But that touches on a
point of substance which I shall consider later.
I understand your lordships all to be of the opinion that the pleading of the ordinary or popular innuendo is permissible, but
do not intend that the House should rule on whether it is necessary. I agree that the point does not arise directly in this case, and,
therefore, I too shall reserve my judgment on it. But I make the comment that, if it is not necessary, it is nevertheless a form of
pleading universally used from the earliest times until 1949, and I can see nothing in the new rule that should alter so well
established a practice.
My lords, I have made a very long preliminary to the consideration of the pleading point in this case. Your lordships were
invited from the Bar to deal in detail with all the difficulties of pleading involved in that point and that have recently come to the
fore in other cases and I have thought it right to do so. I must now state how in the light of what I have said generally I should
decide the point at issue. Paragraph 4 of the statement of claim is as follows:

4. By the said words the defendants meant and were understood to mean that the affairs of the plaintiffs and/or its
subsidiaries were conducted fraudulently or dishonestly or in such a way that the police suspected that their affairs were so
conducted.

The Court of Appeal considered this paragraph to be defective and I agree with them. This does not involve any sort of criticism
of the learned pleader who drafted his statement of claim at a time when it was possible to take almost any view of the points I
have been canvassing. It is plain now that para 4 must be treated as in form a plea of a legal innuendo. But in substance it is not
a legal innuendo because no extrinsic facts are pleaded: general knowledge is, as I have indicated already, not an extrinsic fact
for the purpose of RSC, Ord 19, r 6(2), but is matter, not requiring to be proved, in the light of which the jury can interpret the
publication. In substance the paragraph is a plea of a popular innuendo, and the confusion between substance and form makes it
embarrassing. But I cannot with respect agree with the Court of Appeal that the way in which the judge treated this point is by
itself a ground for a new trial. He went by the substance of the paragraph and left it to the jury as an ordinary innuendo, not a
legal one. Perhaps he ought to have insisted on an amendment in the form, but he stated the course that he was going to take and
neither counsel offered any objection to it. I cannot think that the jury could have been in any way misled. There has been some
discussion about whether the plaintiffs will have to amend before proceeding to a new trial. That is for them to say. In the light
of all this discussion they would perhaps be wise not to take indulgence for granted.
172
I turn now to the main ground for ordering a new trial. This was that the judge misdirected the jury by failing to tell them
that the words were not capable of bearing one or more of the defamatory meanings alleged in para 4 of the statement of claim. It
is admitted that the words are capable of some defamatory meaning, and I think that it is undoubtedly defamatory of a company
to say that its affairs are being inquired into by the police. But para 4 alleges that the words meant
that the affairs of the plaintiffs and/or its subsidiaries were conducted fraudulently or dishonestly or in such a way that
the police suspected that their affairs were so conducted.

This is saying that the words mean either that the plaintiffs were guilty of fraud or that they were suspected of fraud. If it is
permissible to distinguish between these two meanings, then for reasons which I shall give as I proceed I should hold that the
words are capable of the latter meaning but not of the former, and I should on this basis agree with the Court of Appeal that the
jury should have been so directed and that, since they were not, there should be a new trial. But counsel for the appellants has
submitted that it is not right so to distinguish.
In the first place, he relies on what are called the rumour cases. I agree, of course, that one cannot escape liability for
defamation by putting the libel behind a prefix such as I have been told that or It is rumoured that , and then asserting
that it was true that one had been told or that it was in fact being rumoured. You have, as Horridge J said, in a passage that was
quoted with approval by Greer LJ, in Cookson v Harewood ([1931] All ER Rep 533 at p 536; [1932] 2 KB 478 at p 485), to
prove that the subject-matter of the rumour was true. But this is not a case of repetition or rumour. I agree with the distinction
drawn by Horridge J on this point, though not necessarily with his limited view of the effect of the libel in that case. Anyway,
even if this is to be treated as a rumour case, it is still necessary to find out what the rumour is. A rumour that a man is suspected
of fraud is different from one that he is guilty of it. For the purpose of the law of libel a hearsay statement is the same as a direct
statement, and that is all there is to it.
The real point, I think, that counsel for the appellants makes is that whether the libel is looked at as a statement or as a
rumour, there is no difference between saying that a man is suspected of fraud and saying that he is guilty of it. It is undoubtedly
defamatory, he submits, to say of a man that he is suspected of fraud, but it is defamatory only because it suggests that he is guilty
of fraud: so there is no distinction between the two. This is to me an attractive way of putting the point. On analysis I think that
the reason for its attraction is that as a maxim for practical application, though not as a proposition of law, it is about three-
quarters true. When an imputation is made in a general way, the ordinary man is not likely to distinguish between hints and
allegations, suspicion and guilt. It is the broad effect that counts and it is no use submitting to a judge that he ought to dissect the
statement before he submits it to the jury. But if, on the other hand, the distinction clearly emerges from the words used, it cannot
be ignored. If it is said of a manI do not believe that he is guilty of fraud but I cannot deny that he has given grounds for
suspicion, it seems to me to be wrong to say that in no circumstances can they be justified except by the speaker proving the
truth of that which he has expressly said that he did not believe. It must depend on whether the impression conveyed by the
speaker is one of frankness or one of insinuation. Equally in my opinion it is wrong to say that, if in truth the person spoken of
never gave any cause for suspicion at all, he has no remedy because he was expressly exonerated of fraud. A mans reputation
can suffer if it can truly be said of him that although innocent he behaved in a suspicious way; but it will suffer much more if it is
said that he is not innocent.
It is not therefore correct to say as a matter of law that a statement of suspicion imputes guilt. It can be said as a matter of
practice that it very often does so, 173because although suspicion of guilt is something different from proof of guilt, it is the
broad impression conveyed by the libel that has to be considered and not the meaning of each word under analysis. A man who
wants to talk at large about smoke may have to pick his words very carefully, if he wants to exclude the suggestion that there is
also a fire; but it can be done. One always gets back to the fundamental question: what is the meaning that the words convey to
the ordinary man; a rule cannot be made about that. They can convey a meaning of suspicion short of guilt; but loose talk about
suspicion can very easily convey the impression that it is a suspicion that is well founded.
In the libel that the House has to consider there is, however, no mention of suspicion at all. What is said is simply that the
plaintiffs affairs are being inquired into. That is defamatory, as is admitted, because a mans reputation may in fact be injured by
such a statement even though it is quite consistent with innocence. I daresay that it would not be injured if everybody bore in
mind, as they ought to, that no man is guilty until he is proved so, but unfortunately they do not. It can be defamatory without it
being necessary to suggest that the words contained a hidden allegation that there were good grounds for inquiry. A statement
that a woman has been raped can affect her reputation, although logically it means that she is innocent of any impurity:
Youssoupoff v Metro-Goldwyn-Mayer Pictures Ltd. So a statement that a man has been acquitted of a crime with which in fact he
was never charged might lower his reputation. Logic is not the test. But a statement that an inquiry is on foot may go further and
may positively convey the impression that there are grounds for the inquiry, ie, that there is something to suspect. Just as a bare
statement of suspicion may convey the impression that there are grounds for belief in guilt, so a bare statement of the fact of an
inquiry may convey the impression that there are grounds for suspicion. I do not say that in this case it does; but I think that the
words in their context and in the circumstances of publication are capable of conveying that impression. But can they convey an
impression of guilt? Let it be supposed, first, that a statement that there is an inquiry conveys an impression of suspicion; and,
secondly, that a statement of suspicion conveys an impression of guilt. It does not follow from these two suppositions that a
statement that there is an inquiry conveys an impression of guilt. For that, two fences have to be taken instead of one. While, as
I have said, I am prepared to accept that the jury could take the first I do not think that in a case like the present, where there is
only the bare statement that a police inquiry is being made, it could take the second in the same stride. If the ordinary sensible
man was capable of thinking that wherever there was a police inquiry there was guilt, it would be almost impossible to give
accurate information about anything: but in my opinion he is not. I agree with the view of the Court of Appeal.
There is on this branch of the case a final point to be considered. It is undoubtedly the law that the judge should not leave
the question libel or no libel to the jury unless the words are reasonably capable of a defamatory meaning. But if several
defamatory meanings are pleaded or suggested, can the judge direct the jury that the words are capable of one meaning but not of
another? The point is important here, because the defendants admit that the words are defamatory in one sense but dispute that
they are defamatory in the senses pleaded in the statements of claim, and contend that the judge should have so directed the jury.
Both counsel for the appellants appear at one time to have argued in the Court of Appeal that the function of the judge was
exhausted when he ruled that the words were capable of being defamatory; and that it was not for him to inquire whether they
were or were not capable of any particular defamatory meaning. But later they abandoned the point; and, therefore, did not
initiate the discussion of it here. Nevertheless there was considerable discussion of it, because some of your lordships at one time
felt that it was a point which ought to be considered. In the result I think that all your lordships are 174 now clearly of the
opinion that the judge must rule whether the words are capable of bearing each of the defamatory meanings, if there be more than
one, put forward by the plaintiff. This supports indirectly my view on the desirability of pleading different meanings. If the
plaintiff can get before the jury only those meanings which the judge rules as capable of being defamatory, there is good reason
for having the meanings alleged set out precisely as part of the record.
For the reasons that I have given earlier, I agree that there must be a new trial on the ground of misdirection: but I should in
any event have considered that there should be a new trial on the issue of damages as they are, in my opinion, ridiculously out of
proportion to the injury suffered.

Appeals dismissed.

Solicitors: Zeffertt, Heard & Morley Lawson (for the appellants); Swepstone, Walsh & Son (for the respondents, Associated
Newspapers Ltd); Simmons & Simmons (for the respondents, Daily Telegraph Ltd).

C G Leonard Esq Barrister.


[1963] 2 All ER 175

Britt v Buckinghamshire County Council


TOWN AND COUNTRY PLANNING

COURT OF APPEAL
SELLERS, HARMAN AND PEARSON LJJ
6, 7, 8 FEBRUARY 1963

Town and Country Planning Amenity Notice to abate injury to amenity by condition of vacant site or open land Use of land
in agricultural area as car dump Established use since before 1947, but no express planning permission granted Validity of
notice Town and Country Planning Act, 1947 (10 & 11 Geo 6 c 51), s 33(1).

The power conferred by s 33a of the Town and Country Planning Act, 1947, to require the abatement (without compensation) of a
condition of a vacant site or open land b that is injurious to the amenity of an area extends to requiring the abatement by a
landowner of an active use, or a condition brought about by such use, as well as to requiring the abatement of a condition not
attributable to any active use by the landowner; and the power is exercisable, unless planning permission for the use has been
granted, notwithstanding that the use is an established use existing before 1 July 1948, and thus is outside the control of other
provisions of Part 3 of the Act of 1947, and that the exercise of that other control (if it had been applicable) would have entitled
the landowner to compensation (see p 178, letters b, g and h, p 179, letters g and h, p 180, letter g, and p 182, letter c, post).
________________________________________
a Section 33(1), (2) are printed at pp 176, 177, post
b The statutory power extends in terms to any garden, vacant site or other open land, but the word garden is not descriptive of the land
concerned in the present case

Statute Construction Statutory instrument Use of regulations for interpretation of section of statute Regulations modifying
statutory provisions rendered applicable by the section Town and Country Planning Act, 1947 (10 & 11 Geo 6 c 51), s 33(2).

Section 33(2) of the Town and Country Planning Act, 1947, conferred power to modify by regulations certain provisions of the
Act made applicable for the purposes of s 33(1). Modifications were prescribed by the Town and Country Planning (General)
Regulations, 1948, reg 7, which took effect contemporaneously with the coming into operation of s 33.

Held In construing s 33 regard might be had to reg 7 of the regulations of 1948 (see p 177, letter f, p 180, letter a, and p 182,
letter i, post).
Dictum of Upjohn LJ in Stephens v Cuckfield Rural District Council ([1960] 2 All ER at p 718) distinguished.
Observations on inaccuracy of marginal note to s 33 as rendering it of no assistance on interpretation (see p 183, letters b
and c, and p 178, letter c, post).
Appeal dismissed.
175

Notes
By the Caravan Sites and Control of Development Act, 1960, ss 48, 50(4) and Sch 4, as from 29 August 1960, the words in the
manner prescribed by regulations under this Act in s 33(1), and s 33(2), of the Town and Country Planning Act, 1947, have been
repealed. Regulation 7 of the Town and Country Planning (General) Regulations, 1948 (SI 1948 No 1380), has been revoked by
the Town and Country Planning (General) (Amendment) Regulations, 1960 (SI 1960 No 1475). Provisions as to appeal against a
notice served under s 33(1) of the Act of 1947 are now contained in Sch 3, paras 812, to the Act of 1960(40 Halsburys Statutes
(2nd Edn) 1110, 1111). The Town and Country Planning Act, 1947, s 23(3) (in part), (4) and (5), and s 24 (in part), have also
been repealed as from 29 August 1960, by the Act of 1960.
As to power to require abatement of injury to amenity by condition of garden, vacant site or other open land, see Halsburys
Laws (3rd Edn) 399, para 506.
For the Town and Country Planning Act, 1947, s 33, see 25 Halsburys Statutes (2nd Edn) 541.

Case referred to in judgments


Stephens v Cuckfield Rural District Council [1960] 2 All ER 716, [1960] 2 QB 373, 124 JP 420, [1960] 3 WLR 248, 3rd Digest
Supp, affg, [1959] 1 All ER 635, [1959] 1 QB 516, [1959] 2 WLR 480.

Appeal
This was an appeal by the plaintiff landowner and a cross-appeal by the defendant local planning authority from an order of
Widgery J, dated 4 July 1962. Since before 1947, Robert George Graham Britt, the plaintiff landowner, had used certain land
which he owned and occupied in an otherwise agricultural area at Prestwood, Buckinghamshire, as a car-dump, this making the
land an eyesore. On 1 September 1958, the defendant local planning authority, Buckinghamshire County Council, served on the
landowner, inter alia, a notice under s 33 of the Town and Country Planning Act, 1947, requiring him to cease this use of the land.
The landowner brought this action against the local planning authority for a declaration, inter alia, that the notice was invalid.
The local planning authority counter-claimed for a declaration that the notice was valid. Widgery J, declared that the notice was
not invalid, and dismissed the landowners claim.

V G Wellings for the landowner.


Desmond Ackner QC and David Trustram Eve for the local planning authority.

8 February 1963. The following judgments were delivered.

SELLERS LJ. As far back as 1958, if not earlier, the local planning authority, the defendant county council, in the performance
of their public duty and in the interests of the community, decided to exercise the powers which they thought that they had under
the Town and Country Planning Act, 1947. In 1963 the validity of the various steps which they took is still being called in
question before this court. I will allow that bare statement of fact to stand without comment. In the course of this appeal happily
some of the matters that have been in contention have been abandoned, and this court is called on to decide one short point only
under one section of the Act of 1947. It relates to a notice given by the local planning authority on 1 September 1958, in respect
of a piece of land fronting on the easterly side of Hampden Road, Prestwood, Buckinghamshire, which was, and is, owned and
occupied by the plaintiff landowner.
Section 33 of the Town and Country Planning Act, 1947, is as follows:

(1) If it appears to a local planning authority that the amenity of any part of the area of that authority, or of any
adjoining area, is seriously injured by the condition of any garden, vacant site or other open land in their area, then, subject
to any directions given by the minister, the 176 authority may serve on the owner and occupier of the land, in the manner
prescribed by regulations under this Act, a notice requiring such steps for abating the injury as may be specified in the
notice to be taken within such period as may be so specified. (2) In relation to any notice served under this section, the
provisions of sub-ss. (3) to (5) of s. 23 of this Act, and of s. 24 of this Act shall, subject to such exceptions and
modifications as may be prescribed by regulations under this Act, apply as those provisions apply in relation to an
enforcement notice served under s. 23.

No direction limiting the scope of sub-s (1) has been given by the minister as could have been done if it was desired to curtail its
effect. Under powers given by the Act of 1947, a regulation was made modifying s 33(2); namely, reg 7 of the Town and Country
Planning (General) Regulations, 1948. Regulation 7(1) provides for the manner of the service of a notice under s 33(1), and para
(2) reads as follows:

The following exceptions and modifications shall be made in sub-ss. (3) to (5) of s. 23 and s. 24 of the Act in their
application to any such notice as is referred to in para. (1) of this regulation, namely: (a) in sub-s. (3) of s. 23, para. (a) of
the proviso shall be omitted; (b) for heads (a) and (b) of sub-s. (4) of s. 23, there shall be substituted the following heads:
(a) if satisfied that the condition of the land to which the notice relates is such as results in the ordinary course of events
from operations or a use for which permission was granted under this Part of this Act, shall quash the notice to which the
appeal relates; (b) if not so satisfied, but satisfied that the requirements of the notice exceed what is necessary for abating
the injury, shall vary the notice accordingly.

I have restricted my citation to those provisions and have not read the whole regulation, because it is only that portion which
becomes relevant in the task before this court of construing the scope of s 33. The regulations came into force in 1948 at the
same time as the Town and Country Planning Act, 1947, and, having regard to the terms of s 33(2), the regulations must, I think,
be read in conjunction with that section, however unusual it may be to insert modifications in this way. It is not, I think, using the
regulations quite in the manner criticised by Upjohn LJ, in his judgment in Stephens v Cuckfield Rural District Council ([1960] 2
All ER 716 at p 718; [1960] 2 QB 373 at p 380).
Section 33 is within Part 3 of the Act of 1947, which is headed Control of Development, etc. Permission to develop land.
Section 12 commences this Part, and a number of sections sets out the circumstances in which permission can be given and the
method by which the control is to be exercised. Section 23 provides for the enforcement of planning control, and, by sub-s (4),
any person on whom an enforcement notice is served may, within a specified period, appeal against the notice to a court of
summary jurisdiction, and reg 7, to which I have referred, adapts s 23 to the circumstances of s 33. Then, prior to s 26, there is
inserted a heading Additional powers of control, under which falls s 33. On the face of it, the power to abate serious injury to
the amenity of an area would appear to be additional to rights to prevent user on some other ground.
The contention which was advanced before us was that which was argued also before Widgery J (from whom this appeal
comes), that is to say, that s 33 only applies

to a case where the injury to amenity results from the condition of the land as opposed to the use to which land is put,
and that, where the injury is a necessary result of the particular use to which the land is put, the planning authority must
take steps to determine that use under the earlier provisions of the Act (paying compensation in appropriate cases) and
cannot proceed under s. 33.
177

What was submitted was that regard could be had only to what one of the learned counsel described as the inactive condition and
not to the active condition brought about by the use. In construing this matter, I think that the regulation, modifying as it does the
section of the Act and being read with it, makes it clear that that contention cannot prevail and that the learned judge was right in
the conclusion to which he camealthough he himself did indicate that, apart from authority, he might have been inclined to take
a different view. Quite apart from that additional assistance, the words of s 33 read in their ordinary meaning do not limit the
word condition in any way. In its ordinary meaning, the condition would be the state in which it was, irrespective of how it
was brought about, the condition of any garden, vacant site or other open land in their area. If there were any doubt as to that, I
think that it is resolved in looking at the modification brought about by the regulation, which contemplates that operations or use
may seriously injure amenities. Attention was drawn to the sidenote, Power to require proper maintenance of waste land, etc.
This is a case, I think, which demonstrates how unreliable it would be, even if permissible, to pay attention to the sidenote. It
seems to me to be inconsistent with the precise wording of the section itself, to which the word maintenance is inappropriate.
It was in accordance with s 33 that the notice was given to which I have earlier referred. The circumstance which gives rise
to the argument is that there is a small part of the land occupied by the landowner which is not subject to other provisions of the
Act of 1947. The local planning authority have relied on the additional powers of control in s 33 in respect of all the land,
including the small part otherwise immune, and in other respects and in respect of the other land they have relied on notices given
under s 23, dealing with the substantive control of the development of land. But it was said that the power ought not to be
exercised because it was, in effect, using s 33 to terminate the user of part of the land which had acquired rights which could not
be interfered with, and to do so without compensation. It appears that, before the passing of the Act of 1947, the landowner had
some years before (as the judgment of Widgery J shows) used this area of otherwise peaceful and attractive agricultural land in
the heart of Buckinghamshire to carry on a business of a somewhat strange character. He leaves there, apparently for a very long
time, a collection of old motor vehicles, damaged and beyond use, taking part of one to make up another, and so forth. There is
no question, on the facts of this case, as the local planning authority thought and as the judge also upheld, indeed as counsel for
the landowner had to admit, that the whole of this area is rightly described as an eyesore. It infringes the provisions of s 33 in
that it seriously injures the amenities of that locality. In those circumstances, the order was rightly made. No objection is made
to this particular notice save that it is not within the power of the local planning authority to make it because it would interfere
with an established right which was beyond the control of the other sections of the Act of 1947. It seems to me that that is not a
tenable argument. If there had been, on the facts of this case, a use established for which permission had been granted under this
Part of the Act, then that would have been an answer to the notice which was given, but without thatand it was not contended
that such a permission had been grantedI think that this case must fail, the learned judge came to the right conclusion, the
notice was good and full effect should be given to it.
There is a cross-notice asking that the declaration which the learned judge made should be altered in a minor form from a
declaration that the notice to which I have made reference was not invalid to an affirmative declaration that it was valid. That has
not been disputed by the learned counsel on the other side and I am of the opinion, with all respect to the meticulous refinement
of the learned judges view, that the order should be varied accordingly. Subject to that small variation, I would dismiss the
appeal.
178

HARMAN LJ. Hard indeed are the paths of local authorities in striving to administer the town and country planning legislation
of recent years. It is a sorry comment on the law and those who administer it that, between the years 1947 and 1960, they had
succeeded in so bedevilling the whole administration of that legislation that Parliament was compelled to come to the rescue and
remove a great portion of it from the purview of the courts. Not for nothing was I offered a book yesterday called
Encyclopaedia of Planning. It is a subject which stinks in the noses of the public, and not without reason. Local authorities,
until they have been recently rescued have had practically to employ conveyancing counsel to settle these notices which they
serve in the interests of planning the countryside or the towns which they control. Instead of their trying to make this thing
simpler, lawyers succeeded day by day in making it more difficult and less comprehensible, until it has reached a stage where it is
very much like the state of the land which the plaintiff landowner has brought about by his operationsan eyesore, a wilderness,
and a scandal. It may be hoped that this is one of the last of the actions of this sort where declarations are asked in respect of the
multitude of notices served by the poor bewildered local planning authority striving to do their best to cure an eyesore glaring in
the face of the countryside.
By the time the argument before us came to an end, the numerous points agitated before the judge below and, indeed, to
some extent opened to us had all faded away, and we were left with this one question under s 33 of the Town and Country
Planning Act, 1947. It has never seemed to me that there was any merit in the contention that, in some way or other, a limitation
had to be put on the words of this sectionwhich exactly fit, on the face of them, the state of things which was to be found here.
Some limitation (it was said) had to be put on them because the Control of development sections, which begin with s 23,
formed a kind of code, and as the earlier ones provided a scheme for preventing various users of land and for stopping abuses
which were not allowed by the local planning authority and giving compensation to those whose rights were taken away, one
must not, by having recourse to s 33, take a short cut, get away from all those and simply say: This is an eyesore: it is doing
serious injury to the amenity of the countryside: abate it. It is said that the man who is told to abate an eyesore ought to be
compensated for it. That seems to me to be a most astonishing doctrine. I can see no reason why the landowner, who has for
years made the country hideous by his goings-on, should not be made to put his house in order, and no reason at all why he
should be paid by the public for doing it. However, that was the plea that was urged before us, and to some extent it found favour
with the learned judge below, but I cannot think that his view was right. He did, in fact, bow to authority, Stephens v Cuckfield
Rural District Council, and decide that s 33 did fit the case here, and in that, in my judgment, he was right.
If help were wanted to aid the construction of s 33 and show that it means what I think that it saysnamely, that a use, as
well as a non-use, can be prohibited under that sectionthen I think it would be legitimate to look at the Town and Country
Planning (General) Regulations, 1948, which were made in 1948 and which came into existence at the same time as the Act itself.
My Lord has read them; I shall not re-read them; but it was said in Stephens v Cuckfield RDC (where Lord Parker CJ had applied
that principle), that the court there doubted whether that was a correct thing to do. What was said was this ([1960] 2 All ER at p
718; [1960] 2 QB at p 381):

We doubt very much whether it is right to construe the words of the section by reference to regulations made under
powers therein contained

If that were the real process, I should agree with that comment, but in this case 179 there is a process which I have not seen
anywhere else. A power is given to the minister by the Act itself to modify another section of the Act so that, when the minister
does produce that modification (as he did in the 1948 regulations), that regulation becomes in fact part of the Act. It is like an
amending section of the Act; so that, in my judgment, reference can be made to that regulation because it is embodied in the Act
itself and, having a quasi-parliamentary validity, is a good indication of the wishes of the legislature, just as much as if it were
enacted in the Act itself. I do not myself think that aid is necessary. I should have arrived at the same conclusion without it; but,
if it be necessary, is not a question of construing an Act by subsequent regulations, in the ordinary sense of those words.
I agree with what my Lord has said with regard to the cross-notice.
For these reasons, I think that the judge reached the right conclusion and that this appeal fails.

PEARSON LJ. In this appeal we are in the end concerned only with the notice referred to as notice H, which was served
under s 33 of the Town and Country Planning Act, 1947. [His Lordship referred to other notices, served under s 23 of the Act of
1947, with regard to which the plaintiff landowner had not pursued his appeal, and continued:] The contention, and the only
contention, argued in this appeal appears most conveniently in the learned judges judgment, where he said this, referring to the
argument of counsel for the landowner:

Counsel for the landowner does not contend that the facts of this case do not justify the allegation contained in notice
H, nor does he dispute that the land in question is a garden, vacant site or other open land within the meaning of the
section. He attacks notice H on the broad ground that s. 33 applied only to a case where the injury to amenity results from
the condition of the land as opposed to the use to which the land is put, and that, where the injury is a necessary result of
the particular use to which the land is put, the planning authority must take steps to determine that use under the earlier
provisions of the Act (paying compensation in appropriate cases) and cannot proceed under s. 33.

That is a contention as to the power conferred by s 33 of the Act of 1947, and it involves a consideration of the construction and
effect of s 33. Section 33 has already been read and I need not repeat it. I would entirely concur with what has already been said,
that, on the wording of that section, taken by itself without any extraneous aid to construction, quite plainly the words seriously
injured by the condition of any garden, vacant site or other open land in their area must refer to the actual condition, however it
may have been caused. Whether it results from inactivity or whether it results from some activity, it is, nevertheless, the
condition of the land. Therefore, the natural and ordinary meaning of the words is in favour of the decision which the learned
judge has given.
If I go into further questions of construction and extraneous aids, it is not with a view to introducing further complications
into the construction of this Act but merely with a view to showing that the natural and ordinary meaning of the wording in s 33
ought to be adhered to. In the first place, as counsel for the local planning authority has pointed out to us, there is no reason for
not giving s 33 its natural and full extent and operation, because it is a section designed to enable a local planning authority to
protect the amenity of part of its area. Moreover, certain safeguards are provided by the section itself. The power exists subject
to any directions given by the minister, and there are safeguards imported by sub-s (2) to which I am about to refer. Secondly, I
come to s 33(2), and I will consider it at the outset by itself without bringing 180 in the ministers regulation made under it,
because, in my view, s 33(2) by itself, even without any regulation, gives support to the natural and ordinary meaning of the
words in sub-s (1). The point is a little complicated, but I think that it is worth making. I will first of all read s 33(2):

In relation to any notice served under this section, the provisions of sub-ss. (3) to (5) of s. 23 of this Act, and of s. 24
of this Act shall, subject to such exceptions and modifications as may be prescribed by regulations under this Act, and of s.
24 of this Act shall, subject to such exceptions and modifications as may be prescribed by regulations under this Act, apply
as those provisions apply in relation to an enforcement notice served under s. 23.

It is to be observed that the subsections of s 23 which are to apply subject to any exceptions and modifications which the minister
may introduce include s 23(4). It is sub-s (4) which is material for the present purpose. Before going to that, I have to look back
to s 12(2), which defines the expression development as meaning

the carrying out of building, engineering, mining or other operations in, on, over or under land, or the making of any
material change in the use of any buildings or other land.

I have only read the principal part of the definition; the other parts are not material for the present purpose. Thus, the word
development includes operations and includes any material change in the use of the land. I then go to s 23(4), which, in
relation to the enforcement of planning control with respect to developments of land, contains this provision:

If any person on whom an enforcement notice is served under this section is aggrieved by the notice, he may, at any
time within the period mentioned in the last foregoing sub-section, appeal against the notice to a court of summary
jurisdiction for the petty sessional division or place within which the land to which the notice relates is situated; and on any
such appeal the court(a) if satisfied that permission was granted under this Part of this Act for the development to which
the notice relates, or that no such permission was required in respect thereof, or, as the case may be, that the conditions
subject to which such permission was granted have been complied with, shall quash the notice to which the appeal relates.

I need not read (b) and (c) of that provision. The application of s 23(4) of the Town and Country Planning Act, 1947, for the
purposes of s 33, albeit with modifications to be made by regulations, raises an inference that a notice under s 33 may affect a
permitted development and, therefore, may relate to a condition which is caused by operations taking place on the land or to a use
being made of the land. Therefore, even if we paused there and did not proceed to consider the regulations, in my view s 33(2) in
itself, when taken in conjunction with the other provisions to which I have referred, does afford some support for the learned
judges construction of s 33; but then, thirdly, we have to pass on to the peculiar position of regulations made by the minister for
the purposes of s 33(2). I need not read that subsection again, but I should refer to s 111(1) of the Act of 1947, because that
confers the power on the minister to make regulations, and then, by sub-s (2), it provides that regulations shall be laid before
Parliament, and if either House resolves that the regulations be annulled they are to be annulled accordingly. Section 120(2)
provides that the Act of 1947 shall come into force on the appointed day, and the appointed day is defined in s 119(1) as such
day as the minister may by order appoint.
Therefore, we have the position that regulations are required to be made under this Act. The relevant regulations are the
Town and Country Planning (General) Regulations, 1948 (SI 1948 No 1380). They are expressed to be made in exercise of the
power conferred on the minister by various sections of the Act of 1947, including s 33. Regulation 1 provides These regulations
shall come into force on the appointed day. Therefore, both the Act and the 181 regulations would come into force on the
same day. Then there is reg 7, which contains provisions under s 33. Regulation 7(2) provides that:

The following exceptions and modifications shall be made in sub-ss. (3) to (5) of s. 23 and s. 24 of the Act in their
application to any such notice as is referred to in para. (1) of this regulation.

That was a notice served under s 33. Then under head (b) of that paragraph, it is provided:

For heads (a) and (b) of sub-s. (4) of s. 23, there shall be substituted the following heads:(a) if satisfied that the
condition of the land to which the notice relates is such as results in the ordinary course of events from operations or a use
for which permission was granted under this Part of this Act, shall quash the notice to which the appeal relates; (b) if not so
satisfied, but satisfied that the requirements of the notice exceed what is necessary for abating the injury, shall vary the
notice accordingly.

If it is permissible to take that provision into account, it shows quite clearly and unmistakably that the conditions referred to in s
33(1) must include a condition which results from operations on the land or of use made of the land. The only problem is
whether it is permissible to take that provision of the regulations into account. There is a quite peculiar and exceptional position
arising here. I do not say that there may not be other cases, but at any rate I think that they must be cases which arise very rarely.
That which was to come into force and did come into force on the appointed day was a composite instrument consisting of the
provisions of the Act of 1947 as modified by the provisions of the regulations. The statutory force of this composite instrument is
derived from s 33(2), rather than from the regulations themselves. The regulations make their own provisions, but, in the end, it
is a composite instrument which has to be construed as such. Then it becomes quite clear from the terms of s 23(4), as modified
by reg 7(2)(a) and (b), that a notice under s 33 may refer to a condition which results from operations taking place on the land or
from a use made of the land.
I should refer to a passage in the judgment of this court in Stephens v Cuckfield Rural District Council ([1960] 2 All ER at p
718; [1960] 2 QB 373 at pp 380, 381). Upjohn LJ delivering the judgment of the court, said:

In the court below the learned Lord Chief Justice expressed the view that the prima facie meaning of open land in s.
33 covered operations on or user of any open land in the sense of land which was not built on [See [1959] 1 All ER at p
637; [1959] 1 QB at p 524]. He was to some extent influenced by the regulations made by the minister under s. 33(2),
which showed that in the view of the minister open land might include land on which some business was carried on. We
doubt very much whether it is right to construe the words of the section by reference to regulations made under powers
therein contained, especially when such regulations are directed solely to procedural matters such as conferring rights of
appeal in certain limited circumstances from an order of the local authority.

I would fully share that doubt in so far as the regulations are considered only as an instrument made under the Act. In the present
case, however, the argument has been carried at least one stage furtherit may be several stages furtherand our attention has
been called to the peculiar position arising under s 33(2) and reg 7, para (2), which create, as I have said, a composite instrument
consisting of provisions of the Act modified by provisions of the regulations. When that composite instrument is construed as
such, the modified provisions of s 33(4), applying to notices under s 33, must have their due effect on the interpretation of s
33(1).
There are one or two other points to be mentioned briefly. First of all, reference was made to the marginal note to s 33.
There again I ought to read 182 something that was said by this court in Stephens v Cuckfield Rural District Council ([1960] 2
All ER at p 720; [1960] 2 QB at p 383):

While the marginal note to a section cannot control the language used in the section, it is at least permissible to
approach a consideration of its general purpose and the mischief at which it is aimed with the note in mind.
Assuming that one can take into account the marginal note to that limited extent, I find that the marginal note here is in these
words: Power to require proper maintenance of waste land, etc This marginal note is on its own merits of no assistance
whatever in construing or understanding or applying s 23. It is clearly inaccurate in two important respects. The expression
waste land is not appropriate to include a garden. The expression Power to require proper maintenance is far from accurate,
because the power of requiring the removal or abatement of an injury to amenity falls very far short of a power to require proper
maintenance. I, therefore, derive no assistance for the present purpose from this marginal note, although it is fair to add that it
would help anyone who was looking for the section to find it.
There was a further point made by counsel for the local planning authority which I will mention briefly. He pointed out that
whereas one might find in certain earlier provisions a set of co-ordinated and interdependent provisions, one finds s 26 and
following sections introduced by the headnote Additional powers of control; and it is, I think, well established that it is
permissible to take a headnote into consideration. That is only a small matter herea small makeweightbut it tends to show
that one can construe s 33 to some extent by itself, and that its construction ought not to be controlled by the earlier provisions of
s 23, s 24 and s 25. Then it was also mentioned that the Caravan Sites and Control of Development Act, 1960, contains certain
provisions which we did not examine in any detail but which, it appears, may relieve hardship or cure defects which existed or
were created under the Act of 1947. If there were certain hardships arising under s 33, they can simply be accepted and
recognised as hardships which could be created under that Act but not of sufficient importance to affect the proper construction.
For those reasons, I agree that this notice H should be upheld as a valid notice, properly issued in exercise of the powers
conferred on the local planning authority by s 33 of the Act of 1947.

Appeal dismissed. Declaration that the notice was valid. Leave to appeal to House of Lords refused.

Solicitors: S Sydney Silverman (for the landowner); Sharpe, Pritchard & Co agents for Clerk to Buckinghamshire County
Council (for the local planning authority).

Henry Summerfield Esq Barrister.


183
[1963] 2 All ER 184

Bland v Cowan
LEISURE AND LICENSING

QUEENS BENCH DIVISION


LORD PARKER CJ, ASHWORTH AND WINN JJ
27 MARCH 1963

Gaming Betting Street or public place Receiving bets Negotiation of bets by agent of bookmakers in factory Betting
slips thrown over factory gate by agent to manager of bookmakers in street Manager frequented street for the purpose of
receiving betting slips Whether that was for purposes of receiving bets within Street Betting Act, 1906 (6 Edw 7 c 43), s 1(1).

B, an agent of the company of which the appellant was manager, negotiated bets in a factory. The appellant was wont to frequent
the street outside the factory gates, where B would throw to him a roll of paper over the gates. This roll contained the betting
slips which evidenced the bets made by punters in the factory with B on behalf of the company. The appellant was convicted of
frequenting the street for the purpose of receiving bets contrary to s 1(1) of the Street Betting Act, 1906 a. On appeal,
________________________________________
a The relevant part of s 1 is set out at p 186, letter h, post

Held In the context or paying or receiving or settling bets in s 1(1) of the Street Betting Act, 1906, the phrase receiving
bets referred to a financial transaction resulting from a betting contract, viz, receiving the losses resulting from the bets, and did
not refer to the physical receipt of slips of paper evidencing betting contracts, viz, in the present case, the betting slips (see p 187,
letters c and i, post); therefore the appellant was not guilty of the offence charged.
Appeal allowed.

Notes
The Street Betting Act, 1906, was repealed as from 28 March 1963, by the Betting, Gaming and Lotteries Act, 1963, s 57(1) and
Sch 8; for the replacing provisions of the new Act, see s 8(1), and cf s 52(3) as regards second or subsequent offences.
As to frequenting streets for the purposes of betting, see 18 Halsburys Laws (3rd Edn) 196, 197, para 388; and for cases on
the subject, see 25 Digest (Repl) 465, 363366.
For the Street Betting Act, 1906, s 1, see 10 Halsburys Statutes (2nd Edn) 778.
For the Betting and Gaming Act, 1960, s 3, see 40 Halsburys Statutes (2nd Edn) 333.

Cases referred to in judgments


Cassidy v Langmuir 1935 SC (J) 65, 25 Digest (Repl) 470, 346.
Williamson v Wright 1924 SC (J) 57, 25 Digest (Repl) 470, 344.

Case Stated
This was a Case Stated by justices for the county borough of Wolverhampton in respect of their adjudication as a magistrates
court sitting at Wolverhampton on 3 October 1962.
On 15 October 1962, the appellant Percy Hubert Bland was arrested by the respondent, William Crosby Cowan, a police-
constable, and charged with having on 15 October 1962, frequented a certain street called Marston Road in the county borough of
Wolverhampton on behalf of Lowe Brothers (Turf Accountants) Ltd bookmakers, of 364, Dudley Road, Wolverhampton, for the
purpose of receiving bets contrary to s 1 of the Street Betting Act, 1906, and s 6(1) b of the Betting and Gaming Act, 1960.
________________________________________
b Section 6(1) of the Act of 1960 increase the penalty for an offence under s 1(1) of the Act of 1906

The following facts were found. (a) The appellant was manager of the betting office of a firm of bookmakers, Lowe
Brothers (Turf Accountants) Ltd situate at 364, Dudley Road, Wolverhampton (hereinafter called the bookmakers) operating as
such with the necessary permit and licence under the Betting and 184 Gaming Act, 1960; (b) Frank Terence Brannon (hereinafter
called the agent) was an employee of Villiers Engineering Co Ltd and produced his card showing that he was authorised in
accordance with s 3(1) of the Betting and Gaming Act, 1960, to receive or negotiate bets as servant or agent of the bookmakers
and that he was in the habit of collecting and receiving bets from fellow employees within the factory of his employers at
Marston Road, Wolverhampton; (c) On 8 October, 9 October and 10 October 1962, the appellant drove into Marston Road in a
motor car, alighted from it and stood in the street outside the gates of the factory; (d) On each occasion the agent was on the
factory side of the gates of the factory and he threw or dropped over the gates to the appellant a roll containing slips of paper,
known as betting slips, bearing the names of horses running in races on the days in question which he had received from punters
within the factory. No money changed hands on any of these occasions; (e) On 15 October 1962, after the appellant had received
twenty-four betting slips in this manner he was arrested by the respondent and admitted receiving the betting slips but stated that
he did not realise he had committed an offence; (f) The slips bore the names of horses running in races at Dunstall Park,
Wolverhampton, on 15 October together with instructions regarding the terms of the bets being placed; (g) The appellant was
charged with the said offence and under caution replied Nothing to say; (h) The appellant collected or received the slips on
behalf of the book-makers and took them to their betting office; (i) No money changed hands on any of the occasions referred to
in the evidence given before the justices, and the agent accounted for the money later each day at the bookmakers offices; (j) ;
(k) At the conclusion of the prosecutions case a submission of no case to answer was made to the justices by the appellants
solicitor who contended that the betting slips had been passed out by an accredited agent within the factory and that they were not
bets received by or effected with the appellant; that the betting slips were being transmitted by the appellant who was acting in a
similar capacity to that of a postman taking bets through the post. The justices overruled this submission and the appellant
thereupon gave evidence and called witnesses on his own behalf; (1) The appellant admitted that he had carried out the practice
referred to in sub-paras (c), (d) and (e) hereof on numerous occasions by arrangement with the agent and that the latter collected
bets within the factory on behalf of the bookmakers; (m) The appellant produced a book of registered agents which included the
name of F Brannon (the agent); (n) The appellant admitted in cross-examination that the slips of paper which he received from
the agent were bets; (o) There was a conflict of evidence between the appellant on the one hand and his witnesses, the agent and
Thomas William Lowe (a director of the bookmakers) on the other, as regards the moment of time when the bets were regarded
as on or, as the justices understood that term, accepted as binding by both parties to the transaction; (p) The appellant
maintained that the bets were on or binding when they reached the bookmakers office whereas his said witnesses stated that
this occurred when the bets were received by the agent within the factory.
It was contended by the appellant (1) that the intention of the Street Betting Act, 1906, was to prohibit betting transactions
between a punter and a book-maker or bookmakers agent in a street; (2) that the bets had been legally received from punters by
the properly authorised agent within the factory; (3) that the bets were made when so received by the agent; (4) that the appellant
was merely acting in the role of a postman or messenger when collecting the betting slips for delivery to the office and that he
was not receiving bets within the meaning of s 1 of the Street Betting Act, 1906.
It was contended by the respondent (a) that the slips of paper bearing the names of horses were bets received by the agent in
the factory as such; (b) that their character had not been changed when they were received in Marston Road, Wolverhampton, by
the appellant; (c) that the appellant had frequented Marston Road and that he had received the bets from the agent whilst he, the
appellant, 185was in such road which was a street within the meaning of the Street Betting Act, 1906.
The justices were of opinion that (i) the appellant, whatever his purpose, was frequenting a street (Marston Road) on behalf
of the bookmakers as there was evidence of four specific occasions and reference to others. This was admitted by the appellant
and they understood that this point was not in dispute; (ii) the rolls or slips of paper or betting slips which the appellant received
from the agent who was inside the factory were bets and that the appellants reception of them in such street completed the
offence with which he was charged, as this was the purpose for which he frequented the street; (iii) in view of these findings they
considered that the question whether the appellant was acting merely as a postman or messenger was not material, and in this
connexion they had in mind that the position concerning postal betting was legalised by the repeal of the Betting Act, 1853, by s
1 of the Betting and Gaming Act, 1960, which did not however alter the situation regarding the offence under review; (iv) the
bets were on or binding on both parties when they were received by the agent within the factory but this did not affect their
findings referred to in (i) and (ii) above.
The justices, therefore, convicted the appellant of the offence charged and fined him 5, allowing him fourteen days for
payment. The appellant now appealed.
The cases noted below were cited in argument in addition to those referred to in the judgment c.
________________________________________
c Stovell v Jameson [1939] 4 All ER 76, [1940] 1 KB 92, R v Way, [1949] 2 All ER 365

D A McI Kemp for the appellant.


M K Harrison-Hall for the respondent.

27 March 1963. The following judgments were delivered.

LORD PARKER CJ. The appellant was the manager of the betting office of Lowe Brothers (Turf Accountants) Ltd
Bookmakers. The bookmakers quite lawfully had in the factory of Villiers Engineering Co Ltd an agent, a man called Brannon,
authorised for that purpose in accordance with s 3 of the Betting and Gaming Act, 1960. He would receive or negotiate bets in
the factory as servant or agent of the bookmaker. In order to ascertain what the agent had done, the appellant was wont to go
outside the factory gates, and there Brannon, the agent, would throw over the gates a roll containing slips of paper, which were
called the betting slips. Those bore the names of the horses, running in races on the day, which he had received from punters
within the factory. The appellant had done this, and admitted that he had done it, on many occasions, and there was little doubt
that he did frequent the street, the sole question being whether he frequented the street for the purpose of receiving bets. The
justices found that the bets were on in the sense that they were accepted as binding between punters and bookmaker in the
factory, and that when the betting slips were thrown over the gates, concluded contracts had been made. Nevertheless, the
justices took the view that the appellant was receiving bets when he obtained the betting slips over the factory gates, which by
that time had become the mere record of contracts made on behalf of the bookmakers by this agent Brannon. The section in
question, s 1(1) of the Street Betting Act, 1906, reads as follows:

Any person frequenting or loitering in streets or public places, on behalf either of himself or of any other person, for
the purpose of bookmaking, or betting, or wagering, or agreeing to bet or wager, or paying or receiving or settling bets
shall

be guilty of an offence.
As it seems to me, the sole question here is what is meant by receiving bets in that phrase or paying or receiving or settling
bets.
There are three possible meanings; the first is the one which I understand to 186 be that which the justices accepted, namely,
that the phrase receiving bets, covered the physical receipt of the slips of paper which evidenced the contracts made by
Brannon; the second possible meaning, which is the one advocated by counsel for the appellant, is that receiving bets means
accepting bets in the sense of accepting an offer and entering into a contract. If that be the true meaning, then on the finding of
the justices that it was Brannon within the factory grounds who had received bets in that sense and this appellant was not guilty
of the offence charged. Some support for that view is certainly to be obtained from the Betting and Gaming Act, 1960, which in
many places, and I need only refer for this purpose to s 3, uses the words receive bets within the phrase receive or negotiate
bets. Thus s 3(1) it is provided that:

No person shall by way of business receive or negotiate bets as servant or agent to another bookmaker or to the Board
unless

certain conditions are fulfilled. To the same effect are the words in sub-s (2), and indeed in the proviso to sub-s (1) of s 2, and in
other places throughout the Act of 1960. The third possible meaning, and the one which I think is correct, is that receiving bets
in the context of sub-s (1) of s 1 of the Act of 1906 is referring to a financial transaction resulting from the contract which has
been entered into. It will be observed in the first place that the earlier part of that subsection is dealing with the formation of the
contract:

Any person frequenting or loitering for the purpose of bookmaking, or betting, or wagering, or agreeing to bet or
wager,

whereas the words that come after the comma and within the one phrase or paying or receiving or settling bets, are pointing to
a later period of time when not only the contract has been negotiated and formed, but also the transaction is being wound up.
It seems to me that the phrase is dealing in the first place with paying the amount of winnings of the punter; secondly, with
receiving the amount of losses incurred by the punter; and, thirdly, with settling the account between them, that latter provision
covering the case where money does not pass but the matter becomes one of account or cross account. It seems to me that in the
context of s 1(1) of the Act of 1906, which is quite a different context to that in which the phrase receive or negotiate bets
appears in the Act of 1960, the phrase is dealing purely with the financial settlement once the contract is entered into and the
event has been determined. That, as I say, seems to me the most natural meaning, but it is to be observed that in two cases in
Scotland it has been assumed, without being decided, that somebody who, like the appellant in the present case, is acting purely
as a messenger to convey a record of a betting transaction entered into, to his principal, is guilty of receiving bets. The two cases
are, first, a case in 1924, Williamson v Wright; the other is the case of Cassidy v Langmuir. In both those cases the real question
was whether the messenger, who was operating in a motor car, could be said to be frequenting or loitering in the street; it was
assumed in both cases and never argued that, if he was frequenting or loitering, it was for the purposes of receiving bets.
At the same time Lord Anderson in the former case said (1924 SC (J) at p 60):

That he was in the burgh for the purpose of receiving bets, and that he did receive bets on that day I have no doubt
whatever. And I think that the magistrate was well justified in drawing the inference that bets were received on that
occasion.

He alone of the members of the court dealt with the point at all; it was never argued; it had been assumed. For may part, for the
reasons which I have stated, I think that the natural meaning of receiving bets in this context is to receive the losses resulting
from the bets.
187
I would add only that it may be, and I am not deciding the question, that this appellant could have been properly charged
with frequenting this street on behalf of his principals, Messrs Lowe, for the purpose of bookmaking, words which are very
wide indeed and would cover, I conceive, every aspect connected with a bookmakers business, including the knowledge of
betting contracts entered into by an agent, which knowledge he might well require in order to lay off some of the bets. In these
circumstances I have come to the conclusion that the justices were wrong, and that this appeal succeeds.

ASHWORTH J. I agree.

WINN J. I agree.

Appeal allowed.

Solicitors: Gibson & Weldon agents for Challinor & Roberts, Smethwick (for the appellant); Sharpe, Pritchard & Co agents for
Town clerk, Wolverhampton (for the respondent).

N P Metcalfe Esq Barrister.


[1963] 2 All ER 188

Re Michelhams Will Trusts


TRUSTS

CHANCERY DIVISION
BUCKLEY J
21, 22 MARCH 1963

Trust and Trustee Variation of trusts by the court Restraint of marriage Arrangement involving Swiss contracts that might
operate in restraint of marriage Whether arrangement should be approved Variation of Trusts Act, 1958 (6 & 7 Eliz 2 c 53), s
1.

Property was held on trust in two equal shares, subject to an annuity to Lord M for life, as to one share for Lord M for life and
after his death for any wife of his for her life, and thereafter for any child or children of Lord M as he should appoint or, in
default of appointment, for any child or all children of his equally and, subject thereto, for Lord M absolutely. The other share
was held on similar trusts in favour of Lord Ms brother, and any wife, child or children of his. Lord M and his brother applied
for variation of the trusts. Lord M was sixty-five and his brother was fifty-nine years of age. Neither was married at the time of
the application, and each deposed that he had no wish to marry; and neither applicant had had children. Approval was sought on
behalf of any future wife or unborn child of either of the applicants for an arrangement whereby trust property should be
transferred in agreed proportions to the applicants absolutely, insurance policies being effected which would ensure that, if either
applicant married, certain sums would become available to replace the funds so transferred. The policies included a stipulation
that the insurers should be indemnified by a Swiss bank if the policy moneys became payable. Both applicants lived in
Switzerland. The Swiss bank proposed to give the indemnity on terms that it, in turn, should be indemnified by one or other of
the applicants in case the policy moneys became payable.

Held Although by virtue of the counter-indemnities given by the applicants to the Swiss bank the scheme might be regarded as
tending to discourage the applicants from marrying, yet the doctrine of public policy, whereby a contract in restraint of marriage
was unenforceable, did not affect the validity of the proposed arrangement, for, even if the counter-indemnities were not
enforceable by the Swiss bank (which was a question of Swiss law), that would not relieve the Swiss bank from its obligation to
indemnify the insurers; accordingly the court would approve the arrangement under the Variation of Trusts Act, 1958, s 1, on
behalf of any future wife or unborn child of either applicant, the arrangement being, on the facts, calculated to benefit such
possible beneficiaries (see p 191, letter i, and p 193, letter a, post).
188

Notes
As to the jurisdiction to vary trusts, see 38 Halsburys Laws (3rd Edn) 10291031, para 1772.
For the Variation of Trusts Act, 1958, s 1, see 38 Halsburys Statutes (2nd Edn) 1130.

Cases referred to in judgment


Baker v White (1690), 2 Vern 215, 23 ER 740, 12 Digest (Repl) 278, 2133.
Hartley v Rice (1808), 10 East, 22, 103 ER 683, 12 Digest (Repl) 278, 2137.
Lowe v Peers (1768), 4 Burr 2225, Wilm 364, 98 ER 160, 12 Digest (Repl) 278, 2131.

Adjourned Summons
This was an application by originating summons dated 4 May 1960, by the Right Hon Herman Alfred, second Baron Michelham
of Hellingly, and the Hon Jack Herbert Michelham who were beneficially interested under the will dated 27 September 1918, of
the first Baron Michelham, deceased, a settlement dated 22 October 1926, and the will dated 19 November 1925, of Aimee
Geraldine Almy, deceased. The application was that the court might, pursuant to the Variation of Trusts Act, 1958, approve, with
or without modification, on behalf of any future wife and unborn issue of either of the applicants who might become interested
under the trusts of the above-mentioned wills and settlement, an arrangement varying the trusts thereof. The respondent was
Coutts & Co the sole trustee of the wills and settlement. The facts appear in the judgment.
The case noted belowa was cited in argument in addition to those referred to in the judgment.
________________________________________
a Robinson v Ommanney (1883), 23 Ch D 285

R W Goff QC and G B H Dillon for the applicants.


E I Goulding QC and M J Fox for the respondents.

22 March 1963. The following judgment was delivered.

BUCKLEY J. This is an application under the Variation of Trusts Act, 1958, to vary the trusts of the will of the first Lord
Michelham and of a settlement dated 22 October 1926, which was made as a consequence of certain compromise negotiations
which took place in litigation following Lord Michelhams death, and to vary the trusts of the will of the lady who was Lord
Michelhams wife, who subsequently remarried and became Mrs Almy. The only persons who at present have any beneficial
interests under any of those trusts are the two applicants, the present Lord Michelham and his brother, Mr Michelham.
Under the will of the first Lord Michelham, his residuary estate is held, in the events which have happened, on trust to pay
an annuity to the present Lord Michelham during his life, and, subject thereto, the residue is held in equal shares for Lord
Michelham and his brother, each share being settled and held on trust either for Lord Michelham or for his brother, as the case
may be, for life with remainder in each case to any wife of the life tenant who may survive him during her life. Then there is a
power for Lord Michelham or his brother, as the case may be to appoint among his children or issue, and a trust in default of
appointment in the ordinary form for his children who attain the age of twenty-one or, being female, marry, with an ultimate trust
on failure of all the preceding trusts for Lord Michelham or his brother, as the case may be, absolutely.
Under the settlement, certain chattels were settled, which are now mainly represented by investments of the proceeds of sale
thereof, although some of the chattels remain unrealised. There are two funds of settled property under the settlement, one of
which is called the pictures fund and the other the works of art fund. The pictures fund is subject to a trust for Lord
Michelham for life, with remainder to his sons successively in tail male, with remainder to Mr Michelham for life, with
remainder to his sons successively in tail male. If those trusts fail, the fund will belong to Lord Michelham and Mr Michelham in
equal shares in consequence of the provisions of their mothers will. The works of art fund is held on trust for Mr Michelham for
life, with remainder to his sons successively in tail male, with remainder to Lord Michelham for his life, with remainder 189 to
his sons successively in tail male, and, if all those limitations fail, that fund will be divisible between Lord Michelham and his
brother equally.
Under the will of Mrs Almy, there is a fund which is held on trusts identical with the trusts of the pictures fund.
Lord Michelham is now sixty-two years of age. Mr Michelham is now fifty-nine years of age. Lord Michelham has been
married. His wife died in 1961, but in fact Lord Michelham and his wife had been living apart for many years before that. There
is evidence that Lord Michelham has no wish or desire ever to remarry, and that there is no likelihood that he will ever do so. Mr
Michelham is a bachelor, and he has deposed to the fact that he has no wish or desire ever to marry and that there is no likelihood
that he will ever do so. If those present intentions are maintained, the only persons who will ever have an interest in this fund are
the two applicants. But there is, of course, always the possibility that one or other of the applicants might change his mind, in
which case he might leave a widow surviving him who would take an interest in his settled share of his fathers residuary estate,
and he might leave issue interested under the trusts of the will of the first Lord Michelham, or a son who would take an interest
under the trusts of the settlement or of Mrs Almys will.
In these circumstances, a scheme has been propounded, the effect of which I can state quite shortly. Each of the funds with
which I am concerned is at present invested partly in investments which are exempt from tax so long as the person interested in
them is domiciled and usually resident out of this country, and partly in investments which are not so exempt. There are pictures,
which are subject to the trusts of the pictures fund, which are valued at a little over 3,000. There are other works of art, which
are subject to the trusts of the works of art fund, which are valued at rather over 17,000. Those pictures and works of art are
thought to be such as would be exempt from duty, in any event if they were not sold following the death of the beneficiary
entitled to an interest in them. The greater part of the investment representing each of the funds is in investments which are not
exempt, except in the case of the fund subject to the trusts of Mrs Almys will, where the greater part of the fund is in exempt
securities and a smaller part is in securities that are not exempt. It is proposed that all this property should be transferred, in
proportions which have been agreed between Lord Michelham and his brother, to them absolutely, but that insurance policies
should be effected which would ensure that, in certain events, certain sums would become available to replace the funds so
handed over to them. The amounts to be secured by those policies have been calculated in this way. The prospective rate of duty
payable on the death of each of the applicants has been estimated, and that part of each fund which is invested in securities that
are not exempt from tax in the way which I have mentioned has been assumed to be liable to duty at that rate, the remainder of
the fund which is in exempt securities being assumed not to be liable to duty. The amount of the fund which would remain after
payment of duty if the present tenant for life were to die forthwith has been ascertained, and that net sum has been written up by
ten per cent to arrive at the figure to be secured by the relevant insurance policies. So the policies are designed to ensure
repayment to the trustees of a fund ten per cent more than would be left if the tenant for life were to die today and if that duty
were only to be exacted on that part of the trust fund which is invested in securities that are not exempt.
The policies are to be payable in the following events. The policies taken out to secure repayment in respect of the two
settled shares of the testators residuary estate are to mature in the event of Lord Michelham, in the one case, and Mr Michelham
in the other case, marrying at any time after the scheme comes into operation. The policies that are intended to deal with the
position in respect of the pictures fund and the works of art fund, and the fund subject to the trusts of Mrs Almys will, will be
policies that will mature in any of the following events (whichever shall first happen), that is to say, the death of Lord Michelham
having had a son, the death of Mr Michelham having had a son, the birth of a 190 posthumous son to Lord Michelham, or the
birth of a posthumous son to Mr Michelham.
Now there are two points which have been ventilated on the scheme. One relates to the quantum of the sums to be secured
by the policies; the other is a question of general application whether there is some element in this scheme which, because of a
tendency in the scheme to discourage Lord Michelham or Mr Michelham from marrying, might conflict with public policy.
I will deal, first of all, with the question of quantum. The present position with regard to Lord Michelham and Mr
Michelham in relation to their domicil and residence is this. Lord Michelham has for upwards of twenty years resided in Geneva.
He has had business interests in many parts of the world. He has not had any home in this country for many years. There is
evidence before me which affords good grounds for concluding that the right view today is that his domicil is Swiss, and,
although, if the issue of domicil were ever contested in the future, other evidence might be produced which would result in a
different conclusion, I am content to proceed on the footing that Lord Michelhams domicil should, for present purposes, be
regarded as being in Switzerland. Mr Michelham has recently acquired a residence in Geneva. Previously his life has been
nomadic as regards residence. But it appears that he has now got a settled home in Geneva, and, in his case also, the evidence
satisfies me that I can proceed to consider this scheme on the footing that he is at present domiciled in Switzerland. Each of them
says that his intention is to continue to reside in Switzerland, and that he has no intention of making a home anywhere else. But,
of course, that intention might change hereafter, and, if either of these applicants were to lose his domicil of choice in Geneva, as
the law stands at present his domicil of origin would revive unless he acquired some other domicil of choice, and the domicil of
origin of each of these gentlemen is in this country. Moreover, without a change of domicil, either of them might cease to be
resident in Switzerland and might become resident in this country. And those possibilities might be greater than they are today if
the only event with which I am really concerned, that is to say, either of them marrying, were to occur, for it might be that, for
reasons connected with the marriage, there would be a change of heart about domicil or about residence.
As matters stand, if either Lord Michelham or his brother were to die, that part of each fund which is invested in exempt
securities would escape duty, and, on that footing, the amount to be provided in the events with which I am concerned by the
policies would be more than what would be left of the funds in which that applicant is interested, if he were to die forthwith and
duty were to be exacted at the rate which is expected.
It was pointed out by counsel who appeared for the trustees that, if either Lord Michelham or his brother were to marry, the
trustees might feel obliged to alter their investment policy and invest a larger proportion of the funds in exempt securities, in
which case the margin of benefit provided by the sums secured by the policies would probably be decreased and might even be
eliminated altogether.
I have got to look at this scheme as a whole, and at the probable events as a whole, and I have not got to consider whether
the scheme is bound to confer a benefit on everyone who may become interested under the trusts in every event, but whether in
the probable events it is calculated to confer a benefit. It seems to me that I have got to take into consideration the probable
investment policy of the trustees in the future, and the possibility either that the domicil of one or other of these applicants might
be established not to be Swiss, or that there might be a change of domicil or there might be a change of residence such as to put
an end to the exemption in respect of the exempt parts of the fund. Balancing all the considerations it is fair to say that the
scheme is one which is calculated to confer a benefit on possible unascertained or unborn beneficiaries and, on the quantum
aspect of the matter, I am satisfied, although this is perhaps a case which is a little nearer to the line than some.
191
I turn now to the question of public policy. A contract which is in general restraint of marriage is unenforceable on the
grounds of public policy, whether it contains an express undertaking that the subject will not marry, or whether it is merely a
contract of a kind which, on financial or other grounds, will tend to discourage the subject from marrying. Thus, in Baker v
White a widow who gave a bond of 200 payable in the event of her remarrying was held to be entitled to have such bond
delivered up and cancelled; in Lowe v Peers where the defendant had covenanted not to marry any woman other than the plaintiff
and to pay the plaintiff 1,000 in the event of his marrying any other woman, he was held not to be liable to make the payment
notwithstanding that he had married another woman on the ground that the covenant was illegal as being a restraint on
matrimony. See also Hartley v Rice, where the plaintiff had entered into a wager with the defendant that the plaintiff would not
marry within six years. The first and third of these cases were cases which did not involve a direct obligation not to marry, but
tended, for financial reasons, to discourage marriage, and they were decided on the ground that for that reason the court would
not countenance the contract because it offended against the public interest.
Before I develop this, I should mention another fact. The terms on which the insurers are prepared to grant the policies
payable in the event of Lord Michelham and Mr Michelham marrying include a stipulation that they shall be indemnified in the
event of the policy moneys becoming payable under either of these policies. It is proposed that they shall be indemnified by a
Swiss bank, and the proposed form of the deed of indemnity has been produced. It contains a recital that the Swiss bank is
prepared to grant the indemnity on terms that the Swiss bank is itself indemnified in the one case by Lord Michelham and in the
other case by Mr Michelham. It is this counter-indemnity which gives rise to this question of public policy because it is
suggested that, looking at the scheme as a whole, it is one which involves Lord Michelham or Mr Michelham undertaking a
financial liability which will mature in the event of his marrying, and therefore it is suggested that the arrangement is one which
conflicts with the principle of public policy that I have mentioned. It is said that the counter-indemnity to be given to the Swiss
bank must tend to discourage Lord Michelham and Mr Michelham from marrying.
The trustees, who are not opposing this application but thought it their duty to bring this point to the notice of the court,
posed this question: whether, having regard to the scheme as a whole, the insurers might not legitimately, and I say
legitimately for I am not concerned with ethics but with legal obligation, resist a claim under the policies which are the subject-
matter of these indemnities because of this element in the scheme? The policies will be issued in response to proposals by Lord
Michelham and Mr Michelham, and they will pay the premiums out of their own resources. There will be no consideration
moving from the trustees to the insurers, but the policies will be under seal, and no difficulty can arise on this ground in respect
of the trustees power to sue on the policies. As between the insurers and Lord Michelham and Mr Michelham, however, the
premiums will not be the only consideration, for, as I have indicated, Lord Michelham and Mr Michelham are to procure the
indemnities of the Swiss bank. What arrangements the Swiss bank may choose to make to protect itself seem to me to be of no
interest to the insurers and no concern of theirs. The nature of any such arrangements which the Swiss bank may see fit to make
could not qualify in any way the obligation of the Swiss bank to indemnify the insurers, and a fortiori I think it could not qualify
the obligation of the insurers to pay under the policies. Even if I assume that the counter-indemnities to be given by Lord
Michelham and Mr Michelham would be unenforceable by the Swiss bank, this would not, in my judgment, relieve the Swiss
bank of liability or the insurers of liability under the policies. But I have no reason to suppose that the Swiss 192 bank would be
unable to enforce the counter-indemnities. These will, no doubt, be governed by Swiss law, and I think that I am entitled to
assume that the Swiss bank will satisfy itself that whatever protection it requires will be protection of which it can avail itself.
For these reasons, I think that, although it is quite proper that the trustees should bring the matter to the attention of the court, the
case is not one in which the doctrine of public policy can in any way affect the validity or effectiveness of the scheme.
In these circumstances, I do not think that it is necessary for me to deal with the various other points that were touched on in
the argument, but I will just mention what they were. First, it was suggested that, Lord Michelham having been already married,
the doctrine of public policy was not one which would affect him, and there is some support for that view in some of the
authorities that were read to me. Secondly, it was suggested that Mr Michelham, who many years ago acquired Cuban nationality
and still retains that nationality, was not a person to whom the doctrine would apply. And, thirdly, it was suggested that, by
analogy to the reasoning which has been employed in some of the cases in which a similar doctrine has been applied to
dispositions of property either under wills or settlements where interests have been made subject to defeasance on marriage, it is
relevant to look at the intention of the parties generally and see whether the arrangement is one which is really directed to
preventing marriage or whether it is directed to some other proper and prudent object. If that were a relevant consideration in the
present case, and I do not say whether it is or is not, it seems to me that the aim of the present arrangement is one which can be
accepted as proper, for it is in essence this: the trustees are being asked to distribute to Lord Michelham and his brother a fund
which they can only distribute by committing a breach of trust, subject, of course, to the protection which the consent of the
beneficiaries and the order of this court will give them. The object of the scheme, so far as the insurance policies and the
indemnities that will support them are concerned, is that Lord Michelham and his brother will replace in the hands of the trustees,
or procure to be reimbursed to the trustees, some part of what they are getting in that way in the event of any other persons
coming into existence who might have a beneficial interest under the trusts, and, in those circumstances, it would be only right
and proper that they should make such reimbursement. So that it seems to me that the scheme is one which is a very proper one
for them to enter into, even if it does involve some obligations which may be said to tend to discourage either of them from
marrying.
In these circumstances, I am prepared to approve the arrangement.

Order accordingly.

Solicitors: Herbert Oppenheimer & Vandyk (for the applicants); Farrer & Co (for the respondents).

Jenifer Sandell Barrister.


193
[1963] 2 All ER 194

Cope v United Dairies (London) Ltd


ADMINISTRATION OF JUSTICE; Legal Aid and Advice

QUEENS BENCH DIVISION


MEGAW J
15 MARCH 1963

Legal Aid Costs Dismissal of action for want of prosecution Legal aid certificate discharged before dismissal of action
Whether there had been a trial or hearing of action Whether defendants entitled to costs incurred prior to discharge of assisted
persons certificate Legal Aid and Advice Act, 1949 (12 & 13 Geo 6 c 51), s 2(2)(e) Legal Aid (General) Regulations, 1962
(SI 1962 No 148), reg 18(1).

In April, 1961, a legal aid certificate was issued to the plaintiff to bring an action in the High Court against the defendants for
damages for personal injuries. A writ was issued and pleadings delivered. In April, 1962, his legal aid certificate was discharged,
and he thereafter took no further steps in the action. On the defendants application by summons the master made an order in
June, 1962, on the ground of want of prosecution that the action be dismissed with costs, unless the plaintiff issued a summons
for directions within twenty-one days, and that the defendants costs be taxed and paid by the plaintiff. The defendants bill of
costs, brought in for taxation, included the whole of their costs of the action. On taxation all costs referable to the period between
the granting and the discharge of the legal aid certificate were disallowed. By s 2(2)(e) a of the Legal Aid and Advice Act, 1949,
where a person receives legal aid in connexion with any proceedings, his liability by virtue of an order for costs made against him
with respect to the proceedings is not to exceed the amount, if any, which it is reasonable for him to pay; and by reg 18(1) of the
Legal Aid (General) Regulations, 1962 b, where such an order is made, the determination of the amount of liability in accordance
with s 2(2)(e) is to be made at the trial or hearing of the action. On application for an order to review taxation,
________________________________________
a Section 2(2)(e) is set out at p 196, letter e, post
b The material terms of reg 18(1) are set out at p 197, letter a, post

Held (i) The determination of costs in an action to which an assisted person was a party involved two phases, viz, (first) the
determination by the court in its discretion whether an order for costs should be made against the assisted person, and (second) a
separate determination of the amount of the liability of the assisted person for costs (see p 198, letters a and b, post); the second
phase was required by reg 18(1) of the Legal Aid (General) Regulations, 1962, to be done, and could be done only, at the trial or
hearing of the action, but, since on the true construction of reg 18 the coming into effect of the dismissal of the action under the
order of June, 1962, was not a trial or hearing of the action within reg 18(1), there was no jurisdiction, either at the time when
the order of June, 1962, was made or subsequently, to make a determination of the amount of the plaintiffs liability for costs
incurred during the currency of his legal aid certificate (see p 197, letter g, and p 198, letters d and g, post).
(ii) The order of June, 1962, however, on its true construction, neither determined the amount of the plaintiffs liability for
costs during the currency of the legal aid certificate, nor required the taxing master to assess such costs, and accordingly the
taxing master could not properly allow any costs against the plaintiff in respect of that period (see p 198, letter g, post).
Application dismissed.

Notes
As to the award of costs against an assisted person, see 30 Halsburys Laws (3rd Edn) 502, 503, para 933.
For the Legal Aid and Advice Act, 1949, s 2, see 18 Halsburys Statutes (2nd Edn) 535.
For the Legal Aid (General) Regulations, 1962, reg 18(1), see 5 Halsburys Statutory Instruments (1st Re-issue) 243.
194

Cases referred to in judgment


Blatcher v Heaysman [1960] 2 All ER 721, [1960] 1 WLR 663, 3rd Digest Supp.
Wozniak v Wozniak [1953] 1 All ER 1192, [1953] P 179, [1953] 2 WLR 1075, 3rd Digest Supp.

Review of taxation
The defendants applied under r 35 of the Supreme Courts Cost Rules, 1959, for a review of the taxation of their costs in an action
for personal injuries commenced against them by the plaintiff in the Queens Bench Division which was dismissed by Master
Ritchie for want of prosecution on 8 June 1962. The master ordered that the defendants costs be taxed and paid by the plaintiff.
Megaw J, heard the application in chambers on 14 March 1963, and delivered judgment in open court at the request of the
defendants. The facts are set out in the judgment.
The cases noted belowc were cited during the argument in addition to those referred to in the judgment and were supplied by
the courtesy of counsel.
________________________________________
c Arnott v Amber Chemicals Ltd (19 May 1953), The Times, 20 May 1953, MacCarthy v Agard, [1933] All ER Rep 991, [1933] 2 KB 417,
Wilson v Metcalfe, 1826), 1 Russ 530

J H Hames as amicus curiae.


A Lipfriend and Eleanor Platt for the defendants.

Cur adv vult

15 March 1963. The following judgment was delivered.

MEGAW J read the following judgment. On 26 April 1961, a legal aid certificate was issued to the plaintiff in respect of
proceedings by him against the defendants, claiming damages for personal injuries said to have been sustained while he was
employed by the defendants. A writ was issued and pleadings were delivered. On 27 April 1962, the plaintiffs legal aid
certificate was discharged. The plaintiff thereafter took no further steps in the action, but no application was made by him or on
his behalf for discontinuance. On 4 June 1962, a summons was issued by the defendants solicitors in the following terms. It
asked for

an order that this action be dismissed with costs for want of prosecution, the plaintiff having failed to issue a summons
for directions pursuant to R.S.C., Ord. 30, r. 1, and that the defendants costs be taxed and paid by the plaintiff.

The summons was heard by Master Ritchie on 8 June 1962. The plaintiff did not appear and was not represented. The
defendants were represented by their solicitors, who fairly and properly informed the master that the plaintiff had had a legal aid
certificate and that it had been discharged. The master made an order in the following terms:

It is ordered that, unless the plaintiff issues a summons for directions within twenty-one days from the date of this
order, this action be dismissed with costs for want of prosecution and the defendants costs be taxed and paid by the
plaintiff, and that the costs of this application be the defendants costs in any event.

The plaintiff was thereafter notified of the order and of the intended taxation of costs.
When the defendants solicitors brought in their bill of costs for taxation, they included therein the whole of the costs of the
action, including the costs incurred by them during the period when the plaintiff had his legal aid certificate. The taxing officer
refused to allow any items in the bill referable to the period between the granting and the discharge of the legal aid certificate.
The balance which he allowed, referable to the period after the discharge of the legal aid certificate, was 46 5s 8d. The
defendants raised objections to the disallowances. On review, the senior taxing master overruled the objections. The defendants
now ask for a review under r 35 of the Supreme Court Costs Rules, 1959. I have 195 been very much assisted by the careful and
able arguments presented yesterday in chambers by counsel for the defendants, and by counsel who was instructed by the Law
Society on my suggestion that it would be helpful if counsel were to be instructed as amicus curiae, in view of the nature of the
issues raised. At the request of the defendants, I adjourned the application into open court for judgment.
The defendants submit that the senior taxing master exceeded his jurisdiction. They say that the order made by Master
Ritchie was clear and unambiguous; and that, even if the taxing master thought that it was wrong or ultra vires, he had no option
but to comply. He was not entitled to disallow certain items on the ground that they were wrongly or irregularly made the subject
of taxation. If the order were wrong, the only remedy would have been by way of appeal from that order to the judge in
chambers. No such appeal had been made. The defendants further submitted that Master Ritchies order was not wrong,
irregular or ultra vires.
Counsel for the Law Society agreed that the taxing master could not properly refuse to carry out an order for taxation, in
whole or in part, because he considered it to be wrong or ultra vires, and that the same applies to this court on a review. With that
proposition, I agree. Counsel instructed by the Law Society submitted, however, that what the senior taxing master did in this
case was right; not because he had any jurisdiction to refuse to carry out a taxation as directed, but because, in the circumstances,
on the true construction of Master Ritchies order of 8 June 1962, it required taxation only of those costs which were incurred
outside the period during which the legal aid certificate was in force. In my judgment, that contention is right.
Section 2(2) of the Legal Aid and Advice Act, 1949, provides as follows:

Where a person receives legal aid in connexion with any proceedings (e) his liability by virtue of an order for costs
made against him with respect to the proceedings shall not exceed the amount (if any) which is a reasonable one for him to
pay having regard to all the circumstances, including the means of all the parties and their conduct in connexion with the
dispute.

Section 2(3) provides:

Regulations shall make provision as to the court, tribunal or person by whom the amount referred to in para. (e) of the
last foregoing subsection is to be determined and the extent to which any determination thereof is to be final.

Regulation 13(1) of the Legal Aid (General) Regulations, 1962, which deals with the position of a person who has once been
legally aided but whose certificate has, as here, been discharged, provides:

Subject to the provisions of this regulation, a person whose certificate is revoked shall be deemed never to have been
an assisted person in relation to the claim or proceedings to which the certificate related, and a person whose certificate is
discharged shall, from the date of discharge, cease to be an assisted person in the claim or proceedings.

That paragraph is, however, qualified as regards costs by reg 13(6). The relevant part of that paragraph provides:

Where a certificate has been discharged and where he [that is the person who had the certificate] continues to
assert or dispute the claim or to take, defend or be a party to the proceedings to which the certificate related (b) those
provisions of s. 2 of the Act which relate to an assisted persons liability by virtue of an order for costs made against him
shall apply in so far as the costs were incurred while he was an assisted person.

The effect is that, in relation to the plaintiffs liability for the defendants costs incurred in the period in which he was an assisted
person, he is to be treated, for 196 the purposes of s 2(2)(e) and s 2(3) of the Act, and reg 18 made thereunder, as an assisted
person. Regulation 18(1) is as follows:

Where an order for costs is made against an assisted person, the determination of the amount of his liability for costs
in accordance with s. 2(2)(e) of the Act shall be made at the trial or hearing of the action, cause or matter.

Then there is a proviso which is here irrelevant. I should also read reg 18(5):

Where an assisted person serves notice of discontinuance or where an order for costs is made against him by reason of
his default of appearance or defence or by reason of summary judgment, or upon application by him for leave to
discontinue, he shall be liable for the full amount of the costs, unless he makes an application for determination of the
amount of his liability under the provisions of s. 2(2)(e) of the Act, in which case the court may make any such order for
payment as may be made in determining an assisted persons liability by virtue of an order for costs made against him at a
trial or hearing of an action, cause or matter, and shall to the same extent be final.

That paragraph has no direct application here, first, possibly, because it applies to an assisted person and not to one who has
ceased to be an assisted person; and, secondly, because, perhaps by inadvertence, it does not refer to the termination of
proceedings by an order for dismissal for want of prosecution. There is thus no provision, in the circumstances of this case,
making the plaintiff liable for the full amount of the costs unless he makes application for the determination of the amount of his
liability under s 2(2)(e) of the Legal Aid and Advice Act, 1949. It may well be that such a provision would not be appropriate in
the case of a person whose certificate has been discharged, since he might have no legal advice and could not fairly be expected
to know of, and assert, his right to seek a reduction of his liability for the full amount of costs incurred while he was an assisted
person. Regulation 18(5) is, however, relevant in one way to the present issue. It throws light on the meaning of the trial or
hearing of the action, cause or matter in reg 18(1). Paragraph (5) uses those same words in, I think, deliberate contrast with
other methods of putting an end to proceedings, such as discontinuance, default of appearance or summary judgment. That
indicates that the dismissal of an action for want of prosecution is not comprehended within the words trial or hearing of the
action, cause or matter in reg 18(1). It follows that there never has been a trial or hearing of the action in this case. Therefore,
the mandatory provision of reg 18(1) for the determination of the amount of the plaintiffs liability for costs incurred during the
period covered by the legal aid certificate could not be complied with.
Counsel for the defendants submitted that, although there was not a trial or hearing of the action on 8 June 1962, when
Master Ritchie made the order for dismissal of the action at the end of twenty-one days, unless the plaintiff meanwhile should
issue a summons for directions, nevertheless there was a notional trial or hearing of the action when the twenty-one days
expired and the condition subsequent was fulfilled. He said that trial or hearing of the action meant no more than the final
determination of the matter. He pointed out that, if it were otherwise, there would be real difficulties in the way of a non-
assisted person, in circumstances such as the present, obtaining a determination of the amount of costs payable by the plaintiff in
respect of the legal aid certificate period. He said that the suggestion of counsel for the Law Society that the defendants could
have themselves issued a summons for directions and obtained directions and thereafter a judgment by default, was unrealistic. It
would necessarily involve the incurring of further, probably irrecoverable, costs. That may, indeed, be so on the regulations as
they stand, and it may well be unsatisfactory. But I am unable to hold that the coming into effect of the dismissal of the action
twenty-one days after the order of 8 June 1962, was a trial or hearing of the action. It is inconsistent with any ordinary, or, I
think, 197any possible, meaning of the words. It is inconsistent with the indication of the meaning of the words to be derived
from reg 18(5).
Counsel for the Law Society, rightly as I think, submits that, where there is a determination of costs in an action to which an
assisted person is a party, that determination involves two phases, though they sometimes take place at the same time. The first
phase is the determination by the court, in its discretion, whether or not it is proper that an order for costs should be made against
the assisted person. This phase, by reason of s 1(7) of the Act, is carried out without reference to any consideration as to the party
being, or having ever been, an assisted person. The second phase, which arises in relation to a person who is, or has been,
assisted, is the separate determination as to the amount, if any, of the liability for costs which the assisted person should be
ordered to pay. Frequently, in fact, these two phases fall to be dealt with at different stages. Thus, where costs are awarded
against an assisted person on an interlocutory application, the amount must not be assessed at that stage. This is clearly
established by Wozniak v Wozniak. The doctrine of the two phases is implicit in the judgment of the Court of Appeal in Blatcher
v Heaysman. It is reflected in the opening words of reg 18(1), and, I think, in s 2(2)(e) of the Act.
For the reasons which I have given, if Master Ritchie purported to make an order determining the amount of the plaintiffs
liability for costs in respect of the period when he had a legal aid certificate, he had no jurisdiction to do so, because this could
only be doneand had to be doneat the trial or hearing of the action. Further, if he purported to do so, I should not regard
an order for costs in the form of the order made by the learned master as constituting a determination of the amount of the
plaintiffs liability for costs in respect of the legal aid certificate period. There is, in the order, no determination of amount. If,
indeed, that was the meaning and effect of the order, the taxing master would have been wrong in failing to carry it out, and I
should have had no power to uphold the taxing masters decision. The plaintiffs only remedy would be to seek leave to appeal
out of time and to ask that the order as to costs be set aside or varied. However, I agree with the submission of counsel for the
Law Society that that was not the meaning or effect of Master Ritchies order. All that the master was asked to do by the
defendants summons, and all that he did and purported to do by his order was, first, to make the general order as to costs,
referable to the period subsequent to the discharge of the legal aid certificate; and, secondly, in relation to the costs of the period
covered by the legal aid certificate, to deal with the first phase, with which alone he had jurisdiction to deal. He was not invited
to make, and could not properly make, and did not purport to make, any determination of the amount of the plaintiffs liability for
costs incurred during the currency of the legal aid certificate. Such a determination has never been made, and, in view of the
course of the proceedings, could never have been made under the regulations as they now are. The senior taxing master,
therefore, has properly given effect to Master Ritchies order in accordance with its true meaning. He could not himself
determine the amount of the costs during the legal aid period, and he could not properly allow any such costs on taxation, since
they had never been assessed as required by the Legal Aid and Advice Act, 1949, and the Legal Aid (General) Regulations, 1962.
Master Ritchies order did not require him to do so.
I cannot accept counsel for the defendants argument that, prima facie, an assisted person is obliged to pay the whole costs,
and that it is up to him, if he sees fit, to invoke the special protection given by the Act of 1949 to an assisted person. Indeed,
there was no time in the course of these proceedings when he could have required such a determination. The special protection is
the responsibility of the court, whether or not the assisted person, or previously assisted person, invokes it. It is only by virtue of
the special provisions of reg 18(5) that 198 it ever falls on the assisted person to invoke that protection. Regulation 18(5) is not
applicable here. Accordingly, though perhaps for rather different reasons from those given by the senior taxing master, I hold that
his disallowance of the items which he disallowed on this bill of costs was right.
I should add that it will be clear from what I have already said that, in my view, the regulations, and in particular reg 18(1)
and (5), may merit reconsideration, especially with regard to the time and manner of the determination of a previously assisted
persons costs where, after the discharge of his legal aid certificate, the action is dismissed for want of prosecution. If I am right
in this judgment, the regulations may involve hardship on the non-assisted party. If I should be wrong in this judgment, the
formerly assisted person, with no available legal advice, may be deprived of the protection which I think that the Legal Aid and
Advice Act, 1949, intended that he should have.

Application dismissed. Leave to appeal to the Court of Appeal.

Solicitors: Blount, Petre & Co (for the defendants); Secretary of the Law Society (for the Law Society).

Mary Colton Barrister.


[1963] 2 All ER 199

N W Robbie & Co Ltd v Witney Warehouse Company Ltd


COMPANY; Charges

LIVERPOOL ASSIZES
WIDGERY J
23, 24 JANUARY, 4 FEBRUARY 1963

Company Debenture Floating charge Receiver and manager appointed Crystallisation of floating charge Goods
supplied by company on credit before and after charge became fixed Set-off Purchasers of goods obtaining, after receiver
appointed, assignment of debts due from company which arose after the debenture was issued but before receiver appointed
Whether the assigned debts could be set-off against the purchasers indebtedness to plaintiff company for goods bought.

On 26 January 1960, the plaintiff (a company incorporated in Ireland) issued to its bankers a debenture securing all moneys from
time to time due to them from the company by a floating charge on the assets of the company. Between 24 May 1961, and 6 July
1961, the plaintiff company sold on credit to the defendants goods to the value of about 95. On 6 July 1961, the bank, as
debenture-holders, appointed a receiver and manager of the property comprised in the debenture. After 6 July 1961, the plaintiff
company, with the approval of the receiver and manager, sold further goods on credit to the defendants to the value of 1,251 6s
1d. The goods so supplied formed part of the assets of the plaintiff company at the date when the receiver was appointed, so that
the floating charge had attached to them. Between the beginning of November, 1960, and the end of January, 1961, the plaintiff
company had bought goods on credit from a third company, a subsidiary of the same parent company as the defendants, in respect
of which 852 18s 4d remained owing. On 6 October 1961, the benefit of the debt of 852 18s 4d was assigned by the third
company to the defendants, and due notice of the assignment was given to the plaintiff company. The plaintiff company sued the
defendants for 1,346 6s 1d (viz the total of 95 and 1,251 6s 1d), being the price of goods sold and delivered by the plaintiff
company between 24 May and 6 July 1961, and after 6 July 1961. The defendants sought to set-off the debt of 852 18s 4d,
admitting the balance, 493 7s 9d, to be due.

Held (i) The charge created by the debenture, which became a fixed charge on the appointment of the receiver and manager on
6 July 1961, attached to the debt of 95 owing by the defendants in respect of goods sold and delivered before 6 July 1961, and
accordingly that debt was not due between the same parties in the same right at the date (6 October 1961) of the 199 assignment
(since the debenture-holders had become entitled to an interest in the debt); therefore, there could be no partial set-off of the 852
18s 4d as against the 95 (see p 202, letter f, post).
(ii) But the fixed charge created by virtue of the debenture on the appointment of the receiver and manager was not
transferred from goods of the plaintiff company, sold on credit with his approval, to the indebtedness for the purchase price;
accordingly the defendants, as assignees of the plaintiff companys debt for 852 18s 4d, were entitled to a set-off of that sum
against their indebtedness for the price of goods supplied to them by the plaintiff company on and after 6 July 1961, and,
therefore, the plaintiff company was entitled to recover only the 493 7s 9d, which sum included the 95 referred to at (i) ante
(see p 203, letters e and f, post).

Notes
As to the effect of a floating charge becoming fixed, see 6 Halsburys Laws (3rd Edn) 476, para 920; and for cases on the subject,
see 10 Digest (Repl) 781783, 50705088.
As to set-off being confined to claims between same persons in same right, see 34 Halsburys Laws (3rd Edn) 399, para 679;
and for cases on the subject, see 40 Digest (Repl) 407, 408, 1321, 409, 38 et seq, 411, 412, 6872.

Case referred to in judgment


Biggerstaff v Rowatts Wharf Ltd [1896] 2 Ch 93, 65 LJCh 356, 74 LT 473, 9 Digest (Repl) 559, 3702.

Action
The plaintiff company, NW Robbie & Co Ltd sued the defendants, Witney Warehouse Co Ltd for 1,346 6s 1d, the price of goods
sold and delivered. By its statement of claim, which was indorsed on the writ in the action dated 4 January 1962, the plaintiff
company claimed the total sum of 1,346 6s 1d and gave particulars showing sales on sixteen dates, the last date being 12
September 1961. The first four items were24 May 29 7s 7d; 28 June 24 2s 4d; 4 July 42 7s 3d; and 17 July 44 6s 8d.
By their defence the defendants admitted 493 7s 9d to be still owing to the plaintiff company and claimed set-off of the
balance, amounting to 852 18s 4d, for the price of goods sold and delivered by English Spinners Ltd to the plaintiff company,
which debt was assigned on 6 October 1961, to the defendants. Particulars of the debt were given, consisting of four items,
totalling 1,602 18s 4d, credit being given for 750 paid. The four items were11 November 1960, 538 7s 6d; 16 December
1960, 248 5s 11d; 13 January 1961, 291 3s 6d and 31 January 525 1s 5d.
By its reply the plaintiff company pleaded that it was subject to the law of Ireland, and, among other matters, that the
defendants were not entitled to the set-off that they claimed, that the Bank of Ireland were holders of a debenture dated 26
January 1960, securing all moneys due to the bank from the plaintiff company by a floating charge on the assets of the plaintiff
company and that a receiver and manager of the undertaking and assets of the plaintiff company had been appointed on 6 July
1961, by the bank as debenture-holder.
The facts were agreed in letters exchanged between the parties solicitors, and it was also agreed (i) that the law of Ireland as
to debentures, receivers and their powers is the same as the law of England, and (ii) that the law of Ireland as to assignment of
debts is the same as English law as it was before 1 January 1926.
The cases noted belowa were cited in argument in addition to the case referred to in the judgment.
________________________________________
a Parsons v Sovereign Bank of Canada [1913] AC 160, Bennett v White, [1910] 2 KB 643

J H Bamber for the plaintiff company.


H C Easton for the defendants.

Cur adv vult

4 February 1963. The following judgment was delivered.

WIDGERY J delivered the following judgment. In this action the plaintiff, a company incorporated in Ireland, sues the
defendants for the 200 sum of 1,346 6s 1d, the price of goods sold and delivered to the defendants. The defendants admit
liability in the sum of 493 7s 9d, but seek to meet the balance of the claim, namely 852 18s 4d, by pleading a set-off of equal
amount. The only issue for me to decide is the validity of this set-off, but this issue has not proved to be one entirely suitable for
trial in the Queens Bench Division on circuit, as the relevant principles are more familiar in the Chancery Division, and the
opportunity for research in the authorities is limited. No oral evidence has been given at the trial, both counsel having been
content with the following statement of facts, which I am told is agreed.
On 26 January 1960, the plaintiff company issued a debenture to the Bank of Ireland to secure the plaintiff companys
overdraft on its current account with the bank. So far as the law of Ireland is material to this case, it is the same as the law in
force in England prior to 1 January 1926. The debenture appears to be in common form, and provides in para 3:

The company hereby as beneficial owner charges with such payments its undertaking and all its property and assets
present and future including its uncalled capital for the time being and goodwill, and also as a specific charge, the
hereditaments and premises specified in the schedule hereto.

The conditions to which the debenture was expressed to be subject include the following:

(1) This debenture is one for securing the payment of all moneys for the time being due by the company to the bank.
The debenture is to rank as a first charge on the property within mentioned and such charge is to be as regards the
companys lands and premises for the time being and all its uncalled capital for the time being a specific charge and as
regards all other property and assets of the company a floating security but so that the company is not to be at liberty to
create any mortgage or charge on its property for the time being in priority to or pari passu with the said debenture.

Condition (10) reads:

At any time after the principal moneys hereby secured become payable, the registered holder of this debenture may
appoint by writing any person to be a receiver and manager of the property charged by this debenture and may from time to
time remove any such receiver and manager, and any receiver and manager so appointed shall have power:
1. To take possession of, collect and get in the property charged by the debenture, and for that purpose to take any
proceedings in the name of the company or otherwise as may seem expedient.
2. To carry on or concur in carrying on the business of the company.
3. To sell or concur in selling, let or concur in letting any of the property charged by this debenture, and to carry any
such sale into effect by deed in the name and on behalf of the company, or otherwise, to convey the same to the purchaser.
4. To make any arrangement or compromise which he or they shall think expedient in the interest of the debenture-
holders.
And all moneys received by such receiver and manager shall, after providing for the matters specified in s. 24(8)(i), (ii)
and (iii) of the Conveyancing Act, 1881, and for the purposes aforesaid, be applied in or towards satisfaction of the said
debenture, and the foregoing provisions in the condition shall take effect as and by way of variation and extension of the
provisions of s. 19 to s. 24, inclusive, of the said Act, as amended by the Conveyancing Act, 1911, which provisions so
varied and extended shall be regarded as incorporated herein. The receiver and manager so appointed shall be the agent of
the company and the company shall be solely responsible for such receiver and managers acts or defaults.

Following the issue of this debenture the plaintiff company continued to trade, and on and after 24 May 1961, the plaintiff
company sold on credit to the 201 defendants the goods which are the subject of this claim. On 6 July 1961, the Bank of Ireland
appointed a receiver and manager of the property comprised in the debenture to which I have referred.
Notwithstanding the appointment of this receiver the plaintiff company continued to supply goods on credit to the
defendants, and I am told that I may assume that this was done with the approval of the receiver, and that the goods so supplied
formed part of the assets of the plaintiff company at the date when the receiver was appointed. The value of the goods so
supplied and dates of supply are correctly set out in the statement of claim.
Between November, 1960, and January, 1961, the plaintiff company bought goods on credit from English Spinners Ltd to
the total value of 1,602 18s 4d. Payment on account reduced the amount of this debt to 852 18s 4d. English Spinners Ltd are a
company associated with the defendants in the sense that both are subsidiaries of the same parent company, and on 6 October
1961, the benefit of this debt, due from the plaintiff company to English Spinners Ltd was assigned by deed to the defendants.
Due notice of this assignment having been given to the plaintiff company, the assignment amounted to a legal assignment of the
debt to the defendants, and this is the debt pleaded as a set-off in this action.
Certain propositions of law are not in issue and can be stated as follows. First, the defendants are entitled to this set-off,
notwithstanding that their title to the debt set-off was derived from an assignment made by the original creditors, if immediately
following the assignment the debt claimed and the debt set-off existed between the same parties in the same right. Secondly, the
existence of a floating charge on the plaintiff companys assets would not in itself amount to an assignment to the debenture-
holders of the debt claimed so as to prevent the claim and set-off from existing between the same parties in the same right; for
that proposition see Biggerstaff v Rowatts Wharf Ltd. Thirdly, the appointment of the receiver and manager caused the floating
charge created by the debenture to crystallise or become fixed on the assets of the plaintiff company at the date of the
appointment. The debt represented by the first three items b in the statement of claim and amounting to some 95 existed at the
date when the receiver was appointed. It was subject to this fixed charge, and thus became assigned to the debenture-holders.
The debt subsequently assigned by English Spinners Ltd to the defendants cannot be set-off against this part of the plaintiff
companys claim, because this part of the debt claimed had become vested in the debentureholders before the right of set-off was
acquired against the plaintiff company. Fourthly, the floating charge created by the debenture became fixed on the assets of the
plaintiff company existing at the date of the appointment of the receiver, but does not of itself attach to assets of the plaintiff
company subsequently acquired.
________________________________________
b See p 200, letter e, ante, where the first four items are stated

On these agreed facts, and accepting those principles, the plaintiff company resists the defendants right to set-off the debt of
852 18s 4d against the plaintiff companys claim. Counsels contention on behalf of the plaintiff company is that where the
receiver and manager sells, on behalf of the company, assets on which a former floating charge has become fixed, the charge
attaches to the proceeds of sale of those assets, as it formerly attached to the assets themselves. He argues that if the plaintiff
company through the agency of the receiver had sold the goods to the defendants for cash, the charge which formerly attached to
those goods would automatically attach to the proceeds of sale in the receivers hands; and he submits that if the goods are sold
on credit, the charge attaches to the debt due to the plaintiff company for the price of the goods. Accordingly, he contends that
when the goods subsequent to item 3 in the statement of claim were sold to the defendants, the resultant debt was automatically
charged or assigned to the debenture-holders, and that this prohibits the defendants from 202 relying on the set-off of a debt
subsequently acquired by the defendants. Counsel for the plaintiff company has not referred me to any textbook or authority for
this contention, but says that this result must follow if the receivers power to continue the business of the plaintiff company and
realise the assets of the plaintiff company is to exist as a practical possibility.
Counsel for the defendants disputes the plaintiff companys contention that when the goods subject to a fixed charge are sold
by the receiver, the charge is transferred to the proceeds of sale. He concedes, as I understand his argument, that if the receiver
sells such goods for cash, that cash must be paid by the receiver to the debenture-holders, and will not be available for the
satisfaction of the plaintiff companys unsecured creditors; but he says that this result flows from the fact that the conditions of
the debenture required the receiver to hold moneys so received on trust for the debenture-holders, and does not depend on any
notional transfer of the original charge to the proceeds of sale. When the sale of the plaintiff companys assets is on credit, he
contends that there are no proceeds of sale to which the trust created by the debenture can attach, and he says there is nothing in
the debenture which subjects the debt due to the company to any such trust, because the debt is not moneys received within
condition 10. He contends that the debt is due to the plaintiff company and not due to the receiver, and cannot be subject to any
trust of which the receiver is trustee. In result, so the argument goes, the debt due to the plaintiff company from the defendants,
as a result of trading subsequent to the appointment of the receiver, is a debt in which the debenture-holders have no present
interest at all. Consequently, the debt claimed and the debt to be set-off exist between the same parties in the same right, and the
set-off claim should be allowed.
Having reflected on these arguments I have decided that counsel for the defendants is right. Where a receiver carries on the
companys business in the name of the company, those with whom he deals should not lightly be deprived of the rights of set-off
to which they would have been entitled if the business were being carried on by the company on its own account.
The principle for which counsel for the plaintiff company contends, namely, that the debenture-holders charge
automatically attaches to debts due to the company as a result of such trade, is not supported by authority or by any principle of
law or of equity which I recognise. Accordingly in my judgment that contention fails, the set-off is a valid one, and the amount
for which judgment should be given is 493 7s 9dc only.
________________________________________
c The sum of 95 (see p 202, letter f, ante) was included in the 493 7s 9d

As the plaintiff company is in Ireland it would seem convenient to have payment out in this country. There will be judgment
for the plaintiff company with costs up to the date of payment in. The defendants will have their costs after the date of payment
in. Accounts for costs will be set-off, and any balance due to the defendants will be deducted from the amount paid into court
before the balance of the amount is paid out to the plaintiff company.

Judgment accordingly.

Solicitors: Boote, Edgar & Co, Manchester (for the plaintiff company); Skelton & Co, Manchester (for the defendants).

K Buckley Edwards Esq Barrister.


203
[1963] 2 All ER 204

National Provincial Bank Ltd v Hastings Car Mart Ltd and others
LAND; Property Rights: EQUITY

CHANCERY DIVISION
CROSS J
4, 5, 27 MARCH 1963
Husband and Wife Deserted wifes right to remain in occupation of matrimonial home Registered land Overriding interest
Deserted wife, and children, remaining in occupation of husbands dwelling-house Transfer of house to private company in
which husband principal shareholder Mortgage by company Mortgagees unaware of husbands desertion Right of
mortgagees to vacant possession to enforce security Whether wifes equity was right in rem Whether an overriding interest
Land Registration Act, 1925 (15 & 16 Geo 5 c 21), s 70(1) (g).

A deserted wifes equity or claim to remain in occupation of the matrimonial home is simply a right to appeal to the court for
protection against unconscionable conduct on the part of the husband or the husbands successor in title, but it is not a right in
rem; the wifes equity is not, therefore, an overriding interest within s 70(1)(g) a of the Land Registration Act, 1925 (see p 208,
letter f, p 209, letter d, post).
________________________________________
a The terms of s 70(1)(g) are set out at p 207, letter h, post

Westminster Bank Ltd v Lee ([1955] 2 All ER 883) considered.


In 1957 the husband deserted the wife. The wife and the four infant children of the marriage continued to reside in the
matrimonial home, of which the husband was registered proprietor with absolute title. The husband went to live with his mother,
some half a mile away. In 1958 the husband mortgaged the property to the bank to secure a loan and further advances, and the
mortgage was duly registered in the charges register. The bank did not know that the husband had deserted the wife. In 1959 a
company was formed to take over the husbands business and properties; in the memorandum of association the address stated as
the husbands was the address where his mother lived. The company became the registered proprietor of the matrimonial home,
and charged it and other property to the bank, the husband giving a guarantee to the bank for the companys indebtedness, and the
former charge on the matrimonial home being discharged. The new charge was duly registered in the charges register. In 1961
the wife obtained a decree of judicial separation and an order for permanent alimony, which was made on the basis that the
husband was providing the wife and children with a rent-free home at the matrimonial home. At this time the bank learnt that the
husband had deserted the wife and had left her in occupation of the matrimonial home. The company having failed to comply
with a notice from the bank calling in its debt, the bank issued a summons for possession of the mortgaged property. At the date
of the hearing there was pending an application by the wife, who was in receipt of national assistance, for an order under s 2 of
the Matrimonial Causes (Property and Maintenance) Act, 1958, setting aside the conveyance of the matrimonial home by the
husband to the company on the ground that it had been made with the intention of defeating her claim against him for financial
relief.

Held The bank was entitled to an order for possession, but as it would not be right for the court to compel the wife to give up
possession before the determination of her application under s 2 of the Act of 1958, the order would be for possession in three
months, with liberty to the wife to apply for an extension if, without fault on her part, her application had not by then been
determined.

Notes
As to deserted wifes right to remain in the matrimonial home, see 19 Halsburys Laws (3rd Edn) 849851, para 1388; and for
cases on the subject, see 3rd Digest Supp.
204
As to overriding interests, see 23 Halsburys Laws (3rd Edn) 179184, paras 346350.
For the Land Registration Act, 1925, s 20(1), s 70(1)(g), see 20 Halsburys Statutes (2nd Edn) 962, 1002.
For the Law of Property Act, 1925, s 199(1)(ii)(a), see ibid, 822.
For the Matrimonial Causes (Property and Maintenance) Act, 1958, s 2, see 38 Halsburys Statutes (2nd Edn) 443.

Cases referred to in judgment


Hunt v Luck [1901] 1 Ch 45, 70 LJCh 30, 83 LT 479, affd CA, [19003] All ER Rep 295, [1902] 1 Ch 428, 71 LJCh 239, 86 LT
68, 35 Digest 469, 2044.
Westminster Bank Ltd v Lee [1955] 2 All ER 883, [1956] Ch 7, 3rd Digest Supp.

Adjourned Summons
This was an application under RSC, Ord 55, r 5A, by originating summons dated 4 July 1962, and issued by the plaintiffs,
National Provincial Bank Ltd for an order for possession of freehold land with the dwelling-house erected thereon known as 124,
Milward Road, Hastings, in the county of Sussex, the title to which was registered at HM Land Registry under title No Ht 15956
under and by virtue of a registered mortgage and general charge dated 17 December 1959, made between the first defendant,
Hastings Car Mart Ltd of the one part and the plaintiffs of the other part.
The defendants were (1) Hastings Car Mart Ltd (2) Gordon Ainsworth, director and principal shareholder of the company,
and (3) Marjorie Patty Ainsworth, the wife of the second defendant, who was deserted by him in 1957 and who had lived ever
since at the said premises with the four children of the marriage.
The facts appear in the judgment.
The cases noted belowb were cited in argument in addition to those referred to in the judgment.
________________________________________
b Bendall v McWhirter [1952] 1 All ER 1307, [1952] 2 QB 466, Mornington Permanent Building Society v Kenway, [1953] 1 All ER 951,
[1953] Ch 382, Woodcock (Jess B) & Son Ltd v Hobbs, [1955] 1 All ER 445, [1955] 1 WLR 152, Grace Rymer Investments Ltd v Waite,
[1958] 2 All ER 777, [1958] Ch 831, Churcher v Street, [1959] 1 All ER 23, [1959] Ch 251

D H Mervyn Davies for the plaintiffs, the bank.


K Bruce Campbell for the third defendant, the wife.
The first defendant, the company, and the second defendant, the husband, did not appear and were not represented.

Cur adv vult

27 March 1963. The following judgment was delivered.

CROSS J read the following judgment. This is a summons by mortgagees, National Provincial Bank Ltd for an order for
possession of a house, No 124, Milward Road, Hastings, of which the first defendants, Hastings Car Mart Ltd are the registered
proprietors. The other defendants are Gordon Ainsworth, who is the principal shareholder and managing director of the first
defendants, and Marjorie Patty Ainsworth, his wife. The company and the husband have not entered appearances to the
summons, but the wife, who has been in sole occupation of the house since she was deserted by her husband some five years ago,
resists the banks claim in reliance on what has come to be called the equity of a deserted wife.
The husband and the wife were married on 21 April 1948, and have four children now aged fourteen, twelve, nine and six.
On 14 August 1956, the husband was registered as proprietor of No 124, Milward Road, with an absolute title subject to a charge
dated 25 July 1956, and registered on 14 August 1956, in favour of the Co-operative Building Society. The husband and the wife
and their children lived together in the house until 17 August 1957, when the husband deserted the 205 wife, leaving her and the
children at 124, Milward Road, and going himself to live with his mother at No 13, Devonshire Road, Hastings, which is about
half a mile away and was also owned by him. The husband carried on business as a motor car dealer at No 7, Bank Buildings,
Hastings, under the name of The Car Mart and banked with the Hastings branch of the plaintiff bank, the manager of which at all
material times was a Mr Mumford. On 8 July 1958, the bank lent him 1,000 for the purposes of his business and took as
security for that loan and any other advances which it might make to him, inter alia, a second charge on No 124, Milward Road,
which was registered on 24 July 1958. Number 124, Milward Road, appeared in the banks records as the husbands address and
Mr Mumford, who was cross-examined on the affidavit which he made in these proceedings, says (and I accept it) that he did not
know or suspect that the husband was not still living there with his wife. The husband was not in all respects a satisfactory
customer and Mr Mumford often had to get in touch with him on the telephone at short notice to discuss the state of his account.
Sometimes when he was not at 7, Bank Buildings, Mr Mumford would be told that he could probably get in touch with him at 13,
Devonshire Road. Mr Mumford knew that that was his mothers house and he saidand again I accept itthat it did not occur
to him that there was anything odd in the husband being there on those occasions and not at 124, Milward Road. On 27 February
1959, the wife brought a petition for judicial separation against her husband, but Mr Mumford did not know of this at the time.
On 27 April 1959, the mortgage to the Co-operative Building Society was paid off and the banks mortgage became a first
mortgage. By November, 1959, the husband owed the bank nearly 6,000 which was secured by mortgages on 124, Milward
Road, and 7, Bank Buildings, and a second mortgage on 13, Devonshire Road. The husband wished to transfer his business and
properties to a company and the bank was prepared to accept the company as its principal debtor in place of the husband if he
guaranteed the companys account. In pursuance of this arrangement, the first defendants, Hastings Car Mart Ltd were
incorporated on 23 November 1959, the subscribers to the memorandum being the husband, whose address was given as 13,
Devonshire Road; his mother of the same address; and a Mr Jones. Then on 17 December 1959, Mr Ainsworth conveyed 7, Bank
Buildings, and 124, Milward Road, to the company and the company on the same day charged the two properties to the bank to
secure payment of all moneys from time to time owing by it to the bank, which at once advanced it enough to discharge the
existing debt of the husband. The husband at the same time signed a guarantee in respect of the companys indebtedness to the
bank. In that guarantee his address was given as 13, Devonshire Road, the reason for this being, probably, that the clerk in the
bank who prepared the guarantee form for execution took the husbands address from the memorandum of association of the
company, a copy of which was at the bank. Mr Mumford did not himself see the guarantee at this time. He was given to
understand by the husband that he had an informal tenancy of 124, Milward Road, from the company. The company was duly
registered as proprietor of 124, Milward Road. The charge given by the husband on 8 July 1958, was discharged and the charge
given by the company on 17 December 1959, was entered in the charges register first by notice of deposit of the land certificate
and later by registration.
On 13 March 1961, the wife obtained a decree of judicial separation from her husband. At about this time the husband had
asked the bank for an advance to meet the costs of the proceedings and it was then that Mr Mumford learnt that the husband had
deserted the wife and left her in occupation of 124, Milward Road. On 2 May 1961, an order for permanent alimony was made
against the husband. This order was made on the basis that he was providing the wife and children with a home rent free at 124,
Milward Road. He is seriously in arrear with his payments under the order. On 2 November 1961, the bank served notice on the
company calling in its debt which was then 2,308 odd. The company 206 did not comply and on 10 April 1962, the bank served
a second notice stating that in default of payment of what was then owing it would proceed to exercise its rights as mortgagees of
124, Milward Road. The summons for possession which is now before me was issued on July 4.
The wife is legally aided and is in receipt of national assistance. She has made an application under s 2 of the Matrimonial
Causes (Property and Maintenance) Act, 1958, for an order setting aside the conveyance of 124, Milward Road, by the husband
to the company as having been made with the intention of defeating her claims against him for financial relief. This application
is still pending.
Those being the facts, what is the law? The case of Westminster Bank Ltd v Lee was, up to a point, a similar case to this, but
there the property in question was not registered land. In that case Upjohn J, first considered whether the deserted wifes claim to
remain in occupation of the matrimonial home could in any circumstances bind a purchaser of the house from the husband. On
that point he took the view that he was bound by the authorities to hold that a purchaser might be bound by it. The mortgage in
that case was an equitable mortgage and therefore it was necessary to decide whether or not the wife acquired on being deserted
an equitable interest in the house which would have priority over the banks equitable mortgage which was later in date even
though the bank had no notice that the mortgagor had deserted his wife. On this point the judge held that the wife did not acquire
an equitable interest in the house but had only a mere equity, analogous to a right to set aside or rectify a conveyance for fraud
or mistake, which only binds a subsequent purchaser, even if his title is merely equitable, if he has notice of it. The judge was
next faced with the principle discussed in the case of Hunt v Luck. A prudent purchaser or mortgagee inspects the property which
he is buying or on which he is advancing money and, if he finds anyone other than the vendor or mortgagor in occupation, asks
them whether they claim any, and if so what, rights with regard to the property. If a purchaser or mortgagee fails to do this, he is
taken to have knowledge of anything which he would have discovered if he had done so (see the Law of Property Act, 1925, s
199(1)(ii)(a)). Upjohn J held that this principle had no application to the case before him. The fact that the only person in the
house when the prospective mortgagee visited it was the mortgagors wife would not of itself suggest that she had been deserted
by her husband. That fact would only emerge if the mortgagee went on to ask her whether her husband had deserted her, and the
judge refused to hold that it was the duty of anyone who bought or advanced money on the house of a man whom they knew to
be married to embark on inquiries of that sort. There was nothing in that case to suggest that the bank had notice of any fact
which might lead them to think that the mortgagor had deserted his wife, and so the judge was able to come to the conclusion,
which he clearly thought was consonant with good sense and justice, that the wife could assert no right against the bank.
In this case, however, the land is registered land and I am faced with s 70(1)(g) of the Land Registration Act, 1925, which
includes among the over-riding interests to which all registered land is deemed to be subject

The rights of every person in actual occupation of the land or in receipt of the rents and profits thereof, save where
inquiry is made of such person and the rights are not disclosed.

Overriding interests are, speaking generally, matters which are not usually shown on title-deeds or mentioned in abstracts of title
and as to which, in consequence, it is not possible to form a trustworthy record on the register. As to such matters, persons
dealing with registered land must obtain information outside the register in the same manner and from the same sources as people
dealing with unregistered land would obtain it. Paragraph (g) is obviously 207 intended to give effect in the case of registered
land to the principle discussed in Hunt v Luck and to ensure that the rights of persons in actual occupation of registered land, even
though such rights are not noted on the register, shall not be destroyed by a transfer of the property to a new registered proprietor.
In this connexion it is to be remembered that s 20(1) of the Act of 1925 provides that in the case of freehold land registered with
an absolute title, a disposition of the registered land or of a legal estate therein, for valuable consideration shall, when registered,
confer on the grantee the legal estate expressed to be created subject (a) to the incumbrances and other entries (if any) appearing
on the register, and (b) to the overriding interests (if any) affecting the estate transferred or created, but free from all other estates
and interests whatsoever.
Is the equity of a deserted wife an overriding interest? The deserted wife is clearly a person in actual occupation of the
registered land and her claim to remain in occupation, even though described as an equity and not as an equitable interest in the
land, is, according to the case of Westminster Bank Ltd v Lee, a claim which may in some circumstances bind a purchaser. It is at
first sight hard to see how one can avoid the conclusion that it is a right within the meaning of s 70(1)(g) of the Act of 1925, even
though such a conclusion would involve the ridiculous consequence that the position of a deserted wife is stronger if the
matrimonial home is registered land than it is if it is unregistered land, since in the former case her equity will bind a purchaser
without notice. It occurred to me in the course of the argument that one might read into s 70(1)(g) after the word thereof some
such words as of whom a prudent purchaser will inquire as to his rights. But that would be a substantial gloss to put on the
paragraph and, what is more, to do so might be said logically to entail the result that a purchaser who knew all the facts would
nevertheless take free from the wifes equity, for if it is a right relating to the land but yet not an overriding interest, s 20(1) would
make the transfer override it.
A better way out of the apparent impasse is, I think, to go a little further than it was necessary for Upjohn J to go in the case
of Westminster Bank Ltd v Lee, though not, maybe, further than he would have gone had he been faced with the problem which
faces me, and to hold that though in some cases a purchaser from the husband may not have an unqualified right to turn the wife
out of the house, yet her equity is not anything which can properly be described as a right in rem at all but is simply a right to
appeal to the court for protection against unconscionable conduct on the part of the husbands successor in title. A husband who
has deserted his wife cannot be allowed to treat her at once as a trespasser in the matrimonial home. In the normal case it would
not be right to allow him to turn her out unless he provided her with suitable alternative accommodation. Equally, it would not be
right to allow a successor in title of the husband to turn the wife out simply because he is a successor in title and not the husband
himself. If the husband goes bankrupt or gives the house away, the trustee in bankruptcy or the donee has no higher moral claim
to turn the wife out without providing her with another home than the husband himself had. The same may well sometimes be
true of a bona fide purchaser. In this case, for example, the sale of the house by the husband to the company, though the wife is
alleging that it was made in order to defraud her, may, for all I know, have been made in perfect good faith. I cannot think,
however, that any court would allow the company to turn Mrs Ainsworth out of the house without providing her with another
home. It would say that as the company had full knowledge of the position, and was indeed the alter ego of the husband, it had
no higher rights than he would have had. But to say that some successor in title of the husband has no higher right to possession
of the house than the husband would have had is one thing: to say that the wife has a correlative 208 legal right in respect of the
house is another. In the light of s 70(1)(g) of the Land Registration Act, 1925, and the doctrine of notice as worked out by
conveyancers in relation to dealings with unregistered land, to treat a deserted wife as acquiring on desertion some legal or
equitable right in respect of the house may result in grave injustice to innocent purchasers.
I have already pointed out the difficulties arising in the case of registered land from s 70(1)(g). If this was a case of
unregistered land, there would be comparable difficulties under the doctrine of notice. Neither Mr Mumford, nor, so far as I
know, any other servant of the bank, knew at the time of the mortgage that the husband had deserted the wife, but the bank had in
its possession as one of the documents of the mortgagors title the memorandum of association which gave the husbands address
as 13, Devonshire Road, whereas Mr Mumford believed him to be living with his wife at 124, Milward Road. If one were to hold
that the wife had acquired on being deserted some right in respect of the property which could prevail against anyone other than a
bona fide purchaser without notice in the sense in which that word is used by conveyancers, it may may well be that one would
have to hold that the bank was fixed with knowledge of a fact, namely, the giving by the husband of 13, Devonshire Road, as his
address in the memorandum of association, which should have led them to inquire whether he still lived with his wife at 124,
Milward Road. I should have been sorry to have been forced to come to such a conclusion.
I propose, therefore, to hold that the fact that a wife who is deserted may be able successfully to resist a claim for possession
made by a successor in title of the husband does not constitute an overriding interest. I am glad to see that that is also the view
of the Chief and Senior Land Registrars (see Curtis and Ruoff; Law and Practice of Registered Conveyancing, pp 127 and 128).
It might, perhaps, be desirable to enact that a wife on being deserted in the matrimonial home could register the fact in the
charges register in the case of registered land or in the land charges register in the case of unregistered land, and that, if she did
so, a purchaser or mortgagee should be bound by her equity even if he had no actual notice of the fact of desertion. But until
that happens I think that more harm than good will be done by treating the wifes equity as a right relating to or binding the land
as opposed to the conscience of the successor in title. I would add that, so far as I can see, this view is not inconsistent with the
actual decisions in any of the reported cases though it is, of course, inconsistent with what is said by some of the judges who gave
or took part in the decisions. As Upjohn J, pointed out in the case of Westminster Bank Ltd v Lee, it is impossible to reconcile the
numerous dicta on the subject.
It follows, therefore, that in my judgment the bank is entitled to an order for possession. In the ordinary way, it would be an
order for possession within twenty-eight days, but there is in this case the peculiar feature of the wifes pending application under
s 2 of the Matrimonial Causes (Property and Maintenance) Act, 1958. It is not for me to say whether or not any order could be
made under that section which would affect the banks rights as well as the companys rights, but I do not think that it would be
right for me to compel the wife to leave the house while those proceedings are pending providing that she prosecutes them
diligently. In fact the bank has said very fairly that, whatever its strict rights may be, it is prepared to allow the wife to stay in the
house for a further three months. That should, I think, be sufficient for the hearing of the wifes application and so I shall make
an order for possession in three months. There will be liberty to the wife to apply if the application is heard 209 within that time
and results in an order affecting the banks right to possession, or if the application, through no fault of hers, cannot be disposed
of in that period.

Order accordingly.

Solicitors: Wilde, Sapte & Co (for the plaintiffs); Perring & Co (for the third defendant, the wife).

Jacqueline Metcalfe Barrister.


[1963] 2 All ER 210

Arrowsmith v Jenkins
CRIMINAL; Criminal Law

QUEENS BENCH DIVISION


LORD PARKER CJ, ASHWORTH AND WINN JJ
6 MARCH 1963

Highway Obstruction Wifully obstructs Offence constituted where intention was to do act that caused the obstruction,
though no intention to do wrong Highways Act, 1959 (7 & 8 Eliz 2 c 25), s 121.
It is sufficient to constitute the offence of wilfully obstructing the free passage along a highway contrary to the Highways Act,
1959, s 121(1)a that the accused, without lawful authority or excuse, intentionally (viz, by exercise of his free will) does
something or omits to do something, which act or omission causes an obstruction or the continuance of an obstruction (see p 211,
letter g, post).
________________________________________
a The terms of s 121(1) are stated at p 211, letter e, post

Notes
The decision in the present case should be considered in relation to the words wilfully obstructs, with that in Eaton v Cobb
([1950] 1 All ER 1016) as well as other cases on s 72 of the Highway Act, 1835, which s 121 of the Highways Act, 1959,
supersedes.
As to wilfully obstructing the passage along a highway, see 19 Halsburys Laws (3rd Edn) 286, para 455, notes (m) and (o);
and for cases on the subject see 26 Digest (Repl) 468470, 15691586.
For the Highways Act, 1959, s 121, see 39 Halsburys Statutes (2nd Edn) 542.

Case Stated
This was an appeal by way of Case Stated from a decision of the court of Quarter Sessions for the county of Lancaster sitting at
Liverpool on 13 July 1962, dismissing an appeal by the appellant, Pat Arrowsmith, against her conviction on 7 May 1962, at the
magistrates court for the county borough of Bootle of having on 13 April 1962, without lawful authority or excuse wilfully
obstructed the free passage along Nelson Street, Bootle, a highway, by standing thereon and causing others to congregate,
contrary to s 121 of the Highways Act, 1959. The facts appear in the judgment.
The appellant contended, among other contentions, that it was not sufficient to establish merely that the highway had been in
part rendered less convenient or less commodious, but that there must be shown to have been mens rea on the part of the
appellant and that such obstruction was in fact intentional or the result of a conscious act of the appellant with her mind directed
to the possible causing of such obstruction and wilfully disregarding the possible consequences of her action in addressing a
meeting on the highway; and that on the proper interpretation of s 121 of the Highways Act, 1959, it was necessary for the
prosecution to establish that the alleged obstruction followed from a knowingly wrongful act on the part of the appellant, but that
in fact the appellant was addressing a meeting at the place in question in exercise of a supposed legal right established by long
practice. The respondent contended that any conscious action on the part of the appellant which rendered the highway less
commodious and convenient to use constituted a wilful obstruction of the highway.
The cases noted belowb were cited during the argument.
210
________________________________________
b Gully v Smith [1883] 12 QBD 121, Smith v Barnham, (1876), 34 LT 774 at p 776, Homer v Cadman, (1886), 50 JP 454, Younghusband v
Luffig, [1949] 2 All ER 72 at p 80, [1949] 2 KB 354 at p 369

B Wigoder for the appellant.


F J Nance for the respondent.

6 March 1963. The following judgments were delivered.

LORD PARKER CJ stated the nature of the proceedings and continued. The short facts are that Nelson Street is an ordinary
public highway linking two main roads and providing direct access from a main dock entrance to the casualty department of
Bootle hospital. The appellant addressed a meeting there from about 12.35 pm until 12.55 pm. For the first five minutes of that
period the carriageway and pavements were completely blocked by people listening to her address. Thereafter, a passageway for
vehicles was cleared by the police, and a fire-engine and other vehicles were guided through the crowd. The appellant, at the
request of the police officers, assisted by means of a loudspeaker, as a result of which the carriage way remained only partly
obstructed from 12.40 until 12.55 when the appellant finished speaking.
There was a finding that if it had not been for the fact that the appellant was speaking, the crowd would have dispersed, and
the highway would have cleared. For my part, I think that the appellant feels she is under a grievance which she puts in effect in
this wayWhy pick on me? There have been many meetings held in this street from time to time. The police, as on this
occasion, have attended those meetings and assisted to make a free passage and she says that there is no evidence that anybody
else has ever been prosecuted. Why pick on me?, she says. That, of course, is nothing to do with this court. The sole question
here is whether she has contravened s 121(1) of the Highways Act, 1959. It provides that:

If a person, without lawful authority or excuse, in any way wilfully obstructs the free passage along a highway he shall
be guilty of an offence and shall be liable in respect thereof to a fine not exceeding forty shillings.

For my part, I am quite satisfied that this provision on its true construction is providing that if a person without lawful authority
or excuse intentionally, as opposed to accidentally, that is by an exercise of his or her free will, does something or omits to do
something which will cause an obstruction or the continuance of an obstruction, he or she is guilty of an offence. Counsel for the
appellant has sought to argue that if a person acts in the genuine belief that he or she has lawful authority to do what he or she is
doing, then if an obstruction results he or she cannot be said to have wilfully obstructed.
Quite frankly, I do not fully understand that submission. It is difficult certainly to apply it here. I imagine it can be put in
this way, that there must be some mens rea in the sense that a person will be guilty only if he knowingly does a wrongful act. For
my part, I am quite satisfied that that consideration cannot possibly be imported into the words wilfully obstructs in this
enactment. If anybody by an exercise of free will does something which causes an obstruction, then I think that an offence is
committed; there is no doubt that the appellant did that in this case.
For my part I am quite satisfied that the Quarter Sessions were right, and I would dismiss this appeal.

ASHWORTH J. I agree.

WINN J. I agree.

Appeal dismissed.

Solicitors: Peter Kingshill agent for Casson & Co, Manchester (for the appellant); Sharpe, Pritchard & Co agents for A J E
Taylor, Bootle (for the respondent).
Shireen Irani Barrister.
211
[1963] 2 All ER 212

Covington v Wright
CRIMINAL; Criminal Law: TRANSPORT; Road

QUEENS BENCH DIVISION


LORD PARKER CJ, ASHWORTH AND WINN JJ
8 MARCH 1963

Road Traffic Stage carriage Fare Intent to avoid payment of fare Fare from the place whence passenger started Bus
passenger buying a ticket and travelling beyond fare stage for which ticket valid Whether additional fare an excess fare from
the place to which ticket valid or the full fare from the start less the amount paid Whether dishonest intent necessary London
Passenger Transport Act, 1936 (26 Geo 5 & 1 Edw 8 c cxxxi), s 91(1), (2) (a).

The respondent bought a 5d ticket on an omnibus, and, having been told by the conductor that it was valid to Salters Hill,
remained on the omnibus after it reached that place and declared that she was going to Crystal Palace, which was a stage beyond
Salters Hill. The bus having resumed its journey, the conductor asked the respondent for 3d for the excess fare. She refused to
pay; she also refused the conductors request to give her name and address. He did not ask her to produce or deliver up a ticket
for the journey to Crystal Palace. He then stopped the omnibus in order to get police assistance; where upon the respondent
alighted and hurried away up the road. On apprehension she stated Ive come to Crystal Palace for the last twelve months and I
only pay 5d. The fare for a ticket from the place where the respondent boarded the bus to Crystal Palace was 6d and the fare
from Salters Hill to Crystal Palace was 3d, which was the minimum fare. The respondent was charged, on two informations, (i)
that having travelled on an omnibus she left it without paying her fare and with intent to avoid payment contrary to sub-s (2)(a) a
of s 91 of the London Passenger Transport Act, 1933; and (ii) that being a passenger on an omnibus she did on request of an
officer or servant of the London Transport Executive fail to produce or deliver up a ticket showing that her fare was paid or pay
the fare from the place whence she started or give the officer or servant her name and address, contrary to sub-s (1) b of s 91 of
the Act of 1936. On appeal against the dismissal of the two informations on the footing that there was no case to answer,
________________________________________
a Section 91(2), so far as material, is as follows: If any person(a) having travelled on any stage carriage of the board leaves or
attempts to leave it without having paid his fare and with intent to avoid payment; or (b) having paid his fare for a certain distance
knowingly and wilfully proceeds by stage carriage of the board beyond that distance without paying the additional fare for the
additional distance and with intent to avoid payment thereof he shall be liable on summary conviction to a fine
b The material terms of s 91(1) are set out at p 214, letter f, post

Held (i) The words pay his fare from the place whence he started in sub-s (1) of s 91 of the London Passenger Transport Act,
1936, meant pay the full fare, less what the passenger had already paid, from the place where the passenger got on the bus;
therefore, since the respondent had been asked for 3d, instead of 1d (which was all that was required to make up the full fare), the
respondent had not offended against sub-s (1), but
(ii) The words with intent to avoid payment of a fare (viz, of the full fare) in s 91(2)(a) did not necessarily import a
dishonest intention, but merely that the passenger intended not to pay what in fact turned out to be proper fare; there was ample
evidence that the respondent left the bus intending to avoid paying the full fare (though perhaps not dishonestly), and accordingly
the case would be remitted with a direction to determine the charge under s 91(2)(a).
Per Curiam: Section 91(1) of the Act of 1936 does not require each request mentioned in it to be put separately; it is
sufficient if in all the circumstances of the case the conductor or inspector makes known that the passenger must comply with one
or other of the requests (see p 214, letter g, post).
212

Notes
Section 91(2)(b) of the London Passenger Transport Act, 1936, is concerned with a different situation from sub-s (2)(a) and one
that involves a dishonest mind (see p 215, letter f, post). Provisions similar to s 91(1) and (2) of the Act of 1936, in the case of
railways, are made by s 5(1) and (3) of the Regulation of Railways Act, 1889; 19 Halsburys Statutes (2nd Edn) 865. For cases
under those enactments see 8 Digest (Repl) 119, 120, 761775.
As to avoiding payment of fares on public service vehicles, see 33 Halsburys Laws (3rd Edn) 411, para 672, text and notes
(e) and (f).
For the London Passenger Transport Act, 1936, s 91, see 24 Halsburys Statutes (2nd Edn) 947.

Case Stated
This was a Case Stated by one of the metropolitan stipendiary magistrates for the county of London in respect of his adjudication
as a magistrates court sitting at Lambeth on 10 September 1962. On 1 June 1962, two informations were preferred by the
appellant, William Edward John Covington, an authorised officer acting on behalf of London Transport Executive, against the
respondent, Margaret Wright, that on 12 April 1962 (i) having travelled by a stage carriage, to wit a motor omnibus, she did leave
it without paying her fare and with intent to avoid payment thereof, contrary to s 91(2)(a) c of the London Passenger Transport
Act, 1936, as amendedd; and (ii) being a passenger on a stage carriage, to wit a motor omnibus, she did on request of a servant of
London Transport Executive fail to produce or deliver up a ticket showing that her fare had been paid, or pay the fare from the
place whence she started or give her name and address, contrary to s 91(1) e of the Act of 1936.
________________________________________
c For the material terms of s 91(2)(a) see note *, p 212, ante
d Section 91 of the Act of 1936 has been amended (i) expressly by the British Transport Commission Act, 1950, s 41, which increased the
maximum amount of fines; (ii) implicitly as a result of the Transport Act, 1947, s 14, and the schemes delegating functions of the British
Transport Commission to the London Transport Executive, so as to substitute references to the executive for reference to the board; and
(iii) as from 1 September 1962, by the Transport Act, 1962, s 32, Sch 2, Pt 3, and the Transport Act, 1962 (Commencement No 1) Order,
1962, SI 1962 No 1788, which substitute references to the London Transport Board
e For the material terms of s 91(1), see p 214, letter f, post

The two informations were heard together by consent, when the magistrate found the following facts. At about 6.40 pm on
12 April 1962, the respondent boarded between stages a No 2 omnibus going towards Crystal Palace. She asked for a 5d ticket
which the conductor issued to her and for which she paid, the conductor stating that it was valid as far as Salters Hill. At Salters
Hill the respondent did not alight, and, when asked by the conductor if she were going to alight, she ignored him. Being asked
again, she said Im going to Crystal Palace (which is a stage beyond Salters Hill). The conductor rang the bell and the bus
continued its journey. The conductor then asked the respondent for threepence for the excess fare; the respondent refused to pay.
He then asked her for her name and address, which she refused to give. He told her that he would have to call the police if she
refused to do this, but she ignored him. He caused the omnibus to stop outside Gipsy Hill police station, which he entered to
obtain police assistance. By the time when the conductor returned with a police-constable, the respondent had left the omnibus
and was hurrying up the road, having gone about thirty yards from the ominibus. The conductor tried to stop her, but she would
not stop for him. When the police-constable asked her if she refused to pay the excess fare, she said Im not paying my fare.
Ive come to Crystal Palace Parade for the last twelve months and I only pay 5d. The constable asked her for her name and
address, but she refused to give them, and she also refused at first his request to go to the police station. Forty-five minutes later
at the police station the respondent did give her name and address and she paid the 3d excess fare (for which the conductor 213
issued a ticket). She then said to the constable. The reason I didnt pay was the way he treated me and showed me up in front of
the passengers. As a result of the incident the omnibus was delayed for fifty minutes. The correct fare for the respondent was
6d. The minimum excess fare was 3d. If the respondent had originally stated that she wished to go to Crystal Palace she would
have been issued with a 6d ticket.
As regards the first information the magistrate was invited to find that there was evidence that the respondent left the bus
without having paid her fare and with intent to avoid payment (contrary to s 91(2)(a) of the Act of 1936) not that she had paid her
fare for a certain distance and had proceeded beyond that distance with the intention of avoiding payment (contrary to s 91(2)(b)
of that Act). As regards the second information (under s 91(1)), the magistrate intimated in the Case Stated that no request was
made to the respondent to produce or deliver up her ticket or pay her fare from the place from which she started, which was 6d.
The only request made by the conductor was for an excess fare of 3d, the fare chargeable as an additional fare for travelling
beyond the distance for which the respondent had paid, which was not an offence with which she had been charged. The
magistrate intimated to the appellants solicitor, at the end of the case for the prosecution, that so far as the two charges preferred
against the respondent were concerned he did not see that her actions were wrong. Accordingly he held that the respondent had
no case to answer on the charges and dismissed both informations. The appellant now appealed to the Divisional Court.

E H P G Wrightson QC for the appellant, the prosecutor.


F P Shier for the respondent, the passenger.

8 March 1963. The following judgments were delivered.

LORD PARKER CJ stated the charges laid by the two informations, summarised the facts and continued: section 91(1) of the
London Passenger Transport Act, 1936, though the wording is not particularly happy, is intended to deal, and does deal, with the
ordinary case where a conductor is concerned whether a passenger has paid any fare or his full fare, and is making inquiries. In
my judgment this subsection is making it an offence, in effect, to obstruct the conductor in doing his duty of seeing that people
pay proper fares. It says:

Every passenger by any stage carriage shall on request by an officer or servant of the board either produce and
if so requested deliver up a ticket showing that his fare is paid or pay his fare from the place whence he started or give the
officer or servant his name and address

In my judgment, that does not mean that the servant of the board, the conductor or inspector, can go up to anybody and just ask
them for their name and address; equally, I do not think that it means that he has got to go and put each request separately: Will
you show me your ticket? Will you deliver up your ticket? Will you pay the fare? Will you give your name and address? It is
sufficient if in all the circumstances of the case the conductor or inspector makes known that the passenger must comply with one
or other of the requests.
Here it has been said, and I think that this influenced the magistrate, that the respondent was never expressly asked to
produce or deliver up a ticket. That is true but, of course, in the circumstances of this case, the conductor knew full well that she
had not got one, and it was idle and in my view completely unnecessary to convey to the respondent the alternative of producing
or delivering up a ticket. The real difficulty as I see it, however, is that when he gave her the alternative of paying, the alternative
which he gave her was the payment of 3d so that in all she would have paid from the place where she started 8d, whereas the
proper fare from the place where she started was 6d, and on that view only 1d could have been demanded.
Counsel for the appellant has urged that the fare from the place whence he 214 started in the subsection must mean the
fare from the place where the passenger started on the extra stage, the place where the original ticket terminated. That, I confess,
produces a great strain of language, but counsel urges that it is necessary to read it in that way, because otherwise pay his fare
from the place whence he started would mean that having paid 5d, the respondent would than have to pay an extra 6d. In my
judgment, however, that is quite the wrong approach, because as it seems to me pay his fare must mean: make up the full
payment of his fare. The matter becomes really plain when one considers that this provision is something which has been
brought forward in almost identical language from the earlier Railway Acts f, and covers such matters, as the cases show, of
travelling first class on a third class ticket g. It really would be idle in those circumstances to say that pay his fare meant that
the passenger had to pay the full first class fare over and above the lower fare that he had paid. It seems to me there that pay his
fare must mean pay the full fare less what he has already paid, and read in that way, which it seems to me is the natural way,
there is no need to strain the language of the words the place whence he started. The place whence he started means the place
where he got on the bus. Applying that construction to the present case, this respondent was not given the option of making up
the fare of 5d to the full fare of 6d which would involve only 1d; it was 3d that was demanded of her. In my judgment the
magistrate was perfectly right at the end of the prosecutions case in saying that on no view could this respondent be brought
within that subsection.
________________________________________
f Section 5(1) of the Regulation of Railways Act, 1889, is, mutatis mutandis, identical with s 91 of the Act of 1936, and s 5(3) of the former
Act is very similar to s 91(2) of the Act of 1936. Section 5 of the Act of 1889 replaced similar provisions in s 103 of the Railway Clauses
Consolidation Act, 1845
g See, for example, Gillingham v Walker, (1881), 44 LT 715; Noble v Killick, (1891), 60 LJMC 61

So far as sub-s (2) is concerned, there are two provisions which are material; para (a) is dealing with a contravention of this
sort. It says:

If any person (a) having travelled on any stage carriage of the board leaves or attempts to leave it without
having paid his fare and with intent to avoid payment thereof
The learned magistrate in the present case felt, as I understand it, that this respondent should have been charged under para (b), h,
in that she was proceeding beyond the distance covered by the original ticket without paying the additional fare. In my judgment,
however, the provision covered by para (b) is dealing with quite a different situation to that covered by para (a), namely, the case
of a traveller who knowingly and wilfully proceeds beyond the distance covered by the ticket, in other words, somebody who has
a dishonest mind. In other words, the provision in para (a) is dealing with the case of somebody who leaves or attempts to leave
without paying his fare, which must mean his full fare, and with intent to avoid payment. The words and with intent to avoid
payment do not in my judgment mean that the prosecution have to prove some dishonest motive, but merely that the passenger
intended not to pay what in fact turns out to be the proper fare. It does not involve any dishonest motive at all; the passenger
might have done it, as indeed the respondent may have done it in the present case, in the genuine belief that the conductor was
wrong, and that it was only a 5d fare in all. Viewed in that way, I think that the prosecution were fully entitled to say to
themselves: if we proceed under para (b) we have got to prove that this woman is dishonest or was dishonest at the time when
she was telling the police officer:
________________________________________
h For the material terms of para (b) see note *, p 212, ante

Ive come to Crystal Palace Parade for the last twelve months and I only pay 5d. The reason I didnt pay was the
way he treated me and showed me up in front of the passengers.
215

Accordingly in so far as the learned magistrate said that this respondent ought to have been charged under para (b) I think that he
was wrong.
There is, however, a suggestion, I think, in para 4 of the Case Stated that one of the reasons why he dismissed this
information at the end of the prosecutions case was that he was not satisfied that there was any real evidence as to her intent to
avoid payment. In so far as that means a dishonest intent, the learned magistrate may have been right, but in my judgment, as I
have already said, with intent to avoid payment does not involve any dishonest motive, and viewed in that way there was, of
course, ample evidence to show that she left the bus with intent to avoid payment.
In those circumstances, in my judgment this case should go back to the learned magistrate with a direction that he should
proceed and determine the information under s 91(2)(a).

ASHWORTH J. I agree.

WINN J. I agree.

Case remitted accordingly

Solicitors: Stephen G Jones, solicitor, London Transport Board (for the appellant); Arthur E Eves & Jones (for the respondent).

N P Metcalfe Esq Barrister.


[1963] 2 All ER 216

R v Clow
CRIMINAL; Road Traffic, Criminal Procedure

COURT OF CRIMINAL APPEAL


LORD PARKER CJ, ASHWORTH AND WINN JJ
25 MARCH 1963

Road Traffic Dangerous driving Causing death by dangerous driving Indictment charging driving at a speed and in a
manner dangerous to the public Whether bad for duplicity Road Traffic Act, 1960 (8 & 9 Eliz 2 c 16), s 1(1).

Criminal Law Indictment Duplicity Whether separate offences may be charged conjunctively if they arise out of the same
incident.

The appellant was charged with causing death by dangerous driving, contrary to s 1 of the Road Traffic Act, 1960 a, the
particulars of the indictment being that he caused the death of one F-C by the driving of a motor vehicle on a road at a speed and
in a manner which was dangerous to the public having regard to all the circumstances of the case. A submission that the charge
was bad for duplicity was overruled by the trial judge, and the appellant was convicted. On appeal,
________________________________________
a Section 1(1) so far as material, is set out at p 217, letter h, post

Held The appellant had been rightly convicted and the indictment was properly drawn, because, even if the offences charged
were separate offences, it was permissible to charge them conjunctively where, as in the present case, the matter related to one
single incident (see p 219, letters e and f, post).
Dicta of Lord Coleridge J, in R v Jones, Ex p Thomas ([1921] 1 KB at p 635), of Avory J, in R v Surrey JJ., Ex p Witherick
([1931] All ER Rep at p 808), and of Humphreys J, in Thomson v Knights ([1947] 1 All ER at p 113), applied.
Appeal dismissed.

Notes
As to duplicity in indictments, see 10 Halsburys Laws (3rd Edn) 390, para 707; 33 Halsburys Laws (3rd Edn) 623, para 1048;
and for cases on the subject, see 14 Digest (Repl) 248251, 21462174.
216
As to causing death by dangerous driving, see 33 Halsburys Laws (3rd Edn) 622, para 1047.
For the Road Traffic Act, 1960, s 1, see 40 Halsburys Statutes (2nd Edn) 711.
Cases referred to in judgment
Cotterill v Lempriere (1890), 24 QBD 634, 62 LT 695, 54 JP 583, 42 Digest 728, 1504.
R v Courtley (1958) JCL 200.
R v Jones, Ex p Thomas [1921] 1 KB 632, 90 LJKB 543, 124 LT 668, 85 JP 112, 42 Digest 873, 217.
R v Slater, Ex p Bowler (1903), 67 JP 299, 26 Digest (Repl) 553, 2233.
R v Surrey JJ, Ex p Witherick [1931] All ER Rep 807, [1932] 1 KB 450, 101 LJKB 203, 146 LT 164, 95 JP 219, Digest Supp.
R v Wells JJ, etc, Ex p Clifford (1904), 91 LT 98, 68 JP 392, 42 Digest 873, 216.
R v Wilmot [1933] All ER Rep 628, 149 LT 407, 97 JP 149, 24 Cr App Rep 63, Digest Supp.
Thomson v Knights [1947] 1 All ER 112, [1947] KB 336, [1947] LJR 445, 176 LT 367, 111 JP 43, 2nd Digest Supp.

Appeal
This was an appeal by Rowland Charles Clow against his conviction at Aylesbury Assizes on 27 November 1962, before Widgery
J and a jury, of causing death by dangerous driving. He was fined 250 and disqualified for holding a driving licence for five
years. The appeal was on the ground that the indictment, which had charged that, on 29 July 1962, he caused the death of
Carolyn Cecile Fox-Carlyon by the driving of a motor vehicle on a road at a speed and in a manner which was dangerous to the
public having regard to all the circumstances of the case, was bad for duplicity.
The authority and cases noted belowb were cited during the argument in addition to the cases referred to in the judgment.
________________________________________
b Archbolds Criminal Pleading, Evidence and Practice (35th Edn), para 2839; R v Perry, (1945), 31 Cr App Rep 16; R (Bowden) v Belfast JJ,
[1952] NI 91

C J S French for the appellant.


D H Wild for the Crown.

25 March 1963. The following judgment was delivered.

LORD PARKER CJ delivered the following judgment of the court. The appellant was convicted at Aylesbury Autumn Assizes
of causing death by dangerous driving, and was sentenced to be fined 250 and to be disqualified for five years. He now appeals
against his conviction by leave of the single judge on a point which we are told has given rise to a difference in practice
throughout the country. It concerns the form of the indictment, which, in this case, charged the appellant with causing the death
of a certain person by driving a motor car at a speed and in a manner dangerous to the public having regard to all the
circumstances of the case. Objection was taken before the learned judge that that charge was bad for duplicity. That submission
was overruled but is renewed before this court.
Section 1(1) of the Road Traffic Act, 1960, is in this form:

A person who causes the death of another person by the driving of a motor vehicle on a road recklessly, or at a speed
or in a manner which is dangerous to the public, having regard to all the circumstances of the case, including

It is said that that subsection lays down three separate offences, namely, causing death by driving (a) a motor car recklessly; (b) at
a speed dangerous to the public; (c) in a manner dangerous to the public. This court confess that, free from authority, they would
be inclined to take the view that that subsection lays down only two offences, causing death by reckless driving having regard to
all the circumstances of the case, and causing death by dangerous driving whether as 217 a result of speed or in some other
manner, again in all the circumstances of the case. The matter, however, is not free from authority, because, though s 1 lays down
a new offence of causing death by reckless or dangerous driving, driving in these ways has been the subject of legislation for a
long time past. In R v Wells, etc JJ, Ex p Clifford, a Divisional Court were concerned with the Motor Car Act, 1903, s 1(1),
providing that:

If any person drives a motor car on a public highway recklessly or negligently, or at a speed or in a manner which is
dangerous to the public, having regard to all the circumstances of the case, including the nature, condition and use of the
highway, and to the amount of traffic which actually is at the time, or which might reasonably be expected to be, on the
highway, that person shall be guilty of an offence

The applicant in an application for a writ of certiorari had been summoned and convicted of driving at a speed or in a manner
dangerous to the public. The court took the view in that case that the section created four separate offences, and that a summons
charging one alternatively to the other was bad for duplicity. That, again, was followed in R v Wilmot, where the particulars of
the offence alleged were that the appellant drove a motor car recklessly, or at a speed or in a manner which was dangerous. Again
it was held that there were three separte offences charged in the alternative and that the count was bad for duplicity.
As against that, there are a series of decisions to the effect that, whether these are separate offences or not, they can be
charged conjunctively. The first of those cases is R v Jones, Ex p Thomas. This was again a summons under s 1 of the Motor Car
Act, 1903, and the applicant was charged that he drove recklessly and at a speed which was dangerous to the public, having
regard to all the circumstances. It was held that, as the driving of the car was one indivisible act which might constitute both the
offences charged, the charge was not bad for duplicity. This was a court which consisted of Lord Coleridge J, Avory and Salter
JJ. Lord Coleridge, who gave the leading judgment, referred to R v Wells and other cases. After going through them, he
approved R v Wells, where offences were charged in the alternative, but he went on to say ([1921] 1 KB at p 635):

Where the offences charged consist of one single act they may be made the subject of a single court. Here the
defendants act was one and indivisiblethe act of driving which might be both reckless and at a speed dangerous to the
public.

That case was followed by Avory J, himself sitting again in a Divisional Court with Swift and Humphreys JJ, in R v Surrey JJ, Ex
p Witherick. There the charge was that the applicant drove a motor car without due care and attention or without reasonable
consideration. The court held that that was bad as charging the matters in the alternative. In giving his judgment, Avory J went
through the cases to which I have referred, Jones case and Wells case and said ([1931] All ER Rep at p 808.; [1932] 1 KB at p
452):

[The cases] clearly illustrate the distinction which is to be drawn between charging offences in the alternative and
charging that a man may by one act have committed two offences. In R. v. Jones, Ex p. Thomas it was held that a man
might properly be convicted of driving recklessly and at a speed dangerous to the public; since the act of driving was one
indivisible act
Finally, to complete the matter, in Thomson v Knights, the Divisional Court were concerned with a conviction for being
unlawfully in charge of a motor vehicle whilst under the influence of drink or a drug to such an extent as to be incapable of
having proper control of the vehicle. Again, it was contended that that charge 218 was bad for duplicity, and it is to be observed
that that eminent criminal judge, Humphreys J who had been party to the earlier decision of R v Surrey JJ, Ex p Witherick, said
this ([1947] 1 All ER at p 113; [1947] KB at p 338):

I would only observe that, in my view, the cases on the subject were admirably explained in the judgment of LORD
COLERIDGE in R. v. Jones, Ex p. Thomas, where he dealt with the case of R. v. Wells, etc. JJ., Ex p. Clifford on which
counsel for the appellant naturally relied, and also Cotterill v. Lempriere and R. v. Slater, Ex p. Bowler. He pointed out that
in each of those cases there were two separate offences charged of which a person had been convicted in one conviction,
which everybody knows is wrong and contrary to law. He pointed out that in the case with which he was then dealingof
a person charged under one of the sections of the Motor Car Act, 1903, with driving recklessly and at a speed dangerous to
the publichaving regard to the circumstances of the case the real offence was driving in a manner dangerous to the public
by reason of one or other of two things, and that a conviction in which the defendant was convicted of those supposed two
offences was really a conviction for one single act. The same reasoning applies to the present case. I think the appellant
was convicted of the offence of being in charge of a motor car whilst intoxicated or under the influence of something
which made him incapable of having proper control of the vehicle.

Accordingly, however illogical it may seem, the line of authority is clear, and is supported by Lord Coleridge, Avory J,
Humphreys J and others to the effect that, even if these are separate offences, yet it is permissible to charge them conjunctively as
in the present case if the matter relates to one single incident, as of course it does in the present case, the death of the unfortunate
lady concerned. In those circumstances, this court has no hesitation in upholding the ruling of the learned judge in this case, who,
incidentally, followed a similar ruling of Stable J in May, 1958, at Oxford Assizes in R v Courtley. The court has come to the
conclusion that the indictment here was properly drawn and although it may be that, if it was drawn merely in the form in a
manner dangerous, it would have covered the introduction of the element of speed, in fact the reference to speed was in fairness
to the defence who thereby would realise that one of the allegations going to be made was that of excessive speed. This appeal is
dismissed.

Appeal dismissed.

Solicitors: Registrar, Court of Criminal Appeal (for the appellant); Wilkins & Son, Aylesbury (for the Crown).

N P Metcalfe Esq Barrister.


219
[1963] 2 All ER 220

Jayne v National Coal Board


TORTS; Statutory Duty: INDUSTRY

MONMOUTHSHIRE WINTER ASSIZES


VEALE J
25 FEBRUARY 1963

Coal Mining Statutory duty Breach Impracticability of avoiding or preventing contravention Shot-firing Shot-firers
duty to ensure all persons out of danger zone Shot-firer detailing man to check, but not checking personally Meaning of
impracticable Coal Mines (Explosives) Order, 1956 (SI 1956 No 1767), Schedule, reg 35(2) (b) Mines and Quarriers Act,
1954 (2 & 3 Eliz 2 c 70), s 157.

Impracticability is a conception different from that of impossibility; the latter is absolute, the former introduces at all events some
degree of reason and involves some regard for practice (see p 223, letter f, post). For the purposes of s 157 a of the Mines and
Quarries Act, 1954, which makes it a defence in proceedings based on an alleged contravention of that Act to prove that it was
impracticable to avoid or prevent the contravention, impracticability is not confined to the consequences of insufficiency of the
state of knowledge at the time, for example failure to take a precaution against a danger not then known to exist or a precaution
not yet invented, but extends to methods of avoiding a danger which would cause a danger greater than that avoided (see p 224,
letters a to d, and p 223, letter a, post).
________________________________________
a For the material terms of s 157, see note (3), p 223, post

On 23 March 1960, the plaintiff who was employed by the defendants at their colliery as a repairer was one of a team of six
men engaged in ripping and packing at the roadhead of the supply road serving the coal face. A shot-hole having been prepared
at about 6.30 pm, the men sat down at the roadhead to eat. When the shot-firer had arrived and charged the hole, he let it be
known that he was ready to fire, and sent three of the men down the supply road to act as sentries. All the men dispersed. The
shot-firer believed that all the men had gone out of danger. He did not himself check that this was so, but he sent one of the men
back to look. The man gave a perfunctory look down the road, and reported that all was clear. The shot-firer fired and the blast
injured the plaintiff, who in fact, moving slowly, had just passed the shot-hole. Regulation 35(2)(b) of the Coal Mines
(Explosives) Regulations, 1956b, provided that no shot-firer should fire any shot unless he had ensured that all persons had
withdrawn from the danger zone or had taken proper shelter. By s 157 of the Mines and Quarries Act, 1954, it was a defence in
proceedings based on a contravention of regulations under the Act to prove that it was impracticable to avoid or prevent the
contravention. In an action by the plaintiff for damages for breach of statutory duty under reg 35(2)(b),
________________________________________
b For the terms of reg 35, see note (2), p 222, post

Held The duty imposed by reg 35(2)(b) was an absolute duty and the shot firer was in breach of it; s 157 of the Mines and
Quarries Act, 1954, did not afford a defence as, on the facts, there were additional precautions which the shot firer could have
taken but did not take (eg, he could have himself seen that the men had left the danger zone), and, therefore, it had not been
impracticable to have avoided the contravention.
Notes
As to breach of statutory duty by a mine-owner, see 24 Halsburys Laws (3rd Edn) 566, 567, para 1136, and as to the defence that
it was impracticable to avoid the breach, see ibid, 568, para 1138, text and note (p); and for cases on the subject, see 3rd Digest
Supp.
For the Mines and Quarries Act, 1954, s 157, see 34 Halsburys Statutes (2nd Edn) 627.

Cases referred to in judgment


Brown v National Coal Board [1960] 3 All ER 594, [1961] 1 QB 303, [1960] 3 WLR 892, affd, [1962] 1 All ER 81, [1962] AC
574, 3rd Digest Supp.
220
Edwards v National Coal Board [1949] 1 All ER 743, [1949] 1 KB 704, 2nd Digest Supp.
Marshall v Gotham Co Ltd [1954] 1 All ER 937, [1954] AC 360, 3rd Digest Supp.
Sanderson v National Coal Board [1961] 2 All ER 796, [1961] 2 QB 244, [1961] 3 WLR 73, 3rd Digest Supp.

Action
In this action, Roderick Hewet Jayne, the plaintiff, claimed damages against the National Coal Board, the defendants, for
personal injuries caused by the negligence and/or breach of statutory duty of the defendants, their servants or agents at their
Hafodyrynys colliery, Monmouthshire.
The following facts are summarised from the judgment of Veale J. At the material time the plaintiff was employed by the
defendants as a repairer in the big vein seam, B3 district of the Hafodyrynys colliery. The long wall face in the seam was
supplied by two roads, one of which, known as the supply road, was at the right of the face. This road was supported by steel
arches and was a relatively wide road affording ample headroom. The ripping lip was at the road-head of the supply road and in
order to get to the face it was necessary to bend down under the lip. The face had a comparatively low roof and could be
traversed only by crawling or bending almost double. On the afternoon shift on 23 March 1960, both coaling and repairing or
ripping were taking place, the plaintiff being one of a team of six repairers engaged in ripping and packing. Shots were fired in
the ripping during the shift with the object of getting down stone so that another steel arch could be put into position. The
plaintiff was extending the pack and the stone for his packing was provided from the debris shot down from the ripping lip. By
6.30 pm the pack had been partly extended, but the shots which had been fired in the ripping lip had failed to do their job
completely and it was necessary to fire a further shot. A hole for this shot was bored and the deputy and one Lewis, the shot-firer,
were summoned. The men then had their food, the team of repairers sitting down by the pack. When Lewis came to the
roadhead he allowed the men to go on with their food while he charged the shot-hole. He also ran out a stretch of cable and the
men saw him doing this. He then let it be known that he was ready to fire and all the men, including the plaintiff, knew that he
was going to fire a shot. He ordered three of the men to go down the supply road to act as sentries. All the men got up and
dispersed; except for the sentries and the plaintiff, they went along the face. Lewis believed that all the men, except the sentries,
had gone or were going along the face, but he took no steps to check or see them away from the danger zone. Meanwhile the
plaintiff went the other way into the road, moving slowly. He wanted to take off his coat and hang it on a cog stick in the side of
the road and then to return along the face; there was nothing unreasonable in his so acting, but he acted lethargically. Lewis sent
one of the men named Jarvis back to the roadhead to see whether all the men had gone. Jarvis gave a perfunctory glance down
the road. From his probable position, crouched under the ripping lip, he would have been unable to see the top or upper part of
the road until some considerable distance along it. He saw a light or reflection which he took to be the sentries well down the
road, but which was in fact the reflection of the plaintiffs cap lamp while the plaintiff, standing upright, was hanging his coat on
the cog stick. Jarvis returned and reported to Lewis that all was clear. The rest of the cable was unrolled and Lewis fired within a
very short time. At that moment the plaintiff had just passed the shot-hole and was bent down getting under, or having just got
under, the low roof on his way to the face. He was seriously injured by the blast from the shot.
By para 3 of his statement of claim the plaintiff gave the following particulars of his allegations of breach of statutory duty
as follows:

(a) The said Lewis failed, before firing the said shot, to determine the danger zone; contrary to reg. 35(1) of the Coal
Mines (Explosives) 221Regulations, 1956c. (b) The said Lewis fired the said shot when he had not posted sentries or
placed appropriate fences as required by reg. 35(2)(a) of the said regulations and contrary thereto. (c) The said Lewis fired
the said shot when he had not ensured that all persons (in particular the plaintiff) had withdrawn from the danger zone or
taken proper shelter; contrary to reg. 35(2)(b) of the said regulations.
________________________________________
c For the terms of reg 35, see note (2), post

By heads (d) and (e) of para 3 the plaintiff also alleged negligence on the part of Lewis and on the part of Jarvis.
By para 3 of their defence the defendants denied the allegations of negligence and breach of statutory duty and pleaded, in
the alternative, that if they were in breach of reg 35(2)(b) it was impracticable to avoid or prevent such contravention in that the
plaintiff extinguished his cap lamp and lay down to sleep in a place of concealment which could not be identified; and the
defendants stated that they would rely on s 157 of the Mines and Quarries Act, 1954 d. The plaintiffs also pleaded contributory
negligence and breach by the plaintiff of s 89 and s 90 of the Act of 1954 and of reg 9 of the Coal and Other Mines (General
Duties and Conduct) Regulations, 1956.
________________________________________
d For the material terms of s 157, see note (3), post

B Gibbens QC and C Pitchford for the plaintiff.


R V Cusack QC and A G Davies for the defendants.

25 February 1963. The following judgment was delivered.

VEALE J having reviewed the evidence and found the facts (summarised at p 221, letter c, ante), continued. In these
circumstances I must consider the law that is applicable to the facts as I have found them. The Coal Mines (Explosives)
Regulations, 1956, lay down a rigorous code of procedure for firing shots e. Onerous duties are placed on shot-firers, and I have
no doubt deliberately onerous duties, for shot-firing is a great source of danger unless strictly controlled. The regulations are
designed for the protection of the shot-firer himself, for the protection of workmen, including stupid workmen, and indeed for the
protection of the mine itself.
________________________________________
e The Coal Mines (Explosives) Regulations, 1956, are set out in the schedule to the Coal Mines (Explosives) Order, 1956, SI 1956 No 1767;
for a summary, see 14 Halsburys Statutory Instruments (1st Re-issue) 82

The plaintiff alleges a breach of reg 35 of the regulations in three respects f. Under reg 35(1) it is said that Lewis did not
ascertain the danger zone. I think that he did, but even if he did not it did not cause this accident. Under reg 35(2)(a) it is said
that Lewis did not post sentries. He detailed sentries, though I doubt whether technically he posted them; but this is not the case
of a man who went past the place where a sentry should have been; it is the case of a man who never left the danger zone at all. I
do not think that any possible breach of reg 35(2)(a) had anything to do with this accident. The crux of this case is reg 35(2)(b):
________________________________________
f Regulation 35 is as follows: (1) Any shot-firer proposing to fire a shot shall before firing determine the danger zone likely to be created.
(2) No shot firer shall fire any shot unless he has(a) at each entrance to that danger zone either posted a sentry or placed an appropriate
fence conspicuously marked with the words danger and shot firing; (b) ensured that all persons have withdrawn from that zone or have
taken proper shelter; and (c) himself taken proper shelter

No shot firer shall fire any shot unless he has (b) ensured that all persons have withdrawn from that zone or have
take proper shelter.

This is an absolute duty both in the sense that it is imposed by regulation and in the sense that the duty is to ensure. There is no
doubt whatever in my mind that Lewis was in breach of this duty and it was a direct cause of this accident.
That is not the end of this case, because the defendants rely on s 157 of the Mines and Quarries Act, 1954. Under s 102(8)
of the Coal Mines Act, 1911, 222defendants had to show that it was not reasonably practicable to avoid or prevent the breach.
Under s 157 of the Act of 1954 the defendants must show that it was impracticable to avoid or prevent the contravention g.
There is a difference between impracticable and not reasonably practicable. In Brown v National Coal Board when the case
was before the Court of Appeal, Holroyd Pearce LJ said ([1960] 3 All ER at p 609; [1961] 1 QB at p 332):
________________________________________
g Section 157, so far as material, is as follows: It shall be a defence in any legal proceedings to recover damages in so far as the
proceedings are based on an allegation of a contravention in relation to a mine or quarry, of (a) a provision of this Act, of an order
made thereunder or of regulations to prove that it was impracticable to avoid or prevent the contravention.

The escape clause under the Act of 1954 is somewhat more restricted than that under the Act of 1911. It is slightly
more difficult to show that avoidance of the breach was impracticable than to show that it was not reasonably
practicable. The difference of meaning between the two expressions is hard to define; and further definitions of words that
in themselves express a shade of meaning are not helpful. But the difference, though hard to define, exists, and in a border
line case it might produce a difference in the result. The word reasonably has a slight tendency to modify the word
practicable. I respectfully agree with the words of LORD REID in Marshall v. Gotham Co. Ltd. [[1954] 1 All ER 937 at
p 942; [1954] AC 360 at p 373]: But, in my judgment, there may well be precautions which it is practicable but not
reasonably practicable to take.

In the same case in the House of Lords, Lord Denning said ([1962] 1 All ER 81 at p 91; [1962] AC 574 at p 598):

The escape clause in the Act of 1954 (s. 157) is a little different from that in the Act of 1911 (s. 102(8)). Previously
the owner might escape liability for damages if it were shown that it was not reasonably practicable to avoid or prevent
the breach. Now he escapes only if he proves that it was impracticable to avoid or prevent it.

The distinction therefore exists, but it is slight.


The word practicable may also be contrasted with the word possible. There is, in my view, clearly a difference between
these two words. The distinction between physically possible and reasonably practicable was referred to by Asquith LJ, in
Edwards v National Coal Board ([1949] 1 All ER 743 at p 747; [1949] 1 KB 704 at p 712). Had the legislature intended to lay
down possibility as a test, it would have been simple to use the word impossible in s 157; but they did not do so. In my view,
impracticability is a conception different from that of impossibility: the latter is absolute, the former introduces at all events
some degree of reason and involves at all events some regard for practice.
There are, therefore, three theoretical categories into which a set of circumstances requiring precautions may fall, namely,
impossibility, impracticability and not being reasonably practicable. Indeed, if one adds the standard of reasonableness
required at common law, there are four categories. There is, of course, no doubt that the boundaries of practicability are wider
than the boundaries of reasonable care as was emphasised by Donovan LJ, in Sanderson v National Coal Board ([1961] 2 All
ER 796 at p 800; [1961] 2 QB 244 at p 253) and in Edwards v National Coal Board, to which I have already referred, Tucker LJ
distinguished reasonable practicability and reasonable care. Into which category a set of circumstances does fall is a question
of degree, and one of fact, which has to be decided 223 in each case. My own view is that the gap between impossibility and
impracticability is not wide, but is nevertheless wider than the gap between impracticability and not being reasonably
practicable.
It is submitted on behalf of the plaintiff that impracticability amounts almost to impossibility, the distinction being that
practicable means possible in the state of knowledge at the time, the only limitation being the state of knowledge at the time.
I do not accept this argument. It is, I would have thought, clearly impracticable to take precautions against a danger which could
not be known to be in existence, or to take a precaution which has not yet been invented. I think, however, that impracticable
means more than this. For instance, if one imagines that as a result of firing a shot in a ripping a dangerous stone is exposed in
the roof, it might be immediately possible to remove it by boring and firing another shot; but to do so might well cause greater
danger in the surrounding roof than the stone itself. It would be, no doubt, possible to support such a stone forthwith with a
support which was in use supporting the roof elsewhere. I would without hesitation hold both these possible methods of dealing
with the danger to be impracticable. I find it harder to envisage circumstances in which a contravention could be excused
because it was not reasonably practicable to prevent it, but in which the same contravention could not be excused because it
was not impracticable to prevent it. It might be that a border line case referred to by Holroyd Pearce LJ, in Brown v National
Coal Board ([1960] 3 All ER at p 609; [1961] 1 QB at p 332) could arise in circumstances similar to those in Sanderson v
National Coal Board ([1961] 2 All ER 796 at p 800; [1961] 2 QB 244 at p 253) where appropriate evidence was calledno
evidence was called by the defendants in Edwards caseshowing the expenditure which would be involved in taking
precautions throughout the pit, the coalfield, or the country, or in circumstances where a danger such as a lack of fencing was of
very limited duration. In such a case the decision might be balanced on a knife edge, and the words to be applied as a test, not
reasonably practicable on the one hand, impracticable on the other, might affect the result. However this may be, except in the
case of what must now be very stale claims, the relevant word under the legislation relating to coalmines is impracticable,
though in other statutes the words practicable and reasonably practicable exist side by side, for instance, in s 4 and s 63 and
in s 28 and s 29 of the Factories Act, 1961,h.
________________________________________
h For s 4 and s 63 (practicable), see 41 Halsburys Statutes (2nd Edn) 248, 304; for s 28 and s 29 (reasonably practicable), see ibid, pp
271, 272

Returning now to the facts of the present case, the onus is on the defendants to prove that it was impracticable for Lewis to
avoid or prevent the contravention of his duty to ensure that all persons had withdrawn from the danger area. On the facts that I
have found, the answer to the question posed by counsel for the defendantswhat additional precaution could Lewis have taken?
is simple. There were many additional precautions which he could have taken. I need not enumerate them, but he could at the
least have done what he should have done, namely, seen for himself that all the men had left the danger zone. In my judgment
the defendants have failed to bring themselves within s 157. Were it necessary to do so, I would hold both Lewis and Jarvis
guilty of negligence at common law. Further, in my judgment I do not find any of the allegations of contributory negligence
made against the plaintiff to be proved, let alone any contravention of s 89 and s 90 of the Act of 1954, i, and reg 9 of the Coal and
Other Mines (General Duties and Conduct) Regulations, 1956, j. In my judgment the plaintiff is entitled to succeed in this action.
224
________________________________________
i 34 Halsburys Statutes (2nd Edn) 581, 582
j 14 Halsburys Statutory Instruments (1st Re-issue) 39

[His Lordship referred briefly to agreed medical reports and awarded the plaintiff 600 general damages and 455 2s agreed
special damages.]

Judgment for the plaintiff.

Solicitors: T S Edwards & Son, Newport, Monmouthshire (for the plaintiff); Colborne, Coulman & Lawrence, Newport,
Monmouthshire agents for F W Dawson, National Coal Board, Cardiff (for the defendants).

A G Davies Esq Barrister.


[1963] 2 All ER 225

Jordan v Burgoyne
CRIMINAL; Criminal Law

QUEENS BENCH DIVISION


LORD PARKER CJ, ASHWORTH AND WINN JJ
19 MARCH 1963

Public Order Public meeting Threatening, abusive or insulting words Breach of peace Group of persons in audience
intent on obstructing speaker Words in fact insulting to them Speaker must take his audience as he finds them Whether
contravention of Public Order Act, 1936 (1 Edw 3 & 1 Geo 6 c 6), s 5.

The respondent was addressing a crowd of about five thousand people at a public meeting, amongst whom were two hundred or
three hundred young persons, positioned together immediately in front of the speakers platform, who intended to prevent any
meeting being held at all if they could. That group contained Jews, supporters of the campaign for nuclear disarmament and
communists. The respondent used words insulting to that group. The appellant, a superintendent of the Metropolitan Police,
stopped the meeting and the police had the greatest difficulty in restoring order, twenty people being arrested for offences
involving breaches of the Queens peace. The respondents appeal to quarter sessions against his conviction under s 5 of the
Public Order Act, 1936a, of using insulting words at a public meeting whereby a breach of the peace was likely to be occasioned
was allowed on the ground that, though the words used were highly insulting, they were not likely to lead ordinary, reasonable
persons attending the meeting to commit breaches of the peace. On appeal by the prosecution,
________________________________________
a Section 5 is set out at p 227, letter b, post

Held For the purposes of s 5 of the Public Order Act, 1936, a person had to take his audience as he found them and, if the
words that he used to a particular audience, or part of it, were in fact likely to provoke a breach of the peace, then he would be
guilty of an offence under s 5 (see p 227, letter d, post); in the present case the words used by the respondent were intended to be
and were deliberately insulting to the body of persons who were being restrained by the police and there had been a clear
contravention of s 5 of the Act of 1936.
Appeal allowed.

Notes
As to offensive conduct in a public place conducive to a breach of the peace, see 10 Halsburys Laws (3rd Edn) 582, para 1081;
and for a case on the subject, see 15 Digest (Repl) 785, 7359.
For the Public Order Act, 1936, s 5 see 5 Halsburys Laws (2nd Edn) 1091.

Case Stated
This was a Case Stated by the chairman of the County of London Quarter Sessions to whom John Colin Campbell Jordan (Mr
Jordan), appealed against a conviction and sentence of two months imprisonment imposed at Bow Street Magistrates Court on
20 August 1962, on an information laid by George Burgoyne a superintendent of the Metropolitan Police, that, on 1 July 1962,
Mr Jordan 225 at Trafalgar Square at a public meeting used insulting words whereby a breach of the peace was likely to be
occasioned, contrary to s 5 of the Public Order Act, 1936. The following facts were found. On 1 July 1962, there was a public
meeting in Trafalgar Square. At about 3.0 pm, there was a crowd of about two thousand people, of which two hundred or three
hundred young people were positioned together immediately in front of the speakers platform. This group contained many Jews,
supporters of the campaign for nuclear disarmament and communists. That group intended to prevent any meeting being held at
all. Between the crowd and the speakers platform was a line of police. The present appellant, George Burgoyne, was
responsible for maintaining the Queens peace, particularly in the vicinity of the speakers platform which was on the plinth at the
base of Nelsons column. There was disorder throughout the whole of the meeting mainly, but not exclusively, among the group
of two hundred or three hundred young people, who repeatedly attempted to attack the platform and the speakers and their
supporters. In other parts of the crowd the disorder was mainly confined to heckling. The meeting was addressed by one John
Hutchyns Tyndall. During the course of Mr Tyndalls address, the present appellant stopped the meeting on four occasions to
restore order. After Mr Tyndall had addressed the meeting, it was addressed by Mr Jordan who read a speech to the crowd. At
about 5.15 pm, the crowd had grown to about five thousand people. There was considerable opposition from other parts of the
crowd during the speeches, but the main disorder was concentrated in the group at the front. At about 5.15 pm, Mr Jordan used
words which included the following: more and more people every day are opening their eyes and coming to say with us:
Hitler was right. They are coming to say that our real enemies, the people we should have fought, were not Hitler and the
national socialists of Germany but world Jewry and its associates in this country. There was complete disorder, an outcry and a
general surge forward by the crowd towards the speakers platform. When this happened, the present appellant stopped the
meeting. The police had the greatest difficulty in restoring order. Twenty persons present at the meeting were arrested for
offences involving breaches of the Queens peace. The majority of these arrests were made in the dispersal of the crowd after
5.15 pm Quarter Sessions allowed Mr Jordans appeal against conviction, and the present appellant now appealed.

Graham Swanwick QC and W M Howard for the appellant.


The respondent, Mr Jordan, appeared in person.

19 March 1963. The following judgments were delivered.

LORD PARKER CJ stated the facts, and continued. Quarter sessions, in giving their opinion, said

4. We found as a fact that a great many of the words used by [Mr. Jordan] were highly insulting but were not likely to
lead ordinary reasonable persons attending the meeting in Trafalgar Square to commit breaches of the peace by committing
assaults and accordingly allowed [Mr. Jordans] appeal against conviction. 5. The question for the opinion of the High
Court is as follows: Whether the words in s. 5 of the Public Order Act, 1936, Where-by a breach of the peace was likely to
be occasioned can properly be construed to mean likely to lead to a breach of the peace by the ordinary citizen in the
circumstances of the case.

Speaking for myself, I have great difficulty in understanding what quarter sessions were intending to convey. It seems to me,
however, that what they had in mind was a hypothetical audience of ordinary reasonable citizens, whatever their creed, faith, race
or political views might be. In other words, they were eliminating from the audience anybody who was, for instance, intent on
breaking up the meeting whatever words the speaker used. It may be that that is what 226 quarter sessions had in mind, but, even
if that be so, I cannot myself, having read the speech, imagine any reasonable citizen, certainly one who was a Jew, not being
provoked beyond endurance, and not only a Jew but also a coloured man and quite a number of people of this country who were
told that they were merely tools of the Jews and that they had fought in the war on the wrong side, and matters of that sort.
Be that, however, as it may, there is no room here, in my judgment, for any test whether any member of the audience is a
reasonable man or an ordinary citizen, or whatever epithet one might like to apply. This is a public order Act, something to keep
public order in public places, and it provides, by s 5, that:

Any person who in any public place or at any public meeting uses threatening, abusive or insulting words or behaviour
with intent to provoke a breach of the peace or whereby a breach of the peace is likely to be occasioned, shall be guilty of
an offence.

This is, as I have said, a public order Act, and if in fact it is apparent that a body of persons are present (let me assume in Mr
Jordans favour that the persons present are a body of hooligans), yet if words are used which threaten, abuse or insultall very
strong wordsthen the speaker must take his audience as he finds them, and, if those words to that audience, or that part of that
audience, are likely to provoke a breach of the peace, then the speaker is guilty of an offence. I will assume in Mr Jordans
favour that this body of young persons was a body of hooligans (though I am not saying that they were), and I assume that they
came with the preconceived idea of preventing him from speaking, yet the police prevented them from obstructing the speaker,
the police enabled Mr Jordan to speak, and then in his opening words directed to those two hundred or three hundred people he
said:

As for the red rabble here present with us in Trafalgar Square it is not a very good afternoon at all. Some of them are
looking far from wholesome, more than usual I mean. We shall of course excuse them if they have to resort to smelling-
salts or first-aid. Meanwhile, let them howl, these multiracial warriors of the Left. It is a sound that comes natural to them,
it saves them from the strain of thinking for themselves.

Those were words which were intended to be and were deliberately insulting to the body of persons who were being restrained by
the police, and on that and that alone it seems to me that there was a clear contravention of s 5 of the Act of 1936.
Mr Jordan, who has conducted his own case with great skill and industry, has been inclined to elevate this case into a cause
clbre in the sense that, if he is convicted here, there is some inroad into the doctrine of free speech. It is, in my judgment,
nothing of the sort. A man is entitled to express his own views as strongly as he likes, to criticise his opponents, to say
disagreeable things about his opponents and about their policies, to do anything of that sort, but what he must not doand these
are the words of the Acthe must not threaten, he must not be abusive and he must not insult them, insult in the sense of hit by
words. It seems to me this is a perfectly clear case, that Mr Jordan was guilty of the offence charged, and that this appeal must be
allowed and the Case must go back to quarter sessions with a direction to dismiss the appeal.

ASHWORTH J. I agree.

WINN J. I agree.

Appeal allowed. Case remitted.

Solicitors: Solicitor, Metropolitan Police.

N P Metcalfe Esq Barrister.


227
[1963] 2 All ER 228

Doughty and another v Inland Revenue Commissioners


TAXATION; Estate Duty

CHANCERY DIVISION
UNGOED-THOMAS J
7, 8 FEBRUARY, 7 MARCH 1963

Estate Duty Passing of property Discretionary trust Replaced after death by entitlement to definite benefit Finance Act,
1894 (57 & 58 Vict c 30), s 1, s 2(1)(b).

Estate Duty Passing of property Settlor a trustee of settlement Property not included under s 2(3) but included under s 1 or
s 2(1)(b) Whether duty exigible Finance Act, 1894 (57 & 58 Vict c 30), s 2(3).

In 1949 the settlor made a settlement by cl 3 of which he directed that, during his lifetime, the income of the trust fund thereby
constituted should be held on discretionary trusts for his grandchildren, the children of his son D (who at the date of the
settlement had two children, P E and P A), and subject thereto should be accumulated as an accretion to capital. By cl 4 of the
settlement the income and capital of the trust fund were to be held on trust after the settlors death for such of his grandchildren
living at the death of D as being male attained the age of twenty-one or being female attained that age or married. By cl 5, if the
settlor predeceased D, income was to be applied during Ds lifetime at the trustees discretion for the benefit of the grandchildren
and the children of grandchildren who predeceased D. Clause 6 provided that if, at the settlors death, there were no
grandchildren other than P E and P A two-thirds of the trust fund should be held on trust for such of them as should attain the age
of twenty-five years and if both should do so in equal shares and that cl 4 and cl 5 should apply only to the remaining one-third of
the trust fund provided that neither P E nor P A nor any child of either of them should take any benefit under cl 4 or cl 5 if at the
date of Ds death there should be any other grandchildren. The settlor was a trustee of the settlement until his death in 1958. D
survived the testator, but had, at the settlors death, no children other than P E and P A, both of whom were then over the age of
twenty-one but under the age of twenty-five. On the question whether estate duty was exigible under s 1 and s 2(1)(b) of the
Finance Act, 1894, on the trust fund, on the settlors death, it being common ground that no distinction for estate duty purpose
was to be drawn between the two-thirds and the one-third of the trust fund,

Held Estate duty was exigible for the following reasons


(i) the change which took place on the settlors death was (a) not merely a change of a source of title, but also a change of
possession or enjoyment and, being a change from the possibility of interest under a discretionary trust to an entitlement to a
definite portion of income, constituted a passing within s 1 a of the Act of 1894 (see p 234, letter c, and p 233, letter d, post; Scott
v Inland Revenue Comrs ([1936] 3 All ER 752); Re Hodsons Settlement, Brookes v A-G ([1939] 1 All ER 196); and dictum of
Lord Evershed MR, in Re Parkes Settlement Trusts, Midland Bank Executor & Trustee Co Ltd v Inland Revenue Comrs ([1956]
1 All ER 833 at p 838) applied), and was (b) the replacement of one set of trusts by another set of trusts and the determination of
one set of interests and the commencement of another set of interests and this constituted a passing within s 2(1)(b) b of the Act of
1894 (see p 234, letter f, post).
________________________________________
a Section 1, s 2(1)(b), s 2(3), are set out at p 232, letters g to i, post
b Section 1, s 2(1)(b), s 2(3), are set out at p 232, letters g to i, post

(ii) where property fitted both a description within s 2(1) and another description within s 2(3) the notional non-inclusion of
the property by virtue of its fitting a description in s 2(3) c did not prevent its being included as property passing under the other
description that fitted it under s 1 or 228 s 2(1); accordingly s 2(3) did not exempt the trust fund from liability to duty (see p 236,
letter i, post).
________________________________________
c Section 1, s 2(1)(b), s 2(3), are set out at p 232, letters g to i, post

Notes
As to the meaning of passing on death, see 15 Halsburys Laws (3rd Edn) 10, para 14; as to property deemed to pass, see ibid,
1228, paras 1949; and for cases on the subject see 21 Digest (Repl) 1338, 42138; as to non-inclusion of trust property, see 15
Halsburys Laws (3rd Edn) 38, para 74; and for a case on the subject, see 21 Digest (Repl) 94, 440.
For the Finance Act, 1894, s 1, s 2(1)(b), s 2(1)(d), s 2(3), see 9 Halsburys Statutes (2nd Edn) 348, 350, 353.

Cases referred to in judgment


A-G v Farrell [1931] 1 KB 81, 99 LJKB 605, 143 LT 639, 21 Digest (Repl) 29, 108.
Escoigne Properties Ltd v Inland Revenue Comrs [1958] 1 All ER 406, [1958] AC 549, [1958] 2 WLR 336, 3rd Digest Supp.
Hodsons Settlement, Re, Brookes v A-G [1939] 1 All ER 196, [1939] Ch 343, 108 LJCh 200, 160 LT 193, 21 Digest (Repl) 31,
117.
Parkes Settlement Trusts, Re, Midland Bank Executor & Trustee Co Ltd v Inland Revenue Comrs [1956] 1 All ER 833, [1956] 1
WLR 397, 21 Digest (Repl) 36, 136.
Public Trustee v Inland Revenue Comrs [1960] 1 All ER 1, [1960] AC 398, [1960] 2 WLR 203, 21 Digest (Repl) 18, 58.
Scott v Inland Revenue Comrs [1936] 3 All ER 752, [1937] AC 174, 106 LJCh 36, 21 Digest (Repl) 12, 38.

Adjourned Summons
By an originating summons dated 31 May 1962, Donald Arthur Doughty and Roger Limmer Harrison, the plaintiffs, the present
trustees of a settlement made on 6 May 1949, between Arthur Kirby Doughty (the settlor) of the one part and the settlor and the
said Donald Arthur Doughty of the other part, applied for the determination of the following questions: Whether on the true
construction of the said settlement and s 1 and s 2 of the Finance Act, 1894, and in the events which had happened estate duty
became payable under s 1 or under s 2(1)(b) or s 2(1)(d) on the death of the settlor in respect of any of the property subject to the
trusts of the settlement at the date of his death on the footing that such property passed or was deemed to pass on such death, and
if so whether the proportion of such property on which estate duty was so payable was:(a) the whole, or (b) two-thirds, or (c)
any and what other proportion of such property.
The material provisions of the settlement and the relevant facts are set out in the judgment.
The cases noted belowd were cited during the argument in addition to those referred to in the judgment.
________________________________________
d A-G v Lloyds Bank Ltd [1935] All ER Rep 518, [1935] AC 382, Burrell v A-G, [1936] 3 All ER 758, [1937] AC 286, Westminster Bank Ltd
v A-G, [1939] 2 All ER 72, [1939] Ch 610, Oakes v New South Wales Commissioner of Stamp Duties, [1953] 2 All ER 1563, [1954] AC 57,
Parker v Lord Advocate, [1960] 1 All ER 20, [1960] AC 608

Heyworth Talbot QC and R Instone for the plaintiffs.


Arthur Bagnall QC and E Blanshard Stamp for the Crown.

Cur adv vult

7 March 1963. The following judgment was delivered.

UNGOED-THOMAS J read the following judgment. This case raises a question of liability to estate duty under s 1 and s 2 of
the Finance Act, 1894. The plaintiffs are the present trustees of a settlement dated 6 May 1949, made by Arthur Kirby Doughty.
The original trustees were the settlor and his son, Donald Arthur Doughty. The settlor died on 12 April 1958, and the primary
beneficiaries under the settlement were the settlors grandchildren, being the children of his son Donald. There were two such
grandchildren at the 229 time of the settlement, namely, Patricia and Peter. Both are still living and none has been born since, but
there is a possibility of more such grandchildren. At the time of the settlement Patricia and Peter were minors and at the settlors
death they were over the age of twenty-one and under the age of twenty-five.
I will now give a resume of the general effect of the settlement and then I will quote the relevant parts of the settlement.
Clause 3 disposed of the beneficial interests during the settlors life. The settlor gave the income of the trust fund on
discretionary trusts for the benefit of the grandchildren and directed any income not so applied to be accumulated as accretion to
capital. Clause 4 provided for the position after the settlors death. Under that clause the trust fund was given to the
grandchildren living at Donalds death who attained the age of twenty-one, or, if female, married, with a provision for
substituting deceased grandchildrens children in their parents place. Clause 5 provided for the position if the settlor predeceased
Donald, which is what happened. Under that clause income was given during Donalds life for the benefit of the grandchildren
and the children of such grandchildren as predeceased Donald in such proportions as the trustees should decide. There is no
provision in that clause for the accumulation of any undistributed income. Clause 6 dealt with the position after the settlors
death if there were then no granchildren except Patricia and Peter, which was the position. By sub-cl (i) two-thirds of the trust
fund was given for such of themPatricia and Peteras shall attain the age of twenty-five years and if both of them shall
attain that age in equal shares and under sub-cl (ii) one third of the trust fund remained subject to the provisions of cl 4 and cl 5,
subject, however, to the proviso that neither Patricia nor Peter nor any child of Patricia or Peter should take any benefit under the
provisions of cl 4 and cl 5 if at Donalds death there should be any grandchildren other than Patricia and Peter.
I now turn to the precise terms of the settlement. After reciting that the settlor was entitled to and was the registered holder
of shares in a company known as F Doughty & Sons Ltd it defined the trust fund in these terms:

The trust fund shall mean be and include all and any of the following that is to say (a) the said shares (b) all or any
bonus shares received by the trustees in respect of the said shares which ought to be treated by the trustees as capital as a
result of a capitalisation of profits on the part of the company or otherwise (c) all or any moneys added to the trust fund
under and by way of the express provisions to accumulate hereinafter contained (d) all moneys and investments paid or
transferred to and accepted by the trustees as additions to the trust fund (e) the investments and property from time to time
representing the said shares and each of the foregoing.

Then it provided that

The grandchildren shall mean and include all and any of the grandchildren of the settlor being the children male and
female of the settlors son namely the said Donald Arthur Doughty party hereto.

I will now read cll 3-8:

3. The trustees shall during the life of the settlor stand possessed of the trust fund upon the trusts and with and subject
to the powers and provisions following that is to say: (i) Upon trust to pay or apply the income thereof or such part or parts
of such income as the trustees in their absolute and uncontrolled discretion (having regard to the age and requirements of
the grandchildren) shall from time to time think fit for or towards the maintenance education or benefit of all or any of the
grandchildren. (ii) Upon trust to accumulate such parts (if any) of the income of the trust fund as shall not be paid or
applied as aforesaid in the way of compound interest by investing the same and the resulting income thereof from time to
time in any of the investments hereinafter authorised with power to vary such investments and all accumulations so
produced and the investments for 230 the time being representing the same shall be an accretion to follow the destination
of the property from which they arose and as one fund with such capital for all purposes.
4. From and after the death of the settlor and subject to the provisions of this deed hereinafter contained the trustees
shall stand possessed of the trust fund (both as to capital and as to income) upon trust for such of the grandchildren as shall
be living at the date of the death of the said Donald Arthur Doughty (party hereto) who being male shall attain the age of
twenty-one years or being female shall attain that age or marry and if more than one in equal shares
5. Provided always that notwithstanding anything hereinbefore contained if the settlor shall predecease the said
Donald Arthur Doughty the trustees shall (subject to the provisions of this deed) during the residue of the life of the said
Donald Arthur Doughty apply the income of the said trust fund for or towards the maintenance education or benefit of all
or any of the following persons namely the grandchildren and the children or child of any of the grandchildren who shall
predecease the said Donald Arthur Doughty leaving issue them him or her surviving in such manner and proportions in all
respects as the trustees in their absolute and uncontrolled discretion shall decide
6. (i) Provided always that notwithstanding anything hereinbefore contained if at and by the date of the death of the
settlor there shall have been no grandchildren other than or apart from the said Patricia Evelyn Doughty and Peter Arthur
Doughty the trustees shall from and after the settlors death stand possessed of two equal third parts of the trust fund upon
trust for such of them the said Patricia Evelyn Doughty and Peter Arthur Doughty as shall attain the age of twenty-five
years and if both of them shall attain that age in equal shares
(ii) If the trusts of sub-clause (i) of this clause shall come into operation the provisions of cl. 4 and cl. 5 hereof shall
remain and be operative only so far as regards the remainder of the trust fund provided always that notwithstanding
anything hereinbefore contained neither of them the said Patricia Evelyn Doughty and Peter Arthur Doughty nor any child
of the said Patricia Evelyn Doughty and Peter Arthur Doughty shall take any benefit under the provisions of cl. 4 and cl. 5
hereof if at the date of the death of the said Donald Arthur Doughty there shall be grandchildren or a grandchild in addition
to or apart from the said Patricia Evelyn Doughty and Peter Arthur Doughty or the survivor of them.
7. In the event of all the preceding trusts hereof failing then subject and without prejudice to such trusts and any and
every exercise by the trustees of the discretions vested in them the trustees shall stand possessed of the trust fund upon trust
for the said Donald Arthur Doughty (party hereto) absolutely.
8. Provided further that nothing in this deed contained shall be deemed to prejudice or otherwise overrride the
operation of s. 31 of the Trustee Act, 1925 from and after the death of the said Donald Arthur Doughty.

Before considering the application of the Finance Act, 1894, to the settlement it will be convenient to refer to a question of
construction which arose on the proviso to cl 6(ii) of the settlement which applied in the circumstances to one third of the trust
fund. The plaintiffs argued that the proviso only applied as from Donalds death and that the intermediate income of the one-third
from the settlors death to Donalds death consequently had to be applied in accordance with cl 5 for the benefit of grandchildren
and children of deceased grandchildren in such proportions as the trustees should decide. As, however, Patricia and Peter were
then the only beneficiaries within cl 5 immediately after the settlors death it was argued that they then, between them, became
entitled to that income. The 231 Crown pointed out that the plaintiffs construction failed to give any effect to the reference to cl
5 in the proviso which excluded the operation of that clause during the life of Donald if at his death there should be a grandchild
other than Patricia and Peter. The Crown accordingly submitted that the proviso precluded the trustees from paying any income
under cl 5 to Patricia or Peter; that Patricia and Peter had contingent interests in one-third of the trust fund, contingently on (a)
attaining twenty-one (b) surviving Donald and (c) there not being at Donalds death any other grandchild; that as Patricia and
Peter had attained twenty-one before the settlors death the intermediate income for the period from the settlors death to
Donalds death became payable to them under s 31 of the Trustee Act, 1925, notwithstanding that, apart from that section any
implied accumulation of income during Donalds life would offend the rule against perpetuities. With regard to the two-thirds of
the trust fund given under cl 6(i):

for such of them the said Patricia Evelyn Doughty and Peter Arthur Doughty as shall attain the age of twenty-five
years and if both of them shall attain that age in equal shares,

the intermediate income from the settlors death until that age should be reached was likewise payable to them under s 31 of the
Trustee Act, 1925. Both parties thus came to the same conclusion, although by different routes, namely, that on the settlors death
the income of the one-third of the trust fund, like the income of the two-thirds of the trust fund became payable to Patricia and
Peter, although subject in the case of the one-third to divesting provisions, which were agreed to be immaterial for present
purposes. It thus became common ground to the parties in this case that for the purpose of applying the Finance Act, 1894, on the
settlors death to this settlement no distinction was to be drawn between the two-thirds and the one-third and that the whole of the
trust fund should be treated as governed by the Finance Act, 1894, in the same way as that Act applied to the two-thirds.
Now I turn to the application of the Finance Act, 1894, to the two-thirds. The Crown contends first that the property passed
on the settlors death within s 1 of the Finance Act, 1894, and alternatively that it passed under s 2(1)(b), or in the further
alternative under s 2(1)(d). Counsel for the plaintiffs disputes that these provisions apply at all to the case and further says that,
even if they do, this case is excepted from their operation by s 2(3). I will now read s 1, s 2(1)(b) and s 2(3). There is no need for
me to read s 2(1)(d).

1. In the case of every person dying after the commencement of this Part of this Act, there shall, save as hereinafter
expressly provided, be levied and paid, upon the principal value ascertained as hereinafter provided of all property, real or
personal, settled or not settled, which passes on the death of such person a duty called Estate duty at the graduated rates
hereinafter mentioned and the existing duties mentioned in the First Schedule to this Act shall not be levied in respect of
property chargeable with such Estate duty.
2(1). Property passing on the death of the deceased shall be deemed to include the property following, that is to say:
(b) Property in which the deceased or any other person had an interest, ceasing on the death of the deceased, to the
extent to which a benefit accrues or arises by the cesser of such interest
(3) Property passing on the death of the deceased shall not be deemed to include property held by the deceased as
trustee for another person, under a disposition not made by the deceased, or under a disposition made by the deceased more
than [five years] before his death where possession and enjoyment of the property was bona fide assumed by the
beneficiary immediately upon the creation of the trust and thenceforward retained to the entire exclusion of the deceased or
of any benefit to him by contract or otherwise.
232

It was common ground in view of Public Trustee v Inland Revenue Comrs that exceptionally a passing might occur under s 1
which was not within s 2. In particular references were made to the observation of Lord Simonds ([1960] 1 All ER 1 at p 7;
[1960] AC 398 at p 413) that

Section 2 may well not be a definition section in the sense that any property escapes the charge which is not included
in it, though I have not for myself been able to think of any property which would not be so included,

and to the observation of Lord Radcliffe ([1960] 1 All ER 1 at p 10; [1960] AC 398 at p 418):

Section 2 seems to be a natural sequel of s. 1. Its purpose is to explain and refine upon the meaning of the words
property which passes on death which s. 1 had declared to be subject to the charge of duty.

I will deal first with the application to this case of passing on death under s 1 and then with the application to this case of s 2(1)
(b). Immediately before the settlors death the trust fund was held on discretionary trusts to apply the income for the benefit of
grandchildren and to accumulate the surplus as accretion to capital, which was the subject of contingent gifts; and immediately
after the settlors death it was held on trust to pay the income to Patricia and Peter.
In my view Scott v Inland Revenue Comrs, establishes that a change of beneficial interest such as took place in this case
from the possibility of benefit under a discretionary trust to an entitlement to payment of a definite portion of income constitutes
a passing under the Finance Act, 1894, and Re Hodsons Settlement, Brookes v A-G, establishes that a change from a contingent
interest in the income or capital of accumulations to an entitlement to payment of a defined portion of the income of
accumulations also constitutes such a passing. I will refer to the passage from the speech of Lord Russell of Killowen in Scott v
Inland Revenue Comrs ([1936] 3 All ER 752 at pp 756, 757; [1937] AC 174 at pp 182, 183). I need not refer to the facts, which
are sufficiently indicated in the quotation.

On the death of the sixth earl the seventh earls estate in tail male became, instead of an estate in remainder, an estate
in tail male in possession. He became entitled to receive the whole income of the estate, which immediately before the
death of the sixth earl was primarily applicable for the benefit of the objects of the discretionary trust. That, in my opinion,
was a change of hands in the beneficial title or possession of the property as a whole, occurring on the death of the sixth
earl, which constituted a passing of the property on that death within the meaning of s. 1 of the Finance Act, 1894. In the
Court of Appeal ROMER, L.J., pointed out in detail the contrast between the beneficial enjoyment of the property before
and after the death of the sixth earl, and I agree with him when he says that on that death the beneficial enjoyment passed to
the seventh earl from the class of objects of the discretionary trust, and none the less because the seventh earl was himself a
member of the class. As MAUGHAM, L.J., expresses it, the seventh earl had himself during the sixth earls life no interest
in any defined slice or portion of the property (his only interest being the possibility of benefit under the discretionary
trust); but on the death of the sixth earl the discretionary trust ended and the seventh earl became entitled in possession to
the property as a whole.

It seems to me that Lord Maughams observations in that case apply equally to this case.
I quote the following passage from the Hodson case ([1939] 1 All ER 196 at p 202; [1939] Ch 343 at p 357). Again, I need
not set out the facts. The judgment in the case was delivered by Clauson LJ, and 233 it was the judgment of the court consisting
of Sir Wilfrid Greene MR, Scott and Clauson LJJ:

Up to the death, the income was subject to a trust for accumulation for the ultimate benefit of the lady and her
children. Upon the death, this trust ceased, and the lady became thenceforward for the rest of her life the sole beneficiary
in the income of the fund. It is true that before the death the lady had an interest of a contingent character in the income of
the fund, since she had a right to have it accumulated so as to provide a fund the fruit of which would, if she survived Mr.
Hodson, come to her; but this circumstance would not prevent the change in the beneficial possession of the property as a
whole which took place on Mr. Hodsons death from operating as a passing of the property on his death.

It is to my mind clear from these authorities that the change which took place in this case on the settlors death was not merely a
change of source of title but also a change of possession or enjoyment in accordance with the requirement indicated in Lord
Eversheds judgment in Re Parkes Settlement Trusts, Midland Bank Executor & Trustee Co Ltd v Inland Revenue Comrs ([1956]
1 All ER 833 at p 838).
Property passes in accordance with s 2(1)(b) if it is

property in which the deceased or any other person had an interest ceasing on the death of the deceased, to the extent
to which a benefit accrues or arises by the cesser of such interest.

The property in this case is the trust fund and the objects of the discretionary trust had an interest in that property (see A-G v
Farrell). Counsel for the plaintiffs, however, sumitted that that interest did not cease on the settlors death, nor did any benefit
arise or accrue by the cesser of such interest. This submission rested on the contention that what happened on the settlors death
was that Patricias and Peters interests under the discretionary trust were enlarged into some greater interests and therefore did
not cease at all. However, what occurred on the settlors death was the replacement of one set of trusts by another set of trusts
and the determination of one set of interests and the commencement of another set of interests. The discretionary trust and trust
for accumulation ceased and the benefit which arose was a benefit to Patricia and Peter equally and it extended to the income of
the whole of the trust fund.
I, therefore, conclude that the trust fund passed on the settlors death under s 1 and s 2(1)(b) or, if not under s 2(1)(b), then
under s 1 alone. It is consequently unnecessary to consider the crowns alternative submission that if these sections did not apply
then s 2(1)(d) applied.
It is, however, submitted that despite this conclusion the plaintiffs, by reason of s 2(3) are not liable to estate duty. The
Crown conceded that if s 2(3) applied to the facts of this case then it would apply irrespective of whether the passing was under s
1 or s 2. Counsel for the plaintiffs submits that s 2(3) applies wherever the person on whose death property passes is a trustee
(subject only to the exception in case of a disposition made by the trustee within five years before his death). Thus, if A gives
property to B on trust for C for life and then to D estate duty would be payable on Cs death, but if A made C the trustee and gave
the property on exactly the same trusts, namely, to C on trust for C for life and then to D, no estate duty would be payable on Cs
death. Similarly, if A settled property outside the five years period by a disposition under which A held on trust for B during the
life of A and then for C, no estate duty would be payable on the death of A. This interpretation means that estate duty is avoided
if at his death the deceased happens to be a trustee of the settlement although the holding of that office at that time has no
relevance whatsoever to the passing of the beneficial interest upon which estate duty is made payable. The exclusion from
liability on such a ground is as whimsical as it would be on the ground that 234 the deceased held any other office or was married
or a bachelor or was red-headed or bald. It is not, therefore, surprising that so far as I am aware this is the first occasion on which
this submission has been made, although it would have provided a complete answer to the Crowns claim in, for example, Re
Hodsons Settlement. It was suggested that it might not have been relied on before the Arnholz case, Public Trustee v Inland
Revenue Comrs, because it was not realised before that case that s 2(3) would apply to a passing under s 1; but it was not relied
on there, although the taxpayers very contention was that s 2 exclusions extended to s 1 passing. In that case Lord Radcliffe
([1960] 1 All ER 1 at p 11; [1960] AC 398 at p 419) observed:

No one has ever questioned that the rules of inclusion or exemption contained in sub-s. (2) and sub-s. (3) apply to all
property within, or that would otherwise be within, the charge imposed by the terms of s. 1.

Nevertheless if the true construction of sub-s (3) leads to the conclusion for which counsel for the plaintiffs contends that
conclusion must be accepted.
My first impression on reading sub-s (3) is that it is directed to the passing from a trustee of property held by him and which
necessarily passes from him on his death because he can no longer continue to hold it as trustee. Counsel for the plaintiffs,
however, argued that if the words are considered so far as relevant, phrase by phrase, they apply to this case. Thus he says (1)
that the property was held by the settlor until his death as trustee for another person and (2) that the disposition was made by him
more than five years before his death and possession and enjoyment was assumed and retained by the beneficiary to the exclusion
of the settlor in accordance with the requirements of the section. This is a simple and attractive submission but, of course, it is
achieved without considering the phrases in relation to each other and the subsection as a whole or in relation to their context in
the Act.
Counsel for the Crown, on the other hand, sought to govern the construction of the subsection by its context in the Act.
Whilst conceding that whether a passing was under s 1 or s 2 was immaterial to the operation of sub-s (3), he contended that sub-
s (3) was limited in its operation to qualifying sub-s 2(c). He indicated that sub-s 2(c) made property the subject of a disposition
inter vivos by the deceased liable to estate duty at his death unless it was made five years before his death and possession and
enjoyment was immediately assumed by the donee and thenceforward retained to the entire exclusion of the deceased or of any
benefit to him. He suggested that sub-s (3) was directed to ensuring that the appointment of the settlor as trustee should not be
treated as transgressing this requirement of entire exclusion. To the question, why, if this explanation is correct, sub-s (3) was not
inserted as part of sub-s 2(c), he answers:because sub-s (2)(c) was drafted in a referential form. To the question why should
there be included in sub-s (3) a reference to a disposition not made by the deceased when sub-s (2)(c) is concerned with a
disposition made by the deceased, counsel answers:because otherwise it might be argued that property held by the deceased as
trustee for another person under a disposition not made by the deceased was property passing on the deceaseds death. I find both
these answers less convincing than the objections to which they are made. In my view this argument is not compatible with
either the arrangement of the subsections or the contents of sub-s (3).
Counsel for the Crown submitted alternatively that in this case the property passing on the settlors death was not excepted
by sub-s (3) because the property not only passed because it was held by the settlor as trustee but also because of a change of
beneficial interest which occurred under the settlement on the settlors death. Reference was made to Escoigne Properties, Ltd v
Inland Revenue Comrs, but as that case turned on the very different wording of 235 s 42 of the Finance Act, 1930, relating to
stamp duty, I do not find it of assistance in this case. Nor is this submission self-evident. There is, to my mind, no escaping the
requirement that it is on the meaning of the words used in sub-s (3) that this case must be decided. The opening words of the
subsection are Property passing on the death of the deceased shall not be deemed to include . It is not, therefore, in its terms
directed to excluding property which has already been included. It is a clarifying rather than an excepting provision and it is no
objection to its interpretation that it may refer to property which in any event might be considered not to be property passing on
the death. It is not disputed that, on the footing of the decision that there is here a passing under s 1 or s 2(1)(b), the relevant
property which so passed on the settlors death was the trust fund and that this trust fund the settlor held as trustee for another
person.
The words Property passing on the death of the deceased shall not be deemed to include property held by the deceased as
trustee must be construed as a whole and not divided up into disconnected fragments. The phrase property held by the
deceased as trustee does not in its context mean property held by him at any time, even although not so held immediately before
his death. The phrase read separately is elliptical. Read in conjunction with the rest of the sentence it is seen to refer to property
which was held by the deceased as trustee at the time of his death, which might in ordinary language be described as passing
from him on his death. To the question When held by the deceased as trustee?, the answer is On the death of the deceased,
which is the only reference to time in the opening words of sub-s (3); which leads up to the reference to holding by the deceased
as trustee; and to which the holding impliedly refers. To the question, From whom does the property passing pass?, the
answer is, From the person who immediately before his death held as trustee; because that is the only reference to anyone
holding the property. If this is correct, then sub-s (3) applies only to property held by the deceased as trustee and moving from
him as such on his death. But, so far on this analysis, because the trust fund in this case was so held and passed, it can still be
said that this case falls within the requirement of sub-s (3). In accordance with the analysis the provision in sub-s (3), however, is
this:Property passing shall not be deemed to include property held by the deceased as trustee and moving from him as such
on his death. Property so described is not excluded from property previously included or indeed excluded at all. Nor is it
deemed not to be included. It is merely not deemed to be included. It contrasts markedly with the opening words of sub-s (2),
viz, Property passing on the death of the deceased shall be included only, if.
Property passing shall be deemed to include under s 2(1) property answering a description in that subsection and shall
not be deemed to include property answering a description in s 2(3): and the same property may answer descriptions in both
subsections. Thus the property passing on the settlors death in this case both under s 2(1)(b) shall be deemed to include and
under s 2(3) shall not be deemed to include the trust fund. Subsection (1), however, states categorically that property passing
shall be deemed to include property answering to a certain description, whereas sub-s (3) merely states that property passing
shall not be deemed to include property answering other descriptions. In each case the description of the property is the reason
for inclusion or non-inclusion. Thus in this case, property passing shall not be deemed to include the trust fund described in
accordance with sub-s (3) and so not included by reason of that description. But property passing shall be deemed to include the
trust fund described in accordance with sub-s (1)(b) and so included by reason of that description. Non-inclusion under one
description under sub-s (3) does not, in my view, prevent inclusion under another description whether under s 2(1) or otherwise.
Section 1 and s 2, so far as this case has been concerned with them, thus make a coherent and rational scheme and one which
accords with first impressions. It appears to me that the construction to the 236 contrary depended on treating sub-s (3)
independently of sub-s (1), and phrases in sub-s (3), including in particular the phrase property held by the deceased as trustee,
independently of their context in the sub-section. Its strength lay in its simplicity; its weakness was that its effect flouted
common sense and the general scheme of the Act and, in my view, the principles of construction.
It therefore appears to me that s 2(3) does not prevent the property passing in this case under s 1 or s 2(1)(b), and
accordingly being liable to estate duty.

Declaration accordingly.

Solicitors: Swepstone, Walsh & Son (for the plaintiffs); Solicitor of Inland Revenue.

Jacqueline Metcalfe Barrister.


[1963] 2 All ER 237

Mills v Mills
ADMINISTRATION OF JUSTICE; Legal Aid and Advice

COURT OF APPEAL
ORMEROD, DONOVAN AND DIPLOCK LJJ
7 MARCH 1963

Legal Aid Costs Assisted persons liability to pay costs Certificate limited to defending claims for ancillary relief in divorce
petition Whether assisted person liable to pay costs of petition Legal Aid and Advice Act, 1949 (12, 13 & 14 Geo 6 c 51), s
2(2)(e).

Where a person is issued with a legal aid certificate limited to part of the issues in an action or petition, he is not protected by s
2(2)(e)a of the Legal Aid and Advice Act, 1949, in respect of costs incurred in that part of the action or petition to which his legal
aid certificate does not extend (see p 240, letter i, and p 241, letters b and d, post).
________________________________________
a Section 2(2) is set out at p 239, letter b, post

Appeal dismissed.

Notes
As to costs awarded against an assisted person, see 30 Halsburys Laws (3rd Edn) 502, 503, para 933.
For the Legal Aid and Advice Act, 1949, s 2, see 18 Halsburys Statutes (2nd Edn) 535.

Appeal
This was an appeal by the husband from an order of His Honour Judge Herbert, sitting as a Special Commissioner for Divorce on
13 November 1961, whereby he ordered the husband, who had been granted a legal aid certificate to defend his wifes petition for
dissolution of her marriage limited to her claim for alimony and maintenance including application to determine costs, to pay the
costs of the petition. The facts are set out in the judgment of Diplock LJ.

T A Coningsby for the husband.


Neil Taylor for the wife.
7 March 1963. The following judgments were delivered.

DIPLOCK LJ delivered the first judgment at the request of Ormerod LJ. This appeal raises a short and difficult point of
construction under the Legal Aid and Advice Act, 1949. The circumstances under which it arises are these. The appellant
husband was respondent to a petition for dissolution of marriage by his wife in which she asked for alimony pendente lite,
maintenance and costs, as well as for dissolution of the marriage. The husband was granted a legal aid certificate in a limited
form. That certificate certified that he was entitled to legal aid to be heard as respondent in the High Court of Justice, Probate,
Divorce and Admiralty Division (Divorce) proceedings entitled Rhoda Jean Mills, petitioner, and Thomas Leonard Lanty Mills,
respondent, in respect of the claim in the prayer of petition for alimony and maintenance, and to include application to determine
liability for costs. The certificate was thus limited to defending on those matters, and not to defending the petition for the decree
of dissolution. The matter came on in November, 1961, before Judge Herbert, sitting as commissioner in the Divorce Division
here, and he granted a decree 237 nisi of dissolution, the question of alimony and maintenance being referred to some other
judge. He made an order for costs against the husband, whose certificate did not include defending the application for dissolution
of the marriage. The learned commissioner felt that he was entitled to make an order for costs in this form on the true
construction of s 2(2)(e) of the Legal Aid and Advice Act, 1949; but, as the husband was not represented at the hearing of the
petition, the order for costs was made without argument.
It has apparently long been the practice in legal aid cases in the Divorce Division to grant legal aid certificates in the limited
form which was granted in this case, a form which is authorised by reg 6 of the Legal Aid (General) Regulations, 1962; and it has
always been the general practice that, when a certificate is granted in that form, para (e) of s 2(2) of the Legal Aid and Advice
Act, 1949, is treated as covering the costs incurred on the undefended petition for dissolution of marriage. There is, however, or
had previously been, no express decision on the matter, and several applications were made to Mr Commissioner Herbert to seek
to persuade him to vary his order, in particular as there had in the meantime been a decision in the contrary sense by Judge
Rawlins, sitting as commissioner in the Divorce Division. Mr Commissioner Herbert came to the conclusion on the final
application, in October, 1962, that he had no right to vary his order because he was functus officio, as plainly he was, and it has
not been sought to be argued before us, that, in that respect at any rate, he was not right. He did, however, give leave to appeal
from his earlier order as to costs made on 13 November 1961, so that the matter could be considered by this court as this is a
matter on which there are now two conflicting decisions.
The point is quite a short one, and turns on the true construction of the word proceedings in s 2(2) of the Legal Aid and
Advice Act, 1949. I think that I must approach that section by looking first at s 1 of the Act, which deals with the Scope and
general conditions of legal aid in connexion with proceedings, and it starts in sub-s (1):

This and the three next following sections provide for, and (save as hereinafter mentioned) relate only to, legal aid in
connexion with proceedings before courts and tribunals in England and Wales

Subsection (2) provides:

Unless and until regulations otherwise provide, the proceedings in connexion with which legal aid may be given are
any proceedings of a description mentioned in Part 1 of Sch. 1 to this Act, except proceedings mentioned in Part 2 of that
Schedule.

The only assistance which one gets here as to the meaning of proceedings is to be found in para 2 of Part 1 and in para 6 of Part
2 of that schedule. Paragraph 1 sets out Proceedings in any of the following courtsand then some twelve or so different
courts are specified. Paragraph 2 says Proceedings before any person to whom a case is referred in whole or in part by any of
the said courts. The only relevance of thatand it is not a very strong guideis that it does, at any rate, suggest that
proceedings may be something smaller than the whole action, cause or matter, for, if proceedings meant, and meant only, the
whole action, cause or matter, para 2 would be otiose. Then we come to para 6 of Part 2 of the Schedule. Part 2, containing
Excepted Proceedings which I need not enumerate, finishes up with para 6: Proceedings incidental to any proceedings
mentioned in this Part of this Schedule. That again looks as if proceedings were an expression narrower than the whole
action, cause or matter.
I return to s 1(3), which reads as follows:

Subject to the provisions of this section, the proceedings in connexion with which legal aid may be given may be
varied by regulations, and the regulations may describe the proceedings to be included or excluded by 238 reference to the
court to tribunal, to the issues involved, to the capacity in which the person requiring legal aid is concerned, or otherwise.

There again, one does not get a great deal of assistance, but the provision about describing the proceedings by reference to the
issues involved does, at any rate, suggest that proceedings may have a more limited meaning than the whole action, cause or
matter. I go to s 2(2). Section 2 deals with Financial conditions of legal aid. Subsection (2) reads as follows:

Where a person receives legal aid in connexion with any proceedings(a) the expenses incurred in connexion with
the proceedings, so far as they would ordinarily be paid in the first instance by or on behalf of the solicitor acting for him,
shall be so paid except in the case of those paid direct from the legal aid fund as provided by this Part of this Act; (b) his
solicitor and counsel shall not take any payment in respect of the legal aid except such payment as is directed by this Part
of this Act to be made out of the legal aid fund; (c) he may be required to make a contribution to the legal aid fund in
respect of the sums payable thereout on his account; (d) any sums recovered by virtue of an order or agreement for costs
made in his favour with respect to the proceedings shall be paid to the legal aid fund; (e) [which is the paragraph relied on
by the husband] his liability by virtue of an order for costs made against him with respect to the proceedings shall not
exceed the amount (if any) which is a reasonable one for him to pay having regard to all the circumstances, including the
means of all the parties and their conduct in connexion with the dispute.

Subsection (3) goes on to provide for determination of the costs payable.


There is one other section of the Act to which I should refer before turning back to that subsection, and that is s 12(2).
Section 12 deals with the making of regulations, and provides as follows in sub-s (2):

Without prejudice to the foregoing subsection or any other provision of this Act authorising the making of regulations,
regulations may(a) make provision as to the proceedings which are or are not to be treated as distinct proceedings for the
purposes of legal aid, and as to the apportionment of sums recoverable or recovered by virtue of any order for costs made
generally with respect to proceedings treated as distinct.

It seems to me that that subsection clearly contemplates that the expression proceedings, at any rate as used in that paragraph, is
something narrower than the whole action, cause or matter. Finally, I think that I should turn to two of the regulations contained
in the Legal Aid (General) Regulations, 1962, not for the purpose of construing the Act, for regulations made under the Act,
cannot, in my view, assist in construing the Act, but just to see under what power the certificate granted to the husband was made.
That was given under reg 6(1), which provides:

A certificate may be issued in respect of (b) the whole or part of(i) proceedings in a court of first instance

There is one other regulation to which I would also refer, because the learned judge referred to it in his judgment. That is reg 9,
dealing with Amendment of Certificates. That provides, so far as relevant:

(1) The appropriate area committee may amend a certificate (b) when, in their opinion, it has become desirable
either for the certificate to extend to (iv) other proceedings being part of the same action, cause or matter to which
the certificate relates.

It is plain there that the draftsman of the regulations considered that a certificate could be issued in respect of part of an action,
cause or matter, as, indeed, this certificate was issued in the present case.
Counsels argument on behalf of the husband was this, that, in s 2(2) the 239 expression proceedings means the whole of
the action, cause or matter in respect of which a certificate was issued, even if the certificate is restricted and limited to part of the
action, cause or matter. He then says that the proceedings in the present case were the petition filed by the wife, which asked
for various kinds of relief, and, those being the proceedings, it matters not that the husbands legal aid certificate was limited to
contesting only part of the relief sought in those proceedings. So, he says that, in sub-s (2)(e) of s 2 of the Legal Aid and Advice
Act, 1949, where it provides:

his liability by virtue of an order for costs made against him with respect to the proceedings shall not exceed the
amount (if any) which is a reasonable one for him to pay,

the proceedings there referred to are the petition filed by the wife in which she asked for various kinds of relief, including
dissolution of marriage. The order made by the learned judge was an order for costs made against the husband with respect to
those proceedings, and, accordingly, was an order for costs to which sub-s (2)(e) applied.
I have a good deal of sympathy with that argument where it applies to petitions for dissolution of marriage in which a
petitioner, even in an undefended petition, has to prove his or her case and costs are incurred, and, if I could find some way of
treating proceedings in para (e) in a different sense from the word proceedings in other parts of the same subsection, I would
gladly do so; but, as a matter of construction, that seems to me to be quite impossible. Consequently, the result of the
construction contended for by counsel for the husband would be this: not merely where an assisted person has a limited
certificate limited to disputing part, and part only, of the claim, would he be entitled not to have an order for costs made against
him except under the conditions laid down in para (e), but he would also be entitled under para (b) and para (d) to have paid out
of the legal aid fund the costs of the very matter in respect of which he was not given a certificate. The result of that construction
would mean, as far as I can see, that there would be no power on the part of a legal aid committee to give a certificate limited to
part of the proceedings in a court, for such attempted limitation would be a brutum fulmen. It would follow, too, I think, that reg
6(1)(b), so far as it purports to confer such a power on a legal aid committee, would be ultra vires, because, once it is granted that
proceedings in s 2(2) mean the whole action, cause or matter, then it must follow that a person who receives a limited
certificate is a person who receives legal aid in connexion with the proceedings, and, accordingly, para (a) relating to the
expense incurred in connexion with the proceedings, para (b) forbidding his solicitor and counsel to take any payment except
payment out of the fund, and para (d) providing that any sums recovered by virtue of an order or agreement for costs made in his
favour shall be paid to the legal aid fund, would apply to the whole of the action, cause or matter, and not merely to those
parts of it in respect of which the legal aid certificate was granted.
It seems to me that this is a section where the word proceedings in the Act is ambiguous, although the indications to which
I have referred do suggest that, in other sections at any rate, it bears a meaning narrower than the whole of an action, cause or
matter. The considerations directly arising under s 2(2), to which I have just referred, in addition to the earlier indications in the
Act, which I have dealt with at the beginning of this judgment, satisfy me that Mr Commissioner Herbert was right in his
construction of the Act, and that, where a person is issued with a legal aid certificate limited to part of the issues in an action or
petition, he is not protected by s 2(2)(e) of the Act in respect of costs incurred in that part of the action or petition to which his
legal aid certificate does not extend.
I would, therefore, dismiss this appeal. I appreciate that, in doing so, we are deciding that the practice which has been
adopted for a long time now by the 240 Law Society is not a practice which is authorised by the Act. It seems to me that, having
regard to the peculiar nature of the Divorce Division and the jurisdiction exercised there, where a petitioner, even when the matter
is not defended, has to prove his or her case, and will incur costs which will be recoverable against a respondent who does not
defend it, it may be desirable that those concerned with this matter should consider whether or not appropriate provision to deal
with this type of case might not be made by regulation or otherwise; but, the law standing as it does, I would dismiss this appeal.

ORMEROD LJ. I agree, and there is nothing I wish to add to the reasons which have been given by Diplock LJ. It may be that,
in view of the considerations that he has expressed, regulations may be considered to deal with the particular circumstances
which arise in these cases. However, that is not for us. We are concerned to construe the Act and interpret the law as it is.

DONOVAN LJ. I feel myself that the good sense of this matter is that public funds ought not to be at risk for the costs of any
proceedings for which no legal aid has been granted. No legal aid was granted to the husband in respect of the proceedings
which consisted of the wifes prayer for divorce. It was limited to such proceedings as dealt with her prayer for alimony and
maintenance. Then does the wording of s 2(2) of the Legal Aid and Advice Act, 1949, require, nevertheless, that the husband
shall be treated as legally aided in respect of both proceedings? This result will not follow unless the word proceedings in s
2(2) must refer, and refer only, to the totality of the proceedings. That view is certainly susceptible of argument, but I do not
think that it is the true construction. As Diplock LJ has said, the Act in several places indicates that proceedings may be
proceedings incidental to the whole action. I need not repeat the examples which he gave. The husbands construction,
moreover, would lead to some odd results. Thus, under s 2(2)(d), a legally aided person would have to pay the legal aid fund
costs awarded to him on an issue on which he had succeeded, albeit that, on that particular issue, he was not legally aided.
It is urged that it is anomalous that the husband, who is legally aided on certain issues because he is impecunious, should,
nevertheless, have a full order for costs made against him on another issue. This would seem to be an argument against the issue
of restricted certificates at all; but the point cannot really affect the construction of s 2(2). It is really a complaint against the
ordinary law applicable to those who are not legally aided.
For those reasons, I agree that this appeal fails and should be dismissed.

Appeal dismissed. Leave to appeal to House of Lords refused.

Solicitors: Asher, Fishman & Co (for the husband); Ambrose Appelbe (for the wife).
Henry Summerfield Esq Barrister.
241
[1963] 2 All ER 242

Royal Mutual Benefit Building Society v Sharman and others


BANKING AND FINANCE

CHANCERY DIVISION
PLOWMAN J
15, 19, 20, 22, 28 MARCH 1963

Building Society Meeting Notice convening meeting Rules entitling all members to notice of all meetings but only holders of
completed shares to vote Enactment requiring notice of special meeting to be sent to members qualified to vote Notice of
special meeting sent to members holding completed shares only Omission of notice to holders of uncompleted shareholders not
accidental Whether meeting validly convened Building Societies Act, 1960 (8 & 9 Eliz 2 c 64), s 35(1)-(3).

By the rules of a building society all members, including holders of uncompleted shares (ie shares on which less than 10 had
been paid), were entitled to notice of all meetings of the society, and holders of completed shares (but not, it seemed, holders of
uncompleted shares) were qualified to vote thereat. Section 35 of the Building Societies Act, 1960, by sub-s (1), required notice
of any meeting of a building society at which a special resolution was to be proposed to be sent to every member qualified to
vote, and, by sub-s (2), provided that subject to this requirement notice of any meeting should be sent to every member, except
that, subject to the rules of the society, notice need not be sent to a member not holding shares to a value of 25. A notice
convening a special meeting of the society for the purpose of passing certain resolutions as special resolutions was sent only to
the holders of completed shares.

Held The meeting was not validly convened, and accordingly the resolutions passed at it were invalid, because
(i) the societys rules entitled all members, whether they held completed or uncompleted shares, to receive notice of
meetings and the requirement of the rules was not affected by sub-s (2) of s 35 of the Building Societies Act, 1960, which applied
subject to the rules, and was not overridden by sub-s (1), which was an enlarging and not a restricting enactment; and
(ii) omission to give notice in the present instance was not accidental within s 35(3) of the Building Societies Act, 1960.

Notes
Section 35 of the Building Societies Act, 1960, was replaced, on 1 October 1962, by s 66 of the Building Societies Act, 1962.
As to the requirement that rules must provide the manner of calling meetings, see 3 Halsburys Laws (3rd Edn) 558, para
1122; and as to persons entitled to notice, see Cum Supp to Halsburys Laws (3rd Edn), Vol 3, para 1175 B.
For the Building Societies Act, 1960, s 35, see 40 Halsburys Statutes (2nd Edn) 80.

Adjourned Summons
This was an application by originating summons dated 18 January 1963, by the Royal Mutual Benefit Building Society for the
determination of the following question, among other questions:(4) whether on the true construction of the relevant rules of the
society, and in the events which had happened, the resolutions numbered 1, 2 and 3 set out in the notice dated 26 May 1962,
convening a special general meeting of the society held on 19 June 1962, or any, and if so which, of such resolutions were
properly passed by the society in general meeting or were invalid. The defendants were (1) Albert William Sharman, who
represented persons who were repaid the whole or part of the amounts of their subscriptions on their shares in the society less a
thirty per cent deduction on account of the interim rate of loss declared by the directors on 23 May 1945; (2) Mary Rachel
Sorrell, who represented persons who agreed to accept payment of the whole or part of their subscriptions on their shares less a
deduction of thirty per cent on account of such interim rate of loss in full 242 settlement of their shareholdings; (3) Charles
Herbert John Alder who represented persons who agreed to payment of the whole or part of their subscriptions on their shares
less a deduction of 33 1/3 per cent in full settlement of their shareholdings; (4) Winefride Mary Dutton, who represented members
of the society who subscribed for shares prior to 31 December 1944; (5) Lillian Muggridge who represented members of the
society who subscribed for shares after 1 January 1947, and (6) Stanley Newing and (7) Alice Newing, who represented members
of the society who held one or more uncompleted share or shares.
The case is reported solely on the construction of s 35 of the Building Societies Act, 1960.

K B Suenson-Taylor for the plaintiffs.


B Clauson for the first defendant.
J W Bygott-Webb for the second defendant.
David Stanford for the third defendant.
G Hesketh for the fourth defendant.
Martin Roth for the fifth defendant.
C J Slade for the sixth and seventh defendants.

28 March 1963. The following judgment was delivered.

PLOWMAN J having declined to answer questions 1 and 2 of the originating summons and having answered question 3, read
the terms of question 4 (which are printed at p 242, letter h, ante) and continued. The fact underlying question 4 is that notices of
this meeting were not sent to the members of the society holding uncompleted shares. The meeting in question was called to pass
special resolutions, three in number; the first, to alter the rules of the society; the second, to vote a payment of 25,000 to Mr
Burley in recognition of his past services to the society; and the third dealing with the refunding to members who had suffered
deduction on withdrawing their shares of the sums so deducted.
The question which I have to decide really depends, I think, on the point whether the holders of uncompleted shares were
entitled to notice of this meeting. The meeting in question was one called to alter the rules, among other things. The rule which
deals with the alteration of rules is r 41, which says:

Any rule may be altered or rescinded, and any additional rule may be made by the vote of three-fourths of the
members present at a special meeting called for the purpose, of which meeting notice specifying the proposed alteration,
rescission, or addition shall be given to the members by letters sent through the post not less than seven days previous to
such meeting, and such notice shall be deemed sufficient.

I am concerned with the question, who are the members referred to in that rule to whom letters are to be sent? The relevant rules
to which I shall refer on this question are first of all r 3, part of which provides that funds were to be raised by shares, to be
called uncompleted and completed shares; secondly, r 4(1), which in its first sentence provides Every person subscribing for a
share shall be a member. Then r 5, dealing with uncompleted shares, says this:

The subscriptions upon uncompleted shares shall be four shillings per share, and shall be due every twenty-eight days,
and shall be paid in advance, and may be withdrawn in accordance with r. 9. When the subscriptions upon an uncompleted
share amount to 10, the share shall, on the written request of the member, be treated as completed, but until such request,
shall be classed as uncompleted as regards the right of withdrawal and the payment of interest.

Then r 13 opens with these words Advances shall be made to members holding completed or uncompleted shares on mortgage.
That is clearly a reference to holders of uncompleted shares as members. The rule which deals with notices of meetings is r 23,
which provides: Notices of all meetings of the 243 members shall be sent to all members at least seven days before such
meetings. Voting is dealt with in r 28, which is in these terms:

Every member of six months standing who holds not less than one completed share and who is not in arrears with any
payment to the society shall be qualified to nominate or vote for any person to be elected by the members or to vote on any
question submitted to any meeting, and every such member shall have one vote.

That rule suggests that uncompleted shareholders have no vote. It seems to me, so far as the question of notices of meetings is
concerned, that on the rules it is as plain as a pikestaff that all members, whether they hold completed shares or uncompleted
shares, are entitled to notices of meetings.
The question then arise whether that situation has been affected by s 35 of the Building Societies Act, 1960, which was the
relevant statutory provision in force at the material time. Subsection (1) of that section is as follows:

(1) Written notice of any meeting of a building society which specifies the intention to propose a resolution as a
special resolution at the meeting shall be sent to every member qualified to vote on a special resolution at the meeting.

I pause there to say that, as I have already said, the resolutions with which I am concerned were special resolutions. Subsections
(2) and (3) of s 35 are as follows:

(2) Subject to the provisions of sub-s. (1) of this section, written notice of any meeting of a building society shall be
sent to every member of the building society except that, subject to the rules of the building society, notice of a meeting
need not be sent to a member who does not hold shares in the building society to a value of twenty-five pounds or more at
the end of the financial year preceding that in which notices are sent or, if the building society was established in the
calendar year in which notices are being sent, at the time at which the notices are sent.
(3) The accidental omission to give notice of a meeting to, or the nonreceipt of notice of a meeting by, any person
entitled to receive notice shall not invalidate the proceedings at that meeting.

I will say a word about those subsections in reverse order. So far as sub-s (3) is concerned it seems to me impossible, on the
evidence which has been filed in this matter and which I am told is the whole of the relevant evidence available, to hold that the
omission to give notice of this meeting to the holders of uncompleted shares was accidental. With regard to sub-s (2), it is to be
noted that the exemption of necessity of giving notices to members holding less than 25 is expressly made subject to the rules
of the building society. In the present case, as I have already said, in my judgment the rules require notice of meetings to be
given to holders of uncompleted shares and therefore sub-s (2) is not applicable. Then sub-s (1), which I will read again
provides:

Written notice of any meeting of a building society which specifies the intention to propose a resolution as a special
resolution at the meeting shall be sent to every member qualified to vote on a special resolution at the meeting.

First of all it is to be noted that that subsection is not purporting to cut down in any way the provisions of the rules. It does not
say that, whatever the rules may say to the contrary, notice of meetings need not be given to persons who are not entitled to vote.
I think that the explanation of sub-s (1) is to be found in the argument which counsel for the sixth and seventh defendants
submitted, namely, that sub-s (1) is not a restricting, but an enlarging, subsection in that it applies, where a special resolution is
concerned, to persons who would otherwise be within sub-s (2) in the sense that they would be among the category of persons
who, under that subsection, need not be given notice of meetings. 244I do not regard sub-s (1) as overriding the rules of the
society in that regard and, therefore, even though the holders of uncompleted shares may not have been entitled to vote at this
meeting, nevertheless in my judgment they were entitled to notice of it. That being the case, the failure to give them notice, not
being accidental within the meaning of s 35(3) it seems to me to follow that that meeting was invalidly held and therefore that the
resolutions passed at that meeting were not properly passed.
[His Lordship then dealt with the remaining questions raised by the originating summons.]

Declaration accordingly.

Solicitors: McKenna & Co (for all parties).

Jacqueline Metcalfe Barrister.


[1963] 2 All ER 245

R v Kent Justices and another


Ex parte Crittenden
TOWN AND COUNTRY PLANNING

QUEENS BENCH DIVISION


LORD PARKER CJ, ASHWORTH AND WINN JJ
8 MARCH 1963

Town and Country Planning Caravan site Licence Existing site Condition as to number of caravans on site imposed by
local planning authority Whether local authority issuing site licence can impose condition beyond scope of planning condition
Caravan Sites and Control of Development Act, 1960 (8 & 9 Eliz 2 c 62), s 3(3), s 29(4).

Magistrates Jurisdiction Caravan site Appeal against condition imposed by local authority in site licence Condition same
as that imposed by local planning authority Whether magistrates had power to vary condition Caravan Sites and Control of
Development Act, 1960 (8 & 9 Eliz 2 c 62), s 7(1).

From 1939 onwards, and at the commencement of the Caravan Sites and Control of Development Act, 1960, a site was used for
caravans, no more than thirteen of which had been on the land at any one time. The owner of the site (the applicant) applied to
the local authority for a site licence. His application was referred by the local authority to the local planning authority (ie, to
itself in its capacity as local planning authority under delegated powers), which granted planning permission subject to a
condition that the number of caravans to be stationed on the site at any one time should not exceed thirteen. On appeal the
minister upheld the condition imposed on the planning permission. The local authority issued to the applicant a site licence under
the Act of 1960, subject to a similar condition as to the number of caravans. On appeal to the justices under s 7 a of the Act of
1960 against the condition of the site licence, the justices decided that they had no jurisdiction to vary the site licence condition in
view of the condition imposed on the planning permission. The applicant applied for an order of mandamus to the justices to
determine his appeal to them.
________________________________________
a Section 7(1) provides, so far as material, as follows:Any person aggrieved by any condition (other than the condition referred to in s 5(3)
of this Act) subject to which a site licence has been issued to him in respect of any land may appeal to a magistrates court ; and the
court if satisfied (having regard amongst other things to any standards which may have been specified by the minister under s 5(6)) that the
condition is unduly burdensome, may vary or cancel the condition.

Held (i) (Per Lord Parker CJ and Winn J) the planning permission (whether conditional or not) was the necessary prerequisite
of the issue of the site licence, and the site licence had, on the true construction of the Act of 1960, s 3(3) b, s 5 and s 29(4)c to be
confined within the limits of the planning permission; accordingly the justices had no jurisdiction under s 7 so to vary the
condition of the site licence as to extend the licence to any use beyond the scope of the planning permission (see p 248, letter b,
and p 253, letters f and g, post);
245
________________________________________
b The relevant terms of s 3(3) and s 29(4) are printed at p 247, letter i, post
c The relevant terms of s 3(3) and s 29(4) are printed at p 247, letter i, post

(ii) (per Ashworth J) although the applicant was not precluded from appealing to the justices under s 7 of the Act of 1960,
yet, as the condition of the planning permission had been upheld by the minister, the same local authority could not be held to
have imposed an unduly burdensome condition (within s 7(1)) by issuing the site licence subject to the same condition as had
been considered right for planning purposes (see p 250, letters a and h, post).
Motion dismissed.

Notes
As to issue of caravan site licences, see 37 Halsburys Laws (3rd Edn) 405, 406, para 517; as to appeals against conditions
imposed in a site licence, see ibid, p 410, para 524.
For the Caravan Sites and Control of Development Act, 1960, s 3, s 7 and s 29, see 40 Halsburys Statutes (2nd Edn) 1070,
1073, 1088.

Motion for mandamus


This was an application by way of a motion by Jack Crittenden, the owner and occupier of a caravan site, for an order of
mandamus directed to the Kent justices directing them to hear and determine the applicants appeal under the Caravan Sites and
Control of Development Act, 1960, s 7, against a condition subject to which a site licence was issued by the local authority, and
local planning authority, Beckenham Borough Council, that the number of caravans to be stationed on the site at any one time
should not exceed thirteen. The facts are set out in the judgment of Lord Parker CJ.

Nigel Bridge for the applicant.


J H R Newey for the respondents.

8 March 1963. The following judgments were delivered.

LORD PARKER CJ. The applicant is the owner and occupier of about two acres of land known as Pond Cottage Caravan Site
at West Wickham in Kent. At all material times from 1939 down to the present time that land has been used as a caravan site, but
at no time have more than thirteen caravans been on the land at any one time. Accordingly, on the coming into force of the
Caravan Sites and Control of Development Act, 1960, d, the site was an existing site within s 13(a), and the group of ss 1320
came into play. In particular, by s 17 it is provided:
________________________________________
d 29 August 1960

(1) This section shall apply to any application for a site licence in respect of an existing site which is made within two
months of the commencement of this Act, or within such longer period as the local authority to whom the application is
made may, having regard to the special circumstances of the case, allow, other than an application in respect of a site which
has at the date of the application the benefit of a permission for the use of the land as a caravan site granted under Part 3 of
the Act of 1947 otherwise than by a development order. (2) On the making of an application to which this section applies,
the local authority to whom the application is made shall take any steps required for transmitting the application to the local
planning authority and the local planning authority may grant permission for the use of the existing site as a caravan site
under Part 3 of the Act of 1947 as if the application for the site licence were an application for such permission (and as if
compliance with s. 36 and s. 37 of the Town and Country Planning Act, 1959 (which impose requirements to be complied
with before certain applications for planning permission are entertained), were not required).
Application was duly made by the applicant within the said two months, and the local authority, namely, the Council of the
Borough of Beckenham, referred the question of planning consent to the local planning authority which, incidentally, was that
council, acting under delegated powers from Kent County Council. As local planning authority, the council thereupon granted
planning permission for the use of the land as a caravan site, but subject to the condition that the 246 number of caravans to be
stationed on the site at any one time should not exceed thirteen. The applicant appealed to the minister under s 16 of the Town
and Country Planning Act, 1947 (see s 17(5) of the Act of 1960), but the minister upheld the condition imposed by the local
planning authority. Thereupon the Beckenham borough council as local authority issued to the applicant a site licence under the
Act of 1960 subject to a number of conditions, including as the first that which was the subject of the planning permission,
namely, that the number of caravans to be stationed on the site at any one time should not exceed thirteen. Section 7 of the Act of
1960 provides for an appeal to the justices against conditions in a site licence, and empowers the justices to vary any condition if
they are satisfied that it is unduly burdensome. The applicant appealed to the justices against the first condition limiting the use
of the site to thirteen caravans. At the hearing, counsel for the local authority took a preliminary point that the justices had no
power to vary the condition so as to permit an increased number of caravans to come to the site. On 3 December 1962, the
justices upheld this contention holding that, since they had no jurisdiction to vary the condition in the way required by the
applicant, they would not proceed with the appeal. It is in respect of this refusal to proceed further that the present order of
mandamus is sought. May I say at the outset that the justices clearly went into the matter with the greatest care and put their
reasons into writing. The document admirably sets out the rival contentions and the justices decision, and they are to be
congratulated on the manner in which it is presented.
The point at issue here, which is a pure point of law, is whether a local authority, in issuing a site licence and imposing
conditions, is bound to act within the terms of the planning permission (including any conditions attached thereto) the existence
of which is a condition precedent to the issue of a site licence. To take the present case, could the local authority have imposed a
condition providing not for thirteen caravans or less being on the site at any one time, but providing for, say, fifty caravans? If
not, then the justices had no jurisdiction in the sense that they could not vary upwards the number of caravans permitted on the
site. The answer to this question depends, I think, on a true construction of the Act of 1960 in the light of the Act of 1947.
It is, I think, well recognised that the Act of 1947 did not give a sufficient control over the use of land as caravan sites, and
that the object of Part 1 of the Act of 1960 was to give added control. The real question, as I see it, is whether that new control is
a wholly independent control, in the sense that it can provide for the use of land as a caravan site outside the limitations imposed
by planning control, or whether it only operates within the boundaries of the planning permission. I confess that, unless
constrained to the contrary, I should have thought that the latter represented the true position. To have two controls covering in
any respect the same ground, and permitting the added and later control to enlarge the user while the original and earlier control
still remains in force seems quite unreal. To take an example, can it really be said that the scheme of legislation permits the local
authority to impose a condition that the caravans be painted red when the planning permission has provided that they shall be
green?
As against this, it is only right to say that nowhere in the Act is this expressly stated. Section 5 sets out the conditions which
may be imposed, and makes no provision to the effect that they cannot exceed in scope the conditions of the planning permission.
Section 3(3), however, provides:

(3) A local authority may on an application under this section issue a site licence in respect of the land if, and only if,
the applicant is, at the time when the site licence is issued, entitled to the benefit of a permission for the use of the land as a
caravan site granted under Part 3 of the Act of 1947 otherwise than by a development order.

And with this must be read s 29(4):

Any reference in this Part of this Act to permission granted under Part 3 247of the Act of 1947 for the use of land as a
caravan site shall be taken as a reference to such permission whether or not restricted in any way or subject to any
condition or limitation, and any reference in this Part of this Act to such permission shall include a reference to permission
deemed to be granted under the said Part 3.

It follows that the reference to permission in s 3(3) includes permission subject to any condition. Accordingly, it is the
permission with the condition which is the necessary prerequisite to the issue of the site licence. On a literal reading it can, no
doubt, be said that this permission with the condition is merely the key which permits the issue of the site licence. On the other
hand, no violence is done to the language of the subsection by construing it as meaning that just as the site licence must be
confined to the user of the land or part thereof the subject of the planning permission so it must be within the limits of the
condition on which alone that user is permitted.
It is, however, said that there are other considerations which should lead the court to the opposite conclusion. Reference is
made to s 4, s 5(3)and s 20(4), as indicating that the legislature has expressly provided, where so required, that the conditions in
the site licence shall follow conditions in the planning permission. Section 5(3) and s 20(4), however, do not, as it seems to me,
assist, since the conditions which are to be, or may be, included thereby in the site licence are not conditions of the planning
permission. Section 4 on the other hand does make conditions as to time in the site licence correspond with those in the planning
permission, and to this extent supports the argument. In my judgment, however, that provision is in essence providing an
exception to what is the general principle, namely, that a site licence is without duration (see also s 10). Reference again is made
to the different considerations involved in the two controls. Thus, whereas planning permission is dependent on planning
considerations in which the ministers discretion is unfettered (see s 16 of the Act of 1947), a site licence and the conditions
attached thereto are concerned with sanitary and other considerations (see, in this connexion, s 5(6) and the model standards) and
the justices determine such matters. Again, it is pointed out that the enforcement provisions and penalties under the two controls
are different. All this is perfectly true but, as it seems to me, it does not determine the real question, namely, whether the area of
the respective controls are represented by two concentric circles, that of the site licence being within the area of the planning
control, or whether, as the applicant contends, the areas are two independent circles.
A further argument put forward concerns the circumstance that, in the present case, we are concerned not only with an
existing site but with a site which has both before and after the appointed day been used for caravans and, therefore, under the
Act of 1947 required no permission. True, only thirteen caravans have ever been there at one time but, subject to such an
intensification of user as might amount to a material change of use, the applicant could have increased the number. The Act of
1960, however, has made a permission, the written document, a condition precedent to the issue of a site licence and, accordingly,
in the present case the only method of challenging the ministers condition is for the applicant to challenge any enforcement
notice that might be served should he station more than thirteen caravans on the land. But he says that he is willing to run the
risk of an enforcement notice if he can get the condition of the site licence varied. Unless he can get that varied, he will be
prosecuted for breach of the condition in the site licence and never be able to challenge the ministers decision.
I am far from suggesting that the ministers decision in the present case is unenforceable but, be that as it may, this Act is not
dealing solely with the comparatively small number of existing sites but is providing machinery for the future, when, if
circumstances change, an applicant who is not satisfied with his existing planning permission can apply for an extended one. The
point is not an easy one but, on the whole, I have come to the conclusion that the justices were right and I would refuse this
application.
248

ASHWORTH J. The Caravan Sites and Control of Development Act, 1960 (hereinafter called the Act of 1960), was passed,
as appears from the preamble, to make further provision for the licensing and control of caravan sites. Experience had shown the
need for more effective control of such sites than was possible under the Town and Country Planning Act, 1947, or the Public
Health Acts. Accordingly, there was introduced by Part 1 of the Act of 1960 a scheme of further control summarised in s 1(1),
which provides that:

Subject to the provisions of this Part of this Act, no occupier of land shall after the commencement of this Act cause or
permit any part of the land to be used as a caravan site unless he is the holder of a site licence (that is to say, a licence under
this Part of this Act authorising the use of the land as a caravan site) for the time being in force as respects the land so
used.

By s 3 of the Act, the issue of site licences was entrusted to local authorities. Subsection (3) of that section restricts their powers
in this respect by providing that:

A local authority may on an application under this section issue a site licence in respect of the land if, and only if, the
applicant is, at the time when the site licence is issued, entitled to the benefit of a permission for the use of the land as a
caravan site granted under Part 3 of the Act of 1947 otherwise than by a development order.

The permission referred to in that subsection is hereinafter called planning permission. The requirement that planning permission
should have been granted before a site licence can be issued is, in my view, a plain indication of Parliaments intention that the
new scheme of control should be linked with the already existing system of planning control. A further, and possibly stronger,
indication of that intention is to be found in s 22(1) of the Act of 1960, which provides that

before a local planning authority grant permission for the use of land as a caravan site under Part 3 of the Act of
1947 they shall, unless they are also the authority having power to issue a site licence for that land, consult the local
authority having that power.

In passing, it may be noted that, in the present case, Beckenham Borough Council were both the local planning authority (under
delegated powers) and also the local authority for the purpose of issuing a site licence. Section 5 of the Act of 1960 empowers a
local authority to attach conditions to site licences, including a condition restricting the total number of caravans stationed on the
land at any one time. It was in pursuance of this power that the local authority in the present case imposed a condition restricting
the total number of caravans to thirteen. Section 7 confers on any person aggrieved by any condition (other than one dealing with
the display on the site of a copy of the licence) a right of appeal to the local magistrates court, and sub-s (1) provides that

the court, if satisfied (having regard amongst other things to any standards which may have been specified by the
minister under sub-s. (6) of the said s. 5) that the condition is unduly burdensome, may vary or cancel the condition.

The present motion arises out of a refusal by the local magistrates to entertain an appeal by the applicant against the condition
restricting the total number of caravans to thirteen. They took the view that the planning permission granted to the applicant,
containing a condition restricting the total number of caravans to thirteen, constituted the planning permission required as a
condition precedent to a site licence, and that it was not open to them to vary the condition in the site licence by inserting a larger
number than thirteen.
On behalf of the applicant, it was contended that there is no provision in Part 1 of the Act of 1960 to the effect that a
condition as to the number of caravans on a site contained in a site licence must not conflict with a corresponding condition in the
relevant planning permission. It was pointed out that, in s 4(1) of the Act 249 of 1960, Parliament expressly enacted that a
condition in a planning permission as to duration is to be included in the same terms in the site licence; in such a case, the holder
of the site licence could not hope to appeal successfully under s 7 against that condition. Moreover, s 7(1) itself only precludes
an appeal against a condition requiring display of the licence, and, in the absence of clear words limiting the right of appeal
against other types of condition, such right of appeal must, as a matter of construction, be regarded as unfettered. Further, s 20(4)
contains, in effect, a limitation on the right of appeal in respect of a condition providing for the termination in due course of the
use of the land as a caravan site. In my view, there is great force in these contentions and, if the decision in this case involved
only a matter of construction of the Act of 1960, I would be in favour of the applicant. On general principles it would, in my
view, be wrong to imply in Part 1 of the Act a provision that a condition in a site licence is not to conflict with a corresponding
condition in the relevant planning permission or, alternatively, that a condition in a site licence is not to be less restrictive than the
corresponding planning condition. To make such an implication would, in my view, be to legislate in a matter where Parliament
has failed to do so, and I am all the less willing to make it when there are express provisions amongst which the implied
provisions would naturally have been included if that had been Parliaments intention.
The view already expressed does not, however, dispose of the matter. I have already mentioned the link between the new
system of control introduced by the Act of 1960 and the already existing planning control and, quite apart from such indications
as are to be found in the Act of 1960 itself, I should have regarded it as self evident that the two forms of control are intended to
be operated in harmony and not in conflict. It is true, as counsel for the applicant pointed out, that to some extent different
considerations are applicable to planning permission from those applicable to a site licence. There are differences in procedure
for the purpose of obtaining the one as compared with the other, different methods of appeal, different methods of enforcement.
But in spite of these differences there is, in my view, an overriding purpose common to them both, namely, the prevention of
indiscriminate development, and in some respects the jurisdictions, so to speak, overlap. The present case is a good example. If
for planning purposes it was considered right that the total number of caravans on the site should not exceed thirteen, it seems to
me impossible to hold that the inclusion of a similar restriction in the site licence involves the imposition of an unduly
burdensome condition. If consultations between a planning authority and a local authority had been required in this case in
pursuance of s 22(1) of the Act of 1960, it may be supposed that the number of caravans to be permitted on the site would have
been one of the most important features discussed, and the object of such consultation would be to a large extent frustrated if the
number fixed in the site licence exceeded that to which planning permission restricted the occupier. From the planning point of
view, the restriction to thirteen caravans was upheld by the minister, on appeal to him, and for my part I do not see how the self-
same local authority (which had granted the planning permission) could be thought unreasonable if they adopted the same
restrictive figure for the site licence.
For these reasons, while I think that the Act of 1960 did not, on its true construction, preclude the applicant from appealing
against the condition in question, I think that the justices were right in holding that, in the circumstances, the condition could not
be regarded as unduly burdensome.

WINN J. The point raised is whether or not the justices were right in law in declining to accept jurisdiction to entertain that
appeal. I think myself that the exceptionally lucid and helpful reasoning which the justices have set out in their grounds for
refusal to entertain the appeal, for which I would express my indebtedness, leads strictly rather to the conclusion that the decision
of the justices 250 was that, if they entertained the appeal, they would in law have no option but to reject it than to the conclusion
that they had no jurisdiction to entertain the appeal; the distinction is without substance. What this court must decide is whether
or not the justices had power to vary the condition challenged.
The relevant facts are simple. The Pond Cottage Caravan site is an established site which had been in use prior to 1939.
The maximum number of caravans that have ever been on the site at any one time is thirteen. Pond Cottage Caravan site was an
existing site within the definition in para (a) of s 13 of the Act of 1960; it did not have at the commencement of the Act the
benefit of any planning permission granted under the Town and Country Planning Act, 1947. Within two months of the
commencement of the Act of 1960, the occupier of the site, the applicant, duly made an application in writing in pursuance of s
3(2) of the Act of 1960, for a site licence to the authority in whose area the site is situated, Beckenham Borough Council. That
application, therefore, fell to be dealt with as prescribed in s 17 of the Act of 1960. Within the period laid down in s 17, the
borough council under its own delegated power as planning authority granted planning permission and imposed on that
permission, inter alia, a condition that the number of caravans to be stationed on the site at any one time should not exceed
thirteen. When the applicant got that permission, he exercised his right to appeal under s 16 of the Town and Country Planning
Act, 1947, as applied by s 17(5) of the Act of 1960, to the minister, and in due course there was an inquiry into that appeal. The
outcome was that, on 26 January 1962, the minister dismissed the appeal against the planning condition limiting the number of
caravans on the site to thirteen. Thereafter, on 17 May 1962, the council issued a site licence to the applicant to which were
attached twenty-one conditions, of which the first and for the applicant the most important was that the number of caravans on
the site should not exceed thirteen. Feeling thereby aggrieved, the applicant sought to appeal to the magistrates court for the
petty sessional area in which the site lies.
The issue resolves itself as follows:Can a site-holder on whose planning permission to use a site for caravans there has
been imposed a condition limiting the number of caravans which may be maintained on the site and on whose site licence a like
condition has been imposed, obtain from the justices a relaxation of the limit imposed by the site licence condition?
The pattern of the legislation in the Caravan Sites and Control of Development Act, 1960, so far as relevant, may be
sketched in the following broad lines. Section 1 prohibits the use of land as a caravan site unless the occupier holds a site licence,
and imposes penalties for such use unless a licence be held; such a licence is expressly referred to as authorising the use of land
as a caravan site. Section 9 imposes penalties for use of land in contravention of conditions attached to such a licence. Section
3 provides that applications for site licences are to be made to the local authority in whose area the land is situated, and requires
the authority to issue on application a site licence if, but only if, the applicant is entitled to the benefit of a permission for the use
of the land as a caravan site granted under Part 3 of the Act of 1947, namely, the Town and Country Planning Act, 1947,
otherwise than by a development order. The section expressly provides that such a licence shall be issued within two months
after delivery of such particulars as are required to be delivered by the applicant in any case where the applicant is then entitled to
the benefit of such a permission; but in any case where it is not until a subsequent date that the applicant becomes entitled to such
a permission, the licence is to be issued within six weeks after the date when he becomes so entitled. Section 5 empowers the
local authority to attach to the licence such conditions as it thinks necessary or desirable. By force of the definition section, s
29(4),

Any reference in this Part of this Act to permission granted under Part 3 of the Act of 1947 for the use of land as a
caravan site shall be taken as a 251 reference to such permission whether or not restricted in any way or subject to any
condition or limitation

It follows that, on a literal reading of s 3(3) and of s 5, the local authority is empowered to issue a site licence if the applicant
has a planning permission for the use of land as a caravan site which is subject to such conditions as the one which is relevant in
this case, namely, a condition that not more than thirteen caravans be allowed to stand on the site, and to attach only such
conditions as it thinks necessary or desirable in such interests as are there mentioned, namely, the interests of persons dwelling on
the site in caravans, or any other class of persons or of the public at large. There is no express requirement that any planning
condition relating to manner as distinct from duration of user of the site be incorporated.
Section 7(1) of the Act of 1960 provides that any person aggrieved by any condition (other than a particular condition
expressly excluded by the section) which has been attached to a site licence, may within twenty-eight days after the issue of the
licence appeal to a magistrates court which, if satisfied that the condition is unduly burdensome, may cancel or vary it. Site
licences are in general to be of unlimited duration, but are required by s 4, in the case of any site which has a planning permission
for only a fixed period, to be stated to expire at the end of the same period. The question of jurisdiction for decision by the
magistrates and by this court must be a matter of construction of the whole Act.
An examination of the Act of 1960 reveals plainly, in my opinion, that its purpose was to institute a new control over the use
of land for the reception of caravans more stringent and more readily enforced than planning control. Its effect is to require not
only that a new use of land for this purpose shall be subject to the necessity of obtaining a prior grant of planning permission, but
also that land which had been used for receiving caravans before the appointed day (which was the critical date for the purposes
of the principal Act of 1947), or had been used since that date otherwise than by virtue of permission granted under the Act of
1947, should obtain planning permission before receiving a site licence. This seems to me to be the effect of the fasciculus of s
13 to s 18 of the Act of 1960; in particular, it is to be observed that s 13 defines existing site as comprising land in respect of
which permission for use as a caravan site has been granted under Part 3 of the Act of 1947 and land in respect of which
permission for such use was not required at the commencement of the Act by virtue of s 12(5) of the Act of 1947. Section 17
requires that, except in cases where the local authority seek otherwise to get rid altogether of the use of the land as a caravan site,
consideration must be given to a grant of planning permission for the continued use of such existing sites for the reception of
caravans, unless the site already has the benefit of such a permission. Section 16 is of only temporary effect and should not be
invoked as a guide to construction.
It seems to me that those provisions requiring that, for existing sites no less than for new sites, planning permission be
obtained and making the benefit of such a permission a condition sine qua non for the grant of a site licence would be otiose if
the intention of the legislature had been to empower the local authority to issue an unlimited authority to use any site in respect of
which there was a planning permission, however circumscribed, enabling some caravans to be kept on it. There is a distinction
between a decision limiting the extent of use of land as a caravan site and decisions controlling the manner in which such
permitted use is exercised. The fact that s 17(2) and (5) were enacted implies that Parliament intended that recipients of a site
licence should first acquire a planning permission granted after the commencement of, and so in relation to the existence of, the
new control; the permission and the new control were to be complementary. I cannot accept as tenable any suggestion that
Parliament can have intended to enable site licensing authorities or magistrates to operate their function of fixing conditions of
licences in any manner capable of producing a conflict with decisions of a planning authority or of the minister given in his 252
planning capacity; s 33(8) certainly does not favour any such view. On the contrary, it appears to me that Parliament has
indicated plainly enough, by the sections mentioned, a twofold intention (i) that the planning authority, or the minister on an
appeal, should determine what pieces of land may be used as caravan sites and the extent of such use for any piece of land,
having regard in so deciding to the new control; and (ii) that the authorities entrusted by the new Act with administrative and, in
particular, hygienic control should have power in respect only of land so approved, and only within the limit of the approved
extent of user, both to authorise and to control by the machinery of conditions the manner of use for and by caravans.
I see no indication to the contrary in either s 4 or s 20 or s 5(3) or s 5(6), on all of which counsel for the applicant relied in
the course of his most able and helpful submissions. In my judgment, s 4 is concerned solely with the duration of a licence, and
is not an instance of an express incorporation from the planning permission into the licence of a condition affecting extent or
manner of user during the currency of the licence; therefore, it does not invoke the maxim expressum facit cessare tacitum.
Similarly, s 20, and particularly s 20(4), operate in special circumstances where there will not be any condition imposed on the
planning permission other than a duration condition, but the licence may have conditions of the kind referred to in s 19 designed
to achieve a gradual cesser or user complying with that limitation of duration. Section 5(3), but not s 5(6), requires inclusion of a
particular condition for protection of a specific interest; s 5(6) is merely directory and cannot in good sense be taken to mean that
a density which the minister has specifically prohibited for a particular site on a planning appeal may be treated as approved by
force of his general approval of certain model standards of density.
I appreciate that it is true that the applicant may not be susceptible to effective control by force of the town and country
planning condition alone. The enforcement of planning control is a wholly different procedure from that by which conditions on
a site licence may be enforced; it is not only different, but far more dilatory or susceptible of delays. Parliament, I think, also
appreciated that distinction. In my judgment, the local authority when dealing with the applicants application for a site licence
were bound in law to treat him as having only the benefit of a permission to use his site for the reception of thirteen caravans and
were unable, therefore, to issue to him a licence authorising any greater extent of use; therefore, they were bound to attach the
condition limiting the number of caravans to thirteen. The words in s 3(3) of the Act of 1960 only if the applicant is entitled to
the benefit of a permission have, in my opinion, the same meaning as: only if and to the extent that the applicant is entitled
to the benefit of . It follows, as I think, that the justices could not in law remove or modify that condition, and I would refuse
the motion for mandamus.

Motion dismissed.

Solicitors: Garber, Vowles & Co (for the applicant); Town clerk, Beckenham (for the respondents).

W A H Beckett Terrell Esq Barrister.


253
[1963] 2 All ER 254

R v Hurford
R v Williams
CRIMINAL; Criminal Law

COURT OF CRIMINAL APPEAL


LORD PARKER CJ, ASHWORTH AND WINN JJ
26 MARCH 1963

Criminal Law Forgery Obtaining property under, upon or by virtue of a forged instrument Hire-purchase proposal form for
motor vehicle signed at office of finance company Signature forged Motor vehicle subsequently released by dealers
Whether obtained by virtue of the forged proposal form Forgery Act, 1913 (3 & 4 Geo 5 c 27), s 7 (a).

If, with intent to defraud, a man uses an instrument that is to his knowledge forged in order to start a train of events whereby he
intends to obtain and does obtain, property, he is guilty of an offence under s 7(a) a of the Forgery Act, 1913 (see p 257, letter a,
post).
________________________________________
a Section 7, so far as material, is set out at p 256, letter f, post

The appellant H, in order to obtain (in conjunction with the second appellant), a lorry on hire-purchase from dealers, was
sent by the dealers to the local office of a finance company, where he signed on 20 January 1960, a proposal form in the name of
L, with whom both appellants were associated, but without Ls authority to sign his name. The appellants testified that they took
delivery of the lorry on 19 January 1960 (viz, before the proposal form was signed), and there was no positive evidence to
contradict this. The appellants paid 200 deposit. The dealers witness testified that they would not have released the lorry until
the finance company confirmed the proposal. L was credit-worthy, which the appellants were not. It was found (the jury being
entitled to weigh the true business position against the evidence of the appellants as to date) that the lorry was handed over to the
appellants subsequently to Hs signing the proposal form. On appeal against conviction of obtaining the lorry by a forged hire-
purchase form the appellants contended that the dealers had not been induced by the proposal form to part with the lorry.

Held Possession of the lorry having been delivered to the appellants after the signing of the forged proposal form, the
appellants had obtained the lorry by virtue of the forged proposal form, and were rightly convicted of an offence under s 7(a) of
the Forgery Act, 1913.
Appeals dismissed.

Notes
As to obtaining goods by virtue of a forged instrument, see 10 Halsburys Laws (3rd Edn) 870, 871, para 1686.
For the Forgery Act, 1913, s 7, see 5 Halsburys Statutes (2nd Edn) 987.

Appeals
There were appeals by William Henry Hurford and Douglas Philip Williams against their convictions at Newport Assizes on 23
November 1962, before Roskill J and a jury, of obtaining goods on a forged instrument. They were each sentenced to four years
imprisonment. The facts are set out in the judgment of the court.
The authorities noted belowb were cited during the argument.
________________________________________
b Shorter Oxford Dictionary, p 2362, By virtue of, New Standard English Dictionary, Vol 2, p 2657, By virtue of, Archbolds Criminal
Pleading, Evidence and Practice (35th Edn), para 1932

C N Pitchford for the appellants.


A G Davies for the Crown.
26 March 1963. The following judgment was delivered.

LORD PARKER CJ delivered the following judgment of the court. The appellants were convicted at Newport Assizes of
obtaining a motor lorry by a forged hire-purchase form and they were each of them sentenced by Roskill J, to four years
imprisonment. It is against these convictions that they now appeal on a point of law as to the meaning of s 7 of the Forgery Act,
1913, and also on a point of mixed law and fact.
254
The facts were as follows: at some time in January, 1960, the appellants were minded to get a lorry on hire-purchase; they
went to some dealers, Watts (Factors) Ltd near Gloucester who held repossessed vehicles on behalf of finance firms. They saw
a man called Morgan there and also a lorry in which they were interested. Mr Morgan took them upstairs to the manager, a man
called Coggins, and it seems reasonably clear that the appellants were not in a position to pay for the lorry, but that they were in a
position to pay the sum of 200 as a deposit. Presumably they expressed a wish to purchase the lorry on hire-purchase terms, and
Mr Coggins suggested that they should go to the local office of United Dominions Trust Ltd. The appellant Hurford, who had
given the name of Lunn to Mr Morgan, then went to the local office of United Dominions Trust Ltd and signed a hire-purchase
proposal form in the name of E Lunn. One of the issues before the jury was whether that was a forgery or whether Lunn, with
whom both appellants were associated at some time, had authorised the proposal form to be signed in his name. The jury quite
clearly came to the conclusion that the appellant Hurford had no authority to sign in Lunns name and that the document was,
accordingly, a forgery. It seems reasonably clear that that form was signed on 20 January 1960. Thereafter, it appears that
inquiries were made concerning Lunn, who undoubtedly, unlike the appellants, was credit-worthy. A report to that effect was
apparently made by Mr German of the local office to his head office; ultimately, on 28 January a hire-purchase agreement was
entered into. At some time the appellants obtained possession of the vehicle, and the issue at once arose, when did the dealers,
Watts (Factors) Ltd part with the vehicle? Both appellants said that they had paid the deposit and obtained delivery of the vehicle
on 19 January. If that was right (as the judge emphasised to the jury time and time again) or might be right, then, whatever the
meaning of s 7 of the Forgery Act, 1913, the delivery would have preceded the forged instrument and the vehicle could not have
been obtained to use the words of the section, under, upon, or by virtue of the forged instrument.
The first complaint which is made is that there was no evidence to go to the jury on which they could find what was for the
prosecution to prove, namely, that the delivery of the vehicle had at any rate been after the forged instrument came into existence.
As regards that, as I have said, the appellants gave positive evidence that they took delivery on 19 January. As against that, there
was no positive evidence; nobody representing Watts (Factors) Ltd was in a position to say that the vehicle had been handed over
not on 19 January but on 20 January or at some later date after the forged instrument had come into existence. Mr Coggins in his
evidence said that he was not in a position to recollect when it was delivered; it could have been on 19 January or it could have
been at any day up to and including 28 January but in re-examination he said, and I quote the words which appear to have been
given in the course of argument by counsel for the Crown from his note: That would be after we made the arrangement and they
had confirmed it. We would not release it until then. He was clearly thereby saying: that would be after United Dominions
Trust had confirmed it, in the sense that it was all right then for Watts (Factors) Ltd to part with the vehicle; they would not
release it until then.
The learned judge in dealing with this matter said this to the jury:

Members of the jury, you are perfectly entitled to apply your own good common sense. Can you conceive with the
background history of that lorry, knowing its past history, knowing the rather confused position between U.D.T. on the one
hand and Watts Factors on the othercan you conceive when somebody in Watts Factors sent those men across to U.D.T.
to arrange finance that Watts Factors would hand over that lorry without clearance from U.D.T.? You may think it is
contrary to ordinary common sense. Most people have had experience of hire-purchase in one form or another. 255You
are perfectly entitled to bring your own knowledge and common sense into being in deciding this question

Counsel for the appellants has complained of that passage in more than one respect. He says that what was the usual practice, as
spoken to by Mr Coggins is not evidence of what happened on a particular occasion; secondly, that, even if it did amount to some
evidence, little weight should have been given to it on what was clearly a vital question, and the jury should have been warned
how they should regard that evidence; and, thirdly, that the jury should not have been told that they were entitled to use their own
knowledge.
This court feels that the position was not as satisfactory as it might have been; in particular, somebody from United
Dominions Trust might have been called to state that they did give clearance, and only after the forged instrument had come into
existence; but that was never done. Nevertheless, it does seem to this court that the jury were entitled to weigh the evidence of
the appellants as to 19 January against what was not merely the usual practice but what must have been the true business position.
Here were men who could not pay more than a deposit and who wanted hire-purchase terms. They are then sent round to United
Dominions Trust and, as a matter of business, it is almost impossible to believe that the dealers would part with the vehicle
without getting some clearance, as Mr Coggins put it, from United Dominions Trust; in particular, they would want to know
whether they were selling the vehicle to the appellants, or whether they were selling it, which they would have been, to United
Dominions Trust. Accordingly, the clearance that one would expect would be clearance in the sense that a hire-purchase
agreement had been entered into, which would have been clearance on 28 January or on 22 January when Lunns credit-
worthiness had been investigated, or at the earliest on 20 January when a proposal form had been signed. In any event, that
clearance from the business point of view could not have been on 19 January and must have been on a date on or after 20 January
at a time when the forged instrument had come into existence. Accordingly, so far as that point is concerned, this court is
satisfied that the appeals fail.
The other point depends on the true construction of s 7 of the Forgery Act, 1913. That is a section which re-enacts part of
the Forgery Act, 1861, and provides, so far as it is material, that:

Every person shall be guilty of felony who, with intent to defraud obtains property (a) under, upon, or
by virtue of any forged instrument whatsoever, knowing the same to be forged

The indictment in the present case charged the appellants with obtaining from Watts (Factors) Ltd a Dennis Jubilant motor lorry
by virtue of a forged instrument. It is curious that at no time since 1861 has the point which arises here been considered. The
point really is this: does by virtue of in that section mean that there must have been a handing over by the person from whom
the property was obtained with the knowledge of, and in reliance on, the forged instrument? Put another way: must the forged
instrument either wholly or partially have induced that person to part with the property?
Counsel for the appellants contends that that is the natural meaning and the true meaning. He suggests that really this
section must be equated with the obtaining by false pretences which is a misdemeanour, and that the legislature here were merely
making a false pretence by means of a forged instrument not merely a misdemeanour but a felony. Looked at in this way, he
would say that the proper direction to the jury would be exactly the same as in the case of a false pretence, and that the
prosecution must satisfy the jury that it was the false pretence, in this case the forged instrument, which induced another to part
with the property. Counsel for the Crown, on the other hand, maintains that the legislature by this section were casting the net
very much wider, and he would say that the proper approach is not to look at the man who parts with the property and ask oneself
what did he rely on, was he induced by the forged instrument? 256but is to approach the matter from the angle of the alleged
offender and ask oneselfdid he start a train of events by means of the forged instrument which culminated in his obtaining the
property? Those shortly are the rival contentions.
This court has come to the conclusion that those words by virtue of, following as they do the words under and upon,
should be given the very widest meaning. It seems to this court that if, with intent to defraud, a man uses an instrument to his
knowledge forged to start a train of events whereby he intends to obtain, and does obtain, property, he is guilty of an offence.
The learned judge, in dealing with this matter, ruled in the course of the argument c as to the meaning to be given to the
section and, in addressing the jury, said:
________________________________________
c In giving his ruling, given at the end of the prosecutions case and before the appellants gave evidence in their defence, Roskill J said It
seems to me that I have got to construe s 7(a) of the Forgery Act, 1913. All three learned counsel have been unable to supply me
with any relevant authorities on the true construction of the words by virtue of. Therefore, I deal with the matter as one of construction
What does by virtue of any forged instrument mean? Does it mean in knowledge of the existence of or in reliance upon? I find it
very difficult to think the language used in s 7 was intended by Parliament in 1913 to mean the same as the legal phrase in reliance upon.
On the contrary it seems that the words by virtue of have some wider connotation. It is always difficult and perhaps dangerous where
Parliament has used a set of words to paraphrase them and use another set of words; but if I had to find another set of words to explain what
I believe to be the true meaning of the phrase used in the Act of Parliament, it would be these, that something happens by virtue of a forged
instrument if it happens in consequence of that forged instrument. To put the matter negatively, rather than positively, that something would
not have happened had not that forged instrument come into existence.

I said, and by my ruling you are bound on that, I did not think that was right d, and something can be done or obtained
by virtue of a forged instrument if in fact the person concerned got the lorry by means of the existence of or in consequence
of there being in existence this forged instrument.
________________________________________
d That referred to the submissions made on behalf of the appellants that led to the ruling summarised in footnote (1) above. In his
summing-up, Roskill, J., identified the essence of the contention in the following words, They submitted, you remember, there has to be
direct reliance by the person who gives up the lorry upon the forged instrument. He then continued I said, and by my ruling as
quoted in the text above

It might possibly be said that that direction to the jury is not entirely in accordance with the construction which this court has put
on the section. But, in reading the summing-up as a whole, it seems perfectly clear that what the court thinks is the right
construction was fully conveyed to the jury, namely, the question whether they felt sure that, with intent to defraud, the appellants
had brought into existence and used this forged instrument in order to start a train of events whereby they intended to obtain, and
did obtain, the lorry in this case.
For these reasons, this court has come to the conclusion that the appeals fail and should be dismissed. Before parting with
this case, may I thank counsel on both sides for their arguments.

Appeals dismissed.

Solicitors: Registrar, Court of Criminal Appeal (for the appellants); Nash, Field & Co agents for J W Davies, Newport (for the
Crown).

N P Metcalfe Esq Barrister.


257
[1963] 2 All ER 258

J M Allan (Merchandising) Ltd v Cloke and another


LEISURE AND LICENSING: CONTRACT

COURT OF APPEAL
LORD DENNING MR, DANCKWERTS AND DAVIES LJJ
5 MARCH 1963

Gaming Lawful and unlawful gaming Clubs Roulette Royale Fixed payment required on each bet, but regardless of stake,
payable before spin of wheel Whether payment of a fixed sum determined before the gaming began Betting and Gaming
Act, 1960 (8 & 9 Eliz 2, c 60), s 16(7) (b).

Contract Illegality Unlawful purpose Participation in common purpose Illegality of purpose not known to parties
Ignorance of law no defence Hire of equipment for unlawful gaming believed by parties to be lawful Whether contract of
hiring void.

The plaintiffs hired to the defendants, who were interested in the management of a proprietary club, a roulette-table, a roulette-
wheel and a croupiers rake at a weekly rent payable quarterly in advance. They supplied the defendants with a booklet
containing the rules of the game to be played in the club which was in the form of roulette, under which a small charge of 6d was
payable on any bet, regardless of the stake, and was collected before the croupier spun the wheel. The booklet stated that it was a
game designed to be conducted in accordance with the provisions of the Betting and Gaming Act, 1960. The defendants did not
pay the second instalment of the rent on the ground that the agreement was illegal, though at the date the agreement was entered
into neither party knew that the game was unlawful. By virtue of s 16(1)(c), and (7)(b), of the Betting and Gaming Act, 1960,
gaming in a club was lawful if, apart from any annual subscription for membership of the club, the only other payment required
for a person to take part in the gaming was of a fixed sum of money determined before the gaming began.

Held The contract was illegal and no action lay to recover the rent, because
(i) the demand of 6d before every spin of the wheel was not a fixed sum of money determined before the gaming began
within the meaning of s 16(7)(b) of the Act of 1960, since gaming began when the session started and there was not a fresh start
of the gaming at every spin of the wheel (see p 260, letter e, and p 262, letters b, c and e, post).
Quinn v MacKinnon ([1963] 1 All ER 570) considered.
(ii) where two people together had the common design to use a subject-atter for an unlawful purpose, so that each
participated in the unlawful purpose, then the contract for its hiring for that use was illegal in its formation, and it was no answer
for them to say that they did not know that the purpose was unlawful (see p 261, letters a and h, and p 262, letter i, post).
Waugh v Morris ((1873), LR 8 QB 202) distinguished.
Appeal dismissed.

Notes
Section 16 of the Betting and Gaming Act, 1960, has been repealed, and has been replaced, as from 31 March 1963, by s 32 of
the Betting, Gaming and Lotteries Act 1963.
As to unlawful gaming, see Supplement to 18 Halsburys Laws (3rd Edn), para 369A, 3; and for cases on the subject, see 22
Digest (Repl) 448450, 292302.
For the Betting and Gaming Act, 1960, s 16, see 40 Halsburys Statutes (2nd Edn) 345.

Cases referred to in judgments


Quinn v Mackinnon [1963] 1 All ER 570, [1963] 2 WLR 391.
Waugh v Morris (1873), LR 8 QB 202, 42 LJQB 57, 27 LT 265, 12 Digest (Repl) 264, 2041.
258

Appeal
The plaintiffs J M Allan (Merchandising) Ltd brought an action in the county court for 65 against the defendants George Henry
Cloke and John Henry William Whitehead. By their particulars of claim the plaintiffs pleaded that, by an agreement in writing
made on 1 June 1962, between the plaintiffs and the defendants, the plaintiffs let to the defendants a roulette-table, a roulette-
wheel and a croupiers rake for one year from 1 June 1962, at a weekly rent of 5, payable every thirteen weeks in advance, and
that the defendants failed to pay the 65 due on 31 August 1962. The defendants in fact took possession of the equipment and the
game was played at the club until mid-July. On 27 August 1962, the plaintiffs informed the defendants that, consequent on a
decision of a metropolitan magistrate, the rule under which sixpence was paid on each bet was null and void. The defendants
thereon returned and equipment. The defendants pleaded by their defence that the plaintiffs well knew that the defendants had
the conduct and supervision of the Oakwell Park Country Club; they pleaded (by para 3) that the agreement of 1 June 1962, was
made for the express purpose of providing the defendants with the equipment necessary for conducting and permitting the
playing by members of the club of a game of chance known as Roulette Royale; (by para 4) that one Moss on behalf of the
plaintiffs represented orally to the defendants that the game was most lucrative and would produce revenue, that the defendants
entered into the agreement in reliance on this representation, and accordingly that it was a fundamental term or condition of the
agreement, or the plaintiffs warranted, that the game could lawfully be played in the club in accordance with its rules; and (by
para 5) that the conduct and playing of the game in accordance with those rules was in fact illegal, and (by para 6) that the
defendants returned the chattels to the plaintiffs and elected to rescind the agreement or that the agreement was invalid and
unenforceable. The action was tried at Westminster County Court before His Honour Judge Herbert QC who delivered a reserved
judgment on 13 December 1962; he found that the plaintiffs let and the defendants took the equipment for the sole purpose of
playing Roulette Royale, and that both parties honestly thought that the game was lawful, but that the game was unlawful and
that the defendants were entitled to return the equipment and refuse to pay anything further. He dismissed the plaintiffs claim
and they appealed.

Leonard Lewis for the plaintiffs.


Michael Hoare for the defendants.

5 March 1963. The following judgments were delivered.

LORD DENNING MR. The defendants, Mr Cloke and Mr Whitehead, are gentlemen interested in the management of a
proprietary club, the Oakwell Park Country Club. The plaintiffs J M Allan (Merchandising) Ltd are interested in the supply of
roulette-tables. Their trade name, we are told, is the Casino Supply Co. On 1 June 1962, the plaintiffs let to the defendants a
roulette-table, a roulette-wheel and a croupiers rake for one year from 1 June 1962, at a weekly rent of 5 payable quarterly in
advance. The first payment of 65 was made on 1 June 1962, but the next payment due on 31 August 1962, was not made. The
defendants said that the agreement was illegal. The plaintiffs now sue for that amount. The facts are simple. In order to induce
this contract of letting, the plaintiffs prepared a booklet of rules of a game which they called Roulette Royale: Casinos most
exciting game, which is on the general pattern of the game of roulette but with variations so as to ensure that all the winnings go
to the players and no part of them to the bank. The outstanding rule on which this case depends is the first rule:

A small charge of 6d. is payable on any bet, regardless of the stake, and is collected before the croupier spins the
wheel.

On the back of the card it says:


259

Roulette-royale is a game designed to be conducted in accordance with the provisions of the Betting and Gaming Act,
1960.

The plaintiffs clearly represented to the defendants that they could use this roulette table in accordance with those rules charging
6d a time before every spin of the wheel and thereby make substantial profits and pay the rental for the table. The judge said that
the express purpose between the parties for which the table was intended was this game of roulette-royale and not for any other
game played in accordance with different rules.
Accepting, as I do, the view that neither of these parties knew that this game was unlawful, the question is: Is the game of
roulette-royale, played at a club like this in accordance with the rules, unlawful or not? That depends on the proper interpretation
of the Betting and Gaming Act, 1960. Under that Act, it is quite clear that, at a bona fide club, the members are entitled to
indulge in gaming activities, on these conditions: First, the chances in the game must be equally favourable to all the players;
second, all the winnings must be distributed amongst those playing, and furthermore (and this is the condition that comes into
operation in this case), apart from any annual subscription for membership of the club, the only other payment required for a
person to take part in the gaming must be a fixed sum of money determined before the gaming began. Those are the material
words in s 16(7)(b) of the Act of 1960. The question in this case is whether the small charge of 6d referred to in the rules is a
payment required for a person to take part in the gaming which is a fixed sum of money determined before the gaming
began.
It has been argued before us that gaming begins on every occasion when the wheel is spun and, therefore, that it is perfectly
legitimate to have 6d paid to the organisation before every spin of the wheel. This is a point which was left open by the
Divisional Court in the recent case of Quinn v MacKinnon. I am quite clear that the sum of 6d before every spin of the wheel is
not a sum of money determined before the gaming begins. In my judgment, the gaming begins when the players come into the
room and sit down to play. The word gaming is defined in s 28(1) of the Act of 1960. It is the playing of a game of chance
for winnings in money or moneys worth. The emphasis in this connotation is on the playing; it is when the playing begins,
when the session begins, that the sum must be determined. The sum is, so to speak, the entrance money which must be fixed and
pre-determined before the players take part in the game. I find myself in agreement with the opinion which the magistrate
expressed in Quinn v Mackinnon (See [1963] 1 All ER at p 572) that the payment envisaged by s 16(7)(b) was a single payment
similar to table money for bridge which covered all the rubbers played at one session by a player, it being left to the players
choice how long he remains at the table. In order to be lawful, the sum paid by the members of the club who take part in this
game of roulette-royale must be a pre-determined sum, determined before the playing at that session starts. It must be a sum, not
necessarily paid, but at least determined at that time. It is not sufficient for it to be determined before each spin of the wheel. In
my judgment, this game of roulette-royale is unlawful, whether played as in Quinn v MacKinnon, where there was 6d paid with
every stake, or, as has been canvassed in this case, 6d before every spin of the wheel. In each of those cases it is unlawful. It is
only lawful if there is a pre-determined amount before the session in question starts. It may be an afternoon session or an evening
session, but it must be a period which may fairly be regarded as an entire session in itself and not merely part of a session. It
must cover the full period of the entertainment on that occasion.
That is not the end of the case. It certainly was an unlawful game, but counsel for the plaintiffs argues before us that the
parties to this letting were ignorant of its unlawfulness; and that, in these circumstances, as it is a contract which could be
lawfully performed, it is not to be regarded as illegal unless they had a 260wicked intention to break the law. He relied on the
well known judgment of Blackburn J, in Waugh v Morris ((1873), LR 8 QB 202 at p 208). I desire to say that where two people
together have the common design to use a subject-matter for an unlawful purpose, so that each participates in the unlawful
purpose, then that contract is illegal in its formation; and it is no answer for them to say that they did not know the law on the
matter. I would take a comparable case where there is a common design to use a subject-matter for an immoral purpose. If a
landlord lets a flat to a prostitute at a rent beyond any normal commercial rent a, or if he lets her a brougham of a specially
intriguing nature, it may fairly be inferred that it was their common design that it should be used for an immoral purpose. The
letting is unlawful, and he cannot recover the rent or hire. It is different with the washerwoman who washes the clothes of the
prostitute or the butcher who supplies her with meat. They may know of her trade but they charge her normal commercial prices.
There is there no common design; there is no participation in the immoral purpose, but merely knowledge of it, and that is no bar
to recovering the price. Likewise with an unlawful purpose, active participation debars, but knowledge by itself does not. As I
read Waugh v Morris, there was there no participation in any unlawful purpose and the plaintiff could recover. In this case,
however, there was participation. The common design was that a game should be played which was in fact unlawful. I hold that
such a contract is unlawful; and it is no answer for those concerned to say that they did not know the law. It would be an easy
way round the law if we were to permit a person to excuse himself by saying that he did not know the law. Here the judge was, I
think, quite right in holding that there was an express purpose of both that this roulette-table and appurtenances should be used
for this unlawful game. The contract was, in my judgment, illegal in its inception, and no action lies to recover the rent.
________________________________________
a Cf R v Thomas, [1957] 2 All ER 181 at pp 182, 183

I would, therefore, dismiss this appeal.

DANCKWERTS LJ. In this case, the plaintiffs hired out to the defendants a roulette-table, a roulette-wheel and a croupiers
rake under a contract for a year at a rent of 5 a week payable quarterly in advance. It is quite plain that the purpose of the
contract was to enable the defendants to engage in a game of chance called roulette-royale, which was to be played according to
the rules which were supplied by the owners of these chattels. Under those rules, it is provided that a small charge of 6d is
payable on any bet, regardless of the stake, and is collected before the croupier spins the wheel. It is said that that amount of 6d
was to be charged on each occasion when the bets were laid, and that it is not 6d for each bet if more bets than one are
permissible. On the terms of the rule, I should have thought that it was plain that it was 6d on each bet, but it is said that there is
no evidence that more than one bet was accepted from a player for any particular spinning of the wheel, though I should have
thought that it was unlikely in the game of roulette that bets were restricted in that way. However, we have to deal with it on the
footing that one bet only might be accepted and, therefore, only 6d on each spinning of the wheel might be exacted from the
players in the game. If that game was illegal, I agree with Lord Denning MR that the letting was for an illegal purpose and it is
no answer to say that the parties believed it to be a legal game and they cannot be allowed to rest on the plea that they were
ignorant of the law.
The question then is: was it an illegal game? That depends, as it seems to me, on s 16(1) and (7) of the Betting and Gaming
Act, 1960, and s 28(1), containing the definition of gaming, though I doubt whether the definitions really add very much to the
point which we have to decide. Section 16(1) provides for gaming to be lawful if conducted in accordance with conditions of
which one is for the 261 stakes being paid to players as winnings and another (condition (c)) is that no other payment in money
or moneys worth is required for a person to take part in the gaming. That is a plain prohibition. In sub-s (7), we find a
relaxation of condition (c) which is to apply in the case of gaming which is carried on as an activity of a club, and the provision
there (s 16(7)(b)) is:

That, apart from any annual subscription for membership of the club, the only other payment required for a person to
take part in the gaming was of a fixed sum of money determined before the gaming began.

To exact 6d each time a bet is laid by the players engaged in a game of chance is not, in my view, the fixing of a sum of money
determined before the gaming began. It seems to me that the gaming begins when the parties sit down at the table to indulge in
the game of chance, and that there is not a fresh start of the gaming in that sense beginning when one spinning of the wheel has
resulted in the payment of bets and so on and another one is about to start. Actually it seems to me that the gaming starts when
the parties lay their bets in the first round, whatever may be the correct term, of the session, as it has been called, on engaging in
this game. Therefore, I think that the terms of s 16(7)(b) were not complied with in the present case where the 6d payment was
effected each time a bet was made when the parties laid their bets.
I agree with the view of Lord Denning MR that this, therefore, was an illegal game, and I agree that the decision of the
learned county court judge was right and that the appeal must be dismissed.

DAVIES LJ. I also agree. As I understand s 16(7)(b) of the Betting and Gaming Act, 1960, what it permits is, as Lord Denning
MR, has said, something in the nature of an entrance fee for the particular session, if that is an apt word, rather than an ad hoc
charge on each bet transacted or made during the course of the session. It would obviously be difficult in some circumstances to
decide what is or what is not a session. But the argument of counsel for the plaintiffs, submitting as he does that each spin of the
wheel in a roulette party constitutes a separate game, would necessarily mean that, if another game were being played like vingt-
et-un or poker, the statute would permit an ad hoc charge before each deal. It appears to me to be plain that that is entirely
contrary to the words and the intention of the statute.
There was, indeed, another point which was touched on by counsel for the defendants in his argument. But it would not, I
think, be proper to base any judgment in this court on it. If it were, then that point would be covered by the decision of the
Divisional Court in the recent case of Quinn v MacKinnon. As my Lords have said, the rules of this roulette-royale provide that a
small charge of 6d is payable on any bet; and, as Lord Parker CJ pointed out in the case referred to ([1963] 1 All ER at p 573), if
one participant makes one bet and another makes three or four bets on any spin of the wheel, then the requirement of equality
would be defeated if the man who made one bet only had to pay a 6d fee and the man who made four bets had to pay 2s. That
would be another reason for holding that this was an illegal game. But this case does not appear to have been conducted on that
basis, and counsel for the plaintiffs rightly pointed out that there was no evidence as to the permissibility of more than one bet in
this roulette royale, though one cannot help thinking that it would be an odd form of roulette if a person was limited to one bet
only on each spin.
With regard to the other point taken by counsel for the plaintiffs on the decision of the Court of Queens Bench in Waugh v
Morris, it seems to me that, as my Lord has said, the principle enunciated in that case can have no bearing here. The learned
county court judge has made an express finding that the contract here could not be performed without breaking the law, and that
the intention of the parties when they made the contract was to do the very thing which the learned 262 county court judge has
held, and which this court is of opinion, is prohibited by law. In those circumstances, Waugh v Morris can in my judgment have
no application whatever.
I agree that this appeal fails and should be dismissed.

Appeal dismissed.

Solicitors: Burnett L Elman (for the plaintiffs); Tearle & Herbert Jones (for the defendants).

F Guttman Esq Barrister.


[1963] 2 All ER 263

Brown v Roberts and another


INSURANCE

QUEENS BENCH DIVISION


MEGAW J
12, 13, 14, 15, 22 MARCH 1963

Insurance Motor insurance Third-party risks Passengers negligence injuring third party Owner not insured against
third-party liability of passenger Whether breach of statutory duty to insure against third-party risks Whether there was a
user of the vehicle by the passenger within Road Traffic Act, 1930 (20 & 21 Geo 5, c 43), s 35(1).

A pedestrian was struck and injured by a van door negligently opened by a passenger in the van. The owner and driver of the van
was insured against liability to third parties arising out of the use of the vehicle by himself or by any person driving it with his
permission. He was not insured against the third-party liability of passengers. In an action by the pedestrian against the owner of
the van for breach of statutory duty,

Held The duty under s 35(1)a of the Road Traffic Act, 1930, to insure against the third-party liability of any person permitted to
use a motor vehicle applied only to a user of the vehicle by a person in whose relationship to the vehicle there was a sufficient
element of controlling, managing or operating the vehicle, and did not extend to a person who was merely a passenger and who
had, as a passenger, control of a door as he entered or alighted (see p 269, letter d, post); accordingly no breach of statutory duty
was established.
________________________________________
a Section 35(1) is set out at p 266, letter b, post

Notes
As to compulsory third-party insurance, see 33 Halsburys Laws (3rd Edn) 464, para 788; and for cases on the subject, see 29
Digest (Repl) 537539, 36863696.
For the Road Traffic Act, 1930, s 35(1), see 24 Halsburys Statutes (2nd Edn) 602; and for s 201 of the Road Traffic Act,
1960, see 40 Halsburys Statutes (2nd Edn) 882, 883.

Cases referred to in judgment


Elliott v Grey [1959] 3 All ER 733, [1960] 1 QB 367, [1959] 3 WLR 956, 124 JP 58, 3rd Digest Supp.
Ellis (John T) Ltd v Hinds [1947] 1 All ER 337, [1947] KB 475, [1947] LJR 488, 176 LT 424, 2nd Digest Supp.
Gifford v Whittaker [1942] 1 All ER 604, [1942] 1 KB 501, 111 LJKB 461, 166 LT 324, 2nd Digest Supp.
Monk v Warbey [1935] 1 KB 75, 104 LJKB 153, 152 LT 194, Digest Supp.

Action
This was an action against the first defendant for damages for negligence and against the second defendant for damages for
negligence and breach of statutory duty under the Road Traffic Act, 1930, s 35(1). Default judgment was entered against the first
defendant only, with damages to be assessed by the judge under RSC, Ord 36B, r 4. The facts are set out in the judgment.
263

R E G Howe and P E C White for the plaintiff.


S Chapman QC and John D Stocker for the second defendant.

Cur adv vult

22 March 1963. The following judgment was delivered.

MEGAW J read the following judgment. The plaintiff, Mrs Edith Rebecca Brown, in this action claimed damages against the
first defendant, Mrs Hilda Roberts, and the second defendant, Mr Reginald Ernest Nicholls, in respect of personal injuries which
she suffered in an accident which occurred when she was walking along the pavement in Green Street, Enfield, at about five
oclock in the afternoon of 28 January 1960. The plaintiff alleges that the accident happened because the first defendant suddenly
and unexpectedly opened the door of a motor van in which she was travelling as a passenger, at the moment when the van drew
up at the kerb; the door struck the plaintiff, and she was thrown sideways across the pavement and fell, fracturing her left hip
bone. As a result the plaintiff has been seriously incapacitated and has had to give up her employment. The owner of the motor
van, an Austin A35 van, was the second defendant, Mr Nicholls. He had been driving the van immediately before the accident
and was in the drivers seat at the moment when the accident occurred. He had, by chance, met the first defendant, who is his
aunt, a short time previously, and had offered her a lift in his van to take her to a fish and chip shop in Green Street. The place
where the van stopped at the kerb, to enable the first defendant to alight, was directly in front of the fish and chip shop. The van
drew up on its proper side of the road, facing in the same direction as the plaintiff was walking.
The first defendant did not enter an appearance in the action and judgment was entered against her by default, with damages
to be assessed. In accordance with RSC, Ord 36B, r 4, it falls to me to assess those damages. I was told at the outset of the
hearing that the first defendant was present in court though she was not represented; she had in fact been served with a subpoena
on behalf of the second defendant. I was also told that it was understood that the first defendant asserted that she had not been
served with the writ and that she had known nothing of the proceedings against her. I thought it right that some inquiry should be
made as to that matter, though it was not for me to decide whether there was any ground for the judgment against her to be set
aside. The first defendant says that she has no recollection of having seen or been served with the writ. It is said for the plaintiff
that she was duly served. I mention that matter for two reasons. First, although the first defendants negligence or otherwise is a
vital issue in relation to the liability or otherwise of the second defendant, and therefore I have to make findings with regard
thereto, those findings relate solely to the action against the second defendant. Secondly, whatever may be the merits or demerits
of the first defendants contention that she was not served, the mere fact that I shall assess the damages against her on the basis of
the existing judgment is not intended to prejudice any application to set aside the default judgment against her, if she should see
fit to make such application.
I propose to deal with the facts quite shortly. The plaintiffs evidence as to having been suddenly struckat the moment she
knew not by whatas she was walking normally and without hurry along the pavement, was corroborated to the full by Mrs
Rose Anthony, a friend of the plaintiff, who had been walking with her, but at the time of the accident was a foot or so behind her
and a little to her left: that is, towards the inner side of the pavement. Mrs Anthony was able to add that what struck the plaintiff
was the door of the van. It is true that Mrs Anthony was mistaken as to the direction in which the van door opened. She was
confident that it was hinged at the rear of the door, so that it opened backwards. The second defendant says, and I accept it, that
the door is hinged at the front, close to the windscreen, and that it opened forwards. That error does not lead me to have any
doubt as to the substantial accuracy of the rest 264 of Mrs Anthonys evidence. She and the plaintiff were both, in my judgment,
not only truthful but reliable witnesses as to the essence of what happened. The second defendant said that he saw the first
defendant opening the door a few inches and putting her head out to have a look. The first he knew of an accident was a little
later when he heard a thud on the pavement. He had no idea at the timehe had no idea even later when the plaintiff was taken
away in the ambulancethat there was any suggestion that she had been struck by the van door. The first defendant also said
that when the van drew up she certainly did not fling open the door or jump out. She suffers from arthritis and could not have
moved quickly. She opened the door a little way to see if anyone was about. She saw no one who might be interfered with by the
opening of the door. She neither saw nor felt anyone striking the door, nor the door striking anyone. She saw the plaintiff lying
on the pavement. I am satisfied on the evidence that the first defendant opened the door without taking due and proper care for
pedestrians on the pavement, and that as a result of that negligence the plaintiff was struck by the door, knocked down and
sustained her injuries. There was no negligence on the part of the plaintiff.
The case against the second defendant is put on two grounds. The first ground is that the second defendant was personally
negligent in failing to prevent his passenger, the first defendant, from acting carelessly in opening the door, and in failing to give
warning to pedestrians that this was going to happen. I can find no evidence to support that allegation. There was no evidence
that the second defendant knew, or should have known, that his passenger was likely to behave in this way. Even if he saw, when
it happened, that the first defendant was flinging the door open suddenly and carelessly, he had no opportunity to stop her or to
warn anyone before the accident happened, almost instantaneously. The claim on that ground fails.
Much more difficult questions are raised by the second ground; and they are questions solely of law. It is desirable that I
should say at once that I have reached the conclusion in the end that the plaintiff fails on the second ground also. The policy of
insurance which was held by the second defendant, in purported compliance with his obligation under s 35(1) of the Road Traffic
Act, 1930, was a so-called commercial motor vehicle policy. It gave cover to the insured himself, the second defendant, for
third party liability: that is, Liability at law for compensation for death of or bodily injury to any person caused by or arising
out of the use of the van. Had the second defendant himself been negligent, causing the plaintiffs injuries, the policy would
have given the plaintiff a right to recover her damages from the insurers. The policy also gave cover in respect of the third-party
liability of any person driving the van on the order of or with the permission of the second defendant, subject to certain
immaterial provisos. The first defendant was not driving the van. The policy did not purport to give cover in respect of the legal
liability of any passenger carried in the van, for bodily injury to any third party caused by the negligence of the passenger while
mounting in or travelling in or getting out of the van. The policy therefore does not purport to give any rights to the plaintiff in
the circumstances of this case. It is contended on behalf of the plaintiff that because of the absence of such cover the second
defendant was in breach of his statutory duty under s 35(1) of the Road Traffic Act, 1930, b, that that breach of statutory duty has
caused the plaintiffs loss and that therefore the second defendant is personally liable to the plaintiff for the whole of the damages
which she can prove, arising out of the accident.
________________________________________
b Replaced, as from 1 September 1960, by s 201(1) of the Road Traffic Act, 1960, unaltered

If the first proposition were right, that is that there was a breach of statutory duty as alleged, the second and third
propositions would be established. The decision of the Court of Appeal in Monk v Warbey is applicable. It is here proved that the
first defendant would be wholly unable to pay, except to the 265 most trivial extent, the very substantial damages to which the
plaintiff is entitled against her. The crux of the matter is the first proposition, which involves what appears to me to be a difficult
question of the true construction of the words of s 35(1) of the Act of 1930. I am greatly indebted to both Mr Howe for the
plaintiff and Mr Chapman for the second defendant for their very careful, thorough and lucid arguments. Section 35(1) of the Act
of 1930, (1) is as follows:

Subject to the provisions of this Part of this Act, it shall not be lawful for any person to use, or to cause or permit any
other person to use, a motor vehicle on a road unless there is in force in relation to the user of the vehicle by that person or
that other person, as the case may be, such a policy of insurance or such a security in respect of third-party risks as
complies with the requirements of this Part of this Act.

The third-party risks which must be covered are specified in s 36(1) of the Act of 1930, c. I read the parts of that subsection
which are for the moment relevant.
________________________________________
c Replaced, as from 1 September 1960, by s 203(1), (2) and (3)(a) of the Road Traffic Act, 1960

36(1). In order to comply with the requirements of this Part of this Act, a policy of insurance must be a policy which
(a) is issued by a person who is an authorised insurer within the meaning of this Part of this Act; and (b) insures such
person, persons or classes of persons as may be specified in the policy in respect of any liability which may be incurred by
him or them in respect of the death of or bodily injury to any person caused by or arising out of the use of the vehicle on a
road: .

Counsel for the plaintiffs submission is that the first defendant was at the time of the accident using a motor vehicle, namely, the
second defendants van, on the road, and that she was doing so with the permission of the owner, the second defendant. She was
using it because she was a passenger in it. She was using it, on the road, as a means of transport between the place where she
entered the vehicle and her destination in Green Street. Therefore, the second defendant was in breach of the duty which s 35(1)
imposed on him, because he permitted her thus to use his vehicle on the road without there being in force, in relation to that use
by her, a policy of insurance which covered the risks specified in s 36(1) of the Act of 1930; namely, the liability which might be
incurred by a passenger, being a user of the vehicle on the road, in respect of the death of, or bodily injury to any person,
including the plaintiff, caused by or arising out of the use of the vehicle on a road.
Counsel for the plaintiff contends that use as a verb, and use or user as a noun, are words which in their natural and
ordinary meaning are of wide import. In the ordinary employment of language, to use is entirely apt and appropriate to cover
the concept of a person availing himself of a motor vehicle in order to travel as a passenger therein from place to place. Counsel
for the plaintiff referred to definitions of the verb to use given in, I think, the Concise Oxford Dictionary: to employ for a
purpose; to handle as an instrument; to put into operation; to avail oneself of. He also referred to the judgment of Lord
Parker CJ, in Elliott v Grey, a case in which the Divisional Court decided, in criminal proceedings, that the owner of a motor
vehicle was using that motor vehicle on the road, although the motor vehicle in question had broken down and was left in the
road outside the owners house in a condition in which it could not be mechanically propelled because the engine would not
work, the battery had been removed and there was no petrol in the tank. In his judgment ([1959] 3 All ER at p 736; [1960] 1 QB
at p 372), Lord Parker indicates that the word use is equivalent to have the use of, or, perhaps, to have the advantage of a
vehicle as a means of transport.
266
It is clear, and it is conceded by counsel for the second defendant that there may be more than one person who is using a
vehicle at any given time, for the purpose of s 35(1). It is clear further, from Elliott v Grey and other authorities, that the element
of driving the vehicle is not an essential element of using. Nor, of course, is use of the vehicle on a road confined to the
owner of the vehicle.
The Act of 1930 itself, as counsel for the plaintiff points out, indicates that to use is not co-terminous with to drive or
to be in charge of. For example s 15(1) refers d to any person who when driving or when in charge of a motor vehicle on a
road. Section 50 also refers e to any person in charge of a vehicle and contains the phrase likely to cause danger to other
persons using the road. There are many other similar instances. Using the road, counsel for the plaintiff submits, is obviously
an entirely general phrase. There is no reason, he suggests, why using should be given any narrower, or any artificial, meaning
in s 35(1).
________________________________________
d See now s 6(1), (2) of the Road Traffic Act, 1960
e See now s 16 of the Road Traffic Act, 1960

Counsel for the second defendant, on the other hand, submitted that the proposition advanced was a startling one. It would
mean, he said, that very many people, probably millions, since 1930 have been committing a criminal offence every time that
they have entered a bus or a taxi-cab, unless it should turn out that there was in force an insurance policy covering the possible
liability of passengers to third parties for negligence while the passengers are travelling in, or getting into or out of, the bus or
taxi-cab. If a passenger in a private car is using that car on the road, a passenger in a bus is equally using that bus, being a
motor vehicle, on the road. If there were no policy in force to cover the passengers third-party liability, the passenger, as a user,
would be himself guilty of a criminal offence, if counsel for the plaintiffs construction is right. So also, says counsel for the
second defendant, with various other provisions of the Act of 1930. The most striking example, perhaps, is s 3(1), which
providesf that it shall not be lawful to use on any road a motor vehicle which does not comply with the voluminous and detailed
Motor Vehicles (Construction and Use) Regulations, 1955, which have been made under that section. Is it to be imagined, asks
counsel for the second defendant, that Parliament intended, or has enacted, that any passenger in a vehicle, public service or
otherwise, should be guilty of a criminal offence if it turns out that the vehicle in which he happens to be a passenger does not
comply in all respects with the regulations of 1955? Yet this would be so if the phrase use on any road a motor vehicle in s 3(1)
meant what counsel for the plaintiff says the corresponding words meant in s 35(1). The difference in the order of the words in
the two sections is not material.
________________________________________
f See now s 64(2) of the Road Traffic Act, 1960

Counsel for the second defendant also referred to various other provisions of the Act of 1930 which he said were pointers, of
greater or less value, towards the conclusion that use in s 35(1) does not and cannot mean what counsel for the plaintiff says it
means. I do not propose to refer to those interesting and intricate arguments and counsel for the plaintiffs equally interesting
answers, except to mention proviso (ii) to s 36(1)(b). That proviso maintains, by way of exception to the proviso g, the obligation
to have third-party insurance in respect of risks of death of or injury to passengers in the case of inter alia vehicles used for hire
or reward; but the proviso excludes the necessity, in respect of vehicles not covered by the exception, to have insurance for third-
party liability
________________________________________
g See now the exception enacted by the proviso to s 203(3)(a), (4)(a) of the Road Traffic Act, 1960

in respect of the death of or bodily injury to persons being carried in or on or entering or getting on to or alighting
from the vehicle at the time of the occurrence of the event out of which the claims arise.
267

Counsel for the second defendant is, I think, entitled to say that that is an extraordinarily clumsy way of saying persons using the
vehicle as passengers, if use has the connotation for which counsel for the plaintiff contends. In other words, it may well be
that the legislature avoided the simpler phrase because, if the word use had been applied to persons who were passengers, it
might have extended use in other parts of the Act of 1930 further than the legislature intended. Further, it may be asked, why
should Parliament exclude an obligation to insure against the liability of one passenger for injury caused by him to another
passenger, but still make it obligatory to insure against the liability of a passenger to some third person who is not a passenger?
Counsel for the second defendant then says that the meaning of use in s 35(1) must necessarily be narrower than the
plaintiff contends. It is fair to say, I think, that he recognised some difficulty in defining with precision the scope of the more
limited meaning. That is not necessarily a fatal objection, though it is, of course, not altogether satisfactory to find a provision of
this sort, involving the everyday incidents of life and also giving rise to criminal liability, incapable of precise definition.
Counsel for the second defendants exposition was that, for the purposes of use of a vehicle on the road there must be at least
an element on the part of the user, of controlling, managing or operating the vehicle, as a vehicle.
Counsel for the plaintiff replied to counsel for the second defendants submissions, as regards the extraordinary
consequences of the acceptance of his submission, in two ways. First, he pointed out that, once it had been held by the courts that
a passenger is a user for the purposes of s 35(1), insurance companies will take steps to see that their policies for the future cover
such risks. Indeed, they will have to do so in order to give the required certificate of third party insurance to enable their assured
to obtain road fund licences. Secondly, as regards the possible criminal responsibility of a large number of morally innocent
persons, he points out that the courts have said more than once that discretion has to be exercised in the launching of prosecutions
in matters of this nature; and that, if one who is morally innocent but technically guilty is unfortunately prosecuted, an absolute
discharge can be given. The possible wide extension of technical, criminal responsibility is not, he contended, a valid ground for
reading the word use in a narrower sense than its natural and ordinary meaning.
As regards counsel for the second defendants submission that use means control, manage or operate, counsel for the
plaintiff contended that even if, contrary to his contention, the word did have that limited meaning, nevertheless the plaintiff
would still succeed because the control or management or operation should not be confined to the vehicle as a whole, but should
apply to any part of it. Here, he said, the first defendant was at the relevant time controlling, managing or operating the part of
the van, namely the door, which caused the accident.
I was referred to numerous decided cases which, it was agreed, were no nearer than on the fringe of the present case. I do
not find it necessary to refer to any of them otherwise than to mention that in two of them only could it be suggested that the
question whether passengers are users is even touched on. Those two cases are Gifford v Whittaker ([1942] 1 All ER 604 at p
605; [1942] 1 KB 501 at p 504), where Viscount Caldicote CJ, is, as I understand the passage, expressing no view of his own, but
is merely summarising counsels argument; and John T Ellis Ltd v Hinds ([1947] 1 All ER 337 at p 341; [1947] KB 475 at p 487),
where there are certain dicta of Humphreys J but I do not think that the learned judge had the present problem in mind.
I prefer counsel for the second defendants submissions. In my judgment, it would be inconsistent with the indications to be
derived from the remainder 268 of the Act of 1930, if I were to hold that use a motor vehicle on the road in s 35(1) included the
mere relationship of a passenger to the motor vehicle or to a particular part of the motor vehicle. I am particularly impressed by
the argument founded on s 3(1) of the Act of 1930, h. It is, I think, inconceivable that Parliament intended that one who was a
passenger in, say, a public service vehicle should be regarded as using that vehicle on a road, so that he would be guilty of a
criminal offence if the vehicle were not properly constructed. It is, I think, difficult to suppose that Parliament intended to
exclude the necessity of insuring against the liability of one passenger to another passenger for negligence in the course of a
journey, yet intended to include the necessity of insuring against the liability of a passenger to a third party who is not a
passenger. Further, if counsel for the plaintiffs submission were right, it would, as I see it, necessarily and logically follow that
the owner of goods carried in someone elses motor vehicle would be using that vehicle on the road, and so would be guilty of
an offence if there were a non-compliance, by the owner or driver of the vehicle or anyone else, of any obligation, not only in
relation to insurance, which is made by the Act a duty of a person who [uses] a motor vehicle on a road.
________________________________________
h See now s 64(2) of the Road Traffic Act, 1960

In my judgment, counsel for the second defendant is right in his contention that a person does not use a motor vehicle on
the road for the purposes of s 35(1) of the Act of 1930, i unless there is present, in the person alleged to be the user, an element of
controlling, managing or operating the vehicle at the relevant time. Precisely what the extent of that element may be, it is
unnecessary to seek to define. There was no such element present in the relationship between the first defendant and the second
defendants van. I do not accept that the control or management or operation of a door of the vehicle by the passenger entering or
alighting amounts to the necessary control or management or operating of the vehicle.
________________________________________
i See now s 201(1) of the Road Traffic Act, 1960

Accordingly, in my judgment, the plaintiffs claim against the second defendant fails.
I have in any event to deal with the question of damages as against the first defendant, Mrs Roberts, for reasons which I
have already given. Had it been necessary for me to assess damages against the second defendant, the amount of damages would
not be difficult.
[His Lordship referred to the plaintiffs injury, earnings (6 14s 9d net, weekly) and age (60), stated that special damages
down to 13 March 1963, were agreed at a figure of 1,093 9s 6d, and awarded for pain and suffering and loss of amenities and of
future earnings a further 1,500, and continued:] Accordingly, I assess the damages against the first defendant at 2,593 9s 6d
Had I held that there was liability on the part of the second defendant, I should have awarded the same against him. As it is, it
may well be that this unfortunate lady, for whom everyone must have the greatest sympathy, will receive little or nothing by way
of compensation for her injuries.

Judgment against the first defendant only for 2,593 9s 6d.

Solicitors: Russell, Jones & Walker (for the plaintiff); L Bingham & Co (for the second defendant).

Mary Colton Barrister.


269
[1963] 2 All ER 270

Unity Finance Ltd v Woodcock


CONSUMER; Consumer credit

COURT OF APPEAL
LORD DENNING MR, DANCKWERTS AND DAVIES LJJ
8 MARCH 1963
Hire-Purchase Guarantee Recourse agreement Dealer undertaking to pay finance company hire-purchase price less
amount already paid if hire-purchase agreement terminated Whether contract of guarantee Purchase price of goods below
300 Hirer making default in payment of instalments Finance company retaking possession of goods when more than one-
third of price paid Release of hirer Effect on agreement of guarantee Hire-Purchase Act, 1938 (1 & 2 Geo 6 c 53), s 11(1)
(2).

By a recourse agreement in writing dated 5 November 1957, a finance company, at the request of a dealer, arranged for financing
hire-purchase sales of motor vehicles by him. By the agreement, the dealer undertook that, if a hire-purchase agreement entered
into by the finance company at his request should be determined by either party or in any manner whatsoever, the dealer would,
at the request in writing of the finance company, inter alia, pay to the company the total hire-purchase price and other sums under
the hire-purchase agreement less the amount already paid to the company by the hirer. In October, 1959, the finance company
entered into a hire-purchase agreement for a motor car with a hirer at the request of the dealer, the hire-purchase price of the car
being 223 and the agreement being one within the Hire-Purchase Acts, 1938 and 1954. In May, 1960, when less than one-third
of the hire-purchase price had been paid by the hirer, he fell into arrears with his instalments and the finance company terminated
the agreement by notice in writing to him. In August, 1960, the hirer paid a further sum bringing the amount paid by him to over
one-third of the hire-purchase price. In September, 1960, the finance company retook possession of the car, which was damaged,
and thereby, by virtue of s 11 a of the Hire Purchase Act, 1938, the hirer was released from liability. The finance company
claimed to recover from the dealer under the recourse agreement the amount outstanding under the agreement with the hirer.
________________________________________
a Section 11 so far as material, provides: (1) Where goods have been let under a hire-purchase agreement and one-third of the hire-purchase
price has been paid , the owner shall not enforce any right to recover possession of the goods from the hirer otherwise than by action.
(2) If an owner recovers possession of goods in contravention of the foregoing subsection, the hire-purchase agreement, if not previously
determined, shall determine, and(a) the hirer shall be released from all liability under the agreement and shall be entitled to recover from
the owner in an action for money had and received all sums paid by the hirer under the agreement or under any security given by him in
respect thereof, and (b)

Held The finance company were not entitled to recover any sum from the dealer under the recourse agreement, because, where
the finance company (as here) determined the hiring for the hirers default the recourse agreement became a contract of guarantee
(see p 272, letter i, to p 273, letter a, p 273, letter g, and p 274, letter c, post), and the dealers liability as guarantor had been
ended (i) by the finance companys having, by their conduct in retaking the car in breach of s 11(1) of the Hire-Purchase Act,
1938, released (under s 11(2)) the hirer from liability under the hire-purchase agreement (see p 273, letters e and h, and p 274,
letter e, post) and (ii) (per Lord Denning MR) by the finance companys not having give the dealer promptly notice of their
termination of the hire-purchase agreement, as that failure had prejudiced the dealers position (see p 273, letter d, post).
Appeal allowed.

Notes
As to the nature of a guarantee and of a contract of indemnity, see 18 Halsburys Laws (3rd Edn) 411, para 767, and 528, para
973.
270
As to agreements between dealers and finance companies, see 19 Halsburys Laws (3rd Edn) 529, para 850.
For the Hire-Purchase Act, 1938, s 11, see 22 Halsburys Statutes (2nd Edn) 1026.

Case referred to in judgments


Yeoman Credit Ltd v Latter [1961] 2 All ER 294, [1961] 1 WLR 828, 3rd Digest Supp.

Appeal
This was an appeal by the defendant, Arthur H Woodcock, from a judgment of His Honour Judge Lind-Smith given on 20
December 1962, at the Birmingham County Court, ordering the defendant to pay to the plaintiffs, Unity Finance Ltd, 145 10s
under a recourse agreement entered into by the defendant with the plaintiffs on 5 November 1957. The facts are set out in the
judgment of Lord Denning MR.
The cases noted belowb were cited during the argument in addition to that referred to in the judgments.
________________________________________
b Lakeman v Mountstephen (1874), LR 7 HL 17, Coutts & Co v Browne-Lecky, [1946] 2 All ER 207, [1947] KB 104, Reliance Car Facilities
Ltd v Roding Motors, [1952] 1 All ER 1355, [1952] 2 QB 844

R T Monier-Williams for the dealer.


Bernard Caulfield QC and C Stuart-White for the finance company.

8 March 1963. The following judgments were delivered.

LORD DENNING MR. This is an interesting and difficult case. It arises out of an agreement between a finance company and a
dealer. As long ago as 5 November 1957, the plaintiff finance company, Unity Finance Ltd, made an agreement with the
defendant dealer, Mr Woodcock, in anticipation of his afterwards putting forward to them hire-purchase transactions. The
agreement said:

In consideration of the company at any time hereafter agreeing to purchase any motor vehicle or other goods and at
the request of the dealer thereafter to let to any person under a hire-purchase agreement the terms of which had been made
known to the dealer any such motor vehicle or other goods it is mutually agreed as follows:1. That if any such hire-
purchase agreement shall be determined by either party or in any manner whatsoever the dealer will at the request in
writing of the company (a) pay to the company the total hire-purchase price and other sums under the said hire-purchase
agreement less the amount already paid to the company by the hirer. (b) Repossess or assist the company to repossess the
motor vehicle or other goods forming the subject of the hire-purchase agreement. 2. That the dealers liability hereunder
shall be in no way affected by the death or bankruptcy of the hirer or by the company giving time or indulgence to the hirer
or making any composition or agreeing to any alterations in the terms of the said hire-purchase agreement providing the
said alterations shall be brought to the notice of the dealer. 3. If the dealer shall carry out the terms of cl. 1 hereof and
promptly pay the sums due thereunder such title as the company may have in the said motor vehicle or other goods shall
pass to the dealer but the company shall not be liable to the dealer for any defect in the said title.

That is what is commonly called a recourse agreement. It is often taken by hire-purchase companies from dealers.
We can now go straightway to 28 October 1959, when the dealer made arrangements with a hirer, Mr Jones, whereby the
hirer took on hire-purchase a Wolseley 18 horse power saloon motor car which had been made in 1947. The dealer put before the
hirer the hire-purchase form of the finance company, and he signed it. The cash price was 190, and the hire-purchase price was
223 made up as follows: 50 to be paid down in cash; rentals of 7 3s 4d a month for twenty-four 271 months, starting on 26
November 1959; and 1 option to purchase. The form contained the clauses familiar in hire-purchase agreements. The hirer
signed the form as hirer and paid his 50 down in cash; and a Mr Aston signed as guarantor. That hire-purchase agreement came
within the Hire-Purchase Acts, 1938 and 1954, with this result: in case the hirer fell into arrear with his instalments, then, if he
had already paid more than one-third of the hire-purchase price of 223, that is, if he had paid more than 74 6s 8d, then by
reason of s 11 of the Hire-Purchase Act, 1938, the finance company would not be entitled to recover possession of their own
accord. They could recover the car only by action at law. If they should retake it unlawfully without bringing an action for the
purpose, the hirer would be released from all liability under the agreement, and would be entitled to recover back from the
owners all the sums which he had paid. And so would any guarantor. But if less than one-third had been paid, that is, if the hirer
had paid less than 74 6s 8d, the finance company were entitled, in case of default, to retake the car without the necessity of an
action at law.
It so happened that the hirer did not pay his instalments regularly. He fell into arrears and, on 22 May 1960, the finance
company gave him notice of termination. At that date he had paid only 68 10s altogether (50 cash down and three payments of
6 10s, 6 and 6 at various dates). So he had paid less than one-third of the hire-purchase price when the finance company
wrote to him on 22 May 1960, in these words:

As you have made default in payment of rentals due under the above agreement, we hereby give you notice that, in
accordance with the terms of the said agreement, we now put an end to the hiring thereunder and have determined, subject
to the Hire-Purchase Acts of 1938/54, forthwith to seize and retake possession of our property.

The finance company did not, so far as the evidence goes, give any notice or intimation to the dealer that they had determined the
agreement. At that time, 22 May 1960 (inasmuch as less than one-third had been paid), the finance company were in a position to
repossess the car peaceably. But before they did so, the hirer, on 31 August 1960, paid a further sum of 8 in addition to what he
had already paid, so that his total payments came to 76 10s, which was more than one-third of the hire-purchase price. It was
paid apparently to another office of the finance company. The result was that the finance company were not entitled thereafter to
recover possession of the car except by action at law. Yet they did so. They broke the law. On 22 September 1960, they
themselves found the car, much damaged, abandoned on a piece of waste ground, and they towed it (because that was the only
way that they could get it along) to the dealers place; and they left it there in his drive because that was the only place where
they could put it. A few days later, on 27 September 1960, the finance company for the first time gave notice to the dealer, and
demanded of him the amount which they said was owing to them under the recourse agreement. They demanded the sum of 154
12s 6d. The dealer did not pay. The finance company at that time took no action against him, and the car remained in his drive
for over two years. Then the dealer, rather unwisely, in June, 1962, got his solicitors to write to the finance company saying that,
unless the vehicle was removed from his garden, it would be sold to defray storage. That stirred the finance company into action,
and within a very few days they issued the writ in this action claiming under the recourse agreement the sum of 145 10s. The
judge has awarded them that sum, and the dealer appeals to this court. The question in the case is whether the finance company
in the circumstances that have happened can recover this amount.
It seems to me that the principal question is whether the recourse agreement was an agreement of guarantee; for, if it is, then
clearly the dealer is discharged. I think that it is a guarantee. In form it is an agreement of repurchaseto 272 repurchase the car
on request should the hire-purchase agreement be determined. But, in substance, it is an agreement under which the dealer
guarantees the finance company that they will receive the total hire-purchase price. The hirer is the principal debtor; he is the
person who is primarily bound to pay the hire-purchase price (less the nominal option fee). So long as the hirer keeps up the
instalments and the hiring remains in force, the dealer comes under no liability. But, if the hirer should make default and the
hiring should be determined, the dealer would become bound to make up the total hire-purchase price. The dealers obligation is
only a secondary obligation, dependent on the hirers default. I realise that there are circumstances where the hirer may himself
determine the agreement without being in default. He may return the car and pay up all that is required of him for the purpose,
without being in default at all. In those circumstances, the recourse agreement is only a contract of repurchase and not a contract
of guarantee. But in cases where the finance company determine the hiring on account of the hirers default, the recourse
agreement is nothing more nor less than an agreement of guarantee, and the dealer is entitled to the benefit of all the laws in
favour of those who undertake suretyship for anotherexcept in so far as they are excluded by the contract.
In this case, the conduct of the finance company has much prejudiced the position of the dealer. In the first place, when they
gave a notice of termination to the hirer on 22 May 1960, they did not give notice promptly to the dealer as they should have
done if they wished to hold him to the agreement. Meanwhile, the car in the intervening time had been very much damaged
before they gave him notice. In the second place, they have unlawfully retaken the car, contrary to s 11(1) of the Hire-Purchase
Act, 1938, and have thereby released the hirer and the other guarantor from all liability. They cannot afterwards come down on
the dealer for the amount. It is clear that there have been far too many acts done by the finance company, which are prejudicial to
the dealer, for them to be entitled to insist on the guarantee.
For these reasons, I think the appeal should be allowed and judgment entered for the dealer.

DANCKWERTS LJ. I agree, and I think that I can put my reasons quite shortly. The agreement of 5 November 1957, is called,
it is true, an indemnity, but the law is entitled to look at the terms of the agreement to see whether that is a proper description or
not. In my opinion, it is not a proper description. In my opinion, this is an agreement, though not an actual guarantee at the
moment when it is executed, providing for a guarantee in the future or a series of guarantees which are to be the liability of the
dealer if and whenever the finance company agree to let on hire-purchase a car in accordance with the terms recited in the
agreement. In that event, the obligations of the guarantor, that is, the dealer, begin. But what has happened in the present case is
that, by the default of the finance company and a breach by them of the provisions of s 11(1) of the Hire-Purchase Act, 1938, they
have caused, by virtue of the statute, a termination of the agreement and the elimination of any liability by the hirer to pay money
due originally under the hire-purchase agreement and, further than that, they have made any moneys which have already been
paid recoverable, and any moneys paid by the guarantor are to be recoverable. In my view, that put an end to the liability of the
dealer as guarantor. It was a step which entirely altered his position, obviously to his detriment, and it is not protected so far as I
can see by any of the terms of cl 2 of the agreement which provides that the liability is not to be affected by giving time or
indulgence, and so on. It is not giving time or anything of that sort. It is the effect of the statute owing to the breach by the
finance company of its provisions. Thereupon, in my view, the liability of the dealer came to an end. Furthermore, as the dealer
is the guarantor and could recover any moneys he paid under the provisions of the statute from the finance company, it would be
absurd that he should pay and 273 then immediately recover the sums from the finance company. Therefore, on that ground also
his obligations must be put an end to. Furthermore, I feel some doubt whether, when the hire-purchase agreement is determined
in this way under the Act, it ever becomes anything for which the guarantor, the dealer, can be liable under the terms of his
agreement. Clause 1 does not appear to me to be particularly apt to deal with that circumstance because the determination which
has brought the hiring agreement to an end is not a determination by the finance company but a determination by virtue of the
statute.
On all those grounds I agree that the dealer was under no liability under this agreement in respect of the present transaction,
and I agree that the appeal must be allowed and judgment entered for the dealer.

DAVIES LJ. I also agree. In Yeoman Credit Ltd v Latter, to which we were referred, the majority of this court thought that the
document under consideration in that case was not a guarantee, but imposed a primary liability on the person sued. The
document in this case, in my opinion, is a guarantee. It is, perhaps, not without interest to note that the author of Goode on Hire-
Purchase Law and Practice, at p 379, referring in some detail to these recourse agreements, as they are called, has this to say on
the point:

Since an indemnity in its normal form is not, in fact, an indemnity at all in the legal sense but a contract of guarantee,
it is governed by the usual principles applicable to contracts of guarantee.

As this was a contract of guarantee, that, as I think and as my lords have said, puts an end at once to the finance companys case.
For, once the finance company illegally took possession of this motor car, there was nothing to guarantee. Under s 11(2) of the
Hire-Purchase Act, 1938, the hirer was released from all liability under the agreement, and, had he so chosen, he could have
recovered any moneys that he had paid and any guarantor would be entitled to recover anything that he had paid. In my
judgment, it is implicit in the section that, on the discharge of the hirer by the illegal repossession by the owner, a guarantor too is
released from any further liability.
Even if the document were not a guarantee, I should still be of opinion that counsel for the dealers submission is correct for
a similar reason. The wording of the document is in these terms:

the dealer will at the request in writing of the company (a) pay to the company the total hire-purchase price and
other sums under the said hire-purchase agreement less the amount already paid to the company by the hirer.

In the events that happened, no hire-purchase price and no moneys fell to be paid to the finance company under the agreement.
Therefore, for the same reason, as I think, the finance company, having by their own act discharged the hirer from the agreement,
failed to show that any sums were payable to them by the dealer.
I agree, therefore, that this appeal succeeds.

Appeal allowed.

Solicitors: Simpson, Palmer & Winder agents for Reece, Davies & Cox, Birmingham (for the dealer); Margetts & Ritchie,
Birmingham (for the finance company).

F Guttman Esq Barrister.


274
[1963] 2 All ER 275

Brady and another v St Margarets Trust Ltd


CONSUMER; Consumer credit: CONTRACT

COURT OF APPEAL
LORD DENNING MR, DANCKWERTS AND DAVIES LJJ
13 MARCH 1963

Hire-Purchase Damages for breach of contract No repudiation of contract by hirer Hirers breach by not paying two
instalments and not keeping goods in repair Termination of agreement by owner Goods resold by owner Measure of
damages.

The plaintiffs hired from the defendants, a hire-purchase finance company, a second-hand Ford Zephyr motor car for which the
cash price was 515. The agreement, which was in the ordinary form, contained, amongst other clauses, an agreement by the
hirers to pay the monthly instalments and interest at ten per cent on any arrears, and to keep the motor car in good order, repair
and condition, the finance company having the right to retake possession if there was default in payment. The plaintiffs paid the
initial payment of 130. Thirty monthly instalments of 15 14s 5d were payable, of which they paid the first three instalments
after some pressure; but when two further instalments were overdue, the defendants retook the car and refused to accept payment
of the arrears. The defendants offered a new agreement, if there was deposit of 100, but the plaintiffs could not pay that. The
defendants subsequently sold the car for 125. A claim by the plaintiffs for damages for the defendants wrongful retaking of the
car was dismissed; but the defendants, on their counterclaim, were awarded (i) the arrears of instalments and damages for
repudiation of the hire-purchase agreement, together with (ii) damages for the failure of the plaintiffs to keep the car in good
order, repair and condition, on the ground that the defendants had acted reasonably and were fully justified, in the circumstances,
in retaking the car.

Held (i) The defendants were only entitled to the arrears of instalments, up to the date when they determined the agreement,
and interest at ten per cent, since there had not been a repudiation of the agreement by the plaintiffs but only a failure on their part
to pay the instalments (see p 277, letter e, and p 278, letters b and c, post).
Financings Ltd v Baldock ([1963] 1 All ER 443) applied.
(ii) The defendants were entitled also to damages for breach of the agreement to keep the car in good order, repair and
condition, the amount of which should be assessed on evidence of the condition of the car at the time when the agreement was
made and on how far the plaintiffs had defaulted in their duty to keep it in a reasonable condition (see p 277, letters h and i, and p
278, letters b and f, post).
Per Davies LJ: it would be most dangerous, in the case of a vehicle which was second-hand at the time of the hire-purchase
agreement, for the court, in assessing the damages for depreciation and damages for breach of contract to repair, to pay attention
either to the price which was agreed to be paid for the vehicle in the first instance or to the price that was obtained for the vehicle
after it had been retaken by the hire-purchase company (see p 278, letters f and g, post).
Appeal allowed.

Notes
As to the owners right to damages on determination or repudiation of a hire-purchase agreement, see 19 Halsburys Laws (3rd
Edn) 550, 551, para 891; and for cases on the subject, see 17 Digest (Repl) 154, 522, 523.

Cases referred to in judgments


Campbell Discount Co Ltd v Bridge [1961] 2 All ER 97, [1961] 1 QB 445, [1961] 2 WLR 596, revsd HL sub nom Bridge v
Campbell Discount Co Ltd [1962] 1 All ER 385, [1962] AC 600, [1962] 2 WLR 439, 3rd Digest Supp.
275
Financings Ltd v Baldock [1963] 1 All ER 443, [1963] 2 WLR 359.
Overstone Ltd v Shipway [1962] 2 All ER 52, [1962] 1 WLR 117.
Yeoman Credit Ltd v Waragowski [1961] 3 All ER 145, [1961] 1 WLR 1124, 3rd Digest Supp.

Appeal
This was an appeal by the second plaintiff, Mrs Jeretha Chambers, from an order of His Honour Judge Herbert QC dated 6
December 1962, and made at the Westminster County Court, dismissing the plaintiffs claim for damages for breach of a hire-
purchase agreement by the defendants (a finance company), and awarding the defendants on their counterclaim 243 0s 11d for
the plaintiffs breach of the agreement. The facts are set out in the judgment of Lord Denning MR.
The cases noted belowa were cited during the argument in addition to those referred to in the judgments.
________________________________________
a Smith v Baker [1891] AC 333, United Dominions Trust Ltd v Bycroft, [1945] 3 All ER 455

G E Moriarty for the second plaintiff.


J P Harris for the defendants.

13 March 1963. The following judgments were delivered.

LORD DENNING MR. On 1 June 1961, the defendants, a hire-purchase finance company, let out a Ford Zephyr car to two
hirers, Mr Brady and Mrs Chambers. The cash price was 515, of which the hirers paid 130 down, and there were to be paid
thirty monthly instalments of 15 14s 5d, the first instalment to be paid on 5 July 1961. The hire-purchase agreement was in the
ordinary form. The hirers agreed to pay the instalments and interest at ten per cent on any arrears; to keep the goods in good
order, repair and condition; if there was default in payment, the defendants were to have the right to retake possession; and there
was a minimum payment clause of two-thirds. I need not go into the rest of the agreement. The terms are very familiar. The
hirers were not good in making payments. They did not pay promptly even their first instalments. But, after a good deal of
pressure, on 29 September 1961, they paid all the instalments due up to that time; that is, the instalments due on 5 July, 5 August
and 5 September were paid. Then two months instalments fell into arrear; the 5 October and 5 November payments were not
paid, so that there was 31 8s 10d outstanding. On 9 November 1961, the defendants wrote a letter to each of the hirers, in which
they drew attention to the amount unpaid and went on:

Failing a remittance within the next five days the agreement will be terminated, and the vehicle collected. Legal
proceedings will then be commenced for the recovery of all moneys due under the agreement.

A remittance was not received. One of the hirers, Mrs Chambers, was away and did not get back until 22 November 1961, and
then, I think, on that very day, she sent the other hirer with some money to pay into the bank so that she could draw a cheque in
favour of the defendants. While he was away, the defendants representative came and collected the car. She offered to pay, but
he said that he had no authority to accept payment and he took the car. On the very next day, 23 November 1961, she went to see
them and offered to pay the arrears of 31 8s 10d and the cost of collection, which was I think 5 or 7. But the defendants felt
that they could not accept the payments. The only thing the defendants offered to do was to make a new agreement with a
deposit of 100 cash down, which the hirers could not pay. In those circumstances, the defendants retained the car and, after a
time, they sold it for 125.
Then the hirers brought this action against the defendants. They said that the defendants had wrongfully taken possession of
the car. That claim is not insisted on now in view of the evidence which was given at the trial. But there was a counterclaim by
the defendants. They counterclaimed for the instalments in arrear; for failure to keep in repair; and for the two-thirds under the
minimum payment clause. When the case came on for hearing in the county court, the 276 defendants (no doubt because of
Campbell Discount Co Ltd v Bridge) withdrew their counterclaim on the minimum payment clause and, with the leave of the
judge, amended the counterclaim so as to claim damages at common law. The judge awarded them damages. He based himself
on Yeoman Credit Ltd v Waragowski and Overstone Ltd v Shipway, and held that, in the circumstances of this case, the defendants
were quite reasonable in retaking possession. He said:

I have, I think, to decide whether on the facts it was a reasonable step for the owners to take to re-possess the car and
put an end to the agreement. I do not find it easy to appreciate what matters are relevant in considering whether the
defendants were reasonable in exercising their undoubted legal rights of re-possession. True, the plaintiffs had for a short
time, from Sept. 29 to Oct. 5, been out of debt to the defendants. But looking at the history of delays and excuses, and the
final failure to pay any attention to three reminders, I am of the opinion that the defendants were fully justified in retaking
the car on Nov. 22.

He eventually gave judgment on the counterclaim for the sum of 243 0s 11d.
The judge gave his judgment before Financings Ltd v Baldock was decided. In view of that case, it is now plain that it is
erroneous to apply the test whether the owners acted reasonably or not. The question is whether there was a repudiation by the
hirer which was accepted by the hire-purchase company; or whether there was simply a failure on his part to pay any instalments,
on which they exercised their contractual right to determine the agreement. Looking at this case in the light of that decision, I
must say that I do not think that there was a repudiation. This was simply a case of a failure by the hirers to pay the instalments.
There were breaches of contract on their part; but, in all the circumstances, there was no evidence on which it could be said that
they repudiated it. So far from repudiating it, they were anxious to pay up the arrears and get the car back. The defendants,
therefore, cannot claim damages for repudiation, but only the arrears up to the date when they determined the hiring, with, of
course, interest at ten per cent It was said that this point was not taken before the county court judge. It could hardly have been
taken before him, seeing that Financings Ltd v Baldock had not then been decided. But the point was before him in this sense,
that there was an issue as to damages on the basis of what was said in Yeoman Credit Ltd v Waragowski and Overstone Ltd v
Shipway. Now that that has been shown to be not altogether accurate, it is, I think, open to the court to decide the case according
to the law as it has since been declared to be, seeing, of course, that all the evidence on the matter has been brought before the
court. And the law is that, if the owners determine the hiring and retake the car for non-payment of instalments, they can recover
for breaches up to the date of determination but not thereafter.
In addition to the arrears, I think that the defendants are entitled to damages for breach of contract to keep the car in good
order, repair and condition. There is some difficulty in our dealing with these damages because the evidence was not fully
explored in the county court. I think that it must go back for the sum to be assessed. But I would like to say that it is not right to
assess damages under this head on the basis suggested to us by counsel for the defendants, namely, that the initial price is to be
taken as evidence of the condition of the car. The price in these hire-purchase agreements is no guide to the condition of a car.
There should be evidence by the hire-purchase company to show the condition of the goods at the time when the agreement was
made and to show how far the hirer has defaulted under it. As I read this clause, the hirers duty is to keep the car in the
condition in which it might reasonably be expected to be if he had looked after it properly. He need not put it in a better
condition than it was 277 when he hired it. He need only keep it in the condition in which a reasonably minded hirer would keep
it. Thus, he would repair it if there was an accident, and he would do the immediate repairs in the course of running the car, but
no more. The hire-purchase company should give evidence of any default on his part in that duty. On that being done, the
damages for breach of contract to repair can be assessed, but there is no sufficient material before the court on which the figure
can be assessed here. I think that the case ought to go back to the county court.

DANCKWERTS LJ. I agree. It seems to me that the case is covered by the principle of Financings Ltd v Baldock and,
accordingly, the consequences follow which have been mentioned by Lord Denning MR. I also agree with the remarks which
have been made by my Lord on the subject of the liability of the hirer under the agreement to keep in repair.

DAVIES LJ. I also agree. But it is, perhaps, only fair to the learned judge that I should add a word or two, since it is fairly clear
that, in deciding this case, he directed himself in accordance with what he thought were the principles enunciated in Overstone
Ltd v Shipway, to which I was a party. Having had an opportunity of considering the judgment in that case again and the
observations of this court in the later case of Financings, Ltd v Baldock ([1963] 1 All ER at p 453), it is, I think, right that I
should put on some sort of white sheet and say that, in my view, although I am not for one moment suggesting that Overstone
Ltd v Shipway was wrongly decided on its facts, the observations made in the latter case about some remarks which I made in the
earlier case were fully justified. The earlier case ought to be considered in the light of what was said in Financings Ltd v Baldock
and the interpretation put on, at any rate, my observations by my Lord and Upjohn LJ and Diplock LJ. The judgment in
Financings Ltd v Baldock was not given until 18 December 1962, and Judge Herbert gave his judgment in the present case on 6
December 1962, so that he did not have an opportunity of seeing and reading the observations of this court in the later case.
The only other thing I wish to say, agreeing as I do entirely with what has been said by my Lord, is with regard to the
question of repairs. It seems to me that it would be most dangerous, in the case of a vehicle which was second-hand at the time
of the hire-purchase agreement, for the court, in assessing the damages for depreciation and damages for breach of contract to
repair, to pay attention either to the price which was agreed to be paid for the vehicle in the first instance or to the price that was
obtained for the vehicle after it had been retaken by the hire-purchase company. There ought to be evidence as to what its general
condition and value were at the inception of the contract, and similar evidence as to what its general condition and value were at
the termination of the contract.

Appeal allowed; case remitted for damages to be assessed.

Solicitors: Breeze, Benton & Co (for the second plaintiff); G A Fry & Co (for the defendants).

F Guttman Esq Barrister.


278
[1963] 2 All ER 279

Gold v Introductions Ltd


CIVIL PROCEDURE

COURT OF APPEAL
LORD DENNING MR, DONOVAN AND RUSSELL LJJ
19 MARCH 1963

County Court Costs Payment into court Successive payments in Acceptance by plaintiff after more than four days after
third payment in Whether plaintiff entitled to costs up to date of last payment in County Court Rules, 1936, Ord 11, r 7, r 9, r
11, Ord 47, r 1.

The plaintiff claimed 307 1s 3d commission from the defendants for sales of pigs. The defendants by their defence alleged that
commission only became payable after the buyers had paid for the pigs, and they made three successive payments into court
(totalling 83) as and when, according to their accounts, commission became due. The last payment in was made on 2 October
1962. On 29 November 1962, ie, after more than the four days provided for by CCR Ord 11, r 9(a), the plaintiff gave notice that
he accepted the aggregate amount paid into court in satisfaction of his claim. The county court judge refused to award the
plaintiff his costs up to the date of payment in on the ground that CCR Ord 11, r 11 a, did not enable that to be done where the
notice of acceptance was given after the four days.
________________________________________
a Order 11, r 11, is as follows: If a plaintiff fails to give notice of acceptance within the time limited by r 7 or r 9 of this order, he may give
notice of acceptance subsequently, but the money in court shall not be paid out without an order of the court, and the court may order the
plaintiff to pay any costs reasonably incurred by the defendant since the date of payment into court, including the costs of attending court to
obtain the order

Held The plaintiff was entitled to his costs up to the date of receipt of the last notice of payment into court because, on the true
construction of CCR Ord 11, r 11, the substance of r 7 and r 9 were imported into r 11, with the consequence that, on notice on
acceptance of money paid into court being given by the plaintiff after the time limited by r 7 and r 9 (viz, in this case more than
four days after the receipt of notice of payment in), r 9(c)(ii) became applicable and under it the plaintiff was entitled to lodge his
bills for costs incurred up to the time of receipt of notice of payment into court and was liable to be ordered by the court to pay all
costs subsequent to the date of payment into court (see p 280, letter i, and p 281, letters f and g, post).
Per Lord Denning MR and Danckwerts LJ: alternatively, if the true construction of CCR Ord 11, r 11, was not as stated
above, the present case was one to which CCR Ord 47, r 1 b applied, under which the court had an overriding discretion as to
costs, and, as the plaintiff could not know what amount was due for commission until the defendants told him for how many pigs
payment had been made, he should have his costs up to the date of the last payment in (see p 281, letters a, b and g, post).
________________________________________
b Order 47, r 1, so far as material, is set out at p 281, letter b, post

Notes
As to liability for costs on payment of part claim, see 9 Halsburys Laws (3rd Edn) 231, 232, para 526.
For CCR Ord 11, r 7, r 9, and r 11, see County Court Practice (1963), 349, 351, 352.

Appeal
This was an appeal by the plaintiff from that part of an order of His Honour Judge Herbert QC made on 5 February 1963, at
Westminster County Court, refusing to award costs to the plaintiff on the ground that he did not give notice to take out payments
made by the defendants into court within four days. The facts are set out in the judgment of Lord Denning MR.

David Calcutt for the plaintiff.


Peter Pain for the defendants.
279

19 March 1963. The following judgments were delivered.

LORD DENNING MR. On 7 August 1962, the plaintiff brought an action in the county court against the defendants claiming
commission. He claimed 307 1s 3d, and he based it on commission at 8 for every pig he sold to customers in Kenya and
Rhodesia. The defendants within eight days, namely, on 15 August 1962, paid in a sum of 59 1s 3d, together with the scale
costs of 5, and, moreover, they put in a defence in which they admitted the commission of 8 a pig was payable, but said that it
was only payable when the pigs themselves were paid for by the customers; and, therefore, that the amount claimed by the
plaintiff was not due. The money was not taken out of court by the plaintiff and the action went on. There were applications for
particulars of the pleadings on each side. As time went on, the defendants paid more money into court. On 19 September 1962,
they paid in another 16, and on 2 October they paid in another 8, so they paid into court 83 altogether, admitting liability. The
plaintiff did not take it out within the four days which is provided in Ord 11, r 9(a), of the County Court Rules, and the action
continued, in this sense, that particulars were delivered by each side. Then, on 29 November 1962, the plaintiff gave notice that
he accepted the sum of 83 paid into court in satisfaction of his claim, and, that having been done, the whole question is: To what
costs is the plaintiff entitled? We have been taken through the rules of the county court on the matter, and it is quite plain from
Ord 11, r 11, that the court can order, and has ordered, the plaintiff to pay the defendants costs from the time of payment in, that
is from 2 October 1962 (when they had paid in 83), onwards. But it is said that the plaintiff cannot recover any costs in the
matter; it is even suggested that he could not get the scale costs of 5, because it is said that the rules do not provide for it. The
county court judge has so held: he has held that

the effect of Ord. 11, r. 11, is to prevent money in court from being paid out without leave and to enable the court to
award costs to the defendant. I do not think it enables the court to award costs to the plaintiff where notice is not given in
four days.

The question is whether or not the judge is right in that matter. It would, indeed, be surprising (the plaintiff having brought
his action quite properly and having carried it on for quite a long time and a payment into court having eventually been made
which he accepts) that he should not get costs up to the time of payment in. I do not think that I need go through all the details of
the rules because it seems to me that, if one reads them through with r 11, the earlier r 7 and r 9 are, in effect, imported into r 11.
CCR Ord 11, r 11, should be read in this way:

If a plaintiff fails to give notice of acceptance within the time limited by r. 7 or r. 9 of this Order, he may give notice of
acceptance subsequently [and if he does give notice of acceptance subsequently, the provisions of r. 7 and r. 9 respectively
shall apply], but the money in court shall not be paid out without an order of the court and the court may order the plaintiff
to pay any costs reasonably incurred by the defendant since the date of payment into court, including the costs of attending
court to obtain the order.

If r 11 is read in that way, importing into it r 9, it becomes plain that, even though in this case the notice of acceptance was not
given until after the four days, nevertheless, everything thenceforward has to happen as it would have happened under r 9. The
registrar must send notice to the parties, and the action is stayed, subject to this, that, under r 9(c)(ii), the plaintiff may lodge for
taxation a bill of the costs incurred by him before the receipt of notice of payment into court. In other words, the plaintiff can
still get his costs up to the time of payment into court, but it is subject to the sanction that, when he is late and accepts after the
four days, he is liable to be ordered to pay all the costs of the defendant from the date of the payment in.
On that interpretation of the rules, which I think is the right interpretation, it seems to me quite plain that the plaintiff in this
case is entitled to his costs 280 up to the date of payment in which he accepted (that is, the last payment of 2 October), and that
the defendants should have their costs thereafter. But I would like to say that there is an alternative way of looking at the matter.
Even if my interpretation of Ord 11, r 11, is not correct, even if the words which I have read are not to be imported, then there
would be a situation where the rules do not specifically provide what is to be done about costs, and CCR Ord 47, r 1, would
apply. That rule says:

Subject to the provisions of any Act or rule. the costs of proceedings in a county court shall be in the discretion of the
court

If that alternative approach is right, then it is a matter of discretion for the court; and it seems to me that, as a matter of discretion,
the same order should be made. The plaintiff himself could not know what amount was due for commission on these pigs until
the defendants themselves, who would have the knowledge, told him which of the pigs had been duly paid for. He was not in a
position to know how many had been paid for until the particulars which were delivered on 6 October 1962. If it is a matter of
discretion, I think that the ordinary rule should apply, that the plaintiff should have his costs up to the date of the payment in
which was accepted, and the defendants should have their costs thereafter.
I would, therefore, allow this appeal. The order of the judge should be altered so as to enable the plaintiff to have his costs
up to the date of payment in.

DONOVAN LJ. I agree. CCR Ord 11, r 11, is one of a group of rules dealing with payments into court in county court actions,
and the costs which shall be paid or may be claimed in such cases where the plaintiff accepts the amount paid in in satisfaction of
his claim. It would, I think, be wrong in these circumstances to construe r 11 in isolation so as to deprive a plaintiff of all his
costs if he did not observe the time limits in r 7 and r 9. I think that the better construction is to construe r 11 as in the nature of a
proviso to r 7 and r 9, leaving the plaintiff whatever rights to costs those rules give him, but imposing on him these new
conditions: first, that the money in court is not to be paid out without an order of the court, and, second, that the plaintiff may be
ordered to pay the costs reasonably incurred by the defendant after the time of payment in. Subject to those conditions, r 7 and r
9 will apply, with the result in the present case that the plaintiff may lodge under r 9 his bill of costs, as that rule allows.
If this view is wrong, I agree with what Lord Denning MR has said about an overriding discretion in the court.

RUSSELL LJ. The use of the word but shows that Ord 11, r 11, is to be read as if it were an annexure to r 7 and r 9
respectivelyan annexure in this form, that, if the notice of acceptance is given after four days, the difference shall be, first, that
there shall not be payment out without order, and, second, that the court may award to the defendant his costs after payment in,
including his costs of attending for the order to pay out. This seems to me not only a sensible but also a permissible construction;
otherwise r 11 contains no provision, for example, for a stay of proceedings on a notice of acceptance being given late, nor for
any method of giving notice of acceptance. It follows that the plaintiff is entitled to his taxed costs under r 9(c)(ii), the payment
in here being necessarily treated as at the date of the last of the sums paid in.
I prefer for my own part to say nothing as to the alternative claim under Ord 47, which does not arise.

Appeal allowed.

Solicitors: Tarlo, Lyons & Co (for the plaintiff); Montague & Co (for the defendants).

F Guttman Esq Barrister.


281
[1963] 2 All ER 282

Cooper and another v Williams and another


CIVIL PROCEDURE

COURT OF APPEAL
LORD DENNING MR, DANCKWERTS AND DAVIES LJJ
25, 26 FEBRUARY 1963

Fatal Accident Writ Irregularities Representative capacity not shown Fatal Accidents Acts not mentioned Claim settled
No statement of claim delivered and no particulars under s 4 of Act of 1846 given Settlement approved by court Stay of
proceedings ordered Whether order nullity Fatal Accidents Act, 1846 (9 & 10 Vict c 93), s 3, s 4.

Practice Stay of proceedings Removal of stay Fatal accident Action commenced by one dependant in personal capacity
Consent order embodying settlement Application by another dependant to remove stay Discretion of court to order removal.

On 4 December 1959, W was killed in a road accident. He left a widow and an illegitimate infant daughter. On 1 March 1960, a
writ was issued on behalf of the infant daughter by her mother and next friend claiming damages for the death of W from injuries
received by the negligence of the defendants, their servants or agents. The writ did not show that the action was brought in a
representative capacity, the indorsementa did not so state and did not mention the Fatal Accidents Act, 1846. The particulars
required by s 4b of that Act were not delivered, the action being settled on 18 May 1960, before a statement of claim was
delivered. The widows solicitors were kept informed of the proceedings on behalf of the infant daughter, but they took no steps
to ensure that the widows claim should be included. The settlement was effected by a consent order, staying all further
proceedings in the action except for the purpose of enforcing the order. On 17 July 1961, the widow commenced an action on her
own behalf under the Fatal Accidents Acts, 1846 to 1959, but abandoned it, having regard to s 3 c of the Act of 1846. She applied
for leave to intervene in the infant daughters action and to set aside the consent order on the ground that it was a nullity. The
order was set aside by the judge in chambers on the ground that it was an irregularity amounting to a nullity, because it was a
breach of natural justice, and the widow was given leave to intervene. On appeal by the defendants,
________________________________________
a Cf Bowler v John Mowlem & Co Ltd ([1954] 3 All ER 556)
b Section 4 is set out at p 284, letter a, post
c Section 3, so far as material, is set out at p 283, letter i, post

Held (i) The irregularities in the indorsement on the writ, and the failure to deliver the particulars required by s 4 of the Fatal
Accidents Act, 1846, did not vitiate the subsequent proceedings, and, accordingly, the consent order was not a nullity (see p 286,
letter d, p 287, letter i, and p 288, letter d, post).
(ii) The court had jurisdiction to remove the stay imposed by the consent order, and (Davies LJ dubitante), in the
circumstances, the stay should be removed, the consent order should be set aside and the widow should be allowed to intervene in
the infants action (see p 287, letter d, p 288, letter a, and p 289, letter f, post).
Appeal dismissed.

Notes
As to setting aside a consent judgment, see 22 Halsburys Laws (3rd Edn) 792, para 1672; and for cases on the subject, see Digest
(Practice) 621, 622, 25782589.
As to persons for whose benefit an action may be brought under the Fatal Accidents Acts, see 28 Halsburys Laws (3rd Edn)
37, 38, para 32; and for cases on the subject, see 36 Digest (Repl) 209, 10981105.
For the Fatal Accidents Act, 1846, s 3, see 17 Halsburys Statutes (2nd Edn) 7.

Cases referred to in judgment


Avery v London and North Eastern Ry Co [1938] 2 All ER 592, [1938] AC 606, 107 LJKB 546, 159 LT 241, Digest Supp.
282
Bean v Flower (1895), 73 LT 371, 21 Digest (Repl) 223, 217.
Appeal
This was an appeal by the defendants, George Frederick Williams and British Road Services Ltd from an order of Megaw J, in
chambers, dated 6 June 1962, whereby he set aside the order of Mr District Registrar Cox, dated 19 April 1962, made at the
Birmingham District Registry, and ordered that the applicant widow, Barbara Watkins, be granted leave to intervene in the action
by the infant plaintiff, Sheila Anne Cooper, against the defendants, and that the infant plaintiff have leave to re-amend the
indorsement on the writ naming the applicant as dependant on behalf of whom the action had been brought.
The infant plaintiff, suing by her mother and next friend, Hilda Mary Thornton, issued on 1 March 1960, a writ bearing the
following indorsement

The plaintiffs claim is for damages for the death of Frederick Leslie Watkins from injuries received by the negligence
of the defendants their servants or agents.

The writ did not mention the Fatal Accidents Acts nor show that the plaintiff sued in a representative capacity. No statement of
claim was delivered, but, no action having been begun on behalf of the deceaseds (Mr Watkins) widow, the infants action was
settled by a consent order, made by Mr District Registrar Cox in Birmingham District Registry on 18 May 1960, approving terms
of settlement and staying all further proceedings except for the purpose of enforcing the order. The facts are set out at p 284,
letter e, post.
The cases noted belowd were cited during the argument in addition to those referred to in the judgments.
________________________________________
d Rice v Reed [1900] 1 QB 54, Kinch v Wallcott, [1929] All ER Rep 720, [1929] AC 482, Craig v Kanssen, [1943] 1 All ER 108, [1943] KB
256, Marsh v Marsh, [1945] AC 271, Hilton v Sutton Steam Laundry, [1945] 2 All ER 425, [1946] KB 65, Woolfenden v Woolfenden,
[1947] 2 All ER 653, [1948] P 27, Hill v Luton Corpn, [1951] 1 All ER 1028, [1951] 2 KB 387, Stebbings v Holst & Co Ltd [1953] 1 All ER
925, Green v Rozen, [1955] 2 All ER 797, Jeffrey v Kent County Council, [1958] 3 All ER 155

Bernard Caulfield QC and F Blennerhasset for the defendants.


William Stabb for the applicant widow.
P J M Kennedy for the infant plaintiff.

26 February 1963. The following judgments were delivered.

LORD DENNING MR. Mr. Frederick Leslie Watkins was fatally injured in a road accident on 4 December 1959. He was
driving a car. His wife, Mrs Barbara Watkins, who was with him in the car, was herself injured. So she clearly had a claim for
her own personal injuries. In addition, she was dependent on her husband and had a claim for the loss which she had suffered by
his death. In addition to the widow, Mr Watkins left an illegitimate child, who was also dependent on him. She was his child by
another woman. Her name was Sheila Anne Cooper and she was almost ten years old. As a result of the death of Mr Watkins,
which was clearly caused by the negligence of the drivers of two lorries, a claim arose under the Fatal Accidents Acts, 1846 to
1959, for and on behalf of the two dependants. Under the Act of 1846, as originally enacted, the right of action for the death was
vested in the executors or administrators of the deceased person. Section 2 provided:

Every such action shall be for the benefit of the wife, husband, parent, and child of the person whose death shall have
been so caused, and shall be brought by and in the name of the executor or administrator of the person deceased; and in
every such action the jury may give such damages as they may think proportioned to the injury resulting from such death to
the parties respectively for whom and for whose benefit such action shall be brought

Section 3 is very important. It read:

that not more than one action shall lie for and in respect of the same subject-matter of complaint
283

Section 4 provided:

In every such action the plaintiff on the record shall be required, together with the declaration, to deliver to the
defendant or his attorney a full particular of the person or persons for whom and on whose behalf such action shall be
brought, and of the nature of the claim in respect of which damages shall be sought to be recovered.

Such was the Act of 1846. But it was incomplete. There was not always an executor or administrator, or he did not bring an
action. So, in 1864, the Act was amended by the Fatal Accidents Act, 1864, of which s 1 provides:

If and so often as it shall happen that there shall be no executor or administrator of the person deceased, or that
there being such executor or administrator no such action shall within six calendar months after the death of such
deceased person have been brought such action may be brought by and in the name or names of all or any of the
persons (if more than one) for whose benefit such action would have been, if it had been brought by and in the name of
such executor or administrator; and every action so to be brought shall be for the benefit of the same person or persons, and
shall be subject to the same regulations and procedure, as nearly as may be, as if it were brought by and in the name of such
executor or administrator.

So it is quite plain under the Act of 1864 that it is not necessary for the executor or administrator himself to bring the action; but,
if he has not done so, any one of the dependants may do so.
Such being the statutory provisions, I turn to what happened in this case. The death of Mr Watkins was, as I have said, on 4
December 1959. The applicant widow went to a firm of solicitors in Lichfield. The plaintiff infant and her mother went to a firm
in Stafford. The widows solicitors took steps to get a grant of administration on her behalf, but it took a little time and they did
not get the grant until 11 March 1960. Meanwhile, however, the infants solicitors were very anxious to press on with the claim
on behalf of the infant. They did not wait for the widow to take proceedings as administratrix but issued a writ on behalf of the
infant by her mother and next friend. They issued the writ on 1 March 1960, before the widow obtained the grant. The plaintiff
was Sheila Anne Cooper spinster an infant (suing by her mother and next friend) . The defendants were Mr Williams, a
lorry driver, and British Road Services Ltd. The indorsement was

The plaintiffs claim is for damages for the death of Frederick Leslie Watkins from injuries received by the negligence
of the defendants their servants or agents.
That writ was issued, no doubt, with a view to early settlement of the infants claim. They seem to have arranged to settle it for
450. It would need the approval of the court. But there was some delay in making the application. The reason was given in a
letter of the infants solicitors to the widows solicitors of 29 March 1960. They said that

there has been a certain amount of subsequent delay because the other solicitors raised one or two points as to the
procedure under the Fatal Accidents Acts which have now been sorted out We are keeping you informed in view of the
provisions of s. 3 of the Fatal Accidents Act, 1846, as you might like to consider whether your client should join in the
application So that we can place the whole of the facts before the court perhaps you would let us know the name of
your client and also the names of any children.

One would have thought that the widows solicitors would thereupon have looked up s 3 and have discovered that there could be
only one action under the Fatal Accidents Act, 1846, and would, therefore, have seen to it that a claim was included on behalf of
the widow. But they did not do so. They replied on 31 March 2841960: Our clients name is Barbara Watkins. There were no
children of the marriage. On 1 April 1960, the infants solicitors replied saying that they were obliged, and added simply, We
can now give the court full information as to other dependents. On 6 May 1960, the infants solicitors wrote to the widows
solicitors telling them that they were applying on 18 May 1960, to the registrar for his approval to the settlement of our clients
claim under the Fatal Accidents Acts. Even then the widows solicitors did not realise that the widows claim ought to be
included. They replied on 7 May 1960, saying:

If the registrar mentions the question of any other claims in this case, we should be very much obliged if you will
inform him that Mrs. Watkins is still receiving attention in respect of her facial injuries and therefore it is not yet possible to
assess her claim.

It would seem that the widows solicitors still did not realise that the claim for dependency ought to be included in the infants
action. The infants solicitors replied, therefore, on 9 May 1960:

You will of course appreciate that we wrote to you in view of the position with regard to more than one claim being
made under the Fatal Accidents Act.

Still the widows solicitors did nothing. On 18 May 1960, the application came before the District Registrar at Birmingham. No
statement of claim had been delivered. There was, therefore, no particular as required by s 4 of the Act of 1846 of the person
or persons for whom and on whose behalf the action was brought or of the nature of the claim. But an affidavit was filed by the
infants solicitor, in which he said that The deceased left a widow and No action has been commenced by or on behalf of the
widow.
Such was the state of the proceedings on 18 May 1960, when the infants solicitors and the defendants solicitors appeared
before the registrar at Birmingham and sought approval to settle the infants claim for 450. Everyone present knew of the
existence of the widow. She was named in the affidavit, and we were told that her name was called to see if she would appear.
Nevertheless, the settlement was approved without any provision being made for her. The order approved the terms of settlement
and went on to order:

That all further proceedings in this action be stayed (except for the purpose of enforcing this order) upon the terms
following: (i) that the defendants do pay the mother and next friend of the plaintiff the sum of 36. (ii) That the
defendants do pay the further sum of 414 to be invested for the benefit of the infant plaintiff (iii) That
the defendants do pay to the infant plaintiff her costs of this action (iv) That upon payment by the defendants
they be discharged from any further liability in respect of the plaintiffs claim in this action. And that the parties may be at
liberty to apply.

Now that action was one action within s 3 of the Act of 1846. It was brought on behalf of the infant. The widows claim was
not included at all. The consent order was made on 18 May 1960, staying the action. On the very next day, 19 May 1960, the
widows solicitors wrote to the defendants solicitors about her claim for her facial injuries. They still did not mention her claim
for dependency. But the defendants solicitors in their reply made it clear that, if she should make any claim for dependency, they
would contend that it was excluded by the settlement. They replied by return of post on 20 May 1960:

You are no doubt aware that proceedings have already been issued and disposed of in respect to claims under the Fatal
Accidents Acts;

and when at length the widows solicitors did make a claim under the Fatal Accidents Acts, they said explicitly on 18 October
1960, that liability would not be accepted for the claim, having regard to s 3 of the Fatal Accidents Act, 1846.
285
On 17 July 1961, the widow, as administratrix, herself brought an action under the Fatal Accidents Acts, 1846 to 1959, on
her own behalf, but she was met by the defence that only one action lay in respect of the death of Mr Watkins. So she has not
pursued that action, but has applied for leave to intervene in the infants action and to set aside the consent order. She wishes to
be added as a plaintiff in the infants action, or at least to be named as one of the persons for whose benefit the action is brought.
In support of this application, counsel for the widow urged that the consent order was a nullity. He did not suggest that the writ
itself was a nullity. True it is that the widow was not included in the claim, but that is not a ground for saying it is a nullity. It is
quite plain from Avery v London and North Eastern Ry Co, that one dependant can bring an action by herself for her own benefit
alone, at any rate when the others do not want to come in; and it is easy to envisage cases where one dependant claims alone in
complete ignorance of the existence of any other dependants. It would be most unfortunate if it were held that, because one
dependant was overlooked, the whole action was a nullity, for, if a new action had to be brought, it might be barred by the
Limitation Act, 1939. Counsel for the widow relied, therefore, on the failure to deliver the particulars required by s 4 of the Act
of 1846. This failure was so serious an irregularity, he said, that it vitiated the subsequent proceedings. He also relied on the
irregularities in the indorsement of the writ. The indorsement did not say, as it should under RSC, Ord 3, r 4, that it was brought
in a representative capacity. It did not even mention the Fatal Accidents Act, 1846. I cannot myself regard these irregularities as
sufficient to vitiate the subsequent proceedings. They could all have been cured by a statement of claim which set out the
statutory particulars. An irregularity which can be cured is never a nullity and, if the writ with its indorsement was not a nullity, I
do not see that the consent order can be.
But that is not the end of the matter. There is still the question whether it is possible to remove the stay and set aside the
consent order, for if this were done, there could be nothing to prevent the widow from intervening. For some time the effect of a
stay has been a matter of doubt. In the Annual Practice, 1963, at p 3182, it says, under the heading Effect of a Stay of
Proceedings:

Two views may be taken: first, that it is a discontinuance, and therefore cannot be removed; secondly, that it is not
equivalent to a discontinuance, but may be removed if proper grounds are shown.
The point was left open in the case to which we were referred of Bean v Flower. Of the two views, I am of the opinion that the
effect of a stay is that it is not equivalent to a discontinuance, or to a judgment for the plaintiff or the defendants. It is a stay
which can be, and may be, removed if proper grounds are shown.
So in this case the question, it seems to me, ultimately is this: Are there proper grounds for removing this stay so as to
enable the widow to come in and be made a plaintiff in the action, or at all events a person, or one of the persons, for whose
benefit the action is brought? On this point some things appear plain. First, the widows solicitors made a mistake. They did not
realise that, if she were to make any claim of dependency, she ought to be named in the action as one of the persons for whom it
was brought. Unfortunately, they did not insist on that being done. They seem to have thought that she could pursue her claim
for dependency separately from the infants action. In that they were clearly wrong. Secondly, the infants solicitors made a
mistake. I am satisfied that, if one of the dependants brings an action under the Fatal Accidents Acts (which is the only action
that can be brought), it is the duty of that person (just as it is clearly the duty of an executor or administrator) to take all
reasonable steps to see that all those dependents of the deceased person who desire to claim for their loss are informed of the
action and named as persons on whose behalf it is brought. I realise that, in this case, the infants solicitors wrote letters which
286 should have put the widows solicitors on inquiry. But, even so, more could have been done to make sure that she was
brought into the action. Lastly, what about the defendants solicitors? I cannot think that they were under any mistake. They
were well aware that Mr Watkins left a widow, and that the effect of the consent order might be to bar any claim she might have
for dependency; for, within two days after the order, they wrote to her solicitors telling them that the issue had been disposed of.
The defendants were the people who stood to benefit by the mistakes of others. I do not say that it was their duty to see that the
widow was brought into the action, but I should have thought that they were taking a risk in taking a consent order without her
presence.
In these circumstances, is there proper ground for the court to remove the stay? I think that there is; and for much the same
reasons as influenced Megaw J, when he held, or at any rate indicated, that the consent order was a nullity. He plainly thought
that the consent orderand the stayought not to be allowed to stand, because it would

lead here in all the circumstances to what would be a breach of natural justice if the order were allowed to stand,
whether or not the widow would have a claim against the infant or against her advisers, who, knowing that the order was
going to be asked for, took no steps to preserve her interests.

I would not agree that the breach of natural justice makes it a nullity, but I do think that it is proper ground on which the court can
and should set aside the order of the district registrar and remove the stay and allow the widow to intervene. The learned judge
made an order to that effect. He allowed the widow to intervene, but protected the infants interests. I see no reason to interfere
with his order. I would dismiss the appeal.

DANCKWERTS LJ. I am of the same opinion. The action took a peculiar course. In the first place it was started very quickly,
and before the applicant widow had a chance to obtain letters of administration, which she was proceeding, through her solicitors,
to obtain. The action never proceeded as far as statement of claim. If it had proceeded as far as that, I think it likely that,
perhaps, somebody would have looked at the Fatal Accidents Acts, 1846 to 1959, and discovered that the form of the action was
not that which was required by the statute in certain respects, and would have seen that particulars had to be given in accordance
with s 4 of the Act of 1846. What happened instead was that a short cut was taken to give effect to the settlement which had been
reached between the defendants and their insurance company and the persons who were advising the plaintiff infant and her next
friend. Like many short cuts, it seems to me that this turned out to be the longest way round. When it came before the learned
registrar, it appears that everybody knew that there was a widow, and I am rather surprised that the proceedings were not
adjourned to give the widow a chance, through her advisers, to state her attitude; but they were not. The registrar thought fit to
make the order. I think that it is probable that none of the persons concerned, the widows solicitors or the infants solicitors, or
the learned registrar, appreciated that the effect of making an order staying all further proceedings in the action on terms might
result in the possibility of the widow, putting forward her claim, being defeated; but that was done. An order was made in what is
known as the Tomlin forme staying all further proceedings on the terms agreed, subject to the possibility of proceedings being
taken to enforce the order.
________________________________________
e Cf Practice Note, [1927] WN 290

The proceedings were not, in my opinion, a nullity, but it is obvious that they were irregular in several respects. I am quite
satisfied that a stay of this kind is not equivalent to the position when a judgment has been given; but it is a process which can be
removed for good cause. In my experience, stays have often been removed when the facts required that step to be taken, and I
287 would point out that it is also possible to set aside a consent order in proper conditions. If the stay is once removed, it seems
to me that the deficiencies in the action which amount to irregularities, but which do not make the action a nullity, can be
corrected by amendment and by the delivery of a statement of claim and the particulars required by the Act. The question is
whether the stay should be removed, and whether leave to make these amendments should be given, and, on the whole, I think
that this is a case in which the discretion to do so should be exercised in favour of the widow. The result of that is that the action
will continue, and I would like to hear any views which may be expressed by counsel in this respect, because it seems to me that
the effect of Megaw Js order and the effect of the removal of the stay and the leave to amend, which purports to give the widow
leave to intervene, means that the action continues by the infant plaintiff. I am not quite sure in what way the widow intervenes,
except, I suppose, that she obtains an order in continuation of the proceedings so that her claim is kept alive in the action in the
representative capacity, but that, perhaps, requires some further discussion. I am in favour of the appeal being dismissed, for the
reasons which I have stated.

DAVIES LJ. I agree that neither the indorsement on the writ nor the order made in this casethe consent order staying these
proceedingswas a nullity. I also agree that the court has power if it thinks fit for good cause to remove a stay. The only matter
in which I have had some hesitation, as opposed to the views taken by my Lords, is the question whether, in all the circumstances
of this case, the court ought to exercise its power to remove the stay and allow the action to continue.
It is perfectly plain that, in the correspondence prior to the order of the district registrar, the solicitors acting for the plaintiff
infant were keeping in close contact with the solicitors instructed by the applicant widow, and were apprising them by and large
of what was going on. For example, after earlier correspondence, on 22 February 1960, the infants solicitors suggested that the
proceedings should be taken in the name of the administratrix, though they were only contemplating then a claim on behalf of the
infant. To that they had the reply that the grant had not yet been made, and the widows solicitors added: We note your remarks
as to your proceedings and shall be glad to co-operate as far as we can. Then, on 29 March 1960, the infants solicitors informed
the widows solicitors that a writ had been issued on 1 March and that there had been points raised about the procedure under the
Fatal Accidents Acts, and they continue with these words:

We are keeping you informed in view of the provisions of s. 3 of the Fatal Accidents Act, 1846, as you might like to
consider whether your client should join in the application.
The word application may be not altogether appropriate, since that was the application which was going to be made for the
approval of the settlement. Then they ask, in the postscript, the names of the widow and of any children. On 6 May 1960, the
infants solicitors again informed the widows solicitors that, on 18 May they were going to apply to the registrar for his
approval to the settlement of our clients claim under the Fatal Accidents Acts. Finally, so far as concerns the letters, on 9 May
1960, still nine days before the matter came before the registrar, the infants solicitors write:

You will of course appreciate that we wrote to you in view of the position with regard to more than one claim being
made under the Fatal Accidents Act. You will no doubt give the position your consideration from that point of view.

The widows solicitors did nothing at all. And it is to be observed that, at the hearing before the district registrar, an affidavit by
the solicitor acting for the infant was put in in which the existence of the widow, the fact that no action 288 had been started by
her or on her behalf and the fact that she had no children were specifically brought to the attention of the learned registrar; and,
indeed, it appears that the solicitors suggested to the registrar that he might like to adjourn the proceedings in order to give the
widow an opportunity, if she was so advised, of intervening or taking some steps in the matter. The registrar did not adjourn it,
but made the order. As I understand the statute, the infant was not obliged to claim on behalf of the widow. That is implicit in
the decision in Avery v London and North Eastern Ry Co.
What, in the circumstances, ought the court to do? The matters which have been weighing on my mind are, first, the fact
that the widows solicitors, whether through negligence or whether through ignoranceprobably the latterwere taking no steps
at all at that stage to safeguard the position of their client. Indeed, as late as October, 1960, when they had been in
correspondence with the insurance companys solicitors and their attention had been directed to the Fatal Accidents Act, 1846,
they were still, as appears from the letter of 14 October 1960, under the impression that the name of the Act was the Fatal Injuries
Act, which does not suggest that they paid any very careful attention to the provisions of the Act or to its title, even as late as that.
The widow, if the court refused to reopen this matter in her favour, might, I apprehendand this, of course, is not a matter for the
court to decidehave a clear case against her own solicitors. That was one of the matters that has been troubling me. The
second matter is this, that the particular ground on which this court is deciding to support the judge in his decision and to set
aside the registrars order does not, as I understand the notice of appeal, appear in the notice of appeal anywhere. What is more,
when the question arose in the course of the argument, counsel for the widow, possibly because he was keener on the submission
he was making as to the nullity of the proceedings and the order made by the registrar, did not appear to be particularly interested
in the submission which he was asked to make and rather ran away from it.
However, there it is. My lords have come to the conclusion that, in all the circumstances of this case, it is right that the
widow should be allowed to intervene and that the action should be allowed to go on. I do not hold such strong views on the
facts of this matter that I feel I ought to dissent from the order proposed, and I merely record the fact that, during the argument, I
have had the greatest doubts whether, in all the circumstances, the widow ought not to be left to pursue such remedy as she may
have against somebody other than the defendants in this action. I agree that the appeal should be dismissed.

Appeal dismissed. Leave to appeal to the House of Lords refused.

Solicitors: Barlow, Lyde & Gilbert agents for Duggan, Elton & James, Birmingham (for the defendants); Gascoin & Co agents
for Hinkley & Birch, Lichfield (for the applicant widow); Wallace, Copland & Co, Stafford (for the infant plaintiff).

F Guttman Esq Barrister.


289
[1963] 2 All ER 290

The Sitala
Owners of Motor Vessel Niceto de Larrinaga v Owners of Steam Tanker
Sitala
SHIPPING

PROBATE, DIVORCE AND ADMIRALTY DIVISION


HEWSON J ASSISTED BY CAPT G P MCCRAITH AND CAPT D A G DICKENS, TRINITY MASTERS
12, 13, 14 FEBRUARY 1963

Shipping Collision Radar Fog Ascertainment of position of vessel Immoderate speed Necessity to use radar properly
with seamanlike prudence Collision Regulations, 1948, (SI 1953 No 1557, Sch 1), r 16 (b).

To rely on ascertainment of position as an excuse for not stopping engines in fog, as otherwise required by r 16(b) a of the
Collision Regulations, 1948, imposes on a vessel a duty to ascertain the other vessels course and speed and probable closest
position of approach, and to observe these things with accuracy; special care must be taken in the application of the provisions of
r 16(b) to vessels using radar when they are approaching, or are within an area of intersection of, shipping routes where
alterations of courses of a diversity of shipping to widely separated destinations may be possible, if not probable (see p 292, letter
e, post).
________________________________________
a Rule 16(b) is set out at p 292, letter b, post

If the extended and accurate look-out which is provided by radar is relied on to justify immoderate speed, care must be taken
to see that radar is used properly and with seamanlike prudence on the indications and inferences which are given by it, or may be
drawn from the data supplied by it (see p 292, letter i, to p 293, letter a, post).

Notes
Rule 16(b) of the International Regulations for Preventing Collisions at Sea, 1948, is contained in Sch 1 to the Collision
Regulations (Ships and Seaplanes on the Water) and Signals of Distress (Ships) Order, 1953 (SI 1953 No 1557); 20 Halsburys
Statutory Instruments (1st Re-ssue) 118.
As to when the position of a vessel is ascertained under the Collision Regulations, see 35 Halsburys Laws (3rd Edn) 633,
para 946; as to the requirement of moderate speed in fog, see ibid, p 627, paras 936, 937; and as to the use of radar, see ibid, p
631, para 942.

Case referred to in judgment


Gunnar Knudsen, The [1962] 1 All ER 315, [1962] P 151, [1962] 2 WLR 517.

Action
This was a collision action for damages brought by the plaintiffs, the owners of the motor vessel Niceto de Larrinaga, against the
defendants, the owners of the steam tanker Sitala. The following facts are taken from the judgment. The Niceto de Larrinaga
was a single screw motor vessel of 8,869 tons gross, 491 feet in length and sixty-six feet in beam. She was fitted with a diesel
engine of about 7,800 horse power. At the material time she was laden with a cargo of about ten thousand tons of iron ore and
was drawing twenty-nine feet forward and twenty-nine feet three inches aft. She was under the British flag, and was on a voyage
from Freetown to Ijmuiden. The Sitala was a single screw tanker of over forty-nine thousand tons gross, 851 feet in length and
117 feet in beam. She was fitted with steam turbines of twenty-four thousand horse power. She was in ballast, drawing about
twenty-six feet one inch forward and thirty feet six inches aft. Her displacement in that trim was about sixty thousand metric
tons. She was on a voyage from Rotterdam to Malta. The collision occurred on 23 September 1961, at about 1905 hours time on
board the Niceto de Larrinaga. The equivalent time on board the Sitala was 1932 1/2, Central European Time. The place of
collision was about nine miles six degrees (true) from the Casquets Lighthouse in the English Channel. There was no wind; there
was dense fog, and the tide at 290 the time of collision was flood, setting north-easterly, of a force of about two knots. According
to her statement of claim and other documents the Niceto de Larrinaga, on whose behalf no evidence was called, was on a course
of 062 degrees (true) and with her engines at full ahead was making about thirteen to 13 1/2 knots through the water. Single
prolonged blasts for fog were being sounded in accordance with the regulations, a look-out was posted forward and a careful
watch was being maintained on her radar. When the echo of a vessel, which proved to be the Sitala, was observed on the radar,
distant about five miles and bearing fine on the port bow, the course was altered to 066 degrees (true). She continued on this
course until about four minutes before the collision, when the Sitala was observed to be approaching rapidly, still fine on the port
bow, and her course was altered ten degrees to starboard, her engines were reduced to half speed ahead and a signal of one short
blast was sounded on the whistle. Shortly thereafter, she sounded a further signal of one short blast, which was repeated after a
short interval. About one minute later, her course was altered a further ten degrees to starboard. Two or three minutes before the
collision, when the Sitala appeared to be so close that collision was inevitable, her wheel was put hard-a-starboard. The Sitala
came on at speed and with her stem struck the port bow, the angle of blow at collision being about sixty-five degrees leading aft
on the Niceto de Larrinaga, which was heading about 120 degrees (true), the Sitala being on her initial course of 235 degrees
(true). The speed of the Niceto de Larringaga at the time of collision was about eleven to twelve knots. The Sitala was equipped
with three radar sets, one of which was a true motion memory radar set. She was on a course of 235 degrees (true) and her speed
was 13 1/2 knots through the water. She first became aware of the Niceto de Larrinaga about twenty-two minutes before the
collision, but her master came to the conclusion that the Niceto de Larrinaga was on a course about opposite and parallel to his.
About 4 1/2 minutes before the collision occurred, when the ships were 1 3/4 miles apart, the Sitala heard one long blast from the
Niceto de Larrinaga. She did not stop her engines and she kept her course and speed because her master said that he considered
the position of the Niceto de Larrinaga to be ascertained. Both he and the fourth officer, who was on the bridge with him, relied
solely on an assessment of the positions of the two vessels by looking into the radar, and, by such observations, they concluded
that the Niceto de Larrinaga was on an opposite and parallel course. His Lordship found that at no time from first observing the
echo in the radar were the two ships on opposite and parallel courses. At 8 1/2 miles they were in fact converging at an angle of
about seven degrees and no subsequent observations by either the master or the fourth officer of the Sitala indicated such a
convergence, or that, if the vessels kept on their original courses, they would in fact pass each other at a distance much closer
than one mile. About 2 1/2 minutes before the collision, the Sitala observed on her radar the Niceto de Larrinaga turn hard-a-
starboard. About 1 1/2 minutes before the collision, half speed astern was ordered on the Sitala, followed by full speed astern at
about the same time as the collision. At the hearing, the plaintiffs, who had originally contended that the collision was caused
solely by the improper navigation of the Sitala, by leave amended their statement of claim to allege that both vessels were equally
to blame, and they claimed judgment against the defendants and their bail for a moiety of the damage sustained. Counsel for the
plaintiffs admitted that the speed of the Niceto de Larrinaga was not moderate in fog, that the proper steps were not taken to
reduce speed, that the handling of the radar watch was unsatisfactory, and that to make a small series of alterations of course to
starboard was improper when the other vessels approach was known by radar.
The cases noted belowb were cited during argument in addition to that referred to in the judgment.
291
________________________________________
b The Peter Benoit (1915), 85 LJP 12, 13 Asp MLC 203, The Billings Victory, (1949), 82 Ll L Rep 877, The Nora, [1956] 1 Lloyds Rep 617,
The Miguel de Larrinaga, [1956] 2 Lloyds Rep 530, The Dea Mazzella, [1958] 1 Lloyds Rep 10, The Verena, [1960] 2 Lloyds Rep 286,
The Kurt Alt, [1962] 2 All ER 27

Roland Adams QC and Barry Sheen for the plaintiffs, the owners of the Niceto de Larrinaga.
H V Brandon QC and A Stewart-Richardson for the defendants, the owners of the Sitala.

14 February 1963. The following judgment was delivered.

HEWSON J stated the facts, and continued. As the master of the Sitala has made the point that he had ascertained the position
of the Niceto de Larrinaga, I must say something about it. It was obvious that what he had in mind was r 16(b) of the Collision
Regulations, 1948, which reads as follows:

A power-driven vessel hearing, apparently forward of her beam, the fogsignal of a vessel the position of which is not
ascertained, shall, so far as the circumstances of the case admit, stop her engines, and then navigate with caution until
danger of collision is over.

I dealt with this question of ascertainment a year or two ago in The Gunnar Knudsen ([1962] 1 All ER 315 at p 319; [1962] P 151
at p 156). It will be sufficient if I read a short passage from the report:
As I have indicated, it needs much more than a distance and bearing at a particular time. The minimum amount of
knowledge required is that one must know the other vessels course and speed and, it may even be, her probable future
course and speed. It is not necessary for me to decide that in this case.

To rely on ascertainment of position as an excuse for not stopping the engines in these circumstances in my view imposes on
a vessel a duty to ascertain the others course and speed and probable closest position of approach, and to observe these things
with accuracy. It seems to me that especial care must be taken in the application of the provisions of r 16(b) to vessels using
radar when they are approaching, or are within an area of intersection of, shipping routes such as the Casquets, where alterations
of courses of a diversity of shipping to widely separated destinations may be possible, if not probable. It must be remembered
that radar does not foretell intention, and that all these are circumstances which must be considered. These requirements and
circumstances, as I see it, were never heeded by those on board the Sitala and, in my view, she cannot rely on the provisions of r
16(b). A close quarter situation was arising, and she continued at full speed after hearing one long blast for another two minutes,
at the end of which period she stopped her engines, not apparently because of some belated compliance with r 16(b), but because
she heard the one short blast signal from the Niceto de Larrinaga. She was never aware that the Niceto de Larrinaga altered ten
degrees to starboard on two occasions about four and three minutes before the collision, respectively, and that may well have
been because the master, who was relying on his memory radar, did not keep himself sufficiently frequently informed. It must be
remembered, too, that he was ignorant whether or not the Niceto de Larrinaga was equipped with, and using, radar. It is quite
obvious that the master of the Sitala was relying on his radar to justify his speed.
I think that I have said enough to indicate that, in my view, the Sitala was proceeding at a grossly excessive speed, as, of
course, was the Niceto de Larrinaga. It seems to me that the Sitala was at fault for not stopping her engines when she heard the
long blast, but I go further and say that she was at fault for not stopping her engines earlier than that moment. Proper radar
observations ought to have indicated that the vessels would pass much closer than one mile and, being informed by radar of the
rapid approach of the Niceto de Larrinaga, the Sitala should have reduced her way to a moderate speed in these conditions of
dense fog. High speeds at collision cause much greater damage than low speeds. High speeds before collision give less time to
appreciate properly the development of the situation. Therefore, if radar is relied on it must be properly used. If reliance is place
upon the extended and accurate look-out which is provided 292 by radar to justify immoderate speed, care must be taken to see
that one uses the radar properly and with seamanlike prudence on the indications and inferences which are given by it, or may be
drawn from the data supplied by it.
Counsel for the plaintiffs submits that this case is dominated by the grossly excessive speeds of both vessels. He says that
this is a circumstance which completely overshadows all the other incidents, and that it is not a case where niceties should be
considered in assessing the proportion of blame. Well, really, no case is. Counsel admits that the starboard wheel action on board
the Niceto de Larrinaga was misguided. That, in my view, seems to me somewhat of an understatement. An alternation of course
in fog without full and reliable information of the heading and speed of the other vessel has been heartily condemned in these
courts on numerous occasions, and I see no reason in this case to withhold a similar condemnation. Had intelligent use been
made of the radar, I venture to think that such starboarding as was done by the Niceto de Larrinaga on this occasion would never
have been attempted. It must be admitted that excessive speed holds its dominating position in this case. The excessive speed of
both these two large and heavy vessels approaching at twenty-six to twenty-seven knots until shortly before the collision greatly
increased the damage. It also greatly abbreviated the period of time available for reflection and repentance. Nevertheless, the
starboarding indulged in by the Niceto de Larrinaga brought the vessels into contact, while the speed of the two brought them
into violent contact.
I have considered the provisions of s 1(1) of the Maritime Conventions Act, 1911, c and I have them well in mind, and I must
take account of all the circumstances of the case, including all circumstances which contribute to the collision and the damage. I
must consider the excessive speeds of both, the speed of each at the collision, and I must consider what, in my view, was a wholly
unjustified alteration of course to starboard on the part of the Niceto de Larrinaga, which, in my view, is another principal
contributory cause of the collision and damage, and that is a circumstance which cannot lightly be brushed aside. After
considering all these matters, I have come to the conclusion that there is no doubt that some differentiation must be made in this
case in the degrees of blame and, after doing the best that I can in all the circumstances, I find the Niceto de Larrinaga sixty per
cent to blame and the Sitala forty per cent.
________________________________________
c Section 1(1), so far as material, provides: Where by the fault of two or more vessels, damage or loss is caused to one or more of those
vessels the liability to make good the damage or loss shall be in proportion to the degree in which each vessel was in fault: Provided
that(a), if, having regard to all the circumstances of the case, it is not possible to establish different degrees of fault, the liability shall be
apportioned equally; and (b) nothing in this section shall operate so as to render any vessel liable for any loss or damage to which her fault
has not contributed

Judgment accordingly.

Solicitors: Corner & Co agents for Lightbounds & Co, Liverpool (for the plaintiffs); Bentleys, Stokes & Lowless (for the
defendants).

N P Metcalfe Esq Barrister.


293
[1963] 2 All ER 294

R v Harris
R v Turner
LEISURE AND LICENSING: CRIMINAL; Criminal Law

COURT OF CRIMINAL APPEAL


LORD PARKER CJ, ASHWORTH AND WINN JJ
4, 5 MARCH 1963

Gaming Fraud in wagering Betting transactions with bookmaker Bookmaker induced to accept bets by false pretence as to
identity No winnings paid out by bookmaker Whether an offence under Gaming Act, 1845(8 & 9 Vict c 109), s 17.
It is an essential ingredient in an offence charged under s 17 of the Gaming Act, 1845 a, that the money won should have been
obtained (see p 296, letter g, post).
________________________________________
a Section 17, so far as material, is set out at p 296, letter a, post

H had telephoned a bookmakers office, pretending to be a Sgt H from barracks in the neighbourhood and stating that he had
collected a few bets from the boys. He asked to be put on a commission basis, telling the bookmaker that he had been
recommended by a friend, and proceeded to place bets. Two of the horses won, the winnings amounting to about 79. The
bookmaker did not pay. H was charged, together with another man, on an indictment containing three counts of which the first
two were (i) winning money to themselves and others from a bookmaker by fraud in wagering, and (ii) an attempt by false
pretences to get the bookmaker to hand over that sum of money. On appeal against conviction,

Held H was not guilty of the offence of winning money by fraud in wagering, charged in the first count, because he did not
obtain the 79, but on that count the court would substitute a verdict of guilty of an attempt (see p 296, letter h, post); accordingly
the second count must be quashed as it in effect alleged the same offence (see p 297, letter a, post).
Appeal allowed in part.

Notes
As to winning money, etc, by fraud at gaming, see 10 Halsburys Laws (3rd Edn) 822, para 1587; and for cases on the subject, see
15 Digest (Repl) 1194, 1195, 12,12612,136.
For the Gaming Act, 1845, s 17, see 10 Halsburys Statutes (2nd Edn) 755.

Case referred to in judgment


R v Moss (1856), Dears & B 104, 26 LJMC 9, 28 LTOS 109, 20 JP 757, 7 Cox, CC 200, 15 Digest (Repl) 1195, 12,136.

Appeals
These were appeals by William Henry Harris and Francis Reginald Turner against conviction at Worcester City Quarter Sessions
on three counts relating to winning money by fraud in betting on horses. They were each sentenced to twelve months
imprisonment on counts 1 and 3, and to six months imprisonment on count 2, the sentences to run concurrently. The facts are set
out in the judgment of the court. The case against the prisoner Turner, was, substantially, that he had participated and agreed with
the prisoner Harris in the acts constituting the offences, and accordingly his position was immaterial to the report of the legal
issue on counts 1 or 2. Applications for leave to appeal against sentence were refused after the hearing of the appeals.
The authority and cases noted belowb were cited during the argument in addition to the case referred to in the judgment.
________________________________________
b 10 Halsburys Statutes (2nd Edn) 755 and notes, R v Manning, (1883), 12 QBD 241, R v Plummer, [1902] 2 KB 339, R v Leon, [1945] 1 All
ER 14, [1945] KB 136, R v Clucas (No 1), [1949] 2 All ER 40, [1949] 2 KB 226, R v Clucas (No 2), [1959] 1 All ER 438, R v Prater,
[1960] 1 All ER 298, [1960] 2 QB 464

M D L Worsley for the appellants.


Paul Wrightson for the Crown.
294

5 March 1963. The following judgment was delivered.

LORD PARKER CJ delivered the following judgment of the court. These two appellants were convicted at Worcester City
Quarter Sessions on three counts relating to the same transaction, namely, the winning of money by fraud in betting on horses.
Count 1 was a count framed under s 17 of the Gaming Act, 1845. It was in this form:

You are each of you charged with fraud in wagering, for having on May 19, 1962, in the City of Worcester and county
of the same city by fraud in wagering on the events of horse-races [won] to the said William Henry Harris and others from
Maurice Edward Hatt the sum of 79 18s. 5d.

The second count alleged an attempt by false pretences to get Maurice Edward Hatt, the bookmaker, to hand over that sum of
money, and the third count alleged a conspiracy between the two of them to defraud the bookmaker. The prosecution case was
that the appellant Harris telephoned this bookmakers office, and pretended to be a Sgt Harris from Norton Barracks, Worcester,
stating that he had collected a few bets from the boys. He asked to be put on a commission basis, telling the bookmaker that he
had been recommended by a friend, and proceeded to place bets. Two, I think, of the horses won, and thereafter the appellant
Harris telephoned on three occasions. No doubt the bookmaker had failed to pay, being suspicious, and it was finally arranged
that the appellant Harris would call on him. It was said that he had repeated over the telephone the false pretence c that he was
Sgt W H Harris of Norton Barracks, and, indeed, had gone further and had stated that his army number was 257725. The
bookmaker was not satisfied and communicated with the police. The prosecution clearly framed these counts on the supposition
that, in regard to count 1, the count under the Gaming Act, it was unnecessary to show that money had been obtained, and that it
was sufficient if money had been won in the sense that a debt of honour had been created in the bookmaker to pay that sum of
money. Count 2 was on the basis that, the horse having won, the appellants had tried to persuade the bookmaker to pay over the
money by repeating the false pretences. Count 3, as I have said already, was the conspiracy. So far as the appellant Turner is
concerned, it is conceded on both sides that the case against him was not a strong one. At the same time, it is undoubtedly true
that there was evidence on which the jury, who saw him, were entitled to convict. Counsel for the appellants, to whom the court
is indebted for his argument, has been very carefully through the transcript, both of the summing-up and of the evidence, and he
has brought to our attention a number of matters in which he criticises the recorder in his summing-up and alleges that counsel
for the Crown, in cross-examination, made misrepresentations as to the evidence that had already been given, and continued to
press those misrepresentations when he ought not to have done so. Those and other matters have been brought to the attention of
the court. The court has carefully considered them, and has come to the conclusion that there is nothing in those matters which
would entitle this court to interfere, or, indeed, to say that it was unsafe to allow the convictions to stand. That disposes really of
the convictions for conspiracy against both these appellants.
________________________________________
c In his summing-up the recorder said that the second charge, that of attempting to obtain money by false pretences rested, on what the
appellant Harris said after the races, viz, that he again told the bookmaker that he was Sgt Harris of Norton Barracks in order to induce the
bookmaker to pay over the 79 winnings

But legal argument has been advanced as to counts 1 and 2. It is said with regard to count 1 that it is an essential part of the
offence under s 17 of the Gaming Act, 1845, that the money should have been won in the sense that it should have been paid
over. Accordingly, what these two appellants did could be, on that view, no more than an attempt. The words of the section, so
far as they are relevant, are these:
295

Every person who shall, by any fraud or unlawful device or ill practice in wagering on the event of any game,
sport win from any other person to himself, or any other or others, any sum of money or valuable thing, shall be
deemed guilty of obtaining such money or valuable thing from such other person by a false pretence, with intent to cheat or
defraud such person of the same, and, being convicted thereof, shall be punished accordingly.

The point really narrows down to this, whether win from any other person to himself any sum of money or valuable
thing means obtain money. It is curious that this point has never yet been decided. In R v Moss, which was as long ago as
1856, the point was reserved. What had happened there was that the defendant was charged for that he did win from one Bernard
a certain sum of money with intent to cheat, etc. The jury convicted, and the point was reserved for the Court of Crown Cases
Reserved whether there was a defect in the indictment in that it had not alleged that the money won was in fact the money of the
said Bernard. In the course of the argument, Bramwell B, is reported as saying that ((1856), 7 Cox, CC at p 201):

The offence under the statute is not the actual obtaining of any specific money or chattel, but the winning of
money that is of the right to have money; that is, an abstraction, not a thing in nature, with regard to which you may say
whose property it is.

Erle J then said ((1856), 7 Cox, CC at p 201): According to my recollection no money was actually obtained in the car cheating
cases at Brighton. That occurred during the course of the argument, but when the judgment was delivered, Pollock CB said
((1856), Dears & B at p 108):

Whether the word winning is used in this statute in the limited sense in which winning is used in the mining districts,
and means actually getting and obtaining the money, or in its more general sense of obtaining a title to a sum of money by
becoming the winner of a stake, no ambiguity can arise if the offence is stated as it is in this indictment in the very words of
the statute

In other words, the court there reserved this very question and refused to pronounce on it.
The court has come to the conclusion that it is an essential ingredient in this offence that the money won should have been
obtained. Not only does the court think an obtaining of the money is a more apt description of win from any person to himself
any sum of money, but it would seem that the words which follow, shall be guilty of obtaining such money by a false
pretence, with intent to cheat, etc, are reciting the obtaining of such money. It is not deeming the winning to be an obtaining; it
is deeming what has been obtained to have been obtained by a false pretence. Read in that way, it would follow that neither of
the appellants could be guilty of the offence as laid, in that they never got the 79 in question. At the same time, there is no doubt
that this court has powerd, which it will use, to substitute a verdict of the attempt. Count 2, as I have already said, alleges an
attempt to obtain by false pretences, and, accordingly, in the view that this court has taken as to count 1, count 2 is really
superfluous. It is in effect alleging the same thing, and the court in those 296 circumstances thinks that it is right that the verdict
on that count should be quashed. Accordingly, in the judgment of this court, count 3 remains; count 2 is quashed, and there is
substituted for count 1 the attempt.
________________________________________
d Under s 5(2) of the Criminal Appeal Act, 1907

Appeal allowed in part.

Solicitors: Registrar, Court of Criminal Appeal (for the appellants); Tree, Hemming & Johnston, Worcester (for the Crown).

N P Metcalfe Esq Barrister.


[1963] 2 All ER 297

Burns v Currell
CRIMINAL; Road Traffic

QUEENS BENCH DIVISION


LORD PARKER CJ, ASHWORTH AND WINN JJ
28 MARCH 1963

Road Traffic Motor vehicle Go-Kart Mechanically propelled vehicle with engine at rear Tubular frame mounted on four
small wheels and equipped with seat, steering-wheel and column, and silencer Brakes operating on rear wheels only Whether
vehicle intended or adapted for use on roads Intended Adapted Road Traffic Act, 1960 (8 & 9 Eliz 2 c 16), s 253(1).

In the expression intended or adapted for use on roads in s 253(1) of the Road Traffic Act, 1960 a, the test of being intended
is whether a reasonable person looking at the vehicle would say that one of its uses would be a road use (see p 300, letter e, post).
________________________________________
a Section 253(1), so far as material, is set out at p 299, letter c, post

The appellant drove a mechanically propelled vehicle known as a Go-Kart on a road. The Go-Kart had its engine at the rear,
had a tubular frame mounted on four small wheels and was equipped with a single seat, steering-wheel and column, and an
efficient silencer. It had brakes which operated on the rear wheels only, and it had no horn, springs, parking-brake, driving-mirror
or wings. The evidence was that the appellant had used the Go-Kart on the road on one occasion only, and there was no evidence
that other people used Go-Karts on the road. The appellant was convicted of certain offences against the Motor Vehicles
(Construction and Use) Regulations, 1955, and the Road Traffic Act, 1960. On appeal on the question whether the Go-Kart was a
motor vehicle and a motor car within the meaning of s 253(1) and (2) of the Act of 1960,
Held The convictions must be quashed, because, applying the test stated at letter d above, there was not sufficient evidence
before the justices to prove beyond a reasonable doubt that any reasonable person looking at the Go-Kart would say that one of
its uses would be a use on the road (see p 300, letter f, post), nor that it was fit or apt for use on the road, although it was capable
of such use (see p 301, letter a, post).
Dictum of Lord Justice-Clerk Thomson in Woodward v James Young (Contractors) Ltd (1958 SC (J) at p 33) followed.
Per Curiam: (i) the word adapted in the expression intended or adapted for use on roads in s 253(1) of the Road Traffic
Act, 1960b, is used in its adjective sense as meaning fit or apt (see p 301, letter a, post).
________________________________________
b Regulation 44 deals with the braking system of a motor car, and reg 46 deals with wings on motor cars

(ii) In reg 44 and reg 46 of the Motor Vehicles (Construction and Use) Regulations, 1955, the expression motor car means
a motor vehicle which is a motor car and not a motor car which is not a motor vehicle (see p 301, letter e, post).
Appeal allowed.

Notes
As to the meaning of motor vehicle, see 33 Halsburys Laws (3rd Edn) 417, para 680.
297
For the Road Traffic Act, 1960, s 253, see 40 Halsburys Statutes (2nd Edn) 922.

Cases referred to in judgment


Daley v Hargreaves [1961] 1 All ER 552, [1961] 1 WLR 487, 125 JP 193, 3rd Digest Supp.
Macdonald v Carmichael 1941 SC (J) 27, [1941] SLT 81.
Maddox v Storer [1962] 1 All ER 831, [1962] 2 WLR 958.
Woodward v James Young (Contractors) Ltd 1958 SC (J) 28, [1958] SLT 289, 3rd Digest Supp.

Case Stated
This was a Case Stated by justices for the borough of High Wycombe in the county of Buckinghamshire in respect of their
adjudication as a magistrates court sitting at High Wycombe on 31 October 1962. On 8 September 1962, the respondent, Charles
Currell, preferred seven informations against the appellant, David Charles Burns, six of which charged that the appellant on 8
September 1962, unlawfully used a motor car on Halifax Road, High Wycombe, contrary to regs 9, 10, 16(1)(b), 19, 44 and 46 of
the Motor Vehicles (Construction and Use) Regulations, 1955, and s 64(2) of the Road Traffic Act, 1960, the seventh charging
that, on the same date and on the same road, he unlawfully used a motor car, contrary to s 201 of the Act of 1960.
The following facts were found. On 8 September 1962, the appellant was seated in a self-propelled vehicle known as a Go-
Kart at the extreme end of Halifax Road. At the material time and place, Halifax Road was a cul-de-sac which served two
factories. That part of the road on which the vehicle was being driven by the appellant had not been adopted by the local
authority, but it had footpaths, a concrete surface and street lighting. At the end of the road where the appellant was, there was an
open piece of waste-ground on one side, and at the end of the road there was a factory which on that day was closed. Halifax
Road led off from a public road, and was a road to which the public had access. The engine of the Go-Kart was running, the Go-
Kart was moving, and the appellant was driving it. The Go-Kart was mechanically propelled with the engine at the rear. It had a
tubular frame mounted on four small wheels, and was equipped with a single seat, steering-wheel and column. It had no horn,
springs, parking-brake, driving-mirror or wings, but was equipped with an efficient silencer. It had brakes which operated on the
rear wheels only. Its maximum speed was approximately forty miles per hour. The Go-Kart was not constructed or adapted to
carry passengers other than the driver or a load. The appellant had owned the Go-Kart for about seven or eight months, and
before 8 September 1962, it had never been used on any public road. It had been conveyed to the material part of Halifax Road
on an Austin Countryman and was removed in the same manner.
It was contended on behalf of the appellant that the Go-Kart was not constructed, intended or adapted for use on roads to
which the public had access, but was constructed and intended to be used for sporting purposes on private property. The
appellant at no time intended to use the Go-Kart on a road to which the public had access. It was intended to be used on private
property occupied by a club near Tring or on such other non-public place as might be found. The Go-Kart was constructed for
use on private property and not on roads to which the public had access. It was not a motor vehicle or a motor car within s 253 of
the Road Traffic Act, 1960. It was contended on behalf of the respondent that the Go-Kart was a motor vehicle, namely, a motor
car, within s 253 of the Act of 1960. The justices were of the opinion that the Go-Kart was a motor vehicle and a motor car
within s 253 of the Act of 1960, and found all the charges proved. They gave the appellant an absolute discharge on payment of
4s costs on each of the six informations and fined him 5 on the seventh information.
The appellant now appealed.
298
The cases noted belowc were cited during the argument in addition to those referred to in the judgment of Lord Parker CJ.
________________________________________
c Keeble v Miller [1950] 1 All ER 261, [1950] 1 KB 601, Flower Freight Co Ltd v Hammond, [1962] 3 All ER 950, [1963] 1 QB 275

C Stock for the appellant.


J R V McAulay for the respondent.

28 March 1963. The following judgments were delivered.

LORD PARKER CJ. This is an appeal by way of Case Stated from a decision of justices for the borough of High Wycombe,
who convicted the appellant on seven informations, six of which alleged offences against the Motor Vehicles (Construction and
Use) Regulations, 1955, and s 64(2) of the Road Traffic Act, 1960, and the seventh an offence against s 201 of the Road Traffic
Act, 1960, viz, using on the road a motor vehicle when there was not in force in relation to the user of the motor vehicle a policy
of insurance or other security. The particular breaches of the regulations and the offences generally are immaterial. The real
question here is whether the vehicle concerned, which is known as a Go-Kart, is a motor vehicle within the meaning of s 253(1)
of the Road Traffic Act, 1960. That subsection provides: In this Act motor vehicle means a mechanically propelled vehicle
intended or adapted for use on roads
[His Lordship stated the facts, and continued:] These words intended or adapted for use on roads and the words which
appear in the Act in another connexion constructed or adapted for a certain use are always giving rise to trouble. So far as the
cases are concerned with this form of words, the court has been referred to Daley v Hargreaves, which concerned the use on a
road of two dumpers, mechanically propelled vehicles used in the ordinary way for the construction of works. The way that case
went was rather special; the real case made by the prosecution was that, because the vehicle was found at the moment of the
alleged offence on the road, it must be taken to have been intended for use on the road. It is quite clear that that submission of the
prosecution was rejected by the court. Indeed, if it had been acceded to, the appeal would have been dismissed instead of being
allowed. The court referred to a Scottish case, MacDonald v Carmichael in which the court of justiciary came to the conclusion
that similar dumpers did not come within the words intended or adapted for use on the road, and, therefore, were not motor
vehicles. The court felt constrained, if possible, to follow that case. Salmon J who gave the leading judgment, said this ([1961] 1
All ER at p 556):

My conclusion might, and probably would, have been different if the findings had shown that the dumpers were
reasonably suitable for being driven along the public roads in transit or for the purpose of carrying material from one site to
another. Nor must it be thought that I am acceding to the appellants submission that the intention referred to in the
relevant sections is the manufacturers intention alone. It may be that the legislature had no particular persons intention in
view, whether manufacturers, wholesalers, retailers, owners or users. Intended for use on roads may mean no
more than suitable or apt for use. I prefer, however, to express no concluded view on this point but to base my decision,
following Macdonald v. Carmichael, on the ground that in no event does the evidence of very limited user in this case
establish that the vehicles were intended for use on roads within the meaning of the statutes concerned.

I myself added ([1961] 1 All ER at p 556):

I would only like to emphasise that it must not be taken as the result of this decision that dumpers of the type used in
this case are not motor vehicles intended or adapted for use on the road.
299

In other words, that case was decided on the basis that, whatever the words intended or adapted for use on roads exactly meant,
it had not been proved in that case that the dumpers were so intended or adapted.
Since MacDonald v Carmichael, there has been another case before the Court of Session, Woodward v James Young
(Contractors) Ltd. That case concerned the ordinary farm tractor, and the court had no hesitation in holding that such a vehicle
was intended or adapted for use on the road. The court distinguished that case from MacDonald v Carmichael on the basis that
the dumpers were really a bit of plant on wheels, whereas everyone knew the function of a tractor, which not only could be used
but was commonly used on roads. It is to be observed that Lord Blades referred (1941 SC (J) at p 38) with approval to the
decision of the Sheriff-Substitute, who had taken the view that, in deciding whether the mechanically propelled vehicle was
intended for use on roads, that meant intended for use on roads for use for ordinary road purposes.
Thus, in the ordinary case, it seems to me that there will be little difficulty in saying whether a particular vehicle is a motor
vehicle or not. But to define exactly the meaning of the words intended or adapted is by no means easy. For my part, I think
that the expression intended, to take that word first, does not mean intended by the user of the vehicle either at the moment of
the alleged offence or for the future. I do not think that it means the intention of the manufacturer or the wholesaler or the
retailer; and it may be, as Salmon J, said in Daleys case ([1961] 1 All ER at p 556), that it is not referring to the intention as such
of any particular purpose. Salmon J suggested that the word intended might be paraphrased as suitable or apt. It may be
merely a difference of wording, but I prefer to make the test whether a reasonable person looking at the vehicle would say that
one of its users would be a road user. In deciding that question, the reasonable man would not, as I conceive, have to envisage
what some man losing his senses would do with a vehicle; nor an isolated user or a user in an emergency. The real question is: is
some general use on the roads contemplated as one of the users? Approaching the matter in that way at the end of the case, the
justices would have to ask themselves: has it been proved beyond a reasonable doubt that any reasonable person looking at the
Go-Kart would say that one of its uses would be use on the road? For my part, I have come to the conclusion that there really
was no such evidence before them as to satisfy them on that point according to the ordinary standard of proof. The evidence was
that the appellant had used this vehicle on this day alone and that he had never used it before. There was no evidence that other
people used these vehicles on the road, nor is it suggested by the justices that they came to their conclusion, as they would be
entitled to up to a point, on their own experience and knowledge. As I have said, all that they had before them was that a Go-Kart
had been used on a road to which the public had access on this one occasion. Looked at in that way, so far as this matter of
intended is concerned, I do not think that the justices had any material on which they could feel sure so as to be able to convict.
So far as the other word adapted is concerned, as was pointed out in Maddox v Storer, the word adapted is used through
the Act of 1960 in a number of different contexts. Sometimes it is used as an alternative to constructed or adapted, and it seems
clear, and, indeed, it has been so held for a very long time, that adapted there means altered. On the other hand, as it was
pointed out in Maddox v Storer, it is used in other contexts in this Act, in particular when it stands alone as clearly meaning apt
or fit, in other words in an adjectival sense. Here in this context, my own view is, though I think that it is perhaps unnecessary
to decide it in this case, that adapted, when used disjunctively with intended and not with the word constructed, is used in
its adjectival sense. That was the view expressed in one of the Scottish 300 cases: see the Lord Justice-Clerk (Lord Thomson) in
Woodward v James Young (Contractors) Ltd (1958 SC (J) at p 33). If, of course, it means altered, there is no question of it
having been altered in the present case; but if, as the Lord Justice-Clerk felt, and as I am inclined to feel, it does mean fit and
apt, again I do not think that it was proved that the Go-Kart was fit or apt. It was undoubtedly capable of being used on the
road, but adapted adjectivally means considerably more than that:
This again, in my view, is a case similar to Daley v Hargreaves, where the prosecution have not proved their case, but I am
far from saying that, on proof of a general user, not this sole use but a general user, of Go-Karts on roads, that a bench of justices
would not be fully entitled to hold that it was a motor vehicle.
I ought to mention one other point, that, in regard to two of these regulations, reg 44 and reg 46, the offence consists in
having, not a motor vehicle, but a motor car on the road without a particular form of equipment. Counsel for the respondent has
referred us to s 253(2) of the Act, which is defining a motor car, and it says:

In this Act motor car means a mechanically propelled vehicle, not being a motor cycle or an invalid carriage, which
is constructed itself to carry a load or passengers.

Counsel says that that is quite independent of motor vehicle, that the genus here is merely a mechanically propelled vehicle and
that within that genus a motor vehicle means one thing and a motor car means another. I confess that, to my mind, that would
lead to most extraordinary results, but when one considers that these offences were under the regulations, the matter as it seems to
me becomes clear because these regulations are dealing with motor vehicles, and, indeed, they are made under s 64(1) of the Act
of 1960, which only gives the minister power to make regulations in regard to motor vehicles. Accordingly, as it seems to me,
when in reg 44 and reg 46 one has the use of the expression motor car, it must mean a motor vehicle which is a motor car and
not, as is suggested, a motor car which is not a motor vehicle.
For these reasons, I think that this appeal succeeds and that the convictions must be quashed.

ASHWORTH J. I agree, and I would only make the same reservations as those which my Lord has proposed.

WINN J. I agree with both the judgments which have been given.

Appeal allowed.
Solicitors: J B Izod agent for Reynolds, Parry-Jones & Crawford, High Wycombe (for the appellant); Sharpe, Pritchard & Co
agents for Clerk to the Buckinghamshire County Council (for the respondent).

Shireen Irani Barrister.


301
[1963] 2 All ER 302

Re Automatic Telephone & Electric Co Ltds Application


COMPETITION

COURT OF APPEAL
WILLMER, HARMAN AND UPJOHN LJJ
25, 26 FEBRUARY, 28, 29 MARCH 1963

Restrictive Trade Practices Agreement Registration Telephone manufacturers agreement for equal division of business
offered by the Postmaster-General Manufacturers agreement consequent on a Crown agreement between the manufacturers
and the Postmaster-General Postmaster-General agreeing by Crown agreement to order from manufacturer notified by
committee appointed by the manufacturers Manufacturers agreement providing for allocation of orders and the putting
forward of the selected manufacturers name Whether manufacturers agreement contained new restrictions not in Crown
agreement Whether two agreements complementary and manufacturers agreement outside scope of Act, as Crown not bound
by Act Restrictive Trade Practices Act, 1956 (4 & 5 Eliz 2 c 68), s 6(1), s 9(1).

Under an agreement (the Crown agreement) with the Postmaster-General, eight manufacturers undertook to supply telephone
apparatus in accordance with orders placed by him, and for that purpose to establish a committee. The Postmaster-General was
not to be concerned with the method of appointment of the committee or with the method of performing its functions, and he
undertook to place his orders (except in certain circumstances) with the manufacturer nominated by the committee for the
particular order. In connexion with the carrying out of this agreement the manufacturers entered into a further agreement (the
TAM agreement) among themselves providing for the appointment and functioning of the committee, and for the nomination
of the manufacturer who was to fulfil a particular order to be by unanimous decision of the committee, or, failing unanimity,
according to the quota standing of the respective manufacturers based on a division of the business among them in equal shares.

Held (Assuming, but not deciding, that the TAM agreement did introduce new restrictions beyond those in the Crown
agreement) the Crown agreement and the TAM agreement were in substance complementary, and, as the Crown agreement could
not be operated without the TAM agreement, any rendering void of the latter agreement by virtue of the Restrictive Trade
Practices Act, 1956, s 20 and s 21, would prejudice the Crown, with the consequence that, as the Crown was not bound by the Act
of 1956, the TAM agreement, like the Crown agreement, was outside the ambit of the Act of 1956 and was not registrable
pursuant to s 9(1) and s 6(1) thereof (see p 308, letters a and e, p 310, letter b, and p 313, letters h and i, post).
Per Willmer and Harman LJJ: the TAM agreement did not introduce any new restrictions not already accepted by the
contractors under the Crown agreement, since it provided only the method of selection of the one nominee manufacturer, whose
name was to be submitted under the Crown agreement, and was the machinery for making the Crown agreement work (see p 306,
letter i, to p 307, letter a, p 309, letter g, and p 309, letter i, to p 310, letter a, post).
Decision of Wilberforce J ([1962] 2 All ER 207) reversed.

Notes
As to agreements subject to registration under the Restrictive Trade Practices Act, 1956, see 38 Halsburys Laws (3rd Edn) 98,
99, para 126.
As to the principle that the Crown is not bound by statute unless it is expressly or by necessary implication so provided, see
36 Halsburys Laws (3rd Edn) 430432, para 652.)
For the Restrictive Trade Practices Act, 1956, s 6 and s 9, see 36 Halsburys Statutes (2nd Edn) 937, 942.
302

Case referred to in judgments


A-G v Hancock [1940] 1 All ER 32, [1940] 1 KB 427, 109 LJKB 243, 164 LT 52, 2nd Digest Supp.

Appeal
The manufacturers, the contractors, who were parties to two agreements, one with the Postmaster-General (the Crown agreement)
and one among themselves, appealed against an order of Wilberforce J made on 3 April 1962, and reported [1962] 2 All ER 207,
whereby a declaration was made that the contractors agreement among themselves was an agreement to which Part 1 of the
Restrictive Trade Practices Act, 1956, applied and was subject to registration under s 9 of the Act. They sought a declaration that
the agreement was not one to which Part 1 of the Act applied and was not subject to registration. The grounds of appeal were
inter alia: (i) that no restriction as to any of the matters set out in s 6(1) of the Act was accepted by the contractors under the
agreement among themselves; (ii) that the fact that a particular order would be placed with one of the contractors rather than
another resulted from a restriction accepted by the Postmaster-General under the Crown agreement and not from any restriction
accepted by the contractors; (iii) that if the contractors were under any restriction as to any of the matters set out in s 6(1) of the
Act it was accepted under the Crown agreement and not under their agreement among themselves: (iv) that cl 4(1) of the Crown
agreement imposed an obligation on the contractors towards the Crown to procure through the medium of the committee set up as
required by cl 3(1) thereof that on every occasion on which the Postmaster-General notified his intention to place an order a
nomination was made, and conferred on the Crown a right to have such nomination made on each such occasion; (v) that the
Crown agreement therefore obliged the contractors to make provision in their agreement for a nomination to be made on
occasions when the contractors were not unanimous as to which of the contractors was to be nominated; and, alternatively to
these grounds (vi) that the Crown was not bound by the Restrictive Trade Practices Act, 1956, which must be construed as if it
applied to agreements other than those conferring rights on the Crown which might be affected by registration of the agreement;
(vii) that the contractors were unable to perform their obligation to the Crown under cl 3(1) and cl 4(1) of the Crown agreement
without entering into such an agreement or agreements having the effect of excluding all the contractors except one from
nomination on each occasion; (viii) that, if as the judge held any such agreement imposed any restriction within s 6(1) of the Act,
the contractors could not perform such obligation without entering into an agreement which would be registrable unless excluded
from registration by the principle in (vi); (ix) that, if such agreement were registered, under the Act it must be referred to the
Restrictive Practices Court which might declare such restriction void and impose an injunction under s 20(3); (x) that in such
event the contractors would be unable to perform their obligations to the Crown, and the right of the Crown under cl 4(1) of the
Crown agreement to a nomination through a committee would be prejudicially affected, and (xi) that accordingly the rights of the
Crown might be affected by the registration of the contractors agreement, and the Act therefore ought to be construed as if it did
not apply to that agreement.
The cases noted belowa were referred to in argument in addition to that cited in the judgment.
________________________________________
a Clark v Downs, Clark v Mawby [1931] All ER Rep 157, The Moorcock, (1884), 14 PD 64

H A P Fisher QC and J F Lever for the manufacturers, the contractors.


Sir Milner Holland QC and J P F E Warner for the Registrar of Restrictive Trading Agreements.

Cur adv vult


303

29 March 1963. The following judgments were delivered.

WILLMER LJ. This is an appeal from a judgment of Wilberforce J, given on 3 April 1962, on an originating summons under s
13(2) of the Restrictive Trade Practices Act, 1956, brought on behalf of eight companies engaged in the manufacture of telephone
apparatus. By the originating summons the companies (to whom I will refer hereafter as the contractors) ask for a declaration
that a certain agreement made between themselves in relation to the supply of telephone apparatus to the Postmaster-General is
not an agreement falling within Part 1 of the Act so as to be registrable thereunder.
The agreement in question, which was called The Telephone Manufacturers Agreement (referred to hereafter as the
TAM agreement) was made on 27 February 1957, and was expressed to be consequential on a prior agreement (referred to
hereafter as the Crown agreement) made between the contractors and the Postmaster-General for the supply of telephone
apparatus. The Crown agreement, in pursuance of which the TAM agreement was made, was dated 4 June 1952, but has since
been replaced by a new agreement of 30 September 1960, which, so far as relevant to the present case, is in the same terms as the
agreement of 1952. By cl 8 of the TAM agreement this was deemed to have come into force on the same date as the Crown
agreement of 1952, and was to continue for the period of any renewal, extension or modification thereof into which the
contractors might enter with the Postmaster-General.
It is necessary to refer to s 6 of the Restrictive Trade Practices Act, 1956, the material part of which provides as follows:

(1) Subject to the provisions of the two next following sections, this Part of this Act applies to any agreement between
two or more persons carrying on business within the United Kingdom in the production or supply of goods being an
agreement under which restrictions are accepted by two or more parties in respect of the following matters, that is to say:
(a) the prices to be charged, quoted or paid for goods supplied, offered or acquired, or for the application of any process of
manufacture to goods (c) the quantities or descriptions of goods to be produced, supplied or acquired (e) the persons
or classes of persons to, for or from whom goods are to be supplied or acquired
(3) In this Part of this Act agreement includes any agreement or arrangement, whether or not it is or is intended to be
enforceable (apart from any provision of this Act) by legal proceedings, and references in this Part of this Act to restrictions
accepted under an agreement shall be construed accordingly; and restriction includes any negative obligation, whether
express or implied and whether absolute or not.

Section 9(1) provides that

every agreement to which this Part of this Act applies shall be subject to registration thereunder.

The questions arising for decision are twofold: (i) whether the TAM agreement is registrable on the ground that the contractors
thereby accepted any restriction, as defined by s 6(3) of the Act of 1956, to which they were not already subject under the Crown
agreement: (ii) if so, whether the interests of the Postmaster-General are prejudicially affected by the TAM agreement so as to
remove that agreement from the ambit of the Act.
It is common ground that, in accordance with the doctrine enunciated by Wrottesley J, in A-G v Hancock ([1940] 1 All ER
32 at p 40; [1940] 1 KB 427 at p 439), the Restrictive Trade Practices Act, 1956, does not bind the Crown. In these
circumstances it has been conceded on behalf of the registrar that the Crown agreement, to which the Postmaster-General was a
party, is not registrable under the Act. It is the registrars case, however, that the TAM agreement, to which the Postmaster-
General was not 304 a party, introduced new restrictions binding the contractors over and above those which they accepted in the
Crown agreement. It has been submitted that, since these restrictions applied only as between the contractors themselves, it could
not be said that they prejudicially affected the interests of the Postmaster-General. Accordingly, it is said, there is no reason why
the Act of 1956 should not apply to the TAM agreement so as to render it registrable thereunder.
The essential part of the Crown agreement for the purposes of this case is contained in cll 2, 3 and 4 thereof, the material
portions of which are as follows:

2. The contractors shall manufacture and supply to the Postmaster-General in accordance with orders placed by him
during the contract period telephone apparatus of such kinds and descriptions as the Postmaster-General may from time to
time prescribe in the manner hereinafter provided and on and subject to the conditions hereinafter contained.
3.(1) The contractors shall appoint a committee and the committee shall appoint a secretary. The contractors shall
within fourteen days from the date of these presents cause the Postmaster-General to be notified in writing of the name of
the secretary and of the address at which communications may be sent to him
(2) The Postmaster-General shall not be concerned with the method of appointment of members of the committee or to
see that each of the contractors is at any time represented thereon or with the deliberations of the committee or the method
in which it shall perform its functions or with any failure on its part so to do. For all the purposes of these presents the
Postmaster-General shall be entitled (notwithstanding any objections raised or representations made to him by any of the
contractors) to act upon any notification made to him by any person who holds or under the provisions hereinbefore
contained is deemed to hold the office of secretary of the committee of any decision of the committee and such notification
shall be deemed correctly to represent such decision as aforesaid.
4.(1) The Postmaster-General shall from time to time notify the secretary of the committee of any orders which he
proposes to place with the contractors in accordance with these presents and the committee shall within fourteen days of
the receipt of such notification as aforesaid inform the Postmaster-General with which of the contractors each of the orders
referred to in the notification is to be placed and the Postmaster-General shall (subject as is hereinafter provided) place the
order accordingly within a reasonable time unless within two months after the receipt of such information the Postmaster-
General shall advise the committee that he does not intend to proceed with the order. If and so far as the committee shall
not so inform the Postmaster-General within such period of fourteen days as aforesaid the Postmaster-General may place
each such order with such one of the contractors as he shall in his discretion think fit and such contractor shall execute such
order upon the terms and conditions herein contained.

Provision was made in certain other clauses of the agreement whereby the Postmaster-General was to have the right in specified
circumstances to employ a contractor otherwise than in accordance with the agreement; but by cl 12(1) it was provided that,
except as otherwise mentioned in the agreement, the Postmaster-General should not order for delivery during the contract period
any telephone apparatus otherwise than from one of the contractors in accordance with the provisions of the agreement. Thus the
general scheme of the Crown agreement was that the Postmaster-General should place his orders with a single contractor whose
name the committee was bound to supply. It was left to the contractors to set up their own machinery, under a separate agreement
between themselves, for the purpose of determining which contractors name 305 should be put forward by the committee in any
particular case. It was for this purpose that the contractors entered into the TAM agreement.
Clause 1(i) of the TAM agreement provides for the setting up of a committee to consist of officials nominated by each of the
contractors. Clause 1(ii) provides that the committee shall appoint a secretary who shall, on receipt of the notification of orders
passed to him by the Postmaster-General under the Crown agreement, call a meeting of the committee to consider such orders,
and shall notify the Postmaster-General as to the contractor or contractors to whom the respective orders have been allocated.
The clause continues:

The secretary shall notify the Postmaster-General with respect to the allocation of orders in accordance with the
unanimous decision of the committee, but if a unanimous decision as to the allocation of a particular order has not been
made by the committee up to the time within which the secretary must notify the Postmaster-General according to the
[Crown agreement] the secretary shall be authorised to notify the Postmaster-General that the order has been allocated to
the party who is entitled thereto by virtue of the quota standing of the respective parties.

Clauses 3 and 4 of the TAM agreement provide as follows:

3. Division of business. Subject always to the provisions of cl. 4 hereof it is agreed that the basis of the division of
business to which this agreement applies shall be equal shares to each of the parties hereto.
4. Allocation. The allocation of business under this agreement shall be made so that each party shall receive the share
of the total business to which it is entitled under this agreement providing always that the Postmaster-Generals
requirements for supply and delivery shall be paramount.

Lastly, it is necessary to refer to cl 6 of the agreement, which provides that a record shall be kept of all allocations made and
orders received in such a manner that it shall be possible to ascertain at any time the position of each party in relation to the
others.
The learned judge decided both questions in favour of the registrar and against the contractors, and accordingly held that the
TAM agreement was registrable under the Act of 1956. As to the first question he came to the conclusion that the TAM
agreement did introduce new restrictions on the contractors other than those which they had accepted under the Crown
agreement. On the second question he held that these new restrictions did not prejudicially affect the rights or interests of the
Postmaster-General, and he therefore concluded that there was no reason why the TAM agreement should not be registered under
the Act. In reaching this conclusion he founded himself largely on cl 3(2) of the Crown agreement, and more particularly on the
provision that

the Postmaster-General shall not be concerned with the deliberations of the committee or the method in which it
shall perform its functions or with any failure on its part so to do.

It is obvious that, if the contractors can succeed on either of the questions argued, they must be entitled to a declaration that the
TAM agreement is not registrable under the Act of 1956. I have come to the conclusion, for reasons which I will attempt briefly
to explain in a moment, that the contractors are entitled to succeed on the second question. This makes it unnecessary to reach
any final decision with regard to the first question. But since the matter has been fully argued I think that it is right for me to give
expression to the view at which I should have been disposed to arrive on this point. I thought that Mr Fishers argument on this
point was forceful and convincing, and, much as I respect the view at which the learned judge arrived, I should have been
disposed, had it been necessary, to decide this point also in favour of the contractors. Looking at the substance of the matter it
does not seem to me true to say that the TAM agreement did introduce any new restriction or restrictions not already 306
accepted by the contractors under the Crown agreement. The essential restriction whereby only one name was to be put forward
to the Postmaster-General had already been accepted under the terms of the Crown agreement. The purpose and effect of the
TAM agreement was no more than to provide for the method of selection of that one name; in the absence of unanimous
agreement it was provided that a quota system should be adopted based on the principle of equality. This, in the picturesque
phrase used by Mr Fisher, did not have the effect of closing any door which was previously open to the contractors; its effect was
merely to open a door through which the selected contractor might pass. It may well be that, if the Crown agreement stood by
itself, and if no TAM agreement had ever been made, no contractors name could ever be put forward to the Postmaster-General
except as the result of unanimous agreement. But that of itself would involve a restriction whereby the name of only one
contractor could be put forward to the Postmaster-General. Such a restriction would none the less be a restriction even if the
name of the selected contractor was put forward only as the result of unanimous agreement between them. It seems to me that at
the best for the registrar the contractors under the Crown agreement accepted an obligation to agree on a single name to be put
forward if they were to avoid a breach of cl 4(1). The fact that the second sentence of that clause made provision for what the
Postmaster-General would be entitled to do in the event of such a breach does not in my judgment impair the force of the
restriction which the contractors accepted thereunder.
I do not, however, decide the case on this ground. I am prepared to assume that I am wrong, and that the effect of the TAM
agreement was to introduce a new restriction or restrictions not previously accepted by the contractors under the Crown
agreement. Assuming this to be so, I find it impossible to avoid the conclusion that any new restriction accepted by the
contractors under the TAM agreement must prejudice the rights and interests of the Postmaster-General. In relation to this part of
the case we found ourselves in a curious situation, in that it was the contractors who submitted that the interests of the Crown (in
the shape of the Postmaster-General) would be prejudicially affected, whereas counsel for the registrar was concerned to argue
that they would not. Had we been disposed to think that there was no possibility of the interests of the Postmaster-General being
affected, I think it would have been necessary to adjourn the case for further argument, so as to enable the Postmaster-General to
be separately represented for the purpose of making his own submissions. Having regard, however, to the view which I have
already expressed it is happily not necessary to take this course.
The argument on behalf of the registrar, which was accepted by the learned judge, was that cl 3(2) of the Crown agreement
makes it impossible to contend that the Postmaster-General could be prejudicially affected by the operation of the TAM
agreement. For that clause provides that the Postmaster-General is not to be concerned with the deliberations of the committee or
the method in which it shall perform its functions or with any failure on its part so to do. This is of course perfectly true, but I do
not think that it is sufficient to warrant the conclusion contended for. The Postmaster-General was very much concerned that the
committee should be set up, and that when set up it should perform its task of putting forward the selected name whenever the
Postmaster-General notified his intention of placing an order. By the Crown agreement the contractors bound themselves to see
that this was done, and if they failed to do so they would have been in clear breach of the agreement. It was contemplated, and
indeed required, by the Crown agreement that the contractors should set up the committee, and should make provision for the
putting forward of a single selected name. It was thus necessary in order to implement the Crown agreement that the contractors
should enter into a further agreement between themselves. Without such further agreement between the contractors the Crown
agreement could not be effectively worked. This being so, it seems to me that the Crown 307 agreement and the TAM agreement
are necessarily complementary to each other. Though contained in separate documents, they are not really severable, but are
hopelessly mixed up together.
The matter may, I think, be tested in this way. If the TAM agreement is held to be registrable, it must go before the
Restrictive Practices Court and may be held to be contrary to the public interest. Should that occur, no method of operating the
Crown agreement would remain, except in so far as the contractors on any particular occasion might be able to reach unanimous
agreement for submitting the name of the selected contractor. That, as it seems to me, would be a wholly unrealistic possibility.
Should it thus prove impossible to operate the Crown agreement, it must, I think, be presumed that this would prejudicially affect
the interests of the Postmaster-General. It is, moreover, to be remembered that it would not be open to the contractors to make a
new agreement between themselves providing for an alternative method of selecting the contractor whose name is to be put
forward to the Postmaster-General. For, if the TAM agreement were held to be void, the contractors would be subject to an order
under s 20(3) of the Act of 1956 restraining them from making any other agreement to the like effect. Any agreement whereby
they restricted themselves to putting forward a single name, whatever alternative method of selection was provided, would be in
my judgment an agreement to the like effect. This would be so even if they agreed that a name should only be put forward as the
result of unanimous agreement. For they would still be restricted by the obligation envisaged in the Crown agreement to put
forward only a single name. The result of the TAM agreement being held to be void would therefore be to leave the Crown
agreement almost wholly ineffective. Since the Crown agreement must be presumed to have been entered into for the benefit, not
only of the contractors, but also of the Postmaster-General, such a result must be regarded as prejudicial to the interests of the
Postmaster-General.
For these reasons I find myself with all respect unable to agree with the conclusion arrived at by the learned judge. I would
accordingly allow the appeal and hold that the contractors are entitled to the declaration sought.

HARMAN LJ read by Willmer LJ). The Restrictive Trade Practices Act, 1956, with which this appeal is concerned, was, as its
long title shows, an Act dealing with the prohibition of restrictive trading agreements when found contrary to the public interest.
Its machinery was to appoint a registrar, the respondent here, and establish a court before which he could bring restrictive
agreements as defined by s 6 of the Act of 1956. These agreements had under the Act to be registered, and it was provided that
they should be presumed to be contrary to the public interest unless absolved by the court, and if not so absolved be void. The
agreement mentioned in the heading to the summons and here called the TAM agreement has been registered under the Act of
1956 but has not yet been brought by the registrar before the Restrictive Practices Court. In the meanwhile the parties to it, the
contractors, have, as they are entitled to do under s 13(2) of the Act of 1956, applied to the High Court for a ruling whether or no
that agreement has been properly registered. The learned judge below came to the conclusion that it had been so registered
coming under s 6(1)(c) of the Act. Against this decision the contractors appeal.
The first thing here to be noticed about the Act of 1956 is that it does not apply to the Crown. This is admitted. It follows
that the emanation of the Crown constituted by the Postmaster-General is free to make his arrangements with his suppliers
without regard to the prohibition of restrictive practices prescribed by the Act. The Postmaster-General is by far the largest user
of telephone equipment in the country and he has chosen, as he is free to do, to confine his orders to the circle of manufacturers
represented by the eight appellant contractors. He is minded to confine his purchases to this circle and presumably has good
reason to do so, though this is not the concern of the court. He has for 308 this purpose negotiated an agreement of 30 September
1960, and now current which has been called the Crown agreement, expressed to be made between himself and the eight
appellant contractors, whereby they bind themselves during the contract period to manufacture and supply to him such kinds and
descriptions of telephonic apparatus as he shall prescribe. Having selected the eight contractors, the Postmaster-General has
thought it right to leave the selection of the particular contractor or contractors who will fulfil any given order to them and the
Crown agreement provides for the manner in which this is to be done. It binds the contractors to set up a committee with a
secretary to be a channel of communication, to whom he will notify his orders. On receipt of them, the committee is bound to
meet and through the secretary inform him of the selected contractor. The Postmaster-General binds himself, subject to a number
of exceptions which I do not think that I need describe in detail, to place the order with the selected contractor at prices specified
in the Crown agreement. In the Crown agreement the Postmaster-General expressly declines all responsibility for the
appointment and methods of the committee, especially its method of selecting the chosen contractor, and confines himself to
dealings through the secretary alone. The Crown agreement in fact expired on 31 March last, but it is treated as continuing
pending negotiations now current for a further agreement on the same lines.
This, the Crown agreement, is not registrable or subject to attack by the registrar. The agreement under criticism is what I
have called the TAM agreement, to which the Crown is not a party. This latter agreement, dated 27 February 1957, and expressed
to be made between the eight contractors, starts by reciting the former Crown agreement of 1952 (which is treated by the parties
as now referring to the agreement of 1960) and is expressed to be made in consideration of that agreement and of the mutual
covenants of the parties there set out. It is argued on one side by the contractors that this agreement is in fact no more than the
provision of machinery for carrying the Crown agreement into effect and that it obliges the contractors to do nothing that is not
either already explicit or at least implied in the Crown agreement. Alternatively, it is argued that, even if restrictions are imposed
on the parties by the TAM agreement, it is none the less immune from attack because to prohibit it must of necessity affect the
rights of the Crown and thus infringe the Crowns immunity.
The contrary argument supported by the registrar is that, viewed in isolation, the TAM agreement does undoubtedly impose
restrictions on the parties; that these go beyond any restrictions necessarily inherent in the Crown agreement; and that the latter
can subsist independently of the former which is therefore open to attack under the Act. In particular it is said that in at least one
respect the TAM agreement imposes a restriction not to be found in the Crown agreement, viz, an agreement by each of the
contractors to abide by a decision of a majority in making the selections.
It seems to me that the two points made on behalf of the contractors are really only one. It was the policy of the Crown, as
shown by the Crown agreement, having selected the eight most suitable candidates, to leave it to them which individual should
fulfil each order subject always to the exceptions by which the Postmaster-General reserved his freedom of action, and to the
absolute right conferred on him by cl 4 of the Crown agreement within a limited time to decline to proceed with any order. It is
to be supposed that the Postmaster-General regarded it as an advantage to be relieved of the detailed work of scanning
competitive estimates and that this was one of the objects the agreement set out to achieve. The contractors, on the other hand,
bound themselves to set up the committee and through it to select the candidate. It was argued nevertheless for the registrar that
the abdication by each contractor of his right to compete for the order was a further restriction. It was said that, if names could
only be put forward by the unanimous agreement of the parties, there would be no objection. This is a nicety that I do not
understand. Moreover, it would in effect 309 defeat the whole object of the Crown agreement, the purpose of which was to
oblige the contractors by setting up a committee to enable the process of selection to be effective. The contractors chose to adopt
the quota principle or in other words share all orders equally between themselves, but they might, as was pointed out, have
adopted any other form of selection.
I must say that it seems to me, looking at it broadly, that the two agreements are complementary and that the TAM
agreement is merely a machine for making the Crown agreement work. Had the TAM agreement been, as it might have been,
amalgamated with or been expressed as a schedule to the Crown agreement, I take it that the registrar would have admitted that
he could not interfere. I cannot see that the fact that in form there are two agreements and not one makes in substance any
difference. The two are so intimately connected that to interfere with the TAM agreement is in effect to frustrate in whole or in
part the Crown agreement, and thus to interfere with the freedom of contract of the Crown. It is true to say that the Crown
disclaims any control over the committee and its methods, but to insist that the committee can only act when it is unanimous is in
effect to hamstring the whole arrangement.
It is said that the word shall where used in cl 3 and cl 4 of the Crown agreement is not truly mandatory and could not be
enforced as such. No doubt that is true, for the agreement itself provides that, if the selected name be not presented within the
limited time, that merely frees the Postmaster-General from his obligation to limit his choice. None the less the words have a
contractual force and, if the contractors fail to establish a committee and appoint a secretary and submit a selected name, they
would be in breach of contract and could be sued in damages. The TAM agreement, as it seems to me on a fair reading, is merely
a regulation of the method by which the eight contractors carry out their contractual obligations, and is exempt from registration
along with the Crown agreement.

UPJOHN LJ. Willmer LJ has set out the relevant clauses of the Crown agreement and the TAM agreement and the relevant
sections of the Restrictive Trade Practices Act, 1956, and I need not repeat them. The general scheme of the Crown agreement
provides that the Postmaster-General shall purchase at fixed prices a large number of items of telephone equipment set out in the
Sch 2 thereto at the prices therein mentioned, which range from 1s 5d for a resistor spool to 199 18s 3d for a switchboard
component. Most of the items, however, are small and under 2 in value. The agreement contains clauses for variation of prices
to deal with increases in the wage structure and variations in costs of material.
No doubt that was a most important part of the Crown agreement, but another important part and the only one with which
we are concerned is that the contractors are to establish a committee which will appoint a secretary to whom the Postmaster-
General will notify orders that he is about to place. Within fourteen days the secretary has to nominate the particular contractor
with whom the order is to be placed by the Postmaster-General. In implementation of the original Crown agreement of 1952, the
contractors entered into the TAM agreement defining the powers of the committee so that the secretary would have power to
nominate the contractor within fourteen days. It was superseded by a TAM agreement of 27 February 1957, which is the
agreement the subject-matter of this application.
The whole question in this appeal is whether the TAM agreement is registrable with the Registrar of Restrictive Trading
Agreements. It is quite clear and is not in dispute that, if the TAM agreement does contain relevant restrictions accepted by two
or more parties, it is (ignoring for the moment the rights of the Crown) subject to registration and, on being registered, must in
due course inevitably come before the Restrictive Practices Court, which must declare that the restrictions therein contained are
contrary to the public interest and are void 310 unless such restrictions satisfy the somewhat stringent requirements of s 21 of the
Act of 1956.
Now counsel for the registrar concedes that the Crown is neither expressly nor by necessary implication bound by the
provisions of the Act of 1956 and in such case the law is clear. If an Act of Parliament would otherwise devest the Crown of its
property, rights, interests or prerogative, it is not to be construed as applying to the Crown, per Wrottesley J, in A-G v Hancock
([1940] 1 All ER at p 40; [1940] 1 KB at p 439). Counsel further concedes that in these circumstances the Crown agreement is
not subject to registration under the Act, not merely because the Crown is a party to the Crown agreement but because, in the
circumstances of this case, if the Crown agreement is subject to registration and so to the jurisdiction of the Restrictive Practices
Court, the property, rights, interests or prerogative of the Crown may be devested.
Counsel for the contractors submits two points. First, he says, apart altogether from the fact that the Crown is a party to the
Crown agreement, that the TAM agreement is not subject to registration, for he contends that no restriction is thereby accepted by
the contractors: the argument is that it is in the Crown agreement that the contractors accept the relevant restriction, viz, that
instead of all being free to quote times for completion or rates of manufacture (the prices being fixed) for any order, only one
contractor named by the secretary of the committee becomes entitled to quote for the order. Then it is said that the TAM
agreement contains no further restriction accepted by the contractors, because it provides merely machinery to work out the
provisions of the Crown agreement to enable the secretary to perform the functions provided by that agreement. The TAM
agreement merely lays down a method of selection or identification of the nominated person. Indeed, it was urged that, far from
creating any additional restrictions, it merely opens the door to the lucky contractor nominated by the secretary. The learned
judge did not accept this view, but held that the Crown agreement was merely an agreement to agree and that the restrictions were
in fact contained entirely in the TAM agreement. For my part and for the reasons which I shall mention later I am unable to
accept the views of the learned judge on that matter.
Secondly, it was said that, if that be wrong, then, although the Crown is not a party to the TAM agreement, its property,
rights and interests will be affected for the reason that, if the TAM agreement is registered and its restrictive provisions found to
be contrary to the public interest by the Restrictive Practices Court and therefore void, the Crown agreement cannot work, for the
secretary cannot operate the quota arrangement; therefore, it is submitted that the TAM agreement cannot be subject to
registration. The learned judge did not accept this view.
For the registrar, on the first point, counsel points out that, while the Crown agreement provides for the appointment of a
committee and secretary, it is completely silent as to the constitution and powers of the committee and the duties and powers of
the secretary, so that, until the contractors enter into some further agreement, there is nothing in the Crown agreement to enable
the contractors to instruct the secretary to nominate any particular contractor in the absence of the unanimous agreement of the
contractors. It is not until some further agreement between the contractors is entered into that they accept the restriction that, if
they do not agree unanimously, some machinery will provide for the solution of the obligation cast on the committee by cl 4 of
the Crown agreement to nominate the contractor to fulfil the particular order. That machinery might provide for orders to be
fulfilled in rotation or in any other way; in fact the method chosen is that the contractor having the highest quota standing should
be the nominated contractor. Therefore, argues counsel for the registrar, the TAM agreement does contain restrictions accepted
by the contractors and is subject to registration. Counsel subjected the relevant 311 clauses of the Crown agreement to critical
examination, but I think that I can deal with his submissions on that matter more conveniently on the second point: viz, if the
TAM agreement is prima facie subject to registration, does that affect the property, rights and interests of the Crown or, as
counsel for the registrar prefers to put it, if it is subject to registration, is it so impossible to disentangle the TAM agreement from
the Crown agreement that the Crowns interest must be prejudiced. He submits that the TAM agreement can be disentangled.
He points out that in cl 3(2) of the Crown agreement the Postmaster-General is careful to provide that he is not to be
concerned with the method of appointment of members of the committee or with the method in which it shall perform its
functions or with any failure on its part to do so. He goes on to point out that in cl 4(1), while it is contemplated that the
contractors through the secretary will nominate the person to accept any particular orders, he submits that they are not bound to
do so and he relies on the last sentence of that sub-clause which entitles the Postmaster-General to place the order with such one
of the contractors as he shall in his discretion think fit if the secretary fails to make a nomination. Relying on these clauses,
counsel for the registrar says that cl 4 merely gives a right to the committee to make a nomination and that the failure of the
secretary to make a nomination within fourteen days involves no breach of the clause. The Postmaster-General, he submits,
could not sue for damages if the contractors failed to nominate. The Crown agreement would work perfectly well according to its
tenor without any supplementary contract between the contractors, viz, it would work whenever the contractors unanimously
agree to nominate a contractor for a particular order. When they do not so agree, the Postmaster-General can by the express
provisions of cl 4 place the order where he pleases amongst the contractors. And so he submits the operation of the Crown
agreement is not in the least degree affected by the non-existence of the TAM agreement and so the Crown is not affected if there
is in fact a TAM agreement which is found to be contrary to public policy by the Restrictive Practices Court. Therefore, he
submits that the agreement is subject to registration and consequential review by that court. Those are the arguments.
I turn to consider the proper construction and effect of the Crown agreement and the TAM agreement. By cl 3 of the Crown
agreement the contractors shall appoint a committee and the committee shall appoint a secretary. By cl 4 the committee
shall inform the Postmaster-General which of the contractors is to receive the order. These words are clear and mandatory.
The obligation on the contractors to set up machinery for the selection of the contractor to fulfil the order in the absence of
unanimous agreement is, in my judgment, as essential a part of the Crown agreement as that relating to the fixing of prices. This
is a business agreement and the Postmaster-General wants not only to have agreed fixed prices but to have a clear answer within
fourteen days of the name of the contractor who is to fulfil each particular order and so avoid all the trouble and expense of
inquiring of contractors about dates of delivery or rates of manufacture of these, in the main, small and inexpensive telephone
components. The Postmaster-General is content to leave the allocation of individual orders to be decided by these well known
contractors of high standing, bearing in mind, of course, the number of escape clauses in the Crown agreement whereby the
Postmaster-General is entitled, putting it very shortly, if he is not satisfied with the offered dates of completion, rates of
manufacture or capacity of the contractor to execute the order satisfactorily, to place the order with a second nominee (cl 4(3)) or,
in exceptional circumstances, with another contractor (cl 4(4)), or even with an outside company (cl 4(5)). But the contractors
are, in my judgment, clearly bound to make some agreement inter se to implement the provisions of cl 3 and cl 4 of the Crown
agreement, and commit a breach of contract which goes to its roots if they do not.
I cannot accept the argument of counsel for the registrar to the contrary. Clause 3(2) is in my judgment against his
argument. The object of that sub-clause 312 is to make it clear that the Postmaster-General is not to be concerned with the
committee, its appointment or methods in any way. That is to be the entire responsibility of the contractors and any failure of the
contractors to appoint a committee or any failure on the part of the committee to perform its functions is not to be attributed in
any way to the Postmaster-General. He is entitled to the entire performance of the clauses relating to the appointment and
functions of the committee by the contractors without any assistance from himself.
Then with regard to the last sentence of cl 4(1) on which counsel for the registrar naturally strongly relied, I do not think for
one moment that the effect of cl 4(1) is to give merely a right to (as distinct from casting an obligation on) the committee to
nominate and on failure to do so there is no breach of any stipulation. I think that the reason for inserting that sentence is that, if
there is a failure to nominate, there might be a genuine doubt as to the obligations and rights of the Postmaster-General under cl
12(1). It is necessary to make it clear that on a failure to nominate he has the right to go elsewhere, but in theory he might sue the
contractors for breach of their obligation to nominate instead. This provision does not make the mandatory words to which I
have referred above merely permissive.
On the other hand, I am quite unable to accept the argument of counsel for the contractors that all the restrictions are
contained in the Crown agreement and none in the TAM agreement. If I am right in the conclusion that I have reached that it is
an essential term of the Crown agreement that a committee with power to select the particular contractor must be set up by the
contractors so that the contractors inter se must enter into a further agreement to implement the Crown agreement, it follows that
the Crown agreement and the TAM agreement are complementary and must be read together in order to see what restrictions are
accepted by the contractors. It gives a wholly incomplete picture to look at the Crown agreement and say that the only restriction
is that a committee is to be set up. The complete picture is only known, and the restrictions accepted by the contractors can only
be ascertained and defined, when it is known what arrangements they have made inter se so as to enable the committee and its
secretary to comply with the provisions of cl 4 of the Crown agreement. From the point of view of the contractors the Crown
agreement and the TAM agreement constitute one agreement. Clause 8 of the TAM agreement makes this clear. In the
circumstances of this case the disparity in dates between the Crown agreement and the TAM agreement matters not. That is
explained as a matter of history. To put the matter into a nutshell, it seems to me plain that, had the purchasing party in the
Crown agreement been a private person and not the Postmaster-General, then, ignoring altogether the obvious fact that by reason
of the fixing of prices the Crown agreement would have had to be registered, both agreements would have had to be registered.
This brings me to the real question whether the Crown will be prejudiced by registration of the TAM agreement with the
registrar with the consequential result that one day its provisions will be brought before the Restrictive Practices Court. From
what I have already said it must necessarily follow that, if the TAM agreement is registered, the rights and interests of the Crown
will be prejudiced. Suppose the TAM agreement is registered and it is declared that the restrictions which permit the secretary to
operate the provisions of cl 1(ii) and cl 3 of the TAM agreement are void and injunctions granted accordingly: it would make the
Crown agreement basically inoperative. The Postmaster-General, for the reasons already given, simply will not get what he
bargained for in the Crown agreement. Accordingly, it seems to me quite plain that, if the TAM agreement were to be registered,
the Crown would be prejudiced. But as the Crown is not bound by the Act it must follow, in accordance with the principle
enunciated by Wrottesley Jb, that the TAM agreement is not subject to registration and I would so declare.
313
________________________________________
b A-G v Hancock, [1940] 1 All ER at p 40; [1940] 1 KB at p 439

By some oversight the originating summons did not claim rectification of the register by expunging the TAM agreement
from it, but, as I understood counsel for the registrar, he did not object to an order for rectification because the registrar himself
has no power to expunge any entry from the register. I would make an order rectifying the register by expunging the TAM
agreement and allow the appeal accordingly. Let me add that had I reached a contrary conclusion I should not have been
prepared to determine this matter unless the Postmaster-General had been given an opportunity of appearing and expressing his
views.

WILLMER LJ. Perhaps I may add that I find myself in agreement with what my lord has just said with regard to an order
rectifying the register.

Appeal allowed. Declaration as prayed. Application for leave to appeal to House of Lords adjourned and on 29 March
withdrawn.

Solicitors: Bristows, Cooke & Carpmael (for the contractors); Treasury Solicitor.

F A Amies Esq Barrister.


[1963] 2 All ER 314
General and Finance Facilities Ltd v Cooks Cars (Romford) Ltd
TORTS; Other Torts: CIVIL PROCEDURE

COURT OF APPEAL
PEARSON AND DIPLOCK LJJ
25 MARCH, 1 APRIL 1963

Detinue Judgment Forms of judgment Judgment for return of chattel or recovery of its value and damages for its detention
Entitlement of plaintiff to separate assessments of value and damages Distinctions between remedies in conversion and in
detinue.

A successful plaintiff in an action in detinue who obtains judgment for the return of the detained chattel or recovery of its value
and damages for its detention is entitled to have assessed separately (i) the value of the chattel at the date of the assessment, and
(ii) the damages sustained by the plaintiff up to that date (see p 317, letter b, and p 320, letters d and e, post).
Rosenthal v Alderton & Sons Ltd ([1946] 1 All ER 583) applied.
Observations by Diplock LJ, on the forms of judgment in an action in detinue (see p 319, letter b, et seq, post).

Notes
As to form of judgment in detinue and its enforcement, see 38 Halsburys Laws (3rd Edn) 801803, paras 1340, 1345; and for
cases on the subject, see 43 Digest 530, 531, 654660, 670672.

Cases referred to in judgments


Chilton v Carrington (1855), 13 CB 730, 24 LJCP 78, 24 LTOS 258, 139 ER 735, subsequent proceedings, 16 CB 206, 24 LJCP
153, 21 Digest (Repl) 678, 1690.
Cohen v Roche [1927] 1 KB 169, 95 LJKB 945, 136 LT 219, 3 Digest (Repl) 11, 81.
Ellis v Stenning [1932] All ER Rep 597, [1932] 2 Ch 81, 101 LJCh 401, 147 LT 449, 21 Digest (Repl) 457, 1576.
Hymas v Ogden [1905] 1 KB 246, 74 LJKB 101, 91 LT 832, 21 Digest (Repl) 678, 1694.
Jones v Dowle (1841), 9 M & W 19, 1 Dowl NS 391, 11 LJEx 52, 152 ER 9, 3 Digest (Repl) 46, 324.
Reeve v Palmer (1858), 5 CBNS 84, 28 LJCP 168, 141 ER 33, 3 Digest (Repl) 115, 358.
Rosenthal v Alderton & Sons Ltd [1946] 1 All ER 583, [1946] 1 KB 374, 115 LJKB 215, 2nd Digest Supp.
Sachs v Miklos [1948] 1 All ER 67, [1948] 2 KB 23, [1948] LJR 1012, 3 Digest (Repl) 115, 361.
314
Scarth, Re (1874), 10 Ch App 234, 44 LJ Bcy 29, 31 LT 737, 4 Digest (Repl) 344, 3124.
Strand Electric and Engineering Co Ltd v Brisford Entertainments Ltd [1952] 1 All ER 796, [1952] 2 QB 246, 3rd Digest Supp.
Whiteley Ltd v Hilt [1918] 2 KB 808, 87 LJKB 1058, 119 LT 632, 3 Digest (Repl) 113, 345.

Interlocutory Appeal
This was an appeal by the successful plaintiffs in an action in detinue from an assessment of their damages by Master Clayton on
20 July 1962, under a judgment signed in pursuance of leave given by Master Diamond under RSC, Ord 14, the judgment being
signed on 13 April 1962, and in the terms stated at p 316, letter b, post.
The facts appears in the judgment of Pearson LJ.

John Deby for the plaintiffs.


Basil Webb for the defendants.

Cur adv vult

1 April 1963. The following judgments were delivered.

PEARSON LJ. This case relates to a mobile crane, which the plaintiff owners in November, 1958, let on a hire-purchase
agreement to a company called All Star Cars, Ltd. The cash price was 1,000; the hire-purchase price was 1,120; All Star Cars
Ltd paid a deposit of 300 and by September, 1959, had paid instalments amounting to 377 2s 6d, so that the remainder of the
hire-purchase price was 442 17s 6d. In breach of the hire-purchase agreement All Star Cars Ltd purported to sell the crane to a
man called Gooch, and afterwards in the year 1960 Gooch purported to sell it to Romford Scrap and Salvage Co Ltd for 200.
Then Romford Scrap and Salvage Co Ltd instructed the defendant repairers to do certain repairs to the crane, and delivered it to
them for that purpose. The repairs seem to have been carried out, and the defendants account dated 24 October 1960, and
addressed to Romford Scrap and Salvage Co Ltd was for 488, including 250 for a 5LW Gardiner engine. The plaintiffs, who
had lost trace of the crane, found where it was, and their managing director, Mr Saunders, went to see it in April, 1961. I will
refer later to his evidence about the condition of the crane.
On 8 May 1961, there was a letter from the plaintiffs solicitors to the defendants saying:

Dear Sirs, Re Thorneycroft crane OMF 347. I am instructed by my clients, General and Finance Facilities, Ltd. that
you are in possession of the above mobile crane. This crane is owned by my clients who let it on hire-purchase to All Star
Cars, Ltd. who later disposed of the crane in breach of their hire-purchase agreement. This crane subsequently came into
the possession of Romford Scrap and Salvage Co., Ltd., who of course had no rights to the crane at all, who sent it to you
for repairs. My clients wish to repossess the crane at present at your premises and request that you treat this letter as a
demand that you deliver up the said crane to them or their representative forthwith, failing which proceedings will be
instituted against you for its return.

The reply to that letter, dated 15 May was in these terms:

With reference to your letter concerning the above we must point out that we have a lien on this vehicle as already
noted to your client, General and Finance Facilities, Ltd., and when this is discharged we will willingly deliver up the
vehicle.

The lien, however, was not effective against the plaintiffs.


On 2 June 1961, the plaintiffs issued their writ in this action claiming: (i) The return of the crane or 2,000, its value; (ii)
damages for conversion; (iii) 315loss of hire for the crane for thirty-three weeks from 15 May 1961, until 12 January 1962, at
40 per week: 1,320. That was a grossly inflated and exorbitant claim. In this appeal the plaintiffs counsel has said that they
desire to recover only the 442 17s 6d which would make up the full hire-purchase price. On 1 September 1961, the plaintiffs
solicitors wrote a letter to the repairers solicitors, setting out a great many defects in the crane.
The plaintiffs made an application under RSC, Ord 14, for leave to sign final judgment, Master Diamond granted leave, and
the judgment signed on 13 April 1962, was

that the plaintiff do have return of the mobile crane OMF 347 or recover against the defendants its value and damages
to be assessed.

The wording of the judgment is not so clear as the wording of RSC, Ord 13, r 6(2), which as against a defendant who has failed
to enter an appearance entitles the plaintiff to

enter interlocutory judgment against him for the return of the goods or their value to be assessed, damages to be
assessed and costs.

Nevertheless this judgment must have the same effect of requiring both the value of the goods and the damages to be assessed,
because the value is unascertained until it is assessed. The difficulty which has arisen is that, in form at any rate, there has only
been an assessment of damages and not an assessment of the value of the goods. The order of Master Clayton, which is dated 20
July 1962, was, so far as material for the present purpose, in these terms:

Upon hearing counsel for the plaintiffs and defendants upon the plaintiffs application to assess the amount of
damages pursuant to the judgment of Master DIAMOND dated Apr. 13, 1962. It is ordered that the amount of damages be
assessed at 150.

We have been informed that after the learned master had made his assessment and the parties and their counsel had retired and
considered it, they returned to him and asked for some elucidation. There seems to be no written record of what was said, but the
recollection of defendants counsel, who was there at the time, is that plaintiffs then counsel asked whether the plaintiffs were to
have the crane returned to them and the learned master said No. We cannot be sure of the exact words, but it seems clear that
that was the effect of what was said.
Plaintiffs counsel sought to attack the assessment mainly on the following grounds: (i) that there should have been separate
figures for the value of the goods and for the damages, so as to enable the plaintiffs, if they preferred the return of the goods to
payment of the assessed value, to apply for a writ of delivery; (ii) that the assessment of the damages should have included a
reasonable hiring rent for the crane on the ground that the defendants had made use of it after the demand and refusal in May,
1961 (reliance being placed on Strand Electric and Engineering Co Ltd v Brisford Entertainments Ltd); (iii) that the assessment
of the damages should have included a sum in respect of the depreciation of the crane after the demand and refusal in May, 1961;
and (iv) that on any basis the assessment of 150 was too low.
I will deal first with grounds (ii), (iii) and (iv). Ground (ii) fails, because there was, so far as appears from the notes, no
evidence that the defendants had made any use of the crane after May, 1961, or, indeed, at any time. Ground (iii) is right in
principle, because, if the crane has depreciated after the wrongful refusal to return it, the plaintiffs have lost the difference
between the value of the crane at the time when it should have been returned and its diminished value at the date of assessment.
The plaintiffs should have (a) the return of the crane or payment of its value as assessed and (b) damages for the depreciation, if
any, which has occurred since May, 1961. We do not know for certain what the learned 316 masters figure of 150 includes.
There was a conflict of evidence as to the value of the crane at several dates. The defendants witnesses put the value at 350 in
May, 1961, and at 300 in July, 1962. The plaintiffs witness, their managing director, considered that the crane was worth only
scrap value in July, 1961, and early in 1962 and in April and May, 1962. He had said that it was in working order in April, 1961.
Without knowing exactly what the learned master had in mind, I can say only that it has not been established that he took any
wrong view as to value. It is not unfair to bear in mind that the plaintiffs in their statement of claim had claimed 2,000 for the
value of the crane and 1,320 for loss of hire. On the evidence that was an absurd claim.
On the other hand the first ground of appeal, namely, that there should have been separate figures for the value of the goods
and for the damages, is in my view correct. The judgment of 13 April 1962, being a judgment in detinue for the return of the
goods or their value, and for damages, did not divest the plaintiffs of their ownership: Rosenthal v Alderton & Sons Ltd ([1946] 1
All ER 583 at pp 584, 585; [1946] 1 KB 374 at pp 377, 378). They were entitled to have the value of the goods assessed, and still
are so entitled. After the assessment of the value, it will be open to them to apply for leave to issue a writ of delivery in order to
recover the crane itself and not its value (see RSC, Ord 42, r 1; Ord 48, r 1, and Appendix H, forms 10, 10A and 11; also the note
headed Enforcement of judgment in detinue in the notes under RSC, Ord 13, r 6, in the Annual Practice, 1963). Such an
application will not necessarily succeed, as the power to give leave for the issue of a writ of delivery is discretionary. Counsel for
the defendants referred to Whiteley Ltd v Hilt and Cohen v Roche. Whatever the prospects of success may be, the plaintiffs have
the right to make the application for a writ of delivery, if they think fit. They are also entitled to have an assessment of the value
of the crane, before they decide whether they will make the application or not.
Therefore, in my view it is necessary to allow this appeal, to set aside the existing assessment, and to remit the case to the
learned master for a new assessment to be made with separate figures for the value and the damages. I have come to this
conclusion with regret, as presumably the condition of the crane has further deteriorated and its present value is not likely to be
such as to justify continuance of litigation.

DIPLOCK LJ. This appeal raises a neat point as to the remedies available to a plaintiff who sues for the wrongful detention of
goods. The plaintiffs by a specially indorsed writ claimed the return of a mobile crane index No OMF 347 or its value and
damages for detaining the same. They pleaded their title to the crane and relied on a demand for its delivery up dated 8 May
1961. The prayer included an alternative claim for damages for conversion.
There are important distinctions between a cause of action in conversion and a cause of action in detinue. The former is a
single wrongful act and the cause of action accrues at the date of the conversion; the latter is a continuing cause of action which
accrues at the date of the wrongful refusal to deliver up the goods and continues until delivery up of the goods or judgment in the
action for detinue. It is important to keep this distinction clear, for confusion sometimes arises from the historical derivation of
the action of conversion from detinue sur bailment and detinue sur trover; of which one result is that the same facts may
constitute both detinue and conversion. Demand for delivery up of the chattel was an essential requirement of an action in
detinue and detinue lay only when at the time of the demand for delivery up of the chattel made by the person entitled to
possession the defendant was either in actual possession of it or was estopped from denying that he was still in possession. Thus,
if there had been an actual bailment of the chattel by the plaintiff to the defendant the latter was estopped from asserting that he
had wrongfully delivered the chattel to a third person 317 or had negligently lost it before demand for delivery up and the
plaintiff could sue in detinue notwithstanding that the defendant was not in actual possession of the chattel at the time of the
demand (see Jones v Dowle; Reeve v Palmer). Alternatively the plaintiff could sue in conversion for the actual wrongful delivery
of the chattel to the third person, though not for its loss. On the other hand an unqualified refusal to comply with a demand for
delivery up of a chattel made by the person entitled to possession may also amount to conversion, but only if the defendant at the
time of the refusal was in actual possession of the chattel. If he has wrongfully delivered it to a third person before the date of the
demand the prior wrongful delivery constitutes the conversion, not the subsequent refusal to comply with the demand (see Sachs
v Miklos ([1948] 1 All ER 67 at p 68; [1948] 2 KB 23 at p 29)). But even where, as in the present case, the chattel is in the actual
possession of the defendant at the time of the demand to deliver up possession, so that the plaintiff has alternative causes of
action in detinue or conversion based on the refusal to comply with that demand, he has a right to elect which cause of action he
will pursue (see Rosenthal v Alderton & Sons Ltd ([1946] 1 All ER 583 at p 585; [1946] 1 KB 374 at p 379)) and the remedies
available to him will differ according to his election.
The action in conversion is a purely personal action and results in a judgment for pecuniary damages only. The judgment is
for a single sum of which the measure is generally the value of the chattel at the date of the conversion together with any
consequential damage flowing from the conversion and not too remote to be recoverable in law. With great respect to the dictum
of Lord Goddard CJ, in Sachs v Miklos, this is not necessarily the same as the measure of damages for detinue, where the same
act constitutes detinue as well as conversion, although in many cases this will be so. This dictum was based on the headnote to
Rosenthal v Alderton, which, in my view, misrepresents the effect of the last paragraph of the actual judgment. The law is in my
view correctly stated in the current edition of Salmond on Torts at pp 287 and 288. A judgment for damages for conversion does
not, it is true, divest the plaintiff of his property in the chattel (see the analysis of the cases in Ellis v Stenning). The judgment,
however, does not entitle the plaintiff to the assistance of the court or the executive, videlicet the sheriff, in recovering possession
of the chattel.
On the other hand the action in detinue partakes of the nature of an action in rem in which the plaintiff seeks specific
restitution of his chattel. At common law it resulted in a judgment for delivery up of the chattel or payment of its value as
assessed, and for payment of damages for its detention. This, in effect, gave the defendant an option whether to return the chattel
or to pay its value, and if the plaintiff wished to insist on specific restitution of the chattel he had to have recourse to Chancery
(see Re Scarth, per Mellish LJ ((1874), 10 Ch App 234, at p 235)). The Common Law Procedure Act, 1854, s 78, a, gave the
court power to order delivery up of the chattel by the defendant without giving him the option to pay its value as assessed. Such
an order was enforceable by execution and if the chattel could not be found distraint could be had on the defendants lands and
goods until he delivered up the specific chattel, or, at the option of the plaintiff, distraint could be had of the defendants goods
for the assessed value of the chattel. This, in effect, where the court thought fit to make such an order, gave the plaintiff an option
to insist on specific restitution of his chattel if the defendant did not 318 deliver it up voluntarily; but this remedy was not
available unless and until the value of the chattel had been assessed (see Chilton v Carrington). This remedy continues to exist
under the modern law, but if the plaintiff does not wish to exercise his option to recover the assessed value of the chattel the
assessment of its value is no longer a condition precedent to an order for specific restitution (see Hymas v Ogden; RSC, Ord 48, r
1). In addition to an order for specific restitution of the chattel or for payment of its value as assessed the plaintiff was always
entitled to damages for wrongful detention of the chattel.
________________________________________
a See 18 Halsburys Statutes (2nd Edn) 447

In the result an action in detinue today may result in a judgment in one of three different forms: (i) for the value of the
chattel as assessed and damages for its detention; or (ii) for return of the chattel or recovery of its value as assessed and damages
for its detention; or (iii) for return of the chattel and damages for its detention. A judgment in the first form is appropriate where
the chattel is an ordinary article in commerce, for the court will not normally order specific restitution in such a case, where
damages are an adequate remedy (see Whiteley Ltd v Hilt ([1918] 2 KB 808 at pp 819, 824)). A judgment in this form deprives
the defendant of the option which he had under the old common law form of judgment of returning the chattel; but if he has failed
to do so by the time of the judgment the plaintiff, if he so elects, is entitled to a judgment in this form as of right (cf RSC, Ord 13,
r 6). In substance this is the same as the remedy in conversion although the sum recoverable, as I have indicated, may not be the
same as damages for conversion, for the cause of action in detinue is a continuing one up to the date of judgment and the value of
the chattel is assessed as at that date (see Rosenthal v Alderton & Sons Ltd). A final judgment in such a form is for a single sum
of money. A judgment in the second form gives to the defendant the option of returning the chattel, but it also gives to the
plaintiff the right to apply to the court to enforce specific restitution of the chattel by writ of delivery, or attachment or
sequestration as well as recovering damages for its detention by writ of fieri facias (RSC, Ord 42, r 6). This is an important right
and it is essential to its exercise that the judgment should specify separate amounts for the assessed value of the chattel and for
the damages for its detention, for if the plaintiff wishes to proceed by writ of delivery for which he can apply ex parte (Ord 48, r
1) he has the option of distraining for the assessed value of the chattel if the chattel itself is not recovered by the sheriff. He
would be deprived of this option if the value of the chattel were not separately assessed. A judgment in the third form is unusual
but can be given (see Hymas v Ogden). Under it the only pecuniary sum recoverable is damages for detention of the chattel. Its
value need not be assessed and the plaintiff can obtain specific restitution of the chattel only by writ of delivery, attachment or
sequestration. He has no option under the writ of delivery to distrain for the value of the chattel.
In the ordinary way where an action goes to trial the issues of liability, assessment of value of the chattel, and damages for
its detention, are dealt with at the hearing, and final judgment in one or other of the above forms is entered. In the present case,
however, proceedings were brought under RSC, Ord 14, and on 13 April 1962, the plaintiffs, on leave obtained from Master
Diamond, signed final judgment

that the plaintiff do have return of the mobile crane OMF 347 or recover against the defendant its value and damages
to be assessed.

No appeal was brought against this judgment. It is a final judgment granting the plaintiffs relief in the second form referred to
above and entitling the plaintiffs, 319on the assessment of the value of the chattel and of the damages for its detention, to select
and enforce their appropriate remedies by execution.
The assessment of the value of the chattel and of the damages for its detention was referred to Master Clayton under RSC,
Ord 36B, r 2 and r 6. It thereupon became his duty to assess both these amounts separately at the date of the assessment (see
RSC, Ord 36B, r 7) and to certify each amount (RSC, Ord 36B, r 3). He did not do this; what he did instead was to order that
the amount of damages be assessed at 150. He made no separate assessment of the value of the crane. We are informed that
after the hearing counsel returned and asked the learned master whether he intended that the plaintiffs should have their crane
back and that he replied No. It would seem, therefore, that the sum of 150 was intended to include both the value of the crane
and damages for its detention, and I agree with Pearson LJ that it was open to him on the evidence to come to that conclusion.
But an order assessing a single sum of money would be justifiable only under a judgment in the first form discussed above,
whereas the plaintiffs entitlement to a judgment in the second form discussed above was already res judicata under the earlier
judgment of Master Diamond. Master Clayton had no jurisdiction to deprive the plaintiffs of that and the consequent options
which a judgment in that form secures to a plaintiff. His sole jurisdiction and his duty under Master Diamonds order was to
assess separately both the value of the crane and the damages for its detention and to certify these two amounts.
Although I share the regret of Pearson LJ that this litigation should be further protracted, the matter is one of substance, not
merely of form, for Master Claytons order prevents the plaintiffs from exercising an option to which they are in law entitled
under the judgment of Master Diamond, videlicet of choosing whether to seek specific restitution of the crane or to recover its
value. The matter must accordingly go back to Master Clayton with a direction to assess separately (i) the value of the crane at
the date of the assessment and (ii) the damages sustained by the plaintiffs by its detention up to the date of the assessment.
Appeal allowed. Case remitted accordingly.

Solicitors: Adam Shale & Garle (for the plaintiffs); Clive Lipman, Ilford, Essex (for the defendants).

Henry Summerfield Esq Barrister.


320
[1963] 2 All ER 321

Bernays v Prosser
AGRICULTURE: LANDLORD AND TENANT; Tenancies

COURT OF APPEAL
LORD DENNING MR, DANCKWERTS AND DAVIES LJJ
26 MARCH 1963

Agriculture Agricultural holding Tenancy Agreement to let land for period of one year Whether land let for an interest
less than a tenancy from year to year Agricultural Holdings Act, 1948 (11 & 12 Geo 6 c 63), s 2(1).

A tenancy of agricultural land for one year certain is an interest less than a tenancy from year to year within the meaning of s 2(1)
of the Agricultural Holdings Act, 1948 a, and so, by virtue of s 2(1), the tenant has the protection accorded to a contract of
tenancy by that Act (see p 323, letters d and g, and p 324, letters d to f, post).
________________________________________
a Section 2(1), so far as material, provides: where under an agreement made on or after 1 March 1948, any land is let to a person for use
as agricultural land for an interest less than a tenancy from year to year then, unless the letting was approved by the minister before
the agreement was entered into, the agreement shall take effect, with the necessary modifications, as if it were an agreement for the letting
of the land for a tenancy from year to year

Gladstone v Bower ([1960] 3 All ER 353) distinguished.


Dictum of Pearson LJ, in Lower v Sorrell ([1962] 3 All ER at p 1085) applied.
Appeal dismissed.

Notes
As to the minimum term that may be created in respect of an agricultural holding, see 1 Halsburys Laws (2nd Edn) 256, para
556.
For the Agricultural Holdings Act, 1948, s 2, see 28 Halsburys Statutes (2nd Edn) 29.

Cases referred to in judgments


Doe d Clark v Smaridge (1845), 7 QB 957, 14 LJQB 327, 6 LTOS 172, 115 ER 748, 31 Digest (Repl) 486, 6114.
Gandy v Jubber (1865), 9 B & S 15, 5 B & S 485, 29 VP 645, 122 ER 911, 31 Digest (Repl) 382, 5098.
Gladstone v Bower [1960] 3 All ER 353, [1960] 2 QB 384, [1960] 3 WLR 575, 3rd Digest Supp.
Lower v Sorrell [1962] 3 All ER 1074, [1963] 2 WLR 1.
Wright v Tracey (1874), IR 8 CL 478.

Appeal
This was an appeal by the landlord from a judgment of His Honour Deputy Judge S J Havard Evans at Pontypridd County Court
on 21 January 1963, dismissing her action for the possession of a smallholding of approximately four acres of land and a building
which had been let to the tenant for a period of one year. The landlord claimed that the letting was not a contract of tenancy
within the provisions of the Agricultural Holdings Act, 1948.
The cases noted belowb were cited during the argument in addition to those referred to in the judgments.
________________________________________
b Reed v Dawson [1954] 3 All ER 498, [1955] 1 QB 214, Rutherford v Maurer, [1961] 2 All ER 775, [1962] 1 QB 16

Peter Langdon-Davies for the landlord.


W N Francis for the tenant.

26 March 1963. The following judgments were delivered.

LORD DENNING MR. This appeal raises a short but interesting point under the Agricultural Holdings Act, 1948. For a great
number of years the tenant of an agricultural holding has been protected by statute. He cannot be turned out except on having
twelve months notice. It is quite clear that this protection extends to a tenant who has had an agricultural holding for a fixed
term of two years or more (see s 3 of the Act of 1948). It is equally clear that this protection extends to a tenant who has had only
a six months term or a nine months term or even a 364-day term (see s 2 of the Act of 1948). But tenants who have a fixed term
of between one and two years have no such 321 protection. It has been held by this court that there is a lamentable gap in the
statute. A tenant of an agricultural holding for a fixed period between one and two years is not entitled to the protection of the
Act of 1948 at all. He has to go out of his holding at the end of his term without any protection: see Gladstone v Bower. The
gap has not been closed by Parliament even to this day, although we are told there is an agricultural bill before Parliament at the
moment. Accepting that to be the law, we have to consider here what is the position of a tenant for a fixed term of one year. Is
such a tenant entitled to protection under the Agricultural Holdings Act? The agreement in this case is quite simple:

Memorandum of agreement made this first day of April, 1961, between Mr. C. Prosser of the one part, and the
Woodland Management Association, acting as woodland agents for and on behalf of the owners of the Caerphilly
Woodlands, of the other part. Whereby it is agreed that the smallholding of Tyn-y-Parc including approximately four acres
of land and the building known as the Beast House, shall be leased to Mr. Prosser for a period of one year to Mar. 31,
1962, on payment of a rental of 5s. per week (i.e. 13 for the complete period). This agreement is subject to review at the
end of this period and may be renewed at the discretion of the owners.
The then owners transferred to the present plaintiff their interest in the property and she claims possession. Owing to the gap
which has been found in the statute, the tenant in this case, in order to be protected by the Agricultural Holdings Act, 1948, must
bring himself within s 2(1) of the Act; that is to say, he must show that a tenancy for one year certain is an interest less than a
tenancy from year to year.
What, then, is the interest of a tenant from year to year? At one time the impression prevailed in Westminster Hall that, even
at common law, a tenancy from year to year was a tenancy for two years at least. It was supposed that it could not be determined
at the end of the first year but at the end of the second year at the earliest. This impression seems to have been derived from a
passage in Bacons Abridgmentc which was accepted as correct by a strong Court of Exchequer Chamber when they prepared
their undelivered judgment in Gandy v Jubber ((1865), 9 B & S 15 at p 18). But on examining the passage in Bacons
Abridgment, it seems to me that that refers only to a case where a man makes a lease for a year and thereafter from year to
year. On such words the tenancy clearly continues beyond the first year and must go on for two years at the least, because it
cannot be determined earlier. When there is a simple lease from year to year without more said, or a simple holding over on a
tenancy from year to year, it is now well established that it can be determined by six months notice given during the first year
to expire at the end of the first year: see Doe d Clarke v Smaridge. True it is that the Agricultural Holdings Act, 1948, now
requires twelve months notice in the case of an agricultural holding, so that it cannot be determined at the end of the first year,
but only at the end of the second year at the earliest. But that is special to agricultural holdings. It does not affect the quality at
common law of a tenancy from year to year. And it is the common law tenancy which provides the yardstick. It is the measure
by which to judge whether an interest is less than a tenancy from year to year. This court so held in Gladstone v Bower and we
are bound by it.
________________________________________
c 7th Edn, Vol 4, pp 838, 839

Seeing that a common law tenancy from year to year can be determined at the end of the first year, it is said that a tenant
for one year certain has a tenancy which is equal to, and not less than, a tenancy from year to year, because each is only
certain to last for one year. This contention derives considerable support from an Irish case in 1874, Wright v Tracey, when the
Court of Exchequer Chamber in Ireland had to consider similar words in the Irish Landlord and 322 Tenant Act, 1870. The
majority of the court, including Palles CB held that a term of one year certain was not less than a tenancy from year to year. I
cannot agree with this view. It seems to me absurd to say that a tenancy for one year certain is equal to a tenancy from year to
year. It is not equal to it: and, as it is not equal to it, it must be less than it. A tenancy from year to year is not only a tenancy
for one year certain. It is something more, because the tenant, unless a notice to quit has been given, has a right to stay on after
the end of the year and so on from year to year. Counsel for the tenant gave a vivid illustration. Take the first day after the first
year, the 366th day. If the tenancy is for one year certain and the tenant stays on into the 366th day, he is a trespasser. But if
the tenancy is from year to year, and he stays on into the 366th day, he is not a trespasser but a man who has an indefeasible
right to a further years tenancy unless there has been a notice to quit. This view of a tenancy from year to year is supported by
the recent decision of this court in Lower v Sorrell. If the report is carefully examined, it is quite apparent that it was essential to
decide that, when a tenancy is for one year certain, it is a tenancy which is less than a tenancy from year to year, and the tenant is
entitled to the protection of the Agricultural Holdings Act, 1948. The reasons given by Pearson LJ in that case ([1962] 3 All ER
at p 1085) are convincing. I hold, therefore, that a tenancy for one year certain is less than a tenancy from year to year, and that
the tenant has the protection given by s 2(1) of the Agricultural Holdings Act, 1948. He is entitled to stay there until he has
twelve months notice to quit. So I find myself in agreement with the judgment of the county court judge.
It was suggested to us that the recent case of Lower v Sorrell is inconsistent with the decision of this court in Gladstone v
Bower. I do not think that it is. A tenancy for a fixed term of eighteen months may be greater than a tenancy from year to year,
whereas a fixed term of twelve months is less than it. I confess, however, that I find Gladstone v Bower a difficult case. It is
clearly contrary to anything that Parliament intended. I hope that it may soon be remedied.
I would, therefore, dismiss the appeal.

DANCKWERTS LJ. The point which we have to decide is really quite a short one. It depends on the provisions of s 2(1) of the
Agricultural Holdings Act, 1948. We have here a case where there has been produced a tenancy for one year certain, and the
question is whether that interest is an interest less than a tenancy from year to year, as expressed in the subsection to which I have
referred. Unrestricted by authority, I should have had no hesitation in saying that a tenancy from year to year, which amounts to a
tenancy for a year certain plus a further interest to continue the tenancy year after year unless it has been brought to an end by
notice of a proper kind, must be a greater interest than a tenancy for one year certain, and that a tenancy for one year certain must
be an interest less than a tenancy from year to year. It is said that we are prevented from deciding in that way by, at any rate, two
decisions. There is the Irish case of Wright v Tracey, decided in 1874 on the Irish Landlord and Tenant Act, 1870. It is to be
observed that, apart from the question of whether the decision is strictly binding on us, there are certain features which to my
mind make it clear that we should not apply that case. In the first place, it was a decision by a majority of four judges to three,
that is by the odd judge in seven, and the reasoning of the minority who gave judgments appeals to me much more than that of
the majority, notwithstanding that it includes Palles CB and Whiteside CJ. There is another point. The language of the two
statutes is not really quite the same. Section 69(2) of the Irish Act of 1870 provides as follows:

Where any tenancy at will, or less than a tenancy from year to year, is created by a landlord after the passing of this
Act ,
323

then certain things follow. In s 2(1) of the English Act which we have to construe, the language is:

where under an agreement made on or after Mar. 1, 1948, any land is let to a person for use as agricultural land for
an interest less than a tenancy from year to year

Those words for an interest seem to me to strengthen very much the case for the result which I think is the right one. An
interest for a tenancy from year to year is, it seems to me, necessarily greater than an interest which must come to a definite end
at the end of the twelve months, because, in the case of a tenancy from year to year, the tenant not only has an interest for one
year certain, but he has an interest which will continue year after year unless it is brought to an end by notice at the end of the
first year, or any other year, of course, later on.
It seems to me that Gladstone v Bower, which dealt with a tenancy for eighteen months certain, is of a different kind and in
no way compels us to reach a different result from that which I have stated in the present case; in other words, I think that it is
plainly distinguishable.
In my view, the right result is expressed in the words of Pearson LJ, in Lower v Sorrell ([1962] 3 All ER at p 1085), which
expresses exactly the view which I take in the present case. What he says is:
A tenancy from year to year is essentially a tenancy for an indefinite period which runs on from year to year until it is
stopped by a notice to quit expiring at the end of some year, which may be the first or any later year but very often is a year
much later than the first. That is obviously a greater interest than a tenancy for one year, because a tenancy which may last
for more than a year, and will do so unless a notice to quit is served at an early stage, has a larger content than a tenancy
which cannot last for more than a year, unless extended by a further agreement.

That seems to me to state the true proposition with great clarity and I adopt Pearson LJs words with respect.
I, therefore, also agree that the appeal should be dismissed.

DAVIES LJ. I agree with both judgments and do not wish to add anything.

Appeal dismissed. Leave to appeal to the House of Lords refused.

Solicitors: Robbins, Olivey & Lake agents for Burges, Salmon & Co, Bristol (for the plaintiff); Elfyn David & Hamblen, Cardiff
(for the defendant).

F Guttman Esq Barrister.


324
[1963] 2 All ER 325

Bohnel v Bohnel (No 2)


FAMILY; Divorce

PROBATE, DIVORCE AND ADMIRALTY DIVISION


CAIRNS J
11 MARCH 1963

Divorce Estoppel Desertion Just cause for leaving Previous cruelty charge dismissed Facts relied on as giving just
cause for leaving substantially same as those relied on to support charge of cruelty Issues not the same No estoppel.

The parties were married in 1943 and in December, 1958, the wife left the husband. In January, 1959, she presented a petition for
divorce on the ground of cruelty, alleging that the husband had constantly desired and shown his desire to dress and behave like a
woman. The suit was undefended and the court found that, although the husband had a sexual abnormality which had caused
injury to the wifes health, and although his conduct might have justified the wife in leaving, yet it did not amount to cruelty,
since he could not help his abnormality and had tried unsuccessfully to conceal it from her. The wifes petition was accordingly
dismissed. In 1961 the husband presented a petition for divorce on the ground of the wifes desertion. The wife by her answer
alleged that she had just cause for leaving and set out substantially the same facts which she had previously relied on as
constituting cruelty. The husband applied to strike out the wifes answer.

Held The wifes answer would not be struck out, since the issue of just cause had not been decided in the cruelty suit.
Warren v Warren ([1962] 3 All ER 1031) and Fisher v Fisher ([1959] 3 All ER 131) considered.

Notes
As to estoppel as a bar in divorce, see 12 Halsburys Laws (3rd Edn) 277, para 533, notes (s), (t), 294, para 581; and for cases on
the subject, see 27 Digest (Repl) 375, 376, 30893098.

Cases referred to in judgment


Bright v Bright [1953] 2 All ER 939, [1954] P 270, [1953] 3 WLR 659, 117 JP 529, 3rd Digest Supp.
Dixon v Dixon [1953] 1 All ER 910, [1953] P 103, [1953] 2 WLR 748, 3rd Digest Supp.
Fisher v Fisher [1959] 3 All ER 131, [1960] P 36, [1959] 3 WLR 471, 3rd Digest Supp.
Glenister v Glenister [1945] 1 All ER 513, [1945] P 30, 114 LJP 69, 172 LT 250, 109 JP 194, 27 Digest (Repl) 367, 3040.
Thompson v Thompson [1957] 1 All ER 161, [1957] P 19, [1957] 2 WLR 138, 3rd Digest Supp.
Warren v Warren [1962] 3 All ER 1031, [1962] 1 WLR 1310.
Winnan v Winnan [1948] 2 All ER 862, [1949] P 174, 27 Digest (Repl) 375, 3098.

Summons adjourned into open court


In this case the husband appealed against an order of the Reading District Registrar refusing to strike out the wifes answer.
The parties were married in 1943 and there were two children of the marriage. In 1959 the wife presented a petition for
divorce on the ground of the husbands cruelty, the particulars of cruelty being that the husband had desired and shown his desire
to behave like a woman. The suit was undefended and came before His Honour Judge Armstrong sitting as a special
commissioner in divorce at Bournemouth on 18 August 1959, when the petition was dismissed. The wife appealed and the Court
of Appeal dismissed her appeal ([1960] 2 All ER 442). In February, 1962, the husband presented a petition for divorce alleging
that the wife had deserted him on 15 December 1958. By her answer the wife admitted that she left the matrimonial home on 15
December 1958, but denied desertion and pleaded that she had just cause for leaving. In support of her allegation of just cause,
the wife set out substantially the same facts as those on which she had relied to 325 support her charge of cruelty. The husband
applied to the district registrar at Reading, Mr Registrar Chilton, to strike out the wifes answer on the ground that the wife was
estopped from alleging again in a new guise the conduct which had been found not to constitute cruelty. On 7 February 1963, the
registrar dismissed the application and the husband now appealed.

Brian T Neill and R L C Hartley for the husband.


W Kee for the wife.

11 March 1963. The following judgment was delivered.

CAIRNS J. The cruelty alleged in the wifes suit was as follows ([1960] 2 All ER at p 443):
The nature of the wifes case is that since the year 1945 the [husband] has constantly desired and shown his desire to
dress and behave like a woman. In 1945 the [husband] possessed a suitcase filled with womens clothes. In 1948 he
acquired a pair of womens shoes. In 1955 at the matrimonial home the [wife] discovered the [husband] attempting to
conceal womens clothes in and under a wardrobe. On this occasion the [husband] possessed unguents and powder more
usually possessed by women than men. In 1958 at the matrimonial home the [wife] found the [husband] dressed like a
woman and the [husband] told the [wife] he wished to live with another man. The [husband] is repelled by sexual
intercourse.

At the hearing of the wifes suit, which was undefended, the wife gave evidence that the husband had behaved in that way. The
learned commissioner who tried the suit did not accept all her evidence but he did find that the husband had this sexual
abnormality of wishing to dress and behave like a woman, that the wife became aware of it and that in 1955 she had a nervous
breakdown largely because of it. But the commissioner rejected the charge of cruelty because he found that the husband could
not help his abnormality, that he did not flaunt it before the wife, but rather tried unsuccessfully to conceal it from her, and that
the real trouble between the two was that after he developed this tendency his wife had little sexual attraction for him. So the
commissioner dismissed the wifes petition.
She appealed to the Court of Appeal and her appeal was dismissed. Willmer LJ giving the leading judgment, held that the
commissioner had applied the right tests and at the end of his judgment he stressed the fact that the wife had remained with the
husband for eight months after the last incident complained of. Harman LJ and Sellers LJ agreed, though both expressed some
dissatisfaction with the evidence, and Sellers LJ said that he did not feel at all convinced that the case which the wife wished to
put forward was fully advanced. She was not then represented by counsel who appeared for her before me.
In the present suit the wife sets up substantially the same conduct on the husbands part that she relied on before, though her
allegations are somewhat expanded and modified, probably to bring them into line with the evidence that she gave and with the
commissioners findings of fact. She has added some further matters, such as sullen and morose behaviour.
Counsel for the husband puts his case in this way. The wife can only justify her leaving the matrimonial home if she can
show that the husband ill-treated her. If ill-treatment causes injury to health, then it is cruelty. Here it was found that the
husbands conduct did cause injury to health and an apprehension of further injury. Her petition failed because the commissioner
found in effect that there was no ill-treatment. Therefore, she cannot set up the same conduct again and say that it was ill-
treatment. Further, the Court of Appeal have said that, because the wife stayed with the husband for eight months after
discovering in 1958 that the husband still had his abnormal tendencies, she cannot have been unduly disturbed by that discovery
and she, therefore, ought not now to be allowed to say that she was so disturbed by them as to be justified in leaving.
326
The authority chiefly relied on by counsel for the husband is Warren v Warren. There the wife was alleging constructive
desertion after failing in a charge of cruelty and Scarman J ordered part of her particulars of expulsive conduct to be struck out.
He said ([1962] 3 All ER at p 1032):

The question of res judicata in divorce proceedings has been so frequently canvassed of late years that I hesitate to add
to its discussion. I doubt if the law on the topic has yet reached finality, but it is, I think, certain that where a charge of a
matrimonial offence has failed in divorce proceedings and the party who made the charge endeavours to repeat it in a
subsequent divorce suit, the court will not, as a rule, allow him to do so.

Then ([1962] 3 All ER at p 1033):

but if the wife bases her charge of desertion on a case of cruelty, which has been fully considered and unequivocally
rejected in previous divorce proceedings, it seems to me that she is estopped.

The situation in that case was, however, quite different from that in the present case, as is clear from this passage in the judgment
of Scarman J ([1962] 3 All ER at p 1033):

It is conceded, and it is plain, that the greater part of the conduct now alleged as ending the matrimonial life was the
subject of a decision by MARSHALL, J., in January, 1960. I have read his judgment. Some of the wifes allegations he
accepted, some he rejected. He reviewed the matrimonial history as a whole, balancing the evidence of the husband
against that of the wife, and assessing the value and significance of each. He concluded that such evidence of violence,
abuse, and quarrels as he accepted did not approach legal cruelty. He asked himself what was the explanation of the failure
of this marriage and his answer was as follows: In my judgment, it is a classic case of incompatibility, a wife quick in
temper, quick to assert what she wants, and who soon discovered that she was able to get her way with a husband who, I
find, was not quick tempered, who was prepared to put up with a good deal, but who from time to time found his control
breaking down.
In my respectful opinion, the judge dealt fully and clearly with the matrimonial life as a whole and at the end of a
careful judgment not only rejected the wifes case of cruelty but essayed an explanation of the collapse of the marriage
quite inconsistent with the plea now to be found in her answer. [I stress those last words.] There is, of course, no
necessary identity of subject-matter between a charge of cruelty and a charge by the same spouse made on the same facts of
expulsive conduct, or just cause for withdrawal from cohabitation. But if, on a comparison of the two charges and an
analysis of the judgment of the court dismissing the cruelty charges, it is plain that in the subsequent proceedings the
spouse alleging expulsive conduct or just cause is in truth putting forward her failed case of cruelty as being expulsive or
just cause because it was cruel, she ought, in my opinion and having regard to what I believe to be the principles in these
matters, to be estopped from so doing.

Now in the present case the learned commissioner did not reject the wifes case as to the cause of the breakdown of the
marriage. Broadly he accepted it. Indeed he said at the end of the judgment:

In my view I have reluctantly come to the conclusion that it does not amount to legal cruelty. It might be that various
matters fully justified the wife, as she did, in leaving the husband in December of last year.
327

That sentence is no doubt obiter, but at least it shows that the learned commissioner was not conscious that any part of his
decision was inconsistent with the view that the wife had just cause for leaving.
In the judgment in Warren v Warren there is no express reference to Fisher v Fisher, but it is clear from the last passage that
I read from his judgment that Scarman J must have had that case in mind. In Fisher v Fisher the husband petitioned in
September, 1954, for a decree of nullity on the ground that the marriage had not been consummated. The wife by her answer
denied the husbands allegation and cross-prayed for dissolution of the marriage on the ground of her husbands cruelty. The
commissionera found that the marriage had been consummated, and that the allegations made by the wife about the husbands
conduct were proved, but that they did not amount to cruelty; and, therefore, both the husbands petition and wifes cross-prayer
were dismissed. The wife left the matrimonial home in October, 1954, and had never returned. In November, 1957, the husband
petitioned for divorce on the ground of his wifes desertion. The wife denied the desertion and cross-petitioned for a divorce on
the ground of constructive desertion. In her answer she relied on the allegations which she had raised in the previous proceedings
and on some further allegations of expulsive conduct which were not raised in the previous proceedings. It was held that there
could be no estoppel per rem judicatam where the subjectmatter of litigation in previous proceedings was different, and, since
constructive desertion was a fresh issue not determined in the previous proceedings, the wife was entitled to have her allegations
heard and assessed in the later proceedings. Thompson v Thompson and Winnan v Winnan were applied and a dictum of Willmer
LJ, in Bright v Bright ([1953] 2 All ER at p 949; [1954] P at p 288) was disapproved. The decision of the special commissioner
was reversed (I may say in parenthesis that I notice in the report of the judgment of Scarman J, in Warren v Warren ([1962] 3 All
ER at p 1032) that the learned judge refers to Bright v Bright and, if it had been necessary for me to consider that part of his
judgment in the present case, I should have had to consider whether, having regard to the disapproval of that particular dictum by
the Court of Appeal in Fisher v Fisher, that part of the judgment of Scarman J might need further consideration but in the view
that I take of the present case that does not arise to be decided).
________________________________________
a Sir Reginald Sharpe QC

In Fisher v Fisher the short judgment of Harman LJ puts the matter very succinctly and in a way which is applicable to the
present case. He says ([1959] 3 All ER at p 138; [1957] P at p 51):

It seems to me that there has been a confusion of terms. That is a very potent source of deception and it arises here
again. It can be truly said that this wife has made unfounded charges of cruelty against her husband; that may mean either
that the court rejected the facts alleged or that, though it accepted them, it decided they did not amount to cruelty as that
word is understood in the Divorce Court.
The second is the present case. It follows that these charges of cruelty may not be repeated as such. That is a very
different matter from saying that those same facts did not amount to matters relevant to an issue of constructive desertion
which, as my Lord has said, has never been tried at all.
It would indeed be an astonishing result of the confusion in which words can result if the wife, on an issue which has
never been before the court, were compelled to be silent about matters which the court has already declared that she has
truly stated. That she should be kept silent even from true words on a new issue is an astonishing plea. With all respect to
the very 328 energetic argument which we have heard, I think that this court would be landed in an absurdity if we
prevented the wife from telling the truth about her case.

He agreed that the appeal should be allowed. In my view, these principles as enunciated by Harman LJ, apply even more forcibly
where the wife is setting up, not a case of constructive desertion, but merely a defence of just cause for leaving. I do not think
that Fisher v Fisher is to be interpreted as meaning that there can never be an estoppel unless the causes of action in both suits are
precisely the same. The matter is put very clearly in Thompson v Thompson by Morris LJ, as follows ([1957] 1 All ER at p 173;
[1957] P at p 42):

In ordinary litigation a party may be defeated by a plea of res judicata. If in a suit it can be shown that the precise
dispute between the parties has already been adjudicated on in an earlier suit between the same parties where the same
dispute was also in issue as a basis for reliefthen there may be a successful plea of res judicata. An estoppel may also
arise, even if in successive suits between the same parties the causes of action are different, if it can be plainly shown that a
precise point was a direct issue in the first suit and was decided. These principles are, I think, of general application and
may apply in matrimonial proceedings.

It was, I think, on the basis of the second form of estoppel mentioned in that quotation that Scarman J decided as he did in
Warren v Warren.
In the present case the precise point now raised by the wife in her answer has not been decided. It would be wrong to
suggest, and was not suggested by counsel for the wife, that it has been effectively decided in the wifes favour by the remarks
made by the learned commissioner at the end of his judgment.
I am of the opinion that the analysis of counsel for the husband is defective for this reason. In order to justify the wife in
leaving her husband she does not necessarily have to show that her husband ill-treated her. It has never been held that only
conduct aimed at the wife would give her good cause for leaving her husband. For instance, reasonable belief in the other
spouses adultery may amount to just cause for leaving: see Glenister v Glenister.
As to the argument based on the wifes remaining for eight months in the matrimonial home after the final incident, I do not
think that the words of Willmer LJ ([1960] 2 All ER at p 447), are to be read as meaning that her mind was quite unaffected by
the incident. What he was considering was whether the effect was so grave as to cause injury or apprehension of injury to health.
I see no reason why the wife should be debarred by this passage in the judgment from saying that she was justified in leaving the
husband because of this incident, taken with what had gone before and with the morose and unkind behaviour which she now
says followed it and which was not alleged in the previous proceedings, perhaps because it was not behaviour of such a nature as
to be likely to injure health.
If I had come to the conclusion that, as between the parties, the wife should be debarred from making these allegations in her
answer, I should have had to go on to consider whether the court could refuse to entertain them consistently with its statutory
duty to be satisfied of desertion before pronouncing a decree on that ground and I should further have had to decide whether the
matter was so clear as to justify striking out the answer or part of it rather than leaving a plea of estoppel raised in the reply to be
considered by the judge of the hearing: see Dixon v Dixon. But these points do not arise because in my view the 329 husband
has failed to establish that the matters relied on by the wife are res judicatae at all. The appeal will, therefore, be dismissed with
costs.

Appeal dismissed.

Solicitors: Robbins, Olivey & Lake agents for Harris & Cartwright, Slough (for the husband); Howlett & Clarke (for the wife).

A T Hoolahan Esq Barrister.


[1963] 2 All ER 330

Crystall v Crystall
FAMILY; Ancillary Finance and Property

COURT OF APPEAL
WILLMER, HARMAN AND DAVIES LJJ
29, 31 OCTOBER 1962

Husband and Wife Property Summary proceedings Jurisdiction Chose in action Loan by wife to husband Married
Womens Property Act, 1882 (45 & 46 Vict c 75), s 17 Matrimonial Causes (Property and Maintenance) Act, 1958 (6 & 7 Eliz 2
c 35) s 7.

Costs Appeal to Court of Appeal Appeal as to costs without leave of the trial judge Discretion of trial judge Respondent
assisted person whose contribution was assessed at nil Jurisdiction of Court of Appeal Means of husband to be regarded
Supreme Court of Judicature (Consolidation) Act, 1925 (15 & 16 Geo 5 c 49), s 31(1) (h) Legal Aid and Advice Act, 1949 (12
& 13 Geo 6 c 51), s 2(2)(e).

In proceedings under s 17 of the Married Womens Property Act, 1882, a wife put forward claims falling into five categories, of
which the first was a claim to an interest in the proceeds of sale of the matrimonial home, and the fourth was a claim to 90
which she had lent to the husband towards the legal costs of the purchase of the matrimonial home. The registrar found that the
acquisition of the home was a joint venture to which the wife contributed, but that the 90 was a loan by her. He awarded her
240 (viz, 150 in respect of the home, and the 90), and ordered the husband to pay seventy-five guineas costs. The husband
was an assisted person, whose contribution was assessed at nil. The husband had no capital assets and at the date of the hearing
of the appeal was earning 12 weekly. On appeal, leave to appeal under s 31(1)(h) of the Supreme Court of Judicature
(Consolidation) Act, 1925, not having been allowed

Held (i) A claim by one spouse against the other for repayment of money lent was not a matter which was within the
jurisdiction of the court under s 17 of the Married Womens Property Act, 1882, for the money lent was not property in dispute
within the section; accordingly, payment of the 90 could not be awarded in these proceedings under s 17, as distinct from
proceedings to enforce the contractual obligation to pay (see p 332, letter b, and p 334, letters a and g, post).
(ii) There was a right of appeal without the leave of the trial judge, since the appeal was not an appeal as to costs only within
s 31(1)(h) of the Supreme Court of Judicature (Consolidation) Act, 1925 (see p 333, letter b, and p 334, letters c and g, post).
Harris v Aaron ((1877), 4 ChD 749) distinguished.
(iii) In exercising his discretion as to costs the registrar was bound by s 2(2)(e) of the Legal Aid and Advice Act, 1949, to
have regard to the means of all parties, including the husbands means; in the circumstances no order as to costs of the
proceedings before the registrar should have been made against the husband (see p 333, letter i, and p 334, letter e, post).
Per Willmer LJ (Davies LJ, concurring): whatever one might think of the conduct of an assisted party it would still not be
right [in view of s 2(2)(e) of the Legal Aid and Advice Act, 1949] to make an order for costs against him which would be
unreasonable having regard to his means (see p 333, letter g, post, and p 334, letter g, post).
Appeal allowed in part.
330

Notes
As to the scope of orders under the Married Womens Property Act, 1882, s 17, see 19 Halsburys Laws (3rd Edn) 900, 901, para
1492; and for cases on the subject, see 27 Digest (Repl) 263265, 21192133.
As to the right of appeal on costs to the Court of Appeal, see 30 Halsburys Laws (3rd Edn) 423, para 798; and for cases on
the subject, see Digest (Practice vol) 924, 925, 46714686.
As to awards of costs against assisted person, see 30 Halsburys Laws (3rd Edn) 502, 503, para 933.
For the Married Womens Property Act, 1882, s 17, see 11 Halsburys Statutes (2nd Edn) 804, 805; and for the Matrimonial
Causes (Property and Maintenance) Act, 1958, s 7, see 38 ibid, 450, 451.
For the Supreme Court of Judicature (Consolidation) Act, 1925, s 31(1)(h), see 5 Halsburys Statutes (2nd Edn) 359; and for
the Legal Aid and Advice Act, 1949, s 2(2)(e), see 18 Halsburys Statutes (2nd Edn) 535.

Cases referred to in judgment


Forbes-Smith v Forbes-Smith and Chadwick [1901] P 258, 70 LJP 61, 84 LT 789, 27 Digest (Repl) 572, 5285.
Griggs v Petts [1939] 4 All ER 39, [1940] 1 KB 198, 161 LT 317, Digest Supp.
Harris v Aaron (1877), 4 ChD 749, 46 LJCh 488, 36 LT 43.

Appeal
The husband appealed against an order of Mr Registar Long made on 15 December 1961, whereby he allowed in part a claim by
the wife under s 17 of the Married Womens Property Act, 1882. The husband sought to have the order of the registrar discharged
and a declaration made that the wife had no interest in the property in respect of which the claim was made. His grounds of
appeal were: (i) that the registrar was wrong in all the circumstances of the case in finding that the wife had any interest in the
house at 28, Strafford Avenue, Barkingside, which had been the matrimonial home and which had been sold for a sum of which
the balance after discharge of the mortgage was about 707; (ii) that he was wrong in making any order in respect of a sum of
90, which the wife submitted was a contribution by her towards the legal costs incurred in connexion with the purchase of the
home in 1956, but which the registrar found was a loan, the husbands contention being that a loan did not constitute property
recoverable in proceedings under the Act of 1882, and (iii) that the registrar exercised his discretion wrongly in the matter of
costs, and his order for costs ought to be discharged. The registrars findings included the following(3) that there was an
intention on the part of the husband and wife to acquire the matrimonial home jointly: (4) that they contributed to the joint
venture in proportion to their respective incomes; (6) that the sum of 90 paid by the wife in 1956 was paid by her and accepted
by the husband as a loan and not as a contribution to the joint venture, the money having its origin in a legacy received by the
wife, and (9) that the sum of approximately 707 (the total proceeds of the transactions in regard to the matrimonial home)
included the sum of 90 lent by the wife and originally appropriated to the solicitors costs of the purchase of the matrimonial
home, and that the 90 being in the nature of a charge on the proceeds from the series of transactions, though forming an
identifiable and severable fund, remained part of the property in dispute.

M P Picard for the husband.


A J Phelan for the wife.

31 October 1962. The following judgments were delivered.

WILLMER LJ referred to the proceedings, and held that the registrars decision on the first question relating to the wifes
interest in the house, was essentially a conclusion of fact and, as there was evidence on which the registrar could reach his
conclusion, the husbands appeal on this question failed. His Lordship continued: I come, then, to the second ground of appeal,
viz, with 331 regard to the 90. Here, as it seems to me, the registrar has gone wrong. I have struggled hard to make sense of the
registrars findings, but I find it impossible to reconcile his finding numbered 6, that this was a loan and not a contribution to the
joint venture, with his finding numbered 9, that the 90 formed an identifiable and severable fund which remained part of the
property in dispute. It seems to me that, if this was a loan it was a loan; and, if it was a loan, it cannot be part of the property in
dispute. The husband, no doubt, is under a liability to repay the loan; but that is not a matter which can be dealt with as a
question of property within the jurisdiction of a court acting under s 17 of the Married Womens Property Act, 1882. Nor does
the Matrimonial Causes (Property and Maintenance) Act, 1958, make it any more possible to bring that matter within the purview
of these proceedings. For the application of that Act (I refer to s 7) is restricted to cases where there is a question between
husband and wife as to the title to, or possession of, property. That Act is therefore no more directed to the right to recover loans
as between husband and wife than was the original Act of 1882. It seems to me that that is the beginning and the end of this
matter. If this was a loan, the property in the money which the wife handed over to the husband passed to the husband, subject to
his contractual liability to repay. It is in my judgment quite impossible to say, as the registrar sought to say, that it formed any
part of the property in dispute.
The registrar might conceivable have arrived at a different conclusion, depending on the interpretation which he put on the
facts. It would have been, I suppose, open to him to find that, in contributing this 90, the wife was not making a loan, but was
contributing something to the joint venture. However, he did not so find. He found specifically that it was a loan, and not a
contribution to the joint venture. There was certainly evidence on which he could so find; for the wife in her affidavit had herself
put this forward as being a loan, and that was corroborated by the husband in the course of his cross-examination and re-
examination when he gave his oral evidence. That being so, it was no doubt difficult for the registrar to avoid the finding at
which he did arrive, that the 90 was a loan. That finding, it seems to me, is quite fatal to the wifes claim to have it treated as
part of the property in dispute.
What advantage either party is going to obtain from litigating this particular point I find it difficult to understand. The
conclusion at which I have arrived does nothing to get rid of the husbands contractual liability to repay the money that he
borrowed from his wife. All I decide is that this is not a matter which can be entertained in these proceedings. By succeeding on
this point here the husband only exposes himself to the possibility of other proceedings to recover the money lent. Whether such
proceedings will bear any fruit is a different matter, bearing in mind the fact that the husband has now lost his business and has
become a weekly wage earner with very attenuated means. The effect of this decision, however, is that the registrars judgment
for 240 cannot, in my view, be supported. The right of the wife to recover in these proceedings must be limited to 150.
It might be said, and indeed the point was canvassed during the course of the argument, that if the 90 goes out of the case it
might be proper to take as the wifes share of the property in dispute a quarter of the whole 707. This would have the effect of
slightly increasing the amount which she would be entitled to recover in these proceedings. But there has been no cross-appeal
on that point, and I do not think that it is open to us to alter the figure of 150 at which the registrar arrived.
That leaves only the third ground of appeal, as to costs. That, in itself, sub-divides into two questions. First, is the appeal
competent, ie, is there jurisdiction in this court to entertain such an appeal? Secondly, if so, is it a proper case in which this court
should interfere with what was prima facie an exercise of his discretion by the registrar who tried the case?
332
We have had some discussion whether this appeal is competent, having regard to the provisions of s 31(1)(h) of the Supreme
Court of Judicature (Consolidation) Act, 1925. That sub-section provides:

(1) No appeal shall lie (h) without the leave of the court or judge making the order, from an order of the High
Court or any judge thereof made with the consent of the parties or as to costs only which by law are left to the discretion of
the court.

This is not a case in which leave was either sought or given. In those circumstances, as I have said, the question arises whether
the appeal is competent. In my judgment the appeal is competent, because this is not an appeal as to costs only. I do not think
that it falls within the words of the subsection. The appeal, in so far as it relates to costs, is only part of the larger appeal against
the order as a whole. Certainly if and in so far as any appeal to this court succeeds it must be competent for this court to deal
with any order for costs made below. Even if this court were not disposed, in the event, to interfere with the main part of the
order under appeal, it would still be competent to deal with the appeal in so far as it relates to costs. For it is not possible to say
of such an appeal that it is an appeal relating to costs only. The view which I have expressed is, I think, supported at any rate by
two cases to which we have been referred, Forbes-Smith v Forbes-Smith and Chadwick, and Griggs v Petts. I do not think that it
is necessary to refer to the facts of those cases; but, as I have said, I think that they do lend support to the view which I have
expressed.
The appeal, therefore, being competent, there comes the difficult question whether we ought to interfere with the registrars
exercise of his discretion. That is a course which this court must necessarily be slow to take. The matter is complicated,
however, by the fact that both parties to this dispute are legally aided, and, as I have said, the husbands contribution so far as his
own costs are concerned was assessed at Nil. That means that the National Assistance Board regarded him as being a person
substantially without means. We do know that the proceeds of sale of the house have all disappeared, so that there is no fund in
his possession out of which any order could be discharged.
It was the duty of the registrar, when dealing with the question of costs, to follow s 2(2)(e) of the Legal Aid and Advice Act,
1949, which lays down that, where a person receives legal aid, his liability by virtue of an order for costs made against him with
respect to the proceedings shall not exceed the amount, if any, which is a reasonable one for him to pay having regard to all the
circumstances, including the means of all the parties and their conduct in connexion with the dispute. It is, I think, impossible to
avoid the conclusion that one of the circumstances, and indeed a most compelling circumstance, is the means of the party
himself. Whatever one may think of the conduct of a party it would still not be right to make an order for costs against him
which was unreasonable having regard to his means. This husband, we are told, is now earning about 12 a week. He has no
longer any capital assets. In those circumstances it seems to me that in making the order which he made the registrar cannot have
taken into consideration all the circumstances of the case. The order for costs in the sum of seventy-five guineas against a man in
his position is wholly unrealistic, and does not amount to a proper exercise of discretion. The registrar has, as I see it, purported
to exercise his discretion without any materials to justify the order which he made. In such circumstances, in my judgment, this
court is bound to interfere with the order for costs. In my view this was eminently a case in which no order for costs ought to
have been made against the husband.
It follows, therefore, that in my judgment this appeal succeeds, first, as to the 90 and, secondly, as to the order for costs, but
fails in respect of the registrars finding that the wife is entitled to an interest in the house.
333

HARMAN LJ. I agree. [His Lordship agreed that the appeal relating to the wifes claim to an interest in the house failed and
continued.] As to the 90, I am sorry to say that it is wholly outside s 17 of the Married Womens Property Act, 1882, even after
the extension of the Act in 1958. The dispute between husband and wife for the purposes of that section must be a dispute about
some property which belongs either to the one or to the other, and about which they are in dispute. There is no such property
here. There is a chose in action, a debt, if the wife is right; there is no debt if the husband is right. He does not say he is entitled
to any property. She says she has a claim against him in debt, and that is not a matter which can be agitated under s 17. If the
registrar felt himself able to conclude that this was one of the contributions made by the wife to the joint venture, well and good,
that is all right, but he categorically stated that it was not, and there was certainly evidence which pointed to the fact that both of
them were minded to regard it as being only in the nature of a loan, and, therefore, the registrar could not interfere.
As to the costs, I feel no doubt at all that the appeal lies. It was not an appeal as to costs only. The case in the Chancery
Division of Harris v Aaron, which was cited to us, was a case where the respondent desired to have the order altered as to costs
alone, he being quite content with the rest of it, and the Court of Appeal said, You cannot do that, but that is not at all in point
in this case. In fact we are varying the substantive order made below: it was a genuine application and not a sham, and therefore
it was competent for us to deal with the costs along with the rest of the application.
If the appeal be competent, then I am in agreement with my Lord that the registrar, having regard to the terms of the Legal
Aid and Advice Act, 1949, did not properly exercise his discretion; the wife was not wholly successful in the first court, she has
not been wholly successful in this court, and the proper order, I think, is that there should be no order as to costs.

DAVIES LJ. I agree, and I am sorry that we are having to interfere with the decision of the registrar as to the 90. The dispute
as to this sum seems rather pointless. On the facts found by the registrar (although we are not concerned with any subsequent
proceedings) there could be no shadow of defence to an action by the wife, in, I suppose, the county court, for this sum of money,
an action which one or both sides would again, I suppose, conduct at the public expense. But there it is. I suppose that one
practical consideration in that regard (and it is not very much of a consolation to this unfortunate wife) is that, as things now
stand, it seems that the value to the wife herself, as opposed to the Legal Aid Fund, of any judgment, or the payment of any sum
by the husband, is likely to be very little indeed. Apart from those two observations I agree with everything that has fallen from
my Lords.

Registrars order varied by reducing the sum payable by the husband from 240 to 150 and by directing that there should be no
order as to costs below.

Solicitors: W R Millar & Sons (for the husband); Ronald Brooke & Co (for the wife).

F A Amies Esq Barrister.


334
[1963] 2 All ER 335

Richter v Wilson
LANDLORD AND TENANT; Rent

COURT OF APPEAL
WILLMER, DONOVAN AND DIPLOCK LJJ
5 APRIL 1963

Rent Restriction Possession House required by landlord Intention to install married couple as tenants of the upper floor
Married couple looking after landlord in old age Two households, not a single household Whether dwelling-house reasonably
required by landlord for his own occupation within Rent and Mortgage Interest Restrictions (Amendment) Act, 1933 (23 & 24
Geo 5 c 32), Sch 1, para (h).

The landlord of a dwelling-house occupied the ground floor and let the upper floor to a tenant, whose tenancy was subject to the
Rent Acts. The landlord was eighty-three years of age and partially blind. A married couple, who lived next door, were of help to
him in looking after him. Wishing to install them as his tenants in order that they should be able more easily to look after him, he
brought proceedings for recovery of possession of the upper floor on the ground that it was reasonably required by him for
occupation as a residence for himself within the Rent and Mortgage Interest Restrictions (Amendment) Act, 1933, Sch 1, para
(h).

Held The test whether the landlord required the upper floor as a residence for himself was whether, if he recovered possession,
there would be two households or a single household in the house; on the facts the married couple would be a separate household,
and accordingly, if the landlord installed them in the upper floor of the house, he would not occupy that floor as a residence for
himself, and he was not, therefore, entitled to recover possession (see p 338, letters d, g and i, and p 339, letter a, post).
Appeal allowed.

Notes
As to a landlords right to recover possession of a controlled dwelling-house for his own occupation, etc, see 11 Halsburys Laws
(3rd Edn) 822, 823, para 1604; and for cases on the subject, see 31 Digest (Repl) 705708, 79377958.
For the Rent and Mortgage Interest Restrictions (Amendment) Act, 1933, Sch 1, para (h), see 13 Halsburys Statutes (2nd
Edn) 1066.

Cases referred to in judgments


Bloomfield v Westley 3 July 1962, not reported.
Smith v Penny [1946] 2 All ER 672, [1947] 1 KB 230, [1947] LJR 271, 31 Digest (Repl) 707, 7951.
Utting v Haynes 7 March 1960, CLYB 1960, para 2745.

Appeal
The tenant appealed against an order of His Honour Judge Granville-Smith sitting in Edmonton County Court on 20 September
1962, ordering that the landlord should recover possession of premises known as the first floor of 52, Hardwick Road, Palmers
Green, N13, from the tenant within four months. The ground of appeal was that the judge was wrong in law in holding that the
landlord, as landlord of premises controlled by the Rent Acts (viz, the Acts of 19201939, and the Rent Act, 1957), required them
for occupation as a residence for himself within the meaning of the Rent and Mortgage Interest Restrictions (Amendment) Act,
1933, Sch 1, para (h).

R J Southan for the tenant.


P Panto for the landlord.
5 April 1963. The following judgments were delivered.

WILLMER LJ. In this case it is impossible not to be sympathetic with both parties, for this is one of those unhappy cases in
which it is quite impossible to come to any conclusion without incurring the risk of causing hardship to one or other of them. The
question, however, has to be decided which of the parties must be subjected to the hardship. The appeal is from a judgment of
His Honour Judge Granville-Smith given at Edmonton County Court on 20 September 1962. 335The case concerned possession
of certain premises at 52, Hardwick Road, Palmers Green, in North London. The plaintiff was the owner and landlord of those
premises, and himself occupied the ground floor. The defendant was his tenant and occupied the upper floor. It is common
ground that the premises are protected premises to which the Rent Acts apply.
The landlord desires to obtain possession of the upper floor. He is an elderly man, said to be eighty-three years of age, and
has not been enjoying good health; in particular one of the troubles from which he suffers is that he is at least partially blind.
What he desires is to obtain possession of the upper floor in order that he may install there as his tenants a certain Mr and Mrs
King, who at present live next door, and who have been of great assistance to him in looking after him. It is said (and this has
been supported by medical evidence) that he is a man who requires a good deal of looking after, and that Mr and Mrs King could
look after him much better if they were actually living in the same house.
In those circumstances, the landlord invokes para (h) of Sch 1 to the Rent and Mortgage Interest (Amendment) Act, 1933,
and seeks to obtain possession in pursuance of the rights conferred by that paragraph. The opening words of Sch 1 and para (h)
provide:

A court shall, for the purposes of s. 3 of this Act, have power to make or give an order or judgment for the recovery of
possession of any dwelling-house to which the principal Acts apply or for the ejectment of a tenant therefrom without proof
of suitable alternative accommodation (where the court considers it reasonable so to do) if (h) the dwelling-house is
reasonably required by the landlord (not being a landlord who has become landlord by purchasing the dwelling-house or
any interest therein after July 11, 1931) [that date has now been altered] for occupation as a residence for(i) himself; or
(ii) any son or daughter of his over eighteen years of age; or (iii) his father or mother.

That is followed by a proviso dealing with the question of hardship if the paragraph is held otherwise to apply; as it is not
material to this appeal, I need not read that part of the paragraph.
The county court judge came to the conclusion that this case did fall within para (h), and that the landlord could be said
reasonably to require the upper floor for occupation as a residence for himself; and on that ground he decided the first point in
favour of the landlord. He then went on to consider the question of hardship, and whether it was reasonable to make an order. In
the end he decided that the greater hardship lay with the landlord, and he made an order for possession in four months time.
There is an appeal as to that part of the case; but the tenant appeals to this court against so much of the judgment as decided that
the landlord brought himself within para (h) at all.
I think that it is clear that, if the landlord is to succeed, it can only be on the basis of requiring the premises for occupation as
a residence for himself. He cannot bring himself within either sub-para (ii) or sub-para (iii). He has to show that he requires the
premises in question as a residence for himself. Quite plainly, the expression himself must include all the normal emanations
of himself. For instance, if a landlord is a married man, he must be entitled to include with himself his wife and any children of
his who are resident with him in the same household, at any rate up to the age of eighteen; sons and daughters over eighteen years
of age are dealt with separately. It is clear, therefore, that one cannot construe the word himself strictly as applying physically
only to the landlord himself; it must include any normal emanations of himself. The case has been decided by the judge on the
basis that, in the circumstances of this case, Mr and Mrs King really are emanations of the landlord himself, so that, if and when
they become tenants of the upstairs accommodation, it can fairly be said that the landlord himself will be residing there through
them.
Curiously enough, there is not very much authority on the point. We were 336 referred, however, to a decision of this court
in Smith v Penny ([1946] 2 All ER 672; [1947] 1 KB 230). That case was rather different from the present in that it concerned a
man who owned a house which he required for the occupation of his family. He himself had to live elsewhere, being the licensee
of licensed premises; but, having this house, he wished to obtain possession of it in order to accommodate his family. His family,
as it happened, consisted of two small children; but he had arranged with married friends of his, a Mr and Mrs Caley, to go and
live with the children and look after them. Basically it was a case in which he required the accommodation for his children, ie,
persons who could fairly be said to be an emanation of himself; and this court decided that in those circumstances such a plaintiff
does reasonably require the accommodation as a residence for himself. Both Scott LJ and Somervell LJ, made it clear in their
judgments that the word himself must include a mans wife and children, though in that particular case there was no question of
a wife because the plaintiff concerned had the misfortune to be separated from his wife. He did have children, however, and the
married couple, Mr and Mrs Caley, were members of the household in the sense that they were taking the place of the mother of
the children so as to form part of the plaintiffs household. That, it will be appreciated, is a very different case from the present
one.
A case nearer on its facts, which apparently was not considered by the county court judge, is Bloomfield v Westley. In that
instance, the landlorda wanted to let the premises in question, which consisted of the top part of the house of which he occupied
the lower part, to a niece and her husband; she wanted to do so partly because she wanted their company and thought that they
would be able to assist in looking after her. On 337 the facts this court came to the conclusion that that was not a case where the
landlord could be said to require the premises for occupation as a residence for himself.
________________________________________
a The facts in Bloomfield v Westley (decided on 3 July 1962, by Lord Denning MR Donovan and Pearson LJJ), were thus stated by Lord
Denning: The plaintiff [who was the owner of No 44, Broxholm Road, London, SE27] is a lady aged 59. She has been a widow for nearly
eleven years. She is not in good health; she suffers from rheumatism and has had some heart trouble. She has been living alone of late in
the ground floor flat, and what she wants to do, if she gets possession of the top floor, is to have her niece there. She said, if I get
possession of top floor I would have my niece with baby and husband; they are now living with her mother in cramped conditions. I would
hope she would be company for me. Further on, she said when recalled, niece would have her own kitchen. I sleep on first floor. Niece
would have separate flat. She is my godchild. I would help with shopping and minding baby. Husband do jobs for me in house. That is
all the material evidence really on this point. His Lordship said the question was whether the top floor was reasonably required by the
plaintiff as a residence for herself. The County Court judge (His Honour Judge Barrington, sitting at Lambeth County Court) had found that
the plaintiff did not require the top floor for herself. It was argued that that was wrong because the plaintiff required an extension of her
own premises so as to accommodate additional members of the family. Lord Denning MR continued: I can understand that there may be
cases where a landlord or landladyit may be a young couple who have an addition to their family by way of children, or it may be a
couple who want to take in an aged parentcould quite reasonably claim an extension to their premises on the ground that the extension
was reasonably required as a residence for a member of the family, I quite agree that that is a reasonable and a likely way in which this Act
[the Rent and Mortgage Interest Restrictions (Amendment) Act, 1933, Sch 1, para (h)] might operate. My trouble in this case is to bring the
facts within such a principle as was suggested. I would agree that if this niece had been a member of the family before and was coming
back to live with her aunt, there might be more in it; or if there was clear evidence that she was coming, so to speak, to be a member of the
family, to act as a nurse or as a daughter helping her mother, it might be different. But on the facts of this case, when the plaintiff has said
that the niece would have her own kitchen and would have her own separate flat, it seems to me that the judge must have found that the
niece was going to be in separate occupation of the top floor on her own account and not as a member of the aunts family on that view
of the facts [the plaintiff] does not require it as a residence for herself; she requires it as a residence for her niece, and that is not one of the
conditions under which the court can make an order for possession under the Rent Acts. Donovan and Pearson LJJ agreed. The appeal
was dismissed

The county court judge was referred also to Utting v Haynes which was a decision of His Honour Judge Baxter. It is an
extremely brief report, but it looks as though the facts were somewhat similar to the facts of this case. There the judge did find
that residence by the proposed tenants would be residence by the landlord, so that the landlord could fairly be said to require
the premises as a residence for himself. In the present case the judge thought that he ought to follow the decision of His Honour
Judge Baxter. I am bound to say that I find it a little difficult, if the case is accurately reported, to agree with the conclusion to
which the judge came in that case. However that may be, it is not a decision which binds us; we are free to make up our own
minds in the light of the circumstances of this particular case. There is no doubt that, if the landlords wish is carried out, Mr and
Mrs King in the present case would become the tenants of the plaintiff as landlord, with all the rights and obligations attaching to
the relationship of landlord and tenant. They would have the right to sole occupation of the premises let to them, and that would
involve the right to exclude the landlord from those premises. It seems to me that in circumstances such as are envisaged in this
case, it is very difficult to say that the landlord would then be occupying the premises as a residence for himself. I think that
counsel for the tenant put the case correctly when he said that the test was whether, if the plan were carried out, there would be
one household or two households. If Mr and Mrs King were living as a separate household from the landlord, it appears to me
that it would be impossible to say that the landlord was using the accommodation occupied by them as a residence for himself.
If, on the other hand, Mr and Mrs King were fairly to be regarded as forming one household with the landlord, then the position
would be otherwise.
If that is the right test, then the judges findings in this case is conclusive against the landlord, for he found in terms that Mr
and Mrs King would have their own separate household. That finding, in my judgment, was the only finding at which he could
arrive on the evidence before him, since it was made clear in the evidence that they would be leading their own separate lives;
they would not only reside but also would feed separately, and the only value of their occupation of the premises would be that
Mrs King would be immediately available to help the plaintiff and look after him as required. It was contemplated, however, that
they would become tenants, and that they would pay rent, although the amount of such rent had not so far been fixed. In such
circumstances, as I have already indicated, it seems to me that the judge came to the only possible finding of fact when he found
that they would be separate households. If they would be separate households, I think that that concludes the case, for it is
impossible for us to hold in such circumstances that the landlord would be occupying the upper floor as a residence for himself,
or that the accommodation is required as a residence for himself.
In my judgment, therefore, the county court judge did not come to the right conclusion, and this appeal should be allowed.

DONOVAN LJ. The county court judge considered this case with obvious care and sympathy for both parties; but I think that as
a matter of law it was impossible for him to conclude that the plaintiff required the premises for occupation as a residence for
himself. The truth is that he required them as a residence for strangers, viz, Mr and Mrs King and their family, and none the less
because this would enable Mrs King to look after the landlord more easily. But Mr and Mrs King were to be a separate
household, as indeed the judge found, and they were to pay rent for their accommodation. It is impossible in those circumstances
338 to bring this case within para (h) of Sch 1 to the Act of 1933, and I agree that the appeal must be allowed.

DIPLOCK LJ. I agree, and have nothing to add.

Appeal allowed.

Solicitors: Neil Maclean & Co (for the tenant); J Burt Sugar & Co (for the landlord).

F A Amies Esq Barrister.


[1963] 2 All ER 339

Muspratt v Johnston
LANDLORD AND TENANT; Rent, Tenancies

COURT OF APPEAL
LORD DENNING MR, DANCKWERTS, DAVIES LJJ
21 MARCH 1963

Rent Restriction Sub-tenancy Unlawful sub-tenant of part of premises with rateable value below 40 Whole premises
decontrolled by Rent Act 1957 Sub-tenancy becoming lawful by waiver of breach by landlord in 1958 Whether landlord
entitled to recover possession from sub-tenant Increase of Rent and Mortgage Interest(Restrictions) Act, 1920(10 & 11 Geo 5
c 17), s 15(3).

In 1940 a dwelling-house was let to a tenant at a weekly rent for a year. At the end of the year the tenant became a statutory
tenant and remained in occupation. In the tenancy agreement the tenant had agreed not to sub-let without the landlords consent
in writing. In 1956 the tenant, without the landlords consent, sub-let two of the upstairs rooms (of which the rateable value at all
material times was less than 40) to a sub-tenant. On 6 July, 1957, the whole dwelling-house, having a rateable value exceeding
40, became decontrolled by virtue of s 11(1) of the Rent Act, 1957. The tenant remained in possession of the dwelling-house
under the provisions of Sch 4 to the Act of 1957. In a letter dated 21 June, 1958, the tenant disclosed to the landlord that the two
rooms were sub-let, but the landlord took no steps to object to the sub-letting and continued to accept the tenants rent. The
tenant left the dwelling-house in 1961. In an action by the landlord, claiming possession of the two rooms from the sub-tenant, it
was found that the landlord had waived the unlawfulness of the sub-letting.

Held the sub-tenant was not protected by the Rent Acts and the landlord was entitled to possession, because
(i) the waiver only operated so as to render the sub-letting lawful as from the date of the waiver, viz, not until after 21 June,
1958 (see p 341, letter e p 342, letter i, and p 343 letter d, post), and
(ii) the tenants interest in the dwelling-house for the purposes of s 15(3) of the Act of 1920 a determined on 6 July, 1957,
when the Rent Act, 1957, came into operation and, as at that date the sub-letting was unlawful, s 15(3) of the Act of 1920 had not
rendered the sub-tenant a tenant of the landlord (see p 342, letter b, and p 343, letters a and f, post).
________________________________________
a Section 15(3) is set out at p 341, letter g, post

Legge v Matthews ([1960) 1 All ER 595) distinguished.
Appeal dismissed.

Notes
As to the protection under the Rent Acts of a sub-tenant of premises forming part of a superior letting, see 23 Halsburys Laws
(3rd Edn) 826828, para 1611; and for cases on the subject, see 31 Digest (Repl) 711714, 7974-7988.
For the Increase of Rent and Mortgage Interest (Restrictions) Act, 1920, s 15 (3), see 13 Halsburys Statutes (2nd Edn) 1019.

Cases referred to in judgments


Carter v Green, [1950] 1 All ER 627, [1950] 2 KB 76, 31 Digest (Repl) 712, 7981.
Cheeseman v Bagnall, [1962] 2 All ER 195, [1962] 1 WLR 622.
339
Legge v Matthews, [1960] 1 All ER 595, [1960] 2 QB 37, [1960] 2 WLR 620, 3rd Digest Supp.

Appeal.
This was an appeal by the sub-tenant, Marjorie Johnston, from that part of a judgment of His Honour Judge Dow, at Clerkenwell
County Court, given on 28 September 1962, by which he ordered that the sub-tenant give possession of two rooms situate on the
first floor of 26, Coldfall Avenue, Muswell Hill, in the County of London to the landlord. The facts are set out in the judgment of
Lord Denning MR.
The cases noted belowb were cited during the argument in addition to those referred to in the judgments.
________________________________________
b Ross T Smyth & Co Ltd v TD Bailey Son & Co, [1940] 3 All ER 60; Norman v Simpson, [1946] 1 All ER 74; [1946] KB 158; Maley v Fearn
[1946] 2 All ER 583; Oak Property Co Ltd v Chapman [1947] 2 All ER 1; [1947] KB 886; Re Lower Onibury Farm, [1955] 2 All ER 409;
sub nom Lloyds Banks Ltd v Jones, [1955] 2 QB 298; City of Westminster Properties (1934), Ltd v Mudd [1958] 2 All ER 33; [1959] 1 Ch
129

LA Blundell, QC and Gerald Butler for the sub-tenant.


Dingle Foot QC and W H Dunn for the landlord.

21 March 1963. The following judgments were delivered.

LORD DENNING MR. On May 23, 1940, Mrs Muspratt, the landlord, let to Mrs Maling, the tenant, a house, No 26, Coldfall
Avenue, Muswell Hill, for one year at 25s a week. After the end of the year the tenant stayed on as a statutory tenant. She stayed
there for years and did not quit until November, 1961. When she went she left behind a sub-tenant of two of the rooms upstairs, a
Mrs Johnston. Mrs Muspratt, the landlord, now seeks to evict the sub-tenant, Mrs Johnston. But the sub-tenant claims the
benefit of the Rent Restriction Acts. She says that she is entitled to stay there because the two rooms are within the limits of the
Rent Acts (below 40 rateable value) and were lawfully sub-let to her. The case involves an inquiry into the history of the sub-
letting. The original agreement of tenancy contained a provision by which the tenant agreed not to

assign this agreement or underlet sublet or part with the possession of the premises or any part thereof during the
tenancy without the previous written consent of the landlord but such consent shall not be unreasonably withheld

That provision continued, of course, during the statutory tenancy. In breach of it the tenant, in 1956, sub-let two of the upstairs
rooms to the sub-tenant without getting the landlords consent. So the original sub-letting was clearly unlawful; and it remained
unlawful until the landlord got to know of it in the middle of 1958. Then the tenant disclosed it to her in a letter dated June 21,
1958. It was the first intimation the landlord had of the sub-letting:

Two years ago I emptied two rooms upstairs and let them unfurnished to a young widow, and her two little girls. She
is a very nice superior girl, and I love the children, we have always got on well. Before we came to terms I told her you
wanted the house, and that I was looking out. She said she would take that chance. She has known since January I was
writing to Basildon [for a council house]. I have told her that I am now waiting for the word go, so she had better find
something too, she said she was staying put, she had nowhere else to go, and that was that. This worried me so I walked
to Crouch End Citizens Advice Bureau, and stated these facts. She, the young lady at the bureau, advised me to write to
[the sub-tenant], and put down these facts and be prepared to receive notice to quit from you that perhaps you would not
object to her staying in the rooms. If you do I can speak highly of her, I charged her 25s
340

Now that was a plain intimation to the landlord that there had been a sub-letting. Yet the landlord never complained of it. She
did not take any steps to object to the sub-letting; she accepted the rent without demur and did nothing to qualify her acceptance.
The county court judge has held that, by so doing, she waived the unlawfulness of the sub-letting. We have been referred to the
cases on waiver, particularly Carter v Green, and I think that the result of them is that, in these Rent Act cases, there is not a
waiver of an unlawful sub-letting from the mere acceptance of rent or the mere failure to write out a qualification at once. There
has to be such a degree of acquiescence that a consent to a sub-letting can be inferred. This case is very near the line, but, on the
whole, I think that there is just sufficient evidence from which the judge could infer a waiver, and I am not disposed to differ
from his finding on it.
What is the effect of this waiver? It clearly renders the sub-letting lawful; but from what date is it rendered lawful? Counsel
for the sub-tenant has urged us to hold that it is rendered lawful as from the beginning in 1956. He says that, on an unlawful sub-
letting, there is only one single breach, once for all; and that a waiver of the unlawfulness must mean a waiver of that one Initial
breach. It cannot mean anything else; and, if the initial breach is waived, then it must be a lawful sub-letting from the beginning.
Despite the logical force of his argument, I cannot agree with him. It seems to me quite plain that, until the landlord got to know
of the sub-letting, it was an unlawful sub-letting; and I do not think that what was unlawful for two years becomes retrospectively
lawful by waiver. It is settled law that, when there is a breach of the covenants of a lease which gives rise to a forfeiture, and the
landlord forfeits, the lease is only determined as from the date of the re-entry for the forfeiture. The unlawfulness does not relate
back to the original breach. So, here, I would hold that, when there is a waiver, it only operates so as to render the sub-letting
lawful as from the date of the waiver. It does not relate back so as to make the sub-letting lawful ab initio. I hold that this sub-
letting was unlawful, certainly from 1956 right through to 1958, and did not become lawful until after 21 June 1958.
It is now necessary to fit these findings into the framework of s 15(3) of the Increase of Rent and Mortgage Interest
(Restrictions) Act, 1920, which is the only provision by which the sub-tenant would be entitled to remain. It says:
Where the interest of a tenant of a dwelling-house to which this Act applies is determined, either as the result of an
order or judgment for possession or ejectment, or for any other reason, any sub-tenant to whom the premises or any part
thereof have been lawfully sub-let shall, subject to the provisions of this Act be deemed to become the tenant of the
landlord on the same terms as he would have held from the tenant if the tenancy had continued.

The subsection in its very terms applies only where the interest of a tenant of a dwelling-house to which this Act applies is
determined. What is the date of determination in this case? The tenant was a contractual tenant from 1940 to 1941, and
thereafter she was a statutory tenant until 6 July 1957. She was, throughout all those seventeen years, the tenant of a dwelling-
house to which this Act applies. But on 6 July 1957, the Rent Act 1957, came into operation; and the Rent Acts ceased to apply
to the whole house, because it was over 40 rateable value: see s 11(1) of the Act of 1957. It seems to me that, on that date, the
interest of the tenant determined within the meaning of s 15(3) because she was no longer a tenant of a dwelling-house to
which the Acts applied. True it is that thereafter she acquired a new interest, because she remained on in the house by virtue of
the transitional provisions in Sch 4 to the Act of 1957. Under these standstill provisions, as they are called, the tenant had the
right to remain on in possession for fifteen months, and could not thereafter be turned 341 out except on a six months notice to
quit. But, during this standstill period, she was not a controlled tenant. Her tenancy was not a tenancy to which the Rent Acts
applied: see Cheeseman v Bagnall. She was a person with a temporary limited right to possession and no more. She might,
perhaps, be described as a standstill tenant, but she was no longer a tenant of a dwelling-house to which this Act applies. Her
interest for the purpose of s 15(3) had determined on 6 July, 1957. And at that date the sub-letting of the two rooms upstairs
was unlawful. So the sub-tenant never came within the protection of s 15(3).
I am confirmed in this view by considering what would be the position if the tenant had first sub-let these two upstairs
rooms to the sub-tenant in the year 1958, whilst she was a standstill tenant, and had even obtained the consent of the landlord to
the sub-letting. It is clear that the Rent Acts would not apply to that sub-letting. True, the rateable value of the two rooms was
under 40, but the sub-letting would be unprotected, because no new letting after 6 July, 1957, is protected by the Act: see s
11(2) of the Act of 1957. If a new sub-letting since 1957 is not protected, I do not see why a waiver since that date should give
any protection. We were referred to Legge v Matthews, but that case is quite distinguishable because there the ground floor was
lawfully sub-let in 1953 and was itself within the protection of the Acts. The whole house had a rateable value of 48 and was
taken out of the Acts on 6 July, 1957; but the ground-floor had a rateable value of 16 and remained within the Acts. It was,
therefore, a plain case for the operation of s 15(3) because, at the date when the interest of the tenant of the whole house
determined, namely on 6 July, 1957, the ground floor was lawfully sub-let to the sub-tenant; and the sub-tenant became the
direct tenant of the head landlord; but I should have thought that this direct tenancy would not come into operation until the
standstill tenants period of grace had expired. Counsel for the sub-tenant sought to say that s 41 of the Housing Repairs and
Rents Act, 1954, could be applied here in favour of the sub-tenant just as it was in Legge v Matthews. But I do not think so. The
superior letting here was the letting to the tenant and it came to an end on 6 July, 1957. And as at that date the two rooms
upstairs were not lawfully sub-let to the sub-tenant, she can take no benefit from s 41.
I rest my judgment on this, therefore, that the sub-letting to the sub-tenant was unlawful in its inception and it never became
lawful before 6 July, 1957, when the Rent Acts ceased to apply to the house. She is, therefore, not entitled to the protection of the
Acts. She has had quite a long tenure against the landlord and the time has now come when she must go. I find myself in
agreement with the county court judge. I would dismiss this appeal.

DANCKWERTS LJ. I feel great doubt whether there was any real waiver at all in this case in the circumstances, and, in
particular, having regard to the terms of the letter written by the tenant on 21 June, 1958. It seems to me that the basis of the
continuance of the tenant and her sub-tenant in occupation was that the continuance should be precarious because it was on the
footing that the landlord wanted possession of the premises and the tenant was making every endeavour to move out. It seems to
me a sad state of affairs and of the law if a landlord, recognising the difficulties of getting accommodation at the present time,
grants a tenant indulgence and is thereby held to have forfeited his right to recover possession of his property, apparently for ever,
or at any rate for present purposes. But it seems that we are bound by the findings of the county court judge that there was a
waiver. Be it so, I agree with the judgment of Lord Denning, MR, that the waiver only took effect from some date after 21 June,
1958, and that it does not validate a sub-letting before the receipt of the letter of that date. I reach the same conclusion as that
which my Lord 342 has expressed, and I do not find it necessary to add to the arguments and considerations which he has
expressed.

DAVIES LJ. I also agree. On the point of waiver, we were referred to Carter v Green. In that case, there is an observation in
the judgment of the court given by Cohen LJ, which I think does not help the tenant at all. At the end of the judgment he says
([1950] 1 All ER at p 631; [1950] 2 KB at p 85):

In these circumstances it seems to us that the position is that the judge has found that the landlord knew of the sub-
letting before 1 January 1948, and it is plain from the evidence that he received substantial payments of rent without in any
way qualifying his receipt thereof between that date and 6 October 1948. We think, therefore, that we are bound to
conclude that, even if the sub-demise was originally unlawful, the landlord had waived the illegality before he commenced
the present action and thus the sub-tenancy had become a lawful sub-tenancy.

That, as I think, is entirely different from the facts of the present case. What the court were considering there was whether, after
the acceptance of rent, the sub-demise thereafter became lawful. Counsel for the sub-tenant is concerned to say here that, by
reason of the acceptance of rent when the landlord knew of the unlawful sub-letting, that rendered the letting lawful ab initio, to
use counsels phrase. In my opinion, it did no such thing.
With regard to the other point, it is in the first place, I think, doubtful whether the tenant, after the passing of the Rent Act,
1957, remained a tenant at all. I do not discuss the point, as it is not necessary for the purpose of the decision of this case. But it
seems to me that the observations of Pearce LJ, in Legge v Matthews and of Paull J, in Cheeseman v Bagnall suggest that, after
1957, it was not a true tenancy.
I think, as my Lord has indicated, that this case really turns on the application of s 15(3) of the Increase of Rent and
Mortgage Interest (Restrictions) Act, 1920. As I think, the interest of the tenant in a dwelling-house to which the Rent Acts
applied determined as such in July, 1957, by operation of the Rent Act, 1957. At that date there was no lawful sub-letting.
Whatever interest remained in the tenant thereafter during the standstill period, whether, as counsel for the sub-tenant called it,
she was a standstill tenant or, as counsel for the landlord would have it, a statutory licensee, it was not an interest in a dwelling-
house to which the Rent Acts applied. Therefore, as I think, no application of s 41 of the Housing and Repairs Act, 1954, can
help the sub-tenant in relation to the date of November, 1961, when her landlord at last and finally went out. It seems to me that
it is implicit in counsel for the sub-tenants argument here that the introductory words of s 15(3) of the Act of 1920 must be
treated as relating to quite separate dates, namely, the coming into force of the Act of 1957, on the one hand, and the final
departure of the tenant in November, 1961, on the other. That seems to me to be quite impossible.
I, therefore, agree that this appeal fails.

Appeal dismissed.
Solicitors: Lipson Rumney & Co (for the sub-tenant); Dale & Newbery, Feltham (for the landlord).

F Guttman Esq Barrister.


343
[1963] 2 All ER 344

Malyon v Plummer
CIVIL PROCEDURE: QUANTUM

COURT OF APPEAL
SELLERS, PEARSON AND DIPLOCK LJJ
8, 11, 25 MARCH 1963

Fatal Accident Damages Assessment Deductions from damages Husband and wife sole directors of company Whether
and extent to which wifes salary derived from matrimonial relationship Fatal Accidents Act, 1846 (9 & 10 Vict c 93), s 2.

Fatal Accident Damages Assessment Deductions from damages Insurance money paid as a result of the death Insurance
money paid to company on death of director Increase in value of shares inherited by deceaseds widow and sole co-director
Whether amount of benefit derivable by widow from insurance payment to company deductible from damages payable to the
widow Fatal Accidents Act, 1959 (7 & 8 Eliz 2 c 65), s 2(1).

The plaintiff and her husband, M, were the sole directors and shareholders of a limited company in which M held 999 shares and
the plaintiff held the remaining one share. They had married some eight years before the company was formed. There were two
children of the marriage who were infants. The plaintiff and M each received a salary from the company, but the trial judge
found that the services rendered to it by the plaintiff were part-time, casual, and rendered without any set routine. Her salary as
director and secretary of the company which, after the first year of trading, was in 1958 600 and in 1960 800 per annum, was
paid into Ms bank account and was used with his salary for the general benefit of the family. The company had taken out a
personal accident insurance policy on the lives of M and the plaintiff, the sum insured on Ms death being 2,000. M was killed
as a result of a car accident caused by the defendants negligence. On Ms death, his 999 shares in the company passed under his
will to the plaintiff. The value of each share immediately before M.s death was 14s 6d, but this increased to 2 14s 6d on the
payment to the company of the 2,000 under the insurance policy. The shares were valued on an assets basis at the increased
figure for estate duty purposes, and, if the company had ceased trading immediately on the deceaseds death, the plaintiff would
have been entitled to receive the 2,000 in cash on distribution. The plaintiff endeavoured to continue the companys business,
but found it impossible both to run the company and to attend to her home and children, and intended to wind up the company.
In an action by the plaintiff for damages under the Fatal Accidents Acts, 1846 to 1959, the trial judge assessed damages on the
basis of an annual figure (1,375) for the benefits derived by the plaintiff and her children from the deceased via his bank
account, and a multiplier (thirteen) for that figure. On the questions whether deductions should be made in respect of the
plaintiffs salary from the company and of the policy moneys,

Held (i) The excess of the plaintiffs salary from the company over the market value of her services was a benefit derived from
the plaintiffs relationship to the husband as his wife and thus was a benefit lost by his death and to be taken into account in
assessing her dependency on him and the damages recoverable by her under s 2 a of the Fatal Accidents Act, 1846 (see,
particularly, p 356, letter c, post); accordingly only 200 of her salary (200 pa being the market value of the services rendered by
the plaintiff to the company) should be deducted from the annual figure (1,375) on which the assessment of damages was based
(see p 349, letters h and i, p 351, letters f and g, and p 357, letters d and e, post).
________________________________________
a Section 2 is set out at p 349, letter b, post

Burgess v Florence Nightingale Hospital for Gentlewomen Management Committee ([1955] 1 All ER 511) distinguished.
(ii) The insurance policy moneys (2,000), although paid to the company 344 for its benefit, fell within the words of s 2(1) b
of the Fatal Accidents Act, 1959, and accordingly were to be left out of account in assessing the plaintiffs damages; therefore,
the value of the 999 shares should be assessed at 14s 6d each, not 2 14s 6d each, with the consequence that the plaintiff had to
give credit for 650 only, not 2,650 (see p 352, letter g, p 350, letter e, and p 358, letter a, post).
________________________________________
b Section 2(1) is set out at p 350, letter b, post

Green v Russell ([1959] 2 All ER 525) distinguished.


Decision of Mocatta J ([1962] 3 All ER 884) varied on (i) and affirmed on (ii).

Notes
As to actions under the Fatal Accidents Acts and deductions from damages, see 28 Halsburys Laws (3rd Edn) 100, 101, para 110,
and 103, 104, para 113; and for cases on the subject, see 36 Digest (Repl) 211214, 11111132.
For the Fatal Accidents Act, 1846, s 2, see 17 Halsburys Statutes (2nd Edn) 5; and for the Fatal Accidents Act, 1959, s 2, see
39 Halsburys Statutes (2nd Edn) 942.

Cases referred to in judgments


Blake v Midland Ry Co (1852), 18 QB 93, 21 LJQB 233, 18 LTOS 330, 118 ER 35, 17 Digest (Repl) 185, 806.
Burgess v Florence Nightingale Hospital for Gentlewomen Management Committee [1955] 1 All ER 511, [1955] 1 QB 349,
[1955] 2 WLR 533, 3rd Digest Supp.
Franklin v South Eastern Ry Co (1858), 3 H & N 211, 31 LTOS 154, 157 ER 448, 36 Digest (Repl) 213, 1128.
Green v Russell [1959] 2 All ER 525, [1959] 2 QB 226, [1959] 3 WLR 17, 3rd Digest Supp.
Hadley v Baxendale (1854), 9 Exch 341, 23 LJEx 179, 23 LTOS 69, 156 ER 145, 17 Digest (Repl) 91, 99.
Hull v Great Northern Ry Co of Ireland (1890), 26 LR Ir 289, 36 Digest (Repl) 212, 2006.
Monarch SS Co Ltd v Karlshamns Oljefabriker (A/B) [1949] 1 All ER 1, [1949] AC 196, [1949] LJR 772, 2nd Digest Supp.
Salomon v Salomon & Co Ltd, Salomon & Co Ltd v Salomon [189599] All ER Rep 33, [1897] AC 22, 66 LJCh 35, 75 LT 426, 9
Digest (Repl) 30, 11.
Sykes v North Eastern Ry Co (1875), 44 LJCP 191, 32 LT 199, 36 Digest (Repl) 212, 1117.
Weller v OBrien [1962] 3 All ER 65, [1962] 1 WLR 885.

Appeal
This was an appeal by the defendant against the assessment of damages by Mocatta J, on 3 October 1962, reported [1962] 3 All
ER 884, being an assessment of damages in an action brought by the plaintiff, Gertrude Doris Malyon, widow and administratrix
of her husband, suing on behalf of herself and her two infant children, for the damages for which she and her two infant children
were entitled by reason of the death of her husband, Frederick Parker Malyon, the father of the children who, at the time of his
death, was forty years of age and whose death was caused by the defendants negligent driving of a motor car. The accident
occurred on 21 December 1960, and the husband died on 6 January 1961. The damages were assessed at 17,348 of which 123
was undisputed (being 10 for damage to the husbands car not covered by insurance, 50 for equipment in the car at the time of
the accident and 63 for funeral expenses), which sum of 123 was allocated to the plaintiffs claim under the Law Reform
(Miscellaneous Provisions) Act, 1934, which was agreed at 400. The balance of 17,225 represented the financial loss which
Mocatta J found to have been occasioned to the plaintiff, suing on behalf of herself and the children under the Fatal Accidents
Acts, 1846 to 1959.
345
The plaintiff married the husband in 1947. There were two sons of the marriage, born in 1949 and 1953. The husband, after
working for several commercial firms in East Anglia, started in 1955 a business of selling and distributing portable electrical
machines and accessories. To carry on this business he formed a private company, F P Malyon Ltd. The capital was one
thousand 1 shares, of which he held 999 and the plaintiff held the remaining share. The business, so it was found by Mocatta J
was that of selling and distributing in East Anglia various commodities, mainly portable electrical machines. The company held
on important sole distribution agency in the area for a concern called Grinding Improvements Ltd and a number of other non-
exclusive agencies. When the company started it held this sole agency and one non-exclusive agency. At the time of the
husbands death the number of non-exclusive agencies had increased to four and the number of customers on the books, namely,
builders and farm machinery suppliers, was 480. It was a one-man business. The husband operated the business from their house
which was near Peterborough. He worked a six-day week. He would get up at half-past six or seven oclock, would deal with
the post and would prepare his itinerary for the day, and would go out in his car after breakfast, delivering goods, taking orders,
canvassing customers and travelling around the area of the Home Counties and East Anglia. He would come home usually about
7.30 pm and would enter up any orders which the plaintiff had taken, presumably by telephone, during the day and any orders
which he himself had taken. A Mrs Nightingale was employed by the company as a book-keeper; she did part-time work in the
mornings only. In the plaintiffs evidence in chief she described Mrs Nightingales part in the business as follows:

Telephone; any correspondence which was necessary; parcelling up, say, anything vital to go off or anything cropped
up, and attend to the correspondence generally.

The plaintiff had the house and two small boys to look after. The boys used to come home for mid-day meal, and she had also to
prepare the evening meal. In April, 1959, the husband acquired a caravan on hire-purchase. This was kept near the sea. During
the school holidays the plaintiff spent much of her time at the caravan with the boys, and thus was away from the business and
could not help with it. In answer to the learned judges request for clarification with regard to the plaintiffs duties in relation to
the business, counsel supplied information, which the learned judge recorded in a note as follows:

Telephone etc.; inquiries by telephone when husband out or away; if someone phoned and ordered she would take it
down and occasionally packed up goods and despatched them, if required. Telephone was not a manned line. She went to
caravan in school holidayscomparable to doctors wife. No set routine.

In the course of the companys trading year, which ended on 31 May the husband drew money from the company and paid it into
his bank account, which was his sole bank account, not a joint bank account of the husband and wife. Out of this bank account
he drew sums in cash to provide the plaintiff with a housekeeping allowance of 10 weekly, and he also drew other sums for
housekeeping and family purposes. The average annual net sum drawn out and expended for the benefit of the plaintiff and the
children (after deduction of the proportion expended for the benefit of the husband) was 1,328 4s 9d. This figure the learned
judge increased 1,375 per annum to allow for future increases in the sums that would have been available, and so would have
been expended. Substantially the companys available balance at the end of each financial year was used by allocating salaries to
the husband and to the plaintiff, and no dividend was paid, only a small net profit or net loss being carried forward. In the first
year of trading the plaintiff was credited with only 29, and later years were as follows: 1958, husband 1,600, plaintiff 600;
1959 husband 1,450, plaintiff 600; 1960, husband 1,800, plaintiff 800. The husband died about the middle of 346 the
financial year ending 31 May 1961. The trading and profit and loss account for the year ending 31 May 1960, showed as
directors salaries 1,800 for the husband and 800 for the plaintiff and a net profit of 114 3s 9d. Substantially the whole of
the amount allocated as directors salaries at the end of the year had already been drwan out by the husband in the course of the
year, had been paid into his bank account and had been used for housekeeping and family purposes and general purposes.
After the husbands death on 6 January 1961, the plaintiff carried on the business. She was still carrying it on at the date of
the trial on 3 October 1962, but then intended to liquidate it. She had the help at first of Mrs Nightingale, and later a Mrs Hurry
took the place of Mrs Nightingale. For about three months in the summer of 1961 the plaintiff had a travelling salesman, but he
did not give satisfaction. The plaintiff could not drive a car, and thus had to carry on the business by correspondence. The
accounts for the year ended 31 May 1961, showed a reduced turn over, the amount of the sales being 16,370 as compared with
22,048 in the previous year. Directors salaries also were reduced, being 550, for the husband and 350 for the plaintiff, as
compared with 1,800 and 800 in the previous year. That left a net profit of 9 3s 5d. There was a credit balance of 1,653 2s
10d in the profit and loss appropriation account, after taking into account 2,000 received from the policy of insurance mentioned
at letter g, below, and this was carried forward. The accounts for the year ended 31 May 1962, showed a further reduction in
turnover, the sales being 9,231 as compared with 16,370 in 1961. The plaintiffs salary was 1,200. There was a credit
balance of 1,573 in the profit and loss appropriation account, and this was carried forward.
In the plaintiffs evidence at the trial she said that she intended to liquidate the business immediately. In answer to a
question why she was going to do that, she said that she found the business, the home and the boys, to be more than she could
manage. The boys required more and more attention, rather than less and less, and she found that business encroached on the
lives of the boys to such an extent that it would be better if she could get a part-time job which would take her out when they
were out and leave her fresh and available when they were at home.
Under the husbands will the plaintiff received the 999 shares which he had held in F P Malyon Ltd. Credit was given for
these at a price of 14s 6d, making in all 650. The defendant claimed that the value of the shares was 2 14s 6d each because, on
the death by accident of the husband the company received 2,000 from Royal Insurance Co Ltd and with this addition the shares
were worth another 2 each. By a personal accident policy dated 12 December 1960, between Royal Insurance Co Ltd and F P
Malyon Ltd the insurers had undertaken obligations in respect of the lives insured, viz, those of the husband and the plaintiff,
including obligation to pay 2,000 in the event of the husbands death from accidental injury. On the husbands death by accident
the 2,000 was paid to F P Malyon Ltd was brought into its profit and loss appropriation account as compensation received for
goodwill on the death of F P Malyon Esq and was used as part of its capital in the continuation of its business by the plaintiff in
what proved to be an unsuccessful attempt to maintain the profits of the business.

Martin Jukes QC and Kenneth Jupp for the defendant, the appellant.
D P Croom-Johnson QC and R I Kidwell for the plaintiff, the respondent.

Cur adv vult

25 March 1963. The following judgments were delivered.

SELLERS LJ whose judgment was read by Pearson LJ referred to the entitlement to damages for which Mocatta J had assessed
damages, and to the figures at which he had assessed damages under the Fatal Accidents Acts, 1846 to 1959 and continued: The
total financial benefit which the plaintiff 347 wife and her two children enjoyed prior to her husbands death was 1,328 per
annum. The learned judge increased that to no more than 1,375 per annum on the prospect of some slight improvement in
earnings in the future and on which sum he assessed the damages. This was on the basis that the wife and children were solely
dependent on the husband for that sum.
The first ground of appeal is that, in so finding, the learned judge was wrong; that of the 1,328, the wife contributed 668
(being 800 less tax), and that 668, therefore, ought to have been deducted from the 1,375, making the net figure for
dependency 707 only. [His Lordship stated the facts as to the history of the business (see p 346, letters b to d, ante), and the
essentially casual nature of the work done by the wife therein, and continued:] Mocatta J stated ([1962] 3 All ER at p 890; [1963]
1 QB at p 429)

The novel feature of the present case, distinguishing it from any previous authority to which I have been referred, is
the creation of the so-called one-man company and its interposition between the earnings won by the [husband] and the
receipt of part of them by the plaintiff for use in the home. The work done by the plaintiff in connexion with the business
was little more than nominal and, on the facts here, the payment of a sum to her by way of salary as a director instead of a
direct payment by the husband out of a larger salary of his own was mere machinery, no doubt carrying independent tax
advantages. It is not for me to say anything about the latter point which does not concern me here. Whilst the company is
no doubt a separate legal entity, I can see nothing in the principle of Salomon v. Salomon & Co., Ltd., Salomon & Co., Ltd.
v. Salomon, which prevents me from finding and holding, as I do, that the salary which the plaintiff received via [her
husbands] bank account and which was used in purchasing pro tanto the benefits for the plaintiff and her children with
which I have dealt, derived solely from the relationship of husband and wife between her and the deceased. That salary
and the benefits which it purchased were lost on the termination of the relationship by the death of the husband.

His Lordship referred to the amounts credited to husband and wife in the accounts of the business (see p 346, letter i, ante), and
continued: In approaching the question involved I think that it should be contemplated in regard to another contingency. If it had
been the wife who had been killed, could the husband have successfully claimed dependency on her for 800 less tax, because
that sum was shown to be allocated to her in the books of F P Malyon, Ltd? I think not.
In Monarch SS Co Ltd v Karlshamns Oljefabriker (A/B) ([1949] 1 All ER 1 at p 19; [1949] AC 196 at p 232), Lord Du Parcq
made these general observations on damages:

I do not doubt the wisdom of the judges who, in Hadley v. Baxendale and the many later cases which interpreted or
explained that classic decision, have laid down rules or principles for the guidance of those whose duty it is, as judges or
jurymen, to assess damages. When those rules or principles are applied, however, it is essential to remember what my
noble and learned friend LORD WRIGHT ([1949] 1 All ER at p 13; [1949] AC at pp 222, 223), and LORD HALDANE,
L.C., in the passagec cited by him have emphasisedthat in the end what has to be decided is a question of fact, and,
therefore, a question proper for a jury. Circumstances are so infinitely various that, however carefully general rules are
framed, they must be construed with some liberality and not too rigidly applied. It was necessary to lay down principles
lest juries should be persuaded to do injustice by imposing an undue, or, perhaps, an inadequate, 348liability on a
defendant. The court must be careful, however, to see that the principles laid down are never so narrowly interpreted as to
prevent a jury, or judge of fact, from doing justice between the parties. So to use them would be to misuse them.
________________________________________
c In British Westinghouse Electric & Manufacturing Co Ltd v Underground Electric Ry Co of London Ltd, [191113] All ER Rep 63 at 69;
[1912] AC 673 at p 688

Essentially is it so under the Fatal Accidents Act, 1846, s 2:

Every such action shall be for the benefit of the wife, husband, parent, and child of the person whose death shall
have been so caused, and shall be brought by and in the name of the executor or administrator of the person deceased; and
in every such action the jury may give such damages as they may think proportioned to the injury resulting from such death
to the parties respectively for whom and for whose benefit such action shall be brought; and the amount so recovered, after
deducting the costs not recovered, from the defendant, shall be divided amongst the beforementioned parties in such shares
as the jury by their verdict shall find and direct.

In my opinion, the interposition of F P Malyon Ltd if that is how it should be regarded, does not prevent the court assessing
truly the loss which the plaintiff has suffered. The husbands business, F P Malyon Ltd has been destroyed by the loss of the
husband, and it is clear that its revenue was, in substance, derived from him. The decision in Salomon v Salomon & Co Ltd need
not blind one to the essential facts of dependency and require a finding of fact which is contrary to the true financial position as
distinct from an artificial or fictitious one.
Burgess v Florence Nightingale Hospital for Gentlewomen Management Committee was relied on by the defendant before
the judge and before us, but I find that a very different case which has no relevance in the present dispute. The plaintiff might
well have earned nothing if she had not been married or would not have earned it in that company or in that way, but then no
question of dependency would have arisen. The question is what, if anything, did she in fact earn so as to supplement the family
fund, in this case so as to produce a larger income from the business. If someone else had had to be employed to do her work, the
services would have had to be paid for from the revenue of the business and, being an expense, would have reduced the profits.
On behalf of the plaintiff it was submitted that her allocation came from F P Malyon Ltd because her husband performed the
services which brought it about and she was dependent on him for the whole sum credited to her by the company, and Franklin v
South Eastern Ry Co was quoted in support. The plaintiff Franklin was paid 3s 6d a week as a light porter at St Thomass
Hospital. His son, aged twenty-one, who was killed on the defendants railway, through the defendants negligence, had for some
time done the fathers work for him as the father was ill. It was held that the fact that the son had assisted the father to the value
of 3s 6d a week was sufficient to maintain the action. Indeed, it was not necessary to establish an actual benefit; a reasonable
expectation of benefit from the son would have been sufficient. I think that that case and the observations of Pollock CB do lend
support to the argument, but, on the present facts, I am of opinion that the plaintiffs services, which were on paper so
handsomely remunerated, should not in this cold calculation of financial loss be treated as of no value. She was a director and
the secretary of the company with at least nominal duties as such, and from time to time she rendered assistance of a clerical
nature. I would assess this, on such meagre evidence as is available, at 200 with no likelihood of increase in services to be
rendered and reduce the dependency on the husband, therefore, to 1,175. This would give a total sum of 15,275 under the Fatal
Accidents Acts, less such amounts of the husbands estate which have to be taken into consideration. (His Lordship stated the
facts as to the shares received by the wife under the will (see p 347, letter g, ante), 349and the insurance policy, and continued:]
These brief facts are relied on by the defendant to remove the sum of 2,000, which so obviously was insurance money and
which in this indirect way benefited the estate of the husband to which the plaintiff succeeded, from the effect of s 2(1) of the
Fatal Accidents Act, 1959, which is as follows:

In assessing damages in respect of a persons death in any action under the Fatal Accidents Act, 1846, or under the
Carriage by Air Act, 1932, there shall not be taken into account any insurance money, benefit, pension or gratuity which
has been or will or may be paid as a result of the death;

and then there is a definition is sub-s (2) of that section: insurance money includes a return of premiums.
Mocatta J held that the wife benefited to the extent of 2,000 from the insurance money which became payable on the death
of her husband and that the Act of 1959 applied to that benefit and that it did not fall to be set off against the financial loss. He
thought it irrelevant that the company had continued trading and that some part of the 2,000 may have been lost. The defendant
relied on Green v Russell, and in particular on some passages in the judgment of Romer LJ ([1959] 2 All ER at p 533; [1959] 2
QB at pp 243, 244), with which Hodson LJ agreed. That decision was under the then appropriate provision, s 1 of the Fatal
Accidents (Damages) Act, 1908, and the majoirity held that the expression paid or payable on the death of the deceased under
any contract of assurance could not be accepted as descriptive of the sum received. The trial judge, Ashworth J ([1958] 3 All ER
44 at p 49; [1959] 1 QB at p 42) and Pearce LJ ([1959] 2 All ER at p 536; [1959] 2 QB at p 248), in the Court of Appeal thought
that it could. Although the main purpose of the Act of 1959 was, it would appear, to enlarge the scope of exemptions, I think that
the different phraseology now supports a wider view and does away with some of the rather narrow controversies which had
previously arisen. The increased value of 2 a share unquestionably arose from the insurance money and that benefit accrued to
the plaintiff. I am in complete agreement with the learned judge on this issue.
In the result, I would allow the appeal to the extent of reducing the damages to 14,748, of which 123 will be recovered
under the Law Reform (Miscellaneous Provisions) Act, 1934.

PEARSON LJ stated the facts and continued. On those facts, the first question in the appeal arises in this way. The figure of
what is conveniently called the dependency was found by the learned judge to be 1,375 per annum. The relevant dependency
is the hypothetical future dependency, that is to say the amount of the benefits which would have been received by the dependants
from the deceased husband if he had continued to live, but, in computing it, the past dependency is naturally taken as the basis
subject to any necessary adjustments. In ascertaining the past dependency, the learned judge took the full amount of the sums
drawn from the husbands bank account and applied for the benefit of the dependants, notwithstanding the fact that the sums paid
into that bank account in each year of the companys trading included the sum allocated in the accounts at the end of the year to
the plaintiff as directors salary. It is conceded that all the money so allocated to the plaintiff must be assumed to have been
included in the sums drawn out and applied for the benefit of the dependants.
For the defendant, it was contended, in reliance on Burgess v Florence Nightingale Hospital for Gentlewomen Management
Committee, that so much of the alleged dependency as consisted of the plaintiffs salary should be excluded, on the ground that it
was not attributable to the husband and wife relationship, but 350 to some other relationship. There was some difficulty in
finding a plausible definition of the other relationship. In the Burgess case, the wife and the husband had been professional
dancing partners, and the loss sustained by the husband by being deprived of his dancing partner was a separate and important
head of damage. In the present case, it would be inappropriate ad unconvincing to say that the plaintiff has lost her salary by
being deprived of her co-director or managing director or travelling salesman. The fact is that she has lost her husband, who was
the family breadwinner. However, the defendant still has the very simple argument that, in so far as the plaintiff was earning her
own living, she was to that extent not dependent on her husband. She was doing work for the company and assuming
responsibility as director and secretary, and was reasonably entitled to be remunerated for her services. If her husband had lived,
this state of affairs would presumably have continued. Therefore, prima facie the amount of the plaintiffs salary should be
deducted from the dependency. On behalf of the plaintiff, two answers are put forward. First, it is suggested that the plaintiffs
opportunity of earning the salary was entirely derived from the husbands activities in carrying on the business and that, in
consequence of his death, the plaintiff has been deprived of that opportunity. Secondly, it is suggested that the plaintiffs salary
was wholly or in part fictitious, being really part of the husbands earnings but nominally credited to the plaintiff in order to
secure tax advantages. Both these suggestions seem to me sound in principle, but only partially acceptable on the facts of this
case. It is clear from the evidence and from the findings of the learned judge, to which Sellers LJ has referred, that the wifes
services to the company, even when the assumption of responsibility as director and secretary is taken into account, were worth
much less than the salary which was allocated to her. She was, however, rendering useful services to the company and was
reasonably entitled to some remuneration. It is also clear from the evidence that, by the death of her husband, she has lost the
opportunity of receiving a comparatively large salary for rather minor services, but she has not been deprived of her earning
capacity, as appears (a) from the fact that she had by the date of trial carried on the business for a year and a half profitably
though with diminishing turnover and (b) from her expectation of being able to earn money by part-time work in the future.
In my view, the figure of dependency, 1,375 per annum, should be reduced, not by the excessive amount of salary which
was nominally allocated to her, but by so much of it as she really earned, that is the amount of a reasonable remuneration for her
services, including her assumption of the responsibility as director and secretary. I agree that a reasonable remuneration would be
at the rate of 200 per annum net, and the dependency should be reduced accordingly.
The other question in this appeal arises under s 2(1) of the Fatal Accidents Act, 1959, which provides that

In assessing damages in respect of a persons death in any action under the Fatal Accidents Act, 1846 there shall
not be taken into account any insurance money, benefit, pension or gratuity which has been or will or may be paid as a
result of the death.

F P Malyon Ltd had taken out with Royal Insurance Co Ltd a personal accident insurance policy, which provided (inter alia) that,
if the husband lost his life by accidental bodily injury, the insurance company should pay to the insured, which was F P Malyon
Ltd the sum of 2,000. Consequently, when the husband was killed, that sum of 2,000 became payable to F P Malyon Ltd and it
was in fact paid to that company. It was a sum of insurance money and it was to be paid, and was in fact paid, as a result of the
death of the husband. Therefore, under the terms of the section it is not to be taken into account in assessing the damages. One
of the accounting operations which have to be performed in the assessment of the damages in this case is to value the 351 shares
in the company, because the plaintiff has to give credit for the value of the 999 shares which she inherited from her husband.
There is no dispute as to the facts: if the 2,000 is left out of account, the companys shares were, at the date of the husbands
death, worth 14s 6d each: if the 2,000 is taken into account, the companys shares were, at the date of the husbands death,
worth 2 14s 6d each. The enactment requires the 2,000, being insurance money payable as the result of the death of the
husband, to be left out of account. Therefore, in the assessment of damages the value of the shares is to be reckoned as 14s 6d
each. That seems to me a very clear conclusion from the application of the enactment in its literal sense, which is also a
reasonable sense. The enactment does not say who the payer or payee is to be, and, therefore, does not create any limited class of
payers or payees. Any payment of insurance money as a result of the death is to be left out of account in the assessment of the
damages. If some such payment would in any case not have come into the calculation, the enactment makes no difference. Any
such payment which otherwise would have come into the calculation is excluded by the enactment.
Counsel for the defendant sought to impose a limitation on the class of payees by citation of a passage in the majority
judgment of the Court of Appeal in Green v Russell ([1959] 2 All ER at p 533; [1959] 2 QB at pp 243, 244), where Romer LJ
said, with reference to s 1 of the Fatal Accidents (Damages) Act, 1908,

The payer would normally be the insurer having regard to the words paid under any contract of assurance or
insurance. As to the payee, it seems clear that as the payment is relevant to the assessment of damages under the Fatal
Accidents Acts, 1846 to 1958, the payee or payees envisaged by the section must mean the estate or dependants of the
deceased employee.

That passage, however, was dealing with a different enactment and a different problem. Section 1 of the Act of 1908 provided
that

In assessing damages in any action under the Fatal Accidents Act, 1846, there shall not be taken into account
any sum paid or payable on the death of the deceased under any contract of assurance or insurance

The sum of insurance money falling due on the death of the deceased was, under the contract of insurance, payable to the
employer and did not belong in law or in equity to the estate of the deceased employee. It was, however, in accordance with the
intention and expectation of all concerned paid over by the employer to the estate of the deceased employee. It was held that the
payment under the contract of insurance was for the benefit of the estate and that the section applied to it. In the present case,
there is a simpler situation. Payment of the insurance money is made to the company for the benefit of the company, and, if taken
into account, it would affect very materially the value of the shares in the assessment of damages for the death of the husband. I
see no reason for reading into the enactment which applies in this case any provision for a limited class of payees.
In my view, therefore, the 2,000 should be left out of account in the assessment of damages, so that the shares in F P
Malyon Ltd should be taken to have been worth 14s 6d each and not 2 14s 6d each, at the relevant time, and the plaintiff has to
give credit for only 650 and not 2,650.
For these reasons, I agree that the appeal should be allowed to the extent stated in my Lords judgment, and that the order
should be as proposed by him.

DIPLOCK LJ. When, after the lapse of centuries, Parliament in 1846 reintroduced the principle of blood-money, it was, as the
courts soon decided, in order to protect the children of the poor rather than to punish the wrongdoer. Section 2 of the Fatal
Accidents Act, 1846, provided that the right of action in respect of the death of the deceased should be for the benefit of the
wife, husband, parent, and child of the deceaseda category to which the Fatal Accidents Act, 1959, has addedand enacted
352

the jury may give such damages as they may think proportioned to the injury resulting from such death to the parties
respectively for whom and for whose benefit such action shall be brought.

It has, however, long been established, despite these wide words, first, that the pecuniary loss to the persons for whose benefit the
action is brought is the only damage recoverable (see Blake v Midland Ry Co, and, secondly, that the pecuniary loss recoverable
is limited to the loss of a benefit in money or moneys-worth which, if the deceased had survived, would have accrued to a person
within the defined relationship to the deceased and would have arisen from that relationship and not otherwise (see the analysis of
the cases in Burgess v Florence Nightingale Hospital for Gentlewomen Management Committee).
The pecuniary loss which the court has to assess is a loss which will be sustained in the future. This involves making two
estimates, videlicet (i) what benefit in money or moneys-worth arising out of the relationship would have accrued to the person
for whom the action is brought from the deceased if the deceased had survived but has been lost by reason of his death, and (ii)
what benefit in money or moneys-worth (subject to certain statutory exceptions) the person for whom the action is brought will
derive from the death of the deceased which would not have been enjoyed had the deceased lived. The difference between these
two estimates is the measure of damages recoverable under the Fatal Accidents Acts, 1846 to 1959.
Because in most cases the most reliable guide as to what would happen in the future if the deceased had lived is what did in
fact happen in the past when he was alive, the common and convenient way of making the first estimate where the deceased at
the time of his death was the breadwinner of the family is (a) to ascertain what annual benefit in money or moneys-worth in fact
accrued to the person for whom the action is brought from the deceased and arising out of the relationship before the death of the
deceased, (b) to assess the extent (if any) to which that benefit would be likely to have increased or diminished in value in the
future if the deceased had lived, (c) to assess the number of years for which that benefit would have been likely to have continued
if the deceased had not been killed by the tortious act of the defendant, and (d) to apply to the annual benefit, assessed under (a)
and (c) and generally called the dependency, the appropriate multiplier derived from (c), allowance being made for the present
receipt of a capital sum in respect of annual losses which would be sustained in the future. But the fact that it is convenient to
have recourse to the past for guidance as to what would have been likely to happen in a hypothetical future which, owing to the
death of the deceased, will never occur, must not blind one to the fact that one is estimating a loss which will be sustained in the
future. The learned judge adopted this common method in the present case. It is conceded that it was an appropriate method, and
the only issue in respect of the first estimate is as to the amount of the dependency of the widow.
The second estimate is in general, as a result of statutory enactment, limited to pecuniary benefits to the person for whom
the action is brought derived from the estate of the deceased, although the allowance for the chance of a young widows
remarrying which is sometimes made in selecting the appropriate multiplier in the first estimate is really a deduction for a benefit
which the widow will derive from the death of the deceased which she would not have enjoyed had the deceased lived. In the
present appeal, however, the only question which arises in relation to the second estimate is the extent of the prohibition
contained in s 2 of the Fatal Accidents Act, 1959, against taking into account in assessing damages any insurance money
which has been or will or may be paid as a result of the death.
The dependency of the widow. The family income for several years before the death of the husband was almost wholly
derived from the profits of a private 353 company, F P Malyon Ltd in which the deceased husband held 999 shares of 1 each and
the plaintiff one share. The only two directors were the husband and the plaintiff. She was also secretary of the company.
During the course of the year moneys were transferred from the companys bank to that of the husband and recorded in the
companys books as advances against directors salaries, being debited as to part to the husband and as to a smaller part to the
plaintiff. She had no banking account of her own and the family expenses were met exclusively out of the husbands banking
account which was kept in funds in this way. At the end of the companys financial year the husband and the plaintiff decided
what salaries should be credited to them by the company in respect of the past year. These amounts were credited in the
companys books to the respective accounts of the directors and the company accounted to the Inland Revenue for the PAYE
income tax under Sch E payable by each in respect of the salaries so credited.
F P Malyon Ltds business, the merchanting of abrasive tools, was, in effect, the one-man business of the husband. It
depended, as events have shown, on his personal skill and connexions as the sole salesman, but he prudently chose to conduct it
through the legal machinery of a limited liability company, and, as is commonly done to obtain the tax advantage of earned
income relief, the profits of the business were distributed not in the form of dividends but in the form of salaries to himself and to
his wife. That the amount so available for distribution should be apportioned to the wife as well instead of to the husband alone
also has advantages from the point of view of income tax, and, provided that the wife does in fact perform services for the
company, this is a legitimate way of conducting ones affairs. If, however, the wife performs no services and the purported
service agreement between the company and the wife is a sham, this becomes a fraud on the revenue. No question of a fraud on
the revenue, however, arises in the present case, for the plaintiff did perform some services for the company, whose business was
conducted at the family home. The nature and extent of those services was dealt with very sketchily in the evidence and I must
return to them shortly. At the trial, the case presented on her behalf was that she received from her husband out of his banking
account for the benefit of herself and her children a sum which the learned judge estimated at between 1,300 and 1,400 a year.
No complaint is made about the figure of 1,375 which he arrived at as being the annual sum which she would have continued to
receive from the husbands banking account for the benefit of herself and her children if the husband had survived. The issue was
whether the total amount which she would have received from this source or some lesser annual sum was the amount of the
dependency. It was argued on behalf of the defendant that, as the accounts of the company disclosed, the husbands banking
account was ostensibly fed from two sources, viz, the salaries payable by the company to the plaintiff as well as to the husband
and that, to the extent to which the moneys from that account which was expended for the benefit of herself and her children were
derived from payments into that account by the company in respect of her salary, these formed no part of the dependency of
herself and her children on the husband.
It is, I think, beyond argument that, had the plaintiff, as so often happens today, been employed by some independent
employer, her salary, even though paid into her husbands banking account, would form no part of the dependency, for it would
continue after her husbands death. It would not be a benefit arising out of the relationship of husband and wife which she would
lose on his death. It seems to me to be equally clear that, if she were genuinely employed at the market rate of wages for the
services which she performed, whether directly by her husband or by a one-man company which he controlled, the position
would be no different. Her salary would not arise from the relationship of husband and wife, but out of the relationship of
employer and employee. If one looks to the future, as the Fatal Accidents Act, 1846, requires, it would not be a benefit which the
plaintiff would lose on her husbands death, for her earning capacity would 354 remain unimpaired and she could continue to
earn similar wages from another employer. If one looks to the past for the purpose of making the common estimate of the
dependency, her wages, although paid to her by her husband, should be ignored. The family income would have been
augmented to that extent by her own efforts, not those of her husband, for, if she had not been performing the services, her
husband would have had to pay wages to someone else for the services, and the amount available from him for the family
expenses would have been correspondingly reduced. It would be the converse of the position in Sykes v North Eastern Ry Co.
On the other hand, the existence of the relationship of employer and employee between the deceased and the person for whom the
action is brought is not conclusive that no part of the salary paid is a benefit arising from the relationship of husband and wife or
parent and child as the case may be, which, if lost by reason of the death of the deceased, can be taken into account in assessing
the damages recoverable under the Fatal Accidents Acts, 1846 to 1959.
In one of the early cases under the Fatal Accidents Act, 1846, Franklin v South Eastern Ry Co, a father earned 3s 6d a week
for carrying coals. The work was in fact done for him by his son, the deceased, gratuitously, and the court held that there was
here a benefit or dependency of 3s 6d per week which the father lost on the death of the son. If, 3s 6d being the market rate,
the father had paid the son 6d per week, the son forgoing the balance from motives of filial piety, it seems to me plain that there
would have been a benefit or dependency of 3s per week (see Burgess v Florence Nightingale Hospital for Gentlewomen
Management Committee ([1955] 1 All ER at p 517; [1955] 1 QB at p 360), and the passage there cited from the judgment of
Palles CB, in Hull v Great Northern Ry Co of Ireland ((1890), 26 LR Ir 289 at p 293)).
The same principle would apply to the converse case where a husband employs a wife at a salary greater than the market
rate for the services which she in fact renders if the motive for the payment of the excess is the connubial relationship between
them. In such a case there are two elements in the salary. In part, it is payment for services rendered by the wife as employee to
employer; as to balance, it is a benefit to the wife arising from the relationship of husband and wife. As to the first part,
representing the market value of the wifes services, it is not a benefit in money which she will lose on the death of her husband,
for the reasons which I have already discussed but as regards the balance, representing the excess of the actual salary paid to her
over the market value of the services actually rendered, this does arise from the relationship of husband and wife. It is merely a
wayadvantageous and, so far as I am aware, not unlawful under income tax lawof meeting part of the family expenses out of
the husbands resources. This part of the wifes salary is a benefit in money which she would lose on the death of her husband,
for she could not thereafter earn more than the market rate of payment for her services from another employer; or, looking to the
past to make the common estimate of the dependency, if she had not been performing the services for which the total salary
was paid, her husband would have paid someone else only the market rate for performing them, and the balance would have been
available from him for the family expenses. In such a case, it seems to me that the court, in assessing the damages to which the
widow is entitled under the Fatal Accidents Acts, 1846 to 1959, should include in the pecuniary benefit arising from the
relationship of husband and wife which she has lost on the death of the deceased that part, and only that part, of the salary
previously paid to her by her husband which was in excess of the market rate of salary for the services which she actually
performed. There was in fact a decision to this effect by Barry J on 21 May 1962, in Weller v OBriend, but this is unreported
and was not drawn to the attention of Mocatta J, in the present case.
355
________________________________________
d Reported on another point, [1962] 3 All ER 65

I have so far dealt with the law on the assumption that the contract of employment is between husband and wife. In the
present case, the contract of employment of the plaintiff was not with the husband himself but with the company. Does this make
any difference? Counsel for the defendant has strongly contended that it does. Each case must depend on its own facts; but the
Fatal Accidents Acts do, I think, require the courts to approach the question What benefit in money or moneys-worth arising out
of the relationship of husband and wife would have accrued to the widow from the deceased if the deceased had survived but has
been lost by reason of his death? as a jury would approach that question, realistically, and should not be astute to defeat the
object of the Acts by reliance on legal fictions. In the present case, the husband and wife were the sole directors and shareholders
of the company. They were its directing will and mind, with no duty to exercise it in the interests of anyone but themselves as
sole shareholders. If they decided that, because of their relationship of husband and wife, it was in their interests that the
company should pay to the plaintiff a salary greater than the value of her services to the company instead of distributing the
excess in dividends or salary to the husband, it seems to me that that excess of her actual salary over the market value of her
services is a benefit which arises from the relationship of husband and wife and, if lost by reason of his death, should be taken
into account in assessing the damages recoverable under the Fatal Accidents Acts, 1846 to 1959.
Applying these principles to the present case, how does the evidence stand? The husband and the plaintiff determined at the
end of each financial year what their respective salaries should be. In 1958 and 1959, the plaintiffs salary was 600 as compared
with 1,600 and 1,450 to the husband; in 1960, the last complete year before the accident, the plaintiffs salary was 800, the
husbands 1,800. On this state of facts, the onus, in my view, lay on the plaintiff to prove the extent to which what, on its face,
was a payment to the plaintiff arising out of an employer and employee relationship between the company and herself was in
reality a pecuniary benefit to her arising out of the connubial relationship between her husband and herself which she had lost on
the death of her husband. As was said by Palles CB, in Hull v Great Northern Ry Co of Ireland ((1890), 26 LR Ir at p 294):

if that loss [i.e., the loss claimed under the Fatal Accidents Acts] depends upon something which may or may not
exist, but the existence of which has not been proved, and cannot reasonably be inferred from the facts in proof, the
plaintiff must fail, and if the fact in question be one peculiarly within the knowledge of the plaintiff, and not of the
defendants, the difficulty of inferring it, when not directly proved, is increased.

The plaintiff in the present case should have led evidence as to the precise nature of the services which she in fact performed, the
time which she devoted to those services, and the current rates of salary in the district for comparable services. She did not do
this and, in the result, the material from which to draw any inference is exiguous. She described her work as:

Telephone; any correspondence which was necessary, parcelling up, say, anything vital to go off or anything cropped
up, and attend to the correspondence generally.

When asked Were you part-time or full-time?, she replied Part-time, I would presume. Her husband was out all day
canvassing for, and delivering, orders but worked morning and evening before he left and after he returned. The only other
employee was a part-time book-keeper at a salary of about 200 per annum. There were two small boys, and there was some
evidence which was not gone into in any detail that, during the school-holidays, the plaintiff absented herself from her home,
where the business was conducted, to stay with the boys in a caravan, but how often and for what periods did not emerge, nor
what arrangements, if any, were made for doing her work for the business while she 356 was absent. The learned judge, no doubt
because so little stress was laid on the matter in the evidence, seems to me, with respect, to have misapprehended it in his
judgment, for he appears to have regarded the other employee as responsible for the correspondence with occasional assistance
from the plaintiff, whereas the evidence was that the other employee only did the book-keeping and that the plaintiff attended to
the correspondence. It must, I think, have been this misapprehension which led him to say that the work done by the plaintiff in
connexion with the business was little more than nominal. This, on the evidence, is, I think, an understatement.
On the other hand, exiguous though the evidence is, I think that there is sufficient to enable the court to draw the inference
that not all of the salary of 600 or 800 a year representing the market value of her services to the company and arose from the
employer and employee relationship. It represents 12 or 16 a week, which is out of scale for part-time and intermittent work of
the kind which she described, particularly when contrasted with the 4 per week or so paid to the part-time book-keeper. Her
work may have been more responsible, and she had in addition responsibilities as a director and as secretary of the company. The
evidence does, I think, compel the inference that a substantial part of her salary arose from the husband and wife relationship, and
represented a convenient machinery through which she received moneys from her husband via the company for meeting the
household expenses. Making the best estimate I can on the material available, and bearing in mind the view which the judge who
had seen her took of the extent of her services, although, as I think, misapprehending in part the effect of her evidence, I think
that a fair jury estimate would be to ascribe 200 per annum of her total salary as payment for the services which she rendered to
the company and thus as arising from the relationship of employer and employee, and to be left out of account in estimating the
dependency. This is a benefit which she has not lost by the death of the deceased. Her earning capacity is unimpaired and
indeed, as she said in evidence, she is contemplating undertaking part-time work for some other employer. This means that the
dependency to which the learned judge applied a multiplier of thirteen years falls to be reduced by 200 and the damages
reduced by 2,600.
The insurance money. This is a short point which admits of little elaboration. The company took out and paid the premiums
on an accident insurance policy on the lives of both husband and wife. On the death of the husband, the sum insured on his life,
namely, 2,000, was paid to the company. The plaintiff inherited the husbands shares in the company. They were valued at the
date of his death on an assets basis, which it is conceded was the correct method of valuation. It is conceded also that the shares
were a benefit in moneys-worth which the plaintiff derived from the death of the husband and which she would not have enjoyed
if the husband had lived. The principal asset of the company of which account was taken in valuing the shares was the sum of
2,000 receivable by the company under the accident insurance policy on the death of the husband. The only question is whether
s 2 of the Fatal Accidents Act, 1959, requires that this sum shall not be taken into account in assessing the damages payable in
respect of the husbands death as being insurance money which has been or will or may be paid as a result of the death.
I do not think that Green v Russell, to which we were referred, assists in the solution of this problem. It was decided on the
wording of the Fatal Accidents (Damages) Act, 1908, which was in different terms. I must confess that I have felt more doubt
than my brethren as to the answer. The alternative views are: (i) that what should have been taken into account was the value of
the shares; the insurance money received by the company and credited in its books to compensation for loss of goodwill was
not taken into account qua insurance money but was merely an asset of the company which affected the value of its 357 shares;
(ii) that since the benefit received by the widow was the shares, and the only method of valuing that benefit was by valuing the
assets which lay behind the shares of which the principal asset was the insurance money in the hands of the company, the
insurance money would be taken into account qua insurance money in estimating the value of the benefit received by the widow.
The choice between these alternative views seems to me to be fairly evenly balanced. My brethren prefer the latter and I am not
disposed to disagree, more particularly as I suspect that the only reason that the policy was taken out by the company instead of
the husband and wife was for tax reasons, and I should be reluctant to decide that she must lose 2,000 on a technicality resulting
from the ingenuity of her husband and his accountant in which I do not suppose that she played any active part.

Appeal allowed in part. Judgment below varied. Balance of award remaining unpaid in to be paid into court; thereafter whole
sum to be paid out to plaintiffs solicitors. Leave to appeal to House of Lords refused.

Solicitors: Smiles & Co agents for Mossop & Bowser, Wisbech (for the defendant); Greenwoods, Peterborough (for the plaintiff).

Henry Summerfield Esq Barrister.


[1963] 2 All ER 358

Heywood and another v BDC Properties Ltd


LAND; Land Registration
CHANCERY DIVISION
PENNYCUICK J
23 APRIL 1963

Land Charge Vacation of entry in register Interlocutory motion Land charge registered by defendants in respect of estate
contract by specified correspondence Action by plaintiffs for declaration that no contract existed Motion for order for
vacation of entry granted Land Charges Act, 1925 (15 & 16 Geo 5 c 22), s 10(8).

Where a land charge has been registered in respect of an estate contract alleged to have been created by specified correspondence,
and the correspondence clearly shows that no binding contract for the sale of the land has been entered into thereby, an order for
vacation of the registration of the land charge may properly be made (pursuant to s 10(8) of the Land Charges Act, 1925) on
application by interlocutory motion in an action.
Re Engalls Agreement ([1953] 2 All ER 503) distinguished.

Notes
As to vacation of land charges pursuant to order of court, see 23 Halsburys Laws (3rd Edn) 77, 78, para 161, text and notes (e)-
(h); and for a case on the subject, see 38 Digest (Repl) 882, 926.
For the Land Charges Act, 1925, see 20 Halsburys Statutes (2nd Edn) 1063.

Case referred to in judgment


Engalls Agreement, Re [1953] 2 All ER 503, [1953] 1 WLR 977, 38 Digest (Repl) 882, 926.

Motion
This was an application by notice of motion dated 28 March 1963, by the plaintiffs, Sidney George Heywood and Claude Henry
George Heywood in an action commenced by writ dated 28 December 1962, against the defendants, BDC Properties, Ltd. The
motion was for an order vacating the registration at HM Land Registry under the provisions of the Land Charges Act, 1925 (as
amended) of two land charges, class C(iv) numbered 102220/62 and 102221/62 against the plaintiffs respectively, both of which
had been registered in respect of an estate contract alleged to have been created by correspondence between 25 June and 14
September 1962, affecting the land mentioned below.
By their statement of claim the plaintiffs alleged among other matters, that in or about June, 1962, the defendants agreed
subject to contract to purchase 358 from the plaintiffs certain land, known as sites 4 to 13, Wear House Estate, Exeter; that it was
at all times the intention of the plaintiffs and the defendants that neither of them should be bound to sell or buy the land unless
and until a contract for sale and purchase thereof should be concluded between them by exchange of contracts; and that no such
exchange had taken place and no such contract had been concluded. The plaintiffs claimed a declaration that no contract for the
sale and purchase of the land subsisted between the plaintiffs and the defendants, and an order vacating the registration of charges
under the Land Charges Act, 1925, by the defendants against the plaintiffs in respect of an alleged contract for the sale and
purchase of the land, and damages for the defendants registration of, and failure to apply for vacation of, such charges. The
defendants by their defence traversed the allegation in the statement of claim.

J P Brookes for the plaintiffs.


P A W Merriton for the defendants.

23 April 1963. The following judgment was delivered.

PENNYCUICK J. By this motion the plaintiffs, Sidney George Heywood and Claude Henry George Heywood, seek against
BDC Properties, Ltd.

an order vacating the registration at H.M. Land Registry under the provisions of the Land Charges Act, 1925 (as
amended), of two land charges class C (iv) [giving the numbers] against the plaintiffs respectively

The action was commenced by writ dated 28 December 1962, and the statement of claim has been delivered. I will refer in a
moment to the defence.
The action arises out of certain negotiations for the sale by the plaintiffs to the defendants of a piece of land known as sites 4
to 13, Wear House Estate, Exeter. Those negotiations, so far as the material before me on this motion goes, are contained in a
bundle of correspondence which begins on 25 June 1962, and ends on 14 September 1962. I will not go through that
correspondence in detail. It will be sufficient for me to say that it is clear beyond argument that nothing in that correspondence
creates a binding contract between the plaintiffs and the defendants. The whole correspondence consists of negotiations subject
to contract, for the sale of the land, those negotiations being conducted on the basis that a contract would in due course be entered
into and that the parties would not be bound until a contract was entered into.
On 27 September 1962, the defendants caused to be registered at the Land Registry land charges which purported to be in
respect of an estate contract imposed or created by the correspondence between Messrs Beach & Beach, solicitors for the
defendants, and Messrs Dunn & Baker, solicitors for the plaintiffs, from 25 June 1962, to 14 September 1962. That is the
correspondence to which I have referred and which, as I have said, plainly on the face of it does not create any contract. The
evidence shows that the plaintiffs are in negotiation for the sale of certain adjoining land and that those negotiations are being
held up for the reason that the adjoining land includes a small strip which belongs to the Admiralty. The Admiralty has agreed,
subject to contract, to sell that land to the plaintiffs but the Admiralty is unwilling to proceed with the completion of the sale of
that land until the estate contract in regard to the land, the subject of this action, is vacated.
The present state of the action is that the plaintiffs delivered their statement of claim on 22 January. The defence is a short
document signed by the solicitors for the defendants. The only grounds of defence are as follows:

The defendants deny that there was any contract between themselves and the plaintiffs subject to contract. [I am not
clear what that means.] The defendants will contend that all the terms relative to this matter vis vis themselves and the
plaintiffs had been agreed and that they were entitled to register their interests under the Land Charges Act, 1925.
359

So far as the evidence on this motion goes there is an affidavit sworn on behalf of the defendants by Mr George Levy, a director
of the defendant company, who contends that the defendant company properly entered into a binding contract with the plaintiffs.
Then he takes a procedural point and proceeds:

I say that those documents [exhibited to the plaintiffs affidavit, that is, the correspondence to which I have referred]
by no means exhaust the relevant evidence in this case, and the defendants will, upon discovery (if the same be ordered),
disclose further or other documents relating to the matters in issue, but I submit that it would be oppressive if the
defendants were compelled by means of the motion now before the court, to give discovery at short notice and before the
close of pleadings.

Mr Levy thus refers to other documents but does not in any way specify them or indicate the existence of any document outside
the exhibited correspondence which might be regarded as establishing a binding contract contrary to the plain terms of the
correspondence which the plaintiffs exhibited.
It seems to me that in the circumstances it would be proper for this court by way of interlocutory relief on this motion to
discharge the entry in the registry. The claim in the action also includes a declaration and damages. If the defendants wish they
can set up by way of counterclaim the existence of a binding contract and if this claim succeeds and in the meanwhile the land
has been lost to them no doubt they will have their remedy in damages. I do not think, however, that there is any reason why the
defendants should be allowed to keep on foot pending the trial of the action, land charges based on the correspondence, which
quite plainly does not support the existence of a contract for the sale of the land which is the subject of the land charges.
I have been referred to Re Engalls Agreement, in which Vaisey J, refused an application by way of summons to discharge
the registration of an estate contract and pointed out most properly that this matter was consequential on the determination of the
question whether a contract had been made and should be dealt with upon the decision of that question. Vaisey J declared in that
case that there was an arguable question whether or not a contract had been entered into and that question would in due course
have to be decided in the action. I do not think that there is anything in that decision which makes it inappropriate by way of
interlocutory order to discharge the registration of an estate contract where, on the material before the court, including the
correspondence on which the registration of the land charges is based and the evidence in the affidavits, there is no prima facie
case of any kind for saying that any contract ever has been entered into. I propose accordingly to make an order vacating the
registration of the land charges.

Order accordingly.

Solicitors: Arnold Carter & Co agents for Dunn & Baker, Exeter (for the plaintiffs); Beach & Beach (for the defendants).

Jenifer Sandell Barrister.


360
[1963] 2 All ER 361

Re Birmingham Association of Building Trades Employers Agreement


COMPETITION: CONSTRUCTION

RESTRICTIVE PRACTICES COURT


MOCATTA J, SIR STANFORD COOPER, MR W L HEYWOOD AND MAJOR-GENERAL W E V ABRAHAM
29, 30, 31 OCTOBER, 1, 2, 5, 6, 7, 8, 9, 12, 13, 14, 15, 16, 19, 20, 21, 22, 23, 26, 27, 28, 29, 30 NOVEMBER, 3, 4, 5, 6, 7, 10, 11, 12, 13, 14 DECEMBER 1962, 10
APRIL 1963

Restrictive Trade Practices Reference Building trades association Standard forms of contract Bills of quantities
Daywork charges Recommendations and rules of association concerning use of forms, against tendering for contracts without
bills of quantities, against submitting priced bills of quantities with tenders, and for use of standard schedules of daywork
charges Whether restrictions within Restrictive Trade Practices Act, 1956, s 6 Whether removal of restrictions would deny
public substantial benefit Restrictive Trade Practices Act, 1956 (4 & 5 Eliz 2 c 68), s 6(1), (7), s 21(1) (b).

The Birmingham Association of Building Trades Employers (hereinafter called the Association) was one of 252 local
associations in England and Wales which were grouped into ten regional federations which in turn were affiliated to the National
Federation of Buildings Trades Employers (the National Federation). Members could be either individuals or companies, and
on joining a local association they automatically became members of the regional and National Federations. The constitution of
the National Federation provided that its council should have exclusive right to negotiate standard conditions of contract, and that
individual members should conform with the national rules and with instructions issued by the council. The rules of the local
associations also provided that the rules of the regional and National Federations were binding on individual members. In 1962
there were 13,846 members of the National Federation, thought to represent about two-thirds of the building industry. The
industry was highly competitive as to prices.
There were ten standard forms before the court, consisting of four main contract forms (which had had their origin in 1903
as result of collaboration between the National Federation and the Royal Institute of British Architects), two forms relating to war
damage repairs, two sub-contract forms (one of which, issued under the sanction of the National Federation and approved by the
National Federation of Plastering Contractors was the blue form), a form of tender for nominated suppliers and a Conditions
of Estimate form for use in relation to small works. All the forms stated that they were issued under the sanction of the National
Federation and in some cases other bodies, except the Conditions of Estimate form, which stated that it was the copyright of
the Federation. There was evidence from architects and others that the forms were well drafted and fair to all parties. At the date
of the reference the rules of the National Federation (r 26) and of the Association (r 51) were in identical terms. Rule 26 (here
divided for convenience into four parts) required that all members of the National Federation (i) shall conform with the
requirements and recommendations made from time to time in regard to any form of contract formally adopted by the [National]
Federation; (ii) that otherwise members shall on all occasions press for the use of such conditions of contract as may be issued
under the approval of the [National] Federation; (iii) Provided that in exceptional circumstances the council of the [National]
Federation may authorise the use of any other form of contract ; (iv) but on no consideration shall a member sign a form of
contract which has been objected to or declined by the [National] Federation on account of its arbitrary and inequitable
conditions unless the council shall otherwise decide. Between the date of reference and the hearing, part (ii) of r 26 was
amended by adding the words where such conditions 361 are appropriate. On 19 November 1962, during the hearing, the
Association sent to its members a circular stating that the only recommendation made with regard to standard forms was that
members should endeavour to persuade the building owner or his professional adviser to use the standard forms of contract either
unaltered or altered as little as possible, but, that if the persuasion were unsuccessful members were free to sign other forms; and
that if the Association had made or had appeared to make any other recommendation it was withdrawn. Similar circulars were
sent out at the same time by the Midland and the National Federations. The wording of the rules was to be altered as soon as
constitutionally possible. The National Federation admitted that apart from the rules and prior to 19 November 1962, the
Association had recommended its members to use the standard forms, and the court found as a fact that there had been
recommendations in regard to nine of the ten forms (viz, all except the blue form) that members should use these forms and no
others in the cases of contracts of the types for which they were designed. When the blue form was introduced its use had been
clearly stated to be non-obligatory, and the recommendation was no more than to press for its use. It was admitted that parts (i),
(iii) and (iv) of r 26 gave rise to restrictions which could not be justified. In relation to (ii) the court accepted that, owing to the
complexity of building operations and the relationship between those engaged in them, special skill and experience were needed
in the drafting of building contracts, and that the standard forms were well drafted and fair to all parties. In practice, however,
part (ii) of r 26 was a dead letter and members frequently entered into contracts on other forms without first pressing for the use
of the standard forms. Public authorities often had their own forms and would not be affected by pressure from builders, while
most private owners were advised by architects, whose advice was not affected by r 26 or pressure from builders and would not
be altered by the abolition of r 26.
At the date of the reference a rule, known as the national quantities rule, which was included in both the Associations and
the National Federations rules forbade members to tender in competition for contracts exceeding 4,000 without bills of
quantities being supplied, and required that, in the case of contracts for the repetitive construction of small houses, the bills of
quantities should be prepared in accordance with a specified code. The National Federations version of the rule was later altered
to provide for a figure of 8,000, and the Associations version was altered to provide for such a figure as should from time to
time be determined by the National Federation. The national quantities rule of the Association further provided that members of
the Association tendering for work outside its area were to conform to the rules of the relevant local association. In general, any
building owner inviting a tender for works in excess of 8,000 would have bills of quantities prepared whether the rule existed or
not, but this would not be so in the case of repetitive work, work where most of the cost arose from prime cost items provided by
sub-contractors, work which had to be done at speed, or projects to be designed as well as built by the contractor. The minimum
cost of a bill of quantities was 2 1/2 per cent of the contract value. It was not necessarily the case that tenders were lower where
bills of quantities were provided, nor was a reduction in the tender always sufficient to offset the cost of preparing the bill. In
practice the existence of the rule had only a marginal effect on the use of bills of quantities, and in those instances of large
contracts for which bills of quantities were not required tenders had been received which were satisfactory in both number and
price.
A further rule of the Association (r 29) forbade members to submit priced bills of quantities (known as priced schedules)
with their tenders, unless they had the consent of the presidents committee. There was little evidence as to the extent to which
this rule was observed.
362
At the time of the reference the Association had sought to maintain recommendations connected with national schedules of
daywork charges; these had since been abandoned and replaced by new recommendations. Daywork was work charged for on a
time and materials basis with a percentage addition for profits, and it could arise either when additions or variations were made to
existing contracts or in relation to jobbing or maintenance work. In each case the National Schedules, which had been prepared
by an independent accountant and after discussion with the Royal Institution of Chartered Surveyors, provided methods of
calculating the prices to be charged. The original recommendation had been that members should quote for daywork only at the
prices set out in the National Schedules, but this had been widely disregarded. During the hearing (by circular dated 23
November 1962) a new recommendation was made that the National Schedules should be used in the absence of agreement
between builder and client as to daywork charges for general building work; but that, if members were asked to quote for
daywork in advance, they were free to quote such rates as they thought fit. There was evidence that some builders charged for
daywork at less than the rates given in the National Schedules.
On a reference by the Registrar of Restrictive Trading Agreements under s 20(1) and s 20(2)(a) of the Restrictive Trade
Practices Act, 1956a, the Association denied that the recommendations as to the standard forms, r 26, the national quantities rule
(r 29), and the new recommendation as to the National Schedules gave rise to any restrictions within s 6(1) b of the Act. It was
admitted that the original recommendations as to the National Schedules gave rise to such a restriction. In the alternative the
Association 363 sought to justify any such restrictions under s 21(1)(b) of the Actc.
________________________________________
a Section 20, so far as relevant, provides: (1) The court shall have jurisdiction, on an application made in accordance with this section in
respect of any agreement of which particulars are for the time being registered under this Part of this Act, to declare whether or not any
restrictions by virtue of which this Part of this Act applies to the agreement (other than restrictions in respect of matters described in paras
(b) to (d) of s 8(8) of this Act) are contrary to the public interest.
(2) An application to the court under the foregoing subsections may be made(a) in any case, by the registrar
(3) Where any such restrictions are found by the court to be contrary to the public interest, the agreement shall be void in respect of these
restrictions; and the court may make such order as appears to the court to be proper for restraining all or any of the persons party to
the agreement (a) from giving effect to the agreement in respect of those restrictions;
(5) The powers of the court under this section shall not be affected by the determination of an agreement effected after the commencement of the
proceedings, and where an agreement is varied after the commencement of the proceedings, the court may make a declaration and, if it
thinks fit, an order under sub-s (3) or sub-s (4) of this section, either in respect of the agreement as at the commencement of the proceedings
or in respect of the agreement as varied, or both.
b Section 6(1) provides: Subject to the provisions of the two next following sections, this Part of this Act applies to any agreement between
two or more persons carrying on business within the United Kingdom in the production or supply of goods, or in the application to goods of
any process of manufacture, whether with or without other parties, being an agreement under which restrictions are accepted by two or more
parties in respect of the following matters, that is to say:
(a) the prices to be charged, quoted or paid for goods supplied, offered or acquired, or for the application of any process of manufacture of
goods;
(b) the terms or conditions on or subject to which goods are to be supplied or acquired or any such process is to be applied to goods;
(c) the quantities or descriptions of goods to be produced, supplied or acquired;
(d) the processes of manufacture to be applied to any goods, or the quantities or descriptions of goods to which any such process is to be applied;
or
(e) the persons or classes of person to, for or from whom, or the areas or places in or from which, goods are to be supplied or acquired, or any
such process applied.
c Section 21(1), so far as relevant, provides: For the purposes of any proceedings before the court under the last foregoing section, a
restriction accepted in pursuance of any agreement shall be deemed to be contrary to the public interest unless the court is satisfied of any
one or more of the following circumstances, that is to say
(b) that the removal of the restrictions would deny to the public as purchasers, consumers or users of any goods other specific and substantial
benefits or advantages enjoyed or likely to be enjoyed by them as such, whether by virtue of the restriction itself or of any arrangements or
operations resulting therefrom; and is further satisfied (in any such case) that the restriction is not unreasonable having regard to the
balance between these circumstances and any detriment to the public or to persons not parties to the agreement (being purchasers,
consumers or users of goods produced or sold by such parties, or persons engaged or seeking to become engaged in the trade or business of
selling such goods or of producing or selling similar goods) resulting or likely to result from the operation of the restriction.

Held (i) (As regards the standard forms other than the blue form) (a) the original recommendations with regard to these were
specific within the meaning of that word in s 6(7) of the Restrictive Practices Act, 1956, with the consequence that Part 1 of the
Act applied as if the constitution of the Association contained a term by which each member agreed to comply with the
recommendations; that agreement gave rise to restrictions under s 6(1)(b), which, as the original recommendations had been
abrogated, could not be defended under s 21(1)(b) (see p 370, letters c and f, post).
(b) There was jurisdiction by virtue of s 20(5) to make an order in relation to restrictions arising out of the original implied
agreement, and the court would declare them to be contrary to the public interest (see p 370, letter h, post).
(c) (As regards part (ii) of r 26 relating to the standard forms, parts (i), (iii), (iv) all being declared on admissions contrary to
the public interest (see p 371, letter a, post)) part (ii) of r 26, whether in its amended or unamended form, contained a restriction
(to the effect that, where conditions of contract appropriate to work had been issued under the approval of the National
Federation, building work would not be carried out under other conditions of contract, unless members should have pressed for
use of the approved conditions) and the court was not satisfied that, although the approved forms were useful forms, the removal
of the restriction would deny the public as building owners specific or substantial benefits for (among other reasons) the
usefulness of the forms did not depend to any significant extent on the existence or operation of r 26 and there was no evidence
leading the court to consider that their use would be affected by the abolition of r 26 (see p 371, letter e, p 372, letter d, and p 372,
letter i, to p 373, letter a, post); accordingly the restrictions accepted under r 26 would be declared to be contrary to the public
interest.
(ii) (As regards the national quantities rule) the rule gave rise to a negative obligation (viz, not to carry out general building
work under contracts for work exceeding 8,000 in value for persons calling for tenders in competition for the work without
supplying bills of quantities) which by virtue of s 6(3) was a restriction and fell within s 6(1)(e); this restriction did not confer
any specific and substantial benefit on the public as building owners, the removal of the restriction would not deny any such
benefit to the public, and the restriction would be declared to be contrary to the public interest (see p 376, letters b and h, and p
378, letter h, post).
(iii) (As regards r 29) r 29 gave rise to a restriction within s 6(1)(e), which for the same reasons as the national quantities
rule would be declared to be contrary to the public interest (see p 379, letter d, post).
(iv) As regards the National Schedules of Daywork Charges (it being admitted that the original restrictions under this head
could not be defended), the preferable construction of the new recommendation (of 19 November 1962) was that it applied only
if no agreement as to charges for daywork had been 364 made, and thus could take effect only after the work had been carried out
and would have no contractual effect; accordingly, the new recommendation was not of a price to be charged within s 6(1) and
did not give rise to any restriction falling within the jurisdiction of the court (see p 382, letters f and g, and p 383, letter b, post).
Re Blanket Manufacturers Associations Agreement ([1959] 2 All ER 630) considered.
Per Curiam: if, however, the National Schedules did give rise to a restriction within s 6(1), (7), the respondents had not
shown that the requirements of s 21(1)(b) were satisfied in relation to the restriction and it would have been declared to be
contrary to the public interest (see p 384, letter f, post).

Notes
As to the presumption of a restrictive trade agreement being contrary to the public interest, and as to grounds justifying
restrictions, see 38 Halsburys Laws (3rd Edn) 113, para 149, and p 115, para 151.
For the Restrictive Trade Practices Act, 1956, s 6, s 21, see 36 Halsburys Statutes (2nd Edn) 937, 954.

Cases referred to in judgment


Blanket Manufacturers Associations Agreement, Re [1959] 2 All ER 630, LR 1 RP 271, [1959] 1 WLR 1148, 3rd Digest Supp.
Chemists Federations Agreement, Re [1958] 3 All ER 448, LR 1 RP 75, [1958] 1 WLR 1192, 3rd Digest Supp.
Craven-Ellis v Canons Ltd [1936] 2 All ER 1066, [1936] 2 KB 403, 105 LJKB 767, 155 LT 376, Digest Supp.
Net Book Agreement 1957, Re [1962] 3 All ER 751, [1926] 1 WLR 1347.
Practice Direction [1958] 3 All ER 520, LR 1 RP 116, 3rd Digest Supp.
Tyre Trade Register Agreement, Re [1963] 1 All ER 890, [1963] 1 WLR 367.

Reference
Pursuant to the Restrictive Trade Practices Act, 1956, s 20(2)(a), the Registrar of Restrictive Trading Agreements referred to the
Restrictive Practices Court an agreement between the members of the Birmingham Association of Building Trade Employers.
The facts and the relevant documents are set out in the judgment of the court.

H A P Fisher QC, J B Elton and R K Batstone for the respondents.


T G Roche QC and R H W Dunn QC for the registrar.

Cur adv vult

10 April 1963. The following judgment was delivered.

MOCATTA J read the following judgment of the court. This reference, initiated by notice dated 15 December 1959, concerns an
agreement between the members of the Birmingham Association of Building Trades Employers (hereinafter called the
Association), but the National Federation of Building Trades Employers (hereinafter called the National Federation) appeared
as respondents in the reference representing all parties to the agreement.
The reason for what might seem a somewhat unusual representation order is to be found in the elaborate and interlocking
nature of the organisations of builders throughout England and Wales. Thus there are 252 local associations, including that of the
Birmingham builders. These in turn are grouped into ten regional federations, the Association being one of forty-six local
associations affiliated to the Midland Federation of Building Trades Employers (hereinafter called the Midland Federation).
The National Federation is, by its constitution, defined as a federal union of employers in the building industry composed of
regional and area or local groups, affiliation of members taking place through the section covering the locality in which the firm
in membership is situated. Members of local associations are also automatically members of the regional and National
Federations. The constitution of the National Federation provides (a) by r 15 that its council shall have exclusive rights in regard
to (inter alia) 365negotiations of standard conditions of contract of all kinds and with all concerned (b) by r 54 that members
shall conform individually with the requirements of national rules and with such instructions as the council may issue from time
to time in the exercise of its powers under r 15 and (c) by r 56 that the constitution and rules of all sections (which expression
includes regional federations and local associations) shall be consistent with the constitution and rules of the National Federation
and in particular shall contain provisions whereby instructions issued by the council in the exercise of its power under r 15 shall
be binding on all members forthwith. Rule 19 of the constitution of the Association states that each member of the Association
being automatically a member of the regional and National Federations, the rules of these federations shall, so far as they apply to
individuals, be binding on members of the Association. In addition all applications for membership of the Association must, by
r 5, be made on an official printed form containing a declaration that the applicant will abide by the rules or regulations of the
Association.
There were on 1 January 1962, 13,846 members of the National Federation, 2,082 members of the Midland Federation and
about two hundred members of the Association. A member may be an individual or a large public company. Of the 13,846
members, 8,250 employed under ten men and a further four thousand employed less than fifty men. It is not known with any
precision what proportion of the building industry as a whole is represented by the National Federation, but it is thought to be
about two-thirds. Except for a very small percentage of building work the industry is highly competitive as to prices.
The hearing of this reference, which occupied no less than thirty-three days, was concerned with four major topics which
were to a considerable extent disconnected and called for individual treatment. They were (i) recommendations or rules relating
to a number of standard forms, mainly of building contracts (ii) a rule of the Association and also of the National Federation
known as the national quantities rule, forbidding members in certain cases from tendering in competition for contracts exceeding
at the date of the reference 4,000 and from 2 May 1962, 8,000 in value without bills of quantities being supplied to those
invited to tender (iii) a rule of the Association forbidding the submission of priced schedules with tenders without the consent of
the presidents committee and (iv) recommendations of the Association as to (a) tendering in competition for daywork rates in
respect of daywork arising under building contracts and (b) as to charges for building work of a jobbing or maintenance
character, sometimes known as daywork or jobbing work. Changes were made in the recommendations of the Association on (i)
and (iv) during the course of the hearing and over much of the field the parties were at issue whether the recommendations or
rules gave rise to any restrictions within s 6 of the Restrictive Trade Practices Act, 1956. In most, though not all, cases the
respondents further sought to justify under s 21(1)(b) of the Act.
We deal in the first place with the various issues arising in relation to the standard forms, ten of which are before the court.
Two of these relate to the repair of war-damaged property and are now almost obsolete. Very little evidence was given in relation
to them. The four most important forms, each bearing the title Agreement and schedule of conditions of building contract,
have been prepared in slightly differing language depending on (a) whether the building owner is a local authority or not and (b)
whether bills of quantities form part of the contract or not. Thus, there are two forms for use by local authorities, one to be used
when bills of quantities form part of the contract and the other when they do not. The other two forms apply mutatis mutandis in
the case of private building owners. We call these four forms the main contract forms. Then there are two standard forms of sub-
contract, a form of tender to be used by nominated suppliers and, finally, a short document entitled Conditions of estimate for
use in connexion with small works. The first six of the above-mentioned forms state that they are
366

issued under the sanction of the Royal Institute of British Architects [hereinafter called the R.I.B.A.], the National
Federation of Building Trade Employers and the Royal Institution of Chartered Surveyors [hereinafter called the
R.I.C.S.].

The form of tender bears a very similar statement. One of the sub-contract forms, known as the buff form, states that it is

issued under the sanction of and approved by the National Federation of Building Trade Employers and the Federation
of Associations of specialists and Sub-Contractors

and the other, known as the blue form, that it is issued under the sanction of the National Federation and approved by the
National Federation of Plastering Contractors. The form entitled Conditions of estimate states that it is the copyright of the
National Federation.
The main forms of contract had their origin as far back as 1903 when, as the result of collaboration between the RIBA and
the National Federation, the first standard form of building contract was issued. Revised versions were issued in 1909 and 1931
and in the latter year there was set up a permanent standing committee of the National Federation and the RIBA, which later
came to be known as the Joint Contracts Tribunal (hereinafter called the JCT). In 1947 its membership was widened to include
representatives of the RICS, the body representing quantity surveyors, and in 1957 it was further enlarged by the inclusion of
representatives of six bodies representing local authorities, namely, the Association of Municipal Corporations, the County
Councils Association, the Urban District Councils Association, the Rural District Councils Association, the London County
Council and the Metropolitan Boroughs Association. The JCT contains no direct representation from the body of private building
owners, but it is claimed, with some justification, that such direct representation is impracticable and that the interests of private
building owners are looked after by the members from the RIBA and RICS. There is no representative on the JCT from any
government department. In 1937 an edition of the main contract forms adapted for use by local authorities was issued and the
forms for use by private building owners were reissued in revised form in 1939. There have been minor revisions of all four
main contract forms since then, the forms current at the time of the hearing of the reference having been issued in 1957. During
the year ending 31 March 1961, over ninety-one thousand of the main contract forms were sold. As regards the form of tender to
be used by nominated suppliers, this was negotiated between the National Federation, the RIBA and the RICS and issued in 1956.
The purpose of this form is to ensure that the architect invites quotations from prospective nominated suppliers on terms which
are not at variance with the terms of the main contracts. It was not explained in evidence why a similar form of tender has not
been produced for use by nominated sub-contractors whose function is, apart from supplying materials, to execute and complete
sub-contract works. As regards the two forms of sub-contracts neither the JCT nor the RIBA was responsible for their drafting or
issue. The buff form, designed for use as between the builder and sub-contractors nominated by the architect, was first published
in 1936, the current edition having been issued in 1950. The blue form, designed for use as between the builder and sub-
contractors not nominated by the architect was published in 1956. During the year ending 31 March 1961, 47,637 copies of the
buff form and 6,032 of the blue form were sold. Finally the form entitled Conditions of estimate for use in small works was
recently prepared by the National Federation without consultation with any other body and first published in July, 1961. During
the first four months after publication twenty-six thousand copies were sold.
Having thus briefly described the ten standard forms with which we are concerned and their origins and use, it is necessary
next to state the restrictions 367 or alleged restrictions that arise in relation to them. The position in regard to these is somewhat
complicated since not only did the Registrar twice amend his answer on this matter during the course of the hearing, but the
Association, the Midland Federation and the National Federation on and after 19 November 1962, during the course of the
hearing, issued certain circulars to their members in consequence of which the respondents amended their reply. At the date of
the reference r 26 of the Association and r 51 of the National Federation were in identical terms. Between the date of the
reference and the hearing each was amended by the insertion of the words where such conditions are appropriate. We set out
the language of these rules as amended, having inserted therein for ease of reference numbers dividing the rules into four parts:

All members of the Federation shall (i) conform with the requirements and recommendations made from time to time
in regard to any form of contract formally adopted by the Federation (ii) and otherwise members shall on all occasions
press for the use of such conditions of contract as may be issued under the approval of the Federation where such
conditions are appropriate. (iii) Provided that in exceptional circumstances the council of the Federation, direct or upon
recommendation of any regional federation, may authorise the use of any other form of contract, subject to such conditions
by way of amendment as may appear necessary; (iv) but on no consideration shall a member sign a form of contract which
has been objected to or declined by the Federation or by any section of account on of its arbitrary and inequitable
conditions, unless the council of the Federation shall otherwise decide.

Somewhat similar but more far-reaching provisions appeared in r 27(a) of the Midland Federations rules, but the respondents did
not seek to defend this rule in so far as it differed from the rules set out above.
On 19 November 1962, which was the sixteenth day of the hearing before this court, there was sent to all members of the
Association on the authority of the presidents committee the following circular:
Notice to all members of the Birmingham Association of Building Trades Employers. (Issued by the Birmingham
Association of Building Trades Employers to all its members). 1. Members will be familiar with r. 51 of the rules of the
National Federation of Building Trades Employers, r. 27(a) of the Midland Regional Federation of Building Trades
Employers and r. 26 of the Birmingham Association of Building Trades Employers. 2. It has been suggested that the
Birmingham Association of Building Trades Employers has made a specific recommendation to its members that they are
not on any occasion to carry out building work except on the terms and conditions of the standard forms of contract issued
under the approval of the National Federation. 3. This notice is to make it clear that the only recommendation made by the
Association in relation to the standard forms is that members shall on all occasions press for the use of these forms where
they are appropriate; that is to say that members shall endeavour to persuade the building owner or his professional adviser
to use the standard forms of contract unaltered or failing that altered as little as possible. If the pressure is unsuccessful,
then members are free to sign the form of contract proposed by or on behalf of the building owner. A similar
recommendation is made in relation to the standard forms of sub-contract and of tender. 4. If and in so far as at any time
the Birmingham Association of Building Trades Employers has made or appears to have made any different
recommendation concerning the standard forms, it is withdrawn.

Very similar circulars, in terms scheduled to the re-amended reply, issued by the Midland and National Federations on 16
November 1962, were sent to each member of the Association on 23 November 1962. The three circulars were confirmed by the
councils of the respective organisations. We were informed that owing to 368 the requirements of the respective constitutions of
the Association and the two Federations it was not possible to alter the wording of the three rules other than at meetings which
could not be assembled before the termination of the hearing of the reference, but that amendments to bring the rules into accord
with the circulars would be moved as soon as was constitutionally possible.
Apart from arguments based solely on the rules and the circulars, the Registrar contended that down to 19 November 1962,
when the Association issued its circular, there existed an express or implied recommendation by the Association to its members to
use the standard forms of contract, that by virtue of s 6(7) of the Restrictive Trade Practices Act, 1956, d the constitution of the
Association had to be read as if it contained a term by which each member of the Association agreed to comply with such
recommendation and that such a term gave rise to a restriction within s 6(1)(b) of the Act e, as made applicable to building work
by s 36(2) of the Actf This express or implied recommendation was, according to the argument of the registrar, more rigid than
the recommendation in the circulars to press for the use of the forms where these were appropriate and, although it had been
abrogated by the issue of the circulars, the registrar urged that not only had we jurisdiction to deal with the restriction arising out
of the implied agreement to comply with the recommendation, but that we should in the exercise of our discretion under s 20(5)
of the Actg deal with that restriction.
________________________________________
d Section 6(7) of the Restrictive Trade Practices Act, 1956, provides: Where specific recommendations (whether express or implied) are
made by or on behalf of a trade association and its members or to any class of its members, as to the action to be taken or not taken by them
in relation to any particular class of goods or process of manufacture in respect of any matter described in the said sub-s (1), this Part of this
Act shall apply in relation to the agreement for the constitution of the association notwithstanding any provisions to the contrary therein, as
if it contained a term by which each such member, and any person represented on the association by any such member, agreed to comply
with those recommendations and any subsequent recommendations made to them by or on behalf of the association as to the action to be
taken by them in relation to the same class of goods as process of manufacture and in respect of the same matter.
e The terms of s 6(1)(b) are stated in note +, p 363, ante
f Section 36(2) provides: This Act applies to the construction or carrying out of buildings, structures and other works by contractors, as it
applies to the supply of goods, and for the purposes of this Act any buildings, structures or other works so constructed or carried out shall be
deemed to be delivered at the place where they are constructed or carried out.
g The terms of s 20(5) are set out in note *, p 363, ante

In answer to these arguments the respondents admit that apart from the rules and prior to 19 November 1962, when the first
circular was issued, the Association recommended its members to use the standard forms. They say, however, that the
recommendation was ambiguous and was thus not a specific recommendation (whether express or implied) within s 6(7) of the
Act in that the recommendation was merely to use the standard forms and did not state what was to happen if the other parties to
the proposed building contracts would not use them. Alternatively it was argued that, since a building contract cannot be entered
into unless both building owner and builder are in agreement, the recommendation, even when translated by the words of the Act
into an implied agreement between members of the Association to use the standard forms, could not as a matter of construction
amount to more than an agreement between members to press for their use, and that accordingly the recommendation prior to 19
November 1962, was no different from that contained in the circular of that date. Reliance was further placed on oral evidence
given by architects, quantity surveyors and members of the Association that prior to 19 November 1962, they had always
understood the recommendation not to be a rigid one that only the standard forms must be used, but to be a recommendation to
the effect set out in the circular, namely, to press for the use of the standard forms. Certainly if the recommendation were a rigid
one it was not obeyed.
369
What then are the facts as to the pre-19 November 1962, recommendation? There is no doubt at all but that both the
National Federation and the Association recommended members to use the standard forms and a recommendation to this effect in
relation to all forms of contract mutually agreed between the RIBA and the National Federation was duly registered under the Act
by the latter. In addition to this and the statements on the various forms already mentioned as to their sponsorship, the Registrar
relied on a number of resolutions and letters in bundles 8 L, M and N. The importance of these was not as a support for the
recommendation to use standard forms, which was admitted, but as containing express or implied recommendations not to
contract other than on the terms of such forms. Having carefully considered these documents we find as a fact that they did
contain express or implied recommendations to use all of the ten standard forms, save the blue form, and no others in the case of
contracts of the types for which they were designed. In other words the recommendations in relation to these nine forms were of
a rigid character and went beyond the recommendation in the circular of 19 November 1962. We consider the recommendations
were unambiguous and specific within the meaning of that word in s 6(7) of the Act and in consequence of that subsection each
member of the Association impliedly agreed to comply with such recommendations. Such agreement clearly, in our judgment,
gave rise to restrictions under s 6(1)(b) of the Act.
As regards the blue form, namely the form for non-nominated sub-contractors, when this was introduced in 1956 its use was
clearly stated to be in no way obligatory. In these special circumstances we think that the relevant recommendation was no more
than to press for its use and did not differ in any way from the recommendation in the circular of 19 November 1962. In relation
to this form, therefore, we deal later in this judgment with the position under the latter recommendation.
Since the original recommendation in relation to the nine forms has been abrogated, it is clear that the restrictions arising
from the agreement implied by the statute to comply with that recommendation cannot be defended h under s 21(1)(b) of the
Restrictive Trade Practices Act, 1956. Counsel for the respondents argued, however, that we had no jurisdiction to deal with the
implied agreement to comply with the original recommendation, since by s 6(7) the implied agreement was to comply with that
recommendation and, in the words of the subsection, any subsequent recommendationsas to the action to be takenin
relation to the same class of goods or process of manufacture and in respect of the same matter. There was, he said, only one
implied agreement and the court could only deal with it in relation to the current recommendation. In our judgment, however, the
circular of 19 November 1962, constituted a variation of the original recommendation and gave rise to a new implied agreement
varying the original one. Accordingly s 20(5) of the Act applies and we have jurisdiction to make an order in relation to the
restrictions arising out of the original implied agreement. We think it right, pursuant to the discretion given by that subsection to
declare such restrictions contrary to the public interest.
________________________________________
h The terms of s 21(1)(b) are set out in note *, p 364, ante

We now pass to consider r 26 of the Association both with and without the amending words where such conditions are
appropriate. For the purposes of ease of reference in argument, this somewhat lengthy rule was divided into the four parts which
we have indicated by the use of numbers. It was made clear by counsel for the respondents that had time permitted the
Association would have taken the necessary constitutional steps to delete parts (i) and (iii) of this rule with the twofold object of
getting rid of restrictions which could not be justified under s 21(1)(b) and also avoiding any adverse implications as to the
meaning of part (ii) deriving from those two parts. We have, however, to deal 370 with the rule both as it was at the date of the
reference and as it now is with the amendment we have mentioned. As regards part (i) it was admitted by counsel for the
respondents in his final speech that this gave rise to a restriction and he did not seek to justify this. As regards part (iii) a similar
attitude was adopted by the respondents throughout the hearing. As regards part (iv) it was admitted that this gave rise to a
restriction and in his final speech counsel for the respondents was unable to justify it. Accordingly all three restrictions must be
declared contrary to the public interest.
That leaves part (ii) for consideration. Counsel argued for the respondents that this part both in its unamended and amended
forms on its true construction meant no more than is stated in the circular issued on 19 November 1962. The recommendation to
members contained therein, namely, to press for the use of the standard forms where appropriate but, if the pressure is
unsuccessful, leaving members free to sign the form of contract proposed by or on behalf of the building owner, was originally
said on behalf of the respondents, when translated by the Restrictive Trade Practices Act, 1956, into an implied agreement by
members of the Association to the same effect, not to give rise to a restriction under s 6(1)(b). In support of this argument it was
said that the only restriction arising was one on tendering on terms other than those in one of the standard forms where
appropriate and our attention was drawn to the use of the word offered in s 6(1)(b) and its absence from the next sub-paragraph.
In his final speech, however, counsel for the respondents did not seriously contest that restrictions within s 6(1)(b) arose and in so
doing we think that he was right in view of the provision in s 6(3) that a restriction includes any negative obligation, whether
absolute or not. We accept the statement of the restriction arising from the recent recommendation in relation to the main
standard forms appearing as No 5 in Part I of the amended schedule to the further re-amended answer, reading as follows:

Where conditions of contract appropriate to the work have been issued under the approval of the [National]
Federation, not to carry out building work under other conditions of contract unless members shall have pressed for the use
of the said approved conditions.

The statement may require some re-drafting in relation to some of the other standard forms, particularly the two forms of sub-
contract. Since part (ii) of the rule, in its amended or unamended form, at the lowest contains a similar restriction, it is
convenient at this stage to consider the case advanced under s 21(1)(b) in justification of this restriction.
It is said, and we accept it, that owing to (i) the complexity of most building operations, where hardly any two are alike
owing to differences between sites and building owners requirements, (ii) the complicated relationships arising concerning not
only builder and building owner, but also architects, quantity surveyors, and sub-contractors and (iii) the need for making
provision in advance for the almost inevitable changes and variations that take place in the course of construction, special skill
and experience are required in drafting building contracts. That skill and experience, representative of the great majority of
parties interested, has been brought to bear on the production of the seven standard forms issued with the sanction of the RIBA,
since they are the work of the JCT, which is a standing body constantly considering the current forms and from time to time
revising them in order to effect improvements and keep abreast of developments. Indeed, new and much re-drafted editions of
each of the four main contract forms, resulting from several years work by the JCT, were finally approved by its constituent
bodies during the hearing of the reference and we were told that they would by now be available to the public. The public as
building owners benefit, it was said, by the availability of such forms and by their use wherever appropriate because they are fair
to all parties and well known to builders. The latter point was said to be of importance in keeping tender prices down, since 371
builders invited to tender on unfamiliar conditions are, as we find, apt to raise their tenders in order to cover unknown contractual
contingencies. Similarly the use of well-drafted and well-known forms was said to lessen the likelihood of disputes with
consequential waste of time and money. Many similar arguments were advanced in relation to the three forms not resulting from
the deliberations of the JCT, namely the two sub-contract forms and the conditions of estimate, it being stressed in relation to the
two former that it was important, as was admitted by the registrar, that sub-contract forms should be geared into the main contract
forms. We were not invited to examine many of the detailed provisions of the ten forms before us or any of the provisions of the
newly drafted forms, but there was a body of evidence called comprising architects, quantity surveyors and builders to the effect
that whilst no standard form could be perfect, the forms before us were well drafted and were fair to the parties concerned.
We accept on the evidence that the two standard forms before us are useful documents and, in view of the present
composition of the JCT, we have little doubt that the four newly drafted forms will also be useful documents. What we have to
be satisfied of under s 21(1)(b) of the Restrictive Trade Practices Act, 1956, however, before considering its balancing provisions,
is whether the removal of the restrictions under consideration, namely, those arising from the recommendation that members of
the Association should press for the use of one of the standard forms where appropriate, would deny to the public as building
owners specific and substantial benefits or advantages enjoyed or likely to be enjoyed by them by virtue of the restrictions. We
are far from satisfied as to this for a number of reasons. First, on the evidence, part (ii) of the rule, whether interpreted on the
lines of the recommendation of 19 November 1962, or more stringently, is in practice a dead letter. There was a substantial
weight of evidence that members of the Association frequently enter into contracts on other forms, such as those used by the
Birmingham Corporation and other local authorities in and near the area of the Association, without first pressing in any way for
use of the appropriate standard form. The same can be said of contracts with government departments who have their own form.
Moreover, it was well established that the standard forms are often used with alternations made by the architects concerned,
which are readily accepted by builders without prior compliance with the rule. We do not think that these cases can be
distinguished on the grounds that for them the standard forms are not appropriate, since the principal witnesses called for the
respondents urged that there was an approved form appropriate to every circumstance. Secondly, building owners can be divided
into three categories; government departments or nationalised bodies; local authorities; private building owners. The two former
categories have their own expert advisory staff and, in many cases, their own standard forms. They are unlikely to be affected in
any way by the pressure from individual builders called for by the rule when such builders are invited to tender. Certainly there
is no reason to suppose that those local authorities who at present use the standard forms of the RIBA will be any less likely to do
so in the future if the rule went. As for private building owners, in the vast majority of cases they will rely on the advice of their
architects as to the form of contract to be used. It was common ground amongst the witnesses that architects should not allow
their advice to their clients as to the best form or conditions of contract to be influenced by pressure from builders and there was
unanimity amongst the architects called for the respondents that the advice of architects to their clients on these matters was not
affected by the existence of the rule and would not be affected by its abolition. We consider that neither the usefulness nor the
use of the two standard forms before us depends to any significant extent on the existence or operation of this rule and that its
abolition would deny no benefit or advantage to the public as building owners.
In what we have said above on s 21(1)(b) we have mainly dealt with the evidence relating to the seven forms sanctioned by
the RIBA, but our conclusion 372 applies equally to the two forms of sub-contracts and to the conditions of estimate. It is
sufficient to say that no evidence was adduced leading us to think that the use of any of these forms would be affected in any way
by the abolition of the rule. It follows that the restrictions accepted by members of the Association under the rule, both in its
original form and as amended, and the restrictions arising from the agreement implied by s 6(7) by reason of the recommendation
of 19 November 1962, must be declared contrary to the public interest. A similar declaration must be made as regards the
restrictions arising from the agreement by members of the Association to abide by r 27(a) of the Midland Federations rules.
We think it right to add a few observations on certain matters which counsel for the respondents raised in relation to the
position that would obtain in the event of our reaching the conclusions that we have reached in relation to the standard forms. He
suggested that the JCT might have to come to an end or that at any rate the National Federation would no longer be able to
participate in its work. On behalf of the Registrar it was categorically stated that he was not of this view and that he did not
envisage any undertaking that might be given consequent on our decision having this result. As regards two other matters,
namely, whether it would be proper for the National Federations name to continue to appear as a sponsor of the standard forms
and whether members of the Association could in future properly seek, as a body, to persuade local authorities to adopt the use of
the standard forms in preference to their own forms, certain remarks and suggestions were made on behalf of the registrar, which
were no doubt noted by the respondents. There will be time to consider all these matters before the final order is drawn up. For
our part we only say that, as at present advised, we see no reason to dissent from what was said on behalf of the registrar in
relation to the JCT.
We now pass to the second major topic arising, namely, what is known as the national quantities rule. At the date of the
notice of reference r 28 of the Association dealing with this matter read as follows:

Members shall not tender in competition for contracts exceeding 4,000 in total value without bills of quantities being
supplied. In the case of contracts for the repetitive construction of small dwelling-houses the bills of quantities shall be
prepared in accordance with the principles of the code for the measurement of building work in small dwelling-houses.
This instruction shall not apply to contracts for repairs or contracts for painting or decorating only. Any member asked to
tender without quantities contrary to this rule must give notice to the secretary at once. When any member of the
Association is invited to tender for a job outside the district of the Association, he shall conform to the rules of the local
Association in the district where the job is situated regarding the submission of tenders in the absence of quantities.
(Members invited to tender for such jobs should apply to the secretary of the Birmingham Association for information as to
the rule in operation in the district concerned.)

The National Federation had at the date of the notice of reference a rule identical with the first paragraph of r 28, but on 2 May
1962, the limit of 4,000 was increased to 8,000. On 11 September 1962, the first sentence of r 28 of the Association was
amended to read:

Members shall not tender in competition for contracts exceeding in total value such amount as shall from time to time
be determined by the National Federation of Building Trades Employers without bills of quantities being supplied.

A rule to the effect of the present one was first given national application in 1943 with a limit of 1,500. This limit was raised to
3,000 in 1951, to 4,000 in 1956 and, finally, to 8,000 in 1962.
In relation to this rule it was strongly argued on behalf of the respondents that members of the Association did not by
agreeing to the rule accept a restriction in 373 respect of any of the matters coming within sub-paras (a) to (e) of s 6(1) of the
Restrictive Trade Practices Act, 1956, i as rendered applicable to building work by s 36(2). In the alternative the respondents
sought under s 21(1)(b) to justify any restriction arising, save only for the separate restriction arising from the penultimate
paragraph of the rule as to which the respondents called no evidence.
________________________________________
i The terms of s 6(1) are stated in note +, p 363, ante

In order that we may determine whether any relevant restrictions arise out of the rule, a matter which is somewhat intricate
and not free from difficulty, we must first explain what is meant by and the general uses of bills of quantities, or quantities as they
are sometimes referred to for short. Bill or bills of quantities is the name given to an analytical and detailed statement,
usually of very great length, prepared by a quantity surveyor on the basis of the document known as The Standard Method of
Measurement of Building Works from drawings supplied by the architect concerned. The statement shows after many
preliminariesincluding a specification and details of the terms and conditions of the proposed contractthe quantities and
quality of materials and labour required for the different items of work which it is calculated will be necessary for the proposed
structure. The whole document is usually split up into several numbered bills dealing with, in addition to the preliminaries and
general items, each trade, eg, excavator and concreter, bricklayer and drainlayer, founder and smith, etc. Each bill is provided
with a blank column in which the builder who tenders is to insert his price for each item; thus by the addition of these figures the
builder can arrive at the total amount of his tender and the building owner can check the make-up of the total sum quoted. This is
perhaps the most widely used form of a bill of quantities. Another form of bill is the pre-priced bill of quantities. This is
similar to an ordinary bill in that items of work are set out under trades; no quantity is given for each item of work and the unit
price for each item is filled in by the building owner inviting tenders or by the architect or quantity surveyor acting on his behalf.
Firms tendering are required to quote percentages on or off the unit prices for the various trades and when the work is carried out
complete measurement is necessary to work out the final account. Various other alternatives and variations exist and a pre-priced
list of items can be included as one part only, eg, plastering, of a bill covering many different trades.
The rule, therefore, in effect requires that when tenders are invited from members of the Association in competition for
building work other than repairs or painting or decoration only, members shall not tender unless the person inviting the tender,
who will be the prospective building owner or his architect, quantity surveyor or other agent acting on his behalf, provides the
builders invited with appropriate bills of quantities. At this stage it is only necessary to add by way of explanation of the purpose
of one of the purposes of the rule that the preparation of a tender or quotation by builders is in many cases greatly facilitated by
the provision of bills of quantities.
The registrar has submitted that, if the rule gives rise to a relevant restriction at all, it must be stated in three separate ways in
order to cover the provisions of the rule relating to (a) building contracts generally (b) building contracts for the repetitive
construction of small dwelling-houses and (c) building contracts for work to be done outside the district of the Association. In
this he is no doubt right and nothing turns on this point. For the respondent, however, it is argued that as the rule only restricts
the freedom of members to make an offer and as, in addition, only one offer from all builders answering the invitation to tender
can be accepted, the restriction does not fall directly within any of sub-paragraphs (a) to (e) of s 6(1) of the Restrictive Trade
Practices Act, 1956. In the schedule to the registrars pleadings the restriction was put as falling within sub-paras (b) and (e), but
in his final speech counsel for the registrar argued that the restriction in addition fell within sub-paras (a) and (c) and, in so far as
he 374 exhibited any preference, he placed sub-para (a) in the forefront of his argument. Counsel for the respondents raised some
objection to counsel for the registrars departing from or adding to his pleading in this way and drew our attention to a Practice
Direction of this court. It had, however, been clear to the respondents from the moment that the registrar delivered his answer
that he was contending that the national quantities rule gave rise to a relevant restriction and we do not consider that the Practice
Note, which was clearly designed with the primary purpose of assisting the court to make a declaration in precise terms, should
be regarded as any bar to additional formulations of alleged restrictions once the agreement and one or more formulations of the
restriction said to arise therefrom have been pleaded. If such additional formulations were to necessitate the calling of further
evidence, which could only very rarely be the case, or required time for the proper preparation of an argument in reply, the court
would no doubt consider granting an adjournment. This was offered, but not asked for, in the present case and since the court
admittedly has jurisdiction to deal with any relevant restriction arising out of the registered particulars of the agreement, we
proceed to consider the various alternative formulations of the restriction advanced on behalf of the registrar.
Counsel for the registrar first argued that compliance with the rule would prevent members quoting any price at all for
relevant building work when bills of quantities were not supplied and that, therefore, a restriction arose falling within sub-para (a)
[of s 6(1) of the Restrictive Trade Practices Act, 1956]. In answer to this counsel for the respondents contended that a restriction
against quoting at all could not fairly be said to be a restriction as to the price to be quoted and that counsel for the registrars
arguments involved a wholly artificial construction of sub-para (a). In our judgment the sub-paragraph on its true construction
relates to restrictions as to the quantum to be charged, quoted or paid, such as provisions for maximum or minimum prices, and
has no application here. We also reject a subsidiary argument that the rule gives rise to a restriction falling within sub-para (a)
because it prevents quotations of prices in excess of 8,000. We do not think that this is what the rule says; it imposes no fetter
on the size of the quotation, but in certain circumstances it provides that there shall be no quotation at all, which is a very
different thing.
The next argument advanced for the registrar was that the rule gave rise to a restriction under sub-para (b) since it related to
the terms or conditions on or subject to which building work was to be done. The rule does not, however, contain any reference
to the terms or conditions of the building contract; it applies to the circumstances in which members are permitted to tender or
quote in respect of projected building work, leaving the members completely free as to the terms or conditions on or subject to
which the work is to be carried out if a tender is accepted. It is true that two of the main contract forms approved by the RIBA
are drafted to be used when bills of quantities do form part of the contractual documents and can thus be said to constitute part of
the terms or conditions of the contract. It is, however, equally possible, as is exemplified by the other two main contract forms
approved by the RIBA, for bills of quantities to be supplied and used for the purpose (inter alia) of tenders or quotations without
such bills forming any part at all of the terms or conditions of the contract entered into once a tender has been accepted. We
reject the argument under sub-para (b).
As regards sub-para (c) [of s 6(1) of the Restrictive Trade Practices Act, 1956] the argument for the registrar is somewhat
similar to that under sub-para (a). Counsel for the registrar expressed the negative obligation for the purposes of this sub-
paragraph as one not to carry out general building work of a value of more than 8,000 when the building owner has invited
competitive tenders without supplying bills of quantities. In our judgment this argument is unsound. The rule does not place any
restriction on the quantity of building work to be 375 carried out; it says that in certain circumstances no tender shall be made and
accordingly no work at all shall be done. We think these matters are outside the true construction of sub-para (c).
We now come to sub-para (e) [of s 6(1) of the Restrictive Trade Practices Act, 1956] which, so far as material to this case
and bearing in mind its application to building work by s 36(2), can be restated as the persons or classes of persons for whom
buildings or structures are to be constructed or other works carried out by contractors. What is said in order to bring a restriction
arising from the rule within those words is that the members of the Association having agreed to the rule thereby accept a
negative obligation implied from it not to carry out general building work, under contracts for work exceeding 8,000 in value,
for persons calling for tenders in competition for that work without supplying bills of quantities.
Counsel for the respondents first answer to that argument is that the formulation is not sufficiently definite to bring the
individual building owners in question within the word persons in the sub-paragraph. To come within the word the individual
must, he said, be named or described. We do not think that there is anything in this objection; provided that individuals are by the
restriction sufficiently described for it to be possible to say to whom it applies, as is the case here, then those individuals are in
our judgment covered by the word persons. The more serious objection is that the rule in certain circumstances prohibits the
making of tenders, whilst the sub-paragraph refers to the construction of buildings or structures and the carrying out of other
works. Whilst, therefore, counsel for the respondents admits that the rule imposes a restriction, he says that it is an irrelevant one
for the purposes of the Restrictive Trade Practices Act, 1956, since it relates to tendering and not to the carrying out of building
work. He also says that the express restriction is too far removed from the carrying out of the actual work to be within the sub-
paragraph, since many builders may tender for a building project, but until one out of the many is accepted, no building work can
be carried out. He argued that if the rule did not give rise to a restriction falling within sub-para (a), which is the only one of the
five sub-paragraphs expressly dealing with quotations, the court should not stretch the natural meaning of any of the other sub-
paragraphs to cover a restriction on quotations. The very detail of each of the sub-paragraphs leads to the conclusion that each
should be construed narrowly rather than widely. For the latter submission he relied on some dicta ([1959] 2 All ER at p 632; LR
1 RP at p 283) of Lord Evershed MR, in Re Blanket Manufacturers Associations Agreement.
There is force in these arguments and we have not found this matter easy to determine. It is, however, provided in s 6(3) of
the Restrictive Trade Practices Act, 1956, that a restriction includes any negative obligation whether express or implied. It is
clear that, once a builder is prohibited by the rule from tendering in response to an invitation from a building owner for a building
project exceeding 8,000 in total value if he does not supply bills of quantities, the builder cannot carry out that project. This
being so we do not think it is stretching the language of the rule or of s 6(3) of the Act to say that the rule does give rise to the
implied negative obligation previously set out. If it does, the restriction clearly falls within sub-para (e). This seems to us the
short answer to the point. Apart from it, the argument of remoteness seems to us unrealistic; all the sub-paragraphs apply to
restrictions taking effect before contracts are made and we see no reason in principle for excluding from the ambit of sub-para (e)
an express restriction on tendering to certain persons which must have the result of preventing those thus restricted from actually
carrying out for such persons the work for which tenders are invited.
We now pass to the respondents case in seeking to justify the restrictions under s 21(1)(b). They pleaded that the removal
of the restrictions would deny 376 to the public as building owners specific and substantial benefits or advantages in two
respects: (i) a bill of quantities provides an accurate computation for the builder of the work involved and he will not therefore
add something to his tender price in order to protect himself against uncertainty as to the calculation of the amount of the work.
The provision of bills of quantities thus conduces to lower tenders in individual cases. (ii) If bills of quantities are not provided
in the case of larger contracts, now reasonably taken at over the value of 8,000, all builders tendering would have to take out
their own bills of quantities and would thus be involved in extra expense which would lead to increased overheads in the building
industry generally and would have to be recovered in enhanced prices for building work carried out. Over and above these two
pleaded points it was argued that it was advantageous to the building owner that all builders tendering should do so on the same
basis. It is important to remember that the rule only applies to tenders in competition for contracts in excess of 8,000 in value.
It appeared clearly from the evidence adduced by both sides that, with certain exceptions which we mention later, bills of
quantities would, on the advice of the architect or other person concerned to advise the building owner, be provided when inviting
tenders for almost all building contracts in excess of that value whether the rulejexisted or not, partly because of the complexity
and detail involved and partly because of the traditional division of functions between architect and quantity surveyor in this
country. The rule if observed by builders could, therefore, have effect in a relatively small field, though owing to the absence of
statistical evidence as to the number of building contracts in excess of 8,000 in value put out to competitive tender without the
provision of bills of quantities, we are quite unable to make any estimate of the aggregate value or importance of building
projects affected by the rule. Within the field to which the rule by its language applies there are, however, a number of
exceptional cases. The type of such cases most frequently arising is exemplified by a scheme for the building of a large number
of small and relatively cheap dwelling-houses of similar design. The aggregate value of such a scheme may exceed 100,000,
but the building work involved is simple and repetitive. Secondly, there are cases where the aggregate cost is very considerable,
but it is largely made up of prime cost items provided by sub-contractors, whilst the general building work by itself is both cheap
and simple. Thirdly, there may be cases where from the building owners point of view speed is of the essence of the matter and
he cannot afford the time involved in the preparation of elaborate bills of quantities. Finally, of recent years there have been
developments in a new direction whereby the building owner calls for competitive tenders for a project to be both designed and
built by the contractor. In such cases bills of quantities cannot ex hypothesi be provided by the building owner, whereas on the
language of the rule they should be. We should add that the minimum cost to a building owner for the preparation of bills of
quantities is 2 1/2 per cent on the contract value.
________________________________________
j Viz, the quantities rule set out at p 373, letters f to i, ante

As to the suggested specific and substantial benefit to the individual building owner in the shape of a lower price if he
provides bills of quantities with his invitation to tender, whilst this may be so in some cases, we are by no means satisfied that is
is invariably or even generally the case and there was evidence that some smaller builders and some builders specialising in
building large numbers of small houses quoted better prices without the provision of bills of quantities. Apart from this, even if
builders in general quote lower prices when tendering on bills of quantities, the building owners derive no financial benefit from
this if the cost of providing the bills of quantities exceeds the reduction in the tender price due to the provision of the bills. We do
not find it proved that the individual building owner benefits in the way alleged by reason of the rule.
As regards the more general economic argument, for this to be made good it would be necessary that we should be satisfied
on the evidence of four separate 377 matters, namely, (i) that without the rule more tenders would be submitted by builders than
are submitted at present for contracts for which no bills of quantities are supplied; (ii) this would cause an appreciable increase in
builders costs; (iii) this increase would be passed on to the public as building owners, and (iv) the increase thus passed on would
exceed the saving in expenditure by building owners in not having bills of quantities prepared.
Whilst we think (ii) above is made out, the other three points were not proved to our satisfaction. It would be wrong to
describe the national quantities rule as a dead letter in practice, since there was some evidence that on occasions insistence on it
had caused architects to change their minds and have bills of quantities prepared, whilst on other occasions observance of the rule
by builders had reduced the number of tenders submitted in response to an invitation. These were, however, on the evidence as a
whole relative rarities and insignificant in total against the evidence of witnesses, both architects and non-professional men, who
spoke of the many occasions when government departments and agencies, local authorities and private building owners had put
out contracts to tender to values greatly in excess of 8,000 or the previously prevailing limits, both in the Birmingham area and
elsewhere up and down the country, without the provision of bills of quantities and had received tenders satisfactory both in
number and price. Such instances in general related to repetitive work, or jobs where the actual building work was cheap and
simple. If we consider in addition the unknown number of normal non-repetitive contracts over 8,000 in value, in respect to
which as we have said it rarely occurs to an architect not to advise the provision of bills of quantities, we must conclude that the
rule, on the evidence, has only a marginal effect at present on the use of bills of quantities and we think the frequency of such use
if the rule went would be only marginally affected. The general position is, we are satisfied, that spoken to by Mr Waters, a
distinguished architect called by the respondents, who is chairman of the JCT. In every case with which he is concerned as
architect he considers whether in his opinion it would be in his clients interest or not to have bills of quantities prepared and acts
accordingly. His intention is to do the same whether the rule continues in existence or not and he would expect other architects to
do likewise. We are thus unable to find on the evidence in relation to the general economic argument that the respondents
establish the crucial first step in their case, namely, that if the rule itself went there would be any appreciable change in the use of
bills of quantities by building owners. As regards the additional argument that it was advantageous to the building owner that all
builders tendering should do so on the same basis, we are unable to find that the rule at present confers any specific and
substantial benefit on the public as building owners under this heading. Both the chairman of the National Federation and the
principal Birmingham builders giving evidence stated that the case for the rule depended on the general economic argument and
would not accept the suggestion made by one of the architects called for the respondents that the provision of bills of quantities
was some insurance against skimped work or bad quality.
Our conclusion on the evidence is that the restrictions arising from the rule do not at present and have not in the recent past
conferred any specific and substantial benefit on the public as building owners and that the removal of the restrictions would not
deny any such benefit to the public. The restrictions must, therefore, be declared contrary to the public interest.
The third topic calling for our decision is closely related to the national quantities rule and can be shortly dealt with. Rule
29 of the Association provides as follows:

Priced schedules must not be submitted with tenders by members of the Association, without the consent of the
presidents committee. Any member requested to submit the priced-in schedule with the tender must report the matter
immediately to the secretary.
378

Priced schedules in this rule means priced bills of quantities, ie bills of quantities with prices inserted by the builder against
each item in the bills. In practice, once a building owner is minded to accept a particular tender, his architect examines the
builders priced bills of quantities (which he will call for if they have not been submitted with the tender) and if no contract is
made with that builder, he similarly examines the next selected tenderers priced bills. Building owners are never required to
place building work or have building work done before examining the priced bills of quantities if they wish to do so.
Very little evidence was given before us on this topic because the respondents did not seek to justify any restriction arising
from the rule. They limited their argument to denying that the rule gave rise to any relevant restriction and admitted that in this
respect, the same considerations applied to it as to the national quantities rule. As such little evidence was given, we were left
somewhat in the dark as to the extent to which the rule is observed, though it seemed clear that when priced schedules were
submitted with tenders the practice of most architects was only to open those accompanying the tenders which they were minded
to accept in order to preserve the confidential character of the remainder. In our judgment, this particular rule gives rise to a
restriction as to persons within sub-para (e) of s 6(1) of the Restrictive Trade Practices Act, 1956, for the same reasons as led us
to that conclusion in regard to the national quantities rule, which we need not repeat. The restriction must be declared contrary to
the public interest.
The fourth and last topic in this long and intricate case related to certain recommendations made to members of the
Association arising out of a document called the National Schedules of Daywork Charges for General Building Work (hereinafter
called the National Schedules). Here the recommendations as they existed at the beginning of the hearing have been rescinded,
entirely fresh recommendations have been made and the statement of case has been drastically amended. In consequence, the
respondents (a) agreed that the original recommendations, now rescinded, gave rise to restrictions within s 6 of the Act and have
not sought to resist a declaration by the court that they were contrary to the public interest; (b) submit that as a matter of law the
new recommendations give rise to no restrictions to which the Act applies; and (c) seek, in the alternative, if they are wrong on
(b), to defend the restrictions arising from the new recommendations under s 21(1)(b).
We must first state what the original recommendations were and explain to what they related. Daywork in the building
industry is a technical term and has been defined in the agreed statement of facts, if only loosely, as work which is charged and
paid for by time and materials used and not by measurement. So far as this reference is concerned, the term issued in relation to
building work done under two very different sets of circumstances. Thus, what is called daywork on contracts arises out of
variations or additions made in the course of carrying out an ordinary fixed price or lump sum building contract. If such work is
by agreement priced before it is undertaken, the work ceases to be distinguishable as daywork. It not infrequently happens,
however, that though the parties agree the work should be done, it is not possible, owing to its uncertain or novel character or
extent, to price it in advance and agree upon a lump sum for it based on measurement of the estimated quantities and the
estimated time and labour involved. In such cases the work has to be charged and paid for after the event. Part I of the National
Schedules is intended for use in such circumstances. It is headed Daywork on Contracts, it has a sub-title stating that it is a
schedule of charges for building work executed as daywork in conjunction and concurrently with work under a fixed price
contract and provides that the basis of charging shall be the aggregate of four schedules dealing respectively with labour,
materials, direct charges and percentage additions for overhead charges and profit. Part II of the National Schedules is headed
Daywork on Jobbing Work has a sub-title 379 stating that it is a schedule of charges for building work of a jobbing or
maintenance character of over 10 in value carried out on a labour and materials costs basis and provides that the ultimate charge
shall be the aggregate of five schedules dealing respectively with labour, materials, direct charges, percentage additions for
overhead charges and profit, and sub-contractors accounts. Part II is intended to apply to work done under running contracts for
the maintenance of buildings or to jobbing work such as when a builder is called in to do ad hoc repairs or decorations.
Prior to 1933 local associations of building employers had their own daywork schedules, but in that year, as the result of
protracted negotiations between the National Federation and the Chartered Surveyors Institution, the first National Schedule of
Daywork Charges consisting of a short document under three heads was issued as an agreement intended to be of national
application. During the war the national schedule as it then existed was found to be quite inadequate for the vast amount of
building work that, as a result of war damage, had to be carried out on a prime cost or daywork basis. Accordingly, the National
Federation negotiated with the War Damage Commission a basis for charging which would be accepted as proper cost within the
meaning of the War Damage Act, 1943. This War Damage Daywork Schedule appeared in a pamphlet issued by the War Damage
Commission for the guidance of claimants under the Act of 1943 and comprised a comprehensive definition of prime cost, to
which were to be added percentage additions to cover overheads and profit. After the war the National Federation instructed an
independent accountant, Mr John Unwin, to conduct an investigation into the building industry and to advise on the proper
accountancy practice in calculating and charging for such on-costs with a view to producing a completely new edition of the
National Schedule. As the result of two reports by Mr Unwin, which made use of the definition of prime cost in the War Damage
Commission pamphlet, and long discussions with the RICS, the National Schedules we have described, divided into two parts,
were published in 1950. Only a few very minor amendments have been made since that date.
Provision for dealing with daywork arising out of building contracts is made in each of the four main forms of building
contracts approved by the RIBA, which were current at the time of the hearing. Thus, cl 9(c) thereof provided that where extra
work could not properly be measured and valued, the builder should be allowed daywork prices at the rates, if any, inserted by the
builder in the bills of quantities or in the tender or, when no such rates had been inserted, at the rates stated in the National
Schedule of Daywork Charges. This was a reference to Part 1 of the National Schedules under discussion. On the evidence
before us, daywork arising out of lump sum or fixed price building contracts amounted in value to only about one per cent of the
total value of such contracts.
The position as regards Part 1 of the National Schedules at the date of the notice of reference and at the beginning of the
hearing before us was that the Association recommended its members, when invited to quote for daywork rates with their tenders,
to quote only the rates in Part I and thus to eliminate competition on rates for daywork of this category. Similarly, at the said date
and time the Association recommended its members as regards daywork arising in connexion with maintenance and jobbing work
to charge in accordance with the rates in Part 2 of the National Schedules. The restriction to which the latter recommendation
gave rise by virtue of s 6(7) of the Restrictive Trade Practices Act, 1956, was pleaded by the registrar in the following terms, of
which no criticism was made at the hearing on behalf of the respondents:

Not to carry out or offer to carry out building work of a jobbing or maintenance character of over 10 in value carried
out on a labour and materials costs basis except at the prices determined in accordance with the basis of charging set out in
Part II of the National Schedules of Daywork Charges for General Building Work.
380

The respondents statement of case sought, under s 21(1)(b) of the Act, to justify the restrictions arising from these
recommendations. In the course of the evidence it appeared, however, that members of the Association freely departed from both
recommendations; and on 23 November 1962, the Association withdrew both recommendations and issued a new one by a
circular communicated to each of its members in the following terms:

Notice. To members of the Birmingham Association of Building Trades Employers issued by the Birmingham
Association of Building Employers. 1. National Schedules of Daywork Charges, Part I. In the light of developments in the
Associations case now before the Restrictive Practices Court it has been decided to withdraw the advice given by this
Association to members in its letter dated June 8, 1950, namely, that when invited to submit daywork rates with tenders,
members should indorse their tenders: Daywork rates shall be those operating from June 1, 1950, as agreed between the
R.I.C.S. and the N.F.B.T.E. 2. National Schedules of Daywork Charges, Part 2. Likewise this Associations letter dated
Aug. 10, 1950, in which it was stated that the Association had decided to adopt Part 2 of the National Schedules of
Daywork Charges in conjunction with the attached Notes is now withdrawn. 3. In substitution for the above the
Association now makes the following recommendation: The National Schedules of Daywork Charges, Parts 1 and 2, and
the Explanatory Notes Serial No. 1 are recommended for use in the absence of any agreement between builder and client as
to daywork charges for general building work. 4. If members are invited to quote rates for work to be executed as
daywork in conjunction and concurrently with work under a fixed price contract or for building work of a jobbing or
maintenance character they are free to quote such rates as they think fit. 5. Save as above, no recommendation is made to
members as to the National Schedules and Part 2 of the Schedules is no longer adopted by the Association except in the
sense that is recommended as in para. 3 above.

The National and Midland Federations on 20 November 1962, each issued circulars making the same recommendation to all
members as in para 3 of the above-quoted circular of the Association and adding that in so far as either Federation had made or
might appear to have made any other recommendations in relation to the National Schedules of Daywork Charges the same were
thereby withdrawn.
On 5 December 1962, pursuant to leave of the court, the respondents amended their statement of case and pleaded that the
original recommendations as to daywork had been withdrawn, that the new recommendation gave rise to no relevant restriction,
but that, if it did, the removal of such restriction would deny to the public as building owners specific and substantial benefits or
advantages. Counsel for the respondents admitted that in these circumstances any restrictions arising from the original
recommendations could not be defended. We postpone until later consideration of what order should be made in regard to them.
As regards the new recommendation, however, counsel for the respondents submitted that it did not come within s 6(7)
because, to use the words of the subsection, it was not in respect of any matter described in sub-s (1) of s 6. The circular which
we have quoted made it clear that members, if asked to quote rates for daywork falling within Parts 1 or 2 of the National
Schedules were free to quote any rates they pleased and that no recommendation applied to such circumstances. The effect of the
recommendation was, he said, only that if no agreement had been made as to charges for daywork, then members should seek to
charge the building owner in accordance with the rates appropriate under Part 1 or Part 2. The recommendation, if complied
with, could therefore only take effect after the work had been carried out. In such circumstances an attempt by the builder to
charge such rates could have no contractual effect, and, as a matter of law, was outside and not in respect of any matter described
in s 6(1). In support of his argument, he strongly relied upon the decision of the Court of 381 Appeal in Re Blanket
Manufacturers Associations Agreement. There the question for decision, as defined by the court, was whether an agreement
(dealing with what was described as sanctity of contracts) between two or more persons, being manufacturers of the same type
of goods, not to cancel or vary any contract of sale of goods made or to be made in the future by any of those persons with their
customers without the consent of some third party, was the acceptance of a restriction which fell within any one or more of sub-
paras (a) to (c) of s 6(1). The Court of Appeal, affirming the decision of this court, decided that it was not. Lord Evershed MR
([1959] 2 All ER at p 632; LR 1 RP at p 282), stressing the use of the words to be in the subparagraphs of s 6(1)to be
charged, to be applied, to be produced, etcsaid that they indicated that the draftsman was looking to a point of time when
the contract is about to be entered into and is not contemplating the case where the contract has already been made. Romer LJ
([1959] 2 All ER at p 632; LR 1 RP at p 283), accepted that the sub-paragraphs in question in s 6(1) were confined to agreements
regulating the pre-contractual position of members of an association. Pearce LJ ([1959] 2 All ER at p 633; LR 1 RP at p 284), put
the position very shortly when he said:

In my view, s. 6(1), in so far as it relates to the prices and other terms or conditions of sale, is directed to restrictions
prior to the contract for the sale of goods. The sanctity of contracts restriction only affects the situation after the contracts
of sale have been entered into. It is a restriction, but it is not a restriction that comes within the terms of paras. (a), (b) or
(c), on a fair construction of their language.

Although the Court of Appeal were there dealing with a very different agreement from the agreement implied between members
as the result of s 6(7) if it applies to this recommendation, we think that the reasoning of the judgments we have quoted has some
relevance here. When the recommendation says in the absence of any agreement between builder and client as to daywork
charges, the word agreement may be used to cover both an express and an implied agreement or only the former. If it be
given the wider meaning, the recommendation can only have any content or application in those rare cases where the law gives a
party a remedy by way of a quantum meruit, there being neither an express agreement nor an implied agreement by the other
party to pay anything: see, for example, Craven-Ellis v Canons Ltd. It seems more probable that the recommendation on its true
construction is meant to apply where there is no express agreement. But in such cases where daywork has been carried out at the
request of the building owner, the law implies an agreement by the building owner to pay a reasonable price and, if the builder
puts in his bill charging for the work on the basis of the National Schedules, this will have no contractual effect. If in the
circumstances the rates in the National Schedules are reasonable, the building owner will have to pay them, but this will be
because they are reasonable and not because they are in the National Schedules. Further, the charge made in compliance with the
recommendation will not be a price to be charged for the work, nor will the rates in the National Schedules constitute terms or
conditions on or subject to which the building work is to be carried out. The work will already have been carried out under an
agreement implied by law that the building owner shall pay a reasonable price. At the most, compliance with the
recommendation by the builder will be an attempt to vary the existing contractual position between himself and the building
owner and the decision of the Court of Appeal establishes, so far as this court is concerned, that an implied agreement between
members of an association as to this is not within s 6(1). Finally, if on the true construction of the recommendation it only
applies (contrary to what we think is the preferable construction) where there 382 is neither an express nor an implied agreement
as to daywork charges for building work, that is to say in those rare circumstances where the builders claim to remuneration only
arises on a quantum meruit in the true legal meaning of that term, we are unable to see how the legal position is any different. A
charge based on the National Schedules would again have no legal effect unless reasonable; and the implied agreement,
consequent on the recommendation, to make such a charge, which could only be made after the work had been completed, would
again, on the reasoning of the Court of Appeal, fall outside s 6(1).
For these reasons, we conclude that the new recommendation does not give rise to any restriction falling within the
jurisdiction of this court.
In case we should be held on appeal to be wrong in this conclusion, and at the request of the parties, we proceed to state our
view on the case advanced in the alternative by the respondents that the removal of the restriction arising from the new
recommendation would deny to the public as building owners specific and substantial benefits or advantages enjoyed or likely to
be enjoyed by them by virtue of the restriction. It might be thought that because of the very novelty of the new recommendation
and the lack of experience of its working, it was ipso facto impossible for the respondents to establish such a case to our
satisfaction. We should, however, be reluctant to decide against the respondents on this ground alone, because we would not
wish to hold that a newly imposed restriction can never be justified. The first benefit or advantage pleaded for the respondents is
that the National Schedules have been agreed between the National Federation and the RICS and the rates are reasonable to the
building owner. The former part of this plea is correct, but the latter we do not find to be proven. We doubt very much whether it
is possible to determine uniform rates for daywork common to builders throughout England and Wales, both big and small, which
are reasonable to the building owner. Certainly the accountancy evidence advanced to support this part of the respondents case
was inadequate to lead us to the conclusion for which the respondents argued. In saying this we do not mean in any way to
criticise the frankness, conscientiousness, or knowledge of building accountancy problems of Mr Jones of the Advisory Service
for the Building Industry, but in the circumstances we think that he attempted an impossible task. Whilst in the middle of
conducting an investigation into building costs with a view (inter alia) to the possible preparation of new National Schedules and
before he had completed examination of the answers to questionnaires issued to a sample of the building industry, he was
requested at very short notice to prepare a proof in support of the respondents case based on the answers to the questionnaires.
There were far too many uncertainties in evidence prepared in such circumstances for it to carry conviction. In particular, the
sample of the building industry taken was not satisfactorily selected, nor was there any follow-up of the three-fifths of the sample
membership questioned who had not answered the questionaires at the date of the hearing, nor was there any evidence of the
capital employed on daywork. There were other numerous and well-founded criticisms of the respondents accountancy evidence
which we need not detail, but our experience in this case indicates how unsatisfactory and wasteful in time it is to seek to justify
schedules of charges or prices without adopting the usual procedure in these references of a joint accountancy investigation by
accountants chosen respectively by the registrar and the respondents concerned.
The next benefit or advantage pleaded is that Part I of the National Schedules is referred to and recognised in cl 9(c)(ii) of
each of the four main forms of building contract settled by the JCT and also in the Birmingham Corporation contract. This is
true, but we do not accept it as relevant to the issue we have to decide. When cl 9(c)(ii) has application, the matter is outside the
scope of the new recommendation because in such cases the parties will have made an express agreement prior to the daywork
being carried out as to the rates to be charged.
Thirdly, it is pleaded that where no agreement has been made as to the charges 383 for daywork, (a) the charge is more
likely to be reasonable to the building owner if he is charged on the basis of the National Schedules than on some other basis, and
(b) disputes are less likely to arise and, if they do arise, are likely to be more easily settled. We do not think that (a) is made out,
for the reasons indicated in dealing with the first plea. In addition there was evidence from members of the Association that they
sometimes charge less than the National Schedules rates and there was further evidence from the Ministry of Public Building and
Works and the London County Council that when tenders are invited for daywork of either category, lower prices are achieved
than would be the case under the National Schedules. This suggests that many building owners might well be charged less than
the National Schedules rates if there were no recommendation as to such rates in operation. As to (b), whilst there was some
evidence that the use of the National Schedules reduced the incidence of disputes and made them easier to settle, this was far too
limited to satisfy us that in this respect compliance with the recommendation conferred a substantial benefit on the public as
building owners which would be denied if the recommendation was withdrawn.
The final plea in justification was mainly repetitive, but was based on the hypothesis that if a restriction arose from the
recommendation and was declared contrary to the public interest, it was unlikely that a schedule containing percentages would
continue to be negotiated between and issued by the National Federation and the RICS, with consequent loss of specific and
substantial benefits to the public as building owners. Since, as already stated, we are not satisfied that the recommendation
results in the public as building owners enjoying a substantial benefit, this plea also fails even if the hypothesis is made out. We
would emphasise, however, that in our view on the evidence the chief usefulness of the National Schedules lay not in the stated
percentage additions for profits and overheads, but in the classified breakdown or analysis of the costs for work done under
various different headings, a method of ascertaining a proper charge dating back to the time of the War Damage Commission
which would be unaffected by any decision of this court.
In the result, if, contrary to our view, the current recommendation falls within s 6(7) of the Restrictive Trades Practices Act,
1956, we should have declared the restriction arising therefrom to be contrary to the public interest.
We now return to the original recommendations as to the National Schedules. There was some ambiguity in the original
recommendation as to daywork falling within Part 2 of the National Schedules, reflected in the answers of several witnesses, who
were not clear in their minds whether the recommendation was intended to apply (a) only when there had been no express
agreement on charges prior to the execution of the daywork or (b) was also intended to prevent any agreement on or quotation of
charges before the work was carried out other than charges ascertained in accordance with the rates in Part 2. The witnesses were
particularly uncertain on this in relation to building work of a maintenance character. Since the original recommendation has
been rescinded and the respondents have not sought to defend it and no criticism was made of the formulation of the resulting
restriction by the registrar, there would at first sight appear to be no difficulty in a declaration being made that the restriction so
formulated is contrary to the public interest. If, however, on its true construction the original recommendation as to Part 2
applied only when there had been no express agreement on charges prior to the execution of the daywork, such recommendation
did not in our judgment fall within s 6(7), for the same reasons as we have given above in relation to the new recommendation.
We are inclined to the view that such was the true construction of the original recommendation as to Part 2 and that accordingly
no declaration can be made in relation to it. For similar reasons, we think that whilst a declaration should be made that the
restriction arising out of the original recommendation in relation to Part 1 of the National Schedules is contrary to the public
interest, further consideration should be given to the formulation of that restriction. These matters were not canvassed at the 384
hearing and, in concentrating on the new recommendations, admissions were made on behalf of the respondents in relation to the
old recommendations which may, per incuriam, have gone too far. We will hear counsel further on these points if they should so
desire.
In conclusion, it remains for us to state that the registrar listed four further restrictions which had been abandoned since the
date of the notice of reference and before the hearing. These must be declared contrary to the public interest.
[After discussion with counsel concerning any declaration as to the recommendation concerning Part 2 of the National
Schedules and a short adjournmentk, His Lordship continued:] Counsel for the respondents has on behalf of the Association,
although they are not nominally parties, because the reference, as we have stated in our judgment, is one in which there is a
representative respondent in the shape of the National Federation, affirmed the admissions that were made on the pleadings and
during the course of the hearing, and in no way wishes to re-argue those matters.
________________________________________
k In the course of the discussion His Lordship raised the question whether there was jurisdiction to make a declaration, in that the matter
rested on admission of the respondents that there was a registrable restriction, that restriction being where the day work basis (viz, charge by
cost of material and cost of labour together with a percentage) was contractually agreed before the work was done. After the adjournment
reference was made by counsel for the respondents to a passage in the judgment in Re Net Book Agreement, 1957 ([1962] 3 All ER at p 767,
letter f) where the court proceeded on the basis of an admission. Counsel affirmed the admission on behalf of the Association, no admission
having been or being made so far as the National Federation was concerned. Counsel for the registrar accepted this and His Lordship then
continued as reported in the text

He has also explained to the court a matter which was not gone into in any detail during the hearing, namely, that the
members of the Association always regarded themselves as having agreed not to quote for dayworks of a jobbing character other
than on the basis of Part 2 of the National Schedules.
In view of those matters, and counsel for the registrar agreeing, the court thinks it proper that a declaration should be made
in relation to members of the Birmingham Association only to the effect that the recommendation in its original form in relation
to Part 2 of the National Schedules is contrary to the public interest, that declaration being limited to the application of Part 2
prior to the carrying out of the actual work in question.
[Counsel for the registrar asked for an injunction against fifty former members of the Association (since 1958 some fifty
members had given up membership, of whom four were individuals who had died) on whose behalf counsel for the respondents
had no authority to give undertakings. Those who were represented (180 members) were prepared to give the undertaking.
Counsel for the respondents referred to The Tyre Trade Register Agreement ([1963] 1 All ER at p 911), and The Chemists
Federations Agreement ([1958] 3 All ER at p 463). Mocatta J held that an injunction should be granted against the persons not
represented, but with the exception of any who had died, and that it should not be made forthwith but only when the order had
been drawn up.]

Order, including declaration, accordingly.

Solicitors: Joynson-Hicks & Co (for the respondents); Treasury Solicitor.

Mary Colton Barrister.


385
[1963] 2 All ER 386
W v W (No 4)
FAMILY; Other Family

PROBATE, DIVORCE AND ADMIRALTY DIVISION


CAIRNS J
14 MARCH 1963

Nullity Pregnancy at date of marriage by person other than petitioner Application for order for blood tests Matrimonial
Causes Act, 1950 (14 Geo 6 c 25), s 8(1)(d) RSC, Ord 50, r 3.

The court has no power to order a wife to undergo a blood test against her will, or to order a child to undergo such a test, in order
to determine the paternity of the child.
Liff v Liff (otherwise Rigby) ([1948] WN 128), and H v H and C ((1962), The Times, 23 March), considered.
The husband brought a petition for nullity of marriage based on the ground, enacted in s 8(1)(d) of the Matrimonial Causes
Act, 1950, that at the time of the marriage the respondent was pregnant by some person other than the petitioner. He applied for
an order that tests should be made of his own blood and that of the respondent and of her infant child (aged about eleven months)
in order to show that he was not the father of the child. It was not suggested that the court had inherent jurisdiction to make the
order.

Held The application would be refused, for such an order could not be made under RSC, Ord 50, r 3, nor was there other
statutory jurisdiction under which it could be made.

Notes
As to evidence of blood-groups to prove pregnancy by some person other than the petitioner, see 12 Halsburys Laws (3rd Edn)
225, para 421, note (c); and for cases on the subject, see 27 Digest (Repl) 283, 2267.
As to the jurisdiction conferred by RSC, Ord 50, r 3, see 30 Halsburys Laws (3rd Edn) 387, para 722.
For the Matrimonial Causes Act, 1950, s 8(1)(d), s 32(1), see 29 Halsburys Statutes (2nd Edn) 397, 417.

Cases referred to in judgment


H v H and C (1962), The Times, 23 March.
Liff v Liff (otherwise Rigby) [1948] WN 128, 27 Digest (Repl) 283, 2267.
Russell v Russell [1924] AC 687, 93 LJP 97, 131 LT 482, 27 Digest (Repl) 318, 2649.
Tudor Accumulator Co Ltd v China Mutual Steam Navigation Co Ltd [1930] WN 200, Digest (Practice) 661, 2806.

Adjourned summons
The husband, who had presented a petition for nullity under s 8(1)(d) of the Matrimonial Causes Act, 1950, on the ground that the
wife was at the time of the marriage pregnant by some person other than himself, applied for an order for tests to be made of his
own blood and that of the wife and of the child in order to determine the paternity of the child. The wife opposed the
application.

R L Bayne-Powell for the husband.


R Shulman for the wife.

14 March 1963. The following judgment was delivered.

CAIRNS J. The petition is presented under s 8(1)(d) of the Matrimonial Causes Act, 1950, on the ground that the wife was
pregnant at the time of the marriage by some person other than the husband. The marriage was on 7 October 1961. The child
was born on 19 April 1962. It is, therefore, obvious that the wife was pregnant at the time of the marriage. The husband alleges
that the wife had sexual intercourse with another man at about the time when the child was probably conceived. He admits,
however, that he himself had sexual intercourse with her before marriage and at a time close enough to the date of conception for
it to be quite possible for him to be the father. He says that the only possibility of his being able to prove that the child is not his
is for the blood tests to be made. He concedes that these tests may be quite inconclusive but he says that the tests may show that
the child cannot be his and that he ought to have this opportunity of establishing his case.
386
So far as is known no such application as this has ever been made in this country before. There are two reported cases in
which the results of blood tests voluntarily undergone by the parties have been put in evidence in order to prove that a husband
petitioner in a divorce suit was not the father of a child born to the wife for the purpose of establishing the wifes adultery. One
such case is Liff v Liff (otherwise Rigby), decided by Lord Merriman P, and the other was H v H and C, decided by me. It
happens that in both cases the evidence was similar. The husband and wife were both of blood-group O and the child was of
group A. There was medical evidence that a child of group A could not be produced by two parents both of group O. Lord
Merriman P, accepted this as corroborative evidence and granted a decree of nullity. And in my case, so far as appears from the
report, and so far as I remember, it was the only evidence of adultery admissible against the wife, and I accepted it as proving that
she had committed adultery.
Counsel for the wife argues that blood tests cannot prove the paternity of a child and that it has not been established in this
case that the tests could assist the court at all. It is clear that the tests could not prove that any particular person was the father,
but it would appear that if the medical evidence were similar to that given in Liff v Liff and H v H and C, it might establish that
the husband was not the father. At this stage I should be prepared to assume that it could be of value in that way.
Next counsel for the wife says that the court will not assist a party to bastardise a child. As to this I think that the modern
policy of the law is to favour the production of any cogent evidence which will help to establish the true paternity of a child. The
abolition of the rule in Russell v Russell by legislation now reproduced in s 32(1) of the Matrimonial Causes Act, 1950, is a good
illustration of this. But however convenient it may be that blood test evidence should be made available, I must now turn to the
question whether the wife can be compelled against her will to undergo such a test.
Counsel for the husband says, first, that there must be a large proportion of cases under s 8(1)(d) where the husband can
prove that he was not responsible for the pregnancy only by means of a blood test. I see no reason to suppose that this is so. I
should have thought that the number of cases in which a woman who is pregnant at the time of marriage and has had sexual
intercourse with her future husband and with another man, both at about the time of conception, would probably not be great;
certainly not so great as to support the view that Parliament in passing s 8(1)(d) must be taken to have authorised compulsory
blood tests.
It was not contended, and in my view could not be contended, that there is inherent jurisdiction in the court to order such
tests. If they can be ordered, some statutory authority must be found for them. Counsel for the husband contends that authority is
to be found in RSC, Ord 50, r 3, which reads as follows:

It shall be lawful for the court or a judge, upon the application of any party to a cause or matter, and upon such terms
as may be just, to make any order for the detention, preservation, or inspection of any property or thing, being the subject
of such cause or matter, or as to which any question may arise therein, and for all or any of the purposes aforesaid, to
authorise any persons to enter upon or into any land or building in the possession of any party to such cause or matter, and
for all or any of the purposes aforesaid to authorise any samples to be taken, or any observation to be made or experiment
to be tried, which may be necessary or expedient for the purpose of obtaining full information or evidence.

Counsel for the husband says that under that rule the court can order samples 387 to be taken, that is to say samples of blood,
and experiments to be made, that is to say laboratory experiments on the samples. He referred me to Tudor Accumulator Co
Ltd v China Mutual Steam Navigation Co Ltd, where two members of the Court of Appeal said that what can be inspected under
this rule is a physical thing, which, says counsel for the husband, is an apt description of human blood. Then he draws an
analogy with RSC, Ord 37A, which provides for the appointment of a court expert. Under r 5 of that Order experiments are
authorised and under r 11 expert includes medical men. So, says counsel, medical experiments are clearly contemplated by
RSC, Ord 37A, and there is no reason why they should not equally be contemplated by RSC, Ord 50, r 3.
If this argument is right, it would mean that in the case of a plaintiff alleging that he had suffered some internal injury by the
defendants negligence, it would be open to the court to direct that an extensive exploratory operation should be made by a
surgeon on the plaintiffs body against his will. I cannot for a moment suppose that the rules have any such effect. So far as I
know, it has never been suggested that even a simple medical inspection can be ordered under RSC, Ord 50, r 3. I do not
consider that the word thing in the context of that rule includes the body or blood of a living person, nor that the taking of a
sample includes the taking of a sample of blood from the veins of an unwilling human being. In my opinion, far more precise
wording would be needed to enable the court to authorise any such interference with a persons body. No doubt RSC, Ord 37A,
authorises some kinds of medical experiment, but I see no reason to suppose that it was intended to authorise any kind of surgical
operation, however slight.
Medical inspections have, of course, in some circumstances been authorised by statute. A familiar example was under the
old Workmens Compensation legislation, where a claimant for compensation or a person in receipt of compensation had to
submit to such inspections from time to time; see Workmens Compensation Act, 1925, s 17 and s 18. It may be noted that under
those sections the only penalty for refusal to submit was loss of the compensation payments; in such a case as the present, if a
blood test could be ordered, the order would be valueless to the husband unless it could be enforced by attachment. Still more
familiar in this Division are the medical inspections provided for in r 24 of the Matrimonial Causes Rules, 1957; but those are not
inspections to which a party is under any compulsion to submit.
Counsel for the husband says that the operation of taking a sample of blood is of the most trivial character, involving no
danger or discomfort and that the interference with the person of the wife would therefore be de minimis. If I considered this to
be a sustainable contention I should have wished to have medical evidence to support counsels statement, but it is conceded that
the operation involves puncturing the skin and extracting some small quantity of blood. Obviously to do this to an unwilling
person would be an assault unless authorised by law and I can find no such authority.
Counsel further says that the wife has no real objection, conscientious or medical, to a blood test as such, for she was at an
earlier stage willing to undergo a blood test on the terms that unless it showed that the husband was not the father of the child
there should be a reconciliation. Whether this was a reasonable attitude or not and whether the wifes present unwillingness to be
tested is grounded on conscientious objection or physical distaste or a mere whim, or on reluctance to run the risk of providing
the husband with evidence that would establish his case, seems to me to be quite immaterial. It is not for her to explain why she
has declined to be tested. It is for the husband to satisfy the court that she is bound to submit to the test and, in my opinion, he
has failed to do so.
All the reasons which I have given for not compelling the wife to submit to a blood test apply with still more force to the
child, who is not a party to the suit, 388and who is too young to consent or object to a test being made on her. The application
will, therefore, be dismissed.

Application dismissed.

Solicitors: Gouldens (for the husband); Benjamin & & Benjamin (for the wife).

A T Hoolahan Esq Barrister.


[1963] 2 All ER 389

Fisher v Raven
Raven v Fisher
CRIMINAL; Criminal Law: BANKRUPTCY

HOUSE OF LORDS
LORD DILHORNE LC, LORD EVERSHED, LORD MORRIS OF BORTH-Y-GEST, LORD HODSON AND LORD DEVELIN
11, 12, 13 MARCH 1 MAY 1963

Criminal Law Obtaining credit by fraud Contracts to make paintings from photographs or snapshots Deposits paid
Failure to carry out work Whether credit obtained Debtors Act, 1869 (32 & 33 Vict c 62), s 13(1).

Bankruptcy Undischarged bankrupt Obtaining credit to the extent of 10 or upwards without disclosure Contracts to make
paintings from photographs or snapshots Deposits paid Failure to carry out work Whether obtaining credit Bankruptcy
Act, 1914 (4 & 5 Geo 5 c 59), s 155 (a).

In the Debtors Act, 1869, s 13(1)a, and the Bankruptcy Act, 1914, s 155(a) b and s 156(a)c, the phrases obtained credit or
obtains credit alike refer to the obtaining of credit in respect of the payment or repayment of money only, and the receipt of
money on a promise to render services or deliver goods in the future is not an obtaining of credit (see p 395, letter b, p 394, letter
g, and p 393, letter i, to p 394, letter a, post).
________________________________________
a Set out at p 391, letters c and e, post
b Set out at p 391, letters c and e, post
c See p 392, letter e, post

R v Laker ((1949), 34 Cr App Rep 36) applied.


R v Ingram ([1956] 2 All ER 639) disapproved; dictum of Lord Goddard CJ in Osborn v Barton ((1949), 66 (Pt 1) TLR at p
117) criticised.
The appellant, F, was an undischarged bankrupt. He offered to make paintings for people from any photographs or
snapshots. He would hand over a card to each customer which showed K Fisher in the centre, and at the foot presented by Mr
Maurice Fisher. Thirteen persons placed orders with him, and he obtained deposits from them ranging from 5 to 237. He
supplied a receipt which had K Fisher at the top and the address of the appellant and his wife below. In some instances, he
promised that the work would be completed within a specified time. Some of the customers thought that he was the principal,
others that he was merely a representative, of a firm K Fisher. The appellant failed to execute the orders, and the customers did
not receive their paintings. On appeal (a) by the appellant against convictions of obtaining credit by fraud other than false
pretences, contrary to s 13(1) of the Debtors Act, 1869 d and (b) by the Crown against the quashing of convictions of obtaining
credit when an undischarged bankrupt, contrary to s 155(a)e of the Bankruptcy Act, 1914,
________________________________________
d Set out at p 391, letters c and e, post
e Set out at p 391, letters c and e, post

Held For the reason stated at letter d, above, there had not been an obtaining of credit within s 13(1) of the Debtors Act, 1869,
or s 155(a) of the Bankruptcy Act, 1914.
Decision of the Court Of Criminal Appeal (sub nom R v Fisher, [1963] 1 All ER 744) reversed on holding (i) and affirmed
on holding (ii).

Notes
As to obtaining credit by fraud other than false pretences, see 2 Halsburys Laws (3rd Edn) 632634, para 1254; and for cases on
the subject, see 15 Digest (Repl) 1195, 1196, 12,13712,156.
As to obtaining credit when an undischarged bankrupt, see 2 Halsburys Laws (3rd Edn) 630, 631, para 1252(17); and for
cases on the subject, see 5 Digest (Repl) 1129, 1130, 99089107.
For the Debtors Act, 1869, s 13(1), see 2 Halsburys Statutes (2nd Edn) 298.
389
For the Bankruptcy Act, 1914, s 155(a), see 2 Halsburys Statutes (2nd Edn) 437.

Cases referred to in opinions


Osborn v Barton (1949), 66 (Pt 1) TLR 115, 94 Sol Jo 15, 5 Digest (Repl) 1129, 9103.
R v Dawson, R v Wenlock [1960] 1 All ER 558, [1960] 1 WLR 163, 124 JP 237, 44 Cr App Rep 87, 3rd Digest Supp.
R v Dent [1955] 2 All ER 806, [1955] 2 QB 590, [1955] 3 WLR 297, 119 JP 512, 99 Sol Jo 511, 39 Cr App Rep 131, 15 Digest
(Repl) 1173, 11,850.
R v Ingram [1956] 2 All ER 639, [1956] 2 QB 424, [1956] 3 WLR 309, 120 JP 397, 40 Cr App Rep 115, 5 Digest (Repl) 1127,
9086.
R v Jones [1898] 1 QB 119, 67 LJQB 41, 77 LT 503, 15 Digest (Repl) 1163, 11,746.
R v Laker (1949), 34 Cr App Rep 36, 5 Digest (Repl) 1129, 9097.

Appeals
These were appeals by Maurice Fisher and by Dt-Sgt George Patrick Raven, the prosecutor, respectively from the decision of the
Court of Criminal Appeall (Ashworth, Winn and Lyell JJ) dated 15 February 1963, and reported [1963] 1 All ER 744, sub nom R
v Fisher.
The appellant, Maurice Fisher, appealed by leave of the Court of Criminal Appeal against his convictions under s 13(1) of
the Debtors Act, 1869, the court certifying that a point of law of general public importance was involved, namely, whether the
receipt of money on a promise to render services or deliver goods in the future is an obtaining of credit within the section.
The Crown appealed by leave of the Court of Criminal Appeal against the courts decision quashing the convictions of the
appellant under s 155(a) of the Bankruptcy Act, 1914, the court certifying that a point of law of general public importance was
involved, namely, whether the receipt of money on a promise to render services or deliver goods in the future is an obtaining of
credit within the meaning of s 155(a) of the Bankruptcy Act, 1914.

Neil Lawson QC and J Lloyd-Eley for the appellant.


Sebag Shaw QC and W M F Hudson for the Crown.

Their Lordships took time for consideration

1 May 1963. The following opinions were delivered.

LORD DILHORNE LC. My Lords, the appellant, Maurice Fisher, was tried at Essex Quarter Sessions and on 12 October 1962,
was convicted on fourteen counts of obtaining credit by fraud other than false pretences contrary to s 13(1) of the Debtors Act,
1869, and on thirteen counts of obtaining credit when an undischarged bankrupt contrary to s 155(a) of the Bankruptcy Act, 1914.
He appealed to the Court of Criminal Appeal and that court upheld the convictions for offences under s 13(1) of the Debtors
Act, 1869, and quashed the convictions under the Bankruptcy Act, 1914. The appellant has now, with the leave of the Court of
Criminal Appeal, appealed to your Lordships against the convictions under the Debtors Act, 1869, and the Crown has, also with
the leave of the Court of Criminal Appeal, appealed against the quashing of the convictions under s 155(a) of the Bankruptcy Act,
1914.
In relation to the appellants appeal, the Court of Criminal Appeal certified that a point of law of general public importance
was involved, namely, whether the receipt of money on a promise to render services or deliver goods in the future is an obtaining
of credit within the meaning of s 13(1) of the Debtors Act, 1869. In relation to the Crowns appeal, the Court of Criminal Appeal
gave a similar certificate stating that the point of law involved was whether the receipt of money on a promise to render services
or deliver goods in the future is an obtaining of credit within the meaning of s 155(a) of the Bankruptcy Act, 1914. 390Before
your Lordships, the argument was solely directed to these two questions.
The appellant was adjudicated bankrupt in March, 1960, with a deficiency of 28,000. The evidence given at his trial shows
that between November, 1960 and December, 1961, he had called at the homes of fourteen persons and had obtained on these
visits sums varying from 5 to 237 in return for his undertaking to supply to them paintings from photographs given to him for
that purpose. His practice was to hand to a prospective customer a card which showed K Fisher in the centre and at the foot
presented by Mr Maurice Fisher and to give a receipt headed K Fisher at the top and with the address of the appellant and his
wife below. At his trial the question was raised whether credit was in fact given by his customers to him, as distinct from his
wife, but that issue was not raised before your Lordships. In addition to the fourteen cases the subject of the counts in the
indictment, 198 cases of similar conduct were taken into consideration before sentence.
Section 13 of the Debtors Act, 1869, reads as follows:

Any person shall in each of the cases following be deemed guilty of a misdemeanour, and on conviction thereof shall
be liable to be imprisoned for any time not exceeding one year (1) If in incurring any debt or liability he has obtained
credit under false pretences, or by means of any other fraud.

Section 155 of the Bankruptcy Act, 1914, in so far as it is material, reads as follows:

Where an undischarged bankrupt(a) either alone or jointly with any other person obtains credit to the extent of ten
pounds or upwards from any person without informing that person that he is an undischarged bankrupt he shall be
guilty of a misdemeanour.

For that offence the maximum sentence is two years imprisonment.


In respect of thirteen of the fourteen cases the subject of the indictment, charges were preferred in respect of each case under
the Debtors Act, 1869, and under the Bankruptcy Act, 1914. For instance, the first count was a charge under the Debtors Act,
1869, for obtaining credit to the amount of 30 from one Squier and the second count under the Bankruptcy Act, 1914, of
obtaining credit to that extent from him. In the fourteenth case (count five) the amount alleged to be obtained by credit was only
5, so that it was not possible to prefer a charge under s 155(a) of the Bankruptcy Act, 1914.
The contention of the Crown was that the words obtained credit in s 13 of the Debtors Act, 1869, and obtains credit in s
155 of the Bankruptcy Act, 1914, did not mean obtaining credit only in respect of the payment of money. It was argued on behalf
of the Crown, that credit was obtained within the meaning of the enactments where two persons have entered into reciprocal
obligations for the payment (that is the provision) of money or moneys worth and one party performs his obligations or part
of them in anticipation of performance by the other. Where money was paid by one party in reliance on the promise of the other
party to do work or supply goods, then it was said that credit was obtained for the value of the work or goods promised, and that
the value of the work or goods was to be assumed to be equal in value to the amount of money paid.
In this way it was sought to justify a charge of obtaining credit to the amount of x, when x was the amount actually
received by the person charged on his promise to execute work or to supply goods. The appellant on the other hand contended
that whether or not it was appropriate to describe in ordinary language a promise to render services or deliver goods in the future
in return for an immediate payment of money as involving an obtaining of credit by the recipient of the money, the provisions of
the statutes meant, and only meant, the obtaining of credit in respect of the payment of money.
391
Section 3 of the Debtors Act, 1869, provides that words and expressions defined or explained in the Bankruptcy Act, 1869,
shall have the same meaning in the Debtors Act, 1869. The Bankruptcy Act, 1869, which received the Royal Assent the same day
as the Debtors Act, 1869, did not define credit or obtained credit, but in s 31, which was concerned with what might be
proved in bankruptcy, the meaning of liability was defined as follows:

Liability shall for the purposes of this Act include any compensation for work or labour done, any obligation or
possibility of an obligation to pay money or moneys worth on the breach of any express or implied covenant, contract,
agreement, or undertaking, whether such breach does or does not occur, or is or is not likely to occur or capable of
occurring before the close of the bankruptcy, and generally it shall include any express or implied engagement, agreement
or undertaking, to pay, or capable of resulting in the payment of money or moneys worth, whether such payment be as
respects amount fixed or unliquidated; as respects time present or future, certain or dependent on any one contingency or
on two or more contingencies; as to mode of valuation capable of being ascertained by fixed rules, or assessable only by a
jury, or as matter of opinion.

The word liability in s 13 of the Debtors Act, 1869, must therefore be read as including the meanings given to it by s 31 of the
Bankruptcy Act, 1869 (now replaced by s 30 of the Bankruptcy Act, 1914).
Section 156 of the Bankruptcy Act, 1914, reads as follows:

If any person who has been adjudged bankrupt or in respect of whose estate a receiving order has been made(a) in
incurring any debt or liability has obtained credit under false pretences or by means of any other fraud he shall be guilty
of a misdemeanour.

This offence is punishable with one years imprisonment.


While it is curious that in 1914 Parliament should have created in relation to those adjudged bankrupt or in respect of whose
estates a receiving order had been made, a criminal offence in the identical terms of s 13 of the Debtors Act, 1869, which applies
equally to persons subject to the Bankruptcy Act, 1914, as to others, it is, I think, inconceivable that Parliament intended that a
different interpretation should be given to the words obtained credit in the Bankruptcy Act, 1914, from that given to the same
words in the Debtors Act, 1869. Nor, in my view, is there any valid ground for giving the words obtained credit in s 156(a) of
the Bankruptcy Act, 1914, a different meaning from that given to the words obtains credit in s 155(a) of the same Act. In my
opinion the words obtains credit and obtained credit in the Debtors Act, 1869, and in the Bankruptcy Act, 1914, were
intended to have, and have in all instances, the same meaning.
No case decided before 1956 was cited to your Lordships in support of the proposition that the meaning to be given to these
words was more extensive than the obtaining of credit in respect of the payment of money.
In R v Laker the appellant had agreed to get an engine to generate electricity for the owner of a cottage. Some days later he
again saw the owner and told her that he had ordered the engine and asked her to let him have a cheque for 125, which he said
was the price of the engine, explaining that he wanted to pay the manufacturers of the engine in advance. He obtained the cheque
but used the money to pay his debts and he neither returned the money nor delivered the engine. He was convicted of obtaining
credit by fraud contrary to s 156(a) of the Bankruptcy Act, 1914. Pritchard J delivering the judgment of the Court of Criminal
Appeal (Lord Goddard CJ Cassels and Pritchard JJ) said ((1949), 34 Cr App Rep at p 41) that the court was of the opinion that
there had been no obtaining of credit. What was meant by obtained credit was not discussed, but this decision 392 can only
have been based on the view that obtained credit in this section meant, and meant only, the obtaining of credit in respect of
some monetary sum. If obtained credit meant the obtaining of credit for the rendering of services or delivery of goods, then it
would follow that this case was wrongly decided.
In Osborn v Barton a bankrupt had been charged before the magistrates with an offence under s 155 of the Bankruptcy Act,
1914, on the following facts. He had advertised goods for sale on the terms cash with order. A firm sent him 25 for some
goods and he delivered them. The firm then sent him 100 for some more goods. He kept the money and did not deliver the
goods. The magistrates dismissed the charge and the prosecution appealed. The Divisional Court dismissed the appeal. In the
course of his judgment, with which Lewis J and Cassells J agreed, Lord Goddard CJ said ((1949), 66 (Pt 1) TLR at p 117):

So far as we can see he did not obtain credit at all. He did obtain 100, but that is not obtaining credit. Obtaining
credit means obtaining something on a promise to do something in the future. The defendant was to send the goods at
once.

The report of this case does not show that R v Laker was cited. If the ratio decidendi was just that he had not obtained credit
within the meaning of the statute when he obtained 100, then the decision is in line with R v Laker. The later observations made
by Lord Goddard CJ do suggest that he then felt that obtaining credit meant more than obtaining credit in respect of the
payment of money and also that the court attached importance to the fact that the goods were to be sent at once and not at some
time in the future. If, and in so far as, the decision depended on the fact that the goods were to be delivered at once and not in the
future, it would appear difficult to reconcile it with that in R v Jones where it was held that the defendant who ordered and
consumed a meal in a restaurant and was then unable to pay for it was rightly convicted of obtaining credit by fraud under s 13 of
the Debtors Act, 1869.
The case mainly relied on by the Crown was R v Ingram. The appellant was an electrician who had obtained contracts from
a number of shopkeepers for the erection or renovation of electric neon signs on their premises for an agreed payment. He
obtained from a number of shopkeepers part-payment of the price in advance but apart from minor, insignificant matters of a
preparatory nature, he did not do the work contracted for and he did not return the money paid to him by the shopkeepers. He
was charged on a number of counts and convicted of obtaining credit by fraud contrary to s 13(1) of the Debtors Act, 1869.
Donovan J delivered the judgment of the Court of Criminal Appeal, of which Lord Goddard CJ and Streatfeild J were the other
members, dismissing the appeal. He pointed out that the appellant had contended that there was no obtaining of credit, unless the
obligation towards another person who had given credit was to pay him money in the future. He thought that considerable light
was thrown on the problem by the statutory definition of the word liability contained in s 31 of the Bankruptcy Act, 1869, and
said that ([1956] 2 All ER at p 641; [1956] 2 QB at p 430):

when s. 13(1) of the Debtors Act, 1869, speaks of credit in incurring any debt or liability, it covers the case where
credit is obtained in incurring any liability to pay not only money but moneys worth.

With the greatest respect to the learned judge, I do not think that it follows that, because of the wide meaning to be given to the
word liability, a wider meaning than credit in respect of the payment of money is to be given to the words obtained credit.
To commit an offence against the section credit has to be obtained and in its ordinary significance, in my view, the expression
obtained 393 credit connotes the obtaining of credit in respect of the payment of money and no more. To constitute the offence
there must be the obtaining of credit in particular circumstances, namely, in incurring a debt or liability and by particular
methods, namely, under false pretences or by means of any other fraud. I do not think that the fact that the definition of
liability shows that there can be a wide variety of circumstances in which the offence can be committed is any ground for
interpreting the words obtained credit more widely than their natural significance imports.
R v Laker was cited to the court, but was not referred to in the judgment, in R v Ingram. I find it not possible to reconcile
the two cases.
In R v Dawson Finnemore J, delivering the judgment of the court (Finnemore, Davies and Hinchcliffe JJ) referred to the
cases Osborn v Baton and R v Ingram and said ([1960] 1 All ER at p 567):

We think the principles are not really difficult. If you buy goods and the man to whom you send the money does not
send the goods back to you or hand them over, it does not mean you have given him credit. He may be guilty of other
frauds, but he is not guilty of obtaining credit. On the other hand, if by arrangement you hand over to a person the
purchase money for the goods with the arrangement between you that the goods are not going to be delivered until some
time in the future, then that may be a case, as in R. v. Ingram, of credit being enjoyed by the person who has got to supply
the goods. We do not think that this case was that at all.

It would seem that the observations of the learned judge in regard to R v Ingram were obiter. It does not appear that the courts
attention was drawn to R v Laker and I do not think that this case assists in the determination of the questions which your
Lordships have to decide.
The Court of Criminal Appeal in the present case held that they were bound by the decision in R v Ingram in relation to the
offences under s 13 of the Debtors Act, 1869, and said that they would, if not so bound, have come to the same conclusion and
that in essentials the facts of the present case cannot be distinguished from those in R v Ingram. Winn J in delivering the
judgment of the court, saidf that it seemed clear to the court that the credit obtained was correlative to the postponement of
performance of the liability incurred.
________________________________________
f See [1963] 1 All ER 744 at p 749, letters f, g, where Winn J speaking of the ratio decidendi of R v Ingram said, It seems clear that the court
regarded the credit obtained as correlative to the postponement of performance of the liability incurred. This court adopts that reasoning as
the foundation of its judgment

I doubt whether this is an accurate test in relation to the obtaining of credit. Whether or not credit is obtained does not
depend on the length of time it was intended to be enjoyed or was enjoyed. In the case where a person consumes a meal in a
restaurant he obtains credit despite the fact that he is expected to pay immediately on consumption of the meal (R v Jones).
The Court of Criminal Appeal in this case held (See [1963] 1 All ER 744, holding (ii)) that the words obtains credit in s
155(a) of the Bankruptcy Act, 1914, only meant credit in respect of payment or repayment of a sum of money. In my view they
were right in that conclusion. It follows that unless there are valid grounds for holding that obtained credit in s 156(a) has a
different meaning from that which obtains credit has in s 155(a) it should be construed similarly in both sections and also in s
13 of the Debtors Act, 1869, the language of which is identical with that of s 156(a).
It is also to be noted that the Debtors Act, 1869, is entitled An Act for the Abolition of Imprisonment for Debt, for the
punishment of fraudulent debtors, 394and for other purposes. Section 13 of this Act is included in Part 2 which is headed
Punishment of Fraudulent Debtors. The long title of the Act and the heading of Part 2 support the view that the Act was
intended to deal only with those who owe money, debtors in the ordinary sense of the word, and the view that obtained credit in
s 13 means, and means only, credit for the payment of money.
As I have said, I do not think tha the wide meaning which can be placed on the word liability makes it necessary to give
the words obtained credit an extended meaning. Bearing in mind that the words obtains credit and obtained credit in s 13
of the Debtors Act, 1869, and in ss 155(a) and 156(a) of the Bankruptcy Act, 1914, should be given the same meaning unless the
context makes it clear that a different meaning should be given (which is in my view not the case) and also bearing in mind that
obtains credit in s 155(a) means credit in respect of payment or repayment of money, I have come to the conclusion that in all
three sections that is what is meant. It follows that R v Ingram was wrongly decided.
I realise that if your lordships agree with my opinion the result may be that some fraudulent persons will escape justice. In
R v Dent it was held by the Court of Criminal Appeal that a statement of intention about future conduct, whether or not it be a
statement of existing fact, is not such a statement as will amount to a false pretence in criminal law. Where it is not possible, as it
was not in the case of R v Dent, to sustain a charge of obtaining money by false pretences by alleging that the accused falsely
pretended that he was carrying on a genuine business, it follows that the person who obtains money fraudulently on promising to
render services or to deliver goods cannot be convicted either of obtaining money by false pretences or of obtaining credit by
fraud.
While it may be that consideration should now be given to closing this gap in the criminal lawand one possible solution
might be to change the law so that a false pretence need no longer be a pretence as to an existing factone cannot on this account
give a more extended meaning now to obtained credit or obtains credit.

LORD EVERSHED. I agree with the opinion just expressed by the noble Lord on the Woolsack.

LORD MORRIS OF BORTH-Y-GEST. My Lords, I also agree.

LORD HODSON. My Lords, I also agree. My noble and learned friend Lord Devlin, who is unable to be present today, has
asked me to say that he also agrees.

Appellants appeal against convictions under s 13 of the Debtors Act, 1869, allowed. Crowns appeal against the quashing of the
convictions under s 155(a) of the Bankruptcy Act, 1914, dismissed.

Solicitors: Jas H Fellowes & Son (for the appellant); Sharpe, Pritchard & Co agents for T Hambrey Jones, Chelmsford (for the
Crown).

C G Leonard Esq Barrister.


395
[1963] 2 All ER 396

Liverpool and County Discount Co Ltd v AB Motor Co (Kilburn) Ltd


CONSUMER; Consumer credit: CONTRACT

COURT OF APPEAL
LORD DENNING MR, DANCKWERTS AND DAVIES LJJ
25 MARCH 1963

Hire-Purchase Warranty Truth of statements in proposal form Measure of damages.

A printed hire-purchase proposal form for motor vehicles issued to dealers by a finance company contained a schedule for the
entry of particulars of the hirer, including his name, address, length of residence, whether householder, status (married or single),
occupation, name and address of employers (no questions asked) and length of time in present employment. The form also
contained a sales-note for signature by the dealers, which was addressed to the finance company and included the following
statement: In consideration of your agreeing to buy the vehicle we hereby certify and warrant that the statements contained
in the above schedule are true. The hirer, having told the defendant dealers that he wished to buy one of their vehicles, the
cash price of which was 259, on hire-purchase, purported to supply the required personal particulars, and the dealers entered
them on the form. The particulars supplied by the hirer were untrue. He gave the dealers another car in part-exchange, on which
they allowed him 52, and after he had signed the form he was permitted to take away the vehicle that he was to acquire on hire-
purchase. The dealers manager signed the sales-note and sent the completed form to the finance company, who accepted the
proposal and paid the dealers the balance of the purchase price, viz, 207. The hirer and the vehicle disappeared and the vehicle
was not recovered for a long time. In an action by the finance company against the dealers for damages for breach of warranty,
the county court judge awarded the finance company 136 10s, viz, the 207 together with 6 (expenses in tracing and collecting
the vehicle) and 8 10s (in respect of lost hire charges, which, if the agreement had run its full course, would have amounted to
45 10s), less 85 (the agreed value of the vehicle after its recovery). On appeal,

Held (i) The dealers had warranted the truth of the statements in the schedule and were liable to the finance company in
damages for breach of warranty (see p 398, letter e, and p 399, letter e, post).
(ii) The measure of damages for breach of the warranty was the reasonably foreseeable loss resulting from the breach, which
included the loss of the vehicle, and the county court judge had not erred in principle in assessing the damages (see p 399, letter
b, post).
Principle in Hadley v Baxendale ((1854), 9 Exch 341) applied.
Appeal dismissed.

Notes
As to collateral agreements between parties to hire-purchase agreements, see 19 Halsburys Laws (3rd Edn) 529, paras 850, 851.
As to the ordinary measure of damages for breach of contract, see 11 Halsburys Laws (3rd Edn) 240, paras 409 et seq; and
for cases on the subject, see 17 Digest (Repl) 9199, 99154.
As to what is a warranty on a sale of goods, see 34 Halsburys Laws (3rd Edn) 42, para 66; and as to measure of damages for
breach, see ibid, 157, para 263.

Case referred to in judgments


Hadley v Baxendale (1854), 9 Exch 341, 23 LJEx 179, 23 LTOS 69, 156 ER 145, 17 Digest (Repl) 91, 99.

Appeal
This was an appeal by the defendants, motor car dealers, from a judgment of His Honour Judge McIntyre QC on 18 January
1963, at the Willesden County Court, awarding the plaintiffs, a finance company, 136 10s for breach of a 396 warranty by the
dealers of the truth of statements of the hirer in a proposal form for the hire-purchase of a motor vehicle. The facts are stated in
the judgment of Lord Denning MR, post.

M J Anwyl-Davies for the appellants, the dealers.


H K Woolf for the respondent, the finance company.

25 March 1963. The following judgments were delivered.

LORD DENNING MR. This is a most unfortunate case for the two parties before the court today. On the one hand there is a
reputable finance company and, on the other hand, there are reputable dealers. They have been defrauded by a fraudulent hirer.
The question is: who is to bear the loss? The finance company have sued the dealers on a clause in an agreement whereby the
dealers warrant to the finance company that certain statements made by the hirer are true. The finance company sue the dealers
on that warranty.
On 22 November 1960, a man calling himself Paul Jackson went to the dealers premises and brought with him a Bedford
van. He wanted to acquire a 1952 Hillman Minx car at a cash price of 259 and give in part-exchange his Bedford van at 52,
making a balance of 207. He had not got the money to pay that sum at the time, so he wanted to acquire the van on hire-
purchase. The dealers manager got out the form of the finance company and took down particulars from the hirer. These
particulars turned out afterwards to be a cock and bull story. His name, he said, was Paul Jackson, his address he gave as
14, Bosworth Road, Kensington, W.10. He said that he was married; that he lived there 2 1/2 years and that he was the
householder; that his occupation was a tally-clerk at the docks; that his employers were the Port of London Authority, East
India Dock, and that he had been in their employment for 22 years. Those were the particulars which he gave. The manager
filled in the hire-purchase form. The hirer asked him to make arrangements about insuring the Hillman Minx. He said that he
would get temporary insurance: and the hirer said that he would bring in the money the next day. After all the details had been
filled in on the form, the manager himself, in order to induce the finance company to enter into the contract, namely, to buy the
car from him and to let it out on hire-purchase to this man, filled in the sales-note. One has only to read the sales-note to
realise that finance companies are becoming very anxious to protect themselves against the sort of frauds which have become
familiar to anyone who sits in this court. For instance, the price is not a genuine price or the vehicle taken in part-exchange is not
worth its declared value; or no cash is paid. This sales-note is signed by the dealers manager in favour of the finance company
and says this:

We agree to sell to you the vehicle at the total cash price shown and as described in the schedule above and upon your
acceptance of this transaction the property in such vehicle shall immediately pass to you. [These are the important words:]
In consideration of your agreeing to buy the vehicle we hereby certify and warrant.

I need not read the warranties in detail. The first is: That the initial payment shown above has been received by us in cash.
The second is: That the total cash price shown in the schedule is the true and fair price, and so forth. Then the fourth warranty,
which is the particular one that concerns us here, says:

That the statements contained in the above schedule and in this sales-note are true and that the vehicle is correctly
described in the schedule.

Included in those statements are the particulars of the hirer, which I have already read, including his address, 14, Bosworth Road,
his occupation, and so forth. That document being submitted to them, the finance company accepted it, signed it on 24
November 1960, and paid the 207 to the dealers.
Mr Jackson, if that was his name, took the Hillman Minx away on 22 November, immediately after he had signed those
documents. He left his Bedford van with the dealers and has never been seen since. He did not come the next day to pay 397 the
insurance as he promised. He just disappeared. A representative of the dealers went round to see if he could find him, but when
he went round to this address which was given, 14, Bosworth Road, he could not find anyone there who knew of a Paul Jackson
living in the house. They said the name did not ring a bell. He asked the milkman. He did not know of Paul Jackson either,
but when the Bedford van was described, the milkman said he knew the Bedford van, because it had been parked there. Paul
Jackson disappeared: and the Hillman Minx disappeared with him for a long time. No payments were made of any instalments.
Eventually the finance company put the matter into the hands of two reputable collecting agents. The car was not discovered
until February, 1962. It was then found in very bad condition in the possession of a Mr Bradburn from whom it was recovered.
It has now been established, not only that Paul Jackson was not known at 14, Bosworth Road, but that he had never been a
householder there nor lived there for two and a half years; he had never been a tally-clerk in the docks and he had never been
employed by the Port of London Authority. The particulars he gave were all wrong. Whereupon the finance company sued the
dealers under their warranty that the statements contained in the above schedule are true.
Are the finance company entitled to recover on this warranty? It has been argued before us that this is only a warranty, that
the dealers have set down accurately what Paul Jackson told them, that the dealers did not warrant the truth of what Paul
Jackson told them but that they warranted only that they had recorded accurately what he told them. This is a matter of the
construction of this sales-note. I am quite clearly of opinion that this is a warranty, not only that Paul Jackson made these
statements to the dealers, but also a warranty that they are true. In order to induce the finance company to enter into this
arrangement of buying the car and letting it out on hire, the dealers say: Here is this man: he has told us he is a householder; he
has told us he has been employed by the Port of London Authority for twenty-two years; and we warrant that what he told us is
true. Counsel for the dealers raised one special point. In the printed form the hirer had to state the name and address of
employers (no inquiries made). That was all in print. The answer in handwriting was this: Port of London Authority, East
India Dock. It was said by counsel that no inquiries made meant that the dealers had themselves not made any inquiries of the
employers to see if the particulars were accurate. I do not read the printed question in that way at all. It only means, as the judge
held that it meant, that the finance company were saying to the hirer: We are not going to trespass on your private affairs so far
as to make inquiries of your employers. It does not affect the scope of this warranty in the slightest. Then it was said: This is
not a warranty; it is not part of the contract at all. It is an innocent misrepresentation. This is again a question of the intention of
the parties. Having regard to the wording We hereby certify and warrant this is true, I have no doubt that it was a warranty.
The next question is: what are the damages? Counsel for the dealers said rightly that this was quite different from a
warranty of goods as, for instance, that they are in good condition. This is a warranty of the particulars describing an individual
who is going to enter into a contract. What are the damages that flow from it? Here again the ordinary test must apply. What are
the consequences which might reasonably be expected to be foreseen in consequence of a breach of it? It seems to me the
consequences can plainly be foreseen. If instead of getting a householder and an employee with twenty-two years service, they
get a man of straw who cannot be traced, of whom no one knows anything, it seems to me that the reasonable consequence to be
foreseen is the loss of the car, as indeed it was lost and only recovered after a long and difficult search. It is a loss which they
have involved themselves in by being induced to enter into this transaction.
Then it is said that the finance company did not take steps as quickly as they should: they did not tell the police as soon as
they ought to have done: they 398 ought to have taken steps more quickly to get back this car; and if they had done so, it might
have been recovered in a couple of months and the damages would have been very small. This point was put before the judge. It
is a question of fact whether the finance company did all that was reasonable in the circumstances to mitigate any damages
flowing from this breach of warranty. The judge found they did what was reasonable and assessed the damages in this way. He
took the 207 which the finance company had paid out in cash to the dealer, he allowed them for the loss of a proportion of their
finance charges, 8 10s, and 6 for the cost of collection, which totals 221 10s, and then he deducted the sum of 85 which the
parties had agreed as representing value of the car when it was recovered. The resulting figure came to the damages of 136 10s.
It seems to me that the judge has not in any way misdirected himself. He simply asked himself: what are the damages which
naturally and properly flow from the breach of this warranty? The warranty in effect was: the hirer here is a householder, a man
who has been in the same employment for twenty-two years. In truth he was a man of straw, flitting away without trace. I see no
error in the way the judge approached this case and I would, therefore, dismiss the appeal.

DANCKWERTS LJ. I agree. I do not find any ambiguity at all in the obligations which were undertaken by the dealers in the
present case. They entered into those obligations in these words: In consideration of your agreeing to buy the vehicle, we
hereby certify and warrant. That suggests to me an intention to enter into a warranty and I cannot see anything in the subsequent
provisions which limit that in any way. Therefore, they accepted the obligation of warranting that the statements contained in
the schedule and sales-note are true and that the vehicle is correctly described in the schedule. That means that they are
warranting that those statements are the truth and nothing but the truth, and if they enter into an obligation of that sort, it is their
own fault. It may be that the dealers did not read the provisions which they were warranting as carefully as they might have
done. Like most of the awkward provisions in this kind of contract, these provisions appear in very small print. If a dealer does
not make himself acquainted with what he is undertaking, as I said before, it is his own fault. One feels less sympathy for a man
who is getting money out of it than for the ordinary credulous hirer who takes a hire-purchase vehicle, usually a second-hand one,
on terms which are very often onerous. In the circumstances I can see no reason for differing from the learned county court
judge. I agree also on the question of damages.

DAVIES LJ. I also agree. Counsel for the dealers in his attractive argument placed great emphasis on one point. In his
submission, to saddle the dealers with liability, as it were, for the truth of the statements made by the hirer would be to impose on
them from a business point of view a quite unreasonable and impracticable task. He drew a piteous picture of the dealers
searching the register of births and marriages to ascertain whether the biographical details in the schedule were correct,
examining the deeds or the tenancy agreement of the residence of the hirer, and, I suppose, asking questions of the employer,
despite the fact that there is something in the nature of an undertaking in the document not to make any such inquiry, inquiring of
bankers, inquiring of finance companies, and, indeed, trying to prove negatives, eg that the hirer had no bank account and no
obligations to other finance companies. I think, speaking for myself, that there is very great force in those considerations. They
cannot, however, alter the plain meaning of the words, and if a liability of this sort is onerous for the dealers, the remedy lies in
their own hands. They must not enter into these agreements. The dealers are not a small garage: we were told that they are a
branch of a very large organisation. If, therefore, they can persuade hire-purchase companies to amend their agreements, so
much the better.
So far as concerns the damages, I can see no respect in which the learned judge has erred. Counsel for the dealers said that
the facts did not fall within the 399 principles of Hadley v Baxendale. I think that they do. If the question were asked: what is
likely to be the damage caused to the finance company if the statements made by the hirer turn out to be untrue and the hirer a
rogue, the answer would be that the finance company will in all probability lose the money that they pay to the dealers and very
likely will lose the motor car as well. It is on that principle, as I understand, that the learned judge has directed himself and
assessed the damages. I agree that this appeal should be dismissed.

Appeal dismissed.

Solicitors: William Charles Crocker (for the appellants); H Stock (for the respondents).

F Guttman Esq Barrister.


[1963] 2 All ER 400

Note
Re Westminster Bank Ltds Declaration of Trust
TRUSTS

CHANCERY DIVISION
WILBERFORCE J
2 MAY 1963

Trust and Trustee Administration of trust Child-bearing Leave to administer trusts on footing that married woman past
child-bearing Married woman entitled to protected life interest Thrice married and having had two children by first
marriage, but none for twenty-seven years Medical evidence of incapacity to have further children.

Notes
As to presuming whether a woman is past child-bearing, see 15 Halsburys Laws (3rd Edn) 347, para 627; and for cases on the
subject, see 22 Digest (Repl) 170172, 15591579.
As to the protection of trustees by applying for and acting under orders of the court, see 38 Halsburys Laws (3rd Edn) 1023,
1024, para 1766.

Adjourned summons
The plaintiff was born on 11 June 1911. She had married three times; first, on 6 February 1932, and there had been two children
of this marriage, born on 18 March 1933, and 19 September 1935; after the dissolution of this marriage in 1946, she had married
on 5 April 1947, and this second marriage had been dissolved in May, 1960, at the end of which month she had married again.
There had been no child of the second or third marriages. The third marriage was subsisting. The plaintiff was entitled to
protected life interests under two declarations of trust dated 12 February 1925, and 27 July 1925. She applied to the court by
originating summons dated 21 March 1963, for (1) an order that the ninth and tenth defendants, Westminster Bank Ltd and
Barclays Bank Ltd each the trustee of a declaration of trust, might be at liberty to deal with the settled one-third share of the
plaintiff under each declaration of trust on the footing that no further child would be born to her, and (2) an order under s 1 of the
Variation of Trusts Act, 1958, approving a proposed arrangement. The summons was supported by, among other evidence,
evidence of the plaintiffs doctor, who had attended her for fifteen years, that, menopause having, according to the plaintiff,
occurred, the plaintiff could have no further children.
The cases cited belowa were referred to in argument.
400
________________________________________
a Groves v Groves (1863), 9 LT 533, Re Millners Estate, (1872), LR 14 Eq 245, 42 LJCh 44, 26 LT 825, Browne v Taylor, (1872), WR 190,
Re Summers Trusts, (1874), 30 LT 377, Re Allasons Trusts, (1877), 36 LT 653, Re Belts Settled Estates, (1877), 25 WR 901, Croxton v
May, (1878), 9 Ch D 388, Re Taylors Settlement, (1881), 43 LT 795, Re Warrens Settlement, (1883), 52 LJCh 928, Re White, White v
Edmond, [1901] 1 Ch 570, Re Wohlgemuth, Public Trustee v Wohlgemuth, [1948] 2 All ER 882, [1949] Ch 12

J A Brightman QC and Martin Nourse for the plaintiff.


D S Chetwood for the first eight defendants.
J A Armstrong for the ninth and tenth defendants.

2 May 1963. The following judgment was delivered.

WILBERFORCE J. Having taken into account the circumstances of this case I think that it is proper to make the order asked
by para (1) of the originating summons. I have taken into account the age of the lady, her matrimonial history, the dates on which
her existing children were born, her evidence as to any possibility regarding the birth of further children and the not very
satisfactory medical evidence. Taking all those factors together, it seems to me that this is a case in which the court should
authorise the trustees to deal with her share on the footing that she will have no further child.
A number of cases has been cited to me by counsel for the plaintiff and the cases have been analysed very helpfully by him.
They are of some assistance, and I would only say, with regard to them, that, having regard to the dates on which they were
decided, I think that the courts would now be rather cautious in accepting them as laying down any sort of rule as to any
particular age. It seems to me that at this time one must take into account the physiological changes in human beings as a result
of improvements in health and also the different circumstances of family planning. Moreover, it should be borne in mind that
more accurate medical evidence is nowadays available, and ought to be put before the court, in dealing with an application of this
sort. Although I derive assistance from the cases, I cannot extract from them any rule which covers this or similar cases. I deal
with this case on the facts at present before me.
There is one other circumstance which it is necessary to take into account, though I would probably be prepared to authorise
the distribution without it, and that is that a large portion of the trust fund in any event is to remain in the hands of the trustees for
a certain period in case a charge for estate duty may arise. The only result of the order which I make is to protect the trustees and
not to extinguish any right that any future child might have. It is right to consider that this fund will be available if, by any
chance, a child should be born in future. Apart, however, from that consideration I make the order sought in para 1 of the
summons in the circumstances of this ladys age and family and matrimonial history.
[His Lordship further approved the arrangement proposed and made the order sought by para (2) of the originating
summons.]

Order accordingly.

Solicitors: Macfarlanes (for the plaintiff, the third, ninth and tenth defendants), Allen & Overy (for the first, second, fourth to
eighth defendants).

Jacqueline Metcalfe Barrister.


401
[1963] 2 All ER 402

Abraham v Jutsun
PROFESSIONS; Lawyers: CIVIL PROCEDURE

COURT OF APPEAL
LORD DENNING MR, HARMAN AND PEARSON LJJ
23 APRIL 1963

Solicitor Costs Payment by solicitor personally Notice of complaint and opportunity to answer it Order set aside in
absence of such notice or opportunity Supreme Court Costs Rules, 1959, r 8(1)(c), (2).

If an order is sought under the courts inherent jurisdiction against a solicitor that he should pay costs personally, he must be
given an opportunity to meet the complaint (dictum of Lord Wright in Myers v Elman, [1939] 4 All ER at p 508, applied), and, if
the case is within r 8(1)(c) of the Supreme Court Costs Rules, 1959, provision to this effect is made by r 8(2); accordingly, if no
such opportunity is given to the solicitor before the order is made, the order will be set aside on appeal.
Observations on the duty of an advocate in regard to the taking of doubtful points of law (see p 404, letter b, post).

Notes
As to the jurisdiction to order a solicitor to pay costs personally, see 36 Halsburys Laws (3rd Edn) 188, 189, paras 270, 271; and
for cases on the subject, see 42 Digest 337, 338, 38103818.

Case referred to in judgments


Myers v Elman [1939] 4 All ER 484, [1940] AC 282, 109 LJKB 105, 162 LT 113, Digest Supp.

Appeal
An appeal by way of Case Stated, brought by Leonard Wallace Abraham against the dismissal of two informations preferred by
him before justices for the county of Glamorgan against the respondent Eric Jutsun, was allowed by a Divisional Court of the
Queens Bench Division on 25 October 1962, who thereupon ordered that the respondents solicitor, Campbell Puntan, should
pay personally fifteen guineas towards the costs of the prosecution. The solicitor, the present appellant, appealed against so much
of the order as directed that he should personally pay costs. The facts appear from the judgment of Lord Denning MR.

Gerald Gardiner QC, John Wilmers and J Milnes Holden for the appellant solicitor.
Aubrey Myerson for the respondent, the prosecutor on the appeal to the Divisional Court.
23 April 1963. The following judgments were delivered.

LORD DENNING MR. The story arises out of proceedings in the Magistrates Court at Cowbridge in Glamorganshire. It
appears that on 9 January 1961, a man called Eric Jutsun was supposed to have committed two offences in regard to a Ford motor
car. It was said that on that day he aided and abetted another man, perhaps his brother, Peter Jutsun, to drive a car without a
proper driving licence and also on the same day that he allowed the car to be used without a proper insurance certificate. The
summonses were issued by the prosecution on 14 February 1961; the informations were then laid, but they were not served on
Eric Jutsun for many months: they were not served on him until 18 December 1961, and they did not come on for hearing before
the magistrates until 30 January 1962. At that hearing the superintendent of police, Mr Abraham, conducted the prosecution and
Mr Puntan, the appellant solicitor, appeared for the accused, Eric Jutsun. Evidence in support of the summons was called. The
appellant did not cross-examine; but after the prosecution closed their case, he addressed the court. He referred to the length of
time which had elapsed between the date of the offences charged, 9 January 1961, and the actual hearing, which was over twelve
months later on 30 January 1962, and he mentioned the hardship to the accused owing to the delay, because evidence which
would have been available was no longer available. Then he referred to s 244 of the Road Traffic Act, 4021960 (which was the
material section in regard to the insurance charge) in which it said that

summary proceedings may be brought [note the word brought] within a period of six months from the date of
the commission of the alleged offence.

The appellant argued that the words may be brought should be interpreted as meaning that the proceedings may be brought
before the court and heard within the statutory period of six months. He referred also to s 104 of the Magistrates Courts Act,
1952 (which was the relevant section in regard to the other charge in connexion with the driving licence), which says that

a magistrates court shall not try an information unless the information was laid within six months from the
time when the offence was committed.

Before the appellant had fully developed his submission the clerk to the justices rose and spoke to the justices. We do not know
what he said. Then the clerk to the court asked the police superintendent why the summonses had not been served immediately
after they had been issued. The police superintendent said that the police had not been able to trace the defendant. He added that
in his view the proceedings were not out of time. The appellant then got up and told the court that Eric Jutsun was in fact serving
a term of imprisonment in Swansea gaol at the time, so it should have been quite easy for them to serve him. There was a short
discussion between the clerk and the justices, and the justices announced that both summonses would be dismissed. The
prosecution asked for a case to be stated. The Case Stated said that the magistrates dismissed the summonses on the ground that
they were out of time.
When the case stated came before the Divisional Court on 25 October 1962, the Divisional Court had no difficulty in
allowing the appeal because the relevant word in each case (the word brought in the one section and the word laid in the
other) refers to the time when the information is laid; and in both these summonses the information was laid on 4 February 1961,
well within the six months, indeed within five weeks of the offences. Therefore, the appeal was allowed.
One would have thought it might have been the end of the case, but when counsel for the prosecution asked for costs and
pointed out that Mr Jutsun was legally aided, the judges of the Divisional Court evidently took the view that the appellant in the
magistrates court had taken a thoroughly bad and unmeritorious point: and that the bad point had had the effect of causing all
the costs in the Divisional Court. Thereupon an order was made directing that the appellant should pay fifteen guineas personally
towards the costs of the prosecution.
The appellant now appeals against that order and I am afraid that the Divisional Court acted per incuriam in making the
order which they did. True it is that the High Court has a summary jurisdiction over solicitors in pursuance of which they have
power to order them to pay costs, if the costs have been caused by the solicitors misconduct. That was always the case at
common law; but at common law it always was held that no such order should be made unless fair notice was given to the
solicitor of the matter alleged against him as misconduct, and he was given a fair opportunity of being heard in answer. That rule
of the common law has now been given the force of rules of court in the Supreme Court Costs Rules, 1959, r 8(1)(c) and (2),
which are material here. It is quite plain that the Divisional Court did not give the appellant an opportunity of being heard. He
was away in Swansea: his agent was conducting the case in London, and he was given no opportunity of meeting such a charge
against him, as he should have been.
On that ground alone we must, I think, interfere with the decision of the Divisional Court. But I think it only fair to the
appellant to say that his evidence on affidavit, which is not challenged, makes it quite plain that he was not in the 403 least degree
guilty of any misconduct. The points which he took were fairly arguable. The one point on the word brought had a good deal
to be said for it. The other point on the word laid had much less to be said for it; and the appellant said much less. The long
delay gave merit to points which would otherwise appear unmeritorious. As it turned out, both points were bad points; but the
appellant was not the judge of that. The magistrates had their clerk to advise them on the law. He was to advise them whether
the points were good or bad. It was not for the advocate to do so. Appearing, as the appellant was, on behalf of an accused
person, it was, as I understand it, his duty to take any point which he believed to be fairly arguable on behalf of his client. An
advocate is not to usurp the province of the judge. He is not to determine what shall be the effect of legal argument. He is not
guilty of misconduct simply because he takes a point which the tribunal holds to be bad. He only becomes guilty of misconduct
if he is dishonest. That is, if he knowingly takes a bad point and thereby deceives the court. Nothing of that kind appears here.
I would hold, therefore, as a matter of procedure, that the Divisional Court were unfortunately in error in not hearing the
appellant; and, as a matter of substance, that, when the facts are ascertained, there was no misconduct in the appellant in taking
the point which he did take.
I think, therefore, that this appeal should be allowed.

HARMAN LJ. That the appellant was not called on to defend his conduct is enough to make it clear that this appeal must be
allowed; but the further point on which my Lord has pronounced is of more importance I think. The court under its inherent
jurisdiction and under the rules can always control a solicitor and can go to the length of making him pay personally costs which
have been thrown away, but thrown away either by his misconduct or by his negligence, which is only a negative form of
misconduct. Now if it be misconduct to take a bad point, a new peril is added to those of the legal profession, and unless a bad
point be taken knowing it to be bad and concealing from the court, for instance, an authority which shows it clearly to be a bad
point, then it would be a very dangerous doctrine indeed to say that the advocate ought to be mulcted in the costs because he took
a point which failed. I would, therefore, allow the appeal.

PEARSON LJ. I agree that the appeal should be allowed, and there are two quite clear points here on which plainly it must be
allowed. The first is that the solicitor who was ordered to pay costs personally had not been given an opportunity to meet the
complaint and say what he wished to say on the matter. That such an opportunity should be given is plain on the authority of
Myers v Elman and the material passage is in Lord Wrights speech ([1939] 4 All ER at p 508; [1940] AC at p 318). That is the
position as it would be under the inherent jurisdiction of the court. There is also now in the Supreme Court Costs Rules, 1958, r
8(1) and (2), a special provision which ensures, in some cases at any rate, which include this case, the right of the solicitor to be
heard in his own defence to give any explanation which he may have before any such order is made against him. That is the first
point. The second point is that an affidavit which has been put in by the appellant in this case is unchallenged and is accepted,
therefore, as correct. It is quite plain that there was no misconduct or default on his part in this particular case.
Speaking for myself, I would rather leave the matter there and not go into the question whether there ever could, in some
hypothetical circumstances, be a case in which it might constitute misconduct for a solicitor appearing as an advocate to take a
bad point. It could obviously only be in an extreme case that such a conclusion could be reached, but I think that it is better not
to consider in advance what the position might be in other hypothetical cases which 404 are not before this court. I would,
therefore, prefer to decide this appeal solely on the two points which have been mentioned, namely, that no opportunity was given
to the appellant to explain the position, and, secondly, that when the explanation is afforded in the affidavit it is quite plain there
was no fault on the appellants part in this case.

Order of the Divisional Court set aside.

Solicitors: Waterhouse & Co (for the appellant solicitor); Lewin, Gregory, Mead & Sons (for the respondent prosecutor).

F Guttman Esq Barrister.


[1963] 2 All ER 405

Preston (otherwise Putynski) v Preston (otherwise Putynska) (otherwise


Basinska)
FAMILY; Other Family

COURT OF APPEAL
ORMEROD, DONOVAN AND RUSSELL LJJ
25, 26, 27, 28 FEBRUARY, 4 APRIL 1963

Marriage Foreign marriage Validity Marriage in Germany of domiciled Poles Husband a member of Polish forces in
Polish camp in Germany Camp under control of allies Allies in belligerent occupation of Germany Ceremoney of marriage
performed by chaplain in accordance with Roman Catholic rites Whether marriage valid as English common law marriage
abroad.

In June, 1945, the husband and the wife went through a ceremony of marriage at a Polish camp at Nordheim in Germany before a
Roman Catholic priest who was the camp chaplain. Both the husband and the wife were born of Polish parents and were
domiciled in Poland. At the date of the marriage the husband was a member of the Polish army, but was not taking any
immediate part in its activities; he was at a camp which existed under the control of the British army in occupied territory in
Germany for the reception of Polish military personnel, and was not a member of the belligerent occupying forces. The wife was
a civilian, and the evidence did not indicate that she was in the camp for any other reason than her marriage. The camp itself was
under the control of the allied forces of the United Kingdom, the USA, USSR and France, who were in belligerent occupation of
Germany. The husband and the wife had some dealings with the Germans, but clearly did not intend to subject themselves to
German law with regard to their marriage. The marriage was not valid by German law, since it had not been registered before
1950. On an issue whether the marriage was recognised as valid by English law,

Held The marriage was valid by English common law, since the exception in favour of members of occupying forces to the rule
that both parties to a marriage abroad must be taken to have subjected themselves to the law of the country where the marriage
took place extended in the present case to the husband for, although he was not a member of the army occupying forces, he was
(i) (per Ormerod LJ) a person who was in a foreign country as part of an organisation commonly set up when there was
hostile occupation and who was there for the purposes of such occupation (see p 411, letters g and h, post) or
(ii) (per Donovan LJ agreeing with the conclusions of Cairns J at first instance) a person who, although not a member of the
army occupying forces, was associated with those forces (see p 412, letter e, post) or
(iii) (per Russell LJ) a member of a group of allied soldiers organised under military command for the military purposes of
allies, one of whom was in actual control of the conquered territory in which the group was situated (see p 415, letter g, post).
405
Taczanowska v Taczanowski ([1957] 2 All ER 563) and Ruding v Smith ((1821), 2 Hag Con 371) applied.
Kochanski v Kochanska ([1957] 3 All ER 142) criticised.
Decision of Cairns J ([1962] 3 All ER 1057) affirmed.

Notes
It is not easy to state with precision the character of the camp in which the husband was at the time of the marriage or his function
or position militarily; the statement at p 405, letter e, ante, follows the judgment of Ormerod LJ (see p 409, letter g, post), but this
should be considered with that of Russell LJ (see p 414, letter h, post), while Donovan LJ expressly agreed with the findings of
fact in the court below (see p 412, letter e, post, and [1962] 3 All ER at pp 1059, 1062, letter e).
As to the lex loci celebrationis governing the forms and ceremonies of marriage, see 7 Halsburys Laws (3rd Edn) 93, para
168, note (1); and for cases on the subject, see 11 Digest (Repl) 462464, 953971.
As to the validity of a marriage at common law, see 7 Halsburys Laws (3rd Edn) 94, para 170, note (r).

Cases referred to in judgments


Berthiaume v Dastous [1929] All ER Rep 111, [1930] AC 79, 99 LJPC 66, 142 LT 54, 11 Digest (Repl) 462, 954.
Kochanski v Kochanska [1957] 3 All ER 142, [1958] P 147, [1957] 3 WLR 619, 3rd Digest Supp.
Lazarewicz (otherwise Fadanelli) v Lazarewicz [1962] 2 All ER 5, [1962] P 171, [1962] 2 WLR 933.
Merker v Merker [1962] 3 All ER 928, [1962] 3 WLR 1389.
Pilinski v Pilinska [1955] 1 All ER 631, [1955] 1 WLR 329, 3rd Digest Supp.
Ruding v Smith (1821), 2 Hag Con 371, 1 State Tr NS 1054, 161 ER 774, 11 Digest (Repl) 465, 982.
Starkowski v A-G [1953] 2 All ER 1272, [1954] AC 155, [1953] 3 WLR 942, 3rd Digest Supp.
Taczanowska (otherwise Roth) v Taczanowski (Krystek cited) [1956] 3 All ER 457, [1957] P 301, [1956] 3 WLR 935, revsd CA,
[1957] 2 All ER 563, [1957] P 314, [1957] 3 WLR 141, 3rd Digest Supp.
Interlocutory Appeal
The parties went through a ceremony of marriage on 3 June 1945, in Germany. They later came to England, where the husband
had now acquired a domicil of choice. They separated in 1959 and on 19 October 1960, the husband presented a petition seeking
a declaration that the marriage was null and void on the grounds that it was not celebrated in accordance with German law and
had not been validated since, and that it had not been celebrated in circumstances which would make it valid notwithstanding
non-compliance with the lex loci celebrationis. The wife by her answer denied that the marriage was invalid. Her effective case
was that the marriage was valid as an English common law marriage, though in the alternative she claimed that it was valid by
Polish law; and she cross-prayed for divorce on the ground of the husbands cruelty and adultery. On 3 July 1961, Phillimore J
directed by consent the trial of the issue as to the validity of the marriage. Cairns J held ([1962] 3 All ER 1057) that the marriage
was valid by English common law as the parties had not intended, and were not obliged, to submit themselves to German law.
The husband now appealed. The facts are stated in the judgment of Ormerod LJ.

R L Bayne-Powell for the husband.


George Dobry for the wife.

Cur adv vult


406

4 April 1963. The following judgments were delivered.

ORMEROD LJ. This appeal is from a decision of Cairns J that a marriage between the parties in this case (whom I will call the
husband and wife), celebrated in Germany on 3 June 1945, was a valid marriage. The parties are of Polish origin. The
husband was called Putynski, but in 1955 he became a naturalised Englishman and changed his name to Preston. He is now
domiciled in England. On 19 October 1960, the husband filed a petition for a declaration that the marriage was null and void as it
was not celebrated in accordance with the lex loci celebrationis. The wifes case is that the marriage was an English common law
marriage, and therefore valid. She said further that the marriage was valid according to Polish law. That submission was not
pressed on behalf of the wife, and indeed, in my judgment, there is no substance in it. It was not in issue that the marriage was
not in accordance with German civil law, by which the only valid marriage is one performed in the presence of a civil registrar.
For the sake of completeness, I should add that the wife in her answer alleged cruelty and adultery on the part of the husband, and
cross-prayed for a decree nisi on those grounds. On 3 July 1961, Phillimore J directed a trial of the issue of the validity of the
marriage, which is the issue with which the court is now concerned.
The marriage was performed in a camp at Nordheim by a Roman Catholic priest. It is not in issue that he was an
episcopally ordained priest. Nor is it questioned that the parties were capable of contracting a valid marriage, and believed that
they had done so. There were no children, but the parties cohabited as husband and wife for many years. The nature of the camp
and the status of the husband at the time of the marriage are relevant questions. The husband was born in Warsaw in February,
1924, and was domiciled in Poland. After the war broke out he joined the Polish underground movement, which later became the
Polish Home Army and was recognised as a belligerent army by His Majestys Government in 1944. He took the army oath in
1942, and in 1943 went to an officers training school and became a cadet corporal, which entitled him to wear the uniform and
badges of his rank. He took part on 1 August 1944, in the Polish rising against the Germans. He was at Legjonowo, where the
rising failed after two weeks. He then went to Praga, but was evacuated from there by the Germans, and came eventually to
Poznan, in German-occupied Poland. He worked in a laundry there and met the wife, who was born in Berlin of Polish parents,
but had always lived in Poland from the age of two and was domiciled there. The parties became attached to one another and
wanted to marry, but there were difficulties, although a priest gave them his blessing before they left Poznan. According to the
finding of the learned judge, they left Poznan to escape the Russian advance, and went to Dessau and then to Leipzig. Although
the parties were not then married they appear to have been behaving as a married couple. In April or May, 1945, the Americans
occupied Leipzig, and, about the end of May, the parties got permission to go to Nordheim. This is the description of the camp
given by the learned judge ([1962] 3 All ER at p 1059):

This was a military camp, set up by the Americans but shortly afterwards taken over by the British, for the reception
of Polish military personnel. The Commandant was a Polish officer and the purpose of the camp was to train men and
women who had served in the Polish forces to fit them for service in Germany or Italy or elsewhere. In May, 1945, the
organisation of the camp was rudimentary and it was some time before regular drill, etc., began; but the camp had a
military purpose and was not a mere reception centre for displaced persons.

Evidence of the conditions in the camp was given by Mrs Osuchowska, the commandant of the womens block. Mrs
Osuchowska had been a prisoner of war in German hands. She said that the wife was treated as a single woman and was housed
in her block, where the husband was allowed to visit her, but 407 only as a finance. The learned judge found this to be so, and
found too that the parties did not pass as husband and wife when they first came to the camp. Mrs Osuchowska said, too, that
anyone who wished to leave the camp or to be away for a night had to give notice, but this was because of rations. There was
some elementary military training in the camp, and this developed as time went on. It appears, too, that the camp was in charge
of a Polish military officer, although there was no evidence of his place in the chain of command of the Polish army. The judge
came to the conclusion that the purpose of the camp was to train men and women who had served in the Polish forces to fit them
for service in Germany or Italy or elsewhere. He concluded that it was a military camp and not, as was the conclusion of Sachs J,
in Kochanski v Kochanska, a camp for displaced persons (a conclusion arrived at on different evidence). It may not be important
to be able to describe the camp exactly. There is little evidence to show what discipline, if any, was exercised over the inmates of
the camp. There is, however, in my judgment, enough evidence to show that the camp was at least a part of the organisation of
the allied armed forces, that its inmates were associated with, if not members of, those forces, and that preparations were being
made to integrate them more closely with one or another of the Allied armies.
A further question to be decided is the status of the husband at the time of the marriage. The judge concluded that he was a
member of the Polish Armed Forces. He said this ([1962] 3 All ER at p 1059):

In my view, the husband was at the material time a member of the Polish forces. I base this conclusion on three
factors: (i) the evidence of General Pelczynski, former chief of staff to the Polish commander-in-chief, as to his technical
status; (ii) evidence of Mrs. Osuchowska, commandant of a womans block at the camp, that it was always a military camp;
(iii) the husbands unquestioned wearing of uniform with badges of rank in the camp.

According to the evidence of the General most of the members of the Home Army who took part in the Warsaw rising were taken
prisoner by the Germans, but some mixed with the civilian population. He said that the Home Army was part of the Polish forces
commanded by the Polish commander-in-chief from London. It would appear, therefore, that there can be no doubt that the
husband was at one time a member of the Polish forces. On 19 January 1945, however, the Home Army was dissolved, and the
question arises whether the husband then ceased to be a soldier. In re-examination the General said this:
Q.General, did this dissolution apply to all soldiers who were then in Poland, in Germany and elsewhere? A.With
the exception of those who have been staying living in Poland. Q.Did it apply to those who were in Poland or to those
who were outside Poland? A.The order dealt with dissolution of the Home Army as such and could be applied only to
those soldiers who have been in Poland under the commandant of Polish commanders and obviously could not be applied
to those soldiers who have been prisoners of war, and therefore outside the power and scope of the Polish commanders, or
could not be applied to those soldiers who were deported to Germany and were under German power and in the hands of
the Germans. Q.Pursuant to that order would any discharge formalities have to be gone through? A.What do you
mean by formalities in this case? Q.Would anybody have to tell a soldier, you are now discharged? A.Yes. Q.
Then two more questions, General. You mentioned that during the rising some soldiers were accepted without swearing-
in? A.Yes. Q.Having regard to the records, can you tell my Lord whether Corporal Putynski was accepted to the
Army without swearing-in? A.On the contrary. Corporal Putynski has been accepted to the Army and has made 408 his
oath. He has been sworn-in. Q.In Germany, you were asked about this: there were four zonesBritish, American,
French and Russian. In which zone did the Polish troops serve as an occupying force as you said? A.In the British
Zone.

Then learned counsel said: I have no further questions. Then the learned judge said:

I have one question I want to ask you. I want you to consider the case of a man who had been a sworn member of the
Home Army, who after the Warsaw rising had been removed by the Germans to Poznan, and who after that had been able
to go himself to Dessau, and then to Leipzig, and then to Nordheim. If anybody had asked such a man after Jan. 19, 1945,
Are you at present a member of the Polish Home Army?, what would the true answer have been? (The witness):
According to my opinion the true answer of such a man should have been, Yes, I am a member of the Home Army, and
that is why it is because when being on German territory such a man could not have been officially informed about the
order of Jan. 19, 1945. He could not be informed or notified. (CAIRNS, J.): Yes, thank you, General.

The second reason given by the judge, that the camp was a military camp, supports the Generals evidence in that it seemed
unlikely that the husband (and consequently the wife) would have been admitted to the camp if he had not been regarded as a
member of the Polish Army.
The judge relied further on the wearing by the husband of uniform with badges of rank as a cadet corporal. To my mind,
that is not a strong reason. The uniform in question was a nondescript affair made from a tunic and greatcoat (which became the
breeches) which he had obtained from an American soldier. In evidence he gave as his reason for wearing it that it was easier to
get about the country on his motor-cycle in uniform than otherwise. He had added the badges of his rank as he considered
himself entitled to wear them.
It is difficult to discover from the evidence whether the husband at the relevant time was under the compulsion of any army
discipline. It is equally difficult to determine exactly why he was in Germany. There is no evidence that he was moved to
Germany as part of a compulsory labour scheme, or, indeed, that he went there for any other reason than a desire to be lost, or at
least not to be recognised as having taken part in the Warsaw rising. In my judgment, this case must be considered on the basis
that at the relevant date the husband was a member of the Polish Army but was taking no immediate part in its activities. He was,
however, an inmate of a camp which existed under the control of British Army authorities for the reception of Polish military
personnel.
There is little or no evidence about the status of the wife, but, in view of the judgment of this court in Taczanowska v
Taczanowski, this does not seem to be material. We know, as I have already said, that the husband met her when they were both
working in a laundry in German-occupied Poland.
It is on these facts that the questions of law must be considered. The husbands case is that the marriage was invalid because
there was no compliance with the lex loci celebrationis. There was indeed no attempt to comply with it, although it may well be
that the civilian population in that part of Germany was still sufficiently organised to have made compliance possible. The wifes
case is that the circumstances were such that compliance with the lex loci was unnecessary. The facts of the case are different
from those in Taczanowska v Taczanowski. There the husband was a member of the Polish Army stationed in Italy as part of the
forces in hostile occupation. The wife was under no form of military discipline. She was a Polish refugee living in a convent in
Rome. In that respect, the cases appear to be similar, as the evidence about the wife in this case does not indicate that she was in
the camp at Nordheim for any reason other than her 409 intended marriage. Counsel for the husband submitted that
Taczanowska v Taczanowski must be wrong, as it was contrary to decisions of the House of Lords. The questions to be decided
there, however, were questions of capacity and not of form. The court is bound by Taczanowska v Taczanowski so far as it
applies. In any event, I should find it difficult to accept that, although a husband for sufficient reason is under no obligation to
observe the lex loci, the position would be different if the wife were under such an obligation.
It is not easy to extract from the authorities the correct principle to be applied. The earliest case which we need to consider
is Ruding v Smith. That was a husbands claim to set aside a marriage between British subjects celebrated in a private house at
the Cape of Good Hope on 22 October 1796. The officiating clergyman was the chaplain of the British forces occupying the
colony, which had capitulated in the previous year, and the marriage was by licence or permission of the then commander of
those forces. Apart from the fact that the wife was born in India and came to the Cape in 1796, nothing seems to be known about
her. The husband, however, was a captain in the 12th Regiment of Foot and came to the Cape on his way to the East Indies, in
accordance with orders. The husband claimed that the marriage was void as the lex loci had admittedly not been observed; but it
was held that it was valid. At the end of his judgment Lord Stowell said this (2 Hag Con at p 394):

In my opinion, this marriage (for I desire to be understood as not extending this decision beyond cases including
nearly the same circumstances) rests upon solid foundations. On the distinct British character of the partieson their
independence of the Dutch law, in their own British transactionson the insuperable difficulties of obtaining any marriage
conformable to the Dutch lawon the countenance given by British authority, and British ministration to this British
transactionupon the whole country being under British dominionand upon other grounds to which I have adverted; and
I therefore dismiss this libel, as insufficient, if proved for the conclusion it prays.

It seems probable that, in coming to his decision, Lord Stowell had in mind the fact that it was impossible in the circumstances
for a marriage to be contracted between two British subjects which was in conformity with lex loci. He referred in the passage
which I have just cited to the insuperable difficulties of obtaining any marriage conformable to the Dutch law. But he seems
too to have been taking into account the fact that the Cape was conquered territory and the husband was a soldier of the
occupying country. In an earlier passage (2 Hag Con at p 382) he says this:

I am perfectly aware that it is laid down generally in the authorities referred to that the laws of a conquered country
remain till altered by the new authority. I have to observe first that the word remain has, ex vi termini, a reference to its
obligation upon those in whose usage it already existed and not to those who are entire strangers to it in the whole of their
preceding intercourse with each other. Even with respect to the ancient inhabitants, no small portion of the ancient law is
unavoidably superseded by the revolution of government which has taken place.
In the more recent case of Taczanowska v Taczanowski, the husband was an officer serving in a Polish unit stationed in Italy
as part of the forces of belligerent occupation. The wife was a civilian Polish refugee living in a convent in Italy and the marriage
was in a parish church in Rome celebrated by a Polish Army chaplain serving with the Polish Second Corps. It was held in that
case that, although there was no attempt to conform with the lex loci and that such conformity would have been comparatively
easy, the marriage was nevertheless valid as an English common law marriage. The basis of the judgments in this court was 410
that, although parties who marry in a foreign country must be considered to have submitted to the laws of that country so far at
least as the form of marriage is concerned, such a rule did not apply in the case of a soldier serving in a force of belligerent
occupation.
In Kochanski v Kochanska Sachs J had to consider a marriage of parties living in the same camp as the camp in this case.
The marriage was within a few days of the one we have to consider and was performed by the same priest but in a Roman
Catholic chapel in the vicinity. There was no attempt to observe the lex loci. The husband was a Polish army officer who had
been a prisoner of war in German hands until he was liberated by the Russian advance, when he went (in May, 1945) to the camp
at Nordheim. Sachs J found (on other evidence than that before us) that the camp was a displaced persons camp, and that, in the
circumstances, the husband could not be deemed to have submitted himself to the lex loci, although he did not consider him to be
a member of the occupying forces. In my judgment Sachs J went further than the cases warrant, although it may well be, on the
evidence of the status of the camp before us, that his result was justified.
The only other decision to which I need refer is that of the President, Sir Jocelyn Simon, in Merker v Merker. The parties
there were both Polish, serving in the Polish Armoured Division, part of the allied armies of occupation. The case came within
the decision in Taczanowska v Taczanowski, but the President said in the course of his judgment ([1962] 3 All ER at p 934) that
Taczanowska v Taczanowski was confined to marriages within the lines of an army of hostile occupation.
It is clear from a consideration of the above cases that the courts should be reluctant to extend the instances where
compliance with the lex loci is unnecessary to constitute a valid marriage; but, essentially, marriage is a matter of agreement
between the parties to it. Countries have from time to time prescribed regulations which must be complied with if the marriage is
to be considered valid. Some of these regulations deal with capacity, others with form only. The canons of international comity
demand that, on questions of form, one country should recognise the marriage laws of another, and in consequence it is now well
established that persons choosing to marry in a foreign country shall be deemed to submit themselves to the marriage laws of that
country. This rule does not apply in all circumstances, but it is not enough for the parties to say or to show by their actions that
they do not intend to submit to those laws. I would not attempt any exhaustive definition of the circumstances in which the rule
does not apply, but a clear case, in my judgment, was that of Taczanowska v Taczanowski where the husband was in Italy because
of military service. I think, too, that the rule would not apply to persons who are in a foreign country as part of the organisation
necessarily or at least commonly set up when there is hostile occupation. This exception, in my judgment, would not of necessity
include inmates of what has been described as a displaced persons camp. In this case the parties were in the camp at Nordheim,
where the husband could receive military training or else be directed to a place appropriate to the duties required of him. The
camp existed as part of the organisation set up for the purposes of the hostile occupation of Germany and the inmates of it were
there for the purposes of such occupation. In these circumstances, there was, in my judgment, no obligation on the parties to
observe the lex loci, and the learned judge was right in holding that this was an English common law marriage and therefore
valid.
I would dismiss the appeal.

DONOVAN LJ read by Ormerod LJ). This husband whose name was originally Putynski, went through a ceremony of marriage
with the wife on 3 June 1945, at Nordheim in West Germany. Each wanted to marry the other, and, after the ceremony, each
considered that they had done so. Each was a 411 Pole and Roman Catholic; and the ceremony was performed publicly by a
Roman Catholic priest. The parties then lived together as man and wife for some fourteen years. The husband now desires his
freedom, and, to that end, has presented a petition for a decree of nullity. He says that the marriage had been void all along
because the ceremony did not comply with a provision of the local German law that marriages must be registered before a
German civil registrar, which this marriage was not.
The respondent wife has cross-petitioned for a decree of divorce on the grounds of the husbands cruelty and adultery, which
petition has still to be heard. The advantage to the husband of obtaining a decree of nullity is that he may avoid some of the
obligations of maintenance which he would otherwise have to bear; and it would be unreal to suppose that he is not well aware of
that situation.
The husbands petition has necessitated inquiries of the Foreign Office and the War Office (extensive in the case of the
Foreign Office) concerning the political and military situation in Germany at the end of the War. The hearing of the husbands
petition occupied four days before Cairns J and the hearing of his appeal has occupied another four days before us. There are no
merits in the husbands petitionhe is simply tired of his wife; and he revealed himself before the learned judge as an
unscrupulous witness. As a witness the wife, apparently, was not much better.
It is, I think, unfortunate that difficult questions of fact and law have to be resolved in such an undeserving case. The
husband is seeking to get rid of his wife on the merest technicality; and there must be a danger in such a case that one will
unconsciously stretch a doubtful point against him to the detriment of the relevant law, which may thereby acquire less precision
than it ought to have. The learned judge, if I may say so, considered this case with obvious care and impartiality. I agree with the
findings of fact and his conclusion of law. And, lest by adding something to his judgment I may prejudice some future and more
deserving case, I say no more. I think that the appeal should be dismissed.

RUSSELL LJ. I will, first, attempt a summary of the cases touching on this kind of question prior to the decision now appealed
from.
First, Taczanowska v Taczanowski in this court. The husband was an officer serving in the ordinary way in the Polish
Second Corps stationed in Italy. That corps was a part of forces in belligerent occupation of Italian territory in the expressed
view of the Foreign Office. The wife was a civilian Polish refugee living in a convent in Rome. The marriage was in a parish
church in Rome. The priest was a Polish army chaplain serving with the Polish Second Corps. No attempt was made to comply
with the formalities required by the lex loci, though compliance would have been simple. Both intended to marry and thought
that they had. Karminski J, accepted that the husband was a member of a military force in occupation of a foreign state, and saw
nothing in the judgment in Ruding v Smith to indicate that Lord Stowell would necessarily have taken a different view of that
case if the transaction had not concerned British subjects; but, in the end, he (in effect) followed Pilinski v Pilinska, and held the
marriage void. In the Court of Appeal the wife no longer opposed the marriage. For the appeal it was argued that the lex loci of
marriage formalities need not apply to a marriage in an occupied country by a member of the occupying forces, on the basis of
the Ruding case. Hodson LJ pointed out the general rule that persons entering into a marriage are to be taken to subject
themselves to the lex loci as to formalities. He held that that general rule did not apply to the case of marriage entered into in an
occupied country by a member of the occupying forces who did not demonstrate an intention so to subject himself. He based his
view on what was undoubtedly a leading aspect of the judgment in Ruding v Smith, that is to say, the position of a conquering
army in relation 412 to the laws of the conquered country. There being no ground for applying the lex loci, he applied the lex
fori, and found that there was what is known as an English common law marriage validly contracted. Parker LJ said that the rule
of the applicability of the lex loci is based on this, that by contracting marriage in the locus the parties must be taken to subject
themselves to that law; but he saw no ground in principle for taking a member of a military force in belligerent occupation of a
conquered country to be thus subjecting himself, unless there was evidence that he did. This view was related also to the same
aspects of the judgment in Ruding v Smith. Accordingly the lex loci was not applicable, nor was the lex domicilii, but the English
common law was. Ormerod LJ put the matter slightly differently. He stressed the fact that, not only was the husband a member
of an occupying army, but was in Italy under military compulsion, and should not therefore be deemed to have submitted to the
local law.
It is to be observed that the wife was not in military service at all. To decide the present case against the marriage on the
ground that the wife was not in military service would be to say that Taczanowska v Taczanowski was wrongly decided, which is
a course not open to us.
Secondly, Pilinski v Pilinska (Mr Commissioner Latey). The husband was a Polish soldier stationed in Germany. The wife
was a Polish civilian. The marriage was in a parish church, the priest being the regimental chaplain of the husband. The
argument that the marriage was void for non-compliance with the formalities required by the lex loci, which succeeded, was not
opposed. The point raised in Taczanowska v Taczanowski) was not canvassed at all. It may well be that the facts required a
decision in the contrary sense, in light of the later decision in Taczanowska v Taczanowski; on the other hand, it may be that there
were indications that the parties sought (but failed) to comply with the lex loci.
Thirdly, Kochanski v Kochanska (Sachs J). This involved a marriage on 10 June 1945. The husband was a Polish army
officer in 1939, was later captured by the Germans, escaped, and joined (or joined up with) the Polish Home Army, fought with
that army in the Warsaw rising, was again captured by the Germans, and confined as such in a prisoner of war camp in Germany.
From there he was liberated by the Russian advance in April, 1945, and went in May, 1945, to this same Nordheim camp, where
he was the officer commanding one of the blocks or battalions under the colonel camp commandant. He was without doubt at
all times an officer in the Polish armed forces, and clearly no dissolution or disbandment of the Polish Home Army in January,
1945, could have affected that status. The Polish wife was a nurse at the German prisoner of war camp hospital, and not a
member of any Polish military forces. She (as was Mrs Preston) was taken in at Nordheim Camp, and they were married by the
military chaplain of the camp who officiated at the Preston marriage. The marriageseven days later than the Preston marriage
was in the Nordheim village Roman Catholic parish church, which was then being used as the equivalent of a camp church,
and not in the camp itself. It would, I apprehend, be natural for the camp chaplain and the inmates to prefer, as soon as possible,
to use a consecrated church for services rather than a makeshift chapel in a camp block or an open air altar.
In that casewhere Sachs J held that the lex loci did not apply and the marriage was validthe only argument advanced
was for the husband. The argument was that the marriage was valid, not because it complied with the lex loci (which it clearly
did not), but because (a) the parties did not intend to subject themselves to the lex loci, and (b) their situation was such that they
should not be deemed to have so subjected themselves, and there was therefore a valid English common law marriage in respect
of which the wifes desertion justified a decree of divorce.
413
Sachs J proceeded on the footing that the general rule of the applicability of the lex loci is based on the presumption that the
parties have subjected themselves to that law, a presumption that is rebuttable. He took the view, on the evidence before him, that
(in contrast with Taczanowska v Taczanowski) the husband could not be said to be a member of an occupying military force, or of
any military force. The reason for the second view, and a fortiori for the first, apparently was that the colonel in command of the
camp was not a link in a normal chain of command with any recognisable military superior. It was for that reason that he referred
to the camp simply as a displaced persons camp. He then approached the problem on different lines. He considered that the
category of exceptions from the general rule of the applicability of the lex loci to marriage formalities was not limited to cases of
virtual impossibility of compliance (for one reason or another), and cases involving members of belligerently occupying armed
forces. He referred to circumstances of organised self-contained communities who might be held understandably to abstract
themselves from the regimen and laws of a hated conquered people on whose soil they found themselves, and of whom it would
(in effect) be therefore wrong to maintain the presumption or assumption that they subjected themselves or submitted to the lex
loci.
Fourthly, Lazarewicz (otherwise Fadanelli) v Lazarewicz (Phillimore J). This case was plainly rightly decided on the ground
that the parties did intend to subject themselves to the lex loci, and need not be further discussed.
Fifthly, Merker v Merker (Sir Jocelyn Simon P). Both parties were in military service with the Polish Armoured Division,
part of the allied armies of occupation in Germany. The marriage was in the local Roman Catholic parish church, the unit
chaplain officiating. They intended to marry, and the facts demonstrated that they did not intend to subject themselves to the
matrimonial lex loci. The case was therefore clearly covered by the decision in Taczanowska v Taczanowski. In refuting an
argument (which seems to have been unnecessarily advanced) that the decision in Taczanowska v Taczanowski gave carte blanche
to foreigners to elect whether to be bound by the lex loci, the President said that that decision was confined to marriages within
the lines of a foreign army of occupation (an enclave, so to speak, within which it was reasonable to hold that the local law has no
applicability), or, referring to Kochanski v Kochanska, of persons in a strictly analogous situation to the members of such army,
such as members of an organised body of escaped prisoners of war.
Sixthly, Starkowski v A-G. This merely recognised the general rule as to the lex loci stated by the Judicial Committee in
Berthiaume v Dastous ([1929] All ER Rep 111 at p 114; [1930] AC 79 at p 83). The circumstances of the marriage were such
that there were no conceivable grounds for departing from that rule, nor was it suggested.
Turning now to the facts in the present case, in my view the evidence establishes that the parties made no attempt to subject
themselves to the lex loci of marriage formalities; that the husband was at the time of the marriage a soldier in the Polish armed
forceshis rank being in fact that of cadet-corporal; that the camp was a military camp of which the purpose was the collection
of Polish soldiers hitherto at large in Germany, whether or not former prisoners of war, so as to organise them under military
command; that the camp was so established and organised under the auspices of the allied armed forcesboth the United States
Army in whose zone the camp was and the British Army under whose direction the camp came; that the purpose of the allies in
so doing (and indeed in providing rations for the camp) was not merely to avoid the inconvenience and suffering of roaming
bands of displaced persons, but was the military purpose of collecting and training military personnel for allocation to and
incorporation in ordinary service units in due course for purposes of consolidating conquest.
414
Now it is clear that, in those circumstances, the husband was not a member of belligerent occupying forces in the same way
as was the husband in Taczanowska v Taczanowski. He was not on the establishment of any normal military formation or staff.
There is no evidence that the inmates of the camp under the colonel commandant were regarded as responsible for the
maintenance of order in any part of Germany other than the camp itself. Leaving aside for the moment the decision in Kochanski
v Kochanska, do those differences justify or require a decision different from that of this court in Taczanowska v Taczanowski? I
do not think so. I take this court in that case to have based its decision mainly on the objections (pointed out in Ruding v Smith)
to the view that the members of the forces of a belligerent in a conquered territory must be subject to the local laws; and to have
decided that, while such persons may deliberately seek to marry according to the local law and thereby opt to be governed by it,
if they do not do so, their peculiar position excepts them from it. It is a matter of intention in the limited sense that they may opt
in. If, as I think, that is the principle behind the exception to the ordinary rule as applied in Taczanowska v Taczanowski, it must,
it seems to me, apply to members of those forces in that territory notwithstanding that they are not part of a normal fighting
formation or staff. Suppose in the same zone a camp for the collection of liberated United States army prisoners of war; or a
transit or pool camp for United States army personnel; or a United States army convalescent centre. In all such cases it would I
think have been correct to regard the inmates for present purposes as members of the United States forces in belligerent
occupation of the zone, which included Nordheim. Again, it would be right to have so regarded any members of the British army
who had contact with the Nordheim Camp in the course of its supervision, although the zone was the United States Army Zone.
All would be members of the conquering allied forces. Similarly, it seems to me proper, and within the principle, to include in
the exception these allied Polish soldiers who were organised in a military collecting, training or transit campcall it what you
willwhich was serving the overall allied purpose of conquest. It was argued that United States troops in the Nordheim area
were only to be excepted from the general rule if they were at the relevant moment members of a normal military formation or
staff with occupational duties. Not only does this seem to me to ignore the fact that the husband in Ruding v Smith was not such,
but further it introduces undesirable niceties of distinction between, for example, those in a convalescent camp who came there
from a strictly occupational unit or staff and those who did not. The true view I think is that all members of a group of allied
soldiers organised under military command for the military purposes of allies, one of whom is in actual control of the conquered
territory in which that group is situated, are within the exception to the general rule; they are in the privileged position of
members of a belligerent army occupying conquered territory so far as concerns the lex loci of marriage formalities.
In so far as the decision of Sachs J, in Kochanski v Kochanska is to be taken as deciding that the exception extends to any
displaced persons camp organised in arms length separation from the inhabitants of the conquered territory, I think that it goes
too far. (The position of the husband and the character of Nordheim Camp amply justify the result in that case, and I think that
the finding that it was not a military camp and that the inmates were not part of armed forces overlooked the universal principle
that soldiers gathered together as such are subject militarily to the commands and discipline of the senior officer among them, and
overlooked further the allied military purpose of the Nordheim camp, though on this latter point the evidence may have been
deficient).
I can well see that the arguments for exceptiong from the general rule, for example, the inmates of a displaced persons camp
(as distinct from military 415 personnel in a military camp) on a conquered territory of the former conquerors are persuasive,
when they organise themselves or are organised in a kind of apartheid. If the true ground of the general rule is the
inconvenience, disorder and conflict which will follow from departure from the matrimonial lex loci, then much of that goes
when there is found an orderly system within the group of marriages and marriage records, and inconvenience and disorder may
follow if such system is ignored in deference to the lex loci. If, on the other hand, the true ground of the general rule is the
recognition, as a matter of international comity, of the right of a state to lay down the formalities requisite for marriage within its
boundaries, then it may be argued that little heed may be paid to such comity in the case of a state so reduced by conquest that it
has no practical means of exerting authority or discipline over such a displaced persons camp as is under consideration. But, for
my part, I consider that encroachment on the general rule, by way of exception, is not to be justified except on some point of
principle. It is not a point of principle merely to say that in all the circumstances it would be reasonable to recognise a person as
entitled to ignore the lex loci of marriage formalities. The encroachment should not be extended beyond the military context.
While I have thought it right to express my views on Kochanski v Kochanska and its implications, the case of an ordinary
displaced persons camp is not, of course, before us.
Accordingly, in my judgment, Cairns J was correct in holding that the parties did not seek to subject themselves to the lex
loci of marriage formalities, and were not bound thereby, and that, accordingly, the marriage was not void for failure to comply
with that law. The marriage was, moreover, valid as what is referred to as an English common law marriage, the nature of which
is referred to by the President in Merker v Merker. I would only add this comment on what will be accepted by English law as
marriage. When considering any lex loci concerning the formalities required for marriage, it must be borne in mind that the
subject-matter is obstacles to marriage erected by that law. Once the lex loci is rejected there is no lex loci, and no formal
obstacles are erected. This, it seems to me, may well leave it open to a court in this country to recognise as marriage abroad that
which by the general law of Christendom was recognised as constituting the basic essence of the marriage contractthe contract
per verba de praesenti without further formalities. Here, in fact, the marriage was entered into in the presence of an officiating
episcopally ordained priest. I do not think myself that that was an essential to its recognition as a valid marriage (see on this
question Cheshires Private International Law, 6th Edn, pp 342, 343); but this point did not arise for decision or argument.
I agree that the appeal fails.

Appeal dismissed. Leave to appeal to the House of Lords refused.

Solicitors: Cooper, Bake, Fettes, Roche & Wade (for the husband); Martin, Nicholson, Hortin & Nash (for the wife).

Henry Summerfield Esq Barrister.


416
[1963] 2 All ER 417

Re An Agreement Between the Members of the British Paper and Board


Makers Association (Incorporated)
COMPETITION

RESTRICTIVE PRACTICES COURT


MEGAW, P, SIR STANFORD COOPER, MR W L HEYWOOD AND MR D V HOUSE
19, 20, 21, 22, 25, 26, 27, 28, 29 MARCH, 1, 2, 29 APRIL 1963

Restrictive Trade Practices Reference Waste paper Purchase from local authorities Recommended minimum price
guarantee Whether removal of restriction would deny public substantial benefit Restrictive Trade Practices Act, 1956 (4 & 5
Eliz 2 c 68), s 21(1) (b).

The British Paper and Board Makers Association was composed of a large number of manufacturers in the United Kingdom, of
paper or board, in which waste paper was an important raw material. If waste paper were not available, the continuance of
manufacture would involve the use of straw or wood pulp, the latter having to be imported and being substantially more
expensive than waste paper. It was economically difficult, if not impossible, for mills to stock large quantities of waste paper,
and a shortage involving the use of an alternative raw material would, therefore, result in higher manufacturing costs which might
have to be passed on to the public. The import of paper and board from abroad would also lead to higher prices, and the import
of waste paper was impracticable. Manufacturers bought their waste paper principally either from waste paper merchants or from
local authorities, many of whom separated suitable types of waste paper from other refuse, and sold it as baled mixed waste
paper, or Grade 39, which was one of the grades classified in an agreement between the Association and the British Waste
Paper Association. Contracts between manufacturers and local authorities varied considerably; in some cases the manufacturers
undertook to buy all the authoritys waste paper of stated grades for a specified period of years, in others, the manufacturer was
obliged to take it for a period of five years, with a proviso enabling the manufacturer to limit the quantity at any time on three
months notice, so that, if the limitation were not to be agreed, the contract would be terminated; and a number of local authorities
had no contract at all. In general, contracts provided that the price to be paid would be the current market price. For many years
the Association had recommended its members to guarantee to local authorities minimum prices which were agreed annually by a
committee of the Association. It appeared that all members did in fact give at least the recommended guarantee, though certain of
them gave a more extensive guarantee.
On a reference by the Registrar of Restrictive Trading Agreements under s 20(1) and s 20(2)(a) of the Restrictive Trade
Practices Act, 1956a, the Association sought to justify the restriction as to the guaranteed 417 minimum price under s 21(1)(b)b
of the Act. They contended that, because of past experience, local authorities, who were reluctant to start or continue the work of
waste paper salvage unless they were reasonably assured that it would not involve a loss to the ratepayers, were apprehensive that
a fall in the price of waste paper might make their salvage operations uneconomic; that they regarded the guaranteed minimum
price as a protection against this; that without the guaranteed minimum price many local authorities would be likely to abandon
waste paper salvage which would lead to a shortage of waste paper which would necessitate the use by manufacturers of much
more expensive, imported, raw materials or at least would lead to a substantial increase in the price of waste paper, so that the
price of the end-products to the public would be increased.
________________________________________
a Section 20, so far as relevant, provides: (1) The court shall have jurisdiction, on application made in accordance with this section in
respect of any agreement of which particulars are for the time being registered under this Part of this Act, to declare whether or not any
restrictions by virtue of which this Part of this Act applies to the agreement (other than restrictions in respect of matters described in paras
(b) to (d) of s 8(8) of this Act) are contrary to the public interest.
(2) An application to the court under the foregoing subsection may be made(a) in any case by the Registrar
(3) Where any restrictions are found by the court to be contrary to the public interest, the agreement shall be void in respect of these restrictions;
and the court may make such orders as appears to the court to be proper for restraining all or any of the persons party to the
agreement (a) from giving effect to the agreement in respect of these restrictions:
b Section 21(1), so far as relevant, provides: For the purposes of any proceedings before the court under the last foregoing section, a
restriction accepted in pursuance of any agreement shall be deemed to be contrary to the public interest unless the court is satisfied of any
one or more of the following circumstances, that is to say (b) that the removal of the restriction would deny to the public as
purchasers, consumers or users of any goods other specific and substantial benefits or advantages enjoyed or likely to be ejoyed by them as
such, whether by virtue of the restriction itself or of any arrangements or operations resulting therefrom

In a letter dated 16 January 1959, enclosing a revised waste paper schedule of prices (which included a description of Grade
39), circulated to its members of the Association, the Association requested members in the interests of the industry as a whole
to strictly adhere to the descriptions of standardised grades and the maximum and minimum prices contained in the schedule.
The Association did not on this reference deny that the schedule gave rise to restrictions under the Act of 1956, but contended
that the letter was irrelevant on this reference on the ground that the restrictions arose out of an agreement between the
Association and the British Waste Paper Association which was the subject-matter of another reference c and did not affect
dealings between members and local authorities, apart from the identification of Grade 39.
________________________________________
c See Re An Agreement between the British Waste Paper Association and the British Paper and Board Makers Association (Incorporated) p
424, post

Held (i) The restriction as to the guaranteed minimum price was deemed by s 21(1) of the Act of 1956 to be, and was declared
to be, contrary to the public interest, the Association having failed to satisfy the court of the circumstances stated in s 21(1)(b),
because the court was satisfied that all the larger purchasers of waste paper from local authorities would, even if the
recommendation of a guaranteed minimum price were removed, in some way ensure that the great majority of local authorities
would continue to salvage waste paper (see p 422, letter e, and p 423, letter a, post), and that even if in a period of falling prices
the smaller purchasers so acted as to cause the local authorities to cease to salvage waste paper, that would not lead to any
appreciable shortage of waste paper or any increase in the price of the end-product, and that it was unlikely that more than a few
of those smaller purchasers would act in such a way as to cut off, more or less irrevocably, a source of supply of waste paper on
which they would be likely to desire to draw in the future when demand revived (see p 423, letter a, post).
(ii) It was unnecessary for the court to deal in this reference with the restrictions arising on the revised schedule circulated in
January, 1959, because it did not contain any recommendations to members of the Association as to their dealings with local
authorities, and the alleged restrictions contained in it were the subject-matter of another reference to the court (see p 424, letters
b and c, post).

Notes
As to the presumption of a restrictive trade agreement being contrary to the public interest, and as to grounds justifying
restrictions, see 38 Halsburys Laws (3rd Edn) 113, para 149, 115, para 151, 118, para 155 and 119, para 156.
418
For the Restrictive Trade Practices Act, 1956, s 21, see 36 Halsburys Statutes (2nd Edn) 954.

Reference
Pursuant to the Restrictive Trade Practices Act, 1956, s 20(2)(a), the Registrar of Restrictive Trading Agreements referred to the
Restrictive Practices Court an agreement between the members of the British Paper and Board Makers Association
(Incorporated), herein referred to as the Association. The facts and the restrictions are set out in the judgment of the court.

J L Arnold QC and R O C Stable for the British Paper and Board Makers Association (Incorporated).
R H W Dunn QC and R A Barr for the Registrar.

Cur adv vult

29 April 1963. The following judgment was delivered.

MEGAW P read the following judgment of the court. The essential facts in this reference can be stated shortly and simply. The
Association has as members a large number of companies engaged in the United Kingdom in the manufacture of paper or of
board or of both. Waste paper is an important raw material in the manufacture of many kinds of board and, to a lesser degree, of
certain kinds of paper. The percentage of waste paper which is used in the manufacture of these finished products varies with the
particular product. In the manufacture of board, the percentage is high. If waste paper were not available, the continuance of
manufacture would involve the use of some other raw material. The only possible substitutes would be wood pulp or straw.
Wood pulp has to be imported. Its cost, delivered at the mills in the United Kingdom, is much greater than the cost of waste
paper at the present time, and the same sort of ratio has applied in the past. The present market price, delivered in London, of
baled mixed waste paper is in the region of 9 a ton; whereas mechanised wood pulp costs nearly 30 a ton, and unbleached kraft
pulp costs about 43 a ton. A combination of these two would be required if wood pulp were to be substituted for waste paper.
For a number of reasons, it is economically difficult, if not impossible, for the mills to stock large quantities of waste paper
against the possibility of periods of scarcity of supply. Accordingly, if for any reason it were necessary for the board or paper
mills to use alternative raw material, the cost of manufacturing the end-products of paper and board would be higher; indeed, as
an extreme case, manufacture in this country might cease to be economically possible. A part, at least, of any additional cost
might have to be passed on to the purchasers of paper and board in the form of higher prices. The import of paper and board
from abroad would also mean higher prices. The import of waste paper from abroad is not practicable.
The mills buy their wate paper principally from two sources. The first source is waste paper merchants; the second is local
authorities. Many local authorities, in their capacity as refuse collectors, arrange to separate the suitable types of waste paper
from other refuse, in some instances to sort it into different grades, to compress into bales the grade which is described as mixed
waste paper and to sell it, either directly or through merchants, to mills which require it as raw material. The type of waste
paper with which this reference is primarily concerned is that which is described as baled mixed waste paper. It is also
described as Grade 39, being one of the grades classified in an agreement between the Association and an association of waste
paper merchants, which is now known as the British Waste Paper Association. The other classes of waste paper which are sold
separately by some local authorities consist of newspapers, books and magazines, and cardboard cartons. For these classes,
ordinarily, a higher price is paid than for baled mixed waste paper.
Accurate figures are not available. The figures which we shall give cannot be taken as more than approximate. In 1961 the
total quantity of baled mixed waste paper consumed by board and paper mills which are members of the Association was of the
order of 647,849 tons. Of this quantity, about 434,519 tons were purchased from merchants, out of which about eighty thousand
tons 419 had been purchased by the merchants from local authorities. About 202,133 tons were purchased direct from local
authorities by mills, or by merchants who are wholly owned subsidiaries of the respective mills. All these figures are subject to
the qualification that the totals under each head should probably be a little greater, because the figures are taken from the answers
to a questionnaire sent out on behalf of the Association, to which a small, but unspecified number of members did not see fit to
reply. Some further figures disclosed by the table prepared from the answers to this questionnaire should be given, in respect of
purchases by mills or their subsidiaries direct from local authorities. By far the largest purchaser is Thames Board Mills Ltd with
a total of 120,880 tons. Next comes C Davidson Ltd with 26,020 tons. The Reed Paper Group took 9,880 tons, New Merton
Board Mills Ltd 9,358 tons, and St Annes Board Mill Ltd 4,697 tons. These quantities are mainly, if not entirely, used in board
manufacture. These purchases direct from local authorities thus come to 170,835 tons out of the total direct purchases from local
authorities by all mills, paper or board, of 202,133 tons. Radcliffe Paper Mill, which may, perhaps, be assumed to be mainly
concerned with paper manufacture, bought twenty thousand tons from local authorities. That leaves only 11,298 tons, divided
among thirty-four other mills. Only four of these showed, in their returns, purchases direct from local authorities of over one
thousand tons each. Only ten out of the remaining thirty showed any purchases at all direct from local authorities. As regards
many of these fourteen, the direct purchases from local authorities were only a small proportion of their purchases from
merchants. We were told that many of the smaller mills were paper manufacturers, but no details were vouchsafed in evidence.
In the upshot, and having regard to the figures which we have mentioned, we do not think that it matters.
The arrangements between the various mills and the various local authorities for the purchase of the latters offerings of
waste paper vary greatly. We have seen in evidence a number of contracts between local authorities and mills or between local
authorities and merchants who are subsidiary companies of mills. In some instances, the mills undertake to buy all the waste
paper of stated grades which the local authority offers over a period of years. In other casesand this applies to the contracts of
Thames Board Mills, Ltdthe contract is expressed as obliging the mill to take all the local authoritys offering for a period of
five years, but with a proviso which enables the mill to limit the quantity at any time on three months notice, so that, if the
limitation were not to be agreed, the contract would be terminated. A number of the local authorities, it is said, have no contract
at all. Presumably this means that they have no formal or written contract with the mills which take their supplies.
As regards the price to be paid for waste paper, the contractual stipulations which we have seen, again, vary. Broadly, the
price to be paid is the current market price. It is here that the restriction with which we are primarily concerned comes in. For
many years past the Association has recommended to its members, the mills, that, in their purchases of baled mixed waste paper
from local authorities, they should guarantee to the local authority a minimum price. The minimum price to be recommended is
agreed by the waste paper committee of the Association in or about October or November of each year, and it is to be applicable
for the local authorities next following financial year; that is from the following 1 April to 31 March thereafter. Since
November, 1950, the minimum price guaranteed has remained at 6 10s per ton. It is for the individual mills to offer the
guaranteed minimum price. Normally they notify it individually to the local authorities with which they respectively deal in or
about November or December. So far as we know, all members of the Association in fact give at least the recommended
guarantee. Certain mills are prepared to give a more extensive guarantee. Thus, in a contract made in 1960 between a large local
authority and a mill, the contract is for five years; it obliges the mill to take all the waste paper salvaged; the price is to be the
market price with a guaranteed 420 minimum of 8 per ton for the first three months of the contract and thereafter at 6 10s for
the whole of the remainder of the contract. It is conceded by the Association that the recommendation of the offer of a fixed
minimum price is a restriction within the terms of the Restrictive Trade Practices Act, 1956, by virtue of s 6(7) of the Act d.
________________________________________
d Section 6(7) is set out at p 428, letter i, post

The Association seeks to support the restriction under s 21(1)(b) of the Act. The registrar denies that it can be so supported,
and also disputes the Associations contention that, if the case under s 21(1)(b) should be established, the restriction is not, on the
balance of public interest, unreasonable. In order to satisfy the requirements of s 21(1)(b), the Association has to show that the
removal of the restriction would deny a specific and substantial benefit or advantage to the public as purchasers, consumers or
users of goods. The public, and the only public, here in question is the public who buy the end-products manufactured out of
waste paper; that is, those types of paper and board in the manufacture of which waste paper supplied by local authorities is an
ingredient. The specific and substantial benefit or advantage which it is claimed would be lost is the benefit or advantage to the
public of being able to obtain these end-products at a cheaper price, if the restriction were allowed to remain, than if the
restriction were to be condemned.
The Associations contentions can be summarised as follows: (i) Local authorities are reluctant to start, or continue, the
work of waste paper salvage unless they are reasonably assured that it will not involve a loss to the ratepayers. (ii) Because of
past experience and for other reasons local authorities are very apprehensive of the possibility of a fall in the price of waste paper
which may occur suddenly and may make their salvage operations uneconomic. (iii) Local authorities regard the guaranteed
minimum price, which has been repeated year after year, as a real and valuable protection against the danger of their salvage
operations becoming seriously uneconomic. (iv) If the local authorities were not provided with a guaranteed minimum price,
they would be likely, in many instances, to abandon waste paper salvage, particularly if there was a period of surplus supply of
waste paper and falling prices. Considerable capital expenditure is involved in the provision of plant and buildings needed for
waste paper salvage. (v) The abandonment of waste paper salvage by a substantial number of local authorities would be likely to
result, within a comparatively short period, in a shortage of waste paper, and, in foreseeable future conditions of a general rise in
demand, the insufficiency would be a serious matter. (vi) The impact of such shortage would not be avoided, or substantially
lessened by a resumption of waste paper salvage by local authorities which had previously abandoned their salvage operations.
This is principally because of the difficulty of persuading householders to co-operate in the separation of waste paper from other
refuse, especially on a stop and start basis, and the unwillingness of local authorities to incur capital expenditure on an enterprise
of this nature, if they regard it as hazardous. (vii) Such a shortage of waste paper would necessitate the use by mills, in whole or
in part, of much more expensive, imported, raw materials, or, at least, would increase substantially the price of waste paper
because of the shortage; so that the price of the end-products, paper and board, would be increased by reason of increased costs of
manufacture. We do not find it necessary to review in any detail the elaborate evidence which has been submitted, which
included a lengthy historical review of the waste paper market and of the activities of the Association from the era of war-time
controls onwards and of the Waste Paper Recovery Association Ltd, a wholly owned subsidiary of the Association, which has
been responsible for the conduct of propaganda directed towards the local authorities in connexion with the supply of waste
paper. The fact that we refrain from reviewing the evidence in detail should not be regarded as indicating any disrespect for the
witnesses or their evidence. We refrain because we are satisfied that the Associations contentions fail on one short issue which is
an integral and vital part of their case.
421
Assuming that everything else in the Associations case as outlined above is correct, nevertheless the attempt to uphold the
recommendation of a guaranteed minimum price under s 21(1)(b) must fail unless it is established that the enforced abandonment
of the recommendation would be likely to lead to a substantial reduction in the quantity of waste paper supplied by local
authorities. It is necessary for us, for that purpose, to consider what would probably be done by the mills, as manufacturers of
board and paper, if it were no longer lawful for the Association to recommend to its members a guaranteed minimum price for
purchases from local authorities. On that matter, we have had the advantage of the evidence of representatives of three
manufacturers, members of the Association, called on behalf of the Association. Colonel Davidson, managing director of C
Davidson & Sons Ltd of Aberdeen, made it clear that his company, or its wholly owned subsidiary, which as a merchant buys
from local authorities, would itself give to the local authorities such guarantee as would be necessary to make them content to
continue their waste paper salvage. The evidence of Mr Bremmer, a representative of the New Merton Board Mills Ltd was to
the same effect. Mr Curtis, a director of Thames Board Mills Ltd felt unable to say what his company would do. He said that it
was a matter of company policy which had not been considered. He made it clear, however, that his company attached vital
importance to the continuation of supplies by local authorities. He said:

We would have to establish our future conduct with the local authorities on what we saw as a result of the removal of
the restriction. There might be many ways in which it could be handled, but I think we would have to experiment and deal
with the situation as we saw it at the time.

We regard it as likely that Thames Board Mills Ltd whether by offering a guarantee of a minimum price or in some other way
which would satisfy the local authorities from whom they take their supplies, could and would ensure that at any rate the great
majority of these local authorities would continue to salvage waste paper. We were not impressed by the points submitted by
counsel for the Association in his final speech in support of the contention that Thames Board Mills Ltd might be expected to act
in a different way from that which we have mentioned. We are satisfied that all the large purchasers from local authorities,
whether they be board or paper mills, would follow the same course. As we have already said, the six largest mills took, in 1961,
about 190,000 tons out of 202,000 tons purchased direct from local authorities.
What, then, of the fourteen other mills which made purchases direct from local authorities in 1961? No representative of
any of these mills was called to give evidence. The submissions with regard to them are founded on the evidence of Sir Herbert
Hutchinson, who was director general of the Association from October, 1951, till September, 1960. His view was that, whatever
the larger mills might do in the way of giving individual price guarantees to local authorities, there would be a serious possibility
that, in certain circumstances, a substantial number of the smaller millswhich it is said are mostly paper millswould not
follow that course, preferring their short-term interest to the long-term interest of the industry to keep alive waste paper salvage
by the local authorities. The circumstances which Sir Herbert visualised were a period of falling prices for waste paper in a time
of surplus and a prospect of indefinite continuance of that state of affairs. If the Associations recommendation disappeared, said
Sir Herbert, he feared that the small mill owner, suspecting that his competitors were not offering a minimum price guarantee,
and were, therefore, getting, or might get, waste paper cheaper, would himself refuse to give any guarantee of a minimum price
satisfactory to the local authorities from which he drew supplies; so that those local authorities would abandon their waste paper
salvage. From the figures we have given as to the quantity of the sales by local authorities to these fourteen mills (about eleven
thousand tons out of a total consumption in 1961 of about 648,000 tons of this type of waste paper), and from a comparison 422
of the amounts of the purchases of these mills from merchants and local authorities respectively, we have come to the conclusion,
first, that even if the circumstances visualised by Sir Herbert Hutchinson were to come about, and even if every one of these mills
so acted as to cause its supplying local authorities to cease to salvage waste paper, it would not lead to any appreciable shortage
of waste paper, or any increase in the price of the end-product. Secondly, we do not think it likely that, at most, more than a few
of the fourteen mills would act in such a way as to cut off, more or less irrevocably, a source of supply of waste paper on which
they would be likely to desire to draw in the future when demand revived.
Accordingly, the Association fails to establish an essential element in its case under s 21(1)(b) of the Act. It is unnecessary
for us to express a view on the other issues which would have arisen.
[His Lordship rejected an invitation by counsel for the Association to reject the evidence of an economist on behalf of the
registrar on the ground that he was untruthful, and continued:] We have dealt so far with the principal restriction, the
recommendation of a guaranteed minimum price by members of the Association to local authorities who sell baled mixed waste
paper to those members. The registrar contends that the agreement between the members of the Association contains a number of
other restrictions, set out in detail in the Schedule to the registrars amended answer. The registrars contention is founded on the
fact that, in a circular letter to all members, dated 16 January 1959, there was enclosed a revised waste paper Schedule of prices
setting out thirty-nine grades of waste paper, some of the thirty-nine grades containing sub-divisions, with maximum and
minimum prices shown for each grade or sub-division, and with the heading All free from waxed paper or waxed board and all
contraries not suitable for cold re-pulping processes, and with the footnote: The above prices are ex merchants warehouse.
No loading or other charges are allowed. The letter of 16 January 1959, concludes with a paragraph saying:

Members are requested, in the interests of the industry as a whole, to strictly adhere to the descriptions of standardised
grades and the maximum and minimum prices contained in the Schedule.

The maximum and minimum prices and the stipulation as to prices being ex warehouse were abandoned in July, 1962.
The registrar says that six separate restrictions are involved, apart from the minimum price guarantee to local authorities.
The Association has not, in this reference, sought to contend that these are not restrictions within the Act or that they are
defensible under any of the provisions of s 21(1) of the Act. It contends, however, that they are not restrictions which are relevant
in this reference, because they do not in any way affect the dealings between members of the Association and local authorities,
apart from the mere identification of the meaning of Grade 39, from the Schedule of descriptions, in relation to the minimum
price guarantee to local authorities for that grade. It contends that these other restrictions, in so far as they are restrictions under
the Act, arise out of an agreement between the Association and the British Waste Paper Association (an association of waste paper
merchants) which is the subject-matter of a separate reference by the registrar e. It is contended for the Association that the letter
of 16 January 1959, in the surrounding circumstances, should be construed as enjoining members to observe the descriptions of
standardised grades and the prices in the Schedule which were relevant only in respect of their dealings with merchants; and that
these recommendations, arising out of an agreement which is not in issue here, but is in issue in another reference, should not be
considered here, even though they happen to appear in a document which is registered amongst the documents giving rise to this
reference.
________________________________________
e See Re An Agreement between the British Waste Paper Association and the British Paper and Board Makers Association (Incorporated) , p
424, post

The only matters which have given us some cause for doubt with regard to 423 that submission are, first, that the minimum
price for grade 39 in the Schedule of descriptions was the same as the guaranteed minimum price to local authorities, namely, 6
10s; and, secondly, that in October, 1954, and in March, 1955, some members of the Association seem to have thought that both
the minimum and maximum prices in the schedule then agreed with the Merchants Association applied also to sales by local
authorities. We are, however, assured that this was a misunderstanding by members. We feel able to accept that assurance.
Accordingly, we agree with the submission for the Association on this matter and hold that, despite the wording of the letter of 16
January 1959, it is not necessary for us to deal in this reference with alleged restrictions which are, in reality and effect, the
subject-matter of another reference before the court. We treat this reference as being confined to recommendations to members
of the Association as to their dealings with local authorities, and we hold that the letter of 16 January 1959, does not, on its true
interpretation, contain any recommendation to members of the Association as to their dealings with local authorities, but merely
identifies the subject-matter of the recommended minimum price guarantee as being grade 39 in the Schedule of descriptions. It
was for that purpose alone that the letter was included among the registered documents in this reference.
For the reasons which we have given, we hold that the only restriction with which we are properly concerned in this
referencenamely, the recommendation to members to offer a minimum price guarantee to local authorities for the purchase of
baled mixed waste paperis contrary to the public interest.

Order accordingly.

Solicitors: Slaughter & May (for the British Paper and Board Makers Association (Incorporated)); Treasury Solicitor.

Mary Colton Barrister.


[1963] 2 All ER 424

Re An Agreement between the British Waste Paper Association and the


British Paper and Board Makers Association (Incorporated)
CONTRACT: CONSUMER; Consumer credit

RESTRICTIVE PRACTICES COURT


MEGAW, P, SIR STANFORD COOPER, MR W L HEYWOOD AND MR D V HOUSE
3, 4, 5, 29 APRIL 1963

Restrictive Trade Practices Reference Waste paper suppliers Schedule of standard descriptions Standard descriptions not
to preclude special arrangements for any special quality or description of waste paper Descriptions Waste paper to be free
from waxed paper or waxed board and from all unsuitable contraries except by special arrangement Whether restrictions
Whether removal of restrictions would deny public substantial benefit Restrictive Trade Practices Act, 1956 (4 & 5 Eliz 2 c 68),
s 6(1)(b)(c), s 21(1) (b).

Waste paper was an important raw material in the manufacture of board and some types of paper. The British Waste Paper
Association, previously known as the Waste Paper Merchants Association, consisted of about 104 waste paper merchants
supplying roughly sixty per cent of waste paper used by paper mills. The British Paper and Board Makers Association included
in its membership the great majority of paper manufacturers. Waste paper required careful selection and grading. By an
amended agreement between the two Associations provision was made for their members conforming with resolutions of a joint
committee, (a) that standard descriptions of the various grades of waste paper are accepted in accordance with the schedule of
standard descriptions hereto annexed but such standard descriptions shall not preclude any special arrangement made or sought to
be made between a member of either of the Associations for any special quality or description of waste paper, and (b) that
except by special agreement, all waste paper of every grade shall be free from waxed paper or waxed board 424 and from all
contraries not suitable for cold repulping processes. Contraries were undesirable things, such as dirt, ink bottles and other
extraneous matter and paper treated with certain chemicals.
On a reference by the Registrar of Restrictive Trading Agreements under s 20(1) and s 20(2)(a) of the Restrictive Trade
Practices Act, 1956a, the Associations denied that provisions (a) and (b), above, gave rise to restrictions within s 6(1) of the Act b;
in the alternative, if they did give rise to restrictions within s 6(1) the Associations sought to justify them under s 21(1)(b) b or, if
only provision (a) was so justifiable, they sought to justify provision (b) under s 21(1)(g) c.
________________________________________
a Section 20, so far as relevant, provides: (1) The court shall have jurisdiction, on application made in accordance with this section in
respect of any agreement of which particulars are for the time being registered under this Part of this Act, to declare whether or not any
restrictions by virtue of which this Part of this Act applies to the agreement (other than restrictions in respect of matters described in paras
(b) to (d) of s 8(8) of this Act) are contrary to the public interest.
(2) An application to the court under the foregoing subsection may be made(a) in any case, by the Registrar
(3) Where any such restrictions are found by the court to be contrary to the public interest, the agreement shall be void in respect of these
restrictions; and the court may make such orders as appears to the court to be proper for restraining all or any of the persons party to
the agreement (a) from giving effect to the agreement in respect of those restrictions;
b Section 6(1), so far as relevant, provides: Subject to the provisions of the two next following sections, this Part of this Act applies to any
agreement between two or more persons carrying on business within the United Kingdom in the production or supply of goods, or in the
application to goods of any process of manufacture, whether with or without other parties, being an agreement under which restrictions are
accepted by two or more parties in respect of the following matters, that is to say: (b) the terms or conditions on or subject to which
goods are to be supplied or acquired or any such process is to be applied to goods; (c) the quantities or descriptions of goods to be produced,
supplied or acquired;
c Section 21(1), so far as relevant, provides: For the purposes of any proceedings before the court under the last foregoing section, a
restriction accepted in pursuance of any agreement shall be deemed to be contrary to the public interest unless the court is satisfied of any
one or more of the following circumstances, that is to say (b) that the removal of the restriction would deny to the public as
purchasers, consumers or users of any goods other specific and substantial benefits or advantages enjoyed or likely to be enjoyed by them as
such, whether by virtue of the restriction itself or of any arrangements or operations resulting therefrom; (g) that the restriction is
reasonably required for purposes connected with the maintenance of any other restriction accepted by the parties, whether under the same
agreement or under any other agreement between them, being a restriction which is found by the court not to be contrary to the public
interest upon grounds other than those specified in this paragraph

Held (i) Provision (a) of the agreement did not give rise to a restriction within s 6(1)(c) of the Act, since descriptions in s 6(1)
(c) referred to kinds of goods, not to nomenclature of goods, and provision (a) of the agreement did not restrict the kinds of
goods to be supplied or acquired (see p 429, letter i, to p 430, letter b, post); nor was it a restriction within s 6(1)(b) of the Act,
since there was nothing restrictive in agreeing in advance that, if a particular code should be used to describe types of goods, the
meaning set out in the code should apply, so long as it was apparent that the persons whom the agreement bound were not in any
way, expressly or by implication, deterred from entering into transactions which fell outside the scope of the code (see p 430,
letter d, post);
(ii) Provision (b) of the agreement did not give rise to a restriction within s 6(1), because there was nothing to prevent any
member of the Association from agreeing to sell to, or buy from, another member of either Association any waxed paper or
waxed board and it was left entirely open to any member to sell or buy waste paper containing contraries, if for any reason he
should see fit to do so (see p 430, letter f, post), and because provision (b) probably did not go further than the general law, it
probably being a breach of contract, by virtue of s 14 of the Sale of Goods Act, 1893, to deliver waste paper 425 containing
contraries to a manufacturer unless the contract specially so provided (see p 430, letter g, post).
(iii) Assuming, however, that provisions (a) and (b) of the agreement were restrictions within the Act of 1956, and that they
had the meanings indicated in (i) and (ii) ante, they would not have been justified under s 21(1)(b) of the Act since no specific
and substantial benefit or advantage would be lost by the public (the persons who purchased, consumed or used paper or board in
the manufacture of which waste paper was used as a raw material) by the abandonment of the restrictions, because, even if the
abandonment of the restrictions would be likely to lead to some unquantified increase in the costs of manufacture of board or
paper, there was no evidence to justify the conclusion that such an increase in costs would be likely to lead to an increase in the
price of the products or to the cessation of manufacture, or to enable the court to determine whether such increase, if any, in price
would be trivial or greater than trivial (see p 431, letter f, post).

Notes
As to the meaning of restriction, see 38 Halsburys Laws (3rd Edn) 99, para 127, and as to the presumption of a restrictive trade
agreement being contrary to the public interest, and as to grounds justifying restrictions, see ibid, 113, para 149; 115, para 151;
118, para 155 and 119, para 156.
For the Restrictive Trade Practices Act, 1956, s 6 and s 21, see 36 Halsburys Statutes (2nd Edn) 938, 954.

Reference
Pursuant to the Restrictive Trade Practices Act, 1956, s 20(2)(a), the Registrar of Restrictive Trading Agreements referred to the
Restrictive Practices Court an agreement between the British Waste Paper Association and the British Paper and Board Makers
Association (Incorporated). The facts and the restrictions are set out in the judgment of the court.

R I Threlfall and A P Graham-Dixon for the British Waste Paper Association.


J L Arnold QC and R O C Stable for the British Paper and Board Makers Association (Incorporated).
R H W Dunn QC and R A Barr for the Registrar.

Cur adv vult

29 April 1963. The following judgments were delivered.

MEGAW P read the following judgment of the court. The British Waste Paper Association, which at the date of the reference
and up to 5 December 1961, was known as the Waste Paper Merchants Association, is an association comprising in its
membership a large number of companies, firms, and possibly individuals, carrying on business in the supply of waste paper to
mills which manufacture paper and board. Waste paper is an important raw material in the manufacture of board and some types
of paper. There are about 104 members of the Association, including the majority of the large waste paper merchants and many
of the small merchants. Some of the members are subsidiary companies of mill-owning companies. No accurate figures were
made available to the court of the percentage of waste paper supplied to mills by merchants who are members of the Association.
A witness, Mr R V Hough, who has great experience in the trade, estimated it at roughly sixty per cent. We accept this as a
reasonable estimate. The British Paper and Board Makers Association includes in its membership at least the great majority of
the companies or firms which manufacture paper or board in their mills. About 1937, at the instance of the Board of Trade, steps
were taken to grade various types of waste paper into categories in order to facilitate the control which might be necessary in
wartime. During the war, price and other controls were in operation. After the end of government control, the two Associations
were desirous that an effort should be made to secure by agreement a system for suitable and adequate sorting of waste paper to
meet the requirements of the manufacturers of paper and board.
There are many different types of waste paper with many different physical and chemical characteristics. Some of these
types are unsuitable as components of any 426 kind of paper or board, and their use, by inadvertence or ignorance, may cause
serious damage to the finished product. Many other types of waste paper may be suitable for manufacture of some kinds of paper
or board, but unsuitable for other kinds. Careful selection and grading is necessary. The manufacturer must not only know what
types he requires for the purpose of particular manufacture; he must also be sure that the merchant from whom he buys knows
what it is that is being asked for, and supplies that specification and not something different. Moreover, there is the danger that
waste paper, though itself apparently of a suitable type, may contain what are called contraries. These may consist of things
which are obviously undesirable, such as dirt, ink bottles, pieces of string, or other extraneous matter. They may also include
what are technically called pernicious contraries; for example, paper treated with certain chemicals, which may not be easily
identifiable, which may either invariably be harmful to the finished product or be harmful if their presence is not known so that
special treatment can be applied in their use. The difficulties caused by pernicious contraries are likely to increase as technical
developments in the manufacture of paper and board continue, since new processes of manufacture, or new kinds of paper and
board, may result in more material coming back into the waste paper market, with dangers, which may not be generally realised,
affecting their use as a raw material for the fresh manufacture of paper and board. The two Associations and their members are
aware of these dangers, and research is being carried on, particularly on the subject of pernicious contraries. It is apparent that it
is important, from the point of view of the manufacturers of paper and board, that merchants selling waste paper to them should
use care in selection and grading, and that, when a manufacturer negotiates for the supply of waste paper or orders waste paper
from a merchant, everything that is reasonably possible should be done towards ensuring that errors or misunderstandings do not
occur and that the merchant will know what the manufacturer is asking for, and will not supply something different which, if used
in the manufacture, may cause defects in the end products. Moreover, a widespread knowledge of the types of waste paper likely
to be required serves a useful purpose in enabling merchants to acquire stocks and to sort and classify them in a way which is
most likely to enable them to meet the manufacturers needs promptly and satisfactorily.
These factors were no doubt in the minds of the two Associations when, on 11 January 1950, a Schedule of Descriptions
of waste paper was agreed between them. That schedule contained thirty-nine items, each with a number, a name and a more
detailed description. Thus, item 1 was Cream Shavings, and its description was

best white and cream, writing shavings, envelope cuttings and ivory card cuttings unprinted, free from coated paper
and free from mechanical.

(Mechanical means mechanical wood pulp). A few months later maximum prices were agreed between the Associations for
certain of the items in the schedule of descriptions. In 1957, all the items in the schedule of descriptions, which by that time had
grown to forty-seven as a result of new items or sub-divisions, became the subject-matter of maximum and minimum prices. On
12 August 1957, an agreement was made between the two Associations which stipulated as follows:

(1) The Waste Paper Merchants Association agrees that its members should conform to the findings of the joint
committee set out in Sch. 2 hereto and any amendments thereof from time to time effected between the Associations, and
the British Paper and Board Makers Association agrees to recommend its members to comply with the findings of the said
joint committee.

Thus, if any of the provisions of Sch 2 were restrictions within s 6(1) of the Restrictive Trade Practices Act, 1956, s 6(6) applied
as regards the members of the Waste Paper Manufacturers Association and s 6(7) as regards members of 427 the other
associationd. Schedule 2 provided as follows:
________________________________________
d Section 6(6) and s 6(7) provide: (6) This Part of this Act shall apply in relation to any agreement made by a trade association as if the
agreement were made between all persons who are members of the association or are represented thereon by such members and, where any
restriction is accepted thereunder on the part of the association, as if the like restriction were accepted by each of those persons.
(7) Where specific recommendations (whether express or implied) are made by or on behalf of a trade association to its members or to any class
of its members, as to the action to be taken by them in relation to any particular class of goods or process of manufacture in respect of any
matter described in [s. 6(1)], this Part of this Act shall apply in relation to the agreement for the constitution of the association
notwithstanding any provision to the contrary therein, as if it contained a term by which each such member, and any person represented on
the association by any such member, agreed to comply with those recommendations and any subsequent recommendations made to them by
or on behalf of the association as to the action to be taken by them in relation to the same class of goods or process of manufacture and in
respect of the same matter.

Resolutions of the joint committee at the date hereof which have been authorised or adopted by the Associations: (1)
Standard descriptions of the various grades of waste paper are accepted in accordance with the schedule of standard
descriptions hereto annexed but such standard descriptions shall not preclude any special arrangement made or sought to be
made between a member of either of the Associations for any special quality or description of waste paper. (2) Waste paper
of the standard descriptions shall be sold at prices which are not greater than the maximum prices and not lower than the
minimum prices agreed from time to time and the present maximum and minimum prices are set forth in the schedule of
prices hereto annexed. (3) Except by special agreement, all waste paper of every grade shall be free from waxed paper or
waxed board and from all contraries not suitable for cold repulping processes. (4) The prices set forth in the schedule of
prices are calculated ex works and no loading or other extra charges are to be made.

By a further agreement between the two Associations which became effective on 16 July 1962, although it is dated 23 October
1962, the provisions of Sch 2 were substantially amended, and the original agreement was itself amended. As a result of the
latter amendment, both Associations accepted the revised Sch 2 as binding on their respective members as from 16 July 1962, so
that s 6(6) now is applicable to both Associations and their members, in so far as the provisions of Sch 2 are restrictions within s
6(1) of the Act. The amendment of Sch 2 involved the deletion of para (2) and para (4) thereof, which are the provisions relating
to prices. Neither Association has sought, in these proceedings, to justify those provisions as to prices under s 21 of the Act, and
the court accordingly declares that the provisions of para (2) and para (4) of Sch 2 to the agreement of 12 August 1957, are
contrary to the public interest.
The contested issues in this reference thus relate to the provisions of para (1) and para (3) of Sch 2. With regard to each of
them, each of the Associations puts forward two alternative submissions: first, that neither of them constitutes a restriction within
s 6(1) of the Act; and, secondly, that, if they are such restrictions, then each of them is justified under s 21(1)(b) of the Act; with
the further alternative submission that, if the para (1) restriction is so justified, then the para (3) restriction would be justified by s
21(1)(g), if not justified in itself by s 21(1)(b).
The first submission gives rise to questions of law. In order to decide whether a provision is or is not within s 6(1), it is first
necessary to ascertain the meaning of the provision. There is, perhaps, some ground for the view that the words in para (1) of Sch
2 to the agreement are not clear or precise. Standard descriptions of the various grades of waste paper are accepted
Accepted for what purpose? but standard descriptions shall not preclude any special arrangement for any special
quality or description of waste paper. What is meant by a special arrangement? The court is not prepared to say that these
words, which have been accepted and acted on for a number of years by the two Associations, have no meaning. Slightly
different formulae were put forward by 428 counsel for the registrar and for the Associations to express the effect of the words in
para (1); but the difference is in words, not in substance. The court is prepared to accept the interpretation substantially as put
forward by counsel for the British Waste Paper Association, as follows:

In dealings between a member of one Association and a member of that Association or of the other Association, the
number or name of the respective grades of waste paper as set out in the schedule of descriptions shall be understood as
incorporating the extended definition set out in the third column of the schedule of descriptions.

There is no reason to think, on the evidence, that the members of the Associations have acted on any different interpretation of
para (1).
It follows that, if a member of the Merchants Association negotiates with another member of that Association, or (as would
be more usual) with a member of the Manufacturers Association, for the sale of a grade of waste paper by reference to the
number or name appearing in the first or second column of the schedule of descriptions, then both parties to the negotiation
understand and accept that the goods referred to are goods which correspond with the detailed description in the third column;
and that the goods to be delivered under any resulting contract shall be such goods. The paragraph is thus interpreted as applying
only in transactions to which members of the Associations are parties. Further, the words special arrangement are treated as not
connoting any inhibition on, or any special formalities in respect of, negotiations or contracts between members of the
Associations in respect of any type or grade of waste paper which falls outside the forty-seven grades at present included in the
schedule of descriptions. The proviso beginning with the word but in para (1) is treated as merely emphasising that the
preceding part of the paragraph is not to be regarded as compelling or inviting members to confine their transactions to the grades
presently included in the schedule of descriptions. Special arrangement is really indicative only of the fact that there is at
present no agreed specification of the required contents of any grade which members may wish to buy or sell outside the grades
defined in the schedule of descriptions. Had there been any fair inference from the proviso, and in particular from the words
special arrangement, that any difficulties or special formalities were intended to be created or involved in the sale and purchase
of types of waste paper outside the grades in the schedule of descriptions, there would have been a restriction within s 6(1) of the
Act.
However, taking the interpretation which is, in effect, the interpretation accepted by the Association and the registrar, it is
still submitted on behalf of the registrar that there is here a restriction which falls within s 6(1) of the Act. In his answer, and in
the argument on his behalf, the registrar relies primarily on s 6(1)(c), which refers to restrictions accepted in respect of the
quantities or descriptions of goods to be supplied or acquired. He contends that there is here such a restriction because the
members of both Associations are, by the agreement, limited as to the manner in which they shall describe goods of the grades in
the schedule of descriptions. The validity of that argument depends on the meaning of the word descriptions in s 6(1)(c) of the
Act. There is prima facie a simple charm about the registrars argument, because the whole provision is expressed to relate to
descriptions. The first possibly relevant definition of description in the Shorter Oxford Dictionary is: The action of setting
forth in words by mentioning characteristics. If this is the meaning in s 6(1)(c), the registrars argument is right. For the
Association, on the other hand, it is contended that the meaning here is that which is given in the second possibly relevant
definition in the same dictionary: The combination of qualities or features that marks out or describes a particular class: hence,
a sort, kind or variety.
In the opinion of the court, that second meaning is the meaning of the word in s 6(1)(c). The quantities or descriptions
means the quantities or kinds. 429What para (c) of s 6(1) is dealing with is restrictions as to the quantities of goods and as to
the kinds of goods; not as to the quantities and the nomenclature of goods. This view is reinforced by other provisions in Part I of
the Act, where it would be strange, indeed, if the same word had a different meaning. Thus, in s 8(3)(a) and (b), it would appear
that goods of the same description means no more than goods of the same kind. In s 8(5), and again in s 8(7), it seems clear
that the descriptions of goods means nothing more than the kinds of goods. There is, then, no restriction within s 6(1)(c) in
para 1, because there is no restriction as to the kinds of goods to be supplied or acquired.
The registrar, however, seeks to rely also on s 6(1)(b), which relates to restrictions accepted in respect of the terms or
conditions on or subject to which goods are to be supplied or acquired. He contends that it is a restriction within that paragraph
if there is advance agreement as to the contractual definition of particular words which may be used in anticipated contracts.
There are, indeed, cases in which such an advance agreement might well involve restrictions within s 6(1)(b). In the opinion of
the court, however, that is not the case here. There is nothing restrictive in agreeing in advance that, if a particular code should be
used to describe types of goods, the meaning set out in the code shall apply, so long as it is apparent, as is the case here on the
interpretation which we accept, that the persons whom the agreement binds are not in any way, expressly or by implication,
deterred from entering into transactions which fall outside the scope of the code. Accordingly, in the opinion of the court, para
(1) of Sch 2 to the agreement does not involve or include anything which is a restriction within s 6(1) of the Act.
Paragraph (3) of Sch 2 to the agreement also does not include any restriction within s 6(1) of the Act, for two reasons. First,
it does no more than refer to something which is already incorporated in the heading to the schedule of descriptions, making it
clear that each of the descriptions is to be read as though it contained the words: Free from waxed paper or waxed board and all
contraries not suitable for cold repulping processes. There is nothing in the provisions of para (3) to prevent or hinder any
member of either Association from agreeing to sell to, or buy from, another member of either Association (as he might, indeed,
wish to do) any waxed paper or waxed board. It is merely made clear that the scheduled grades by definition do not include such
material. It is also left entirely open to any member to sell or buy waste paper containing contraries, if for any reason he should
see fit to do so. Secondly, it seems from the evidence, probable, that s 14(1) and (2) of the Sale of Goods Act, 1893, would
render it a breach of contract, in the absence of special provision in the contract, for a merchant to deliver to a manufacturer waste
paper containing contraries. It is not a restriction within the Act of 1956 to impose by agreement a term which goes no further
than is already provided by general statute. Paragraph (3) is not a restriction. It is merely a warning notice of something which,
in the circumstances, is already covered by the existing law.
That is sufficient to dispose of this reference; but, since another court might take a different view of the questions of law, it
seems desirable, and it is in accordance with the wishes of the parties, that we should deal with the submissions made under s
21(1)(b) of the Act. Although we shall express our conclusions at no great length, we have considered this part of the case with
no less care than we should have devoted to it if a different conclusion had been reached on the first part of the case. For this
purpose, we assume that the restrictions (as we shall call them in this part of the judgment, though we have held that they are not
restrictions within the Act) have the meaning which we have already indicated. It was on the basis of that meaning that the
Associations sought to defend them and that the registrar opposed them. Both Associations, in their submissions to the court,
treated the relevant public, for the purposes of s 21(1)(b), as being the persons who purchase, consume or use paper or board in
the manufacture of which waste paper is used as a raw material. They assert that, as a result of these 430 restrictions, the
purchasers, consumers and users of paper or board obtain these goods, and are likely hereafter to obtain them, at a lower price
than they would be likely to have to pay if the restrictions were to be abrogated. That is the specific and substantial benefit or
advantage which the Associations contend would be lost by the abandonment of the restrictions.
The case was put in the form of three propositions: (i) The existence of the restrictions engenders flexibility in the supply of
waste paper, by grades, by merchants who are members of the Merchants Association, and to mills which are members of the
Manufacturers Association; and further, within these limits, the existence of the restrictions eases sorting and stockpilingthat
is, sorting by the merchants and stockpiling by the millsand cuts down staff costs. (ii) Against that background of their
usefulness, the removal of the restrictions would be likely, on the balance of probability, to lead to increased costs of the materials
used by the mills which make board and paper from waste paper, and also would be likely to lead to an increase in the other costs
of manufacture of the mills. (iii) On the balance of probability, it is likely that those higher costs would lead to higher prices of
the relevant board and paper products. Alternatively, if competing products having an equivalent use were so priced that it was
found impossible to pass on to the consumer the increased cost of making paper and board containing waste paper and some mills
were put out of business then, since the available substitutes are more expensive than products now made from home-supplied
waste paper, the consuming public would be injured by being forced to buy those more expensive products.
We have carefully considered these propositions and the evidence bearing on them. While at least propositions (i) and (ii)
have undoubted attractions, and have some evidence, although vague and not substantiated by any figures or supporting
calculations, we are unable to hold, on the evidence before us, that proposition (iii) is established. Assuming in favour of the
Associations that the abandonment of the restrictions, despite any action which individual mills or merchants might, and possibly
would, legitimately take toward achieving the same effect, would be likely to lead to some unquantified increase in the costs of
manufacture of board and paper, we have no evidence which would justify us in the conclusion that such an increase in costs
would be likely to lead to an increase in the price of the products or to the cessation of manufacture, or in concluding whether
such increase, if any, in price would be trivial or greater than trivial. We have no evidence as to the extent, relative or actual, of
the increase in costs; we have no evidence whether such increase would be capable of being absorbed by the manufacturers by
accepting lower levels of profits; we have no evidence whether or not the nature of the existing or anticipated competition for the
sale of the products would be likely to make the manufacturers anxious to decrease their profit margins in face of increased costs,
rather than to increase their prices. In these circumstances, we should be bound to hold that, if the terms of the agreement here in
question were restrictions within s 6(1) of the Act, they have not been justified under s 21(1)(b). Accordingly, the alternative
argument in favour of para (3) of Sch 2, under s 21(1)(g) of the Act, would not arise.
In the result, we hold that the provisions of para (1) and para (3) of Sch 2 are not restrictions within Part I of the Act; and
that the provisions of para (2) and para (4), which have been abandoned voluntarily since July, 1962, are contrary to the public
interest.
[Counsel for the Association applied for costs against the registrar under r 76 of the Restrictive Practices Court Rules,
1957,e, on the ground that the registrar had occasioned the expenditure of substantial costs in the defence of an agreement which
had no economic significance, and that that constituted unreasonable conduct. Counsel said that, by a letter dated 18 March
1962, the registrar had said that he 431 was not prepared to consider making any representation to the Board of Trade under s 12
of the Act of 1956,f on the basis that if his representation was rejected the Associations would attempt to justify the restrictions
before the court, but that he would be prepared to make a representation if the agreement were so varied that only restrictions
falling within s 6(1)(c) were accepted thereunder. Counsel for the registrar said that the registrar had acted perfectly reasonably
and that it was inconsistent for the Associations to ask on the one hand for the agreement to be referred to the Board of Trade on
the ground that it was of no economic significance, and on the other hand to say in the formal pleadings on the record that the
agreement conferred a specific and substantial benefit.]
________________________________________
e Rule 76, so far as material, provides: Where it appears to the court that any party has been guilty of unreasonable conduct the court
may make an order for costs against him.
f Section 12, so far as material, provides: (1) The Board of Trade may, upon the representation of the registrar, give directions authorising
him to remove from the register particulars of such agreements of which particulars are for the time being entered therein as appear to the
board to be of no substantial economic significance.

MEGAW J. The circumstances which would have justified an order for costs under r 76 do not, in the opinion of the court, exist,
and there will be no order as to costs.

Order accordingly.

Solicitors: Beddington, Hughes & Hobart (for the British Waste Paper Association); Slaughter & May (for the British Paper and
Board Makers Association (Incorporated)); Treasury Solicitor.

Mary Colton Barrister.


[1963] 2 All ER 432

Charterhouse Credit Company Ltd v Tolly


CONTRACT: CONSUMER; Consumer credit

COURT OF APPEAL
ORMEROD, UPJOHN AND DONOVAN LJJ
21, 22, 23, 24 JANUARY, 15 MARCH 1963

Contract Exception clause Exception clause ineffective Fundamental breach of party seeking to rely on clause Subsequent
affirmation of contract by other party Whether clause could be relied on to defeat affirming partys claim for damages.

Hire-Purchase Damages for breach of contract Hirers breach by not paying three instalments Termination of hiring by
owners Goods re-sold by owners Fundamental breach by owners by delivery of unroadworthy car Election by hirer to
affirm contract Measure of damages of owners and hirer.

If a party to a contract had broken a fundamental term, an election by the other party to affirm the contract and sue for damages,
does not of itself re-entitle the party in breach to rely on an exemption clause which he has disentitled himself from relying on by
his breach of the fundamental term of the contract (see p 438, letter h, p 439, letter a, p 442, letter g, and p 445, letter g, post).
Karsales (Harrow) Ltd v Wallis ([1956] 2 All ER 866), Pollock & Co v Macrae (1922 SC (HL) 192) and Wallis, Son and
Wells v Pratt and Haynes ([191113] All ER Rep 989) applied.
A hire-purchase agreement, made on 30 December 1960, between a finance company and the hirer, for the hire-purchase of
a second-hand motor car at a hire-purchase price of 468 12s payable by an initial payment of 90 and twenty-three monthly hire
rentals provided that the hirer should have no claim against the finance company arising directly or indirectly out of any defect
in the vehicle. The car, when delivered to the hirer, had among other defects, a serious defect in the back axle, which had been
badly repaired after a previous accident and would have cost about 40-50 to repair. Although the car could in fact be driven on
the road, this defect made it completely unroadworthy and unfit for use on the road. On discovery of the defect the hirer, who
had made the initial payment of 90, decided to have it repaired himself, and asked for an extension of time to pay the monthly
432 rentals. The finance company gave the hirer three months, but the hirer fell ill and so did not get the car repaired. The hirer
paid none of the monthly rentals, and on 24 April 1961, when three rentals totalling 47 4s were in arrear, the finance company
terminated the hiring and re-took the car, as permitted by the agreement. A few days later the finance company sold the car for
250. Before discovering the defect the hirer had driven the car twice from Greenwich to Norwood and back. In an action in
which the finance company claimed, and the hirer counterclaimed, damages,

Held (i) The finding of the county court judge that there had been a fundamental breach of the hire-purchase agreement by the
finance company (viz, by the delivery of the car with a defect which made it unroadworthy, which would involve the hirer in
what would be for him a substantial sum for its repair) would not be disturbed, and this breach entitled the hirer to repudiate the
contract and disentitled the finance company from relying on the protection of the exemption clause (see p 437, letter b, p 441,
letter d, and p 444, letter f, post).
(ii) The hirer having by his conduct elected to affirm the contract, and the finance company having then lawfully terminated
it, the damages to which the finance company were entitled were the aggregate of the rentals in arrear at the date when the hiring
was terminated, viz, 47 4s (see p 437, letter f, p 441, letters g and h, and p 446, letter e, post).
Financings Ltd v Baldock ([1963] 1 All ER 443) applied.
(iii) The hirer not being excluded by the exemption clause from recovering damages from the finance company, as the
exemption clause did not avail the finance company in view of their breach of a fundamental term of the hire-purchase agreement
(see p 432, letter f ante), was entitled to such damages for the finance companys breach as would place the hirer in the same
position as if the hire-purchase agreement had been performed (see p 439, letter f, post); in the particular circumstances of this
case these damages would be assessed at 132 4s (viz, the initial payment of 90, together with 47 4s for three rental payments,
less 5 for the use of the car on two journeys) (see p 439, letter h, p 443, letter g, p 444, letter b, and p 446, letter g, post).
Yeoman Credit Ltd v Apps ([1961] 2 All ER 281) considered and not followed.
Per Upjohn LJ: If the finance company had not terminated the hiring they would have been entitled to be paid the rental for
the remainder of the hiring, and the hirers measure of damages would have been the amount required to put the vehicle into a
proper state of repair together with damages for loss of use while it was being put in repair (see p 443, letter e, post.
Observations as to the construction and effect of the exempting clause (see p 438, letter c, and p 445, letter h, post).
Appeal allowed.

Notes
The damages recovered by the hirer (143 4s in all) included 11 for new tyres, but this element was regarded as de minimis and
there was some divergence of opinion with regard to it; see p 440, letter d, and p 444, letter c, post.
As to the owners right to damages on the determination or repudiation of a hire-purchase agreement, see 19 Halsburys
Laws (3rd Edn) 550, 551, para 891; and for cases on the subject, see 17 Digest (Repl) 154, 522, 523. As to actions by hirers
against owners for damages, see 19 Halsburys Laws (3rd Edn) 555, para 899.
As to the implied condition of fitness in hire and hire-purchase contracts, see 2 Halsburys Laws (3rd Edn) 123, 124, para
237; 19 ibid, 532, 533, para 858; and for cases on the subject, see 3 Digest (Repl) 9597, 231250; 26 Digest (Repl) 666, 35, 36.
433
As to exemption clauses in hire-purchase contracts, see 19 Halsburys Laws (3rd Edn) 531, para 855, text and note (s), and
generally, see Supplement to 8 Halsburys Laws (3rd Edn), para 215A.

Cases referred to in judgments


Alexander v Railway Executive [1951] 2 All ER 442, [1951] 2 KB 882, 3 Digest (Repl) 93, 225.
Anglo-Saxon Petroleum Co Ltd v Adamastos Ltd [1957] 2 All ER 311, [1957] 2 QB 233, [1957] 2 WLR 969, CA, on appeal sub
nom Adamastos Shipping Co Ltd v Anglo-Saxon Petroleum Co Ltd [1958] 1 All ER 725, [1959] AC 133, [1958] 2 WLR 688,
HL, 3rd Digest Supp.
Astley Industrial Trust Ltd v Grimley (ante, p 33).
Bridge v Campbell Discount Co Ltd [1962] 1 All ER 385, [1962] AC 600, [1962] 2 WLR 439.
Financings Ltd v Baldock [1963] 1 All ER 443, [1963] 2 WLR 259.
Hadley v Baxendale [1854], 9 Exch 341, 23 LJEx 179, 23 LTOS 69, 156 ER 145, 8 Digest (Repl) 151, 956.
Karsales (Harrow) Ltd v Wallis [1956] 2 All ER 866, [1956] 1 WLR 936, 26 Digest (Repl) 666, 35.
Morelle Ltd v Wakeling [1955] 1 All ER 708, [1955] 2 QB 379, [1955] 2 WLR 672, 13 Digest (Repl) 288, 1065.
Overstone Ltd v Shipway [1962] 1 All ER 52, [1962] 1 WLR 117.
Pinnock Bros v Lewis & Peat Ltd [1923] 1 KB 690, 92 LJKB 695, 129 LT 320, 2 Digest (Repl) 450, 187.
Pollock & Co v Macrae 1922 SC (HL) 192, [1922] SLT 510, 39 Digest (Repl) 468, q.
Robinson v Harman (1848), 1 Exch 850, 18 LJEx 202, 13 LTOS 141, 154 ER 30, Digest 443 (Repl) 857.
Smeaton Hanscomb & Co Ltd v Sassoon I Setty Son & Co [1953] 2 All ER 1471, [1953] 1 WLR 1468, 3rd Digest Supp.
Spurling (J) Ltd v Bradshaw [1956] 2 All ER 121, [1956] 1 WLR 461, 3 Digest (Repl) 84, 196.
Strand Electric and Engineering Co Ltd v Brisford Entertainments Ltd [1952] 1 All ER 796, [1952] 2 QB 246, 3rd Digest Supp.
Wallis, Son and Wells v Pratt and Haynes [191113] All ER Rep 989, [1911] AC 394, 80 LJKB 1058, 105 LT 146, 39 Digest 477,
996.
Yeoman Credit Ltd v Apps [1961] 2 All ER 281, [1962] 2 QB 508, [1961] 3 WLR 94, 3rd Digest Supp.
Yeoman Credit Ltd v Waragowski [1961] 3 All ER 145, [1961] 1 WLR 1124, 3rd Digest Supp.

Appeal and cross-appeal


This was an appeal by the defendant, Joseph Edward Tolly (hereinafter called the hirer), against a decision of His Honour
Judge Cohen given at Croydon County Court on 7 May 1962. The plaintiffs, Charterhouse Credit Co Ltd (hereinafter called the
finance company) cross-appealed.
By their particulars of claim, dated 28 June 1961, the finance company claimed 127 12s under an agreement for the hire-
purchase of a car by the hirer, on which agreement they alleged that the hirer had made default in paying monthly rentals. The
finance company claimed 127 12s under the agreement, as being a sum equal to the hire rentals due down to the date when they
had taken re-possession of the car together with such sum as would make those up to seventy-five per cent of the total rentals;
alternatively the finance company claimed the 127 12s as damages, being the total charges (together with costs of repairs to the
re-possessed car) less the amount paid by the hirer and the proceeds of sale of the car, which the finance company had sold after
repossession.
The hirer, by his defence pleaded that it was an implied term of the agreement 434 that the car should be as reasonably fit for
the purpose for which it was hired as reasonable care and skill could make it, and that, in breach of this fundamental term of the
agreement, the car was unroadworthy and unsafe. He alleged that he had taken the car back to the dealers who supplied it, and
that they had refused to make it roadworthy; that he did not thereafter use the car and refused to accept it in its then condition,
and that he notified the dealers of its condition. He denied liability and counterclaimed for 90, the amount of the initial payment
which had been credited to him; he also counterclaimed for damages, alleging, among other matters, that he had bought two new
tyres for the car at a cost of 11.
The county court judge awarded the finance company the 127 12s which they claimed, less 30 for acceleration of
payment, and awarded the hirer 81 damages for defects (including 11 for the two new tyres).

Leonard Caplan QC and Jonathan Sofer for the hirer.


John Shaw for the finance company.

Cur adv vult

15 March 1963. The following judgments were delivered.

DONOVAN LJ. The dispute arises out of a hire-purchase agreement. On 30 December 1960, the hirer, Mr Tolly, entered into
such an agreement with the finance company, Charterhouse Credit Co Ltd for the hire-purchase of a secondhand Vauxhall motor
car. The cash price was 410. The hire charges were 57 12s. There was the usual conditional option to purchase for a final
payment of 1. The hire-purchase price was therefore 468 12s. This was to be paid by an initial payment of 90 plus twenty-
three monthly hire rentals of 15 14s 8d and one final such rental of 16 14s 8d.
The hirer paid the initial payment of 90. He paid, however, none of the monthly instalments. He refused to do so on
finding, when he took the car on to the road, that it had a serious defect in the back axle. This, apparently, had been damaged in a
previous collision and badly repaired. According to the findings of the learned county court judge the hirer contemplated having
the car properly repaired himself. The cost would have been in the neighbourhood of 50, but he could not meet this outlay and
keep up the monthly instalments as well. He, therefore, asked the finance company for some indulgence in the matter of time,
and in all some three months was given. In the meantime the hirer fell ill and had to go to hospital. The car remained unused in
his front-garden. He had, in fact, driven it on two occasions only from Norwood to Greenwich and back; and his son-in-law had,
after the second of these journeys, partially stripped the back axle and discovered the botched repair.
No monthly instalment having been paid, the finance company, pursuant to their right under the hire-purchase agreement in
these circumstances so to do, terminated the hiring and re-took possession of the car on 24 April 1961, and a few days later it was
sold by the company for 250. On 28 June 1961, the finance company commenced the proceedings which have led to this appeal
and cross-appeal. By their particulars of claim they asked for a further sum from the hirer of 127 12s 0d, and they described this
as

damage being the total hiring charges less the amount paid and proceeds of sale of the said car.

This claim was rested on the common law. There was a clause in the hire-purchase agreement a purporting to give the finance
company a right, in effect, to recover a larger sum sufficient to make the hirers payments up to three-fourths of the total hiring
charges under the agreement; but at the hearing before the county court judge the finance company, though they had pleaded the
clause, abandoned any claim under it, no doubt because they apprehended that it would be regarded as penal. The hirer delivered
a defence alleging that the finance 435 company were in breach of a fundamental term in the contract that the car should be
reasonably fit for the purpose for which it was hired, and that, on discovering that the car was, as he alleged, unusable,
unroadworthy and unsafe, he had refused to accept it in that state. He also counterclaimed for the return of his initial payment of
90 as being money paid for a consideration which had wholly failed; for the cost of two new tyres which he had fitted to the car,
and for damages.
________________________________________
a Clause 14, which is summarised by Upjohn LJ see p 441, letter f, post

It is not now disputed that at common law it was an implied term of the contract that the car should be reasonably fit for the
purpose for which it was hired (Yeoman Credit Ltd v Apps).
The findings of the learned county court judge were these: (i) There had been a fundamental breach of the contract on the
part of the finance company in hiring out a car to the hirer which was not fit to be driven, completely unroadworthy, and not
fit to be used on the road. (ii) The hirer had not taken up the position that by this fundamental breach the finance company had
repudiated the contract, and that he himself, therefore, would no longer be bound by it. (iii) Instead he had elected to treat the
contract as still subsisting, and as still binding on him, without prejudice, however, to his right to sue the finance company for
damages in respect of their fundamental breach of the contract. (iv) In the circumstances the finance company were entitled to
recover as damages the sum they claimed, namely, 127 12s, subject, however, to a deduction of 30 for accelerated payment,
that is, 97 12s. (v) The hirer on his counterclaim was entitled to recover a sum of 81, made up as follows: 45, the estimated
cost of repairs to the back axle; 25 for other defectsthere were some others, though less seriousand 11 for two new tyres
which the hirer had fitted, and of which the finance company had the benefit when they re-took the car and sold it.
Before us, the hirer, who is the appellant, contends first that the damages awarded to the finance company were computed on
a basis which is wrong in law. Pursuant to the recent decision of this court in Financings Ltd v Baldock the finance company
were entitled to receive only those instalments which were in arrear when the company terminated the contract. He next contends
that the damages awarded to him on his counterclaim were too little, and were again computed on a basis which is wrong in law.
He asks for damages equal to the initial instalment of 90, the three monthly instalments totalling 47 4s which he admits he
must pay under the contract; and 11 for the two new tyres.
The finance company resists these contentions, and on its cross-appeal has argued(a) that even though there was an
implied term in the contract as to the reasonable fitness of the car for hiring purposes, nevertheless there had been no fundamental
breach of the contract; (b) alternatively, that cl 5 of the contract precluded the hirer from relying on any such breach; and (c) that,
in any event, the damages awarded to the hirer on his counterclaim were too high, having been computed on a wrong principle.
It will be convenient to deal with these rival contentions in a different order, and to take first the question whether the
finance company was in fundamental breach of the contract. It was urged on its behalf that a car might deserve all the epithets
levelled against this particular car by the county court judge, and its delivery to a hire-purchaser still be a fulfilment of the
contract. For example, a car with defective steering might be unfit to be used and quite unroadworthy, and yet the task of
putting it right might be simple and inexpensive. Nobody would describe such a defect as involving a fundamental breach of the
contract. This I would accept; but I am not disposed to interfere with the county court judges finding in the present case. A
defect in the back axle can be an extremely serious and dangerous matter, and when the hirer agreed to take a car on hire-
purchase from the finance company the last thing that he would want would be a 436 car with such a defect involving what
would be for him a substantial outlay on repairif he wished to repair. The judge committed no error of law when he said, in
effect, that delivery of such a car was not a performance of the contract at allin other words, that there was a fundamental
breach by the finance company of their contractual promise. I would not say the question is purely one of fact. The facts, of
course, must be found, but whether they amount to such a fundamental breach involves legal considerations, and I think that the
question is, at the least, a mixed question of fact and law. I agree with the county court judges decision that such a breach here
occurred.
He went on to find, as I have said, that the hirer, nevertheless, elected to treat the contract as still on foot. Against this
finding there is now no appeal by the hirer. The effect is, of course, that he must pay for such breaches of the contract as he
himself may have committed. These consist, however, solely of his failure to pay the three monthly instalments of hire due at the
date when the finance company terminated the hiringa total of 47 4s. Pursuant to the recent decision of this court in
Financings Ltd v Baldock, this is all that the finance company can recover. Counsel for the finance company contends that this
decision is wrong, and that it is in conflict with two earlier decisions of this court, namely, Yeoman Credit, Ltd v Waragowski and
Overstone Ltd v Shipway, in each of which cases damages were awarded on a basis similar to that adopted by the county court
judge in the present case. It is unnecessary to recite the facts of those two cases. They are epitomised in the judgments in
Financings, Ltd v Baldock; and the root distinction which they exhibit is that they were cases where the hirer had evinced an
intention to be no longer bound by the contractin other words, had repudiated it. In such a case, of course, the other party had
the right to recover as damages what he lost by such repudiation, namely, what he would have received in hire charges had the
contract been allowed to run its course. The case is quite different where the owner of the goodsin the present case the finance
companyterminates the contract. There can then be no breach of the promise to pay instalments falling due thereafter, for the
contract has ceased to exist before they became due. The facts of the present case are, so far as they bear on the true measure of
the finance companys damages, essentially no different from those in Financings Ltd v Baldock, and the same result must follow.
On this point, therefore, the hirers appeal succeeds and the finance companys damages must be reduced to the sum of 47 4s.
I turn now to the hirers own claim to damages for the finance companys fundamental breach of contract. Here two issues
arise. First, is the claim competent at all? Second, if it is, what is the true measure of the damages?
The competence of the claim is challenged by the finance company on the ground that it is excluded by the terms of the
contract. Clause 5 thereof, so far as at present material, reads thus:

The [finance company] do not supply the vehicle with or subject to any warranties or conditions either express or
implied by statute
Pausing there for the moment, counsel for the hirer says this does not hurt him in the least; for the hirer is relying not on any
express warranty or condition, nor on any warranty or condition implied by statute. He is relying on the condition as to a chattel
being reasonably fit for the purpose for which it was hired out which is implied by the common law. But cl 5 goes on:

and the hirer shall have no claim against the [finance company] arising directly or indirectly out of any defect in the
vehicle and shall indemnify and keep indemnified the [finance company] their successors and assigns in respect of any
third party claim so arising

Counsel for the finance company says that this part of the clause excludes the present counterclaim of the hirer. Counsel for the
hirer says that it does not, 437on the ground that on its true construction it refers only to claims arising out of a mishap caused by
some defect in the vehicle whether those claims be by the hirer or by some third party. The first part of the clause dealing with
warranties and conditions is self-contained and comprehensive, and what follows deals with something else. Accordingly, as the
counterclaim is based on the breach of a condition, and the first part of cl 5 admittedly does not touch this case, the clause is no
bar to the claim.
These rival arguments are, of course, arguments as to the true construction of cl 5, and there is force in the contentions of
both sides. Counsel for the hirer fortifies his case by a reference to a concluding proviso b to cl 5 dealing with the situation where
the goods hired out have a hire-purchase price not exceeding 300. This, he says, is unintelligible, and I agree, although the hirer
has signed on the face of the agreement a printed clause to the effect that cl 5 had been explained to him and he fully understands
its effect. The authorities show that an exempting clause of this character must be clear and precise if it is to be given effect (see,
for example, Anglo-Saxon Petroleum Co Ltd v Adamastos Ltd ([1957] 2 All ER 311 at pp 318, 320; [1957] 2 QB 233 at pp 269
272.). It is contended for the hirer that cl 5 is neither clear nor precise.
________________________________________
b The proviso was as followsProvided That if the hire-purchase price of the vehicle does not exceed 300 this clause shall be limited in
application to the condition as to fitness of the vehicle for a particular purpose implied by s 8(2) of the Hire-Purchase Act (which is hereby
expressly excluded) and (in the case of a second-hand vehicle) to any condition as to the vehicle being of merchantable quality.

I do not find it necessary to determine the true construction of this clause, for even if it bears the construction contended for
by the finance company, I am of the opinion that it is of no avail to them in this case. As has been often said in recent years, a
fundamental breach of contract, that is, one which goes to its very root, disentitles the party in breach from relying on the
provisions of an exempting clause (see hereon the remarks of Denning LJ, in Karsales (Harrow) Ltd v Wallis ([1956] 2 All ER
866 at pp 868 et seq), where he refers to the principle and cites various earlier authorities which propound it). That principle will
apply here, unless an argument which counsel for the finance company has put forward against this result is an argument which
ought to be sustained. It is this: that when the hirer elected not to treat the fundamental breach as ending the contract, but chose
instead to treat the contract as still subsisting, then he affirmed or approbated the contract with the result that he is bound by
cl 5 as much as by any other clause of the contract. So that when the hirer comes to sue for damages for the breach, cl 5 may be
relied on by the finance company to defeat the claim. I do not think that anything turns on the precise label to be attached to the
hirers action in treating the contract as still subsisting despite the finance companys fundamental breach. Whether one calls it
affirming or approbating, the contract can make no difference. If he had elected instead to treat the contract as at an end, and
had sued forthwith for damages for the fundamental breach, it is not contested that cl 5 would have been of no avail to the finance
company. The question is whether the position in this respect is reversed simply because the hirer chose his alternative remedy,
that is, not to interfere with the contract. Counsel for the hirer argues that the position is no different, but that the condition as to
reasonable fitness of the chattel for the purpose for which it was wanted remains a condition whichever remedy is pursued; that a
breach of it remains a fundamental breach whichever remedy is pursued, and that against such a breach an exempting clause such
as cl 5 is of no avail.
The point is, apparently, free from direct authority, but, on principle, the election by the hirer of one remedy for the
fundamental breach, instead of another remedy, ought, as I see it, to make no difference to the ineffectiveness of an exempting
clause in face of such a breach. However this may be, two decisions of the House of Lords exist where one party to a contract
elected to treat it as still subsisting despite a fundamental breach by the other, and succeeded in 438 obtaining damages for such
breach despite the existence in the contract of an exempting clause similar to cl 5 here. One is Pollock & Co v Macrae; and the
other Wallis, Son and Wells v Pratt and Haynes. The contention that I am now considering was not specifically raised in either
case, but it is impossible to think that, if valid, it would have been overlooked not only by the parties sued, but by all the courts
before which the two cases came. I think, therefore, that the argument of the finance company on this matter ought to be rejected.
That leaves as the final matter to be considered the true measure of the damages to be awarded to the hirer on his counterclaim.
Ignoring for the moment the cost of the two new tyres, the county court judge adopted as the measure of damages the
estimated cost of the necessary repairs to the car, saying that the hirers counterclaim was similar to a claim for a diminution of
the price. Counsel for the hirer says that while such a measure of damage might be appropriate to the breach of a contract of sale,
it is not appropriate to a contract of hire, such as the present contract was when the fundamental breach was committed. This
proposition is not contested by counsel for the finance company, and, in my view, it is right. If I hire a car which turns out to be
useless for the purpose by reason of defects, it is not for me to repair the car; and to treat the estimated cost of repairs as the
measure of my damage does not really meet the requirements of the case. But while both parties here are agreed that the county
court judges measure of the damages was a wrong one, they are not agreed what measure is the right one. For the hirer it is
argued that he should get by way of damages all the hiring charges which he has paid less some small deduction for the small use
which he had of the car, that is, the two journeys to Greenwich and back. It is conceded that this use of the car, although
minimal, precludes him from claiming the total hire payments as being money paid for a consideration which has wholly failed.
For the finance company, on the other hand, it is contended that the true measure of damage is the cost of hiring a similar car up
to the time when the contract was terminated.
This point again seems to be free from direct authority, but an application of basic principle does, I think, yield the true and
just solution. The general rule in cases of breach of contract is that a plaintiff is entitled to be placed, so far as money can do it, in
the same position as he would have been in had the contract been performed (per Parke B, in Robinson v Harman ([1848], 1 Exch
850 at p 855)). This involves considering what would have been the hirers position had there been no continuing breach of the
contract by the finance company. He would then have had a suitable car delivered to him on hire-purchase terms. He would have
been liable for the initial payment plus three monthly instalments. These payments would have been much higher than payments
for mere hire alone, on account of the eventual option to purchase for the purely nominal sum of 1. The hirer has, in fact, paid
the initial payment of 90, and under this judgment must pay the three instalments amounting to 47 4s. For this outlay,
however, he has received nothing, owing to the finance companys breach of contract, except the two rides to Greenwich. What
is required to put him, so far as money can, in the same position as if the contract had been performed? To my mind, it is a sum
equal to the cost of hiring a similar car on similar terms as to the eventual option to purchase for 1. There is no reason why one
should not adopt as the figure of that cost what the hirer actually has to pay to the finance company for the like hiring in the
present case, to wit, 90 and 47 4s less a small sum for the two journeys he made in the car, which I would put at 5. The net
figure thus becomes 132 4s.
I reject counsel for the finance companys argument that all to which the hirer is entitled is the cost of hiring another similar
car. That would not of itself 439 be restitutio in integrum, for the contract here was not one of simple hire but of hire-purchase.
Furthermore, it leads to this absurdity, that, if a friend had lent the hirer a car for nothing for the period in question the damages
which he would have suffered from the finance companys breach would be treated as nila result which counsel for the finance
company agreed would follow. Counsel put his argument for the hirer on other alternative grounds. Thus the general rule of
restitutio in integrum of itself required damages equal to all the hirers abortive payments, less some deduction for the slight use
that he had of the car. Or, that since the finance company had put their own value on the hiring for the period in question,
namely, 90 and 47 4s, there was no reason why that should not be taken as the measure of the hirers loss, subject again to
some discount for the use which he made of the car. In this connexion counsel for the hirer referred to Strand Electric and
Engineering Co Ltd v Brisford Entertainments Ltd. Beyond mentioning them I do not examine these alternatives. They lead to
the same result in money as the measure I have already described and adopted.
With regard to the tyres, the hirer said in evidence, which the county court judge appears to have accepted, that he could not
get the car insured until two new tyres were put on at a cost of 11. There was little argument before us regarding this item, the
parties treating it as standing or falling within the larger claim.
One final matter should be noticed. In his judgment the learned county court judge in estimating the hirers damages
purported in one respect to follow the judgment of this court in Yeoman Credit Ltd v Apps. That was a case very similar to the
present, and Holroyd Pearce LJ upheld the view of the county court judge as to the measure of damage due to the hirer, namely,
the cost of putting the car in proper repair. Harman and Davies LJJ felt some difficulty about the matter, but were content to
accept the same view. The question which has troubled this court is whether or not the decision binds us to employ the same
measure when computing the hirers damages here on his counterclaim. Counsel on both sides before us have contended that the
decision on this point in the case cited was come to per incuriam. If so, the explanation may be that the real contest was whether
Mr Apps should recover the whole of the money he had paid as being for a consideration which had wholly failed (as the county
court judge had held), and that the court heard no argument in favour of a measure of damages different from that which the
county court judge said that he would have adopted if, contrary to his view, the case was one for the award of damages. Be that
as it may, I have come to the conclusion that the decision ought not to be regarded as laying down any general principle in this
regard, and that for the reasons I have already given, the measure of the hirers damages should be that above stated.
I would, therefore, allow the hirers appeal and dismiss the finance companys cross-appeal. The judgment below should,
accordingly, be varied by reducing the finance companys damages to 47 4s and increasing the hirers damages to 143 4s.

UPJOHN LJ. Subject to one small point (11 for the cost of new tyres), I agree with the judgment which has been delivered by
Donovan LJ and as he has fully set out the facts I shall not advert to them further.
As I recently stated in Astley Industrial Trust Ltd v Grimley (Ante, p 33, at 46, 47), in my judgment there are normally two
stipulations to be implied in the hiring or hire-purchase of a motor vehicle in circumstances such as are before us. The first
implied stipulation is that the lender must deliver for hire that which he has contracted to lend; this is a fundamental term of the
contract and the breach of it entitles the hirer at once to repudiate the contract. The second implied 440 stipulation is that the
lender must deliver a vehicle as fit for the purpose for which it is hired as reasonable skill and care can make it. In complete
contrast to the Astley case (Ante, p 33, at 46, 47) we, in this case, are concerned with the first implied stipulation and only in a
very minor degree with the second (on the question of new tyres).
As I pointed out in Astley Industrial Trust Ltd v Grimley (Ante, at 46), the question whether or not the motor vehicle
delivered complies with this fundamental obligation of the lender is very largely a matter of fact and degree and must depend on
the circumstances of each case, and I indicated one or two factors that would be relevant. The learned county court judge in this
case propounded the principles which should guide this matter completely accurately. He said:

These cases are never easy to decide. It is always a question of degree. In some cases the vehicle suffers from a
number of faults which, if taken together, add up to a fundamental breach. In another case you have one item which is
defective and which is so important as to destroy the basis of the contract.

That is plainly right. Then he discussed the facts, this being a case where there was one really serious breach, and he came to the
conclusion that the defective condition of the back axle was such that it did amount to a fundamental breach entitling the hirer to
treat the agreement as repudiated.
I would regard this, on the facts, as a border-line case, but the learned judge considered the matter very carefully, and I
would not seek to disturb his finding on this matter. The learned county court judge also found, and it has not been challenged
before us, that the hirer did not treat the contract as repudiated, but affirmed it. Indeed, he was anxious to remedy the trouble to
the back axle, but his difficulty was that he could not pay for the repairs and at the same time pay the instalments, and, as he was
ill for most of the relevant time, in fact he paid neither. Accordingly, the finance company, as they were lawfully entitled to do
under cl 10 of the terms and conditions of the hire-purchase agreement terminated the contract and repossessed the motor vehicle.
Thereupon the finance company became entitled under cl 14 of the Terms and Conditions to demand from the hirer: (i)
rentals up to the date of recovery of the goods; (ii) damages for breaches of the agreement other than the covenant to pay rent
(which does not arise); and (iii) such further sums as might be necessary to make up the amount already paid by the hirer to the
equivalent of the total rental attributable to the minimum period of hire c. As the last sub-clause was in the nature of a penalty and
irrevocable, the finance company abandoned their claim under that sub-clause. Accordingly, the finance company were entitled
to the three months instalments in arrear which amounted to 47 4s, and no more; but the learned county court judge gave the
finance company damages at common law for the hirers failure to carry out the contract. However, the hirer never repudiated
the agreement, he merely failed to pay certain instalments, which gave the finance company the right to terminate the agreement
but not to treat it as repudiated by the hirer. It seems to me that this case is completely covered by the recent decision of this
court, to which I was a party, in Financings Ltd v Baldock. Counsel for the finance company sought to say that this decision was
wrong and conflicted with the earlier cases of Yeoman Credit Ltd v Waragowski and Overstone Ltd v Shipway, but I cannot for
one moment accept that. I distinguished those cases in my judgment in Financings Ltd v Baldock ([1963] 1 All ER at pp 448,
449) and therefore propose to say no more about them. On the facts of this case it was the finance company who were at fault,
and it was very fortunate for them that the hirer did not accept their complete failure to carry out their part of the contract as a
repudiation. I would reduce the judgment on the claim accordingly.
441
________________________________________
c The minimum period of hire was eighteen months

In fairness to the learned county court judge, let it be said that when the matter was before him Financings Ltd v Baldock
had not been decidedd.
________________________________________
d The county court hearing was on 7 May, 1962; the Court of Appeal gave judgment in Financings Ltd v Baldock on 18 December 1962

Now I come to the counterclaim. The hirer submits that he is entitled to claim the loss which he has suffered as the result of
the failure of the finance company to perform their part of the contract. The finance company rely on the clause of exclusion of
liability contained in cl 5 of the terms and conditions of the hire-purchase contract. My Lord has cited it (Ante, p 46), and I shall
not quote it again; in my judgment, the finance company cannot rely on it. The authorities establish that where there is a breach
of a fundamental term the person in breach cannot rely on clauses of exclusion to protect him as against the other party; but the
finance company said with some force that that is so, no doubt, when the innocent party treats the contract as repudiated, yet if he
elects to affirm the contract, then he must take the benefit of the contract subject to all its provisions, including a clause of
exclusion, and he can no longer plead that the other party, though in breach of a fundamental term, cannot rely on a clause of
exclusion. That is not, I think, an easy question, and there appears to be no authority, where the matter has been expressly
decided. If I am right in the analysis of this fundamental term, that it really stems from the fact that the finance company must
lend that which they contract to lend and not something which is essentially different, it seems to me that the principle must apply
whether it is a case of repudiation accepted by the hirer or whether he affirms the contract and sues for damages. Let me take a
simple example. Suppose the lender contracts to hire a tractor to a farmer and in purported performance of that contract delivers,
not a tractor, but three fine Suffolk punches which take the fancy of the farmer and for which he can find a use. Can it be said
that by the farmers affirming the contract and accepting these horses in lieu of the tractor the lender can escape under some
clause of exclusion so that the farmer can make no claim for damages for the loss which he suffers by taking these horses in lieu
of a tractor? In principle I cannot see how that can be right. He is entitled to have the horses, and the lender having delivered
them in purported pursuance of his contract is in no position to complain of that, but surely the hirer can still say that the lender
has wholly failed to carry out his part of the bargain by failing to lend that which he contracted to lend. The law will not allow a
clause of exclusion to be pleaded in answer to such a claim. Of course, the hirer has to make allowances for the value of the hire
of the horses which he has accepted, but no more. I agree with my Lord that Pollock & Co v Macrae and Wallis, Son and Wells v
Pratt and Haynes (both cases in the House of Lords) show by necessary implication that that is the law. Accordingly, it seems to
me that the finance company here cannot rely on the clause of exclusion contained in cl 5 of the contract.
What, then, is the measure of damages which the hirer has suffered in this case? He has contracted to hire, in the event
which has happened, a motor car for three months. He cannot complain, I think, that he no longer has the option to purchase it,
because it was his own breach of the contract which entitled the lender to determine the hiring; but, having had the motor car for
three months, he has had practically no use from it. My Lord has assessed the use which he has had at 5, which I regard as a
figure on the high side, but from which I will not dissent. He is, therefore, entitled in principle, to claim for the loss he has
suffered by reason of the finance companys breach.
He might, of course, have hired another motor car in its place while it was being repaired; then, no doubt, he would have
been entitled to claim the charges he would have had to pay for that other motor car and also the amount of the repairs which he
has actually had done. He might, in addition, have some further claim in that the motor car was not available to take him on
some important mission 442 known to the finance company. However, that does not arise in this case. True it is that he was ill
and probably would not have been able to make much use of the car in any event, but that cannot be a defence in the mouth of the
wrongdoer, for, as my Lord has pointed out, he might have been able to have a free loan of a motor car from a friend to go about
his lawful occasions, and that could not possibly reduce the damages to nil. On the other hand, it is perfectly true, of course, that
the hire-purchase rent contains a very substantial element represented by the option to purchase at the end of the hiring for the
sum of 1 (for the loss of which right, for the reasons I have given, the hirer cannot complain), and, in a sense, the monthly rental
does not measure the value of the hire per month. But the assessment of damages has never been an exact science; it is
essentially practical, and a contracting party who has wholly failed to deliver that which he has contracted to lend cannot
complain if the court takes a somewhat severe view of his failure to implement his promise and makes the punishment fit the
crime. This case, in my judgment, will lay down no principle of law in the assessment of damages, though it may form a useful
guide in similar cases. But for the fact that the car moved briefly and sporadically during the hire, the hirer could have reclaimed
the money that he has paid as on a total failure of consideration. What is the practical answer? The hirer, in the event brought
about by the finance company who terminated the contract, has paid 90 plus 47 4s for the hire of the car. He has had 5 use.
His loss is, therefore, 137 4s less 5, which equals 132 4s for which he is, in my view, entitled to judgment. Let me point out
that the finance company need not have terminated the agreement; they did so of their own volition. If they had not done so the
position would have been entirely altered. The finance company would have been entitled to be paid the rental for the remainder
of the hiring, and the hirer would, no doubt, have paid 1 for the exercise of his option. The hirers measure of damages would
then have been entirely different; it would have been the amount required to put the vehicle into a proper state of repair together
with damages for loss of use while it was being put in repair; the measure of damages would be analogous to that in the case of
breach of a contract of sale and purchase.
We have, of course, been referred to Yeoman Credit Ltd v Apps, and that case does give rise to some difficulty on the
question of damages. Neither side sought to support the decision on damages, and it was suggested that it was given per
incuriam. I doubt whether it is open to us to say that, having regard to the recent decision of Morelle Ltd v Wakeling; but decided
cases are only authorities for the principles of law that they decide. The assessment of damages must vary so much with the
particular circumstances of each case that justice can only be done by applying the broad principles laid down in Hadley v
Baxendale in cases of breach of contract to the particular circumstances of each case. I find nothing in the judgment of Holroyd
Pearce LJ, in Yeoman Credit Ltd v Apps, which lays down any different principle of assessment of damages; but I must examine
his judgment in a little more detail.
It is quite clear from the extract from the county court judges judgment quoted in the report of Yeoman Credit Ltd v Apps
([1961] 2 All ER at p 290) that his assessment of 100 damages was on the hypothetical basis that the agreement would continue
in force and the hirer would complete the purchase. I respectfully agree with the mode of assessment on that footing and would
only add (as I have already pointed out) that he would also be entitled to damages for loss of use while the vehicle was
undergoing repair. Then the learned lord justice went on to point out that subsequently the hirer rejected the car and, in the
circumstances of that case, held he was entitled to do so, notwithstanding that he had kept it for five months. We are not
concerned with that question. But having then reached that conclusion so that the hirer lost the right to exercise the option to
purchase, the 443 learned lord justice did not consider the impact of these altered circumstances on the assessment of damages,
but awarded him 100 damages. I cannot find any principle of law laid down in that conclusion. It is, indeed, clear that the
assessment of damages where the lender is in breach will vary according to circumstances and will depend on whether (i) the
hirer is entitled to reject the vehicle and does so, or (ii) being entitled to reject he does not do so but elects to affirm the contract
and ultimately to purchase, or (iii) the hiring is terminated by notice pursuant to some provision in the contract.
In this case the hirer expended nothing on repairs, so that he suffered no loss on that heading as the finance company have
repossessed the car. His measure of damages is his loss on the transaction, and, taking a robust view of that loss, I would call it
the sum of 132 4s already mentioned.
With regard to the sum of 11 required to replace the defective tyres with new ones, I would think that sum irrecoverable by
the hirer for two reasons. First, the defect must have been apparent to him, and he cannot have relied on the implied obligation of
fitness, of which stipulation alone the failure to have roadworthy tyres was a breach, as by inference, the learned county court
judge, in my judgment, correctly held. Secondly, liability for breach of that obligation (not being a fundamental term) was
probably excluded by cl 5; but, as my Lord has said, this matter was really treated in argument as de minimis, so that I do not
dissent from the order proposed.

ORMEROD LJ. The first question is whether there has been a breach of a fundamental term of the contract of hiring, and, if so,
its effect on the contract. The learned county court judge has found that there was to be implied into the contract a term that the
car was to be fit for the purpose for which it was hired and that there was a breach of that term. The car was not fit to be used on
the road and therefore there was a fundamental breach entitling the defendant to repudiate or rescind. These findings involve
questions of mixed law and fact, and this court is entitled, so far as they are questions of law, to consider them. In my judgment,
the findings were right. There was an implied term that the car would be fit for the purpose for which it was hired. The car was
not so fit and there was, therefore, a breach of the implied term, which was a fundamental one. The breach therefore entitled the
hirer to repudiate; but the judge has found, and again I agree with him, that the hirer did not repudiate the contract. All he did
was to ask for time to do the repairs, a course of conduct only consistent with treating the contract as in being.
The case must be considered, therefore, on the basis that, although there was a breach by the finance company of
fundamental term of the contract, the hirer chose to treat the contract as in being. It was terminated later by the finance company,
under the terms of the contract, because the hirer had made default in his payments.
It has been submitted on behalf of the finance company that if the contract was still in being in spite of the breach of the
fundamental term, then cl 5 of the contract, the exclusion clause, applied and the hirer could have no claim against the finance
company in respect of the breach. It is necessary, therefore, to determine, first of all, whether the hirer has a claim for damages,
irrespective of cl 5, and irrespective of the fact that he has not repudiated the contract. There is no direct authority on this
question, which has caused me some difficulty. It seems reasonable, on the face of it, that if a hirer chooses that a contract should
continue in being he should be bound by all its terms, including the exclusion clause; but in Karsales (Harrow) Ltd v Wallis
([1956] 2 All ER 866 at pp 868, 869) Denning LJ said:

The plaintiffs say that there can be no such implication in this case in view of the express terms of cl. 3(g). The law
about exempting clauses, however, has been much developed in recent years, at any rate about printed 444 exempting
clauses, which so often pass unread. Notwithstanding earlier cases which might suggest the contrary, it is now settled that
exempting clauses of this kind, no matter how widely they are expressed, only avail the party when he is carrying out his
contract in its essential respects. He is not allowed to use them as a cover for misconduct or indifference or to enable him
to turn a blind eye to his obligations. They do not avail him when he is guilty of a breach which goes to the root of the
contract. It is necessary to look at the contract apart from the exempting clauses and see what are the terms, express or
implied, which impose an obligation on the party. If he has been guilty of a breach of those obligations in a respect which
goes to the very root of the contract, he cannot rely on the exempting clauses. I would refer in this regard to what was said
by ROCHE, J., in Pinnock Bros. v. Lewis & Peat Ltd. ([1923] 1 KB 690 at p 695), to the judgments of DEVLIN, J., in
Alexander v. Railway Executive, and Smeaton Hanscomb & Co., Ltd. v. Sassoon I. Setty Son & Co., and a recent case in
this court, J. Spurling, Ltd. v. Bradshaw, and the cases there mentioned. The principle is sometimes said to be that the
party cannot rely on an exempting clause when he delivers something different in kind from that contracted for, or has
broken a fundamental term or a fundamental contractual obligation. However, I think that these are all comprehended
by the general principle that a breach that goes to the root of the contract disentitles the party from relying on the
exempting clause. In the present case the lender was in breach of the implied obligation that I have mentioned. When the
defendant inspected the car prior to signing the application form the car was in excellent condition and would go: whereas
the car which was subsequently delivered to him was no doubt the same car but it was in a deplorable state and would not
go. That breach went to the root of the contract and disentitles the lender from relying on the exempting clause.

This case is not directly in point, as the issue was whether there had been a breach of a fundamental term, and we are considering
what remedies remain to a hirer who has chosen to treat the contract as still in being in spite of such a breach, but the judgment of
Denning LJ gives support to the view that in the present case the hirer has not foregone his claim for damages, even although he
has chosen not to repudiate the contract of hiring. There is further support in the decisions in two Scottish cases considered by
the House of Lords, Pollock & Co v Macrae, and Wallis, Son and Wells v Pratt and Haynes although in neither case is the point
now in question specifically raised.
I have come to the conclusion that in spite of the fact that the hirer continued to treat the contract as in being after he had
discovered the breach of the fundamental term, he should not be in any worse position with regard to his claim for damages, and
in those circumstances, even if the proper construction of cl 5 be that it would exclude the hirers claim, it does not apply. It is
unnecessary, therefore, to decide what the proper construction of cl 5 should be, and I do not so decide, but I should say that as at
present advised, I lean towards the construction contended for by counsel for the hirer. If it is intended in a contract to exclude
certain contingencies, it should be done in clear and unambiguous terms. I confess that I find the terms e of cl 5 anything but
clear and unambiguous, although the hirer has signed a statement that he has understood them. It would seem, therefore, that
both parties are entitled to damages, and the remaining questions are the measure of damages in each case. The learned judge
held that the finance company were entitled to 97 12s damages, this being 127 12s claimed by the finance company as the
difference between what they would have got under the agreement if it had not been determined, and the 250 obtained 445 on
resale after possession. From this the judge deducted the further sum of 30 for accelerated payment. The hirer submits that
these damages are assessed on the wrong basis, and that the only sum for which he is liable is the total of instalments in arrear at
the date of the termination of the contract, a sum of 47 4s. The submission was based on the decision of this court in
Financings, Ltd v Baldock and counsel for the finance company agreed that it would be difficult to distinguish the present case
from that one, but submitted that, in view of the two earlier decisions of Yeoman Credit Ltd v Waragowski and Overstone Ltd v
Shipway, the decision in Financings Ltd v Baldock should not be follwed. Lord Denning MR in his judgment in Financings Ltd
v Baldock distinguished the two earlier authorities in this way ([1963] 1 All ER at p 446):
________________________________________
e See ante, p 437, letters g to i, and p 438, letter b

So much for the matter of principle. But now for the authorities. They are Yeoman Credit, Ltd. v. Waragowski and
Overstone, Ltd. v. Shipway and a passage at the end of my own speech in Bridge v. Campbell Discount Co. Ltd. [[1962] 1
All ER 385 at p 401; [1962] AC 600 at p 632]. When those cases are examined, it will be found that there was not only a
failure to pay past rentals, but there was a repudiation by the hirer of his obligation to pay future rentals. Thus in Yeoman
Credit, Ltd. v. Waragwoski the hirer, by his long non-payment for six months, evinced an intention no longer to be bound
by his contract. In Overstone, Ltd. v. Shipway, the hirer had written a letter saying: I cannot afford to carry on paying for
this car. In Bridge v. Campbell Discount Co. Ltd. he had written saying: I am very sorry, but I will not be able to make
any more payments. Once the hirer repudiated his liability for future rentals, the owners were entitled to treat the
repudiation as itself a breach going to the root of the contract; and, on accepting it as such, they were entitled to regard the
hiring as at an end and retake the vehicle.

The principle in Financings Ltd v Baldock should be applied here, and I would allow the hirers appeal by reducing the damages
for which he is liable to 47 4s.
The learned county court judge determined that the finance company were liable to pay 81 to the hirer in respect of their
breach of the hiring contract, regarding himself as bound to follow the decision in Yeoman Credit, Ltd v Apps. I confess that this
question has caused me some little difficulty, but, as Upjohn LJ has just said, decided cases are only authorities for the principles
of law that they decide. Like him I find nothing in the judgments in Yeoman Credit Ltd v Apps laying down any principle of
damages differing from that laid down in Hadley v Baxendale. I agree with my brethren as to the sum payable by the finance
company.
Appeal allowed; damages awarded to the finance company reduced to 47 4s, and damages awarded to the hirer increased to
143 4s; cross-appeal dismissed.

Solicitors: Ormerod, Morris & Dumont, Croydon (for the finance company); Peter Kingshill, Norwood (for the hirer).

Henry Summerfield Esq Barrister.


446
[1963] 2 All ER 447

Wilson v Wilson
FAMILY; Ancillary Finance and Property: LAND; Property Rights

COURT OF APPEAL
ORMEROD, DONOVAN AND RUSSELL LJJ
18, 21 MARCH, 9 APRIL 1963

Husband and Wife Property Matrimonial home Discretion of court Conveyance to spouses jointly Trust for sale and
trust of proceeds of sale etc for grantees as joint tenants Mortgage repayable by instalments Separation of spouses
Matrimonial home sold Mortgage repayments made by husband before and after separation Entitlement, as between spouses,
to proceeds of sale of house Married Womens Property Act, 1882 (45 & 46 Vict c 75), s 17.

By a conveyance on sale dated 21 February 1957, a freehold dwelling-house was conveyed to a husband and wife as joint tenants
subject to a building society mortgage upon trust to sell the same with power to postpone the sale thereof and [to] hold the net
proceeds of sale and other money applicable as capital and the net rents and profits thereof until sale upon trust for themselves as
joint tenants. The purchase price of the house had been provided as to 750 by the husband from his own resources and family
loans, and as to the balance of 1,600 by the mortgage, both husband and wife being mortgagors, but the mortgage payments to
the building society were in fact made by the husband. The parties both resided in the house as their matrimonial home until, in
July, 1959, the wife left the husband. On 31 March 1961, the house was sold. Before the sale the husband had repaid (by
instalments paid both before and after the separation) about 400 under the mortgage. After payment of the balance of the
mortgage and all expenses, the net proceeds of sale amounted to 1,001. On a summons under the Married Womens Property
Act, 1882, s 17,

Held If, at the time of a conveyance of property that passed beneficial interests therein to husband and wife, it was clearly
shown what interest was conferred on each and that each was to have that interest whatever circumstances might arise in future,
then either s 17 of the Married Womens Property Act, 1882, did not confer any discretion to disturb the interests so taken or, if it
did (per Donovan LJ at p 451, letter f, post), the discretion should be exercised to effect the intention (see p 450, letter b, p 451,
letter g, and p 452, letter c, post); in the present case the trust of the proceeds of sale declared in the conveyance showed that the
parties had determined at the outset that those proceeds should be shared equally in any event, and accordingly the wife was
entitled to an equal share, subject, however, to an adjustment, in respect of mortgage repayments made by the husband after the
separation in July, 1959, by deducting one half of those payments from the wifes share of the proceeds and adding these
deductions to the husbands share (see p 450, letters e and g, p 451, letter i, to p 452, letter a, and p 454, letters d and e, post).
Hine v Hine ([1962] 3 All ER 345) explained and dictum of Lord Denning MR at p 347, letter e, criticised.
Dictum of Romer LJ in Cobb v Cobb ([1955] 2 All ER at p 700) applied.
Appeal allowed.

Notes
If a decree of divorce or nullity of marriage has been pronounced (as was the case here), it is possible to approach the trusts of
land purchased and conveyed to spouses as purchasers on trusts for themselves as joint tenants, on the basis that there is a
settlement which the court has power to vary under s 25 of the Matrimonial Causes Act, 1950; see Brown v Brown ([1959] 2 All
ER 266). On such an application the question of responsibility for the break-down of the marriage becomes relevant, but it is not
relevant on an application under s 17 of the Married Womens Property Act, 1882 (see p 454, letters a and b, post).
447
As to the exercise of jurisdiction in summary proceedings between husband and wife as to property, see 19 Halsburys Laws
(3rd Edn) 900, 901, para 1492; and for cases on the subject, see 27 Digest (Repl) 263265, 21192133.
For the Married Womens Property Act, 1882, s 17, see 11 Halsburys Statutes (2nd Edn) 804.

Cases referred to in judgments


Brown v Brown [1959] 2 All ER 266, [1959] P 86, [1959] 2 WLR 776, 3rd Digest Supp.
Cobb v Cobb [1955] 2 All ER 696, [1955] 1 WLR 731, 3rd Digest Supp.
Hine v Hine [1962] 3 All ER 345, [1962] 1 WLR 1124.
Jones v Maynard [1951] 1 All ER 802, [1951] Ch 572, 27 Digest (Repl) 152, 1112.
Kelner v Kelner [1939] 3 All ER 957, [1939] P 411, 108 LJP 138, 27 Digest (Repl) 657, 6183.
Rimmer v Rimmer [1952] 2 All ER 863, [1953] 1 QB 63, 3rd Digest Supp.
Short v Short [1960] 3 All ER 6, [1960] 1 WLR 833, 3rd Digest Supp.
Smith v Smith [1945] 1 All ER 584, 114 LJP 30, 173 LT 8, 27 Digest (Repl) 647, 6101.

Appeal
Doris Mary Wilson appealed against the decision of Mr Registrar Townley Millers awarding her only 45 on her claim under the
Married Womens Property Act, 1882, s 17, against her husband, Leslie Walter Wilson, that she was entitled to a half share in
1,001, the net proceeds of sale of their house. The purchase price of the house, which was bought in February, 1957, more than
eight years after the marriage, was provided as to 750 by the husband and by some family loans and as to 1,600 by a building
society on mortgage, both spouses being mortgagors. The husband paid off instalments of the mortgage amounting in all to about
400.

A B Ewbank for the appellant wife.


W A B Forbes for the respondent husband.
Cur adv vult

9 April 1963. The following judgments were delivered.

ORMEROD LJ. This is an appeal from a decision of Mr Registrar Townley Millers given on 9 October 1962. The learned
registrar was asked to decide an issue under s 17 of the Married Womens Property Act, 1882, as to the disposal of a fund of
1,001 deposited in a bank as the result of the sale of a bungalow in Colchester known as Charnwood, which was the
matrimonial home of the parties prior to the break up of their marriage. The wife claimed that she was entitled to a half share of
the money but was awarded 45 only, and it is against that award that she appeals.
The parties were married on 4 September 1948, and there are two children, a girl and a boy, born in 1950 and 1953
respectively. For some years after the marriage the parties lived with the husbands parents, but early in 1957 Charnwood was
bought and conveyed to them in their joint names by a deed dated 21 February 1957. The parties lived at Charnwood with
their children until July, 1959, when the wife left the husband, alleging that he had treated her with cruelty. She filed a petition
for divorce on the ground of cruelty on 3 February 1960, and in December, 1962, a decree nisi was granted to her. Charnwood
was sold on 31 March 1961, and the proceeds were held jointly by the solicitors to the parties. They are now held by a bank and
their proper distribution is the subject of the appeal. The application was made under s 17 of the Married Womens Property Act,
1882, which states that in any question between husband and wife as to the title to or possession of property either party may
apply to the court and the court may make such order with respect to the property in dispute as it thinks fit.
448
The conveyance of 21 February 1957, was made between the vendor of the first part, the mortgagees of the second part, and
the husband and wife, thereinafter called the purchasers, of the third part. After various recitals, the deed witnessed as follows:

The purchasers hereby declare as follows (a) the purchasers shall hold the said property upon trust to sell the same
with power to postpone the sale thereof and shall hold the net proceeds of sale and other money applicable as capital and
the net rents and profits thereof until sale upon trust for themselves as joint tenants.

The position in law appears to be sufficiently defined. I have referred to the relevant part of s 17, which is the statutory provision
governing the matter. The question has arisen, in this and in other cases, whether the words of the section are sufficiently wide to
give the court power to make an order irrespective of any rights which may have accrued to the parties, if in the opinion of the
court it is fair and just to do so. A number of authorities has been cited to us, but I think it will be sufficient to refer to two of
them only. They are Cobb v Cobb and Hine v Hine. I would refer first of all to a passage from the judgment of Romer LJ, in
Cobb v Cobb. He said ([1955] 2 All ER at p 700):

I know of no power that the court has under s. 17 to vary agreed or established titles to property. It has power to
ascertain the respective rights of husband and wife to disputed property and frequently has to do so on very little material;
but where, as here, the original rights to property are established by the evidence and those rights have not been varied by
subsequent agreement, the court cannot in my opinion under s. 17 vary those rights merely because it thinks that, in the
light of subsequent events, the original agreement was unfair.

In Hine v Hine Lord Denning MR, said ([1962] 3 All ER at p 347):

It seems to me that the jurisdiction of the court over family assets under s. 17 is entirely discretionary. Its discretion
transcends all rights, legal or equitable, and enables the court to make such order as it thinks fit. This means, as I
understand it, that the court is entitled to make such order as appears to be fair and just in all the circumstances of the
case.

Later Lord Denning said this ([1962] 3 All ER at p 347):

Two principles have, however, emerged in exercising this discretion. The first is that when it can clearly be seen that
the parties intended that one piece of property or one amount of money should belong to one or the other in any event, that
intention should prevail. The second principle only applies where no such intention appears. I venture to state the
principle in the words which I used in Rimmer v. Rimmer [[1952] 2 All ER 863 at p 869; [1953] 1 QB 63 at p 74].

Lord Denning then cited a passage from his judgment in Rimmer v Rimmer ([1952] 2 All ER 863 at p 869; [1953] 1 QB 63 at p
74), which provided for the exercise of the discretion of the court if the circumstances warranted it. Pearson LJ in his judgment
in the same case, said this ([1962] 3 All ER at p 350):

In my judgment, however, the fact that the husband and wife took the property in joint tenancy does not necessarily
mean that the husband should have a half interest in the proceeds of the sale now in contemplation. The parties agreed,
expressly or by implication from the creation of the joint tenancy, that the house should be the matrimonial home and
should belong to both of them (technically to each of them in its entirety) and that, on the death of one, it would belong to
the other by right of survivorship. They did not, however, make any agreement, or have any common intention, 449what
should happen in the event of the marriage breaking up and the property then being sold. That event was outside the
contemplation of the parties. The proper division of the proceeds of sale in that event is left to be decided by the court in
this application under s. 17.

The principle appears to be that, if no clear intention is to be seen that at the time of the conveyance the rights created by the
conveyance are to continue in any event, then on the break up of the marriage it would be necessary under s 17 to determine those
rights. If, however, the intention shown or to be inferred was that the rights then passing to the parties respectively should
continue in any event, then s 17 would not give the court any discretion to disturb them. It is necessary, therefore, to examine the
facts from that point of view. The husband has said and the wife has agreed that one of the factors, although not necessarily the
dominant factor, in the decision to put the house in the names of husband and wife as joint tenants was that the payment of estate
duty would be avoided in the event of the death of one of them. That on the face of it would appear to indicate that there was a
real transfer of those rights to the spouse in question, but it is doubtful whether that is enough having regard to the decision in
Hine v Hine, where one of the considerations was certainly the avoidance of estate duty. The question remains, however, whether
in the particular circumstances of this case and having regard particularly to the provisions of the conveyance, the inference
should be drawn that the rights which were being transferred to the wife were to be hers whatever should happen. This is not a
question of presumption. The conveyance and the terms of the declaration are clear. The only question is whether there can be
implied from that declaration a transfer to the wife, not only for the duration of the marriage or whilst the house shall be a joint
matrimonial home, but in any event. This has not been an easy question to decide and, indeed, it is matter on which courts may
take different views, but in my judgment the words of the declaration contained in the conveyance leave little doubt that the
husband was providing for his wife, whatever circumstances should arise in the future. The purchasers declared that they should
hold the net proceeds of the sale as joint tenants. There seems to be no need for such a declaration unless the intention is that the
wifes interest should continue in any event. The learned registrar in coming to his decision felt that he was bound by the
judgments in Hine v Hine. This means, as I understand it, that he was of the view that the proper inference to be drawn from the
facts was that any rights created should subsist only so long as the house in question was the matrimonial home of the parties. I
have come to a different view as to the inference to be drawn from the conveyance and, therefore, to a different view as to the
final destination of the moneys which are the balance of the proceeds of sale. Since preparing this judgment, I have had an
opportunity of reading the one about to be delivered by Russell LJ and agree that the husband should be given credit for any
mortgage payments made by him since the break up of the marriage. Subject to that, I would allow the appeal and divide the
money equally between husband and wife.

DONOVAN LJ. In Hine v Hine s 17 of the Married Womens Property Act, 1882, was construed by the Court of Appeal as
conferring an unfettered discretion on the court in cases coming within it. This discretion, said Lord Denning MR ([1962] 3 All
ER at p 347) transcended all rights, legal or equitable, and enabled the court to make such order as it thought fit, that is to say,
such order as appeared to be fair and just in all the circumstances. With Lord Dennings judgment I agreed ([1962] 3 All ER at p
348). Pearson LJ took, I think, the same view of the construction of the section, saying ([1962] 3 All ER at p 350) that the fact
that the husband and wife in that case took the property in joint tenancy did not necessarily mean that the husband should have a
half share in the proceeds of sale once the marriage had 450 broken down. That event was outside the contemplation of the
parties. The proper division of the proceeds of sale, therefore, had to be decided by the court once an application was made under
s 17; and the court had to do this by attributing to the parties a reasonable intention at the time that the transaction took place.
In Rimmer v Rimmer Romer LJ had expressed, I think, a similar view when he said ([1952] 2 All ER at p 870; [1953] 1 QB
at p 76):

Cases between husband and wife ought not to be governed by the same strict considerations, both at law and equity, as
are commonly applied to the ascertainment of the respective rights of strangers when each of them contributes to the
purchase price of property.

This again was said in relation to the construction of s 17. But in Cobb v Cobb ([1955] 2 All ER 696 at p 700) the same learned
lord justice said:

I know of no power that the court has under s. 17 to vary agreed or established titles to property. It has power to
ascertain the respective rights of husband and wife to disputed property and frequently has to do so on very little material;
but where, as here, the original rights to property are established by the evidence and those rights have not been varied by
subsequent agreement, the court cannot in my opinion under s. 17 vary those rights merely because it thinks that, in the
light of subsequent events, the original agreement was unfair.

This statement has given rise to some difficulty in the past, but the decision in Hine v Hine ([1962] 3 All ER 345 at p 347) was
arrived at in the full face of it, and in my opinion is not really in conflict with it. For it is to be noticed that in Hine v Hine
([1962] 3 All ER 345 at p 347) Lord Denning followed what he said about the absolute discretion conferred by s 17 by these
words ([1962] 3 All ER 345 at p 347):

Two principles have, however, emerged in exercising this discretion. The first is that, when it can clearly be seen that
the parties intended that one piece of property or one amount of money should belong to one or the other in any event, that
intention should prevail.

Whether one treats this as a limitation on the discretion under s 17, or whether one treats it as defining the manner in which that
discretion ought normally to be exercised in that particular kind of case, is not of prime importance. I think that the latter view is
right. Accordingly, if Romer LJs dictum regarding established rights ([1955] 2 All ER 696 at p 700) simply means (as I think it
does) that when the parties have agreed how the property should be owned or shared, come what may, those rights will not be
interfered with under s 17, then there is no dissonance between what he said in Cobb v Cobb ([1955] 2 All ER 696 at p 700) and
what this court said in Hine v Hine ([1962] 3 All ER 345 at p 347).
The position in the present case is as follows. By the conveyance the husband and wife, who were the purchasers
thereunder, were to hold the property on trust to sell the same with power to postpone the sale thereof and to hold the net
proceeds of sale and other money applicable as capital and the net rents and profits thereof until sale on trust for themselves as
joint tenants. Thus the entire beneficial interest in the property was declared at the outset. While it was the matrimonial home
the spouses owned it as joint tenants. If it should be sold they were to hold the net proceeds of sale as joint tenants, which I
construe as meaning that on a division of these proceeds, the division was to be in equal shares. The house might be sold in the
course of the marriage or on the break up of the marriage. The trusts declared are wide enough to cover both events and no
distinction is made between them. I have some doubt whether the break up of the marriage was really contemplated, but in view
of the express and unconditional language of the trust in relation to the proceeds of sale, I have come, with 451 some hesitation,
to the view that this case should be regarded as one where from the outset it was agreed between the parties that, come what may,
the proceeds of sale of the house should be shared equally. Thus the case comes within the principle enunciated by Lord Denning
MR in Hine v Hine ([1962] 3 All ER 345 at p 347) in the passage which I have quoted. I, therefore, think that this appeal should
be allowed and that the wife should be declared to be an equal owner with her husband of the proceeds of sale, with, however, the
adjustment in favour of the husband to be indicated by Russell LJ.
Finally, I would observe that in Hine v Hine ([1962] 3 All ER 345 at p 347) the court did not, as I recollect the matter, have
the conveyance before it, nor any declaration of trust in relation to the proceeds of sale, such as we have in this case. The court
was simply told that the parties had been joint tenants of the house.

RUSSELL LJ. I do not propose to add to the many attempts in other cases to generalise as to the scope of the jurisdiction under
s 17 of the Married Womens Property Act, 1882, except in a negative sense. Where husband and wife have made clear in a
formal manner what are the nature and quantum of their beneficial interests in the property in question, that section does not in
my judgment entitle the court to substitute other beneficial interests, or to deprive one in favour of the other of that which is
clearly the property of the one. There are many statements in the cases to that effect. It would indeed be surprising if that were
the effect of a section which is primarily designed to afford a summary procedure for the disentangling and settling of property
disputes between spousesand not necessarily spouses whose marriage has broken down; and I observe that the predecessor of s
17 (s 9 of the Married Womens Property Act, 1870) was confined to property made the separate property of the wife by the Act
of 1870. As was said by Sir Boyd Merriman P, in Kelner v Kelner ([1939] 3 All ER 957 at p 958; [1939] P 411 at p 412):

I emphasise that this is an application under the Married Womens Property Act, 1882, s. 17. Under that Act, after
whatever inquiries the court may think it necessary to make, the question remains: whose property is this? Is it the
husbands or is it the wifes, or does it belong to them jointly? It differs by the whole breadth of heaven from the procedure
for variation of settlement which can be invoked on dissolution of marriage.
For my part I think that some of the post-war applications of the section have led to difficulties and uncertainties which justify a
consideration by the House of Lords of its true scope.
In the present case the conveyance was to the husband and wife as purchasers as joint tenants on trust for sale. They then
proceeded in the same document expressly to declare what were to be their beneficial interests in the property and the proceeds of
any sale thereofnamely, that they were to be beneficial joint tenants. As I read the judgment below and the affidavits, there was
no evidence to justify the conclusion that this declaration of beneficial interests was in any way due to mistake or a
misunderstanding of instructions. The fact that the purchase price was produced as to 750 by the husband out of his own
resources and by family loans, and the balance of 1,600 by a building society mortgage under which both husband and wife
were mortgagors, and that the husband paid off instalments totalling 400 odd is entirely consistent with the beneficial trust
declared by them.
Suppose in those circumstances that the beneficial trust expressly declared of the proceeds of sale had been for the husband
and wife in equal undivided shares, there would in my judgment have been no jurisdiction under the section to make an order in a
different sense, any more than there would have been such jurisdiction in an ordinary action by the wife claiming half the net
proceeds 452 of sale. (I observe that in an ordinary action involving disputes as to property between husband and wife there is
scope for a broad view to be taken in a proper case: eg, Jones v Maynard, approved by this court in Rimmer v Rimmer, was not a
s 17 case: there the intricacies of operations on a joint account were solved on a half and half basis.) Can the position be
different if the declared beneficial trusts are for a joint tenancy? In my judgment, no. I may point out that this question does not
involve resort to any presumption of gift: the beneficial trusts are express, and the presumption of gift could be relevant here
only to half the mortgage instalments paid by the husband, and as to those paid by him while they were living together, there can
be no doubt that the husband intended the wife to benefit from them.
The beneficial joint tenancy conferred on the wife an equitable right or title in respect of the property and the proceeds of
sale whenever sold, under which she was entitled to the whole together with the husband until severance inter vivos by either, and
to the whole alone on survival before such severance, and was further entitled at any time by unilateral act to sever the joint
tenancy and confer on herself and on him a right to an undivided one half share. The conferring of the beneficial joint tenancy
necessarily and clearly involved these beneficial rights and interests. Had either taken steps by way of severance (for example,
by assigning his or her beneficial interest by way of charge or perhaps by written notice of desire to sever under the Law of
Property Act, 1925, s 36(2)) while they were living together, or after the separation before the house was sold, or after the house
was sold before the s 17 summons, the court would in my judgment plainly have been in the same situation as if the original
beneficial trusts had been in undivided shares. I mention this only to stress the fact that in my judgment there is for present
purposes no distinction to be drawn between a clearly declared beneficial joint tenancy and a clearly declared beneficial interest
in undivided shares. Each is a clearly defined and formally agreed and declared beneficial title which, in my judgment, the court
has no more right to override under the summary procedure of s 17 than it has in an action.
The registrar took the view that the decision of this court in Hine v Hine being binding on him (as it is on us) must be
followed, and that following it meant that the husband was entitled to have back out of the proceeds of sale the money he put in,
which would exhaust the net proceeds of sale. He also allotted some 45 to the wife, but for present purposes this can be
ignored. We do not know the exact form of the conveyance in Hine v Hine except that the legal estate was conveyed to husband
and wife as joint tenants. We have ascertained that the conveyance was not in fact before this court at all. The court therefore
was not considering the case before us of an express trust with the beneficial interests formally declared at the time of the
purchase by the husband and wife: the statement in one judgment ([1962] 3 All ER at p 348, letter d) that the holding as joint
tenants gave to the husband an equal undivided share was a slip of the tongue. For that reason the registrar was in error in
considering that he was bound to follow Hine v Hine, and we are not bound by it.
For my part I venture to think that some of the observations which were made in the judgments in Hine v Hine require to be
read with careful regard to their context. I do not think that it would be correct to say that the court has power under s 17 to
transcend all rights, legal or equitable, without at least excepting a case such as this where beneficial trusts (whether joint tenancy
or undivided shares) are clearly and precisely and expressly declared and defined in a form which does not (it seems to me)
permit of any qualification by reference to a breakdown in the marriage.
It would, I think, be unfortunate if it were otherwise, for it would mean in the present case (if the decree granted to the wife
stands) that the husband by treating 453 his wife with cruelty would have been able to reclaim the beneficial interest which he
undoubtedly conferred on her. Were it, on the other hand, the wife who was responsible for the break up of the marriage, it would
seem that the husband might apply to vary the trusts of the settlement constituted by the conveyance by excluding the wifes
beneficial interest: see Smith v Smith and Brown v Brown. In making those comments it must not be thought that they are in any
way a reason for my decision. Authority establishes that the responsibility for the break up of a marriage is not a relevant factor
in a decision under s 17; and in the present case the husband is in fact in process of seeking to appeal from the decree granted to
the wife and to adduce fresh evidence on that appeal.
It was represented that those cases (such as Short v Short) in which it has been held that s 17 empowers the court to refuse or
adjourn an application by the husband for possession of the matrimonial home, admittedly his property in law and equity,
demonstrate a jurisdiction to override clear legal or equitable titles. But it seems to me that the legal and equitable title of a
husband is not absolute but is vis--vis his wife limited, in that in general law he has not an absolute right to eject the wife. The
refusal to order possession under s 17 is, therefore, not the overriding of an unassailable title, but the recognition of a defect in the
title.
In the result in my judgment the appeal should be allowed and it should be declared that the wife is entitled to half of the net
proceeds of sale of the house. I think, however, that there must be some adjustment in respect of mortgage instalments paid by
the husband between the time when the wife left the matrimonial home in July, 1959, and the sale of the house in March, 1961: I
do not think that the presumption of gift can continue to apply after the separation, nor consequently that the husband can be
taken to have given to the wife the benefit of half these post separation payments; he is in respect of these payments in the
ordinary position of a joint mortgagor redeeming a mortgage and entitled to contribution from the co-mortgagor in proportion to
their interests, and from her half of the proceeds of sale, half of such payments should be deducted and added to his half. If
necessary, there must be an inquiry to ascertain the amount.

Appeal allowed: Order accordingly. Leave to appeal to the House of Lords.

Solicitors: Eric P Hanney (for the wife); Bischoff & Co (for the husband).

Henry Summerfield Esq Barrister.


454
[1963] 2 All ER 455

Sharpe v E T Sweeting & Son Ltd


TORTS; Negligence

YORK ASSIZES
NIELD J
25, 26 FEBRUARY, 7 MARCH 1963

Independent contractor Negligence Builder Fault in construction by builders of house on local authoritys estate Personal
injury sustained by tenants wife Whether principle inDonoghue(orMcAlister)vStevenson applicable.

Negligence Duty to take care Proximity of relation Builders of house for occupation by local authority tenant Whether
principle inDonoghue(orMcAlister) vStevenson applicable.

The defendants were builders who, under an agreement with a local authority in 1950, constructed a number of houses on an
estate belonging to the authority. In erecting, to the designs of the local authority, a concrete canopy over the front door of one of
the houses of which the plaintiffs husband became tenant in 1951, the builders negligently committed a fault of construction in
circumstances in which they could not reasonably have expected that there would be any intermediate examination likely to
reveal the defect. In 1959 the canopy collapsed owing to this defect and injured the plaintiff. In an action by the plaintiff against
the defendants for damages for personal injuries caused by the defendants negligence it was common ground that the local
authority, as the landlord and free-holder, was not liable to the plaintiff, and it was found that the defendants had been
independent contractors, not agents of the local authority, with the consequence that the landlords immunity did not extend to
them.

Held The defendants constructed the canopy expecting the doorway to be used by the occupiers of the house, and there was
sufficient proximity of relation between the plaintiff and the defendants for the latter to owe to her a duty of care in accordance
with the principle of Donoghue (or McAlister) v Stevenson, which was applicable notwithstanding that the canopy, being part of
the house, partook of the nature of realty; therefore, as the fall of the canopy caused the plaintiffs injuries and was due to the
defendants want of reasonable care, they were liable to her in damages (see p 459, letter h, p 460, letter d, p 463, letter e and p
464, letter e, post).
Donoghue (or McAlister) v Stevenson ([1932] All ER Rep 1); Haseldine v C A Daw & Son Ltd ([1941] 3 All ER 156) and
Gallagher v McDowell Ltd ([1961] NI 26) applied.
Per Curiam: the fact that the owner of land is also the builder does not remove the owners immunity a, but when the builder
is not the owner he enjoys no such immunity (see p 463, letter e, post).
________________________________________
a The principle of immunity is stated at p 461, letter f, post

Otto and Otto v Bolton and Norris ([1936] 1 All ER 960) and Bottomley v Bannister ([1931] All ER Rep 99) considered.

Notes
As to persons answerable for negligence in relation to work, see 28 Halsburys Laws (3rd Edn) 61, para 58; and for cases on the
subject, see 36 Digest (Repl) 8088, 428470.
As to landlords immunity from liability, see 23 Halsburys Laws (3rd Edn) 572, para 1246; and for cases on the subject, see
31 Digest (Repl) 194197, 32523302.

Cases referred to in judgment


Billings (AC) & Sons Ltd v Riden [1957] 3 All ER 1, [1958] AC 240, [1957] 3 WLR 496, 3rd Digest Supp.
Bottomley v Bannister [1931] All ER Rep 99, [1932] 1 KB 458, 101 LJKB 46, 146 LT 68, 36 Digest (Repl) 80, 429.
Cavalier v Pope [1906] AC 428, 75 LJKB 609, 95 LT 65, 12 Digest (Repl) 52, 283.
Deyong v Shenburn [1946] 1 All ER 226, [1946] KB 227, 115 LJKB 262, 174 LT 129, 2nd Digest Supp.
455
Donoghue (or McAlister) v Stevenson [1932] All ER Rep 1, [1932] AC 562, 101 LJPC 119, 147 LT 281, 36 Digest (Repl) 85,
458.
Gallagher v McDowell Ltd [1961] NI 26.
Haseldine v Daw (CA) & Son Ltd [1941] 3 All ER 156, [1941] 2 KB 243, 111 LJKB 45, 165 LT 185, 31 Digest (Repl) 105, 2491.
Heaven v Pender (1883), 11 QBD 503, 52 LJQB 702, 49 LT 357, 47 JP 709, 36 Digest (Repl) 7, 10.
Herschtal v Stewart and Arden Ltd [1939] 4 All ER 123, [1940] 1 KB 155, 109 LJKB 328, 161 LT 331, 36 Digest (Repl) 82, 448.
Lane v Fox [1897] 1 QB 415, 66 LJQB 193, 76 LT 135, 45 WR 261, 31 Digest (Repl) 195, 3261.
OConnor v Swan and Edgar Ltd The Times, 6 March 1963, 107 Sol Jo 215.
Otto (M) and Otto (E) v Bolton and Norris [1936] 1 All ER 960, [1936] 2 KB 46, 105 LJKB 602, 154 LT 717, 7 Digest (Repl)
347, 46.
Travers v Gloucester Corpn [1946] 2 All ER 506, [1947] KB 71, 115 LJKB 517, 175 LT 360, 110 JP 364, 36 Digest (Repl) 83,
453.

Action
In this action Violet Robinson Sharpe, the plaintiff, claimed damages against E T Sweeting & Sons Ltd the defendants, for
personal injuries caused by the negligence of the defendants, their servants or agents. On 8 July 1959, the plaintiff was injured
when the concrete canopy over the front door of the house which she occupied together with her husband, the tenant, collapsed.
The house was one of a number of houses constructed by the defendants under an agreement made in 1950 with the Corporation
of Middlesbrough on land belonging to the corporation. The corporation were the landlords and were not parties to the action.
The facts and the material parts of the pleadings are set out in the judgment.
The cases noted belowb were cited during the argument in addition to those referred to in the judgment.
________________________________________
b Grant v Australian Knitting Mills Ltd [1935] All ER Rep 209, [1936] AC 85, Buckner v Ashby & Horner Ltd [1941] 1 KB 321, 337

S E Brodie and Christopher Robson for the plaintiff.


J F S Cobb QC for the defendants.

Cur adv vult

7 March 1963. The following judgment was delivered.


NIELD J read the following judgment. By an agreement in writing dated 2 September 1950, the defendants E T Sweeting & Son
Ltd agreed with the Corporation of Middlesbrought to build for the corporation twenty-two houses of four different types at
Saltersgill, an estate belonging to the corporation and being with other estates about to be developed for housing purposes. It was
a term of the agreement that the defendants should execute and perform the work as described in the tender, specification, bills of
quantities and drawings annexed to the agreement and to the complete satisfaction of the borough engineer for the time being of
the corporation. The twenty-two houses in question were part of a project for the erection of ninety houses on the Saltersgill
estate and the corporation had further projects for the erection of four hundred houses on adjacent and other estates in
Middlesbrough. The corporation agreed with five contractors including the defendants for the building of the houses at
Saltersgill, and they agreed with some thirty contractors for the building of houses in different parts of Middlesbrough. The
corporation also executed some building work themselves by direct labour. In connexion with all these projects the corporation
employed three clerks of works. Of the twenty-two houses which the defendants agreed to build under the agreement of 2
September 1950, four in a terrace were Nos 6, 8, 10 and 12, Gleneagles Road. These were two pairs of houses with adjoining
from doors and over each pair of front doors was a reinforced concrete canopy to the design of the corporation. The method of
constructing the canopies and placing them in position was as follows. When the 456 brick-work of the houses had reached an
appropriate level above the pairs of front doors a wooden frame or shuttering was placed in position above the pairs of front
doors and propped up. Into the frame was poured concrete to the required depth, a re-inforcement of wire mesh was then placed
on the concrete and thereafter further concrete was poured on top of the wire mesh. The wire mesh included wire of two
thicknesses, the thicker being known as 5 SWG (which means 5 Standard Wire Gauge) and the thinner as 10 SWG. The proper
method of setting the wire mesh in position involved two requirements; first, that the wire mesh should be set high up towards the
top of the concrete and, secondly, that the thicker mesh, 5 SWG, should be at right angles to the wall of the house and the thinner
mesh, 10 SWG, parallel with the wall. As appears from one of the drawings, the canopies were three feet in width; eleven inches
extended from the outer wall back towards the inside of the house, and two feet one inch protruded from the wall outwards. The
length of each canopy over the pair of doors was fifteen feet. Nowhere in the agreement of 2 September 1950, or specification or
other document annexed to it, nor in either of the two drawings designing the canopy is there indicated the manner or position in
which the wire mesh is to be placed in the concrete. This was, however, indicated in a drawing of a similar canopy which was
provided by the corporation in relation to a house of a different type from Nos 6, 8, 10 and 12, Gleneagles Road, but to be built
by the defendants. Furthermore, Mr Walker, one of the corporations clerks of works, at a time before the building of the two
pairs of houses referred to, told Mr Bland, at that time the defendants general foreman, that the wire mesh must be at the top of
the concrete.
On 22 June 1951, the plaintiffs husband went into possession as the first tenant of the house, No 12, Gleneagles Road, and
the plaintiff went with him. On 8 July 1959, at about 7.30 in the evening the plaintiff decided to go outside the house at No 12,
Gleneagles Road for some fresh air. She went to the front door and remembers no more until waking up in hospital. Her
husband on that evening saw her go to the door and then heard a crash and a shout of Tommy from her. He hurried to the front
door and saw that the canopy was lying on the path and that she was lying there too with one foot caught under the canopy. With
the help of a neighbour, the plaintiffs husband and his son lifted the canopy from the plaintiffs foot and she was taken to
hospital.
On 9 July 1959, an assistant in the architects section in the office of the Middlesbrough borough engineer, a Mr Marsh,
went to No 12, Gleneagles Road, to make an inspection. He found that the canopy had fallen and that the reasons for this were
that the wire mesh had been inserted below the centre line of the concrete and that the thicker wire, the 5 SWG, had been placed
parallel with instead of at right angles to the outer wall of the house. The house and the canopy should have had a minimum life
of sixty years. An investigation was then begun to discover the situation as to the other canopies so as to seek to avoid other
accidents. The canopy over Nos 6 and 8, Gleneagles Road, manufactured and erected by the defendants, was taken down and
found to have similar faults to those found in the canopy over Nos 10 and 12, Gleneagles Road. Other canopies made and
erected by contractors other than the defendants were inspected and found to be correctly re-inforced with wire mesh. These then
are the facts of the case.
The plaintiff in the action claims damages for personal injuries on the grounds that they were brought about by the
negligence of the defendants, their servants or agents, and it is important to notice how the claim and the defence are pleaded. In
para 3 of the amended statement of claim the particulars of negligence are these:

(1) The plaintiff will rely upon the fall of the said concrete canopy as evidence of negligence. (2) The defendants,
their servants or agents failed to re-inforce the said concrete canopy in a safe skilful or workmanlike manner, in that: (a)
the said reinforcing wire mesh was set in the said concrete 457 canopy below centre so that it served no useful purpose as
re-inforcement against tensional stresses; and (b) the reinforcing wire mesh was placed in the said concrete in such a way
that the thick gauge wire was parallel with the face of the brickwork of the said premises and the thin gauge wire was at
right angles to the said brickwork.

In the amended defence the defendants first assert, in para 1, that the amended statement of claim discloses no cause of action. It
is then said, in para 2:

The defendants will allege that the said premises were built in accordance with detailed plans and specifications
provided by the Middlesbrough Corporation, whose representative exercised close supervision and control over the
construction at every stage;

and in para 3:

The defendants admit that the concrete canopy over the front door of 12, Gleneagles Road was constructed by them on
the site of the premises during the course of the work of building construction. Save as aforesaid para. 2 of the amended
statement of claimc is not admitted.
________________________________________
c Paragraph 2 set out the alleged circumstances of the accident

The first question for determination, and it is one of fact, is what was the cause of the collapse of the canopy? This presents
no difficulty and the evidence on the matter called on behalf of the plaintiff is not challenged by the defendants. There were two
reasons for its collapse: the first that the wire mesh was placed at too low a level in the concrete, the second that the thicker
mesh, 5 SWG, was placed parallel to and not at right angles to the wall of the house. But for these defects in construction as I
find them to be, the canopy should have remained safely in position for many, perhaps sixty, years from the time of building. The
central issue between the parties here is whether or no the principles laid down in the case of Donoghue (or McAlister) v
Stevenson are applicable to the facts of the present case. It must, however, be considered whether, assuming that such principles
are applicable, the plaintiff has shown on the balance of probabilities that the defendants, owing a duty of care to the plaintiff,
failed in that duty and were by their servants or agents negligent so as to cause the injuries suffered by the plaintiff. On behalf of
the defendants it is submitted that the defendants were agents of the Corporation of Middlesbrough and could not therefore be
liable since the owners of realty are admittedly excluded from liability d; that there was no duty of care owed by the defendants to
the plaintiff since the proximity test, as it has been called, is not satisfied and that the defendants, their servants or agents, were
not negligent. Were the defendants then the agents of the Corporation of Middlesbrough, that is to say, acting under the control
of the corporation? Counsel for the defendants pointed out that in Travers v Gloucester Corpn it was conceded that the builders
were the agents of the corporation to build the houses concerned. This appears in counsels argument in that case ([1947] KB at p
76); I do not, however, think that this can guide me in the present problem. I must decide on the evidence whether the
corporation was exercising control over the work or whether the defendants were independent contractors. At the time the
corporation themselves built houses by direct labour or they agreed with contractors that the contractors should build for them,
and the defendants were competent and reliable builders. The three clerks of works employed by the corporation could not and
did not watch, supervise and control the building of the four hundred houses being built in Middlesbrough in various stages of
construction. They were there in truth to see that the corporation had their moneys worth, but not to direct the defendants
operations. I am satisfied that the defendants were not the agents of the Corporation 458 of Middlesbrough for the purpose of
building the houses in question. It is significant that in para 1 of the amended statement of claim there appears this passage:
________________________________________
d See, eg Travers v Gloucester Corpn [1946] 2 All ER at p 513; [1947 KB at p 86, citing Cavalier v Pope, [1906] AC at p 430

The said premises were built for the said corporation by the defendants who at all material times carried on business
and were employed by the said corporation as independent building contractors.

That was admitted in the defence. I take the view that on the pleadings this point, namely that the defendants were merely agents
of the corporation, is not open to the defendants. However, I find against the defendants on this point on the evidence.
Is it then shown by the plaintiff that the defendants owed her a duty, still assuming that the principles in Donoghue (or
McAlister) v Stevenson apply? The proximity test is put in Lord Atkins speech in that case in this way ([1932] All ER Rep at p
11; [1932] AC at p 580):

Who, then, in law is my neighbour? The answer seems to be persons who are so closely and directly affected by my
act that I ought reasonably to have them in contemplation as being so affected when I am directing my mind to the acts or
omissions which are called in question.

The test was further examined by Tucker J in Herschtal v Stewart and Arden Ltd when the learned judge came to the conclusion
that the mere existence of a reasonable possibility of intermediate examination was not sufficient to destroy the proximate
relationship on which liability for negligence rested. He said ([1939] 4 All ER at p 135; [1940] 1 KB at p 170):

the mere existence of the opportunity for examination is not sufficient to break the chain or destroy the proximate
relationship. I think that Donoghues case ought not to be so limited as to be confined only to the precise words used by
LORD ATKIN in the concluding part of his opinion, which I have already pointed out were not necessary for the decisions
in that particular case. I think that one has to look at the whole basis of the judgment and apply it to the facts of the
particular case e. Accordingly, I think that the defendants are liable in this case, and I think that they are liable on the
basis that they were the plaintiffs neighbours in law and were supplying this dangerous article for his user when, and in
circumstances in which, they did not and could not, reasonably have anticipated, that there would be any such intermediate
examination as would be likely to reveal a defect such as existed in this motor car.
________________________________________
e The passage which follows is in [1939] 4 All E.R. at p. 136; [1940] 1 K.B. at p. 172

Counsel for the plaintiffs in the present case points to the evidence that the pouring in of the concrete after the setting up of
the frame would take about twenty minutes and he says that that was the only opportunity for inspection. In my judgment it is
clear that the defendants, their servants or agents constructed and put up the canopy expecting the doorway to be used by the
occupier of the house, his family and visitors, and in circumstances in which they did not and could not have reasonably
anticipated that there would be any such intermediate examination as would be likely to reveal defects such as existed. Support
for the view that the defendants did owe a duty to the plaintiffs is in a limited measure to be found in the decision in A C Billings
& Sons v Riden. Again assuming that the defendants owed a duty, are they shown to have been negligent? The defendants assert
that there was no negligence and their case is pleaded, on this aspect of the matter, in para 2 of the amended defence to which I
have already referred (See p 458, letter b, ante), where it is said that the premises were built in accordance 459 with the detailed
plans and specifications provided by the Middlesbrough Corporation, whose representative exercised close supervision and
control over the construction at every stage. In argument it was submitted that, the defendants duty being to take reasonable care
they had discharged that duty by building in accordance with the corporations detailed requirements and under their supervision
and that the work of re-inforcing concrete was outside their competence as builders. I cannot accept these contentions as having
validity. As a fact, as I have already pointed out, the defendants general foreman, Mr Bland at the time in question, knew that the
wire mesh should be inserted towards the top of the concrete. He had discussed the reasons for this with Mr Walker, clerk of
works. Further than that the defendants had one drawing which showed the position in which the wire mesh should be placed in
the concrete. If the defendants, who were competent builders, regarded the corporations specifications, drawings or general
requirements as inadequately defined, their duty was to ask the corporation for further particulars. I think that no such request
was made for the reasons, first, that, as Mr Best said in his evidence, the defendants as experienced builders knew how the wire
mesh should be placed; and, secondly, that a drawing had reached them which showed them that position. The evidence satisfies
me beyond any doubt that the defendants servants were negligent in placing the wire mesh too low in the concrete and in placing
the thicker wire parallel to instead of at right angles to the outer wall of the house. It is not disputed that such negligence caused
the canopy to fall.
I now turn to consider the issue whether or not the principles laid down in Donoghue (or McAlister) v Stevenson are
applicable to the circumstances in the present case. The issue is one, according to my researches and, it seems, to those of
counsel, which has not hitherto been decided in England. The principles to which I refer are best set forth in the well known
passage from the speech of Lord Atkin, which is as follows ([1932] All ER Rep at p 11; [1932] AC at p 590):

At present I content myself with pointing out that in English law there must be and is some general conception of
relations giving rise to a duty of care, of which the particular cases found in the books are but instances. The liability for
negligence, whether you style it such or treat it as in other systems as a species of culpa, is no doubt based on a general
public sentiment of moral wrongdoing for which the offender must pay. But acts or omissions which any moral code
would censure cannot in a practical world be treated so as to give a right to every person injured by them to demand relief.
In this way rules of law arise which limit the range of complainants and the extent of their remedy. The rule that you are to
love your neighbour becomes in law: You must not injure your neighbour, and the lawyers question, Who is my
neighbour? receives a restricted reply. You must take reasonable care to avoid acts or omissions which you can reasonably
foresee would be likely to injure your neighbour.

Then Lord Atkin continues to answer the question, Who is my neighbour? to which question I have already referred f, and then
he refers ([1932] All ER Rep at p 11; [1932] AC at p 580) to the decision in Heaven v Pender. He says ([1932] All ER Rep at p
11; [1932] AC at p 581):
________________________________________
f See p 459, letter c, ante

A. L. SMITH, L.J., saysg: The decision in Heaven v. Pender was founded upon the principle that a duty to take care
did arise when the person or property of one was in such proximity to the person or property of another that, if due care
was not taken damage might be done by the one to the other. I think that this sufficiently states the truth if proximity 460
be not confined to mere physical proximity, but be used, as I think it was intended, to extend to such close and direct
relations that the act complained of directly affects a person whom the person alleged to be bound to take care would know
would be directly affected by his careless act. That this is the sense in which nearness or proximity was intended by
LORD ESHER is obvious from his own illustration in Heaven v. Pender of the application of his doctrine to the sale of
goods.
________________________________________
g In Le Lievre v Gould, [1893] 1 Q.B. 491 at p. 504.

For the defendants it is contended that to apply the principles of Donoghue (or McAlister) v Stevenson to the present case
would be to extend those principles, and that such extension cannot be justified. In this regard there were quoted the observations
of Du Parcq LJ in Deyong v Shenburnh. Du Parcq LJ said ([1946] 1 All ER at p 229; [1946] 1 KB at p 233):
________________________________________
h [1946] 1 All ER 226, particularly at p 229; [1943] 1 KB 227, particularly at p 233

There are well known words of LORD ATKIN in Donoghue v. Stevenson as to the duty towards ones neighbour and
the method of ascertaining who is ones neighbour. It has been pointed out (and this only shows the difficulty of stating a
general proposition which is not too wide) that, unless one somewhat narrows the term of the proposition as it has been
stated, one would be including in it something which the law cannot support. It is not true to say that wherever a man finds
himself in such a position that unless he does a certain act another person may suffer, or that if he does something another
will suffer, then it is his duty in the one case to be careful to do the act and in the other case to be careful not to do the act.
Any such proposition is much too wide. One has to find that there has been a breach of duty which the law recognises, and
to see what the law recognises one can only look at the decisions of the courts.

The Corporation of Middlesbrough, owners of the realty, are not parties in the proceedings for the reason that it is agreed
that the corporation as owners enjoy an immunity. This principle was explained in the case of Bottomley v Bannister, in which
Scrutton LJ said ([1931] All ER Rep at p 102; [1932] 1 KB at p 468) that in the absence of an express contract the landlord of an
unfurnished house is not liable to his tenant, or a vendor of real estate to his purchaser, for defects in the house or land rendering
it dangerous or unfit for occupation, even if he constructed the defects himself or is aware of their existence. Greer LJ in that
case, said ([1931] All ER Rep at p 106; [1932] 1 KB at p 477):

The law is stated by LORD ATKINSON in Cavalier v. Pope ([1906] A.C. 428 at p. 432.): It is well established that
no duty is, at law, cast upon a landlord not to let a house in a dangerous or dilapidated condition, and further, that if he does
let it while in such a condition, he is not thereby rendered liable in damages for injuries which may be sustained by the
tenant, his (the tenants) servants, guests, customers, or others invited by him to enter the premises by reason of this
defective condition A purchaser of the freehold is, in my judgment, in no better position than a tenant. No case was
cited to us in which a tenant or purchaser has ever recovered against a lessor or vendor, either by implied contract or in tort,
by establishing the liability of the vendor or landlord for injuries sustained through the house or its fixtures being unsafe at
the date of the sale or of the lease.

This decision was reached by the Court of Appeal before the decision in Donoghue (or McAlister) v Stevenson. However,
thereafter another case came before the court, Otto and Otto v Bolton and Norris. In that case, which was decided 461 by
Atkinson J the learned judge, referring to Cavalier v Pope and other cases, said ([1936] 1 All ER at p 966; [1936] 2 KB at p 54):

Now unless the law there laid down has clearly and plainly been declared to be wrong in Donoghue v. Stevenson, it is,
of course, binding upon me. That was a case dealing with chattels, and there is not a word in the case from beginning to
end which indicates that the law relating to the building and sale of houses is the same as that relating to manufacture and
sale of chattels. Bottomley v. Bannister is, in fact, referred to by LORD ATKIN without any relevant criticism. Clearly,
contractually, the law is different, in that on the sale of a chattel there may be, and very often is, an implied warranty of
fitness, but there is never an implied warranty of fitness in the sale of a house.

It is not easy to understand entirely the reason for what one might call the owners immunity. Counsel for the defendants urged
the view that perhaps the chief reason was that it was unfair that an owner should be called on to meet a claim made after a lapse
of years. He urged too that logically for the same reason such immunity should be enjoyed also by builders. For myself I doubt
the cogency of this reason. It is true that it might be difficult after a lapse of years to know whether a dangerous defect in a house
was due to age or interference or negligent building. But that difficulty should surely be capable of being met by applying the
general principles of the onus of proof under which the plaintiff would be required to prove that the dangerous defect was due to
negligent building and by excluding other causes. However, the rule as to the owners immunity is too deeply embedded in the
common law to be capable of disturbance by this court. None the less I would say that whilst it is contended that it is right to
disallow an extension of the principles in Donoghue (or McAlister) v Stevenson it is certainly right in my judgment to disallow
any extension of the principles relating to owners immunity. I respectfully adopt the reasoning of Lord MacDermott CJ in the
Northern Ireland case of Gallagher v McDowell Ltd, where, considering the question of the extension of the principles in
Donoghue (or McAlister) v Stevenson he said ([1961] NI at p 44):

Will its extension to building contractors cause or lead to some mischief which would justify the rejection of such an
extension? The attitude that any enlargement of the field of tortious liability is always to be regarded as a step in the right
direction is not one to be commended. Some gap between morality and law is inevitable and, if the gap is not too large,
may be for the benefit of both codes. On the other hand, the changes to be expected in a progressive society call, from time
to time, for such adjustments in the domain of legal responsibility as will promote justice and fair dealing. Those are the
terms which may mean different things in different epochs, but from the point of view of what is generally regarded as just
and fair today I see no reason why a responsibility similar to that of the manufacturers and repairers of chattels should not
fall upon the builders of houses, or why, if it does so fall, harm to trade or commerce or some other important facet of the
life of the community should be apprehended. The immunities of the landowner seem to me to strengthen rather than
weaken these conclusions.

I refer later to that decision again. Counsel for the defendants submits that the cases of Bottomley v Bannister and Otto and Otto
v Bolton and Norris 462were decided on the footing that the defendants in these cases were the builders, and not on the footing
that they were the owners. I am not able to accept this contention and in my view the contrary is the case. In Bottomley v
Bannister the whole basis of the decision rested on the fact that the defendants were owners of the house which they sold. I refer
in particular to the words of Greer LJ who said ([1931] All ER Rep at pp 106, 107; [1932] 1 KB at p 477):

It was strenuously contended by counsel for the plaintiffs that they were entitled to retain the judgment by reason of
the principles involved in two classes of decisions. First, there were those cases concerned with the liability of a contractor
doing work negligently on another persons premises to people with whom he had no contractual relations who were
damaged by reason of his defective work The other cases are those which deal with the liability of a vendor of some
chattel, which is dangerous in itself, who gives no warning to the purchaser which might be passed on to the user of the
article. After the decision in Cavalier v. Pope and Lane v. Fox I cannot hold that these decisions apply to cases between
landlord and tenant or to cases between landlord and persons using the premises by licence of the tenant. The result is
unsatisfactory, because in the present case, if the landlords, instead of doing the work themselves, before they sold the
house, had done it afterwards as contractors to Mr. Bottomley, they would have been liable if there were sufficient evidence
that it were negligence on their part to have installed the boiler without a flue.

In a sentence, I consider the law to be this: the fact that the owner is also the builder does not remove the owners immunity, but
when the builder is not the owner he enjoys no such immunity.
It was submitted on behalf of the defendants that the principles in Donoghue (or McAlister) v Stevenson are applicable only
in the case of chattels and their manufacture, repair or installation and do not apply in relation to realty. The observations of
Scrutton LJ to this effect in Bottomley v Bannister are obiter and I am of the view with great respect that they should not be
followed without reservation in these days. I appreciate that this view may be in part at variance with that of Gorman J in
OConnor v Swan and Edgar Ltd. I am not in possession and cannot be aware of the full facts or submissions made in that case
which was shortly reported yesterday. Counsel for the plaintiff in the present case pointed out several cases which he submitted
showed that Scrutton LJs dictum had not been followed; and among these cases was that of Haseldine v C A Daw & Son Ltd,
where the lift was attached to the realty. The last reported case to which I refer, and I have referred to it already in passing, is that
of Gallagher v McDowell Ltd. In that case the Court of Appeal of Northern Ireland was confronted with a problem closely akin
to that which confronts me. It is, I think, convenient to read the headnote:

A dwelling-house was erected by the defendants, a firm of building contractors, for the Northern Ireland Housing
Trust and on completion was inspected and passed by the trusts architect. The plaintiffs husband was the first tenant of
the house and he and the plaintiff went into occupation six days after the house had been inspected and passed. Eighteen
months later the plaintiff was walking over the wooden floor in one of the rooms wearing ordinary high-heeled shoes when
she fell and was injured by reason of the heel of her shoe having gone through the floor. Examination showed 463 that a
hole in the floor board had been repaired by the insertion of a plug of wood shaped to fit the hole and this plug had given
way under the plaintiffs heel. In an action for negligence by the plaintiff against the defendants, expert evidence was led
by the plaintiff to show that the method adopted to repair the hole was not a proper method and that in any event the plug
had not been fitted in a workmanlike manner. At the conclusion of the case for the plaintiff the trial judge withdrew the
case from the jury and gave judgment for the defendants on the ground that the defendants owed no duty in tort to the
plaintiff. HELD by the Court of Appeal, reversing the decision of the trial judge: (i) that there was sufficient evidence to
go to the jury that the hole had been made and plugged, or at any rate plugged, by the defendants; (ii) that the defendants
owed a duty to the plaintiff, as a lawful user of the house they had constructed, to take reasonable care in the repair of the
hole; (iii) that, on the evidence, although the defendants and their workmen could have anticipated the examination of the
house which did in fact take place, they could not reasonably have anticipated that such inspection would expose the
danger, and that accordingly there was no sufficient reasonable possibility of intermediate examination to absolve the
defendants from their duty to take care

and it was held that the principles of Donoghue v Stevenson applied. The opinions of Lord MacDermott CJ, Black LJ and Sheil J
are persuasive only, but in my opinion, and I say so respectfully, the judgments are a correct exposition of the law in England.
My conclusion is that the principles in Donoghue (or McAlister) v Stevenson apply to the present case in the light of the
facts and that for the reasons stated the plaintiff is entitled to damages.
[His Lordship then considered the medical evidence, stated that there was no special damage and assessed general damages
at 850.]

Judgment for the plaintiff.

Solicitors: Doberman, Richardson & Broady, Middlesbrough (for the plaintiff); Cohen, Jackson & Scott, Stockton-on-Tees (for
the defendants).

G M Smailes Barrister.
464
[1963] 2 All ER 465

James v James
FAMILY; Ancillary Finance and Property: BANKRUPTCY

PROBATE, DIVORCE AND ADMIRALTY DIVISION


SIR JOCELYN SIMON P AND WRANGHAM J
30 JANUARY 1963
Magistrates Husband and wife Maintenance Maintenance order Enforceability of order Bankruptcy of husband on own
petition Arrears of maintenance not provable in bankruptcy Leave of bankruptcy court not required Magistrates Courts
Act, 1952(15 & 16 Geo 6 & 7 Eliz 2 c 55), s 74(6), as substituted by Maintenance Orders Act, 1958 (6 & 7 Eliz 2 c 39), s 16(1)
Bankruptcy Act, 1914 (4 & 5 Geo 5 c 59), s 7(1).

Bankruptcy Debt Value incapable of being fairly estimated Arrears of maintenance Enforcement of arrears a matter of
discretion Debt not provable Bankruptcy Act, 1914(4 & 5 Geo 5 c 59), s 7(1), s 30(6).

By a maintenance order made in 1960 the husband was ordered to pay to the wife 2 per week for herself and 1 10s per week
for the child of the marriage. In June, 1962, the collecting officer complained to the justices on behalf of the wife that the
husband was in arrears to the extent of 138 10s. Thereafter, but before the hearing of the complaint, the husband was
adjudicated bankrupt on his own petition. At the hearing the husband admitted that he had made no payment under the order
since February, 1962, but contended that the arrears were provable in his bankruptcy and that, by virtue of s 7(1) of the
Bankruptcy Act, 1914a, they could not now be the subject of proceedings against him except by the leave of the bankruptcy
court. He also said that he was living on 14s 2d a week and that he would not apply for national assistance. The justices were of
the opinion that his failure to pay the arrears was due to his wilful refusal or culpable neglect within the meaning of s 74(6) of
the Magistrates Courts Act, 1952, as substituted by s 16(1) b of the Maintenance Orders Act, 1958, and made a suspended
committal order. On appeal by the husband,
________________________________________
a Section 7(1), so far as material, is set out at p 467, letter b, post
b The material terms of s 74(6), as substituted, are set out at p 466, letter h, post

Held Arrears of maintenance did not constitute a debt provable in bankruptcy, because the value of the debt was incapable of
being fairly estimated within the meaning of s 30(6) c of the Bankruptcy Act, 1914, as the enforcement of such arrears was at the
discretion of the magistrates court; therefore the wife (and on her behalf, the collecting officer) had not ceased to have any
remedy against the debtor husband except with the leave of the bankruptcy court, and the justices were entitled in their discretion
to make the suspended committal order.
________________________________________
c Section 30(6) is set out at p 467, letter i, to p 468, letter a, post

Kerr v Kerr ([189599] All ER Rep 865) applied.

Notes
As to the effect of bankruptcy on orders for alimony, see 12 Halsburys Laws (3rd Edn) 430, para 965; and for cases on the
subject, see 27 Digest (Repl) 675, 676, 64366439; see also 2 Halsburys Laws (3rd Edn) 322, para 615, note (s); and 4 Digest
(Repl) 329, 29862993.
For the Bankruptcy Act, 1914, s 7(1), s 30(1), (3), (6), see 2 Halsburys Statutes (2nd Edn) 336, 364, 365.
For the Magistrates Courts Act, 1952, s 87, see 32 Halsburys Statutes (2nd Edn) 488.
For the Magistrates Courts Act, 1952, s 74(6), as substituted by the Maintenance Orders Act, 1958, s 16(1), see 38
Halsburys Statutes (2nd Edn) 706.
For the Maintenance Orders Act, 1958, s 16(1), s 18(1), see 38 Halsburys Statutes (2nd Edn) 706, 708.

Cases referred to in judgments


Kerr v Kerr [189599] All ER Rep 865, [1897] 2 QB 439, 66 LJQB 828, 77 LT 29, 27 Digest (Repl) 675, 6437.
465
Paquine v Snary [1909] 1 KB 688, 78 LJKB 361, 100 LT 220, 27 Digest (Repl) 255, 2051.
Watkins v Watkins [1896] P 222, 65 LJP 75, 74 LT 636, 27 Digest (Repl) 630, 5906.

Appeal
This was a Case Stated by the Gloucester City Justices under s 87 of the Magistrates Courts Act, 1952, to determine whether or
not there was evidence that the husbands failure to pay arrears of maintenance to the wife was due to his wilful refusal or
culpable neglect within the meaning of the Magistrates Courts Act, 1952, s 74(6), as replaced by the Maintenance Orders Act,
1958, s 16(1).

Conway Clifford for the appellant husband.


B E J Shiner for the respondent wife.

30 January 1963. The following judgments were delivered.

SIR JOCELYN SIMON P. This is an appeal by a husband by way of Case Stated under s 87 of the Magistrates Courts Act,
1952. He seeks to question in point of law an order made by the Gloucester City Justices on 6 July 1962, whereby they sentenced
the husband to imprisonment for six weeks though postponing the issue of the warrant of committal for a period of six weeks.
The matter arose in this way. A maintenance order was originally made on 29 February 1960. It was varied on 16
September 1960, so that the wife was granted maintenance of 2 weekly for herself and 1 10s weekly for the child of the
marriage. On 15 June 1962, the collecting officer on behalf of the wife complained that the husband was in arrears to the extent
of 138 10s. The husband admitted at the hearing that he owed that sum, and that he had made no payment under the order since
9 February 1962, although at that date there must already have been arrears. His reason for not complying with the order and not
making any payment since 9 February was that he had contracted to buy by instalments a freehold property for the conduct of a
club. This business was not successful. The husband had drawn 3 to 4 a week for himself; but the expense of buying the
premises and of conducting the club had been such, he claimed, as to disable him from making any payments for the wife or
child. Before the collecting officers complaint could be heard on 29 June 1962, the husband was adjudicated bankrupt on his
own petition. He registered at the employment exchange, but asserted that he was not available to meet prospective employers
because he was required to visit the official receiver. He was living on 14s 2d a week, but would not apply for national
assistance.
On those facts the justices were of opinion that the husbands default was due to his wilful refusal or culpable neglect and
they made their suspended committal order. The words wilful refusal or culpable neglect come from the Magistrates Courts
Act, 1952, s 74(6), which readsd:
________________________________________
d As substituted by s 16 (1) of the Maintenance Orders Act, 1958; 38 Halsburys Statutes (2nd Edn) 706

A magistrates court shall not impose imprisonment in respect of a default to which a complaint under this section
relates unless the court has inquired in the presence of the defendant whether the default was due to the defendants wilful
refusal or culpable neglect, and shall not impose imprisonment as aforesaid if it is of opinion that the default was not so due

In my view, that places on the defendant the onus of showing to the satisfaction of the justices that the default was not due to his
wilful refusal or culpable neglect. The justices, in posing a point of law for us at the suit of the husband, have asked whether
there was evidence on which they could hold that the husband had satisfied them that his default was not due to his wilful refusal
or culpable neglect. The proper question was whether there was evidence on which they could hold, as they did, that the default
was due to the wilful refusal or culpable neglect of the husband. If this were still a live issue, it would have been necessary 466
to refer the matter back to the justices for an amendment of the Case Stated; but nothing now arises as to the form in which the
question is asked.
It appears that the argument put forward before the justices on behalf of the husband and which it was desired to raise before
this court by way of Case Stated was as follows. The husband had been adjudicated bankrupt. By s 7(1) of the Bankruptcy Act,
1914:

no creditor to whom the debtor is indebted in respect of any debt provable in bankruptcy shall have any remedy
against the property or person of the debtor in respect of the debt unless with the leave of the court.

The court is the bankruptcy court (see s 167). By s 30:

(1) Demands in the nature of unliquidated damages arising otherwise than by reason of a contract, promise, or breach
of trust, shall not be provable in bankruptcy (3) Save as aforesaid, all debts and liabilities, present or future, certain or
contingent, to which the debtor is subject at the date of the receiving order, or to which he may become subject before his
discharge by reason of any obligation incurred before the date of the receiving order, shall be deemed to be debts provable
in bankruptcy.

It was argued before the justices and it was proposed to argue before us that the liability of the husband under the maintenance
order was thus provable in his bankruptcy; so that the wife and the collecting officer ceased to have any remedy against the
person of the debtor except with leave of the bankruptcy court. Alternatively put, the husband was not guilty of wilful refusal or
culpable neglect in failing to pay direct a liability which should be proved in the bankruptcy.
There is, however, authority binding on us, in my view, that arrears of maintenance do not constitute a debt provable in
bankruptcy. By s 30(6) of the Bankruptcy Act, 1914:

If, in the opinion of the court, the value of the debt or liability is incapable of being fairly estimated, the court may
make an order to that effect, and thereupon the debt or liability shall, for the purposes of this Act, be deemed to be a debt
not provable in bankruptcy.

In Kerr v Kerr, the Queens Bench Divisional Court held that arrears of alimony payable by a debtor to his wife, which
accrued before the making of a receiving order against the debtor, are not provable by the wife in the debtors bankruptcy. That
was a decision under provisions of the Bankruptcy Act, 1883, s 37, which corresponds word for word with s 30 of the Act of
1914. It followed Watkins v Watkins, a decision of the Court of Appeal concerned with maintenance in the Divorce Court, where
Lopes LJ said ([1896] P at p 230):

The object of the legislature was to compel the husband to make such a provision for the wife out of his monthly or
weekly earnings for her support as would keep her respectable and be in substitution for that support to which she would
have been entitled had she continued his wife, and which would not have been capable of being alienated or anticipated by
her.

In Paquine v Snary the Court of Appeal was concerned with the appointment of a receiver by way of equitable execution
and not specifically with a provision of the Bankruptcy Act, 1883; but it was held that the court had no jurisdiction, on the
application of a judgment creditor of a married woman who had obtained a separation and maintenance order from her husband,
to appoint a receiver of the maintenance. Farwell LJ said ([1909] 1 KB at p 691):

Permanent alimony under the Divorce Act, 1857, in cases of judicial separation had always been inalienable, and in
Watkins v. Watkins it was 467 held that permanent maintenance ordered under the Divorce Act, 1866, in cases of divorce
was equally inalienable, and the reasons given for that decision apply in their entirety to the present case.

What impelled the Queens Bench Division in Kerr v Kerr to hold that the arrears under the order for alimony were incapable of
being fairly estimated, so that they fell within the provision of the Act of 1883, s 37, which corresponds to s 30(6) of the Act of
1914, was that arrears are completely under the control of the court and it is entirely at the discretion of the court how far arrears
of alimony which have accrued shall be enforced. It seems to me that that applies equally to maintenance. I am reinforced in that
opinion by the fact that in Paquine v Snary the Court of Appeal held that the reasoning in Watkins v Watkins, on which Kerr v
Kerr was founded, was equally applicable to maintenance in a court of summary jurisdiction.
Counsel for the husband felt it right to draw our attention to those authorities, which he accepted were conclusively against
him. He faintly adjured us to disregard then on the ground that the wording of s 30(1) and of s 30(3) of the Act of 1914 is plain.
There are three things to say about that. First, the authorities are binding on us. Secondly, the Act of 1883 was re-enacted by the
legislature in 1914 in the light of the decisions that I have cited. Thirdly, it is not sufficient merely to point to s 30(1) and s 30(3)
without adverting also to sub-s (6).
The result was that counsel for the husband, who had not been concerned in the case before, abandoned the point raised by
the Case Stated. He sought leave, however, to raise a further point and asked us to refer the case back for amendment so that it
might be properly raised before us. We heard him de bene esse. The point that he desired to argue was that on the facts found the
justices had wrongly exercised their discretion in suspending the warrant for committal. It does not appear that any such
submission was made to the justices; if it was not, they can hardly be asked to make any finding on it now. It is, however,
unnecessary to determine this, as in my view the main argument is untenable. It was this. The justices did not send the debtor to
prison forthwith, which would have been understandable as a punishment for his wilful refusal or culpable neglect in failing to
comply with the order. What they did was to make a suspended committal order. The object of that was clearly to bring pressure
on him to make good the default; but on the face of the evidence he was at the time of the order, whatever may have been the
position earlier, incapable of discharging those arrears. Therefore, there were no facts entitling the justices to exercise the
discretion that they did to make a suspended committal order. That, says counsel for the husband, is a question of law. Without
expressing any concluded opinion, I am inclined to think that a submission that there is no evidence to support a finding does
raise a question of law. However, it is one thing to establish that that is a matter of law; it is quite another to establish that there is
no evidence on which the discretion could be exercised in the way that it was. It is not disputed that it would have been within
the discretion of the justices to order the debtor to prison forthwith as a punishment for his refusal or neglect. The question then
is, does the mere suspension of the order amount to a wrongful exercise of the discretion? That seems to be a pretty startling
proposition. There are two purposes in the code which we are considering. The first is to punish debtors who have failed to
comply with their liabilities through wilful refusal or culpable neglect, and thereby to deter other debtors who may be tempted to
take the same attitude. But the second is to bring pressure to bear on debtors to make good their defaults. I personally cannot see
any impropriety or wrongful exercise of discretion if the justices say: This is a case which is deserving of a prison sentence, but
we will give you a last chance to see if you cannot after all discharge your liabilities. It might be, for example, that the
husbands creditors would lend him the money 468 to discharge his liability and keep him thereby out of prison, so that he could
earn money for the discharge of his other liabilities to them. It might be that he would be able to raise money elsewhere on
proper disclosure that he is an undischarged bankrupt. But, in any case, a conclusive answer to the husbands argument is to be
found in s 18(1) of the Maintenance Orders Act, 1958. On a suspended committal order the justices have the opportunity of
reviewing whether to send the husband to prison in the light of what has happened since they made their original order. The
warrant is not to be issued unless notice is given to the defendant so that he has an opportunity for showing that there are grounds
for not issuing the warrant. It is, therefore, open to a debtor in the position of the husband to come back to the court, and say I
have tried every expedient to discharge my debt, and in those circumstances it would not be right to send me to prison. That,
however, has not yet arisen. Therefore, having considered counsels ingenious argument, it seems to me that the answer to any
question that we were asked on those lines by the justices is so clear that no useful purpose would be served by sending back the
case for that point to be further raised by Case Stated, even assuming that it was originally argued before the justices.

WRANGHAM J. I agree. If one ignores for a moment the very substantial procedural difficulties which occur in this case and
considers only the merits of the matter, the position is that this is in effect an appeal against that which the Gloucester City
magistrates did when they heard the complaint of the collecting officer on 6 July 1962. The appeal against what they did is
really, or has been at some time or other, threefold. First, it was said, in effect, that the magistrates had no right to issue a warrant
of commitment under s 64 of the Magistrates Courts Act, 1952, because the husband was not then in default in paying a sum. At
that stage he owed something over 130, so that prima facie he was in default; but it is said that he was not in default, because the
arrears which had accrued under the maintenance order had been discharged as a result of his adjudication in bankruptcy on his
own petition on 29 June. The short answer to that, as Sir Jocelyn Simon P, has pointed out, is that there is authority which binds
us to hold that arrears under such an order as this are not affected by an adjudication in bankruptcy. The second prong of the
attack was that, even if the husband were in default, so that the magistrates had power under s 64 of the Magistrates Courts Act,
1952, to issue a warrant of commitment, under s 74(6), as substituted by s 16(1) of the Maintenance Orders Act, 1958, they were
not entitled to impose imprisonment because the evidence did not show that the default was due to the husbands wilful refusal or
culpable neglect. Ignoring for a moment the matter of onus to which my Lord has referred, it seems to me sufficient to say that
there was ample evidence to show that the default was due to the husbands culpable neglect and wilful refusal. The third prong
of the attack was that in imposing the imprisonment which they have done, after considering the matters which they were
required to consider under s 74 as so amended, they exercised their discretion wrongly because there was no evidence from
which they could infer that a suspended sentence of committal could possibly produce the payment of any money and, therefore,
any advantage to the complainant or collecting officer who set the proceedings in motion. It is conceded in argument that there
was evidence which would have justified the justices in imposing a sentence of imprisonment to operate immediately. One
wonders whether at the hearing a contention was advanced on behalf of the husband that he should be sent to prison at once
rather than on a warrant of committal suspended for six weeks. I suspect not, and, if not, then this question was never really
raised before the justices at all. That might well have been a substantial difficulty in the way of raising this contention if the
correct procedure had been adopted in relation to this matter. But, as my Lord has pointed out, it would be idle really to explore
that matter further because it is quite plain that if the justices could properly impose an immediate sentence of imprisonment, they
equally could properly impose a sentence of imprisonment suspended for a period, even if the grounds for their hope that 469
during that period the husband might escape the sentence by somehow or other producing the money were rather slenderly based.
One does not know on what the justices did base any such hope, but it does occur to me that they may have thought that a man
who told them that he was living on 14s 2d a week and would not apply for national assistance had access to some financial
resources about which they had not been adequately informed; and that they may have thought that in those circumstances there
was some chance that their sentence might operate not merely as a punishment which the husband deserved, but also as a means
of securing financial advantage to the complainant. If they did think that, I can see no conceivable reason why they should not,
nor do I think that they are open to any criticism whatever. In those circumstances I agree with the order that my Lord has
proposed.

Appeal dismissed.

Solicitors: Rutland & Crauford agents for Frank H Green & Co Cheltenham (for the husband); Gibson & Weldon agents for
Keith Scott & Co Gloucester (for the wife).

A T Hoolahan Esq Barrister.


[1963] 2 All ER 470

Clarke v National Insurance and Guarantee Corporation Ltd


INSURANCE

QUEENS BENCH DIVISION


DAVIES LJ SITTING AS AN ADDITIONAL JUDGE
2, 3, 9 APRIL 1963

Insurance Motor insurance Exception clause Liability of insurers excluded if car being driven in unsafe or unroadworthy
condition Steering, braking and control of car seriously impaired when being driven at speed by reason of overloading.

The plaintiff was the owner of a four-seater car which was involved in a collision while he was driving it with eight adult male
passengers. By reason of the overloading, the steering, braking and control of the car were seriously impaired, and it was unsafe
to drive it at the speed at which it was being driven at the time of the collision. By an exception clause in an insurance policy
issued by the defendants the defendants were not liable in respect of any accident, injury, loss, damage and/or liability caused,
sustained or incurred while the car was being driven in an unsafe or unroadworthy condition.

Held The defendants had failed to establish that the exception clause applied, and were liable under the policy since, although
the plaintiff was obviously negligent in overloading the car and in driving it so overloaded at the speed at which he did, the
misuse of the vehicle did not alter its condition as a vehicle, the overloading and manner of driving it when overloaded being
extrinsic to its condition (see p 474, letters e and f, post).

Notes
As to terms in insurance policies relating to the condition of the vehicle, see 22 Halsburys Laws (3rd Edn) 357, para 733; and for
cases on the subject, see 29 Digest (Repl) 533, 534, 36703675.

Cases referred to in judgment


Barrett v London General Insurance Co Ltd [1935] 1 KB 238, 104 LJKB 15, 152 LT 256, 29 Digest (Repl) 533, 3672.
Brown v Zurich General Accident and Liability Insurance Co Ltd [1954] 2 Lloyds Rep 243, 29 Digest (Repl) 520, 3652.
Jones and James v Provincial Insurance Co Ltd (1929), 46 TLR 71, 29 Digest (Repl) 533, 3670.
Trickett v Queensland Insurance Co Ltd [1935] All ER Rep 729, [1936] AC 159, 105 LJPC 38, 154 LT 228, 29 Digest (Repl)
533, 3673.
Weir v Aberdeen (1819), 2 B & Ald 320, 106 ER 383, 29 Digest (Repl) 223, 1648.
470

Action
In this action the plaintiff, James Thomas Clarke, claimed a declaration that he was entitled to be indemnified by the defendants,
the National Insurance and Guarantee Corporation Ltd with whom he had a policy of insurance in respect of his car, against all
liability for damages and costs arising out of an accident involving the car on 25 December 1957, and he further claimed 350 in
respect of damage caused to his car by the accident. The defendants counterclaimed for an indemnity against their third-party
liability under a judgment obtained against the plaintiff in respect of the same accident. The facts are set out in the judgment.

R B Gibson for the plaintiff.


N H Curtis-Raleigh for the defendants.

Cur adv vult

9 April 1963. The following judgment was delivered.

DAVIES LJ read the following judgment. This is a claim on a motor car insurance policy issued by the defendants to the
plaintiff in respect of his four-seater Ford Anglia car. The General Exceptions clause in that policy, so far as is material,
provides as follows:

The [defendants] shall not be liable in respect of (i) any accident, injury, loss, damage and/or liability caused,
sustained or incurred while any motor vehicle in respect of or in connexion with which insurance is granted under this
policy is (e) being driven in an unsafe or unroadworthy condition.

In the very early hours of Christmas morning, 1957, the plaintiff, accompanied by no less than eight adult male passengers, was
driving his car down West Hill, Highgate, when, in circumstances to which it will be necessary more particularly to refer
hereafter, it came into violent collision with a Renault Dauphine car, which was being driven up the hill by a Mr Viala. In the
collision a Mr Harris, one of the passengers in the plaintiffs car (and others of his passengers as well) suffered personal injuries
and Mr Viala was killed. Mr Harris brought an action for damages against both the present plaintiff and the personal
representatives of Mr Viala. On the trial of that action, Lawton J on 20 February 1961, found that Mr Clarke was wholly to
blame for the accident, and gave judgment against him in favour of Mr Harris for 2,579 9s 6d, and costs. In a separate action
brought by the personal representatives of Mr Viala against Mr Clarke under the Fatal Accidents Acts and the Law Reform
(Miscellaneous Provisions) Act, 1934, there was, on 13 March 1961, by agreement judgment for the plaintiffs in that action
against Mr Clarke for 8,500, with costs.
The defendant insurance company had, on 20 January 1958, repudiated liability under the policy and, accordingly, the
plaintiff issued a writ claiming (i) a declaration that he was entitled to be indemnified against all liability for damages and costs
arising out of the accident, and (ii) 350, the value of his car, which was a total loss. The value of the car is now agreed at 325.
The defendants, in their defence, rely on this clause in the policy which I have already set out. They contend that, by reason of
the fact that there were nine persons in the Anglia at the material time, the steering, braking and control of the car were seriously
impaired and that, therefore, at the time of the accident, the car was being driven in an unsafe and/or unroadworthy condition.
Consequently, they claim that they are not liable to indemnify the plaintiff. The defence proceeds to point out, rightly, that, under
the Road Traffic Acts, they are obliged to satisfy the Viala judgment (that being, of course, a judgment in favour of a third party,
whereas Mr Harris was a passenger), but seeks to set off and/or counterclaim the amount of that judgment against the plaintiff
under a term in the policy which provides that

the insured shall repay to the [defendants] all sums paid by the 471[defendants] which the [defendants] would not
have been liable to pay but for the

provisions of the Road Traffic Acts.


The point for decision in this case can be shortly stated. Did the overloading of this Anglia carthe gross overloading, as
Lawton J calls itand its consequent actual or potential effect on the brakes, steering and general control of the car result in its
being driven in an unsafe and/or unroadworthy condition? The facts of the accident are fully set out in the judgment of Lawton
J and it is not necessary to set them all out again. The essential findings were that the Anglia was grossly overloaded. It was a
two-door saloon constructed and intended to carry altogether no more than four grown-up persons. There were eight men in it in
addition to the driver. Mr Clarke was in the drivers seat; another man sat in the front passengers seat, with a third man on his
lap. Three men sat on the back seat, with three more on their laps. The heights of the men ranged between five feet seven inches
and six feet, and their weights between 10 1/2 and 16 1/2 stone. Their total weight was a little over thirteen hundredweight (the
unladen weight of the car being, according to a specification put in before me, 15 1/4 hundredweight). West Hill, Highgate, has a
gradient of one in eight at the top and one in eighteen near the spot, some four to five hundred yards lower down, where the
collision occurred. In this stretch of road there are two or three bends.
The learned judge found that the Anglia was being driven at a dangerous speed and on an erratic course, and there is no
doubt that he was of the opinion that the gross overloading would cause difficulty in steering round bends and would make it
difficult for the driver to steer a proper course. Counsel for the plaintiff submits that this last matter was an inference drawn by
the judge rather than a finding of primary fact, and that it ought not to affect the decision of the present case. But that does not, I
think, matter, since I have had the evidence of two expertswho largely agree with each otheras to the effects of overloading
such as occurred here. And I accept their evidence. Mr Porte was the defendants expert. In his view, such overloading must
affect the overall efficiency of the brakes and would materially affect the steering according to the speed of the vehicle. As to the
brakes, the stopping distance would be increased by between thirty and fifty per cent. The steering would be adversely affected
by the raising of the centre of gravity of the vehicle owing to four of the passengers sitting on the knees of others. This would
tend to cause rolling or heeling over on any change of direction. At over twenty-five miles per hour, there would be some
difficulty. At forty miles per hour, the car would become dangerous owing to snakingan expression which both experts used,
meaning, I suppose, a wriggling progression. Mr Porte added that, in his view, the presence of the extra men in the back of the
car would necessarily push forward the drivers seat, which is adjustable. The distance between the back of the drivers seat and
the leading edge of the back seat, is only five inches when the drivers seat is fully back, and nine inches when it is fully forward.
If the drivers seat were pushed forward in this way the driver would not be able to put proper pressure on the pedals. Mr Davies,
for the plaintiff, differed very little from Mr Porte. He agreed with Mr Portes figures about the effect on the braking and the
steering. It is true that he had himself, as a test, driven a similar car with the same number of people in it (although weighing a
little less than the plaintiffs cargo) for 2 1/2 miles without difficulty. I suspect, however, that, on this drive, he drove with great
circumspectionmore in the manner of Agag than that of Jehu.
This evidence can, in my view, be summed up by saying that such overloading would be calculated to make the driver lose
control. It might well be more difficult to apply proper pressure to the brake pedal. Much heavier braking than normal would be
required to slow or stop the vehicle on the level, still more so when going down hill. And the steering would be adversely
affected at anything over twenty-five miles per hour. The defendants have, therefore, in my 472 judgment, established that, by
reason of the overloading, the steering, braking and control of the car were seriously impaired. The question remains for
decision: was this vehicle in an unsafe or unroadworthy condition?
It is agreed by both learned counsel, to whom I am most indebted for their interesting arguments, that the point is one free
from any direct authority. It is also agreed (a) that it is for the defendants to show that the exceptions clause covers the facts of
the case, and (b) that, in case of doubt, the clause must be construed against the defendants. Counsel for the defendants did,
indeed, suggest that assistance might be derived from the principles of marine insurance law, according to which, of course, if a
ship is overladen she is unseaworthy (see, for example Weir v Aberdeen). But is it clear that great caution must be exercised in
applying principles of marine insurance to motor insurance; for very different considerations may apply. In Barrett v London
General Insurance Co Ltd, Goddard J had to consider the application of an exceptions clause, in terms almost identical with those
of the present one, to a case where the brakes of a car had failed at the moment of the accident, but there was no evidence as to
any defect in the brakes at any earlier time. The learned judge assimilated the position of a motor car to that of a ship, and held
that a car must be roadworthy at the start of its journey but that there is no warranty or condition that it should so continue. But
in Trickett v Queensland Insurance Co Ltd ([1935] All ER Rep 729 at p 732; [1936] AC 159 at p 165), Lord Alness, in delivering
the opinion of the Judicial Committee, said this with regard to Barrett v London General Insurance Co Ltd:

All that their Lordships find it necessary to say regarding that judgment is that, while not questioning the conclusion
reached by the learned judge on the facts of the case, they find great difficulty in agreeing with the reasons on which that
conclusion was based. They are not able to assimilate, as did the learned judge, the position of a ship at sea with that of a
motor-car on land, and to apply rigidly the same code of law to both cases. For reasons which are too obvious to be
stressed in detail, their Lordships think the analogue imperfect, and, indeed, misleading. They are of opinion that the
argument based by the appellant on the identity of the conditions which govern the seaworthiness of a ship at sea and the
roadworthiness of a car on land is unsound.

In my opinion, therefore, it would be wrong to treat the decisions on unseaworthiness as of any assistance in construing a clause
such as that now under discussion.
Nor, conversely, do I think that any great assistance is to be derived from the words of reg 73 of the Motor Vehicles
(Construction and Use) Regulations, 1955a, which, so far as is material, provides that
________________________________________
a SI 1955 No 482

Every motor vehicle and all parts and accessories of such vehicle shall at all times be in such condition, and
the number of passengers carried by such vehicle shall at all times be such that no danger is caused or is likely to be
caused to any person on the vehicle or on a road:

This regulation clearly distinguishes between the condition of the vehicle and its parts and accessories on the one hand and the
number of passengers carried on the other. But it would not, in my judgment, be proper to construe the clause in this policy by
reference to that regulation.
It is agreed by both learned counsel that to be hit by this exceptions clause the unsafety or unroadworthiness must be, to use
their word, something intrinsic in the vehicle. The defendants contend that, even though there was no evidence here that there
was anything wrong with the brakes or steering of the car if 473 properly loaded and properly used, it is, nevertheless, established
that this overladen car was unsafe and unroadworthy for normal functioning. It is argued, in my view rightly, that it is not
necessary to show that the vehicle was completely unsafe or completely unroadworthy. And, in this connexion, reference was
made to Brown v Zurich General Accident & Liability Insurance Co Ltd. That case concerned not an exceptions clause but a
condition in a policy in the familiar terms to take all reasonable steps to maintain in efficient condition. It was held that to
allow the treads of the front tyres to be worn right down was a breach of the conditions. See also Jones and James v Provincial
Insurance Co Ltd, where the foot brake had been removed. So, it is submitted, the fact that the vehicle can be used if proper care
and skill is exercised does not prevent it from being unsafe and unroadworthy for the purposes of such use as is to be normally
expected on the highway. The opposite contention is that the words condition of the vehicle means condition of the vehicle,
and do not include any consideration of the use to which it is being put or the manner in which it is used. In all the authorities
cited in the arguments an actual defect in the vehicle itself was under discussion. A number of examples were given in argument
of various possible abuses of a car which would render its use on the highway to some degree unsafe and could be said to
increase the risk but as to which no one would say that they affected the condition of the vehicle. Moreover, it is pointed out that,
on the evidence, this vehicle, even overloaded as it was, might have been driven safely if it had been kept down to a speed
appropriate to its load.
The point is not, in my opinion, an easy one to decide; but it is not capable of a great deal of argument or exposition. On the
whole, I have come to the conclusion that the defendants have failed to establish that the exceptions clause applies. The plaintiff
was obviously negligent in overloading the car and obviously negligent in driving it so overloaded at the speed at which he did.
But it seems to me that the misuse of the vehicle which occurred in this case did not alter its condition as a vehicle. To say that,
in the circumstances, the vehicle was unsafe to drive, at any rate above a certain speed, is not the same as to say that the vehicle
was in an unsafe or unroadworthy condition. The overloading was something extrinsic to the condition of the vehicle as, of
course, was the manner of driving it when overloaded. In this case there was nothing wrong with the vehicle as such. The cause
of the trouble was its negligent use. One can well understand that insurance companies would wish to guard themselves against
such extraordinary happenings as those in the present case, and it may well be that, by appropriate words, they may be able to do
so.
In the result, the plaintiff is, in my opinion, entitled to judgment on the claim and counterclaim.

Judgment for the plaintiff.

Solicitors: Neil Maclean & Co (for the plaintiff); L Bingham & Co (for the defendants).

Mary Colton Barrister.


474
[1963] 2 All ER 475

R v Hammond
CRIMINAL; Sentencing

COURT OF CRIMINAL APPEAL


LORD PARKER CJ, HAVERS AND EDMUND DAVIES JJ
29 APRIL 1963

Criminal Law Sentence Quarter sessions Committal to quarter sessions for sentence Detention Youthful offender under
seventeen at date of conviction, over seventeen at date of sentencing No notification that detention centre available for persons
under seventeen Court of summary jurisdiction, therefore, without jurisdiction to impose sentence of detention Sentence of
borstal training imposed by quarter sessions Whether quarter sessions had jurisdiction to impose sentence of detention
Criminal Justice Act, 1948 (11 & 12 Geo 6 c 58), s 20(5).

The appellant at a date when he was under seventeen years of age pleaded guilty before magistrates to housebreaking and larceny
and was committed to quarter sessions for sentence. A sentence of detention in a detention centre was thought fit, but, the
appellant being under seventeen at the time of his appearance before quarter sessions, and no notification, pursuant to s 18(1) of
the Criminal Justice Act, 1948, that a detention centre was available for the detention of persons under seventeen having been
received, quarter sessions remanded him in custody for a month at the end of which time he would have attained seventeen.
When the appellant came before quarter sessions he was sentenced to borstal training, because of s 20(5) a of the Act of 1948,
under para (a)(ii) of which the appeal committee was authorised merely to deal with him in any manner in which the court of
summary jurisdiction might have dealt with him. On appeal on the ground that that enactment authorised in the circumstances
sentence to detention in a detention centre,
________________________________________
a The material part of s 20(5) is set out at p 476, letter f, post

Held The words deal with him in any manner in which the court of summary jurisdiction might have dealt with him in s
20(5) of the Criminal Justice Act, 1948, did not confer on quarter sessions jurisdiction to impose a sentence that could not have
been imposed by the court of summary jurisdiction at the time when they decided to commit the appellant to quarter sessions;
accordingly, quarter sessions had no jurisdiction to sentence the appellant to detention at a detention centre.
Appeal dismissed.

Notes
On 1 August 1963, s 18 of the Criminal Justice Act, 1948, is repealed, and s 20(5)(a) is amended, this latter amendment being in a
respect that is not material to this decision; see Criminal Justice Act, 1961, s 41 and Sch 4, and Criminal Justice Act, 1961
(Commencement No 2) Order, 1963 (SI 1963 No 755). Section 20 as re-enacted is set out in Sch 6 to the Act of 1961. Provision
regarding detention of young offenders is made in s 4-s 7 of the Act of 1961, which are to come into operation on 1 August 1963.
As to the committal of youthful offenders from magistrates courts for sentence, see 25 Halsburys Laws (3rd Edn) 226, 227,
para 421, and, as to the power of sentencing, see ibid, p 229, para 423, text and notes (1)-(0); and for cases on the punishment of
youthful offenders, see 14 Digest (Repl) 588592, 58525899.
For the Criminal Justice Act, 1948, s 18 and s 20, see 28 Halsburys Statutes (2nd Edn) 367, 369; and for the Criminal
Justice Act, 1961, ss 4-7, 41, Sch 4, Sch 6, see 41 Halsburys Statutes (2nd Edn) 134137, 145, 150, 157.

Appeal
This was an appeal by George David Hammond against a sentence of borstal training imposed on him by the chairman at West
Sussex Quarter Sessions on 14 February 1963. The facts appear in the judgment of the court.

J Grove Hull for the appellant.


M J P Macnair for the Crown.
475

29 April 1963. The following judgment was delivered.

LORD PARKER CJ delivered the following judgment of the court. This appellant pleaded guilty at Horsham Magistrates
Court to housebreaking and larceny, and was committed under s 28 of the Magistrates Courts Act, 1952, to West Sussex Quarter
Sessions for sentence, where he asked for six other offences to be taken into consideration. The deputy chairman on that
occasion, which was on 3 January 1963, came to the conclusion, despite this boys record, that it would be sufficient to commit
him to a detention centre for three months, and he made that order. It was pointed out to him later on the same day that his power
to make such an order depended on the court having been notified by the Secretary of State that a detention centre was available
for the reception of a boy of his age, the age being under seventeen. That appears from s 18(1) of the Criminal Justice Act, 1948.
In fact, no notification had been made that there was any detention centre available for youths under seventeen, and the deputy
chairman thereupon remanded the prisoner in custody until the end of the month, when he would be seventeen, so that the court
then sitting could make a detention order, it being assumed that there was a detention centre available for youths of seventeen and
upwards. Accordingly, the prisoner was brought up again on 14 February and then the chairman of the court likewise was
minded to send this prisoner to a detention centre, but his attention was drawn quite rightly to s 20(5) of the Criminal Justice Act,
1948, and as a result he came to the conclusion that he had no power to make the order, notwithstanding that the prisoner was
then seventeen and he took what seemed to be the only alternative course of sentencing this boy to borstal training.
Counsel, who has said everything that can be said for this appellant, has submitted that the chairman on 14 February was
wrong, and that he did have power to make an order of detention. Whether he has such power or not depends on the true
construction of sub-s (5) of s 20 of the Act of 1948. That reads, so far as it is material:

Where an offender is so committed for sentence as aforesaid, the following provisions shall have effect, that is to say
(a) the appeal committee or court of quarter sessions may (i) if satisfied [of certain matters] sentence him to borstal
training; or (ii) in any case, deal with him in any manner in which the court of summary jurisdiction might have dealt with
him

It seems to this court that the words there are perfectly plain; might have dealt with him refer to the period when the lower
court decided not to sentence him but to commit him. In other words, quarter sessions could deal with him in any way in which
the court of summary jurisdiction might have dealt with him if they had not committed. That, as the court thinks, is perfectly
plain, and it is to be observed that if the suggested alternative construction had been intended, namely in any way in which the
magistrates might have dealt with him at the time when he was before the quarter sessions, quite a different formula would no
doubt have been used. An illustration is to be found in s 29(3) of the Criminal Justice Act, 1948, dealing with committal for
sentence in respect of indictable offences which have been tried summarily.
Section 29(3) provides, so far as is material:

Where an offender is so committed for sentence as aforesaid, the following provisions shall have effect, that is to say:
(a) the appeal committee or court of quarter sessions shall inquire into the circumstances of the case, and shall have power
to deal with the offender in any manner in which he could be dealt with by a court of quarter sessions before which he had
just been convicted of the offence on indictment.

Those words are entirely different from the words of sub-s (5) of s 20 of the Act of 1948, and, as I have already said, the court is
clearly of the opinion that the words in the latter subsection refer to the time when the justices decided to commit. In these
circumstances, the chairman on 14 February was quite correct in 476 coming to the conclusion that he had no power to make an
order for detention. In so far as the sentence of borstal training is concerned, the court is clearly of the opinion that this was fully
justified; this prisoner has five previous convictions; he has twice been put on probation, and he has been to an approved school.
Further, the Prison Commissioners report that he is unlikely to settle to training without conditions more secure than those
normally available at an approved school. This appeal is dismissed.

Appeal dismissed.

Solicitors: Registrar, Court of Criminal Appeal (for the appellant); Walmsley & Stansbury agents for J E Dell & Loader,
Shoreham (for the Crown).

N P Metcalfe Esq Barrister.


[1963] 2 All ER 477

R v Brixton Prison (Governor) and Another, ex parte Enahoro


CONSTITUTIONAL; Crown: INTERNATIONAL; International Criminal Law

QUEENS BENCH DIVISION


LORD PARKER CJ, HAVERS AND EDMUND DAVIES JJ
25 APRIL 1963

Extradition Discharge of fugitive Not returned within one month after committal May, unless sufficient cause is shown
to the contrary, order the fugitive to be discharged May construed to mean shall Fugitive Offenders Act, 1881 (44 & 45
Vict c 69), s 7.

Extradition Fugitive offender Return by warrant of Home Secretary on expiration of fifteen days May, if he thinks it just,
by warrant order fugitive to be returned May not mandatory Discretion of Home Secretary Fugitive Offenders Act, 1881
(44 & 45 Vict c 69), s 6.

Constitutional Law Minister Statutory responsibility Debate on Ministers decision before decision implemented Whether
a surrender of statutory responsibility to Parliament Fugitive Offenders Act, 1881 (44 & 45 Vict c 69), s 6.

The true effect of s 7 a of the Fugitive Offenders Act, 1881, is that, when a fugitive is found to have been detained for one month
or more from the time of his committal, it is for the Secretary of State to account for the delay, and that, unless he satisfies the
court that the detention in all the circumstances for that period is reasonable, then the court should discharge the fugitive (see p
479, letter i, post).
________________________________________
a Section 7, so far as material, is set out at p 479, letter c, post

Dictum of Lord Parker CJ in Re Shuter (No 2) ([1959] 3 All ER at p 484) applied.


On 13 December 1962, the applicant was committed to prison under the Fugitive Offenders Act, 1881, s 5, to await his
return to Nigeria. The offences which he was alleged to have committed were treason and conspiracy to effect an unlawful
purpose. On 15 January 1963, the Divisional Court dismissed an application on his behalf for a writ of habeas corpus and also
for relief under s 10 of the Act of 1881. On 6 February 1963, the appeals committee of the House of Lords refused his
application for leave to appeal to the House of Lords. On 12 February 1963, a petition by the applicant was considered by the
Home Secretary who also received a memorandum from the member of Parliament in whose constituency the applicant had been
arrested, and on 5 March 1963, a deputation of members of Parliament brought certain matters to the Home Secretarys attention.
On 14 March 1963, the Home Secretary, being satisfied that he had properly investigated all the matters raised for his
consideration, decided under s 6 of the Act of 1881 b, that it was just to order the applicants return to Nigeria. However, before
issuing the warrant, he decided that he should report the matter to Parliament.
477
________________________________________
b Section 6, so far as material, is set out at p 481, letter b, post

The matter was debated in the House of Commons on 26 March 1963, and was adjourned until 10 April 1963, when the
House made its decision by a division. Immediately thereafter, the Home Secretary received notice of the intention to make an
application to the Divisional Court that the applicant be discharged from custody under s 7 of the Act of 1881 on the ground of
delay and detention exceeding one month, and he took no further steps to sign the warrant. The applicant also applied for a writ
of habeas corpus. On the hearing of the applications, no complaint was made of any detention up to and including 14 March
1963.

Held The applications must be refused, because


(i) on the true construction of s 6 of the Act of 1881, there was no reason for giving the word may in the phrase may, if he
thinks just, a mandatory meaning, and, therefore, if the Home Secretary were satisfied that it was just that the fugitive should be
returned, it was still a matter of discretion for him not only whether he should issue the warrant, but also when he should issue it
(see p 481, letter g, post); further the Home Secretary, in inviting the House of Commons to express a view whether his decision
was also the view of the majority of the House, had not surrendered his statutory responsibility under s 6 to Parliament, and, since
what had been done was chiefly attributable to the applicants supporters and had been for his assistance, so far as possible, the
Home Secretary had shown sufficient cause to the contrary within s 7 of the Act (see p 482, letter d, and p 483, letters a and b,
post).
(ii) the grounds for the writ of habeas corpus were the same as for the application for discharge and it had been shown that
the applicant was held under a valid warrant and nothing in the present proceedings was capable of challenging that warrant (see
p 482, letters h and i, post).
Applications refused.

Notes
As to surrender of a fugitive, see 16 Halsburys Laws (3rd Edn) 589, para 1224; and as to discharge of a fugitive by a superior
court, see ibid, 589, 590, para 1225.
For the Fugitive Offenders Act, 1881, s 6, s 7, see 9 Halsburys Statutes (2nd Edn) 899, 900.

Case referred to in judgments


Shuter (No 2), Re [1959] 3 All ER 481, [1960] 1 QB 142, [1959] 3 WLR 652, 123 JP 534, 3rd Digest Supp.

Motions
Pursuant to a committal order made at Bow Street Magistrates Court on 13 December 1962, under s 5 of the Fugitive Offenders
Act, 1881, the applicant, Anthony Eronsele Enahoro, was committed to Brixton Prison on that date to await his return to Nigeria.
Counsel for the applicant now moved (i) for an order for his discharge pursuant to s 7 of the Act of 1881, and (ii) for a writ of
habeas corpus. The facts are set out in the judgment of Lord Parker CJ.
The cases noted belowc were cited during the argument in addition to that referred to in the judgment of Lord Parker CJ.
________________________________________
c Barnard v National Dock Labour Board [1953] 1 All ER 1113, [1953] 2 QB 18, Vine v National Dock Labour Board [1956] 3 All ER 939,
[1957] AC 488, Zacharia v Republic of Cyprus [1962] 2 All ER 438

Dingle Foot QC, E F N Gratiaen QC and Kenneth Potter for the applicant.
The Attorney General (Sir John Hodson QC) and J R Cumming-Bruce for the respondents, the Governor of Brixton Prison and
the Secretary of State for Home Affairs.

25 April 1963. The following judgments were delivered.

LORD PARKER CJ. On 13 December 1962, an order was made at Bow Street Magistrates Court under s 5 of the Fugitive
Offenders Act, 1881, committing the applicant to Brixton prison with a view to his return in custody to 478 Nigeria. That order
was made as a result of a warrant for the applicants arrest issued in Nigeria on 27 November 1962, alleging charges of treason
and conspiracy to effect an unlawful purpose. On 15 January 1963, this court dismissed an application on behalf of the applicant
for a writ of habeas corpus and also for relief under s 10 of the Act of 1881. On 6 February of this year, the appeals committee of
the House of Lords refused an application for leave to appeal to the House of Lords. The applicant is still in custody at Brixton
Prison, and two applications by way of motion now come before this court, both moved by counsel on his behalf. The first
motion, a motion of 10 April 1963, asks that the applicant be discharged from custody under s 7 of the Fugitive Offenders Act,
1881. The second motion, dated 19 April 1963, asks for a writ of habeas corpus. It is, I think, convenient in the first instance to
deal with the application under s 7 of the Fugitive Offenders Act, 1881. That section provides, so far as it is material, as follows:

If a fugitive who, in pursuance of this Part of this Act, has been committed to prison in any part of Her Majestys
dominions to await his return, is not conveyed out of that part within one month after such committal, a superior court
may, unless sufficient cause is shown to the contrary, order the fugitive to be discharged out of custody.

So far as is known, there has only been one other case under s 7 of the Act; that is Re Shuter (No 2). The only materiality for
this purpose of that decision is as to the meaning of those words may, unless sufficient cause is shown to the contrary, order the
fugitive to be discharged out of custody. In the first place, the court came to a decision that the word may should be
interpreted as mandatory, so that, in the absence of sufficient cause being shown to the contrary, there was no further discretion;
the fugitive had to be discharged. The second matter which is important is that it had been argued in that case that sufficient
cause to the contrary must relate to something outside the control of the relevant authorities, or some strong and compelling
reason such as the serious illness of the fugitive. The court, however, took the view that that interpretation was too narrow, and
that the words unless sufficient cause is shown to the contrary brought matters of reasonableness under consideration. In
giving judgment in that case, I said ([1959] 3 All ER at p 484; [1960] 1 QB at p 148):
Accordingly, it seems to me that, when one is construing the words in in s. 7, unless sufficient cause is shown to the
contrary, one is entitled to take into account questions such as reasonableness in all the circumstances. Once one gets as
far as that, as a matter of construction, it seems to me quite plain that, on the facts disclosed in Mr. Rushfords affidavit, no
one could say that he was not acting perfectly reasonably in making the arrangements which he did. One cannot shut ones
eyes to the fact that this is only a matter of a day or twos delay and that the alleged offences are serious offences. There is
also the fact that the applicants own solicitor was informed of the arrangements and took no objection. Of course that
latter observation is subject to the comment that he might well not have had s. 7 in mind, but, when one is thinking of this
matter in terms of reasonableness, it is some evidence that what was being done was perfectly reasonable and that the
applicant was not being detained for an unreasonable time.

Approaching the matter in that way, it would seem that the true effect of s 7 is that, when a fugitive is found to have been
detained for more than thirty days from the time of his committal, it is for the Secretary of State to account for the delay, and that,
unless he satisfies the court that the detention in all the circumstances for the period exceeding thirty days is reasonable, then this
court should discharge the fugitive.
In the present case, as I have already said, the matter first of all came before this court; application was made to the Appeals
Committee of the House of 479 Lords for leave to appeal and that was refused, and no conceivable complaint is made of any
delay, certainly up to that time, 6 February. Following that, on 12 February a petition was put in by the applicant, and that was
considered by the Home Secretary. Not only that, but a memorandum was received by him from the Member of Parliament in
whose constituency the applicant had been arrested. On 5 March a deputation of members of Parliament brought certain matters
to the attention of the Home Secretary, and so the matter progressed until, on 14 March the Home Secretary made up his mind. In
para 5 of an affidavit sworn in these proceedings, he says:

On Mar. 14 I was satisfied that I had properly investigated all the matters which had been raised for my consideration
by or on behalf of the applicant and by members of Parliament, and I decided under s. 6 of the Fugitive Offenders Act,
1881, that it was just to order the return of the applicant to Nigeria.

Once more it is only right to say that no complaint is made of any detention up to and including 14 March. Everything that had
been done up to that stage had certainly been in the interests of the applicant, and to ensure that he would suffer no injustice if he
was sent back to Nigeria. It is in respect of the period subsequent to that date that complaint is made. It is said that no sufficient
cause has been shown for that further detention.
What in fact happened was this. The Home Secretary, having come to that decision on 14 March decided that he should,
before issuing his warrant, report the matter to Parliament and allow a debate to take place. He says in para 6 of his affidavit
and I do not propose to read it all

I am answerable to Parliament for the discharge of my statutory duty under s. 6. My decision did not accord with the
representations urged upon me by a number of members of Parliament of all parties, and the case raised problems
concerning the discharge of that duty in relation to a person charged with political offences in the territory of an
independent sovereign member of the Commonwealth. I considered that it was both my duty and in the interests of the
applicant that I should report my decision to Parliament before ordering his return. A draft warrant ordering his return had
already been prepared and was ready for my signature. Accordingly I made a statement announcing my decision to the
House of Commons on Mar. 14. It was clear to me that it was the wish of the House of Commons that the House should
debate my decision before I ordered the return of the applicant

He then goes on to say that a debate was due to take place on 21 March that on that date there was no question relating to his
decision before the House, and that, at the conclusion of the discussion, he gave an assurance that there would be no order for the
return of the applicant pending what is referred to as the decision of the House of Commons provided that the matter could be
brought to an early conclusion. The debate took place in the House of Commons on 26 March and was adjourned. On 10 April
the adjourned debate was resumed and concluded, and the House made its decision by a division about 8 pm. However, at 8.30
pm, the Home Secretary received notice of the intention to make the application which is now before the court, and, accordingly,
he took no further step to sign the warrant. Pausing there, I confess that, for my part were it not for the very able and ingenious
argument of counsel for the applicant, I should consider this a thoroughly impudent application. Everything as it seems to me
that has been done from the date of his committal until today has been as much in the interests of the applicant as could be.
Every possible step has been taken to see that it would be just that he should be sent back, and if one may use the expression,
everybody seems to have leant over backwards in his favour. However, if he has a legal right to be discharged now, he is,
whatever I might think of the nature of the application, of course fully entitled to be discharged.
480
The way in which the matter is put is this. It is said that, under s 6, it is the duty of the Home Secretary once he has come to
a decision that it is just that the fugitive should be returned, to issue forthwith a warrant for that purpose. Section 6 of the
Fugitive Offenders Act, 1881, provides that:

Upon the expiration of fifteen days after a fugitive has been committed to prison to await his return, or if a writ of
habeas corpus or other like process is issued with reference to such fugitive by a superior court, after the final decision of
the court in the case, (1) a Secretary of State may, if he thinks it just, by warrant under his hand order that fugitive
to be returned to the part of Her Majestys dominions from which he is a fugitive ;

that, I think, is all that is relevant for this purpose. The argument, as I understand it, runs thus, that may, if he thinks it just, by
warrant order the return is mandatory in the same way as may is mandatory under s 7, and that, accordingly, once the Home
Secretary has decided that it is just that the fugitive should be returned, he must by warrant under his hand order the return. If
that be right, so runs the argument, then, on 14 March when the Home Secretary decided that it was just to order the return, he
should have signed the warrant. Observe again on the merits that, if he had, the fugitive would presumably be back in Nigeria;
but, be that as it may, it is said that anything that happened after 14 March that is after the Home Secretary had decided that it was
just that he should be returned, cannot amount to any sufficient cause to the contrary within s 7.
For my part, I am quite satisfied that that argument ultimately depends on whether it is right to treat the word may in s 6 as
mandatory. Re Shuter (No 2), to which I have referred, is a decision of this court that, in connexion with s 7, may is mandatory.
For my part, however, I am quite satisfied that, in s 6, it cannot be read in that sense. Section 7, as I have already said, is dealing
with the liberty of the subject, and the prevention of his being detained for an unduly long period. After thirty days he must be
discharged unless sufficient cause is shown. Section 6, on the other hand, so far as time is concerned, is not saying that
something must happen after a certain time, but is saying that something shall not happen before a certain time has expired,
namely, the person is to be allowed fifteen days from a committal or the termination of habeas corpus proceedings before the
Secretary of State can take any steps, and, for my part, I can see no reason for giving may a mandatory meaning in that
connexion. He must of course first be satisfied that it is just, and, if he is not satisfied that it is just, then no question of a warrant
being signed arises.
If, on the other hand, he is satisfied that it is just, it is still a matter of discretion for him not only whether he should issue the
warrant, but when he should issue the warrant. If he does delay, if he leaves the prisoner in custody without either releasing him
or signing a warrant for his return, then s 7 comes into operation for the protection of the fugitive. If that be right, then, as it
seems to me, one has to look to see what happened after 14 March to see whether what took place then amounts to a sufficient
cause to the contrary under s 7. With regard to that, counsel for the applicant urges that the true effect of para 6 of the Home
Secretarys affidavit is that the Home Secretary was either surrendering to Parliament or sharing with Parliament what, under s 6
of the Fugitive Offenders Act, 1881, was his statutory responsibility. He said that action by the Home Secretary in a manner
which the law does not permit could not on any view disclose sufficient cause to the contrary within s 7. It is well settled that
certainly no person made responsible for a judicial decision can delegate his responsibility. Equally, in certain cases of
administrative decisions there is no power to delegate, and I would assume without deciding it that here the Home Secretary,
albeit the 481 decision whether to sign the warrant may be treated as an administrative decision, has no power to delegate that
responsibility. Assuming that, however, I can see no ground, accepting literally everything that is said in para 6 of the Home
Secretarys affidavit, for thinking that he has delegated to a majority of the House of Commons or shared with the majority of the
House of Commons what is his statutory responsibility.
The matter can really be considered in two ways. One can consider his duties under s 6 of the Fugitive Offenders Act, 1881,
as twofold, first, to ascertain whether it is just that the fugitive should be returned, and, secondly, whether in the exercise of his
discretion he should sign a warrant, and, if so, when. Approached in this way, it could be said and, indeed, has been said by
counsel for the respondents that all that he was inviting Parliament to express a view on was as to the signing of the warrant. For
my part, I think that that is the wrong approach. I think, to be realistic, that he was inviting Parliament to express a view whether
his decision that it was just was the view also of the majority of the House. But even so, for my part I can see no reason why he
should not. Granted that it is his duty to decide, granted that he has decidednot expressed as a preliminary or provisional
opinion, but has decidedthat it is just, there is no reason why he should not invite the expression of further opinion, and, if
influenced by it, change his mind. Looked at in that way, I cannot see that, by inviting Parliament or by allowing Parliament to
express an opinion on his decision, he was in any way surrendering or sharing with Parliament what was his statutory
responsibility. Accordingly, as it seems to me and as I said at first, everything that has been done in this case has been so far as
possible to assist the applicant, and the Home Secretary has shown sufficient cause to the contrary within s 7 of the Fugitive
Offenders Act, 1881.
Finally, there is a further point on which this court has been asked to adjourn the case, in order that the permission of the
House of Commons should be obtained for this court to consider what was said in the debate in question. As I understand it,
counsel for the applicant would seek, from reading passages in Hansard, to show that, in the course of the debate, other ministers
may have intervened, that other ministers, that is ministers other than the Home Secretary, may have expressed a view which was
for the Home Secretary to express or have given an assurance which only the Home Secretary could properly give, and that in
some way the Home Secretary had surrendered or shared with that minister or ministers what was his responsibility, in other
words, that there has been some wrongful delegation. The court has refused to adjourn to look into that matter because, and this
is my own opinion, it seems to me that, at the end of the day, even if all that were shown, it is completely irrelevant to the
application under s 7.
It is, however, said to be relevant to the application for a writ of habeas corpus, and with that, the second application before
the court, I have not dealt. So far as the habeas corpus application is concerned, it was put in on 19 April some nine days after the
other motion and, it seems to me, as a pure adjunct. It asks for the writ of habeas corpus, and it states as grounds, the grounds set
forth in the affidavits of the applicant and of his solicitor which are the affidavits in support of the application which I have
already considered under s 7. No further ground is advanced for a writ of habeas corpus. For my part, I find it very difficult to
see how there could be. This matter of habeas corpus has already been before the court. It has been shown that the fugitive is
held under a valid warrant, and nothing that has been advanced in these proceedings seems to me capable of challenging the
warrant under which he is held. I cannot see that an application for habeas corpus could succeed in the present case, certainly not
on the matters advanced in connexion with the s 7 application. Accordingly, as it seems to me, it would be idle to adjourn this
case. Even if counsel for the applicant is right in what he suggests might be found if we looked at Hansard, yet I am quite
satisfied that it would be wholly irrelevant in these applications.
482
Accordingly, I have come to the opinion that both applications fail and should be refused.

HAVERS J. I agree with the judgment given by my Lord and with the reasons which he has given for it. I am satisfied by the
affidavit of the Secretary of State that sufficient cause has been shown to the contrary within the meaning of s 7 of the Fugitive
Offenders Act, 1881. I agree that both these applications should be dismissed.

EDMUND DAVIES J. I agree. I would only add that, speaking for myself, I find difficulty in conceiving how a clearer or more
sufficient cause could be shown within s 7 of the Fugitive Offenders Act, 1881, than where the lapse of more than a month since
committal is overwhelmingly, if not indeed entirely, attributable to (a) the supporters of the fugitive himself and his advisers and
other well-intentioned parties to prevent or delay his return; and (b) the unimpeached and unimpeachable desire of the Home
Secretary to give due, proper and reasonably prompt consideration to those representations. That is clearly what had happened in
the present case. It follows that, in my judgment, like that of my brethren, the relief now sought under s 7 should, and indeed
must, be refused.

Applications refused. Leave to appeal to the House of Lords refused, but certificate in respect of the application under s 7 of the
Act of 1881 granted under s 1 of the Administration of Justice Act, 1960, on the question of the true interpretation of the word
may in s 6 of the Act of 1881d.
________________________________________
d Application for leave to appeal to the House of Lords was duly made and, on 15 May was refused

Solicitors: Hatchett Jones & Co (for the applicant); Treasury Solicitor (for the respondents).

N P Metcalfe Esq Barrister.


[1963] 2 All ER 483

Practice Direction
(Probate: Summons for directions; Subsequent applications
PRACTICE DIRECTIONS
PROBATE, DIVORCE AND ADMIRALTY DIVISION (PROBATE).
14 MAY 1963

Probate Practice Summons for directions Subsequent applications to be by notice RSC, Ord 30, r 7(3).

Attention is drawn to the provisions of RSC, Ord 30, r 7(3), which provisions a are reproduced in Ord 25, r 7(3) of the revised
Rules of the Supreme Court coming into force on 1 January 1964. The practice of giving notice by issuing a fresh summons to
deal with interlocutory matters subsequent to the summons for directions should be discontinued.
________________________________________
a See Tristram and Cootes Probate Practice (21st Edn) 676

B Long, Senior Registrar


14 May 1963.
483
[1963] 2 All ER 484

Simpsons Motor Sales (London) Ltd v Hendon Corporation


HOUSING

HOUSE OF LORDS
LORD EVERSHED, LORD COHEN, LORD JENKINS, LORD HODSON AND LORD GUEST
18, 19, 20, 21, 25, 26 MARCH, 6 MAY 1963

Housing Compulsory purchase order Notice to treat and notice of entry served Delay Abandonment Ultra vires
Original housing project not implemented Redevelopment at some future date as part of larger area contemplated instead
Whether notices continued to be enforceable Whether change of circumstances gave rise to equity to restrain enforcement of
notices Housing Act, 1936 (26 Geo 5 & 1 Edw 8 c 51), s 74.

The Hendon corporation made on 25 March 1952, a compulsory purchase order under s 74 of the Housing Act, 1936, for the
acquisition of land (the North Road site) occupied by the appellant company (which, with their predecessor in title to the land,
are included here in the description Simpsons). The compulsory purchase order was confirmed by the Minister of Housing and
Local Government on 30 July 1952. Notice to treat was served on 13 August 1952, and no objection to it was taken. On 22
October 1953, notice of entry was served by the corporation. On 11 August 1958, the corporation wrote to the Central Land
Board, to whom Simpsons had applied for immediate payment for loss of development rights on compulsory acquisition, that at
that time it was not the intention of the corporation to proceed to acquire the land. The Central Land Board informed Simpsons
that the corporation were not proceeding with the acquisition of the North Road site. On 25 August 1958, Simpsons asked the
corporation to confirm that they had abandoned the compulsory purchase. On 13 November 1958, the corporation replied that
they intended to proceed with the acquisition of the site. On the question whether the notice to treat and notice of entry continued
to be effective or whether the corporation were disabled from enforcing them, either because their present intentions were not the
purposes for which the compulsory purchase order was granted, or because of delay or abandonment of rights under the notice to
treat, or because to enforce the notices would be contrary to principles of equity,

Held The corporation were entitled to enforce the notice to treat of 1952 and the notice of entry of 1953 because
(i) (in regard to the contention of ultra vires), although after 1955 the actual use to which the North Road site would be put
became vague if not visionary, the corporation had not at any time before the issue of the writ determined on a use of the site
which would be outside the scope of Part 5 of the Housing Act, 1936, or on its use for redevelopment as distinct from the purpose
of providing housing accommodation under Part 5 of the Act of 1936 (see p 489, letter i, to p 490, letter a, post).
(ii) (in regard to delay) the delay was not in the circumstances such as to disentitle a local authority, having public duties to
perform, from enforcing their rights (see p 491, letter a, post).
(iii) (in regard to abandonment) there had not been communications between Simpsons and the corporation (the letter of 11
August 1958, having been written to the Central Land Board, not to Simpsons) such as would amount to an abandonment by the
corporation of their rights (see p 493, letter b post).
(iv) (in regard to equitable right) if an equitable right to restrain the corporation from enforcing the notices were to arise, it
would have to be shown either that there had been some bad faith, misconduct or abuse of powers on the part of the corporation
or that Simpsons had so altered their position on faith that there would be speedy acquisition of the North Road site that the long
delay had put them into an unfair position; but there was 484 no evidence that either of these factors existed (see p 494, letter c,
post).
Decision of the Court of Appeal ([1962] 3 All ER 75) affirmed.

Notes
The Housing Act, 1936, Part 5, which dealt with the provision of housing accommodation by local authorities ha.s been almost
wholly repealed and replaced by Part 5 of the Housing Act, 1957.
As to waiver of rights under a notice to treat, see 10 Halsburys Laws (3rd Edn) 67, para 105; and for cases on the subject,
see 11 Digest (Repl) 178, 455, 185, 519, 231, 954.
As to acquisition of land for housing purposes, see 19 Halsburys Laws (3rd Edn) 688, 689, para 1101; and for cases on the
subject, see 26 Digest (Repl) 699703, 114137.
For the Housing Act, 1936, s 72, s 74, see 11 Halsburys Statutes (2nd Edn) 514, 517; and for the Housing Act, 1957, s 92, s
97, replacing those sections, see 37 Halsburys Statutes (2nd Edn) 399, 402.

Cases referred to in opinion


Galloway v Mayor and Commonalty of London (1866), LR 1 HL 34, 35 LJCh 477, 14 LT 865, 30 JP 580, 11 Digest (Repl) 118,
112.
Grice v Dudley Corpn [1957] 2 All ER 673, [1958] Ch 329, 121 JP 466, [1956] 3 WLR 314, 3rd Digest Supp.
Tiverton & North Devon Ry Co v Loosemore (1884), 9 App Cas 480, 53 LJCh 812, 50 LT 637, 48 JP 372, 11 Digest (Repl) 231,
954.
Appeal
This was an appeal from a decision of the Court of Appeal (Ormerod, Upjohn and Pearson LJJ) made on 7 June 1962, and
reported [1962] 3 All ER 75, allowing the appeal of the respondent corporation from an order of Buckley J dated 25 January
1962, made against the corporation and which declared that a notice to treat dated 13 August 1952, and a notice of entry dated 22
October 1953, served on the appellants by the corporation in pursuance of the Borough of Hendon (North Road, Edgware)
Compulsory Purchase Order, 1952, were no longer of legal effect. The facts are summarised in the judgment of Upjohn LJ
[1962] 3 All ER at pp 7780, and sufficiently appear in the opinion, post.

R E Megarry QC and J D F Moylan for the appellants.


J T Molony QC and F H B W Layfield for the corporation, the respondents.

Their Lordships took time for consideration

6 May 1963. The following opinion was read.

LORD EVERSHED (read by Lord Cohen). My Lords, I have found this somewhat intricate case one of no little difficulty and
one in which, having regard to the increase in the value of land during the past ten years, it is impossible not to feel some
sympathy for the appellantsand not the less so since they were successful at first instance before Buckley J But I think it fair
also to the respondents (to whom I shall hereafter refer as the corporation) to pay at least sympathetic regard to the difficulties
which faced the corporation and which were at least in some measure due to the delaying tactics from 1952 to 1954 of the
appellants.
The relevant facts are fully and clearly stated in the report of the case before the Court of Appeal (Ormerod, Upjohn and
Pearson LJJ). I understand from the report that the narrative was in substance taken from the judgment of the Court of Appeal
delivered by Upjohn LJa. In the circumstances I do not propose to narrate the facts again though to some episodes in the story it
will be necessary for me to make more detailed reference hereafter. At the trial neither side chose to call any oral evidence. As
regards matters of fact, therefore, the case fell to be decided on the considerable bundle of documents which has 485 comprised
Appendix II before your lordships and consists entirely, as I understand, of documents (or of copies of documents) produced by
the corporation. Although the bundle was labelled Agreed Correspondence it consists in fact for the greater part of Minutes
and Reports of the council of the corporation and of committees of the council (or of extracts therefrom) and of reports and other
memoranda made by officers of the corporation and submitted to the council or its committees. It is from this material that, so far
as necessary, conclusions must be reached on such matters of fact as the attitude and intention from time to time of the appellants
and the corporation respectively and of the reasons for, and the effect of, the delay that undoubtedly occurred.
________________________________________
a The facts, as stated by Upjohn LJ, are set out in [1962] 3 All ER at pp 77 to 80

My lords, I will now refer to certain essential facts and dates which are not the subject of any controversy and then attempt
to formulate as a single question (which I think can fairly be done) the problem which is submitted for your Lordships decision.
The appellants are a limited company which has at all material times conducted the business of second-hand motor car dealers on
a piece of ground slightly less than half an acre in extent situated at the junction of North Road with a main road formerly part of
the Roman highway of Watling Street known further south as the Edgware Road but in the borough of Hendon called Burnt Oak
Broadway. I shall hereafter refer to the site as the North Road site. Until the month of August, 1959, the freehold of the site
belonged to Mrs Simpson the mother of the principal officer of the company and was let by her to the company; but on 20 August
1959, the property was conveyed by Mrs Simpson to the company. In the bundle Appendix II certain of the letters were, naturally
and inevitably, written to or by Mrs Simpson (or persons acting on her behalf) and others were written to or by the company (or
persons acting on its behalf). It has, however, been agreed throughout the litigation that no significance for any purpose can be
attached to the different interests of Mrs Simpson and the company. I shall therefore hereafter refer to the appellants as
Simpsons and in my references to the documents use that name indifferently to mean Mrs Simpson or the company. The North
Road site was at all material times substantially if not entirely unbuilt on.
On 13 August 1952, the corporation by their town clerk served on Simpsons a notice to treat in respect of the North Road
site stating that the corporation required to purchase such site for the purposes and under the provisions of the Housing Acts,
19361949, and the Acquisition of Land (Authorisation Procedure) Act, 1946. The service of this notice followed the
confirmation by the Minister of Housing and Local Government on 30 July 1952, of a compulsory purchase order in respect of
the North Road site, known as the Borough of Hendon (North Road, Edgware) Compulsory Purchase Order, 1952. The
essential terms of the order are these:

the.. corporation are under s. 74 of the Housing Act, 1936, hereby authorised to purchase compulsorily for the
purpose of providing housing accommodation under Part 5 of the Housing Act, 1936, the land which is described ,

being the North Road site. No objection was taken by Simpsons to the notice to treat, so that there was no inquiry held following
its service, and in accordance with the terms of the Acquisition of Land (Authorisation Procedure) Act, 1946, the notice became
finally effective and incapable of challenge at the end of six weeks from its date (see Sch I, Part 4, s 15 and 16 of the Act of
1946). About a year later, namely, on 22 October 1953, the corporation by their town clerk served on Simpsons a notice of
intended entry on the North Road site.
It is not open to doubt that at the date of the notice to treat and equally at the date of the notice of entry it was the real and
immediate intention of the corporation to use the North Road site for the purpose of erecting thereon at first nine, and later
twelve, residential flats and they had in the year 1953 accepted a tender for such purpose and also had taken steps towards the
borrowing of the 486 moneys which they would require therefor. It is equally clear that at the date of the issue of the writ in this
action on 13 August 1959, the project for the erection of flats on the North Road site had (and I adopt the word used in argument
by the learned counsel for Simpsons) evaporated. In its place no other project had come into existence either precise in
character or intended to be made immediately effective. On the contrary it is, I think, clear that the corporation had concluded (1)
that the development of the North Road site as a separate entity was not practical or realistic, (2) that the development of the
North Road site would be and have to be undertaken as part of and in conjunction with the development of a substantially larger
area of which the North Road site formed but a part, and (3) that in the circumstances the actual development of the North Road
site would not be likely to occur for some appreciable timenot, in the language used by the housing committee in October,
1957, in the foreseeable future. When this change of plan became known to Simpsons and the corporation were asked by
Simpsons agents in August, 1958, whether in truth the corporation were no longer proceeding with the acquisition of the North
Road site the council of the corporation considered the question put to them and then resolved to proceed with the acquisition of
the North Road site and Simpsons were so informed by the town clerk on 13 November 1958; and thereafter the town clerk
sought to obtain from Simpsons or their solicitors a draft form of appropriate contract.
Though it will be necessary for me, as I have earlier stated, to retread to some extent the pathway from 1952 to 1959 and
note more particularly some of the landmarks on it, the brief statement which I have made of the contrast between the view in
1952 and that in 1959 is, I believe, a fair one both for Simpsons and the corporation and enables me fairly to formulate the
problem thus: In the circumstances were Simpsons entitled to claim, as they did in their writ, that the notice to treat of 1952 is no
longer properly available to the corporation so as to enable them to acquire the North Road site? Such is my formulation of the
problem in the shape of a single question and that is the question which your lordships have to answer Yes or No. But it is
also appropriate and useful to subdivide the single question thus: Has the delay since August, 1952, been such, or has the
corporations change of intention been such, as severally or together to disable the corporation from proceeding under the notice
to treat? Has the corporations conduct been such as to amount to an effective abandonment of their rights under the notice to
treat? In light of all the circumstances of the case ought Simpsons now in equity to be relieved of any further obligation under
the notice to treat; so that if and when in truth the corporation wish positively to acquire the North Road site they must start
afresh and (consequentially) pay to Simpsons compensation based on present day land values?
The subdivision which I have made of my single question is intended to show what are the grounds on which Simpsons
invite your lordships to give an answer favourable to themselves. They are one or more of the following: undue delay on the
corporations part, abandonment by the corporation of the rights vested in them by virtue of the notice to treat, disablement by the
corporation from acting on the notice because their present intentions are outside the scope of the purposes permitted by the
notice and the compulsory purchase order and, finally, because on a proper view of all the circumstances of the case it is now
contrary to the principles of equity that the corporation should be able to proceed on their notice to treat to acquire the North
Road site.
My Lords, I do not doubt that, if Simpsons are able to establish one or more of the grounds which I have stated, then they
are entitled to succeed. That such is the law affecting a body which like the corporation is invoking statutory powers to acquire
compulsorily the land of a private owner was stated and justified by Upjohn LJ ([1962] 3 All ER at p 81; [1963] 1 Ch at p 82), in
delivering the judgment of the Court of Appeal in the 487 present case where he founded himself on his own analysis in deciding
as a judge of first instance the case of Grice v Dudley Corpn. I think it unnecessary to say more than that I fully accept as correct
the learned lord justices analysis. I have also in mind the references to which your lordships attention was drawn in the
speeches in your lordships House of Lord Cranworth LC in Galloway v Mayor & Commonalty of London ((1866), LR 1 HL 34
at p 43), and of Earl Cairns, Lord Blackburn and Lord Bramwell in Tiverton & North Devon Ry Co v Loosemore ((1884), 9 App
Cas 480 at pp 489, 496). In truth the principles of law applicable were not really in dispute in the present case; the question was
as to their application in light of all the facts. I add only this on the law that I think it right to say, as counsel for the corporation
submitted, that as regards delay the court will and should be somewhat less strict in the case of a local authority exercising the
powers and duties relating to housing and like matters than it was in the past in cases of railway corporations, which were
regarded as private undertakers seeking to make profits for themselves.
Counsel for Simpsons in his forceful argument relied on each of the several heads or grounds which I have stated, but he
made it clear, particularly in his reply, that in the end of all he rested his case primarily on the last-mentioned of such heads,
namely, the existence in his favour of an equity, an equitable right to be relieved from any further obligation under the notice to
treat. I note also and repeat that on each of the three other heads, delay, abandonment and what was called in argument the ultra
vires point, both courts below found concurrently on the facts adversely to Simpsons. I agree with these conclusions. But in
justice to counsel for Simpsons argument and because (as he strongly contended) all these matters bear also on his claim to
equitable relief, I must say something on each.
I have thought it convenient to take first the so-called ultra vires point, which is at least the most capable of precise
formulation and disposal. Put more accurately, the point is this: that where a body such as the corporation obtains a power of
compulsory acquisition which is expressed or limited by reference to a particular purpose then it is not legitimate for the body
concerned to seek to use such power for some different purpose or (to use the language of Lord Cranworth LC in Galloways case
((1866), LR 1 HL at p 43)) for a collateral purpose.
In the present case it is first essential to have clearly in mind the purpose for which the compulsory purchase order was
obtained and, therefore, for which the notice to treat was served. The relevant words occur in the order and have been already
cited but I repeat themfor the purpose of providing housing accommodation under Part 5 of the Housing Act, 1936; and
express reference was made to s 74 of that Act. It is right accordingly that I should make such references to the Housing Act,
1936, as, in the light of the arguments addressed to us, are relevant to the question. The title to Part 5 of the Act (as it had been
amended) is Provision of Housing Accommodation. It is therefore distinct for present purposes from Part 3 which bears the
heading Clearance and Redevelopment. Section 71 (being the first section in Part 5) imposes a duty on every local authority
periodically to review housing conditions in its area and to frame proposals in regard thereto. Section 72 is directed to the
method of provision of housing accommodation and by sub-s (1) provides as follows:

A local authority may provide housing accommodation(a) by the erection of houses on any land acquired or
appropriated by them; (b) by the conversion of any buildings into houses; (c) by acquiring houses; (d) by altering,
enlarging, repairing or improving any houses or buildings which have, or an estate or interest in which has, been acquired
by the local authority.

By s 73 power is conferred on local authorities to acquire land for the provision of housing accommodation and s 74 is directed to
the mode of such acquisition, 488including the means of compulsory purchase orders. The proviso to s 74(2) is in the following
terms:

Provided that a local authority shall not be authorised to purchase any land compulsorily for those purposes unless it
appears to the minister that it is likely to be required for those purposes within ten years from the date on which he
confirms the compulsory purchase order.

There follow, in s 79 and s 80 provisions giving powers to a local authority to deal with land acquired for the purpose of housing
accommodation together with supplementary powers in connexion therewith. It is sufficient for present purposes to note that
such powers extend to the laying out and construction of streets and open spaces and (with the consent of the minister) to the
provision and maintenance of places of worship, recreation grounds, shops and workshops as may be appropriate for the benefit
of the occupants of the contemplated houses.
Although, as I have earlier observed, it was the present intention of the corporation in 1952 to use the North Road site for
the purpose of erecting flats thereon, the terms of the compulsory purchase order (and of the notice to treat) clearly do not, in my
opinion, limit the use of the North Road site, when acquired, to such a use. Had there been a public inquiry, it might possibly
have been otherwise. But it is, in my opinion, clear that the corporation, having acquired the North Road site, would be entitled
to use it for any of the purposes comprehended within Part 5 of the Act of 1936. In fact as late at any rate as November, 1955, the
original project may be said to have persisted for, in a schedule to the borough treasurers report to the council of 1 November
1955, the North Road site was shown as appropriated for twelve flats and the symbol N attached to the reference indicated that
the project was not to be deferred. In fact, however, the attainment of the project by this time had been frustrated for reasons for
which, as counsel for the appellants conceded, the corporation could not be regarded as blameworthy. To this aspect of the matter
I shall have to return when I come to the questions of delay and abandonment. In fact the project for the immediate erection of
flats evaporated and thenceforth, as I have earlier stated, the North Road site was treated by the corporation as appropriate for
development or redevelopment as part of a substantially larger area. The reference to redevelopment is first found in the
councils resolution of 10 October 1955, and thereafter recurs from time to time. The word is no doubt particularly appropriate to
schemes of the kind contemplated in Part 3 of the Housing Act, 1936, and also to schemes under the Town and Country Planning
legislation. And so it was contended by counsel for the appellants that, at any rate from the end of 1955 onwards, the corporation
must be treated as intending and indeed (so far as they came to any decision) deciding that the North Road site should be utilised
for redevelopment under the town and country planning legislation or, perhaps, under Part 3 of the Housing Act, 1936, as distinct
from the purpose of providing housing accommodation under Part 5 of the last-mentioned Act; and that such use being (to use
Lord Cranworths word) collateral ((1866) LR 1 HL at p 43) to the originally specified purpose is inevitably beyond the powers
conferred by the compulsory purchase order.
My lords, I have been unable to accept the argument. I do not take up your lordships time by further reference to the
numerous documents including eight plans which were put before your lordships. It is to my mind sufficient to say that, although
from the beginning of 1956 onwards the actual use to which the North Road site would ultimately be put became vague if not
visionary, it cannot be said that in all that occurred during this period the corporation at any time determined on a use of the site
which would be outside the purposes of Part 5 of th Housing Act, 1936, or on a use of any of the suggested redevelopment area
which would be inconsistent with the North Road site being utilised in fact by the corporation for any purpose other than the
purpose of providing 489 housing accommodation within the scope of Part 5 of the Housing Act, 1936; and I note that the
discussions which did from time to time take place in regard to the North Road site continued to be treated as appropriate to the
councils housing committee.
It is right, however, that before leaving this part of the case I should refer to the language used by Upjohn LJ in the report of
the judgment of the Court of Appeal ([1962] 3 All ER at p 80; [1963] 1 Ch at p 81, 82).

It is only necessary to add that the resolutions passed in 1958 and 1959 show that the corporation do intend one
day to acquire the North Road area (including, of course, the North Road site) and to redevelop it for housing purposes.
Simpsons do not challenge the bona fides of that intention on the part of the corporation.

On the face of the language cited it would appear to have been the view of the Court of Appeal that the learned counsel for
Simpsons had conceded that, right up to the date of the writ, the corporation bona fide intended to use the North Road site for
housing purposes. If such was the view of the Court of Appeal, then in justice to counsel for Simpsons it must be treated as clear
that no such concession was made or intended. True it is that Simpsons by their counsel have never suggested and do not suggest
that the corporation throughout acted otherwise than in good faiththat they never pretended to have one purpose in view while
entertaining in fact another. But it is and has been Simpsons contention that the corporation in fact not only abandoned their
their original project as regards the North Road site, but also, having done so (in so far as they had any intention at all in regard to
it), no longer in truth or reality intended the use of the site to be for the a provision of housing accommodation.
I have said so much in justice to counsel for Simpsons (and counsel for the corporation has not contested counsel for
Simpsons statement) but none the less, for the reasons which I have given, I reject what has been called the ultra vires point
submitted on Simpsons behalf. And having done so, as it seems to me, the point ceases to have further relevance. In other
words, I cannot think if the point is not made good, that it can be relevant to the existence of the alleged equity. At the date of the
writ either the corporation had exceeded their powers or they had not; and if, as I think, the latter is the true view, then I know of
no principle or logic which would justify the court in restraining the corporation from invoking their existing powers because (as
it was suggested) they had at least come near to having exceeded them.
I turn to the question of delay: and since on such question (which is essentially one of fact) the conclusions of both courts
below have been concurrently in the corporations favour, I shall deal shortly with it. It is not in doubt (and counsel for Simpsons
so conceded) that the delay which occurred during the period described by Upjohn LJ ([1962] 3 All ER at p 77; [1963] 1 Ch at p
61) as the first phase, that is down to November, 1954, should be regarded as having been due to Simpsons. It may well be, as
counsel for the corporation suggested, that had it not been for such delaying tactics on Simpsons part the projected building of
the flats would have become a reality by the end of 1954. As it was, but through no fault of the corporation, the project could not
then be effected. It is to be noted that at this time Simpsons acquired another property at Colindale not far distant and suggested
to the corporation that for the purpose of their proposed housing the Colindale site should take the place of the North Road site.
It is clear also that the corporation had to give attention to certain instructions from the central government relating to economy.
In the circumstances already observed the corporation proceeded to consider the North Road site in relation to the more general
idea of redevelopment of a more substantial area. So there passed the period of rather more than 490 four-and-a-half years until
the date of the issue of the writ. My lords, it cannot, in my opinion, be fairly said of the corporation that during this period they
were guilty of inanition or procrastination. As I have earlier observed, we are here concerned with a responsible local authority
having public duties to perform and not with the promoters of an intended profit-making business like a railway company. In the
circumstances I am quite unable to take on this question any different view from that which commended itself to the two courts
below and I therefore reject the argument for Simpsons based on delay.
I come then to the suggestion of abandonment; and though, again, I think the question substantially one of fact and observe
that on it the courts below have again found concurrently for the corporation, I must deal a little more fully with it, or rather with
certain specific incidents relevant to it; because these incidents have been particularly (and naturally) stressed in the argument for
equitable relief.
I have already referred, on the question of delay, to what happened during the period called phase one in the judgment of
the Court of Appeal; and to the fact that the failure to make effective the project which was originally in the mind of the
corporation was the fault of Simpsons rather than of the corporation. Indeed, as I have also mentioned, it appears that as late as
November, 1955, the schedule to a report made by the borough treasurer showed the North Road site as one still intended to be
used for building flats and showed also that the project was not to be delayed. There can, I conceive, be no question then of
anything approaching abandonment of their notice to treat on the part of the corporation in phase one. I pass accordingly to the
next period called phase two in the judgment of the Court of Appeal ([1962] 3 All ER at p 78; [1963] 1 Ch at p p 61, 62). The
first significant incident in this period is dated 6 February 1956, when there was a resolution of the council recording that the
development of the North Road site as a separate entity was not a sensible or practical idea and which accordingly rescinded the
resolutions passed in 1953 for obtaining the loan of the necessary money and for authorising the town clerk to take legal
proceedings against Simpsons in order to effect possession. It will be appreciated that at this time the idea, and indeed the duty,
of the corporation to consider the redevelopment of a larger area than the North Road site had entered on the scene, and so from
now on the North Road site was inevitably regarded as part of the larger area.
In the meantime there had been enacted the Town and Country Planning Act, 1954, by s 33 of which provision was made for
varying the compensation to be paid to persons whose land was being compulsorily taken, if such acquisition took effect within
the period of five years after the private owner had put the question to the local authority. Accordingly and in light of that
legislation I find that on 9 July 1956, the corporation resolved that the acquisition of the North Road area should be effected
within five years from that date. I have now reached what the Court of Appeal called the end of phase two: and in my opinion it
is no less clear in regard to this phase than it was in regard to phase one that so far there had been nothing done or left undone by
the corporation which could fairly amount to abandonment of their rights under the notice to treat. I, of course, appreciate that
the original project for building twelve flats had evaporated and that in its place the development of the North Road site was to
be undertaken as part of the development of a larger area. But I have already, I hope, made good the point that the purpose for
which the original compulsory purchase order was made cannot be limited to what I will call the flat project, but must extend
to any utilisation of the site within Part 5 of the Housing Act, 1936.
I come then to the succeeding and final phase in the story when, according to the records, events did undoubtedly occur
which I must concede are of real significance on this question of abandonment. The first such incident is the 491 resolution of
the council dated 7 October 1957: for by that resolution the council rescinded the resolution previously referred to of 9 July
1956, recording that it could no longer be regarded as realistic to envisage the acquisition of the North Road site within the five-
year period. Indeed at this time it is clear enough that the North Road site, as part of an area ultimately to be taken over by the
corporation, had dropped to the bottom of the list of such areas which were under consideration by the council. In the same
month there is the report to the housing committee by one of the corporations officers in which is found the phrase that the
acquisition of this North Road site is something not to be regarded as likely to be effective in the foreseeable future. Then, on
11 November 1957, at a further meeting of the council resolutions are passed which record the fact that there is no likelihood of
the acquisition of the North Road area in the foreseeable future, and that such acquisition should therefore be deferred.
These resolutions and other events were, of course, unknown to Simpsons, who appear to have supposed that the council
were still debating the possibility of exchanging for the purposes of their requirements the newly acquired Colindale site for the
North Road site. Indeed it seems that, at the end of the year 1955, Simpsons had made application to the Central Land Board for
the purpose of fixing the sum of compensation that should be payable to them out of the so-called 300,000,000 fund. It is in
relation to this application that the last, and perhaps the most striking, of all the incidents occurred. In the month of August,
1958, Simpsons having inquired from the Central Land Board what progress was being made in regard to their application, the
Central Land Board in turn made inquiry of the town clerk of the corporation and, as a result, on 11 August 1958, received from
the town clerk a reply which concluded with the highly significant sentence: I may say that at the present time it is not the
intention of the corporation to proceed to acquire the North Road site. There was some argument before your lordships of the
exact meaning which should be given to this sentence, particularly whether the words at the present time should be related to
the intention or to the word acquire. For my part I cannot feel any real doubt that the words I have quoted mean what on the
face of them they appear to say, namely, that the corporation had not then (that is at the date when the letter was written) any
present intention to proceed with the acquisition of the North Road site. And this indeed was how the Central Land Board
themselves read the letter; for in their letter to Simpsons of 19 August they stated that, according to the information then available
to them, the corporation are not proceeding with the acquisition of the North Road site. As a result Simpsons not unnaturally
wrote on 25 August to the corporation to inquire whether, as they understood, the fact was that the corporation had no longer any
intention of acquiring the site. As earlier recorded, on the initiative of the town clerk (who, as Upjohn LJ ([1962] 3 All ER at p
80) pointed out, evidently felt some disquiet about the way in which the matter was being dealt with) the whole question was
fully and carefully considered by the housing committee and later by the council which on 10 November 1958, finally resolved to
proceed with the acquisition of the North Road site and instructed the town clerk so to inform Simpsons. As also earlier stated,
the town clerk did so inform Simpsons and thereafter pressed them to prepare and submit to him a draft form of appropriate
contract.
It cannot, I think, be in doubt that in a case such as the present a local authority may abandon a right to purchase
compulsorily land in respect of which a compulsory purchase order has been obtained: and that if such abandonment is accepted
by the owner then the compulsory purchase order ceases to be effective. As is pointed out in one of the cases, the quasi
contractual relationship which the notice to treat has caused to come into existence is then determined. I also accept counsel for
the appellants contention that the private owner might not be willing to accept the abandonment, but could proceed to enforce his
claim 492 under the notice to treat; and that in such event it would not be open to the local authority to rely on their own
wrongful act of abandonment to defeat the private owners claim. But nothing of that kind occurred in the present case. As was
pointed out by Upjohn LJ in the Court of Appeal ([1962] 3 All ER at p 82; [1963] 1 Ch at p 85), it may well be that, if the town
clerks letter of 11 August 1958, had been addressed not to the Central Land Board but to Simpsons, it would have been open to
Simpsons forthwith to reply to the effect that they accepted the abandonment of the acquisition of the North Road site which the
town clerks letter indicated. But the letter was not in fact written to Simpsons and, although it cannot be doubted that during this
period that which had originally been a presently intended project had become vague and visionary, yet on the whole I cannot
find in the incidents which occurred and the communications which passed between Simpsons and the corporation anything
which could fairly amount to an abandonment by the corporation of their rights under the compulsory purchase order. I agree,
therefore, on this matter with the view of the courts below.
There remains therefore only the question of alleged equitable right which, as I earlier stated, counsel for Simpsons had put
in fact in the forefront of his case. It was indeed on this ground that Buckley J decided in Simpsons favour. In reaching his
conclusion he asked himself the question whether the minister would have confirmed a compulsory purchase order in terms
similar to that originally put before him in 1952 had it been submitted by the corporation in 1959; and it was the view of the
learned judge that he would not. With all respect to the learned judge I agree with the Court of Appeal that such a test ought not
to be used in determining whether Simpsons are entitled to the equity which they claim. The compulsory purchase order of 1952
became finally effective and not subject to challenge, and I cannot, therefore, think that an equity arises to have it, in effect, set
aside on the ground that a similar compulsory purchase order might not have been confirmed by the minister seven years later.
However, as the Court of Appeal said ([1962] 3 All ER at p 84; [1963] 1 Ch at pp 86, 87) the learned judge did not really found
his judgment on this factor but on the general change of circumstances.
I did for myself attach at one time some importance to the proviso to s 74(2) of the Housing Act, 1936, which I have earlier
citedb, to the effect that the minister should not confirm a compulsory purchase order submitted to him unless it then appeared
reasonably likely that the land proposed to be acquired would be so acquired within ten years. I had thought that, although in
1952 clearly the minister would have been rightly informed of an intention to acquire the North Road site very well within the
ten-year period, subsequent events had shown that such intention had long since ceased to operate; so that, as it were, the premise
on which the compulsory purchase order had originally been granted had been destroyed. Counsel for Simpsons did not however
himself adopt or press this point and I think, on reflection, that it is of no greater validity than the reasoning which influenced
Buckley J.
________________________________________
b Page 489, letter a, ante

I should, I hope, be the last to suggest that the proper application and, where necessary, extension of equitable principles in
the present day and age, when so much human activity is governed and controlled by legislation, should be restricted and I have
in mind the picturesque language once used by Harman LJ when he said that he thought that equity was not presumed to be of an
age past childbearing. But on reflection I have been unable to accept counsel for Simpsons argument. To accept it would appear
to me to involve considerations of this kind and no more: that Simpsons had indeed substantial points on which they had argued
for the view that the notice to treat had ceased to be effective but that such substantial points had not in the end been quite
substantial enough, and that, therefore, as a kind of consolatory award Simpsons 493 should be granted under the guise or
heading of equitable relief the result which they had on his other grounds failed to obtain. Put alternatively and briefly it seems
to me that it would amount to no more than saying that in all the circumstances it would be perhaps rather hard luck for Simpsons
now to have to part with their North Road site at a price which no longer represented modern prices. It would, in my opinion, be
no more than palm tree justice. I cannot think that this can suffice for the establishment and enforcement of an equity. If such an
equitable right is to be found then it must, as I conceive, be based on the view that to permit the corporation to continue to
enforce their rights under the original compulsory purchase order must in some real sense be against good conscience. In order to
achieve such a result it seems to me that it would be necessary to show one or both of the following: that there had been on the
part of the corporation something in the nature of bad faith, some misconduct, some abuse of their powers; that there had been on
the part of Simpsons some alteration of their positionsomething must have been done or not have been done by them on the
faith and in the belief that there would be a speedy acquisition of the North Road site; in other words that they had in some sense
been put into an unfair position because of the long period which had elapsed since the service of the notice to treat.
My lords, I can find no sign of either of the considerations which I have attempted to state. It is not suggested on Simpsons
part that the corporation had at any point been guilty of any bad faith towards them nor, as I have earlier observed, have I been
able to find anything which could fairly be called misconduct or abuse of powers on their part. Equally it seems to me from a
reading of the documents that Simpsons were in truth quite content from first to last that they should eventually be dispossessed
of the North Road site. They did not at the time of the service of the notice to treat make any objection; they have continued ever
since to carry on their business of dealing in second-hand motor cars and at no stage have they suggested that their business
activities were placed in any difficulty by the presence or pendency of the notice to treat.
My lords, I do not think the matter is capable of lengthy exposition but, for the reasons which I have attempted to state, I
have felt compelled to the conclusion that there is here no such equity as Simpsons have attempted to assert and that there would
be no good ground for now restraining the corporation from giving effect to the notice to treat of 1952. I would therefore dismiss
the appeal.

LORD COHEN. My lords, speaking now on my own behalf, I concur in the judgment which I have just read.

LORD JENKINS. My lords, I concur in the opinion which has just been read by my noble and learned friend Lord Cohen on
behalf of Lord Evershed.

LORD GUEST. My lords, I also concur. Lord Hodson, who is unable to be present here this morning, has asked me to say that
he also concurs.

Appeal dismissed.

Solicitors: Straker & Primbak (for the appellants); Town clerk, Hendon (for the corporation, the respondents).

C G Leonard Esq Barrister.


494
[1963] 2 All ER 495

Dorling v Honnor Marine Ltd and Another


INTELLECTUAL PROPERTY; Copyright: LEISURE AND LICENSING

CHANCERY DIVISION
CROSS J
6, 7, 8, 13, 14, 15, 18 MARCH, 10 APRIL 1963

Copyright Infringement Boat plans Plans for constructing dinghy and parts of dinghy Design not registered Dinghies
and parts built without consent of owner of copyright Plans artistic works Reproduction in three-dimensional form Whether
reproduction constituted infringement of copyright Copyright Act, 1956 (4 & 5 Eliz 2 c 74), s 9(8), s 10(2), (3), s 49(1), (7).

Licence License to build boats Revocation Notice denying grant of licence and requiring cessation of licensed actions
forthwith Whether revocation of licence Effect.

Licence Assignment Personal licence Whether Copyright Act, 1956(4 & 5 Eliz 2 c 74), s 49(7) applicable to purported
assignment of licence.

A letter from the grantor of a revocable licence to the licensee stating that the licensee has no licence and ordering him to stop
forthwith doing what the licence permits him to do has the effect of revoking the licence, though if the licensee continues to do
what the licence permits he cannot be treated as a wrongdoer for so continuing, until the expiry of the period which would have
constituted reasonable notice of revocation (see p 502, letter b, post).
Principle in Minister of Health v Bellotti ([1944] 1 All ER 238) applied.
In September, 1959, the plaintiff, who was the designer of a sailing dinghy called a Scorpion, gave an informal licence to
the second defendant, who was a boat builder, to build Scorpions (including constructional kits of parts) and supplied him with
the plans of the boat. The shape or configuration of the completed boat was not registered as a design under the Registered
Designs Act, 1949. After negotiations between the parties with a view to their jointly building Scorpions had broken down, the
plaintiff on 29 June 1960, wrote to the second defendant saying that he had not been given a licence to build Scorpions and
requiring him forthwith to cease using the design and building the boats. Between 30 June and 5 September 1960, the second
defendant completed his orders in hand and accepted further orders. On 5 September the second defendants business, together
with all existing contracts, licences and orders, was transferred to the first defendants, a company incorporated for that purpose,
and the first defendants carried out outstanding orders received by the second defendant before that date. The company accepted
further orders and sold or offered for sale Scorpions or kits of parts of Scorpions accompanied by building instructions, including
photographs of some of the parts. In an action by the plaintiff for injunctions to restrain the defendants from building Scorpions
or parts of Scorpions without the plaintiffs consent and from infringing the plaintiffs copyright in the plans, it was common
ground that the plans were an artistic work within the Copyright Act, 1956, that the plaintiff was prima facie entitled to restrain
anyone from reproducing them or a substantial part of them without his consent, that reproduction included their conversion into
a three-dimensional form, and that he could have registered the shape or configuration of a Scorpion boat as a design under the
Registered Designs Act, 1949, but could not have registered a design in respect of the plans. It was admitted also that there had
been industrial application, within s 10(2) of the Copyright Act, 1956. The court found as a fact a that persons who were not
experts in relation to boats would not have felt any strong conviction that a completed Scorpion was a three-dimensional version
of the plan, but that some of the parts, together constituting a substantial part of the whole boat, would have appeared to be
reproductions of the corresponding drawings on the plans.
495
________________________________________
a See p 504, letter c, post

Held (i) the action must be dismissed


(a) as against the second defendant, because the reasonable period for notice of revocation of the licence was six months and
accordingly, under the principle stated at p 495, letter c, the second defendant would have been entitled to build, etc, Scorpions
until the end of 1960, and in fact he had not himself built Scorpions since 5 September 1960, when his business was assigned to
the first defendants (see p 502, letter e, post); and
(b) as against the first defendants, because the provisions of s 10(2) b of the Copyright Act, 1956, were satisfied and
accordingly, by s 10(3)c, the plaintiff, as he had exploited his copyright in the plans industrially, ceased to be entitled to
protection against the doing of anything which would, if a design had been registered in respect of the boat under the Registered
Designs Act, 1949, have been within the scope of the design copyright; accordingly, since the making and selling of the parts of
the boat for the purposes of their being put together by purchasers was something which would have infringed design copyright,
the plaintiff was not protected against it (see p 506, letters d and h, post).
________________________________________
b Subsection (2) and the material provisions of sub-s (3) of s 10 are set out at p 504, letter h, to p 505, letter b, post
c Subsection (2) and the material provisions of sub-s (3) of s 10 are set out at p 504, letter h, to p 505, letter b, post

(ii) the licence given to the first defendant was not assignable but was personal to the first defendant, and s 49(7) of the
Copyright Act, 1956, did not enable it to be assigned to the first defendants (see p 502, letter h, and p 503, letter b, post).
(iii) each of the parts making up a kit of parts was an object for the purposes of s 9(8) d of the Copyright Act, 1956, and, by
virtue of s 49(1), s 9(8) would not save the making of such parts as a non-expert would see to be reproductions of a substantial
part of the plans from being an infringement of the copyright in the plans (see p 504, letter e, post); moreover as a design could
not have been registered under the Registered Designs Act, 1949, in respect of the component parts of the boat, the parts were not
articles within s 10(2)(b) and (c) of the Copyright Act, 1956 (see p 506, letter c, post).
________________________________________
d The terms of s 9(8) are set out at p 503, letter i, post

Notes
As to infringement of copyright by reproduction, see 8 Halsburys Laws (3rd Edn) 426429, paras 776778 and Supplement
thereto.
As to certain designs excluded from copyright, see 8 Halsburys Laws (3rd Edn) 379381, para 696 and Supplement thereto.
For the Copyright Act, 1956, s 1, s 3, s 9(8), s 10, s 48, s 49, see 36 Halsburys Statutes (2nd Edn) 70, 76, 90, 9193, 142,
145.
For the Registered Designs Act, 1949, s 1, s 7, see 17 Halsburys Statutes (2nd Edn) 723, 729.
For the Registration of Business Names Act, 1916, s 8, see 25 Halsburys Statutes (2nd Edn) 1151

Case referred to in judgment


Minister of Health v Bellotti [1944] 1 All ER 238, [1944] KB 298, 113 LJKB 436, 170 LT 146, 60 TLR 228, CA, 11 Digest
(Repl) 581, 163.

Action
By his amended statement of claim in an action (the Chancery action) commenced by writ issued on 6 January 1961, the plaintiff,
Taprell John Dorling, sought (1) an injunction to restrain Honnor Marine Ltd and Timothy Patrick Honnor (Colonel Honnor),
the defendants, and each of them by themselves or by their 496 servants or agents from manufacturing, offering for sale, selling
or dealing in boats or parts of boats known as Scorpions or any boat or parts of boats of substantially the same design, without
the consent of the plaintiff, (2) an injunction to restrain the defendants and each of them by themselves or by their servants or
agents from infringing the plaintiffs copyright in plans relating to a sailing dinghy called the Scorpion by reproduction in
material form of parts of boats and from selling, offering for sale or by way of trade exhibiting such parts in public, (3) an
injunction to restrain the defendants and each of them by themselves or by their servants or agents from infringing the plaintiffs
copyright in the said plans by reproduction in the material form of photographs of boats or parts of boats or from by way of trade
exhibiting in public any such photographs, (4) an inquiry as to the damages sustained by the plaintiff by reason of breach of
confidence and contract and infringment of copyright and conversion and payment by the defendants of all sums found due on
such inquiry, (5) a declaration that the manufacture and sale of sailing dinghies known as Scorpions or parts thereof by the
defendants constituted an infringement of the plaintiffs copyright in the said plans. The defendants were (1) Honnor Marine Ltd,
a company incorporated on 8 July 1960, for the purpose of taking over the second defendants business of boat building and (2)
Colonel Honnor. A motion by the plaintiff on 6 January 1961, for interlocutory injunctions was stood over until the trial of the
action on the defendants giving undertakings to the same effect.
The Chancery action was heard together with an action (the Queens Bench action) commenced in the Queens Bench
Division by writ dated 23 November 1960, and transferred by order to the Chancery Division, in which Colonel Honnor was
plaintiff and the plaintiff in the Chancery action was defendant. In the Queens Bench action, in which Colonel Honnor claimed
684 odd for work done and material supplied to the order of the defendant (in connexion with Scorpion kits and parts
despatched by the plaintiff to the defendants customers) Colonel Honnor was given liberty to sign final judgment for 487 13s
2d, with leave to the defendant to defend as to the balance, and subject to a stay of execution.
The proceedings are reported in relation to the Chancery action only. The facts are set out in the judgment.
The cases cited belowe were referred to in argument in addition to that referred to in the judgment.
________________________________________
e Stenor Ltd v Whitesides (Clitheroe) Ltd [1947] 2 All ER 241, (1948), 65 RPC 1, Saltman Engineering Co Ltd v Campbell Engineering Co
Ltd (1948), 65 RPC 203, Aberdale Cycle Co Ltd v Charles Twigg & Co (1952), 69 RPC 137, Martin-Baker Aircraft Co Ltd v Canadian
Flight Equipment Ltd [1955] 2 All ER 722, [1955] 2 QB 556, Nichrotherm Electrical Co Ltd v Percy [1956] RPC 272, Terrapin Ltd v
Builders Supply Co Ltd [1960] RPC 128, Ackroyds (London) Ltd v Islington Plastics Ltd [1962] RPC 97

A D Russell-Clarke and W Bruce Spalding for the plaintiff.


D H Mervyn Davies for both defendants.

Cur adv vult

10 April 1963. The following judgment was delivered.

CROSS J read the following judgment. This judgment relates to two actions. The first, which I will call the Chancery action, is
between Taprell John Dorling (whom I will call Mr Dorling) as plaintiff and Honnor Marine Ltd (which I will call the company)
and Timothy Patrick Honnor (whom I will call Colonel Honnor) as defendants. By the Chancery action Mr Dorling seeks to
restrain Colonel Honnor and the company from selling boats or parts of boats called Scorpions of which Mr Dorling was the
designer. The second action, which I will call the Queens Bench action, is between Colonel Honnor as plaintiff and Mr Dorling
as defendant. By it Colonel Honnor claims a liquidated sum for work done and materials supplied to the order of Mr Dorling.
The Queens Bench action was transferred to this Division and the two actions were tried together.
Mr Dorling carries on business under the name of Collins Marine Equipment, but he was not at any material time
registered in that name under the Registration of Business Names Act, 1916. When the story starts in 1958, Mr Dorlings
business consisted solely in the sale of fittings for boats which he conducted from an office in the house of a friend, a Mr
Churchouse, in Barnes. 497Mr Dorling had recently designed a fourteen foot sailing dinghy, which he named the Scorpion, of
which he had made a prototype model. He joined with some friends in forming a so-called class association, consisting of
ownersor, rather, at that date, prospective ownersof Scorpions, the main object of which was to further the interests of the
Scorpion class of boats. In order to keep up the standard of the boats, the principle was laid down, which was embodied in r 7
of the Associations constitution, that professional boat builders should not be allowed to build Scorpions unless licensed to do
so by Mr Dorling and on payment of a designers royalty which was fixed at two guineas. It was further provided that Mr
Dorling should arrange for so-called kits of parts of Scorpions to be machined by a competent firm and that, if licensed
builders did not use these kits but built themselves from scratch, they should pay a further licence fee. Any amateurs, on the
other hand, could buy a copy of the plans from the class association for a small sum and make the boat themselves either from
scratch or from a kit of parts on payment of only the designers royalty. It was arranged that a company named Kitboats Ltd
should make the kits of parts of a second edition of the Scorpion to be known as Mark II, on the plans of which Mr Dorling
was working, and at about the beginning of 1959 Mr Dorling had granted licences to two boat builders, Moores of Wroxham and
the Aln Boatyard.
It is at this point that Colonel Honnor comes into the story. He was then carrying on business as a boat builder at Ivybridge
in Devonshire, and Mr Dorling was anxious that he should take up Scorpions and should become the licensed builder of them
in the West Country. A letter which he wrote to Colonel Honnor on 2 March 1959, contains the following paragraph:

The class association is looking for builders, one in each of various areas, who are able to produce boats with a good
finish. I should be very glad to hear whether you might be interested, and whether you might consider building a boat on
spec. so to speak, which, if you cannot find a customer for amongst your own circle, the class association would assist, by
passing on future inquiries.

Colonel Honnor replied on 9 March that he was interested and would like to have a copy of the class rules and the timber
schedule, so that he could give the idea further thought. On 9 April a Captain Calvert, who had ordered a kit of parts from
Kitboats Ltd asked Colonel Honnor whether he would build a Scorpion from this kit. Colonel Honnor agreed to do so and Mr
Dorling, who had in fact suggested the idea to Captain Calvert, was very glad that Colonel Honnor should build a Scorpion and
satisfy himself as to its good qualities. After some delay Colonel Honnor obtained Captain Calverts kit of parts from Kitboats
Ltd and began about August to build the boat, Mr Dorling having previously sent him some building instructions which he
supplemented with further instructions at the end of August. Meanwhile, Kitboats Ltd had given up making Scorpion kits and
Mr Dorling had entrusted the making of them to a firm at Richmond. He and the class association wanted the Scorpion to be
shown at the Boat Show to be held in London in January, 1960. The exhibition rules did not permit a boat to be shown by a class
association, and so Mr Dorling arranged to show it under his firm name of Collins Marine Equipment. He told meand I accept
itthat hitherto he had not intended to take any financial interest in the development of the Scorpion beyond the receipt of the
designers royalty, but that the necessity of taking the exhibition space in his firm name suggested to him that he might himself
engage in the building of Scorpions.
At the end of September, Colonel Honnor, having nearly completed the building of Captain Calverts model, suggested to
Mr Dorling that he should come down to Ivybridge to see the boat and advise him on the final details. There is a conflict of
evidence as to what happened at this meeting. Mr Dorling said 498 that, although he was prepared to give Colonel Honnor a
licence to build Scorpions, Colonel Honnor did not in fact signify his willingness to accept one. Colonel Honnor, on the other
hand, said that the first thing that he told Mr Dorling when he met him up at the station was that he was delighted with the boat
and had decided to build Scorpions, and from that moment onwards he considered that he had Mr Dorlings licence to do so. I
have no hesitation in preferring Colonel Honnors account to Mr Dorlings. In the first place, it is confirmed by Mr Churchouse
who said that when Mr Dorling returned to London he expressed delight that Colonel Honnor had agreed to take up the
Scorpion. Secondly, it is confirmed by some of the letters which Mr Dorling himself subsequently wrote. I would refer to one
dated 10 December 1959, to the Aln Boatyard, in which he says, among other things:

Moores of Wroxham are pricing their finished boats at 165 on the basis of the previous (Kitboats) Kit, which cost
them so much extra time in fiddling about with replacements and so on. Col. Honnor, at Ivybridge is producing at 150 as
he considers 165 a bit too conservative. He is wildly enthusiastic about the boat.

Later on, as will appear, negotiations took place between Mr Dorling and Colonel Honnor with a view to their co-operating in the
building of Scorpions, but there is, to my mind, no doubt that Mr Dorling gave Colonel Honnor a licence to build them on his
own account long before there was any idea of their being jointly interested in building them. There is no doubt that at that time,
at any rate, it was usual for licences by designers to boat builders to be given informally either orally or by letter without any
document defining their terms.
The firm at Richmond did not prove satisfactory as makers of kits, and by the beginning of November Mr Dorling and the
class association needed to find some other firm from whom they could be sure of getting some satisfactory kits made in time for
the show. In his difficulty Mr Dorling turned to Colonel Honnor who approached a firm of timber merchants, Reeves, Fox &
Elliott Ltd of Totnes, with whom he often did business. Only three kits were needed for the show, but an order for so few was not
an economic proposition, and Colonel Honnor, who had great faith in the Scorpion, was prepared to finance an order for
twenty-five. On 7 December Reeves quoted a price of 45 5s for each kit less 2 1/2 per cent discount for prompt payment. After
reporting to Mr Dorling, Colonel Honnor accepted this quotation, and it was agreed that the retail price to be quoted for them
should be 47 10s, Mr Dorling being very insistent that the price of the first batch of Scorpions should be kept as low as
possible. There is no doubt that Colonel Honnor was the only person in contractual relations with Reeves, Fox & Elliott, Ltd.
Whether as between himself and Mr Dorling, Mr Dorling had any legally enforceable interest in the kits or any right to have them
or any of them sub-sold to him at any particular figure is the main issue in the Queens Bench action to which I shall return later.
To enable Reeves, Fox, & Elliott Ltd to machine the kits as quickly and as satisfactorily as possible, Mr Dorling supplied to
Colonel Honnor for him to hand on to Reeves, Fox & Elliott Ltd not only the plans of the Scorpions in their then form, but also
an exercise book in his handwriting containing sketches and instructions. Some of the material in the exercise book which was
not in the plans as then existing was incorporated in the final plans of the Mark II which Mr Dorling got out later. There is
some material in the exercise book which would be useful to anyone, whether amateur or professional boat builder or timber
merchant, who was building his first Scorpion or his first kit of parts for Scorpions, and which is not in fact to be found
anywhere else. Then in the middle of December Colonel Honnor, anticipating that after the boat show he might receive a number
of orders for Scorpions, made arrangements for extending his premises by building a new hut. This hut was finished by about
March, 1960, and cost about 500. Reeves, Fox & Elliott Ltd succeeded 499 in producing two kits in time for the boat show and
Colonel Honnor came up to London to help Mr Dorling and Mr Churchouse at it. The Scorpion was a success at the show, and
Colonel Honnor and Mr Dorling conceived the idea that it might be to their mutual advantage to join forces in the building of
Scorpions and to form a company with this object, in which Mr Churchouse (who was prepared to put in some money) should
also have shares. When Colonel Honnor returned to Devon in January after the show, Mr Dorling went with him and stayed as a
guest at his house for some weeks during which the discussions continued. Mr Churchouse also gave up his existing job and took
a house in Devon for himself and his wife. Mr Dorling, or, rather, Collins Marine Equipment, received a number of orders for
kits of parts of Scorpions at or shortly after the show and Colonel Honnor also received some orders in January and February.
It was arrangedor, rather, I think it had always been understoodbetween Colonel Honnor and Mr Dorling that Mr Dorlings
customers should be supplied from some of the twenty-five kits ordered from Reeves, Fox & Elliott Ltd and ten kits were
despatched to these customers in the last week in March. On 31 March 1960, Colonel Honnor received a bill from Reeves, Fox
& Elliott Ltd for 484 17s 6d for these kits and the crates in which they were sent and paid it, less the 2 1/2 per cent discount, a
few days later. When and how much Mr Dorling was paid by his customers, I do not know.
At about this time Colonel Honnor decided that a business association between him and Mr Dorling in the building of
Scorpions would not be a success, and refused to continue the negotiations which had been proceeding over the last three
months. It therefore became necessary to decide what Mr Dorling should pay him for the kits and crates supplied by Reeves, Fox
& Elliott Ltd and also what part they were respectively going to play in the future in the building of Scorpions. An attempt was
made to submit the matter to the arbitration of a Mr Webb, the chief executive of The National Federation of Ship and Boat
Builders, but in the end this came to nothing. Finally, on 29 June 1960, Mr Dorling, who had meanwhile set up a boat building
establishment for himself in the West Country, caused his then solicitors to write to Colonel Honnor a letter which, after truly
stating that Colonel Honnor was building Scorpions and having kits of parts of Scorpions made for him, continued as
follows:

We write to inform you that this must stop immediately as you have no rights whatsoever to this design, as Mr.
Dorling has never assigned the rights to you nor has he ever given you a licence to construct these boats and their fittings or
arrange for these kits to be made. We therefore request on behalf of our client (a) that you stop using this design
immediately, (b) that you stop building any boats in accordance with this design and the fittings for use in them, (c) that
you stop building or having built any constructional kits, and/or stop being instrumental in having them constructed, (d)
that you return to us immediately any copies of Mr. Dorlings plans that you have.

Colonel Honnor, having taken legal advice, took no notice of this letter. As well as completing the orders for Scorpions which
he had in hand he accepted four further ordersone for a boat and three for kits of partsbetween 30 June and 5 September
1960. On 11 August he sent Mr Dorling a bill for 682 18s 5d in respect of the kits and crates for which Reeves, Fox & Elliott
Ltd had sent him a bill at the end of March and for certain other goods provided and for labour done for Mr Dorling, to which I
will refer in detail later.
Honnor Marine Ltd the company which is one of the defendants to the Chancery action, was incorporated on 8 July 1960,
for the purpose of taking over Colonel Honnors business. At all material times Colonel Honnor held fifty per cent of the shares,
the remainder being held by Mr Churchouse and a Mr Tweed. On 10 November 1960, the three gentlemen signed an agreement
whereby Colonel Honnor agreed to assign to the company as from 5 September the goodwill, plant and stock in trade of his boat
building business together with the benefit 500 of all existing contracts, licences and orders for the consideration there
mentioned, and Mr Churchouse and Mr Tweed agreed to lend the company certain sums there referred to. In fact the business
had been carried on in the name of the company since 5 September. As well as carrying out certain orders for Scorpion kits
received by Colonel Honnor before that date the company, between 5 September and the end of the year, itself took and executed
a number of similar orders and one order for the building of a Scorpion. The company had a stand at the boat show held at
Earls Court in January, 1961, and there offered for sale Scorpions and kits of parts of Scorpions. Colonel Honnor and later
the company had from time to time sent to Mr Dorling credit notes in respect of the designers fee of two guineas on the boats or
kits sold by them, but Mr Dorling never accepted payment of such fees.
Mr Dorling not having paid the bill sent to him on 11 August and a further bill for 1 5s 6d sent to him subsequently,
Colonel Honnor issued the writ in the Queens Bench action on 23 November 1960, claiming payment of 684 3s 11d. Mr
Dorling issued the writ in the Chancery action on 6 January 1961, and at once applied to the court by motion for interlocutory
injunctions. The motion was stood over until the trial on the defendants giving undertakings to the same effect as the injunctions
claimed. [His Lordship then stated the items making up the plaintiffs (ie, Colonel Honnors) claim in the Queens Bench action,
considered the admissions, evidence and contentions of the parties and gave judgment for the plaintiff for the removal of a stay of
execution on a judgment for 487 odd that Colonel Honnor had been given liberty to sign on 19 January 1961, with a stay of
execution, and for a further sum to be calculated in accordance with His Lordships decision. He continued:] Turning now to the
Chancery action, it follows from my finding that in September, 1959, Mr Dorling had granted Colonel Honnor a licence to build
Scorpions that Colonel Honnor cannot be liable for any sales of boats or kits effected before he received the letter of 29 June
1960. Mr Dorling claims that on receipt of that letter Colonel Honnor was bound to cease to accept any further orders for
Scorpions, or kits of parts of Scorpions, and that, as he did not do so, he and, a fortiori, the company are liable to him for
infringement of the copyright in the plans or, alternatively, for misuse of confidential information. I will deal first with the claim
based on breach of confidence. This is based entirely on the view that such of the material in the exercise book as is not
contained in the final plans stands on a different footing from the plans themselves. The plans can be obtained from the class
association by anyone on payment of a fee, but it is said that the extra information in the exercise book was confidential
information entrusted to Colonel Honnor for the sole purpose of getting Reeves, Fox & Elliott Ltd to make the twenty-five kits
and that even if neither he nor the company is guilty of infringement of the copyright in the plans, they have improperly availed
themselves of this confidential information by causing Reeves, Fox & Elliott Ltd (who had the exercise book) to make
Scorpion kits for them after 29 June 1960, without Mr Dorlings consent. I cannot accept this view of the status of the extra
information contained in the exercise book. Mr Dorling intended Reeves, Fox & Elliott Ltd to be the regular makers of
Scorpion kits. If the final plans had been available when the orders for kits were given and Reeves, Fox & Elliott Ltd having
studied the plans, had asked for some further information such as was contained in the exercise book, I have no doubt that Mr
Dorling would have given it to them at once. Indeed, I have no doubt that if some amateur, who had bought the plans with a view
to building a boat from scratch himself, had asked Mr Dorling for some further information such as is contained in the exercise
book, he would have given it to him without hesitation. It is, in my judgment, impossible to treat the information in the exercise
book as in a different category from the information in the plans. Accordingly, I think that the questions to be decided in the
Chancery action are, first, how far, if at all, what was done by Colonel Honnor or the company after 29 June 1960, was 501
covered by Mr Dorlings licence, and, if and so far as it was not, was it an infringement of any copyright in the plans?
On the first point the defendants admit that the licence was revocable. They say, however, first, that it was not revoked,
since the letter of 29 June denies in terms that it ever existed, and, secondly, that in any event it could not be revoked without
notice and that, in the circumstances, a notice of at least six months was needed. The case of Minister of Health v Bellotti shows
that if a licensor purports to revoke a revocable licence but gives insufficient notice of revocation, the licensee cannot ignore the
revocation and treat the licence as still subsisting. On the other hand, he cannot be treated as a wrongdoer for continuing to do
what the licence permitted him to do until the expiry of the period which would have constituted reasonable notice. The principle
of that case must, I think, apply equally if the licensor instead of giving insufficient notice gives no notice of revocation. Here it
is true that Mr Dorling was still more at fault for he wrongly denied that he had ever granted Colonel Honnor any licence at all.
But it would, I think, be unreasonable to draw a distinction between a letter from the licensor which says You have no licence to
do what you are doing. Stop at once, and a letter which says I summarily revoke your licence to do what you are doing. Stop
at once. Minister of Health v Bellotti shows that the latter letter is not a nullity, however unreasonable it is to give no notice.
That being so, I am not prepared to hold that the former letter is a nullity. The stop at once is a brutum fulmen in either case,
and the licensor shows his wish to bring the licensees activities to a stop as soon as he can just as clearly by the first form of
letter as by the second.
In my judgment, therefore, Mr Dorling, by the letter of 29 June 1960, revoked Colonel Honnors licence to build
Scorpions as soon as he could, though Mr Dorling could not treat Colonel Honnor as a wrongdoer for building Scorpions
without a licence until the expiration of a reasonable period. On the question of a reasonable period, I accept Colonel Honnors
view that six months would have been reasonable. It was submitted on behalf of Mr Dorling that the most that Colonel Honnor
could be permitted to do was to complete any orders taken before 29 June which were uncompleted on that day. I cannot agree.
Colonel Honnor had spent a substantial sum on the faith of his licence and had advertised himself as a builder of Scorpions. I
think that he was entitled to not less than six months notice.
If Colonel Honnor had then simply continued to accept and execute orders for Scorpions up to the end of 1960 and had
then given them up, Mr Dorling could not have complained. What happened, however, was that Colonel Honnor, after accepting
and executing some orders himself, on 5 September transferred his business, including his trade in Scorpions, to the company
which executed Colonel Honnors uncompleted orders, accepted and executed further orders itself and would have continued to
deal in Scorpions after the end of 1960, but for the issue of the writ in the Chancery action. A licence is, prima facie, personal
to the licensee and there is nothing in the facts of this case to justify the conclusion that Colonel Honnors licence to build
Scorpions was assignable: on the contrary, the licence was given to him because of his personal reputation as a good builder.
It is true that at the relevant time he held fifty per cent of the shares in the company and no doubt he could and did see to it that
the company maintained the same standard. If Mr Dorling had been on good terms with him, it may very well be that he would
have readily agreed to a transfer of the licence to the company. In my judgment, however, even if the licence had still been on
foot, Colonel Honnor could not have transferred it to the company without Mr Dorlings consent. In fact, however, the licence
had been revoked before 5 September 1960, and Colonel Honnor was only entitled to build Scorpions at that date because Mr
Dorling had given him no period of notice. A fortiori, therefore, he could not authorise the building of Scorpions by the
company.
502
On this part of the case counsel for the defendants called in aid s 49(7) of the Copyright Act, 1956, which runs as follows:

Where the doing of anything is authorised by the grantee of a licence, or a person deriving title from the grantee, and it
is within the terms (including any implied terms) of the licence for him to authorise it, it shall for the purposes of this Act
be taken to be done with the licence of the grantor and of every other person (if any) upon whom the licence is binding.

I cannot see how this provision helps them. It is not, as I read it, dealing with assignments of licences at all. The situation which
is envisaged is not that the grantee of the licence has put someone else in his place as grantee and has himself dropped out of the
picture, but that the grantee of the licence, instead of doing everything necessary for the exploitation of the licence personally, has
employed an agent or sub-contractor to do some of the necessary work for him. In that case, unless the licence on its true
construction obliged the grantee to do the work in question personally, the agent or sub-contractor is taken to have the licence of
the grantor. In this case, even if Colonel Honnor was still the grantee of the licence within the meaning of the subsection on 5
September 1960 (which I do not think he was), he did not then employ the company as an agent or sub-contractor. He purported
to put it in his place as grantee of the licence.
In the light of these conclusions in Mr Dorling entitled to any relief against Colonel Honnor in the Chancery action?
Colonel Honnor was, I think, entitled to accept and execute the orders which he accepted and executed between 29 June and 5
September 1960. Since then he has not dealt in Scorpions personally at all. His action in purporting to assign his licence to the
company may well have been a breach of one of its implied terms, but no relief is asked against him on that footing, naturally
enough, since Mr Dorling denied that he ever gave him a licence. In any case, s 8 of the Registration of Business Names Act,
1916, would have been an obstacle in the way of Mr Dorling claiming any such relief since it would have been founded on a
breach of contract. It follows, therefore, in my judgment, that, so far as Colonel Honnor himself is concerned, the Chancery
action must fail.
I turn finally to consider whether the company (which I have held not to have been protected by any licence from Mr
Dorling) infringed the copyright in the plans. It is common ground (a) that the plans were an artistic work within the meaning
of that phrase in the Copyright Act, 1956 (see s 3(1)(a) and s 48(1)); (b) that, by virtue of s 1(2), s 3(5)(a) and s 49(1) of the Act,
Mr Dorling as the owner of the copyright in the plans is prima facie entitled to restrain anyone from reproducing the plans or a
substantial part of them in a material form without his consent, and (c) that reproduction includes the conversion of the plans or a
substantial part of them into a three-dimensional form (see s 48(1)). It is argued, however, that although the company has
converted the plans or a substantial part of them into a three-dimensional form without Mr Dorlings consent, it has nevertheless
not infringed his copyright, because s 9(8) and, further, or in the alternative, s 10(3)(a) apply to the case. Section 9(8) is in the
following terms:

The making of an object of any description which is in three dimensions shall not be taken to infringe the copyright in
an artistic work in two dimensions, if the object would not appear, to persons who are not experts in relation to objects of
that description, to be a reproduction of the artistic work.

I know nothing whatever about boats or plans of boats. I have seen a completed Scorpion. I have not seen a kit of parts of
a Scorpion, but I have seen a brochure issued by the company called Instructions for building from the kit of parts, which
contains photographs of the chief components of the kit, both separate and in various stages of assembly. I am, therefore, I think,
in nearly as good a position as if I had seen a kit of parts in the flesh. The plans 503 of the boat itself, as opposed to those of the
spars, mast, rigging, and so on, consist (a) of a sheet headed General arrangement which includes outline drawings of the
completed boat seen from above and also sideways, showing in places what would only be visible if the outer skin of the boat
was removed, and (b) of two sheets largely filled with outline drawings of the component parts of the boat. My task, as I see it, is
to say whether the boat and the parts, or, if not the boat, then all or some of the parts, appear to me to be a conversion of the plans
or the relevant part of them into a three-dimensional form. It is a very difficult task because I know that in fact the boat which I
have seen and the parts of which I have seen photographs were based on these plans. In these circumstances, one is inevitably
tempted to observe resemblances which one might well not have observed at all if one did not know that the boat was based on
the plans. Making allowances for this as best I can, the conclusion to which I come is that some of the partssufficient together
to constitute a substantial part of the whole boatwould have appeared to a non-expert, who did not know that they were in fact
based on the plans, to be reproductions of the corresponding drawings on the plans, but that he would not have felt any strong
conviction that the completed boat was a three-dimensional version of the plans. In the completed boat many of the parts are
invisible, and I do not think that a non-expert looking at it without knowing that it was in fact based on the plans would be able to
say with any confidence either that it was the boat delineated in the drawings on the sheet headed General arrangement or that
it was made up of the parts appearing on the next two sheets.
What follows from this conclusion? It was submitted on behalf of the company that in the circumstances of this case the
object for the purpose of s 9(8) was the boat, and that, if the boat would not appear to non-experts to be a three-dimensional
reproduction of the plans, copyright in the plans would not be infringed. I cannot accept this submission. Each of the parts
making up the kit of parts is itself an object and under s 49(1) one has to read in after the words a reproduction of the artistic
work in s 9(8) the words or a substantial part thereof. Therefore, in my judgment, s 9(8) does not save the making of the parts,
which a non-expert would see to be reproductions of a substantial part of the plans, from being an infringement of the copyright
in the plans.
I must next set out the relevant parts of s 10 of the Act and also some parts of the Registered Designs Act, 1949, and of the
rules made under it, without which s 10 is not intelligible. Section 10 is as follows:

(1) Where copyright subsists in an artistic work, and a corresponding design is registered under the Registered
Designs Act, 1949 it shall not be an infringement of the copyright in the work (a) to do anything, during the subsistence
of the copyright in the registered design under the Act of 1949, which is within the scope of the copyright in the design
(2) Where copyright subsists in an artistic work, and(a) a corresponding design is applied industrially by or with the
licence of the owner of the copyright in the work, and (b) articles to which the design has been so applied are sold, let for
hire, or offered for sale or hire, and (c) at the time when those articles are sold, let for hire, or offered for sale or hire, they
are not articles in respect of which the design has been registered under the Act of 1949, the following provisions of this
section shall apply. (3) Subject to the next following subsection, (a) during the relevant period of fifteen years, it shall not
be an infringement of the copyright in the work to do anything which, at the time when it is done, would have been within
the scope of the copyright in the design if the design had, immediately before that time, been registered in respect of all
relevant articles In this subsection the relevant period of fifteen years means the period of fifteen years beginning with
the date on which articles, such as are mentioned in para. (b) of the last preceding 504 subsection, were first sold, let for
hire, or offered for sale or hire in the circumstances mentioned in para. (c) of that subsection; and all relevant articles, in
relation to any time within that period, means all articles falling within the said para. (b) which had before that time been
sold, let for hire, or offered for sale or hire in those circumstances. (4) For the purposes of sub-ss. (2) and (3) of this
section, no account shall be taken of any articles in respect of which, at the time when they were sold, let for hire, or
offered for sale or hire, the design in question was excluded from registration under the Act of 1949 by rules made under
sub-s. (4) of s. 1 of that Act (5) The power of the Board of Trade to make rules under section thirty-six of the Act of
1949 shall include power to make rules for the purposes of this section for determining the circumstances in which a design
is to be taken to be applied industrially. (6) In this section, references to the scope of the copyright in a registered design
are references to the aggregate of the things, which, by virtue of s. 7 of the Act of 1949, the registered proprietor of the
design has the exclusive right to do (7) In this section corresponding design, in relation to an artistic work, means a
design which, when applied to an article, results in a reproduction of that work.

By the Copyright (Industrial Designs) Rules, 1957, made under s 36 of the Act of 1949 and s 10(5) of the Act of 1956, it is
provided that a design shall be taken to be applied industrially for the purposes of s 10 if it is applied (inter alia) to more than fifty
articles all of which do not together constitute a single set of articles as defined by s 44(1) of the Registered Designs Act, 1949.
The parts of the Registered Designs Act, 1949, which are relevant to this case are:

Section 1(3): In this Act the expression design means features of shape, configuration, pattern or ornament applied
to an article by any industrial process or mans, being features which in the finished article appeal to and are judged solely
by the eye, but does not include a method or principle of construction or features of shape or configuration which are
dictated solely by the function which the article to be made in that shape or configuration has to perform.
Section 7(1): The registration of a design under this Act shall give to th registered proprietor the copyright in the
registered design, that is to say, the exclusive right in the United Kingdom and the Isle of Man to make or import for sale or
for use for the purposes of any trade or business, or to sell, hire or offer for sale or hire, any article in respect of which the
design is registered, being an article to which the registered design or a design not substantially different from the
registered design has been applied, and to make anything for enabling any such article to be made as aforesaid, whether in
the United Kingdom or the Isle of Man or elsewhere.
Section 44(1): In this Act article means any article of manufacture and includes any part of an article if that
part is made and sold separately.

The Designs Rules, 1949, r 26(3) provides that there shall be excluded from registration under the Act of 1949 designs to be
applied to printed matter primarily of a literary or artistic character including, among other things, plans.
The general effect of s 10 is clear enough. If the owner of the copyright in an artistic work exploits it industrially, he does
not lose his copyright under the Act of 1956, but, whether or not he actually registers the design which corresponds to the artistic
work under the Registered Designs Act, 1949, he ceases to be entitled to protection under the Act of 1956 in respect of matters
which areor, if he had registered the design, would have beeninfringements of the design copyright. When the period of
design copyright expires, his copyright protection under the Act of 1956 goes on, but only in respect of non-industrial use. In this
case Mr Dorling registered no design under the Registered Designs 505 Act, 1949, at any material time, and it is conceded that
more than fifty Scorpions had been made with his licence by September, 1960. It is common ground between the parties that
he could have registered the shape or configuration of the completed boat as a design under the Act of 1949. It appears, indeed,
from Appendix C to the Report of the Departmental Committee on Industrial Designs published in 1962 that designs are not
uncommonly registered in respect of boats. It is also common ground that he could not have registered a design in respect of the
plans, since such a registration was excluded by r 26(3) of the Designs Rules, 1949, or in respect of the shapes of the component
parts of the boat because their shapes were dictated solely by the function which they have to perform as parts fitting together to
make the whole boat and also because they are not sold separately. It follows, I think, that the parts are not articles within the
meaning of s 10(2)(b) and (c) of the Copyright Act, 1956. The articles there referred to must, as it seems to me, be articles in
respect of which a design could have been registered under the Act of 1949, and in this case a design could have been registered
only in respect of the completed boat.
The company, however, argues, and I think rightly, that the conditions of s 10(2) are nevertheless satisfied. Copyright
subsists in the plans which are an artistic work. The design which consists in Mr Dorlings conception of the shape of the boat is
a design corresponding to the plans because the result of applying that conception to the material out of which the boat is made
results in the production of an article which is a reproduction of the plans. It is true that it is not a reproduction which itself
infringes the copyright in the plans because, in my judgment, a non-expert would not recognise the boat (as opposed to the parts)
as a reproduction of the plans. I see no sufficient warrant for inserting the word infringing before the word reproduction in s
10(7). Indeed, it was not argued that I should do so. The other conditions of sub-s (2), namely, industrial application, non-
registration of the design and sale of the articlesthat is to say, the boatsare admittedly fulfilled. If sub-s (2) applies, then
under sub-s (3) Mr Dorling cannot complain of anything done or threatened to be done by the company which would have been
an infringement of the design copyright if the design had been registered. What the company did and, but for the undertaking,
would have continued to do, was to sell and offer for sale Scorpions and kits of parts of Scorpions accompanied by elaborate
building instructions including photographs of some of the parts separately and in the course of being assembled with an
intimation that the plans could be obtained by a purchaser from the class association.
Counsel for the company submitted that a kit of parts was something for enabling Scorpions to be made within the
meaning of s 7 of the Registered Designs Act, 1949. Counsel for Mr Dorling submitted that those words referred to objects such
as moulds or tools specially adapted for use in the production of the articles to which the design in question related. I have no
doubt that that was the sort of thing which the legislature had in mind in using those words. It does not, however, follow that the
words do not also include parts of the article in question. But, quite apart from those words, it seems to me that a prohibition
against making and selling an article of a particular character must at least extend to the making and selling of the component
parts of the article for the purpose of their being put together by the purchaser. It would be very odd that if Mr Dorling registered
his design in relation to the boat, he would secure design copyright with regard to it and at the same time remain entitled to
artistic copyright in respect of the industrial use of the plans by way of manufacture and sale of kits of parts. As to the
photographs of parts of the kit, it is perhaps open to doubt whether a two-dimensional reproduction of a three-dimensional
reproduction of an artistic work is an infringement of the copyright in that work. Assuming, however, that it is, the photographs
were part and parcel of the offer of the kit for sale and would, therefore, in my judgment, have 506 been covered by the design
copyright if Mr Dorling had registered the design in respect of the boat.
In the result, therefore, I have come with some regret to the conclusion that the Chancery action fails against the company as
well as against Colonel Honnor. I say with some regret for the following reason. I have held that Colonel Honnor had a
licence from Mr Dorling; that the attitude which Mr Dorling adopted in his letter of 29 June 1960, was unwarranted and that
Colonel Honnor was entitled to take the rest of the year to wind up his activities as a builder of Scorpions. But Colonel
Honnor, though he stopped building Scorpions himself, went on, or, but for the undertakings, would have gone on, making
Scorpions or kits of parts of Scorpions through the medium of the company in defiance of Mr Dorlings wishes and after a
reasonable period of notice for the revocation of his licence had expired. This conduct does not commend itself to me and I am,
therefore, sorry that I am obliged to give effect to the technical and quite unmeritorious defence raised by the company under s 10
of the Copyright Act, 1956. With the best will in the world, however, I cannot see an answer to it.
It follows, in my judgment, that the Chancery action fails.

Order accordingly.

Solicitors: Jaques & Co agents for Bond, Pearce, Eliott & Knape, Plymouth (for the plaintiff); Gregory, Rowcliffe & Co agents
for Watts, Anthony, Yeo & Segar, Plymouth (for both defendants).

Jacqueline Metcalfe Barrister.


[1963] 2 All ER 507

The Bramley Moore


Alexandra Towing Co Ltd v Owners of Dumb Barge Millet and Others
SHIPPING

PROBATE, DIVORCE AND ADMIRALTY DIVISION


CAIRNS J
3, 4 APRIL 1963

Shipping Limitation of liability Towage Collision between ship and dumb barge in tow by tug Tug and tow in different
ownership Loss of dumb barge Ship and tug found equally to blame Whether tug owners entitled to limit liability by sums
calculated by reference to tonnage of tug alone Merchant Shipping Act, 1894(57 & 58 Vict c 60), s 503(1) (d) (ii), as amended
by Merchant Shipping (Liability of Shipowners and Others) Act, 1958 (6 & 7 Eliz 2 c 62), s 2(1).

On 12 December 1959, the dumb barge Millet, which was being towed by the steam tug Bramley Moore (which had another
dumb barge made fast to her starboard side) came into collision with, and was sunk by, the motor vessel Egret in the River
Mersey. The barges did not belong to the owners of the Bramley Moore, but were being towed under a contract incorporating the
Liverpool Conditions of Towage, which provided that, on hiring of the tug for towage services, the master and crew became the
servants of the hirer, and that the tug owners should be held harmless and indemnified by the hirer against all claims. In an action
in rem arising out of the collision, the Egret and the Bramley Moore were found equally to blame, and judgment was given for
the owners of the Millet on their claim against the Egret and judgment for the owners of the Egret on their counterclaim against
the owners of the Bramley Moore for half the damages sustained by the owners of the Egret, including damages paid by them to
the owners of the Millet and others. The owners of the Millet were absolved from blame.
The owners of the Bramley Moore brought this action against the owners of the Millet and of the Egret claiming, pursuant to
s 503(1) of the Merchant Shipping Act, 1894, as amended a, limitation of their liability to sums calculated by reference to the
tonnage of their tug alone. The owners of the Millet 507 delivered a formal defence, but the owners of the Egret, while admitting
that the owners of the Bramley Moore were entitled to limit liability, contended that the limit should be calculated by reference to
the combined tonnages of the Bramley Moore and the two barges, or, alternatively, the combined tonnages of the Bramley Moore
and the barge Millet.
________________________________________
a Section 503(1), as amended, and so far as material, is set out at p 509, letter g, post

Held The limits of liability of the owners of the Bramley Moore should be governed by the tonnage of the Bramley Moore
alone (see p 512, letter d, post).
The Ran, The Graygarth ([1922] P 80), and The Freden (1950), 83 Ll L Rep 427) considered.
Semble: if the owners of the Egret had brought an action in personam against the owners of the Bramley Moore alleging
negligent navigation of the Bramley Moore and of the Millet and had succeeded, the owners of the Bramley Moore might have
had to limit their liability separately in respect of the Bramley Moore and in respect of the Millet (see p 512, letter d, post).

Notes
As to limitation of tonnage in case of tug and tow, see 35 Halsburys Laws (3rd Edn) 778, para 1196; and for cases on the subject,
see 41 Digest 916, 8071; 917, 918, 80868091.
For the Merchant Shipping Act, 1894, s 503, see 23 Halsburys Statutes (2nd Edn) 656.
For the Merchant Shipping Liability of Shipowners and Others Act, 1958, s 2, see 38 Halsburys Statutes (2nd Edn) 1093.
Cases referred to in judgment
Alde, The [1926] P 211, 95 LJP 163, 136 LT 187, 17 Asp MLC 157, 41 Digest 918, 8090.
Ant, The (1924), 19 Ll L Rep 211.
Freden, The (1950), 83 Ll L Rep 427.
Harlow, The [1922] P 175, 91 LJP 119, 126 LT 763, 15 Asp MLC 498, 41 Digest 916, 8071.
Ran, The, Graygarth, The [1922] P 80, 91 LJP 113, 126 LT 675, 15 Asp MLC 517, 41 Digest 792, 6527.

Action
This was a limitation action brought by the plaintiffs, Alexandra Towing Co Ltd the owners of the tug Bramley Moore, against
the first defendants, the owners of the dumb barge Millet, and the second defendants, the owners of the motor vessel Egret. The
facts are set out in the judgment.
The cases noted belowb were cited during the argument in addition to those referred to in the judgment.
________________________________________
b Union Steamship Co v Aracan (Owners), The American and The Syria [1874] LR 6 PC 127, 2 Asp MLC 350, The Quickstep (1890), 15 PD
196, 6 Asp MLC 603, The Englishman and The Australia [1894] P 239, 7 Asp MLC 603

Barry Sheen for the plaintiffs.


H W M Sellers for the first defendants.
R J H Collinson for the second defendants.

4 April 1963. The following judgment was delivered.

CAIRNS J. On 12 December 1959, the steam tug Bramley Moore was proceeding southbound in the River Mersey with the
dumb barge Buckwheat made fast to her starboard side and the dumb barge Millet in tow astern, when the motor vessel Egret,
proceeding northbound, came into collision with the Millet, which sank, with the result that one of the crew of the Millet lost his
life and two others were injured. The barges did not belong to the owners of the Bramley Moore, but were being towed under a
contract incorporating the usual Liverpool Conditions of Towage. These provide, inter alia:

On hiring of the tug for towage services, the master and crew thereof become the servants of and identified with the
vessel or vessels towed and their owners, and are under the control of the persons in charge of such 508 vessel or vessels
during the performance of the contract, the tug owners only undertaking to provide motive power ,

and then later: the tug owners shall be held harmless and indemnified by the hirer against, inter alia, all claims. The
owners of the Millet brought an action in rem against the owners of the Egret. The owners of the Egret delivered a defence and
counterclaim, in which they joined the owners of the Bramley Moore as defendants to the counterclaim. The action was heard by
Hewson J who, on 4 October 1961, made a decree in which he pronounced the Egret and the Bramley Moore equally to blame
and gave judgment for the owners of the Millet on their claim against the Egret and judgment for the owners of the Egret on their
counterclaim against the owners of the Bramley Moore for half the damages sustained by the owners of the Egret, including any
damages paid by them to the owners of the Millet or others. The owners of the Millet were absolved from blame c.
________________________________________
c See The Egret [1961] 2 Lloyds Rep 224

The present action is a limitation action brought by Alexandra Towing Co Ltd the owners of the Bramley Moore, the
plaintiffs, against the owners of the Millet, the first defendants, and the owners of the Egret, the second defendants, and all others
interested. The plaintiffs, by their statement of claim, claim to limit their liability to sums calculated by reference to the tonnage
of their tug alone. The first defendants delivered a formal defence; the second defendants, while admitting in their defence that
the plaintiffs are entitled to limit liability, raised the contention that the limit should be calculated by reference to the combined
tonnages of the tug and the two barges, or, alternatively, the combined tonnages of the tug and the barge Millet.
An issue was directed to be tried under RSC, Ord 25, r 2, on the point of law thus arising. On the issue coming on for
hearing before me, it was agreed by all parties that, once this issue was decided, no further dispute would remain, and I was
invited to dispose of the whole action by making, under RSC, Ord 25, r 3, such order as may be just, which in this case will be a
decree declaring the proper limits of the plaintiffs liability with the usual consequential directions. I see no reason why I should
not comply with this request. The relevant statutory provisions are contained in s 503 of the Merchant Shipping Act, 1894, as
amended by subsequent Acts, and in particular by s 2(1) of the Merchant Shipping (Liability of Shipowners and Others) Act,
1958. The relevant words in s 503, as amended, are as follows:

(1) The owners of a ship shall not (d) where any loss or damage is caused to any property through the act
or omission of any person (whether on board the ship or not) in the navigation or management of the ship be liable to
damages beyond the following amounts (ii) an aggregate amount not exceeding [a certain sum] for each ton of their
ships tonnage.

I have confined my citation to the parts relating to loss or damage to property, because the question of limitation in relation to
claims for the loss of life and personal injuries has ceased to be of practical importance.
Counsel for the plaintiffs says that his clients are the owners of the tug Bramley Moore; loss or damage was caused to
property; it was caused through an act or omission of a person in the navigation of the tug, and, therefore, he says, the plaintiffs
are not liable beyond a sum calculated on their ships tonnage, that is, the tonnage of the tug. It is true, says counsel, that s 3 of
the Act of 1958 extends the right of limiting liability to persons whose relation to a ship is other than that of owners, for example,
charterers, managers, master and crew, but, he says, there is nothing in that section to extend the liability of shipowners to any
higher figure than that based on their own ships tonnage. There appears to be no decision of this court on the point raised by the
second defendants. Circumstances must quite frequently have arisen in which this contention might have 509 been advanced, if
it is a valid one, and, indeed, some of the authorities to which I was referred are cases where the facts were very similar to those
of this present case, and yet the higher limit of liability now contended for by the second defendants was never put forward.
However, as counsel for the second defendants says, it may have been overlooked, or there may have been some reason, not
apparent from the reports, why it was not advanced. What is clear from the authorities is that, where tug and tow are in the same
ownership, it is sometimes necessary for their tonnages to be aggregated for the purpose of fixing the limit of liability. Counsel
for the plaintiffs argues that the very fact that the factor of common ownership is emphasised in these cases indicates that
aggregation is not required if the ownership is separate. Counsel for the second defendants, on the other hand, argues that
rationally the same limits should apply whether the ownership is single or divided.
The first case that should be mentioned is The Ran, The Graygarth, a decision of the Court of Appeal. There, a vessel was
damaged through collision with a barge in tow of a tug, by reason solely of the negligence of those in charge of the tug. The tug
and the barge were in the same ownership. The owners of the damaged vessel sued the owners of the barge in rem. Hill J found
the defendants liable as owners of the tug, and in subsequent limitation proceedings held that they were entitled to limit liability
according to the tonnage of the tug. The Court of Appeal held that the judgment should have been a judgment against the owners
of the barge by reason of improper navigation of the barge by the defendants servants on the tug and, accordingly, that the limit
of liability should be based on the tonnage of the barge. It may be noted that, although the tug and tow were in common
ownership, the Court of Appeal did not suggest that the tonnages should be aggregated, and that it was on the basis that the tow
was negligently navigated, and that the action was an action in rem against the tow, that the tonnage of the tow was held to be the
right basis for fixing the limit of liability.
In The Harlow, where tug and tow were again in the same ownership, Sir Henry Duke P, held that the combined tonnages
should be taken. There, however, it was found that there was negligence both on the part of the crew of the tug and also on the
part of the lightermen of the barges in tow. The President distinguished The Ran, The Graygarth on the grounds that in that case
the defendants were not sued as owners of the tug but only as owners of the tow.
In The Ant, the plaintiffs in a limitation action were the owners of a tug by whose negligence the barge that she was towing
came into collision with another vessel, and both colliding vessels were damaged. A number of issues were debated, but,
although counsel very experienced in Admiralty matters appeared for the defendants, it was never suggested that the limit of
liability should be calculated otherwise than on the tonnage of the tug alone.
In The Alde, damage was caused to a steamship by the negligent navigation of a barge (not in tow). In a limitation action by
the owners of the barge, it was contended by the owners of the steamship that the collision was contributed to by the negligence
of those on board a grain sucker belonging to the same owners as the barge, and that the tonnage of the grain sucker should be
added to that of the barge in fixing the limit of liability. Bateson J refused to accede to this argument on the ground that any
negligence of the crew of the grain sucker was not negligence in navigation. I do not think that this decision is of any assistance
in the present case. It may be noted that Bateson J said ([1926] P at p 215; 17 Asp MLC at p 159):

Mr. Dunlops last contention was that even if the plaintiffs prove that the damage was caused solely by the navigation
of the Alde, nevertheless, 510on the principle laid down in The Ran, The Graygarth, and The Harlow, the limit of the
plaintiffs liability must be the tonnage of both the Alde and the Turbo. I think that those cases are consistent and show that
damage caused by a barge in tow due to the improper navigation of the tug and of the tow comes within the section [s.
503], and that both vessels have to provide the amount of their joint tonnage,

but he clearly had in mind only the case of a tug and tow in the same ownership.
In The Freden, a tug and three barges which it had in tow all belonged to the plaintiffs in the limitation action. The
defendants contended that the tonnages of the tug and all the barges should be aggregated. The plaintiffs admitted that the
collision with the defendants vessel was caused by the negligent navigation of the tug and of one of the barges, namely, the barge
that had actually come into contact with the defendants vessel. Pilcher J held that the presence of the other two barges had not
contributed to the collision, and, on that basis, held that only the tonnages of the tug and the colliding barge should be taken into
account. He said this ((1950), 83 Ll L Rep at p 430):

Consequently, on that finding of fact, and applying the principle laid down in The Harlow, I have come to the
conclusion that the plaintiffs are entitled to limit their liability to the statutory limit of the George Livesey and of the
Bittern. Both of these vessels being in the same ownership, each of them falls within the wording of the appropriate
section, s. 503 of the Merchant Shipping Act, 1894. My judgment, therefore, in this case is that the plaintiffs are entitled to
the decree of limitation which they seek, based upon the tonnage of the George Livesey and the Bittern

The George Livesey was the tug and the Bittern was the colliding barge. Counsel for the plaintiffs laid stress on the words, both
of these vessels being in the same ownership, as indicating that, if the tug and barge had been separately owned, Pilcher J would
not have added their tonnages together.
Certainly these cases do not suggest that aggregation of tonnages is the proper course in any case where tug and tow are
separately owned.
Counsel for the second defendants was unable to cite any authority which supports the view for which he contends. His
argument can, I think, be reduced to the following propositions: (i) that it would be rational to treat the flotilla as a unit, seeing
that it was being navigated under the control of the tug which alone had the motive power; (ii) that, according to the terms of the
towage between the tug owners and the barge owners, the master and crew of the tug were the servants of the barge owners, who
gave an indemnity against all claims to the tug owners; (iii) that, so far as the barge Millet is concerned, it was the actual
instrument of the damage and was being navigated by the master and crew of the Bramley Moore; (iv) that it would be an unjust
and arbitrary rule which would make the amount recoverable by the Egret from the Bramley Moore dependent on the chance of
whether the barges belonged to the owners of the Bramley Moore or not especially where, in the latter case, ultimate liability will
fall on the barge owners under an indemnity which they have given to the tug owners; (v) that the judgment of Hewson J ([1961]
2 Lloyds Rep at p 227), shows that the presence of the barge, encumbering manoeuvre, brought about the collision; and (vi) that
the provisions of s 3 of the Act of 1958, while not directly relevant, indicate a policy in the legislature to move away from
ownership as the essential connexion between the party claiming to limit liability and the ship or ships whose tonnage falls to be
taken into account.
I think that the answers to these propositions are as follows: (i) To treat the flotialla as a unit would be contrary to what was
decided in The Ran, The Graygarth and The Freden. (ii) The towage contract governs only the 511 relations between the tug
owners and the barge owners, and in relation to the Egret is res inter alios acta. (iii) The owners of the Bramley Moore were not
sued as being responsible for the navigation of the Millet; the counterclaim was by way of an action in rem in respect of the
navigation of the tug. (iv) The limitation of liability does sometimes lead to arbitrary results, but the court cannot adopt a
construction which would strain the language of the Act in order to achieve what might appear to be a greater degree of fairness.
(v) The mere presence of the barges can have no more effect on the limitation assessment than would the presence in the river of
another vessel, separately owned, making navigation more difficult. (vi) Section 3 of the Act of 1958 is directed to enlarging the
class of persons who can limit liability, and cannot have been intended indirectly to amend the antecedent law by raising the
limits of liability of shipowners.
Counsel for the first defendants supported the contentions of the plaintiffs, because it is, of course, in the interests of the
owners of the Millet that the sum in respect of which they are liable to indemnity the owners of the Bramley Moore should be
kept down.
For the reasons which I have given, I am of the opinion that the limits of liability of the plaintiffs should be governed by the
tonnage of the Bramley Moore alone.
It is possible that, if the owners of the Egret had brought an action in personam against the owners of the Bramley Moore,
alleging negligent navigation of that vessel and of the Millet, and had succeeded on both grounds, the owners of the Bramley
Moore would not have been able to limit their liability to figures based on the tonnage of the Bramley Moore alone. I apprehend
that the result might have been that they would have had to limit separately in respect of the Bramley Moore and, under s 3 of the
Act of 1958, in respect of the Millet. If that is so, then it would appear that any hardship suffered by the owners of the Egret from
the result of the present action might have been avoided if they had proceeded in a different way in the first instance. However,
there may be good reasons why they did not do so, and it is not for me to form any concluded opinion on what the rights of the
parties would have been if they had done so.
I decide the issue before me in favour of the plaintiffs and, in accordance with the request of the parties, I make a declaration
limiting the plaintiffs liability accordingly. As I understand that the claims for loss of life and personal injuries have been settled
at figures less than the difference between their limit of liability under s 503(1)(i) and that, under s 503(1)(ii), it is unnecessary
for the decree to deal with the higher figure. The usual ancillary directions will be incorporated in the decree.

Declaration accordingly.

Solicitors: Hill, Dickinson & Co (for the plaintiffs); Laces & Co (for the first defendants); Weightman, Pedder & Co (for the
second defendants).

N P Metcalfe Esq Barrister.


512
[1963] 2 All ER 513

Chapman v Honig
ADMINISTRATION OF JUSTICE; Contempt of Court: ADMINISTRATION OF JUSTICE; Courts

COURT OF APPEAL
LORD DENNING MR, PEARSON AND DAVIES LJJ
7 FEBRUARY, 1 MAY 1963

Contempt of Court Proceedings no longer pending Contempt by subsequent victimisation of witness Witness given notice to
quit by his landlord against whom witness had given evidence Action by tenant for damages Whether civil action for damages
lay.

House of Lords Appeal to Leave to appeal Applicant a legally assisted person in Court of Appeal Consideration of
position of respondent in House of Lords as to costs.

In an action against a landlord his tenant was subpoenaed by the landlords opponent in litigation and gave evidence against the
landlord. The next day the landlord gave the tenant notice to quit. In an action by the tenant against the landlord for damages the
county court judge found that the landlords motive in giving the tenant notice was desire to punish the tenant for having given
evidence against him. The county court judge awarded damages to the tenant on the basis that an action for damages for
contempt of court was maintainable in lawa. On appeal,
________________________________________
a This was not the cause of action pleaded, but the substance of the decision, see p 519, letters f to i, and p 523, letter i, post

Held (Lord Denning MR dissenting). An action for damages did not lie for contempt of court in the present case, because (a)
the object of the courts jurisdiction to punish for contempt of court was the protection of the administration of justice, not the
protection of the individual affected (see p 521, letter e, p 524, letter g, and p 526, letter d, post), and (b) the same act (viz, in the
present case, the giving of notice to quit) could not be both lawful and unlawful, viz, valid in the sphere of contract and wrongful
in the sphere of tort (see p 523, letter f, p 524, letter i, and p 526, letter d, post).
Dicta of Lord Denning MR in A-G v Butterworth ([1962] 3 All ER at pp 329, 330) considered.
Per Lord Denning MR and Davies LJ: an act, in the present case victimisation of a witness, may constitute contempt of
court irrespective of whether other people get to know of it; it is not essential that the act should be done publicly (see p 517,
letters a to d, and p 526, letter d, post).
Per Pearson LJ, Davies LJ concurring: it may well be correct as a general proposition that there can never be a right of
action for damages for contempt of court (see p 523, letter e, and p 526, letter d, post).
Observations on the granting of leave to appeal to the House of Lords where the appellant is legally aided (see p 526, letter
g, post).
Appeal allowed.

Notes
As to contempt by interference with the course of justice, see 8 Halsburys Laws (3rd Edn) 79, para 11; and as to contempt in
relation to witnesses, see ibid, p 14, para 22; and for cases on the subject, see 16 Digest (Repl) 28, 29, 208234, 32, 266270, 39,
328336.
As to new causes of action in tort, see 37 Halsburys Laws (3rd Edn) 112, para 187.

Cases referred to in judgments


Allen v Flood [18959] All ER Rep 52, [1898] AC 1, 67 LJQB 119, 77 LT 717, 62 JP 595, 1 Digest (Repl) 33, 254.
American Bank and Trust Co v Federal Reserve Bank of Atlanta, Georgia (1921), 256 US 350.
Atkinson v Newcastle and Gateshead Waterworks Co (1877), 2 ExD 441, 46 LJEx 775, 36 LT 761, 38 Digest (Repl) 51, 263.
A-G v Butterworth [1962] 3 All ER 326, LR 3 RP 327, [1962] 3 WLR 819.
513
Beresford v Royal Insurance Co Ltd [1938] 2 All ER 602, [1938] AC 586, 107 LJKB 464, 158 LT 459, 29 Digest (Repl) 397,
2995.
Bradford Corpn v Pickles [18959] All ER Rep 984, [1895] AC 587, 64 LJCh 759, 73 LT 353, 60 JP 3, 1 Digest (Repl) 34, 261.
Christie v Davey [1893] 1 Ch 316, 62 LJCh 439, 36 Digest (Repl) 294, 403.
Crofter Hand Woven Harris Tweed Co Ltd v Veitch [1942] 1 All ER 142, [1942] AC 435, 111 LJPC 17, 166 LT 172, 2nd Digest
Supp.
Groves v Lord Wimborne [18959] All ER Rep 147, [1898] 2 QB 402, 67 LJQB 862, 79 LT 284, 24 Digest (Repl) 1045, 169.
Hargreaves v Bretherton [1958] 3 All ER 122, [1959] 1 QB 45, [1958] 3 WLR 463, 1 Digest (Repl) 28, 218.
Hemmings v Stoke Poges Golf Club Ltd [191819] All ER Rep 798, [1920] 1 KB 720, 89 LJKB 744, 122 LT 479, 31 Digest
(Repl) 45, 2036.
Hollywood Silver Fox Farm Ltd v Emmett [1936] 1 All ER 825, [1936] 2 KB 468, 105 LJKB 829, 155 LT 288, 36 Digest (Repl)
302, 479.
Keeble v Hickeringill (1706), 3 Salk 9, 11 Mod 74, 130, 91 ER 659, 2 Digest (Repl) 295, 38.
Monk v Warbey [1934] All ER Rep 373, [1935] 1 KB 75, 104 LJKB 153, 152 LT 194, 50 Ll L Rep 33, 29 Digest (Repl) 537,
3686.
Pasmore v Oswaldtwistle Urban District Council [18959] All ER Rep 191, [1898] AC 387, 67 LJQB 635, 78 LT 569, 62 JP 628,
38 Digest (Repl) 163, 41.
Phillips v Britannia Hygienic Laundry Co Ltd [1923] All ER Rep 127, [1923] 2 KB 832, 93 LJKB 5, 129 LT 777, 42 Digest 870,
197.
R v Odhams Press Ltd, Ex p A-G [1956] 3 All ER 494, [1957] 1 QB 73, [1956] 3 WLR 796, 16 Digest (Repl) 28, 215.
Ridge v Baldwin [1963] 2 All ER 66, [1963] 2 WLR 935.
Roberts v J and F Stone Lighting and Radio Ltd (1945), 172 LT 240, 1 Digest (Repl) 30, 234.
Solomons v R Gertzenstein Ltd [1954] 2 All ER 625, [1954] 2 QB 243, 38 Digest (repl) 255, 638.
Sorrell v Smith [1925] All ER Rep 1, [1925] AC 700, 94 LJCh 347, 133 LT 370, 42 Digest 985, 156.

Appeal
This was an appeal by the defendant landlord, Maurice Honig, from a decision of His Honour Judge Baxter made on 12
December 1962, at the Bow County Court, giving judgment for the plaintiff for 50 damages and costs. The facts appear in the
judgment of Lord Denning MR.
The case noted belowb was cited in argument in addition to those in the judgments.
________________________________________
b Candler v Crane, Christmas & Co [1951] 1 All ER 426; [1951] 2 KB 164

The appellant landlord appeared in person.


Gerald Butler for the respondent tenant.

Cur adv vult

1 May 1963. The following judgments were read.

LORD DENNING MR Mr Maurice Honig owns a house, No 218, Romford Road, Stratford, and lets it out in flats. His father,
Emanuel Honig, collects the rents and is closely associated with his son in the management of the properties. The son has sought
to disclaim responsibility for his fathers acts but on the judges findings it is plain that the two were acting in combination in all
that took place.
The story starts with the flat on the second floor which was occupied by Mr and Mrs Harrand and their three children. On 3
November 1960, the landlord gave Mr Harrand notice to quit to expire on 3 December 1960. But a few days later, on 5147
November 1960, long before the notice expired, whilst the Harrands were out for the day, Emanuel Honig, the father, entered the
flat, piled all their furniture into one of the rooms, installed someone else as tenant and padlocked the doors against the Harrands.
The Harrands managed to find accommodation for their three children with relatives, and they themselves had to borrow a car to
sleep in. They slept in it for eleven nights. Meanwhile their furniture was put into the garden, where it deteriorated and much of
it disappeared. Mr Harrand brought an action against the landlord, Mr Maurice Honig, for trespass. It was tried by His Honour
Judge Baxter on 22 June 1962. He held that although the acts were done by the father, Emanuel Honig, the father and son were
very closely in communication one with the other. He drew the inference that the son had complete knowledge of what was
going on and held the son responsible for the fathers conduct. He reproached them for causing these people and their three
children to be without a home on a cold November evening. It was, he said, a high-handed defiance of the plaintiffs legal
rights. And he awarded the sum of 200 as exemplary damages. The landlord appealed to this court and on 15 November 1962,
his appeal was dismissed. Ormerod LJ said it was a shocking performance on the part of the landlord or his father or both of
them.
The story now turns to the flat on the first floor which was occupied by Mr and Mrs Chapman and their family. Mr
Chapman had been tenant since 1959. He had seen something of what happened on the second floor: and Mr Harrand wanted
him to give evidence in the case which I have described. The tenant, fearing what might befall him if he gave evidence against
his landlord, did not go voluntarily to the court. He was subpoenaed to do so: and only gave evidence in obedience to the
subpoena. He gave evidence on 22 June 1962, at the hearing before Judge Baxter. On the very next day, 23 June 1962, the
landlord served on the tenant notice to quit his first floor flat on 28 July 1962. The reason why he did this was simply because
the tenant had given evidence for Mr Harrand. The object of the landlord was, the judge found, to punish or victimise the tenant
for having given evidence.
The tenant did not vacate the premises on July 28. He remained there. On 15 August whilst he was at work and his wife
and daughter at home, Emanuel Honig, the father, came and put padlocks on the doors of two of the rooms and a staple on the
third. The tenant was called back from work. The police were called too. The tenant said to the police: I was subpoenaed to
give evidence against the landlord. He lost the case and gave me notice to quit. The police asked Emanuel Honig: Why do
you want him out? Emanuel Honig replied: He gave evidence against me in court and you dont do that to landlords. In
making that statement, the father was, as the judge found, the mouthpiece of the son and fully authorised by him to make
explanation. The tenant said he did not intend to leave the premises. The padlocks were removed from the doors. But
thenceforward the tenant was careful to see that one or other of the family remained in the flat lest any attempt should be made to
lock the doors in their absence. He consulted solicitors and was granted an emergency legal aid certificate. His solicitors at once
applied for an injunction and a few days later Judge Sir Alun Pugh granted an injunction restraining the landlord from trespassing
on the flat or placing padlocks on the doors or cutting off the electricity. But nevertheless the tenant found that his wife, Mrs
Chapman, was so greatly disturbed by what had happened that for the sake of her health he felt obliged to leave the flat. He
moved his furniture and left on 27 August 1962. But he proceeded with his action to recover damages. It came for trial before
Judge Baxter on 19 November 1962.
In his pleadings the tenant asserted that the notice to quit was invalid because it was served on him in contempt of court.
And he claimed damages for trespass and breach of covenant for quiet enjoyment. The landlord sought to justify the notice to
quit. He said that he gave it because the rent was in arrear and he produced a so-called rent book in support. The judge rejected
this suggestion. 515He found that father and son were both lying. The tenant was not in arrear with his rent and the so-called
rent book had all the marks of being freshly written up for the purposes of the action. The judge summarised his conclusion in
these words:

When faced with a service of a notice to quit on the day after the previous hearing, and a persistent course of lying to a
process server and to me, with the utilisation of a false document, I have no hesitation in rejecting the landlords evidence
as to the motive for serving the notice to quit and in finding that he served the notice because the tenant had given evidence
for the plaintiff Harrand in his action against the landlord. His object was to punish or victimise the tenant for having given
evidence.
The judge held that the act was a contempt of court and he awarded the plaintiff 50 damages. The landlord appeals.
To my mind the whole case depends on this one point: When the landlord served this notice to quit, was his act lawful or
unlawful? If it was lawful, so that the notice to quit was good, then I think that the tenant must fail. He cannot complain of being
evicted if his tenancy had been validly terminated: see Hemmings v Stoke Poges Golf Club Ltd. But if it was unlawful so that the
notice to quit was bad, then the tenant must succeed: for in that event the conduct of the landlord, in putting padlocks on the
doors and attempting to evict the tenant, was undoubtedly a trespass and a breach of covenant for quiet enjoyment.
It would, I think, be fatal for the tenant to concede that the notice to quit was lawfully served on him. And I did not
understand him ever to concede it. For once such a concession was made, the plaintiff would be met by the proposition that an
act, lawful in itself, cannot be rendered unlawful by being done with a bad motive: see Bradford Corpn v Pickles, and Allen v
Flood. But before ever you apply that proposition, you must first decide whether the act was lawful in itself. And then you
will find that, often enough, its lawfulness depends on the motive with which it is done: see the essay of Professor Ames in
Selected Essays On The Law Of Torts, p 150. If a landowner shoots a gun at a target on his own land, you might say that it is an
act lawful in itself. But you would be wrong. If he does it for the express purpose of injuring his neighbouras by disturbing
his silver foxes in the breeding seasonand does in fact injure him, it is an actionable wrong: see Keeble v Hickeringill, per Holt
CJ; Hollywood Silver Fox Farm Ltd v Emmett. So also if a person exercises a contractual right (as if a master dismisses his
servant on sufficient notice or if a landlord gives his tenant a correct notice to quit), you might say that it is an act lawful in
itself. But again you would be wrong. It is true that when a person exercises a contractual right, it is nearly always lawful, no
matter that he may be actuated by spite or malevolence; see per Wills J in Allen v Flood ([1898] AC at p 46). But circumstances
may arise when it is unlawful. The Supreme Court of the United States had such a case before them in 1921, American Bank and
Trust Co v Federal Reserve Bank of Atlanta, Georgia ((1921), 256 US 350, see at p 358), where Holmes J delivered an instructive
judgment. We have, I believe, such a case before us today. For the landlord here was exercising his contractual right in contempt
of court, and that is to my mind unlawful, and being unlawful, he can acquire no rights under it.
Let me first show that it was unlawful. On the judges findings the landlord gave this notice to quit and attempted to evict
the tenant vindictively in order to punish the tenant for having given evidence against him. That is in itself a contempt of court
a criminal offenceand punishable accordingly (see A-G v 516 Butterworth), and, being done by father and son in a combination
to injure, it may also have been a conspiracy: see Crofter Hand Woven Harris Tweed Co Ltd v Veitch. It was in any case
unlawful. My brother Pearson has, however, some doubt about it. He thinks that the victimisation of a witness is not a contempt
of court in itself. It is only a contempt if other people are likely to get to know of it and be deterred from giving evidence in
other actions. If that is right, it would mean this, that if the tenant proclaims his grievance on the housetops, telling everyone
about it, the landlord is guilty of contempt. But if the tenant should keep his suffering to himself, without telling his neighbours
why he is evicted, the landlord does no wrong. That cannot be right. Let me suppose next that the landlord himself does the
proclaiming on the housetops. He calls the neighbours together and tells them (as he told Judge Baxter here) that he is evicting
the tenant because he is in arrear with his rentwhich is utterly falseand the neighbours believe it. By thus adding deceit to
victimisation, does it mean that he is not guilty of a contempt? I cannot believe it. In my judgment the victimisation of a witness
is a contempt of court and unlawful, irrespective of whether other people get to know of it or not. It is a gross affront to the
dignity and authority of the court and a grievous wrong to the individual affected.
Once it is held that the act of the landlord, in giving this notice to quit, was unlawful, it must follow that the notice itself was
invalid and will not be enforced in the courts. Just as no criminal can enforce rights resulting from his own crime (see Beresford
v Royal Insurance Co Ltd), so also no contemnor can enforce rights resulting from his own contempt. Just as the dismissal of a
police officer is invalid if he has not been heard in his defence (see Ridge v Baldwin) so also a notice to quit is invalid if it is
given in contempt of court. This seems so obvious as not to need demonstration. But as it is so vital a part of the case I must
proceed to prove it. And I would do so by taking a case of contempt by intimidation. Suppose a landlord threatens a tenant that,
if he gives evidence against him, he will give him notice to quit. When the tenant goes into the witness box, he asks the judge,
naturally enough, for protection: Am I bound to give evidence? My landlord says that, if I do, he will give me notice to quit.
What answer is the judge to make? I should have thought that the judge could assure the tenant with all the authority of the law
behind him: You need not fear what the landlord may do. The arm of the court is strong enough to protect you. The law can, I
think, grant an injunction to stop the landlord carrying out his threat: for the simple reason that the court can restrain an unlawful
act, whereas it cannot a lawful one (see Sorrell v Smith ([1925] All ER Rep 1 at pp 14, 23; [1925] AC 700 at pp 730, 747)). I do
not believe that the law is so weak that it cannot protect the tenant, nor that the judge has to tell him: I cannot stop the landlord
giving you notice to quit. No court can do so. And what is more, at his bidding, this court must itself evict you. It is all very
well to say that the tenant can apply to the Divisional Court and get the landlord rebuked for contempt, and even fined or
imprisoned. But what good does that do to the tenant if at the end of it allnay, even when he is on his way to the Divisional
Courtthe landlord can give him notice to quit and turn him out? The tenant might well give to the judge the retort courteous:
If the law cannot protect me, then I am sorry but I cannot give evidence before you because I cannot risk being turned into the
street. No one, I trust, would blame him for such refusal. No judge, I hope, would commit him for contempt. But if the law
can protect him, as I think it can, it would be proper for the judge to compel him to answer.
The principle on which this case falls to be decided is simply this. No system of law can justly compel a witness to give
evidence and then, on finding him victimised for doing it, refuse to give him redress. It is the duty of the court 517 to protect the
witness by every means at its command. Else the whole process of the law will be set at naught. If a landlord intimidates a
tenant by threatening him with notice to quit, the court must be able to protect the tenant by granting an injunction to restrain the
landlord from carrying out his threat. If the landlord victimises a tenant by actually giving him notice to quit, the court must be
able to protect the tenant by holding the notice to quit to be invalid. Nothing else will serve to vindicate the authority of the law.
Nothing else will enable a witness to give his evidence freely as he ought to do. Nothing else will empower the judge to say to
him: Do not fear. The arm of the law is strong enough to protect you.
It is said, however, that to hold the notice invalid is a pointless exercise, because the landlord can give another notice next
day or next week or next month and that notice will be valid. I do not agree. If the landlord has been guilty of such a gross
contempt as to victimise a tenant, I should have thought that any court would hold that a subsequent notice to quit was invalid
unless he could show that it was free from the taint. The landlord can at least be required to purge his contempt before being
allowed to enforce the contractual rights which he has so greatly abused. The tenant, of course, has to pay his rent and perform
his covenants: so there is no injustice in requiring the landlord to clear his conscience.
The case was put of the valet who gives evidence against his master in a divorce suit. Next day the master, out of spite,
dismisses him by a months notice. Clearly the notice is unlawful. But the servant cannot stay on against the masters will. The
law never enforces specifically a contract for personal service. But what are the damages? They would, I think, be such damages
as a jury might assess to recompense him for the loss of the chance of being kept on longer, if he had not been victimised. Thus
only can the law give adequate redress, as it should, to an innocent person who has been damnified for obeying its commands.
Such would be the measure in the case of compensation under the Witnesses (Public Inquiries) Protection Act, 1892: and it
should be the measure applied by the courts of law. Witnesses before the courts should be just as much entitled to redress as
witnesses before statutory inquiries: see A-G v Butterworth ([1962] 3 All ER 326, cf at p 330).
I find no help from the cases that show that no action lies against a person who perjures himself in the witness box. That is a
special rule laid down so that witnesses may not be deterred by fear of having actions against them. I find no help from the cases
where a statutory offence gives rise to a penalty but not a civil action. These turn on the true interpretation of the statute. I find
some help from the cases which hold that a witness who wilfully disobeys a subpoena may be liable to an action: for they show
that a contempt of court may on occasion give rise to a civil action.
The truth is, however, that this is a new case. None like it has ever come before the courts so far as I know. But that is no
reason for us to do nothing. We have the choice before us. Either to redress a grievous wrong, or to leave it unremedied. Either
to protect the victim of oppression, or to let him suffer under it. Either to uphold the authority of the law or to watch it being
flouted. Faced with this choice I have no doubt what the answer should be. We cannot stand idly by. The law which compels a
witness to give evidence is in duty bound to protect him from being punished for doing it. That was the view of Judge Sir Alun
Pugh when he granted an injunction. It was the view of Judge Baxter when he gave damages of 50. It is my view too. I would
not turn the tenant away without remedy. I would dismiss this appeal.

PEARSON LJ. The landlord owns a number of properties including 218, Romford Road, Stratford, which is a house let out in
tenement flats. The plaintiff was one of his tenants. He gave evidence under subpoena for another 518 tenant, named Harrand, in
an action by Harrand in the county court against the landlord. Harrand obtained judgment against the landlord for 193 10s
damages for trespass and conversion. That was on 22 June 1962. The landlord gave to the plaintiff a notice to quit which was
dated 23 June and was served on 25 June and expired on 28 July 1962. So far as the terms of the tenancy were concerned, the
notice to quit was duly given and effective to terminate the tenancy. The tenant, however, with his family remained in
occupation. On 15 August the landlords father and another man were putting padlocks on the doors of the rooms in the flat for
the purpose of excluding the plaintiff and his family from the flat. The tenant and a policeman arrived and there was a
discussion, and the tenant and his family remained in occupation of the flat for the time being. Afterwards they vacated the flat.
The tenant brought this action against the landlord. In the particulars of claim he alleged that the notice to quit had been
served by the landlord on the tenant because the tenant had given evidence against the landlord in the Harrand action, that
therefore the service of the notice was in contempt of court and that it was invalid. On the basis of the invalidity of the notice to
quit and the continuance of his tenancy, he claimed damages for subsequent acts of interference with his occupation as
constituting trespass and breaches of the covenant for quiet enjoyment. He also claimed injunctions and a declaration. There
was, however, in the particulars of claim no claim for damages for the alleged contempt of court. At the trial there was an issue
of fact as to the motive which prompted the landlord to serve the notice to quit. The learned judge rejected the landlords
evidence that the tenant was in arrears with his rent and that that was the reason for the service of the notice to quit. The judge
found that the landlords motive was his desire to punish the tenant for giving evidence against the landlord in Harrands action.
For brevity I will refer to that as the vindictive motive.
Evidently there was, after the conclusion of the evidence, a discussion between the judge and counsel as to the legal
consequences of contempt of court in such a case as this and as to the effect of A-G v Butterworth. Although damages for
contempt of court had not been claimed in the particulars of claim, the tenants counsel was permitted to argue and claim that the
tenant was entitled to such damages. The landlords counsel made two concessions, viz: (1) that, if the service of the notice to
quit was prompted by the vindictive motive, it was contempt of court, and (2) that if there was contempt of court, the landlord
was liable in damages. Both concessions were made on the ground that they were required by the decision and reasoning of this
court in A-G v Butterworth. The question whether they were so required will need to be investigated with a view to clarifying the
law. The fact that the concessions were made creates difficulties in dealing with this appeal.
As to the second concession the judge said that counsel for the landlord conceded that if I found contempt his client would
have to pay damages, and that counsel

added as the defendant had to do in A.-G. v. Butterworth, a concession which, in view of the nature of the
proceedings in that case, was not as helpful as no doubt it was intended to be. LORD DENNING, M.R.s observation was
perhaps obiter, but in view of the very character of the judgment and the emphasis of his remarks, I am entitled to regard it
as virtually binding on me, and the phrase may well have redress should be construed as meaning, especially in view of
counsels concession, has a right of redress.

The judge decided that the claim for damages for contempt of court was admissible in law, and that general damage without proof
of direct financial loss was a sufficient foundation for such a claim and he gave judgment for the plaintiff 519 for 50 damages
for contempt of court. The judge then considered it was unnecessary to decide whether the notice to quit, given in contempt of
court, was invalid. He said: I do not think I need consider that complication, especially as the plaintiff has left the premises.
On 15 January 1963, the landlord gave a notice of appeal raising several points. At the hearing of the appeal the landlord
appeared in person; he explained what had happened in the court below and made it sufficiently clear that he wished to contend
that he had no liability in damages, but he did not carry his argument any further than that. The tenants counsel presented an
argument in support of the judgment for damages for contempt of court, but, as I understood, he did not in the end seek to argue
that the notice to quit was invalid. He relied on the two concessions made by counsel for the landlord in the court below. I think
that the first concessionnamely, that the service of the notice to quit, if it was prompted by vindictive motive, was in contempt
of courtstill stands, although the sufficiency of the evidence may be doubted, as will appear later. The second concession
namely, that there is a liability in damages for the contempt of courthas been treated as not binding on the landlord, because it
was made under a misunderstanding of the nature of the proceedings in A-G v Butterworth. The question whether there is such a
liability in damages is therefore an open question and it is to be decided in the appeal.
In A-G v Butterworth there was no civil action and no claim for damages. Mr Greenlees, who held certain offices in a
branch of a trade union, had given evidence in a case before the Restrictive Practices Court and members of the branch, who were
displeased at the evidence which he had given, called on him to answer for it at meetings of the branch and purported to deprive
him of his offices. The Attorney General applied by motion to the Restrictive Practices Court for writs of attachment or
committal against the members concerned. It was found that some of them had been actuated by the vindictive motive (i.e., to
punish Mr Greenlees for having given evidence). The Restrictive Practices Court, holding that, except in cases where the court
itself was scandalised, an act could not amount to contempt unless it was calculated to prejudice the trial of pending proceedings
or to interfere with persons returning home therefrom, dismissed the motions. On appeal it was held that contempt of court was
not confined to pending cases, and that victimisation of a witness, being an interference with the proper administration of justice
as a continuing process in deterring potential witnesses from giving evidence in future cases, was contempt of court whether it
was done while the proceedings were still pending or after they had finished. That is a sentence taken from the headnote and it
gives a convenient short summary of the decision of the Court of Appeal, but it needs to be supplemented by reference to certain
passages in the judgment of Donovan LJ which have an important bearing on the present question. He said ([1962] 3 All ER at p
332):

The question to be decided here, as in all cases of alleged contempt of court, is whether the action complained of is
calculated to interfere with the proper administration of justice.

He said ([1962] 3 All ER at p 333):

R. v. Odhams Press, Ltd., Ex p. A.-G. makes it clear that an intention to interfere with the proper administration of
justice is not an essential ingredient of the offence of contempt of court. It is enough if the action complained of is
inherently likely so to interfere. A newspaper article accusing a man of crime after proceedings have been begun and
before his trial plainly answers that description. But there may be other actions where the likely effect is not self-evident
and further inquiry will have to be 520 made. The present case is an instance. The respondents were within their legal
rights in seeking to relieve Greenlees from his honorary posts. But if the object of doing so was not merely to exercise that
right for the good of the branch but to punish him for the evidence which he gave before the Restrictive Practices Court,
and if the taking of such revenge was calculated to interfere with the administration of justice, then it will be no answer for
the respondents to say that, while intending to punish Greenlees, still they had no intention of interfering with the
administration of justice. Thus far the case is no different from R. v. Odhams Press, Ltd., Ex p. A.-G.. But where it differs
is that, in order to determine the likely effect of what the respondents did, one has to inquire into their motives The
issue of fact remains: was the action calculated to interfere with the administration of justice? I agree that in this kind of
case it must be proved by the Crown that knowledge of the revenge taken on one who has given evidence is likely to come
to the knowledge of potential witnesses in future cases. I think that the Crown has proved that here as a matter of
reasonable inference

Those passages show that for the purpose of deciding whether a contempt of court has been committed in a case of this kind, the
determining factor is not harm done to the individual but harm done to the future administration of justice. The distinction is not
likely to have much practical importance in relation to criminal proceedings of the usual character for contempt of court; in any
ordinary case the victimisation of a person for giving evidence would become known, and would tend to deter potential witnesses
from giving evidence in other actions and so would prejudice the future administration of justice. The distinction is, however,
important in relation to the question arising in this appeal whether there is any civil liability to pay damages to the individual
concerned. It is possible to imagine a case in which there would be victimisation of a witness and yet there would be no
contempt of court. The object of the courts jurisdiction to punish for contempt of court is the protection of justice and not the
protection of the individual affected. The adverse effect on the individual does not constitute a contempt of court unless there is
the additional element of probable interference with the course of justice. I should mention here, incidentally, that the evidence in
this case may have fallen short of proving that there was contempt of court, though the point is covered by the first concession
made by landlords counsel in the court below and that concession still stands.
The tenants counsel has relied on passages in the judgment of Lord Denning MR in A-G v Butterworth ([1962] 3 All ER at
p 329). He said:

I have no hesitation in declaring that the victimisation of a witness is a contempt of court, whether done whilst the
proceedings are still pending or after they have finished. Such a contempt can be punished by the court itself, before which
he has given evidence; and, so that those who think of doing such things may know where they stand, I would add that if
the witness has been damnified by it, he may well have redress in a civil court for damages.

After referring to the Witnesses (Public Inquiries) Protection Act, 1892, Lord Denning MR said ([1962] 3 All ER at p 330):

Why were courts of justice thus excluded? Not because the victimisation of witnesses before the court was any less
reprehensible, but because the courts have their own machinery at hand for dealing with victimisation, namely, their own
power to bring offenders before them for contempt of court, and, I would add, the remedy to a person aggrieved by action
for the wrong done.
521

Those passages were clearly obiter dicta because no question of civil liability arose for decision in that case. As obiter dicta,
when read together and fairly construed, they contain a warning of the risks incurred by a person who acts in contempt of court
and do not in my view say or imply that there is necessarily in every case of contempt of court of the kind which we are
considering a civil right of action for damages. In many cases there would be such a right. The act of victimisation may be itself
a tort or a breach of contract, eg, trespass, assault, libel, slander, wrongful dismissal or breach of covenant for quiet enjoyment.
In some cases the vindictive motive would of itself render unlawful an act which without such motive would have been lawful.
For instance, in a defamation action the vindictive motive would be malice defeating a plea of privilege; it would make a
prosecution malicious: it might cause some act by an occupier of land, which would otherwise have been a lawful use of the
land, to be an act of nuisance (Christie v Davey; Hollywood Silver Fox Farm Ltd v Emmett); and it might cause some concerted
activity, which might otherwise have an an innocent combination in defence or furtherance of trade interests, to be an actionable
conspiracy. On the facts of A-G v Butterworth there was a possibility that Mr Greenlees might have had a cause of action for
conspiracy, but it did not arise for decision and was not investigated. There might well be other cases in which the vindictive
motive would render unlawful some act which would otherwise be lawful.
There is a special difficulty in the present case. The act complained of, the service of the notice to quit, was on the face of it
a lawful exercise of a contractual right, duly implemented in accordance with the provisions of the tenancy agreement and
effective to terminate the tenants estate and to convert the landlords interest from a reversion to an estate in possession.
Common experience is that, when the validity of an act done in purported exercise of a right under a contract or other instrument
is disputed, the inquiry is limited to ascertaining whether the act has been done in accordance with the provisions of the contract
or other instrument. I cannot think of any case in which such an act might be invalidated by proof that it was prompted by some
vindictive or other wrong motive. Motive is disregarded as irrelevant. A person who has a right under a contract or other
instrument is entitled to exercise it and can effectively exercise it for a good reason or a bad reason or no reason at all. If the rule
were different, if the exercise of such a right were liable to be overthrown, in an action brought at any time within the limitation
period, by proof that the act was done with a wrong motive, there would be great unsettlement of property titles and commercial
transactions and relationships. I think that it was conceded by the tenants counsel in the course of his argument that the notice to
quit must be considered to have been valid for the purpose of terminating the tenants estate. I am not insisting on the
concession, because it may have been withdrawn in later discussion. In any case that is in my judgment the right conclusion on
the point.
What follows? The notice to quit was valid: it was effective for its purpose of requiring the tenant to quit the premises: it
terminated the tenants right of possession; the landlord was entitled under the tenancy agreement to give the notice and he gave
it in all respects in conformity with the provisions of the tenancy agreement. For those purposes, in those connexions, in those
aspects, the giving of the notice was a lawful act. How then could it be unlawful for some other purpose or in some other
connexion or in some other aspect as between the same parties? Of course it could be lawful as between these parties, the tenant
and the landlord, and yet unlawful as between the landlord and the court on the ground that it was calculated to interfere with the
administration of justice. But in my view the same act cannot be as between the same parties 522 both a lawful exercise of a
contractual right and at the same time unlawful as being tortious and giving rise to an action for damages. No such complication
has yet existed in the law, and it is not necessary or desirable to introduce it.
I have considered a number of cases in which the court had to decide, in relation to some particular enactment, whether an
individual, adversely affected by breach of a statutory duty, had a right of action for damages against the person who had
committed the breach. Such cases include Atkinson v Newcastle and Gateshead Waterworks Co, Groves v Lord Wimborne,
Pasmore v Oswaldtwistle Urban District Council, Phillips v Britannia Hygienic Laundry Co Ltd, Monk v Warbey, Solomons v R
Gertzenstein Ltd. The answer depends on the construction of the particular enactment, ie, on the intention which it manifests.
Here there is no enactment which is directly relevant and I can only consider, perhaps in a rather metaphorical way, what
intention is to be inferred from the nature and exercise of the jurisdiction. So far as I know, no individual ever has been awarded,
or has even claimed, damages or other compensation for contempt of court until the present case. The jurisdiction exists and is
exercised alio intuitu, for the protection of the administration of justice and not for the protection of individuals. So to speak, the
hypothetical enactment should be notionally construed as not conferring on an individual affected by a contempt of court, any
right of action for damages for the contempt of court as such, although of course he may have a right of action for damages on
other grounds.
It is not necessary, however, for the determination of the present case to decide as a general proposition that there can never
be a right of action for damages for contempt of court. The general proposition might well be correct, but in the present case it is
enough to say that there can be no such right of action in respect of an act which, as between the tenant and the landlord, has been
done in exercise of a right under a contract or other instrument and in accordance with its provisions. The main reason is that the
same act as between the same parties cannot reasonably be supposed to be both lawful and unlawfulin the sphere of contract,
valid and effective to achieve its object, and in the sphere of tort, wrongful and imposing a tortious liability. Two ancillary
reasons can be given. First, the supposed liability could easily be evaded. For instance, a landlord wishing to punish his tenant
for giving evidence against him could wait for a suitable interval and then give notice to quit and put forward some plausible
excuse for desiring a change of tenant. Secondly, whenever there was a continuing contractual right, the damages could not be
much more than nominal. For instance, if the landlord had not given the offending notice to quit when he did, the tenant would
have retained his tenancy subject to the landlords right to terminate it at any time by duly giving notice to quit. The tenant could
not reasonably be allowed to gain security of tenure by giving evidence against his landlord.
Accordingly, I am of opinion that the alleged right of action for damages was not established, and the appeal should be
allowed and there should be judgment for the landlord.

DAVIES LJ. The facts of this case, the nature of the proceedings before the learned county court judge and his decision thereon
have already been stated by my lords. It is clear that, although the action started as one of trespass, what the judge did was to
award to the tenant damages for contempt of court. It is also clear that, in so doing and in holding that the facts of this case gave
523 rise to such a cause of action, he based himself on the observations made obiter by Lord Denning MR in AG v Butterworth
([1962] 3 All ER 326 at pp 329, 330).
One cannot help but sympathise with the proposition that in general a person injured by a wrongful act should have a
remedy in damages against the wrong-doer. But it has to be considered whether, in the first place, that proposition is universally
true, and, secondly, whether in the circumstances of this case the landlords action in serving a notice to quit was as against the
tenant, wrongful at all. It is, no doubt, true that in most cases a person injured by a criminal offence has a right of action against
the criminal. That is because most crimes are torts. Acts of criminal violence to person or property would be trespasses; larceny
would be conversion; most frauds would give rise to an action of deceit; and so on. But not all crimes give rise to a cause of
action. For example, it is well established that perjury does not give rise to a cause of action at the suit of a person injured by the
perjury; see the decision of Lord Goddard CJ in Hargreaves v Bretherton, and the authorities there cited. It is true that there may
be special features relating to the offence of perjury which might make it difficult to permit of an action based on it. But this line
of authority shows that there is no general rule that all crimes give rise to a cause of action.
Equally relevant to this inquiry is the great body of case law dealing with the question whether the commission of an act
forbidden or made punishable by statute gives a cause of action to a person injured by the act. On this question it is notoriously
difficult to enunciate any guiding principle. The authorities are discussed in the dissenting obiter judgment of Somervell LJ in
Solomons v R Gertzenstein Ltd ([1954] 2 All ER at pp 629632; [1954] 2 QB at pp 253256). As examples may be cited the well
known case of Groves v Lord Wimborne, the alma genetrix of so much litigation under the provisions of the Factories Acts, on the
one side of the line, and, on the other, Phillips v Britannia Hygienic Laundry Co Ltd. Perhaps the nearest that one gets to a
statement of principle is in the words of Atkin LJ in the last cited case, in a passage adopted by Somervell LJ in Solomons case
([1954] 2 All ER at p 631; [1954] 2 QB at p 255). Atkin LJ said ([1923] All ER Rep at p 133; [1923] 2 KB at p 842):

Therefore, the question is whether these regulations, having regard to the circumstances in which they were made and
to which they relate, were intended to impose a duty, which is a public duty, or whether they were intended also to impose a
duty enforceable by an individual aggrieved.

It is, of course, implicit in this principle that not in every case is an individual who has been injured by a wrongful act entitled to
sue, even though the wrongful act is prohibited or made punishable by statute. And the principle can, in my judgment, be applied
in the present case by inquiring whether the concept of, and proceedings for, contempt of court are concerned with the
preservation of the inviolability of the administration and course of justice and its proper conduct or whether, in addition, they are
intended in all cases to give a remedy in damages to an individual injured by the contempt.
On the facts of the present case, I find it extremely difficult to see how it can be said that the tenant had any civil remedy
against the landlord, if it is conceded, as it was, at any rate at one time in the argument, conceded, that the notice to quit was valid
as between the parties; for all that followed on its expiry was, again as between the parties, lawful. Pearson LJ has pointed out
the odd consequences that would follow if it could be said that the giving of the notice to quit was at the same time lawful and
unlawful. And, presumably, if it was unlawful, its giving or implementation could be prevented by injunction, as 524 indeed, in a
sense, happened in the present case. But it is difficult to see how in a case such as this the exercise of a legal right could properly
be restrained by injunction. And quite apart from any question of injunction, one wonders, as Pearson LJ has indicated, for how
long the exercise of a legal right can, by reason of its amounting to a contempt, remain, as between the parties, one which gives
rise to a claim for damages. To take an example almost on all fours with the present facts: suppose a valet gives evidence against
his master in a divorce suit. On the next day the master, in order to punish his valet, terminates his service by proper notice, say,
one month. What damages has the valet suffered? And if the notice were bad, for what period of time will any subsequent notice
given by the master to the valet entitle the latter to claim damages?
There is one somewhat interesting form of action which may be thought not altogether dissimilar to that alleged to exist
here. A party to an action who has obtained the issue of and served a subpoena may bring an action for damages against a
witness who fails to attend on the subpoena, on proof of actual damage sustained owing to the witnesss non-attendance: see
Bullen and Leake, Precedents of Pleadings (11th Edn, 1959), p 658, and 15 Halsburys Laws of England (3rd Edn), p 435. It is
not, I think, necessary to quote the early authorities on this subject, since they are referred to in the judgment of Asquith J in
Roberts v J & F Stone Lighting and Radio, Ltd, which, so far as I know, is the latest reported case on this branch of the law. In
the course of his judgment in that case Asquith J said this ((1945), 172 LT at p 241):

It is important to note, first, that the cause of action is not maintainable without proof of special damage, that is, actual
damage; and, secondly, that in the decided cases the special damage recoverable has invariably been limited to costs thrown
away through the witnesss non-attendance. In almost all of them the plaintiff, on finding the witness absent, either elected
to be non-suited or withdrew the record.

By a parity of reasoning, if such a cause of action as that given effect to by the learned county court judge were to exist, it seems
to me that it would be essential for the plaintiff to prove special damage. I cannot see that there was any such special damage
here. Indeed, as I read the judgment of Judge Baxter, he so found. He said:

It is sufficient, I think, to say that the plaintiff has been the victim of a wrongful act, a criminal offence, and has been
injured. In the circumstances of a case such as this I do not think that it is necessary for a plaintiff to show special
pecuniary damage. Here the plaintiff found himself with the worry and anxiety of a notice to quit in an area where
accommodation is extremely difficult to find; he was faced with ejectment by the landlords agents, his wife became ill on
account of the inconvenience and anxiety and at last he felt obliged to leave the flat he had occupied for two years.

In any event, however, an action against an absent witness is, in my judgment, quite different from an action such as the present.
It is true that a subpoena is an order of the court, obtained on application of the party. But once it is issued and served, the
witness has no right either as against the court which issued the subpoena or against the party who obtained it or against anyone
else to absent himself. This state of affairs is entirely different from the present case where, as between himself and the plaintiff,
the defendant did no more than exercise his legal rights.
Reference was made in the course of the argument to the Witnesses (Public Inquiries) Protection Act, 1892, by which it was
enacted, putting it shortly, that it should be a misdemeanour, punishable by fine or imprisonment, for anyone to threaten, punish,
damnify or injure any person for having given evidence or on account of the evidence given by him at a public inquiry, as defined
in the Act. By s 4 of the Act the court before which any person is convicted of an offence under the Act may
525

award to complainant any sum of money which it may think reasonable by way of satisfaction or compensation for
any loss of situation, wages, status, or other damnification or injury suffered by the complainant through or by means of the
offence

This somewhat unusual enactment cannot in my judgment assist the tenants contention in the present case. It does not, of
course, apply to witnesses or to evidence given in courts of law; and I can see no ground for inferring from this Act, which
applies only to public inquiries, that there was or is in existence any power in any proceedings to compensate in the like
circumstances witnesses in courts of law.
One minor, and perhaps not important point, remains in connexion with the present case. Had proceedings for contempt
been taken against this landlord, the county court would, as the learned judge himself pointed out, have been unable to deal with
the matter. A contempt of this sort is outside its jurisdiction. It would seem a little odd then that the judge, though prevented
from deciding whether the defendant should be punished for contempt, should nevertheless be entitled to decide that a contempt
had been committed and to award damages therefor.
I would respectfully desire to express my agreement with the judgment of Pearson LJ save in one respect. The only matter
on which I venture to differ from him, agreeing as I do in this respect with Lord Denning MR is that in my judgment it is not
essential, in order to constitute a contempt, that the act should be done publicly or publicised in any way. A threat to, or the
punishment of, a witness as such would in my view amount to a contempt even though the knowledge of the threat or punishment
were confined solely to the witness and the contemnor.
On the whole of the matter, I am of the opinion that on the facts of this case the plaintiff had no cause of action. If the
defendants conduct rendered him a contemnor, that fact made him liable to be punished as such by a Divisional Court of the
Queens Bench Division, but did not render him liable in damages to the plaintiff. What the position might be in a case where the
act of contempt was without any legal justification whatsoever it is not necessary to decide.
In agreement with Pearson LJ I would allow the appeal and dismiss the tenants claim.

Appeal allowed.

Counsel for the tenant asked for leave to appeal to the House of Lords. After hearing argument Lord Denning MR said: We have
considered your application. This is a case in which we would certainly in the ordinary way, on account of its general
importance, have given leave to appeal to the House of Lords, but in view partly of the small amount of money involved, and, of
much more importance, in view of the present position of the law whereby an unassisted person may find himself saddled with a
great amount of costs which would not be paid by the legally aided person, we do not give leave to appeal. If the law should be
altered in future, the position will be different, but we do not think, save in exceptional circumstances, that an unassisted person
should be taken to the House of Lords and have to find a large sum of money out of his own pocket c
________________________________________
c See also, on this aspect of applications for leave to appeal to the House of Lords, Searle v Warbank, [1947] 1 All ER at p 13, letter H; [1947]
AC at p 346, per Viscount Maugham.

Leave to appeal to the House of Lords refused.

Solicitors: Lipson, Rumney & Co (for the tenant).

F Guttman Esq Barrister.


526
[1963] 2 All ER 527

Malden and Coombe Corporation v Bennett


LOCAL GOVERNMENT

QUEENS BENCH DIVISION


LORD PARKER CJ, HAVERS AND EDMUND DAVIES JJ
1 MAY 1963

Rates Rateable occupation Joint rateable occupation Husband separated from wife Wife remaining in actual occupation
of matrimonial home Husband remaining rateable occupier and paying wife interim alimony Husband informing rating
authority that future demands for rates to be served on wife Whether rateable occupation could have changed from that of the
husband to a joint rateable occupation.

The respondents husband was the sole rateable occupier of a house. In November, 1960, he left her, but she remained in
occupation of the house where she was still living. Her husband was paying her a weekly sum under an order for alimony
pendente lite, and he informed the appellants, the rating authority, by letter that all future demands for rates on the house should
be served on the respondent. The appellants sought to recover rates from him for the year 1961/62, but the justices refused to
issue a distress warrant. On appeal by the appellants against the refusal of the justices (differently constituted to those
determining the summons against the husband) to order the respondent to pay the arrears of rates,

Held The respondent was not liable for the arrears of rates because the husband remained a rateable occupier of the house after
he left the respondent (see p 529, letter c, post), and, just as there would not be joint rateable occupation when husband and wife
were living in a house but not cohabiting, so also they did not become joint rateable occupiers when they were not occupying the
same houses (see p 529, letter i, post).
Cardiff Corpn v Robinson ([1956] 3 All ER 56) applied.
Appeal dismissed.

Notes
As to rateable occupation by a spouse, see 32 Halsburys Laws (3rd Edn) 21, para 23; and for cases on the subject, see 38 Digest
(Repl) 482, 52; 509, 216219.

Cases referred to in judgments


Beckwith v Barnet UDC (Collector) (1950), 43 R & ITR 346.
Cardiff Corpn v Robinson [1956] 3 All ER 56, [1957] 1 QB 39, [1956] 3 WLR 522, 120 JP 500, 38 Digest (Repl) 482, 52.

Case Stated
This was a Case Stated by justices for the county of Surrey in respect of their adjudication as a magistrates court sitting at
Kingston-upon-Thames on 16 November 1962. On 23 October 1962, complaint was made by the appellants, Malden and
Coombe Borough Council (being the local rating authority), that the respondent, Agnes Esther Swainson Bennett, of 21,
Linkside, New Malden, being a person duly rated and assessed in a rate made on 22 May 1961, in the sum of 67 4s payable in
two half-yearly instalments in respect of 21, Linkside for the period from 1 April 1961, to 31 March 1962, had not paid that sum
or any part of it. The following facts were found. At the time of making the rate for 1960/61, the name of Herbert George
Bennett, husband of the respondent, was entered in the appellants records as the occupier of the premises, but, on 29 December
1960, after he was known to have left the premises, he informed the appellants that all future demands for rates on 21, Linkside
should be served on the respondent. The rate for the year 1961/62 was made on 22 March 1961, and, on 1 April 1961, a demand
for the rate for the first half-year 1961/62 was served on the respondent. On 1 October 1961, a demand for the rate for the second
half-year 1961/62 was served on the respondent. By March, 1962, no payment whatsoever had been made by the respondent and
a demand was, therefore, served on Mr Bennett on 22 March 1962, in respect of the full years rates for 1961/62. Mr Bennett
made no 527 payment in response to this demand. A summons was issued by the appellants and, on 29 May 1962, a court
composed of justices of the petty sessional division of Kingston-upon-Thames (on which none of the present justices sat), having
heard evidence given by Mr Bennett concerning the terms on which the respondent resided at 21, Linkside, determined that he
was not in occupation thereof and, therefore, not liable for payment of the general rate. The court, therefore, declined to issue a
distress warrant as requested by the appellants. The appellants appealed against this decision by way of Case Stated, but were
unable to proceed because of procedural difficulties in timing. As late as 13 August 1962, solicitors for the respondent offered
the payment of 5s. per week on behalf of the respondent. The appellants were not prepared to accept this offer, and sought
further information, their request still being outstanding. Mr Bennett had left the premises on 29 November 1960. The
respondent was living at 21, Linkside with her daughter, and she had no other home to go to and would, therefore, expect to
remain at 21, Linkside. The respondent had changed the locks on the premises, and Mr Bennett had no key, but the furniture at
21, Linkside belonged to him. The respondents only income was 4 a week allowed to her by Mr Bennett under an order for
interim alimony pending suit made by Mr Registrar Forbes and dated 5 March 1962. This order contained no reference
whatsoever to the payment of rates. A letter dated 29 November 1960, written by Mr Bennett was produced but, in the absence of
Mr Bennett, the justices could not say whether this did or did not contain the basis of an agreement between Mr Bennett and the
respondent. The respondent denied that she had ever accepted the liability apparently placed on her by that letter to pay the rates
on 21, Linkside. The four conditions necessary to establish liability for payment of the rates referred to in John Laing & Son Ltd
v Kingswood Assessment Committee ([1949] 1 All ER 224; [1949] 1 KB 344) were fulfilled by the respondent. Her position
approximated to that of the wife in Cardiff Corpn v Robinson ([1956] 3 All ER 56; [1957] 1 QB 39).
It was submitted on behalf of the respondent that, though a decision given by a different bench of justices to the effect that
Mr Bennett was not liable for the payment of the rate might appear to embarrass the court, nevertheless the court should not
allow that prior judgment to influence them at this hearing and should find on the basis of Cardiff Corpn v Robinson ([1956] 3 All
ER 56; [1957] 1 QB 39) that Mr Bennett was liable and that the respondent was not, therefore, rateable. It was submitted on
behalf of the appellants that, on the basis of Cardiff Corpn v Robinson ([1956] 3 All ER 56; [1957] 1 QB 39) it was arguable
whether or not Mr Bennett could be considered in rateable occupation of 21, Linkside. However, the matter before the court was
whether the respondent could be rated in respect of her occupation and she, in examination and cross-examination, had given
replies which indicated that, by living at 21, Linkside, she fulfilled the four conditions of rateable occupation based on John
Laing & Son Ltd v Kingswood Assessment Committee ([1949] 1 All ER 224; [1949] 1 KB 344). For differing reasons it might be
that there were grounds for holding that Mr Bennett or the respondent could be held to be in rateable occupation. In view of the
previous decision by a court composed of different justices but acting for the same division that Mr Bennett was not in rateable
occupation and, as the respondent fulfilled the four conditions of rateable occupation, then the warrant against her should be
issued as requested by the appellants.
The justices were aware of the fact that a court composed of their colleagues had dismissed the case against Mr Bennett, but
were not aware of the evidence given in the previous case or of the reasons which prompted their colleagues to come to the
decision. They considered submissions made on behalf of the appellants that dual responsibility for rates might exist (both Mr
Bennett and the respondent being liable), and the submission on behalf of the respondent that 528 this was not so. They inclined
to the view that dual responsibility could exist in the present circumstances, but could find no recorded case nor any specific
enactment on the point. There being a doubt in their minds on the law, they decided that the respondent was entitled to the
benefit of the doubt and dismissed the application. The appellants now appealed. The case noted below a was cited during the
argument in addition to the case referred to in the judgment of Lord Parker CJ.
________________________________________
a Bendall v McWhirter [1952] 1 All ER 1307; [1952] 2 QB 466

J R Phillips for the appellants.


Evan Stone for the respondent.

1 May 1963. The following judgments were delivered.

LORD PARKER CJ. This is an appeal by way of Case Stated from a decision of justices for the county of Surrey sitting at
Kingston-upon-Thames who dismissed an application by way of complaint for non-payment of rates. [His Lordship stated the
facts, and continued:] If one goes back to the position before November, 1960, it is clear that Mr Bennett alone was the rateable
occupier of 21, Linkside, New Malden. That he remained a rateable occupier of 21, Linkside after he had left the respondent is,
in my judgment, abundantly clear from the judgment of this court in Cardiff Corpn v Robinson. In that case a husband left his
wife in the matrimonial home and went to live elsewhere. The rating authority sought to levy the rates on him, and the contention
was that, as from the time he left her, she was in the exclusive occupation and alone was the rateable occupier. The court there,
although it did not hold expressly that the wife was not a rateable occupier, held quite definitely that the position of the husband
had never changed and that he was a rateable occupier. He was discharging his obligation to maintain her and thereby deriving a
beneficial use from the house. As Donovan J said at the end of his judgment ([1956] 3 All ER at p 58; [1957] 1 QB at p 46):

by turning his occupation into account in the way he has done, this respondent remains in rateable occupation.

Accordingly Mr Bennett, after he left the respondent, continued to be a rateable occupier. Although counsel, who has said all that
can be said on behalf of the appellants, has not expressly admitted that, he has gone so far as to say that that probably is the case;
but his argument really is that, as from the moment when the husband left, while the husband may well have remained in rateable
occupation, yet there was such a change of circumstances that the respondent also became a rateable occupier of the premises. In
other words, counsel for the appellants says that a time came when, although the husbands position in relation to the premises
had not changed, the respondents had, and that, as from that time, there was a joint rateable occupation, in which case it is
undoubtedly true that the rating authority could recover the whole rates from one or the other.
Counsel have been unable to refer to any case of joint occupation other than, of course, that of partners. For my part, I see
very great difficulties in any such relationship. It is quite clear that there is no joint rateable occupation when husband and wife
are living together. It is also, I think, clear that there is no joint rateable occupation when husband and wife are living in the same
house, albeit not cohabiting: see Beckwith v Barnet UDC (Collector). It would, accordingly, be extraordinary if the position
changed, and when the husband and wife were not occupying the same premises at all, yet they became joint rateable occupiers.
I have come to the conclusion that that is a quite impossible state of affairs, and, though sorry for the rating authority in the
present case, 529I am quite satisfied that the justices were right and that this appeal should be dismissed.

HAVERS J. I agree with the judgment given by my Lord and the reasons for it, and I do not think that I can usefully add
anything.

EDMUND DAVIES J. I also agree.

Appeal dismissed.

Solicitors: Town clerk, New Malden (for the appellants); Gregsons (for the respondent).

N P Metcalfe Esq Barrister.


[1963] 2 All ER 530

Re British Steel Founders Associations Agreement


COMPETITION

RESTRICTIVE PRACTICES COURT


MEGAW P, BUCKLEY J, LORD CAMERON, SIR STANFORD COOPER, MR W L HEYWOOD AND MR D V HOUSE
26 APRIL 1963

Restrictive Trade Practices Reference British Steel Founders Association Determination of certain restrictions before
reference Guidance prices restrictions declared contrary to policy interest, and undertaking given Ancillary restrictions
removed from register in pursuance of directions of Board of Trade Whether court should make order as regards the ancillary
restrictions Restrictive Trade Practices Act, 1956 (4 & 5 Eliz 2 c 68), s 12(1), s 20(1).

Where the Restrictive Practices Court has declared some, but not all, restrictions under an agreement referred to the court to be
contrary to the public interest and has adjourned the reference, directions may validly be given by the Board of Trade under s
12(1) of the Restrictive Trade Practices Act, 1956 a, for the removal of the remaining restrictions from the register, and, if they are
given, the court may, on the adjourned hearing, refrain from making any order under s 20(1) of the Act b.
________________________________________
a Section 12(1) provides: The Board of Trade may, upon the representation of the Registrar, give directions authorising him to remove from
the register particulars of such agreements of which particulars are for the time being entered therein as appear to the Board to be of no
substantial economic significance.
b Section 20(1), so far as relevant, provides: The court shall have jurisdiction, on application made in accordance with this section in respect
of any agreement of which particulars are for the time being registered under this Part of this Act, to declare whether or not any restrictions
by virtue of which this Part of this Act applies to the agreement are contrary to the public interest.

Notes
As to directions by Board of Trade authorising removal from register of particulars of agreements of no economic significance,
see 38 Halsburys Laws (3rd Edn) 107, 108, para 138. As to orders of the Restrictive Practices Court, see ibid, 132, para 184.
For the Restrictive Trade Practices Act, 1956, s 12, s 20, see 36 Halsburys Statutes (2nd Edn) 946, 952.

Case referred to in judgment


British Constructional Steelwork Assocns Agreement, Re [1959] 1 All ER 428, LR 1 RP 199, [1959] 1 WLR 306, 3rd Digest
Supp.

Adjourned reference
Pursuant to s 20(2)(a) of the Restrictive Trade Practices Act, 1956, the Registrar of Restrictive Trading Agreements referred to the
Restrictive Practices Court the agreement between the members of the British Steel Founders Association. On 14 January 1963,
the court made an order declaring that certain restrictions 530 which had been terminated before the date of the reference were
contrary to the public interest, but adjourned the question of certain ancillary restrictions which, pursuant to directions of the
Board of Trade on 31 December 1962, under s 12 of the Act of 1956, had been removed from the register. The facts are set out in
the judgment of the court.

Walter Gumber for the British Steel Founders Association.


R H W Dunn QC and R A Barr for the Registrar.

26 April 1963. The following judgment was delivered.

MEGAW P delivered the following judgment of the court: The matter before the court arises in this way. On 10 September
1959, reference was made by the registrar of an agreement of the British Steel Founders Association. The agreement, broadly
speaking, contained two types of restriction falling within s 6 of the Restrictive Trade Practices Act, 1956. The first of those
types was what has been called guidance prices, and the second was what has been called ancillary recommendations, which were
concerned with standard terms and conditions, and certain technical matters. Some restrictions had been terminated before the
notice of reference. Originally, the Association sought to justify the guidance prices under s 21(1)(b) of the Act and the ancillary
recommendations under s 21(1)(g). The registrar in his answer contended that justification could not be made within the terms of
the Act, and he also submitted that, on balance of public interest, the restrictions should be condemned. On 3 September 1962,
the registrar issued a notice for a final order in the common form asking that all the restrictions should be condemned. On 8
October 1962, that matter came before the court. At that stage, the draft order which the court was asked to make involved a
division of the restrictions into two parts. First, the restrictions in respect of guidance prices, which the Association did not then
seek to support and in respect of which, in lieu of an injunction, an undertaking was then given by the Association involving an
undertaking that they would not enter into any agreement to the like effect. Secondly, with regard to the restrictions in the
ancillary recommendations, it was mentioned to the court that the registrar was then considering making representations to the
Board of Trade under s 12 of the Act inviting the Board of Trade to give directions to the registrar to remove those restrictions
from the register on the ground that they were of no substantial economic significance. The court on that occasion directed that
the question of the ancillary restrictions should stand over until the first application day of the following term.
The registrar thereafter made representation to the Board of Trade under s 12 of the Act in relation to substantially all the
ancillary restrictions. On 31 December 1962, the Board of Trade gave a direction which has been placed before the court, under
which it was said that the Board of Trade, in pursuance of the power conferred on it by s 12(1), gave the following directions:

The Registrar of Restrictive Trading Agreements is authorised to remove from the register kept by him under the Act
particulars now included therein of the said agreement and all variations thereof being particulars relating solely to the
recommended conditions of sale now subsisting,

that is, the ancillary restrictions. Counsel on behalf of the registrar submitted that the effect of that direction was mandatory on
the registrar, and that he had no option but to comply with it, and that, accordingly, the restrictions in question were so far as the
registrar was concerned, properly and necessarily removed from the register.
On 14 January 1963, the matter came before the court again. The court on that occasion made an order declaring that the
restrictions which had been terminated before the date of reference to the court were contrary to the public interest, but the court
at that stage had it in mind that there was a possible difficulty in relation to s 12 of the Act, and it was directed that the question of
the ancillary restrictions should be adjourned for a date to be fixed for hearing by a full court. It is 531 in accordance with that
adjournment that the matter has come before the full court.
It was submitted on behalf of the registrar that the court had no jurisdiction to inquire into the validity of a direction made or
purporting to be made by the Board of Trade under s 12 of the Act, but it was said that the court had jurisdiction in its discretion
to deal with the matter under s 20(1), even where such a direction had been given. The court was invited in the exercise of its
discretion either to make no order or to stand the matter over generally, which would have the same effect. It was submitted that
the discretion ought not to be exercised in any other way having regard to the direction given by the Board of Trade. Counsel for
the registrar did not feel it necessary or desirable to make any submission as to the true construction of s 12. He cited a number
of orders made by the court, beginning with the order in Re British Constructional Steelwork Assocns Agreement, in which the
court, without the question of construction having been raised in any form, acceded to the application of the registrar and of the
respondents in that case to adjourn the hearing in relation to certain restrictions in order to enable the registrar to refer it to the
Board of Trade under s 12. The position in that case was comparable to the position in the present case, in that a part of the
restriction had been declared by the court to be against the public interest and the representation which was contemplated and
subsequently made under s 12 was in relation to all the restrictions which remained subsisting.
Counsel for the Association adopted the registrars submissions, save that he contended that, where any restrictions had been
removed from the register under s 12, the court did not have a discretion, but had no jurisdiction whatever to deal under s 20 with
the removed restrictions. He submitted further that, if it was open to the court to examine the validity of the directions of the
Board of Trade under s 12, the true conclusion was that those directions were intra vires and valid. He based this on two
alternative arguments: (i) that in s 12 the word agreements means only subsisting agreements, and that, where restrictions had
already been abandoned or had been removed from the register, the only agreement remaining was an agreement which did not
contain the terms thus abandoned or removed from the register. Therefore, he said, assuming that the powers of the Board of
Trade are limited to directions as to an agreement as a whole, the directions of the Board of Trade in this case related to the whole
subsisting agreement as it was at the date of making of the representation to the Board of Trade. (ii) Alternatively, he contended
that, on its true construction, s 12 enabled the Board of Trade to give directions as to a part of a registered agreement.
In the absence of argument to the contrary effect, while we have felt, and feel, considerable doubt as to the true construction
of s 12, we do not think it right to hold in the circumstances of this case that it precludes the giving of the directions which the
Board of Trade have given, bearing in mind that, in this case, the directions given relate, and relate only, to all the restrictions still
subsisting, in the sense that all the other restrictions in the agreement as originally registered have already been dealt with by the
court under s 20(1). Accordingly, the court will make no order.

No further order.

Solicitors: Allen & Overy (for the British Steel Founders Association); Treasury Solicitor.

Mary Colton Barrister.


532
[1963] 2 All ER 533

Caravans & Automobiles Ltd v Southall Borough Council


TOWN AND COUNTRY PLANNING

QUEENS BENCH DIVISION


LORD PARKER CJ, HAVERS AND EDMUND DAVIES JJ
3 MAY 1963

Town and Country Planning Enforcement notice Validity Occupier Tenant of caravan site permitting companies in
which he had interest to occupy parts of site for displaying and parking caravans and trailers Site not sub-let to companies
One company making money payment to tenant Enforcement notice served on one company as occupier of site Town and
Country Planning Act, 1947 (10 & 11 Geo 6 c 51), s 23(1).

The appellants and W Co Ltd carried on business on a site, of which G was tenant, and used it for the display and parking of
caravans and trailers. G was a shareholder of both the appellants and W Co Ltd and he was also a director of W Co Ltd. G had
not granted any sub-tenancy or other interest of any part of the site to either the appellants or W Co Ltd but they both carried on
business from the site with Gs consent, the appellants had signboards on the site with Gs knowledge and they made a money
payment to him. The respondents served an enforcement notice under s 23 of the Town and Country Planning Act, 1947 a, on the
appellants as occupiers of the site requiring them to discontinue the use of the site for the display and parking of caravans and
trailers. On appeal by the appellants against their conviction of an offence against s 24(3) of the Act of 1947 in not complying
with the terms of the enforcement notice,
________________________________________
a Section 23(1), so far as material, provides: If it appears to the local planning authority that any development of land has been carried out
after the appointed day without the grant of permission required in that behalf under this Part of this Act the local planning authority
may within four years of such development being carried out, serve on the owner and occupier of the land a notice under this section.

Held The appeal must be allowed and the conviction quashed because occupier in s 23(1) of the Act of 1947 must mean all
the occupiers of the site and, on the facts, either G was the occupier of the whole site, or the appellants and W Co Ltd were the
occupiers of it, and only the appellants had been served with the enforcement notice (see p 535, letter d, and p 535, letter i, to p
536, letter a, post).
Appeal allowed.

Notes
Section 23(1) of the Town and Country Planning Act, 1947, has been repealed and re-enacted in different form by s 45(1) of the
Town and Country Planning Act, 1962, which came into force on 1 April 1963.
As to service of enforcement notices, see 37 Halsburys Laws (3rd Edn) 332, 333, para 438.
For the Town and Country Planning Act, 1947, s 23(1), see 25 Halsburys Statutes (2nd Edn) 524; and for the replacing
Town and Country Planning Act, 1962, s 45(1), see 42 ibid, 1015.

Case Stated
This was a Case Stated by justices for the county of Middlesex in respect of their adjudication as a magistrates court sitting at
West Ealing on 6 December 1962.
On 9 October 1962, an information was preferred by the respondents, Southall Borough Council, against the appellants,
Caravans & Automobiles Ltd that they had continued to use certain land for the display and parking of caravans and trailers in
contravention of a notice dated 3 April 1962, served on them as the occupiers requiring them to discontinue that use of the land
and without planning permission, contrary to s 24(3) of the Town and Country Planning Act, 1947. The following facts were
found. The notice was served on the appellants as occupiers of the land. The freehold of the land was held by the London Co-
operative Society Ltd and a Mr Harry Gold was the tenant of the land 533 under a tenancy agreement made between him and the
freeholders on 2 December 1957. No enforcement notice was served on Mr Gold. The respondents knew that Mr Gold was the
tenant of the land, and had at no time disputed this. Mr Gold was a shareholder of the appellants and of the West End Caravan Co
Ltd. He was also a director of the West End Caravan Co Ltd but not of the appellants. The appellants traded under the name of
the Greenford Caravan Co, and there were signboards on the land which carried the name Greenford Caravan Co. The
appellants, and also the West End Caravan Co Ltd carried on business from the land, such business being the display and parking
of caravans and trailers. Mr Gold had not granted any sub-tenancy or other interest of any part of the land to either the appellants
or the West End Caravan Co Ltd but both the appellants and the West End Caravan Co Ltd carried on business from the land with
Mr Golds consent and the appellants signboards were on the land with Mr Golds knowledge and consent. The appellants made
a money payment to Mr Gold. Since 30 May 1962 (the date referred to in the enforcement notice), the land has been used for the
display and parking of caravans and trailers by the appellants and by the West End Caravan Co Ltd.
It was contended by the appellants that they were not the occupiers of the land and that the enforcement notice and the
information should have been served not on them but on Mr Gold. They were not guilty of an offence under s 24(3) of the Town
and Country Planning Act, 1947, and not guilty of the offence charged in the information. It was contended by the respondents
that the appellants were the occupiers of the land and guilty of the offence charged in the information.
The justices convicted the appellants and fined them 25. The appellants now appealed.
The cases noted belowb were cited during the argument.
________________________________________
b R v St Pancras Assessment Committee (1877), 2 QBD 581, Fairman v Perpetual Investment Building Society [1923] AC 74, Excelsior Wire
Rope Co Ltd v Callan [1930] All ER Rep 1, [1930] AC 404, Westminster City Council v Southern Ry Co Railway Assessment Authority and
Smith & Son Ltd [1936] 2 All ER 322, [1936] AC 511, Hartwell v Grayson Rollo and Clover Docks Ltd [1947] KB 901

A de Piro for the appellants.


R M Yorke for the respondents.

3 May 1963. The following judgments were delivered.

LORD PARKER CJ. This is an appeal by way of Case Stated from a decision of justices for the county of Middlesex sitting at
West Ealing who convicted the appellants of an offence against s 24(3) of the Town and Country Planning Act, 1947, in not
complying with the terms of an enforcement notice. May I say at the outset that, for my part, I have the greatest sympathy with
the justices in this case, and it may be that more came out in evidence than appears in the Case. However, so far as this court is
concerned, they are confined to the facts as they appear in the Case. [His Lordship stated the facts, and continued:] The Act of
1947, as is well known, provides, by s 23, that where a local planning authority is satisfied that some development has taken
place within the preceding four years, and that includes non-compliance with any limitation or condition, they may serve on the
owner and occupier of the land a notice under the section, and that is the enforcement notice. What is said on behalf of the
appellants is that service made on them (and the question of service on the owner did not arise) was not a service on the
occupier of the land. Counsel for the appellants has referred the court to a number of cases concerning the law of occupiers, and
also to rating cases as to who can be said to be in rateable occupation. Without deciding the matter, I myself have very grave
doubts whether any of those cases are of any assistance in considering who is the occupier for the purposes of the Town and
Country Planning Act, 1947. It seems to me that different considerations are involved when one is thinking of occupiers
liability, when one is considering rateable occupation and when one is considering town and country planning cases. In so far as
this case is concerned, I find it unnecessary 534 to decide what exact definition should be applied to the word occupier for the
purposes of town and country planning legislation. I find it unnecessary for this reason. As it seems to me on the facts as found,
either Mr Gold himself was the occupier, as being the person in full control of the premises, allowing companies in which he has
an interest to occupy for certain periods certain parts of the site for business purposes, or else he has parted with control to his
companies. If he has parted with control to his companies, giving them separate, exclusive interests in different parts of the site,
then both caravan companies are in occupation, each in occupation of its part. There is no suggestion here that there was a joint
occupation by two caravan companies of the whole of the site, in which case it may be that the service of the notice on one of the
joint occupiers of the whole would be sufficient. Accordingly, it seems to me that either the true view is that Mr Gold was the
occupier of the whole site, or that different parts of the site were occupied by the two different caravan companies, in which case
only the occupier of part of the site has been served.
Counsel for the respondents has argued that occupier in s 23(1) is the person that anyone walking along the road and
looking at the site would say was the occupier, and that here the only person who had signboards or advertisements up was the
appellants, and, therefore, that one was entitled to say that the appellants were the occupiers. For my part, I cannot accede to that
argument. Whatever occupier means for this purpose, it must, I think, mean all the occupiers who are occupiers of different
parts of the site the subject of the enforcement notice. Here, it seems that only one occupier was served, if it can be said that the
caravan companies were truly in occupation, or if they were not, Mr Gold as the occupier was not served.
Accordingly, I would allow this appeal and quash the conviction.

HAVERS J. I agree, and as we are differing from the justices I would add a few observations of my own. I share the sympathy
that my Lord has expressed for the justices in this case. On this site the appellants were carrying on a business, namely, the
display and parking of caravans and trailers. They had their signboard exhibited on the land, and they were there clearly under
some sort of contract or licence with Mr Gold, who was the tenant of the whole site and they made a money payment to Mr Gold.
As my Lord has said, if those were the whole of the facts, I should not have hesitated in saying, with the justices, that the
appellants were in occupation of the site. Those, however, were not the whole of the facts, because the Case finds that, in
addition to the business being carried on by the appellants on the land, there was another company, the West End Caravan Co Ltd
of which Mr Gold was a director which was similarly carrying on business on the land. No notice was served on Mr Gold, who
is the undoubted tenant of the whole site, and I feel constrained to come to a similar conclusion as my Lord, that, on the facts as
found in this case, either the position is that Mr Gold is the man who has control of the whole site, allowing these two companies
to occupy parts of it for the purpose of carrying on their respective businesses in which Mr Gold is interested, or, alternatively,
that each of these two companies, by permission of Mr Gold, is occupying some part, and some undefined part, according to the
findings in effect of the Case, of the site. Section 23(1) of the Town and Country Planning Act, 1947, requires the authority, if
they desire to exercise their powers, to serve the enforcement notice on the owner and occupier of the land. Owner is defined
in s 119(1), the interpretation section, but occupier is not. I do not myself find assistance from the authorities to which we have
been referred which deal with the common law liability of an occupier, either before the Occupiers Liability Act, 1957, or under
that Act, or from the authorities which deal with rating cases. Beneficial occupation, from the point of view of rating, is
something different from occupation under the Town and Country Planning Acts. I am forced to a similar conclusion to that to
which my Lord has come, that, there being no notice served 535 on Mr Gold and no notice served on all the occupiers of this
piece of land, the findings of the justices cannot be supported.

EDMUND DAVIES J. I agree. I share the sympathy expressed with the justices who had to deal with this case and from whom
we are now differing. I confess to being mystified, however, that the respondents chose to serve an enforcement notice on these
particular appellants in view of the power vested in them by s 106 of the Town and Country Planning Act, 1947, to obtain any
information which they felt they needed in relation to the exact use enjoyed, and occupation, of this particular site in relation to
which they already possessed considerable knowledge. They were familiar with Mr Harry Gold, and with the fact that he was the
tenant of the site in respect of which the notice was served.

Appeal allowed.

Solicitors: Shurman & Bindman (for the appellants); Town clerk, Southall (for the respondents).

N P Metcalfe Esq Barrister.


[1963] 2 All ER 536

Cumberland Court (Brighton) Ltd v Taylor


LAND; Sale of Land, Mortgages

CHANCERY DIVISION
UNGOED-THOMAS J
9 OCTOBER 1962

Sale of Land Title Mortgage Discharge of charge by way of legal mortgage by receipt indorsed Receipt dated two days
after conveyance to vendors predecessor in title Notice served by vendor requiring completion within twenty-eight days Time
made the essence of the contract by condition of sale Whether vendor able and ready to complete National Conditions of Sale
(17th Edn), condition 22 Law of Property Act, 1925 (15 & 16 Geo 5 c 20), s 87, s 115(3).

An agreement for the sale of freehold land free from encumbrances incorporated condition 22 a of the National Conditions of Sale
(17th Edn). That condition, so far as material, provided that at any time after the completion date, either party being himself
able, ready and willing to complete might give to the other party notice in writing requiring completion of the contract in
conformity with the condition and that on service of such notice it should become a term of the contract, in respect of which time
should be of the essence, that the party to whom the notice was given should complete within twenty-eight days after service.
The abstract of title disclosed, among other documents, a legal charge dated in 1947 made by a predecessor in title of the vendors.
A receipt by the chargee by way of legal mortgage had been indorsed on the legal charge, but was dated two days after the date of
the conveyance by the mortgagor to a purchaser from him. On the present sale objection was taken, the time for delivering
requisitions having passed, that a good title had not been shown, as the receipt operated as a transfer since it was given two days
after the conveyance. After the date for completion of the present sale, the vendors served on the purchaser notice under
condition 22. The purchaser did not complete within the twenty-eight days allowed by the notice. In proceedings by the vendors
for, among other relief, a declaration that by reason of the defendant purchasers breach of agreement the vendors were no longer
bound to perform the agreement,
________________________________________
a The material terms of condition 22 are set out at p 537, letter i, post

Held (i) the words where there is no right to keep the mortgage alive, the receipt does not operate as a transfer in s 115(3) b of
the Law of Property Act, 1925, did not prevent the receipt in the present case, operating as a transfer (see p 541, letter d, post);
but, if it did so operate, then, although the legal charge did not create a legal estate, the doctrine of feeding the estoppel would
apply, with the consequence that the chargee by way of 536 legal mortgage would not be in a position to exercise any rights that
he might have had under the legal charge c (see p 541, letter i, and p 542, letter d, post); accordingly, the requirement of condition
22 that the vendors should be able and ready to complete was satisfied, and the vendors were entitled to a declaration that by
reason of the purchasers breach the vendors were no longer bound to perform the agreement of sale.
________________________________________
b Section 115(3)is printed at p 541, letter b, post
c The material terms of s 87 are set out at pp 541, 542, post

(ii) the objection did not go to the root of the title, and the time for requisitioning having passed, was not maintainable by the
purchaser (see p 542, letter f, post).
Observations of Joyce J in Pryce-Jones v Williams ([1902] 2 Ch at p 522) considered.
(iii) time became the essence of the contract under condition 22 (see p 543, letters c and d, post) or alternatively because the
period of notice under that condition was in the circumstances reasonable.
Dictum of Danckwerts J in Re Barrs Contract ([1956] 2 All ER at p 857) applied.

Notes
As to the statutory receipt to be indorsed on a mortgage, see 27 Halsburys Laws (3rd Edn) 406, 407, para 783; and as to its
operating as a transfer, see ibid, p 407, 408, para 786.
As to the doctrine of feeding the estoppel, see 15 Halsburys Laws (3rd Edn) 220, 221, para 418; and for cases on the
subject, see 21 Digest (Repl) 362364, 10621080.
As to the consequences of requisitions being too late where a vendors title is wholly bad, see 34 Halsburys Laws (3rd Edn)
246, 247, para 410; and for cases on the subject, see 40 Digest (Repl) 93, 698709.
As to making time of the essence of a contract, see 36 Halsburys Laws (3rd Edn) 322324, paras 468, 469; and for cases on
the subject, see 42 Digest 511514, 770786.
For the Law of Property Act, 1925, s 87, s 115, see 20 Halsburys Statutes (2nd Edn) 617, 682.

Cases referred to in judgment


Barrs Contract, Re, Moorwell Holdings Ltd v Barr [1956] 2 All ER 853, [1956] Ch 551, 40 Digest (Repl) 121, 944.
Pryce-Jones v Williams [1902] 2 Ch 517, 71 LJCh 762, 87 LT 260, 40 Digest (Repl) 89, 672.
Weg Motors Ltd v Hales [1961] 3 All ER 181, [1962] Ch 49, 3rd Digest Supp.

Action
By an agreement in writing dated 22 February 1961, made between Cumberland Court (Brighton) Ltd (the vendors) of the one
part and Jack Taylor)the defendant) of the other part, it was agreed that the plaintiffs should sell to the defendant purchaser for
18,500, certain freehold property known as Cumberland Court, situate at Nos 150 and 151, Kings Road, Brighton, and certain
furniture, fixtures and fittings. In accordance with the agreement the purchaser paid 1,850 by way of deposit to Messrs B
Jackson & Co of Brighton, as stakeholders. The date fixed by the agreement for completion was 22 May 1961, and the
agreement incorporated the National Conditions of Sale (17th Edn). Condition 22 of the National Conditions, so far as material,
was as follows:

(1) At any time on or after the completion date, either party, being himself able, ready and willing to complete, may
(without prejudice to any other right or remedy available to him) give to the other party or his solicitor notice in writing
requiring completion of the contract in conformity with this condition.
(2) Upon service of such notice as aforesaid it shall become and be a term of the contract, in respect of which time
shall be of the essence thereof, 537that the party to whom the notice is given shall complete the contract within twenty-
eight days after service of the notice (exclusive of the day of service); but this condition shall operate without prejudice to
any express right of either party to rescind the contract in the meantime.

The vendors title showed that the property sold had been charged by a legal charge dated 3 May 1947, and made between Sidney
Walters and Lloyds Bank Ltd to secure a sum not exceeding 6,500; and that a conveyance on sale dated 22 May 1950, by Sidney
Walters as beneficial owner conveyed the property to Richard Lewis Craseby in fee simple, the conveyance containing no
reference to the legal charge. The legal charge had indorsed on it a receipt, dated 24 May 1950, stating that the moneys thereby
secured had been repaid to Lloyds Bank, Ltd.
The date fixed for completion by the agreement was by mutual agreement postponed to 5 June. On 9 June completion not
having taken place, the vendors served on the purchaser a notice in writing purporting to be a notice under condition 22. The
purchaser did not complete within the period of twenty-eight days specified in that condition, but on the last day of the period,
namely, 7 July he registered the agreement as a land charge class C (iv) in the register of land charges.
The vendors, by their statement of claim dated 14 July 1961, claimed: (1) a declaration that by reason of the breaches by the
purchaser of the agreement the vendors were no longer bound to perform the same; (2) a declaration that the deposit of 1,850
had become forfeited to the vendors; (3) a declaration that the vendors pursuant to the agreement were at liberty to resell the
property comprised therein and that if the vendors should resell the property within six months of 8 July 1961, they were entitled
(on crediting the said deposit) to recover from the purchaser the amount of any loss occasioned to the vendors by expenses of or
incidental to such resale or by diminution in the price; (4) an order that the registration of the land charge be vacated. By para 3
of his defence and counterclaim dated 30 October 1961, the purchaser alleged that the vendors were not on 8 June 1961, or at any
subsequent date able willing and ready to complete the contract on the ground that at all material times the property comprised in
the agreement was subject to, inter alia, the legal charge dated 3 May 1947, and that by virtue of s 115 d of the Law of Property
Act, 1925, the legal charge was transferred to Sidney Walters (the vendor under the conveyance dated 22 May 1950). The
purchaser pleaded further (by para 4) that in a telephone conversation on or about 5 July 1961, between his solicitors and the
vendors solicitors, his solicitors inquired what steps the vendors solicitors were taking to discharge or release the property from
the legal charge and that they said that they were taking no steps on the ground that no requisition had been raised on the matter,
and that by a letter dated 6 July 1961, the vendors by their solicitors declined to clear the matter up and required the purchaser to
complete the purchase by 7 July 1961, and that in the premises the vendors were at all material times in default and were not able
ready and willing to complete the agreement. Further and in the alternative the purchaser pleaded (para 5) that the service of the
notice dated 8 July 1961, requiring him to complete within twenty-eight days was unreasonable and was void. The purchaser
stated (para 10) that he would, so far as necessary, rely on s 49 of the Law of Property Act, 1925, and he counterclaimed specific
performance of the agreement, further or alternatively damages, alternatively repayment of his deposit.
________________________________________
d The material terms of s 115 are set out at pp 540, 541, post

The vendors by their reply dated 13 November 1961, admitted that the property was subject to the legal charge dated 3 May
1947, until 24 May 1950, and pleaded (1) that by virtue of the Law of Property Act, 1925, s 76(1)(a), Sch 2, 538Pt 1e, the
conveyance dated 22 May 1950, was deemed to include a covenant by the then vendor with the then purchaser that
notwithstanding anything done or omitted by the then vendor he had full power to convey as beneficial owner, freed and
discharged or otherwise by the then vendor indemnified against the said legal charge, and that the vendor would after the date of
the conveyance on the request and at the cost of the then purchaser or any person deriving title under him execute and do all
things for further assuring the property to them as they should reasonably require; (2) that by virtue of s 76(6) f of the Act of 1925
the benefit of the said implied covenant was annexed to the property and enforceable by the owner of the property for the time
being; (3) that the vendors were at all material times the successors in title to the said purchaser and the present purchaser would
if he had properly completed the contract have been a successor in title to the former purchaser; and (4) that in the premises the
receipt dated 24 May 1950, and the payment of the moneys secured by the legal charge operated to feed by way of estoppel the
estate conveyed or purported to be conveyed by the conveyance dated 22 May 1950, and thereby to vest in the former purchaser
the legal estate in fee simple in the property freed and discharged from the legal charge and the moneys thereby secured. Further
or alternatively the vendors pleaded by their reply that, by virtue of condition 8 of the National Conditions the defendant was,
subject to requisitions, deemed to have accepted the title, that in respect of the delivery of requisitions time was of the essence of
the contract and that that time had expired. The plaintiffs also denied that the service of the notice dated 8 July 1961, was
unreasonable or void.
________________________________________
e 20 Halsburys Statutes (2nd Edn) 583, 874
f 20 Halsburys Statutes (2nd Edn) 590

Michael Albery QC and C A Brodie for the plaintiff vendors.


The defendant purchaser did not appear and was not represented.

9 October 1962. The following judgment was delivered.

UNGOED-THOMAS J. By a written agreement made on 22 February 1961, between the plaintiffs (the vendors) and the
defendant (the purchaser), the purchaser agreed to buy, for 18,500, the property known as Cumberland Court, 150/151, Kings
Road, Brighton, together with the furniture, fixtures and fittings. I understand that 1,000 of the 18,500 was apportioned to the
furniture, fixtures and fittings.
The agreement provided that on its signing the purchaser should pay 1,850 deposit to Messrs Jackson as stakeholders. That
deposit was accordingly paid. The agreement provided, by cl 9, that completion should take place on or before 22 May 1961.
Clause 10 of the agreement incorporated the seventeenth edition of the National Conditions of Sale. The relevant condition on
which this action largely turns is condition 22; the first two paragraphs of that condition are the relevant paragraphs. Paragraph 1
provides that:

at any time on or after the completion date, either party, being himself able, ready and willing to complete, may
(without prejudice to any other right or remedy available to him) give to the other party or his solicitor notice in writing
requiring completion of the contract in conformity with this condition.

Paragraph 2 provides that:

upon service of such notice as aforesaid it shall become and be a term of the contract, in respect of which time
shall be of the essence thereof, that the party to whom the notice is given shall complete the contract within twenty-eight
days after service of the notice (exclusive of the day of service); but this condition shall operate without prejudice to any
express right of either party to rescind the contract in the meantime.

The following sub-paragraph of the condition is not material to this action. Notice under condition 22, dated 8 June 1961, was
given by the vendors and served on 9 June. The twenty-eight days notice, by the condition provided, 539expired on 7 July and
on that date the purchaser registered a land charge. In those circumstances, the vendors claim, first, a declaration that, by reason
of the breaches by the purchaser of the agreement, the vendors are no longer bound to conform to the same; secondly, that the
deposit has become forfeited to the vendors; thirdly, that the vendors are at liberty to re-sell the property; fourthly, that the
registration of the land charge be vacated; fifthly, an inquiry as to damages.
There is a counterclaim in this case, but the purchaser has not appeared here today; that, therefore, has not been proceeded
with and I am not concerned with it. It will be noted that in condition 22 of the National Conditions of Sale, the requirements are,
first, that the written notice requiring completion shall be given on or after the completion date; no difficulty turns on that in this
case; secondly, that the vendors were themselves able, ready and willing to complete; that is the subject of dispute. The third
provision is that on the service of the notice it shall become and be a term of the contract, in respect of which time shall be of the
essence, that the party to whom the notice is given shall complete within twenty-eight days. In answer to that it is suggested in
the defence that, despite the express provision which I have just read, the notice must be a reasonable notice.
First then, with regard to the requirement that the vendors were able, ready and willing to complete. In answer to this
requirement it is pleaded in the defence that the vendors were not able, ready and willing to complete, because no good title had
been shown; and it is said that no good title had been shown for two reasons. First, it is said that, as disclosed by the abstract of
title, a conveyance of this property made on 22 May 1950, was not made free, as intended, from an outstanding legal charge. The
property was subject to a legal charge made on 3 May 1947; the receipt for the moneys secured by this legal charge, was dated 24
May 1950, and that receipt was indorsed on the legal charge. In view of the fact that the receipt was dated two days after the
conveyance it is pleaded that the receipt did not operate to free the property from the legal charge, but operated as a transfer
under s 115 of the Law of Property Act, 1925, to the vendor named in the 1950 conveyance, who, in fact, was the person who
paid off the moneys secured by the legal charge.
Counsel for the vendors has submitted that, on the true construction of s 115, the receipt did not operate as a transfer, but as
a discharge. Section 115(1), (2) are as follows:
(1) A receipt indorsed on, written at the foot of, or annexed to, a mortgage for all money thereby secured, which states the
name of the person who pays the money and is executed by the chargee by way of legal mortgage or the person in whom the
mortgaged property is vested and who is legally entitled to give a receipt for the mortgage money shall operate, without any
reconveyance, surrender, or release(a) Where a mortgage takes effect by demise or subdemise, as a surrender of the term, so as
to determine the term or merge the same in the reversion immediately expectant thereon; (b) Where the mortgage does not take
effect by demise or subdemise, as a reconveyance thereof to the extent of the interest which is the subject-matter of the mortgage,
to the person who immediately before the execution of the receipt was entitled to the equity of redemption; and in either case, as
a discharge of the mortgaged property from all principal money and interest secured by, and from all claims under the mortgage,
but without prejudice to any term or other interest which is paramount to the estate or interest of the mortgagee or other person in
whom the mortgaged property was vested.
(2) Provided that, where by the receipt the money appears to have been paid by a person who is not entitled to the immediate
equity of redemption, 540the receipt shall operate as if the benefit of the mortgage had by deed been transferred to him
Then sub-s (2) goes on to refer to the exceptions with which we are not concerned in this case. Subsection (3) says:

(3) Nothing in this section confers on a mortgagor a right to keep alive a mortgage paid off by him, so as to affect
prejudicially any subsequent incumbrancer; and where there is no right to keep the mortgage alive, the receipt does not
operate as a transfer.

Counsel for the vendors concedes that as the payment by the 1950 vendor was after the date of the conveyance it comes within
subsection (2); but he submits that the words in subsection (3), where there is no right to keep the mortgage alive, the receipt
does not operate as a transfer, following as they do on a semi-colon in the subsection, are words of general application. Here, he
says, there is no right to keep the mortgage alive and, therefore, the receipt does not operate as a transfer. But the words, no
right to keep the mortgage alive appear to me to refer to the earlier words in the same subsection, namely, a right to keep
alive a mortgage paid off by him ie by the mortgagor as defined by s 205(1)(xvi) of the Act which does not include the 1950
vendor at the date of the receipt. I do not read the words in the latter part of the subsection as being words of general application,
but as subsidiary to the words which are used in the previous part of the subsection. They do not therefore apply to the 1950
vendor who was not the mortgagor at the date of the receipt.
Then counsel for the vendors, says that, in any case, if there were any legal estate outstanding in the vendor in 1950 by
reason of the receipt having been effected two days after the conveyance, to which I have referred, that legal estate would have
vested in the then purchaser, and would be conveyed to the present purchaser by way of estoppel. In Williams on Vendor and
Purchaser (4th Edn), at p 1096, this paragraph appears:

As previously explained, if the conveyance to the purchaser contained a precise averment of the vendors seisin in fee
or other right, sufficient to work an estoppel at law, then if the vendor had not the estate specified at the time of conveyance
but afterwards acquired it, the same would immediately pass in effect to the purchaser and his successors in title without
any further conveyance, by reason of the doctrine that the acquisition of the legal estate feeds the estoppel. An estate by
estoppel of this kind would be available in favour of the purchaser and his successors in title, as against all persons
claiming the whole or any part of the vendors after-acquired estate by any title derived from him, whether gratuitously or
for value and whether for a legal or an equitable interest. But the legal estate would not so pass as against any person not
bound by the estoppel.

Here, in the conveyance of 22 May 1950, which was abstracted, there was a recital that: The vendor is seised in fee simple in
possession of the property with which we are concerned in this case, free from incumbrances and the conveyance
was by the vendor, to hold the premises unto the purchaser in fee simple. In those circumstances, it appears to me that the
principle as set out in the passage which I have read from Williams on Vendor and Purchaser applies.
But there is, in fact, no outstanding legal estate here at all, because a charge by way of legal mortgage does not create a legal
estate. The Law of Property Act, 1925, s 87, does not provide for the creation of a legal estate in such circumstances, but merely
confers the same remedies as would obtain if a legal estate were created. Section 87 provides:

(1) Where a legal mortgage of land is created by a charge by deed expressed to be by way of legal mortgage, the
mortgagee shall have the same 541 protection, powers and remedies (including the right to take proceedings to obtain
possession from the occupiers and the persons in receipt of rents and profits, or any of them) as if(a) where the mortgage
is a mortgage of an estate in fee simple, a mortgage term for three thousand years without impeachment of waste had been
thereby created in favour of the mortgagee; and (b) where the mortgage is a mortgage of a term of years absolute, a
subterm less by one day than the term vested in the mortgagor had been thereby created in favour of the mortgagee.

That enactment was considered in the case of Weg Motors Ltd v Hales ([1961] 3 All ER 181 at p 190; [1962] Ch 49 at pp 73, 74).
There, Lord Evershed MR said ([1961] 3 All ER 181 at p 190; [1962] Ch 49 at pp 73, 74):

It is said that although this instrument was expressed to be made by charge by way of legal mortgage, the effect of
s. 87 of the Law of Property Act, 1925, was to vest a term in the mortgagee. In our judgment this is precisely what the
section does not do. It provides that the mortgagee shall have the same protection and so forth as if a mortgage term had
been created in his favour. That is to say that he has in fact no term. Moreover, s. 87(2) enables a mortgagee to convert his
mortgage into a charge by way of legal mortgage and provides that in that case the mortgage term shall be extinguished.
This seems to us conclusive that the chargee by way of legal mortgage has no term.

Further, if the chargee by way of legal mortgage has no legal estate at all, there is no legal estate outstanding to be got in and,
although the doctrine of feeding the estoppel is directed to a case where there is a legal estate outstanding, a fortiori an estoppel
would similarly apply where, in fact, there is no legal estate outstanding at all. Consequently, the chargee by way of legal
mortgage, would not be in a position, nor would anybody claiming be in a position, to exercise any rights, powers or remedies
which he has by way of legal mortgage.
Finally, counsel for the vendors contends, with regard to that part of the defence with which I am dealing, that this is not a
matter which goes to the root of the title; that as the requisitions were out of time in this case, the purchaser is in no position to
take the objection to which I have referred. In Pryce-Jones v Williams Joyce J apparently considered ([1902] 2 Ch at p 522) that,
as the purchaser in that case obtained possession of the property and would get a perfectly good equitable title and could not be
disturbed, it was not a matter which went to the root of the title. Again, it seems to me that this contention is well-founded.
The other ground of objection to the ability of the vendors to complete in accordance with, and thus to satisfy the
requirements of, condition 22 of the National Conditions of Sale, turns on two legal charges, dated 22 May 1950. It was said that
these had not been discharged and that they had not been abstracted. I have heard evidence on this matter from Mr Ernest
Richard Tee, who had the conduct of these proceedings on behalf of the vendors solicitors, and I am completely satisfied from
his detailed and careful evidence that not only were the legal charges in fact discharged, but that the receipts of the discharge
were in fact duly abstracted for the purchasers solicitors and there was no defect at all in the abstract in that respect. Those
defences, therefore, in my view, fail.
The final defence raised was that the twenty-eight days notice was not reasonable. On this part of the case I was referred to
Re Barrs Contract, Moorwell Holdings Ltd v Barr. There Danckwerts J stated ([1956] 2 All ER at p 856 [1956] Ch at p 556):

Apart from the provisions of any plain and clear conditions of sale, the law about making time of the essence of the
contract for the purpose of completion, as is accepted on both sides, is subject to the following conditions.
542

Then he states the conditions but it will be seen that those conditions only apply where there are no plain and clear conditions of
sale which govern the situation. That is developed in a passage ([1956] 2 All ER at p 857; [1956] Ch at p 557), where
Danckwerts J quotes from Harman Jg, who quoted from Fry Jh, who said:
________________________________________
g In Smith v Hamilton [1950] 2 All ER 928 at p 933; [1951] Ch 174 at p 181
h In Green v Sevin (1879), 13 ChD 589 at p 599

That which is not of the essence of the original contract is not to be made so by the volition of one of the parties,
unless the other has done something which gives a right to the other to make it so. You cannot make a new contract at the
will of one of the contracting parties.

The position here is, of course, that both parties have agreed to make time of the essence of the contract in the circumstances
which are stated in condition 22 of the National Conditions of Sale, which I have read. The condition provides that on the notice
being served on or after the completion date (and that was done in this case), either party (the vendors in this case) being himself
able, ready and willing to complete (which I decide he was in this case), it shall become a term of the contract in respect of which
time shall be of the essence thereof, that the party to whom the notice is given shall complete the contract within twenty-eight
days of the service. Time automatically became of the essence of the contract, under para 2 of condition 22.
I have heard a good deal of evidence showing particulars of dates when the abstract was delivered, requisitions were
delivered, completion was postponed and the circumstances in which all that happened. It is unnecessary for me now, in view of
what I have said about time being of the essence of the contract by reason of condition 22, to go into all the details. Suffice it to
say that I am satisfied that in the circumstances of this case the notice which was given was a perfectly reasonable notice.
The result, therefore, is that the vendors, in my view, succeed in this action. The relief which they have claimed and which I
have already indicated is, in my view, the relief to which they are entitled, and I shall make declarations accordingly and make an
order that the registration of the land charge referred to in the case be vacated. There will be an inquiry as to damages.

Judgment for the vendors.

Solicitors: Arbeid & Co (for the plaintiff vendors).

Jacqueline Metcalfe Barrister.


543

[1963] 2 All ER 544

R v Westminster Betting Licensing Committee, ex parte Peabody Donation


Fund (Governors) and Another
LEISURE AND LICENSING

QUEENS BENCH DIVISION


LORD PARKER CJ, HVERS AND EDMUND DAVIES JJ
1 MAY 1963

Licence Betting office Application for licence Insertion of notice of application in sporting newspaper with limited
circulation in licensing area Whether application complied with Betting and Gaming Act, 1960 (8 & 9 Eliz 2 c 60), Sch 1, para
6.

The respondent wished to obtain a licence for a betting shop. In accordance with the provisions of the Betting and Gaming Act,
1960, he made his application to the appropriate authority, notified the local authority and the police, and caused a notice to be
posted up outside the entrance to the premises. In order to comply with Sch 1, para 6, to the Act of 1960 a, under which an
applicant for a betting shop licence had to cause to be published by advertisement in a newspaper circulating in the appropriate
authoritys area a notice of the making of the application, he inserted the notice of the application in The Sporting Life which had
a limited circulation in the area. The applicants applied for orders of certiorari to quash the decision of the local betting licensing
committee granting the respondent a licence on the ground that The Sporting Life, although a newspaper for the purposes of para
6, was not a newspaper circulating in the appropriate authoritys area.
________________________________________
a Paragraph 6, so far as material, is set out at p 545, letter d, post

Held The respondent had inserted an advertisement as required by para 6 of Sch 1 to the Act of 1960 and the applications must
be dismissed, because the words of para 6 could not be read as meaning a local newspaper (see p 545, letter h, and p 546, letters f
and h, post), and there was no national newspaper which could be said to be read by the general public of the area and not by a
limited class of the general public (see p 545, letter d, and p 546, letters g and h, post).
Applications dismissed.

Notes
As to advertising applications and the grant and renewal of permits and licences, see Supplement to 18 Halsburys Laws (3rd
Edn), para 392D (4).
For the Betting and Gaming Act, 1960, Sch 1, see 40 Halsburys Statutes (2nd Edn) 363.

Motions for certiorari


These were applications by way of motion by the Governors of the Peabody Donation Fund and by Mary Fae Callaghan for
orders of certiorari to bring up and quash a decision of the respondents, the Westminster Betting Licensing Committee, dated 9
October 1962, whereby they granted a licence for a betting shop to the respondent, Richard Frederick Best, pursuant to the
Betting and Gaming Act, 1960, in respect of the ground floor of premises at 79, Great Peter Street, Westminster, SW1. The facts
are set out in the judgment of Lord Parker CJ.
The authority, enactments and cases noted belowb were cited during the argument.
________________________________________
b Eddy and Loewes The New Law of Betting and Gaming (1st Edn) 47, Trustee Act, 1925, s 27(1), Clean Air Act, 1956, Sch 1, para 2,
Boydell v Drummond (1808), 2 Camp 157, Re Bracken (1890), 43 ChD 1

Ian McCulloch for the applicant.


J R Phillips for the respondent Best.
B T Wigoder for the respondents the Westminster Betting Licensing Committee.
544

1 May 1963. The following judgments were delivered.

LORD PARKER CJ. In these proceedings, counsel moves on behalf of two applicants, first the Governors of the Peabody
Donation Fund, and secondly Mrs Mary Fae Callaghan, for orders of certiorari to quash a decision of the Betting Licensing
Committee for Westminster, dated 9 October 1962, whereby they granted a licence for a betting shop to the respondent, Mr Best,
the applicant before them. The sole question here, as I understand it, is whether the proper machinery for advertising the
application was complied with. The application was in fact advertised in The Sporting Life, but the applicants and, indeed, many
others, did not take in The Sporting Life, and did not see the advertisement. However, they did get to hear of the application and
appeared at the hearing. It is only right to say that I am quite satisfied that the justices were prepared to hear and consider any
objections which were made. The applicants say: be that as it may, if the proper machinery of advertisement was not complied
with, then the justices had no jurisdiction, and they say that the advertisements in The Sporting Life in the issue of 6 September
1962, was not a compliance with the provisions of the Act.
The question of advertisements is dealt with in Sch 1, para 6, to the Betting and Gaming Act, 1960. So far as it is relevant to
these proceedings, that paragraph reads:

Not later than fourteen days after the making of any such application as aforesaid to the appropriate authority, the
applicant shall cause to be published by means of an advertisement in a newspaper circulating in the authoritys area a
notice of the making of the application the applicant shall also cause a like notice to be posted up outside the entrance,
or on the site of the proposed entrance, to the premises

The respondent had had a licence for a betting shop in premises in Greycoat Place; he was giving up those premises, and he
wished to obtain a licence in respect of the premises with which we are now concerned, namely, the ground floor of 79, Great
Peter Street. He made his application to the appropriate authority; he notified the local authority and the police; he caused a
notice to be posted up outside the entrance and then, in order to comply with the provision that there should be an advertisement
in a newspaper circulating in the authoritys area, he inserted the notice of the application in The Sporting Life.
The only point that counsel for the applicants can take in the present case is to say that, while he admits that The Sporting
Life is a newspaper for the purposes of para 6, yet it was not a newspaper circulating in the area. So far as the facts are concerned
as to that, the evidence is that The Sporting Life does circulate in the City of Westminster on weekdays to the extent of some
eighteen hundred copies. Counsel puts his argument in this way: he says, and this is perfectly true, that the object of
advertisement is to inform all those persons who are interested of the making of the application. Those people will be either rival
bookmakers or residents or those working in the locality, and he would say that the only way that one can properly inform all
those different types of person is by inserting the advertisement either in a local newspaperand there is a local newspaper
circulating in Westminsteror in the alternative in a newspaper which is read by the general public of Westminster, and not
merely, as he would put it, by the limited class that would read The Sporting Life. For my part, I am quite unable to read the
words in para 6 of Sch 1 as meaning a local newspaper. It does not say so, and it is to be observed that Parliament when it so
intended, as in Sch 12 to the Highways Act, 1959, specifically stated that the advertisement should be in a local newspaper. As
regards the alternative suggestion that it should be one read by the general public and not by a limited class, I find it very difficult
to see how that really advances the matter at all. There is no national newspaper which one could say is read by the general
public of Westminster. It may well be that, if one puts the advertisement in one of the national newspapers, it is not one which
will be read 545 by most of the people affected. I do not know, but the answer, as it seems to me, is that Parliament has left this
deliberately vague. It is to be observed that, of those two classes of people who are affected, namely, competitors and those who
wish to object to the location of the proposed betting shop, competition and location being the two matters referred to in para 20
(b) of Sch 1 which entitle the justices to refuse an application, the insertion of an advertisement in The Sporting Life was the
fairest and best way of drawing the attention of the respondents competitors to the application, whereas the advertisement on the
door of the proposed premises was best designed to bring the matter to the attention of those residing and working in the locality.
It seems to me that Parliament has left this procedure deliberately vague, and for my part I am quite unable to say that this
advertisement was not an advertisement in a newspaper circulating in the area.
On those short grounds, I have come to the conclusion that these applications fail, and should be dismissed.

HAVERS J. I agree. The only point taken by counsel for the applicants is that the respondent, who was applying for a betting
office licence, failed to comply with one of the preliminary requirements imposed on such an applicant by the Betting and
Gaming Act, 1960, Sch 1. Counsel contended that he had failed to cause to be published by means of an advertisement in a
newspaper circulating in the authoritys area a notice of the making of the application. Counsel asks us to interpret those words
as meaning either a local newspaper or a newspaper which did not circulate amongst a limited class of persons but amongst the
public generally. No doubt few people will expect that The Sporting Life is found on the breakfast table of most of the occupants
of the Peabody Trust premises, but, on the other hand, one main source of possible objectors are rival bookmakers, and, as my
Lord has said, it is difficult to imagine a more suitable medium for calling the attention of the rival bookmakers to the application
than its publication in The Sporting Life. On the other hand, no doubt it can be said that that is perhaps not the most suitable
medium for calling the attention of persons who either live or work in the Westminster area. I find it impossible to read into para
6 of Sch 1 the word local. Where Parliament has desired to prescribe publication in a local newspaper, it has done so, as was
pointed out, in Sch 12 to the Highways Act, 1959, where the words local newspaper circulating in the area are used. I agree
entirely with the interpretation which my Lord has put on the Act of 1960, and I find it impossible to say that it must be read in
the way in which counsel for the applicants is asking us to read it.
This was, in my view, an advertisement in a newspaper circulating in the authoritys area, and I agree, therefore, that these
applications must be dismissed.

EDMUND DAVIES J. In agreeing with my brethren, I must not be taken as expressing any view as to the suitability of The
Sporting Life for the purpose for which it was used by the respondent. Whether or not some other provision in the Betting and
Gaming Act, 1960, for advertisements might better safeguard the public weal than the expression which is to be found in para 6
of Sch 1 is neither here nor there. It is sufficient for me to say that I agree with my brethren that the advertisement inserted in
The Sporting Life in this particular case was one which, in my judgment also, complied with the requirements of para 6.

Applications dismissed.

Solicitors: Sherwood & Co (for the applicants); Campbell, Hooper & Co (for the respondent Best); A W Richmond (for the
respondents, the Westminster Betting Licensing Committee).

N P Metcalfe Esq Barrister.


546
[1963] 2 All ER 547

Car and Universal Finance Co Ltd v Caldwell


SALE OF GOODS

QUEENS BENCH DIVISION


LORD DENNING MR, SITTING AS AN ADDITIONAL JUDGE
5, 8 APRIL 1963

Sale of Goods Title Fraud Voidable title Election of seller to avoid contract for fraud Whether seller must communicate
election to purchaser if the latters title is to be avoided Acquisition of voidable title by second purchaser Hire-purchase
transaction Dealer with notice of defect in title to goods acting as general agent for finance company Whether finance
company obtained a good title to goods Sale of Goods Act, 1893 (56 & 57 Vict c 71), s 23.

Where a seller of goods has a right to avoid the contract for fraud, he sufficiently exercises his election if he at once, on
discovering the fraud, takes all possible steps to regain the goods, even though he cannot find the rogue or communicate with him
(see p 550, letter h, post).
Dictum of Lord Blackburn in Scarf v Jardine ((1862), 7 App Cas at p 361) not followed.
On the evening of 12 January 1960, C sold a motor car to N, and accepted a cheque in part payment of the price. When he
presented the cheque to the bank the next morning it was dishonoured, and he thereupon went to see the police. The police
informed him that there was a warrant out for the arrest of N. C asked them to recover the car and he also asked the Automobile
Association to trace it. The car was found on 20 January 1960, being driven by a director of M Ltd a firm of motor car dealers,
but they claimed to have bought it on 15 January from N and that it was their property. M Ltd had notice from which they could
infer that N had acquired the car fraudulently, having had a similar transaction with him the previous week. Immediately after
acquiring the car, M Ltd filled up hire-purchase forms by which the car was sold to a finance company, G & C Ltd. On 29
January 1960, C demanded the return of the car from M Ltd but they claimed to have a good title to it. Eventually, in August,
1960, the car was bought by another finance company, C & U Ltd who bought it in perfect good faith without notice of any
defect in title. In 1962 C obtained judgment against M Ltd for the return of the car or its value. The court found that G & C Ltd
entrusted M Ltd as dealers, with the conduct of all negotiations and placed themselves entirely in the hands of the dealers. On the
trial of an issue whether the car belonged to C & U Ltd or to C.

Held The title to the car was vested in C (the original owner) because
(i) C had avoided the contract of sale of the car to N on 13 January 1960, when he asked the police to get the car back, so
that the later purported sales of the car to M Ltd and to C & U Ltd (the second finance company) were ineffective to pass the
property in it to them (see p 550, letter i, post);
(ii) alternatively, as the circumstances of the case showed that M Ltd were the general agents of the first finance company, G
& C Ltd (see p 552, letters f and g, post), G & C Ltd were affected by the notice which M Ltd had of the defect of title to the car,
so that G & C Ltd did not get a good title; and, as the contract in any case was avoided on 29 January 1960, the second finance
company, C & U Ltd had no better title than M Ltd (see p 552, letter i, post).

Notes
As to the position where a seller of goods has a voidable title, see 34 Halsburys Laws (3rd Edn) 81, 82, para 126; and for cases
on the subject, see 39 Digest 532535, 14431460.
For the Sale of Goods Act, 1893, s 23, see 22 Halsburys Statutes (2nd Edn) 1001.
547

Cases referred to in judgment


Dixon v Winch [1900] 1 Ch 736, 69 LJCh 465, 82 LT 437, 21 Digest (Repl) 445, 1501.
Hop and Malt Exchange and Warehouse Co, Re, Ex p Briggs (1866), LR 1 Eq 483, 35 Beav 273, 35 LJCh 320, 14 LT 39, 55 ER
900, 9 Digest (Repl) 94, 423.
Jones v Gordon (1877), 2 App Cas 616, 47 LJBcy 1, 37 LG 477, 6 Digest (Repl) 131, 974.
Navulshaw v Brownrigg (1852), 2 De GM & G 441, 21 LJCh 908, 20 LTOS 25, 42 ER 943, 1 Digest (Repl) 393, 545.
Reese River Silver Mining Co Ltd v Smith (1869), LR 4 HL 64, 39 LJCh 849, 10 Digest (Repl) 959, 6597.
Scarf v Jardine (1862), 7 App Cas 345, 51 LJQB 612, 21 Digest (Repl) 299, 633.

Issue
On 15 February 1962, John David Balfour Caldwell obtained an order by default directing judgment to be entered against
Motobella Co Ltd for the return of a Jaguar motor car or its value of 975. Pursuant to a writ of delivery issued by Mr Caldwell,
the Sheriff of Hampshire in June, 1962, seized the car. Claim to possession of the car was made by Car and Universal Finance
Co Ltd. On a summons taken out by the sheriff, Master Harwood, on 26 July 1962, ordered that an issue be tried in the High
Court, in which Car and Universal Finance Co Ltd were to be the plaintiffs and Mr Caldwell the defendant, and that the sheriff
sell the car through the agency of the plaintiffs and pay the net proceeds therefrom into court. The plaintiffs claimed a declaration
that the title in the car was vested in them at the inception of the issue and the defendant claimed that the title to the car and the
right to its immediate possession vested in him. The facts are set out in the judgment.
The authority and cases noted belowa were cited during the argument in addition to these referred to in the judgment.
________________________________________
a 1 Halsburys Laws (3rd Edn) 225, para 535, Sein v Ryan (1862), 15 Moo PCC 230, Clough v London and North Western Ry Co (1871), LR
7 Exch 26, Whitehorn Bros v Davison [190810] All ER Rep 885, [1911] 1 KB 463, Wells v Smith [1914] 3 KB 722, Financings Ltd v
Stimson [1962] 3 All ER 386

Norman C Tapp for the plaintiffs.


A L J Lincoln for the defendant.

8 April 1963. The following judgment was delivered.

LORD DENNING MR. This case raises the familiar question, which of two innocent persons is to suffer for the fraud of a
third? Mr Caldwell, the defendant, lived at Bosham and was the owner of a Jaguar car. On Saturday, 9 January 1960, he
advertised it in a local paper for sale. On the next day, Sunday, 10 January he was visited by two men who wished, as they said,
to buy it for a price of 975, and they paid down 10 in cash as a deposit. The defendant was unwilling to let the car go out of his
possession except for cash. However, on the Tuesday evening, 12 January he was induced to let them take this Jaguar car away.
They left with him a Hillman car of much less value and also a cheque for 965 drawn on the Midland Bank Ltd 114, London
Road, Brighton, and signed For and on behalf of Dunns Transport. W Foster F Norris. Those were the names they gave
Foster and Norris. The very next morning, 13 January by the time when the bank opened at Brighton at ten oclock in the
morning, the defendant was there to present the cheque. It was not met. The manager told him that it could not be met from that
account, and went on to tell him that they had had a previous case of the same thing, and he advised the defendant to go to the
Shoreham police. The defendant took a taxi at once and went there. The police produced a photograph of a wanted man, who
was the very man who had induced the defendant to part with his car. The police immediately put out a wireless call to all the
police patrols trying to find this 548 car and track it down. They told the defendant that there was a warrant out for the arrest of
the man Norris in the name of Rowley. They had been watching his house for several days and had not found him. Having thus
seen the police, the defendant in the afternoon telephoned the office of the Automobile Association and they sent out a call to all
their patrols, trying to find the car. It was eventually found. On 20 January 1960, that is, just a week later, at Bosham it was
being driven by a Mr Brian Charles Allen, a director of a firm of motor car dealers called Motobella Co Ltd but he claimed that
that company had bought it and that it was their property. On 29 January 1960, the defendants solicitors demanded of Motobella
Co Ltds solicitors the return of the car but Motobella Co Ltd claimed to have a good title. The man who obtained the car
(Rowley alias Norris) from the defendant was arrested and pleaded guilty to obtaining cars (including this one) by false
pretences. The defendant sued Motobella Co Ltd for the return of the car. They put up the defence in their pleadings that they
had bought it in good faith and that it was theirs. The action came on for trial in 1962. It was not defended by Motobella Co Ltd
and the defendant got judgment against them. When the defendant sought to take the car under the judgment, a company called
the Car and Universal Finance Co Ltd claimed that it belonged to them. The car seems to have been transferred by Motobella Co
Ltd on 15 January 1960, to a finance house called G & C Finance Corporation Ltd and in due course, on 13 August 1960, it was
transferred to another finance company, Car and Universal Finance Co Ltd the plaintiffs. So an issue has been directed to be
tried: does this car belong to the defendant, the original owner, or does it belong to the plaintiffs, who have purchased it in good
faith themselves and without notice of anything wrong with it? That is the issue which I have to try.
Many of the facts here are agreed. The car was obtained from the defendant by fraud by means of a worthless cheque. It
was not a case of larceny by a trick. It was a case of obtaining goods by false pretences. The property in the car passed to Norris
and Foster on 12 January 1960, when they took it away. But the transaction was voidable. It was voidable for the fraud. So it
was open to the defendant to avoid it provided that he did so before a third person bought it in good faith and without notice of a
defect in the title. The principal question is whether the defendant did succeed in avoiding the sale by the steps which he took of
going to the bank, the police and the A A, before the rogue sold it to Motobella Co Ltd. It is said by counsel for the plaintiffs that
a man from whom goods have been obtained by false pretences cannot avoid the transaction unless he does an act which
unequivocally shows his election to avoid it; and, furthermore, communicates his election to the other side, that is, to the other
party to the contract. The avoidance does not, it is said, take place until it is communicated. In this case, therefore, the avoidance
did not take place on the morning of 13 January when the defendant went to the police. It would not take place until the
defendant discovered Mr Norris and Mr Foster and communicated his election to them. Counsel for the plaintiffs conceded that it
was avoided by 29 January 1960, but by that time, he said, Motobella Co Ltd had sold it to G & C Finance Corporation Ltd who
had acquired a good title.
In support of this supposed requirement of communication counsel for the plaintiffs referred me particularly to the judgment
of Lord Blackburn in Scarf v Jardine, where Lord Blackburn says that a man does not elect to avoid a contract simply because in
his own mind he decides to avoid it or even makes a note in his own book about it. An election, he indicated ((1862), 7 App Cas
at p 361), is not completed until he

has communicated it to the other side in such a way as to lead the opposite party to believe that he has made that
choice. [Then] he has completed his election and can go no further.
549

Counsel for the plaintiffs also referred me to a passage in Benjamin on Sale, which appears in the second edition in 1873 by Mr
Benjamin, and it is also in the present edition, the eighth edition, at p 441. It says:

The rescission is the legal consequence of his election to reject it, and takes date from the time at which he announces
this election to the opposite party.

Those passages do seem to support counsel for the plaintiffs contention that the defendants acts on the morning of 13 January
did not amount to an election to avoid the contract, because they were not communicated to the opposite party. I quite appreciate
the force of that argument, but I am afraid that I cannot accept it. In point of principle, it seems to me that a seller can avoid a
contract by an unequivocal act of election which demonstrates clearly that he elects to rescind it and no longer to be bound by it.
It is sufficient if he asserts his intention to rescind in the plainest and most open manner competent to him. I take those words
from the speech of Lord Hatherley LC in Reese River Silver Mining Co Ltd v Smith ((1869), LR 4 HL 64 at p 74). Lord
Hatherley goes on to give communication as one of the ways in which an election can be demonstrated: but not, I think, in such a
way as to make it essential, so as to make it a sine qua non. I would ask this simple question: How is a man in the position of
defendant ever to be able to rescind the contract when a fraudulent person absconds as Norris did here? If his right to rescind is
to be a real right, when the rogue absconds, it must be sufficient if he does all that he can in the circumstances unequivocally to
make it known. It is not sufficient for him, of course, to keep it in his own mind or write down a note in his own private sitting
room; but conduct such as we have here, namely, telling the bank, the police and the A A, Find this car if you possibly can. Get
it back. It is mine, seems to me an unequivocal act of rescission.
If you take comparable cases of election, none of them requires communication as an essential pre-requisite. Take
forfeiture, which Lord Blackburn took as his guide. If a lessor elects to determine a lease for forfeiture, it is sufficient for him to
issue a writ for possession. The forfeiture dates from the issue of the writ, not from the time it is served. Or if the lessor does an
act which, if done by a stranger, would be a trespass, it is an entry. The forfeiture dates from the time of the re-entry, even though
the lessee is away and does not know anything about it. Next take ratification. Where a person has a right to ratify an
unauthorised act made professedly on his behalf by an agent, any unequivocal act of affirmation is sufficient. Again, take
repudiation. If a contract is repudiated by a party and the other has a right to elect whether to accept it or not, any unequivocal
act clearly evincing his election is sufficient. Take finally the affirmation of a contract. Counsel for the defendant referred me to
Re Hop and Malt Exchange and Warehouse Co Ex parte Briggs, where it was held that an affirmation by instructing a broker to
re-sell (which was an unequivocal act) was held to be sufficient affirmation even though not communicated.
I hold, therefore, that, where a seller of goods has a right to avoid the contract for fraud, he sufficiently exercises his election
if he at once, on discovering the fraud, takes all possible steps to regain the goods, even though he cannot find the rogue or
communicate with him. That is what the defendant did here by going to the police and asking them to get the car back. I,
therefore, hold that, on 13 January the contract of sale to these rogues was avoided and the defendant then became the owner of
the car again. It was only after he avoided it (so that it was once again his property) that these rogues purported to sell it to
Motobella Co Ltd and then Motobella Co Ltd purported to sell it to G & C Finance Corporation, Ltd. Those sales were
ineffective to pass the property because it had already re-vested in the defendant.
550
But in case I am wrong on that point, I must go on to consider the remaining facts. After this rogue Norris got the car, he
took it to the dealers called Motobella Co Ltd and purported to sell it to them. Motobella Co Ltd had notice that he had acquired
it fraudulently. Only a week before they had had a similar transaction with him. They probably did not have details of the actual
fraud, but they had notice from which they could infer that he had not come by it honestly. They had no better title than Norris.
Now Motobella Co Ltd were agents or dealers for a finance company, G & C Finance Corporation, Ltd. Immediately after
acquiring the Jaguar, Motobella Co Ltd filled in one of the hire-purchase forms which had been issued to them by G & C Finance
Corporation, Ltd. They proposed a hirer named Alfred Henry Knowles and invited G & C Finance Corporation Ltd to buy the
car from them and to let again on hire-purchase to Knowles. G & C Finance Corporation Ltd accepted that proposal at its face
value and paid Motobella Co Ltd 950 on the faith of it.
There is a question whether Mr Knowles ever existed. An address was put in the form giving his address at 30, Palace
Green, Old Addington, Surrey. Inquiries have since been made for Mr Knowles there and nothing is known of him. He has
never been at that address. 30, Palace Green, is occupied by a very respectable gentleman who had nothing to do with this
transaction whatsoever. The occupation of Knowles was put in the form as a greengrocer. He was said to be self-employed,
and his business address was given as Surrey Street, Croydon. That is Croydon market. There is no shop there occupied by
Knowles. If there was ever such a person at all, he can only have kept a stall there. When demands were made from Knowles for
payment, Knowles paid nothing. Knowles is a very dubious character, if he ever existed. But there it is. G & C Finance
Corporation Ltd on 15 January 1960, accepted the document at its face value and paid over the money.
A few weeks later, in March, 1960, Motobella Co Ltd put forward to G & C Finance Corporation Ltd another transaction for
the self-same Jaguar car. It wore the appearance of their taking the car back from Knowles and putting forward another hire-
purchase agreement, this time with a Miss P Stockwin. G & C Finance Corporation Ltd accepted that proposal too. According to
the documents Miss Stockwin (if she existed) also fell into arrear: and G & C Finance Corporation Ltd authorised Motobella Co
Ltd to re-possess the vehicle. Motobella Co Ltd then, on behalf of G & C Finance Corporation Ltd seem to have sold it to a
dealer in Brighton named David McGhie, trading as Brunswick Car Sales. This dealer, on 10 August 1960, put forward a hire-
purchase proposal to the plaintiffs; and, on 13 August 1960, the plaintiffs bought it in perfect good faith without notice of any
defect in title. They paid Mr McGhie 595 in cash and let it out on hire-purchase. They are the claimants here.
In case I am wrong on the first point, the question is whether G & C Finance Corporation Ltd (when they bought this car
from Motobella Co Ltd on 15 January 1960) bought it in good faith and without notice of the sellers defect of title. Section 23
of the Sale of Goods Act, 1893, says:

When the seller of goods has a voidable title thereto, but his title has not been avoided at the time of the sale, the buyer
acquires a good title to the goods, provided he buys them in good faith and without notice of the sellers defect of title.

I desire to say that I am not prepared to attribute to G & C Finance Corporation Ltd any actual knowledge of anything wrong. It
has been often said that blundering or negligence does not amount to bad faith. It does not itself mean that there was notice of
any defect of title. There are several cases on that point starting with Navulshaw v Brownrigg, and including, of course, the well-
known passage in Jones v Gordon in Lord Blackburns judgment ((1877), 2 App Cas 616 at p 629). It would be wrong in this
case to say that it had been proved that G & C Finance Corporation, 551Ltd had themselves notice of any defect of title or
themselves did not buy in good faith. But what is said is that they must be fastened with the knowledge of Motobella Co Ltd.
Motobella Co Ltd undoubtedly knew a great deal about this matter. They certainly knew that there was something wrong with
the title to this car. They had had a previous transaction with the rogue which put them on inquiry, or should have done. But are
G & C Finance Corporation Ltd to be fixed, so to speak, with the knowledge of Motobella Co Ltd?. So that they did not get a
good title.
I am quite clearly of opinion that the dealers, Motobella Co Ltd were in many respects the agents of the finance house G &
C Finance Corporation, Ltd. G & C Finance Corporation Ltd supplied Motobella Co Ltd with the hire-purchase forms. G & C
Finance Corporation Ltd thenceforward left everything to them. They left them to inquire into the title of the car which they were
going to let on hire. If there was any question of seeing the log book, it was for them to see to it. If there was any question as to
the existence or stability of the hirer, Knowles, they left it to them. All negotiations with Knowles, everything to do with the title,
was left to the dealers. The representatives of G & C Finance Corporation Ltd Mr Morley and Mr Rice, were quite frank on the
matter. They simply dealt with the hire-purchase documents as they came in; and, if they were apparently in good order, they
accepted them. They referred, it is true, to the hire-purchase register to see if there was any previous hire-purchase dealings with
the car; and they looked up their own records to see if they had had any transaction with the hirer. But otherwise they left
everything to the dealers.
In my judgment, in all matters between G & C Finance Corporation Ltd and the supposed hirer, Knowles, the dealers were
the agents of the finance company. But were they the agents of the finance company to investigate the title to this car? That is
the question. Were they the agents of the finance company to see that the title was in order? This is a very difficult question. But
the circumstances of this case show to my mind that the finance company entrusted the dealers with the conduct of all the
negotiations and placed themselves entirely in the hands of the dealers. I am not prepared to limit the agency simply to the
dealings with the hirer. It seems to me here that the dealers were more in the nature of general agents under a general agency
such as was spoken of in the case to which counsel for the plaintiffs very properly and helpfully drew my attention, namely,
Dixon v Winch. When these dealers put forward a document to the finance company, knowing that the finance company were
going to let the car out on hire-purchase to the supposed man Knowles, the dealers were the agents of the finance company to see
that the title was in order. There was a general agency which covered the investigation of the title to this car. It follows that the
knowledge of Motobella Co Ltd the dealers, of the defect of title is fastened on G & C Finance Corporation Ltd and they did not
acquire a good title to the car. They had no better title than Motobella Co Ltd.
So I put my judgment on two grounds. First, that the defendant did exercise his election to rescind on the morning of 13
January 1960, before there was any resale by Norris; thereupon the property was re-vested in the defendant and it has never gone
from him since. But if that were wrong, if there were still a voidable title to it in Norris after 13 January (and it was not avoided
until 29 January 1960), nevertheless, I hold that G & C Finance Corporation Ltd when they bought on 15 January 1960, did not
acquire a good title to the goods because they are affected by the notice which their agents, the dealers, had of the defect of title
and so they themselves did not get a good title. It is admitted that, in any case, the transaction was avoided by 29 January 1960,
long before the plaintiffs bought the car, so the plaintiffs have no better title than G & C Finance Corporation, Ltd. This makes it
unnecessary for me to go into the other points which were raised. 552I hold that the defendant is entitled to this car and I decide
the issue accordingly. I believe that it has been agreed that the proceeds are to take the place of the car on the sale.

Order accordingly.

Solicitors: Saunders, Sobell, Leigh & Dobin (for the plaintiffs); Amery-Parkes & Co (for the defendants).

F Guttman Esq Barrister.


[1963] 2 All ER 553

United Marketing Company v Hasham Kara


COMMONWEALTH; Commonwealth countries: CIVIL PROCEDURE

PRIVY COUNCIL
LORD EVERSHED, LORD HODSON AND DONOVAN LJ
28, 29 JANUARY, 8 APRIL 1963

Privy Council Practice Fresh point Fact and law Evidence must be such as to establish that facts would support new plea
Fresh point of law not readily allowed even if facts beyond dispute.

The practice of the Judicial Committee is to refuse to allow a point not taken in a lower court to be argued unless satisfied that the
evidence on which they are asked to decide establishes beyond doubt that the facts, if fully investigated, would have supported
the new plea; and, even if the facts were beyond dispute and no further investigation of fact were required, a fresh point of law
will not readily be allowed to be argued without the benefit of the judgments of the judges in the court below; accordingly, where
a fresh point of law is raised on which no direct authority in this country is available, and on which there are conflicting decisions
in other countries, the points will not be entertained (see p 554, letter i, to p 555, letter a, and p 555, letter c, post).
Connecticut Fire Insurance Co v Kavanagh ([1892] AC 473, 480) followed and dictum of Lord Birkenhead LC in North
Staffordshire Ry Co v Edge ([1920] AC 254 at pp 263, 264) applied.

Notes
As to practice of Judicial Committee of the Privy Council to entertain arguments on points of law not raised in the court below,
see 9 Halsburys Laws (3rd Edn) 392, 393, para 920; and for cases on the subject, see 16 Digest (Repl) 174178, 588627.

Cases referred to in judgment


Archambault v Archambault [1902] AC 575, 71 LJPC 131, 87 LT 404, PC, 16 Digest (Repl) 174, 597.
Connecticut Fire Insurance Co v Kavanagh [1892] AC 473, 61 LJPC 50, 67 LT 508, 8 TLR 752, PC, 16 Digest (Repl) 176, 607.
Lewis Ltd v Norwich Union Fire Insce Co Ltd [1916] SALR App D 509, 29 Digest (Repl) 476, 1793.
North Staffordshire Ry Co v Edge [1920] AC 254, 89 LJKB 78, 122 LT 385, 8 Digest (Repl) 195, 1253.
Sacks v Western Assurance Co (1907), 5 Transvaal LR (Witwatersrand High Court) 257.

Appeal
This was an appeal from an order of the Court of Appeal for Eastern Africa (OConnor P, Gould, Ag V P and Crawshaw JA) of 15
September 1960, which dismissed an appeal by the appellants from a decree of the Supreme Court of Kenya (Templeton J), dated
27 April 1959, by which the appellants were ordered to pay damages to the respondent for breach of duty and for negligence in
failing, as agents, to renew a policy of insurance on behalf of the respondent in respect of a shop and contents belonging to the
respondent which were destroyed by fire.
The respondent was lessee of a shop, 2646, Bazaar Road, Nairobi, which was destroyed by fire together with the bulk of its
contents on the night of 9 April to 55310 April 1956, at a time when no policy of insurance was in force covering the stock-in-
trade and furniture contained in the building. The appellants were a partnership carrying on business in Nairobi as insurance
agents, and were the chief agents of Jubilee Insurance Co Ltd (herein called the insurance company). The contents of the
respondents shop had been insured with the insurance company under a policy dated 17 November 1956, for 50,000/-,
apportioned as to 40,000/- to stock-in-trade and 10,000/- to furniture. The respondents case was that the appellants were under
contractual obligation to procure the renewal of the policy and that, since the policy was not in force at the time of the loss, he
was unable to claim under the policy and had suffered damage. At the trial there was a conflict of evidence concerning an alleged
arrangement in 1950, between the respondent and a Mr Thanawalla on behalf of the appellants, that the appellants would renew
the respondents insurance without further instructions from the respondent, debiting his account with the premiums. The trial
judge decided the conflict of evidence in favour of the respondent and the Court of Appeal of Eastern Africa affirmed his
decision. Thus there were concurrent findings of fact. On this appeal the appellants sought that these concurrent findings of fact
should be reversed and contended that the agreement relied on by the respondent was not proved.
Before the Judicial Committee the appellants sought also to raise, as a ground of appeal that had not been relied on in either
court below, that, even if the agreement were proved, damages could not be more than nominal since the respondent was in
breach of a condition of the policy of insurance in the following terms:

7. Warranted that the insured keeps and during the currency of the policy shall keep a complete set of books of
account and stock sheets or stock books, showing a true and accurate record of all business transactions and stock in hand,
and such books shall be locked in a fireproof safe, or removed to another building at night, and all times when the premises
are not actually open for business.
This formed the ground of appeal referred to in His Lordships opinion as the second ground of appeal, on which ground only the
appeal is reported.

F P Neill for the appellants.


J G Le Quesne QC for the respondents.

8 April 1963. The following judgment was delivered.

LORD HODSON stated the nature of the appeal, explained how the appeal arose and the issues raised by the respondents case
at the trial, and reviewed the evidence and the conflict of evidence. Having intimated that there were concurrent findings of fact
in favour of the respondent by the trial judge and the Court of Appeal of Eastern Africa, and having stated their lordships opinion
that the appellants failed to show that the concurrent findings of fact ought to be reversed, His Lordship continued: The second
ground of appeal, depending on the allegation that the respondent was in breach of a condition of his policy, is said to be
established by his own evidence that he kept stock books, that the stock books relating to the goods destroyed by fire were left on
the shelf in the shop which caught firethat the books were burned and that usually they took their books to their homes at night
and put them in the safe, but on this night they were left in the shop.
Their lordships are of opinion that the appellants should not be allowed to take this point at this stage. In the first place the
point could have been met by evidence that if the claim had been made against the insurance company under a subsisting policy
the insurance company would not have relied on the breach of the condition, or possibly by some other evidence. Their lordships
would not depart from their practice of refusing to allow a point not taken before to be argued unless satisfied that the evidence
on which they are asked to decide establishes beyond doubt that the facts, if fully investigated, would have 554 supported the
new plea (Connecticut Fire Insurance Company v Kavanagh, and Archambault v Archambault).
Even if the facts were beyond dispute and no further investigation of fact were required, their lordships would not readily
allow a fresh point of law to be argued without the benefit of the judgments of the judges in the court below. In this case the
appellants have relied in support of their submission that there was a breach of condition on two South African cases, Lewis, Ltd
v Norwich Union Fire Insurance Co Ltd and Sacks v Western Assurance Co, which on similar facts support their submission, but
their lordships are not prepared to say that the point is too plain for argument to be required on it. The arguments and judgments
in these two cases indicate that at any rate in the United States of America there are conflicting decisions on this topic, and no
direct authority in this country was available so far as the researches of the appellants were able to show. Accordingly their
lordships would not, even if the question were a bare question of law, entertain the submission that the respondents claim is to be
defeated by reason of his breach of a condition of his contract of insurance with the insurance company and they would follow
the guidance given by Lord Birkenhead LC in North Staffordshire Ry Co v Edge, when he said ([1920] AC at pp 263, 264):

The efficiency and the authority of a Court of Appeal, and especially of a final Court of Appeal, are increased and
strengthened by the opinions of learned judges who have considered these matters below. To acquiesce in such an attempt
as the appellants have made in this case is in effect to undertake decisions which may be of the highest importance without
having received any assistance at all from the judges in the courts below.

The Lord Chancellor went on to say that there might be very exceptional cases where new matters might be considered ([1920]
AC at p 264), but their lordships do not regard this case as requiring such exceptional treatment.
Accordingly their lordships will humbly advise Her Majesty that this appeal be dismissed, subject to a variation in the
amount of damages which by an oversight have been wrongly assessed.

Appeal dismissed.

Solicitors: Goodman, Derrick & Co (for the appellants); T L Wilson & Co (for the respondent).

C G Leonard Esq Barrister.


555
[1963] 2 All ER 556

Re A I Levy (Holdings) Ltd


COMPANY; Insolvency

CHANCERY DIVISION
BUCKLEY J
13 MAY 1963

Company Winding-up Compulsory winding-up Disposition of property Authority to company to sell leasehold interest in
premises Winding-up order not yet made on pending petition Whether jurisdiction to authorise disposition Companies Act,
1948 (11 & 12 Geo 6 c 38), s 227.

A company held a lease of premises by the terms of which the landlord was enabled to forfeit the lease if the company were
compulsorily wound-up. On 28 November 1962, the landlord instituted proceedings against the company for possession of the
premises, arrears of rent and mesne profits, and obtained judgment on 9 January 1963. In the meantime the company had
received an offer to purchase the lease. The landlord did not enforce the judgment, in order to allow the company to sell its
leasehold interest. A petition to wind-up the company was presented by a creditor on 4 March 1963, and, no winding-up having
yet been made, the company applied to the court for an order, pursuant to s 227 a of the Companies Act, 1948, authorising the
company to sell the lease. The circumstances were such that the proposed sale would be for the benefit of the creditors of the
company.
________________________________________
a The relevant provisions of s 227 of the Companies Act, 1948, are set out at p 557, letter a, post

Held There was jurisdiction under s 227 of the Companies Act, 1948, to authorise the sale notwithstanding that a winding-up
order had not yet been made; and in the circumstances the company would be granted liberty to sell the leasehold interest in the
premises, such sale not to be avoided by s 227 if a winding-up order were made in relation to the company (see p 560, letter g,
post).
Carden v Albert Palace Assocn ((1886), 56 LJCh 166) and dictum of Pennycuick J in Re Douglas (Griggs) Engineering Co
Ltd ([1962] 1 All ER at p 501) applied.
Re Miles Aircraft Ltd ([1948] 1 All ER 225) not followed.

Notes
As to retrospective effect of winding-up order on dispositions made after the commencement of a winding-up, see 3 Halsburys
Laws (3rd Edn) 628, para 1239; and for cases on the subject, see 10 Digest (Repl) 904906, 61426152.
For the Companies Act, 1948, s 227, see 3 Halsburys Statutes (2nd Edn) 646.

Cases referred to in judgment


Carden v Albert Palace Assocn (1886), 56 LJCh 166, sub nom Re Albert Palace Assocn Ltd Carden v Albert Palace Assocn Ltd
55 LT 831, 10 Digest (Repl) 905, 6149.
Douglas (Griggs) Engineering Ltd Re [1962] 1 All ER 498, [1963] Ch 19.
Howard Sunderland Ltd, Re (1948), unreported.
International Life Assurance Society, Re, Gibbs & Wests Case (1870), LR 10 Eq 312, 59 LJCh 667, 18 WR 970, 10 Digest (Repl)
1148, 7993.
Miles Aircraft Ltd, Re, Barclays Bank, Ltds Application [1948] 1 All ER 225, [1948] Ch 188, [1948] LJR 1133, 10 Digest (Repl)
906, 6158.
Wiltshire Iron Co, Re, Ex p Pearson (1868), 3 Ch App 443, 37 LJCh 554, 10 Digest (Repl) 904, 6142.

Summons
This was an application by summons dated 23 April 1963, by A I Levy (Holdings) Ltd for leave to sell the leasehold premises
known as Nos 40 and 41, Market Place, Kingston-on-Thames, Surrey, comprised in and demised by a lease dated 30 June 1960,
pursuant to s 227 of the Companies Act, 1948. The facts appear in the judgment.

Muir Hunter for the company.


Ian Edwards-Jones for the petitioning creditor.
556

13 May 1963. The following judgment was delivered.

BUCKLEY J. This is an application by A I Levy (Holdings) Ltd a company which is sought to be wound up, for leave to sell
certain leasehold premises and the application comes before the court under s 227 of the Companies Act, 1948, which provides
that:

In a winding-up by the court any disposition of the property of the company made after the commencement of the
winding-up, shall, unless the court otherwise orders, be void.

The question has been raised whether it is open to me to make an order validating the disposition by the company of these
leasehold premises at the present stage, the winding-up proceedings being pending but no winding-up order having yet been
made. The company owns a number of leasehold shops and I am concerned with one of those, 40 and 41, Market Place,
Kingston-on-Thames, which is held under a lease by the terms of which the landlords will be in a position to forfeit the lease in
the event of the company being put into compulsory winding-up.
On 28 November 1962, the landlords instituted proceedings against the company for possession of the premises, for a
certain sum as arrears of rent, and for mesne profits. On 9 January 1963, they obtained judgment in that action. In the meantime
the company had obtained an offer from a prospective purchaser for the lease at the price of 3,000. In those circumstances, the
landlords agreed to hold their hands under their judgment with a view to the company continuing in negotiation for the sale of the
lease and making application to the landlords for a licence to assign. In the events which have happened, the company has come
to terms with the proposing purchaser for the sale of the leasehold property to him at the price of 3,000. But, of course, the
claim of the landlords for their arrears of rent and their costs in that action have to be met. There is, moreover, a charge on the
leasehold interest securing the sum of 360 due to a body corporate which made a loan to the company. The solicitors who acted
for the company in the negotiations for the sale of the lease also claim to have a lien of some kind in respect of costs which have
to be met. There is evidence before me which says that 3,000 is the proper price for this leasehold, and it is clear that if this sale
does not go through, and the company is put into liquidation, the asset will become valueless. Consequently, the transaction is
one which, it seems to me, must on any possible view be beneficial to the creditors of the company in that the proceeds of sale
will discharge the claim of the landlords and a net balance will be receivable by the company out of the proceeds of sale after the
claims of the secured creditors and the landlords, and possibly the solicitors in respect of their costs, have been satisfied, so that
there will also be a benefit to the other unsecured creditors of the company. Accordingly, the transaction appears to me to be a
beneficial one, which, if I have jurisdiction to facilitate the bargain by making an order under s 227, I ought to assist. The
landlords are not content to deal with the matter in this way unless an order is made now (that is to say, before the petition comes
before the court for hearing and before any winding-up order can be made) validating the payment to them of the sum which they
will receive in respect of the arrears of rent and costs due to them.
Section 227 of the Companies Act, 1948, has in one form or another formed part of the companies winding-up legislation
since it appeared as s 153 of the Companies Act, 1862. I have been referred to various authorities in which the jurisdiction has
been either exercised or commented on, the earliest of which is Re International Life Assurance Society, Gibbs & Wests Case,
where Sir R Malins V-C cited ((1870), LR 10 Eq at p 323) a passage from Lord Cairns judgment in Re Wiltshire Iron Co Ex p
Pearson, which I need not read. That passage indicated a broad view of the nature of the jurisdiction conferred by the section and
indicated 557 that it was designed to preserve the value of the assets of a company for the benefit of the people interested in the
assets notwithstanding the pendency of winding-up proceedings, in order that the company might not be unduly hampered in
carrying out transactions which might be for the benefit of those interested in the value of its assets. Sir R Malins V-C, in the
course of his judgment, said this ((1870), LR 10 Eq at p 324):

but when the matter is brought before the court, it must have regard to all the surrounding circumstances, and if
from all the surrounding circumstances it comes to the conclusion that the transaction should not be void, it is within the
power of the court, under the 153rd section, to say that the transaction is not void.

That is a reference to the section in the Act of 1862 corresponding to s 227 of the Act of 1948. It is a statement of jurisdiction in
broad and general terms, which does not suggest that it is confined to making an order after a winding-up order has been made.
In Re International Life Assurance Society however the winding-up order had already been made, and, therefore, the case does
not give me any authoritative assistance in deciding the point which has been raised before me.
The next case that I should mention is a decision of Chitty J in Carden v Albert Palace Assocn, in which that learned judge,
having been satisfied that the transaction there in question was for the benefit of all possible parties, did make the order under s
153 of the Act of 1862 before any winding-up order had been made in respect of the company. It is right to say that in that case
the learned judge is not shown by the report to have heard any argument about the extent of the jurisdiction, although he seemed
to have had some doubts about it in his mind because he started his judgment as reported with these words ((1886), 56 LJCh at p
167):

The difficulty in making an order before a winding-up is that it is impossible to know whether all the parties interested
are before the court. As the transaction is one which I am informed must be for the benefit of all possible parties, I will
make an order both in the action and in the winding-up.

The reference to the action is irrelevant for present purposes.


However, in the year 1948, a case came before Vaisey J in which the question of jurisdiction was raised. That was Re Miles
Aircraft Ltd Barclays Bank, Ltds Application in which the applicants, who held an equitable charge on the assets of the company,
applied to the court under s 173 of the Companies Act, 1929, for an order that a contract entered into by the company to sell
certain premises should not be void. The contract in question was made on the day after the presentation of the petition to wind-
up the company. Owing to successive adjournments that petition had not yet been effectively heard. The petitioning creditors
opposed the making of the order. Vaisey J after saying that the question raised before him was whether he had jurisdiction in
those circumstances to make the order, read s 173 of the Act of 1929 and s 153 of the Companies Act, 1862. Then he said this
([1948] 1 All ER at p 226; [1948] Ch at p 190):

In my judgment, the object of the section, both in its original and in its present form, is that, if a winding-up order is
made, any transaction which has been entered into since the commencement of the winding-upwhich, in the present case
would be the date of the presentation of the petitionis subject to review by the liquidator. If that be the true object of the
section, I have great difficulty in seeing how I have jurisdiction to adjudicate on what must be an incomplete knowledge of
the facts. It is not right that I 558 should anticipate the performance of his duty by the liquidator, if a liquidator is
subsequently appointed, and it seems to me that to attempt to deal with the matter, not only by implication, but also entirely
conditionally, is something which the section does not enable me to do. If there is never a winding-up, my order would be
without operation. It would be completely otiose if the petition before me is ultimately dismissed or withdrawn, but, if the
petition results in the making of a winding-up order, it will be I think for the liquidator to deal with the matter as he thinks
proper.

Then his Lordship referred to Carden v Albert Palace Assocn and of that case, he said ([1948] 1 All ER at p 226; [1948] Ch at p
191):

That case, so far as it has attracted legal attention, is cited in only a casual way in STIEBELS COMPANY LAW AND
PRECEDENTS (3rd Edn.), p. 830, and is not referred to in BUCKLEY or PALMER. Indeed it has never been regarded as
an authority for anything. On general principles, I feel bound to say that, unless there is in progress a winding-up by the
court I am powerless to make any order under this section and I must leave the matter to the liquidator after his
appointment. The difficulty in saying that there is a winding-up now in progress seems to me to be that, if the petition is
ultimately dismissed or withdrawn, there never will have been a winding-up by the court. All that we have now is a
contingent future possible winding-up, and I do not think the section is so framed as to give me jurisdiction to adjudicate
on this matter, so to speak, in advance. I must, therefore, dismiss this summons with costs.

I am told that in a later case in 1948, Re Howard Sunderland Ltd which is unreported, Roxburgh J on an application which
was made after the date of the winding-up order, refused to validate a transaction under s 227, or its predecessor, on the ground
that the applicant, having regard to the nature of the transaction and to the circumstances which existed when he entered into the
transaction with the company, ought to have applied to the court under the section at the date of the transaction, that is to say,
before the winding-up order had been made. Whether Roxburgh J was referred to Re Miles Aircraft, Ltd I do not know, but it is
evident that he took a different view of the nature of the jurisdiction from that expressed by Vaisey J in that case.
I also find that in Re Douglas (Griggs) Engineering Ltd, it is recorded, at the end of the judgment ([1962] 1 All ER at p 501;
[1963] Ch at p 24), that Pennycuick J made an order under s 227 notwithstanding that at the date of that order no winding-up
order had yet been made. There is no indication in the report that the learned judge was referred to the decision of Vaisey J in Re
Miles Aircraft Ltd. But, in the light of the decision of Chitty J in Carden v Albert Palace Assocn and to the order of Pennycuick J
in Re Douglas (Griggs) Engineering Ltd, and of what I have been told of the view of Roxburgh J expressed in an unreported case,
I am bound to look at the decision of Vaisey J in Re Miles Aircraft Ltd with considerable care to see whether it is in fact an
authority which I ought to feel myself bound to follow to the conclusion that I have no jurisdiction to make the order asked for in
the present case.
In Re Miles Aircraft Ltd the circumstances were not such that it was possible to contend that the transaction in question was
obviously for the benefit of all the creditors. Indeed, the petitioning creditor opposed the making of the order.
For myself, with the utmost respect to the learned judge, I find it difficult 559 to accept his statement ([1948] 1 All ER at p
226; [1948] Ch at p 190) that the object of the section is to ensure that any transaction entered into after the commencement of
the winding-up shall be subject to review by the liquidator. It appears to me that the object of the section b is to protect the
interests of the creditors from the possibly unfortunate results which would ensue from the presentation of a petition, and to
protect their interests as much during the period when the petition is pending as after an order has been made on it. What the
section provides in its present terms is that any disposition of the property of the company made after the commencement of the
winding up shall be void in the winding-up of the company unless the court otherwise orders; that is to say, if and when the
company comes to be put into liquidation the transaction is to be as if it had never taken place. It does not appear to me, with the
utmost respect to Vaisey J that the language of s 227 of the Companies Act, 1948, necessarily requires that an order under it
should be made in respect only of a company which is being wound up by the court at the date when the order under s 227 is
made, that is to say, that an order under s 227 should be made only after the date of the winding-up order. If that were the true
effect of the section, the present case would demonstrate that the section is ill-designed to meet a kind of risk to the creditors of a
company against which one would have expected it to be intended to protect them.
________________________________________
b Companies Act, 1948, s 227

Although it is true that in Carden v Albert Palace Assocn and in Re Douglas (Griggs) Engineering Ltd, the extent of the
jurisdiction does not seem to have been the subject of argument, the learned judges in those cases seem to have felt themselves
able to exercise the jurisdiction although no winding-up order had been made, and, as I have pointed out, the matter was one to
which Chitty J at any rate, seems to have given at least a passing thought.
In these circumstances, this being a case in which it appears to me to be manifest that the transaction is one which must
benefit the unsecured creditors of the company if in due course a winding-up order is made, the reason which affected the mind
of Vaisey J that is to say that the liquidator should be given an opportunity to investigate the matter and bring it before the court
representing the interests of all the creditors, does not affect my mind, for I do not think that the liquidator could make the
position clearer to me than it is at the present time on the facts. With the greatest respect to Vaisey J from whose view as to
jurisdiction I think I am, in the circumstances, at liberty to differ, I hold that I have jurisdiction and that on the facts of this case I
should exercise my jurisdiction here and now, notwithstanding that no winding-up order has yet been made. Accordingly, I will
make an order, the consequence of which will be, that the company will be in a position to effect the sale of these leasehold
premises without any fear of the purchaser finding that the transaction is avoided hereafter. I will say that the company shall be
at liberty to sell Nos 40/41, Market Place, Kingston, comprised in the demise dated 30 June 1960, and that such sale shall not be
avoided by s 227 in the event of a winding-up order being made in respect of the company. I will also say that the company shall
be at liberty to pay out of the proceeds of sale of the leasehold premises to the landlords such sum as may be necessary to
discharge in full their claim for arrears of rent and the costs of their action and to the secured creditor who has a charge on the
leasehold property such an amount as that creditor may be entitled to by virtue of that charge.
I have very little information about the solicitors claim for a lien for costs, and I do not feel able to give any particular
direction about it. If the solicitor 560 is entitled to a lien, he will be in the position of a secured creditor to that extent in the
winding-up and can have recourse to his remedy as such.

Order accordingly.

Solicitors: Franks, Charlesly & Co (for the company); Taylor, Willcocks & Co agents for Hamways, Croydon (for the petitioning
creditor).

Jenifer Sandell Barrister.


[1963] 2 All ER 561

Chetwynd-Talbot (Countess of Shrewsbury) v Chetwynd-Talbot (Earl of


Shrewsbury)
FAMILY; Divorce

PROBATE, DIVORCE AND ADMIRALTY DIVISION


ORMROD J
8, 10 APRIL 1963

Divorce Discretion Denial of adultery Adultery found in earlier suit Petitioner continuing in subsequent suit to refuse to
admit adultery Petition to include prayer for discretion in respect of adultery found Discretion statement unnecessary All
relevant material to be put before court Discretion exercised as denial of adultery not in circumstances dishonest
Matrimonial Causes Rules, 1957 (SI 1957 No 619), r 4(4), r 28.

In 1958 the husband presented a petition for divorce on the ground of the wifes adultery with L and prayed for the exercise of the
courts discretion in respect of his own adultery. The wife denied adultery and cross-prayed for divorce on the ground of the
husbands adultery with M. The wifes answer contained no prayer for discretion. The suit was heard in 1959, and the trial judge
found that the wife had committed adultery with L, but refused to exercise the courts discretion in the husbands favour. The
wife maintained her denial of adultery and did not ask for the courts discretion to be exercised in her favour. In consequence the
prayers in the petition and the answer were rejected, and the petition was dismissed.
The wife now petitioned for divorce on the ground of the husbands adultery with M since the trial in 1959. The petition
was not defended and the adultery was found proven. The wifes petition did not include a prayer for the courts discretion to be
exercised in her favour.

Held (i) the finding of adultery against the wife in the earlier suit was res judicata, and, therefore, she could not now obtain a
decree unless the court exercised its discretion in her favour; accordingly her petition should have included a prayer for discretion
as required by the Matrimonial Causes Rules, 1957, r 4(4), but leave would be granted to amend her petition by including a
prayer that the courts discretion should be exercised in respect of adultery found (see p 563, letter d, and p 569, letters a and d,
post).
North v North, post p 570, distinguished;
(ii) r 28 of the Matrimonial Causes Rules, 1957, applied only to cases where the party seeking discretion admitted adultery
(see p 564, letter h, post), and accordingly the wife was not under obligation to file a discretion statement in the present case.
Dicta in North v North, post p 570, applied.
(iii) in the present case all the material facts relevant to the exercise of discretion were before the court in the judgment of
the trial judge in the previous suit, and the case was one in which, apart from the factor of the wifes maintaining her denial of the
adultery of which she had been found guilty, the courts discretion should be exercised in her favour; the present was not a case in
which her denial must be regarded as deliberately dishonest, and accordingly in the exercise of the courts discretion a decree nisi
would be pronounced (see p 568, letters b and i, post).

Notes
As to praying for the courts discretion and lodging a discretion statement, see 12 Halsburys Laws (3rd Edn) 311, para 622; 319,
para 641; and for cases on the subject, see 27 Digest (Repl) 426, 427, 35723581.
561
For the Matrimonial Causes Act, 1950, s 4(2) proviso, see 29 Halsburys Statutes (2nd Edn) 394.
For the Matrimonial Causes Rules, 1957, r 4(4), r 28, see 10 Halsburys Statutory Instruments (1st Re-issue) 221, 231.

Cases referred to in judgment


Alexandre v Alexandre (1870), LR 2 P & D 164, 39 LJP & M 84, 23 LT 268, 27 Digest (Repl) 372, 3074.
Apted v Apted and Bliss [1930] P 246, 99 LJP 73, 143 LT 353, 27 Digest (Repl) 428 3588.
Bainbridge v Bainbridge [1934] P 66, 103 LJP 19, 150 LT 197, 27 Digest (Repl) 585, 5456.
Blunt v Blunt [1943] 2 All ER 76, [1943] AC 517, 112 LJP 58, 27 Digest (Repl) 429, 3589.
Brooke v Brooke [1912] P 136, 81 LJP 75, 106 LT 766, 27 Digest (Repl) 584, 5439.
Jackson v Jackson [1947] WN 287.
North v North post, p 570, (1946), LJo 457.
Pretty v Pretty [1911] P 83, 80 LJP 19, 104 LT 79, 27 Digest (Repl) 430, 3602.
Wickins v Wickins [1918] P 265, 87 LJP 155, 119 LT 268, 87 JP 155, 27 Digest (Repl) 413, 3411.
Wilkinson v Wilkinson and Seymour (1921), 126 LT 29n, 27 Digest (Repl) 428, 3586.
Wilson v Wilson [1920] P 20, 89 LJP 17, 122 LT 223, 27 Digest (Repl) 427, 3584.

Petition
This was an undefended suit in which The Rt Hon Nadine Muriel, Countess of Shrewsbury, petitioned for divorce on the ground
that her husband, The Rt Hon John George Charles Henry Alton Alexander Chetwynd, Earl of Shrewsbury, had committed
adultery with a Miss Aileen Mortlock between December, 1959, and the date of the hearing.
The facts appear in the judgment.

J Stirling QC and P A Harmsworth for the wife.


Colin Duncan for the husband.

Cur adv vult

10 April 1963. The following judgment was delivered.

ORMROD J read the following judgment. The adultery has been amply proved and I am satisfied that Lord Shrewsbury and
Miss Mortlock are and have been living together as man and wife during the greater part of this period and are now in fact living
together in Madeira. A decree of dissolution would follow as a matter of course but for one matter which gives rise to
considerable difficulty. Counsel, who appears for Lady Shreswbury, has, as always, been of the utmost assistance to me in
resolving it.
The difficulty arises in this way. In 1958 Lord Shrewsbury filed a petition against his wife, in which he charged her with
adultery with Mr Anthony Lowther and prayed for the exercise of the discretion of the court in respect of his own adultery with
Miss Mortlock. By her answer, Lady Shrewsbury denied the adultery and later by her amended answer she in turn charged her
husband with adultery with Miss Mortlock, and asked for a decree of divorce. Her amended answer contained no prayer for the
exercise of the discretion of the court in her favour. These suits were heard by Collingwood J who gave judgment on 21
December 1959, after a long hearing. In his judgment the learned judge, after a very full and careful review of the evidence,
found that Lady Shrewsbury had committed adultery with Mr Lowther and that Lord Shrewsbury had committed adultery with
Miss Mortlock. He further held that in his original discretion statement Lord Shrewsbury had markedly failed to reveal the full
extent of his adultery 562 and had put forward in extenuation of his adultery a false and misleading account of his relationship
with his wife over a period of many years. In the event the learned judge declined to exercise his discretion in Lord Shrewsburys
favour. He was not asked to consider the question of exercising his discretion in favour of Lady Shrewsbury, who maintained her
denial of the adultery to the end. In consequence he rejected both prayers and dismissed the petition.
In the present suit Lady Shrewsbury relied on adultery subsequent to the order of Collingwood J but has again denied that
she has committed adultery with Mr Lowther, notwithstanding the finding to the contrary by Collingwood J After full
consideration by her counsel her petition contains no prayer for the exercise of the courts discretion under r 4 of the Matrimonial
Causes Rules, 1957, and no discretion statement has been filed on her behalf.
Counsel for Lady Shrewsbury very properly made it clear from the beginning of his submission that it was no part of his
case to invite me directly or indirectly to review the findings of Collingwood J. Quite obviously I have no jurisdiction to
interfere with that decision, nor could I contemplate the possibility of doing so for one moment. As between the parties the issue
of Lady Shrewsburys adultery is res judicata and I must deal with the present suit on the basis of that finding, namely, that Lady
Shrewsbury has committed adultery with Mr Lowther. The result is that, notwithstanding my finding of adultery against Lord
Shrewsbury in this suit, Lady Shrewsbury is not entitled as of right to a decree of dissolution. She is, in my judgment, plainly
within the proviso to s 4(2) of the Matrimonial Causes Act, 1950, and, therefore, can succeed in obtaining a decree of dissolution
only if the court is prepared to exercise its discretion in her favour in respect of the adultery that has been found against her in the
first suit.
In my judgment, therefore, the petition in its present form is technically defective for non-compliance with r 4(4) of the
Matrimonial Causes Rules, 1957. This is an appropriate case within the rule and the prayer to the petition should be amended,
in my view, to comply with the terms of the rule, notwithstanding the dicta of the Court of Appeal in North v North which is
unreported except briefly, and inaccurately in a note to Jackson v Jackson. In that case the court was not dealing with the
situation which arises when adultery has been found against the petitioner in a previous suit so that that issue is already res
judicata at the time when the second petition is filed. North v North deals with the different situation where a party praying for
relief and denying adultery is none the less found in the same suit to have committed adultery. In such circumstances a prayer for
the exercise of the discretion in respect of adultery still in issue would not be appropriate within r 4(4).
The absence of a discretion statement raises quite different considerations. The procedure of the discretion statement was
first suggested by Lord Merrivale P, in Apted v Apted and Bliss ([1930] P at p 262) and subsequently introduced by a rule of
court. Its purpose is to provide the court in a convenient form with the facts which are relevant to the exercise of the courts
discretion. It is not a piece of procedural magic. As long ago as 1946 the Court of Appeal held in the clearest terms that the
absence of a discretion statement in no way debarred the court from exercising its discretion in a proper case. The discretion is
given by the statute and cannot be restricted or fettered by rules of court. I am referring again to North v North, in which I
appeared as a junior and I am most grateful to counsel for Lady Shrewsbury and those who instruct him for providing me with a
full transcript of the judgment. It is to be hoped that an opportunity can be found even at this length of time to report this useful
judgment more fully. I may add that an unsuccessful application was made to the Appeal Committee of the House of Lords for
leave to appeal from the decision of the Court of Appeal in that case, and in the circumstances I think it right to read the judgment
of Bucknill LJ 563 which was concurred in in the event by both the other members of the court. Bucknill LJ said a:
________________________________________
a Post, p 574

I agree, and merely add a very few words on two points which occur to me. The first is this, that the Matrimonial
Causes Act gives the judge an absolute discretion to grant a decree or not if the petitioner has himself or herself been found
guilty of adultery. Section 4 of the Matrimonial Causes Act, 1937, repeats the very words which occur in the original Act
of 1857, and it seems to me that if counsel for the wifes argument were right, and if the discretion were cut down in this
way, that unless a discretion statement was filed the judge could not grant a decree in such a case as this, the discretion
would be limited by the rules. I myself cannot think that a rule can limit a discretion given to a judge by statute.
The other point is this. I myself, speaking entirely for myself, doubt very much whether r. 4 of the Matrimonial
Causes Rules, 1944, which deals with the necessity of filing a discretion statement in appropriate cases, applies to cases of
this kind. Rule 28 uses the words: Every party to a matrimonial cause praying that the court shall exercise its discretion.
Now, one cannot pray that the court shall exercise its discretion in a case where one does not oneself admit the adultery,
and to say that after the judge has found adultery that has been proved a discretion statement must be filed seems to me to
be putting a wholly unnecessary expense on the party who is successful. The party may very well persist in the statement,
But I have not committed adultery. It is quite true that the judgment would appear to show that that allegation was
circumstantially proved, but I think that it would be most harsh and unjust to force the husband to file a discretion
statement admitting adultery which he in fact denies. I cannot believe for a moment that the rules were intended to do
anything of the kind, and the result is that this miserable document which is called a discretion statement is filed in which
the husband says, I do not think that I did commit adultery because I was physically incapable of doing so, but the learned
judge has found that I did, and I therefore ask him to exercise the discretion in my favour. What the value of that
document is to anybody I entirely fail to see. It is merely putting the parties to wholly unnecessary expense, and I myself
think that it is a wrong interpretation of the rules.

Somervell LJ and Morton LJ concurred in that view. This decision of the Court of Appeal is, of course, based on two premises.
First, that the party concerned denies the adultery which has been found against him, and secondly, that all the material facts
which are relevant to the exercise of the discretion have been put before the court in some way other than in a discretion
statement.
In the present case if Lady Shrewsbury were to be forced to file a discretion statement it would be no less miserable a
document than that which was filed in North v North. As in that case so in this all the material facts are before the court in the
shape of the judgment of Collingwood J which has been read to me in full. In my judgment, r 28 of the Matrimonial Causes
Rules, 1957, which deals with the filing of a discretion statement, has no application where the party, who is obliged to rely on
the courts discretion in respect of adultery found against her, still denies it. Rule 28, in my judgment, only applies where the
party asks for discretion in respect of admitted adultery. This emerges from the wording of the rule itself. I realise that the views
which I have just expressed sit rather awkwardly on the rules in their present form. This is inevitable because, when the rules
were drafted, I do not for one moment think that either the North v North situation or the situation which exists in the present case
were in the contemplation of the draftsman. In these circumstances, the exercise of my discretion is in no way fettered by
procedural obstacles and I must now consider whether 564 on all the facts of the present case I ought or ought not to grant Lady
Shrewsbury a decree of dissolution.
The basic principles on which this discretion is exercised at the present time are set out in Viscount Simons speech in Blunt
v Blunt. It is not, I think, perhaps inappropriate in the present case for me once again to refer to those passages, very well known
though they are. The Lord Chancellor said ([1943] 2 All ER at p 78; [1943] AC p 524):

The scope within which the divorce court has thought it right to exercise a favourable discretion has been greatly
widened since the pronouncement of Lord Penzance b, and in contrast with his view it has for long been the practice to
regard the discretion as unfettered. It is impossible to lay down strict rules for its exercise, and to attempt to do so would
really be to restrict the liberty conferred by the language of the statute. I would adopt the view of the Court of Appeal,
expressed a quarter of a century ago, in Wickins v. Wickins per SWINFEN EADY, M.R. ([1918] P. at p. 272.), that:
where Parliament has invested the court with a discretion which has to be exercised in an almost inexhaustible variety of
delicate and difficult circumstances, and where Parliament has not thought fit to define or specify any cases or classes of
cases fit for its application, this court ought not to limit or restrict that discretion by laying down rules within which alone
the discretion is to be exercised, or to place greater fetters upon the judge of the Divorce Division than the legislature has
thought fit to impose The utmost that can be properly done is to indicate the chief considerations which ought to be
weighed in appropriate cases as helping to arrive at a just conclusion. In Wilson v. Wilson, SIR HENRY DUKE, P., in
dealing with the particular case before him, mentioned four circumstances which, in his view, warranted the exercise of the
judicial discretion in the petitioners favour, and these four considerations were referred to with approval by LORD
BIRKENHEAD, L.C., when he was sitting in the divorce court and deciding Wilkinson v. Wilkinson and Seymour, the
Kings Proctor showing cause. These four points are: (a) the position and interest of any children of the marriage; (b) the
interest of the party with whom the petitioner has been guilty of misconduct, with special regard to the prospect of their
future marriage; (c) the question whether, if the marriage is not dissolved, there is a prospect of reconciliation between
husband and wife; and (d) the interest of the petitioner, and in particular the interest that the petitioner should be able to
remarry and live respectably. To these four considerations I would add a fifth of a more general character, which must
indeed be regarded as of primary importance, viz., the interest of the community at large, to be judged by maintaining a
true balance between respect for the binding sanctity of marriage and the social considerations which make it contrary to
public policy to insist on the maintenance of a union which has utterly broken down. It is noteworthy that in recent years
this last consideration has operated to induce the court to exercise a favourable discretion in many instances where in an
earlier time a decree would certainly have been refused.
________________________________________
b In Morgan v. Morgan and Porter, (1869), L.R. 1 P. & D. at p. 647.

Applying Lord Simons five considerations to the facts of the present case my conclusions are as follows. So far as the children
are concerned, there are six of them, ranging in age from the eldest girl, who is now twenty-four, to the younger boy, now aged
four and a half. The four younger ones are in the care of their mother under an order made in the Chancery Division. It cannot
be to the advantage of any of them that they should continue to live with the knowledge that their father is living in adultery with
Miss Mortlock. If that state of affairs is to continue it would be, to my mind at least, an embarrassment to them and 565 possibly
an obstacle to the future development of their relations with their father. The second consideration does not arise in the present
case because so far as the evidence goes Lady Shrewsbury has no wish to marry Mr Lowther. The third considerationthe
prospect of reconciliationis relevant in the present case. I am quite satisfied that there is not the remotest prospect of
reconciliation between the parties. Lady Shrewsbury said that after the previous suit she had made overtures to her husband by
letter, which had been ignored. In the circumstances, any other response would have been surprising. The fourth consideration
also is material, that is Lady Shrewsburys own future. If this petition is dismissed, it is difficult to see how she can ever be in a
position to remarry so long as her husband is alive.
The fifth considerationthe interest of the community at largeraises questions which are as far-reaching as they are
nebulous. It is difficult to see what interest of the community is served by preserving this marriage in its present skeletal form.
This last consideration does, however, embrace what is to my mind the real difficulty in the present case. It is manifestly in the
public interest that decrees of dissolution should not be obtained contrary to the justice of the case. From this it follows that it
is essential that the party seeking relief should put before the court all the material facts so that the court can come to a just
conclusion. This means, in the words of Sir William Jowitt, A-G, in his argument for the Kings Proctor in Apted v Apted and
Bliss ([1930] P at p 248):

The first essential for the favourable exercise of the discretion is complete candour on the part of the petitioner

It might well be a proper application of Viscount Simons principles to refuse to exercise the discretion in favour of a party who
has wilfully misled the court by falsely denying his or her adultery or by deliberately concealing it. On the other hand, it is well
established that even in such cases a last minute disclosure may justify the court in exercising its discretion favourably as is
sometimes done where an intervention by the Queens Proctor has been successful in bringing concealed adultery to light.
In Brooke v Brooke Sir Samuel Evans P, summarised the position in these words ([1912] P at p 144):

The result of the authorities appears to me to be that upon an intervention by the Kings Proctor, even where material
facts have been deliberately suppressed, or withheld, at the original hearing, the court must consider all the facts then
established by either the Kings Proctor, or the petitioner, before deciding whether it will rescind the decree nisi, or exercise
a discretion under s. 31 of the Matrimonial Causes Act, 1857, or adopt some immediate course which would necessitate a
further inquiry in order to ascertain the true facts I would respectfully deprecate any attempt to fetter this discretion by
any strict or inelastic rules laid down either in this court, or in any appellate court, other than such as may be essential in
principle by reason of the provisions of the Acts of Parliament which confer the discretion upon the tribunal to which it is
entrusted.
It must not be thought that I deem the withholding or suppression of material facts from the court on the hearing of a
petition as unimportant. On the contrary, I think it is of grave importance that they should be laid before the court from the
first, especially when the court is going to be asked to apply itself to the question of discretion under s. 31 of the
Matrimonial Causes Act, 1857. Where they are not, the court ought not to, and will not, pass over lightly the omission to
disclose them, whether the omission be intentional, or deliberate, or (as is sometimes said) fraudulent; or whether it be due
to some more innocent reason (as is alleged in the present case). And it must not be assumed that it is only where the
omission is deliberate, or fraudulent, that the court may rescind the decree nisi.
566

Those passages were referred to with approval by Sir Boyd Merriman P, in Bainbridge v Bainbridge ([1934] P at p 75).
These cases do not, however, cover the facts of the present case for Lady Shrewsbury has maintained her denial of adultery
before me. Does this fact preclude me from exercising my discretion in her favour? There is at least one case in which the court
has exercised its discretion in favour of a petitioner who maintained her denial of her own adultery to the end after being found
guilty of it by a jury. That case is Pretty v Pretty which was also referred to with approval by Sir Boyd Merriman P, in
Bainbridge v Bainbridge. It is, however, a very exceptional case and its facts are very far from the facts of the present case.
It is, therefore, necessary to consider the present case on principle. There is no rule of law to the effect that the court may
not grant a decree of dissolution to a party found guilty of adultery unless he or she makes an admission of the adultery which he
or she in fact denies. In North v North Bucknill LJ in the passage that I have already read said that where a party against whom
adultery was circumstantially proved persists in his denial it would be most harsh and unjust to force [him] [to admit]
adultery which he in fact denies. Moreover, no one would suggest for a moment that a party found guilty of cruelty which he
has strenuously contested must be refused a decree of dissolution, unless he can be brought to admit his cruelty at the last
moment.
The foundation of the whole of the modern practice relating to discretion lies in s 7 of the Matrimonial Causes Act, 1860, the
relevant parts of which are as follows:

Every decree for a divorce shall in the first instance be a decree nisi, not to be made absolute till after the expiration of
such time, not less than three months from the pronouncing thereof, as the court shall be general or special order from time
to time direct; and during that period any person shall be at liberty, in such manner as the court shall by general or special
order in that behalf from time to time direct, to show cause why the said decree should not be made absolute by reason of
the same having been obtained by collusion or by reason of material facts not brought before the court; and, on cause being
so shown, the court shall deal with the case by making the decree absolute, or by reversing the decree nisi, or by requiring
further inquiry, or otherwise as justice may require;.

The rest of the section is not material. Failure to put the material facts before the court exposes the petitioner to the risk of
intervention and rescission of his decree, particularly where the material facts amount to a discretionary bar. The procedure of
the discretion statement was introduced as a means of ensuring that the petitioner puts the material facts before the court at the
proper time. On the material facts coming before the court, the court must adjudicate on them. As long ago as 1870 Lord
Penzance in Alexandre v Alexandre said ((1870), LR 2 P & D at p 167):

But there is nothing, as it seems to me, in s. 7 of the statute c, that would justify the court, on the mere proof that some
material facts had been kept back, in refusing the petitioner his divorce, if it appears upon all the facts being known that he
is in all other respects entitled to it. The suppression of a material fact is a sufficient reason to justify a third person, or the
Queens Proctor, in intervening; but when the material fact, whatever it may be, is brought to light and placed before the
court, there is nothing in s. 7 which would authorise the court to withhold a decree if, upon the whole, supposing the fact
had been brought to light in the first instance, the petitioner would have been entitled to it. Therefore, the court has to
consider whether, upon the whole, there is anything to justify it in withholding this 567 decree, treating the facts now
disclosed as if they had been brought before it in the first instance.
________________________________________
c (Matrimonial Causes Act, 1860.

Sir Samuel Evans in the passage that I have read from Brooke v Brooke ([1912] P at p 144) expressed the same view.
In my judgment, therefore, the primary question in the present case is whether Lady Shrewsbury has brought all the material
facts before the court. By placing the judgment of Collingwood J before me she has put before me all the material facts,
including the finding of adultery against her. The court is, therefore, now in possession of all the material on which to decide
whether or no this marriage should be dissolved.
The only valid ground on the facts of the present case on which, in my judgment, I should be justified in refusing to exercise
my discretion in Lady Shrewsburys favour is that she had been guilty of perjury in the first suit and consequently in the present
suit. I use the word perjury advisedly because it is one thing for a judge having heard all the evidence to come to the conclusion
that he must infer that adultery has taken place in spite of the denial on oath of the party charged and another to establish as a
positive fact that the denial is a lie. Not to be believed is one thing; it is another to be a liar. Of course, inter partes, the findings
of fact by a judge are conclusive of the truth of the facts found. In that sense the party whose evidence is rejected must be
regarded as having told an untrue story, but judges and juries are not infallible and such a finding by itself cannot, in my
judgment, be conclusive of the truth in an absolute sense. In Bainbridge v Bainbridge the wife petitioner had been found guilty
of adultery by Horridge J on an intervention in 1917. In subsequent proceedings before Lord Merrivale she again denied the
adultery which had been proved against her. In his judgment in the later proceedings ([1934] P at p 68), Lord Merrivale felt
entitled to say that he believed her denial in spite of the finding in the previous proceedings against her. Let me, however, make
it perfectly clear that in the present case I am not impugning Collingwood Js decision nor attempting any re-assessment of Lady
Shrewsburys credibility. If I were to be asked to do either of those things it would mean in effect re-trying the whole of the
previous suit. There are, of course, cases in which it is established as a positive fact that a petitioner has lied. This may be
proved by documentary evidence or other evidence which does not depend on the veracity of another witness. In such cases the
court would, I imagine, hesitate a long time in the public interest before acceding to the prayer of such a party. In other cases the
evidence of a petitioner may be so obviously unsatisfactory that the court can come to a confident conclusion that he is lying and
so refuse to exercise discretion in his favour. In others, notwithstanding proof beyond reasonable doubt, a scintilla of doubt may
remain.
I must now ask myself whether the petitioner in the present case is to be regarded as a perjurer in the strict sense. I have
most carefully considered the judgment of Collingwood J and it appears to me that strong though the evidence against her was on
the learned judges assessment of the witnesses, his decision turned on his assessment of respective credibility. There is,
therefore, a possibility that Lady Shrewsbury was telling the truth. It would not be right, therefore, for me to regard her denial of
adultery before me as necessarily deliberately dishonest. As I have already said, the facts in the present case bring it beyond any
question within the principles of Blunt v Blunt subject only to the petitioners steadfast denial of her adultery. Having been found
guilty of that adultery she must rely on the discretionary jurisdiction and put all the material facts before the court. This, in my
judgment, she has done. It is my duty to exercise my discretion on all the facts as they are before me, including Collingwod Js
finding of adultery which stands and Lady Shrewsburys continued 568 refusal to admit it. In the exercise of my discretion I
have come to the conclusion that justice requires that I should pronounce a decree nisi.
In conclusion I wish to emphasise two points. First, in the earlier suit Colling-Wood J was not asked to consider whether or
not to exercise the discretion in favour of Lady Shrewsbury and it is, therefore, unnecessary for me to consider what effect, if any,
a refusal to exercise the discretion in the first suit would have had in the second suit. Secondly, if Lady Shrewsburys denial of
her adultery as found in the previous suit is a bar to the exercise of the discretion in the present suit, it would be no less a bar to a
suit by her in the future. If, therefore, a reconciliation had taken place followed by gross cruelty on the part of the respondent,
Lady Shrewsbury could only obtain relief by making what, on her view of the matter, would be a dishonest admission of adultery.
I cannot think that the proviso to s 4(2) of the Matrimonial Causes Act, 1950, which simply says that the court shall not be bound
to pronounce a decree in favour of a petitioner who is found to have committed a matrimonial offence himself, could possibly
have such an effect.
I think that the prayer of the petition should be amended. It would mean slightly altering the conventional from to allege in
respect of adultery found. In the exercise of my discretion, therefore, I shall pronounce a decree nisi. I certify that I am satisfied
with the arrangements as to the children, whom I have already said are wards of court and, therefore, I need say no more about
them. I will make an order so far as maintenance is concerned in the terms of the minute which has been handed to me, and that
can be incorporated in the decree. I shall make an order for costs against the respondent. I give leave to make the final
amendment to the petition and dispense with re-service.

Order accordingly.

Solicitors: Withers, Nicholl, Manisty & Co (for the wife); Stephenson, Harwood & Tatham (for the husband).

A T Hoolahan Esq Barrister.


569
[1963] 2 All ER 570

North v North
FAMILY; Divorce

COURT OF APPEAL
MORTON, BUCKNILL, SOMERVELL LJJ
18 JULY 1946

Divorce Discretion Denial of adultery Prayer for discretion and discretion statement Whether required where party
maintains belief that he or she has not committed adultery Matrimonial Causes Rules, 1944(SR & O 1944 No 389), r 4(3)(e), r
28(1).

The parties were married in November 1935. In November 1936, the husband went to a dinner after which, instead of going
home, he took a woman to a hotel for the night with a view to committing adultery; owing to the amount of alcohol that he had
consumed, however, he did not know next morning whether or not he had in fact committed adultery. The husband told the wife
of the incident and the parties continued to live together, a child being born to them in November, 1937. In 1944 the wife filed a
petition for divorce on the grounds of the husbands cruelty and adultery with two named women, the incident in November,
1936, not being made the subject of a charge of adultery. The husband denied both cruelty and adultery and cross-prayed for
divorce on the ground of the wifes adultery. The trial judge dismissed the wifes charge of cruelty but found that the husband
had in fact committed adultery on the occasion in November, 1936. The trial judge also found that the wife had committed
adultery. The husband having, by the leave of the court, amended his answer by adding a prayer for discretion and filed a
discretion statement in respect of the incident in November, 1936, the court, in the exercise of its discretion granted him a decree
nisi, notwithstanding that the husband in his discretion statement maintained his belief that he had not in fact committed adultery.
On appeal by the wife,

Held the trial judge had rightly exercised his discretion and the appeal would be dismissed.
Per Curiam: it was doubtful whether the rules requiring a party to include a prayer for discretion and to file a discretion
statement applied to a case, such as the present, where the husband denied that he had committed adultery (see p 574, letters f and
g, and p 575, letter a, post).

Notes
As to praying for the courts discretion and lodging a discretion statement, see 12 Halsburys Laws (3rd Edn) 311, para 622; 319,
para 641; and for cases on the subject, see 27 Digest (Repl) 426, 427, 35723581.

Case referred to in judgments


Blunt v Blunt, [1943] 2 All ER 76, [1943] AC 517, 112 LJP 58, 27 Digest (Repl) 429, 3589.

Appeal.
This was an appeal by the wife against an order of Hodson J, dated 28 March 1946, granting a decree nisi of divorce to the
husband.
The parties were married on 15 November 1935, and a child was born on 11 November 1937. On 24 March 1944, the wife
presented a petition for divorce on the grounds of the husbands cruelty and adultery. The allegations of cruelty were set out in
para 5 to para 25 of the petition, and para 7 read as follows:

That at the end of the year 1936 the [husband] arranged with the [wife] to return from a Masonic dinner in London by
the last train from London to Reading, arriving just after 11 pm so that he would be home by midnight. The [wife] retired
to bed and on waking in the early hours of the morning, about 2 am, finding the [husband] had not returned, became frantic
and agitated, and was fearful of a railway accident. The [husband] subsequently admitted to the [wife] that he had stayed
the night with a woman in London. The conduct of the [husband], as outlined above, affected the [wifes] health.
570

The allegation in this paragraph was not relied on as part of the charge of adultery. Adultery was alleged in the petition with two
other women.
In his answer the husband denied the charge of cruelty and the charge of adultery and cross-prayed for a decree of divorce
on the ground of the wifes adultery with a named person. The husband did not include any prayer for the exercise of the courts
discretion in respect of any adultery on his part.
At the hearing of the suit before Hodson J, the wife did not proceed with the charge of adultery in respect of one of the
named women but proceeded with the charge of cruelty and the charge of adultery in respect of the other named woman. In his
judgment, delivered on 28 March 1946, Hodson J, dismissed the charge of cruelty against the husband but found that he had
committed adultery on the occasion referred to in para 7 of the petition. Hodson J, also found that the wife had committed
adultery. The learned judge then gave leave to the husband to amend his answer to include a prayer for discretion and also gave
leave to the husband to file a discretion statement and extended the time for doing so for seven days. On 3 April 1946, the
husband duly amended his answer and filed a discretion statement and on 10 April 1946, Hodson J, granted the husband a decree
nisi. The wife now appealed.

J P Eddy, K C,and H S Simmons for the wife.


R E Manningham-Buller, K C, and R F Ormrod for the husband.

18 July 1946. The following judgments were delivered.

MORTON LJ, stated the facts and continued: In the passage in his judgment dealing with the finding of adultery on the part of
the husband Hodson J, says this:

It is not alleged as a charge of adultery but as a charge of crueltyas part of the conduct of the husband which is said
to have affected the wifes health. It appears that at the end of 1936 he had been to a dinner in London and was intending
to come back to Reading by the last train which presumably left Paddington round about midnight. On that particular night
he got drunk and he took a woman to the Great Western Hotel. He had his luggage with him and although he does not
remember registering, he engaged a room, a double room for himself and this woman, in which there were two beds. He
says that he woke up in the morning and found that the woman had gone. Now that incident depends entirely on his
evidence and his statement to his wife. He told his wife no doubt something to the effect that he had slept in this room
without going into details. He treated the matter on the footing that he had behaved badly and he said in his evidence that
his wife took it very well, and she appears to have taken it very well. The question, of course, which I have to consider is
what finding I am to make about it. His answer to the question whether in fact he committed adultery with this woman is:
I do not believe I did, and that does not seem to me to be a very effective answer on the facts which I have stated. I
cannot help thinking that that was the way in which the husband himself looked at that position when he was telling his
wife about itwhether I committed adultery or not the facts are as I have described them. You have a legitimate
grievance against me, and nice questions as to whether through my drunken condition I was able to have intercourse with
this woman, with whom I intended to have intercourse, really do not carry the matter very much further. That in substance
is the view that I take. Having gone there with the intention of committing adultery, and having been unable to dispose of
the matter any further than by saying, I really was very drunk but I do not believe I did, without my saying anything
involving that I am finding him guilty of perjury, I do not think that I am able to accept that as a sufficient explanation of
the incident to prevent me drawing the inference which I think I must draw on those facts, namely, that he has committed
adultery.
571

Now, as the learned judge says, he does not find the husband guilty of perjury. That part of his judgment comes to this. He says:
I draw the inference that the husband committed adultery. I think he believes that he did not, and I am satisfied that he told his
wife about spending a night with this woman in a hotel soon after it happened. There is no doubt that after that disclosure on the
part of the husband the parties continued to live together for years as man and wife, and in fact a child was conceived and born
after it.
The learned judge, having arrived at these conclusions, at the end of his judgment said this:

The charge of adultery against the wife is proved; but as I have found the husband guilty of adultery in the
circumstances disclosed by himself, but not put forward as making a finding by me necessary, I have been asked for leave
to amend the answer by asking for the discretion of the court to be exercised in the husbands favour. The facts being
known to the wife and being condoned by her, it seems that there is no reason for me to withhold the exercise of my
discretion, and therefore the answer put in by the husband ought to be amended.

Then, after further discussion counsel then acting for the husband asked leave to put in a discretion statement in addition to the
leave to amend, and the learned judge gave that leave and extended the time for seven days.
Subsequently, on 2 April 1946, the husband signed a discretion statement, and that statement was duly lodged on the next
day, 3 April. Counsel for the wife has criticised the wording of this discretion statement, but to my mind his criticisms have no
substance. The husband says still:

It is my belief that I did not commit adultery that night, because I do not think I was physically capable of doing so.

The discretion statement then goes on to say that in view of the finding of adultery he asks the court to exercise its discretion in
his favour. The husbands answer was also amended on 3 April by inserting a prayer for the exercise of the courts discretion.
Subsequently, on 10 April, the judge granted a decree nisi to the husband.
At the hearing of this appeal counsel for the wife relied on two grounds for saying that the learned judges decision ought to
be reversed. In the first place, he says that the learned judge had no power to exercise his discretion in favour of the husband
because the statute and rules governing the matter were not observed. Secondly, he says that the learned judge, even if he had
that discretion, exercised it wrongly.
I shall deal first very shortly with the second of these arguments. In my view, it is a wholly unfounded argument. The
learned judge had ample material on which he could exercise his discretion, and there are no grounds whatever on which this
court could interfere with the exercise of that discretion; and so the decision stands. I do not think that there is any need to refer
to the various authorities cited by counsel for the wife, but if authority were required it is to be found very clearly set out in Blunt
v Blunt.
I now come to counsel for the wifes first point, which is based on the wording of the statutes and rules. I first turn to s 99 of
the Supreme Court of Judicature (Consolidation) Act, 1925, which is as follows (18 Halsburys Statutes (2nd Edn) 511):

(1) Rules of court may be made under this Act for the following purposes:(a) For regulating and prescribing the
procedure (including the method of pleading) and the practice to be followed in the Court of Appeal and the High Court
respectively in all causes and matters whatsoever in or with respect to which those courts respectively have for the time
being jurisdiction
572
Pursuant to that section, rules were made in 1944 which were the Matrimonial Causes Rules, 1944. Rule 4(3)(e) reads as follows
(Now r 4(4) of the Matrimonial Causes Rules, 1957, 10 Halsburys Statutory Instruments (First Re-issue) 221):

The petition shall conclude with a prayer setting out particulars of the relief claimed including in appropriate cases
a prayer that the court will exercise its discretion to grant a decree nisi notwithstanding the adultery of the petitioner during
marriage.

It is common ground between the counsel before us that that rule applies equally to either a petition or an answer; but it is to be
observed that the words appear in appropriate cases. It seems to me that in a case where the petitioner is saying, I have not
committed adultery, it would be most inappropriate for him to conclude with a prayer that the court will exercise its discretion
to grant a decree nisi notwithstanding the adultery of the petitioner during the marriage.
I now turn to r 28(1), which is as follows:

Every party to a matrimonial cause praying that the court shall exercise its discretion to grant a decree nisi
notwithstanding that partys adultery shall lodge in the Divorce Registry a statement (in these rules called a discretion
statement) signed by him or his solicitor stating that the court will be asked to exercise its discretion on his behalf
notwithstanding his adultery and setting forth particulars of the acts of adultery committed and of the facts which it is
material for the court to know for the purpose of the exercise of its discretion.

In the present case all the facts which it was material for the court to know for the purpose of the exercise of its discretion had
come out in the course of the hearing, and the filing of what is usually called a discretion statement was, in the circumstances of
this particular case, a pure formality. Nevertheless, the rules must no doubt be observed, and I am quite willing to assume in
counsel for the wifes favour that it was absolutely necessary in the circumstances of this case for the husband to lodge a
discretion statement. No time is specified in the rules within which a discretion statement is to be lodged, and the husband to my
mind has complied completely with the rule which I have just read. Then r 28(2) is this:

Where the application for the registrars certificate under r 30 is made by the party praying for the discretion,

and so on. I need not read that, because, in the present case the application for the registrars certificate under r 30 was not made
by the husband, but by the wife. Turning to r 30, it says:

(1) The petitioner or any party who is defending a matrimonial cause shall, before setting down the cause for trial or
hearing, refer the pleadings and proceedings in the cause to the registrar for his certificate that the pleadings and
proceedings are in order and for directions as to the place of trial or hearing,

and so on. That was done in the present case by the wife or her solicitor, and it does not seem to me that that rule is material for
the present purpose.
We were also referred to r 14 with regard to the amendment of the husbands answer, which was allowed by the judge:

(1) No supplemental petition shall be filed and no petition shall be amended without leave,

and to r 19:

Any originating summons, notice of an application for ancillary relief, summons, pleading or other document may be
amended by leave subject to any directions as to re-service and as to consequential amendment of pleadings already filed.
573

For my part I cannot see that there has been any infringement whatsoever of the rules in the present case.
I would add this, in case it is material, that under r 81:

Subject to the provisions of these rules and of any statute, the Rules of the Supreme Court shall, notwithstanding the
provisions of Ord 68 thereof, apply with the necessary modifications to the practice and procedure in any material cause or
matter to which these rules relate.

It would appear to be in the power of the court to enlarge the time for the doing of any act. But, as I have said, I cannot see that
any time is laid down in the rules, certainly not in any of the rules to which we have been referred, for the filing of the discretion
statement.
I can quite appreciate that, as a result of the trial, the wife now finds herself in a most unfortunate financial position, and I
can well appreciate that she was most desirous of obtaining a reversal of the learned judges decision on any ground that might be
available; but I must say, speaking for myself, that I do not think that I have ever heard a more ill-founded and baseless appeal,
since I first joined this court in October, 1944. The appeal will be dismissed, with costs.

BUCKNILL LJ. I agree, and merely add a very few words on two points which occur to me. The first is this, that the
Matrimonial Causes Act gives the judge an absolute discretion to grant a decree or not if the petitioner has himself or herself been
guilty of adultery. Section 4 of the Matrimonial Causes Act, 1937, repeats the very words which occur in the original Act of 1857
and it seems to me that if counsel for the wifes argument were right, and if the discretion were cut down in this way, that unless a
discretion statement was filed the judge could not grant a decree in such a case as this, the discretion would be limited by the
rules. I myself cannot think that a rule can limit a discretion given to a judge by statute.
The other point is this. I myself, speaking entirely for myself, doubt very much whether r 4(3)(e) of the Matrimonial Causes
Rules, 1944, which deals with the necessity of filing a discretion statement in appropriate cases, applies to cases of this kind.
Rule 28(1) uses the words:

Every party to a matrimonial cause praying that the court shall exercise its discretion.

Now, one cannot pray that the court shall exercise its discretion in a case where one does not oneself admit the adultery, and to
say that after the judge has found that adultery has been proved a discretion statement must be filed seems to me to be putting a
wholly unnecessary expense on the party who is successful. The party may very well persist in the statement, But I have not
committed adultery. It is quite true that the judgment would appear to show that that allegation was circumstantially proved, but
I think that it would be most harsh and unjust to force the husband to file a discretion statement admitting adultery which he in
fact denies. I cannot believe for a moment that the rules were intended to do anything of the kind, and the result is that this
miserable document which is called a discretion statement is filed in which the husband says, I do not think that I did commit
adultery because I was physically incapable of doing so, but the learned judge has found that I did, and I therefore ask him to
exercise the discretion in my favour. What the value of that document is to anybody I entirely fail to see. It is merely putting
the parties to wholly unnecessary expense, and I myself think that it is a wrong interpretation of the rules. I could not more
heartily agree with what my lord has said about this appeal having not a vestige of substance in it, and I think that this appeal
should be dismissed.

SOMERVELL LJ. I do not wish to add anything on the subject of this appeal, and I approve of the reasons which have been
given for dismissing it. 574I should only like to say that I associate myself with the doubts which have been expressed by my
brothers whether r 4(3)(e) applies to the position which has arisen in the present case at all. I think that the appeal should be
dismissed, with costs.

MORTON LJ. I should like to add that, having heard the judgment of Somervell LJ, I share his feeling that r 4(3)(e) and r 28(1)
were not really intended to apply to a case like the present, where the party denies adultery. I assumed that they applied for the
purposes of my judgment. I did not express any view that they do apply, and I do not think that they do.

Appeal dismissed.

Solicitors: Leader, Henderson & Leader (for the wife); Dawson & Co (for the husband).

A T Hoolann Esq Barrister.


[1963] 2 All ER 575

Hedley Byrne & Co Ltd v Heller & Partners Ltd


TORTS; Negligence

HOUSE OF LORDS
LORD REID, LORD MORRIS OF BORTH-Y-GEST, LORD HODSON, LORD DEVLIN AND LORD PEARCE
25, 26, 27, 28 FEBRUARY, 4, 5, 6, 7 MARCH, 28 MAY 1963

Negligence Duty to take care Statements Information or advice Knowledge of informant or advisor that he was being
trusted or that reliance was placed on his skill or judgment Whether duty to exercise care imposed on person giving
information or advice.

Bank Statement in answer to inquiry Financial references given honestly but without due care Bank giving references to
other bank Use by second banks customer expected First bank financing customer whose reference given Whether first
bank liable to second banks customer.

If, in the ordinary course of business or professional affairs, a person seeks information or advice from another, who is not under
contractual or fiduciary obligation to give the information or advice, in circumstances in which a reasonable man so asked would
know that he was being trusted, or that his skill or judgment was being relied on, and the person asked chooses to give the
information or advice without clearly so qualifying his answer as to show that he does not accept responsibility, then the person
replying accepts a legal duty to exercise such care as the circumstances require in making his reply; and for a failure to exercise
that care an action for negligence will lie if damage results (see p 583, letter d, p 588, letter i, to p 589, letter a, p 590, letter b, p
594, letter c, p 598, letter a, p 601, letter b, p 606, letter h, p 610, letters e to h, and p 617, letter g, post).
Cann v Willson ((1888), 39 ChD 39), Fish v Kelly ((1864), 17 CBNS 194), approved.
Nocton v Lord Ashburton ([191415] All ER Rep 45), Robinson v National Bank of Scotland (1916 SC (HL) 154), and view
of Denning LJ dissenting in Candler v Crane, Christmas & Co ([1951] 1 All ER, see, eg, at p 432, letter a) applied.
Candler v Crane, Christmas & Co ([1951] 1 All ER 426) and Le Lievre v Gould ([1893] 1 QB 491) disapproved.
A bank inquired by telephone of the respondent merchant bankers concerning the financial position of a customer for whom
the respondents were bankers. The bank said that they wanted to know in confidence and without responsibility on the part of the
respondents, the respectability and standing of E Ltd and whether E Ltd would be good for an advertising contract for 8,000 to
9,000. Some months later the bank wrote to the respondents asking in confidence the respondents opinion of the respectability
and standing of E Ltd by stating whether the respondents 575 considered E Ltd trustworthy, in the way of business, to the extent
of 100,000 per annum. The respondents replies to the effect that E Ltd was respectably constituted and considered good for its
normal business engagements were communicated to the banks customers, the appellants. Relying on these replies the
appellants, who were advertising agents, placed orders for advertising time and space for E Ltd on which orders the appellants
assumed personal responsibility for payment to the television and newspaper companies concerned. E Ltd went into liquidation
and the appellants lost over 17,000 on the advertising contracts. The appellants sued the respondents for the amount of the loss,
alleging that the respondents replies to the banks inquiries were given negligently, in the sense of misjudgment, by making a
statement which gave a false impression as to E Ltds credit. Negligence was found at the trial and contested on appeal; the
appeal was determined, however, on the assumption that there had been negligence, but without deciding whether there had or
had not been negligence.
Held Although in the present case, but for the respondents disclaimer, the circumstances might have given rise to a duty of
care on their part, yet their disclaimer of responsibility for their replies on the occasion of the first inquiry was adequate to
exclude the assumption by them of a legal duty of care, with the consequence that they were not liable in negligence (see p 586,
letter h, p 595, letter a, p 599, letter d, p 613, letter d, and p 618, letter c, post).
Robinson v National Bank of Scotland (1916 SC (HL) 154) applied.
Semble (per Lord Reid, Lord Morris Of Borth-Y-Gest and Lord Hodson) in the absence of special circumstances requiring
particular search and consideration on the part of a bank giving to another bank a reference concerning a customers credit-
worthiness there is no legal duty on the replying bank beyond that of giving an honest answer (see p 594, letter i, to p 595, letter
a, p 586, letter g, and p 600, letter b, post; cf p 613, letter b, and p 618, letter d, post).
Decision of the Court of Appeal ([1961] 3 All ER 891) affirmed, but not on the same ground.

Notes
The statement of principle at p 575, letter f ante represents an endeavour to combine the reasons of all the opinions delivered, but
is based principally on the formulation made by Lord Reid; attention is drawn in this connexion to Lord Devlins words regarding
the circumstances which give rise to a relation where a legal duty to exercise care in giving information existsI am prepared,
he said, to adopt any one of your lordships statements as showing the general rule; and I pay the same respect to the statement
by Denning LJ in his dissenting judgment in Candler v Crane, Christmas & Co [1951] 1 All ER 426, see eg, at p 433) about
circumstances in which he says a duty to use care in making a statement exists (see p 611, letter c, post).
It may be convenient to summarise here the characteristics principally distinguishing this case from other actions for
negligence, namely, that it was an action for negligence in word, not deed, causing financial loss, not physical damage, and to
mention, by way of distinction, some other causes of action (apart from defamation) on which liability in damages for
misstatement may be founded, viz, breach of contract or fiduciary relation and fraud. The present decision shows that a duty to
exercise proper care may arise either out of special relationship of a general character, eg, the relation of solicitor and client or of
banker and customer, or out of a particular relationship created ad hoc (see, eg, p 611, letter g, post).
As to the arising of a duty to take reasonable care, see 28 Halsburys Laws (3rd Edn) 7, para 4, p 20, para 17; and for cases
on the subject, see 36 Digest (Repl) 1218, 3479.
576
As to bankers answering inquiries as to a customers financial position, see 2 Halsburys Laws (3rd Edn) 241, para 455; and
for cases on the subject, see 3 Digest (Repl) 344, 11151117.

Cases referred to in opinions


Banbury v Bank of Montreal [191819] All ER Rep 1, [1918] AC 626, 87 LJKB 1158, 119 LT 446, 36 Digest (Repl) 14, 59.
Candler v Crane, Christmas & Co [1951] 1 All ER 426, [1951] 2 KB 164, 36 Digest (Repl) 17, 75.
Cann v Willson (1888), 39 ChD 39, 57 LJCh 1034, 59 LT 723, 35 Digest 33, 242.
Coggs v Bernard (1703), 1 Sm LC (13th Edn) 175, 1 Salk 26, 1 Com 133, Holt, KB 13, 2 Ld Raym 909, 3 Salk 11, 92 ER 107,
36 Digest (Repl) 32, 144.
Dartnall v Howard (1825), 4 B & C 345, 6 Dow & Ry KB 438, 3 LJOSKB 246, 107 ER 1088, 12 Digest (Repl) 251, 1947.
De la Bere v Pearson Ltd [19047] All ER Rep 755, [1908] 1 KB 280, 77 LJKB 380, 98 LT 71, 36 Digest (Repl) 44, 235.
Derry v Peek (1889), 14 App Cas 337, 58 LJCh 864, 61 LT 265, 54 JP 148, revsg Peek v Derry (1887), 37 ChD 541, 57 LJCh
347, 59 LT 78, 9 Digest (Repl) 127, 685.
Donoghue (or McAlister) v Stevenson [1932] All ER Rep 1, [1932] AC 562, 1932 SC (HL) 31, 101 LJPC 119, 147 LT 281, 36
Digest (Repl) 85, 458.
Everett v Griffiths [1920] 3 KB 163, 89 LJKB 929, 123 LT 280, 84 JP 161, affd HL, [1921] 1 AC 631, 90 LJKB 737, 125 LT 230,
85 JP 149, 33 Digest 267, 1862.
Fish v Kelly (1864), 17 CBNS 194, 144 ER 78, 42 Digest 108, 1032.
George v Skivington (1869), LR 5 Exch 1, 39 LJEx 8, 21 LT 495, 39 Digest 441, 705.
Gladwell v Steggall (1893), 5 Bing NC 733, 8 Scott 60, 8 LJCP 361, 132 ER 1283, 34 Digest 549, 80.
Glanzer v Shepard (1922), 233 NY 236,.
Grant v Australian Knitting Mills [1935] All ER Rep 209, [1936] AC 85, 105 LJPC 6, 154 LT 18, 36 Digest (Repl) 86, 461.
Haseldine v Daw & Son Ltd [1941] 3 All ER 156, [1941] 2 KB 343, 111 LJKB 45, 165 LT 185, 36 Digest (Repl) 29, 125.
Heaven v Pender (1883), 11 QBD 503, 52 LJQB 702, 49 LT 357, 47 JP 709, 36 Digest (Repl) 7, 10.
Heilbut, Symons & Co v Buckleton [191113] All ER Rep 83, [1913] AC 30, 82 LJKB 245, 107 LT 769, 9 Digest (Repl) 259,
1643.
Herschel v Mrupe 1954 (3) SA 464 (South African Law Reports).
Heskell v Continental Express Ltd [1950] 1 All ER 1033, 17 Digest (Repl) 131, 380.
Le Lievre v Gould [1893] 1 QB 491, 62 LJQB 353, 68 LT 626, 57 JP 484, 36 Digest (Repl) 9, 27.
Low v Bouverie [1891] 3 Ch 82, 60 LJCh 594, 65 LJ 533, 43 Digest 852, 3002.
Morrison Steamship Co Ltd v Greystoke Castle (Cargo Owners) [1946] 2 All ER 696, [1947] AC 268, [1947] LJR 297, 176 LT
66, 2nd Digest Supp.
Nocton v Lord Ashburton [191415] All ER Rep 45, [1914] AC 932, 83 LJCh 784, 111 LT 641, 42 Digest 107, 1017.
Old Gate Estates Ltd v Toplis and Harding and Russell [1939] 3 All ER 209, 161 LT 227, 1 Digest (Repl) 790, 3180.
Parsons v Barclay & Co Ltd [190810] All ER Rep 429, 103 LT 196, 3 Digest (Repl) 344, 1115.
577
Pasley v Freeman (1789), 3 Term Rep 51, 2 Smith LC (12th Edn) 71, 100 ER 450, 1 Digest (Repl) 27, 205.
Perlman v Zoutendyk 1934 CPD 151 (South Africa, Cape of Good Hope Provincial Division).
Plowright v Lambert (1885), 52 LT 646, 43 Digest 978, 4180.
Robinson v National Bank of Scotland 1916 SC 46, affd HL, 1916 SC (HL) 154, 26 Digest (Repl) 33, 63.
Rutter v Palmer [1922] All ER Rep 367, [1922] 2 KB 87, 91 LJKB 657, 127 LT 419, 3 Digest (Repl) 78, 159.
Scholes v Brook (1891), 63 LT 837, affd CA, 64 LT 674, 35 Digest 53, 477.
Shiells v Blackburne (1789), 1 Hy Bl 158, 126 ER 94, 3 Digest (Repl) 71, 111.
Skelton v London and North Western Ry Co (1867), LR 2 CP 631, 36 LJCP 249, 16 LT 563, 36 Digest (Repl) 28, 121.
Tournier v National Provincial and Union Bank of England [1923] All ER Rep 550, [1924] 1 KB 461, 93 LJKB 449, 130 LT 682,
32 Digest 70, 985.
Ultramares Corpn v Touche (1931), 255 NY 170.
Wilkinson v Coverdale (1793), 1 Esp 74, 12 Digest (Repl) 251, 1945.
Woods v Martins Bank Ltd [1958] 3 All ER 166, [1959] 1 QB 55, [1958] 1 WLR 1018, 3 Digest (Repl) 182, 324.

Appeal
This was an appeal from an order of the Court of Appeal (Ormerod, Harman and Pearson LJJ) dated 18 October 1961, and
reported [1961] 3 All ER 891, dismissing an appeal by the appellants, Hedley Byrne & Co Ltd from the judgment of McNair J
dated 20 December 1960, dismissing the claim of the appellants against the respondents, Heller & Partners Ltd who were
merchant bankers, for alleged negligence or breach of duty in the giving of two bankers references relating to the credit of a
customer of the respondents, called Easipower Ltd The facts are summarised in the opinion of Lord Reid.

Gerald Gardiner QC, D G A Lowe and B H Anns for the appellants.


J G Foster QC, J M Shaw and L J Blom-Cooper for the respondents.

Their Lordships took time for consideration

28 May 1963. The following opinions were delivered.

LORD REID. My Lords, this case raises the important question whether and in what circumstances a person can recover
damages for loss suffered by reason of his having relied on an innocent but negligent misrepresentation. I cannot do better than
adopt the following statement of the case for the judgment of McNair J:

This case raised certain interesting questions of law as to the liability of bankers giving references as to the credit-
worthiness of their customers. The [appellants] are a firm of advertising agents. The [respondents] are merchant bankers.
In outline, the [appellants] case against the [respondents] is that, having placed on behalf of a client, Easipower, Ltd., on
credit terms substantial orders for advertising time on television programmes and for advertising space in certain
newspapers on terms under which they, the [appellants], became personally liable to the television and newspaper
companies, they caused inquiries to be made through their own bank of the [respondents] as to the credit-worthiness of
Easipower, Ltd. who were customers of the [respondents] and were given by the [respondents] satisfactory references.
These references turned out not to be justified, and the [appellants] claim that in reliance on the references, which they had
no reason to question, they refrained from cancelling the orders so as to relieve themselves of their current liabilities.
578

The appellants, becoming doubtful about the financial position of Easipower Ltd got their bank to communicate with the
respondents who were Easipower, Ltds bankers. This was done by telephone and the following is a contemporaneous note of the
conversation which both parties agree is accurate:

Heller & Partners, Ltd. Minute of telephone conversation. Call from National Provincial Bank, Ltd., 15, Bishopsgate,
E.C.2. 18.8.58. Person called: L. Heller. re Easipower, Ltd. They wanted to know in confidence, and without
responsibility on our part, the respectability and standing of Easipower, Ltd., and whether they would be good for an
advertising contract for 8,000 to 9,000. I replied the company recently opened an account with us. Believed to be
respectably constituted and considered good for its normal business engagements. The company is a subsidiary of Pena
Industries, Ltd., which is in liquidation, but we understand that the managing director, Mr. Williams, is endeavouring to
buy the shares of Easipower, Ltd., from the liquidator. We believe that the company would not undertake any
commitments they are unable to fulfil.

Some months later the appellants sought a further reference and on 7 November 1958, the city office of National Provincial
Bank Ltd wrote to the respondents in the following terms:

Dear Sir, We shall be obliged by your opinion in confidence as to the respectability and standing of Easipower, Ltd.,
27, Albemarle Street, London, W.1 and by stating whether you consider them trustworthy, in the way of business, to the
extent of 100,000 per annum, advertising contract. Yours faithfully,

On 11 November 1958, the respondents replied as follows:

CONFIDENTIAL
For your private use and without responsibility on the part of the bank or its officials.
Dear Sir, In reply to your inquiry of 7th instant. We beg to advise: Re E...................... Ltd. Respectably constituted
company, considered good for its ordinary business engagements. Your figures are larger than we are accustomed to see.
Yours faithfully, Per pro. Heller & Partners, Ltd.

National Provincial Bank communicated these replies to their customers, the appellants, and it is not suggested that this was
improper or not warranted by modern custom. The appellants relied on these statements and as a result they lost over 17,000
when Easipower Ltd went into liquidation.
The appellants now seek to recover this loss from the respondents as damages on the ground that these replies were given
negligently and in breach of the respondents duty to exercise care in giving them. In his judgment McNair J said:

On the assumption stated above as to the existence of the duty, I have no hesitation in holding (1) that Mr. Heller was
guilty of negligence in giving such a reference without making plainas he did notthat it was intended to be a very
guarded reference, and (2) that properly understood according to its ordinary and natural meaning the reference was not
justified by facts known to Mr. Heller.

Before your lordships the respondents were anxious to contest this finding but your lordships found it unnecessary to hear
argument on this matter being of opinion that the appeal must fail even if Mr Heller was negligent. Accordingly I cannot and do
not express any opinion on the question whether Mr Heller was in fact negligent. But I should make it plain that the appellants
complaint is not that Mr Heller gave his reply without adequate knowledge of the position, nor that he intended to create a false
impression, but that what he said was in fact calculated to create a false impression and that he ought to have realised that. The
same applies to the respondents letter of 11 November.
579
McNair J gave judgment for the respondents on the ground that they owed no duty of care to the appellants. He said:

I am accordingly driven to the conclusion by authority binding upon me that no such action lies in the absence of
contract or fiduciary relationship. On the facts before me there is clearly no contract, nor can I find a fiduciary
relationship. It was urged on behalf of the [appellants] that the facts that Easipower, Ltd. were heavily indebted to the
[respondents] and that the [respondents] might benefit from the advertising campaign financed by the [appellants], were
facts from which a special duty to exercise care might be inferred. In my judgment, however, these facts, though clearly
relevant on the question of honesty if this had been in issue, are not sufficient to establish any special relationship involving
a duty of care even if it was open to extend the sphere of special relationship beyond that of contract and fiduciary
relationship.

This judgment was affirmed by the Court of Appeal both because they were bound by authority and because they were not
satisfied that it would be reasonable to impose on a banker the obligation suggested.
Before coming to the main question of law it may be well to dispose of an argument that there was no sufficiently close
relationship between these parties to give rise to any duty. It is said that the respondents did not know the precise purpose of the
inquiries and did not even know whether National Provincial Bank Ltd wanted the information for its own use or for the use of a
customer: they knew nothing of the appellants. I would reject that argument. They knew that the inquiry was in connexion with
an advertising contract, and it was at least probable that the information was wanted by the advertising contractors. It seems to
me quite immaterial that they did not know who these contractors were: there is no suggestion of any speciality which could
have influenced them in deciding whether to give information or in what form to give it. I shall therefore treat this as if it were a
case where a negligent misrepresentation is made directly to the person seeking information, opinion or advice, and I shall not
attempt to decide what kind or degree of proximity is necessary before there can be a duty owed by the defendant to the plaintiff.
The appellants first argument was based on Donoghue (or McAlister) v Stevenson. That is a very important decision, but I
do not think that it has any direct bearing on this case. That decision may encourage us to develop existing lines of authority, but
it cannot entitle us to disregard them. Apart altogether from authority I would think that the law must treat negligent words
differently from negligent acts. The law ought so far as possible to reflect the standards of the reasonable man, and that is what
Donoghue (or McAlister) v Stevenson sets out to do. The most obvious difference between negligent words and negligent acts is
this. Quite careful people often express definite opinions on social or informal occasions, even when they see that others are
likely to be influenced by them; and they often do that without taking that care which they would take if asked for their opinion
professionally, or in a business connexion. The appellants agree that there can be no duty of care on such occasions, and we were
referred to American and South African authorities where that is recognised, although their law appears to have gone much
further than ours has yet done. But it is at least unusual casually to put into circulation negligently-made articles which are
dangerous. A man might give a friend a negligently-prepared bottle of home-made wine and his friends guests might drink it
with dire results; but it is by no means clear that those guests would have no action against the negligent manufacturer. Another
obvious difference is that a negligently-made article will only cause one accident, and so it is not very difficult to find the
necessary degree of proximity or neighbourhood between the 580 negligent manufacturer and the person injured. But words can
be broadcast with or without the consent or the foresight of the speaker or writer. It would be one thing to say that the speaker
owes a duty to a limited class, but it would be going very far to say that he owes a duty to every ultimate consumer who acts on
those words to his detriment. It would be no use to say that a speaker or writer owes a duty, but can disclaim responsibility if he
wants to. He, like the manufacturer, could make it part of a contract that he is not to be liable for his negligence: but that
contract would not protect him in a question with a third party at least if the third party was unaware of it.
So it seems to me that there is good sense behind our present law that in general an innocent but negligent misrepresentation
gives no cause of action. There must be something more than the mere misstatement. I therefore turn to the authorities to see
what more is required. The most natural requirement would be that expressly or by implication from the circumstances the
speaker or writer has undertaken some responsibility, and that appears to me not to conflict with any authority which is binding
on this House. Where there is a contract there is no difficulty as regards the contracting parties: the question is whether there is a
warranty. The refusal of English law to recognise any jus quaesitum tertio causes some difficulties, but they are not relevant here.
Then there are cases where a person does not merely make a statement, but performs a gratuitous service. I do not intend to
examine the cases about that, but at least they show that in some cases that person owes a duty of care apart from any contract,
and to that extent they pave the way to holding that there can be a duty of care in making a statement of fact or opinion which is
independent of contract.
Much of the difficulty in this field has been caused by Derry v Peek. The action was brought against the directors of a
company in respect of false statements in a prospectus. It was an action of deceit based on fraud and nothing else. But it was
held that the directors had believed that their statements were true although they had no reasonable grounds for their belief. The
Court of Appeal held that this amounted to fraud in law, but naturally enough this House held that there can be no fraud without
dishonesty and that credulity is not dishonesty. The question was never really considered whether the facts had imposed on the
directors a duty to exercise care. It must be implied that on the facts of that case there was no such duty. But that was
immediately remedied by the Directors Liability Act, 1890, which provided that a director is liable for untrue statements in a
prospectus unless he proves that he had reasonable ground to believe and did believe that they were true.
It must now be taken that Derry v Peek did not establish any universal rule that in the absence of contract an innocent but
negligent misrepresentation cannot give rise to an action. It is true that Lord Bramwell said ((1889), 14 App Cas at p 347): To
found an action for damages there must be a contract and breach, or fraud; and for the next twenty years it was generally
assumed that Derry v Peek decided that. But it was shown in this House in Nocton v Lord Ashburton that that is much too widely
stated. We cannot therefore now accept as accurate the numerous statements to that effect in cases between 1889 and 1914, and
we must now determine the extent of the exceptions to that rule.
In Nocton v Lord Ashburton a solicitor was sued for fraud. Fraud was not proved, but he was held liable for negligence.
Viscount Haldane LC dealt with Derry v Peek and pointed out ([191415] All ER Rep at p 49; [1914] AC at p 947) that while the
relationship of the parties in that case was not enough, the case did not decide

that where a different sort of relationship ought to be inferred from the circumstances, the case is to be concluded by
asking whether an action for deceit will lie There are other obligations besides that of honesty the breach of which may
give a right to damages. These obligations depend on 581 principles which the judges have worked out in the fashion that
is characteristic of a system where much of the law has always been judge-made and unwritten.

It hardly needed Donoghue v Stevenson to show that that process can still operate. Lord Haldane ([191415] All ER Rep at
p 50; [1914] AC at p 950) quoted a passage from the speech of Lord Herschell in Derry v Peek ((1889), 14 App Cas at p 360)
where he excluded from the principle of that case

those cases where a person within whose special province it lay to know a particular fact has given an erroneous
answer to an inquiry made with regard to it by a person desirous of ascertaining the fact for the purpose of determining his
course.

Then ([191415] All ER Rep at p 53; [1914] AC at p 954) he explained the expression constructive fraud and said ([191415]
All ER Rep at p 53; [1914] AC at p 954):

What it really means in this connexion is not moral fraud in the ordinary sense, but breach of the sort of obligation
which is enforced by a court which from the beginning regarded itself as a court of conscience.

He went on ([191415] All ER Rep at p 53; [1914] AC at p 955) to refer to breach of special duty and said ([191415] All ER
Rep at p 53; [1914] AC at p 955):
If such a duty can be inferred in a particular case of a person issuing a prospectus, as, for instance, in the case of
directors issuing to the shareholders of the company which they direct a prospectus inviting the subscription by them of
further capital, I do not find in Derry v. Peek an authority for the suggestion that an action for damages for
misrepresentation without an actual intention to deceive may not lie.

I find no dissent from these views by the other noble and learned Lords. Lord Shaw of Dunfermline also quoted the passage
which I have quoted from the speech of Lord Herschell, and, dealing with equitable relief, he approved ([191415] All ER Rep at
p 62; [1914] AC at p 971) a passage in an argument of Sir Roundell Palmer a:
________________________________________
a In Peek v Gurney (1873), LR 13 Eq 79, at p 97) which concluded ([191415] All ER Rep at p 62; [1914] AC at p 971

in order that a person may avail himself of relief founded on it he must show that there was such approximate relation
between himself and the person making the representation as to bring them virtually into the position of parties contracting
with each other;

an interesting anticipation in 1873 of the test of who is my neighbour.


Lord Haldane gave a further statement of his view in Robinson v National Bank of Scotland a case to which I shall return.
Having said that in that case there was no duty excepting the duty of common honesty, he went on to say (1916 SC (HL) at p
157):

In saying that I wish emphatically to repeat what I said in advising this House in the case of Nocton v. Lord Ashburton,
that it is a great mistake to suppose that, because the principle in Derry v. Peek clearly covers all cases of the class to which
I have referred, therefore the freedom of action of the courts in recognising special duties arising out of other kinds of
relationship which they find established by the evidence is in any way affected. I think, as I said in Noctons case, that an
exaggerated view was taken by a good many people of the scope of the decision in Derry v. Peek. The whole of the
doctrine as to fiduciary relationships, as to the duty of care arising from implied as well as express contracts, as to the duty
of care arising from other special relationships which the courts may find to exist in particular 582 cases, still remains, and
I should be very sorry if any word fell from me which should suggest that the courts are in any way hampered in
recognising that the duty of care may be established when such cases really occur.

This passage makes it clear that Lord Haldane did not think that a duty to take care must be limited to cases of fiduciary
relationship in the narrow sense of relationships which had been recognised by the Court of Chancery as being of a fiduciary
character. He speaks of other special relationships, and I can see no logical stopping place short of all those relationships where it
is plain that the party seeking information or advice was trusting the other to exercise such a degree of care as the circumstances
required, where it was reasonable for him to do that, and where the other gave the information or advice when he knew or ought
to have known that the inquirer was relying on him. I say ought to have known because in questions of negligence we now
apply the objective standard of what the reasonable man would have done.
A reasonable man, knowing that he was being trusted or that his skill and judgment were being relied on, would, I think,
have three courses open to him. He could keep silent or decline to give the information or advice sought: or he could give an
answer with a clear qualification that he accepted no responsibility for it or that it was given without that reflection or inquiry
which a careful answer would require: or he could simply answer without any such qualification. If he chooses to adopt the last
course he must, I think, be held to have accepted some responsibility for his answer being given carefully, or to have accepted a
relationship with the inquirer which requires him to exercise such care as the circumstances require.
If that is right then it must follow that Candler v Crane, Christmas & Co was wrongly decided. There the plaintiff wanted to
see the accounts of a company before deciding to invest in it. The defendants were the companys accountants and they were told
by the company to complete the companys accounts as soon as possible because they were to be shown to the plaintiff who was
a potential investor in the company. At the companys request the defendants showed the completed accounts to the plaintiff,
discussed them with him, and allowed him to take a copy. The accounts had been carelessly prepared and gave a wholly
misleading picture. It was obvious to the defendants that the plaintiff was relying on their skill and judgment and on their having
exercised that care which by contract they owed to the company, and I think that any reasonable man in the plaintiffs shoes
would have relied on that. This seems to me to be a typical case of agreeing to assume a responsibility: they knew why the
plaintiff wanted to see the accounts and why their employers, the company, wanted them to be shown to him, and agreed to show
them to him without even a suggestion that he should not rely on them.
The majority of the Court of Appeal held that they were bound by Le Lievre v Gould and that Donoghue v Stevenson had no
application. In so holding I think that they were right. The Court of Appeal have bound themselves to follow all rationes
decidendi of previous Court of Appeal decisions, and, in face of that rule, it would have been very difficult to say that the ratio in
Le Lievre v Gould did not cover Candlers case. Lord Denning, who dissented, distinguished Le Lievre v Gould on its facts, but,
as I understand the rule which the Court of Appeal have adopted, that is not sufficient if the ratio applies; and this is not an
appropriate occasion to consider whether the Court of Appeals rule is a good one. So the question which we now have to
consider is whether the ratio in Le Lievre v Gould can be supported. But before leaving Candlers case I must note that Cohen LJ
(as he then was), attached considerable importance to a New York decison Ultramares Corporation v Touche, a decision of
Cardozo CJ. But I think that another decision of that great judge, 583Glanzer v Shepard, is more in point because in the latter
case there was a direct relationship between the weigher who gave a certificate and the purchaser of the goods weighed, who the
weigher knew was relying on his certificate: there the weigher was held to owe a duty to the purchaser with whom he had no
contract. The Ultramares case can be regarded as nearer to Le Lievre v Gould.
In Le Lievre v Gould a surveyor, Gould, gave certificates to a builder, who employed him. The plaintiffs were mortgagees of
the builders interest and Gould knew nothing about them or the terms of their mortgage; but the builder, without Goulds
authority, chose to show them Goulds report. I have said that I do not intend to decide anything about the degree of proximity
necessary to establish a relationship giving rise to a duty of care, but it would seem difficult to find such proximity in this case
and the actual decision in Le Lievre v Gould may therefore be correct. The decision, however, was not put on that ground: if it
had been Cann v Willson would not have been overruled. Lord Esher MR held that there was no contract between the plaintiffs
and the defendant and that this House in Derry v Peek had ([1893] 1 QB at p 498) restated the old law that, in the absence of
contract, an action for negligence cannot be maintained when there is no fraud. Bowen LJ gave a similar reason: he said
([1893] 1 QB at p 501):

Then Derry v. Peek decided this further pointviz. that in cases like the present (of which Derry v. Peek was itself an
instance) there is no duty enforceable in law to be careful.

He added that the law of England ([1893] 1 QB at p 502)

does not consider that what a man writes on paper is like a gun or other dangerous instrument; and, unless he intended
to deceive, the law does not, in the absence of contract, hold him responsible for drawing his certificate carelessly.
So both he and Lord Esher held that Cann v Willson was wrong in deciding that there was a duty to take care. We now know on
the authority of Donoghue v Stevenson that Bowen LJ was wrong in limiting duty of care to guns or other dangerous instruments,
and I think that, for reasons which I have already given, he was also wrong in limiting the duty of care with regard to statements
to cases where there is a contract. On both points Bowen LJ was expressing what was then generally believed to be the law, but
later statements in this House have gone far to remove those limitations. I would therefore hold that the ratio in Le Lievre v
Gould was wrong and that Cann v Willson ought not to have been overruled.
Now I must try to apply these principles to the present case. What the appellants complain of is not negligence in the
ordinary sense of carelessness, but rather misjudgment in that Mr Heller, while honestly seeking to give a fair assessment, in fact
made a statement which gave a false and misleading impression of his customers credit. It appears that bankers now commonly
give references with regard to their customers as part of their business. I do not know how far their customers generally permit
them to disclose their affairs, but even with permission it cannot always be easy for a banker to reconcile his duty to his customer
with his desire to give a fairly balanced reply to an inquiry; and inquirers can hardly expect a full and objective statement of
opinion or accurate factual information such as skilled men would be expected to give in reply to other kinds of inquiry. So it
seems to me to be unusually difficult to determine just what duty, beyond a duty to be honest, a banker would be held to have
undertaken if he gave a reply without an adequate disclaimer of responsibility or other warning. It is in light of 584 such
considerations that I approach an examination of the case of Robinson v National Bank of Scotland.
It is not easy to extract the facts from the report of the case in the Court of Session. Several of the witnesses were held to be
unreliable and the principal issue in the case, fraud, is not relevant for present purposes. But the position appears to have been
this. Harley and two brothers Inglis wished to raise money. They approached an insurance company on the false basis that
Harley was to be the borrower and the Inglis brothers were to be guarantors. To satisfy the company as to the financial standing
of the Inglis brothers Harley got his London bank to write to McArthur, a branch agent of the National Bank of Scotland, and
McArthur on 28 July 1910, sent a reply which was ultimately held to be culpably careless but not fraudulent. Robinson, the
pursuer in the action, said that he had been approached by Harley to become a guarantor before the inquiry was made by Harley
but he was disbelieved by the Lord Ordinary who held that he was not brought into the matter before September. This was
accepted by the majority in the Inner House and there is no indication that any of their lordships in this House questioned the
finding that the letter of 28 July 1910, was not obtained on behalf of Robinson. Harley and the brothers Inglis did not proceed
with their scheme in July, but they resumed negotiations in September. The company wanted an additional guarantor and Harley
approached Robinson. A further reference was asked and obtained from McArthur on 1 October about the brothers Inglis, but no
point was made of this. The whole case turned on McArthurs letter of 28 July 1910. After further negotiation the company
made a loan to Harley with the brothers Inglis and Robinson as guarantors. Harley and the brothers Inglis all became bankrupt
and Robinson had to pay the company under his guarantee. Robinson sued the National Bank of Scotland and McArthur. He
alleged that McArthurs letter was fraudulent and that he had been induced by it to guarantee the loan. He also alleged that
McArthur had a duty to disclose certain facts about the brothers Inglis which were known to him, but this alternative case played
a very minor part in the litigation. Long opinions were given in the Court of Session on the question of fraud, but the alternative
case of a duty to disclose was dealt with summarily. The Lord Justice Clerk (Lord Scott Dickson) said (1916 SC at p 63):

It appears to me that there was no such duty of disclosure imposed upon Mr. McArthur towards the pursuer as would
justify us in applying the principle on which Noctons case was decided,

Lord Dundas referred (1916 SC at p 67) to cases of liability of a solicitor to his client for erroneous advice and of similar liability
arising from a fiduciary relationship and said such decisions seem to me to have no bearing on or application to the facts of the
present case. He also drew attention to the last sentence of the letter of 28 July 1910, which he said would become important if
fraud were out of the case. That sentence is: The above information is to be considered strictly confidential and is given on the
express understanding that we incur no responsibility whatever in furnishing it. Lord Salvesen, who dissented, did not deal with
the point and Lord Guthrie merely said (1916 SC at p 85) that here there was no fiduciary relationship. In this House an unusual
course was taken during the argument. I quote from the Session Cases report (1916 SC (HL) at pp 154, 155):

after counsel for the respondents had been heard for a short time EARL LOREBURN informed him that their
lordships, as at present advised, thought that there was no special duty on McArthur towards the pursuer; that the
respondents were not liable unless McArthurs representations were dishonest; and that their lordships had not been
satisfied as yet that the representations 585 were dishonest that under the circumstances the House was prepared to
dismiss the appeal; but that they considered that the pursuer had been badly treated, though he had not any cause of action
at law, and that, therefore, their lordships were disposed to direct that there should be no costs of the action on either side.
EARL LOREBURN said that counsel might prefer to argue the case further and endeavour to alter these views, but of
course he would run the risk of altering their lordships views as to the legal responsibility as well as upon the subject of
costs.

Counsel thenwisely no doubtsaid no more, and judgment was given for the bank but with no costs here or below.
That case is very nearly indistinguishable from the present case. Lord Loreburn regarded the fact that McArthur knew that
his letter might be used to influence others besides the immediate inquirer as entitling Robinson to found on it if fraud had been
proved. But it is not clear to me that he intended to decide that there would have been sufficient proximity between Robinson
and McArthur to enable him to maintain that there was a special relationship involving a duty of care if the other facts had been
sufficient to create such a relationship. I would not regard this as a binding decision on that question. With regard to the banks
duty Lord Haldane said (1916 SC (HL) at p 157):

There is only one other point about which I wish to say anything, and that is the question which was argued by the
appellant, as to there being a special duty of care under the circumstances here. I think the case of Derry v. Peek in this
House has finally settled in Scotland, as well as in England and Ireland, the conclusion that in a case like this no duty to be
careful is established. There is the general duty of common honesty, and that duty of course applies in the circumstances of
this case as it applies to all other circumstances. But when a mere inquiry is made by one banker of another, who stands in
no special relation to him, then, in the absence of special circumstances from which a contract to be careful can be inferred,
I think there is no duty excepting the duty of common honesty to which I have referred.

I think that by a contract to be careful Lord Haldane must have meant an agreement or undertaking to be careful. This was a
Scots case and by Scots law there can be a contract without consideration: Lord Haldane cannot have meant that similar cases in
Scotland and England would be decided differently on the matter of special relationship for that reason. I am, I think, entitled to
note that this was an extempore judgment. So Lord Haldane was contrasting a mere inquiry with a case where there are special
circumstances from which an undertaking to be careful can be inferred. In Robinsons case any such undertaking was excluded
by the sentence in McArthurs letter which I have quoted b and in which he said that the information was given on the express
understanding that we incur no responsibility whatever in furnishing it.
________________________________________
b Page 585, letter h, ante

It appears to me that the only possible distinction in the present case is that here there was no adequate disclaimer of
responsibility. Here, however, the appellants bank, who were their agents in making the enquiry, began by saying that they
wanted to know in confidence and without responsibility on our part, ie on the part of the respondents. So I cannot see how the
appellants can now be entitled to disregard that and maintain that the respondents did incur a responsibility to them.
The appellants founded on a number of cases in contract where very clear words were required to exclude the duty of care
which would otherwise have flowed from the contract. To that argument there are, I think, two answers. In the case of a contract
it is necessary to exclude liability for negligence, but in this case the 586 question is whether an undertaking to assume a duty to
take care can be inferred; and that is a very different matter. Secondly, even in cases of contract general words may be sufficient
if there was no other kind of liability to be excluded except liability for negligence: the general rule is that a party is not
exempted from liability for negligence unless adequate words are usedper Scrutton LJ in Rutter v Palmer ([1922] 2 KB 87 at
p 92; cf [1922] All ER Rep 367 at p 370). It being admitted that there was here a duty to give an honest reply, I do not see what
further liability there could be to exclude except liability for negligence: there being no contract there was no question of
warranty.
I am therefore of opinion that it is clear that the respondents never undertook any duty to exercise care in giving their
replies. The appellants cannot succeed unless there was such a duty and therefore in my judgment this appeal must be dismissed.

LORD MORRIS OF BORTH-Y-GEST (read by Lord Hodson). My Lords, the important question of law which has concerned
your lordships in this appeal is whether in the circumstances of the case there was a duty of care owed by the respondents, whom
I will call the bank, to the appellants, whom I will call Hedleys. In order to recover the damages which they claim Hedleys
must establish that the bank owed them a duty, that the bank failed to discharge such duty, and that as a consequence Hedleys
suffered loss.
An allegation of fraud was originally made but was abandoned. The learned judge held that the bank had been negligent,
but that they owed no duty to Hedleys to exercise care. The Court of Appeal agreed with the learned judge that no such duty was
owed, and it was therefore not necessary for them to consider whether the finding of negligence ought or ought not be upheld. In
your lordships House the legal issues were debated and again it did not become necessary to consider whether the finding of
negligence ought or ought not be upheld. It is but fair to the bank to state that they firmly contend that they were not in any way
negligent and that they were prepared to make submissions by way of challenge of the conclusions of the learned judge.
Hedleys were doing business with a company called Easipower, Ltd. In August, 1958, Hedleys wanted a bankers report
concerning that company who then had an account with the bank. [In November, 1957, Hedleys had received a report about the
company which had been given by another bank though not by direct communication.] Hedleys banked at a Piccadilly branch of
National Provincial Bank, Ltd. Hedleys asked that a report concerning Easipower Ltd should be obtained. The Piccadilly branch
communicated with the City office of their bank, the National Provincial. The National Provincial City office telephoned the
bank on 18 August 1958, and it is common ground that the representative of the National Provincial said that they wanted to
know in confidence and without responsibility on the part of the bank as to the respectability and standing of Easipower Ltd
and whether Easipower Ltd would be good for an advertising contract for 8/9,000. To that oral inquiry the bank then gave an
oral answer. In due course the answer then given was communicated by the Piccadilly branch of the National Provincial to
Hedleys. It was communicated orally and a letter of confirmation from that branch (dated 21 August 1958) was sent to Hedleys.
The letter had the headings Confidential and For your private use and without responsibility on the part of this bank or the
manager. The oral answer which the bank had given to the City office of the National Provincial was passed on with the
prefatory wordsIn reply to your telephoned inquiry of 18 August bankers say:. There was a later inquiry. On 4 November
1958, in a letter to the Piccadilly branch of the National Provincial, Hedleys wrote: I have been requested by the Directors to
again ask you to check the financial structure and status of Easipower Limited: Hedleys made some particular references and
concluded their letter with the words: I would be appreciative if you could make your check as 587 exhaustive as you
reasonably can. In a letter dated 7 November and headed Private and Confidential the City office of the National Provincial
asked the bank for their opinion in confidence as to the respectability and standing of Easipower, Ltd and asked the bank to
state whether they considered Easipower Ltd trustworthy, in the way of business, to the extent of 100,000 per annum,
advertising contract. The bank replied in a letter dated 11 November and sent to the City office of the National Provincial. The
letter had the headings Confidential and For your private use and without responsibility on the part of this Bank or its
officials. On 14 November the Piccadilly branch of the National Provincial wrote to Hedleys (heading their letter Confidential.
For your private use and without responsibility on the part of this Bank or the Manager) and, with the prefatory words: In reply
to your inquiry letter of 4 November Bankers say, passed on what the bank had stated in their letter to the City office of the
National Provincial.
It is, I think, a reasonable and proper inference that the bank must have known that the National Provincial were making
their inquiry because some customer of theirs was or might be entering into some advertising contract in respect of which
Easipower Ltd might become under a liability to such customer to the extent of the figures mentioned. The inquiries were from
one bank to another. The name of the customer (Hedleys) was not mentioned by the inquiring bank (National Provincial) to the
answering bank (the bank): nor did the inquiring bank (National Provincial) give to the customer (Hedleys) the name of the
answering bank (the bank). These circumstances do not seem to me to be material. The bank must have known that the inquiry
was being made by someone who was contemplating doing business with Easipower Ltd and that their answer or the substance of
it would in fact be passed on to such person. The conditions subject to which the bank gave their answers are important, but the
fact that the person to whom the answers would in all probability be passed on was unnamed and unknown to the bank is not
important for the purposes of a consideration of the legal issue which now arises. It is inherently unlikely that the bank would
have entertained a direct application from Hedleys asking for a report or would have answered an inquiry made by Hedleys
themselves: even if they had they would certainly have stipulated that their answer was without responsibility. The present
appeal does not raise any question as to the circumstances under which a banker is entitled (apart from direct authorisation) to
answer an inquiry. I leave that question as it was left by Atkin LJ in Tournier v National Provincial & Union Bank of England
when he said ([1923] All ER Rep 550 at p 561; [1924] 1 KB 461 at p 486):

I do not desire to express any final opinion on the practice of bankers to give one another information as to the affairs
of their respective customers except to say that it appears to me that, if it is justified, it must be upon the basis of an implied
consent of the customer.

The legal issue which arises is therefore whether the bank would have been under a liability to Hedleys if they had failed to
exercise care. This involves the questions whether the circumstances were such that the bank owed a duty of care to Hedleys, or
would have owed such a duty but for the words without responsibility, or whether they owed such a duty but were given a
defence by the words without responsibility, which would protect them if they had failed to exercise due care.
My lords, it seems to me that if A assumes a responsibility to B to tender him deliberate advice there could be a liability if
the advice is negligently given. I say could be because the ordinary courtesies and exchanges of life would become impossible
if it were sought to attach legal obligation to every kindly and friendly act. But the principle of the matter would not appear to be
in doubt. If A employs B (who might, for example, be a professional man such as an accountant or a solicitor or a doctor) for
reward to give advice, and if the advice is negligently 588 given, there could be a liability in B to pay damages. The fact that the
advice is given in words would not, in my view, prevent liability from arising. Quite apart, however, from employment or
contract there may be circumstances in which a duty to exercise care will arise if a service is voluntarily undertaken. A medical
man may unexpectedly come across an unconscious man, who is a complete stranger to him, and who is in urgent need of skilled
attention: if the medical man, following the fine traditions of his profession, proceeds to treat the unconscious man he must
exercise reasonable skill and care in doing so. In his speech in Banbury v Bank of Montreal Lord Atkinson said ([191819] All
ER Rep at p 18; [1918] AC at p 689):

It is well established that if a doctor proceeded to treat a patient gratuitously even in a case where the patient was
insensible at the time and incapable of employing him to do so, the doctor would be bound to exercise all the professional
skill and knowledge he possessed or professed to possess, and would be guilty of gross negligence if he omitted to do so.

To a similar effect were the words of Lord Loughborough in the much earlier case of Shiells v Blackburne when he said ((1789),
1 Hy Bl 158 at p 162):

If a man gratuitously undertakes to do a thing to the best of his skill, where his situation or profession is such as to
imply skill, an omission of that skill is imputable to him as gross negligence.

Compare also Wilkinson v Coverdale. I can see no difference of principle in the case of a banker. If someone who was not a
customer of a bank made a formal approach to the bank with a definite request that the bank would give him deliberate advice as
to certain financial matters of a nature with which the bank ordinarily dealt the bank would be under no obligation to accede to
the request: if however they undertook, though gratuitously, to give deliberate advice (I exclude what I might call casual and
perfunctory conversations) they would be under a duty to exercise reasonable care in giving it. They would be liable if they were
negligent although, there being no consideration, no enforceable contractual relationship was created.
In the absence of any direct dealings between one person and another, there are many and varied situations in which a duty
is owed by one person to another. A road user owes a duty of care towards other road users. They are his neighbours. A duty
was owed by the dock owner in Heaven v Pender. Under a contract with a shipowner he had put up a staging outside a ship in
his dock. The plaintiff used the staging because he was employed by a ship painter who had contracted with the shipowner to
paint the outside of the ship. The presence of the plaintiff was for business in which the dock owner was interested, and the
plaintiff was to be considered as having been invited by the dock owner to use the staging. The dock owner was therefore under
an obligation to take reasonable care that at the time when the staging was provided by him for immediate use it was in a fit state
to be used. For an injury which the plaintiff suffered, because the staging had been carelessly put up, he was entitled to succeed
in a claim against the defendant. The chemist in George v Skivington sold the bottle of hair wash to the husband knowing that it
was to be used by the wife. It was held on demurrer that the chemist owed a duty towards the wife to use ordinary care in
compounding the hair wash. In Donoghue (or McAlister) v Stevenson it was held that the manufacturer of an article of food,
medicine, or the like, was under a duty to the ultimate consumer to take reasonable care that the article was free from defect
likely to cause injury to health.
My lords, these are but familiar and well-known illustrations, which could be multiplied, which show that irrespective of
any contractual or fiduciary relationship and irrespective of any direct dealing, a duty may be owed by one person to 589 another.
It is said, however, that where careless (but not fraudulent) misstatements are in question there can be no liability in the maker of
them unless there is either some contractual or fiduciary relationship with a person adversely affected by the making of them or
unless through the making of them something is created or circulated or some situation is created which is dangerous to life, limb
or property. In logic I can see no essential reason for distinguishing injury which is caused by a reliance on words from injury
which is caused by a reliance on the safety of the staging to a ship, or by a reliance on the safety for use of the contents of a bottle
of hair wash or a bottle of some consumable liquid. It seems to me, therefore, that if A claims that he has suffered injury or loss
as a result of acting upon some misstatement made by B who is not in any contractual or fiduciary relationship with him the
inquiry that is first raised is whether B owed any duty to A: if he did the further inquiry is raised as to the nature of the duty.
There may be circumstances under which the only duty owed by B to A is the duty of being honest: there may be circumstances
under which B owes to A the duty not only of being honest but also a duty of taking reasonable care. The issue in the present
case is whether the bank owed any duty to Hedleys and if so what the duty was.
Leaving aside cases where there is some contractual or fiduciary relationship there may be many situations in which one
person voluntarily or gratuitously undertakes to do something for another person and becomes under a duty to exercise
reasonable care. I have given illustrations. Apart from cases where there is some direct dealing, there may be cases where one
person issues a document which should be the result of an exercise of the skill and judgment required by him in his calling and
where he knows and intends that its accuracy will be relied on by another. In this connexion it will be helpful to consider the case
of Cann v Willson. The owner of some property wished to obtain an advance of money on mortgage of the property and applied
to a firm of solicitors for the purpose of finding a mortgagee. Being informed by the solicitors that for the purpose of finding a
mortgagee he should have a valuation made of the property he consulted the defendants and asked them to make a valuation.
They surveyed and inspected the property and then made a valuation which they sent to the solicitors. The solicitors then
particularly called the defendants attention to the purpose for which the valuation was wanted and to the responsibility they were
undertaking. The defendants stated that their valuation was a moderate one and certainly was not made in favour of the borrower.
The valuation and representations so made by the defendants to the solicitors were communicated to the plaintiff (and a co-trustee
of his) by the solicitors. The plaintiff (and his co-trustee, who died before the commencement of the action) then advanced
money to the owner on the security of a mortgage of his property. Chitty J held on the evidence (a) that the defendants were
aware of the purpose for which the valuation was made and (b) that the valuation was sent by the defendants direct to the agents
of the plaintiff for the purpose of inducing the plaintiff and his co-trustee to lay out the trust money on mortgage. The owner
made default in payment and the property proved insufficient to answer the mortgage. The plaintiff alleged that the value of the
property was not anything like the value given by the defendants in their valuation. Chitty J held that the valuation as made was
in fact no valuation at all. In those circumstances the claim made was on the basis that the plaintiff has sustained loss through the
negligence, want of skill, breach of duty and misrepresentation of the defendants. Chitty J held the defendants liable. His
decision was principally based upon his finding that the defendants owed a duty of care to the plaintiff. It had been argued that
there was also liability in the defendants in contract (referred to in the judgment as the first ground) and on the ground of fraud
(referred to as the third ground). At the end of his judgment Chitty J said ((1888), 39 ChD at p 44):
590

I have entirely passed by the question of contract. It is unnecessary to decide that point. I consider on these two last
groundsand if I were to prefer one to the other it would be the second groundthat the defendant is liable for the
negligence.

In the course of his judgment he said ((1888), 39 ChD at p 43):

It is not necessary, in my opinion, to decide the case with reference to the third point, but even on the third point I
think the defendants are liableand that is what may be termed fraudulent misrepresentation.
Chitty J then (ie on 7 June 1888) referred to the judgment in the Court of Appeal in Peek v Derry. That judgment was reversed in
the House of Lords ((1889), 14 App Cas 337) on 1 July 1889. Chitty J compared the situation with that which arose in Heaven v
Pender. He pointed out that in that case there was ((1888), 39 ChD at p 42)

no contractual relation between the plaintiff and the dock owner, and there was no personal direct invitation to the
plaintiff to come and do the work on that ship, yet it was held that the dock owner had undertaken an obligation towards the
plaintiff, who was one of the persons likely to come and do the work to the vessel, and that he was liable to him and was
under an obligation to him to use due diligence in the construction of the staging.

Chitty J went on, therefore, to hold as the defendants had knowingly placed themselves in the position of sending their
valuation direct to the agents of the plaintiff for the purpose of inducing the plaintiff then they in point of law incurred a duty
towards him to use reasonable care in the preparation of the document. He likened the case to George v Skivington and
continued ((1888), 39 Ch D at p 43):

In this case the document supplied appears to me to stand upon a similar footing and not to be distinguished from that
case, as if it had been an actual article that had been handed over for the particular purpose of being so usued. I think,
therefore, that the defendants stood with regard to the plaintiffquite apart from any question of there being a contract or
not in the peculiar circumstances of this casein the position of being under an obligation or duty towards him.

My lords, I can see no fault or flaw in this reasoning and I am prepared to uphold it. If it is correct, then it is submitted that in the
present case the bank knew that some existing (though to them by name unknown) person was going to place reliance on what
they said and that accordingly they owed a duty of care to such person. I will examine this submission. Before doing so I must,
however, further consider Cann v Willson. It was overruled by the Court of Appeal in Le Lievre v Gould. The latter case, binding
on the Court of Appeal, in turn led to the decision in Candler v Crane, Christmas & Co. It is necessary therefore to consider the
reasons which governed the Court of Appeal in Le Lievre v Gould in overruling Cann v Willson. I do not propose to examine the
facts in Le Lievre v Gould: nor need I consider whether the result would have been no different had Cann v Willson not been
overruled. Lord Esher MR said ([1893] 1 QB at p 497):

But I do not hesitate to say that Cann v. Willson is not now law. CHITTY, J., in deciding that case acted upon an
erroneous proposition of law which has been since overruled by the House of Lords in Derry v. Peek ((1889), 14 App Cas
337.) when they restated the old law that, in the absence of contract, an action for negligence cannot be maintained when
there is no fraud.
591

Bowen LJ said ([1893] 1 QB at p 499) that he considered that Derry v Peek had overruled Cann v Willson. He considered that
Heaven v Pender gave no support for that decision, because it was no more than an instance of the class of case where one who,
having the conduct and control of premises which may injure those whom he knows will have a right to and will use them, owes
a duty to protect them. He said ([1893] 1 QB at p 501):

Then Derry v. Peek decided this further pointviz., that in cases like the present (of which Derry v. Peek was itself an
instance) there is no duty enforceable in law to be careful.

He followed the view expressed by Romer J in Scholes v Brook that the decision of the House of Lords in Derry v Peek by
implication negatived the existence of any such general rule as laid down in Cann v Willson. The reasoning of A L Smith LJ in
overruling Cann v Willson was on similar lines.
The inquiry is thus raised whether it was correct to say that Derry v Peek had either directly or at least by implication
overruled that part of the reasoning in Cann v Willson which led Chitty J to say that quite apart from contract and quite apart from
fraud there was a duty of care owed by the defendants to the plaintiffs. My lords, whatever views may have been held at one time
as to the effect of Derry v Peek, authoritative guidance as to this matter was given in your lordships House in 1914 in the case of
Nocton v Lord Ashburton. In his speech in that case Viscount Haldane LC said ([191415] All ER Rep at p 49; [1914] AC at p
947):

The discussion of the case by the noble and learned lords who took part in the decision appears to me to exclude the
hypothesis that they considered any other question to be before them than what was the necessary foundation of an
ordinary action for deceit. They must indeed be taken to have thought that the facts proved as to the relationship of the
parties in Derry v. Peek were not enough to establish any special duty arising out of that relationship other than the general
duty of honesty. But they do not say that where a different sort of relationship ought to be inferred from the circumstances
the case is to be concluded by asking whether an action for deceit will lie. I think that the authorities subsequent to the
decision of the House of Lords shew a tendency to assume that it was intended to mean more than it did. In reality the
judgment covered only a part of the field in which liabilities may arise. There are other obligations depend on principles
which the judges have worked out in the fashion that is characteristic of a system where much of the law has always been
judgmade and unwritten.

After a review of many authorities Lord Haldane said ([191415] All ER Rep at p 53; [1914] AC at p 955):

But side by side with the enforcement of the duty of universal obligation to be honest and the principle which gave the
right to rescission, the courts, and especially the Court of Chancery, had to deal with the other cases to which I have
referred, cases raising claims of an essentially different character, which have often been mistaken for actions of deceit.
Such claims raise the question whether the circumstances and relations of the parties are such as to give rise to duties of
particular obligation which have not been fulfilled.

Lord Haldane, pointed out that from the circumstances and relations of the parties a special duty may arise: there may be an
implied contract at law or a fiduciary obligation in equity. What Derry v Peek decided was that 592 the directors were under no
fiduciary duty to the public to whom they had addressed the invitation to subscribe. (I need not here refer to statutory enactments
since Derry v Peek). In his speech in the same case ([191415] All ER Rep at p 501; [1914] AC at p 959) Lord Dunedin pointed
out that there can be no negligence unless there is a duty, but that a duty may arise in many ways. There may be duties owing to
the world at large: alterum non laedere. There may be duties arising from contract. There may be duties which arise from a
relationship without the intervention of contract in the ordinary sense of the term, such as the duties of a trustee to his cestui que
trust or of a guardian to his ward.
Lord Shaw in his speech pointed out ([191415] All ER Rep at p 61; [1914] AC at p 970) that Derry v Peek

was an action wholly and solely of deceit, founded wholly and solely on fraud, was treated by this House on that
footing alone and that this being so what was decided was that fraud must ex necessitate contain the element of actual
moral delinquency. Certain expressions by learned lords may seem to have made incursions into the region of negligence,
but Derry v. Peek as a decision was directed to the single and specific point just set out.

Lord Shaw formulated the following principle ([191415] All ER Rep at p 62; [1914] AC at p 972):

Once the relations of parties have been ascertained to be those in which a duty is laid upon one person of giving
information or advice to another upon which that other is entitled to rely as the basis of a transaction, responsibility for
error amounting to misrepresentation in any statement made will attach to the adviser or informer although the information
and advice have been given, not fraudulently, but in good faith.

Lord Parmoor in his speech said ([1914] AC at p 978) in reference to Derry v Peek:

That case decides that in an action founded on deceit and in which deceit is a necessary factor, actual dishonesty,
involving mens rea, must be proved. The case, in my poinion, has no bearing whatever on actions founded on a breach of
duty in which dishonesty is not a necessary factor.

My lords, guided by the assistance given in Nocton v Ashburton I consider that it ought not to have been held in Le Lievre v
Gould that Cann v Willson was wrongly decided. Independently of contract there may be circumstances where information is
given or where advice is given which establish a relationship which creates a duty not only to be honest but also to be careful.
In his speech in Heilbut, Symons & Co v Buckleton Lord Moulton ([191113] All ER Rep 83 at p 92; [1913] AC 30 at p 51)
said that it was of the greatest importance to

maintain in its full integrity the principle that a person is not liable in damages for an innocent misrepresentation, no
matter in what way or under what form the attack is made.

That principle is, however, in no way impeached by recognition of the fact that if a duty exists there is a remedy for the breach of
it. As Lord Bowen said in Low v Bouverie ([1891] 3 Ch 82 at p 105):

the doctrine that negligent misrepresentation affords no cause of action is confined to cases in which there is no duty,
such as the law recognises, to be careful.

The inquiry in the present case, and in similar cases, becomes therefore an inquiry as to whether there was a relationship between
the parties which created a duty and if so whether such duty included a duty of care.
593
The guidance which Lord Haldane gave in Nocton v Ashburton was repeated by him in his speech in Robinson v National
Bank of Scotland. He clearly pointed out that Derry v Peek did not affect (a) the whole doctrine as to fiduciary relationships (b)
the duty of care arising from implied as well as express contracts and (c) the duty of care arising from other special relationships
which the courts may find to exist in particular cases.
My lords, I consider that it follows and that it should now be regarded as settled that if someone possessed of a special skill
undertakes, quite irrespective of contract, to apply that skill for the assistance of another person who relies on such skill, a duty of
care will arise. The fact that the service is to be given by means of, or by the instrumentality of, words can make no difference.
Furthermore if, in a sphere in which a person is so placed that others could reasonably rely on his judgment or his skill or on his
ability to make careful inquiry, a person takes it on himself to give information or advice to, or allows his information or advice
to be passed on to, another person who, as he knows or should know, will place reliance on it, then a duty of care will arise.
I do not propose to examine the facts of particular situations or the facts of recent decided cases in the light of this analysis,
but I proceed to apply it to the facts of the case now under review. As I have stated, I approach the case on the footing that the
bank knew that what they said would in fact be passed on to some unnamed person who was a customer of National Provincial
Bank, Ltd. The fact that it was said that they, ie National Provincial Bank Ltd wanted to know does not prevent this
conclusion. In these circumstances I think that some duty towards the unnamed person, whoever it was, was owed by the bank.
There was a duty of honesty. The great question, however, is whether there was a duty of care. The bank need not have
answered the inquiry from National Provincial Bank, Ltd. It appears, however, that it is a matter of banking convenience or
courtesy and presumably of mutual business advantage that inquiries as between banks will be answered. The fact that it is most
unlikely that the bank would have answered a direct inquiry from Hedleys does not affect the question as to what the bank must
have known as to the use that would be made of any answer that they gave but it cannot be left out of account in considering what
it was that the bank undertook to do. It does not seem to me that they undertook before answering an inquiry to expend time or
trouble in searching records, studying documents, weighing and comparing the favourable and unfavourable features and
producing a well-balanced and well-worded report. (I quote the words of Pearson LJ ([1961] 3 All ER at p 902, letter e; [1962]
1 QB at p 414)). Nor does it seem to me that the inquiring bank (nor therefore their customer) would expect such a process. This
was, I think, what was denoted by Lord Haldane in his speech in Robinson v National Bank of Scotland when he spoke of a
mere inquiry being made by one banker of another. In Parsons v Barclay & Co Ltd Cozens-Hardy MR expressed the view that
it was no part of a bankers duty, when asked for a reference, to make inquiries outside as to the solvency or otherwise of the
person asked about or to do more than answer the question put to him honestly from what he knew from the books and accounts
before him. There was in the present case no contemplation of receiving anything like a formal and detailed report such as might
be given by some concern charged with the duty (probably for reward) of making all proper and relevant inquiries concerning the
nature, scope and extent of a companys activities and of obtaining and marshalling all available evidence as to its credit,
efficiency, standing and business reputation. There is much to be said, therefore, for the view that if a banker gives a reference in
the form of a brief expression of opinion in regard to credit-worthiness he does not accept, and there is not expected 594 from
him, any higher duty than that of giving an honest answer. I need not, however, seek to deal further with this aspect of the matter,
which perhaps cannot be covered by any statement of general application, because in my judgment the bank in the present case,
by the words which they employed, effectively disclaimed any assumption of a duty of care. They stated that they only
responded to the inquiry on the basis that their reply was without responsibility. If the inquirers chose to receive and act upon the
reply they cannot disregard the definite terms upon which it was given. They cannot accept a reply given with a stipulation and
then reject the stipulati present case, by the words which they employed, effectively disclaimed any assumption of a duty of care.
They stated that they only responded to the inquiry on the basis that their reply was without responsibility. If the inquirers chose
to receive and act upon the reply they cannot disregard the definite terms upon which it was given. They cannot accept a reply
given with a stipulation and then reject the stipulation. Furthermore, within accepted principles (as illustrated in Rutter v Palmer
the words employed were apt to exclude any liability for negligence.
I would therefore dismiss the appeal.

LORD HODSON. My Lords, the appellants, who are advertising agents, claim damages for loss which they allege they have
suffered through the negligence of the respondents, who are merchant bankers. The negligence attributed to the respondents
consists of their failure to act with reasonable skill and care in giving references as to the credit-worthiness of a company called
Easipower Ltd which went into liquidation after the references had been given, so that the appellants were unable to recover the
bulk of the costs of advertising orders which Easipower Ltd had placed with them.
The learned judge at the trial found that the respondent bankers had been negligent in the advice which they gave in the form
of bankers references, the appellants being a company which acted in reliance on the references and suffered financial loss
accordingly, but that he must enter judgment for the respondents since there was no duty imposed by law to exercise care in
giving these references, the duty being only to act honestly in so doing. The respondents have at all times maintained that they
were in no sense negligent, and further that no damage flowed from the giving of references, but first they took the point that,
whether or no they were careless and whether or no the appellants suffered damage as a result of their carelessness, they must
succeed on the footing that no duty was owed by them. This point has been taken throughout as being, if the respondents are
right, decisive of the whole matter. I will deal with it first, although the underlying question is whether the respondent bankers,
who at all times disclaimed responsibility, ever assumed any duty at all. The appellants depend on the existence of a duty said to
be assumed by or imposed on the respondents when they gave a reference as to the credit-worthiness of Easipower Ltd knowing
that it would or might be relied on by the appellants or some other third party in like situation. The case has been argued first on
the footing that the duty was imposed by the relationship between the parties recognised by law as being a special relationship
derived either from the notion of proximity introduced by Lord Esher MR in Heaven v Pender, or from those cases firmly
established in our law which show that those who hold themselves out as possessing a special skill are under a duty to exercise it
with reasonable care.
The important case of Donoghue (or McAlister) v Stevenson shows that the area of negligence is extensive, for as Lord
Macmillan said ([1932] All ER Rep at p 30; [1932] AC at p 619):

The grounds of action may be as various and mainfold as human errancy, and the conception of legal responsibility
may develop in adaptation to altering social conditions and standards. The criterion of judgment must adjust and adapt
itself to the changing circumstances of life. The categories of negligence are never closed . Where there is room for
diversity of view is in determining what circumstances will establish such relationship between the parties as to give rise on
the one side to a duty to take care and on the other side to a right to have care taken.
595

In that case the necessary relationship was held to have been established where the manufacturer of an article, ginger beer in a
bottle, sold by him to a distributor in circumstances which prevented the distributor or the ultimate purchaser or consumer from
discovering by inspection any defect. He is under a legal duty to the ultimate purchaser or consumer to take reasonable care that
the article is free from injurious defect. No doubt that was the actual decision in that case and indeed it was thought by
Wrottesley J in Old Gate Estates Ltd v Toplis and Harding and Russell that he was precluded from awarding damages in tort for a
negligent valuation made by a firm of valuers, which knew it was to be used by the plaintiffs, since the doctrine of Donoghue v
Stevenson was confined to negligence which results in danger to life, limb or health. I do not think that this is the true view of
Donoghue v Stevenson, but the decision itself, although its effect has been extended to cases where there was no expectation as
contrasted with opportunity of inspection (see Grant v Australian Knitting Mills Ltd, and to liability of repairers (see Haseldine v
Daw & Son Ltd, has never been applied to cases where damages are claimed in tort for negligent statements producing damage.
The attempt so to apply it failed as recently as 1951 when in Candler v Crane, Christmas & Co the Court of Appeal by a majority
held that a false statement made carelessly as contrasted with fraudulently by one person to another, though acted on by that other
to his detriment, was not actionable in the absence of any contractual or fiduciary relationship between the parties and that this
principle had in no way been modified by the decision in Donoghue v Stevenson. Cohen LJ one of the majority of the court,
referred to the language of Lord Esher MR in Le Lievre v Gould who, repeating the substance of what he had said in Heaven v
Pender said ([1893] 1 QB at p 497; [1951] 1 All ER at p 445; [1951] 2 KB at p 199): If one man is near to another or is near to
the property of another, a duty lies upon him not to do that which may cause a personal injury to that other, or may injure his
property. Asquith LJ the other member of the majority of the court held that the neghbour doctrine had not been applied
where the damage complained of was not physical in its incidence to either person or property. The majority thus went no further
than Wrottesley J in the Old Gate Estates case save that injury to property was said to be contemplated by the doctrine expounded
in Donoghue v Stevenson. It is desirable to consider the reasons given by the majority for their decision in the Candler case for
the appellants rely on the dissenting judgment of Denning LJ in the same case. The majority, as also the learned trial judge, held
that they were bound by the decision of the Court of Appeal in Le Lievre v Gould in which the leading judgmet was given by
Lord Esher MR and referred to as authoritative by Lord Atkin in Donoghue v Stevenson.
It is true that Lord Esher MR refused to extend the proximity doctrine so as to cover the relationship between the parties in
that case and the majority in Candlers case were unable to draw a valid distinction between the facts of that case and the case of
Le Lievre v Gould. Denning LJ however, accepted the argument for the appellant which has been repeated before your lordships,
that the facts in Le Lievre v Gould were not such as to impose a liability, for the plaintiff mortgagees, who alleged that the
owners surveyor owed a duty to them, not only had the opportunity but also had stipulated for inspection by their own surveyor.
The defendants employee, who prepared the accounts in Candlers case, knew that the plaintiff was a potential investor in the
company of which the accounts were negligently prepared and that the 596 accounts were required in order that they might be
shown to the plaintiff. In these circumstances I agree with Denning LJ that there is a valid distinction between the two cases. In
Le Lievre v Gould it was held that an older case of Cann v Willson was overruled. That is a case where the facts were in pari
materia with those in Candlers case and Chitty J held that the defendants were liable because (a) they, independently of contract,
owed a duty to the plaintiff, which they failed to discharge, and (b) they had made reckless statements on which the plaintiff had
acted. This case was decided before this House in Derry v Peek overruled the Court of Appeal on the second proposition, but the
first proposition was untouched by Derry v Peek and, in so far as it depended on the authority of George v Skivington, the latter
case was expressly affirmed in Donoghue v Stevenson although it had often previously been impugned. It is true that, as Asquith
LJ pointed out in referring to George v Skivington, the hair wash put into circulation, knowing it was intended to be used by the
purchasers wife, was a negligently compounded hair wash, so that the case was so far on all fours with Donoghue v Stevenson,
but the declaration also averred that the defendant had said that the hair wash was safe. I cannot see that there is any valid
distinction in this field between a negligent statement, eg an incorrect label on a bottle, which leads to injury and a negligent
compounding of ingredients which leads to the same result. It may well be that at the time when Le Lievre v Gould was decided
the decision of this House in Derry v Peek was thought to go further than it did. It certainly decided that careless statements
recklessly but honestly made by directors in a prospectus issued to the public were not actionable on the basis of fraud and
inferentially that such statements would not be actionable in negligence (which had not in fact been pleaded), but it was pointed
out by this House in Nocton v Lord Ashburton that an action does lie for negligent misstatement where the circumstances disclose
a duty to be careful. It is necessary in this connexion to quote the actual language of Viscount Haldane LC ([191415] All ER
Rep at p 53; [1914] AC at pp 955, 956):

Such a special duty may arise from the circumstances and relations of the parties. These may give rise to an implied
contract at law or to a fiduciary obligation in equity. If such a duty can be inferred in a particular case of a person issuing a
prospectus, as, for instance, in the case of director issuing to the shareholders of the company which they direct a
prospectus inviting the subscription by them of further capital, I do not find in Derry v. Peek an authority for the suggestion
that an action for damages for misrepresentation without an actual intention to deceive may not lie. What was decided
there was that from the facts proved in that case no such special duty to be careful in statement could be inferred, and that
mere want of care therefore gave rise to no cause of action. In other words, it was decided that the directors stood in no
fiduciary relation, and, therefore, were under no fiduciary duty to the public to whom they had addressed the invitation to
subscribe. I have only to add that the special relationship must, whenever it is alleged, be clearly shown to exist.

So far I have done no more than summarise the argument addressed to the Court of Appeal in Candlers case to which effect
was given in the dissenting judgment of Denning LJ with which I respectfully agree in so far as it dealt with the facts of that case.
I am, therefore, of opinion that his judgment is to be preferred to that of the majority, although the opinion of the majority is
undoubtedly supported by the ratio decidendi of Le Lievre v Gould, which they cannot be criticised for following. This, however,
does not carry the 597 appellants further than this, that, provided that they can establish a special duty, they are entitled to
succeed in an action based on breach of that duty.
I shall later refer to certain cases which support the view that apart from what are usually called fiduciary relationships such
as those between trustee and cestui que trust, solicitor and client, parent and child or guardian and ward there are other
circumstances in which the law imposes a duty to be careful, which is not limited to a duty to be careful to avoid personal injury
or injury to property but covers a duty to avoid inflicting pecuniary loss provided always that there is a sufficiently close
relationship to give rise to a duty of care. The courts of equity recognised that a fiduciary relationship exists in almost every
shape, to quote from Field J in Plowright v Lambert ((1885), 52 LT 646 at p 652). He went on to refer to a case c, which had
said that the relationship could be created voluntarily, as it were, by a person coming into a state of confidential relationship with
another by offering to give advice in a matter, and so being disabled thereafter from purchasing.
________________________________________
c Tate v Williamson (1866) 2 Ch App 55

It is difficult to see why liability as such should depend on the nature of the damage. Lord Roche in Morrison S S Co Ltd v
Greystoke Castle (Cargo Owners) ([1946] 2 All ER 696, at p 700; [1947] AC 265 at p 280) instanced damage to a lorry by the
negligence of the driver of another lorry which while it does no damage to the goods in the second lorry causes the goods owner
to be put to expense which is recoverable by direct action against the negligent driver.
It is not to be supposed that the majority of the Court of Appeal, who decided as they did in Candlers case, were unmindful
of the decision in Nocton v Lord Ashburton, to which their attention was drawn, but they seem to have been impressed with the
view that, in the passage which I have quoted, Lord Haldane had in mind only fiduciary relationships in the strict sense, but in my
opinion the words need not be so limited. I am fortified in this opinion by examples to be found in the old authorities such as
Shiells v Blackburne, Wilkinson v Coverdale and Gladwell v Steggall, which are illustrations of cases where the law has held that
a duty to exercise reasonable care (breach of which is remediable in damages) has been imposed in the absence of a fiduciary
relationship where persons hold themselves out as possessing special skill and are thus under a duty to exercise it with reasonable
care. The statement of Lord Loughborough in Shiells v Blackburne is always accepted as authoritative and ought not to be
dismissed as dictum, although the plaintiff failed to establish facts which satisfied the standard he set. He said ((1789), 1 Hy Bl
at p 162.):

If a man gratuitously undertakes to do a thing to the best of his skill, where the situatio or profession is such as to
imply skill, an omission of that skill is imputable to him as gross negligence.

True that proximity is more difficult to establish where words are concerned than in the case of other activities and mere casual
observations are not to be relied on, see Fish v Kelly, but these matters go to difficulty of proof rather than principle.
A modern instance is to be found in the case of Woods v Martins Bank Ltd where Salmon J held that on the facts of the case
the defendant bank, which had held itself out as being advisers on investments (which was within the scope of its business) and
had not given the plaintiff reasonably careful or skilful advice, so that he suffered loss, was in breach of duty and so liable in
damages, even though the plaintiff might not have been a customer of the bank at the 598 material time. True that the learned
judge based this part of his conclusion on a fiduciary relationship which he held to exist between the plaintiff and the bank and
thus brought himself within the scope of the decision in Candlers case by which he was bound. For my part I should have
thought that even if the learned judge put a strained interpretation on the word fiduciary, which is based on the idea of trust, the
decision can be properly sustained as an example involving a special relationship.
I do not overlook the point forcefully made by Harman LJ in his judgment ([1961] 3 All ER at pp 902, 903; [1962] 1 QB at
p 415), and elaborated by counsel for the respondent before your lordships, that it may in certain cases appear to be strange that
whereas innocent misrepresentation does not sound in damages yet in the special cases under consideration an injured party may
sue in tort a third party whose negligent misrepresentation has induced him to enter into the contract. As was pointed out by Lord
Wrenbury, however, in Banbury v Bank of Montreal ([191819] All ER Rep 1 at p 28; [1918] AC 626 at p 713), innocent
misrepresentation is not the cause of action but evidence of the negligence which is the cause of action.
Was there then a special relationship here? I cannot exclude from consideration the actual terms in which the reference was
given and I cannot see how the appellants can get over the difficulty which these words put in their way. They cannot say that the
respondents are seeking to, as it were, contract out of their duty by the use of language which is insufficient for the purpose, if the
truth of the matter is that the respondents never assumed a duty of care nor was such a duty imposed on them.
The first question is whether a duty was ever imposed and the language used must be considered before the question can be
answered. In the case of a person giving a reference I see no objection in law or morals to the giver of the reference protecting
himself by giving it without taking responsibility for anything more than the honesty of his opinion which must involve without
taking responsibility for negligence in giving that opinion. I cannot accept the contention of the appellants that the responsibility
disclaimed was limited to the bank to which the reference was given, nor can I agree that it referred only to responsibility for
accuracy of detail. Similar words were present in the case of Robinson v National Bank of Scotland (1916 SC (HL) 154 at p
159), a case in which the facts cannot, I think, be distinguished in any material respect from this. Moreover in the Inner House
the words of disclaimer were, I think, treated as not without significance. In this House the opinion was clearly expressed that
the representations made were careless, inaccurate and misleading, but that the pursuer had no remedy since there was no special
duty on the banks representative towards the pursuer. This conclusion was reached quite apart from the disclaimer of
responsibility contained in the defender banks letters. Viscount Haldane LC recalled the case of Nocton v Lord Ashburton in the
following passage (1916 SC (HL) at p 157):

In saying that I wish emphatically to repeat what I said in advising this House in the case of Nocton v. Lord Ashburton
that it is a great mistake to suppose that, because the principle in Derry v. Peek clearly covers all cases of the class to which
I have referred, therefore the freedom of action of the courts in recognising special duties arising out of other kinds of
relationship which they find established by the evidence is in any way affected. I think, as I said in Noctons case, that an
exaggerated view was taken by a good many people of the scope of the decision in Derry v. Peek. The whole of the
doctrine as to fiduciary relationships, as to the duty of care arising from implied as well as express contracts, as to the 599
duty of care arising from other special relationships which the courts may find to exist in particular cases, still remains, and
I should be very sorry if any word fell from me which should suggest that the courts are in any way hampered in
recognising that the duty of care may be established when such cases really occur.
This authority is, I think, conclusive against the appellants, and is not effectively weakened by the fact that the case came to an
end, before the matter had been fully argued, on the House intimating that it was prepared to dismiss the appeal without costs on
either side, since the pursuer had in its opinion been badly treated. Since no detailed reasons were given by the House for the
view that a bankers reference given honestly does not in the ordinary course carry with it a duty to take reasonable care, that
duty being based on a special relationship, it will not, I hope, be out of place if I express my concurrence with the observations of
Pearson LJ who delivered the leading judgment in the Court of Appeal and said ([1961] 3 All ER at p 902; [1962] 1 QB at pp
414, 415):

Apart from authority, I am not satisfied that it would be reasonable to impose on a banker the obligation suggested, if
that obligation really adds anything to the duty of giving an honest answer. It is conceded by counsel for the plaintiffs that
the banker is not expected to make outside inquiries to supplement the information which he already has. Is he then
expected, in business hours in the banks time, to expend time and trouble in searching records, studying documents,
weighing and comparing the favourable and unfavourable features and producing a well-balanced and well-worded report?
That seems wholly unreasonable. Then, if he is not expected to do any of those things, and if he is permitted to give an
impromptu answer in the words that immediately come to his mind on the basis of the facts which he happens to remember
or is able to ascertain from a quick glance at the file or one of the files, the duty of care seems to add little, if anything, to
the duty of honesty. If the answer given is seriously wrong, that is some evidenceof course, only some evidenceof
dishonesty. Therefore, apart from authority, it is to my mind far from clear that the banker, in answering such an inquiry,
could reasonably be supposed to be assuming any duty higher than that of giving an honest answer.

This is to the same effect as the opinion of Cozens-Hardy MR in Parsons v Barclay & Co Ltd ((1910), 26 TLR 628 at p 629;
cf [190810] All ER Rep 429 at pp 432, 433) cited as follows:

His Lordship said he wished emphatically to repudiate the suggestion that, when a banker was asked for a reference of
this kind, it was any part of his duty to make inquiries outside as to the solvency or otherwise of the person asked about, or
to do anything more than answer the question put to him honestly from what he knew from the books and accounts before
him. To hold otherwise would be a very dangerous thing to do and would put an end to a very wholesome and useful
practice and long established custom which was now largely followed by bankers.

It would, I think, be unreasonable to impose an additional burden on persons, such as bankers, who are asked to give references
and might, if more than honesty were required, be put to great trouble before all available material had been explored and
considered.
It was held in Low v Bouverie that if a trustee takes on himself to answer the inquiries of a stranger about to deal with the
cestui que trust, he is not under a legal obligation to do more than to give honest answers to the best of his actual knowledge and
belief, he is not bound to make inquiries himself. I do not think that a banker giving references in the ordinary exercise of
business should be in any worse position than the trustee. I have already pointed out 600 that a banker like anyone else may find
himself involved in a special relationship involving liability, as in Woods v Martins Bank Ltd, but there are no special features
here which enable the appellants to succeed.
I do not think that it is possible to catalogue the special features which must be found to exist before the duty of care will
arise in a given case, but since preparing this opinion I have had the opportunity of reading the speech which my noble and
learned friend Lord Morris Of Borth-Y-Gest has now delivered. I agree with him that if in a sphere where a person is so placed
that others could reasonably rely on his judgment or his skill or on his ability to make careful inquiry such person takes it on
himself to give information or advice to, or allows his information or advice to be passed on to, another person who, as he knows,
or should know, will place reliance on it, then a duty of care will arise.
I would dismiss the appeal.

LORD DEVLIN (read by Lord Pearce): My Lords, the facts of this case, stated sufficiently to raise the general point of law, are
these. The appellants, being anxious to know whether they could safely extend credit to certain traders with whom they were
dealing, sought a bankers reference about them. For this purpose their bank, National Provincial Bank Ltd approached the
respondents who are the traders bank. The respondents gave, without making any charge for it and in the usual way, a reference
which was so carelessly phrased that it led the appellants to believe the traders to be creditworthy when in fact they were not.
The appellants seek to recover from the respondents the consequent loss.
Counsel for the respondents has given your lordships three reasons why the appellants should not recover. The first is
founded on a general statement of the law which, if true, is of immense effect. Its hypothesis is that there is no general duty not
to make careless statements. No one challenges that hypothesis. There is no duty to be careful in speech, as there is a duty to be
honest in speech. Nor indeed is there any general duty to be careful in action. The duty is limited to those who can establish
some relationship of proximity such as was found to exist in Donoghue (or McAlister) v Stevenson. A plaintiff cannot therefore
recover for financial loss caused by a careless statement unless he can show that the maker of the statement was under a special
duty to him to be careful. Counsel submits that this special duty must be brought under one of three categories. It must be
contractual; or it must be fiduciary; or it must arise from the relationship of proximity, and the financial loss must flow from
physical damage done to the person or the property of the plaintiff. The law is now settled, counsel submits, and these three
categories are exhaustive. It was so decided in Candler v Crane, Christmas & Co and that decision, counsel submits, is right in
principle and in accordance with earlier authorities.
Counsel for the appellants agrees that outside contractual and fiduciary duty there must be a relationship of proximitythat
is Donoghue v Stevensonbut he disputes that recovery is then limited to loss flowing from physical damage. He has not been
able to cite a single case in which a defendant has been held liable for a careless statement leading, otherwise than through the
channel of physical damage, to financial loss; but he submits that in principle such loss ought to be recoverable and that there is
no authority which prevents your lordships from acting on that principle. Unless counsel for the appellants can persuade your
lordships of this, his case fails at the outset. This therefore is the first and the most fundamental of the issues which the House is
asked to decide.
Counsel for the respondents second reason is that, if it is open to your lordships to declare that there are or can be special or
proximate relationships outside the categories he has named, your lordships cannot formulate one to fit the case of a banker who
gives a reference to a third party who is not his customer; and he 601 contends that your lordships have already decided that point
in Robinson v National Bank of Scotland. His third reason is that if there can be found in cases such as this a special relationship
between bankers and third parties, on the facts of the present case the appellants fall outside it; and here he relies particularly on
the fact that the reference was marked Strictly confidential and given on the express understanding that we incur no
responsibility whatever in furnishing it.
My lords, I approach the consideration of the first and fundamental question in the way in which Lord Atkin approached the
same sort of questionthat is, in essence the same sort, though in particulars very differentin Donoghue v Stevenson. If
counsel for the respondents proposition is the result of the authorities, then, as Lord Atkin said ([1932] All ER Rep at p 12;
[1932] AC at p 582):
I should consider the result a grave defect in the law and so contrary to principle that I should hesitate long before
following any decision to that effect which had not the authority of this House.

So before I examine the authorities, I shall explain why I think that the law, if settled as counsel for the respondents says that it is,
would be defective. As well as being defective in the sense that it would leave a man without a remedy where he ought to have
one and where it is well within the scope of the law to give him one, it would also be profoundly illogical. The common law is
tolerant of much illogicality especially on the surface; but no system of law can be workable if it has not got logic at the root of it.
Originally it was thought that the tort of negligence must be confined entirely to deeds and could not extend to words. That
was supposed to have been decided by Derry v Peek. I cannot imagine that anyone would now dispute that, if this were the law,
the law would be gravely defective. The practical proof of this is that the supposed deficiency was, in relation to the facts in
Derry v Peek, immediately made good by Act of Parliament. Today it is unthinkable that the law could permit directors to be as
careless as they liked in the statements that they made in a prospectus.
A simple distinction between negligence in word and negligence in deed might leave the law defective but at least it would
be intelligible. This is not, however, the distinction that is drawn in counsel for the respondents argument and it is one which
would be unworkable. A defendant who is given a car to overhaul and repair if necessary is liable to the injured driver (a) if he
overhauls it and repairs it negligently and tells the driver that it is safe when it is not; (b) if he overhauls it and negligently finds it
not to be in need of repair and tells the driver that it is safe when it is not; and (c) if he negligently omits to overhaul it at all and
tells the driver that it is safe when it is not. It would be absurd in any of these cases to argue that the proximate cause of the
drivers injury was not what the defendant did or failed to do but his negligent statement on the faith of which the driver drove
the car and for which he could not recover. In this type of case where if there were a contract there would undoubtedly be a duty
of service, it is not practicable to distinguish between the inspection or examination, the acts done or omitted to be done, and the
advice or information given. So neither in this case nor in Candler v Crane, Christmas & Co (Denning LJ noted the point
([1951] 1 All ER at p 432; [1951] 2 KB at p 179) when he gave the example of the analyst who negligently certifies food to be
harmless) has counsel for the respondents argued that the distinction lies there.
This is why the distinction is now said to depend on whether financial loss is caused through physical injury or whether it is
caused directly. The interposition of the physical injury is said to make a difference of principle. I can find neither logic nor
commonsense in this. If irrespective of contract, a doctor negligently advises a patient that he can safely pursue his occupation
and he cannot and the 602 patients health suffers and he loses his livelihood, the patient has a remedy. But if the doctor
negligently advises him that he cannot safely pursue his occupation when in fact he can and he loses his livelihood, there is said
to be no remedy. Unless, of course, the patient was a private patient and the doctor accepted half a guinea for his trouble: then
the patient can recover all. I am bound to say, my lords, that I think this to be nonsense. It is not the sort of nonsense that can
arise even in the best system of law out of the need to draw nice distinctions between borderline cases. It arises, if it is the law,
simply out of a refusal to make sense. The line is not drawn on any intelligible principle. It just happens to be the line which
those who have been driven from the extreme assertion that negligent statements in the absence of contractual or fiduciary duty
give no cause of action have in the course of their retreat so far reached.
I shall now examine the relevant authorities and your lordships will, I hope, pardon me if, with one exception, I attend only
to those that have been decided in this House, for I have made it plain that I will not in this matter yield to persuasion but only to
compulsion. The exception is the case of Le Lievre v Gould, for your lordships will not easily upset decisions of the Court of
Appeal if they have stood unquestioned for as long as seventy years. The five relevant decisions of this House are Derry v Peek,
Nocton v Lord Ashburton, Robinson v National Bank of Scotland, Donoghue v Stevenson, and Morrison S S Co Ltd v Greystoke
Castle (Cargo Owners). The last of these I can deal with at once for it lies outside the main stream of authority on this point. It
is a case in which damage was done to a ship as the result of a collision with another ship. The owners of cargo on the first ship,
which cargo was not itself damaged, thus became liable to the owners of the first ship for a general average contribution. They
sued the second ship as being partly to blame for the collision. Thus they were claiming for the financial loss caused to them by
having to make the general average contribution although their property sustained no physical damage. This House held that they
could recover. Their lordships did not in that case lay down any general principle about liability for financial loss in the absence
of physical damage; but the case itself makes it impossible to argue that there is any general rule showing that such loss is of its
nature irrecoverable.
I turn back to the earlier authorities beginning with Derry v Peek. The facts in this case are so well known that I need not
state them again. Nor need I state in my own words the effect of the decision. That has been done authoritatively by this House
in Nocton v Lord Ashburton. I quote Viscount Haldane LC as stating most comprehensively the limits of the decision (nothing
that his view of the case is fully supported by Lord Shaw ([191415] All ER Rep at p 61; [1914] AC at p 970) and Lord Parmoor
([1914] AC at p 978)) as follows ([191415] All ER Rep at p 49; [1914] AC at p 947):

The discussion of the case by the noble and learned Lords who took part in the decision appears to me to exclude the
hypothesis that they considered any other question to be before them than what was the necessary foundation of an
ordinary action for deceit. They must indeed be taken to have thought that the facts proved as to the relationship of the
parties in Derry v. Peek were not enough to establish any special duty arising out of that relationship other than the general
duty of honesty. But they do not say that where a different sort of relationship ought to be inferred from the circumstances
the case is to be concluded by asking whether an action for deceit will lie.

There was in Derry v Peek, as the report of the case shows, no plea of 603 innocent or negligent misrepresentation and so their
lordships did not make any pronouncement on that. I am bound to say that had there been such a plea I am sure that the House
would have rejected it. As Lord Haldane said, their lordships must be taken to have thought that there was no liability in
negligence. But what their lordships may be taken to have thought, though it may exercise great influence on those who
thereafter have to form their own opinion on the subject, is not the law of England. It is impossible to say how their lordships
would have formulated the principle if they had laid one down. They might have made it general or they might have confined it
to the facts of the case. They might have made an exception of the sort indicated by Lord Herschell ((1889), 14 App Cas at p
360) or they might not. This is speculation. All that is certain is that on this point the House laid down no law at all.
Clearly in Le Lievre v Gould it was thought that the House had done so. Lord Esher MR ([1893] 1 QB at p 498), treated
Derry v Peek as restating the old law that, in the absence of contract, an action for negligence cannot be maintained when there
is no fraud. A L Smith LJ stated the law in the same way ([1893] 1 QB at p 504). This is wrong and the House in effect said so
in Nocton v Lord Ashburton.
My lords, I need not consider how far thereafter a court of equal authority was bound to follow Le Lievre v Gould. It may be
that the decision on the facts was correct even though the reasoning was too wide. There has been a difference of opinion about
the effect of the decision: compare Asquith LJ in Candler v Crane, Christmas & Co ([1951] 1 All ER at 426 at p 441; [1951] 2
KB 164 at p 193) with Denning LJ ([1951] 1 All ER at p 434; [1951] 2 KB at p 181). Nor need I consider what part of the
reasoning, if any, should be held to survive Nocton v Lord Ashburton. It is clear that after 1914 it would be to Nocton v Lord
Ashburton and not to Le Lievre v Gould that the lawyer would look in order to ascertain what the exceptions were to the general
principle that a man is not liable for careless misrepresentation. I cannot feel, therefore, that there is any principle enunciated in
Le Lievre v Gould which is now so deeply embedded in the law that your lordships ought not to disturb it.
I come now to the case of Nocton v Lord Ashburton which both sides put forward as the most important of the authorities
which your lordships have to consider. The appellants say that it removed the restrictions which Derry v Peek was thought to
have put on liability for negligent misrepresentation. The respondents say that it removed those restrictions only to a very limited
extent, that is to say, by adding fiduciary obligation to contract as a source of special duty; and that it closed the door on any
further expansion. I propose, therefore, to examine it with some care because it is not at all easy to determine exactly what it
decided. Lord Haldane began his speech by saying ([191415] All ER Rep at p 47; [1914] AC at p 943):

Owing to the mode in which this case has been treated both by the learned judge who tried it and by the Court of
Appeal, the question to be decided has been the subject of some uncertainty and much argument.

He went on to say that the difficulties in giving relief were concerned with form and not with substance. The main difficulty, I
think, lies in discovering from the statement of claim what the cause of action was. Lord Ashburton sought relief from the
consequences of having advanced money on mortgage to several persons of whom the defendant Nocton was one. The statement
of claim consists of a long narrative of events interspersed with complaints. Although in the end the vital fact was that Nocton
was Ashburtons solicitor, there is no allegation of any retainer and nothing is pleaded in contract. The fact that Nocton was a
solicitor emerges only in the framing of the complaint in para 13 where it was said that Noctons advice to make the advance of
65,000 was not that of a solicitor 604 advising his client in good faith but was given for his own private ends. The relief
asked for in respect of this transaction is a declaration that the plaintiff was improperly advised and induced by the defendant
Nocton whilst acting as the plaintiffs confidential solicitor to advance 65,000. In para 31 to para 33 of the statement of claim
it is related that the plaintiff was asked to release part of his security for the loan; and it is said that the defendant Nocton in
advising the plaintiff to execute the said release allowed the plaintiff to believe that he was advising the plaintiff independently
and in good faith and in the plaintiffs interest. No separate relief was sought in respect of this transaction.
Until the case reached this House no substantial point of law was raised. Neville J at the trial held that the only issue raised
by the statement of claim was whether the defendant Nocton was guilty of fraud and that the plaintiff had failed to prove it. The
Court of Appeal agreed with the judges view of the pleadings. Cozens-Hardy MR said that if damages had been claimed on the
ground of negligence, the action would have been practically undefended. But it was then too late to amend the statement of
claim if only because a new cause of action would have been statute-barred. On the facts the Court of Appeal reversed in part the
judges finding of fraud, holding that there was fraud in relation to the release.
In this House at the conclusion of the appellants argument the respondents counsel was told that the House was unlikely to
differ from the judgment of Neville J on fraud. The pith of the respondents argument is reported as follows ([1914] AC at p
943):

Assuming that fraud is out of the question, the allegations in the statement of claim are wide enough to found a claim
for dereliction of duty by a person occupying a fiduciary relation. In the old cases in equity the term fraud was frequently
applied to cases of a breach of fiduciary obligation.

He was then stopped.


It can now be understood why Lord Haldane regarded the question as one of form rather than of substance. The first
question which the House had to consider was whether the statement of claim was wide enough to cover negligence. Lord
Parmoor thought that it was and ([1914] AC at p 978; [191415] All ER Rep at p 63) decided the appeal on that ground. So I
think in the end did Lord Dunedin ([191415] All ER Rep at p 58; [1914] AC at p 965), but he also expressed his agreement with
the opinion of Lord Haldane. Lord Haldane, with whom Lord Atkinson concurred, thought that possibly negligence was covered
but he did not take the view that the statement of claim must be interpreted either as an allegation of deceit or as an allegation of
negligence. He said ([191415] All ER Rep at pp 48, 49; [1914] AC at p 946):

There is a third form of procedure to which the statement of claim approximated very closely, and that is the old bill in
Chancery to enforce compensation for breach of a fiduciary obligation. There appears to have been an impression taht the
necessity which recent authorities have established of proving moral fraud in order to succeed in an action of deceit has
narrowed the scope of this remedy. For the reasons which I am about to offer to your lordships, I do not think that this is
so.

The Lord Chancellor then went on to examine Derry v Peek in order to determine exactly what it had decided.
I find most interest for present purposes in the speech of Lord Shaw of Dunfermline. He held that the pleadings disclosed
([191415] All ER Rep at p 59; [1914] AC at p 967)

a claim for liability upon a ground quite independent of fraud, namely, 605of misrepresentations and misstatements
made by a person entrusted with a duty to another, and in failure of that duty.

He posed what he considered to be the crucial question ([191415] All ER Rep at p 60; [1914] AC at p 968): What was the
relation in which the parties stood to each other at the time of the transaction. He stated ([191415] All ER Rep at p 60; [1914]
AC at p 969) that the defendant was Lord Ashburtons solicitor and so under a duty to advise. He concluded in the following
terms ([191415] All ER Rep at p 62; [1914] AC at p 972):

Once the relations of parties have been ascertained to be those in which a duty is laid upon one person of giving
information or advice to another upon which that other is entitled to rely as the basis of a transaction, responsibility for
error amounting to misrepresentation in any statement made will attach to the advisor or informer, although the information
and advice have been given not fraudulently, but in good faith. It is admitted in the present case that misrepresentations
were made; that they were material; that they were the cause of the loss; that they were made by a solicitor to his client in a
situation in which the client was entitled to rely, and did rely, upon the information received. I, accordingly, think that that
situation is plainly open for the application of the principle of liability to which I have referred, namely, liability for the
consequences of a failure of duty in circumstances in which it was a matter equivalent to contract between the parties that
that duty should be fulfilled.

Lord Shaw does not anywhere in his speech refer to the relationship as being of a fiduciary character. Lord Haldane laid down
the general principle in much the same terms. He said ([191415] All ER Rep at p 50; [1914] AC at p 948):

Although liability for negligence in word has in material respects been developed in our law differently from liability
for negligence in act, it is none the less true that a man may come under a special duty to exercise care in giving
information or advice. I should accordingly be sorry to be thought to lend countenance to the idea that recent decisions
have been intended to stereotype the cases in which people can be held to have assumed such a special duty. Whether such
a duty has been assumed must depend on the relationship of the parties, and it is at least certain that there are a good many
cases in which that relationship may be properly treated as giving rise to a special duty of care in statement.

It is quite true that Lord Haldane applied this principle only to cases of breach of fiduciary duty. But that was inevitable on the
facts of the case since upon the view of the pleading on which he was proceeding it was necessary to show equitable fraud.
In my judgment the effect of this case is as follows. The House clearly considered the view of Derry v Peek, exemplified in
Le Lievre v Gould, to be too narrow. It considered that outside contract (for contract was not pleaded in the case), there could be
a special relationship between parties which imposed a duty to give careful advice and accurate information. The majority of
their lordships did not extend the application of this principle beyond the breach of a fiduciary obligation, but none of them said
anything at all to show that it was limited to fiduciary obligation. Your lordships can therefore proceed on the footing that there
is such a general principle and that it is for you to say to what cases, beyond those of fiduciary obligation, it can properly be
extended.
I shall not at this stage deal in any detail with Robinson v National Bank of Scotland. Its chief relevance is to counsel for the
respondents second point. All that need be said about it on his first point is that it is no authority for the 606 proposition that
those relationships which give rise to a special duty of care are limited to the contractual and the fiduciary. On the contrary, it is a
clear authority for the view that Lord Haldane did not mean the general principle he stated in Nocton v Lord Ashburton to be
limited to fiduciary relationships. He said (1916 SC (HL) at p 157) that he wished emphatically to repeat what he had said in
Nocton v Lord Ashburton that it would be a great mistake to suppose that the principle in Derry v Peek affected the freedom of
action of the courts in recognising special duties arising out of other kinds of relationship. He went on (1916 SC (HL) at p 157):

The whole of the doctrine as to fiduciary relationships, as to the duty of care arising from implied as well as express
contracts, as to the duty of care arising from special relationships which the courts may find to exist in particular cases, still
remains, and I should be very sorry if any word fell from me which should suggest that the courts are in any way hampered
in recognising that the duty of care may be established when such cases really occur.

I come next to Donoghue (or McAlister) v Stevenson. In his celebrated speech in that case Lord Atkin did two things. He
stated ([1932] All ER Rep at p 11; [1932] AC at p 580) what he described as a general conception and from that conception he
formulated ([1932] All ER Rep at p 20; [1932] AC at p 599) a specific proposition of law. In between he gave a warning ([1932]
All ER Rep at p 13; [1932] AC at p 584)

against the danger of stating propositions of law in wider terms than is necessary, lest essential factors be omitted in a
wider survey and the inherent adaptability of English law be unduly restricted.

What Lord Atkin called a general conception of relations giving rise to a duty of care is now often referred to as the principle of
proximity. You must take reasonable care to avoid acts or omissions which you can reasonably foresee would be likely to injure
your neighbour. In the eyes of the law your neighbour is a person who is so closely and directly affected by your act that you
ought reasonably to have him in contemplation as being so affected when you are directing your mind to the acts or omissions
which are called in question. The specific proposition arising out of this conception is that ([1932] All ER Rep at p 20; [1932]
AC at p 599)

a manufacturer of products, which he sells in such a form as to show that he intends them to reach the ultimate
consumer in the form in which they left him, with no reasonable possibility of intermediate examination, and with the
knowledge that the absence of reasonable care in the preparation or putting up of the products will result in an injury to the
consumers life or property, owes a duty to the consumer to take that reasonable care.

Now it is not in my opinion a sensible application of what Lord Atkin was saying for a judge to be invited on the facts of any
particular case to say whether or not there was proximity between the plaintiff and the defendant. That would be a misuse of a
general conception and it is not the way in which English law develops. What Lord Atkin did was to use his general conception
to open up a category of cases giving rise to a special duty. It was already clear that the law recognised the existence of such duty
in the category of articles that were dangerous in themselves. What Donoghue v Stevenson did may be described either as the
widening of an old category or as the creation of a new and similar one. The general conception can be used to produce other
categories in the same way. An existing category grows as instances of its application multiply, until the time comes when the
cell divides.
607
Lord Thankerton and Lord Macmillan approached the problem fundamentally in the same way, though they left any general
conception on which they were acting to be implied. They inquired directly ([1932] All ER Rep at pp 22, 30; [1932] AC at pp
603, 619, 620) whether the relationship between the plaintiff and the defendant was such as to give rise to a duty to take care. It
is significant, whether it is a coincidence or not, that the term special relationship used by Lord Thankerton is also the one used
by Lord Haldane in Nocton v Lord Ashburton. The field is very different but the object of the search is the same.
In my opinion the appellants in their argument tried to press Donoghue v Stevenson too hard. They asked whether the
principle of proximity should not apply as well to words as to deeds. I think that it should, but as it is only a general conception it
does not get them very far. Then they take the specific proposition laid down by Donoghue v Stevenson and try to apply it
literally to a certificate or a bankers reference. That will not do, for a general conception cannot be applied to pieces of paper in
the same way as to articles of commerce, or to writers in the same way as to manufacturers. An inquiry into the possibilities of
intermediate examination of a certificate will not be fruitful. The real value of Donoghue v Stevenson to the argument in this case
is that it shows how the law can be developed to solve particular problems. Is the relationship between the parties in this case
such that it can be brought within a category giving rise to a special duty? As always in English law the first step in such an
inquiry is to see how far the authorities have gone, for new categories in the law do not spring into existence overnight.
It would be surprising if the sort of problem that is created by the facts of this case had never until recently arisen in English
law. As a problem it is a by-product of the doctrine of consideration. If the respondents had made a nominal charge for the
reference, the problem would not exist. If it were possible in English law to construct a contract without consideration, the
problem would move at once out of the first and general phase into the particular; and the question would be, not whether on the
facts of the case there was a special relationship, but whether on the facts of the case there was a contract.
The respondents in this case cannot deny that they were performing a service. Their sheet anchor is that they were
performing it gratuitously and therefore no liability for its performance can arise. My lords, in my opinion this is not the law. A
promise given without consideration to perform a service cannot be enforced as a contract by the promisee; but if the service is in
fact performed and done negligently, the promisee can recover in an action in tort. This is the foundation of the liability of a
gratuitous bailee. In the famous case of Coggs v Bernard, where the defendant had charge of brandy belonging to the plaintiff
and had spilt a quantity of it, there was a motion in arrest of judgment for that it was not alleged in the declaration that the
defendant was a common porter, nor averred that he had anything for his pains. The declaration was held to be good not-
withstanding that there was not any consideration laid. Gould J said:

The reason of the action is, the particular trust reposed in the defendant, to which he has concurred by his assumption,
and in the executing which he has miscarried by his neglect.

This proposition is not limited to the law of bailment. In Skelton v London & North Western Ry Co Willes J applied it generally
to the law of negligence. He said ((1867), LR 2 CP at p 636):
608
Actionable negligence must consist in the breach of some duty if a person undertakes to perform a voluntary act,
he is liable if he performs it improperly, but not if he neglects to perform it. Such is the result of the decision in the case of
Coggs v. Bernard.

Likewise in Banbury v Bank of Montreal, where the bank had advised a customer on his investments, Lord Finlay LC said
([1918] AC at p 654): He is under no obligation to advise, but if he takes upon himself to do so, he will incur liability if he does
so negligently.
The principle has been applied to cases where as a result of the negligence no damage was done to person or to property and
the consequential loss was purely financial. In Wilkinson v Coverdale the defendant undertook gratuitously to get a fire policy
renewed for the plaintiff, but, in doing so, neglected formalities, the omission of which rendered the policy inoperative. It was
held that an action would lie. In two similar cases the defendants succeeded on the ground that negligence was not proved in
fact. Both cases were thus decided on the basis that negligence was not proved in fact. Both cases were thus decided on the basis
that in law an action would lie. In the first of them, Shiells v Blackburne, the defendant had, acting voluntarily and without
reward, made an entry of the plaintiffs leather as wrought leather instead of dressed leather, with the result that the leather was
seized. In Dartnall v Howard the defendants purchased an annuity for the plaintiff but on the personal security of two insolvent
persons. The court, after verdict, arrested the judgment on the ground that the defendants appeared to be gratuitous agents and
that it was not averred that they had acted either with negligence or dishonesty.
Many cases could be cited in which the same result has been achieved by setting up some nominal consideration and suing
in contract instead of in tort. In Coggs v Bernard Holt CJ put the obligation on both grounds. He said ((1703), 2 Ld Raym at p
919):

Secondly it is objected, that there is no consideration to ground this promise upon, and therefore the undertaking is but
nudum pactum. But to this I answer, that the owners trusting him with the goods is a sufficient consideration to oblige him
to a careful management. Indeed if the agreement had been executory, to carry these brandies from the one place to the
other such a day, the defendant had not been bound to carry them. But this is a different case, for assumpsit does not only
signify a future agreement, but in such a case as this, it signifies an actual entry upon the thing, and taking the trust upon
himself. And if a man will do that, and miscarries in the performance of his trust, an action will lie against him for that,
though nobody could have compelled him to do the thing.

De la Bere v Pearson Ltd is an example of a case of this sort decided on the ground that there was a sufficiency of
consideration. The defendants advertised in their newspaper that their city editor would answer inquiries from readers of the
paper desiring financial advice. The plaintiff asked for the name of a good stockbroker. The editor recommended the name of a
person whom he knew to be an outside broker and whom he ought to have known, if he had made proper inquiries, to be an
undischarged bankrupt. The plaintiff dealt with him and lost his money. The case being brought in contract, Vaughan Williams
LJ thought ([19047] All ER Rep at p 756; [1908] 1 KB at p 287) that there was sufficient consideration in the fact that the 609
plaintiff consented to the publication of his question in the defendants paper if the defendants so chose. For Barnes P, the
consideration appears to have lain in the plaintiff addressing an inquiry as invited ([19047] All ER Rep at p 757; [1908] 1 KB at
p 289). In the same way when in Everett v Griffiths the Court of Appeal was considering the liability of a doctor towards the
person he was certifying, Scrutton LJ ([1920] 3 KB at p 191), said that the submission to treatment would be a good
consideration
My lords, I have cited these instances so as to show that in one way or another the law has ensured that in this type of case a
just result has been reached. But I think that today the result can and should be achieved by the application of the law of
negligence and that it is unnecessary and undesirable to construct an artificial consideration. I agree with Sir Frederick Pollocks
note on the case of De la Bere v Pearson Ltd, where he wrote in Pollock on Contract (13th Edn) 140 (note 31) that the cause of
action is better regarded as arising from default in the performance of a voluntary undertaking independent of contract.
My lords, it is true that this principle of law has not yet been clearly applied to a case where the service which the defendant
undertakes to perform is or includes the obtaining and imparting of information. But I cannot see why it should not be: and if it
had not been thought erroneously that Derry v Peek negatived any liability for negligent statements, I think that by now it
probably would have been. It cannot matter whether the information consists of fact or of opinion or is a mixture of both, nor
whether it was obtained as a result of special inquiries or comes direct from facts already in the defendants possession or from
his general store of professional knowledge. One cannot, as I have already endeavoured to show, distinguish in this respect
between a duty to inquire and a duty to state.
I think, therefore, that there is ample authority to justify your lordships in saying now that the categories of special
relationships, which may give rise to a duty to take care in word as well as in deed, are not limited to contractual relationships or
to relationships of fiduciary duty, but include also relationships which in the words of Lord Shaw in Nocton v Lord Ashburton
([191415] All ER Rep at p 62; [1914] AC at p 972) are equivalent to contract that is, where there is an assumption of
responsibility in circumstances in which, but for the absence of consideration, there would be a contract. Where there is an
express undertaking, an express warranty as distinct from mere representation, there can be little difficulty. The difficulty arises
in discerning those cases in which the undertaking is to be implied. In this respect the absence of consideration is not irrelevant.
Payment for information or advice is very good evidence that it is being relied on and that the informer or adviser knows that it
is. Where there is no consideration, it will be necessary to exercise greater care in distinguishing between social and professional
relationships and between those which are of a contractual character and those which are not. It may often be material to
consider whether the adviser is acting purely out of good nature or whether he is getting his reward in some indirect form. The
service that a bank performs in giving a reference is not done simply out of a desire to assist commerce. It would discourage the
customers of the bank if their deals fell through because the bank had refused to testify to their credit when it was good.
I have had the advantage of reading all the opinions prepared by your lordships and of studying the terms which your
lordships have framed by way of definition of the sort of relationship which gives rise to a responsibility towards those who act
on information or advice and so creates a duty of care towards them. I do not 610 understand any of your lordships to hold that it
is a responsibility imposed by law on certain types of persons or in certain sorts of situations. It is a responsibility that is
voluntarily accepted or undertaken either generally where a general relationship, such as that of solicitor and client or banker and
customer, is created, or specifically in relation to a particular transaction. In the present case the appellants were not, as in Woods
v Martins Bank Ltd the customers or potential customers of the bank. Responsibility can attach only to the single act, ie, the
giving of the reference, and only if the doing of that act implied a voluntary undertaking to assume responsibility. This is a point
of great importance because it is, as I understand it, the foundation for the ground on which in the end the House dismisses the
appeal. I do not think it possible to formulate with exactitude all the conditions under which the law will in a specific case imply
a voluntary undertaking, any more than it is possible to formulate those in which the law will imply a contract. But in so far as
your lordships describe the circumstances in which an implication will ordinarily be drawn, I am prepared to adopt any one of
your lordships statements as showing the general rule; and I pay the same respect to the statement by Denning LJ in his
dissenting judgment in Candler v Crane, Christmas & Co ([1951] 1 All ER 426 at p 433; [1951] 2 KB 164 at p 179) about the
circumstances in which he says a duty to use care in making a statement exists.
I do not go further than this for two reasons. The first is that I have found in the speech of Lord Shaw in Nocton v Lord
Ashburton and in the idea of a relationship that is equivalent to contract all that is necessary to cover the situation that arises in
this case. Counsel for the appellants does not claim to succeed unless he can establish that the reference was intended by the
respondents to be communicated by National Provincial Bank Ltd to some unnamed customer of theirs, whose identity was
immaterial to the respondents, for that customers use. All that was lacking was formal consideration. The case is well within the
authorities that I have already cited and of which Wilkinson v Coverdale is the most apposite example.
I shall therefore content myself with the proposition that wherever there is a relationship equivalent to contract there is a
duty of care. Such a relationship may be either general or particular. Examples of a general relationship are those of solicitor and
client and of banker and customer. For the former Nocton v Lord Ashburton has long stood as the authority and for the latter
there is the decision of Salmon J in Woods v Martins Bank Ltd which I respectfully approve. There may well be others yet to be
established. Where there is a general relationship of this sort it is unnecessary to do more than prove its existence and the duty
follows. Where, as in the present case, what is relied on is a particular relationship created ad hoc, it will be necessary to
examine the particular facts to see whether there is an express or implied undertaking of responsibility.
I regard this proposition as an application of the general conception of proximity. Cases may arise in the future in which a
new and wider proposition, quite independent of any notion of contract, will be needed. There may, for example, be cases in
which a statement is not supplied for the use of any particular person, any more than in Donoghue v Stevenson the ginger beer
was supplied for consumption by any particular person; and it will then be necessary to return to the general conception of
proximity and to see whether there can be evolved from it, as was done in Donoghue v Stevenson, a specific proposition to fit the
case. When that has to be done, the speeches of your lordships today as well as the judgment of Denning LJ to which I have
referredand also, I may add, 611the proposition in the Restatement d, and the cases which exemplify it, will afford good
guidance as to what ought to be said. I pefer to see what shape such cases take before committing myself to any formulation, for
I bear in mind Lord Atkins warning, which I have quoted, against placing unnecessary restrictions on the adaptability of English
law. I have, I hope, made it clear that I take quite literally the dictum of Lord MacMillan, so often quoted from the same case,
that ([1932] All ER Rep at p 30; [1932] AC at p 619) the categories of negligence are never closed. English law is wide
enough to embrace any new category or proposition that exemplifies the principle of proximity.
________________________________________
d See 65 Corpus Juris Secundum title Negligence, pp 428, 429, 20, which begins A false statement negligently made may be the basis of a
recovery of damages for injury or loss sustained in consequence of reliance thereon, the American rule, in this respect, being more liberal
than the rule in England.

I have another reason for caution. Since the essence of the matter in the present case and in others of the same type is the
acceptance of responsibility, I should like to guard against the imposition of restrictive terms notwithstanding that the essential
condition is fulfilled. If a defendant says to a plaintiff:Let me do this for you, do not waste your money in employing a
professional, I will do it for nothing and you can rely on me, I do not think that he could escape liability simply because he
belonged to no profession or calling, had no qualifications or special skill and did not hold himself out as having any. The
relevance of these factors is to show the unlikelihood of a defendant in such circumstances assuming a legal responsibility and as
such they may often be decisive. But they are not theoretically conclusive, and so cannot be the subject of definition. It would
be unfortunate if they were. For it would mean that plaintiffs would seek to avoid the rigidity of the definition by bringing the
action in contract as in De la Bere v Pearson Ltd and setting up something that would do for consideration. That to my mind
would be an undesirable development in the law; and the best way of avoiding it is to settle the law so that the presence or
absence of consideration makes no difference.
Your lordships attention was called to a number of cases in courts of first instance or of appeal which it was said would
have been decided differently if the appellants main contention was correct. I do not propose to go through them in order to
consider whether on the facts of each it should or should not be upheld. I shall content myself with saying that in my opinion Le
Lievre v Gould and all decisions based on its reasoning (in which I specifically include, lest otherwise it might be thought that
generalia specialibus non derogant, the decision of Devlin J in Heskell v Continental Express Ltd ([1950] 1 All ER 1033 at p
1044)) can no longer be regarded as authoritative; and when similar facts arise in the future, the case will have to be judged
afresh in the light of the principles which the House has now laid down.
My lords, I have devoted much time and thought to considering the first reason given by counsel for the respondents for
rejecting the appellants claim. I have done so, not only because his reason was based on a ground so fundamental that it called
for a full refutation, but also because it is impossible to find the correct answer on the facts to the appellants claim until the
relevant criteria for ascertaining whether or not there is a duty to take care have been clearly established. Once that is done their
application to the facts of this case can be done very shortly, for the case then becomes a very simple one.
I am satisfied for the reasons which I have given that a person for whose use a bankers reference is furnished is not, simply
because no consideration has passed, prevented from contending that the banker is responsible to him for what 612 he has said.
The question is whether the appellants can set up a claim equivalent to contract and rely on an implied undertaking to accept
responsibility. Counsel for the respondents second point is that in Robinson v National Bank of Scotland this House has already
laid it down as a general rule that in the case of a banker furnishing a reference that cannot be done. I do not agree. The facts in
that case have been stated by my noble and learned friend Lord Reid and I need not repeat them. I think it is plain on those facts
that the bank in that case was not furnishing the reference for the use of the pursuer; he was not a person for whose use of the
reference they were undertaking any responsibility, and that quite apart from their general disclaimer. Furthermore, the pursuer
never saw the reference; he was given only what the Lord Justice-Clerk described (1916 SC at p 58) as a gloss of it. This
makes the connexion between the pursuer and the defendants far too remote to constitute a relationship of a contractual character.
On the facts of the present case counsel for the respondents has, under his third head, argued for the same result. He
submits, first, that it ought not to be inferred that the respondents knew that National Provincial Bank, Ltd were asking for the
reference for the use of a customer. If the respondents did know that, then counsel submits that they did not intend that the
reference itself should be communicated to the customer; it was intended only as material upon which the customers bank could
advise the customer on its own responsibility. I should consider it necessary to examine these contentions were it not for the
general disclaimer of responsibility which appears to me in any event to be conclusive. I agree entirely with the reasoning and
conclusion on this point of my noble and learned friend Lord Reid. A man cannot be said voluntarily to be undertaking a
responsibility if at the very moment when he is said to be accepting it he declares that in fact he is not. The problem of
reconciling words of exemption with the existence of a duty arises only when a party is claiming exemption from a responsibility
which he has already undertaken or which he is contracting to undertake. For this reason alone, I would dismiss the appeal.

LORD PEARCE. My Lords, Viscount Haldane LC in Nocton v Lord Ashburton ([191415] All ER Rep 45 at p 50; [1914] AC
932 at p 948) said:

Although liability for negligence in word has in material respects been developed in our law differently from liability
for negligence in act, it is none the less true that a man may come under a special duty to exercise care in giving
information or advice. I should accordingly be sorry to be thought to lend countenance to the idea that recent decisions
have been intended to stereotype the cases in which people can be held to have assumed such a special duty. Whether such
a duty has been assumed must depend on the relationship of the parties, and it is at least certain that there are a good many
cases in which that relationship may be properly treated as giving rise to a special duty of care in statement.
The law of negligence has been deliberately limited in its range by the courts insistence that there can be no actionable
negligence in vacuo without the existence of some duty to the plaintiff. For it would be impracticable to grant relief to everybody
who suffers damage through the carelessness of another.
The reason for some divergence between the law of negligence in word and that of negligence in act is clear. Negligence in
word creates problems different from those of negligence in act. Words are more volatile than deeds. They travel fast and far
afield. They are used without being expended and take effect 613 in combination with innumerable facts and other words. Yet
they are dangerous and can cause vast financial damage. How far they are relied on unchecked (by analogy with there being no
probability of intermediate inspectionsee Grant v Australian Knitting Mills Ltd) must in many cases be a matter of doubt and
difficulty. If the mere hearing or reading of words were held to create proximity, there might be no limit to the persons to whom
the speaker or writer could be liable. Damage by negligent acts to persons or property on the other hand is more visible and
obvious; its limits are more easily defined and it is with this damage that the earlier cases were more concerned. It was not until
1789 that Pasley v Freeman recognised and laid down a duty of honesty in words to the world at largethus creating a remedy
designed to protect the economic as opposed to the physical interests of the community. Any attempts to extend this remedy by
imposing a duty of care as well as a duty of honesty in representations by word were curbed by Derry v Peek.
In Cann v Willson it had been held that a valuer was liable in respect of a negligent valuation which he had been employed
by the owner of property to make for the purpose of raising a mortgage, and which the valuer himself put before the proposed
mortgagees solicitor. Chitty J there said ((1888), 39 ChD at pp 42, 43):

It seems to me that the defendants knowingly placed themselves in that position, and in point of law incurred a duty
towards him to use reasonable care in the preparation of the document called a valuation. I think it is like the case of an
articlethe supply of the hairwash in the case of George v. Skivington.

George v Skivington was later approved in Donoghue (or McAlister) v Stevenson. Thus in the case of economic damage alone he
was drawing an analogy from a case where physical damage to the wife of a purchaser was held to give rise to an action for
negligence.
Cann v Willson was, however, overruled by Le Lievre v Gould on the ground, erroneous as it seems to me, that it could not
stand with Derry v Peek. The particular facts in Le Lievre v Gould justified the particular decision, as Denning LJ explained in
Candler v Crane, Christmas & Co. But the ratio decidendi was wrong since it attributed to Derry v Peek more than that case
decided. In Nocton v Lord Ashburton this House pointed out that too much had been ascribed to Derry v Peek. Viscount Haldane
LC said ([191415] All ER Rep at p 49; [1914] AC at p 947):

The discussion of the case by the noble and learned lords who took part in the decision appears to me to exclude the
hypothesis that they considered any other question to be before them than what was the necessary foundation of an
ordinary action for deceit. They must indeed be taken to have thought that the facts proved as to the relationship of the
parties in Derry v. Peek were not enough to establish any special duty arising out of that relationship other than the general
duty of honesty. But they do not say that where a different sort of relationship ought to be inferred from the circumstances
the case is to be concluded by asking whether an action 614 of deceit will lie. I think that the authorities, subsequent to the
decision of the House of Lords shew a tendency to assume that it was intended to mean more than it did. In reality the
judgment covered only a part of the field in which liabilities may arise. There are other obligations besides that on honesty,
the breach of which may give rise to damages. These obligations depend on principles which the judges have worked out
in the fashion that is characteristic of a system where much of the law has always been judge-made and unwritten.

Lord Haldane spoke to a like effect in Robinson v National Bank of Scotland (1916 SC 154 at p 157):

I think, as I said in Noctons case that an exaggerated view was taken by a good many people of the scope of the
decision in Derry v. Peek. The whole of the doctrine as to fiducary relationships, as to the duty of care arising from
implied as well as express contracts, as to the duty of care arising from other special relationships which the courts may
find to exist in particular cases, still remains and I should be very sorry if any word fell from me which should suggest that
the courts are in any way hampered in recognising that the duty of care may be established when such cases really occur.

Lord Haldane was thus in terms preserving unencumbered the area of special relationships which created a duty of care; and he
was not restricting the area to cases where courts of equity would find a fiduciary duty.
The range of negligence in act was greatly extended in Donoghue v Stevenson on the wide principle of the good neighbour
sic utere tuo alienum non laedas. It is argued that the principles enunciated in Donoghue v Stevenson apply fully to negligence
in word. It may well be that Wrottesley J in Old Gate Estates Ltd v Toplis and Harding and Russell put the matter too narrowly
when he confined the applicability of the principles laid down in Donoghue v Stevenson to negligence which caused damage to
life, limb or health. But they were certainly not purporting to deal with such issues as, for instance, how far economic loss alone
without some physical or material damage to support it, can afford a cause of action in negligence by act (see Morrison
Steamship Co Ltd v Greystoke Castle (Cargo Owners) where it was held that it could do so). The House in Donoghue v
Stevenson was, in fact, dealing with negligent acts causing physical damage and the opinions cannot be read as if they were
dealing with negligence in word causing economic damage. Had it been otherwise some consideration would have been given to
problems peculiar to negligence in words. That case, therefore, can give no more help in this sphere than by affording some
analogy from the broad outlook which it imposed on the law relating to physical negligence.
How wide the sphere of the duty of care in negligence is to be laid depends ultimately on the courts assessment of the
demands of society for protection from the carelessness of others. Economic protection has lagged behind protection in physical
matters where there is injury to person and property. It may be that the size and the width of the range of possible claims has
acted as a deterrent to extension of economic protection. In this sphere the law was developed in the United States in Glanzer v
Shepard, where a public weigher employed by a vendor was held liable to a purchaser for giving him a certificate which
negligently 615 overstated the amount of the goods supplied to him. The defendant was thus engaged on a task in which, as he
knew, vendor and purchaser alike depended on his skill and care and the fact that it was the vendor who paid him was merely an
accident of commerce. This case was followed and developed in later cases.
In the Ultramares case however, the court felt the undesirability of exposing defendants to a potential liability in an
indeterminate amount for an indefinite time to an indeterminate class. It decided that auditors were not liable for negligence in
the preparation of their accounts (of which they supplied thirty copies although they were not aware of the specific purpose,
namely, to obtain financial help) to a plaintiff who lent money on the strength of them. In South Africa, under a different system
of law, two cases show a similar advance and subsequent restriction (Perlman v Zoutendyk and Herschel v Mrupe.
Some guidance may be obtained from the case of Shiells v Blackburne. There a general merchant undertook, voluntarily and
without reward, to enter a parcel of the goods of another, together with a parcel of his own of the same sort, at the Customs House
for exportation. Acting, it was contended, with gross negligence, he made the entry under a wrong denomination, whereby both
parcels were seized. The plaintiff failed on the facts to make out a case of gross negligence. But Lord Loughborough said
((1789), 1 Hy Bl at p 162):
where a bailee undertakes to perform a gratuitous act, from which the bailor alone is to receive benefit, there the
bailee is only liable for gross negligence; but if a man gratuitously undertakes to do a thing to the best of his skill, where
his situation or profession is such as to imply skill, an omission of that skill is imputable to him as gross negligence. If in
this case a ship-broker or a clerk in the Custom-House, had undertaken to enter the goods, a wrong entry would in them be
gross negligence, because their situation and employment necessarily imply a competent degree of knowledge in making
such entries.

Heath J said ((1789), 1 Hy Bl at p 161):

the surgeon would also be liable for such negligence, if he undertook gratis to attend a sick person, because his
situation implies skill in surgery; but if the patient applies to a man of a different employment or occupation for his
gratuitous assistance, who either does not exert all his skill, or administers improper remedies to the best of his ability, such
person is not liable.

In Gladwell v Stegall an infant plaintiff, ten years old, recovered damages for injury to health from a surgeon and apothecary
who had treated her. She did not sue in contract but brought an action ex delicto alleging a breach of duty arising out of his
employment by her, although it was her father to whom the bill was made out. In Wilkinson v Coverdale Lord Kenyon accepted
the proposition that a defendant who had gratuitously undertaken to take out an insurance policy and who did it negligently could
be liable in damages. In those cases there was no dichotomy between negligence in act and in word, nor between physical and
economic loss. The basis underlying them is that if persons holding themselves out in a calling or situation or profession take on
a task within that calling or situation or profession they have a duty of skill and care. In terms of proximity one might say that
they are in particularly close proximity to those who, as they know, are relying on their skill and care, although the proximity is
not contractual.
616
The reasoning of Shiells v Blackburne was applied in Everett v Griffiths ([1920] 3 KB 163, particularly at pp 182, 217)
where the Court of Appeal held that a doctor owed a duty of care to a man by whom he was not employed but whom he had a
duty to examine under the Lunacy Act, 1890. It was also relied on by Denning LJ in his dissenting judgment in Candler v Crane,
Christmas & Co. He reached the conclusion that in respect of reports and work that resulted in such reports there was a duty of
care laid on ([1951] 1 All ER at p 433; [1951] 2 KB at p 179)

those persons such as accountants, surveyors, valuers and analysts, whose profession and occupation it is to examine
books, accounts, and other things and to make reports on which other peopleother than their clientsrely in the ordinary
course of business.

The duty is in his opinion owed (apart from contractual duty to their employer) ([1951] 1 All ER at p 434; [1951] 2 KB at pp 180,
181)

to any third person to whom they themselves show the accounts, or to whom they know their employer is going to
show the accounts so as to induce him to invest money or take some other action on them.

He excludes strangers of whom they have heard nothing and to whom their employer without their knowledge may choose to
hand their accounts, and continues ([1951] 1 All ER at p 434; [1951] 2 KB at p 181):

The test of proximity in these cases is: did the accountants know that the accounts were required for submission to the
plaintiff and use by him?

(It is to be noted that these expressions of opinion produce a result somewhat similar to the Restatement para 552 e. I agree with
those words. In my opinion they are consonant with the earlier cases and with the observations of Lord Haldane.
________________________________________
e Compare p 612 note (206), ante

It is argued that so to hold would create confusion in many aspects of the law and infringe the established rule that innocent
misrepresentation gives no right to damages. I cannot accept that argument. The true rule is that innocent misrepresentation per
se gives no right to damages. If the misrepresentation was intended by the parties to form a warranty between two contracting
parties, it gives on that ground a right to damages (Heilbut, Symons & Co v Buckleton. If an innocent misrepresentation is made
between parties in a fiduciary relationship it may, on that ground, give a right to claim damages for negligence. There is also in
my opinion a duty of care created by special relationships which, though not fiduciary, give rise to an assumption that care as
well as honesty is demanded.
Was there such a special relationship in the present case as to impose on the respondents a duty of care to the appellants as
the undisclosed principals for whom National Provincial Bank Ltd was making the inquiry? The answer to that question depends
on the circumstances of the transaction. If, for instance, they disclosed a casual social approach to the inquiry no such special
relationship or duty of care would be assumed (see Fish v Kelly). To import such a duty the representation must normally, I think,
concern a business or professional transaction whose nature makes clear the gravity of the inquiry and the importance and
influence attached to the answer. It is conceded that Salmon J 617 rightly found a duty of care in Woods v Martins Bank Ltd, but
the facts in that case were wholly different from those in the present case. A most important circumstance is the form of the
inquiry and of the answer. Both were here plainly stated to be without liability. Counsel for the appellants argues that those
words are not sufficiently precise to exclude liability for negligence. Nothing, however, except negligence could, in the facts of
this case, create a liability (apart from fraud to which they cannot have been intended to refer and against which the words would
be no protection since they would be part of the fraud). I do not, therefore, accept that, even if the parties were already in
contractual or other special relationship, the words would give no immunity to a negligent answer. But in any event they clearly
prevent a special relationship from arising. They are part of the material from which one deduces whether a duty of care and a
liability for negligence was assumed. If both parties say expressly (in a case where neither is deliberately taking advantage of the
other) that there shall be no liability, I do not find it possible to say that a liability was assumed.
In Robinson v National Bank of Scotland also the correspondence expressly excluded responsibility. Possibly that factor
weighed with Lord Haldane when he said (1916 SC (HL) at p 157):

But when a mere inquiry is made by one banker of another, who stands in no special relation to him, then, in the
absence of special circumstances from which a contract to be careful can be inferred, I think there is no duty excepting the
duty of common honesty to which I have referred.

I appreciate counsel for the appellants emphasis on the general importance to the business world of bankers references and the
desirability that in an integrated banking system there should be a duty of care with regard to them, but on the facts before us it is
in my opinion not possible to hold that there was a special duty of care and a liability for negligence.
I would, therefore, dismiss the appeal.

Appeal dismissed.

Solicitors: Evill & Coleman (for the appellants); Franks, Charlesly & Co (for the respondents).

C G Leonard Esq Barrister.


618
[1963] 2 All ER 619

Curwen v James and Others


QUANTUM: CIVIL PROCEDURE

COURT OF APPEAL
SELLERS, HARMAN AND PEARSON LJJ
3, 8 APRIL 1963

Fatal Accident Damages Assessment Deductions from damages Re-marriage of widow after trial but before expiry of time
for serving notice of appeal Chance of re-marriage considered by trial judge, but no evidence of possibility of re-marriage
given at trial Whether damages should be reduced by Court of Appeal.

Court of Appeal Evidence Further evidence Damages awarded to widow under Fatal Accidents Act, 1846 (9 & 10 Vict c 93)
Re-marriage of widow after trial but before expiry of time for service of notice of appeal RSC, Ord 58, r 9(2).

In June, 1959, C, aged 22, was killed in a collision between a motor-bicycle which he was riding and a motor-car. Cs widow
brought an action for damages under the Fatal Accidents Act, 1846, and the Law Reform (Miscellaneous Provisions) Act, 1934,
against the executors of the driver of the motor-car. The action was heard in January, 1962, when the widow, who had no
children, was aged 24. The trial judge found that C was one-third to blame for the accident, that the amount by which the widow
would have been likely to benefit in the future was 6 a week, and assessed the damages at 4,000, saying that the widow was a
presentable young lady who would have opportunities, if she were so minded, of re-marriage and that it was right to make some
real diminution in the amount of damages awarded because of that factor. Owing to the fact that she had broken down when
giving evidence at the trial, the widow had not been asked about the possibility of re-marriage. In March, 1962, before the expiry
of the time for giving notice of appeal, she re-married. On appeal by the defendants against the amount of damages awarded,
they sought leave under RSC, Ord 58, r 9(2) a, to adduce evidence of the widows re-marriage on the ground that, if it was
granted, they would contend that, by her re-marriage, the widow had not lost the financial support assessed by the judge. The
defendants tendered no evidence as to the amount by which the re-marriage was benefiting the widow.
________________________________________
a Order 58, r 9(2), is set out at p 621, letter g, post

Held The appeal would be allowed and the amount of damages awarded reduced because where, as here, the re-marriage
occurred soon after the trial, the case should be decided on the known fact of marriage, of which evidence would be admitted on
the appeal (see p 622, letter c, p 623, letter g, and p 624, letter a, post); accordingly, the damages assessed would be reduced by
one half to 2,000, a figure which, in the opinion of the Court of Appeal, was as much as the trial judge would have awarded had
he known that the re-marriage was likely to take place as early as it did (see p 622, letter h, and p 623, letter h, post).
Per Harman LJ and Pearson LJ: we do not decide what the position would have been if the re-marriage had occurred after
the expiry of the time limited for serving notice of appeal (see p 624, letter e, and p 623, letter g, post).
Appeal allowed.

Notes
As to the reduction of damages under the Fatal Accidents Acts on account of re-marriage of the widow, see 28 Halsburys Laws
(3rd Edn) 102, para 111, text and note (o); and for cases on the subject, see 36 Digest (Repl) 221224, 11761194.
As to the power of the Court of Appeal to receive further evidence, see 30 Halsburys Laws (3rd Edn) 468, 469, para 884;
and for cases on the subject, see Digest (Practice) 776778, 33983420.

Cases referred to in judgments


A-G v Birmingham, Tame and Rea District Drainage Board [191113] All ER Rep 926, [1912] AC 788, 82 LJCh 45, 107 LT 353,
76 JP 481, 16 Digest (Repl) 119, 44.
619
Bradberry, Re, National Provincial Bank Ltd v Bradberry [1942] 2 All ER 629, [1943] Ch 35, 112 LJCh 49, 167 LT 396, 23
Digest (Repl) 429, 4992.
Brown v Dean [190810] All ER Rep 661, [1910] AC 373, 79 LJKB 690, 102 LT 661, 13 Digest (Repl) 449, 720.
Bwllfa and Merthyr Dare Steam Collieries (1891) Ltd v Pontypridd Waterworks Co [19003] All ER Rep 600, [1903] AC 426, 72
LJKB 805, 89 LT 280, 11 Digest (Repl) 136, 199.
Ladd v Marshall [1954] 3 All ER 745, [1954] 1 WLR 1489, 3rd Digest Supp.
Mead v Clarke Chapman & Co Ltd [1956] 1 All ER 44, [1956] 1 WLR 76, 3rd Digest Supp.

Appeal and application


This was an appeal by the defendants, Frank Warwick James, Kathleen Patricia Boyes and William Gorley, executors of William
James, deceased, against the assessment by Marshall J at Carlisle Assizes on 24 January 1962, of damages which he awarded to
the plaintiff, Catherine Veronica Curwen, administratrix of Peter Michael Curwen, deceased, against them under the Fatal
Accidents Act, 1846, and the Law Reform (Miscellaneous Provisions) Act, 1934. The defendants also applied for leave to adduce
additional evidence, namely, the plaintiffs re-marriage on 7 March 1962, to John Moore. The facts are set out in the judgment of
Sellers LJ.

G W Guthrie Jones for the defendants.


A D Karmel QC and Anthony Thompson for the plaintiff.

8 April 1963. The following judgments were delivered.

SELLERS LJ. The plaintiff in this action was the widow of Peter Michael Curwen, aged twenty-two, who met with a fatal
accident when he was riding a motor bicycle on 19 June 1959. He had a pillion rider with him. He was travelling very fast, as
the judge found, and came into collision with a motor-car which turned across the course of the motor-cycle. The car was being
driven by a man, now deceased, whose executors are the defendants to the action. The learned judge found that the plaintiffs
husband was one-third to blame and the defendants (as executors) two-thirds to blame. Therefore, the widow was only entitled to
recover two-thirds of whatever amount the learned judge thought appropriate for the damages which were sought to be recovered
under the Fatal Accidents Act, 1846 and the Law Reform (Miscellaneous Provisions) Act, 1934. There were no children. The
wife at the time of the trial in January, 1962, was twenty-four. The learned judge in reaching his conclusion took what, I think,
can be said to be a quite generous figure as to the amount by which the plaintiff would have been likely to benefit in the future
that is to say, 6 a week. On that figure of 6 a week, the learned judge arrived at a figure of total compensation, apart from
apportionment of liability, of 4,000. In so doing, he did say that the plaintiff was a presentable young lady, and that he had
little doubt that she would have opportunities, if she were so minded, of starting afresh in married life with someone else, and
that, in a case of this kind, he thought that it would be right (as indeed it was) for him to make some real diminution in the figures
which he would award because of that.
The first submission on behalf of the defendants is that the amount awarded was too much, that the learned judge erred in
the amount which the plaintiff had probably lostthe 6 a weekand that the sum of 4,000 was in any event too great. But
another point is taken which has proved much more troublesome, partly because it was not taken wholly in accordance with the
practice in these matters. That is, perhaps, understandable because there was a desire to save costs, and there was some
correspondence between the respective solicitors. First of all, on 7 March 1962, notice of appeal was givenI think it is said the
judgment had only been perfected that very dayon the ground that the learned judge had made a wholly erroneous estimate of
the damages and that the damages 620 were excessive, but then, on 27 April 1962, a further notice was given, said to be a
supplementary notice of appeal, and in that form of notice (which was not a motion, as is customary) the plaintiff was informed
that the defendants were going to ask for leave to adduce evidence in addition to the evidence given before Marshall J. That
evidence was (as stated in the notice) that, on 7 March 1962, the very day of the first notice of appeal, the plaintiff was married to
John Moore, a fitter aged twenty-two, at the parish church in the parish of St Jamess, Whitehaven, in the county of Cumberland.
It was said that, if that evidence was granted, the defendants would contend that, by reason of her said marriage, the plaintiff has
not lost the financial support assessed by the learned judge. I said that that notice had come about by letters passing between the
solicitors; but there is to my mind one serious omission in the information there which the defendants wished to put before this
courtthat nothing is said about the earnings of Mr John Moore, to whom the plaintiff was married on 7 March 1962. It is a
factor which throws light on the matter, anticipated by the judge in his judgment, of the plaintiff being a presentable young lady
who might be inclined to marry and to have opportunities of marriage. The marriage is something certain which was not, and
could not have been, known at the trial.
One of the difficulties in the case as I see it is that in circumstances perhaps understandable, although I do not think that they
are altogether excusable, the issue as to the probabilities of the future, apart from such evidence as the plaintiff herself presented
by her appearance to the learned judge, was not investigated in the evidence. Apparently the plaintiff when giving evidence
broke down in the witness-box and collapsed in tears, and the examination or the cross-examination of her was not pursued. If
she had been asked, she might have been able to tell the learned judge, not truly the fact that she was married, but sufficiently
close to that to indicate to him that she was engaged and intended to be married in March, and perhaps even the day might have
been fixed, 7 March 1962. It might well have saved this appeal and have saved any further enquiry into the matter. On the other
hand, this may all have developed after the hearing and the learned judge would have been no wiser than he was at the trial with
regard to what the future had in store.
The rule with regard to this particular matter is of old standing. It is RSC, Ord 58, r 9(2), but it has had its predecessor in
effect in earlier rules. It reads as follows:

The Court of Appeal shall have full discretionary power to receive further evidence upon questions of fact, either by
oral examination in court, by affidavit, or by deposition taken before an examiner or commissioner: Provided that in the
case of an appeal from a judgment after trial or hearing of any cause or matter upon the merits, no such further evidence
(other than evidence as to matters which have occurred after the date of the trial or hearing) shall be admitted except on
special grounds.

The special grounds which would affect our discretion here, although not strictly applicable, have been often stated in the courts
and have been well summarised fairly recently in the decision of this court in Ladd v Marshall([1954] 3 All ER 745 at p 748) by
Denning LJ. Lord Loreburn LC had said in Brown v Dean ([190810] All ER Rep 661 at p 662; [1910] AC 373 at p 374):

When a litigant has obtained a judgment in a court of justice he is by law entitled not to be deprived of that
judgment without very solid grounds ,

and that is in the public interest. The grounds which are set out as special grounds, as I have said, were stated in Ladd v Marshall
([1954] 3 All ER 745 at p 748), and it might be said that they have all been fulfilled in this case by the nature of the evidence, but
I doubt whether by the detail of the evidence, which the defendants seek to adduce. 621There is the citation relied on from the
speech of Lord Gorell in A-G v Birmingham, Tame and Rea District Drainage Board ([191113] All ER Rep 926 at p 939; [1912]
AC 788 at p 801), where he says:

Under the Judicature Acts and rules the hearing of an appeal from the judgment of a judge is by way of re-hearing, and
the court has power to give any judgment and to make any order which ought to have been made, and to make such further
or other order as the court may think fit The court also has power to take evidence of matters which have occurred after
the date of the decision from which the appeal is brought It seems clear, therefore, that the Court of Appeal is entitled
and ought to re-hear the case as at the time of re-hearing

In that case, they sought to adduce fresh evidence of something which they had done afterwards which altered the effect of the
injunction which had been granted.
In the present case, although the proceedings are not wholly regular, it is desirable that the court should decide the matter on
the known fact of the marriage rather than that it should remain decided on an uncertainty for the future as it stood before the
learned judge. That is a factor which would justify this court admitting the new evidence to that effect in the particular
circumstances of this case. Having said that, I find that it is unsatisfactory (and it has a very direct bearing on the case) that,
although the court has the information now that the plaintiff has re-married, it has no information as to the amount by which the
re-marriage is benefiting her, and I think (as learned counsel for the defendants rather invited us to do) that it is open to this court
to look at this question quite generally in the light of that established fact and apply its mind to its effect on the amount which
might properly have been awarded by the judge had he known of the marriage but not known how much Mr John Moore was
earning or anything about the state of his health. Why the court should be left in that position I do not quite know; but the party
who must suffer by it must be the defendants, whose obligation it is to put more definite evidence before the court. We were
referred to some observations to this effect from Parker LJ towards the end of his judgment in Mead v Clarke Chapman & Co Ltd
([1956] 1 All ER 44 at p 49). Having introduced that fact of the marriage, although I do this not without some doubt as to its
strict regularity having regard to the way the defendants have advanced their case, I am prepared to review this amount in the
light of that added circumstance. It does not enable one to say on this calculation that the widow, the plaintiff, has been put back
into the same position as if her husband who was killed had still been alive, but it does enable one to say that the matter as to the
probability of marriage, to which the judge applied his mind, has actually occurred, and one would assume, although again it is a
little unsatisfactory not to know more definitely, that the fact of marriage would lead to the conclusion that there is some benefit
to be gained financially by the plaintiff, and that she would have some of the hardship of the loss of her husbands earnings
ameliorated by the benefits she gets from the new marriage.
Applying very broad standards of loss appropriate to the injury and looking into the future of the plaintiff, I think that the
best the court can doit is very much a jury matteris to say that the learned judge assessed the damages at twice as much as
they ought to have been and that, if he had had the knowledge that the marriage was likely to accrue as early as it did in fact
accrue, he would have awarded, or could reasonably have awarded, no more than 2,000 damages. That will have to be reduced
by one-third, and, in my view, the amount to be entered for the plaintiff by way of damages for the loss of her husband in this
accident against the defendants should be two-thirds of 2,000, plus the appropriate sum for the funeral expenses, which has been
agreed.
In those circumstances, I would allow the appeal and vary the order accordingly.
622

HARMAN LJ. I agree in the result. There is an important principle here involved and it is that the court should never speculate
where it knows. That is a principle which is involved where for instance there is a change in value in property. I refer in that
connexion to the well known case, Bwllfa and Merthyr Dare Steam Collieries (1891) Ltd v Pontypridd Waterworks Co , where
there was an arbitration about the value of some coal, and Lord MacNaghten in his speech said this ([190003] All ER Rep at p
603; [1903] AC at p 431):

If the question goes to arbitration, the arbitrators duty is to determine the amount of compensation payable. In order
to enable him to come to a just and true conclusion it is his duty, I think, to avail himself of all information at hand at the
time of making his award, which may be laid before him. Why should he listen to conjecture on a matter which has
become an accomplished fact? Why should he guess when he can calculate? With the light before him, why should he
shut his eyes and grope in the dark?

A similar conclusion was reached by Uthwatt J in Re Bradberry, National Provincial Bank Ltd v Bradberry, where the question
was about the valuation of the annuity of an annuitant who in fact had died during the course of the proceedings and before the
valuation came to be made. The learned judge says this ([1942] 2 All ER at p 635; [1943] Ch at p 42):

The death of an annuitant before the courts decision enables the court exactly to measure what was in fact given to
him. Why should the court neglect known facts and put itself in the position of a prophet who, when he knows all the facts,
projects himself to an earlier date and predicts as the span of life of a person known to be dead the length of life of the
hypothetical person who lives his actuarial life? Few would trouble to acquire the mantle of Elijah to make such a
prophecy. In my view, one who did so would be only wasting his coupons.

Mr Ralph Sutton KC the editor of the reports, puts in an explanatory note at the bottom ([1943] Ch at p 42) about clothing
coupons, which have thus taken their place in the Law Reports. But the principle is the same. Why should we, when we know
that the plaintiff has married, pretend that we do not know it and assess the damages, as we are assessing them anew here, on the
footing that she may or may not marry? As we know the truth, we are not bound to believe in a fiction. So, in my judgment, the
fact of her re-marriage within the time within which an appeal could properly be brought (I say nothing about what would happen
after that time), just like the fact of death within such a time, seems to me to be a matter with which the higher court should
concern itself.
As to the evidence, it has been quite insufficient. As to the burden, it is on those who wish to support the new facts. The
defendants have not done so here with any accuracy or with any adequacy, and, that being so, it may be that they do not reap the
full result which would come from more certainty. Doing the best we can with the inadequate materials before us, I agree in the
conclusion that my lord has reached.

PEARSON LJ. I agree. It appears from RSC, Ord 58, r 3(1), that An appeal to the Court of Appeal shall be by way of re-
hearing As my lord has pointed out, in A-G v Birmingham, Tame & Rea District Drainage Board ([191113] All ER Rep 926
at p 939; [1912] AC 788 at p 801), Lord Gorell dealt with this matter, and he said in terms

It seems clear, therefore, that the Court of Appeal is entitled and ought to re-hear the case as at the time of re-hearing

623

It follows, therefore, that, once the evidence of the new event (that is to say, the event which has occurred after the date of the
trial or hearing) has been admitted, the new event should be taken fully into account and the decision should be given in the light
of all the evidence, including the evidence as to the new event which has occurredin this case, the re-marriage of the plaintiff,
who brought the action under the Fatal Accidents Act, 1846.
The really essential decision, therefore, is on the question whether the new evidence should be admitted or not. There, one
has the provisions of RSC, Ord 58, r 9(2):

The Court of Appeal shall have full discretionary power to receive further evidence upon questions of fact [in certain
ways which are specified]: Provided that in the case of an appeal from a judgment after trial or hearing of any cause or
matter upon the merits, no such further evidence (other than evidence as to matters which have occurred after the date of
the trial or hearing) shall be admitted except on special grounds.

I read that as meaning that, where evidence is sought to be adduced of a new event (in the sense that I have indicated), there is a
discretionary power to admit it or to refuse to admit it, but that no special grounds are required. I suppose that the special
grounds are those referred to in the well known case of Ladd v Marshall ([1954] 3 All ER 745 at p 748); but, if one looks at
those special grounds, they would seem all to be fulfilled in this case.
However, I think it right to emphasise what has already been pointed outthat, in this case, the event in question occurred
quite soon after the trial or hearing, and in fact within the time limited for serving the notice of appeal. This case, as it stands, is
limited to a case on those facts, and I do not propose to say what the position would be if the event had occurred at some later
stage. I do feel anxiety on this subject, because the normal rule in accident cases is that the sum of damages falls to be assessed
once for all at the time of the hearing. When the assessment is made, the court has to make the best estimate it can as to events
that may happen in the future. If further evidence as to new events were too easily admitted, there would be no finality in such
litigation. There are quite often uncertain matters which have to be estimated and taken into account to the best of the ability of
the judge trying the action. For instance, to give obvious examples, the court often has to consider whether an operation on the
patient is likely to be successful or not, and, if so, how successful, and what result it is likely to achieve and, therefore, whether
the plaintiff is likely to suffer from a continuing disability or not. That is a very important matter in the assessment of damages,
and quite often the continuing disability is foreseen merely as an uncertain future possibility, or at most as a future probability.
Another question which very frequently has to be taken into account in assessing damages in accident cases is whether and when
osteo-arthritis is likely to supervene. We are not in this case faced with the problem of when the new event has to occur in order
that the evidence of it may be properly admitted as a matter of discretion under RSC, Ord 58, r 9(2). That is a matter to be
considered in the future. On that aspect, this is a comparatively easy case, because the new event occurred within a short time
after the hearing.

Damages reduced.

Solicitors: Speechly, Mumford & Soames agents for Saul & Lightfoot, Carlisle (for the defendants); Lees, Smith & Crabb agents
for Gaitskell, Dodgson & Bleasedale, Whitehaven (for the plaintiff).

Henry Summerfield Esq Barrister.


624
[1963] 2 All ER 625

H West & Son Ltd and Another v Shephard


TORTS; Negligence

HOUSE OF LORDS
LORD REID, LORD TUCKER, LORD MORRIS OF BORTH-Y-GEST, LORD DEVLIN, LORD PEARCE
18, 19, 20, 21 MARCH, 27 MAY 1963

Negligence Damages Personal injury Measure of damages General damages Objective and subjective elements
Assessment by comprehensive sum Unconsciousness of injured person Woman permanently bedridden Slight sign of
consciousness Never likely to be able to use award.

(i) General damages for personal injuries are to compensate for results that have actually been caused, which may consist both of
physical loss (eg, loss of the use of a limb) which is an objective element of damages, and of pain and suffering, of frustration and
the affliction of awareness of the loss and of loss of expectation of life, which form a subjective element, and in relation to both
these elements, the period of probable duration (which may be the period of expectation of life of the plaintiff) is relevant to be
taken into consideration (see p 633, letter d), p 636, letter f, p 637, letter c, p 634, letter g, p 643, letter c, and p 630, letter d,
post); an injured person who is rendered continuously unconscious is spared the subjective element, and thus cannot recover
compensation for it, but a court is not required to segregate and assess separately the objective and subjective elements, both of
which may properly be covered by one comprehensive sum (see p 628, letter i, p 633, letter g, p 634, letter f, p 643, letter d, and p
630, letter d, post).
(ii) In assessing general damages for personal injuries there may be items of need which should be taken into account (eg,
future needs for special treatment or for transport), but, apart from that, the consideration of what use a plaintiff will thereafter
make of the money recovered as damages is irrelevant (see p 633, letter h, p 642, letter f, and p 630, letter d, post; cf p 641, letter
i, and p 629, letter a, post).
The appellants were liable for negligence that caused grievous injuries to the respondent, a woman who, at the time of the
accident in 1959, was forty-one years of age and the mother of three children. As a result of her injuries she became permanently
bedridden and in need of continuous nursing attention in hospital; she was unable to speak, but could appreciate the difference
between articles of food that she liked or disliked, showing her likes and dislikes by facial expressions; she could show some sign
of recognition of relatives and of members of the nursing staff, and she could respond to commands by moving her right hand.
Her expectation of life was seven years from the accident. She was maintained in hospital under the National Health Service.
Damages were assessed at the trial under four heads, viz, (a) 500 as agreed special damages; (b) 600 for future loss of earnings;
(c) 500 for loss of expectation of life; (d) 17,500 for general damages, of which 2,500 were attributable to the possibility of
the respondents being aware of her present condition. On appeal as to (d), that is, as to general damages only,

Held (Lord Reid and Lord Devlin dissenting): the award of general damages should stand, there having been no error in
principle (see (i) above) in assessment and, though the figure of 17,500 might seem high, it was neither unreasonable nor so
excessive as to justify an appellate courts interference, having regard to the fact that, on the evidence, the respondents state was
such that she might be aware that she had lost all the amenities of life (see p 634, letter i, p 635, letter i, to p 636, letter a, p 646,
letter c, and p 630, letter d, post).
Wise v Kaye ([1962] 1 All ER 257) approved.
Per Lord Morris of Borth-y-Gest and Lord Pearce (Lord Tucker concurring): the guidance given inBenham v Gambling
([1941] 1 All ER 7) was intended to apply solely to the assessment of damages in respect of a special head of damages, loss of
expectation of life, arising in consequence 625 of the Law Reform (Miscellaneous Provisions) Act, 1934, s 1 (see p 632, letter h,
p 644, letter e, and p 630, letter d, post; compare p 640, letter h, to p 641, letter a, and p 629, letter h, post).
Per Lord Pearce: the loss of happiness is not a correct guide to reasonable compensation (see p 645, letter d, post; cf p 628,
letter e, and p 629, letter g, post).
Appeal dismissed.

Notes
Lord Reid and Lord Devlin held that the general damages should be reduced. Perhaps the principal divergence between their
view and that of the majority was that the degree of moderation, adopted by the assessment decided on in Benham v Gambling
([1941] 1 All ER 7), should be extended to the objective element in assessing the general damages in the present case (see p 629,
letter i, and p 641, letter e, post).
As to the measure of damages for personal injury, see 11 Halsburys Laws (3rd Edn) 255, para 427; and for cases on the
subject, see 36 Digest (Repl) 200, 10531057, 229, 12091234

Cases referred to in the opinions


Admiralty Comrs v Susquehanna (Owners), The Susquehanna,[1926] All ER Rep 124, [1926] AC 655, 95 LJP 128, 135 LT 456,
17 Asp MLC 81, 33 Com Cas 1, 41 Digest 802, 6624.
Benham v Gambling, [1941] 1 All ER 7, [1941] AC 157, 110 LJKB 49, 164 LT 290, 36 Digest (Repl) 231, 1227.
Davies v Powell Duffryn Associated Collieries, Ltd (No 2), [1942] 1 All ER 657, [1942] AC 601, 111 LJKB 418, 167 LT 74, 36
Digest (Repl) 231, 1229.
Flint v Lovell,[1934] All ER Rep 200, [1935] 1 KB 354, 104 LJKB 199, 152 LT 231, 36 Digest (Repl) 200, 1055.
Greta Holme, The, [1895-9] All ER Rep 127, [1897] AC 596, 66 LJP 166, 77 LT 231, 8 Asp MLC 317, revsg SC sub nom The
Emerald, The Greta Holme, [1896] P 192, CA, 17 Digest (Repl) 83, 56.
Halliday or Reid v Lanarkshire Traction Co, 1934 SC 79, 1934 SLT 54.
Hebridean Coast, The, Owners of Steamship Lord Citrine v Owners of Motorship or Vessel Hebridean Coast, [1961] 1 All ER 82,
[1961] AC 545, [1961] 2 WLR 48, 3rd Digest Supp.
McGrath Trailer Equipment Pty, Ltd v Smith, [1956] VLR 738.
Mediana (Owners) v Comet (Owners), The Mediana, [1900-3] All ER Rep. 126, [1900] AC 113, 69 LJP 35, 82 LT 95, 9 Asp
MLC 41, HL, 17 Digest (Repl) 76, 10.
Mills v Stanway Coaches, Ltd, [1940] 2 All ER 586, [1940] 2 KB 334, 109 LJKB 648, 163 LT 68, 36 Digest (Repl) 229, 1211.
Oliver v Ashman, [1961] 3 All ER 323, [1962] 2 QB 210, [1961] 3 WLR 669, 3rd Digest Supp.
Phillips v London & South Western Ry Co, (1879), 5 QBD 78, 41 LT 121, 43 JP 749, previous proceedings, (1879), 4 QBD 406,
subsequent proceedings, 5 CPD 280, 49 LJQB 223, 42 LT 6, 44 JP 217, CA, 17 Digest (Repl) 190, 860.
Quinn v Leatham, [1900-3] All ER Rep 1, [1901] AC 495, 40 LJPC 76, 85 LT 289, 65 JP 708, 42 Digest 971, 30.
Reid v Lanarkshire Traction Co, Ltd, 1934 SC 79, 1934 SLT 54.
Rose v Ford, [1937] 3 All ER 359, [1937] AC 826, 106 LJKB 576, 157 LT 174, 36 Digest (Repl) 229, 1210.
Rowley v London & North Western Ry Co, (1873), LR 8 Exch 221, 42 LJ Ex 153, 29 LT 180, 17 Digest (Repl) 185, 808.
Slater v Spreag, [1935] All ER Rep 900, [1936] 1 KB 83, 105 LJKB 17, 153 LT 297, 36 Digest (Repl) 229, 1209.
Wise v Kaye, [1962] 1 All ER 257, [1962] 1 QB 638, [1962] 2 WLR 96.
626

Appeal.
This was an appeal from a decision of the Court of Appeal (Lord Denning, MR, Upjohn and Diplock, LJJ) given on Oct 4, 1962,
upholding a decision of Paull J, dated May 3, 1962, whereby damages of 19,100 were awarded to the respondent for serious
injuries received by her in an accident due to the negligence of the second appellant in the course of his employment by the first
appellant. General damages were assessed at 17,500, of which 2,500 (the excess of 17,500 over the 15,000 awarded inWise
v Kayea) were awarded on the basis that the respondent might have some realisation of her condition, a factor not present in Wise
v Kaye.
________________________________________
a [1962] 1 All ER 257; [1962] 1 QB 638

By her statement of claim delivered on June 7, 1961, the respondent alleged that on Nov 25, 1959, she was knocked down,
when crossing a street, by a motor lorry belonging to the first appellants and driven by their servant, the second appellant; that
she received serious head injuries as a result of which she had been unconscious ever since the accident. Negligence was held
proved at the trial. The judgment of Paull J, included the following:

Then comes the extremely difficult question, what about general damages? Quite recently there has been a case, Wise
v Kayea. In Wise v Kaye very much the same situation developed as here, namely, you had a woman who was then aged
twenty, who was reduced in one sense to an even worse condition than this woman has been, because there she had really
no conscious life at all, and in those circumstances, there not being very much evidence in regard to expectation of life, the
judge, Finnemore J, gave 15,000 as a reasonable figure for general damages, and the Court of Appeal upheld that figure.
Now, there is a factor in this case which was absent in that case, and that is that here you have a woman who has recovered
to the extent that she may well appreciate, at least to some extent, the condition in which she is. She does know the
difference between food which is pleasant and food which is unpleasant. She does know the difference between different
colours and she does know the difference between different numbers up to nine, and she does recognise her relations, too.
What then? From the point of view of damages is she in a better condition or a worse condition? My own view is that she
is in a worse condition, for the simple reason that having the ability to do these things means that she must realise to some extent
the condition in which she is lying. Nobody can tell; she cannot express it to anybody, but she knows what she likes and what she
does not like. She can express that facially, and, if she can do that, she must have some sort of knowledge of the condition into
which she has been put by this accident. I just do not know what sort of damages to award a woman in this condition. If she had
a longer life expectation, I think that damages would come well up into the twenties by way of thousands of pounds, because I
think that there can be no sadder case than a case of this kind. However, I have got to take into account that she may well die
within five years. Clearly she has lost all the amenities of life, clearly she has got to be looked after, and she may well recognise
the condition in which she is, and in her mind may be the most appalling thoughts as to the conditioning which she is. I do not
know. I think that in a case of this sort the proper sum to award for general damages is 17,500, which plus 1,600, b, makes
19,100.
________________________________________
b Viz, the aggregate of 500, special damages, 600, loss of earnings, and 500, loss of expectation of life

C P Harvey, QC, and T M Eastham for the appellants.


Mark Littman, QC, and A L J Lincoln for the respondent.

Their Lordships took time for consideration


627

27 May 1963. The following opinions were delivered.

LORD REID. My Lords, I have had an opportunity of reading the speech which has been prepared by my noble and learned
friend Lord Devlin and I am in general agreement with it. I need not set out the facts of this case and I can go straight to the
question of general importanceWhat is the basis on which damages for serious injuries are awarded? The determination of that
question in the ordinary case where the injured person is fully conscious of his disability will go far to decide how to deal with a
case like Wise v Kaye where the injured person was wholly unconscious with no prospect of ever regaining consciousness or like
the present case where the respondent is only conscious to a slight extent.
In the ordinary case of a man losing a leg or sustaining a permanent internal injury, he is entitled to recover in respect of his
pain and suffering: if he is fortunate in suffering little pain, he must get a smaller award. So it is not disputed that where an
injured person does not suffer at all because of unconsciousness he gets no award under this head. Nothing was awarded in
Wises case and nothing has been awarded in this case. On the other hand no one doubts that damages must be awarded
irrespective of the mans mental condition or the extent of his suffering where there is financial loss. That will cover the cost of
treatment or alleviation of his condition just as much as it covers the cost of repairing or renewing his property. And it will cover
loss of earning power: there may be a question whether some deduction should be made where his outgoings will be less than
they would have been if there had been no accident, so as to reach his net financial loss, but that does not arise in the present case.
The difficulty is in connexion with what is often called loss of amenity and with curtailment of his expectation of life. If there
had been no curtailment of his expectation of life, the man whose injuries are permanent has to look forward to a life of
frustration and handicap and he must be compensated, so far as money can do it, for that and for the mental strain and anxiety
which results. But I would agree with Sellers LJ in Wises case ([1962] 1 All ER at p 263; [1962] 1 QB at p 651) that a brave man
who makes light of his disabilities and finds other outlets to replace activities no longer open to him must not receive less
compensation on that account.
There are two views about the true basis for this kind of compensation. One is that the man is simply being compensated for
the loss of his leg or the impairment of his digestion. The other is that his real loss is not so much his physical injury as the loss
of those opportunities to lead a full and normal life which are now denied to him by his physical conditionfor the multitude of
deprivations and even petty annoyances which he must tolerate. Unless I am prevented by authority I would think that the
ordinary man is, at least after the first few months far less concerned about his physical injury than about the dislocation of his
normal life. So I would think that compensation should be based must less on the nature of the injures than on the extent of the
injured mans consequential difficulties in his daily life. It is true that in practice one tends to look at the matter objectively and
to regard the physical loss of an eye or a limb as the subject for compensation. But I think that is because the consequences of
such a loss are very much the same for all normal people. If one takes the case of injury to an internal organ, I think that the true
view becomes apparent. It is more difficult to say there that the plaintiff is being paid for the physical damage done to his liver or
stomach or even his brain, and much more reasonable to say that he is being paid for the extent to which that injury will prevent
him from living a full and normal life and for what he will suffer from being unable to do so.
If that is so, then I think it must follow that if a mans injuries make him wholly unconscious so that he suffers none of these
daily frustrations or inconveniences, he ought to get less than the man who is every day acutely conscious of what he suffers and
what he has lost. I do not say that he should get nothing. This is not a question that can be decided logically. I think that there
are two elements, what he has lost and what he must feel about it, and of the two I think the 628 latter is generally the more
important to the injured man. To my mind there is something unreal in saying that a man who knows and feels nothing should
get the same as a man who has to live with and put up with his disabilities, merely because they have sustained comparable
physical injuries. It is no more possible to compensate an unconscious man than it is to compensate a dead man. The fact that
the damages can give no benefit or satisfaction to the injured man and can only go to those who inherit the dead mans estate
would not be a good reason for withholding damages which are legally due. But it is, in my view, a powerful argument against
the view that there is no analogy between a dead man and a man who is unconscious and that a man who is unconscious ought to
be treated as if he were fully conscious.
It is often said that it is scandalous that it should be cheaper to kill a man than to maim him, and that it would be monstrous
if the defendant had to pay less because in addition to inflicting physical injuries he had made the plaintiff unconscious. I think
that such criticism is misconceived. Damages are awarded not to punish the wrong-doer but to compensate the person injured,
and a dead man cannot be compensated. Loss to his estate can be made good, and we can give some compensation to those
whom he leaves behind. Perhaps we should do more for thembut not by inflating the claim of the dead mans executor, for
then the money may go to undeserving distant relatives or residuary legatees or even to the Treasury if he dies intestate and
without heirs. And it is already the case that it may benefit the defendant to injure the plaintiff more severely. If he is injured so
severely that he can only live a year or two at most the damages will be much less than if he is less severely injured so that he
may survive for many years. And that brings me to the other matter of loss of expectation of life.
There is no dispute about the claim for loss of expectation of life in this case, but the appellant relies on Benham v
Gambling, so I must deal with the matter at least in a general way. One must have in mind the position when Benhams case
came before this House. It had been established in Rose v Ford that a plaintiff is entitled to damages for the tortious shortening
of his expectation of life whether or not he knew that this expectation had been curtailed. But the measure of damages was left so
vague that in practice this head of claim got out of hand. In Mills v Stanway Coaches, Ltd ([1940] 2 All ER 586 at p 593; [1940]
2 KB 334 at p 346) Lord Goddard said that damages awarded under this head have increased and are increasing and ought as I
think to be diminished. Then in 1941 (Benham v Gambling [1941] 1 All ER 7; [1941] AC 157) this House decided that these
damages must be diminished. I cannot interpret that decision as anything other than a decision based on policy, but justified by
assumptions more philosophical than legal. The decision stands that damages under this head must be limited to a low
conventional figure and I would be the last to question that. But I do not accept that the decision has established that damages
under other heads must be assessed by reference to a hedonistic profit and loss account of happiness and unhappiness, or pleasure
and pain. It is now a rule of law that, if a man is cut off in the prime of life, then no matter how bright his prospects only a
conventional sum of 500 or so can be awarded in respect of his lost years. But, if it were true that no mans future on earth is
worth on balance more than that, I do not see how we would justify awards of 10,000 and more for loss of amenity. So I would
not apply what Viscount Simon, LC, said ([1941] 1 All ER at p 13; [1941] AC at p 168) to any other class of case. All that I
would take from Benhams case is that in assessing damages on an objective basis, independently of what the injured person
knew or felt, a low figure was taken. That is some justification for taking a moderate figure for the objective element in a claim
by a living person for loss of amenity and attaching more importance to what he knows and feels about his deprivation than to his
actual injuries.
629
Coming to the facts of this case I would accept the sum of 1,600 which has been awarded to cover special damage, loss of
earnings and loss of expectation of life, but I would reduce the general damages of 17,500. I would consider separately the
objective and the subjective the element arising from the respondents injuries. Accepting that in view of her shortened
expectation of life 17,500 would be a fair sum if the respondent were fully conscious of her position, I would think that not
more than 5,000 of that ought to be attributed to the actual physical injuries, and then the question is to what extent the
respondent is conscious and suffering. Unfortunately we have nothing to go by except three medical reports, and on this matter
they do not take us very far. It would seem that the respondent has some, but not very much, appreciation of her surroundings
and she seems to suffer no pain. I think that perhaps 4,000 would be appropriate here. Then perhaps insufficient attention has
been given to expense which her husband may incur in tending her and providing amenities, if her condition should improve
slightly. So I would substitute a figure in the region of 10,000 for the sum of 17,500 which has been awarded.

LORD TUCKER. My Lords, the opinion which I am about to read is that of my noble and learned friend Lord Morris of Borth-
y-Gest. I am in complete agreement with the reasoning and conclusions of that opinion.
LORD MORRIS OF BORTH-Y-GEST, (read by Lord Tucker): My lords, the grievous injuries in respect of which, as has been
held, liability rests on the appellants, were sustained by the respondent on Nov 28, 1959. She was then forty-one years of age, the
wife of a devoted husband and the mother of three children. On that date she was removed to hospital in an unconscious
condition. Her clinical state suggested that there was intracranial bleeding and an operation was performed on the same day in
order to evacuate haemorrhages. There was also bruising of the underlying brain. Following the operation her level of
consciousness lightened somewhat, but she did not regain full consciousness. Later investigations suggested a marked degree of
cerebral atrophy on the right side of the brain: there was asymmetry between the two hemispheres and a diffuse abnormality
such as is often seen after head injuries with extensive cerebral contusion. On 1 March 1961, she was transferred to another
hospital. In reference to that date it was said that she was conscious and responded in a negative manner to examination, but that
she lay quite detached from her environment: there was no response to external painful stimuli except slight withdrawal by the
right arm, and she did not respond to any requests. A hospital report stated that all four limbs were spastic, the arms more so
than the legs and there was no voluntary movement present beyond eye movements to follow a moving object. By 22
November 1961, there had been a gradual increase in her awareness of, and response, to her surroundings and a little voluntary
movement was returning to the head and right arm. She could see and hear and she appeared to understand simple requests
though very slowly. In medical language her state was said to be that of post-traumatic spastic quadriplegia and intellectual
deficit. A report dated 16 February 1962, recorded that there has been a slight improvement in her mental condition in that she
showed some signs of recognition of relatives and members of the nursing staff and had responded to commands by moving her
right hand. She could appreciate the difference between articles of food that she liked or disliked, her means of indication being
by means of facial expressions. Her husband, a witness at the trial, noticed that in November 1961, there was some improvement
in her condition. Though she remains unable to speak she has been able to indicate colour in that when her husband mentioned a
colour she touched a coloured card which was held up in front of her: she could just lift her hand from the bed cover and extend
a finger so as to touch the card. When her husband mentioned a number up to but not beyond the number nine she could touch a
card with the mentioned number on it. Accepting the husbands evidence the learned judge at the trial held that, though she could
not speak at all 630 she must be able, to a certain extent, to understand what was said to her.
She needs continuous nursing care in hospital with hourly or two-hourly feeding by tube. Although her general condition
(apart from her neurological condition) is good, she is at great risk from the development of chest and other infections. She will
require full-time hospital nursing for the period that her life continues and there is no prospect of any further improvement in her
condition. On a consideration of the evidence, and balancing the possibilities, the learned judge held that she had an expectation
of life somewhere in the neighbourhood of five years. On that basis her life from the date of the accident for a period of over
seven years will have been lived in the condition, and subject to the deprivations, which I have summarised.
The award of damages made by the learned judge at the trial on 3 May 1962, included (a) a sum of 500, the agreed special
damages which covered the respondents loss of earnings down to the date of trial; (b) 600 for future loss of earnings; (c) 500
for loss of expectation of life, and (d) 17,500 general damages. In the Court of Appeal no challenge was made of the sums
awarded under (a), (b) and (c) above. The challenge which was made of the amount of the general damages failed.
My lords, the damages which are to be awarded for a tort are those which so far as money can compensate, will give the
injured party reparation for the wrongful act and for all the natural and direct consequences of the wrongful act ( Admiralty
Comrs v Susquehanna (Owners), The Susquehanna (Per Viscount Dunedin, [1926] All ER Rep 124 at p 127, [1926] AC 655 at p
661)). The words so far as money can compensate point to the impossibility of equating money with human suffering or
personal deprivations. A money award can be calculated so as to make good a financial loss. Money may be awarded so that
something tangible may be procured to replace something else of like nature which has been destroyed or lost. But money cannot
renew a physical frame that has been battered and shattered. All that judges and courts can do is to award sums which must be
regarded as giving reasonable compensation. In the process there must be the endeavour to secure some uniformity in the general
method of approach. By common assent awards must be reasonable and must be assessed with moderation. Furthermore, it is
eminently desirable that so far as possible comparable injuries should be compensated by comparable awards. When all this is
said it still must be that amounts which are awarded are to a considerable extent conventional.
In the process of assessing damages judges endeavour to take into account all the relevant changes in a claimants
circumstances which have been caused by the tortfeasor. These are often conveniently described as heads of damage. In his
judgment in the much-litigated cause of Phillips v London & South Western Ry Co, Cockburn CJ, referred to some of these. He
mentioned the bodily injury sustained ((1879), 4 QBD at 407) as well as the pain undergone ((1879), 4 QBD at 407), the
effect on the health of the sufferer ((1879), 4 QBD at 407), and items of expense incurred and pecuniary loss suffered. If there
has been some serious physical injury which, as the result of skilled medical attention, has happily not necessitated the enduring
of pain, then it will follow that there will be no question of including in an award any sum as compensation for the enduring of
pain. If someone has been made unconscious so that pain is not felt the like result will follow. Damages are awarded as a fair
compensation for that which has in fact happened and will not arise in respect of anything that has not happened.
In the case to which I have referred Cockburn CJ, made no mention of a head of damages which in recent years has figured
prominently in personal injuries cases viz, damages to give compensation for the fact that the injuries will result in a material
shortening of the length of life. Such damages formed an element in an award made by Acton J, in Flint v Lovell. When that
case 631 went to the Court of Appeal, Roche LJ, said ([1934] All ER Rep at p 206; [1935] 1 KB at p 367) that the admission of
such a head of damage as a separate and independent head was novel and difficult of application and he added ([1934] All ER
Rep at p 206; [1935] 1 KB at p 367): In the hearing of very many cases of personal injuries I do not remember this head of
damage being so treated. The Court of Appeal held that Acton J, had been entitled to take the plaintiffs diminution of
expectancy of life into account. That decision of the Court of Appeal was approved in your lordships House in Rose v Ford. As
a result of the Law Reform (Miscellaneous Provisions) Act, 1934, (See 1, 9 Halsburys Statutes (2nd Edn) 792), a cause of action
which entitles a person to claim damages for loss of an expectation of life survives for the benefit of his estate.
In the years that followed the decisions in Flint v Lovell and Rose v Ford there were wide variations in the amounts which
were awarded in reference to this head of damages whether the claims were made by, and in the lifetime of, those whose lives had
been shortened or were made because causes of action survived where death had occurred. In Benham v Gambling Viscount
Simon, LC, said ([1941] 1 All ER at p 9, [1941] AC at p 161):

It has been recognised by judges who have had to deal with the many cases under this head which have fallen to be
decided in the last few years that the measurement of this head of damage in terms of money is a very difficult matter.

He added that it must be accepted that in cases where a victims life has been shortened ([1941] 1 All ER at p 9, [1941] AC at p
162) some figure to represent the loss suffered by the deceased through the shortening of his life may be included in the
damages and he said that judges had drawn attention to the need for authoritative guidance on the subject of how to arrive at it
([1941] 1 All ER at p 9, [1941] AC at p 162). Such authoritative guidance was given by your lordships House. It was given in
reference to that particular head of damages. The speech of Viscount Simon was only concerned with that head and dealt with no
other. Viscount Simon spoke more than once of assessing damages under this head. He said ([1941] 1 All ER at p 9, [1941]
AC at p 162):

The present appeal raises the problem of the assessment of damage for loss of expectation of life before this House
for the first time and it is indeed the only issue with which we are now concerned.
My lords, I have never understood that the speech of Viscount Simon was calculated to prescribe for judges a new method of
approach in assessing what damages to award for some bodily injury. If, for example, damages fall to be assessed for the loss of
a leg, I cannot think that it was ever intended that guidance should be sought for in the speech of Viscount Simon in Benham v
Gambling. Indeed I think that Viscount Simon was indicating as much when he said that damages which would be proper for a
disabling injury may well be much greater than for deprivation of life ([1941] 1 All ER at p 13; [1941] AC at p 168). It may be
that a thesis can be formulated leading to the conclusion that the reasoning which guided Viscount Simon towards the result
which he announced is reasoning which logically should be applied when bodily deprivations are the subject-matter for a
monetary award. If this be so then here is an occasion for the reminder that logic is not the life of the law. The guidance given in
Benham v Gambling was, I consider, solely designed and intended to apply to the assessment of damages in respect of the rather
special head of damages for loss of expectation of life.
I pass now to a consideration of the submissions which were made in regard to the award of 17,500 as general damages. It
was submitted that there was a 632 wrong approach in law in arriving at that figure, and alternatively it was submitted that, if the
approach was not wrong in law, the figure of 17,500 was excessive and reflected a wholly erroneous assessment.
An interesting argument was addressed to your lordships which analysed those elements of general damages which may be
regarded respectively as consolatory and as compensatory in their nature. My lords, in reference to a judicial process which must
so often be undertaken such as that of the assessment of damages for personal injuries I would favour simplicity of expression
and an absence to the greatest extent possible of any elaborate or complex formulae. I consider that it is sufficient to say that a
money award is given by way of compensation and that it must take into account the actual consequences which have resulted
from the tort.
Certain particular questions have been raised. How are general damages affected, if at all, by the fact that the sufferer is
unconscious? How are they affected, if at all, if it be the fact that the sufferer will not be able to make use of any money which is
awarded?
The first of these questions may be largely answered if it is remembered that damages are designed to compensate for such
results as have actually been caused. If someone has been caused pain then damages to compensate for the enduring of it may be
awarded. If, however, by reason of an injury someone is made unconscious either for a short or for a prolonged period with the
result that he does not feel pain then he needs no monetary compensation in respect of pain because he will not have suffered it.
Apart from actual physical pain it may often be that some physical injury causes distress or fear or anxiety. If, for example,
personal injuries include the loss of a leg, there may be much physical suffering, there will be the actual loss of the leg (a loss the
gravity of which will depend on the particular circumstances of the particular case) and there may be (depending on particular
circumstances) elements of consequential worry and anxiety. One part of the affliction (again depending on particular
circumstances) may be an inevitable and constant awareness of the deprivations which the loss of the leg entails. These are all
matters which judges take into account. In this connexion also the length of the period of life during which the deprivations will
continue will be a relevant factor (see Rose v Ford). To the extent to which any of these last-mentioned matters depend for their
existence on an awareness in the victim it must follow that they will not exist and will not call for compensation if the victim is
unconscious. An unconscious person will be spared pain and suffering and will not experience the mental anguish which result
from knowledge of what has in life been lost or from knowledge that life has been shortened. The fact of unconsciousness is
therefore relevant in respect of, and will eliminate, those heads or elements of damage which can only exist by being felt or
thought or experienced. The fact of unconsciousness does not, however, eliminate the actuality of the deprivations of the
ordinary experiences and amenities of life which may be the inevitable result of some physical injury.
If damages are awarded to a plaintiff on a correct basis, it seems to me that it can be of no concern to the court to consider
any question as to the use that will thereafter be made of the money awarded. It follows that if damages are assessed on a correct
basis, there should not then be a paring down of the award because of some thought that a particular plaintiff will not be able to
use the money. In assessing damages there may be items which will only be awarded if certain needs of a plaintiff are
established. A particular plaintiff may have provision made for some future form of transport: a particular plaintiff may have to
have provision made for some special future attention or some special treatment or medication. If, however, some reasonable
sum is awarded to a plaintiff as compensation for pain endured or for the loss of past or future earnings or for ruined years of life
or lost years of life, the use to which a plaintiff puts such sum is a matter for the plaintiff alone. A rich man, merely because he is
rich and is not in need, is not to 633 be denied proper compensation: nor is a thrifty man merely because he may keep and not
spend.
With this approach I turn to the question whether there is any error in principle in the assessment of damages made by Paull
J. He referred to the case of Wise v Kayewhich was a case where a young woman, twenty years of age, received serious brain
injuries in an accident in May, 1958. Thereafter she remained in hospital in a helpless and unconscious condition. The medical
evidence established that there was no prospect of her recovery and that she would die at some unspecified time of an intercurrent
complaint which would take place during her illness. At a trial on Feb 3 1961, she was awarded the sum of 15,000 as general
damages in addition to amounts of damages for loss of earnings and for loss of probable future earnings and for loss of
expectation of life. On 1 December 1961, the Court of Appeal by a majority upheld the award of 15,000 general damages.
In his judgment in the present case Paull J, pointed out that there was the factor, absent in Wise v Kaye, that the respondent
might well appreciate, at least to some extent, the condition in which she was and for that reason was probably in a worse
condition than was the plaintiff inWise v Kaye. After stating that had the respondent in the present case had a longer expectation
of life he would have awarded higher damages, Paull J, said:

However, I have got to take into account that she may well die within five years. Clearly she has lost all the amenities
of life, clearly she has got to be looked after, and she may well recognise the condition in which she is, and in her mind
may be the most appalling thoughts as to the condition in which she is. I do not know. I think that in a case of this sort the
proper sum to award for general damages is 17,500.

The learned judge was clearly approaching some matters on an objective basis and others on a subjective basis. Experienced
judges are well accustomed to weigh up all relevant matters and then to express a comprehensive conclusion, and it would not be
profitable and would only be burdensome if they were expected to segregate those matters which they have viewed objectively
and those which they have viewed subjectively and then to make precise and detailed allocations of money for each separate
component matter which they had considered.
My lords, leaving aside for the moment the question whether the amount is, as an amount, excessive, I can see no fault in the
approach of the learned judge. It is necessary to have in mind the matters for which he was awarding these general damages.
Accepting the estimate as to the respondents expectation of life, damages were to be given to cover a period of over seven years.
At the age of forty-one everything that life held for her was taken away from her. For a period of about seven years instead of
having lifes activities and amenities she will have mere existence but little else, save that, to the extent that I have described, she
may have the torment of a realisation of her helplessness. If in some degree she has processes of thought, she has the agony and
frustration of being unable to convey her thoughts or to give them expression. All these matters constitute grave and sombre
deprivations for which in my view she is entitled to receive substantial compensation.
Unless the approach of the majority in the Court of Appeal in Wise v Kaye was erroneous, I detect no error of approach on
the part of Paull J, in the present case. I have already expressed the opinion that the decision in Benham v Gambling was not
calculated to lay down a new basis for the assessment of general damages and should not be regarded as having done so. On the
matters which are relevant in the present appeal I respectfully express my concurrence with the views of the majority in Wise v
Kaye and with the reasoning which 634 guided the powerful judgments of Sellers LJ, ([1962] 1 All ER at p 259; [1962] 1 QB at p
644) and Upjohn LJ ([1962] 1 All ER at p 265; [1962] 1 QB at p 655).
With the question of principle which was decided inWise v Kaye as to the basis to be taken in assessing future loss of
earnings your lordships are not concerned. The recorded evidence in that case was not very explicit as to the reduction in the
expected span of life: apparently the doctors could not give an estimate, though it was accepted that the plaintiff could not be
expected to endure for anything approaching a normal span of life. There were some similarities but also some differences
between the condition of the plaintiff in that case and the condition of the respondent in the present case. Sellers LJ, thus
described the condition of Miss Wise ([1962] 1 All ER at p 260; [1962] 1 QB at p 646):

There is no hope of real recovery and but little hope even of a degree of consciousness, although the plaintiff still
lives, breathing with periodic aid from a tube in the lungs, revealing faint signs of a waking and sleeping rhythm and
maintaining a steady weight. The plaintiff cannot eat or drink but her body can obtain nourishment from food fed to her
and so she lives on.

I am of the opinion that there was no error of approach on the part of Paull J, in the present case and that the approach of the
majority (on the matters now in issue) in the Court of Appeal inWise v Kaye should be indorsed. The remaining question is
whether the award of 17,500 was excessive. As the case undoubtedly has many features of comparison withWise v Kaye ([1962]
1 All ER at p 260; [1962] 1 QB at p 646) the issue may be raised whether the sum of 15,000 in that case was excessive. InWise
v Kaye an argument was addressed to the Court of Appeal that the award was in any event too high. That submission was
rejected. Sellers LJ ([1962] 1 All ER at p 264; [1962] 1 QB at p 653), said ([1942] 1 All ER at pp 664, 665; [1942] AC at p 617):

The learned judge has assessed a sum of 15,000 for these physical injuries with all their attendant deprivations. The
circumstances are unprecedented. The only factor I can see which would operate to modify the sum awarded is that the
plaintiff may not live very much longer, but on the other hand she may. Damages of this character must be fair and
reasonable and I do not feel able to say that this award is not.

My lords, unless there is revealed some error in principle, on what basis can the figure of 17,500 be attacked? The attack
must be on the basis that to someone apprised of the facts the figure seems startling and suggests that it cannot be right. The
well-known principles which guide an appellate court were expressed by Lord Wright in his speech inDavies v Powell Duffryn
Associated Collieries, Ltd (No 2) in the course of which he said:

It is not enough that there is a balance of opinion or preference. The scale must go down heavily against the figure
attacked if the appellate court is to interfere whether on the ground of excess or insufficiency.

The difficult task of awarding money compensation in a case of this kind is essentially a matter of opinion of judgment and of
experience. In a sphere in which no one can predicate with complete assurance that the award made by another is wrong the best
that can be done is to pay regard to the range and limits of current thought. In a case such as the present it is natural and
reasonable for any member of an appellate tribunal to pose for himself the question as to what award he himself would have
made. Having done so, and remembering that in this sphere there are inevitably differences of view and of opinion, he does not
however proceed to dismiss as wrong a figure of an award merely because it does not correspond with the figure of his own
assessment. My lords, the figure of 17,500 may seem high but I am not persuaded that the learned judge who fixed it and those
who 635 approved of it are shown to have been in error. I ought not to differ from them unless I am persuaded that they have
awarded or allowed a figure which is shown to be unreasonable and excessive and out of proportion having regard to the injuries
sustained. I ought not to overrule their discretion and their act of judgment unless I am so satisfied. As I am not, I would dismiss
the appeal.

LORD DEVLIN, (read by Lord Pearce): My lords, this is in effect an appeal from the decision of the Court of Appeal inWise v
Kaye, in which the Court of Appeal by a majority upheld an award of 17,400 general damages to a plaintiff, who had been made
completely unconscious by the accident for which the defendant was responsible and was presumed (correctly as the event
showed) to remain so for the rest of her life. Because of her unconsciousness she suffered nothing, and because she was cared for
by the state, there was no claim for the cost of nursing her and keeping her alive. Amounts of 2,000 and 400 were allocated
respectively to loss of future earnings and loss of expectation of life. The difference of opinion in the Court of Appeal arose over
the residual figure of 15,000.
The present case was pleaded as one of unconsciousness. But the evidence which your lordships have already noted shows
that the respondent has some command over her faculties and Paull J, considered that she may well appreciate to some extent
the condition in which she is. This state is expected to last till death. Paull J, therefore applied the principle inWise v Kaye and
took as the appropriate residual figure the same figure of 15,000; but added to it 2,500 because of some sort of knowledge
which she must have of her condition. His judgment was upheld in the Court of Appeal and the lords justices did not add
substantially to the reasoning of the majority in Wise v Kaye.
The case raises a fundamental question on the nature of damages for personal injury. There must be compensation for
medical expenses incurred and for loss of earnings during recovery; these are easily quantified, whether as special or as general
damage. Then there is compensation for pain and suffering both physical and mental. This is at large. It is compensation for
pain and suffering actually experienced. Loss of consciousness, however caused, whether by the injury itself or produced by
drugs or anaesthetics, means that physical pain is not experienced and so has not to be compensated for; and this must be true
also of mental pain. Then there is or may be a temporary or permanent loss of a limb, organ or faculty. Whether it is the limb
itself that is lost or the use of it is immaterial. What is to be compensated for is the loss of use and the deprivation thereby
occasioned. This deprivation may bring with it three consequences. First, it may result in loss of earnings and they can be
calculated. Secondly, it may put the victim to expense in that he has to pay others for doing what he formerly did for himself; and
that also can be calculated. Thirdly, it produces loss of enjoyment, loss of amenities as it is sometimes called, a diminution in the
full pleasure of living. This is incalculable and at large. This deprivation with its three consequences is something that is
personal to the victim. You do not, for instance, put an arbitrary value on the loss of a limb, as is commonly done in an accident
insurance policy. You must ascertain the use to which the limb would have been put, so as to ascertain what it is of which the
victim has actually been deprived.
What has to be considered in the present case is the method of compensation for the third of these consequences, loss of
enjoyment or pleasure. There is here an almost total loss of use of all the faculties or limbs, but compensation under this head
must be assessed in the same way as it would be for a partial loss of a single limb or faculty. The degree is different, but not the
principle.
There are two ways in which this loss of enjoyment can be considered. It can be said that from beginning to end it is really
all mental suffering. Loss of enjoyment is experienced in the mind and nowhere else. It may start with acute distress at the
inability to use a limb in games or exercise as before or just in getting about, and may end with a nagging sense of frustration. If
this is the true view, then total 636 unconsciousness as in Wise v Kaye relieves all mental suffering, and nothing can be recovered
for a deprivation which is not being experienced.
The other way to look on the deprivation of a limb is as the loss of a personal asset, something in the nature of property. A
limb can be put both to profitable use and to pleasurable use. In so far as it is put to profitable use, the loss is compensated for by
calculating loss of earnings and not by assessing mental pain. On the same principle, it can be said, a sum must be assessed for
loss of pleasurable use irrespective of whether there is mental suffering or not. It used at one time to be thought that damages
could not be given for the loss of use of property that was not profit-earning, but that idea has not survived The Greta Holme and
The Mediana.
My lords, as might be expected, English law has not come down firmly in favour of either of these two ways to the
exclusion of the other. It favours a compound of both, as was agreed in argument and as I shall show later by reference to the
authorities. The elements to be compounded have been called the objective and the subjective. The loss of property element is
objective; it requires some sort of valuation that is in no way dependent on the victims sense of loss. The other element is
subjective because it depends entirely on mental suffering actually experienced. Is the main,or, at least, a very substantial
element in the compound, the objective, so that an evaluation must be made of it with an addition for mental suffering when
proved? Or is the main element mental suffering laid on an objective bedrock, so that some sum is always recoverable even
where there is no mental suffering at all? Paull J, has proceeded on the former view. He has taken 15,000,the same as in
Wise v Kaye,as a sum determined objectively for the loss of all faculties. To this he has added 2,500 for suffering caused to
the respondent by her partial appreciation of her state. Since the learned judge approached the problem in this way, I think that
you lordships are bound to inquire whether Wise v Kaye was in principle rightly decided. In my opinion it was not, and the
appropriate figure in that case should have been about one-tenth of what it was, as Diplock LJ, thought ([1962] 1 All ER at p 277;
[1962] 1 QB at p 674). But even if there were no error in principle, I should consider that the sums awarded both in Wise v Kaye
and in the present case were inordinately high and should be reduced on that ground alone. It is convenient that I should express
my views on this second point before I turn to the important question of principle.
Phillips v London and South Western Ry Co is a case that is often quoted as showing the right direction to be given to a jury
when they are awarding compensation for personal injuries. In his direction to the jury, Field J, said:

it has been pointed out for centuries, and it is the principle of foreign jurisprudence as well as ours, that in actions
for personal injuries of this kind, as well as in many others, it is wrong to attempt to give an equivalent for the injury
sustained. I do not mean to say that you must not do it, because you are the masters and are to decide; but I mean that it
would operate unjustly, and in saying so I am using the language of the great Baron Parke (In Blake v Midland Ry Co,
(1852), 18 QB 93) whose opinion was quoted with approval in Rowleys case ((1873), LR 8 Exch 221 at p 231). Perfect
compensation is hardly possible, and would be unjust.
In Rowleys case Brett J ((1873), LR 8 Exch 221 at p 231), had approved of and laid down the rule as declared by Baron Parke (In
Blake v Midland Ry Co, (1852), 18 QB at p 105) that the jury

must not attempt to give damages to the full amount of a perfect compensation for the pecuniary injury, but must take
a reasonable view of the case, 637and give what they considered under all the circumstances a fair compensation.
The direction of Field J, based on this rule, was approved by Cockburn CJ (Phillips v London & South Western Ry Co, (1879), 4
QBD 406 at p 407).
What is meant by compensation that is fair and yet not full? I think it means this. What would a fair-minded man, not a
millionaire, but one with a sufficiency of means to discharge all his moral obligations, feel called on to do for a plaintiff whom by
his careless act he had reduced to so pitiable a condition? Let me assume for this purpose that there is normal consciousness and
all the mental suffering that would go with it. It will not be a sum to plumb the depths of his contrition, but one that will enable
him to say that he has done whatever money can do. He has ex hypothesi already provided for all the expenses to which the
plaintiff has been put and he has replaced all the income which she has lost. What more should he do so that he can hold up his
head among his neighbours and say with their approval that he has done the fair thing?
I think that he would say in an extreme case like this that he would provide such a sum as would ensure that for the rest of
her life the plaintiff would not within reason want for anything that money could buy. That would not be perfect; it would not be
full; but it would be as much as money could fairly do. Of course, the extent of a plaintiffs wants depends on the standards of
life to which he or she is accustomed. The law requires that the standard to be taken should be that of a person of average means,
for it does not permit more to be paid to the rich or less to the poor. What amount annually does the average person in this
country have to spend on himself alone and purely on pleasure? One or two hundred pounds perhaps: not more. Suppose the
defendants gave the plaintiff that much again. Suppose that he doubled, trebled or even quadrupled it. That would cover all the
ordinary pleasures that comparative wealth can bring and the defendant cannot be expected to provide a margin large enough to
satisfy fantastic cravings. A defendant who on these terms provided 1,000 a year would be doing, I think, more than a fair-
minded man should be required to do. It would produce in this case a figure of 7,000. The award of 17,500 is two-and-a-half
times that and is based on only partial appreciation of loss. If the learned judge had said that there was total appreciation, he
could not have given less than 20,000 and might have given as much as 30,000. This sort of figure bears no proper relation to
a fair standard. It is far more than a fair-minded plaintiff would ask for from a defendant of reasonable means; and the defendant
who volunteered it would rightly be thought to be giving his money away.
There are three factors in this particular case, not by any means always present in this type of case, which should keep the
damages awarded to the respondent comparatively small. The first is that the respondents life has been cut short. If another
twenty years of life had to be allowed for, the figure of 7,000 would have been 20,000. The second is that grave injuries of this
sort are very often accompanied by severe and prolonged and permanent physical suffering. The third is that no part of the very
heavy medical expenses incurred has to be borne by the defendant. The National Health Service apparently makes no claim. I do
not pause to inquire whether this abstention springs from the state of the law or from a decision of policy. If it is due to the
former, the position could easily be remedied, but there may be good reasons why it is thought preferable that the loss should be
borne by the taxpayers generally. I should imagine that the cost of hospital treatment with all the assiduous nursing required in
this case might easily be 2,000 or 3,000 a year. This for seven years might well equal the total figure of 17,500 awarded in
this case; and twenty years in a private nursing home, which is not an impossible thing to happen, would be a very formidable
item in an award.
My lords, I think, with respect, that the sum awarded in this case represents an attempt to arrive at equivalent compensation;
and that is what the authorities 638 condemn as unjust. Injustice may seem a strong word to use; but the injustice and the
hardship are there, as Diplock LJ, observed in Wise v Kaye ([1962] 1 All ER at p 274; [1962] 1 QB at p 960), even though they
are spread among many. A sum of the order of 15,000 and upwards is not, to speak bluntly, one that any judge or jury would
have awarded purely as a solatium if they thought that it had to come out of the pocket of a person of average means. Cockburn
CJ, said, in Phillips case ((1879), 4 QBD at p 407):

The attempt to award compensation in damages might be attended with ruinous consequences to defendants who
cannot always, even by the utmost care, protect themselves against the carelessness of persons in their employ.

This consideration, it may be thought, does not apply to insurance companies. But the general body of premium-paying policy-
holdersI refer again to the judgment of Diplock LJ, in Wise v Kaye ([1962] 1 All ER at p 274; [1962] 1 QB at p 670)are no
richer than the general body of railway travellers who in the end would have to pay the damages awarded in such cases as Rowley
and Phillips in which the judges laid down the standard which should still govern. The distinction that they drew between
fairness and fullness is now in danger of being lost altogether. The quest after perfect compensation results only in the piling up
of massive sums, which the plaintiffs themselves can probably never use, and which serve only to express the sense of pity which
judges as well as juries must feel for the tragedy of broken lives.
I turn now to consider the part which the objective element should play in the total award. If this were a matter on which the
House could gain no guidance from the authorities, I could see much scope for argument about the relative importance of the
objective and the subjective. But slowly and painfully English law has evolved ways of assessing the incalculable and it is
important that they should be followed and applied as far as possible so that the law may be coherent. I am satisfied on the
authorities that the objective element should be rated low. They are not authorities that are directly applicable; if they were, there
would be no division of opinion in this House. But they are authorities which in my opinion cannot be evaded without grave
injury to the structure of the law of damages for personal injuries and so I think they ought to be followed and applied. As they
are also the authorities which permit the objective element to be considered at all, I must examine them with some care.
This is not a problem that has arisen very frequently. Deprivation is almost always accompanied by knowledge of it in the
victim. So the traditional way of assessing compensation has been by reference to the feelings of the victim. I cannot agree that
judges and juries usually approach the matter by asking themselves objectively what is the value of a leg; they think of what it
must feel like to be a cripple and they award what is often called a solatium. Until medicine had progressed sufficiently to keep
unconscious persons alive for an indefinite period, there was never any need in the case of the living for drawing a hard and fast
distinction between the objective and the subjective. The problem arose only in the case of the dead when a sum had to be
awarded to compensate for loss of expectation of life. What has to be compensated for in this assessment is a total loss of
enjoyment of all the faculties, a complete loss of the pleasure of living. When the victim knows his fate, he will suffer from the
distress which, except in the most saintly or philosophical, is caused by the prospect of death; and for that clearly he must be
compensated. But what if he never knows his fate? It has been decided that he still must have some compensation, which should
be moderate. The doctrine, I think, originated in Scotland and Lord Sands took the view that the objective element grew out of
the subjective. InReid v Lanarkshire Traction Co (1934 SC 79 at p 84) he said that

while the doctrine of an award in respect of the shortening of life may 639 have originated in the theory of mental
disquiet about the prospect or the possibility of death that doctrine is now a matter positivi juris irrespective of the
presence or absence of evidence as to the sufferers state of mind in the particular case.

But, he said, he should warn the jury that the weight to be given to this element must be moderate and they must not consider
what price the man would have put on his life (1934 SC at p 83).
The problem of the separation of the two elements first arose in England because of the Law Reform (Miscellaneous
Provisions) Act, 1934, (See s 1; 9 Halsburys Statutes (2nd Edn) 792), which allowed the executors of a dead man to sue for
damages for personal injury, the right to which had accrued before the mans death. Clearly, if he had suffered from the prospect
of death, damages could be recovered, as they could be for any other form of physical or mental suffering. But if death came
suddenly, could the executors recover anything at all? It was said that they could not on two grounds. One ground was that there
was no mental suffering in such a case and this was the ground adopted by MacKinnon J, in Slater v Spreag ([1935] All ER Rep
900 at p 903; [1936] 1 KB 83 at p 89) and Humphreys J, in Rose v Ford ([1937] 3 All ER 359; [1937] AC 826, noted at p 827).
This amounted to a denial that there was any objective element in the assessment. The other ground was the technical one that
damages for loss of expectation of life were the same thing as damages for injury inflicted by death and that an old rule of the
common law, left intact by the Act of 1934, forbade a recovery of damages in such a case. Both these arguments were negatived
by this House in Rose v Ford ([1937] 3 All ER 359; [1937] AC 826, noted at p 827). I accept this decision as an authority
binding on your lordships that there is an objective element in damages for loss of enjoyment of life, whether it is caused by
death or by maiming or by any other form of physical injury. This does not exclude the assessment of compensation for mental
suffering in addition, where that can be proved. In Flint v Lovell Greer LJ ([1934] All ER Rep at p 202; [1935] 1 KB at p 359),
gave mental disquietude at the prospect of an early death as an element of damage. Lord Roche spoke to the same effect in Rose
v Ford ([1937] 3 All ER at p 379; [1937] AC at p 859) and Pearce LJ, in Oliver v Ashman ([1961] 3 All ER 323 at p 332; [1962] 2
QB 210 at p 231).
In Rose v Ford both Lord Wright ([1937] 3 All ER at p 373; [1937] AC at p 850) and Lord Roche ([1937] 3 All ER at p 380;
[1937] AC at p 860) stressed the importance of the objective element being assessed at a moderate figure. In Mills v. Stanway
Coaches, Ltd, Goddard LJ, said ([1940] 2 All ER 586 at p 594; [1940] 3 KB 334 at p 348) that the court should think in hundreds
and not in thousands. Just how moderate the figure should be was settled by this House in Benham v Gambling, when the trial
judges award of 1,200 was reduced to 200. This case was intended to set and has set a standard of uniformity for the
assessment of damage for loss of expectation of life where there is no mental suffering. The depreciation of the currency has
raised the figure of 200 to 500; and that is the figure which the judge awarded in the present case under this head.
Counsel for the respondent, in seeking to sustain the award of 17,500 in this case, relies on these authorities in part. He
argues, I think rightly, that if irrespective of suffering damages can be recovered for deprivation of enjoyment caused by death,
they must also be recoverable for deprivation of enjoyment caused by injury short of death. But he asks the House to reject the
view that such damages should be moderate in the sense in which moderation was understood in Benham v Gambling. He asks
the House to restrict the Benham v Gambling principleI shall call it that for the sake of convenience although, as I have pointed
out, it is only the culminating authority of those that 640 enjoin moderationto cases where the deprivation of enjoyment is
caused by death. I regard this argument as quite illogical. I do not say that Benham v Gambling is binding on your lordships and
concludes this case. Your lordships have been reminded of Lord Halsburys protest in Quinn v Leatham ([19003] All ER Rep 1
at p 7; [1901] AC 495 at p 506) against authorities being quoted for propositions that may seem to follow logically from them.
Although I see no logical distinction at all, I agree that it is open to your lordships to say that you dislike theBenham v Gambling
principle and will not extend it. But I am unwilling on two grounds to take that rather drastic course. The first is that I think it
would introduce a distortionfor your lordships cannot drive Benham v Gambling out of the field in which it operatesthat
would make the law not merely illogical but repugnant to common sense. The second is that, although I think that the reasoning
inBenham v Gambling can be criticised and the figure set may be too small and the uniformity imposed too rigid, it is
fundamentally a decision on the right lines.
On the first ground, I can see no distinction, logical or otherwise, between sudden death and death preceded by a period of
unconsciousness, long or short. The injury that mortifies the limbs and cuts off the faculties has in both cases the same effect on
the power of enjoyment; in each case the deprivation is absolute. Death is often preceded by some period of unconsciousness.
For how long must the period last in order that the victims estate may benefit by the higher measure? For days, for weeks or for
months? I find it, with respect, repugnant to common sense and to justice that if the victim dies at once the estate benefits only
by a few hundreds but that if the body is kept alive and inert when the mind is dead the amount should grow and grow until it
reaches a sum such as 15,000. In this case if, as in Wise v Kaye, there had been no spark of consciousness, the damages would
be thirty times what they would have been if the plaintiff had died at once.
Secondly, I said that I thought Benham v Gambling was fundamentally on the right lines. The objective element should be
very moderately assessed. Even in cases where it is undoubtedly property which is being dealt with, such as the loss of the use of
a ship, the law, where no loss of profit can be found, will attempt only a conventional calculation, eg, loss of interest on capital:
see The Hebridean Coast. Limbs and faculties cannot be turned into cash as property can. If it were not that the objective
element has already by the authorities been given a place in the assessment, I should question whether it ought to be there at all. I
think that deprivation should be measured mainly, if not wholly, by the sense of loss. I cannot help feeling that the contrary view
is coloured by the thought that a wrongdoer should be made to pay damages commensurate with the gravity of the physical injury
he has inflicted rather than with the suffering he has caused. This, as my noble and learned friend Lord Reid has said, is a variant
of the argument that it should not be cheaper to kill than to maim. That argument is wholly fallacious. If a man chooses
deliberately to maim rather than to kill, he would pay the penalty for the crime. The amount of harm done by negligence owes
nearly everything to accident and very little to the state of mind of the doer: the barest negligence can cause the gravest injury
and the most culpable little or none at all.
The consideration, also mentioned by my noble and learned friend, that the sum awarded will in all probability be of no use
to the victim is in my opinion a legitimate and cogent argument in support of the extremely moderate assessment made in
Benham v Gambling. It is not a consideration which should be elevated into a principle. For reasons which have already been
fully stated by your lordships it cannot be said that an award otherwise appropriate ought to be cut because of the unlikelihood
that the victim will be able to use or dispose of it 641 all. But as a sound practical reason for reducing in the Benham v Gambling
type of case the part played in the assessment by the objective element, it has great force.
Accordingly, I consider that where there is no knowledge of the deprivation, nothing more than a conventional sum of the
same order as that assessed on the Benham v Gambling principle should be awarded for deprivation of faculties. Indeed, where,
as in Wise v Kaye the injury causes total unconsciousness which lasts until death, I can see no justification for two awards, one
for deprivation, which although inflicted on a living victim is not experienced by him, and the other, for loss of expectation of life
that is also not experienced. To my mind there is then only one deprivation and the passing of unconsciousness into death does
not make a second one.
I think that the sum awarded for mental suffering should in this case be generously assessed. The extent to which this
respondent has experienced a sense of loss is very difficult to ascertain, but I think that the presumption should be against the
appellant. The difficulty arises because she cannot express whatever it is that may be in her mind; she cannot speak or make
herself understood. It was the act of the appellant that destroyed these faculties: onmia praesumuntur contra spoliatorem. But as
I have reason to believe that the majority of your lordships intend to uphold the award of the learned judge, I do not find it
necessary to decide on an alternative figure.
I would allow the appeal.

LORD PEARCE. My Lords, the appellants seek to use the respondents condition as the foundation for two arguments in
extinction or diminution of damages claimed in respect of her injuries and pain and loss of amenities.
First, it is argued that such damages are given as compensation or consolation, and therefore, when the respondents
condition is so bad that they cannot be used by her to compensate or console, they should either be greatly reduced or should not
be awarded at all. No authority is cited in favour of such a proposition, nor can I see any principle of common law that supports
it. The argument contains the assumption, which in my opinion is fallacious, that the court is concerned with what happens to the
damages when they have been awarded. The court has to perform the difficult and artificial task of converting into monetary
damages the physical injury and deprivation and pain and to give judgment for what it considers to be a reasonable sum. It does
not look beyond the judgment to the spending of the damages. If it did so, many difficult problems would arise. Similar sums
awarded for similar suffering may produce wholly different results. To a poor man, who is thereby enabled to achieve some
cherished object such as the education of his family, the sum awarded may prove to be a more than adequate consolation. To a
man who already has more money than he wants, it may be no consolation at all. But these are matters with which the court is
not concerned. Whether the sum awarded is spent or how it is spent is entirely a matter for the plaintiff or the plaintiffs legal
representatives. If the plaintiffs personal ability to use or enjoy the damages awarded for injury and pain and loss of amenity
were a condition precedent to their award, it would be impossible for the executors of an injured person to obtain such damages.
Yet they did so in Rose v Ford and Benham v Gambling and many other cases.
The second argument is founded onBenham v Gambling and would affect the whole basis of damages awarded for personal
injury, apart, of course, from economic loss with which the argument is not concerned. Substantial damages are not awarded, it is
said, for physical injury simpliciter, but only for the pain and suffering and general loss of happiness which it occasions.
Therefore the deprivation of a limb can only command any substantial compensation in so far as it results in suffering or loss of
happiness; and where there is little or no 642 consciousness of deprivation there can be little or no damages. For this argument
the appellants rely on Benham v Gambling and on the minority judgment of Diplock LJ, in Wise v Kaye ([1962] 1 All ER at p
270; [1962] 1 QB at p 663).
The practice of the courts hitherto has been to treat bodily injury as a deprivation which in itself entitles a plaintiff to
substantial damages according to its gravity. In Phillips v London & South Western Ry Co Cockburn CJ, in enumerating the
heads of damage which the jury must take into account and in respect of which a plaintiff is entitled to compensation said
((1879), 4 QBD at pp 407, 408):

These are the bodily injury sustained; the pain undergone: the effect on the health of the sufferer according to its
degree and its probable duration as likely to be temporary or permanent; the expenses incidental to attempts to effect a cure
or to lessen the amount of injury; the pecuniary loss

In Rose v Ford ([1937] 3 All ER at p 379; [1937] AC at p 859) Lord Roche said: I regard impaired health and vitality not
merely as a cause of pain and suffering but as a loss of a good thing in itself. If a plaintiff has lost a leg, the court approaches
the matter on the basis that he has suffered a serious physical deprivation no matter what his condition or temperament or state of
mind may be. That deprivation may also create future economic loss which is added to the assessment. Past and prospective
pain and discomfort increase the assessment. If there is loss of amenity apart from the obvious and normal loss inherent in the
deprivation of the limb for instance, the plaintiffs main interest in life was some sport or hobby from which he will in future be
debarred, that too increases the assessment. If there is a particular injury to the nervous system, that also increases the
assessment. So too with other personal and subjective matters that fall to be decided in the light of common sense in particular
cases. These considerations are not dealt with as separate items but are taken into account by the court in fixing one inclusive
sum for general damages.
There are no rigid rules for the assessment of damages, but the approach of judges in directing juries or themselves and the
approach of advocates in address sing courts or arranging settlements have been on the lines that I have set out. Each year for
many decades past some thousands of cases have been argued and decided on those lines with a reasonable measure of success.
Although it is not possible to get a complete consistency in human decisions, there has been a general standard of fair
assessments in the many cases decided daily on those traditional lines. It was certainly not understood by the profession
generally that the decision of Benham v Gambling had altered them or that it had been so intended. I agree with the words of
Sellers LJ, inWise v Kaye ([1962] 1 All ER at pp 262, 263; [1962] 1 QB at p 650) when in referring to physical injury and pain he
said:

These are, however, well-known heads of claims which have throughout the years been translated into moneyno
doubt on what might be called a conventional figureand which the courts have sought to assess, and I think have
succeeded in keeping, on a reasonable basis in the interests of both the recipient and the payer, and on a reasonable
adjustment between one claimant and another. A comparative value has been assessed according to the gravity and
duration of the infliction on the plaintiff before the court.
I cannot speak for my brethren, but I have not, either at the Bar or on the Bench in dealing with such assessments, had
in mind the happiness or unhappiness of a claimant except in the most general way. The inner world or the inner life or,
should it be said, the soul of a person is not, in my view, a matter for investigation in a court of law in order to justify an
award of damages to a person living and I have not understood that Benham v Gambling has so stated or inferentially so
decided.
643

The fact that the learned lord justice and I and the legal profession in general have not hitherto so understood (although upwards
of twenty years have passed since the decision) does not decide the question, but it is some indication that the guidance, if
guidance was intended, was far from clear.
InBenham v Gambling this House was called on to answer a particular problem that had recently caused grave difficulty in
the courts. It had little direct connexion with the daily cases concerned with injuries that disable the living body. The problem
simply stated was Is life a boon? And, if so, what is the money value of all that which we lose by death? From 1934 (See Law
Reform (Miscellaneous Provisions) Act, 1934, s 1; 9 Halsburys Staututes (2nd Edn) 792) onward every person, be it an infant in
arms or an aged cripple, who was killed by negligence, had, through his personal representatives, a claim for damages for the loss
of his expectation of life. These claims were supported by varying evidence designed to give speculative illumination on what
might have been the future material, social and temperamental prospects of the deceased and the resulting value of life to him.
As might be expected, the wide divergence of views as to the value of our leases of life, whether forfeited near their beginning or
end, or in the middle, led to awards which varied very widely and unpredictably. Into this unseemly chaos Benham v Gambling
brought consistency at the inevitable expense of withdrawing the consideration of such damages, in effect, from the judge or jury.
It imposed a small conventional figure within narrow limits. This figure was a great deal lower than that at which many of us
would have set the value of human living. But, although this might seem a hardship to plaintiffs and a leniency to defendants, it
could fairly be said that in the majority of cases the plaintiffs were no longer alive to resent the hardship and their executors owed
the very existence of their claims to the then recent Act of 1934. To the living plaintiffs there was hardship in the decision to the
minds of those who attach a high value to life. From the terms of Viscount Simons opinion and the fact that the rest of their
lordships added no observations, I think that the House was addressing itself solely to clarifying the peculiar and difficult
problem which was before it and to imposing order on chaos in that particular aspect of the law. Having referred to the
divergences which that problem had produced Viscount Simon continued ([1941] 1 All ER at p 11; [1941] AC at p 165):

The House is now set the difficult task of indicating what are the main considerations to be borne in mind in assessing
damages under this head.

He dealt ([1941] 1 All ER at p 12; [1941] AC at p 166) with the fallacy of assuming

that human life is continuously an enjoyable thing so that the shortening of it calls for compensation to be paid to the
deceaseds estate on a quantitative basis. The ups and downs of life, its pains and sorrows as well as its joys and pleasures
all that makes up lifes fitful feverhave to be allowed for in the estimate The question thus resolves itself into that
of fixing a reasonable figure to be paid by way of damages for the loss of a measure of prospective happiness. Such a
problem might seem more suitable for discussion on an essay on Aristotelian ethics than in the judgment of a court of law,
but in view of earlier authorities we must do our best to contribute to its solution.

In that peculiar problem the only possible test which could be made was a consideration of whether there was a general balance
of happiness in life which had been lost. It was an imprecise and unsatisfactory test, but no other test was available. And the
courts have to do the best they can with the available material in assessing damages.
I can see no trace of any intention to give guidance on the more practical and 644 wholly different question, a commonplace
of the courts, which had caused no acute difficulties, namely what was the proper method of assessment for varying degrees of
physical deprivation to plaintiffs who in any event, whether whole or maimed, would continue to live lifes fitful fever with its
ups and downs. Many of the observations are quite unsuitable and inapt to the latter class of case. Had Viscount Simon intended
the fixing of a reasonable figure to be paid by way of damages for the loss of a measure of prospective happiness to supersede
the existing practice of the law for generations past and to apply to the familiar assessments for loss of limbs or amenities by
living plaintiffs, he must surely have said so. He must have added to his comment on the suitability of the problem for discussion
in an essay on Aristotelian ethics, the fact that the problem had been dealt with tolerably successfully for generations by judges
and juries alike. He would also, I think, have contrasted the fact that before any damages could be obtained for a shortened life,
the plaintiff must first establish on balance a positive measure of happiness with the case of injuries to living plaintiffs where
there cannot conceivably be such a requirement. For it is obvious that even the most miserable of pessimists is entitled to
damages for deprivation of a limb.
The loss of happiness of the individual plaintiffs is not, in my opinion, a practicable or correct guide to reasonable
compensation in cases of personal injury to a living plaintiff. A man of fortitude is not made less happy because he loses a limb.
It may alter the scope of his activities and force him to seek his happiness in other directions. The cripple by the fireside reading
or talking with friends may achieve happiness as great as that which, but for the accident, he would have achieved playing golf in
the fresh air of the links. To some ancient philosophers the former kind of happiness might even have seemed of a higher nature
than the latter, provided that the book or the talk were such as they would approve. Some less robust persons on the other hand
are prepared to attribute a great loss of happiness to a quite trivial event. It would be lamentable if the trial of a personal injury
claim put a premium on protestations of misery and if a long face was the only safe passport to a large award. Under the present
practice there is no call for a parade of personal unhappiness. A plaintiff who cheerfully admits that he is as happy as he was,
may yet receive a large award as reasonable compensation for the grave injury and loss of amenity over which he has managed to
triumph.
I venture to think that an alteration of the current principles of assessing damages for personal injury would be an
embarrassment to a practice which in spite of its difficulties does in the main produce a just result. Common law courts should
not lightly abandon a method of estimation that works reasonably well and achieves a certain amount of precision, for a method
that is nebulous, variable and subjective. I cannot read Benham v Gambling as having by implication intended such a result. I
agree with the observations of Sellers LJ ([1962] 1 All ER at p 259; [1962] 1 QB at p 644), and Upjohn LJ ([1962] 1 All ER at p
265; [1962] 1 QB at p 655), in Wise v Kaye and also of Herring, CJ ([1956] VLR 738 at p 739), and Barry, J ([1956] VLR at p
739), in McGrath Trailer Equipment Pty Ltd v Smith.
Even so, it is contended that such a situation as arose in Wise v Kaye where a plaintiff became and remained unconscious
over the years with no prospect of regaining consciousness should be equated to a living death and should fall within the
principle of Benham v Gambling. Although it is tempting and easy to make that equation, I do not think that it is necessary or
just to do so, and I agree with the majority decision inWise v Kaye to that effect. Benham v Gambling artificially and drastically
limited the liability of defendants in respect of loss of expectation of life. But I would not extend that artificial limitation to any
claims for loss of some or even all of the amenities of 645 living during a plaintiffs life, however low that life may have been
brought. The present case is, however, very different fromWise v Kaye(80). Here there is no continuous, total, lack of
consciousness. The doctors give no guidance as to how far the respondent is conscious of her plight, how far she is plunged in
misery or free from any feeling at all. I think that a reasonable juror would on balance find it more probable that she is leading a
miserable existence and that she is at least to some extent, and probably to a very large extent, aware of her plight.
It is clear from Benham v Gambling that the conventional sum awarded to her for loss of expectation of life covers every
aspect of her loss from the date when her death may be anticipated, except in so far as she suffers during her lifetime from a
knowledge of that loss of expectationa knowledge which I cannot assume in this case. It follows that her damages for her
injuries and for her suffering and deprivation of all that is good in life are confined to a period of about seven years from the date
of the accident. It was further contended that in any event, even on the traditional lines of approach to personal injuries, 17,500
was too high a sum for seven years and that some lesser sum such as 10,000 would be adequate. In view of the limited period
the damages were, in my opinion, on the high side. But seven years is a substantial time in the light of such a devastation of the
human amenities. In my opinion, the award cannot be said to be so excessive that this House should interfere. The learned judge
referred to the award of 15,000 in Wise v Kaye. That award was, I think, high on the particular facts of that case, but I should be
inclined to agree with the majority judgments that it was not so high as to call for interference. We should be doing less than
justice to the experience of the learned judge, if we assumed that he was taking the figure in Wise v Kaye as an established figure
to which he merely had to make some necessary adjustment in order to find the figure in the present case. But he was, I think,
referring to that case, which was very different from the present case, as a check to his own view of the damages in the present
case founded on his own wide experience. That he was entitled to do.
I would accordingly dismiss the appeal.

Appeal dismissed.

Solicitors: Hugh-Jones & Co (for the appellant); J I Humphreys & Co (for the respondent).

C G Leonard Esq Barrister.


646
[1963] 2 All ER 647

Finbow v Air Ministry


AGRICULTURE: LANDLORD AND TENANT; Tenancies

QUEENS BENCH DIVISION


MCNAIR J
27, 28, 29 MARCH, 1 APRIL 1963

Licence Licence to occupy and use, as agricultural land, land forming part of airfield Grantor a government department
Part of land under requisition, and rest owned, by grantor Agreement containing covenants usual in tenancy agreements, but
expressly excluding creation of tenancy Same protection conferred on grantee as tenant of agricultural holding, other than
conditional security of tenure Whether tenancy created.

Agriculture Agricultural holding Licence to occupy Approval by Minister of Agriculture Document declaring ministers
approval of all licences by specified government departments Approval stated to be given under repealed enactment
Enactment re-enacted in slightly different terms Whether document effective to give approval Whether approval required to
be given ad hoc Agricultural Holdings Act, 1948 (11 & 12 Geo 6 c 63), s 2(1).

The following principles at least are established by the authorities for guidance in determining whether an agreement is to be
construed as creating a tenancy of land or a licence(1) the agreement must be construed as a whole, and the relationship must
be determined by the law and not by the label which the parties put on it, though the label is a factor to be taken into account in
determining the true relationship; (2) the grant of exclusive possession, if not conclusive against the view that there is a mere
licence as distinct from a tenancy, is a consideration of the first importance; (3) in all cases where an occupier has been held to be
a licensee there has been something in the circumstances such as a family arrangement, an act of friendship or generosity, or such
like, to negative any intention to create a tenancy (see p 653, letter h, to p 654, letter a, post).
Facchini v Bryson ([1952] 1 TLR 1386) and Addiscombe Garden Estates Ltd v Crabbe ([1957] 3 All ER 563) considered.
A document dated 1 September 1948, and executed under the seal of the Minister of Agriculture and Fisheries, stated that
the minister thereby approved the grant by specified government departments, including the Secretary of State for Air, of every
licence to occupy and use as agricultural land any land belonging to those authorities for any interest or estate. The ministers
approval was stated to be given in exercise of the powers conferred on him by the Agriculture Act, 1947, s 40, although that had
been replaced, in slightly different terms, by the Agricultural Holdings Act, 1948, s 2. On 3 March 1950, a farm forming part of
an airfield was partly owned and partly under requisition by the Secretary of State. By an agreement dated 3 March 1950, the
Secretary of State granted to the plaintiff licence to occupy and use certain land, comprising a few acres of land then still in
requisition (the red land) and some 227 acres of land then held in fee simple by the Secretary of State (the green land),
making 237 acres in all, as agricultural land for a year certain and thereafter from year to year but so that nothing therein
contained should be deemed to create a tenancy of the farm either at law or in equity. The agreement contained exceptions and
reservations of timber and minerals and covenants, including a covenant for rent, commonly to be found in agricultural tenancies,
a covenant by the plaintiff to maintain the visibility from the air of fencing erected across a runway, and provided, by cl 7, that
the parties should have the same rights and liabilities under the Act of 1948 as if the agreement had created a tenancy within that
Act except that s 24 and s 25 should not apply. In making the agreement dated 3 March 1950, the Secretary of State relied on the
document dated 1 September 1948, for the ministers approval. In 1955 the fee simple in the red land was purchased by the
Secretary of State. On 8 September 1958, notice to quit the farm, which was admittedly an effective notice if the plaintiff were
only a licensee, was served on the plaintiff on behalf of the Secretary of 647 State. On 2 October 1958, the plaintiff served a
purported counter-notice pursuant to s 24 of the Act of 1948 on the basis that he had an agricultural tenancy protected by that Act.

Held The defendants, the Air Ministry, were entitled to possession of the land since the agreement of 1950 created a licence and
not a tenancy for the following reasons
(i) although, if the agreement of 1950 had been made by a private land owner, not by the Crown and without special
circumstances, it would have created a tenancy, yet in the circumstances of this case the agreement should be construed as
creating a licence, having regard particularly to the following factors(a) that the land was part of a Royal Air Force airfield; (b)
that at the date of the agreement the Secretary of State had no power in law to create a tenancy of the red land (no relevant
distinction being drawn in the agreement between the red land and the green land) and the subsequent acquisition of the fee
simple in the red land by the Secretary of State did not affect the construction of the agreement for the purpose of ascertaining
therefrom the intention of the parties to it at its date; (c) that the grantor of the licence was a manifestation of the Crown; (d) that
the Secretary of State relied on the approval of the Minister of Agriculture, Fisheries, and Food in entering into the agreement,
and (e) cl 7 of the agreement showed that the intention of the parties was to confer by means of a licence two of the main
protections of the Agricultural Holdings Act, 1948, viz, compensation for improvements and for disturbance, but to deny a third,
namely, conditional security of tenure (see p 654, letters b, e and h, and p 655, letters a and c, post).
(ii) notwithstanding the falsa demonstratio of the document of 1 September 1948, in referring to the then repealed s 40 of the
Agriculture Act, 1947, instead of to s 2 of the Agricultural Holdings Act, 1948, the document should have effect according to its
manifest intention, and accordingly it gave the approval of the minister under s 2 to the granting of the licence (see p 656, letters
a and b, post; Re Lockwood, Atherton v Brooke, [1957] 3 All ER 520, applied); moreover the approval that might be given under
s 2 was not limited to an approval ad hoc of only one agreement at a time, but could be a general approval of agreements by
specified authorities (see p 657, letter c, post), and accordingly s 2(1) did not cause the agreement to take effect as an agreement
for a tenancy (see p 657, letter g, post).

Notes
As to the creation of a tenancy of agricultural holding, see 1 Halsburys Laws (3rd Edn) 256, para 556. As to determining
whether agreement creates licence or lease, see 23 ibid, 427, para 1022; and for cases on the subject, see 30 Digest (Repl) 527
536, 16491711. As to correcting documents by rejecting repugnant words, see 11 Halsburys Laws (3rd Edn) 413, para 670, text
and note (o); and for cases on the subject, see 17 Digest (Repl) 279, 280, 857873.
For the Agricultural Holdings Act, 1948, s 2, see 28 Halsburys Statutes (2nd Edn) 29.

Cases referred to in judgment


Addiscombe Garden Estates Ltd v Crabbe [1957] 3 All ER 563, [1958] 1 QB 513, [1957] 3 WLR 980, 3rd Digest Supp.
Bracey v Read [1962] 3 All ER 472, [1962] 3 WLR 1194.
Errington v Errington & Woods [1952] 1 All ER 149, [1952] 1 KB 290, 3rd Digest Supp.
Facchini v Bryson [1952] 1 TLR 1386, 3rd Digest Supp.
Foster v Brooks (1950), 156 Estates Gazette 92.
Gilbey v Rush [1906] 1 Ch 11, 75 LJCh 32, 93 LT 616, 54 WR 71, 40 Digest (Repl) 815, 2940.
Lewisham Metropolitan Borough & Town Clerk v Roberts [1949] 1 All ER 815, [1949] 2 KB 608, 113 JP 260, sub nom Roberts v
Lewisham Borough Council, [1949] LJR 1318, CA, 17 Digest (Repl) 443, 124.
648
Lockwood, Re, Atherton v Brooke [1957] 3 All ER 520, [1958] Ch 231, 3rd Digest Supp.
Minister of Agriculture & Fisheries v Matthews [1949] 2 All ER 724, [1950] 1 KB 148, 2 Digest (Repl) 147, 1136.
R v Paddington & St Marylebone Rent Tribunal, Ex p Bell London & Provincial Properties Ltd [1949] 1 All ER 720, [1949] 1
KB 666, 113 JP 209, 31 Digest (Repl) 730, 8125.
Ward v Scott (1950), 66 (pt 1) TLR 340, [1950] WN 76, 2 Digest (Repl) 14, 61.
Webb v Austin (1844), 7 Man & G 701, 8 Scott NR 419, 13 LJCP 203, 3 LTOS 282, 135 ER 282, 30 Digest (Repl) 382, 257.

Action
This was an action by Archibald Finbow, the plaintiff, against the Air Minstry a, the defendants, for, in effect, determination of the
question whether an agreement dated 3 March 1950, whereby the defendants granted to the plaintiff licence to occupy and use as
agricultural land a farm forming part of an airfield now vested in fee simple in the defendants, created an agricultural tenancy of
the farm within the meaning of the Agricultural Holdings Act, 1948, and so entitled him to the conditional security of tenure
provided by that Act.
________________________________________
a Viz, the authorised government department for being sued pursuant to s 17(3) of the Crown Proceedings Act, 1947

L A Blundell QC and J T Plume for the plaintiff.


The Solicitor General (Sir Peter Rawlinson QC) and J R Cumming-Bruce for the defendants.

1 April 1963. The following judgment was delivered.

MCNAIR J. This case raises important and difficult questions as to the rights of farmers who have been granted by documents
which, on their face purport to be licences, the right to occupy and use as agricultural land portions of service airfields no longer
used for active service operations, of which originally possession has been taken by the service authorities pursuant to reg 51 of
the Defence (General) Regulations, 1939, and which have either remained on requisition or passed into the fee simple ownership
of one or other of the service departments or of the Minister of Agriculture, Fisheries and Food by compulsory purchase.
The case before me relates to one particular area which formed part of the Royal Air Force aerodrome at Leiston in the
county of Suffolk; but I am informed that the case has been brought in the nature of a test action in the hope that the decision of
the court in this case may assist in the solution of the difficulties which have arisen between the Ministry of Agriculture, Fisheries
and Food and other occupiers of agricultural land in similar circumstances.
In general the issues raised by this litigation may be stated as follows:(i) Is the agreement under which the plaintiff is
granted in terms a licence to occupy and use as agricultural land in law to be construed as a tenancy? If so, admittedly the tenant
would obtain the protection of conditional security of tenure granted by the Agricultural Holdings Act, 1948. (ii) If the agreement
is on its true construction merely a licence, does it by reason of s 2 of the Act of 1948 take effect as if it were an agreement for
letting the land for a tenancy from year to year? If so, the occupier would enjoy the same protection as if it had been, in fact, a
tenancy. (iii) Or is it preserved as a licence by reason of the fact that the grant [of the licence] was approved by the minister
[i.e., the Minister of Agriculture, Fisheries and Food] before the agreement was entered into within the meaning of the
aforementioned s 2 of the Act of 1948? (iv) Does it make any difference that at the date of the agreement part of the land covered
by the agreement was still under requisition only and not in the fee simple ownership of the Crown or the relevant department?
(v) What effect, if any, should be given to the fact that during the currency of the agreement, that part of the land which was held
on 649 requisition passed into the fee simple ownership of the Crown or the appropriate government department?
The case came before me on a statement of agreed facts and certain agreed documents and has involved a wide authorities
on either side, for which I am indebted to counsel. In the documents and in this judgment, that part of the land which at the date
of the material agreement, namely, 3 March 1950, was under requisition, amounting to 9 1/2 acres, is referred to as the red land;
the remainder of the land which, at that date was in the fee simple ownership of the airfield authority, namely, the Secretary of
State for Air, is referred to as the green land. The red and green land together amounted to 237 acres or thereabouts. It is, I think,
conceded that for the purpose of the solution of the legal problems involved, the exact proportion of the red and green lands is
immaterial so long as the red land does not fall to be disregarded (as it clearly does not) under the principle de minimis non curat
lex.
The material facts may be shortly summarised as follows:
(1) By a licence agreement dated 16 April 1947, the minister (which expression therein and hereinafter in this judgment
means the Minister of Agriculture, Fisheries and Food) granted to Mr Finbow, the plaintiff in this action, a licence to occupy 150
acres, or thereabouts, forming part of the Royal Air Force airfield at Leiston for 362 days from 15 October 1946, these 150 acres
all being land of which possession had been taken under the Defence (General) Regulations and was still so held.
(2) By a similar agreement dated 2 January 1948, a licence was granted to the plaintiff for a period of 362 days to occupy
233 acres, or thereabouts, forming part of the said airfield, this land still being held in possession by virtue of requisition under
the aforesaid regulations.
(3) During the currency of this licence, the Secretary of State for Air purchased the green land for an estate in fee simple.
(4) On 1 September 1948, a duly authorised officer of the minister, under the seal of the minister, executed a document
(hereinafter referred to as the approval) in the following terms:

The Minister of Agriculture and Fisheries in the exercise of the powers conferred on him by s. 40 of the Agriculture
Act, 1947, hereby approves the grant by any of the government departments or persons specified in the schedule hereto
(hereinafter referred to as the scheduled authorities) of every licence to occupy and use as agricultural land any land
which belongs to any of the scheduled authorities for any estate or interest whatever and which is at the time of the grant of
such licence being managed on behalf of the scheduled authorities by a county agricultural executive committee.

Schedule
The Commissioners for executing the Office of Lord High Admiral of the United Kingdom of Great Britain and
Ireland.
His Majestys Principal Secretary of State for the War Department.
The Secretary of State for Air.
The Minister of Supply.

Section 40 of the Agriculture Act, 1947, contains provisions substantially, but not in detail, the same as the provisions of s 2 of
the Agricultural Holdings Act, 1948; but by an apparent oversight the official responsible for executing this document on 1
September 1948, had failed to observe or remember that s 40 of the Act of 1947 had, in fact, been repealed as from 30 July 1948,
by virtue of s 98 of and Sch 8 to the Act of 1948.
(5) On 3 March 1950, the agreement (hereinafter referred to as the 1950 agreement) with which this litigation is
concerned, was entered into between the Secretary of State for Air (hereinafter referred to as the airfield authority) acting by
Richard Cook Ridley, Chairman of the East Suffolk Agricultural 650 Executive Committee of the first part, the minister acting by
the said Richard Cook Ridley of the second part and the plaintiff in this action of the third part.
(6) The powers and interests of the airfield authority over and in the land which was the subject of the 1950 agreement were
at the date of the agreement being managed by the East Suffolk Agricultural Executive Committee on behalf of the airfield
authority.
(7) The airfield authority entered into the 1950 agreement in reliance in so far as the agreement affected land already
belonging to him upon the approval of the minister hereinbefore referred to and obtained no other approval of the grant
contained in the 1950 agreement.
(8) On 7 November 1955, the red land was purchased by the airfield authority, for an estate in fee simple and has since
remained in the ownership of the airfield authority.
(9) On 8 September 1958, the regional controller [of the airfield authority] wrote to the plaintiff as follows:

Dear Sir, You will know that the government has decided that land which was compulsorily purchased by a
government department, but is no longer required for government purposes, shall be offered for sale. Leiston airfield is to
be offered for sale in accordance with this policy. With regard to the land occupied by you on this airfield, the ministry is
advised that, in view of the terms of the licence granted to you, it would not be practicable to sell the land subject to and
with the benefit of the licence. We regret that it is accordingly necessary to terminate your licence in order that sale may be
effected. We accordingly enclose a notice terminating the licence under which you occupy and use land containing about
237 acres, forming part of Leiston Airfield. We are sorry that, for the reasons which we have explained, it has become
necessary to disturb your occupation of the land. Yours faithfully, (Signed) S. G. WARDLE for J. Perrin, Regional
Controller.

The notice to quit enclosed was dated 18 September 1958.


By para 5 of the defence to counterclaim, the plaintiff admits that if he has only a licence to occupy and use the lands the
subject of the 1950 agreement, this notice would be a valid and effective notice to terminate such licence.
(10) On 2 October 1958, solicitors acting for the plaintiff on the basis that his occupation of the land was by virtue of an
agricultural tenancy protected by the Agricultural Holdings Act, 1948, served on the regional controller a counternotice
pursuant to s 24 of the Agricultural Holdings Act, 1948, in the following terms:

To the Secretary of State for Air. I hereby give you a counter-notice that I require sub-s. 1 of s. 24 of the Agricultural
Holdings Act, 1948, to apply to the notice to quit the house farm and premises and land containing 237 acres or thereabouts
forming part of Leiston airfield at Theberton and Leiston in the county of Suffolk which I hold as tenant thereof and which
notice to quit is dated Sept. 18, 1958.
Dated 2 October 1958. (signed) A Finbow (Tenant).

This notice, if effective, would give to the plaintiff conditional security of tenure.
I now turn to the provisions of the 1950 agreement, which is a long agreement extending over five pages of closely typed
provisions. After reciting that the airfield authority is seised in fee simple in possession of the green land and has taken
possession of the red land under the Defence (General) Regulations which it intended to purchase before the expiration of its
requisition powers and that it intended that after the red land has been purchased it should be transferred to the minister, the
agreement states by way of recital (3) that

The airfield authority has agreed to grant to [the plaintiff] licence to occupy and use as agricultural land the land
hereinafter described during such period as the airfield authority has any estate interest or powers in or over 651 the same
and the minister [viz., the Minister of Agriculture, Fisheries and Food] has agreed to grant [the plaintiff] a similar
licence as from the time when the said land becomes vested in him and [the plaintiff] has agreed to farm the same on the
terms and conditions hereinafter contained.

Clause 1 contains somewhat elaborate provisions for the giving of a notice by the airfield authority that the farm has been
transferred to the minister and the effect of such notice as substituting the minister for the airfield authority as grantor and
provides by sub-cl (5) as follows:
After the notice mentioned in sub-cl (2) hereof has been given to [the plaintiff] the Minister of Agriculture and [the
plaintiff] shall except as provided in cl. 7 [a reference to the exclusion of s. 24 and s. 25 of the Act of 1948] have the same
rights against and liabilities to each other as if the agreement had created a tenancy of the land hereinafter described and the
Minister of Agriculture had been the successor in title of the airfield authority to the freehold reversion of the said land
immediately expectant on the determination of such tenancy.

In fact, no such notice was given as the land was never transferred from the airfield authority to the minister. In the events which
have happened therefore, the rights and liabilities arising under the agreement are rights and liabilities of and as between the
airfield authority and the plaintiff.
The main grant is contained in cl 2 in the following terms:

The grantor hereby grants to [the plaintiff] licence to occupy and use as agricultural land all that land containing 237
acres or thereabouts forming part of the Royal Air Force airfield at Leiston in the county of East Suffolk (hereinafter called
the farm) and more particularly described in the schedule hereto and delineated on the plan annexed hereto and thereon
edged pink Together with a right of way at all times by day and by night and for all purposes (in common with the grantor
and all persons who now have or may hereafter have the like right) on foot and with animals and vehicles over the roads
and tracks coloured brown on the said plan Except and Reserved as hereinafter mentioned from Oct. 11 1949 to Oct. 11,
1950 and thereafter from year to year but so that nothing herein contained shall be deemed to create a tenancy of the
farm either at law or in equity.

The schedule describes the land as arable land; the plan annexed has apparently been mislaid; but on the plan annexed to the
statement of claim, which for the purposes of showing the red land is accepted as accurate, the green land is shown as being criss-
crossed with concrete runways which I understand are still there.
Clause 3 sets out certain exemptions and reservations of timber, mines and minerals and game, reserving to the grantor and
all persons authorised by him or the committee the exclusive rights to preserve the game, wild fowl, hares, rabbits, foxes and to
hunt, course, fish, shoot and sport on the farm. These reservations and exceptions, I am informed and accept, are the exceptions
and reservations commonly to be found in any agricultural tenancy. Clause 4 provides for the termination of the agreement on
11 October in any year by either the airfield authority or the plaintiff giving to the other of them twelve months notice in writing
of his intention so to do.
The plaintiffs covenants are contained in cl 5, and broadly they may be described as the typical covenants which would be
found in an agricultural tenancy, for example:

To pay to the grantor the annual sum of 230 5s. 0d. (herein referred to as the rent) [sub-cl. (1)]; To pay to the
grantor the amount (if any) which might reasonably have been expected to be payable in addition to rent by an incoming
tenant in respect of things done prior to the said Oct. 11, 1949 for the purposes of the cultivation of the farm and in respect
of seeds 652 tillages growing crops and other similar matters under a lease of the farm granted immediately before the said
Oct. 11, 1949 [sub-cl. (2)] To farm the farm in accordance with the rules of good husbandry as provided by s. 11 of the
Agriculture Act, 1947 , [sub-cl. (7)] Not to commit or suffer any wilful or voluntary waste spoil or destruction on the
farm [sub-cl. (14]; Not to assign or underlet the benefit of this agreement or part with the possession of the farm in any way
whatsoever [sub-cl. (17)]; To permit the grantor and persons authorised by him to enter on the farm at all reasonable times
to view the state and condition thereof [sub-cl. (21)]; On the determination of this agreement peaceably and quietly to
deliver up possession of the farm to the grantor [sub-cl. (22)].

Among the plaintiffs covenants is one, namely, sub-cl (6), which may be of significance as suggesting the continued use of the
airfield for emergency landings. This sub-clause requires the plaintiffs

to maintain all portions of fencing which are or may from time to time be erected across a runway plainly visible
from the air by covering the same with brushwood or some other suitable and clearly discernible material.

Clauses 6 and 7(1) and (2) deal with the grantors obligation as to repairs and in cl 7 is to be found this significant provision:

Save as hereinafter mentioned the grantor and the farmer shall have the same rights and be subject to the same
liabilities under the said Act of 1948 as if this agreement had created a contract of tenancy within the meaning of that Act
and the Minister of Agriculture shall have the same powers under that Act notwithstanding anything herein contained
Provided that ss. 24 and 25 of the said Act shall not apply to this agreement.

Further references to rent are to be found in cl 9 and cl 10.


Considering the agreement as a whole, it seems to me that the parties have clearly attempted by means of this agreement, to
create a contractual position under which the plaintiff has in substance all the rights of a tenant of an agricultural holding under
the Agriculture Holdings Act, 1948, save for the vital exclusion of the right to security of tenure under s 24 and s 25. They have
attempted to do so by means of a licence and not by means of a tenancy. The first question to be determined is whether in law
they have achieved this result.
Counsel for the plaintiff, in support of his argument that the 1950 agreement must be construed as a tenancy and not a mere
licence, referred the court to a number of authorities such as Errington v Errington & Woods; Facchini v Bryson; Addiscombe
Garden Estates Ltd v Crabbe; Bracey v Read where the courts were concerned with the question whether or not documents which
were expressed to be licences should in law be construed as creating tenancies so as to confer on the tenant rights conferred on
tenants under the Rent Acts or the Landlord and Tenant Act, 1954. So far as I recall, I was referred to no case in which the
question debated arose under the Agricultural Holdings Acts, but it seems to me the principles laid down in these authorities
would equally apply to the present problem. In my judgment these authorities do establish the following principles at least:(i)
that the agreement must be construed as a whole and that the relationship is determined by the law and not by the label which the
parties put on it, though the label is a factor to be taken into account in determining the true relationship (see Facchini v Bryson;
Addiscombe Garden Estates Ltd v Crabbe); (ii) that the grant of exclusive possession, if not conclusive against the view that there
is a mere licence as distinct from a tenancy, is at any rate a consideration of the first importance (see Jenkins LJ in the
Addiscombe case ([1957] 3 All ER at p 571; [1958] 1 QB at p 528)); (iii) that in all the cases where an occupier has been held to
be a licensee
653

there has been something in the circumstances such as a family arrangement, an act of friendship or generosity, or
such like, to negative any intention to create a tenancy. In such circumstances it would be obviously unjust to saddle the
owner with a tenancy, with all the momentous consequences that that entails nowadays, when there was no intention to
create a tenancy at all (see DENNING, L.J., in Facchinis case ([1952] 1 T.L.R. at p. 1389.)).
Accordingly, if this had been a case as between a private land-owner and a farmer relating to ordinary farming land with no
special circumstances, I should have been prepared to hold that the 1950 agreement did create a tenancy. There are a number of
circumstances in the present case, however, which in their totality have led me to the conclusion that this agreement should in law
be construed as creating a licence and not a tenancy. First, the nature of the land. It is described as forming part of the Royal Air
Force airfield. Though it may be inferred that at the date of the agreement the land was not being currently used as an airfield, it
is, I think, legitimate for the court to take judicial knowledge of the fact that in present days emergencies may suddenly arise in
which it may be necessary, in the national interest, to re-activate an airfield. The agreement, by cl 5(6), contemplates that the
runways may be required for emergency use even while the land is being occupied by the plaintiff, and further, though cl 5(14),
which imposes on the plaintiff the obligation not to commit or suffer any wilful or voluntary waste spoil or destruction on the
farm is primarily directed to deal with waste in the strict sense as between landlord and tenant, it would, in my judgment, also
operate to prevent the tenant from destroying and ploughing up the existing runways. Secondly, the airfield authority had no
power in law to create a tenancy of the red land (see Minister of Agriculture & Fisheries v Matthews and Lewisham Metropolitan
Borough and Town Clerk v Roberts. In the latter case Denning LJ says this ([1949] 1 All ER at p 824; [1949] 2 KB at p 622):

it is necessary to consider the nature of the power to requisition land. It is only a power to take possession of land.
It is not a power to create any estate or interest in any land Once possession is taken the Crown can exercise all the
powers incident to possession, such as to license other people to use the premises but it cannot grant a lease or create
any legal interest in the land in favour of any other person because it has itself no estate in the land out of which to carve
any interest.

As regards rights and obligations, no distinction is made in the agreement between the red land and the green land. If the airfield
authority had no power to grant the tenancy of the red land and did not so intend, it is difficult to understand why he should be
assumed or held to have intended to create a tenancy of the green land. It is true that subsequent to the date of the 1950
agreement, the airfield authority acquired a fee simple title to the red land, but, in my judgment, the force of the point that he had
no power to create a tenancy of the red land at the date of the agreement, was not met by cases such as Webb v Austin, where the
lessor had no title to the land he purported to demise at the date of the agreement but subsequently acquired it, since in the
present case one is seeking to ascertain the intention of the parties at the time of the execution of the 1950 agreement. Thirdly,
the fact that the grantors are two manifestations of the Crown. Whereas in the cases under the Rent Acts and the Landlord and
Tenant Act, 1954, the courts, when they held that a document purporting to be a licence was in law a tenancy, were giving effect
to the public policy embodied in those Acts of conferring security of tenure and preventing the evasion of those Acts (see
Denning LJ in Facchinis case ([1952] 1 TLR at p 1390), I do not feel that the same considerations apply where the grantors are
manifestations of the Crown, especially when one 654 is the Minister of Agriculture, Fisheries and Food, the authority
responsible for the administration of the Agricultural Holdings Acts. Fourthly, the agreed fact that the airfield authority relied on
the approval of the minister in entering into the agreement is, or may be, relevant.
In my judgment the real clue to the true intention of the 1950 agreement is to be found in cl 7 which provides as follows:

Save as hereinafter mentioned the grantor and [the plaintiff] shall have the same rights and be subject to the same
liabilities under the said Act of 1948 as if this agreement had created a contract of tenancy within the meaning of that Act
and the Minister of Agriculture shall have the same powers under that Act notwithstanding anything herein contained
Provided that s. 24 and s. 25 of the said Act shall not apply to this agreement.

This clause, coupled with the earlier provisions as to the licences, seems to me quite plainly to express an intention to confer on
the plaintiff by means of a licence, two of the main protections conferred by the Agricultural Holdings Act, 1948, namely,
compensation for improvements and compensation for disturbance, while denying to him the third, namely, conditional security
of tenure. This latter exclusion seems to be essential for the protection of the airfield authority. I can find nothing in the
authorities which in the special circumstances of this case prevents the court from giving effect to this plain intention. I,
therefore, hold that the 1950 agreement does not on its true construction create a tenancy.
I now turn to the question of whether or not there has been an effective approval of the licence by the minister, so as to
prevent the licence taking effect under the Agricultural Holdings Act, 1948, as an agreement for the letting of the land for a
tenancy from year to year. This turns on s 2 of the Act of 1948 which provides (omitting immaterial words) as follows:

(1) Subject to the provisions of this section, where under an agreement made on or after Mar. 1, 1948 [the date when s.
40 of the Agricultural Act, 1947, came into effect] any land is let to a person for use as agricultural land for an interest less
than a tenancy from year to year, or a person is granted a licence to occupy land for use as agricultural land, and the
circumstances are such that if his interest were a tenancy from year to year he would in respect of that land be the tenant of
an agricultural holding, then, unless the letting or grant was approved by the minister before the agreement was entered
into, the agreement shall take effect, with the necessary modifications, as if it were an agreement for the letting of the land
for a tenancy from year to year: Provided that this sub-section shall not have effect in relation to an agreement for the
letting of land, or the granting of a licence to occupy land, by a person whose interest in the land is less than a tenancy from
year to year and has not by virtue of this section taken effect as such a tenancy.

The approval relied on by the Crown in this case is the approval contained in or evidenced by the document dated 1 September
1948, under the seal of the minister. This document opens as follows:

The Minister of Agriculture and Fisheries in exercise of the powers conferred on him by s. 40 of the Agriculture Act,
1947 hereby approves.

As stated earlier, s 40 of the Act of 1947, was repealed as from 30 July 1948, by virtue of s 98 of and Sch 8 to the Act of 1948, so
that on 1 September 1948, the minister had no powers under s 40 of the Act of 1947.
If it had been executed in its present form on any date between 1 March and 30 July 1948, it would have been continued in
force so far as it could have been made under s 2 of the Act of 1948, by virtue of sub-s (1) of s 100 of the Act of 1948. But it is
not so saved by reason of its date of execution. Is it therefore to be regarded as a nullity?
The document was in the circumstances in which it was executed, plainly intended by the minister to be an approval in the
exercise of his statutory powers. 655These powers at the date of the instrument were to be found only in s 2 of the Act of 1948.
To hold that the misdescription of his powers rendered the document a nullity, would, in my judgment, defeat the plain intention
of the minister to be deduced from the circumstances and the date of its execution. It is, in my judgment, a plain case for the
application of the maxim falsa demonstratio non nocet and of the principle embodied in the maxim magis valeat quam pereat.
There is a total inconsistency and repugnancy between the ministers manifest intention and the literal effect of the document,
and, in my judgment, the former should prevail. In this connexion, reference may be made to Re Lockwood, Atherton v Brooke,
where Harman J was confronted with a rather similar problem arising out of mistake in a statute and approved a citation from
Maxwell on Interpretation of Statutes (now 11th Edn, p 243) to the following effect:
[The authorities] would seem rather to establish that the judicial interpreter may deal with careless and inaccurate
words and phrases in the same spirit as a critic deals with an obscure or corrupt text, when satisfied, on solid grounds, from
the context or history of the document, or from the injustice, inconvenience, or absurdity of the consequences to which it
would lead that the language thus treated does not really express the intention and that this amendment probably does.

In line with this decision and perhaps more closely relevant, is the decision of the Court of Appeal in Foster v Brooks. In
that case a tenant claiming compensation for disturbance gave a notice on 21 February 1949, headed The Agricultural Holdings
Acts, 192347 after these Acts had been repealed. The notice was in the following form:

I hereby give you notice that I claim under, and in accordance with, s. 30 of the Agricultural Holdings Act, 1947, to be
paid on the termination of my tenancy of the holding compensation

Sir Raymond Evershed MR is reported to have said:

It must be borne in mind that the intention of Parliament plainly was that agricultural tenants should get, as a matter of
course, disturbance compensation equivalent to a years rent, when the landlord gave notice to quit. The court must,
therefore, have regard to this intention and I take the view that the reference in the notice to the 1947 Act ought to be
rejected as surplusage and not sufficient to vitiate the document

Singleton and Jenkins LJJ agreed. The same result may possibly be arrived at by another route. In Ward v Scott Danckwerts J as
a second ground for his decision, gave effect to a tenants counter-notice which by mistake referred to the repealed s 31(1) of the
Act of 1947, instead of the current s 24(1) of the Act of 1948 by reliance on s 100(9) of the Act of 1948 which provides as
follows:

Any document referring to a former enactment relating to agricultural holdings shall be construed as referring to
the corresponding provision of this Act.

With great respect I feel the force of the contrary argument that s 100(9) cannot be relied on to give effect to a document of a
class, namely a notice, specifically referred to in s 100(1). For completeness I should add that although the ambit of s 40 of the
Act of 1947 by referring to a holding as defined in the Act of 1923 is not identical with the ambit of s 2 of the Act of 1948, the
difference for present purposes is, in my judgment, immaterial and would not prevent s 2 of the Act of 1948 being considered as a
corresponding section.
It is next submitted on behalf of the plaintiff that s 2 does not contemplate a blanket approval but requires an approval given
ad hoc after consideration of the terms of the proposed agreement or at least the circumstances relating to the 656 particular
letting or grant. It was argued that, if a blanket approval to letting or grants by the named service authorities were held to be
valid, the minister would equally have power in advance to approve all lettings or grants by whomsoever made and that this, in
effect, would enable the minister to deprive s 2 of any effect at all. This argument seems to me to be fallacious since, if a blanket
approval of all lettings and grants were attempted, it would be clear that the minister had not applied his mind to any relevant
circumstances. In the present case by confining his approval to lettings or grants by specific named authorities it is clear that he
had applied his mind to the general circumstances relating to such grants. The decision in R v Paddington and St Marylebone
Rent Tribunal, Ex parte Bell London & Provincial Properties Ltd, relied on by counsel for the plaintiff, seems to me to turn on
the particular provisions and policy of the Furnished Houses (Rent Control) Act, 1946, and does not assist in the solution of the
present problem. Nor does Gilbey v Rush contain any general statement of principle. I can find nothing in the language used in s
2(1) or in the policy of the Act of 1948 to be deduced from its provisions which requires or supports the submission that the
approval must be given ad hoc. If anything, the use of the words requiring the approval of the letting or grant rather than of
the terms of the proposed agreement seems to point the other way and suggest that the minister may approve lettings or grants of
particular types without considering the particular proposed letting or grant. The form of words used follows a very familiar
statutory form (see, for instance, s 1 and s 2 of the Ships and Aircraft (Transfer Restriction) Act, 1939 b, and it has, so far as I
know, never been doubted that the appropriate authority could under such provisions issue general sanctions and approvals
relating to particular categories of cases.
________________________________________
b 26 Halsburys Statutes (2nd Edn) 225, 226

Though on the view which I have formed it is unnecessary for me to express any concluded opinion on the point, I should
refer to the argument advanced on behalf of the Crown that s 2 of the Act of 1948, does not apply to the facts of this case by
reason of the terms of the proviso previously set out c. It was submitted that, as the minister had no interest in the red land, the
grant in the present case was a grant by a person whose interest in the land is less than a tenancy from year to year. As at
present advised, I should not be prepared to accept this submission as it seems to me that where the word interest is used earlier
in the section it plainly refers to a legal interest or estate. Furthermore, I do not consider that a person who has no interest can be
said to have an interest less than a tenancy from year to year.
________________________________________
c Page 655, letter g, ante

I, therefore, hold on the issues stated earlier in this judgment: (1) that the 1950 agreement is not to be construed as a tenancy
but as a licence; (2) and (3) that the licence did not take effect as a tenancy by reason of s 2 of the Act of 1948 since a valid
approval to the grant was given by the minister; (4) that the fact that part of the land was under requisition at the date of
execution of the agreement is a factor in considering whether the agreement as a whole created a tenancy, and (5) that no effect is
to be given to the fact that during the currency of the agreement that part of the land which was on requisition passed into the fee
simple ownership of the Crown. Accordingly there must be judgment for the defendants, the Air Ministry, the appropriate
authorised department, on the claim and judgment for the Air Ministry for possession of the land and for mesne profits at the rate
of 230 5s per annum from 11 October 1959, until delivery of possession.

Order accordingly.

Solicitors: Ellis & Fairbairn (for the plaintiff); Treasury Solicitor (for the defendants).

Mary Coulton Barrister.


657
[1963] 2 All ER 658
Parsons v B N M Laboratories Ltd
CIVIL PROCEDURE: QUANTUM: CONTRACT

COURT OF APPEAL
SELLERS HARMAN AND PEARSON LJJ
11, 12, 13, 14, 15 FEBRUARY, 4 APRIL 1963

Damages Measure of damages Incidence of income tax Contract Wrongful dismissal Whether income tax on lost
earnings and whether unemployment benefit to be taken into account in assessing damages Finance Act, 1960 (8 & 9 Eliz 2 c
44), s 37, s 38, Sch 4, para 13.

Damages Mitigation Unemployment benefit Breach of contract Whether unemployment benefit to be taken into account in
assessing damages for wrongful dismissal.

Statute Construction Marginal notes Whether marginal notes may be used for interpretation of enactment.

The plaintiff was engaged by the defendants at a salary of 2,000 per annum for three years from 15 March 1961. In April, 1962,
the defendants, in breach of the contract, informed the plaintiff that they would employ him only until the end of July, 1962. The
plaintiff, accepting the repudiation of the contract, left the defendants employment on 31 May 1962. He received 59 2s 6d in
unemployment benefit before acquiring new employment at a salary of 1,600 per annum in August, 1962. In an action for
damages for wrongful dismissal damages were assessed for loss of salary until 15 March 1964, without deduction for income tax
or unemployment benefit. On appeal,

Held (i) (Sellers LJ dissenting) at common law the principle of British Transport Commission v Gourley applied to the
assessment of damages for breach of a contract of service, and accordingly income tax at the standard rate should be taken into
account by way of deduction from the plaintiffs loss of annual earnings, the damages awarded to him as compensation for that
loss not being taxable in his hands (see p 671, letter e, p 678, letter f, and p 681, letter d, post); and s 37 and s 38 of the Finance
Act, 1960, and para 13 of Sch 4 thereto did not exclude the application of the common law principle in the present case, because
they did not render taxable damages that amounted to less than 5,000 (see p 674, letter c, and p 682, letter d, post).
(ii) the unemployment benefit (59 2s 6d) should also be taken into account by way of deduction in assessing the plaintiffs
damages because the general principle was that a plaintiff was entitled to have made good only the loss that he had suffered as
result of the wrong done to him and that loss was diminished by the unemployment benefit received by the plaintiff, which
benefit was not an exception to the general principle, since (a) (per Sellers LJ) contribution to the insurance was made by the
employer and the benefit directly mitigated the plaintiffs loss of earnings and was an asset that was sufficiently direct (see p 669,
letter i, and p 670, letter f, post), (b) (per Harman LJ) it was payable by the state as a result of contributions which the employer
as well as the employee were bound to make, and was not analogous to insurance moneys payable under a voluntary contract
made by the employee but not by the employer (see p 675, letter i, to p 676, letter a, and p 676, letter e, post), and (c) (per
Pearson LJ) the receipt of the unemployment benefit was not a matter too remote to be taken into account, and ought properly to
be taken into account, in mitigation of damages (see p 684, letters c and e, post).
British Transport Commission v Gourley ([1955] 3 All ER 796) applied.
Stewart v Glentaggart ([1963] SLT 119) applied on (i) above.
Payne v Railway Executive ([1951] 2 All ER 910) not followed on (ii) above.
Per Curiam: if income tax would be payable not only on the lost earnings but also on the damages recovered, the two
amounts of tax should in general, 658as a matter of practice, be left by way of set-off one against the other and should not be
taken into account in assessing the damages, though (per Pearson LJ), there may be exceptional cases (see p 675, letter e, p 680,
letter d, and p 681, letter c, post; cf p 669, letter b, post).
Dictum of Lord Hunter in Stewart v Glentaggart ([1963] SLT at p 121) disapproved.
Per Harman LJ: side-notes to sections of Acts of Parliament are inadmissible on the interpretation of the sections of the Acts
(see p 674, letter e, post).
Appeal allowed.

Notes
As regards the deduction of national insurance contributions from earnings when damages are being assessed, see Cooper v Firth
Brown Ltd p 31, ante, where the cause of action was in tort; but the decision was reached on the basis of the principle in British
Transport Commission v Gourley ([1955] 3 All ER 796). The deductibility of any national insurance employees contributions
was not, it seems, raised in the present case.
As to collateral benefits in assessing damages for breach of contract or in tort, see 11 Halsburys Laws (3rd Edn) 240, para
408; and for cases on the subject, see 17 Digest (Repl) 80, 81, 2738.
As to damages for wrongful dismissal, see 25 Halsburys Laws (3rd Edn) 521, 522, para 993; and for cases on the subject,
see 34 Digest 104106, 772792.
For the Finance Act, 1960, s 37, s 38 and Sch 4, para 13, see 40 Halsburys Statutes (2nd Edn) 458, 459, 491.

Cases referred to in judgments


Admiralty Comrs v Susquehanna (Owners), The Susquehanna [1926] All ER Rep 124, [1926] AC 655, 95 LJP 128, 135 LT 456,
41 Digest 802, 6624.
Beach v Reed Corrugated Cases Ltd [1956] 2 All ER 652, [1956] 1 WLR 807, 3rd Digest Supp.
Beak v Robson [1943] 1 All ER 46, [1943] AC 352, 112 LJKB 141, 169 LT 65, 25 Tax Cas 33, 28 Digest (Repl) 232, 1008.
Billingham v Hughes [1949] 1 All ER 684, [1949] 1 KB 643, [1949] LJR 1147, 17 Digest (Repl) 80, 32.
Bradburn v Great Western Ry Co (1874), LR 10 Exch 1, 44 LJEx 9, 17 Digest (Repl) 108, 222.
British Transport Commission v Gourley [1955] 3 All ER 796, [1956] AC 185, [1956] 2 WLR 41, 3rd Digest Supp.
Browning v War Office [1962] 3 All ER 1089, [1963] 1 WLR 52.
Diamond v Campbell-Jones [1960] 1 All ER 583, [1961] Ch 22, [1960] 2 WLR 568, 3rd Digest Supp.
Edison, The [1933] All ER Rep 144, sub nom Liesbosch, Dredger v Edison, SS [1933] AC 449, 102 LJP 73, 149 LT 49, Digest
Supp.
Fairholme v Firth and Brown Ltd (1933), 149 LT 332, 49 TLR 470, Digest Supp.
Hall & Co Ltd v Pearlberg [1956] 1 All ER 297, [1956] 1 WLR 244, 3rd Digest Supp.
Herring v British Transport Commission [1958] TR 401, 37 ATC 382.
Houghton Main Colliery Co Ltd, Re [1956] 3 All ER 300, [1956] 1 WLR 1219, 3rd Digest Supp.
Island Tug & Barge Ltd v SS Makedonia (Owners) [1958] 1 All ER 236, [1958] 1 QB 365, [1958] 2 WLR 256, [1957] 2 Lloyds
Rep 49, 3rd Digest Supp.
Jacobs v London County Council [1950] 1 All ER 737, [1950] AC 361, 114 JP 204, 26 Digest (Repl) 472, 1604.
Jordan v Limmer and Trinidad Lake Asphalt Co Ltd [1946] 1 All ER 527, [1946] KB 356, 115 LJKB 379, 175 LT 89, 36 Digest
(Repl) 200, 1058.
659
London Investment and Mortgage Co Ltd v Inland Revenue Comrs, London Investment and Mortgage Co Ltd v Worthington
(Inspector of Taxes) [1958] 2 All ER 230, [1959] AC 199, [1958] 2 WLR 842, 38 Tax Cas 86, 28 Digest (Repl) 37, 168.
McGhie (Thomas) & Sons Ltd v British Transport Commission [1962] 2 All ER 646, [1963] 1 QB 125, [1962] 3 WLR 380,
[1962] TR 209.
Monk v Redwing Aircraft Ltd [1942] 1 All ER 133, [1942] 1 KB 182, 111 LJKB 277, 166 LT 42, 2nd Digest Supp.
Morahan v Archer and Belfast Corpn [1957] NI 61.
Payne v Railway Executive [1951] 2 All ER 910, [1952] 1 KB 26, 36 Digest (Repl) 201, 1061, affg, [1951] 1 All ER 1034.
Phipps v Orthodox Unit Trusts Ltd [1957] 3 All ER 305, [1958] 1 QB 314, [1957] 3 WLR 856, 3rd Digest Supp.
Praet (Julien) et Cie, S/A v H G Poland Ltd [1962] 1 Lloyds Rep 566.
Scruttons Ltd v Midland Silicones Ltd [1962] 1 All ER 1, [1962] AC 446, [1962] 2 WLR 186, [1961] 2 Lloyds Rep 365.
Spencer v Macmillans Trustees [1958] TR 417, 1958 SC 300, [1959] SLT 41, 3rd Digest Supp.
Stewart v Glentaggart Ltd [1963] SLT 119.
Telemachus, The, Tantalus (Owners) v Telemachus (Owners) [1957] 1 All ER 72, [1957] P 47, [1957] 2 WLR 200, [1956] 2
Lloyds Rep 490, 3rd Digest Supp.
Vaughan v Archie Parnell & Alfred Zeitlin Ltd (1940), 23 Tax Cas 505, 28 Digest (Repl) 29, 130.
West Suffolk County Council v W Rought Ltd [1956] 3 All ER 216, [1957] AC 403, [1956] 3 WLR 589, 120 JP 522, 3rd Digest
Supp.

Appeal
This was an appeal by B N M Laboratories Ltd the defendants, against an order of Master Jacob assessing damages at 1,200
(viz, 1,100 in respect of lost salary and 100 in respect of lost commission) on an interlocutory judgment signed against the
defendants for damages to be assessed in an action by Anthony Mesnard Parsons, the plaintiff, for damages for wrongful
dismissal. The grounds of appeal were that the 1,200 ought to be reduced by deducting from it (i) 320 in respect of income tax
at the standard rate, which would have been payable on the plaintiffs lost salary and commission if he had been paid them, and
(ii) 59 2s 6d in respect of unemployment benefit that the plaintiff has received for the interval before he entered new
employment.

Sir Andrew Clark QC and J F Kingham for the defendants (the appellants).
Ashton Roskill QC, H H Monroe QC and Adrian Hamilton for the plaintiff, the respondent.

Cur adv vult

4 April 1963. The following judgments were read.

SELLERS LJ. The plaintiff in this action sought no more of the court than that it should assess the damages due to him from the
defendants by reason of their wrongful termination of a simple agreement of service.
The plaintiff is an experienced and well qualified chemist and was employed by the defendants as their chief chemist and
technical adviser under an agreement dated 11 February 1961, at a salary of 2,000 per annum plus a specified commission for a
term of three years from 15 March 1961, and thereafter until determined by either party giving to the other three months notice
in writing not to expire before 15 March 1964. By letter of 19 April 1962, the defendants informed the plaintiff that they would
employ him only until the end of July, 1962, after which date they would cease to pay any further salary. The plaintiff accepted
that breach and repudiation of the agreement by the defendants and left their employment on 31 May 1962. He was paid full
salary and commission 660 up to that date and for the balance of his agreement up to 15 March 1964, he claimed damages.
A more simple and conventional common law issue it would be hard to find. There was no defence. The plaintiff signed
interlocutory judgment against the defendants for damages to be assessed by a master, and when the matter came before Master
Jacob, with two experienced junior counsel to assist him with the evidence and issues, one might have assumed that the
assessment would have been economically and expeditiously concluded to the satisfaction of both parties.
On 27 August 1962, the plaintiff had obtained another appointment in the research laboratory of Unilever Ltd on a monthly
basis at a salary of 1,600 per annum and the prospects were that the employment would continue for a long time, but without
any real likelihood of any increase of salary. In calculating the difference between the contractual salary and the new salary, that
is 400, for the balance of the period to 15 March 1964, about eighteen and a half months, the learned master, be it noted, had
regard to the gross earnings or, as I would prefer to say, the earnings in each case and made no deduction of tax, although
presumably some tax would arise on the 1,600 a year then being earned elsewhere by the plaintiff so as to reduce its net benefit
to the plaintiff, if that is the way in which it has to be regarded. The judgment was for 1,100 loss of salary and 100 loss of
commission, a modest total of 1,200, but the defendants gave notice of appeal alleging that the damages should only have been
820 17s 6d.
Both sides have appeared before us with leading counsel, the respondent, the plaintiff, employing a second leader with
particular experience in our revenue law. The major trouble has arisen over income tax. The defendants have submitted that
320 in respect of tax should be deducted from the sum assessed for loss of salary and commission. The minor dispute, scarcely
less strenuously fought on both sides, concerns a sum of 59 2s 6d which the plaintiff received as unemployment benefit during a
period when he was registered at the employment exchange, before acquiring the new employment, for which the defendants
claim a further credit.
It seems to have been readily accepted that British Transport Commission v Gourley was a decision to be applied to the
assessment of damages in a case such as this, unless some provisions of the Finance Act, 1960, have a contrary effect. Master
Jacob said so in terms and the submissions before us progressed on that basis, although I ventured to question it in the course of
the argument. In Phipps v Orthodox Unit Trusts Ltd, not insignificantly an interlocutory appeal on particulars in relation to tax
liability in a case of wrongful dismissal, Jenkins LJ said:

Since that decision [that is British Transport Commission v. Gourley] it has been established that where a claim is
made for damages, whether for personal injuries or for wrongful dismissal, the income tax and surtax liability of the
plaintiff is an essential element in the calculation of damages.

It is with all deference to that and my brethrens views that I would question whether the Gourley decision is so far-reaching, and
in particular whether it is binding in a case of wrongful dismissal. The real question is whether the opinion of Lord Goddard CJ
with the undoubted weight of his authority and with some general approval of some of the other noble Lords, is more than an
obiter dictum.
In West Suffolk County Council v W Rought Ltd the decision in Gourleys case was applied and Lord Morton put the decision
of the House in these terms ([1956] 3 All ER at p 221; [1957] AC at p 413):
661

It seems to me that the reasoning which led this House to its decision in British Transport Commission v. Gourley is
equally applicable to the present case. In British Transport Commission v. Gourley the House had to decide what was the
actual monetary loss suffered by the injured man by being deprived of earnings which he would have made if he had not
been injured. In the present case the House has to decide under item (3) (B) what was the actual monetary loss suffered by
the respondents by being deprived of profits which they would have made in the period of nine months already mentioned.
I can see no reason why, if the incidence of taxation was a relevant factor in British Transport Commission v. Gourley, it is
not also a relevant factor in the present case.

Neither Gourleys case nor Roughts case was a case of damages for breach of contract, and I do not think that the decision in
Roughts case in itself interprets Gourleys case as applying to such a case as the present.
The width of the argument for the defendants called in question, to my mind, the very basis on which the decision in
Gourleys case rests, but the present question is only whether Gourleys case applies to a case of wrongful dismissal. Under the
Finance Act, 1960, provision is made for the taxation of sums acquired by an employee in the circumstances specified, which
would include damages for breach of a service agreement, in excess of 5,000. The argument was, as I understand it, that where
such a sum fell to be assessed the court in order to apply the dominant rule of law expressed in Gourleys case must, after
assessing the amount properly due, add on the tax which the plaintiff would have to pay on that sum and award the gross sum so
that the plaintiff would in the result, after payment of the tax, receive the appropriate or correct compensation.
I am tempted to ask what has gone wrong with the common law that it can seriously be submitted as a necessary corollary to
one of its principles that a defendant can be called on to pay the tax imposed on damages received by a plaintiff. If this argument
had been adumbrated or foreseen in Gourleys case it might well have influenced those noble lords who had previously held
otherwise to have adhered to their original views. The mere statement of the argument seems to me to reveal how extraneous and
remote taxation may be in the assessment of damages. At no period in the history of our common law can I contemplate a judge
directing himself or a jury to impose on a defendant the tax, or the best estimate that can be made of it, which a plaintiff would
have to pay on an award of damages. The Finance Act, 1960, in making the special provisions here relevant in regard to tax
cannot have had any such possibility in contemplation.
Gourleys case and Billingham v Hughes were actions in tort where the earnings of professional men which had been lost or
diminished by reason of the defendants negligence had to be assessed. Gourleys case changed the law and established in such a
case that in calculating the damages the incidence of tax should be taken into account. It was a new application of the old
principle of law that the wrongdoer should make good to a plaintiff the financial loss which he has suffered, or will probably
suffer, as a result of the defendants wrongdoing.
That is not this case, which concerns two parties to a contract. Close as the analogy may be between the loss of earnings,
either from a contract of service or otherwise, by reason of the negligence of a third party and the loss of earnings due to breach
by a party to the contract which brought about the loss, I am not satisfied that the same considerations apply. A third party is not
bound by the letter of a contractual bond and the approach to the assessment of damages 662 against a third party permits of a
more general and broader view as established by Gourleys case. Earl Jowitt said in Gourleys case ([1955] 3 All ER at p 800;
[1956] AC at p 199):

There may well be a difference between actions for personal injuries and actions for wrongful dismissal in regard to
the obligations of the plaintiff to pay tax on the amount of damages received; and cases on the one topic may, therefore, be
a dangerous guide to follow on the other. DU PARCQ, J., in the course of his judgment, said a: I should be reluctant to
give a decision which would seek to alter an inveterate practice unless I were convinced that the practice is inconsistent
with principle, and unjust, and I am not so convinced in this case. On the contrary, I am of opinion that it is right in
principle, to have no regard, in assessing damages as between master and servant, to the servants liability to the Crown,
which is truly res inter alios acta.
________________________________________
a In Fairholme v Firth and Brown Ltd (1933), 149 LT at p 333.

Notwithstanding the limited ground of differentiation I do not think it can be said that Earl Jowitt was expressing any opinion on
the particular problem which might arise between master and servant. Lord Goddard CJ it is true, in terms as well as in his
reasoning treats the wrongful dismissal action and the personal injuries action without distinction and Lord Radcliffe and Lord
Somervell agreed in general terms with Lord Goddards speech. Lord Tucker said ([1955] 3 All ER at p 810; [1956] AC at p
215):

I think the true answer is that expenditure whichalthough not actually a charge on earningsis imposed by law as a
necessary consequence of their receipt is relevant to the ascertainment of the loss suffered by the party injured.

Lord Reid though that a deduction of tax would be justified because the employer who had wrongly dismissed a servant would
have to pay the loss but ([1955] 3 All ER at p 809; [1956] AC at p 214) he will have to pay someone else to perform the
services. However that may be, the indications in the present case are that the defendants were reducing staff and that the
plaintiff was not replaced, and that surely would often be the case where there was unjustified dismissal.
The conclusion which I reach on this decision, and I hope I show proper appreciation, is that the opinions on the application
of the majority decision to a case of wrongful dismissal are obiter dicta and, although of great weight, are not binding on this
court. I do not think that the doctrine of precedent or stare decisis should be pressed so far as to include obiter dicta, however
close the reasoning of the actual decision may be to its extended application. If unfettered, as I believe I am, I would not apply
the reasoning of Gourleys case to the present case but would decide it on the same lines as Du Parcq J in Fairholme v Firth and
Brown Ltd ((1933), 149 LT at p 333), from which Earl Jowitt cited the passage I have quoted above. Atkinson J decided similarly
in Jordan v Limmer and Trinidad Lake Asphalt Co Ltd.
The justification for regarding tax in assessing damages is at least more questionable when looked at in terms of contract.
The employer has at all times during the subsistence of a service agreement to pay the employee the contractual sum, no
contractual provision is made for any deduction of or reduction by tax. It is true that the sum the employee receives when it
comes to be distributed by him has to meet the requirements of income tax which for the convenience of administration under our
PAYE scheme is, in effect, handed back to the employer to pay over to the Inland Revenue on behalf of the employee, the
taxpayer. It seems unnecessary to say that that is the employees 663 expenditure and payment to meet his contribution to the
services he receives from the state and to which he may have to make further contribution by way of surtax. If his salary were
less, the employee would no doubt pay less tax, but whatever he pays in taxes in no way reduces the contractual obligation on the
employer to pay the whole of the agreed sum.
A further distribution from the sum earned may be enforced by the employer taking the employees contribution towards the
weekly stamp which is the payment towards the national health and welfare services. Yet another distribution of earnings in the
majority of cases goes directly or indirectly in the payment of rates, that is for the local services which are provided and enjoyed
and for which a compulsory levy is made on the occupiers of property. Not one of these, or any other expenditure or distribution
of earnings, is the concern of the employer or serves to reduce his liability to his employee for the sum due under a service
agreement in any way. There is a distinction between an employees earnings and his distribution of them, and distribution,
however made, does not and cannot diminish or affect the contractual earnings. It is the distinction made by Donovan LJ in
Browning v War Office ([1962] 3 All ER 1089 at p 1093), between an asset and a liability.
If the weekly, or monthly, or quarterly sums are in fact paid under a contract of service the employer pays the full rate of
salary. Whatever is allocated for income tax is sent to the Inland Revenue. If these sums are not paid because any agreement to
pay them has been broken by the employer, then the damages are based on the earnings over whatever period there is default and
they are grossed up into a sum appropriate to an immediate payment of the whole after all factors relating to mitigation of the
damages have been taken into account. In such circumstances income tax might appropriately have been made payable by the
employee on the damages received, for he will have had his earnings so computed, and the tax could be assessed on the basis of
probable liability, as Gourleys case ([1962] 3 All ER 1089 at p 1093) contemplates it will be where Gourleys case applies, or on
some other basis, and either paid by the employer to the revenue or collected direct from the taxpayer, but as it chances (and it
may possibly be a flaw or omission in our taxation law) the grossed up lost income is treated as a capital sum and is not,
subject to some recent provisions which call for consideration, exigible to income tax. This is a fortuitous circumstance which
has nothing to do with the employer or his contract of service with the employee and cannot, as I see it, enure to his benefit so as
to result in his contract being treated as one not to pay the sum stipulated but some lesser sum which might vary between a
number of employees all having a common form of contract for, say, 2,000 a year but each having a different tax liability. The
argument which would give this undoubted gain to an employer, and the greater the tax liability the greater his gain, has gone so
far, and as I see it so oddly, that for an employee to ask to be rewarded in the terms of his contract has been spoken of as punitive
on a defendant. If tax is taken into consideration the employer gains, that is he pays less than he has contracted to pay, and the
employee loses the benefit of no tax on a capital sum which a perhaps charitable revenue law gives him.
It may be irrelevant in assessing damages, as Tucker LJ said in Billingham v Hughes ([1949] 1 All ER at p 686; [1949] 1 KB
at p 648), to consider whether a party who has broken his contract has benefited by his breach by paying less by way of damages
than if he had paid the salary due under his agreement. It would, of course, be a benefit to deduct the tax and it leaves no room
for suggestions of punitive imposition if the contract in its terms is sought to be enforced. However, it cannot, I think, be
irrelevant in the assessment to adhere to the contract and ascertain what payment the contract required. There is no other
contractual sum available for consideration
Gourleys case is said to apply to commercial contracts, to damages for 664 defamation of character, and, I understand, in all
cases where loss of earnings becomes a factor for assessment. Where there is an available market for goods, can the defaulting
buyer or seller look beyond the difference in market price, or the defaulting charterer look beyond the terms of either his contract
or the freight market, and ask that the incidence of tax in any given case should mitigate his liability or the undefaulting
contracting party ask that the defendant should meet his tax liability, so that he suffers no loss thereby? There is nothing
surprising that a man not in default should receive a benefit by the incidence of taxation, but that the benefit given to a plaintiff
by not taxing his damages which represent future earnings should accrue to a defendant, so as to reduce his liability, is indeed
surprising. It gives the benefit to the wrong person.
The assessment of damages for breach of a service agreement or other contract is an everyday commonplace issue in the
courts, so often to be decided in a county court, where the sums involved are small, or, as here, before a master, and it seems
undesirable that the matter should become if not wholly impracticable at least complicated and expensive, as the introduction of
tax problems may well make it.
Although no objection was apparently made here, one can imagine without excess of fancy that a plaintiff having been
dismissed from employment and seeking other employment might feel a prejudice to himself or to others and might be reluctant
therefore to disclose his tax position, his private income or his wifes income affecting his own liability to income tax or surtax or
the fact that he supported a dependent relative. It may be that some of such information is embraced within a code number and,
clothed in that anonymity, would be known to a defendant, a defaulting employer, but it is quite another thing to have to reveal it
in open court in order to establish a claim for damages for breach of contract. Particulars and discovery may be necessary if the
burden of tax is a legitimate issue, and this court has pointed out that such requirements may be oppressive: (see Phipps v
Orthodox Unit Trusts, Ltd per Jenkins LJ and Parker LJ.
Therefore, far from extending the principle of Gourleys case as the defendants urge us to do, I would hold that the decision
strictly viewed is restricted to the assessment of damages against a third party in tort and has no application when the assessment
of damages arises between the parties to such a contract as the one now under consideration. It would, if applied, as I see it, have
the effect of making a new and different contract between the parties, and this the courts have consistently refused to do.
The argument in favour of the appeal went, as I have already indicated, as far as to apply Gourleys case even if thee award
of damages were to be the subject of taxation. First, it was said, deduct income tax and where appropriate surtax also, and on that
basis calculate the damages on the net income and so apply the dominant rule of law that a wrongdoer is liable for such damage
as the plaintiff has sustained or Lord Goddard CJs, statement ([1955] 3 All ER at pp 804, 805; [1956] AC at p 206) that

the basic principle so far as loss of earnings and out-of-pocket expenses are concerned, is that the injured person
should be placed in the same financial position so far as can be done by an award of money, as he would have been had the
accident not happened.

Then having arrived at that sum, if the damages are to be subject to a tax in whole or in part, a sum must be added as the other
side of the equation so that the plaintiff eventually will receive the correct sum after the tax on his damages has been met. Tax
in these circumstances, it is submitted, is to be taken into account on both sides. I find this a wholly unacceptable argument.
Gourleys case proceeded on an admission and acceptance that the damages 665 awarded would not be subject to tax and there is
no indication in the speeches that any of their lordships were anticipating or contemplating the position which would arise if tax
were to be levied on the damages awarded.
The plaintiffs counsel refuted the equation argument, pointing out that the prerequisites of an equation would be absent,
as one side would be an ascertained or known tax liability and the other side a notional and uncertain assessment and like would
not be compared with like. If this issue calls for our consideration, that is if any damages would be taxable and on the
assumption that Gourleys case applies to a case of contract, I would not accede to the defendants argument but would restrict
Gourleys case to its circumstances, which were that the plaintiffs earnings, of which he was or would be deprived by the
accident, would have been subject to tax and any damages he received as compensation for this loss would not be taxed. This is
the view taken by both the courts in Scotland in Spencer v Macmillans Trustees and in Morahan v Archer and Belfast Corpn in
Northern Ireland, and by Buckley J in Diamond v Campbell-Jones and Phillimore J in Thomas McGhie & Sons Ltd v British
Transport Commission in our own courts. In Julien Praet et Cie, S/A v H G Poland Ltd Mocatta J with his wide experience of
commercial law and the assessment of damages for breach of commercial agreements, refused to take into consideration the
incidence of taxation which in that case would have been Belgian taxation.
Lord Hunters opinion in Stewart v Glentaggart Ltd gives support to the defendants argument and may have been the origin
of it. The learned judge rejected the view that mere chargeability to tax in respect of damages is itself sufficient to exclude the
operation of the principle in Gourleys case or that actual liability for tax excluded it, and he regarded the addition of the tax to
the damages, where tax is imposed on them, as simply a corollary to Gourleys case.
Lord Reid pointed out in Gourleys case that the assessment of damages is a practical matter and an element to be considered
is whether bringing in the matter of liability to tax would seriously increase the duration and expense of trials. I feel real
apprehension that this extension which is proposed would frequently increase the duration and expense of trials, and, as far as a
contract of service is concerned, I would ask, for what purpose?merely to reduce the obligation which a defendant had
expressly undertaken. These courts are not unfamiliar with the intricate arguments of counsel learned in our revenue law in its
ever-changing aspects, and if this is to be the law, the assessment of damages may frequently require such expert assistance.
After all the courts endeavour, the Inland Revenue, who would not have been parties to the suit, might take a different view and
the damages enhanced by tax might be proved to be all wrong. The court should not be placed in such a position nor, I would
add, should the litigant who can produce a simple contract and make a simple request to have as damages what he ought to have
had as salary, subject to the long-established requirement that he should act reasonably to mitigate the lost earnings. As Lord
Greene MR said in Monk v Redwing Aircraft Ltd ([1942] 1 All ER 133 at p 135; [1942] 1 KB 182 at p 186),

The [plaintiffs] salary is really one element in ascertaining what the just measure of damage is, and another element
in ascertaining the just measure of damage is the salary which he in fact earned in another employment. Nevertheless, the
damages claimed are damages, and not, so to speak, the balance of an account.

What in the present case should be taken into account in mitigation of the loss of earnings calls for later consideration. But I do
not see that income tax can be 666 regarded in the light of mitigation of damages. Tax is the converse of earnings. It is the
taxpayers disbursement or distribution, albeit obligatory, of part of his earnings, and just like any other expenditure it leaves him
less to spend on other things, but an obligation to pay 2,000 a year or its equivalent by way of damages cannot, in my view, be
treated as an obligation to pay less than 2,000 when the loss occasioned by breach of the agreement comes to be assessed.
I turn now to the effect of the Finance Act, 1960. Master Jacob took the view that the Act made damages for wrongful
dismissal taxable and therefore he held that Gourleys case did not apply. The defendants say that in this the learned master was
wrong, that for amounts not in excess of 5,000 damages are not chargeable to tax and therefore, whether they are right or not on
their first wide submission, they are right in this particular case on this second argument. The relevant parts of the Act are as
follows. Section 37 provides:

(1) Subject to the provisions of this and the next following section, income tax shall be charged under Sch. E in
respect of any payment to which this section applies which is made to the holder or past holder of any office or
employment, or to his executors or administrators, whether made by the person under whom he holds or held the office or
employment or by any other person.
(2) This section applies to any payment (not otherwise chargeable to income tax) which is made, whether in pursuance
of any legal obligation or not, either directly or indirectly in consideration or in consequence of, or otherwise in connexion
with, the termination of the holding of the office or employment upon any change in its functions or emoluments, including
any payment in commutation of annual or periodical payments (whether chargeable to tax or not) which would otherwise
have been made as aforesaid
(4) Any payment which is chargeable to tax by virtue of this section shall be treated as income received on the
following date, that is to say: (a) in the case of a payment in commutation of annual or other periodical payments, the date
on which the commutation is effected; (b) in the case of any other payment, the date of the termination or change in respect
of which the payment is made, and shall be treated as emoluments of the holder or past holder of the office or employment
assessable to income tax under Sch. E; and any such payment shall be treated for all the purposes of the Act of 1952 as
earned income.

Section 38 provides:

(3) Tax shall not be charged by virtue of the last foregoing section in respect of a payment of an amount not exceeding
five thousand pounds, and in the case of a payment which exceeds that amount shall be charged only in respect of the
excess: Provided that where two or more payments in respect of which tax is chargeable by virtue of that section, or would
be so chargeable apart from the foregoing provisions of this subsection, are made to or in respect of the same person in
respect of the same office or employment, or in respect of different offices or employments held under the same employer
or under associated employers, this subsection shall apply as if those payments were a single payment of an amount equal
to that aggregate amount; and the amount of any one payment chargeable to tax shall be ascertained as follows, that is to
say (a) where the payments are to tax shall be ascertained as follows, that is to say (a) where the payments are treated as
income of different years of assessment, the said sum of five thousand pounds shall be deducted from a payment treated as
income of an earlier year before any payment treated as income of a later year; and (b) subject as aforesaid, the said sum
shall be deducted rateably from the payments according to their respective amounts.
(4) The person chargeable to tax by virtue of the last foregoing section 667 in respect of any payment may, before the
expiration of six years after the end of the year of assessment of which it is treated as income, by notice in writing to the
surveyor claim any such relief in respect of the payment as is applicable thereto under Sch. 4 to this Act; and where such a
claim is duly made and allowed, all such repayments and assessments of tax shall be made as are necessary to give effect
thereto.
(5) For the purposes of this section and of Sch. 4 to this Act offices or employments in respect of which payments to
which the last foregoing section applies are made shall be treated as held under associated employers if, on the date which
is the relevant date in relation to any of those payments, one of those employers is under the control of the other or of a
third person who controls or is under the control of the other on that or any other such date.

Turning to the fourth schedule, that is headed Reliefs in respect of tax under s 37. Paragraph 12 and para 13 of that schedule
are as follows:

12. Any reference in the foregoing provisions of this schedule to a payment in respect of which tax is chargeable
under s. 37 of this Act is a reference to so much of that payment as is chargeable to tax after deduction of the relief
applicable thereto under sub-s. (3) of s. 38 of this Act.
13. In this schedule payment of compensation for loss of office means a payment made(a) in pursuance of an
order of a court in proceedings for wrongful dismissal or otherwise for breach of a contract of employment, or by way of
settlement of such proceedings or of a claim in respect of which such proceedings could have been brought; or (b) by way
of compensation for the extinguishment of any right the infringement of which would be actionable in such proceedings;
and any question whether, and to what extent, a payment is or is not a payment of compensation for loss of office shall be
determined according to all the circumstances and not (or not exclusively) by reference to the terms on which it is
expressed to be made.

It is clear that these provisions are applicable to payments made to an employee as damages for breach of a contract of service.
This view was accepted by both sides and, were damages for such a breach not included, the provisions could easily be evaded.
The argument for the defendants was simply this, that s 37(1), which stipulates that income tax shall be charged under Sch E
in respect of any payment to which the section applies, is made subject to the provisions of the next following section. Section
38(3) stipulates that tax shall not be charged by virtue of the last foregoing section in respect of a payment of an amount not
exceeding 5,000 and therefore it is said the provisions have no effect in the present case, where the damages cannot be more
than 1,200. The directness of that argument is apparent but it seems to me to disregard the proviso to sub-s (1) and I am
persuaded by the argument of the plaintiffs counsel and particularly, if I may say so, by that of Mr Monroe, that the provisions
should not be construed in that limited way. The effect of these provisions taken as a whole is to treat the payments to which they
refer as income receipts, not, as previously, for instance in the case of damages, as a capital sum. This is of general effect and
includes the first 5,000 in the class of income receipts and the scope of tax, although it may be relieved in whole or in part from
bearing any actual tax. The proviso throws light on the nature and effect of this small self-contained code and supports, if it does
not require, the view that payments under 5,000 are income receipts for tax purposes. The proviso and its provision for
aggregation make it appear that if a man had two separate contracts of service with the same employer or with associated
employers and the appropriate damages under each in the event of breach would be 4,000, if they were both broken in the same
year the two sums would be treated as a single payment of 8,000 and the excess of 3,000 over 5,000 would be chargeable to
tax rateably, that is on 1,500 in respect of the damages recovered 668 under each contract. If this occurred there would be
double taxation of the damages in effect. So long as there are circumstances in which that can arise, if that be right, the payments
under each separate contract must, I think, be income receipts which are subject to tax unless relief is to be found within the
provisions.
The sum, therefore, awarded in this case is to be regarded as an income receipt within the scope of tax, but as one which
escapes actual taxation because of the relief given. I would not say, as the learned master said, that it necessarily follows that the
principles of Gourleys case could no longer be applied, for in this particular case no tax would in fact arise on the damages but,
since tax might be payable on sums awarded for breach of contract if less than 5,000 (for example in the case of aggregation)
and certainly if over 5,000, it would seem to me that the only satisfactory rule would be to ignore taxation altogether in the
assessment of the damages. Therefore by reason of these provisions of the Finance Act, 1960, in my opinion the sum does not
fall within the principle of Gourleys case if I am wrong in my view that Gourleys case does not apply to a case of a contract of
service at all. This is no doubt a fortuitous result of the provisions of the Finance Act, 1960, which are relied on, but it serves its
purpose in the issues raised in this appeal.
On both the grounds which I have indicated I would uphold the decision of Master Jacob that no tax is deductible from the
sum of 1,200 which he assessed as compensation.
There is, however, no doubt that the plaintiff had to act reasonably to mitigate the loss which his unemployment created. He
had family responsibilities and he very properly received unemployment benefits to which he was entitled and these amounted to
59 2s 6d. Master Jacob took the view that this sum should not be deducted, holding, I think, that it was to be likened to an
insurance policy held by a plaintiff and that it was a benefit to which the plaintiff whilst unemployed would be entitled whether
wrongfully dismissed or not. The benefit which the plaintiff obtained was just that provision made by employer, employee and
the state to bridge over or attenuate the hardships of a period of unemployment by providing a weekly payment when earnings
are not forthcoming. With the employer as a contributing party I do not regard the payment as comparable to benefits received
under a purely personal insurance policy or by way of pension or benefit arising out of a mans employment. In the one case the
employee has paid a premium and in the other he has a contract of employment and his services given under it form the reward
(equivalent to a premium) which entitles him to the benefit.
I can see no reason why a defaulting party should obtain the benefit of a payment for which he has not paid any part of the
premium or given the services which command the benefit. In such matters I would apply the decision of this court in Payne v
Railway Executive. That case was applied in Canada in support of a similar question to that which we have to consider here, but
although I was a party to the decision in Payne v Railway Executive, at first instance, with respect I would not so apply it, but
would distinguish it. I would adhere to what was said in Payne v Railway Executive and by Donovan LJ in Browning v War
Office ([1962] 3 All ER 1089 at p 1093), but I think that where, as here, the employer has made a contribution to the
unemployment insurance he should get the benefit of it, if he finds it necessary to put one of his employees into unemployment,
even in circumstances where he is liable to compensate him in damages.
Browning v War Office, a decision of this court to a contrary effect to Payne v Railway Executive, was referred to and in
response we were invited by learned counsel for the plaintiff to apply to the majority decision the trenchant 669 observations of
Viscount Simonds in Scruttons, Ltd v Midland Silicones Ltd. It is no doubt a judicial duty, if occasion demands, to pass criticism
on a lower court, but in relation to my colleagues in this court I do not feel called on to disapprove but would with all deference
feel free to disagree. I would still holdas I think I maythat Brownings disability pension was not, in substance,
distinguishable from Paynes benefit, and I would have dealt with them both in the same way and would not have regarded Payne
v Railway Executive as affected by Gourleys case, which was quite a different case. I find it difficult to appreciate, as I have
said, that it can be in any way regarded as punishing a wrongdoer because the courts do not give him the benefit of an injured
partys prudence or thrift or contractual provision. It seems to me unjust to relieve him of his obligation and to deprive a plaintiff
of compensation by reason of so fortuitous and extraneous a circumstance. The benefit arises from provision made for the future
quite outside the employment which has wrongfully been brought to an end or interrupted and is not something which a plaintiff
is called on to do to mitigate the loss which would otherwise arise from unemployment. It is in a totally different category.
Since Payne v Railway Executive under the common law, the Fatal Accidents Act, 1959, s 2, has provided that there shall not
be taken into account in assessing damages in respect of a persons death in any action under the Fatal Accidents Act, 1846, any
pension or gratuity which has been or will or may be paid as a result of the death. I would regard it as a retrograde step for the
common law now to go back on what Payne v Railway Executive in the Court of Appeal decided. The same section also excludes
in the assessment of damages any benefit under the National Insurance Acts, but benefits under these Acts have previously been
treated on a special basis in respect of common law claims and I do not feel that it is necessary to give any indirect effect to such
statutory provisions in dealing with damages for wrongful dismissal.
Here the employer has played some part in making the provision and I would hold that the plaintiff should give credit for the
59 2s 6d just as he gives credit for his actual earnings in order to mitigate the loss. It directly mitigates the loss of earnings and
is in effect a partial substitute for them. It is clearly an asset, sufficiently direct, and not, as is a tax, a liability.
I would therefore allow the appeal to the extent that the damages should be reduced by 59 2s 6d.

HARMAN LJ. This appeal arises out of an award to the plaintiff of a sum of damages for wrongful dismissal by the defendants.
That the plaintiff was wrongfully dismissed was not denied. He was in receipt of a salary of 2,000 a year and some commission
under an agreement which was not to expire earlier than 15 March 1964. He was dismissed at the end of May, 1962. The
question of damages was referred to the master, who awarded the plaintiff the sum of 1,200, that being the masters calculation
of the gross loss which the plaintiff had suffered. The master declined to make either of two deductions which the defendants
claimed he should have made, namely, first the estimated sum of tax which the plaintiff would have suffered on the lost earnings,
and secondly the unemployment benefit he received. These two sums have been agreed at 320 and 59 2s 6d respectively and
those are the sums here at issue.
As to the first, the decision of the House of Lords in British Transport Commission v Gourley undoubtedly altered the law,
to an extent to be hereafter discussed, by admitting in the calculation of damages in respect of loss of earnings in a case of tort the
income tax factor, that is to say the amount of income tax which the plaintiff would have had to pay if he had not been prevented
from acquiring the earnings. I think it is right to say that the decision of the majority 670 clearly assumed, if it did not decide,
that the same principle would apply to damages in respect of wrongful dismissal, that is to say a case of breach of contract. This
factor appears in the argument of counsel for the appellants ([1956] AC at p 188), and although Earl Jowitt in his speech ([1955]
3 All ER at p 800; [1956] AC at p 189) reserves his opinion on the point, expressing the view that there may well be a difference
in this respect between actions for personal injuries and actions for wrongful dismissal, yet Lord Goddard CJ in his speech
expressly states that the principles which he sets out in the form of an imagined charge to a jury would be applicable to wrongful
dismissal actions as well as to actions in tort ([1955] 3 All ER at p 807; [1956] AC at p 210). With this opinion Lord Radcliffe
and Lord Somervell agreed, as I think did Lord Tucker, as witness his words ([1955] 3 All ER at p 810; [1956] AC at p 215):

I think the true answer is that expenditure whichalthough not actually a charge on earningsis imposed by law as a
necessary consequence of their receipt is relevant to the ascertainment of the loss suffered by the party injured.

Speaking for myself, I cannot see any valid distinction in this respect between the two causes of action.
Since the decision of the House of Lords it has been accepted in the profession that the Gourley principle is applicable to
cases of wrongful dismissal. I refer in particular to the observations of Jenkins LJ in Phipps v Orthodox Unit Trusts Ltd where
the lord justice treats the matter as established. Consistently with this view it was, as I understand, not assumed before the
learned master below in the present case that, but for the passing of s 37 and s 38 of the Finance Act, 1960, the Gourley principle
would have been applicable to these damages and would have caused their reduction by the sum of 320. The learned master
assumed that this was the position, as indeed did counsel before us. It was the starting point of their arguments. Nevertheless it
has seemed right to Sellers LJ to express the view that Gourleys case has no application to wrongful dismissal cases. As at
present advised I find myself unable to accept this view, which indeed I doubt whether it is open to this court in this case to take.
I hope that I shall be excused from dealing with the matter further because for myself before entertaining any such opinion I
should have wished to reconstitute the court and hear the matter argued. Accordingly what follows is based on the view that until
the passing on 29 July 1960, of the Finance Act of that year the 320 must be deducted; but it is said, and the master so held, that
s 37 and s 38 of the Act of 1960 removed damages for wrongful dismissal from the ambit of Gourleys case. It is therefore
necessary to come to a conclusion what it is that Gourleys case decides. As I understand it, the case decides that in an action
whether in tort for injuries deriving from negligence or in contract for wrongful dismissal, where the loss of the plaintiff consists
wholly or in part in a loss of earnings, and where the sum to be awarded in damages will not suffer deduction of tax in his hands,
then in ascertaining the plaintiffs loss his tax liability, that is to say the sum which he would have had to pay in income tax
including surtax in acquiring the lost earnings, must be deducted from his loss in assessing the damages. It is stated in the
headnote ([1956] AC at p 185) at para 3:

Per LORD GODDARD (LORD RADCLIFFE and LORD SOMERVELL OF HARROW agreeing). Such damages are
awarded as compensation, not restitution, and must be decided by the application of reasonable common sense, taking all
matters which might have affected the plaintiffs tax liability into account. The same principles would be applicable in
wrongful dismissal actions.

The principle involved is simple, namely that damages are a recompense to the 671 victim and not a punishment of the
wrongdoer, and therefore arguments to the effect that it is wrong to allow an advantage to the wrongdoer owing to the fact that
the tax gatherer will take part of the victims damages are irrelevant. On the other hand, to allow the plaintiff to acquire a sum
which under to-days conditions he never could have obtained would put him in a better position financially than if the accident
had not happened. Of course the assessment of damages is a jury question. The assessment of the amount of tax which the
plaintiff might have to pay is a matter to some extent of guesswork; so indeed are other factors such as the continuance of his life
and health and the solvency of his employer. Nevertheless the assessment must be made as best it may be on broad lines. As
Lord Goddard CJ said ([1955] 3 All ER at p 806; [1956] AC at p 208):

The assessment of general damages in these cases is always a matter of difficulty. I do not think that restitutio in
integrum has any application to general damages. The plaintiff receives compensation and not restitution. If he has lost an
eye or a limb he can be compensated by money, but that will not restore what he has lost. So, too, if his earning capacity is
lessened or destroyed, the loss cannot be measured so as to ensure that he is no worse off in the future than he was in the
past and, indeed, if it turned out that the amount of his disability was less than was anticipated at the trial, he might even be
over-compensated. On this matter I would quote LORD DUNEDIN in Admiralty Comrs. v. Susquehanna (Owners), The
Susquehanna ([1926] All ER Rep 124 at p 127; [1926] AC 655 at p 661.): If the damage be general the quantification
of such damage is a jury question For a jury question no rigid rules, or rules that apply to all cases, can be laid down,
but in each set of circumstances certain relevant considerations will arise which, were the matter before a judge, it would
be the duty of the judge in the case to bring before the jury. A judge sitting without a jury must act on the same principles,
and bear in mind the same considerations, as he would direct a jury to do as to the fair amount to award both for pain and
suffering, and for loss of earning capacity, and I think it would be well to remember that it has always been laid down that
damages cannot be a perfect compensation.

Gourleys case was followed in the next year by West Suffolk County Council v W Rought Ltd which applied the principle to an
award of compensation for loss of profit during an interruption of trade. Here again there was evidence which satisfied the court
that the award would not be taxable in the plaintiffs hands.
Since that time Gourleys case has been consistently applied, in cases of industrial accident in particular. It has not been
applied, as I understand it, in commercial cases to awards for loss of profit. For instance, in Hall & Co Ltd v Pearlberg it was
held that lost rent and similar items of loss which would be taxable in the recipients hands as profits ought not to be diminished
on the Gourley principle, because that would be to charge them twice, while a sum of damages which would not be taxable in the
hands of the plaintiff ought to have the proper Gourley proportion deducted from it. A similar result was reached in Morahan v
Archer and Belfast Corpn, a case about trading profits. So again in Diamond v Campbell-Jones. The last paragraph of the
headnote reads as follows:

That since any damages recovered by the plaintiff were liable to attract tax as part of the profits or gains of his
business, he was entitled to a sum in damages equal to the gross amount of the profit, the difference between the 672
purchase price and the market value of the property, which he was likely to have made if there had been no breach of
contract.

Buckley J deals with it in one paragraph ([1960] 1 All ER at p 592; [1961] Ch at p 37):

Such a profit in the hands of the plaintiff would undoubtedly have attracted income tax, and counsel for the defendants
claims that in accordance with the decision in British Transport Commission v. Gourley the defendants should only be
liable for damages equivalent to the net profit which the plaintiff would have retained after paying tax. In Gourleys case,
however, it was conceded that no part of the sum awarded as damages would be subject to tax. Similarly, the decision in
West Suffolk County Council v. W. Rought Ltd., where the principle of Gourleys case was applied to an assessment of loss
of profit as part of the compensation on a compulsory purchase, proceeded expressly on the basis that such assessment
would not be liable to income tax. In the present case, on the other hand, it appears to me that any damages recovered by
the plaintiff are liable to attract tax as part of the profits or gains of his business (London Investment and Mortgage Co.,
Ltd. v. Inland Revenue Comrs., London Investment and Mortgage Co., Ltd. v. Worthington (Inspector of Taxes). I was also
referred in this connexion to Vaughan v. Archie Parnell & Alfred Zeitlin Ltd.. If the damages would be taxable in the
hands of the plaintiff, in order to give him the degree of indemnity to which he is entitled, I must, I think, award him a
gross sum in damages equal to the gross amount of the profit which he would be likely to have made, had there been no
breach of contract.

There are other instances to the same effect.


There is no doubt that all that Gourley decided was that where the damages in the plaintiffs hands would not attract tax then
the principle applied. The House of Lords did not consider cases where such profits were taxable and expressed no opinion on
the point and there is no decision, except those at first instance to which I have alluded, which deals with the point.
Here come in, so far as actions for wrongful dismissal are concerned, the Finance Act, 1960, s 37 and s 38 of which were
held by the master to exclude in all cases the operation of the Gourley principle. It is therefore essential to examine these
sections. Section 37(1) has already been read b and I do not repeat it beyond saying that it is made expressly subject to s 38.
Similarly, I do not re-read sub-s (2) and sub-s (4)(b), which my lord has already read c. So far s 37 has dealt with the persons
coming within the mischief of the tax. It is, however, expressly made subject to s 38, which proceeds to state the cases in which
tax shall not be charged. Subsection (1) I may pass over. Subsection (2) deals with employment outside the United Kingdom;
and sub-s (3) is in terms which my lord has already read and I shall not repeat them. There is a Sch 4 to the Act of 1960 dealing
with administrative matters described as Reliefs in respect of tax under s 37. By para 13 of that schedule payment of
compensation for loss of office is defined under sub-para (a) as
________________________________________
b See p 667, letter c, ante
c See p 667, letter c, ante

a payment made in pursuance of an order of a court in proceedings for wrongful dismissal or otherwise for breach
of a contract of employment, or by way of settlement of such proceedings or of a claim in respect of which such
proceedings could have been brought.

It is said that the primary object of these two sections was to discourage what has become known as the golden handshake, that
is to say the very large payments recently made to retiring directors and others, very much to be preferred by them because
masquerading as capital sums they escape income tax. This perhaps 673 regrettable state of the law dates from the decision of
the House of Lords in Beak v Robson, a decision which surprised the profession at the time. The sections proceed, no doubt in
order to prevent what would have been a simple device for avoidance, to cover certain damages for wrongful dismissal. This
makes clear the view of the legislature that but for the sections such damages are not taxable in the hands of the recipient, which
accords I believe with the practice of the Crown.
Now the damages in the present case were but 1,200 and therefore it is clear that by reason of s 38(3) tax will not be
charged on them. It would therefore seem at first sight that, this being a sum of damages not subject to tax in the recipients
hands, the conditions of Gourleys case are satisfied and the principle applicable. It is argued, and the master so held, that this is
not so. If I understood the argument, it was that this was a payment of a class, namely damages for wrongful dismissal, made a
taxable subject-matter by s 37 and that it was the nature of the payment rather than the question whether tax turned out to be
payable which was decisive. Parliament, it is said, intended to remove all payments for loss of office from the capital to the
income category and that this 1,200 was in that latter category and so therefore to be regarded as subject to exactly the same rule
as if it had exceeded 5,000. This argument is too metaphysical for me. I am entirely unable to accept it. It does not result from
a proper construction of the two sections. As I have already said, s 37(1) is expressed to be subject to s 38 and the latter by sub-s
(3) provides tax shall not be charged. A sum of under 5,000 therefore is not a payment to which this section applies within s
37(1) nor is it a payment chargeable to tax under this section within sub-s (7). The fact that the provisions of s 38 are described
in Sch 4, para 12, as Reliefs seems to me to make no difference at all, nor, in my opinion, do the words of the side-note
Exemptions and Reliefs have any effect. I must, however, here say that I have always been brought up to believe that to
interpret an Act of Parliament by the side-notes to the sections is quite inadmissible, although there are judicial pronouncements
seeming to show that judges have not always refrained, as in my judgment they should, from giving some weight to them.
If I am right so far, that is enough to dispose of this appeal, but the greater part of the time of the court during four days
argument was devoted to a discusion whether as a matter of principle the excess over 5,000 of damages for wrongful dismissal,
or for that matter any award of damages which will be taxable in the hands of the recipient, is thereby excluded from the Gourley
principle. Any view that I may take on this point would be mere obiter dictum and perhaps not worth expressing, but should I be
wrong on the first point so that these damages ought to be treated as in the income category the question would become relevant
and in deference to the argument addressed to us I think I should say something on the subject. We have been supplied with an
elaborate opiniond delivered in the Outer House of the Court of Session on 10 January 1963, in the course of assessing damages
for breach of a managing directors agreement, in which large sums were involved and which involved the sections of the
Finance Act, 1960, that I have been discussing, and the question whether the sections so far as they applied to the excess over
5,000 precluded the operation of the Gourley principle. Lord Hunter posed the two following possible solutions between which
he said he had to choose ([1963] SLT at p 120):
________________________________________
d See Stewart v Glentaggart, Ltd [1963] SLT 119

First, the court might take the view that as Parliament has expressly imposed some measure of taxation on such
damages, then the Gourley principle should not be applied at all to damages which are within the 1960 definition, even
though they may be less than the exempt amount and so not taxable Secondly, the court might carry the Gourley
principle 674 of assessment of damages to its logical conclusion and go through two calculations: first of all the net
amount of damages, as reduced by Gourley, should be calculated, and then a sum should be ascertained of such an amount
as would leave the plaintiff with the net amount after taking into account the plaintiffs liability to tax under the 1960
provisions.

That is set out in a transcript that we have of Lord Hunters opinion, which has now I think been reported in the Scottish Law
Reports. His Lordship came to the conclusion that in a case where no tax is in fact payable in respect of the damages by reason
of reliefs a refusal to apply Gourleys case is unjustifiable. With this, as I have said already, I find myself in agreement. Lord
Hunter goes on to conclude ([1963] SLT at p 121) that it would be right to carry the principle to what he calls (and I agree) its
logical conclusion and to provide for the tax on both sides, that is to say, to make a deduction from the damages on the Gourley
lines and then to add to them such a sum as would provide for the tax exigible on them. This however is a counsel of perfection
and no doubt adds greatly to the difficulty of assessing damages. It was pointed out, for instance, that the first calculation is
purely hypothetical because the profit would never be earned nor the tax become payable, but the second is an estimate of what
will be an actually assessed sum when the matter has come before the Inland Revenue. Thus the comparison, it is said, is not of
like with like. Moreover, in order to make the second calculation it is essential to know the amount of the damages, and if this is
going to depend on the amount of the tax the calculation can never be made. It is a further argument against this principle that
the amount of the tax under Sch E on the damages is to be levied in a single year and will therefore inevitably be quite different
from the estimated tax on the lost earnings spread over the years, however many they may be, of the lost employment.
On the whole I incline to the view that this conclusion of his lordship, though logical, is impracticable, and that it is better,
where the Crown has taken a hand and actually taxed the damages in the recipients hands, to leave the two taxes to set
themselves off one against the other. There may be some roughness in this justice but it does at least make an end of the matter.
The law, as Lord Wright said in The Edison ([1933] All ER Rep 144 at p 158; [1933] AC 449 at p 461), cannot take account of
everything.
There was a third possible result suggested to us, namely to treat the tax imposed by the Finance Act, 1960, on the damages
as too remote and to ignore it but to apply the Gourley principle to the amount awarded. Otherwise, it is said, the law is defeating
the object of Parliament, which was to render payments of this sort unattractive by taxing them. If the defendants and not the
plaintiff are in effect made to pay the tax which Parliament imposed, that is to fly in the face of Parliament. I cannot accept this
view. It would defeat the main principle enunciated in Gourleys case, namely that the victim is to be compensated so far as
money will do it. If this award is to be diminished by taking off the tax he would have paid if he had earned it and then to be
diminished again by being taxed as damages under s 37, the plaintiff would not get his just recompense. Moreover the same
subject-matter would be twice taxed.
On the whole, therefore, I would incline to the view that, as seems to be the practice in commercial loss of profit cases, no
account should be taken of tax at either end. This may be a matter largely dictated by expediency but it strikes me as preferable
in an imperfect world to an over-assiduous search after perfection. To make the punishment fit the crime is no doubt a sublime
object but it may land the searcher in the end in the ridiculous.
Now the second point. If this point were free of authority I should feel little doubt that the grand principle in Gourleys case
ought to apply here also. Unemployment insurance is a sum receivable by contract made by the employed man and his employer
each of whom contributes to the state on the footing that 675 if and when the servant is unemployed the state will make good part
of his earnings to him. I do not think that such a payment is truly analogous to insurance moneys, as in the leading case of
Bradburn v Great Western Ry Co, where there was a purely voluntary contract made by the plaintiff. This is a contribution which
he is bound to make with the very object of mitigating the damage which inability to work will do him. It is just as if his
employer continued to pay part of his wages. The loss he suffers is pro tanto diminished and therefore cannot be charged against
the wrongdoer.
It is argued, however, that on the cases this way of reasoning is not open, and Payne v Railway Executive was cited. That
was a case of sailors disability pension and it was held that no account should be taken of it, largely, as I read the case, because
the Minister of Pensions had the right to withhold or reduce the pension if damages were awarded. Nevertheless it is well
arguable that this case is binding on us. I do not myself think that it is because a disability pension which the minister has a
discretion to withhold is not at all analogous in my view to unemployment pay, which is a subject of contract. However that may
be, this court has recently declined to follow that case in Browning v War Office. That again was a case of a disability pension
and the majority of the court felt entitled to set aside Payne v Railway Executive. Lord Denning accepted the ground to which I
have alluded and felt at liberty to discard the other ground in that it conflicted with the principle of Gourley. Diplock LJ held
expressly that Payne v Railway Executive must be taken to have been overruled by Gourley. Whether I should have had the
courage to arrive at this conclusion it is not for me to say, but I think that the decision does justify the court in feeling free to
reach its own conclusion. On that footing I feel that unemployment pay no less than sick pay or payment of wages during
disability under the contract of employment must be brought into account.
I would therefore allow the appeal on both heads.

PEARSON LJ. The learned master has assessed the damages for wrongful dismissal at 1,200. That is the gross amount of the
lost salary and commission, that is to say the difference between the gross salary and commission which he would have earned, if
his employment had continued for the residue of the agreed term, and the actual gross remuneration which he was able to earn in
other employment in the same period. The lost salary and commission would have been subject to income tax, the amount of
which, as found by the master and accepted by the parties, would have been 320. Moreover in the period of about three months
between the date when the plaintiff left the defendants service in consequence of the wrongful dismissal and the date when the
plaintiff obtained other employment he received unemployment benefit amounting to 59 2s 6d. The defendants contend that the
assessment of 1,200 damages should be reduced by deducting from it, first the sum of 320 for the tax which would have been
payable on the lost salary and commission, and secondly the sum of59 2s 6d for the unemployment benefit received.
The main question in this appeal is whether the deduction of 320 in respect of tax should be made. The learned master
would have made this deduction on the authority of British Transport Commission v Gourley but for the view which he took as to
the effect of certain provisions of the Finance Act, 1960. His view can be summarised very shortly in this way: the principle of
Gourleys case is not applicable unless two conditions are fulfilled, namely, first that the lost earnings or profits would have been
taxable, and secondly that the damages awarded will not be taxable; the effect of the relevant provisions of the Finance Act, 1960,
is to make damages for wrongful dismissal taxable; and consequently the rule in Gourleys case is not applicable. I am for the
moment using the convenient word taxable, which may need examination at a later stage.
676
In opening the appeal counsel for the defendants attacked the Masters view on two grounds: first, that the principle of
Gourleys case has a much wider application, so that it would not be excluded, even if the damages were taxable, and secondly,
that under the relevant provisions of the Finance Act, 1960, the damages in this case are not taxable. I will begin by considering
the first ground.
What is the principle of Gourleys case? Undoubtedly the decision was based partly on the familiar general principle that

in cases such as this the tribunal should award the injured party such a sum of money as will put him in the same
position as he would have been in if he had not sustained the injuries:

(per Earl Jowitt ([1955] 3 All ER at p 799; [1956] AC at p 197): see also Lord Goddard CJ ([1955] 3 All ER at pp 804, 805;
[1956] AC at p 206) and per Lord Reid ([1955] 3 All ER at p 808; [1956] AC at p 212). But that general principle does not
belong distinctively to the Gourley case. On the contrary it has been established, if not from time immemorial, at any rate since a
time long before the year 1955. Indeed it follows necessarily from the obvious truism that damages are intended to compensate
the plaintiff for the damage which he has sustained by reason of the defendants wrongdoing.
The distinctive element of the House of Lords decision in Gourleys case, involving a departure from previous decision of
lower courts, is that, in a case where the lost earnings would have been taxable and the damages awarded will not be taxable, the
notional incidence of taxation on the lost earnings is to be taken into account in reduction of the gross amount of the plaintiffs
loss and consequently in reduction of the amount of damages. The fact that the damages awarded would be immune from
taxation was not an accidental or unimportant feature in Gourleys case: it was a material factor and one of the bases of the
decision. That appears from passages in the speeches. Earl Jowitt said ([1955] 3 All ER at p 799; [1956] AC at p 197):
It was agreed by counsel on both sidesand I think rightly agreedthat the respondent would incur no tax liability in
respect of the award of 37,720, or alternatively of 6,695

and then he said ([1955] 3 All ER at p 800; [1956] AC at pp 198, 199):

The first case in which this question arose for decision was Fairholme v. Firth and Brown Ltd.. That was a case in
which a company had wrongfully dismissed their managing director. The damages were assessed at 18,000, and the
issues for determination were (a) Whether any sum awarded to the plaintiff by way of damages would be subject to British
income tax and/or surtax. (b) If not, whether this fact and the fact that the plaintiff would have been liable for income tax
and surtax if the money had been paid under the agreement, should be taken into account in assessing damages. Point (a)
was not in fact argued, it being agreed by counsel on both sides that the damages would not be subject to tax; and
accordingly DU PARCQ, J., assumed, without deciding the point, that no tax would be exigible on the amount of the
damages, and pointed out that this was the foundation for the argument on point (b). I express no opinion what the answer
to point (a) would have been if it had been the subject of a judicial decision. There may well be a difference between
actions for personal injuries and actions for wrongful dismissal in regard to the obligation of the plaintiff to pay tax on the
amount of damages received; and cases on the one topic may therefore be a dangerous guide to follow on the other.

Lord Goddard CJ said ([1955] 3 All ER at p 804; [1956] AC at p 205):

The parties agreed that, under the present law, no part of the sum 677 awarded as damages was subject to income tax
or surtax and the appeal proceeded on this footing.

Lord Reid said ([1955] 3 All ER at p 807; [1956] AC at p 211):

It is not disputed that, whatever sum is awarded as damages, it will not be subject to income tax or surtax.

Lord Radcliffe said ([1955] 3 All ER at p 810; [1956] AC at p 215) that he had had the opportunity of considering in advance the
speech of Lord Goddard and he agreed with it. Lord Somervell of Harrow ([1955] 3 All ER at p 812; [1956] AC at p 218) also
expressed agreement with the speech which Lord Goddard had delivered.
In West Suffolk County Council v W Rought Ltd there was a letter from the Inland Revenue stating that they did not regard
the compensation in a case of that kind in respect of temporary loss of profits as liable to taxation for income tax purposes. Lord
Morton said ([1956] 3 All ER at p 220; [1957] AC at p 412):

My Lords, the question whether any sum awarded to the respondents under item (3)(B) is or is not liable to income tax
is not before the House, but I see no reason to doubt that the view of the Board of Inland Revenue, as expressed in this
letter, was correct, and I have formed the opinion which I am about to express upon the footing that this is so.

In later cases it has been stated or assumed that the principle of the Gourley case is applicable only if the lost earnings or profits
are taxable and the damages are not. To reduce the length of this judgment, I will not cite the passages but merely give the
references to them: Morahan v Archer and Belfast Corporation, per Curran LJ; Herring v British Transport Commission, per
Donovan J ((1937), 37 ATC 382 at p 383); Island Tug & Barge Ltd v S S Makedonia (Owners) per Pilcher J ([1958] 1 All ER 236
at pp 243, 244; [1958] 1 QB 365 at pp 377, 378); Spencer v Macmillans Trustees per the Lord Justice-Clerk ([1959] SLR 41 at p
49) and per Lord Mackintosh ([1959] SLR at p 53); Diamond v Campbell-Jones per Buckley J; Thomas McGhie & Sons Ltd v
British Transport Commission, per Phillimore J. In my opinion the view stated in those later cases as to the requisites for
application of the Gourley principle is clearly correct, being deducible from the passages which I have cited above from the
speeches of the House of Lords in the Gourley case and in the Rought case.
The practice also has been consistent with that view. On the one hand the Gourley principle has been applied regularly in
accident cases, which are very numerous, and normally it does not cause any difficulty or complication. The figures of all the
relevant earnings (for instance the pre-accident and post-accident earnings, and the lost earnings up to trial for inclusion in the
special damages and the estimated lost earnings after trial to be taken into account in assessing the general damages) are agreed
or ascertained or estimated as net figures after deduction of tax under the PAYE system. The figures of actual net earnings are
readily available. There is not usually any significant investment income, which might complicate the tax computation. No
doubt there must be some cases in which more complicated tax computations are involved, but I think that they are infrequent.
On the other hand, Mr Roskill, counsel for the plaintiff, has confirmed from his wide experience my impression that in the
assessment of damages in commercial cases or cases of breach of contract other than wrongful dismissal the Gourley
678principle has not been applied, and the incidence of taxation, whether on the lost profits or on the damages to be awarded, has
been ignored. In such cases both the lost profits and the damages to be awarded have the character of taxable subject-matter, and
rough justice is done and a great expenditure of time and costs is saved by ignoring the tax on both sides so that in effect the tax
on the lost earnings is set off against and cancelled out by the tax on the damages. The actual amounts of the tax (if any) to be
paid on the one side and the other would depend on the special circumstances of the particular case and might differ widely, but
no attempt is made to ascertain the actual difference and adjust the damages accordingly.
In relation to the practical aspects of the question, an instructive case, affording an instance of the complex and difficult
problems which might arise if tax on the lost profits or lost receipts and tax on the damages had to be ascertained and compared,
is Julien Praet et Cie, S/A v H G Poland, Ltd. Damages had to be assessed for wrongful termination of an agency agreement
relatinn to Belgian motor insurance. Questions were raised as to the incidence of Belgian taxation on the lost profits and on the
damages, and there was conflicting evidence as to the relevant Belgian law. Mocatta J cited ([1962] 1 Lloyds Rep at pp 594,
595) Spencer v Macmillans Trustees, Herring v British Transport Commission and Diamond v Campbell-Jones, and said ([1962]
1 Lloyds Rep at p 596):

I, therefore, apply the law as it has been stated in the three cases I have cited, notwithstanding that its application may
in this, as in other cases, be open to the theoretical objections that it achieves but rough justice and does not provide a
completely satisfactory solution to all the consequences flowing from the breach of contract. In awarding damages to the
underwriters I accordingly take no account of Belgian tax.

He went on to say:

In case I be wrong in this conclusion and the matter goes further, I fear it is my duty to express my conclusions on the
disputed points arising under Belgian tax law;

and he went on to do so.


Having regard to what was said in the House of Lords in Gourleys case and the Rought case and to what has been said in the
later cases, and to the practice, I consider it is impossible to maintain that there can be derived from Gourleys case any principle
requiring taxation to be taken into account in assessing damages in a situation where both the lost earnings or profits and the
damages are taxable. The Gourley case does not deal with that situation. It is open to the defendants to contend, as they have
contended, that in that situation taxation should be taken into account in assessing damages in one or other of two alternative
ways: (A) Taxation should be taken into account on both sides, that is to say both the notional taxation on the lost earnings or
profits and the expected taxation on the damages should be ascertained, and the sum of damages awarded should be such sum as
will after deduction of tax leave a net sum equal to the plaintiffs net financial loss: (B) Taxation should be taken into account on
one side only, that is to say the notional taxation on the lost earnings or profits should be taken into account so as to arrive at the
amount of the plaintiffs net financial loss, and that should be the amount of the damage awarded, no regard being had to any
expected taxation of the damages.
Any such contention is fully open, but has to be established on general principles and on its own merits without any
assistance from Gourleys case or from Roughts case.
679
In my judgment no such contention has been established in the present case. It is possible that in some other case there
might be special hardship or other exceptional features which would require taxation to be taken into account in one of the ways
suggested for the purpose of doing justice in the particular case. For instance, if in some future case a large sum of damages for
wrongful dismissal is awarded, the effect of s 37 and s 38 of the Finance Act, 1960, will be that the first 5,000 of the damages
will be free of tax and the excess over 5,000 will be subject to tax, and that discrepancy will raise an awkward problem as to the
effect, if any, of such taxation on the assessment of the damages. I am not seeking to deal with any such case as that, because the
problem will have to be dealt with when it arises and cannot profitably be considered as an abstract problem without the
particular facts. The arguments in the present case have rested solely on general considerations, and it has not been suggested
that the present case has any special hardship or exceptional feature. It is appropriate therefore to deal with the defendants
contention on broad general grounds.
The contention in its second form (B) can be quickly rejected, because there is no authority to support it and because it is on
the face of it manifestly one-sided and unjust. In any ordinary case, at any rate, it would be unfair to take into account taxation so
as to reduce the plaintiffs loss to a net figure of x, and then to award damages of x with no corresponding addition to cover the
taxation of the damages.
The contention in its first form (A) is more debatable. It has the support of Lord Hunters judgment in the recent and not yet
reported case of Stewart v Glentaggart Ltd,e and it has the attraction of appearing to produce perfect justice. Nevertheless there
are serious objections to it. First, it is not supported by any authority except Stewart v Glentaggart, Ltdf. Secondly, the further
calculations and arguments required would tend to increase the length and expense of trials, and that is an important
consideration because the assessment of damages is an every-day matter. Thirdly, the data required for assessing, or estimating
with a reasonable degree of accuracy, the amount of the tax on the damages might not be available for a substantial time after the
wrongful dismissal, and in such a case either a quick and inaccurate estimate of the tax on the damages would have to be made or
the trial of the action for wrongful dismissal would have to be postponed. Postponement of a trial is in many cases inimical to
justice, because it tends to impair the witnesses recollection of the events. Fourthly, in some cases the amount of the tax on the
damages would be decided as between the plaintiff and the defendant in the absence of the Inland Revenue. The decision would
not be res judicata as against the Inland Revenue, but if a question of principle were involved (as opposed to facts and figures
which could be adjusted) the Inland Revenue would be confronted with a judicial decision as an authority, and would have to go
to a higher court in order to challenge it, and if it were a decision of the House of Lords there would be no higher court. Fifthly, it
is no part of, at any rate, the normal functions of a court of law to increase the amount of an award of damages so as to protect the
plaintiff at the expense of the defendant against the incidence of taxation which the legislature has thought fit to impose. In
Island Tug & Barge Ltd v S S Makedonia (Owners), Pilcher J after referring to the view expressed in The Telemachus, said this
([1958] 1 All ER at pp 244, 245; [1958] 1 QB at pp 379, 380):
________________________________________
e Subsequently reported, [1963] SLT 119, see at p 121
f Subsequently reported, [1963] SLT 119, see at p 121

I do not feel that any principle laid down in British Transport Commission v. Gourley compels, or even inclines,
me to take the view that a salvor who is already generously rewarded on well-known principles 680 should have his award
further increased so as to indemnify him wholly or in part for the fact that the fiscal law to which he is subject requires him
to pay tax on the profit element in his salvage award. If it is right that awards made to salvors, which are already assessed
on generous lines, should be exempted in whole or in part from liability to tax, this seems to me to be a matter for
consideration by the legislature. In my humble opinion, it is not the business of the court to defeat or mitigate the effect of
the fiscal laws of this or any other country, more especially where this can be done only at the expense of the owners of the
salved property.

In my judgment these objections are sufficient to show that, as a general rule at any rate, in a case where both the lost earnings or
profits and the damages are taxable, no account should be taken of taxation in assessing the damages. The present practice of
ignoring the taxation in such a case is sound and should not be disturbed. That is my conclusion, subject to a proviso that there
may be exceptional cases in which a departure from the practice may be required for the doing of justice in special circumstances.
The present case is not exceptional.
On this general question as to the true nature and extent of the principle in Gourleys case, there are two further points. First,
I am assuming, as did the learned master, and I think also counsel arguing this appeal, that the principle of Gourleys case applies
to the assessment of damages for lost earnings or profits in actions for wrongful dismissal as well as in actions for personal
injuries. Support for this assumption is afforded by passages in the speeches of the House of Lords in Gourleys case ([1955] 3
All ER at pp 800, 805, 807, 809, 810, 811, 812; [1956] AC at pp 199, 207, 210, 213, 215, 218), and by Beach v Reed Corrugated
Cases Ltd, Re Houghton Main Colliery Co Ltd, and Phipps v Orthodox Unit Trusts Ltd in the Court of Appeal. Secondly, I am
inclined to think the right evaluation of Gourleys case is to say that it was dealing with a very special case of tax discrepancy or
tax anomaly, and was not intended and should not be construed as generally requiring or authorising the incidence of taxation to
be taken into account in the assessment of damages. The tax discrepancy or tax anomaly is that, whereas the lost earnings or
profits would have been taxable, the damages which are provided in replacement of themas counsel vividly said, to fill the hole
made by the wrongful actare free of tax. The facts of Gourleys case show that, if the notional taxation on the lost profits or
earnings had not been taken into account in assessing the damages, the plaintiff would have received an enormous fortuitous
windfall in addition to compensation.
Now I come to the other ground on which counsel for the defendants has attacked the masters decision that the sum of
320, representing the notional tax on the lost earnings, should not be deducted from the gross sum of 1,200 in assessing the
damages. Counsel says that the damages in this case are not taxable, and in my view he is plainly right on this point. The
reasoning of the master in his judgment, and the arguments of counsel for the plaintiff in this appeal, have been to the effect that
the damages in this case, although by reason of their smallness in amount they will not in fact bear tax, are nevertheless under the
provisions of the Finance Act, 1960, made chargeable to tax, brought within the scope or range of taxation, converted from
capital payments to income payments for tax purposes and rendered taxable subject-matter. In my view, however, this argument
is disposed of by the wording of the Act and by its evident intention. The first part of s 37(1) is in these terms:
Subject to the provisions of this and the next following section, income 681 tax shall be charged under Sch. E in
respect of any payment to which this section applies which is made to the holder or past holder of any office or
employment

Subsection (2) defines the application of the section. The first part of s 38(3) provides:

Tax shall not be charged by virtue of the last foregoing section in respect of a payment of an amount not exceeding
five thousand pounds, and in the case of a payment which exceeds that amount shall be charged only in respect of the
excess

When Parliament has expressly provided that tax shall not be charged in respect of a payment such as this it is in my view
impossible to contend successfully that a payment such as this is chargeable to tax. Section 38(3) does not create a relief from
tax in the ordinary sense. It leaves a small or medium-sized payment of the relevant character outside the scope of the taxation
imposed by the Act; such a payment is not brought into charge, and does not become taxable subject-atter. The evident intention
of s 37 and s 38 is to bring within the scope of income tax and surtax large sums paid to employees on the termination of their
employments, because such sums have hitherto been tax-free. It was necessary to cover payment of large damages for wrongful
dismissal, because the machinery of a wrongful dismissal followed by a claim for damages could easily be used for the purpose
of arranging and effecting such a payment. It was not, however, intended to impose tax on damages for wrongful dismissal in an
ordinary case, where the damages are small or medium-sized. Such damages remain tax-free.
On this ground I would allow the appeal so far as it relates to deduction of the 320 from the 1,200.
There remains the question whether there should be a further deduction of 59 2s 6d in respect of the unemployment benefit
received by the plaintiff in the period of about three months of unemployment following his leaving the defendants employment
in consequence of the wrongful dismissal. The question can be put in this form: Are the sums of unemployment benefit received
by the plaintiff to be brought into account in reduction of the damages for wrongful dismissal, or are they too remote to be
brought into account for this purpose?
I have looked in vain for any precise or decisive test to apply in determining a question such as this. A combination of two
tests was used by the Court of Exchequer in Bradburn v Great Western Ry Co, and by Sellers J at first instance in Payne v
Railway Executive and as the first ground of decision by the Court of Appeal in that case. The two tests are: (i) to consider
whether the accident was the causa causans or only a causa sine qua non of the plaintiffs receipt of the sum in question (ie, the
sum which the defendant claims to have taken into account in reduction of the damages) and (ii) to consider whether the payment
in question results from the plaintiffs own thrift or hard work, so that the defendant, the wrongdoer, should not be allowed to
appropriate it for his own benefit.
On appeal in Payne v Railway Executive Cohen LJ after citing a passage from the judgment of Sellers J said ([1951] 2 All
ER at p 912; [1952] 1 KB at p 36):

I respectfully agree with the judges conclusion and his reasoning. It seems to me that the accident in this case was
not the causa causans of the receipt by the first plaintiff of the disability pension, but the causa sine qua non. The causa
causans was his service in the Royal Navy.

There was a second ground of decision based on the discretionary character of 682 the pension. Singleton LJ ([1951] 2 All ER at
p 914; [1952] 1 KB at p 40) was prepared to adopt the reasoning of Baron Pigott, but preferred to base his judgment on the fact
that the pension was discretionary, so that it could be withheld or reduced. Birkett LJ agreed with both judgments, and thereby
adopted both grounds of decision.
In Browning v War Office a majority of the Court of Appeal (the Master of the Rolls and Diplock LJ with Donovan LJ
dissenting) declined to follow the first ground of decision in Payne v Railway Executive. The Master of the Rolls said it could be
discarded, although the discarding of one of the grounds of a previous decision is difficult to maintain in view of the unanimous
House of Lords decision in Jacobs v London County Council. The Master of the Rolls also said ([1962] 3 All ER at p 1091):

The general principle undoubtedly is that the plaintiff should be compensated, so far as money can do it, for the
pecuniary loss or loss of earnings (or of earning capacity, I care not how it is put) which he has suffered or will suffer by
reason of the injury. He should recover for his loss, but for no more than his loss. If he can earn money elsewhere, he
should do so. The award of damages is made to compensate him, not to punish the wrongdoer. That is now settled by
British Transport Commission v. Gourley. He should, therefore, give credit for all sums which he receives in diminution of
his loss, save in so far as it would not be fair or just to require him to do so. The difficulty is to say when it is or is not fair
and just to take the receipts into account. The cases give some guidance on the point.

Then the Master of the Rolls mentions some of the decided cases and then he says ([1955] 3 All ER at p 808; [1956] AC 185 at p
212):

Apart from such exceptional cases, however, the injured person must, I think, give credit for any sums which he
receives as of right in consequence of his injury.

Diplock LJ declined to follow the first ground of decision in Payne v Railway Executive for the reasons that reference to causa
causans and causa sine qua non was merely productive of confusion and that reference to what the wrongdoer should not be
allowed to appropriate involved a ratio decidendi contrary to the later decision of the House of Lords in Gourleys case, which
raised the same issue of principle, whether the damages for negligence were punitive or compensatory.
I do not think it is possible to say that the first ground of decision in Payne v Railway Executive was overruled by the House
of Lords in Gourleys case, in which no mention was made of Payne v Railway Executive. Nevertheless I think that it can be said
that the decision and reasoning of the House of Lords in Gourleys case involved a new approach to the question of remoteness in
cases such as this, and the new approach leads to a conclusion that the first ground of decision in Payne v Railway Executive is no
longer of compelling authority.
The principal passages dealing with remoteness in Gourleys case are in the speech of Lord Reid. He said ([1955] 3 All ER
at p 808; [1956] AC 185 at p 212):

The general principle on which damages are assessed is not in doubt. A successful plaintiff is entitled to have awarded
to him such a sum as will, so far as possible, make good to him the financial loss which he has suffered, 683and will
probably suffer, as a result of the wrong done to him for which the defendant is responsible . But the general principle
is subject to one qualification. A loss which the plaintiff has suffered, or will suffer, or a compensatory gain which has
come, or will come, to him following on the accident may be of a kind which the law regards as too remote to be taken into
account ([1955] 3 All ER at p 809; [1956] AC at p 214) I do not think that it is possible to formulate any principle by
which it can be determined what is and what is not too remote. MAYNE ON DAMAGES (11th Edn.), p. 151, refers to
Matters completely collateral, and for a general description of what is too remote I cannot find better words, but I do not
think that every case can be solved merely by applying those words to it Another element to be considered is whether
bringing in the matter of liability to tax would seriously increase the duration and expense of trials; for practical as well as
theoretical considerations weigh in determining what is too remote.

In the end one comes back to the question: Is the plaintiffs receipt of unemployment benefit a matter too remote to be taken
into consideration in ascertaining his net loss resulting from the wrongful dismissal? The common-sense answer is that of course
it is not too remote. It is not completely collateral. The dismissal caused the plaintiff to become unemployed, and therefore
entitled, as a matter of general right under the system of state insurance and not by virtue of any private insurance policy of his
own, to receive unemployment benefit. The effect of the dismissal was not to deprive him of all income but to reduce his income
by substituting unemployment benefit for his salary. It would be unrealistic to disregard the unemployment benefit, because to do
so would confer on the plaintiff, to the extent of 59 2s 6d, a fortuitous windfall in addition to compensation. Counsel for the
defendants also put his argument in this way: the plaintiff has a duty to mitigate the damage, resulting from the wrongful
dismissal, by seeking other employment and drawing unemployment benefit in the meantime. I think that is correct, although the
word duty is used in a special sense, meaning only that the plaintiff cannot charge the defendant with any part of the loss which
the plaintiff could have avoided by taking reasonable steps.
I would therefore allow the appeal on both the points involved.

Appeal allowed: damages reduced to 820 17s 6d. Leave to appeal to the House of Lords.

Solicitors: Joynson-Hicks & Co agents for Waltons, Luton, Beds (for the defendants); McKenna & Co (for the plaintiff).

Henry Summerfield Esq Barrister.


684
[1963] 2 All ER 685

Midland Bank Ltd v R V Harris Ltd


BANKING AND FINANCE

QUEENS BENCH DIVISION


MEGAW J
8 MARCH 1963

Bank Cheque Holder in due course Collecting bank Cheques delivered by customer for collection for his own account
Cheques not indorsed Customer by arrangement allowed to draw against uncleared cheques Cheques presented by collecting
bank but dishonoured Collecting bank had allowed customer to draw against the cheques Whether collecting bank holder in
due course Bills of Exchange Act, 1882 (45 & 46 Vict c 61), s 2, s 8(3) Cheques Act, 1957 (5 & 6 Eliz 2 c 36), s 2.

There was an agreement between the plaintiff bank and S that S should be allowed to draw on his current account with the bank
against uncleared cheques that had been credited to his account, including in particular the two cheques next mentioned. These
two cheques were drawn by the defendants, one cheque being payable to L Garage and the other cheque being payable to L
Garages Ltd in each case or order, on Lloyds Bank Ltd and were dated 7 September and 8 September 1961. They were
handed by S, who traded as L Garage, to the plaintiff bank on those dates for collection. The aggregate of the cheques was
3,380. On 14 September 1961, the two cheques were returned to the plaintiff bank marked Payment countermanded by order
of drawer, without having been paid. The plaintiff bank sued the defendant drawers of the cheques, who contended that the
plaintiff bank was not holder of either cheque as they were not indorsed and that s 2 a of the Cheques Act, 1957, did not confer
on the plaintiff bank the status of being a holder of the cheques.
________________________________________
a Section 2 is set out at p 686, letter i, post

Held The plaintiff bank was holder of the cheques, and holder in due course (see p 687, letter i, post), for the following reasons

(i) since the Cheques Act, 1957, the description indorsee in the definition of holder in s 2 of the Bills of Exchange Act,
1882, included the plaintiff bank, on which s 2 of the Act of 1957 conferred the rights that it would have had if S had indorsed the
cheques in blank on delivery to the plaintiff bank (see p 687, letter c, post).
(ii) if, however, (i) above were wrong, the two cheques were, by virtue of s 2 of the Act of 1957, s 8(3) of the Act of 1882,
and the definition of bearer in s 2 of the Act of 1882, payable to bearer in the hands of the plaintiff bank, and thus the bank was
holder of them within s 2 of the Act of 1882 (see p 687, letters d and e, post).

Notes
As to the meaning of holder of a negotiable instrument, see 3 Halsburys Laws(3rd Edn) 146, para 224, text and note (1); as to
rights of banker as holder for value of cheque, see 2 ibid, 178, para 338.
For the Bills of Exchange Act, 1882, s 2, s 8(3), s 29, see 2 Halsburys Statutes (2nd Edn) 507, 511, 520.
For the Cheques Act, 1957, s 2, see 37 Halsburys Statutes (2nd Edn) 53.

Preliminary Issue
The plaintiff bank, Midland Bank Ltd sued the defendants, R V Harris Ltd by action begun by specially indorsed writ issued on
18 October 1961, for principal (3,380) and interest from 13 October to 18 October 1961 (2 6s 3d) in respect of two cheques,
viz, (i) a cheque for 1,640 dated 7 September 1961, and drawn by the defendants on Lloyds Bank Ltd Aston Road,
Birmingham, payable to Lye Garage or order, and (ii) a cheque for 1,740 dated 8 September 1961, drawn by the defendants on
Lloyds Bank Ltd payable to Lye Garages Ltd or order. The plaintiff bank further claimed interest at five per cent from the date
of the writ until judgment. By its statement of claim, as amended, the plaintiff bank also alleged that it became and was holder in
due course of the cheques and on 13 October 1961, duly presented them for payment, but that they were dishonoured, payment
685 thereof having been countermanded by the defendants, of which dishonour the defendants had notice in writing dated 13
October 1961. The plaintiff bank further pleaded, without prejudice to the generality of the foregoing, that alternatively the
plaintiff bank became and was holder for value or the holder of each of the cheques. The plaintiff bank also alleged that at all
material times no company named Lye Garages, Ltd existed, whereby the payee of the second cheque was a fictitious or non-
existent person, and that in the premises the second cheque was deemed to be payable to bearer and the plaintiff bank was at the
date of the writ in possession of it. By further particulars delivered on behalf of the plaintiff bank in December, 1962, the bank
pleaded that it was at all material times banker for a Mr M D Smith, who traded under the style of Lye Garage, and granted him
limited overdraft facilities on his current account; that the two cheques were paid in by him and their amounts were credited by
the plaintiff bank to his current account; and that the plaintiff bank had debited his account in a sum of 6,380 on 14 September
1961, which sum included the two cheques in issue. The defendants by their defence admitted drawing the cheques as alleged,
that the cheques were presented for payment on 13 October 1961, and that they were dishonoured, payment having been
countermanded as alleged, and admitted that the defendants had had written notice of dishonour dated 13 October 1961. They
denied, however, that the plaintiff bank was holder in due course of the cheques at the commencement of the action. By order
dated 21 December 1962, the action was transferred to the commercial list, and it was ordered that in the instance of each of the
two cheques the question whether the plaintiff bank was the holder in due course should be tried as a preliminary issue. His
Lordship (Megaw J), stated with regard to the evidence before him that it was not necessary to review the evidence relating to the
conduct of Mr Smiths account with the plaintiff bank because it was conceded (or, if not conceded, it was the clear effect of the
evidence which on that aspect was unchallenged) that there was an agreement between the plaintiff bank and Mr Smith that the
latter should be allowed to draw against uncleared cheques which had been credited to his account, including specifically the two
cheques with which the action was concerned.

Norman Tapp for the plaintiff bank.


Maurice Megrah for the defendants.

8 March 1963. The following judgment was delivered.

MEGAW J stated the issue, reviewed the evidence, intimated that in view of his conclusion on a question of fact it was
unnecessary for him to consider one of the points of law raised by counsel for the plaintiff bank, and continued: In the end two
points, and two points only, were raised by counsel on behalf of the defendants. The first point was that the plaintiff bank was
not holder of either cheque. In support of this argument counsel founded himself on the definition of holder in s 2 of the Bills
of Exchange Act, 1882. That definition is: Holder means the payee or indorsee of a bill or note who is in possession of it, or
the bearer thereof. The bank was not the payee of the cheques. There was no indorsement of cheques. Hence the bank was not
indorsee. The cheques were not payable to bearer, nor indorsed in blank. Hence the bank could not be the bearer thereof. The
bank had not required Mr Smith to indorse the cheques, no doubt because of the provisions of the Cheques Act, 1957. So far
counsels argument appears to be irresistible. However, he then runs into trouble in the shape of s 2 of the Cheques Act, 1957.
That section reads:

A banker who gives value for, or has a lien on, a cheque payable to order which the holder delivers to him for
collection without indorsing it, has such (if any) rights as he would have had if, upon delivery, the holder had indorsed it in
blank.

On the face of it the purpose and the meaning of the section is clear. A bank is not to be deprived of any rights which it would
otherwise have as the holder of a cheque merely because the cheque is not indorsed. Counsel for the defendants 686 says that
that applies to rights and not to status. In other words, it does not make a bank a holder, but merely gives it the rights of a holder.
Therefore, says counsel, s 29 of the Act of 1882 does not make the bank a holder in due course, because it applies only to one
who is a holder and does not apply to one who is not a holder, even though by statute he may have the rights of a holder.
Counsel for the defendants frankly accepts that his argument deprives s 2 of the Act of 1957 of all meaning and effect, but,
he says, that that should not impair the validity of his argument. In my judgment, counsels argument, although ingenious, is
unsound. Section 2 of the Act of 1957 says expressly that the banker is to have such rights as he would have had if the cheque
had been indorsed in blank. Indorsee is not itself defined by the Act of 1882, though indorsement is defined b. I take the
view that, in the definition of holder in s 2 of that Act, indorsee means a person who has the rights which are given by statute
in respect of a bill or cheque by virtue of an indorsement. Since the Cheques Act, 1957, the word is not limited to one who has
received a cheque with a signature actually written on the back. By reason of s 2 of the Act of 1957 indorsee includes one who
has the rights which would have been given to that person by virtue of an indorsement having been written on the back. If that
were wrong, I should still hold that the plaintiff bank was holder of the cheques within the definition in s 2 of the Act of 1882.
That definition includes not only an indorsee but also a bearer. Bearer is defined by s 2 of the Act of 1882 as the person in
possession of a bill or note which is payable to bearer. By s 8(3) of the Act of 1882 a bill is payable to bearer if the only or last
indorsement on it is an indorsement in blank. Section 2 of the Act of 1957 gives to the banker the same rights as he would have
had if, on delivery, the holder, Mr Smith, had indorsed the cheque in blank.
________________________________________
b By s 2 of the Act of 1882 indorsement means an indorsement completed by delivery

It follows that by express enactment as applicable to the circumstances of this case the two cheques here are in the hands of
the plaintiff bank payable to bearer. There is here no question which can be described as a question of status rather than of rights.
It is solely a matter of rights. Hence the bank is a bearer for the purposes of the definition of holder in s 2 of the Act of 1882.
Hence the bank is a holder of both cheques.
The other point which was, at least for a time, raised by counsel for the defendants was that the bank had lost or abandoned
any rights which it might have had as holder in due course by handing the cheques over to Mr Smith after they were returned
dishonoured and after the bank had debited Mr Smiths account with their value. Counsel conceded that s 62 of the Act of 1882
could not be invoked in his favour. He also conceded that he could not at this stage succeed as he accepted, being on the
defendants) that the bank had parted with the cheques to Mr Smith.
Mr Robertsc said in evidence that he had no specific recollection what was done with these cheques after their dishonour,
but that the usual practice of the bank would have been to send them to Mr Smith with a formal printed slip. However, Mr
Roberts was not in charge of the branch on 14 September 1961, when the cheques were received back dishonoured. Mr Brooks,
the manager, had returned from holiday then. Mr Brooks was asked no questions about this matter. Having regard to the fact that
the cheques were produced from the possession of the bank, I am unable to hold that the defendants have laid the foundation for
any argument on this point, apart from any other difficulties, and I think that they might have been formidable, in the way of the
defendants in establishing the point.
________________________________________
c An officer of the bank who, when temporarily in charge of the branch of the bank at which Mr Smiths account was kept, orally arranged for
him to draw against uncleared effects

Accordingly I hold on the preliminary issue that the bank is and was at all material times holder in due course of both the
cheques sued on.
687
[Counsel for the defendants having conceded that in view of the decision on the preliminary issue judgment should be
entered for the plaintiff bank in the action, His Lordship gave judgment for the bank accordingly.]

Judgment for the plaintiff bank for 3,382 6s 3d with interest.

Solicitors: Beckingsales & Brashiers agents for Wm Bache & Sons, West Bromwich (for the plaintiff bank); Kingsford, Dorman
& Co agents for Glaisyer, Porter & Mason, Birmingham (for the defendants).

K Diana Phillips Barrister.


[1963] 2 All ER 688

R v Gill
CRIMINAL; Criminal Law

COURT OF CRIMINAL APPEAL


LORD PARKER CJ, HAVERS AND EDMUND DAVIES JJ
29 APRIL 13 MAY 1963

Criminal Law Duress Defence Burden of proof.

Where an accused person desires to rely on the defence of duress, he must, either, by the cross-examination of the prosecution
witnesses or by evidence called on his behalf, or by a combination of the two, place before the court such material as makes
duress a live issue fit and proper to be left to the jury; but, once he has succeeded in doing that, it is for the Crown to destroy that
defence in such a manner as to leave in the jurys minds no reasonable doubt that the accused cannot be absolved on the grounds
of the alleged compulsion (see p 691, letter d, post).
Dictum of Lord Goddard CJ in R v Steane ([1947] 1 All ER at p 817) explained.
Observations on the practice of including in the indictment in cases of larceny the name of the owner of the property (see p
690, letter c, post).
Appeal dismissed.

Notes
As to coercion or duress as a defence to criminal responsibility, see 10 Halsburys Laws (3rd Edn) 290, 291, para 539; and for
cases on the subject, see 14 Digest (Repl) 7476, 346361.

Cases referred to in judgment


Hibbert v McKiernan [1948] 1 All ER 860, [1948] 2 KB 142, 117 LJR 1521, 205 LT Jo 278, 112 JP 287, 15 Digest (Repl) 1086,
10,752.
MGrowthers Case (1746), Fost 13, 18 State Tr 391, 168 ER 8, 14 Digest (Repl) 74, 350.
R v Carr-Briant [1943] 2 All ER 156, [1943] KB 607, 112 LJKB 581, 169 LT 175, 107 JP 167, 29 Cr App Rep 76, 14 Digest
(Repl) 495, 4791.
R v Dossi (1918), 87 LJKB 1024, 13 Cr App Rep 158, 14 Digest (Repl) 246, 2114.
R v Purdy (1946), 10 JCL 182.
R v Shiartos, 29 September 1961, unreported (Central Criminal Court).
R v Steane [1947] 1 All ER 813, [1947] KB 997, 116 LJR 969, 177 LT 122, 111 JP 337, 32 Cr App Rep 6, 14 Digest (Repl) 33,
47.

Appeal
This was an appeal by Samuel James Gill against his conviction at Bedford County Sessions on 3 January 1963, before the
deputy-chairman and a jury (i) of common law conspiracy and (ii) of larceny pursuant to that conspiracy. He was sentenced to a
total of two years imprisonment. The facts are set out in the judgment of the court.
The authority and case noted belowa were cited during the argument in addition to the cases referred to in the judgment.
688
________________________________________
a Archbolds Criminal Pleading, Evidence and Practice (35th Edn), para 45, R v Bourne, (1952), 36 Cr App Rep 125

J F Kingham for the appellant.


H W Sabin for the Crown.

Cur adv vult

13 May 1963. The following judgment was delivered.

EDMUND DAVIES J at the request of Lord Parker CJ read the following judgment of the court. The appellant was charged at
Bedford County Sessions and convicted of (i) conspiring with James Lockett and other persons unknown to steal a lorry and its
load, and (ii) with larceny pursuant to that conspiracy. Against those convictions he now appeals by leave of the full court. The
appellant, who was employed as a lorry-driver by A E Meeks Ltd himself testified that he was approached by a group of men (of
whom his co-accused Lockett was not one) who suggested that he should steal a valuable load from his employers and hand it
over to them in return for a payment of 1,000, and that he agreed to do this. It was arranged that he would leave the loaded lorry
in a car-park in Bristol, that during his absence it would be driven away, and that he would then falsely report to the police that it
had been stolen without his knowledge. On this evidence, the learned deputy-chairman rightly told the jury that the conspiracy
charge was clearly established on the appellants own testimony, and (subject to two matters later to be mentioned) no question
now arises as to the correctness of that direction or of the conviction on that count. According to the appellant, however,
although the lorry and its load were in fact later stolen by his fellow-conspirators, this was done not pursuant to the conspiracy to
which he had been a party, but wholly against his will. He testified that he repented of the conspiracy the day after he entered
into it, and that, when Reg (one of his fellow-conspirators) and three others arrived at his home by arrangement to collect him
for the purpose of the theft being effected, he told them that he was not going through with it. They thereupon threatened
physical violence both to him and to his wife, one of them flourishing a crowbar and another showing him a bottle of petrol, and,
in great fear for the safety of his wife and himself, he obeyed their orders to accompany them to his employers premises. They
dropped him outside, he went into the yard and then, still in fear, collected his lorry and drove it to a point on the M1 near St
Albans, where he was forced by threats to leave the lorry. It was then promptly driven away and has never since been recovered.
When later seen by the police, however, the appellant signed two statements in which he confessed that he had been a party to the
larceny and made no mention of having been subjected to duress.
Three points are now raised in this appeal against his convictions, and we proceed to deal with them in turn. First, it was
urged that the conviction on the conspiracy court is bad in that, although the appellant was charged with conspiring with James
Lockett and with other persons unknown, Lockett was acquitted by the jury. There is nothing in this point, for there was ample
evidence provided by the appellant himself that, despite his absolving of Lockett and the acquittal of that accused, he had in fact
conspired with other persons unknown, as charged, and of that the jury were fully entitled to convict him. Indeed, on the
evidence any other verdict would have been perverse.
The second point relied on by the appellant was that, whereas he was employed by A E Meeks Ltd in the larceny count the
property involved was, by a slip, described as belonging to A E Hicks, Ltd, and that, although the appellant himself raised this
matter after the jury had returned their verdict, no amendment of the indictment was ordered. Clearly this should have been done,
as it could have been even after verdict (R v Dossi. But, as no amendment was in fact made, it is now submitted that the
misdescription of the owner in both counts renders the conviction on each invalid. This court rejects that submission. By r 6(1)
of Sch 1 to the Indictments Act, 1915, it is provided that:
689

The description of property in a count in an indictment shall be in ordinary language and such as to indicate with
reasonable clearness the property referred to, and if the property is so described it shall not be necessary (except when
required for the purpose of describing an offence depending on any special ownership of property or special value of
property) to name the person to whom the property belongs or the value of the property.

There was in both counts a sufficient description of the property referred to for it to be identified with certainty, and the addition
of the proprietors name was, having regard to the nature of the charges, in each case mere surplusage. As Lord Goddard CJ said
in Hibbert v McKiernan ([1948] 1 All ER 860 at p 862; [1948] 2 KB 142 at p 151),

at the present day allegations concerning the ownership of stolen property are, except in a few exceptional cases,
treated as immaterial.

Having said that, however, this court must certainly not be taken as discouraging the present useful and most desirable (and
sometimes essential) practice of including in the indictment the owners name, where known, or as approving of the failure to
amend in the present case, where the obvious errors were actually pointed out by the accused man himself.
The third and most interesting point taken relates only to the larceny count, it being submitted that the learned deputy-
chairman wrongly directed the jury that it was for the appellant to establish that he was acting under duress. The account given
by the appellant himself makes it very doubtful whether such a defence was strictly open to him, inasmuch as there was a time
after the alleged threats when, having been left outside his employers yard and having then entered it, he could presumably have
raised the alarm and so wrecked the whole criminal enterprise. In MGrowthers Case, Lee LCJ, directed the jury that, to
establish a plea of duress, the defendant must have resisted or fled from the wrongdoer if that were possible. Seemingly, the
position under American law is the same, as appears from the statement in Professor Rollin Perkins Criminal Law that,

The excuse (of compulsion) is not available to someone who had an obviously safe avenue of escape before
committing the prohibited act.

The issue of duress was, nevertheless, left to the jury in the present case, and that may well have been the prudent course.
Having been left, did the burden rest on the Crown conclusively to destroy this defence, in the same way as it is required to
destroy such other defences as provocation or self-defence? Or was the appellant required to establish it, on the balance of
probabilities? For the latter view, reliance was placed on the judgment of Lord Goddard CJ in R v Steane ([1947] 1 All ER 813 at
p 817; [1947] KB 997 at p 1005), in the course of which he said:

before any question of duress arises, a jury must be satisfied that the prisoner had the intention which is laid in the
indictment. Duress is a matter of defence and the onus of proving it is on the accused. As we have already said, where an
intent is charged on the indictment, it is for the prosecution to prove it, so the onus is the other way.

On the other hand, in R v Purdy ((1946), 10 JCL 182 at p 186), where a British prisoner of war was charged with treason, Oliver J
directing the jury on the defence of duress, said:

If you believe, or if you think that it might be true, that he only did that because he had the fear of death upon him,
then you will acquit him on that charge, because to act in matters of this sort under threat of death is excusable.
690

Similarly, in R v Shiartosb, where duress was relied on by an accused charged with arson, Lawton J directed the jury that:
________________________________________
b 19 September 1961, unreported, Central Criminal Court

If, in all the circumstances of this case, you are satisfied that what he did he did at pistol point and in fear of his life,
he is entitled to be acquitted. If, although you are not satisfied, you think it might well be that he was forced at pistol point
to do what he had to do, then again you should acquit him, because the prosecution would not have made you feel sure that
what he did he did maliciously.

In our judgment, the law on this matter is to be found correctly stated in Dr Glanville Williams Criminal Law (2nd Edn), p
762, para 247, in this way:

although it is convenient to call duress a defence, this does not mean that the ultimate (persuasive) burden of
proving it is on the accused . But the accused must raise the defence by sufficient evidence to go to the jury; in other
words, the evidential burden is on him.
The Crown are not called on to anticipate such a defence and destroy it in advance. The accused, either by the cross-examination
of the prosecution witnesses or by evidence called on his behalf, or by a combination of the two, must place before the court such
material as makes duress a live issue fit and proper to be left to the jury. But, once he has succeeded in doing this, it is then for
the Crown to destroy that defence in such a manner as to leave in the jurys minds no reasonable doubt that the accused cannot be
absolved on the grounds of the alleged compulsion. It is true that this approach appears to conflict with the literal reading of the
passage from Lord Goddard CJs judgment in R v Steane ([1947] 1 All ER at p 817; [1947] KB at p 1005). It is to be observed,
however, that that passage was obiter, in that the real decision there was that it was for the Crown to prove the specific intent laid
and that, in the particular circumstances of that case, an inference could not be drawn that the appellant intended the natural
consequences of his act. We agree with Dr Glanville Williams c that the dictum must be read as relating only to what the author
calls the evidential burden cast on the accused, and not to the ultimate (or persuasive) burden placed on the Crown of
destroying the defence of duress where it has been substantially raised.
________________________________________
c Criminal Law (2nd Edn), p 762, para 247

We now turn to consider the summing-up in the present case. Two passages, in particular, are said wrongly to have placed
the burden of establishing the alleged duress on the shoulders of the accused. In one of these, the deputy-chairman said:

He [the appellant] has got to satisfy you that the threat was such, and he was in such fear, that he really had lost all
will of his own, so that he was not acting of his own volition, of his own will.

Later he continued: It is for [the appellant] to prove to you the facts on which you could find in his favour on the basis that he
was acting under duress. Taking these two passages in isolation, it has been submitted for the appellant (a) that the deputy-
chairman was there wrongly placing the ultimate (or persuasive) burden of proof on the appellant, and (b) that, assuming that a
burden of any kind rested on the appellant, the deputy-chairman erred in failing to indicate that such burden was of the less
onerous kind indicated in such cases as R v Carr-Briant.
The court has anxiously considered these submissions. In the light of them, had the two passages complained of stood
alone, we should have felt compelled 691 to quash the conviction on the larceny count, for the reasons already indicated in this
judgment. But acceptance of the submissions would involve taking these two passages out of their context and failing to consider
the summing-up as a whole. That at the outset the deputy-chairman correctly directed the jury as to the general burden of proof is
not challenged, and that direction was repeated several times. Furthermore, each of the two passages complained of was
followed immediately by a direction which must have made clear to the jury that the ultimate (or, to use Dr Glanville Williams
word, persuasive) burden was throughout on the Crown. Thus, the second passage was immediately succeeded by the
following direction:

You have got to look at the whole of the evidence. If, on the whole of the evidence, you are left in a real doubt about
this [that is, whether the [appellant] acted freely or under compulsion] you would find for the [appellant] and say Not
Guilty. If, on the whole of the evidence, you are satisfied that he did intend to do that act and his mind went with it, then
he would be guilty.

And almost the last words the jury heard from the deputy-chairman before they retired were these:

Those are the sort of considerations you must weigh up and say whether you are satisfied that the prosecution has
proved its case so that you feel certain about it, that he was acting of his own free will when he was taking that lorry and
handed it over to those men. If you do, you would find him guilty on the second count. If you are not so satisfied, then
you would find him not guilty.

Such a direction was impeccable and could have left the jury in no doubt as to the manner in which they were to consider their
verdict on the larceny count. Taking the summing-up as a whole, the conclusion this court has come to is that the verdict ought
not to be disturbed. This appeal against the conviction on both counts must, accordingly, be dismissed.

Appeal dismissed. Leave to appeal to the House of Lords refused.

Solicitors: Registrar, Court of Criminal Appeal (for the appellant); Giffen, Couch & Archer, Luton (for the Crown).

N P Metcalfe Esq Barrister.


692
[1963] 2 All ER 693

Punton and Another v Ministry of Pensions and National Insurance (No 2)


SOCIAL SECURITY

QUEENS BENCH DIVISION


PHILLIMORE J
1, 2, 3, 13 MAY 1963

National Insurance Benefit Determination of claims and questions Finality of decision of National Insurance Commissioner
Jurisdiction of High Court on originating summons to determine whether commissioners decision wrong in law Exercise of
courts discretion Unemployment benefit National Insurance Act, 1946 (9 & 10 Geo 6 c 67), s 43(1) RSC, Ord 54A r 1A, r 4
National Insurance (Determination of Claims and Questions) Regulations, 1948 (SI 1948 No 1144), eg, reg 10, reg 11, reg 15
Tribunals and Inquiries Act, 1958 (6 & 7 Eliz 2 c 66), s 11(1).

National Insurance Benefit Disqualification Exception Not directly interested Not belonging to grade or class of
workers any of whom directly interested National Insurance Act, 1946 (9 & 10 Geo 6 c 67), s 13(1), proviso (a), proviso (b).

As a result of stoppages of work by two categories of skilled workers, platers and shipwrights, who were involved in a
demarcation dispute in the shipyard where the plaintiffs were employed as platers helpers, a recognised grade of semi-skilled
workers, the plaintiffs, together with other platers helpers assisting the platers, lost their employment, the plaintiff Punton for
five weeks commencing in March, 1961, and the plaintiff Croxford for eleven weeks commencing in June, 1961. The plaintiffs
appeals to the National Insurance Commissioner against the disallowance of their claims to unemployment benefit were
dismissed on the ground that the plaintiffs were disqualified for receiving unemployment benefit by s 13(1) a of the National
Insurance Act, 1946; the commissioner decided that proviso (a) and proviso (b) to s 13(1) did not except the plaintiffs from
disqualification, because they had failed to prove that none of the platers helpers was directly interested in the dispute, inasmuch
as on the evidence it was quite certain that the amount of work available for platers helpers must depend on the outcome of this
dispute. By originating summons, amended pursuant to a decision b of the Court of Appeal (reported [1963] 1 All ER 275), the
plaintiffs applied for the determination of the question whether, on the facts found by the commissioner, he had come to the
correct decision in law.
________________________________________
a Section 13(1) of the National Insurance Act, 1946, is set out at p 696, letters c and d, post; s 43 is summarised at p 697, letter i, post
b See [1963] 1 All ER at pp 278, 279. The plaintiffs had appealed against an order striking out their summons as frivolous and vexatious,
and, on their being given leave to amend the summons so as to make it clear that they were no longer challenging the findings of fact made
by the commissioner, their appeal was allowed cf, p 696, letter h, post

Held (i) the relevant statutory provisions (viz, National Insurance Act, 1946, s 13(1) and s 43 c, Tribunals and Inquiries Act,
1958, s 1, s 9, s 11(1) and Sch 1, para 11 d and the material regulationse under the Act of 1946) did not clearly exclude the
supervisory jurisdiction of the court to make declarations such as that sought on this originating summons, and, accordingly, the
court had jurisdiction to grant the relief claimed (see p 701,
________________________________________
c Section 13(1) of the National Insurance Act, 1946, is set out at p 696, letters c and d, post; s 43 is summarised at p 697, letter i, post
d As to the Act of 1958, see p 699, letter b, post
e Principally the relevant provisions of National Insurance (Determination of Claims and Questions) Regulations, 1948 (SI 1948 No 1144);
see p 698, letter e, post

Observations of Lord Denning MR and Upjohn LJ in Punton v Ministry of Pensions and National Insurance ([1963] 1 All
ER at pp 278, 279) and of Lord Goddard CJ in Pyx Granite Co Ltd v Ministry of Housing and Local Government ([1959] 3 All
ER at p 8) applied.
693
Barnard v National Dock Labour Board ([1953] 1 All ER 1113) and Taylor v National Assistance Board ([1956] 2 All ER
455) considered.
(ii) the declaration sought would, however, be refused because
(a) the decision of the National Insurance Commissioner had not been shown to be wrong in law, for, when a plater required
help, he and his helper worked in effect as a team, so that the platers helper had a direct interest in the volume and type of work
available to the plater (see p 705, letter a, post).
(b) even if the court had come to the conclusion that the commissioner was wrong, it was doubtful whether the discretion
conferred by RSC, Ord 54A, r 4, to make the declaration sought should be exercised in a case such as this, for the question was
one of mixed law and fact on which judges might differ and this jurisdiction did not fit conveniently into a scheme designed to
give speedy decisions on claims for unemployment benefit (see p 705, letters c and h, post).
Dictum of Viscount Simonds in Pyx Granite Co Ltd v Ministry of Housing and Local Government ([1959] 3 All ER at p 7)
considered.
Per Phillimore J: I am not impressed by the argument that even if this court declared the law, no effect could be given to that
declaration, for it would be the duty of the minister to take appropriate steps, if necessary by seeking amendment of the
regulations, to give effect to it (see p 701, letter h, post).

Notes
As to declaratory judgments and the discretionary nature of the remedy, see 22 Halsburys Laws (3rd Edn) 746751, para 1610,
para 1611; and for cases on the subject, see 30 Digest (Repl) 182, 293295.
As to the occasions when procedure by originating summons is applicable, see 30 Halsburys Laws (3rd Edn) 303305, para
559.
As to the finality of the National Insurance Commissioners decision and as to the availability of certiorari to quash, see 27
Halsburys Laws (3rd Edn) 682, para 1233.
As to disqualifications affecting persons who have lost employment because of a stoppage of work due to a trade dispute at
their place of employment, see 27 Halsburys Laws (3rd Edn) 731, para 1326.
For the National Insurance Act, 1946, s 13(1), s 43(1)(3), see 16 Halsburys Statutes (2nd Edn) 689, 690, 724.
For the National Insurance (Claims and Payments) Regulations, 1948, regs 1, 2, 5, 9, and the National Insurance
(Determination of Claims and Questions) Regulations, 1948, regs 10, 11, 15, 16, 18, 19, see 15 Halsburys Statutory Instruments
(1st Re-issue) 191193, 196, 218, 220, 223.
For the Tribunals and Inquiries Act, 1958, s 1, s 9, s 11(1), Sch 1, para 11, see 38 Halsburys Statutes (2nd Edn) 200, 201,
207, 211, 215, 217 (notes).

Cases referred to in judgment


Andrews v Mitchell [19047] All ER Rep 599, [1905] AC 78, 74 LJKB 333, 91 LT 537, 25 Digest (Repl) 347, 287.
Barnard v National Dock Labour Board [1953] 1 All ER 1113, [1953] 2 QB 18, [1953] 2 WLR 995, 3rd Digest Supp.
Cooper v Wilson [1937] 2 All ER 726, [1937] 2 KB 309, 106 LJKB 728, 157 LT 290, 101 JP 349, 30 Digest (Repl) 171, 219.
Francis v Yiewsley and West Drayton UDC [1957] 3 All ER 529, [1958] 1 QB 478, 122 JP 31, 3rd Digest Supp.
Hanson v Radcliffe UDC [1922] All ER Rep 160, [1922] 2 Ch 490, 91 LJCh 829, 127 LT 509, 86 JP 144, 30 Digest (Repl) 169,
203.
Lee v Showmens Guild of Great Britain [1952] 1 All ER 1175, [1952] 2 QB 329, 3rd Digest Supp.
Pyx Granite Co Ltd v Ministry of Housing and Local Government [1959] 3 All ER 1, [1960] AC 260, 123 JP 429, [1959] 3 WLR
346, 3rd Digest Supp.
694
Taylor v National Assistance Board [1956] 2 All ER 455, [1956] P 470, [1956] 3 WLR 290, revsd CA, [1957] 1 All ER 183,
[1957] P 101, [1957] 2 WLR 189, affd HL, [1957] 3 All ER 703, [1958] AC 532, [1958] 2 WLR 11, 3rd Digest Supp.
Vine v National Dock Labour Board [1956] 3 All ER 939, [1957] AC 488, [1957] 2 WLR 106, 3rd Digest Supp.

Summons
The plaintiffs, whose claims for unemployment benefit had been refused by the insurance officer, by the local tribunal in
Birkenhead, and finally on 20 September 1961, by the National Insurance Commissioner to whom they had appealed, applied by
originating summons under RSC, Ord 54A, r 1A, for the determination of the question whether, on the true construction of s
13(1) of the National Insurance Act, 1946, and on the facts found by him, the commissioner came to correct decisions in point of
law in deciding certain questions. These questions (A) were, in the case of the plaintiff Eric Edward Punton, (i) whether on the
construction of proviso (a) to s 13(1) he was a person directly interested in a trade dispute which caused a stoppage of work
between 9 March 1961, and 15 April 1961, referred to in the commissioners decision CU.22/61 and (ii) whether on the
construction of proviso (b) to s 13(1) he was a person belonging to a grade or class of workers of which, immediately before the
commencement of the said stoppage, there were members employed at his place of employment, any of whom were directly
interested in the said trade dispute; and (B) were, in the case of the plaintiff James Charles Croxford, similar questions in respect
of a stoppage of work between 10 June 1961, and 28 August 1961, referred to in the commissioners decision CU.22/61. The
plaintiffs sought also declarations that, on the true construction of proviso (a) and proviso (b) to s 13(1), and on the facts found by
the commissioner, each of them was entitled to unemployment benefit. The facts appear in the judgment.
The cases noted belowf were cited during the argument in addition to those referred to in the judgment.
________________________________________
f Smith v Hancock [1894] 2 Ch 377, Barraclough v Brown [18959] All ER Rep 239, [1897] AC 615, Gophir Diamond Co v Wood [1902] 1
Ch 950, R v Northumberland Compensation Tribunal, Ex p Shaw [1952] 1 All ER 122, [1952] 1 KB 338, Healey v Minister of Health
[1954] 3 All ER 449, [1955] 1 QB 221, Bagettes Ltd v GP Estates Co Ltd [1955] 3 All ER 451, [1956] Ch 290

Gerald Gardiner QC and F B Purchas for the plaintiffs.


J R Cumming-Bruce and Miss J A H G Cochrane for the defendants.

13 May 1963. The following judgment was delivered.

PHILLIMORE J read the following judgment. This originating summons has been described by counsel on both sides as a test
action. The plaintiffs seek a declaration that they are entitled to unemployment benefit, in the case of Punton in respect of a
period of about five weeks in the spring of 1961 and in the case of Croxford in respect of a period of eleven weeks in the summer
of 1961.
The plaintiffs were, at the times in question, employed as platers helpers in the shipyard at Birkenhead. They were semi-
skilled workers and members of the National Union of General and Municipal Workers. Their claims had their origin in the
demarcation dispute between the platers, as members of the Boilermakers Union, and the shipwrights, as members of the
Shipwrights Union. These two categories of skilled tradesmen each claimed the right to do certain work. On 2 February 1961,
the dispute having been submitted to arbitration, an award was made giving some of the disputed work to each category. The
dispute, however, continued, and survived an explanatory award, and on or about 7 March 1961, certain platers refused to carry
out work in the shop side by side with the shipwrights and they were dismissed. Other platers were brought in in their place, with
the same result, and a stoppage of five weeks followed during which the plaintiff Punton, with other platers helpers, was thrown
out of work. In June the dispute flared up again, but this time it was the shipwrights who 695 refused to handle work which had
been done in the shop by the platers, with the help of the platers helpers. The shipwrights withdrew their labour, and a stoppage
followed, during which the plaintiff Croxford and other platers helpers lost their employment.
The insurance officer, whose duty it was under the provisions of the National Insurance Act, 1946, to determine their claims
to unemployment benefit, disallowed the claims of the plaintiffs and of the other platers helpers involved in the stoppages, and
the Birkenhead Tribunal, to which they appealed, upheld this decision. The plaintiffs then appealed to the National Insurance
Commissioner, Mr R G Micklethwait QC who heard evidence and gave his decision on 20 September 1961. The question which
he had to decide turned on the provisions of s 13(1) of the Act of 1946, which reads as follows:

A person who has lost employment in an employed contributors employment by reason of a stoppage of work which
was due to a trade dispute at his place of employment shall be disqualified for receiving unemployment benefit so long as
the stoppage of work continues, except in a case where, during the stoppage of work, he has become bona fide employed
elsewhere in the occupation which he usually follows or has become regularly engaged in some other occupation:
Provided that this subsection shall not apply in the case of a person who proves(a) that he is not participating in or
financing or directly interested in the trade dispute which caused the stoppage of work; and (b) that he does not belong to a
grade or class of workers of which, immediately before the commencement of the stoppage, there were members employed
at his place of employment any of whom are participating in or financing or directly interested in the dispute.

The commissioner dismissed the appeals, holding in each case that the plaintiff, or his association, had proved that none of the
platers helpers had participated in or financed the trade dispute which caused the stoppage but had failed to prove that they had
no direct interest in that dispute.
The originating summons which I had before me was filed on 28 March 1962, which is over six months later. In its original
form it made no reference to the findings of the commissioner, but asked the court to determine in the case of each plaintiff
whether he was a person directly interested in the trade dispute which caused the stoppage of work whereby he lost employment,
and prayed for the declaration which I have indicated. Counsel who appeared both for the Minister of Pensions and National
Insurance and the insurance officer took out a summons to strike out the originating summons on the grounds that it was frivolous
and vexatious, and succeeded before the master and the judge in chambers. The matter went to the Court of Appeal, where
counsel for the plaintiffs and their association conceded that he could not ask for a retrial of the facts, and the summons was
amended by substituting for the question previously included, the question: Whether, on the facts as found by the
commissioner, the commissioner came to the correct decision in point of law. This amendment having been made, it was clear
that the summons was no longer frivolous or vexatious and the appeal was allowed, the plaintiffs being ordered to pay the costs.
I heard the amended summons on 1 May 2 and 3, when I had the benefit of extremely able arguments on both sides.
Learned counsel made it quite clear that whatever decision I gave would be tested in the Court of Appeal and possibly in the
House of Lords. Accordingly, I thought it right to reserve my judgment. The matter being somewhat complex, counsel for the
minister allowed me to hear the argument of counsel for the plaintiffs on the merits before developing his preliminary objection
that this court has no jurisdiction to grant the relief claimed. He also dealt with the merits, so all the matters that the parties
desire to raise have been fully ventilated.
696
The issues for my decision are as follows: (i) Whether this court has jurisdiction to grant the relief claimed by the summons.
(ii) Whether the Court of Appeal decided on 28 November 1962, on the interlocutory appeal, that the court has such jurisdiction,
so that I am bound by their finding. (iii) Whether the commissioner came to a correct decision in point of law. (iv) Whether,
assuming that the court has jurisdiction and that the commissioners decision was wrong, the court ought, in the exercise of its
discretion, to grant the relief claimed.
I have come to the conclusion that the court has jurisdiction to grant the relief claimed, and in any event I think that I am
bound by the observations in the Court of Appeal to that effect. In my judgment, however, the decision of the commissioner was
correct in law. Accordingly, the last question does not strictly arise, but, since it has been fully argued, I ought perhaps to deal
with the argument and express a view. On the facts of this case, even if I had come to the conclusion that the commissioner was
wrong, I would venture to doubt whether this court, in the exercise of its discretion, ought to grant the declaration sought by the
plaintiffs.
In stating my reasons for so holding, I shall deal first with the question of jurisdiction. The argument presented by counsel
on behalf of the minister falls into three parts, (A), (B) and (C) as follows: (A) Reference to the provisions of the National
Insurance Act, 1946, and of the Tribunals and Inquiries Act, 1958, shows a clear parliamentary intention that the High Court
should have no jurisdiction to examine the decisions of the National Insurance Commissioner save by certiorari. (B) If the court
were to grant the declaration sought by the plaintiffs, the minister would be faced with two conflicting decisions. The decision of
the commissioner not having been quashed, there is no machinery by which the declaration of this court could be implemented by
an award and payment of benefit. This proceeding is in those circumstances virtually pointless. (C) The decisions of this court in
which it has asserted its supervisory jurisdiction over inferior tribunals by the grant of a declaration do not extend to a case of this
sort, where the commissioner fulfils all the tests of a judicial body and has acted within his jurisdiction. In developing these
arguments, counsel for the minister first examined the provisions of the Act of 1946 and the regulations made under it. Section
11 of the Act of 1946 establishes the right to benefit, subject to certain conditions, and in particular those relating to contributions.
Section 13, to which I have already referred, sets out certain conditions which will disentitle the claimant to benefit. Section 28
makes provision for claims to benefitin particular, they are to be made in the prescribed manner. The prescribed manner is to
be found in the National Insurance (Claims and Payments) Regulations, SI 1948 No 1041. Regulation 2 provides that claims
must be made in writing to the minister. Regulation 5 stipulates the information which the claimant must give. Regulation 9
provides for payment following an award by the determining authority which, as appears from the definition in reg 1, may
comprise an insurance officer, a local tribunal, or the National Insurance Commissioner. Thus, counsel emphasises that an award
by the determining authority is a necessary precursor to payment of benefit.
Returning to the National Insurance Act, 1946, s 43(1) provides for regulations to govern the determination of questions by
the minister or by a person or by a tribunal appointed or constituted in accordance with the regulations. Subsection (3) provides
in regard to the regulations that questions as to the right to benefit shall not be determined by the minister but in the first instance
by an officer appointed by him. This officer is to be given the right to determine the question himself or to refer it to a local
tribunal. There is also to be an appeal from the officer to the tribunal. Provision is to be made for appeals from the tribunal to the
National Insurance Commissioner, or a deputy commissioner, or a tribunal 697 presided over by such a commissioner. A proviso
to sub-s (3) excludes certain questions, notably, those concerned with contributions, from determination by the insurance officer,
the tribunal, or the commissioner. These, I was told, are called the ministers questions. In regard to these questions, provision is
made by sub-s (4) for regulations to enable an appeal from the minister to the High Court on a point of law, such appeal to be
final. This latter appeal is governed by RSC, Ord 55B, and is subject to a time limit of twenty-one days, there being no further
appeal to the Court of Appeal.
Counsel for the minister argues that Parliament has enacted a complete scheme for the disposal of disputed questions and
has provided both for the ministers questions and for non-minister questions. The former are specifically subject to a limited
right of appeal in the High Court, while the latter are left to the insurance officer, the local tribunal and, on appeal from that body,
the commissioner and his tribunal. It must, he says, be a necessary inference that Parliament did not intend that the decisions of
the commissioner shall be subject to the supervision of the High Court. If the High Court is to exercise the jurisdiction contended
for by the plaintiffs, anyone dissatisfied with the commissioners decision can apply to the High Court. The only time limit
fettering an application for a declaration is that provided by the statutory limitation and, if the jurisdiction exists, the plaintiffs can
take the matter to the House of Lords. Such a result is, he says, so at variance with the procedure specified in the Act of 1946 that
it is a necessary inference that Parliament cannot have intended that a claimant should be allowed to take the course contended
for by the present plaintiffs. The matter is rendered even more clear by an examination of the National Insurance (Determination
of Claims and Questions) Regulations, 1948, SI 1948 No 1144. It is important to observe the time limits imposed. Thus reg 10
requires the insurance officer, so far as practicable, to determine any question within fourteen days of its submission. Under reg
11, the claimant must give notice of appeal to the local tribunal within twenty-one days of the decision of the insurance officer.
By reg 15, an appeal to the commissioner must be brought within three months. Regulation 16 provides for the hearing before
the commissioner. Without detailing its provisions, it is perhaps sufficient to say that they are designed to ensure a careful and
judicial hearing, provision being made for oral evidence and representation of the parties, and for assistance to the commissioner
when difficult questions of law or fact arise. Counsel for the minister argues that the procedure provided for by the Act of 1946
and the regulations is designed to ensure the speedy decision that is obviously desirable in connexion with claims of this sort. In
the case of the ministers questions, where provision is made for limited appeal to the High Court on a point of law, the time for
appeal is twenty-one days. In the chain of appeal from the insurance officer to the commissioner strict time limits are provided.
Surely, he contends, the inevitable inference must be that Parliament did not intend that the claimant, having failed before the
commissioner should be fettered only by the statutory limitation and that, subject to this, he should be allowed at any time to
claim a declaration in the High Court, which puts the matter, subject to any necessary leave, through three further courts.
Before reading these regulations, it is convenient to mention the argument on point (B), namely, that any declaration of this
court could not be implemented. Apart from the contention that the decision of the commissioner must remain and that,
accordingly there would be no award, counsel for the minister relies on reg 18 and reg 19, g. Under these regulations, provision is
made for review in certain circumstances. He points out that review will only give a right to benefit with a very limited
retroactive effect. It was, he said, suggested in the Court of Appeal in the present case that a decision of the High Court that the
commissioners decision was wrong might confer a right of review, thus enabling 698 an award. To this his answer is that reg 19
would only allow benefit over a period of six months prior to the change of circumstances relied on h and this could not be said to
have occurred prior to the filing of the present summons on 28 March 1962, and, in consequence, could not avail the plaintiffs,
since their claims do not relate to periods after September, 1961.
________________________________________
g Ie, of the National Insurance (Determination of Claims and Questions) Regulations, 1948, SI 1948 No 1144
h Regulation 19, so far as material, provides: (1) Where on review a decision is revised so as to make benefit payable, the decision given
on the review shall have effect as follows:(a) in the case of unemployment benefit ; as from the date of the application for the review;
Provided that if in any case the claimant proves(i) that on a date earlier than the date on which the application for the review was
made he was entitled to benefit; and (ii) that throughout the period between the earlier date and the date on which the application for
review was made he was entitled to benefit; and (ii) that throughout the period between the earlier date and the date on which the
application for review was made, there was good cause for delay in making the application; he shall not be disqualified for receiving any
benefit to which he would have been entitled in respect of the said period.

The Tribunals and Inquiries Act, 1958, by s 11(1), confers the right to move by certiorari, and it is conceded that the
plaintiffs could have done so in this case. They are now out of time and were out of time when this summons was issued. It is to
be observed that s 9 provides for appeal to the High Court from certain inferior tribunals and reference to para 11 of Sch 1 shows
that this appeal was not conferred in the case of the National Insurance Commissioner. On the contrary, that tribunal is entrusted
by s 1 to the supervision of the council to be appointed under the Act of 1958. Here again, it is contended, is strong evidence that
Parliament did not intend that the decision of the commissioner should be subject to proceedings such as the present. Turning to
point (B), counsel for the minister pointed out that where certiorari was involved and the decision of the commissioner was
quashed, there was an appeal still before him and he could accordingly make an order pursuant to the decision of this court.
Where this court merely granted a declaration, the decision of the commissioner remained. Accordingly, there could not be an
award when there was no means of implementing the declaration. The only way in which effect could be given to the decision,
or so it is said, would be for the minister to apply to the Treasury for payment out of public funds, Parliament to validate the
payment when the next appropriation is made, and inasmuch as that payment would, in such circumstances, come out of public
funds instead of the insurance fund, this court ought not to assume power to make a declaration in relation to a statute concerned
with payments out of the insurance fund.
The submission made to me on point (C) was to the effect that, with one exception, in every case where the court has
exercised its supervisory jurisdiction by the grant of a declaration it has done so either because the inferior tribunal had acted
without jurisdiction so that its decision was a nullity, or in order to enable the plaintiff to obtain his contractual rights. Thus,
where a contract between the parties imported submission to statutory rules and the decision of a statutory body, this court has
held that the rules formed part of the contract and that the statutory body could not give itself jurisdiction by misconstruing its
rules, and has granted a declaration. There is no case, it is said, where this court has held that it has jurisdiction to make a
declaration on the ground that a statutory body with jurisdiction has made an error of law in its exercise. Counsel for the minister
cited Andrews v Mitchell and Cooper v Wilson as cases where the inferior tribunal had acted without jurisdiction, and Hanson v
Radcliffe UDC and Lee v Showmens Guild of Great Britain as contract cases. He also cited Barnard v National Dock Labour
Board as a case where the inferior tribunal had acted without jurisdiction. In that case, the board had delegated its disciplinary
powers to the port manager, a fact which was only ascertained on 699 discovery in the action. Thus, the plaintiff could not have
succeeded in proceedings for certiorari. The Court of Appeal held that the board was not entitled so to delegate its powers and
that the order suspending the plaintiff from work and pay was a nullity. Counsel relied on the following passages from the
judgment of Singleton LJ ([1953] 1 All ER at p 1116; [1953] 2 QB at p 35):

It is submitted on behalf of the National Dock Labour Board that in the order of 1947 there is a complete code on
matters of discipline, and that the only way in which decisions of the board can be questioned is by writ of certiorari.
There is great force in this submission, and there is a body of authority to support it. It cannot be right to say that whenever
a tribunal such as the local board or the appeal tribunal makes a mistake the court can grant a declaration such as is sought
in the present case. That would lead to endless confusion. The courts have, however, power to grant a declaration or an
injunction in certain cases to prevent an injustice.

Subsequently he said ([1953] 1 All ER at p 1118; [1953] 2 QB at p 38):

In the present case, if the question is not one of jurisdiction, it is certainly closely akin to it. The local board had no
jurisdiction to delegate; the port manager had no jurisdiction to adjudicate; each purported so to do; and, as in Cooper v.
Wilson a writ of certiorari was of no use.

It is, however, to be observed that Denning LJ and Romer LJ put the matter in much wider terms. Denning LJ said ([1953] 1 All
ER at p 1119; [1953] 2 QB at p 42):

This is not a case of a tribunal which has a lawful jurisdiction and exercises it; it is a case of a man acting as a tribunal
when he has no right to do so. These courts have always had jurisdiction to deal with such a case. The common law courts
had a regular course of proceeding by which they commanded such a person to show by what warrantquo warrantohe
acted. Discovery could be had against him, and, if he had no valid warrant, they ousted him by judgment of ouster. In
modern times proceedings by quo warranto have been abolished and replaced by declaration and injunction: see s. 9 of the
Administration of Justice (Miscellaneous Provisions) Act, 1938. Side by side with the common law jurisdiction of quo
warranto the courts of equity have always had power to declare the orders of a usurper to be invalid and to set them aside.
So, at the present day we can do likewise. We can declare that the suspension ordered by the port manager was unlawful
and void. We can declare it to be the nullity which in law it was.

Romer LJ said ([1953] 1 All ER at pp 1121, 1122; [1953] 2 QB at p 42):

As to counsels preliminary point, prima facie it is the right of everyone in this country who is involved in a legal
dispute to have that dispute determined by Her Majestys courts. That right can be taken away sometimes by contract,
subject to certain safeguards, and certainly by statute; but except to the extent to which it is taken away (and here we are
only concerned with parliamentary intervention), that prima facie right remains.

Finally, counsel for the ministry referred me to Taylor v National Assistance Board. In that case, the National Assistance Board,
in assessing the wifes disposable income for the purposes of the grant of a civil aid certificate included the sums due to be paid
to her under an order for alimony pendente lite. The 700 learned President granted a declaration that she was entitled not to have
such sums included. Treasury counsel had argued that there was no jurisdiction. It was suggested that that case might be
distinguished from the present case on the ground of the Presidents observations ([1956] 2 All ER at p 460; [1956] P at p 495).
It was also sought to distinguish that case on the basis that the court was dealing with an administrative rather than a judicial
decision.
In replying to this argument, counsel for the plaintiffs pointed out that, inasmuch as it had been conceded that certiorari was
available to the plaintiffs, the question was really one of procedure. Nowhere in the National Insurance Act, 1946, were there
express words excluding the supervisory jurisdiction of this court in relation to the decisions of the commissioner. If his
decisions could be tested by certiorari, why not also by the present procedure which had the advantage of enabling discovery? In
relation to the argument that the declaration sought would be ineffective, he relied on RSC, Ord 25, r 5, and said i, that the
plaintiffs were entitled to have their rights declared irrespective of whether they could or could not enforce them. As to the
enforcement of my declaration, counsel for the plaintiffs suggested that the ministry could, if they so desired, find a way to give
effect to the decision of this court and that payment out of public funds would not be objected to by the plaintiffs if provision out
of the insurance fund was impossible. He relied on the views expressed by Denning LJ and Romer LJ in Barnards case ([1953]
1 All ER at pp 1119, 1121, 1122; [1953] 2 QB at pp 42, 46) and the decision of the President in Taylors case. Counsel for the
plaintiffs pointed out that the latter case went to the Court of Appeal and to the House of Lords, in neither of which courts was the
point of jurisdiction taken. Both courts assumed that there was jurisdiction. I was referred by him to Vine v National Dock
Labour Board, in which the House of Lords, following Barnards case, granted a declaration that the dismissal of the plaintiff
was invalid for want of jurisdiction, and to Francis v Yiewsley and West Drayton UDC, in which the Court of Appeal granted a
declaration that an enforcement notice served by the defendants was invalid. He relied strongly on Pyx Granite Co Ltd v
Ministry of Housing and Local Government. In all those cases the argument which was addressed to me in the present case on
behalf of the ministry was advanced, albeit admittedly in connexion with other statutes. In each case it was rejected, the courts
reiterating that the subjects recourse to Her Majestys courts for the determination of his rights is not to be excluded except by
clear words.
________________________________________
i RSC, Ord 25, r 5 reads: No action or proceeding shall be open to objection, on the ground that a merely declaratory judgment or order is
sought thereby, and the court may make binding declarations of right whether any consequential relief is or could be claimed, or not

In my judgment, the statute and statutory provisions on which counsel for the minister relies do not contain clear words
excluding the supervisory jurisdiction of the court to make declarations such as that sought by this summons. There is no
authority, as Lord Goddard, pointed out in the Pyx case ([1959] 3 All ER at p 8; [1960] AC at p 290),

for saying that if an order or decision can be attacked by certiorari the court is debarred from granting a declaration in
an appropriate case. The remedies are not mutually exclusive

I am not impressed by the argument that even if this court declared the law, no effect could be given to that declaration. It would
be the duty of the minister to take appropriate steps, if necessary by seeking amendment of the regulations, to give effect to it.
Moreover, the matter is, I think, concluded for me by the 701 opinions expressed in the Court of Appeal on the hearing of the
interlocutory appeal by the plaintiffs against the dismissal of their originating summons as being frivolous and vexatious.
Counsel for the minister tells me, and I accept, that once counsel for the plaintiffs had asked for leave to amend the originating
summons so as to raise the question whether on his findings the commissioner had erred in law, he stated that he would waive the
question of jurisdiction for the purpose of this summons, since it was obvious that, in the light of the amendment, it could not be
suggested that the plaintiffs summons should be struck out as frivolous or vexatious. In the course of his judgment, Lord
Denning, speaking of the amendment, said ([1963] 1 All ER at p 278):

When the case is put that way, it seems to me to be a very proper matter for determination by originating summons for
a declaration. Indeed, it is a sensible and modern way of approach. When an originating summons is framed so as to raise
a distinct and important point of law, there is not the slightest possible ground for saying that it is frivolous and vexatious.
Counsel for the minister acknowledged as much in his opening remarks when he said that he did not challenge that the
High Court has jurisdiction to make a declaration that the determination of the commissioner was bad in law.

He dealt with the amendment and described it, and added that, on that amendment being made, the jurisdiction of the court would
be clear. Upjohn LJ said ([1963] 1 All ER at p 279):

I agree with the order proposed. It appears that the High Court has jurisdiction to correct the decisions of inferior
tribunals by declaration, and, if necessary, injunction, where it is alleged that the inferior tribunal has made some mistake
of law: see Barnard v. National Dock Labour Board, where this court held that the subject is not confined necessarily to
proceedings by way of certiorari. Where the alleged mistake of law is said to consist in some misconstruction by the
tribunal of some statutory rule or order, it is a convenient, speedy and cheap procedure to raise the question and to ask for a
declaration by taking proceedings by originating summons under R.S.C., Ord. 54A.

Diplock LJ is more guarded. He said ([1963] 1 All ER at pp 279, 280):

I merely desire to add that, in concurring in the order made, I do not wish it to be thought that, without further careful
examination, I necessarily assent to the proposition that a declaration lies as an alternative remedy wherever certiorari
would lie. I think that it must depend, or may at any rate depend, on the statutory terms in which jurisdiction is conferred
on the inferior tribunal and on the statutory effect of its decision. That, however, is a matter which is not appropriate for
discussion on summary proceedings such as these.

It may well be that the Master of the Rolls misunderstood the quality of the admission which counsel for the minister intended to
make, but I do not think that he founded his expression of view on that. I think that he and Upjohn LJ were expressing a firm
opinion on the jurisdiction of the court. Whatever counsel may or may not admit, jurisdiction is a question for the court itself.
Nor do I think that the views expressed can be considered as obiter: the question whether the court has jurisdiction to grant the
relief claimed must be directly relevant to the question whether a summons is vexatious and frivolous. I conclude, therefore, that
irrespective of the view which I have expressed, I am bound to find that this court had jurisdiction to grant the relief sought.
702
I turn now to the merits of this appeal. The vital findings of the commissioner are as follows j:
________________________________________
j Decision CU.22/61, dated 20.9.61

Para. 4. The claimant and each of the others whose right to benefit depends on these appeals are platers helpers.
They accordingly belong to a recognised grade of semi-skilled workers, as is shown by an agreement made in 1955 to
which their association was a party. It is common ground that during the period stated at the head of this decision they lost
employment by reason of a stoppage of work due to a trade dispute at their place of employment, a shipyard. The dispute
however was between the platers and the shipwrights, who are skilled tradesmen. (It also of course involved the
employers.) The disputing tradesmen however were contending in their own interests and not either for or against those of
the platers helpers, and neither the platers helpers nor their association, a general association including labourers also,
took part in the dispute or did anything to affect the result.
Para. 5. Traditionally the main place of work of shipwrights is on the berth, and that of the platers is in the shops in
the shipyard. Their systems of work are very different. One plater works with one platers helper; there are therefore about
the same number of platers engaged on the relevant work as there are platers helpers.

The commissioner then went on to contrast the position of shipwrights and of their semi-skilled assistants, the erectors, pointing
out that the proportion of the latter to the shipwrights whom they assist is about one to five. Having reviewed the history of the
long-standing dispute between the shipwrights and the platers and the circumstances of the stoppages which gave rise to the
present claims, the commissioner then recited the arguments before him before making his conclusions of fact in para 11. Having
easy for platers helpers to transfer to the position of erectors, he states:

Further, I am fully satisfied that, if the platers contentions in the dispute had prevailed to the full extent, all the
disputed work would have been done by platers and their helpers; whereas if the shipwrights contentions had prevailed to
the full extent, that same work would have been done by shipwrights, with perhaps very minor assistance from others, and
no help at all from platers helpers. The result of the dispute either way would therefore have involved a transfer of work
as between platers helpers and shipwrights and only to a lesser extent between platers helpers and erectors. And nobody
has suggested that it would be easy or even possible for a platers helper to transfer into the grade of shipwright. In my
judgment, especially in view of the clearly proved differences in the established systems of work referred to in para. 5
above, the only possible conclusion from the whole evidence is that the amount of work available for platers helpers must
inevitably have been substantially affected by the outcome of the dispute.

The learned commissioner then deals with the Act of 1946 and refers to previous decisions of the commissioners or deputy
commissioners, finding in reference to one of these:
Para. 13. I am not satisfied that this criticism of the decision is justified, but in any event this case goes much
further, since I am satisfied that it is quite certain that the amount of work available for platers helpers must depend on the
outcome of this dispute.

Finally, after expressing his approval of the decision reached by a tribunal of commissioners in 1960 k, he concludes:
________________________________________
k Decision R(U)1/60

Para. 14. Indeed, without the assistance of any authority, I should have come to the conclusion that, on the findings
of fact already recorded by me in this case, it cannot possibly be held that any of the 703 claimants or their association have
proved that neither they nor any of the other platers helpers were directly interested in the dispute in question.

In criticising this decision, counsel for the plaintiffs, in the course of his opening, suggested that the commissioners had been
misinterpreting the provisions of s 13(1) of the Act of 1946 in that they have come to interpret interested as equivalent to
directly interested. He referred me to a number of previous decisions of the commissioners and in the course of so doing he
asked for the text of a decision, CU 16/59, in the case of a claim by a platers helper. On this being produced, it became apparent
that the deputy commissioner in that case, the facts of which were not very dissimilar to those in the present case, had given the
most careful consideration to the distinction between directly and indirectly interested and had concluded in favour of the
claimant. That decision had been reviewed by the tribunal of commissioners in decision R(U) 1/60, the decision to which the
commissioner referred with approval in the present case. The tribunal, dealing in that case with the claim of a shipwright, held
that the claimant was directly interested and that the previous decision in 1959 (CU 16/59) was wrongly decided. Counsel for the
plaintiffs accordingly amended his general criticism and said that it was clear that there are two schools of thought among
commissioners and that it was desirable in such circumstances that they should receive an authoritative decision from this court.
I confess that this argument did not appeal to me. I see no reason to suppose that the decision of the tribunal in 1960 is not being
loyally applied by all the commissioners, or that the commissioner in the present case, who had before him the 1960 decision
which reported the facts of the 1959 decision, was not addressing his mind to the difference between a direct and an indirect
interest. Whether his decision is right is another matter.
Counsel further argued as follows: In considering the meaning of the words directly interested in the dispute one must be
very careful not to confuse the dispute with the original cause of the trouble, and one must be careful to define the parties to the
dispute. Here, as he says, the first dispute was between the employers and the seven platers who were told to do the work that the
previous seven had refused to do. He suggests that the refusal of the second lot of seven men arose simply out of loyalty and to
avoid the term blackleg. He says it was simply loyalty between men of the same union. The second dispute was between the
shipwrights and the employers, and the platers were not themselves involved, let alone the platers helpers.
It must be noted that this argument does not appear to have been put before the commissioner, with the result, presumably,
that evidence was not directed to it, and I cannot think that it should be open to the plaintiffs when invoking the supervisory
jurisdiction of this court. In any event, it seems to me somewhat unreal in the circumstances to attempt to divorce the
circumstances of the dispute from the root cause of the dispute which prompted the seven platers and the shipwrights to refuse to
do certain work. Alternatively, there were two disputes which caused one stoppagethe dispute between the platers and the
shipwrights and the dispute between the employers and the platers in the case of the March stoppage, and that between the
employers and the shipwrights in the later stoppage.
Counsel for the plaintiffs further submitted that on a normal use of language, any interest of the platers helpers must be
indirect, the direct interest being the rights acquired or lost by the platers. He contended that the interest of anyone other than a
party to the dispute must be indirect even when that party is claiming on behalf of another, or on behalf of all workers. The
interest must be indirect unless the claimant stands to gain what the party to the dispute is claiming. No doubt the construction of
the section is not entirely easy; participating in or financing involve active support by the claimants, and it is to be observed
that the third category is directly interested in, not has a direct interest in. I suggested to counsel that it might be argued that
directly interested 704might, in the context, also be held to involve some sort of active support. However, learned counsel
both indicated that they did not regard this as arguable and that the words must be treated as equivalent to has a direct interest
in. Indeed, it is clear from these decisions that the commissioners have always so treated them. This being the position, I am
not satisfied that the commissioner was wrong. When a plater requires help, he and his helpers work in effect as a term. It seems
to me that, in those circumstances, the platers helper has just as much a direct interest in the volume of work and type of work
available for platers as the platers have themselves. Certainly it would be difficult for the platers helper to prove that his interest
differed from that of the plater, and the commissioner has found that the platers helpers failed to do so in this case.
Finally, I turn to the question of discretion. This is a test action on which a number of other claims depend. I do not know
their number or extent, but it is clear that the issue is regarded by the plaintiffs union as important. Moreover if they are right,
these claimants, who are working men, have been deprived of what is due to them. On the other hand, the supervisory
jurisdiction of the court ought not, I think, to be invoked whenever an inferior tribunal has made an error of law. The
commissioner had to decide a mixed question of law and fact within his jurisdiction. The distinction between a direct and an
indirect interest on the facts of an individual case is one on which judges are likely to differ. There is no doubt that this
jurisdiction does not fit conveniently into a scheme designed to give speedy decisions on claims for unemployment benefit. In
Barnards case, Singleton LJ said ([1953] 1 All ER at p 1116; [1953] 2 QB at p 35):

It cannot be right to say that whenever a tribunal makes a mistake the court can grant such a declaration as is
sought in the present case. That would lead to endless confusion.

In the Pyx Granite case in the House of Lords, Viscount Simonds said ([1959] 3 All ER at p 7; [1960] AC at p 287)

The appropriateness of the remedy was the final point on this part of the case. It was urged that, even if the court had
jurisdiction to make the declaration claimed, it was a discretionary jurisdiction which should not be exercised in this case.
My Lords, this plea should not, in my opinion, prevail. It is surely proper that, in a case like this involving, as many days
of argument showed, difficult questions of construction of Acts of Parliament, a court of law should declare what are the
rights of the subject who claims to have them determined. I do not dissent from the contention of the respondents that,
where the administrative or the quasi-judicial powers of the minister are concerned, declaratory judgments should not
readily be given by the court. But here, if ever, was a case where the jurisdiction could properly be invoked. It might even
be thought surprising that the minister should not be glad to have such questions authoritatively determined.

Similarly, in the House of Lords in the case of Vine v National Dock Labour Board, reference was made to the care with which
this remedy should be used.
If these expressions are to be given any meaning, I venture to doubt whether the discretion conferred by RSC, Ord 54A, r 4
should be exercised in a case such as this. For those reasons, I must refuse the declarations that are sought.
Declarations refused.

Solicitors: Rowley Ashworth & Co (for the plaintiffs); Solicitor, Ministry of Pensions and National Insurance.

K Diana Phillips Barrister.


705
[1963] 2 All ER 706

Re W (J C) (an infant)
FAMILY; Children

CHANCERY DIVISION
PENNYCUICK J
1, 8 MAY 1963

Infant Guardianship of Infants Acts Custody Division of custody rights Order of magistrate giving legal right to custody
of infant to father and possession to mother Word custody denoted all rights and duties towards infant Whether court had
no jurisdiction under the Acts to divide custody Guardianship of Infants Act, 1886 (49 & 50 Vict c 27), s 5.

Custody in s 5a of the Guardianship of Infants Act, 1886, denotes rights and duties in relation to an infant regarded as an
indivisible whole, viz, the charge of the infants person coupled with the right to determine the manner of the infants upbringing;
accordingly there is no jurisdiction under that section to give legal custody to one parent and care and control (i.e., charge of
the person of the infant) to the other (see p 710, letter b, post).
________________________________________
a Section 5 is set out at p 708, letter h, post

An infant, the only child of the marriage, was born on 2 April 1961. Early in 1962 the parents separated. On 30 March
1962, the mother issued a summons under the Guardianship of Infants Acts seeking custody of the infant and provision towards
the infants maintenance. On 12 April 1962, the father issued a cross-summons. At the hearing of the summonses the stipendiary
magistrate made an order giving custody of the infant to the father, but ordering that the mother should have possession of the
infant until further order; and he gave directions as to times when the father might have the infant. On appeal by the mother, on
the ground that the order was wrong in law,

Held For the reasons stated at letter b above the order ought to be varied; and the case would be remitted accordingly.
Appeal allowed.

Notes
As to rights of mother and father to custody, see 21 Halsburys Laws (3rd Edn) 191193, paras 425, 427; and for cases on the
subject, see 28 Digest (Repl) 609, 610, 11721177.
For the Guardianship of Infants Act, 1886, s 5, s 9, see 12 Halsburys Statutes (2nd Edn) 943, 945; for the Custody of
Children Act, 1891, s 4, see ibid, 948; for the Guard.ianship of Infants Act, 1925, s 3, s 7, see ibid, 955, 958.
For the Matrimonial Causes Act, 1950, s 26, see 29 Halsburys Statutes (2nd Edn) 413.

Cases referred to in judgment


Agar-Ellis, Re, Agar-Ellis v Lascelles (1878), 10 ChD 49, 48 LJCh 1, 27 WR 117, CA, 28 Digest (Repl) 642, 1362.
E, Re [1955] 3 All ER 174, [1956] 1 Ch 23, 28 Digest (Repl) 607, 1171.
Wakeham v Wakeham [1954] 1 All ER 434, [1954] 1 WLR 366, 3rd Digest Supp.
P (an infant), Re 18 May 1951 (unreported).

Appeal
This was an appeal by notice of appeal dated 4 August 1962, by the mother of an infant, Mrs Barbara Tattersall Wainwright, from
an orderb, dated 19 July 1962, of the stipendiary magistrate of the City of Leeds, whereby the custody of the infant was given to
the father but it was ordered that the mother should have possession of the infant until further order save that the father should be
entitled to have the infant from 11 am to 6 pm one day in each week and for the period of a fortnight during the summer holidays.
The grounds of appeal were that the decision of the stipendiary magistrate was against the weight of the evidence, and that he was
wrong in law in giving custody to the father and care and control to the mother. The facts appear in the judgment.
706
________________________________________
b The order was made under the Guardianship of Infants Acts, 1886 and 1925, and the Guardianship and Maintenance of Infants Act, 1951

B J H Clauson for the appellant mother.


Donald Herrod for the respondent father.

Cur adv vult

8 May 1963. The following judgment was delivered.

PENNYCUICK J. This is an appeal from an order dated 19 July 1962, made under the Guardianship of Infants Acts by the
stipendiary magistrate for the city of Leeds. The appeal relates to an infant, John Christopher Wainwright, to whom I will refer as
the infant. The appeal is that of the mother, Mrs Barbara Tattersall Wainwright, and the respondent is the infants father,
George David Wainwright. The appeal raises a question of law of some general importance and for that reason I am giving this
judgment in open court.
The father and the mother were married on 12 July 1960. The mother was then a widow with one son, Michael, born on 27
April 1956. During the marriage, the father and mother lived in a house belonging to the mother in St Anns Road, Leeds. The
infant was born on 2 April 1961, and is the only child of the marriage. Differences arose between the father and the mother
towards the end of 1961. Those differences appear to have been partly personal and partly financial; the particular matters of
disagreement appear to have been the position of the mothers mother and the position of her elder son, Michael. Early in 1962
the father and mother separated, the father going to his family at Huddersfield and the mother remaining at St Anns Road, Leeds,
with Michael and the infant. The father has visited the infant regularly there. The father wishes the infant to live with him at his
parents house in Huddersfield.
On 30 March 1962, the mother issued a summons under the Guardianship of Infants Acts in the Leeds magistrates court
seeking custody of the infant and provision towards its maintenance. On 12 April 1962, the father issued a cross-summons. The
learned stipendiary heard the two summonses on 3 May and 5 July 1962. At these hearings the father and the mother, and three
other witnesses called by the father, were examined and cross-examined at considerable length and a transcript of the evidence
has been read on this appeal. For reasons which will be apparent, I do not propose to go into the evidence beyond stating that it
disclosed nothing of a startling nature and no ground for saying that either the father or mother is unfit to have charge of the
infant. On 19 July 1962, the learned stipendiary gave judgment in the following terms:

I have been unable to find any authority as to whether or not magistrates courts have the powers sometimes exercised
in the High Court of Justice in making an order for the custody of a child with conditions, in particular, to grant custody in
law to the father of a child of the marriage with a provision that he shall not remove the child from the care and control of
the mother, unless or until he obtains the permission of a court to do so. I can find no record of any instances in this court
of there having been made any custody order of the kind to which I have referred; whereas I am of the opinion that such an
order would be very desirable in this case.
My main object is to discover, as far as I can, what decision is the most likely to result in the greatest benefit to the
child John Christopher Wainwright who is now under two years of age.
So far as I am aware of the practice of the High Court of Justice in these matters, it is most unusual to deprive the
mother of a very young child of, at least, the care and control of the child. Only in cases of extreme necessity, having
regard to the paramount consideration, that is to say, the childs welfare, would a child so young as John Christopher
Wainwright be taken from his mother. There is no evidence before me as to suggest any necessity to deprive Barbara
Tattersall Wainwright of the possession of her son.
On the other hand, it is, in my opinion, in the interests of John Christopher Wainwright that his father, George David
Wainwright, should have a practical and effective interest in and, so far as possible, be able to plan 707 ahead and provide
for his childs further education, so as to prepare for the time when John Christopher Wainwright shall have become old
enough to benefit from the best education available. For this purpose, George David Wainwright requires at least some
promises of certainty that he can prepare for such education in a reasonable hope of his desires being effective, and, should
the circumstances warrant it because of dispute between the parents, to have the chance of proving to the satisfaction of the
court that his proposals are in the best interests of his son.
I therefore order that the custody of John Christopher Wainwright be granted to George David Wainwright as the
father, but that John Christopher Wainwright shall not be removed from the care and control of Barbara Tattersall
Wainwright, his mother, save as provided for in the access defined hereby, that is to say, that George David Wainwright
shall be entitled to the possession of his son from eleven in the morning until six in the evening on one day in each week,
and for the period of a fortnight during the school holidays in the summer of each year.
Since neither party to these proceedings wholly succeeds or wholly fails in this application for the custody of John
Christopher Wainwright, I make no order as to costs.

In accordance with that judgment, the learned stipendiary made an order that the custody of the infant be given to the father and

that [the father] shall have the right of access to the said infant (by consent): the said mother to have possession of the
said child until further order. The said father to have the child one day each week between the hours of 11 a.m. and 6 p.m.
and for a fortnight during the summer holidays.

Against this order, the mother appeals. The grounds of appeal are set out as follows:

(1) That the decision of the stipendiary magistrate granting the custody of the child of the marriage to the said George
David Wainwright was against the weight of evidence. (2) That the stipendiary magistrate was wrong in law in making an
order giving the custody of the child of the marriage to the said George David Wainwright and the care and control of the
child of the marriage to the said Barbara Tattersall Wainwright.

Counsel for the mother has confined his argument to the second ground of appeal, which is a matter of law.
The jurisdiction of the magistrates court arises under the Guardianship of Infants Acts, 1886 to 1925. The primary purpose
of the Act of 1886, it appears, was to give further rights to the mother in respect of guardianship. Section 5 of the Act of 1886 is
as follows:

The court may, upon the application of the mother of any infant make such order as it may think fit regarding the
custody of such infant and the right of access thereto of either parent, having regard to the welfare of the infant, and to the
conduct of the parents, and to the wishes as well of the mother as of the father, and may alter, vary, or discharge such order
on the application of either parent, or, after the death of either parent, of any guardian under this act

and then the section goes on to deal with costs. By s 9, the court is defined as meaning in England the High Court or the
county court of the district in which the respondent resides, but under s 7 of the Guardianship of Infants Act, 1925, the court is
extended so as to include a court of summary jurisdiction. The Acts do not contain any definition of the word custody and one
must therefore consider what is meant by that word where it is employed in the Acts.
At common law, the father of an infant had exclusive custody of the infant. Custody, in the first place, comprised or,
perhaps more accurately, denoted 708 charge of the infants person. The first meaning of custody in the Oxford English
Dictionary is safe keeping, Custody also includes as an inseparable element the right to determine the manner of the infants
upbringing as regards religion, education and other matters (see Re Agar-Ellis, Agar-Ellis v Lascelles). Those rights, of course,
carried corresponding duties. The notion of divided custody was, so far as I have been able to discover, unknown to the common
law.
The Chancery Division, in the exercise of its inherent jurisdiction, took on itself the charge of its wards and in the exercise
of this jurisdiction has been accustomed to make a variety of orders with regard to all the matters involved in custody, including
the charge of an infants person, religion, education and other matters. The wardship of itself involves in one sense the division
of custody, and the directions in appropriate cases may involve a further division.
The Probate, Divorce and Admiralty Division has statutory power to make such provision as appears just to it with regard to
the custody, maintenance and education of the children of the marriage (see s 26 of the Matrimonial Causes Act, 1950, replacing s
35 of the Matrimonial Causes Act, 1857). In the exercise of those statutory powers, the courts have from time to time given what
is now described as legal custody to one parent and what is now described as care and control to the other parent. This is a
well-recognised form of order. In Wakeham v Wakeham Denning LJ said ([1954] 1 All ER 434 at p 436):
Cases often arise in the Divorce Court where a guilty wife deserts her husband and takes the children with her, but the
father has no means of bringing them up himself. In such a situation the usual order is that the father, the innocent party, is
given the custody of the child or children, but the care and control is left to the mother. That order is entirely realistic. By
giving the father the custody, it recognises that he, the innocent party, is at least entitled to a voice in the bringing up of the
child or children, and also to the consideration of the court when any question arises as to what is to be done for the child.
The paramount, but not the only, consideration when the court comes to consider the matter is, of course, the welfare of the
child, but the fathers views are also entitled to consideration, and that is why the order for custody should be given to him,
although, solely for practical reasons, the mother may have the care and control.

That form of order is, so far as I have been able to ascertain, a modern development. Indeed, until the beginning of this century
an adulterous wife was not even allowed access.
As regards terminology, the word custody is of immemorial antiquity. The expression custody and controlor custody
or control is also of immemorial antiquity, apparently as a synonym for custody. On the other hand, the expression care and
control appears to be of quite modern origin and so far as I can discover has no statutory sanction. Again, as a matter of
language, it will be seen that where an order is expressed to give legal custody to one parent and care and control to the other,
the result is that the person who takes the legal custody does not take what is normally the basic element of custody, namely,
charge of the infants person. In this Division, in the exercise of its inherent jurisdiction, the more usual practice of the court
today, I understand, is for the order to be silent as regards custody and to be expressed as giving care and control to the party
who is to have charge of the infants person. Orders expressed to give custody to such persons are, however, sometimes made,
and there is not uniformity of practice as regards the terms used to express charge of the infants person. In particular, I
understand that where an order for maintenance is sought in respect of a ward, it is the practice for the summons to be intituled
In the matter of the Guardianship of 709 Infants Act, and an order made for custody in order to support the order for
maintenance (see s 3(2) of the Guardianship of Infants Act, 1925).
I now return to s 5 of the Guardianship of Infants Act, 1886. This section confers on the court jurisdiction to make such
order as it may think fit regarding the custody of infants and the right of access thereto. There is no jurisdiction under the section
to make any other order (see Re E (an infant)).
It seems to me that the word custody in this context naturally denotes rights and duties in relation to an infant regarded as
an indivisible whole, that is to say, the charge of the infants person coupled with the right to determine the manner of the infants
upbringing. The word is used in the sense known to the common law. In 1886 the notion of divided custody would, I think, have
been relatively unfamiliar and if the intention of the Act had been to confer on the court jurisdiction to divide up custody between
more than one person, one would have expected to find quite different language. The words of s 35 of the Matrimonial Causes
Act, 1857, would have afforded a precedent. It will be observed that if there were jurisdiction to divide custody at all, there
would be no logical reason for stopping at the simple division between what are now called legal custody and care and
control respectively. The jurisdiction would extend to such matters as residence and education, and thus would approximate in
many respects to that exercised by this Division under its inherent jurisdiction.
The construction which I have put on s 5 of the Guardianship of Infants Act, 1886, is, I think, confirmed by the circumstance
that the jurisdiction under the Act is exercisable not only by the High Court but also by the county courts, subsequently extended
to the magistrates courts. One would rather expect that jurisdiction to be of a relatively summary nature and not to cover the
more elaborate orders sometimes made in this Division. On this point, see, too, Re E (an infant) where Roxburgh J said ([1955] 3
All ER at p 176; [1956] 1 Ch at p 26):

Therefore, I do not think for a moment that it is a mere accident that the powers conferred under the Guardianship of
Infants Acts are less extensive than the powers which the Chancery Division of the High Court of Justice enjoys by virtue
of its inherent jurisdiction over infants.

The point appears to be free from direct authority. I have been referred by the learned registrar to the note of an unreported
judgment in Re P (an infant) on 18 May 1951, in which Roxburgh J allowed appeal from magistrates under the Guardianship of
Infants Acts stating that it was unsatisfactory to divorce custody from care and control, but no argument on the scope of the
magistrates jurisdiction under the Acts appears to have been addressed to him.
Some light on this point is perhaps thrown by two statutory provisions subsequent to the Act of 1886. The Custody of
Children Act, 1891, s 4, provides as follows:

Upon any application by the parent for the production or custody of a child, if the court is of opinion that the parent
ought not to have the custody of the child, and that the child is being brought up in a different religion to that which the
parent has a legal right to require that the child should be brought up, the court shall have power to make such order as it
may think fit to secure that the child be brought up in the religion in which the parent has a legal right to require that the
child should be brought up.

That section contains an express power to separate custody from decision on religion, which would be unnecessary if the
custody were severable under s 5 of the Act of 1886.
Then s 7(5) of the Guardianship of Infants Act, 1925, provides:

Where an order made by a court of summary jurisdiction under the Guardianship of Infants Act, 1886, as amended by
this Act, contains a 710 provision committing to the applicant the legal custody of any infant, a copy of the order may be
served on any person in whose actual custody the infant may for the time being be, and thereupon the provision may,
without prejudice to any other remedy open to the applicant, be enforced under s. 34(2) of the Summary Jurisdiction Act,
1879,c, as if it were an order of the court requiring that person to give up the infant to the applicant.
________________________________________
c See now the Magistrates Courts Act, 1952, s 54 (3), (4); 32 Halsburys Statutes (2nd Edn) 464

That subsection describes custody under an order of the court as legal custodya different use of the expression from that
considered aboveand then refers to some other person as having actual custody. The contrast is evidently between lawful
custody and unlawful possession and affords some indication that custody under an order automatically carries with it the
right to possession.
The difficulty of jurisdiction could not be circumvented by expressing the order as awarding custody to the father subject
to the proviso that he shall allow the mother to have care and control. No argument on those lines has been advanced before
me. This observation is not, of course, addressed to any undertaking which may be voluntarily given by the party to whom
custody is awarded.
I see force in the comment of the learned stipendiary that it is in the interests of the infant that the father should have a
practical and effective interest in its education. I think, however, that this point is to a great extent met by the following
considerations, namely (1) the fact that the mother had the custody would not prevent the father from making plans for the
infants education: (2) any order relating to an infant is in its nature subject to review, and where the custody of an infant is given
to one parent it is always open to the other parent to make a further application to the court; (3) it is also open to either parent to
apply to the Chancery Division by way of wardship proceedings, and that is probably the right course in a case of any
complexity; (4) if one parent has legal custody and the other care and control and they are unable to agree, a further application
by one or other to the court is probably inevitable in any case.
For the reasons which I have sought to give, I have come to the conclusion that the order dated 19 July 1962, should be
varied. It seems to me that, in all the circumstances, the proper course is to remit the case to the learned stipendiary. Counsel
invites me to exercise the discretion myself by awarding custody to the mother. I do not, however, think that I ought to do this. It
is true that the stipendiary has found that there is no evidence to suggest any necessity to deprive the mother of the possession of
her child. On the other hand, there is equally no complaint against the father. The order only gives care and control to the mother
on the footing that the father has the legal custody. I have not myself seen either of the parents and in all the circumstances I
think that it would be better that the learned stipendiary should make the decision rather than that I should do so on the material
before me and without having seen either party.

Appeal allowed.

Solicitors: Stafford Clark & Co agents for Simpson & Co Leeds (for the appellant); Smiles & Co agents for Marklands, Leeds
(for the respondent).

Jenifer Sandell Barrister.


711
[1963] 2 All ER 712

Note
Sumner v William Henderson & Sons Ltd
CIVIL PROCEDURE

COURT OF APPEAL
SELLERS, DONOVAN AND RUSSELL LJJ
20, 21, 22 MAY 1963

Practice Special case Hypothetical facts Questions of law stated for the opinion of the court on the basis of assumptions of
fact Unsuitability of procedure Decision set aside.

Notes
As to the stating of questions of law by way of special case, see 30 Halsburys Laws (3rd Edn) 393, para 732.
As to preliminary points of law, see ibid, p 392, para 730; and for cases concerning this procedure, see Digest (Pleading) 48
53, 390-427.

Cases referred to in judgment


Davie v New Merton Board Mills Ltd [1959] 1 All ER 346, [1959] AC 604, [1959] 2 WLR 331, 3rd Digest Supp.
Windsor Refrigerator Co Ltd v Branch Nominees Ltd [1961] 1 All ER 277, [1961] Ch 375, [1961] 2 WLR 196, 3rd Digest Supp.

Appeal
In an action begun by writ issued on 27 July 1960, the plaintiff, Isaac Sumner, claimed damages under the Fatal Accidents Acts,
1846 to 1959, in respect of the death of his wife, which was alleged to have been caused by the negligence or breach of duty of
the defendants, their servants or agents. On 22 June 1960, the plaintiffs wife was employed by the defendants as restaurant
supervisor in the defendants store; a fire broke out and she was asphyxiated by smoke when in the course of her employment.
The defendants denied negligence. The parties stated certain questions of law in the form of a special case for the opinion of the
court. By para 9 of the special case the court was asked

for the purposes of this special case, to assume the truth of each of the allegations of the plaintiff and defendants set
forth in paras. 5, 6, 7 and 8 of the special case

which paragraphs contained extracts from the statement of claim and the defence. The particular allegations in the statement of
claim concerned the electrical installation and the construction of the building, it being alleged by the plaintiff that a fault in an
electrical cable caused the fire and that a cause of his wifes death was that parts of the store had been so installed that the fire
spread with great rapidity. Paragraph 11 of the special case was as follows

The court is asked to consider each of the foregoing questions and in respect of any such question which it may decide
should be answered in the negative to order that the plaintiff may not adduce at the trial of the action any evidence which is
solely directed to showing that negligence on the part of the company or firm referred to in that question caused or
contributed to the death of [the plaintiffs wife].

The origin of the fire was not agreed between the parties. At the hearing before Phillimore J reported [1963] 1 All ER 408, His
Lordship determined the questions of law raised by the special case, holding that the defendants would be liable in law for any
negligence of certain contractors and electrical engineers. The defendants appealed.

J M Davies QC and R J H Collinson for the defendants, the appellants.


R M Bingham QC and G H Wright for the plaintiff, the respondent.

22 May 1963. The following judgment was delivered.

SELLERS LJ stating the decision of the court, said. At the outset of this appeal the court expressed doubts as to the desirability
and value of the 712 procedure which has been adopted by the parties in submitting the Special Case, and even as to its propriety
in seeking the opinion of the court on what seemed to be no more, or little more, than a consultative case on points of law.
[His Lordship referred briefly to the facts from which the action arose, read the provisions of para 9 of the special case
which have been set out previously a, stated that no facts were admitted, and that any fault or negligence of the defendants or of
any of those engaged by them to do the work was denied. His Lordship said that the court was asked, on the assumptions that it
had to make, in effect, whether, the defendants would be liable for a negligent act or omission, which caused the death of the
plaintiffs wife, on the part of the architects, the builders, the consulting electrical engineers or the sub-contractors for the
electrical work; and, after reading para 11 of the special case b, His Lordship continued:] With some hesitation we allowed the
defendants to open the appeal, which seeks to reverse the judgment of Phillimore J which answered four questions against the
contention of the defendants and held that they were liable in law for the negligence of those to whom they had delegated their
duty. As the argument has advanced, it was urged before us that in the light of the decision in Davie v New Merton Board Mills
Ltd the learned judges decision was wrong. That authority and earlier authorities make the problem a complex one and the
position in law may depend on the precise faults or negligence, if any, which are ultimately found in the case. It does not seem to
us possible or, if possible, appropriate to express a general opinion on the law which might be of no effect or erroneous on a
certain view of the facts or which would have to be alternative with regard to a variety of views of the facts.
________________________________________
a See p 712, letter f, ante
b See p 712, letter g, ante

In Windsor Refrigerator Co Ltd v Branch Nominees Ltd ([1961] 1 All ER 277 at p 283; [1961] Ch 375 at p 396) Lord
Evershed MR said at the end of his judgment:

I repeat what I said at the beginning, that the course which this matter has taken emphasises as clearly as any case in
my experience the extreme unwisdomsave in very exceptional casesof adopting this procedure of preliminary issues.
My experience has taught me (and this case emphasises the teaching) that the short cut so attempted inevitably turns out to
be the longest way round.

Harman LJ in concurring, said that he agreed particularly with the last observation of Lord Evershed MR ([1961] 1 All ER at p
283; [1961] Ch at p 396):

The number of conditions which my Lord has found it necessary to use to fence in the expression of this courts
opinion shows at once the undesirability of this kind of procedure. It is highly undesirable that the court should be
constrained to tie itself in so many knots, and in the end merely say: Well, if this was thus, then that was so.

Those observations apply very aptly, I think, to the present dispute.


In the present case no facts have been agreed and what the outcome of the evidence will be is most uncertain. It does not
seem to us in the interests of either party that a hypothetical decision should be reached now. It might tie one or other of the
parties to a decision which the facts might reveal as erroneous. Time is running out against possible claims against other parties,
either directly by the plaintiff or as claims over for indemnity by the defendants, and we do not think that a decision now on the
matters raised will satisfactorily relieve the parties from protecting themselves accordingly.
In our view the procedure by this Special Case was wrong: no judgment should have been given under it and it cannot
properly be further pursued. We therefore, on this ground only, set aside the judgment given, so that the parties are free 713 to
argue the legal position on established facts, and we proceed no further with this appeal.

Judgment set aside.

Solicitors: Lawrence, Graham & Co agents for Weightman, Pedder & Co Liverpool (for the defendants); Mace & Jones,
Liverpool (for the plaintiff).

Henry Summerfield Esq Barrister.


[1963] 2 All ER 714

J & C Moores Ltd v Commissioners of Customs and Excise


TAXATION; VAT and Customs and Excise: INTELLECTUAL PROPERTY; Copyright: LEISURE AND LICENSING

COURT OF APPEAL
LORD DENNING MR, HARMAN AND PEARSON LJJ
16, 17 MAY 1963

Purchase Tax Wholesale value Stationery Football coupons printed by subsidiary of pools promoters Copyright material
used in compilation of coupons Royalties paid by pool promoters for non-exclusive licence to reproduce or authorise
reproduction of copyright material Whether a copyright element should be included in wholesale value Finance (No 2) Act,
1940 (3 & 4 Geo 6 c 48), s 21, Sch 8, paras 3, 4.

Copyright Licence Licensee authorising printer to print football pool coupons including copyright material Whether any
copyright in coupons or title to copyright in printer Copyright Act, 1956 (4 & 5 Eliz 2 c 74), s 49(7).

The appellants, who were a subsidiary of a football pools company, printed football coupons for the company at an agreed price
of cost of production plus twelve and a half per cent In the compilation of the coupons the football pools company made use of
chronological fixture lists prepared by F Co Ltd. In 1940 the appellants agreed with the revenue authorities that the coupons
were subject to purchase tax on delivery to the football pools company under the classification of stationery, and that their
wholesale valuea for this purpose was the price at which they were sold. In 1959 F Co Ltd obtained a declaration in the
Chancery Divisionb that they were entitled to copyright in the fixtures lists. After the judgment, F Co Ltd granted to the pools
company a non-exclusive licence to reproduce or authorise the reporduction of any part of the fixture lists on payment of a
royalty. The revenue authorities now sought to add to the wholesale value (determined in accordance with s 21(1) of the Finance
(No 2) Act, 1940, and Sch 8, cf, paras 3, 4) the amount of the royalty payments on the ground that the value of the coupons had
been increased by the inclusion of a copyright element, viz, the right derived by the appellant printers through the football pools
company (see s 49(7) of the Copyright Act, 1956) to reproduce the fixtures lists.
________________________________________
a Wholesale value was defined in s 21(1) of the Finance (No 2) Act, 1940, for which see p 717, letter a, post
b Football League Ltd v Littlewoods Pools Ltd [1959] 2 All ER 546; [1959] Ch 637

Held Nothing should be added by reason of any alleged copyright element to the wholesale value of the coupons on which
purchase tax was charged, which value was rightly estimated as cost to the printers together with a reasonable profit (see p 718,
letter d, post), because
(i) the goods sold and chargeable to purchase tax were the printed coupons and the copyright formed no part of them, being
an incorporeal right distinct from the coupons and being vested as to the fixture lists in F Co Ltd and as to the rest in the football
pools company (see p 717, letter g, p 718, letter a, and p 720, letter b, post), and
(ii) the football pools company was the only possible purchaser (apart from a waste paper merchant) of the coupons in the
open market which had to be 714 assumed for the purposes of purchase tax, since a notional sale to a retail trader (such as had to
be imagined for the purposes of the tax), would lay the retail trader open, if he resold, to an action for infringement of copyright
in the fixtures lists, and it should not be supposed that the football pools company would pay more for the coupons to the
appellant printers than the football pools company were paying, since the football pools company were already paying F Co Ltd
for the licence to use material from the fixtures lists (see p 721, letters e and f, p 720, letter c, and p 718 letter c, post).
Appeal allowed.

Notes
As from 1 April 1963, s 21(1) of the Finance (No 2) Act, 1940, and Sch 8, paras 3, 4, are repealed by the Purchase Tax Act, 1963;
see now ibid, s 3(1) (wholesale value) and Sch 2, paras 3-5.
As to wholesale value, see 33 Halsburys Laws (3rd Edn) 240, para 416.
For the Finance (No 2) Act, 1940, s 21(1), Sch 8, see 21 Halsburys Statutes (2nd Edn) 1247, 1262.
For the Copyright Act, 1956, s 49(7), see 36 Halsburys Statutes (2nd Edn) 147.

Cases referred to in judgments


Football League Ltd v Littlewoods Pools Ltd [1959] 2 All ER 546, [1959] Ch 637, [1959] 3 WLR 42, 3rd Digest Supp.
Hill (William) v Ladbrooks 19 November 1962, not reported.
Morris (B) Ltd v Lunzer [1942] 1 All ER 77, [1942] 1 KB 356, 111 LJKB 653, 166 LT 143, 2nd Digest Supp.

Appeal
This was an appeal by J & C Moores Ltd from a judgment of Megaw J dated 31 January 1963, upholding an interim award stated
in the form of a Special Case, pursuant to the Arbitration Act, 1950, s 14 and s 21(1)(b), by Ashton Roskill QC a referee
appointed under the Finance (No 2) Act, 1940, s 21(2), whereby it was adjudged that the Commissioners of Customs and Excise,
the respondents, were entitled to make an addition to the wholesale value of the football coupons printed, sold and delivered by
the appellants by reason of the value of the right to reproduce, print and publish therein and thereby the copyright fixture lists of
the Football League Ltd and the Scottish Football League. The grounds of appeal were: (1) that the learned judge misdirected
himself as to the nature of the open market contemplated by the Finance (No 2) Act, 1940, s 21, Sch 8, as amended, and as to the
assumptions to be made accordingly concerning an open market for the sale of football coupons by the appellants to Littlewoods
Pools Ltd and H Littlewood Ltd; (2) that the learned judge did not sufficiently take into account the significance of the fact that
whereas by the said Sch 8 specific statutory provision was made for the computation of a price for the purpose of charging
purchase tax in respect of patents and designs, no such provision was made in respect of copyright; (3) that there was no evidence
on which the learned judge could find that the value of the coupons sold by the appellants to Littlewoods Pools Ltd and H
Littlewood Ltd had been increased by virtue of copyright protection at the material time; (4) that the learned judge misdirected
himself as to the value to the appellants of the right to reproduce the lists of football fixtures, which value in fact did not differ
from the value of any other printing order; (5) that the learned judge did not strictly construe the provisions of the relevant taxing
Act, namely, the Finance (No 2) Act, 1940, and Sch 8 thereto, which were neither clear nor unambiguous, and which on the
interpretations placed on them by the learned judge would contain at least one wholly superfluous paragraph; (6) that on the true
construction of the relevant statutory provisions, the learned judge ought to have found that there was no entitlement to make an
addition for the purpose of charging purchase tax by virtue of the copyright payments made to the Football League Ltd 715 and
the Scottish Football League, to the value of the football pool coupons printed by the appellants.
The facts appear from the judgment of Lord Denning MR.
The authority and cases noted belowc were cited in argument in addition to the cases referred to in the judgments.
________________________________________
c 28 Halsburys Laws (2nd Edn) 315, para 585, Re Sir William Thomas Paulin, Re Percy Crossman [1935] 1 KB 26, revsd [1936] 1 All ER
762, North Central Wagon & Finance Co Ltd v Fifield [1953] 1 All ER 1009

Sir George Honeyman QC and D H Hene for the appellants.


R E Borneman QC and F Hallis for the respondents.

17 May 1963. The following judgments were delivered.

LORD DENNING MR. We are concerned here with the purchase tax on football coupons. Littlewoods Pools Ltd distribute
ordinary football pool coupons; H Littlewood Ltd distribute fixed odds coupons; but nothing turns on the distinction. I will
call both companies Littlewoods and refer to both sets of coupons as coupons. The coupons have been printed for
Littlewoods by an associated company of theirs J & C Moores Ltd whom I will call Moores. The question is: what purchase
tax is payable by Moores on these coupons?
It is common ground that these coupons are chargeable with purchase tax. They come under the heading of stationery,
and are liable to a twenty-five per cent purchase tax. It is also common ground that, when Moores supply the coupons to
Littlewoods, the transaction is to be treated as if it were a chargeable purchase, and the purchase tax becomes due at the time
when the coupons are delivered by Moores to Littlewoods. That follows from the decision of this court in B Morris Ltd v Lunzer.
Those matters being common ground the whole question in the case is: what is the wholesale value of the coupons?for it is
on the wholesale value that the tax is to be calculated.
For many years the wholesale value was calculated, by agreement between the commissioners and Moores, at the full cost of
production (including overheads) plus twelve and a half per cent profit. In 1959, however, it was held in the Chancery Division
that the Football League Ltd have copyright in their lists of matches, and that accordingly Littlewoods, when they made use of
these fixture lists for their coupons, were guilty of infringement of copyright (see Football League Ltd v Littlewoods Pools Ltd).
In consequence of that decision, Littlewoods, in order to be able to use and make the coupons at all, had to seek the permission of
the Football League Ltd. Accordingly, they entered into an agreement with the Football League Ltd whereby they were given
permission to use the fixture lists for ten years on the terms that they were to pay a royalty to the Football League Ltd The
Football League Ltd gave Littlewoods authority to reproduce, or to authorise to be reproduced, the fixture lists or any parts of
them. They could use the lists only for the purpose of promoting their own football pools. They could not use the lists for
anybody elses football pools at all. Littlewoods had to pay a royalty to the Football League Ltd based on the number of coupons
they printed or the gross stakes received. The Commissioners of Customs and Excise, knowing that a royalty is being paid, claim
that it gives them a ground for increasing the purchase tax on the coupons. They say that the royalty shows that the wholesale
value is more than what they had previously agreed. In addition to the cost plus twelve and a half per cent, they seek to add a
sum for the copyright element of 3s 9d a thousand football pool coupons, and 4 2/3d a thousand fixed odds coupons.
The point comes down to this: when ascertaining the wholesale value of coupons, is it right to treat them (to use a useful
phrase) as loaded with the copyright element? This depends on the true application of s 21(1) of the Finance (No 2) Act, 1940.
It says:
716

(1) The wholesale value of any goods in respect of which tax is chargeable shall be taken to be the price which in the
opinion of the commissioners the goods would fetch, on a sale made at the time when the tax in respect of the goods
becomes due by a person selling by wholesale in the open market in the United Kingdom to a retail trader carrying on
business in the United Kingdom only, if no tax were chargeable in respect of the sale and it were made in the circumstances
specified in Sch. 8 to this Act.

In Sch 8, which is there referred to, there are certain circumstances to be assumed in making the calculation. For instance, it is to
be assumed that the price is to include the cost of delivery to the buyer at his place of business, that the price is the sole
consideration for the sale, and so forth. All of those assumptions can easily be made. There is, in addition, a series of
provisionsd in Sch 8 dealing with goods which are the subject of patents, or of registered design, or trade mark, or get-up. Under
those provisions the price is, in some circumstances, to be taken as loaded by reason of the patent element, the design element,
the trade mark element, or the get-up element. Significantly enough, there is no such provision in regard to the copyright
element. There is nothing to say that the price is to be loaded with the copyright element.
________________________________________
d See para 3, para 4 and para 5 of Sch 8 to the Finance (No 2) Act, 1940

In order to solve this problem, it is necessary to remember the nature of copyright. It is an incorporeal right, quite distinct
from the paper on which it is written. When an author sets down his words in writing he acquires copyright in the written
expression of this thought, but not in the thoughts themselves nor in the paper on which they are written. He may sell the paper
to another, but he does not thereby sell the copyright. Unless he expressly assigns copyright, he can stop anyone thereafter
reproducing his words, even after he has sold the piece of paper.
The copyright in the fixture lists has been throughout vested in the Football League Ltd for the Football League Ltd have not
assigned it to anyone. The copyright in the rest of the coupon (namely the compilation of the coupon as a whole) has been
throughout vested in Littlewoods (see a recent case in this court, William Hill v Ladbrookse). When Moores printed the coupons
they were, of course, not guilty of any infringement of copyright. The reason was that they had got the permission of the owners
of the copyright. The permission was given in this way: so far as the fixture lists were concerned, it was derived through
Littlewoods from the Football League, Ltd (see s 49(7) of the Copyright Act, 1956, f). So far as the rest of the coupon was
concerned, Moores, of course, had the permission of Littlewoods, because Littlewoods had commissioned them to print the
coupons. So Moores were not guilty of any infringement of copyright when they printed the coupons. But Moores had no title to
any copyright themselves. They had nothing to sell except the printed coupons with no copyright attached to them in any way.
Counsel for the commissioners argued that the coupons had copyright protection attached to them. He spoke of copyright
protection as if it was a magic formula adding to the value. I beg leave to say that, when Moores printed the coupons, there was
no copyright attached to them, nothing adhered to them which added to their value in the hands of Moores at the time of delivery
to Littlewoods. All that can be said is that Moores were not infringing copyright when they sold the coupons.
________________________________________
e 19 November 1962, not reported
f Section 49(7) is as follows: Where the doing of anything is authorised by the grantee of a licence, or a person deriving title from the
grantee, and it is within the terms (including any implied terms) of the licence for him to authorise it, it shall for the purposes of this Act be
taken to be done with the licence of the grantor and of every other person (if any) upon whom the licence is binding.

Purchase tax is chargeable on the goods. The word goods is defined g as being the same as in the Sale of Goods Act,
1893. By s 62 of the Act of 1893, goods
717
________________________________________
g By s 41(1) of the Finance (No 2) Act, 1940

include all chattels personal other than things in action and money, and in Scotland all corporeal movables except
money.

It seems to me quite plain that the goods here were the coupons, the printed coupons, and the copyright formed no part of them.
The value which is to be ascertained is the value of the printed coupons without the copyright. The value is ascertained by asking
a hypothetical question: what price would those goods fetch on a sale in the open market at the time of delivery by Moores to
Littlewoods if they were sold by wholesale to a retail trader? The section requires the commissioners and the referee and the
courts to make an hypothesisan hypothesis of an open market when there is none, and of a retail trader when there is none
and then to say what the goods would fetch on a sale by wholesale. It is an impossible hypothesis to make. There is no open
market. The only possible purchaser is Littlewoods or a waste paper merchant. There is no retail trader because there is no
trader who would buy these coupons to sell again. Nevertheless the attempt has to be made. The courts have often had to
attempt impossible hypotheses in other branches of the law, and so must they do so here.
In the circumstances I think that the wholesale value can be ascertained only by asking the question: what is the fair price
for Moores to charge Littlewoods for these goods, remembering always that Moores can only sell the coupons without the
copyright. The fair price is, I think, cost plus a reasonable profit. That was the method of ascertaining the wholesale value before
1959 and I think that it remains the right method. Nothing is to be added in respect of the copyright element.
I am confirmed in this view by considering Sch 8 to the Finance (No 2) Act, 1940. Take the patent provision. Paragraph 3
of Sch 8 says:

Where the goods to be valued are goods made in accordance with a patented invention or are goods to which a
registered design has been applied, it shall also be assumed for the purpose of computing the price aforesaid that the buyer
under the sale mentioned in s. 21 of this Act is not the patentee or the proprietor of the design and has not paid any sum or
given any consideration by way of royalty or otherwise in respect of the patent or design and, on payment of the price, will
be entitled to deal with the goods free from any restriction as regards the patent or design.

It is quite clear from that paragraph that if you have an article which is the subject of patent or registered design you are to take
the value as loaded with the added patent element or the design element. It is most significant, however, that that provision has
not been applied to copyright. It may be an oversight of the legislature; I know not. But one thing is clear: Parliament realised
that, unless some such provision was made in regard to the incorporeal rights of patents and designs, the value would not be
loaded with it. Copyright is likewise an incorporeal right. Seeing that Parliament has not thought it right to include copyright in
this schedule, I think that the fair inference is that the value is not loaded with any copyright element, for if Parliament had so
intended, it would surely have provided for it, just as it has with patents and designs.
I much respect the judgment of Megaw J and, indeed, of the referee, but I find myself unable to agree with their conclusion.
I would allow the appeal.

HARMAN LJ. I am in agreement and, like my lord, I am concerned in my own mind for I cannot see what justified in their
minds the decision of the referee and the learned judge below for whose decisions in such matters I have the greatest respect. I
cannot help thinking that the mental gymnastics to which this court has been urged throughout this case, to which standing on
ones head is nothing, are all caused by the fact that purchase tax here has been applied to a subject-matter to which it does not
really fit. These coupons are not articles for trade purposes on hire purchase or sale in the ordinary way. Therefore, there being
no seller and no buyer in the ordinary sense of the word, no wholesaler and no 718 retailer, all these persons have to be conjured
up by counsel for the commissioners out of the air which he then loads with a copyright value which never existed in the seller at
all.
Littlewoods had been carrying on their trade for many years. That trade was to attract the public to make bets on the result
of football matches. For this purpose they disseminated at one time broadcast and afterwards, in a somewhat more disciplined
way, to the public, sheets showing the fixtures for the Saturday, or whatever the day might be in question, and information on the
sheets printed to show the members of the public into whose hands these bits of paper might come where they were to fill in their
noughts and crosses, and how to sign on the dotted line at the bottom. Littlewoods formed for the purpose of printing these
sheets a subsidiary company. They kept the matter in the family. They did not go to an outside printer. As between the two
associated companies it was agreed that, as an accounting matter, the printers should be able to charge their customers (that is
what Littlewoods were) with the cost of printing these sheets, and a profit of twelve and a half per cent on the cost. In 1940 the
commissioners accountants persuaded the printers that these bits of paper were goods which were subject to purchase tax as
being stationery. I gather that they were so classified, because stationery is a bit of paper on which one is asked to fill in
something, or to put down something, and that there was enough blank left in the so-called coupon to make it into a piece of
stationery, and to treat it as goods which were subject to purchase tax accordingly. Whether that was right or not is now, I
suppose, far too late to consider.
When one looks at the Finance (No 2) Act, 1940, one sees h that the tax is to be charged on the wholesale value of all
chargeable goods bought under chargeable purchases. The Act then goes on to say what are chargeable purchases i and what are
chargeable goodsj. One can only work those in here by already starting on a hypothetical career. Never mind. It was agreed
between the commissioners and the printers that the purchase tax should be based on the same figure as appeared in the books of
the two associated companies; namely, cost of production plus twelve and a half per cent, and no doubt a considerable sum was
paid accordingly. So things went on till the year 1959 when, after many years, Football League Ltd woke up to the fact that they
might get a slice of the very large sums flowing into the coffers of the pools promoters. It was a splendid idea. The Football
League Ltd suddenly saw an enormous new income to be gathered from that source. The lists which they published of these
fixtures was copyright material (as indeed it plainly is, if I may say so, with respect to the judge) and a necessary thing for the
pools promoters to use. They could not construct their coupons without it, or set up the competition. The league therefore sued
for breach of copyright and won: as a result if the pools promoters wanted to make use of these lists they had to pay royalty, and
they very soon did.
________________________________________
h See s 18 (1) of the Act of 1940, as substituted by the Finance Act, 1944, s 10
i See s 18 (2) of the Act of 1940, as substituted by the Finance Act, 1944, s 10
j Section 19 (1) of, and Sch 7 to, the Act of 1940, have been repealed; see the the Finance Act, 1958, s 1, Sch 2

Now there enters another power, the Commissioners of Customs and Excise, who conceived that they too might cash in on
this bonanza, and they put forward a proposal that the whole of the value of the royalty paid by Littlewoods to Football League
Ltd should be added to the price of the coupons. The coupon, they said, has increased in value because it has now got a copyright
element in it. Therefore, if it has increased in value it ought to be increased in price. The measure of the increase is the amount
of the royalty. The commissioners thereupon demanded that that amount should be added to the notional price of the coupons
and become subject to tax.
Counsel for the commissioners says (and his argument was described in the court below as unanswerable) that the goods
must be sold in the state in which they are. When asked what he means by that he says that the state in which they 719 are is that
they are goods having copyright protection. This is where I absolutely part company with counsel and those who think like him.
The printers are persons who shelter under the umbrella of their customer, Littlewoods. Littlewoods have obtained from Football
League Ltd a non-exclusive licence to reproduce, and to authorise to be reproduced, the whole or any part of the fixture lists.
Littlewoods authorise to be reproduced by the printers, just as though the printers were their agents, certain parts of the fixture
lists week by week. That means that the printers can print these parts that appear. Otherwise, after the decision of the Chancery
Court, they might have an injunction given against them any week on Friday. What have they got? They have got protection for
themselves. They do not own any copyright. They do not own any rights at all in the matter. All they have is protection. What
have they got to sell to Littlewoods? Exactly the same piece of paper as before. What price are they going to ask? The same
price as before, the cost plus twelve and a half per cent. What is it, I ask, that they have that they could possibly sell to
Littlewoods or anybody else? Nothing. Littlewoods themselves are the owners of the copyright. It is not possible to sell back to
a man that which he already owns. They could not give any other customer any rights in the matter. If they purported to sell
these bits of paper to anyone else, not only would they break their contract with Littlewoods, but the person to whom they sold
would be immediately liable to an action by Football League Ltd for breach of copyright. Therefore, with all respect to those
who differ, I cannot see that this case begins to get on its feet at all. I cannot see that there is any case for the commissioners to
say that they are entitled to, as they call it, the loaded price, or the value, I know not which, of these pieces of paper with some
hypothetical value attached to that, because the printers can print them with impunity for delivery to the licensees of the
copyright.
In my judgment, the claim is quite untenable and this appeal must be allowed.
PEARSON LJ. I agree. At this stage of the case it will be sufficient to state and rely on a short point which, to my mind, has
emerged quite clearly from the careful and comprehensive arguments which have been presented in this appeal, and from the
discussion which has taken place.
The Football League Ltd and the Scottish Football League have copyright in their fixture lists which have been complied, as
was explained to us, with great care and skill. Football pool coupons incorporate parts of these fixture lists, and so would
infringe the copyright unless there were an appropriate licence. In the present case there is an appropriate licence granted by the
Football League Ltd on its own behalf and on behalf of the Scottish League to Littlewoods and other football pool promoters. In
fact the agreement was oral, but its terms are sufficiently set out in a draft written agreement which for some reason was never
executed. That shows, by cl 1, that Football League Ltd on behalf of itself and the Scottish Football League, granted to each of
the pool promotersthere were four of them concerned, and one of them was Littlewoodsa non-exclusive licence to reproduce
and to authorise to be reproduced the fixture lists, or any part or parts of them, for the purpose only of promoting pools conducted
by each such respective pools promoter. The pools promoter concerned in this case is Littlewoods. Littlewoods had the right to
authorise reproduction, and they did authorise reproduction, of parts of the fixture lists in their football pool coupons to be printed
by Moores, who are the complainants in this case. Moores were authorised to produce football pool coupons for use for
Littlewoods pools only and for no other purpose. Moores had a secondary right derived from Littlewoods primary right.
The relevant section is s 21 of the Finance (No 2) Act, 1940. I venture to say that that section means what it says, and has to
be interpreted literally, and has to be applied according to its terms. When so interpreted and applied, it supplies the answer to
the present question. The whole sub-s (1) which applies 720 here has already been read k, so I will extract only the most material
words. It provides that the wholesale value of the goods shall be taken to be the price which the goods would fetch on a sale
made by a person selling by wholesale in the open market to a retail trader. There are several persons or transactions which are
hypothetical, notional and imaginative, but that, to my mind, does not make it difficult to apply this provision. Assume that there
is a person selling by wholesale the goods concerned. Assume that there is an open market, and let it be as open as anyone
pleases. Let there be hundreds of retail traders ready and willing to buy. Then the question arises: what price is any retail trader
in that hypothetical market going to pay for these goods? As appears from the definitions in s 41, a retail trader is a person
selling by retail, and selling by retail means selling goods by way of business otherwise than by wholesale, and selling by
wholesale means selling goods of any class to a person who carries on a business of selling goods of that class. When one works
out the ultimate result of those definitions, I think that it comes to this: that a retail trader is a person who buys for the purpose of
selling to someone else who is normally the ultimate purchaser. Then one has to consider that any one of these supposed retail
traders who buys these goods, these football coupons, is going to sell to such a purchaser. When he decides what price he is
going to pay he must consider: To whom am I going to sell, and what is the end use to which that ultimate purchaser is going to
put the goods?
________________________________________
k Page 717, letter a, ante

For all these goods, which are Littlewoods pools coupons and made by virtue of the non-exclusive licence conferred on
Littlewoods by the Football League Ltd and the Scottish Football League, the only possible end use is as football coupons of
Littlewoods pools. Therefore, the only possible ultimate purchaser is Littlewoods themselves. Now they have, under the
agreement with the Football League Ltd and the Scottish Football League, the primary right to use these football coupons for
their purpose, and they will not be willing to pay something more than they have already paid, the full amount, to the Football
League Ltd and the Scottish Football League. They are not going to be willing to pay something extra to a retail trader who has
bought from the person, Moores, on whom Littlewoods themselves conferred the derivative secondary right. Littlewoods have
already paid for the copyright, they already have the copyright, and they are not going to pay any more for acquiring the same
right over again. Therefore, the retail trader will not be able to afford in the price which he pays to the wholesale seller anything
extra by reason of the alleged copyright protection which these goods are said to enjoy. It is true they enjoy copyright protection,
but only in a very limited sense which would not enhance the price payable in the hypothetical transaction which is envisaged by
s 21(1).
For that reason I agree that the appeal should be allowed.

HARMAN LJ. I should like to add to what I said, that I concur with my lord in his views l about para 3 and para 4 of Sch 8 to
the Finance (No 2) Act, 1940, as to patented goods and goods to which a trade mark applies. It may very well be that the
commissioners thought that they might get tax in this way, because they do it in the case of those goods. What they forgot was
that there is no provision entitling them so to do. The fact that provisions were needed for those purposes, or the legislature
thought that they were needed, is, in my judgment, a very strong pointer to the answer that without such statutory provision no
such claim can be made.
________________________________________
l See p 717, letter c, and p 718, letters e to h, ante

Appeal allowed. Leave to appeal to the House of Lords.

Solicitors: Jaques & Co agents for North, Kirk & Co Liverpool (for the appellants); Solicitor of Customs and Excise (for the
respondents).

F Guttman Esq Barrister.


721
[1963] 2 All ER 722

Inland Revenue Commissioners v Coutts & Co


TAXATION; Estate Duty

HOUSE OF LORDS
LORD REID, LORD JENKINS, LORD MORRIS OF BORTH-Y-GEST, LORD HODSON AND LORD GUEST
2, 3, 4, 8, 9, 10 APRIL, 29 MAY 1963

Estate Duty Exemption Property which has borne duty on death of spouse Derivative settlement Marriage settlement
followed by derivative settlement by widow of defeasible, but otherwise absolute, interest in settled estates Estate duty paid on
death of husband on settled estates comprised in marriage settlement Death of widow Passing of property settled by
derivative settlement Whether exemption in respect of estate duty on settled estates extended to duty on the passing of the
defeasible interest on the widows death Finance Act, 1894 (57 & 58 Vict c 30), s 5(2).

By a marriage settlement dated 12 May 1914, the Revesby Estates were settled on trusts, as events happened, for S for life with
remainder to the second and younger sons of a brother of S, with ultimate remainder to S S died in 1916, leaving his defeasible
interest in the remainder (ie, his ultimate remainder in default of a second son being born to his brother) to his widow, B. Estate
duty was paid on Ss death. B conveyed the defeasible interest that she was given by S to trustees, settling it by a settlement
made in 1923 on discretionary trusts during her life, and on further trust under which, after Bs death in 1957, her daughter
became entitled to a life interest. Ss brother survived B; at her death he was a widower and had never had a son.
Estate duty was claimed in respect of a passing of the defeasible interest comprised in the 1923 settlement on the death of B,
the respondent trustees contending against this that, duty having been paid on Ss death in respect of the whole settled estates, the
payment now claimed was not exigible, since by s 5(2) of the Finance Act, 1894, duty would not again be payable in respect
thereof, viz, of the settled estates, and that this exemption extended to duty on the defeasible interest comprised in the 1923
settlement.

Held Although the property that passed on the death of B was the property settled by the 1923 settlement, viz, Ss defeasible
interest in the settled estates, not the whole interest in those estates, yet the exemption conferred by s 5(2) of the Finance Act,
1894 (viz, that estate duty should not be payable in respect of the property settled by the 1914 settlement) applied, because estate
duty payable on the passing of the defeasible interest was duty payable in respect of the property settled by the 1914 settlement
for the purposes of s 5(2), it being irrational that, if payment of duty in respect of a whole should exempt a subsequent passing of
the whole, it should not also exempt the subsequent passing of something less than the whole (see p 728, letter i, to p 729, letter
a, p 731, letter g, and p 732, letter d, post).
Re Studdert ([1900] 2 IR 281) and Inland Revenue Comrs v Priestley ([1901] AC 208) considered and applied.
Decision of the Court of Appeal ([1962] 2 All ER 521) affirmed.

Notes
As to exemption for prior payment of estate duty on settled property, see 15 Halsburys Laws (3rd Edn) 49, 50, para 97; and for
cases on the subject, see 21 Digest (Repl) 44, 179192.
For the Finance Act, 1894, s 1, s 2(1)(a), s 2(1)(b), s 5(2), and s 7(7), see 9 Halsburys Statutes (2nd Edn) 348, 350, 356,
364.

Cases referred to in opinions


A-G v Dodington [1897] 2 QB 373, 66 LJQB 684, 77 LT 299, 61 JP 644, 21 Digest (Repl) 45, 179.
Christie v Lord Advocate [1936] 1 All ER 443, [1936] AC 569, 105 LJPC 65, 154 LT 441, 21 Digest (Repl) 19, 66.
Coutts & Co v Inland Revenue Comrs [1953] 1 All ER 418, [1953] AC 267, [1953] 2 WLR 364, 21 Digest (Repl) 21, 74.
722
Cowley (Earl) v Inland Revenue Comrs [18959] All ER Rep 1181, [1899] AC 198, 68 LJQB 435, 80 LT 361, 63 JP 436, 21
Digest (Repl) 11, 36.
Inland Revenue Comrs v Priestley [1901] AC 208, 70 LJPC 41, 84 LT 700, affg sub nom Re Studdert [1900] 2 IR 281, 400, 21
Digest (Repl) 15, 51.
Midland Bank Executor & Trustee Co Ltd v Inland Revenue Comrs [1959] 1 All ER 180, [1959] Ch 277, [1959] 2 WLR 77, 21
Digest (Repl) 13, 41.
Public Trustee v Inland Revenue Comrs [1960] 1 All ER 1, [1960] AC 398, [1960] 2 WLR 203, 21 Digest (Repl) 17, 58.
Studdert, Re see Inland Revenue Comrs v Priestley supra.

Appeal
This was an appeal by the Commissioners of Inland Revenue from an order of the Court of Appeal (Lord Evershed MR Upjohn
and Diplock LJJ) dated 17 April 1962, and reported [1962] 2 All ER 521, affirming an order of Buckley J dated 22 June 1961 and
reported [1961] 3 All ER 158, determining that property comprised in a settlement dated 14 December 1923, was exempt from
estate duty on the death in 1957 of the principal beneficiary under a life discretionary trust, except as regards a slice of the
property not material to this report. The facts appear in the opinion of Lord Reid.

E I Goulding QC and K J T Elphinstone for the appellants.


Lionel Edwards QC and J P F E Warner for the respondents.

Their Lordships took time for consideration

29 May 1963. The following opinions were delivered.

LORD REID. My lords, Lady Beryl le Poer Trench married Captain Stanhope in 1914 when he made a settlement of his
Revesby estate, which I shall call the 1914 Settlement. He was killed in action in 1916. Thereafter she married Lieutenant
Commander Gilbert, RN, and their daughter is now Mrs Lee. In 1923 Lady Beryl made a settlement which I shall call the 1923
Settlement. In 1931 she married again and she died in 1957. The respondents (herein called the taxpayers) are the trustees
under both settlements, and in 1961 an originating summons was issued on their application to determine whether the property
comprised in the 1914 settlement or any interest therein was chargeable with estate duty in connexion with the death of Lady
Beryl.
Under the 1914 settlement in the events which happened the settled estate was held for Captain Stanhope during his life with
remainder to the second and younger sons of the Earl of Stanhope his brother and ultimate remainder to himself. Lord Stanhope
has survived Lady Beryl but has never had a second son a. So Captain Stanhope was entitled at his death to dispose of an
absolute interest in the estate subject to defeasance in the event of a second son being born to Lord Stanhope. He bequeathed that
interest to his widow and she by the 1923 Settlement conveyed it to be held on discretionary trusts during her life and on her
death for Mrs Lee for her life. I need only mention that under the 1914 Settlement Lady Beryl was entitled to 200 per annum
pin-money during her husbands life and thereafter to a jointure of 1,500 reduced on her second marriage to 1,000 per annum.
No question now arises about this.
________________________________________
a At her death he was a widower and had never had a son; Lord Stanhope was born on 12 November 1880

On Captain Stanhopes death estate duty was paid on the whole of the Revesby estate except a slice appropriate to cover
the pin-money. Admittedly duty is now due on that slice and I need not consider it further. The question raised in this appeal is
whether, as regards the rest of the estate or any interest in it, any estate duty is now payable by reason of Lady Beryls death. The
723 taxpayers maintain that s 5(2) of the Finance Act, 1894, applies to relieve them. That subsection is as follows:
(2) If estate duty has already been paid in respect of any settled property since the date of the settlement, the estate
duty shall not, nor shall any of the duties mentioned in the fifth paragraph of Sch. 1 to this Act, be payable in respect
thereof, until the death of a person who was at the time of his death or had been at any time during the continuance of the
settlement competent to dispose of such property

The application of that subsection is now limited by s 14 of the Finance Act, 1914, but that limitation does not touch the present
case and it is not disputed that the subection must be construed, where it still applies, without reference to this later limitation.
The subsection first requires that estate duty shall have been paid since the date of the settlement in respect of the settled
property. That requirement is admittedly satisfied: estate duty was paid on the principal value of the property after Captain
Stanhopes death in 1916. Then there come the exempting words to which I must return; and finally the exemption is to last until
the death of a person who had been competent to dispose of such property. Such property must mean the settled property
mentioned at the beginning, ie, the Revesby estate and Lady Beryl was not competent to dispose of it. She could not prevent the
property being taken by a second son of Lord Stanhope if and when he came into existence. So the exemption applies on her
death if the exempting words are wide enough to cover this case.
The exemption applies to any claim for estate duty in respect thereof, ie, in respect of the settled property, the Revesby
estate. The Crowns claim is set out in their notice of appeal to the Court of Appeal. The first part of the claim is in respect of the
pin-money and I need not set it out because it is admitted. Then comes the claim which is in dispute:

(3) that estate duty is also chargeable in connexion with the death of (Lady Beryl) in respect of an absolute interest
subject only to the entailed interests of future younger sons of the Right Honourable James Richard Earl Stanhope, K.G.,
under the said settlement dated May 12, 1914, in all the property held upon the trusts of that settlement at the date of the
death of [Lady Beryl].
(4) that the plaintiffs are liable for such duty as trustees of the said settlement dated Dec. 14, 1923.
(5) provided that by reason of s. 7(10) of the Finance Act, 1894, estate duty is not chargeable in respect of the said
qualified or defeasible interest in so much of the said property as is itself chargeable with estate duty as aforesaid

(ie the pin-money slice). The Crowns contention is that a claim in respect of an absolute interest in the settled property subject
to defeasance is not a claim in respect of the settled property, and is therefore not within the scope of the exemption. Anyone
ignorant of the wealth of authority that has encrusted s 1, s 2 and s 5 of the Act of 1894 might be pardoned for thinking that this is
a short and simple question of construction of the phrase in respect of in this context, and he might think it strange that the
statute should give exemption from the larger claim in respect of the whole property but withhold exemption from a smaller
claim in respect of an interest in that property. But the question cannot now be treated as either short or simple.
The taxpayers have two alternative answers. First they say that, although Lady Beryl had only a defeasible interest in the
settled property, yet on her death the settled property itself passed: if that is right it is not disputed that the exemption would
apply. Secondly they say that in respect thereof means in respect of the settled property or of any interest in it: again, if that is
right, it is not disputed that the exemption applies. So I proceed to consider 724 what it was that passed on Lady Beryls death.
Was it the settled estate or was it the defeasible interest in that estate which belonged to Lady Beryl and was settled by her in
1923?
There is no authority directly in point and I am afraid that I cannot avoid going back to s 1 and s 2 of the Finance Act, 1894.
They are:

1. In the case of every person dying after the commencement of this Part of this Act, there shall, save as hereinbefore
expressly provided, be levied and paid, upon the principal value ascertained as hereinafter provided of all property, real or
personal, settled or not settled, which passes on the death of such person a duty, called Estate duty, at the graduated rates
hereinafter mentioned, and the existing duties mentioned in Sch. 1 to this Act shall not be levied in respect of property
chargeable with such estate duty.
2.(1) Property passing on the death of the deceased shall be deemed to include the property following, that is to say:

(a) Property of which the deceased was at the time of his death competent to dispose;
(b) Property in which the deceased or any other person had an interest ceasing on the death of the deceased, to the
extent to which a benefit accrues or arises by the cesser of such interest;

As has often been pointed out, property passing was not a term of art in 1894. The word is not defined in the Act and its
meaning must be found from its context in the Act. In the case of unsettled property both title and possession change hands on
the death of the owner, but the title to settled property remains unchanged in the trustees and what changes hands is the beneficial
enjoyment. As Lord Macnaghten pointed out in Earl Cowley v Inland Revenue Comrs ([18959] All ER Rep 1181 at p 1188;
[1899] AC 198 at p 212) in the common case of one life-tenant succeeding another the life-interest of the former ceases on his
death for good and all.

You might just as well strike the word settled out of s. 1 altogether as hold that the section does not embrace a case
where a limited interest under a settlement ceases on death and passes to no one.

Cowleys case must now be read in light of what was said in Public Trustee v Inland Revenue Comrs. That case decides that s 1
and s 2 are not mutually exclusive. Section 2(1)(b) applies where a life-interest ceases and another life-tenant comes into
possession: but still the settled property passes. But first you have to determine what was the settled property. In Cowleys case
the settlors had mortgaged the Mornington estates before resettling them, and it was held that the property which passed was not
the Mornington estates but the equity of redemption, because that and that alone had been settled and had been enjoyed by the
deceased life-tenant.
Applying that to the facts of this case, what happened on Lady Beryls death was that the enjoyment of the property settled
in the 1923 Settlement changed hands. The discretionary trust ceased and Mrs Lee became entitled. The property comprised in
the 1923 Settlement was not the Revesby estate. Lady Beryl had no power to settle that estate. It remained under the 1914
Settlement. What Lady Beryl had and what she settled in 1923 was her defeasible interest in it and it was the enjoyment of that
settled property which passed on her death. That settled property would have vanished if a second son had been born to Lord
Stanhope.
On what ground then do the taxpayers maintain that it was the Revesby estate which passed on Lady Beryls death? In the
first place they say, and the Crown do not deny, that liability for estate duty would have been the same 725 if Lady Beryl had not
made the 1923 settlement but had died possessed of her defeasible interest. So I must consider what the position would then have
been. The defeasible interest which she owned would simply have passed to her executor. Why then should we hold that
something greater passed than what the executor in fact took? I find nothing in the Act or in any authority in this House which
even suggests, let alone requires, that we must disregard the property which in fact passed and hold that something else passes or
must be deemed to have passed. And so to hold could produce a most unjust result. In the present case the possibility of a
second son being born to Lord Stanhope may be remote, and the distinction between the defeasible interest and absolute
ownership may seem shadowy; but in the case of a much younger man, who already had one son, the probability of defeasance
would be great and the value of the defeasible interest would be small. It would be most inequitable to make the passing of such
a defeasible interest the occasion for payment of duty, perhaps at a high rate, on the whole value of the estate: the duty might be
several times as much as the value of the interest which passed. Things of that kind do happen in the realm of estate duty but
only where the terms of the Act leave no alternative. It is to be noted that in this case it is the Crown who contend for the
reasonable result, and the taxpayers who contend for the unreasonable result, because on the facts of this case they might gain a
collateral advantage and would suffer no hardship.
The taxpayers case is really based on a false analogy between what happened in this case and what happens when a life-
interest ceases. As I have already said, when a life-interest ceases no right of property passes at all. The right of the deceased
life-tenant comes to an end and another right formerly in expectancy becomes a right in possession. These two events are treated
quite differently in the Act. The taxpayers found on the decision in this House in Christie v Lord Advocate. There Mrs Norman
was entitled under her fathers trust disposition and settlement to one-tenth of the income of the trust estate, but on the death of
her mother her right was to be restricted to a right to receive 600 per annum. She died before her mother and on her death her
children became entitled under the trust disposition and settlement to the same partially defeasible right. When she died her share
of the trust income was 2,800 so the children took an immediate right to 2,800 per annum which was due to be restricted to
600 per annum on the death of their grandmother. The Revenue claimed duty on the one-tenth part of the trust estate which was
producing the income of 2,800. The case was made difficult and complicated by the then accepted doctrine in Cowleys case
that, if s 1 of the Finance Act, 1894, applied, s 2(1)(b) could not apply. In light of what we now know from the Public Trustee v
Inland Revenue Comrs it would be perfectly straightforward. By reason of the cesser of Mrs Normans interest s 7(7) would
apply so that duty must be paid on the principal value of the property which yielded the income of 2,800; and that was what this
House decided, although they put their decision on a passing of the tenth part of the trust estate under s 1.
Where an interest ceases, whether it be a life-rent or other interest ceasing on the death of the deceased, and a benefit
thereby accrues to another beneficiary who becomes entitled to the income previously enjoyed by the deceased, the Act takes no
account of the fact that the real value of the interest which ceased or the benefit which accrued may be very much less than the
capital value of the property which yields the income. It does not matter that the interest which ceased would have come to an
end at an early date even if the deceased had survived. I ventured to draw attention to the injustice which this can involve in
Coutts & Co v Inland Revenue Comrs. But there is no reason to extend that 726 injustice to a case such as the present would
have been if Lady Beryl had not settled her defeasible interest. Then no interest would have ceased on her death: her interest
would have passed to her executor. Then s 7(5) would have applied and duty would have been payable on the capital value of
that defeasible interest. So I reject the argument based on analogy between the cesser of an interest which comes to an end on the
death of the deceased and the passing to the executor of an interest which does not come to an end on the deceaseds death.
There was much argument about the case of Midland Bank Executor & Trustee Co Ltd v Inland Revenue Comrs and I must
deal with it. The facts were very complicated, but I think that it is sufficient to say that a testator had directed accumulation of
the income of his estate until 1958. But in 1941 all who had an interest made an agreement with regard to the income during the
remainder of the period of accumulation. The case was decided on the basis that they had made a revocable settlement of this
income whereby one-fifth of it became payable to Geoffrey Harrison during his life. He died in 1955 and his right under this
derivative settlement came to an end. Under the testators will he had only an interest in expectancy which might have come into
possession in 1958. It was admitted that some estate duty was payable in respect of his deathbut the question was on what was
it payable? The Revenue claimed that duty was payable on the one-fifth part of the testators estate which yielded the income
which Geoffrey had enjoyed for some fourteen years before his death, but this claim did not succeed. It was held that the
property which passed was the property comprised in the derivative settlement from which alone came Geoffreys right to the
income which he had been receiving. That property was the right to receive the income between the date of the settlement and
1958 and nothing else. The fact that the income came to the settlement from the testators estate was irrelevant.
Applying that to Lady Beryls 1923 Settlement it would mean that, when the income from that settlement changed hands on
her death, what passed was the property comprised in the 1923 Settlementthe defeasible interestand not the Revesby estate
from which the income originated. I have already said that in my opinion that is right. But I must take the matter a little further.
My noble and learned friend Lord Jenkins gave some examples in his judgment ([1959] 1 All ER at p 188; [1959] Ch at p 289). I
need only notice the two simplest. A settles property on B for life with remainder to Bs children. B then either assigns his life-
interest to C or settles it on X for life with remainder to Xs children. Then C or X dies in Bs lifetime. My noble and learned
friend thought that in either case duty would only be exigible in connexion with the death of C or X on the value of the residue of
Bs life-interest at the death of C or X and would not be exigible on the corpus of the whole estate settled by A. This was not
attacked by counsel on either side in this case and I see no reason to differ from my noble and learned friend. Counsel sought to
distinguish such cases from the present case on the ground that all the interests mentioned in the Midland Bank case were bound
to come to an end some day whereas Lady Beryls interest might never come to an end. I do not see any distinction in principle,
and in practice a defeasible interest might end next year, whereas a life-interest might last half a century or more. I am therefore
of opinion that when Lady Beryl died what passed was her defeasible interest and not the property comprised in the 1914
settlementthe Revesby estate itself.
Before leaving this part of the case I should note that counsel for the taxpayers argued that to decide against his contention
would open the door to tax evasion. But counsel for the Crown made light of this suggestion, and if the Crown see no substantial
risk of evasion I do not think that we need pursue the topic.
Now I must turn to the taxpayers alternative argument that, although the claim for duty is only a claim based on the passing
of an interest in the settled 727 property, it is nevertheless a claim for duty payable in respect of the settled property within the
meaning of s 5(2). In respect of is not at all a precise phrase. It is used several times in the Finance Act, 1894, and one
naturally first inquires what it means in other sections. But little help is to be got there. I doubt very much whether it always
means the same thing: the context seems to require a narrower meaning in s 9 than in s 7 (10). Then what would be the
reasonable meaning?
The Crown say that the exemption can only have been intended to apply to devolutions under the settlement itself and
cannot have been intended to apply when an interest in the settled property passes under a disposition by the owner of that
interest. I have been unable to discover any particular reason for that limitation of the scope of the subsection. If it applies at all
when only an interest in the settled property passes and the settled property itself does not pass then I think it must apply
generally. So the question must be whether it ever applies when only an interest passes and the settled property itself does not
pass. If estate duty has been paid in respect of the whole property and there is to be any subsequent exemption at all it would
seem quite irrational that the exemption should apply to the larger claim for duty when the whole property passes but not to the
smaller claim when something less than the whole property passes.
The taxpayers found on the decision in Inland Revenue Comrs v Priestley reported in Ireland as Re Studdert. There a wife
settled her property on her husband for his life then to herself for her life and in default of issue to such persons as she might
appoint or her next-of-kin. She predeceased her husband without issue and appointed the property to relatives. Section 7(6) of
the Act of 1894 applied, the persons accountable chose to pay duty immediately and duty was paid on the value of the settled
property less the value of the husbands life-interest. Then the husband died and as he was in receipt of the whole income the
whole property passed. The Revenue claimed duty in respect of this passing but it was held that s 5(2) applied and that no duty
was payable on the husbands death.
I find it difficult to determine precisely the ratio decidendi in this House but in my view it is unnecessary to go beyond the
decision itself for the purposes of the present case. Section 5(2) cannot apply unless estate duty has already been paid in respect
of the settled property. In Priestleys case the whole interest in the settled property did not pass on the wifes death. Her interest
was an indefeasible interest but nevertheless an interest in expectancy because her husbands life-interest was unaffected by her
death and did not pass on her death. And the duty paid on her death was not calculated on the principal value of the settled
property but on the principal value less the value of the surviving husbands life-interest. So the decision necessarily involved a
decision in law that a payment of duty in respect of something less than the whole interest in the settled property and based on
something less than the principal value of the property can nevertheless be a payment of duty in respect of the settled property
within the meaning of s 5(2). That proposition may not appear very clearly from what was said in this House, but I think that it
does appear reasonably clearly from the judgments of Palles CB and Holmes LJ which were approved in this House. The only
distinction that I can see between Priestleys case and the present case is this. In Priestleys case payment of duty in respect of
less than the whole interest in the settled property was effective to exempt from a later claim for duty in respect of passing of the
whole property. In this case the question is whether payment of duty in respect of the whole of the settled property on Captain
Stanhopes death is effective to exempt from a later claim in respect of the passing of less than the wholea defeasible interest.
If payment on the passing of less than the whole exempts from a later claim on the passing of the whole, I can see no possible
ground on which it could be held that payment in respect of 728 the whole does not exempt from a later claim in respect of the
passing of less than the whole. So I am of opinion that on this part of the case the taxpayers are right and therefore this appeal
must be dismissed.
My Lords, my noble and learned friend Lord Morris Of Borth-Y-Gest is unable to be present this morning and he has asked
me to say that he has read the speech which I have just delivered and that he agrees with it.

LORD JENKINS. My Lords, this is an appeal from a judgment of the Court of Appeal dated 17 April 1962, affirming by a
majority (Lord Evershed MR and Upjohn LJ, Diplock LJ dissenting) a judgment of Buckley J dated 22 June 1962, which decided
adversely to the Crown the question whether the property comprised in a settlement dated 12 May 1914, and made on the
marriage of Captain Stanhope and Lady Beryl Groves (both since deceased) and in a voluntary settlement made by Lady Beryl on
14 December 1923, or any interest therein, was chargeable with estate duty in connexion with the death of Lady Beryl, and if so
for the determination of the extent (if any) of the plaintiffs liability for such duty as trustees of the settlements of 1914 and 1923
and as executors of Lady Beryl.
The facts are sufficiently summarised in the following extract from the headnote to a report of the case in transcribing which
I have substituted names for initial letters where appropriate:

By a settlement dated May 12, 1914 (hereinafter called the 1914 settlement), made on the marriage of Lady Beryl
Groves and The Hon. Captain Richard Philip Stanhope, certain hereditaments (the settled estates) were settled in the
events that happened (a) to the use that Lady Beryl should receive a yearly rentcharge of 200 during the joint lives of
herself and Captain Stanhope, and subject thereto, (b) to the use of Captain Stanhope for life with remainder (c) to uses to
secure a yearly rentcharge of 1,500 (reducible to 1,000 on remarriage) to Lady Beryl for life if she survived Captain
Stanhope and subject thereto to trustees (d) on trust for the second and subsequent sons of The Right Hon. James Richard
Earl Stanhope successively in tail male (hereinafter called the paramount limitations) with remainder (e) on trust for
Captain Stanhope in fee simple. On Sept. 15, 1916, Captain Stanhope was killed in action, having by his will devised and
bequeathed his residuary estate to Lady Beryl absolutely; his fee simple interest in remainder defeasible on the birth of a
second son to the Right Hon. James Richard Earl Stanhope forming part of such residue. On his death, estate duty was
paid in respect of the whole of the settled estates less the slice referable to the yearly rentcharge of 200. On July 15,
1917, Lady Beryl remarried and on Dec. 14, 1923, executed a voluntary settlement whereby she settled all her interest in
the settled estates, subject to the yearly rentcharge of 1,000 and the paramount limitations: (a) on trust during a term of 99
years terminable on her death to hold the annual income on discretionary trusts for the benefit of herself and her issue and,
subject thereto, in the events that happened (b) on trust for her daughter Mrs. Lee for life with remainder (c) on trust for
Mrs. Lees sons in tail male with remainders over. On May 1, 1957, Lady Beryl died leaving her daughter Mrs. Lee and
Mrs. Lees infant son surviving her, and a claim was made for duty under the 1923 settlement in respect of an absolute
interest subject only to the paramount limitations in all the property (the settled estates) held on the trusts of the 1914
settlement at Lady Beryls death. At the date of the proceedings The Right Hon. James Richard Earl Stanhope, aged 80,
was still alive but had never had a son.

In these circumstances the trustees (the taxpayers) applied to the court 729 for the determination of the question whether the
exemption from estate duty afforded by s 5(2) of the Finance Act, 1894, was applicable to the case. It is necessary at this point to
refer to s 5(2) of the Act of 1894 which is in these terms:

(2) If estate duty has already been paid in respect of any settled property since the date of the settlement, the estate
duty shall not, nor shall any of the duties mentioned in para. 5 of Sch. 1 to this Act, be payable in respect thereof, until the
death of a person who was at the time of his death or has been at any time during the continuance of the settlement
competent to dispose of such property.

It will next be convenient to deal with the relatively minor matter of the pin-money slice, which was disposed of by Lord
Evershed MR in the following passage from his judgment ([1962] 2 All ER at p 523; [1963] Ch at p 161):

In my view it is not, for the purposes of this present case, necessary for this court to give an expository definition of
the word paid as used in the subsection. For the purposes of the present case it is sufficient to say that, upon the death in
1916 of Captain Stanhope, estate duty was not in fact paid nor regarded as payable in respect of the relevant slice of the
settled estate: with the inevitable result that quoad such slice, the exemption provisions of s. 5(2) of the Act of 1894 can,
upon Lady Beryls death, have no application.

I entirely agree with the reasoning and conclusions of the Master of the Rolls, Upjohn LJ ([1962] 2 All ER at pp 527, 528; [1963]
Ch at pp 165167), and Buckley J ([1961] 3 All ER at p 171; [1962] Ch at pp 328, 329), on this part of the case and need not
enlarge on it further, except to mention that counsel for the Crown conceded that the cesser of the 1,000 jointure rentcharge on
the death of Lady Beryl was exempt for the reason that the slice of income which it represented paid duty on the death of
Captain Stanhope.
I should next return to the main issue, that is to say, the question whether (apart from the pin-money slice) duty was
exigible on the death of Lady Beryl in respect of the Revesby Estates or some interest therein notwithstanding s 5(2) of the Act of
1894. Continuing the argument in the Court of Appeal counsel for the taxpayers contended that there was a passing of the
whole of the Revesby Estates on the death of Lady Beryl, as seems to have been the case according to the commonly accepted
meaning in estate duty language of the operation of changing hands; but that if that view was accepted there must inevitably be
an exemption under the terms of s 5(2), this being the very property which paid duty on the death of Captain Stanhope. Upjohn
LJ found it impossible to accept this argument, and I do not propose to pursue it further ([1962] 2 All ER 528; [1963] Ch at p
168).
There was considerable discussion in the Court of Appeal of the case of Midland Bank Executor & Trustee Co Ltd v Inland
Revenue Comrs, leading Upjohn LJ to say ([1962] 2 All ER at p 529; [1963] Ch at p 170) that he rejected the view that there
was any passing of the estates as a whole on the death of Lady Beryl and that he thought it could not be denied that as the estates
as a whole had not passed there had been a passing of the defeasible interest on the death of Lady Beryl. A little later in his
judgment ([1962] 2 All ER at p 530; [1963] Ch at p 171) Upjohn LJ imputes to counsel for the Crown an argument on the
following lines:

The defeasible interest is a new or different property which has not previously passed under the 1914 Settlement and
on which duty has not, therefore, previously been paid. Put in a slightly different phraseology but 730 to the same legal
effect counsel for the Crown [as interpreted by UPJOHN, L.J.], says that the only property that passed on Lady Beryls
death was the defeasible interest, and that is a property which she had been competent to dispose of and indeed did dispose
of in 1923. Therefore [according to this part of counsel for the Crowns argument] the final condition of s. 5(2) is not
satisfied.

A little later Upjohn LJ said, with regard to counsel for the Crowns argument to which he had just referred and a counter
argument put forward by counsel for the taxpayers ([1962] 2 All ER at p 530; [1963] Ch at p 171):

I cannot accept the Crowns argument. In my judgment the natural meaning of the words of the subsection that after
payment of duty on the settled property on the first life, duty is not again payable in respect thereof is that duty is not
payable on that property or in respect of any interest therein.

Upjohn LJ went on to develop this aspect of the argument, referring in particular ([1962] 2 All ER at p 531; [1963] Ch at p 172)
to Palles CBs judgment in the Irish case of Re Studdert ([1900] 2 IR 281 at pp 283290), which came before the House of Lords
as Inland Revenue Comrs v Priestley and there received full approval. Reference should also be made to Attorney General v
Dodington and in particular to the judgment of Rigby LJ ([1897] 2 QB at p 381) which placed a wide construction on the word
disposition and the phrase in respect of which and was specially commended in your lordships House in the Priestley case.
In conclusion I would quote the following passage from the judgment of Buckley J as providing a clear and succinct
statement of the case as it stood before him ([1961] 3 All ER at pp 170, 171; [1962] Ch at p 328):

Lady Beryl was not, at her death, or at any time during the subsistence of the settlement competent to dispose of any
part of the settled estates. However, in consequence of the 1923 settlement of Lady Beryls qualified interests in the settled
estates a claim for duty will, unless s. 5(2) is applicable, have arisen on her death in relation to the interests comprised in
the 1923 settlement. If that duty could, if exigible, properly be described as payable in respect of the settled estates,
exemption will be available under the subsection. It appears to me, on the authority of Inland Revenue Comrs. v. Priestley
([1961] 3 All ER at pp 170, 171; [1962] Ch at p 328), that the duty in question can be properly so described. It follows
that, in my judgment, the subsection is applicable and the claim for exemption is well founded.

I should add that I find myself in complete agreement with the opinion just delivered by my noble and learned friend Lord
Reid.
I would accordingly dismiss this appeal.

LORD HODSON. My Lords, I also agree with the opinion of my noble and learned friend Lord Reid.

LORD GUEST. My Lords, two questions were debated before this House. First, what was the nature of the property which
passed under s 1 of the Finance Act, 1894, on Lady Beryls death in 1957. Second, whether the exemption under s 5(2) of the Act
of 1894, as amended by s 14 of the Finance Act, 1914, is available to the respondents.
On the first question I have had the advantage of reading the speech of my noble and learned friend Lord Reid and I agree
for the reasons given by him that on Lady Beryls death the defeasible interest passed. On this view it then becomes necessary to
consider the application of s 5(2) of the Finance Act, 1894. 731This involves a pure question of the construction of the terms of
this section, and this depends, as Buckley J (See cf [1961] 3 All ER at p 170; [1962] Ch at p 327), said on by which door you
enter. It was matter of admission that on Captain Stanhopes death the whole Revesby estates passed and that Lady Beryl was
never competent to dispose of these estates. Two different approaches were suggested. The crown argued that as estate duty was
claimed on the defeasible interest and as Lady Beryl was in terms of the section competent to dispose of the defeasible interest
and did in fact dispose of them by her 1923 settlement, s 5(2) was not applicable, even though the property on which estate duty
was paid in 1916 on Captain Stanhopes death was the whole Revesby estates. The respondents (the taxpayers) on the other hand
contended that the section must be read from the beginning, that estate duty had already been paid on the Revesby estates, that
the claim for estate duty on the defeasible interests was a claim for estate duty in respect of the Revesby estates and that as Lady
Beryl was never competent to dispose of the Revesby estates, s 5(2) was applicable. Thus stated, the short question, by no means
easy of answer, is the meaning of in respect of in s 5(2). If it is confined to the precise property on which estate duty was paid
on the first death, then the taxpayers contention fails. But the phrase in respect of denotes some imprecise kind of nexus
between the property and the estate duty. Estate duty may, in my opinion, truly be said to be payable in respect of the Revesby
estates when estate duty is claimed in respect of the defeasible interests only. In the present case estate duty was paid on the
whole Revesby estates and the claim is made in respect of an interest in these estates which was defeasible. I incline to the
construction put forward by the taxpayers. Some support for this may be obtained from the case of Re Studdert reported in the
House of Lords sub nom Inland Revenue Comrs v Priestley. In that case property was settled on trust for B for life and after his
death for his wife A for life and after the death of the survivor in the event which happened in trust as A should appoint. A
predeceased B and on her death estate duty was paid on her reversionary interest. The courts in Ireland held s 5(2) to apply on
Bs death. This decision was affirmed by the House of Lords. I quote from Palles CB ([1900] 2 IR at p 288):

Assuming, then, that the payment of 97 1s. 4d. was made in respect of a partial interest in the settled property, I am
still of opinion that that payment was in respect of the settled property within s. 5(2), because, on ordinary principles of
construction, the whole must include a part, and because the taxing section (i.e., s. 1), by using the words save as
hereinafter expressly provided, renders it necessary to construe s. 5(2) as part of that taxing section; and if, on the true
construction of both together, the case is not within the words of the enactment the omission cannot be supplied. In other
words, if we read settled property so as to exclude a part of it, we, for the purpose of extending taxation, omit to give to
that expression its full forcea construction which I hold to be impossible in a taxing enactment.

Holmes LJ in the Court of Appeal in Ireland [[1900] 2 IR 400 at p 413] said:

Therefore no estate duty was payable on that event if estate duty had already been payable in respect of such property.
There only remains the narrow pointwhether, within the meaning of the subsection, duty was so payable upon the death
of Mrs. Studdert. It is, in my opinion, quite clear that it was. If it were not payable in respect of the property comprised in
the settlement, in respect of what was it payable? It is said on behalf of the Crown that it was only payable upon a
reversionary interest in such property; but it was not for that reason the less payable in respect of the property comprised in
the settlement.
The observations of Palles CB were expressly approved by the Lord 732 Chancellor, the Earl Of Halsbury, in the House of Lords
([1901] AC at p 211).
These observations are in point in the present case. If the whole includes the part, then payment of estate duty on the
defeasible interest is payable in respect of the Revesby estates. The present case is really a fortiori of Priestley in that payment of
estate duty on the whole Revesby estates enfranchises the claim for estate duty payable on the passing of the defeasible interest.
I would dismiss the appeal.

Appeal dismissed.

Solicitors: Solicitor of Inland Revenue (for the Crown); Caprons & Crosse (for the taxpayers).

C G Leonard Esq Barrister.


[1963] 2 All ER 733

Church of Jesus Christ of Latter-Day Saints v Henning (Valuation Officer)


LOCAL GOVERNMENT

HOUSE OF LORDS
LORD REID, LORD EVERSHED, LORD MORRIS OF BORTH-Y-GEST, LORD DEVLIN AND LORD PEARCE
27, 28 MARCH, 30 MAY 1963

Rates Exemption Place of public religious worship Mormon temple Use for special ceremonies Admission only to
Mormons of good standing Rating and Valuation (Miscellaneous Provisions) Act, 1955 (4 & 5 Eliz 2 c 9), s 7(2).

In addition to chapels for ordinary services open to all members of the public of any denomination, the Mormon Church
possessed temples, of which there was only one in England; this was not open to the public nor to every Mormon, only to
Mormons of good standing. A Mormon of good standing was one whose spiritual and secular qualities entitled him in the view
of a bishop to a recommend to a president; if the president were satisfied that the recipient of the recommend were entitled to
the appellation by his personal qualities, he indorsed the recommend, but until the indorsement was appended the recipient had
no right to enter the temple. The temple was used for the performance of ceremonies that were sacred, but not secret. About five
thousand Mormans in good standing attended ceremonies at the temple each year.

Held The temple was not exempt from rates under s 7(2) of the Rating and Valution (Miscellaneous Provisions) Act, 1955,
because the words places of public religious worship therein did not include places which, though they were from the
worshippers point of view public as opposed to domestic, yet in the more ordinary sense were not public since the public were
excluded (see p 741, letters c and g, post).
Barnes v Shore ((1846), 1 Rob Eccl 382), Freeland v Neale ((1848), 1 Rob Eccl 643) and Nesbitt v Wallace ([1901] P 354)
distinguished.
Dictum of Lowe J in Association of the Franciscan Order of Friars Minor v City of Kew ([1944] VLR 199) approved.
Decision of the Court Of Appeal (sub nom Henning (Valuation Officer) v Church of Jesus Christ of Latter-Day Saints [1962]
3 All ER 364) affirmed.

Notes
As to the exemption from rates of places of public religious worship and church halls and chapel halls, see 32 Halsburys Laws
(3rd Edn) 43, 44, para 59; and for cases on the subject, see 38 Digest (Repl) 540, 541, 367376.
For the Places of Religious Worship Act, 1812, see 7 Halsburys Statutes (2nd Edn) 1275; for the Places of Worship
Registration Act, 1855, see ibid, 1287.
For the Poor Rate Exemption Act, 1833 (repealed), see 20 Halsburys Statutes (2nd Edn) 29.
For the Rating and Valuation (Miscellaneous Provisions) Act, 1955, s 7(2), see 35 Halsburys Statutes (2nd Edn) 392.
733

Cases referred to in judgment


Association of the Franciscan Order of Friars Minor v City of Kew [1944] VLR 199.
Barnes v Shore (1846), 1 Rob Eccl 382, Brod & F 44, 4 Notes of Cases 593, 10 Jur 887, 163 ER 1074, affd PC sub nom Shore v
Barnes (1848), Brod & F 49, 19 Digest (Repl) 562, 4035.
Cole v Police Constable 443A [1936] 3 All ER 107, [1937] 1 KB 316, 106 LJKB 171, 155 LT 498, 19 Digest (Repl) 248, 95.
Farnworth v Chester (Bishop) (1825), 4 B & C 555, 7 Dow & Ry KB 56, 4 LJOSKB 14, 107 ER 1166, 19 Digest (Repl) 395,
1962.
Freeland v Neale (1848), 1 Rob Eccl 643, 6 Notes of Cases 252, 11 LTOS 535, 12 Jur 635, 163 ER 1164, 19 Digest (Repl) 561,
4025.
Jones v Mersey Docks, see Mersey Docks v Cameron infra.
London County Council v Erith Parish (Churchwardens, etc) & Dartford Union Assessment Committee and Others [1893] AC
562, 63 LJMC 9, 69 LT 725, 57 JP 821, 38 Digest (Repl) 481, 47.
Mersey Docks v Cameron, Jones v Mersey Docks (1865), 11 HL Cas 443, 20 CBNS 56, 6 New Rep 378, 35 LJMC 1, 12 LT 643,
29 JP 483, on appeal from SC sub nom Mersey Docks & Harbour Board v Jones, Same v Cameron (1861), 30 LJMC 185,
239 Ex Ch, 38 Digest (Repl) 545, 395.
Nesbitt v Wallace [1901] P 354, 19 Digest (Repl) 358, 1516.

Appeal
this was an appeal from an order of the Court of Appeal (Lord Denning MR Donovan and Pearson LJJ), dated 10 July 1962, and
reported [1962] 3 All ER 364, allowing an appeal by the valuation officer (the respondent to the present appeal) by way of Case
Stated pursuant to Lands Tribunal Act, 1949, s 3(4) and s 11, and RSC, Ord 58A, from a decision of the Lands Tribunal (Mr. J P
C Done), given on 16 January 1961, and reported sub nom Godstone RDC v Henning (Valuation Officer) and Church of Jesus
Christ of Latter-Day Saints (1961), 8 RRC 189. By its decision the Lands Tribunal had dismissed two appeals by the respondent
from decisions of the Southern Surrey Local Valuation Court given on 2 December 1959, determining that the hereditament,
consisting of a number of buildings and the grounds occupied therewith, used by the appellants for their activities, the principal
building being known as The Temple or The London Temple and hereinafter referred to as the temple, should be entered in
the valuation list for the rating area of Godstone rural district, as exempt. The respondent conceded that the temple was a place
of religious worship. The question raised by the appeal was whether the hereditament was exempt from liability to be rated by
virtue of s 7(1), (2) of the Rating and Valuation (Miscellaneous Provisions) Act, 1955, other questions referred to in the decision
of the Lands Tribunal no longer being in issue.

Gerald Gardiner QC and H H V Forbes for the appellants.


P R E Browne QC and J R Phillips for the respondent.

Their Lordships took time for consideration

30 May 1963. The following opinions were delivered.

LORD REID. My Lords, I have had an opportunity of reading the speech about to be delivered by my noble and learned friend
Lord Pearce. I agree with it and I shall therefore move that this appeal should be dismissed.

LORD EVERSHED. My Lords, I have found the problem presented by this case to be one of extreme difficulty and though, in
the end, I have not come to a conclusion different from that entertained by the rest of your lordships, I have felt grave doubts on
the matter. The result of this appeal will, of course, be that although no rates are, I understand, levied in respect of any of the
chapels of that accepted branch of the Christian faith known as the Mormon, 734rates will be exigible in respect of their temple
at Godstone; and this result emerges from the use of the phrase place of public religious worship in the Rating and Valuation
(Miscellaneous Provisions) Act, 1955. I confess that I am not for myself satisfied that such was in truth the legislative intention,
and I accordingly state my doubts and the reasons for them in case the matter should at some time come up for further
consideration in Parliament.
The problem is capable of simpledeceptively simple, as it seems to meformulation; for it is confined to the proper sense
and meaning to be given to the phrase already mentioned place of public religious worship in s 7 of the Act of 1955. The
phrase occurs four times in the sectionthree times in the singular and once (in sub-s (2)(a) of the vital paragraph for present
purposes) in the plural. It is, however, as I conceive, clear that nothing in the particular contents in which the phrase occurs can
be regarded as determining the true intent of the epithet public as applied to the noun which follows, namely, worship.
My lords, I have had the advantage of reading in advance the speech prepared by Lord Pearce and I do not, therefore, make
any restatement of the facts. It is sufficient for me to say that the temple at Godstone in Surrey, which is the subject of the
present appeal, is the only religious edifice of its kind in England and that for the services performed therein only those members
of the Mormon Church who are in possession of a recommend of the local bishop indorsed by the temple President are
admitted. No other person, whether a member of the Mormon Church or not, is permitted to enter the temple whether for the
purpose of taking part in or observing the services or for the purpose of seeing the interior of the temple. It cannot therefore be in
doubt that if by the use of the epithet public in the phrase public religious worship is meant that the services are open to the
general public in the same sense as that intended by the use of the word public in relation to a park or a theatrical or other
similar entertainment, the appellants fail to satisfy the requirement of the subsection for exemption from rating liability. But
according to the ordinary sense of the word, does public have this significance when applied to such a matter as religious
worship? What in such a context is meant by the word worship? I am content to take the relevant meaning given in the
Shorter Oxford Dictionary, namely, the actions or practices of displaying reverence or veneration to a being regarded as Divine
by appropriate rite or ceremonies; and it must be essential to the participation of any worshipper that his words and actions
are addressed and directed to the Deity. If this be right, then the rites or ceremonies, the worship which they constitute, are in
no true sense addressed to or directed to the public, nor would the presence of any member of the public, if admitted to the
temple, be in any sense relevant to the true intent and purpose of the worship. There is therefore, as it has seemed to me, a
strong case for the view that by the phrase public religious worship is meant and intended forms or ceremonies of worship
distinct from private religious worship, namely, the religious devotions which a man may offer up in the privacy of his
bedroom or which are confined to the members of a family or household. In the latter case the worship is conducted alone or in
the intimate presence only of the worshippers family or household. By contrast in public worship the individual worshippers
may come from widely different places and may be personally entirely unknown to one another. In other words, there is, as it has
seemed to me, a strong case (as a matter of language) for the contrast expressed by Goddard J in the case of Cole v Police
Constable 443A ([1963] 3 All ER 107 at p 119; [1937] 1 KB 316 at p 334) congregational as distinct from private and family
devotion.
I think, further, that substantial support is lent to this view by two considerations. The first such consideration is this. The
Rating and Valuation (Miscellaneous Provisions) Act, 1955, requires that, in order to qualify for exemption from rating, the
places of public religious worship must also be certified as 735 required by law as places of religious worship. The latter
part of the requirement involves reference to the Act, 18 & 19 Vict c 81, known as the Places of Worship Registration Act, 1855,
which enables every place of meeting for religious worship not appertaining to the established church had not certified under the
Act, 52 Geo III c 155,a to be certified in writing to the Registrar of Births, Deaths and Marriages in England. The temple in the
present case has been so certified; but it does not of course follow from the terms of the Act of 1855 that it is therefore to be
regarded as a place of public religious worship. I observe, however, the reference to the Act of 1812, 52 Geo III c 155. The
predecessor of the Rating and Valuation (Miscellaneous Provisions) Act, 1955, was the Poor Rate Exemption Act, 1833(3 & 4
Will IV c 30). Prior to the passing of that Act, although it appears that in practice places of worship belonging to the established
church had not been rated, the exemption did not apply to places of worship of other religious denominations. The Act of 1833
was passed pursuant to the then current principle of religious toleration. Its terms were, save in one important respect, akin to
those of the Act of 1955. Exemption from rating was granted for or in respect of any churches meeting houses or premises
or such part thereof as shall be exclusively appropriated to public religious worship and which (other than premises of the
established church) shall be duly certified for the performance of such religious worship according to the provisions of any Act
or Acts now in force. As counsel for the appellants forcibly observed in opening the case before your lordships, the phrase
such religious worship can in the context have no other meaning than public religious worship. The relevant Act then in
force was the Places of Religious Worship Act, 1812(52 Geo Iii c 155), already referred to and mentioned in the Places of
Worship Registration Act, 1855. By s 2 of the Act of 1812 it was (stating the matter briefly) provided that no congregation or
assembly for religious worship of protestants at which more than twenty persons (other than the immediate family and servants of
the owner of the premises) should participate should be permitted unless and until the place of such meeting should have been
certified as therein provided. Reading together the relevant terms of the Acts of 1833 and 1812, there is at the least much support
for the view that by public religious worship in the former Act was meant religious ceremonies of the kind indicated in the Act
of 1812, that is to say, ceremonies at which the participants numbered more than twenty persons other than the immediate family
and servants of the owner of the premises and that the admissibility of members of the public to the ceremonies was not regarded
as relevant or as constituting the test for their qualification in public religious worship.
________________________________________
a Places of Religious Worship Act, 1812.

I do not forget that s 11 of the Act of 1812 (which I understand to be still in force) made it an offence to hold any form of
religious meeting behind locked doorsa provision, as my noble friend Lord Pearce observes in his speech, that was no doubt
enacted for reasons of public security. But I cannot think that the presence of that section provides assistance for the solution of
the present problem. Those responsible for the conduct of the services at the Godstone temple may, by barring entry to any but
those Mormons having a recommend, be committing an offence under s 11 of the Act of 1812. But that is something wholly
distinct from rating liability and I cannot think that Parliament in 1833 by its exemption from places of public religious worship
intended obliquely to exclude illegal assemblies from the benefit of the Poor Rate Exemption Act, 1833.
It is true that the language of the Act of 1955 differs from that of the Act of 1833 in that in relation to the requirement of
certification in the Act of 1955 the words religious worship are read without the qualification such. But, if the inference to
be drawn from the terms of the Act of 1833 is that which I have suggested, then must it be supposed that Parliament in 1955
intended in so indirect a manner to make so substantial a change in the meaning of the 736 phrase public religious worship?
The second consideration to which I have above referred is derived from the three ecclesiastical cases referred to and
distinguished by the Court of Appeal in their judgments in the present case, namely, Barnes v Shore, Freeland v Neale and
Nesbitt v Wallace. I cite the following passage from the judgment of the Dean of the Arches in the last of the cases ([1901] P at p
366):

But then it is said that the public do not go there, and cannot go there, and that, therefore, to read the services of the
Church of England there is not a public reading. I think that is a misapprehension of the word public in this connexion.
This appears clear from Freeland v. Neale (4), a case decided on the same ground as the case of Barnes v. Shore. The
learned judge in Freeland v. Neale, after observing that the assemblage in the Wardens Chapel at Sackville College could
in no sense be regarded as a private family, says ((1848), 1 Rob. Eccl. at p 651): In Barnes v. Shore I said, what I now
repeat, that where two or three are gathered together who do not strictly form a part of a family there is a congregation, and
the reading to them of the services of the Church is a reading in public, and, as I understand, it does not matter how the
congregation got there, whether by invitation or otherwise.

It is undoubtedly true, as the learned judges in the court below observed, that these cases were concerned with a matter
wholly different and distinct from the question of rating. They were concerned with the canon law and, in Nesbitt v Wallace,
more particularly with the question: What is a reading in public of the Book of Common Prayer having regard to the exclusive
right of the incumbent of a parish to say who shall read in public from the Book of Common Prayer within his parish?
Nevertheless, as it seems to me, the cases cited and particularly the language of the Dean of the Arches in the passage which I
have quoted ([1901] P at p 366) from his judgment do support the view of the meaning which the word public may properly
have in relation to religious worship. The ecclesiastical law is, after all, part of the law of England and Parliament may not
unreasonably be supposed in the Act of 1955 to have used the word public in the formula public religious worship in a sense
compatible with the meaning properly belonging to it in ecclesiastical law; and all the more so if the inference which I have
suggested can be drawn from the language used by Parliament in the Poor Rate Exemption Act, 1833.
For the reasons which I have endeavoured to state I have felt, and still feel, considerable doubt of the true meaning intended
by Parliament in the Rating and Valuation (Miscellaneous Provisions) Act, 1955, to be given to the formula public religious
worship. But, as I have stated at the beginning of this opinion, I am not in the end of all prepared to take a view different from
that which has commended itself to all your lordships and to all the members of the Court of Appeal. I therefore agree that the
appeal must be dismissed.

LORD MORRIS OF BORTH-Y-GEST. My Lords, the broad general purpose of the Poor Rate Exemption Act, 1833 (the
provisions of which are reflected in s 7 of the Rating and Valuation (Miscellaneous Provisions) Act, 1955), was to give
exemptions from rates so that all places of religious 737 worship should have the same exemptions as had in fact previously
existed in the case of the established church. Exemptions from rates are enjoyed in respect of the appellants seventy-five
chapels. The temple is admittedly a place of religious worship and sacred ordinances are there performed. It might seem strange,
therefore, if the exemption which is applicable in the case of the chapels is not available for the temple, but the statutory
exemption can only apply if the terms of the statute, which are satisfied in the case of the chapels, are also satisfied in the case of
the temple. The temple is certified as required by law as a place of religious worship, but to qualify for exemption it must be a
place of public religious worship and the question is whether the temple can properly be so described. I consider that there is a
distinction between private or domestic or family worship on the one hand and public religious worship on the other. In my view
the conception of public religious worship involves the coming together for corporate worship of a congregation or meeting or
assembly of people, but I think that it further involves that the worship is in a place which is open to all properly disposed
persons who wish to be present.
For the reasons expressed in the speech of my noble and learned friend Lord Pearce, which I have had the privilege of
reading in advance and with which I agree, I feel constrained, though with a measure of regret, to come to the conclusion that the
temple is in a different position from the chapels and that it does not satisfy the statutory requirements for exemption.

LORD DEVLIN. My Lords, I also have had the advantage of reading in advance the speech my noble and learned friend Lord
Pearce is about to deliver. I agree with it and I agree that the appeal should be dismissed. I find it unnecessary to say anything of
my own.

LORD PEARCE. My Lords, the question for your lordships is whether the temple of the Church of Jesus Christ of Latter-Day
Saints built at Godstone in Surrey is exempt from rating by virtue of s 7(2)(a) of the Rating and Valuation (Miscellaneous
Provisions) Act, 1955. That subsection exempts

Places of public religious worship which belong to the Church of England or to the Church in Wales or which are
for the time being certified as required by law as places of religious worship.

The temple is duly certified under the Places of Worship Registration Act, 1855, as a place of religious worship. If, therefore, it is
also a place of public religious worship, it is entitled to exemption. The Lands Tribunal held that it was so entitled, but the Court
of Appeal reversed that decision.
The Mormon Church has in the United Kingdom seventy-five chapels and some 16,000 adherents. The services at these
chapels are open not only to members of the Mormon Church but also to members of the public, and are admittedly exempt from
rating. In these respects they are in line, we are told, with the other Christian churches of various denominations and also with
synagogues and mosques. The temple, however, is a special building. There is only one temple in the United Kingdom and
certain sacred ceremonies are performed therein. It is not open to the public or even to every Mormon, but only to a Mormon of
good standing. Such a person is defined in the agreed statement of facts as one whose spiritual and secular qualities entitle him
in the view of a local Bishop, to a Recommend to a President who, if in turn satisfied that the holder is entitled by his personal
qualities to the appellation of Mormon of good standing, indorses the Recommend. Until that indorsement is appended the
recipient has no right to enter the temple. About 5,000 Mormons in good standing, inclusive of persons from abroad, attend the
ceremonies at the temple each year. Authoritative booklets, which were in evidence, show that the ordinances and blessings
performed in the temple are regarded as sacred and not publicly revealed and that the temple is not to be confused with the
regular church meeting-houses where public worship is conducted. The President of the Mormon Church is quoted as
answering the question What is the difference between your temple and your other church edifices by the words As all
members of the Church know, the answer is that temples are built for the performance of sacred ordinances, not secret but sacred.
A temple is not a public house of worship. It is erected for special purposes. Indeed after a temple is dedicated, only members of
the church in good standing may enter. These quotations show that the temple is not, in popular parlance, a place of public
religious worship. It does not, however, follow that the temple is not within the intention of the words of the subsection in their
context. Two alternative views 738 of that intention are possible; and either view can produce unsatisfactory results in certain
imaginable circumstances.
One view, which the appellants put forward, regards the nature of the worship from the standpoint of the individual who
attends the meeting rather than the nature of the meeting itself. The individual is worshipping in a congregation as opposed to
worshipping in private devotion or in the domestic setting of family prayers. To him the presence of many strangers, albeit drawn
from a select circle of his own church, constitutes public religious worship. For this view there is some support to be found in
three ecclesiastical cases which were concerned with the behaviour of the individual, namely the preacher who conducted the
service. This notion is conveniently summed up in the words of Goddard J in Cole v Police Constable 443A ([1936] 3 All ER at
p 119; [1937] 1 KB at p 334) where there is congregational worship as distinct from private and family devotion. The defect of
this view is that it would have to regard the meeting place of a small esoteric circle of some twenty or thirty persons worshipping
secretly behind closed doors as a place of public religious worship, a conclusion which offends against ones common sense.
The opposing view regards the nature and quality of the meeting itself rather than its aspect in the eyes of its individual
components. If the public are excluded from the meeting, then the meeting-house is not a place of public religious worship. This
view also has authority to support it. Its defect is that it must regard a meeting drawn perhaps from a very wide circle as failing
to constitute public religious worship merely because the public beyond that wide circle are not admitted.
There was some discussion in argument as to the Church of Englands power to exclude notorious evil livers, but such
exclusions are beside the point. For this purpose the admission of the public means, I think, the admission of those members of
the public who are reasonably suitable, who come in reverence not mockery, and who are prepared to behave in reasonable
conformity to the requirements of the religion which they are visiting eg, by covering their heads where that is required or by
removing their shoes on entering a mosque.
After the passing of the Poor Relief Act, 1601, churches were in fact not rated. It matters not whether this was because, as
has been suggested in argument, it was considered that churches were sacred and therefore not taxable, or because the incumbent
was not in occupation or at least not in beneficial occupation, or simply as a matter of good sense, since the church was the parish
church and the parish church and the parish was the unit of collection. In order to extend to the places of worship belonging to
other religious denominations an exemption similar to that enjoyed in practice by churches of the Church of England, the Poor
Rate Exemption Act, 1833, provided that no person should be rated

for or in respect of any churches meeting-houses or premises or such part thereof as shall be exclusively
appropriated to public religious worship and which (other than churches, District churches, and Episcopal chapels of the
Established Church) shall be duly certified for the Performance of such Religious Worship according to the provision of
any Act or Acts now in force.

This was the genesis of the words public religious worship in the context of rating. These words were repeated in the Act of
1955 and unless the context requires or unless in the intervening years there have been legal decisions or other matters affecting
their meaning, they bear the same meaning in 1955 as they bore in 1833.
In 1833 the situation relating to the legality of religious meetings, other than those of the Church of England, was as follows.
The Places of Religious Worship Act, 1812, s 1, had provided that

no congregation or assembly for religious worship of protestants (at which there shall be present more than twenty
persons besides the immediate 739 family and servants of the person in whose house or upon whose premises such
meeting shall be had) shall be permitted or allowed unless or until the place of such meeting shall have been
certified to the relevant bishop, archdeacon or justices.

(This was later superseded by certification with the Registrar General under the Places of Worship Registration Act, 1855.) There
was apparently no adjudication on whether such a certification of the place was properly required by the person so certifying and
whether in fact the religious meetings at that place would acquire the dimensions requiring a certificate. There was in that Act no
concept of public religious worship. The certification was to make lawful that which would otherwise be unlawful. When the
Act of 1833 was passed with the added concept of public religious worship conferring exemption from rates, it was natural that it
would apply only to those places that had been lawfully certified. But that certification by itself was not a sufficient ground for
exemption unless the religious worship was also public. Perhaps an indication of how the matter would strike the legislator in
1833 may be gained from some observations of Abbott CJ in 1825 in Farnworth v Bishop of Chester ((1825), 4 B & C 555 at p
568):

I have always understood it to be a general rule of law that no person can be authorised to preach publicly within a
chapel to which all the inhabitants of the district may have a right to resort without the consent of the clergyman to whom
the care of souls is given. I do not speak of a chapel belonging to a private individual where service is performed for the
convenience of his family and friends but of a chapel open to all the inhabitants of a certain district.

At that date I think that the place in which the meetings were held would be the dominant consideration and the important
difference was between worship in private houses and worship in public places, rather than between the public and private
worship of the individual.
The Act of 1812, s 11, had made it an offence, no doubt for reasons of political security, to hold religious meetings with the
doors barree so as to prevent persons entering in, and it might be suggested that since all meetings must thus be open to the
public, the admission of the public cannot in the Act of 1833 have been intended as a criterion of public religious worship. On
the other hand that argument would render the word public otiose. I think it more likely that in the Act of 1833 the concept of
public religious worship in the sense that it was open to the public was intended as a sine qua non of exemption.
In the context other than that of rating, the Act of Uniformity, 1662, s 1 had used the expression a place of public worship
in providing for the use of the Book of Common Prayer. And in a similar context s 13 refers to any church chappell or other
public place of or in any colledge or hall in either of the universities. But I cannot regard this as bearing on the intention to be
assigned to a statute dealing with exemption from rating. The cases of Barnes v Shore and Freeland v Neale and more recently
Nesbitt v Wallace, dealt with the question of what constituted a reading in public of the Book of Common Prayer. From those
cases it appears that when the meeting comprised persons outside the domestic household, even when those persons were merely
guests invited from other households, any reading by a curate was a reading in public. But those cases, as the Court of Appeal
said in the present case, were alio intuitu. When one is considering whether a curate is exceeding his rights under the canons of
his calling by reading in public outside his curacy, the test is wholly different from that where the question is whether there is
public religious worship which justifies exemption from rates. The curate may well be reading in public, although he is not
reading to what could be described as a public meeting. I do not regard the observations in those cases as helpful in deciding the
meaning of 740 the words of a statute concerned with ratinga subject which has a wider horizon than the question of
professional behaviour of individuals. Furthermore those observations cannot have had effect in 1833, since they occurred after
that date, and they were neither sufficiently relevant nor important to have an effect on the mind of the legislature in 1955 or to
lead it, when repeating the words of the Act of 1833, to do so with a different intention.
By the Act of 1833 the legislature was intending to extend the privileges of exemption enjoyed by the Anglican churches to
similar places of worship belonging to other denominations. Since the Church of England worshipped with open doors and its
worship was in that sense public, it is unlikely that the legislators intended by the word public some more subjective meaning
which would embrace in the phrase public religious worship any congregational worship observed behind doors closed to the
public. I find it impossible therefore to hold that the words places of public religious worship include places which, though
from the worshippers point of view they were public as opposed to domestic, yet in the more ordinary sense were not public
since the public was excluded.
This view accords with that of Lowe J in the Association of the Franciscan Order of Friars Minor v City of Kew where in
considering a statutory exemption for land used exclusively for public worship he held inter alia that the chapel of an Enclosed
Order of Carmelite Nuns was not exempted. He said b: In my opinion worship to be public must be open without
discrimination to the relevant public. By the word relevant he left open the question of how universal and indiscriminating
must be the admission of the public. The question is one of fact, and there may clearly be difficult questions whether some
discrimination may be insufficient to deprive the worship of its public character.
________________________________________
b [1944] VLR at p 202. Lowe J continued after relevant public, with the words but also be performed in public. The words represented
by the three dots n the quotation in the text are not only.

Furthermore it is less likely on general grounds that Parliament intended to give exemption to religious services that exclude
the public since exemptions from rating, though not necessarily consistent, show a general pattern of intention to benefit those
activities which are for the good of the general public. All religious services that open their doors to the public may, in an age of
religious tolerance, claim to perform some spiritual service to the general public. Jones v Mersey Docks & Harbour Board
exploded the supposition on which many earlier cases proceeded that lands held for public purposes were not rateable (see
per Lord Herschell LC in London County Council v Erith Parish (Churchwardens, etc) ([1893] AC 562 at p 585)). Nevertheless
in considering ambiguous words in a statute granting exemption from rating one cannot wholly disregard what must have
weighed with the legislators, namely, considerations of fairness and public benefit.
I do not find anything unreasonable in denying to the Mormon church the public benefit of an exemption of its temple, to
which it will not allow the public to have access for worship, while according such exemption to its many chapels which, like
those of other denomination, do admit the public.
I agree therefore with the judgment of the Court of Appeal and I would dismiss the appeal.

Appeal dismissed.

Solicitors: Devonshire & Co (for the appellants); Solicitor of Inland Revenue (for the respondent).

C G Leonard Esq Barrister.


741

[1963] 2 All ER 742

Vaughan v Vaughan
FAMILY; Ancillary Finance and Property

PROBATE, DIVORCE AND ADMIRALTY DIVISION


SIR JOCELYN SIMON, P AND CAIRNS J
23, 24 APRIL 1963

Magistrates Husband and Wife Maintenance Wilful neglect to maintain child Desertion Wife in state of desertion
Jurisdiction to make orders for custody and maintenance of child Matrimonial Proceedings (Magistrates Courts) Act, 1960 (8
& 9 Eliz 2 c 48), s 1(1)(h), s 2(1)(d), (h), s 4(1).

Magistrates Husband and wife Order Non-cohabitation clause Finding of wilful neglect to maintain No other
circumstances making separation order necessary Matrimonial Proceedings (Magistrates Courts) Act, 1960 (8 & 9 Eliz 2 c
48), s 2(1)(a).

Since the coming into operation of the Matrimonial Proceedings (Magistrates Courts) Act, 1960 a, a magistrates court may,
under s 4(1), make a matrimonial order relating to the legal custody or maintenance of a child of the family in proceedings at the
suit of the wife under s 1, notwithstanding that she is in desertion, and may include in the order a finding that there has been
wilful neglect on the part of the husband to maintain the child (see p 744, letter g, and p 745, letter h, post).
________________________________________
a Ie, since 1 January 1961, see p 745, note (10), post.

A non-cohabitation clause is inappropriate to a justices order based on the ground of wilful neglect to provide reasonable
maintenance, if there is no other circumstance which makes a separation order necessary for the protection of the wife or children
(see p 744, letter i, and p 746, letter a, post).

Notes
Leading considerations relevant in deciding whether a non-cohabitation clause should be included in or excluded from a
matrimonial order were stated by Sir Jocelyn Simon P in Corton v Corton ([1962] 3 All ER 1025), which was a case where
persistent cruelty was alleged; the exclusion of the non-cohabitation clause in the present case is, it seems, an application of the
first of these leading considerations in a case where the matrimonial order was sought on the ground of wilful neglect to provide
reasonable maintenance.
As to maintenance for a child, see 12 Halsburys Laws (3rd Edn) 488, para 1087; and for cases on the subject, see 27 Digest
(Repl) 708, 709, 67566762.
As to when non-cohabitation orders are made, see 12 Halsburys Laws (3rd Edn) 267, para 510, note (t); and for cases on the
subject, see 27 Digest (Repl) 696 et seq, 6647 et seq, 704, 705, 67266728.
For the Matrimonial Proceedings (Magistrates Courts) Act, 1960, s 1(1)(h), s 2(1)(d), (h), s 4(1), see 40 Halsburys Statutes
(2nd Edn) 396, 400, 405.

Cases referred to in judgment


Jolliffe v Jolliffe (1963), 107 Sol Jo 78.
Naylor v Naylor [1961] 2 All ER 129, [1962] P 253, 125 JP 358, [1961] 2 WLR 751, 3rd Digest Supp.
Young v Young [1962] 3 All ER 120, [1962] 3 WLR 946.

Appeal
In this case the wife appealed and the husband cross-appealed against an order of the Llanidloes justices dated 9 August 1962.
The parties were married on 3 July 1946, and there were four children between four and thirteen and a half years of age.
The husband was at first a seaman, but in 1957 or 1958 he obtained a shore job. The wife complained that from that time
onwards the husband drank to excess, that when in drink he assaulted her, and that she threatened to go if he continued his habit
of drinking. On 12 May 1962, there was a quarrel in which the husband told the wife to leave, and she thereupon left. Ten days
later the wife returned to collect some clothes; there was a further quarrel and the wife then left for good.
The wife caused a summons to be issued against the husband on her complaints that he had been guilty of persistent cruelty
towards her, that he had deserted 742 her and that he had been guilty of wilful neglect to provide reasonable maintenance for
herself and the children. The justices dismissed the wifes complaint of cruelty; they also dismissed the wifes complaints of
desertion and wilful neglect to provide reasonable maintenance for herself, holding that the husbands conduct had not been
sufficiently grave to compel the wife to leave and that she had, therefore, left the husband without cause. The justices, however,
found that the husband had been guilty of wilful neglect to provide reasonable maintenance for the children and made an order
against the husband. They included in the order or provision that the wife was no longer bound to cohabit with the husband.
The wife now appealed against the dismissal of her complaints and the husband cross-appealed against the finding of wilful
neglect to maintain the children. The husband did not dispute that the wife was entitled to an order for maintenance under the
Guardianship of Infants Acts, 1886 and 1925, nor did he dispute the amount of maintenance awarded.

D W Powell for the wife.


H W J ap Robert for the husband.

24 April 1963. The following judgments were delivered.

SIR JOCELYN SIMON P stated the facts, considered the wifes appeal, which he held should be dismissed, and, turning to the
husbands appeal, continued: The justices, having refused to accept the husbands excuses for non-payment, were entitled in my
view to come to the conclusion that he had failed to provide reasonable maintenance for the children. The question, therefore,
arises, namely, whether the justices were in law entitled to make a maintenance order on the ground of wilful neglect to maintain
the children under the Matrimonial Proceedings (Magistrates Courts) Act, 1960, or only a maintenance order under the
Guardianship of Infants Acts, 1886 and 1925, without any finding of wilful neglect to maintain the children? That depends on
how far Naylor v Naylor is applicable since the coming into force of the Matrimonial Proceedings (Magistrates Courts) Act,
1960. We were referred to Young v Young ([1962] 3 All ER at p 125) where it was said:

If the husband was guilty of wilful neglect to maintain the child then, whatever the conduct of the wife, the justices
should make an order for the maintenance of the child.

But that left it open whether the order should be made under the Matrimonial Proceedings (Magistrates Courts) Act, 1960, or the
Guardianship of Infants Acts, 1886 and 1925. In Naylor v Naylor Lord Merriman P concluded a careful argument by saying
([1961] 2 All ER at p 144; [1962] P at p 275):

In our opinion, a deserting wife cannot obtain an order on a complaint of neglect to maintain the children.

However, he made it clear ([1961] 2 All ER at p 143; [1962] P at p 273) that he was dealing with the law as it existed prior to the
Act of 1960, which had not come into force at the material time. Naylor v Naylor decided that, where a wife was in desertion, the
justices could not make an order for the custody and maintenance of children under the Summary Jurisdiction (Separation and
Maintenance) Acts, 1895 to 1949, though they could make such orders under the Guardianship of Infants Acts, 1886 and 1925.
They would be bound to dismiss such a wifes complaint of wilful neglect to maintain the children: a deserting wife is not
entitled to an order under any of the provisions of the Summary Jurisdiction (Separation and Maintenance) Acts, because a
matrimonial order, even if in respect of the children, is made in favour of the wife; and a deserting wife is not entitled to a
matrimonial order in her favour. It is important to note, though, that Naylor v Naylor did not say that a deserting wife was not
entitled to complain that her husband had wilfully neglected to maintain the children of the marriage; only that she was not
entitled 743 to an order on that ground. Still less did it say that a deserted husband was not capable of the offence of wilfully
neglecting to provide reasonable maintenance for the children.
It is against that background that one must look at the Act of 1960. By s 1(1) a married woman may apply for an order on
various causes of complaint, which include

(h) being the husband, has wilfully neglected to provide reasonable maintenance for the wife or for any child of the
family who is, or would but for that neglect have been, a dependant.

Section 2(1) provides:

on hearing a complaint under s. 1 the court may make an order ( referred to as a matrimonial order)
containing any one or more of the following provisions (d) a provision for the legal custody of any child of the family
who is under the age of sixteen years (h) a provision for the making by the defendant for the maintenance of any
child of the family, of payments by way of a weekly sum not exceeding in respect of any one child the sum of fifty
shillings.

So far the Act of 1960 does not differ materially from the Acts which were under consideration in Naylor v Naylor. But s 4(1) of
the Act of 1960 is a new provision:
Where the court has begun to hear a complaint under s. 1 of this Act then, whether or not the court makes the
order for which the complaint is made, the court may make a matrimonial order containing any provision such as is
mentioned in paras. (d) to (h) of s. 2(1).

In other words, if the court has begun to hear a wifes complaint, then, irrespective of whether it decides that the complaint is
justified, it may make a provision for the childrens custody or maintenance.
Naylor v Naylor decided that a deserting wife was not entitled to an order under the Acts of 1895 and 1920, b, on the ground
of wilful neglect to maintain the children, because she was a deserter. It followed that the justices could not make a finding of
wilful neglect to maintain the children, because the wife was not entitled to any order resulting from such a finding. Today,
however, the justices would not be bound to dismiss the wifes complaint of wilful neglect to maintain the children on the ground
that as a deserter she was not entitled to any order; a deserting wife may obtain an order for the custody or maintenance of the
children. It, therefore, seems to me that there is nothing to preclude the justices from making a finding on the complaint of a wife
that the husband has wilfully neglected to maintain the children, even though they should find also that she is in desertion. It
follows that the justices were, in my view, entitled to find in the present case that the husband had wilfully failed to provide
reasonable maintenance for the children.
________________________________________
b Summary Jurisdiction (Married Women) Act, 1895, and the Married Women (Maintenance) Act, 1920; 11 Halsburys Statutes (2nd Edn)
849, 863.

The justices have, however, included in their order as drawn up in favour of the wife a provision that she was no longer
bound to cohabit with her husband. It does not appear from the clerks note that they said anything about that matter when they
announced their decision in court; and it may well be, therefore, that the provision was inserted per incuriam. A non-cohabitation
clause is inappropriate to a justices order based on the ground of wilful neglect to maintain, when there are no other
circumstances which make a separation order necessary for the protection of wife or children: see Jolliffe v Jolliffe.
744
My conclusion is that the wifes appeal should be dismissed; that the husbands cross-appeal should be allowed only to the
extent of the deletion of the non-cohabitation clause; but that otherwise the justices adjudication and order should stand.

CAIRNS J considered the wifes appeal, agreed that it should be dismissed, and continued: Turning now to the husbands
appeal, there was, in my view, evidence to support the justices finding of wilful neglect to maintain the children. It is true that
not much attention seems to have been devoted to this matter during the taking of the evidence, but it was uncontradicted that the
husband had paid to his wife for herself and the children only one sum of 10 during a period of eight weeks up to the date of the
wifes complaint, and the real case that he was setting up was, not that he had adequately maintained the children, but that his
failure to maintain them was not wilful because he had been sick and unable to work. The justices did not accept his case with
regard to that matter and, once that goes, it seems to me clearly that they were entitled to say that he had wilfully neglected to
maintain his children. However, the justices findings involved that the wife was a deserter and under a decision of this court in
Naylor v Naylor she would not be entitled to an order for custody or maintenance of the children under the law as it stood before
the coming into force of the Matrimonial Proceedings (Magistrates Courts) Act, 1960, except under the Guardian ship of Infants
Acts, 1886 and 1925. It is clear that an order for the custody and maintenance of the children can now be made under s 4(1) of
the Act of 1960. Such an order could be made without any finding of wilful neglect to maintain or of any other of the causes set
out in s 1 of the Act on the part of the husband and the real question in the present case is whether that finding of wilful neglect to
maintain should be allowed to stand as part of the order or should be deleted from the order, and I ask myself why should it be
deleted from the order? There is no ground for saying that it was a wrong finding. It appears to me that even as the law stood at
the time of Naylor v Naylor there was nothing to prohibit the inclusion of such a finding in the order of the court. All that Lord
Merriman P said in relation to this matter was ([1961] 2 All ER at p 144; [1962] P at p 275):

In our opinion a deserting wife cannot obtain an order on the complaint of neglect to maintain the children.

If a formal order dismissing the wifes complaint were drawn up by the magistrates court, even at the time of Naylor v Naylor, it
seems to me that it might well have said that the court adjudged that there had been wilful neglect to maintain the children, but
that the court further adjudged that the wife was in desertion and on that ground dismissed her complaint. However that may be
in relation to the law as it stood before the beginning of 1961 c, it seems to me that now that an order can plainly be made under s
4(1) of the new Act, there is no reason why this finding should not be included in the order if it is established to the satisfaction of
the justices that there was wilful neglect to maintain. The husband says that he wants it removed from the order because of the
slur on him of such a finding. This seems to me to be a good reason why it should not be removed, for to remove it would
suggest that this court thought that it was unjustified. For these reasons I agree that the husbands appeal should be dismissed.
745
________________________________________
c The appointed day for the commencement of the Matrimonial Proceedings (Magistrates Courts) Act, 1960, was 1 January 1961; see
Matrimonial Proceedings (Magistrates Courts) Act, 1960 (Commencement) Order, 1960, SI 1960 No 2223.

I further agree that this is not an appropriate case for the inclusion of a non-cohabitation clause in the order and that that
should be deleted.

Order accordingly.

Solicitors: Jaques & Co agents for Emrys Jones & Co, Welshpool (for the wife); Field, Roscoe & Co agents for E P Careless &
Co Llandrindod Wells (for the husband).

A T Hoolahan Esq Barrister.


[1963] 2 All ER 746

TvT
FAMILY; Industrial relations

COURT OF APPEAL
ORMEROD, DONOVAN AND RUSSELL LJJ
25, 26, 27 MARCH, 2 MAY 1963

Divorce Sodomy Consent Reality of consent Wife allowing acts of sodomy because of husbands persuasion that it was
normal between married couples and her marital duty Wife regarding practice as unnatural but not wrong Whether such
consent barred her from obtaining a divorce on the ground of sodomy Matrimonial Causes Act, 1950 (14 Geo 6 c 25), s 1(1).

A husband, aged twenty-four, represented to his wife, aged eighteen, that if he desired sodomy she ought to submit to it, that his
friends wives allowed it, that it was normal between married couples, and that submission to it was part of her matrimonial duty;
he thereby persuaded her to submit to sodomy by him on her on three occasions. The husband honestly believed the
representations that he made. The wife testified that she thought instinctively that the practice was unnatural but not wrong, that
she hated it, but allowed it, her husbands arguments prevailing with her on the three occasions. On petition by her for divorce on
the ground of sodomy,

Held (Ormerod LJ dissenting): the wifes consent to the sodomy was not a real consent, she having been persuaded into
accepting that the practice was normal between married couples and that it was her marital duty to allow it, and not knowing that
the act was one that was wrong on her part; thus there was not such consent on her part as would bar her from obtaining a decree,
since, owing to the husbands persuasion, she had not complete freedom of choice (see p 755, letters f and g, p 763, letter d, and p
765, letter a, post), notwithstanding the husbands genuine belief in what he had said and the absence of physical compulsion by
him (see p 753, letter f, and p 763, letter b, post).
Dictum of Hodson LJ in Bampton v Bampton ([1959] 2 All ER at p 767) applied.
Statham v Statham ([1928] All ER Rep 219) considered and distinguished.
Per Donovan LJ: a real consent involves knowledge of the relevant factors bearing on the question whether consent
should be forthcoming or not, and one of the chief of these is whether the act to which the consent is sought is right or wrong (see
p 755, letter g, post).
Appeal allowed.

Notes
The basis of the judge-made rule that consent to sodomy is a bar to divorce on that ground is that the party seeking the decree
may not take advantage of her own wrong (see p 751, letter h, post); approaching the present case from the aspect of the quality
of the wifes consent, Donovan LJ concluded that her consent was not such that she was seeking to take advantage of her wrong
(see p 753, letters g to i, post). With this may be compared some observations of Russell LJ made from the aspect of the effect of
the husbands conduct on the availability to him of the defence of the wifes consent, viz, that it was not open to him to maintain
the wifes consent as a bar when he had persuaded her that the practice was not a matrimonial offence (see p 764, letter h, post).
Russell LJ also, however, approached the present case initially 746 from the aspect that the wifes state of mind was the primarily
relevant factor (see p 763, letter c, post).
As to consent as a defence to a charge of sodomy, see 12 Halsburys Laws (3rd Edn) 281, para 544; and for cases on the
subject, see 27 Digest (Repl) 294, 2394 and 3rd Digest Supp.

Cases referred to in judgment


Bampton v Bampton [1959] 2 All ER 766, [1959] 1 WLR 842, 3rd Digest Supp.
Churchman v Churchman [1945] 2 All ER 190, [1945] P 44, 114 LJP 17, 73 LT 108, 27 Digest (Repl) 379, 3131.
Curtis v Curtis (1858), 1 Sw & Tr 192, 27 LJP & M 73, 31 LTOS 272, 164 ER 688, affd (1859), 4 Sw & Tr 234, 27 Digest (Repl)
543, 4908.
D B v W B [1935] P 80, sub nom B v B [1935] All ER Rep 428, 104 LJP 25, 152 LT 419, 99 JP 162, 27 Digest (Repl) 697, 6664.
Davidson v Davidson [1953] 1 All ER 611, [1953] 1 WLR 387, 117 JP 152, 3rd Digest Supp.
Douglas v Douglas [1950] 2 All ER 748, [1951] P 85, 27 Digest (Repl) 380, 3135.
Lawson v Lawson [1955] 1 All ER 341, [1955] 1 WLR 200, 119 JP 279, 3rd Digest Supp.
Statham v Statham [1928] All ER Rep 219, [1929] P 131, 98 LJP 113, 140 LT 292, 27 Digest (Repl) 294, 2394.
Woodbury v Woodbury [1948] 2 All ER 684, [1949] P 154, [1949] LJR 40, 27 Digest (Repl) 383, 3154.

Appeal
On 23 November 1960, the wife filed a petition for divorce alleging cruelty by acts not of a sexual nature. On 15 November
1961, the petition was amended and allegations of cruelty consisting of sexual malpractices were made; in addition the wife
petitioned for a decree on the ground of sodomy committed on her by her husband on three occasions. By his answer the
husband admitted that he had committed sodomy as alleged but pleaded consent by the wife and alternatively condonation, but
the issue of condonation did not arise. On 11 November 1962, the trial judge (Wrangham J) found that the allegations of cruelty
were not made out, and, found that there had been consent to the acts of sodomy (see p 752, letter h, to p 753, letter a post, where
an extract from his judgment is set out), and dismissed the petition. The wife appealed.

J Stirling QC and C T Reeve for the wife.


John Latey QC and S Seuffert for the husband.

Cur adv vult

2 May 1963. The following judgments were delivered.

ORMEROD LJ referred to the issue and pleadings and continued: The questions which we have to consider are whether, in the
first place, there was a consent by the wife to the sodomy, and, secondly, whether on the evidence the acts alleged amounted to
cruelty on the husbands part. By cruelty I refer only to the allegations of cruelty based on sexual malpractices, as this court is
not required to pronounce on what the learned judge has described as the non-sexual acts of cruelty.
The parties were married on 4 June 1959, the wife being eighteen and the husband twenty-four. Prior to meeting the
husband, the wife had no previous experience of sex, but it was agreed that sexual intercourse took place before marriage as a
result of which the wife became pregnant, and a child was born on 8 January 1960. The wife said in her evidence, and it was not
denied by the husband, that prior to the marriage the husband asked the wife if he could perform the act of sodomy, and she
refused. The parties lived together for a year in Earls Court Road and then took a flat in Lennox Gardens. The marriage lasted
eighteen months, and, on 18 October 1960, the wife left the husband, taking the child with her.
747
In the early part of the marriage the parties were happy enough, and apparently in love one with the other. Early in the
marriage, however, when sexual intercourse was taking place between the parties, the husband suggested that sodomy should be
committed, but the wife on many occasions refused. She said that she permitted it only on three occasions in all, each time when
sexual intercourse in the normal way was not practicable, and that she did not like the practice of sodomy. It was painful and she
did not enjoy it; but her husband told her that other wives of her acquaintance made no objection and that it was her wifely duty
to do it, and that she appears to have accepted. She said in her evidence that she thought that it was not a natural thing to do
although she did not think that it was wrong.
The salient points in the evidence of the wife on this question are these:

Q.It was first mentioned? A.Yes. First mentioned just after we got married. Q.In what form was it
mentioned? A.He said he wanted to bugger me again. Q.Were those his words? A.Yes. Q.Where did this
conversation take place? A.In our bedroom in Earls Court. Q.Yes. In bed or out of bed? A.In bed. Q.What was
your response to that? A.No. I did not like it. Q.Did he say anything to that? A.He more or less implied that it did
not really matter whether I liked it or not; it was my duty, I was his wife. Q.I must ask you this. What was your reaction
to it; did you enjoy it or not? A.No, I did not. I hated it. I found it very uncomfortable, and very painful. Q.Did you
say anything about that to your husband? A.Yes, I did. Q.How long after? A.While it was happening, and
afterwards, and he said that in time it would not hurt so much, and I said I did not care, I did not want it.

She went on in her evidence to say that she did not allow sodomy to be committed except on the three occasions already referred
to, and, when asked what she believed when the husband said that his friends wives allowed it, her answer was: I was not sure
what to believe. I thought that if what he said is true it must be true, and later she said that when she refused his attitude was
sulky and he used to behave like a child who was refused a sweet.
It is to be remembered, however, that, although the wife appears to have accepted the husbands statements that sodomy was
an act which was practised by his friends with the consent of their wives, she herself does not appear to have regarded it as
wholly natural. The transcript contains this passage:

Q.Tell me, in relation to some of that that you say your husband asked you to do; did it not ever occur to you that it
was wrong? A.By instinct, yes, possibly it did; I think it did, not wrong, but not natural. Q.Well, not natural. Did it
upset you at all being asked to do these things which you regarded as not natural? A.Yes, it did upset me.

It is probable that those questions and answers relate to other sexual practices as well as to the sodomy.
Those, I think, are the material passages in the evidence which the learned judge had in mind when he considered whether
there had been consent on the part of the wife, and in his judgment he says this:

In this case I am satisfied that this particular wife had complete freedom to choose whether she would permit sodomy
or would not permit sodomy, and in the exercise of that complete freedom, she did permit it, in other words, she consented
to it. I might add that I have seen her in the witness box, and she is a young woman of character and personality who
would not easily be dominated either by this husband or any other. It follows that I find that the sodomy was consented to,
and that she cannot rely on that for relief in this case. Of course, the fact that this sodomy was committed, though it cannot
be relied on as I think as a substantive ground for relief, remains as a fact against which the other matters in this case have
to be assessed.
748

The question which we have to consider is whether there was any real consent on the part of the wife to the admitted acts of
sodomy. By the Matrimonial Causes Act, 1950, s 1, the wife may obtain a decree on the ground of the husbands sodomy after
the marriage, and it has been held that sodomy is a ground for divorce when practised by a husband on his wife and not on a third
person. The authorities establish, however, that sodomy is not a ground for a decree when practised on a wife if the wife has been
a consenting party, and therefore the question to be considered is whether the wife in fact has given any real consent to the acts
alleged. Two decisions of this court appear to be relevant. They are Statham v Statham and Bampton v Bampton. In Statham v
Statham the issues were whether there had been any act of sodomy, and, if so, whether the wife had consented to it. There was
certainly not more than one act, and the question then was whether the wife had consented. Greer LJ says this ([1928] All ER
Rep at p 223; [1929] P at p 144):

Section 178 of the Judicature (Consolidation) Act, 1925, a, provides as follows: 178(1). On a petition for divorce it
shall be the duty of the court to satisfy itself so far as it reasonably can both as to the facts alleged and also as to whether
the petitioner has been accessory to or connived at or condoned the adultery or not, and also to inquire into any counter
charge which is made against the petitioner. It is curious that this section only speaks of the petitioner as being accessory
to or conniving at or condoning adultery, that is to say, the cause of divorce mentioned in sub-s (a) of s. 176, and there is
nothing about connivance or condonation of the causes of divorce mentioned in the first part of sub-s. (b), which include
sodomy, but it is difficult to suppose that the court is bound to grant a decree for dissolution of marriage to a wife who
upon her own evidence has consented to the act of which she complains. The rules with regard to condonation and
connivance as a bar to a decree for divorce were well established in the practice of the ecclesiastical courts before the Act
of 1857; see the cases cited in Rayden on Divorce (2nd Edn) 107110 and especially Curtis v. Curtis. In my judgment,
these rules apply to cases in which the ground alleged for divorce is sodomy.
________________________________________
a See now s 4 of the Matrimonial Causes Act, 1950.

Later in the same judgment Greer LJ says this ([1928] All ER Rep at p 224; [1929] P at p 145):

It is quite clear that the act was not done, as she alleges in her petition, by force and against her will. In cross-
examination she stated that her husband was forty-nine and that she was only twenty, and when asked whether she tried to
prevent him she said No, I did not try to prevent him. I did as I was told. I did not know much about things when I was
married. This falls far short of showing that she did not understand that what she was consenting to was wrong. She must
have known that it was wrong, improper, and unnatural, and she does not venture to say that she did not. There is no
indication of any sort of duress exercised by the husband, and it is noteworthy that when he suggested doing it again, she
maintained her refusal without difficulty. In my judgment the learned judge ought to have dismissed the claim for divorce
on the ground that the wife was a willing party in the act on which she founded her claim.

It is true in the present case to say that, although the wife said that she thought the practices were unnatural, she said that she did
not think them wrong and there is no evidence to show that she thought them improper.
Russell LJ said this ([1929] P at p 156):

Her evidence makes it clear (1) that her husband explained to her 749 quite plainly the exact physical act which he
wished to do upon her; (2) that she assented and placed her body at his disposal for the purpose of that act being
committed; (3) that no compulsion of any kind was brought to bear upon her; and (4) that the only reason given by her for
refusing subsequent invitations to a similar act was fear of pain. It thus appears that the wife was a consenting party to the
only act upon which a decree for divorce could be founded. It is immaterial whether or not she knew that the act to which
she consented was called sodomy or that it was a crime. The act was accurately explained to her beforehand, she must
have known that it was against nature, yet she consented of her free will to its commission.

Lord Hanworth MR said ([1928] All ER Rep at p 221; [1929] P at p 139):

Her evidence proves that what was desired was explained to her, and although in its very essence every person must
realise that such an act is against nature, she accepted it until she found that it caused her pain.

In Bampton v Bampton the question was whether the wife had consented to the husband committing sodomy on her. It was
held that she had consented and that such consent was an absolute bar to relief on the ground of sodomy. At the outset of his
judgment Hodson LJ said ([1959] 2 All ER at p 767):

The question which falls for decision (and it is the main ground of appeal) is whether the consent of the wife to the act
of sodomy, as opposed to the cruelty, was a bar to her obtaining relief; and, if such consent is a bar, whether it is right to
say that there was here a real consent. In considering the second question, the relationship of husband and wife lies in the
background. It should be recognised that the relationship of husband and wife being what it is, and the obligations of one
to the other being what they are, it is not readily to be taken against a wife in a situation such as exists in this case,
especially a young wife, that there has been a real consent. If the same acts were under consideration between two persons,
not being husband and wife, where this relationship did not exist, the presence or absence of consent would have to be
considered in a different way.

The finding of Barnard J that there had been consent was upheld. It is to be noted that Hodson LJ used for the first time the
phrase real consent. It is not easy to define the limits of the adjective real in this connexion. It may well be that real
consent means nothing more than consent. If a husband commits sodomy on his wife by force, she cannot, of course, be said
to have consented. Nor can she be said to have consented if her acquiescence has been obtained by fraud. It is not suggested in
this case, however, that there was any fraud on the part of the husband. The judge accepted the husbands evidence that he
believed that the wives of his friends consented to, and that some of them enjoyed, the act of sodomy. It was argued, therefore,
that the wife was deceived, innocently enough, by her husbands representations that his friends wives consented to the practice
and it was her duty as a wife to consent also. I do not accept this argument. Had she been deceived, however innocently, by her
husband as to the nature of the act she was committing, it might have been true to say that she was not consenting to it. There
can, I think, be no doubt on the evidence that she knew the nature of the act, and the effect of her husbands statements was that
she believed that other mens wives were agreeable to the performance of this act. I have already quoted the passage in her
evidence, (p 748, letter g, ante), in which she referred to this act, and probably others, as being not wrong but not natural. I
quote further from her evidence:

Q.From the time you and your husband were having these relations, prior to the marriage, was there ever any
discussion about intercourse 750 other than in the normal way? A.Yes; on one occasion. Q.About when was that after
you first started having relations with him? A.That was, I suppose, about the April of the following year. Q.April of
1959? A.Yes. Q.Well, you must tell us what was said about it; what was the discussion, what was discussed? A.He
said he wanted to bugger me, and I said No.

I find it impossible to believe, in the light of her evidence, that she did not know the nature of the act that was committed on her
before it was committed for the first time. After the first time there can be no doubt, in my judgment, that she knew and agreed to
it. She may have been persuaded against her better judgment, and she may have agreed in the belief that other mens wives did
so, and that she should do so too. These matters do not make her agreement any less a consent. She knew that she was accepting
that an act of sodomy should be committed on her, and I see no reason for disturbing the finding of the learned judge that she
consented, although this view differs from that held by my brethren.
On behalf of the wife, counsel also submitted that the learned judge was wrong in not holding that the sexual conduct of
which the wife complained amounted to cruelty. His submission in the first place was that the judge was wrong in his findings of
fact as to certain of the sexual malpractices. He submitted, too, that the husbands continued requests to be allowed to commit
sodomy should have been held to be cruelty. For my part, I do not accept these submissions and I would dismiss the appeal.

DONOVAN LJ. Subsection (1) of s 1 of the Matrimonial Causes Act, 1950, read with s 4 of the same Act, provides, inter alia,
that a wife may be granted a decree of divorce on the ground that her husband has, since the celebration of the marriage, been
guilty of sodomy or bestiality. The husband in this case admits that after the marriage he committed sodomy on his wife. He also
admits that he sought an orgasm in his wifes mouth. This latter perversion was alleged in the wifes petition as an act of cruelty
and not of bestiality. Wrangham J refused any relief to the wife. As to the sodomy, he finds that she consented to it. As to the
other perversion, he finds that the husband sought to do this only once, that the request was refused, and that there was no
evidence whatsoever that the refusal was not completely accepted by the husband. Accordingly, he found that this act did not
amount to cruelty. The learned judge was wrong in saying there was no evidence whatsoever that the refusal was not accepted.
The wife gave evidence to that effect, and this evidence was not specifically rejected by the judge. Nevertheless, there is, I agree,
no ground for interfering with his finding against cruelty.
To return to the sodomy; the Act of Parliament does not say that consent by a wife to such an act by her husband shall be a
bar to a decree of divorce. This law is judge-made, and it finds expression in Statham v Statham and in Bampton v Bampton, both
decisions of the Court of Appeal. The reasoning behind such a rule I take to be this: that sodomy is an act in which two people
take part; that prima facie each is as guilty as the other of the crime; so that when a wife who has committed sodomy with her
husband seeks a divorce on the ground of the sodomy, she is pleading an act which is a wrong not only on the part of her husband
but on her part, too, and she ought not to be allowed to take advantage of her own wrong. Counsel for the husband accepted this
view of the reason for the rule, and none other has been suggested. So stated it is obvious that the rule must permit of exceptions.
The wife who is forced to submit by violence or by threats, and the wife who is deliberately deceived into thinking that as
between spouses the act is right, are among them. It is conceded that any consent of the wife obtained by these means is not a
real consent, and does not bar her from relief. The present is not such a case. It is a case where 751 a wife was innocently
deceived by her husband into thinking that, if he desired sodomy, she ought to submit to him; that this practice was widespread
among married couples, was part of normal love-making; and that submission to it was part of her matrimonial duty. The judge
finds that the husband sincerely believed all this, which no doubt made his representations to his wife all the more persuasive.
The question now is whether a consent of the wife obtained by these means is a real consent which bars her from relief.
The wife gave evidence to the following effect. Her husband had asked her to submit to buggery by him before the
marriage, and she had refused. Shortly after they were married, he renewed the request, and she refused again, saying she did not
like it. His response was that it did not matter whether she liked it or not, it was her duty as his wife to submit. When she did
allow him to do what he wanted, she hated it; it was very uncomfortable and very painful. She told her husband so, but he said
that in time it would not hurt so much. Her reply was that she did not care about thisshe did not want it. She suffered him to
commit sodomy on her on three occasions only, and, on the other occasions when she would not comply with similar requests by
him, her husband said that it was a normal thing; everyone did it and so should she. He said his friends wives allowed it. In the
result, the wife thought that what her husband said in this respect was true. On the occasions when she refused to permit sodomy,
her husband became sulky and would not speak to her. In the result she felt, perhaps, that she was being silly, that she was
wrong, that sodomy was normal, and that she was not being a good wife in refusing it. Instinctively she thought that sodomy was
wrong, or at any rate not natural, and it upset her being asked to submit to it.
The husbands evidence on the topic was to the effect that the wife was never keen on this practice, but said All right, if
you want to you can, and on the other occasions when she objected, he accepted her objection. He was under the impression
that sodomy was one of those things one did. He had had this impression since the age of twenty-one or twenty-two, and, since
that age, regarded sodomy as a normal part of love-making between married persons, and not merely as a method of sexual relief
at times when other such relief was impossible. He himself liked it. He agreed that, prior to his having sexual relations with his
wife, she was sexually wholly inexperienced, and knew nothing about the practice of sodomy until he explained it to her. He
asserted that it took place more than three times during the marriage.
The wifes evidence to the effect that her husband told her that sodomy was her duty if he wished it, and that it was normal
between married couples, was not challenged. Indeed, it would have been difficult to do so successfully in view of the husbands
own evidence.
The learned judge refers to the methods of persuasion adopted by the husband in these terms:

I accept that persuasive arguments were placed before her by her husband, and that those arguments prevailed when
they ought not to have done. The persuasive arguments, I think, were these; that this was the kind of thing other married
couples did, that some women at any rate enjoyed it, and that in the case of these other married couples the wives permitted
it. I accept that the husband honestly believed in the truth of that which he was saying to his wife; in those circumstances,
no question of fraud arises. Having placed those arguments before his wife, he seems to me to have allowed her either to
accept them or to reject them. It was said that where a young wife is, in effect, told that her matrimonial obligations
include the commission of sodomy she should be held not to consent to sodomy but merely to submit to it. I can imagine
cases where the matter was so put, that a wife, particularly a young wife, might be held to be merely submitting to the
domination of the husband; it seems to me to be a question of fact in 752 any particular case, and a question of degree. In
this case I am satisfied that this particular wife had complete freedom to choose whether she would permit sodomy or
would not permit sodomy, and in the exercise of that complete freedom, she did permit it, in other words, she consented to
it.

It will be seen that the learned judge omits, from his statement of the husbands arguments, the argument that sodomy was a
normal thing between spouses, and was part of the matrimonial duty of the wife if the husband wished it. He refers to this aspect
of the matter only by way of repeating one argument put forward for the wife. The persuasive arguments of the husband which
the judge considers are simply that some other married couples did it, some wives permitted it, and some women enjoyed it. He
does not deal with the effect of the representation that it was a matital obligation on the part of the wife and that it was her duty to
submit; matters which bear heavily on the quality of the consent thereby obtained. The judge simply said that there might be
such a case leading to submission to a dominant husband, and this would not be consent; but, he says, this wife had complete
freedom of choice.
With the greatest respect to the learned judge, I fear that I am unable to agree. The wifes unchallenged evidence is that she
was told that it was her marital duty to allow her husband to do this thing, whether she liked it or not, and she believed him,
though disliking the act. I do not think that it can be said, in these circumstances, that she had complete freedom of choice. Yet
it is on the basis that she had such freedom that the learned judge finds that she consented and withholds relief.
The question still to be considered and decided in this case is whether a wife who is persuaded into believing that sodomy is
normal between married couples, and who, although disliking it, submits to her husbands desire for sodomy, under the belief,
induced by him, that it is part of her marital duty to submit, is to be debarred from relief when she discovers the truth, on the
ground that she consented to the act. Curiously enough, it is conceded that she would not be so debarred if her belief had been
induced by her husbands fraud or wilful deceit; but it is said that here the husbands belief was genuine and sincere that sodomy
between spouses was normal and proper, and that makes all the difference.
I regret that I am unable to follow this reasoning. What we have to consider is the quality of the wifes consent. Was it a
real consent or not? What difference can it make to the quality of this wifes consent whether her husbands beliefs on the
question of sodomy and his consequent representations to her between spouses were genuine or false? What seems to me to be
chiefly important on this issue is whether the wife knew that she was being asked to do something which, even as between
husband and wife, was wrong; though I do not think that she would need to know that it was also a crime. If the basis of the rule
barring relief in the case of consent be that a wife ought not to be allowed to plead her own wrong, then it seems to me to follow
that she ought to know at the time the act was committed that it was wrong. The wife here said that she did not know that it was
wrong, though she thought it was unnatural, and this evidence was unchallenged and uncontradicted; and it must be remembered
that we are here dealing with a young girl aged only eighteen at the time who admittedly knew nothing of sodomy until her
husband explained it to her.
A consent given by the wife to the perpetration of this act on her by her husband ought not to be held to be effective to bar
her claim for a divorce unless it be a real consent (Bampton v Bampton). By real consent, I mean a consent given with
knowledge of the factors which ought first to be considered; and in particular that it is a wrongful act even between spouses.
Consent is an act of reason accompanied with deliberation, the mind weighing, as in a balance, the good and evil on each side.
See Story, Commentaries On Equity Jurisprudence (3rd English Edn, 1920) $S 222. There was no such consent in this case.
753
How does the matter stand on the authorities? There are two only, Statham v Statham, and Bampton v Bampton. In Statham
v Statham a wife petitioned for divorce, founding on a single act of sodomy by her husband. She also alleged acts of cruelty both
sexual and non-sexualthese apparently being intended as grounds for a judicial separation should her claim for divorce fail.
The case was tried before a jury who found, inter alia, that the act of sodomy had been committed. But there was no
corroboration of the wifes story. The husband denied it, and the jury was not warned by the trial judge of the desirability of
corroboration, whether the wife were an accomplice or not. This failure on the part of the trial judge to warn the jury against the
danger of finding that sodomy had been committed in the absence of corroboration of the wifes story, was held to be fatal to the
jurys verdict. So far as sodomy was concerned, therefore, the wifes case was at an end, but there remained the question whether
there should be a new trial. In relation to her charges of cruelty, the court held that there should not, for reasons which are set out
in the judgment of Russell LJ. As regards the sodomy, the court held that, as she had consented, she could not obtain a decree of
divorce in respect of it, and accordingly dismissed her petition for divorce. There seems to have been no issue between the
parties on the subject-matter of this consent. Lord Hanworth MR said ([1928] All ER Rep at p 221; [1929] P at p 139):

The wife was an accomplice. Furthermore, there is, according to her own story, strong evidence of consent on the part
of the wife. Her evidence proves that what was desired was explained to her, and although in its very essence every person
must realise that such an act is against nature, she accepted it until she found that it caused her pain.

Greer LJ said ([1928] All ER Rep at p 224; [1929] P at p 145):


This falls short of showing that she did not understand that what she was consenting to was wrong. She must have
known that it was wrong, improper and unnatural, and she does not venture to say that she did not. There is no indication
of any sort of duress exercised by the husband, and it is noteworthy that when he suggested doing it again, she maintained
her refusal without difficulty. In my judgment, the learned judge ought to have dismissed the claim for divorce on the
ground that the wife was a willing party to the act on which she founded her claim.

Russell LJ who agreed with the judgments both of the Master of the Rolls and of Greer LJ said ([1929] P at p 156):

Her evidence makes it clear (1) that her husband explained to her quite plainly the exact physical act which he wished
to do upon her; (2) that she assented and placed her body at his disposal for the purpose of that act being committed; (3)
that no compulsion of any kind was brought to bear upon her; and (4) that the only reason given by her for refusing
subsequent invitations to a similar act was fear of pain. It thus appears that the wife was a consenting party to the only act
upon which a decree for divorce could be founded. It is immaterial whether or not she knew that the act to which she
consented was called sodomy or that it was a crime. The act was accurately explained to her beforehand, she must have
known that it was against nature, yet she consented of her free will to its commission. In these circumstances it is
impossible for her to obtain a decree for divorce based solely upon an act of the husband to which she was a consenting
party. The husband is entitled, in my opinion, to have the petition for divorce dismissed.

There is here, I respectfully think, no analysis of the necessary elements of an 754 effective consent for present purposes.
Furthermore stress is laid in Stathams case by Greer LJ on the absence of any sort of duress ([1928] All ER Rep 219 at p 224;
[1929] P 131 at p 145) and by Russell LJ on there having been no compulsion of any kind ([1929] P at p 156). As I hope to
show, compulsion of a potent kind existed here; but for the moment it is enough to say that to my mind Statham v Statham is
certainly not an authority for saying that, if a wife knows or thinks that sodomy by her husband is against nature, but nevertheless
submits, that is enough to prove a real consent to the act on her part.
The second authority is Bampton v Bampton. In that case Hodson LJ posed the question ([1959] 2 All ER at p 767) as
being: Was there a real consent? on the part of the wife. She had submitted to several acts of sodomy by her husband over a
period of two years, and said she did not repel him because he was always so affectionate at these times, and because she was
worried that she might lose him if she did not let him try. The Court of Appeal held in the circumstances that the trial judge was
entitled to find that there had been a real consent. No definition of such a consent was given, but it is to be observed that there
was no such misrepresentation of a wifes marital duty as there was in the present case; nor any insistence on sodomy being
normal as between spouses.
In the circumstances, I am clear in my own mind that the present case may be considered free from any constraint of judicial
authority. We are bound, of course, by the two cases cited to hold that consent is a bar to relief, although the Act of Parliament
does not say so. Whether such a condition ought to be read into the Matrimonial Causes Act, 1950, is a matter that the House of
Lords may one day have to consider. If the consent must be a real consent, then, in my view, it is clear that one cannot
conclude that, merely because the wife submitted, or said, Well, if you like to try, you can, the problem is solved. This may be
consent, but it is not necessarily real consent. The addition of that adjective involves looking closely at the circumstances under
which the wife submitted. If she consented as the result of some fraudulent misrepresentation made to her, or as the result of
force or threats of force, it is conceded (and I think rightly) that this would not be a real consent. For then the wife would not
have full freedom of choice. But there can be other pressures brought to bear on a wife just as potent as those just mentioned and
making just as large an inroad on freedom of choice. Indeed, to tell a wife that what happened here was normal, that it was part
of her marital duty to submit, and to submit whether she liked it or not, is to bring very heavy moral pressure on her; and that is
what happened here. A real consent involves, as I say, a knowledge of the relevant factors bearing on the question whether
consent should be forthcoming or not, and one of the chief of these is whether the act to which consent is sought is right or
wrong. The uncontradicted evidence here is that the wife was deceived into thinking that it was not wrong, and it is immaterial,
in my opinion, that that gross misrepresentation was innocent on the husbands part.
Accordingly, my view is that there was no real consent on the part of the wife in this case. Moreover, since she did not
know that a matrimonial sin had been committed against her, no question of condonation can arise. From the time that she
discovered the true nature of what had been done to her she refused to have anything more to do with her husband. I would allow
the appeal and pronounce a decree.

RUSSELL LJ. I will consider first the authorities dealing with the question in what circumstances (apart from condonation) is a
wife debarred from a decree when she establishes that her husband has committed sodomy on her, a fact which under s 1(1) of the
Matrimonial Causes Act, 1950, entitles her to present a petition for divorce. It is established by a decision of this court 755 in
Statham v Statham, that, although s 4 requires the court to be satisfied that, where the ground of the petition is adultery, the
petitioner has not in any manner been accessory to or connived at or condoned the adultery, and, where the ground is cruelty, has
not in any manner condoned the cruelty, and makes no reference to cases in which the ground is sodomy or bestiality,
nevertheless by analogy consent by the wife to sodomy on herself must be regarded as an absolute bar to relief. The question of
the wifes consent in that case became relevant under two heads: first, whether, she being a consenting accomplice, the jury
should have been warned against the danger of accepting her evidence of the offence (which was denied by the husband); second,
whether the correct course was to order a new trial on the ground of misdirection, or simply to set aside the decree in her favour
on the ground that her consent raised an absolute bar against relief. The wife was aged twenty and the husband forty-nine. The
wifes evidence was that there was one occasion only; the husband explained to her what he was suggesting; she did not try to
prevent him but adopted an attitude convenient for the act; her resistance due to pain came only after the technical attainment of
the act, and thereafter for the same reason she refused suggestions of repetition. She said of the one occasion: I did not try to
prevent him. I did as I was told. I did not know much about things when I was married. As reported there does not seem to
have been argument that she had not consented, or that her evidence because of consent would not, had the case been a criminal
prosecution, have called for a warning to the jury about lack of corroboration. Lord Hanworth MR when considering the
question of the direction to the jury, said ([1928] All ER at p 221; [1929] P at p 139):

There is, according to her own story, strong evidence of consent on the part of the wife. Her evidence proves that
what was desired was explained to her, and although in its very essence every person must realise that such an act is against
nature, she accepted it until it caused her pain.

Later, when considering whether a new trial should be ordered, he included as one factor in deciding merely to rescind the wifes
decree ([1928] All ER Rep at p 223; [1929] P at p 143) the evidence of the consent of the wife to the act at the time. Greer LJ
decided in terms that the rules as to connivance and condonation apply when the ground alleged is sodomy, notwithstanding the
form of the statute, having remarked ([1928] All ER Rep at p 224; [1929] P at pp 144, 145) that

it is difficult to suppose that the court is bound to grant a decree for dissolution of marriage to a wife who upon her
own evidence has consented to the act of which she complains.

He continued ([1928] All ER Rep at p 224; [1929] P at p 145):


It seems to me impossible to say that where two people commit sodomy together, either of them is entitled to ask the
court for a decree based on an act to which he or she was a party The evidence given by the wife in this case gives rise
to the question whether the judge ought not to have come to the conclusion that she was a willing party to the act of which
she complained, and was, therefore, disentitled to a decree of divorce in respect of it. It is quite clear that the act was not
done, as she alleges in her petition, by force or against her will This [by which GREER, L.J., referred to the evidence]
falls far short of showing that she did not understand that what she was consenting to was wrong. She must have known
that it was wrong, improper, and unnatural, and she does not venture to say that she did not. There is no indication of any
sort of duress exercised by the husband, and it is noteworthy that when he suggested doing it again she maintained her
refusal without difficulty. In my judgment the learned judge ought to have 756 dismissed the claim for divorce on the
ground that the wife was a willing party to the act on which she founded her claim.

Russell LJ in considering whether, if the sodomy were proved the wife could obtain a divorce, said that it was necessary for that
purpose to scrutinise carefully the evidence of the wife. He summarised the result of such scrutiny in four sentences ([1929] P at
p 156), of which the third was:

Her evidence makes it clear (3) that no compulsion of any kind was brought to bear upon her.

He then proceeded ([1929] P at p 156):

It thus appears that the wife was a consenting party It is immaterial whether or not she knew that the act to which
she consented was called sodomy or that it was a crime She must have known that it was against nature, yet she
consented of her free will to its commission. In these circumstances it is impossible for her to obtain a decree for divorce
based solely upon an act of the husband to which she was a consenting party.

In Davidson v Davidson the husband appealed to the Divisional Court from a finding by magistrates of persistent cruelty.
The main complaint of the wife was that the husband had committed a number of acts of sodomy on her, and she said that she
consented; she was only a very young woman and she alleged that the husband persuaded her that it was normal sexual
intercourse or at any rate a kind of normal sexual intercourse, and that it was only when she discovered that it was not normal that
she refused to allow him to do it further. The husband denied sodomy. There was no corroboration of the wifes complaints. A
new trial was ordered on the grounds, inter alia, that the justices did not appear to have directed their minds to the desirability
(though not absolute necessity) of corroboration in cases of this kind, and further ([1953] 1 All ER at p 613) they do not seem to
have considered at all the question, which must arise in charges of this kind, of acquiescence by the wife. Reference was then
made to the reference by Lord Merriman P in D B v WB ([1935] All ER Rep 428 at p 429; [1935] P 80 at p 84), to acts of this
sort which cannot have occurred without at least the assent of the wife. Karminski J (with whom Collingwood J concurred) then
said ([1953] 1 All ER at p 613):

It is true that the assent of a wife, especially a young wife, cannot be a true assent if the acts are forced on her, either
literally in the physical sense, or by some fraudulent persuasion that such conduct was only one of the normal incidents of
married life and this was unobjectionable. But that aspect of acquiescence, which, in my view, is vital in these cases, does
not seem to have been considered by the justices at all.

Lawson v Lawson was also in this court, and was a case in which the justices found that the wife had not consented to the
acts of sodomy of which she complained. The report does not disclose the evidence on which that finding was based. Lord
Goddard CJ said ([1955] 1 All ER at p 343):

In the present case the justices found that the wife was not a consenting party. That means that the wife, being forced
by her husband, submitted to these acts, but that she was not a willing party if she was not a consenting party in the
sense of having voluntarily allowed her husband to satisfy himself in this unnatural way .

Later he referred ([1955] 1 All ER at p 344) to the Statham case as one which

depended on very peculiar and special facts. There is no doubt that there the husband explained to the wife what he
desired to do to her and 757 she readily consented That is very different from the present case, where the woman,
although she may have submitted, was not a consenting party.

Hodson LJ considered ([1955] 1 All ER at p 344) that it was open to the justices on the evidence to find that

the wife was not a consenting party and that the acts were inflicted on her against her will.

He also remarked ([1955] 1 All ER at p 344) that in the Statham case no compulsion of any kind was put on the wife.
Finally, Bampton v Bampton, in this court, was a case in which consent by the wife to sodomy on herself was held to bar her
from relief, following the Statham decision. The facts need not be set out in detail. Sometimes the wife permitted the act, and
sometimes she refused it. When she permitted it, or did not fight it off, it was partly because her husband was specially
affectionate to her in relation to the acts and she was glad of that affection, and he bragged of what other women did, and she was
worried in case she might lose him if she did not let him try. Hodson LJ said ([1959] 2 All ER at p 767) the question was whether
there was real consent, and that in considering that question the relationship of husband and wife lay in the background. He
continued ([1959] 2 All ER at p 767):

It should be recognised that the relationship of husband and wife being what it is, and the obligations of one to the
other being what they are, it is not readily to be taken against a wife especially a young wife, that there has been real
consent.

Hodson LJ was unable to disagree ([1959] 2 All ER at p 768) with the finding that on the evidence there had been real consent;
he said ([1959] 2 All ER at p 768) There was no question of consent being compelled by fraud or duress; there was no threat,
nothing of that kind. Sellers LJ said ([1959] 2 All ER at p 772):

One of the strong points advanced was that the motives of the wife were motives which might well affect a
wife who felt a duty and obligation towards her husband, and a desire to retain him on some sort of matrimonial basis and
to make the marriage work. Whether it be the right view to take or not that the motives might be said to be excusable or
understandable, what they do clearly establish is that, for reasons which the wife thought good, she did eventually consent
to several acts of this type of intercourse I think that the facts completely negative, and indeed there is no
suggestion of, any force or threat.
Harman LJ considered ([1959] 2 All ER at p 772) that the evidence of consent was a great deal stronger than in the Statham case.
Against the background of that authority it is necessary to consider in some detail the evidence in this case to see whether it
demonstrates a situation in which this wife must be barred from the relief to which the statute prima facie entitles her in respect
of the sodomy committed on her by the husband. In considering that evidence, I bear in mind the view of Hodson LJ ([1959] 2
All ER at p 767), that it is not readily to be taken against a wife that there has been a real consent.
The wife gave the following evidence about the sodomy. She said that she and the husband had met in July, 1958, and
became engaged in December; that she was eighteen and he twenty-three or twenty-four; that she knew about sex in the sense of
knowing the facts of life, but had no experience, and that at some stage they started sexual intercourse. She said that on one
occasion before the marriage, about April, 1959, the husband said that he wanted to bugger her and that she had said no, for she
did not think it right, and did not want it; 758that he said that his last girl friend had always allowed it, and that he thought she
too ought to allow it. She said that it did not take place. She gave evidence concerning another occasion soon after the marriage
in June, 1959:

He said again that he wanted to bugger me (in those words) when in bed. Q.What was your response to that? A.
No. I did not like it. Q.Did he say anything to that? A.He more or less implied that it did not really matter whether I
liked it or not; it was my duty, I was his wife He mentioned his girl friends again

She said that it did happen on that occasion, which she put at August, 1959, when she would presumably have been about four
months pregnant. She said:

I hated it. I found it very uncomfortable and very painful. Q.Did you say anything about that to your husband? A.
Yes, I did. Q.How long after? A.While it was happening and afterwards, and he said that in time it would not hurt
so much, and I said I did not care; I did not want it.

She said that it happened on two other occasions, one in September, 1960, and one earlier just after the baby was born. She said
that apart from those occasions the subject was mentioned all the time whenever he wanted to make love to me I
suppose. She was asked:

Q.Apart from those three occasions, did you ever allow it? A.No. Q.When he asked for it on other occasions
and you did not allow it; first of all what did he say? A.He said it was normal; everyone did it, and I should do it. Q.
When you say everyone; did he specify anyone in particular? A.He said his friends wives allowed it. Q.You told my
Lord he said to you that his friends wives allowed it; that everybody did it, that you ought to allow it; what did you believe
about that? A.I was not sure what to believe. I thought if what he said was true, it must be true. Q.Just answer Yes
or No to this question. At that time did you discuss it with anybody? A.No. Q.Now, you told my Lord what he said,
that you ought to allow it and so on; when you did not allow it, what did he do apart from sayingwhat was his attitude?
A.He used to behave like a child who was not given a sweet when it wanted one. Q.In what way? A.He was very
sulky; he would not talk to me. He would not let me go near him. Q.Are you speaking of while you were still in bed
when this conversation was taking place or after? A.Yes, while we were still in bed. Q.When he said he would not let
you go near him; what do you mean, you were in bed with him? A.Because I felt maybe I was being silly with him, that
I was wrong, and that it was normal, and I was not being a good wife. I would sometimes try and apologise, and say I am
sorry, I just do not like it, and he would turn his back on me, and he would just behave as though I was not there. Q.On
the occasions you have refused and that was his reaction, did you and he have normal intercourse? A.No. Q.How
often on an average did you have normal intercourse throughout the marriage? A. Well, once, possibly twice, a week. Q.
You described his attitude towards you in bed after your refusals; what was his attitude on the whole the following day
after you had refused? A.He was very bad tempered, and would find fault with whatever I did.

The wife said that after the baby was born she explained to the husband that the gynaecologist had warned against intercourse for
about two months until after she had had a check-up. She said:

He said in that case he wanted to bugger me and I felt I was being unfair to my husband if I did not allow him to, so I
allowed him.

She said also that he had asked her to do this previously since the birth. The actual incident she put in March. She was asked in
connexion with the third episode in September, 1960:
759

Why did you allow it on that occasion? and she replied: To keep the peace, because he got to the stage that
whatever I did was wrong, and I thought maybe I could help matters if I allowed it this time.

When asked whether she said anything to her husband about her feelings on these other occasions, she replied: I said I still
hated it and I could not see how I could ever come to like it at all. Referring both to sodomy and oral intercourse at the same
time, she said:

I did not know whether what he told me was true or right. I disliked it and I did not feel it was right, but if he said it
was right, I felt I should trust him; he was my husband.

Immediately before that, in connexion with that oral intercourse, she had said that when she said No to the husbands
suggestions,

he said I was being silly he would try to persuade me just by saying he wanted it, it was my right and my duty
as a wife to obey him; I had in the marriage service had obey left in and he said that is what I should do.

Her evidence was that the husband was trying to get her to pleasure him by taking his penis in her mouth until his semen
was ejaculated into her mouth. She said that she took it in a few times, but never kept it till ejaculation, which she refused to
allow. Later, when she referred to a practice which she said he requested that she should during intercourse insert her finger in
his rectum, she said she did it because again he said it was my duty to do so. Thus far, I would be prepared to infer that her
evidence about references to duty related also to sodomy, though on a strict reading of her evidence so far she has not associated
an express call to duty with sodomy.
In cross-examination she denied that sodomy took place many times before and after marriage when she was menstruating.
She was asked:
Q.Now, would this put it fairly, that although you did not like it, to please him you permitted itreluctantly? A.
On three occasions, yes, very reluctantly. Q.Well, I am not suggesting that you did like it, or that you ever said so, that is
not your husbands case; but your permission did go to this extent that on occasions you in fact applied some vaseline to
that area, did you not? A.No. Q.Are you sure? A.No. I am absolutely sure I did not. Q.Again, I think I must
put this as there appears to be some difference between you about it. Your husband says, what we in fact say here is he can
remember two occasions when you were unwilling to permit him to try to do this? A.I Q.And that on those two
occasions he left it there, he did not do anything more about it; would you agree with that? A.I am not quite with you.
Can you phrase it again, please. Q.Yes. Your husband says there were only two occasions when you refused altogether
definitely to permit it? A.Oh no, many more, many more. Q.And on those occasions he accepted your refusal and did
nothing more about it? A.There were a lot more occasions when I refused, many more.

In the course of cross-examination it was suggested to the wife that the husband had suggested genital kissing (and that it had
been acceptable to her), but had only once suggested ejaculation of his seed into her mouth; she insisted in answer that the latter
had been often suggested. It was not suggested that she ever allowed the latter. She denied that her husband ever inserted his
finger into her rectum so as to stimulate her, but agreed that at his request she did it to him. The gist of the cross-examination so
far as now relevant was as follows:

Q.You did so because you knew it caused him pleasure ? A.He said it was my duty to give him pleasure. I
thought if that was my duty it is right I should do so.

The judge then asked some questions:

Q.In relation to some of that that you say your husband asked you 760 to do, did it not ever occur to you that it was
wrong? A.By instinct, yes, possibly it did. I think it did, not wrong, but not natural. Q.Well, not natural. Did it upset
you at all being asked to do these things which you regarded as not natural? A.Yes, it did upset me. Q.If your
evidence to me is correct, you were asked a great many times? A.Yes, I was. Q.For a variety of things which you
regarded as wrong, or rather regarded as unnatural? A.Yes. Q.Well now, I understand, of course, that at first you
might try to accommodate yourself and accept your husbands guidance, I understand that; but as time went on, did not you
realise that what he was asking you to do was simply disgusting you more and more? A.Yes, I did. Q.Tell me this.
Your mother, you told me, had told you about what was called the facts of life when you were a girl? A.Yes, Q.Why
did not you go and ask your mother whether this might be right or wrong? A.Because I was much too embarrassed to.
Q.You were a young girl, your mother had discussed with you matters of sex before in order to tell you the facts of life.
Why did not you go and ask her if a girl was really expected to put up with this kind of thing? A.As I say, because I was
too embarrassed, and because also I did not feel it was right to discuss what my husband and I did in the privacy of our
bedroom with anyone, whether it was my mother or with anyone; I felt what happened there must surely be private. Q.
Well now, take the most horrible, much the most horrible of all I should have thought, one I can refer to by saying to you
the swallowing. A.Yes. Q.You must have realised surely that no woman could be expected to put up with that kind
of thing, did not you? A.No. I did not realise because my husband used to say to me, and mention some names which I
would rather not say, he used to say so and so, Louise, and so on, used to do it. Q.Well, the fact that you accepted what
he said as true, that meant that some other couples at any rate had been prepared to disclose the sort of things they did in
bed, did not it? A.Yes, well (a pause). Q.Well, you do not need to imitate them, but knowing that, did not you
think it would be practicable, at least, to ask your mother if you could be expected to put up with that sort of thing? A.
No. Q.You did not? A.No. I could not have gone to my mother and asked her. I just did not feel it was right. I had
married the man, it was not right to go to my mother and ask her.

The husband said in evidence that sodomy in fact first took place on three or four occasions before marriage when she was
menstruating. He said:

She did not like the idea I explained other people did it and so on, and that a girl friend of mine had done it and
liked it, and she finally said: All right then, try it, which I did.

He said that it often happened after marriage when she was not in a condition for ordinary intercourse; that sometimes she
applied vaseline. He said:

She was never keen when I asked her if we could do it, but she always used to say: All right, if you want to you can,
and that is how it started and it went on from there.

He said that there were one or two occasions when she persisted in her objection, and he said: All right. He said of the other
practices that she co-operated without saying whether she liked or disliked them, and that she never complained of the way in
which he treated her. In cross-examination he said in relation (inter alia) to sodomy: My wife has (now) been told very
different things by her parents. The husband in many passages insisted that he regarded sodomy on a wife as a perfectly
normal thing. He said that he did not exercise an awful lot of persuasion on her to permit sodomy; he had no scruples about it
because I was under the impression in fact that it was one of those things that one didthough he said that he had not done it
habitually with others. He said that the wife did not like it but did not mind it; that when asked if 761 she was enjoying it she
would indicate that she would do so at some future time. He said that he did not consider that a wife ought to consent to sodomy
any more than she ought to consent to ordinary intercourse. On the very few occasions, according to him, when he wanted to
commit sodomy and she refused, he said he was not really angry, I do not think, I probably (pause) would say that I was a bit
upset frustrated.
Now, taking the transcript alone, there are on various matters considerable differences between husband and wife,
particularly the frequency with which sodomy was accepted by the wife. The judge dealt only in a very general fashion with their
reliability as witnesses, and, of course, it must be appreciated that his remarks are referable not only to the evidence about
sodomy but also to the alleged events of non-sexual cruelty, which occupied a good deal of the trial. Early in his judgment, he
said this:

It is plain that whatever the result of this case may be, there was a very large measure of truth indeed in both the
evidence of the wife and the evidence of the husband, as there was for that matter in the evidence of the independent
witnesses on the one side and on the other. Indeed, I would go further and say that I am satisfied that all the witnesses
before me have done on the whole their best. So far as the husband is concerned, he had a most unattractive story to tell. I
was impressed by the candour with which he told it, and, although I do not seek for one moment to suggest that the wife
was untruthful, I am disposed on the whole to prefer her husbands evidence to hers where the two are in conflict. I am not
sure that even that preference extends to every portion of the evidence, but in general it does, and I would like to to be
understood that in so far as the wifes account diverges from what I believe to be the true facts of this case, it is not because
she has wilfully tried to deceive me, but because in a matter in which emotions are as deeply involved as they are in a case
of this kind it is hard to put oneself outside the arena and determine coldly what is the precise truth of the affair.

The judge dealt with the evidence of sodomy by a summary of what each said, without saying which he accepted on points
of divergence; except that he did say that he did not think the husband knew she objected when he asked for sodomy. He
accepted the husbands evidence that in substance her attitude was: You can, if you want to. As I read his judgment, he did not
reject her evidence that she did it unwillingly, finding both the idea and the act distasteful. Nor, it seems to me, did he reject her
evidence, which was scarcely challenged in cross-examination, that it was in one way or another represented to her by the
husband that participation in sodomy was to be expected of a wife as a normal part of married life and matrimonial obligations.
Indeed, his judgment, I think, proceeds on the basis that this was so, for he said this:

I accept that persuasive arguments were placed before her by her husband, and that those arguments prevailed when
they ought not to have done. The persuasive arguments, I think, were these: that this was the kind of thing other married
couples did, that some women at any rate enjoyed it, and that in the case of these other married couples the wives permitted
it. I accept that the husband honestly believed in the truth of that which he was saying to his wife; in those circumstances,
no question of fraud arises. Having placed those arguments before his wife he seems to me to have allowed her either to
accept them or to reject them. It was said that where a young wife is in effect told that her matrimonial obligations include
the commission of sodomy she should be held not to consent to sodomy but merely to submit to it. I can imagine causes
where the matter was so put, that a wife, particularly a young wife, might be held to be merely submitting to the
domination of the husband; it seems to me to be a question of fact in any particular case, and a question of degree.
762

The judge took the view that the wife had in this case complete freedom of choice in that, on her own evidence, she sometimes
refused and sometimes permitted the act, and in that she was not the sort of person who was dominated by the husband, and that
consequently it was the inescapable conclusion that she consented and was barred from relief.
Now is it the law in the light of authority that relevant consent exists even when a young and inexperienced wife is induced
to participate by representations by the husband whom she loves that it is her wifely duty? Must there be physical compulsion of
some kind, or threats, or some degree of dominance of the husband over the wife, or dishonesty on the part of the husband in
making those representations? I think not. I cannot see any ground as a matter of policy for saying that a wife who is persuaded
to participate by such representations is not to be regarded as consenting if the husbands state of mind is fraudulent, but is to be
so regarded if he speaks in innocence or ignorance. It must be her state of mind that is relevant, rather than his. However
innocent may be the husbands state of mind, his matrimonial offence is absolute under the statute; but the more he thinks that the
practice is normal and right the more likely, I should have thought, that his representations would be persuasive, with no sense of
guilt to hamper him. If a wife is led to believe by her husband that that is the situation (even though before marriage she thought
that it was not right, and continued to feel that it was not natural) and permits the acts for that reason, and for that reason only, I
think that she is not consenting in a manner which ought to debar her from relief, but is submitting to the most persuasive type of
pressure by her husband, the fulcrum for which is the very relationship of husband and wife. The fact that her antipathy to the act
leads her sometimes (I care not whether rarely or frequently), despite the pressure of such persuasion, to refuse, does not mean, in
my view, complete freedom of choice. It only means that sometimes she yields to the pressure of the persuasion in spite of
antipathy, and sometimes she does not.
I am not to be taken as saying that the same result would follow if a wife formed the view that acquiescence in sodomy was
part of her married lot and duty independently and without being persuaded to that view by the husband. The very fact that such
persuasion comes from the husband will commonly be most compelling. Nor am I to be taken to say that, if a wife so persuaded
participates in the practice willingly in the sense that she finds it agreeable, she is not really consenting. Such circumstances are
not present here. We were warned of the dangers of going beyond the matters which are normally required to vitiate consent in
other branches of the law. It was said that many cases may arise in which a wife will seek to escape the bar by alleging that the
words or conduct of a husband amounted to persuasion that matrimonial duty included sodomy, and that this would be contrary to
the public interest. I am not impressed by this argument. Presumably the same danger exists in cases in which the husband
unsuccessfully denies sodomy, or unsuccessfully denies the representations, without establishing that the representations were
made in the innocent belief that they were true. Moreover, if the public interest be regarded, I should have thought that a
constriction of the circumstances in which a husband can inflict this degradation on his wife and be subsequently immune from
its consequences as a matrimonial offence is to be welcomed as a discouraging factor.
Am I prevented from arriving at that conclusion by authority binding on this court? I think not. I need not repeat the facts
in the Statham case, where there was no evidence of the wife being persuaded by the husband that what he proposed was a
normal ingredient of love-making between married couples, or part of her wifely duty and not wrong. General remarks in the
judgments must be considered in the light of the particular facts for decision, and it would, in my judgment, not be right to take
the Statham case as a decision of this 763 court that appreciation that the act is unnatural is sufficient, short of compulsion by
force or fear, to raise the absolute bar of consent. In Davidson v Davidson the divisional court was considering whether there was
ground for a new trial and so decided. In referring to fraudulent persuasion it may well have been that the court did not address
their minds to the possibility of innocent persuasion, and this cannot be taken as a decision by the Divisional Court that innocent
persuasion by the husband can never vitiate consent; nor was Lawson v Lawson such a decision. In Bampton v Bampton, though
there were references to lack of compulsion, duress and threats, and references to a wife perhaps feeling a duty and obligation to
her husband as not being inconsistent with consent, there was no suggestion that the wife was led to that feeling by the
persuasions of the husband that it was normal and right and part of her duty, or even that she thought that it was not wrong.
Moreover, in part at least, she welcomed these occasions as producing in him demonstrations of affection which were welcome to
her.
I consider therefore that authority is not against my conclusions. This court in the Statham case imported this question of
consent from the general rules as to connivance established in the practice of the ecclesiastical courts before the Matrimonial
Causes Act, 1857, which rules in relation to adultery are expressly stated in the statutes. On this basis it is to be observed that the
application of the bar of connivance at adultery is based on two general principles: one is that volenti non fit injuria (see per
Denning LJ in Douglas v Douglas ([1950] 2 All ER 748 at p 752; [1951] P 85 at p 96)); the other is that the complaining spouse
must come to the court with clean hands and that it would be unconscionable to give relief to one who has been willingly blind to
or has encouraged the matrimonial offence (see per Tucker LJ in Woodbury v Woodbury ([1948] 2 All ER 684 at p 690; [1949] P
154 at p 165)). I observe also the use of the phrase corrupt intention in connexion with connivance: see the judgment of this
court in Churchman v Churchman. Though such considerations applicable to the subject of adultery are not necessarily
applicable to the subject of sodomy on a wife, nevertheless I cannot but feel that they are of some relevance and lend some slight
support to the view which I have formed. It scarcely lies in the mouth of the husband to say of the wife volenti non fit injuria,
when he has successfully persuaded her that it was no such thing. It scarcely lies in his mouth to say that her hands are not clean,
and therefore, it would be unconscionable to grant her relief in respect of the matrimonial offence, when the state of her hands is
due to his having successfully persuaded her that it was not a matrimonial offence. It scarcely lies in his mouth to label as corrupt
the state of her mind when he had successfully induced in it the view that acquiescence was a normal wifely gesture.
The question of condonation I can deal with shortly, without reference to the facts. Condonation involves an intention to
forgive a wrong done to one spouse by the other. Granted the view of the facts of this case already outlined I see no place for
condonation. Thanks to the husband implanting in his wifes mind his views of what was proper and normal, she was persuaded
by him that the sodomy he committed on her was the very opposite of a wrong of which she was entitled to complain and was
something which required no forgiveness. It seems to me impossible in those circumstances to talk of the wife condoning the
sodomy. Of course, on the facts, her eyes were not opened until after she had left the husband, when, in his own words, she was
told very different things by her parents.
764
In my judgment, the wife is not debarred from a decree on the ground of the husbands sodomy on her, and her appeal
should be allowed. I agree with my brethren on the question of cruelty.

Appeal allowed. Decree nisi to date from date of this judgment. Leave to appeal to House of Lords refused.

Solicitors: Dehn & Lauderdale (for the wife); Hart-Leverton & Co (for the husband).

Henry Summerfield Esq Barrister.


[1963] 2 All ER 765

Morton v Confer
CRIMINAL; Road Traffic

QUEENS BENCH DIVISION


LORD PARKER CJ, HAVERS AND EDMUND DAVIES JJ
16 MAY 1963

Road Traffic Being in charge of motor vehicle when unfit to drive through drink No intention of driving until fit to do so
Burden of proof Road Traffic Act, 1960 (8 & 9 Eliz 2 c 16), s 6(2)(i).

The respondent was found at 10.50 pm slumped over the wheel of his car asleep by a police constable. The constable woke the
respondent who immediately turned on the ignition. The constable switched off the ignition and took the key away. The
respondents speech was slurred, his breath smelt of alcohol, and, when he got out of the car, he was unable to stand without
assistance. Just over an hour later, a doctor called by the police found him to be unfit to drive, and an analysis of blood and urine
samples indicated that he had consumed nearly twelve pints of beer or seven-tenths of a bottle of spirits. The respondent had left
a club eleven miles from his home at 10.00 pm. He stated on oath that, about one mile from home, the drinks hit him, that he
immediately stopped the car, and that it was his intention not to continue his journey until he felt fit to do so. An information
charging the respondent with being in charge of a motor car on a road when he was unfit to drive in that he was under the
influence of drink or drugs to such an extent as to be incapable of having proper control of the vehicle, contrary to s 6(2) of the
Road Traffic Act, 1960, was dismissed, the justices having accepted the respondents evidence. On appeal,

Held The appeal must be allowed and the Case remitted because, under s 6(2)(i)a of the Act of 1960, a burden was put on the
respondent to show that, on a balance of probabilities, the circumstances were such that there was no likelihood of his driving the
vehicle so long as he remained unfit to drive (see p 767, letter b, and p 768, letters d and e post), and the justices were not entitled
to find that defence proved merely by accepting what the respondent said but should have considered whether, having regard to
all relevant evidence, including the respondents testimony, they were satisfied that there was no likelihood of the respondents
driving the vehicle so long as he remained unfit to drive through drink (see p 767, letter d, and p 768, letters a and e, post).
________________________________________
a Section 6(2)(i) is set out at p 767, letter b, post

Appeal allowed

Notes
As to being under the influence of drink when in charge of a motor vehicle, see 33 Halsburys Laws (3rd Edn) 627, 628, para
1058.
For the Road Traffic Act, 1960, s 6(2), see 40 Halsburys Statutes (2nd Edn) 718

Case Stated
This was a Case Stated by justices for the county of Huntingdon in respect of 765 their adjudication as a magistrates court sitting
at Huntingdon on 12 December 1962.
On 7 September 1962, the appellant Arthur Thomas Morton, a police inspector, preferred an information against the
respondent, Clayson Edwin Confer, charging that, on 6 September 1962, when in charge of a motor car on a road called The
Brington to Old Weston Road, Brington, in the county of Huntingdon, he was unfit to drive in that he was under the influence of
drink or drugs to such an extent as to be incapable of having proper control of the vehicle, contrary to s 6(2) of the Road Traffic
Act, 1960.
The following facts were found. At 10.50 pm on 6 September 1962, a police officer found the respondent sitting slumped
over the wheel in the nearside driving seat (a left-hand drive) of his car, asleep. The car was stationary on the nearside of the road
but two feet six inches from the nearside kerb, and its headlights were dipped. The engine was switched off. The ignition key
was in the lock. The police constable awakened the respondent. The respondent then turned on the ignition, and the constable
switched the ignition off and took the key. The constable talked to the respondent, he found the respondents speech very slurred,
his breath smelt of alcohol and, when he got out of the car, he was unable to stand without assistance. Later, at 12.02 am at the
police station, the respondent was seen by a doctor called on behalf of the police, who was of the opinion that the respondent was
unfit to drive or to be in charge of a car through drink. The respondent said that he was of the same opinion. Blood and urine
samples taken at 12.35 am on analysis indicated that the respondent had a blood alcohol concentration equal to a minimum intake
of between 11 1/2 and twelve pints of beer or between nineteen and 19 1/2 fluid ounces (that is, 7/10ths of a bottle) of spirits. The
respondent at all material times was under the influence of drink to such an extent as to be incapable of having proper control of
the vehicle and was unfit to drive through drink. The respondent had left a club, in which he had drunk alcoholic liquor, at about
10.00 pm that evening. The club was about eleven miles from his home at Brington. When arrested, the respondent was about
one mile from his home. The respondent on oath stated that he had suddenly become aware of the effect of the alcohol he had
consumed (he said the drinks hit him), realised he was unfit to drive, and immediately stopped in the position in which he was
found by the police constable. The respondent further stated that he had no intention of driving at all until he felt that he was fit
to do so. He further stated that he had not considered pulling his car off the road or into a gateway, and that he did not know why
he had switched on the ignition after the police constable had arrived.
It was contended on behalf of the respondent that the special provision in s 6(2)(i) and (ii) of the Road Traffic Act, 1960, was
intended to meet such a case as this, and that, as the respondent had no intention of driving his car whilst unfit to drive and had
not driven it when so unfit, it could not be said that he was in charge of the car and the charge should, accordingly, be dismissed.
The justices dismissed the information and the appellant now appealed.
The cases noted belowb were cited during the argument.
________________________________________
b R v Carr-Briant [1943] 2 All ER 156, [1943] KB 607, Thaw v Segar [1962] SLT (Sh Ct) 63

Paul Wrightson QC for the appellant.


The respondent did not appear and was not represented.

16 May 1963. The following judgments were delivered.

LORD PARKER CJ. This is an appeal by way of Case Stated from a decision of justices for the county of Huntingdon, sitting
at Huntingdon, who dismissed an information preferred by the appellant, a police inspector, against the respondent, that, when in
charge of a motor car on a road, he was under the influence of drink or drugs to such an extent as to be incapable of having
proper control of the vehicle, contrary to s 6(2) of the Road Traffic Act, 1960. [His Lordship stated the facts, and continued:]
This, so far as I know, is the first case in this country which has arisen under s 6(2) of the Road Traffic Act, 1960. 766It is well
known, of course, that, theretofore, a man might be summoned and convicted of being in charge of a car because nobody else was
in charge of it, and yet he had no intention of using it at all. Subsection (2) of s 6 now includes these wordsand they are the
only ones which are material for this case:

A person shall be deemed for the purposes of this subsection not to have been in charge of a motor vehicle if he
proves(i) that at the material time the circumstances were such that there was no likelihood of his driving the vehicle so
long as he remained unfit to drive through drink or drugs

Accordingly, a burden is put on the defence, and, under the principles on which we work, it would be the lesser degree of proof,
namely, that, on a balance of probabilities, the circumstances were such that there was no likelihood of his driving the vehicle so
long as he remained unfit to drive. In my judgment, it is open to any court to consider as one of the circumstances what the
mans intention was. It cannot be said that the justices may only look at outside circumstances; they are entitled, as it seems to
me, to consider what the mans intention was. In the present case, the respondent said that he had no intention of driving at all
until he felt he was fit to do so, and that evidence was accepted by the justices.
The sole question then, as it seems to me, is whether, having accepted the respondents statement, the justices were entitled
to say on a balance of probabilities that the circumstances were such that there was no likelihood of his driving the vehicle so
long as he remained unfit. In my judgment, the justices were not entitled to find that defence proved merely by an acceptance of
what the respondent said. They would have to be satisfied further that not only was that his intention, but that there was no
likelihood that that intention would be departed from. The point becomes very clear here. The respondent stated that he had no
intention of driving until he felt that he was fit to do so. The justices would, therefore, have to be satisfied not only that that was
his intent, but that there was no likelihood of his making a mistake as to his feeling fit to do so. I am far from saying that there
are not cases when the justices might have such faith in the person concerned that they would be entitled to say not only that that
was his intent, but that they were satisfied that that intent would have been carried out, and, therefore, there would be no
likelihood of his driving until he was fit. But, so far as the present case is concerned, the justices do not make that finding
specifically. They may or may not have applied their minds to it; there is no evidence that they did. Indeed, the fact that the
respondent, when awakened, automatically used the ignition key and switched on the ignition showed the danger of the justices
accepting the intention, and indicates that that is a thing which was likely to happen albeit that he was not fit to drive.
In my judgment, the Case ought to be remitted to the justices with the opinion that I have endeavoured to state, and they
should consider the matter with a view to saying whether, in all the circumstances, they are satisfied of the second ingredient,
namely, that there was no likelihood that the intent of which the respondent had spoken to them c would not be carried out.
________________________________________
c The respondent had stated that he had no intention of driving at all until he felt that he was fit to do so (see p 766, letter f, ante).

HAVERS J. I agree. Section 6(2) of the Road Traffic Act, 1960, provides a locus poenitentiae for a person who is charged with
being in charge of a motor vehicle while unfit to drive through drink or drugs. The section provides that such a person shall be
deemed, for the purposes of this subsection not to have been in charge of a motor vehicle if he proves two things. The material
one for the purpose of this appeal is

that at the material time the circumstances were such that there was no likelihood of his driving the vehicle so long as
he remained unfit to drive through drink or drugs.
767

The respondent stated on oath that he had no intention of driving at all until he felt that he was fit to do so, and the justices
accepted that evidence. Intention of the person in charge of a car is a relevant piece of evidence which has to be considered
together with the other circumstances of the case. I agree with my Lord that it is not sufficient for a person in charge, in seeking
to discharge the onus which the subsection lays on him, merely to prove that he had such an intention. It seems to me that the
justices thought that that was the decisive factor in the case, accepting the contention of the respondent that he did not intend to
drive at all until he was fit to do so, and that they overlooked the words in s 6(2) which were that

A person shall be deemed for the purposes of this subsection not to have been in charge of a motor vehicle if he
proves(i) that at the material time the circumstances were such that there was no likelihood of his driving the vehicle so
long as he remained unfit to drive through drink or drugs

There were other circumstances in this case which tended to show that the intention, though no doubt a perfectly honest
intention, might not have been carried out. The respondent was in the car, the ignition key was in the lock, and, as soon as he was
aroused by the police officer, he switched the ignition on. In these circumstances, I am satisfied that the justices never really
directed their minds to the true question which they had to decide, namely, whether the respondent had satisfied the burden which
rested on him to prove that, at the material time, the circumstances were such that there was no likelihood of his driving the
vehicle so long as he remained unfit to drive through drink or drugs. I agree, therefore, that this appeal should be allowed, and
that the Case should go back to the justices for further hearing.

EDMUND DAVIES J. I agree with the course proposed, namely, that this matter be remitted to the justices for the purpose of
clarifying that which is now obscure in the Case Stated.

Appeal allowed. Case remitted.

Solicitors: Sharpe, Pritchard & Co agents for A C Aylward, Huntingdon (for the appellant).

N P Metcalfe Esq Barrister.


768
[1963] 2 All ER 769

Birt v Swansea Justices


LEISURE AND LICENSING

QUEENS BENCH DIVISION


LORD PARKER CJ, HAVERS AND EDMUND DAVIES JJ
16 MAY 1963

Licensing Occasional licence Applicant the nominee of a club licence Justices on-licence granted to applicant as nominee
of proprietory club Whether same applicant entitled to apply for occasional licence in respect of premises separate from club
Licensing Act, 1961 (9 & 10 Eliz 2 c 61), s 35(2) (a).

A justices on-licence was granted under s 35 of the Licensing Act, 1961 a, to the appellant, a joint proprietor and manager of a
club, as the nominee of the club, subject to the provisions of s 35(2)(b), that no intoxicating liquor should be sold to any person
otherwise than to a member of the club. Thereafter, the appellant applied in his own name for the appropriate consent which he
required in order to obtain an occasional licence for a hall, which was in the same premises as the club and also owned by him,
but which was run quite separately from the club. On appeal against the refusal of the justices to grant him the appropriate
consent,
________________________________________
a Section 35, so far as material, is set out at p 770, letters b to f, post.

Held The consent leading to the grant of the occasional licence had been rightly withheld because a person to whom a licence
was issued as a nominee of a club had not, under s 35(2)(a) of the Act of 1961, any right to apply in his own name for an
occasional licence (see p 771, letter i, and p 772, letter a, post).
Appeal dismissed.

Notes
As to the licensing of club premises, see 5 Halsburys Laws (3rd Edn) 276, 277, para 651, and Supplement, para 651C.
For the Licensing Act, 1961, s 35, see 41 Halsburys Statutes (2nd Edn) 615.

Case Stated
This was a Case Stated by justices for the county borough of Swansea in respect of their adjudication as a magistrates court
sitting at Swansea on 15 November 1962. The appellant, Arthur Ernest Birt, applied to the justices under s 148 of the Licensing
Act, 1953, for the appropriate consent to enable him to obtain an excise licence under s 151 of the Customs and Excise Act,
1952. The justices refused the application and the appellant now appealed. The facts are set out in the judgment of Lord Parker
CJ at p 770, letters f, et seq, post.
the cases noted belowb were cited during the argument.
________________________________________
b Patersons Licensing Acts (71st Edn), pp 1175 (notes to s 35), 1206 (Sch 8, para 3).

A G W Coulthard for the appellant.


The respondents did not appear and were not represented.

16 May 1963. The following judgments were delivered.

LORD PARKER CJ. This is an appeal by way of Case Stated from a decision of justices for the county borough of Swansea
who refused an application made by the appellant for an occasional licence. I should say at once that the court has been invited
to, and has departed considerably from, the Case, because objection has been taken to certain words in the Case Stated, and
arguments therein set out, which the appellant claims are wrong.
Before stating the facts as the court understands them now, it is, I think, necessary to refer to the relevant legislation.
Section 151(1) of the Customs and Excise Act, 1952, provides that:

Where the appropriate consent has been obtained by the holder of a retailers 769 on-licence, the Commissioners shall
grant to him an excise licence (hereinafter in this section referred to as an occasional licence) authorising the sale by him
of any intoxicating liquor to which his retailers licence extends at such place other than the premises in respect of which
his retailers licence was granted, during such period not exceeding in Great Britain three weeks or in Northern Ireland
three days at one time, and between such hours, as may be specified in the consent

By s 148(1) of the Licensing Act, 1953, it is provided that, for the purposes of s 151 of the Customs and Excise Act, 1952,
appropriate consent means consent granted by justices of the peace under s 148 of the Act of 1953, and provision is then made
for the manner of application. Finally, by s 35 of the Licensing Act, 1961, provision is made for the licensing of club premises.
By sub-s (1):

The authority to sell any intoxicating liquor which is conferred by an excise licence taken out for club premises by the
club shall include authority for the club to supply that liquor to or to the order of members, notwithstanding that in law the
supply is not and does not involve a sale; and a justices licence for club premises may be granted to the club and shall have
effect accordingly, and (notwithstanding that the club is registered in respect of other club premises) sub-s. (3) of s. 25 of
this Act shall not apply to the supply of intoxicating liquor under the authority of the licence.

Then, by sub-s (2), which is for this purpose the most material provision, it is provided that:

Any excise licence for the sale of intoxicating liquor in club premises which is to be taken out by the club, and any
justices licence for club premises which is to be granted to a club, shall be taken out or granted in the name of an officer of
the club nominated for the purpose by or on behalf of the club; and in relation to any premises for which a licence is so
taken out or granted(a) the rights and obligations of the holder of the licence under the enactments relating to the sale of
intoxicating liquor and to licensed premises shall attach to the person in whose name the licence is, and those enactments
shall apply as if he were, as holder of the licence, in occupation of the premises

The facts here are that, on 17 April 1962, the appellant applied for a licence in respect of premises known as Langland
Court, and in his application he recites that he is a club proprietor, joint proprietor and manager of a club carrying on business at
Langland Court, and that he is nominated for that purpose by or on behalf of the club. He goes on to ask for the grant to the club
of a justices on-licence authorising the club to apply for and hold an excise licence to sell by retail any intoxicating liquor for
consumption on the premises of the club. Pursuant to that application he was, on 17 May 1962, granted at a special licensing
session what is called throughout a club licence. It grants

unto Arthur Ernest Birt of the club premises hereinafter mentioned, this justices club licence, authorising such person
to hold an excise licence to sell by retail at the club premises situated at Langland Court, Langland.

In a Schedule of conditions, these words appear:

Subject to the provisions of s. 35(2)(b) of the Licensing Act, 1961, no intoxicating liquor shall be sold to any person
otherwise than to a member of the said club.

To get one point out of the way, I should say that it is beyond doubt, and, indeed, is admitted, that this licence was not a licence
granted to the appellant as an individual, but was a club licence granted to the club, and merely in the name of the appellant as the
nominee of the club. What happened was that thereafter, on 15 November 1952, the appellant applied for the appropriate consent
which he required in order to obtain an occasional licence. Part of the Langland Court premises consists also of a hall called
Tudor Hall. The whole premises are owned by the appellant. Part, as I have said, is run as a club and the other part consists of
this Tudor Hall. The latter is run quite separately from the club, it has a music and dancing licence and in the past occasional
licences in respect of it have been 770 granted, but granted to other licensees with on-licences. The appellant thought that it
would be convenient, rather than to employ somebody else to supply liquor in the Tudor Hall, if he got an occasional licence to
do so. He claimed that, being an holder of an on-licence, or to put it another way, being a person in whose name an on-licence
had been granted, he was entitled to the appropriate consent for the purposes of obtaining an occasional licence. The justices
refused, and this court has been referred to a shorthand note of extracts of what occurred. One of the complaints of the appellant
is that the justices may have appeared then to be considering the wrong matters, or to be giving reasons which were not strictly
accurate, and there is a complaint that, when the Case is Stated, different reasons are given.
For my part, I do not propose to go into that matter. I am really concerned to see whether the justices were right or wrong
whatever the reasons may be in refusing this licence. Section 35 of the Act of 1961, to which I have referred, is quite clearly
intended to deal in the main with members clubs and, as one would expect, legislation is provided for someone nominated by the
members club to be responsible and who will have the rights and obligations of the holder of the licencesomeone, in other
words, to whom sanctions will attach. Counsel for the appellant is really driven to say, and he does not shrink from saying, that
the nominee of a members club in whose name the licence is issued is himself entitled by virtue of the wording of sub-s (2)(a) to
go into the market in competition with others who hold retailers on-licences and obtain an occasional licence, and not only
obtain an occasional licence but use that occasional licence to make a profit for himself. For my part, that only has to be stated to
make one realise that there must be something wrong, and, in my judgment, the reason why it is wrong is that sub-s (2)(a), in
giving to the nominee certain rights and obligations, only does so in respect of the licensed premises. In my judgment, the
provisions of sub-s (2)(a) do not attract rights which a licensee may have in respect of other premises, in other words, to obtain an
occasional licence for other premises. Subsection (2)(a) is dealing only with rights and obligations in relation to the premises to
which the licence was granted.
This, of course, is not a members club, it is a proprietory club, and the appellant is the proprietor; but there is nothing to
prevent him from applying as the nominee of his own club under s 35. No doubt many proprietors have already done so; not only
does it cost them less under s 35, but it obviates the necessity which would otherwise arise of getting licensing planning
permission. Granted, however, that the proprietor is entitled to apply for a licence under s 35, he must of course take it within the
provisions of s 35, and, if s 35 does not give him power to get an occasional licence, then he must face that consequence. If he
requires an occasional licence, he must apply, not as nominee of a club under s 35, but in his own name for an ordinary retailers
on-licence. I should add that counsel for the appellant has referred us to s 1 and s 4 of the Licensing Act, 1961. Under s 1,
provision is made for new forms of licences in respect of restaurants, and what are called residential licences and residential and
restaurant licences. He points out that, by s 4(5), the right of the licensee of such licences to obtain an occasional licence is
specifically dealt with, and dealt with in a manner restricting the form of occasional licence. He says that, if there was intended
to be any departure from the ordinary principle that the holder of an on-licence is entitled with the appropriate consent to an
occasional licence, that would have been specifically dealt with. I have fully considered that point, but it seems to me that, on the
plain wording of s 35(2)(a), there is no right in a man to whom a licence is issued as a nominee of a club to apply in his own
name for an occasional licence. In my judgment, the justices came to the right conclusion, and I would dismiss this appeal.

HAVERS J. I agree. I agree with the interpretation which my Lord has put on s 35 of the Licensing Act, 1961. The important
words which, in my view, 771qualify sub-s (2)(a) are: in relation to any premises for which a licence is so taken out or
granted. I agree, with the conclusion at which my Lord has arrived and the reasons which he has given. I agree, therefore, that
this appeal should be dismissed.

EDMUND DAVIES J. I agree, and in so doing would like to express my indebtedness to learned counsel for the clarity with
which he has brought to the courts attention the various quite difficult and intricate statutory provisions.

Appeal dismissed.
Solicitors: Theodore Goddard & Co agents for T B Walker-Jones & Gardner, Swansea (for the appellant).

N P Metcalfe Esq Barrister.


[1963] 2 All ER 772

Theodoropoulas v Theodoropoulas
FAMILY; Divorce

WINCHESTER ASSIZES
SIR JOCELYN SIMON P
11, 12 MARCH 1963

Divorce Evidence Privilege Conversation between parties with view to reconciliation Conversation privileged Evidence
of third party overhearing conversation inadmissible.

The privilege against disclosure accorded in matrimonial proceedings to communications made by spouses to, eg, probation
officers, whose services have been enlisted with a view to reconciliation, extends to communications between spouses themselves
with a view to their reconciliation, and extends also to the evidence of an independent witness who was present when those
communications were made and who overheard or read them (see p 774, letter g, post).
At the hearing of a petition by the wife and a cross-petition by the husband for divorce, on the ground in each case of
cruelty, which was denied, the husband sought to call as a witness a Mrs G, in order that she should testify to an alleged
conversation which she had had with the wife at the husbands request with a view to the reconciliation of the parties, and to
cross-examine the wife on the conversation. The husband also sought to give evidence himself of, and to cross-examine the wife
on, an alleged conversation between the husband and the wife, who had approached him without the knowledge of her solicitor
with a view to reconciliation; and the husband sought further to call as a witness a Mr B whom he alleged had been present
during part of this conversation.

Held Both the conversations were privileged from disclosure, and accordingly the evidence sought to be adduced was
inadmissible.

Notes
As to privilege for communications with view to reconciliation, see 12 Halsburys Laws (3rd Edn) 375, para 818; and for cases on
the subject, see 27 Digest (Repl) 368, 30443046.

Cases referred to in judgment


Broome v Broome (Edmundson cited) [1955] 1 All ER 201, [1955] P 190, [1955] 2 WLR 401, 3rd Digest Supp.
Henley v Henley (Bligh cited) [1955] 1 All ER 590, n, [1955] P 202, 119 JP 215 n, 3rd Digest Supp.
McTaggart v McTaggart [1948] 2 All ER 754, [1949] P 94, [1949] LJR 82, 27 Digest (Repl) 368, 3044.
Mole v Mole [1950] 2 All ER 328, [1951] P 21, 27 Digest (Repl) 368, 3046.

Petition
In these proceedings the wife petitioned and the husband cross-petitioned for a decree nisi of divorce on the ground of cruelty,
which was denied by each party.

R Hughes for the wife.


A C Munro Kerr for the husband.
772

12 March 1963. The following judgment was delivered.

SIR JOCELYN SIMON P stated that the case had caused him anxiety because he found himself unable to accept the
unsupported evidence of either the husband or the wife and had to rely on probabilities supported by some evidence of
independent witnesses, and continued: Before I review the evidence which has led me to my conclusion, I must refer to other
evidence which was tendered and which I ruled inadmissible, reserving my reasons. It relates to two alleged conversations. The
first took place when a Mrs Gray had a discussion with the wife after she had left the husband; this approach was made at the
request of the husband with a view to reconciliation. Counsel for the husband sought to cross-examine the wife as to what was
said between her and Mrs Gray and to call Mrs Gray as a witness. The second conversation was said to have taken place in the
following circumstances. It was sought on behalf of the husband to allege that after the petition was filed the wife called on the
husband, emphasised that that visit was without the knowledge of her solicitor and that he should be kept in ignorance of it, and
then discussed financial terms on which she might consider returning home. Counsel wished to cross-examine the wife as to the
approach which was said to have taken place, to ask the husband questions about it, and to call a Mr Brewer, who was said to
have been present during part of the conversation. Counsel outlined at my request the nature of the evidence and also its purpose,
namely, that the wife in making overtures herself for a reconciliation, showed that she was not terrified of her husband, as she had
suggested in evidence that she was, and that her motives both in leaving and in offering terms on which she might come back
were of a purely mercenary nature.
Counsel for the husband accepted that statements made to a probation officer whose services have been enlisted with a view
to reconciliation are privileged, if one of the parties objects to their disclosure (McTaggart v McTaggart; Mole v Mole). He
argued, however, that the rule does not extend to a private individual whose services are enlisted towards the parties
reconciliation; certainly not to conversations between the parties themselves; and least of all to a witness fortuitously present and
overhearing part of any such communication. I think that it is, therefore, important to ascertain the object of the privilege which
has been conceded in the authorities. In McTaggart v McTaggart, Denning LJ said ([1948] 2 All ER at p 756; [1949] P at p 97):

The law favours reconciliation, and the court will not take on itself a course which would be so prejudicial to its
success.

That is to say, will not take on itself to insist on statements made in the course of an attempt at reconciliation being disclosed.
If a probation officer should be compelled to give evidence as to what was said in the course of negotiations, it would
mean that, when he attempted reconciliation, he would not be told the truth, or, at all events, not the whole truth. The
parties would have at the back of their minds the thought that whatever they said might be given in evidence against them
or for them, and would colour their statement accordingly.

In Mole v Mole, Bucknill LJ said ([1950] 2 All ER at p 329; [1951] P at p 23):

one must bear in mind that, in matrimonial disputes, the state is also an interested party and is more interested in
reconciliation than in divorce, and if the rule as to privilege tends to promote the prospects of reconciliation, I think it ought
to be applied.

It seems to me that the reasoning in each case applies with equal force to a private individuals attempts at reconciliation as to
those of an official person.
In the second place, privilege has been conceded to a wide range of persons. To quote again from Mole v Mole, Denning LJ
said ([1950] 2 All ER at p 329; [1951] P at p 24):
773
I take it that the principle of McTaggart v. McTaggart applies not only to probation officers, but also to other persons
such as clergy, doctors, or marriage guidance counsellors, to whom the parties resort with a view to reconciliation when
there is a tacit understanding that the conversations are without prejudice.

Thus, communications were held to be privileged to a local officer of the Soldiers, Sailors, and Airmens Families
Association in Broome v Broome (Edmundson cited), and to the vicar of the parties parish in Henley v Henley (Bligh cited).
In the third place, although there are persons, such as those whom I have mentioned, who will have a certain inherent
standing as conciliators, the privilege attaches to them because they are attempting reconciliation in the case, and not because of
their standing. No doubt, when a probation officer or a SSAF. A representative or a clergyman is approached, the law will
readily infer that the parties have gone to him with a view to reconciliation and on the tacit understanding that nothing said
should afterwards be used against them; but, equally, where it is proved that any private individual is enlisted specifically as a
conciliator, in my view the law will aid his or her efforts by guaranteeing that any admissions or disclosures by the parties are
privileged in subsequent matrimonial litigation. I was, therefore, quite clear that evidence of the conversation between the wife
and Mrs Gray was inadmissible.
Moreover, it seems to me that the principle on which the cases I have referred to were decided is just as applicable to
communications between the parties themselves with a view to reconciliation as to those conducted through intermediaries. In
the case of without prejudice communications with a view to settlement of actual or pending litigation, the privilege equally
attaches whether the communication is directly between the parties, or through an intermediary such as a solicitor. In McTaggart
v McTaggart, Cohen LJ clearly regarded ([1948] 2 All ER at p 755; [1949] P at p 97) such cases of without prejudice
negotiations as analogous to those in question. Referring to one of such cases, he said ([1948] 2 All ER at p 755; [1949] P at p
97):

In that case the court had to do with solicitors correspondence and the litigation was actually pending, but it seems to
me that the principle applies to negotiations to avoid threatened litigation, and a fortiori to negotiations between parties
themselves.

It follows, in my judgment, that privilege attaches to communications between the spouses themselves when made with a
view to reconciliation. It also extends to excluding the evidence of an independent witness who was fortuitously present when
those communications were made and who overheard or read them. I therefore ruled that all the evidence tendered, whether by
way of cross-examination of the wife, or in chief from the husband or by calling Mr Brewer, was inadmissible.
[His Lordship then reviewed the evidence, found that each party had been guilty of cruelty to the other party, the husband
being the more blameworthy, and in the exercise of the courts discretion granted a decree nisi of divorce to both parties.]

Decrees nisi for the husband and the wife.

Solicitors: Norris & Gillett, Bournemouth (for the wife); Preston & Redman, Bournemouth (for the husband).

Deirdre McKinney Barrister.


774
[1963] 2 All ER 775

Re Wonderland, Cleethorpes
East Coast Amusement Co Ltd v British Railways Board
LANDLORD AND TENANT; Tenancies

HOUSE OF LORDS
VISCOUNT SIMONDS, LORD REID, LORD MORRIS OF BORTH-Y-GEST, LORD HODSON AND LORD PEARCE
1, 2, 30 MAY 1963

Landlord and Tenant New tenancy Business premises Improvements Effect in determining rent Whether improvements
made before the current tenancy should be disregarded in determining rent payable on the grant of a new tenancy Landlord
and Tenant Act, 1954 (2 & 3 Eliz 2 c 56), s 34(c).

In para (c) of s 34 of the Landlord and Tenant Act, 1954, the phrase the tenant refers to the tenant in the capacity in which he
makes his application to the court for a new lease, viz, as tenant under the current tenancy which he seeks to have renewed;
accordingly the effect of improvements made during a tenancy prior to the current tenancy are not required by s 34(c) to be
disregarded in determining the rent at which the new tenancy is to be granted on the tenants application (see p 777, letters e and
f, p 778, letter e, and p 780, letter a, post; cf p 779, letter h, post).
Tenants of land erected on their site in 1926 two structures, a Wonderland and a Big Dipper. In 1938 they were granted
a new lease of the site and of all buildings thereon for twenty-one years from 25 March 1939. Before the expiration of that
tenancy (called the current tenancy) they applied to the court for a new tenancy pursuant to s 26 and s 29 of the Landlord and
Tenant Act, 1954. On the question whether the structures erected in 1926 were improvements within s 34 (c) of the Act of 1954,
with the consequence, if they were, that they would be disregarded in determining the rent of the new tenancy,

Held The structures of 1926 should not be disregarded in determining the rent, because they were erected before the beginning
of the current tenancy, not during that tenancy.
Decision of the Court Of Appeal ([1962] 2 All ER 92) affirmed.

Notes
As to the determination of rent of a new tenancy, see 23 Halsburys Laws (3rd Edn) 898, 899, para 1725; and for cases on
compensation for improvements, see 31 Digest (Repl) 628, 7329 and 3rd Digest Supp.
For the Landlord and Tenant Act, 1954, s 26, s 29, and s 34, see 34 Halsburys Statutes (2nd Edn) 411, 413, 418.

Cases referred to in judgment


Clift v Taylor [1948] 2 All ER 113, [1948] 2 KB 394, [1948] LJR 1822, 31 Digest (Repl) 632, 7410.
Lawrence v Sinclair [1949] 1 All ER 418, [1949] 2 KB 77, [1949] LJR 1003, 31 Digest (Repl) 631, 7402.
Reid v Reid (1886), 31 ChD 402, 55 LJCh 294, 54 LT 100, 27 Digest (Repl) 95, 704.

Appeal
This was an appeal from a decision of the Court of Appeal (Upjohn and Diplock, LJ; Lord Evershed MR dissenting), dated 22
February 1962, and reported in [1962] 2 All ER 92, upholding a decision of Pennycuick J dated 23 November 1961, and reported
in [1962] 1 All ER 68.
The appellant company (the tenants) was incorporated on 8 May 1912. Under several leases and tenancy agreements
granted by predecessors in title of the respondents (the landlords), the tenants acquired piecemeal an area of rather more than
three acres adjoining the promenade at Cleethorpes. In or about 1912 a structure known as the Big Dipper was erected, and in
1926 the tenants removed this to another part of the premises and re-erected it there. In the same year they erected the main hall
known as Wonderland. By a new 775 lease dated 4 November 1938, the London and North Eastern Railway Company, the
predecessor of the landlords, granted to the tenants the area that they then occupied and all the buildings, structures and erections
for the time being thereon for the term of twenty-one years from 25 March 1939 at a rent of 2,500 yearly. In 1941 the property
was requisitioned and in 1946 it was de-requisitioned. On 1 September 1959, the tenants made a request for the grant of a new
tenancy pursuant to s 24 of the Landlord and Tenant Act, 1954, the terms that they proposed being an annual rent of 2,500, a
term of fourteen years, and otherwise the same terms as were contained in the lease of 1938. By originating summons, dated 17
December 1959, the tenants applied to the High Court for the grant of a new tenancy of the premises pursuant to Part 2 of the
Landlord and Tenant Act, 1954.

L A Blundell QC and K R Bagnall for the appellants, the tenants.


Sir Andrew Clark QC and I L R Romer for the respondents, the landlords.

Their Lordships took time for consideration. May 30. The following opinions were read.

30 May 1963. The following opinions were delivered.

VISCOUNT SIMONDS. My Lords, this appeal raises a question of construction of a section of the Landlord and Tenant Act,
1954, on which there has been a difference of opinion in the Court of Appeal. The relevant facts are not in dispute. The appellant
company (the tenants) under a lease dated 4 November 1938, held certain premises at Cleethorpes in the county of Lincoln for
a term of twenty-one years which was due to expire on 25 March 1960. On 1 September 1959, invoking the provisions of s 24 of
the Landlord and Tenant Act, 1954, they requested the respondents (the landlords) to grant them a new tenancy of the premises.
One of the proposed terms was that the new annual rental should be 2,500. This rental was considerably below the then
economic rent of the premises and could not be supported unless in determining it certain improvements carried out before the
grant of the 1938 lease ought to be disregarded. The improvements were very substantial and consisted of an erection known as
the Big Dipper and the main hall known as Wonderland. They had in fact been made as long ago as 1926 butand here is
the crucial pointthey had been made by the tenants who were in occupation of the premises under earlier leases granted by the
predecessors in title of the landlords.
The landlords conceded that under the Act of 1954 the tenants were entitled to a new tenancy, but claimed that the
improvements to which I have referred should not be disregarded in fixing the rental. Accordingly the tenants commenced these
proceedings to have that question determined. It was decided against them by Pennycuick J, and by the Court of Appeal (1) and,
notwithstanding a dissenting judgment by Lord Evershed MR, in the latter court, I concur in the unanimous opinion of your
lordships that it was rightly so decided.
The issue is to be resolved on a consideration of s 34 of the Landlord and Tenant Act, 1954, which is as follows:

The rent payable under a tenancy granted by order of the court under this Part of this Act shall be such as may be
agreed between the landlord and the tenant or as, in default of such agreement, may be determined by the court to be that at
which, having regard to the terms of the tenancy (other than those relating to rent), the holding might reasonably be
expected to be let in the open market by a willing lessor, there being disregarded(a) Any effect on rent of the fact that the
tenant has or his predecessors in title have been in occupation of the holding. (b) Any goodwill attached to the holding by
reason of the carrying on thereat of the business of the tenant (whether by him or by a predecessor of his in that business),
(c) Any effect on rent of any improvement carried out by the tenant or a predecessor in title of his otherwise than in
pursuance of an obligation to his immediate landlord, (d) In the 776 case of a holding comprising licensed premises, any
addition to its value attributable to the licence, if it appears to the court that having regard to the terms of the current
tenancy and any other relevant circumstances the benefit of the licence belongs to the tenant.

The immediately relevant clause is (c), and the question is whether the Big Dipper and Wonderland were improvements
carried out by the tenant or a predecessor in title of his within the meaning of that clause. It is urged on the one hand by the
tenants that they were at the date of their application under s 24 of the Act of 1954 the tenants of the premises and that it was they
who had in fact carried out the improvements in question: therefore, they say, the language of the section is unambiguously apt
to cover their case. On the other hand, it is said by the landlords that the context imperatively demands that some qualification
must be imposed on the relevant words, at any rate if the words the tenant are to be read as indicating nomination of the person
who is making the application: it could not be supposed that in determining the new rent improvements were to be disregarded
that had been carried out by him in some other capacity than that of tenant or, if as a tenant, under some earlier tenancy with an
intervening tenancy of a third party. If so, no other qualification could be suggested than that improvements carried out by the
tenant must mean those carried out by him under the tenancy which he sought to have renewed.
In the course of the argument stress was laid on one side and the other on the anomalies that might arise from these
alternative constructions and assistance was sought to be derived from other sections of the Act of 1954. But, my lords, I do not
find it possible to get much assistance from other sections and I am no more than usual moved by an argument from anomalies
which unfailingly arise on legislation of this character. For me the critical question isWhat is meant by the tenant and I
reject the premise, on which the tenants must rely, that the phrase includes the individual tenant in any other capacity than that in
which he makes his application to the court, ie, as tenant under the current tenancy which he seeks to have renewed. Part 2 of the
Act of 1954 (within which s 34 falls) begins and ends with the tenancy then current which is to be extended or renewed. When
that tenancy was created, a new relation was established between landlord and tenant. It is to be assumed that, when it was
established and the old chapter, if there was one, was closed, the parties took into account what were then their respective rights
and liabilities and founded on them accordingly. A new chapter then began and it is only with what thereafter happened that the
court is concerned. This is perhaps only another way of saying that in the context the words carried out by the tenant import
carried out during the current tenancy, but I arrive at that conclusion on a consideration of the meaning which must be given to
the tenant.
Learned counsel for the landlords urged that the same result could be reached by having regard to the meaning of the word
improvement, which he said could only refer to something done after the start of the current tenancy. It may well be that in the
context of s 34 (c) this is a valid argument, but I prefer not to rest on it, for it is easy to see what difficulties might arise if it was
sought to adopt such a meaning generally in an interpretation of the Act.
This view is in some measure supported by the following words, or his predecessors in title, on which Upjohn LJ ([1962]
2 All ER at p 96; [1962] Ch at p 719), largely relied. For if the tenant includes the current tenant in any capacity in which he
may at an earlier date have carried out improvements, as freeholder, squatter, licensee, or tenant, an extension to his predecessors
in title would clearly be a quite unreasonable impairment of the landlords rights. This argument would have less force if (as
Lord Evershed MR was disposed to think) the relevant words were apt to limit the improvements done by the tenants or their
predecessors in title under some tenancy. But I 777 have been unable to find any ground for thus limiting the scope of the words,
if they are to be limited at all.
Both Pennycuick J, and the majority of the Court of Appeal were assisted to their conclusion by a consideration that the
contention of the tenants would lead to what they called a quasi-confiscation of rights vested in the landlord long before the
passing of the Act by, in effect, retrospective legislation. I, too, my lords, should have been influenced by this consideration if I
had felt any real doubt about the true construction of the section. Undoubtedly the Act of 1954 in important respects derogates
from the common law rights of the landlord: he is no longer master of the situation to grant or deny a new lease to his tenant,
but, if there is any ambiguity about the extent of that derogation, the principle is clear that it is to be resolved in favour of
maintaining common law rights unless they are clearly taken away: see Clift v Taylora. Nor can the retrospective operation of
the Act of 1954 be ignored. Again it is true that the Act of 1954 operates in the sense that I have mentioned to deprive the
landlord of his vested right to dispose as he will of the reversion on the current tenancy. But it is carrying it further to deprive
him of the benefit of the improvements on his land already vested in him on the grant of that tenancy. In this connexion I would
refer to the observations of Bowen LJ on Reid v Reid ((1886), 31 ChD 402 at pp 408, 409).
________________________________________
a Per Scott LJ [1948] 2 All ER 113 at p 116; [1948] 2 KB 394 at p 400.

I must mention, lest it be thought that I have forgotten it, the case of Lawrence v Sinclair on which the tenants relied. But,
with great respect to Lord Evershed MR who also found support for his view in that decision, I do not think it necessary to say
more than that it was a decision on a different section of a different Act (the Landlord and Tenant Act, 1927) in relation to a
different subject-matter and that, even if it was rightly decided, on which I do not feel called on to pronounce, I cannot regard it
as having any, much less any decisive, bearing on the construction of s 34 (c) of the Act of 1954.
In my opinion this appeal must be dismissed with costs.

LORD REID. My Lords, I concur.

LORD MORRIS OF BORTH-Y-GEST. My Lords, the appellant company (the tenants) erected the structure known as
Wonderland in the year 1926. In the same year they erected the Big Dipper on its present site. (It had been originally erected
in 1912 on a different part of the land.) The leases or tenancies which were held by the tenants in 1926 subsequently expired.
The predecessor of the respondents (the landlords) then granted a new lease to the tenants. That lease, which was dated 4
November 1938, was for a term of twenty-one years from 25 March 1939, at a rent of 2,500 a year. It was a lease of the area
then occupied by the tenants and all the buildings, structures and erections for the time being erected thereon. The term of
twenty-one years was due to expire on 25 March 1960. It is not in controversy that the tenants who duly (on 1 September 1959)
made a request for a new tenancy in accordance with s 26 of the Landlord and Tenant Act, 1954, and applied to the court for a
new tenancy, are entitled to a new tenancy. On their application to the court the provisions of s 29 became applicable which
require that, subject to the provisions of the Act of 1954, the court shall make an order for the grant of a tenancy comprising
such property at such rent and on such other terms as are provided in the Act of 1954. The landlords served no counter-notice
and did not oppose the tenants application (see s 26(6) and s 30) and accordingly it became the duty of the court in granting a
new lease to determine (in default of agreement) (a) the property to be comprised in the new tenancy (see s 32), (b) the duration
of the new tenancy (see s 33), (c) the rent under the new tenancy (see s 34), and (d) the other terms of the new tenancy.
778
No question arises in the present case as to the property to be comprised in the new tenancy and that part of s 32 is
applicable which provides that an order for the grant of a new tenancy is to be

an order for the grant of a new tenancy of the holding; and in the absence of agreement between the landlord and the
tenant as to the property which constitutes the holding the court shall in the order designate that property by reference to
the circumstances existing at the date of the order.

The holding is defined (see s 23(3)) to mean the property comprised in the tenancy (subject to a qualification which is not for
the present purposes material).
The tenancy in the present case is the tenancy for a term of twenty-one years from 25 March 1939. That was the current
tenancy (see s 26(1)) when the tenants made their request for a new tenancy and when they applied to the court for a new
tenancy. By virtue of the provisions of the Act of 1954 (see ss 24, 26 and 64) that tenancy has not come to an end but is
continuing. The duration of the new tenancy has yet to be determined and the limited issue now arising concerns the
determination of the rent of the new tenancy. The issue was raised on an application by summons. The summons was adjourned
to be heard in court. The question raised was:

Whether on the true construction of s. 34 of the Landlord and Tenant Act, 1954, any and if so which of the buildings
and structures or part or parts thereof now upon the land comprised in a lease dated the 4th day of November, 1938, and
made between [the landlords] and [the tenants] are improvements for the purposes of sub-s. (c) of the said section.

The parties reached agreement in regard to certain improvements, and your lordships have not been concerned with them, but in
regard to the works of erection or re-erection, which took place in 1926 and to which I have referred, the decision of the court
resulted in a declaration that no works carried out prior to the commencement of the current tenancy can be an improvement for
the purposes of s 34 (c) of the Landlord and Tenant Act, 1954. On appeal to the Court of Appeal the appellants asked for a
declaration that all the improvements carried out by them as tenants were improvements for the purposes of sub-s (c). The appeal
failed and the order made by Pennycuick J, was affirmed.
My lords, the limited issue now arising is denoted by the terms of the declaration to which I have referred. Does s 34(c)
refer to works carried out by the tenant or a predecessor in title of his during the currency of the current lease or does it refer to
works carried out by the tenant or a predecessor in title of his during the currency of either the current or any prior lease?
My lords, the tenancy that is now continuing is the current tenancy which was created by the lease on 4 November 1938, and
which began on 25 March 1939. The holding (see s 23) is the property comprised in that tenancy. That must be the property as
it existed at the beginning of the current tenancy together with any physical improvements carried out during the tenancy. The
tenant who is referred to in s 34(c) is the tenant of that property. If what is to be disregarded is the effect from a rent point of
view of any improvements carried out by the tenant that, I consider, denotes any improvements carried out by him in respect of
the property comprised in the current tenancy. That involves a consideration whether between the start and the end of the period
of the current tenancy improvements have been carried out by him. It seems to me therefore that the reference in s 34(c) is a
reference only to works carried out during the currency of the current lease.
I agree that the appeal fails.
779

LORD HODSON. My Lords, I agree that this appeal should be dismissed for the reasons given by my noble and learned friend
Viscount Simonds.

LORD PEARCE. My Lords, I have had the advantage of reading the opinion of my noble and learned friend Viscount Simonds:
I entirely agree with it and have nothing further to add.

Appeal dismissed.

Solicitors: Godfey Warr & Co agents for John Barkers, Great Grimsby (for the appellants); M H B Gilmour (for the
respondents).

C G Leonard Esq Barrister.


[1963] 2 All ER 780

R v Yule
CRIMINAL; Criminal Law: PROFESSIONS; Lawyers

COURT OF CRIMINAL APPEAL


LORD PARKER CJ, HAVERS AND WIDGERY JJ
29, 30 MAY 1963

Criminal Law Fraudulent conversion Solicitor Moneys received by solicitor from clients in respect of counsels and
shorthand-writers fees Fees not paid, but used for solicitors own use and benefit Whether an offence under Larceny Act,
1916 (6 & 7 Geo 5 c 50), s 20(1) (iv) (b).

Criminal Law Indictment Defect Particulars of offence Not amended at trial No embarrassment caused to accused
Applicability of proviso to Criminal Appeal Act, 1907 (7 Edw 7 c 23), s 4(1).

Solicitor Fees Fees received by solicitor for counsel and shorthand-writers misappropriated by him for his own benefit and
use Gross dishonesty admitted but intention ultimately to pay asserted Whether an offence under Larceny Act, 1916 (6 & 7
Geo 5 c 50), s 20(1) (iv) (b) Solicitors Act, 1957 (5 & 6 Eliz 2 c 27), s 66 Solicitors Accounts Rules, 1945 (SR & O 1944 No
781), r 9(2).

The appellant, a former solicitor, received from a client a cheque for 1,663 9s 7d in payment of his bill. The bill included 639
17s as disbursements for counsels fees. These fees were not paid to counsel. The appellant paid the cheque into his own bank
account. He was charged under s 20(1)(iv)(b) of the Larceny Act, 1916 a, with fraudulent conversion, the particulars alleging that
the appellant, having received the cheque on account of the client, fraudulently converted part of the proceeds thereof to his own
use and benefit. Admissions by the appellant in evidence at the trial showed, so the court decided, that the 639 17s was
fraudulently converted by the appellant to his own use. On appeal against conviction the appellant contended (i) that, in effect, r
9(2) of the Solicitors Accounts Rules, 1945b, required that he should pay the cheque into his own bank account, and that
thereupon the money became his own, and (ii) that the charge was wrongly laid under s 20(1)(iv)(b), as distinct from s 20(1)(iv)
(a)a, for the particulars alleged receipt of the cheque on behalf of the client, whereas the client had drawn the cheque.
________________________________________
a Section 20(1), so far as material, is set out at p 783, letters e and f, post.
b Rule 9(2), so far as material, is set out at p 783, letter h, post.

Held (i) the fact that a particular sum was paid into a particular banking account by a solicitor, albeit pursuant to statutory
obligation, did not affect the rights of persons interested in the sum or any duty of the solicitor either towards his client or
towards third persons in regard to the disposal of the sum; the relevant provisions of the Solicitors Act, 1957, and of the
Solicitors Accounts Rules, 1945, were domestic matters, and it remained a question of fact in each case whether money was so
received as to bring the case within s 20(1)(iv)(b) of the Larceny Act, 1916 (see p 784, letters a and b, post); in the present case
the 639 17s was shown to have been fraudulently converted by the appellant to his own use (see p 784, letter g, post).
780
(ii) although the proceeds of the cheque were received by the appellant as to part (639 17s) for or on account of another
person within the wording of s 20(1)(iv)(b), so that a charge under that enactment lay, yet the cheque was not received on account
of the client as alleged in the particulars; therefore, although the statement of offence under s 20(1)(iv)(b) was unobjectionable,
the particulars were defective and should have been amended at the trial, but as the appellant had not been embarrassed in his
defence by the particulars, the court would apply the proviso to s 4(1) c of the Criminal Appeal Act, 1907, and would dismiss the
appeal (see p 785, letters a, b and e, and p 786, letters f and i, post).
________________________________________
c Section 4(1), so far as material, provides: provided that the court may, notwithstanding that they are of opinion that the point raised in the
appeal might be decided in favour of the appellant, dismiss the appeal if they consider that no substantial miscarriage of justice has actually
occurred.

R v Bottomley ((1922), 16 Cr App Rep 184) explained.


Dictum of Isaacs CJ in R v Thompson ([1914] 2 KB at p 105), and R v McVitie ([1960] 2 All ER 498) applied.
Per Curiam: sub-paragraph (a) of s 20(1)(iv) of the Larceny Act, 1916, is concerned with cases where money is entrusted by
a transferor to another and an obligation is imposed by the transferor to pass on the money to a third person; sub-paragraph (b) on
the other hand, is appropriate for cases where money is received by a person with an obligation to hold it on account of another,
whether or not that obligation is imposed by the initial transferor who passed the money to him (see p 785, letters d and e, post).
Appeal dismissed.

Notes
The case also related to shorthand-writers fees, the position in regard to which was not different for the purposes of the case (see
p 782, letter f, post), but, as the judgment is substantially confined to an instance in regard to counsels fees, the headnote has
been similarly limited.
As to fraudulent misappropriation of property, see 10 Halsburys Laws (3rd Edn) 792, 793, para 1534; as to criminal liability
of a solicitor, see 36 Halsburys Laws (3rd Edn) 58, para 82; and for cases on the subject, see 15 Digest (Repl) 1110, 11,034
11,036.
For the Larceny Act, 1916, s 20, see 5 Halsburys Statutes (2nd Edn) 1022.
For the Solicitors Act, 1957, s 66, see 37 Halsburys Statutes (2nd Edn) 1108.
For the Solicitors Accounts Rules, 1945, r 9, see 20 Halsburys Statutory Instruments 195.

Cases referred to in judgment


R v Bottomley (1922), 127 LT 847, 87 JP 26, 16 Cr App Rep 184, 14 Digest (Repl) 662, 6712.
R v McVitie [1960] 2 All ER 498, [1960] 2 QB 483, 124 JP 404, [1960] 3 WLR 99, 44 Cr App Rep 201, 3rd Digest Supp.
R v Thompson [1914] 2 KB 99, 83 LJKB 643, 110 LT 272, 78 JP 212, 9 Cr App Rep 252, 14 Digest (Repl) 251, 2168.

Appeal and application


This was an appeal by Thomas Christopher Yule on a certificate under s 3 (b) of the Criminal Appeal Act, 1907, against his
conviction at the Central Criminal Court on 1 February 1963, before the Recorder of London sitting with a jury, on nine counts of
fraudulent conversion. He also applied for leave to appeal against his conviction on a further count of fraudulent conversion.
The facts are set out in the judgment of the court.
On 30 January and 1 February 1963, submissions were made on behalf of the appellant, at his trial before the Recorder of
London, in the absence of the jury. The initial submission by counsel on behalf of the appellant was that in no instance was
money ever received for or on behalf of the party named in the indictment 781(viz, the client who drew the cheque in question).
In two or more instances moneys claimed on a bill for shorthand-writers fees had been paid and formed part of the moneys
referred to in the count of the indictment. Counsel for the appellant sought to satisfy the recorder that shorthand-writers fees
were to be regarded, for the purposes of the indictment, in the same light as counsels fees. He referred to Cocks v Bruce, Searl
and Good ((1904), 21 TLR 62; 42 Digest 364, 4158), as showing (as the recorder accepted) that a solicitor, although in the
position of agent for his client in this connexion, accepted in law personal liability to pay a shorthand-writer employed. There
was, however, no legal liability on a solicitor for payment of counsels fees. The cheques that the appellant received were all paid
into his own account, an office account subject to his sole control. Counsel for the Crown referred to Sadd v Griffin ([1908] 2
KB 510; 77 LJKB 775; 99 LT 502; 42 Digest 156, 1558), as showing that counsels fees entered as disbursements in a bill of
costs were not recoverable if they had not been paid so that, as the disbursements for counsels fees entered in the bills in this
case were not therein stated not yet to have been paid, the money which they represented was not money that the solicitor would
have been entitled to recover, and thus should be regarded as money of the client, the drawer of the cheque. Counsel referred to s
66d of the Solicitors Act, 1957. He submitted that where a cheque was paid by a client in response to a bill including counsels
fees, it must have been paid on the understanding both of the client and of the solicitor that the proceeds were to be applied partly
for counsels fees. The recorder, in the course of his summing up directed the jury as follows:
________________________________________
d Section 66, so far as material, provides: For the purposes of the remuneration of a solicitor, his bill of costs may include costs payable in
discharge of a liability properly incurred by the solicitor on behalf of the party to be charged with the bill (including counsels fees)
notwithstanding that those costs have not been paid before the delivery to that party of that bill

If you are sure in each case that the cheque delivered, or caused to be delivered to [the appellant] by the client
concerned, was delivered on the mutual understanding, shared between them, that the proceeds, or the appropriate part of
the proceeds of each cheque, would be applied to the payment of counsels fees, then until that payment was made to
counsel by [the appellant] the proceeds of these cheques, so far as appropriate to counsels fees, was held by [the appellant]
on account of the clients concerned; and [if you are sure] that instead of applying the proceeds to that purpose of paying
counsels fees and shorthand-writers [the appellant] used those proceeds for his own purposes, he did in law convert that
money to his own use and benefit

The foregoing summary in relation to proceedings at the trial shows the treatment of money paid for shorthand-writers fees to
have been on the same footing, in the direction in the summing-up, made after argument, as counsels fees.
The authorities and cases noted below e were cited during the argument on the appeal in addition to the cases referred to in
the judgment.
________________________________________
e Stephens History of Criminal Law (1883), Vol 3, p 159 Russell on Crime (11th Edn), Vol 2, pp 1266 et seq, Archbolds Criminal Pleading,
Evidence and Practice (35th Edn), para 1901, R v Jones (1948), 33 Cr App Rep 11, R v Dent [1955] 2 All ER 806, [1955] 2 QB 590, R v
Pilkington (1958), 42 Cr App Rep 233.

Victor Durand QC and D W T Price for the appellant.


J M G Griffith-Jones and J C Mathew for the Crown.

Cur adv vult


30 May 1963. The following judgments were delivered.

WIDGERY J read the following judgment of the court: The appellant, Thomas Christopher Yule, a former solicitor, was
convicted at the Central Criminal Court on ten charges of fraudulent conversion of property, and 782 he was sentenced by the
recorder of London to two years imprisonment concurrent on counts 1 to 3 and 5 to 10, and to twelve months imprisonment
consecutive on count 4, making a term of three years in all. He now appeals against his conviction on nine of those counts,
namely, 1 to 3 and 5 to 10 inclusive, on a certificate of the trial judge, these nine counts all being counts relating to moneys
received by the appellant from clients in respect of fees payable to counsel and to shorthand-writers. The appellant also seeks
leave to appeal on the remaining count, that is count 4, against his conviction, but this application is on grounds which do not
arise unless he succeeds in his appeal on the first-mentioned nine counts.
I can take the circumstances of count 1 as being typical of the nine counts for this purpose. The indictment on count 1
showed as the statement of offence Fraudulent conversion of property, contrary to s 20(1)(iv)(b) of the Larceny Act, 1916, and
the particulars of the offence are shown as

Thomas Christopher Yule on or about July 5, 1954, within the jurisdiction of the Central Criminal Court, having
received a cheque for 1,663 9s. 7d. for or on account of Thomas Hedley & Co., Ltd., fraudulently converted part proceeds
thereof, namely, the sum of 639 17s. to his own use and benefit.

Exhibit 1 of the exhibits shows the bill which was rendered by the appellant to Messrs Thomas Hedley & Co Ltd who were his
clients in respect of the sum referred to in count 1. The bill shows a total indebtedness from the client of 1,663 9s 7d, and in the
disbursements appear certain items due for counsels fees which total 639 17s. In substance, the charge against the appellant on
this count was that he had fraudulently converted that part of the money paid to him by the client which was attributable to
counsels fees. Section 20(1) of the Larceny Act, 1916, provides that:

Every person who (iv)(a) being entrusted either solely or jointly with any other person with any property in
order that he may retain in safe custody or apply, pay, or deliver, for any purpose or to any person, the property or any part
thereof or any proceeds thereof; or (b) having either solely or jointly with any other person received any property for or on
account of any other person; fraudulently converts to his own use or benefit, or the use or benefit of any other person, the
property or any part thereof or any proceeds thereof; shall be guilty of a misdemeanour

The charge in this case, as I have already indicated, is laid under sub-para (b) of that subsection. Counsel appearing on
behalf of the appellant argues as a matter of law that the receipt by the appellant of the cheque referred to in count 1 cannot
amount to a receiving on account of any other person within s 20(1)(iv)(b) so as to make its subsequent appropriation an offence
under that subsection. He has referred us to s 66 of the Solicitors Act, 1957, and also to the Solicitors Accounts Rules, 1945,
having particular regard to r 9 of those rules. Rule 9(2) provides that:

Notwithstanding the provisions of these rules, a solicitor shall not pay into a client account clients money held or
received by him (c) which is paid to him expressly on account of costs incurred, in respect of which a bill of costs or
other written intimation of the amount of the costs has been delivered, or as an agreed fee, or on account of an agreed fee,
for business undertaken or to be undertaken.
Counsel for the appellant contends that, by virtue of those rules, the cheque received by the appellant which is the subject of
count 1 was required to be paid into the appellants own account and not into the client account. The argument is that the effect
of these rules, which have the force of law, is to require the cheque to be paid into that account, and that thereupon the money
represented by the cheque either is, or becomes, the appellants own money which he can dispose of without being subject to any
criminal sanction under s 20 of the Larceny Act at all. The 783 court has considered this argument, but finds itself unable to
accept it. The fact that a particular sum is paid into a particular banking account, albeit pursuant to a statutory obligation, does
not affect the right of persons interested in that sum or any duty of the solicitor either towards his client or towards third parties
with regard to disposal of that sum. The provisions of the Solicitors Act, 1957, and the Solicitors Accounts Rules, 1945, to
which I have referred are domestic matters affecting the conduct of the solicitors practice only, and it is a question of fact in each
case whether money received by a solicitor from his client has been so received as to bring the case within the ambit of s 20(1)
(iv)(b) of the Act of 1916.

In the present case, the fact that the money paid to the appellant as counsels fees was intended by the lay client to be applied
for that purpose and the fact that that money had been dishonestly used by the appellant for his own purpose was not in dispute,
though the appellant did contend that it was his intention to pay counsel ultimately. I would refer in this connexion to the
evidence of the appellant himself, where the recorder was putting certain questions to him. The first question reads:

It follows from that, does it, that these cheques were delivered to you with the idea and understanding in both clients
minds and yours that part of the proceeds were to be used to pay counsels fees? You would not question that? A.No, of
course. That was the intention for which the money was given. Q.And the counsels fees were never paid? A.No. Q.
Payments out of pocket does not mean that the counsel have received their fees yet but it means they are going to be
paid? A.Oh, certainly. [Then counsel for the prosecution takes up the questioning and refers the appellant to p. 55 of
one of the exhibits, whereupon the appellant gives this answer:] On your question of receiving counsels fees from clients,
and failing to pay counsel, which was, in fact gross, not negligence, but gross dishonesty on my part, there was never an
intention never to settle those fees. I was invited to make an offer to settle them three months before I was reported to the
Law Society. I made an offer, but it was refused. Before I could renew my offer the papers were reported to the Law
Society. I had made arrangements to discharge all these liabilities.

Then the recorder says: Your failure to pay counsels fees was gross dishonesty on your part?, and the appellant answered
Yes. In the opinion of this court, those admissions by the appellant show that that part of the proceeds of the cheque attributed
by the client to counsels fees was received by the appellant for and on account of counsel concerned, and was fraudulently
converted by the appellant to his own use. It follows that, if these particulars had been laid in the indictment and the jury had
been properly directed on them, the jury must inevitably have convicted of an offence under s 20(1)(iv)(b).
Counsel for the appellant points out, however, that the particulars were not so laid. Count 1, he says, does not allege receipt
of 639 17s for and on account of counsel, but alleges receipt of a cheque for 1,663 9s 7d for and on behalf of the client who
drew the cheque. Counsel contends that, although the charge might perhaps have been laid under s 20(1)(iv)(a) on the footing
that the proceeds of the cheque had been entrusted to the appellant by the client in order that he might pay or deliver part thereof
to counsel, the charge cannot be laid under s 20(1)(iv)(b) on the footing that the cheque was received for and on account of the
client. Counsel for the Crown contends that the charge in this case was properly laid under s 20(1)(iv)(b) and, furthermore, that it
was proper to allege in the charge that the cheque had been received for and on account of the client, Thomas Hedley & Co Ltd.
He has drawn our attention to the practice which has been observed for many years by Crown counsel when settling indictments
under this section of showing what one might call a preference for proceeding under sub-para (b) rather than under sub-para (a),
and in his submission not only is it proper to use sub-para. (b) in the circumstances and in the way in which it has 784 been used
in this case, but he invites us to say that sub-para (b) is, in effect, an all-embracing clause and may be used in cases even though
sub-para (a) would also be appropriate. He has, as I say, referred us to the practice prevailing for many years to treat these two
sub-paragraphs in this way. This court recognises that there is an element of common ground between sub-para (a) and sub-para
(b), but cannot recognise in the facts of this case that it is proper or appropriate to regard the receipt by the solicitor as being a
receipt for or on account of his client. The plain fact of the matter here as this court sees it is that, in this case, the money was
received by the appellant for and on behalf of counsel for whom it was intended. The courts attention has been drawn to R v
Bottomley in which case one of the matters contended before this court was that s 20(1)(iv)(b) was not appropriate in a case
where money was received from an individual and was to be held on account of that same individual. In Bottomleys case this
court has held that, where the facts justified the allegation, it was perfectly proper to use s 20(1)(iv)(b) in such a case, but, as we
see it, there is no authority in R v Bottomley for the proposition that in each and every case money received by a solicitor from a
client will be held on account of that client. Indeed, in this case as we see it, the money was to be held on account of counsel and
not of the client.
We would draw attention to the structure of s 20(1)(iv) as we see it, and point out that sub-para (a) appears to be concerned
with cases where money is entrusted by a transferor to another with intent that it should be passed on to a third party by the
transferee. Indeed, sub-para (a) is concerned with cases where an obligation to pass on to a third party is imposed by the person
who transfers the money in the first instance. Sub-paragraph (b), on the other hand, is appropriate for cases where money is
received by a person with an obligation to hold it on account of another, whether or not that obligation is imposed by the initial
transferor who passed the money to him. In this case, therefore, the facts justified a charge under s 20(1)(iv)(b), and the
statement of offence in the indictment is unobjectionable. However, in the view of this court the particulars of the offence were
defective and ought to have been amended at the trial.
The question then arises whether this is a proper case for the application of the proviso to s 4(1) of the Criminal Appeal Act,
1907. There is no lack of authority for the proposition that the proviso may be used in cases where the indictment was defective
as to the particulars of the offence charged. Thus, in R v Thompson an indictment was bad for duplicity but it was nevertheless
held that, as the prisoner had not been embarrassed or prejudiced in his defence, no substantial miscarriage of justice had actually
occurred; the proviso was accordingly applied. There is a passage in the judgment of the court read by Isaacs CJ which sets out
the principle to be applied in these cases and to which I will refer. It reads that ([1914] 2 KB at p 105):

One of the objects of s. 4 was to prevent the quashing of a conviction upon a mere technicality which had caused no
embarrassment or prejudice. Whilst giving the right of appeal upon any wrong decision of any question of law, the object
of the legislature was that justice should be done in spite of a wrong decision and that the court should not interfere if it
came to the conclusion that, notwithstanding the wrong decision, there had been no substantial miscarriage of justice. The
court must always proceed with caution when it is of opinion that a wrong view of the law has been taken by the judge
presiding at the trial, but when it is apparent, and indeed undisputed, as it is and must be in this case, that no embarrassment
or prejudice had in fact been suffered in consequence of the pleader having made the manifest error above mentioned, the
court must act upon the proviso in this section of the Act.
785

That decision has been followed more recently by a court of five judges in R v McVitie where both the facts and the decision
are adequately summarised in the headnote, as follows. The appellant was convicted on an indictment, the statement of offence
in which charged him and three other men with Possessing explosives, contrary to s 4(1) of the Explosive Substances Act,
1883. The particulars of offence in the indictment stated that the four men had in their possession a certain explosive substance
under such circumstances as to give rise to a reasonable suspicion that it was not in their possession for a lawful object. On
the arraignment, the particulars of the offence alone were read. The appellant appealed against his conviction on the ground that
by reason of the omission of the word knowingly from the particulars of the offence, he had been tried and convicted in respect
of an offence unknown to the law. He had in fact admitted that he knew that he had explosives in his possession, and it was
conceded that he was in no way embarrassed by the omission. The appellant contended that the indictment was not merely
defective but bad, in that it disclosed no offence, and that that was a substantial miscarriage of justice precluding the application
of the proviso to s 4(1) of the Criminal Appeal Act, 1907. It was held that if the words necessary for giving reasonable
information in s 3(1) of the Indictments Act, 1915, imported an objective test, the word knowingly should have been included
in the particulars of the offence, but that since the offence was accurately described in the statement of offence, the omission of
that word from the particulars did not make the indictment bad, but merely defective or imperfect, so that the application of the
proviso was to be considered on the basis that the indictment disclosed a known offence with incomplete particulars. It was
further held that, considering the matter on that basis, and as no embarrassment or prejudice had been caused to the appellant by
the omission of the word knowingly from the particulars and the arraignment, and the essential ingredient of knowledge had
been established, no substantial miscarriage of justice had occurred.
In the opinion of this court, the present case is a proper one for the application of the proviso, if it is clear that the appellant
has not been embarrassed in his defence by the defective particulars and has not lost an opportunity which was fairly open to him
of being acquitted. In the court below, counsel for the appellant led no evidence with regard to these nine counts, and he relied on
his submission of law to which I have already referred. He now contends that, had the particulars been amended in the court of
trial so as to conform to the evidence, he would have cross-examined certain prosecution witnesses and in particular certain
senior barristers clerks with a view to showing that solicitors commonly defer the payment of counsels fees, and that it is
accepted in the profession that the obligation to pay counsels fees is a matter of honour binding on the solicitor and on no one
else. He, therefore, submits that the appellant would be prejudiced if the proviso were applied and this opportunity of cross-
examination were lost. This court, however, is of the opinion that, having regard to the appellants own admissions, no such
evidence could possibly have affected the verdict of the jury, because nothing turns on the particular nature of a solicitors
obligation to pay counsels fees, and to defer a settlement with counsel is quite a different matter from dishonestly
misappropriating moneys held for that purpose.
Accordingly, the court in this case will apply the proviso and will dismiss the appeal, and will also refuse the application for
leave to appeal on count 4.

Appeal dismissed. Application for leave to appeal refused.

Solicitors: Wontner & Son (for the appellant); Director of Public Prosecutions (for the Crown).

N P Metcalfe Esq Barrister.


786
[1963] 2 All ER 787
Mixnams Properties Ltd v Chertsey Urban District Council
TOWN AND COUNTRY PLANNING

COURT OF APPEAL
WILLMER, DANCKWERTS AND DIPLOCK LJJ
6, 7, 8, 23 MAY 1963

Town and Country Planning Caravan site Licence Condition Validity Conditions not relating to physical use of land,
but to control of rents, of contracts with caravan owners, of security of tenure and of normal terms of a tenancy Uncertainty
Unreasonableness Caravan Sites and Control of Development Act, 1960 (8 & 9 Eliz 2 c 62), s 5(1).

Conditions imposed on the grant of a site licence under the Caravan Sites and Control of Development Act, 1960 a, if they are to
be valid, must be such as do not seek to impose some substantial alteration of the law governing the position of the licensee
beyond the immediate scope and object of the Act, viz, control of the physical condition or use of the site (see p 797, letters b, c
and d, and p 804, letter h, post; contrast p 792, letter g, post).
________________________________________
a For the principally relevant provisions of the Act of 1960, viz, s 5(1), see p 795, letters e to i, post.

Principle stated by Denning LJ in Pyx Granite Co Ltd v Minister of Housing and Local Government ([1958] 1 All ER at p
633) applied.
National Assistance Board v Wilkinson ([1952] 2 All ER 255) considered, and dictum of Sir John Romilly MR in Minet v
Leman ((1855), 20 Beav at p 278) applied.
A site licence under the Caravan Sites and Control of Development Act, 1960, was issued subject to conditions which
included the following:

(i) The site rents, which are to be inclusive of all services except electricity, shall be agreed with the council.
(ii) Security of tenure, subject to similar conditions appertaining to a statutory tenancy of a dwelling-house under the
Rent Acts, shall be granted to all caravan occupiers.
(iii) Site rules shall be restricted to those items normally covered by a tenancy agreement, and necessary for the good
administration of the site.
(iv) There shall be no restriction on caravan occupiers as to from whom they purchase the commodities that they
require, or on the callers that they may have for purposes of trade or pleasure.
(v) No premium is to be charged for any caravan occupier entering the site, or any restriction as to the make or
supplier of the caravan. Existing caravan occupiers purchasing new caravans shall not be compelled in any way to
purchase a particular make of caravan or from a particular dealer.
(vi) There shall be no restriction imposed on the caravan occupiers of the formation of, or membership of, any form of
tenants association, political party or other organisations.

The site was subject to existing use rights, so that planning permission for use as a caravan site was not required. On appeal
against an order dismissing an action claiming a declaration that the conditions were void,

Held The conditions were void (Willmer LJ dissenting as to conditions (iv)-(vi)) for the following reasons
(i) (as to condition (i)) because this condition was ultra vires and void as unreasonable (viz, such as Parliament could not
have intended should be imposed), for it sought to impose provisions of rent control, which were not within the ambit of the
Caravan Sites and Control of Development Act, 1960, or (per Diplock LJ) it did not relate to the physical use to be made 787 of
the land (see p 791, letters g and h, p 793, letter b, p 797, letters b, c and d, and p 804, letter i, to p 805, letter a, post).
Kruse v Johnson ([18959] All ER Rep 105) applied.
(ii) (as to condition (ii)) because this condition was void for uncertainty or sought to impose indirectly rent restrictions (see p
793, letters c and d, and p 797, letter f, post), or (per Diplock LJ) its subject-matter did not relate to the physical use of the land
(see p 805, letter e, post).
(iii) (as to condition (iii)) because this condition was void for uncertainty (see p 793, letter c, p 797, letter h, and p 805, letter
g, post).
(iv) (as to conditions (iv-(vi)) because they were outside the purview of the Act of 1960, as (per Diplock LJ) not relating to
the physical use of the land, and any good part of each condition being inseverable (see p 797, letter f, p 805, letter i, and p 806,
letters b, e and f, post).
Appeal allowed.

Notes
As to conditions which may be imposed on the issue of a licence for a caravan site, see 37 Halsburys Laws (3rd Edn) 407409,
para 521.
For the Caravan Sites and Control of Development Act, 1960, s 5(1), see 40 Halsburys Statutes (2nd Edn) 1072.

Cases referred to in judgment


A-G v Denby [1925] 1 Ch 596, 94 LJCh 434, 133 LT 722, 89 JP 145, 26 Digest (Repl) 629, 2780.
Fawcett Properties Ltd v Buckingham County Council [1960] 3 All ER 503, [1961] AC 636, [1960] 3 WLR 831, 125 JP 8, 3rd
Digest Supp.
Gloucester (Bishop) v Cunnington [1943] 1 All ER 61, [1943] KB 101, 112 LJKB 151, 168 LT 68, 31 Digest (Repl) 642, 7492.
Kruse v Johnson [18959] All ER Rep 105, [1898] 2 QB 91, 67 LJQB 782, 79 LT 647, 62 JP 469, 13 Digest (Repl) 239, 639.
Leach v R [1912] AC 305, 81 LJKB 616, 106 LT 281, 76 JP 203, 22 Cox, CC 721, 7 Cr App Rep 158, 14 Digest (Repl) 523,
5070.
May v Waters [1910] 1 KB 431, 79 LJKB 250, 102 LT 180, 74 JP 125, 25 Digest (Repl) 371, 19.
Minet v Leman (1855), 20 Beav 269, 24 LJCh 545, 25 LTOS 57, 19 JP 260, 52 ER 606, on appeal, 7 De GM & G 340, 24 LJCh
545, 25 LTOS 284, 44 ER 133, 42 Digest 674, 862.
Nash v Finlay (1901), 85 LT 682, 66 JP 183, 20 Cox, CC 101, 38 Digest (Repl) 177, 93.
National Assistance Board v Wilkinson [1952] 2 All ER 255, [1952] 2 QB 648, 116 JP 428, 3rd Digest Supp.
Pilling v Abergele UDC [1950] 1 All ER 76, [1950] 1 KB 636, 114 JP 69, 38 Digest (Repl) 231, 474.
Powell v May [1946] 1 All ER 444, [1946] KB 330, 115 LJKB 236, 174 LT 275, 110 JP 157, 13 Digest (Repl) 241, 664.
Pyx Granite Co Ltd v Ministry of Housing and Local Government [1958] 1 All ER 625, [1958] 1 QB 554, [1958] 2 WLR 371,
122 JP 182, CA, revsd [1959] 3 All ER 1, [1960] AC 260, [1959] 3 WLR 346, 123 JP 429, HL, 3rd Digest Supp.
R v Salisbury (Bishop) [1901] 2 KB 225, 70 LJKB 593, 84 LT 553, 65 JP 531, 19 Digest (Repl) 288, 543.
Seward v The Vera Cruz (1884), 10 App Cas 59, 54 LJP 9, 52 LT 474, 49 JP 324, 5 Asp MLC 386, 1 Digest (Repl) 169, 557.
Smith v Hunt (1885), 54 LT 422, 50 JP 279, 16 Cox, CC 54, 25 Digest (Repl) 370, 13.

Appeal
The plaintiffs appealed against an order of Ormerod LJ sitting as an additional judge of the Queens Bench Division, made on 12
February 1963, dismissing an action for a declaration that six conditions attached to a site licence dated 78824 October 1961,
issued by the defendants to the plaintiffs were ultra vires and of no effect. The grounds of appeal were that the lord justice
misdirected himself in holding that the conditions were not ultra vires and of no effect.
The cases noted belowb were cited in argument in addition to those referred to in the judgment.
________________________________________
b Gentel v Rapps [19003] All ER Rep 152, [1902] 1 KB 160, Goldsmiths Company v Wyatt [19047] All ER Rep 542, [1907] 1 KB 95,
Roberts v Hopwood [1925] All ER Rep 24, [1925] AC 578.

D G H Frank for the plaintiffs.


A Garfitt for the defendants.

Cur adv vult

23 May 1963. The following judgments were delivered.

WILLMER LJ. The plaintiffs, as holders of a site licence issued by the defendants in respect of a caravan site at Staines Lane,
Chertsey, brought this action claiming a declaration that certain conditions sought to be imposed by the defendants are ultra vires
and of no effect. No objection is taken to this form of action being resorted to for the purpose of testing the validity of the
conditions. On the contrary, the action has been described to us as a friendly action, and we have been informed that the
defendants are just as anxious as the plaintiffs to obtain a decision as to the validity of their conditions.
The action was tried by Ormerod LJ sitting as an additional judge of the Queens Bench Division. By his judgment, given
on 12 February 1963, he decided that, having regard to the wide power to impose conditions which is contained in the relevant
statutory enactment, the conditions objected to could not be said to be ultra vires. He accordingly dismissed the action, and the
plaintiffs now appeal to this court.
It will be convenient first to refer to the relevant statutory provisions, which are contained in the Caravan Sites and Control
of Development Act, 1960. Section 1(1) provides that after the commencement of the Act no occupier of land shall cause or
permit any part of the land to be used as a caravan site unless he is the holder of a site licence; and by sub-s (2) penalties are
prescribed for contravention of this requirement. section 3(1) provides that application for the issue of a site licence must be
made to the local authority in whose area the land is situated. By sub-s (3) that authority may issue a licence if, and only if, the
applicant is, at the time when the licence is issued, entitled to the benefit of planning permission under Part 3 of the Town and
Country Planning Act, 1947. If the applicant is entitled to the benefit of such planning permission, then by sub-s (4) the issue of
a site licence by the local authority is mandatory; similarly, under sub-s (5), if the applicant becomes entitled to the benefit of
planning permission after the application for a site licence has been made. Subsection (6) forbids a local authority to issue a site
licence to any person who, to their knowledge, has held a site licence which has been revoked within the last three years. This is
a provision on which some reliance was placed by the defendants as showing that, unlike a planning permission (which operates
in rem), a site licence is personal to the licensee.
The power to impose conditions on the issue of a site licence is contained in s 5, sub-s (1) of which is in the following terms:

A site licence issued by a local authority in respect of any land may be so issued subject to such conditions as the
authority may think it necessary or desirable to impose on the occupier of the land in the interests of persons dwelling
thereon in caravans, or of any other class of persons, or of the public at large; and in particular, but without prejudice to the
generality of the foregoing, a site licence may be issued subject to conditions(a) for restricting the occasions on which
caravans are stationed on the land for the purposes of human habitation, or the total number of caravans which 789 are so
stationed at any one time; (b) for controlling (whether by reference to their size, the state of their repair or, subject to the
provisions of sub-s. (2) of this section, any other feature) the types of caravans which are stationed on the land; (c) for
regulating the positions in which caravans are stationed on the land for the purposes of human habitation and for
prohibiting, restricting, or otherwise regulating, the placing or erection on the land, at any time when caravans are so
stationed, of structures and vehicles of any description whatsoever and of tents; (d) for securing the taking of any steps for
preserving or enhancing the amenity of the land, including the planting and replanting thereof with trees and bushes; (e) for
securing that, at all times when caravans are stationed on the land, proper measures are taken for preventing and detecting
the outbreak of fire and adequate means of fighting fire are provided and maintained; (f) for securing that adequate sanitary
facilities, and such other facilities, services or equipment as may be specified, are provided for the use of persons dwelling
on the land in caravans and that, at all times when caravans are stationed thereon for the purposes of human habitation, any
facilities and equipment so provided are properly maintained.

The plaintiffs placed some reliance on sub-s (6), which gives power to the Minister of Housing and Local Government to specify
model standards with respect to the lay-out of, and the provision of facilities, services and equipment for, caravan sites, and
requires that a local authority, in deciding what conditions to impose, shall have regard to any standards so specified. We were
shown a leaflet published by the minister specifying model standards in pursuance of this subsection; but the matters dealt with
therein are remote from the conditions sought to be imposed in the present case; and I confess that I do not find the leaflet of
much assistance in deciding the issues which fall to be determined here. Reference should also be made to s 7, which provides
that any person aggrieved by a condition subject to which a site licence has been issued to him may, within twenty-eight days of
the date on which the licence was issued, appeal to the local magistrates court, and that court, if satisfied that the condition is
unduly burdensome, may vary or cancel it. Lastly, it is necessary to refer to s 9, which provides that an occupier of land who
fails to comply with a condition attached to a site licence held by him shall be guilty of an offence, for which penalties are
prescribed. It is, I think, important to bear in mind, when considering the validity of conditions sought to be imposed, that breach
thereof could have penal consequences.
In the present case the site had been in use for many years as a caravan site, and it was claimed (and not denied) that it was
subject to existing use rights, so as not to require planning permission. The defendants were consequently bound to issue a site
licence, and it is in this context that the validity of the conditions sought to be imposed has to be considered. A large number of
conditions was attached to the site licence in this case, but objection is taken to six of them only, viz, Nos (28) to (33), which
come under the heading: (E) Conditions of Tenancy. It is, I think, desirable to set out in full the terms of the disputed
conditions. They are as follows:

(28) The site rents, which are to be inclusive of all services except electricity, shall be agreed with the council. (29)
Security oftenure, subject to similar conditions appertaining to a statutory tenancy of a dwelling-house under the Rent Acts,
shall be granted to all caravan occupiers. (30) Site rules shall be restricted to those items normally covered by a tenancy
agreement, and necessary for the good administration of the site. (31) There shall be no restriction on caravan occupiers as
to from whom they purchase the commodities that they require, or on the callers that they may have for purposes of trade
or pleasure. (32) No premium is to be charged for any caravan occupier entering the site, or any restriction as to the make
or 790 supplier of the caravan. Existing caravan occupiers purchasing new caravans shall not be compelled in any way to
purchase a particular make of caravan or from a particular dealer. (33) There shall be no restriction imposed on the caravan
occupiers of the formation of, or membership of, any form of tenants association, political party or other organisations.

The case for the plaintiffs was that, wide as are the words of s 5, they must be read as subject to some limitation and cannot be
construed as conferring an unfettered discretion on local authorities to impose whatever conditions they like. Ormerod LJ
appears to have accepted that the section cannot be read as conferring an unfettered discretion, but he concluded that the only
fetter is the right conferred by s 7 on any person aggrieved to appeal to the magistrates if any condition imposed is considered to
be unduly burdensome. I find myself unable to accept this view, and indeed counsel for the defendants has not sought to support
it. It must, I think, be clear that the question whether a condition is burdensome or not is of no necessary relevance to the inquiry
whether it is or is not within the powers conferred. A condition which is clearly intra vires may yet be unduly burdensome, and
the converse is equally true. In this court it has been accepted that, in spite of the wide terms in which it is expressed, the power
to impose conditions which is conferred by s 5 of the Act of 1960 must be subject to some limitation. Counsel for the defendants
admitted, for instance, that a local authority would not be within its powers in seeking to impose a condition contrary to law. He
also conceded that the powers under the section must be limited to matters within the ambit of the statute, though he made it clear
that the defendants took a different view from that of the plaintiffs as to what the ambit of the statute is.
In the light of the authorities cited to us, I think it is possible to say that such a statutory power to impose conditions, even
though expressed in the widest terms, must be held to be subject to limitation in four well defined respects. (1) The conditions
must not be such as to effect a fundamental alteration in the general law relating to the rights of the persons on whom they are
imposed unless the power to effect such an alteration is expressed in the clearest possible terms. This is no more than an
application of the general principle that no statute is to be construed as effecting a substantial alteration in the law beyond what it
expressly declares; see National Assistance Board v Wilkinson for a very clear statement of the principle involved. (2) The power
to impose conditions must be limited by reference to the subject-matter of the statuteie, the conditions must be such as fairly
fall within the ambit of the statute. Thus in Pyx Granite Co Ltd v Ministry of Housing and Local Government in relation to the
analogous power to impose conditions conferred by s 14 of the Town and Country Planning Act, 1947, Denning LJ in an oft
quoted dictum said ([1958] 1 All ER at p 633; [1958] 1 QB at p 572): The law says that those conditions, to be valid, must
fairly and reasonably relate to the permitted development. Subject to a difference of view as to what is the ambit of the Act in
question here, this principle is accepted by both sides. (3) The conditions must not be unreasonable, ie, such as Parliament
clearly cannot have intended should be imposed. In this respect it seems to me that the construction of a statutory power to
impose conditions should be approached in the same way as that of a power to make bye-laws. The principles stated in Kruse v
Johnson appear to me to be equally applicable. It is to be remarked that in that case Lord Russell Of Killowen ([18959] All ER
Rep at p 110; [1898] 2 QB at p 99) was at pains to emphasise that in considering unreasonableness, bye-laws made by a local
authority ought to be benevolently interpreted. In the light of this he went on to consider 791 in what sense bye-laws must be
regarded as unreasonable before they can properly be held to be ultra vires. He said ([18959] All ER Rep at p 110; [1898] 2 QB
at p 99):

If, for instance, they were found to be partial and unequal in their operation as between different classes, if they were
manifestly unjust, if they disclosed bad faith, if they involved such oppressive or gratuitous interference with the rights of
those subject to them as could find no justification in the minds of reasonable men, the court might well say that Parliament
never intended to give authority to make such rules, and that they are unreasonable and ultra vires. But it is in this sense,
and in this sense only, as I conceive, that the question of unreasonableness can properly be regarded.

I would apply a similar test in considering the conditions sought to be imposed in the present case. (4) Although I do not think it
is strictly a question of vires, it must be accepted that a condition may be held void for uncertainty, and consequently
unenforceable, if it is ambiguous or uncertain in its application. It is to be remembered that, in the event of non-compliance with
one of the conditions, penal consequences may follow. It is therefore important that the conditions sought to be imposed should
be unambiguous and free from uncertainty.
I do not think that there can be much room for doubt as to the principles underlying these four classes of limitation on the
power of a local authority under s 5 of the Act of 1960 to attach conditions to the grant of a site licence. The difficulty here, as so
often happens, lies in the application of well established principles to the circumstances of the particular case. It is necessary to
examine each of the six conditions here objected to on its own merits, and in the light of the principles which I have already
stated. But, before doing so, I think that it is necessary to form some view as to the ambit of the statute in question here. It was
strenuously argued for the plaintiffs that this statute covers no more than (a) matters of public health and sanitation, and (b)
matters relating to town and country planning. We were accordingly pressed to say that a condition imposed by a local authority,
in order to be valid, must be relevant to one or other of these aspects, since otherwise it would not be within the ambit of the
statute. At least, it was argued, the power to impose conditions must be limited to matters affecting the physical use of the land. I
do not take such a narrow view of the scope of the statute. In my judgment, the opening words of the long title, viz, An Act to
make further provision for the licensing and control of caravan sites, indicate that the statute is intended to be of general
application, and to cover all aspects of the regulation of caravan sites. I do not think, therefore, that the six conditions here
objected to, or any of them, can be held to be bad, as not being within the ambit of the statute, merely because they relate to
conditions of tenancy, and are not directly relevant either to public health or planning.
I turn, therefore, to consider individually the conditions here objected to. Condition No (28) is the one which to my mind
presents the greatest difficulty. I was at one time disposed to think that it should be held void for uncertainty, in that there could
be no certainty that agreement as to site rents would be reached. But on reflection I do not think that this is a valid objection.
The condition means no more than that site rents must be approved by the local authority. A more substantial objection is that the
imposition of this condition amounts to an attempt, contrary to the general law, to introduce a system of rent control in relation to
a type of living accommodation, viz, caravans, to which Paraliament has never seen fit to extend it.
I do not think, however, that the matter is quite so easy as that. There is at least something to be said for the view that a
condition limiting site rents to be charged to a figure specified in the condition might not be outside the powers of the local
authority, notwithstanding that it might be said to involve some degree of rent control. If the figure specified in the condition
were thought to be burdensome, 792the site licensees would have their remedy by way of appeal to the magistrates court. But
the vice of the condition sought to be imposed here is that it leaves the site licensees with no redress. The condition cannot be
said to be burdensome until the local authority has refused to agree the site rents proposed to be charged. Against such a
refusal there is no right of appeal. The only appeal is against the condition, and this must be brought within twenty-eight days of
the licence being issued. I have, therefore, come to the conclusion that condition No (28) is ultra vires on the ground that it is
unreasonable in the sense explained by Lord Russell CJ in Kruse v Johnson (See p 791, letter h, ante). It would enable the local
authority to behave in a completely arbitrary manner, and could indeed be used to render the site licence virtually valueless to the
licensee. It seems to me that such a condition is one that Parliament can never have intended should be imposed.
As to conditions Nos (29) and (30), I am in the end left in no doubt that both are objectionable, and should be held to be
unenforceable. Both, as it seems to me, should be held void for uncertainty. To draw up terms similar to those appertaining to a
statutory tenancy of a dwelling-house under the Rent Acts, or covering those items normally covered by a tenancy agreement
might be theoretically possible. But such a document would require exceedingly careful drafting, and would necessarily be most
complicated. A successful prosecution for breach of such a condition would, I should have thought, be quite out of the question.
This is enough to render both these conditions invalid. But in addition I entertain no doubt but that condition No. (29) is ultra
vires in that it is designed to introduce, by indirect means, and contrary to the general law, a system of control analogous to that
under the Rent Acts, which have never been applied to caravans.
That leaves conditions Nos (31), (32) and (33), all of which to my mind are quite unobjectionable, and well within the
powers conferred on a local authority under s 5 of the Caravan Sites and Control of Development Act, 1960. All these conditions
appear to me to do no more than protect caravan occupiers from exploitation by the licensees of the site and preserve their
freedom of contract and other basic rights of citizenship. This seems to me to fall well within the ambit of the statute, as this is
expressed in the words already quoted from its long title. I can see nothing contrary to the general law in such a condition, still
less anything unreasonable. Any unreasonableness is that of the plaintiffs in objecting to such conditions, and in seeking to take
advantage of their position so as to exploit the caravan occupiers to whom sites are let.
The result of this is that in my judgment the appeal should be allowed in relation to conditions Nos. (28), (29) and (30), all
of which should be pronounced void. In relation to conditions Nos (31), (32) and (33), on the other hand, the appeal should in
my view be dismissed. My brethren, however, take a different view, and the order of the court will accordingly be as they may
direct.

DANCKWERTS LJ. In this action the plaintiffs, who are the owners and occupiers of some land at Staines Lane, Chertsey,
which is used as a caravan site, claim against Chertsey Urban District Council, who are the appropriate licensing authority under
the Caravan Sites and Control of Development Act, 1960, a declaration that certain conditions attached to a licence granted to the
plaintiffs are ultra vires and of no effect.
There were no less than thirty-seven conditions attached to the licence, but the conditions of which the plaintiffs complain in
the action are those numbered (28) to (33) inclusive. The contention of the plaintiffs is that the imposition of these conditions is
not within the powers conferred on the defendants by the Act of 1960. That is the main contention of the plaintiffs, though the
question whether these conditions are void because they are unreasonable, and the question whether 793 they are void for
uncertainty, have also been discussed in the course of the hearing. The action was heard by Ormerod LJ sitting as an additional
judge of the Queens Bench Division on 12 February 1963, and the lord justice held that these conditions were validly imposed,
and dismissed the action.
The conditions in question are grouped together as: (E) Conditions of Tenancy, a description which I understand refers to
the terms on which the inhabitants of the caravans on the site are to be required or permitted to occupy the caravans. It seems to
me that tenancy is not an accurate description of the rights which the occupiers will acquire in respect of the caravans. I think
that their rights cannot be more than those of licensees. But, of course, it is possible for the plaintiffs, as the owners of the site, to
grant tenancies of the portions of the land on which the caravans stand. The conditions are as follows:

(28) The site rents, which are to be inclusive of all services except electricity, shall be agreed with the council. (29)
Security of tenure, subject to similar conditions appertaining to a statutory tenancy of a dwelling-house under the Rent
Acts, shall be granted to all caravan occupiers. (30) Site rules shall be restricted to those items normally covered by a
tenancy agreement, and necessary for the good administration of the site. (31) There shall be no restriction on caravan
occupiers as to from whom they purchase the commodities that they require, or on the callers that they may have for
purposes of trade or pleasure. (32) No premium is to be charged for any caravan occupier entering the site, or any
restriction as to the make or supplier of the caravan. Existing caravan occupiers purchasing new caravans shall not be
compelled in any way to purchase a particular make of caravan or from a particular dealer. (33) There shall be no
restriction imposed on the caravan occupiers of the formation of, or membership of, any form of tenants association,
political party or other organisations.

The Caravan Sites and Control of Development Act, 1960, is an extension of, and a substitution for, s 269 of the Public
Health Act, 1936. Certain provisions of the Town and Country Planning Act, 1947, which are contained in Part 2 of the Act of
1960 have since been incorporated in the Town and Country Planning Act, 1962, leaving Part 1, which deals with caravan sites
(the material part of the Act) operative for the purposes of this action. In Pilling v Abergele Urban District Council the
Divisional Court of the Kings Bench Division held that s 269 of the Public Health Act, 1936, was confined to matters of health
and sanitation, so that the local authority were not entitled to take matters of amenity into account in refusing a licence.
The Act of 1960 obviously covers a wide field, and is also designed to provide more satisfactory methods of enforcing
control over caravan sites. It is described in the preamble as:

An Act to make further provision for the licensing and control of caravan sites [and certain other purposes therein
described] and for connected purposes.

Section 1(1) provides that:

Subject to the provisions of this Part of this Act, no occupier of land shall after the commencement of this Act cause or
permit any part of the land to be used as a caravan site unless he is the holder of a site licence (that is to say, a licence under
this Part of this Act authorising the use of land as a caravan site) for the time being in force as respects the land so used.

Subsection (2) imposes substantial penalties for contravention of sub-s (1). Subsection (3) defines occupier, and sub-s (4)
provides that:

In this Part of this Act the expression caravan site means land on which 794 a caravan is stationed for the purposes of
human habitation and land which is used in conjunction with land on which a caravan is so stationed.

Section 2 contains certain exemptions by reference to Sch 1 which are not material. Section 3 provides for the method of
application for a licence. Subsection (3) of this section provides that:

A local authority may on an application under this section issue a site licence in respect of the land if, and only if, the
applicant is, at the time when the site licence is issued, entitled to the benefit of a permission for the use of the land as a
caravan site granted under Part 3 of the Act of 1947 otherwise than by a development order.

Under sub-s (5), if the applicant becomes entitled to planning permission, the authority have to issue a site licence in respect of
the land within six weeks unless there is agreement for a longer period. Subsection (6) is of interest because it provides that:
795
Notwithstanding anything in the foregoing provisions of this section, a local authority shall not at any time issue a site
licence to a person who to their knowledge has held a site licence which has been revoked in pursuance of the provisions of
this Part of this Act less than three years before that time.

Section 4 provides for duration of licences.


Section 5 is the section around which the main argument revolves. This section provides as follows:

(1) A site licence issued by a local authority in respect of any land may be so issued subject to such conditions as the
authority may think it necessary or desirable to impose on the occupier of the land in the interests of persons dwelling
thereon in caravans, or of any other class of persons, or of the public at large; and in particular, but without prejudice to the
generality of the foregoing, a site licence may be issued subject to conditions(a) for restricting the occasions on which
caravans are stationed on the land for the purposes of human habitation, or the total number of caravans which are so
stationed at any one time; (b) for controlling (whether by reference to their size, the state of their repair or, subject to the
provisions of sub-s. (2) of this section, any other feature) the types of caravan which are stationed on the land; (c) for
regulating the positions in which caravans are stationed on the land for the purposes of human habitation and for
prohibiting, restricting, or otherwise regulating the placing or erection on the land, at any time when caravans are so
stationed, of structures and vehicles of any description whatsoever and of tents; (d) for securing the taking of any steps for
preserving or enhancing the amenity of the land, including the planting and replanting thereof with trees and bushes; (e) for
securing that, at all times when caravans are stationed on the land, proper measures are taken for preventing and detecting
the outbreak of fire and adequate means of fighting fire are provided and maintained; (f) for securing that adequate sanitary
facilities, and such other facilities, services or equipment as may be specified, are provided for the use of persons dwelling
on the land in caravans and that, at all times when caravans are stationed thereon for the purposes of human habitation, any
facilities and equipment so provided are properly maintained.

Subsection (2) provides that:

No condition shall be attached to a site licence controlling the types of caravans which are stationed on the land by
reference to the materials used in their construction.

Section 5(6) provides that:

The minister may from time to time specify for the purposes of this section model standards with respect to the lay-out
of, and the provision of facilities, services and equipment for, caravan sites or particular types of caravan site; and in
deciding what (if any) conditions to attach to a site licence, a local authority shall have regard to any standards so
specified.

The Minister of Housing and Local Government in fact produced in 1960 a set of model standards. But these do not seem to me
to add anything to the solution of the questions before us. It is plain that it is not compulsive on a local authority to adopt these
standards. The local authority, no doubt, must look at them and consider them, but having done that, it seems clear that the
authority may reject them completely.
Section 7 may be of some importance as it enables any person aggrieved by any condition attached to a licence to appeal
within twenty-eight days of the issue of the licence to petty sessions; and that court, if satisfied that the condition is unduly
burdensome, may vary or cancel the condition. It seems to be accepted that a petty sessions court in such a case is not concerned
with the validity of the condition in question, and that such an appeal would not be appropriate in the present case. Section 8
gives the local authority power to alter conditions attached to site licences. Counsel for the plaintiffs appeared not to regard this
as a beneficial provision, but as a weapon in the hands of the authority enabling them to avoid the effect of bad drafting rendering
their first efforts void for uncertainty.
Section 9 provides substantial penalties for breaches of condition. This, no doubt, is an essential feature of the Act to secure
the compliance which was so easily evaded under the Public Health Act, 1936, but it renders the provisions of the statute
provisions of a penal nature with the consequential strictness of interpretation. It should be noticed that, though the licence is a
personal matter, provisions for transfer are contained in s 10. This should be compared with the position of permissions under the
Town and Country Planning Acts, which are sometimes said to be attached to the land and matters in rem. Section 30 repeals s
269 of the Public Health Act, 1936, as regards caravan sites. Part 2 having been taken out of the Act, Part 1 remains as a code
containing the present provisions as to the licensing of caravan sites.
Read literally, the words of s 5(1):

A site licence issued by a local authority in respect of any land may be so issued subject to such conditions as the
authority may think it necessary or desirable to impose on the occupier of the land

appear to confer on the authority the widest discretion to impose conditions as long as they are bona fide thought to be

in the interests of persons dwelling thereon in caravans, or of any other class of persons, or of the public at large.

The words without prejudice to the generality of the foregoing prevent the particular considerations (a), (b), (c), (d), (e) and (f),
which are thereafter mentioned in the section, having any restrictive effect, and exclude the ejusdem generis rule. Of course, the
curious provision in sub-s (2) has a restricting effect so far as it goes. These provisions may give some indication of the objects
which the legislature has in mind.
There is a principle, however, in the interpretation of statutes which is stated in Maxwell On Interpretation Of Statutes (11th
Edn) at p 78 as follows:

Presumption against implicit alteration of law. One of these presumptions is that the legislature does not intend to
make any substantial alteration in the law beyond what it explicitly declares, either in express terms or by clear implication,
or, in other words, beyond the immediate scope and object of the statute. In all general matters outside those limits the law
remains undisturbed. It is in the last degree improbable that the legislature would overthrow fundamental principles,
infringe rights or depart from the general 796 system of law, without expressing its intention with irresistible clearness, and
to give any such effect to general words, simply because they have a meaning that would lead thereto when used in either
their widest, their usual or their natural sense, would be to give them a meaning other than that which was actually
intended. General words and phrases, therefore, however wide and comprehensive they may be in their literal sense, must,
usually, be construed as being limited to the actual objects of the Act.

Examples of these propositions are National Assistance Board v Wilkinson, Leach v R and R v Bishop of Salisbury.
Applying this principle to the present case, it is argued that the terms of conditions Nos (28), (29) and (32), in particular, are
objectionable, because they seek to impose provisions of rent control in a special Act dealing with a special subject. It is pointed
out that it is most improbable that Parliament would have intended to give power to extend the system of rent control to caravan
sites by a side wind in a statute of 1960, when by the Rent Act, 1957, Parliament had shown an intention to reduce the system of
rent control and artificial security of tenure. Moreover, it has been pointed out that, when Parliament intends such control shall
apply, specific provisions to that effect are included in the special Act, eg, in s 43 of the Housing Act, 1949, and s 37 of the
Housing Repairs and Rents Act, 1954.
I find these contentions convincing. This Act of 1960 is undoubtedly a special Act concerned with the control of caravan
sites. The object of the Act appears to me to be to control the physical conditions of a caravan site so as to prevent its becoming
in the nature of a public nuisance. I think that the wide powers which were given to the local authority were given to the
authority to enable them to secure the provision of proper facilities in respect of sanitation and amenities so that the site should
not become a menace to the health of the occupants of the caravans or other persons who might be affected by the site, and also
to prevent the site becoming an eyesore, or in any other way a nuisance to the members of the public who may dwell in the
neighbourhood or pass through the locality. I cannot believe that Parliament intended to confer on the authority power to impose
on the owner of the site the complications of rent control so as to restrict his power to make contracts with the persons who wish
to use the caravan sites. On this footing, all of the conditions Nos (28) to (33) seem to me to go beyond the powers which were
conferred by the Act on the authority, and they are therefore void because they are ultra vires.
Some at least of these conditions seem to me to be vulnerable on the ground of uncertainty. I do not agree with the
suggestion made that condition No (28) was too uncertain. It seems to me to mean no more than that the rents were to be
approved by the defendants, and a term on these lines is by no means uncommon in contracts. But conditions Nos (29) and (30)
are, in my view, quite impossible to apply. Difficulties would arise at once if it were sought to apply the complicated provisions
of the Rent Acts to caravan sites merely occupied by persons under licences with various amenities enjoyed by persons in
common. And what does similar conditions mean in this connexion? As regards condition No (30), it seems to me to be
impossible to say what items are normally covered by a tenancy agreement in this connexion. Conditions Nos (31), (32) and (33)
may be well intentioned, but they affect the personal or social lives of the caravan dwellers rather than the amenities of the site.
They are not only far outside the objects of the Act, as it seems to me, but also might be very difficult to apply in practice. For
these reasons I would allow the appeal and make the declaration which is asked for by the writ.

DIPLOCK LJ. The question in this appeal is whether certain conditions attached by the defendants to a site licence issued to the
plaintiffs as occupiers of 797 a caravan site are ultra vires the powers of a local authority under s 5(1) of the Caravan Sites and
Control of Development Act, 1960. The six disputed conditions, which I shall have to examine in detail, are incorporated in the
site licence under the heading Conditions of tenancy, and purport to prescribe and restrict the terms of the contracts into which
the occupiers may enter with persons dwelling, or desirous of dwelling, in caravans stationed on the site. The plaintiffs contend
that the Act does not confer on the local authority power to impose any, or alternatively certain, of these conditions. Failure to
comply with a condition attached to a site licence is a criminal offence. The power to impose conditions is thus in effect a power
to make subordinate legislation analogous to a power to make bye-laws. (See per Lord Denning in Fawcett Properties Ltd v
Buckingham County Council ([1960] 3 All ER at p 517; [1961] AC at p 679).) The validity of such conditions is thus to be tested
by the same principle as the validity of bye-laws.
The power to impose conditions on occupiers of caravan sites is conferred on local authorities by s 5 of the Act of 1960 in
wide terms which, if read literally and in isolation, would authorise the imposition of all the disputed conditions and many more
fanciful conditions which have been suggested in the course of the argument; but it is a well-settled rule of construction that
expressions used in particular sections of statutes must be read secundum subjectam materiam, with the result that, having regard
to the context in which they appear and the subject-matter to which the statute relates, they are to be read in a less extended sense
than their literal meaning. This is a rule of general application, and not restricted to statutory provisions which confer powers of
subordinate legislation. A clear example is to be found in Smith v Hunt and May v Waters, where the wide words in s 6 of the
Ground Game Act, 1880: No person having a right of killing ground game under this Act or otherwise were construed as
limited by the subject-matter of the remainder of the Act so as not to include the owner of the land or his tenants or licensees of
the sporting rights. A more recent example is to be found in National Assistance Board v Wilkinson. Lord Goddard CJ there
([1952] 2 All ER at p 259; [1952] 2 QB at p 659) cited with approval the statement of Sir John Romilly MR in Minet v Leman
((1855), 20 Beav at p 278):

the general words of the Act are not to be so construed as to alter the previous policy of the law, unless no sense or
meaning can be applied to those words consistently with the intention of preserving the existing policy untouched

The previous policy of the law, which was relevant in National Assistance Board v Wilkinson, was that a husband was not bound
to maintain an adulterous wife. The National Assistance Act, 1948, which, be it noted, made sweeping changes in the poor law,
contains a section which, on the face of it, expressly imposed an unqualified liability on a husband to maintain his wife not
generally but for the purposes of this Act c. The next following section empowered the National Assistance Board who had
given assistance to a person to recover sums from d any other person who for the purposes of this Act is liable to maintain the
person assisted. This, therefore, was not a case where general words were relied on as effecting a change in the general law as
to the liability of a husband to maintain an adulterous wife. The contention was that a limited alteration had been made in the
general law in the special case where a wife had received national assistance so as to enable not the wife herself, or other persons
who had supplied her with necessaries, but the National Assistance Board only to recover from the 798 husband expenses to
which they had been put for her maintenance, It was nevertheless held that the general words of an Act dealing with the special
subject-matter of poor law (or national assistance) were to be construed so as not to alter the previous general policy of the law
that a husband was not liable, whether directly or indirectly, to maintain an adulterous wife, since a more limited sense or
meaning could be applied to them consistently with the intention of preserving the existing policy untouched. It is indeed an
illustration of the converse of the familiar maxim generalia specialibus non derogant, which is illustrated by such cases as Seward
v The Vera Cruz (Owner) and Bishop of Gloucester v Cunnington. The cases which I have cited are all illustrative of the general
rule of construction applicable to all statutes that the legislature is presumed not to have intended to make any substantial
alteration in the law beyond the immediate scope and object of the statute.
________________________________________
c National Assistance Act, 1948, s 42(1); 16 Halsburys Statutes (2nd Edn) 968.
d National Assistance Act, 1948, s 43; 16 Halbsburys Statues (2nd Edn) 969.

This general rule of construction has a particular application to clauses in statutes by which Parliament authorises other
persons or authorities to exercise powers of subordinate legislation whether directly by means of bye-laws or (as in the present
case) indirectly by subjecting what would otherwise be lawful activities of citizens to conditions the breach of which gives rise to
penal sanctions. Parliament can make whatever changes it likes in the law, but subordinate legislative authorities can make only
such changes in the law as Parliament has expressly empowered them to make.
The various special grounds on which subordinate legislation has sometimes been said to be voideg, because it is
unreasonable; because it is uncertain; because it is repugnant to the general law or to some other statutecan, I think, today be
properly regarded as being particular applications of the general rule that subordinate legislation, to be valid, must be shown to be
within the powers conferred by the statute. Thus, the kind of unreasonableness which invalidates a bye-law is not the antonym of
reasonableness in the sense in which that expression is used in the common law, but such manifest arbitrariness, injustice or
partiality that a court would say: Parliament never intended to give authority to make such rules; they are unreasonable and
ultra vires; Kruse v Johnson ([18959] All ER Rep at p 110; [1898] 2 QB at p 99) per Lord Russell Of Killowen CJ. Bye-laws
have in the past been declared void for uncertainty; see Nash v Finlay ((1901), 85 LT at p 683) and A-G v Denby. Some doubt
is cast on the correctness of uncertainty as a separate ground of invalidity by the speeches in the House of Lords in the recent
case of Fawcett Properties Ltd v Buckingham County Council but if the courts can declare subordinate legislation to be invalid
for uncertainty as distinct from unenforceable, as in the case of a clause in a statute to which it is impossible to ascribe a
meaning, this must be because Parliament is to be presumed not to have intended to authorise the subordinate legislative authority
to make changes in the existing law which are uncertain. Invalidity on the ground of repugnancy to the general law or to some
other statute, of which ground there is a very useful exposition in Powell v May, is merely an application to clauses which
empower a subordinate authority to make changes in the law, of the general rule of construction that the legislature is presumed
not to have intended to make any substantial alteration in the law beyond the immediate scope and object of the statute which
confers the power. If this presumption applies, as it does where Parliament is itself making changes in the law by the direct
exercise of its own sovereign powers, it applies a fortiori to the construction of clauses by which Parliament delegates its
sovereign powers to persons over whose exercise of the delegated powers Parliament itself retains no direct control.
The initial problem in the present case is thus to ascertain what is the 799 immediate scope and object of the Caravan Sites
and Control of Development Act, 1960. What is it a law about? Its subject-matter, as stated in the relevant part of the preamble,
is

to make further provision for the licensing and control of caravan sites, to authorise local authorities to provide and
operate caravan sites.

It is thus an Act to make changes in the law relating to its subject-matter as it existed before it came into force. What changes it
is intended to effect cannot be properly ascertained without considering what the pre-existing law on this subject-matter was. At
common law a person entitled to possession of land was entitled to use it for stationing caravans as he thought fit subject to such
general restraints as were imposed by common law rules as to nuisance. He was also entitled to license other persons to station
caravans on his land on whatever terms he thought fit.
Before the Act of 1960 these common law rights had been curtailed by Parliament in two different ways. By s 269 of the
Public Health Act, 1936, the use of land for stationing (inter alia) caravans for more than forty-two consecutive days, or for more
than sixty days in any twelve months, was prohibited unless the occupier of the land held a licence issued by the local authority.
The local authority was entitled to refuse a licence, or to attach to any such licence such conditions as they thought fit with
respect to certain specified matters relating to what may be broadly described as public health. There was a right of appeal to the
justices on the part of an aggrieved occupier against the refusal of a licence or any condition attached to a licence. This section
expressly limited the kind of condition which the local authority could validly attach to a licence; and it was held in Pilling v
Abergele Urban District Council, construing the section secundum subjectam materiam, that, in determining whether or not to
issue a licence, the local authority were not entitled to take into consideration questions other than those of sanitation and public
health.
This decision was purely one on the construction of the Public Health Act, 1936. It was not based on the concurrent
existence by 1950 of the further statutory provisions contained in the Town and Country Planning Act, 1947, which also affected
the use of land as a caravan site. That Act controlled the development of land which includes (inter alia) the making of any
material change in the use of land. It did not control the continuation of an existing use of land, and planning permission
authorising a change in the use of land (unlike a licence under s 269 of the Public Health Act, 1936) operated in rem, and enured
to the benefit of all persons from time to time interested in the land. Under the Town and Country Planning Act, 1947, land
which had not previously been used as a caravan site could not be so used without planning permission issued by the local
planning authority, who were authorised to grant such permission either unconditionally or subject to such conditions as they
think fit. Despite the wide words in which the power to impose conditions was conferred in the Town and Country Planning
Act, 1947, any conditions so imposed to be valid must fairly and reasonably relate to the permitted development; Pyx Granite
Co Ltd v Minister of Housing and Local Government ([1958] 1 All ER at p 633; [1958] 1 QB at p 572), per Denning LJ. This
statement of the law was approved by the House of Lords in Fawcett Properties Ltd v Buckingham County Council ([1960] 3 All
ER at pp 509, 515, 522; [1961] AC at pp 674, 678, 685) already referred to, and is, in my view, of general application mutatis
mutandis to similar provisions in statutes which authorise subordinate authorities to attach conditions in their grant of a licence,
or permission to do acts of a specified character.
The state of the law at the date of the passing of the Caravan Sites and Control of Development Act, 1960, was thus that
certain land had, for planning purposes, the status of land which could be used as a caravan site, either because 800 permission
so to use it had been granted by the local planning authority under the Town and Country Planning Act, 1947, or because it had
been used for that purpose before that Act came into force so that planning permission for its use as a caravan site was
unnecessary. The occupier of land possessing the necessary status was entitled to use it as a caravan site for certain limited
periods in each year without any further licence or permission; but he could not lawfully use it as a caravan site for more
extended periods without obtaining a licence, personal to himself, from the local authority (which would not necessarily be the
local planning authority) and complying with such conditions relating to public health and sanitation as the local authority,
subject to appeal to the justices, thought fit to attach to the licence. The occupier of land which did not possess the necessary
status could not use it as a caravan site until he obtained planning permission from the local planning authority giving it the
status of land which could be used as a caravan site, to which permission the planning authority could attach such conditions
relating to planning considerations as they thought fit; but such planning permission did not obviate the necessity of his obtaining
also the licence personal to himself from the local authority under s 269 of the Public Health Act, 1936.
It would seem to me to be clear that neither the local authority under the Public Health Act, 1936, nor the local planning
authority under the Town and Country Planning Act, 1947, could attach either to a licence or to a planning permission any
conditions regulating the contractual relations between the occupier of the caravan site and the persons whom he permitted to
station caravans on the site, except in so far as those conditions fairly and reasonably related, in the case of the licence, to public
health or sanitation and (in the case of the planning permission) to the way in which the land was to be used by the persons whom
the occupier permitted to station caravans thereon. It is also relevant to observe that Parliament had for more than forty years
before 1960 imposed restrictions on the freedom of contract of parties to tenancy agreements, and other contracts not amounting
to tenancies, for the use of premises for human habitation. These restrictions have been imposed by a long series of statutes
generically referred to as the Rent Acts; and where Parliament has sought to extend restrictions of this kind to contracts to
which they did not previously apply, it did so by an Act specifically relating to this subject-matter (compare Furnished Houses
(Rent Control) Act, 1946; Crown Lessees (Protection of Sub-Tenants) Act, 1952). These Acts did not apply to caravans, which
are capable of being moved from one place to another. Parliament, up to 1960, had thus treated public health, planning and the
control of contracts for the use of premises for human habitation as three separate legislative subject-matters.
It is against that common law and legislative background that the Caravan Sites and Control of Development Act, 1960, has
to be construed. Part 1 with which this appeal is concerned, and Part 2, have been drafted as if each Part was a separate and self-
contained Act, and Part 2 has recently been repealed and its provisions incorporated in the Town and Country Planning Act, 1962.
The Act maintains the previous dichotomy between planning permission which operates in rem and affects the status of land,
and a licence to use the land as a caravan site (called in the Act a site licence) which is personal to the occupier of the land.
Save in minor respects, most of which are of a transitional character, Part 1 of the Act effects no substantial alteration in the
existing law as respects planning permission for the use of land as a caravan site. It is concerned primarily with the amendment
of the law relating to the licence to use the land as a caravan site which is personal to the occupier.
A site licence is defined in s 29(1) and s 1(1) as a licence authorising the use of the land as a caravan site, and a
caravan site is defined in s 1(4) as

land on which a caravan is stationed for the purposes of human habitation 801 and land which is used in conjunction
with land on which a caravan is so stationed.

A site licence is thus a licence controlling the physical use to which land may be put. Section 1(1) of the Act enacts that:

no occupier of land shall cause or permit any part of the land to be used as a caravan site unless he is the holder
of a site licence for the time being in force as respects the land so used.

A breach of this prohibition is a criminal offence. The prohibition is personal to the occupier, who is defined, in effect, as the
person entitled to possession of the land, and who would thus at common law be entitled to licence other persons to station
caravans on the land. The prohibition does not extend to the licensees of the occupier, but the section is concerned with the
occupiers contractual relations with his licensees, for he commits an offence if he permits his licensees to use the land as a
caravan site. Section 2 and Sch 1 set out a number of exceptions to the general prohibition contained in s 1 by specifying a
number of circumstances in which the use of land as a caravan site does not require a site licence. Section 3, which deals with
applications for site licences, makes it clear that a site licence cannot be issued by the local authority (which may or may not be
the local planning authority) in respect of land which has not got planning permission for its use as a caravan site, but that the
occupier of land which has such planning permission has a vested right to be granted a site licence in respect of that land, unless
he is disqualified by reason of his having had a previous site licence revoked.
Section 5 is the crucial section. I must cite the first part of sub-s (1) in full:

A site licence issued by a local authority in respect of any land may be so issued subject to such conditions as the
authority may think it necessary or desirable to impose on the occupier of the land in the interests of persons dwelling
thereon in caravans, or of any other class of persons, or of the public at large; and in particular, but without prejudice to the
generality of the foregoing, a site licence may be issued subject to conditions.

There follow six different matters to which conditions may relate. They are not restricted (as they were under s 269 of the Public
Health Act, 1936) to matters relevant to public health and sanitation; they include matters relevant to lay-out and amenities. They
are, however, all conditions relating to the way in which the land is to be physically used and things to be done physically on the
land.
This enumeration of particular kinds of conditions which may be imposed on the occupier of the land is not to be read as
restrictive of the local authoritys general power to impose conditions under the earlier words in the subsection; but it has at least
this negative significance, that there is nothing in the enumerated kinds of conditions to indicate that Parliament intended to
authorise the local authority to impose conditions regulating the contractual relationship between the occupier and persons whom
he permitted to station caravans on the land save in respect of the way in which he permitted his licensees to use the land
physically for that purpose or to do things on the land in connexion with that use.
The local authority in this appeal rely on the general words:

such conditions as the authority may think it necessary or desirable to impose on the occupier of the land in the
interests of persons dwelling thereon in caravans.

They contend that this gives them power to impose any conditions which they bona fide consider to be desirable in the interests
of persons dwelling on the land in caravans, even though they do not relate to the physical use of the land or to things to be done
physically on it. The words in the interests, etc, are words which restrict the local authoritys discretion. They do not extend it;
and the issue comes back to the question: What are the implied limitations on the wide words such conditions as the local
authority may think it necessary or desirable 802to impose on the occupier of land in the context of conditions to be attached to
a licence authorising the use of land for a particular purpose?
Subsections (2) to (5) of s 5 throw no further light on the matter; but sub-s (6) authorises the minister to specify model
standards with respect to the lay-out of, and the provision of facilities, services and equipment for, caravan sites, and requires
the local authority to have regard to these standards in deciding what (if any) conditions to attach to a site licence, as must the
magistrates on an appeal against conditions under s 7. It is clear that the minister has no power to specify any standards with
respect to the terms of the contracts to be entered into between the occupier and persons whom he licenses to station caravans on
the land in accordance with any requirements of the site licence as regards lay-out and the provision of facilities, services or
equipment. If power to impose conditions relating to so wide and important a subject-matter was intended to be conferred on
local authorities by sub-s (1), it would be surprising that Parliament should have deprived the minister of power to give effective
guidance about it, whilst giving him power to give guidance about conditions relating to other more limited subject-matters.
Section 7 gives to an occupier a right of appeal to a magistrates court against a condition on the ground that it is unduly
burdensome. The local authority rely on this provision in support of their contention that the wide words of s 5(1) are not cut
down by necessary implication. Here, they say, is a safeguard against any oppressive exercise by a local authority of their wide
powers. But the existence of a right of appeal to some other court or authority cannot enlarge the powers of the local authority;
see Pilling v Abergele Urban District Council (already referred to), where there was a similar right of appeal. It is merely a
safeguard against the oppressive, though intra vires, use of those powers which Parliament has conferred on them. The right of
appeal, it is to be noted, must be exercised within twenty-eight days of the issue of the licence. After that time has elapsed, the
occupier has no remedy in respect of an intra vires condition attached to his site licence, however unduly burdensome it may be.
Section 8 empowers local authorities to alter the conditions attached to a site licence, and contains similar provisions for
appeal to the magistrates which is subject to similar time limits. But it gives the occupier no right to apply for a variation, and no
right of appeal against the local authoritys refusal to do so. Section 9 is the section which makes failure to comply with a
condition attached to a site licence a criminal offence by the occupier of the land, and provides in certain events for the
revocation of the licence. Here is the penal sanction which makes the conditions of a site licence subordinate legislation
analogous to that contained in bye-laws. There is, however, a distinction between an offence under this section for non-
compliance with a condition attached to a site licence, and an offence under s 1(1) for using land as a caravan site without a site
licence. The offence under s 1 can be committed by the occupier vicariously by permitting the land to be so used, and the
section (as already pointed out) by necessary implication limits the occupiers freedom of contract with licensees whom he has
authorised to use the land. A condition attached to a site licence under s 5, on the other hand, is imposed on the occupier
personally, and creates an absolute obligation with which he is personally bound to comply. Such a condition may be imposed or
varied at a time when licensees of the occupier are already using the land, and compliance with the condition may be inconsistent
with the terms of a contract into which he has entered with a licensee whom he has authorised to use the land. What then is the
effect of such a condition on the contractual rights and duties of the occupier and his licensees?
Both these matters are dealt with directly and specifically in s 12 of the Caravan Sites and Control of Development Act,
1960. Subsection (1) deals with consequences of s 1 of the Act on the contractual relationship between the occupier 803 and his
licensees by making it a statutory condition of every licence that the licence may be determined by the occupier if the licensee
does anything which would constitute an offence under s 1 if the licensee was the occupier. Subsection (2) deals with the
consequences of conditions imposed under s 5 on the contractual relations between the occupier and his licensees by conferring
on the occupier a statutory right to enter on the land and do anything reasonably required for the purpose of complying with any
condition attached to the site licence. In my opinion, assistance in construing the Act of 1960 is to be derived from the provisions
of s 12 in two respects. In the first place, it is some indication that, where Parliament intended to regulate the contractual
relations between the occupier and his licensees, it did so expressly, and that accordingly s 12 is exhaustive in this respect. In the
second place, the remedy which is given to the occupier to secure compliance with conditions attached to a site licence indicates
that the subject-matter of such conditions is limited to things to be done physically on the land itself.
Section 24 empowers local authorities themselves to provide and manage caravan sites. Subsection (5) empowers them to
acquire compulsorily (inter alia) land which is in use as a caravan site where it appears to them that it should in the interests of
the users of caravans be taken over by the local authority. If they think it desirable that sites for caravans should be available at
low charges, or with security of tenure, they can acquire existing caravan sites under this subsection on payment of compensation
to the persons interested in the land. It seems unlikely that Parliament, though it incorporated this express provision in the Act of
1960, intended by the general words of s 5(1) to empower the local authority to attain the same object without paying any
compensation by attaching to site licences conditions regulating the sums to be charged and other terms, not affecting the actual
use of the land, to be granted by the occupier to his licensees.
The provisions of Part 1 of the Act of 1960, to which I have specifically referred, are those which, in my view, throw the
most light on what is its immediate scope and object. I can find nothing in them, or in any of its other provisions, which contains
any hint of an intention on the part of Parliament to do more than to impose, by a system of site licensing, control over the
physical use of land for stationing caravans and over things to be done physically on that land in connexion with such use. This
control is imposed personally on the occupier of land; and to the extent that it is intended to affect the contractual relations
between him and persons whom he licenses to station caravans on land in his occupation, Parliament has in s 12 of the Act of
1960 made express provision as to what are to be the consequences on such contractual relations of the control so imposed on
him. In particular I can find no indication of any intention by Parliament to depart from its previous practice of treating the
control of contracts for the use of premises for human habitation as a separate legislative subject-matter to be dealt with by a
different series of statutes from those dealing with the control of the physical use to which land is to be put from the point of view
of public health or planning.
The wide general words of s 5(1) empowering local authorities to impose conditions on occupiers of land to be attached to a
site licence issued to them must, in my view, be read subject to the implied limitation that such conditions must fairly and
reasonably relate to the physical use to be made of the land for stationing caravans and things to be done physically on the land in
connexion with that use.
I accordingly turn to the disputed conditions:

(28) The site rents, which are to be inclusive of all services except electricity, shall be agreed with the council.

This condition is, in my view, ultra vires for more than one distinct reason. The first is that it does not fairly and reasonably
relate to the physical use to be made 804 of the land or anything to be done physically on the land. The use of the land is exactly
the same whether the rent is agreed at one figure or another. The condition is simply an attempt to impose rent control in respect
of a type of living accommodation to which Parliament, in statutes dealing with that subject-matter, has not thought fit to apply it.
It is a condition purporting to alter the general principles of the law in a respect which is outside the subject-matter of the Act.
The second ground of invalidity is that it does not itself fix the rents which are to be charged; it purports to empower the
defendants to determine the rents at some later date, or possibly to vary their determination from time to time. But it is implicit in
the statutory provisions for appeal against conditions within a limited time after the issue of the site licence that any power of this
kind should be exercised in the condition itself, and not subsequently or in some other way, for this would deprive the occupier of
any remedy if the defendants exercised their power to fix rents in a way which was unduly burdensome. Thirdly, the retention of
an unlimited power to decide what rents the defendants will agree without any right of appeal gives the defendants power for
practical purposes to prevent the occupier from using the land as a caravan site, although he is entitled, as of statutory right, to a
licence authorising him to do so.
Condition (29) is as follows:

Security of tenure, subject to similar conditions appertaining to a statutory tenancy of a dwelling-house under the Rent
Acts, shall be granted to all caravan occupiers.

This condition, whose very terms indicate that its subject-matter is not the physical use to be made of the land, but a different
subject-matter which Parliament has dealt with in the Rent Acts, is in my view ultra vires on the general ground already
discussed. Furthermore, since the security of tenure appertaining to a statutory tenancy under the Rent Acts is based on the
control of and the exercise of a judicial discretion by the county court, which would have no jurisdiction to deal with a caravan, it
is difficult to ascribe any intelligible meaning to the condition. I do not, however, find it necessary to decide whether it would be
void for uncertainty if it were not ultra vires on the more general ground.
Condition (30) is as follows:

Site rules shall be restricted to those items normally covered by a tenancy agreement, and necessary for the good
administration of the site.

A condition relating to site rules, which is not a term of art, is capable of being construed as fairly and reasonably relating to
the physical use to be made of the land and things to be done physically on the land in connexion with that use. A condition as to
the contents of site rules is not, therefore, ultra vires. But condition (30) is, in my view, invalid on a different ground, viz, that it
is so uncertain that it is not possible to ascribe any intelligible meaning to the words those items normally covered by a tenancy
agreement. I agree with Danckwerts LJ (whose experience in this field is so much greater than mine), that it is quite impossible
to say what is and what is not normally covered by a tenancy agreement.
Condition (31) is as follows:

There shall be no restriction on caravan occupiers as to from whom they purchase the commodities that they require,
or on the callers that they may have for purposes of trade or pleasure.

The first part of this condition about purchasing commodities does not seem to me to be fairly and reasonably related to the
physical use to be made of the land or things to be done physically on the land in connexion with that use. It relates to activities
which do not take place on the land at all. It is, in my view, ultra vires. The latter part does relate to what is to take place on the
land, and if it stood alone it might be intra vires. But in the context in which it appears it is, I think, intended to be ancillary to
the first part, and inserted for the purpose of preventing 805 the occupier from evading the effect of the first part of the condition
by indirect means. One may sympathise with the evident desire of the local authority to amend, as between the occupier and his
licensees, the general law about covenants in restraint of trade; but this is a consideration which ought not to influence them in
imposing conditions to be attached to a site licence; see Pilling v Abergele UDC, already referred to. Although my mind has
wavered, I do not think that latter part of the condition is severable from the first part, and I would agree with Danckwerts LJ in
declaring the whole of this condition ultra vires.
Condition (32) is as follows:

No premium is to be charged for any caravan occupier entering the site, or any restriction as to the make or supplier of
the caravan. Existing caravan occupiers purchasing new caravans shall not be compelled in any way to purchase a
particular make of caravan or from a particular dealer.

The prohibition on the charging of premiums is, in my view, invalid on the same general grounds as the conditions as to rent and
terms of tenancy. So also is the prohibition of restrictions on the supplier from whom a caravan is to be bought. The fact that a
caravan of a particular type has been bought from one supplier rather than another does not affect in any way the physical use to
be made of the land or things to be done on it. There remains the prohibition of restrictions on the make of caravan to be used on
the site. If this part of the condition stood alone, it might be construed as relating to the type of caravan which may be stationed
on the land, and thus fairly and reasonably to relate to the physical use to be made of the land. But the type of caravan to be
stationed on the land is specifically dealt with in a separate condition, condition (7), in the group of conditions which are headed
Site, lay-out, roads, etc. This part of condition (32) seems to me to be inseverable from the remainder of the condition, and to be
directed to the same objects. I would declare the whole of condition (32) ultra vires.
Condition (33) is as follows:

There shall be no restriction imposed on the caravan occupiers of the formation of, or membership of, any form of
tenants association, political party or other organisations.

This condition is ultra vires. However praiseworthy it may be to attempt to preserve the fundamental human rights of
association, a condition attached to a site licence is, for the reasons already indicated, no place to do it.
For my part I would declare the whole of conditions (28), (29), (30) and (33) ultra vires and void; condition (31) void as to
that part of it which prohibits the imposition of restrictions as to from whom caravan occupiers shall purchase commodities, and
condition (32) void as to that part of it which prohibits the charging of premiums, and also as to that part of it which prohibits the
imposition of restrictions (i) as to the suppliers of a caravan, and (ii) as to compelling existing caravan occupiers to purchase a
caravan from a particular dealer. I, like Danckwerts LJ would declare the whole of the disputed conditions ultra vires and void.

Appeal allowed. Leave to appeal to the House of Lords granted.

Solicitors: Wilkinson, Howlett & Moorhouse (for the plaintiffs); Champion & Co (for the defendants).

F A Amies Esq Barrister.


806

[1963] 2 All ER 807

Re British Basic Slag Ltds Agreements


COMPETITION

COURT OF APPEAL
WILLMER, DANCKWERTS AND DIPLOCK LJJ
13, 14, 15, 27 MAY 1963

Restrictive Trade Practices Agreement Registration Negative obligation implied from positive clause Agreement
restricting goods produced, supplied or acquired Marketing company owned by eight steel manufacturers Agreements with
own members Undertaking by marketing company to acquire members basic slag and apportion sales fairly between them
No obligation on marketing company to acquire more basic slag than it could dispose of Implied restriction on purchase from
other members Restrictive Trade Practices Act, 1956 (4 & 5 Eliz 2 c 68), s 6(1)(c).

Restrictive Trade Practices Agreement Registration Arrangement Marketing company owned by eight steel
manufacturers Separate similar agreements by manufacturers with company for marketing through company Arrangement
between steel manufacturers presumed from communications before the separate agreements were sealed Restrictive Trade
Practices Act, 1956 (4 & 5 Eliz 2 c 68), s 6(1), (3).

Eight steel manufacturing companies were shareholders of B Ltd; each appointed a director to its board and each entered into a
ten years agreement in the same form with B Ltd. These agreements were made on 16 March 1954. The renewing of the
agreements previously subsisting had been discussed at a number of board meetings of B Ltd beginning in July, 1953. Drafts had
been prepared and circulated by B Ltd and, each company having executed its agreement, the agreements were sealed on behalf
of B Ltd pursuant to a resolution of a board meeting of B Ltd attended by five of the nominated directors. Under its agreement
each steel company undertook (by cl 1) to sell to B Ltd the whole of the basic slag which the vendor company produced, owned
or controlled, provided that nothing therein contained should impose on B Ltd any obligation to purchase any basic slag of which
it could not dispose. Each steel company undertook (by cl 5) not to sell basic slag to any other person without B Ltds consent.
B Ltd undertook (by cl 16) to use its best endeavours to dispose of the whole of the basic slag that each steel company produced
and to allocate and apportion all deliveries of basic slag between the companies and other companies entering into similar
arrangements in an equitable and reasonable manner. Each agreement was indorsed with a memorandum listing companies
which, in fact, it was intended should execute similar agreements. It was conceded that by cl 5 each steel company accepted a
restriction within s 6(1) of the Restrictive Trade Practices Act, 1956.

Held (i) though cl 16 of each agreement was positive in terms, it impliedly imposed a negative obligation, viz, a restriction that
B Ltd would not buy from members, other than the particular vendor steel company, such a quantity of basic slag as would
disable B Ltd from disposing of the vendors fair proportion of the total; accordingly, since B Ltd accepted such a restriction
(within s 6(1)(c) or (e) of the Restrictive Trade Practices Act, 1956) and it was conceded that by cl 5 the vendor steel company
accepted a restriction, the conditions of s 6(1) were satisfied and the agreements of 1954 were subject to registration under s 9 of
the Act of 1956 (see p 813, letter b, p 817, p 817, letters h and i, and p 816, letter g, post).
(ii) the word arrangement in s 6(3) of the Act of 1956 was to be construed in its ordinary sense, and, where each of two
parties by communicating with the other intentionally aroused in the other an expectation that he would act in a particular way,
each incurred a moral obligation so to act, and there would be an arrangement, viz, something whereby the parties to it accepted
mutual rights and duties, within the meaning of the 807 word arrangement in s 6(3); in the present case there was such an
arrangement between the steel companies to enter into the agreements of 1954, in view of the communications between nominees
of the companies at board meetings of B Ltd prior to the sealing of the agreements, and accordingly the particulars of the
arrangement should not be deleted from the register under the Act of 1956 (see p 814, letter c, p 816, letter d, and p 819, letters e
and f, post).
Dictum of Upjohn J in Re Austin Motor Co Ltds Agreements ([1957] 3 All ER at p 69) considered and applied.
Decision of Cross J ([1962] 3 All ER 247) affirmed.

Notes
As to agreements to which Part 1 of the Restrictive Trade Practices Act, 1956, applies, see 38 Halsburys Laws (3rd Edn) 98, 99,
paras 126, 127; and as to the power of the High Court to declare whether an agreement is registrable, see ibid, 101, para 130.
For the Restrictive Trade Practices Act, 1956, s 6, see 36 Halsburys Statutes (2nd Edn) 937.

Cases referred to in judgments


A-G v Seccombe [1911] 2 KB 688, 80 LJKB 913, 105 LT 18, 42 Digest 737, 1607.
Austin Motor Co, Ltds Agreements, Re [1957] 3 All ER 62, LR 1 RP 6, [1958] Ch 61, 3rd Digest Supp.
Blanket Manufacturers Assocns Agreement, Re [1959] 2 All ER 630, LR 1 RP 271, 3rd Digest Supp.Balfour v Balfour [1918
19] All ER Rep 860, [1919] 2 KB 571, Rose & Frank Co v J R Crompton & Bros Ltd [1924] All ER Rep 245, [1925] AC
445, Cross-land v Hawkins [1961] 2 All ER 812, [1961] Ch 537, Re Automatic Telephone & Electric Cos Application
[1962] 2 All ER 207, LR 3 RP 98.

Appeals
The first applicants (British Basic Slag Ltd) appealed against an order of Cross J made on 6 July 1962, whereby an originating
summons (1960 B No 4358) issued by the first applicants was dismissed and it was declared that each of the agreements made by
the first applicants separately with eight steel manufacturing companies (seven of them the second applicants) was one to which
the Restrictive Trade Practices Act, 1956, applied and that it was subject to registration under s 9 of the Act. The grounds of
appeal were as follows: (i) that the judge was wrong in holding that under cl 16 of the agreements the first applicants accepted
any restriction in respect of the quantities of goods (not being goods supplied under the agreement) to be acquired by them; (ii)
that the judge was wrong in holding that under cl 16 of the agreements the first applicants accepted any restriction in respect of
the persons or classes of persons from whom goods were to be acquired by them; (iii) that the judge was wrong in holding that
under the agreements any restriction within s 6(1) of the Restrictive Trade Practices Act, 1956, was accepted by the first
applicants otherwise than under terms of which no account might be taken by virtue of s 7(2) of the Act.
The second applicants (seven steel manufacturing companies) appealed against an order of Cross J made on 6 July 1962,
dismissing an originating summons (1960 C No 3911) issued by the second applicants, and sought an order that the register
maintained under s 1(2) of the Restrictive Trade Practices Act, 1956, should be rectified pursuant to s 13(1) by the removal of all
the particulars registered under reference R 1024, being a memorandum a of an 808 arrangement between the steel manufacturing
companies leading to the marketing of basic slag through the Basic Slag, Ltd. The grounds of appeal were as follows: (i) that the
judge misdirected himself in holding that it was sufficient to constitute an arrangement within the meaning of s 6 of the Act that
the parties to it should have communicated with one another in some way, and that as a result of the communication each should
have intentionally aroused in the other an expectation that he would act in a certain way; (ii) that the judge should have held that
the term arrangement in s 6 of the Act connoted an arrangement under which the parties accepted definite mutual rights and
obligations, binding on them at least in honour; and accordingly there could be no arrangement within s 6 of the Act under which
restrictions were accepted by two or more parties in the case unless each applicant had, as against the other applicants, a right to
the execution by each other applicant of a bilateral agreement with the first applicant (British Basic Slag Ltd), containing
restrictions on such applicant, and assumed a corresponding obligation towards the other applicants to execute such agreement;
(iii) that on the evidence before the judge each of the second applicants entered into its bilateral agreement with the first
applicants (British Basic Slag Ltd) as a matter of independent commercial judgment and not as the result of any agreement with
any other applicant, and the execution of its bilateral agreement was not conditional on the execution of a similar agreement by
any other applicant; and on the evidence the judge should have determined that there was no such arrangement as was set out in
the particulars registered under Reference R 1024 in the register maintained under s 1(2) of the Restrictive Trade Practices Act,
1956, and should have ordered the register to be rectified by the removal of the particulars.
________________________________________
a The relevant terms of the memorandum were:(1) Arrangement made on or about 22 July 1953, between the undermentioned steel
manufacturers for the common marketing of basic slag through British Basic Slag Ltd upon the terms set out in the agreement annexed,
evidenced by minutes of board meetings of British Basic Slag Ltd held on 22 July 1953, 15 September 1953, 12 November 1953, and 14
January 1954, copies of which are also annexed. (2) Each of the said steel manufacturers executed an agreement in the said form on the
dates shown.

There followed the dates (16 March 1954, as regards seven companies, and 19 July 1955, as regards one) and the names of
the steel manufacturing companies. The agreement referred to was an agreement of 16 March 1954.
By a respondents notice dated 23 October 1962, the Registrar of Restrictive Trading Agreements gave notice of an
additional ground on which he would contend that the order of Cross J dated 6 July 1962, should be affirmed, viz, that, once the
eight agreements dated 16 March 1954, had been executed, an arrangement between the parties to those agreements come into
existence under which restrictions were accepted by those parties in respect of matters specified in s 6(1) of the Act of 1956.

Sir Lionel Heald QC and D A M Kemp for the applicants.


Arthur Bagnall QC and J P F E Warner for the Registrar of Restrictive Trading Agreements.

Cur adv vult

27 May 1963. The following judgments were delivered.

WILLMER LJ. This is an appeal from a judgment of Cross J given on 6 July 1962, relating to two originating summonses
taken out under s 13 of the Restrictive Trade Practices Act, 1956. Both summonses relate to certain agreements made between
various companies carrying on business as steel manufacturers and a company called British Basic Slag Ltd (hereinafter referred
to as Basic) for the sale of fertilisers including basic slag by the former to the latter. By the first summons, 1960 B No 4358,
Basic seeks a declaration, pursuant to s 13(2) of the Act, that the agreements in question, all of which are in the same form, are
not subject to registration under s 9(1). By the second summons, 1960 C No 3911, the steel manufacturing companies apply
under s 13(1) of the Act for an order that the register be rectified by deletion therefrom of particulars of these agreements
previously registered on their behalf. In both cases the judge decided adversely to the applicants. In the first case 809 he held
that the agreements between the various steel manufacturing companies and Basic were agreements falling within s 6(1) of the
Act, and as such were subject to registration under s 9. In the second case he held that the particulars of the agreements could not
be deleted from the register, because the agreements had been entered into in pursuance of an arrangement between the
respective steel manufacturing companies within the menaing of s 6(3) of the Act.
It will be convenient to refer at once to the relevant provisions of s 6 of the Restrictive Trade Practices Act, 1956, which are
as follows:

6.(1) Subject to the provisions of the two next following sections, this Part of this Act applies to any agreement
between two or more persons carrying on business within the United Kingdom in the production or supply of goods, or in
the application to goods of any process of manufacture, whether with or without other parties, being an agreement under
which restrictions are accepted by two or more parties in respect of the following matters, that is to say:
(a) the prices to be charged, quoted or paid for goods supplied, offered or acquired, or for the application of any process
of manufacture to goods:
(b) the terms or conditions on or subject to which goods are to be supplied or acquired or any such process is to be
applied to goods;
(c) the quantities or descriptions of goods to be produced, supplied or acquired;
(d) the processes of manufacture to be applied to any goods, or the quantities or descriptions of goods to which any
such process is to be applied; or
(e) the persons or classes of persons to, for or from whom, or the areas or places in or from which, goods are to be
supplied or acquired, or any such process applied.

(3) In this Part of this Act agreement includes any agreement or arrangement, whether or not it is or is intended to be
enforceable (apart from any provision of this Act) by legal proceedings, and references in this Part of the Act to restrictions
accepted under an agreement shall be construed accordingly; and restriction includes any negative obligation, whether
express or implied and whether absolute or not.

The facts giving rise to the making of the agreements and the registration of the particulars were very fully stated by the judge,
and are conveniently set out in the reports of the case. It would serve no useful purpose for me to set out all the details again, and
I therefore content myself with a brief summary.
Basic slag is a by-product manufactured from waste material left over from the manufacture of steel. When ground to a fine
powder it is a valuable fertiliser for agricultural purposes, and has been used as such for many years. Before the first world war a
number of steel manufacturing companies had a contract with a German firm for the disposal of the basic slag resulting from
their manufacture of steel. In pursuance of this contract the German firm had installed plant at the various companies steel
works for grinding the slag. During the war the German firms business in this country was ordered to be wound up pursuant to
the Trading with the Enemy Act, 1916. The steel manufacturing companies then conceived the idea of themselves marketing
their basic slag in common, and to this end joined in forming a company (of which Basic is the successor) for the purpose of
taking over the German firms business in this country. The steel manufacturing companies who were parties to the scheme
became shareholders in the new company in proportion to the amount of their average deliveries of basic slag during the last
three years before the war. Each of the steel manufacturing companies appointed a director to the board of the new company; and
each purchased the plant which had been installed by the German firm at 810 its respective works. Once the new company was
formed each of the steel manufacturing companies entered into a ten years agreement with it for the sale to the new company of
the basic slag produced at its works. Substantially the same arrangement has continued till the present day, the agreements being
renewed from time to time as they expired. Over the years there have been changes in the membership of Basic and its
predecessor company, some of the original steel manufacturing companies having dropped out of the scheme and others having
joined in.
The agreements with which we are now concerned, or at least most of them, were executed on 16 March 1954, and were
expressed to continue in operation till 30 September 1964. The question of renewing the previously current agreements, and the
form which the new agreements should take, was discussed at a number of board meetings of Basic beginning on 22 July 1953.
The relevant extracts from the minutes of these meetings are fully set out in the judges judgment, and formed part of the
particulars entered on the register by Basic on behalf of the member companies on the coming into force of the Restrictive Trade
Practices Act, 1956. It will be necessary to refer to some of these minutes hereafter when I come to consider the question raised
by the second summons, viz, whether there was an arrangement between the steel manufacturing companies within the
meaning of s 6(3) of the Act.
I propose, however, to consider first the question whether the agreement made between each of the steel manufacturing
companies as vendor and Basic as purchaser is an agreement to which s 6(1) of the Act applies. This depends on whether on
the true construction of the agreement the parties thereto accepted restrictions in respect of any of the matters specified in paras
(a) to (e) of that subsection. Clause 1 of the agreement, omitting immaterial words, provided:

The vendor shall sell and the purchaser shall purchase subject to the terms and conditions hereof the whole of the
fertilisers (as hereinafter defined) which the vendor shall produce own or control Provided that nothing herein
contained shall be deemed to impose on the purchaser any obligation to purchase from the vendor any fertilisers of which it
cannot dispose.

By cl 2 fertilisers were defined to mean, amongst other things, basic slag ground or otherwise processed as a fertiliser.
Clause 3 entitled the vendor to use or sell any portion of its make of fertilisers for the manufacture of steel or iron or to dispose of
any fertilisers which the purchaser should refuse to take. By cl 4 the vendor was not to be bound to sell to the purchaser any
specific quantity provided that it sold to the purchaser the whole of the fertilisers which it produced, owned or controlled. Clause
5, under which the vendor clearly accepted a relevant restriction, provided as follows:

The vendor shall not without the written consent of the purchaser (except as aforesaid) sell or dispose of any basic slag
ground or unground capable of use as a fertiliser to any other person, firm, company, or body and will refer to the purchaser
all inquiries which it may receive for fertilisers.

Clauses 6, 7 and 8 made provision for the method of delivery of fertilisers sold in pursuance of the agreement. In substance it
was provided that delivery should be made direct to the consignees named by the purchaser, ie, to the buyers to whom the
purchaser disposed of the goods. There was to be no delivery to the purchaser, who would never handle the goods itself; in the
result a sale would only take place when, and in so far as, the purchaser succeeded in disposing of the goods. Clause 13 regulated
the price to be paid by the purchaser to the vendor, and provided that this was to be based on the realised price obtained by the
purchaser on resale of the goods, less certain deductions. Clause 16, which is the only clause whereby it is suggested that the
purchaser accepted any relevant restriction, was in the following terms:
811

The purchaser shall use its best endeavours to dispose of the whole of the fertilisers which the vendor shall produce
and subject to any government regulations to allocate and apportion in an equitable and reasonable manner between the
vendor and other vendors who have entered into similar agreements all deliveries of the fertilisers.

Lastly, it is necessary to refer to the fact that a memorandum was indorsed on the agreement in the following terms:

The following is a list of the companies which have executed agreements in this form which are in force at the date
hereof.

A list of other steel manufacturing companies executing similar agreements was then set out. It must be accepted that this
memorandum cannot be taken as strictly accurate; it must be regarded as a proleptic statement of the companies which it was
intended should execute similar agreements.
It being conceded that under cl 5 the vendor accepted a restriction falling within s 6(1) of the Act, the question is whether
any relevant restriction was accepted by the purchaser. The judge construed cl 16 as imposing a restriction on the right of Basic
to buy such quantities of fertilisers as it likes from persons other than the vendor. This he held to be a restriction in respect of the
quantities of goods (not being goods supplied under the agreement) to be acquired by Basic, ie, a restriction falling within para
(c) of s 6(1). He expressed his conclusion in this way ([1962] 3 All ER at p 253; LR 3 RP at p 194):

The right which Basic would have apart from the agreement to buy fertilisers from any source is restricted by the
agreement in that (a) it cannot buy from non-members and (b) it cannot buy from members other than the vendor in
question, such a quantity of fertilisers as will disable it from buying from the vendor his fair proportion of the total
purchases.

The argument for the applicants has been that the judge erred in construing cl 16 as imposing a restriction on Basic from
buying what quantities of fertilisers it likes to buy. Clause 16, it is said, is concerned not with buying but with disposing of the
fertilisers acquired. It may, it is true, have the result of making it difficult for Basic to dispose of fertilisers acquired from other
sources without breaking its contract with the vendor, but it is claimed that this would not be a restriction within s 6(1) of the Act.
In this connexion reliance was placed on the decision of this court in Re Blanket Manufacturers Agreement. In that case a
trade association made a recommendation accepted by its members to the effect that no manufacturer should agree to the
breaking of any contract by reduction of price or other procedure. This court held that the recommendation affected only the
rights and obligations of the members after they had entered into their contracts, and was not a restriction accepted under any
agreement in respect of prices to be charged or any of the other matters set out in s 6(1). So here it has been contended that cl 16
cannot be construed as imposing any restriction on Basic in respect of quantities of goods to be acquired. I confess that I have
not found the argument for the applicants altogether easy to follow in this respect. I derive some comfort from the fact that my
difficulty appears to be one that is shared both with the judge and with counsel for the registrar. But it seems to me that the
argument, so far as I have been able to follow it, applies only in relation to the right of Basic to buy fertilisers from non-members.
Counsel for the registrar has indeed conceded that he cannot support the view expressed by the judge in this respect. He himself
attempted to formulate an alternative restriction which he contended was imposed by cl 16 in relation to Basics purchases from
non-members. But it did not seem to me that this was any happier than the restriction formulated by the judge.
812
Assuming, however, that it is not possible to extract from cl 16 any restriction on Basic in relation to purchases from non-
members, the position is quite different in relation to purchases from members other than the vendor. For, as pointed out already,
purchases under this and similar agreements take place only as and when Basic is able to dispose of the fertilisers offered. Basic
cannot buy from members who have entered into similar agreements unless it disposes of what is bought. Thus it is true to say
that it is restricted by the agreement from buying from members, other than the particular vendor, such a quantity as will disable
it from disposing of the vendors fair proportion of the total. This seems to me to amount plainly to a restriction accepted under
the agreement in respect of quantities to be acquired within para (c), and possibly also in respect of the persons from whom the
goods are to be acquired within para (e) of s 6(1). Accordingly, although I do not wholly accept his reasoning, I do not find it
possible to say that the judge came to a wrong conclusion on this part of the case.
I pass, therefore, to consider the question raised by the second summons, viz, whether the respective member companies
entered into their agreements with Basic in pursuance of an arrangement between themselves within the meaning of s 6(3) of
the Act. The judge concluded that they did, and in those circumstances he held that the particulars were rightly entered on the
register and should not be deleted therefrom. Having pointed out that the subsection included an arrangement, whether or not it
was or was intended to be enforceable by legal proceedings, he expressed his view as follows ([1962] 3 All ER at p 255; LR 3 RP
at p 196):

As I see it all that is required to constitute an arrangement not enforceable in law is that the parties to it shall have
communicated with one another in some way and that as a result of the communication each has intentionally aroused in
the other an expectation that he will act in a certain way. If that is right then as it seems to me the member companies made
an arrangement that they would each of them execute the relevant agreement with Basic.

This conclusion is attacked as erroneous both in point of law in point of fact. As to the former, it is said that in order to constitute
an arrangement something more is required than that one party should intentionally arouse in the other an expectation that he
will act in a certain way. In order to constitute an arrangement it is contended that there must be mutuality in the acceptance of
rights and obligations. In this connexion reliance is placed on Re Austin Motor Co Ltds Agreements, where Upjohn J said
([1957] 3 All ER at p 69; LR 1 RP at p 19; [1958] Ch at p 74):

Whether enforceable at law or not, it seems to me that an arrangement must at least connote an arrangement whereby
the parties to it accept mutual rights and obligations.

As to the facts, reliance is placed on the uncontradicted evidence given by representatives of the various companies
nominated as members of the board of Basic, the effect of which may be summarised as follows. The decision of each company
to execute its agreement with Basic was based, not on any prior agreement or consultation with any other company, but solely on
the consideration that commercial interests of the company would best be served by continuing to dispose of its basic slag to
Basic. This evidence was never challenged in cross-examination, and must therefore, it is said, be accepted as conclusive. All
that was admitted by the witnesses was that it was never in the mind of any of the representatives of the member companies that
any other member company would enter into an agreement with Basic in a different form.
I find myself unable to accept either branch of the argument for the applicants. I do not think that the construction which
they seek to put on the word arrangement gives effect to the plain meaning of s 6(3) of the Restrictive Trade 813 Practices Act,
1956. Nor do I think that the answers of the witnesses, on which so much reliance is placed, present a realistic view of the facts
in the light of the totality of the evidence as to the circumstances in which the agreements between Basic and the member
companies came to be executed.
To deal first with the meaning of the subsection, I think it is highly significant that Parliament did not see fit to include any
definition of arrangement. I infer from this that it was intended that the word should be construed in its ordinary or popular
sense. Though it may not be easy to put it into words, everybody knows what is meant by an arrangement between two or more
parties. If the arrangement is intended to be enforceable by legal proceedings, as in the case where it is made for good
consideration, it may no doubt properly be described as an agreement. But the statute clearly contemplates that there may be
arrangements which are not enforceable by legal proceedings, but which create only moral obligations or obligations binding in
honour. This seems to me to be entirely consistent with the dictum of Upjohn J ([1957] 3 All ER at p 69; LR 1 RP at p 19; [1958]
Ch at p 74), to which I have already referred. Nor do I consider that there is any inconsistency between that and the view
expressed by the judge in the present case. For, when each of two or more parties intentionally arouses in the others an
expectation that he will act in a certain way, it seems to me that he incurs at least a moral obligation to do so. An arrangement as
so defined is therefore something whereby the parties to it accept mutual rights and obligations.
As to the facts of the present case, I agree with the submission made on behalf of the registrar that it is wholly unrealistic to
say that the companies never communicated or consulted with each other about the making of their agreements with Basic. This
may be true enough in the sense that they did not write letters to each other about it. But each company had its nominee as a
member of the board of Basic, and it is clear that these nominees not only had, but used, the opportunity to discuss with each
other what their companies were going to do in relation to renewing their agreements with Basic. In this connexion it appears to
me that the minutes of the board meetings of Basic which led up to the making of the present agreements are themselves highly
significant. On 22 July 1953, it is recorded that the

directors present indicated that their companies would be willing to renew membership for a further period of ten
years.

In so indicating the directors were speaking for their respective companies, and in signifying their companies willingness to
renew membership they can only have meant that they were willing to continue for a further period the arrangement already
existing. On 14 January 1954, a first draft of the proposed new agreement was before the board, and it is recorded that

various amendments were requested and subject to the approval of these amendments by the directors individually the
board gave their approval

The contrast between the collective approval of the board and the individual approval by the directors is illuminating; for in
giving their individual approval the directors can only have been speaking for their respective companies. This record of the way
in which the agreements of 1954 were negotiated gives point to the admission made in cross-examination by Mr Miller, the
secretary of Basic, to the effect that Basic is essentially a common selling organisation for its members production. The plain
meaning of this, it may be thought, is that by setting up and maintaining the existence of Basic, and by entering into contracts
with it in identical or similar form, the companies were giving effect to an arrangement between themselves for a common selling
organisation for their production.
The same conclusion follows in my view from a consideration of the terms of the agreement itself, especially cl 16, the
effect of which has already been considered in relation to the first question on this appeal. This clause, by its reference 814 to
other vendors who have entered into similar agreements, clearly contemplates that there must have been some arrangement for
the execution of similar agreements by other vendors, ie, by the other companies who were parties to the scheme. Unless there
were such an arrangement there could be no point in the individual companies agreeing that Basic should allocate and apportion
the deliveries of fertilisers in an equitable and reasonable manner as between themselves and other vendors. But, if any doubt
remains, the memorandum indorsed on the agreement must be, I should have thought, conclusive. For, as already indicated, this
really amounts to a proleptic statement as to the companies which have arranged to execute agreements in similar terms.
In these circumstances, construing the word arrangement in its ordinary sense, and giving effect to the realities of the
situation, I think it is impossible to avoid the conclusion that the various companies here entered into their agreements with Basic
in pursuance of an arrangement between themselves to do so. If that be right, it is accepted that in doing so they accepted
restrictions in respect of matters falling within s 6(1) of the Act. It follows that, in my judgment, on this point of the case also the
judge came to a right conclusion, and I would accordingly dismiss the appeal.

DANCKWERTS LJ. Section 6(1) of the Restrictive Trade Practices Act, 1956 (Part 1), provides that:

Subject to the provisions of the two next following sections, this Part of this Act applies to any agreement between
two or more persons carrying on business within the United Kingdom in the production or supply of goods, or on the
application to goods of any process of manufacture, whether with or without other parties, being an agreement under which
restrictions are accepted by two or more parties

in respect of certain matters which are then set out in the subsection. By s 9(1) (subject to the provisions of that section) every
agreement to which Part 1 of the Act applies is to be subject to registration thereunder.
The first question, therefore, is whether in the present case there is an agreement or agreements to which Part 1 of the
Act applies. Agreement is defined in s 6(3):

(3) In this Part of this Act agreement includes any agreement or arrangement, whether or not it is or is intended to be
enforceable (apart from any provision in this Act) by legal proceedings, and references in this Part of this Act to restrictions
accepted under an agreement shall be construed accordingly; and restriction includes any negative obligation, whether
express or implied, and whether absolute or not.

It is easy to see why so wide a description was included in the Act, with the object of rendering evasion of the purposes of the Act
impossible or at any rate difficult, even with skilled advice. After all, long experience of the ingenuity exercised in respect of the
Revenue Acts had shown the way. Hence the relevancy of such cases as A-G v Seccombe (an estate duty case, in which it was
held that in the words benefit to him by contract or otherwise the words or otherwise indicated something ejusdem generis
with contract, ie, an enforceable arrangement); and the observations in that case by Hamilton J (afterwards Lord Sumner), are of
great interest in regard to the problem before this court. He said ([1911] 2 KB at p 701):

In the present case, I am satisfied that there was no contract that in consideration of the gift the donor should be
permitted to reside and be maintained by the donee, in part or in whole, in the house; nor was there any understanding to
that effect in the sense of an arrangement, or as it is called an honourable understanding, which is not legally enforceable.

In the present case, of course, it is clear that the arrangement to which the 815 Act applies is not limited to enforceable
contracts, and extends to arrangements which have no legal force, because the Act expressly says so. Criticisms which were
made in respect of the observations of Upjohn J in Re Austin Motor Cos Agreements ([1957] 3 All ER at p 69; LR 1 RP at p 19;
[1958] Ch at p 74):

Whether enforceable at law or not, it seems to me that an arrangement must at least connote an arrangement whereby
the parties to it accept mutual rights and obligations,

seem to me to be mistaken. It is implicit in the judges statement that mutual rights and obligations include rights and
obligations which are not enforceable by any legal process. However, the provisions of the Restrictive Trade Practices Act, 1956,
in this respect, seem to me to be calculated to drive any accurately minded lawyer to despair. Once the ascertainable ambit of
arrangements, rights or obligations, which are legally enforceable is left behind, one flounders in a morass of inexactitudes.
There is no path or guide which can be followed except that it must be an arrangement between two or more persons carrying on
business in the United Kingdom.
What is the limit of looseness of association which may be tolerated in the application of the Act? It may be that it is
impossible to lay down any principle for application and that each case must be decided on the particular circumstances of the
case; and heaven help the lawyer who has to advise a clienta form of assistance which I gather has not always been at hand in
the present case. In my opinion, the judge reached the right conclusion, and there was an arrangement between the companies
concerned in the present case. A company exists and was formed originally for the purpose of providing a common marketing
body for the basic slag produced by the companies. The scheme was intended to operate for the benefit of the steel companies in
order to enable them to dispose of the basic slag. I reject the argument that the history of the company is not relevant for the
present purpose. For the purposes of the scheme the companies became members of British Basic Slag Ltd and were represented
by their directors on the board of that company. The scheme would, I think, have collapsed if the individual companies had gone
their different ways and entered into varying agreements. In fact the companies, whose names are contained in a memorandum
indorsed on the agreements, all executed similar agreements and each knew that the others had done so. This was done, as it
seems to me, clearly under an arrangement between the companies within the meaning of s 6 of the Act.
The question then remains whether under the arrangement restrictions were accepted by two or more parties in respect of the
matters set out in s 6(1) of the Act. The question is made more difficult by reason of the peculiar and almost contradictory
provisions contained in various sections of the Act. I must confess that I have found considerable doubt and difficulty on this part
of the case, but I am not prepared to differ from the result reached by the judge or the conclusions of my brethren on this part of
the case. I agree that the appeals should be dismissed.

DIPLOCK LJ. In the process of steel-making there are produced as by-products basic slag and mineral phosphates. These, after
undergoing a further process of grinding, can be used either alone or admixed as fertilisers. The seven steel companies who are
appellants in the second appeal (whom I will call the members) and the Steel Company of Wales all sell the whole of their
production of these fertilisers to British Basic Slag, Ltd (whom I will call Basic) under contracts in similar terms to that
between Colvilles Ltd (whom I will call Colvilles) and Basic. This contract (which I will call the vertical contract) is the
subject-matter of the first appeal in which Basic is the appellant. The members and another steel company, which at one time
sold all its production of fertilisers to Basic but no longer does so, are the sole shareholders in Basic and each of the members in
practice nominates one of its own directors to be a member 816 Basics board. The facts material to the two appeals as found by
Cross J are set out in the reports of this case below and I need not repeat them.

The First Appeal: the Vertical Contract.


The question in the first appeal is whether the vertical contract is one to which Part 1 of the Restrictive Trade Practices Act,
1956, applies and so subject to registration under s 9. The provisions of the Act which are directly relevant are s 6(1), (2) and (3);
s 7(2) and s 8(3). The vertical contract is a bipartite agreement intended to create legally enforceable rights and obligations. The
effect of s 6(1) and s 7(2) is that, in order to see whether it is an agreement to which Part 1 of the Act applies, one must look at
the vertical contract to see whether it imposes on each party in respect of any of the matters enumerated in s 6(1) a restriction
which is not a restriction relating exclusively to the goods supplied in pursuance of the vertical contract. If it does, then one must
see whether all of such restrictions are of the kind described in s 8(3)(a) or (b). If all are, Part 1 of the Act does not apply to the
vertical contract but, if any one of them is not, then Part 1 of the Act does apply to it. By s 6(2) a restriction includes any
negative obligation whether express or implied and whether absolute or not.
The vertical contract is one for the sale by Colvilles to Basic of the whole of Colvilles production of fertilisers during a
period of ten years subject to the important limitation contained in the proviso to cl 1 that Basic is under no obligation to
purchase from Colvilles any quantity of fertiliser of which Basic cannot dispose. Clauses 7 and 8 dealing with delivery of
fertilisers under the contract and cl 13 dealing with the price make it clear that the intention of the parties is that, in relation to the
contract goods, Basic is to act solely as a middle-man disposing of (ie, finding sub-purchasers for) fertilisers produced by
Colvilles, that its purchases from Colvilles are to be limited to fertilisers required to satisfy sub-contracts and that it alone is
entitled to determine when, where and in what quantities fertilisers are to be delivered by Colvilles to Basic or to Basics order
for the purpose of fulfilling such sub-contracts between Basic and its sub-purchasers.
Clause 16 contains two covenants by Basic. The first is a covenant to use its best endeavours to dispose of the whole of the
fertilisers which Colvilles shall produce during the period of the contract. The second, which qualifies the first, is:

to allocate and apportion in an equitable and reasonable manner between the vendor [viz., Colvilles] and other vendors
who have entered into similar agreements all deliveries of the fertilisers.

The other vendors referred to are identified by a memorandum attached to the vertical contract as being the other members and
the other steel company which formerly sold all its production of fertilisers to Basic. This last-mentioned company can for the
purposes of the present appeals be ignored. In the context of the vertical contract deliveries of fertilisers in cl 16 means
deliveries of fertilisers to Basic under the vertical contract and similar contracts.
I agree with the submission of counsel for the registrar that this second contract by Basic which is positive in form involves
by necessary implication a negative obligation on Basic which, amending somewhat the wording suggested by him, I myself
would prefer to express as follows: viz, in any period when the quantity of fertilisers for which Basic can find sub-purchasers is
less than the total quantity of fertilisers produced by Colvilles and the other members, not to acquire fertilisers (ie, not to call for
deliveries of fertilisers to fulfil sub-contracts made by Basic) from other members in such quantity as would result in Basics
being unable (viz, because it could not find sub-purchasers therefor) to acquire from Colvilles an equitable and reasonable
proportion of the total quantity of fertilisers produced by Colvilles and the other members.
817
An alternative way of putting the negative obligation in any such period would be: not to supply to sub-purchasers
fertilisers acquired by Basic from other members in such quantities as would result in Basics being unable to find sub-purchasers
for such quantity of fertilisers produced by Colvilles as constituted a fair and reasonable proportion of the total quantity of
fertilisers produced by all members.
This is a restriction accepted by Basic with respect to some of the matters set out in s 6(1), para (c), viz, the quantities or
descriptions of goods to be produced, supplied or acquired. Looked at in the first way it is a restriction on the quantity of
fertilisers produced by other members to be acquired by Basic. Looked at in the alternative way it is a restriction on the quantity
of fertilisers produced by other members to be supplied by Basic to sub-purchasers. Looked at either way it is a restriction which
does not relate exclusively to goods to be supplied pursuant to the vertical contract, ie, fertilisers produced by Colvilles.
Counsel for the registrar also contended that in the context of the vertical contract the covenant by Basic to use its best
endeavours to dispose of the whole of the fertilisers which Colvilles shall produce of itself involved a negative obligation falling
within the terms of s 6(1), para (c). I do not think it necessary to decide this, for I share the doubts expressed by Willimer LJ
about it. This covenant in the absence of the covenant about allocation of deliveries from other members involves no implied
negative obligation on Basics acquiring fertilisers from other sellers: it need not sell what it acquires. It is true that, in a period
in which it could not find sub-purchasers for the total quantity of fertilisers produced by Colvilles, Basic would commit a breach
of this covenant if it supplied to a sub-purchaser fertiliser not produced by Colvilles in fulfilment of a sub-contract for sale of
fertilisers of a description and quality which could have been satisfied by fertilisers produced by Colvilles; but I should prefer to
reserve the question whether this can properly be categorised as a restriction with respect to the quantities of goods to be supplied
within the meaning of s 6(1), para (c) of the Act.
It is conceded by counsel for the registrar that no clause of the vertical contract other than cl 16 contains a restriction
accepted by Basic which does not relate exclusively to the goods to be supplied pursuant to the vertical contract. It is conceded
by counsel for the applicants that the vertical agreement contains restrictions accepted by Colvilles which do not relate
exclusively to the goods to be supplied pursuant to the contract; in particular, in cl 3, a restriction on the quantities of goods to
which a process of manufacture is to be applied by Colvilles and, in cl 5, a restriction on the supply by Colvilles to third parties
of certain descriptions of goods, which Basic has not undertaken to buy.
Although the restriction accepted by Basic in cl 16 is a restriction

accepted by the party acquiring the goods, in respect of the sale, or acquisition for sale, of other goods of the same
description

within the meaning of s 8(3) of the Act, neither of the restrictions accepted by Colvilles in cl 3 and cl 5 is such a restriction as is
referred to in that subsection and accordingly the vertical agreement is not exempted from registration under Part 1 of the Act.
For this reason, which is substantially the same as one of those given by the judge, I would dismiss the first appeal.

The Second Appeal: the Horizontal Arrangement.

Agreement is defined in s. 6(3) of the Act as including


any agreement or arrangement, whether or not it is or is intended to be enforceable (apart from any provision of this
Act) by legal proceedings, and references in this Part of this Act to restrictions accepted under an agreement shall be
construed accordingly.

At about the same time in 1953 in the circumstances described in the judgment of Cross J each of the members entered into
an agreement in terms identical with those of the vertical contract, under which each agreed with Basic not to sell any 818
fertilisers to anyone except Basic. The only question in the second appeal is whether this was a pure coincidence or whether it
was done under an arrangement between the members, which has been conveniently referred to as the horizontal arrangement.
That it was pure coincidence offends credulity, and in doing justice to the second appeal one is confronted with a linguistic
difficulty. As Sir Lionel Heald, with his unconscious flair for the mot juste, found in opening the facts, it is not easy to talk
naturally about what went on between the members without using as he did the word arrangement. This was no Freudian slip
of the tongue but because in ordinary parlance there is no word more apt to describe it.

Arrangement is not a term of art; and in s. 6(3) of the Act I agree with my lords that it bears the meaning that an
ordinary educated man would ascribe to it. It involves a meeting of minds because under s. 6(1) it has to be an
arrangement between two or more persons and, since it must be an arrangement under which restrictions are accepted
by two or more parties, it involves mutuality in that each party, assuming he is a reasonable and conscientious man, would
regard himself as being in some degree under a duty whether moral or legal to conduct himself in a particular way or not to
conduct himself in a particular way as the case may be, at any rate so long as the other party or parties conducted
themselves in the way contemplated by the arrangement.

No necessary or useful purpose would be served by attempting an expanded and comprehensive definition of the word
arrangement in s 6(3) of the Act. Cross J said ([1962] 3 All ER at p 255; LR 3 RP at p 196):

all that is required to constitute an arrangement not enforceable in law is that the parties to it shall have
communicated with one another in some way and that as a result of the communication each has intentionally aroused in
the other an expectation that he will act in a certain way.

I think that I am only expressing the same concept in slightly different terms if I say without attempting an exhaustive definition,
for there are many ways in which arrangements may be made, that it is sufficient to constitute an arrangement between A and
B, if (i) A makes a representation as to his future conduct with the expectation and intention that such conduct on his part will
operate as an inducement to B to act in a particular way; (ii) such representation is communicated to B, who has knowledge that
A so expected and intended, and (iii) such representation or As conduct in fulfilment of it operates as an inducement, whether
among other inducements or not, to B to act in that particular way.
On the evidence in the present case it is plain beyond a peradventure that the knowledge of each member acquired at the
board meetings of Basic from statements made by the nominees on that board of his fellow members that each of his fellow
members was going to enter into a contract with Basic in the terms of the vertical contract, or at any rate that any of his fellow
members who entered into a contract for the sale of fertilisers to Basic would do so on substantially the same terms as of those of
the vertical contract, operated as an inducement to each member himself to enter into a contract with Basic in the same terms as
those of the vertical contract. If this is not an arrangement I do not know what is.
It was, however, contended on behalf of the applicants that the observations of Upjohn J in Re Austin Motor Co Ltds
Agreements ([1957] 3 All ER at p 69; LR 1 RP at p 19; [1958] Ch at p 74) showed that the expression arrangement in s 6(3) of
the Restrictive Trade Parctices Act, 1956, must involve mutual rights and obligations. The issue in that case was not what was
comprised in the concept of rights and obligations not enforceable in law, but whether having regard to the evidence of the
history of a bipartite agreement alleged to be registrable under Part 1 of the Act any rights or obligations, whatever was
comprised in that concept, were acquired by or imposed upon third persons who were not parties to that bipartite agreement.
Upjohn J held 819 on the evidence before him in that case that no rights or obligations were acquired or imposed by the bipartite
agreement by or on anyone but the two parties to it and that the agreement was exempted from registration under s 8(3) of the
Act. I see no reason for doubting the correctness of his decision, but the observations relied on appear to me to have no relevance
to the issue in the second appeal before this court. I would uphold the judgment of Cross J and dismiss this appeal also.

Appeals dismissed. Leave to appeal to the House of Lords granted to both parties.
Solicitors: Bird & Bird (for the applicants); Treasury Solicitor (for the Registrar of Restrictive Trading Agreements).

F A Amies Esq Barrister.


[1963] 2 All ER 820

Re Rainbow Tours Ltd


COMPANY; Insolvency

CHANCERY DIVISION
BUCKLEY J
7, 8, 9, 31 MAY 1963

Company Attachment of debt Garnishee order absolute obtained by judgment creditors against company Order made after
judgment creditors were given notice of meeting of creditors to consider winding-up Debt attached paid to judgment creditors
Winding-up resolution passed and liquidator appointed after debt so received Whether judgment creditors entitled to retain
moneys recovered under the garnishee order as against the liquidatory Effect of s 325(1) of, and grounds for relief under
proviso(c) to s 325(1) of, Companies Act, 1948, (11 & 12 Geo 6 c 38).

From October, 1961 onwards the respondents demanded payment of moneys owing to them by R Ltd. On 9 March 1962, the
respondents obtained judgment in the Queens Bench Division against R Ltd in a sum of 1,233 19s and 14 1s 6d costs. They
attempted to levy execution by writ of fieri facias but the sheriff made a return of nulla bona. On 30 March 1962, the respondent
discovered who R Ltds bankers were. A meeting of R Ltd was held on 2 April 1962, to consider its financial position, at which
meeting the respondents and two other creditors only were present. On 3 April 1962, the respondents obtained a garnishee order
nisi on the balance standing to R Ltds credit at its bank. The order was made absolute on 17 April 1962. It contained an
undertaking by the respondents not to part with the proceeds before 17 May 1962, except to a duly appointed liquidator or with
the leave of the court. on 25 April 1962, the bank paid 576 2s 7d to the respondents. The respondents received from R Ltd on 14
April 1962, notice of a meeting of creditors to be held on 30 April 1962, on which date R Ltd went into creditors voluntary
liquidation. On the question whether s 325(1)a of the Companies Act, 1948, disentitled the respondents from retaining the 576
2s 7d as against the liquidator,
________________________________________
a The relevant provision of s 325 of the Companies Act, 1948, are set out at p 822, letter g, post.

Held The benefit of the execution, which by s 325(1) of the Companies Act, 1948, a creditor was disentitled from retaining,
was confined to the charge on the companys property occasioned by the execution, so far as that charge remained operative;
therefore, the respondents having succeeded in obtaining the 576 2s 7d before the liquidator had got in that sum or had sued for
it were entitled to retain that sum for their own benefit, the terms of respondents undertaking not being such as to make the
receipt of the 576 2s 7d not a receipt for their own use and benefit (see p 825, letters d and f, g, p 826, letter g, and p 827, letter
a, post).
Re Andrew ([1936] 3 All ER 450) considered and applied.
Per Curiam: if the decision expressed above were wrong, the discretion conferred by proviso (c) to s 325 of the Companies
Act, 1948, would not be 820 exercised in favour of the respondents, because (a) diligence alone would not entitle a judgment
creditor to relief under the proviso, (b) any delay of the respondents in not applying immediately for a garnishee order absolute
was not due to any pressure or request on the part of R Ltd whose opposition to the order did not prejudice the respondents, and
(c) the fact that the garnishing of R Ltds bank balance precipitated the winding-up was not a ground for granting relief from the
effect of s 325(1) (see p 827, letter h, and p 828, letter g, post).
Re Grosvenor Metal Co Ltd ([1949] 2 All ER 948) and Re Suidair International Airways Ltd ([1950] 2 All ER 920)
considered.

Notes
As to the restriction of creditors retaining the benefit of execution or attachment as against liquidators see 6 Halsburys Laws (3rd
Edn) 690, para 1377; and for cases on execution, see 10 Digest (Repl) 10161021, 69907027.
For the Bankruptcy Act, 1914, s 40, s 41, see 2 Halsburys Statutes (2nd Edn) 376, 377.
For the Companies Act, 1948, s 227, s 228, s 325, s 326, see 3 Halsburys Statutes (2nd Edn) 646, 647, 707, 708.

Cases referred to in judgment


Andrew, Re, Official Receiver v Standard Range & Foundry Co Ltd (No 2) [1936] 3 All ER 450, 155 LT 886, sub nom Re
Andrew, Ex p Official Receiver (Trustee) (No 2) [1937] Ch 122, 106 LJCh 195, [19367] B & CR 205, 5 Digest (Repl) 878,
7338.
Armorduct Manufacturing Co Ltd v General Incandescent Co Ltd [191113] All ER Rep 141, [1911] 2 KB 143, 80 LJKB 1005,
104 LT 805, 10 Digest (Repl) 1081, 7483.
George v Thompsons Trustee [1949] 1 All ER 554, [1949] Ch 322, 5 Digest (Repl) 879, 7347.
Godwin, Re, The Trustee v Morris [1934] All ER Rep 140, [1935] Ch 213, 104 LJCh 189, 152 LT 392, 5 Digest (Repl) 877, 7336.
Grosvenor Metal Col Ltd, Re [1949] 2 All ER 948, [1950] Ch 63, 10 Digest (Repl) 1018, 7009.
Lupkovics, Re, Ex p Trustee v Freville [1954] 2 All ER 125, [1954] 1 WLR 1234, 5 Digest (Repl) 880, 7354.
National United Investment Corpn, Re [1901] 1 Ch 950, 70 LJCh 461, 84 LT 766, 10 Digest (Repl) 995, 6856.
Suidair International Airways Ltd, Re [1950] 2 All ER 920, [1951] Ch 165, 10 Digest (Repl) 1021, 7024.

Adjourned Summons
This was an application by originating summons dated 6 November 1962, issued by Kenneth Russell Cork, the liquidator of
Rainbow Tours Ltd (the company) which was in creditors voluntary liquidation for (i) a declaration that the respondents, Blue
Cars Continental Coach Cruises Ltd were not entitled to retain, pursuant to s 325, sub-s (1), proviso (a) and sub-s (2) of the
Companies Act, 1948, the benefit of an execution levied by them on the property of the company by attachment of a sum of 576
2d 7d standing to the credit of the companys account at the National Provincial Bank Ltd by means of a garnishee order absolute
dated 17 April 1962. The garnishee order absolute was made in an action brought in the Queens Bench Division by the
respondents against the company, in which they had obtained a judgment dated 9 March 1962. The grounds on which the
declaration was sought on the present summons was that the attachment had not been completed by receipt of the debt attached
by the respondents before they received notice on 14 April 1962, of the calling of a meeting of creditors for 30 April 1962, at
which a resolution for voluntary winding-up of the company was to be proposed. The liquidator also sought by the summons an
order that the execution put in force by the attachment should be stayed, and a declaration that the applicant as liquidator was
entitled to recover from the 821 respondents the sum of 576 2s 7d, which the respondents had received on or about 25 April
1962, together with interest at the rate of 5 per cent per annum from 9 May 1962, until payment of judgment, and an order for
payment of the sums.

Muir Hunter for the applicant liquidator.


J G Monroe for the respondents.

Cur adv vult

31 May 1963. The following judgment was delivered.

BUCKLEY J read the following judgment. On 9 March 1962, the respondents, Blue Cars Continental Coach Cruises Ltd
obtained judgment against Rainbow Tours Ltd (whom I will call the company) in the Queens Bench Division in a sum of
1,233 19s and 14 1s 6d costs. They attempted to levy execution of this judgment by a writ of fieri facias, but this was abortive,
as the sheriff made a return of nulla bona. Shortly after this the respondents learned from another creditor of the company that
the company proposed to arrange an informal meeting of its principal creditors to consider its financial position. Such a meeting
in fact occurred on 2 April 1962. The respondents and two other creditors only were represented at this meeting. The managing
director of the company then explained the financial position of the company and stated that the intention was that the company
should continue to carry on business in the hope that sufficient money would be earned to pay all the companys creditors in full.
The business of the company was the arranging of holiday tours for travellers. The companys main outlay for the 1962 season,
consisting of the cost of printing brochures and of advertising, had then already been incurred. The respondents were not
satisfied by the companys proposals made at this meeting. On the next day, 3 April they obtained a garnishee order nisi on the
balance standing to the companys credit at a branch of the National Provincial Bank, Ltd. This order was made absolute on 17
April 1962, and on 25 April 1962, the bank paid 576 2s 7d to the respondents pursuant to the order. Meanwhile, on 14 April
1962, the respondents received notice from the company under s 293 of the Companies Act, 1948, of a meeting of creditors to be
held on 30 April 1962. On the latter date the company went into a creditors voluntary winding-up.
The question is whether, in these circumstances, the respondents are entitled, as they contend, to retain the moneys which
they have received from the bank, or whether, as the liquidator contends, he is entitled to recover these moneys from the
respondents for the benefit of the creditors generally. The answer depends on the meaning and effect of s 325 of the Companies
Act, 1948, which, so far as relevant, provides as follows:

(1) Where a creditor has attached any debt due to the company, and the company is subsequently would up, he
shall not be entitled to retain the benefit of the attachment against the liquidator in the winding-up of the company
unless he has completed the attachment before the commencement of the winding-up: Provided that (c) the rights
conferred by this subsection on the liquidator may be set aside by the court in favour of the creditor to such extent and
subject to such terms as the court may think fit. (2) For the purposes of this section, an attachment of a debt shall be
deemed to be completed by receipt of the debt.

The respondents contended, first, that the amount paid by the bank to them is not, according to the true interpretation of the
section, the benefit of the attachment of the sum standing to the companys credit with the bank within the meaning of the
section; and, secondly, that in any event this is a case in which the court should exercise its discretion under proviso (c) to s
325(1) so as to relieve the respondents from any obligation they might otherwise be under to repay the amount in question to the
liquidator.
I will deal first with the former of these contentions. A section in the terms of s 325 of the Act of 1948 (omitted proviso (c))
was first introduced into the code of 822 legislation dealing with the liquidation of companies in the year 1929. It is in the same
terms with appropriate modifications as the Bankruptcy Act, 1914, s 40. Before 1929 there was no statutory provision to this or
any like effect applicable to the winding-up of a company. In the case of a compulsory winding-up what is now s 227 of the Act
of 1948 (deriving from the Companies Act, 1862, s 153) rendered every disposition of the liquidated companys property made
after the commencement of the winding-up void, and what is now s 228 of the 1948 Act (deriving from the Companies Act, 1862,
s 163) rendered void every attachment, sequestration, distress or execution put into force after the commencement of the
winding-up. A compulsory winding-up normally is deemed to have commenced at the time of the presentation of the petition.
These sections and their predecessors, however, had no application to a voluntary winding-up. In a voluntary winding-up the
court had a discretionary jurisdiction to restrain a judgment creditor from levying or completing execution; but, unless he was so
restrained, a judgment creditor was perfectly entitled to enforce his judgment by execution, notwithstanding that by doing so he
obtained an advantage over other unsecured creditors, whether he initiated his execution before or after the commencement of the
voluntary winding-up or before or after receiving notice of an intention to wind the company up. See, for example, Armorduct
Manufacturing Co Ltd v General Incandescent Co Ltd, where the court refused to restrain an execution by a judgment creditor
levied on the day on which a resolution for a voluntary winding-up was passed because the judgment creditor had been induced
to stay his hand in respect of levying execution by a trick practised on him on behalf of the company. It was said that before
liquidation the rights of the other creditors were subject to the legal right of the judgment creditor to enforce his judgment by
execution, and that they were not entitled to take advantage of the companys wrongful act and so to participate therein for their
own benefit. This jurisdiction did not enable the court to order a judgment creditor who, before the matter came before the court,
had succeeded in obtaining some satisfaction of his judgment debt by means of an execution to repay any sums so received by
him, even if the execution remained uncompleted. Exactly the same position obtained in respect of attachments as in respect of
executions. This, the respondents contended, remains the position except so far as it may have been altered by the enactment of s
325.
Section 325, however, does not, the respondents pointed out, make executions or attachments which at the date specified in
the section remain uncompleted void: it does not even say that they shall be ineffective as against the liquidator. It provides that
the judgment creditor shall not be entitled to retain what is called the benefit of the execution or attachment. This expression
has been considered in a number of cases and until the decision of the Court of Appeal in Re Andrew, Official Receiver v
Standard Range & Foundry Co Ltd (No 2), there was a conflict of judicial opinion about its meaning. That conflict has, however,
been resolved in Re Andrew. The order of events in that case were (1) judgment; (2) execution; (3) conditional withdrawal of the
sheriff on terms of payment by the debtor to the creditor of certain instalments, some of which were duly paid; (4) an act of
bankruptcy; (5) re-entry by the sheriff and sale; (6) receiving order. The sheriff paid the proceeds of the sale to the trustee and
none of these were in issue in the case. The question was whether, notwithstanding the Bankruptcy Act, 1914, s 40, the judgment
creditor was entitled to retain the instalments which had been paid. He had received these before he had notice of the act of
bankruptcy, indeed, before the act of bankruptcy had been committed. The Court of Appeal ([1936] 3 All ER at p 460; [1937] Ch
at p 129) cited with approval passages from a judgment of the Divisional Court in Re Godwin, The Trustee v Morris ([1934] All
ER Rep 140 at p 144; [1935] Ch 213 at p 218) where it was said that the 823 phrase the benefit of the execution on the true
construction of the section refers solely to the protection obtained by an execution creditor by reason of his execution, and does
not describe any payments to an execution creditor whether by the sheriff or by the judgment debtor. The Court of Appeal
expressed its own viewb that, to the extent that a creditor has been paid his debt under and in virtue of an execution, the debt is
pro tanto discharged, and to that extent there is nothing on which s 40 can operate. The judgment proceeds ([1936] 3 All ER at p
465; [1937] Ch at p 135):
________________________________________
b Re Andrew, [1936] 3 All ER at p 464; [1937] Ch at p 135.

The operation of the section in such cases is limited to cases where there is at the date of the receiving order, or when
the creditor has notice of a bankruptcy petition or of an act of bankruptcy, still on foot a subsisting execution, and is limited
to the balance for which the execution is still operative. In respect of that balance it is true that there is a benefit of the still
incomplete execution which may be affected by the operation of s. 40, sub-s. (1).

I draw attention to the use of the word may. Then a little later it continues ([1936] 3 All ER at p 465; [1937] Ch at p 136):

Section 40, in our opinion, can only apply if or to the extent that there is a subsisting execution which is still operating
to charge the debtors goods, and it cannot operate in so far as goods have already been sold and the proceeds applied to the
partial discharge of the debt, or where payments on account have been made by the debtor in partial discharge of the debt in
order to avoid seizure or sale, or to induce a temporary withdrawal by the sheriff. In our opinion, to the extent that the debt
has been actually discharged, it is impossible to apply to money so paid the appellation of the benefit of the execution.
The money so paid has, in our opinion, become the money of the creditor with the result of wiping out pro tanto the debt.
The benefit of the execution can then only refer, in our opinion, to the charge still remaining under the still subsisting
execution for the balance of the debt.

This language is appropriate to apply to partial discharge of the debt at any time, whether before or after the date of the receiving
order or any other event mentioned in the section.
Counsel for the applicant, however, contended that the court should be understood to be speaking of the state of affairs at the
time of or antecedent to any of those events. If this were so, the word may, to which I have drawn attention, would seem to me
to be rather inappropriate. Counsel for the applicant further contended that a construction of s 40 which would allow an
execution creditor to retain moneys received after the commencement of a voluntary winding-up in consequence of his execution
would introduce a surprising breach in the general rule that as from the commencement of a winding-up, subject to the special
rights of preferential and secured creditors, all creditors are put on an equality, and would occasion a surprising difference
between bankruptcy and compulsory winding-up, on the one hand, and voluntary liquidations on the other; and, I think he might
have added, between executions levied on goods and attachments, to the latter of which s 326 of the Companies Act, 1948, and s
41 of the Bankruptcy Act, 1914, do not apply. He drew attention to proviso (c) to s 325(1) and said that this inferred that the
section conferred some right on a liquidator against a judgment creditor which was capable of being set aside or modified to the
creditors advantage, namely, as he contended, the right as from the commencement of the winding-up or the earlier date
mentioned in s 325, sub-s (1), proviso (a), to deny the creditor the benefit of his execution. His contention may, I think, be fairly
stated as being that the position crystallises on the commencement of the winding-up or such earlier date as is mentioned in
proviso (a) to s 325(1), so that as from that moment, if the execution or attachment has not then been completed, the judgment
creditor cannot, as against the liquidator, any 824 longer benefit from his execution or attachment unless the court so permits
under proviso (c).
On the the other hand, counsel for the respondents contended that an attachment constituted the judgment creditor a secured
creditor (see Re National United Investment Corporation) and that on the true construction of s 325 of the Companies Act, 1948,
it was this security which constituted the benefit of the attachment (see Re Andrew). He said that there was nothing in s 325 to
deprive a judgment creditor of his right, where a company was in voluntary liquidation, to enforce his judgment by execution
unless restrained by the court from doing so; and that s 325 did not relate to any fruits of the execution, that is to say, satisfaction
or partial satisfaction of his debt, which the judgment creditor might succeed in gathering before he was so restrained. He
emphasised that there was no provision in the Act requiring a judgment creditor to restore any such fruits to the liquidator in any
event.
In my judgment, the respondents contentions are sound. In the light of the decision in Re Andrew, it is not, I think,
permissible to interpret the benefit of the execution as though the phrase were the benefits of the execution or any benefit of
the execution. The words have been given a clear and restricted meaning, confined to the charge on the companys property
occasioned by the execution so far as that charge is for the time being operative. To the extent that the judgment creditor
succeeds in obtaining satisfaction of his debt without coming into collision with the liquidator, that charge is discharged and pro
tanto the benefit of the execution within the meaning of s 325 of the Companies Act, 1948, ceases to exist. By attachment of a
debt the garnishor, to the extent that he succeeds in obtaining payment from the garnishee, procures satisfaction of his judgment
out of an asset of the company which the liquidator has not yet got in. The process is one which of itself does not occasion any
collision between the garnishor and the liquidator. It might be otherwise if the liquidator in the name of the company were to sue
the garnishee for the debt before payment by the garnishee to the garnishor and the garnishee were to interplead. Nothing of this
kind happened in the present case. Moreover, the section does not provide, at any rate, in terms, that the creditor shall be
disentitled to retain the benefit of the execution or attachment as from the commencement of the winding-up or any other
particular time. It appears to me that the effect of the section must be considered in the circumstances existing at any moment at
which the creditor seeks to enforce or assert his rights in virtue of the execution or attachment against the liquidator or the
liquidator seeks to do something which he could not do if in fact the creditor then retained the benefit of the execution or
attachment. If before that moment the creditor obtains satisfaction or partial satisfaction of his debt, thereupon the benefit of the
execution is pro tanto spent and will no longer exist when that moment arrives.
Accordingly, in my judgment, the respondents are entitled, subject possibly to a consideration which I will now mention, to
retain the sum which they have received from the bank under the garnishee order. The garnishee order absolute contained an
undertaking given by the respondents through their solicitors not to part with the proceeds before 17 May 1962, except to a duly
appointed liquidator of the company or with the leave of the court. The question arises whether in these circumstances the
judgment debt has pro tanto been discharged or whether the 576 2s 7d was received by the respondents in some kind of
fiduciary capacity. In this connexion counsel relied on Re Lupkovics, Ex p Trustee v Freville. In that case a judgment creditor
obtained an order which was in effect a garnishee order under which a sum was on 29 October 1952, ordered to be paid to the
judgment creditor on her solicitors undertaking to retain it for six months and to indemnify the quasi-garnishee against any claim
in respect of that sum by a trustee in 825 bankruptcy of the judgment creditor. On 20 January 1953, a bankruptcy petition was
presented against the judgment debtor on which a receiving order was made on 4 March 1953. Upjohn J said this ([1954] 2 All
ER at 129):

Counsel for the wife has a further point. He says that the words benefit of the execution in s. 40(1) refer, not to the
fruits of the execution, but only to the charge remaining under the still subsisting execution for the balance of the debt: see
Re Andrew, Official Receiver v. Standard Range & Foundry Co., Ltd. (No. 2); and he says that the order has been complied
with, that there has been payment, the Ministry have been discharged of their indebtedness to the bankrupt and there is
nothing on which the benefit of the execution can operate, but, having regard to the words of sub-s. (2), he still has to say,
and he does say, that the attachment has been completed by receipt of the debt and that is really the whole question which I
have to determine. Did the order of Oct. 29 and payment to the solicitors in pursuance thereof constitute a receipt of the
debt? That must mean, in my judgment, receipt of the debt by the debtor or her duly authorised agents. In my judgment,
there is no receipt of the debt until, at all events, as between the judgment creditor, the judgment debtor and the garnishee,
there has been an unconditional payment to the judgment creditor or her duly authorised agents. In this case there has been
a payment to the judgment creditor or her duly authorised agents, but they are bound to hold the sum not solely as her
agents but, as stakeholders or trustees as security for the benefit of the Ministry of Health. If, instead of ordinary payment
to the wifes solicitors, payment into court for six months had been ordered, George v. Thompsons Trustee would have
been directly in point. If, instead of payment to the wifes solicitors, payment had been made to third parties as
stakeholders, I cannot see how the wife could have said there was receipt of the debt. It can make no difference that
payment was made to her solicitors, they receiving the money not solely as her agents, but as stakeholders or trustees
holding it in medio to meet a possible claim against the garnishee. In my judgment, there was no receipt of the debt while
the money or investments representing the same remained in the solicitors hands on the terms of the order, and, therefore
the attachment was not completed before the date of the receiving order.

That case differs from the present case in the important respect that the fund never reached the hands of the judgment creditor.
The object of the undertaking given by the respondents as garnishors in the present case may have been to ensure that if on any
ground a liquidator should thereafter succeed in establishing a right to recover the amount of the garnisheed debt from the
respondents a sufficient fund would be available; but the terms of the undertaking are not, in my opinion, such as to make receipt
of the 576 2s 7d by the respondents anything less than receipt of that sum for their own use and benefit. If the liquidator were to
succeed in recovering that amount, I apprehend that the position as between the respondents and the liquidator would be as
though no garnishee order had ever been made, but in setting about recovering it the liquidator must, in my judgment, accept that
the respondents as judgment creditors have in fact procured payment of the amount to them, not as stakeholders or in any such
fiduciary capacity, but as judgment creditors by means of an attachment. I should perhaps mention that it appears from exhibited
correspondence that in fact the money has been held and is still held by the respondents solicitors, but this is not on account of
anything contained in the order. The solicitors are merely the agents in this respect of the respondents and the fact that the money
is in their hands and not in those of the respondents is of no significance.
826
Accordingly, for the reasons that I have given, the respondents are, in my judgment, right on their first point. In case,
however, I should be held elsewhere to be wrong in that conclusion, I will state my view on the second point. Proviso (c) to s
325(1) was first enacted in 1947 and came into operation at the commencement of the Companies Act, 1948. In Re Grosvenor
Metal Co Ltd Vaisey J has held that under this proviso the court is enabled to do what is right and fair according to the
circumstances of each case, that is to say, as I understand it, right and fair as between the judgment creditor and the other persons
having claims on the assets of the company in the winding-up.
The relevant facts in this connexion are that from about October, 1961, onwards the respondents from time to time
demanded by telephone that the company should pay the amount due to them. They also periodically rendered statements
showing the amount due. On 1 February 1962, the respondents solicitors wrote to the company demanding payment. On 3
February the company replied, acknowledging liability, expressing regret for their default in payment and stating that they were
not then in a position to pay but that they hoped to be able to do so in May. In that letter the company stated that they had in hand
certain moneys representing deposits paid by clients in respect of tours in the summer of 1962 which the company was not then
prepared to disburse. On 12 February 1962, the respondents solicitors wrote to the company again demanding payment within a
week and threatening proceedings. No payment having been made, the respondents issued their writ on 28 February 1962, and
signed judgment on 9 March 1962. They thereupon levied their abortive execution by a writ of fieri facias. Meanwhile with the
object of discovering to what bank account the company paid deposits received from clients, the respondents arranged that an
employee of theirs should make a booking with the company. The employee drew a cheque in favour of the company on 15
March 1962, which was in due course cleared and returned cancelled to the drawers bank on or about 30 March 1962. Before
the last-mentioned date the respondents had become aware of the intention to hold the informal meeting of the companys
creditors which took place on 2 April 1962. As I have already said, they obtained their garnishee order nisi on the following day.
In these circumstances the respondents invoke proviso (c) to s 325(1) on four grounds: (a) that the respondents alone among
those concerned as creditors of the company took any steps to enforce their claims against the company and ought not to be
deprived of the fruits of their diligence; (b) that the respondents were induced to refrain for a critical period from enforcing their
claim by promises of payment out of the results of anticipated trading and by the move to hold a meeting of creditors of the
company; (c) that the company was continuing to incur fresh liabilities at a time when it was unable to meet its existing liabilities
and the action of the respondents by precipitating winding-up has left the company with a smaller deficiency than would have
been the case if it had continued in business; (d) that it is improbable that the said sum of 576 2s 7d would have been available
in the said bank account at the commencement of the winding-up even if the respondents had not taken their garnishee
proceedings.
As to the first of these grounds, diligence alone cannot, I think, entitle a judgment creditor to relief under the proviso. Any
judgment creditor may be said to be more diligent than any other creditor who could have obtained judgment but has not done so:
nevertheless, the section would apply to him. It does not, in my judgment, lie in his mouth to say that what Parliament has so
enacted is not right and fair.
On the second ground I was referred to Re Grosvenor Metal Co Ltd and to Re Suidair International Airways Ltd. In the
former case Vaisey J absolved the company of any trickery, but considered that the judgment creditors had been persuaded or
induced or requested to stay their hands in the matter of 827 the execution, but for which the execution would have been
completed before the commencement of the winding-up. The facts of that case do not appear from the report with particularity.
It seems, however, that the pressure by the company was exerted after the judgment had been obtained. In Re Suidair
International Airways Ltd, on the other hand, Wynn Parry J evidently formed the view that not only had the company been guilty
before action of stalling the creditors claims by promises and evasions but also it had sought and obtained leave to defend the
creditors action on the debt by means of evidence disputing the debt which the company had already by correspondence clearly
admitted. The judge said that the company had less than no merits, and he was obviously of the opinion that the company had
not acted in good faith. In the present case, while it is true that the company was in default in paying the respondents debt when
it became due and told the respondents that the company hoped to be able to make payment in May, it does not appear that the
company did anything which effectively influenced the respondents to postpone issuing their writ. Nor do I think that the
evidence shows that the company did anything which had the object or effect of making the respondents hold their hand in
respect of their attachment proceedings. The respondents could not attach the bank account until they knew where it was. This
they did not know until 30 March or there-abouts. That they did not then immediately apply for a garnishee order nisi may have
been due in part to the fact that 30 March in 1962 was a Friday and in part to the fact that the creditors meeting was to be held on
the following Monday. At any rate, there is nothing to show that the delay between 30 March and 3 April was due to any pressure
or request on the part of the company: it seems to have been the result of a free exercise of the respondents own discretion. The
company attempted to resist the attachment of the bank balance and to this end filed an affidavit stating that the balance on the
bank account represented deposits paid by clients of the company which would have to be returned to the clients in consequence
of the liquidation of the company. This opposition was unsuccessful and did not, so far as I can see, prejudice the respondents in
the least.
As regards the third ground, it is true that the attachment precipitated the winding-up, but I cannot see why this should be
considered to render the operation of s 325(1) other than right and fair. Whether the deficiency will be less in consequence of the
company having gone into liquidation when it did than would have been the case if it had continued in business is, I think,
speculative.
The fourth ground appears to me to be a somewhat oblique assertion that if the respondents had not attached the bank
balance, the company would, by some means or other (and I think improper means are hinted at), have rendered it inaccessible to
its creditors. There is no evidence justifying such a suggestion.
In my judgment, there is no good ground on which in the present case the respondents could seek to be relieved under s
325(1)(c) from the effects of s 325(1) if, contrary to the view I have earlier expressed, that subsection is applicable.

Application dismissed.

Solicitors: Herbert Oppenheimer, Nathan & Vandyk (for the applicant); Sydney Morse & Co (for the respondents).

Jennifer Sandell Barrister.


828
[1963] 2 All ER 829

Moss v Moss
CRIMINAL; Criminal Evidence

QUEENS BENCH DIVISION


LORD PARKER CJ, HAVERS AND WIDGERY JJ
21, 30 MAY 1963

Criminal Law Evidence Spouse Competence as witness Decree of judicial separation Competence of one spouse to give
evidence against other spouse charged with criminal offence.

In criminal cases, subject to certain common law and statutory exceptions, a spouse is incompetent to give evidence against the
other, and the incompetence continues after a decree absolute for divorce or a decree of nullity (where the marriage annulled was
merely voidable) in respect of matters which arose during the coverture (see p 831, letter g, and p 832, letter f, post) and applies
to the spouses notwithstanding a decree of judicial separation (see p 832, letters d and f, post).
Dictum of Lord Alvanley in Monroe v Twisleton ((1802), Peake Add Cas at p 220) and of Lord Goddard CJ in R v Algar
([1953] 2 All ER at p 1383) explained.
Appeal allowed.

Notes
As to competency of spouses as witnesses in criminal proceedings, see 10 Halsburys Laws (3rd Edn) 483, 484, paras 883, 884;
and for cases on the subject, see 14 Digest (Repl) 519521, 50205054, 523, 50705078, and 22 Digest (Repl) 413, 414, 4451
4456.

Cases referred to in judgment


Barker v Dixie (1736), Lee temp Hard 264, 95 ER 180, 17 Digest (Repl 190, 872.
Davis v Dinwoody (1792), 4 Term Rep 678, 100 ER 1241, 14 Digest (Repl 519, 5023.
Dawes v Creyke (1885), 30 ChD 500, 54 LJCh 1096, 53 LT 292, 27 Digest (Repl) 110, 811.
Doker v Hasler (1824), Ry & M 198, 171 ER 992, subsequent proceedings, (1825), 2 Bing 479, 22 Digest (Repl) 414, 4452.
Monroe v Twisleton (1802), Peake Add Cas 219, 170 ER 250, 22 Digest (Repl) 413, 4451.
OConnor v Marjoribanks (1842), 4 Man & G 435, 11 LJCP 267, 134 ER 179, subsequent proceedings (1843), 12 LJCP 161, 22
Digest (Repl) 414, 4453.
R v Algar [1953] 2 All ER 1381, [1954] 1 QB 279, [1953] 3 WLR 1007, 118 JP 56, 37 Cr App Rep 200, 14 Digest (Repl) 523,
5077.
Shenton v Tyler [1939] 1 All ER 827, [1939] Ch 620, 10 LJCh 256, 160 LT 314, CA, revsg [1938] 4 All ER 501, 22 Digest (Repl)
414, 4454.
Waite v Morland (1888), 38 ChD 135, 57 LJCh 655, 59 LT 185, 27 Digest (Repl) 110, 815.

Case Stated
This was a Case Stated by justices for the county of Middlesex acting in and for the petty sessional division of Spelthorne in
respect of their adjudication as a magistrates court sitting at Feltham on 28 January 1963. The respondent husband, Eric Tom
Moss, preferred an information against the appellant wife, Jean Violet Moss, charging that, between 24 November 1961, and 20
November 1962, in the County of Middlesex, she persistently made telephone calls without reasonable cause and for the purpose
of causing annoyance to the husband, contrary to s 66 of the Post Office Act, 1953. After the case for the husband had been
closed on 17 December 1962, counsel for the wife submitted, inter alia, that, as the parties were man and wife, the husband was
not a competent witness against the wife. The justices adjourned the hearing of the information and on the adjourned hearing on
28 January 1963, found the following facts. The parties were married on 7 May 1955. At 829 about Christmas, 1960, they
ceased to cohabit and never resumed cohabitation. On 9 April 1962, the wife was granted a decree of judicial separation from the
husband by the Probate, Divorce and Admiralty Division on the grounds of the husbands adultery. The wife remained so
judicially separated at the time of the telephone calls of which complaint was made.
It was contended on behalf of the wife that, as the husband and herself were husband and wife, he was not competent to give
evidence against her by reason of the common law and the provisions of s 2 of the Evidence Amendment Act, 1853, and s 4 of
the Criminal Evidence Act, 1898. It was contended on behalf of the husband that, as the coverture came to an end on the
pronouncement of a decree of judicial separation, the common law and the statutory provisions, whereby the husband became a
competent witness against the wife, were no longer applicable.
The justices were of the opinion that, as the coverture had come to an end on the pronouncement of a decree of judicial
separation, the husband was a competent witness against the wife in relation to acts occurring after the decree for the purposes of
prosecuting the information. They ruled against the wifes submission and adjourned the further hearing of the information in
order that the point of law might be submitted to the High Court of Justice by way of Case Stated.

M J Anwyl-Davies for the wife.


J M Drinkwater for the husband.

Cur adv vult

30 May 1963. The following judgment was delivered.

HAVERS J read the following judgment at the request of Lord Parker CJ: This is an appeal by way of Case Stated against a
ruling by the justices for the county of Middlesex acting in and for the petty sessional division of Spelthorne made in the course
of the hearing of an information preferred by the respondent husband against the appellant wife that she, between 24 November
1961, and 20 November 1962, in the County of Middlesex, persistently made telephone calls without reasonable cause and for
the purpose of causing annoyance to the husband, contrary to s 66 of the Post Office Act, 1953. [His Lordship stated the facts,
and continued:] It is clear that there is an essential difference between a decree absolute for divorce and a decree for judicial
separation under the Matrimonial Causes Act, 1950. The decree absolute for divorce terminates the marriage; the decree for
judicial separation does not, and, therefore, the spouses still remain husband and wife. However, the spouse in whose favour the
decree for judicial separation is granted is no longer under an obligation to cohabit with the other spouse. The court also has
power, where a decree of judicial separation is granted to the wife, to order payment of alimony to her. Section 21 of the
Matrimonial Causes Act, 1950, contains provisions in every case of judicial separation with regard to the wifes after-acquired
property and the liability of the husband for necessaries supplied for the use of his wife if any alimony ordered to be paid has not
been duly paid.
At common law, one spouse was not competent to give evidence against the other except in the case of offences against the
person or liberty of the other party to the marriage. The earliest statement which has been found on this subject appears in Cokes
Commentary On Littleton, Book 1, Ch 1, $S 1, at folio 6 (b), published in the year 1628. This note states:

It hath been resolved by the justices, that a wife cannot be produced either against or for her husband quia sunt duae
animae in carne una, and it might be a cause of implacable discord and dissension between the husband and the wife, and
a meane of great inconvenience.

The reasons given for the common law rule have not always been the same. Lord Hardwicke, in Barker v Dixie said that this rule
and the rule forbidding one spouse to give evidence in favour of the other existed in order to preserve 830 the peace of families.
In Butlers Nisi Prius, p 286, cited arguendo in Davis v Dinwoody, the reason for the rule is said to lie in the legal policy of
marriage. Lord Kenyon CJ in Davis v Dinwoody ((1792), 4 Term Rep at p 679), said that, quite apart from interest, the ground
of both rules was the presumption of bias.
Whatever be the true reason for the rule, there can be no doubt that it has existed for a very long time. Moreover, the
incompetency extended after the marriage was dissolved by divorce or terminated by the death of one of the spouses in respect of
anything which happened while the marriage was subsisting. In Monroe v Twistleton Lord Alvanley laid down that, where a man
and wife are divorced, the wife was not competent to prove a contract made by the husband previous to divorce or anything else
which happened during the coverture. In Doker v Hasler Best CJ followed the decision of Lord Alvanley. Both these cases were
approved by the Court of Common Pleas in OConnor v Majoribanks where the incompetence was extended to the widows
personal representatives. The reason for the rule was stated in that case by Tindal CJ and Maule J to be the necessity of
preserving the confidence of the conjugal relation. These cases were cited without criticism or dissent by the Court of Appeal in
Shenton v Tyler
The incompetence has now been held to extend to a case where a marriage has been lawfully contracted but is voidable at
the suit of one of the parties and a decree of nullity is subsequently pronounced. In such a case, one former spouse is not
thereafter competent to give evidence against the other in respect of criminal offences committed by that other spouse during
coverture: R v Algar. As regards civil proceedings, the common law rule was altered by the Evidence Act, 1853, but s 2 of that
Act provided that nothing therein should render any husband competent or compellable to give evidence for or against his wife,
or any wife competent or compellable to give evidence for or against her husband, in any criminal proceedings. The Act
implicitly recognised the existence of the rule at common law.
There has been a considerable number of modern statutory exceptions to the common law rule which allowed a wife to be a
competent but not a compellable witness. The Criminal Evidence Act, 1898, s 4, deals with some of the cases, and there have
been a number of modern instances since. It is not, however, necessary to refer to them, as the present case comes within none of
the categories. Section 12 and s 16 of the Married Womans Property Act, 1882, have no application to the facts of this case.
In criminal cases, therefore, subject to the common law and modern statutory exceptions mentioned above, a spouse remains
incompetent to give evidence against the other, and the incompetence continues after a decree absolute for divorce or a decree of
nullity (where the marriage annulled was merely voidable) in respect of matters which arose during the coverture.
Counsel for the husband, however, contended that the common law rule only applied while the wife was under coverture,
and that she was only under coverture while she was sub potestate viri. He further contended that a decree of judicial separation
brought the coverture to an end, and that thereafter the husband or wife was a competent witness against the other in respect of
any matter which arose after the date of the decree. Counsel for the husband sought to derive some assistance for this argument
from the fact that Lord Alvanley, in Monroe v Twisleton ((1802), Peake, Add Cas at p 220), said that the divorced wife was
competent to prove any fact after the divorce but not to prove a contract or anything else which happened during the coverture.
He also relied on the use by Lord Goddard CJ of similar 831 words in R v Algar ([1953] 2 All ER at p 1383; [1954] 1 QB at p
287). It is clear, however, that both Lord Alvanley and Lord Goddard were speaking of cases in which there had been a divorce,
and were using the words during coverture as a convenient phrase to describe the period of time when the marriage was
subsisting. Counsel for the husband also referred to Dawes v Creyke (cited without disapproval in Waite v Morland). This case,
however, was merely a decision on the construction of the words during the coventure in a marriage settlement.
Further, counsel for the husband contended that, if regard is paid to the reason for the common law rule, given in OConnor
v Marjoribanks, namely, to preserve the confidence of the conjugal relation, there is no room for the application of the rule after a
decree for judicial separation. He contended that a decree of judicial separation under which the spouses lived apart caused just
as much a disruption of the conjugal relationship as a decree absolute of divorce, and that there was no reason why thereafter one
spouse should not be competent to give evidence against the other in criminal proceedings in respect of matters which arose after
the decree. If this argument is correct, it would apply also to spouses who were living apart under non-cohabitation orders made
by justices under the Matrimonial Proceedings (Magistrates Courts) Act, 1960, and to spouses living apart under a separation
agreement.
I am unable to accept the argument of counsel for the husband though it is not lacking in attraction. The common law rule
has existed at any rate since 1628. Prior to the Matrimonial Causes Act, 1857, the Ecclesiastical Courts had for centuries granted
decrees of divorce a mensa et thoro which did not dissolve the marriage and were equivalent to the modern decree of judicial
separation. I know of no authority, and none was cited to us, in which it was held that, after the pronouncement of such a decree
either by an Ecclesiastical Court before 1857 or thereafter by the Court for Divorce and Matrimonial Causes or its successor the
Probate, Divorce and Admiralty Division of the High Court of Justice, one spouse has been held a competent witness against the
other in criminal proceedings (apart from the common law and statutory exceptions). It has also to be borne in mind that, after a
decree for judicial separation, there still remains the opportunity for reconciliation, which it is the policy of the law to encourage.
In my judgment, the common law rule applies to the spouses notwithstanding a decree of judicial separation, and neither
spouse is a competent witness against the other in criminal proceedings so long as the marriage subsists. The justices, therefore,
were in error in ruling that the husband was a competent witness against the wife in support of the information, and the Case
should be remitted to them for further hearing with a direction to this effect

WIDGERY J. I agree.

LORD PARKER CJ. I also agree.

Appeal allowed and Case remitted. Certificate that point of law of general public importance but leave to appeal to House of
Lords refused.

Solicitors: Sharpe, Pritchard & Co agents for Horne, Engall & Freeman, Egham (for the wife); Bentley, Taylor & Goodwin (for
the husband).

Henry Summerfield Esq Barrister.


832
[1963] 2 All ER 833

Practice Note
R v Southgate
PRACTICE DIRECTIONS

COURT OF CRIMINAL APPEAL


EDMUND DAVIES, MARSHALL AND LAWTON JJ
20 JUNE 1963

Criminal Law Trial Jury Direction to jury Erroneous direction in law Whether duty of counsel to draw the direction of
trial judge to error.

Notes
As to the duty of counsel in relation to bring to the courts notice any irregularity in a criminal trial, see 3 Halsburys Laws (3rd
Edn) 77, para 119.

Appeal
The appellant Edmund Thomas Southgate was tried at Nottingham Assize on 21 March and 22, 1963, before Phillimore J and a
jury on an indictment for non-capital murder, and invited a verdict of manslaughter on the grounds of provocation. When
directing the jury the trial judge paraphrased the direction on provocation in R v Duffy ([1949] 1 All ER 932), and later directed
that the question for the jury was whether there was in the circumstances of the case anything which in law could justify a
reasonable man in doing what the appellant did; and he repeatedly used the word justify in the course of his direction. At the
end counsel for the defence drew the trial judges attention to the repeated use of that word, which seemed to be a departure from
the words of s 3 of the Homicide Act, 1957 a. The trial judge immediately accepted that justify was not the right word, and so
directed the jury, who convicted the appellant of murder.
________________________________________
a Section 3 reads, so far as material, where on a charge of murder there is evidence on which a jury can find that the person charged was
provoked (whether by things done or by things said or by both together) to lose his self-control, the question whether provocation was
enough to make a reasonable man do as he did shall be left to be determined by the jury See also, as regards the law concerning
provision, 10 Halsburys Laws (3rd Edn) 712, para 1363.

A E James QC and A R Arneil for the appellant. There appears to be some doubt among members of the Bar with regard to
their duty in drawing the attention of the trial judge to an erroneous statement of law made during the course of the summing-up.
There is no doubt that it is counsels duty to draw attention to an erroneous statement of fact, and with some diffidence what
seemed to be a department from well settled law was mentioned to the judge at the end of the summing-up in this case.

A E James QC and A R Arneil for the appellant.


W A Sime QC and I C R McCullough for the Crown.

20 June 1963. The following judgment was delivered.

EDMUND DAVIES J giving the judgment of the court, said that counsel, in the circumstances, had adopted a course which was
commendable and desirable. [His Lordship then considered the merits of the appear and dismissed it, applying the proviso to s
4(1) of the Criminal Appeal Act, 1907.]

Solicitors: Cleaver & Son, Derby (for the appellant); Director of Public Prosecutions (for the Crown).

N P Metcalfe Esq Barrister.


833
[1963] 2 All ER 834
Corporation of London and Others v Appleyard and Another
PERSONAL PROPERTY

QUEENS BENCH DIVISION


MCNAIR J)
25, 28, 29, 30 JANUARY, 27 FEBRUARY 1963

Personal Property Finding Possession Building operation Workmen in cellar of demolished building finding bank notes
in safe built in wall True owners unknown Whether freeholders, building leaseholders or finders entitled to notes Whether,
if finders entitled, they would be bound in law to account to their employers, who were independent contractors Equitable title
of freeholders by virtue of a provision of the building lease.

On 30 June 1961, two workmen in the course of their employment by W Ltd building contractors, on a building operation on the
site of a demolished building found in the cellar an old wall safe built into the wall. Inside the safe was a wooden box containing
bank notes, issued in 1943 or 1944, to the value of 5,728. The true owner of the notes was never found. The freeholders of the
site of the building were the Corporation of London. Before the building operation had begun V Ltd had purchased the residue of
a lease of the premises expiring in 1964, and Y Ltd had financed that purchase. On 27 January 1961, the Corporation (as
freeholders), V Ltd (as leaseholders) and Y Ltd (as contractors) entered into an agreement in the form of a building lease under
which Y Ltd undertook to erect a new building on the site and the Corporation undertook to grant a lease for ninety-nine years
from 25 March 1962. Clause 15 of the agreement provided Every relic or article of antiquity rarity or value which may be found
in or under any part of the site shall belong to [the Corporation] and immediately upon finding of any such relic or article of
antiquity or rarity or value the same shall be carefully and without avoidable damage removed by [Y Ltd] and delivered to [the
Corporation]. By a letter dated 23 March 1961, V Ltd confirmed to Y Ltd that V Ltd held the property on trust for Y Ltd. In
June, 1961, Y Ltd entered into a building contract with W Ltd for the construction of the new building, W Ltd being in law
independent contractors. On an issue who was entitled to the bank notes found, in default of their true owner being traced,

Held The corporation were entitled to the notes as against anyone other than the true owner for the following reasons
(i) because the safe, being built into the wall, had formed part of the demised premises and, accordingly, it and its contents
were in the de facto possession of either Y Ltd or V Ltd one or other of whom was in possession of the premises, and thus had a
better title to the notes than the finders (see p 838, letter h, post).
Principle stated by Lord Russell Of Killowen CJ in South Staffordshire Water Co v Shareman ([18959] All ER Rep 259 at p
261) applied.
Bridges v Hawkesworth ((1851), 21 LJQB 75) and Elwes v Brigg Gas Co ((1886), 33 ChD 562, considered.
(ii) the bank notes were articles of value within cl 15 of the agreement of 27 January 1961, that expression not being in the
context restricted to articles whose value depended on their antiquity or rarity, and accordingly cl 15 conferred on the Corporation
an equitable title to the notes as against Y Ltd and V Ltd (see p 840, letters e, and g, post).
Per Curiam: (a) if contrary to holding (i), supra, the finders had been entitled to the bank notes, they would have been
accountable in law to W Ltd as their principals, for the bank notes, and not to Y Ltd since the finders were the servants of W Ltd
there being no general rule that for all purposes the servant of an independent contractor was to be regarded as the servant of the
person (Y Ltd) by whom the independent contractor was engaged (see p 839, letter e, post).
834
(b) possession of the premises at the material time was in Y Ltd rather than V Ltd either because Y Ltd were in actual
possession, or because V Ltd held the lease as trustee for Y Ltd either in consequence of a resulting trust arising from their having
financed the purchase of the lease or by reason of the letter of 23 March 1961 (see p 839, letter g, post).

Notes
As to the right of the finder of a chattel, see 29 Halsburys Laws (3rd Edn) 368, 369, para 729 and 2 ibid, 100, 101, para 200; and
for cases on the subject, see 3 Digest (Repl) 6770, 83105.

Cases referred to in judgment


A-G v Goddard (1929), 98 LJKB 743, 1 Digest (Repl) 546, 1700.
Balfour v Barty-King (Hyder & Sons (Builders) Ltd third parties) [1957] 1 All ER 156, [1957] 1 QB 496, [1957] 2 WLR 84, 3rd
Digest Supp.
Bridges v Hawkesworth (1851), 21 LJQB 75, 18 LTOS 154, 3 Digest (Repl) 68, 85.
Cohen, Re, National Provincial Bank Ltd v Katz [1953] 1 All ER 378, [1953] Ch 88, [1953] 1 WLR 303, 3rd Digest Supp.
Elwes v Brigg Gas Co (1886), 33 ChD 562, 55 LJCh 734, 55 LT 831, 3 Digest (Repl) 69, 88.
Grafstein v Holme and Freeman (1958), 12 DLR (2d) 727, [1958] OR 296, [1958] OWN 161, affg 9 DLR (2d) 444, 3 Digest
(Repl) 71, 72*.
Hannah v Peel [1945] 2 All ER 288, [1945] KB 509, 114 LJKB 533, 3 Digest (Repl) 68, 87.
Morison (Morrison) v Thompson (1874), LR 9 QB 480, 43 LJQB 215, 30 LT 869, 38 JP 695, 1 Digest (Repl) 548, 1712.
Parker v McKenna (1874), 10 Ch App 96, 44 LJCh 425, 31 LT 329, 3 Digest (Repl) 154, 179.
Reading v A-G [1951] 1 All ER 617, [1951] AC 507, 1 Digest 549, 1720.
Regal (Hastings) Ltd v Gulliver [1942] 1 All ER 378, 9 Digest (Repl) 523, 3447.
South Staffordshire Water Co v Sharman [18959] All ER Rep 259, [1896] 2 QB 44, 65 LJQB 460, 74 LT 761, 3 Digest (Repl)
69, 89.
Title of the Finder, The [1899], 33 ILT 225.
Willey v Synan (1936), 57 CLR 200.

Issue
By an order of Master Harwood dated 5 April 1962, made on an interpleader summons taken out under RSC, Ord 57, r 5, by the
Commissioner of Police for the City of London, it was ordered that an issue should be tried, the question to be which of five
claimants was entitled to certain money deposited by the commissioner with Barclays Bank Ltd and accruing interest and two
Scottish bank notes. The order directed that three of the claimants, namely, the Corporation of London (the Corporation),
Venture Property and Development Co Ltd (Venture) and Priest Marians & Co Ltd (Priest Marians) should be plaintiffs, and
the other two claimants, Norman Appleyard and Stephen Falan, should be defendants. By an order dated 25 October 1962, made
with consent, Yorkwin Investments Ltd (Yorkwin) were added as a claimant and as fourth plaintiff. Points of claim were
delivered by the first, second and fourth plaintiffs; the third plaintiffs withdrew their claim. Each defendant filed an affidavit in
support of his claim. The issue was tried on an agreed statement of facts.
The following summary of the relevant facts is taken from the judgment. On 30 June 1961, the two defendants, who were
workmen employed by Wates Ltd were working in the cellar of an old building at Nos 26 and 27 Bevis Marks, in the City of
London, which had been demolished to make way for a new building. The celler extended under the footway of Bevis Marks. In
that cellar the defendants were engaged in cutting a key-way into one of the walls of the cellar 835 so as to key in cement as part
of the foundations of the new building, when they found an old wall-safe built into a recess in the old wall. They broke open the
rusty door of the safe, which appeared to have remained undisturbed for many years, and found within a wooden box which,
when opened, was found to contain a large number of Bank of England notes and two Scottish notes, all issued in 1943 or 1944
and amounting in value to 5,728. They handed over the notes to the City of London police. The proceeds of the notes, other
than the two Scottish notes, were deposited in an interest-bearing account with Barclays Bank, Ltd. The true owner of the notes
was never traced.
Since at least 1888, the Corporation had been owners in fee simple of the land and buildings known as Nos 26 and 27 Bevis
Marks (hereinafter referred to as the premises). By two leases dated 2 May 1889, the Corporation let the premises to one
Samuel Samson for a term of eighty years expiring on 24 June 1964. Some time before 1935 the residue of each of the terms so
created was assigned to Priest Marians. On 14 July 1959, Venture and Yorkwin, being minded to redevelop the site of Nos 26 and
27 Bevis Marks, together with other neighbouring properties to which they had or were acquiring title, either as leaseholders or
freeholders, procured an assignment by Priest Marians to Venture of the residue of each of the aforesaid leases, the consideration
for the assignment, namely 65,000, being supplied by Yorkwin as an associated company of Venture. On 27 January 1961, an
agreement in the form of a building lease was entered into between the Corporation as freeholders and Venture as the present
leaseholder of (inter alia) Nos 26 and 27 Bevis Marks and Yorkwin as contractors. Under this agreement a Yorkwin undertook to
erect on the composite site including Nos 26 and 27 a new building to the satisfaction of the city surveyor by 25 March 1962, and
the Corporation agreed to grant a new lease of the site for ninety-nine years from 25 March 1962, at a yearly rental of 12,000.
On 23 March 1961, by letter of that date, Venture confirmed to Yorkwin that Nos 26 and 27 Bevis Marks, then registered in their
names, were held by them on trust and to the order of Yorkwin as at 31 December 1960, the latter having found the purchase
consideration. In June, 1961, Yorkwin entered into a building contract on the standard RIBA form with Wates Ltd for the
construction of a new building on the site, and thereafter work commenced.
________________________________________
a Clause 15 of this agreement, which is relevant to the issue decided in this case, is set out at p 839, letter i, post.

E A Machin for the first plaintiffs (the Corporation).


A E Holdsworth for the second and fourth plaintiffs (Venture and Yorkwin).
R A W Sears for the defendants (for finders).
The third plaintiffs, having discontinued proceedings, did not appear and were not represented.

27 February 1963. The following judgment was delivered.

MCNAIR J having summarised the facts regarding the discovery and handing over of the notes by the defendants to the police,
stated the issue and continued: The issue raises a fundamental question of law as to the rights of finders of chattels as against
persons in possession of premises in or under or on which the chattels are found. If this question is answered in favour of the
defendants, the finders, a further question arises whether the finders, as servants of Wates Ltd who were the independent
contractors of Yorkwin Investments Ltd (hereinafter referred to as Yorkwin), were in law obliged to hand the notes over to
Yorkwin as their principals. If Venture Property and Development Co Ltd (hereinafter referred to as Venture) or Yorkwin,
either as the parties in possession or as principals of the finders, were in the first instance entitled to the notes, the final question
to be determined is whether, under the terms of a building lease between the Corporation of the City of London (hereinafter
called the Corporation) as freeholders and Yorkwin as contractors or Venture as guarantors, the property in the notes passed 836
to the Corporation, or whether Yorkwin or Venture are under a contractual obligation to cause the notes to be handed over to the
Corporation. The Corporation claimed no title to the notes as owners in fee simple, but relied on a derivative title through
Venture or Yorkwin. [His Lordship then stated the relevant facts relating to Nos 26 and 27 Bevis Marks, which are set out at p
836, letter c, ante, and continued:] Though no issue in fact was raised as between Venture and Yorkwin, it is clear in law, and I so
hold, that, at any rate by the date when the building operation commenced, Yorkwin were in lawful possession of the premises in
question either as lessees, as the consequence of the resulting trust created in their favour in July, 1959, by the furnishing of the
purchase money by them, or under the declaration of trust contained in the letter of 23 March 1961, or by reason of their entry in
fact into possession with the leave of the Corporation pursuant to the building lease, and of Venture for the purpose of the
building operations. If, however, I am wrong in this conclusion, it is clear that Venture were in possession of the premises. For
the solution of the first question which I have to decide, it is immaterial whether Venture or Yorkwin were in possession.
The first question which I have to decide raises the fundamental question of law as to the rights of finders of chattels as
against persons in possession of premises in or under or on which the chattels are found. The three leading English authorities on
this topic are Bridges v Hawkesworth, Elwes v Brigg Gas Co, and South Staffordshire Water Co v Sharman. These cases were the
subject of a valuable critical analysis by Professor A L Goodhart in an article entitled Three Cases on Possession in 3
Cambridge Law Journal 195. These cases themselves and the discussion on them in subsequent decisions in this country, for
example, in Hannah v Peel (jewellery found in a crevice of a window sill), and abroad, for example, in Ontario in Grafstein v
Holme and Freeman (chattels found in rubbish in a basement), raise many fine possible point of distinction between (i) chattels
found upon premises, for example, notes found on the floor of a shop as in Bridges v Hawkesworth; and (ii) chattels found under
land, for example, a prehistoric boat as in Elwes v Brigg Gas Co; and (iii) chattels found embedded in the land, for example, rings
found embedded in a pool of water on land as in the South Staffordshire Case. In my judgment, however the facts of this case so
clearly fall within the principle laid down by Lord Russell Of Killowen CJ and Wills J in the South Staffordshire case, sitting as a
Divisional Court on appeal from a county court, that no purpose would be served by any lengthy examination of the finer
problems involved.
The facts in the South Staffordshire case may be shortly stated as follows. The South Staffordshire water company were
owners in fee simple in possession of land on which was situated a pool known as the Minister Pool. A workman employed by
the South Staffordshire water company to clean out the pool found two rings in the mud at the bottom of the pool. Lord Russell
Of Killowen CJ with whom Wills J agreed, accepted ([18959] All ER Rep at p 261; [1896] 2 QB at pp 46, 47) as the ground of
his decision the following statement to be found in Pollock And Wright On Possession In The Common Law, p 41:

The possession of land carries with it in general, by our law, possession of everything which is attached to or under
that land, and, in the absence of a better title elsewhere, the right to possess it also. And it makes no difference that the
possessor is not aware of the things existence It is free to anyone who requires a specific intention as part of a de facto
possession to treat this as a positive rule of law. But it seems preferable to say that 837 the legal possession rests on a real
de facto possession, constituted by the occupiers general power and intent to exclude unauthorised interference.

Later, the learned chief justice stated this principle in slightly different terms as follows ([1896] 2 QB at p 47; [18959] All ER
Rep at p 261):

The general principle seems to me to be that where a person has possession of a house or land, with a manifest
intention to exercise control over it, and the things which may be upon or in it, then, if something is found on that land,
whether by an employee of the owner or by a stranger, the presumption is that the possession of that thing is in the owner
of the locus in quo.

I do not regard this passage as being intended to qualify or extend the principle stated in Pollock And Wright, though in terms the
words upon or in used by the chief justice are wider than the words attached to or under appearing in Pollock And Wright. In
Re Cohen, National Provincial Bank, Ltd v Katz, the following passage occurs in the judgment of Vaisey J ([1953] Ch at pp 92,
93; cf [1953] 1 All ER at p 380, letter d):

There is a consistent line of cases which shows that there is a legal presumption that the owner of land is the owner of
chattels found on the land; and for that proposition see South Staffordshire Water Co. v. Sharman.

In both these passage the term owner is used to denote the person in lawful possession.
On the facts here, the notes were found in a wooden box within a safe built into the wall of the old building. It seems to me
to be clear that the safe, in those circumstances, formed part of the demised premises. If so, Yorkwin or Venture, being in lawful
possession of the premises, were in de facto possession of the safe, even though they were ignorant of its existence.
It was argued, though I think rather faintly, that possession of the safe did not involve possession of the wooden box inside
it, still less of the notes within the box. To accept this argument would be to introduce a wholly unnecessary and unreasonable
refinement. If the prehistoric boat in Elwes v Brigg Gas Co, though its existence was unknown, was in the possession of the
person who as owner was in possession of the land under which the boat was found, so too would be the contents of a locker in
the boat. If the rings embedded in the mud at the bottom of the Minister pool in the South Staffordshire case were in the
possession of the person who as owner was in possession of the land on which the pool was situated, the result would, in my
judgment, have been the same if the rings had been found in a purse or other container found in the mud. For completeness, I
should add that in Hannah v Peel the contest was between the owner in fee simple of the premises and the finder, and this contest
was decided in favour of the finder on the basis that the owner had never been in possession. This case, therefore, does not assist
the solution of the present problem.
In my judgment, the notes having been found within the safe, which itself formed part of the demised premises, the party in
possession of the premises, whether it be Venture or Yorkwin, had, in the absence of any evidence as to the true ownership of the
notes, a better title thereto than the finders.
The conclusion which I have thus stated renders it strictly unnecessary for me to decide the second question, namely,
whether, assuming the finders have a title, they, as servants of Wates, who were the independent contractors of Yorkwin, were
under a legal obligation to hand the notes over to Yorkwin as being their principals. If the finders had been directly employed by
Yorkwin, I think there is little doubt that the finders would be under that legal obligation.
838
As it seems to me, the cases of A-G v Goddard and Reading v A-G, though in fact they arose out of the dishonesty of a
servant or persons in analogous positions, are merely illustrations of the general principle that a servant who receives property or
money, whether honestly or corruptly, by reason of his employment is accountable therefor to his master as principal: see
Morison (Morrison) v Thompson, per Cockburn J ((1874), LR 9 QB 480 at p 483); Parker v McKenna, per Lord Cairns ((1874),
10 Ch App 96 at p 118) and James LJ ((1874), 10 Ch App at p 124); and Story, Commentaries On The Law Of Agency (9th Edn)
$S 224. In this connexion, I was also usefully referred to a valuable decision of Palles CB reported under the name of The Title
of the Finder, where he says this:

I do not decide this case [a case of a porter in a bank finding notes in the bank when sweeping out the premises] on the
ground laid down by LORD RUSSELL in Sharmans case. I decide it on the ground of the relation of master and servant
and that it was by reason of the existence of that relationship and in the performance of the duties of that service that the
plaintiff acquired possession of the property . I hold that the possession of the servant of the bank was the possession of
the bank itself

This judgment was adopted by Dixon J in Willey v Synan ((1936), 57 CLR 200 at p 217); see too Regal (Hastings) Ltd v Gulliver,
where the principle was applied in the absence of fraud.
The finders in this case, however, were not the servants of Yorkwin but the servants of Wates Ltd who were engaged by
Yorkwin as independent contractors to construct the building. As at present advised, it would seem to me that, on the hypothesis
involved in the present question, the finders would be accountable to Wates Ltd their employers, and not to Yorkwin. Though in
some cases involving the carrying out of dangerous operations the person who engages an independent contractor is responsible
for the negligent acts of the servants of that independent contractor, there is no general principle that for all purposes the servant
of an independent contractor is to be regarded as the servant of the person by whom the independent contractor was engaged.
Were it otherwise, the whole of the argument and the judgment of Lord Goddard CJ in Balfour v Barty King (Hyder & Sons
(Builders) Ltd third parties) would be beside the point. Though, as stated previously, I do not consider it necessary to decide
finally whether the possession of the premises was in Yorkwin or in Venture, as at present advised, I should be prepared to hold
that at the material time the possession was in Yorkwin either on the ground that Yorkwin, as the building owners, were in actual
possession, or on the ground that Venture held the leases as trustees for Yorkwin either as the consequence of a resulting trust in
favour of Yorkwin arising from the fact that Yorkwin provided the consideration for the assignment of the lease to Venture, or by
reason of the declaration of trust contained in the letter b of 23 March 1961.
________________________________________
b See p 836, letter e, ante.

There remains the final question whether the Corporation are, by virtue of the agreement by way of building lease dated 27
January 1961, entitled to the notes as against Yorkwin or Venture or both. This question primarily depends for its solution on the
construction of cl 15 of that agreement, which provides as follows:

Every relic or article of antiquity rarity or value which may be found in or under any part of the site or in or upon any
remains of former buildings standing thereon shall belong to the Corporation and immediately upon 839 finding of any
such relic or article of antiquity or rarity or value the same shall be carefully and without avoidable damage removed by
and at the expense of the contractors [which expression by definition means Yorkwin] and delivered to the city surveyor or
as he shall direct.

The bank notes were clearly articles of value within the ordinary and natural meaning of that expression. On behalf of Yorkwin
and Venture it was argued: (i) that inasmuch as all property found of which the owner is unknown belongs to the person in
possession, this clause is to be regarded as a clause cutting down the possessors common law right in favour of the Corporation,
and must therefore be strictly construed; (ii) that as the phrase article of value is ambiguous, it must be construed against the
Corporation as proferentes; and (iii) that, on the principle of construction summarised in the phrase noscitur a sociis the phrase
article of value must, having regard to the presence in the clause of the words relic or article of antiquity or rarity be limited
to articles (a) which have a corporeal value of their own or (b) which have a special value to a collector or have a curiosity value.
Although the matter is largely one of first impression, I can see no reason for departing from the general rule that words are
to be taken to be used in their ordinary and natural sense, unless there is something in the instrument itself to show that the words
are not used in that sense. Bank notes are, in their ordinary meaning, articles of value. Many relics, for example pieces of
Roman pottery, have no corporeal value, if I understand the expression correctly, in themselves, but have a value dependent on
extraneous factors, for example, age and historical association. Many articles of antiquity or rarity have a value to others than
collectors, and have a value other than curiosity value. There are no words in the clause to confine articles of value to articles
which have a value dependent on their antiquity or rarity. Again, it is to be observed that treasure trove, which is also mentioned
in the clause, is not limited to articles of value to a collector or antique articles: see 2 Halsburys Laws (3rd Edn) 101, para 201,
note (m). Furthermore, the phrase shall be carefully and without avoidable damage removed, which was relied on as indicating
that the words related to articles of a fragile nature by reason of their antiquity, would equally apply to modern bank notes which
had been damaged by fire or water. A modern gold watch or a hoard of smuggled Swiss watches would, as I think, be within the
plain intention of the clause, which is, in substance, to secure that any adventitious profit derived from the finding of articles,
whether relies, antique or rare articles, or articles of value, should go to the Corporation. I see no reason for cutting down the
generality of the words articles of value.
In my judgment, the clause confers on the Corporation at least an equitable title to the notes as against Yorkwin, if, as I am
inclined to think Yorkwin were in possession, or against Venture, who, by cl 23, guarantees to the Corporation the due
performance and observation by Yorkwin of all the agreements and stipulations contained in the lease. I, accordingly, hold that
the Corporation are entitled, as against anyone but the true owner, to the fund now representing the notes, together with interest
thereon.

Judgment for the Corporation, the first plaintiffs.

Solicitors: The Comptroller and City Solicitor (for the first plaintiffs); Lawrance Messer & Co (for the second and fourth
plaintiffs); Guillaume & Sons, Weybridge (for the defendants).

Mary Colton Barrister.


840
[1963] 2 All ER 841

W v W (No 4)
FAMILY; Other Family

COURT OF APPEAL
WILLMER, DANCKWERTS AND DIPLOCK LJJ
16, 17 MAY 1963

Nullity Pregnancy at date of marriage by person other than petitioner Application for order for blood tests Supreme Court
of Judicature (Consolidation) Act, 1925 (15 & 16 Geo 5 c 49), s 32, s 103(1) Matrimonial Causes Act, 1950 (14 Geo 6 c 25), s
8(1)(d) Matrimonial Causes Rules, 1957 (SI 1957 No 619), r 24; RSC, Ord 37A, r 1, Ord 50, r 3.

On a petition for nullity of marriage under s 8(1)(d) of the Matrimonial Causes Act, 1950, on the ground that at the time of the
marriage the wife was pregnant by some person other than the husband, the court has no power to order the wife, or a child born
since the marriage, to undergo a blood test, for the following reasons
(i) because, as regards the inherent jurisdiction of the court, argument based on such medical inspection as could be obtained
in the limited kind of nullity suit known to the ecclesiastical courts (viz, a suit based on incapacity or impotence), was precluded
by s 32 of the Supreme Court of Judicature (Consolidation) Act, 1925 a (under which reference to the old practice could be made
only where no special provision was contained in, eg, rules of court), since the Matrimonial Causes Rules, 1957, r 24 made
special provision as to medical inspection in that type of nullity suit (see p 844, letter b, p 845, letter i, and p 846, letter c, post);
moreover (per Diplock LJ), s 103(1) of the Act of 1925 b would limit any ordering of medical inspection under the inherent
jurisdiction to the like purpose as formerly, viz, to ascertaining impotence or incapacity (see p 846, letter i, post).
________________________________________
a Section 32 is printed at p 844, letter c, post
b Section 103(1) is printed at p 846, letter g, post.

(ii) because, though r 24 conferred power to appoint medical inspectors in a nullity suit based on wilful refusal to
consummate marriage (a ground introduced contemporaneously with that now enacted in s 8(1)(d) of the Act of 1950), yet it did
not confer any such power in a suit under s 8(1)(d), and expressio unius est exclusio alterius (see p 844, letter i, and p 846, letter
f, post).
(iii) because RSC, Ord 50, r 3, was not apt for the purpose of conferring power to take a blood test (see p 845, letters f and i,
and p 846, letter b, post), nor did RSC, Ord 37A, confer power for that purpose.
Decision of Cairns J ante, p 386, affirmed.

Notes
As to evidence of blood-groups to prove pregnancy by some person other than the petitioner, see 12 Halsburys Laws (3rd Edn)
225, para 421, note (c); and for a case on the subject, see 27 Digest (Repl) 283, 2267.
As to medical inspection in a nullity suit where a party refuses inspection, see 12 Halsburys Laws (3rd Edn) 364, para 785;
and for cases on the subject, see 27 Digest (Repl) 512514, 45484569.
As to the jurisdiction conferred by RSC, Ord 50, r 3, see 30 Halsburys Laws (3rd Edn) 387, para 722.
For the Supreme Court of Judicature (Consolidation) Act, 1925, s 32, see 5 Halsburys Statutes (2nd Edn) 361; and for s
103(1), see 18 ibid, 516.
For the Matrimonial Causes Act, 1950, s 8(1)(d), see Halsburys Statutes (2nd Edn) 397.
For the Matrimonial Causes Rules, 1957, r 24, see Halsburys Statutory Instruments (1st Re-issue) 229.
For RSC, Ord 37, r 1, and Ord 50, r 3, see the Annual Practice.

Cases referred to in judgments


Briggs v Morgan (1820), 2 Hag Con 324, 3 Phillim 325, 161 ER 758, 27 Digest (Repl) 280, 2249.
De v Ag (falsely calling herself De), (1845), 1 Rob Eccl 279, 163 ER 1039, 27 Digest (Repl) 273, 2187.
841

Appeal
The petitioner husband appealed against an order of Cairns J made in chambers on 14 March 1963, and reported at p 386, ante,
dismissing the husbands application in a nullity suit for an order requiring the wife and her child to submit to a blood test on the
grounds that the court had not jurisdiction to order a blood test save where statute conferred such power, that RSC, Ord 50, r 3,
did not extend to permitting the court to order a blood test, nor was there otherwise any statutory jurisdiction to order a blood test.
The grounds of appeal were that the learned judge was wrong in law (a) in holding that the court had no inherent jurisdiction to
order a blood test, (b) in holding that RSC, Ord 50, r 3, did not extend to permitting the court to order a blood test, and (c) in
holding that to order a blood test would be to order an assault.
The cases noted belowc were cited in argument in addition to the cases referred to in the judgments.
________________________________________
c Greenstreet (falsely called Cumyns) v Cumyns (1812), 2 Hag Con 332, 2 Phillim Ecc 10, Brown v Brown, (1828), 1 Hag Ecc 523, Liff v Liff
(otherwise Rigby), [1948] WN 128, H v H & C, The Times, 23 March 1962, A v A (orse L), The Times, 10 April 1962

R L Bayne-Powell for the husband.


R Schulman for the wife.

17 May 1963. The following judgments were delivered.

WILLMER LJ. This is an appeal from an order made in chambers by Cairns J on 14 March 1963, whereby he dismissed an
application brought by the husband in a nullity suit for an order requiring the wife and her child to submit to a blood test. The
petition is one brought under s 8(1)(d) of the Matrimonial Causes Act, 1950, which gives a right to relief in the case where the
respondent was at the time of the marriage pregnant by some person other than the petitioner. It is not in issue that at the time of
the marriage the wife here was pregnant: in fact a child has subsequently been born. The difficulty in the case is caused by the
fact that the husband admits that he himself had sexual intercourse with the wife before marriage. He alleges that the wife also
had sexual intercourse with another man or men. The point has been made that in such a case the task of a petitioner in proving
his case is an extremely difficult one, bearing in mind the presumption of legitimacy. The husband accordingly asks for an order
for a blood test, on the basis that otherwise it will be practically impossible for him to prove his case.
It has been accepted in the course of the argument that, if a blood test were ordered, it would not necessarily be conclusive.
As I understand it, there are many cases in which the taking of a blood test proves nothing; but it is said that, if a blood test were
ordered, there is at least a chance that valuable, and possibly conclusive, evidence might be obtained to enable the court to arrive
at the right decision. When it was proposed to the wife that she and the child should be submitted to a blood test, she refused
except on terms which the husband did not feel disposed to accept. In those circumstances the present application has been made
to the court.
The application was dealt with quite shortly by Cairns J. I think that it will be useful to read a brief report of his judgment,
which reads ((1963), 107 Sol Jo 237; [1963] 3 All ER 386):

CAIRNS, J., said that the husband conceded that blood tests might be quite inconclusive but that they might, on the
other hand, show that the child could not have been his and that he ought to have the opportunity of establishing his case.
So far as was known, no such application as that in the present summons had ever been made in England before. It could
not be contended that there was an inherent jurisdiction in the court to order blood tests. Such tests involved the puncture
of the skin and the extraction of blood from the veins. For the court to order such a procedure to be carried out upon an
unwilling person precise statutory authority would be required. 842In his lordships view no such authority existed. The
court could not, therefore, compel the wife to submit to a blood test. This also applied to the child, who was not a party to
the suit and who was too young to consent or object to a test being made upon her.

Accordingly, the summons was dismissed, but leave to appeal was granted.
I think that there can be no doubt that, without an order of the court, what the court is now being asked to order would prima
facie be an unlawful act. It is not suggested that the Matrimonial Causes Act, 1950, the Act which regulates the procedure in this
jurisdiction, confers any statutory power to order a blood test or make an order of the kind sought here. In that connexion I think
it is not altogether irrelevant to refer, as I invited learned counsel this morning to refer, to s 2 of the Road Traffic Act, 1962, which
does make provision, in proceedings against a driver charged with being unfit to drive, for receiving evidence as to the quantity
of alcohol contained in the blood, as ascertained by analysis or measurement of a specimen of blood taken from him with his
consent by a medical practitioner. The section goes on to qualify this new right with a number or rather stringent conditions. I
refer to that section as showing the view which it would appear Parliament takes with regard to a blood test on a person unwilling
to have one. No power is conferred to make an order against a person who does not consent. It is thus clearly recognised, as it
seems to me, by the legislature that without statutory power a test certainly could not be ordered as against an unwilling party.
That prompts the comment that it would be strange indeed if in this jurisdiction, without any express statutory authority, such an
order can be made and can be enforced against a person unwilling to submit to a test.
It has been contended, however, that the court is vested with power to make the order asked for, and the case is put on two
grounds. First, reliance is placed on the inherent jurisdiction of the court: secondly, or in the alternative, it is said that power
exists under rules of court, and reference has been made both to the Matrimonial Causes Rules, 1957, and to certain of the Rules
of the Supreme Court.
I will deal first with the inherent jurisdiction of the court, which it is said enables such an order to be made. Before 1857 the
kind of nullity suit in question here was something quite unheard of. The only suit known to the ecclesiastical courts was a suit
brought on the ground of alleged incapacity or impotence. It is clear that, generally speaking, the ecclesiastical courts before
1857 assumed a power to order medical inspection of the parties in cases of that sort. We have been referred to several cases
going back to the year 1812 where a procedure of that sort appears to have been recognised as appropriate to be adopted. In the
case of men, the examination seems to have been an ordinary medical examination. In the case of women, the old books appear
to show that the appropriate course was to direct an inquiry by a jury of matrons. In either case what was involved can be
described as some sort of medical examination. How far the practice went back in history we have not investigated, but I suspect
that it went back a good deal before the year 1812, which was the date of the first case to which we were referred. It does not
appear to have been a practice very commonly adopted. I base that statement on a remark made by Sir William Scott in Briggs v
Morgan to which we were referred. That case was heard in 1820, and it appears from what was said in that case that the type of
medical examination considered was then something of a rarity. I think it probably became more common later on, because in
the well known case of De v Ag (falsely calling herself De), which was heard in 1845, medical inspection seems to have
been accepted as part of the normal routine. But I venture to emphasise again that in those days the only question within the
jurisdiction of the ecclesiastical courts was that of incapacity or impotence.
843
It has been argued, however, that, as Parliament has extended the grounds on which decrees of nullity can be sought, so the
inherent power of the court must be regarded as being extended so as to enable some sort of medical examination to be ordered in
relation to the other types of nullity suit now available; and in the present case the order for a blood test is sought on the basis that
such a test is merely one type of medical examination. In my judgment that contention is subject to the fatal objection that s 32
of the Supreme Court of Judicature (Consolidation) Act, 1925, precludes reference to the old practice of the ecclesiastical courts
in this respect. That section, which is the successor of a similar section in the Supreme Court of Judicature Act, 1873, which
itself was the successor of another similar section in the Matrimonial Causes Act, 1957, provides as follows:

The jurisdiction vested in the High Court and the Court of Appeal respectively shall, so far as regards procedure and
practice, be exercised in the manner provided by this Act or by rules of court, and where no special provision is contained
in this Act or in rules of court with reference thereto, any such jurisdiction shall be exercised as nearly as may be in the
same manner as that in which it might have been exercised by the court to which it formerly appertained.

It seems to me to be clear from that section that reference to the old practice of the court, to which the jurisdiction formerly
appertained, is only permissible in cases where no special provision is contained in this Act or in rules of court with reference
thereto. In relation to the one type of nullity suit which was known to the old ecclesiastical courts, viz, a suit on the ground of
incapacity, rules of court have now been made dealing with the power to order medical inspection.
That brings me, therefore, to the second part of the argument of counsel for the husband, viz, that which is based on rules of
court. The appropriate rule is r 24 of the Matrimonial Causes Rules, 1957. This is an immensely long rule, and I think I may be
forgiven for not reading it in full. It makes provision in the first place for medical inspection in cases brought on the ground of
impotence or incapacity, ie, cases brought on the only ground known to the ecclesiastical courts; therefore, in relation to that
particular matter, the practice of the court is now governed by rules of court. That, as it seems to me, precludes reference to the
practice prevailing in the old ecclesiastical courts in relation to that type of suit. Rule 24 also deals, however, with one, at any
rate, of the new types of nullity suit, viz, a suit on the ground of wilful refusal to consummate the marriage. Again in that case
power is given to appoint medical inspectors. Oddly enough, the rule in relation to proceedings on the ground of wilful refusal to
consummate the marriage is differently phrased from that in relation to cases on the ground of impotence or incapacity. In
relation to wilful refusal to the rule merely provides that the party concerned shall be at liberty to submit himself for examination
to the inspector. But those are the only two kinds of nullity suit which are dealt with by the rule. The rule is completely silent as
to any form of medical inspection, whether by blood test or otherwise, in relation to cases brought under s 8(1)( d) of the
Matrimonial Causes Act, 1950. That circumstance, it seems to me, is of itself somewhat arresting, bearing in mind that this
ground of relief was introduced by the self-same Act, indeed in the self-same section of the Act, as the ground of wilful refusal to
consummate the marriage. Both grounds were introduced into our law for the first time by the Matrimonial Causes Act, 1937. In
those circumstances it seems to me that not only can the husband get no help out of r 24 of the Matrimonial Causes Rules, 1957,
but the fact that all reference to this type of nullity suit is omitted from that rule is a circumstance which is very much against the
submission now made to us. I make that observation basing myself on the well worn maxim expressio unius est exclusio
alterius.
Nothing daunted, however, counsel for the husband seeks to obtain some 844 assistance from the Rules of the Supreme
Court. He is, of course, entitled to invoke the Rules of the Supreme Court in pursuance of r 82 of the Matrimonial Causes Rules,
1957, but he is entitled to do so only subject to certain limits. Rule 82 says:

Subject to the provisions of these rules and of any enactment, the Rules of the Supreme Court shall, notwithstanding
the provisions of Ord. 68 thereof, apply with the necessary modifications to the practice and procedure in any cause or
matter to which these rules apply.

In those circumstances, before one can look at the Rules of the Supreme Court, one has got to get over the opening words of that
rule, viz, subject to the provisions of these rules and of any enactment. It is contended, however, that, as the Matrimonial
Causes Rules, 1957, are silent with regard to any form of medical inspection in relation to suits for nullity of the type in question
here, help can be sought in the Rules of the Supreme Court.
I do not find the rules to which we were referred very helpful. The two rules to which we were referred were RSC, Ord 50, r
3 and Ord 37A, r 1. The latter deals with the appointment of a court expert to inquire and report on any question of fact or of
opinion not involving questions of law or construction. It seems to me that that is very remote from the kind of application that is
made in this case. Something much more specific would be required to justify the court in making an order against an unwilling
party for a blood test. Nor do I think any greater assistance is to be derived from the provisions of Ord 50, r 3, which says:

It shall be lawful for the court or a judge, upon the application of any party to a cause or matter, and upon such terms
as may be just, to make any order for the detention, preservation, or inspection of any property or thing,

and it goes on to provide that certain experiments may be conducted in relation to such property or thing. When I asked counsel
for the husband what the property or thing was in this case in relation to which he required experiments to be conducted, the
answer which I received was: The blood of the wife and the child. It does not seem to me that a provision relating to any
property or thing is very apt to confer power to take a blood test such as is being asked for in this case.
In those circumstances I do not think that there is any power in the court to make the order asked for. I do not think such
power can be derived from the previous practice of the ecclesiastical courts. There is certainly no direct statutory authority; and
for the reasons which I have given I do not think that any power is contained in any of the rules to make the order asked for.
Accordingly I am satisfied that the judge came to the right conclusion, and in my judgment this appeal should be dismissed.

DANCKWERTS LJ. In my opinion also the judge reached the right conclusion. To compel persons to submit to a blood test
without their consent seems to me a very serious interference with personal liberty and rights. Very convincing reasons would
have to be shown before I could conclude that such a power was within the inherent jurisdiction of the court. For the reasons
which have been stated by my Lord, and which I need not repeat, I am satisfied that such a power in the court does not exist. So
far as the Rules of the Supreme Court are concerned, it is clear to me that they do not extend to and really are not relevant to me
matter which is before us. I agree that the appeal should be dismissed.

DIPLOCK LJ. I too agree. The question in this appeal is not strictly one of the jurisdiction of the court, but of the powers
which the court can exercise in the course of exercising its jurisdiction in a suit for nullity based on one of the 845 grounds first
introduced in 1937 as a ground for nullity of marriage. The Matrimonial Causes Rules, 1957, to which Willmer LJ has referred,
contain elaborate provision for medical inspection in suits for nullity where the suit is based on one or other of two different
grounds, viz, the old ecclesiastical law ground of impotence or incapacity and, one of the new grounds introduced in 1937, of
wilful refusal to consummate the marriage. It contains no provision at all for medical inspection in a suit for nullity based on the
ground relied on in the present case, viz, that the wife was at the time of the marriage pregnant by some person other than the
husband. Counsel for the husband sought to rely on RSC, Ord 50 and Ord 37A but I do not think it necessary to say more in
relation to that than that I entirely agree with Willmer LJ that they have got nothing whatever to do with this sort of thing.
In those circumstances, counsel for the husband is driven to rely on what he calls the inherent jurisdiction of the court,
meaning thereby the practice and procedure which the ecclesiastical courts adopted before 1857. As my Lord has pointed out,
one can rely on that practice and procedure only where no special provision is contained in the Supreme Court of Judicature
(Consolidation) Act, 1925, or rules of court made thereunderwhich have the effect of law. Accordingly, if the matter of
medical inspection is dealt with by rules of court, there is no power to go back to the previous procedure and to rely on that.
Counsel for the husband, it seems to me, in seeking to rely on the previous procedure, is confronted by two difficulties, either of
which would be fatal to him. The first difficulty, to which my Lord has referred, is that at the time before 1857 the only ground
of nullity as to which medical evidence was relevant was impotence or incapacity. It is quite true that the ecclesiastical court had
certain powers (to which I shall advert in a moment) to order inspection of the parties in a suit for nullity on the grounds of
impotence or incapacity, but to order inspection only for the purpose of ascertaining whether the party concerned, who was a
party to the suit, was impotent or incapable. When the Matrimonial Causes Act, 1857, was passed, and subsequently the
Judicature Acts, the only power which the ecclesiastical courts previously exercised to order inspection of the parties, viz, to see
whether a party was impotent or incapable, was dealt with, and exhaustively dealt with, by rules of court, and still continues to be
dealt with by the present r 24, para (1) of the Matrimonial Causes Rules, 1957. In my view, therefore, it is not possible to go
behind that rule and to say that there is some wider inherent power in the court to order inspection in cases not covered by the
rule.
The second difficulty with which counsel for the husband seems to me to be confronted is this. Section 103(1) of the
Supreme Court of Judicature (Consolidation) Act, 1925, provides:

Save as is otherwise provided by this Act or by rules of court, all forms and methods of procedure which, under or by
virtue of any law, custom, general order or rules whatsoever, were formerly in force in any of the courts the jurisdiction of
which is vested in the High Court or the Court of Appeal respectively, and which are not inconsistent with this Act or with
rules of court, may continue to be used in the High Court and the Court of Appeal respectively in the like cases and for the
like purposes as those in and for which they would have been applicable in the former respective courts.

I have already pointed out the first difficulty, that this matter of medical inspection is otherwise provided for by the rules.
Secondly, there is the difficulty that it is only in the like cases and for the like purposes. At the time of the ecclesiastical courts,
as I say, this procedure for medical inspections was available only for the purpose of ascertaining impotence or incapacity.
There remains the third difficulty with which counsel for the husband is confronted. All that he can rely on is the procedure
which was adopted before 1857, and it appears from Shelford On Marriage And Divorce (and is confirmed by a case to which I
shall refer in a moment) that, while the practice of the ecclesiastical 846 court and the power of ordering inspection which it
exercised was, in the case of a man alleged to be impotent, a medical inspection by surgeons, in the case of a woman the only
examination to which she could be compelled to submit was an inspection by matrons, in effect equivalent to the procedure of
inspection by a jury of matrons which used to be followed in the criminal law. That procedure is set out in Shelford On Marriage
And Divorce (1841 Edn), at pp 204 and 205, where it is set out in Latin d. But in Briggs v Morgan, to which we were referred,
Sir William Scott refers to the usual method of proof. The only factual mode of proof in the ecclesiastical court was, as appears
from the report, an inspection by matrons and that was the only compulsory power which the court had in relation to inspection
of a woman. It is suggested that the case of D v A in 1845, some four years after that edition of Shelford, shows that there was a
power to compel a woman to submit to a medical examination as opposed to examination by matrons. While it is true that in that
case the woman did in fact submit to medical examination, there is nothing in the case that I can see which suggests that that was
done otherwise than voluntarily, or that the court had any power to compel it. If, therefore, counsel for the husband can rely at all
(and I do not think that he can) on the procedure applicable before the Act of 1857, what he could ask for is not what he desires, a
blood test, but an inspection by a jury of matrons. I agree that this appeal should be dismissed.
________________________________________
d The passage is quoted in note (x) on pp 204, 205 of Shelford, from Oughtons Ordo Judicorum tit 217

Appeal dismissed.

Solicitors: Gouldens (for the husband); Benjamin & Benjamin (for the wife).

F A Amies Esq Barrister.


[1963] 2 All ER 847

Note
R v Gould
CRIMINAL; Road Traffic

SUSSEX ASSIZES
BRABIN J AND A JURY
2 APRIL 1963

Road Traffic Dangerous driving Causing death by dangerous driving Dangerous driving must be a substantial cause, but
not necessarily the sole substantial cause, of death Road Traffic Act, 1960 (8 & 9 Eliz 2 c 16), s 1.

Notes
As to causing death by dangerous driving, see 33 Halsburys Laws (3rd Edn) 622, para 1047.
For the Road Traffic Act, 1960, s 1, see 40 Halsburys Statutes (2nd Edn) 711.

Cases referred to in judgments


R v Curphey (1957), 41 Cr App Rep 78.
R v Evans [1962] 3 All ER 1086, [1962] 3 WLR 1457.

Trial on indictment
The accused was charged with causing the death by dangerous driving of another motorist with whom he collided in the middle
lane of a three-lane highway. He pleaded not guilty. The case is reported solely with regard to a point of law, raised for the
defence, on which counsel made submissions in the absence of the jury. The point of law was whether on the true construction of
s 1 of the Road Traffic Act, 1960, it was open to a jury to convict of the offence when there was more than one substantial cause
of death and the accuseds conduct was not the sole substantial cause.
847
E J Griew: It follows, in my submission, from R v Curphey that an accused can be convicted of causing death by dangerous
driving under s 1 of the Road Traffic Act, 1960, a, only where his driving was the sole substantial cause of the death, although
there may be other minor causes. Where two motorists are more or less equally responsible for a deat neither can be convicted of
this offence. Had the legislature intended otherwise, words such as causes or substantially contributes would have been used.
Although an accused may be convicted of murder or manslaughter where his act is only one of two or more substantial causes of
death, the present charge is different in kind, and is in effect one of dangerous driving aggravated by consequences which may be
largely fortuitous. There is an irrational element in such an offence, and it should not be extended further than necessary.
________________________________________
a Section 1(1) provides: A person who causes the death of another person by the driving of a motor vehicle on a road recklessly, or at a
speed or in a manner which is dangerous to the public, having regard to all the circumstances of the case, including the nature, condition and
use of the road, and the amount of traffic which is actually at the time, or which might reasonably be expected to be, on the road, shall be
liable on conviction on indictment to imprisonment for a term not exceeding five years

J B R Hazan: The jury in a criminal case should not be called on to decide questions of contributory negligence appropriate
to a civil court. Had a third party been killed in the present case both motorists could have been convicted of causing death by
dangerous driving. It has been decided in R v Evans that, once dangerous driving has been established, questions of degree
become irrelevant. The jury should be directed that it is no defence to show that another motorist was driving equally badly,
provided they are satisfied that the driving of the accused was a substantial cause of the death.

J B R Hazan for the Crown.

E J Griew for the accused.

2 April 1963. The following judgments were delivered.

BRABIN J. I shall direct the jury that the law on this matter is a substantial cause and not the substantial cause in respect of
the offence of causing death.
[Subsequently, the trial having proceeded, the learned judge directed the jury in his summing-up as follows:]

BRABIN J. What is causing death? In this case you may think that the deceased man drove in a manner dangerous to the
public. You may think that, I do not know, for it is you who will decide the matter. But if you find that the accused drove in a
manner dangerous to the public, you may also find that the accident in which the death took place was caused by two persons
driving in a manner dangerous to the public. In order to convict the accused in such circumstances of causing death by dangerous
driving you would have to be sure that his driving was a substantial cause of the death. If it did not measure up to that, to being a
substantial cause of death, then he would be not guilty of causing death by dangerous driving.

Solicitors: Wontner & Sons (for the Crown); Geoffrey B Gush & Co (for the accused).

Mary Colton Esq Barrister.


848
[1963] 2 All ER 849

Gallacher v Gallacher
FAMILY; Divorce

PROBATE, DIVORCE AND ADMIRALTY DIVISION


SCARMAN J
10 APRIL 1963

Divorce Petition Form of petition Presumption of death and dissolution of marriage Prayer for discretion to be included
and discretion statement to be filed in appropriate cases Matrimonial Causes Act, 1950 (14 Geo 6 c 25), s 16(1) Matrimonial
Causes Rules, 1957 (SI 1957 No 619), r 4(4), r 28(1).

Where a petitioner who seeks a decree under s 16(1) a of the Matrimonial Causes Act, 1950, of presumption of death and
dissolution of marriage has been guilty of adultery, the petitioner should include in the petition a prayer for discretion and should
file a discretion statement in accordance with r 4(4) and r 28(1) of the Matrimonial Causes Rules, 1957.
________________________________________
a Matrimonial Causes Act, 1950, s 16(1) provides: Any married person who alleges that reasonable grounds exist for supposing that the
other party to the marriage is dead may, if he is domiciled in England, present a petition to the court to have it presumed that the other party
is dead and to have the marriage dissolved, and the court if satisfied that such reasonable grounds exist, may make a decree of presumption
of death and of dissolution of the marriage.
The proviso to s 4(2) of the Act of 1950 reads, so far as relevant: Provided that the court shall not be bound to pronounce a decree of divorce
and may dismiss the petition if it finds that the petitioner has during the marriage been guilty of adultery

Notes
As to the filing of discretion statement in case of petition for presumption of death and dissolution of marriage, see 12 Halsburys
Laws (3rd Edn) 287, para 565, note (h); and for a case on the subject, see 27 Digest (Repl) 430, 3597.
For the Matrimonial Causes Act, 1950, s 16, see 29 Halsburys Statutes (2nd Edn) 403.
For the Matrimonial Causes Rules, 1957, r 4(4), and r 28, see 10 Halsburys Statutory Instruments (1st Re-issue) 221, 231,
232.

Cases referred to in judgment


Edwards v Edwards 19 December 1946, cited in Practice Note [1948] WN 146.
Parkinson v Parkinson [1939] 3 All ER 108, [1939] P 346, 161 LT 251, 27 Digest (Repl) 372, 3069.

Petition
In this case the wife petitioned for a decree of presumption of death and dissolution of the marriage. The wife married the
husband in 1929 in Scotland. There were four children. In 1939 the husband joined the army. He corresponded with the wife
until the end of 1942 or beginning of 1943. At the beginning of 1943 the wifes and the childrens army allowances ceased to be
paid. The wife wrote to the husband but received no reply. She had not seen or heard from him since that time. In 1950 the wife
started to live with another man and had lived with him ever since. The wife now petitioned for divorce under s 16 of the
Matrimonial Causes Act, 1950, and obtained leave to dispense with service of the petition on the husband. The petition did not
include a prayer by the wife for the exercise of the courts discretion in her favour notwithstanding her adultery.
B Finlay for the wife. In Edwards v Edwards (19 December 1946, cited in Practice Note, [1948] WN 146.) Wallington J not
following Parkinson v Parkinson ([1939] 3 All ER 108; [1939] P 346) held that it was not necessary to include a prayer for the
exercise of the courts discretion or to file a discretion statement in a petition for a decree of presumption of death; in Rayden On
Divorce (8th Edn) at p 291, the view taken was that a prayer for discretion should be included.

10 April 1963. The following judgment was delivered.

SCARMAN J. The requirements for a prayer for the exercise of discretion and for the filing of a discretion statement arise
solely from r 4(4) and r 28(1) of the Matrimonial Causes Rules, 1957. They are not to be found anywhere in the Matrimonial
Causes Act, 1950. The rules draw a distinction between discretion in respect of adultery, and discretion in respect of delay or
other discretionary bars where 849 the prayer for discretion does not have to be in the petition. I think that the practice followed
by Mr William Latey as counsel in Parkinson v Parkinson is correct, and that, under the rules as they now stand, there ought to be
a prayer for the exercise of the discretion and a discretion statement filed. I shall give leave to amend the petition without the
necessity of re-service.
[Counsel then put in the discretion statement. Having heard the evidence, his lordship granted the relief sought.]

Decree nisi.

Solicitors: Walter Bluhm (for the wife).

A T Hoolahan Esq Barrister.


[1963] 2 All ER 850

Re Davenport, ex parte The Bankrupt v Eric Street Properties Ltd and


Another
BANKRUPTCY

COURT OF APPEAL
LORD DENNING MR, HARMAN AND PEARSON LJJ
24 MAY 1963

Bankruptcy Infant Adjudication based on unenforceable debt Whether adjudication should be annulled Bankruptcy Act,
1914 (4 & 5 Geo 5 c 59), s 29(1).

Where an infant has been adjudicated bankrupt on an unenforceable debt, the court has discretion under s 29(1) of the
Bankruptcy Act, 1914, to annul the adjudication; as a matter of discretion, however, the adjudication may well not be annulled if
there are other debts enforceable against the infant.

Notes
As to the position of infants in relation to the bankruptcy law, see 2 Halsburys Laws (3rd Edn) 257, 258, para 477; and for cases
on the subject, see 4 Digest (Repl) 2830, 228252.
As to the annulment of adjudication, see 2 Halsburys Laws (3rd Edn) 354, para 690.
For the Bankruptcy Act, 1914, s 29(1), see 2 Halsburys Statutes (2nd Edn) 362.

Cases referred to in judgment


Re a Debtor (No 564 of 1949), Ex parte Customs and Excise Comrs v The Debtor [1950] 1 All ER 308, [1950] 1 Ch 282, 4
Digest (Repl) 30, 251.

Appeal
This was a motion by way of appeal by Maureen Rosemary Davenport (formerly Briggs), that an order made by Mr Registrar
Cunliffe on 29 January 1963, adjudicating the appellant bankrupt, might be annulled, that the receiving order made on 9 June
1962, might be rescinded and that the petition on which it was founded might be dismissed on the ground, among others, that on
10 August 1960, the date of the underlease containing the guarantee on which judgment was founded and on 26 February 1962,
the date of the judgment, the appellant was an infant. The facts appear from the judgment of Lord Denning MR.

J G Monroe for the bankrupt.


R O C Stable QC for the Official Receiver.
J H Zieger for the petitioning creditors.

24 May 1963. The following judgment was delivered.

LORD DENNING MR. The appellant, a young woman, who was then Maureen Briggs and about eighteen years of age, gave a
guarantee together with her father and mother, guaranteeing the rent of certain premises where a business was being carried on.
The rent fell into arrear, and in February, 1962, 850a writ was issued by the landlord against the tenants and the guarantors,
including the appellant, and on 26 February 1962, judgment was entered accordingly. She was still an infant at that time; she was
under twenty-one and she did not become twenty-one until 16 September 1962. Unfortunately her infancy was apparently not
known to her solicitors: no steps were taken to appoint a guardian ad litem or anything of that kind; and eventually in view of
that judgment debt and the bankruptcy notice, she was made bankrupt. By that time I think she had married and become Mrs
Davenport. Afterwards, when the fact of her infancy became known, an application was made on her behalf of rescind the
receiving order and now she appeals to annul the bankruptcy or to set it aside. It is clear that the debt was unenforceable. It was
not for the infants benefit at all. It was void and could not be ratified.
We have had a discussion on what is the position when an infant is made bankrupt in such a way. It is quite clear that if
there are legally enforceable debts against an infant, then the infant can be made bankrupt. That was settled by the decision of
this court in Re a Debror (No 564 of 1949), Ex parte Comrs of Customs and Excise v The Debtor. Where, however, the debt on
which the petition is founded is an unenforceable debt, then it is equally clear that the order of adjudication can be set aside. It
can be set aside under s 29 of the Bankruptcy Act, 1914, which says:

(1) Where in the opinion of the court a debtor ought not to have been adjudged bankrupt the court may, on the
application of any person interested, by order annul the adjudication.

It seems to me that gives a discretion to the court. It does not mean that where the petitioners debt is an unenforceable debt, the
adjudication is a nullity ab initio. It only means that the court may in its discretion annul the bankruptcy. I can well see that, if
there were other legally enforceable debts, the court might say that the bankruptcy was not to be set aside; but in the present case
there are no other legally enforceable debts. It is a case where the judgment and the bankruptcy have proceeded on a debt which
was unenforceable. In those circumstances under the power given in s 29 of the Act, the bankruptcy ought to be annulled, and all
the proceedings anterior to it. I would allow the appeal accordingly.

HARMAN LJ. I agree and there is nothing I need add.

PEARSON LJ. I agree.

Appeal allowed.

Solicitors: Adler & Perowne (for the bankrupt); Solicitor, Board of Trade (for the Official Receiver); Fruitman & Co (for the
petitioning petitioner).

F G Guttman Esq Barrister.


851
[1963] 2 All ER 852

R v Scaramanga
CRIMINAL; Criminal Procedure

COURT OF CRIMINAL APPEAL


LORD PARKER CJ, HAVERS AND EDMUND DAVIES JJ
29 APRIL, 13, 30 MAY 1963

Criminal Law Indictment Joinder Charges of assault and malicious damage Two persons convicted of joint offences
Offences committed by both independently Whether convictions can stand.

Except where provided by statute, when two persons are jointly charged with one offence, judgment cannot stand against both of
them on a finding that an offence has been committed by each independently (see p 856, letter b, post).
The appellant, who had been drinking in a public house, went with a Mrs S to a nearby restaurant for supper. An argument
ensued between them, in the course of which some damage was done to the restaurant. When the manager asked them to leave,
the appellant grabbed him by the tie and was thereupon forcibly removed from the restaurant. He tried to return but got his arm
trapped in the glass door which was held from the inside by the chef to prevent his re-entry. Mrs S thereupon picked up a flower
pot and threw it through the glass door, which broke, thereby occasioning actual bodily harm to the chef. As a result, the
appellant was able to re-enter the restaurant, where he caused further damage. The appellant and Mrs S were charged jointly on
one count of causing malicious damage (viz, the damage done in argument before the removal of the appellant and damage done
after his return) and on a second count of occasioning actual bodily harm (viz, Mrs Ss throwing the flower pot). They both gave
evidence to the effect that it was the restaurant staff who were responsible, the appellant alleging that he was forcibly ejected for
insufficient reasons and kicked and butted whilst outside. He said that he returned to get his coat, pay his bill and make a
complaint, that his arm was trapped by the door being slammed and that when Mrs S threw the flower pot he fell inside. As to
what happened thereafter, he was acting in self-defence. Mrs S corroborated the appellants account of how he was kicked and
butted outside the restaurant, and said that she threw the flower pot to free him. The jury were directed that if they thought that
this was a joint venture, then they would convict both as being equally responsible. On appeal by the appellant against his
convictions on both counts,

Held (i) The appeal must be allowed and the convictions quashed, because the directions did not distinguish between the
possibility of the individual emotional state of the appellant and of Mrs S leading to their acting jointly, on the one hand, or
independently of each other, on the other hand (see p 854, letter i, post); further, (a) the second count must be quashed, because
there was no evidence that the appellant was acting in concert with Mrs S in breaking the glass of the door and occasioning actual
bodily harm to the chef (see p 855, letter a, post), and (b) the first count must be quashed, because the damage done inside the
restaurant before the appellant was ejected was done in the course of an argument and not in the course of a common design, and
any damage done by him on his return was his own independent act (see p 855, letter b, post).
R v Messingham and Messingham (1830), 1 Mood CC 257, and R v Dovey and Gray (1851), 4 Cox, CC 428, distinguished.
(ii) the proviso to s 4(1) of the Criminal Appeal Act, 1907 a, could not be applied for the purpose of, in effect, substituting
another verdict (see p 856, letter f, post).
852
________________________________________
a The proviso to s 4(1) provides: Provided that the court may, notwithstanding that they are of opinion that the point raised in the appeal
might be decided in favour of the appellant, dismiss the appeal if they consider that no substantial miscarriage of justice has actually
occurred.

Dictum of Hallett J in R v Yindrich ((1953), unreported), applied.
Appeal allowed.

Notes
As to joinder of several offences, see 10 Halsburys Laws (3rd Edn) 391, 392, para 708; and for cases on the subject, see 14
Digest (Repl) 253, 254, 21892212.
As to permissible verdicts, see 10 Halsburys Laws (3rd Edn) 428, para 790.

Cases referred to in judgments


R v Dovey and Gray (1851), 2 Den 86, 4 New Sess Cas 572, 20 LJMC 105, 16 LTOS 539, 15 JP 69, 4 Cox, CC 428, 169 ER 429,
sub nom R v Gray, T & M 411, 14 Digest (Repl) 253, 2196.
R v Hempstead and Hudson (1818), Russ & Ry 344, 168 ER 837, 14 Digest (Repl) 355, 3447.
R v Hurse and Dunn (1841), 2 Mood & R 360, 174 ER 316, 14 Digest (Repl) 90, 520.
R v McVitie [1960] 2 All ER 498, [1960] 2 QB 483, [1960] 3 WLR 99, 124 JP 404, 44 Cr App Rep 201, 3rd Digest Supp.
R v Messingham and Messingham (1830), 1 Mood CC 257, 168 ER 1263, 15 Digest (Repl) 1140, 11,484.
R v Thompson [1914] 2 KB 99, 83 LJKB 643, 110 LT 272, 78 JP 212, 9 Cr App Rep 252, 14 Digest (Repl) 251, 2168.
R v Yindrich 16 June 1953, unreported, Court of Criminal Appeal.Archbolds Crininal Pleading, Evidence and Practice (35th
Edn), para 126, R v Lynn and Debney (1824), 1 C & P 527, OConnell v R (1844), 11 Cl & Fin 155, R v Joachim (1912), 7
Cr App Rep 222, R v Ballysingh (1953), 37 Cr App Rep 28, R v Abbott [1955] 2 All ER 899, [1955] 2 QB 497, R v Martin
[1961] 2 All ER 747, [1962] 1 QB 221, R v Harden [1962] 1 All ER 286, [1963] 1 QB 8.

Appeal
The appellant and Nancy Sapieha were tried at the County of London Sessions before the Chairman (R E Seaton, Esq) on 11
December 1962, being charged on indictment on two counts, (1) that in the night of 24 September 1962, they maliciously
damaged one spirit stove, two bottles of brandy, five bottles of wine, a coffee machine, a table, a door window, two bottles of
liqueur, a flower pot and a wall, the property of Enid Riddell, to the value of 55 18s 10d, and (2) that on 24 September 1962,
they assaulted Bernard Leguin and thereby occasioned him some actual bodily harm. The appellant and Nancy Sapieha were
convicted on both counts. The appellant was sentenced to imprisonment for nine months concurrent on each count; Nancy
Sapieha was not sentenced on 11 December 1962, but on 20 December 1962, was given a conditional discharge and ordered to
pay thirty guineas towards the costs of the prosecution.
This was an appeal by Peter John Scaramanga against his convictions with Nancy Sapieha on the two counts. He also
appealed against his sentence of imprisonment. The facts are set out in the judgment of the court. The case was adjourned for
further argument on Apr 29, 1963, and judgment was reserved on 13 May 1963.

Leonard Lewis for the appellant.


H F Cassel and J G Marriage for the Crown.

Cur adv vult

30 May 1963. The following judgments were delivered.

LORD PARKER CJ read the following judgment of the court: The facts giving rise to this matter are as follows. On the
evening of 24 September 1962, the appellant, who had been drinking in a public house, was joined by Mrs Sapieha and they both
went to a nearby restaurant for supper. According to the prosecution case, they began to argue and to raise their voices. It was
said that he threatened Mrs Sapieha with a knife; that she swore at him; that he threw a glass at her and that she threw the
contents of another glass at him, and that, in 853 the course thereof, some damage was done to the restaurant. When the manager
asked them to leave and not to bother to pay the bill, the appellant grabbed the manager by his tie and was thereupon removed
forcibly from the restaurant. He then tried to return and got his arm trapped in the glass door which the chef was holding from
inside to prevent his re-entry. Thereupon Mrs Sapieha picked up a flower pot and threw it at the glass door which broke,
occasioning actual bodily harm to the chef. As a result, the appellant was able to re-enter the restaurant and, once inside,
smashed up property in the restaurant, including bottles of wine and spirits and a coffee machine. The total damage done both by
Mrs Sapieha and the appellant was in the region of 56, and the chef suffered a small cut above his right eye and a cut two inches
in length on his arm. Both the appellant and Mrs Sapieha gave evidence. The appellants case was that the real persons to blame
for what had happened were the staff of the restaurant; that he was forcibly ejected for no sufficient reason and that, when
outside, he was kicked and butted. He then decided to return, as he said, to get his coat, to pay his bill and make a complaint; that
his arm was trapped by the door being slammed; that he was in great pain and that, when Mrs Sapieha threw the flower pot and
broke the glass of the door, he fell inside. As to what happened when he got inside, he claimed to be acting in self-defence. Mrs
Sapieha corroborated the appellants account of how he got kicked and punched outside the restaurant, and described how, when
his arm got trapped in the door, she threw the flower pot to free him. The appellant and Mrs Sapieha were charged jointly on
both counts, and the prosecution asked the jury to infer that they were throughout acting in concert so as to make the assault and
the malicious damage done to the door by Mrs Sapieha the acts also of the appellant, and the malicious damage done by the
appellant inside the restaurant the act of Mrs Sapieha.
In this appeal, complaint was made as to the direction of the learned chairman in regard to the alleged acting in concert and,
further, it is said that, even on a proper direction, there was no sufficient evidence to support a verdict of being jointly guilty on
these counts. As to the direction, the learned chairman said:

and it is suggested by the prosecution here that this was a joint venture, that there was a common design. What
they say there is that here were two people both excited, so excited that they lost control of themselves and that their
behaviour was such as regards both of them that it resulted in the matters about which you have been listening.

Later, the learned chairman said:

Similarly, of course, if you were satisfied that this was a joint and disgraceful affair you would convict them out of
hand upon that count [that is, of malicious damage] if you thought the damage was inflicted by both in pursuit of this
appalling piece of behaviour and anger at the restaurant. And so with count No. 2 [the assault]: if you thought that this
matter of breaking that window was done in pursuit of this joint escapade, again you may think both were equally
responsible

Finally, he said:
If you are satisfied that this was a disgraceful episode with two people who just lost their heads and deliberately did
these things with the consequence, so far as the throwing of that pot is concerned, of the injuries that Mr. Leguin [the chef]
had, then you will find a verdict of guilty.

In the opinion of this court, these directions were too wide in their terms. That both the appellant and Mrs Sapieha were
guilty of disgraceful behaviour, that they both became excited and both lost self control is undoubted, but the directions do not
distinguish between the possibility of their individual emotional state leading to their acting jointly, on the one hand, or
independently of each other, on the other hand. Before the jury could convict the appellant and Mrs Sapieha on these counts,
they would have to be satisfied that these two people 854 were jointly minded, one with the other, to wreck this restaurant and
inflict harm on the persons who were employed there. Quite apart from this, it seems to this court that the only possible inference
from the evidence was that Mrs Sapieha broke the glass in the door and occasioned actual bodily harm to the chef merely to free
the appellants arm, and that there was no evidence that he was acting in concert with her in doing this. Accordingly, the
conviction of the appellant on count 2 must in any event be quashed.
So far as the joint count of malicious damage is concerned, any damage done inside the restaurant before they were removed
was done in the course of an argument and not in the course of a common design. Further, any damage done by the appellant on
his return into the restaurant was his own independent act, and it could not be said that Mrs Sapieha was acting in concert with
him. There was of course abundant and, indeed, uncontroverted evidence that the appellant and Mrs Sapieha each caused part of
the malicious damage alleged in count 1, and the court adjourned the appeal for further argument whether, on such a joint count
as this, each prisoner could be convicted of independent acts of malicious damage. At the adjourned hearing, counsel for the
prosecution, to whom the court is much indebted for his researches, felt constrained to say that this was impossible. It is
sufficient to refer to only a few of the cases cited to us. In R v Hempstead and Hudson two prisoners, who were employed as
porters by a firm of wholesale stationers, were jointly charged with larceny. On leaving their place of employment one evening
they were stopped and searched. On Hempstead were found twenty-five penknives and on Hudson two, all of which were the
property of their employers. The jury were of the opinion that, although the prisoners were in the same room together, there was
not sufficient evidence to prove that they acted together but, nevertheless, they found the prisoners both guilty. On the question
being reserved, the judges were of the opinion that judgment could not be given against both prisoners. Further, in R v Hurse and
Dunn, the two prisoners each uttered three bad sixpences, made in the same mould, and of the same metal, to shopkeepers living
within a short space of each other. On it appearing that there was no proof that they were together at either of the utterings,
Erskine J called on counsel for the prosecution to elect in respect of which of the prisoners and which of the utterings he intended
to proceed. There being two joint counts, counsel for the prosecution elected to proceed against one prisoner on one count and
the other prisoner on the other count. The case then proceeded, and the jury were directed that, if they thought the prisoners were
acting in concert in the utterings charged, they should convict on the whole indictment, but if they thought they were uttering
independently of each other they might convict one prisoner on one count and the other prisoner on the other. A somewhat
similar position arose in receiving cases. Thus in R v Messingham and Messingham, the defendants, mother and son, were
charged jointly with receiving stolen pork. On the evidence, it emerged that the son first received the pork and then passed it on
to his mother, both knowing at the time that it was stolen. Both prisoners having been found guilty, the question was reserved
whether the mother had been properly convicted. The judges were unanimous in their opinion that, to uphold the verdict, a joint
receipt must be proved and, accordingly, the conviction of the mother was quashed. Again, in R v Dovey and Gray, the jury
returned a verdict of guilty against both prisoners on a joint count of receiving, but stated that they found Dovey received all the
property first and that later Gray received part of the property. It was held on appeal, per Jervis CJ ((1851), 4 Cox, CC at p 429),
that, according to the principle in Messinghams case, the allegation having been satisfied by evidence of a separate receipt by
Dovey, the evidence affecting the woman ought not to have been left to the jury. Accordingly, Grays conviction was quashed.
Indeed, it was probably because of this last 855 mentioned case that the legislature thought it necessary to pass s 14 of the
Criminal Procedure Act, 1851, which is now reproduced in s 44(5) of the Larceny Act, 1916. This provides as follows:

If on the trial of any two or more persons indicted for jointly receiving any property it is proved that one or more of
such persons separately received any part of such property, the jury may convict upon such indictment such of the said
persons as are proved to have received any part of such property.

In our judgment, except where provided by statute, when two persons are jointly charged with one offence, judgment cannot
stand against both of them on a finding that an offence has been committed by each independently. Accordingly, it would seem
that the conviction of the appellant for malicious damage should, prima facie, be quashed. It was, however urged on behalf of the
prosecution that, in the light of Messinghams case and Doveys case, the court should ascertain what malicious damage was first
inflicted and, if the author were the appellant, uphold his conviction on that ground. This court, however, is clearly of opinion
that such a course is not open to them. The cases above referred to were decided on the basis that, once it was shown that A had
received all the property in question, the evidence that B subsequently received that property or part of it ought not to have been
left to the jury. That principle, as it seems to us, has no application to the present case.
Finally, it was urged that the court should apply the proviso to s 4(1) of the Criminal Appeal Act, 1907, on the ground that no
substantial miscarriage of justice has actually occurred. No doubt one of the objects of the proviso was to prevent the quashing
of a conviction on a mere technicality provided, as is undoubtedly the case here, that no embarrassment or prejudice to the
defendant was caused thereby. Reference in this connexion was made to R v Thompson, where the proviso was applied
notwithstanding that the count on which the defendant had been convicted was bad for duplicity, and also the more recent case of
R v McVitie, in which again the proviso was applied notwithstanding a defect in the indictment. We, however, know of no case in
which the court has applied the proviso for the purpose of, in effect, substituting another verdict. The only power in this court to
substitute a verdict is that contained in s 5(2) of the Act of 1907, which power is limited to a case in which the jury could on the
indictment have found the defendant guilty of some other offence. A similar position arose in R v Yindrich. The defendant had
been charged with larceny as a servant, on which charge it was open to the jury to convict, as they did, of embezzlement. It
appeared, however, that the defendant had committed the offence of fraudulent conversion, and it was argued that the proviso
could be applied on the ground that, owing to the technical nature of the distinctions between these offences, no substantial
miscarriage of justice had occurred. In giving the judgment of the court, Hallett J said:

we do not think that in any view it would be right to do that In the first place, it would seem that if we did do that,
we should be doing that which we could not do under s. 5(2) of the Act [of 1907], the very next section; that is to say, we
should be substituting a conviction for another offence for a conviction which resulted from the verdict of the jury.

After reading s 5(2) he went on:

It will be observed that the power of substitution only exists where the jury could on the indictment have found him
guilty of what I will call the substituted offence. Here, as I have pointed out, s. 44 of the Larceny Act did not enable the
jury to find this appellant guilty of fraudulent conversion, and there is no other section which would enable them to do so.
Therefore, we 856 think it would be clearly wrong on any view to do under the proviso to s. 4(1) what we could not do
under s. 5(2).
Accordingly, this court feels constrained, though with considerable reluctance, to quash the appellants convictions on both
counts.

Appeal allowed.

Solicitors: Goodman, Monroe & Co (for the appellant); Solicitor, Metropolitan Police (for the Crown).

N P Metcalfe Esq Barrister.


[1963] 2 All ER 857

Macdonald v Macdonald
FAMILY; Ancillary Finance and Property

COURT OF APPEAL
WILLMER, DANCKWERTS AND DIPLOCK LJJ
16 MAY 1963

Divorce Maintenance Wife Variation of order Back-dating of order No payment made under order Re-marriage of
wife Order varied to nominal sum back-dated to date of re-marriage Whether jurisdiction to back-date order and thereby to
remit arrears Matrimonial Causes Act, 1950 (14 Geo 6 c 25), s 28.

The High Court has jurisdiction to back-date an order under s 28 of the Matrimonial Causes Act, 1950, varying or discharging a
maintenance order, even if that results indirectly in maintenance already accrued due being remitted (see p 859, letter d, post).
Dicta of Scarman J in W v W (No 3) ([1962] 1 All ER at p 740) and of Karminski J in Young v Young (No 2) ([1961] 3 All
ER at p 795) applied.

Notes
As to jurisdiction to vary maintenance orders, see 12 Halsburys Laws (3rd Edn) 444, 445, para 999; and as to claim for
maintenance by divorced wife who re-marries, see ibid, 432, para 970; and for cases on the subject, see 27 Digest (Repl) 622
624, 58035840.
For the Matrimonial Causes Act, 1950, s 28, see 29 Halsburys Statutes (2nd Edn) 414.

Cases referred to in judgment


W v W (No 3) [1962] 1 All ER 736 at p 740, SC sub nom W v W (No 4), [1962] P 131.
Young v Young (No 2) [1961] 3 All ER 793, [1962] P 218.

Appeal
The wife appealed against an order made on 7 March 1963, by Cairns J sitting in chambers, dismissing her appeal from an order
of Mr Registrar Lawton varying an order for maintenance of the wife dated 4 January 1962.
The facts are set out in the judgment of Willmer LJ.

S Seuffert for the appellant, the wife.


Roger Gray for the respondent, the husband.

16 May 1963. The following judgment was delivered.

WILLMER LJ. This is a wifes appeal from an order made in chambers by Cairns J on 7 March 1963, whereby he dismissed
her appeal from an order of Mr Registrar Lawton of Leeds, relating to a matter of maintenance. The registrars order appealed
from was an order for a nominal rate of 1s per annum. At the date when it was made the husband, against whom it was made,
was in arrears to a considerable extent under a previous maintenance order. The order made by the registrar, although made on 1
February 1963, was back-dated to 16 August 1962. The practical effect of the order complained of was, therefore, to remit
arrears outstanding under the previous order. This appeal has been brought on the basis that, as a matter of law, the High Court
has no jurisdiction to 857 remit arrears of maintenance already outstanding. No other ground of appeal is put forward; that is to
say, it has not been argued that, if power exists in law, the power has been wrongly exercised in this case as a matter of discretion.
The relevant facts can, I think, be shortly stated. The parties were divorced, the decree nisi having been made on 25 October
1961, and the decree absolute on 31 January 1962. In anticipation of the decree absolute, there had been proceedings for
maintenance, as a result of which the wife obtained an order for maintenance at the rate of 8 per week from the date of the
decree absolute, the order being made on 4 January 1962. On 16 August 1962, the wife re-married. It appears from an affidavit
sworn by the husband that the wife married a medical practitioner, who was believed to be well able to maintain her. That has not
been contradicted on the part of the wife. The husband had not, as I understand it, made any payments under the order up to then,
and he has not made any payments since. There were, therefore, arrears outstanding at the date of the wifes re-marriage, and
until the further order was made the arrears went on accumulating. We are told that, in fact, the arrears from 16 August 1962, up
to the date of the hearing, amounted to 191. There were proceedings before local justices in Yorkshire, with whom the original
order had been registered, for enforcement of the order. Those took place at some time in September, 1962, and were adjourned
by the justices to enable the husband to take legal advice. The reason for that, I understand, was that it then came to light that the
wife had re-married and that her new husband was in a perfectly good position to maintain her. In the result, these proceedings
were brought by the husband asking for discharge of the maintenance order. The learned registrar, however, as I have said, made
a nominal order of 1s a year and by backdating it to the date of the marriage, in effect remitted the amount of the arrears which
had accumulated since that date.
Our attention has been drawn to the fact that, whereas by s 76 of the Magistrates Courts Act, 1952, power is specifically
given to a court of summary jurisdiction to remit the whole or any part of a sum due under an order, no such power is, at any rate
expressly, conferred on the High Court by the relevant section of the Matrimonial Causes Act, 1950. The relevant section dealing
with variation and discharge of orders for maintenance is s 28 of the Act of 1950. The explanation of this difference of treatment
as between courts of summary jurisdiction and the High Court may well be, as was suggested by Diplock LJ in the course of the
argument, that express power to remit had to be given to the justices for the very reason that they had no jurisdiction to make
retrospective orders, whereas it is not in dispute that the High Court has jurisdiction to make retrospective orders.
In support of the appeal, however, three cases have been brought to our attention. I make this comment on those three cases,
that one of them turned out to be a decision on the facts which, so far as it went, seemed to me to be rather against the wife than
in her favour; the second is wholly against the contention argued for on this appeal; and the third does not seem to have very
much relevance to the question at issue on this appeal.
The first two cases, however, are undoubtedly relevant, and to these I shall now refer. One was W v W (No 3) ([1962] 1 All
ER 736 at p 740; [1962] P 131). That was a decision of Scarman J in which he held that as the court had no express power to
remit arrears of maintenance it would be wrong, on the facts of the particular case, to seek to achieve the same result and get rid
of outstanding arrears by adopting the device of discharging the existing maintenance order. The fact that the learned judge put it
in that way, that it would be wrong on the facts of the particular case to achieve the result by that means, seems to me to indicate
that, in his view at any rate, in an appropriate case, it would be perfectly proper to do it if the facts justified it. The other case was
that of Young v Young (No 2).
858
That was a decision of Karminski J The main point in that case related to the recovery of maintenance said to have been
wrongly paid; but in the course of his judgment, the learned judge dealt with the very point that has arisen in this case. He said
([1961] 3 All ER at p 795; [1962] P at p 222):

I feel no doubt that in exercising its powers to vary maintenance under s. 28(1) of the Act of 1950, the court has power
to go back in date beyond the date of the application to vary. Indeed, on the present application, which was filed on Mar. 7,
1961, Mr. Registrar FORBES discharged the order for maintenance with effect from Sept. 27, 1959 [that is a very
considerable time earlier], and I have no doubt that he had power so to order. But this does not help me to determine the
question of the courts power to order a refund of moneys already paid under this order.

Apart from the fact that, in this case, the learned registrar made a nominal order, whereas in that case he discharged the previous
order altogether, the situation envisaged was precisely the same. It seems to me, therefore, that we have the weight of authority
of these two experienced judges of the Probate, Divorce and Admiralty Division in favour of the proposition that, so far as power
is concerned, the High Court has power to back-date its orders even if that does result indirectly in maintenance already accrued
due being remitted or written-off.
In taking the course which he did in this case, Cairns J as it seems to me, was only following the guidance given by those
two learned judges. In effect, this is, I think, an appeal from the statement made by Karminski J in the passage which I have just
read. I am certainly far from persuaded that Karminski J was wrong in the view he took. I see no reason to restrict the power
conferred on the High Court by s 28 of the Matrimonial Causes Act, 1950, in the way suggested. In my judgment, no grounds
have been shown for saying that the learned judge in the present case came to a wrong conclusion, and I would accordingly
dismiss the appeal.

DANCKWERTS LJ. I agree completely with the judgment that has just been given.

DIPLOCK LJ. So do I.

Appeal dismissed.

Solicitors: Baldwin & Co agents for Willey, Hargrave & Co, Leeds (for the appellant, the wife); Wilkinson, Kimbers & Staddon
agents for W E Clayton-Smith & Son, Pontefract (for the respondent, the husband).

F A Amies Esq Barrister.


859
[1963] 2 All ER 860

Videan and Another v British Transport Commission


TORTS; Negligence, Tortious Liability

COURT OF APPEAL
LORD DENNING MR, HARMAN AND PEARSON LJJ
23, 24, 25, 26, 29 APRIL, 28 MAY 1963

Child Negligence Trespasser Foreseeability Whether duty owed to child injured on railway line adjacent to home at
railway station Negligent driving of trolley by railway employee.

Negligence Rescue Railway Negligent driving of trolley by railway employee Child trespassing on line Stationmaster
killed in rescuing child Liability of railway.

The infant plaintiff, the youngest son of the stationmaster of a small railway station who lived in the station house with his wife
and four children, made his way on to the railway line not far from the barrow crossing for porters with barrows. To be on the
railway line was prohibited to everyone except railway employees on their lawful occasions. At the same moment as the infant
plaintiff was seen on the line by his father, the stationmaster, and a porter, they saw a motor trolley driven by an employee of the
defendants, approaching him along the railway line. The stationmaster and the porter signalled the trolley driver to stop, but he
only slowed down and only at the last moment did he see the infant plaintiff. In an effort to save his son, the stationmaster leaped
from the platform on to the line and saved the infant plaintiff, who was badly injured, but was himself killed instantaneously. In
an action for damages by the infant plaintiff for his injuries, and under the Fatal Accidents Acts, 1846 to 1959, and the Law
Reform (Miscellaneous Provisions) Act, 1934, by the plaintiff widow in respect of the death of her husband, the trial judge found
that the trolley driver was negligent in not keeping a proper look-out, in travelling too fast, and in not applying his brakes hard
enough and soon enough, and dismissed both claims, the infant plaintiffs on the ground that he was a trespasser and the widows
on the ground that a rescuer could be in no better position than the rescued. On appeal,

Held (i) the infant plaintiff was not entitled to recover damages because, on the facts, he was a trespasser on the railway line
(see p 864, letter d, p 868, letter i, and p 872, letter h, post) and the trolley driver owed him no duty since his presence on the line
was not reasonably foreseeable (see p 871, letter h, p 876, letter d, and p 867, letter f, post),
(ii) the widow was entitled to recover compensation in respect of the death of her husband, the stationmaster, because
(a) (per Lord Denning, MR), the occurrence of an emergency (not necessarily the emergency that in fact happened) was
foreseeable by the driver of the trolley, who should have realised that someone might be put in peril if the trolley approached too
fast and without a proper look-out being kept, and accordingly the trolley driver owed a duty to any person who attempted to
rescue another from a danger thus created (see p 868, letters a and b, post).
(b) (per Harman and Pearson, LJJ), the presence of the stationmaster, an employee of the defendants, on the track was
something that was reasonably foreseeable by the driver of the trolley, and accordingly he owed a duty to the stationmaster, who
was not a trespasser on the line (see p 872, letters b and c, and p 876, letters g and h, post).
Semble: if the presence of trespassers on land is known, or reasonably foreseeable, some duty of care is owed by an
occupier of the land or contractor or other person lawfully on the land (see p 866, letter a, p 870, letter i, and p 874, letter h, post);
that duty is (per Lord Denning, MR), a duty to use reasonable care, or (per Pearson, LJ), a duty, substantially less than the duty
owed to a lawful visitor, to treat the trespasser with common 860 humanity (see p 866, letter a, and p 875, letter f, post), but (per
Lord Denning MR, Pearson LJ, differing) this principle applies only where the person alleged to owe the duty conducts
operations on the land (see p 867, letter b, post; cf, p 873, letter g, and p 830, letter e, post).
Robert Addie & Sons (Collieries), Ltd v Dumbreck ([1929] All ER Rep 1); Buckland v Guildford Gas Light & Coke Co
([1948] 2 All ER 1086); Hardy v Central London Ry Co ([1920] All ER Rep 205), and Mourton v Poulter ([1930] All ER Rep 6)
considered.
Per Lord Denning MR, the right of the rescuer to maintain an action for negligence is an independent right, not derived from
that of the victim (see p 867, letter i, post).
Haynes v G Harwood & Son ([1934] All ER Rep 103) and Baker v T E Hopkins & Son, Ltd ([1958] 3 All ER 147)
considered.

Notes
As to the standard of care in relation to trespassing children, see 28 Halsburys Laws (3rd Edn) 17, 18 para 15; and for cases on
the subject, see 36 Digest (Repl) 120, 121, 600611.
As to the effective cause of negligence, see 28 Halsburys Laws (3rd Edn) 2729, para 25; and for cases on the subject, see
36 Digest (Repl) 33, 34, 150157, and 151, 794, 795.

Cases referred to in judgments


Addie (Robert) & Sons (Collieries), Ltd v Dumbreck,[1929] All ER Rep 1; [1929] AC 358; 98 LJPC 119; 140 LT 650; 36 Digest
(Repl) 120, 604.
Baker v TE Hopkins & Son, Ltd, [1958] 3 All ER 147; [1958] 1 WLR 993;on appeal,[1959] 3 All ER 225; [1959] 1 WLR 996,
CA.
Billings (AC) & Sons, Ltd v Riden, [1957] 3 All ER 1; [1958] AC 240; [1957] 3 WLR 496; 3rd Digest Supp.
Bolton v Stone, [1951] 1 All ER 1078; [1951] AC 850; 36 Digest (Repl) 18, 79.
Bourhill v Young, see Hay (or Bourhill) v Young, infra.
Buckland v Guildford Gas Light and Coke Co, [1948] 2 All ER 1086; [1949] 1 KB 410; 113 JP 44; 20 Digest (Repl) 234, 149.
Burchell v Hickisson, (1880), 50 LJQB 101; 36 Digest (Repl) 69, 374.
Commissioner for Railways (NSW) v Cardey, (1960), 104 CLR 274.
Cooke v Midland Great Western Ry of Ireland, [190810] All ER Rep 16; [1909] AC 229; 78 LJPC 76; 100 LT 626; 36 Digest
(Repl) 118, 590.
Davis v St Marys Demolition and Excavation Co, Ltd, [1954] 1 All ER 578; [1954] 1 WLR 592; 3rd Digest Supp.
Donoghue (or McAlister) v Stevenson, [1932] All ER Rep 1; [1932] AC 562; 101 LJPC 119; 147 LT 281; 36 Digest (Repl) 85,
458.
Edwards v Railway Executive, [1952] 2 All ER 430; [1952] AC 737; 36 Digest (Repl) 121, 611.
Excelsior Wire Rope Co, Ltd v Callan, [1930] All ER Rep 1; [1930] AC 404; 99 LJKB 380; 142 LT 531; 94 JP 174; 36 Digest
(Repl) 118, 592.
Farrugia v Great Western Ry Co, [1947] 2 All ER 565; 36 Digest (Repl) 16, 68.
Gallagher v N McDowell, Ltd, [1961] NI 26.
Grand Trunk Ry Co of Canada v Barnett, [1911] AC 361; 80 LJPC 117; 104 LT 362; 36 Digest (Repl) 71, 379.
Hardy v Central London Ry Co, [1920] All ER Rep 205; [1920] 3 KB 459; 89 LJKB 1187; 124 LT 136; 36 Digest (Repl) 120,
603.
Hay (or Bourhill) v Young, [1942] 2 All ER 396; [1943] AC 92; 111 LJPC 97; 167 LT 261; 36 Digest (Repl) 16, 66.
Haynes v G Harwood & Son, [1934] All ER Rep 103; [1935] 1 KB 146; 104 LJKB 63; 152 LT 121; 36 Digest (Repl) 151, 795.
861
Hillen and Pettigrew v ICI (Alkali), Ltd, [1935] All ER Rep 555; [1936] AC 65; 104 LJKB 473; 153 LT 403; 36 Digest (Repl) 71,
377.
Hughes v Lord Advocate, [1963] 1 All ER 705; [1963] 2 WLR 779.
Jenkins v Great Western Ry Co, [191113] All ER Rep 216; [1912] 1 KB 525; 81 LJKB 378; 105 LT 882; 36 Digest (Repl) 64,
346.
Latham v Richard Johnson & Nephew, Ltd, [191113] All ER Rep 117; [1913] 1 KB 398; 82 LJKB 258; 108 LT 4; 77 JP 137; 36
Digest (Repl) 49 262.
Liddle v North Riding of Yorkshire County Council, [1934] All ER Rep 222; [1934] 2 KB 101; 103 LJKB 527; 151 LT 202; 36
Digest (Repl) 120, 607.
Lowery v Walker, [1910] 1 KB 173; 79 LJKB 297; 101 LT 873, CA; revsd [190810] All ER Rep 12; [1911] AC 10; 80 LJKB
138; 103 LT 674, HL; 36 Digest (Repl) 55, 302.
Lynch v Nurdin, (1841), 1 QB 29; 10 LJQB 73; 5 JP 319; 113 ER 1041; 36 Digest (Repl) 33, 150.
Miller v South of Scotland Electricity Board, 1958 SC (HL) 20: [1958] SLT 229; 20 Digest (Repl) 237, *91.
Mourton v Poulter, [1930] All ER Rep 6; [1930] 2 KB 183; 99 LJKB 289; 143 LT 20; sub nom Moulton v Poulter, 94 JP 190; 36
Digest (Repl) 71, 380.
Overseas Tankship (UK), Ltd v Morts Dock and Engineering Co, Ltd, [1961] 1 All ER 404; [1961] AC 388; [1961] 2 WLR 126;
[1961] 1 Lloyds Rep 1 3rd Digest Supp.
Pearson v Coleman Bros, [1948] 2 All ER 274; [1948] 2 KB 359; [1948] LJR 1781; 36 Digest (Repl) 55, 303.
Rich v Commissioner for Railways (NSW), (1959), 101 CLR 135.
Roe v Ministry of Health, [1954] 2 All ER 131; [1954] 2 QB 66; [1954] 2 WLR 915; 3rd Digest Supp.
Slater v Clay Cross Co, Ltd, [1956] 2 All ER 625; [1956] 2 QB 264; [1956] 3 WLR 232; 3rd Digest Supp.
Tebbutt v Bristol and Exeter Ry Co, (1870), LR 6 QB 73; 40 LJQB 78; 23 LT 772; 35 JP 341; 8 Digest (Repl) 104, 681.
Thompson v Bankstown Corpn, (1952) 87 CLR 619; 69 NSWWN 64; 18 LGR 148; 26 Digest (Repl) 350, 248.
Wagon Mound, The, see Overseas Tankship, ante.

Appeal.
This was an appeal by the plaintiffs, Mabel Joy Videan suing as administratrix of Dennis John Videan, deceased, and Richard
Dennis Videan, an infant suing by his mother and next friend, from a judgment of Stephenson J, dated Nov 23, 1962, dismissing
the first plaintiffs claim under the Fatal Accidents Acts, 1846 to 1959, and the Law Reform (Miscellaneous Provisions) Act,
1934, in respect of the death of her husband, Dennis John Videan, and the second plaintiffs claim for damages for injuries. The
facts are set out in the judgment of Lord Denning MR.
The cases noted belowa were cited during the argument in addition to those referred to in the judgments.
________________________________________
a Adams v Naylor, [1946] 2 All ER 241, [1946] AC 543 Searle v Wallbank, [1947] 1 All ER 12; [1947] AC 341, Darling v A-G [1950] 2 All
ER 793; Dunster v Abbott [1953] 2 All ER 1572

DP Croom-Johnson, QC, and Michael Hoare for the plaintiffs.


NR Fox-Andrews, QC, and CWC Ross-Munro for the defendants.

Cur adv vult

28 May 1963. The following judgments were delivered.

LORD DENNING, MR. There is a small country station at North Tawton on the main line between Okehampton and Exeter.
The stationmaster was Mr 862 Videan, who lived in the station house with his wife and four small boys, aged seven, five, three
and two. They had been living there for three years, the youngest being born there. The station house was on the up side of the
line, and their garden and chicken run on the other side. There was a footbridge for passengers and a barrow crossing for porters
with barrows. Mrs Videan used sometimes to take the pram with a child across the barrow crossing, and one of the staff
occasionally took a child across by the hand. But the children by themselves usually went by the footbridge. Mrs Videan said:
that the children were always trained to do that, they were not allowed to run about on the tracks; but they did run out on to the
platformthat was quite another matter. On Sunday morning, July 26, 1959, the stationmaster was having a day off, and Porter
Canniford was in charge. The stationmaster was going to take his family to Exeter for the day. His wife was busy in the house
getting everything ready for their outing. The stationmaster was outside playing with the children until the time came to go.
Suddenly they found that Richard, the youngest, aged two years and two months, was missing. The stationmaster looked into the
door. His wife said to him, Where is Richard? He said, Isnt he with you? She said: Bring him in when you find him. I
want to get him ready. The stationmaster went out, through the booking hall into the booking office, and asked Porter
Canniford, Have you seen Richard? The porter said No. The stationmaster went out on to the platform followed by the
porter. They saw the little boy in the four foot of the up line, a little way from the barrow crossing, and at the same moment, on
that very line, a trolley coming towards him. It was the sort of trolley railwaymen use nowadays for getting about the line, driven
by a petrol motor and capable of considerable speeds. It was then by the home signal, about 120 yards away, coming fast along
the line towards the place where the little boy stood. The stationmaster and the porter held their hands up, signalling the trolley
driver to stop. But still he came on. They ran forward. The stationmaster was in front, running as fast as he could; the porter a
little behind. The trolley driver started to slow down, but not very much. He seemed not to appreciate the danger. Still he came
on. He cannot have seen the child. Only at the last moment did he see him. Only then did he apply his brake. The porter said:
as he got very close, I could see he was really hanging on to the handbrake, pulling as hard as he could. But it was too late.
Than the stationmaster, in a desperate effort to save his son, leaped from the platform on to the line, in the very path of the trolley.
He saved his sonit seems that he pushed him down below the trolleybut he was himself killed in the instant. The little boy
was badly injured. He was five and a half months in hospital, but has recovered a good deal. The trolley driver was named
Souness. He said he was driving at about twenty-five miles an hour; and that, just before he got to the home signal, he looked out
and saw the stationmaster and porter putting their hands up, running along the platform. He thought that they had a message for
him. So he changed down into second gear and applied the brakes lightly. Afterwards, he happened to look on the track and
there he saw the boy. He then applied the brakes as hard as he could. The judge found that, after seeing the men, Souness
travelled for something like two hundred yards before he appreciated that there was an emergency.
The judge found that Souness was at fault in these respects: (i) In not keeping a proper look-out: If Souness had been
keeping a proper look-out, he would have seen the boy before he saw the men. (ii) In travelling too fast: If he had been driving
at a reasonable speed in the wet conditions, he would have slowed down quicker after seeing the men and perhaps would have
been driving more slowly before seeing them. (iii) He did not apply his brakes hard enough soon enough: He should have
applied the brake hard before he did, and so would probably have avoided running down the boy and his father. I take this to
mean that Souness did not take proper heed of the signals of the two men.
863
He should have realised that their signals meant an emergency stopstop at onceand yet he did not do so. The plaintiff widow
now claims compensation under the Fatal Accidents Acts, 1846 to 1959, in respect of the death of her husband; and the infant
plaintiff Richard, by his mother as his next friend, claims damages for his injuries. If they should succeed, the judge has assessed
the widows compensation at 6,348, and the childs at 400. But the judge has rejected both claims. He held that, even though
Souness was at fault, the infant, Richard, has no claim because he was a trespasser; and that the widow, in respect of her
husbands death, has no claim either, because the rescuer can be in no better position than the rescued.
It is, of course, important to decide whether the infant was a trespasser or not; for much of the law depends on it. Before
1957 we always had to inquire whether a person coming on to land was an invitee, a licensee or a trespasser. But we have now
only to inquire whether he was lawfully there or not. If he was lawfully present, the occupier owes him the common duty of care
which is described in the Occupiers Liability Act, 1957. If he was not lawfully there, the occupier owes him the same duty as he
has always owed to a trespasser; for the law as to trespassers has not been affected by the Act of 1957. Counsel for the plaintiffs
asked us to hold that the two-year old infant was not a trespasser. He said that, as one of the stationmasters children, the infant
certainly would have a licence to be on the platform, and his straying a few yards from it on to the line would not make him a
trespasser; and he cited Pearson v Coleman Brothers. I cannot accept this view. It seems to me that, when this infant strayed
from the platform on to the barrow crossing and thence some ten or twelve feet up the line, he was a trespasser; for he went
beyond the bounds of any licence that he had: see Jenkins v Great Western Ry Co, and Hillen and Pettigrew v ICI (Alkali), Ltd
and Edwards v Railway Executive.
It has commonly been supposed that the occupier of land owes no duty towards a trespasser to take care for his protection.
To make the occupier liable, said Lord Hailsham, LC, in Robert Addie & Sons (Collieries), Ltd v. Dumbreck:

There must be some act done with the deliberate intention of doing harm to the trespasser, or at least some act done
with reckless disregard of the presence of the trespasser.

This rule seems fair enough if you put all trespassers in the same bag as burglars or poachers and treat them all alike. But as soon
as you realise that a trespasser may be innocent of any wicked intenthe may be a child too young to do wrong or a grown-up
who has lost his way you find that the rule works most unfairly. Hence the shifts to which generations of judges have been put to
escape the rule. They have time and again turned a trespasser into a licensee so as to give him a remedy for negligence when
otherwise he would have none. Notably, in Cooke v Midland Great Western Ry of Ireland, and Lowery v Walker, where persons
came on to the land without any permission in fact, but were held, by a fiction, to be there by the leave and licence of the
occupier. In recent years, a new way has been found to mitigate the harshness of the old rule about trespassers. It has been done
by observing that it only applies when it is sought to make the occupier liable, as occupier, for the condition of his premises. It
leaves untouched his duty towards his neighbour to conduct his activities with reasonable care. Thus he may be liable as
neighbour for negligence when he would not be liable as occupier. I drew attention to this distinction in 864 a speech which I
delivered in the House of Lords in Miller v South of Scotland Electricity Board (1958 SC (HL) 20 at p 37), and it has been made
manifest by an illuminating trilogy of cases in the High Court of Australia, namely, Thompson v Bankstown Corpn, Rich v
Commissioner for Railways (NSW), and Commissioner for Railways (NSW) v Cardey. But I must needs make it good today.
To do so, let me first consider what is the duty owed by a contractor or anyone else (not being the occupier) who conducts
activities on land towards a person who is lawfully there. It is settled by the highest authority that a contractor doing work on
premises is under a duty to use reasonable care to prevent damage to persons whom he may reasonably expect to be affected by
his work: see AC Billings & Sons, Ltd v Riden, recently applied in an interesting case in Northern Ireland, Gallagher v N
McDowell Ltd in 1961 by Lord McDermott and his brethren. And when he is sued as contractor it is no answer for him to say
that, if he were occupier, he might not be liable. Suffice it that, doing work, as he is, on the land, he must use reasonable care in
the doing of it: seeTebbutt v Bristol & Exeter Ry Co. Let me next consider a case where the occupier instead of employing a
contractor to do the work, himself does it and conducts the activities on the land. Is he under any the less duty than a contractor
would be? Clearly not. There is neither rhyme nor reason why he should be relieved because he does it himself. He is under a
duty to use reasonable care just as a contractor is; and it has been so held by this court in Slater v Clay Cross Co, Ltd. So there is
no distinction in the duty of occupier or contractor towards persons who are lawfully present on the land. But what about the
duty towards trespassers? The cases show that here, also, when you are considering activities taking place on land, no distinction
is ever drawn between the duty of a contractor towards a trespasser and the duty of the occupier towards him. In Addie v
Dumbreck and Edwards v Railway Executive, it was the occupier himself who conducted the activities on the land. In Excelsior
Wire Rope Co Ltd v Callan, and Mourton v Poulter, it was not the occupier but the contractor or someone else who carried on the
activity. But no one has ever suggested that there is a legitimate ground for distinguishing between them. If a contractor is in
some circumstances under a duty to a trespasser, so also in like circumstances should the occupier be. Suppose the Excelsior
Wire Rope Co, Ltd had been occupiers and worked the haulage machinery, just as they did, regardless of the children; or that Mr
Poulter had owned the land and felled the tree, just as he did, without warning the children; surely the result would have been the
same? They could not escape by the plea We are occupiers when they themselves were doing the work.
Seeing, therefore that, in regard to activities on land, no distinction can legitimately be drawn between the duty of an
occupier and the duty of a contractor or anyone else who carries on the activity, the question remains: What is the duty? He is
clearly under a duty towards all persons lawfully on the land. He owes them a duty to use reasonable care in doing his work not
to injure them. But what is his duty towards a trespasser? The answer, I think, is to be found by applying the test of
foreseeability which is so amply established in our law by Donoghue v Stevenson, Hay (or Bourhill) v Young and Overseas
Tankship (UK), Ltd v Morts Dock & Engineering Co, Ltd (The Wagon Mound). The true principle is this: In the ordinary way the
duty to use reasonable 865 care extends to all persons lawfully on the land, but it does not extend to trespassers; for the simple
reason that he cannot ordinarily be expected to foresee the presence of a trespasser. But the circumstances may be such that he
ought to foresee even the presence of a trespasser; and then the duty of care extends to the trespasser also. Childrens cases
afford a good illustration. As I said in Millers case (1958 SC (HL) at pp 37, 38):

he ought to have children in contemplation if he knows that they are in the vicinity or are likelythen or laterto
be attracted to the spot. It does not avail him to say: They are trespassers and wash his hands of all responsibility. He
cannot take refuge in the thought: I am not going to bother about themthey have no permission to be there. He must
bother about them if he knows or ought to know that they are likely to be affected by what he is doing. He must take
reasonable care to prevent injury to them.

Once he foresees their presence, he owes them the common duty of care, no more and no less. I would not restrict it to a duty to
treat them with common humanity, for I do not know quite what that means. I prefer to say that he is to take reasonable care.
This simple test (which is based on foreseeability) is sufficient to explain all the cases, though not all the statements
contained in them. For you must remember that, in applying the test of foreseeability, you have to consider all the circumstances
of the particular case: (i) You must take into account, for instance, the gravity and likelihood of the probable injury. Ultra-
hazardous activities require a man to be ultra-cautious in carrying them out. The more dangerous the activity, the more he should
take care to see that no one is injured by it. Thus, an electricity company which carries highly dangerous current across the
countryside may reasonably be expected to foresee that children may be tempted to trespass near them, and should take steps to
guard against injury, even to them: see Buckland v Guildford Gas Light & Coke Co, and Thompson v Bankstown Corpn ((1952),
87 CLR at pp 628632), per Dixon CJ, and Williams, J (ii) You must take into account also the character of the intrusion by the
trespasser. A wandering child or a straying adult stands in a different position from a poacher or a burglar. You may expect a
child where you would not expect a burglar. Thus, in Cooke v Midland Great Western Ry of Ireland, the company might
reasonably be expected to foresee that children would trespass and meddle with the turn-table; whereas in Grand Trunk Ry Co of
Canada v Barnett, the company could not reasonably he expected to foresee that a man would jump a lift on a train. (iii) You
must also have regard to the nature of the place where the trespass occurs. A public highway is different from private property or
even from a railway line. You may reasonably be expected to foresee that a child may get on to a cart which you leave
unattended in the street (see Lynch v Nurdin): or to run behind a lorry (Farrugia v Great Western Ry Co). But you cannot
reasonably be expected to foresee that children will get on to a railway line, when they are not in the habit of doing so: see
Jenkins v Great Western Ry Co and Edwards v Railway Executive. (iv) You must also take into account the knowledge which
the defendant has, or ought to have, of the likelihood of trespassers being present. Thus, in Excelsior Wire Rope Co, Ltd v
Callan, it was well known to the company that, when the machine was going to start, it was extremely likely that children would
be there. The 866 signalman had only to turn round to see them. Whereas in Addie v Dumbreck, the machine was down a hill
where the children could not be seen and it was not so likely that they would be there: see Mourton v Poulter per Scrutton LJ
([1930] All ER Rep at p 9; [1930] 2 KB at p 190). In short, all the circumstances must be taken into account in order to see
whether the consequence is within the risk created by the negligence: see Roe v Ministry of Health ([1954] 2 All ER 131, at p
138; [1954] 2 QB 66 at 85), quoted by Lord Pearce in Hughes v Lord Advocate ([1963] 1 All ER 705 at p 715).
The principle that I have stated applies only where an occupier or a contractor or anyone else conducts activities on land. It
does not apply where an occupier has done no work on the land; for then his liability is as occupier and nothing else. I am not
disturbed by the suggestion that it is difficult to distinguish between a mans activities on land and the static condition of
premises. I should have thought that, whenever an occupier does things on land, whether he runs a moving staircase, or puts a
bull into a field, or drives a railway engine, or uses land as a cinder tip, or even digs a hole, he is conducting activities on the land
and he is under a duty of care, even to trespassers, if he ought to foresee their presence; and he is none the less under that duty
because he is an occupier. Applying these principles, it cannot be said that the defendants were carrying on a highly dangerous
activity. They were carrying on the simple operation of driving a trolley along a railway line. The railway line was not open to
the public. It was prohibited to everyone except such persons as a porter or platelayer on his lawful occasions. It could not
reasonably be foreseen that a trespasser would be there. Not even a child trespasser could be foreseen, for there is no evidence
that children were in the habit of trespassing there at all. Counsel for the plaintiffs did submit, however, with considerable force,
that, on the particular facts of this case, Souness ought to have realised that the child was there, just as Excelsior Wire Rope Co,
Ltd ought to have realised that the child, Callan, was there. He relied particularly on the judges finding that, after seeing the men
signalling, Souness travelled for something like two hundred yards before he applied his brakes hard. He ought to have
appreciated that there was an emergency and seen the child on the line. I have had some hesitation on this point, but on the
whole, I think that this argument attributes too much foresight to Souness. He could not reasonably be expected to foresee that a
child was there, whereas the man inCallanscase could. I hold, therefore, that the infants claim fails.
I turn now to the widows claim in respect of the death of her husband. In order to establish it, the widow must prove that
Souness owed a duty of care to the stationmaster, that be broke that duty, and that, in consequence of the breach, the stationmaster
was killed. Counsel for the defendants says that the widow can prove none of these things. All depends, he says, on the test of
foreseeability; and, applying that test, he puts the following dilemma: If Souness could not reasonably be expected to foresee the
presence of the child, he could not reasonably be expected to foresee the presence of the father. He could not foresee that a
trespasser would be on the line. So how could he be expected to foresee that anyone would be attempting to rescue him?
Counsel for the defendants points out that, in all the rescue cases that have hitherto come before the courts, such as Haynes v G
Harwood & Son, and Baker v T E Hopkins & Sons, Ltd, the conduct of the defendant was a wrong to the victim or the potential
victim. How can he be liable to the rescuer when he is not liable to the rescued?
I cannot accept this view. The right of the rescuer is an independent right, and is not derived from that of the victim. The
victim may have been guilty 867 of contributory negligenceor his right may be excluded by contractual stipulationbut still
the rescuer can sue. So, also, the victim may, as here, be a trespasser and excluded on that ground, but still the rescuer can sue.
Foreseeability is necessary, but not foreseeability of the particular emergency that arose. Suffice it that he ought reasonably to
foresee that, if he did not take care, some emergency or other might arise, and that someone or other might be impelled to expose
himself to danger in order to effect a rescue. Such is the case here. Souness ought to have anticipated that some emergency or
other might arise. His trolley was not like an express train which is heralded by signals and whistles and shouts of Keep clear.
His trolley came silently and swiftly on the unsuspecting quietude of a country station. He should have realised that someone or
other might be put in peril if he came too fast or did not keep a proper look-out; and that, if anyone was put in peril, then
someone would come to the rescue. As it happened, it was the stationmaster trying to rescue his child; but it would be the same if
it had been a passer-by. Whoever comes to the rescue, the law should see that he does not suffer for it. It seems to me that, if a
person by his fault creates a situation of peril, he must answer for it to any person who attempts to rescue the person who is in
danger. He owes a duty to such a person above all others. The rescuer may act instinctively out of humanity or deliberately out
of courage. But whichever it is, so long as it is not wanton interference, if the rescuer is killed or injured in the attempt, he can
recover damages from the one whose fault has been the cause of it.
I would, therefore, allow the appeal of the widow and dismiss the claim of the infant. The judge assessed the damages at
6,348 for the widow and 400 for the infant. Counsel for the plaintiffs submitted that these figures were too low. I do not think
that we can interfere with the sum for the widow. Judgment should be entered for the full 6,348. There is no need to consider a
sum for the infant for he is, in my opinion, not entitled to any award.

HARMAN LJ. So far as the facts of this case are known, there does not seem to me to be any relevant dispute about them. It is
true that the driver of the trolley gave differing accounts at varying times about the place where he first saw the infant plaintiff on
the line and about where he first applied his brakes. but in the end I do not think that these details matter, for, at any rate before
us, it was conceded by counsel for the defendants that the case must be decided on the footing, first, that the driver was not
keeping an adequate look-out, and, secondly, that he was going faster than he ought prudently to have gone in the circumstances,
which were that the line and the wheels of the trolley were wet, and that stopping took a longer time than would have been
required in dry conditions. What is not known is how the infant plaintiff came to be where he was. There does not seem to have
been any available way by which he could have got on to the platform and so down on to the barrow crossing and the line, and
the most likely theory, as the judge found, was that he crawled somehow under one of the locked gates shutting off the station
yard (on the side of the station buildings away from the line) from the goods yard. If the infant managed to do this, he could have
gone round the goods yard and on to the line at the barrow crossing.
The first question to be answered is whether the infant on the line was a trespasser. If he was not, then he was a visitor
within the meaning of the Occupiers Liability Act, 1957, and the common duty of care defined by that Act would apply to him.
If he was a trespasser, then the Act is inapplicable for it covers only those who are lawfully on the site of the accident. It seems to
me that the infant undoubtedly was a trespasser. It is, I suppose, just possible to hold that he might lawfully be on the platform, if
he could have got there, for there was evidence that the children, or some of them, had been seen on the platform, having got
there, I suppose, through the booking hall, which communicated with the fathers house, but I can find no trace of any invitation
or 868 permission to him to cross the line. There was no evidence at all of any child unaccompanied having been seen to cross
the line.
In Jenkins v Great Western Ry Co, it was held that there was an invitation to children to play on a pile of wooden railway
sleepers near the line, but no evidence that the companys servants knew that children were in the habit of getting on the line, and
it was held that the invitation to play on the sleepers was confined to that spot and did not extend to the line beyond them. The
court distinguished Cooke v Midland Great Western Ry of Ireland, where there was a defined track inviting children to go to the
turntable which was a dangerous object. Cozens-Hardy, MR, held ([191113] All ER Rep at p 220; [1912] 1 KB at p 531) the
case to be founded on the knowledge imputed to the railway companys servants of the user by the children of the particular
place, and Farwell LJ, said ([191113] All ER Rep at p 221; [1912] 1 KB at p 534):

When the invitation or leave and licence depends not on actual but on imputed knowledge, it appears to me that it
must be limited by the knowledge which is proved to exist in the landowner over whose land and on whose land the
accident has taken place. The invitation inCookes case was to play with the turntable which was defective, and by reason
of that defect the child was hurt. In the present case such invitation as has been proved was to play on the pile of sleepers.
I doubt very much if it could be shown to have extended to a child of such tender years so as to relieve the parents of all
responsibility for letting the child go; but I pass that by. The pile of sleepers was not an unnatural place for children to play
on, and one would be sorry to compel railway companies or other landowners to be churlish enough to refuse admission to
children to a place where perhaps it is safer for them away from the high road in these days of the motor car. However that
may be, the only leave and licence in the present case was for the children to play on the pile of sleepers. Beyond the pile
of sleepers there is the siding. There is no finding of the jury about that; and, so far as I follow, there is no evidence to
prove that any child was ever seen playing on the siding.
The lord justice doubted whether there could be such an invitation extended to a child only two and a half years old, and the same
observation is to be found in Latham v R Johnson & Nephew, Ltd ([191113] All ER Rep 117 at p 126; [1913] 1 KB 398 at p
414), where Hamilton LJ, says much the same thing, holding that there might be a condition attached, even if there was such a
licence, that a child so young should be accompanied by an adult. He cites the judgment of Lindley, LJ, in Burchell v Hickisson
((1880), 50 LJQB 101 at p 102), where he said:

The defendant never invited such a person as the plaintiff to come unless he was taken care of by being placed in
charge of others
In Latham v Johnson, the cause of the injury to the child was a stone lying among others in a heap, and this was held not to be an
allurement. I can find no trace of any allurement here, and, indeed, it was not suggested. The investigation, therefore, must start
from the premise that the infant was not lawfully on the line, but was a trespasser.
On this footing, the defendants submit that a child trespasser must take property on which he goes and also normal
operations on that property as he finds them; that this trolley coming down the line was a normal operation on the property and
that this is the end of this part of the case. This view does, 869indeed, obtain some support from the decision inHardy v Central
London Ry Co, where the Court of Appeal held that there was no invitation to a little boy who hurt his hand at the top of the
moving staircase by putting it on the moving band. The Court of Appeal held that, though children frequented the station, the
railway companys servants did all that they could to drive them away, and that the children knew that they were not allowed to
be there. Having found that the little boy was a trespasser, the court concluded, almost as a matter of course, that he had no
rights, because, although there was a moving object on the station, namely, the band on the staircase, still this was a normal
operation which did not constitute an allurement, and, therefore, gave the child no rights.
The cases, however, do show that there may be a duty even to trespassing children in some circumstances. I leave out for
this purpose Excelsior Wire Rope Co, Ltd v Callan, because that case, as it seems to me, had nothing to do with occupiers of
property nor with trespassers, although the children are sometimes so described. The defendants there who brought the
dangerous thing (a wheel with a wire rope running on it) onto the land were not occupiers of the land, but simply had a licence to
put the thing where it was. Having done this, a duty of care arose in them towards people whom they allowed to approach it: see
per Lord Atkin ([1930] All ER Rep at p 5; [1930] AC at p 412). They knew that children habitually played with the rope and
maintained an insufficient watch against them so doing at the moment when it was put into motion. Robert Addie & Sons
(Collieries) Ltd v Dumbreck, which was decided the other way and seems difficult to distinguish on its facts from the Excelsior
case turned, I think, on the older view that occupiers of land had no duty towards a trespasser. I do not for myself concur in the
view that a different degree of liability rests on an occupier and some other persons doing some act on the land. The occupier, if
he does the like acts, has the same responsibility as the outsider. In both cases, foreseeability is the test.
Negligence in law involves a duty: see per Scrutton, LJ in Mourton v Poulter ([1930] All ER Rep 6 at 8; [1930] 2 KB 183 at
p 188). See also Lord Porters observations in Hay (or Bourhill) v Young ([1942] 2 All ER 396 at p 409; [1943] AC 92 at p 116),
where he says this:

In the case of a civil action there is no such thing as negligence in the abstract: there must be neglect of the use of care
towards a person towards whom the defendant owes the duty of observing care. And I am content to take the statement of
Lord Atkin in Donoghue v Stevenson ([1932] All ER Rep 1 at p 11; [1932] AC 562 at p 580) as indicating the extent of the
duty.

The fact, then, that the driver was going too fast or not keeping an adequate look-out will not amount in law to negligence unless
the person injured by these acts was somebody to whom a duty was owed Mourton v Poulter shows that the defendant owes a
duty even to a trespasser not to do any act which will alter the condition of the land to the injury of the trespasser of whose
presence he knows without giving warning. That was a case where men felling a tree failed adequately to warn children who, as
they knew, were watching the operation, that the tree was about to fall, even though the children had no right to be there. The
tree cutter was not the occupier of the land, but that, in my judgment, is irrelevant; the like duty would have been incumbent on
the occupier doing the work himself.
The cases seem to go even further, and find that there may be a liability to a trespasser whose presence, although you do not
know of it, may be probable or suspected. In Buckland v Guildford Gas Light & Coke Co, the defendants 870 were found liable
because they ought to have known that a girl passing by on a neighbouring footpath would be likely to climb a tree which
concealed a dangerous object. This appears in the headnote, para (3):

even if the girl had been a trespasser vis--vis the owner or occupier of the field, she would still in the
circumstances of the case, have belonged to that class of persons whom Lord Atkin had called neighbours, for, although
as a general rule a trespasser on land would not be regarded as being within the class of neighbours, yet the class of
persons who must be regarded as neighbours from the point of view of the defendants was not of rigid necessity the same
as the class of persons who must be regarded as invitees or licensees from the point of view of the owner or occupier of the
land, and in the circumstances of the case the defendants ought reasonably to have had in contemplation that an easily
climbable tree visible to a bright and healthy school girl from a nearby footpath, even though in climbing it she would be a
trespasser vis--vis the owner or occupier of the field, would present an alluring attraction to her.

This case was followed by Ormerod J, in Davis v St Marys Demolition and Excavation Co, Ltd. The Scottish case ofMiller v
South of Scotland Electricity Board was a case decided on the Scottish equivalent of demurrer. The defendant Electricity Board
was not the occupier of the partly demolished house but had left there a live electric cable, with which a child trespassing on the
house had come into contact. This, therefore, was not a case about the rights of an occupier, but observations were made in the
House of Lords which indicate that foreseeability and not occupation was the test of the liability of the defendants. In Hillen and
Pettigrew v ICI (Alkali) Ltd, the appellants were stevedores injured by falling into a barge whose hatch covers gave way. They
were held to be trespassers because the invitation to them did not extend to the use of the hatch covers. Here again, the case
seems to turn on foreseeability, as appears from the speech of Lord Atkin, where he says this ([1935] All ER Rep at p 558; [1936]
AC at p 70):

The barge-owners had no reason to contemplate such a use: they had no duty to take any care that the hatch when
covered was safe for such a use; they had no duty to warn anyone that it was not fit for such use. I know of no duty to a
trespasser owed by the occupier of land other than when the trespasser is known to be present, abstaining from doing an act
which if done carelessly must reasonably be contemplated as likely to injure him, and, of course, abstaining from doing
acts which are intended to injure him.

This part of the case, then, seems to turn on whether it was reasonably foreseeable by the driver that the infant would be on
the line. There seems no doubt that the driver did not in fact see the infant till it was too late; no doubt he did not see that for
which he was not looking. In other words, he had no reason to suppose that the infant would be there. If that be so, he owed the
infant no duty because his presence there was not reasonably foreseeable: if so, his carelessness does not amount to negligence in
law. On this part of the case, I agree with the learned judge.
The fathers case seems to have attracted much less attention than the sons. The judge, I think, decided against the father on
the ground that he could not be in a better position than his son was, and the burden of counsel for the defendants argument was
similar, namely, that, if the trolley driver had no reason to expect the presence of the infant on the line, still less reason had he to
expect to find the father there. I do not think that the two cases stand or fall together like this. These trolleys are not a part of the
regular train service which runs (or ought to run) at stated hours and with the arrival of which the employees of 871 the
defendants must be taken to be familiar. The trolleys are occasional visitors, with no stated times and no warning of their
approach. It is, to my mind, most significant that it is an instruction to trolley drivers that they must approach stations with care.
The inference from this is that they must take care that there are no persons on the line, more especially railway servants engaged
in maintenance and like duties. One of these servants was the dead stationmaster. He was a person whose presence on the track
was well within the contemplation of the driver. He could not be said to be a trespasser. If the infant had suffered nothing and
action had been brought on behalf of the father alone, I do not see what answer the defendants could have to a claim for vicarious
liability for the negligent act of their servant, the trolley driver. The fact that the father acted rather as a father than as
stationmaster seems to me to obscure the issue. The infant might not have been his son but a child of a passenger. It would
clearly be within the scope of the stationmasters employment to take all steps to rescue such a child. It is not necessary that the
exact event should be foreseeable. The presence of the stationmaster, one of the defendants employees, on the track was within
the sphere of contemplation. Whether, if the rescuer had been a member of the public, there would have been liability, I leave out
of account.
It is, perhaps, rather a different point of view to hold that the emergency justified the fathers presence on the line. In the
policemans case, Haynes v Harwood, the policeman dashed into the highway to stop the horse which was a menace to children
on the highway. It may be said to be different in that such children were lawfully on the highway and were not trespassers, but
the emergency is the like and the rescuer has an independent right. Similarly, in the other rescue case, that of the doctor, Baker v
TE Hopkins & Son, Ltd, the emergency was created by two men who had gone down the well in defiance of their employers
instructions. None the less, the court held that, the defendants having created the dangerous state of affairs by using a petrol
pump down the well, the possible injury to the men was within their contemplation, and that they must also be held to anticipate
that a rescuer would behave as he did. Barry J, makes some observations about this ([1958] 3 All ER at p 152):

The original premise is no doubt correct. On a similar principle, the motorist driving along the highway does not owe a
duty of care to the world at large, but only to those whom he might reasonably contemplate would be affected by his
negligence. The essence of the Haynes v Harwood doctrine, as I understand it, is that among the persons who fall within
that class, are those who might attempt to rescue the man or woman placed in acute peril by the negligent act complained
of.

In my judgment, therefore, though the infants action fails, the widows succeeds, and the appeal should be allowed to that
extent.

PEARSON LJ. There has been in this appeal some challenge of the learned judges findings of fact, but, in my opinion, they
were fully justified by the evidence and should be accepted. In order to bring out clearly the questions of law affecting the infant
plaintiffs claim, I will summarise the facts shortly in this way: (i) The infant plaintiff was trespassing on the track. (ii) Up to the
moment when he was seen by the stationmaster and the porter, the presence of the infant plaintiff on the track was not known to,
or reasonably to be anticipated by, the defendants or any of their servants or agents. (iii) Souness, the driver of the trolley, was, in
relation to any person to whom he owed a duty of care, negligent in three respects, namely: (a) In his approach to the station he
was driving too fast, at about twenty-five miles per hour when the proper speed would have been not more than fifteen miles per
hour; (b) he was not keeping 872 a sufficiently careful look-out; (c) he did not respond sufficiently to the attempts of the
stationmaster and the porter to communicate with him by hand signals; if he had been driving with due care and skillespecially
with sufficient alertnesshe would have realised that something was amiss, and he would have promptly taken the precaution of
stopping or slowing down to a very slow speed. (iv) Neither Souness nor anybody else acted in reckless disregard of the
presence of the infant plaintiff. Each of the defendants servantsSouness, the stationmaster,and the porteras soon as he
became aware of the presence of the infant plaintiff, did everything he could to save him from harm. Prima facie, on those facts
the defendants had no liability to the infant plaintiff, because he was a trespasser, and the defendants did not owe him any
ordinary duty of care, and the defendants did not act with reckless disregard of his safety: Robert Addie & Sons (Collieries), Ltd v
Dumbreck; Edwards v Railway Executive.
The principal arguments presented on behalf of the infant plaintiff were that (i) although the defendants as occupiers of the
land might owe no duty of care to a trespasser, in respect of the static condition of the land, yet the defendants as a person
engaged in current 873 operations on the land owed to the trespasser the ordinary duty of care, as one neighbour to another, and
(ii) Souness was not the occupier, and did not enjoy the occupiers partial exemption from liability, and so owed an ordinary duty
of care to the trespasser, as one neighbour to another, and he acted in breach of this duty, and the defendants were victoriously
responsible for his breach of duty. These arguments are based on passages in judgments decided under the law prevailing before
the Occupiers Liability Act, 1957, came into force, when the courts were striving to do justice in particular cases in which the
excessive rigidity of the then existing rules as to occupiers liability was tending to produce injustice. Now that the Act has
provided rules as to occupiers liability which are flexible and in line with the general law of negligence, I think that there is no
longer any need for this branch of the law to be encumbered with artificial distinctions and complexities.
I will first of all consider some prominent features of these arguments, and will then endeavour to define the principles
governing liability to trespassers, so far as is necessary for flee decision of this appeal. First, surely it must be a heresy to suggest
that occupation of land is a ground of exemption from liability. On the contrary, occupation of land is a possible ground of
liability, inasmuch as the occupier by virtue of his right of control may reasonably bear some responsibility for the state of the
land and for what is done on it. There is no good reason for ascribing less liability to the occupier doing work on land than to
some other person doing the same work on the same land. It is true that the trespasser commits his wrong against the occupier,
but the occupier is not entitled to have his revenge by being less mindful of the trespassers safety than some other person ought
to be in the same situation. Secondly, I do not think that there is any sound basis of principle for differentiating sharply between
liability for the static condition of the land and liability for current operations on the land. If the trespasser has to take the land as
he finds it, so, also, he ought to have to take the current operations as he finds them. Normally, a person who does something on
the land is more likely to incur liability than a person who lets things be, but there is no difference of duty involved; conduct,
whether active or passive, gives rise to liability if and only if it constitutes a breach of the duty that is owing (whether that duty be
to show ordinary care, or not to show reckless disregard of a persons safety, or to attain some other standard of conduct).
Moreover, the distinction between static condition and current operations, if it had to be applied as a regulator of liability, would
be very difficult to apply reasonably. For instance, does a moving stairway at an underground railway station, and does a bull
grazing in a field, belong to static condition or to current operations? If a hole is dug on Monday, and on Tuesday somebody falls
into it, is the accident due to the static condition prevailing on Tuesday or to the operations which took place on Monday?
Thirdly, it seems to me that the expression duty to a neighbour is more appropriately used as an aid to ascertaining whether or
not there is a duty of care owing by one person to another rather than as a definition of the content of such a duty. Any person to
whom any duty of care is owing is a neighbour, but the content of the duty varies according to the circumstances. By content of
the duty, I mean the nature and extent of the duty or (if there is any difference) the nature and extent of the conduct, active or
passive, which is required for the performance of the duty. The person whose liability is in question may have insured his
neighbour, or may have invited or permitted him to come on to his land, or done his best, unsuccessfully, to keep him off or to
chase him off the land. Those circumstances may very materially affect the content of the duty.
The trespasser differs from lawful visitors in two main respects. First, he has no right to enter on the land, or, having
entered, to remain there. Secondly, so long as he is an unknown and merely possible trespasser, his presence and his movements
are unpredictable. The lawful visitor, coming and remaining as of right, is expected to come and be there; also he is likely to
come at a normal hour, and to enter by the proper entrance, and to go to, and normally to remain at, the part of the premised
where he has business or with which he is concerned. By contrast, the unknown and merely possible trespasser may come at any
time or may never come at all; if he does come, he may walk, break, creep or climb into the premises at any place and go by any
route to any part of the premises and remain for any length of time. As his presence and movements are unpredictable, he is not
within the zone of reasonable contemplation (Hay (or Bourhill) v Young ([1942] 2 All ER 396 at pp 399, 401, 403, 404, 406, 409;
[1943] AC 92 at pp 98, 101, 104, 107, 116)), and he is not a neighbour (Donoghue v Stevenson ([1932] All ER Rep 1 at p 11;
[1932] AC 562 at p 580)) to the occupier or to any other person working or present on the land, and no precautions can
reasonably be required to be taken by such persons for his safety. In this connexion, all such personsthe occupier himself, his
servants and agents, his licensees, his invitees, and anybody coming on to the land as of rightare in the same position of owing
no duty of care to the unknown and merely possible trespasser. Such persons are entitled to farm lands, operate quarries and
factories, run express trains at full speed through stations, fell trees and fire shots without regard to the mere general possibility
that there might happen to be in the vicinity a trespasser who might be injured. Such persons do not have to cease or restrict their
activities in view of that possibility, which is too remote to be taken into account and could not fairly be allowed to curtail their
freedom of action. InBolton v Stone ([1951] 1 All ER 1078 at p 1082; [1951] AC 850 at p 860), Lord Porter said:

The quantum of danger must always be a question of degree. It is not enough that there is a remote possibility that
injury may occur. The question is: Would a reasonable man anticipate it?

The unforeseeability of trespassing has been referred to and relied on in Grand Trunk Ry Co of Canada v Barnett ([1911] AC 361
at p 369), and in Hillen and Pettigrew v ICI (Alkali), Ltd ([1935] All ER Rep 555 at p 558; [1936] AC 65 at p 69), per Lord Atkin.
If, however, the presence of the trespasser is known to, or reasonably to be anticipated by, the person concerned (whether he be
the occupier, or a servant or agent of the occupier, or his invitee or licensee, or a person coming on to the land as of right), that
person owes some duty of care to the trespasser, because he must treat even a trespasser with common humanity. The trespasser
is a neighbour, though an underprivileged neighbour. It must be said, both on principle and on authority, that the duty of care
owing 874 to a trespasser by the person who knows of, or ought reasonably to anticipate, his presence is radically different from
the duty of care owing to a lawful visitor. The authority will be mentioned later.
The clearest way of demonstrating the difference is by referring to s 2 (2) of the Occupiers Liability Act, 1957. The
common duty of care, owing by the occupier to any lawful visitor, is

to take such care as in all the circumstances of the case is reasonable to see that the visitor will be reasonably safe in
using the premises for the purpose for which he is invited or permitted by the occupier to be there.

If that provision were adapted to apply to the case of a trespasser, the resulting duty would be

to take such care as in all the circumstances of the case is reasonable to see that the trespasser will be reasonably safe
in using the premises for the purposes for which he is trespassing.

That cannot be right; it would plainly be absurd that the occupier or anyone else working or present on the land should have to
make the state of the land and operations or activities thereon safe for trespassers in their trespassing. In this respect also, the
position of the occupier should be no more favourable than the position of anyone else working or present on the land. It is true
that the wrong of trespass is committed only against the occupier and not as against such other persons, but still as against them
the trespasser has no business to be or to remain on the land and should not be able by his trespassing to curtail their freedom of
action, except to the extent that they must treat him with common humanity.
The two principles governing liability to trespassers seem to have emerged clearly. If the person concerned does not know
of, or have good reason to anticipate, the presence of the trespasser, that person owes to him no duty of care because he is not
within the zone of reasonable contemplation and is not a neighbour. If the person concerned knows of, or has good reason to
anticipate, the presence of the trespasser, that person owes to the trespasser a duty of care which is substantially less than the duty
of care which is owing to a lawful visitor, because the duty to a trespasser is only a duty to treat him with common humanity and
not a duty to make the land and operations thereon safe for the trespasser in his trespassing. If those are the principles, there is no
longer any excessive rigidity in the law. Lord Denning MR, in his judgment has pointed out that, in applying the test of
foreseeability, you have to consider all the circumstances of the particular case, taking into account (inter alia) the gravity and
likelihood of the probable injury, the character of the intrusion by the trespasser, the nature of the place where the trespass occurs,
and the state of the defendants knowledge. Also there is flexibility, and some scope for development of the law, in determining
what constitutes breach (or performance) of the duty which is owing to trespassers whose presence is known or reasonably to be
anticipated. There is not (in general at any rate) a duty to cease operations or to change their character; for instance, to cease
felling trees, or to cease keeping cattle on the land, or to reduce the speed of trains to, say, fifteen miles an hour. But it is a breach
of the duty to show reckless disregard of the presence of a trespasser: Lowery v Walker ([190810] All ER Rep 12 at 14; [1911]
AC 10 at p 12) (putting a savage horse in the field without any warning to persons who habitually walked across it: this may
perhaps be explained as a case of licensor and licensee, but it is a good example of reckless disregard); Hardy v Central London
Ry Co ([1920] All ER Rep 205 at pp 209, 210, 212; [1920] 3 KB 459 at pp 467, 470, 473, 474); Addie v Dumbreck ([1929] All
ER Rep at p 4; [1929] AC at p 365):

Towards the trespasser the occupier has no duty to take reasonable care for his protection or even to protect him from
concealed danger. The trespasser comes on to the premises at his own risk. An occupier is in such a case 875 liable only
where the injury is due to some wilful act involving something more than the absence of reasonable care. There must be
some act done with the deliberate intention of doing harm to the trespasser, or at least some act done with reckless
disregard of the presence of the trespasser.

Mourton v Poulter ([1930] All ER Rep 6 at 9; [1930] 2 KB 183 at pp 190, 191) (felling a tree with trespassing children standing
round: duty to give warning before doing a dangerous act); Liddle v North Riding of Yorkshire County Council ([1934] All ER
Rep 222 at pp 226, 227; [1934] 2 KB 101 at pp 109112); Edwards v Railway Executive ([1952] 2 All ER at pp 436, 437, 438;
[1952] AC at pp 745, 746, 747, 748, 749, 750). Lord Goddard said ([1952] 2 All ER at p 437; [1952] AC at pp 747, 748):

I can see no evidence at all of any licence in this case, and the injured child was a mere trespasser It follows,
therefore, that, unless there is evidence of wilful or reckless behaviour on the part of the motorman in failing to stop the
train before it ran over the boy, he can have no case. There is no evidence to support such a charge against the driver. A
man can only act recklessly with regard to the safety of another if he knows of, or has reason to believe in, the presence of
that other.

I do not think that the phrase reckless disregard, illustrative though it is, should be considered as an exhaustive statement
or as an inflexible formula. However, there is no need to pursue this question further in the present case, as it is quite clear that
(i) up to a late stage of this tragic incident, nobody knew or had reason to anticipate the presence of the infant plaintiff on the
track, so that there was until a late stage no duty of care, and (ii) there was not at any stage any reckless disregard of his presence,
or any conduct showing lack of common humanity, or any failure to perform the duty owing to known trespassers, however that
duty should be defined. I would affirm the decision of the learned judge in respect of the claim of the infant plaintiff.
I now come to the appeal of the plaintiff widow, who claims damages for the death of her husband caused, as she contends,
by the negligence of Souness acting as the servant of the defendants. It is clear from the evidence and the learned judges
findings that Souness, in his approach to the station, was acting negligently in relation to anyone to whom he owed a duty of care,
and that the conduct of Souness in this respect caused the accident. The only disputable question is whether Souness owed any
relevant duty of care to the deceased. The defendants argument, evidently accepted by the learned judge, has been that the
position of the rescuer could not be any better than the position of the person rescued, and that, as the infant plaintiffs trespass
was unforeseeable, so the act of his father in trying to rescue him was unforeseeable, and that, therefore, both the infant plaintiff
and his father were outside the zone of reasonable contemplation and the scope of duty. That would no doubt have been a
formidable argument if the deceased had been only a father rescuing his son. But the deceased was the stationmaster, having a
general responsibility for dealing with any emergency that might arise at the station. It was foreseeable by Souness that, if he
drove his trolley carelessly into the station, he might imperil the stationmaster, as the stationmaster might well have some proper
occasion for going on to the track in the performance of his duties. For this purpose, it is not necessary that the particular
accident which happened should have been foreseeable. It is enough that it was foreseeable that some situation requiring the
stationmaster to go on the line might arise, and, if any such situation did arise, a careless approach to the station by Souness with
his vehicle would be dangerous to the stationmaster. On that ground, I hold that Sounesss careless approach to the station was a
breach of a duty owing by him to the deceased as stationmaster, and it caused the accident, and that, consequently, the defendants
are liable to the widow and her 876 appeal should be allowed. I agree that this court has no ground for interfering with the
learned judges assessment of the widows damages at 6,348.

Plaintiff widows appeal allowed. Infant plaintiffs appeal dismissed. Leave to appeal to the House of Lords.

Solicitors: Russell Jones & Walker (for the plaintiffs); MHB Gilmour (for the defendants).

F Guttman Esq Barrister.


[1963] 2 All ER 877

Woodhead v Bates
CIVIL PROCEDURE

CHANCERY DIVISION
UNGOED-THOMAS J
20, 31 MAY, 18 JUNE 1963

Practice Payment out of court Payment to nominees Solicitors on behalf of client Nominees of administrators RSC, Ord
55, r 2(1).

In the Chancery Division payment out of a fund in court may be authorised in the normal course to a nominee of the person
entitled to the fund on production of a signed, written authority by the person so entitled, or without such authority if the judge is
personally satisfied that the nominee has been duly nominated to receive payment; the authorisation of payment to the nominee
should be recorded in the order authorising payment (see p 878, letter i, to p 879, letter a, post).

Notes
As to payment out of court, see 30 Halsburys Laws (3rd Edn) 359, 360, para 669.

Summons
By a judgment dated 9 November 1938, on motion for judgment in default of defence the court declared that a partnership
formerly subsisting between the plaintiff and his father, Joe Woodhead, in a business of haulage contractors was dissolved by the
death of the said Joe Woodhead on 30 July 1936, and ordered accounts and inquiries appropriate to the dissolution of partnership.
On 17 February 1939, it was ordered that the goodwill of the business should be sold and that the proceeds should be paid into
court to the credit of the action. On 22 July 1947, a receiver and manager was appointed to get in the assets of the partnership
business. Pursuant to these orders divers sums were paid into court to the credit of the action, and at the time of the application
next mentioned there were in court funds consisting of 4,152 12s 8d money on deposit, 8,000 British Transport 3 per cent
Guaranteed Stock 1968/73, and 9,559 0s 10d 2 1/2 per cent Consols.
By summons dated 9 October 1962, the plaintiff applied for an order that all proceedings in the action should be stayed and
that the funds in court should be dealt with as provided in a draft payment schedule annexed to the summons, viz, that the money
on deposit and any interest should be paid to a bank for the credit of the joint account of the solicitors for the applicant and the
solicitors for the defendant, and that the stock and consols should be divided into four equal parts and one such part should be
paid to each of four named beneficiaries. By affidavit sworn on 19 November 1962, the plaintiff deposed that no partnership debt
remained unpaid, that the action had been compromised on certain terms which included terms that the costs of both parties to the
action were to be paid and that the residue of the moneys and securities in court were to be divided equally between the four
named beneficiaries.
The plaintiff and the defendant were the administrators of the estate of Joe Woodhead, deceased. The four named
beneficiaries were his four children and were entitled to his estate in equal shares.
His Lordship (Ungoed-Thomas J) gave judgment on the summons of 9 October 1962, in chambers on 31 May 1963, but
subsequently adjourned the matter into open court for delivery of judgment on 18 June 1963.
877

Jeremiah Harman for the plaintiff.


The defendant was not represented on the application.

18 June 1963. The following judgment was delivered.

UNGOED-THOMAS J read the following judgment. This is a partnership action in which an application is made for payment
out of moneys in court standing to the credit of the action. The partnership affairs have been completely wound up, and all that
remains is to secure the payment of the moneys in court to the persons entitled to them. The partners in this case were the
plaintiff and his father, Joe Woodhead. Joe Woodhead died, and the personal representatives of his estate are the plaintiff and the
defendant.
The plaintiff and the defendant have asked the court to provide for payment out in accordance with a payment schedule in
part to National Provincial Bank Ltd at Leeds to the joint account of the solicitors of the plaintiff and the defendant, and as to the
remainder of the funds in court, to the plaintiff, the defendant, and two other named persons in four equal shares.
Difficulty arose about the payment out as requested by the parties in this case, in view of the established practice of the
Chancery Division as stated in the note headed Payment to Nominee in the Annual Practice, 1963, at p 1475. It is there stated
that

it is not the practice to pay out funds, however small, to nominees of persons entitled, in the absence of a deed of
assignment or power of attorney in their favour,

subject to certain exceptions with which we are not concerned in this case.
There are two aspects to the application for payment out requested in this case: first, there is the provision for payment to
the solicitors and, secondly, there is provision for payment to four named persons. I am told that these four persons are in fact
beneficially entitled to the estate of Joe Woodhead, deceased, but I am not concerned with this because the court is not
administering Joe Woodheads estate.
Some difficulty was apparently felt about the payment being made to solicitors without providing for taxation of costs, and
some difficulty was also felt about the other payment being made otherwise than to the administrators of Joe Woodheads estate.
As clients can require the taxation of solicitors costs if desired and as in other litigation (and, indeed, in non-litigious practice)
payment is habitually made to solicitors on behalf of clients, it is a little difficult to appreciate why a person sui juris should not
require payment to be made to his solicitors, if the court is satisfied that the lay client does require payment to be made in such
manner.
Similarly, with regard to the estate of Joe Woodhead, there are before me both the administrators of that estate; they want
payment to be made direct to the four named persons. It seems to me that that is the administrators responsibility. The court
being satisfied that the persons who require payment to be made to their nominees are the persons who are entitled to that
payment (as the administrators are in this case) and it being satisfied that the nominees are truly nominated for that purpose by
the persons so entitled, it is very difficult to appreciate why the persons entitled to the fund should not have payment made in the
way which they desire and to the persons to whom they desire it made.
The practice, therefore, has been the subject of consideration by the Chancery judges, and it has been considered that, in the
normal course, payment out may be authorised to a nominee of the person entitled to the fund no production of a signed, written
authority by the person so entitled, by without such authority if the judge is personally satisfied that the nominee has been duly
nominated to receive payment by the person so entitled. It is desirable, however, to record that the nominee is to receive payment
on the nomination of the person entitled to payment. The Queens Bench Division form of payment schedule itself provides for
such a record, but the Chancery Division form of payment 878 schedulea does not so provide. Accordingly, this authorisation of
payment to the nominees by the person entitled has to be recorded in the order authorising payment.
________________________________________
a See Supreme Court Funds Rules, 1927, r 6, Appendix, Form No 4.

Therefore, I authorise the payment out to the nominees in accordance with the payment schedule in this case, and there is to
be included in the body of the order authorising such payment words to this effect:

The plaintiff in his personal capacity and as one of the joint administrators of the estate of Joe Woodhead, deceased,
and the defendant, as the other administrator of the said testate, being between then entitled to the funds in court to the
credit of this action and the judge being satisfied that the plaintiff and the defendant have nominated the persons named in
the payment schedule hereto the receive the payments thereof appearing opposite their names, order that the fund in court
be dealt with as provided in the said payment schedule.

Then, as requested, I order that all further proceedings in the action by stayed.

Order accordingly.

Solicitors: Ward, Bowie & Co agents for C W Nelson & Co, Leeds (for the plaintiff); Eland Hore Patersons agents for Lupton &
Fawcett, Leeds (for the defendant).

Jacqueline Metcalfe Barrister.


[1963] 2 All ER 879

Ilkiw v Samuels and Others


TORTS; Tortious Liability: CIVIL PROCEDURE

COURT OF APPEAL
WILLMER, DANCKWERTS AND DIPLOCK LJJ
22, 23 MAY 1963

Master and Servant Liability of master Liability to third person for acts of servant Act within course of employment
Authorised act in prohibited manner Lorry driver permitting workman to drive lorry Employers prohibition on allowing
anyone but lorry driver himself to drive.

Pleading Damage Special damage Need to plead Personal injuries Loss of wages between accident and trial not
pleaded as special damage No power to include in general damages.

A lorry driver, W, who had strict instructions from his employers not to allow the lorry to be driven by anybody else, had backed
the lorry into a warehouse in a position under a conveyor belt used for delivering sacks of sugar into the lorry. On completion of
the loading W had to move the lorry forward to enable him to cover the load with a tarpaulin and to make way for any other lorry
needing to load. Notwithstanding his instructions and without making any inquiry as to the workmans competence, he allowed a
workman, S, who was not the servant of Ws employers but had been engaged on loading, to move the lorry, showing him how to
start the engine and remaining in the back of the lorry while S drove it. S had never driven a lorry before; he had no driving
licence and, though he thought himself competent to move the lorry, he was quite incompetent to manoeuvre a heavily loaded
vehicle in the confined space. He drove the lorry a few yards forward but could not stop it when W should to him to stop, and it
ran into the base of two conveyor belts, between which the plaintiff was standing, injuring him. The accident happened in March,
1955. The injuries comprised puncture wounds in the left thigh, causing torn muscles there and leaving a permanent weakness
preventing the plaintiff from doing really heavy work again, and probably some exacerbation of disc lesions of the spine giving
rise to permanent pain. The plaintiff was fifty years of age at the time of the trial (eight years after the accident) and his loss of
earning capacity as result of the accident was put at 200 pa. The judge found the employers 879 liable for the negligence of W
and awarded 4,077 damages, including 77 agreed special damage. The special damage was agreed on the basis of loss of
wages for four months following the accident. No other special damage (eg, in respect of loss of earnings at 200 pa) was
pleaded. In giving judgment, however, the judge said that he must give some effect to the fact that the plaintiff had lost wages in
the past and was going to go on losing them in the future.

Held (i) the employers of W were liable in negligence for the following reasons
(a) (per Willimer and Danckwerts LJJ, Diplock LJ dubitante) their servant, W, was negligent in allowing the workman S to
drive without making any inquiry as to his ability to drive (see p 883, letter e, and p 888, letter a, post; cf p 888, letter f, post), and
(per Diplock and Willimer LJJ) W being employed to take charge and control of the lorry while engaged on the task, the lorry
was driven negligently while in his charge and control (see p 888, letter i, and p 884, letter a; cf p 888, letter b, post); and
(b) Ws negligence arose in the course of his employment, since he was employed to have charge and control of the lorry
while engaged on the task, and at the time of the accident he was so engaged, the prohibition on allowing anyone else to drive
being merely a prohibition on the mode of doing his job, a prohibition of conduct within the sphere of employment, disregard of
which did not take the negligent act of driving outside the course of his employment (see p 885, letters c and d, p 889, letter e,
and p 888, letter c, post).
(ii) the damages must be reduced to 2,077, because on the facts either there had been a wholly erroneous estimate of the
plaintiffs loss or the damages awarded contained a substantial, though concealed, element of loss of earnings between the date of
the accident and the date of the trial (eight years later), which constituted special damage and which could not be recovered, as it
had not been pleaded (see p 887, letter f, p 888, letter d, and cf, p 891, letter c, post).

Notes
As to negligence of drivers of vehicles who are servants or agents, see 28 Halsburys Laws (3rd Edn) 71, 72 para 71; and for
cases on the subject, see 34 Digest 138143, 10831131.
As to a masters liability in negligence where his servant has allowed a stranger to carry out his duty, see 25 Halsburys
Laws (3rd Edn) 541, para 1028; and for cases on the subject, see 34 Digest 127131, 9811006; 138143, 10831131.
As to pleading special damage, see 30 Halsburys Laws (3rd Edn) 12, para 25; and for cases on the subject, see 17 Digest
(Repl) 113, 260267.

Cases referred to in judgments


Canadian Pacific Rail Co v Lockhart [1942] 2 All ER 464, [1942] AC 591, 111 LJPC 113, 167 LT 231, 2nd Digest Supp.
Gwilliam v Twist [18959] All ER Rep 200, [1895] 2 QB 84, 64 LJQB 474, 72 LT 579, 59 JP 484, 34 Digest 142, 1117.
Limpus v London General Omnibus Co (1862), 1 H & C 526, 32 LJEx 34, 7 LT 641, 27 JP 1477, 34 Digest 129, 989.
Marsh v Moores [1949] 2 All ER 27, [1949] 2 KB 208, [1949] LJR 1313, 113 JP 346, 3rd Digest Supp.
Ormrod v Crosville Motor Services Ltd [1953] 2 All ER 753, [1953] 1 WLR 1120, 3rd Digest Supp.
Plumb v Cobden Flour Mills Co Ltd [1914] AC 62, 83 LJKB 197, 109 LT 759, 34 Digest 288, 2415.
Ricketts v Thomas Tilling Ltd [1915] 1 KB 644, 84 LJKB 342, 112 LT 137, 34 Digest 142, 114.
880

Appeal
The third defendants appealed against an order of Phillimore J made at Lincoln Assizes on 8 February 1963, awarding 4,077
damages to the plaintiff in an action for damages for negligence in respect of personal injuries sustained by the plaintiff. The
grounds of appeal included the following: (i) that the judge ought not to have found and was wrong to have found negligence at
common law on the part of the second and third defendants; (ii) that the judge ought not to have found and was wrong to have
found that at the material time the second defendant was acting in the course of his employment by the third defendants and he
ought to have found and held that at the material time the second defendant was acting outside the scope of his employment so
that the third defendants were not liable in law for any negligence on his part; (iii) that the judge drew the wrong inferences from
the facts admitted or proved in the action and failed to draw the right or reasonable inferences from those facts; and (iv) that in all
the circumstances the basis of the general damages awarded by the judge was an excessive and wholly erroneous estimate, and
the judge erred or acted on some wrong principle or misdirected himself in making the award for general damage.
The cases noted belowa were referred to in argument in addition to those cited in the judgments.
________________________________________
a Booth v Mister (1835), 7 C & P 66, Engelhart v Farrant & Co and T J Lipton [1896] 2 QB 240, Beard v London General Omnibus Co
[19003] All ER Rep 112, [1900] 2 QB 530, Fleming v Bank of New Zealand [1900] AC 577, Samson v Aitchison [1912] AC 844, Joseph
Rand Ltd v Craig [1919] 1 Ch 1, Pratt v Patrick [1924] 1 KB 488, SS Horntestrom v SS Sagaporack [1927] AC 37, Powell v Streatham
Manor Nursing Home [1935] All ER Rep 58, [1935] AC 243, Flint v Lovell [1934] All ER Rep 200, [1935] 1 KB 354, Hewitt v Bonvin
[1940] 1 KB 188, Twine v Beans Express Ltd [1946] 1 All ER 202, Anglo-Cyprian Trade Agencies Ltd v Paphos Wine Industries Ltd [1951]
1 All ER 873, Hayward v Pullinger & Partners Ltd [1950] 1 All ER 581, British Transport Commission v Gourley [1955] 3 All ER 796,
[1956] AC 185.

B Caulfield QC and A C H de Piro for the third defendants.


W P Grieve QC and D A L Smout for the plaintiff.

23 May 1963. The following judgments were delivered.

WILLMER LJ. This is an appeal from a judgment of Phillimore J given at Lincoln Assizes on 8 February 1963, whereby in an
action brought by the plaintiff for personal injuries he found in the plaintiffs favour against the third defendants to the action, and
awarded damages amounting to 4,077. The odd 77 represents the amount of agreed special damage. The action arises out of
an accident in which the plaintiff was involved, and which took place at his place of work, viz, the British Sugar Corporation
warehouse at Brigg, on 17 March 1955. I emphasise the date of the accident, as indeed the judge did, for something like eight
years had elapsed between the date of the accident and the date when the case was tried. The plaintiff unfortunately was struck
and injured by a lorry belonging to the third defendants, which at the time was manoeuvring inside the warehouse where the
plaintiff was working, after picking up a load of sugar. The action was brought against three defendants, but in the event the
plaintiff proceeded only against the third defendants. The first defendant was a fellow employee of the plaintiff with the British
Sugar Corporation; he was actually the man who was driving the lorry at the time of the accident, which took place in
circumstances to which I will presently refer. The second defendant was the regular driver of the lorry, and as such an employee
of the third defendants, who were the owners of the lorry. Having now disposed of the first and second defendants, I think that it
may be convenient to refer hereafter to the third defendants simply as the defendants.
There was a considerable dispute how in fact the accident came to occur. I do not go into that because the judge heard the
evidence and came to the clearest possible finding of fact how the accident occurred. For the purposes of this appeal the judges
findings of fact in that respect have been accepted. I can, therefore, summarise the nature of the accident quite briefly. The lorry
had 881 backed into the warehouse into a position under a conveyor belt which was in use for the purpose of delivering the sacks
of sugar to the lorry. The driver, whose name was Waines, was standing on the back of the lorry engaged in stacking the sacks of
sugar in the ordinary course of his work. The moment came when the loading of the lorry had been completed, and it was then
necessary to move it forward away from the conveyor belt for two reasonsfirst, so as to enable Waines to sheet the load with a
tarpaulin, and, secondly, so as to clear the conveyor belt and make room for any other lorry which might want to back in there to
take another load. At this point Samuels (the first defendant, to whom I have already referred), who had been engaged on the
work of loading this lorry, intervened and offered to move the lorry for Waines. Waines allowed him to do so, and went so far as
to tell him how to start the engine of the lorry. He made no inquiry whatsoever as to Samuels competence to drive the lorry, and
(as was very quickly apparent) Samuels was in fact quite incompetent to do so. He did succeed in starting the engine, got into
gear, and moved the lorry a few yards forward, but he then found that he could not stop it; the lorry ran into the base of two
conveyor belts, between which the plaintiff was standing, with the result that it caused injury to the plaintiff. Throughout this
time Waines, the regular driver, remained in the back of the lorry. I should here state and emphasise that Waines was under strict
instructions from the defendants (his employers) that he was not in any circumstances to allow anyone else to drive his lorry.
On those facts the judge came to the conclusion that Waines was guilty of negligence. He said:

I have no doubt that in allowing Samuels to do so [i.e; to drive the lorry] Waines was acting in flat disregard of those
instructions. It is quite clear that this accident was caused by the negligent driving of Samuels; that in allowing him to
drive without inquiry Waines was negligent, and that that negligence caused this accident.

The judge then went on to consider whether, in acting as he did, Waines was acting in the course of his employment so as to
render the defendants liable for his negligence. He referred to a number of authorities that had been cited to him, and came to his
conclusion which he expressed as follows:

I have no hesitation in concluding in this case that, in accepting Samuels offer to drive this lorry a short distance,
Waines was doing a job that he was authorised to do, albeit doing it in an unauthorised way, indeed in a way which was
directly contrary to the instructions of his employer. Moreover, I am satisfied that he gave his permission and remained on
the lorry whilst this manoeuvre was carried out in close conjunction with this man Samuels who was driving it, and it
might be said that he was still in control of the lorry. In those circumstances, in accordance with the principle to which I
have referred, I have no doubt that these defendants are liable for the negligence of Waines. This is the exact equivalent of
a case where an employer tells an employee to drive from A to B, and the employee gets someone else to drive part of the
way. Thus, the employee is doing the job that is authorised, albeit, as I have said, in an unauthorised way. In such
circumstances I think it is clear law that an employer is liable for the negligence of his employee.

Having thus found that the defendants were liable for the negligence of Waines, the judge proceeded to consider the question of
damages. He arrived at the conclusion that the general damages ought to be assessed at 4,000, and, adding the special damage
which had been agreed, he arrived at the result which I have stated.
On this appeal three points have been argued before us. First, it is said that the judge was wrong to find that Waines was
guilty of any negligence. Secondly, it has been contended that, even if Waines was guilty of negligence, his action was not within
the course of his employment so as to render the defendants liable.
882
Thirdly, it has been contended that in any case the damages awarded by the judge were excessive having regard to the nature
of the plaintiffs injuries and all the circumstances of the case. I will deal with those points in that order.
First, as to the question whether Waines was guilty of negligence. This was a matter which it was necessary for the judge to
find if liability was to be fastened on to the defendants, for the defendants could not be liable for the plaintiffs injuries otherwise
than through the negligence of Waines. It is quite clear, as the judge pointed out, that the man Samuels was guilty of negligence,
and there can be no doubt that his negligence was a case, and a substantial cause, of the accident. But the defendants, of course,
cannot be made liable for the negligence of Samuels, who was not their servant. As I have already stated, the judge has found
that Waines, too, was guilty of negligence, and that his negligence was also a cause of the accident. Oddly enough (although one
might be forgiven for thinking that this was a plain question of fact) a considerable number of authorities has been cited to us in
relation to this question. They are mostly cases where, in varying circumstances, vehicles have been permitted to be driven by
unauthorised persons. I hope that I shall not be thought to be lacking in respect for the interesting argument which has been
addressed to us on this aspect of the case if I do not refer to all the cases which have been cited; but I do not think that any
principle of law is involved in this aspect of the present case. The only difficulty here lies in applying perfectly well known
principles of the law of negligence to the somewhat unusual circumstances of this peculiar accident.
Treating it as a question of fact, I am perfectly satisfied that the judge came to the right conclusion, and for the reason that he
gave, viz, that Waines was negligent in allowing, without making any inquiry, Samuels to drive. Certain it is that he made no
inquiry. Had he done so, he might have ascertained (as was the fact) that Samuels had never driven a lorry before; in fact, I do
not think that he had even driven anything before except in a field, and that was some years before the accident with which we
are concerned took place. He had never driven on the road, and he held no driving licence. Yet here he was being invited to
drive a very heavily loaded lorry, in a confined space, where the difficulties of manoeuvring might well have taxed the skill of
even an experienced driver, in circumstances in which there were clearly obstructions such as the conveyor belts on the
warehouse floor, and where there were men moving about. It is difficult to imagine anything more dangerous than allowing an
unknown man to take on the job of moving a heavily loaded vehicle like this in such circumstances. It seems to me that, not
having inquired, Waines can be in no better position than he would have been if he had known of Samuels incompetence.
As it appears to me, that is sufficient to dispose of this aspect of the case. An interesting problem might well have arisen as
to what the result might have been if Samuels had in fact been a competent driver who had happened to drive the lorry
negligently on this occasion. That particular problem is not before us in this case; but I think it may very well be that even in that
event Waines could be held to be guilty of negligence on the ground stated in Ricketts v Thomas Tilling Ltd. That was a case
where an omnibus driver wrongly allowed his conductor to drive the omnibus, and actually sat alongside the conductor when he
did it. The conductor drove negligently, with the result that an accident happened. The employers were held liable, not on the
ground of the conductors negligence, but on the ground of the drivers negligence. The way it was put by Buckley LJ was this
([1915] 1 KB at p 646):

It seems to me that the driver, who was authorised to drive, had the duty to prevent another person from driving, or, if
he allowed another person to drive, to see that he drove properly.

Substantially to the same effect was this observation made by Pickford LJ ([1915] 1 KB at p 650):
883
It seems to me that the fact that he allowed somebody else to drive does not divest him of the responsibility and duty
he has towards his masters to see that the omnibus is carefully, and not negligently, driven.
It may be, therefore, that on that ground also it would be proper in this case to hold that Waines was guilty of negligence; but that
he was, and was rightly so found by the judge, I have no doubt.
I come then to the second question, viz, whether the negligence of Waines was such as to render the defendants liable. That
depends on whether, when he behaved as he did, he was acting in the course of his employment. This is a familiar problem in the
law, and the general principle is stated in Salmond On Torts (13th Edn), art 36, at p 122:

A master is not responsible for a wrongful act done by his servant unless it is done in the course of his employment. It
is deemed to be so done if it is either (i) a wrongful act authorised by the master, or (ii) a wrongful and unauthorised mode
of doing some act authorised by the master.

That is the general principle involved. There is a good deal of authority on the subject, much of which was brought to the
attention of the judge. He in fact directed himself in accordance with the statement of principle given by Lord Thankerton in
delivering the judgment of the Privy Council in Canadian Pacific Rail Co v Lockhart. In the course of giving that judgment Lord
Thankerton himself quoted ([1942] 2 All ER at p 467; [1942] AC at p 599) a lengthy extract from Salmond On Torts, a passage
following immediately after that which I have already quoted. The judge has himself repeated the whole of that extract in his
judgment, and in those circumstances I do not think that anything is to be gained by my reciting it again in the course of my
judgment.
As I have said, a number of other cases was cited, both in the court below and before us. I content myself with referring to
two of them only, and I restrict myself to two because I cite them only for the statements of general principle contained therein,
recognising (as I do) the danger of referring to other cases merely because they are similar to the present case on the facts. In
Marsh v Moores there is a most useful statement of the principle involved by Lynskey J ([1949] 2 All ER at p 31; [1949] 2 KB at
p 215). He there said:

It is well settled law that a master is liable even for acts which he has not authorised provided that they are so
connected with the acts which he has authorised that they may rightly be regarded as modes, although improper modes, of
doing them. On the other hand, if the unauthorised and wrongful act of the servant is not so connected with the authorised
act as to be a mode of doing it, but is an independent act, the master is not responsible, for in such a case the servant is not
acting in the course of his employment but has gone outside it.

The other case to which I would refer is the very old and well known case of Limpus v London General Omnibus Co, and I
refer to that case for a statement of principle by Willes J ((1862), 1 H & C at p 539). That case, it will be remembered, is the case
familiar to students in which, completely contrary to his express instructions, an omnibus driver so drove his omnibus as to
obstruct an omnibus belonging to a rival company, with the result that an accident happened. Willes J said ((1862), 1 H & C at p
539):

It may be said that it was no part of the duty of the defendants servant to obstruct the plaintiffs omnibus, and
moreover the servant had distinct instructions not to obstruct any omnibus whatever. It my opinion those instructions are
immaterial. If disobeyed, the law casts upon the master a 884 liability for the act of his servant in the course of his
employment; and the law is not so futile as to allow a master, by giving secret instructions to his servant, to discharge
himself from liability.

I refer to that simply as authority for the proposition that the mere fact that the act complained of was done in disobedience to
express instructions is of no necessary materiality in deciding whether or not the act was within the course of the employment. It
seems to me that those authorities to which I have referred leave no doubt whatsoever as to the principle which is involved in this
case. Once again, as is not uncommon, the difficulty is to apply the principle to the unusual facts of the case.
In the end I have come to the conclusion without any hesitation that the judge was right in the conclusion at which he
arrived, and for the reasons which he gave. The driver of the vehicle, Waines, as I see it, was employed, not only to drive, but
also to be in charge of his vehicle in all circumstances during any such times as he was on duty. That means to say that, even
when he was not himself sitting at the controls, he remained in charge of the lorry, and in charge as his employers representative.
His employers must remain liable for his negligence so long as the vehicle was being used in the course of their business. As I
understand the authorities, the employers escape liability if, but only if, at the time of the negligent act, the vehicle was being
used by the driver for the purpose of what has been called a frolic of his own. That is not this case. Here, at the material time,
this vehicle was in fact being used in the course of the defendants business. In those circumstances it appears to me that there is
no ground on which the defendants can escape liability for Waines negligence. So far as liability is concerned, therefore, I arrive
at the same conclusion as that at which the judge arrived.
I pass, therefore, the consider the question of damages. I hope that it is not necessary for me to do more than summarise the
injuries sustained by this unfortunate plaintiff. The judge has described the injuries in some detail; and, although I am mindful of
the fact that his statement of the plaintiffs injuries has been criticised on behalf of the defendants, I do not think it is necessary
for me to do more than summarise them. The immediate injuries caused by the accident appear to have consisted of what have
been described as puncture wounds in the left thigh. These in turn resulted in tearing of the muscles of the thigh, with a
consequent wasting of the whole thigh. There has resulted some degree of permanent weakness in that left thigh, and it is, I
think, important to observe that the judge said that he saw the demonstration which the plaintiff gave in the witness-box, and was
satisfied that a very considerable degree of weakness did exist in the muscles of the left thigh. The judge was satisfied that, as the
plaintiff contended, that weakness in the thigh must prevent him from ever doing really heavy work again. I emphasise the
importance of heavy work in this case because, of course, that is what he was engaged on before the accident; his job was to
handle these bags of sugar, the weight of which might run up to as much as two hundredweight.
Over and above that injury to the left thigh, there was at least a suggestion (and indeed rather more than a suggestion) that
the accident resulted in an injury to the plaintiffs spine in the shape of disc lesions. Unfortunately, owing to the way in which
the case developed and the way in which the medical reports were obtained, it never became possible to say whether these disc
lesions were the result of a pre-existing weakness in the mans spine, or whether they were directly due to the accident. Although
it could not be stated for certain, it seems to have been accepted that there was a strong probability that they were at any rate
exacerbated by the accident. The effect of the injury or damage to the spine has been that there is not only permanent weakness,
but also some degree of permanent pain which, as I understand it, is transmitted down the whole of the left leg through the sciatic
nerve. As I follow it, anybody suffering 885 from that disability is never wholly without pain, so that in the case of this plaintiff
the pain from which he suffers is probably with him for the rest of his life.
Owing to the weakness of the left leg, and the inability to do heavy work, it must, I think, be obvious that there is a loss of
earning capacity in the case of this plaintiff. That was put as being equivalent to about 200 a year, and the judge seems rather to
have accepted thatat least he does not find against itas a sort of measure of the mans disability. But I think that it is to be
remembered that the plaintiff is now a man fifty years of age; and, although a certain amount of guesswork must necessarily be
involved, I think that it can probably be said with a fair degree of safety that there could be only a limited number of years in the
future during which in any case he could have gone on doing the kind of heavy work which he had been in the habit of doing
before the accident. It appears to me that in the not too distant future the time would in any case come when there would be
bound to be a falling off in the earnings of the plaintiff. On the other hand, if it is right to connect the spinal injury with the
accident (as the judge was inclined to do), I should be disposed to think that the degree of pain, and the permanence of the pain
which is to be expected, would be something which of itself would be sufficient to warrant a substantial award of damages.
The case, however, is by no means easy. For one thing, it has been pressed on us that the judge has put the plaintiffs
injuries rather too high; he has apparently accepted as certain that the spinal injury was in fact caused (or at least exacerbated) by
the accident. He said in terms:

I have no doubt whatever that the major trouble from which this man suffers, i.e., the pain which he gets all down his
leg, and which he says he is never without even when in bed, is due to this accident, at any rate to the extent that it has been
exacerbated by it.

As to that, it is said that on the basis of the medical reports that is putting it much too high; the doctor who examined the plaintiff
not very long before the trial was not prepared to be anything like so dogmatic as to any connexion between the accident and the
present state of this mans back. Up to a point, I think that that criticism is well founded. On the other hand, having regard to the
nature of the accident and its serious character, it does seem to me that there is quite a high degree of probability of spinal injury
in the case of a man struck in this way. Although, therefore, the judge may possibly have put the matter too high, and been a little
to dogmatic about it, I do not think it is possible to disregard the spinal injury, because, as it seems to me, there is at least a
probability that that was in part at least caused by the accident; it certainly accounts for some of the trouble from which this man
now suffers.
Another, and to my mind rather worse, difficulty arose out of the way in which the case was conducted. I have already
indicated that special damage was agreed at 77. That special damage was agreed on the basis of a loss of wages for a period of
four months following the accident. No other claim for special damage was ever put forward in all the eight years between the
accident and the time when the action was tried. Inspite of that, however, when the case came to trial, some evidence was
admitted (although some mild objection was registered) to show that, throughout the eight years that had elapsed since the
accident before the trial, this man had been suffering from loss of earnings to the extent of about 200 a year, and also that in the
course of those eight years he had had no less than 466 days off work for the purpose of receiving treatment for his injuries and
the results of his injuries.
The judge quite rightly in the course of the evidence expressed the view that this alleged loss of earnings in the past, not
having been pleaded by way of special damage, could not be treated as such. He said, for instance, at p 21 of the transcript, in the
course of the evidence given by Mr Bodin:
886

As far as that is concerned, I am not going to have it put as special damage. If I give effect to it, I shall give effect to it
in a different way.

When, however, the judge came to give judgment, this is what he said, and it is this passage which, I think, is open to a good deal
of objection; it has certainly caused me difficulty. Having set out the plaintiffs injuries, the judge then went on as follows:

I have to consider what is a proper sum to compensate him for his injuries. The defendants, although put in very
considerable difficulty, as I have said, by the failure to plead matters of special damage which ought to have been so
pleaded, have not sought an adjournment, and I must deal with this case broadly on the material before me. I think, in
assessing the general damages, I must give some effect to the loss this man has sustained due to his injuries in the course of
treatment which he has had to have from time to time, and also to the fact that he is undoubtedly losing wages as a result
has lost them in the past, and is going to go on losing them in the future.

If I thought that, in coming to that result, the judge was doing no more than taking into consideration this mans over-all loss of
earning capacity, past, present and future, as merely one factor to be considered amongst all the other factors in awarding a global
sum by way of general damages, I should certainly have hesitated before saying that we ought to interfere with that approach.
But the case which has been put forward (and which, I think, succeeds) is that, whatever the judge may have said, he must in
effect have allowed substantially the whole of the loss of wages claimed over the past eight years as a quantified sum. What is
said is that he has in effect awarded that as though it were special damage which had been properly pleaded. I am driven to the
conclusion that the judge must be done something substantially like that, for I find it difficult to see how otherwise he could
possibly have arrived at the figure of 4,000 which he awarded by way of general damages. As I have already indicated, this
does not strike me as a case in which there is much room for any large sum to be awarded in respect of future loss of earnings,
having regard to the age which this plaintiff has already attained. In the circumstances, as it appears to me, the very large figure
which the judge arrived at must either have contained a very substantial (although concealed) element of past loss of earnings,
which were never pleaded as special damage, or else the figure awarded is a wholly erroneous estimate of the plaintiffs loss. On
either view, this court would be not only justified in interfering, but bound to interfere.
Having regard to what I think was the error in the judges approach, it seems to me that the question of damages is at large,
and it is for this court to award a new and proper figure which will fairly compensate the plaintiff for the loss which he has
properly proved. Taking the view that I do that the judge has rather exaggerated the seriousness of this mans injuries and the
degree of probability that the main trouble from which he now suffers was directly attributable to the accident; and having regard
also to the view which I have formed that the judge (whatever he has said) has in fact included a large amount in respect of past
loss of earnings, I think that the proper course is to reduce the general damages awarded by half, and I would award the sum of
2,000. On that ground, therefore, in my judgment the appeal should be allowed.

DANCKWERTS LJ. I agree. The principles which have to be applied in cases of this type are really well established. The
clear statement in Salmond On Torts has been approved more than once. But, as has often been pointed out, the real difficulty
is to apply those principles to the facts of a particular case, or to decide which part of the principles there stated are to govern the
application of the facts in the particular case. Waines was not supposed to allow another person to drive, and he had strict orders
on that point. The prohibition, of 887 course, is one of the circumstances that is not necessarily enough to exonerate the
employer from liability for his mistakes. Waines was supposed to be in control of the vehicle. To allow a person whose ability to
drive was unknown to him, a person whose English was not entirely satisfactory, and who (he must have known) had no
familiarity with the particular lorry in question, seems to me plainly negligent; for a person who was unfamiliar with such things
as the brakes, the clutch and other features in a confined space appears to me very plainly a most undesirable person to drive in
those circumstances.
The position on the lorry in which Waines was, rendered it very difficult for him to control the man who was driving in fact,
and yet it is clear that he must be held responsible for the mans acts so that, if mistakes are made, they are the mistakes of
Waines. Waines was not supposed to let the other man drive, but it is clear that he was using the lorry in the course of the
employment in which he was engaged; and it seems to me that it cannot be regarded as something outside the employers
liability. They must be responsible for the acts done in the present case since it was a way of doing the job for which Waines was
employed, though it was in the circumstances an unauthorised way of doing it. Accordingly, it seems to me that the defendants
cannot escape liability in the present case.
On the other point as to the amount of the damages, I agree with what has been said by Willmer LJ and I do not desire to add
anything on that point.

DIPLOCK LJ. The judge found against the defendants in this case on two grounds. The first was that the defendants servant,
Waines, was negligent in permitting Samuels to drive the defendants lorry without making inquiries as to Samuels ability to
drive, and that this negligence caused the accident. The second ground was expressed as follows:

Moreover, I am satisfied that he [i.e., Waines] gave his permission and remained on the lorry whilst this manoeuvre
was carried out in close conjunction with this man Samuels who was driving, and it might be said that he [i.e., Waines] was
still in control of the lorry.

I myself might have found difficulty in upholding the judgment on the first ground for several reasons. I am not wholly
convinced that, in an age when most adults have experience in driving motor vehicles, it shows a lack of reasonable care not to
ask a person, who himself volunteers to move a lorry a yard or two, whether he is competent to do so. I think a reasonable man
might assume that the offer would not be made by someone who did not regard himself as competent to do so. But, even if it
showed lack of reasonable care to fail to make the inquiry, I do not think that it is established that such failure was an effective
cause of the accident. On the evidence, Samuels thought himself competent to perform this simple operation, and I do not doubt
that, had he been asked, he would have said that he was competent to do so.
Thirdly, the negligence found by the judge was negligence by Waines in his selection of the person whom he authorised to
drive the lorry. But to make this particular negligent act of Waines negligence for which the defendants are vicariously liable
seems to me to involve the tacit assumption that the selection of a person to drive the lorry was one of the things which Waines
was employed by the defendants to do. I do not think that it was. He was expressly prohibited from permitting anyone to drive
the lorry besides himself.
I would, however, affirm the judgment under appeal on the second and much broader ground. In my view, the defendants
liability does not depend on the fact that Samuels was an inexperienced driver who had never driven a lorry in a confined space
before, but on the fact that the lorry was driven negligently while being used for the purposes of the defendants business under
the control of the defendants servant, Waines, he being their servant employed by them to take charge and control of the vehicle
while engaged on the task which was being performed when the accident took place. In my view, their liability would have been
the same if Samuels had been a highly experienced driver, provided that 888 his negligent driving on this occasion was the cause
of the plaintiffs injuries.
Many authorities have been cited about the vicarious liability of a master for the torts of his servants, for the law is nearly
always most obscure in those fields in which judges say: The principle is plain, but the difficulty lies in its application to
particular facts. The principle as set out in Salmond On Torts in the passages already cited by my lord b is clear; and in
approving those passages in Canadian Pacific Rail Co v Lockhart ([1942] 2 All ER at p 467; [1942] AC at p 599) Lord
Thankerton also cited with approval the dictum of Lord Dunedin in Plumb v Cobden Flour Mills Co Ltd ([1914] AC at p 67) that
________________________________________
b See p 884, letter c, ante.

there are prohibitions which limit the sphere of employment, and prohibitions which only deal with conduct within
the sphere of employment.

In cases such as this, where there is an express prohibition, the decision into which of these two classes the prohibition falls
seems to me to involve first determining what would have been the sphere, scope, course (all these nouns are used) of the
servants employment if the prohibition had not been imposed. As each of these nouns implies, the matter must be looked at
broadly, not dissecting the servants task into its component activitiessuch as driving, loading, sheeting and the likeby
asking: What was the job on which he was engaged for his employer? and answering that question as a jury would.
In the present case it appears to me that the job on which Waines was engaged for his employers was to collect a load of
sugar at the sugar factory and transport it to its destination, using for that purpose his employers lorry of which he was put in
charge. The express prohibition was against permitting anyone else to drive the lorry in the course of performing this job. This,
it seems to me, was a prohibition on the made in which he was to do that which he was employed to do, a prohibition dealing
with conduct within the sphere of employment.
That this is the correct approach to prohibitions imposed by an employer on a servant whom he has put in charge of a
vehicle to be used for the purposes of the employers business was established a hundred years ago in Limpus v London General
Omnibus Co in 1862. Gwilliam v Twist is at first sight an exception; but, as was pointed out by Buckley LJ in Ricketts v Thomas
Tilling Ltd, the only matter argued in that case was agency of necessity; were it properly argued today, the decision might have
been different. In Ricketts v Thomas Tilling Ltd, the driver of an omnibus had permitted the conductor to drive. The conductor
has no authority from his employers to drive, and the driver had no authority to permit him to do so, although it does not appear
that there was an express prohibition. The employers were held liable, not for the negligence of the conductor (for it was outside
the scope of his employment to drive the bus) but for the negligence of the driver in not performing his duty of driving or
controlling the driving (per Buckley LJ ([1915] 1 KB at p 648)), or

the responsibility and duty he had towards his masters to see that the omnibus is carefully, and not negligently, driven

(per Pickford LJ ([1915] 1 KB at p 651)).


The duty, as put by Buckley LJ is the general duty of care owed by the servant, as the person in charge of the vehicle, to the
plaintiff, his neighbour in the Atkinian sense c; the duty referred to by Pickford LJ is the contractual duty owed by the servant to
his master under the contract of employment; but, where the vicarious liability of a master for the acts of his servant is concerned,
these are two sides of the same coin. A person who makes use of a vehicle for the purposes of his business is under a duty in tort
so to control it that it is 889 driven with reasonable care while being used for that purpose. If he delegates the performance of the
acts which give rise to this duty to his servant, he is vicariously liable if the servant fails to perform it. In this sense he may be
said to delegate the duty though he cannot divest himself of it, as his continuing vicarious liability shows. The test whether the
master has in this sense delegated the duty to his servant is whether the servant owes to the master a contractual duty to perform
the masters duty owed to his neghbours arising out of those acts. If, as in Ricketts case, and in the present case, the master puts
the vehicle in the charge and control of his servant to be used for the purposes of the masters business, he thereby delegates to
the servant his duty so to control it that it is driven with reasonable care while being used for that purpose; and an express
prohibition on allowing any other person to drive it whilst being used for that purpose is no more than a direction as to the mode
in which the servant shall perform the duty. It is a prohibition dealing with conduct within the sphere of employment.
________________________________________
c See Donoghue (or McAlister) v Stevenson [1932] All ER Rep 1, per Lord Atkin, at p 11; [1932] AC 562 at p 580.

Different considerations might apply if the servant, or some stranger, used the vehicle for some other purpose than his
masters business, and the accident occurred while the vehicle was being used for that other purpose; but in the present case the
accident occurred while the vehicle was being used for the purpose of the defendants business; it was being moved for the
purpose of facilitating the completion of the loading by sheeting the sacks. Waines, as the person in charge and control of the
lorry, was under a duty to the plaintiff so to control it that it was driven with reasonable care. He could not divest himself of that
duty to the plaintiff by authorising Samuels to move the lorry unless he also abandoned his right to control the lorry, which he
was under no duty to the plaintiff (though he was to his master) to retain. But he did not purport to abandon his right to control
the lorry; on the contrary, it was in the exercise of his right of control of the lorry that he authorised Samuels to move it, told him
how to start it, and shouted to him to stop. In my view he remained responsible for its being driven by Samuels with reasonable
care.
It is true that it may be said that this is to treat Waines as vicariously liable for the negligence of Samuels; see Ormrod v
Crosville Motor Services Ltd ([1953] 2 All ER at p 754), per Singleton LJ. And it is contended that, Waines having no authority
from the defendants to delegate the driving of the lorry to another person, he cannot make them vicariously liable for the
negligence of someone to whom he has purported to delegate this. But this, I venture to think, is fallacious. He did not delegate
his duty owed to the plaintiff in tort so to control the vehicle that it was driven with reasonable care because in law he could not
delegate that duty without abandoning his right of control of the lorry.
The duty in tort of which he was in breach was, in my view, a duty delegated to him by the defendants under his contract of
employment, and for that breach the defendants are vicariously liable notwithstanding that it resulted from his breach of an
express prohibition by the defendants against permitting any other person to drive, for that prohibition did not limit the sphere of
his employment, but dealt with the conduct of Waines within that sphere. I agree, therefore, that on the question of liability, this
appeal should be dismissed.
As regards the question of damages, I would put it in this way. Special damage in the sense of a monetary loss which the
plaintiff has sustained up to the date of trial must be pleaded and particularised. In this case special damages were so pleaded and
particularised at the sum of 77 odd. Shortly before the trial, the special damage (as so particularised) was agreed at 77 by
letter. In my view, it is plain lawso plain that there appears to be no direct authority, because everyone has accepted it as being
the law for the last hundred yearsthat one can recover in an action only special damage which has been pleaded, and, of course,
proved. In the present case, evidence was called at the trial the effect of which 890 was that the plaintiff had sustained special
damage of a very much larger sum, amounting, I think it would work out at, to something like 2,000at any rate, a very much
larger sum than 77. This was not pleaded, and no application to amend the statement of claim to plead it could be made because
of the agreement already arrived at at the sum of 77 for special damage. The evidence about the loss of earnings in excess of
77 was admissible, not as proof of special damage (which had not been pleaded) but as a guide to what the future loss of
earnings of the plaintiff might be.
The judge, in the passage to which my Lord has referred, did say that he was not going to take it into account as special
damage. But it seems to me to be plain from the passages in his judgment to which my Lord has alluded, as well as from the
figure of 4,000 general damages which the judge awarded, that he must have included in those general damages some
unspecified sum, the precise amount of which I do not know (and it may be that the judge himself did not quantify it even
mentally) is respect of the plaintiffs loss of earnings before the date of trial in excess of the 77 pleaded which he was entitled to
recover. In so doing, in my view, the judge was guilty of an error in law, save to the extent to which he was entitled to take into
account the actual loss as a guide to loss in the future. That does mean that the matter of damages is at large for this court to
consider.
I agree with what my lord has said that in any event the judge seems to have had greater confidence in the aetiology and
prognosis of these injuries than the specialist had in his report; for whereas the judge says (as my Lord has pointed out) that he
has no douct whatever that the pain which the plaintiff suffers is due to the accident, at any rate to the extent that it has been
exacerbated by it, the highest the surgeon put it was:

It is difficult to deny that this pain may have arisen from an exacerbation of these degenerative disc lesions as a result
of the accident.

At a later stage, the judge said:

I am quite satisfied that the pains that he sustains from the degenerative condition of his back are bound to
deteriorate,

whereas the surgeon had said:

It is anticipated that this pain in the left groin may deteriorate if the patient is engaged in heavy and vigorous
work,

and the plaintiff has been awarded damages on the basis that he will not in future engage in heavy and vigorous work.
Speaking for myself, I might in the light of the medical report have been inclined to give a lesser figure than that which has
commended itself to my Lords; but I agree with them that, in the circumstances of this case, 2,000 is a proper sum to award.

Appeal allowed on issue of quantum of damages only (damages reduced to 2,077).

Solicitors: Clifford Turner & Co agents for Alick Altman & Co, Nottingham (for the third defendants); Sharpe, Pritchard & Co
agents for William Baines, Brigg (for the plaintiff).

F A Amies Esq Barrister.


891
[1963] 2 All ER 892

Warwick Film Productions Ltd and Another v Eisinger and Others


CIVIL PROCEDURE

CHANCERY DIVISION
WILBERFORCE J
2 APRIL 1963
Practice Parties Co-plaintiffs Striking out one plaintiff and adding that party as defendant Copyright Infringement
Licensees commencing infringement action and joining owners of copyright as co-plaintiffs Copyright owners withdrawing
retainer of solicitors for the co-plaintiffs and applying to be struck out as co-plaintiffs and to be added as defendants Whether
and on what terms order should be made Copyright Act, 1956 (4 & 5 Eliz 2 c 74), s 19(3) RSC, Ord 16, r 11.

Under a copyright agreement dated 17 March 1960, which related to a book, the first plaintiffs (the licensees), a film
production company, were licensees of the second plaintiffs (the publishers), a publishing company. The licensees commenced
an action in April, 1960, alleging the infringement of copyright by the defendants. In accordance with the agreement the
licensees joined the publishers as co-plaintiffs. In February, 1960, before the action was set down for trial, the publishers, finding
an indemnity for the costs of the action offered by the licensees not acceptable, withdrew their retainer to the solicitors acting for
the co-plaintiffs and instructed fresh solicitors. The publishers applied by summons under RSC, Ord 16, r 11 a, that their names
might be struck out as plaintiffs and inserted as defendants. The licensees did not object to the making of the order provided that
the publishers were added as defendants in order to comply with s 19(3) b of the Copyright Act, 1956. The defendants objected.
________________________________________
a RSC, Ord 16, r 11, provides: No cause or matter shall be defeated by reason of the misjoinder or non-joinder of parties and the court may
in every cause or matter deal with the matter in controversy so far as regards the rights and interests of the parties actually before it. The
court or a judge may, at any stage of the proceedings, enter upon or without the application of either party, and on such terms as may be
just, or order that the names of any parties improperly joined, whether as plaintiffs or defendants, be struck out, and that the names of any
parties, whether plaintiffs or defendants who ought to have been joined, or whose presence before the court may be necessary in order to
enable the court effectively and completely to adjudicate upon and settle all the questions involved in the cause or matter be added
b Section 19(3) of the Copyright Act, 1956, provides: Where an action is brought either by the owner of the copyright or by the exclusive
licensee, and the action, in so far as it is brought under s 17 of this Act, relates (wholly or partly) to an infringement in respect of which they
have concurrent rights of action under that section, the owner or licensee, as the case may be, shall not be entitled, except with the leave of
the court, to proceed with action, in so far as it is brought under that section and relates to that infringement, unless the other party is either
joined as a plaintiff in the action or added as a defendant.

Held There was jurisdiction under RSC, Ord 16, r 11, in a suitable case to grant the relief sought, and in the present case the
court would make the order sought, on terms c safeguarding the defendants position, since it would be wrong to leave the action
in a paralysed condition with the consequence that a fresh action would have to be brought by the licensees (see p 895, letter e,
and p 896, letters e and f, post).
________________________________________
c The terms imposed are stated at p 896, letters f and g, post

Re Kent Coal Concessions Ltd ([1923] WN 398) applied.

Notes
As to the effect of misjoinder or non-joinder, see 30 Halsburys Laws (3rd Edn) 394, para 735; and for cases on the subject, see
Digest (Practice) 423431, 11901266.
For the Copyright Act, 1956, s 19, see 36 Halsburys Statutes (2nd Edn) 107.

Cases referred to in judgment


A -G v Cooper (1837), 3 My & Cr 258, 40 ER 923, Digest (Practice) 427, 1233.
892
Brown v Sawer (1841), 3 Beav 598, 10 LJCh 656, 93 LT 158, Digest (Practice) 428, 1236.
Kent Coal Concessions Ltd, Re, Burn v Kent Coal Concessions Ltd [1923] WN 328, Digest (Practice) 428, 1238.
Mathews, Oates v Mooney, Re [1905] 2 Ch 460, 74 LJCh 656, 93 LT 158, Digest (Practice) 428, 1236.
Miller v Smith (1839), unreported.

Adjourned Summons
Warwick Film Productions Ltd (hereinafter called the licensees), a film production company, were licensees of the second
plaintiffs, William Hodge & Co Ltd (hereinafter called the publishers), the owners of the copyright, under a copyright
agreement dated 17 March 1960. The agreement related to a book, The Trials of Oscar Wilde, by Professor Montgomery
Hyde. A film was made based on the book. Clause 8 of the agreement provided:

If at any time during the subsistence of this agreement the copyright of the book shall be infringed and [the publishers]
after receiving written notice of such infringement from [the licensees] refuse or neglect to take proceedings in respect of
the infringement, [the licensees] shall be entitled to take proceedings in the joint names of themselves and [the publishers]
on giving [the publishers] a sufficient and reasonable security to indemnify them against any liability for cost.

On 26 April 1960, the licensees, joining the publishers as co-plaintiffs, issued a writ against the defendants, Joseph Eisinger,
Robert Goldstein, Gregory Ratoff and Vantage Films Ltd. They alleged by their statement of claim infringement by the
defendants of (a) the copyright in The Trials of Oscar Wilde, which copyright was owned by the publishers and in which the
licensees had certain rights, and (b) the copyright, of which the licensees alone were the owners, in another literary work, Oscar
Wilde, Three Times Tried. The alleged infringement was by the defendants participation in the production and exhibition of a
film entitled Oscar Wilde. The claim was for an injunction, delivery up, and damages. In July, 1960, Twentieth Century-Fox
Film Company Ltd were added as fifth defendants.
This was a summons taken out by the publishers under RSC, Ord 16, r 11, asking that their name might be struck out as a
plaintiff and inserted as a defendant with consequent amendments to the writ and pleadings.

Douglas Falconer for the first plaintiffs, the licensees.


G W Tookey QC and Christopher Mayson for the second plaintiffs, the publishers.
M Stranders QC and J P Neill for the first, second, third and fourth defendants.
F E Skone James for the fifth defendants.

2 April 1963. The following judgment was delivered.

WILBERFORCE J having reviewed the course of proceedings and stated the facts, continued: This is a summons taken out by
the second plaintiffs in the action (the publishers) that their names may be struck out as plaintiffs and inserted as defendants
with consequent amendments to the writ and pleadings. As originally framed, the summons asked that the first plaintiffs (the
licensees) should pay the costs of the publishers incurred by reason of their joinder in the action, that being based on a
contention that the action had been commenced and prosecuted without the proper authority of the publishers. In support of that
some voluminous evidence was filed, exhibiting many pages of correspondence, and the matter was debated before the master.
However, today the publishers do not pursue that line of attack and they put their application now on the much simpler ground
that they are no longer willing to run in harness with licensees, and they have withdrawn their retainer to the solicitors who were
formerly acting for both plaintiffs in the action.
893
The framework of the proceedings up to date is as follows. The writ was issued on 26 April 1960, by the present plaintiffs
against the first four defendants, of whom it appears that one has since died. A statement of claim was delivered which showed
that the action was based on certain alleged breaches of copyright. The plaintiffs relied in fact on two separate rights. In the first
place in the first paragraphs of the statement of claim they refer to the copyright in a literary work, The Trials of Oscar Wilde,
written by Professor Montgomery Hyde, and it was said in relation to that that the publishers were the owners of the copyright
and that the licensees had certain rights in the nature of a licence by virtue of an agreement made in 1960. Then the statement of
claim went on to plead that the licensees alone were and are the owners of rights of copyright in another literary work called
Oscar Wilde: Three Times Tried, written by one Millard, and infringement was claimed in respect of that. I need not deal with
the rest of the statement of claim. The normal form of relief was sought: injunction, delivery up and damages. That has been
followed in due course by defences put in on behalf of the defendants and a reply.
There was a motion for interim relief which came before Danckwerts J in May, 1960, and that motion, it appears, was
brought by the licensees alone, the publishers being unwilling to be associated with it on account of the possibility that they
might have to give an undertaking in damages; but no relief was, in the event, granted on that motion.
Then in July, 1960, Twentieth Century-Fox Film Co Ltd were added as the fifth defendants. The matter went on with
various particulars and other matters until about 21 February 1962, when, in correspondence passing between the plaintiffs, the
publishers found that an indemnity for the costs of the action, which was offered by the licensees and which was under discussion
between them, was not acceptable to them. As a result of that, quite shortly afterwards, on or about 1 March 1962, the publishers
withdrew their retainer to the solicitors acting for the co-plaintiffs and instructed fresh solicitors of their own, in respect of whom
a notice of change was duly given.
The application is brought in accordance with RSC, Ord 16, r 11, which refers to the names of parties improperly joined
which, it is said, may be struck out, but it seems clear from authority that under that rule it is possible in appropriate cases for
plaintiff to be struck out, in cases where the plaintiffs disagree, and for the name of the plaintiff so struck out to be brought in as a
defendant. Several cases were referred to before me. The first was Re Mathews, Oates v Mooney. That was a case before
Swinfen Eady J where there was an application on behalf of the defendants to have proceedings stayed as between one of several
co-plaintiffs and themselves on the ground that a compromise had been arranged, and what was said with regard to situations
similar to the present was no doubt obiter, but the judge did say that in a proper case the right course was, in case of a difference
between co-plaintiffs, that the name of one of them should be struck out as plaintiff and added as defendant. He made a reference
to the necessity of providing security for the defendants costs. That was followed in a case before the Court of Appeal, Re Kent
Coal Concessions Ltd, Burn v Kent Coal Concessions Ltd which was a debenture-holders action being brought by seven
plaintiffs on behalf of themselves and all other holders of debentures in the defendant company. There were various motions in
the nature of interlocutory proceedings and as a result a statement of claim was delivered asking only for costs. Then certain
further pleadings were delivered which introduced fresh accusations of a personal character against the defendants, on which two
of the plaintiffs desired to withdraw from the action altogether on the ground that larger issues were involved than had been
contemplated when they consented to join in it. Lawrence J held that they were not entitled to withdraw, but the Court of Appeal
held otherwise and, following certain earlier 894 cases including Re Mathews they said that in a proper case an order could be
made and that the power to strike out plaintiffs from an action could be exercised when it was found that a paralysis of the action
would take place if the order were not made. They made the order on an undertaking by the appellants to abide by such order as
the court might make in regard to any claim by the defendants against them in respect of costs. Those were the cases relied on,
on behalf of the applicants.
The defendants on their side also relied in the main on two authorities. The first was A-G v Cooper which was a relators
action. It was asked there that certain of the names of those who had joined in as relators should be struck out, without, be it
noted, their names being added as defendants. The basis of that decision was that an application of that kind would not be
granted, even though the defendants would not be prejudiced, unless it appeared that without the alteration justice would not be
done or that the case could not be so conveniently prosecuted if the alteration were not made. That was not an application for the
names of the struck-out plaintiffs to be brought in as defendants, and I think it is also right to say that the court took a rather
stricter view at that date as regards maintaining the record than would be taken in present times. The other case was Brown v
Sawer, where one of co-plaintiffs refused to proceed. The other plaintiff, not, in other words, the refusing plaintiff, obtained an
order to amend by making the refusing plaintiff a defendant. Lord Langdale MR gave a judgment which referred to A-G v
Cooper and he said he thought that it was an appropriate case for the substitution to be effected ((1841), 3 Beav at p 600).
From those cases it seems to me that there is undoubted jurisdiction in a suitable case to grant the relief which is asked and
the only question is whether on the facts and in the circumstances the courts discretion should be exercised. In this case it is to
be noted that the licensees do not object to the order sought, provided that the publishers are added as defendants, and that the
position is related to s 19 of the Copyright Act, 1956 which, in certain circumstances, that may or may not be relevant here,
contemplates that a person in the position of the publishers, namely, the owner of the copyright, shall be present in the action as a
defendant. However that may be, the licensees do not for their part object to their co-plaintiffs being removed from their side of
the action.
On the other hand, all the defendants object to the proposed change. They put their case in two ways. First, they say quite
generally that no case is made by the publishers for the relief which is asked and that in order to support such a case some
motivated argument supported by evidence must be before the court. As to that, one has to face the fact, in my view, that the
publishers have withdrawn their retainer from their solicitors, thus making it impossible for the action to proceed as it is, and one
must consider the consequences, in the circumstances which prevail, of leaving the action in that position. So I think that it is
necessary to look into the whole circumstances of the case.
As to that (and this is the second limb of the defendants argument) they rely on the evidence which has in fact been filed
and practically the whole of it was ready on behalf of the defendants. It seems to me relevant to observe that that evidence was
put in in support of the summons as it was originally framed, which was that the publishers be removed on the footing that the
licensees had no right to bring or keep them in and that the licensees should pay the costs of the publishers incurred by their
joinder in the action. The evidence goes into, at considerable length, the question whether the publishers and their legal advisers
knew what was being done and whether they did or did not give their implied authority and countenance to the conduct of the
action by the licensees.
The defendants say with reference to that, that the publishers are under a contractual obligation to the licensees to allow their
association with the action 895 and that their present step is a step taken in breach of their contractual obligations; but that seems
to me quite clearly disputed on the evidence and it is the case that the other parties to the alleged contract do not, as they might,
seek to hold the publishers to it, or, as they might, ask that they may continue the action using the publishers namea course
which I note was taken in Miller v Smith, which is referred to briefly in a note to Brown v Sawer. So as regards that part of the
evidence it seems to me that the defendants are really relying on facts which were put in evidence in order to canvass the mutual
claims and obligations of the plaintiffs themselves inter se.
As regards the defendants own position, a solicitor acting for the first four defendants drew attention in his affidavit to the
costs that they have incurred in the action up to date, which are very considerable, and to certain procedural steps and safeguards
which would have to be taken if the publishers application were granted. Those are serious matters which must be seriously
taken into account.
It is finally relevant in considering the merits of this application to have regard to the fact that the action could have been
commenced with the publishers in the position of a defendant and that indeed at any timethis is made plain in several of the
lettersthe defendants might have had to face that situation, namely, of having the publishers put in as defendants. Indeed, the
solicitor for the licensees, in his affidavit, said in terms that the effective plaintiffs in the action are the licensees and that the
joinder of the publishers is, in his view, a procedural matter.
In those circumstances it seems to me that it would be wrong to leave the action in a paralysed condition, so that a fresh
action would have to be brought by the licensees in relation not only to the copyright in which the publishers are involved, but
also in relation to the other copyright in which the publishers are not involved, if the defendants position can be properly
safeguarded.
So I propose to make the order asked for on the following terms, if complied with the by publishers: first, that the
defendants be given liberty to amend their pleadings as they may be advised and that the costs of those amendments should be
borne by the publishers; secondly, that the publishers should give an undertaking to the court as regards the defendants costs to
date, in the form which was approved by the Court of Appeal in Re Kent Coal Concessions Ltd; and, thirdly, that, in view of the
facts that the publishers are registered in Scotland and that there is no sufficient evidence of their assets or business within this
jurisdiction, they should provide security for the defendants costs incurred to date. As regards the costs of the first to fourth
defendants, that should be in the sum of 2,650, being the figure given on their behalf as representing an estimate of the taxed
costs of the first to fourth defendants down to date. As regards the fifth defendant, I have no figure, and the security will be as
directed by the master in the absence of agreement. Thirdly, the order will be subject to the publishers giving an undertaking to
give such discovery as the defendants would have been entitled to had the publishers remained plaintiffs and the issues had been
such as they were at the date when they were struck out as plaintiffs and inserted as defendants. The costs of the present
application will fall on the publishers.

Order accordingly.

Solicitors: Nicholas & Co (for the licensees); Radcliffes & Co (for the publishers); Goodman Derrick & Co (for the first, second,
third and fourth defendants); Joynson-Hicks & Co (for the fifth defendants).

Jenifer Sandell Barrister.


896
[1963] 2 All ER 897

R v Allan and Others


CRIMINAL; Criminal Law

COURT OF CRIMINAL APPEAL


EDMUND DAVIES, MARSHALL AND LAWTON JJ
11, 12, 13, 18 JUNE 1963

Criminal Law Affray When a person can properly be convicted of being a principal in the second degree to an affray.

Criminal Law Principal Principal in second degree Whether encouragement required before an accused person can be
regarded as a principal in the second degree.

Before a jury can properly convict an accused person of being a principal in the second degree to an affray, they must be
convinced by the evidence that, at the very least, he by some means or other encouraged the participants; to hold otherwise would
be, in effect, to convict a man on his thoughts, unaccompanied by any physical act other than the fact of his mere presence (see p
900, letter i, to p 901, letter, a, post).
Dicta of Cave J and of Lopes J in R v Coney ((1882), 8 QBD at pp 539, 552) applied.
Encouragement in one form or another is a minimal requirement before an accused person may properly be regarded as a
principal in the second degree to any crime (see p 901, letter a, post).
Dictum of Devlin J in National Coal Board v Gamble ([1958] 3 All ER at p 209) explained.
Appeals allowed.

Notes
As to principals in the second degree, see 10 Halsburys Laws (3rd Edn) 297, 298, para 554; and for cases on the subject, see 14
Digest (Repl), 8789, 487513, 91, 92, 533542.
As to affrays, see 10 Halsburys Laws (3rd Edn) 584, para 1086; and for cases on the subject, see 15 Digest (Repl) 785, 786,
7360, * 4915-*4919.

Cases referred to in judgment


National Coal Board v Gamble [1958] 3 All ER 203, [1959] 1 QB 11, [1958] 3 WLR 434, 122 JP 453, 42 Cr App Rep 240, 3rd
Digest Supp.
R v Cohen and Bateman (1909), 73 JP 352, 2 Cr App Rep 197.
R v Coney (1882), 8 QBD 534, 51 LJMC 66, 46 LT 307, 46 JP 404, 15 Digest (Repl) 790, 7411.
R v Murphy (1833), 6 C & P 103, 2 Nev & MMC 149, 172 ER 1164, 15 Digest (Repl) 945, 9084.
R v Young (1838), 8 C & P 644, 173 ER 655, 14 Digest (Repl) 94, 566.
Wilcox v Jeffery [1951] 1 All ER 464, 115 JP 151, 2 Digest (Repl) 184, 122.Shorter Oxford English Dictionary (2nd Edn), p 405
Countenance.

Appeals
These were appeals by George Donald Allan, James Joseph Boyle, and Albert Ballantyne against their convictions with Michael
Mooney and others at Lincoln Assizes before Thesiger J and a jury on 11 February 1963, of making an affray. The appellants
Allan and Boyle also appealed against their sentences of four years imprisonment and the appellant Mooney against that part of
his sentence relating to a recommendation for deportation. The facts are set out in the judgment of the court.

Anthony Cripps QC and Alan de Piro for the appellants Allan and Ballantyne.
Geoffrey Lane QC and R D Lymbery for the appellants Boyle and Mooney.
J A Grieves QC and E F Jowitt for the Crown.

Cur adv vult


897

18 June 1963. The following judgment was delivered.

EDMUND DAVIES J read the following judgment of the court. After a trial lasting nine days at the Lincoln Assizes in February
last, the appellants Allan, Boyle and Ballantyne were convicted of making an affray, the appellant Mooney having pleaded guilty
on the second day of the trial. The appellants Allan and Boyle now appeal against conviction and their sentences of four years
imprisonment, the appellant Ballantyne against conviction only. The appellant Mooney appeals only against the recommendation
that he be deported to his native Ireland. Having regard to the decision at which this court has arrived on the point of law on
which the first three appellants rely, the facts need not be gone into in very great detail. During the evening of Saturday, 8
September 1962, a fight with knives took place between a number of Somalis and white men in streets in the vicinity of the
Oswald Hotel in Scunthorpe. During that evening, a Somali named Yusef was stabbed to death in Allanby Street and this court
last week dismissed the appeal of John Gow against his conviction for non-capital murder. The fight appears to have lasted at
most some fifteen minutes, a large number of people collected in Frances Street, West Street and Allanby Street, where the
principal fighting occurred, and, during its course, the appellants Allan, Ballantyne and Mooney received knife wounds. Despite
the submissions of learned defence counsel, the court has come to the conclusion that, in the case of each of the appellants, there
was sufficient evidence to justify the jury coming to the conclusion, on a proper direction, that they had all unlawfully fought and
made an affray. The case against the appellant Allan was a good deal less strong then that against the other appellants, but, even
so, we should not have interfered with his conviction had the jury been properly directed.
The legal point that calls for determination is as to the learned judges direction regarding the proof necessary before an
accused person may properly be convicted of making an affray, but the criticism made on it involves consideration of matters
which have an importance which reaches beyond the confines of the law as to affrays. The learned judge had caused to be typed
out copies of his direction which were handed to the jury and, one may take it, this was carefully studied by them and operated to
guide their deliberations, as it was intended to do. The direction was in the following terms:

1. An affray is a fight between two or more people in a public street that is likely to terrify passers-by or residents. 2.
Every person who is a party to an affray is guilty of an offence. 3. Mere presence at the fight does not make a person a
party to it, even if he does nothing to stop it; nor is he a party to it if he tries to stop the fight and restore the Queens Peace,
or if he tries to defend someone from being hurt, because it is lawful to intervene for this purpose. 4. Every person is,
however, a party and guilty who: (i) agrees that such a fight should take place and, in pursuance of that agreement, is later
present at it; or (ii) without such agreement unlawfully joins in such a fight, or, being present, chose to remain present,
either (a) knowing that his continued presence encouraged the fight, or (b) intending to join in the fight if his help was
needed by his side.

No criticism has been, or properly could be, levelled against para 1, para 2, para 3 and para 4(i) of that direction, nor as to the
direction in para 4(ii) that a person unlawfully joining in an affray is guilty, notwithstanding the absence of prior agreement.
Paragraph 4(ii)(a) is, however, attacked as containing what learned counsel described as a concealed dichotomy. This highly
alarming concept fortunately does not call for comment, and, we can turn forthwith to consider the more telling attack made on
para 4(ii)(b). In effect, it amounts to this: that the learned judge thereby directed the jury that they were in duty bound to convict
an accused who was proved to have been present and witnessing an affray if it was also proved that he nursed an intention to join
in if help was needed by the side which he favoured, and this notwithstanding that he did 898 nothing by words or deeds to
evince his intention and outwardly played the role of a purely passive spectator. It was said that, if that direction is right, where A
and B behave themselves to all outward appearances in an exactly similar manner, but it be proved that A had the intention to
participate if needs be, whereas B had no such intention, then A must be convicted of being a principal in the second degree to the
affray, whereas B should be acquitted. To do that, it is objected, would be to convict A on his thoughts, even though they found
no reflection in his actions. For the Crown, on the other hand, it is contended that the direction was unimpeachable, and that, in
the given circumstances, a jury doing its duty would be bound to convict A of aiding and abetting in an affray, even though he
uttered no word of encouragement and acted throughout in exactly the same manner as all the other spectators of what was
happening.
The locus classicus on this branch of the law is the majority decision of the Court of Crown Cases Reserved in R v Coney,
whereby the conviction of mere spectators of a prize-fight as being aiders and abettors in the fight was quashed. Before
proceeding to quote from Hale (1 Hale, PC 439) and from Fostera, Cave J said ((1882), 8 QBD at p 539):
________________________________________
a Fosters Crown Law (1792), p 350.

Now it is a general rule in the case of principals in the second degree that there must be participation in the act, and
that, although a man is present whilst a felony is being committed, if he takes no part in it, and does not act in concert with
those who commit it, he will not be a principal in the second degree merely because he does not endeavour to prevent the
felony, or apprehend the felon.

The opening words of the learned trial judge in para 3 of his direction accurately adopted this passage, but the difficulty arises, as
we have said, in relation to the following paragraph (para 4). In R v Murphy ((1833), 6 C & P 103 at p 104), Littledale J directed
the jury as follows:

I am of opinion that persons who are at a fight, in consequence of which death ensures, are all guilty of
manslaughter, if they encouraged it by their presence; I mean, if they remained present during the fight. I say, that if they
were not casually passing by, but stayed at the place, they encouraged it by their presence, although they did not say or do
anything If the death occurred from the fight itself, all persons encouraging it by their presence are guilty of
manslaughter

Commenting on this direction, Cave J had this to say in R v Coney ((1882), 8 QBD at p 543):
This summing-up unfortunately appears to me capable of being understood in two different ways. It may mean either
that mere presence unexplained is evidence of encouragement, and so of guilt, or that mere presence unexplained is
conclusive proof of encouragement, and so of guilt. If the former is the correct meaning, I concur in the law so laid down;
if the latter, I am unable to do so. It appears to me that the passage tending to convey the latter view is what was read by
the chairman in this case to the jury, and I cannot help thinking that the chairman believed himself, and meant to direct the
jury, and at any rate I feel satisfied that the jury understood him to mean, that mere presence unexplained was conclusive
proof of encouragement, and so of guilt; and it is on this ground I hold that this conviction ought not to stand.

With this judgment that great master of the criminal law, Stephen J expressed his agreement ((1882), 8 QBD at p 548). The basis
of the majority decision was neatly expressed in the short judgment of Lopes J which contains this passage ((1882), 8 QBD at p
552):
899

I cannot hold, as a proposition of law, that the mere looking on is ipso facto a participation in or encouragement of a
prize-fight. I think there must be more than that to justify a conviction for an assault. If, for instance, it was proved that a
person went to a prize-fight, knowing it was to take place, and remained there for some time looking on, I think that would
be evidence from which a jury might infer that such person encouraged, and intended to encourage, the fight by his
presence.

Finally, Hawkins J had this to say ((1882), 8 QBD at p 557):

In my opinion, to constitute an aider and abettor some active steps must be taken by word, or action, with the intent to
instigate the principal, or principals Non-interference to prevent a crime is not itself a crime. But the fact that a person
was voluntarily and purposely present witnessing the commission of a crime, and offered no opposition to it, though he
might reasonably be expected to prevent and had the power so to do, or at least to express his dissent, might under some
circumstances offer cogent evidence upon which a jury would be justified in finding that he wilfully encouraged and so
aided and abetted. But it would be purely a question for the jury whether he did so or not.

Applying these passages to the direction in the present case, we have come to the conclusion that, in effect, the trial judge
here dealt with facts which, at most, might provide some evidence of encouragement as amounting to conclusive proof of guilt.
The jury were in terms told that a man who chooses to remain at a fight, nursing the secret intention to help if the need arose, but
doing nothing to evince that intention, must in law be held to be a principal in the second degree and that, on these facts being
proved, the jury would have no alternative but to convict him. In our judgment, that was a misdirection. As Cave J said in R v
Coney ((1882), 8 QBD at p 540), Where presence is prima facie not accidental it is evidence, but no more than evidence, for the
jury, and it remains no more than evidence for the jury even when one adds to presence at an affray a secret intention to help.
No authority in support of the direction given in the present case has been cited to us. The passage in R v Young ((1838), 8 C & P
644 at p 652) cited in R v Coney ((1882), 8 QBD at p 541), is incomplete, and reference to the report itself makes clear that
Vaughan J was there dealing with presence at a fight as the result of a previous arrangement. The only other case cited by the
Crown, Wilcox v Jeffery turned on special facts very different from the present, and is one from which we think no general
principle can be deduced. In the present case, the trial judge dealt with matters of evidence from which encouragement (and,
therefore, guilty participation) might be inferred ifbut only ifthe jury thought fit to do so as necessarily amounting in law to
proof that guilt was established. In our judgment, this amounted to a misdirection, and one, unfortunately, of a basic kind.
This court considers that there is a further ground for criticism of the directions to the jury. By the third direction, they were
rightly told that proof of mere presence at an affray is not sufficient ipso facto to establish guilt as a principal in the second
degree. So far, so good. By para 4(ii)(a) of the directions, the element of encouragement was dealt with. But, owing to the
somewhat complex grammatical construction of that paragraph and the use of the disjunctive between part (a) and part (b)
thereof, when the jury came to deal with para 4(ii)(b) they may well have been led into thinking that the element of
encouragement was a matter with which they need not then concern themselves at all. So to think would be wrong. In our
judgment, before a jury can properly convict an accused person of being a principal in the second degree to an affray, they must
be convinced by the evidence that, at the very least, he by some means or other encouraged the participants. To hold otherwise
would be, in effect, as counsel 900 for the appellants rightly expressed it, to convict a man on his thoughts, unaccompanied by
any physical act other than the fact of his mere presence. Indeed, in our judgment, encouragement in one form or another is a
minimal requirement before an accused person may properly be regarded as a principal in the second degree to any crime. It is
true that, in National Coal Board v Gamble ([1958] 3 All ER 203 at p 209; [1959] 1 QB 11 at p 24) Devlin J referring to R v
Coney said:

it would be wrong to conclude that proof of encouragement is necessary to every from of aiding and abetting.

But when the facts of that case are examined, it does not, in our judgment, affect the validity of the proposition which we have
stated, for there the accused had intentionally supplied an article essential to the perpetration of the crime charged, and this
Devlin J described ([1958] 3 All ER at p 210; [1959] 1 QB at p 24) as an a fortiori case to that of voluntary presence. In that
case, there was what was judicially described as proof of intent to aid, which may properly be regarded as but one of the
numerous possible manifestations of encouragement.
In the light of these criticisms, we have considered whether, in the circumstances, it would be proper to apply the proviso to
s 4(1) of the Criminal Appeal Act, 1907. But counsel for the Crown, with characteristic candour, intimated that, should we hold
(contrary to his submissions) that what was here said to the jury amounted to a misdirection in the respects already indicated, he
felt unable to invite this court to say that it was clear that no substantial miscarriage of justice had flown therefrom. We think that
the attitude of counsel for the Crown is right. For the proviso to operate where there has been a wrong direction on the law,
the Crown have to show that, on a right direction, the jury must have come to the same conclusion (per Channell J in R v Cohen
and Bateman), and that the Crown concede they cannot here show. This was in many respects a complex and difficult case, as
well as a long one. The witnesses were numerous, there were not unimportant discrepancies even on the prosecution side as to
the parts alleged to have been played by the various accused, and in their two-hour deliberations the jury must often have turned
to the typed legal directions as to a sheet-anchor. Profitless though it is to conjecture how their minds worked, it is by no means
fanciful to entertain the suspicion that their thought-process may well have been:

The accused were present at the scene of the affray and chose to remain there, and we are satisfied that they intended
to intervene if they thought the Somalis were getting the better of the whites; the judge has said that, on those facts, we
must hold that they were parties to the affray; we, therefore, find them guilty.

For these reasons, in our judgment the misdirection was one in respect of which the proviso cannot properly be invoked. It
follows that the appeals of Allan, Boyle and Ballantyne succeed, that their convictions are quashed and that they are discharged.
[His Lordship considered the appellant Mooneys appeal against the recommendation that he be deported and said that the
court would allow his appeal by removing the recommendation.]
Appeals allowed.

Solicitors: Registrar, Court of Criminal Appeal (for the appellants); Director of Public Prosecutions (for the Crown).

N P Metcalfe Esq Barrister.


901
[1963] 2 All ER 902

Shepherd and Another v Lomas


AGRICULTURE; LANDLORD AND TENANT; Tenancies

COURT OF APPEAL
LORD DENNING MR, HARMAN AND PEARSON LJJ
20, 21 MAY 1963

Agriculture Agricultural holding Notice to quit Non-compliance with notice to remedy breaches of terms of tenancy within
period of six months Some breaches requiring supply of materials by landlords Materials not supplied Failure of tenant to
remedy any breaches Validity of notice Agricultural Holdings Act, 1948 (11 & 12 Geo 6 c 63), s 24(2) (d).

By a notice in writing dated 13 September 1961, the landlords of an agricultural holding served a notice on the tenant to remedy
breaches of his tenancy agreement within a period of six months. One group of the breaches alleged concerned hedging, ditching
and clearing, which the tenant had to do with his own labour, and the other group concerned the making good of gates and fences,
for which the landlords were under an obligation to provide the materials. The tenant requested the landlords to provide the
materials, but they were not supplied until a week before the six months notice was due to expire. The tenant failed to remedy
any of the breaches alleged, and the landlords served on him a years notice to quit, relying on the provisions of s 24(2)(d) of the
Agricultural Holdings Act, 1948 a. The matter having been referred to arbitration under the Act, the arbitrator found that the
period of six months was reasonable in respect of the group of breaches which required no materials to be provided by the
landlords and that six months would have been reasonable in respect of the second group of breaches, but, as the landlords failed
to provide the materials for the other group of breaches, the six months period was not, as things turned out, a reasonable period.
________________________________________
a Section 24(2)(d), so far as material, is set out at p 904, letter b, post.

Held (i) the notice was not void ab inito, because, when it was served, six months was a reasonable period within s 24(2)(d) of
the Act of 1948 in which to remedy all the breaches alleged (see p 904, letter h, p 906, letter g, and p 907, letter g, post), and
(ii) the tenants failure to comply with the notice in not remedying those breaches which he could have remedied without
materials being supplied by the landlords was a failure on which the landlords could rely as entitling them under s 24(2)(d) to
serve an effective notice to quit (see p 905, letter d, p 906, letter i, and p 908, letter b, post).
Appeal dismissed.

Notes
Section 24(2)(d) of the Agricultural Holdings Act, 1948, is amended as from a day to be appointed by s 19(1) of the Agriculture
(Miscellaneous Provisions) Act, 1963.
As to the operation of a notice to quit an agricultural holding, see 1 Halsburys Laws (3rd Edn) 282285, paras 599601; and
for cases on the subject of notices to quit agricultural holdings, see 2 Digest (Repl) 813, 1852.
For the Agricultural Holdings Act, 1948, s 24, see 28 Halsburys Statutes (2nd Edn) 46.

Cases referred to in judgments


Fox v Jolly [1916] 1 AC 1, 84 LJKB 1927, 113 LT 1025, 3 Digest (Repl) 540, 6633.
Pannell v City of London Brewery [1900] 1 Ch 496, 69 LJCh 244, 82 LT 53, 31 Digest (Repl) 540, 6634.
Price v Romilly [1960] 3 All ER 429, [1960] 1 WLR 1360, 3rd Digest Supp.

Appeal
This was an appeal by the tenant from a decision of His Honour Judge Addleshaw given on 8 March 1963, at the Congleton
County Court, on a Special Case 902 stated by an arbitrator under the Agricultural Holdings Act, 1948, s 77 and Sch 6. The facts
are set out in the judgment of Lord Denning MR.
The authorities and cases noted belowb were cited during the argument in addition to the cases referred to in the judgments.
________________________________________
b Woodfall on Landlord and Tenant (26th Edn), Vol 1, p 957, 23 Halsburys Laws (3rd Edn) 676, para 1401, note (s), Hopley v Tarvin Parish
Council, (1910), 74 JP 209, Blewett v Blewett, [1936] 2 All ER 188.

L A Blundell QC and Christopher Priday for the tenant.


P G Langdon-Davies for the landlords.

21 May 1963. The following judgments were delivered.

LORD DENNING MR. This case relates to an agricultural holding which is situated at Overton Hall, Biddulph, in
Staffordshire. The farm was let to Mr Lomas, the tenant, as long ago as 1941. There was a clause in the tenancy agreement
under which the tenant agreed on his part

to keep all roads hedges fences walls gates gateposts bridges ditches drains and watercourses in good and
tenantable repair and condition and so leave the same at the end of the tenancy, the landlord providing the tenant with
timber and other material in the rough for the repair of gates and fences and rubble stone for walls.
There was also a separate covenant by the landlords to provide timber in the rough for the repair of gates and fences and rubble
stone for the repair of walls. On 13 September 1961 the landlords served on the tenant a notice to remedy breaches. The notice
said:

We hereby give you notice that we require you to remedy within the period of six months from the date of service of
this notice the breaches whereof particulars are set out below of the terms or conditions of your tenancy of the holding
known as Overton Hall Farm and situate at Biddulph in the County of Stafford, which you hold of us as tenant being
breaches which are capable of being remedied of terms or conditions which are not inconsistent with your responsibilities
to farm the said holding in accordance with the rules of good husbandry.

Then there follow thirty breaches which are alleged against the tenant. They fall into two groups: (i) One group concerned
hedging and ditching and clearing. The landlords did not have to provide any materials for this work. The tenant, of course, had
to do it with his own labour. (ii) The other group concerned the making good of gates and fences. Those require quite a lot of
material, and the landlords were under an obligation to provide it. That notice was served on 13 September 1961, so that the six
months would expire on 13 March 1962. A fortnight later, on 29 September 1961, the tenants solicitors replied:

We also acknowledge receipt of copy notice to remedy defects which we have discussed with our client. Our client is
naturally willing to perform those parts of the tenancy agreement which are due to be performed by him, and will therefore
remedy those defects which he is bound to remedy, but, with the exception of those relating to the clearance of the gorse
and the bushes, the tenant is not in a position to remedy them until the landlord observes his part of the agreement.

It is quite plain that the solicitors there, on reading the notice, were saying this: Our client will get on with the work for which
materials are not required, but he cannot do the other work until the landlord provides the materials. In that letter and later
letters, the tenants solicitors requested the landlords to provide the materials, namely, the timber for the repair of the gates and
fences, but the landlords did not comply with the request. The letters of the tenants solicitors were left unanswered for weeks.
No satisfaction was obtained. The materials were not supplied until 5 March 1962, just a week before the six months notice was
due to expire.
The six months notice expired on 13 March 1962. Nine days later, on 22 March 903 1962, the landlords served on the
tenant a years notice to quit, to expire on 25 March 1963. They claimed that they were entitled so to do by reason of s 24(2)(d)
of the Agricultural Holdings Act, 1948, which enables a landlord to serve notice to quit (without obtaining the consent of the
Agricultural Land Tribunal) where

at the date of the giving of the notice to quit the tenant had failed to comply with a notice in writing served on him by
the landlord requiring him within a reasonable time or within such reasonable period as was specified in the notice to
remedy any breach by the tenant that was capable of being remedied of any term or condition of his tenancy which was not
inconsistent with the fulfilment of his responsibilities to farm in accordance with the rules of good husbandry, and it is
stated in the notice to quit that it is given by reason of the matter aforesaid

The tenant disputed the validity of the notice to quit. He said that it was bad because the notice to remedy breaches was bad. The
period specified in the notice was not a reasonable period. Alternatively, the tenant was discharged by the landlords failure to
provide the materials. The tenant served a counter-notice; and the matter went, in accordance with the statutory provision, to an
arbitrator, who has stated a Special Case for the opinion of the court. The arbitrator made this finding c.:
________________________________________
c In para 11 of the Special Case

I find that the tenant did not comply with the requirements of the notice in that only a few of the breaches referred to
in the notice were remedied within the period specified therein and I further find that the period of time specified in the
notice was reasonable in respect of the breaches for the remedy of which the landlords were not required to supply any
materials and was unreasonable with regard to those for the remedy of which the landlords were liable to provide
materials.

We have had a good deal of argument as to the true construction of that finding. We have been referred to the statements of case
which the landlords and the tenant put in, to the contentions recorded in the Special Case, and to the correspondence which
passed between the solicitors. After considering them all, I think that the true interpretation of this finding of the arbitrator is that
six months was reasonable in respect of the first group (where no materials were required), and that six months would also have
been reasonable in respect of the second group (where materials were required), provided always that the landlords did their part
and supplied the materials; but that, in the events that happened, the period turned out to be unreasonable for the second group
because of the landlords failure to provide the materials.
So interpreting the finding, I do not think that the notice to remedy breaches was void ab initio. It was good in the
beginning, because six months was reasonable for all the breaches, not only the first group but also the second group, seeing that
it could fairly be assumed that the landlords would do their part and fulfil their covenant to provide the materials. So the answer
to the first question: Was the notice void ab initio? is: No..
The second question was this:

Having regard to my findings, does the tenants failure to comply with the requirements for which the period specified
was reasonable constitute a failure to comply with the notice for the purpose of s. 24(2)(d) of the Agricultural Holdings
Act, 1948?

Put in other words, it comes to this: assuming that the notice was good in the beginning because a reasonable period was
specified, how does it stand now that it turns out that, in regard to the second group of breaches, the tenant is excused from doing
those within the time because of the landlords own failure to provide materials? Does it mean that the landlords cannot rely on
the notice? 904This is a difficult question. It depends on whether the notice can be split up. The landlords have specified in the
notice some breaches on which, in the result, they cannot rely, but there are others on which they can rely. Does this make the
whole notice bad? To answer this question, I would draw an analogy from the cases on what is now s 146 of the Law of Property
Act, 1925 (which bears a close resemblance to s 24(2)(d)), particularly Pannell v City of London Brewery Co, and Fox v Jolly. It
is quite plain that, even though a notice under s 146 specifies some matters which cannot be relied on as breaches, nevertheless
that does not disentitle the landlord from relying on the rest of the notice. So, also, it seems to me that, under the Agricultural
Holdings Act, 1948, even though some of the matters that are specified in the notice turn out not to be breaches, or the tenant
afterwards is excused from performing some of them, the landlord is still entitled to rely on the others. It follows that, in this
case, although the tenant was excused from remedying the second group of breaches (because the landlords did not provide
materials), nevertheless he ought to have remedied the first group. He was not entitled to ignore the whole notice and do nothing.
He should have got on with the part which specified quite clearly what he should doas, indeed, his solicitors in the letter of 29
September 1961 said that he would do. He was given a reasonable time to do that work and he did not do it. In those
circumstances, he failed to comply with the notice and must suffer the consequences. The notice was good, and the tenants
failure to comply with the first group was a failure on which the landlords can rely. The answer to the second question is in the
affirmative.
I would only add this: in considering what is a reasonable time for any of the breaches, the arbitrator should always take
into account the rest of the breaches specified in the notice. It may often be quite reasonable for the tenant to postpone some
work till other work has been done; or to wait until he has men to spare for the task; or to abide a suitable season. The time
specified ought to be time to do them all. In this very case, the arbitrator might have found that, having regard to the total amount
of work required, the term specified for the first group was unreasonable. But he has not done so; he has found it was reasonable.
We are bound by that finding, which means that the notice was good in respect of those breaches; and, it not having been
complied with, the notice to quit was good.
I think that the county court judge answered the questions of the arbitrator rightly, and I would dismiss this appeal.

HARMAN LJ. The tenant has been farming this land at least since 1941. The landlord has died and the applicants are his
representatives. It is quite clear that, in the course of time, the tenant had let down the ditches, the hedges and the walls and gates
on this farm to a very large extent, and he was served with a notice in September, 1961, setting out a long list of matters in
respect of which he was in default under his covenant in the lease, which my lord has read, and another covenant to farm in a
proper and husbandlike manner. In respect of the breaches of covenant which related to walls and gates, the landlords were under
an obligation themselves to provide the material for doing the work which they asked to have done. It was not suggested by the
tenant at any time that, except for one or two items at the very most, he was not in default in all these respects, subject, of course,
to the correlative liability of the landlords where materials were involved. This was an annual tenancy in the ordinary way of
farm tenancies with a years notice to quit. In respect of agricultural land, Parliament has thought fit to put a great number of
obstacles in the landlords way, preventing him from disturbing the quiet of his tenants holding except in what are very
stringently limited conditions. When one looks at the Agricultural Holdings Act, 1948, one finds, in s 23(1), that a tenant must
always have twelve months notice to quit, and one finds in s 24 that, except with the consent of the Agricultural Land Tribunal, a
landlord cannot give a good notice to quit unless he can prove 905 that he is within one of the sub-paras of s 24(2). Sub-
paragraph (d) says that the landlord must satisfy the court that, at the date of the notice to quit, the tenant had failed to comply
with a notice in writing served on him by the landlord. Here there has been a notice served by the landlords. What has that got to
do in order to be an adequate notice? It has to require him within such reasonable period as is specified in the notice to remedy
any breach by the tenant specified in the notice which is capable of being remedied. Here, a six months period was specified,
and there was a long list of breaches also specified which were capable of being remedied. There may have been one about the
re-making of a gate which was in doubt, the tenant arguing that there had never been a gate in that position and that he was,
therefore, under no liability. Apart from that, subject to the question of the supply of materials, he was bound to do all that the
notice called on him to do. Ones sympathies are with the tenant here, because the landlords behaved very badly. They did not
answer letters; they did not supply the timber they were asked to supply and they were generally, through their agents, thoroughly
dilatory in dealing with the whole matter. If there was some relief against forfeiture which could be given, one would be glad to
do it, but the Agricultural Holdings Act, 1948, unlike the Law of Property Act, 1925, is all or nothing. Either the notice to quit is
good and out you go, or it is bad and you need not go.
What has been argued here is that the notice on the face of it is bad. The arbitrator considered the question of time, and
found that the time specified in the notice which concerned those matters which did not require materials to be supplied by the
landlords was reasonable, but that it was not reasonable as to the matters which required the supply of materials. Here the
controversy has largely been as to what the arbitrator meant when he said that. Counsel for the tenant very boldly opened his
case by jettisoning all the reasons in his notice of appeal, and saying that his case was that the time given always had been a bad
time and that it could be seen at the very start that six months would not be enough to do the whole of these thirty items, and,
therefore, that the tenant was entitled from the start to ignore the whole notice because it was bad and the breaches were breaches
on which the landlords could not rely because they had not given sufficient time for the repair of the whole of them. That was a
formidable argument, but to me, as to my lord, having looked as far as I can into such materials as we have, namely, the Stated
Case and the points of claim and defence put in by the parties and also certain letters between the solicitors when the Case Stated
was being settled, it does seem quite clear that nobody thought of that point before and that everybody was concentrating on the
point that, as the timber had not been supplied promptly, the time allowed was unreasonable. I think, like my lord, that, in the
end, that is all the arbitrator meant. The notice, therefore, was not bad on its face.
If that is right, then the position is this. The notice is served on 13 September 1961, and gives six months. When that time
expires, if the notice has not been complied with and the landlords are entitled to rely on it, they are in a position under the Act of
1948 to serve an effective notice to quit. What was the position at the date when they served the notice to quit? So far as
concerns all that part of the notice which involved the supply by them of timber or rubble stone, they could not rely on the notice.
But there remained a substantial list of things which the arbitrator had found the tenant had had reasonable time to do and which
had not been done and, therefore, in my judgment the tenant had failed to comply with the notice in writing served on him. I do
not think that the notice need be good in every single respect in order that the landlords can rely on it. It is enough if there are
substantial portions of it to which they can point as defects in performance on the part of the tenant. That is consistent with the
cases under the Conveyancing and Law of Property Act, 1881, to which Lord Denning MR has referred, which show quite clearly
that a notice served by a landlord, which contains within it matters which he cannot enforce, whether because there was no
covenant or 906 because the covenant had not been broken, is not invalidated thereby and that he may rely on other parts of it in
respect of which the tenant is in default. This machinery is intended to keep tenants up to the mark and, if they are given a
reasonable period within which to comply with a notice, they have no cause to complain if the landlord says: You are in breach
of your tenancy agreement and I give you notice to quit. In so far as the landlord overstates his right or does not satisfy, as here,
a condition precedent to the exercise of it, the tenant is entitled to take no notice, but that does not seem to me to excuse the
tenant from that which he can do and ought to do under the covenant and which he is given notice to do. Therefore, I think that
the learned county court judge was right.
There was cited to the learned judge a case which he thought bound him in the matter, Price v Romilly, a decision of Diplock
J, but the decision in that case was that, where a notice is served and the tenant complies with a great deal of it, it is no good him
coming to the court and saying: Substantially I have complied and those bits with which I have not complied ought not to be
fatal to me. I think that they ought not to be fatal to him, because there ought to be some relief against forfeiture, but, as the Act
of 1948 stands at present, there is no such relief. But it did appear in that case that there was a surprising anomaly, because it was
held by the arbitrator in respect of one of those breaches given in the notice that the time allowed for remedying was not
sufficient. If counsel for the tenants point were right, that would invalidate the whole notice and the decision must have been the
other way. The point was not taken and was just no more than mentioned by the learned judge. I am far from saying that, if it
had been taken, the judge would not have been entitled to decide as he did, and, if we decided as counsel for the tenant asks us to
do here, we should in effect be saying that Diplock Js decision was per incuriam. I do not see anything wrong with that decision.
I do not think that it was binding on the county court judge in this case because, where it really impinges on the present case, the
point was not canvassed. However that may be, there is nothing inconsistent with what we have decided here.
I would dismiss the appeal.
PEARSON LJ. I agree. At all material times the tenant was in breach of his tenancy. There were continuing breaches of his
obligations in respect of the upkeep of gates, fences and ditches. In that situation, the landlords served notice to remedy the
breaches. The effect of that notice was not to impose any new obligation to remedy breaches but to prepare the way for the
landlords to serve an effective and operative notice to quit if the tenant failed to comply with the notice which required him to
remedy all the breaches within a period of six months. That notice, when it was served, was a good and valid notice. It was at
that time reasonable to specify six months as the period within which all those breaches had to be rectified. On that point, there is
the important letter, which my lord has read from the tenants solicitors of 29 September 1961, and there is also the finding of the
arbitrator in para 11 of the Special Case, and I agree with what my lord has said as to the true construction of that finding.
Whatever it might have appeared to mean at first sight, it is right to take into account the parties statements of case under para 6
of Sch 6 to the Agricultural Holdings Act, 1948, and to take into account also the parties contentions as they are set out in the
Special Case, and, holding regard to those aids to interpretation, I think that it is quite clear that para 11 of the Special Case does
bear the meaning which my Lord has put on it.
The position, then, was that there was an initially valid notice in respect of all the breaches concerned. Then the landlords
failed to comply with their obligation to provide material for making good fences and gates. What was the effect of that? It
plainly was to disentitle the landlords from taking advantage of their 907 own wrong by seeking to rely on the tenants failure to
make good gates and fences for the purpose of enabling the landlords to serve an effective notice to quit. But that, in my view, is
all the effect which that failure of the landlords to perform their obligation ought to have. The position still remains that the
tenant was at all material times and still is in breach of his obligation to make good the hedges and ditches. Therefore, the
requirements of s 24(2)(d) of the Act of 1948 are satisfied, because, at the date when the notice to quit was given, on 22 March
the tenant had failed to comply with the landlords notice in writing within the period of six months in so far as that notice
required the tenant to remedy his breaches in respect of the hedging and ditching. Therefore, as the requirement of para (d) was
satisfied, the landlords were entitled to serve an effective notice to quit, and they did so.
Therefore, I agree that the decision of the learned county court judge should be upheld.

Appeal dismissed. Leave to appeal to the House of Lords refused.

Solicitors: Warmingtons agents for H P & H C Rigby, Sandbach (for the tenant); Bowcock & Pursaill, Leek (for the landlords).

F Guttman Esq Barrister.


[1963] 2 All ER 908

Roles v Nathan and Others


Roles v Corney and Others (Consolidated Actions)
TORTS; Tortious Liability

COURT OF APPEAL
LORD DENNING, MR, HARMAN AND PEARSON LJJ
13, 14, 15 MAY 1963

Occupier Negligence Visitor Common duty of care Chimney sweeps killed by carbon monoxide fumes while sealing up
sweep-hole Warnings by occupier disregarded Special risks ordinarily incident to calling Whether occupier liable
Occupiers Liability Act, 1957 (5 & 6 Eliz 2 c 31), s 2(2), (3) (b), (4) (a).

A building was centrally-heated by a boiler in which coke was used as a fuel, there being an old system to carry away the smoke
and fumes, which included a horizontal flue running from the boiler under the floor to a vertical flue which went up a chimney.
The smoke and fumes had to descend about two feet four inches into the horizontal flue and then pass along it for seventy feet.
In the vertical flue there was a sweep-hole, about twelve inches in diameter and nine feet above the ground. It was sometimes
difficult to get the boiler lighted up, the difficulty being to get a draught going along the flues. In December, 1958, the fire was lit
and there was a lot of smoke. A boiler engineer was consulted, who said that the flues needed cleaning. Two chimney sweeps
were called in, and, ignoring the engineers warning of the danger from fumes, one of them crawled into the horizontal flue. The
fire was let out and the chimney sweeps cleaned out the flues, but when the fire was re-lit there was further trouble with fumes
and smoke. Another expert was called in who advised that the fire should be withdrawn and told everyone present to get out into
the fresh air. The chimney sweeps said that they did not need any advice, but eventually were more or less dragged out by the
expert. Later, the expert made his inspection and gave his advice, and, in the presence of the chimney sweeps and of C, the
occupiers son-in-law (who was looking after the building, the occupier being ill), advised that, inter alia, the sweep-hole was to
be sealed up before the boiler was lit again, and that the chimney sweeps, while doing the sealing, ought not to stay too long in
the alcove. The following day, the fire was re-lit by the caretaker. By the evening, the chimney sweeps had not finished sealing
up the sweep-hole, and C arranged with them to finish the work the next morning. The next morning, both the chimney sweeps
were found dead by the sweep-hole. 908Apparently, they had returned the previous night to complete their work and had been
overcome by carbon monoxide fumes. In actions for damages brought by their widows against the occupier, the trial judge held
that the occupier was in breach of the common duty of care under s 2(2) of the Occupiers Liability Act, 1957 a, but found that
the chimney sweeps were guilty of contributory negligence. On appeal by the occupier,
________________________________________
a Section 2(2) is set out at p 912, letter e, post.

Held Pearson LJ, dissenting): the warnings given to the chimney sweeps by the expert on behalf of the occupier of the danger
which in fact killed them were enough to enable them to be reasonably safe, and therefore the occupier had discharged under s
2(4)(a) b of the Occupiers Liability Act, 1957, the common duty of care that he owed to the chimney sweeps (see p 914, letters a
and b, and p 915, letters c and d, post).
________________________________________
b Section 2(4)(a) is set out at p 913, letter e, post.

Per Lord Denning MR (Harman LJ dubitante): the occupier was under no duty of care to the chimney sweeps in regard to
the danger that caused their deaths, since the danger was a special risk, ordinarily incident to the exercise of their calling, against
which the occupier might expect them to guard, within s 2(3)(b) c of the Occupiers Liability Act, 1957, the sweeps having been
repeatedly warned about the danger (see p 913, letters b and d, and p 914, letters h and I, post).
________________________________________
c Section 2(3)(b) is set out at p 912, letter g, post.

Appeal allowed.

Notes
As to the common duty of care, see 28 Halsburys Laws (3rd Edn) 43, 44, para 39.
For the Occupiers Liability Act, 1957, s 2, see 37 Halsburys Statutes (2nd Edn) 834.

Cases referred to in judgment


Christmas v General Cleaning Contractors Ltd [1952] 1 All ER 39, [1952] 1 KB 141, affd sub nom General Cleaning
Contractors Ltd v Christmas, [1952] 2 All ER 1110, [1953] AC 180, 3rd Digest Supp.
Greene v Chelsea Borough Council [1954] 2 All ER 318, [1954] 2 QB 127, [1954] 3 WLR 12, 118 JP 346, 3rd Digest Supp.
London Graving Dock Co Ltd v Horton [1951] 2 All ER 1, [1951] AC 737, 36 Digest (Repl) 54, 296.
Indermaur v Dames (1866), LR 1 CP 274, 35 LJCP 184, 14 LT 484, affd (1867), LR 2 CP 311, 36 LJCP 181, 16 LT 293, 31 JP
390, Ex Ch, 36 Digest (Repl) 46, 246.

Appeal
This was an appeal by the defendants, Eric Corney, James Wood and Laurence Marks, the executors of Herbert Abraham Nathan,
deceased, from a judgment of Elwes J dated 24 January 1963. The plaintiffs, Mary Ellen Roles and Annie May Roles,
commenced separate actions as administratrices of estates of their dead husbands for damages under the Fatal Accidents Acts,
1846 to 1908, and the Law Reform (Miscellaneous Provisions) Act, 1934, against Herbert Abraham Nathan (trading as
Manchester Assembly Rooms), the occupier of the assembly rooms where their husbands, who were chimney sweeps, had died
on 12 December 1958. Mr Nathan having died, the proceedings were by order continued against the defendants as his executors,
and the actions were consolidated. Elwes J found that the occupier and the chimney sweeps were equally negligent, and awarded
the first plaintiff 263 10s and the second plaintiff 2,263 10s.

W D T Hodson QC and K W Dewhurst for the defendants.


J S Watson QC and E Sanderson Temple for the plaintiffs.

15 May 1963. The following judgments were delivered.

LORD DENNING MR. This case arises out of a tragic accident which took place on Friday, 12 December 1958, when two
chimney sweeps were overcome by 909 fumes, and died in the basement of the Manchester Assembly Rooms. Their widows
bring actions against the occupier, Mr Nathan, claiming that he was at fault and in breach of the duty of care which is now laid
down by the Occupiers Liability Act, 1957. It is very unfortunate that this case was tried so long after the accident. The actions
were not commenced until 2 1/4 years after the accident, and not tried till four years after. In the intervening time the caretaker, a
most important witness, had disappeared without trace. So the court, after this length of time, is in great difficulty in ascertaining
the facts.
In the Assembly Rooms there was a central heating boiler in which coke was used as a fuel. The boiler had been
manufactured in 1929, and was thus nearly thirty years old. There was an old system of flues to carry away the smoke and
fumes. One of these was a horizontal flue twenty-four inches in diameter which ran from the boiler for seventy feet along under
the floor. That led into a vertical flue eighteen inches in diameter which went up a chimney eighty feet in height. In the
horizontal flue there was an inspection chamber under a slab in the floor. In the vertical flue there was a sweep-hole about
twelve inches in diameter and nine feet about the ground. It was sometimes very difficult to get this boiler lighted up. The
difficulty was to get a draught going along the flues. When it was first lighted, smoke and fumes went into the atmosphere,
which cleared off when the fire got going well. In April, 1958, some repairs were carried out, and a firm of builders gave this
advice: If it smokes, light a fire at the foot of the flue to create a draught. In December, 1958, the occupier himself was ill in
hospital, and his son-in-law, Mr Corney, was looking after the Assembly Rooms for him. On 9 December 1958, the boiler was to
be going for the winter. The fire was lit. There was a lot of smoke. They called in Mr Gardner, a boiler engineer. He said that
the flues needed cleaning. So they called in two chimney sweeps to sweep the flues. They were the brothers Donald Roles and
Joseph Roles.
Now coke, when burning, gives off carbon monoxide gas, which is very dangerous because it cannot be seen or smelt. It is a
stealthy killer. This should be known to everyone who has anything to do with boilers. On the Tuesday, when the sweeps
arrived, Mr Gardner warned them of the danger from the fumes, but they took no notice. He described how they acted:

Donald crawled into the horizontal flue. I told him he should take care and not go in when the thing was just open.
Give it a chance to clear out a bit, but he said he knew, he was a flue cleaner for many years, and he knew what he was
doing. So we do not interfere with people who know what they are doing.

The fire was let out, and on the Wednesday the sweeps cleaned out the flue. On the Thursday, the boiler was lit up again, but still
there was trouble with the fumes and the smoke. So they called in an expert, Mr Collingwood. When Mr Collingwood got there
he saw the fumes and smoke, and he gave another warning; he advised that the fire should be withdrawn at once. He told
everyone to get out of the place altogether and get some fresh air. The two flue sweeps were not prepared to accept Mr
Collingwoods advice. He described what took place:

I ordered everybody out, and they just jumped into the hole, and that was it. I went over to them and warned them
against this, and I got foul language.

Mr Collingwood went on to say:

I ordered them out and told them the danger of the gas, and the use of it, and they said that they knew a damn sight
better; they had been in this business all their life; they did not need my advice.

He said that he repeated his warning two or three times, and added: they eventually came out, under duress. I had to more
or less drag them out After Mr Collingwood had made his inspection, the boiler was lit again. He created a draught by
lighting a bit of paper at the bottom of the vertical shaft 910 so as to get it hot. But that was only a temporary expedient. He
advised a permanent remedy, either a new flue or an induction fan in the base of the chimney. A new flue would cost a lot of
money. So they decided on an induction fan. This could not be provided at once, so they would have to light it up for a time
without the fan. Mr Collingwood then gave another warning which is crucial in this case. He advised the two sweeps, Mr
Corney and everyone there that the two vent holes (the inspection chamber and the sweep-hole) were to be sealed up before the
boiler was lit up. He said:

I repeated this warning quite a few times to everyone that was there, not only the Roles brothers. That was to
everyone in the placeof the danger of these gases.

The attitude of the sweeps, he said, throughout was that they were experts, and they really knew better than Mr Collingwood
himself. It does appear, however, that Mr Collingwood did contemplate the possibility that the boiler might be lit before the
holes were sealed up. He said that, from his own practical experience, he thought that he could seal the holes with the fire on.
He thought that the sweeps could have done it, but said:

It depends entirely how long they were in there. I myself could have gone in there and sealed them off and come back
out again, but I certainly would not have stayed in there any length of time.

He advised the sweeps, he said, while they were sealing up, not to stay too long in the alcove.
So much for the warnings that were given. On Friday, 12 December the fire was lighted. We do not know by whom, but
perhaps by the caretaker. The sweeps were working there that day with the fire on. On Friday evening, Mr Corney and Mr
Gardner went there. The sweeps had very nearly completed their work, but they had not finished sealing up the sweep-hole in the
vertical shaft. Mr Corney said: The job was not complete inasmuch as the cover over the flue hole was not cemented on. He
asked the sweeps about it. They said they had not enough cement to do it. It was too late to complete it then, but they would get
the necessary cement the following morning from a builders yard and complete the job. So Mr Corney left thinking they would
come back the next morning (the Saturday morning) to finish off the job. The judge asked him:

With the fire still alight? A.I did not think that could have been done with the fire alight. I was under the
impression the fire would be out in the morning, when they came back to do the job.

It now turns out that the sweeps must have got their cement that evening, and that they must have come back later that night.
Maybe the caretaker let them in. They came back and attempted to seal up the sweep-hole with an old dustbin lid and with
cement. The fire was on at the time. Whilst they were doing this, they were overcome by fumes and died. Their bodies were
found next morning. One body was just in the alcove, and the other a little way out, as though he had been trying to pull his
workmate out of the alcove. One of them had on his nose a mask of wadding with a nose clip. And so they died. When the
police inspected the boiler between 8 and 9 oclock in the morning, the fire was found to be brightly burning.
It is quite plain that these men died because they were overcome by fumes of carbon monoxide. It would appear to a layman
that the fumes must have come from the sweep-hole, but the judge on the evidence thought that they probably came from the
boiler. But I do not think that it matters. The fumes came from the boiler or the sweep-hole or both. The question is whether
anyone was at fault. The judge found Mr Corney guilty of negligence because he failed to take such care as should have
ensured that there was no fire until the sweep-hole had been sealed. He said: unhappily, he did not tell the caretaker to
draw the fire, or at any rate not to stoke it up. On this account he held that 911 Mr Corney was at fault, and the occupier liable.
But he found the two sweeps guilty of contributory negligence, and halved the damages. The judge said:

That negligence [of the chimney sweeps] consisted in the knowledge that there was gas about, or probably would be,
the way they ignored explicit warnings, and showed complete indifference to the danger which was pointed out to them in
plain language, and this strange indifference to the fact that the fire was alight, when Mr. Collingwood had said it ought not
to be, until the sweep-hole had been sealed.

The occupier now appeals and says that it is not a case of negligence and contributory negligence, but that, on the true
application of the Occupiers Liability Act, 1957, the occupier was not liable at all. This is the first time that we have had to
consider that Act. It has been very beneficial. It has rid us of those two unpleasant characters, the invitee and the licensee, who
haunted the courts for years, and it has replaced them by the attractive figure of a visitor, who has so far given no trouble at all.
The Act has now been in force six years, and hardly any case has come before the courts in which its interpretation has had to be
considered. The draftsman expressed the hoped that the Act would
________________________________________
d See Salmond On Tort (13th Edn) at p 512, p 513, note 51.

replace a principle of the common law with a new principle of the common law: instead of having the judgment of
WILLES, J. e construed as if it were a statute, one is to have a statute which can be construed as if it were a judgment of
WILLIES, J.
________________________________________
e In Indermaur v Dames, (1866), LR 1 CP 274 at p 288.

It seems that his hopes are being fulfilled. All the fine distinctions about traps have been thrown aside and replaced by the
common duty of care.
The Act says, in s 2(2):

The common duty of care is to take such care as in all the circumstances of the case is reasonable to see that the visitor
[note the visitor, not the premises] will be reasonably safe in using the premises for the purposes for which he is invited or
permitted by the occupier to be there.

That is comprehensive. At the circumstances have to be considered. But the Act goes on to give examples of the circumstances
that are relevant. The particular one in question here is in s 2(3):

The circumstances relevant for the present purpose include the degree of care, and of want of care, which would
ordinarily be looked for in such a visitor, so that (for example) in proper cases (b) an occupier may expect a person,
in the exercise of his calling, will appreciate and guard against any special risks ordinarily incident to it, so far as the
occupier leaves him free to do so.

That subsection shows that Christmas v General Cleaning Contractors, Ltd is still good law under this new Act. There a window
cleaner (who was employed by independent contractors) was sent to clean the windows of a club. One of the windows was
defective; it had not been inspected and repaired as it should have been. In consequence, when the window cleaner was cleaning
it, it ran down quickly and trapped his hand, thus causing him to fall. It was held that he had no cause of action against the club.
If it had been a guest who had his fingers trapped by the defective window, the guest could have recovered damages from the
club. But the window cleaner could not do so. The reason is this: The householder is concerned to see that the windows are safe
for his guests to open and close, but he is not concerned to see that they are safe for a window cleaner to hold on to. The risk of a
defective window is a special risk, but it is ordinarily incident to the calling of a window cleaner, and so he must take care for
himself, 912and not expect the householder to do so. Likewise, in the case of a chimney sweep who comes to sweep the
chimneys or to seal up a sweep-hole. The householder can reasonably expect the sweep to take care of himself so far as any
dangers from the flues are concerned. these chimney sweeps ought to have known that there might be dangerous fumes about
and ought to have taken steps to guard against them. They ought to have known that they should not attempt to seal up the
sweep-hole whilst the fire was still alight. They ought to have had the fire withdrawn before they attempted to seal it up, or at
any rate they ought not to have stayed in the alcove too long when there might be dangerous fumes about. All this was known to
these two sweeps; they were repeatedly warned about it, and it was for them to guard against the danger. It was not for the
occupier to do it, even though he was present and heard the warnings. When a householder calls in a specialist to deal with a
defective installation on his premises, he can reasonably expect the specialist to appreciate and guard against the dangers arising
from the defect. The householder is not bound to watch over him to see that he comes to no harm. I would hold, therefore, that
the occupier here was under no duty of care to these sweeps, at any rate in regard to the dangers which caused their deaths. If it
had been a different danger, as for instance if the stairs leading to the cellar gave way, the occupier might no doubt be
responsible, but not for these dangers which were special risks ordinarily incidental to their calling.
Even if I am wrong about this point, and the occupier was under a duty of care to these chimney sweeps, the question arises
whether the duty was discharged by the warning that was given to them. This brings us to s 2(4), which says:

In determining whether the occupier of premises has discharged the common duty of care to a visitor, regard is to be
had to all the circumstances, so that (for example)(a) where damage is caused to a visitor by a danger of which he had
been warned by the occupier, the warning is not to be treated without more as absolving the occupier from liability, unless
in all the circumstances it was enough to enable the visitor to be reasonably safe

We all know the reason for this subsection. It was inserted so as to clear up the unsatisfactory state of the law as it had been left
by the decision of the House of Lords in London Graving Dock Co Ltd v Horton. That case was commonly supposed to have
decided that, when a person comes on to premises as an invitee, and is injured by the defective or dangerous condition of the
premises (due to the default of the occupier), it is, nevertheless, a complete defence for the occupier to prove that the invitee
knew of the danger, or had been warned of it. Supposing, for instance, that there was only one way of getting into and out of
premises, and it was by a footbridge over a stream which was rotten and dangerous. According to Hortons case, the occupier
could escape all liability to any visitor by putting up a notice: This bridge is dangerous, even though there was no other way by
which the visitor could get in or out, and he had no option but to go over the bridge. In such a case, s 2(4)(a) makes it clear that
the occupier would not be liable. But if there were two footbridges, one of which was rotten, and the other safe a hundred yards
away, the occupier could still escape liability, even today, by putting up a notice: Do not use this footbridge. It is dangerous.
There is a safe one further upstream. Such a warning is sufficient because it does enable the visitor to be reasonably safe.
I think that the law would probably have developed on these lines in any case; see Greene v Chelsea Borough Council
([1954] 2 All ER 318 at p 325; [1954] 2 QB 127 at p 139), where I ventured to say:

knowledge or notice of the danger is only a defence when the plaintiff is free to act on that knowledge or notice so
as to avoid the danger.
913

But the subsection has now made it clear. A warning does not absolve the occupier unless it is enough to enable the visitor to be
reasonably safe. Apply s 2(4) to this case. I am quite clear that the warnings which were given to the sweeps were enough to
enable them to be reasonably safe. The sweeps would have been quite safe if they had heeded these warnings. They should not
have come back that evening and attempted to seal up the sweep-hole while the fire was still alight. They ought to have waited
till next morning, and then they should have seen that the fire was out before they attempted to seal up the sweep-hole. In any
case they should not have stayed too long in the sweep-hole. In short, it was entirely their own fault. The judge held that it was
contributory negligence. I would go further and say that, under the Act, the occupier has, by the warnings, discharged his duty.
I would, therefore, be in favour of allowing this appeal and entering judgment for the defendants.

HARMAN LJ. The facts of this case remain lamentably obscure, partly because the two chief actors whose acts or defaults have
caused the claim to be made by their widows (the plaintiffs) are dead, and partly because of the disappearance of the caretaker at
the relevant time of the building where the tragedy occurred. Moreover, there has been a great delay in starting the action, and
the events were over four years old at the time of the trial. This may make it more difficult for the plaintiffs to prove their case,
but, in my judgment, they must accept that disadvantage, which is largely of their own making. [His Lordship stated the facts,
and continued:] The plaintiffs case, as presented to us in this court, rests solely on the fact that, on the Friday, the fire of the
stove was lit by the caretaker (the occupiers agent) in the face of Mr Collingwoods advice not to relight it till the two vents were
sealed. The judge held that there was a breach of duty for which the occupier was liable, but which was mitigated by the
contributory negligence of the sweeps, who knew all the risks as well as the occupier himself. He seems to have thought the
plaintiffs fault to lie not in lighting the fire on Friday, but in a failure to see that it was put out.
The issue in the case depends entirely on the Occupiers Liability Act, 1957. The sweeps were visitors and were,
therefore, owed the common duty of care. This is defined by s 2(2) of the Act in these words:

The common duty of care is a duty to take such care as in all the circumstances of the case is reasonable to see that the
visitor will be reasonably safe in using the premises for the purposes for which he is invited or permitted by the occupier to
be there.

Some of the circumstances are stated in sub-s (3)(b), in these terms:

an occupier may except that a person, in the exercise of his calling, will appreciate and guard against any special risks
ordinarily incident to it, so far as the occupier leaves him free to do so.

Here the person is a chimney sweep, and the first question is: Would such a person appreciate and guard against the risk of
carbon monoxide gas? I should have thought that this was a special risk ordinarily incident to the trade of sweep. There was no
evidence on this point except the words of the sweeps themselves, who said that they knew all about this kind of risk. Clearly,
however, they did not appreciate the degree of risk for they did not guard against it; but I should have thought that the occupier
was entitled to take their word and was entitled to expect that they would take sufficient precautions having regard to the
emphatic warnings of the occupiers agent, Mr Collingwood, given the previous day. He said that he told them of the risks of
these gases more than once. Whether this fact would absolve the occupier from his duty of care, he himself having caused the
fire to be lit in the face of Mr Collingwoods advice may perhaps be doubtful.
In my judgment, the case in the end depends on s 2(4), which is in these terms:
914

In determining whether the occupier of premises has discharged the common duty of care to a visitor, regard is to be
had to all the circumstances, so that (for example)(a) where damage is caused to a visitor by a danger of which he had
been warned by the occupier, the warning is not to be treated without more as absolving the occupier from liability, unless
in all the circumstances it was enough to enable the visitor to be reasonably safe

There seems to me no doubt that the sweeps had been warned by the occupier through his agent, Mr Collingwood, of the danger
which killed them. That, however, as the section say does not without more absolve the occupier from liability. The crucial
question is whether, in all the circumstances, the warning was enough to enable the [visitors] (that is, the sweeps) to be
reasonably safe. In my judgment, it was. The occupier did not request or even authorise the sweeps to close the sweep-hole
while the fire was alight. Mr Corney did not expect the return of the sweeps on Friday night; they told him that they were coming
back in the morning. He had arranged that they should do the work on Saturday morning. He said also that he anticipated that
the fire burning on Friday night would be out by Saturday morning, thus making the work safe. It is true that the caretaker
apparently did not let the fire out, and it is said that Mr Corney failed in his duty because he did not expressly order the caretaker
to do so, nor did he expressly forbid the sweeps to attempt the work with the fire on. Nevertheless, these sweeps knew as much
about the danger as he did. There was no obligation on them to proceed without drawing the fire, they were free to do so, and
they deliberately chose to assume the risk notwithstanding the advice given.
In these circumstances, we ought to determine that the occupier did discharge the common duty of care, and, in my view, the
plaintiff widows cannot succeed in their claim. I would allow the appeal.

PEARSON LJ. I am sorry to be unable to agree with my brethren in the conclusion that they have reached. I think that the
difference of opinion is only as to the interpretation of the evidence, and not as to any question of law.
The arrangements for the disposing of the smoke and fumes from the boiler fire were at all material times defective. The
smoke and fumes had to descend about two feet four inches from the boiler pipe into the flue, and the flue was too wide (twenty-
four inches in diameter) and too long (seventy or eighty feet) and made of unsuitable-material, stoneware, which tends to cause
condensation. Consequently, there was trouble in what I will call the starting period, that is to say, the period from the moment of
lighting the fire until a good fire and a good draught were established. In the starting period, the flue and the chimney were not
properly warmed up, and the smoke and fumes were not sufficiently drawn along the flue to the chimney, and tended to escape
through the boiler itself through any available aperture or leak into the atmosphere of the cellar. The smoke and fumes were
likely to contain carbon monoxide gas. There is, however, no evidence of any accident prior to the fatal accident giving rise to
these proceedings. In April, 1958, the firm of Mr Sheldon, who gave evidence, were called in to deal with this starting trouble.
He said that they had trouble with the boiler, mainly in lighting it, on quite a few occasions. Smoke and fumes escaped into the
cellar. Mr Sheldons firm did repairs, and advised that, when the boiler was lit, there should also be a fire made in a dustbin
alongside the base of it in order to warm it up. On 9 December 1958, there was further trouble. Apparently the boiler was being
started, but it was not going well, and there was a lot of smoke because the smoke was not getting away as it should. Mr Gardner,
a boiler engineer, was called in, and he advised that the flues should be cleaned. Consequently, the two deceased brothers, who
were chimney sweeps, were summoned to do the cleaning. The elder brother, Donald Roles, crawled inside the horizontal flue.
Mr Gardner told him that he should take care and not go in when 915 the inspection clamber had just been opened, but Roles
replied that he knew, as he had been a flue cleaner for many years, and he knew what he was doing. The cleaning of the flue did
not cure the trouble. Accordingly, another expert, Mr Collingwood, was called in on Thursday, 11 December 1958. He found
that there was a lot of amoke in the cellar, the fire having been lit. He advised that the fire should be withdrawn, and that was
done. He also advised everyone to get out of the premises altogether into the fresh air. The two deceased ignored his advice, and
one of them jumped down into the inspection clamber. Mr Collingwood went over to them and warned them against doing that,
but they used foul language and told him that they knew better; that they had been in the business all their life and did not need
advice. Mr Collingwood more or less dragged them into the open air. Mr Collingwood had a discussion with Mr Corney, the
son-in-law of the occupant, who was the proprietor of the Assembly Rooms, and advised that a new flue should be put in, or
alternatively an induction fan should be provided at the base of the chimney to draw the smoke and fumes along the flue into the
chimney.
Now I come to the crux of the case. Mr Collingwood advised that the two access vents which had been opened upthat is
to say, the inspection chamber in the middle of the flue and the sweep-hole in the side of the chimneymust be sealed, and that
this must be done before the boiler fire was lit. That advice was given to everyone who was therecertainly, on the evidence,
to the two deceased and to Mr Corney. As to the caretaker, he either heard the advice, or he should have had it passed on to him
by Mr Corney. There was also advice from Mr Collingwood to the deceased that they were not to stay too long in the alcove
while they were sealing up the sweep-hole in the chimney. I do not think that meant that he expected them to be sealing up the
sweep-hole at a time when the boiler fire was burning, as that would be inconsistent with the previous and main advice that the
access vents should be sealed before the boiler fire was lit up. It would be wise not to stay too long in the alcove whether or not
the fire was burning, as there might be pockets of carbon monoxide lingering in the confined space of the alcove. Mr
Collingwood himself lit a temporary fire for testing the movement of the gases. He lit a fire with paper in the bottom of the
chimney, and the effect of it was to dray the gases along the flue into he chimney. It may be that, for the purposes of this test,
there was also a fire in the boiler, but if so, it was only temporary. Despite the advice which had been given by Mr Collingwood,
some agent of the occupier (presumably the caretaker) on Friday, 12 December lit the boiler fire, got it started and kept it going,
although the access vents, or at any rate one of them, the sweep-hole, had not been sealed. Mr Collingwood had warned the
occupers agent not to do that. There is evidence showing why it was dangerous. Mr Brierley, another expert, in his deposition at
the inquest, which was used as part of his evidence in the action, said this:

The fact that the hot gases must descend two feet four inches and travel along seventy feet of horizontal flue before
entering the vertical flue, will result in the draught being difficult to establish and consequently the boiler being difficult to
light. During the early periods of firing, prior to the draught being established adequately, it is inevitable that the products
of the combustion will be discharged into the basement at any point of leakage from the boiler, flue or chimney. If carbon
monoxide is present in the products of combustion at this stage, lethal concentrations will readily be produced.

On this occasion there was the added defect of a hole in the chimney. The effect of that must have been to diminish still further
the efficiency of the system, and to prolong the starting period in which smoke and fumes were escaping into the atmosphere of
the cellar and evidently putting some carbon monoxide into it. A concentration of four parts in a thousand is enough carbon
monoxide to be lethal to a human being in about ten minutes. By starting the fire in the boiler before the access vents had been
sealed, the occupiers agents were creating that 916 danger unnecessarily for the deceased chimney sweeps who were going to
complete the sealing of the access vents. Mr Corney came to the Assembly Rooms on the afternoon of Friday about 5 to 6
oclock to pay the sweeps. He retained 3 as they had not completed the work, the cover of the sweep-hole not having been
cemented on. They said that they had not the necessary cement to complete the job, and it was too late that evening to obtain any,
and they would, therefore, complete it the following morning when the builders yards were open, and they could get the
necessary cement. Mr Corney thought that the boiler fire would have to be put out on Saturday morning for the work on the
sweep-hole to be done. The boiler fire, however, would have to be lit again on Saturday as the Assembly Rooms were to be used
on Saturday evening, and the central heating would need to be in operation.
In the event, the deceased must have found some cement somewhere on Friday evening. They came back to the Assembly
Rooms on that evening, presumably with the knowledge of the caretaker, and started to fix with cement the cover of the sweep-
hole, although the boiler fire was burning; they were overcome with carbon monoxide fumes, and they died. Their bodies were
found on the following morning in the alcove. There was an overturned bucket of cement. On the face of one of the bodies there
was a breathing mask made of cotton wadding fitted on a small clip covering the nostrils and the mouth, and fastened round the
head with elastic bands. This mask was probably useless.
The questions to be decided arise under s 2 of the Occupiers Liability Act, 1957. Under sub-s (2) of the section, the
occupier has the common duty of care to his visitors; that is to say,

a duty to take such care as in all the circumstances of the case is reasonable to see that the visitors will be reasonably
safe in using the premises for the purposes for which he is invited or permitted by the occupier to be there.

Prima facie there was a breach of that subsection by the occupierthat is to say, the defendant in this casebecause the lighting
of the fire before the sealing of the access vents had been completed created a serious and unnecessary danger for the deceased
who were lawful visitors, invited to the premises for the purpose of doing this work. The lighting of the fire at that time caused
smoke and fumes containing carbon monoxide to enter the air of the cellar, and doubtless to linger in pockets in enclosed places,
including the alcove. Clearly on the balance of probabilities this was the cause (or at any rate, a cause) of the fatal accident.
Subsection (3) of s 2 provides, so far as material:

The circumstances relevant for the present purposes include the degree of care, and of want of care, which would
ordinarily be looked for in such a visitor, so that (for example) in proper cases (b) an occupier may expect that a
person, in the exercise of his calling, will appreciate and guard against any special risks ordinarily incident to it, so far as
the occupier leaves him free to do so.

In my view, such a visitor means a visitor of the relevant class, in this case chimney sweeps. The defendant, as occupier, was
entitled to expect that the deceased, in the exercise of their calling as chimney sweeps, would to some extent appreciate and guard
against the carbon monoxide risk. In fact, it appears that they did have some appreciation of the risk, and tried to guard against it
because the face mask was used. But, in my view, the actual risk in this caseresulting from the use of a defective installation,
when it had a serious additional defect, was not ordinarily incident to the sweeps calling. It was an extraordinary risk, as Mr
Collingwoods vigorous actions and emphatic warnings on Monday, 11 December sufficiently show. Sweeps are not expert
chemists or engineers, and it would be wrong to attribute to them a high degree of expert knowledge. Moreover, the defendant,
as occupier, had not left the deceased free to guard against the risk adequately, if I have correctly understood the true nature of the
risk. As I understand the evidence of Mr Brierley (which the learned judge 917 appears to have accepted), the risk did not consist
of doing the work on the sweep-hole cover while the boiler fire was burning well. At such a time the chimney would be hot, and
would be duly drawing the smoke and fumes from the boiler along the flue into and up the chimney; and at the sweep-hole there
would be an intake of air into the chimney and not an output of smoke and fumes from the chimney into the alcove. The risk
arose, so to speak, from an event of past history. The boiler had been lit, and the dangerous starting period had elapsed, at a time
when the defective installation was rendered still more defective by the hole in the chimney, and the fatal accident shows that
carbon monoxide had been left behind in the alcove. The deceased could not adequately guard against that risk. It can only be
said they they would have had a better chance if they had postponed the completion of the work until Saturday morning because
there would by then have been more time for the lingering pockets of carbon monoxide to disperse. The fire could have been put
out on Saturday morning, but there is no evidence that that would in itself have removed the carbon monoxide from the alcove.
In my view, therefore, the plaintiff widows claims are not defeated by s 2(3)(b) of the Occupiers Liability Act, 1957.
The opening words of s 2(4), of which para (a) is material, read as follows:

In determining whether the occupier of premises has discharged the common duty of care to a visitor, regard is to be
had to all the circumstances, so that (for example)(a) where damage is caused to a visitor by a danger of which he had
been warned by the occupier, the warning is not to be treated without more as absolving the occupier from liability, unless
in all the circumstances it was enough to enable the visitor to be reasonably safe

Substantially the same facts provide the plaintiffs with their answer to the occupiers argument under this subsection. The
deceased received adequate warning from Mr Collingwood, and, if the occupiers agents had heeded the warning, the warning
would have enabled the deceased to be reasonably safe. But the occupiers agents themselves, in disregard of the warning, did
the dangerous act of lighting the fire before the access vents had been sealed. That created the concentration of carbon monoxide
in the alcove. That was something done by the occupiers agents, and it could not be undone by the deceased, however much
they heeded Mr Collingwoods warning. In these circumstances, the warning did not enable the deceased to be reasonably safe;
and in my view, therefore, this paragraph of the subsection does not defeat the plaintiff widows claim. Accordingly, I agree with
the learned judges decision on the liability of the occupant, and do not have to consider his decision as to contributory
negligence because that is not disputed.
I would have dismissed the appeal, but that is a minority view, and of course, the majority view of my brethren will prevail,
so that the appeal will be allowed.

Appeal allowed. Leave to appeal to the House of Lords refused.

Solicitors: L Bingham & Co agents for James Chapman & Co Manchester (for the defendants); Fox & Gibbons Agents for Cecil
Franks & Co Manchester (for the plaintiffs).

F Guttman Esq Barrister.


918
[1963] 2 All ER 919

Re Macandrews Will Trusts


Stephens and Another v Barclays Bank Ltd and Others
SUCCESSION; Gifts
CHANCERY DIVISION
UNGOED-THOMAS J
9. 10 APRIL, 8 MAY 1963

Will Gift over Whether words of contingency should be construed otherwise than literally Share of residuary estate to
testators son, but if my said son shall not leave a child or widow who shall become entitled to an interest in his share,
ultimate gift over Son had no children but survived by widow Gift over subject to interest given to widow.

A testator by his will dated 31 March 1900, directed his trustees to hold a part of his residuary estate (called W.s share) during
the life of his son W on discretionary trusts for one or more exclusively of a specified class including W and W.s wife and issue
(if any), and from and after the death of W on trust to pay the income to his widow (if any), and from and after the death of the
survivor of W and his wife in trust for W.s children who being sons attained the age of twenty-one or being daughters attained
that age or married in equal shares. The testator then directed the trustees, but if my said son W shall not leave a child or widow
who shall become entitled to an interest in his share, to hold W.s share on trust for certain children of another son, I. F.

Held The reference in the condition quoted above to there being no child left fitted as an alternative to the absolute gift, making
a rational scheme of disposition, but the insertion of the further words or widow in the condition quoted was incongruous and
these words, if literally construed, were senseless in the context; having regard to the intention apparent from the will as a whole,
the words or widow should be read as in parenthesis, designed to indicate that the gift over was to be subject to the interest
given to the widow, and, accordingly, W.s share passed on the widows death under the gift over to children of I. F (see p 929,
letters e, h and i, post).
Anon. ((1683), 2 Vent 363) and principle of Maddison v Chapman ((1858), 4 K & J 709) applied.
Per Curiam: in the condition quoted above the word leave was to be construed as have and the words entitled to an
interest meant entitled to a vested interest (see p 923, letter i, to p 924, letter c, and p 924, letters e and g, post).
re Cobbold ([1903] 2 Ch 299 as corrected in [1915] 1 Ch 847 n) applied.

Notes
As to the rule in Lassence v Tierney a, see 39 Halsburys Laws (3rd Edn) 979, para 1482; as to the effect where words are
ambiguous in the context, see ibid, 976977, para 1478; as to departure from literal meaning of words, see ibid, 986, para 1493;
and for cases on the subject, see 44 Digest 579581, 39844023.
________________________________________
a (1849), 1 Mac & G 551.

Cases referred to in judgment


Anon (1683), 2 Vent 363, 1 Ch Ca 190, 86 ER 487.
Bird v Luckie (1850), 8 Hare, 301, 14 Jur 1015, 68 ER 375, 44 Digest 883, 7394.
Boden, Re, Boden v Boden [1907] 1 Ch 132, 76 LJCh 100, 95 LT 741, CA, 44 Digest 578, 3981.
Cobbold, Re, Cobbold v Lawton [1903] 2 Ch 299 (as corrected in [1915] 1 Ch 847n), 72 LJCh 588, 88 LT 745, CA, 44 Digest
1126, 9751.
Evans, Re, Public Trustee v Evans [1920] 2 Ch 304, 89 LJCh 525, 123 LT 735, 36 TLR 674, CA, 44 Digest 578, 3982.
Finch, Re, Abbiss v Burney (1880), 17 ChD 211, 43 LT 20, on appeal, (1881), 17 ChD 224, 50 LJCh 348, 44 LT 267, 44 Digest
583, 4058.
919
Key v Key (1853), 4 De GM & G 73, 1 Eq Rep 82, 22 LJCh 641, 22 LTOS 67, 43 ER 435, 44 Digest 580, 3999.
Lassence v Tierney (1849), 1 Mac & G 551, 2 H & Tw 115, 15 LTOS 557, 14 Jur 182, 44 Digest 554, 3715.
Maddison v Chapman (1858), 4 K & J 709, 70 ER 294, affd (1859), 3 De G & J 536, 28 LJCh 450, 44 ER 1375, 44 Digest 585,
4078.
Maunder, Re, Maunder v Maunder [1902] 2 Ch 875, 71 LJCh 815, 87 LT 262, affd [1903] 1 Ch 451, 72 LJCh 367, 88 LT 280, 44
Digest 1168, 10,112.
Mellish v Mellish (1798), 4 Ves 45, 31 ER 24, 44 Digest 596, 4204.
Noyce, Re, Brown v Rigg (1885), 31 ChD 75, 55 LJCh 114, 53 LT 688, 34 WR 147, 44 Digest 1168, 10,110.
Pearsall v Simpson (1808), 15 Ves 29, 33 ER 666, 44 Digest 1174, 10,157.
Redfern, Re, Redfern v Bryning (1877), 6 ChD 133, 47 LJCh 17, 37 LT 241, 44 Digest 564, 3816.
Whitrick, Re, Sutcliffe v Sutcliffe [1957] 2 All ER 467, [1957] 1 WLR 884, 3rd Digest Supp.

Adjourned Summons
This was an application by originating summons dated 10 December 1962, by the plaintiffs, Rodway Stephens and Travers
Lawrence Mooyaart, the trustees of the will dated 31 March 1900, of William Macandrew, deceased. The plaintiffs sought the
determination of the following question, whether, on the true construction of the above-mentioned will, and in the events which
had happened, the share of William Duncan Macandrew in the residuary estate of the testator was held (a) upon trust for Barclays
Bank Ltd the first defendants, as the personal representatives by representation of William Duncan Macandrew, deceased, or, (b)
upon trust for Katherine Alexis Charles, Rachel Mary Macandrew, the second and third defendants, and the estate of John Angus
Macandrew, deceased, in equal shares absolutely, or, (c) upon some other, and if so, what trusts.
The first defendants were the personal representative by representation of William Duncan Macandrew, deceased, who
survived the testator and died on 28 February 1921, without issue leaving a widow who died on 5 March 1962. The second and
third defendants were the two surviving children of the testators son Isaac Forsyth Macandrew, who died on 23 May 1931 (who
claimed to be beneficially interested under the trusts of the will). The facts and the relevant terms of the will appear in the
judgment.
the following casesb were cited during the argument in addition to those referred to in the judgment.
________________________________________
b
Webb v Hearing, (1617), Cro Jac 415; MLachlan v Taitt. (1860), 2 De GF & J 449; Re Smith, Veasey v Smith, [1947] 2 All ER 708; [1948] Ch
49; Re Follett, Barclays Bank Ltd v Dovell, [1955] 2 All ER 22; Re Bacharachs Will Trusts, Minden v Bacharach, [1958] 3 All ER 618;
[1959] Ch 245.

G B H Dillon for the plaintiffs.


Robert S Lazarus QC and E Wright for the first defendants.
R W Goff QC and N Micklem for the second and third defendants.

Cur adv vult


8 May 1963. The following judgment was delivered.

UNGOED-THOMAS J read the following judgment. In this case the plaintiffs are the present trustees of the will of the testator,
William Macandrew. The first defendant, Barclays Bank Ltd is the personal representative by representation of William Duncan
Macandrew, a son of the testator. The second and third defendants are children of Isaac Forsyth Macandrew, another son of the
testator. The question is whether William Duncans share in the testators residuary estate goes on the death of his wife, who had
a life interest in his share, to the children of Isaac Forsyth, under a gift over in the will. The value of the share is some 21,000.
If the gift over does not take effect, it is common ground that the share falls back into the estate of William Duncan under 920 the
doctrine of Lassence v Tierney). The question, therefore, is whether, in the events that have happened, the gift over takes effect,
and that depends on how words, apparently of contingency, introducing the gift over are to be construed.
The facts are, briefly, as follows. The testators will was dated 31 March 1900. There was a codicil to his will dated 6
March 1903. Incidentially nobody engaged in this case was responsible for drawing up these documents. The testator died on 18
December1905, and probate of the will and codicil was granted on 3 March 1906. William Duncan died on 28 February 1921,
and left a widow, Grace Nina Macandrew, who was his sole executrix and sole devisee and legatee. On 5 March 1962, William
Duncans widow died and Barclays Bank Ltd is her executor and thus personal representative by representation of William
Duncan.
On 23 May 1931, Isaac Forsyth Macandrew died. He had five children, of whom two are the second and third defendants.
Two children died under twenty-one. The fifth child attained twenty-one but died domiciled in South Africa last year. The
second and third defendants had a grant of representation to his estate in South Africa, but as he has no estate here, apart from the
interests with which we are concerned in this will, no grant of representation has so far been obtained in this country.
The will, so far as is material, is as follows. After appointing executors and trustees and giving certain legacies, the testator
proceeded:

To my dear wife Mariana Helen I give an annuity of 150 during her life to be paid to her quarterly on the first days of
January April July and October in every year the first payment to be made on the first of these days which shall happen
after my decease the sum of 100 to be paid to her within three months after probate hereof.

Then, later, he gave his residue to his trustees on trust for sale and to pay his funeral and testamentary expenses and debts in the
usual way, and he directed that they

shall also appropriate set apart and invest such a sum of money as will when invested be sufficient or such securities
as will be sufficient with the income thereof to pay and satisfy the said annuity of 150 hereinbefore given to my said wife
and shall retain the same as a fund to answer the said annuity and shall stand possessed of the residue of the said moneys
and also of the fund set apart to answer the said annuity subject to the same annuity (hereinafter called the residuary trust
funds) in trust as to two equal tenth parts thereof for my son Isaac Forsyth and as to the remaining eight equal tenth parts
in trust for all my other children who shall survive me in equal shares and if only one of my children shall survive me the
whole to be in trust for him or her but subject as to the shares of my sons Isaac Forsyth and William Duncan to the trusts
hereinafter declared concerning the same Provided always that if any of my children shall predecease me and there shall be
more than one living at my death the residuary trust funds shall be divided in such proportions that my son Isaac Forsyth
shall have twice the share that each of my other surviving children shall take.

And then later he proceeded:

Provided always and I hereby declare that if my said son Isaac Forsyth shall survive me my trustees shall retain his
share of and in the residuary trust funds including any share in such trust funds which may accrue to him by reason of the
death of any of his brothers or sisters or failure of their issue upon the trusts following that is to say in trust until my said
son Isaac Forsyth shall be discharged from all liabilities in respect of the Waikari Estate in the Colony of New Zealand to
invest or continue the investments thereof with power to vary such investments and to pay and apply the income thereof
and of the investments for the time being representing the 921 same for the maintenance and support or otherwise for the
benefit of all or such one or more exclusively of the other or others of the said Isaac Forsyth and his wife and issue for the
time being (if any) or if he shall for the time being have no wife or issue living then for the benefit of all or such one or
more exclusively of the others or other of the said Isaac Forsyth and his brothers and sisters nephews and nieces for the
time being in such manner in all respects as my trustees shall in their uncontrolled discretion think fit And from and after
the said discharge of my said son Isaac Forsyth from such liabilities as aforesaid in trust for him absolutely But if he shall
die before such discharge as aforesaid then from and after his decease in trust for all his children who being sons shall
attain the age of twenty-one years or being daughters shall attain that age or marry in equal shares and if there shall be only
one such child the whole to be trust for him or her And if there shall be no child of the said Isaac Forsyth who being a son
shall attain the age of twenty-one years or being a daughter shall attain that age or marry then in trust for all my other
children in equal shares Provided always that the said Isaac Forsyth may by deed or will appoint that the whole or any part
of the income of his said share shall after his death be paid to his widow (if any) during her life or for any less period
Provided always and I hereby declare that if my said son William Duncan shall survive me my trustees shall retain his
share of and in the residuary trust funds including any share in such trust funds which may accrue to him by reason of the
death of any of his brothers or sisters or failure of their issue upon the trusts following that is to say in trust during the life
of my said son William Duncan to invest or continue the investments thereof with power to vary such investments and to
pay and apply the income thereof and of the investments for the time being representing the same for the maintenance and
support or otherwise for the benefit of all or such one or more exclusively of the other or others of the said William Duncan
and his wife and issue for the time being (if any) or if he shall for the time being have no wife or issue living then for the
benefit of all or such one or more exclusively of the others or other of the said William Duncan and his brothers and sisters
nephews and nieces for the time being in such manner in all respects as my trustees in their uncontrolled discretion think fit
and from and after the death of my said son William Duncan to pay such income to his widow (if any) without power of
anticipation for her life And from and after the decease of the survivor of them my said son William Duncan and his wife in
trust for all his children who being sons shall attain the age of twenty-one years or being daughters shall attain that age or
marry in equal shares and if there shall be only one such child the whole to be in trust for him or her but if my said son
William Duncan shall not leave a child or widow who shall become entitled to an interest in his share my trustees shall
stand possessed of such share in trust for my said son Isaac Forsyth and after his death in trust for all his children who
being sons shall attain the age of twenty-one years or being daughters shall attain that age or marry in equal shares and if
there shall be only one such child the whole to be in trust for him or her.

Then, by the codicil, amongst other things, he provided as follows:

And whereas by my said will I have given and bequeathed to my said son Isaac Forsyth Macandrew a share of my
residuary trust funds therein particularly described upon certain trusts and provisions in my said will declared concerning
the same Now I hereby confirm the said gift to my said son but revoke the said trusts and provisions and power to appoint
the income thereof affecting the same and in lieu and stead thereof I direct that my trustees shall stand possessed of the said
share of my residuary trust funds in trust to invest the same in any of the securities authorised by my said will as
investments for trust funds thereunder and to pay the 922 income arising therefrom to my said son Isaac Forsyth
Macandrew during his life and after his decease to pay such income to his wife Katherine Alexis during her life and
widowhood and after the decease of the survivor of them my said son and his said wife or on her second marriage in trust
for all the children of my said son living at his decease and who being sons have attained or shall attain the age of twenty-
one years or being daughters have attained or shall attain that age or shall marry under that age in equal shares except that
my grandson William Forsyth Macandrew shall take a share equal to two of that to be taken by any other grandchild being
his brother or sister Provided that if there shall be only one child of my said son who shall attain a vested interest in the said
share of my residuary trust funds then the whole of such share shall be in trust for that one child and if there shall be no
child of the said Isaac Forsyth Macandrew who shall attain a vested interest then in trust for all my other children in equal
shares.

The crucial words are those which introduce the ultimate gift over of William Duncans share:

but if my said son shall not leave a child or widow who shall become entitled to an interest in his share.

Counsel for the second and third defendants submits that in this sentence leave must be construed as have or have had, that
interest means vested interest, and that accordingly the ultimate gift over, so far as contingent upon the reference to children,
dovetails into the prior gift to children. Thus, apart from the reference to William Duncans widow, if William Duncans children
do not take absolutely, the gift over takes effect absolutely. As William duncans widow does not take absolutely, but only for
life, the ultimate gift over cannot be a disposal of the absolute interest in the alternative to the gift of it to her; but counsel for the
second and third defendants submits that the gift over should be read as subject to her interest, just as, on his construction it
would, of course, be subject to the interest given to William Duncans children.
If leave were to be strictly construed as leaving a child surviving William Duncan, then a child who, being a son, attained
twenty-one, or being a daughter attained that age or married, during William Duncans lifetime and died before him, would have
his or her share divested by the gift over. It is well established that in those circumstances leave is to be construed so as not to
destroy any prior vested interest (Re Cobbold, Cobbold, v Lawton as corrected). In the contingent gift over by the codicil of
Isaacs share to his children, the gift is expressly to his children living at his decease, and this might suggest, particularly as the
codicil confirmed the will, that the testator had a similar intention with regard to the gift of William Duncans share to his
children. The gift to Isaacs children by codicil, however, was restricted to those living at his decease, whereas the gift to a child
of William Duncan by the will would not, even on the strict construction, be divested by his pre-deceasing William Duncan so
long as any one child survived William Duncan. And, in the will, the gifts of Isaacs and William Duncans shares were in such
different terms that it does not seem to me that the terms in which Isaacs share is given are of much assistance. I do not,
therefore, consider that the terms of this will and codicil are a sufficiently strong indication of a contrary intention to displace the
principle of construction stated in Re Cobbold.
If, however, leave is read as have or having had, and entitled to an interest is read as meaning entitled to any
interest, whether vested or contingent, then, once William Duncan had a child, although the child died immediately after birth,
the gift over could not take effect even though the property would not pass to the child. This would be giving the words of
contingency a meaning and effect that would restrict the possibility of the ultimate 923 gift over taking effect, and would do this
further than is necessary to prevent William Duncans children being divested of their vested interests. As the whole purpose of
the Re Cobbold principle is to avoid destroying any prior vested interest, the principle must be restricted to that purpose. Thus, if
leave is to be construed as have or having had, it seems to me to follow, so far as the Re Cobbold principle applies, that
entitled to an interest should be read as entitled to a vested interest. Entitled is commonly interpreted as entitled in
possession (see Re Noyce, Brown v Rigg; Re Maunder, Maunder v Maunder; Re Finch, Abbiss v Burney and the explanation of
Pearsall v Simpson, given in Jarman On Wills (5th Edn), p 765), and its use in the present context appears to me to indicate that
the testator had in mind a greater interest than a contingent interest. If the words child who shall become entitled to an
interest in his share are satisfied by becoming entitled to a contingent interest, then they are surplusage whether leave is
construed strictly or as have or have had, since a child would become entitled to a contingent interest on birth. Of course, a
testator may indulge in surplusage. As Fletcher Moulton LJ said in Re Boden, Boden v Boden c:
________________________________________
c [1907] 1 Ch 132 at p 143

A word may be inserted for the sake of emphasis or for greater clearness There is no presumption that human
beings use the irreducible minimum of words to effect their purpose.

Here, however, if the testator intended in the words of contingency to include a child entitled to a contingent interest, it would be
more emphatic and clearer if he had left out the words of surplusage altogether. They only have significance if they have a
limiting effect and I read them as referring to entitlement to a vested interest.
Counsel for the first defendants points out that in the contingent gift over by the will of Isaacs share in default of his having
specified children the words of contingency dovetail into the earlier gift to the specified children, and argues that when the
testator intended the gift so to dovetail, he employed the appropriate words. He further points out that when the testator wanted
to refer to a vested interest in the words introducing the gift over of Isaacs share, in the codicil, he used the word vested,
namely, in the phrase if there shall be no child of the said Isaac Forsyth who shall attain a vested interest. These arguments are,
to my mind, too remote and unreliable to vitiate a conclusion based on a consideration in accordance with well established
principles of the very words which have to be construed. Thus, on the construction which I favour, the gift over in the absence of
the specified children operates in the event in which the primary gift to them fails.
As, in fact, William Duncan never had a child, the only relevance of the reference to his not leaving a child is with regard to
the construction of the words or widow. But to construe the contingency as if my said son William Duncan shall not leave a
child or widow who shall become entitled to a vested interest in his share does not make it any more sensible in its relation to
the widow. It is obviously immaterial whether the reference is William Duncan shall not have or William Duncan shall not
leave a widow: and as William Duncans widow is only ascertained on his death, whereupon her interest becomes immediately
vested, it is immaterial whether she becomes entitled to an interest, or entitled to a vested interest. The words who shall
become entitled to an interest in his share are surplusage with reference to the widow, but are not, as construed, surplusage with
reference to a child.
Counsel for the first defendants argues that the insertion of the words or widow must have been deliberate as the testator
introduces the disposition in which they occur by the words from and after the decease of the survivor 924 of them my said son
William Duncan and his wife, so that he must have had the earlier death of the widow in mind when he made the gift over
dependent upon William Duncan not leaving a widow. Counsel relies on the omission from the codicil of any reference to widow
in the terms of contingency introducing the gift over of Isaacs share. He further points out that when the testator intended the
residue to be subject to the annuity to his wife, he says so in clear terms, thus indicating that he knew how to make one gift
subject to another. Those are valid considerations, but they cannot be conclusive, particularly if an analysis of the words to be
construed indicates a different conclusion.
On the other hand, it is argued that to make the gift over dependent on the condition of his widow surviving William Duncan
is senseless. Such a condition is not necessary for the protection of the widows life interest; and the gift of a life interest to the
widow would not exhaust the absolute interest the subject of the gift over. The consequential effect that, in the absence of the
specified children, the capital of the share would, on the widows death, pass to William Duncans estate absolutely under the
doctrine of Lassence v Tierney cannot in my view be ascribed to the testators deliberate intention. The testator did not
deliberately intend that, after providing for William Duncans widow and children, William Duncan should be in the same
position as those of the testators other children to whom he gave shares absolutely, since, clearly, if William Duncan had no
children or widow, the gift over would operate. Nor do I think that the testator can have intended that a child of William Duncan
who did not take under the testators will might nevertheless take under some different disposition made by William Duncan.
Thee is thus here, in my view, no positive indication of intention that the property should go to William Duncans estate if he had
a widow or child who did not take as specified. The question therefore is whether, in the absence of such positive indication of
intention, the words of contingency in relation to the widow can and should be construed otherwise than literally. Counsel for the
second and third defendants contends that they should be construed so as to read the ultimate gift over as subject to the widows
interest. He suggests that this can be done either by omitting the words or widow, or by transposing and amplifying the words
or widow so that after share at the end of the condition there appears instead the phrase then or on the death of the widow.
The same effect would result if or widow were read as words in parenthesis to emphasise the priority of her interest and this
appears to me more consonant with the testators language than the suggested omission or the suggested transposing and
amplifying.
I was referred to passages in Jarman On Wills (8th Edn). At p 592 under the heading As to supplying words it is stated:

Where it is clear on the face of a will that the testator has not accurately or completely expressed his meaning by the
words he has used, and it is also clear what are the words which he has omitted, those words may be supplied in order to
effectuate the intention as collected from the context.

At p 603 under the heading As to the transposition of words and clauses it is stated:

It is quite clear says Mr. Jarmand that, where a clause or expression, otherwise senseless and contradictory, can be
rendered consistent with the context by being transposed, the courts are warranted in making that transposition.
________________________________________
d Jarman on Wills (1st Edn), p 437.

At p 606 under the heading As to changing words it is stated:

To alter the language of a testator, as Mr. Jarman points out e, is evidently a strong measure, and one which, in
general, is to be justified only by a clear explanatory context. It often happens, however, that the 925 misuse of some word
or phrase is so palpable on the face of the will, as that no difficulty occurs in pronouncing the testator to have employed an
expression which does not accurately convey his meaning. But this is not enough: it must be apparent, not only that he
used the wrong word or phrase, but also what is the right one; and, if this be clear, the alteration of language is warranted
by the established principles of construction.
________________________________________
e Jarman on Wills (1st Edn), p 441.

It is clear that in every such case, before the court can depart from the words as strictly interpreted in order to give effect to the
testators intention, the court must be satisfied, first that there is a mistake, and secondly what the testators intention was as made
apparent from within the four corners of the will (Re Whitrick, Sutcliffe v Sutcliffe, Mellish v Mellish(1798), 4 Ves 45 at p 49)).
But, as stated by Romer LJ in Re Whitrick ([1957] 2 All ER 467 at p 473):

It is not necessary, of course, to know the precise language which the testatrix must be deemed to have had in her
mind: it is sufficient if one knows she did intend to make a particular provision and what that provision was.

Sellers LJ expressed himself in much the same sense. That was a case of omission in the will, but the observation is, in my view,
applicable generally to all cases of departure from the words as they stand in the will.
Counsel for the first defendants urged on me that there was no ambiguity in the relevant words in this case and that,
therefore, the court was obliged to give effect to this unambiguous meaning despite its folly. In Re Evans, Public Trustee v
Evans, Warrington LJ stated ([1920] 2 Ch 304 at p 320):

if there be no ambiguity, however unfortunate it may be that the intention of the testator shall fail, there is no right in
any court of justice to say those words shall not have their plain and unambiguous meaning.

In that case it was held that, in the gift by the testator of an annuity to his son unless he attemptsto become bankrupt, the
quoted words did not apply to a bankruptcy in invitum. As Knight Bruce, V-C, said in Bird v Luckie ((1850), 8 Hare, 301 at p
306):

no man is bound to make a will in such a manner as to deserve approbation from the prudent, the wise, or the good. A
testator is permitted to be capricious and improvident, and is, moreover, at liberty to conceal the circumstances and the
motives by which he has been actuated in his dispositions. Many a testamentary provision may seem to the world arbitrary,
capricious and eccentric, for which the testator if he could be heard, might be able to answer most satisfactorily.

There is, however, a difference between a plain statement in common language as in Re Evans and the expression of
intention in circumstances which give scope for the proverbial blundering attorneys clerk referred to in Re Redfern, Redfern v
Bryning ((1877), 6 ChD 133 at p 138). In this case, as Re Cobbold, Cobbold v Lawton makes clear, the language cannot be
regarded as a plain statement in common language which precludes mistake in expression.
In Key v Key where the testator gave an estate to SK for life charged with life annuities, and proceeded:

But in case the aforesaid annuitants, or any of them, shall survive the said S.K. I then givethe aforesaid estateunto
B charged with the aforesaid annuities,
926
it was held that the limitation to B should not be construed literally so as to make it dependent on SK being survived by one
of the annuitants. There the words in case the aforesaid annuitants or any of them shall survive the said SK were transposed
and treated as if they were inserted immediately before the word charged. And Knight Bruce LJ ((1853), 4 De GM & G 73 at p
79), stated that the testator mentioned the annuitants and the annuities

for the mere purpose of declaring that the property is to remain liable to the annuities, until their extinction
respectively, not with a view to make the title of any person to the property dependent on the life or death of any
annuitant.

Similarly here, if the testator mentioned the widow for a corresponding purpose the reference to her could be treated in the same
way as the reference to the annuitants was treated in Key v Key. In this case that effect could not be achieved by a mere
transposition of words as in that case; but, as appears from Re Whitrick, Sutcliffe v Sutcliffe, the ascertainment of the testators
intention does not depend on the solution of a verbal jigsaw puzzle.
In Pearsall v Simpson, the testatrix gave money on trust to pay the interest to Richard Stafford for life, and after his death,
in case he shall become entitled to such interest, then over to cousins of the testatrix. It was held that although Richard Stafford
never became entitled to such interest, the gift over took effect. Sir William Grant MR said ((1808), 15 Ves 29 at p 33):

The only question is, whether Richard Staffords taking for life was a condition precedent to the cousins of the
testatrix taking the capital. That would be a most absurd condition undoubtedly; for there is no sense or reason, making the
right of her first cousins depend upon a fact, totally unconnected with any intention as to them. What was it to them,
whether Richard Stafford took, or not. Such a construction is not to be made; unless absolutely necessary It was
doubtful whether Richard Stafford would live to become entitled to the interest. The testatrix, giving the capital over after
his death, recollects that he may not live to take the interest: but, if he does, she makes his death the period at which her
first cousins are to take. It is not a condition precedent, but fixing the period at which the legatees over shall take; if he
ever takes. The words will bear that construction; and the reason of the thing seems to require it.

As in Key v Key, words which literally are words of condition are construed as merely expressing that the gift over is subject to
the prior interest of the named beneficiary. What constrained Sir William Grant MR to his conclusion was that

there is no sense or reason making the right of her first cousins depend upon a fact, totally unconnected with any
intention as to them.

Equally, in the case before me there is no sense or reason making the right of Isaac Forsyth or his children under the gift over
depend on a fact, namely the coming into being of a widow of William Duncan, totally unconnected with any intention as to
them.
In Maddison v Chapman, the testator, who had a wife and two daughters, gave by his will property to be divided equally
between his wife and children when the younger child attained twenty-one years, with a provision that in the event of a child
dying before division her share should go to her surviving sister, but if she died before division leaving issue, then to such issue,
and he provided by codicil that should both my children die in their minority and leave no issue, 927then in such case, and in
such case only, he gave the property over. Both children died without issue, but the elder daughter had attained twenty-one
years. Page-Wood, V-C, laid down the proposition referred to as the rule in Maddison v Chapman ((1858), 4 K & J 709 at p 719):

where there is a limitation over, which, though expressed in the form of a contingent limitation, is, in fact, dependent
upon a condition essential to the determination of the interests previously limited, the court is at liberty to hold that,
notwithstanding the words in form import contingency, they mean no more, in fact, than that the person to take under the
limitation over is to take subject to the interests so previously limited.

There Page-Wood, V-C., was directing his mind to a condition essential to the determination of the interest previously limited,
whereas here we are concerned with a condition essential to the non-commencement of the previous interest and that, counsel for
the first defendant says, makes the rule in Maddison v Chapman inapplicable to this case. The reference in the condition to the
non-commencement of the previous interest given to the children, as construed, dovetails the gift over as an alternative absolute
gift to the absolute gift to the children and the only difficulty arises with regard to the reference to the widow, who is not given an
absolute but only a life interest. Page-Wood, V.-C., immediately after laying down the proposition which I have quoted,
establishes a test for its application. He says):

I apprehend the true way of testing limitations of that nature is this: Can the words which in form import contingency
be read as equivalent to subject to the interests previously limited?.

Later, Page-Wood, V.-C., in explaining why that test was not satisfied in the case before him, states ((1858), 4 K & J 709 at p
720):

The conditionthe death of the eldest daughter before attaining twenty-oneis not merely an event essential to the
determination of the interests previously given to her, but involves a further incident which may or may not have happened
when that estate is determined.

He adds that he is not therefore at liberty on the authority of Pearsall v Simpson and other cases to construe the condition as a
condition over instead of a conditional limitation. He states ((1858), 4 K & J 709 at p 720):

And the reason is sufficiently clear. When I find a testator expressing this varied contingencyby his will giving an
interest which may be determined by a death after minority, and by his codicil making a limitation over, which is only to
take effect in the event of death during minorityit is impossible to know what he intended, or to foresee what he would
have said had it been called to his attention that the two limitations did not coincide. In such a case, so far from feeling the
certainty upon which KNIGHT BRUCE, L.J., acted in Key v. Key, I confess I am quite uncertain as to what the testator
would have done had the case been explained to him.

In the case before me, if William Duncan should not have a widow then the interest given to her would be eliminated and the
condition imports no further incident than is involved in that elimination. The not coming into being of the widow is, like the
death of the widow, a particular example of events which eliminate the interest given to her. The example which Page-Wood, V-
C, gave of satisfying his test was: A limitation to A for life, remainder to B for life, and, upon the decease of B, if A be dead,
then to C in fee, and he observes ((1858), 4 K & J 709 at p 719).
928
There the limitation to C is apparently made contingent upon the event of As dying in the lifetime of B. Nevertheless,
inasmuch as the condition of As death is an event essential to the determination of the interest previously limited to him,
the court reads the devise as if it were to A for life, remainder to B for life, and on Bs death, subject to As life interest (if
any), to C in fee.

There, as expressly indicated, the gift over to C would take effect even if A died before taking any interest at all, and the reference
to the death of A in the lifetime of B, though requiring his death within a specified time, namely the lifetime of B, was a particular
instance of death which determined or eliminated his interest. It may be said that the express words of condition there, namely,
If A be dead, cover As death before A could take any interest, but the illustration nevertheless indicates that it was considered
that non-commencement of an interest was within the rule in Maddison v Chapman. In Anon, after a devise to Katharine and the
heirs of her body and if she died without issue to Jane for life, the testator provided that if Katharine died without issue and Jane
be then deceased, then and not otherwise he gave the land over to J N. Katharine died without issue and Jane survived her and
died. It is reported ((1683), 2 Vent. at p 364):

My Lord Keeper decreed it for J.N. altho Jane survived Katharine; because the words (if Jane be then deceased)
seemed to be put in to express his meaning, that Jane should be sure to have it for her life, and that J.N. should not have it
till she were dead; and also to shew when J.N. should have it in possession.

The parallel between Jane and the widow, and between Katharine and William Duncan, is close. It seems to me that similarly
here the words or widow were inserted to emphasise that the widows interest was not to be prejudiced by the gift over, and that
the gift over was to be subject to the prior disposition.
The fundamental and overriding duty binding the court is to ascertain the intention of the testator as expressed in his will
read as a whole. Such rules of construction as are laid down in Maddison v Chapman are valuable guides for that purpose, but
not more. They are to be construed in the light of their own facts and are not rigid canons of construction which by implication
exclude from the principle of their operation that which they do not in terms expressly include. They are to be construed in due
subjection to the terms of the will. The rule in Maddison v Chapman was directed to the facts of that case, and it seems to me
that this case is within the reasoning and principle on which that case was based. Even if this were not so, I am not precluded
from reading the gift over of William Duncans share as being merely subject to the prior gift, just because the relevant words
introducing the gift over are not within the precise words of the rule in Maddison v Chapman.
In this case, the dominating consideration, to my mind, is that the words or widow, if literally construed, are so completely
senseless in the context of this will, which is not distinguished by deliberate and erratic folly, and that this senselessness occurs in
the kind of passage which gives scope for the blundering attorneys clerk ((1887), 6 Ch D 133 at p 138). They are, literally
construed, an incongruous insertion in what is, otherwise construed, a rational and normal scheme of disposition. I am convinced
by considerations limited within the ambit of the will that a literal construction would not be in accordance with the testators
intention, and I am satisfied that the proper construction of the words or widow would treat them as in parenthesis, designed to
indicate that the gift over was to be subject 929 to the interest given to the widow just as, on construction, the gift over was
subject to the interest given to William Duncans children.

Declaration accordingly.

Solicitors: Burton, Yeates & Hart (for the plaintiffs); Field, Roscoe & Co (for the first defendants); Allen & Overy (for the
second and third defendants).

Jacqueline Metcalfe Barrister.


[1963] 2 All ER 930

I & H Caplan Ltd v Caplan and Another (No 2)


LANDLORD AND TENANT; Tenancies

CHANCERY DIVISION
CROSS J
23, 24, 27, 28 MAY 1963

Landlord and Tenant New tenancy Business premises Right to new tenancy conditional on tenants continuing throughout
proceedings to be tenant under tenancy to which Part 2 of the Act of 1954 applies Continuity of business user Tenant ceasing
to trade from premises and premises left empty for period during pendency of application for new tenancy Tenants intention to
discontinue lines of existing business and, if granted new tenancy, to start another but similar line of business Whether tenant
ceased to occupy premises for the purposes of a business Landlord and Tenant Act, 1954 (2 & 3 Eliz 2 c 56), s 23(1).

It is a continuing condition of a tenants right to a new tenancy under Part 2 of the Landlord and Tenant Act, 1954, that he should
be, throughout the proceedings, tenant under a tenancy to which Part 2 of the Act of 1954 applies; but a tenant does not lose the
protection of Part 2 of the Act of 1954 simply by ceasing physically to occupy the premises, provided that the thread of continuity
of business user is not broken (see p 936, letter e, and p 938, letter f, post).
On 15 February 1960, the applicant companys summons under s 24 of the Landlord and Tenant Act, 1954, for a new
tenancy of the premises on which they then carried on the business of selling mens clothing, ladies hosiery and mens, womens
and childrens footwear, was dismissed. The company thereupon decided both to appeal and to cease trading on the premises.
They inserted notices in the press stating that the shop would close on 9 April 1960, and advertised and conducted a closing-down
sale. Trading on the premises ceased on 9 April 1960, and during the following week all the remaining stock, with the exception
of a quantity of footwear, was removed, the greater part being sold at cost price to another company, which on 16 April 1960,
opened a shop for the same line of business as that formerly carried on by the company. Some fittings were also removed, but the
water, electricity and gas services were not disconnected. The court found that it was the companys intention during the period
between 9 April 1960, and 15 November 1960, to discontinue their former line of business and to use the premises, if they were
granted a new tenancy, solely as a ladies garment shop. On 15 November 1960, the companys appeal against the dismissal of
the originating summons was allowed, and the Court of Appeal referred the case back to the Chancery Division for determination
of a question, in consequence of which the company immediately re-commenced trading at the premises in the same goods as
formerly with the addition of some ladies garments. The landlords now applied for a declaration that the company, either by
going out of possession or occupation of the declaration or by ceasing to trade thereon after Arp. 9, 1960, had lost the protection
of the Landlord and Tenant Act, 1954, Part 2.
Held The tenancy of the company continued throughout to be one to which Part 2 of the Landlord and Tenant Act, 1954,
applied because, notwithstanding that the company had ceased trading at the premises between 9309 April and 15 November
1960, the company had not ceased to occupy the premises for the purposes of a business within the meaning of those words in s
23(1) of the Act of 1954, the facts being such that the thread of continuity of business user had not been broken (see p 939, letter
d, post).
Teasdale v Walker ([1958] 3 All ER 307) distinguished.

Notes
As to tenancies to which Part 2 of the Landlord and Tenant Act, 1954, applies, see 23 Halsburys Laws (3rd Edn) 885, para 1707;
and for cases on the subject, see 3rd Digest Supp.
For the Landlord and Tenant Act, 1954, s 23(1), (3), s 24(1), (3), s 25(1), (6), s 29, s 30(1), s 32(1), s 64, see 34 Halsburys
Statutes (2nd Edn) 408411, 413, 414, 417, 441.

Cases referred to in judgment


I & H Caplan Ltd v Caplan [1961] 3 All ER 1174, [1962] 1 WLR 55, 3rd Digest Supp.
Teasdale v Walker [1958] 3 All ER 307, [1958] 1 WLR 1076, 3rd Digest Supp.

Adjourned Summons
On 14 April 1959, the applicant company, I & H Caplan Ltd applied by originating summons for the grant under s 24(1) of the
Landlord and Tenant Act, 1954, of a new tenancy of business premises at Peel Square, Barnsley, of which the company were
tenants and the respondents, Maurice Caplan and his wife Glaydys Caplan, were landlords. The application came finally on
appeal before the House of Lords (reported [1961] 3 All ER 1174). The case was remitted to the Chancery Division for the
determination of the question whether the company were entitled to the grant of a new tenancy of the premises, or of a part
thereof, regard being had to all relevant events which have happened between 14 April 1959, and the date on which the court
shall determine whether to order the grant of a new tenancy. The respondent landlords applied by summons, in the action still
proceeding as aforesaid, for a declaration

that the applicant company either by going out of possession of occupation of the premises at the corner of Peel
Square and Market Street, Barnsley, or by ceasing to trade thereon after 14 April 1959, have lost the protection of the
Landlord and Tenant Act, 1954, Part 2.

Accordingly His Lordship (Cross J) stated that what the court had to decide on the present summons was, first, whether a tenants
right to call for a new lease or tenancy was subject to a continuing condition that the tenancy remained a tenancy to which Part 2
of the Act of 1954 applied throughout the proceedings, and secondly, if the tenants right was subject to such a continuing
condition, then whether the tenancy in the present case had at all times been within Part 2 of the Act of 1954.
The cases noted belowa were cited during the argument in addition to those referred to in the judgment.
________________________________________
a Evans v Horner [1925] Ch 177, R F Fuggle Ltd v Gadsden [1948] 2 All ER 160, [1948] 2 KB 236, Nichols v Walters [1953] 2 All ER 1516,
King v Taylor [1954] 3 All ER 373, [1955] 1 QB 150, Bagettes Ltd v G P Estates Co Ltd [1956] 1 All ER 729, [1956] Ch 290, Gold v
Brighton Corpn [1956] 3 All ER 442, Bettys Cafs Ltd v Phillips Furnishing Stores Ltd [1958] 1 All ER 607, Lyons v Central Commercial
Properties Ltd [1958] 2 All ER 767.

L A Blundell QC and K R Bagnall for the applicants, the landlords.


Michael Albery QC and Jeremiah Harman for the respondents, the company.

28 May 1963. The following judgment was delivered.

CROSS J. This application is a further stage in proceedings under the Landlord and Tenant Act, 1954, which have already lasted
five years. The applicants, Maurice Caplan and Gladys Caplan, are the freeholders of premises at Peel Square in Barnsley. The
respondents, I & H Caplan Ltd had been their tenants for many years on a yearly tenancy at a rent of 750 a year and 931 had
been carrying on business there as outfitters. Maurice Caplan holds forty per cent of the shares in the company but the company
is controlled by other members of the Caplan family who hold the remaining sixty per cent of the shares. Since about 1957
Maurice and the rest of the family have not seen eye to eye and these proceedings are not the only litigation in which they have
been engaged.
On 23 December 1958, Maurice and Gladys Caplan, whom I will call the landlords, served notice on the company
determining its tenancy on 24 June 1959. That notice was in proper form and there is no doubt that if the company had not
invoked the protection of the Landlord and Tenant Act, 1954, its tenancy would have come to an end with the expiry of the
notice. On 14 April 1959, however, the company issued an originating summons to which the landlords are respondents asking
for an order under s 24 of the Act for a new tenancy which, it was suggested, should be for fourteen years at a rent of 2,300 a
year. The landlords opposed that application on the ground set out in s 30(1)(g) of the Act, namely, that they themselves intended
to carry on business as retail outfitters at the premises on the expiry of the companys tenancy. In answer to that, the company
contended that the landlords had not and could not have such an intention, because they held the freehold not beneficially but as
trustees for their infant children and so could not lawfully carry on business themselves on the premises without an order of this
court enabling them to do it, which order they might in the circumstances very well not obtain. On 15 February 1960, Lloyd-
Jacob J decided that issue in favour of the landlords and dismissed the companys applications for a new lease.
On 29 April 1960, the company gave notice of appeal from that decision; but shortly before thento be precise, on 9 April
the company ceased to trade on the premises and the landlords, having discovered that fact, applied to the Court of Appeal by
motion dated 14 July 1960, for an order that the appeal be struck out on the ground that, even assuming that the landlords
objection under s 30(1)(g) failed, the company could not, in the circumstances, claim a new lease. No order was made on that
motion when it came on in July. The appeal came on on 15 November 1960, and the Court of Appeal reversed the decision of
Lloyd-Jacob J on the ground that the uncertainty whether or not the court would allow the landlords to carry on business on the
premises as trustees for their children made it impossible to say that the landlords had an intention to do so within the meaning
of s 30(1)(g) of the Act.
On the other point raised by the landlords the Court of Appeal took the view that the relevant date for determining the
question whether the company had ceased to occupy the premises for the purposes of the business was the date of their order,
namely, 15 November 1960, and they sent the case back to the Chancery Division to determine the following questions:

(1) Whether at the date of this order and in the events which have happened (including events since the said Feb. 15,
1960) [the date of LLOYD-JACOB, J.s judgment]) the applicants are entitled to the grant of a new tenancy of the said
holding or some and if so which part thereof under s. 24 and s. 29 of the Landlord and Tenant Act, 1954. (2) If the answer
to question (1) is in the affirmative what are the proper terms as regards rent duration or otherwise of such tenancy.
That order did not suit the companys book at all. As soon as its advisers appreciated the effect of what the Court of Appeal was
saying, they took immediate steps to cause the company to start trading on the premises again and in point of fact it has continued
to trade on the premises since shortly after 15 November but the time factor made it very difficult, if not impossible, effectively to
start trading again on that very day.
Accordingly, the company appealed to the House of Lords on this point and 932 on 6 December 1961, the House discharged
the part of the order of the Court of Appeal which I have read and substituted for it the following declaration:

That, in determining whether or not [the company] are entitled to the grant of a new tenancy of premises situate at
Peel Square, Barnsley, and, if so, of which part thereof, regard shall be had to all relevant events which shall have happened
between Apr. 14, 1959, and the date upon which the court shall determine whether or not to grant a new tenancy.

That declaration was carefully framed in order to leave it open to the landlords to argue that the fact that the company was
carrying on business on the premises at the date when the court finally determined whether or not to grant a new tenancy would
not enable the company to claim a new tenancy if in fact there had been some period of time after the issue of the summons when
no part of the premises was occupied for business purposes. Lord Reid, with whom the other lords concurred, dealt with that
point in his speech in the following terms. He said ([1961] 3 All ER 1174 at p 1177):

It only remains to notice a further argument for [the landlords]. They maintain that a tenant is not entitled to the grant
of a new tenancy unless the existing tenancy complied with the requirements of s. 23(1) not only at the date of the
application for a new tenancy and at the date of the order granting a new tenancy but also throughout the intervening
period. They say that if at any time during this interval the tenancy does not comply with these requirements, the
protection of the Act flies off and any proceedings under the Act, such as an application for a new tenancy, abate and
cannot be revived by any subsequent change of circumstances. Again, I do not propose to express any opinion about this.
If the facts proved in the inquiry make this argument relevant, it will be dealt with by the judge to whom the case is
remitted.

The application which I now have before me is not, even yet, the companys originating summons asking for a new tenancy. It is
a summons within that summons taken out by the landlords for the purpose of raising the point reserved by the House of Lords,
and by it the landlords ask for

A declaration that the applicants either by going out of possession or occupation of the premises at the corner of Peel
Square and Market Street, Barnsley or by ceasing to trade thereon after Apr. 14, 1959 have lost the protection of the
Landlord and Tenant Act, 1954, Part 2.

What I have to decide then, as I understand it, is, first, whether a tenants right to call for a new lease lease or tenancy is subject to
a continuing condition that the tenancy remains a tenancy to which Part 2 of the Act of 1954 applies throughout the proceedings,
and, secondly, if the tenants right is subject to such a continuing condition, then has the tenancy in this case at all times been
within Part 2 of the Act?
I must now refer in detail to what appear to me to be the relevant sections of the Act for the purpose of deciding the question
of construction. Section 23(1) and (3) are as follows:

(1) Subject to the provisions of this Act, this Part of this Act applies to any tenancy where the property comprised in
the tenancy is or includes premises which are occupied by the tenant and are so occupied for the purposes of a business
carried on by him or for those and other purposes (3) In the following provisions of this Part of this Act the expression
the holding, in relation to a tenancy to which this Part of this Act applies, means the property comprised in the tenancy,
there being excluded any part thereof which is occupied neither by the tenant nor by a person employed by the tenant and
so employed for the purposes of a business by reason of which the tenancy is one to which this Part of this Act applies.
933

Section 24(1) and (3) say:

(1) A tenancy to which this Part of this Act applies shall not come to an end unless terminated in accordance with the
provisions of this Part of this Act; and, subject to the provisions of s. 29 of this Act, the tenant under such a tenancy may
apply to the court for a new tenancy(a) if the landlord has given notice under the next following section to terminate the
tenancy, or (b) if the tenant has made a request for a new tenancy in accordance with s. 26 of this Act (3)
Notwithstanding anything in sub-s. (1) of this section(a) where a tenancy to which this Part of this Act applies ceases to
be such a tenancy, it shall not come to an end by reason only of the cesser, but if it was granted for a term of years certain
and has been continued by sub-s. (1) of this section then (without prejudice to the termination thereof in accordance with
any terms of the tenancy) it may be terminated by not less than three nor more than six months notice in writing given by
the landlord to the tenant; (b) where, at a time when a tenancy is not one to which this Part of this Act applies, the landlord
gives notice to quit, the operation of the notice shall not be affected by reason that the tenancy becomes one to which this
Part of this Act applies after the giving of the notice.

Section 25(1) and (6) say:

(1) The landlord my terminate a tenancy to which this Part of this Act applies by a notice given to the tenant in the
prescribed form specifying the date at which the tenancy is to come to an end (hereinafter referred to as the date of
termination): Provided that this subsection has effect subject to the provisions of Part 4 of this Act as to the interim
continuation of tenancies pending the disposal of applications to the court.
(6) A notice under this section shall not have effect unless it states whether the landlord would oppose an application
to the court under this Part of this Act for the grant of a new tenancy and, if so, also states on which of the grounds
mentioned in s. 30 of this Act he would do so.

Section 29(1) says:

Subject to the provisions of this Act, on an application under s. 24(1) of this Act for a new tenancy the court shall
make an order for the grant of a tenancy comprising such property, at such rent and on such other terms, as are hereinafter
provided.

Section 30(1) sets out a number of grounds on which the landlord can oppose an application for a new tenancy, on one of which,
as I have said, the landlord tried to rely in this case. Section 32(1) deals with the property to be comprised in the new tenancy. It
runs as follows:
Subject to the next following subsection, an order under s. 29 of this Act for the grant of a new tenancy shall be an
order for the grant of a new tenancy of the holding; and in the absence of agreement between the landlord and the tenant as
to the property which constitutes the holding the court shall in the order designate that property by reference to the
circumstances existing at the date of the order.

Section 33 deals with the duration of the new tenancy; s 34 with the rent to be paid under the new tenancy, and s 35 with the other
terms of the new tenancy. I must refer finally to s 64 which provides for the interim continuation of tenancies pending
determination by the court. That section reads as follows:

(1) In any case where(a) a notice to terminate a tenancy has been given under Part 1 or Part 2 of this Act or a
request for a new tenancy has been made under Part 2 thereof, and (b) an application to the court has 934 been made under
the said Part 1 or the said Part 2, as the case may be, and (c) apart from this section the effect of the notice or request would
be to terminate the tenancy before the expiration of the period of three months beginning with the date on which the
application is finally disposed of, the effect of the notice or request shall be to terminate the tenancy at the expiration of the
said period of three months and not at any other time.
(2) The reference in para. (c) of sub-s. (1) of this section to the date on which an application is finally disposed of shall
be construed as a reference to the earliest date by which the proceedings on the application (including any proceedings on
or in consequence of an appeal) have been determined and any time for appealing or further appealing has expired, except
that if the application is withdrawn or any appeal is abandoned the reference shall be construed as a reference to the date of
the withdrawal or abandonment.

Some points at least are reasonably clear. First, having regard to s 64(1), the notice served by the landlords on 23 December
1958, operated as from 14 April 1959, as a notice to terminate the tenancy at the expiry of three months from the date when the
companys application for a new tenancy is finally disposed of. Secondly, the companys application has not yet been finally
disposed of and, therefore, this interim tenancy is still on foot. Thirdly, the Act makes no provision for the service of any fresh
notice in the event of a tenancy ceasing to be one to which Part 2 of the Act applies after the issue of the application for a new
tenancy. The possibility of a tenancy to which Part 2 of the Act applies ceasing to be such a tenancy is indeed contemplated by s
24(3), which I have read, but that subsection appears to me to refer only to the time before an application for a new tenancy is
made. Under s 24(1) a tenancy of business premises for a fixed period continues after the expiry of that period unless the
landlord serves a statutory notice under s 25(1) and (4). If after the expiry of the fixed period the premises ceased to be used for
business purposes, the landlord can no longer serve a notice under s 25 because, ex hypothesi, the tenancy has ceased to be a
tenancy to which Part 2 of the Act of 1954 applies, and unless something was specifically said about it, it would be uncertain
whether the tenancy extended by s 24(1) after the end of the fixed period went on for ever or came to a sudden conclusion. I
think that it was to meet that difficulty that it is provided in s 24(3)(a) that a continued tenancy is not to end summarily but that
the landlord is to have, as it were, a fresh contractual right to determine it. No corresponding right, however, is given to a
landlord, either in the case of a tenancy for a fixed term or in the case of a periodic tenancy, when the tenancy has been
determined by a proper statutory notice but extended by s 64 until the disposal of the tenants application for a new lease and the
extended tenancy later ceases, during the pendency of the proceedings, to be a tenancy protected by the Act of 1954.
The company argues that as s 29(1) is couched in mandatory terms and as the Act makes no provision for the termination of
the tenancy as extended by s 64 on its ceasing to be a tenancy to which Part 2 of the Act applies, it must follow that, if a summons
for a new tenancy has been taken out in due time by a tenant who at the date of its issue was a tenant under a tenancy protected
by the Act and if the landlord fails to establish any of the defences mentioned in s 30, the tenant has an absolute right to have a
new tenancy granted to him. That tenancy can, of course, only comprise the relevant holding, unless the landlord agrees to
more being included or wishes more to be included in it. On the face of it s 32 contemplates that there may be disputes between
the landlord and the tenant as to the extent of the holding, and the company concedes that if, when the court is asked to settle the
terms of the new tenancy, the landlord can establish that there is in fact no property in existence which qualifies as a
935holding within the meaning of the Act, then the court will not in fact be able to make an effective order in the tenants
favour because there would be no subject-matter to be comprised in the new tenancy. But the fact that at some earlier date after
the issue of the summons there was no holding is, the company says, irrelevant provided only that there is a holding when the
judge addresses his mind to the making of an order under s 29.
The point is not an entirely easy one, but for my part I am not prepared to accept that argument. The only sort of tenant who
is given by s 24(1) a right to apply for a new tenancy is a tenant under a tenancy to which Part 2 of the Act of 1954 applies. If the
tenant ceased to occupy the property for business purposes after the service by the landlord of a statutory notice but before he
himself issued his summons asking for a new tenancy, then, as it seems to me, the landlord could apply to have the summons
dismissed on the ground that the tenant had no locus standi to apply to the court. Section 29(1) is telling the court how to deal
with an application under s 24(1), and that, as I have said, must be an application by a tenant under a protected tenancy. Section
29(1) and s 32 appear to me to contemplate that, if and when the terms of the new tenancy comes to be settled, there will
inevitably be some property to be comprised in it. There may be disputes as to the extent of that property, but the wording of s
29(1) and s 32(1), seem to me to be inconsistent with the notion, which is really implicit in the companys argument here, of the
judge solemnly sitting down to settle the terms of the tenancy and then finding at the end of the day that he cannot make any
effective order because there is no holding at all. As I see it, it is a continuing condition of the tenants right to a new tenancy that
he should be throughout the proceedings tenant under a tenancy to which Part 2 of the Act applies. If this condition ceases to be
fulfilled at any time, then I think that the landlord can apply to have the tenants summons dismissed, just as he could if the
tenancy were not protected by Part 2 of the Act when the application was made. I reach that conclusion on a consideration of the
wording of the Act itself, but I am confirmed in my view by the fact that the contrary view would lead to some astonishing
results, which I can hardly think would be consistent with the policy underlying the Act.
Having decided the question of construction in that way, I must now turn to consider whether in fact the company did
occupy these premises, or some part of them, for the purpose of some business carried on by them for the whole of the period
from 9 April to 15 November 1960. The premises are a corner site consisting of a first floor, a ground floor and a basement. Up
to 9 April the company carried on there the business of selling mens clothing, ladies hosiery and mens, womens and childrens
footwear. The basement was used as a store-room and in particular as a store-room for rubber footwear. The busines was carried
on under the name or style of British and Colonial Footwear Company. This name and also the initials B and C appeared on
the fascia above the shop front. The company had two other retail shops, one at Mexborough where mens wear was sold, and
another at Worksop where boys wear was sold. It also had a warehouse and a wholesale department at Sheffield. When Lloyd-
Jacob J gave his decision in favour of the landlord on 15 February the company at once decided to appeal against it and notice of
appeal was served on 29 April. At the same time the company also decided to cease to trade on the premises. They inserted
notices in the press stating that the shop would close on Saturday, 9 April and advertising a closing-down sale at which they
offered goods at greatly reduced prices. There is no doubt that the greater part of the stock which was left after this sale was
removed from the premises in the following week between 9 April and 16 April. Some of it went to Sheffield and some of it
appears to have gone to Mexborough and some of it to Worksop, but a great deal of it, together with a quantity of shop fittings,
was transferred to another shop 936 in Eldon Street, Barnsley, quite close to the Peel Square shop and that other shop was opened
for trade on Saturday, 16 April. This other shop did not belong to the company but to another company, HCC Clothiers
(Sheffield) Ltd (hereinafter called HCC Clothiers), in which Maurice Caplan has no interest. The business which this other
company carried on at Eldon Street as from 16 April 1960, was the same kind of business as that carried on formerly at the Peel
Square premises, and the fascia over the shop bore the words B & C Manshop. B & C Shoeshop. One of the advertisements
announcing the closing down was in the following terms:

Closing down sale. Prices shattered. All stock must go. New B. and C. Manshop and B. and C. Shoe Shop opening
soon at Eldon Street previously occupied by the Three Cranes.

Notices were pasted on to the windows of the premises in Peel Square, one of which said: All inquiries to new B. & C.
Manshop, Shoeshop. Three Cranes Buildings, Eldon St Barnsley, and another of which said: Closed. All inquiries to: B. &
C. Mans Shop or B & C Shoe Shop. 3 Cranes Building, Eldon St.. The stock transferred from Peel Square to Eldon Street was
sold by the company to H. C. C. Clothiers at cost price so that any profit from its retailing at Eldon Street went to them and not to
the company, and a good many of the staff formerly employed by the company at Peel Square entered the service of H C C
Cothiers at Eldon Street.
These facts give food for thought but I am not in these proceedings concerned to deal in any way with the question whether
Maurice Caplan, as a minority share-holder in the company can complain of those acts of the company, or whether, as the
directors of the company say, by taking steps to eject the company from the Peel Square premises Maurice has, as it were,
estopped himself from complaining of their retaliatory action. For all I know that question may form the subject of yet more
litigation between these parties. All that I am concerned with is whether the company continued throughout the period from April
to November to occupy the Peel Square premises or some part of them for the purposes of a business carried on by it.
There is no doubt that from soon after 9 April 1960, until 15 November 1960, the ground and first floors of the premises at
Peel Square were empty, although the services, namely, the water, electricity and gas were still connected and not cut off at
source. There is, however, a conflict of evidence as to whether the basement also was empty, to which I shall have to return later.
In July, 1960, as I have said, the landlords applied by motion to the Court of Appeal to have the appeal stayed because they
discovered that trading on the premises had ceased. Mr Cyril Caplan, who is a director of the company, made an affidavit on 20
July in answer to the evidence on that motion, to paras 2, 4, 5, 6 and 7 of which I should refer. He said:

2. It is true that the premises the subject-matter of this appeal are at present not in use as a saleshop. After a clearance
sale of stock business temporarily ceased therein on May 28, 1960 [that was a slip, and, I am sure, quite an unintentional
slip, for Apr. 9, 1960] but this was in pursuance of a reorganisation of [the companys] business which was taking place
independently of the hearing of this matter, and there is no intention on [the companys] part of abandoning their business
use of the premises if their appeal to the Court of Appeal is successful. 4. The applicant company is still in occupation of
the said premises. There still remains on the premises the electric light fittings and window lighting, the window backs,
floor fittings and garment rails. Other fittings have been removed to facilitate the change of business on the said premises.
5. The new British and Colonial 937 Manshop and Footwear Shop referred to in the affidavit of Peter Stuart Newton is a
business carried on in Eldon Street by H.C.C. Clothiers (Sheffield), Ltd. The latter company is a separate and distinct
entity from the applicant company although some of the directors and shareholders of the applicant company are directors
and shareholders of H.C.C. Clothiers (Sheffield), Ltd. 6. It is the intention of the applicant company to carry on in the near
future the business of a Ladies Gown Shop in the premises to which the appeal relates having disposed of the stocks
previously held on the premises, the subject-matter of this appeal. 7. In the event of this appeal being successful it is [the
companys] intention to spend a considerable sum of money fitting the premises out as a Ladies Gown Shop.

As I have said, in view of the decision of the Court of Appeal on 15 November the company, as soon as they possibly could,
resumed trading at Peel Square; and they have traded there ever since. The business which they resumed and which they have
been carrying on since November, 1960, comprises the same goods as they sold before 9 April with the addition of some ladies
wear.
There is a dispute of fact on two points. First, whether a quantity of rubber footwear was left in the basement of Peel Square
between April and November; and, secondly, as to what precisely the intentions of the company were between April and
November with regard to the future use of the Peel Square premises if they eventually got a new tenancy. [His Lordship
reviewed the evidence and concluded that a quantity of rubber footwear was left in the basement of the premises and that it was
the companys intention between 9 April and 15 November 1960, to discontinue the old line of business at the premises
altogether and, if and when they were granted a new tenancy, to use the premises simply and solely as a ladies gown shop; and to
market the footwear eventually at the Eldon Street shop. He continued:] Those being the facts as I find them, what is the legal
conclusion? I think it is quite clear that a tenant does not lose the protection of Part 2 of the Landlord and Tenant Act, 1954,
simply by ceasing physically to occupy the premises. They may well continue to be occupied for the purposes of the tenants
business although they are de facto empty for some period of time. One example would be if there was a need for urgent
structural repairs and the tenant had to go out of physical occupation in order to enable them to be effected. Another example is
that which the Court of Appeal had to deal with in Teasdale v Walker. There premises were only occupied during discontinuous
periods: they were closed and empty in the winter and only used in the summer. On the other hand, as the Court of Appeal
pointed out, a mere intention to resume occupation if a new tenancy is granted will not preserve the continuity of business user if
the thread has once been definitely broken. It is to be observed, however, that in Teasdale v Walker the discontinuance of
business occupation was before the tenant applied to the court for a new tenancy. She had let someone else occupy the premises
for business purposes. He had gone out of occupation and given her back the key and the premises were empty when the
summons was issued. In those circumstances the Court of Appeal considered that her expressed intention to start again herself if
and when she got a new tenancy was not enough.
The position may, I think, very well be different if the tenant is using the premises for business purposes at the date of the
issue of the summons but goes out of physical occupation during the proceedings. Suppose that a tenant in occupation for
business purposes applies for a new tenancy. The landlords oppose the application and succeed in the court of first instance. The
tenant then lodges a notice of appeal but his solicitors tell him that his chances are not very rosy, that it is likely that the landlords
will be upheld in their objection and that 938 he will eventually have to give up possession. It seems to me to be not
unreasonable for a tenant in those circumstances to say: Well, I must secure a way of retreat. I must preserve the goodwill of
the business and I must give continuity of employment to my staff. I cannot risk staying on here and being suddenly turned out.
I am going to take some other premises to which I will transferpossibly for ever, but possibly only temporarilymy staff and
my stock. Those premises are not as suitable as those from which I am being turned out. If I succeed, as I hope, in this appeal, I
will go back. Of course, if I fail, I cannot go back and I must go on in the new premises. On those facts I would be very loth to
hold that the cesser of physical occupation deprived the tenant of his rights under Part 2 of the Act of 1954 and, a fortiori, if he
left a quantity of stock on the premises. Of course the example which I have given is more favourable to the tenants than the
facts here because, as I have said, the trade which they hoped to set up if they succeeded in the appeal was not precisely the same
as the trade which they had discontinued although it was allied to it. Again, it may be said that the stock which they left behind,
though it was physically left on the companys premises, was really dedicated to the purposes of H C C Clothiers and would not
have been marketed on the companys premises if matters had turned out as the company thought that they would. I appreciate
the force of those arguments; the facts here are most unusual and I think that this is a border-line case. But on the whole I have
come to the conclusion that the thread of continuity, to use the words of the Court of Appeal in the case of Teasdale v Walker
([1958] 3 All ER 307 at p 315), was not broken in this case.
I propose, therefore, to declare that notwithstanding that they ceased to trade on the premises between 9 April and 15
November 1960, the company did not cease to occupy the premises for the purposes of a business within the meaning of the Act.

Declaration accordingly.

Solicitors: Vizard, Oldham, Crowder & Cash agents for Pye-Smith & Son, Sheffield (for the applicants, the landlords); Kanter,
Jules & Co agents for Keeble Hawson, Steele Carr & Co, Sheffield (for the respondents, the company).

Jacqueline Metcalfe Barrister.


939
[1963] 2 All ER 940

Charles Forte Investments Ltd v Amanda


COMPANY; Shares

COURT OF APPEAL
WILLMER AND DANCKWERTS LJJ AND CROSS J
12, 13 JUNE 1963

Company Shares Transfer Restriction imposed by articles of association Refusal by directors of private company, which
was a holding company, to register transfer of shares by member Remedy Winding-up petition threatened by the aggrieved
member Alternative remedy, eg, by action, available Whether presentation of proposed petition should be restrained by
injunction under the inherent jurisdiction to stay proceedings that were an abuse of the process of the court Companies Act,
1948 (11 & 12 Geo 6 c 38), Sch 1, Table A Pt 2, art 3.

The articles of association of a private company, which was a holding company, had an issued share capital of 750,000 1
ordinary shares and had practical control of a large public company that had independent shareholders, empowered the directors
in their absolute discretion and without assigning any reason therefor to decline to register any transfer of a share. A, who was
a minority shareholder in the private company, holding ten thousand 1 ordinary shares, had been an employee of the public
company and had acquired his shareholding as a result of that employment. A had, however, fallen out with F, who in effect
controlled both companies, and A had consequently resigned the employment. A submitted to the directors of the private
company transfers on sale of four thousand 1 shares in the private company in favour of merchant bankers. The directors
declined to register the transfers, and gave no reason. A having informed the private company that, unless an undertaking was
given to register the transfers (which the directors did not give), A would present a winding-up petition by way of remedy, the
private company applied for an injunction to restrain A from presenting the petition. The proposed petition alleged that the
directors of the private company other than F, were accustomed to act in accordance with his wishes; that there was no ground for
properly refusing to register the transfers, which were transfers on sale, and that refusal of registration was prompted by desire of
F to victimise A; that the refusal to register the transfer was an abuse of the powers of the directors and was oppressive to A; that
the private company was in substance a partnership between its members and that it was not reasonably possible for A to remain
associated with F and the other members and that it was just and equitable that the private company should be would up by the
court.

Held The presentation of the winding-up petition would be restrained by injunction, granted under the inherent jurisdiction of
the court to stay proceedings which were vexatious or an abuse of the court, for the following reasons
(i) a winding-up petition was not the proper remedy in the circumstances of this case, particularly as the shareholder could,
by bringing an action, obtain his real remedy, if he established his case, without risk of inflicting possibly irreparable damage on
other innocent shareholders; moreover, as the shareholder was very much a minority shareholder, the present case was not that of
a deadlock in a quasi-partnership, as, eg, where two principal shareholders in a private company fell out and business could no
longer be carried on (see p 948, letter d and e, p 949, letters f and g, and p 951, letters a and e, post).
Re Cuthbert Cooper & Sons Ltd ([1937] 2 All ER 466) approved and applied.
Re Yenidje Tobacco Co Ltd ([191617] All ER Rep 1050) distinguished.
(ii) (per Willmer LJ) an object of the jurisdiction was to restrain the 940 assertion of doubtful rights in a manner productive
of irreparable damage; in the present case the presentation of the proposed winding-up petition with consequent necessity for
advertisement might cause irreparable damage, while there was no chance of As making good his case, since the power of the
directors to refuse registration enabled them without impropriety to refuse registration of a transfer so as to prevent shares getting
into the hands of strangers, eg, merchant bankers (see p 945, letters d and e, and p 947, letter g, post; cf, p 950, letter g, post).
Dictum of Malins V-C, in Cadiz Waterworks Co v Barnett ((1874), LR 19 Eq at p 196) applied.
Per Willmer and Danckwerts LJJ: the court would assume that directors exercising a power to refuse registration of a
transfer of shares acted in good faith, the onus of proving the contrary being on those who so alleged (see p 950, letter c, and p
946, letter d, post).
Appeal allowed.

Notes
As to exercise of a power to decline to register share transfers, see 6 Halsburys Laws (3rd Edn) 253, para 527; and for cases on
the subject, see 9 Digest (Repl) 388393, 25062526.
For Table A, Pt 2, art 3, of Sch 1 to the Companies Act, 1948, see 3 Halsburys Statutes (2nd Edn) 823.
As to the just and equitable ground for a winding-up order, see 6 Halsburys Laws (3rd Edn) 534, para 1035; for cases on
the subject, see 10 Digest (Repl) 856, 857, 56385645; and for the Companies Act, 1948, s 225(2), see 3 Halsburys Statutes (2nd
Edn). As to who may not petition, see 6 Halsburys Laws (3rd Edn) 537, para 1038; and for cases on the subject, see 10 Digest
(Repl) 870872, 57215752.

Cases referred to in judgments


Cadiz Waterworks Co v Barnett (1874), LR 19 Eq 182, 44 LJCh 529, 31 LT 640, 10 Digest (Repl) 871, 5743.
Coalport China Co, Re [1895] 2 Ch 404, 64 LJCh 710, 73 LT 46, 9 Digest (Repl) 394, 2529.
Company, A, Re [1894] 2 Ch 349, 63 LJCh 565, 71 LT 15, 10 Digest (Repl) 866, 5697.
Cooper (Cuthbert) & Sons Ltd Re [1937] 2 All ER 466, [1937] Ch 392, 106 LJCh 249, 157 LT 545, 10 Digest (Repl) 857, 5645.
Davis and Collett Ltd, Re [1935] All ER Rep 315, [1935] Ch 693, 104 LJCh 340, 153 LT 329, 10 Digest (Repl) 857, 5644.
Dublin North City Milling Co, Re [1909] 1 IR 179, 9 Digest (Repl) 390, * 1061.
Goodson v Grierson [1908] 1 KB 761, 77 LJKB 507, 98 LT 740, 25 Digest (Repl) 426, 83.
Hannans King (Browning) Gold Mining Co Ltd Re (1898), 14 TLR 314, 9 Digest (Repl) 224, 1436.
Merchant Banking Co of London v Hough (1874), WN 230, 10 Digest (Repl) 872, 5744.
Niger Merchants Co v Capper (1877), 18 ChD 557, 10 Digest (Repl) 872, 5745.
Smith and Fawcett Ltd, Re [1942] 1 All ER 542, [1942] Ch 304, 111 LJCh 265, 166 LT 279, 9 Digest (Repl) 389, 2509.
Sutherland (Duke) v British Dominions Land Settlement Corpn Ltd [1926] Ch 746, 59 LJCh 542, 135 LT 732, 18 Digest (Repl)
207, 1801.
Yenidje Tobacco Co Ltd, Re [191617] All ER Rep 1050, [1916] 2 Ch 426, 86 LJCh 1, 115 LT 530, 10 Digest (Repl) 863, 5683.

Interlocutory Appeal
The plaintiff company, Charles Forte Investments Ltd appealed from the decision of Pennycuick J on 15 May 1963, refusing its
interlocutory motion 941 against the defendant shareholder, Amando Bertuzzi Amanda, for an injunction restraining him until
judgment in the action or further order from presenting a petition for winding-up the plaintiff company on the grounds that the
refusal of the plaintiff companys directors to register two transfers of its shares by the defendant constituted an abuse of the
directors fiduciary powers justifying the defendant presenting the petition. The facts are stated in the judgment of Willmer LJ
the grounds stated in the proposed petition being summarised in the headnote at p 940, letter f, ante.

Sir Andrew Clark QC, M M Wheller QC and R A K Wright for the plaintiff company.
R B S Instone for the defendant shareholder.

13 June 1963. The following judgments were delivered.

WILLMER LJ. This is an appeal from a judgment of Pennycuick J given on 15 May 1963, whereby he dismissed a motion
brought by the plaintiff company asking for an interim injunction pending trial restraining the defendant from presenting a
petition to wind up the company. The plaintiff company, which is admittedly a solvent company, is a private company
incorporated on 21 May 1962, whose main object, stated in para 3 (A) of the memorandum of association, is To acquire for
payment in shares the whole of the issued capital of Fortes (Holdings), Ltd. The capital of the plaintiff company is 750,000,
all issued and paid up, divided into 750,000 shares of 1 each. The plaintiff company holds one and a half million ordinary
shares and four million B ordinary shares in Fortes (Holdings) Ltd. We are informed that that holding is sufficient to give the
plaintiff company for practical purposes a controlling interest in Fortes (Holdings) Ltd. The latter company is a public company,
whose shares are dealt with on the Stock Exchange, and which has other shareholders apart from the plaintiff company. Its
directors, however, are the same persons as are directors of the plaintiff company.
The defendant is the holder of ten thousand shares in the plaintiff company, and he acquired those shares in the following
circumstances. In or about the year 1958 he accepted an appointment through Mr Charles Forte with Fortes (Holdings) Ltd and
in consideration of his taking up that appointment he was given ten thousand shares in Fortes (Holdings) Ltd. Unhappily, after a
year or so had passed, the defendant fell out with Mr Charles Forte and ultimately, in or about December, 1960, he resigned his
appointment. There was then a dispute between the defendant and Mr Charles Forte as to the ownership of the shares which were
held by the defendant. Mr Charles Forte claimed that on the termination of the defendants appointment they should be returned
to him. The defendant did not agree with that claim. In the result an action was brought in the Queens Bench Division by Mr
Charles Forte, in which it was alleged that the defendant was under an obligation to re-transfer his shares for a nominal
consideration on the termination of his employment. The action in the event came to nothing, for Mr Charles Forte, when it was
due to come on for hearing, decided not to proceed with it, and consequently the action was dismissed in November, 1962.
In the meantime, however, whilst all this had been going on, the plaintiff company was formed, and in the ordinary course
the defendant, as a shareholder in Fortes (Holdings) Ltd received ten thousand shares in the plaintiff company in exchange for
his previous holding in Fortes (Holdings), Ltd. Being no longer employed by the company, however, and being at loggerheads
with Mr Charles Forte, the defendant was minded to dispose of his shares. On 4 March 1963, he submitted to the board of the
plaintiff company through his stockbrokers three transfers for registration. The first was a transfer dated 1 March 1963, in respect
of one hundred and fifty shares, which was in favour of a Mr Richard Evelyn Fleming and two other named gentlemen, all of
whom were of the address 8, Crosby Square, EC3. The second was a transfer, also dated 1 March 1963, of 3,850 shares. That
was in favour of a body called R F Nominees Ltd of the same address, 8, Crosby Square. As has been pointed out to us in
argument, 942those transferees were readily identifiable, both by their name and by their address, as being nominees of Robert
Fleming & Co Ltd well-known merchant bankers. Both those transfers were transfers for valuable consideration and were duly
stamped with the appropriate ad valorem stamp. The third transfer was dated 4 March 1963, and was in favour of Branch
Nominees Ltd and covered the remaining six thousand shares held by the defendant. That was stamped only with a 10s stamp,
and was indorsed with a certificate to the effect that no beneficial interest passed.
Those transfers were considered by the board of the plaintiff company, who, on or about 14 March 1963, passed a resolution
declining to register the transfers without assigning any reason therefor. At this point it should be stated that the articles of
association of the plaintiff company incorporated art 3 of Pt 2 of Table A, which is in the following terms:

The directors may, in their absolute discretion and without assigning any reason therefor decline to register any
transfer of any share whether or not it is a fully paid share.

Following the boards refusal to register the transfers correspondence ensued between solicitors for the respective parties, and
that correspondence culminated in a letter of 1 May 1963, written by the defendants solicitor to the solicitors to the company. I
read the second and third paragraphs of that letter. It says:

Our client is advised by counsel that in the circumstances of this case such refusal cannot have been based on any
interest of the company, and constitutes an abuse of the directors fiduciary powers which justifies him in presenting a
petition for its winding-up by the court. We are reluctant to take this step without giving your clients an opportunity to
reconsider their attitude. The requisite petition and evidence in support have already been prepared, but we will refrain
from presenting the petition if, within fourteen days from the date of this letter, we receive an undertaking on behalf of the
directors of the company that if the two transfers for the sale of a total of four thousand of our clients shares in favour (a)
of R. F. Nominees, Ltd. and (b) of Mr. R. E. Fleming and others are resubmitted to the company, they will be forthwith
registered. Our client is not at present disposed to press for the registration of the third transfer, which was in favour of a
bank nominee company for holding on his own account.

The plaintiff company countered that letter by issuing the present writ. By the indorsement of the writ the plaintiff company
claims an injunction to restrain the defendant from presenting a petition fo winding-up the company
on the grounds that the refusal of the directors of the plaintiff company to register three transfers of shares in the
plaintiff company[then details of the three transfers are given]constituted an abuse of the directors fiduciary powers
justifying the defendant in presenting a petition for winding-up the plaintiff company.

The company obtained an interim injunction ex parte to cover the period until 14 May when the present motion came on for
hearing. By their notice of motion the plaintiff company now asks for an injunction pending trial in the terms of the indorsement
of the writ.
The plaintiff companys case can, I think, be summarised under three heads. First, it is said that the petition is not sought to
be presented for any proper purpose, but merely as a means of bringing pressure to bear on the directors to reverse the decision at
which the board arrived. It is claimed that such a petition would be an abuse of the process of the court. Secondly, it is said that
there is no evidence of any facts sufficient to substantiate the allegation made against the directors, which is virtually an
allegation of lack of bona fides. On that ground it has been contended that the petition is a petition which is bound to fail, and is
therefore an abuse of the process of the court which should be halted 943 in limine. Perhaps that is not quite the correct
expression, because I suppose what is really said is that it should never reach the threshold at all. Thirdly, it is said that even if
there were evidence to support the allegations put forward, a winding-up petition is not the proper remedy. The proper remedy, if
the defendant has any legitimate grievance, would be an action for rectification of the register or a proceeding by way of motion
under s 116 of the Companies Act, 1948. It is pointed out that if the defendant wants to have the transfer registered, a winding-up
petition will not give him the relief which he seeks. On the other hand, if he brought an action, and assuming he was able to
prove his facts, he could possibly get what he seeks, or alternatively he could obtain damages for the refusal to register the
transfers.
The plaintiff company here invokes the inherent jurisdiction of the court to stay proceedings which are vexatious or an abuse
of the process of the court. It does not rely on any of the Rules of the Supreme Court, but solely on the inherent jurisdiction. The
learned judge held that this jurisdiction should not be exercised unless it were made perfectly clear that the plea could not
succeed. He then proceeded to consider whether in his view it could be said that the plea could not possibly succeed. He
considered, first, the contention that the petition would be bound to fail on its facts, and secondly, the contention that a winding-
up petition was the wrong remedy. As to the latter, he came to the conclusion that in certain circumstances a winding-up petition
could be a legitimate remedy in a case such as the present, or that it was at least arguable that it might be an appropriate remedy
in this case. As to the other point, he in the end felt unable to say that on the facts this petition must necessarily be bound to fail.
Consequently, he came to the conclusion that this was not a case in which he would be justified in invoking the drastic remedy of
halting the proceedings in limine. From that decision the plaintiff company appeals to this court.
I need hardly say that I differ with reluctance from presenting a petition for experienced judge as Pennycuick J though I am
encouraged by the fact that both my brothers in this court take the same view as I do. I am bound to say, however, that I have
found the argument in support of this appeal wholly convincing, and I am satisfied that the learned judge came to a wrong
conclusion.
As to the circumstances in which the inherent jurisdiction of the court may be invoked, I entirely accept the learned judges
caution, which he quoted from the Annual Practice a, that this is a jurisdiction to be exercised with great circumspection. Counsel
for the defendant this morning has properly drawn our attention to an authority, Goodson v Grierson which expresses the test to
be applied as being whether it is impossible for the party concerned to succeed on his claim; but there is something to be said for
the view put forward by counsel on the other side that this is an argument which loses at any rate some of its force in a case
where it can be shown that the defendant has an alternative remedy open to him. We were, however, referred to several cases
where the jurisdiction has been invoked in order to restrain the presentation or prosecution of a winding-up petition. One case
very much in point was Re A Company, in which the petition had actually been presented, but an injunction was sought and
obtained to restrain the advertisement of it. It was found in that case that the petition had been presented for the purpose of
putting pressure on the company, and it was held that that was sufficient to justify an injunction restraining its advertisement.
Vaughan Williams J said ([1894] 2 Ch at p 351):
________________________________________
a Annual Practice, 1963 Edn, p 577, note Inherent Jurisdiciton to RSC, Ord 25, r 44

In my judgment, if I am satisfied that a petition is not pressented in good faith and for the legitimate purpose of
obtaining a winding-up order, but for other purposes, such as putting pressure on the company, I ought to stop it if its
continuance is likely to cause damage to the company.
944

It seems to me that the letter of 1 May 1963, the material part of which I have already read, furnishes the clearest possible
evidence of a desire, and indeed an attempt, in this case on the part of the defendant to put pressure on the company.
Some assistance, I think, is also to be derived from two old cases which were referred to by Sir George Jessel MR in Niger
Merchants Co v Copper, which is reported in a note. The material part of that report was read by the learned judge in the course
of giving his judgment in the present case. I can conveniently re-quote the passage which the learned judge quoted from Sir
George Jessels judgment:

The authorities stand thus: I find there is a case on this point, Merchant Banking Company of London v. Hough, not
fully reported, in which HALL, V.-C., in December, 1874, upheld an injunction previously granted to restrain the
defendants, who had a claim against the plaintiff company, from taking proceedings to wind up the company. MALINS,
V.-C., a few days later, granted a similar injunction in the case of Cadiz Waterwords Co. v. Barnett ((1874), LR 19 Eq 182,
see at p 196) on the ground that it is the object of the court to restrain the assertion of doubtful rights in a mannor
productive of irreparable damage.

I accept, of course, as has been pointed out, that these causes, being cases of creditors petitions, are not the same as the
present case; but those words of Malins V-C ((1874), LR 19 Eq. 182, see at p 196) as it seems to me, are words of general
application and seem to me to be apt to the present case. For in the present case, as I think, there is good reason to think that the
presentation of a winding-up petition, with the consequent necessity for advertisement, might cause irreparable damage. There
can, I think, be no doubt that the allegations in the proposed petition amount to a very grave attack on the directors of the plaintiff
company, who are the same people as the directors of Fortes (Holdings), Ltd. The publicising of this attack could, therefore,
have a damaging effect on Fortes (Holdings) Ltd with the possibility of damage (possibly irreparable) to quite innocent
shareholders in that company. In those circumstances, I do not doubt that this would be a proper case for the exercise of the
inherent jurisdiction of the court, if the plaintiff company can otherwise make good its claim that this is a petition which is bound
to fail and amounts in the circumstances to an abuse of the process of the court.
I pass, therefore, to consider that aspect of the case. It seems to me that that involves a consideration both of the relevant
law and of the facts alleged in this particular case. We have had cited to us a series of authorities, which seem to me to establish
that where directors act under an article in form similar to that which we have here, namely, art 3 of Pt 2 of Table A, their refusal
to register a transfer cannot be questioned in the absence of actual evidence that they have not acted bona fide. It is to be
observed that in most of the authorities cited the article under which the directors were acting was in somewhat narrower terms
than that in question here. Here, as it seems to me, it is in the widest possible form. It speaks of an absolute discretion, which
may be exercised without assigning any reason. In those circumstances, it can be said that what was laid down in the cases
which were cited to us would apply a fortiori to the circumstances of the present case.
I take the cases in historical order as they were cited to us. I do not propose to refer to the individual facts of the individual
cases to which we were referred, for I cite them merely for such statements of principle as I can find in them. The first case was
Re Coalport China Co. In that case I will refer to two brief passages in the judgment of Lindley LJ. He said ([1895 2] Ch at p
407)

It is for those who say that the directors have exercised their power improperly to give some evidence to that effect.
945

He concluded his judgment as follows ([1895] 2 Ch at p 409):

I wish to be cautious in this matter, because I have not the slightest doubt that the court has ample power to control the
refusal of directors, or the exercise by them of their power to refuse, provided there is some evidence which justifies the
court in coming to the conclusion that they have not done their duty; but in the absence of all such evidence the court has
no right to presumeit is contrary to the ordinary principles of justice to do sothat they have done wrong, but it must be
presumed that they have done right.

Much to the same effect was the statement, also by Lindley LJ in Re Hannans King (Browning) Gold Mining Co Ltd ([1898), 14
TLR 314 at p 315). There is only a brief report of what he said in that case, and it is, of course (as is common in the old Times
Law Reports), in oratio obliqua and not an actual quotation. He concluded his judgment in that case by saying:

The court ought, as a matter of honesty between man and man, to presume that the directors were acting within their
powers unless the contrary was proved; but that was not proved by casting unfounded aspersions upon them.

We were also referred to an Irish case, Re Dublin North City Milling Co. I do not think that I need refer to that in detail.
The same principle was followed in that case, and it was said that the transferee complaining of the directors refusal to register
the transfer must allege and prove some indirect motive on their part.
In Duke of Sutherland v British Dominions Land Settlement Corpn Ltd ([1926] Ch 746 at p 756), there was a valuable
dictum by Tomlin J which is worth reading in full:

Prima facie the directors are assumed to act bona fide just as ordinary trustees in exercising powers are assumed to act
bona fide. If anybody alleges the contrary the onus is on him to prove it, and if in fact he adduces no ovidence at the trial
which justifies a conclusion either that there has been no exercise of the discretion or that there has been a mala fide
exercise of the discretion, then the mere fact that the directors have refused to given any reason for the exercise of the
power, and for the manner in which they have exercised it, throws no suspicion on them or in any sense shifts the onus of
proof so as to put upon them the burden of justifying that which they have done.

Lastly, in Re Smith and Fawcett Ltd ([1942] 1 All ER 542 at p 545; [1942] Ch 304 at p 308), the opportunity was taken by Lord
Greene MR to emphasise that an article in form similar to that which we have here is such as to give the directors what it says,
namely, an absolute and uncontrolled discretion.
Those being the principles which have been established by the decided cases, I come now to apply them to the facts which
have been alleged here. I hope I am not unfairly summarising what is sought to be put forward in the proposed petition. I seems
to me the allegations really amount to three. First, there is an allegation that the directors, other than Mr Charles Forte, are
persons who are accustomed to acting in accordance with his wishes. That is to say, they are more or less yes, men;
consequently, their conduct in refusing to register the transfers in question here was not a bona fide exercise of their discretion
but really amounted to a dereliction of duty on their part. Secondly, it is said that the refusal to register the transfers was arrived
at without any inquiry as to the identity of the persons who would be beneficially interested if the transfers were registered.
Thirdly, it is said that there was not, and could not be, any 946 ground on which the directors could properly or reasonably refuse
to register the transfers.
As to the first point, I think that it is fair to say that counsel for the defendant has not seriously argued in support of it. At
any rate, I can say that I have searched in vain through the draft affidavit which has been produced, and which was intended to be
the affidavit supporting the petition, for any evidence to support any such grave charge as has been levelled against the directors
other than Mr Forte.
With regard to the absence of inquiry, there certainly was no evidence to support the allegation made. That perhaps is not
surprising, for it is not a matter of which the defendant could have any knowledge. I have wondered whether it would have been
relevant in any case to make any such inquiry. I seems to me that the directors of a private company, such as that in question
here, would be perfectly entitled in their absolute discretion to refuse the registration of a transfer merely on the ground that the
proposed transferee was a stranger. I can well imagine circumstances in which the board of a private company might properly
and reasonably come to the conclusion that they did not want stranger shareholders in the company. Be that as it may, it does not
seem to me that in the circumstances of this case absence of inquiry, if indeed there was absence of inquiry, is of any significance,
because there was not room for very much doubt as to the identity of the proposed transferees. It was known that the proposed
transferers were one manifestation of the Robert Fleming group, whose activities must have been well known to the directors of
the plaintiff company. I do not think, therefore, that there could be any substance in the allegation that the registration of the
transfers was refused without inquiry.
The third allegation, that there was not and could not be any ground on which the directors could properly refuse, seems to
me to be one that really will not bear examination. I have no desire to speculate about the matter, but it seems to me that there
may have been any number of excellent reasons why the directors of this private company should above all things wish to avoid
shares in the company coming into the hands of stangers. More particularly that may be so if the stangers are a merchant banking
company, or persons connected with a merchant banking company; this is a view which is seems to me they might well quite
properly form. Without in the least imputing any adverse criticism against the proposed transferees or their business integrity, the
mere fact that they were stangers, or the mere fact that they were merchant bankers, might well, in the view of the board, be a
sufficient reason for refusing to register a transfer.
In those circumstances, bearing in mind the very heavy onus which lies on those who allege that the directors refusal to
register a transfer is not bona fide, I can see no possible chance, on the evidence which is at present before us, namely, that
contained in the draft affidavit, of the defendant making good his allegations.
But even if I am wrong about that, there is another ground on which it can be said that this petition is bound to fail; for it
seems to me that this is a case in which a winding-up petition is not the proper remedy. I say that on the assumption that I am
wrong in everything else which I have said so far, and on the assumption that there is sufficient evidence before the court to
justify some at least of the allegations made by the defendant. Even assuming that, I venture to think that a winding-up petition is
not a proper remedy.
The authority on that point is Re Cuthbert Cooper & Sons Ltd. The effect of that decision, which was a decision of Simonds
J has been fully set out in the learned judges judgment in the present case, and the learned judge has also quoted extensively
from the judgment of Simonds J. I do not wish to waste time by merely re-quoting what the learned judge below has already
quotedb. 947I would refer only to one passage. In the course of his judgment Simonds J said ([1937] 2 All ER at p 468; [1937]
Ch at p 399):
________________________________________
b Pennycuick J quoted passages reported in [1937] 2 All ER at pp 468, 469; [1937] Ch at pp 399, 401.

I will not assume that the directors discretion has been improperly exercised. Nor will I assume that the petitioners
could not get the appropriate remedy in the proper proceedings, which would be by motion to rectify, or by action in the
ordinary way in the Chancery Division. But on a petition, I will not exercise my jurisdiction to order the winding-up on
that ground, on the footing that it is for that reason just and equitable.

Certainly in the circumstances of that case Simonds J expressed little sympathy for a proceeding such as that which has been
attempted by the defendant in the present case; but it has been arguedand the learned judge accepted the argumentthat the
decision of Simonds J is distinguishable from the present case, on the ground that there the winding-up petition was launched by
a person who was seeking to be registered as a member of the company, whereas here the petition is by a member who is seeking
to get out of the company and to transfer his shares elsewhere. There is no doubt, of course, that that distinction between the two
cases does exist; but on consideration I cannot think that that distinction makes any difference in principle. In my judgment, the
decision of Simonds J in the Cuthbert Cooper case was a sound decision, one that should be followed, and one which, as I think,
governs this case to the extent of saying that here, as there, the winding-up petition is misconceived even if the allegations
contained in it could be substantiated, having regard to the existence of an alternative and more suitable remedy. It is to be
observed that by bringing an action the defendant, if he has a good case, could not only more easily obtain what he is really after,
but he could do so without incurring the risk of inflicting damage, possibly irreparable damage, on the company, and possibly
also on other innocent shareholders. In those circumstances, it seems to me that that is an additional reason for holding that the
injunction which is sought ought to be granted.
Reference has, of course, been made to s 225(2) of the Companies Act, 1948. That provides:

Where the petition is presented by members of the company as contributories on the ground that it is just and equitable
that the company should be would up, the court, if it is of opinion,(a) that the petitioners are entitled to relief either by
winding-up the company or by some other means; and (b) that in the absence of any other remedy it would be just and
equitable that the company should be wound up; shall make a winding-up order, unless it is also of the opinion both that
some other remedy is available to the petitioners and that they are acting unreasonably in seeking to have the company
would up instead of pursuing that other remedy.

I merely refer to that for the purpose of saying that, in my judgment, it does not apply in the circumstances of the present case;
for it must follow from that I have already said that in the present case not only would there be an alternative remedy open to the
defendant, but also that if the defendant has a good case he is acting unreasonably in seeking to pursue his remedy by way of a
winding-up petition instead of the alternative remedy.
I must, however, deal with the argument presented to us by counsel on behalf of the defendant, who has sought to contend
that, following the principle applied in Re Yenidje Tobacco Co Ltd, the present case is one in which a winding-up order could be
justified on the same grounds as would justify a dissolution of partnership. It has been contended that the plaintiff company in
the present case does really amount in effect to a quasi partnership, being, as counsel for the defendant expressed it, merely a
syndicate for holding shares in Fortes 948(Holdings), Ltd. I can only say that to my mind the case of Re Yenidje Tobacco Co
Ltd, and equally the case cited to us this morning, Re Davis and Collett Ltd, where the same principle was applied, are utterly
remote from the question that has to be dealt with in this case. Re Yenidje Tobacco Co Ltd, for instance, was a case of a two-man
company, and it so happened that the two men fell out with each other and could not get on, with the result that deadlock ensued
and the business of the company could not be carried on. It was in that context that it was held that a winding-up order ought to
be made on the same basis as an order might be made for a dissolution of partnership; but here we are dealing with a shareholder
who can be described as very much a minority shareholder. His whole holding amounts to only one seventy-fifth of the capital of
the plaintiff company, and the four thousand shares at stake in the two transfers now in question amount to an even smaller
fraction of the capital. Nor is he a shareholder who has any part in the running of the plaintiff company. He is merely a small
shareholder. There is nothing, as I see it, in his relationship to the plaintiff company which remotely resembles that of a
partnership such as there was in Re Yenidje Tobacco Co Ltd. I do not think that there is any room for the application of that
principle in the circumstances of the present case.
In those circumstances, I have come to the conclusion that the plaintiff company is entitled to succeed. I should just mention
a possible question which, it may be, would go only to costs, namely, whether the plaintiff company has chosen the right remedy
by bringing an action to restrain the presentation of the petition, instead of waiting till the petition was presented and then
applying to strike it out, which I have no doubt it could have done. The answer which has been given to that is a convincing
answer, namely, that the plaintiff company was entitled to take the course which it has taken so as to avoid what is, after all, the
greatest evil of which it was afraid, namely, the necessity for advertisement. In those circumstances, I do not think that it can be
said that the plaintiff company has chosen a wrong remedy in bringing this action asking for an injunction.
For my part, therefore, I would allow this appeal and would make an order in the terms of the notice of motion.

DANCKWERTS LJ. I agree that the appeal must be allowed.


In my view, the petition, if allowed to proceed, must fail and the presentation of the petition would be an abuse of the
process of the court and should be prevented under the inherent jurisdiction which the court possesses. In substance, the case is
covered by Re Cuthbert Cooper & Sons Ltd, which is not truly distinguishable from the present case. So far as counsel for the
defendant has attempted to rely on the principles of partnership, in my view his arguments are misplaced. The court has treated
the situation of a private limited company with only a few shareholders as resembling a partnership, but counsel for the defendant
has attempted to extend the principle of such cases as Re Yenidje Tobacco Co Ltd to such a degree that the principles of the
Companies Act, 1948, could no longer be operated.
I would refer to the observations of Simonds J in Re Cuthbert Cooper & Sons Ltd ([1937] 2 All ER at p 468; [1937] Ch at p
398). He said:

It has been pressed upon me that I am to be guided by the principles laid down by the Court of Appeal in Re Yenidije
Tobacco Co. Ltd. and followed recently by CROSSMAN, J., in Re Davis and Collett Ltd.. But whether it be a matter of
articles of association or articles of partnership the rights of the parties are determined by those articles, and the question
whether it is right for me to apply the principles of partnership to the question of dissolution, depends on what are the
contractual rights of the 949 parties as determined by the articles of association in this case. Accordingly, when I come to
consider the allegation in the petition I must have regard to the rights of the parties as determined by the bargain into which
they have entered.

I would add that in the present case the defendant is in the position of a minority shareholder in a company which is
governed by its articles and with the small holding which he has it would be most unreasonable to deal with him as if he were a
partner with the rights of dissolution which are conferred by the rules of partnership law in appropriate cases.
In Re Cuthbert Cooper & Sons Ltd, as in the present case, the companys articles gave the directors power to refuse
registration of the transfer without assigning any reason. The authorities over a long period of years have established that the
onus of showing that the decision to refuse registration was made in good faith is not on the directors. It must be shown by those
who attack their decision that the directors did not act in good faith. Willmer LJ has gone through the cases c, and I need not
repeat that labour. The court will, therefore, assume that the directors acted in good faith, and for the purpose of reversing this
assumption the allegations in the petition must be considered as in Re Cuthbert Cooper & Sons, Ltd.
________________________________________
c See p 945, letter i, ante.

In the present case the allegations in the petition relied upon may be summarised as (1) the previous private dispute between
Mr Forte and the defendant, (2) the alleged control of Mr Forte over the company and its directors, and (3) the argument that no
grounds for the refusal to register other than spite against the defendant could exist; and it is alleged that the directors had made
no inquiries. I find those allegations on their face quite insufficient to entitle the defendant to a winding-up order and the case
that it is just and equitable to wind up the company could not be established.
There is also the point referred to by Simonds J ([1937] 2 All ER at p 468; [1937] Ch at pp 399, 400), which has been quoted
by Willmer LJd, that alternative and more appropriate remedies are available to the defendant, that is to say, an action or
summons or motion for rectification of the register of members, which would seem also to provide the defendant with more
efficient methods of proving the allegations if they are well-founded. Moreover, it seems incongruous to me to seek the
destruction of the plaintiff company. I am bound to say that my conclusion in the present case, which is reinforced by the terms
of the letter from the defendants solicitors of 1 May 1963, is that the defendant proposes to adopt the method of a petition to
wind up the company, rather than the other and more appropriate remedies open to him, because he hopes thereby to bring
pressure on the directors to alter their attitude. This is an improper use of the process and unreasonable, and in my view the
plaintiff company is justified in objecting to it, even if the company is not a trading company, for the reasons which have been
already mentioned by Willmer LJ
________________________________________
d See p 948, letter a, ante.

Section 225(2) of the Companies Act, 1948, affords no help to the defendant in the present case. Of course, the remedy
asked for by the plaintiff company is one which is not lightly to be applied, and indeed only in a clear case, but in my opinion this
is a case in which the plaintiff company should be freed from the improper pressure which is being used by the defendant and an
injunction should be granted.

CROSS J. I agree. The defendant asserts that the directors of the plaintiff company did not exercise their power to refuse to pass
the transfers in good faith in what they considered, rightly or wrongly, to be the best interests of the company. He suggests that
they were actuated by spite against him because he 950 had successfully asserted his ownership of the ten thousand shares and
had refused to sell them to Mr Charles Forte at his price. Now if the defendant wishes to make out that case, the proper course
for him to take is to bring an action in the Chancery Division asking for a declaration to that effect and consequential relief by
way of rectification of the register and possibly alternatively by way of damages in the event of the sale of the shares having gone
off; but in the circumstances of this case a petition to wind up the company is not, in my judgment, a form of relief which the
defendant can properly invoke. I find support for that view in the judgment of Simonds J in Re Cuthbert Cooper & Sons Ltd, to
which my Lords have referred. Pennycuick J treated that decision as applying only to the rather unusual case where the
petitioner, though he is a contributory and so has a locus standi to petition, is not on the register and is seeking to get on it. He
thought that what Simonds J said did not apply to the ordinary case such as this where the complainant is a shareholder on the
register who wishes to transfer all or some of his shares to someone else. For my part, I see no ground for limiting the
application of the Cuthbert Cooper case in that way.
Further, I think that in that case there was really more to be said for the propriety of proceeding by way of petition to wind
up than there is in this case, for there the petitioners were between them beneficially entitled to fifty per cent of the shareholding
in the company, and the holders of the other fifty per cent were the persons whose conduct was impugned. There were no other
shareholders interested, and the company was in effect a quasi partnership. Here, on the other hand, there are a number of
outside shareholders who are not, as far as I know, in any way concerned with the dispute between the defendant and the
directors. They would not be affected one way or the other by the success or failure of the defendant in an ordinary action in the
Chancery Division. On the other hand, they might be very much affected by a winding-up order.
It was suggested that s 225(2) of the Companies Act, 1948, which was not in existence in 1937, had altered the position. I
do not agree. That subsection only comes into play if and when the court is satisfied that the petitioners are entitled to relief by
way of a winding-up order. I do not say that there could never be a case in which the facts were such that it would be proper to
launch a winding-up petition in respect of a refusal to register a transfer, but, for the reasons which have been given by my lords,
I am satisfied that this particular petition is bound to fail, and I have, for my part, no doubt that the reason why the defendant
chose to proceed by way of petition rather than by way of action was because he thought that more effective pressure could be
brought to bear on the company in that way than if he sought his proper remedy.
For these reasons, I agree that the appeal should be allowed.

Injunction granted.

Solicitors: Paisner & Co (for the plaintiff company); William Charles Crocker (for the defendant).

Henry Summerfield Barrister.


951
[1963] 2 All ER 952

C H W (Huddersfield) Ltd v Inland Revenue Commissioners


TAXATION; Surtax

HOUSE OF LORDS
LORD REID, LORD JENKINS, LORD HODSON, LORD GUEST, LORD PEARCE
21, 22, 23 MAY, 20 JUNE 1963

Surtax Undistributed income Direction and apportionment Apportionment of actual income of the company in
accordance with the respective interests of the members Shareholders selling all their shares before end of accounting
period to a company not within s 245 Whether original shareholders had any interest after end of accounting period for
purposes of s 245 and s 248(1) Whether apportionment valid Income Tax Act, 1952 (15 & 16 Geo 6 & 1 Eliz 2 c 10), s 245, s
248(1).

Until 29 January 1957, the whole share capital of the appellant company was held by four shareholders; at that date they sold
their shares, nine-tenths being acquired by A Ltd to which s 245 a of the Income Tax Act, 1945, did not apply, and one-tenth by
another company S Ltd On 31 January 1957, the appellant company ceased to trade. On 7 July 1958, the Special
Commissioners made a surtax direction on the appellant company under s 245, and apportioned its actual income for the relevant
accounting period of ten months ending 31 January 1957 (less a small preference dividend paid) on a time basis, ie, as to
303/306ths to the original four shareholders and as to 3/306ths to the two purchasing companies. On the appeal it was conceded
(a) that the point of time to which regard should be had in determining whether s 245 applied to the appellant company was the
end of the relevant accounting period, viz, 31 January 1957, at which time the appellant company was a subsidiary of A Ltd (b)
that the four original shareholders were members within s 248(1) of the Act of 1952. By s 248(1) a of the Income Tax Act,
1952, the apportionment of the actual income of the company shall be made in accordance with the respective interests
of the members.
________________________________________
a The relevant terms of s 245 and s 248(1) are set out at p 953, letter i, to p 954, letter c, post.

Held Lord Guest dissenting): it had not been competent to the Special Commissioners to make the apportionment b to the four
original shareholders because at the crucial date (viz, 31 January 1957, the end of the relevant accounting period) the four
original shareholders had not any interest in the actual income of the company within s 248(1) (as distinct from an interest in
profits accruing day by day during the greater part of the accounting period), as they had sold their shares before the end of the
relevant accounting period (see p 956, letters a and h, p 957, letter h, and p 956, letter e, post; cf, p 960, letter f, and p, 962 letters
d and e, post.)
________________________________________
b The consequence was that, as the apportionment was not competent, the appellant company was a subsidiary of A Ltd for the purposes of s
256(1) of the Income Tax Act, 1952, on 31 January 1957, and was excluded by s 256(1) from the net of s 245. See per Lord Reid at p 954,
letter i, post; and cf, [1962] 3 All ER 243, holding (ii).

Dicta of Lord Russell Of Killowen in F P H Finance Trust Ltd v Inland Revenue Comrs ([1945] 1 All ER at p 497) and of
Lord Atkin in Fattorini Ltd v Inland Revenue Comrs ([1942] 1 All ER at p 626) considered and applied.
Per Lord Reid (Lord Jenkins concurring): the decision of the Court of Appeal c [on the argument of the appellant company
that a reasonable time (for the purposes of s 245(1)) had not been proved to have elapsed before the dividend was declared] was
clearly right (see p 956, letter g, post).
________________________________________
c See [1962] 3 All ER 243, holding (iii).

Decision of the Court of Appeal ([1962] 3 All ER 243) reversed on the ground stated at letter f, above.
952

Notes
As to the power to make surtax directions and apportionments, see 20 Halsburys Laws (3rd Edn) 551, para 1073, and 558, para
1085; and for cases on the subject, see 28 Digest (Repl) 358, 359, 15801586, 364374, 15981632.
For the Income Tax Act, 1952, s 245, s 248(1), s 256(4), see 31 Halsburys Statutes (2nd Edn) 232, 235, 245.

Cases referred to in opinions


Fattorini Ltd v Inland Revenue Comrs [1942] 1 All ER 619, sub nom Thomas Fattorini (Lancashire) Ltd v Inland Revenue
Comrs [1942] AC 643, 111 LJKB 546, 167 LT 45, 27 Tax Cas 328, 2nd Digest Supp.
Fendoch Investment Trust Co v Inland Revenue Comrs, Alporteno Investment Trust Co v Inland Revenue Comrs [1945] 2 All ER
140, sub nom Alporteno Investment Trust Co v Inland Revenue Comrs, Fendoch Investment Trust Co v Inland Revenue
Comrs 114 LJKB 291, 173 LT 35, 27 Tax Cas 53, 28 Digest (Repl) 371, 1625.
FPH Finance Trust Ltd v Inland Revenue Comrs (No 2) [1945] 1 All ER 492, [1946] AC 38, 114 LJKB 203, 172 LT 278, 28 Tax
Cas 209, 28 Digest (Repl) 371, 1624.

Appeal
This was an appeal from an order of the Court of Appeal (Lord Denning MR Donovan and Pearson LJJ) dated 22 June 1962,
reported [1962] 3 All ER 243, dismissing the appellant companys appeal from an order of the High Court (Plowman J), dated 26
July 1961, reported [1961] 3 All ER 551, whereby an appeal by the appellant company by way of Case Stated from a decision of
the Special Commissioners was dismissed and their decision was upheld.

Sir Andrew Clark QC Hubert H Monroe QC and M P Nolan for the appellant company.
Heyworth Talbot QC Alan S Orr QC E Blanshard Stamp and J Raymond Phillips for the Crown.

20 June 1963. The following opinions were delivered.

LORD REID. My Lords, this case arises out of surtax directions made on the appellant company and consequent apportionment
of its income for the accounting period 1 April 1956, to 31 January 1957, under s 245 and s 248 of the Income Tax Act, 1952.
Until 29 January 1957, the whole share capital was held by four persons, two Garsides and two Spencers. They then sold their
shares for 279,500, nine-tenths being acquired by Anglo-French Trust Ltd and one-tenth by Standard Industrial Trust, Ltd.
When the companys accounts for the period in question were made up they showed a net profit of 32,103. On 14 February
1957, these accounts were adopted by the company in general meeting, and it was resolved to confirm payment of a small
preference dividend, which the former shareholders had received, and to pay no further dividend for the period. Then on 7 July
1958, the assessing Special Commissioners made a direction and apportionment of the gross income for the period of 55,819.
They apportioned 54,282 of this to the Garsides and Spencers in respect that they had owned the ordinary shares for 303 days of
the period, and 537 to Anglo-French Trust Ltd and Standard Industrial Trust, Ltd in respect that they had owned the shares for
the remaining three days of the period. The appellants maintain that the commissioners had no right to make this direction and
apportionment. The relevant sections of the Act of 1952 are as follows:

245. With a view to preventing the avoidance of the payment of surtax through the withholding from distribution of
income of a company which would otherwise be distributed, it is hereby enacted that where it appears to the Special
Commissioners that any company to which this section applies has not, within a reasonable time after the end of any year
or other period for which accounts have been made up, distributed to its members, in such manner as to render the amount
distributed liable to be included in the statements to be made by the members of the company of their total income for the
purposes of surtax, a reasonable part of its actual income from all sources for the said year or other 953 period, the
commissioners may, by notice in writing to the company, direct that, for purposes of assessment to surtax, the said income
of the company shall, for the year or other period specified in the notice, be deemed to be the income of the members, and
the amount thereof shall be apportioned among the members.
248(1). Where a direction has been given under s. 245 of this Act with respect to a company, the apportionment of the
actual income from all sources of the company shall be made by the Special Commissioners in accordance with the
respective interests of the members.
256(1). Section 245 of this Act shall apply to any company which is under the control of not more than five persons
and which is not a subsidiary company or a company in which the public are substantially interested.
(2) For the purposes of this section, a company shall be deemed to be under the control of not more than five persons
(c) if(i) on the assumption that the company is a company to which the said s. 245 applies more than half the
income of the company (including any income which has been apportioned to it, or could on either of those assumptions be
apportioned to it, for the purposes of this Chapter) could be apportioned for those purposes among not more than five
persons.
In ascertaining under para. (c) of this subsection whether or not income could be apportioned among not more than
five persons, account shall, in cases where an original apportionment and any sub-apportionment are involved, be taken
only of persons to whom income could be finally apportioned as the result of the whole process of original apportionment
and sub-apportionment.
(4) For the purposes of this section, a company shall be deemed to be a subsidiary company if by reason of the
beneficial ownership of shares therein the control of the company is in the hands of a company not being a company to
which s. 245 of this Act applies, or of two or more companies none of which is a company to which the said s. 245 applies:

Provided that, notwithstanding anything in this subsection, a company which is deemed for the purposes of this section to be
under the control of not more than five persons shall not be deemed to be a subsidiary company unless it can be deemed to be
under the control of not more than five persons only by including among the persons mentioned in para (a), para (b) or para (c) of
sub-s (2) of this section a company to which the said s 245 does not apply and which is not the nominee of any other person.
It was agreed that we must look at the end of the accounting period to see whether s 245 applied to the appellant company. So it
is unnecessary to decide whether that is the universal or general rule, and we can accept this agreement for the purposes of this
case. At that date the appellant company was under the control of Anglo-French Trust Ltd Section 256(1) would exclude the
appellant company if it was a subsidiary company, but the question is whether the proviso to s 256(4) required the appellant
company to be deemed not to be a subsidiary company. If the proviso applies to this case, then s 245 applied. So the question is
what does the proviso mean? Its language is very obscure, and again I think that we can proceed on the agreement of the parties.
They both say that you must begin by assuming that s 245 does apply; and then, on that assumption, you must see whether it
would be competent for the commissioners to apportion more than half the income to persons other than the controlling company,
Anglo-French Trust, Ltd.
954
That takes us straight to s 248(1). That section requires the actual income of the company to be apportioned in accordance
with the respective interests of the members. So there are two questionswho are members and what is meant by their
respective interests. The appellant company agrees that the Garsides and Spencers were members, although they had ceased
to have any interest in the company before the crucial date, the end of the accounting period. It admits that that is the result of the
decision of this House in Fendoch Investment Trust Co v Inland Revenue Comrs; and I think that that must be so. In the present
case the Garsides and Spencers had received 1,000 in preference dividend out of the profits of the period, and it would be
strange if that sum could not be apportioned to them because the new shareholders never had any kind of interest in it.
The question what is meant by respective interests is much more difficult. It cannot mean vested interest in the income
because these sections are dealing with income which the appellant company has withheld from distribution. Such income is
simply part of the asserts of the appellant company until it has decided what to do with the income and no shareholder has any
right to any particular part of the appellant companys assets. At one time it was thought that this meant the interests which
members would get if the income were distributed in the ordinary way as dividend; but FPH Fnance Trust Ltd v Inland Revenue
Comrs (No 2) shows that interests must be given a much wider meaning than that. This company had an odd structure. The
preference shareholders had control and got all surplus assets in a liquidation, while the ordinary shareholders were entitled to all
dividends (except a small preference dividend). But the rights of the ordinary shareholders were worth little because the
preference shareholders could refuse to declare a dividend, accumulate the income, and ultimately get it in a liquidation. Plainly
they had a very real interest in the income, while the ordinary shareholders interest was shadowy. So it was held that on a surtax
direction the real interest could be regarded and the case was sent back to the commissioners to make proper apportionments.
Lord Russell said ([1945] 1 All ER at p 497; [1946] AC at p 51):

They may well ask themselves the questions (i) on whom did it depend whether or not the income should be withheld
from distribution, and (ii) for whose benefit was the distribution withheld or (in other words) who would avoid payment of
surtax by the withholding?

Lord Russell had just referred to the preamble to what is now s 245 of the Income Tax Act, 1952, and he was obviously
speaking of withholding after the end of the accounting period. If those questions are applied in their natural sense to this case it
could not be right to apportion anything to the Garsides and Spencers in respect of their ordinary shares. They had nothing to do
with any failure to distribute after the end of the accounting periodthey were not even shareholders thenand it did not matter
a penny piece to them whether the income was then distributed or withheld.
I must now try to explain the nature of the interest which the Crown says that the Garsides and Spencers had in this income.
During the 303 days when they held control the appellant company was earning income. There is no finding to this effect, but I
am willing to assume that the commissioners were entitled to infer that 303/306 parts of the actual income for the period had been
earned during those 303 days; and I shall further assume, but without expressing any opinion, that that gave them an interest in
that income and therefore in the actual income for the period. In selling their shares they sold that interest. I have no doubt
that the purchasers had a pretty shrewd idea of how much profit had been earned, and that this was reflected in the price of the
shares. If the sellers had sold their interest, how did they still retain it at the crucial 955 date, the end of the period? The
purchasers acquired it and had it at that date: they could have declared a dividend and put the whole of the actual income in their
pockets, excepting the small preference dividend already paid. What the sellers had was the price of the interest: they had sold
an asset full of profit for a capital sum. There are other sections which deal with dividend stripping and the like, and the facts of
this case with regard to the Garsides and Spencers seem to me to be far removed from the mischief of withholding profits from
distribution.
The Crown seeks to bring in this case by an ingenious adaptation of Lord Russells questions. The Crown says that during
those 303 days the Garsides and Spencers could have distributed the profit already earned to themselves, and that they avoided
surtax by not doing so. That would be such a strange thing to penalise that I would require clear words to justify that result. I do
not think that the words of the leading section, s 245, are capable of being so extended and certainly such a case is not within its
apparent scope. The express purpose of the section is to prevent avoidance of surtax by withholding from distribution income
which would otherwise be distributed. I take it that otherwise means but for the attempt to avoid surtax. I suppose that could
include withholding a customary interim dividend, but even that is excluded by the next part of the section: there must be a
failure to distribute within a reasonable time after the end of any year. So a failure to distribute during the year does not bring
the section into operation. That may not be directly relevant to s 248, but I think that it does support the view that interest in s
248 means interest surviving at the relevant time.
The question in Fendochs case was quite different. The section which applied there does not apply here. There the question
was whether any person is or is likely to be able to secure that income present or future will be applied for his benefit, and it
was held that that did not mean on the last day of the year but at any time during the year of assessment. There was no difficulty
there such as would arise here if one read interest as meaning interest which had existed at any time. As I have said, any
interest which the seller once had passed to the purchaser: so, if interest were to include former interest, you would have two
people with interests of the same kind in the same income. I do not see how the commissioners could then simply choose which
of the two was to bear the apportionment. They must exercise their powers in a reasonable manner, and I cannot think that s 248
gives them this choice. I should add that there was some argument about what would happen if control was sold immediately
after the end of the accounting period and before a reasonable time for distribution had elapsed. I do not think it necessary to
discuss that question.
As the appellant company in my judgment succeeds in its main argument, I need not deal with their argument that it had not
been proved that a reasonable time had elapsed. The decision of the Court of Appeal in that matter seems to be clearly right. In
my opinion this appeal should be allowed with costs. The question of law contained in the Case Stated should therefore be
answered in the negative.

LORD JENKINS. My Lords, I concur in the opinion just delivered by my noble and learned friend Lord Reid.

LORD HODSON. My Lords, the relevant sections of the Income Tax Act, 1952, have been cited and the substantial question
which emerges is as to the meaning of s 248(1), which provides that when a direction has been given under s 245 of this Act with
respect to a company, the apportionment of the actual income from all sources of the company shall be made by the Special
Commissioners in accordance with the respective interest of the members.
It is conceded that the individual ordinary shareholders who are concerned, the Garsides and the Spencers, must be regarded
as members of the appellant 956 company by virtue of the decision of this House in Fendoch Investment Trust Ltd v Inland
Revenue Comrs, which decided that members in a parallel context included those who were members at any time during the
relevant period. It is, however, contended by the appellant company that no distribution can be made in accordance with their
interests, for, though members, the shareholders had no apportionable interest in the ordinary shares at the end of the relevant
accounting period. The Crown on the other hand contends that once it is conceded that the Garsides and the Spencers were
members the rest follows and all that is to be done is to ascertain what their respective interests were when they were members
and to make an apportionment accordingly. The Crown points out that members are defined in s 255(2) of the Income Tax Act,
1952, as including any person having a share or interest in the capital or profits or the income of the company. The Garsides and
Spencers were members and received the benefit of the profit in the sale price of the shares, the profit having accrued during the
time when they were in control of the company. Donovan LJ in the Court of Appeal ([1962] 3 All ER at pp 245, 246) found
himself reluctantly compelled to accept the submission of the Crown, but for my part I do not find myself so constrained.
The Garsides and the Spencers were no doubt interested as shareholders in the profits accruing from day to day, but, until the
actual income had been ascertained at the end of the accounting period, there was nothing to distribute unless by way of interim
dividend pending the ascertainment and in anticipation of the profit on the year. No interim dividend was in fact paid. True it is
that the profits were no doubt piling up and these were reflected in the price paid by the purchasers when the Garsides and
Spencers sold their shares before the end of the period, but the section speaks of the actual income of the company not the
profits accruing from day to day. This is the income tax income in the ordinary sense in which those words are used, and this
accords with the language of s 245 which begins by the introductory words

with a view to preventing the avoidance of the payment of surtax through the withholding from distribution of income
of a company which would otherwise be distributed

and which continues by enacting that where it appears to the Special Commissioners that any

company to which this section applies has not, within a reasonable time after the end of any year or other period for
which accounts have been made up, distributed to its members

These words, in my opinion, refer to the same income as the actual income referred to in s 248(1), the subsection with which
your lordships are mainly concerned, and compel me to the conclusion that the words in accordance with the respective interests
of the members must be more narrowly construed than might at first appear. In other words, they do not refer to the interests of
the members in the profits accruing from day to day during their membership, but to their interests in the income tax income of
the company.
The decision in the Fendoch case had to do with apportionment to members in accordance with their respective interests, but
the question was whether the member was one of whom it could be predicated at any time in the fiscal year that they were able
to secure the income or assets of the company to be applied for her benefit. The section there under consideration was s 15 of
the Finance Act, 1939, which deals with special investment companies and special powers with respect to persons able to secure
such benefit. This section is now represented by s 260(1) of the Act of 1952. The question was answered adversely to the
taxpayer on the facts of that case, but it does not seem to me to follow that the questionwhat are the respective interests of the
members? 957in the context of withholding income from distribution must be answered in the same way.
I am assisted in my conclusion in favour of the appellant company by comparison with s 237(1) of the Income Tax Act,
1952, which contains provisions for preventing avoidance of surtax by sales cum dividend. This section is apt to cover
systematic and, not exceptional, sales by individuals. The significance of s 237, which was introduced by the Finance Act, 1927,
is that it makes express reference to income from assets as having accrued from day to day. I contrast with this the language of
s 245 which speaks of

withholding from distribution of income of a company which would otherwise be distributed,


and of a company which

has not, within a reasonable time after the end of any year or other period for which accounts have been made up,
distributed a reasonable part of its actual income from all sources for the year or other period

Even if the appellant company could, as was most probably the case, have distributed its profits by way of interim dividends I do
not see that it can be said that it was withholding, within the meaning of s 245, until the end of the accounting period had been
reached and the profits for this income tax year ascertained. This event did not happen until after 19 February 1957, when the
accounts were passed or at earliest until 31 January 1957, the end of the relevant accounting period. The avoidance of surtax was
thus occasioned by the sale of shares before the end of the accounting period and, if their sale had been within the mischief struck
at by s 237, appropriate steps could have been taken under that section.
In my opinion therefore the contention of the Crown that the commissioners were entitled to apportion the whole of the
income which accrued while the Garsides and Spencers were members fails. The relevant income is not the income from day to
day but the income tax income when ascertained. See the opinion of Lord Atkin in Fattorini Ltd v Inland Revenue Comrs
([1942] 1 All ER 619 at p 626; [1942] AC 643 at p 658):

Actual income does not mean the specific receipts that come in from time to time but the income-tax income as
calculated at the end of the year of assessment.

I would allow the appeal.

LORD GUEST (read by Lord Jenkins). My Lords, I do not propose to rehearse the rather complicated facts in this case but only
to state the points at which I have the misfortune to differ from the majority of your lordships.
The primary question which arises is whether the appellant company is one to which s 245 of the Income Tax Act, 1952,
applies. The appellant company is subject to s 245 if, in apportioning the income of the company under s 248(1), it would be
open to the Special Commissioners to apportion more than half the income to the Garsides and Spencers, because then the
company is not a subsidiary of Anglo-Franch Trust Ltd in terms of s 256. The crucial question therefore revolves around s 248(1)
which is in the following terms:

248(1). Where a direction has been given under s. 245 of this Act with respect to a company, the apportionment of the
actual income from all sources of the company shall be made by the Special Commissioners in accordance with the
respective interests of the members.

It was matter of concession by the appellant company that, not withstanding the fact that the Garsides and Spencers had parted
with their shares before the end of the relevant period, they were nevertheless members within the meaning of s 255(2). This
concession was inevitable as a result of the decision of this 958 House in Fendoch Investment Trust Co v Inland Revenue Comrs.
This case was concerned with s 15(1) of the Finance Act, 1939, which, so far as relevant, is in the following terms:

15.(1) If in the case of any investment company the Special Commissioners are of opinion that any person who is
not a member of the company for the purposes of s. 21 of the Finance Act, 1922, and the enactments relating thereto is, or
is likely to be, able to secure that income or assets, whether present or future, of the company will be applied either directly
or indirectly for his benefit, they may, if they think fit, treat him as a member of the company for the said purposes.

The question arose whether the commissioners were entitled to give a direction to a person who was able to secure that the
income would be applied for his benefit at some time during the year of assessment but was not in a position so to secure at the
end of the year of assessment. This House held that the commissioners could make an apportionment against a person of whom it
could be predicated at any time in the fiscal year that he was able to secure, etc Lord Simonds, who gave the only judgment,
said in the course of his speech ([1945] 2 All ER at p 144):

It was contendedand this is, of course, fundamental to the argumentthat under the principal Act an apportionment
could only be made against a person who was a member (as defined d by s. 21(7)) on the last day of the accounting period.
Section 21(1) of the principal Act provides that the commissioners may direct that: for purposes of assessment to
super-tax, the said income of the company shall, for the year or other period specified in the notice, be deemed to be the
income of the members, and the amount thereof shall be apportioned among the members and para. 8 of Sch. 1 to the
Act provides that the apportionment shall be made in accordance with the respective interests of the members I
find nothing in these words which supports the appellants contention, or would restrict the meaning of members to
persons who were members during the whole period or the last or any other day of it. The language of the section is clearly
apt to include any person who was a member at any time during the period in question.
________________________________________
d Ie, by s 21(7) of the Income Tax Act, 1922.

Section 21(1) of the Income Tax Act, 1922, has its counterpart in s 245 of the Act of 1952 and s 21(7) is the forerunner to s
255(2) of the latter Act. As the Garsides and Spencers were not shareholders either at the end of the relevant period or at the date
of the direction, the concession which must be taken for the purpose of the present case can only have been on the footing that, in
terms of s 255(2), they had at the relevant time an interest in the capital or profits or income of the company. Member
therefore for the purposes of s 245 and s 248(1) includes a person who was a shareholder during the relevant period, but has
ceased to be a shareholder at the end of this period. A direction can be given to him by the commissioners under s 245
presumably because he had a qualifying interest during the time when he was a shareholder in the capital, profits or income of the
company. The appellant company, while agreeing that the Garsides and Spencers were candidates for apportionment as members,
argued that the apportionment under s 248(1) must be nil, because they had no interest in the actual income at the end of the
relevant period. If sound, this indeed would be a remarkable result. It would mean that a shareholder who parted with his shares
during the accounting period could never have a direction made against him and that the only persons against whom directions
could be made would be the shareholders at the end of the relevant period. This, with respect to your lordships who hold the
contrary view, is the negation of 959 what Lord Simonds said in the Fendoch case ([1945] 2 All ER at p 144) and directly
contrary to the concession made by the appellant company in the present case.
I will now attempt to state the argument for the appellant company that a nil direction must be made against them. First,
reliance was placed on the opening words of s 245:

With a view to preventing the avoidance of the payment of surtax through the withholding from distribution of income
of a company which would otherwise be distributed
These words, it was said, indicated that the mischief which the section was designed to correct was the avoidance of surtax
through the withholding of income for an unreasonable time after the end of the relevant period. It was argued that, as there was
no withholding from distribution of the appellant companys income while the Garsides and Spencers were members of the
company, the section never applied. It would be a mistake in my view to place too much reliance on the opening words of the
section and in any event the withholding struck at must always occur after the end of the relevant period, because it is not until
after the expiry of a reasonable time after the end of the period that the commissioners are entitled to give a direction. Moreover,
it is quite unnecessary for the operation of the section that surtax has in fact been avoided. Further, it was said that what was
struck at was the avoidance of surtax by the withholding of income for an unreasonable period and that what had occurred in this
case was the avoidance of surtax by the sale of the shares cum dividend before the end of the period, a situation covered by s 237.
In my view the fact that a scheme to avoid surtax may be circumvented by two different provisions in the Income Tax Act, 1952,
does not mean that one or other of these provisions is inoperable. Moreover, s 237, which is limited in its application (see sub-s
(7)), relates to different matters not to the withholding by a company of the distribution of its income.
I therefore reject the argument based on what has been loosely described as the preamble to s 245 and turn to the operative
part of s 248(1). The appellant company argues that as the Garsides and Spencers had no interest in the actual income at the end
of the relevant period, there could be no apportionment made against them. The question is therefore what interest, if any, did
they have in the actual income. Actual income must, I think, mean incometax income, and it cannot mean the companys
income as it accrues de die in diem, because the time for distribution cannot arrive until the actual income is ascertained at the
end of the relevant period. At this stage it is necessary to refer in some detail to FPH Finance Trust Ltd v Inland Revenue Comrs
(No 2) which was strongly relied upon by the appellant company. In that case the companys share capital was divided into 1,000
ordinary shares and 10,000 preference shares. Under the articles of association the preference shareholders had the voting
control. Until winding up the preference shareholders were only entitled to a dividend: all the profits were to go on distribution
among the ordinary shareholders. On a winding-up the ordinary shareholders were to receive 1,000 and the whole surplus
assets were to go to the preference shareholders. This House held that there was no justification for restricting the interests in
respect of which apportionment might be made under s 21 of the Income Tax Act, 1922, as rights in declared dividends and that
the different interests of members should be considered including their right to undistributed profits in a winding-up. A direction
against the preference shareholders was held valid. The observations of Lord Russell of Killowen I quote from the report in Tax
Cases (28 Tax Cas 245; [1945] 1 All ER at p 497):

The foundation of the power given by the section to the commissioners is the fact that the company, for an
unreasonable time after the end of the 960 period for which accounts have been made up, has refrained wholly, or in
unreasonable measure, from declaring dividends in general meeting, and thus distributing its profits among the persons
entitled, according to their rights in dividends so declared. Nothing would have been easier than to provide that the income
should be apportioned among the same persons, and in the same manner, as if the income in question had been so
distributed by way of dividend. The section, however, does not do this. It first enacts that the income in question is to be
treated as if, instead of being the companys income, it were the income of the members. No notional declaration of
dividend is envisaged at all. The income (which is now envisaged not as the companys income at all, but as income of the
members) then has to be apportioned in accordance with the respective interests of the members. What justification can
there be for restricting the interests which the commissioners may take into consideration, to rights in declared dividends,
when no declaration of dividend, notional or otherwise, is contemplated by the section? I can find none. I can conceive
many cases in which they might well so act, but I cannot assent to the view that they are compelled so to act in all cases.
Obviously everyone who falls within the extended definition of member is not necessarily to be included in the
apportionment. In my opinion the commissioners, in apportioning the income among the members, should determine who
are the persons of whom it can be said (i) that they fall within the definition, and (ii) that they are the persons who, in view
of all their interests in the company, are the persons really interested in the income in question and in what proportions.
Further, I think that, in considering these interests and apportioning the income among members, the commissioners may
properly be guided by the preamble to s 21 and endeavour to make an apportionment appropriate to their interests to those
members for whose benefit, in relation to the avoidance of payment of super-tax (now sur-tax), the distribution of income
has obviously been withheld. They may well ask themselves the questions, (i) on whom did it depend whether or not the
income should be withheld from distribution, and (ii) for whose benefit was the distribution withheld or (in other words)
who would avoid payment of sur-tax by the withholding? If the same individuals figure in each answer, those are
obviously the persons who, according to their interests in the company, own the real and paramount beneficial interest in
the fund in question. Other members may also have an interest therein, but to a smaller extent.
Applying this view to the facts of the present case, there should be no doubt about the broad result. Both questions
admit of one answer onlyviz., Mrs. Latilla and her daughters, whose voting power enabled them (i) to prevent (before
liquidation) any distribution to the ordinary shares beyond a nominal percentage, and thus to enforce the understanding
referred to in the Case Stated; (ii) to wind up the company at any moment and (subject to the payment of 1,000 to the
corporation) get all the surplus assets (including the fund in question) for themselves, or (iii) if ever they wanted the
income paid to them as such, to alter the articles of association by special resolution and (not withstanding art, 47) thereby
increase the dividends payable on the preference shares to any desired amount.

It was argued that if the two questions posed by Lord Russell of Killowen were answered in this case the appellant company
would not figure in the answers. These observations must, however, be considered secundum subjectam materiam and they are
observations made with reference to the particular facts of that case. The position of Mrs Latilla and her daughters was that they
were in control of the company and this entitled them to enforce the arrangement designed to secure for the benefit of these
persons immunity from liability to surtax. The ordinary shareholders were at the mercy of the Latilla family. I 961 cannot think
that the questions posed by Lord Russell who was a party to the decision in the Fendoch case were intended to be of general
application. If they were, it humbly seems to me that they are inconsistent with the speech of Lord Simonds in the Fendoch case.
Tested by these questions no ex-member would ever be liable to a direction under s 245 because after he had ceased to be a
member he would have no say in the withholding of the distribution of the income. If, however, the crucial tests are the first two
tests proposed by Lord Russell, namely (1) that the persons fell under the definition of member and (2) that they are the
persons who in view of all their interests in the company are the persons really interested in the income, I am unable to reconcile
the two cases. It is only if this view is taken of FPH Finance Trust Ltd v Inland Revenue Comrs (No 2) that it is possible to
reconcile the decision with the Fendoch case.
In the present case the commissioners took the view that the apportionment of the appellant companys income should be on
a time basis as between the Garsides, Spencers, Anglo-French Trust Ltd and Standard Industrial Trust, Ltd. This, in my view, is
the only possible apportionment as between members and ex-members. The Garsides and Spencers had an interest in the income
until three days before the end of the relevant period. They were in control of the company until 29 January 1957, the date of the
sale to Anglo-French Trust Ltd and Standard Industrial Trust, Ltd. They could have wound up the company and shared the assets.
They could have distributed the profits of the company as income at any time during that period. By the sale to Anglo-French
Trust Ltd and Standard Industrial Trust Ltd on 29 January 1957, they disabled themselves from distributing the income for the
income period to 31 March but they retained an interest in the profits of the company which were represented in the purchase
price paid to them. In my opinion the interest necessary to qualify the Garsides and Spencers for a direction under s 248(1) thus
exists. The appellant company is therefore one to which s 245(1) applies. In my view there is no material which would justify us
in interfering with the commissioners direction.
I would dismiss the appeal.

LORD PEARCE (read by Lord Hodson). My Lords, the entire share capital of the appellant company was held by the Garsides
and Spencers, whom I will call collectively the vendors, during most of the accounting period in which the company made a
substantial profit. Near the end of the period the vendors sold the shares which were thus big with dividend. One may infer that
the profits made during the accounting period were reflected in the sale price of the shares. Thus the vendors probably received
as capital on the sale of the shares some part at least of the value of the profits on which, had those profits been distributed by
dividend, they would have had to pay surtax. Against this obvious method of avoiding tax there is provision under s 237, which
was expressly designed to deal with sales cum dividend, and the vendors would be caught by the section, unless it is shown that
the avoidance is exceptional and not systematic and there has not been any such avoidance in the preceding three years.
It happened, however, that subsequently the purchasers failed to declare a dividend in respect of the profits in question
within a reasonable time after the end of the accounting period. There is no evidence that their failure was linked up with the sale
transaction or was due to any design of tax avoidance or to anything other than commercial convenience. It is contended by the
Crown that by reason of that failure of the purchasers s 245 applies. That section deals expressly with
962

preventing the avoidance of the payment of surtax through the withholding from distribution of income of a company
which would otherwise be distributed.

Together with s 248(1) it gives the Special Commissioners power to assess for surtax and apportion among the members the
actual income of the company, for the accounting period where it appears to them that any company to which the section applies
has not, within a reasonable time after the end of any year or other period for which accounts have been made up, distributed to
its members, in such manner as to render the amount distributed liable to be included in the statements to be made by the
members of the company of their total income for the purposes of surtax, a reasonable part of its actual income. The words
actual income mean, not the specific receipts of the company from time to time, but the income as calculated for income tax
purposes at the end of the period (Fattorini Ltd v Inland Revenue Comrs). The normal case to which this section applies is that of
the taxpayer who avoids surtax by allowing his company to accumulate undistributed profits of which he may ultimately obtain
the capital benefit by a winding-up or a bonus issue of shares, and the section was clearly not intending to deal with the kind of
case which is now before your lordships.
Since the appellant company failed to distribute to its members within reasonable time, the section operates if this was a
company to which the section applies. That depends on the ownership of its shares. The purchaser of nine-tenths of the shares
from the vendors was another company, whose status would not attract s 245. To decide the question whether the section applies
to this company in these circumstances one looks to the elaborate terms of s 256. It is conceded that for the purposes of the
present case one may paraphrase them by saying that s 245 applies if, on the assumption that it does apply, one could at the
relevant date apportion to the vendors in accordance with s 248(1) more than half its income for the relevant period in accordance
with the respective interests of the members.
It is conceded that the date at which that problem must be considered is at the end of the account period, namely, 31 January
1957. As at that date the Special Commissioners in apportioning the respective interests ascribed approximately 99 per cent to
the vendors, who then had no rights at all in the company, and one per cent to the purchasers who then had complete ownership
and control of it, in whose power it was to distribute the income for the period, and who would receive the benefit of that
distribution.
It is agreed that by reason of the decision of this House in Fendoch Investment Trust Co v Inland Revenue Comrs
members includes former members who held shares at any time during the relevant period. Thus the vendors are eligible for
the apportionment of their interest. Obviously everyone who falls within the extended definition of member is not necessarily to
be included in the apportionment (per Lord Russell of Killowen in the case of F P H Finance Trust Ltd v Inland Revenue Comrs
(No 2) ([1945] 1 All ER at p 497; [1946] AC at p 51)). The appellant company claims that the vendors interest in the ordinary
shares was but a shadowy spes, already defunct by 31 January 1957, a hope of dividend which never fructified, since they had
parted with their whole interest in the appellant company before the time when the accounts would be made up, when the profits
would be ascertained, and when a dividend would be paid. Their interest should therefore either not be included in the
apportionment or should be apportioned as a trifling amount. The Crown, however, contend that the vendors were in control
during the period when the bulk of the profit accrued from day to day, that they have received the benefit of that profit in the sale
price of the shares 963 and that since they were in that position for 99 per cent of the period, their interest was rightly apportioned
at 99 per cent.
The decision in Fendoch Investment Trust Co v Inland Revenue Comrs related to the effect of a different section of a
different Act (s. 15 of the Finance Act, 1939, now s 260 of the Income Tax Act, 1952) which gives to the Special Commissioners
an unfettered discretion to apportion income between members as they may think fit in the circumstances of the case. Under s
245 and s 248(1), however, there is no such discretion and the Special Commissioners are directed to apportion between members
according to their respective interests. No case has been cited where there has been under this section an apportionment to
former members. Lord Russell of Killowen in the case of FPH Finance Trust Ltd v Inland Revenue Comrs ([1945] 1 All ER at p
497; [1946] AC at p 51) which dealt with apportionment between existing members, poses the questions which the
commissioners may ask themselves in making an apportionment:

(i) on whom did it depend whether or not the income should be withheld from distribution, and (ii) for whose
benefit was the distribution withheld or (in other words) who would avoid payment of surtax by the withholding? If the
same individuals figure in each answer, those are obviously the persons who, according to their interests in the company,
own the real and paramount beneficial interest in the fund in question.

Prima facie the obvious answers to the questions posed by Lord Russell Of Killowen are that at the relevant date it was the
purchasers who withheld and the purchasers who benefited by the withholding. Counsel for the Crown, however, contends that
during most of the relevant period the vendors had an interest and were notionally withholding the income as it accrued from day
to day. But it is noticeable that whereas s 237(4) of the Income Tax Act, 1952, expressly says that for the purposes of that section
income shall be deemed to accrue from day to day, there is nothing to that effect in s 245 or s 248. The greater the apportionment
to the vendors, the less must be the apportionment to the purchasers. The chief difficulty of counsels argument is its effect on the
purchasers. He has to concede that, if it is correct, a surtax payer who at the end of an accounting period purchases a company
with large profits accrued and then fails to declare a dividend, and accumulates the profits for a possible winding up or issue of
bonus shares, should have an apportionment of nothing or of some negligible amount. Yet it would be he who is committing the
mischief aimed at and who gets the benefit from it.
In my opinion weight must be given to the intention expressed in the introductory words of the section e. From the
standpoint of sound trading the conservation of dividends is desirable and compulsory distribution is bad. On the other hand
when the conservation is used not for sound economic reasons but for tax avoidance it is undesirable and must therefore be
stopped. Thus two principles conflict. The opening words of the section were inserted, I think, to underline the fact that the
operators of the section must keep in mind the object, namely to stop avoidance by failure of a company to distribute dividends. I
therefore approach with suspicion any use of the section which would disregard the 964 present actual position of the company in
order to catch some past owner of its shares.
________________________________________
e Ie, s 245 of the Income Tax Act, 1952.

The question really turns on what meaning one gives to the word interest. All the existing interest of the vendors had at
the relevant date been sold to the purchasers. Even assuming that interest includes past and present interest I find it difficult to
accept the view that on the relevant date, 31 January 1957, the past interest of the vendors was for the purposes of this section
greater than the present interest of the purchasers, who could control whether or not the appellant company should pay them a
dividend. The argument that the taxpayer had a notional interest in the profits accruing from day to day entirely disregards the
realities. It would be quite unreal to suppose that he could during the period prepare a running account and declare interim
dividends monthly. The company may prosper exceedingly during some months and lose in others, and the possible resulting
anomalies have been pointed out in argument. Moreover the end of the accounting period is the relevant date for the
apportionment. There is force in the argument, put forward by Mr Blanshard Stamp for the Crown, that the taxpayer had an
interest in the profits since he managed to sell that interest for some figure contained in the purchase price; and, if one were
considering the vendors alone, there would be something to commend it. But I find it difficult to see how one can say that when
the vendors inchoate interest in the profits crystallises in the purchasers hands into a power and a right to receive those profits, it
should be valued at one per cent or thereabouts. On any view the interest must surely then be worth more than it was when the
purchasers had it. The only ground for disregarding the substantial value of the purchasers interest is that, instead of holding
shares while the profit was being earned, he has paid cash for the shares in their final and productive phase. But why does that
fact make his interest on 31 January 1957, any the less? Why is his interest smaller because he has paid for it? Even, therefore, if
interest includes past and present interest and requires at the date of apportionment a balance between a past total interest, which
has ceased before the date when it would in practice produce a dividend, and a present total interest, which in fact enables the
holder to withhold or to declare and receive a dividend, the latter seems to me greater than the former on any apportionment. It
would be otherwise if one could read the word interests as meaning interests during the period when the income was being
earned or was accruing. But I see no justification for this. It is not a natural reading of the section (I.e., s 248(1) of the Income
Tax Act, 1952) nor is it necessary to give a sensible effect to any ambiguity. It would make the vendors in the present case liable
in respect of some untaxed profits, but it would in other cases release a purchaser who is committing the actual mischief aimed at.
I find it impossible to hold that the purchasers practical existing interest on 31 January 1957, was less than the taxpayers
historical and vanished interest. The section was not intended to apply to such a case as this and it is not possible without
distortion to make its provisions apt to such a transaction.
I would accordingly allow the appeal.

Appeal allowed.

Solicitors: Malkin, Cullis & Sumption (for the appellant company); Solicitor of Inland Revenue (for the Crown).

C J Leonard Esq Barrister.


965
[1963] 2 All ER 966

Gollins v Gollins
FAMILY; Divorce

HOUSE OF LORDS
LORD REID, LORD EVERSHED, LORD MORRIS OF BORTH-Y-GEST, LORD HODSON AND LORD PEARCE
23, 24, 25, 29, 30 APRIL, 27 JUNE 1963

Divorce Cruelty Factual question Negative conduct Intention Husbands failure to maintain his family Debts Wife
obliged to support family Wifes health affected Whether intention to injure an essential element of matrimonial offence of
cruelty Whether husband had treated wife with cruelty Matrimonial Causes Act, 1950 (14 Geo 6 c 25), s 1(1)(c).

An intention on the part of one spouse to injure the other is not a necessary element of cruelty as a matrimonial offence (so held
by Lord Reid, Lord Evershed and Lord Pearce, see p 973, letter d, p 974, letters b and c, p 976, letter e, and p 991, letter d, post),
though the presence of such an intention, if it exists, is material and may be crucial (see p 974, letter c, p 969, letter d, p 976,
letter g, and p 991, letter c,post).
Whether cruelty, as a matrimonial offence, has been established is a question of fact and degree, which should be determined
by taking into account the particular individuals concerned and the particular circumstances of the case, rather than by any
objective standard; accordingly, in cases where the two spouses are of normal physical and mental health, and the conduct of the
respondent spouse, so considered, is so bad that the other should not be called on to endure it, cruelty is established, and then it
does not matter what was the respondents state of mind, eg, it is immaterial whether the respondents conduct was aimed at the
other spouse or due to unwarranted indifference, attributable, perhaps, to selfishness or laziness (see p 974, letters b and c, p 970,
letter b, p 972, letter g, p 976, letters f, h and i, p 977, letters a and f, and p 992, letters a b and d, post).
Dictum of Denning LJ in Westall v Westall ((1949), 65 TLR at p 337), and Kaslefsky v Kaslefsky ([1950] 2 All ER 398)
disapproved as regards the necessity for proving intention to injure, if cruelty is to be established; Eastland v Eastland ([1954] 3
All ER 159) disapproved.
Lang v Lang ([1954] 3 All ER 571) explained.
The parties were married in 1946 and there were two daughters of the marriage, born in 1947 and 1949. The husband ran
into debt, sold his farm and bought a house on mortgage which he transferred, subject to the mortgage, to his wife who had given
or lent him considerable sums. To maintain the family the wife ran this house, the matrimonial home, as a guest house. The
husband did little or nothing to help her; he could have obtained paid employment, but did not. He was incorrigibly and
inexcusably lazy, but the evidence did not show any wish on his part to harm the wife, nor was he aggressively unkind to her.
Creditors of the husband tried to make the wife pay, and she did pay, some of his debts. His refusal to try to help her or to earn
money, worried her and made her ill. In the result the wife, who was a normal active and capable woman, was reduced to a
physical and mental state where she would no longer be able to maintain herself or her children. On her complaint justices made
a matrimonial order on the ground of her husbands persistent cruelty. On appeal,

Held Lord Morris of Borth-Y-Gest and Lord Hodson dissenting): applying the principle stated at letter d above and ordinary
standards of human conduct, the finding of the justices that the wife had made out her case of persistent cruelty should stand (see
p 974, letter e, p 976, letter i, to p 977, letter a and p 993, letter h, post).
Decision of the Court of Appeal ([1962] 3 All ER 897) affirmed.

Notes
As to the meaning of, and what constitutes, cruelty, see 12 Halsburys Laws (3rd Edn) 269271, paras 514516; and for cases on
the subject, see 27 Digest (Repl) 294296, 23932422.
966

Cases referred to in opinions


Buchler v Buchler [1947] 1 All ER 319, [1947] P 25, [1947] LJR 820, 176 LT 341, 111 JP 179, 27 Digest (Repl) 350, 2899.
Dysart v Dysart (1844), 1 Rob Eccl 106, 470, 5 Notes of Cases, 194, 163 ER 1105, 27 Digest (Repl) 298, 2426.
Eastland v Eastland [1954] 3 All ER 159, [1954] P 403, [1954] 2 WLR 851, 3rd Digest Supp.
Evans v Evans (1790), 1 Hag Con 35, 161 ER 466, 27 Digest (Repl) 294, 2398.
Hadden v Hadden (4 December 1919), The Times, 5 December 1919.
Hall v Hall [1962] 2 All ER 129, [1962] 1 WLR 478, revsd [1962] 3 All ER 518, [1962] 1 WLR 1246.
Harriman v Harriman [190810] All ER Rep 85, [1909] P 123, 78 LJP 62, 100 LT 557, 27 Digest (Repl) 363, 3005.
Horton v Horton [1940] 3 All ER 380, [1940] P 187, 109 LJP 108, 163 LT 314, 27 Digest (Repl) 307, 2543.
Jamieson v Jamieson [1952] 1 All ER 875, [1952] AC 525, 116 JP 226, 3rd Digest Supp.
Jolliffe v Jolliffe (1963), 15 January 107 Sol Jo 78.
Kaslefsky v Kaslefsky [1950] 2 All ER 398, [1951] P 38, 114 JP 404, 27 Digest (Repl) 296, 2413.
Kelly v Kelly (1870), LR 2 P & D 59, 39 LJP & M 28, 22 LT 308, 18 WR 767, 27 Digest (Repl) 298, 2434.
King v King [1952] 2 All ER 584, [1953] AC 124, 3rd Digest Supp.
Lang v Lang [1954] 3 All ER 571, [1955] AC 402, 119 JP 368, [1954] 3 WLR 762, 3rd Digest Supp.
Lauder v Lauder [1949] 1 All ER 76, [1949] P 277, 27 Digest (Repl) 306, 2537.
Russell v Russell [18959] All ER Rep 1, [1897] AC 395, 66 LJP 122, 77 LT 249, 61 JP 771, 27 Digest (Repl) 307, 2551.
Simpson v Simpson [1951] 1 All ER 955, [1951] P 320, 115 JP 286, 27 Digest (Repl) 2998 2447.
Squire v Squire [1948] 2 All ER 51, [1949] P 51, [1948] LJR 1345, 112 JP 319, 27 Digest (Repl) 296, 2415.
Usmar v Usmar [1949] P 1, [1948] LJR 1418, 27 Digest (Repl) 299, 2445.
Waters v Waters [1956] 1 All ER 432, [1956] P 344, 120 JP 105, [1956] 2 WLR 661, 3rd Digest Supp.
Westall v Westall (1949), 65 TLR 337, 27 Digest (Repl) 296, 2416.
Williams v Williams [1963] 2 All ER 994, [1964] AC 698.
Wright v Wright [1960] 1 All ER 678, [1960] P 85, [1960] 2 WLR 499, 3rd Digest Supp.

Appeal
The Ludlow justices had adjudged that the appellant husband was guilty of persistent cruelty to the respondent wife. The
principal facts on which the justices reached their decision were that the husband did not work, provided no money, had debts and
pressing creditors and left the wife to support the household, whereby her health deteriorated. On 5 April 1962, a Divisional
Court of the Probate, Divorce and Admiralty Division (Sir Jocelyn Simon, P., and Cairns J), reported [1962] 2 All ER 366, held
that the husbands conduct did not amount to cruelty. On 26 October 1962, the Court of Appeal (Willmer, Harman and Davies
LJJ), reported [1962] 3 All ER 897, by a majority (Harman LJ dissenting) reversed the decision of the Divisional Court. The
husband appealed.

J B Latey QC and C T Reeve for the appellant.


R J A Temple QC and Joseph Jackson for the respondent.

Their Lordships took time for consideration

27 June 1963. The following opinions were delivered.


967

LORD REID. My Lords, in 1961 the Shropshire justices made an order that the respondent be no longer bound to cohabit with
the appellant her husband. This order was made because the justices found that he had been guilty of persistent cruelty to her.
On appeal a Divisional Court (Sir Jocelyn Simon, P., and Cairns J) made an extensive and valuable analysis of the authorities and
held, chiefly on the authority of Kaslefsky v Kaslefsky and Eastland v Eastland that the facts proved did not amount to cruelty.
Their decision was reversed by the Court of Appeal (Willmer and Davies LJJ Harman LJ dissenting).
We have before us notes of evidence given before the Ludlow justices and a note of their reasons. Both are admirably
prepared, but they cannot be a full equivalent of a transcript of evidence and a judgment in the High Court. So it is even more
necessary for us to be cautious in attempting to substitute a different view of the facts from that taken by the justices. In this case
I think that I might have taken a different view of some of the facts, but I do not find any sufficient grounds for rejecting the view
of the justices: it may well be that, if I had seen the witnesses and heard the whole of the evidence, I would have agreed with
them. So I shall state briefly the facts as they appear to have presented themselves to the justices.
The parties were married in 1946. They were then aged forty-three and thirty-eight. There are two daughters born in 1947
in 1949. The husband owned a farm, but he was unsuccessful and ran into debt. He sold the farm in 1957 and bought a house at
Church Stretton. His wife had given or lent to him considerable sums and he transferred the house to her. The wife has carried
on the house as a guest house for elderly people. The husband did little or nothing to help her in running the house: if he had
chosen to do so, he could have obtained paid employment, but he did not do so. He spent a good deal of time and money in
trying to invent agricultural machinery, but the justices appear to have regarded this as a mere excuse or as a selfish indulgence.
He was incorrigibly and inexcusably lazy, and that has been at the root of the whole trouble. She was an active and capable
woman, and with little assistance she earned some 25 per week from the guest house, but this was little more than enough to
meet expenses. I accept the view of Harman LJ ([1962] 3 All ER at p 903), that all that he has done is to hang up his hat in the
hall. He was being constantly dunned by creditors. She was alleged to be liable for some of his debtsit is not clear why. But
certainly creditors were trying to make her pay and she did pay some of the debts.
There was never any suggestion that he was deliberately trying to hurt her or that there were any violent quarrels. She does
say that her husband was arrogant and bigoted and not a mild man and did not think of anyone else but himself; but the evidence
shows that what worried her and made her ill was his refusal to try to help her or to earn money and clear off his debts. She says:
Many times people have come to the door seeking my husband because of long standing debts and also the county court bailiff
has been to the door many years. This has worried me to death and I cannot stand any more of it. It would seem that she would
have been quite content if she could have put him out of the house and continued to maintain herself and her children without
being troubled by him and his creditors. She did refuse to cook for him or do anything for him, and he left for a time but came
back again. First she tried to get a non-cohabitation order without alleging cruelty, but then her health began to be affected and
she brought the present proceedings. It is true that he did obtain paid employment for a while, but the justices apparently did not
see that as a sign of genuine reform.
So we have a normal active and capable woman, against whom nothing is said, brought to a state of health when she can no
longer earn her living or maintain her children solely by the shiftless and selfish conduct of her husband. We 968 must take it
that the justices found that he was well aware that his conduct was reducing her to a physical and mental state where she would
no longer be able to maintain herself or their children, that if he had chosen to behave as any decent minded man would this
could easily have been avoided, but that in spite of his awareness of the consequences he chose to continue to sponge on her in
selfish idleness. The question is whether the law of England requires that these two shall continue to live together and would
regard her as guilty of the matrimonial offence of desertion if she left him. If that is the law then she must be told that it is her
legal duty as a wife to sink into poverty and ill health and become with her children a charge on the state; but does the law not
permit us to say that such conduct in such circumstances amounts to persistent cruelty?
No one has ever attempted to give a comprehensive definition of cruelty and I do not intend to try to do so. Much must
depend on the knowledge and intention of the respondent, on the nature of his (or her) conduct, and on the character and physical
or mental weaknesses of the spouses, and probably no general statement is equally applicable in all cases except the requirement
that the party seeking relief must show actual or probable injury to life, limb or health. So I shall first exclude the kinds of case
with which I am not dealing, with the caveat that much of what I am going to say may not apply to them. First I exclude cases
where the respondent acted with a desire or intention to hurt. That kind of case was dealt with in Jamieson v Jamieson, and I
need say no more than that if one spouse sets out to hurt the other and causes injury to health the means whereby that happens
can hardly matter. Then there are difficult cases where the conduct complained of which have caused no damage to a spouse
normal in health and temperament, but that does not arise here because this wife was a normal healthy woman; and there are
other cases where the offending spouse suffers from some mental abnormality. The next case which we have to decide, Williams
v Williams ([1963] 2 All ER 994), is such a case and I shall not anticipate what I have to say there. With such cases I associate
cases where the offending spouse is too stupid or obtuse to realise the effect which his conduct is having on his wife or her
husband. In the present case there is nothing in the husbands condition to palliate his conduct and he must be regarded as fully
responsible for it; and no question arises here of the guilty spouse having been provoked in any way.
This appears to me to be a plain uncomplicated case of a husband fully responsible for his conduct, knowing that it was
injuring his wifes health and yet persisting in it, not because he wished or intended to injure her but because he was so selfish
and lazy in his habits that he closed his mind to the consequences. The facts of this case appear to me to go well beyond the
ordinary wear and tear of married lifeadopting Lord Asquiths phrase in Buchler v Buchler ([1947] 1 All ER 319 at p 326;
[1947] P 25 at p 45). So the question must be whether the husbands conduct was of a kind which can in law be called cruel and
whether the law requires an intention to injure before there can be cruelty.
I shall not make any extensive examination of the authorities now. I intend to look at a good many in Williams v Williams
(See p 994, post), and I understand that others of your lordships will do so in this case. Sometimes a distinction is drawn between
conduct which is and conduct which is not cruel in itself. That distinction has some validity. Where for example there is physical
violence of a grave and weighty kind there is no need to look farther, as I shall try to show in dealing with Williams v Williams
(See p 994, post); but more often the conduct must take its colour from the state of mind which lay behind it. Sometimes it is
said that the matter can be left at large: that, although you cannot define cruelty, you can recognise it when you see it: and that,
therefore, the trial judge should not be hampered by legal niceties and refinements. But that appears to me to be placing too great
a burden 969 on the trial judge and to be likely to lead to a multitude of appeals: for without further guidance border-line cases,
which are numerous, will make a different impression on different minds, and moreover a judge must give reasonshe cannot
just say I think that these facts prove cruelty. Nevertheless I would try to reduce tests, rules and presumptions to a minimum.
A judge does and must try to read the minds of the parties in order to evaluate their conduct. In matrimonial cases we are not
concerned with the reasonable man, as we are in cases of negligence. We are dealing with this man and this woman and the
fewer a priori assumptions we make about them the better. In cruelty cases one can hardly ever even start with a presumption
that the parties are reasonable people, because it is hard to imagine any cruelty case ever arising if both the spouses think and
behave as reasonable people.
In the present case I think that the Divisional Court went wrong because they proceeded on the authority of Kaslefsky v
Kaslefsky. They are in no way to blame for that because they were bound to follow it. But I do not regard Kaslefskys case as a
good guide, so I must now examine it and its implications. As was inevitable in the existing state of the law, that case was argued
and decided in an atmosphere of phrases and presumptions more picturesque than of easy practical application. The trouble
began in a way which was not uncommon. The husband after the birth of his son went abroad for two and a half years on was
service. While he was away the wife told him she wanted her freedom. She and the child were living with her mother and he
returned to her there in April, 1946. She was lazy and sluttish and left much of the care of the child to her mother and then to her
husband. She refused sexual intercourse and there were some quarrels about the child; and after a couple of years he met another
woman and went off with her. Then he tried to get a divorce for cruelty. There was some evidence of injury to his health, but it
was not clear that that was really caused by his wifes conduct. There was nothing to show that she knew or must have known
that her conduct was injuring his health, or that the real cause of his leaving her was that he could not stand her conduct any
longer. There was some evidence of failure to treat the child properly, but nothing that I would regard as cruelty. On any view of
the law I would think that cruelty had not been proved.
The case was decided by the learned commissioner on the ground that there was a no evidence before me that those acts
were what are called intentional acts or aimed specifically at the husband. In the Court of Appeal the eminent counsel engaged
could not do other than take the familiar line, the husbands health was not injured merely by the natural development of the
wifes character but by her wilful and unjustifiable conduct towards him a spouse must be presumed to intend the natural and
probable consequences of his or her own acts it is not necessary to prove that the respondent intended to be cruel provided
that her intentional acts amounted in fact to cruelty and then they argued that the test of in some way aimed at, first introduced
in Westall v Westall, was obiter and inconsistent with Squire v Squire. Bucknill LJ with whom Somervell LJ agreed, said ([1950]
2 All ER at p 401; [1951] P at p 44):
________________________________________
a Referred to [1950] 2 All ER at p 399; [1951] P at p 41.

I venture to think that that is one of the tests which has to be applied: is the conduct of the wife unless done for the
express purpose of injuring the health of the husband innocent in the sense that it is justified or justifiable under the
circumstances. In such cases as refusal of sexual intercourse or sheer laziness or neglect of a child I think that is conduct
which is innocent so far as any charges of cruelty is concerned unless it is done for the express purpose of causing injury to
the health of the complaining spouse.
970
That appears to me to go much too far, but I need not develop the point because in the next paragraph he appears to me to have
watered it down by agreeing with what Denning LJ had said in Westall v Westall ((1949), 65 TLR at p 337):

intention is an element in this sense, that there must be conduct which is in some way (my italics) aimed by one person
at the other.

Denning LJ elaborated what he meant by aimed at. First it included action or words actually or physically directed at him
([1951] P at p 46; cf [1950] 2 All ER at p 402). I am not at all sure what this is meant to include. Suppose a husband speaks
derogatory words to a third person in his wifes presence. They are physically directed at her: they impinge on her ears. But in
Kaslefskys case it appears to have been accepted that cruelty to a child in the mothers sight and presence is not aimed at her
unless done with the express purpose of hurting her. And I cannot think that speaking derogatory words to a stranger can be
cruelty, if brutality to the child is not. So this test seems too vague to be satisfactory. Then Denning LJ deals with cases where
there is only misconduct indirectly affecting him or her such as drunkenness, gambling or crime ([1950] 2 All ER at p 402;
[1951] P at p 46). Then he says that there must be in some part an intention to injure the other or to inflict misery on him or
her. But he goes on to say ([1950] 2 All ER at p 402; [1951] P at p 46):

Such an intention may readily be inferred from the fact that it is the natural consequence of his conduct, especially
when the one spouse knows, or it has already been brought to his notice, what the consequences will be, and nevertheless
he does it, careless and indifferent whether it distresses the other spouse or not.

There are many cases where such an inference in justified, but there are many where it is not. The drunkard may know, at least in
his sober moments, what damage he is doing to his wife. The gambler sees his wife and children deprived of proper food and
clothing. But it must be a rare case where either desires or intends to hurt his wife.
Why should we have to drag in intention at all? It seems to me a very poor defence to say I know the disastrous effect on
my wife of what I have been doing. Probably I could have resisted temptation if I had really tried. But my conduct is innocent
because I had not the slightest desire or intention to harm my wife. I have acted throughout from pure selfishness. And the
evidence may make it quite clear that he had no intention at all of causing pain to his wife. If he knew, or the evidence shows that
he must have known, the effect of his conduct, if there was no justification or excuse for it, if the effect was really serious, if the
wife was not unusually sensitive and he had not reached the stage of mental disorder, why does intention matter? As I have said,
I am not dealing with cases where the wife from illness or temperament cannot stand what an ordinary woman could be expected
to tolerate, or where for any reason the husbands responsibility is diminished. I am dealing with cases like this case where
nothing of that sort can be said. If it can, the case becomes much more complicated. And I wish to make it quite clear that I am
not criticising Denning LJ. Sitting in the Court of Appeal he was bound by a host of authorities which do not bind us. Propbably
his attempt at rationalisation went as far as he was entitled to go, but I do not think that anyone would say that the present state of
the law is satisfactory. I shall try to explain why I think that that is so.

Aimed at is a phrase in ordinary use understood by everybody. If you aim at something you intend to hit it, and if
you hit something unintentionally you have not aimed at it. Intention is a state of mind. You cannot see into other peoples
minds, but ordinary people have little difficulty in inferring intention 971 from what a man does and says, viewed in light
of the circumstances. Juries deal daily with the burden of proof of intention, whether it has to be proved beyond reasonable
doubt or on a balance of probability. And a man can intend to kill two birds with one stone.

If it were the law that an intention to hurt is a necessary element in cruelty there would be no difficulty in theory. The only
difficulty would be to evaluate the evidence and decide whether the burden of proving such intention had been discharged. But
no one now supposes that that is the law. Everyone agrees that something short of proof of actual intention will do. This test is
that conduct must be in some way aimed at the petitioner. I must admit that I find that difficult. In real life either you are
aiming at something or you are not: in law either you have proved an intention or you have not. Aimed in some way, however,
takes us into a region where you assume or presume an intention which has not been proved, because it cannot be inferred as a
fact from the evidence. But you do not treat a presumed intention as equivalent to a proved intention. Jamiesons case shows that
a proved intention to hurt can make conduct cruel where a mere presumed intention would not. Presuming an intention which
has not been proved as a fact causes such confusion that I will, with your lordships permission, deal with this matter in some
detail.
Sometimes it is said that a person must be presumed to have intended the natural and probable result of what he did. That, if
taken literally, must mean that it would be irrelevant to prove that in fact he did not intend that result: it would introduce a purely
objective standard not depending at all on the state of his mind. In fact people often intend something quite different from what
they know to be the natural and probable result of what they are doing. To take a trivial example, if I say that I intend to reach the
green, people will believe me although we all know that the odds are ten to one against my succeeding; and no one but a lawyer
would say that I must be presumed to have intended to put my ball in the bunker, because that was the natural and probable result
of my shot. Irrebuttable presumptions have had a useful place in the law of tort in facilitating the change from a subjective to an
objective standard. For a long time it was thought that, at least in theory, intention or mental state of some kind was a necessary
ingredient in negligence. But life would be impossible in modern conditions unless on the highway and in the market place we
were entitled to rely on the other man behaving like a reasonable man. So we now apply a purely objective standard. The other
man may have been doing his best, and he may not realise that his best is not good enough, but if he causes damage by falling
short of the ordinary standard he must pay. In matrimonial affairs we are not dealing with objective standards, and it is not a
matrimonial offence to fall below the standard of the reasonable man (or the reasonable woman). We are dealing with this man
and this woman. There appears to be no recent authority which expressly approves an irrebuttable presumption in such cases, so
I pass to the next stage.
What is most often said is that there is a presumption in cruelty cases that the respondent intended the natural and probable
results of his conduct, but that that presumption can be rebutted. That is equivalent to saying that proof of the natural and
probable consequences transfers the onus of proof, so that now the respondent must prove that he did not intend that result. I
suppose that on this theory the respondent would have to produce evidence, including his own, sufficient to show that on a
balance of probability he did not intend harm to the petitioner, and of course it might be more difficult to prove that he did not
intend a likely result than to prove that he did not intend an unlikely result, but quite often he could prove it. A person accused
does not have to prove his defence beyond reasonable doubt. I think that Denning LJ must have had something of this kind in
mind when he said that such an intention ([1950] 2 All ER at p 402; [1951] P at p 46) may readily be 972 inferred. But in
many authorities the objective test seems to persist. In the present case Willmer LJ stated the propositions of law which he
deduced from the decided cases. He said ([1962] 3 All ER at p 901):

there must always be some element of intention in relation to the impact of the conduct complained of in the other
spouse such an intention may in a proper case be inferred where, for instance, the conduct complained of is persisted in
in circumstances in which any reasonably person would appreciate that it was likely to injure the other spouse. For any
spouse may be presumed to intend the natural consequences of his own behaviour.
That seems to me to be very vague. I do not blame Willmer LJ because he was trying to sum up the authorities and it is
impossible to reconcile a good deal of what has been said in the various decisions. But what is a proper case and when may
the presumption be applied? This is as vague as in some way aimed at. It is said that although this is not a definition it is a
useful yardstick. But it is a very elastic yardstick, and practical people do not use elastic yardsticks except perhaps for nefarious
purposes. All this seems to me merely to distract our attention from the real problem.
It appears to me that the time has come to decide whether or not intention really is a necessary element in cruelty. I shall
have more to say about that in Williams v Williams ([1963] 2 All ER 994; [1964] AC 698). If it is a necessary element then we
must go by the evidence. If the evidence shows, as it often does, where the respondent is obtuse and selfish, that it is extremely
improbably that he had any real intention to hurt his wife, then we must either refuse a remedy or say that something else will do
instead. Judges, bound by the existing state of the law, have shown much ingenuity where justice demands a remedy but where it
is extremely improbably that the respondent had any intention to cause harm. The objective test of what a reasonable man would
have had in mind slips inthough it is quite obvious that no reasonable man could have behaved as the respondent did. Once we
say that the state of this respondents mind does not matter, and proceed, against the weight of evidence, to attribute to him an
intention which he did not have, we base ourselves on a fiction and that is bound to lead to trouble. However useful fictions may
have been in the past in other branches of the law, they seem to me both unnecessary and confusing in this realm of cruelty. I
must pay this tribute to learned judges in recent years: in spite of the difficulties created by all these artificial tests there are very
few reported cases where they have not been circumvented when justice required that. Indeed the only decision cited to us
which, on a more direct approach, would seem to be wrong on the facts is Eastland v Eastland where the test in Kaslefsky v
Kaslefsky was applied too literally. With regard to other decisions a few may be doubtful, because sometimes the full facts do not
emerge very clearly. I must however say a little about Lang v Lang. It is not an easy case. There had to be an agreed single
judgment and such judgments are not infrequently obscure. The husband was undoubtedly cruel; but cruelty was not a ground of
divorce in Victoria, and it was necessary to establish constructive desertion. The Act b required that the respondent has without
just cause or excuse wilfully deserted the petitioner. So the question was whether the constructive desertion was wilful
whether there was animus deserendi. A great deal was said about intention and predeliberately ill-treated his wife. He knew that
this was likely to cause her to leave him, but he desired or hoped that she would not leave. He did not act with the intention of
driving her out, but he acted with the knowledge that that was what would probably happen. There are references to what a
reasonable 973 man would have known; but it is said that man must have known, which I take to mean that it was proper to hold
on the evidence that he did know. So in the result his desire to keep his wife or lack of intention to drive her out was irrelevant.
The Act said nothing about intention: it used the word wilfully. So the decision was that if without just cause or excuse you
persist in doing things which you know your wife will probably not tolerate, and which no ordinary woman would tolerate, and
then she leaves, you have wilfully deserted her, whatever your desire or intention may have been. That seems to me to be in line
with what I am now submitting to your lordships is the law in cases of cruelty.
________________________________________
b Marriage Act, 1928 (No 3726), of the State of Victoria, s 75(a).

I shall now re-state briefly the result at which I have arrived. If the conduct complained of and its consequences are so bad
that the petitioner must have a remedy, then it does not matter what was the state of the respondents mind. That I shall develop
in Williams v Williams ([1963] 2 All ER 994). In other cases the state of his mind is material and may be crucial.
Jamiesons case deals with deliberate intention to cause suffering and Kelly v Kelly shows that a groundless belief that such
conduct is justified is no defence. I do not have to deal with cases where the petitioner is partly at fault or cases like King v King.
Nor do I have to deal with the very difficult class of case where from illness or temperament the petitioner is unduly demanding
or unusually sensitive.
I am dealing with a spouse normal in mind and health who has been reduced to ill-health by inexcusable conduct of the other
spouse persisted in although he knew the damage which he was doing. Theses matters must be clearly proved. For reasons
which I have given I must hold that they have been proved in this case. They appear to me plainly to establish persistent cruelty.
I am therefore of opinion that this appeal should be dismissed.

LORD EVERSHED. My Lords, I concur with the opinion which my noble and learned friend Lord Reid has just expressed and
think accordingly that this appeal should be dismissed.
My lords, I am disposed to agree with the view of Davies LJ ([1962] 3 All ER at p 907), in the Court of Appeal that the
respondents (Mrs Gollins) claim might colloquially be described as a thin case. But the matter in issue being essentially a
question of fact, one considerable difficulty presented to the appellate courts and, now, to your lordships House emerges from the
fact that no full note was taken of the evidence given before the magistrates, nor did the magistrates express their conclusion in
the form of a reasoned judgment. There was available to the Divisional Court and to the Court of Appeal and there is now
available to your lordships only a brief record of the proceedings before the Shropshire court taken, in accordance with the
normal practice in such cases, by the clerk to the magistrates. Certain reasons were, however, clearly formulated for the
magistrates on 5 October 1961, and in the circumstances I have thought it proper and helpful to set out in terms the first five of
such reasons. They were as follows:

(1) That at the time of the marriage the husband was heavily in debt, unbeknown to the wife, and that throughout the
marriage the husband has not made any real or sustained effort to repair his financial situation; but on the contrary has
involved his wife in his financial affairs to her detriment;
(2) That since 1957 the husband, in spite of his wifes pleas for him to do so, has persistently and wilfully refused to
undertake paid employment, and that he has not made any real effort to support his wife and children;
(3) That the constant worry and anxiety over the husbands debts and his wilful refusal to get work has seriously
affected the wifes health, and that she has good reasons to fear permanent impairment of her health, 974which would
affect her capacity to keep herself and her two daughters;
(4) That in September, 1960, the wife told the husband that she could no longer stand the strain imposed by his
behaviour;
(5) That apart from the complaint of the wife, the husband should have known that his conduct was having a serious
adverse effect on his wifes health. We are satisfied that any reasonable man would have appreciated this, and that the
husband must have known that a continuance in his course of conduct would have an injurious effect on his wifes mental
health.

In their sixth reason (which I can properly summarise) the magistrates stated that they rejected entirely the appellants (Mr
Gollins) submission that what he said he had done in looking after the garden and the poultry at the matrimonial home at
Rivenhall was a fair and proper contribution on his part. They conclude by the statement of their view that it was the clear duty
of the appellant to make a real contribution to the support of his family and that his failure to do so was the direct cause of his
wifes ill-health.
Having regard to the terms of their recorded reasons as well as to the record of the evidence which on two occasions the
magistrates heard (and I do not forget that the magistrates had the great advantage of seeing and hearing on both occasions both
the appellant and the respondent) I agree with my noble friend Lord Reid and with the majority of the Court of Appeal in thinking
that no sufficient ground has been shown for disabling the conclusions of fact reached by the magistrates or for holding that the
magistrates in any respect misdirected themselves in regard to the law.
The facts of the case which have been rendered available to your lordships have been fully stated in the opinion of Lord
Reid and I do not repeat them. It is not in doubt, according to the express conclusions of the magistrates, and has indeed not been
disputed, that as a result of the conduct of her husband to which she has been subjected, the health of the respondent has been
substantially affected so that the essential requirement for the establishment of a charge of cruelty, according to the decision of
your lordships House in the case of Russell v Russell, has been satisfied. It also clearly appears that the respondent was and is a
woman of (at least) normal health and intelligence. She was before her marriage a trained nurse and, having regard to her age
(namely 39 and 41) at the time when she respectively bore the two children of her marriage, her physical stamina must have been
certainly no less than average. She was also fortunately possessed of some private means, though it is clear that her resources
have now been wholly involved or expended in relief of her husbands indebtedness or its consequences. She discovered shortly
after her marriage that her husband, then a farmer, was in fact in grave financial difficulties. His farm was sold and the
respondent has since maintained the home not only for her husband but also for herself and her two children of the marriage by
carrying on, without any help whatever financial or otherwise from her husband, a guest house for elderly people. In so doing
she has been from time to time gravely embarrassed by the claims of the creditors of her husband who has never made the
smallest contribution to her home or to the maintenance and up-bringing of the two children; and it is now indeed apparent that, if
she is denied the relief which she has claimed, she will be incapable of further maintenance of the home for herself and her
children and, as my noble friend has stated, will sink into poverty and ill-health and become with her children a charge on the
state. It may, therefore, fairly be said (if these conclusions of fact are justified) in the words of Sir William Scott in the old case
of Evans v Evans ((1790), 1 Hag Con 35 at p 37) that her husbands conduct is such as to show an absolute impossibility that the
duties of the married life can be discharged; or at least in Lord Normands language ([1952] 1 All ER at p 878; [1952] AC at p
535) that the 975 appellant (Mr Gollins) was guilty of unwarranted indifference to the sufferings of his wife.
It was however the view of the Divisional Court, as expressed by Sir Jocelyn Simon P ([1962] 2 All ER 366 at p 376), that
there was no evidence showing that the husbands conduct, however selfish and irresponsible, was aimed at his wife in any
sense and that, therefore, on the authority particularly of the case of Kaslefsky v Kaslefsky the respondents charge of cruelty
could not be sustained.
My Lords, like my noble friend Lord Reid, I must not be taken to be casting doubt on the correctness in fact of the decision
of the Court of Appeal in Kaslefsky v Kaslefsky but (also like my noble friend) I am with all respect to those who take or have
expressed the opposite opinion unable to accept the view that in order to satisfy a charge of cruelty in a matrimonial cause it must
be shown that the conduct of the spouse charged was in some real sense aimed at the other party to the marriage. Such a view
has, as I understand, emerged from the premise that cruelty in matrimonial contests involves essentially a quality of malignity
in some sense on the part of the spouse charged. On this premise appears to have depended, naturally enough, that there must
have been some intention to injure the other party of the spouse charged. On this premise appears to have depended, naturally
enough, that there must have been some intention to injure the other party, in other words that the conduct of the party charged
(whether positive or active on the one hand or, on the other, indifferent, selfish or negative) must in some sense have been aimed
at the other spouse. My lords, for reasons I more fully give in my opinion in the case of Williams v Williams ([1963] 1 All ER
994), I am unable to accept the premise that cruelty in matrimonial proceedings requires or involves of necessity the element of
malignitythough I do not of course doubt that if malignity be in fact established it would be highly relevant to a charge of
cruelty. In my opinion, however, the question whether one party to a marriage has been guilty of cruelty to the other or has
treated the other with cruelty does not, according to the ordinary sense of the language used by Parliament, involve the presence
of malignity (or its equivalent); and if this view be right it follows, as I venture to think, that the presence of intention to injure on
the part of the spouse charged or (which is, as I think, the same thing) proof that the conduct of the party charged was aimed at
the other spouse is not an essential requisite for cruelty. The question in all such cases is, to my mind, whether the acts or
conduct of the party charged were cruel according to the ordinary sense of that word, rather than whether the party charged was
himself or herself a cruel man or woman. Indeed, for reasons which I shall more full state in my opinion in Williams v Williams
([1963] 1 All ER 994) the introduction of the notion of intention into the jurisdiction with which your lordships are now
concerned has led to no little confusion and difficulty which, as I hope, may now be laid to rest. The case where one party to a
marriage sets out by deliberate means to injure the other is no doubt simple; but equally it is, I should think, relatively rare. More
frequent (and in its effects more often hurtful and insidious) is such a case as the present where the intention of the party
charged, if it exists at all, is no more and no less than an intention to gratify his or her purely selfish inclinations. So it is that,
as it seems to me, the courts in adopting the necessity of intention have, inevitably in the cause of justice, had to introduce the
notion of presumed intention and thereby to have introduced into divorce law quasi-philosophical difficulties which have clouded
the law and given to the word cruelty a sense which is not justified by its ordinary significance.
For the reasons which I have attempted to state I therefore conclude with my noble and learned friend Lord Reid, and as the
majority of the court of Appeal thought, that no good reason has been established for disabling the conclusion of the magistrates.
I think in other words that if the reasons formulated by the magistrates were justified, then applying the common sense of the 976
English language and the ordinary standards of human conduct the respondent made good her charge of cruel treatment, and I
think accordingly that this appeal should be dismissed.

LORD MORRIS OF BORTH-Y-GEST. My Lords, though a question whether one person has been guilty of persistent cruelty
to another must ultimately be determined as an issue of fact, it is eminently desirable that the approach to the determination
should be guided by principles of general application. This is not to say that some precise definition of cruelty should, for the
purposes of judicial determinations, be attempted to be given. It has long been recognised that it would be unwise to make any
such attempt (see Jamieson v Jamieson). There are, however, certain conceptions which are basic. Most people have the same
general ideas in mind when they speak of cruelty or of kindness. If someone is giving thought to a question whether a person has
been cruel, he will probably first consider what it is that the person has done or has omitted to do, and will then decide, perhaps
largely as a matter of opinion, whether the act or omission does or does not fall within his conception of cruelty. In the
administration of the law the matter cannot be left to be dealt with on quite such broad lines. Cruelty is a matrimonial offence
and an allegation of cruelty is a serious one. To be found guilty of cruelty involves not only a slur which would be deeply
wounding to any self-respecting person but also involves certain specific consequences. The matters complained of as amounting
to cruelty must therefore extend much beyond the trivial or the casual. They must be serious matters. If they were not they
surely could not be the foundation for an order affecting the matrimonial relationship. When in 1790 Sir William Scott (in Evans
v Evans) said that ((1790), 1 Hag Con at p 37) the causes must be grave and weighty, he laid down a principle which has never
been challenged. Even the particular words that he used in expressing the principle have been employed in subsequent periods
with a measure of tenacious fidelity. The conduct complained of must therefore go beyond what the particular complaining
spouse ought reasonably in all the circumstances to have been expected to tolerate. I use these words because, as Willmer LJ
points out in his judgment ([1962] 3 All ER at p 901) in the present case, it is necessary to consider what was the impact of the
personality and conduct of one spouse on the mind of the complaining party (see Usmar v Usmar and King v King) and also
because, as Pearce J said in Lauder v Lauder ([1949] 1 All ER 76 at p 90; [1949] P 277 at p 308), in a cruelty case the question
is whether this conduct by this man to this woman, or vice versa, is cruelty.
Not only must the matters complained of be serious, but they must also be such as to have caused injury to health or a
reasonable apprehension of such injury (Russell v Russlee).
To these requirements I would add that the acts or conduct or omissions complained of must have been intentional and not
accidental. Some occurrence accidentally brought about by one spouse may cause hurt and misery to the other, but it would not
be rational to convict the former of cruelty.
The range of the reported decisions in cruelty cases, so carefully and usefully reviewed, if I may respectfully say so, in the
judgment of the learned President, ([1962] 2 All ER 366 at pp 368 et seq) reveals that difficulties often arise in those cases where
no positive wish to hurt is apparent and in those cases where conduct causes hurt indirectly. In the difficult spheres covered by
such cases there has naturally been a desire to seek a principle and in this search dependence has often been placed on particular
phrases employed by learned judges in expressing their conclusions in particular cases. In the result it has sometimes happened
that a phrase or an aphorism coined in one 977 case has become doctrine in the next. My lords, I would favour as much
simplicity as is reasonably possible and as much freedom as is reasonably possible from any limiting words or phrases. As
Harman LJ ([1962] 3 All ER at p 903), pointed out in his judgment the only question which in the end arises for decision isDid
the conduct of this husband amount to cruelty to this wife?
Some conduct may be shown to be conduct which is directed to or towards, or which is in relation to, or which is in respect,
of, the other spouse. I am intending my words to be mere servants to express a meaning. Other and different words would do. If
such conduct is wilful and intentional, then I would consider it fair that it should be judged by relation to those standards to which
a reasonable spouse in all the particular circumstances of the particular marriage ought to have conformed. If such conduct is
wilful and intentional then there either would be or there should have been knowledge of any injurious effects resulting from it.
Some cases would be covered by the oft-quoted words used in 1919 by Shearman J in Hadden v Hadden (The Times 5
December 1919): I do not question that he had no intention of being cruel but his intentional acts amounted to cruelty.
I agree therefore with Willmer LJ when he said ([1962] 3 All ER at p 901):

Thus any course of conduct intentionally pursued, provided it has some impact on the other spouse, may in
appropriate circumstances justify a finding of cruelty.

There may however be conduct which does not appear to have been directed to or towards or to have been conduct in relation to
or in respect of the other spouse. In such cases it is sometimes said that the impact of the conduct is not direct. If such conduct
was wilful and intentional (and satisfied the conditions to which I earlier referred) cruelty could be established if there had been
an intention to injure the other spouse or if with knowledge of its effect it was persisted in or continued. A continuance of some
particular conduct after receiving a warning that it was adversely affecting the other spouse could amount to cruelty. Further the
facts and circumstances might be such that any reasonable person must have known that injury to the other spouse would be the
result of the conduct: if so the wilful pursuit of the conduct would be inference involve an intention to cause the injury. The
process of drawing an inference does not involve imputing an intention that did not in fact exist, but involves deducing from
proper material that an intention did exist. If the conduct itself is wilful and intentional and if either there is knowledge that it
will probably harm the petitioner or if, applying the standards of a reasonable person in the circumstances of the particular case,
there must have been such knowledge, then, a charge of cruelty could be maintained. I say could because in the end it must be
shown as a question of fact that the conduct is such that it can properly and rationally be stigmatised by the word cruelty using
that word in its ordinary acceptation.
My Lords, the divergences of view which are revealed in the judgments in the Divisional Court and in the Court of Appeal
seem to me to arise mainly in the application of the law to the facts. The facts are very clearly set out in the judgment of the
learned President ([1962] 2 All ER at pp 368, 369) and I need not refer to them in complete detail. My study of them leads me to
the same conclusion as was reached by the learned President ([1962] 2 All ER at p 376) and Cairns J ([1962] 2 All ER at p 376)
in the Divisional Court and by Harman LJ ([1962] 3 All ER at p 904), in the Court of Appeal. From the time of their marriage in
1946 until the year 1957 the parties lived at the farm. Though the husband farmed unsuccessfully and was considerably in debt
and though the wife was caused much distress by her discovery in 1947 of the grave financial situation then existing, there is no
suggestion that the husband at any time offered any sort of physical harm to her or that he was in any way aggressively unkind to
her to that 978 he was in any intemperate. Nor does the evidence show that at any time he had any wish to harm her. That has
not been suggested. Furthermore the wife was not, in fact, at any time short of money. The farm was sold in 1957. The property
was then purchased which thereafter the wife ran as a guest house. The house was purchsed in his name and there was a
mortgage on it. The mortgage had been partly redeemed by January, 1961, and in 1960 the house was handed over to the wife.
Undoubtedly she had a hard and strenuous and worrying time. She had been involved by her husband in his debts and she had
continuously to work to support herself and the children and to pay mortgage sums. He made no financial contribution. His help
in regard to the running of the guest houses does not appear to have been at all adequate. His activities in seeking to make
inventions of agricultural machinery did not produce any money for the home. In the enterprise which kept the home going he
may have been little more than a passenger. It was the wife who carried the burdens.
It is easy to understand that a hard pressed and worried wife would become exasperated. In September, 1960, a brother of
the husband had a fire at his farm in Buckinghamshire and as a result the husband went there for a time in order to help. It was
while he was there that the wife wrote to him and told him that she could not stand the strain of his debts any longer and she said:
I told him it grieved me to have to take this step but if he did not get work and clear himself of debt after the end of the year, I
should take the steps I have. I asked him to stay away and get work because I could not stand the strain. The husband did not
do as she wished and towards the end of the year he returned home. The wifes first summons then followed. That was in
December, 1960. She alleged wilful neglect by him to provide reasonable maintenance. Cruelty was not alleged. In her
evidence at the hearing on 5 January 1961, the wife made no complaint at all save in regard to money matters. There was no
suggestion or hint at that time that her health was in any way affected. She said The only time I have any bother is over
finance. She did complain of the fact that although her husband in the period after September, 1960, had worked as a farm
labourer (earning from 6 to 8 per week) at his brothers farm in Buckinghamshire he had not sent any money to her.
In his evidence the husband said that the guest house was a joint venture and that he had kept the garden in order and had
done small outside repairs and painting. He considered that the work he had done had saved much expenditure which otherwise
would have been incurred and that he had improved the value of the property. He further said that during the previous four years
he had applied for jobs but had only succeeded in obtaining casual employment for very short periods. The magistrates told the
husband that he ought to find work and that he ought to maintain his wife and children and that he had been guilty of wilful
neglect to maintain them. Their order (requiring him to pay her 3 a week and 1 for each of the two children) was made on 5
January 1961. For some weeks after that date the husband was away at his sisters house and then he did obtain work. He
became employed as a gardener-handyman at a wage of 8 15s a week. He started to pay 15s a week to his wife and afterwards,
apart from one payment of 2, he paid her 1 per week. He used the balance partly to keep himself and partly (to the extent he
said of 4 per week) to pay off his previously incurred debts. In April, 1961, the wife consulted her doctor. In June, 1961, she
ceased to perform regular household duties for him or to have any association with him. In August, 1961, she took out a
summons for the variation of the January order. She asked for the insertion of a non-cohabitation clause on the ground that her
husband had treated her with persistent cruelty. It seems remarkable that she should have complained of cruelty at that stage. In
the earlier period she had her husbands presence and no money from him: at this period she was having nothing to do with him
and he was making a payment (though a very small one) to her. It is true that he was not fully complying with the January order
and was not paying the stated amounts, but his case 979 was that because of his financial position he could not do so and could
not pay more than he was doing. In September, 1961, he took out a cross-summons to revoke the order of 5 January 1961, on the
ground that his wife was in desertion or alternatively on the ground that he was no longer guilty of wilful neglect to maintain for
the reason that he was not (and never had been) in a position to pay the amount ordered. The husband in his evidence in October
said that he had no reason to think that his wife was ill. He did not see the doctor come to the house. His wife had given him the
impression that she did not want him. It was the view of the learned President that the object of the wife in taking out her
summons in August, 1961, for variation of the previous order was to get her husband out of the house. When the summons and
cross-summons came before the justices in October, 1961, they inserted a non-cohabitation clause. They held that since 1957 he
had persistently and wilfully refused to undertake paid employment, that worry over this and the debts had seriously affected the
wifes health, that in September, 1960, she had told him that she could no longer stand the strain, that he should have known that
his conduct was having a serious adverse effect on her health and that he must have known that a continuance in his course of
conduct would have an injurious effect on his wifes mental health. Though the order of the justices in October contains no
express finding of cruelty it must, I think, be taken that their reason for acceding to the application for a non-cohabitation order
was that they considered that the complaint of persistent cruelty made in August, 1961, was made out. At the same time they
decided in favour of the husband to the extent that they reduced the wifes maintenance from 3 a week to 1 a week and they
deleted the requirement to pay 1 a week for each child and furthermore they remitted the arrears.
They therefore impliedly held that the husband was paying as much as in all the circumstances he could pay. Previously
they had told him to get work. Following on that he had obtained work. He had made such payments as he could. At the earlier
date there was no allegation of cruelty. There seems to have been even less justification for it in August than there was in
January.
My lords, the husband may have been ineffectual in money matters and he may well be criticised and blamed for his failures
in the period prior to January, 1961. He was then found guilty of wilful neglect to provide reasonable maintenance for his wife
and his children. Thereafter the events were as I have indicated. Unless a finding of wilful neglect to maintain must in every
case involve a finding of cruelty (which I cannot accept) the question that arises is as the learned President stated it ([1962] 2 All
ER at p 376):

whether the husbands conduct, however reprehensible, can properly be stigmatised by the word cruelty in its
ordinary acceptation.

I do not think that it can and accordingly I would allow the appeal.

LORD HODSON. My Lords, the question raised by this appeal is whether the appellant, George Gollins, was rightly convicted
of cruelty towards his wife Mabel Gollins. I put the matter in this way although there is no conviction recorded against him in
the somewhat strange manner in which this case has proceeded. What has happened is this. The wife who had become
dissatisfied with the husbands failure to make any substantial contribution to the joint home which was successfully maintained
by her efforts sued the husband for wilful neglect to maintain her and her children, making no allegation of cruetly against him,
and obtained an order which he was unable to comply with except to a very limited extent.
The parties were at this time living under the same roof and the wife, unwilling that this should continue, for the first time
raised against him a charge of cruelty on which the justices must have found the wifes allegation proved for they made a
separation order thereby enabling the wife to expel him from the house which, 980while originally his, had been transferred to
her in circumstances which will appear. There was no material change in the situation from the time the first order was made on
5 January 1961 and 5 October 1961, when the separation order was made, the wifes complaint being, as it always had been, that
she was not getting enough money and was worried by her husbands debts and the consequences of those debts for she also had
to suffer the calls and demands of creditors. Indeed it appears that after the first order the husband paid more than he had
previously been able to do because he was, during part of the time, in employment and able to keep himself, which the wife was
no longer willing to do, make payments in reduction of his debts and some payment to his wife under the order. On the second
hearing the wife, having obtained her objective namely, separation, did not resist the husbands application to vary the order
which was reduced from 3 a week to 1 a week and 1 a week for each child. The husband satisfied the justices that he was
unable to pay the arrears which were accordingly remitted. Smarting under the implied finding of cruelty, the husband appealed
to the Divisional Court which allowed the appeal on the overriding ground that the court did not consider that the husbands
conduct, however reprehensible, could properly be stigmatised by the word cruelty in its ordinary acceptation ([1962] 2 All ER
366; see particularly at p 376). The wife appealed to the Court of Appeal and Willmer LJ ([1962] 3 All ER at p 903), with whom
Davies LJ ([1962] 3 All ER at p 907), agreed in substance, held that there was abundant material to justify the finding of cruelty
and that the Divisional Court were therefore in error in substituting a finding of their own. Harman LJ ([1962] 3 All ER at p
903), in a dissenting judgment, expressly agreed with the overriding ground of the Divisional Courts decision to which I have
referred, concluding by saying ([1962] 3 All ER at p 904): Whatever opprobrous names one may give to this husband I do not
think that cruel is one of them. There is no dispute as to the facts and the only question for your lordshps consideration is
whether the Divisional Court was right in reversing the finding of cruelty implicit in the justices second order or whether the
Court of Appeal majority was right in holding the Divisional Court to be wrong in disturbing the order of the justices.
Under the summary jurisdiction Acts of 1895 and of 1960 c, an appeal lies from any order made by a court of summary
jurisdiction under these Acts to the Probate, Divorce and Admiralty Division of the High Court, and rules may from time to time
be made regulating the practice and procedure in such appeals. Rule 73 of the Matrimonial Causes Rules, 1957, provides for
these appeals to be made to the Divisional Court and directs that the court may draw all inferences of fact which might have been
drawn in the magistrates court and may give any judgment and make any order which ought to have been made. Ever since the
passing of the first of these Acts in 1895 the practice has been that the Divisional Court will not reverse any finding of fact by the
magistrates court, unless it appears that such finding is clearly wrong. This practice is manifestly sound, for decisions of justices
are nearly akin to decisions of juries and they do not and cannot be expected to give elaborate judgments showing the process by
which they have reached their decision on fact. Moreover, in so far as these questions depend on the appreciation of the
personalities involved it is well recognised that any appellate court will be reluctant to interfere with decisions reached by those
who have not only heard the evidence but had an opportunity of summing up the characters of the protagonists in the dispute.
________________________________________
c Viz. the Summary Jurisdiciton (Married Women) Act, 1985, and the Matrimonial Proceedings (Magistrates Courts) Act, 1960, which
repealed the earlier legislation.

The justices are, however, in practice required to give reasons for their decision, and these appear to have been carefully
drawn so as to show the conclusion which they reached and the relevant facts which led them to convict the husband of cruelty.
They are as follows:
981

(1) That at the time of the marriage the husband was heavily in debt, unbeknown to the wife, and that throughout the
marriage the husband has not made any real or sustained effort to repair his financial situation; but on the contrary has
involved his wife in his financial affairs to her detriment;
(2) That since 1957 the husband, in spite of his wifes pleas for him to do so, has persistently and wilfully refused to
undertake paid employment, and that he has not made any real effort to support his wife and children;
(3) That the constant worry and anxiety over the husbands debts and his wilful refusal to get work has seriously
affected the wifes health, and that she has good reasons to fear permanent impairment of her health, which would affect
her capacity to keep herself and her two daughters;
(4) That in September, 1960, the wife told the husband that she could no longer stand the strain imposed by his
behaviour;
(5) That apart from the complaint of the wife, the husband should have known that his conduct was having a serious
adverse effect on his wifes health. We are satisfied that any reasonable man would have appreciated this, and that the
husband must have known that a continuance in his course of conduct would have an injurious effect on his wifes mental
health;
(6) We reject entirely the husbands contention that by looking after the garden and poultry at Rivenhall he was
making a sufficient contribution towards maintaining his wife and children, thus leaving him free to devote virtually his
whole time to developing his invention. We consider that a husband in his position in life should as a matter of course
cultivate the garden and do the heavier domestic tasks, such as carrying coal, and that no husband in his position is entitled
to expect his wife to maintain him in return for such normal duties. We also consider that on the facts of this case it was the
clear duty of the husband to obtain work, to maintain himself, and to contribute to the support of his wife and children, and
that his failure to do so was the direct cause of his wifes ill health.

I have already stated that the facts are not in dispute but it is desirable to refer to some of the evidence in order to show what
the wifes case was. She was married to the husband in 1946, she being then thiry-eight and he forty-three years of age. They
had two daughters, born in 1947 and 1949. At the time of the marriage they lived in a farm belonging to the husband and shortly
before the birth of their elder daughter the husband tried to borrow money from the wife to pay for drainage and improvements.
She then learned that the farm was heavily mortgaged, that he owed other debts and could not support a wife and child. The
husband was never free from debt although the wife lent him money to pay off pressing creditors, including 200 in the year
1951 which has not been repaid.
In the year 1957 the farm was sold and a further property Rivenhall was bought in the name of the husband for 3,000 and
subject to a mortgage of 2,250. In 1960 the husband transferred Rivenhall into the wifes name and it so remains, subject to
the mortgage. The wife thereafter ran the house as a guest house for elderly people. This produced an income for her of 1,300 a
year, out of which she said she paid everything and had done so ever since they moved into the place. The husband looked after
the garden and the poultry but his main interest was inventing, in connexion principally with agricultural machinery, which did
not prove remunerative. The wifes complaint was not that she was short of money but that her husband, so far from providing
anything to keep down the mortgage and provide for his family, involved her in his debts so that she was embarrassed by the
demands of creditors and the calls of bailiffs which no doubt would come to the knowledge of the guests.
The wife made no complaint at either hearing of any aggressive conduct on the part of the husband. She stated at the first
hearing that the only time 982 she had any bother with him was over finance and from this statement she never departed. At the
second hearing she said I do not claim that I am short of money. I have to work to pay off the mortgage and keep myself. One
expects the husband to pay. The husband was the one to complain of hostile conduct, saying that he thought that the wife
intended to make life unpleasant for him, and he gave examples of her conduct. One instance of her conduct emerges from the
wifes own evidence, for she complained that the husband was asserting his authority towards his children and making himself a
nuisance. She admitted that she took them out on one occasion to stop him taking them to Nantwich. Latterly she refused to
provide him with food and only cooked his meals at weekends, and he did his own washing and ironing (apart from bed linen).
In this unhappy state of affairs from April, 1961, the wife complained of headaches, agitation and unexplained fits of weeping,
sleeplessness and short periods of amnesia. Her doctor formed the impression that she was suffering from a moderately severe
anxiety state precipitated by her financial and marital difficulties. He felt that if her domestic circumstances did not improve her
psychological condition might well become worse in spite of treatment.
Although the justices nowhere in their reasons use the word cruelty, nor do they find that the complaint that the husband
has been guilty of persistent cruelty was made out, yet they must have formed the opinion that cruelty was proved, for otherwise
it is not to be supposed that they would have made an order the effect of which was to bar the husband from his wifes society.
Although the statute does enable justices to pronounce a separation even on the finding of wilful neglect to maintain, it is not the
practice so to do, for the reasons set out in the judgment of the Court of Appeal in Harriman v Harriman, see the judgment of
Kennedy LJ ([190810] All ER Rep at p 100; [1909] P at p 151):

In my judgment the history of this provision, the language of the context, the very nature of the protection which the
provision gives, and the enactment in (a), d, that it shall have the effect of a decree of judicial separation on the ground of
cruelty unite in showing that the discretion given by Parliament to the court of summary jurisdiction as to the framing of
orders under s. 4 and s. 5, e, is not rightly exercised, if this non-cohabitation provision is inserted in the order made for the
wifes relief, except where the court has been satisfied by evidence, in the words of the Act of 1878, f, that the future safety
of the wife is in peril. I do not mean, in saying this, that the non-cohabitation provision ought to be inserted in the order
where the wife has proved the convictions for assault or the persistent cruelty expressly specified in s. 4 of the Act of 1895.
It is possible that a wilful neglect to maintain may be proved to have been accompanied by occasional acts of dangerous
violence, and so justify the inclusion in the order of the non-cohabitation provision. But I am clearly of opinion that it is
neither in accordance with the intention of the legislature, as appearing in this statute, nor in the interest of the wife or of
public morals, that the provision should be included in an order which is sought for and obtained solely on the ground of
the husbands desertion.
________________________________________
d Ie, s 5(a) of the Summary Jurisdiction (Married Women) Act, 1895.
e Ie, s 4 and s 5 of the Act of 1895.
f Ie, the Matrimonial Causes Act, 1878, s 4.

True that since these words were used Parliament has re-enacted g the Act of 1895 in substantially the same terms, but the
practice laid down in Harrimans case has been consistently followed, cf the judgment of the President in Jolliffe v Jolliffe (107
Sol Jo 78. See also Vaughan v Vaughan, [1963] 2 All ER 742) given on 15 January 1963, after his attention had been drawn 983
to the observations of two of the members of the Court of Appeal on this topic in the present case. Your lordships are therefore in
my opinion bound to deal with this appeal on the footing that the husband has been found guilty of persistent cruelty. This
finding appears to me a finding which is wholly wrong and ought not to be allowed to stand.
________________________________________
g Section 4 and s 5 of the Act of 1895 are re-enacted by the Matrimonial Proceedings (Magistrates Courts) Act, 1960, s 1, s 2.

I have already indicated, I hope sufficiently, that findings of justices in matters of this kind ought not be disturbed where
there is evidence to support them. But I can find no trace of any conduct alleged by the wife against her husband which can be
properly called cruel. It is not I think profitable to use other adjectives by way of explanation, substitution or definition. The
word cruel is enough, is readily understood without more and there is no need to speak of unreasonable, reprehensible or
inexcusable conduct.
True that the courts have taken a stand in that conduct which readily merits the adjective cruel is not to be the basis of relief
unless injury to health or a reasonable apprehension of the same is established. This was established by Russell v Russel, when
by a majority your lordships house decided that the disgraceful conduct of Lady Russell towards her husband, while meriting the
description cruel, did not measure up to the standard set by the ecclesiastical courts in that no physical harm was found to have
been inflicted or was reasonably to be apprehended. Although the members of your lordships House who decided that case were
divided in their judgment, none doubted the validity of the opinion of Sir William Scott in Evans v Evans when he said ((1790), 1
Hag Con at p 37):

This however must be understood, that it is the duty of courts, and consequently the inclination of courts, to keep the
rule extremely strict. The causes must be grave and weighty, and such as show an absolute impossibility that the duties of
the married life can be discharged. In a state of personal danger no duties can be discharged; for the duty of self-
preservation must take place before the duties of marriage, which are secondary both in commencement and in obligation;
but what falls short of this is with great caution to be admitted.

The converse of the decision in Russell v Russell namely, that once injury to health could be attributed to matrimonial discord
then cruelty was proved, has never so far as I know been advanced in this country until this case. Counsel for the wife was at
pains to emphasise that the catalogue of cruelty, like the catalogue of negligence, is never closed. This is true. He relied on the
observations of my noble and learned friend Lord Tucker to that effect in Jamieson v Jamieson ([1952] 1 All ER at p 887; [1952]
AC at p 550) in relation to cases where no physical violence is averred. In such cases, of which Jamiesons case was a not a
typical example, the conduct averred in the pleadings which alone fell for consideration was not on the face of it cruel but the
case was sent for trial because of the averment of deliberate intention and persistence in callous conduct by the offending spouse
although aware of the effect on his wifes health. But for the averment of intent the judgment of the lower house, holding that the
pleaded case would not sustain a charge of cruelty, would have been upheld.
Where physical violence or threats of violence are involved no difficulty arises and the court does not stay to inquire with
what intent (in the absence of insanity) acts are committed. By 1870 it has been accepted that what has been called moral force
combined with injury to health is sufficient without any act of violence, see the decision of Lord Penzance in Kelly v Kelly
((1870), LR 2 P&D 59 at p 76) which has never been disapproved. It is in cases of this kind, where no violence or threat of
violence is involved, that the question of intent becomes important and is sometimes crucial, as it was in Jamiesons case. There
is no indication that 984 this husband ever had the smallest intention of doing any harm to anyone, least of all his wife. In what
way then was he cruel? Was it by omission, for cruelty may be manifested by ommission as readily as by commission. To
neglect a suffering spouse or one who has by misfortune fallen into danger is cruel, one would think, whether or not the other
spouse has had anything to do with bringing about the suffering or the danger. To speak of unwarrantable indifference is only to
illustrate that the cruel man is merciless and the question must always be: was the conduct, be it positive or negative, cruel
according to the meaning of that word as understood by ordinary people? If it be said that it is difficult to draw any line between
what is or is not cruel, I agree, but I see no reason for attempting to establish boundaries on one side of which cruelty lies, on the
other of which it does not. No one has hitherto attempted so to do and at the end of it each person has to make up his own mind
after considering each case as a whole, not by asking whether conduct is cruel and then asking whether it can in any way be
justified. I respectfully adopt the language of my noble and learned friend Lord Reid in King v King when he said, using words
which seem to me to have general application ([1952] 2 All ER at p 593; [1953] AC at p 140):

The question whether the respondent treated her husband with cruelty is a single question only to be answered after all
the facts have been taken into account.

In that case your lordships were invited to consider some recent cases in which the courts had sought to give guidance in those
cases where cruelty was alleged and the acts were not on the face of them necessarily cruel vis--vis the complaining spouse.
The first of these was Horton v Horton, a decision of Bucknill J. The learned judge said ([1940] 3 All ER at p 384; [1940] P at p
193):

Mere conduct which causes injury to health is not enough. A man takes the woman for his wife for better, for worse.
If he marries a wife whose character develops in such a way as to make it impossible for him to live happily with her, I do
not think he establishes cruelty merely because he finds life with her is impossible. He must prove that she has committed
wilful and unjustifiable acts inflicting pain and misery upon him and causing him injury to health.

As Lord Normand pointed out this test may well have been adequate for what fell to be decided in Hortons case but it is not an
adequate test in all cases. It cannot be regarded as a test in cases which come under the head of unwarrantable indifference for
example.
The same comment seems to me to be appropriate in considering the case of Kaslefsky v Kaslefsky. I agree with the
decision, but I find it difficult to see how the aiming test there put forward assists in the conclusion, although it is a valuable
guide in many cases. The majority of the court seems, however, to have taken the view that, although cruelty was not proved, yet
the husband was justified in refusing to live with his wife. That, on the facts of the case (which was quite different from the
Russell case), I also find difficult to understand. The judgment of Denning LJ ([1950] 2 All ER at pp 402, 403; [1951] P at pp
4548), who was the third member of the court, has, as I have indicated, been a very useful guide in considering many border-
line cases, repeating as he did observations of his own in an earlier case of Westall v Westall. Aiming is an aspect of intention.
Moreover I note that your lordships had the opportunity of considering Kaslefskys case not only in 985 Kings case but also in
Jamiesons case. I do not see how differences of judicial opinion are to be avoided in this area. Kings case is an illustration.
Your lordships were divided three to two in favour of the finding of no cruelty, the Court of Appeal was of the same opinion,
being divided two to one, and the learned and experienced trial judge was of the opposite opinion.
I have formed myself a clear opinion that in this case the conduct of the husband cannot justly be called cruel, and I have
looked in vain to see what he did or left undone which qualifies him to be convicted of cruelty to his wife. The case against him
was simple enough. It was that to all intents and purposes he was a man who did not provide for his wifes maintenance and
further got into debt and allowed his wife to suffer with him the inconveniences which followed. It was said, but this was not
proved, that he put her name on a document relating to a debt of his in circumstances amounting to forgery, but I find no positive
act proved against him. Willmer LJ ([1962] 3 All ER at p 903), was of opinion that this failure to provide was enough, but this
ignores the realities of the situation. The husband was not a man of means and the wife, relatively speaking, was a woman of
means. She never attempted to make a case of physical suffering brought about by any failure to provide the necessaries of life.
For a man to fail to support his wife is no doubt a matter which can be dealt with by appropriate proceedings, though I have not
myself previously heard of a case where an impecunious husband was sued by a pecunious wife. In any event, in my opinion no
cruelty was involved and I would allow the appeal.
LORD PEARCE. My lords, in all matrimonial disputes, whether in the magistrates courts or in the divorce court, the word
cruelty has the same connotations. It has a long history behind it. By the Matrimonial Causes Act, 1857, h the divorce court
took over the jurisdiction of the ecclesiastical courts with the added statutory power to grant decrees of dissolution of marriage.
By s 22 it was bound in all suits other than those for dissolution to act and give relief on principles and rules which shall in the
opinion of the said court be as nearly as may be conformable to the principles and rules on which the ecclesiastical courts have
heretofore given relief (see now Supreme Court of Judicature (Consolidation) Act, 1925, s 32 and s 103). The former relief by
decree a mensa et thoro, of which cruelty had been a ground, became relief by a decree for judicial separation (s 7); and a decree
for judicial separation might be obtained either by the husband or the wife on the ground of adultery or cruelty or desertion
without cause for two years and upwards (s 16). The Matrimonial Causes Act, 1937, first empowered the court to grant a decree
of dissolution on the ground that the respondent has since the celebration of the marriage treated the petitioner with cruelty.
From the days of Lord Stowell down to the present it has been acknowledged that to support a finding of cruelty the matter must
be grave and weighty; and in Russell v Russell, this House finally settled that conduct must, in order to constitute cruelty in the
legal acceptance of the term, be such as to cause danger to life, limb, or health, bodily or mental, or to give rise to a reasonable
apprehension of such danger. (See Lord Merriman in Jamieson v Jamieson ([1952] 1 All ER at p 883; [1952] AC at p 544).)
Thus there have long been two safeguards against any extension of relief to cases founded on mere trivialities and
incompatibility.
________________________________________
h 20 & 21 Vict c 85. The commencement of the Act was 1 January 1858

The earlier cases of cruelty dealt in the main with acts of physical violence, and were little concerned with motives or
intentions. In Dysart v Dysart Dr Lushington said ((1844), 1 Rob Eccl at p 116):
986

It is for me to consider the conduct itself and its probable consequences; the motives and causes cannot hold the hand
of the court unless the wife be to blame which is a wholly different consideration.

But it appears clearly from the cases that the courts objective consideration took account of the particular individuals concerned
and the particular circumstances of each case. In Kelly v Kelly, which Lord Merriman described (in Jamieson v Jamieson ([1952]
1 All ER at p 881; [1952] AC at p 540)) as the leading case in England on the subject of cruelty without physical violence, the
court considered a course of tyrannous conduct intended to dominate the wife and bend her to the husbands will. The full court
there found cruelty proved and Lord Penzance expressly stated ((1870), LR 2 P & D at p 72) that the husband says that he has no
desire to injure her and it has never been asserted that he does. In various cases thereafter it was shown that intention to injure
or to be cruel was not a necessary ingredient of cruelty. The word of Shearman J in Hadden v Hadden (The Times, 5
December 1919) express the matter succinctly: I do not question he had no intention of being cruel, but his intentional acts
amounted to cruelty. The same line of thought was followed by the Court of Appeal in the case of Squire v Squire (approved by
Lord Merriman in Jamieson v Jamieson ([1952] 1 All ER at p 881; [1952] AC at 541)). Tucker LJ there said ([1948] 2 All ER 51
at p 53; [1949] P 51 at p 56):

It is to be observed that in the well-known and much quoted case of Russell v. Russell it is not suggested that motive is
a necessary element in cruelty.

In Jamieson v Jamieson Lord Normand having quoted the Lord Presidents observation in the court below ([1952] 1 All ER
at pp 877, 878; [1952] AC at p 535), that

In cases of mental cruelty the guilty spouse must wither intend to hurt the victim or at least be unwarrantably
indifferent as to the consequences to the victim

continued ([1952] 1 All ER at p 878; [1952] AC at p 535):

I do not propose to go into that because I wish to avoid the discussion of hypothetical cases and because I am of
opinion that actual intention to hurt may have in a doubtful case a decisive importance and that such an intention has been
averred here. Actual intention to hurt is a circumstance of peculiar importance because conduct which is intended to hurt
strikes with a sharper edge than conduct which is the consequence of mere obtuseness or indifference.

Those words are inconsistent with the view that intention was an essential element. So too Lord Merrimans opinion. He quoted
with approval ([1952] 1 All ER at p 881; [1952] AC at pp 540, 541) the words of Lord Penzance in Kelly v Kelly ((1870), LR 2 P
& D at p 72), and those of Shearman J in Hadden v Hadden (The Times, 5 December 1919), set out above. My noble and
learned friend Lord Reid said ([1952] 1 All ER at p 886; [1952] AC at p 549):

I do not doubt that there are many cases where cruelty can be established without it being necessary to be satisfied by
evidence that the defender had such an intention, but I do not intend to decide anything about such cases.

Lord Tucker said ([1952] 1 All ER at p 887; [1952] AC at p 550):


987

Every such act must be judged in relation to its attendant circumstances, and the physical or mental condition or
susceptibilities of the innocent spouse, the intention of the offending spouse, and the offenders knowledge of the actual or
probable effect of his conduct on the others health are all matters which may be decisive in determining on which side
of the line a particular act or course of conduct lies.

Thus in all the opinions there appears the view that intention, though it may be a deciding factor in some doubtful cases, is not
essential to cruelty.
There is nothing in King v King which is inconsistent with that view. My noble and learned friend Lord Reid there said
([1952] 2 All ER at p 595; [1953] AC at p 145):

It has long been recognised that a malevolent intention while not essential is a most important element where it
exists.

The case concerned a nagging wife whose husbands conduct was thought by three of their lordships to have given the wife
sufficient provocation and excuse for her behaviour. Two of their lordships thought otherwise. Thus it was a borderline case
which turned on the extent to which the husbands conduct provided an excuse to the wife in the particular facts of the case. It
would be wrong to read words intended to refer to the particular problem before the House as intended to be of universal
application, particularly when the authors of them disclaimed such an intention. My noble and learned friend Lord Reid (who
was one of the majority) having described the case as ([1952] 2 All ER at p 596; [1953] AC at p 146)) a narrow and difficult
case said ([1952] 2 All ER at p 596; [1953] AC at p 146):

I do not intend to try to define cruelty. I doubt whether any definition would apply equally well to cases where there
has been physical violence and to cases of nagging, or to cases where there has been a deliberate intention to hurt and to
cases where temperament and unfortunate circumstances have caused much of the trouble. But in cases like the present,
the wifes conduct must at least be inexcusable after taking everything into consideration.

Lord Normand said ([1952] 2 All ER at p 586; [1953] AC at p 129.):

I have to doubt that the test whether the conduct was wilful and unjustifiable was an adequate test for what remained
to be decided in Hortons case. What is open to question is whether it can be taken to be an adequate test in all cases of
cruelty by nagging accusations. I think it is not always an adequate test, and that BUCKNILL, J., did not put it forward as
a universal and exhaustive test in this type of case If the trial judge, in the exercise of his discretion, comes to the
conclusion that the conduct of the respondent is, notwithstanding the provocation received or the stresses endured, really an
inexcusable offence against the other spouse, his judgment should be respected and treated as conclusive.

In the present case no question of provocation or of bad behaviour by the petitioner arises.
Those words were not, I think, intended to limit, so far as cases of nagging and accusations and the like are concerned, the
generality of any rule that intention is not an essential element in cruelty. An act may be found inexcusable although the actor
had no harmful intention. It has always been clear that an intention to hurt invests any act or conduct with a greater significance.
This axiom is, however, merely a piece of jury wisdom which a sensible tribunal bears in mind when performing its task of
assessing whether a particular act or course of conduct was cruel. It is wrong to exalt it into a criterion of legal principle which
will decide in all cases where there is no physical violence whether 988 conduct can be cruel. It was natural that, with the
increasing incidence of cases where no physical violence was involved, the question of intention should take on an added
importance. For whereas a blow speaks for itself, insults, humiliations, meannesses, impositions, deprivations, and the like may
need the interpretion of underlying intention for an assessment of their fullest significance. And that interpretation may in some
cases turn the scale.
The introduction of divorce on the ground of cruelty by the Act of 1937 appears to have given rise to fears that divorce
might be granted for mere trivialities or incompatibility in spite of the existing safeguards of gravity and injury to health. Instead
of following the simple and direct approach of Dr Lushington ((1844), 1 Rob Eccl at p 116), namely, considering the conduct
itself and its probable consequences the court has in some cases pursued a devious and confusing course. In Kaslefsky v
Kaslefsky it was laid victim. Thus an element of intention was made a necessary ingredient in cruelty.
The germ of the doctrine that conduct is not cruel unless it is aimed at the other party may, I think, be found in Horton v
Horton. There Bucknill J in granting a decree on the ground of cruelty, distinguished between conduct which was caused merely
by ([1940] 3 All ER at p 384; [1940] P at p 193.) the development and manifestation of the wifes character acting so to speak in
its own sphere, and wilful and unjustifiable conduct to him which was an intrusion upon and did violence to his own mode of
living. The former in his judgment was not cruelty, but the latter was.
Later in Westall v Westall, Denning LJ said obiter ((1949), 65 TLR at p 337), that intention is an element in this sense that
there must be conduct which is in some way aimed by one person at the other. Kaslefskys case adopted that dictum, and laid
down that to secure a divorce on the ground of cruelty there must be shown on the part of the respondent conduct which is
aimed at, that is to say actions or words actually or physically directed at, the petitioner or done with intention to injure him or
to inflict misery on him.
The decision on the facts was not of importance. The cumulative effect of the wifes acts appear from the report to have
been somewhere on the border line of cruelty, but the evidence of injury to health was unsatisfactory. Without hearing the
evidence it would be difficult to say whether a decree should or should not be granted. But the general approach laid down has
been treated as binding and has therefore affected later decisions. So far as it laid down that unjustifiable conduct therefore
affected later decisions. So far as it laid down that unjustifiable conduct to the children could not be cruelty unless it was done
for the purpose of wounding the other spouse, its effect has been satisfactorily mitigated by the Court of Appeal in Wright v
Wright and in other respects the reasoning in Waters v Waters has avoided some of the difficulties caused by it.
In Kaslefskys case the court apparently considered that the use of the word treated with cruelty in the Act of 1937 had
imparted a slightly different approach to cruelty (Bucknill LJ ([1950] 2 All ER at pp 401, 402; [1951] P at p 45)). This, as it
seems to me, is erroneous, since I regard the phrase treated with cruelty as being no more than a convenient description of a
situation where there has been cruel treatment of which the respondent was the author. In an article on this subject by the Editor
of the Law Quarterly Review there is the apt observation:

If instead of using the word cruelty Parliament had seen fit to use the word stupidity, which in fact is one of the
major causes of divorces, must it be said that the words treats his wife with stupidity means either that the man is stupid
or that he intends his acts to be stupid. 989I have sympathy with the courts desire in Kaslefaskys case to supply some
mesh that would separate the grain from the chaff: but the introduction of the necessity for a petitioner to prove that
conduct was aimed at him or her has created confusion and difficulty.

Can the words aimed at include acts done without intention but with knowledge (and also perhaps some regret) that they
will strike the other? Can it include acts done with an unreasonable lack of appreciation of the fact that they will strike the other?
Can it include acts done with callous indifference as to whether the acts strike the other or not? In my view the test is not a happy
one from a practical point of view and it can only be made to work if it is patched by presumptions. On such a doctrine the court
is at once faced with the difficulty that much cruelty is purely selfish and is not aimed at the victim nor prompted by any intention
or desire to injure. Not infrequently acts which any reasonable person would regard as cruel acts, or which any reasonable person
would have known to be hurtful and to be injuring the health of the victim, are done by a respondent who is so bigoted, or obtuse,
or insensitive, or self-centred that he or she did not in fact realise that these acts were cruel or injurious or intend that they should
be. If the court insists on proof of intention it must then either allow to the bigot, the obtuse, the insensitive, the self-centred a
free hand to be as cruel as seems reasonable to their bigotry, stupidity, insensitiveness, or selfishness, or else it must make
disingenuous presumptions in order to pay lip service to its insistence on intention.
A court may, as a piece of prima facie reasoning, presume that a person intends the probable consequences of his acts. But if
that presumption is rebuttable and the court insists on proof of intention, then in many cases of cruelty it cannot honestly give
relief against the bigot, the obtuse, the insensitive, the self-centred. To avoid absurdity it may sayI think he must have
realised and must have intended to hurt,well knowing that in truth this particular man did not.
My noble and learned friend Lord Reid, in Jamieson v Jamieson ([1952] 1 All ER at p 887; [1952] AC at p 549.) reserved
his opinion

whether or to what extent it is necessary or proper to impute an intention which did not exist by invoking a legal
presumption that everyone must be supposed to intend or foresee the natural and probable consequences of his acts.
Lord Merriman P always made clear that such a presumption is rebuttable (eg Simpson v Simpson ([1951] 1 All ER 955 at p
962; [1951] P 320 at p 333); Waters v Waters ([1956] 1 All ER 432 at p 440; [1956] P 344 at p 360) and Jamieson v Jamieson
([1952] 1 All ER at p 881; [1952] AC at p 541)). See also Lang v Langi). But some of the expressions in other cases have
seemed to treat the presumption as irrebuttable, which is tantamount to saying that whatever the true intentions of the actor he
must be taken to intend the probable consequence of his act. If an irrebuttable presumption is allowed (and I see no justification
for it save only as an escape of last resort from the absurdities into which insistence on proof of intention can lead) one reaches
by a devious route the same objective test which Dr Lushington applied ((1844), 1 Rob Eccl at p 116.) when he considered the
conduct itself and its probable consequences.
________________________________________
i Per Lord Porter, [1954] 3 All ER 571 at p 579; [1955] AC 402 at p 428.

One may take for example the case of a husband whose wife has become seriously ill abroad and who leaves her timorous,
unable to speak the language, without adequate nursing arrangements or currency, simply because he must return to watch a
football match, which he feels that he cannot miss. In such a case he would probably express and feel some regret at her plight
and feel sorry that he cannot spare her any money from his own 990 comforts and cannot make better arrangements, but he hopes
optimitically that it will all work out somehow. I doubt if any court would say that that was not cruelty; and yet, in order so to
hold, it would, if intention or aiming be a necessary ingredient, have to deem an intention or aim which it knows to be untrue. To
say that unwarrantable indifference will suffice as intention or aim is no satisfactory answer to the problem, since indifference is
the antithesis of intention or aim. In such a case the test of aiming at is clearly inappropriate. If one endows the wife in the
case I have envisaged with a robust and independent nature, a knowledge of the language, a less severe illness, a larger supply of
currency and more adequate nursing arrangements and gives to the husband a more cogent reason for returning to England, the
element of cruelty departs: yet it may return, if he acted with the deliberate intention of hurting her. If the husband was to obtuse
to realise how unkindly he was acting, that may be some excuse, but not (in those circumstances) any adequate excuse such as
would prevent the conduct being cruel. If he appreciated, but was indifferent to her suffering, the cruelty was worse. If he
intended to hurt, it was worst of all. The question whether he aimed it at her is decisive only of those acts of which one can
predicate that they are cruel when intentional but not cruel when unintentional, and it is not possible to form categories of acts
which come exactly within that dubious class. For so often, as in the case envisaged, cruelty is a question of degree.
In my opinion, therefore, the court in Kaslefskys case erred in saying that aiming at the petitioner was an essential
ingredient in cases of cruelty. So, too, in Eastland v Eastland which followed and applied the reasoning in Kaslefskys case.
I do not find the test of the behaviour which the supouse bargains to endure for better, for worse (see Buchler v Buchler
(Per Asquith LJ [1947] 1 All ER at p 326; [1947] P at p 45) and Horton v Horton ([1940] 3 All ER at p 384; [1940] P at p 193)
helpful, since it confuses rather than clarifies. Much of the felicitous and often quoted language of Asquith LJ in Buchler v
Buchler has value and in particular his notion of the reasonable wear and tear of married life, which is a useful reminder of the
fact that in many marriages there are many complaints that could be put forward by either party, but that only grave and weighty
matters causing injury or apprehended injury to health will suffice to support a charge of cruelty or constructive desertion. The
words for better, for worse, however, carry religious associations knit up with the indissolubility of marriage and are of little
help in estimating whether a spouse is legally entitled to the relief provided by Parliament; see the observations of Danckwerts LJ
in Hall v Hall ([1940] 3 All ER at p 384; [1940] P at p 193). It may be that those words influenced the actual decision since, like
Danckwerts LJ ([1962] 3 All ER at p 525), I find it hard to understand how the court on the reported facts came to overrule the
decision of the judge of first instance. Nor do I think that the attempt in Hortons case to exclude from cruelty acts caused merely
by the natural developmenmt of a spouses character within its own sphere in helpful. Allowances must always be made for
temperament, and mere temperamental disharmony simpliciter is not cruelty. But if a temperament which naturally tends to
unkindness or selfishness or callousness develops to a point at which its acts are cruel, whether intentionally or not, it cannot be
right to say that the other spouse must endure it without relief. Nor can one helpfully say that development of character is within
its own sphere if its emanations affect and cause injury to the other spouse. Marriage by its nature causes one party to be affected
by most of the reprehensible conduct on the part of the other, and usually it is obvious that it will be so.
991
When the two parties and the evidence are before the court it is easier to form a view than to pronounce in the abstract. That
view, though it makes allowance for the subjective emotions, temperaments and excuses of the individuals, judges the conduct by
its probable consequences, and thus decides the question of fact and degree whether this conduct between these spouses was
cruel. It is impossible to give a comprehensive definition of cruelty, but when reprehensive conduct or departure from the normal
standards of conjugal kindness causes injury to health or an apprehension of it, it is, I think, cruelty if a reasonable person, after
taking due account of the temperament and all the other particular circumstances would consider that the conduct complained of
is such that this spouse should not be called on to endure it. The judgments of the Court of Appeal in Hall v Hall propounded a
similar test in a case of constructive desertion, save that in such a case there need be no injury to health. A relevant question is
whether the conduct is excusable (per Lord Reid in King v King ([1952] 2 All ER at p 593 AC at p 140) and Willimer LJ in
Usmar v Usmar ([1949] P at p 9): forgivable in the circumstances), for there are two sides to be considered in a case of cruelty
from the petitioners side ought this petitioner to be called on to endure the conduct, from the respondents side was this
conduct excusable?
I agree with Lord Merriman whose practice in cases of mental cruelty was always to make up his mind first whether there
was injury or apprehended injury to health. In the light of that vital fact the court has then to decide whether the sum total of the
reprehensible conduct was cruel. That depends on whether the cumulative conduct was sufficiently weighty to say that from a
reasonable persons point of view, after a consideration of any excuse which this respondent might have in the circumstances, the
conduct is such that this petitioner ought not to be called on to endure it. What on paper may seem little more than a series of
pinpricks said Lord Normand in Jamieeson v Jamieson ([1952] 1 All ER at p 878; [1952] AC at p 536), may present a very
different aspect when it has been developed in evidence. The quality of every piece of conduct is affected by its context and by
that which precedes and follows it. And it is to be remembered that, as my noble and learned friend Lord Reid pointed out in
King v King ([1952] 2 All ER at p 593; [1953] AC at p 140.):

It is not right first to ask whether the respondents conduct was cruel in fact and then to ask whether it can in any way
be justified. The question whether the respondent treated her husband with cruelty is a single question only to be answered
after all the facts have been taken into account.

And ([1952] 2 All ER at p 592; [1953] AC at p 138):

In a case of this character the wifes conduct must be judged in the light of the whole history of the marriage.

It was suggested by counsel for the appellant that we should evolve a formula which would exclude such conduct as that of
the appellant from cruelty unless there was an intention to hurt, and that then having found no such intention we should acquit
him of cruelty. But cruelty is a question of fact and degree, and no legal formula can resolve its peculiar problems. It would no
doubt simplify decisions in accident cases if the law evolved a legal principle that all driving over forth mph is negligent and that
no driving under the speed could be the question of fact. So too with cruelty cases which depend on an even wider variety of
matters than negligence cases. The particular circumstances of the home, the temperaments and emotions of both the parties and
their status and their way of life, their past relationship and almost every circumstance that attends the act or conduct complained
of may all be relevant. I agree with the words of Lord Tucker when he said in Jamieson v Jamieson ([1952] 1 All ER at p 887;
[1952] AC at p 550.):
992
It is in my view equally undesirableif not impossibleby judicial pronouncement to create certain categories of
acts or conduct as having or lacking the nature or quality which render them capable of amounting to cruelty in cases where
no physical violence is averred.

Since the personalities of the parties are an important element in a decision whether conduct whether those two persons has been
cruel, it follows that a court which relies on notes and has not seen the witnesses must use great caution. There are, of course,
cases where it can say that whatever the personalities of the parties of the manner of giving their evidence, it can be asserted that
the conduct was or was not cruel. But often that which the justices saw and heard may properly have turned the scale. In such a
case reversal of their decision is to create, not remedy, injustice. Was there material here on which the justices with the advantage
of hearing and seeing the witnesses could properly reach their conclusion?
Counsel for the appellant seeks to put before us the image of a man who was a good husband in all save his incapacity to
work or to provide money for the home. It is more probable that the justices formed the view that he was not merely lazy but
callous and selfish. Early in the marriage he borrowed her saving and he involved her in his financial affairs to her detriment.
She was a competent, stable person and she managed by hard work in the guest house to keep the family from want. Since 1957
the husband, in spite of his wifes pleas for him to do so, has persistently and wilfully refused to under take paid employment.
He has subjected her to constant worry and anxiety over his debts. Creditors and bailiffs have come to her guest house and
embarrassed her courageous efforts to make a living for the family. He has insisted on remaining on the premises, when under
the circumstances the wife naturally wanted him to depart. Apparently he has actually gone to the length of signing her name to a
promissory note in order to obtain money for his own purposesa course which was obviously likely to cause her great anxiety.
In September, 1960, he was warned that his wife could no longer stand the strain imposed by his behaviour. Moreover, any
reasonable man, as the justices found, would have appreciated that his conduct was having a serious adverse effect on his wifes
health. In the result a capable, well-balanced woman who has endured much stress for several years has been reduced to
headaches, agitation and unexplained fits of weeping, sleeplessness and short episodes of amnesia. The doctor describes her
condition as a moderately severe anxiety state precipitated by her financial and marital difficulties If her domestic
circumstances do not improve her psychological condition may well become worse in spite of treatment. The husband is,
according to the wife, arrogant and bigoted. Whether that description is justified we do not know, but the justices had an
opportunity of getting some impression of his personality. There is created by the evidence and the reasons the picture of a man
who has been, over the years, parasitical, selfish and callous to the physical undoing of a healthy woman working hard to support
herself and her children.
I agree with Willmer ([1962] 3 All ER at p 903) and Davies LJJ ([1962] 3 All ER at p 907), that the conclusion of the
justices was a conclusion which they could properly reach on the evidence and on the reasons given. I also agree with the
reasoning of the majority in the Court of Appeal, save in so far as it was necessarily affected by Kaslefskys case.
I would therefore dismiss the appeal.

Appeal dismissed.

Solicitors: Henry Lee, Bygott & Eccleston, Wem, Salop (for the appellant); J S Winny & Co Craven Arms, Salop (for the
respondent).

C G Leonard Esq Barrister.


993

[1963] 2 All ER 994

Williams v Williams
FAMILY; Divorce: HEALTH; Mental health

HOUSE OF LORDS
LORD REID, LORD EVERSHED, LORD MORRIS OF BORTH-Y-GEST, LORD HODSON AND LORD PEARCE
11, 12, 13, 14, 18, 19 FEBRUARY, 27 JUNE 1963

Divorce Cruelty Mental disorder Respondent of unsound mind Respondent appreciated nature and quality of acts but did
not know that they were wrong McNaghten rules inapplicable as constituting the criterion of defence Mental disorder a
factor in determining whether respondent treated petitioner with cruelty Matrimonial Causes Act 1950(14 Geo 5 c 25), s 1(1).

Insanity of a respondent spouse (in terms of the McNaghten rules, proof that he or she, owing to disease of the mind, was
unaware of the nature and quality of his or her acts, or, if aware of that, did not know the acts to be wrong) is not necessarily a
defence to a suit for divorce on the ground that the respondent treated the petitioning spouse with cruelty (see p 1002, letter h, p
1009, letters c and i p 1028, letter g, and p 1015, letter e; cf, p 1015, letter h, post); but insanity is a factor to be taken into account
in applying the test (which, per Lord Evershed, nevertheless remains an objective test (see p 1009, letter d, post)) whether in all
the circumstances of the case the respondents conduct is of such gravity that he has by his acts treated the petitioner with cruelty
(see p 1004, letters d to g, p 1009, letter d, and p 1029, letter e; cf p 1010, letter i, p 1015, letter d, and p 1016, letter g, post), eg
(per Lord Pearce), if the conduct were such that it would amount to cruelty only if aggravated by intention to hurt, a spouse who
could not form such an intention would not be held to have treated the other with cruelty (see p 1029, letter e, post).
Lissack v Lissack ([1950] 2 All ER 233) approved.
Astle v Astle ([1939] 3 All ER 967); Swan v Swan ([1953] 2 All ER 854); Palmer v Palmer ([1954] 3 All ER 494), and Breen
v Breen (1961 SC 158) not followed.
Observations of Sir J P Wilde in Hall v Hall ((1864), 3 Sw & Tr at p 349); and White v White ([1949] 2 All ER 339),
considered.
A husband by his conduct caused damage to his wifes health. His conduct took the form of deliberately accusing his wife
of adulterous associations, but he thought that such accusations were justified because he believed that they were true, having
been so induced to think by imaginary voices which informed him of them as fact. The wife petitioned for a divorce on the
ground of cruelty. The trial judge found that the husband knew what he was doing in making the accusations, but that he did not
know that they were wrong; the trial judge would have granted a decree, but felt constrained to dismiss the petition because of the
husbands insanity. On appeal from a decision of the Court of Appeal that the second limb of the McNaghten rules afforded the
husband a defence,

Held Lord Hodson dissenting): the fact that the husband did not know that his acts were wrong did not of itself constitute a
defence to a suit for divorce on the ground of cruelty, and accordingly (Lord Morris of Borth-Y-Gest dissenting on the facts of
this case) the appeal should be allowed.
Decision of the Court of Appeal ([1962] 3 All ER 441) reversed.

Notes
As to the law before the present decision concerning insanity as a defence to a charge of cruelty in a divorce petition, see 12
Halsburys Laws (3rd Edn) 273, para 521; and for cases on the subject, see 27 Digest (Repl) 309311, 25742590.
For the Matrimonial Causes Act, 1950, s 1(1), see 29 Halsburys Statutes (2nd Edn) 389, 390.
994

Cases referred to in opinions


Astle v Astle [1939] 3 All ER 967, [1939] P 415, 109 LJP 6, 55 TLR 1045, 27 Digest (Repl) 310, 2584.
Beals v Hayward [1960] NZLR 131, 3rd Digest Supp.
Breen v Breen [1961] SC 158.
Curtis v Curtis (1858), 1 Sw & Tr 192, 27 LJP & M 73, 164 ER 688, affd (1859), 4 Sw & Tr 234, 27 Digest (Repl) 543, 4908.
Dobbie v Dobbie 1953 SC 379, 3rd Digest Supp.
Donaghy v Brennan (1901), 19 NZLR 289, 33 Digest (Repl) 603, * 51.
Dysart v Dysart (1847), 1 Rob Eccl 470, 5 Notes of Cases 194, 163 ER 1105, overruling (1844), 1 Rob Eccl 106, 27 Digest
(Repl) 298, 2426.
Gollins v Gollins [1963] 2 All ER 966, [1964] AC 644.
Hadden v Hadden (1919), The Times, 5 December.
Hall v Hall (1864), 3 Sw & Tr 347, 33 LJPM & A 65, 164 ER 1309, 27 Digest (Repl) 374, 3084.
Hanbury v Hanbury (1892), 8 TLR 559, affg [1892] P 222, 61 LJP 115, 27 Digest (Repl) 374, 3086.
Hayward v Hayward (1858), 1 Sw & Tr 81, 31 LTOS 24, 6 WR 638, 164 ER 638, 27 Digest (Repl) 374, 3082.
Holden v Holden (1810), 1 Hag Con 453, 161 ER 614, 27 Digest (Repl) 300, 2449.
Horton v Horton [1940] 3 All ER 380, [1940] P 187, 109 LJP 108, 163 LT 314, 27 Digest (Repl) 307, 2543.
Imperial Loan Co v Stone [1892] 1 QB 599, 61 LJQB 449, 66 LT 556, 33 Digest 130, 57.
Inglis v Inglis 1931 SC 547.
Jamieson v Jamieson [1952] 2 All ER 875, [1952] AC 525, 116 JP 226, 3rd Digest Supp.
King v King [1952] 2 All ER 584, [1953] AC 124, 3rd Digest Supp.
Kirkman v Kirkman (1807), 1 Hag con 409, 161 ER 598, 27 Digest (Repl) 302, 2465.
Lilley v Lilley [1959] 3 All ER 283, [1960] P 158, 169, [1959] 3 WLR 306, 123 JP 525, 3rd Digest Supp.
Lissack v Lissack [1950] 2 All ER 233, [1951] P 1, 114 JP 393, 27 Digest (Repl) 311, 2587.
MKenzie v MKenzie 1960, SC 322, 3rd Digest Supp.
MLachlan v MLachlan, 1945 SC 382, 27 Digest (Repl) 301, 1093.
McNaghtens Case (1843), 10 Cl & Fin 200, 8 ER 718, sub nom McNaughtons Case, 4 State Tr NS 847, 1 Car & Kir 130n, 14
Digest (Repl) 60, 246.
Marsh v Marsh (1858), 1 Sw & Tr 312, 28 LJP & M 13, 32 LTOS 197, 164 ER 744, 27 Digest (Repl) 303, 2488.
Morriss v Marsden [1952] 1 All ER 925, 33 Digest (Repl) 602, 193.
Palmer v Palmer [1954] 3 All ER 494, [1955] P 4, [1954] 3 WLR 756, 3rd Digest Supp.
R v Windle [1952] 2 All ER 1, [1952] 2 QB 826, 116 JP 365, 36 Cr App Rep 85, 14 Digest (Repl) 61, 253.
Russell v Russell [18959] All ER Rep 1, [1897] AC 395, 66 LJP 122, 77 LT 249, 13 TLR 516, 27 Digest (Repl) 308, 2551.
Sofaer v Sofaer [1960] 3 All ER 468, [1960] 1 WLR 1173, 3rd Digest Supp.
Stapleton v R [1952] ALR 929, 86 CLR 358, 26 ALJ 453, 8 Digest (Repl) 674, 92.
Steuart v Steuart (1870), 8 Macph (Ct of Sess) 821, 42 Sc Jur 480, 27 Digest (Repl) 556, 2007.
Squire v Squire [1948] 2 All ER 51, [1949] P 51, [1948] LJR 1345, 112 JP 319, 27 Digest (Repl) 296, 2415.
995
Swan v Swan [1953] 2 All ER 854, [1953] P 258, 117 JP 519, [1953] 3 WLR 591, 3rd Digest Supp.
Thomas (Watt or Thomas) v Thomas [1947] 1 All ER 582, [1947] AC 484, 1947 SC (HL) 45, [1947] LJR 515, 176 LT 498, 2nd
Digest Supp.
Timmins v Timmins [1953] 2 All ER 187, [1953] 1 WLR 757, 3rd Digest Supp.
Weaver v Ward (1616), Hob 134, 14 Digest (Repl) 54, 187.
White v Pile (1951), 68 WN (NSW) 176.
White v White (1859), Sea & Sm 77, 1 Sw & Tr 591, 1 LT 197, 164 ER 874, 27 Digest (Repl) 307, 2546.
White v White [1949] 2 All ER 339, [1950] P 39, 113 JP 474, 27 Digest (Repl) 310, 2586.
Yarrow v Yarrow [1892] P 92: 61 LJP 69, 66 LT 383: 27 Digest (Repl) 374, 3087.

Appeal
This was an appeal by the petitioner wife, Josephine Williams, against an order of the Court of Appeal (Willmer and Davies LJJ,
Donovan LJ, dissenting) dated 12 July 1962, and reported in [1962] 3 All ER 441, dismissing the appellants appeal from an
order of Mr Commissioner Gallop QC made at Monmouthshire Assizes on 8 February 1962, dismissing the appellants petition
for dissolution of marriage on the ground of cruelty. The legal issue for decision on the appeal is stated, and the facts are briefly
summarised, at the beginning of the opinion of Lord Reid.

J P Comyn QC and A G Davies for the appellant.


John Latey QC and C N Pitchford for the respondent.

Their Lordships took time for consideration

27 June 1963. The following opinions were delivered.

LORD REID. My Lords, this case requires a decision of the question whether an insane person can be held to have treated his
wife (or her husband) with cruelty. The facts are not in dispute and they are clearly stated in the judgment of Mr Commissioner
Gallop QC so I need only state in outline those which are relevant to the issue. The respondent is a miner. For the first ten years
of married life his behaviour was not above reproach but he was never guilty of deliberate cruelty. There is insanity in his family
and in 1954 he began to hear voices and to think that people were after him. He was admitted to hospital as a voluntary patient
for three months and on his return home his wife says that his condition was worse. One night he thought that he heard people
talking about him. He got up and dressed, and went out with a knife looking for the people who were tormenting him. His wife
reported this and he was certified insane and was taken back to hospital. It is found as a fact on medical evidence that from then
until the trial in 1962 he was certifiably insane, and the evidence suggests that this is incurable. He frequently returned home for
weekends, but he was restless and the voices began to say that his wife was a prostitute. Nevertheless in 1958 he was regarded as
a voluntary patient and in March, 1959, he discharged himself and went home. His wife did not want to have him, but says that
she could not stop him coming. For the next nine months he was at home and his conduct during that time caused damage to the
wifes health. This was caused by the voices which told him of men up in the loft of the house and of his wifes persistent
adultery. He persisted in accusing her: if she tried to get away, he would follow her about the house. Sometimes he would climb
up into the loft to find the men. The learned commissioner had no difficulty in holding that the case was proved, unless the
second limb of the McNaghten rules applied. He held that the respondent knew what he was doing in making these accusations,
but that he did not know that they were wrong in any sense of the word. The medical evidence was that he thought that his
accusations were 996 based on sound fact and that he was fully justified in trying to resolve what was otherwise an intolerable
situation.
The learned commissioner in his judgment dealt in some detail with the authorities on the second limb of the McNaghten
rulesa matter to which I shall return later. He said:

If it were res integra I should have thought it virtually impossible to import the second limb of the rule into divorce
law. Is one to find and examine the opinion which a spouse, who was ex hypothesi insane, formed of the blameworthiness
or culpability of the conduct in question. That is if one treats the word wrong as meaning what Windles case said it does
not mean. If one treats the word wrong as what the Court of Criminal Appeal said it does mean, then much of the texture
of cruelty cases has nothing to do with criminal law, or tort or contract for that matter.

But he held that on authority he was bound to dismiss the petition.


The Court of Appeal (Willmer and Davies LJJ Donovan LJ dissenting) held that they were bound to hold that the
McNaghten rules applied, and bound by Palmer v Palmer to hold that the second limb of those rules applied as well as the first.
If that were so, then this petition must be dismissed. The majority further gave reasons why in principle the second limb must
apply and I shall have to deal with those reasons later. So the appeal was dismissed; but leave was granted to appeal to this
House.
In my judgment, before we come to the McNaghten rules at all we must first decide the general question whether insanity in
any sense is a defence to a petition for divorce for cruelty. The position as I see it is this. Before 1857 divorce a mensa et thoro
on the ground of cruelty was a well-established remedy given by the ecclesiastical courts. By the Matrimonial Causes Act, 1857,
the jurisdiction of those courts was transferred to Her Majestys courts: divorce a mensa et thoro became judicial separation, but
the same principles continued to apply. By the Matrimonial Causes Act, 1937, (now consolidated in the Matrimonial Causes Act,
1950), divorce for cruelty was introduced for the first time. The Act requires that the respondent shall have treated the petitioner
with cruelty. There has been much argument about the meaning of the word treated. I attach no importance whatever to that
word. What Parliament did in 1937 was to provide an additional remedy for cruelty. It did not touch the older remedy of judicial
separation which is still available. So, for that remedy, cruelty must have the same meaning today as it had before 1937. It is
incredible that cruelty now has a different meaning according to which remedy is chosen. But quite apart from that it seems to
me obvious that Parliament cannot have intended to alter the former meaning of cruelty. If it had been intended to alter the
meaning, no one in or out of Parliament would have been so foolish as to leave that intention, and the extent of the intended
alteration of meaning, to be inferred from the mere use of the word treated.
Accordingly it is necessary to see what the old law was. We have been referred to no case before 1857 in which there was
actual insanity. The earliest was Hayward v Hayward, and I get little or no help from that case. But ungovernable passion, or a
state of mind which was far from normal, was not unfamiliar, and I think it very helpful to see how those great judges Lord
Stowell and Dr Lushington dealt with this matter.
In Kirkman v Kirkman Lord Stowell (then Sir W Scott) said ((1807), 1 Hag Con at p 414):

This evidence most clearly establishes that the wife is not mistress of her own passions; and the court would be
wanting in due attention to the safety 997 of the injured party in this case if it did not pronounce for a separation as
absolutely necessary for that purpose.

In Holden v Holden Lord Stowell said ((1810), 1 Hag Con at p 458):

the court has had frequent occasions to observe that everything is, in legal construction, saevitia, which tends to
bodily harm and, in that manner, renders cohabitation unsafe; whenever there is a tendency only to bodily mischief, it is a
peril from which the wife must be protected: because it is unsafe for her to continue in the discharge of her conjugal
duties; and to enforce that obligation upon her might endanger her security, and perhaps her life. It is not necessary in
determining this point to inquire from what motive such treatment proceeds. It may be from turbulent passion or
sometimes from causes which are not inconsistent with affection, and are indeed often connected with it, as the passion of
jealousy.

Dr Lushington was even more emphatic. He said in Dysart v Dysart ((1844), 1 Rob Eccl 106 at p 116):

When I find conduct towards a wife likely to prove dangerous to her safety, but not in other cases, I shall consider it
within my cognisance, whatever may have been the cause thereof, whether having arisen from natural violence of
disposition, from want of moral conduct, or from eccentricity. It is for me to consider the conduct itself and its probable
consequences: the motives and causes cannot hold the hand of the court, unless the wife be to blame, which is a wholly
different consideration. In plainer words, even if I were satisfied that conduct dangerous in itself arose from morbid
feelings, out of the control of the husband, I must act, if the danger exists though it is not my province to inquire into or
ascertain such cause.

I find it difficult to believe that either of those great judges, if confronted with a case of insanity, would have said that then he was
powerless to afford any protection. And, apart from incarceration of the insane spouse, counsel were unable to suggest any
possible method of protection other than divorce a mensa et thoro. Even if cohabitation was not enforced the wife would, if she
left her husband, have been penniless in the then state of the law.
In Curtis v Curtis the Judge Ordinary, Sir C Cresswell, said ((1858), 1 Sw & Tr at p 213) after quoting the above passage
from Dysart v Dysart:

If indeed an act of violence were committed under the influence of an acute disorder such as brain fever, and it were
made clear that, the disorder having been subdued, there was no danger of a recurrence of such acts, the case would be
different. But if the result of such a disease has been a new condition of the brain, rendering the party liable to fits of
ungovernable passion which would be dangerous to a wife, then undoubtedly this court is bound to emancipate her from
such peril.

The same learned judge in Marsh v Marsh decreed judicial separation where the husbands acts were committed at a time when
he was suffering from delirium tremens.
In Hall v Hall Lord Penzance (then Sir J P Wilde) said ((1864), 3 Sw. & Tr. at p 349):

I have no doubt that cruelty does not cease to be a cause of suit if it proceeded from violent and disorderly
affections, as said in one case, or from violence of disposition, want of moral control, or eccentricity, as said in another,
or from a liability to become excited in controversy in the language of a third; but madness, dementia, positive disease of
the mind, this is quite another matter. An insane man is likely enough to be dangerous to his wifes personal safety, but the
remedy lies in the restraint 998 of the husband, not the release of the wife. Though the object of this courts interference is
safety for the future, its sentence carries with it some retribution for the past.

A similar result was reached in the Scottish case of Steuart v Steuart. And stress has been laid in other cases on certification of
insanity being an adequate protection. But the facts of the present case show that, owing to modern methods of treatment,
certification is not now always an adequate protection. This respondent was released though still certifiable, and then caused the
injury to her health which the appellant has proved. If the several learned judges, who relied on certification as a protection, had
had to consider circumstances in which it was not, I am by no means sure that they would have reached the same result. They
would have had to weigh the fact that separation is some retribution and is permanent even if the insane man recovers against
the present danger to the wife if no remedy is given. Looking to the paramount importance which was attached to protection they
might well have decreed separation.
There follows a gap in the authorities until 1892. In Yarrow v Yarrow the President (Sir C Butt) said ([1892] P at p 94):

I am by no means surethough I express no opinion upon it nowthat insanity which would entitle an accused to
an acquittal on an indictment for a crime would constitute a valid defence to a suit for divorce on the ground of adultery.

Hanbury v Hanbury was a petition for divorce on the ground of adultery coupled with cruelty. Sir C Butt considered at some
length the question of intermittent or recurrent insanity, where the husband would be entitled to go home from time to time when
he recovered. He said that though the insanity ([1892] P at p 224)

may assume the form of a disease, yet if it is such as to imperil the wifes safety she is entitled to the protection of
the court.

The jury found that the husband was capable of understanding the nature and consequences of his acts and decree was
pronounced. In the Court of Appeal ((1892), 8 TLR 559 at p 560) Lord Esher took this to mean that the husband knew what he
was doing and that he was doing wrong, and said:

If the disease in the mind of the person doing the act was not so great as to make him unable to understand the nature
and consequences of the act which he was doing that was an act for which he would be civilly or criminally responsible to
the law.

He reserved his opinion on the question whether the petitioner would be entitled to divorce if the respondent did not know the
nature of what he was doing or that he was doing wrong. There appear to be no reported cases on this matter between 1892 and
the passing of the Matrimonial Causes Act, 1937, which permitted divorce for cruelty.
I can deal with the later cases more briefly. The first relevant case under the Act of 1937 was Astle v Astle. There the
respondent did not know the nature and quality of those acts done while he was insane, and Henn Collins J held that those acts
could therefore not be held to be cruelty. The learned judge took this view chiefly because ([1939] 3 All ER at p 970; [1939] P at
p 418)

the respondent could not be held answerable outside this court either civilly or criminally for his assaults,

and he thought that the McNaghten rules should apply because they were the test applied in all other courts. In fact counsel
in the present case were unable to find any civil case, other than Hanbury v Hanbury and more recent divorce 999 cases, in which
the McNaghten rules have been applied. But the learned judge was able to grant decree on another ground. And indeed the
present case appears to be the first reported case in which a petitioner has failed to get relief on some ground against a spouse
who was insane.
Astles case was criticised in Squire v Squire, where the view that cruelty must be deliberate, malignant or intended was
rejected. Tucker LJ ([1948] 2 All ER at p 54; [1949] P at p 58), refused to dispose of the case on the ground that the wifes
conduct was not actuated by spite but was due solely to her state of health. My noble and learned friend Lord Evershed, having
pointed out that the wife was not insane, said ([1948] 2 All ER at p 55; [1949] P at p 61):

She remained at all material times responsible for her actions. If therefore her conduct to her husband was in fact
cruel, she cannot, as it seems to me, escape its natural consequences.

A good deal was said about the presumption that a man intends the natural and probable consequences of his acts, a matter to
which I must return.
White v White is an important case. There it was held, as the headnote states, that the mere fact that the respondent was
insane is no defence. The commissioner had held that the wifes mind was diseased and that she was not responsible for her acts
of cruelty. Bucknill LJ ([1949] 2 All ER at p 345; [1950] P at p 47), quoted and followed the judgment of Lord Esher in
Hanburys case which in effect applied the McNaghten rules. This respondents type of insanity did not come within those rules
and was therefore disregarded. Bucknill LJ, ([1949] 2 All ER at p 347; [1950] P at p 51) reserved his opinion on whether the
petitioner would have been entitled to protection if the respondent had come within the McNaghten rules, and indicated that
danger might be removed to some extent because a spouse may be able to defeat a petition for restitution of conjugal rights,
although he fails to prove legal cruelty. Asquith LJ ([1949] 2 All ER at p 347; [1950] P at p 51), gave as one of his reasons a
ground which to my mind is clearly wrong. He thought that if insanity is immaterial it must follow logically, that no state of
mind, intention or knowledge of the aggressor can be relevant. To my mind non sequitur: it is not illogical, though it might be
wrong in law, to say that some conduct is so bad that it is cruel in itself (which seems to have been the view of Dr Lushington in
Dysarts case while other conduct may be equivocal so that it can be excused by illness or insanity but aggravated by deliberate
intention. But he went on to say ([1949] 2 All ER at p 348; [1950] P at p 52):

It is plain that the presence of insanity of some sort or other, without more, will not necessarily afford a defence.
Unfortunately he did not say why that was plain. Of course, if the insanity is of a type which did not in any way cause the cruel
acts that would be plain. But he must have meant more than that, because in Whites case the insanity did give rise to the cruel
acts, and it was only because the insanity was of a type which did not fit the McNaghten rules that he was for granting decrees of
divorce. Denning LJ ([1949] 2 All ER at pp 350352; [1950] P at pp 5660), thought that the McNaghten rules do not apply to
divorce, that this woman, if charged with a crime, would have been found insane by a jury, but that insanity is no defence in
divorce proceedings.
In Lissack v Lissack my noble and learned friend Lord Pearce ([1950] 2 All ER at p 237; [1951] P at p 7) held that insanity
was no defence, but I need not deal with this case because my noble and learned friend will state his view much better than I can.
But Lissacks case was disapproved in Swan v Swan, and we have now to 1000 consider whether in this, Swans case was right. I
shall not attempt to deal with the judgment of my noble and learned friend Lord Hodson further than to note that he said (Cf
[1953] 2 All ER at p 857; [1953] P at p 263) that it was a contradiction in terms to describe as cruel the conduct of a person who
did not know what he was doing, and that the word cruel carried with it implications of guilt. Somervell LJ ([1953] 2 All ER at p
861; [1953] P at p 270), held that insanity was a defence, if the respondent did not know the nature of his acts, but thought that it
would be no defence if he knew the nature of his acts and merely did not know that they were wrong. Jenkins LJ held ([1953] 2
All ER at p 863; [1953] P at pp 271, 272) that a man who does not know what he is doing cannot be guilty of cruelty, but reserved
his opinion as to a man who knows what he is doing but does not know that it is wrong.
Finally in Palmer v Palmer the husband was insane with a delusion that his wife had been unfaithful, and he assaulted her
on a number of occasions. It was held that he knew what he was doing and knew it was wrong, and that both limbs of the
McNaghten rules applied. So it did not matter that according to the medical evidence he might very well think that he was doing
right by correcting her. I am bound to say that I think that this is a case in which a very antiquated view was taken about the way
in which an insane mans mind must in law be deemed to work, and I would not regard some of the opinions expressed as a safe
guide in other cases, whichever way the present case is decided.
We were also referred to a number of Scottish authoritiesquite properly because, as was said in Jamieson v Jamieson there
was no difference in principle as to what was cruelty for the purpose of separation, although there might be differences in the way
in which principles were applied. The cases show that the picture is broadly the same in both countries. With regard to insanity
being a defence the law was not clear. In Inglis v Inglis Lord Moncrieff (1931 SC at pp 548, 549) emphasised the importance of
protection, but Lord Anderson (1931 SC at p 551) thought that cruelty must be intentional or with wilful disregard of
consequences. In MLachlan v MLachlan Lord Moncrieffs view prevailed and this case was followed in Dobbie v Dobbie. In
MKenzie v MKenzie Lord Walker held that insanity was no defence, but in Breen v Breen the Second Division held that insanity
was a defence. This was a case which would not have come within the McNaghten rules, but the McNaghten rules are not part of
the law of Scotland. The defender was certifiable and his acts were committed under the influence of his insanity. Lord Patrick
said that the defender was not responsible for his conduct (1961 SC at p 185)

because that conduct was influenced by his insanity so that he was disabled form forming a rational decision in regard
to it.

Lord Mackintosh said (1961 SC at p 188) that

insanity, if shown to be related to the acts of cruelty complained of, is a good defence.

Lord Strachan said (1961 SC at p 197):

The defender was not able to exert his reason to control his reactions in the normal way. I therefore hold that he was
not responsible for his violent acts and that he cannot be held guilty of cruelty.

There appear to me to be four possible solutions of this problem. From the survey of the authorities, which I have made, I
do not think that any can be said 1001 to be firmly established, and a consideration of the arguments for and against each leaves
me with the opinion that none is in itself wholly satisfactory. But we must choose between them. We could follow the Court of
Appeal in holding that the McNaghten rules must be applied: or we could hold that only the first limb of the McNaghten rules
should be applied: or we could hold that insanity is no defence: or we could hold that insanity is always a defence if it appears
that it had given rise to the cruel acts. I shall first consider the McNaghten rules.
Apart from their inherent defects the McNaghten rules cannot be applied to civil cases in the same way as they are applied in
criminal cases. There are many indications that the rules have often been applied liberally or even loosely in criminal cases.
Much evidence was taken from distinguished and experienced witnesses by the Royal Commission on Capital Punishment and in
their Report they saya:
________________________________________
a Cmd 8932, p 82, para 232.

it was generally agreed that, as the rules are now applied, the great majority of those who have committed a crime
as a result of insanity, and ought not to be held responsible for the act, are in practice found guilty, but insane.

Then they quote from the evidence of many witnesses. For example, Lord Goddard said b:
________________________________________
b Cmd 8932, p 82, para 234.

I think a jury can always be trusted to do justice, where it might be impossible to bring the case strictly within the
McNaghten rules, but everybody would say that the mans acts were the acts of a lunatic,

and then they quote from the evidence of Lord Simon who had been in turn Home Secretary and Lord Chancellor c:
________________________________________
c Cmd 8932, p 82, para 235.

I imagine that it would be conceded that the strict rigour of the rules as to the intellectual test is in practice to some
extent qualified. A British jury, whatever you say, will see that it is qualified if they are really convinced that it is a proper
case.

On the other hand there was evidence that sometimes the formula is applied rigidly in its literal meaning. The Royal Medico-
Psychological Association is quoted as sayingd:
________________________________________
d 5 Cmd 8932, p 86, para 245.

The association is fully aware that if the rules were rigidly interpreted a majority of even insane murderers would be
judged criminally responsible.

This may be an exaggeration, but it seems clear that the accused at least often gets the benefit of the doubt. But it would be
impossible in a divorce case to give the benefit of the doubt to an insane aggressor against the injured spouse.
Then there is a second difference. The second limb of the rules has been interpreted in criminal law so that wrong means
contrary to law and not wrong in the eyes of the accused (R v Windle). Obviously that meaning cannot be applied in a divorce
case and to substitute the meaning morally wrong is to alter the rules substantially.
I have come to be clearly of opinion that it would be wrong to take the McNaghten rules as a test. Not only have these rules
been subject to persistent and powerful criticism for nearly a hundred years but their strict application would lead to capricious
results. It appears to be the general opinion of medical men, who at least have a better understanding of insanity than lawyers,
that there are types of insanity not within the rules which deprive the insane man of choice or responsibility just as much as those
types which are covered by the rules. So if guilt, culpability, or blameworthiness in some degree is to be held a necessary
element in cruelty, I can see no rational basis for holding that if two 1002 persons are in fact equally irresponsible one is to be
divorced because his type of insanity does not come within the rules, but the other is to have a defence because his case is
covered by the rules.
The second possibility is that insanity should be a defence if it is so bad that the insane person does not know what he is
doing, but not otherwise. This test seems straightforward whereas the test that a man knows that his acts are wrong may seem
simple, but in fact is not: it is a test which experts seem to have great difficulty in applying. Moreover I think that many people
would say that if a man does not know what he is doing his acts are not really his acts at all, but they have at least a lingering
suspicion that when a man does know what he is doing he cannot be wholly blameless, whatever the experts may say. I think that
this is a possible half-way house, but I would not much favour it myself. It is still subject to the objection that it discriminates
between people who on evidence are proved to be equally irresponsible by reason of disease of the mind. But I must recognise
that, if we are thinking of protection, a man who does not know what he is doing will, except in cases of epilepsy or temporary
lapses into insanity, generally be detained, whereas a man certifiable for less serious types of insanity may be released at least for
a time.
So it remains for me to choose between the two clear-cut alternativeseither insanity is a defence or it is not. I think that
ultimately the answer must depend on the meaning one gives to the word cruel, and on this there are obviously two opinions,
even among judges. Some think that there cannot be cruelty without some kind of means rea and some think that there can. To
my mind cruelty is a word that can take its meaning from its context: often it connotes blameworthiness, but quite often it does
not. Let me give one or two examples. Even in comparatively recent times practically everyone, including men of the highest
integrity and intelligence who were quite as civilised as any of us, firmly believed that persecution in one form or another was not
only excusable, but was a moral duty. Few would deny that their acts were cruel, but I do not see how we can reasonably blame
them for not having anticipated modern ideas. And is it a misuse of language to call a cat cruel? Again, when we speak of the
cruel sea, no doubt we personify the sea, but do we blame it? So the law cannot just take cruelty in its ordinary or popular
meaning because that is too vague: we must decide what, if any, mental state is a necessary ingredient. I have already dealt with
this in a general way in Gollins v Gollins (See p 966, ante), but perhaps I should restate the argument shortly with special
reference to this case.
If some metal or subjective element is necessary, the first possibility is that there must have been malignity or an intention to
hurt. But that has long been abandoned. Such an intention may be an aggravation, and may justify holding that acts are cruel
where, without that intention, they would not be sufficiently grave and weighty to amount to cruelty: but it is not essential. The
next possibility is that there must be some degree of mens rea, that at least the respondent must be blameworthy for what he did
and that requires careful consideration. But opinions have been expressed that it is enough if his acts were intentional, if he
knew what he was doing, although he could not be held blameworthy. That would be a basis for holding that insanity is a
defence if the insane person did not know what he was doing, but not otherwise. I have already dealt with that and I need not say
any more about it than that that view would lead to this appeal being allowed, because it is clear that this respondent knew what
he was doing but thought by reason of his delusions that he was justified in what he did. So to my mind the alternatives are either
that no mental element is essential or that the respondent must at least be blameworthy.
I have already dealt in Gollins case (See p 966, ante) with the man who knows that he is injuring his wifes health and
persists in his conduct. He is clearly blameworthy, unless he has adequate justification. But what if he did not realise the damage
1003 that he was doing. Again it would be wrong to bring in the reasonable man and what he would have realised: that would
throw no light on whether this man was blameworthy. It would be necessary to consider the particular respondent with all his
limitations. If it could properly be said that he ought to have realised, then he would be blameworthy, because that would be
equivalent to a finding that he had shut his eyes to the consequences of his conduct. Then we come to the really difficult cases if
blameworthiness is to be a test. There are many cases of husbands and wives not insane but either sick in mind or body or so
stupid, selfish or spoilt that they plainly do not appreciate or foresee the harm which they are doing to the other spouse, and
perhaps they are now so self-centred that nothing would ever get the truth into their heads. Certainly allowances have to be
made, particularly when their condition is due to misfortune. But I suppose that no one would now maintain that cruelty cannot
be proved against such a person, if his acts are sufficiently grave and really imperil the other spouse.
It is often untrue that such a man is able to exert his reason so as to control his acts in the normal way or even that he is
capable of forming a rational decision about them. Yet these are often the cases where the other spouse is most in need of
protection. It is difficult in some of these cases to attribute more than a speck or scintilla of blame to the respondent in the sense
that he, not the reasonable man, ought to have realised the consequences of what he was doing and could have done otherwise if
he had tried. If we are to make culpability an essential element in cruelty, we can really only bring in these people by deeming
them to have qualities and abilities which the evidence shows that they do not possess. Surely it is much more satisfactory to
accept the fact that the test of culpability has broken down, and not to treat entirely differently two people one of whom is just
short of and the other just over the invisible line which separates abnormality from insanity.
In my judgment decree should be pronounced against such an abnormal person not because his conduct was aimed at his
wife, nor because a reasonable man would have realised the position, nor because he must be deemed to have foreseen or
intended the harm he did, but simply because the facts are such that, after making all allowances for his disabilities and for the
temperaments of both parties, it must be held that the character and gravity of his acts was such as to amount to cruelty. And, if
that is right for an abnormal person, I see no good reason why the same should not apply to an insane person.
If that be accepted then the appeal must be allowed. I must add that we are not called on to determine the character and
gravity of this respondents acts. No attack was made on the findings of the learned commissioner. I do not express any opinion
about them, but I must not be taken as agreeing that acts like those of this respondent put the wife in such peril as to necessitate
protection, or that sufficient allowance was made for the facts that the respondent was insane and that the appellant knew that his
charges against her were entirely the product of his insane delusions.
LORD EVERSHED MR. My Lords, this case comes before your lordships House raising a single question, apparently but
deceptively limited, namely, whether the second of the so-called McNaghten rules applies in the case of a petition for divorce on
the ground of cruelty with the effect in the present case of defeating the appellant petitioners claim. The facts of the case are
sufficiently stated in the opinion of my noble and learned friend Lord Reid, and I do not repeat them. It is sufficient to say that
the learned commissioner found first that the respondent husband was aware of the nature and quality of the acts which he was
doing (namely, of his persistent accusation of his wife that she was committing promiscuous adultery) but second that he was,
owing to his mental disease, unable to realise that his accusations were false and that accordingly his conduct was wrong. It
should be added that the commissioner had no difficulty in finding that the husbands course of conduct was in fact injurious to
his wifes 1004 health. The learned commissioner expressed the view that if the matter were res integra before him he would
hold that the second McNaghten rule had no application; but he felt himself bound by authority to take a different view and he
therefore found that the wifes claim based on cruelty was answered by the circumstance that the husband was unaware that what
he was doing was wrong. The Court of Appeal by a majority (Willmer and Davies LJJ Donovan LJ dissenting) affirmed this
conclusion, following recent decisions of that court.
For myself I confess that I can feel no doubt that the answer to the specific question raised in the case is that the second
McNaghten rule has no application, with the consequence that the present appeal must be allowed. It will be of some assistance
in regard to what follows if I briefly state the history of what I will call the intrusion of the so-called McNaghten rules into
divorce law. The history may be said to begin with the case of Hall v Hall decided in the year 1864. In that case Sir J P Wilde
((1864), 3 Sw & Tr at p 349), though stating that intemperance and conduct of that kind would be no answer to an allegation of
cruelty, yet thought that insanity might be a different matter. The question did not arise again in the reports until the case of
Hanbury v Hangury in the year 1892. In the result the question did not call for decision, but in the Court of Appeal Lord Esher
MR ((1892), 8 TLR at p 560), expressed the view that the case might well have been different, that is to say, a petitioner might
fail if it were shown that the respondent was unaware of the nature and quality of what he was doing, or alternatively if he
realised such nature and quality but did not know that he was doing wrong. Though Lord Esher made no reference in terms to the
McNaghten rules, the dictum to which I have alluded was clearly a paraphrase of them. Again there is a considerable interval
until the year 1939 (that is, two years after the passing of the Matrimonial Causes Act, 1937) when in the case of Astle v Astle,
the first two McNaghten rules were for the first time specifically mentioned and applied by Henn Collins J ([1939] 3 All ER at p
971; [1939] P at p 419), in reference to his proposition that the word treated in the Act of 1937 denoted conscious action on the
part of the person charged with cruelty. The next date to mention is one of great importance, namely, the year 1949, in which the
case of White v White came before the Court of Appeal. Though in this case, as in the case of Astle, the petition in fact
succeeded, nevertheless Asquith LJ ([1949] 2 All ER at pp 347350; [1950] P at pp 5156), in the course of his judgment treated
it as established that if the spouse charged was unaware through insanity of the nature and quality of his acts it would be a
defence to the petition, and he added that if for such a purpose the first of the McNaghten rules had been invoked it was
inevitable and logical that the second rule should also be available, that is to say, that even if the spouse charged did know the
nature and quality of his acts he would have a defence if, through insanity, he was able to establish that he was unaware that what
he was doing was wrong. This statement of Asquith LJ has since been followed in the Court of Appeal particularly in the two
cases of Swan v Swan and Palmer v Palmer. As my noble and learned friend Lord Reid has pointed out, in none of the reported
cases has the application of these McNaghten rules or either of them in fact so far provided a defence to the claim, but it must be
taken that during the past decade it has been accepted by the Court of Appeal that both McNaghten rules are or may be applicable
in cases of divorce founded on charges of cruelty, and it was this view which commended itself to the majority of the court in the
present case.
As is well known the so-called McNaghten rules consisted of answers given by the judges to questions put to them by your
lordships House in reference to the 1005 proper direction to be given at a criminal trial and particularly at murder trial. My
noble friend Lord Reid has pointed out that of late the so-called rules have not been regarded as entirely satisfactory even in
criminal casesas may indeed be illustrated by the passing of the Homicide Act, 1957. But apart from such criticism, it is my
opinion that the rules ought, as such, to have no application to divorce cases. It was of the essence of the so-called rules that they
should be invoked where the test for liability involved the existence of mens rea (that is, a criminal mind) which by the common
law and in a great many cases of statutory offences is an essential ingredient to a criminal act. Cruelty is not a crime. Therefore,
as it seems to me, it follows that mens rea as understood in our law should not be regarded as being of its essenceany more
than it is generally so regarded in our civil law, except in rare cases as, for example, the tort of malicious destruction of property.
In any case I should venture to think, with all respect to those who take a different view, that the rules and particularly the second
rule can have no application to case like the present. Here the respondent, Mr Williams, was not doing things which were ex faci
criminal or analogous to criminal acts. He was deliberately accusing his wife of adulterous associations, but he thought that such
accusations were justified because he believed that they were true, having been so induced to think by voices which informed
him of the fact.
It is, however, clear that, having regard to the arguments presented to your lordships, it would not be regarded as a sufficient
answer to the present case merely to state that the second McNaghten rule had here no application. The case and the arguments
presented to your lordships House have inevitably raised the general and important questionto what extent is insanity an
answer to a plea of cruelty in a petition for divorce? I should here state that in all that follows I shall assume that the petitioner is
able to establish injury to health or proper anticipation of such injury. Such a premise is essential for the establishment of cruelty,
as was laid down by this House in the case of Russell v Russell. I have already stated that in the present case the facts were found
to establish such injury or anticipated injury, and in all that follows I shall assume the premise similarly established. The question
whether cruelty is established may be said to involve two extremes of view. On the one hand it may be said that the question
whether actions are cruel must be judged wholly objectively, that is to say, whether the actions complained of would be
regarded by any reasonable man if done by one ordinary person to another as being cruel to the spouse affected. To take one of
the illustrations given in the Oxford Dictionary of the meaning of the word cruelty it was stated by an eminent writer of the last
century that this brutal cruelty to make any jest of the weaknesses and sufferings of the patients in a hospital. If therefore a
man is found in fact to be making jests of the weaknesses or sufferings of patients in a hospital, will it be any defence to the
charge, will it, in other words, make his actions not properly described as cruel if it be shown that the jester is for any reason so
insane as to be unaware that he is making a jest or as to be unaware that the persons against whom his remarks are directed are
patients in a hospital? On the extreme objective view the answer will be No. On this view the behaviour of the man in
question will be called properly cruel, whether he is really aware of what he is doing or not. There is, however, another extreme
view which essentially rests on the premise that the test of cruelty is subjective, that is to say, that no one can properly be accused
of cruelty unless there is something opprobrious, something in his behaviour that must be condemned as inexcusable, a test
which, it is said, cannot fairly be satisfied unless he is not only aware of what he is in fact saying or doing but knows or ought to
know that what he is saying or doing is wrong or culpable. Put in another form, on this view it is said that cruelty properly so
called involves essentially an element of malignity on the part of the person called cruel. On this view it would mean that
insanity, if it were such that the actor was unaware of the nature and quality of his acts or thought that such acts were justified
and I assume that if he in a case such as the present 1006 accepted the truth of the hallucinations they would be justifiedwould
disqualify the proper application of the word cruel to the actors conduct.
The length and intricacy of the arguments presented shows how important it now is that your lordships should if possible
give an answer to this difficult and important question of such clarity as will enable Her Majestys judges hereafter to know
properly how to direct themselves in cases of this kind, and as also will enable counsel and solicitors properly to advise their
clients.
A reference to the very many cases cited shows that the problem has been much vexed by what I venture to call quasi-
philosophical discussions on certain other and related questions: for example, whether in order to establish cruelty the acts must
in some real sense be aimed at the person affected, and the related question (if indeed it is not the same question differently
expressed) to what extent must there on the part of the actor be an intention to hurt (or at least to do acts which hurt), whether
such intention be conscious or deliberate or in some cases presumed: for, as the decisions show, the courts have been compelled
in the interests of justice, and applying common sense, to expand the the requisite of intention so as to include presumed intention
(the presumption being sometimes regarded as compelling only and sometimes as conclusive) and thereby to add greatly to what
I have called the quasi-philosophical discussion on the subject, and therefore to its difficulty and uncertainty. It is no doubt true
that in some cases, applying the ordinary standards of sense and language, proof of a deliberate intention on the part of the actor
to hurt may be highly relevant in deciding whether his conduct amounts to cruelty. But, save in such borderline cases, my view
is that the problems whether the conduct challenged was aimed at the person affected or was (or should be treated as)
intentional do not properly arise in the jurisdiction with which we are now concerned, and I venture to think that their presence
and the discussions of them in many of the cases has served seriously to cloud the law. We are here concerned with the formula
stated in the Matrimonial Causes Act, 1937, has treated with cruelty. True it is that before 1937 the formula was (as it now is in
Scotland) guilty of cruelty. As my noble and learned friend Lord Reid has pointed out, the formula guilty of cruelty was
applicable before 1937 to a claim for judicial separation. In my opinion it is impossible to suppose that Parliament in 1937
intended that cruelty as a ground for divorce should have a different significance from that appropriate to the older remedy of
judicial separation. To my mind the variation in the formulation makes no real difference. The use of the word guilty does not,
as it seems to me, import the essentials of crime, any more than it does in such a common context as guilty of negligence. The
phrase has been guilty of cruelty means no more and no less than has acted cruelly to the person making the charge, in other
words, has been cruel to him or her. The formula is therefore in truth the same as that now laid down in the Act, has treated with
cruelty.
I therefore conceive that the first essential thing is to decide what is meant by the word cruelty. The word is one of
common use and is defined as meaning delight in or indifference to pain or misery in others. I observe at once that there is
therefore inevitably involved the case, which must indeed be common if perhaps not the most common of all cases, where the
conduct is founded not on delight in the pain of others but on indifference. The cases indeed show clearly enough that so often
the spouse charged with cruelty is doing little else than indulge his or her own purely selfish desires or instincts. It follows,
therefore, in my opinion, that this essential meaning of the word must dispose of the argument involved in the proposition that the
so-called cruel acts must in some sense be aimed at the other party concerned. If a mans acts are founded on stupidity or on
disregard of anyone else or of anything but his own self-interest it is clear that he none-the-less may fairly and properly be guilty
of cruelty. On this simple ground therefore (though I do not forget the type of case already mentioned, when the presence of
deliberate intention on the actors part to hurt may be decisive in determining whether his conduct is fairly to be described as
cruel) I would reject 1007 entirely the notion that aiming at the injured party or intention to hurt on the actors part is an
essential of cruelty, and I am assisted to that conclusion because of the necessity in any event for the injured party to prove that
his or her health has been in fact affected or that he or she may fairly apprehend such resultin other words, that acts or conduct
of the party charged have, whatever the aim or lack of aim, hit the other spouse. Force, as I venture to think, is given to this
conclusion by considering such phrases treated with stupidity or treated with disrespect. It is not, as I venture to think,
essentially inherent in either phrase that there was any intention on the person said to have treated another with either stupidity
or disrespect. I am also for my part unable to accept the view that there must be present in order to constitute cruelty any
conscious moral obliquity. There is nothing in the word cruelty or in the formula chosen by Parliament to suggest such a thing
and (as Donovan LJ pointed out in his dissenting judgment in the present case ([1962] 3 All ER 441 at p 452; [1963] P 212 at p
233) it would have been easy for Parliament so to provide, if it had been intended that it should be of the essence of cruelty for
the purposes of the Act, either that there should be deliberate intention or that there should be moral obliquity on the part of the
actor. I would add that if intention (whether actual or presumed) be introduced it must in any event be subject to some
qualification such as without lawful excuse. This is shown if the example be taken of a policeman attempting to arrest a
fugitive criminal. In such cases the policeman may justifiably inflict some injury on the person who is seeking to evade arrest.
No doubt if the policeman exceeded what was reasonably necessary or proper for the performance of his duty he might expose
himself to the charge that his action had been cruel. But, if he did inflict injury properly in the execution of his duty, he could not
fairly be called cruel, though it would be clear that the injury inflicted would be rightly regarded as intentional.
In the circumstances it is indeed tempting to say that the test is in all cases purely objective. If a man is seen to be beating
his wife, his child or his dog, the question would be whether, according to the judgment of a reasonable man who saw the
performance, the actor would fairly be said to be treating his wife, his child or his dog with cruelty? There is, however, the
difficulty at once presented to this simple view, viz, if in truth the man accused of cruelty is through insanity quite unaware of
what he is doing, can he fairly be said to be acting cruelly at all any more than it could fairly be said of a sleep-walker who
inflicted some injury on a child or a ketten or puppy in his sleep that he was cruel? It was the view of Donovan LJ ([1962] 3 All
ER 441 at p 452; [1963] P 212 at p 233), that the use of the word treated does in some sense involve knowledge on the part of
the actor of what he is doing. Moreover, as I have already observed, it seems in England from the year 1864 to have been
consistently suggested by the judges that the acts of a madman should be differently treated from acts done in the heat of passion
or intemperance. In other words, it may be said to have been part of our law as indicated by the judges (even if it has not in fact
been applied in any reported cases so as to defeat the plea made) that if a man, who does acts which might otherwise and
objectively regarded be thought to be cruel, did not in truth know the nature and quality of those acts, then it should not be treated
as cruelty. It is also true to say that, if the purely objective test be taken, there will inevitably be created a wide divergence
between the law of England and the law of Scotland on this matter, as the latter has been recently expounded.
On the other hand, my noble and learned friend Lord Reid has in his opinion shown that in the early part of the history of the
law relating to cruelty considerable emphasis was laid on the need to give protection to the sufferer and therefore that, prior to the
decision in Hall v Hall, mental infirmity does not appear to have been regarded as inconsistent with the application of the epithet
cruel to a spouses conduct.
1008
If the decision in this matter rested with me alone I am disposed to think that I should take the view that, on the ordinary
sense of language, a man could not and would not be said to be treating another with cruelty if he was shown, by reason of
mental disease or infirmity, not to be at all aware of what he was doingif, to take an extreme case, a man who was observed to
be beating physically his wife with the utmost severity were proved to be quite unaware that he was doing other than beating his
drawing-room rug. On this particular question it may strictly not be necessary for me to express any concluded opinion. But, as I
have earlier stated, the nature and extent of the arguments presented to your lordships have undoubtedly raised, and raised before
your lordships House for the first time, the whole question of the extent to which insanity may be an answer to a petition for
relief in a matrimonial cause based on an allegation of cruelty; and on this question two of your lordships take one view, and two
of your lordships take another. In the circumstances I have felt it my duty to express a conclusion consistent with one or other of
the divergent views which your lordships entertain. I therefore conclude, though I confess with some hesitation, that the test
whether one charged with treating his or her spouse with cruelty is to be applied wholly objectively, and therefore that proof of
insanity (that is, proof that he or she was unaware through mental disease or disorder of the nature and quality of his or her acts)
is not necessarily an answer to the charge. I say deliberately not necessarily. The mental derangement of the person charged
cannot, as I think, be wholly disregardedcertainly where the sufferer is himself or herself aware of the disorder. But the test
will be still objectivein all the circumstances of the case should it fairly be said that the spouse charged has treated the other
with cruelty? As I have said, since the matter will be judged objectively, prima facie the insanity of the person charged will not
disable an affirmative answer being given to the question. Beyond that, however, I would for myself prefer not to go; for there
may be instances where, in all the circumstances of the case, the objective observer would say that the conduct complained of
was not in truth cruel. I do, however, conclude that generally speaking the conduct of the party charged will not fail to be
properly described as cruel merely because he or she is unaware of the nature and quality of his or her conduct. Certainly if the
conduct is regarded from the point of view of the sufferer it would not, in the ordinary case, be said that he or she was not cruelly
treated; or indeed that the party charged did not in fact act cruelly, though no doubt (unhappily) he or she was through insanity
morally blameless. In reaching eventually this conclusion I have in mind the point made by my noble and learned friend Lord
Reid that a distinction may otherwise unfairly and illogically be drawn between one kind of insanity and anotherto the serious
detriment of the suffererthat is, between the case of the man or woman who through insanity does not know the nature or
quality of his or her acts and the man or woman who, though aware of the nature and quality of his or her acts, nevertheless
through mental disease or disorder genuinely is unaware that they are unjustifiable. I add too that the emphasis given in the older
cases (as pointed out by my noble and learned friend Lord Reid) to the need to give protection to the suffering spouse has
certainly not been qualified by the fact that, in the present day and age, the break-up of a marriage is regarded much less seriously
than it was a hundred years ago.
But whatever be the true and just answer to the question whether a husband or wife has treated his or her spouse with cruelty
if he or she is through insanity unaware of the nature and quality of the acts complained of, I confess that I have no doubt that it
can be no answer to a charge of cruelty on his or her part that, knowing the nature and quality of his or her acts, he or she is,
through insanity, unaware that those acts are wrong. In other words, if the person charged with cruelty knows what he is doing,
knows, that is to say, the nature and quality of his acts, then it is for the court or the jury to say whether the other party has in
truth been cruelly treated thereby, and it will be no answer for the actor to say that he did what he did or failed to do what he
ought to have done because he was 1009 justified on account of some hallucinations based on insanity, or because he otherwise
did in truth assume the facts to be such as would or might have justified his conduct. In such event it does seem to me that, even
if the test of inexcusability be applied, his or her conduct will truly and fairly be said to be inexcusable, though it may perhaps be
explicable.
I add finally this, if it be said on a true view that there must be some element of opprobrium, if there must on the part of the
person charged be something which fairly can be said to be conscious and deliberate wrongdoing, then where does the matter
end? As it seems to me, it cannot be said fairly to end by considering matters merely of insanity. Suppose the case be taken of
one who is a devotee of some extreme religious dogma honestly and conscientitously held. For example, suppose the case of a
man of such strict puritan views that he firmly believes that one who uses bad language is inevitably condemned to hell. He
hears a child use some swear word and in the belief that he will thereby be saving and cannot otherwise save the child from
perdition, he then inflicts on the child the most severe caning or other physical punishment. Can it be denied on the part of the
person so behaving that his conduct has been cruel because (as he says and honestly says) such conduct is based on some
extreme, but sincerely held, religous view? In other words, as I venture to think, it is impossible to stop at cases of insanity; and,
when questions of cruelty arise, problems of the kind which I have mentioned will inevitably also arise, thereby adding to the law
not only an altogether undesirable refinement but also a departure, as I think, from the ordinary sense and meaning of the
language which Parliament has thought fit to use.
I would like on this most vexed and difficult question to express my indebtedness to Dr A L Goodhart for the full and lucid
analysis of the problem contained in the article contributed by him (in 79 Law Quarterly Review 98) in January of the present
year.

LORD MORRIS OF BORTH-Y-GEST. My Lords, the immediate question which is raised in this appeal is whether the
respondent treated the petitioner with cruelty. Linked with that question is the wider one as to the measure of the relevancy if any
of the mental state of the respondent. The problem which is raised may be posed in simple form by taking an imaginary case. If
a respondent repeatedly assaulted a petitioner causing injury to health, would it be legitimate for a court to have regard to the fact
that the assaults were all attributable to insanity in the respondent and to decline to give a decree. If it is legitimate to pay heed to
the fact that a respondent is mentally deranged the further question arises whether there is any set test or formula by which to
assess and measure the extent of the relevancy of the particular mental derangement.
My lords, in King v King in your lordships House it was pointed out that the question whether one spouse has treated the
other with cruelty is a single question, which is only to be answered after all the facts have been taken into account. I was said
that it is not right first to ask whether a respondents conduct was cruel in fact and then to ask whether it can be in any way
justified. In the present case the main issue has by both sides been formulated as being whether insanity is a defence to a charge
of cruelty in matrimonial causes. So formulated there is implicit a suggestion that in a judicial investigation in a cruelty case the
acts complained of may first be isolated so that it may be decided whether they should be characterised as cruel, and that it should
then be decided whether the defence of insanity can be deployed. Such an approach seems to me to be undesirable and to be
contrary to the guidance given in King v King. Any questions which arise concerning the mental health of the parties should, in
my view, be regarded as relevant and integral parts of the inquiry whether one spouse has treated the other with cruelty. If there
is insanity, it is a fact which is to be taken into account. Insanity does not therefore come in as a 1010defence: it comes in
because it is a fact and a circumstance which may loom large in any true and complete narrative of the events which are under
review in an inquiry whether one person has treated another with cruelty. When human conduct is being assessed, all its features
would seem to possess relevancy. In their dealings with each other human beings recognise that there are occasions when in
justice and humanity allowances must be made. I cannot think that it would be helpful ever to seek to catalogue or to define such
occasions. Nor do I think that it would be desirable to confine the conception of cruelty within precisely defined limits. If a jury
is asked to say whether one spouse has treated the other with cruelty, the task should not be beyond the competence of justminded
and reasonable persons who, though lacking knowledge of decided cases, have an equipment of common-sense derived from a
knowledge of human nature and of the ways of the world.
In such an inquiry some questions which arise can be answered with prompt and assured confidence, although a full
explanation of the exact stages of reasoning which warrant the answer may require skill in exposition, analysis and expression.
But it is the answer that is important for the parties: a jury would be called on to give the answer, but do not give their reasons.
If a devoted spouse, by some quite unintentional mishap in the home, caused severe personal injury to the otherno one would
be likely to assert that the one had treated the other with cruelty. The same view would be likely to be taken even if the
unintentional mishap were occasioned by carelessness. The statute does not in terms say that accident is a defence. It need
not do so, because anyone called on to decide whether one person has treated another with cruelty would naturally and without
question consider it to be highly relevant to know whether an occurrence was accidental. If one spouse while genuinely asleep
unwittingly caused severe personal injury to the other, it would be unlikely that anyone would say that the former had treated the
latter with cruelty. If one spouse in a state of self-induced but complete intoxication caused severe personal injury to the other
different considerations might apply. So also some isolated incident might be viewed in one way and a series of incidents might
be viewed in another. I would think therefore that there is no touchstone by which to appraise conduct. In Watt (or Thomas) v
Thomas Lord Thankerton said ([1947] 1 All ER 582 at p 587, letter D; [1947] AC 484 at p 488): The law has no footrule by
which to measure the personalities of the spouses. In some cases one approach may be apposite though in others it would not.
In some cases one test may be adequate while in others it will not be. In King v King Lord Normand said ([1952] 2 All ER 584 at
p 586; [1953] AC 124 at p 129):
I have no doubt that the test whether the conduct was wilful and unjustifiable, as well as injurious, was an adequate
test for what remained to be decided in Hortons case. What is open to question is whether it can be taken to be an
adequate test in all cases of cruelty by nagging accusations. I think it is not always an adequate test, and that BUCKNILL,
J., did not put it forward as a universal and exhaustive test in this type of case. The general rule in all questions of cruelty
is that the whole matrimonial relations must be considered, and that rule is of special value when the cruelty consists, not of
violent acts, but of injurious reproaches, complaints, accusations or taunts. Wilful accusations may be made which are not
true and for which there are no probable grounds, and yet they may not amount to cruelty. To take an obvious example,
they may have been provoked by the cruel conduct of the other spouse. There is in many cases no easy rule, no clear line
of demarcation which divides cruelty from something which does not amount to cruelty. The issue may become one of
great difficulty in which the decision must be largely a matter of the discretion of the judge 1011 who saw and heard the
witnesses and who has considered the conduct of both parties, and the whole circumstances in relation to the temperament
and character of the respondent spouse. If the trial judge, in the exercise of this discretion, comes to the conclusion that the
conduct of the respondent is, notwithstanding the provocation received or the difficulties and stresses endured, really an
inexcusable offence against the other spouse, his judgment should be respected and treated as conclusive. I have used the
word inexcusable, but unpardonable, or unforgivable or grossly excessive would equally convey what I mean.

Later on in his speech Lord Normand referred to the temperaments of the spouses in that case and said e that both parties had
great need of forbearance: he saidf that the husband had behaved with great want of consideration: he said g:
________________________________________
e [1952] 2 All ER at p 587, letter b [1953] AC at p 131
f [1952] 2 All ER at p 588, letter d; [1953] AC at p 133.
g [1952] 2 All ER at p 588, letter h; [1953] AC at p 133.

They were both blameworthy and each was guilty of behaviour which considered without reference to the behaviour
of the other, might be regarded as cruel.

All this serves to illustrate that in a cruelty case the conduct of both parties falls to be assessed as well as everything that is
relevant to the temperament and character of the respondent spouse. In his speech in the same case my noble and learned
friend Lord Reid said ([1952] 2 All ER at p 596; [1953] AC at p 146):

I do not intend to try to define cruelty. I doubt whether any definition would apply equally well to cases where there
has been physical violence and to cases of nagging, or to cases where there has been a deliberate intention to hurt and to
cases where temperament and unfortunate circumstances have caused much of the trouble. But in cases like the present,
the wifes conduct must at least be inexcusable after taking everything into consideration.

My lords, applying these tests I consider that the factor of illness may often be highly relevant in assessing whether conduct has
been cruel. What someone does while in a state of automatism may not unreasonably be said not to be that persons action at all.
What someone does while suffering from a raging physical fever might be regarded as excusable. Is what someone does while
suffering from that form of illness which dethrones reason and withdraws the power of control to be regarded as inexcusable? I
cannot think that physical illness could be regarded as relevant, but that mental illness should be regarded as irrelevant. Evidence
concerning illness may be of high consequence in any survey which is designed to give that complete picture of domestic life
against the background of which judgment is formed as to whether one spouse has treated the other with cruelty. In his speech in
Watt (or Thomas) v Thomas ([1947] 1 All ER 582 at p 591) Lord Simonds approved the clear recognition which he discerned in
the judgment of the Lord Ordinary (Lord Patrick)

of the fact that the picture of the domestic lives of this man and woman must be surveyed as a whole, before a true
judgment can be formed of their possible future relations.

Lord Du Parcq, speaking in that case of an assault which had taken place, said (1947 SC (HL) 45 at p 63):

The assault must be viewed not as an isolated fact, but in its setting, as an incident in a complicated series of marital
relations. The intent and the mental condition of the spouse against whom cruelty is alleged are always important
considerations.
1012

He further said (1947 SC (HL) at p 63):

The fact that the defender was suffering from a form of ill health likely to affect her conduct is not to be regarded as
telling against her. I would not suggest that illness ought to be accepted as an excuse for cruelty but when it is necessary to
appreciate the quality of an act or a course of conduct, as it is when the grave accusation of saevitia is made, the law does
not forbid a judge to have regard to any physical or mental strain under which the accused spouse was labouring.

My lords, I think that it is right to view a charge of cruelty as a grave accusation. Opprobrium is involved if it is held that
one spouse has treated the other with cruelty. It is described as a matrimonial offence not merely in common legal parlance but
also in the Matrimonial Causes Rules, 1957. The conduct of a spouse, who is accused of cruelty and who enjoys normal physical
and mental health, will be judged by any such test as may be applicable having regard to the facts of the particular case. Some
acts may follow a declared express intention to be cruel. Some acts may be such as to reveal or to make it proper to infer an
intention to be cruel. In other cases there may be no such wilful intention but a complete failure, either from indifference or
obtuseness, to appreciate that conduct is causing pain and hurt. These cases may present difficulties for judges though it is not in
this field alone that human beings demand of each other that they must conform to normal decent accepted standards. Whatever
be the analysis philosophically of the oft-quoted phrase of Shearman J in Hadden v Hadden:

I do not question that he had no intention of being cruel, but his intentional acts amounted to cruelty.

I venture to think that it expresses an accepted conception. If there are intentional acts which, viewed objectively, amount to
cruelty, it is no relief for the injured spouse to be told that the offending spouse, being capable of normal rational thought, either
failed to think at all or failed to appreciate. It is neither harsh nor unfair in such circumstances to say that the offending spouse
ought to have known better. Different considerations may apply, however, if someone is suffering from ill health. Such a person
may not know what he is doing. He may not be capable of normal rational thought. If he is not, then I would think it harsh and
unfair to say of him that he ought to have known better. I would regard these as highly relevant considerations in any decision as
to whether one spouse has treated the other with cruelty.
If in a particular case it were held that a spouse afflicted by mental ill health had not treated the other spouse with cruelty, it
might nevertheless be the case that the latter was in peril of suffering some injury or further injury from the former. That might
suggest that there could be cases where a spouse needed protection, but could get neither decree of dissolution nor decree of
judicial separation. Such a consideration does not affect the present problem. Relief on the ground that a respondent is of
unsound mind is given by statute. It is, however, only obtainable if the statutory conditions are satisfied. It would not be
justifiable to extend the meaning of the words has treated with cruelty on account of the fact that in a particular case
unsoundness of mind was not of the nature or duration which would form a ground for a petition.
In some of the early reported cases concerning cruelty there is recognition of the need to give protection. The cases further
show that it was not necessary to prove that acts of cruelty were caused by actual malignity. Thus in Kirkman v Kirkman, Sir
William Scott referred to a suggestion that the wifes acts of violence were caused by jealousy and he said ((1807), 1 Hag Con at
p 410):

All the evidence tends to establish, that there was no foundation, in the 1013 conduct of the husband, for feelings of
that nature. If such feelings were entertained, with or without reason, jealousy is a passion producing effects as violent as
any other passion, and there will be the same necessity to provide for the safety and comfort of the individual. If that safety
is endangered by violent and disorderly affections of the mind, it is the same, in its effects, as if it proceeded from mere
malignity alone: it cannot be necessary that, in order to obtain the protection of the court, it should be made to appear to
proceed from malignity.

That the reference to violent and disorderly affections of the mind was not understood as a reference to insanity is, I think,
shown by the later case of Hall v Hall. In that case, after acts of violence had been proved, the court raised the question whether
a decree ought to be made inasmuch as there was strong evidence that the husband was irresponsible for his actions. It was
objected that the question did not arise on the pleadings for it was said that ((1864), 3 Sw & Tr at p 348) such a fact or defence
must be before the court in a known legal manner. The case was adjourned so that further evidence should be obtained and the
Judge Ordinary (Sir J P Wilde) said ((1864), 3 Sw. & Tr. at p 348) such a fact or defence must be before the court in a known
legal manner. The case was adjourned so that further evidence ):

It was faintly, and with great care not to be too explicit, argued that madness would be no answer, even if pleaded.

The Judge Ordinary distinguished sharply between those causes of conduct that do not proceed from madness and those that
do. He said ((1864), 3 Sw & Tr at p 349):

With danger to the wife in view the court does not hold its hand to inquire into motives and causes. The sources of the
husbands conduct are for the most part immaterial.

This was the same thought as that which had been expressed by Sir William Scott in 1810 in the case of Holden v Holden
((1810), 1 Hag Con 453 at p 458): If bitter waters are flowing, it is not necessary to inquire from what source they spring. The
Judge Ordinary in Hall v Hall made the point very plain when he proceeded to say ((1844) 1 Rob Eccl 106 at p 116):

Thus I have no doubt that cruelty does not cease to be a cause of suit if it proceeded from violent and disorderly
affections, as said in one case, or from violence of disposition, want of moral control or eccentricity, as said in another,
or from a liability to become excited in controversy, in the language of a third: but madness, dementia, positive disease
of the mind, this is quite another matter.

The reference to violent and disorderly affections was a reference to Kirkman v Kirkman in 1807, and the reference to
violence of disposition, want of moral control or eccentricity seems to have been a reference to Dysart v Dysart ((1844) 1 Rob
Eccl 106 at p 116).
My lords, in a field in which your lordships are free to direct the right path I would be well content to be guided by the clear
light of this judgment delivered nearly one hundred years ago. Here also is simplicity of expression. If the conduct complained
of proceeded from madness, dementia, or positive disease of the mind it should not be classed as cruelty. Here also is the
justness of this approach explainedh:
________________________________________
h Hall v Hall (1864), 3 Sw & Tr at p 349.

An insane man is likely enough to be dangerous to his wifes personal safety, but the remedy lies in the restraint of the
husband, not the release of the wife. Though the object of this courts interference is safety for the future, its sentence
carries with it some retribution for the past. In either aspect it would be equally unjust to act on the excesses of a
disordered brain:
1014

in the latter, for the insane is not responsible; in the former, for insanity may be cured, and the danger at an end.
It is to be noted that the Judge Ordinary considered ((1864), 3 Sw & Tr at pp 349, 350) that White v White supported his views.
Reference may also be made to Hayward v Hayward ((1858), 1 Sw & Tr 81, see at p 84), in which case in a wifes suit for
restitution the Judge Ordinary said:

If this lady at the time alleged concealed a dagger or a knife intending to do violence to her husband, being insane, and
she turns out to be insane, I find no authority for holding that that is an answer to a claim for restitution of conjugal rights.
A husband is not entitled to turn a lunatic wife out of doors. He may be rather bound to place her in proper custody, under
proper care, but he is not entitled to turn her out of his house. He is less than ever justified in putting her away, if she has
the misfortune to be insane. On the other hand, she insists she is not insane, and if her assertion can be established, she
would be responsible for those acts of violence, and her husband would be justified in refusing to receive her, or in using
force to restrain her.

In the Scottish case of Steuart v Steuart in 1870 it was recognised that the pursuer was not entitled to succeed if the defender was
insane.
My lords, I am clearly of the opinion that in an inquiry whether one spouse has treated the other with cruelty the mental state
of the parties and more particularly that of the former may be highly relevant: I am also clearly of the opinion that a
consideration of the mental state of the spouse whose conduct is complained of may be a deciding factor in reaching a conclusion
that that spouse has not treated the other with cruelty. I pass therefore to consider whether there is any test or any formula or any
set of words or any question by which to measure the extent of the relevancy of the mental state of a respondent.
The answers of the judges in McNaghtens case were given in a criminal case. I do not consider that they were ever intended
to be applied automatically in a consistorial case. Nor do I find it necessary to consider whether they can have any bearing in
civil actions and in claims for compensation. It is to be observed that in Hall v Hall it did not occur to the Judge Ordinary that his
guidance needed the assistance of, or needed to be linked with, the answers which had been given by the judges to the House of
Lords some twenty-one years earlier. At a much later time references to the so-called McNaghten rules appeared in judgments.
The test of those rules or of some of them may be appropriate in particular cases, but I do not consider that they should be used as
a kind of codification of the law relating to insanity in cruelty cases. I observe that in his judgment in White v White Asquith LJ
said ([1949] 2 All ER at p 348; [1950] P at pp 52, 53) that it would not suffice for a respondent to show that he suffered from a
disease of the mind nor even to show that he would not have committed the acts complained of but for such disease but that in
order to avail a respondent ([1949] 2 All ER 339; [1950] the insanity must, at all events, not fall short of such insanity as would
afford a defence to a criminal charge within the McNaghten rules.
My lords, the view that I have formed is that it is undesirable to seek to use any set form of words or any formula by which
to measure whether someone who is mentally afflicted has treated another with cruelty. The McNaghten rules may often be
helpful. If they are being referred to as useful guides, I can see no justification for picking on one of them to the exclusion of
others. To say that no heed should be paid to any consequence of insanity, other than that of not knowing what one is doing,
seems to me to be a wholly unwarrantable limitation. Ultimately, however, the question is a question of fact, which is to be
decided as a fair-minded reasonable jury would decide it. If someone who 1015 in common parlance was plainly mad did some
act while not knowing what he was doing, I do not think that a jury would say that he was guilty of cruelty. Likewise, if someone
who was plainly mad did some outrageous thing and said that he had done it under divine direction, a jury would not be likely to
say that he was guilty of cruelty. A jury would be likely to think as the Judge Ordinary thought in Hall v Hall ((1864), 3 Sw & Tr
at p 349) that it would be unjust to condemn someone whose conduct resulted from the excesses of a disordered brain. They
would be likely to think (as did the Judge Ordinary) that insanity is a form of illness and that a cure may be effected.
My lords, I think that the reasoning which guided their lordships in Breen v Breen (1961 SC 158, see at pp 166, 170) was
much in tune with the reasoning in Hall v Hall. I notice such expression assuch unsoundness of mind as to make it
impossible or at least unreasonable to say that he was guilty of cruelty and The defender was not able to exert his reason to
control his reactions in the normal way. Though I refer to these passages, I am very averse from seeking to select any set of
words which could be regarded as a rigid or complete test. It has been pointed out that in directing juries in criminal cases in
regard to the onus of proof which lies on the prosecution it is the essence and spirit of the matter that must be conveyed and
explained. There is no magic in the mouthing of some phrase or formula. So here. Did the conduct complained of proceed from
and was it caused by madness? Did the conduct result from the excesses of a disordered brain? Was the respondent responsible
for what he did? Did he know what he was doing? Even if he knew what he was doing was he so affected by his abnormal
mental condition that it caused him to act in a way that someone of normal mental condition would not have acted? Was there
such a defect of reason due to disease of the mind that it would be unreasonable and unjust to say that the respondent had treated
the petitioner with cruelty? Were the respondents actions the result of insane delusions due to disease of the mind (see the fourth
question put to the judges in McNaghtens case? Were the actions of the respondent symptoms of, and were they carried out
under, the influence of insanity? Was the respondent labouring under a defect of reason due to disease of the mind so that the
conduct complained of resulted from his inability to control his actions or reactions in a normal way?
My lords, no one of these questions is intended as a test of general application. They are singly and collectively but
pointers. They are some of the questions that may arise in deciding an issue as to cruelty, for in my judgment if certain conduct
can properly and fairy be said to be the definite result of mental illness (which may indeed only be temporary) it would be
contrary to the fitness of things to stigmatise it as cruelty. I can but record my view that a jury would shrink from finding a
respondent guilty of the matrimonial offence of cruelty in a case where they might very colloquially sum up their conclusion in
the phrase the poor creature is mad. I consider that it is very desirable that an inquiry whether a respondent has treated a
petitioner with cruelty should be regarded as an issue of fact uncomplicated as far as possible by questions of law and released
from anchorage to any phrase or formula. I consider that judges will not find it difficult to adjudge whether conduct has ((1864),
3 Sw & Tr 347 at p 349) proceeded from madness, dementia, positive disease of the mind.
On the facts as found by the learned commissioner and recorded in his careful judgment he was, I think, directed by
authority to the conclusion which he reached. Not being bound by direct authority I would reach the same conclusion. The
respondent, who had a bad family history, developed many of the sharply marked characteristics of a schizoid or a paranoid or
both. He had auditory delusions. He heard voices. They told him that his wife was committing adultery with numbers of men.
In consequence he accused his wife. She tried to 1016 reason with him. She could not convince him that there were no voices.
Distressing as it all must have been for the wife, and injurious to her health as it has been found to have been, I cannot think that
it would have been just to find the respondent guilty of cruelty.
I would dismiss the appeal.

LORD HODSON. My Lords, the issue in this case is whether insanity is a defence to a charge of cruelty. The gravamen of the
case presented by the wife was that the husbands repeated unfounded allegations of adultery against her must eventually have
injured her health. The foremost feature of the malady from which the husband suffered was auditory delusions. Voices were
telling him that his wife was unfaithful and that men were in the house. Sometimes he would climb up into the loft to find the
men, and on one occasion he took a knife and went outside to find the persons, whose voices he was hearing, in order to attack
them. In general the wife spoke of her husband as a kind and good man, but for his drinking, which the commissioner did not
regard as something which imperilled the marriage. The commissioner would, but for the insanity of the husband, have found
that he had treated his wife with cruelty and pronounced a decree of divorce, but felt himself constrained to dismiss the petition
because of the husbands insanity. The wife appealed to the Court of Appeal which by a majority held that the defence of insanity
rightly prevailed and on the hearing in that court and before your lordships it was accepted that but for the insanity of the husband
there was no answer to the charge of cruelty. In agreement with my noble and learned friend Lord Reid I am not to be taken as
confirming the finding that on these facts cruelty was established.
I am also in agreement with my noble and learned friend that Parliament cannot have intended by its method of drafting to
give any different meaning to the word cruelty than that it had previously borne in the ecclesiastical courts. For instance, by
the Matrimonial Causes Act, 1857, which first made cruelty (if coupled with adultery) a ground for divorce the word guilty is
used in association with the word cruelty i. The phrase guilty of persistent cruelty still appears in the Matrimonial
Proceedings (Magistrates Courts) Act, 1960, and the word guilty appears in the Divorce (Scotland) Act, 1938. In the
corresponding English legislation the phrase is used has treated the petitioner with cruelty. Parliament must, however, by its
language have recognised cruelty as connoting blameworthiness, and must by the use of the phrase treated with cruelty have
recognised that a deliberate act must be done to the complaining party by his or her spouse before a remedy can be available.
What, then, does cruelty signify in its natural and ordinary meaning, and how was it understood by those great judges to whom
my noble and learned friend refers? I find nothing in their utterances to lead me to suppose that they would regard a madman as
cruel. References to turbulent passion, natural violence of disposition, want of moral control and eccentricity, as examples of the
motives and causes of cruel conduct, are wholly inappropriate to describe what lies behind the actions of those who have lost
their reason through disease of the mind. Rather are they appropriate to describe the motives of sentient beings subject to human
frailties and passions. It is true that those judges were concerned to afford protection for the suffering partner, for in those days
the wife at any rate was in grave need of protection from any husband who took advantage of his superior position before the law.
She was dependent on him for her sustenance and, even when living with her husband, might not find anyone else willing to give
her credit, even for the necessaries of life. When apart from her husband, the position was even more difficult, for there was then
no presumption that she could pledge his credit.
1017
________________________________________
i Matrimonial Causes Act, 1857, s 27, on the ground that since the celebration [of the marriage] the husband has been guilty of
adultery coupled with such cruelty as without adultery would have entitled [the wife] to a divorce a mensa et thoro

This need for protection no doubt was uppermost in the minds of the judges of the ecclesiastical courts for they were able to
enforce their orders for alimony and provide sustenance for the one to whom the remedy of separation by reason of adultery or
cruelty was available. They were not, however, able in the nature of things to provide physical protection, except to a limited
extent, and pointed out that such could only be secured by the confinement of a violent partner in some secure place. The
position was well summarised soon after the passing of the Matrimonial Causes Act, 1857, by Lord Penzance (then Sir J P Wilde)
in Hall v Hall ((1864), 3 Sw & Tr 347 at p 349.), who said:

I have no doubt that cruelty does not cease to be a cause of suit if it proceeded from violent and disorderly affections
as said in one case, or from violence of disposition, want of moral control, or eccentricity, as said in another or from a
liability to become excited in controversy in the language of a third; but madness, dementia, positive disease of the mind,
this is quite another matter. An insane man is likely enough to be dangerous to his wifes personal safety, but the remedy
lies in the restraint of the court, not the release of the wife. Though the object of this courts interference is safety for the
future, its sentence carries with it some retribution for the past.

After the passing of the Matrimonial Causes Act, 1857, there was some discussion in the courts on the availability of a defence of
insanity to a suit founded on cruelty or adultery, which was influenced by the advice given by the judges in 1843 in the
McNaghten case. The language of Lord Esher MR in Hanbury v Hanbury is in line with that of the McNaghten rules, although
he reserved his opinion on the question whether the rules were applicable (cf Yarrow v Yarrow (an adultery case) and the
Hanbury case at first instance where Sir Charles Butt P considered the possibility of defence of insanity to either charge).
The first case in which the McNaghten rules were referred to eo nomine appears to be Astle v Astle, where Henn Collins J
was of opinion on the basis of these rules that the ([1939] 2 All ER at p 970, letter c; [1939] P at p 52.) respondent would not be
held answerable either civilly or criminally for his assaults. There may be different considerations applicable in civil cases
generally, but there is this paralled between divorce cases (which are civil cases) and criminal proceedings, that the standard of
proof is to all intents and purposes the same.
I come now to the case of White v White and to the judgment of Asquith LJ who was unable ([1949] 2 All ER at p 347, letter
h; [1950] P at p 52) to accept the view that insanity could never be a defence, the suit being one based on cruelty, but added
([1949] 2 All ER at p 348, letter a; [1950] P at p 52.) that it was plain that the presence of insanity of some sort or other without
more would not necessarily afford a defence. He went on ([1949] 2 All ER at p 348, letter b; [1950] P at p 52.) to consider what
type or degree of insanity would suffice as a defence, recognising no doubt that, as has been said, it is hard to determine when
twilight becomes darkness. He drew the inference ([1949] 2 All ER at p 348, letter a; [1950] P at p 52.) that to avail as a defence
the insanity must at all events not fall short of such insanity as would afford a defence to a criminal charge within the McNaghten
rules. He said ([1949] 2 All ER at p 348, letter b; [1950] P at p 53.):

In other words, if the actor knows the nature and quality of his acts (that is to say, knows what he is doing and knows
he is doing wrong), the fact that he is suffering from a disease of the mind is immaterial, even if he would not have
committed the acts complained of but for such disease.
1018
Denning LJ ([1949] 2 All ER at p 352; [1950] P at p 60), was of opinion that, subject to the qualification that a point might be
reached when the conduct was so irrational that it was obvious to the injured party that the conduct was not cruelty but was due to
mental disease, insanity was no defence to a charge of cruelty. Bucknill LJ ([1949] 2 All ER at pp 340 et seq.; [1950] P at pp 46
et seq), did not express any concluded opinion. In 1950 my noble and learned friend Lord Pearce decided in Lissack v Lissack
that insanity was no defence, and to this he held there was no qualification. I need not refer to the cases of Swan v Swan (12) and
Palmer v Palmer, save to say that the view put forward by Asquith LJ in Whites case majority of the Court of Appeal in the
instant case.
Having heard the full argument which has been addressed to your lordships and with all deference to those who take a
different view, I cannot avoid the conclusion that the word cruelty of itself involves an implication of blame-worthiness.
Indeed I agree with Asquith LJ (([1949] 2 All ER at p 347; [1950] P at p 51), in thinking that, if insanity is immaterial, it would
appear to follow that the intention of the aggressor is irrelevant, for the act must then be looked at from the point of view of the
victim, and one looks no further than that. If cruelty is to be excused by insanity, that is because intention is relevant and the
effect of insanity is to negative intention. True it is that there must be degrees of mental illness falling short of insanity, as there
are of other maladies from which the person charged with cruelty may be suffering. In such cases allowance must be made for
the condition of the sick person, as I think all would agree, but once the line is crossed and a condition of complete insanity is
reached the question is not what allowance must be made, but whether a defence has been made out. It is in this way that I
understand the observation of Asquith LJ ([1949] 2 All ER at p 348, letter a, [1950] P at p 52), to the effect that the presence of
insanity of some sort or other will not necessarily afford a defence.
I do not think it necessary to discuss further the much discussed topic of intention which may often be inferred from
inexcusable conduct or unwarrantable indifference. I do not draw any distinction between these two things. If one pushes
someone who cannot swim into deep water, one may expect to be called cruel, and no less so if one sees that person in the water
and ignores his or her cries for help. If insanity is to be an excuse for conduct either positive or negative it must in my opinion be
a complete answer, just as inevitable accident must be an answer. I understood counsel for the appellant to concede that accident
would be a defence, but he did not seek to explain why the same should not be the case where the person accused of cruelty is
insane. From the victims point of view the pain is the same, whether suffering is caused by accident or by the act of a madman.
I do not accept the distinction which the appellant sought to draw between acts objectively cruel and acts or omissions which
only become cruel because of the intention proved, which causes these acts or omissions to be cruel or makes them more likely to
inflict suffering. In the one case it is said insanity is no answer, although in the second case it would or might be relevant. This is
to create an artificial subdivision of cruelty, which I think is unjustifiable, creating two classes of cruel persons, those who are
violent and those who have been called mentally cruel. This division is a convenient one perhaps as a figure of speech, but the
essential elements of cruelty do not differ whatever label is attached.
I do not attempt to define cruelty but would assert that it involves mercilessness and delight in or indifference to pain and
suffering. These qualities are not 1019 present in an insane person, but they may be present in one who is so self-centered and
lacking in control that he behaves cruelly. I get no assistance from the use of the word cruel in a rhetorical or picturesque
sense. An adjective is often used to indicate a quality of the thing described. Thus the sea and fate may be called, as Donovan LJ
noticed ([1962] 3 All ER at p 452, letter h; [1963] P at p 233), cruel, for this adjective is apt to show that each is without mercy.
But to say of a man that he is merciless is to stigmatise him, and cannot be justified if he is out of his mind. Anyone who has had
any experience of hearing in our courts the unhappy disputes between spouses when a charge of cruelty is involved on one side or
the other will be familiar with the bitterness aroused and the resentment felt at the accusation in nearly every case.
When considering the degree of insanity which must be established in order to furnish an answer to the accusation there are
no doubt great difficulties. The so-called McNaghten rules at least have the merit of simplicity, but I recognise that there is no
obvious justification other than that of convenience for their use in any particular class of civil case, and I would not seek to be
bound by any form of words. The first branch of the rules as applied to cruelty has received a wider measure of acceptance than
the second, and no doubt is the easier to apply, for if a man is unconscious of what he is doing he will be the more readily
excused. This test goes some way towards recognising the subjective element in cruelty. I cannot think, however, that it goes far
enough, and it seems that the second branch or its equivalent is required to cover the case of one who is conscious of what he is
doing but through disease of the mind does not know that it is wrong. I think that this is required to cover those cases where,
through his insanity, a person has no power of moral judgment. After all, a man is supposed to be different from a brute beast in
that very respect. The fox which robs a hen roost presumably knows perfectly well what he is doing, but he is following his
natural instincts and it is just as absurd to describe him as cruel as to say that he has committed larceny. It is to me equally
objectionable to apply the adjective cruel to one who, through disease of the mind, does not know that what he is doing is
wrong.
I appreciate that the difficulty in applying the second branch of the rules has been enhanced by a difference of opinion in the
criminal courts. In this country in R v Windle the Court of Criminal Appeal held that in relation to a criminal charge the word
wrong meant contrary to law. As Willmer LJ pointed out ([1962] 3 All ER at p 449; [1963] P at p 227) this is inapt in
relation to an offence such as cruelty. In the High Court of Australia, in Stapleton v R, the test was thought to be whether the
accused had the capacity to distinguish right from wrong according to the standard adopted by reasonable men. Sir Owen Dixon
CJ thought that this conclusion followed from the words used by the judges in formulating their advice in the McNaghten case. I
agree with Davies LJ ([1962] 3 All ER at p 455, letter d; [1963] P at p 238), who thought that the use of the word wrong by
itself is sufficient.
A further difficulty has been felt, as is said, by reason of the unscientific nature of the test. I appreciate that disease of the
mind is a topic on which scientific evidence is commonly required in order to assist judge or jury, but the essential issue is one of
fact, on which juries in civil and criminal cases are required to give a decision. There must be many cases in which the rules have
been applied liberally in favour of the accused person, but I find it difficult to suggest a form of words which would give any
better assistance to a jury of laymen.
In Scotland the question whether insanity is a defence to a charge of cruelty has not always been answered in the same way
and no question of the direct application of the McNaghten rules arises, but the conclusion reached by the 1020 Court of Session
in Breen v Breen is summarised in the language of Lord Patrick as follows (1961 SC at p 182.):

In principle, no blame can be attached to a man who at the time of the acts in question was by reason of alienation of
mind disabled from coming to a rational decision in regard to the acts.

Lord Strachan agreed with this opinion and added (1961 SC at p 197.):

The defender was not able to exert his reason to control his reactions in the normal way. I therefore hold that he was
not responsible for his violent acts and that he cannot be held guilty of cruelty.

I agree that the Scottish Act speaks of guilt but, as I have already said, I do not see that the presence or absence of that word,
which appears in some Acts of Parliament and not in others in association with the word cruelty, can affect the meaning of the
latter word or do more than show that the legislature has thought that guilt was an appropriate word to use in association with
cruelty.
A subsidiary argument was advanced, that it would be wrong to allow the word cruelty to be associated with blame, since
the consequence would be that in such a case as the present there would, unless and until a divorce could be obtained on the
ground of insanity under the Matrimonial Causes Act, 1950, s 1(1)(d), (2), as amended by the Divorce (Insanity and Desertion)
Act, 1958, be no defence to a suit by the husband for restitution of conjugal rights and no means of obtaining maintenance for the
wife so long as she refused to live with her husband. Orders for restitution of conjugal rights being no longer enforced, the only
relevant consequence is the financial one in the event of circumstances arising in which the husband might be in a position to
maintain his wife. In such a case the principles explained in Lilley v Lilley ([1959] 3 All ER 283; [1960] P at p 169.) would
apply, and the question would be whether a de facto separation was imposed on the wife by force of circumstances.
I would dismiss the appeal.

LORD PEARCE. My Lords, the problem of insanity has caused doubt and difficulty in every branch of the law. In the case
before your lordships it is conceded that the respondent knew the nature and quality of his acts and that, unless the second limb of
the McNaghten rules or some equivalent rule applies, the wife is entitled to succeed. It is also conceded that if it does apply, she
is not entitled to relief.
A criminal court cannot disregard the diseased mental condition of the man whom it is proposing to punish. It has a duty to
punish; but it can grant to the insane any immunity that is consistent with that duty, and no individual hardship is thereby created.
The criminal courts in 1843 adopted the McNaghten rules as the best solution to their problem. But the working of those rules is
dependent on medical opinions, which change with advancing knowledge, and in practice they have given rise to doubt and
dissatisfaction.
In the common law courts the problem is different. Immunity to an insane person can only be granted at the expense of an
injured party, and a balance of hardship at once arises. In 1616 it was said that if a lunatick hurt a man he should be answerable
in trespass (Weaver v Ward). Professor Fleming in his book, The Law of Torts (2nd edn), at p 24, says:

Divergent views have been expressed on whether this stringent view should now be modified in conformity with the
change in theory regarding liability for trespass (see SALMOND, STREET, WINFIELD, POLLOCK).

He points out that the New Zealand court in Donaghy v Brennan) favoured the more stringent view but in New South Wales
(White v Pile immunity was 1021 extended to a defendant in terms of the McNaghten rules. In Morris v Marsden, followed in
Beals v Hayward, Stable J steered a course between these extremes and awarded damages for assault against a defendant of
unsound mind, deriving sufficient intent on the defendants part from the fact that his mind directed the blows he struck.
Professor Fleming in The Law of Torts (2nd Edn), at p 25, says:

This conclusion is shared by the better reasoned American cases and has the merit of neither giving undue scope to the
defence of insanity, nor departing from the modern basis of liability for trespass which as we have seen requires intentional
or negligent conduct by the actor.

(see also Salmond on Torts (12th Edn), p 75.)


In contract, however, a different view is taken. The general rule appears to be that a person of unsound mind is bound by his
contract even although his mental condition was such that he did not understand what he was doing, unless he can show that the
other party ws aware of his incapacity. If these two conditions are satisfied the contract is voidable at his option. (See Cheshire
& Fifoot on the Law of Contract (2nd Edn), p 354; Chitty on Contracts (22nd Edn), p 469; and cases there cited.) Lord Esher
MR said in Imperial Loan Co v Stone ([1892] 1 QB 599 at p 601):

I shall not try to go through the cases bearing on the subject; but what I am about to state appears to me to be the result
of all the cases. When a person enters into a contract and afterwards alleges that he was so insane at the time that he did
not know what he was doing and proves the allegation, the contract is binding on him in every respect whether it is
executor or executed as if he had been sane when he made it, unless he can prove further that the person with whom he
contracted knew him to be insane so as not to be capable of understanding what he was about.

If the grounds for setting aside a contract on the ground of insanity were founded on the theory that the insane mans acts were
not really his acts, one would expect the contract to be void not voidable and the knowledge of the other party would seem
irrelevant. But presumably the compromise was evolved in the light of practical commercial considerations.
Thus the common law courts afford no clear or uniform solution.
The divorce court has more similarity to the common law courts, which grant relief, than to the criminal courts which
merely inflict punishment. But it is peculiar in that it deals with the ties that bind two persons together and make it their duty to
live with one another. I find, therefore, no great help in the somewhat uneasy compromises at which the criminal and common
law courts have arrived.
I cannot accept the argument that divorce is partly punitive and should, therefore, look to the criminal law for guidance. The
dissolution or permanent interruption of a union, which is in theory life-long and indissoluble, cannot be justified by any logic.
But the frailties of humanity produce various situations which demand practical relief and the divorce Acts owe their origin to a
merciful appreciation of that demand. Any extension of the area of relief has always been advocated on the ground that there are
situations of hardship that must be alleviated, and has been contested on the ground that to extend relief would create
corresponding hardship to the other party and would weaken the important and sacred institution of matrimony. Never does an
intention to punish enter into the debate; nor is an extension of the grounds of divorce ever advocated or opposed on the ground
that it will extend the area of punishment of errant spouses. It is true that the divorce law of England, following the ecclesiastical
law, is founded on the concept of the matrimonial offence. That concept is used 1022 to give some justification for breaking an
indissoluble union against the will of the offending party. But in the divorce Acts there is nothing that suggests an intention to
punish. If relief is to be granted, each party alike must forfeit the status of matrimony or, in the case of separation, part of that
status. One party loses it voluntarily, the other involuntarily, but the loss is inherent in the granting of relief. A guilty husband
may have to pay maintenance but only in so far as necessity of fairness demands on a consideration of the means of both parties;
and punitive considerations are excluded. Even when a co-respondent pays damages for breaking up a marriage, an apt occasion
for punishment, the damages are confined to compensation for the loss suffered by the husband and are not punitive. Costs are
no more punitive than in any other branch of the law; they are in fact less punitive in that for economic reasons a husband who
has done no wrong may be ordered to pay the costs of his erring wife, and rarely is any order made against her however bad her
conduct has been. So far as a decree contains some retribution for the past the same may be said of a judgment in tort for
damages; but such a judgment is universally distasteful to a defendant whereas a decree of divorce is earnestly desired by a
large proportion of respondents. I do not find anything in the divorce Acts to justify a theory that the law is intended to punish.
They appear to intend a practical alleviation of intolerable situations with as little hardship as may be upon the party against
whom relief is sought.
Certainly the provisions in the Act of 1937, which first allowed divorce on the ground of cruelty alone j, was enacted in order
to alleviate the hardship to respondents and petitioners alike of being tied for life to a marriage that had broken down. It gave as
an alternative to the long-existing remedy of judicial separation on the ground of cruelty, the opportunity of divorce which would
allow to both parties the freedom to remake their lives. There was, I think, a deliberate omission of the word guilty in s 2 of the
Act of 1937 (now s 1 of the Act of 1950). The new section was breaking away from the old idea of insistence on a matrimonial
offence in that it was adding incurable insanity as a ground for divorce. But the words guilty of adultery in the previous Act
were no more a guide to penal intention than are the words guilty of negligence, tortfeasor or, in Scotland, delinquent. I
read the words treated with cruelty as being no more than a convenient description of a situation where one party has treated
another, and that other has been treated with cruelty. I do not see in those words any indication that the state of mind of the actor
was to be a paramount consideration
________________________________________
j See the Supreme Court of Judicature (Consolidation) Act, 1925, s 176 (c), as substituted by the Matrimonial Causes Act, 1937, s 2.

It is to the word cruelty as used in the section to which one must look for guidance. For all purposes under the Act it must,
in my opinion, have the same meaning. The word had had a long previous history which cannot be disregarded. There are many
cases dealing with cruelty throughout the nineteenth century, both before and after the passing of the Act of 1857. Those cases
show that judges looked at the conduct itself regardless of motive. Spouses had a duty to cohabit unless they were separated by
decree. Where one spouse was treated by the other with cruelty it was therefore necessary to relieve the ill-treated spouse by
granting a divorce a mensa et thoro, or, later, a decree of separation. Where that cruelty sprang from a disordered mind, such
relief was all the more necessary unless (which is not always the case) it could be certain that the insane spouse would be
confined in some institution. On the other hand there are passages that show a reluctance to hold a person responsible for acts
that were caused solely by a demented mind. In 1807 in Kirkman v Kirkman, Lord Stowell when referring to the necessity to
provide for the safety and comfort of the individual said ((1807), 1 Hag Con at p 410):
1023
If that safety is endangered by violent and disorderly affections of the mind, it is the same, in its effects, as if it
proceeded from mere malignity alone; it cannot be necessary that in order to obtain the protection of the court, it should be
made to appear to proceed from mere malignity.

He was thus equating cruel acts caused by a disordered mind to acts of deliberate intentional cruelty. In Holden v Holden he said
((1810), 1 Hag Con at p 458):

the court has had frequent occasion to observe that eveything is, in legal construction, saevitia, which tends to
bodily harm, and, in that manner, renders cohabitation unsafe; whenever there is a tendency only to bodily mischief, it is a
peril from which the wife must be protected; because it is unsafe for her to continue in the discharge of her conjugal duties;
and to enforce that obligation upon her might endanger her security and perhaps her life. It is not necessary in determining
this point to inquire from what motive such treatment proceeds. It may be from turbulent passion, or sometimes from
causes which are not inconsistent with affection, and are indeed often connected with it, as the passion of jealously. If
bitter waters are flowing, it is not necessary to inquire from what source they spring. If the passions of the husband are so
much out of his own control, as that it is inconsistent with the personal safety of the wife to continue in his society, it is
immaterial from what provocation such violence originated.

In Dysart v Dysart in 1844 Dr Lushington said ((1844), 1 Rob Eccl at p 116):


When I find conduct towards a wife likely to prove dangerous to her safety, but not in other cases, I shall consider it
within my cognizance, whatever may have been the cause thereof, whether having arisen from natural violence of the
desposition, from want of moral control, or from eccentricity. It is for me to consider the conduct itself, and its probable
consequences; the motives and causes cannot hold the hand of the court, unless the wife be to blame, which is a wholly
different consideration. In plainer words, even if I were satisfied that conduct dangerous in itself arose from morbid
feelings, out of the control of the husband, I must act, if the danger exists, though it is not my province to inquire into or
ascertain such cause.

Thus he is taking a purely objective view of the cruel acts regardless of any intention to be cruel.
In 1858 in Curtis v Curtis Sir Cresswell Cresswell after referring to those observations of Dr Lushington said ((1858), 1 Sw
& Tr at p 213):

If indeed an act of violence were committed under the influence of an acute disorder, such as brain fever; and it were
made clear that, the disorder having been subdued, there was no danger of a recurrence of such acts, the case would be
different. But if the result of such diseases has been a new condition of the brain rendering the party liable to fits of
ungovernable passion which would be dangerous to a wife, then undoubtedly this court is bound to emancipate her from
such peril.

Thus if cruelty came from lack of control due to disease of the brain, he would order a separation. He also, like Lord Stowell and
Dr Lushington, was looking at the acts themselves, regardless of any intention to be cruel. Earlier in the same year in Hayward v
Hayward he had expressed the view that though a lunatic wife threatens her husband with violence he is not entitled to turn her
outdoors but may be rather bound to place her in an institution. These views are not necessarily inconsistent if one takes into
account the practical 1024 difference between the circumstances of a wife and those of a husband. In White v White, Sir
Cresswell Cresswell made an order against a wife who had on several occasions been in confinement as an insane person and
concluded ((1859), 1 Sw & Tr at p 593):

As far as I can judge from the evidence given, I suppose those attacks to have been the consequence, and not the
cause, of her intemperance and the quarrels with her husband. The assaults committed upon him were not proved to have
been productive of any serious bodily injury; but where a woman, either from the effects of drinking or any other cause, is
entirely without power of controlling her passion, and in such a state of mind is in the habit of assaulting her husband, it is
impossible to say that he is not in such danger of bodily injury as entitles him to the protection of the court.

Through all these cases runs a robust and practical approach. The judges are concerned to look at the conduct itself and its
probable consequences to see whether it, the conduct, is cruel without searching into the motives or intentions that gave rise to
it. On occasion an impression is conveyed that there may be a degree of mania that may be a defence, but that impression never
crystallises in a successful defence on that ground.
In 1864 in Hall v Hall Sir J P Wilde said ((1864), 3 Sw & Tr at p 349):

With danger to the wife in view, the court does not hold its hand to inquire into motives and causes. The sources of
the husbands conduct are, for the most part, immaterial. Thus I have no doubt that cruelty does not cease to be a cause of
suit if it proceeded from violent and disorderly affections as said in one case, or from violence of disposition, want of
moral control, or eccentricity as said in another, or from a liability to become excited in controversy in the language of a
third; but madness, dementia, positive disease of the mind, this is quite another matter. An insane man is likely enough to
be dangerous to his wifes personal safety, but the remedy lies in the restraint of the husband not the release of the wife.
Though the object of the courts interference is safety for the future, its sentence carries with it some retribution for the
past. In either aspect it would be equally unjust to act on the excesses of a disordered brain: in the latter, for the insane are
not responsible; in the former for insanity may be cured and the danger at an end.

In Hanbury v Hanbury, however, Butt P in charging a jury said ([1892] P at pp 224, 225.):

Assuming for a moment that these attacks were not brought on the respondent by his own self-indulgence, assuming
that they were the result of hereditary disease, I should still be disposed to hold that acts of cruelty committed in one of
these fits of mania would entitle the wife to the remedy for which she asksseparation from her husband. If the mania is
intermittent and recurrent, the husband is entitled to go home when he recovers from time to timethe wife cannot refuse
him admission to the conjugal home; and if the mania is likely to recur accompanied with violence which would place the
wife in peril, the ordinary protection which she is supposed to obtain by proceedings in lunacy is a delusion, because it
does not protect her against the return home of her husband, who is liable at any moment to become a lunatic.

Again one finds the practical approach rather than the theoretical. The Court of Appeal dismissed the husbands appeal without
calling on the other side. Lord Esher MR (with whom Lindley LJ and Kay LJ concurred) left open the larger question which the
President touched on but did not decide ((1892), 8 TLR at p 560.):
1025
namely, whether, even if the respondents mind hd been such that he did not know the nature of what he was doing
or that he was doing wrong, the petitioner would or would not be entitled to a divorce.

Lord Esher there spoke the language of the McNaghten rules, but he was not, I think, intending to lay down the proposition that if
a defence of insanity was applicable to a divorce suit, it would necessarily be governed by those rules. (See also Yarrow v
Yarrow.)
In more recent years in White v White, the Court of Appeal by a majority held that mere insanity as such was no defence to a
petition based on cruelty, and that any defence on the ground of insanity, if it was to succeed, must at all events not fall short of
the McNaghten rules. Denning LJ however, held that insanity would never be a defence to such a petition, unless at any rate the
petitioner knew at the time at which the respondent committed the acts, that his conduct was due to mental disease; such
knowledge was an element that had not hitherto been introduced into this particular problem, and it may be that it was suggested
by analogy with cases in contract. In Lissack v Lissack it was held in reliance on the older cases to which I have referred and the
observation of Denning LJ in White v White ([1949] 2 All ER at p 352; [1950] P at p 60.) that insanity was no defence. In that
case a man deliberately killed the child of the marriage knowing (but regretting), as the correspondence showed, that it caused
agony to the wife. In Swan v Swan, however, the Court of Appeal held for the first time that the first limb of the McNaghten
rules constituted a defence to a charge of cruelty, but they granted a decree on the ground that a previous act of cruelty had not
been condoned. They were in doubt as to the second limb, and conflicting opinions were expressed obiter. In Palmer v Palmer
obiter opinions were expressed that the second limb constituted a defence. The ensuing grave difficulties in its application are
shown in Sofaer v Sofaer. Finally, in the present case for the first time it was held as the ratio decidendi by a majority of the
Court of Appeal that the second limb applies. Thus the shackles of the McNaghten rules, which have caused so much difficulty
in criminal cases, have been fastened on to divorce suits at a time when the criminal courts are emerging from their confinement.
Your lordships House has now to consider for the first time to what extent, if at all, insanity provides a defence to a petition
based on cruelty.
It is noteworthy that the present suit is the first reported case in the whole history of matrimonial law in which a respondent
has ever succeeded on such a defence.
The practical considerations, which clearly weighed so heavily with the judges of the nineteenth century, may be outlined as
follows: They are more obvious in the case of a wife petitioner but similar considerations (apart from those of finance) apply to a
husband. The case of the wife whose exceptionally strong and loving character is able to regard endurance of the cruelty
resulting from the husbands mental illness as part of the duty which she undertook for better, for worse, need not be considered.
She will not seek relief from the court and no question of any defence will arise. The less robust and more normal wife is in an
unenviable position. The fact that cruelty comes from mental disorder may, to a very few, make it more tolerable. To the
majority it will increase their apprehensions, since there is no limit to which it might not go. If the husband is sufficiently insane,
and is therefore to be given immunity for his acts of cruelty 1026(and also in consequence for his acts of constructive desertion),
but yet is not so continuously insane as to be detained permanently, it is at the very least doubtful, as the law now stands, if she
has any reasonable cause within the meaning of the acts for not living with him. If the wife leaves, she will probably be in
desertion and without home or maintenance. It may be that the court has a discretion and need not make a restitution order
against her, but the sole authority for this is the case of Timmins v Timmins in which there is a powerful dissenting judgment by
Hodson LJ (as my noble and learned friend then was). It may be that she can claim that she is separated by necessity as a
sailors wife is during his duties at sea (see Lilley v Lilley ([1959] 3 All ER 283 at p 289, letter d; cf p 291, letter f [1960] P at p
169) and can therefore obtain maintenance. But this is a difficult matter and dependends on her paying lip service at least to a
willingness to return whenever it may be safe to do so, whereas the truth may well be that after her ordeal she is not prepared to
do so in any event. Under the Guardianship of Infants Act, 1925, she can obtain custody of the children and maintenance for
them, but without maintenance for herself she would not be able to support a home. She may, as sometimes happens in these
cases, find some other man to help and support her and her children. She will then live with him in the hope that she may one
day obtain release under s 1 (d) of the Matrimonial Causes Act, 1950, asking the court for the exercise of its discretion, which in
my experience of such cases has never been refused. Her position is thus unhappy and at the best very precarious. There is no
great alleviation in the fact that in five years she may possibly obtain a divorce on the ground of incurable insanity. The husband
may not be pronounced incurable or there may be interruptions in his period of detention that will defeat a petition. Attempts by
the doctors to cure the patient may well lead to such interruptions and in their attempts any consequent hardship to the wife will
rightly be disregarded. Moreover, to a wife so placed five years will seem a long time.
I accept that in practice, perhaps in the majority of cases were serious cruelty has been inflicted, the respondent will by the
time of the trial be under detention and that in upwards of five years the petitioner may possibly be able to obtain a divorce under
s 1 (d). But if insanity be a defence, it cannot be waived in cases where it creates purposeless hardship. The judge must
investigate it, even if it is not raised, and must, if the defence appears good, refuse a decree to a petitioner however long
suffering. The defence will normally be in the hands of the Official Solicitor and he will be bound to plead insanity and call
evidence in support of it where it appears to be the cause of the cruelty.
Against these hardships on the wifes side one must balance the hardships of the husband. These are indeed heavy. He will
lose his marriage to a wife whom he may love. But that is a misfortune which ex hypothesi will fall on him in any event, and
does not come into these scales. The question here is whether it is really a greater hardship to be severed from her by divorce
than to be tied to her in a marriage which this wife finds intolerable, which makes this wife ill, and from which she wishes to be
free. In my opinion it is not. The real hardship to the husband would be that he was found guilty of cruelty when, apart from his
illness, he may be a kindly man. It is fairly argued that he should not be asked to bear that stigma. But it must be remembered
that in the majority of cases where he continues under treatment it will mean little or nothing to him. In the cases where he
recovers it is seldom that those who know of the decree will not also know of the illness which in the minds of decent people will
obliterate all blame.
In my opinion the frequent hardship to a petitioner so greatly exceeds the more infrequent hardship to a respondent that the
practical social considerations speak 1027 strongly against insanity as a defence to cruelty. It was for that reason that the report
of the Royal Commission on Marriage and Divorce (195155) while admitting that k whichever course is adopted there will be
some hardship recommended that insanity should not be a good defence to a charge of cruelty in matrimonial proceedings.
Unless, therefore, there is a necessary implication from the words of the Act treated with cruelty or from some overriding legal
principle in the light of which they have been enacted, I would not impose on those words either the gloss intending to be cruel
or knowing that it was wrong or even the gloss intending to do the act which was in fact cruel.
________________________________________
k Cmd 9678, para 256.

I am of opinion that the first gloss is not necessary or justifiable. Section 2(c) of the Matrimonial Causes Act, 1937 (now s
1(1)(c) of the Act of 1950) was enacted to alleviate a certain kind of intolerable situation. The Act of 1937 did not introduce any
alteration of the law of cruelty previously in force (see Jamieson v Jamieson). For the reasons set out by the majority in the case
of Gollins v Gollins) neither intention to hurt nor knowledge that the act done is wrong or hurtful is an essential ingredient.
I agree with Donovan LJ, when he said in the present case in the Court of Appeal ([1962] 3 All ER at p 452; [1963] P at p
233):

If Parliament had intended that the defence put forward in the present case should be a valid defence to a petition for
divorce based on cruelty, it would have been simple so to provide, and I should have expected it to have been done in
explicit terms. I cannot think that Parliament would have left it to be implied from the mere words has treated the
petitioner with cruelty In particular, in the case of a man who intends to do something and does it knowing what he is
doing, the word treated does not, in my opinion, imply that he must be able to form a sound judgment upon the question
whether his act is right or wrong. If one looks no further than the words of the statute, I think they afford the husband no
defence. The word cruelty of itself does not assist in this respect. It connotes acts which give unnecessary pain to others
or which are savage or inhuman or merciless, but it certainly does not carry with it the implication that the doer is
conscious that his acts are wrong.

I therefore see no justification for applying the second limb of the McNaghten rules.
Moreover in my opinion insanity should not constitute a defence to suits based on cruelty, even when a man did not know
the nature of his acts. Suppose that an insane man inflicted some cruel torture on his wife not knowing what he was doing. To
the question did he treat his wife with cruelty? no jury would, I venture to think, answer No. Their answer would be Yes,
but they would add a rider that the man was insane at the time and did not know what he was doing. The first part of the answer
would appear to justify the court in giving a decree unless the rider, on an objective view of the circumstances and parties,
removed the act from the area of cruelty.
The argument for holding that a man should not be held to have treated his wife with cruelty if he did not know what he was
doing has an attractive simplicity. But so to hold would create a dividing line which in practice is not easy to apply (even with
medical help), which will at times make the courts powerless to help when help is most needed, and which will cause more
hardship than it alleviates. It is not the dividing line which has been drawn in criminal cases, nor is it that which has been drawn
in cases of contract. It is that which has, after 1028 much doubt, been drawn in cases of tort. For divorce cases it has little
practical justification.
Every man is to some extent at the mercy of his temperament. If one speaks in terms of culpability, there is not much
greater blame on the man who is cruel because he was born without the capacity for self-restraint, or with a sadistic nature, than
on the man who has lost the capacity for self-restraint through disease of the mind. Yet the courts give relief for cruel acts when
innate lack of self-control, or even lack of self-control due to disease, drives men inexorably to cruel conduct. So much is clear
from the earlier cases to which I have referred. And inevitably it must be so if the court is to exercise its duty of protection
between spouses who are otherwise under a duty to live with one another. It may be that to know all is to pardon all; but the
court cannot act on that maxim. If then a decree on the ground of cruelty can be granted against the man who is driven by the
impulse of a diseased brain, because the practical nature of the courts function demands it, why should there be a line, shared
only by the law of tort, which puts those who do not know their acts into a different class from those who cannot avoid their acts?
To say that the acts in the former class are not really their acts does not justify such a distinction, since the same may be said also
of the acts in the latter class. The distinction is one of sentiment rather than logic. Since there is no uniform legal principal that
compels such a distinction, I see no reason why this House should impose it.
In my opinion insanity should, like temperament, and other circumstances, be one of the factors that may be taken into
account in deciding whether a wife is entitled to relief. Where therefore the conduct in question is such that it would not amount
to cruelty in the absence of an actual intention to hurt, an insane man who could form no such intention would not be held to have
treated his wife with cruelty. Where, however the conduct would be held to be cruelty regardless of motive or intention to be
cruel, insanity should not bar relief.
I would allow the appeal.

Appeal allowed.

Solicitors: Collyer-Bristow & Co agents for D Granville-West, Chivers & Dunford, Pontypool (for the appellant); Sidney
Pearlman agent for Athan Morgan & Shibko, Cardiff (for the respondent).

C G Leonard Esq Barrister.


1029
[1963] 2 All ER 1030

Inland Revenue Commissioners v Jamieson


TAXATION; Surtax

HOUSE OF LORDS
LORD REID, LORD JENKINS, LORD HODSON, LORD GUEST, LORD PEARCE
6, 7, 8 MAY, 20 JUNE 1963

Surtax Settlement Power to revoke or otherwise determine settlement Special power of appointment Determination
by the act of any person Provide for the determination of the settlement Whether appointment of whole settled fund to one
beneficiary sui juris would be a determination of the settlement Income Tax Act, 1952 (15 & 16 Geo 6 & 1 Eliz 2 c 10), s 399
(b), proviso (ii).

By a settlement dated 9 August 1950, and irrevocable according to its tenor, the taxpayer settled funds on trusts for his issue and
their wives or husbands as the trustees should appoint by deed, and subject to any such appointment on trust for all his children at
certain ages or times. The taxpayer had, at all material times, three children, all of whom were infants. The income of the settled
funds had been accumulated. He was assessed to surtax in respect of the income of the settled funds for the years 195556,
195657 and 195758 on the ground that the income was to be treated as his by virtue of s 397 a, s 398a and s 399a (b) of the
Income Tax Act, 1952, the settlement being one whose terms provide for the determination of the settlement by the act
of any person within s 399 (b).
________________________________________
a The relevant terms of s 397, s 398 and s 399 are set out at p 1035, letters b to h, post.

Held An appointment made in exercise of the special power of appointment and in favour of, eg, a husband or wife of a
grandchild of the settlor, appointing the settled fund absolutely to the appointee, would bring about a determination of the
settlement by the act of the appointor, and, as the settlement contained the special power of appointment, it did provide for
such determination; accordingly the settlement fell within s 399 (b) of the Income Tax Act, 1952 (not being excluded by proviso
(ii) to s 399), and thus the income of the settled funds was not within the exemption conferred by s 398(2) for irrevocable
settlements, and was assessable for the years in question, 195556 to 195758, as the taxpayers income (see p 1033, letters f and
g, p 1034, letters c and h, p 1036, letter h, and p 1038, letters c and e, post).
Jenkins v Inland Revenue Comrs, ([1944] 2 All ER 491) disapproved.
Decision of the Court Of Appeal (sub nom Jamieson v Inland Revenue Comrs, [1962] 2 All ER 321) reversed.

Notes
The decision in the present case was on the terms of s 397(1), s 398(2) and s 399 (b) and proviso (ii) of the Income Tax Act,
1952, before they were amended by the Finance Act, 1958, s 20(2), (5). These amendments would have excepted the settlement
on grounds immaterial to the point decided (see p 1032, letter F, post).
As to irrevocable settlements for purposes of surtax liability, see 20 Halsburys Laws (3rd Edn) 575, 576, para 1119; and for
cases on the subject, see 28 Digest (Repl) 278291, 12391278.
For the Income Tax Act, 1952, s 397, s 398(2) and s 399, see 31 Halsburys Statutes (2nd Edn) 376378; and for the Finance
Act, 1958, s 20, see 38 Halsburys Statutes (2nd Edn) 497, 4.98.

Cases referred to in opinions


Inland Revenue Comrs v Hamilton-Russell Exors [1943] 1 All ER 474, sub nom Hamilton-Russell Exors v Inland Revenue
Comrs, 25 Tax Cas 200, 28 Digest (Repl) 348, 1540.
Jenkins v Inland Revenue Comrs [1944] 2 All ER 491, 171 LT 355, 26 Tax Cas 265, 28 Digest (Repl) 281, 1248.
Kenmare (Countess) v Inland Revenue Comrs, [1957] 3 All ER 33, [1958] AC 267, 37 Tax Cas 383, 406, [1957] 3 WLR 461, 28
Digest 293, 1287.
Muir or Williams v Muir [1943] AC 468.
1030

Appeal
This was an appeal by the Commissioners of Inland Revenue from an order of the Court of Appeal (Sellers, Donovan and
Pearson LJJ), dated 27 March 1962, and reported [1962] 2 All ER 321, reversing an order, dated 13 July 1961, whereby Plowman
J dismissed an appeal by way of Case Stated of the respondent, Major D A Jamieson, VC, against a determination of the Special
Commissioners relating to assessments to surtax made on the respondent for the years 195556, 195657 and 195758. The
facts, and the question arising, are set out in the opinion of Lord Reid.

Sir Milner Holland QC, A S Orr QC, E Blanshard Stamp and J R Phillips for the appellants, the Crown.
Heyworth Talbot QC and Arthur Bagnall QC for the respondent, the taxpayer.

Their Lordships took time for consideration

20 June 1963. The following opinions were delivered.

LORD REID. My Lords, this appeal relates to three assessments to surtax made on the respondent taxpayer for the years 1955
56, 195657 and 195758. The assessments are in respect of income of funds settled by him in 1950. Under the settlement the
taxpayer had no interest in the settled funds and could never take any interest in them; but elaborate provisions were enacted in
1936 laying down the conditions in which income from settled funds must be treated as income of the settlor for tax purposes.
Those provisions now form s 397, s 398 and s 399 of the Income Tax Act, 1952, and the question is whether those provisions
apply to this case. The Special Commissioners and Plowman J held that they do, but the Court of Appeal has held that they do
not.
The relevant part of the settlement is cl 3 which is in these terms:

3. The trustees shall stand possessed of the said sum of ten thousand pounds and of the investments and property for
the time being representing the same (hereinafter called the trust fund) and of the income thereof upon the following
trusts: (1) In trust for all or such one or more exclusively of the others or other of the following class of persons (that is to
say) the issue (including children hereafter to be born and issue more remote than children) of the settlor and the wives
husbands widows or widowers (whether or not remarried) of such issue in such shares if more than one and either
absolutely or for such successive or other interests or with such trusts and provisions for their respective benefit at the
discretion of the trustees or any other persons and generally in such manner for the benefit of the said class or any of them
as the trustees shall from time to time or at any time by any deed or deeds revocable or irrevocable (with due regard to the
rule against perpetuities) appoint Provided that no such appointment may be made or revoked whether wholly or in part
after the perpetuity date and Provided Further that the trustees may at any time or times by deed wholly or partially release
or restrict the foregoing power of appointment. (2) In default of and until and subject to any such appointment as aforesaid
in trust for all or any of the children or child of the settlor born before Feb. 20, 1975, who shall either attain the age of
twenty-five years or being a daughter shall marry under that age or be living on the perpetuity date and attain the age of
twenty-one years or being a daughter shall marry under that age and if more than one in equal shares.

The case turns on the proper interpretation of s 399 (b) of the Act of 1952, but before coming to that provision I must state in
outline the general scheme of this part of the Act. Section 397 provides that, where any income of a settlement 1031 is paid to or
for the benefit of infant children of the settlor, that income shall be treated as income of the settlor. During the relevant period the
settlor had three infant children but the income was accumulated by the trustees, so that section does not apply to this case. Then
s 398 deals with income which will or may become payable for the benefit of a child, but such income is not to be deemed to be
income of the settlor if the settlement is an irrevocable settlement. This settlement is in fact irrevocable, but s 399 deems an
irrevocable settlement not to be irrevocable in certain cases. So if this settlement falls within the scope of this section the
exception in s 398 does not apply. Section 399 is as follows:

399. For the purposes of this Chapter, a settlement shall not be deemed to be irrevocable if the terms thereof provide
(a) for the payment to the settlor or, during the life of the settlor, to the wife or husband of the settlor for his or her
benefit, or for the application for the benefit of the settlor or, during the life of the settlor, if the wife or husband of the
settlor, of any income or assets in any circumstances whatsoever during the life of any child of the settlor to or for the
benefit of whom any income, or assets representing it, is or are or may be payable or applicable by virtue or in consequence
of the settlement: or (b) for the determination of the settlement by the act or on the default of any person: or (c) for the
payment of any penalty by the settlor in the event of his failing to comply with the provisions of the settlement:
Provided that a settlement shall not be deemed to be revocable by reason only (ii) that it provides for the
determination of the settlement as aforesaid in such a manner that the determination will not, during the lifetime of any
such child as aforesaid, benefit any person other than such a child, or the wife, husband, or issue of such a child; or (iii)

Before coming to the question which your lordships have to determine I should note that proviso (ii) was amended by s 20(5) of
the Finance Act, 1958, by deleting any person other than such a child, or the wife, husband or issue of such a child and
substituting the settlor or the wife or husband of the settlor. Apparently it was recognised that the original form was
unnecessarily wide and that the new form was adequate to deal with the mischief at which these provisions were aimed. The
amendment, however, was not made retrospective. The original form of the proviso did not exclude this settlement from the
scope of s 399, because the trustees can use their powers to benefit the husbands or wives of grandchildren as well as those who
are mentioned in the proviso in its original form; but the new form of the proviso does cover the present case, because the trustees
cannot use their powers to benefit the settlor or the wife of the settlor. It is apparently for that reason that this case is concerned
only with the years before the passing of the Act of 1958.
The trustees could use their power of appointment so as to appoint the whole fund absolutely to a beneficiary who had
attained majority and such beneficiary might be the husband or wife of a grandchild of the settlor. The Crown says that therefore
the settlement provides for the determination of the settlement by the act of the trustees. If that is right then admittedly this
appeal succeeds. But the taxpayer argues,first that the appointment of the whole trust fund is not a determination of the
settlement within the meaning of s 399 (b), and secondly, that even if it is, the settlement does not provide for its determination
it merely confers power to do something which will in fact bring about its determination.
What then is meant by the determination of a settlement by the act or on the default of a person? It appears to me that,
unless the context indicates a more limited meaning, it must include every case where the act of a person brings the settlement to
an end. The trustees are persons, and therefore it must include every case where the trustees do an act authorised by the
settlement which brings it to an end. Exercising a power of appointment is the doing of an act, 1032and, if the exercise of such
power vests the whole trust fund absolutely in a major beneficiary, that appears to me to bring the settlement to an end. If
authority be needed for that I find it in the decision of this House in Countess of Kenmare v Inland Revenue Comrs. It is said,
however, that an absolute appointment of the whole fund does not bring the settlement to an end because after making the
appointment the trustees have to wind up the trust estate by paying debts, etc, and then to convey the net estate to the appointee
and the settlement must survive until that is done. But they would have to do that in every case where the act or default of any
person, whether the settlor or a third person, is such that it makes it the duty of the trustees under the settlement to divest
themselves of the whole trust property. Where the act is an act of some person other than the trustees it might be quite
unexpected and it might take some time before the trustees could wind up the estate. It would surely be impossible to hold that
for that reason the settlement was not determined by that act. I can see no difference in this respect between determination by an
act of the trustees and determination by the act of some other person.
Then it was said that the word determination must be given a narrow meaning, because to give it this wide meaning would
make the section apply to cases to which it cannot have been intended to apply. That may be true. The draftsman of provisions
for the prevention of tax evasion is often faced with this difficulty: if he uses narrow language the ingenuity of taxpayers
advisers will find a way to circumvent it, while if he uses wider language it will catch cases which do not really involve any
element of evasion. The intervention of Parliament in 1958 shows that in 1936 the draftsman had used unnecessarily wide
language, but unfortunately for the taxpayer the amendment made in 1958 does not cover this case. In such a situation I would
be very willing to adopt a restricted meaning if I could find any secure basis for it by ordinary methods of construction, but I
cannot.
The argument is that the operation of a special power of appointment is a fulfilment of the settlement and therefore should
not be regarded as a determination of it. But I cannot find any clear distinction between different kinds of acts by reason that the
provisions of the settlement in fact bring it to an end when they are done. I cannot find any satisfactory test so that some such
acts can be said to be in fulfilment of the settlement and some not. A special power of appointment is in effect an authority to the
trustees to write something into the settlement (see Muir or Williams v Muir). The settlor may himself write into the settlement a
provision which will in a certain event determine the settlementeg if X does a certain actor he may authorise his trustees to
write in something which will determine the settlementeg to appoint the whole trust fund to A absolutely. I do not see why one
should be said to terminate the settlement prematurely, to bring death to the settlement, while the other does not. I can see no
escape from the conclusion that any act which results in the settlement being brought to an end is an act by which the settlement
is determined.
The other main argument for the taxpayer turned on the meaning of the word provide. In this case the settlement provides
means whereby the trustees can do an act which in fact determines the settlement. It is said that that is not providing for the
determination of the settlement by that act: the settlement must expressly provide that it shall come to an end if a certain act is
done. There is some support for that argument in the decision of the Court of Appeal in Jenkins v Inland Revenue Comrs. There
the taxpayers scheme was to lend money to the settlement trustees and then have them use the income of the trust fund to repay
his loan. He would then say that what he received was not income, but a repayment of his loan. Of course he did not baldly put
his scheme on paper. He gave the trustees a general power to borrow and authorised them 1033 to repay any loan out of income.
So they need not have borrowed from him or anyone else, and if they did borrow they need not have repaid the loan out of
income. It just so happened that they did borrow from the settlor and they did use the trust income to repay him. Then the
taxpayer succeeded in his contention that the settlement did not provide that all this should be doneit merely authorised it. In
my opinion that decision was wrong. Lord Greene said that he found no justification for giving to the word provide ([1944] 2
All ER at p 496, letter E; 26 Tax Cas at p 282)

a crabbed or artificial or highly intricate meaning such as would be necessary to enable it to cover the state of facts
which I have mentioned.

No doubt the ordinary meaning of a word is something which each of us must decide according to our experience of the ordinary
use of the English language. I can only say that to my mind a person who contemplates or desires a certain result and gives
powers to trustees which enable that result to be achieved could properly be said in ordinary parlance to provide for that result by
giving those powers. In the present case the settlor must be taken to have known that the terms of his settlement would enable
the trustees to appoint the whole fund absolutely to any person who might in future come within the class of beneficiaries set out
in cl 3. If such an appointment would in law determine the settlement, then the settlor made provision for its being determined in
that way and the settlement so provided. Any other view would simply open the door wide for evasion. All that would have to
be done would be to confer powers in terms sufficiently wide to enable them to be used in several different ways and then to say
that the settlement does not provide for the use of those powers in any particular way.
Finally an argument was founded on the reference in s 398(2) of the Act of 1952 to a settlement which, at the time when the
income is so dealt with, is an irrevocable settlement. It was said that this shows that one must look at the facts at the relevant
time to see whether the settlement was then revocable. That is quite true: a settlement may be revocable today but irrevocable
tomorrow, because the only person with power to revoke has died. That appears to me to have no application to s 399. The
question there is not whether the terms of the settlement provide for its determination during the relevant period, but whether they
provide for is determination at any time by the act of any person. One must therefore look not only at the position today but also
at what the position will be if the settlors children have children who grow up and marry. It is true that today the trustees could
only appoint the trust fund to persons within the scope of proviso (ii), but some day they may be able to appoint to persons not
within the scope of that proviso. And the terms of s 399 are, I am afraid, so wide that that is enough to bring that section into
operation immediately. With some regret I feel bound to hold that this appeal must be allowed and the judgment of Plowman J
restored.
My Lords, my noble and learned friends Lord Guest and Lord Pearce are unable to be present this morning and they have
asked me to say that they both concur.

LORD JENKINS. My Lords, in this case the taxpayer was assessed to surtax for the years of assessment 195556, 195657 and
195758 under the provisions of the Income Tax Act, 1952, relating to settlements on children. Against these assessments he
appealed by way of Case Stated to the High Court (Plowman J) who by an order dated 13 July 1961, dismissed the appeal. From
that order the taxpayer appealed to the Court of Appeal (Sellers, Donovan and Pearson LJJ) who by a majority (Donovan and
Pearson LJJ Sellers LJ dissenting) reversed the order of Plowman J and (by an order dated 27 March 1962) allowed the appeal,
giving leave to the Commissions of Inland 1034 Revenue to appeal to your lordships House. The question at issue in the appeal
now brought before your lordships in pursuance of such leave is whether a settlement made by the taxpayer on 9 August 1950, is
an irrevocable settlement for the purposes of s 397, s 398 and s 399 of the Income Tax Act, 1952. It is common ground that if the
settlement is not irrevocable the taxpayer is assessable to surtax in the figures determined by the Special Commissioners, but that
if the settlement is irrevocable the proper assessments are those set out in the order of the Court of Appeal.
The Crown has conveniently set out in its Case the relevant provisions of the Act of 1952 and I take the liberty of
reproducing them here:

3. The relevant provisions (contained in Chapter 2 of Part 18) of the Income Tax Act, 1952, were at the material times
as follows:
397.(1) Where, by virtue or in consequence of any settlement to which this Chapter applies and during the life of
the settlor, any income is paid to or for the benefit of a child of the settlor in any year of assessment, the income shall, if at
the commencement of that year the child was an infant and unmarried, be treated for all the purposes of this Act as the
income of the settlor for that year and not as the income of any other person
398.(1) Subject to the provisions of this section, for the purposes of this Chapter(a) income which, by virtue or
in consequence of a settlement to which this Chapter applies, is so dealt with that it, or assets representing it, will or may
become payable or applicable to or for the benefit of a child of the settlor in the future (whether on the fulfilment of a
condition, or on the happening of a contingency, or as a result of the exercise of a power or discretion conferred on any
person, or otherwise) shall be deemed to be paid to or for the benefit of that child; and
(2) Where any income is dealt with as mentioned in sub-s.(1) of this section by virtue or in consequence of a
settlement to which this Chapter applies, being a settlement which, at the time when the income is so dealt with, is an
irrevocable settlement(a) the provisions of sub-s.(1) of this section shall not apply to that income
399. For the purposes of this Chapter, a settlement shall not be deemed to be irrevocable if the terms thereof provide
(b) for the determination of the settlement by the act or on the default of any person; Provided that a settlement
shall not be deemed to be revocable by reason only (ii) that it provides for the determination of the settlement as
aforesaid in such a manner that the determination will not, during the lifetime of any such child as aforesaid, benefit b any
person other than such a child, or the wife, husband or issue of such a child; or
________________________________________
b The succeeding words of proviso (ii) were repealed and other words substituted by the Finance Act, 1958, s 20(5); see p 1032, letter e, ante.

400.(1) Where, by virtue of this Chapter, any income tax becomes chargeable on and is paid by the person by
whom a settlement was made or entered into, that person shall be entitled(a) to recover from any trustee or other person
to whom the income is payable by virtue or in consequence of the settlement the amount of the tax so paid; and .

The settlement of 9 August 1950 (the settlement) was made between the taxpayer of the the one part and The Southern
Investment Trust Ltd (the trustees) of the other part, and after reciting that the settlor had one daughter Fiona Mary (born on 20
February 1950) and with a view to the settlement had paid to the trustees a sum of 10,000, went on to state the following
provisions of the settlement relevant to the present appeal:

1. In this deed the expression the perpetuity date means the date of expiration of a period of twenty-one years from
the death of the survivor of the said Fiona Mary Jamieson and the settlors wife Nancy Cecil Jamieson.
1035
3. The trustees shall stand possessed of the said sum of 10,000 and of the investments and property for the time
being representing the same (hereinafter called the trust fund) and of the income thereof upon the following trusts:(1)
In trust for all or such one or more exclusively of the others or other of the following class of persons (that is to say) the
issue (including children hereafter to be born and issue more remote than children) of the settlor and the wives husbands
widows or widowers (whether or not remarried) of such issue in such shares if more than one and either absolutely or for
such successive or other interests or with such trusts and provisions for their respective benefit at the discretion of the
trustees or any other persons and generally in such manner for the benefit of the said class or any of them as the trustees
shall from time to time or at any time by any deed or deeds revocable or irrevocable (with due regard to the rule against
perpetuities) appoint. Provided that no such appointment may be made or revoked whether wholly or in part after the
perpetuity date and Provided Further that the trustees may at any time or times by deed wholly or partially release or
restrict the foregoing power of appointment. (2) In default of and until and subject to any such appointment as aforesaid in
trust for all or any of the children or child of the Settlor born before Feb. 20, 1975, who shall either attain the age of
twenty-five years or being a daughter shall marry under that age or be living on the perpetuity date and attain the age of
twenty-one years or being a daughter shall marry under that age and if more than one in equal shares.

The special commissioners on the hearing of the taxpayers appeal against the said assessments found the following facts: (1) It
was not disputed that the taxpayer was the settlor in relation to the settlement for the purposes of the said s 397, s 398 and s
399. (2) The trustees of the settlement had not exercised the power of appointment given to them by the said cl 3(1) of the
settlement, nor had they released or restricted such power. (3) At all material times there had been three children only of the
taxpayer, who were born on 20 February 1950, 28 May 1951, and 4 July 1956, respectively, and there are no issue of the taxpayer
more remote than children. (4) The income arising under the settlement and forming the subject-matter of this appeal was: for
the year 195556: 1,519; for the year 195657: 1,693 10s; for the year 195758; 1,696. (5) At all material times the trustees
had accumulated all the net income of the trust fund under or by virtue of cl 3(2) and (3) of the settlement and s 31 of the Trustee
Act, 1925.
It has been contended for the taxpayer that on the true construction of s 399 the expression determination of the settlement
connotes the destruction, setting aside, or circumvention of the settlement as distinct from the fulfilment of the purposes of the
settlement; and an appointment of the trust fund on trust absolutely for an appointee who is sui juris, in exercise of the special
power of appointment conferred by the settlement, forms part of the fulfilment of the purposes and the operation of the trusts of
the settlement and is not a determination of the settlement. I find this line of argument impossible to accept. I see no sufficient
justification for the assertion that a settlement is not determined, eg, by an exercise of a power therein contained of appointing
absolutely to some beneficiary or beneficiaries the entirety of the trust fund.
Then it was said on the taxpayers side that the settlement was by its terms irrevocable and accordingly the prohibition in s
399 against the deeming of certain settlements to be irrevocable had no application. I see no sufficient justification for construing
s 399 (b) in this way; or for denying the legislature power to say that this or that irrevocable settlement (ie irrevocable according
to its own tenor) should or must, in order to be irrevocable for the purposes of the enactment, be provided with some additional
qualification. It appears to me to be reasonably plain that what is meant by s 399 (b) is simply that the settlement concerned is
not to count as, or be regarded or treated as (to take three obvious alternatives out of the many that might be chosen), an
irrevocable settlement.
1036
There was a good deal more discussion about the meaning of determination, but it did not, I think, carry the matter any
further. In this connexion reference was made to the important case in your lordships House of Countess of Kenmare v Inland
Revenue Comrs, where the trustees of the settlement were by its terms allowed (subject to certain limitations) to make periodic
withdrawals from the trust fund; and it was held in your lordships House that inasmuch as the withdrawals might in given
circumstances exhaust the entire fund, the exhaustion of the fund in this way would operate to determine the settlement within
the meaning of s 38(2) of the Finance Act, 1938. The effect of the Kenmare principle, if accepted in the present case, would
apparently be to make the trustees power of appointment a provision for the determination of the settlement by the act of
any person within the meaning of s 399 (b), and Plowman J, regarded himself as bound so to hold. The lords justices for one
reason or another declined to regard themselves as bound by the Kenmare case and, of course, were well entitled to take that view
inasmuch as that case related to a different settlement, different legislation, and different circumstances generally. I think,
therefore, that the Kenmare case, so far as it is sought to be applied in the case now before your lordships, should be accorded
strong persuasive force not amounting to binding authority.
I find it unnecessary to do more than notice the cases of Jenkins v Inland Revenue Comrs and Inland Revenue Comrs v
Hamilton-Russell Exors, to which we were referred by counsel in the course of their full and able arguments, but which did not, I
think, in the end materially advance the discussion.
The Crowns contention before the special commissioners is shortly stated in its Case as being to the effect that cl 3(1) of the
settlement provided for the

determination of the settlement by the act of any person [that is to say the trustees] within the meaning of s. 399
(b) of the Income Tax Act, 1952, and so provided in such manner that the determination could benefit persons other than
those described in proviso (ii) to that section, inasmuch as the trustees could make an appointment in favour of the wives,
husbands, widows or widowers of issue of [the taxpayer] more remote than children, and that accordingly the settlement
was not an irrevocable settlement for the purposes of s. 398 of the Act with the result that the said income fell to be treated
as income of [the taxpayer].

It appears to me that these words, unfortunately for the taxpayer, exactly fit his case, though it would seem that they do so only
through a mistake which brought about the inclusion in the trusts of the settlement of too wide a class of beneficiaries.
The sections on which this case turns may be thought unnecessarily obscure, and in the circumstances to bear somewhat
heavily on the taxpayer, but your lordships task, as in any other revenue case, is to construe the provisions of the taxing
enactment according to the ordinary and natural meaning of the language used and then to apply that meaning to the facts of the
case. If by the application of this process the taxpayer is brought fairly within the net, he is caught, otherwise he goes free; but
there must be no straining of language either way.
I confess to some sympathy with the taxpayer, but having applied to this case the principles above indicated to the best of
my ability I have come to a clear conclusion to the effect that the Crown is entitled to succeed in this case, and I would
accordingly allow the appeal.

LORD HODSON. My Lords, I agree that the appeal succeeds for the reasons given by Plowman J, and for those given by
Sellers LJ, in the Court of Appeal ([1962] 2 All ER 321 at pp 323327; [1962] 1 Ch at pp 765771).
1037
The reasoning of this House in Countess of Kenmare v Inland Revenue Comrs leads to the conclusion that the power to
appoint absolutely which is contained in the settlement is a power to determine the settlement by the act of any person within the
language of s 399 (b) of the Income Tax Act, 1952. True it is that the Kenmare case was concerned with the wording of s 38(2) of
the Finance Act, 1938 (now s 404 of the Income Tax Act, 1952), which is not the same totidem verbis as s 399 (b), yet the
wording is to the same effect and each section involves consideration of what is meant by determination of the settlement. In
the Kenmare case the words were may have power in the future to revoke or otherwise determine the settlement or any
provision thereof. In this case the words are if the terms thereof provide for the determination of the settlement by the act
of any person. In the former case the terms of the settlement enabled the trust fund to be exhausted by successive
withdrawals during the lifetime of the settlor so that there would be nothing left for the trusts to operate on and the settlement, it
was held, could be determined. Similarly, in this case the exercise of the power could bring about a determination.
A subsidiary argument was addressed to your lordships to the effect that the settlement did not provide in terms for its
determination and that if the statute had intended determination to be brought about by the exercise of a power of appointment it
could have easily said so. This view appealed to Donovan LJ (See [1962] 2 All ER at p 332; [1962] 1 Ch at p 779) and is
supported by the judgment of Lord Greene MR in Jenkins v Inland Revenue Comrs, a case which I find difficult to distinguish on
the ground suggested, namely, that it involved proof of extraneous facts. In that case the settlor was able to get into his hands the
whole of the trust funds but that was said to have occurred, not by reason of any terms contained in the settlement but rather
because of the extraneous fact that the settlor had lent money to the trustees which they repaid to him out of the income of the
settlement. Like Donovan LJ I find the distinction difficult to perceive and I do not think that the decision in Jenkins case can be
supported.
The third point raised by the taxpayer was to the effect that the exercise of a power of appointment cannot have the effect of
determining a settlement, for it is said that the settlement is not determined until the transfer of the funds, which is not provided
for by the settlement. This argument, I think, is fallacious. The settlement is in truth determined by the execution of the power in
favour of a person sui juris and this is no less true because the trustees may have further duties and rights under the settlement
before their accounts are finally passed. This is clear and is illustrated by the decision of the Court of Appeal in Inland Revenue
Comrs v Hamilton-Russell Exors. Luxmoore LJ there pointed out ([1943] 1 All ER at p 477, letter b; 25 Tax Cas at p 208) that
once funds are at home and belong solely to the beneficiary for his own absolute use and benefit the trusts of a settlement become
unenforceable and ineffective.
Lastly, it was argued that the settlement was saved by the second proviso to s 399 of the Act of 1952, which reads c:
________________________________________
c the Finance Act, 1958, s 20(5); see p 1032, letter e, ante

Provided that a settlement shall not be deemed to be revocable by reason only (ii) that it provides for the
determination of the settlement as aforesaid in such a manner that the determination will not, during the lifetime of any
such child as aforesaid, benefit any person other than such a child or the wife, husband or issue of such a child.

It was argued that, having regard to the facts existing at the material time, that is to say, when the income of the settlement is
being accumulated, the only 1038 persons who could have benefited were the children of the settlor. The answer is as Plowman J
said, that one must look at the settlement. One then sees from the language of the power that persons outside the class can benefit
when the settlement is determined.
I would allow the appeal and restore the judgment of Plowman J

Appeal allowed.

Solicitors: Solicitor of Inland Revenue (for the Crown); Farrer & Co (for the taxpayer).

C G Leonard Esq Barrister.


[1963] 2 All ER 1039
Welford Gravels Ltd v De Voil (Inspector of Taxes)
TAXATION; Income Tax, Assessment

HOUSE OF LORDS
LORD REIDb, LORD JEKINS, LORD HODSON, LORD GUEST AND LORD PEARCE
9, 13, 14, 15, 16, 20 MAY, 20 JUNE 1963
________________________________________
b Lord Reid was unable to be present throughout some of the hearing owing to indisposition.

Income Tax Assessment Valuation of property New property Part of farm sold for development Use as gravel pit
Whether separate valuation or apportioned part of farm value Income Tax Act, 1952(15 & 16 Geo 6 & 1 Eliz 2 c 10), s 82,
para 2, s 84(3), s 108.

In 1956, the owners of farm lands of about 155 acres, which included a sand and gravel pit that had been used for extracting sand
and gravel for farm purposes, obtained planning permission for the extraction of sand and gravel from 23 1/2 acres. They then
sold the 23 1/2 acres to B Ltd who allowed the taxpayers (an associated company) to go into occupation of the site and prepare it,
by laying a concrete apron of half an acre in extent and erecting buildings and installing plant and machinery at a cost of some
25,000. In December, 1957, B Ltd granted the taxpayers a twenty-one years lease of the site at a rent of 60 per annum, with a
provision that nothing in the lease was to authorise the tenants to get or carry away sand or gravel. On the same day, they granted
a separate licence to the taxpayers to extract sand and gravel on payment of a royalty of 4s per cubic yard, and in 1957 some
fifty-six thousand cubic yards. Before the sale of the 23 1/2 acres, the Sch A assessment on the farm as a whole was 108 gross
and 56 net. For 195758, the revenue authorities, having become aware of the sale of the 23 1/2 acres, apportioned the Sch A
assessment of the 155 acres between the remainder of the farm and the 23 1/2 acres, for which the resulting assessment was 16
gross, 8 10s net, but, on subsequently learning of the development of the 23 1/2 acres, made a revised assessment for 195758 of
5,000 gross, 4,375 net for the 23 1/2 acres.

Held The revised assessment was rightly raised for the following reasons
(i) (per Lord Guest, Lord Hodson and Lord Pearce concurring) where a subject substantially different in character came into
existence during a year which was not a year of revaluation, a fresh entry, necessitating revaluation, was justified under Sch A; in
the present case the new extensive machinery and buildings constituted a significant change in the character of the land (the 23
1/ acres) with the consequence that it became a subject substantially different from the gravel pit formerly part of Stows Farm
2
(see p 1047, letters c and e, and p 1045, letter e, post).
1039
Moray Estates Development Co v Inland Revenue Comrs (1951 SC 754; 32 Tax Cas 317) considered and applied.
(ii) (per Lord Jenkins) consequent on the sale and change of occupation of the 23 1/2 acres a new property (within s 84(3) a
of the Income Tax Act, 1952) came into existence for the purposes of Sch A, and the Crown was entitled, although the year was
not a year of revaluation (cf. s 82, para 2 of the Act of 1952), to determine the annual value of the new property for the purposes
of assessment to tax under Sch A, otherwise than by apportioning (pursuant to s 108 b of the Act of 1952) the annual value of
Stows Farm (see p 1045, letters b and c, post).
________________________________________
a Section 82, Sch A., para 2 providesThe annual value for the purposes of this schedule shall, in the case of all lands, tenements,
hereditaments or heritages, of whatever nature and for whatever purpose occupied or enjoyed, and of whatever value, be understood to be
(a) if they are let at a rack rent and the amount of that rent has been fixed by agreement commencing within the period of seven years
preceding the fifth day of April next before the time of making the assessment, the amount of the rent by the year at which they are let; or
(b) if they are not let at a rack rent so fixed, then the rack rent at which they are worth to be let by the year:
Section 84(3) providesThe annual value of any property which has been adopted for the purpose of income tax under Schs. A and b for any
year of assessment shall be taken as being the annual value of that property for the same purpose for the next year of assessment, unless that
year is a year of revaluation: .
Section 108 providesIf, after the making of an assessment under Sch A, the lands are divided into two or more distinct occupations, the
General Commissioners, on the application of the persons respectively interested, shall determine what proportion of the tax shall be paid or
borne by each occupier, and the amount apportioned shall be collected and levied in like manner as if it had been an original assessment.

Decision of the Court Of Appeal, sub nom De Voil (Inspector of Taxes) v Welford Gravels Ltd ([1962] 2 All ER 657)
affirmed.

Notes
As to quinquennial valuation and assessment of new properties for the purposes of Sch A to the Income Tax Act, 1952, see 20
Halsburys Laws (3rd Edn) 59, 60, para 96, and 61, 62, para 100.
For the Income Tax Act, 1952, s 82, s 84 and s 108, see 31 Halsburys Statutes (2nd Edn) 80, 85, 102.

Cases referred to in judgment


Edwards v Bairstow [1955] 3 All ER 48, [1956] AC 14, 36 Tax Cas 207, [1955] 3 WLR 410, 28 Digest (Repl) 397, 1753.
Menzies, Re (1878), 1 Tax Cas 148, 5 R (Ct of Sess)531, 28 Digest (Repl) 382, 820*.
Moray Estates Development Co v Inland Revenue Comrs, 1951 SC 754, 32 Tax Cas 317, 28 Digest (Repl) 144, 428*.
Russell v Scott [1948] 2 All ER 1, [1948] AC 422, [1948] LJR 1265, 30 Tax Cas 394, 28 Digest (Repl) 142, 543.

Appeal
Appeal by the taxpayers, Welford Gravels Ltd from an order of the Court of Appeal (Lord Evershed MR Upjohn and Diplock
LJJ), dated 17 April 1962, and reported [1962] 2 All ER 657, reversing an order of Plowman J dated 4 July 1961, on an appeal by
the Crown by way of Case Stated against a determination of the General Commissioners of Income Tax. The facts are set out in
the opinion of Lord Jenkins.

J G Foster QC and H F Williams for the taxpayers.


Hubert H Monroe QC A S Orr QC and J R Phillips for the Crown.
Their Lordships took time for consideration

20 June 1963. The following judgments were delivered.

LORD JENKINS. My Lords, this case concerns an assessment to income tax under Sch A made on the taxpayers for the year
1957/58 on an estimated annual value of 5,000 gross, 4,375 net, in respect of property described as 1040Gravel pit and
premises, Stows Farm, Tillingham, Essex. The circumstances in which the disputed assessment came to be made are fully set
out in the Case Stated, and I need not repeat them at length in this judgment.
The property in question comprised farm lands extending to 155 acres or thereabouts, and included a very valuable feature
in the shape of a sand and gravel pit of some 23 1/2 acres in extent. The farm had for many years been in the ownership of
Messrs W & R L Procter, who had extracted gravel from the pit for use in connexion with the building of airfields during the
first world war, and, since that war, had from time to time also made some use of gravel so extracted for farm roads and other
farming purposes. An important development took place in 1956, in the shape of a great demand for gravel in connexion with the
construction at Bradwell-on-Sea (in the neighbourhood of Stows Farm) of a nuclear power station. In these circumstances,
Messrs Procter applied for and obtained from the appropriate authority the development permission required for the extraction of
sand or gravel from the 23 1/2 acres already mentioned. In February, 1957, Messrs Procter sold the 23 1/2 acres to a company
called Besbuilt Ltd which bought the land in order that one of its associate companies could extract sand and gravel for
commercial purposes. The sale was completed on 22 February 1957, and immediately thereafter Besbuilt Ltd allowed the
taxpayers to go into occupation of the land and prepare the site for the plant. The apparatus required seems to have been fairly
elaborate, and for details of it reference should be made to the stated Case. There was a concrete apron half an acre in extent with
stanchions for the plant to be attached to and from which it could easily be detached. There were also three small buildings
described as a brick power-house, a brick pump-house and a Nissen hut. These buildings cost some 1,000, and were intended to
be temporary and incidental to the workings. Machinery was installed on the site at a cost of some 24,000. It was the end of
March, 1957, before the apron was down and the machinery was in operation. It was intended that, when the pit had been
worked to the extent proposed, all the machinery, plant and buildings would be removed and the top-soil replaced. On 31
December 1957, Besbuilt Ltd executed a lease to the taxpayers of the whole of the land which it had bought from Messrs Procter
for twenty-one years from 1 March 1957, at the rent of 60 per annum. This document forms part of the Case Stated and need
not be repeated here. A curious feature of the transaction was that the lease provided that nothing that it contained was to
authorise the tenant to get or carry away sand, gravel, ballast or any other mineral from the said land. However, on the same
date, viz, 31 December 1957, Besbuilt Ltd granted a licence under seal to the taxpayers in respect of the land comprised in the
lease. This licence likewise forms part of the Case Stated and, to put it shortly, permits the extraction for sale of sand, gravel,
ballast and other minerals from the land comprised in the lease at 4sf per cubic yard. Some fifty-six thousand cubic yards were
extracted in 1957, and some sixty thousand in 1958, these being in effect the latest available figures. The estimated potential of
this pit was only two hundred and fifty thousand cubic yards, but the taxpayers had an option over further land in the area.
Before the sale of the 23 1/2 acres, the Sch A assessment on Stows Farm containing 155 acres was 108 gross and 56 net. The
Revenue first became aware of the sale of the 23 1/2 acres to Besbuilt Ltd after the 1957/58 assessment had been raised on the
farm as a whole. Then there seems to have been a good deal of confusion as to the true position, but I do not think it necessary to
say more about that than what has been said in this passage at the end of para 4 of the Case:

A fresh additional assessment under Sch. A was raised for the year 1957/58 upon the [taxpayers] on a revised
estimated annual value of 5,000 gross 4,375 net in respect of gravel pit and premises, Stows Farm, Tillingham, Essex,
and this assessment is the subject of the appeal by the [taxpayers].
1041

At the hearing before the commissioners, the following points amongst others were made on behalf of the taxpayers:

(i) that the basis of a Sch. A assessment was a quinquennial valuation of specific property and this could only be
departed from (a) where in connexion with the valuation of such property a relevant and significant factor had been
overlooked in the pre-quinquennial year (which was not so in this case), (b) where there might have been some significant
alteration in the property (although it was doubtful whether this was legally valid) in which case an apportionment could be
made, (c) where a new property had come into being, as distinct from a new use of property, e.g., a new house built on a
bare piece of land. (ii) that the getting of sand and gravel from land was merely making use of the natural potentialities of
the lands. There had been a continuous getting of sand and gravel from the land in question for many years, and the more
intensive getting of sand and gravel from the land in the period relevant to the present appeal did not create a new property.
(iii) that if it was decided there was a new property the basis of the Sch. A assessment was the rack rent of the property.
The rack rent was evidenced by the lease and for the purpose of Sch. A any payments under the ancillary agreement which
represented the measure of damages done to the freehold by waste and did not form part of the rent reserved were not to be
taken into account. (iv) if, however, account had to be taken of the amount payable under the licence, which is obviously a
fluctuating amount, then an average should be taken over the period of the lease and licence which in this case would be
twenty-one years.

On the Crowns side, points taken included the following:

(i) That in the year 1957/58 the area of 23 1/2 acres no longer formed part of Stows Farm but was a separate piece of
property in respect of which tax under Sch. A could only be charged by an assessment on the new occupierss. 105,
Income Tax Act, 1952; (ii) that no annual value had been adopted for that property for any previous year of assessment and
s. 84(3) of the Income Tax Act, 1952, did not apply; (iii) that under s. 82, Sch. A, para. 2 (b), of the said Act the annual
value of the said property must be understood to be the rack rent at which it was worth to be let by the year for use as a
gravel pit; (iv)

The commissioners were asked by the representative for the taxpayers to come to a decision on the preliminary points raised
by him (reserving the evidence of the valuers for later if in fact it was needed), the preliminary points being (i) whether there
must be a new property, that is, a property significantly 1042 different from the twenty-three acres of land plus the sand pit which
existed prior to February, 1957, before an additional Sch A assessment could be raised, (ii) whether there was such a property
here and, if so, what was it and how did it differ from the property that had been there previously, and (iii) if there was a new
property, how should the annual value be determinedin particular, whether it should be limited to the amount of the rent
reserved by the lease. The commissioners agreed to adopt this course and, after consideration, decided (i) that there must be a
new property before an additional Sch A assessment could be raised, and (ii) that there was no new property in this case. The
third point did not, therefore, arise. The commissioners, accordingly, reduced the assessment of 5,000 gross, 4,375 net on the
taxpayers for the year 1957/58 to 16 gross, 8 10s net, being the proportion of the original farm assessment under Sch A.
Perhaps I should make some further reference to the preliminary points raised by the taxpayers. These preliminary points
were duly incorporated in the Case Stated, the first of them being as follows:

Whether there must be a new property, that is a property significantly different from the 23 acres of land plus the sand
pit which existed prior to February, 1957, before an additional Sch. A assessment could be raised;

The second of such points was as follows:

Whether there was such a property here and, if so, what was it and how did it differ from the property that had been
there previously?

Paragraph 8 of the Case says this:

We, the commissioners who heard the appeal, agreed to adopt this course and decided (i) that there must be a new
property before an additional Sch. A assessment could be raised, and (ii) that there was no new property in this case

The inspector of taxes having appealed to the High Court, Plowman J by an order dated 4 July 1961, dismissed his appeal, the
learned judge being of opinion that the commissioners finding of fact ought not to be disturbed. From that order of Plowman J
the Crown appealed to the Court of Appeal (Lord Evershed MR and Upjohn and Diplock LJJ), and that court, by an order dated
17 April 1962 (Upjohn LJ dissenting), allowed the appeal, leave being given to the taxpayers to appeal to your lordships House.
The Master of the Rolls in his judgment in the Court of Appeal has described the question raised in this appeal as a novel one of
considerable difficulty, and I certainly would not dissent from that description of it.
The substantial point at issue, as I understand it, is how land previously assessed as a single unit for the purposes of Sch A
income tax ought to be treated with respect to such tax if part only, but not the whole, of such land is disposed of to some third
party. As the Master of the Rolls puts it ([1962] 2 All ER at p 661; [1963] 1 Ch at p 115):

As is well known, however, there have in fact been no revaluations for a very long period of time. None the less, it is
the case for the Crown that, where, as here, a piece of property which was formerly owned and occupied by a single person
or corporation is then divided both as regards ownership and occupation, there arises at once some new item of property
which the Revenue must be entitled separately to value under the terms of s. 82 of the Act. The case for the Crown goes to
the length of saying that such a right of revaluation arises where an owner who has previously let a substantial area to a
single tenant proceeds to subdivide his land and then to grant, in lieu of the single tenancy, two distinct tenancies of
particular parts of his land. Similarly, and indeed more forcibly (it is submitted), the same result arises where a common
owner or owner-occupier sells outright part of what was formerly a singly occupied piece of property. Finally, it is said
that in any event, and whether the two previous propositions can or cannot be sustained, if on any part of a piece of land
work is donee.g., by way of the erection of buildings or otherwisewhich, in the language of the counsel for the
taxpayers, relevantly and significantly transforms the nature of that piece of property, then such a right of revaluation
arises.

Reference should also be made to the concession ([1962] 2 All ER at p 662; [1963] 1 Ch at p 117) by counsel for the taxpayers,
who conceded that if, as a result of

relevant and significant changes in the nature of any propertye.g., by the building of a house on what had been
vacant landa new item of property came into existence, then a power and a right to make a revaluation for Sch. A
purposes did unquestionably arise

The Master of the Rolls said later ([1962] 2 All ER at p 662; [1963] 1 Ch at p 117):
1043

In the light, however, of the concession of counsel for the taxpayers, I should myself be prepared to hold that the
transformation of the 23 1/2 acres with which we are concerned from its previous use as part of Stows Farm, albeit that a
certain amount of sand and gravel had been extracted therefrom for farm purposes, to its present industrial use supported
by buildings and machinery of a total cost of 25,000, had produced a change in the character of the property no less
relevant and significant than the change that would have been produced by the erection on it of a dwelling-house or
dwelling-houses. I appreciate that the question was dealt with as a matter of fact and that, as the judge said, such matters
are apt to be in the end of all questions of degree. But on the authority of Edwards v. Bairstow, I would hold that the true
and indeed the inevitable inference from all the facts, including the sale to Besbuilt, Ltd., and the fact that planning
permission was required for what was done, is the creation for present purposes of a new item of property.

A little later, the Master of the Rolls said ([1962] 2 All ER at p 663; [1963] 1 Ch at p 118):

If, however, I am not entitled so to reverse the commissioners finding, and since the whole case was exhaustively
argued, then I would also hold that the sale of the 23 1/2 acres to Besbuilt, Ltd., and its occupation by the taxpayers gave
rise to a new property which justified and required separate valuation for the purposes of Sch. A in accordance with the
provisions of s. 82(2) of the Act. On this matter I have had the advantage of reading the judgment to be delivered by
DIPLOCK, L.J., and I agree with his conclusion and his reasons therefor.

The Master of the Rolls went on ([1962] 2 All ER at p 663; [1963] 1 Ch at p 119) to refer to the Scottish case, Moray Estates
Development Co v Inland Revenue Comrs:

As the judge pointed out, that was a case in which part of an owners land had been for many years let to a company
for the purposes of gravel extraction at a royalty, and had for all that time been assessed under Sch. D upon the supposition,
proved erroneous by the Russell v. Scott decision, that it fell within what is now para. (c) of the proviso to s. 82(1). The
Inner House rejected the taxpayers submission that in the circumstances it was necessary to revert to the assessment
appropriate to the entire unit of land before the letting to the gravel company, and held that, on such letting, the site of the
gravel pit had been elevated into a distinct and separate subject of occupation.

The Master of the Rolls, accordingly, expressed himself ([1962] 2 All ER at p 664; [1963] 1 Ch at p 120) in favour of allowing
the appeal and holding that the answer to the second preliminary question formulated by the General Commissioners should have
been in the affirmative.
As already mentioned, Upjohn LJ dissented, giving his reasons in these words ([1962] 2 All ER at p 669; [1963] 1 Ch at p
127):
I reject the Crowns claim to be entitled to revalue on a division of a holding of lands whether on a change of
occupation or on a sale for the basic reason that the whole scheme of valuation depends on a general revaluation in a year
of revaluation. A change of occupation of the whole of a holding between years of revaluation is admittedly irrelevant; and
so, in my judgment, is a division of a holding. Valuation depends not on units of assessment but on physical valuations in
years of revaluation.
1044

As has already appeared, the Master of the Rolls expressed his agreement with Diplock LJs conclusions and his reasons
therefor. It has been suggested that, to be strictly accurate, the Master of the Rolls and Diplock LJ might at some points have
been described as reaching the same conclusion by a different route; but I do not think that anything turns on this. Diplock LJ
began his judgment thus ([1962] 2 All ER at p 670; [1963] 1 Ch at p 129):

This appeal turns on the answers to three questions: (i) whether on the sale and consequent change of occupier in
February, 1957, of 23 1/2 acres of gravel-bearing land which previously formed part of, and were occupied together with,
the remainder of Stows Farm, Tillingham, there came into existence a new property for the purposes of Sch. A to the
Income Tax Act, 1952; (ii) if so, whether the Crown became entitled in a year which was not a year of revaluation to
determine the annual value of that new property for the purposes of assessment to tax under Sch.A; and (iii) if so, whether
it was entitled to determine such annual value otherwise than by apportioning the annual value of Stows Farm as
determined in the last year of revaluation between the 23 1/2 acres and the remainder of Stows Farm. In my opinion, the
answer to each of these questions is: yes.

He went on to make good his three affirmative answers, and, to my mind, did so satisfactorily. His reasoning involved a
somewhat detailed discussion of various provisions of the Income Tax Act, 1952 (including, in particular, s 82, s 84, s 85 and s
108), into which I need not enter, inasmuch as it is fully set out in the judgment of Diplock LJ of which I have expressed my
approval.
I would dismiss this appeal.

LORD HODSON. My Lords, I agree with the opinion of my noble and learned friend, Lord Guest.
My noble and learned friend, Lord Pearce, who is unable to be present today, has asked me to say that he agrees with the
opinion that I am about to read, that is to say, the opinion of my noble and learned friend, Lord Guest.

LORD GUEST (read by Lord Hodson) stated the facts, referred to the taxpayers contention before the commissioners and the
commissioners decision, and continued: Plowman J accepted the finding of the commissioners and dismissed the Crowns
appeal. In the Court of Appeal Lord Evershed MR decided that the transformation of the 23 1/2 acres to its present industrial use
produced a relevant and significant change in the character of the property which justified an assessment of the gravel pit on a
revaluation. Diplock LJ did not deal with this aspect of the case, but based his judgment on the reasoning that the division of the
land previously in a single occupation into two separate ownerships or occupations resulted in two new properties which justified
revaluation. Lord Evershed MR was prepared to follow Diplock LJ only to the extent that he would hold that the division of the
land into two separate ownerships would have resulted in two units justifying a revaluation. He thought that, where the division
was into separate occupations, apportionment was the correct procedure. It will thus be seen that, on this point, the majority of
the Court of Appeal reached the same result but for different reasons. Upjohn LJ held that the Crown were not entitled to revalue
on a division of the lands, whether on a change of occupation or on a sale, for the basic reason that the whole scheme of valuation
depended on a general revaluation in a year of revaluation.
Two questions arise for decision(i) whether the division of the lands into separate occupations justified a revaluation, and
(ii) whether the use of the land as a gravel pit justified a separate entry of the gravel pit at a value arrived at on a revaluation. It
may be convenient to deal with the first contention of the Crown that the division of the land by the sale of one part necessitated a
fresh assessment 1045 and a revaluation. The taxpayers contended that, in view of the terms of s 84 of the Income Tax Act, 1952,
which provided for quinquennial valuations, the gross annual value of the farm which was, prior to 1957, the subject of a Sch A
assessment, could not be varied, and that the only method whereby the annual valuation could in the circumstances be altered was
by an apportionment under s 108 of the Act of 1952. Section 108 is in the following terms:

If, after the making of an assessment under Sch. A, the lands are divided into two or more distinct occupations, the
General Commissioners, on an application of the persons respectively interested, shall determine what proportion of the tax
shall be paid or borne by each occupier, and the amount apportioned shall be collected and levied in like manner as if it had
been an original assessment.

It will be seen that the section covers the case of a division into two or more distinct occupations. There was considerable
discussion as to the meaning of the two expressions after the making of an assessment under Sch A and the amount [of tax]
apportioned shall be collected and levied in like manner as if it had been an original assessment. The rival contentions on the
first expression were, for the Crown, that it referred to the annual assessment and, for the taxpayers, that it referred to the first
assessment in the year of revaluation under s 84 of the Act of 1952. Having regard to the scheme of the Act which provides for
an annual valuation of properties for Sch A purposes, I incline to the view that the Crowns contention is correct. But it does not
appear to me to matter, from the point of view of the construction of the section, which view is correct. Secondly, the Crown
contended that the concluding words of s 108 already referred to indicated that it was the tax, and not the annual value as
contended for by the taxpayers, which was to be apportioned. Again, it appears to me to be of no consequence which
interpretation is followed. If it is the tax which is being apportioned, as the Crown contended, this can only be done by an
apportionment of the gross annual value. If the apportionment has been made, the tax will be collected and levied by a certificate
in terms of s 34 which will state the annual value as apportioned.
The extreme contention for the taxpayers was that as the commissioners had apportioned the annual value in 1957/58 and
1958/59, this was an end of the matter and the commissioners were not, on any view, entitled to revalue the 23 1/2 acres as a
gravel pit on a fresh valuation. I recognise the force of this contention in relation to the question of division of the lands and, in
my view, the correct procedure in the circumstances is for the Revenue to apportion as provided for under s 108, but, if
revaluation is justified on the ground of a change of character in the property, a point later dealt with in this opinion, I see no
reason why the apportionment under s 108 should act as a barrier to the separate assessment of the gravel pit. If it be the case, as
argued for the Crown, that there is a duty on the inspector when the land has been divided into separate occupations to revalue
each separate unit under Sch A, at current values, it is difficult to see the necessity for s 108, because, once the inspector becomes
aware of the division, the commissioners must make separate entries which will render apportionment unnecessary. The Crown
were unable to suggest any valid reason for the retention of s 108 in the Act of 1952. In rejecting the argument for the Crown, I
adopt the reasoning of Upjohn LJ on this point which I find entirely satisfactory. In particular, I am impressed by the
extraordinary results which he says ([1962] 2 All ER at p 668; [1963] 1 Ch at p 127) would follow if the Crowns contention were
sound. These results were accepted by the Crown as accurate, but they denied that they were extraordinary.
Finally, the practice of the Revenue has for the last 120 years been, where there has been division into separate occupations,
to apportion, and the only 1046 justification for this practice that I can find is in s 108. The Crown conceded that, where the
lands were not divided but a new occupier entered into possession, there could not be a fresh valuation (see s 106 of the Act of
1952). I am unable to see the distinction between such a case and a case where the land is divided into two separate occupations.
I see no reason to disturb the existing practice and I reject the Crowns contention on this point.
I pass now to the second point taken by the Crown, that, where there is a relevant and significant change in the character of
the land, a new property comes into existence which has not been previously valued and which must be revalued. I observe from
the contentions of the taxpayers before the commissioners and the commissioners findings previously referred to, that the
taxpayers at that stage were conceding that, if there was a relevant and significant change in the character of the property, a new
property came into existence which necessitated a revaluation (see also Upjohn LJ ([1962] 2 All ER at p 669; [1963] 1 Ch at p
128)). It was on this basis that the case proceeded before the commissioners, and it was on this basis that Lord Evershed decided
the case. Apart, however, from any question of a concession, I am satisfied on a review of the relevant provisions of the Income
Tax Act, 1952, which have already been fully dealt with by your lordships, that, where a subject substantially different in
character comes into existence during a year which is not a year of revaluation, a fresh entry is justified under Sch A which
necessitates a revaluation. Plowman J and Upjohn LJ were not prepared to disturb the finding of the commissioners that no new
property came into existence. I feel no such reluctance. Lord Evershed, in my view, correctly stated the position when he said
([1962] 1 All ER at p 662; [1963] 1 Ch at p 117), after referring to Edwards v Bairstow, that the true and inevitable inference
from all the facts was the creation of a new item of property. The land had not since 1936 (the last year of the quinquennial
revaluation) been used commercially as a gravel pit. The extensive machinery and buildings at a total cost of 25,000 constitute
a significant change in the character of the land, so that it is a different subject from the gravel pit which formerly formed part of
the farm land of Stows Farm.
In Moray Estates Development Co v Inland Revenue Comrs., the Court of Session were faced with a somewhat similar
problem. Proprietors of woodlands which contained a gravel pit leased the gravel pit for less than a year on a royalty basis for
gravel removed. The court affirmed an assessment under Sch A based on the gross annual value as entered in the valuation roll.
The facts in that case would appear to be not far removed from the present. The woodlands, which presumably had included the
gravel pit, had previously been assessed under Sch A, and the gravel pit was opened in 1942. There was a slight complication
that, after 1942, the company had been assessed on the royalties of the gravel pit under Sch D, a course which was subsequently
held to be inadmissible in Russell v Scott. But there is nothing to show that the gravel pit was excised from the remaining
subjects. Thereafter, a separate Sch A assessment was raised on the gravel pit. The arguments for the company before the
commissioners and the Court of Session were to the effect that the gravel pit was not a proper unit of assessment, and that the
whole estate consisting of woodland area and gravel pit should be treated as a unum quid and assessed as such. This is, in effect,
the argument which the taxpayers put forward in the present case, though expressed in a slightly different form. In the result, the
court disposed of that argument as follows (see per Lord President Cooper (32 Tax Cas at p 322)):

No. 1 of Sch. A relates to all lands capable of actual occupation, of whatever nature, and for whatever purpose
occupied or enjoyed, and the annual value for income tax purposes is understood to be the rack-rent at 1047 which they
are worth to be let by the year. In 1942 the appellants elevated this gravel-pit into a distinct and separate subject of
occupation as the terms of the lease show and I should have thought that for income tax purposes (as unquestionably for
rating purposes) it then became a suitable subject for separate assessment from the woodlands which surrounded it or the
estates of which it formed a small part. It was quite properly entered under a separate entry in the valuation roll at a gross
annual value based on the royalties for the previous year. If the assessor for Inverness-shire had been an inland revenue
official this valuation would have been binding upon the Crown (s. 3 of the Lands Valuation (Scotland) Act, 1857; Re
Menzies. Even when the assessor is independently appointed it has long been the practice in Scotland to adopt the
valuation roll as the basis of income tax assessments and I have never known of a case where this has not been done. In
my view there is a heavy onus upon the appellants to show what is wrong with the unit and quantum of valuation which
they have accepted for rating purposes and why some different unit and value should be substituted for the purposes of Sch.
A. If common sense plays any part in income tax matters it is difficult to see why a gravel-pit which for nine years has
been worked commercially to an extent showing royalty profits of the order of nearly 1,200 per annum should be treated
as incapable of being let for more that prairie value.

The company did not advance the argument that a separate entry could not be made in view of the provisions of s 84, and it
may be that, on the facts, such an argument was not open. But, whether the point was in issue or not, I regard the result of the
decision as correct. It would be satisfactory if, as a result of the decision in this case, the administration of income tax law in
cases such as the present will be the same in England and in Scotland.
I would dismiss the appeal.

Appeal dismissed.

Solicitors: Field, Roscoe & Co agents for Dixon, Martell & Batcheldor, Bedford (for the taxpayers); Solicitor of Inland Revenue.

C G Leonard Esq Barrister.


[1963] 2 All ER 1048

Practice Direction
(Patent: Practice and procedure)
PRACTICE DIRECTIONS
10 JULY 1963

Patent Practice and procedure Extension of the term of a patent Advertisement Form RSC, Ord 53A, r 4 (d).

Mr Justice Lloyd-Jacob has directed that in the advertisement to be inserted in the Official Journal (Patents) (a form of which is
set out on p 1311 of the Annual Practice, 1963), the title of the invention is in future to be included.

W F S Hawkins, Chief Master, Chancery Division.


10 July 1963
1048
[1963] 2 All ER 1049

Re Pinion (deceased)
Westminster Bank Ltd v Pinion and Another
CHARITIES: SUCCESSION; Gifts

CHANCERY DIVISION
WILBERFORCE J
8 MARCH, 22 MAY, 27 JUNE 1963

Charity Education Public element Evidence Admissibility of evidence to support or negative existence of educational
value of gift Gift by testator of studio, paintings and old furniture, etc to form exhibition Evidence of experts that collection of
inferior quality and of no public interest Whether gift charitable in law.

A testator by his will and codicils gave his freehold studio and his pictures, one of which he attributed to Lely and some of which
were painted by himself, his antique furniture and some silver, china and miniatures, etc to be offered to the National Trust to be
shown to the public and maintained as a collection, and, if the Trust declined the gift, he authorised his executors to appoint
trustees to carry it out. He gave his residuary estate as an endowment to maintain the collection. The National Trust declined the
gift. There was expert evidence that the collection was of low quality, that the one picture stated by the testator to be the work of
Lely was not by that master, that the studio was squalid and that the testators pictures were bad, but that among the furniture
were a dozen or so genuine English and continental pieces of the seventeenth and eighteenth centuries, which perhaps might be
acceptable as a gift to a minor provincial museum. The experts were of opinion that the collection had no educational value. On
the question whether there was a valid charitable trust,

Held (i) the court ought in the present case to receive evidence to support or negative the existence in the gift of educational
value or public benefit, but it should be cautious in receiving such evidence for, when dealing in the sphere of art or aesthetics,
allowance must be made for the difficulty of making any secure objective judgment (see p 1055, letter b, post).
Royal Choral Society v Inland Revenue Comrs ([1943] 2 All ER 101) and Re Delius Will Trust ([1957] 1 All ER 854)
applied.
(ii) the evidence did not establish with sufficient certainty that no recognisable benefit to the public was secured by the gift,
for there was just enough in the collection, if it were given proper and skilled exhibition, to justify it as making a contribution to
the formation of artistic taste; accordingly the court would declare that the gift constituted a valid charitable trust (see p 1057,
letter f, post).
Dictum of Lord Simonds in National Anti-Vivisection Society v Inland Revenue Comrs ([1947] 2 All ER at p 233) applied.

Notes
As to preservation of objects for public inspection as an educational charitable purpose, see 4 Halsburys Laws (3rd Edn) 219,
para 497.
As to instances of public purposes, see 4 Halsburys Laws (3rd Edn) 228, para 509; and for cases on the subject, see 8
Digest (Repl) 346, 266276.
As to the admission of extrinsic evidence in relation to the ascertainment whether there is a charitable trust, see 4 Halsburys
Laws (3rd Edn) 277, para 576.

Cases referred to in judgment


Delius Will Trusts, Re, Emanuel v Rosen [1957] 1 All ER 854, [1957] Ch 299, 3rd Digest Supp.
Holburne, Re, Coates v Mackillop (1885), 53 LT 212, 1 TLR 517, 8 Digest (Repl) 346, 270.
Hummeltenberg, Beatty v London Spiritualistic Alliance, Re [1923] All ER Rep 49, [1923] 1 Ch 237, 92 LJCh 326, 129 LT 124, 8
Digest (Repl) 314, 4.
Income Tax Special Purposes Comrs v Pemsel [1891] AC 531, 3 Tax Cas 53, HL, 8 Digest (Repl) 312, 1.
1049
National Anti-Vivisection Society v Inland Revenue Comrs [1947] 2 All ER 217, [1948] AC 31, sub nom Inland Revenue Comrs v
National Anti-Vivisection Society, 28 Tax Cas 311, HL, 8 Digest (Repl) 313, 3.
Royal Choral Society v Inland Revenue Comrs [1943] 2 All ER 101, 112 LJKB 648, 169 LT 100, 25 Tax Cas 263, CA, 28 Digest
(Repl) 318, 1400.
Shaws Will Trusts, Re, National Provincial Bank Ltd v National City Bank Ltd [1952] 1 All ER 49, [1952] Ch 163, 8 Digest
(Repl) 330, 124.
Spence, Re, Barclays Bank Ltd v Stockton-on-Tees Corpn [1937] 3 All ER 684, [1938] Ch 96, 107 LJCh 1, 8 Digest (Repl) 344,
259.
Thornton v Howe (1862), 31 Beav 14, 31 LJCh 767, 6 LT 525, 10 WR 642, 54 ER 1042, 8 Digest (Repl) 335, 161.

Cases also cited


Oppenheim v Tobacco Securities Trust Co Ltd [1951] 1 All ER 31; [1951] AC 297.

Adjourned Summons
This was an application by originating summons dated 26 October 1962, by the plaintiff, Westminster Bank Ltd, the sole
executors of the will and two codicils of Arthur Watson Hyde Pinion, deceased (the testator). The application was to determine
whether on the true construction of the will and codicils and in the events which had happened the gifts (a) of the freehold studio
No 22a, Pembridge Villas, London, W 11, together with the contents thereof for the establishment of a museum and (b) of the
residuary estate of the testator as an endowment fund for the maintenance of the museum, or either of the gifts, constituted valid
charitable trusts or were invalid. The defendants were Edith May Pinion, who was a beneficiary under the will and two codicils
of the testator and was entitled to so much of his estate as should pass on his intestacy, and Her Majestys Attorney General. The
facts appear in the judgment.

A J Balcombe for the plaintiff.


G T Hesketh for the first defendant.
B J H Clauson for the Attorney General.
Cur adv vult

27 June 1963. The following judgment were delivered.

WILBERFORCE J read the following judgment. The question raised in this originating summons on which I have reserved
judgment is, briefly, whether a gift in the will and codicil of Arthur Watson Hyde Pinion, the purpose of which is to establish a
museum for the permanent exhibition of the testators own paintings and other objects, is a valid charitable gift.
Mr Hyde Pinion drafted his own will and signed it on 22 February 1956. He described himself as of the studio, 22a,
Pembridge Villas in the County of London, W 11. After giving some legacies he provided as follows:

To my sister Edith May Pinion to receive all the income from my real and personal estate for her life and afterwards to
my trustees to offer my freehold studio 22a Pembridge Villas, London, W.11 to the National Trust with the pictures painted
by myself and others, and my collection of antique furniture to be kept intact in the said studio and shown at an appointed
time by the National Trust in a similar way to their other properties and the income from my real and personal estate to be
applied for the upkeep and maintenance of the said studio and its contents with an income to be paid to a custodian
sufficient to make it worth their while, my sister Edith May Pinion to be the first custodian.

There then follow other provisions relating to the custodianship. He continued:

Having obtained a scholarship at the Patrick Allan Fraser College of Arts at Hospitalfield, Arbroath, Angus, in 1902
after four years tuition.
1050
The paintings done there by myself and afterwards, now in my studio may be considered to have sufficient merit for
preservation along with my copies of portraits of the Hyde family to be kept together with the original portrait of Edward Hyde
first Earl of Clarendon by Lely and the early 17th century portrait of Hamnet Hyde, both of which formerly hung in Hyde Hall,
Hyde, Cheshire and mentioned in Eurwakers East Cheshire. Also the two portraits of myself painted at Hospitalfied and of old
Mrs Munro of the Abbey House, Arbroath, and the portrait I painted of George Herbert Hyde Villiers Earl of Clarendon, K. G., G.
C. M. G., G. C. V. O., when he was Lord Hyde in 1911. Also the portrait of his grandmother, Caroline, Dowager Countess of
Normanton, exhibited in the Royal Academy and the painting of a cardinal, also exhibited in the Royal Academy 1909. Among
the furniture I particularly wish to be retained along with my pictures in the studio are the three needlework chairs 173040 done
by Penelope Hyde and the card table suite formerly in Hyde House, Hyde, Cheshire, and the carved oak chairs with initials I. H.
R. H 1676 formerly in Hyde Hall, Denton, and handed down through each generation to myself. Also I wish to be retained the
silver tea and coffee service with Hyde crest formerly belonging to our grandmother Ann Watson, also the silver cream jug with
her maiden name initials A. B on it, also the silver cup won by our father William Henry Pinion. Also the cabinet containing
collection of old china, ivories and miniatures etc. Any goods and chattels not of an antique nature can be disposed of by my
trustees or removed to a vacant flat for the custodian at 64, St Stephens Gardens, W.2 when vacant possession of one can be
obtained.

He appointed Westminster Bank Ltd and his sister, Edith May Pinion, trustees of his will and he made a first codicil to this will
on 28 July 1961, in which, after revoking the conveyance of his whole estate to his sister, Edith May Pinion, for her life and
giving her an annuity, he said:

Whereas by my will I have bequeathed my freehold studio 22a Pembridge Villas in the County of London where I
have lived since June, 1907, and its contents and the pictures painted by myself, the collection of antique furniture, silver,
porcelain and other objects of art to the National Trust along with the endowment of my estate to maintain the same intact
in the aforesaid studio but revoke this bequest if the National Trust are not willing to accept and carry out these conditions
and instead authorise my executors the Westminster Bank, to appoint a trust who will do so in a small way similar to that of
the Soane Museum in Lincolns Inn Fields.

Then after a provision relating to an annuity, he continued:

Along with the pictures by myself I wish to be kept in my studio all the portraits of Edward Hyde, first Earl of
Clarendon by Lely and Hamnet Hyde formerly in Hyde Hall, Hyde, Cheshire. My grandfather and grandmother James
Lowe Watson and Ann Watson and Jonathan Benison.

There was a second codicil dated 12 August 1961, but this is not material to the present question.
The testator died on 30 August 1961, and Miss Pinion renounced probate and the will was proved by the bank alone. The
studio at 22a, Pembridge Villas, was in the ownership of the testator at his death. It consists of a single storey erection originally
put up as an addition to No 22. The building in itself is undistinguished and shabby, but the studio has been valued for probate at
10,000 on the basis of the development value of the site, which extends to 3,150 square feet. The contents of the studio were
valued for probate at 742. There is a long list of objects of a very miscellaneous character which can roughly be classified as
follows: (a) paintings by the testator himself, of which there are over fifty, the majority being portraits, but there are some small
landscapes and two nude sketches; (b) other paintings, including some originals of the Stuart period, and a 1051 number of
portraits or copy portraits of members of the Hyde family, with which the testator had connexions, including one of Edward
Hyde, first Earl of Clarendon, ascribed by the testator in his will to Lely; and (c) furniture. There are two articles valued at 50
or over, an eighteenth century Italian commode and a Louis XV style commode. A number of pieces are described as Louis XIV
style, Stuart style, Chippendale style and Hepplewhite style. There are the three chairs mentioned in the will described as
three charities of mid-eighteenth century design (no separate value ascribed to them) and there are some articles described as of
the seventeenth century; (d) bric-a-brac, miniatures and curios, including objects of Chinese, Japanese, Burmese, Indian and other
origin and some rugs; (e) china and glass of which those contained in a cabinet are described as mostly damaged. Also, but not
apparently in the studio, there was the silver tea and coffee service and the silver cream jug and cup mentioned in the will. The
bank offered the studio and its contents to the National Trust, as directed by the will, but the Trust did not accept the bequest.
The summons, raising the question as to the validity of the bequest, first came before me on 8 March 1963. There was read
on that occasion an affidavit by Mr F R Perry, an auctioneer and valuer employed by Messers Phillips Son and Neale. He
confirmed that only two of the items in the studio would realise more than 50 each in a sale. He also said this:

The whole collection is far inferior to a collection such as one might find in an antique dealers show room, as the few
genuine antique items in the collection have been so neglected as to render them of comparatively small value. In my
opinion the testators collection would be of no interest or benefit to the public, as a collection, whether housed in its
existing surroundings or exhibited in a museum or other place to which the public has resort, as it is but a haphazard
collection covering no particular period or style. I cannot think of any museum or similar institution which would be
willing to accept such a collection, the quality of the items in which must inevitably be inferior to the articles already on
exhibition in such museum or institution. The cost of exhibiting the testators collection in its present surroundings would
be out of all proportion to any benefit it might confer on the public.

On this evidence it was urged on my by counsel for the first defendant, who is the testators sister and next of kin, that I should
come to the conclusion that the gift was not charitable, since it was lacking the necessary element of benefit to the public. It
seemed to me, however, that if I were to consider an arguments of this kind, it would be necessary for me to have a wider range
of expert evidence as to the artistic or educational value of the collection than the evidence of a valuer for probate, and I was
therefore asked to adjourn this case in order that such evidence should be sought. Counsel for the Attorney General, while
formally objecting that evidence to this effect would be inadmissible, did not oppose the adjournment. On the matter returning to
the court on 22 May 1963, further affidavits were filed by two experts of undoubted authority, Mr P B James CBE, and Mr H C R
Edwards CBE. Both were emphatically of the opinion that the collection, including the testators own paintings, had no
educational value whatever and that they could not constitute a museum or permanent exhibition.
The evidence having been received de bene esse, I have first to decide whether I can take it into account. Counsel for the
Attorney General objected to the reception of any evidence bearing on the artistic or educational worth of the objects included in
the bequest. He submitted that the gift was, on the face of it, of an educational character, and that was that: the gift was within a
recognised category of charitable gifts and the court could not set itself up as a judge of the value of the education provided. This
was particularly the case, he said, because this gift was in the field of the arts, perhaps I should say the fine arts, in which
objective judgments are unattainable. It is no part, he submitted, of the courts 1052 function in deciding whether a gift of artistic
objects is charitable to pronounce judgments of aesthetic value on what the testator has given. This argument, with respect,
appears to me to include a petition principii. I would accept that once it can be established, on a reading of the gift, that it is for
genuinely educational purposes, the inquiry need not be carried any further. As Vaisey J put it: The court ought not to weigh the
respective merits of particular educational methods (Re Shaws Will Trusts, National Provincial Bank Ltd v National City
Bank Ltd ([1952] 1 All ER at p 53; [1952] Ch at p 168)). But the whole question here is whether the gift has an educational
character: that question cannot be answered by a mere reading of the will, even if one them proceeds to a perusal of a catalogue
of the objects given, unless it is the law that a gift of any articles for exhibition is automatically educational. Prima facie,
educational such a gift may be, but then it must be legitimate to resort to evidence to refute or confirm this prima facie
assumption.
The Attorney General sought, as I have said, to uphold the gift as one for educational purposes, but it is possible to consider
it in another way, namely, whether it falls within Lord Macnaghtens fourth category a as a gift for purposes beneficial to the
public. I note that a gift to or for a museum is so classified in 4 Halsburys Laws (3rd Edn), p 229, and probably also in Tudor On
Charities (5th Edn), p 39. Regarding it in this way, I would accept the propositionand I adopt here again the language of
Vaisey J in Re Shaws Will Trusts ([1952] 1 All ER at p 54; [1952] Ch at p 169)that
________________________________________
a See Income Tax Special Purposes Comrs. v Pemsel, [1891] AC 531.

there are many cases where the purpose is so obviously beneficial to the community that to call evidence on the
matter would be absurd.

But is this such a case? In my judgment, it is not. One may contrast the present gift with part of the gift in Re Shaws Will Trusts
([1952] 1 All ER at p 51; [1952] Ch at p 165) which was for

the making of grants [to any body] having for its object the bringing of the masterpieces of fine art within the
reach of the people of Ireland

That carries immediate conviction; the present gift imparts immediate doubt.
A number of authorities was referred to on this point; they fall into two groups. The first group establishes that whether a
gift has the character of being for the public benefit in a manner which the law regards as charitable is to be determined by the
court on the facts of each case, ie, with the aid of relevant evidence; Re Hummeltenberg, Beatty v London Spiritualistic Alliance
([1923] All ER Rep 49; [1923] 1 Ch 237); National Anti-Vivisection Society v Inland Revenue Commissioners. This general
principle is now beyond question. The second group was invoked by counsel for the Attorney General to show that, although that
may be true generally, in the particular case of gifts which, in a broad sense, are for education in the field of art, evidence cannot
be considered, unless, of course, it is sought to show that the gift is contra bonos mores or against public policy. I proceed to
examine these cases.
The first, and that most relied on, is Re Holburne, Coates v Mackillip. In that case a testatrix by her will bequeathed a
collection of pictures, china, plate, books, and articles of vertu to trustees to form an art museum in Bath, to be called the H
Museum, and bequeathed to the trustees a sum of money to be held for the perpetual protection, maintenance, and endowment of
the collection. It appeared that the testatrix intended the museum to be kept in Bath as a public institution for the benefit of the
inhabitants of the city and the public generally. It was held that the gift was a valid gift for charitable purposes. The issues in the
case were first whether the gift was obnoxious to the Charitable Uses 1053 Act, 1735b, as involving a purchase of land, and
second whether the gift was for public purposes only, or whether the museum was to be of a private character. It was not
apparently disputed that the collection was of such merit that its preservation and exhibition would be for the public benefit: it
was evidently a substantial one which had been assembled by Sir Thomas H and may have been well known locally. The
decision upheld the gift as charitable, not on the ground that it was educational but because it was for the benefit of the
inhabitants of Bath.
________________________________________
b 9 Geo 2 c 36.

The next case is Re Spence, Barclays Bank Ltd v Corpn of Stockton-on-Tees, where there was one gift of a collection of arms
and antiques and another gift of money to erect and provide a public hall in which the arms and antiques were to be exhibited.
The main portion of the judgment of Luxmoore J was concerned with the question whether the gift for the hall was good and,
having so decided, the judge said this ([1937] 3 All ER at p 690; [1938] Ch at p 105):

With regard to the gift of the collection of arms and antiques, somewhat different considerations apply. Having regard
to my declaration with regard to the public hall, it is probable that no difficulty will arise. it is necessary, however, to
determine whether it is a good charitable gift. The object of the gift is to allow the collection to be inspected by the public.
This is an educational object, and is, therefore charitable.

There is no indication in the report of the nature of the collection, nor whether there was evidence of its educational value, but the
learned judge must have found or assumed that it had such value unless (which seems unlikely) he was intending to say that a gift
of anything to be inspected by the public is educational.
Then there is Royal Choral Society v Inland Revenue Comrs. The well-known judgment of Lord Greene MR establishes
that education is not limited to teaching or inconsistent with pleasure and that the development of artistic taste is a valuable
objective of education. There are many references in the report to the high standard of the societys work and the Master of the
Rolls evidently thought that this was relevant, and indeed Sir Hugh Allen gave evidence that the performances of the society were
in his view a valuable element in the musical education of the country. That would seem to show that such evidence is relevant.
That was followed by Re Delius Will Trusts, Emanuel v Rosen, where the trust was for the advancement of the musical
works of that composer. The headnote records that it was not in dispute that the standard of Delius works was so high that the
question of his adequacy as a composer did not arise. Roxburgh, J, said this ([1957] 1 All ER at p 857; [1957] Ch at p 306):

I do not find it necessary to consider what the position might be if the trusts were for the promotion of the works of
some inadequate composer. It has been suggested that perhaps I should have no option but to give effect even to such a
trust. I need not investigate that problem, because counsel who have argued before me have been unanimous in the view
that the standard of Delius work is so high that that question does not arise in the present case.

That case at least shows that evidence as to quality may be received and leaves open the question what would be the result if it
could be shown that the work to be encouraged was worthless.
Lastly, in a different field, I was referred to Thornton v Howe, the case on the writings of Joanna Southcote. That case is
comparable, in the field of religion, 1054with the case, in the field of education, of Re Shaws Will Trusts from which I have
already quoted. Once a gift is clearly seen to be in the religious or educational sphere, the court cannot discriminate between
religious or methods of education. It does not prevent the court from ascertaining, if necessary by evidence, whether the gift has
any educational tendency.
In my judgment, therefore, the court can and indeed must in this case receive evidence to support or negative the existence
in this gift of educational value or public benefit. No doubt it must be cautious in the receipt of such evidence: as Lord Simonds
put it, the court will not be astute to defeat on doubtful evidence the avowed benevolent intention of a donor: ( National
Anti-Vivisection Society v Inland Revenue Comrs ([1947] 2 All ER at p 233; [1948] AC at p 65)). Particularly where it is dealing
with a subject matter in the sphere of art or aesthetics it must allow for the difficulty there is in making any secure objective
judgment, for changes in fashion and in taste. It should recognise that the formation of an educated taste is a complex process,
differing greatly as between individuals. It must allow for the differencesvery great differencesof education and taste to be
found among the members of the public who are likely to see the bequest. Nevertheless, making all these necessary allowances,
there must come a point when the court, on the evidence, is impelled to say that no sufficient element of benefit to the public is
shown to justify the maintenance in perpetuity of the subject matter given. A strong and a clear case has to be made before such a
conclusion can be reached. Is this case sufficiently strong and clear? I now proceed to consider the evidence.
I have already referred to Mr Perrys affidavit and I now examine the evidence of Mr James and Mr Edwards. Mr James,
after giving his qualifications, which are undoubted, said that he had inspected the collection, and as to the furniture he said that

the items in the collection could not have been of a lower quality. I have a wide experience and knowledge of the
contents of museums and art galleries throughout this country, and in my opinion no item in that collection except the said
few pieces of furniture would be acceptable to any such museum or art gallery. I am not surprised that the National Trust
has declined the offer of this collection. In particular the pictures and china are quite worthless and the suggestion that they
should be shown in public in London or anywhere else does not bear serious consideration. In my view, the collection has
no educational value whatsoever. I would have expected that a person with the testators voracious appetite for bric-a-brac
would occasionally have acquired some pieces of mediocre quality, but that has not proved to be the case.

Then in his second affidavit he said that he had inspected the silver and also the studio. As to the silver he said: it is a perfect
example of the tastelessness and ugliness of Victorian silver of this date, which in fact is 1895. He then said: The testators
studio it extremely squalid and in very poor surroundings. Mr Edwards has also filed two affidavits. After giving his
qualifications, which are undoubted, he said this:

The pictures consist of the testators own works, nudes and portraits, which by any recognised standard can only by
described as atrociously bad, and also copies after later Stuart portrait painters with a few coeval but negligible works of
the same kind.

He then dealt with their value and continued:

The so-called Lely is not in my opinion by that master. Among the furniture there are a dozen or so genuine English
and continental pieces of the seventeenth and eighteenth centuries which perhaps might be acceptable as a gift to a minor
provincial museumnotably, three eighteenth century 1055 English single chairs covered with contemporary embroidery.
The frames, however, of these chairs are of distinctly mediocre quality. The remainder of the furniture is either spurious or
of low quality or very minor examples of provincial craftsmanship. The items which may be regarded in any was as
desirable would not do more than provide the miscellaneous contents of a single small room. I failed to find among the
china and bric-a-brac one specimen of a quality above what is commonly described as junk. In my view the proposal that
this collection should form a trust is really quite fantastic. I am firmly of opinion that the bequest regarded as an entity
cannot be said to possess any educational value whatever.

In his second affidavit he dealt with the silver in the same way as Mr James and also said that the condition of the studio is
appalling and that the local authority was likely to condemn it.
Both experts were cross-examined on that evidence, but no positive evidence was called to support the merits of the
collection. In neither case was their evidence challenged in so far as it related to the great majority of the objects belonging to the
testator, or as to the collection regarded as a whole, but in each case it was attacked as regards the testators paintings, to a lesser
extent the other paintings, his furniture, or some of it, and his silver. As regards the testators own paintings, it was put that no
expert opinion can be more than an opinion, and a fallible opinion at that. The rejects of one age are the masterpieces of another.
The example was given of Vincent Van Gogh, who only sold one of his paintings during his life for 400 francs, which now might
fetch 40,000. The experts dealt with these suggestions in a very similar, and, to my mind, effective manner. While conceding
that judgment as to artistic merit is not infallible, they said that there is such a thing as the consensus of informed opinion. A case
such as Van Gogh is quite distinct from such a case as the present. Van Gogh was a revolutionary artist, ahead of his time, but
even in his lifetime many informed people existed who considered him a genius. The testator (said Mr Edwards) was in quite a
different category: he was an inconceivably bad academic artist of whom there have been hundreds at any one time. All experts
would agree that artistically the testators paintings were valueless. As to the other pictures, Mr James was clear that the so-
called Lely was a recent copy; that such of the Stuart period paintings as are genuine are badly painted and of no quality.
As to the furniture, Mr James thought that some twelve pieces might be acceptable in a minor museum: they were not
sufficient to constitute a museum in themselves. They were such minor examples that their educational value was practically nil.
For an exhibition not less than fifty pieces are required. Mr Edwards said that to treat twelve pieces as worth exhibiting would be
stretching it very far. There might, on a liberal assessment, be another dozen of provincial craftsmanship but many of these had
been played about with. He could not say with any confidence that any one piece was of metropolitan origin. He could not
conceive of anyone going to be educated by twelve pieces of third-rate furniture. But he, like Mr James, had been directing his
evidence to the merits of the collection as a whole, and, when asked whether some of the best objects could not be picked out, did
not rule out the possibility of some instruction being conveyed if these were properly exhibited and labelled. As to the silver, Mr
James said that the tea set dated from 1895a bad period. Finally, Mr James was asked to consider the possibility that further
items might be added and said that objects of any worth would be ruined by juxtaposition with this junk.
Where then does that evidence lead? I was invited to consider first the possibility that the collection might be maintained as
a whole, as indeed was, substantially, the testators intention. The argument of the next of kin as to this was that such few articles
of merit as there are would be so completely submerged in the sea of junk that nobody could derive any profit from seeing it.
One may accept this, but I do not think that maintenance in toto is the only course open to 1056 the trustees. They have power to
dispose of, or to remove to the custodians flat, any articles not of an antique nature, which evidently gives them a fairly wide
discretion. I think therefore that it would be fair to assume that the testators wishes would be carried out by exhibiting the best
pieces of furnitureperhaps even some furniture less good than this; the Stuart period paintings; some of the better copies of
paintings and those connected with the Hyde family, a family, after all, with a place in history. The testators own compositions
could hardly be excluded, but it might be legitimate not to hang them all at one time. All these objects, it must be assumed,
would be properly labelled in such a way as to convey their date and place of origin and any historical affiliations. An even more
favourable character can be imparted to the collection if one assumes that the trustees were from time to time to purchase
additional items of merit, but I do not think that it is legitimate to make this assumption, not only because the will contains no
power to do this, but because the evidence does not show that there will be enough money in hand after providing for the,
necessarily considerable, administrative expenses.
Taking, then, the middle view of what the trustees are likely to do, ought I to hold that the element of benefit to the public,
educational or otherwise, is so wholly absent that the gift is void? Or is this rather a case of possibly a disproportionate
endowment to achieve a small benefit, but still a case where there is a small benefit to be anticipated for the public? With
considerable hesitation, I think that this is a case of the latter class. It seems to me that there is just enough, given proper and
skilled exhibition, in the collection to make a contribution to the formation of artistic taste to justify it. It may do no more than
interest those who see it in styles of furniture and portraiture and encourage them to go further and to look for better specimens
both of furniture and painting in, say, the Victoria and Albert Museum or the National Portrait Gallerythis irrespective of
whether they might learn something from the testators own struggles with the pictorial medium. The contribution will, no doubt,
be a small one; it may even be out of proportion to the resources locked up in preserving it, but I do not think that the court can
measure the relation of benefit to expenditure and say that the former is or is not a justified use of the latter. Once it is found, as I
feel obliged to find, that the evidence does not establish with sufficient certainty that no recognisable benefit to the public is
secured by the gift, in a field which the law regards as charitable, the court must declare, as I do, that the gift is charitable and
valid.
To reach this conclusion does not involve a disregard of the experts evidence. I feel a considerable sympathy with them in
their refusal to accept the testators gift as measuring up to their standards of what a museum should be. I have little doubt that
the studio, when arranged by the trustees even in the most attractive and selective manner, will not nearly qualify to be called a
museum as Mr James and Mr Edwards understand that word, but I think that the standard for a valid charitable gift is not so
high as that. I cannot say that to provide a room, with a number of objects possessing some degree of historical and artistic
interest, open to the public, will not be a benefit to the public: I think that it may, and I uphold the gift.

Declaration accordingly.

Solicitors: Scadding & Bodkin (for the plaintiff and the first defendant); Treasury Solicitor (for the Attorney General).

Jennifer Sandell Barrister.


1057
[1963] 2 All ER 1058

Wickwar (Inspector of Taxes) v Berry


TAXATION; Income Tax, Deduction in computing profits

COURT OF APPEAL
SELLERS, DONOVAN AND RUSSELL LJJ
12 JUNE 1963

Income Tax Deduction in computing profits Technical education payments Fees for employees (sons of taxpayer) attending
agricultural college Income Tax Act, 1952(15 & 16 Geo 6 & 1 Eliz 2 c 10), s 140(1).

On their leaving school a farmer employed his two sons on his farm as full-time farm labourers receiving wages at normal
agricultural rates, and a year later in each case he sent them to a technical college of a county institute of agriculture, paying them
30s a week for the work that they were able to do on his farm during their course, and full wages when they worked full-time in
the holiday. He paid 167 in fees to the county council institute of agriculture for the purposes of the sons technical education
without imposing any condition that the sum was to be used only for the general purposes of the institute, but it was inferred that
it was in fact used by the institute for its general purposes.

Held The 167 should be allowed, by virtue of s 140(1) a of the Income Tax Act, 1952, as a deduction in computing the profits
of the farmer from his farming business, since the words any payment to be used for the purposes of technical education in s
140(1) covered in their natural meaning this payment, notwithstanding that the purpose for which it was made was the technical
education of particular individuals rather than technical education generally (see p 1061, letters c and h, post).
________________________________________
a Section 140(1) of the Income Tax Act, 1952, provides
Notwithstanding anything in s 137 of this Act, where a person carrying on a trade makes any payment to be used for the purposes of technical
education related to that trade at any university or university college, or at any such technical college or other similar institution as may for
the time being be approved for the purposes of this section by the Minister of Education, the payment may be deducted as an expense in
computing the profits or gains of the trade for the purposes of income tax.

Appeal dismissed.
Notes
As to payments by a trader for technical education deductible in computing profits for income tax purposes, see 20 Halsburys
Laws (3rd Edn) 185, para 322.
For the Income Tax Act, 1952, s 140(1), see 31 Halsburys Statutes (2nd Edn) 138.

Cases referred to in judgment


Cape Brandy Syndicate v Inland Revenue Comrs [1921] 1 KB 64, 12 Tax Cas 358, affd [1921] 2 KB 403, 90 LJKB 461, 125 LT
108, 12 Tax Cas 358, 28 Digest (Repl) 410, 1822.

Cases also cited


Whimster & Co v Inland Revenue Comrs (1925), 12 Tax Cas 623, 1925 SC 20.

Appeal
The respondent taxpayer appealed to the Special Commissioners of Income Tax against an additional assessment in the sum of
167 made on him under Case I of Sch D to the Income Tax Act, 1952, for 195859. That sum was the amount expended by the
taxpayer for the whole-time technical education of his two sons, and the grounds of the appeal were that that expenditure was
allowable as a proper deduction from his profits under s 140 of the Act of 1952. The following facts were found. The taxpayer
lived and carried on business as a farmer at Ashes Farm, Longridge. He had two sons, David and Anthony, who left school
respectively in December, 1956, and December, 1957. On leaving school both sons commenced full-time employment as farm
labourers on the taxpayers farm and they were in such employment at all material times; both were paid wages at the normal
agricultural rate with the necessary statutory deductions. Both sons completed the full time certificated course in agriculture at
the Lancashire County Institute of Agriculture at Winmarleigh College, a technical college approved by the Minister of Education
for the purpose of s 140 of the Act of 1952. David attended the course at the college from September, 1957, to July, 1958,
1058and Anthony from September, 1958, to July, 1959. While attending the course, each received 30s per week wages for the
work they were able to do on the taxpayers farm. During the holidays each worked full time on the farm and was paid normal
wages. In the year of assessment the taxpayer paid 162 fees in respect of David and 5 in respect of Anthony to the county
councils institute for the purposes of the sons technical education through the medium of the courses. Those sums were not paid
subject to a condition that the money was only to be used for the general purposes of the institute, but in the absence of any
evidence showing how money paid to the institute in respect of fees was primarily used or how in particular the money paid by
the taxpayer was in fact used, the commissioners inferred that it was used by the institute for its general purposes.
The taxpayer contended that by virtue of s 140 of the Income Tax Act, 1952, the payment of 167 fees to Lancashire County
Council for the courses in agriculture at Winmarleigh College was properly deductible as an expense in computing his profits or
gains as a farmer for income tax purposes for 195859, and that, when considering an expense under the section, the general rules
as to deductions under s 137 should be disregarded. The Crown contended: (i) that the words where a person makes any
payment to be used for the purposes of technical education in s 140 meant where a person makes any payment for the general
purposes of technical education per se, and that reading s 140 with s 137 the right conclusion to come to was that, for the
purposes of s 140, the payments must be not of a kind where the payer received a direct commercial return either in kind or in
services; (ii) that the expenditure fell on the taxpayer as a parent and not as a farmer, and the words to be used for the purposes
of technical education were not apt to describe the payment of fees for the education of his sons even if the education was in his
own field of trading; (iii) that the taxpayer must establish that Lancashire County Council should only use the payments for
technical education; and (iv) that the effect of the words notwithstanding anything in s 137 was to produce a situation where
there was no specific prohibition in the Income Tax Act, and in such case the expense should be judged on ordinary commercial
principles. Private expenditure would not be a proper deduction on sound commercial principles and it followed that nothing in s
140 contradicted that.
The commissioners decided that the words in s 140 were apt to cover the circumstances of the appeal. They rejected the
interpretation suggested by the Crown that only payments made for the general purposes of secondary education were allowable
as a deduction under s 140 as being too narrow, and they did not think that such payments made for the technical education of
individuals would only be allowable if the payments fulfilled the requirements of s 137. They allowed the appeal and discharged
the assessment. On 7 November 1962, Ungoed-Thomas J dismissed the Crowns appeal by way of Case Stated against that
decision. The Crown appealed to the Court of Appeal.

R E Borneman QC, AS Orr QC, and J R Phillips for the Crown.


E W Griffith for the taxpayer.

12 June 1963. The following judgments were delivered.

DONOVAN LJ delivered the first judgment at the request of Sellers LJ: I need not repeat the facts, which are set out in the Case
and in the judgment below of Ungoed-Thomas J. The words to be construed are the words of s 140(1) of the Income Tax Act,
1952, which allow a deduction, in computing trading profits, in respect of any payment made by a trader which payment isand
here I quote the section:

to be used for the purposes of technical education related to that trade at any university or university college, or at
any such technical college or 1059 other similar institution as may for the time being be approved for the purposes of this
section by the Minister of Education

The institution concerned in this case, viz, the Lancashire County Institute of Agriculture at Winmarleigh College, has been so
approved. The sums totalling 167 paid by the respondent taxpayer to that college were obviously intended by him to be used for
the technical education of his two sons, and one may reasonably assume that they were so used, though no doubt they would lose
their identity as part of the general funds of the institute. The commissioners have drawn the inference that the money was used
by the institute for its general purposes (which are presumably indicated by its name), and this inference is not disputed.
But it is said that s 140(1) has no reference to a case where education is bought for specific persons, as here. The section
refers, and refers only, to payments which are to be used for technical education generally, and which produce no quid pro quo to
the payer. I asked counsel for the Crown to give an illustration of a payment which would qualify as a deduction under s 140,
and he gave the example of a farmer sitting down and writing a cheque to an approved agricultural college and saying in a
covering letter, this is to be used for the purposes of technical education in agriculture, and saying no more. In other words, it
is argued that the purpose of s 140 is simply to enable industry to develop by contributions from traders without any quid pro quo
moving to them.
This argument has found favour with neither the Special Commissioners nor the judge, and I too feel unable to accept it.
The question is one of construction, and the crucial words are, any payment to be used for the purposes of technical education.
For the Crown it is argued that those words connote technical education generally, for otherwise the language would have been,
any payment for technical education. The addition of the words, to be used for the purposes of eliminate a case, it is said,
such as the present, where the payment is for the education of specific persons. I appreciate the argument, but I think that the
suggested interpretation is too strained and too narrow for acceptance. It is to be noted that the section does not require that the
money is to be paid to the university or college direct. So that the section prima facie envisages payments being made to persons
not necessarily being the person who is to provide the education. Who could such other persons be? They might be trustees
administering payments made to them by a trader to be applied for the technical education of a beneficiary; they might be the
very person to be so educated. The fact that the section leaves these possibilities open is, I think, an obstacle in the way of the
Crowns interpretation.
Moreover, if a trader pays money, as this trader did, for the technical education of two employees, I find it difficult to see
why such a payment is not accurately described as a payment for the purposes of technical education. If a father with two boys at
a public school were asked: How much have you paid out for the purposes of education this year?, he is hardly likely to reply:
Nothingthough I have paid out large sums for education. Furthermore, I do not see why the existence of some quid pro quo
for the payer should take a payment out of the section. It is a section confined to a trader, the technical education must be related
to his particular trade, and it must be specially requisite for persons employed in the class of trade to which the particular trade
belongs. Obviously, a quid pro quo is going to accrue to, or at least is expected by, anybody who makes such a payment; that
quid pro duo being in the form of better-equipped employees, whether they be employees whom he has sponsored or some
general pool of employees produced by the efforts of the technical college. It is, I think, more likely that Parliament had this in
mind rather than the encouragement of mere philanthropy. How else, indeed, can money be used for the purposes of education
except by educating persons?
The employees in the present case were the two sons of the taxpayer. Nothing turns on this feature of the case, although no
doubt it is a more attractive case 1060 for the Crown to argue. But it is conceded that their argument would be the same if the
two employees were strangers. Since they are sons, however, the Crown says that it would be anomalous if a deduction from
trading profits could be secured by their father for their college fees, and at the same time he could, but for the fact that the period
of education was nine months instead of two years, also get the child allowance under s 212 of the Act of 1952. But anomalies of
this sort, if this be one, are not rare in this field, and, while they are proper to note, they cannot displace a fair construction of the
language of a particular section designed to confer a particular benefit on a trading taxpayer.
Counsel for the Crown also sought to draw some support for his argument from the language of s 335 of the Act of 1952,
granting deductions from trading profits for moneys expended on scientific research or paid to a scientific research association, or
for sums paid to be used for scientific research to any approved university, college or research institution. I do not detail this
argument, for I have failed to derive any assistance from it: and it appears to have been employed for the first time in this court.
I think that the words which we have to construe in s 140(1) are apt, in their ordinary meaning, to cover the case now before us. I
am not able to subscribe to the view that this is a result so startling that Parliament cannot have intended it, and that therefore the
section should be given a construction which will avoid it; and merely on its merits I find the alternative construction proposed
by the Crown unconvincing. In particular, in my opinion, far too much weight is being rested on the words for the purposes of.
We are asked to construe them as if they meant the same thing as, for the promotion of, and I think that it would be wrong to do
so.
It is perhaps useful to recall what Rowlatt J said in Cape Brandy Syndicate v Inland Revenue Comrs ([1921] 1 KB at p 71;
12 Tax Cas at p 366) about the construction of a taxing Act. He said:

in a taxing Act one has to look merely at what is clearly said. There is no room for any intendment. There is no
equity about a tax. There is no presumption as to a tax. Nothing is to be read in, nothing is to be implied. One can only
look fairly at the language used.

I think that those words are also true in considering whether a taxpayer comes within the terms of a relieving section such as s
140(1). I agree with the Special Commissioners and with the judge that this taxpayer does, and accordingly I would dismiss this
appeal.

RUSSELL LJ. I agree that this appeal should be dismissed, and I wish to add nothing to what has fallen from Donovan LJ.

SELLERS LJ. I also agree with what Donovan LJ has said, supporting as it does the clear judgment of Ungoed-Thomas J from
whom this appeal comes. I would add only that it seems to me that this section was designed to encourage technical education,
and the interpretation which the Special Commissioners, the judge and now this court have put on it would be in complete
harmony with that object, giving relief to those who are prepared to further technical education not merely indirectly, as I
understand the argument for the Crown would have it, but directly; ie, the section is concerned not with to whom the money is to
be paid but with the purpose for which it should be paid, viz, for technical education. These two sons of the taxpayer benefited
by such technical education, and I think that the words are wide enough, as my lord has said, to cover the circumstances of that
expenditure. I, too, would dismiss this appeal.

Appeal dismissed. Leave to appeal to the House of Lords refused.

Solicitors: Solicitor of Inland Revenue; Hatchett Jones & Co agents for Hodgson & Sons, Preston (for the taxpayer).

F A Amies Esq Barrister.


1061
[1963] 2 All ER 1062

Re All Saints, Leamington Priors


ECCLESIASTICAL

COVENTRY CONSISTORY COURT


CHANCELLOR GAGE
21 NOVEMBER 1962

Ecclesiastical Law Ornament Sanctuary lamp Faculty to authorise placing on altar.

The vicar and churchwardens of a parish, with the approval of the parochial church council, petitioned for a faculty to authorise
the placing of a silver sanctuary lamp on the altar of a memorial chapel in the church. One parishioner opposed.
Held A sanctuary lamp was not an illegal ornament, and a faculty for the placing of the lamp on the altar would be granted in
the present case.
Re St Saviours, Walthamstow, ([1950] 2 All ER 812) applied.

Notes
As to the illegality of the ceremonial use of lights, see 13 Halsburys Laws (3rd Edn) 343, para 782, note (e); and for cases on the
subject, see 19 Digest (Repl) 466467, 28832893.
As to a faculty for church furnishings, see 13 Halsburys Laws (3rd Edn) 415, para 922, note (c); and for cases on the
subject, see 19 Digest (Repl) 493, 32253233.

Cases referred to in judgment


St Saviours, Walthamstow, Re [1950] 2 All ER 812, [1951] P 147, 19 Digest (Repl) 467, 2892.

Petition
This was a petition by the vicar and churchwardens of the parish church of All Saints, Leamington Priors, Warwickshire, for a
faculty to place a silver sanctuary lamp on the altar of the memorial chapel in the church. The proposal was approved by the
parish generally, but was opposed by one parishioner.

J A Stancer, solicitor, for the petitioners.


The party opponent appeared in person.

21 November 1962. The following judgment was delivered.

THE CHANCELLOR. The sanctuary lamp in question has been produced and, if I may say so, it is a beautiful object. It was
given by the widow of a former incumbent as a memorial to her husband; It strikes one as a pious thought that she should want to
give such a fitting memorial, and a natural one, in memory of her husband; and everyone, save one, in the parish approves of it.
The donor herself is held in great esteem and affection in the parish, and I am told that it meets with the unanimous approval of
the parochial church council.
One parishioner, McFarlane, has opposed the petition on grounds which I am bound to say are not easy to understand. It is
quite clear to me that there is no reason at all why a sanctuary lamp should not be placed on an altar. It is unusual, no doubt, but
the solicitor for the petitioners has quoted Re St Saviours, Walthamstow, in which Sir Philip Wilbraham-Baker-Wilbraham, Dean
of Arches, decided that it was not illegal to have six lights on the altar, or behind the altar.
The argument of the solicitor for the petitioners is that this lamp is no more than a light to be placed on the altar, and that if it
is legal to have light in the form of a candle on the altar, it is certainly legal to have light in the form of a sanctuary lamp. I agree
with this argument.
If a sanctuary lamp were an illegal ornament, then almost every church in this diocese would be doing wrong, since almost
every church in this diocese has a sanctuary lamp. In my view it is not an illegal ornament, and indeed the objector himself says
that he would raise no objection to the lamp being suspended on chains in the chapel and above the Holy Table. In these
circumstances I am of the opinion that there is nothing in law which prevents me from granting this faculty, and in the exercise of
my discretion I think it is a very proper faculty to grant.
1062
A great deal has been made of the fact that McFarlane is the only person who opposes it, and that the majority of
parishioners would be happy to include this singularly beautiful ornament in their church and to see it upon the altar. Though it
may be unusual, it is found on the altar of many churches, and I shall exercise my discretion in favour of the petitioners, and
grant the faculty in the terms as prayed.

Faculty decreed.

Solicitors: Wright, Hassall & Co Leamington Spa (for the petitioners).

A T Hoolahan Esq Barrister.


[1963] 2 All ER 1063

Heywood and Another v B D C Properties Ltd


LAND; Land Registration

COURT OF APPEAL
WILLMER AND DANCKWERTS LJJ
14 JUNE 1963

Land Charge Vacation of entry in register Interlocutory motion Land charge registered by defendants in respect of an estate
contract evidenced by specified correspondence Action by plaintiffs for declaration that no contract existed and for vacation of
entry in land charges register Motion in action for order for vacation of entry granted Land Charges Act, 1925(15 & 16 Geo
5 c 22), s 10(8) RSC, Ord 55, r 14A.

Where a land charge has been registered in respect of an estate contract alleged to have been created by specified correspondence,
and the correspondence clearly shows that no binding contract for the sale of the land has been entered into thereby, an order for
vacation of the registration of the land charge may properly be made (pursuant to s 10(8) of the Land Charges Act, 1925) on
application by interlocutory motion in an action, even though the relief claimed in the action is merely a declaration that no
contract existed and vacation of the entry of the land charge.
Re Engalls Agreement ([1953] 2 All ER 503) distinguished.
Decision of Pennycuick J (ante p 358) affirmed.

Notes
As to vacation of registration of a land charge, see 23 Halsburys Laws (3rd Edn) 77, 78, para 161; and for a case on the subject,
see 38 Digest (Repl) 882, 926.
For the Land Charges Act, 1925, s 10(8), see 20 Halsburys Statutes (2nd Edn) 1078.
For RSC, Ord 14 and Ord 55, r 14A, see the Annual Practice.

Cases referred to in judgment


Bailey v Ford (1843), 13 Sim 495, 12 LJCh 482, 1 LTOS 286, 36 Digest (Repl) 626, 1900.
Bailey (Malta) Ltd v Bailey, The Times, 29 May 1963.
Engalls Agreement, Re [1953] 2 All ER 503, 38 Digest (Repl) 882, 926.

Appeal
The defendants appealed against an order of Pennycuick J made on 23 April 1962, on interlocutory motion in an action, reported
ante p 358, whereby it was ordered (so far as relevant to be stated here) that the entry of a land charge on the register of land
charges relating to an alleged contract between the plaintiffs and the defendants be vacated. The action was brought for a
declaration that no contract for the sale of certain land subsisted between the plaintiffs and the defendant and for an order
vacating the registration of two land charges both registered in respect of one alleged estate contract comprised in
correspondence. The grounds of appeal were: (i) that the judge had no jurisdiction to make the order as an interlocutory order;
(ii) that, even if there were jurisdiction, the order should not have been made having regard to the pending dispute between the
parties and to all the circumstances of the case; (iii) that the order was made contrary to the practice and procedure of the
Chancery Division; (iv) that 1063 by the making of the order the defendants were deprived of any opportunity to enforce or
obtain an order for specific performance of the contract which they alleged existed if they should succeed at the trial; and (v) that
the order was an unreasonable exercise of the discretion, if any, to make it.

P A W Merriton for the defendants.


J P Brookes for the plaintiffs.

14 June 1963. The following judgments were delivered.

DANCKWERTS LJ delivered the first judgment at the request of Willmer LJ: This is an appeal against an order of Pennycuick
J made on 23 April 1963, by which he directed the vacation of a land charge registered under the Land Charges Act, 1925, on an
interlocutory motion made in an action which had been begun by the plaintiffs, the motion being based on the provisions of s
10(8) of the Land Charges Act, 1925. The land charge in question was an estate contract claiming to refer to a contract made in
correspondence extending between the two dates mentioned in the application for registration, and it was a land charge in class C
(iv) of the register of land charges. The position was that the plaintiffs, the two Heywoods, and the defendants had been
negotiating for the sale and purchase of certain property at a price of 14,000; and the plaintiffs have brought an action for a
declaration that no contract exists for the sale of the property to the defendants and for an order vacating the registration of the
charge under the Land Charges Act, 1925. It appears that, the plaintiffs case being that there is no such contract, they are
negotiating with different purchasers, and the existence of the land charge is a bar to their proceeding successfully with those
negotiations.
Pennycuick J made an order because he considered that it was plain on the correspondence, which was referred to in the land
charge, that no possible successful case could be made out that there was any contract in favour of the alleged purchasers. This,
of course, is a summary method of proceeding, and one of the objections that has been taken is that not only does it mean giving
to the plaintiffs the relief which they seek to obtain at the trial of the action, but also that, if there is any triable point to obtain at
the trial of the action, but also that, if there is any triable point or any arguable point, it is not appropriate to grant relief under the
summary provisions which are envisaged by the Land Charges Act, 1925, s 10(8). That objection is supported by a decision of
Vaisey J in Re Engalls Agreement, where, application being made by originating summons (which is, according to RSC, Ord 55,
r 14A, the appropriate way of proceeding to obtain vacation of a land charge pursuant to s 10(8) of the Act) and there being some
doubtful issue, Vaisey, J, took the view that the proper method of proceeding was by an action and not by this summary
procedure. In Re Engalls Agreement the point at issue was whether the contract which had been made, quite properly, had been
determined by the appropriate procedure open to the vendors in that case. Vaisey J considered that he ought not to deprive the
defendant of his chance to have that tried in an action. In that case no action had been started.
In the present case the plaintiffs issued their writ on 28 December 1962, the statement of claim was delivered on 22 January
1963, and the defendants put in their defence on 8 February 1963. The statement of claim in substance pleaded a case that any
arrangements between the plaintiffs and the defendants were subject to contractie, subject to formal contractwhich, it is
well established, means that there is no final and binding contract until a formal contract has been drawn and has been signed and
exchanged by the respective parties. The defence was

the defendants deny that there was any contract between themselves and the plaintiffs subject to contract.

Pennycuick J found that sentence difficult to understand, but, though it is very inartistically drawn, I can see quite clearly what
the defendants meant. 1064They meant that there was a contract, and that it was not subject to any such condition as I have
already indicated.
Pennycuick J reached the conclusion on correspondence that it was quite impossible for the defendants to establish any
binding contract relating to the sale of this land. As the registration refers to the correspondence, of course, it is simply a matter
of looking at the correspondence to see whether the land charge was justified. I think that the conclusion reached by the judge in
the circumstances was the correct one in that respect. The correspondence which we have in the bundle before us starts with a
letter of 25 June 1962, written by the plaintiffs solicitors to the defendants solicitors, and the first sentence is:

We understand that you act for the above company who have agreed subject to contract, to purchase the above land
from our clients, Messrs. Heywood.

Now that attaches to the whole transaction the phrase subject to contract, and it is never alleges anywhere in the
correspondence that the plaintiffs solicitors were incorrect in making the assumption that they did. It is clear, when the
correspondence is read through, that what happened was that, while the solicitors who had been instructed to draw up a formal
contract were engaged in that operation, the purchasing company found difficulties in getting planning permission, and also in
paying, or at any rate they did not desire to pay as quickly as had been originally planned, and they were making further
negotiations with the object of altering the original terms which had been provisionally agreed between the parties. Eventually
the negotiations were broken off by the plaintiffs solicitors. It is quite plain to me that the judge reached the right conclusion,
and that there was on the correspondence no possible binding contract between the plaintiffs and the defendants. That being so,
of course, the land charge was not really effective and ought not to remain. It is noteworthy that the land charge was actually not
put on the register until 27 September 1962, when difficulties had already arisen and disputes were imminent between the parties.
The next point is whether it is right for an order to be made on a summary application by motion in the action, or whether
the matter ought to be left for decision at the hearing of the action. If there were really an arguable point, it would, I think, be
clear, as Vaisey Js decision established (Re Engalls Agreement, [1953] 2 All ER 503), that it would not be proper to dispose of
the land charge summarily in this manner, and that the matter should be left to be dealt with at the hearing of the action. In this
case, if that were the situation, it would be right and proper, I should have thought, to adjourn the application and to direct it to
come on at the hearing of the action which had been begun by the plaintiffs.
Other objections are taken. It is said on behalf of the defendants that the plaintiffs could have proceeded under RSC, Ord
14, and obtained a summary judgment in the action in that way. I do not think that they were bound to do that, and I do not think
that argument carries the matter any further. Secondly, it is pointed out that the method of proceeding under s 10(8) of the Land
Charges Act, 1925, is the subject of RSC, Ord 55, r 14A, and an originating summons or, if there is already an action on foot, an
ordinary summons is the method which is provided for obtaining the vacation of a land charge. But it is clear in the terms of the
rule in question that the method of proceeding is permissive; a party may take that particular course. Therefore, on the face of it,
it does not seem necessarily to deprive the plaintiffs of the alternative method of proceeding by motion in the action.
A further objection has been taken, however, to procedure by motion in that the time in which it comes before the court is
very much shorter than if the procedure of summons is adopted, which, of course, comes in the first instance before the master in
the Chancery Division, and is adjourned by him usuallycertainly if the parties require itto the judge; whereas, in the case of a
motion, 1065eight days only is allowed. There is no real substance in that, however, because, if the defendants have any real
case and are not able to complete the evidence which they want to bring on the motion on the first hearing of the motion by the
judge or indeed by the master, they can always ask for an adjournment, and such an adjournment would invariably, I think,
according to the practice, be granted. So there is really no substance in the objection that the motion comes before the court too
quickly for the matter to be dealt with adequately.
Finally, another objection is taken that to vacate the land charge in this case is equivalent to giving judgment on an
interlocutory application in the action in favour of exactly the relief which the plaintiffs would obtain at the eventual hearing of
the action in the ordinary way. It seems to me, however, that that is largely a matter which depends on the circumstances, and
partly a matter of convenience. The position is stated very clearly by Sir L Shadwell, V-C, in the comparatively old case of
Bailey v Ford, decided in 1843. In his short judgment he says ((1843), 13 Sim at p 495):

Although the general rule is that the court will not grant, on motion, that relief which ought to be granted at the
hearing, yet it will do so in some cases.

That was a partnership case, in which it was desired to obtain the sale of the partnership property. The Vice-Chancellor continued
((1843), 13 Sim. at p 495):

It appears that the affairs of the partnership are daily growing worse, and there is no reason to infer, from what is
stated in the defendants answer, that they will improve. Under those circumstances, I shall make an order in the terms of
the notice of motion.

The same sort of position arose in the recent case of Bailey (Malta), Ltd v Bailey (The Times, 29 May 1963), where an
application was made for the delivery of documents which it was essential for the company to have in order to continue the
business which was under consideration. In that case the Court of Appeal upheld the order of Pennycuick J granting the relief,
though the objection was taken that it was the same relief as was to be given at the hearing of the action. Therefore, so it seems
to me, this is not a fatal objection and it all depends on the circumstances and the convenience of the matter. The objection, in
my view, fails. In my view, the judge was correct in the course which he took, and I think that the appeal should be dismissed.

WILLMER LJ. I am of the same opinion. It seems to me that this is a clear case and that the judge was plainly right. The
matter which was in fact entered on the register was in these terms:

An estate contract imposed or created by correspondence made between Messrs. Beach and Beach, 5, Maddox Street,
London, W.1, solicitors for B.D.C. Properties, Ltd., and Messrs. Dunn and Baker, of 21, Southernhay East, Exeter, Devon,
from June 25, 1962, to Sept. 14, 1962.

There the correspondence relied on as constituting an estate contract was precisely specified and limited to the period between the
dates given. With regard to that correspondence, the judge in his judgment said (ante at p 359):

I will not go through that correspondence in detail. It will be sufficient for me to say that it is clear beyond argument
that nothing in that correspondence creates a binding contract between the plaintiffs and the defendants. The whole
correspondence consists of negotiations subject to contract, for the sale of the land, those negotiations being conducted on
the basis that a contract would in due course be entered into and that the parties would not be bound until a contract was
entered into.
1066

I entirely agree with what the judge said in that passage; indeed, the contrary has not really been argued, for counsel for the
defendants has admitted that on the correspondence as it stands he cannot contend that a binding contract was entered into. If that
be right, then it seems to me that the entry which is on the register ought not to be allowed to stand.
However, counsel for the defendants has indicated that it may be possible, with the aid of documents which are not at
present available, and possibly with the aid of oral evidence from a witness, to prove that there was some other contract.
Assuming for the sake of argument that he is justified in that contention, it does not seem to me to affect the question whether this
particular entry in these terms ought to be allowed to stand. Agreeing, as I do, with the judge that on the correspondence it is
quite clear that the alleged contract registered was not a contract, I also agree with him in the conclusion he reached that the
previous case of Re Engalls Agreement was clearly distinguishable.
The question then remains whether the plaintiffs have selected the right remedy. That involves partly a question of form, as
I see it, and partly a question of substance. The statute, which is the Land Charges Act, 1925, lays down by s 10(8) that

the registration of a land charge may be vacated pursuant to an order of the court or a judge thereof.

The statute itself does not prescribe any procedure for achieving that object. But RSC, Ord 55, r 14A, does set out a procedure.
However, as Danckwerts LJ has already pointed out, that procedure is only permissive, and in no sense mandatory. What is
contemplated is a procedure by way of summons, either an originating summons or a summons in an action. Here, there being an
action on foot, I apprehend that on the defendants argument the correct procedure would have been by way of a summons in the
action. That being so, it seems to me the merest technicality to say that what can be done by summons cannot be done by way of
motion. As I have pointed out, the statute does not restrict one to any particular method of procedure; nor does the rule, to which
I have referred, lay down that that procedure is to be followed to the exclusion of any other. So much for the question of form.
The matter of substance to which I referred is whether it is proper on an interlocutory applicationthis would apply equally
whether the application were by way of motion or by way of summonsto make an order which has the effect of granting the
plaintiffs substantially the whole of the relief claimed in the action. That is the real complaint of substance which has been put
before us. My lord has already referred to the two cases cited to us, Bailey v Ford and the recent case before this court of Bailey
(Malta), Ltd v Bailey (The Times, 29 May 1963). Those cases, I think, do show that it is only in unusual circumstances that the
court ought to take the step of granting substantially the whole relief claimed in the action on an interlocutory application. But
they equally show that that is a procedure which, in a proper case, is available. In my judgment, having regard to the admitted
fact that the alleged contract registered was no contract at all, this is one of those cases in which it is proper and appropriate to
grant by way of interlocutory application the relief which the judge has granted. In those circumstances I agree that the judge
came to the right conclusion, and that the appeal should be dismissed.

Appeal dismissed.

Solicitors: Beach & Beach (for the defendants); Arnold Carter & Co agents for Dunn & Baker, Exeter (for the plaintiffs).

F A Amies Esq Barrister.


1067
[1963] 2 All ER 1068

Collins v Flynn
LANDLORD AND TENANT; Leases

QUEENS BENCH DIVISION


SIR BRETT CLOUTMAN, VC QC, OFFICIAL REFEREE
6, 7, 8, 16 MAY 1963

Landlord and Tenant Repair Covenant Construction Covenant to repair and renew Renew Repair covers
renewal of part of premises, but not an improvement to eliminate inherent defects.

The inclusion of the word renew in a lessees covenant to repair, amend, renew, the premises demised adds nothing to the
obligation that would be imposed by a covenant to repair without the word renew (see p 1074, letters b and c, post).
Dicta of Buckley LJ in Lurcott v Wakely and Wheeler ([191113] All ER Rep 41 at p 49) and of Atkin LJ in Anstruther-
Gough-Calthorpe v McOscar ([1923] All ER Rep 198 at p 204) considered.
The word repair in such a covenant by a lessee does not impose on the lessee an obligation to do work to eliminate an
inherent defect, even though it affects a subsidiary part only of the premises, where the work will amount to the provision of an
improvement (see p 1074, letters e and f, post).
Dictum of Lord Esher MR in Lister v Lane and Nesham ([1893] 1 QB 212 at p 216) and Wright v Lawson ((1903), 68 JP 34)
and Pembery v Lambdin ([1940] 2 All ER 434) applied.
Lurcott v Wakely and Wheeler ([191113] All ER Rep 41) distinguished.
The lease of a house contained covenants on the part of the lessee well and substantially to repair, amend, renew, etc, the
premises, and on the determination of the term to yield them up so well and substantially repaired, amended and renewed. Owing
to an inherent defect, namely inadequate foundations, a pier carrying one end of a girder supporting a considerable part of the
back main wall, and indirectly part of the side wall, of the house subsided and this necessitated the rebuilding of the pier and
walls with newly designed foundations.

Held The work effected an important improvement of the premises and the lessees covenants did not impose on the tenant
liability to remedy such an inherent defect, which would involve him in rendering up the premises in a different condition from
that in which they were demised.

Notes
As to a tenants obligation under repairing covenants in relation to inherent defects, see 23 Halsburys Laws (3rd Edn) 579, 580,
para 1255; and for cases on the subject, see 31 Digest (Repl) 344, 345, 47524766; 358, 359, 48864897.

Cases referred to in judgment


Anstruther-Gough-Calthorpe v McOscar [1923] All ER Rep 198, [1924] 1 KB 716, 130 LT 691, sub nom Calthorpe v McOscar,
93 LJKB 273, 31 Digest (Repl) 359, 4903.
Lister v Lane and Nesham [1893] 2 QB 212, 62 LJQB 583, 69 LT 176, 57 JP 725, 31 Digest (Repl) 358, 4893.
Lurcott v Wakely and Wheeler [191113] All ER Rep 41, [1911] 1 KB 905, 80 LJKB 713, 104 LT 290, 31 Digest (Repl) 363,
4953.
Morcum v Campbell-Johnson [1955] 3 All ER 264, [1955] 1 QB 106, 3rd Digest Supp.
Pembery v Lamdin [1940] 2 All ER 434, 31 Digest (Repl) 345, 4763.
Sotherby v Grundy [1947] 2 All ER 761, 2nd Digest Supp.
Torrens v Walker [19047] All ER Rep 800, [1906] 2 Ch 166, 75 LJCh 645, 95 LT 409, 54 WR 584, 31 Digest (Repl) 344, 4752.
Wright v Lawson (1903), 68 JP 34, 31 Digest (Repl) 360, 4906.

Preliminary Issue
In proceedings by the landlord against the tenant alleging breaches of covenants well and substantially to repair, amend, renew,
etc, the demised premises and 1068 on the determination of the term to surrender and yield them up so well and substantially
repaired, etc, there was ordered to be determined as a preliminary issue the question whether a tenant under a repairing covenant
was liable for the repair of a subsidiary part of a structure where that part had fallen into disrepair due to its own inherent defects
of construction. The issue was tried on the basis of an agreed statement of facts which, so far as material, are stated in the
judgment, post.

R E Hammerton for the landlord, the plaintiff.


Donald Keating for the tenant, the defendant.

16 May 1963. The following judgment was delivered.

SIR BRETT CLOUTMAN VC QC. In this case I am required to decide a preliminary point in a landlords claim for
dilapidations, namely, whether the landlord can recover the cost of building work which became necessary at 183, Hook Lane,
Welling, Kent, following on a serious settlement of part of the house. The settlement was caused by the inadequate foundations
and the consequent subsidence of a pier which carried one end of a girder, which in turn directly supported a considerable part of
the rear main wall of the house and indirectly part of the side wall. The initial movement of the pier was quite possibly
aggravated by war damage and by defective drains. I am relieved of the necessity of finding the essential facts, because the
parties have agreed that the point should be tried on the basis of an agreed statement of facts which incorporates a detailed letter
dated 3 October 1960, from the landlords surveyor, as well as the lease of 1 October 1937, and the plans and elevations of the
property, being respectively appendices B, C and A to the statement of facts. I read only para 6 of appendix B, the letter in which
the landlords surveyor speaks for surveyors for both parties. He says this:

Having considered all the above facts, we form the opinion that the initial movement had been caused by this pier
having inadequate foundations. The settlement of this pier caused fractures in the wall above, which in effect means that
the side wall was completely independent from the rear wall of the house. Since that initial movement there has been
subsequent movement probably caused by the thrust of the roof, the absence of any direct bond in the brickwork, further
settlement and general distortion, and all of it was quite possibly aggravated by war damage and defective drains.

Taking the rest as read, I merely pause to say that the argument on both sides has been directed entirely to questions of liability
for the consequences of an inherent defect in the premises, and that it has been agreed that I should ignore the element of war
damage entirely, and this, indeed, necessarily follows from the terms of the landlords surveyors letter, appendix B. The relevant
covenants in the lease, appendix A, para 2 and cl 3 and cl 5, are: Well, and substantially to repair, amend, renew, uphold,
support, maintain etc the premises; and at the end or sooner determination of the said term to surrender and yield up the
premises being so well and substantially repaired, amended, renewed etc.
The point was admirably argued by counsel on both sides, who were good enough to summarise their arguments as follows.
Counsel for the plaintiff landlord said that the renewal of the side and rear wall of the house were within the covenant, because,
though due to an inherent defect, (a) it was work of repair necessary to part of the premises only; and (b) there was an express
obligation to renew as well as to repair; and (c) this being repair and renewal of part of the premises only, the landlord did not, as
the result of such repair and renewal, receive back anything different in substance from what was demised. Counsel also
suggested that the damage which occurred was, or should have been, within the contemplation of the parties; but I have nothing
in the agreed statement of facts to support this.
Counsel for the defendant tenant based himself on a passage in the judgment of Lord Esher MR in Lister v Lane and
Nesham ([1893] 2 QB 212 at p 216) where he says that
1069

if a tenant takes a house which is of such a kind that by its own inherent nature it will in the course of time fall into a
particular condition, the effects of that result are not within the tenants covenant to repair.

Secondly, if it was objected that this only related to cases where the whole and not merely part of the building was affected,
counsel said that this doctrine only related to decay by age and not to inherent defect. Thirdly, if it was argued that the word
renew in the present covenant went further than the word repair in all the cases considered, counsel said, first, that as to the
construction of repairing covenants the judgments of Lord Esher MR and Kay LJ in Lister v Lane and Nesham, and those of
Bankes and Scrutton LJJ in Anstruther-Gough-Calthorpe v McOscar were to be preferred to that of Atkin LJ in the latter case:
the point being, shortly, that Bankes LJ for example, attached no importance to the particular words of the covenant ([1923] All
ER Rep at p 200; [1924] 1 KB at p 722), while Atkin LJ saw no reason for construing the words of covenants in leases dealing
with obligations to repair in any other way than one would construe any other covenant ([1923] All ER Rep at p 204; [1924] 1
KB at p 731). Counsel for the defendant tenant says that in the present covenant the insertion of the word renew adds nothing
to the more usual word repair; but if he is wrong as to this, the covenant to renew only involves the renewal of the premises as
demised and not the substitution of something different, as a pier and walls with new foundations (see per Kay LJ in Lister v
Lane and Nesham ([1893] 2 QB at p 218)).
The argument was rendered particularly interesting as each side was able to cite a passage in support of his own contentions
respectively from one or other of the two leading text-books on the law of landlord and tenant, namely, those of Woodfall and
Foa, each of which has been fairly recently edited by leading counsel very experienced in this branch of the law. So counsel for
the landlord relied on the passage in Woodfalls Law Of Landlord And Tenant (26th Edn), p 703, which I may summarise as
follows:

A covenant to repair does not involve a duty to improve the property by the introduction of something different in kind
from that which was demised If a defect in design has given rise to dry rot, the elimination of that existing rot and the
replacement of affected timber is, within the principle of renewal of subordinate parts for this is an obligation imposed by a
covenant to repair, but there is no obligation to undertake the structural alteration which is needed to prevent a recurrence
of the rot, for this is improvement and not repair It is incorrect to say, without qualification, that a repairing covenant
does not oblige the covenantor to remedy inherent defects; it is true only in the sense explained above, and the mere fact
that the renewal of some part is rendered necessary in consequence of an inherent defect does not prevent the renewal of
that part being a matter of repair.

Counsel for the tenant, on the other hand, adopted a passage from Foas Outline of the Laws of Landlord and Tenant/ (8th Edn), p
211:

Where the necessity for repair of a subordinate part (such as a wall or window) of a demised building is shown to be
due, not to age or wear, but entirely to faulty construction, it would seem that such repair is not within the tenants covenant

The learned editor goes on the cite the passage from Lord Eshers judgment in Lister v Lane and Nesham ([1893] 2 QB at p 216;
see letter A, supra), which I have already set out.
The first question is, I think, on the construction of the covenant: does the word renew in the present case add anything to
the obligation to repair? If not, this case can be considered as in line with all the cases to which I have been referred, none of
which contains the word renew, and all of which I 1070 think, contain the word repair. If, however, the word renew does
add something, then does it import a liability to provide an essential improvement, in this case new foundations, without which
the rebuilt walls and pier would be liable to early settlement? Then the second and, I think, the main question is: does the
principle of repair or renewal of subordinate parts extend in the present case to remedying inherent defects?
I will now examine the cases, starting with what seems to be the root case on the second point, namely, Lister v Lane and
Nesham, to which I have already referred. This was the case of the failure of the walls of a cottage built on a mud cill or raft.
The cottage was a small part of a much larger demise comprising the Shot Tower Wharf, the Shot Tower itself and a warehouse,
all in Lambeth. The cottage was condemned by the district surveyor and the only remedy was to pull it down and re-erect it on
seventeen foot piles. Lord Esher MR said ([1893] 2 QB at pp 216, 217), following the words which counsel for the tenant took
as the basis of his argument, and which I have quoted (See p 1070, letter A, ante) above:
However large the words of the covenant may be, a covenant to repair a house is not a covenant to give a different
thing from that which the tenant took when he entered into the covenant. He has to repair that thing which he took; he is
not obliged to make a new and different thing, and, moreover, the result of the nature and condition of the house itself, the
result of time upon that state of things, is not a breach of the covenant to repair.

Kay LJ at the end of his judgment, says ([1893] 2 QB at p 218):

He [the lessee] is not liable under his covenant for damage which accrued from such a radical defect in the original
structure.

Counsel for the landlord says that in that case the whole cottage was affected, so that the reasoning does not affect liability as to
subordinate parts as in the instant case. Further, he says that the damage in this case could and should have been within the
contemplation of the parties, and a diligent prospective lessee could have discovered the defective foundations. Counsel for the
tenant points out that the cottage was really a subordinate part of a much larger demise including the Shot Tower and wharf, and
in any case there is no logical reason for distinguishing between a partial and general failure in a case of inherent defect.
The next case was Wright v Lawson. Here a first floor bay window erected apparently on cantilever beams became
dangerous and was condemned. It could only be safely replaced by supporting it on columns from the ground, and the lessee
built a new window set back in the main wall. Kekewich J did not think that erecting a new bay window supported by columns
could be regarded as the repair of the old bay window. It would be erecting a new bay window of a totally different character,
and the lessee was not bound to do this. I observe that this is no doubt a case of inherent defect in construction and relates only to
a subordinate part of the structure. Torrens v Walker is a case of a lessors covenant to repair of a house that was over two
hundred years old. Following a dangerous structure notice, the London County Council took down the walls from the first floor
upwards, propped up the floors with timber and left the place uninhabitable. The district surveyor said that the house was worn
out and had come to the end of its time ([1906] 2 Ch at p 171; [19047] All ER Rep at p 802). Warrington J in his judgment said
([19047] All ER Rep at p 804; [1906] 2 Ch at p 174):

patching up was of no avail. It had by its own inherent nature fallen into the condition in which it was then found
to be. Then repairs were 1071 impossible, and it was necessary to rebuild the front wall and the greater part of the back
wall, to do which was not in the lessors covenant.

The lessor was not bound to give to the lessee a different thing from that which he had taken at the beginning of the tenancy.
This is a case of the whole house crumbling into decay due to old age.
The next case is Lurcott v Wakely and Wheeler on which counsel for the landlord very much relies. Here again, following a
dangerous structure notice, the lessor and the London County Council took down a front external wall in Hatton Garden and
rebuilt it. The house was very old and the condition of the wall was caused by old age, and the wall could not have been repaired
without rebuilding it. The lessees were held liable to recoup the plaintiff the cost of taking down and rebuilding the wall. In
contrast to Torrens v Walker, it was held that the wall was merely a subsidiary portion of the demised premises, and its restoration
left the rest of the building untouched. The character of the subject-matter of the demise was unchanged (see per Lord Cozens-
Hardy MR ([191113] All ER Rep at p 44; [1911] 1 KB at p 614)). This judgment is, I think, the first in which the principle of
liability for subsidiary parts worn out by age was fully expressed and made the basis of the decisions. Fletcher-Moulton LJ
pointed out ([191113] All ER Rep at p 45; [1911] 1 KB at p 915) that it was the duty of the court to give the full meaning to
each word in the covenant to keep in through repair and good condition. He said ([191113] All ER Rep at p 46; [1911] 1 KB
at p 919) that repair included the replacement of part and that ([191113] All ER Rep at p 48; [1911] 1 KB at p 922) Lister v
Lane and Nesham was different because the lessee was not obliged to give back a totally different structure, merely because the
premises were even at the time of demise in a very bad state of stability, or pay for the cost of foundations which did not exist in
the demised premises. The lord justice added ([191113] All ER Rep at p 49; [1911] 1 KB at p 923) that, if it were necessary to
treat the matter as a question of degree, he would say that this was only the repair of an element in the whole of the demised
premises. Buckley LJ decided ([191113] All ER Rep at pp 50, 51; [1911] 1 KB at pp 926, 927) the matter on the principle that it
was a question of degree and the front wall was a subsidiary part of the main structure and, therefore, a case such as Torrens v
Walker did not apply. He did, however, preface his judgment by saying ([191113] All ER Rep at p 49, [1911] 1 KB at p 923):

Repair and renew are not expressive of a clear contrast. Repair always involves renewalrenewal of a part,
renewal of a subordinate part.

He also said:

Repair is restoration by renewal or replacement of subsidiary parts of a whole. Renewal as distinguished from repair
is reconstruction of the entirety, meaning by the entirety not necessarily, but substantially, the whole subject-matter under
discussion. I agree that if repair of the whole subject-matter has become impossible, a covenant to repair does not carry an
obligation to renew or replace.

Counsel for the landlord relies on this case for both limbs of his argument: that the word renew may have a wider connotation
than the word repair; and that the twenty-four feet front wall of 64, Hatton Garden was only a subsidiary part of the demise.
The case did not, of course, deal with defective construction or inherent defect.
Next comes Anstruther-Gough-Calthorpe v McOscar, to which I have 1072 already referred. This case turns on very
different considerations from those that I have previously mentioned: namely, whether the standard of repair required could be
affected by the fact that, as it is said, the neighbourhood has gone down. I mentioned the case in connexion with the question
whether the particular form of words in repairing covenants was of importance, as to which, as I pointed out, there seems to have
been a slight difference of opinion in the court. Scrutton and Atkin LJJ both approved ([1923] All ER Rep at pp 203, 205, 206;
[1924] 1 KB at pp 729, 733, 734) the doctrine of the obligation on the lessee of the renewal of subsidiary parts, but not of the
whole, as set out by Fletcher-Moulton LJ and Buckley LJ in Lurcott v Wakely and Wheeler. Again the question of an inherent
fault in construction did not arise.
In Pembery v Lamdin, Lister v Lane and Nesham and Lurcott v Wakely and Wheeler are again reviewed, but this case is
important as turning on the effect of an inherent fault in the building rather than mere old age. Here a lessor had covenanted to
keep the external part of the demised premises in good and tenantable repair and condition. Water penetrated into the basement,
which was in no way waterproof, and, therefore, it was unusable as a cocktail bar. The tenant claimed that the landlord was liable
to waterproof the outside walls and to render the place dry and usable. Relying on Lister v Lane and Nesham, the court held that
he was under no obligation to do this. If by the inherent nature of the house it fell into a particular condition, the effects were not
in the covenant to repair. However large the words of the covenant, it was not a covenant to give back a different thing from
what was demised. I think that this case is important, because it is one of inherent vice, like Lister v Lane and Nesham and
Wright v Lawson; but, unlike Lister v Lane and Nesham ([191113] All ER Rep at p 44; [1911] 1 KB at p 914), though like
Wright v Lawson, only a subsidiary part of the structure was affected. Yet, perhaps for the very reason that it was only a case of
inherent defect, the Lurcott v Wakely and Wheeler doctrine of the subsidiary part is not so much as discussed.
The last case that is really in point is Sotherby v Grundy. This was the case of a condemned house, built in or about 1861,
the main walls having been built either without footings or on defective footings. The foundation had settled, and this could have
been avoided only by underpinning and substituting a new foundation. On the authority of Lord Eshers judgment in Lister v
Lane and Nesham, it was held that the tenant was not liable for the cost of demolition. The expenses were incurred because of
the inherent nature of the defect in the premises, and, therefore, did not come within the terms of the repairing covenant. Plainly
the doctrine of liability for the defects in a subsidiary part could have nothing to do with that case. The case, as it seems to me,
was on all fours with Lister v Lane and Nesham. Oddly enough, Lynskey J does introduce it, in what I think is an obiter passage.
He said ([1947] 2 All ER at pp 761, 762): It may be that the inherent nature of a building may result in its partial collapse. One
can visualise the floor of a building collapsing, owing to defective joists having been put in. I do not think Lister v Lane would
be applicable to such a case. In those circumstances, in my opinion, the damage would fall within the ambit of the covenant to
repair, but, as I say, it must be a question of degree in each particular case. These obiter joists, if I may so describe them, were
mentioned both in Lister v Lane ([1893] 2 QB at p 216) and in Lurcott v Wakely ([191113] All ER Rep at p 44; [1911] 1 KB at p
914). Those in Lurcott v Wakely, at all events, were an example of joists that had become rotten and could not be patched up, and
no doubt this was an excellent example of the doctrine of the subsidiary part in the old age cases, which counsel for the tenant
1073 contends has no application to a case of inherent defect dating from the construction of the premises. Lastly, counsel for the
landlord cited the case of Morcum v Campbell-Johnson. This is a Rent Acts case contrasting improvements with repairs, but I do
not think that it advances his argument in any way.
Having reviewed the cases, I will now give my opinion on the questions which I formulated earlier. First, then, in the
construction of the covenant, does the word renew add anything to the obligation to repair? While I am most anxious to give
effect to every word of the covenant, as Atkin LJ insisted in Anstruther-Gough-Calthorpe v McOscar ([1923] All ER Rep at p
204; [1924] 1 KB at p 731), yet every repair does involve a degree of renewal (except, perhaps, tightening a loose screw, as
counsel for the tenant suggested). This was the opinion of Buckley LJ in Lurcott v Wakely and Wheeler ([191113] All ER Rep
at p 49; [1911] 1 KB at p 924), and I feel that I can give a separate meaning to the word renew only by holding that it includes
rebuilding the whole property demised; and I think that if this were intended, much stronger and more specific words would have
to be used. However, this is of no importance, since I regard the word repair as apt to cover the renewal of a part of the
premises, and, therefore, so far as the words of the covenant are concerned I regard the obligation of the defendant lessee as being
similar to that in the cases cited where only the word repair is used.
I now come to the crucial point. Do the words repair and renew import a liability to rebuild with newly designed
foundations and footings the pier supporting the girder, which in turn carries a great part of the rear wall and a part of the side
wall in addition? This is manifestly a most important improvement, which, if executed by the tenant, would involve him in
rendering up the premises in different condition from that in which they were demised, and on the authority of Lord Esher MR in
Lister v Lane and Nesham, I do not think that the tenant is under any such obligation. Furthermore, although a suggestion of
liability for removal of an inherent defect in a subsidiary part seems to have been touched on in Southeby v Grundy, I do not think
that the obiter remarks of Lynskey J as to defective joists have any bearing on the present case.
If it is necessary to go beyond Lister v Lane and Nesham, then I think that the decisions in Wright v Lawson (the bay
window case) and Pembery v Lamdin (the damp basement case) are ample authority for the view that this doctrine of subsidiary
parts does not throw on the lessee an obligation to provide an improvement to eliminate an inherent defect, though affecting only
a part of the building; and I so hold. It follows that the tenant is entitled to succeed on the preliminary issue.

Order accordingly.

Solicitors: Nicholson, Graham & Jones (for the landlord, the plaintiff); Thomas Boyd Whyte (for the tenant, the defendant).

Mary Carlton Barrister.


1074
[1963] 2 All ER 1075

R v Hall
CRIMINAL; Criminal Law

COURT OF CRIMINAL APPEAL


LORD PARKER CJ, WIDGERY AND JOHN STEPHENSON JJ
26 JUNE 1963

Criminal Law Gross indecency With Sexual Offences Act, 1956(4 & 5 Eliz 2 c 69), s 13.

The word with in s 13 of the Sexual Offences Act, 1956 a (which relates to offences of gross indecency by a man with
another), means merely against or directed towards rather than with the consent of (see p 1077, letter i, post), with the
consequence that A can be convicted of committing an offence of gross indecency with B, although B was not a consenting party.
________________________________________
a Section 13 is set out at p 1076, letter d, post.

Dictum of Humphreys J in R v Pearce ([1951] 1 All ER at p 494) applied.


R v Jones ([1896] 1 QB 4) explained.
Appeal dismissed.

Notes
As to gross indecency with a male person, see 10 Halsburys Laws (3rd Edn) 671, para 1284; and for cases on the subject, see 15
Digest (Repl) 901, 902, 86918700.
For the Sexual Offences Act, 1956, s 13, see 36 Halsburys Statutes (2nd Edn) 223.

Cases referred to in judgment


R v Hornby & Peaple [1946] 2 All ER 487, 175 LT 518, 110 JP 391, 32 Cr App Rep 1, 15 Digest (Repl) 902, 8699.
R v Jones [1896] 1 QB 4, 65 LJMC 28, 73 LT 584, 60 JP 89, 15 Digest (Repl) 901, 8691.
R v Pearce [1951] 1 All ER 493, 115 JP 157, 35 Cr App Rep 17, 15 Digest (Repl) 902, 8700.
Cases also cited
ARCHBOLDS CRIMINAL PLEADING, EVIDENCE AND PRACTICE (35th Edn), paras, 2985, 2993, R v Naylor, [1961] 2 All
ER 932, [1962] 2 QB 527.

Appeal
This was an appeal by Frederick John Hall against his conviction at Brighton Borough Quarter Sessions on 18 March 1963, of
being a party to the commission by one Jones of an act of gross indecency with another man, contrary to s 13 of the Sexual
Offences Act, 1956. He was fined 25. The facts are set out in the judgment of the court.

A F B Scrivener for the appellant.


E F Monier-Williams for the Crown.

26 June 1963. The following judgments were delivered.

LORD PARKER CJ delivered the following judgment of the court: On 18 March of this year three men were brought before
Brighton Quarter Sessions, namely, Gulliford, Jones and the appellant Hall. They were charged, Gulliford with committing an
act of gross indecency with Jones, Jones with committing an act of gross indecency with Gulliford, and finally the appellant with
being a party to the commission of an act of gross indecency by Jones with Gulliford. On being put up to plead, Jones pleaded
guilty, Gulliford pleaded not guilty and the appellant pleaded not guilty. The trial, therefore, proceeded of Gulliford and the
appellant. In the result, the jury acquitted Gulliford but convicted the appellant. It is against that conviction that he now appeals
to this court. The facts of the matter need not be gone into at any length. It appears that these three men were in a public lavatory
in Brighton under observation from the police. The police emerged from their hiding place at a point of time when it is said that
Jones and Gulliford were behaving together in a thoroughly indecent manner, and what the prosecution sought to prove was that
what took place had been prompted to some 1075 extent by the appellant or at any rate that he had been within the words of s 13
of the Sexual Offences Act, 1956, a party to the commission of that offence. Gullifords case was that he had been taken by
surprise by Jones; that Jones had approached him and tried to make him behave with Jones in an indecent manner, and that he had
never consented to it at all. By their verdict, the jury must have come to the conclusion that Gullifords story might well be true;
in other words, that he was not party to any indecent approach by Jones. That, I think, is clear. The other thing that is clear is
that, quite apart from the fact that the jury may have heard Jones plead guilty, the overwhelming evidence both of the police, of
Guilliford, and, indeed, of the appellant, was that Jones had approached Gulliford in a thoroughly indecent manner. The sole
question then for the jury was whether the appellant had been a party to that.
In these circumstances, counsel for the appellant has taken a number of points, and, without doing any injustice to him, the
court thinks that it is sufficient to deal with two points. The first is whether, in the circumstances that I have related, the jury
could find the appellant guilty of the offence with which he was charged. The section, which is now s 13 of the Sexual Offences
Act, 1956, repeats s 11 of the Criminal Law Amendment Act, 1885, and says this:

It is an offence for a man to commit an act of gross indecency with another man, whether in public or private, or to be
a party to the commission by a man of an act of gross indecency with another man, or to procure the commission by a man
of an act of gross indecency with another man.

It is urged here that, before the appellant could be held guilty of being a party to the commission by a man of an act of gross
indecency with another man, it must be shown that two men had, acting in concert, been guilty of gross indecency, the one with
the other. Accordingly, it is said, since Gulliford was acquitted, it was impossible for the jury to hold the appellant guilty of being
a party to the commission of an offence. This court has come to the conclusion that the fact the Jones pleaded guilty is for this
purpose neither here nor there. A proper approach is to ask oneself what, if both Jones and Gulliford had pleaded not guilty, the
jury could have done. If the jury, coming to the conclusion that Gulliford was not a consenting party, could not have convicted
Jones, then it seems to this court that Jones plea of guilty cannot enable the jury to hold the appellant guilty.
It is curious that, since 1885, this point has not directly arisen, the point being whether the word with in the section means
with the consent of or whether with there merely means directed to, against, with in that sense. The matter was in fact
left as a question for the court in R v Pearce; but, as appears from that case, it was unnecessary directly to answer that question.
In Pearces case, the appellant had been convicted not of the substantive offence, but of attempting to commit an act of gross
indecency with another man. That other man in fact was not charged and gave evidence for the Crown; he claimed that he did
not consent. One of the questions reserved to the court was whether the word with another male person meant with the
consent of the other person involved. Despite the fact that the conviction there was of an attempt, so that the point did not really
arise, Humphreys J in giving the judgment of the court, did to some extent deal with it. He referred to R v Hornby and Peaple,
where the court had appeared to say, as indeed I think is the view taken in Archbold b, that, before the offence could be
committed, it must have been shown that the two men were acting in concert, the one consenting to the other. Humphreys J dealt
with it in this way; he said ([1951] 1 All ER at p 494):
________________________________________
b Archbolds Criminal Pleading, Evidence and Practice (35th Edn), para 2988.

Obviously the offence is one which, if two persons are to be convicted 1076 of it, must be proved to have been
committed with the consent of both of them. They must be proved to have been acting in concert together.

That seems clear. Then he goes on with these words, and, coming from Humphreys J they certainly deserve to be given very full
weight. He says ([1951] 1 All ER at p 494):

There is nothing to support the proposition that where two persons are jointly indicted for such an offence, one cannot
be convicted and the other acquitted.

Pausing there, that, as it seems to this court, can only mean that with in the section does not mean with the consent of but
must be read in the sense of towards, otherwise it seems that Humphreys Js view could not stand. It is true that that
proposition is followed by the words

On the contrary, R. v. Jones is a direct authority to the contrary.

Unfortunately, when one looks at R v Jones, it cannot be said that that is truly so. What had happened in Jones case was that A
had been charged with committing an act of gross indecency with B in one count, and in the next count B had been charged with
committing an act of gross indecency with A. The third count then alleged that A did procure the commission of an offence of
gross indecency between B and himself. In giving the judgment of the court, Lord Russell Of Killowen CJ said this ([1896] 1
QB at p 6):
Two questions are raised by this case. The first count of the indictment charges Robert Jones with having committed
an act of gross indecency with Harry Lewis Bowerbank; and it was objected that the jury, having acquitted Bowerbank,
who was charged in the second count of the same indictment with the commission of a similar act with Robert Jones, could
not afterwards convict Jones. There is, however, on the face of the indictment nothing to show that the offence, which the
first count charges Jones with having committed with Bowerbank, is the same offence which the second count charges
Bowerbank with having committed with Jones. It does not seem to me that there is anything in the [Criminal Law
Amendment] Act [1885], which requires that both offenders should be before the court before there can be a conviction for
this offence. I think that there is nothing in this objection.

It seems clear, therefore, that Jones case is authority for two things, one that A can be held guilty of committing an act of
gross indecency with B without having B before the court, and, secondly, that, on the facts of that case, the court came to the
conclusion that there was nothing to show that the incidents referred to in counts 1 and 2 were the same. Accordingly, that case is
no authority, as the court understand it, for the proposition which Humphreys J in Pearces case said ([1951] 1 All ER at p 494)
that there was no authority to support, namely, that, where two persons are jointly indicted for such an offence, one cannot be
convicted and the other acquitted. No doubt that statement of Humphreys J was derived from his very considerable experience in
criminal cases, and on what had always been the practice; witness the fact that in these cases there are always separate counts, the
first alleging that A has committed an act of gross indecency with B., and, secondly, that B has committed an act of gross
indecency with A. This court has come to the conclusion that the view taken by Humphreys J is right and should be followed,
and that the word with in s 13 of the Sexual Offences Act, 1956, does not mean with the consent of, but has the somewhat
looser meaning of merely against or directed towards. So far as the first point, therefore, is concerned, the court sees no
reason to interfere.
[His Lordship said that counsel for the appellants second point that the recorder never dealt with the burden of proof as
opposed to the degree of proof 1077 was right but that, on the evidence in the case the proviso to s 4(1) of the Criminal Appeal
Act, 1907, should be applied. He continued:] Accordingly, this appeal is dismissed.

Appeal dismissed.

Solicitors: Registrar, Court of Criminal Appeal (for the appellant); T J Blake, Brighton (for the Crown).

N P Metcalfe Esq Barrister.


[1963] 2 All ER 1078

Mills v Smith (Sinclair Third Party)


INSURANCE

QUEENS BENCH DIVISION


PAULL J
7, 8, 11, 20 FEBRUARY 1963

Insurance Liability insurance Householders comprehensive insurance Damage to premises by settlement caused by tree
roots Whether damage caused by accident.

The assured, who held a householders liability policy indemnifying him against liability for damage to property caused by
accident, was held liable for settlement damage caused to an adjacent house by the root action of a tree in the assureds garden.
On a claim by the assured for a declaration that the insurers were bound to indemnify him under the policy,

Held The insurers were liable to indemnify the assured under the policy, the damage being caused by accident, because an
unexpected event had caused the overstepping of the safety limit and led to damage; the cause of the overstepping of the safety
limit being the roots of the tree penetrating into the soil and draining away the moisture necessary to keep the movement of the
house from overstepping the safety limit (see p 1081, letter h, and p 1082, letter a, post).
Dicta of Viscount Haldane LC and of Lord Shaw Of Dunfermline in Trim Joint District School Board of Management v
Kelly ([1914] AC at pp 676, 706) applied.

Notes
As to the meaning of accident in insurance policies, see 22 Halsburys Laws (3rd Edn) 293, para 585; and for cases on the
subject, see 29 Digest (Repl) 438441, 32303251.

Cases referred to in judgment


Fenton v Thorley & Co Ltd [1903] AC 443, 72 LJKB 787, 89 LT 314, 52 WR 81, 34 Digest 266, 2264.
Fenwick v Schmalz (1868), LR 3 CP 313, 37 LJCP 78, 16 WR 481, 17 Digest (Repl) 284, 911.
Hensey v White [1900] 1 QB 481, 69 LJQB 188, 81 LT 767, 63 JP 804, 48 WR 257, 34 Digest 266, 2261.
Schloss Brothers v Stevens [1906] 2 KB 665, 75 LJKB 927, 96 LT 205, 10 Asp MLC 331, 29 Digest (Repl) 263, 1995.
Trim Joint District School Board of Management v Kelly [1914] AC 667, 83 LJPC 220, 111 LT 305, 34 Digest 270, 2300.
Warner v Couchman [1912] AC 35, 81 LJKB 45, 105 LT 676, 34 Digest 318, 2604.

Action
In this action the plaintiff obtained judgment against the defendant in respect of damage to his house by settlement caused by the
root action of a tree growing in the defendants garden. The defendant claimed an indemnity against the third party, one of the
insurers of the defendant under a householders liability policy. The case is reported only with regard to the third party
proceedings. The facts are set out in the judgment.

J W Wellwood for the plaintiff.


Quentin T Edwards for the defendant.
S H Noakes for the third party.

Cur adv vult


1078

20 February 1963. The following judgment was delivered.

PAULL J. In the third party proceedings in this action, the defendant claims from the third party as one of the insurers of the
defendant as occupier of 315, Mutton Lane, a declaration that the insurers are bound to pay to the defendant all sums which he
may have to pay to the plaintiff as a result of my judgment in the action. The policy is in the usual form of a Lloyds
householders policy, and, as regards the scope of the indemnity given, the alleged applicable words are:

Section 2 (e). Liability to public. All sums for which the assured as occupier may be held legally liable in respect of
claims made by any person for damage to property caused by accident. The liability shall not exceed 25,000 for any one
accident or series of accidents.

It should be noted that, after the words by accident, are inserted the words or by defective sanitary arrangements in or about
the said premises.
The insurers take a very proper attitude towards this claim. They do not desire to rely on any facts that may be peculiar to
this action. For example, they do not desire to take any point as to the knowledge of the defendant as to the likelihood of
damage; they did not seek to press the defendant as to why he desired to cut down the tree in 1948. Further, the insurers do not
suggest that the words by accident are limited by the words following them. They only seek to use those following words as
tending to throw light on the proper interpretation of the words by accident, using them as showing that the cover is very
limited outside what ordinary people would call accidents. The only point that the insurers take, and it is a formidable one, is
that damage brought about by the natural growth of the tree and its natural desire for water cannot be brought within the words
caused by accident. Clearly, most people, if asked the question Was the growth of the tree an accident?, or, Was the
extraction of the water by the tree an accident?, would answer No. On the other hand, if, owing to a weight placed on a table
in a garden one of the legs of the table dropped into the ground which happened to crumble at that spot, and the leg broke, the
same people would say that the dropping of the leg into the ground was an accident, whatever was the cause of the ground
crumbling. Counsel for the third party says that the taking of the water is the cause and that the breaking of the foundations is the
damage. Counsel for the defendant says that the dropping of the foundations into the ground is the accident, and that the
fractures caused by the dropping or, indeed, the cost of mending the result of the dropping, is the damage under the policy.
Curiously enough, although these words have been a standard form of words in a householders policy of insurance for a long
time, there is no authority deciding which is the correct way of looking at the problem, and the insurers are anxious, and properly
anxious, for a decision.
The word accident is difficult to define, indeed, one judge very many years ago said the word was underfinable. The
dictionary definition given in Murrays Oxford Dictionary is (and I leave out immaterial meanings) Anything that happens
without foresight or expectation, or is an unusual effect of a known cause. The application of that definition depends almost
entirely on the point of view from which the particular matter is approached. If, quite unexpectedly, someone coming up to me,
hits me in the face and gives me a black eye, the event, so far as I am concerned, is quite unexpected, yet I would not say that I
had got the black eye by an accident; I would say that I got it by an unprovoked assault. Nor would it be an accident so far as the
attacker is concerned. Yet, under the Workmens Compensation Acts, an injury by an unprovoked attack has been decided to be
an injury by accident for the purpose of those Acts. Before dealing with this aspect of the case, I would say that I have found the
insurance cases quoted to me not very helpful, in that the policies in question were not policies against damage caused by
accident. Where, for instance, insurers have sought to be relieved of liability under all risks policies on the ground that the loss
was through natural deterioration, although that deterioration was brought about by 1079 decay owing to natural causes such as
storms, I think that quite different considerations apply, particularly where the ejusdem generis rule has been applied (see, for
example, Fenwick v Schmalz and Schloss Brothers v Stevens. Again, the word from any accident which may happen. As has
been pointed out in the House of Lords, by accident has a different meaning from by an accident. Counsel for the third
partys proposition that, where the damage is caused in the ordinary course of nature, either to human beings or to property, there
is no accident, is, in my judgment, too wide when the words are caused by accident. It would exclude damage through
exertion, such as rupture or haemorrhage, both of which have been held to be injuries by accident under the Workmans
Compensation Acts.
In my judgment, the key to the problem is in the fact that the insurers have deliberately chosen to use in s 2 (e) the same
words as have been used in the Workmens Compensation Acts. It is to be noted that, in s 3 of the policy, where it is dealing with
injuries to the occupiers servants, the same words are used, and there can be no doubt but that the insurers have in that section
quite deliberately brought in the same concept as has been applied under the Workmens Compensation Acts. I do not think that
it is tenable to say that, where they use the same words in s 2 (e), some other concept can be applied. One section deals with
damage to humans and one with damage to property, but the concept of the type of event giving rise to the claim must be the
same in each case. If a servant while on a journey catches a disease, that is not by accident under the policy, any more than it
would be under the Workmens Compensation Acts if he was in his masters factory. If the brickwork of a house, so to speak,
catches a disease from a treeif, for instance, fungus spreads from the roots to the bricks causing a gradual eating away of the
bricksI should say that that was not damage caused by an accident under s 2 (e). On the other hand, a rupture caused by
overstrain, or a nervous shock received from some cause, is injury by accident under the Workmens Compensation Acts. Does it
then follow that an unexpected fracture caused by the ground crumbling is damage by accident, even though the crumbling is
brought about by a combination of a hot summer and a thirsty tree? I look to see how the courts have looked on the concept
invoked by the words injury by accident, under the Workmens Compensation Acts. The first case in which these words
received attention from the House of Lords is that of Fenton v Thorley & Co Ltd. That was a case where a rupture had been
brought about simply by the man overstraining himself while turning a wheel at work. Lord Macnaghten says ([1903] AC at p
446):

the learned judges of the Court of Appeal held in Hensey v. White, as they have held here, that there was no
accident, because (to quote the leading judgment) there was an entire lack of the fortuitous element. What the man was
doing, it was said, he was doing deliberately, and in the ordinary course of his work, and that which happened was in no
sense a fortuitous event.

Having considered this matter at some length, Lord Macnaghten says ([1903] AC at p 448):

I come, therefore, to the conclusion that the expression accident is used in the popular and ordinary sense of the word
as denoting an unlooked for mishap or an untoward event which is not expected or designed.

Lord Lindley says ([1903] AC at p 453):

Speaking generally, but with reference to legal liabilities, an accident means any unintended and unexpected
occurrence which produces hurt or loss.
1080

In Warner v Couchman, the House of Lords upheld a decision of a county court judge who had held that frostbite was not
injury by accident arising out of and in the course of the mans employment. The whole argument, however, in that case really
turned on whether it arose out of his employment, since it was caused by weather which was common to all persons in that area.
That it was an injury by accident seems to have been assumed and, indeed, was admitted in argument. It is in this case that Earl
Loreburn LC says ([1912] AC at p 38):

I will only say this further: To be perfectly strict and accurate, it is somewhat lax to speak of this statute [the
Workmans Compensation Act, 1906, s. 1(1)] as though it referred to an accident. I am perfectly conscious that I myself, as
well as others, have fallen into that lapsus linguae; but at times it may be apt to confuse ones idea of what is enacted in this
particular Act of Parliament. The Act of Parliament does not speak of an accidentit speaks of injury by accident arising
out of and in the course of the employment.

The case which has always been looked on as being the really decisive case as to the interpretation of the words injury by
accident is Trim Joint District School Board of Management v Kelly. This case was twice argued before the House of Lords, and
the final result was that four law lords were of one opinion and three of the other, showing, I venture to say, how difficult it is to
get clear the concept of injury or damage by accident. It was a case of a deliberately planned, cold-blooded assault by a number
of boys on a schoolmaster, because earlier he had stopped them playing hockey and had caught one of them stealing. The
question was whether Kellys consequent death was caused by injury by accident. By four to three, the law lords held that it was.
Viscount Haldane LC after pointing out that the meaning of the word accident may vary according as the context varies, says
([1914] AC at p 676): If, so far as the workman is concerned, unexpected misfortune happens and injury is caused he is to be
indemnified. Substitute property for workman, damage for injury and insured for he, and the phrase would run:

If, so far as the property is concerned, unexpected misfortune happens and damage is caused the insured is to be
indemnified.

Earl Loreburn, after pointing out the difficulties of the word accident, goes on to say ([1914] AC at p 681): When people use
this word they are usually thinking of some definite event which is unexpected, but it is not so always Lord Shaw Of
Dunfermline says ([1914] AC at p 706):

It has been pointed out more than once that even the term accident does not in any view stand by itself. The
expression is injury by accident, and, as LORD MACNAGHTEN has explained, it is accidental injury that is being dealt
with, and not accident per se.

In this case, of course, one would substitute accidental damage for accidental injury.
Summing-up the effect of these cases and trying to apply them to damage by accident instead of to injury by accident, I
would ask and seek to answer two questions. One: has there been at any moment of time (or at particular moments of time)
some unexpected event (or events) which has (or have) led to damage? My answer to that question is, Yes. It is true that
foundations settle, that is, tend to drop a little at one or more points, and that that settling may be gradual. In this case, however,
there has come a point of time when the movement has overstepped the safety limit, if I may use that expression, and a crack in
the concrete or in the brickwork at or near the south-east corner of the house has started. This may have happened several times,
that is, there may have been more than one crack. In humans, the overstepping of movement may cause a 1081 fracture, a
rupture, or a haemorrhage. In a building, it may cause a fracture of the concrete or of the brickwork. There is no accident until
the overstepping takes place. The second question is: what was the cause of the overstepping of the safety limit? My answer is
that the cause is the nuisance of the roots of the tree penetrating into the plaintiffs soil and draining away the moisture necessary
to keep the movement of the house from overstepping that safety limit. In my judgment, these answers result in judgment in
favour of the defendant for a declaration that the insurers are bound to indemnify him against any sums which he may have to
pay to the plaintiff by reason of my judgment in the action. I should like to add that I am greatly indebted to counsel on each side
for their arguments.

Judgment for the defendant.

Solicitors: Claude Barker & Partners (for the plaintiff); Coode, Kingdon, Cotton & Ward agents for Derrick Bridges & Co
Barnet (for the defendant); Chamberlain & Co (for the third party).

Mary Colton Barrister.


[1963] 2 All ER 1082

Re Adoption Application No 41/61 (No 2)


FAMILY; Children

CHANCERY DIVISION
WILBERFORCE J
15, 16, 22 MAY 1963

Adoption Custody Illegitimate child Conflicting applications Application to High Court for adoption order Putative
fathers application to High Court for custody Concurrent hearings Welfare of infant the guiding consideration on both
applications Legitimacy Act, 1959(7 & 8 Eliz 2 c 73), s 3 Adoption Act, 1958(7 & 8 Eliz 2 c 5), s 7(1)(b).

Adoption Practice Guardian ad litem of illegitimate infant whose adoption is sought Duty to inform court if putative father
intimates to guardian a wish to be heard on question whether adoption order should be made Whether any duty on guardian to
seek out putative father Adoption (High Court) Rules, 1959 (SI 1959 No 479), r 16 (f), Sch 2, para 9.

An illegitimate child was born on 16 September 1961. On 12 October 1961, the putative father applied to a magistrates court for
custody of the child, and on or about 27 October 1961, the child was sent to adopters. On 17 November 1961, the mother signed
her consent to the adoption. On 27 November 1961, the adopters applied to the High Court for an adoption order. On 29 May
1962, the preliminary stages of the adoption proceedings were heard and stood over until the putative father had had an
opportunity to issue a summons for custody in the High Court which he did on 8 June 1962, the application being made under the
Guardianship of Infants Acts by virtue of s 3a of the Legitimacy Act, 1959.
________________________________________
a The Legitimacy Act, 1959, s 3(1), provides: Subject to the provisions of this section, the following enactments relating to the custody of
infants, that is to say(a) s 5 of the Guardianship of Infants Act, 1886 (which enables the court to make, on the application of the mother of
an infant, orders regarding the custody of the infant and the right of access thereto of either parent); and (b) s 16 of the Administration of
Justice Act, 1928 (which enables the court to make orders under the said s 5 on the application of the father of an infant), shall apply in
relation to an infant who is illegitimate as they apply in relation to an infant who is legitimate, and references in those enactments, and in
any other enactment so far as it relates to proceedings under the said s 5, to the father or mother or parent of an infant shall be construed
accordingly.

Held Although under the Adoption Act, 1958, the consent of the putative father was not required for the making of an adoption
order, yet both under that Act (s 7(1)(b)) and under the Guardianship of Infants Acts (under which the putative father was
applying for custody) the welfare of the infant was the guiding consideration; in the present case the proposals of the mother were
more for the welfare of the child than the putative fathers proposals and an adoption order would be made.
1082
Per Curiam: (i) under para 9 b of Sch 2 to the Adoption (High Court) Rules, 1959, the guardian ad litem of an illegitimate
child in respect of whom adoption proceedings are pending is not under obligation to seek out the putative father, but if it comes
to the guardians notice that the putative father wishes to be heard by the court on the question whether the adoption order should
be made, the guardian should inform the court so that the court may consider whether notice of the hearing should be served
under the Adoption (High Court) Rules, 1959, r 16c (see p 1088, letter d, post).
________________________________________
b Paragraph 9 of Sch 2 to the Adoption (High Court) Rules, 1959, provides: Where the infant is illegitmate but no one is liable as the
putative father to contribute to the maintenance of the infant by virtue of any order or agreement, the guardian ad litem shall inform the
court if he learns of any person, claiming to be the father, who wishes to be heard by the court on the question whether an adoption order
should be made.
c Adoption (High Court) Rules, 1959, r 16: On the application being adjourned to the judge, the guardian ad litem shall serve a notice in
Form 8 on the following persons, that is to say:(f) any other person or body who in the opinion of the judge ought to be served with
notice of the hearing of the application.

(ii) in directing that the adoption proceedings in the present case should be stood over until the putative father had issued a
summons for custody the court was merely recognising that he had already taken this step in a magistrates court, but ordinarily
there would be no reason why a putative father should not put his case in adoption proceedings under the rules (see p 1088, letter
f, post).

Notes
As to rights of a father to custody of his child, see 21 Halsburys Laws (3rd Edn) 191, 192, para 425; and for cases on the subject,
see 3 Digest (Repl) 432435, 268291.
As to the procedure leading to an adoption order, see 21 Halsburys Laws (3rd Edn) 234, para 507; and as to arrangements
for adoption, see ibid, 223, para 490; and for cases on the subject, see 28 Digest (Repl) 632638, 13251348.
For the Guardianship of Infants Acts, 1886 and 1925, see 12 Halsburys Statutes (2nd Edn) 942, 945.
For the Adoption Act, 1958, s 4, s 7, s 57, see 38 Halsburys Statutes (2nd Edn) 542, 546, 578.
For the Legitimacy Act, 1959, s 3, see 39 Halsburys Statutes (2nd Edn) 33.
For the Adoption (High Court) Rules, 1959, r 16, see 11 Halsburys Statutory Instruments (1st Re-issue) 186.

Cases referred to in judgment


A (an infant), Re, [1955] 2 All ER 202, [1955] 1 WLR 465, 3 Digest (Repl) 435, 291.
Adoption Application 41/61, Re [1962] 3 All ER 553, [1963] Ch 315.
M (an infant), Re, [1955] 2 All ER 911, [1955] 2 QB 479, 119 JP 535, [1955] 3 WLR 320, 28 Digest (Repl) 636, 1342.
R v Nash, Re Carey (1883), 10 QBD 454, 52 LJQB 442, 48 LT 447, 3 Digest (Repl) 433, 276.

Adjourned Summonses
These were two applications by originating summons: the first, dated 8 June 1962, being under the Guardianship of Infants Acts,
1886 and 1925, in which the applicant was the putative father of an infant and the respondents were the mother, the National
Childrens Adoption Association and the infant. The second dated 27 November 1961, was by a husband and wife for an order
that they might be authorised to adopt the infant. The infant whose guardian ad litem was the Official Solicitor, was the
respondent to that summons. The facts appear in the judgment.

Nigel Warren QC and William Denny for the putative father.


Peter Foster QC and E W H Christie for the mother.
1083
J G Strangman QC and Charles Sparrow for the adopters.
D A Thomas for the Official Solicitor.

Cur adv vult

22 May 1963. The following judgment was delivered.

WILBERFORCE J. Following the procedural decision of the Court of Appeal in July, 1962, and after most regrettable delay,
there now come before me on the substance two summonses. The first is an application under the Guardianship of Infants Acts,
1886 and 1925, and the Legitimacy Act, 1959, dated 8 June 1962. The applicant in that summons is the putative father, and the
respondents are the mother, the National Childrens Adoption Association, and the infant. The second proceeding is an
application to approve an adoption which was commenced on 27 November 1961. These two proceedings have been heard by
me concurrently and, as Danckwerts LJ said ([1962] 3 All ER at p 561; [1963] Ch at p 330), though there have been procedural
difficulties, these have turned out to be not insuperable. Counsel have appeared for the adopters without disclosing the latters
identity, and a report from the Official Solicitor as to the present circumstances of the adoption, but without identifying the
adopters, has been made available to the father. Both father and mother have been cross-examined on their statements, as have
their respective parents, that evidence being treated as available on both applications.
The basic dates are as follows. The mother and the father met in 1960 when the father was aged twenty-two and the mother
twenty-four. The mother became pregnant in December, 1960, and the infant was born on 16 September 1961. On 12 October
1961, the father made an application for custody in a magistrates court and on or about 27 October 1961, the child was sent to
adopters. On 17 November 1961, the mother signed her consent to the adoption and, as I have said, on 27 November the formal
application for adoption was made. On 8 June 1962, the father took out the present custody summons in the High Court, and on
29 May 1962, I heard a preliminary stage of these proceedings on which I gave certain procedural directions. On 30 July 1962,
the appeal was heard by the Court of Appeal.
I have before me now three propositions. The first proposition is that the court should sanction the adoption. The second is
that if it does not do that, it should give the custody of the infant to the mother; and the third proposition is that it should give the
custody to the father.
Before I consider the merits of these three proposals it is necessary to place them in the appropriate legal framework in order
to see what weight ought to be given to the various considerations urged. I should make it clear at the outset that I am not here
concerned with the rights of a legal parent to obtain custody of his child, and I therefore forbear from referring to the various
authorities on that subject. I am concerned only with the position of a putative father. Under the Adoption Act, 1958, the court
has first to see whether the consent of any person whose consent is required to the suggested adoption is lacking: the persons in
question being identified by s 4(1)(a) as every person who is a parent or guardian of the infant. It is clear that the natural father
is not a parent (see Re M (an infant)) and nothing in the Legitimacy Act, 1959, alters this. It has been argued (a new point since
the case went to the Court of Appeal) that the natural father might be a guardian and that by virtue of s 57 of the Adoption Act,
1958, which provides a definition of guardian, he would be a guardian if an order giving him the custody were made under the
Legitimacy Act, 1959, s 3. I am satisfied that this is not so, since the Guardianship of Infants Acts throughout make a clear
distinction between persons appointed guardian and persons given custody. I am therefore of the view that the position
established by Re M (an infant) stands, namely, that the consent of the natural father is not 1084 required; he is merely a person
who may be heard by the court before the decision to approve the adoption is made or refused.
That leaves me with the second and only other relevant requirement that the order if made will be for the welfare of the
infant (the third requirement stated in s 7(1)(c) of the Adoption Act, 1958, is not relevant). The section, apart from a particular
direction given in sub-s (2), does not prescribe what matters have to be considered in this connexion, so that it would seem to me
that the court must take into account all the merits and demerits of the alternative proposals as they seem likely to bear on the
childs welfare: not limiting itself to purely material factors, but considering, as they may bear on the welfare of the infant, such
matters as the natural ties of blood and family relationship. The tie (if such is shown to exist) between the child and his natural
father (or any other relative) may properly be regarded in this connexion, not on the basis that the person concerned has a claim
which he has a right to have satisfied, but, only if, and to the extent that, the conclusion can be drawn that the child will benefit
from the recognition of this tie.
Turning now to the position under the Guardianship of Infants Acts, 1886 and 1925, and the Legitimacy Act, 1959, the right
which is given to the natural father by s 3 of the latter Act is to apply for the custody of the child under the Guardianship of
Infants Acts (including the Administration of Justice Act, 1928). The guiding principle laid down by those Acts is again that of
the welfare of the infant, and it seems to me clear that any proposals of the natural father must be considered to the extent that
they contribute to it. It has long been recognised that a natural father has the right to come to a court of equity and to ask that
court to exercise its jurisdiction over infants in such a manner as to give effect to the natural fathers wishes, and that the court
will, in a proper case, do so. This has been laid down by Sir George Jessel MR in R v Nash, Re Carey, and more recently in Re A
(an infant). Sir R Evershed MR there said ([1955] 2 All ER at p 205) that the judge had acted quite correctly in the exercise of
his discretion by considering the alternatives presented to him on the one side and the other (that was, by the natural father and
the mother respectively) and so coming to a conclusion that in the interests of the child the better plan was that to which he gave
effect in the order made by him. In my judgment this has not been altered by s 3 of the Legitimacy Act, 1959. That Act gives the
putative father a new statutory right to apply to the courts having jurisdiction in guardianship matters for custody and enables
him, if his case is made out, to obtain an order against third parties, but it does not alter the principles on which the court deals
with the substance of his application. It remains, in my judgment, the case that, whether in equity or in statutory proceedings, the
putative father has access to the courts and the right to put forward his plans on their objective merits: his position as putative
father may enable him to urge recognition, in the childs interests, of the ties of blood and of natural affection, but it does no more
for him than that. So strong, but no stronger, was his position in equity before 1959, and I am unable to see that the Legitimacy
Act, 1959, s 3, gave him further rights, whether against the mother or against third persons. So that, whether the court is faced
with adoption proceedings, the natural father being present and giving his views, or with custody proceedings under the
Guardianship of Infants Acts, or with the two concurrently, the court should act in the same way and on the same considerations:
it should take the natural fathers proposals into consideration to the extent that I have endeavoured to state. Of course, the
proposals of the mother must be considered in an analogous way.
Now what are the relevant circumstances here? First, I think that I must form some view as to the manner in which the
mother and father respectively came to their decisions in October, 1961, the one to place the child for adoption, the other 1085 to
seek custody for himself. I hope that I can do so without wounding the feelings of either of these young people, who have faced
a difficult experience. Theirs was a rather short-lived relationship based mainly on physical attraction. As so often happens, the
woman was more emotionally involved than the man. She was in love with him and hoped for marriage. He, and he was quite
honest about this, could not find himself to be in love with her. She became pregnant in December, 1960, and, though physical
relations continued for some months, in April, 1961, he finally convinced her that he was not going to marry her and that the
problem of her and her child was for her to solve. [His Lordship then referred to the fathers statement.] There is much in the
correspondence between them to the effect that he wanted to accept his responsibilities, but I think that she was justified in
believing that, though these phrases might be genuine manifestations of a personal concern, they were not likely to be translated
into future action. From April to September he did not go to see her once. He wrote to her and he sent her money. She was
expressing her loneliness and her anxieties as to the childs future, thinking and telling him of her idea that the child might be
given to adopters. As she puts it the possibility that he would consider keeping the child himself simply did not exist as a
serious proposal until after the child was born. He wanted her to keep it and for him in some undefined way to have a say in its
future. All this continued until on 12 October 1961, without warning, he launched the custody proceedings. I think that there
may well be grounds for believing, as she believed, that a main motive for this step was to bring pressure to bear on her to keep
the child. There may also be substance in the suggestion that it was by then his parents who wanted him to take the child, so that
they would be assured of a grandchild. But even if I discount all this and accept that he has now persuaded himself that he
sincerely wants the child, it is inevitable that the value of his proposal, the strength of its roots in any real attachment to the child,
must be seriously diminished when one views it against the background of his detached, I use no harder word, attitude during her
pregnancy and his view of the child as the product not even of a love affair but of a lesser relationship; and he has only seen the
child three times. I compare this with the process by which the mother reached her decision to place the child for adoption. This
was one which she reached after long and painful internal debates both before and after the birth coupled with the best advice she
could get from relations, friends and experts. I accept entirely the sincerity of the decision. I reject the suggestion which was
made in the course of her cross-examination that, being unable to make up her mind on the merits, she was triggered into it by his
application for custody. I refuse to place any less value on the decision because she changed her mind many times, even up to the
last moment: what mother would not, when faced with the most painful of all choices? Although I am not bound by her views, I
am impressed by the reasons she now gives for continuing to think that, marriage being unattainable, adoption is best for the
child. So that it seems to me, estimating as best I can the quality of the rival decisions, that the mothers was far more closely
related to a genuine and understanding choice of what was best for the child.
I would only add two things at this point. First I accept almost entirely the mothers account of the pre-natal period and of
what took place between her and the father. I think that she understood him and that in her judgment she was tolerant as well as
perceptive. The correspondence indeed speaks for itself. Secondly, having seen and heard both in the box, I am certain that the
cold print of the transcript of evidence, if it ever has to be read, would give a misleading impression of these witnesses. This is
certainly true of one extract of the fathers evidence which has been shown to me and relied on against him, which to my mind
gives a totally false impression.
So much for introductory matters. Now I come to the heart of the question. The mothers case is this, and she is supported
in it by the Official Solicitor as guardian ad litem of the infant: the best thing for the child would have been to be brought up by
its united parents. This has not been possible. The next best 1086 thing, in her view, is for it to be brought up by suitable people
who can take the place of its two parents. I cannot go into the details of the proposed adoption, but this much can be said, that the
child has been taken into a suitable home, with adopters who have everything in their favour: the right age, position in life,
material prospects, and suitability as parents. These facts do not rest on conjecture but on the evidence of eighteen months actual
care. The adopters intention to adopt also a girl is a further point in favour of the adoption. There are risks in everything but this
seems as hopeful an adoption as one could wish to find. The fathers proposition is that he should have custody. At the present
time and for about a year he will be a medical student, supported by a grant and an allowance. After that he expects to get a
house appointment which would involve living in. Over this period of about two years the proposal is that the child should live
with his parents. They have a house of their own, a floor of which would be at his disposal. They are comfortably off and would
be able to employ help. They are willing to settle 2,000 on the child. After, say, two years, the father should be making a good
income and capable of setting up his own establishment.
The mother makes various criticisms of these proposals. It is said that the grandparents are too old to take on the onerous
task of looking after a young child; that the grandmothers health is not good; that the house is not well maintained. I think that
there is some substance in these objections, though I am sure that both are sincere in their present intentions to do the best and in
their belief that they can. But these are minor points compared to the basic objections. What would the position of the child be in
this household, an illegitimate child of their son, bearing another name? What would be the position of the mother? Could she
be denied access to her child? If access were granted, one could foresee great difficulties. The grandparents take an
unfavourable view of her. They consider that she was not a fit person for their son to marry. There would certainly be endless
disputes. Even if the mother could be shut out for a time (and that would be a great hardship to her), what of the future? One
must look forward to the time when there would be other grandchildren which will perhaps lessen the interests of the
grandparents in this child. One must look forward to the possible marriage of the son and other children being born to him. The
whole position in either of these events would radically change and that would open up the prospect of wardship proceedings.
These are not worked-up fears: they are real risks, and they can be summed up by saying that there would be a change from a life
of security free from the stigma of illegitimacy back to that of an illegitimate child who, even if things go well, would have
successive influences in his lifehis grandmother, a nurse, possibly a stepmother and possibly substantial disputes. I give the
father fullest credit for every wish to surmount these difficulties and I have no doubt that he hopes that he will be able to do so,
but they do seem to me to be overwhelming.
There is one other point. The fathers proposals involve a change of regime for the child at this stage in his life and as to
that I have medical evidence from a well-known practitioner in psychological medicine. Having examined the child in question
and expressed his satisfaction with his present condition, he says that There is virtually a consensus in professional circles that a
change of maternal care has potentially damaging results on any child between the ages of about six months and two and a half
years and I would consider that this little boy is currently at about the peak level of risk through any such move. From my point
of view I cannot too strongly condemn this proposal. That report was made on 22 January 1963, and the point seems to me to be
a serious one, but I am reluctant to base my decision on it on account of the delay which has elapsed in the proceedings, though I
might point out that the same point would have been good in January, 1963, when the report was made. I can, I think, leave it out
of account because there is so much else in the case which is decisive to my mind.
I reach the conclusion on the balance of considerations which I have stated that the fathers proposal is not in the interests of
the child; that the adoption is 1087 in the interests of the child, not merely because it is the mothers proposal, but per se. The
third alternative, namely, custody to the mother, does not in that case arise. If it did, and the matter rested solely between the
mother and the father, I would without hesitation come to the conclusion that with a child of this age the mothers case was
considerably the stronger. I do not accept for a moment the argument of the father that, by offering the child for adoption, she has
forfeited her own claims. The decision reached was, it seems to me, wholly to her credit. Indeed, the fact that such an argument
is put forward in this case serves to underline the basic failure of the father to understand the realities of the situation.
There are two other points of general application on which I would like to say a few words. First, apart from the custody
proceedings the putative father is here participating in the adoption proceedings by virtue of r 16 (f) of the Adoption (High Court)
Rules, 1959 (SI 1959 No 479)d. In a case, such as this, where the putative father is not liable to contribute to maintenance under
an order or agreement, his existence is brought to the notice of the court by the guardian ad litem under para 9 of Sch 2 e which
applies when he, the guardian ad litem, learns of any person claiming to be the father who wishes to be heard by the court on the
question whether an adoption order should be made. I would like to make it clear that this provision does not place on the
guardian ad litem the duty to seek out the putative father of any illegitimate child in respect of whom adoption proceedings are
pending, or indeed (in the absence of special circumstances) to make any inquiries as to the existence, whereabouts or attitude of
a putative father. It merely provides that if it comes to his notice that a putative father wishes to be heard by the court on the
question whether an adoption order should be made, he is to inform the court so that the court may consider whether notice of the
hearing should be served under r 16. This is in fact, I understand, the High Court practice, and it is desirable that it should be
generally followed.
________________________________________
d See p 1083, footnote *, ante.
e See p 1083, footnote +, ante.

Secondly, I do not wish it to be thought that this case sets a pattern to be followed in every case where a putative father seeks
to put forward his case against a proposed adoption. In directing, as I did, that the adoption proceedings should stand over until
the father had an opportunity to issue a summons for custody, I was merely intending to give recognition to the fact that he had
already taken this step in another court before the application for adoption was made and to enable him to assert his claim before
it was too late. There is no reason in the ordinary case why a putative father should not put his case in the adoption proceedings
under the rules, whether or not he accompanies his intervention with a formal application for custody if the adoption is refused.
If this is done, there would be no need for the adoption proceedings to be held up, a consequence which, as I would respectfully
agree with Diplock LJ ([1962] 3 All ER at p 564; [1963] Ch at p 335), the courts should do all in their power to prevent.

Order accordingly.

Solicitors: W E Wise & Son (for the putative father); Radcliffes & Co (for the mother and for the National Childrens Adoption
Association); Milles, Day & Co (for the adopters).

Jenifer Sandall Barrister.


_WOW_WOW_WOW_WOW_WOW_WOW_WOW_WOW_WOW_WOW_WOW_WOW_WOW_WOW_WOW_WOW_WO
W_WOW_WOW_WOW_WOW_WOW_WOW_WOW_WOW_WOW_WOW_WOW_WOW_WOW_WOW_WOW_WOW_
WOW_WOW_WOW_WOW_WOW_WOW_WOW_WOW_WOW_WOW_WOW_WOW_WOW_WOW_WOW_WOW_WO
W_WOW_WOW_WOW_WOW_WOW_WOW_WOW_WOW_WOW_WOW_WOW_WOW_WOW_WOW_WOW_WOW_
WOW_WOW_WOW_WOW_WOW_WOW_WOW_WOW_WOW_WOW_WOW_WOW_WOW_WOW_WOW_WOW_WO
W_WOW_WOW_WOW_WOW_WOW_WOW_WOW_WOW_WOW_WOW_WOW_WOW_WOW_WOW_WOW_WOW_
WOW_WOW_WOW_WOW_WOW_WOW_WOW_WOW_WOW_WOW_WOW_WOW_WOW_WOW_WOW_WOW_WO
W_WOW_WOW_WOW_WOW_WOW_WOW_WOW_WOW_WOW_WOW_WOW_WOW_WOW_WOW_WOW_WOW_
WOW_WOW_WOW_WOW_WOW_WOW_WOW_WOW_WOW_WOW_WOW_WOW_WOW_WOW_WOW_WOW_WO
W_WOW_WOW_WOW_WOW_WOW_WOW_WOW_WOW_WOW_WOW_WOW_WOW_WOW_WOW_WOW_WOW_
WOW_WOW_WOW_WOW_WOW_WOW_WOW_WOW_WOW_WOW_WOW_WOW_WOW_WOW_WOW_WOW_WO
W_WOW_WOW_WOW_WOW_WOW_WOW_WOW_WOW_WOW_WOW_WOW_WOW_WOW_WOW_WOW_WOW_
WOW_WOW_WOW_WOW_WOW_WOW_WOW_WOW_WOW_WOW_WOW_WOW_WOW_WOW_WOW_WOW_WO
W_WOW_WOW_WOW_WOW_WOW_WOW_WOW_WOW_WOW_WOW_WOW_WOW_WOW_WOW_WOW_WOW_
WOW_WOW_WOW_WOW_WOW_WOW_WOW_WOW_WOW_WOW_WOW_WOW_WOW_WOW_WOW_WOW_WO
W_WOW_WOW_WOW_WOW_WOW_WOW_WOW_WOW_WOW_WOW_WOW_WOW_WOW_WOW_WOW_WOW_
WOW_WOW_WOW_WOW_WOW_WOW_WOW_WOW_WOW_WOW_WOW_WOW_WOW_WOW_WOW_WOW_WO
W_WOW_WOW_WOW_WOW_WOW_WOW_WOW_WOW_WOW_WOW_WOW_WOW_WOW_WOW_WOW_WOW_
WOW_WOW_WOW_WOW_WOW_WOW_WOW_WOW_WOW_WOW_WOW_WOW_WOW_WOW_WOW_WOW_WO
W_WOW_WOW_WOW_WOW_WOW_WOW_WOW_WOW_WOW_WOW_WOW_ All ER 1963 Volume 2
[1963] 2 All ER 1

Government of Ceylon v Chandris


ADMINISTRATION OF JUSTICE; Arbitration

QUEENS BENCH DIVISION


MEGAW J
14, 15, 18, 25 FEBRUARY 1963

Arbitration Costs Fees of umpire and arbitrators Duty of umpire to tax or settle fees Duty of umpire to consider whether
fees fair and reasonable Arbitration Act, 1950 (14 Geo 6 c 27), s 18(1).

The High Court will not normally intervene in a motion to set aside or remit an award of an umpire on the ground that the umpire
misconducted himself by settling his own and the arbitrators remuneration at a figure which was wholly excessive unless (i) it is
satisfied that the fees can properly be described as extravagant, or (ii) it is apparent that the umpire has seriously misunderstood
his duty as regards the assessment of fees which he demands by his award, whether for himself or, through himself, for the
arbitrators, or for both. Absence of proper assistance from the umpire, in the way of adequate information to the court when the
fees prima facie appear to be out of relation to the work involved and are challenged as being grossly excessive, may be treated as
leading to the conclusion that the umpire has misunderstood his duty (see p 4, letter c, post).
It is desirable that an umpire or arbitrator, in fixing his fees, should do so by reference to considerations which he can put
forward and expect to justify as being reasonable, should a taxation be called for under s 19(1) a of the Arbitration Act, 1950.
Those considerations would normally involve, as at least two major factors, the period of time, whether days or hours, which he
has in fact reasonably devoted to the work which he has done, and the scale of charges for his time so computed (see p 5, letter a,
post).
________________________________________
a Section 19(1), so far as material, provides: If in any case an arbitrator or umpire refuses to deliver his award except on payment of the fees
demanded by him, the High Court may, on an application for the purpose, order that the arbitrator or umpire shall deliver the award to the
applicant on payment into court by the applicant of the fees demanded, and further that the fees demanded shall be taxed by the taxing
officer and that out of the money paid into court there shall be paid out to the arbitrator or umpire by way of fees such sum as may be found
reasonable on taxation

In his award the umpire in a commercial arbitration included an award of the costs of the award (as distinct from costs of the
reference), the aggregate sum so awarded including fees of the two arbitrators and a sum by way of his own remuneration. He
did not attempt to assess the value of the arbitrators services or the remuneration which they could fairly claim to be paid for the
work done, but included in the award the fees which they 1 asked him to include, believing this to be in accordance with the usual
practice in such arbitrations.

Held The umpires jurisdiction to fix the costs of the award derived from s 18(1) of the Arbitration Act, 1950 b, which conferred
power to tax or settle the costs, including arbitrators fees, and both tax and settle involved the application by the umpire of
his own independent mind and judgment to the fees demanded and the work done in order to be satisfied that the fees were fair
and reasonable, bearing in mind the interests of the party who would have to pay them as well as the legitimate interests of the
arbitrators (see p 6, letter e, post); the umpire had not, however, so taxed or settled the arbitrators fees or his own, and
accordingly was guilty of technical misconduct, and the award would be remitted for reconsideration of the provision as to costs
of the award (see p 7, letter f, post).
________________________________________
b Section 18(1), so far as material, provides: Unless a contrary intention is expressed therein, every arbitration agreement shall be deemed to
include a provision that the costs of the reference and award shall be in the discretion of the arbitrator or umpire, who may direct to and by
whom and in what manner those costs or any part thereof shall be paid, and may tax or settle the amount of costs to be so paid or any part
thereof

Per Curiam: if an umpire taxes and settles the costs of arbitrator-advocates, he should distinguish between that part of their
fees which is referable to their judicial functions before they have disagreed and that part which is referable to their functions as
advocates thereafter (see p 7, letter f, post).
Notes
As to costs of an arbitration, see 2 Halsburys Laws (3rd Edn) 47, para 103; and for cases on the subject, see 2 Digest (Repl) 545,
546, 808822.
As to remedies where an umpires or arbitrators remuneration is excessive, see 2 Halsburys Laws (3rd Edn) 49, para 108;
and for cases on the subject, see 2 Digest (Repl) 549, 550, 847853.
For the Arbitration Act, 1950, s 18, see 29 Halsburys Statutes (2nd Edn) 104.

Case referred to in judgment


Appleton v Norwich Union Fire Insurance Society Ltd (1922), 13 Lloyd LR 345.

Motion to set aside award


This was a motion by the respondent shipowner, Mrs Evgenia J Chandris, to set aside or remit an award of an umpire made in the
form of a Special Case under s 21(1)(b) of the Arbitration Act, 1950, in a dispute between the owner and the claimant charterers,
the Government of Ceylon. On 31 May 1953, the owner entered into a charterparty with the charterers whereby she chartered a
vessel to the charterers for a voyage from Burma to Ceylon to carry a cargo of rice in bags. The charterparty incorporated the
Centrocon charterparty arbitration provisions under which, in the event of arbitration, there were to be two arbitrators carrying on
business in London who should be members of the Baltic and engaged in the shipping and/or grain trades. Each side was to
nominate an arbitrator. During or after discharge of the cargo at Colombo, disputes arose. The owner appointed Mr J O Struthers
and the charterers Mr J Chesterman as arbitrators. The arbitrators, being unable to agree after a meeting of about two hours,
appointed Mr F J Norman as umpire. The case is reported only in relation to para 2 of the notice of motion, which alleged that
the umpire misconducted himself by settling his own and the arbitrators fees at a figure which was wholly excessive. The facts
are set out in the judgment.

C S Staughton and A D Coleman for the owner.


E F N Gratiaen QC, B C Sheen and N A Phillips for the charterers.

Cur adv vult


2

25 February 1963. The following judgment was delivered.

MEGAW J. I now come to para (2) of the notice of motion, which reads:

The umpire misconducted himself by settling his own and the arbitrators remuneration at a figure which was wholly
excessive.

The relevant parts of the award are as follows. The second paragraph of para 14 reads:

And I award that the owner does pay the costs of this award amounting to 612 3s. (of which 8 8s. is the cost of
hiring a room for the hearing of the arbitration and 183 15s. and 157 10s. are the fees of the two arbitrators respectively).
I further award that the owner does pay to the charterers their costs of the reference (to be taxed unless agreed).

Paragraph 18 reads:

And I direct that if the charterers shall in the first instance pay the costs of this award then the owner shall repay to the
charterers the amount so paid.

The charterers took up the award, paying the umpire the sum of 612 3s. If the award stands, both on this motion and in the
Special Case which may fall to be heard hereafter, the charterers will be entitled to recover the whole of that sum from the owner,
as well as their taxed costs of the reference.
The first question is whether the court has jurisdiction to consider an allegation of excessive remuneration. Both parties
agreed, through their counsel, that there is jurisdiction. That view is amply supported by the judgment of the Divisional Court in
Appleton v Norwich Union Fire Insurance Society Ltd ((1922), 13 Lloyd LR 345 at p 347). Salter J said:

I do not doubt that where an excessive charge is shown the court would act. Arbitrators are bound to act
conscientiously and with a proper regard to the interests of the parties, and if they dishonestly subordinate the interests of
the parties to their own, and take money from the parties in excess of what is just, I cannot myself doubt that that would be
misconduct.

The Arbitration Act, 1950, itself gives power, in certain circumstances, to have the remuneration of umpires and arbitrators
taxed. Section 18(1) gives to the arbitrator or umpire a discretion as to the costs of the reference and the award. When he awards
costs, he may either leave them to be taxed by the taxing machinery of the court in whole or in part, or he may himself tax or
settle the amount of costs to be so paid or any part thereof. A common practice is for the umpire to separate the costs of the
award (the arbitrators and umpires fees and any expenses which they have incurred) from the costs of the reference, which are
the costs incurred by the party to whom costs are awarded, other than the costs of the award. The costs of the award are taxed or
settled by the umpire. The costs of the reference are left to be taxed by the taxing machinery of the court, if necessary. The
umpire stipulates that the payment to him of the costs of the award shall be a condition of the delivery of the award to the party
requiring delivery of it, with a provision that, if the award be taken up by the party to whom the costs have been given, the other
party shall reimburse him. That was the practice which the umpire at least purported to follow in this case. Section 19(1) of the
Act of 1950 provides that, if the delivery of the award is made conditional on payment to the umpire, a party desiring delivery of
the award may apply to the High Court, which may order the delivery of the award to the applicant on his payment into court of
the full amount of the fees demanded. There is then a taxation of the fees, and the amount in court is paid out according to the
result of the taxation. The umpire can, of course, appear and be heard on such taxation. That is, no doubt, a useful provision, but
it leaves a gap. If the party who turns out to be the successful party has 3 taken up the award and paid the fees, the section does
not apply. The other party, who in fact has to bear the burden of the fees, cannot apply to the court. Thus, under the section as it
now stands, it is largely a matter of chance whether or not a party can make use of this statutory protection in respect of fees
which he claims to be excessive.
As I have said, both sides here agree that, in the present circumstances, the court has jurisdiction. The charterers say,
however, that not only is the onus on the owner to show that the fees demanded are excessive, but also that it is a very heavy
onus. They emphasise the words of Salter J, if they dishonestly subordinate the interests of the parties to their own; and they
contend that, before the court acts, it should be satisfied on the evidence before it that there has been such a dishonest
subordination. In my judgment, the court would not normally intervene in a motion such as this unless (i) it is satisfied that the
fees can properly be described as extravagant; or (ii) it is apparent that the umpire has seriously misunderstood his duty as
regards the assessment of fees which he demands by his award, whether for himself or, through himself, for the arbitrators, or for
both. Absence of proper assistance from the umpire, in the way of adequate information to the court when the fees prima facie
appear to be out of relation to the work involved and are challenged as being grossly excessive, may be treated as leading to the
conclusion that the umpire has misunderstood his duty. It has to be realised that a serious responsibility rests on one who is put in
a position to assess his own reward; particularly since his self-allotted remuneration may fall to be paid by someone who had no
part in his appointment; for example, in the case of the arbitrator appointed by the opposite party, or the umpire appointed, not by
either party, but by the two arbitrators. Of course, if the fee, or the scale of fees, to be charged is made known to both parties and
agreed by them before the appointment is accepted, there would be much less chance of embarrassment and complaint. However
that may be, this much is clear: an umpire or arbitrator, in fixing his fees, must consider not only his own interest, but also the
interest of the party or parties who will ultimately be required to pay those fees.
I was invited by counsel for the owner to take the view that, at least for arbitrations of this nature, where the arbitration
clause expressly provides that the arbitrators shall be Members of the Baltic and engaged in the shipping and/or grain trades,
the court should lay down some sort of appropriate scale, which would take account of the fact that the arbitrators whom the
clause contemplates are gentlemen normally engaged in business and accepting duties as arbitrators or umpires as a kind of
public service for the general good of the trade in which they are normally engaged, rather than as persons who look to
arbitrations as a substantial part of their livelihood. I am not sure that this is an altogether realistic view in relation to arbitrations
of this nature in the circumstances which exist today. It was suggested, further, that, whether or not that view be correct, in the
absence of special circumstances, which would have to be proved by those who have assessed their fees as arbitrators or umpires,
twenty-five guineas per day for the time actually occupied in the hearing should be regarded as the maximum reasonable figure
for this type of arbitration, and that anything at least substantially above that figure should normally be regarded as excessive or
calling for special explanation. It was pointed out that, in this case, the total time actually occupied by the umpire in the hearing
of the arguments was not more than about four or five hours, and that the total fees awarded by the umpire to himself and the
arbitrators, excluding out of pocket disbursements, which were not challenged, were over 500. It was submitted that the
umpires fee should not have exceeded seventy-five guineas and the arbitrators fees fifty guineas each.
I think that it would be undesirable for me to express any general view as to the appropriate scale of fees in arbitrations such
as this. The fees which are fair and appropriate depend on many factors, and generalisations would be 4 unsatisfactory and
dangerous. Nevertheless, I think that I can properly say this: it would be desirable that an umpire or arbitrator, in fixing his fees,
should do so by reference to considerations which he can put forward and expect to justify as being reasonable, should a taxation
be called for under s 19(1) of the Arbitration Act, 1950. Those considerations would normally involve, I apprehend, as at least
major factors, the period of time, whether days or hours, which he has in fact reasonably devoted to the work which he has done,
and the scale of charges for his time so computed. If the fees are challenged, their basis can be clearly stated and examined.
The umpires fee has been fixed by him at 172 16s. That is the total of 612 3s, less the arbitrators fees totalling 341 5s
and less 8 8s for the hire of a room and 89 14s legal charges incurred for, inter alia, the settling of the Special Case. Out of this
172 16s, the umpire paid a further twenty-five guineas to one of the arbitrators, Mr Struthers. That arose in this odd and, I am
bound to say, unsatisfactory way. Mr Chesterman, the other of the two arbitrators, originally requested that the umpire should
include for him a fee of 150 guineas. So did Mr Struthers. When a Special Case was ordered, Mr Chesterman thought that that
entitled him to additional remuneration, and he asked the umpire to include a further twenty-five guineas. Mr Struthers made no
such request. In the award, the umpire stated the arbitrators fees as 183 15s and 157 10s. He sent a cheque for these sums to
Mr Chesterman and Mr Struthers respectively. Mr Struthers replied that, on the first page of the award, his name appeared before
Mr Chestermans; therefore, he claimed, the sum first mentioned in para 14 (the arbitrators names not being there mentioned)
applied to him. The umpire replied, telling Mr Struthers of Mr Chestermans subsequent request for an additional twenty-five
guineas; but the umpire at the same time sent Mr Struthers a cheque for a further twenty-five guineas. It is fair to make it clear
that the umpire did not seek to make this further twenty-five guineas an additional charge against the parties. He paid it,
presumably, out of his own fee of 172 16s. For the purposes of this motion, however, the umpires remuneration must be treated
as being 172 16s, and that of the arbitrators as totalling 341 5s. The owner contends that, prima facie, the umpires fee of 172
16s is excessive in relation to an arbitration where the whole hearing lasted not much more than four hours at most, and where,
moreover, the task of preparation of the draft award was handed over by the umpire to solicitors, whose remuneration the owner
does not challenge. The owner says that, if there are special factors which justify such a fee for so apparently limited an amount
of work, those factors are within the knowledge of the umpire, and it is for him to declare them with sufficient precision if the
prima facie case of excessive fees is to be rebutted. The umpire, in his affidavit put in evidence by the charterers, does indeed
mention various matters, such as that he had to read carefully through the documents put in evidence; and the transcript of the
arguments on two occasions; that he had to have many consultations with the solicitors whom he instructed to settle the draft
award; and that he had extensive correspondence with the parties representatives. It would have been better if the umpire had
seen fit to specify more fully the time which was reasonably occupied by him on the work involved in these aspects, together
with any other special factors. I should, nevertheless, have hesitated long before holding that the umpires fee was extravagant,
and I should have been disposed, had it not been for another matter, to have adjourned the motion in order to give the umpire a
further opportunity, if he saw fit, to supplement the information which he has given. I shall come back to that other matter after I
have considered the question of the fees awarded by the umpire to the arbitrators.
As regards those fees, the evidence before me indicates no more than this: that the two arbitrators, having received and no
doubt read the documents supplied by their respective clients, had a two hour meeting on 2 June 1961, when they discussed these
documents and agreed to disagree. Mr Struthers 5subsequent work seems to have involved preparing two written submissions,
attending to ask for an adjournment on 5 July and attending the hearing on 26 July when counsel presented the owners case. Mr
Chesterman, in addition, prepared and presented the charterers case on 26 July and six months later prepared suggested findings
of fact. The umpire has not supplemented that information, which I have derived from other affidavits. The reason for this lack
of further information is that the umpire, as he makes clear in his affidavit, did not regard it as in any way his business to consider
or assess the value of the services of the arbitrators or the remuneration which they could fairly claim to be paid. He makes it
clear in his affidavit that he, quite simply, included in the award the amounts which they asked him to include, because they asked
for them. This, says the umpire, is in accordance with the usual practice. He says:

It is not the practice for an umpire to settle or to tax the fees of the arbitrators, and I did not settle or tax the fees of Mr.
Chesterman or Mr. Struthers in this case.

I have no doubt that the umpire genuinely believed that this was an accepted and proper practice, and that that is why he followed
it. If, however, that is the usual practice, it is, in my judgment, a practice which is wrong and unjustifiable and which should not
be followed. It is contrary to the meaning and intent of s 18(1) of the Arbitration Act, 1950. It is by virtue of that subsection that
an umpire has power to fix the amount of any costs to be awarded, including the arbitrators fees. The power is to tax or settle.
The umpire, as he himself says, did not purport to tax or settle the arbitrators fees. Therefore, his inclusion of the 341 5s in
respect of them is ultra vires. Both tax and settle involve the application by the umpire of his own independent mind and
judgment to the fees demanded and the work done in order to be satisfied that the fees are fair and reasonable, bearing in mind
the interests of the party who will have to pay them, as well as the legitimate interests of the arbitrators. It is, of course, open to
the umpire, if he does not wish to tax or settle the fees, because he finds it embarrassing or because he feels that he has not
sufficient information, not to tax or settle them under s 18(1), but to leave them to be dealt with outside the costs of the award, by
way of taxation or otherwise.
There may be cases where the umpire can satisfy himself that the fees put forward by the arbitrators have in fact been fixed
and agreed between the parties, whether by the arbitrators having express authority from the respective parties so to agree, or
otherwise. In such a case, an umpire need not go behind the agreement or investigate the agreed fees. There is no suggestion, in
the umpiress affidavit or elsewhere, that there was any such agreement, or authority on the part of the arbitrators to agree so as to
bind the parties, in this case. Indeed, Mr Chestermans twenty-five guineas for the Special Case, not known to Mr Struthers until
after the award was published, would destroy the possibility of any such agreement having existed here. I mention this only
because it was argued for the charterers that the umpire here, though he does not himself say so, must have thought that the
arbitrators, with authority from their respective appointors, had agreed one anothers fees. That being so, it follows that the
umpire has seriously misunderstood his duty as regards the assessment of the arbitrators fees. This amounts to what is
technically misconduct and the inclusion of those fees cannot stand as a part of the award.
I now have to return to the umpires own fees. The umpires attitude shows that, with regard to the arbitrators fees, he did
not understand that his duty was to consider them from the point of view of the interest of the party who would have to pay them,
as well as of the arbitrators themselves. Therefore, as it seems to me, there is at least the possibility that he did not, in the proper
sense, tax or settle his own fees. He may or may not have taken into account the interest of the party who would have to pay the
fees, in carrying out this 6 invidious and delicate task of assessing his own remuneration. It is impossible to be confident that he
did so. In the circumstances, I do not think that it would be right to allow that assessment to stand. The owner submitted that, if
the provision as to the fees in the award should not be allowed to stand, the whole award should be set aside. I do not think that
that is the proper course. Although the umpire has taken a wrong, and unfortunate, view as to the assessment of fees, I see no
reason to think that the fair inference is that he did not approach the decision of the substantive issues in the case in a proper
judicial spirit. Whether he decided the questions of law rightly or wrongly is a matter which remains to be determined hereafter,
and nothing that I have said in my judgment on the motion has any bearing on that question. In my judgment, the award should
be remitted to the umpire for the reconsideration of the fees awarded in the light of my judgment on this motion.
I do not think that I have power to direct the umpire what he should find on such reconsideration. But I tink I may properly
say that, in my view, it would obviously be gravely embarrassing for the umpire now himself to settle or tax his own fees or those
of the arbitrators. I assume, however, that his only desire, on reconsideration, will be to act fairly and properly. If I had taken
any other view as to his attitude, I should have set aside the whole award. The umpire will, I am confident, realise that there is
now open to him a course which will avoid such embarrassment, and which will ensure that there is an impartial review, such as
would not now be possible for him himself to be seen to have applied. That course is to maintain, as costs of the award properly
taxed and settled by him, the two unchallenged items of his out of pocket expenses, namely, 8 8s for hire of a room and 89 14s
for his legal expenses; and to direct that all the remaining costs of the award, as well as the costs of the reference, shall be taxed
unless agreed. In any such taxation, the umpire and the arbitrators would be entitled, as the umpire would have been in a taxation
under s 19 of the Arbitration Act, 1950, to appear and to be heard, in respect of the umpires and the arbitrators fees, on any
taxation or review of taxation.
I, therefore, remit the award for reconsideration of the provisions as to costs of the award contained in the second paragraph
of para 14 of the award. I would only add this. In my view, if in future an umpire taxes and settles the costs of arbitrator-
advocates, he should distinguish between that part of their fees which is referable to their judicial functions before they have
disagreed and that part which is referable to their functions as advocates thereafter. Otherwise, the position on any taxation
which may be required of the costs of the reference may be difficult or impossible, since the taxing officer may have no means of
distinguishing between the two in his taxing of the costs of the reference.

Order accordingly.

Solicitors: Richards, Butler & Co (for the owner); T L Wilson & Co (for the charterers).

Mary Colton Barrister.


7
[1963] 2 All ER 8

Baldwin v Worsman
CRIMINAL; Road Traffic

QUEENS BENCH DIVISION


LORD PARKER CJ, ASHWORTH AND WINN JJ
6 MARCH 1963

Road Traffic Licence Driving licence Groups of vehicles for which driving licence granted Three-wheeled motor vehicle
constructed with means of reversing which not immediately usable by reason of reversing equipment being blanked off Whether
vehicle within group A or group G Motor Vehicles (Driving Licences) Regulations, 1950 (SI 1950 No 333), Sch 2.

The respondent held a driving licence to drive vehicles in group G of Sch 2 to the Motor Vehicles (Driving Licences)
Regulations, 1950. He also held a provisional driving licence to drive vehicles in group A of that schedule. He drove a three-
wheeled motor vehicle constructed to carry more than one person and which had a reversing gear, but the reversing gear could
not be used because it was blanked off, ie, a piece of metal had been put in such a position that the gear lever would not go into
the reverse position. Group G of Sch 2 to the 1950 Regulations covered motor-bicycles or tricycles not equipped with means for
reversing, while group A covered, inter alia, motor cars or motor tricycles equipped with means for reversing. On appeal from
dismissal of an information charging failure to comply with a condition of the respondents provisional licence, viz, a charge
based on the motor vehicles falling within group G,

Held A vehicle constructed with the means of reversing did not cease to be a motor vehicle equipped with means for reversing
within group A of Sch 2 to the 1950 Regulations merely because the means for reversing was not immediately usable by reason
of it being blanked off (see p 9, letters f and h, post); accordingly the case would be remitted to the justices with a direction to
convict.
Appeal allowed.

Notes
As to the groups of vehicles for which driving licences may be granted, see 33 Halsburys Laws (3rd Edn) 458, para 780, note (p).

Case Stated
This was a Case Stated by justices for the City of Bradford in respect of their adjudication as a magistrates court sitting at
Bradford on 21 September 1962. On 7 September 1962, the appellant, Cyril Baldwin, preferred two informations against the
respondent, Fred Worsman, that, on 18 August 1962, in the City of Bradford, being a person to whom a provisional licence had
been granted and not having passed the appropriate test, he failed to comply with the condition subject to which the provisional
licence was granted, in that he used a motor car constructed to carry more than one person on a road called Hammerton Street (a)
when not under the supervision of a person who was present in the motor car with him and who held a licence other than a
provisional licence authorising him to drive a vehicle of the same class as the motor car and who had been the holder of a licence
for at least two years or had passed a test under s 6 of the Road Traffic Act, 1960, and (b) when, not clearly displaying in a
conspicuous manner on the front and on the back of the car a distinguishing mark in the form set out in Sch 7 to the Motor
Vehicles (Driving Licences) Regulations, 1950, contrary to reg 16(3) of the regulations and s 102(3) of the Road Traffic Act,
1960.
The following facts were found. On 18 August 1962, the respondent was driving a Reliant three-wheeled motor vehicle on
Hammerton Street. The vehicle was constructed to carry more than one person, and was not at the material time displaying a
distinguishing mark in the form set out in Sch 7 to the 1950 Regulations. The respondent was in possession of a provisional
licence to drive vehicles of group A of the 1950 Regulations, including a motor tricycle equipped with means 8 for reversing. He
was also in possession of a licence to drive vehicles of group G of the regulations, including a motor tricycle not equipped with
means for reversing. The respondent had not passed the appropriate test for vehicles of group A, and was not under the
supervision of any person who was present in the vehicle with him. The vehicle was constructed with a means for reversing,
namely, a reverse gear, which was blanked off, in that a piece of metal had been inserted into a position by the manufacturer in
its construction which rendered the gear unusable. The gear could be made usable by removing the piece of metal, the time
which would be taken to remove the metal being at least half an hour.
It was contended by the appellant that the vehicle was equipped with means for reversing and, therefore, the vehicle was a
vehicle of group A of the 1950 Regulations. It was contended by the respondent that the vehicle was not equipped with means for
reversing and, therefore, the vehicle was a vehicle of group G of the regulations.
The justices dismissed both informations, and the appellant now appealed.
The case noted belowa was cited during the argument.
________________________________________
a Lawrence v Howlett [1952] 2 All ER 74, DC, WN 308, 1 TLR 1476

J A Cotton for the appellant.


The respondent did not appear and was not represented.

6 March 1963. The following judgments were delivered.

LORD PARKER CJ stated the facts, and continued. The respondent was in fact the holder of a licence to drive vehicles in
group G, and, if this vehicle that he was driving was within group G, he committed no offence. On the other hand, if it was
within group A, he was merely the holder of a provisional licence, and he was in breach in the two respects alleged. The sole
question, therefore, is into which group the vehicle fell. In fact, this was a Reliant three-wheeled motor vehicle which had a
reversing gear, but the reversing gear at the time could not be used, because a piece of metal had been put in such a position that
the gear lever would not go into the reverse position; it was, as it is called, blanked off.
The groups of vehicles are set out in Sch 2 to the Motor Vehicles (Driving Licences) Regulations, 1950, and group A covers,
so far as is material here, a motor car or motor tricycle equipped with means for reversing. Group G on the other hand covers a
Motor-bicycle (with or without side-car) or tricycle not equipped with means for reversing. The short point here on which we
are told that magistrates courts are coming to different conclusions, is whether a vehicle of this sort, with the reversing gear
blanked off so that it cannot be used, is or is not equipped with means for reversing. For my part, I have come to the clear
conclusion that if a vehicle is constructed with the means of reversing and is found on a road with that equipment intact, it does
not cease to be a motor vehicle equipped with means for reversing merely because it is not immediately usable by reason of its
being blanked off.
In my judgment, the magistrates came to a wrong decision; this appeal should be allowed and the Case sent back to them
with a direction to convict.

ASHWORTH J. I agree.

WINN J. I agree.

Appeal allowed. Case remitted.

Solicitors: Wilkinson, Howlett & Moorhouse agents for Town clerk, Bradford (for the appellant).

Shireen Irani Barrister.


9
[1963] 2 All ER 10

Dove v Dove
FAMILY; Divorce

COURT OF APPEAL
ORMEROD, DONOVAN AND RUSSELL LJJ
1, 4 MARCH 1963
Divorce Decree nisi Substitution of decree of judicial separation Discretion Factors to be considered Respondents right
to apply in due course to make decree nisi absolute Appropriate procedure Matrimonial Causes Act, 1950 (14 Geo 6 c 25), s
12(3).

In determining whether to substitute a decree of judicial separation for a decree nisi of divorce the court should take into
consideration, in a case where three months from the earliest time when application could be made for a decree absolute has not
yet elapsed, that the substitution of the order sought in place of the decree nisi may nullify the right of the party against whom the
decree nisi was obtained to apply under s 12(3) of the Matrimonial Causes Act, 1950, for a decree absolute (see p 11, letter h, and
p 12, letters e and i, post); and (per Donovan and Russell LJJ) the proper practice, if the respondent wishes to apply under s 12(3),
is to defer the hearing of the application to substitute a decree for judicial separation until the time has elapsed which will enable
the other spouse to apply for a decree absolute, and then to determine both applications together (see p 12, letters d and h, post).

Notes
As to the substitution of a decree of judicial separation for a decree nisi of divorce, see 12 Halsburys Laws (3rd Edn) 322, para
650; and a case on the subject, see 27 Digest (Repl) 687, 6566.
For the Matrimonial Causes Act, 1950, s 12(3), see 29 Halsburys Statutes (2nd Edn) 400.

Cases referred to in judgments


Davies v Davies [1955] 3 All ER 589, [1956] P 212, [1955] 3 WLR 84, 3rd Digest Supp.
Jeffrey v Jeffrey [1950] 2 All ER 449, [1951] P 32, 27 Digest (Repl) 687, 6566.

Appeal
This was an appeal by a husband, who was respondent to a petition by his wife for divorce, against an order of Payne J, made on
the wifes application, substituting a decree of judicial separation for the decree nisi of divorce already granted on her petition.
The facts are stated in the judgment of Ormerod LJ.

D Armstead Fairweather for the husband.


J A Kirpal for the wife.

4 March 1963. The following judgments were delivered.

ORMEROD LJ. This is a respondent husbands appeal from a decision of Payne J given on 28 June 1962, in these
circumstances: the parties were married and eventually separated, having had six children as the result of the marriage. On the
wifes petition a decree nisi of divorce was pronounced on 2 February 1962. That was on the ground of adultery. The wife in due
course made an application to the court for the decree nisi to be set aside and for an order for judicial separation to be substituted.
The learned judge, after giving the matter careful consideration, decided to make the order asked for. It is sufficient to say,
although this is in no way material to the matter before us on this appeal, that the grounds of the wifes application were, in the
first place, that, owing to the difficulty of finding further accommodation for herself and her six children, she felt it wiser in the
circumstances to remain in her husbands house and that it would be better if she had an order for judicial separation rather than
divorce. The second ground was that she felt it would be in the best interests of the children that their father and mother should
remain married at least until they were old enough to form their own views; and thirdly, although I think that she 10 has not put
this reason very high, that on moral and religious grounds she felt that she would not care to be a party to divorce proceedings.
Be that as it may, the application came before Payne J on 30 May 1962; he adjourned it for twenty-one days, and it came on
finally on 28 June when he considered the matter and made the order sought. Counsel on behalf of the husband first raises the
point that there was no jurisdiction in the court to make this order, having regard to s 12(3) of the Matrimonial Causes Act, 1950.
The effect of that subsection, which is a re-enactment of a similar provision a in the Matrimonial Causes Act, 1937, is that if the
spouse obtaining the decree nisi does not make an application for it to be made absolute three months after the trial, when he or
she could first apply for it to be made absolute, then the other spouse can apply after a further period of three months, and, if the
circumstances warrant, obtain a decree absolute. Counsel for the husbands submission in the first place is that, accepting that
there was jurisdiction in the court to substitute for an order of decree nisi an order for judicial separation, or vice versa, that had
been ousted by inference as an effect of s 12(3). The learned judge did not accept that contention; he expressed the view that if
the legislature had intended to revoke this jurisdiction, it would have been done in a clearer way than by inference from the
subsection; and with that I am fully in agreement. I see no reason, if this jurisdiction existed, for saying that s 12(3), although it
may confer a right on the spouse when an order for decree nisi is made, means that there is no jurisdiction in the court to rescind
that order and substitute another; but that jurisdiction is, I think, not in question. There is no doubt, and neither party has said
anything to the contrary, that this jurisdiction has existed and that the practice has grown up whereby the court should exercise it,
and indeed the principles on which it is based are set out and discussed in Rayden on Divorce b with reference to the helpful
authorities.
________________________________________
a Section 9 of the Act of 1937, which added a new sub-s (3) to s 183 of the Supreme Court of Judicature (Consolidation) Act, 1925
b Rayden on Divorce (8th Edn), 1960, pp 321, 322, para 54, text and notes (a)-(d)

However, there is a further submission which was made by counsel for the husband which does seem to be material, and that
is that the jurisdiction which was in the court prior to the passing of the Matrimonial Causes Act, 1937, c was, of course, the
exercise of discretiona discretion to be exercised on established judicial principles. The submission is that a new factor had
been introduced by s 12(3) which the learned judge in exercising his discretion should take into account. It may be, of course,
that, as the result of the decree nisi, the other spouse has entered into some form of arrangement with some third party. It may be,
as I believe is the case here, that a child is likely to be born in the comparatively near future which would be illegitimate if the
husband is not in a position to marry the prospective mother of the child. Those and many others are matters which the learned
judge may choose to take into consideration, and it seems to me material that he should take into consideration the fact that if
there is a substitution of the one order for the other, it may well be that the circumstances are such that s 12(3) will in effect be
nullified. That is a contention which I am bound to say appeals to me. I think that it is clear that the learned judge did not take
that matter into consideration in coming to the decision. In my view the effect of s 12(3) is to introduce a new factor for
consideration, that new factor being that the guilty spouse has rights which have accrued as a result of the decree nisi being made,
and it is a factor to be taken into account by the learned judge in exercising his discretion. In those circumstances I would allow
this appeal to the extent that I would order that the whole matter be referred back and that there be a new hearing, so that whoever
hears the case shall have an opportunity of taking into account in the exercise of his discretion the matter to which I have
referred.
11
________________________________________
c Ie, under s 183(2) of the Act of 1925; see footnote (1) supra

I should add this: counsel for the husbands further contention was that on the proper construction of s 12(3), in the
circumstances of this case, as the three months had gone by and the wife was in a position to ask for an order for the decree to be
made absolute and had not done so, the proper construction was that she could not ask for a new order because of the effect of s
12(3). I do not think that counsel for the husband urged that submission with any great sense of conviction and all that I need say
is that I reject it.

DONOVAN LJ. I agree that s 12(3) of the Matrimonial Causes Act, 1950, re-enacting the provision introduced by s 9 of the
Matrimonial Causes Act, 1937, does not deprive the court of the jurisdiction, which it has exercised for many years, of altering a
decree nisi into a decree of judicial separation on the application of the person who obtained the decree nisi; but the section does
introduce this new factor, namely, that the respondent may be prejudicially affected if such an application be granted. He or she
will be deprived, in effect, of the value of his or her right to apply after the necessary six months for the decree to be made
absolute. This may well be serious if, for example, children are expected from some new relationship into which the respondent
may have entered relying on his or her prospective divorce. The right way to deal with this situation is to defer such application
to alter the decree nisi to a date when the respondents right to apply for the decree to be made absolute will have accrued. If the
respondent makes such an application, then both applications can be dealt with together in the light of the current facts. This was
not done in the present case. The wifes application was allowed on 28 June 1962, whereas the husbands right to ask for the
decree to be made absolute did not accrue until 6 August 1962. The eventual result in this case may be no different; I do not
know; but the husband is, I think, entitled to be allowed to make his application, and I accordingly agree with the order proposed.

RUSSELL LJ. It is undoubtedly a lawful practice of long standing to permit the substitution of a decree of judicial separation
for a decree nisi of divorce; but when, by s 9 of the Matrimonial Causes Act, 1937, a right was conferred for the first time on the
respondent, after a period (which is now six months) has elapsed from the decree nisi, to apply for it to be made absolute, it
seems to me that a new factor was introduced relative to the decision whether or not to substitute judicial separation for decree
nisi of divorce. I consider that the practice requires to be regulated so as to permit proper weight to be given to that new factor
new at least in 1937. If on such an application by the petitioner the respondent raises no objection, as was the case in Davies v
Davies, then the order can be made in favour of the petitioner even if the six months period has not elapsed. If the six months
have elapsed, then the respondent is in a position to make his, so to speak, cross-application under s 12(3) and the two
applications can be heard together, which is what happened in Jeffrey v Jeffrey. If on the petitioners application, the respondent
indicates that he will wish to apply under s 12(3) when the six months have elapsed (as was the case here), in my view the
petitioners application should be adjourned for a period sufficiently long to enable that to be done. As I read the judgment in this
case, the learned judge took the view that in dealing with the wifes application, no consideration at all should be given to s 12(3),
there being no application before the court under that subsection. This in my view was a wrong approach and the judges order
ought to be set aside. On the merits of the case I say nothing at all. The question will be considered on up-to-date evidence when
the wifes renewed application and the husband respondents application which he proposes to make under s 12(3) come to be
heard together.
12

Appeal allowed; case sent back for re-hearing. Order for judicial separation set aside; decree nisi of divorce restored as from
the original date; respondent husband to have an opportunity to apply for decree nisi of divorce to be made absolute.

Solicitors: Johnson, Jecks & Landons agents for Landons, Brentwood (for the husband); Nehra, Emerson, Naylor & Co (for the
wife).

Henry Summerfield Esq Barrister.


[1963] 2 All ER 13

Lovelock v Margo
LANDLORD AND TENANT; Leases

COURT OF APPEAL
LORD DENNING MR, DANCKWERTS AND DAVIES LJJ
27, 28 FEBRUARY 1963

Landlord and Tenant Lease Forfeiture Relief Breach of covenant Peaceable re-entry by landlord.

Landlord and Tenant Lease Assignment Covenant against assignment without consent Landlord giving unjustified reason
for withholding consent Whether landlord can later justify withholding consent on other grounds.

In May, 1960, the landlord let to the tenant a lock-up shop under a lease of twenty-one years at a yearly rent of 260 payable
weekly. The lease contained a proviso for re-entry in case of non-payment of rent, and a covenant against assignment without the
consent of the landlord. In 1961 the tenant asked the landlord for her consent to assign the lease and produced satisfactory
references of the proposed assignee. The landlord refused consent on the ground that the area of the premises to be transferred
was uncertain. In August, 1961, when 25 rent was due, the landlord peaceably re-entered the premises and excluded the tenant
therefrom. The tenant claimed relief against forfeiture and a declaration that the landlord had unreasonably withheld her consent
to assignment. On appeal by the landlord from an order of the county court judge granting relief against forfeiture and the
declaration claimed, it was contended on behalf of the landlord that there were other points, not taken by the landlord herself,
why it might be reasonable to withhold consent.

Held The order would be upheld because


(i) the right to relief against forfeiture extended to a case where there had been re-entry without action (see p 14, letter f, and
p 15, letters f and g, post).
Howard v Fanshawe ([1895] 2 Ch 581) applied.
(ii) the question whether consent was unreasonably withheld was not an objective question and could not be considered
without regard to the state of mind of the landlord herself as to her reasons for refusing consent (see p 15, letter c, post).
Appeal dismissed.
Notes
As to relief from forfeiture for non-payment of rent, see 23 Halsburys Laws (3rd Edn) 681, 682, para 1409; and for cases on the
subject, see 31 Digest (Repl) 535, 536, 65926608.
As to unreasonable withholding of consent to assign, see 23 Halsburys Laws (3rd Edn) 633, 634, para 1338; and for cases
on the subject, see 31 Digest (Repl) 424427, 55155536.

Case referred to in judgments


Howard v Fanshawe [1895] 2 Ch 581, 64 LJCh 666, 73 LT 77, 31 Digest (Repl) 535, 6599.

Appeal
This was an appeal by the landlord from an order of His Honour Judge Ruttle at Lambeth County Court, dated 17 July 1962,
granting the tenant the declaration that the landlord had unreasonably refused her consent to an assignment of 13 premises at
113A, Kennington Road, Lambeth, of which the tenant was lessee under a lease dated 18 May 1960, and that the tenant be
granted relief against forfeiture of the lease for non-payment of rent on terms that the tenant paid the arrears of rent and half the
landlords costs. The facts are set out in the judgment of Lord Denning MR.

R Shulman for the landlord.


Ashley Bramall for the tenant.

28 February 1963. The following judgments were delivered.

LORD DENNING MR. On 18 May 1960, the landlord let to the tenant a lock-up shop, No 113A, Kennington Road, under a
lease of twenty-one years from 7 March 1960, at a rent of 260 a year payable by weekly payments. There was a proviso for re-
entry in case the rent was not paid. In 1961 25 rent was not paid. I will not go into the reasons for it, but the landlord, on 28
August 1961, made what was said to be a peaceable re-entry; she got in and changed the locks and excluded the tenant from the
premises, claiming that it was done under a right of re-entry for non-payment of rent. The tenant, after a time, brought an action
in the county court claiming, amongst other things, relief against forfeiture, and the judge has given it.
A point was raised before us as a point of law to the effect that the court has not got power to grant relief from forfeiture
when there has been peaceable re-entry. It was said that the court can only grant relief when there has been an action for
forfeiture or an action for ejectment or for possession. In answer to that argument, it is sufficient to say that Stirling J, in Howard
v Fanshawe, pointed out that the jurisdiction

is not confined to cases where the lessor has recovered possession by legal process, but extends to cases where the
lessor has recovered peaceable possession without the assistance of any court

that being a jurisdiction not dependent on statute but the ancient jurisdiction of the Court of Chancery. Indeed, that is borne out
by the County Courts Act, 1959, s 191(3), which is framed on the very basis that that is the law; that the right to give relief
extends to a case where there has been re-entry without action. In my judgment, that point is clearly bad. The judge granted
relief and there was clear jurisdiction to grant it.
The second point is quite different. In this case, the tenant wanted to assign the lease of this fried fish shop quite early in the
term. On 8 August 1961, a letter was written by his solicitors saying that he had exchanged contracts with the proposed
purchaser, a Mr Jacobs, and enclosing references and asking for a licence to assign. The lease contained the usual covenant not to
assign without licence, and, of course, there was imported in it that licence was not to be unreasonably withheld. The matter was
held up because the landlord was abroad. Meanwhile the solicitor for the landlord went into the matter and said that he was
satisfied about it and would recommend the licence to the landlord, when she came home that week-end. When the landlord did
come home, she, or rather her husband, Mr Margo, stated on the premises that he would not give leave to assign. I need only
read the solicitors letter on behalf of the landlord on 24 August:

I confirm that my client has instructed me not to grant the licence to assign the premises requested as she is not
satisfied with the area proposed to be used in connexion with the shop.

That was the only reason given for the refusal to assign. In the action, the tenant claimed a declaration that the landlord had
unreasonably refused her consent to the assignment. The judge went into the matter and considered this question of the extent of
the area. He held that the point taken by the landlord was not a bona fide one. The reason for refusing the licence, he said, was
invalid. Even if there was a bona fide dispute between the parties as to the extent of the 14 premises held by the tenant (which he
did not accept), the tenant was entitled to assign (subject to his references, which were in fact satisfactory), whatever his interest
was under the lease. It seems to me that that reasoning by the judge is correct and unimpeachable. The landlord took a wrong
point and gave an unjustified reason for refusing the licence to assign.
Counsel for the landlord has argued that this is an objective question; that he desires to raise before us other points, not taken
by the landlord herself, why it might be reasonable to withhold consent. It was said that a reasonable time was not given for
consideration. The landlord never said that she needed further time. Then he said that third parties were involved; it would have
an effect on the next door property and the financial position of the tenant and matters of that kindmatters which were not
hinted at by the landlord herself or by her solicitors, and, indeed, there was no hint of it in the evidence. I am quite clearly of
opinion that it is not right to say that this is an objective question, as counsel said. This matter cannot be considered without
regard to the state of mind of the landlord herself as to her reasons for refusing consent. How otherwise can a lessee hope to see
whether he can assign unless he knows the landlords reasons for objection? It seems to me that, when the tenant produced an
assignee, a responsible person with good references to which no objection could be taken, the landlord had no good ground for
objecting. She put forward a bad ground and she has not shown any other before the court, and, in those circumstances, the judge
was perfectly justified in holding that consent had been unreasonably withheld and making a declaration accordingly.
As to the terms of the relief granted, no point is taken before us. The tenant had to pay the rent in arrear, but he only had to
pay half the costs and, on that being paid, he got relief. It seems to me that the judge was entitled to do that. We were told that,
notwithstanding the existence of this action and almost in defiance of it, a few days before the action came on for hearing, the
landlord let these premises to someone else, having got into the premises, as I have said. So much the worse for her. She takes
the risk if she lets them to others in face of a pending law case. If that is what the landlord did, the burden must rest on her. In
the circumstances, I see no fault in the judges judgment and I would dismiss the appeal.

DANCKWERTS LJ. I agree. I only want to add that I am completely satisfied that the decision of Stirling J, in Howard v
Fanshawe correctly stated the law and that that law has not been altered by the effect of the Judicature Acts and, in particular, the
provisions of s 46 of the Supreme Court of Judicature (Consolidation) Act, 1925.

DAVIES LJ. I also agree. I think that this appeal is perfectly hopeless.
Appeal dismissed.

Solicitors: Geoffrey Summers (for the landlord); Henry I Sidney & Co (for the tenant).

F Guttman Esq Barrister.


15
[1963] 2 All ER 16

Francis Day & Hunter Ltd and another v Bron (trading as Delmar Publishing
Co) and another
INTELLECTUAL PROPERTY; Copyright

COURT OF APPEAL
WILLMER, UPJOHN AND DIPLOCK LJJ
20, 21, 22, 25 FEBRUARY 1963

Copyright Infringement Reproduction Essential elements similarity and causal connexion Objective and subjective
questions, but questions of fact No irrebuttable presumption of causal connexion Subconscious copying as possible
infringement Copyright Act, 1956 (4 & 5 Eliz 2 c 74), s 2(5).

The opening bar of a song, In a Little Spanish Town, composed in 1926, was a common-place series of quavers found in
previous musical compositions and was very similar to though not identical with the opening bar of a new song, Why,
composed in 1959. This opening phrase was developed over the remainder of the first eight bars by the use of the same devices
or tricks of composition in both the songs, producing a definite or considerable similarity between them. In a Little Spanish
Town had been extensively exploited in the United States and elsewhere by the publication of sheet music, by the distribution of
records and by broadcasting. The composer of Why was a man of thirty-three who had lived most of his life in the United
States and had played various instruments in dance bands; his evidence was accepted that he had not seen or studied or in his
recollection played the music of In a Little Spanish Town nor had ever to his knowledge heard it, although he admitted that he
might have heard it at a younger age. In an action for breach of copyright in In a Little Spanish Town brought against the
publishers of Why, the trial judge found that he had insufficient factual material in the similarity of the works and in the
inference that the composer of Why had at some time heard In a Little Spanish Town to draw an inference, in the absence of
direct evidence and in face of the composers denial, that he had had sufficient knowledge or memory of In a Little Spanish
Town to have copied it without knowing that he was doing so, rather than that the similarity arose from coincidence, and
accordingly the trial judge held infringement of copyright was not established. On appeal,

Held The decision of the trial judge should stand, for the following reasons
(i) in order to constitute reproduction within s 2(5) of the Copyright Act, 1956, there must be both sufficient objective
similarity between the two works and some causal connexion between the work infringed and the infringers work, viz, it must be
proper to infer derivation of the latter from the former (see p 22, letters a and b, p 24, letter d, p 25, letter b, and p 27, letter i, to p
28, letter a, post).
(ii) the existence of both these elements was a question of fact, the first element (similarity) being an objective issue, and the
second element (causal connexion) being a subjective question (see, eg, p 24, letter g, post) but not to be presumed as a matter of
law merely from proof of possible access by the alleged infringer to the copyright work (see p 29, letter g, and p 23, letter i, to p
24, letter a, post).
(iii) in the present case there was no ground for interfering with the trial judges conclusion of fact (see p 23, letter e, p 26,
letter f, and p 30, letter c, post).
Per Willmer and Diplock LJJ: subconscious copying can constitute an infringement of copyright, for, in order to constitute
infringement, intention or knowledge is not necessary (see p 21, letter g, and p 28, letters c and e, post; and cf p 26, letter i, to p
27, letter a, post); and (per Willmer LJ), if subconscious copying is to be established, there must be proof or strong inference of
de facto familiarity with the work alleged to be copied (see p 21, letter e, post).
16
Dictum of Luxmoore J, in G Ricordi & Co (London) Ltd v Clayton and Waller, Ltd (Macgillivrays Copyright Cases (1928
1935) at p 162); dictum of Judge Learned Hand in Fred Fisher, Inc v Dillingham, ((1924), 298 Fed Rep at p 145), and Edwards
and Deutsch Lithographing Co v Boorman ((1926), 15 Fed Rep (2nd series) 35) considered.
Per Upjohn LJ: where there is evidence from the music itself that there is real practical possibility of independent
composition by the defendant, it requires quite strong evidence to support the view that there has been unconscious copying (see
p 25, letter i, post).
Appeal dismissed.

Notes
As to the meaning of reproduction infringing copyright, see 8 Halsburys Laws (3rd Edn) 426, 427, para 776; and for cases on the
subject, see 13 Digest (Repl) 104121, 451616.
For the Copyright Act, 1956, s 2(5), see 36 Halsburys Statutes (2nd Edn) 73.

Cases referred to in judgments


Austin v Columbia Gramophone Co Ltd (1923), Macgillivrays Copyright Cases, 19171923, p 398, 13 Digest (Repl) 69, 146.
DAlamaine v Boosey (1835), 1 Y & C 288, 4 LJEx 21, 13 Digest (Repl) 115, 564.
Edwards and Deutsch Lithographing Co v Boorman and Others (1926), 15 Fed Rep (2nd series) 35.
Fisher (Fred), Inc v Dillingham and Others (1924), 298 Fed Rep 145.
Hanfstaengl v Empire Palace [1894] 2 Ch 1, 63 LJCh 417, 70 LT 459, subsequent proceedings, (1895), 11 TLR 368, 13 Digest
(Repl) 111, 525.
Hill (William) (Football) Ltd v Ladbroke (Football) Ltd 19th December, 1962, CA, not reported.
Purejoy Engineering Co Ltd v Sykes Boxall & Co Ltd (1955), 72 RPC 89, 13 Digest (Repl) 112, 537.
Ricordi (G) & Co (London) Ltd v Clayton and Waller Ltd (1930), Macgillivrays Copyright Cases 19281935, p 154.

Appeal
The plaintiffs appealed against an order of Wilberforce J, made on 27 July 1962, dismissing the plaintiffs action for alleged
infringement of their copyright in a song called In a Little Spanish Town. The plaintiffs sought an injunction restraining the
defendants from reproducing in any material form a musical work entitled Why or any work which reproduced or was an
adaptation of any substantial part of the plaintiffs song, and an inquiry as to damages for infringement of copyright and for
conversion, and alternatively an account of profits; and an order for delivery up of infringing material. The grounds of appeal
were: (i) that the similarity between the work In a Little Spanish Town and Why was such that the judge ought to have found
as a fact that the latter was derived from the former and was therefore a reproduction of a substantial part of the former; (ii) that
the similarity between the two works was such that the judge ought to have found as a fact that the composer of Why in
composing it must have used even though unconsciously his memory of In a Little Spanish Town; (iii) that, if a work in fact
contained a substantial part of an earlier work, and it was found as a fact that this was due to the composers memory of the
earlier work, the latter work constituted an infringement of the copyright in the earlier work notwithstanding that the composer of
the later work was unaware that, when composing it, he was relying on his memory of the earlier work; (iv) that, where it was
established that a work in fact contained a substantial part of an earlier work, the onus was on the defendant to show that the
similarity was due to some cause other than the conscious or unconscious recollection of the composer; (v) that the judge, having
been prepared to find as a fact that the composer of 17Why had at some time in some circumstances heard In a Little Spanish
Town, having regard to the similarities of the two works ought to have gone on to find as a fact that Why reproduced a
substantial part of the earlier work; and (vi) that the judge gave insufficient weight to the defendants admission that the musical
work in In a Little Spanish Town had been extensively exploited in the United States of America and elsewhere ever since 1926
by the publication of sheet music, by the distribution of gramophone records and by broadcasting.

John G Foster QC and F E Skone James for the plaintiffs.


J L Arnold QC and John E Williams for the defendants.

25 February 1963. The following judgments were delivered.

WILLMER LJ. This is an appeal against a judgment of Wilberforce J given on 27 July 1962, whereby he dismissed an action
brought by the plaintiffs for infringement of their copyright in a song called In a Little Spanish Town (to which I will refer as
Spanish Town). This was composed in 1926, and (as has been admitted by the defendants) was extensively exploited in the
United States of America and elsewhere by the publication of sheet music, by the distribution of gramophone records and by
broadcasting. Unlike many popular songs, Spanish Town appears to have retained its popularity over the years. Records
published in this country (some of them quite recently) were played to us during the course of the hearing, and I was readily able
to recognise the tune as a familiar one which I had heard on frequent previous occasions.
The defendants are the publishers of another song called Why, which was composed in 1959 by Mr Peter de Angelis.
Spanish Town is written in 3/4 time, and Why in 4/4 time. There are other differences between the two works, which were
the subject of a good deal of evidence by musical experts on both sides; but, when the two songs were played to us, it was
immediately apparent, to me at any rate, that the effect on the ear was one of noticeable similarity. This is a matter which is not
without importance, for, as was pointed out by Astbury J, in Austin v Columbia Gramophone Co Ltd ((1923), Macgillivrays
Copyright Cases, 19171923, at pp 415 and 409): Infringement of copyright in music is not a question of note for note
comparison but falls to be determined by the ear as well as by the eye.
The judge included in his judgment a detailed analysis of the musical structure of the two songs. I accept this as correct, and
it is, I think, unnecessary for me to repeat it except in summary form. In each case the essential feature of the song is contained
in the first eight bars, which constitute what has been described as a musical sentence, and in which the main theme is stated. It
is common ground that in the case of Spanish Town these first eight bars constitute a substantial part of the work within the
meaning of s 49 of the Copyright Act, 1956. In Spanish Town a subsidiary and contrasting theme is then introduced, after
which there is a return to the original theme, which is then re-stated with variations. By way of contrast, Why is described as a
thematic song; there is no subsidiary or contrasting theme, but, practically speaking, the whole song is devoted to the
development of the original theme.
Having given his analysis of the musical structure of each song, the judge stated his conclusions as to the points of
similariety or difference under nine headings, which I will summarise. (1) The structure of the two songs is different in the way
that I have already described. (2) The first eight bars being the essential part of Spanish Town, if the theme therein stated has
been borrowed in Why, the fact that it is developed by staying with it, rather than by way of contrast and return, would not
make a significant difference, but might even accentuate the likeness. (3) The theme of Spanish Town is built up of musical
common-places or cliches. The six notes of the first bar are a common-place series, found in other previous musical works, and
the manner in which this phrase is developed during the rest of the first eight bars is by way of some of the commonest tricks of
composition. The result, however, is a combination which gives character and charm to Spanish Town. (4) On a note for note
comparison between the 18 two songs there is a noticeable correspondence, though at no point do more than five consecutive
notes correspond. But the fact that in Why the descent of one-sixth from the first to the second note is immediately followed
by a leap back of a sixth to the original note instead of an arpeggio, as in Spanish Town, constitutes a not insignificant
difference. (5) The harmonic structure of the first eight bars is the same in both cases; but this is completely common-place and
insignificant. (6) Correspondence of notes is not of itself enough to create similarity; time and rhythm are equally important. (7)
In the present case there is a difference in time, but this is not a decisive factor, for in the case of Spanish Town, as with other
popular songs, a change of time from 3/4 to 4/4 does not destroy its substantial identity, or cause loss of recognition. (8) There is,
however, a significant difference in rhythm between the two songs; this is exemplified in the first bar, which in the case of
Spanish Town consists of an even sequence of six quavers, whereas in the case of Why the first note is held for half a bar,
and is followed by a descent to an accented note. (9) There is a significant difference between the two songs in the third and
seventh bars. In Spanish Town these consist of a single held note; in Why these bars each start with the same note as in
Spanish Town, but after an interval of a crotchet there is a drop of a fifth and a fourth respectively to a different held note.
Having stated these various points of similarity and difference (which I wholly accept) the judge expressed the view that, in
relation to the aural appeal of the sentence as a whole, there is an undoubted degree of similarity between the two songs, the only
question being what adjective to put before the word degree. He expressed his conclusion as follows:

On the whole, I think Mr. Palmers word definite or considerable is the right weight to put on the degree of
similarity; it is such that an ordinary reasonably experienced listener might think that perhaps one had come from the
other.

With that conclusion, I entirely agree.


If the matter stopped there, I do not think it could be doubted that there was material on which to base the inference that the
composer of Why deliberately copied from Spanish Town. Were that the right inference, I am satisfied that the degree of
similarity would be sufficient to constitute an infringement of the plaintiffs copyright. But the composer of Why was called as
a witness, and not only denied copying, but denied that he had ever seen the music of Spanish Town, or even consciously heard
it. He was a man of thirty-three years of age, and had lived most of his life in the United States. He stated that he had been
composing music since he was eleven, and had played various instruments in dance bands. In cross-examination he admitted that
at a younger age he might have heard Spanish Town, because he had heard a lot of music, but he adhered to his statement that
he had never consciously studied it, and said that he did not recall ever playing it. The judge accepted his evidence, and I do not
think that we in this court could properly interfere with that finding even if we were invited to do so, which we were not. But the
plaintiffs say that that is by no means the end of the case, for Mr de Angelis could well have copied from Spanish Town
subconsciously. The song having been extensively exploited in the United States, the overwhelming probability (it is said) is that
he must have heard it; and the degree of similarity between Spanish Town and Why is such that an inference of, at any rate,
subconscious copying should be drawn. That, it is contended, would be enough to constitute an infringement of the plaintiffs
copyright. The judge, however, decided that there was not sufficient material to justify the inference that Mr de Angelis copied
the plaintiffs work, even subconsciously; and he accordingly dismissed the action. It is to this point that the present appeal has
been mainly directed.
In approaching the suggestion of subconscious copying on the part of Mr de Angelis, it is to be observed that the Copyright
Act, 1956, nowhere uses the word copying. Section 2(5) provides:
19

The acts restricted by the copyright in a literary, dramatic or musical work are(a) reproducing the work in any
material form; (b) publishing the work; (c) performing the work in public; (d) broadcasting the work; (e) causing the work
to be transmitted to subscribers to a diffusion service; (f) making any adaptation of the work; (g) doing, in relation to an
adaptation of the work, any of the acts specified in relation to the work in paras. (a) to (e) of this subsection.

By sub-s (6), para (b), adaptation in relation to a musical work is defined as meaning an arrangement or transcription of the
work. By s 48(1) reproduction is defined as including reproduction in the form of a record. There is no further relevant
definition of the word, and it has been left to judicial decision to introduce the notion of copying.
Counsel, in presenting his argument on behalf of the defendants, drew attention to the fact that in relation to musical
copyright, under s 2 of the Act of 1956, there are only three forbidden processes, viz, reproduction, arrangement, and
transcription. Arrangement and transcription, he submitted, can be only the result of a conscious and deliberate process; a man
cannot arrange or transcribe without knowing that he is doing so. The judges acceptance of the evidence of Mr de Angelis,
therefore, precludes the possibility of finding any infringement of the plaintiffs copyright by arrangement or transcription. This
submission must, I think, be accepted.
Counsel for the defendants conceded that reproduction could possibly be the result of a subconscious process. But he went
on to submit that reproduction within the section could mean nothing short of identity. Reproduction, under s 49, may be of a
substantial part; but there is no suggestion in the Act of 1956 of any such thing as a substantial reproduction. In the present
case it cannot be said that there is anything approaching identity between the plaintiffs work and that of Mr de Angelis.
Consequently, counsel submitted, there could be no infringement of the plaintiffs copyright, whether conscious or unconscious,
by way of reproduction.
I find myself quite unable to accept this submission, for I can find no warrant for the suggestion that reproduction, within the
meaning of the section, occurs only when identity is achieved. This not only offends against common sense, but, I think, is
contrary to authority. In Austin v Columbia Gramophone Co Ltd ((1923), Macgillivrays Copyright Cases, 19171923, at p 398)
the headnote reads:

Infringement of copyright in music is not a question of note for note comparison, but of whether the substance of the
original copyright work is taken or not.

In that case, Astbury J, quoted from the earlier case of DAlamaine v Boosey ((1835), 1 Y & C at p 302) where it was laid down
that it must depend on whether the air taken is substantially the same with the original. I accept that as a correct statement of
the principle.
On the other side, counsel for the plaintiffs submitted in the first place that Mr de Angeliss denial of copying was wholly
irrelevant. For where, as was said to be the case here, a sufficient degree of similarity is shown, and it is further proved that the
composer of the second work had access to the earlier work in the sense that he must probably have heard it, an irrebuttable
presumption arises that the former has been copied from the latter. No authority was cited in support of this proposition, which,
if well-founded, would eliminate the necessity for any further evidence once similarity coupled with access had been proved. In
my judgment, the proposition contended for is quite untenable; the most that can be said, it seems to me, is that proof of
similarity, coupled with access, raises a prima facie case for the defendant to answer.
Counsel for the plaintiffs contended in the alternative that the degree of similarity found by the judge in the present case was
such as to compel an inference 20 of copying which, even if subconscious, was sufficient to give the plaintiffs a cause of action
for infringement. I confess that I have found the notion of subconscious copying one of some difficulty, for at first sight it would
seem to amount to a contradiction in terms, the word copying in its ordinary usage connoting what is essentially a conscious
process. The text-books on copyright make no reference to the subject, and English authority in relation to it is confined to a
single dictum of Luxmoore J, in G Ricordi & Co (London) Ltd v Clayton and Waller Ltd ((1930), Macgillivrays Copyright
Cases, 19281935, at p 154). Our attention, however, was called to a number of cases in the United States in which the subject
has been discussed, and in some of which a decision in favour of the plaintiff has been based on a finding of subconscious
copying. It appears to me that the question must be considered in two stages, viz, (i) whether subconscious copying is a
psychological possibility; and (ii), if so, whether in a given case it is capable of amounting to an infringement of the plaintiffs
copyright.
As to the first of these questions, it was suggested by counsel for the defendants that medical evidence should always be
required before a finding of subconscious copying could be justified. I cannot think that this is necessary; for the psychological
possibility of subconscious copying was clearly recognised by Luxmoore J and in the various American decisions, which must be
regarded as of high persuasive authority. What Luxmoore J said, in relation to the defendants before him in the Ricordi case
((1930), Macgillivrays Copyright Cases, 19281935, at p 162), was:

If there has been any infringement it must have been subconsciously, because the persons responsible knew the air
complained of so well that they have taken it because they knew it.

Similarly, in two American cases in which the plaintiff succeeded on the ground of subconscious copying, viz, Fred Fisher, Inc v
Dillingham and Edwards & Deutsch Lithographing Co v Boorman, the decision was based on the finding of a high degree of
familiarity with the plaintiffs work. From this emerges the conclusion, which seems to me to be consonant with good sense, that,
if subconscious copying is to be found, there must be proof (or at least a strong inference) of de facto familiarity with the work
alleged to be copied. In the present case, on the findings of the judge, this element is conspicuously lacking.
On the second question, viz, whether any subconscious copying proved could amount to an infringement of the plaintiffs
copyright, it seems to me that all that can be said is that at least the dictum of Luxmoore J envisages the possibility. On this point
I do not think that much help is to be derived from the American decisions which have been cited, since the American statute
under which they were decided is markedly different in its terms. No evidence of American law was adduced, and in its absence
it is not for us to construe the American statute. However (as was pointed out by junior counsel for the plaintiffs) it may be
observed that, in order to establish an infringement of copyright, it is not necessary to prove anything in the nature of mens rea.
The printer, for instance, may be held guilty of infringement though he has no conscious intent.
The conclusion at which I arrive on this part of the case is that subconscious copying is a possibility which, if it occurs, may
amount to an infringement of copyright. But in order to establish liability on this ground, it must be shown that the composer of
the offending work was in fact familiar with the work alleged to have been copied. This view, I think, is not inconsistent with the
submissions put forward by Mr Skone James. In the course of an argument which I found convincing, he submitted that, in
considering whether there has been reproduction, so as to constitute an infringement within the Copyright Act, 1956, it is wholly
irrelevant to inquire whether any copying has been conscious or subconscious. It is for this reason, he modestly suggested, that
the text-books are silent on the subject of subconscious copying. Mr Skone James presented his argument in four propositions
which, if I understood him correctly, may be summarised as 21 follows: (i) in order to constitute reproduction within the
meaning of the Act, there must be (a) a sufficient degree of objective similarity between the two works; and (b) some causal
connexion between the plaintiffs and the defendants work. (ii) it is quite irrelevant to inquire whether the defendant was or was
not consciously aware of such causal connexion. (iii) where there is a substantial degree of objective similarity, this of itself will
afford prima facie evidence to show that there is a causal connexion between the plaintiffs and the defendants work; at least, it
is a circumstance from which the inference may be drawn. (iv) the fact that the defendant denies that he consciously copied
affords some evidence to rebut the inference of causal connexion arising from the objective similarity, but is in no way
conclusive.
If this is the right approach (as I think it is), it becomes a simple question of fact to decide whether the degree of objective
similarity proved is sufficient, in all the circumstances of the particular case, to warrant the inference that there is a causal
connexion between the plaintiffs and the defendants work. This is the way in which, as it seems to me, the judge in the present
case approached the question which he had to decide. He directed himself as follows:

The final question to be resolved is whether the plaintiffs work has been copied or reproduced, and it seems to me that
the answer can only be reached by a judgment of fact on a number of composite elements: The degree of familiarity (if
proved at all, or properly inferred) with the plaintiffs work, the character of the work, particularly its qualities of
impressing the mind and memory, the objective similarity of the defendants work, the inherent probability that such
similarity as is found could be due to coincidence, the existence of other influences on the defendant composer, and not
least the quality of the defendant composers own evidence on the presence or otherwise in his mind of the plaintiffs
work.

In my judgment that was a proper direction, against which no criticism can fairly be brought.
Having so stated the question to be determined, and the matters to be considered, the judge stated his conclusion as follows:

In this case, after taking account of the respective character and similarities of the two works as previously discussed,
and relating this to the fact that there is no direct evidence that Mr. Peter de Angelis ever knew the work of Spanish Town
before he composed Why, I have come to the conclusion that I have not sufficient factual material from which to draw an
inference that he had sufficient knowledge or memory of Spanish Town at the date of composition to justify me in
finding, against his express denial, that in composing Why he copied, without knowing that he did so, Spanish Town or
a part of Spanish Town. Putting it in another way, it does not seem to me that the degree of similarity shown, coupled
with the fact, which I think is as far as it is possible to go by inference, that at some time and in some circumstances Mr. de
Angelis must have heard Spanish Town, is enough to make good the plaintiffs case.

The question, being one of fact, is eminently one for the determination of the trial judge, as I think is recognised in the American
cases which were cited to us. It is to be remembered that the judge not only had the advantage, denied to us, of himself seeing
and hearing the witnesses at first hand; he also had the advantage, which strikes me as being of great importance, of hearing how
the musical experts who were called as witnesses illustrated the technical evidence which they gave by demonstrations, both
vocally and on the piano. Bearing this in mind, and having regard to the judges acceptance of the evidence given by Mr de
Angelis, in my judgment it is impossible for us in this court to say that he reached a wrong conclusion on what was eminently a
question of fact for him.
I should perhaps mention one further consideration which appears to me to be of possible significance, and which was not
dealt with specifically by the judge. I 22 have already referred to the fact that the six quavers which form the opening bar of
Spanish Town are, as the judge observed, a common-place series to be found in other previous musical compositions. Our
attention was drawn, for instance, to an Austrian dance tune composed in the early nineteenth century by Von Lichnowsky, the
opening bar of which is identical with that of Spanish Town. The same sequence of notes is also to be found in a song entitled
Let Us Sing Merrily, although in this case there is a difference of tempo. In these circumstances, the fact that Why begins
with an opening bar containing a similar, though not identical, phrase is of no special significance. By itself it would not be
sufficient to warrant the inference that, if the phrase was copied, it was copied from the plaintiffs work rather than some other
composition. What is significant is the fact that, both in Spanish Town and in Why, the opening phrase enunciated in the
first bar is developed over the remainder of the first eight bars by the use of the same devices or tricks of composition, viz,
repetition followed by a pause, followed again by further repetition with a slight variation. It is this circumstance which produces
the degree of similarity between the two compositions. If it could be said that this method of development was so distinctive and
idiosyncratic as to preclude the possibility that its adoption by the two composers was the result of coincidence, this would be a
very strong argument in the plaintiffs favour. But, as pointed out by the judge, the devices used by the two composers for
developing the phrase stated in the first bar are among the commonest tricks of composition and, I would add, exactly the sort to
be expected from the composer of a popular song. I do not think, therefore, that in the circumstances of this case the fact that Mr
de Angelis developed the opening phrase stated in the first bar by way of the same devices as were employed by the composer of
Spanish Town can be taken as in any sense proof of copying. There is at least an equal probability that his choice of these
devices was the result of coincidence.
In my judgment, no sufficient reason has been shown for interfering with the judges decision, and I would accordingly
dismiss the appeal.

UPJOHN LJ. I agree with the judgment which has just been delivered. When counsel for the plaintiffs opened this appeal, he
invited us in the name of international comity to say that a right of property (ie, copyright) which is the subject of international
convention must be protected in a most special and unique way. We were invited to say that, if similarity in the alleged infringing
work to the original work was established as a fact, and if it was further established that the alleged infringer had had some access
to the original work, then, although a denial of conscious plagiarism was accepted, we were bound, as an irrebuttable
presumption of law, to say that the alleged infringer must have unconsciously copied the original work. The doctrine was said to
be necessary to protect the author of the original work, for otherwise (so it was argued) any infringer could escape the
consequences of plagiarism by denying that he had done so. Alternatively, it was said that, if some undefined higher degree of
similarity between the two works could be provedsomething higher than is necessary to prove similarity in factthen that
would be sufficient to establish a similarity from which we were bound to infer unconscious copying.
Apart from the appeal to international comity, no authority and no text-book has been cited in support of this remarkable
doctrine. Copyright is statutory, and depends on s 2 of the Copyright Act, 1956. No hint of this doctrine appears there. As to
international comity, while it is true that in the United States of America a number of authorities (to which I shall have occasion
to refer later in some detail) accept the doctrine that subconscious or unconscious copying may be inferred in a proper case and
operate as a breach of copyright, not one of those authorities gives any support to this alleged and startling doctrine. The
authorities in question in each case treated the question of unconscious copying as purely a question of inference of fact which
might be drawn in the circumstances of a 23 particular case, and not as a presumption of law. We were not referred to the laws of
any other convention country, and the relevant paragraph (No 4) of the Brussels Convention itself lends no support to the
doctrine. I therefore reject this submission.
The truth is that the plaintiff in a copyright action must show that a substantial part of the original work has been reproduced
(see s 2(5) and s 49 of the Copyright Act, 1956); and, although not expressed in the Act, it is common ground that such a
reproduction, in the words of junior counsel for the plaintiffs, must be causally connected with the work of the original author. If
it is an independent work, then, though identical in every way, there is no infringement. If a true infringer wrongly persuades the
court that it is his own unaided work, the plaintiff fails, as do other plaintiffs when fraudulent defendants unhappily succeed (as,
no doubt, they sometimes do) in persuading the court that they have not been fraudulent. The question in this case, therefore, is
whether there has been a breach of s 2(5); that subsection has been read by my Lord, and I will not read it again.
This is really a question of fact and nothing else, which depends on the circumstances of each case. But it is a question of
fact which must be taken in two stages. The first stage is objective and the second stage subjective. The first question is whether
in fact the alleged infringing workwhich for the sake of brevity I will inaccurately call the defendants work, for though the
composer was a witness, he was not a defendantis similar to the work of the original author, which again for the sake of brevity
I will (with equal inaccuracy) call the plaintiffs work. Is it then proper to draw the inference that the defendants work may have
been copied from the plaintiffs work? This is purely an objective question of fact, and depends in large degree on the aural
perception of the judge, but also on the expert evidence tendered to him; but it is essentially a jury question. A defendant might
in theory go into the witness-box and say that he had deliberately made use of the plaintiffs work, but that it is not an
infringement, either because he did not make use of a substantial part of the plaintiffs work, or because though the plaintiffs
work has been utilised, he has been able so to alter it that it cannot properly be described as a reproduction. The onus is on the
plaintiff to prove the contrary as a matter of purely objective fact, and if he cannot do so then the morally dishonest defendant
will escape the consequence of the allegation of infringement. No such question arises in this case. At this stage similarity has
been found by the judge, and that is not challenged before us. For myself, I think that perhaps I would have used rather stronger
adjectives than definite or considerable similarity, which were the words used by the judge; the adjective close would be
more appropriate, but nothing, I think, turns on that.
The next stage is the subjective stage, and is equally a question of fact, though of course the degree of similarity is most
important in reaching this subjective conclusion. The question at this stage, put bluntly, is: Has the defendant copied the
plaintiffs work, or is it an independent work of his own? Mr Skone James, in an attractive argument, agrees that the plaintiff in
order to succeed must prove a causal connexion between his work and the defendants work; but he submits that, providing that,
on a proper inference from the known facts it is right to assume that the alleged infringing work was derived from the plaintiffs
work, it matters not whether it was done consciously or unconsciously. There is, he submits, no difference in principle between a
conscious act of piracy and an unconscious act of piracy; all that must be established is a causal connexion.
While conscious acts of piracy may be established in the witness-box, unconscious acts of piracy must clearly be a matter of
inference from surrounding circumstances. The alleged infringing work may be an identical reproduction of the original work
with all its idiosyncracies and all the same mistakes. Theoretically and mathematically, that may be a complete coincidence, and
both works may be the product of entirely independent brains; but the judge 24 has to judge of these matters on the balance of
probabilities; and such an identical reproduction may lead him to reject the evidence of the defendant, who otherwise appears to
be an honest witness and to make a finding of conscious piracy. Much less than complete identity may properly lead the judge,
on the balance of probabilities, to reject the evidence of an apparently honest witness on this question. This is a question of pure
fact in every case. It does not arise in this case for the judge accepted the authors evidence that he did not consciously derive the
composition of Why from In a Little Spanish Town, and that has not been challenged before us.
At this stage, therefore, the question is whether, on the facts of the case, it is proper to infer that Mr de Angelis has derived
Why unconsciously from the plaintiffs work, which he must have heard at some earlier time. This again is purely a question
of the proper inference of fact to be drawn from all the relevant and admissible known facts. There may be cases where, if the
circumstances do not justify the conclusion that the defendant, in denying conscious plagiarism, is not telling the truth, yet justify
the conclusion that he must have heard the plaintiffs tune, and subconsciously reproduced it.
I do not pause to recapitulate the facts of this case in any detail, for they have been set out in such meticulous detail in the
judges judgment, and also by my Lord. I draw the conclusion that, though, as I have already stated, the resemblance is a close
one, that resemblance in the circumstances of this case is little evidence of unconscious copying, The judge said this.:

Thirdly, the theme of Spanish Town is made up of common-place elements or, as some witnesses have called them,
clichs. The first six notes are a common-place enough series; they are found in an Austrian country dance and in a song,
Let Us Sing Merrily. The device of repetition, of resting for two bars on a long note and of repetition in sequence, are
the commonest tricks of composition. But many writers of great music have used cliches to produce masterpieces; indeed,
some of them have found in the common-place character of their basic phrase their stimulus. Professor Newman gave
some interesting examples from the music of Mozart, and most writers of popular songs use, and can use, nothing else. No
example was given of precisely this combination having been used in other compositions, though it was apparent that the
musical dictionaries and the experience of the witnesses had been thoroughly combed.

Having heard the arguments of counsel, accompanied by very helpful demonstrations on the piano, I reach the conclusion of
fact that it is not a mere legal or mathematical possibility, but a real live practical possibility that the defendants composition of
Why was an independent composition, apart altogether from Mr de Angeliss denial of conscious plagiarism, which was
accepted. This practical possibility again does not conclude the matter, for the defendants composition may nevertheless be the
result of unconscious memory. But first it is necessary to establish the probability that the defendant has heard the plaintiffs
composition.
The judge had to deal with a difficult situation as to whether Mr de Angelis had heard, or even played, as a youth in a dance
band, the plaintiffs composition. I think that it is possible that, although in perfect good faith he stated the contrary, Mr de
Angelis did hear the music, and possibly played it in his early youth. Each case must depend on its own facts, and it is not
possible to lay down any criteria. But it does seem to me that where, for the reasons which I have given, there is evidence from
the music itself that there is a real practical possibility of independent composition by the defendant, it requires quite strong
evidence to support the view that there may have been unconscious copying. To my mind, the possibility that the defendant had
heard it, or even played it in his early youth, is a quite insufficient ground on which it would be proper to draw the inference of
unconscious copying. It may be that in the future medical evidence will be available to guide us on this point, but in the absence
25 of acceptable and probative medical evidence I think that it requires quite strong evidence, in a case such as thiswhere, as I
have already pointed out, independent composition is a real practical possibilityto establish, as a matter of probability, that Mr
de Angeliss subconscious ego guided his hand.
The cases in the United States to which we have been referred offer some instructive comparison on their facts, although I
do not lose sight of the fact that cases are only authorities for legal propositions; but nevertheless the cases cited are helpful. In
the first case, Fred Fisher, Inc v Dillingham, that great judge, Judge Learned Hand, in giving the famous composer, Mr Jerome
Kern, the benefit of the doubt, said this ((1924) 298 Fed Rep at p 147):
On the whole, my belief is that, in composing the accompaniment to the refrain of Kalua, Mr. Kern must have
followed, probably unconsciously, what he had certainly often heard only a short time before.

That is in marked contrast to the facts of this case. Then in a rather different case, Edwards and Deutsch Lithographing Co v
Boorman, the plaintiffs prepared, printed, published and distributed certain discount tables, the copy-right work. The defendants
published very similar tables. But there it was established that the defendants had sold and handled the plaintiffs publications for
years, and on that the inference was drawn of unconscious copying. Again the facts of that case are very different from the one
before us.
Wilberforce J, put the relevant points to himself quite accurately, and my Lord has read that part of his judgment; and his
summary, which I will venture to repeat, was this:

In this case, after taking account of the respective character and similarities of the two works as previously discussed
and relating this to the fact that there is no direct evidence that Mr. Peter de Angelis ever knew the work of Spanish Town
before he composed Why, I have come to the conclusion that I have not sufficient factual material from which to draw an
inference that he had sufficient knowledge or memory of Spanish Town at the date of composition to justify me in
finding, against his express denial, that in composing Why he copied, without knowing that he did so, Spanish Town, or
a part of Spanish Town.

I entirely agree with that conclusion of fact of the judge in this case. I am not prepared on the evidence in this case to draw any
inference of unconscious copying.
That makes it unnecessary to decide the really interesting question whether if unconscious copying had been properly
inferred on the facts that would have amounted to an infringement and whether junior counsel for the plaintiff is right when he
says there is no difference in law between conscious and unconscious copying. It seems to me that that is an interesting question
on which I would venture to express no opinion, for, as I have said, it does not arise. In support of his argument that the infringer
may be entirely ignorant of knowledge of plagiarism, counsel has pointed out that, normally, the printer and publisher will also be
guilty of infringement, though they have no reason even to suspect that any plagiarism can be suggested. However, this does not
meet my difficulty. Knowledge or suspicion of plagiarism does not necessarily have to be shown against every defendant, but the
plaintiff always has to prove that the alleged infringement is not the independent work of the alleged infringing author or
composer, but is causally connected with the plaintiffs work. The real question is this: Can it be said to be an act of
reproduction, for the purposes of s 2(5) of the Copyright Act, 1956, if the alleged infringing work is not the conscious act of the
infringer? It has been argued that Luxmoore J, in Ricordis case ((1930), Macgillivrays Copyright Cases, 19281935, at p 162)
expressed the view that subconscious copying could be an infringement of 26 copyright; but I do not think that he intended to
express any view on the law at all. For my part, I think that this question, therefore, remains entirely open. I agree that this
appeal must be dismissed.

DIPLOCK LJ. This appeal seems to me to turn entirely on a question of fact: Was the judge entitled, notwithstanding the
similarities between the melodies of the plaintiffs song In a Little Spanish Town and the defendants song Why, to refuse to
infer that the composer of the latter work copied it from the former work? It is conceded on the one hand (as is obvious to the
ear) that the two works show considerable similarities, and on the other hand that the composer of Why did not intentionally
copy it from In a Little Spanish Town; but it was found by the judge that the composer of Why must at some time and in
some circumstances have heard In a Little Spanish Town; and it is contended by the plaintiffs that the only proper inference of
fact is that he must have stored it in his memory and reproduced it without being aware that he was performing an exercise of
recollection and not an act of independent creation.
To this assumed mental feat there has been applied the conveniently ambiguous term subconscious copying; and we have
heard much argument as to whether, if it is established, it constitutes an infringement of the copyright in the work which has been
unconsciously copied. I think that the law is perfectly clear, and that such difficulties as there are in this appeal are solely due to
the absence of any factual information about the mental process involved in subconscious copying. We know not whether it is
rare or common, general or idiosyncratic, nor indeed whether it is possible to remember, not a mere isolated phrase, but a
substantial part of the remembered work without remembering that one is remembering.
First, as to the law; and for this purpose I will assume that it is established that the composer of Why did in fact use his
recollection of a substantial part of the melody of In a Little Spanish Town as the model for his own composition, although
he was unaware that he was doing so, and genuinely thought that Why was his own independent creation. The verb to copy
is not used at all in the Copyright Act, 1956, nor was it in the Copyright Act, 1911. Nevertheless, it is well established that to
constitute infringement of copyright in any literary, dramatic or musical work there must be present two elements: First, there
must be sufficient objective similarity between the infringing work and the copyright work, or a substantial part thereof, for the
former to be properly described, not necessarily as identical with, but as a reproduction or adaptation of the latter; secondly, the
copyright work must be the source from which the infringing work is derived. The necessity for the second element was
expressly laid down by the Court of Appeal in Purejoy Engineering Co Ltd v Sykes Boxall & Co Ltd ((1955), 72 RPC at p 99),
and is, indeed, implicit in all the compilation cases, including the recent case in this court of William Hill (Football) Ltd v
Ladbroke (Football) Ltd, where tables of betting odds were unanimously held not to infringe the copyright in substantially
identical tables because the authors of the later tables, although very familiar with the earlier tables, had in fact worked out the
odds for themselves. But, while the copyright work must be the source from which the infringing work is derived, it need not be
the direct source; see Hanfstaengl v Empire Palace. Mr Skone James, I think, put it with his usual accuracy when he said there
must be a causal connexion between the copyright work and the infringing work. To borrow an expression once fashionable in
the law of negligence, the copyright work must be shown to be a causa sine qua non of the infringing work.
The necessity for a causal connexion between the copyright work and the infringing work, although well established under
the Copyright Act, 1911, either 27 as being implicit in the legal concept of copyright, or in the word reproduce, is, I think,
more easily deduced wording of the current Copyright Act, 1956. Section 1(1) defines copyright in relation to a work as the
exclusive right to do and to authorise other persons to do certain acts in relation to that work; and sub-s (2) defines
infringement as the doing of any of those acts by a person who is not the owner of the copyright or his licensee. The acts,
which are defined in s 2, and include reproducing the work in any material form, if they are to constitute infringement must
thus be done in relation to the work, an expression which connotes a causal connexion between the copyright work and the act
relied on as an infringement. If the existence of the copyright work has no causal connexion with the production of the alleged
infringing work, even though the latter be identical with the former, there is no infringement of copyright.
In my view, however, it is equally clear law that neither intention to infringe, nor knowledge that he is infringing on the part
of the defendant, is a necessary ingredient in the cause of action for infringement of copyright. Once the two elements of
sufficient objective similarity and causal connexion are established, it is no defence that the defendant was unaware (and could
not have been aware) that what he was doing infringed the copyright in the plaintiffs work. This is expressly recognised by s 17
and s 18 of the Copyright Act, 1956, which restrict the remedies available against an innocent infringer, but recognise his liability,
Thus under s 18, which gives to the copyright owner remedies in conversion and detinue in respect of infringing copies of his
work, a defendant who believed and had reasonable grounds for believing that they were not infringing copies is relieved of
any liability in damages, but not of his liability to deliver up any infringing copies in his possession.
Unconscious copying in the sense in which it has been used in the argument postulates, first, such objective similarity
between the copyright work and the alleged infringing work that the latter may properly be said to reproduce the former;
secondly, that there is a causal connexion between the copyright work and the alleged infringing work; thirdly, that the composer
of the alleged infringing work believed (and may indeed have had reasonable grounds for believing) that there was no such causal
connexion. The first two, if established, in my view constitute breach of copyright; the third is irrelevant on liability, although it
may be relevant on remedy.
The real difficulty in this case is one not of law but of fact. It involves an inquiry into the working of the human mind. It
may well be that this is a matter on which expert evidence is admissible; but cases in English courts are normally conducted on
the tacit assumption that, where no question of disease of the mind is involved, the ordinary man, whether sitting in the jury-box
or on the bench, is capable of determining (where it is relevant) what went on in the defendants mind. The present case was so
conducted before Wilberforce J. No expert evidence was called as to how the human memory or musical creative faculties work;
no investigation was made into the mental idiosyncracies of the composer of Why or his methods of composition. The judge
was left to draw the inference of subconscious copying from the evidence (i) of the similarities between the melodies of In a
Little Spanish Town and Why as explained by the conflicting evidence of expert musicians; (ii) of the likelihood of the
composer of Why having at some time heard In a Little Spanish Town; and (iii) of his denial that he had consciously copied
In a Little Spanish Town.
On this state of the evidence, there were three possible explanations of the similarities: conscious copying, unconscious
copying, coincidence. The first the judge rejected. He accepted the denial of the composer of Why. This is a finding of
primary fact, and it depends ultimately on credibility. The plaintiffs do not seek to disturb it. This reduces the possible
explanations to two: unconscious copying, or coincidence. The judge did not reject the possibility that unconscious copying of
musical works can occur. He proceeded to consider, 28in the light of the conflicting expert evidence, which was the more
probable explanation of the similarities, unconscious copying, or coincidence. The relevant similarities were to be found in the
first eight bars of the melody of In a Little Spanish Town which, it is common ground, do constitute a substantial part of that
musical work. They are described clearly (and it is conceded accurately) in Wilberforce Js judgment.
The rival contentions, supported by expert evidence, may be summarised thus: The plaintiffs, conceding that the first bar by
itself was a musical clich in which there was no copyright, contended that the similarities in the use made of the clich in eight
successive bars in each of the two works were too great to be explained by coincidence. The defendants contended that, once a
composer of popular songs had decided to use, as a basis of the theme of a popular song, the musical clich contained in the first
bar, the use which was in fact made of it in both Why and In a Little Spanish Town in the succeeding bars was a device by no
means uncommon in musical composition, and the similarities were readily explicable by coincidence. The judge was not
satisfied that the similarities were due to unconscious copying. This, no doubt, was an inference of fact, but one which depends,
in part at least, on the degree of conviction which the evidence of the respective experts carried, and thus one with which an
appellate court should be slow to interfere. No attempt has been made to demonstrate that he has overlooked or misunderstood
any of the evidence.
How, then, is the case for the plaintiffs put? The procedure of the English courts, says counsel for the plaintiffs, is ill-
adapted to deal with such esoteric problems as subconscious copying. It places too heavy a burden on those who seek to
establish that it has occurred. Copyright is an international proprietary right, and English law should keep in step with foreign
law. They order these things better in more sophisticated (though unspecified) jurisdictions. But the only foreign law to which
we have actually been referred is to be found in the United States cases which Willmer and Upjohn LJJ have discussed; and there,
it seems, the matter is dealt with in the same unsophisticated way as that in which Wilberforce J dealt with this case, without
making it impossible for the courts to find (where the evidence so warrants) that unconscious copying has taken place.
Faced with the difficulty that unconscious copying is by definition not susceptible of direct proof in the present state of
psychological techniques, it must always be a matter of inference from other facts, the first bold submission of counsel for the
plaintiffs was that, if the plaintiff proves (i) the presence of the necessary element of objective similarity between the copyright
work and the alleged infringing work; and (ii) the mere possibility of access to the copyright work by the author of the alleged
infringing work, there is an irrebuttable presumption (ie, a presumption of law) that the author of the alleged infringing work
unconsciously copied the copyright work; or, put more briefly, what cannot be proved must be presumed. With all respect, this is
bad logic as well as bad law. For, unless the law is an asswhich I must ex officio irrebuttably presume that it is notthe
essential, though unexpressed, premise of this proposition is that the similarities cannot be due to coincidence; proof of
possibility of access is thus unnecessary; access as well as unconscious copying can be irrebuttably presumed. But this is merely
a roundabout way of saying that proof of a causal connexion between the copyright work and the alleged infringing work is not a
necessary element in infringement of copyright; and that is not the law.
The alternative submission of counsel for the plaintiffs (although I understood it to be presented as one of law) was, I think,
on analysis merely one as to the proper inferences of fact to be drawn from varying degrees of similarity between the copyright
work and the alleged infringing work. The degree of objective similarity is, of course, not merely important, indeed essential, in
proving the first element in infringement, viz, that the defendants work can properly be described as a reproduction or adaptation
of the copyright work; it is also very cogent material 29 from which to draw the inference that the defendant has in fact copied,
whether consciously or unconsciously, the copyright work. But it is not the only material. Even complete identity of the two
works may not be conclusive evidence of copying, for it may be proved that it was impossible for the author of the alleged
infringing work to have had access to the copyright work. And, once the impossible (viz, copying) has been eliminated, that
which remains (viz, coincidence) however improbable, is the truth; I quote inaccurately, but not unconsciously, from Sherlock
Holmes.
No useful purpose can thus be served by seeking to classify degrees of similarity into categories which must be taken to be
sufficient to prove unconscious copying where access to the copyright work by the author of the alleged infringing work is
proved (i) as a certainty; (ii) as a probability; (iii) as a possibility, and (iv) as an impossibility. That is not how questions of fact
are decided in courts of law, or anywhere else. The answer, as the judge said at the conclusion of an impeccable summary of the
evidence, can only be reached by a judgment of fact upon a number of composite elements. Those elements on which the
judge directed himself have already been read by my Lord, and I need not repeat them. I agree that it is impossible for this court,
which has not heard the evidence or seen the witnesses, to say that the judge came to a wrong conclusion of fact.

Appeal dismissed. Leave to appeal to the House of Lords refused.

Solicitors: Davenport, Lyons & Co (for the plaintiffs); Maltz, Mitchell & Co (for the defendants).

F A Amies Esq Barrister.


[1963] 2 All ER 30
Practice Note
(Notice of Appeal: Setting down: Lodgment of papers)
PRACTICE DIRECTIONS
15 MARCH 1963

Court of Appeal Notice of Appeal Setting down Preparation and lodgment of papers for use of Court of Appeal
Expedition.

On 15 March 1963, after judgments in an appeal from the Westminster County Court, dated 25 January 1963, had been delivered,

15 March 1963. The following judgment was delivered.

LORD DENNING MR made the following statement. Before parting with the case I would like to say it was tried at the county
court on 25 January 1963, and comes into our list for decision today. I am gratified that the solicitors for the appellants were able
to get the papers prepared for us in time for the case. I would like all solicitors to realise that as soon as a notice of appeal is set
down it is necessary for them at once to prepare all the papers and lodge them with the court. The days are long past when they
can say: We have many months before the case will come on. The solicitors in this case remedied the position. They did not
realise how soon the case would come on. I hope all solicitors will realise that cases in this court come on with the greatest
expedition and as soon as a case is entered, the papers must be prepared at once.

Solicitors: Kinch & Richardson agents for Albert G Berry & Co, New Malden (for the appellants); Brignall, White & Orchard,
Stevenage (for the respondent).

F Guttman Esq Barrister.


30
[1963] 2 All ER 31

Cooper v Firth Brown Ltd


CIVIL PROCEDURE; QUANTUM

SHEFFIELD ASSIZES
LAWTON J
19, 20 FEBRUARY 1963

Damages Measure of damages Loss of earnings National insurance contributions Whether assessment to be based on net
earnings after deducting employees contributions.

In assessing, in an action for negligence, the special damages of an employee for loss of earnings, where national insurance
contributions have been currently deducted by his employer, the earnings to be regarded as lost are the net amount after making
deduction for the employees national insurance contributions that would have been deductible by law by the employers from the
earnings, as well as (in accordance with the decision in British Transport Commission v Gourley, infra) income tax on the
earnings.
Dictum of Lord Tucker in British Transport Commission v Gourley ([1955] 3 All ER 796 at p 810) applied.

Notes
By virtue of s 3 of the National Health Service Contributions Act, 1957, it seems that the principle of the present decision must
apply to an employees national health service contributions just as it does to his national insurance contributions. Payment of
contributions for national insurance and for national insurance (industrial injuries) is normally made by affixing a single stamp
(see National Insurance and Industrial Injuries (Collection of Contributions) Regulations, 1948 (SI 1948 No 1274), reg 6(1); see,
generally, as to contributions and their collection 27 Halsburys Laws (3rd Edn) 690, paras 1248, 1249, and pp 699, 700, para
1270.
As to collateral benefits in assessing damages in tort, see 11 Halsburys Laws (3rd Edn) 240, para 408; and for cases on the
subject, see 17 Digest (Repl) 80, 81, 2738.
For s 6 of the National Insurance Act, 1946, see 16 Halsburys Statutes (2nd Edn) 679, 680; and for s 3 of the National
Health Service Contributions Act, 1957, see 37 Halsburys Statutes (2nd Edn) 807809.
For s 2 of the Law Reform (Personal Injuries) Act, 1948, see 25 Halsburys Statutes (2nd Edn) 365.

Cases referred to in judgment


British Transport Commission v Gourley [1955] 3 All ER 796, [1956] AC 185, [1956] 2 WLR 41, 3rd Digest Supp.
Browning v War Office [1962] 3 All ER 1089, [1963] 2 WLR 52.

Action
The plaintiff, Tom Cooper, claimed damages against his employers, the defendants, for their negligence. He was sixty-five years
of age and was injured in an accident at work on 15 November 1960, which was caused by a crane driver employed by the
defendants; the crane driver was young and inexperienced and did not really know how to operate the crane. The defendants
were held liable for their negligence in putting the crane driver in charge of the crane in the circumstances; there was no
contributory negligence on the part of the plaintiff. The case is reported solely on the question whether national insurance
employees contributions, which would have been deducted from the plaintiffs wages if he had earned them, should be taken into
account for the purpose of assessing the special damage recoverable by him for loss of earnings.

H Scott QC and A C Lauriston for the plaintiff.


R Castle-Miller for the defendants.

20 February 1963. The following judgment was delivered.


LAWTON J after stating his findings of fact summarised above, holding the defendants liable in damages for the plaintiffs
injuries, and finding that there was no contributory negligence on the part of the plaintiff, assessed general damages at 600, and
continued. Now arises the point, which I have been told by counsel has so far been undecided, as to how the special damages
should be 31 calculated in this case. As a result of the plaintiffs injuries he has been away from work from time to time, and it
was agreed between counsel that if his wages were calculated on one basis the special damages would be 450, and if they were
calculated on another basis the special damages would be 433. The difference is small. I have been told by counsel for the
defendants, however, that in industry the question raised in a very important one in which, when finance throughout the country
as a whole is considered, very substantial sums are involved. The difference in calculation arises in this way. The defendants
seek to persuade me that, when I am deciding how much the plaintiff has lost by way of wages, I should pay regard solely to what
he would have had in his pay packet if he had not been injured by the accident. In other words, that I should take into account
income tax deductions. That has been decided by the House of Lords in British Transport Commission v Gourley. The
defendants go on to say that I should also take into account the fact that the plaintiff had a statutory deduction from his wages, as
long as he was working, of a sum representing his proportion of the weekly contribution for national insurance. The plaintiff, on
the other hand, says that I should disregard the statutory deduction for national insurance as that has really got nothing to do with
the defendants at all. It was an insurance payment and it was that the plaintiff himself was bound to pay; he was not under a legal
duty to pay it out of his wages.
The basis of the defendants submission is founded on an observation of Lord Tucker in British Transport Commission v
Gourley. Lord Tucker said this ([1955] 3 All ER at p 810; [1956] AC at p 215):

I agree that the phrase res inter alios acta does not assist in the solution of the problem, but the difficulty is, I feel, in
deciding what items of expenditure following the earnings of profits are to be taken into consideration and which are to be
ignored. Such items are clearly distinguishable from those which are incurred in the process of earning the profits and
which have to be deducted in the computation thereof. I think the true answer is that expenditure whichalthough not
actually a charge on earningsis imposed by law as a necessary consequence of their receipt, is relevant to the
ascertainment of the loss suffered by the party injured.

The plaintiff as a person in employment was bound to pay national insurance contributions, and under the provisions of s 6 of the
National Insurance Act, 1946,a the defendants, being liable in the first instance to pay contributions on behalf of the plaintiff,
were entitled to deduct the amount of his contributions from his wages. It is then argued on behalf of the defendants that a
situation arises which comes squarely within Lord Tuckers observations in Gourleys case. My attention was also drawn to the
recent decision of the Court of Appeal in Browning v War Office. In that case the Court of Appeal made it clear that the
principles of British Transport Commission v Gourley are not to be confined solely to questions whether income tax should or
should not be taken into consideration, and that the ratio decidendi of British Transport Commission v Gourley is that damages
should be compensatory and not punitive.
________________________________________
a The right to deduct is conferred by the joint effect of s 6(1) and (3) of the Act of 1946 and the National Insurance and Industrial Injuries
(Collection of Contributions) Regulations, 1948, SI 1948 No 1274, reg 7(2). This provides, so far as relevant,Where the insured person
receives any wages the amount of any contribution paid or to be paid by the employer on behalf of the insured person shall be
recoverable by means of deductions from the wages See also s 5(1) of the National Insurance (Industrial Injuries) Act, 1946

It seems to me that the object of damages is to compensate the plaintiff for what he has lost, and what he has lost is what
would have been in his pay packet when he took it home. As against that, it was submitted on behalf of the plaintiff that I should
bear in mind the provisions of s 2 of the Law Reform 32(Personal Injuries) Act, 1948, which oblige courts to take into account
one half of certain benefits which have been obtained by an injured person; and counsel for the plaintiff argued that, if that were
the position, the defendants could not at the same time claim that they should also have the benefit of the contributions to national
insurance out of which those benefits arise. I feel impelled to take the view that what the plaintiff is entitled to is the loss of the
contents of his pay packet, and it seems to me that when special damages are being calculated there should be deducted the
amount of any national insurance contributions which the plaintiff would have had to make if he had remained in work. In those
circumstances the appropriate sum to be taken into account by way of special damages in this case is the sum of 433. It follows
that the total damages which I award the plaintiff is the sum of 1,033.

Judgment for the plaintiff.

Solicitors: Russell Jones and Walker (for the plaintiff); Harold Jackson & Co, Sheffield (for the defendants).

G M Smailes Esq Barrister.


[1963] 2 All ER 33

Astley Industrial Trust Ltd v Grimley


CONSUMER; Consumer credit

COURT OF APPEAL
ORMEROD, UPJOHN AND PEARSON LJJ
13, 14, 15 NOVEMBER 1962, 8 MARCH 1963

Hire-Purchase Condition Implied condition Fitness Fundamental term that goods must correspond with description
Tipping lorry six years old Lorry initially in state of some disrepair Latent defects Extent of obligation of finance company.

Hire-Purchase Warranty Fitness Implied stipulation that motor vehicle hired should be as fit as reasonable skill and care
can make it Whether a fundamental term of the contract entitling hire-purchaser to terminate contract Whether able to be
excluded by term of contract Necessity for hire-purchaser to have relied on implied obligation.

The extent of any obligation of fitness impliedly undertaken by an owner on the letting on hire or hire-purchase of a specific
chattel, such as a motor vehicle, depends on the contractual intention of the parties, which is to be ascertained from the provisions
of the agreement and the relevant facts; in general it is an implied fundamental term of an agreement between a hirer and a
finance company for the hire-purchase of a motor vehicle that the vehicle hired shall correspond with the description of the
vehicle contracted to be hired (see p 40, letter h, p 44, letter f, p 46, letter d, and p 48, letter d, post).
Dicta of Parker LJ, in Karsales (Harrow) Ltd v Wallis ([1956] 2 All ER at p 870) and of Holroyd Pearce LJ, in Yeoman
Credit Ltd v Apps ([1961] 2 All ER at p 287) considered.
G, who was a haulage contractor, told C, a representative of dealers to whose garage G had gone on a Saturday, that he (G)
wanted a tipping lorry capable of carrying ten tons of hard core or other filling materials at a price between 500 and 700, that
he was trying to expand his business and that he proposed to start work on the following Monday. C showed G some tipping
vehicles and said something to the effect that one of them, which was a Bedford tipping lorry six years old, would be the one
from which G would get the most satisfaction. Its cash price was 525, whereas the price of a new vehicle of the same type
would have been 1,600. G took the lorry for a run and noticed certain defects, in particular, defects of the clutch and the starter.
C said that they could be put right over the weekend and that the lorry would be ready on Monday. G paid a deposit, and signed a
hire-purchase agreement form and a delivery form. The plaintiffs, 33a finance company, accepted the transaction and entered
into the hire-purchase agreement. The hiring was for twenty-four months. The delivery form included an acknowledgment by G
that he had examined the lorry and had satisfied himself that it was in good order and condition. Clause 3 of the agreement
provided that the hirers acceptance of delivery of the lorry should be conclusive that he had examined the vehicle and found it to
be in good order and condition and in every way satisfactory to him; cl 3 further provided that the finance company gave no
warranty as to the state or quality of the vehicle and that any warranty as to description, repair, quality or fitness for any purposes
was excluded. On Monday delivery of the lorry was taken on behalf of G, who later found that the defects had not been
remedied. G returned the lorry to the dealers, and C told him that it would be checked and put right by Saturday. By the
Saturday G lost work and profits estimated at 30. On Saturday G took the lorry under protest, since he found that the repairs
had not been done, but he had arranged to start work with it the following Monday, and drove it about 150 miles to the site of the
work. When he tipped a load there oil gushed out. He took the lorry to a garage where the oil leak was mended, and
subsequently took it again to that garage where it was overhauled and defects were remedied, the total cost being 18 to which
the dealers contributed 9. Subsequently further repairs were required amounting to some 55, including the 18. G could not
find the money for a final expensive repair, and wrote to the plaintiffs terminating the hire-purchase agreement. This letter was
treated as a repudiation of the agreement. The plaintiffs sued G for arrears of rental and for 212 18s 2d damages for termination
of the agreement. G contended that there was an implied condition that the lorry should be as fit for the purpose for which it was
hired as reasonable care and skill could make it, that it was not, and that, therefore, he had been entitled to terminate the
agreement; he counterclaimed 55 odd damages for attempting to remedy defects, and he joined the dealers as third parties
claiming against them damages for breach of an alleged express warranty of fitness and damages for breaches of contract to
repair the lorry. On appeal from a decision of the county court in favour of the plaintiffs that, though there was an implied
condition that the lorry should be usable as a tipping lorry, there had been no breach of it, and in favour of the dealers to the
extent of finding that there had not been any express warranty,

Held (i) The plaintiffs (the finance company) were in the circumstances under an obligation as to the fitness of the tipping lorry,
this obligation being that it was a lorry of the make specified, and a tipper, and a motor vehicle capable of self-propulsion along a
road and of receiving and carrying loads of material (see p 44, letter f); on the facts, however, no breach of this fundamental term
or implied condition was proved, and the plaintiffs were entitled to recover under the hire-purchase agreement (see p 44, letter g,
p 47, letter f, and p 48, letter e, post), moreover Gs acknowledgment that he had examined the lorry and found it in good order
must at least exempt the plaintiffs from contractual responsibility for visible external defects such as the defects in the clutch or
starter or oil leakage on tipping (see p 44, letter i, to p 45, letter a, post).
(ii) no express warranty by the dealers was established; but the promises by the dealers regarding repair of the lorry were
contractual and had been broken, accordingly G was entitled to damages for breach of these contracts, such damages being 30
(in respect of the first (Saturday) repair promise) and 9 (viz, the remaining half of 18 in respect of the second (Monday) repair
promise) (see p 45, letters f, g and i, and p 48, letters c and f, post).
Per Upjohn LJ (Ormerod LJ concurring): in general on a hiring of an ordinary motor vehicle for normal use two stipulations
are to be implied, 34subject to any express terms, in the contract of hiring the second of these is that the vehicle must be as fit
for the purpose for which it was hired as reasonable care and skill can make it, but this stipulation is in the nature of a warranty,
not a condition going to the root of the contract, and the lender can lawfully exclude liability on it, though such exclusion would
not, in an appropriate case, prevent the hirer claiming that the breach frustrated the contract (see p 46, letters b and h, and p 47,
letters e and g, post); in the present case the hirer knew of the principal defects and, having taken the vehicle relying on a
collateral agreement with the dealer to remedy the defects, could not thereafter complain that the lender had failed in some
implied obligation to put right those defects (see p 47, letters h and i, post).
Appeal dismissed against the plaintiffs (finance company); allowed in part as against the dealers.

Notes
Liability for breach of the implied fundamental term cannot be excluded by an exception clause in the contract (see p 46, letter e,
post).
As to implied condition of fitness in hire and hire-purchase contracts, see 2 Halsburys Laws (3rd Edn) 123, 124, para 237;
19 ibid, 532, 533, para 858; and for cases on the subject, see 3 Digest (Repl) 9597, 231250; 26 Digest (Repl) 666, 35, 36. As
to exempting clauses in hire-purchase contracts, see 19 Halsburys Laws (3rd Edn) 531, para 855, text and note (s).

Cases referred to in judgments


Andrews Bros (Bournemouth) Ltd v Singer & Co Ltd [1934] 1 KB 17, 103 LJKB 90, 150 LT 172, Digest Supp.
Brown v Edgington (1841), 2 Man & G 279, Drinkwater 106, 2 Scott, NR 496, 10 LJCP 26, 5 JP 276, 133 ER 751, 39 Digest 470,
940.
Financings Ltd v Baldock [1963] 1 All ER 443, [1963] 2 WLR 359.
Hong Kong Fir Shipping Co Ltd v Kawasaki Kisen Kaisha Ltd [1962] 1 All ER 474, [1962] 2 QB 26, [1962] 2 WLR 474, 3rd
Digest Supp.
Hyman v Nye (1881), 6 QBD 685, 44 LT 919, 45 JP 554, 3 Digest (Repl) 96, 239.
Jones v Page (1867), 15 LT 619, 3 Digest (Repl) 95, 234.
Karsales (Harrow) Ltd v Wallis [1956] 2 All ER 866, [1956] 1 WLR 936, 26 Digest (Repl) 666, 35.
Parkinson v Lee (1802), 2 East, 314, 102 ER 389, 39 Digest 454, 811.
Pollock & Co v Macrae 1922 SC (HL) 192, [1922] SLT 510, 60 Sc LR 11, 39 Digest 468, q, 661, q.
Reed v Dean [1949] 1 KB 188, [1949] LJR 852, 12 Digest (Repl) 704, 5381.
Robertson v Amazon Tug and Lighterage Co (1881), 7 QBD 598, 51 LJQB 68, 46 LT 146, 4 Asp MLC 496, 3 Digest (Repl) 96,
242.
Smeaton, Hanscomb & Co Ltd v Sassoon I Setty Son & Co [1953] 2 All ER 1471, [1953] 1 WLR 1468, 3rd Digest Supp.
Sutton v Temple (1843), 12 M & W 52, 13 LJEx 17, 2 LTOS 102, 150, 152 ER 1108, 3 Digest 95, 231.
Yeoman Credit Ltd v Apps [1961] 2 All ER 281, [1962] 2 QB 508, [1961] 3 WLR 94, 3rd Digest Supp.

Appeal and Cross-appeal


This was an appeal by George Arthur Grimley, the first defendant, and a cross-appeal by the Astley Industrial Trust Ltd, the
plaintiffs, from a judgment dated 4 June 1962, given by His Honour Judge Steel in the Manchester County Court in an action
commenced by writ dated 21 April 1961, for 235 10s 7d alleged to be due under a hire-purchase agreement dated 25 April 1960,
made between the plaintiffs and the first defendant, in which action W 35 Jones (Manchester) Ltd (the dealers) were added, first
as third party and subsequently as second defendants.
The plaintiffs, a finance company, and the first defendant, who was a haulage contractor, entered into a hire-purchase
agreement dated 25 April 1960, relating to a vehicle described as a used Bedford tipper manufactured or first registered in 1954.
The cash price of the vehicle was 525, the initial instalment was 140 and the balance of hire was 450 9s, which was payable
by twenty-four monthly instalments of 18 5s 5d each. Paragraphs 3, 7 and 9 of the agreement, so far as material, were as
follows:

3. The hirers acceptance of delivery of the vehicle shall be conclusive that he has examined the vehicle and found the
same to be complete and in good order and condition and in every way satisfactory to him the [finance company] give
no warranty as to the state or quality of the vehicle, and any warranty as to description, repair, quality or fitness for any
purpose is hereby excluded
7. The hirer may at any time determine this agreement by notice in writing to the owners and thereupon the provisions
of cl. 9 shall apply.
9. If this agreement or the hiring be terminated for any reason before the hirer has become entitled to exercise his
option to purchase hereunder the hirer shall forthwith: (a) deliver up the vehicle ; and (b) pay to the [finance
company] all arrears of hire rentals due and unpaid to the date of return of the vehicle to the [finance company] or as
agreed compensation for depreciation of the goods such further sum as together with all rentals (including first instalment
of rent) previously paid hereunder shall equal two-thirds of the hire-purchase price as specified overleaf, whichever is the
greater

By an agreement of sale made in writing on 25 April 1960, the dealers sold the motor vehicle, the subject of the hire-purchase
agreement, to the finance company for 525, the agreement of sale containing an express term that the vehicle was in roadworthy
condition and had not been misrepresented in any way to the first defendant or to the finance company.
The first defendant paid the initial instalment under the hire-purchase agreement and signed a delivery form which stated:
The vehicle being second-hand is sold in the condition in which it now is, as inspected by the buyer and without warranty. He
paid the first monthly instalment but no other instalment, and by a letter dated 15 July 1960, he wrote to the plaintiffs: I am
obliged to inform you I shall be unable to continue payments on this vehicle Please consider this a termination of agreement.
He subsequently delivered the motor vehicle to the finance company. The finance company sold the vehicle for 150.
By their re-amended particulars of claim the finance company claimed against the first defendant 235 10s 7d as the balance
due under cl 9(b) of the hire-purchase agreement or alternatively 212 18s 2d, as damages for breach of contract and 18 15s 5d,
one monthly instalment due but unpaid, making in all 231 13s 7d. Alternatively, the finance company claimed against the
dealers the said sum of 231 13s 7d, as being the loss caused by the dealers to the finance company by the dealers breach of the
terms of the agreement of sale, and an indemnity against any liability of the finance company to the first defendant. The first
defendant by his defence and counterclaim pleaded (by para 4) that it was an implied condition and/or fundamental term of the
hire-purchase agreement that the vehicle was reasonably fit for the purpose for which it was hired and (by para 5) that in breach
of that condition or fundamental term, on the commencement of the said hiring the vehicle suffered from or shortly thereafter
developed the defects set out therein and thereby was not reasonably fit for the purpose of being driven on a road and loading and
tipping materials. The first defendant pleaded (para 6) that by reason of this breach of contract he repudiated the agreement by
letter dated 15 July 1960, and (para 7) he 36 denied any liability to the plaintiffs. The first defendant also pleaded, in the
alternative, that the claim under cl 9(b) of the agreement was a penalty and that if the finance company were not in breach of
contract as alleged, they were disentitled to the sums claimed by reason of their failure to mitigate their loss. The first defendant
further pleaded (para 10) that by reason of the aforesaid breach of condition or fundamental term he had incurred expenditure and
damage totalling 55 19s in attempting to put the vehicle in a roadworthy and usable condition, and he claimed to set off the said
sum against the finance companys claim and he counterclaimed the said sum.
By a third party notice served on the dealers by the first defendant, the first defendant claimed (i) an indemnity against any
liability which he might be found to owe to the plaintiffs; (ii) damages for breach of an express warranty as to the fitness of the
vehicle for the purpose for which the first defendant required it; and (iii) damages for breaches of contracts to repair the vehicle.
The damages so claimed were quantified as follows: (i) the amount of the deposit and instalment paid under the hire-purchase
agreement, viz, 158 15s 5d; (ii) special damages under para 9, namely, 55 19s (cost of repairs) and 30 (estimated loss of
profit), totalling 85 19s; and (iii) damages for inconvenience limited to a sum which would bring the total amount payable to
400.
By para 2 of their defence to counterclaim the finance company pleaded as follows:

When the said motor vehicle was sold to the [finance company] by [the dealers] it conformed in all respects with
statutory requirements and regulations, it was in roadworthy condition and was not misrepresented to the first defendant.
Further, the first defendant signed a delivery receipt annexed to the said agreement on delivery of the said motor vehicle
and the [finance company] will rely on the terms thereof.

The dealers pleaded a defence to the third party notice but did not deliver any defence to the finance companys claim against
them.
On 4 June 1962, His Honour Judge Steel, in the Manchester County Court, gave judgment for the finance company against
the first defendant for 231 15s 7d, and for the finance company on the first defendants counterclaim, and for the dealers against
the first defendant. By a notice of appeal delivered on 10 July 1962, and amended on 18 October 1962, the first defendant
appealed to the Court of Appeal for an order setting aside the said judgment and entering judgment for the first defendant on the
finance companys claim against him and on his counterclaim and on his claim for an indemnity and damages against the dealers,
or, alternatively for an order remitting the first defendants claim against the dealers to the county court judge to give the dealers
an opportunity of adducing evidence in support of their case. The finance company cross-appealed, with leave out of time, for an
order in favour of the finance company on their claim against the dealers for damages and an indemnity if the first defendants
appeal were successful.

Vaughan Davies for the first defendant.


J Hugill for the plaintiffs.
L A Cohen for the dealers, the third party and second defendants.

Cur adv vult

8 March 1963. The following judgments were delivered.

PEARSON LJ read the first judgment at the request of Ormerod LJ. This is a hire-purchase case relating to a used tipping lorry.
The plaintiffs are the finance company; the first defendant was the hirer; the third parties (also made second defendants at a late
stage of the pleadings) were the dealers. The finance company bought the lorry from the dealers and let it on hire-purchase to the
first defendant. In the action the finance company claimed, in addition to some arrears of hiring rent which were admittedly
owing, a sum of 212 18s 2d as damages for wrongful termination of the hire-purchase agreement by the first 37 defendant. In
his defence the first defendant pleaded that it was it was an implied condition and/or fundamental term that the said lorry was
reasonably fit for the purpose for which it was hired, and that in breach of the condition or fundamental term the lorry was not so
fit and consequently he was entitled to terminate the agreement. He also counterclaimed 55 19s damages for money spent in
attempting to rectify the defects, and sought to set-off that sum against the finance companys claim. The first defendant also
served a third party notice on the dealers, and his claim can be summarised shortly as being (1) for damages for breach of an
express warranty by the dealers as to the fitness of the lorry for the first defendants purpose and (2) for damages for breaches of
contracts to repair the lorry. The finance company pleaded a defence to the first defendants counterclaim and the dealers pleaded
a defence to the third party notice.
The learned county court judge held that as between the finance company and the first defendant there was an implied
condition or fundamental term to the effect that the lorry should be usable as a tipping lorry, but that there was no breach.
Accordingly he gave judgment for the finance company against the first defendant for 212 18s 2d damages as well as for the
admitted sum of 18 5s 5d arrears of hiring rent, and he dismissed the first defendants counterclaim. With regard to the first
defendants claim under the third party notice against the dealers he held that no warranty was given by the dealers, and he
dismissed the whole claim, not dealing specifically with the alleged breaches of contracts to repair the lorry. The first defendant
appeals both against the decision in favour of the finance company on the claim and counterclaim and against the decision in
favour of the dealers as third parties. I should add for the sake of completeness that the finance company joined the dealers as
second defendants, claiming to be indemnified in respect of their loss if the first defendants defence and counterclaim should
succeed, and now the finance company have given a notice of appeal involving the same contingent claim against the dealers.
The question, however, did not arise in the court below, and does not arise in this court unless the first defendants appeal against
the decision in favour of the finance company succeeds.
The main facts are as follows:On Saturday, 23 April 1960, the first defendant, having been attracted by one of the dealers
advertisements, went with his brother to the dealers garage and spoke to a man named Wilfred Callister, whose exact position
does not appear from the evidence but who evidently had authority to act on behalf of the dealers. The first defendant told
Callister that he was looking for a tipping vehicle, and wanted one capable of carrying ten tons of hard core or other filling
materials; that he was already a haulage contractor with a B licence and pretty well established, and wished to expand his
business and therefore wanted a tipping vehicle; and that he had arranged to start work on the Monday morning (25 April 1960)
in connexion with some operations being carried out at Barton Bridge near Manchester. The first defendant was shown several
tipping vehicles, and eventually, as he said in his evidence, he put his cards on the table, telling Callister that he had not been
tipping at all, and his price was between 500 and 700, and that he was asking what would be the best. Callister took him over
to a used Bedford tipping lorry, and said something to the effect that that was definitely the one he would get the most satisfaction
from. The vehicle was about six years old and the cash price was 525, whereas the price of a new lorry of the same type would
have been 1,600. The first defendant with one of the garage mechanics took the vehicle for a run and the first defendant noticed
certain defects, principally that when the vehicle was stopped and re-started the clutch was very fierce and the starter failed to
operate. He mentioned these defects to Callister, and Callister said That can be put right over the week-end and added that his
mechanics would work over the week-end and the vehicle would be ready for work on Monday. Thereupon the first defendant
paid a deposit of 140 and signed a hire-purchase agreement form, putting on it the date 25 April 1960, though he signed it on
Saturday, 23 April. 38That was his offer, and it was accepted by the finance company also signing it, probably on 25 April 1960,
as that date was not altered. Also on 23 April 1960, the first defendant signed a delivery form. I will refer later to the provisions
of the hire-purchase agreement and the delivery form.
On Monday, 25 April the first defendants brother brought the vehicle from the dealers garage, and the first defendant saw it
later in the day and found the same defects in the clutch and the starter and an oil leakage in the tipping gear. He took it back to
the dealers and eventually saw Callister and complained to him of the continued existence of the defects, saying that nothing had
been done. Callister said at first that he would need the vehicle for at least a week, but later said it would be ready by Saturday
(30 April). He said that he would give it a complete check over and that it would be right by Saturday. The first defendant
accordingly left the vehicle at the garage to be checked over and put right.
When he came to take the vehicle on Saturday, 30 April he found it in no better state. In his words the clutch was still
juddering, the starter was exactly the same, and they had not done the gear box. He had, however, arranged to start work on the
following Monday, 2 May on the Llanwern site near Newport in Monmouthshire. He took the vehicle under protest, telling the
dealers that they would hear more of it. He drove the vehicle on a journey of about 150 miles from Manchester to the Llanwern
site, and there is no evidence of the vehicle giving any trouble on that journey. Having arrived at the Llanwern site, he went,
presumably on Monday, 2 May with the vehicle to the quarry from which loads of hard-core were to be obtained. He took the
first load. When he tipped the load, oil gushed out. On the same day, 2 May he took the vehicle into Larcombes garage. A new
gasket was put in the tipping gear box, and this appears to have cured the oil leakage because there is no evidence of any oil
leakage afterwards. The first defendant arranged with Larcombe that the vehicle should be taken in for a full service and
thorough overhaul at the end of the week. During the week the first defendant carried ten loads, whereas he would have expected
to carry seven or eight loads a day, and he said that he was nursing the vehicle.
The work on which he was using the vehicle was very heavy work, such as might wear out even a new tipping lorry in a
short period. Presumably the lorry fetching hard core from a quarry would have to manoeuvre on an irregular and very hard
surface and thus would undergo severe strains and stresses. The learned judge said in his judgment I have evidence that this
type of work can smash a new lorry in six months. He may well have had such evidence, though it does not appear in the notes
of evidence. In the notes of the cross-examination of the first defendant there is this passage:

The work was rough, but I would not say a new lorry could be worn out by it in six months. I know that vehicles have
been abandoned at the Llanwern site, but they are not wrecks after six months. I did see two or three fairly new lorries left
abandoned but I made no inquiries myself for lorries lying about. I agree that I took a six year old lorry for rough work.

The vehicle was taken in to Larcombes garage at the weekend (6 or 7 May), and remained there till the following Thursday (12
May). Larcombe made a thorough examination and found numerous defects. Three of the road wheels were cracked and
unserviceable, the safety bracing on the tipping gear was cracked ninety per cent through, there was a broken main spring and a
broken helper spring, and the hand brake was defective. There was a crack in the main chassis in front of the main spring hanger,
and this was not a new crack, as grime had to be removed to see it. The starter ring had a number of teeth missing or badly worn.
There was a violent judder caused by a mechanical fault in the clutch mechanism. In Larcombes opinion the vehicle was
unroadworthy.
On a Monday, which was probably 9 May after the defects had been ascertained, the first defendant telephoned to the
dealers and mentioned the defects 39 that had been found, and asked them what they would do and whether they would agree to
pay the cost of repairs. The dealers offered to pay half the cost of materials, but were not willing to make any contribution to the
labour cost as they had their own mechanics who could have done the work. Larcombe repaired the vehicle, and the bill was
estimated at 18, and the first defendants brother collected 9 from the dealers. It was necessary to obtain main springs, a helper
spring, and there was a considerable amount of welding and there were four wheels to be replaced. These wheels were bought at
a price of 1 each. After the repairs the vehicle returned to the same very heavy work. It was in Monmouthshire for about seven
weeks, and it operated on the site for about five weeks, but not for any full week.
At intervals in June three disasters occurred. One evening, when the vehicle was operating from the quarry, the engine
mounting bracket collapsed, dropping one side of the engine on the front axle. The vehicle was towed to the nearest garage and
repaired by welding at a cost of 6. In the following week the cab supports collapsed, and the vehicle was again taken to the
nearest garage and repaired by welding and the cost was 10 10s. Other repairs were carried out to the hand brake, and to the
push rods for the clutch, as these push rods had broken by reason of the juddering and shaking. Tyres had to be renewed because
the juddering had caused the wheels to spin, and the total cost of tyre renewals was 22 10s. The final disaster occurred when the
clutch collapsed, and the repairing of it would be expensive. Even then the first defendant, if he had been able to raise the money,
would have got the vehicle out and continued operating. He had spent about 55 on the vehicle between 1 May and 15 July. The
figures which I have mentioned18, 6, 10 10s and 22 10smake a total of 57, which is sufficiently close to the
approximate figure of 55.
The first defendant was not able to raise the money for repairing the collapsed clutch, and he wrote a letter dated 15 July
1960, to the finance company, stating that he was unable to continue payments and terminating the agreement. It has been
assumed throughout these proceedings that this letter constitutes a repudiation of the agreement. The finance company replied on
26 July 1960, indicating that claims would have to be made. They were treating the first defendants letter as a repudiation of the
agreement. Manifestly the first defendant has been unfortunate, and deserves sympathy, but I have to consider only the
contractual rights and obligations of the parties and the alleged breaches of contract. First I will consider the contractual position
as between the first defendant and the finance company.
Counsel for the first defendant contended, as he had to contend in order to succeed against the finance company, that from
the mere fact of the finance company letting this tipping lorry on hire to the first defendant there was to be implied a condition in
his favour, binding on the finance company, that the vehicle should be as fit for his purpose, ie, for use as a tipping lorry, as
reasonable care and skill could make it. The decided cases show that normally in a hiring or hire-purchase agreement the person
who lets goods on hire assumes some contractual responsibility for the fitness of the goods for the purpose for which the hirer
requires them, but the existence and the extent of the obligation depend on the contractual intention of the parties, which is to be
ascertained from the provisions of the particular agreement and from the relevant facts of the situation in relation to which the
agreement was made. There could be a case in which there would be no such obligation at all. Suppose that a customer, proud of
his skill as a mechanic, hired or took on hire-purchase a dilapidated and immobile vehicle, hoping to be able by his own efforts to
put it into good repair and working order, and making it clear to the persons letting it out on hire to him that he would not hold
them responsible either for the existing state of disrepair or for the success of his attempts to rectify it. In such a case no rule of
law would compel the court to defeat the contractual intention of the parties by imposing on the persons letting the vehicle on
hire an obligation as to the fitness of the vehicle 40 for the hirers purpose. Therefore it is not possible to accept as an unqualified
proposition the contention advanced in this appeal on behalf of the first defendant.
I will cite passages from decided cases as showing the general principle involved and as illustrating the matters to be taken
into account in determining the existence and extent of the obligation in a particular case.
In Sutton v Temple it was held that in a demise of land there was no implied obligation that the land should be fit for the
purpose for which it was taken. There are, however, in the judgments some obiter dicta as to the letting of goods. Lord Abinger
said ((1843), 12 M & W at p 61):

The letting of the goods and chattels, as well as the house, implies that the party who lets it so furnished is under an
obligation to supply the other contracting party with whatever goods and chattels may be fit for the use and occupation of
such a house, according to its particular description, and suitable in every respect for his use. Where the party has had an
opportunity of personally inspecting a ready-furnished house by himself or his agent before entering on the occupation of
it, perhaps the objection would not arise; but if a person take a ready-furnished house upon the faith of its being suitably
furnished, surely the owner is under an obligation to let it in a habitable state. Common sense and common justice concur
in that conclusion.

Baron Parke said ((1843), 12 M & W at p 64):

One class of cases is, where the agreement is for a specific chattel in its then state, as in Parkinson v. Lee; there there
is no implied warranty of its fitness or merchantable quality: another class of cases is, where a person is employed to make
a specific chattelthere the law implies a contract on his part that it shall be fit for the purpose for which it is ordinarily
used: and there is also an intermediate class of cases, where goods are ordered for a specific purpose from a person in a
particular department of trade; in that case, also, Brown v. Edgington is an authority for saying, that the law implies an
undertaking that they shall be fit for that specific purpose.

Then Jones v Page, Hyman v Nye and Reed v Dean, are examples of cases where goods are ordered for a specific purpose from a
person in a particular department of trade. In Hyman v Nye, Lindley J, said ((1881), 6 QBD at p 687) that the duty of a jobmaster,
letting out horses and carriages for hire, was to supply a carriage as fit for the purpose for which it is hired as care and skill could
render it. He went on to say ((1881), 6 QBD at p 688):

Nor does it appear to me to be at all unreasonable to exact such vigilance from a person who makes it his business to
let out carriages for hire. As between him and the hirer the risk of defects in the carriage, so far as care and skill can avoid
them, ought to be thrown on the owner of the carriage. The hirer trusts him to supply a fit and proper carriage; the lender
has it in his power not only to see that it is in a proper state, and to keep it so, and thus protect himself from risk, but also to
charge his customers enough to cover his expenses.

Then Mathew J said ((1881), 6 QBD at pp 689, 690):

It appears to me that the question which the jury ought to have been asked was, whether the carriage was, in fact,
reasonably safe when it was hired by the plaintiff. The cases referred to by my brother LINDLEY, seem to show that there
is no distinction in this respect between contracts for the sale and for the hire of an article for a specific purpose, where
trust is reposed in the 41 person who, in the ordinary course of business, sells or lets to hire. The purpose and use, the time
for which the article is intended to be used, seem to me the essential part of the contract Here, the defendant let the
carriage for the purpose of carrying the plaintiff safely. The plaintiff trusted him to select the carriage, horses, and driver,
and there seems to me nothing unreasonable in charging the defendant with a duty which it was certainly in his power to
fulfil, and which from his business he would be presumed to have bound himself to take the proper steps to perform
strictly.

Then in Robertson v Amazon Tug and Lighterage Co, there was no hiring of a chattel but there are dicta on the subject. Bramwell
B, said ((1881), 7 QBD at p 603):
The case seems to me the same as a contract of hiring, and as all contracts when one man furnishes a specific thing to
another which that other is to use. The man so letting and furnishing the thing does not, except in some cases, undertake
for its goodness or fitness, but he does undertake for the condition being such that it can do what its means enable it to do.
Thus, if a man hired a specific horse and said he intended to hunt with it next day, there would be no undertaking by the
letter that it could leap or go fast; but there would be that it should have its shoes on, and that it should not have been
excessively worked or unfed the day before.

Brett LJ said ((1881), 7 QBD at p 606):

When there is a specific thing, there is no implied contract that it shall be reasonably fit for the purpose for which it is
hired or is to be used. That is the great distinction between a contract to supply a thing which is to be made and which is
not specific, and a contract with regard to a specific thing. In the one case you take the thing as it is, in the other the person
who undertakes to supply it is bound to supply a thing reasonably fit for the purpose for which it is made.

Cotton LJ said ((1881), 7 QBD at pp 608, 609):

It has been suggested that the plaintiff is in the same position as the hirer of an ascertained chattel, and the defendants
in the same position as the person who lets the chattel to hire. There is at least a doubt what warranty the law implies from
the relation of hirer and letter to hire of an ascertained chattel.

In Karsales (Harrow) Ltd v Wallis, Parker LJ said ([1956] 2 All ER 866 at p 870):

I think it is the duty of a hire-purchase finance company, which is letting out a chattel on hire-purchase, to ascertain
that the chattel is reasonably fit for the purpose for which it is expressly hired.

In my view that sentence should be understood as stating a general proposition not necessarily applying to every case. For
instance, it cannot apply if (as in the present case) the evident contractual intention of the parties is that the finance company shall
not have an opportunity of ascertaining the fitness of the chattel before the hiring begins. Parker LJ went on to refer to the
exceptions clause in that case, which was cl 3(g), and he said ([1956] 2 All ER at p 871):

But, in my judgment, however extensive the exception clause may be, it has no application if there has been a breach
of a fundamental term. We were referred to a number of cases, including Smeaton Hanscomb & Co., Ltd. v. Sassoon I.
Setty, Son & Co..

After citing from the judgment of Devlin J, in that case, Parker LJ, went on ([1956] 2 All ER at p 871):

Applying that to the facts of this case, it seems to me that the vehicle delivered in effect is not properly described (as
the agreement describes it) 42as a motor vehicle, Buick, giving the chassis and engine number. By that I am not saying
that every defect in a car which renders it for the moment unusable on the road amounts to a breach of a fundamental term;
but where, as here, a vehicle is delivered incapable of self-propulsion except after a complete overhaul and in the condition
referred to by my Lord, it seems to me that it is abundantly clear that there was a breach of a fundamental term and that
accordingly the exceptions in cl. 3(g) do not apply. I think the same result is achieved by saying, in effect, that what was
delivered was not what was contracted for; and I think that Andrews Bros. (Bournemouth), Ltd. v. Singer & Co. Ltd. might
well have been decided on the basis that there had been a breach of a fundamental term.

Then in Yeoman Credit Ltd v Apps, Holroyd Pearce LJ, having referred to a number of decided cases and finally to the decision of
Lewis J, in Reed v Dean that there was in that case an implied undertaking by the defendant that the vessel should be as safe as
care and skill could make it, said ([1961] 2 All ER at p 287):

Those cases clearly establish that such a warranty, condition or undertaking exists in the hiring of a specific chattel
except in the cases where defect is apparent to the hirer and he does not rely on the skill and judgment of the owner.

In my view that sentence should be regarded as directing attention to what are usually the main factors affecting the extent of the
implied obligation, and not as excluding other factors which may be relevant and important in a particular case. Holroyd Pearce
LJ, also said ([1961] 2 All ER at pp 289, 290):

Whether there has been a breach of a fundamental condition of the agreement is a question of degree depending on the
facts. Such a breach is different in weight and gravity from breaches of condition which would come within the exemption.
There may be, as in Pollock & Co. v. Macrae and in the present case, an accumulation of defects which, taken singly, might
well be within an exception clause, but which, taken en masse, constitute such a non-performance, or repudiation, or breach
going to the root of the contract, as disentitles a party to take refuge behind an exception clause intended to give protection
only in regard to those breaches which are not inconsistent with and not destructive of the whole essence of the contract.
One would not lightly come to the conclusion that mere defects in repair, even though numerous, amount to breach of a
fundamental condition of the contract. In the present case, however, I accept the finding of the learned judge on the facts
that: The plaintiffs were in breach of a fundamental term of the contract to hire a motor car suitable for use on the
highway and cannot rely on the exemption clause. There was, to use the words of LORD DUNEDIN a, such a congeries
of defects as to destroy the workable character of the machine.
________________________________________
a In Pollock & Co v Macrae, 1922 SC (HL) 192 at p 200

Now I come to the hire-purchase agreement between the first defendant and the finance company. There is no evidence of
the dealers being for any relevant purpose agents of the finance company, or of the arrangements made between the first
defendant and the dealers having been communicated to the finance company. The express terms of the contract between the first
defendant and the finance company have to be found in the agreement itself. The main points are as follows: (1) The date for
commencement of hiring was Monday, 25 April 1960. The first defendant had made his offer by filling in or having filled in for
him the details in the schedule and by putting his signature on the second page, on Saturday, 23 April. He was intending to drive
the vehicle away on the Monday. 43Presumably the finance company had never seen the vehicle and had no previous
knowledge of it. The first defendant obviously was not relying on the skill or judgment of the finance company to choose a
suitable vehicle for his purpose, or to give him any advice as to his choice, and he was not giving them any opportunity to
examine it or make inquiries about it before the commencement of the hiring. (2) The first defendant described himself as a
haulage contractor and referred to previous hire-purchase transactions. He could be expected to have some knowledge of
vehicles. (3) The vehicle was described as a Bedford tipper manufactured or first registered on 31 March 1954, so that it was six
years old. It is reasonable to assume that a tipping lorry is likely to be subjected to more arduous use, and to be in the common
phrase older for its age, than a motor car. It could not reasonably be expected that a six-year old tipping lorry, not stated to have
been recently overhauled or reconditioned, would be in anything like new condition or free from latent defects. (4) This was a
hire-purchase agreement with a hiring period of twenty-four months, so that immediate readiness for use would not appear to be
so important as in the case of a hiring for a short period. (5) The hirer acknowledged that before signing the agreement he had
examined the goods and satisfied himself that they were in good order and condition. It was true that he had examined them, but
not true that he was satisfied with them. However his dissatisfaction cannot prejudice the finance company, as he made no
complaint to them, and this acknowledgment must have its due effect on the extent of the finance companys obligations under
the agreement. (6) Clause 3 provides:

The hirers acceptance of delivery of the vehicle shall be conclusive that he has examined the vehicle and found the
same to be complete and in good order and condition and in every way satisfactory to him. Except where it is implied by
the Hire-Purchase Acts, 1938 and 1954 the [finance company] give no warranty as to the state or quality of the vehicle,
and, save as aforesaid, any warranty as to description, repair, quality or fitness for any purpose is hereby excluded.

In my view the finance company are not entitled to say that they had no obligation at all under the agreement as to the
fitness of the vehicle for the first defendants purpose. They were letting on hire to him a Bedford tipper, and it had to be a
Bedford tipper, ie a lorry of that make, and a tipper. It had to be an automobile, capable of self-propulsion along a road, and it
had to be capable of receiving and carrying and tipping loads of materials. The condition or fundamental term is to be implied in
this case, as it was in the Karsales case and in the Apps case. The learned county court judge, however, has found that there was
no breach of this condition or fundamental term, and on the facts of this case he was in my view clearly right. The vehicle was
capable of self-propulsion because it propelled itself from Manchester to the vicinity of Newport, Monmouthshire, and thereafter
to and from the Llanwern site, and it was receiving and carrying and tipping loads under very adverse conditions for five weeks.
There was some initial disrepair of a not trivial character, but even after some aggravation, probably, by the first week of
hard use on the Llanwern site it was put right at a cost of 18, of which the dealers paid half. The later disasters and grave
defects were probably caused by the continuing hard use of this six-year old lorry on work which put too great a strain on it. That
is really the explanation of the unfortunate history of this lorry, and that was the view of the learned county court judge and I
agree with him. The initial state of disrepair consisted, largely at any rate, of visible external defects, such as the fierce clutch
and the starter failing to operate and the oil leakage on tipping. The acknowledgment by the first defendant in the hire-purchase
agreement that he had examined the vehicle and found it in good order must at least exempt the finance company 44 from
contractual responsibility for visible external defects. Then what is left of the initial state of disrepair? The first defendant may
well have had good reasons, as against the dealers who had failed to do the repairs which they had promised, for driving the lorry
in its unrepaired state from Manchester to Llanwern and putting it into service there, but the result may have been to cause further
defects, for which the finance company could not in any case be held responsible. It was however not reasonably to be expected
that a six-year old tipping lorry, not stated to have been overhauled or reconditioned, would be wholly free from latent defects.
Larcombes bill at the end of the first week in May was only 18, of which the dealers paid half. If it be assumed that the finance
company had some obligation in respect of repair greater than the obligation to provide a lorry capable of self-propulsion and of
tipping, it is not clear that there was any breach of it. However, in my judgment, having regard to the provisions of the hire
purchase agreement and the facts as stated in it, there was no such greater obligation.
Now as against the dealer the first defendant has in my view failed to establish any express warranty as to the fitness of the
vehicle for his purpose. In the first place the oral evidence does not prove any words which prima facie have the appearance of a
warranty. The first defendant said he wanted a tipping lorry costing 500-700. Callister took the first defendant to this vehicle
and said It is definitely the one you will get the most satisfaction from or words to that effect. That seems to be saying This is
the best vehicle that we can provide at your price, without any suggestion that the dealers are assuming contractual
responsibility for its efficiency. Secondly, the first defendant signed at the request of the dealers a delivery form which contained
the words The vehicle being second-hand is sold in condition in which it now is, as inspected by the buyer and without
warranty. I agree with the county court judge that that is strong evidence of the absence of a warranty.
Finally however there are to be considered the dealers two broken promises with regard to the repair of the vehicle. The
learned county court judge did not deal with this aspect of the case, which was of course of less importance than the other issues.
In my judgment however these promises were contractual promises. The first promise was given on Saturday, 23 April 1960, in
consideration of the first defendant signing the hire-purchase agreement, and it was a promise to remedy the defects by Monday,
25 April so as to enable the first defendant to start his work on the Burton Bridge site. In consequence of the dealers failure to
carry out their promise the first defendant could not do any work on the Burton Bridge site, and lost profits which he estimated at
30, and he was not, so far as appears from the notes of evidence, cross-examined as to that estimate. In my view the first
defendant should have 30 damages against the dealers for breach of the Saturday contract. The figure seems on the generous
side, but there is no basis for fixing any other figure, and it would be unreasonable in the circumstances to order an inquiry as to
damages. Secondly, there was the Monday contract, made on Monday, 25 April to give the vehicle a complete check-over and
have it right by Saturday, 30 April. There was consideration, as the first defendant allowed the dealers to keep the vehicle and
work on it and seek to minimise the damage caused by their breach of the earlier contract. This promise also was unfulfilled, and
consequently the first defendant had to drive the lorry and start using it in its unrepaired condition. Exact assessment of the
damage is not possible, but Mr Larcombes bill was 18, and the dealers have already paid half of it, and the remaining 9 would
be a reasonable assessment of the damages for this breach.
I would dismiss the first defendants appeal against the judgment in favour of the finance company. I would allow in part
the first defendants appeal against the dismissal of his claim against the dealers in the third party proceedings, and I would give
him judgment against them for 39 damages for breaches of contracts.
45

UPJOHN LJ. I agree with the judgment which has just been delivered by Pearson LJ and only desire to add a few words of my
own. My Lord has very fully recited the facts and discussed the authorities and no further reference either to fact or to authority
is necessary.
The question whether there are any implied conditions, warranties or stipulations relating to the hiring (whether it be pure
hire or hire-purchase) of any vehicle, must depend on the circumstances of each case and, as my Lord has pointed out, it is not
possible to lay down a general rule applicable to every hiring. So much must depend on the facts of each case. This case, as I
shall show, is a very good illustration of it. In general on a hiring of an ordinary motor vehicle, whether a car, lorry or a van
which the parties are intending shall be let and hired for normal use on the road for ordinary purposes for which the vehicle
appears to be suited and capable, there are in my judgment, subject to any express terms as to state of repair or condition and so
forth, two stipulations to be implied in the contract of letting and hiring.
First, there is an implied stipulation that the vehicle hired corresponds with the description of the vehicle contracted to be
hired, or to put it in another way, the lender must lend that which he contracts to lend and not something which is essentially
different. Thus, as my Lord has pointed out, a tipping lorry plainly hired for use as such must be capable of self-propulsion to a
reasonable degree and must have a rear compartment capable of being mechanically tipped.
This implied stipulation is a fundamental implied term and breach of it at once gives the hirer the right, if he desires to do so,
to treat the contract as repudiated. Furthermore, being a fundamental term the lender cannot by clauses of exclusion or exception,
however widely phrased, exclude liability for this fundamental term for the simple reason that the law will not permit one of the
contracting parties to escape liability for failure to deliver that which he has contracted to lend by delivery of something which is
essentially different. The question whether or not the motor vehicle delivered complies with this fundamental obligation of the
lender is very largely a question of fact and degree and must depend on the circumstances of each case. Many factors will be
relevant; I attempt no exhaustive catalogue. On the one side its condition so far as apparent to the ordinary inspection by a
layman and its known defects will be relevant. On the other side, of course, most important of all are defects unknown to the
hirer at the date of delivery; then the impact of such unknown defects not merely on the roadworthiness of the car but on its
general condition and ability to perform the tasks for which it is hired; and in deciding whether a somewhat defective vehicle
complies with this fundamental term it is clearly relevant to have in mind the type and expense of repairs and the time required to
put it into reasonable condition. It must not be forgotten that persons resort to hire-purchase because they cannot afford to pay
cash down and may reasonably want the early use of the vehicle in order to earn the hire-purchase rent. Therefore if a large sum
or a long time is required to put a car into a reasonable state of repair that is a most material circumstance in assessing whether a
vehicle does or does not comply with this fundamental term.
The second implied stipulation is that the vehicle must be as fit for the purpose for which it is hired as reasonable skill and
care can make it. I would regard this not as a condition going to the root of the contract but as a stipulation in the nature of a
warranty, that is to say a stipulation breach of which does not thereupon automatically give the hirer the right to terminate the
contract if he desires to do so. It is akin to the stipulation of seaworthiness and I discussed this aspect of the matter in the recent
case of Hong Kong Fir Shipping Co Ltd v Kawasaki Kisen Kaisha Ltd ([1962] 1 All ER 474 at p 483; [1962] 2 QB 26 at p 63),
where I pointed out the danger of using the words condition and warranty except under the Sale of Goods Act, 1893.
Although in my judgment breach of this stipulation does not at once 46 entitle the hirer to treat the contract as at an end, it may
nevertheless in certain circumstances entitle the hirer to do so. Let me give one example: suppose that a vehicle which is to be
let on hire or hire-purchase over a period of two years is at the moment when it ought to be delivered minus four tyres and all its
sparking plugs and the carburettor is unserviceable; that would not entitle, I think, the hirer to treat the contract as repudiated; not
at all events if the lender offers to put on four tyres, the necessary number of sparking plugs and fit a new carburettor, all of
which could be done within a day; no doubt the hirer would be entitled to sue for damages for one days late delivery. On the
other hand if a car which is to be hired at 9 am sharp for the express purpose known to the lender of carrying the hirer 150 miles
to lunch in the country and back that same afternoon, is at 9 am in the defective condition which I have just mentioned, in my
judgment the defects would go to the root of the contract and would clearly frustrate it, for the defects could not be put right in
time to permit the hirer to reach his luncheon engagement in time. He would be entitled therefore to treat the contract as
repudiated if he so desired. It may be that it would also amount to a breach of the fundamental term mentioned earlier for the
lender has contracted to deliver a car capable of starting at 9 am sharp and time is of the essence. This shows that these two
implied stipulations may in some cases tend to merge. But essentially they are separate and distinct and a breach of the
stipulation of fitness alone may in proper circumstances give rise to a frustration of the commercial purpose of the contract which
entitles the hirer to treat it as repudiated by the lender and at an end. On the other hand liability for breach of this stipulation of
fitness when sounding in damages is one which may be excluded by appropriate clauses of exclusion or exception. Such a clause
of exclusion could not however prevent the hirer from claiming that the breach frustrated the contract.
I turn then to apply these propositions of law to the facts of this case. The vehicle was delivered to the hirer in a very
defective condition but Pearson LJ has very fully reviewed the facts and has shown how on those facts it is impossible to treat the
lender as being in breach of the fundamental condition which I have mentioned above. The hirer accepted and used the vehicle
for very heavy work for many weeks and would have gone on using it if he had not run out of money. On those facts I agree with
the lord justice and the county court judge entirely and that matter therefore passes from consideration
On the second implied stipulation (of fitness) it is possible that the hirer might originally have had some claim for damages
against the lender for the defective condition in which he found the vehicle but for the fact that by cl 3 of the agreement the
lender excluded liability as, in my judgment, he lawfully could. But another answer to this claim which depends on the facts of
the case, is, that the hirer never relied on any implied obligation of fitness by the lender when he took the vehicle. He knew of
the principal defects yet took the vehicle relying on the collateral agreement with the dealers to rectify those defects. He gave the
dealers two opportunities to repair the vehicle before he took it away and was then content to take it some 150 miles to the
Llanwern site in South Wales and even when he arrived there and then had trouble he was content to come to a further
arrangement with the dealers with regard to its repair and the latter in fact paid some 9 as a contribution to that repair. If a man
takes a vehicle with known defects and uses it for its ordinary purposes and relies on some collateral agreement with the dealer to
put the defects right he cannot in my view afterwards complain to the lender that the latter (who in fact had no knowledge of the
defects or opportunity of putting them right) has failed in some implied obligation to put it right. In law the answer must be that
the implied obligation is excluded for the hirer never relied on it. It is true that later on defects appeared in the motor vehicle but
that was only after the vehicle, itself a somewhat old vehicle, had been subjected to the most severe treatment at the hands of the
hirer. For these reasons I do not see how the 47 hirer can have any complaint against the lender for the breach of this implied
obligation (had it not been excluded) in the particular circumstances of this case.
I want to add one word on the lenders claim for damages in the light of the recent decision (to which I was a party) in
Financings Ltd v Baldock, which was decided since this case was argued. In this case before the learned county court judge
damages were assessed on the footing that the hirer had repudiated the hire-purchase agreement. An application was made to us
to amend the notice of appeal by raising some question on the measure of damages on the footing that there had been no such
repudiation but at that late stage we refused leave to amend. It follows therefore that this court and the learned county court
judge treated the case, whether rightly or wrongly, as one of repudiation by the hirer and therefore there is no conflict of opinion
between Financings Ltd v Baldock and this case.
As to the claim of the hirer against the dealers, I desire to say only that I concur entirely with the judgment of Pearson LJ. I
agree that this appeal must be dismissed against the finance company but allowed to the extent indicated by my Lord against the
dealers.

ORMEROD LJ. I agree with the judgments which have been delivered and would also dismiss the appeal against the plaintiff
finance company but allow the appeal against the third party (the dealers) to the extent indicated by Pearson LJ. In my judgment
it is a fundamental term of a contract of hiring of this kind that the goods hired must correspond with the description of the goods
which are contracted to be hired. In this case, as Pearson LJ has pointed out, the goods in question consisted of a tipping lorry,
that is, a vehicle capable of self-propulsion and of being mechanically tipped. There can be little doubt that the vehicle complied
with these requirements. Not only was it driven from Manchester to South Wales but was driven to and from the quarry there to
the place where the stone was tipped. It is important to remember also that the first defendant was given an opportunity of trying
out the lorry before he made the agreement and knew, of course, that it was far from being a new one. It is true that the tipping
gear was not as efficient as it might have been when it was first taken over, but the defect was one which was easily remedied and
could not be said to amount to a breach of a fundamental term.
I agree with my brethren, however, that the first defendant has a claim for damages against the dealers in respect of their
failure to comply with their agreement to carry out certain repairs to the vehicle. This aspect of the matter has already been fully
considered by them and no useful purpose will be served in discussing it further.

Appeal dismissed as against plaintiffs; allowed in part as against the third party; judgment for first defendant against the third
party for 39.
Solicitors: J E Lickfold & Sons agents for Taylor, Hindle & Rhodes, Manchester (for the plaintiffs); Kanter, Jules & Co agents for
Boote, Dutton & Whittaker, Manchester (for the first defendant); Gibson & Weldon agents for Grover, Smith & Moss, Manchester
(for the second defendant and third party).

Henry Summerfield Esq Barrister.


48
[1963] 2 All ER 49

Lepre v Lepre
FAMILY; Other Family

PROBATE, DIVORCE AND ADMIRALTY DIVISION


SIR JOCELYN SIMON P, JULY 13, 1961, BEFORE COLLINGWOOD J
3, 4, 5 DECEMBER 1962

Nullity Foreign decree Recognition Contrary to justice Maltese husband marrying English wife at register office in
England Husband a Roman Catholic Marriage valid by English law but voidable by Maltese law Decree of nullity granted
to husband in Malta Whether decree recognised by English court.

L was a Maltese by birth and a Roman Catholic by religion. He came to England and in April, 1955, went through a civil
ceremony of marriage at a register office in Portsmouth with the wife who was an Englishwoman with whom he had lived for
some years. In May, 1955, the parties, together with their child went to Malta, but in 1956 they separated and the wife returned to
England with the child. In 1957 the wife obtained in the magistrates court orders for the maintenance of herself and for the
custody and maintenance of the child. The wifes subsequent application in Malta for registration and confirmation of these
ordersa was adjourned by the Maltese court, L contending that he was not married to the wife since, being a domiciled Maltese of
the Roman Catholic faith, he could not contract a valid marriage in a register office. On 28 March 1960, the Maltese court
pronounced at the suit of L a decree of nullity on the ground that the marriage in England had failed to comply with the canon
law. The wife now petitioned for a declaration that the decree of nullity by the Maltese court was invalid and of no effect, and for
a divorce on the grounds of Ls cruelty and constructive desertion, herself seeking the exercise of the courts discretion in respect
of her own adultery.
________________________________________
a See Maintenance Orders (Facilities for Enforcement) Act, 1920, s 3; 11 Halsburys Statutes (2nd Edn) 857

Held (i) In determining whether or not there was a subsisting marriage to dissolve, the court must necessarily determine
whether the Maltese decree of nullity should be recognised as valid; accordingly there was jurisdiction under RSC, Ord 25, r 5, to
make the declaration sought (see p 53, letter g, post).
Har-Shefi v Har-Shefi ([1953] 1 All ER 783) considered.
(ii) unless the decree of the Maltese court were vitiated by fraud or as being contrary to natural justice, it should be accepted
by the English court as conclusive (see p 56, letter i, to p 57, letter a, post), for the following reasons
(a) because the defect of the marriage in England was by English law a matter of formalities and was thus referable to the
lex loci celebrationis, viz, to English law, and by that law the marriage was valid, with the consequence that at the date of the
commencement of the Maltese nullity suit (which was the relevant point of time) the wife was married and had acquired, and
retained, a Maltese domicil, and accordingly the Maltese court was the court of the common domicil and its decree was valid (see
p 55, letter c, post); moreover it appeared that in Maltese law the defect rendered the marriage voidable, not void, and indeed that
all relevant systems of law would conclude that the marriage was subsisting when the nullity suit in Malta began, so that the wife
was then domiciled with L in Malta (see p 55, letter g, post).
Dictum of Lord Greene MR, in De Reneville v De Reneville ([1948] 1 All ER at p 61) applied.
(b) and because, even if the English marriage were void ipso jure, recognition ought to be accorded to the Maltese decree,
since in like circumstances the English court would assume jurisdiction in nullity if the petitioner alone were domiciled in
England, and, therefore, should concede the like jurisdiction 49 to the Maltese court, L being domiciled in Malta at the
commencement of the Maltese suit (see p 56, letter b, post).
Travers v Holley and Holley ([1953] 2 All ER 794) and Corbett v Corbett ([1957] 1 All ER 621) followed.
Ogden v Ogden ([19047] All ER Rep 86) doubted.
(iii) nevertheless the English court would not recognise the nullity decree of the Maltese court in the present case, because it
was unjust that a system of law should seek to impose as a condition of marriage in another country that the marriage should take
place according to the tenets of a particular faith (Formosa v Formosa, [1962] 3 All ER 419, applied; see p 57, letter g, and p 58,
letter a, post); the marriage was, therefore, subsisting and, cruelty and constructive desertion on Ls part being established, the
court would, in the particular circumstances of the case, exercise its discretion in favour of the wife and grant her a decree nisi of
divorce.

Notes
As to the recognition of a foreign decree of nullity where the marriage was celebrated in England, see 7 Halsburys Laws (3rd
Edn) 117, para 206; as to domicil as the test of jurisdiction, see ibid, 118, para 208, note (r); and for cases on the subject, see 11
Digest (Repl) 485, 486, 10981107.

Cases referred to in judgment


Apt (orse Magnus) v Apt [1947] 2 All ER 677, [1948] P 83, 177 LT 620, 27 Digest (Repl) 67, 459.
Bank of Hindustan, China and Japan, Re, Campbells Case, Hippisleys Case, Alisons Case (1873), 9 Ch App 1, 43 LJCh 1, 29
LT 524, 21 Digest (Repl) 225, 228.
Blamey v Blamey [1902] WN 138, 22 Digest (Repl) 528, 5902.
Chapelle v Chapelle [1950] 1 All ER 236, [1950] P 134, 11 Digest (Repl) 485, 1101.
Chetti v Chetti [190810] All ER Rep 49, [1909] P 67, sub nom Venugopal Chetti v Venugopal Chetti 78 LJP 23, 99 LT 885, 11
Digest (Repl) 460, 941.
Corbett v Corbett [1957] 1 All ER 621, [1957] 1 WLR 486, 3rd Digest Supp.
De Bono v De Bono [1948] 2 SALR 802.
De Reneville v De Reneville [1948] 1 All ER 56, [1948] P 100, [1948] LJR 1761, 11 Digest (Repl) 479, 1075.
Estin v Estin (1947), 334 US 541.
Formosa v Formosa [1962] 3 All ER 419, [1962] 3 WLR 1246.
Harrison v Harrison (1910), 54 Sol Jo 619.
Har-Shefi v Har-Shefi [1953] 1 All ER 783, [1953] P 161, [1953] 2 WLR 690, 3rd Digest Supp.
Igra v Igra [1951] P 404, 11 Digest (Repl) 518, 1324.
Kenward v Kenward [1950] 2 All ER 297, [1951] P 124, 11 Digest (Repl) 460, 942.
Merker v Merker [1962] 3 All ER 928, [1962] 3 WLR 1389.
Ogden v Ogden [19047] All ER Rep 86, [1908] P 46, 77 LJP 34, 97 LT 827, 11 Digest (Repl) 357, 260.
R v Wilkes (1769), 4 Burr 2527, 19 State Tr 1075, sub nom Wilkes v R, 4 Bro Par Cas 360, Wilm 322, 2 ER 244, 14 Digest (Repl)
375, 3674.
Ross Smith v Ross Smith (orse Radford) [1962] 1 All ER 344, [1962] 2 WLR 388.
Salvesen (or Von Lorang) v Austrian Property Administrator [1927] All ER Rep 78, [1927] AC 641, 96 LJPC 105, 137 LT 571,
11 Digest (Repl) 478, 1069.
Travers v Holley and Holley [1953] 2 All ER 794, [1953] P 246, [1953] 3 WLR 507, 3rd Digest Supp.
50
Vassallo v Vassallo [1952] SASR 129.
Williams v North Carolina (1942), 317 US 287.

Petition
In this case the wife, Hilda Lepre, by petition presented on 22 December 1960, sought (i) a declaration that a decree of nullity
pronounced by the Civil Court of Malta on 28 March 1960, [in respect of her marriage in a register office in Portsmouth to the
respondent Joseph Lepre] was invalid and of no effect in the United Kingdom and (ii) a decree of divorce on the ground of the
husbands cruelty and desertion. No prayer for discretion was included in the petition at that time. The husband acknowledged
service and the suit came before Collingwood J on 13 July 1961, when the case was adjourned for argument by the Queens
Proctor. On 16 October 1961, the Queens Proctor was granted leave to intervene in the suit and by his plea dated 25 October
1961, the Queens Proctor alleged that the wife had attempted to obtain a decree contrary to the justice of the case by her failure
to disclose that since 1 January 1957, she had lived and cohabited and frequently committed adultery with one Albert Voitasik.
By her answer to the plea the wife admitted that on five or six occasions she had committed adultery with Albert Voitasik, but
otherwise she denied the allegations in the plea; and she asked for the discretion of the court to be exercised in her favour and
filed a discretion statement. Collingwood J was unable through illness to resume the hearing of the case which now came before
Sir Jocelyn Simon P, His Lordship having made an order under RSC, Ord 72, r 4, nominating himself to try the case. The facts
appear in the judgment.

W A L Raeburn QC and F R N H Massey for the wife.


Brian T Neill for the Queens Proctor.
The respondent did not appear and was not represented.

5 December 1962. The following judgment was delivered.

SIR JOCELYN SIMON P. Here is another of those distressing cases where a domiciled Maltese of the Roman Cathoic faith has
married in a register office. The petitioner, whom I will call the wife prays, first, for a declaration that a decree of nullity of her
marriage pronounced by the Civil Court of Malta on 28 March 1960, is invalid and of no effect in the United Kingdom; secondly,
that her marriage may be dissolved on the ground that the respondent, whom I will call the husband, has treated her with
cruelty and has deserted her; and, thirdly, for ancillary relief. By amendment she prays that the courts discretion to pronounce
the divorce may be exercised in her favour.
The wife was born in this country and has lived most of her life here. She was married first to a Mr Shaw, a domiciled
Englishman. The husband was born and brought up in Malta, and baptised according to the rites of the Roman Catholic Church.
Between 1925 and 1926 he was employed at the naval dockyard at Malta. At some time between 1926 and 1944 he came to the
United Kingdom, and had a cafe in Cardiff; but he got into trouble with the police and left the country. Between May, 1944 and
July, 1946, he was in Malta, and then in Alexandria, and then back in Malta, in each case being employed by the royal navy in
their dockyard establishments. In August, 1946, he came to this country again, and started working at the Portsmouth dockyard.
He there met the wife, who was by then living apart from Mr Shaw, and within a short time the husband and the wife had set up a
joint establishment. On 24 November 1947, a child was born, called David. Some time before 27 April 1951, the husband
abandoned the wife for a time, and she obtained an affiliation order against him: however, they soon came together again. On 17
March 1952, the wife obtained a decree absolute of divorce from Mr Shaw. She and the husband did not, however, get married at
once, though they continued to live together. In April, 1955, the parties decided to go to Malta. The wife had for years been
asking the husband to marry her, and he now agreed. On 16 April 1955, the wife and the husband went through a ceremony of
marriage at a register office at Portsmouth. If English matrimonial 51 law were the only matter to be considered, there could be
no question but that this was a valid marriage. However, even before the wife and the husband had left this country he was
telling her that they were not really married, since he was a Roman Catholic and his church did not recognise a marriage in a
register office.
The parties, together with David, arrived in Malta on 20 May 1955. The husband continued to taunt the wife that she was
not really married. Nor, according to her account, was that the sum of his ill-treatment of her. He repeatedly abused her, accused
her falsely of immoral conduct and struck her. In September, 1956, he told the wife that since she did not like life in Malta, he
would pay her fare and Davids to England, hand her 100 when they got to the airport to give her a start in England, and pay her
2 a week for David when she got there. The wife jumped, as she said, at the chance of returning to England. The husband did
pay the fares; but when the parties arrived at the airport he refused to give the wife any money. Its a good riddance to both of
you, he said. Thats the last you will see of me. The wife has never heard from him since, though he did send two letters to
David. He has provided no support for either of them.
On 2 August 1957, the wife complained to the Portsmouth magistrates that the husband had deserted her, and they made a
maintenance and custody order in her favour. That order, however, could not be enforced against the husband until it had been
registered and confirmed in the Malta courts. The wife, therefore, through local agents, started proceedings to procure this. On 2
December 1957, the Maltese court adjourned the wifes application, the husband having alleged that he was not married to her:
his ground for that assertion was that he was a Roman Catholic Maltese domiciled in Malta and by his personal law he could not
contract a valid marriage anywhere in the world except in the manner stipulated by the canon lawthat is, within the context of
the present case, before the local parish priest or his nominee in a Roman Catholic church; the register office marriage was
therefore invalid.
The Maltese court, as I have said, adjourned the wifes application and directed the husband to take proper nullity
proceedings before the appropriate court to have his marriage declared void. On 16 December 1957, the husband issued his writ
in the civil court of Malta praying for a decree of nullity on the ground that his marriage did not comply with the canon law. The
court appointed a procurator and an advocate to represent the wife, and her evidence on certain matters was taken on commission
in this country. On 29 October 1959, before the proceedings had concluded, the Legitimacy Act, 1959, came into force here. By
that Act the child David became legitimated, provided that the husband had an English domicil at the time that he married the
wife. On the evidence adduced before me I was not satisfied that he had ever acquired an English domicil; but it is unnecessary
and indeed inappropriate to come to a final conclusion on this, affecting as it may Davids status but having no materiality to my
mind as to whether the marriage is a subsisting one. Even if, contrary to my present judgment, the husband had acquired an
English domicil of choice, I am satisfied that he had reverted to his Maltese domicil of origin by the time that the proceedings
there had started.
On 28 March 1960, the Maltese court pronounced a decree of nullity on the ground that the marriage had failed to comply
with the canon law. It held that the husband was at all times domiciled in Malta, and that by the law of his Maltese domicil he
had an incapacity to contract a marriage otherwise than in accordance with canon law. In view of the emphasis placed by counsel
for the wife on the effect of the Maltese decree on the status of the child, I may say that even if David had been legitimated by the
Act of 1959, I am by no means convinced that he would be bastardised by the Maltese decree. The ceremony, though devoid in
the eye of Maltese law of permanent legal effect as between the husband and the wife, may yet by that law have amounted to a
putative marriage so as to legitimate their offspring. That appears to have been the view of the Maltese court in the comparable
case of Chapelle v Chapelle, the Maltese proceedings 52 relating to the child being referred to in a footnote to an article by Mr J
K Grodecki in (1958) 74 LQR at p 233, n 50.
[His Lordship then referred to the pleadings of the wife and of the Queens Proctor and continued:] A preliminary question
arises as to the jurisdiction. There is no difficulty about the prayer for divorce. The wife was resident in England at the
commencement of the proceedings and had then been continuously resident here for over three years; the husband was then and
is now domiciled in Malta. I therefore have jurisdiction by the Matrimonial Causes Act, 1950, s 18(1)(b). But that provision,
though embracing proceedings for nullity, does not extend to proceedings for a declaration under RSC, Ord 25, r 5, as to the
international efficacy of a foreign judgment of nullity. If I am to make such a declaration I must seek jurisdiction on some other
than the statutory ground. In Har-Shefi v Har-Shefi the Court of Appeal held that the court had jurisdiction to make a declaration
under RSC, Ord 25, r 5, where the party seeking such a declaration is domiciled in England. Jurisdiction is, however, tested on
the assumption that the suitor is entitled to the relief claimed. Thus in Har-Shefi v Har-Shefi the wife petitioner was praying for a
declaration that the marriage had been validly dissolved by a foreign court: if successful she would have been domiciled in
England. In Merker v Merker, the foreign decree of nullity for recognition of which the petitioner was praying would have had a
similar effect. But if the wife here is entitled to the declaration she seeks, namely, that the Maltese decree was inefficacious to
annul her marriage, she was not domiciled here at the commencement of this suit, nor is she so domiciled today: she was and is
dependently domiciled in Malta. It may be thought artificial that jurisdiction to declare whether a foreign judgment is to be
recognised should depend on whether the petitioner is putting the claim in the affirmative or the negative; and there seems much
to be said for the contention, persuasively argued by Mr J A C Thomas b and by Mr I M Sinclairc, that the court has jurisdiction to
make such a declaration where the party seeking it is no more than resident in England, though no doubt the discretion to do so
would be exercised with caution.
________________________________________
b (1953), Vol 2 International and Comparative Law Quarterly, p 444
c (1953), 30 British Year Book of International Law, p 524

It is, however, unnecessary to decide that in the present case, as I am satisfied that I have jurisdiction on another ground. I
cannot proceed to consider whether the wife should be granted a decree of divorce without ascertaining whether there is a
subsisting marriage to dissolve, and this involves determining whether the Maltese decree should be recognised as an effective
judgment of nullity. In my view, where determination of the validity of a foreign judgment is a necessary step in proceeding to
adjudication on a matter within the jurisdiction of the court, the court has jurisdiction under RSC, Ord 25, r 5, to make a
declaration on such validity. It is then doing no more than to give formality to what is inherent in and clearly deducible from a
judgment which the court has jurisdiction to pronounce, so as in any event to constitute res judicata: see Alisons Case, per
Mellish LJ ((1873), 9 Ch App at p 25).
I have, therefore, to consider initially whether the decree of the Maltese court of 28 March 1960, should be recognised in
England as annulling the marriage. That itself raises two questions: first, should such a decree in principle be recognised in the
absence of fraud or offence against our notions of justice; secondly, does the decree in the present case in fact offend against our
notions of justice?
On the first question differing views of great weight have been expressed. On the one side stand Willmer J, in Chapelle v
Chapelle and Lord Denning MR and Donovan LJ, in Formosa v Formosa: they would not recognise the decree as effective to
annul the marriage. On the other side are Pearson LJ, 53in Formosa v Formosa, Herbstein J, in De Bono v De Bono, Reed J, in
Vassallo v Vassallo and the distinguished academic figures who have commented on Chapelle v Chapelle: they hold that the
decree ought to be recognised. Those who hold the first view argue in this way: the Maltese court can make a binding and
conclusive decree annulling the marriage if both parties were domiciled in Malta at the commencement of the suit there (Salvesen
(or Von Lorang) v Austrian Property Administrator); the Maltese decree, however, declared the marriage void; the wife,
therefore, never acquired the husbands Maltese domicil by operation of the law, but retained her English domicil throughout; it
follows that the decree of the Maltese court was not a decree of the common domicil and was not binding and conclusive. Those
who take the contrary view, that we should recognise the decree, retort that such a refusal to recognise the Maltese decree as
effectively annulling the marriage involves that the parties remain married in the eye of English law; the wife had thus at the
commencement of the Maltese proceedings a domicil in law dependent on her husbands, which was then admittedly Maltese;
both parties were, therefore, domiciled within the jurisdiction of the Maltese court, which in consequence had jurisdiction over
their marriage. The views expressed by the Court of Appeal in Formosa v Formosa were obiter on this part of the case. I must,
therefore, proceed to give my own judgment guided but not governed by what has been said elsewhere.
The apparent dilemma is, in my view, to be resolved by, first, carefully isolating the legal system to which reference should
be made in order to ascertain whether the Maltese court had a conclusive jurisdiction, and, secondly, bearing constantly in mind
the crucial time for invoking such legal system. That time is unquestionably the commencement of the Maltese proceedings. The
husband being then domiciled in Malta, where was the wife domiciled? This depends on whether the marriage was void or
voidable or valid in the eye of the legal system which should be invoked: if valid or voidable, the wife remained married to the
husband until the pronouncement of the decree of nullity, and therefore took his domicil until that event; if void, she had no legal
tie with the husband and had her own domicil throughout: see De Reneville v De Reneville, per Lord Greene MR ([1948] 1 All
ER at p 60; [1948] P at p 111). In his judgment in that case, in which Somervell LJ concurred, Lord Greene MR also indicated,
first, the meaning of void and voidable in this connexion and, secondly, what legal system should be invoked to determine
whether the marriage was void or voidable. As to the former he said ([1948] 1 All ER at p 60; [1948] P at p 111):

A void marriage is one that will be regarded by every court in any case in which the existence of the marriage is in
issue as never having taken place and can be so treated by both parties to it without the necessity of any decree annulling it:
a voidable marriage is one that will be regarded by every court as a valid subsisting marriage until a decree annulling it has
been pronounced by a court of competent jurisdiction.

And again ([1948] 1 All ER at p 62; [1948] P at p 115):


it would be for the English court, after hearing evidence of French law, to decide whether in French law the
marriage was void or voidable, not merely in a verbal sense, but in the sense of the words as understood in this country,
that is, as indicating or not indicating, as the case might be, that the marriage would be regarded in France as a nullity
without the necessity of a decree annulling it.

Lord Greenes reference to French law in the latter passage results from his conclusion as to the legal system that should be
invoked; as to that he said ([1948] 1 All ER at p 61; [1948] P at p 114):
54

the question whether the marriage is void or merely voidable is for French law to answer. My reasons are as
follows. The validity of a marriage so far as regards the observance of formalities is a matter for the lex loci celebrationis.
But this is not a case of forms. It is a case of essential validity. By what law is that to be decided? In my opinion, by the
law of France, either because that is the law of the husbands domicil at the date of the marriage or (preferably, in my view)
because at that date it was the law of the matrimonial domicil in reference to which the parties may have been supposed to
enter into the bonds of marriage.

There is no question but that our law characterises the marital defect alleged in the present casethat the ceremony did not take
place in a Roman Catholic church in the presence of a priestas a matter of formalities. Its legal result, says Lord Greene, must
in these circumstances be referred to the lex loci celebrationis; that is, to English law. English law says without any doubt that
the marriage was neither void nor voidable but valid. The wife and the husband were, therefore, married at the commencement
of the Maltese proceedings; the wife had acquired and retained the husbands Maltese domicil; the decree was a judgment of the
court of common domicil and should, therefore, be recognised here as binding and conclusive.
De Reneville v De Reneville was concerned with the jurisdiction of the English court to pronounce a decree of nullity.
Where the jurisdiction of a foreign court is in question it may reasonably be argued that we should pay regard not solely to our
own characterisation of the nature of the defect in question, but also to how it is characterised by the foreign court. The same
result, however, ensues. I had no expert evidence of Maltese law; but it is clear from the terms of the Maltese judgment that they
characterised the defect not as one of formalities but as affecting the husbands capacity to marry, and thus referable to the law of
his Maltese domicil. I conclude, however, from the proceedings of the Maltese courts that in Maltese law the marriage was
voidable and not void, in the senses indicated by Lord Greene MR. The wifes maintenance proceedings in Malta were not
dismissed out of hand on the ground that she was not the wife of the husband; on the contrary, they were adjourned so that the
husband could question the validity of the marriage in separate and appropriate proceedings: that, indeed, seems to be a common
feature of these cases. The marriage thus being voidable and not void in Maltese law, the parties had a common domicil in Malta
at the commencement of the Maltese proceedings, and the decree of nullity is binding and conclusive. In truth, there is no real
difficulty in the present case: all the systems of law to which reference could conceivably be madethe lex loci contractus, the
lex domicilii of the husband, the leges domicilii of the wife, the lex causae and the lex foriconcur at the time when the Maltese
proceedings started that the wife was married to the husband and domiciled with him in Malta. If that is so, then, whatever the
words of the decree, it is inadmissible to relate it back so as to destroy the basis of the jurisdiction to make it. To cite again Lord
Greene MR, in De Reneville v De Reneville ([1948] 1 All ER at p 60; [1948] P at p 111):

The fact that a domicil has been acquired by reason of a voidable marriage is a fact the existence of which cannot be
undone by a declaration of nullity.

A fortiori if the marriage is valid by our choice of law rule. To allow otherwise involves a circular argument from which there is
no escape.
But even if this marriage were void ipso jure, so that the husband alone was domiciled in Malta at the start of the
proceedings there, in my judgment we should still accord recognition to the Maltese decree. In the case of a marriage void ipso
jure, such as a marriage fundamentally defective as to formalities, the English court assumes jurisdiction in nullity if the
petitioner alone is domiciled in England: 55De Reneville v De Reneville; Apt (orse Magnus) v Apt; Kenward v Kenward, to cite
only authorities in the Court of Appeal. Moreover, in such circumstances we purport to operate on the status not only of the
petitioner who is domiciled within the jurisdiction but also of the respondent who is not; it is for this reason that we insist that he
or she should be made a party to the proceedings, so as to be bound by our decree. If we ourselves claim a ground of jurisdiction
we must concede a similar ground of jurisdiction to foreign courts: Travers v Holley and Holley, Corbett v Corbett. Therefore
even if the wife were, contrary to my view, domiciled in England at the start of the Maltese proceedings by reason of the nullity
of the marriage, we should none the less concede recognition to the Maltese decree, because we would regard ourselves as
competent to pronounce a decree of nullity of a marriage void ipso jure were the husband domiciled in England and the wife in
Malta. In as far as Ogden v Ogden appears to be to the contrary, it is in my judgment in conflict with the reasoning of the later
authorities in the Court of Appeal to which I have referred and must be taken to be confined to its particular factsif, indeed, it
can today stand at all in the light of the House of Lords decisions in Salvesen (or Von Lorang) v Administrator of Austrian
Property and Ross Smith v Ross Smith (orse Radford). Furthermore such assumption and concession of a binding jurisdiction in
nullity based on the domicil of one party only seems to me to accord with principle. A judgment declaratory of the status of some
subject-matter legally situated within the national and jurisdiction of the court pronouncing the judgment constitutes a judgment
in rem which is universally conclusive. The husband was legally situated within the jurisdiction of the Maltese court because he
was domiciled in Malta. That court was, therefore, competent to declare his status by a decree of nullity; such a decree
constitutes a judgment in rem, and should be regarded universally as conclusive as to his status, that is to say, that he is
unmarried. Counsel for the wife conceded that for the purpose of criminal proceedings in this country we would be bound so to
regard him; but not, he said, for the purpose of matrimonial proceedings, because the wife (on the present hypothesis) was not
domiciled in Malta, so that her status was not within the competence of the Maltese court. The wife therefore, it is claimed,
remains married to the husband, even though he is not married to her: the concept is no more difficult of acceptance than a
finding that a respondent has committed adultery with a co-respondent, but not he with her. Such schizoid situations reflect little
credit on the law, though the latter one is reasonably based on differential admissibility of evidence. But I cannot conceive how
our courts could accept as conclusive the decree of a competent court of the husbands domicil that he is unmarried and, at the
same time, purport to dissolve a marriage to which he is the other party. Moreover, there is high persuasive authority to suggest
that this contention for the wife is not correct: see Williams v North Carolina, a decision of the Supreme Court of the United
States, where comparable recognition problems arise after divorce owing to their rule that a wife retains during marriage a
domicil independent of her husbands. Incidentally, further authority from the same eminent source indicates that recognition of
the nullity decree of the court of the husbands domicil only ought not necessarily to exclude our courts from enforcing in favour
of the wife in appropriate circumstances financial obligations arising out of the marriagefor example, under a prior
maintenance order made in this country: see Estin v Estin.
Therefore, in my judgment, we should accept the Maltese decree as binding and conclusiveprimarily as a decree of the
court of the common domicil at the commencement of the proceedings there, though alternatively as a decree of 56 the husbands
domicil alone at that timeprovided always that it is not vitiated by fraud or contrary to natural justice.
That brings me to the second main issue in this part of the case. I confess that I approach it with some misgiving. We are
concerned here with the decree of a superior court of a Commonwealth country. Its procedure was manifestly solicitous of the
forensic interests of the wife. The code of law applied was an ancient and honoured one. Moreover, limping marriages are
themselves inherently liable to cause hardship and injustice: suppose, for example, the wife had remarried and had offspring in
reliance on the Maltese decreeor, for that matter, the husband. The refusal to recognise an otherwise binding foreign judgment
or rule of law on the ground that it is manifestly unjust is nowadays put as a matter of discretion.

But discretion, when applied to a court of justice, means sound discretion guided by law. It must be governed by rule,
not by humour: it must not be arbitrary, vague, and fanciful; but legal and regular.

See R v Wilkes, per Lord Mansfield ((1769), 4 Burr at p 2539). In short, there must be a reasonable consistency in its exercise. In
Corbett v Corbett, Barnard J recognised a foreign decree of nullity based on two grounds, one apparently identical with that
which constituted the defect in the present case, the otherthat a Jewess was incapable of marrying out of her faithdifficult to
differentiate in principle: see also Igra v Igra. But in Formosa v Formosa, the Court of Appeal, though not expressly adverting
to Corbett v Corbett, was unanimous that a decree pronounced by a Maltese court on a ground identical with that in the present
case and in largely similar circumstances offended so grossly against our notions of justice that it should not be recognised. It is
true that the present case differs in certain details from Formosa v Formosa. The husband here was not proved to have acquired
an English domicil at the time of the marriage, and I am not satisfied that the child would be adversely affected by recognition of
the decree. Corbett v Corbett was similar to the present case in the latter respect, and possibly in the former. But to differentiate
the present case and Corbett v Corbett from Formosa v Formosa on these grounds would be, in my opinion, to introduce idle
distinctions into the law and throw it into confusion. Not least in matters relating to marriage is it incumbent on the law to speak
with a clear, consistent and unequivocal voice. In truth, I do not believe that it was mere cumulation of detail which impelled the
Court of Appeal to their conclusion. I think that the crux of their decision was that it was an intolerable injustice that a system of
law should seek to impose extra-territorially, as a condition of the validity of a marriage, that it should take place according to the
tenets of a particular faith. As Donovan LJ put it ([1962] 3 All ER at p 423):

It ill accords with present-day notions of tolerance and justice that a wife, validly married according to our law, should
be told by a foreign court that she is a mere concubine and her children bastards, simply on the ground that her husband did
not marry her in the church of a particular religious denomination.

I cannot believe that the effect on the status of the children of the union was a decisive factor in the mind of Donovan LJ nor the
further element which he took into account ([1962] 3 All ER at p 423), namely, that the decree was

pronounced in favour of a husband who deserted her and his children and has left them to be supported by the
taxpayer in this country.

In any event that is common to the present case. Just as in Chetti v Chetti, Sir Gorell Barnes P, refused to give effect to an
incapacity to marry outside his caste or religion imposed extra-territorially on the husband by the law of 57 his domicil, so, I
think, the Court of Appeal discerned in Formosa v Formosa an attempt by Maltese law to impose an analogous incapacity based
on creed: they would refuse to recognise the incapacity, so they refused to recognise the domiciliary decree founded on it. If that
is so, the present case cannot be distinguished; and I am bound to hold that the Maltese decree of nullity, although on general
jurisdictional grounds conclusive, should not be accorded recognition because it offends intolerably against the concept of justice
which prevails in our courts.
It follows that the marriage is valid and subsisting. The wife petitions to dissolve it on the ground of the husbands cruelty
and desertion. As for cruelty, I accept her evidence. It is corroborated by an affidavit sworn by Mrs Zarb, the husbands sister,
who lives in Malta: though not entitled in this suit, it bears the stamp of the Government of Malta, and I therefore admitted it on
the authority of Blamey v Blamey, and the cases there cited. Mrs Zarb says that the husband kept the wife so short of money in
Malta that she (Mrs Zarb) used to have to make clothes for her and lend her money even for her public transport fares. On one
occasion she saw the wife in tears and bruised. Just before the wife left for England Mrs Zarb was told by the husband: It is
better that she should leave because if she remains here I shall suffocate her, meaning, I suppose, strangle her. Mrs Zarb also
bears out that the husband sent the wife back to England without a penny. Having also in mind what happened after the parting, I
am satisfied that the husband treated the wife with cruelty.
As for desertion, I have no difficulty in drawing the inference that the husband intended to drive the wife from cohabitation.
In so far as there may have been a residual consent by the wife to the separation, in my view it was procured and nullified by the
fraudulent misrepresentation of the husband as to his intentions of financial support: compare Harrison v Harrison. I, therefore,
hold that the husband deserted the wife and has remained in desertion up to the date of the petition.
There remains the difficult problem of the exercise of the courts discretion in favour of the wife. When she originally filed
her petition, she did not pray for discretion at all. When she did file her discretion statement it was far from a full disclosure of
her adultery. Even in the witness-box she told untruths about it, and it was only as a result of close questioning that she came
finally to disclose the adultery which is now set out in the amended discretion statementwith another man as well as Mr
Voitasik, and a more extensive course of adultery with Mr Voitasik than she admitted at first. The wife is a stupid woman, and I
have also no doubt that one of the reasons for her non-disclosure was that she is bitterly ashamed of her misconduct. I think that
she has now disclosed its full extent. She is anxious to marry Mr Voitasik, and he wants to marry her. This marriage has been
unequivocably repudiated by the husband, and no useful purpose would be served by keeping it in further subsistence. I am,
therefore, though I confess with some hesitation, prepared to exercise the discretion of the court in favour of the wife and to grant
her a decree nisi of divorce.

Decree nisi.

Solicitors: H M Gammans, Portsmouth (for the wife); Queens Proctor.

A T Hoolahan Esq Barrister.


58
[1963] 2 All ER 59

Good v Parry
CIVIL PROCEDURE

COURT OF APPEAL
LORD DENNING MR, DANCKWERTS AND DAVIES LJJ
26, 27 FEBRUARY 1963

Limitation of Action Acknowledgment Debt Claim Limitation Act, 1939 (2 & 3 Geo 6 c 21), s 23(4).

For there to be an acknowledgment of a claim within s 23(4) of the Limitation Act, 1939 a, there must be an admission that
there is a debt or other liquidated amount outstanding and unpaid (see p 61, letter e, and p 62, letters c and e, post).
________________________________________
a Section 23(4) is set out at p 61, letter b, post

In 1949 a landlord let premises to a husband and wife. The husband died and the wife carried on the tenancy. In 1962 the
landlord brought an action claiming more than six years arrears of rent. In 1957 the tenants agent wrote a letter to the landlord
during negotiations for the proposed purchase of the premises by the tenant, in which it was stated that the question of
outstanding rent can be settled as a separate agreement as soon as you present your account. The landlord contended that this
acknowledged the claim for arrears of rent for the purposes of s 23(4) of the Limitation Act, 1939 b, with the consequence that
time would not begin to run until the date of the letter.
________________________________________
b Section 23(4) is set out at p 61, letter b, post

Held The letter was not an acknowledgment within s 23(4) of the Limitation Act, 1939, because there was in it no admission of
any defined amount of rent or of any amount that could be ascertained by calculation being due, nor, indeed, any admission that
there was such a debt in fact (see p 62, letters a, d and e, post).
Skeet v Lindsay ((1877), 2 Ex D 314), and Langrish v Watts ([1903]1 KB 636), considered.
Appeal dismissed.

Notes
As to an action to recover a debt, see 24 Halsburys Laws (3rd Edn) 298, para 590, and as to the form of acknowledgment of debt,
see ibid, pp 299, 300, para 593.
For the Limitation Act, 1939, s 23, see 13 Halsburys Statutes (2nd Edn) 1184.

Cases referred to in judgments


Jones v Bellgrove Properties Ltd [1949] 2 All ER 198, [1949] 2 KB 700, 65 TLR 451, 93 Sol Jo 512, 2nd Digest Supp.
Langrish v Watts [1903] 1 KB 636, 72 LJKB 435, 88 LT 443, 51 WR 503, 19 TLR 359, 32 Digest 370, 541.
Skeet v Lindsay (1877), 2 ExD 314, 36 LT 98, 41 JP 456, 25 WR 322, sub nom Skeat v Lindsay, 46 LJQB 249, 32 Digest 373,
566.
Whitcomb v Whiting (1781), 2 Doug KB 652, 1 Sm LC 635, 32 Digest 353, 362.

Appeal
This was an appeal by the landlord from a judgment of His Honour Judge Potter, given on 5 November 1962, at the Bow County
Court, awarding the landlord 156 0s 7d arrears of rent due from the tenant, but refusing to enter judgment for the landlord for
the amount of arrears claimed, 368 16s 9d on the ground that the difference was statute-barred. The facts are set out in the
judgment of Lord Denning MR.
The cases noted belowc were cited during the argument in addition to those referred to in the judgment.
59
________________________________________
c Tanner v Smart (1827), 6 B & C 603, Moodie v Bannister, (1859), 4 Drew 432, Green v Humphreys, (1884), 26 ChD 474

F Bresler for the landlord.


S O Olson for the tenant.

27 February 1963. The following judgments were delivered.

LORD DENNING MR. This is a claim for rent in arrear. The landlord, Mr Good, let premises to Mr and Mrs Parry as long ago
as 1949. The landlord says that he has not received any rent for very many years. Mr Parry died and his widow, Mrs Parry, who
was joint tenant with him, survived and carried on. Rent of 2 3s 8d per week, it is said, became payable and was never paid.
Eventually, on 6 March 1962, the landlord brought an action against the tenant claiming 604 14s 5d, the amount of rent in
arreard as at 12 February 1962. By way of defence, the tenant said that the cause of action for arrears of rent did not accrue
within six years before the action and was barred by s 2 of the Limitation Act, 1939. It is quite plain that that Act bars any rent
which accrued due more than six years before the action was brought, subject, however, to the question whether there has been an
acknowledgment within s 23 and s 24 of the Act of 1939. The landlord has put before the court a number of letters, of which
only one arises for consideration now, which, he says, was an acknowledgment such as to prevent the statute from running. It is a
letter of 6 September 1957, written by Lieutenant John Parry, RN, the son of the tenant and undoubtedly her authorised agent. I
need not read the whole of it. It says:
________________________________________
d The figures were substantially agreed figures, but they do not correspond with the period of letting and rent without some explanation. In
July, 1961, a sum of 535 8s had been paid in respect of six years rent ending in June, 1961, after deducting Sch A tax and a sum of 49 7s
10d in respect of repairs claimed by the defendant to be deductible. The Sch A tax deducted included 24 8s 3d for the tax year 195455,
which was statute-barred. In the county court the plaintiff recovered judgment for 156 7s made up of the 24 8s 3d and 131 12s 3d,
which latter sum consisted of 82 4s 5d arrears of rent and mesne profits since the date in June, 1961, to which rent had been paid, together
with the 49 7s 10d wrongly deducted for repairs. Thus the agreed sum of 368 16s 9d (see letter f, above), which was to be the amount
recoverable on the basis that there had been acknowledgment of the indebtedness for rent within s 23(4) of the Limitation Act, 1939,
included a substantial sum in respect of rent prior to June, 1955

Dear Mr. Good, Since I have not had the opportunity of seeing you this week to discuss 11 Scotland Road, I am
writing to try and arrive at some satisfactory arrangement. [He discusses the proposed purchase of the house and says:] I
have considered the surveyors letter and am prepared to offer 1,350 as recommended for this property, subject to
contract.
Then he goes on to use this one sentence about the rent: The question of out-standing rent can be settled as a separate agreement
as soon as you present your account. It is said on the landlords behalf that that is an acknowledgment such as to take the case
out of the statute, and that he can recover the amount in arrear. It is agreed that, if it is an acknowledgment, the judgment is to be
for the sum of 368 16s 9d.
The landlord relies on two cases before the Act of 1939, namely, the decision of Cleasby B, in Skeet v Lindsay, and the
observations of Vaughan Williams LJ, in Langrish v Watts ([1903] 1 KB 636 at p 641), where he said:

Looking at the common law cases on the subject, apart from any question of a claim in equity to an account, they
appear to show that, where a defendant in effect has said that he does not think that he owes so much as is claimed, but, if
vouchers are furnished, he will pay whatever is due, that is an acknowledgment of a debt subject to the business operation
of ascertaining the amount being gone through.

It is urged by counsel on behalf of the landlord that this case comes within the 60 words of Vaughan Williams LJ. The words
here mean, he says, that, when the account is presented, the tenant will pay whatever is due, subject to the business operation of
ascertaining the amount. He points out that, at a later stage in the correspondence, some similar operation was gone through.
The law, it seems to me, is now to be taken from the Limitation Act, 1939. Section 23(4) replaces all the old law on the
subject. It says simply this:

Where any right of action has accrued to recover any debt or other liquidated pecuniary claim and the person liable
or accountable therefor acknowledges the claim or makes any payment in respect thereof, the right shall be deemed to have
accrued on and not before the date of the acknowledgment or the last payment.

Section 24 goes on to say that an acknowledgment has to be in writing and signed by the party or his agent. Clearly here the
writing and the agency is satisfied. The whole question is whether, in this letter, there is an acknowledgment of the claim. The
words the claim are not, perhaps, very happy. A person may acknowledge that a claim has been made against him without
acknowledging any indebtedness. It is clear that what the Limitation Act, 1939, means is acknowledges the debt or other
liquidated pecuniary amount. The question is: Is there such an acknowledgment? This statute alters the old law. Previously, in
order to get a case out of the statute, a creditor had to show a writing from which there could be implied a promise to pay; then
his cause of action was on the promise and not on the previous debt, and, so long as the promise was within the six years, all well
and good: see the notes to Whitcomb v Whiting. Nowadays, as the result of this new Act, there is no necessity to look for a
promise, express or implied. There need only be an acknowledgment of a debt or other liquidated amount. That means, I think,
that there must be an admission that there is a debt or other liquidated amount outstanding and unpaid. Even though the debtor
says in the same writing that he will never pay it, nevertheless it is a good acknowledgment. In order to be an acknowledgment,
however, the debt must be quantified in figures or, at all events, it must be liquidated in this sense that it is capable of
ascertainment by calculation, or by extrinsic evidence, without further agreement of the parties. For instance: I admit I owe you
the sum shown in this rent book would be a perfectly good acknowledgment, for it only needs to be calculated. Again, in Jones
v Bellgrove Properties, Ltd, the balance sheet contained the acknowledgment: To sundry creditors 7,638 6s 10d. It was
possible by extrinsic evidence to sort out the various items in that lump sum, and it was held to be a sufficient acknowledgment.
But if the debt is not quantified and is not ascertainable without further agreement, then there is no acknowledgment sufficient to
satisfy the statute.
No doubt a promise in writing by a debtor to pay whatever sum is found due on taking an account is a good
acknowledgment today just as it was before the Act of 1939, provided always that the amount is a mere matter of calculation
from vouchers, or can be ascertained by extrinsic evidence, and is not dependent on the further agreement of the debtor. If Skeet
v Lindsay and Langrish v Watts are to be regarded as good law since the Act of 1939, it can only be on the ground that the amount
was a mere matter of calculation from vouchers. I must say for myself that I feel that Skeet v Lindsay stretched the law to the
uttermost. I doubt whether it was right there to hold, as Cleasby B, seems to have done, that there was an engagement absolutely
to pay.
I come back to the sentence in this case. I agree with the submission of counsel for the tenant, which was accepted by the
county court judge, that this sentence means there may be some rent outstanding and it can be made the subject of an 61
agreement as soon as you present your account. Such being the meaning of it, I am quite satisfied that there is no
acknowledgment, because there is no admission of any rent of a defined amount due, or of any amount that can be ascertained by
calculation. The amount is uncertain altogether. Nor can I regard it as a promise to pay whatever amount may be found due on
taking an account. The tenant clearly reserves the right to examine it and not to be bound except by separate agreement. I agree,
therefore, with the county court judge and would dismiss this appeal.

DANCKWERTS LJ. I agree. The question which the learned county court judge had to decide, and which we have to decide,
is an important one, and the matter has been very fully and carefully argued by counsel before us. I agree with the judgment of
Lord Denning MR and it is really only necessary for me to add a few words. The question depends on the application of only a
few words in s 23(4) of the Limitation Act, 1939, the person liable or accountable therefor acknowledges the claim. I agree
with Lord Denning MR in his interpretation that the claim means any debt or other liquidated pecuniary claim referred to in
the earlier part of the subsection.
The only question then is whether the final sentence in the letter of 6 September 1957, written by the tenants son and on her
behalf, satisfied those words in the statute. The sentence is: The question of outstanding rent can be settled as a separate
agreement as soon as you present your account. I find it impossible to come to the conclusion that that sentence is in any way
an acknowledgment of any debt. It is merely, as it seems to me, an admission that there may be some possible justified claim, but
no admission that there is such a debt in fact. Therefore, I have come to the same conclusion as Lord Denning MR and, in my
opinion, this appeal should be dismissed.

DAVIES LJ. I entirely agree with my Lords, and only refer to one sentence in the judgment of Judge Potter in the court below.
After a very careful review of the authorities, he said this: In my judgment the letter did not acknowledge the claim; it only
acknowledged that there might be a claim. I entirely agree with that part of the county court judges judgment, as well as what
has fallen from my Lords.
I agree that the appeal fails and should be dismissed.

Appeal dismissed.

29 March. Leave to appeal to the House of Lords refused.

Solicitors: Shepherd, Harris & Co (for the landlord); Attwater & Liell (for the tenant).

F Guttman Esq Barrister.


62
[1963] 2 All ER 63

Laurent v Sale & Co (a firm)


CIVIL PROCEDURE

QUEENS BENCH DIVISION


MEGAW J
24 JANUARY 1963

Champerty Defence Assignment of chose in action void for champerty Plaintiffs title to sue depending on the assignment
Whether champerty could properly be raised by way of defence.

Where an essential link in a plaintiffs alleged title to what he claims in an action is a champertous assignment, the illegality of
the assignment for champerty may properly be raised by way of defence (see p 65, letter e, post).
Martell v Consett Iron Co Ltd ([1955] 1 All ER 481) distinguished.
By documents of assignment in 1956, made in the French language but to which English law was declared to be applicable,
K and M purported to assign to L their rights under letters dated 3 July 1953, addressed to K and M by the defendants, in which
the defendants acknowledged irrevocable instructions to pay K and M respectively at stated figures against shipments of goods.
Each document of assignment stated that the consideration for the assignment was to be the payment, by L to K or M
respectively, of a fraction of the total amount which should be paid to L by virtue of the letters of 3 July 1953. The assignments
were made with the knowledge of both the parties to them that L would not be able to enforce claims under the letters unless he
took legal proceedings against the defendants, and with the intention that L should take such proceedings. In an action by L to
enforce the claims assigned to him,

Held The assignments were void for champerty and L had no title to sue.

Notes
As to the purchase of rights of action and the unenforceability of champertous agreements, see 1 Halsburys Laws (3rd Edn) 43,
para 85, and p 41, para 83; and for cases on the unenforceability of champertous agreements, see 1 Digest (Repl) 8083, 607
624, 8587, 644666.

Case referred to in judgment


Martell v Consett Iron Co Ltd [1955] 1 All ER 481, [1955] Ch 363, [1955] 2 WLR 463, 3rd Digest Supp.

Preliminary Issue
On 3 July 1953, the defendants, Sale & Co wrote two almost identical letters, one to Mr Marcelle in Belgium and one to Herr
Kraus in Hamburg. The letters read as follows:

We confirm that against shipment of 860 metric tons of copper wire from Antwerp in respect of [a named company],
we have received irrevocable instructions to pay you $10 [or, in the letter to Herr Kraus, $5] per metric ton shipped and
paid for.

It appeared that on the same date the defendants also wrote a similar letter to a Mr Zehnder in Geneva. On 23 April 1956, Mr
Zehnders solicitors (with whom the plaintiffs solicitor was at that time associated) wrote to the defendants solicitors stating that
they were instructed in connexion with a claim of Mr Zehnder under the defendants letter of 3 July 1953. On 24 April 1956, the
defendants solicitors wrote in reply denying liability. By a document, dated 25 June 1956, made in the French language but
stating that English law should apply thereto Herr Kraus purported to assign to the plaintiff, Ferdinand Laurent, the right to the
sum, namely $4,300, which he claimed to be due to him from the defendants by virtue of their letter dated 3 July 1953, in
consideration of a payment by the plaintiff to Herr Kraus of one quarter of the total amount which should be recovered. A similar
document, except that the amount of the claim was stated to be $8,600, was executed by Mr Marcelle on 8 November 1956, and
63 supplemented by a further document dated 12 June 1959. On 24 June 1959, the plaintiffs solicitor wrote enclosing notice of
the two documents of assignment to the defendants solicitors, and informed them that unless the claims were admitted and
settled proceedings would be instituted against the defendants forthwith. On 1 July 1959, the writ in the action was issued. By
his statement of claim indorsed on the writ the plaintiff claimed 6,450, being the equivalent of $8,600 and $4,300 due
respectively to Mr Marcelle and Herr Kraus and confirmed in writing by the defendants by letters dated 3 July 1953, addressed to
Mr Marcelle and to Herr Kraus. Paragraph 5 and para 6 of the particulars of the statement of claim were as follows:

5. On Nov. 8, 1956, the said Marcelle absolutely assigned to the plaintiff the said debt of $8,600 due to him from the
defendants by an assignment in writing and a deed of assignment expressed to be supplemental thereto dated June 12,
1959.
6. On June 25, 1956, the said Kraus absolutely assigned to the plaintiff the said debt of $4,300 due to him from the
defendants by an assignment in writing.

On 1 October 1959, the defendants delivered their defence, by para 5 of which they pleaded as follows:

5. The defendants do not admit the assignments referred to in paras. 5 and 6 of the statement of claim. If (which is
not admitted) there was any such assignment, the same was and is illegal and champertous, (a) as being an assignment of a
bare right of litigation; and further or in the alternative (b) in that it was an assignment in consideration of a promise by the
assignee to pay to the assignor a proportion of the amount recovered in proceedings to enforce the chose in action thereby
assigned.

By order of Master Clayton, for Master Grundy, dated 26 June 1962, the issue raised by para 5 of the defence was ordered to be
tried as a preliminary issue. His Lordship (Megaw J) found on the facts that the assignments were champertous. The case is
reported for his decision on the point whether the fact that the assignments were champertous could be raised as a defence to the
action.
N Inglis Jones for the plaintiff: First, an agreement is not champertous merely because it involves the transfer of a debt,
which is due to A, and provides that the consideration or part of the consideration of that transfer shall be that A will have a
part of the proceeds of the debt when these proceeds are received by B to whom the debt is transferred. Second, the doctrine of
champerty should not be extended, and, in order that there should be champerty, there must be a lawsuit in the offing; no lawsuit
was in the offing in the present case.
Ronald Bernstein and J S Colyer for the defendants: It is submitted (i) that the plaintiff has no cause of action independently
of the assignments, and consequently (ii) that, if the assignments are champertous, they are illegal or void and no cause of action
arises from them; (iii) that an agreement between a claimant and a stranger whereby the stranger agrees to finance the prosecution
of a claim in consideration of a share of the proceeds is champertous, and (iv) it is immaterial in the case of such an agreement
which of the two parties to it thereafter becomes a plaintiff in an action.

24 January 1963. The following judgment was delivered.

MEGAW J stated the substance of the letters of 3 July 1953, and the material provisions (in an English translation) of the French
document of assignment dated 25 June 1956, read the letter of 24 June 1959, giving notice of assignment, reviewed the course of
events relevant to the claim and referred to the writ and statement of claim. His Lordship then read para 5 of the defence,
mentioned the order dated 26 June 1962, directing the trial of the issue thereby raised as a preliminary issue and stated that he had
not heard argument on sub-para. (a) of para 5 as he was satisfied that sub-para. (b) concluded the issue in 64 favour of the
defendants. His Lordship stated the four propositions that were submitted by counsel for the defendants (as set out at p 64, letter
h, ante), and said that propositions (i) and (ii) were not disputed by the plaintiff and that he (His Lordship) agreed with
propositions (iii) and (iv). His Lordship referred to the submissions on behalf of the plaintiff, agreed with the first submission as
stated at p 64, letter f, ante, and accepted, as regards the second submission, that the doctrine of champerty should not be
extended. His Lordship found on the evidence, however, that the the proper inference to draw was that the assignments in June
and November, 1956, were made by the assignors with the knowledge and intention, shared by the plaintiff, that legal
proceedings would be necessary if anything were to be recovered from the defendants, and that the plaintiffs intention in taking
the assignments was to seek to enforce by litigation against the defendants the supposed rights under the letters of 3 July 1956.
His Lordship found that both the assignments were plainly champertous agreements, and continued:] The assignments being
champertous agreements, it is quite unnecessary for me to go into the various authorities which have been cited. It might have
been of importance had I reached a different conclusion on the facts as to what was in the minds of the parties in making these
assignments. Counsel for the defendants has fairly pointed out that in the case of Martell v Consett Iron Co Ltd, it was held that
the mere fact that an action was being maintained was not a defence. It might be a matter which would arise thereafter, but it
could not be raised by way of defence to the action. In my view, the position is entirely different where the plaintiff in the action
is one who does not have any original title in respect of the matter which he claims. He then has to show how he comes to the
title to be plaintiff at all. That is an essential part of his cause of action. Here the step necessary to establish his title before he
can begin to present the rest of his case is these two assignments from Marcelle and Kraus. If the point be taken by the
defendants, as it has been taken here, that those assignments are champertous agreements, then, in my view, that is a matter which
can properly be taken by way of defence and has to be considered by the court as part of the defence, in the same way as any
other defects, that might be alleged in the assignment on which a plaintiff had to rely to show his title, might be raised by way of
defence.
The mere fact that champerty, if it exists, is illegal and, indeed, can be criminal, is certainly not a reason for refusing to
consider it as part of the defence to an action where, if the agreement is champertous and illegal, it destroys a necessary step in
the plaintiffs title in the action. I am therefore satisfied that the decision in Martell v Consett Iron Co Ltd is not a bar to the
course, which has been taken here, of pleading in the defence that this is a champertous agreement. Accepting as I do the four
propositionsa put forward by counsel on behalf of the defendants, it follows that I decide this issue in favour of the defendants.
________________________________________
a See p 64, letter h, ante

Judgment for the defendants on the issue.

Solicitors: F de B Brisley, Purley (for the plaintiff); Kanter, Jules & Co (for the defendants).

Shireen Irani Barrister.

CORRIGENDUM
The foregoing case, commencing at p 63, ante, was heard before, and decided by, Mr Justice Megaw, to whom the case was
transferred for hearing from Mr Justice Gormans list. It is regretted that in error the case is reported as having been decided by
Mr Justice Gorman, and it is requested that the name of Mr Justice Megaw should be substituted for that of Mr Justice Gorman
on pp 63, 64, ante.
65
[1963] 2 All ER 66

Ridge v Baldwin and others


CRIMINAL; Police

HOUSE OF LORDS
LORD REID, LORD EVERSHED, LORD MORRIS OF BORTH-Y-GEST, LORD HODSON AND LORD DEVLIN
5, 6, 7, 8, 12, 13, 14, 15 NOVEMBER 1962, 14 MARCH 1963

Police County borough police force Chief constable Summary dismissal by watch committee Whether rules of natural
justice applicable to proceedings for dismissal Chief constable previously indicted for alleged criminal offences Acquitted,
but conduct severely criticised by trial judge Appeal against decision of watch committee dismissed by Home Secretary
Whether action by chief constable thereby barred Municipal Corporations Act, 1882 (45 & 46 Vict c 50), s 191(4) Police Act,
1919 (9 & 10 Geo 5 c 46), s 4(1) Police (Appeals) Act, 1927 (17 & 18 Geo 5 c 19), s 2(3) Police (Discipline) (Deputy Chief
Constables, Assistant Chief Constables and Chief Constables) Regulations, 1952 (SI 1952 No 1706), reg 1 and reg 18.

The appellant, who in March, 1958, was nearly fifty-nine years of age, joined Brighton Borough Police Force in 1925; thereafter
he rose in the service and was appointed chief constable in 1956, the appointment being expressed to be subject to the Police
Acts and regulations. In October, 1957, he was suspended from duty after he had been arrested, together with two other officers
of the same police force, on charges which were subsequently the subjects of two indictments, one for criminal conspiracy to
corrupt the course of justice and the other for corruption. At the end of the trial of the first indictment in February, 1958, at which
the appellant had given evidence himself but had called no other witnesses, he was acquitted, but the other two police officers
were convicted. In passing sentence on the other two police officers, the trial judge intimated that they had not had from the
appellant the professional and moral leadership which they should have had. At the trial of the second indictment on 6 March
1958, the prosecution offered no evidence against the appellant, and the judge directed the jury to acquit him, but again he made
certain observations about the appellant. On 7 March 1958, the watch committee held a meeting at which, after considering
matters relating to the appellant, they unanimously dismissed him from his office of chief constable under s 191(4) a of the
Municipal Corporations Act, 1882. The appellant was not present at this meeting, nor was he charged or given notice of the
proposal to dismiss him or particulars of the grounds on which it was based or an opportunity of putting his case. By notice of
appeal dated 12 March 1958, the appellant appealed, under the Police (Appeals) Act, 1927, to the Home Secretary against his
dismissal, the notice stating that it was without prejudice to the validity of the watch committees decision, and reserving right to
contend that the procedure was bad. On 18 March 1958, the watch committee held a special meeting at which the appellants
solicitor requested them to re-consider their decision, particularly with regard to its consequences in relation to the plaintiffs
pension, but by a majority the watch committee adhered to their previous decision. On 5 July the Home Secretary dismissed the
appellants appeal. In October, 1958, the appellant commenced an action against the watch committee, claiming that his
purported dismissal was void, and also claiming payment of salary and pension, or alternatively, damages. His action was
dismissed. On appeal,
________________________________________
a The terms of s 191(4) are set out at p 96, letter h, post

Held Lord Evershed dissenting): (i) the decision of the watch committee on 7 March 1958, to dismiss the appellant was null
and void for the following reasons
(a) In exercising the power of dismissal conferred by s 191(4) b of the Municipal Corporations Act, 1882 (at any rate where
that power was to be 66 exercised on the ground of negligence, which required to be proved c) the watch committee were bound
to observe the principles of natural justice, but in this instance the committee had not observed them, for the appellant had not
been charged nor informed of the grounds on which they proposed to proceed and had not been given a proper opportunity to
present his defence (see p 80, letter g, p 109, letter h, and p 116, letter d, post).
________________________________________
b The terms of s 191(4) are set out at p 96, letter h, post
c The power of dismissal was exercisable in relation to a borough constable whom the watch committee think negligent in the discharge of
his duty or otherwise unfit for the same; there are dicta that the inclusion of the words or otherwise unfit conferred a residual discretion
which might be unfettered, and for the exercise of which no charge was necessary (see p 111, letter c, and p 114, letter g, post)

Baggs case ((1615), 11 Co Rep 93b); Cooper v Wandsworth Board of Works ((1863), 14 CBNS 180); De Verteuil v Knaggs
([1918] AC 557) applied.
Dicta of Atkin LJ, in R v Electricity Commissioners ([1923] All ER Rep at p 161) and of Lord Hewart CJ, in R v Church
Assembly Legislative Committee ([1927] All ER Rep at p 699) considered and explained.
Nakkuda Ali v M F de S Jayaratne ([1951] AC 66) disapproved in part.
and (b) (per Lord Morris of Borth-y-Gest, Lord Reid and Lord Hodson concurring) once there was a report or allegation
from which it appeared that a chief constable might have committed an offence against the discipline code, established by
regulations under the Police Act, 1919, it became a condition precedent to any dismissal based on a finding of guilty of such an
offence that the regulations should in essentials have been put into operation, but the watch committee had not complied with the
regulations, for they preferred no charge against the appellant and gave him no notice and no opportunity to defend himself (see p
102, letters f and g, p 116, letter c, and p 81, letter b, post);
or (c) (per Lord Devlin) compliance with reg 11(1) of the Police (Discipline) (Deputy Chief Constables etc) Regulations,
1952, but not all other of those regulations, was a condition precedent to dismissal, and here there had been no report or inquiry
satisfying reg 11(1) (see p 118, letter h, post).
and (d) the proceedings at the meeting of 18 March 1958, were not a full re-hearing and did not make good the failure on 7
March to observe the rules of natural justice (see p 81, letter a, p 106, letter i, and p 112, letter g, post).
(e) (Lord Devlin dissenting) the consequence of the failure to observe the rules of natural justice was that the decision of 7
March 1958, was void, not merely voidable (see p 81, letter e, p 110, letter a, and p 116, letter h, post; cf p 120, letter e, post).
Wood v Woad ((1874), LR 9 Exch 190), approved.
(f) the same consequence flowed from disregard of the regulations (see p 104, letter h, p 105, letter e, p 81, letter b, and p
116, letters c and d, post).
Andrews v Mitchell ([19047] All ER Rep 599) and Annamunthodo v Oilfields Workers Trade Union ([1961] 3 All ER 621)
applied.
(ii) the decision of 7 March 1958, was a nullity, and the decision of the Secretary of State, although final and binding by
virtue of s 2(3) of the Police (Appeals) Act, 1927, could not make valid that which was a nullity (see p 81, letter i, p 106, letter i,
to p 107, letter a, p 116, letter f, and p 119, letter b, post).
Decision of the Court of Appeal ([1962] 1 All ER 834) reversed.

Notes
In considering the application of the principles of natural justice to cases of dismissal, Lord Reid distinguishes three classes of
cases (i) dismissal of a servant by a master, (ii) dismissal from offices held at pleasure, and (iii) dismissal from an office where
there must be something against a man to warrant his dismissal (see p 71, letter f, post). The present case fell within class (iii).
67
The question of waiver (cf holding (ii) at [1962] 1 All ER p 835), as distinct from the effect of s 2(3) of the Police (Appeals)
Act, 1927, was referred to in the opinions (cf eg, p 81, letter h, and p 116, letter e, post), but it was doubted whether it really
arose.
As to the statutory power to dismiss a member of a police force, see 30 Halsburys Laws (3rd Edn) 103, para 171.
For the Municipal Corporations Act, 1882, s 191, see 14 Halsburys Statutes (2nd Edn) 140.
For the Police Act, 1919, see 18 Halsburys Statutes (2nd Edn) 122.
For the Police (Appeals) Act, 1927, s 2(3), see ibid, 136.
For the Police (Discipline) Regulations, 1952, Sch 1, see 17 Halsburys Statutory Instruments 200.

Cases referred to in opinions


Andrews v Mitchell [19047] All ER Rep 599, [1905] AC 78, 74 LJKB 333, 91 LT 537, 25 Digest (Repl) 347, 287.
Annamunthodo v Oilfields Workers Trade Union [1961] 3 All ER 621, [1961] AC 945, [1961] 3 WLR 650, 3rd Digest Supp.
Baggs Case (1615), 11 Co Rep 93b, 1 Roll Rep 224, 77 ER 1271, 13 Digest (Repl) 217, 382.
Barnard v National Dock Labour Board [1953] 1 All ER 1113, [1953] 2 QB 18, [1953] 2 WLR 995, 3rd Digest Supp.
Blisset v Daniel (1853), 10 Hare, 493, 1 Eq Rep 484, 1 WR 529, 68 ER 1022, 37 Digest (Repl) 609, 1703.
Board of Education v Rice [191113] All ER Rep 36, [1911] AC 179, 80 LJKB 796, 104 LT 689, 75 JP 393, HL, affg, SC sub
nom R v Board of Education, [1910] 2 KB 165, 79 LJKB 692, 102 LT 578, 74 JP 259, CA, 19 Digest (Repl) 630, 206.
Capel v Child (1832), 2 C & J 558, 2 Tyr 689, 1 LJEx 205, 149 ER 235, 19 Digest (Repl) 447, 2644.
Cooper v Wandsworth Board of Works (1863), 14 CBNS 180, 2 New Rep 31, 32 LJCP 185, 8 LT 278, 143 ER 414, 26 Digest
(Repl) 585, 2450.
Cooper v Wilson [1937] 2 All ER 726, [1937] 2 KB 309, 106 LJKB 728, 157 LT 290, 101 JP 349, 30 Digest (Repl) 171, 219.
Dawkins v Antrobus (1881), 17 ChD 615, 44 LT 557, CA, affg, (1879), 41 LT 490, 8 Digest (Repl) 652, 21.
Daws, Re (1838), 8 Ad & El 936, 1 Per & Dav 146, 112 ER 1095, 16 Digest (Repl) 464, 2841.
De Verteuil v Knaggs [1918] AC 557, 87 LJPC 128, 8 Digest (Repl) 691, 38.
Dean v Bennett (1870), 6 Ch App 489, 40 LJCh 452, 24 LT 169, 13 Digest (Repl) 228, 512.
Fisher v Jackson [1891] 2 Ch 84, 60 LJCh 482, 64 LT 782, 19 Digest (Repl) 642, 278.
Fisher v Keane (1878), 11 ChD 353, 49 LJCh 11, 41 LT 335, 8 Digest (Repl) 656, 36.
Hogg v Scott [1947] 1 All ER 788, [1947] KB 759, [1948] LJR 666, 177 LT 32, 111 JP 282, 2nd Digest Supp.
Hopkins v Smethwick Local Board of Health (1890), 24 QBD 712, 59 LJQB 250, 62 LT 783, 54 JP 693, 26 Digest (Repl) 634,
2822.
Kanda v Government of The Federation of Malaya [1962] AC 322, [1962] 2 WLR 1153.
Lapointe v LAssociation de Bienfaisance et de Retraite de la Police de Montreal [1906] AC 535, 75 LJPC 73, 95 LT 479, 25
Digest (Repl) 323, *38.
Liversidge v Anderson [1941] 3 All ER 338, [1942] AC 206, 110 LJKB 724, 116 LT 1, 17 Digest (Repl) 422, 27.
68
Local Government Board v Arlidge [191415] All ER Rep 1, [1915] AC 120, 84 LJKB 72, 111 LT 905, 79 JP 97, HL revsg SC
sub nom R v Local Government Board, Ex p Arlidge, [1914] 1 KB 160, 83 LJKB 86, 109 LT 651, 78 JP 25, CA, 38 Digest
(Repl) 102, 733.
Nakkuda Ali v Jayaratne (M F de S), [1951] AC 66, 2nd Digest Supp.
Osgood v Nelson (1872), LR 5 HL 636, 41 LJQB 329, 13 Digest (Repl) 229, 513.
R v Church Assembly Legislative Committee, Ex p Haynes Smith [1927] All ER Rep 696, [1928] 1 KB 411, 97 LJKB 222, 138 LT
399, 44 TLR 68, 19 Digest (Repl) 240, 2.
R v Darlington School (Governors), (1844), 6 QB 682, 14 LJQB 67, 4 LTOS 175, 115 ER 257, 13 Digest (Repl) 229, 518.
R v Electricity Comrs, Ex p London Electricity Joint Committee Co (1920) Ltd [1923] All ER Rep 150, [1924] 1 KB 171, 93
LJKB 390, 130 LT 164, 20 Digest (Repl) 202, 3.
R v Gaskin (1799), 8 Term Rep 209, 101 ER 1349, 16 Digest (Repl) 382, 1685.
R v Metropolitan Police Comr, Ex p Parker [1953] 2 All ER 717, [1953] 1 WLR 1150, 117 JP 440, 16 Digest (Repl) 461, 2811.
R v Nat Bell Liquors Ltd [1922] All ER Rep 335, [1922] 2 AC 128, 91 LJPC 146, 127 LT 437, 27 Cox, CC 253, 16 Digest (Repl)
469, 2897.
R v Neal [1949] 2 All ER 438, [1949] 2 KB 590, 33 Cr App Rep 189, 14 Digest (Repl) 662, 6717.
R v North, Ex p Oakey [1927] 1 KB 491, 96 LJKB 77, 136 LT 387, 19 Digest (Repl) 345, 1302.
R v Smith (1844), 5 QB 614, 1 Dar & Mer 564, 13 LJQB 166, 21 LTOS 400, 9 JP 5, 114 ER 1381, 16 Digest (Repl) 382, 1686.
R v Stratford-upon-Avon Corpn (1670), 1 Lev 291, sub nom Dighton v Stratford-on-Avon Corpn, 1 Sid 461, 2 Keb 641, 82 ER
1217, 13 Digest (Repl) 232, 554.
R v University of Cambridge (1723), 1 Stra 557, Fortes Rep 202, 2 Ld Raym 1334, 8 Mod Rep 148, 93 ER 698, 8 Digest (Repl)
506, 2281.
Ramshay, Ex p (1852), 18 QB 173, Cox, M & H 589, 21 LJQB 238, 18 LTOS 273, 16 JP 135, 118 ER 65, 13 Digest (Repl) 371,
24.
Russell v Norfolk (Duke) [1949] 1 All ER 109, 12 Digest (Repl) 693, 5321.
Smith v R (1878), 3 App Cas 614, 47 LJPC 51, 38 LT 233.
Spackman v Plumstead Board of Works (1885), 10 App Cas 229, 54 LJMC 81, 53 LT 157, 40 JP 420, HL, affg SC sub nom
Plumstead Board of Works v Spackman, (1884), 13 QBD 878, 53 LJMC 142, 51 LT 757, 49 JP 132, CA, 26 Digest (Repl)
571, 2357.
Terrell v Secretary of State for the Colonies [1953] 2 All ER 490, [1953] 2 QB 482, 8 Digest (Repl) 793, 509.
Urban Houses Co Ltd v Oxford Corpn [1939] 4 All ER 211, [1940] Ch 70, 109 LJCh 38, 162 LT 29, 38 Digest (Repl) 230, 462.
Weinberger v Inglis (No 2) [1918] 1 Ch 517, 87 LJCh 345, 118 LT 769, 34 TLR 337, CA, affd [1919] AC 606, 88 LJCh 287, 121
LT 65, HL, 8 Digest (Repl) 651, 11.
Willis v Childe (1851), 13 Beav 117, 30 LJCh 113, 17 LTOS 12, 51 ER 46, 19 Digest (Repl) 640, 258.
Wood v Woad (1874), LR 9 Exch 190, 43 LJEx 153, 30 LT 815, 2 Asp MLC 289, 8 Digest (Repl) 656, 35.

Appeal
This was an appeal by Charles Field Williams Ridge (the appellant) from a decision of the Court of Appeal (Holroyd Pearce,
Harman and Davies 69LJJ), dated 30 January 1962, and reported [1962] 1 All ER 834, affirming the judgment of Streatfeild J
dated 19 April 1962, and reported [1961] 2 All ER 523, dismissing the appellants claim against the respondent watch committee,
the police authority of the county borough of Brighton, that their dismissal of him under s 191(4) of the Municipal Corporation
Act, 1882, was invalid.
The Court of Appeal held that the watch committee, in dismissing the appellant under s 191(4) of the Act of 1882, were not
bound to apply the Police (Discipline) Regulations, 1952, as those regulations required that, as a condition precedent to acting
thereunder, a report or allegation must be received. In this case, however, the appellant had been dismissed after his acquittal on
a criminal charge following which the trial judge had made certain observations on the fitness of the appellant for office as chief
constable. Such observations, the Court of Appeal held, were not a report or allegation. Further the Court of Appeal held that the
watch committee, exercising their power under s 191(4) of the Act of 1882, were acting in an executive or administrative
capacity, not in a judicial or quasi-judicial nature with the consequence that the rules of natural justice did not apply to their
proceedings for dismissal. The Court of Appeal also held that by appealing to the Home Secretary under the Police (Appeals)
Act, 1927, the appellant had waived his right to bring the action in the courts, notwithstanding the appellant purported to reserve
his right to contend that the decision of the watch committee was a nullity.
It was not contended before the House of Lords that the power of dismissal conferred by s 191(4) of Act of 1882 was
impliedly repealed, by the joint effect of the Police Act, 1919, s 4 and regulations thereunder (cf p 99, letter i, to p 100, letter b,
post); accordingly the decision of the Court of Appeal (see [1962] 1 All ER 834, letter i) on that point stands.

D J C Ackner QC and J L E Macmanus for the appellant.


Neville Faulks QC and P A Harmsworth for the respondents.

Their Lordships took time for consideration


14 March 1963. The following opinions were delivered.

LORD REID. My Lords, the appellant, Mr Ridge, became Chief Constable of the County Borough of Brighton in 1956, after
serving in the Brighton Police Force for some thirty-three years. At a meeting of the watch committee, the police authority, on 7
March 1958, it was resolved that he should be dismissed and he now maintains that that resolution was void and of no effect
because he had no notice of the grounds on which the committee proposed to act and no opportunity to be heard in his own
defence.
The appellant had been arrested on 25 October 1957, and subsequently tried on a charge of conspiring with the senior
members of his force and others to obstruct the course of justice, and had been suspended from duty on 26 October. He was
acquitted on 28 February but the other two members of the force were convicted and in sentencing them the trial judge, Donovan
J made a statement which included grave reflections on the appellants conduct. He was then indicted on a charge of corruption
and was on 6 March acquitted, no evidence having been offered against him. On this occasion Donovan J made a further
statement. On the day following that statement the watch committee met and summarily dismissed the appellant. I shall not deal
further with these matters because my noble and learned friend Lord Morris of Borth-y-Gest intends to do so.
The power of dismissal is contained in s 191(4) of the Municipal Corporations Act, 1882. So far as I am aware that
subsection is the only statutory provision regarding dismissal, and the respondents purported to act under it. It is in these terms:

The watch committee, or any two justices having jurisdiction in the borough, may at any time suspend, and the watch
committee may at any time dismiss, any borough constable whom they think negligent in the discharge of his duty, or
otherwise unfit for the same.
70

The appellant maintains that the watch committee ought to have proceeded in accordance with regulations made under the
Police Act, 1919, s 4(1), which authorised the Secretary of State to make regulations as to, inter alia, the conditions of service of
the members of all police forces in England and Wales. Regulations were duly made but the respondents maintain that they do
not apply to this case. For the moment I shall assume in their favour that that is so and consider whether the Act of 1882 taken by
itself authorised them to do as they did.
The appellants case is that in proceeding under the Act of 1882 the watch committee were bound to observe what are
commonly called the principles of natural justice, that before attempting to reach any decision they were bound to inform him of
the grounds on which they proposed to act and to give him a fair opportunity of being heard in his own defence. The authorities
on the applicability of the principles of natural justice are in some confusion and so I find it necessary to examine this matter in
some detail. The principle audi alteram partem goes back many centuries in our law and appears in a multitude of judgments of
judges of the highest authority. In modern times opinions have sometimes been expressed to the effect that natural justice is so
vague as to be practically meaningless. But I would regard these as tainted by the perennial fallacy that because something
cannot be cut and dried or nicely weighted or measured therefore it does not exist. The idea of negligence is equally
insusceptible of exact definition but what a reasonable man would regard as fair procedure in particular circumstances and what
he would regard as negligence in particular circumstances are equally capable of serving as tests in law, and natural justice as it
had been interpreted in the courts is much more definite than that. It appears to me that one reason why the authorities on natural
justice have been found difficult to reconcile is that insufficient attention has been paid to the great difference between various
kinds of cases in which it has been sought to apply the principle. What a minister ought to do in considering objections to a
scheme may be very different from what a watch committee ought to do in considering whether to dismiss a chief constable. So I
shall deal first with cases of dismissal. These appear to fall into three classes, dismissal of a servant by his master, dismissal from
an office held during pleasure, and dismissal from an office where there must be something against a man to warrant his
dismissal.
The law regarding master and servant is not in doubt. There cannot be specific performance of a contract of service and the
master can terminate the contract with his servant at any time and for any reason or for none. But if he does so in a manner not
warranted by the contract he must pay damages for breach of contract. So the question in a pure case of master and servant does
not at all depend on whether the master has heard the servant in his own defence: it depends on whether the facts emerging at the
trial prove breach of contract. But this kind of case can resemble dismissal from an office where the body employing the man is
under some statutory or other restriction as to the kind of contract which it can make with its servants, or the grounds on which it
can dismiss them. The present case does not fall within this class because a chief constable is not the servant of the watch
committee or indeed of anyone else.
Then there are many cases where a man holds an office at pleasure. Apart from judges and others whose tenure of office is
governed by statute, all servants and officers of the Crown hold office at pleasure and this has even been held to apply to a
colonial judge (Terrell v Secretary of State). It has always been held, I think rightly, that such an officer has no right to be heard
before he is dismissed and the reason is clear. As the person having the power of dismissal need not have anything against the
officer, he need not give any reason. That was stated as long ago as 1670 in R v Stratford-upon-Avon Corpn where the
corporation dismissed a town clerk who held office durante bene placito. The leading case on 71 this matter appears to be R v
Governors of Darlington School, although that decision was doubted by Lord Hatherley LC in Dean v Bennett ((1870), 6 Ch App
489 at p 496) and distinguished on narrow grounds in Willis v Childe. I fully accept that where an office is simply held at
pleasure the person having power of dismissal cannot be bound to disclose his reasons. No doubt he would in many cases tell the
officer and hear his explanation before deciding to dismiss him. But if he is not bound to disclose his reason and does not do so,
then, if the court cannot require him to do so, it cannot determine whether it would be fair to hear the officers case before taking
action. Again that is not this case. In this case the Act of 1882 permits the watch committee to take action only on the grounds of
negligence or unfitness. Let me illustrate the difference by supposing that a watch committee who had no complaint against their
present chief constable heard of a man with quite outstanding qualifications who would like to be appointed. They might think it
in the public interest to make the change but they would have no right to do it. But there could be no legal objection to dismissal
of an officer holding office at pleasure in order to put a better man in his place.
So I come to the third class which includes the present case. There I find an unbroken line of authority to the effect that an
officer cannot lawfully be dismissed without first telling him what is alleged against him and hearing his defence or explanation.
An early example is Baggs case though it is more properly deprivation of the privilege of being a burgess of Plymouth. R v
Gaskin arose out of the dismissal of a parish clerk and Lord Kenyon CJ ((1799), 8 Term Rep at p 210), referred to audi alteram
partem as one of the first principles of justice. R v Smith was another case of dismissal of a parish clerk and Lord Denman CJ
((1844), 5 QB at pp 622, 623), held that even personal knowledge of the offence was no substitute for hearing the officer: his
explanation might disprove criminal motive or intent and bring forward other facts in mitigation, and in any event delaying to
hear him would prevent yielding too hastily to first impressions. Ex parte Ramshay is important. It dealt with the removal from
office of a county court judge and the form of the legislation which authorised the Lord Chancellor to act is hardly
distinguishable from the form of s 191 which confers powers on the watch committee. The Lord Chancellor was empowered if
he should think fit to remove on the ground of inability or misbehaviour but Lord Campbell CJ ((1852), 18 QB at p 190), said
that this was only on the implied condition prescribed by the principles of eternal justice. In Osgood v Nelson objection was
taken to the way in which the Corporation of the City of London had removed the clerk to the Sheriffs Court and Lord Hatherley
LC said ((1872), LR 5 HL at p 649):
I apprehend my Lords that as has been stated by the learned Baron who has delivered in the name of the judges their
unanimous opinion, the court of Queens Bench has always considered that it has been open to that court, as in this case it
appears to have considered, to correct any court or tribunal or body of men who may have a power of this description, a
power of removing from office, if it should be found such persons have disregarded any of the essentials of justice in the
course of their inquiry before making that removal, or if it should be found that in the place of reasonable cause those
persons have acted obviously upon mere individual caprice.

That citation of authority might seem sufficient but I had better proceed further. In Fisher v Jackson, three vicars had power to
remove the master of an endowed school. But unlike the Darlington case the trust deed set out the grounds on which he could be
removedbriefly, inefficiency or failing to set a 72 good exampleand it was held that they could not remove him without
affording him an opportunity of being heard in his own defence. Only two other cases of this class were cited in argument,
Cooper v Wilson and Hogg v Scott. Both dealt with the dismissal of police officers and both were complicated by consideration
of regulations made under the Police Acts. In the former the majority at least recognised that the principles of natural justice
applied and in deciding the latter Cassels J ([1947] 1 All ER at p 792; [1947] KB at p 767), in deciding that a chief constable
could dismiss without hearing him an officer who had been convicted of felony, appears to have proceeded on a construction of
the regulations. Of course, if the regulations authorised him to do that and were intra vires in doing so there would be no more to
be said. I do not think it necessary to consider whether the learned judge rightly construed the regulations, for he did not
expressly or I think by implication question the general principle that a man is not to be dismissed for misconduct without being
heard.
Stopping there I would think that authority was wholly in favour of the appellant, but the watch committees argument was
mainly based on what has been said in a number of fairly recent cases dealing with different subject-matter. Those cases deal
with decisions by ministers, officials and bodies of various kinds which adversely affected property rights or privileges of persons
who had had no opportunity or no proper opportunity of presenting their cases before the decisions were given. And it is
necessary to examine those cases for another reason. The question which was or ought to have been considered by the watch
committee on 7 March 1958, was not a simple question whether or not the appellant should be dismissed. There were three
possible courses open to the watch committeereinstating the appellant as chief constable, dismissing him, or requiring him to
resign. The difference between the latter two is that dismissal involved forfeiture of pension rights whereas requiring him to
resign did not. Indeed, it is now clear that the appellants real interest in this appeal is to try to save his pension rights.
It may be convenient at this point to deal with an argument that, even if as a general rule a watch committee must hear a
constable in his own defence before dismissing him, this case was so clear that nothing that the appellant could have said could
have made any difference. It is at least very doubtful whether that could be accepted as an excuse. But even if it could the watch
committee would in my view fail on the facts. It may well be that no reasonable body of men could have reinstated the appellant.
But as between the other two courses open to the watch committee the case is not so clear. Certainly on the facts as we know
them the watch committee could reasonably have decided to forfeit the appellants pension rights, but I could not hold that they
would have acted wrongly or wholly unreasonably if they had in the exercise of their discretion decided to take a more lenient
course.
I would start an examination of the authorities dealing with property rights and privileges with Cooper v Wandsworth Board
of Works. Where an owner had failed to give proper notice to the board, they had under an Act of 1855 d authority to demolish
any building which he had erected and recover the cost from him. This action was brought against the board because they had
used that power without giving the owner an opportunity of being heard. The board maintained that their discretion to order
demolition was not a judicial discretion and that any appeal should have been to the Metropolitan Board of Works. But the court
decided unanimously in favour of the owner. Erle CJ ((1863), 14 CBNS at p 189), held that the power was subject to a
qualification repeatedly recognised that no man is to be deprived 73 of his property without his having an opportunity of being
heard and that this had been applied to many exercises of power which in common understanding would not be at all a more
judicial proceeding than would be the act of the district board in ordering a house to be pulled down. Willes J ((1863), 14 CBNS
at p 190) said that the rule was of universal application and founded on the plainest principles of justice and Byles J ((1863), 14
CBNS at p 194), said that:
________________________________________
d See the Metropolis Management Act, 1855, s 76

although there are no positive words in a statute requiring that the party shall be heard, yet the justice of the common
law will supply the omission of the legislature.

This was followed in Hopkins v Smethwick Local Board of Health. Wills J ((1890), 24 QBD at pp 714, 715), said:

In condemning a man to have his house pulled down a judicial act is as much implied as in fining him 5: and as the
local board is the only tribunal that can make such an order its act must be a judicial act and the party to be affected should
have a notice given him The judgment of WILLES, J. (in Coopers case [(1890), 14 CBNS at pp 190194.]) goes far
more upon the nature of the thing done by the board than on the phraseology of the Act itself. It deals with the case on
principle: from the nature of the thing done it must be a judicial act and justice requires that the man should be heard.

In the Court of Appeal ((1890), 24 QBD at pp 716, 717) Lord Esher MR in dismissing an appeal expressly approved the
principles laid down in Coopers case.
The principle was applied in different circumstances in Smith v R. That was an action of ejectment on the alleged forfeiture
of a Crown lease in Queensland. The governor was entitled to forfeit the lease if it had been proved to the satisfaction of a
commissioner that the lessee had abandoned or ceased to reside on the land. The commissioner did not disclose to the lessee the
case against him so that he had no opportunity to meet it, and therefore the decision could not stand. The Commissioner was not
bound by any rules as to procedure or evidence but he had to conduct his inquiry according to the requirements of substantial
justice. In De Verteuil v Knaggs the governor of Trinidad was entitled to remove immigrants from an estate on sufficient
ground shown to his satisfaction. Lord Parmoor ([1918] AC at p 560) said that

the acting governor was not called upon to give a decision on an appeal between parties and it is not suggested that he
holds the position of a judge or that the appellant is entitled to insist on the forms used in ordinary judicial procedure

but he had

a duty of giving to any person against whom the complaint is made a fair opportunity to make any relevant
statement which he may desire to bring forward and a fair opportunity to correct or controvert any relevant statement
brought forward to his prejudice.

The duty of an official architect in fixing a building line was stated in somewhat similar terms in Spackman v Plumstead Board of
Works.
I shall now turn to a different class of casedeprivation of membership of a professional or social body. In Wood v Woad,
the committee purported to expel a member of a mutual insurance society without hearing him and it was held that their action
was void and so he was still a member. Kelly CB said of audi alteram partem ((1874), LR 9 Exch at p 196)
74

this rule is not confined to the conduct of strictly legal tribunals but is applicable to every tribunal or body of persons
invested with authority to adjudicate upon matters involving civil consequences to individuals.

This was expressly approved by Lord Macnaghten giving the judgment of the Board in Lapointe v Lassociation de Bienfaisance
etc ([1906] AC 535 at p 540). In that case the board of directors of the association had to decide whether to give a pension to a
dismissed constablethe very point the watch committee had to decide in this caseand it was held that they had to observe
the elementary principles of justice. Then there are the club cases Fisher v Keane and Dawkins v Antrobus. In the former
Jessel MR said ((1878), 11 ChD at pp 362, 363) of the committee:

They ought not as I understand it according to the ordinary rules by which justice should be administered by
committees of clubs, or by any other body of persons who decide upon the conduct of others, to blast a mans reputation for
everperhaps to ruin his prospects for life without giving him an opportunity of either defending or palliating his
conduct.

In the latter case it was held that nothing had been done contrary to natural justice. In Weinberger v Inglis (No 2) a member of
enemy birth was excluded from the stock exchange and it was held that the committee had heard him before acting. Lord
Birkenhead LC said ([1919] AC at p 616):

if I took the view that the appellant was condemned upon grounds never brought to his notice I should not assent to
the legality of that course unless compelled by authority.

He said this although the rule under which the committee acted was in the widest possible termsthat the committee should each
year re-elect such members as they should deem eligible as members of the stock exchange.
I shall not at present advert to the various trade union cases because I am deliberately considering the state of the law before
difficulties were introduced by statements in various fairly recent cases. It appears to me that if the present case had arisen thirty
or forty years ago the courts would have had no difficulty in deciding this issue in favour of the appellant on the authorities which
I have cited. So far as I am aware none of these authorities has ever been disapproved or even doubted. Yet the Court of Appeal
have decided this issue against the appellant on more recent authorities which apparently justify that result. How has this come
about? At least three things appear to me to have contributed. In the first place there have been many cases where it has been
sought to apply the principles of natural justice to the wider duties imposed on ministers and other organs of government by
modern legislation. For reasons which I shall attempt to state in a moment it has been held that those principles have a limited
application in such case and those limitations have tended to be reflected in other decisions on matters to which in principle they
do not appear to me to apply. Secondly, again for reasons which I shall attempt to state, those principles have been held to have a
limited application in cases arising out of war-time legislation; and again such limitations have tended to be reflected in other
cases. And thirdly, there has I think been a misunderstanding of the judgment of Atkin LJ, in R v Electricity Comrs, Ex p London
Electricity Joint Committee Co ([1923] All ER Rep 150 at pp 158 et seq; [1924] 1 KB 171 at pp 198 et seq).
In cases of the kind with which I have been dealing the Board of Works or the governor or the club committee was dealing
with a single isolated case. It was not deciding, like a judge in a lawsuit, what were the rights of the person before it. But it was
deciding how he should be treatedsomething analogous to a judges duty in imposing a penalty. No doubt policy would play
some part in the decisionbut so it might when a judge is imposing a sentence. So it was easy to say that such a body is
performing a quasi judicial task in considering and 75 deciding such a matter and to require it to observe the essentials of all
proceedings of a judicial characterthe principles of natural justice. Sometimes the functions of a minister or department may
also be of that character and then the rules of natural justice can apply in much the same way. But more often their functions are
of a very different character. If a minister is considering whether to make a scheme for say an important new road, his primary
concern will not be with the damage which its construction will do to the rights of individual owners of land. He will have to
consider all manner of questions of public interest and, it may be, a number of alternative schemes. He cannot be prevented from
attaching more importance to the fulfilment of his policy than to the fate of individual objectors and it would be quite wrong for
the courts to say that the minister should or could act in the same kind of way as a board of works deciding whether a house
should be pulled down. And there is another important difference. As explained in Local Government Board v Arlidge a minister
cannot do everything himself. His officers will have to gather and sift all the facts including objections by individuals and no
individual can complain if the ordinary accepted methods of carrying on public business do not give him as good protection as
would be given by the principles of natural justice in a different kind of case.
We do not have a developed system of administrative lawperhaps because until fairly recently we did not need it. So it is
not surprising that in dealing with new types of cases the courts have had to grope for solutions, and have found that old powers,
rules and procedure are largely inapplicable to cases which they were never designed or intended to deal with. But I see nothing
in that to justify our thinking that our old methods are any less applicable today than ever they were to the older types of case.
And, if there are any dicta in modern authorities which point in that direction, then in my judgment they should not be followed.
And now I must say something regarding war-time legislation. The older authorities clearly show how the courts engrafted
the principles of natural justice on to a host of provisions authorising administrative interference with private rights. Parliament
knew quite well that the courts had an inveterate habit of doing that and must therefore be held to have authorised them to do it
unless a particular Act showed a contrary intention. And such an intention could appear as a reasonable inference as well as from
express words. It seems to me to be a reasonable and almost an inevitable inference from the circumstances in which defence
regulations were made and from their subject-matter that at least in many cases the intention must have been to exclude the
principles of natural justice. War-time secrecy alone would often require that and the need for speed and general pressure of
work were other factors. But it was not to be expected that anyone would state in so many words that a temporary abandonment
of the rules of natural justice was one of the sacrifices which war conditions requiredthat would have been almost calculated to
create the alarm and despondency against which one of the regulations was specifically directed. And I would draw the same
conclusion from another fact. In many regulations there was set out an alternative safeguard more practicable in war timethe
objective test that the officer must have reasonable cause to believe whatever was the crucial matter. (I leave out of account the
very peculiar decision of this House in Liversidge v Anderson.) So I would not think that any decision that the rules of natural
justice were excluded from war-time legislation should be regarded as of any great weight in dealing with a case such as this case
which is of the older type, and which involves the interpretation of an Act passed long before modern modifications of the
principles of natural justice became necessary, and at a time when, as Parliament was well aware, the courts habitually applied the
principles of natural justice to provisions like s 191(4) of the Municipal Corporations Act, 1882.
76
The matter has been further complicated by what I believe to be a misunderstanding of a much quoted passage in the
judgment of Atkin LJ, in R v Electricity Comrs. He said ([1923] All ER Rep at p 161; [1924] 1 KB at p 205):
The operation of the writs [of prohibition and certiorari] has extended to control the proceedings of bodies which do
not claim to be and would not be recognised as, courts of justice. Whenever any body of persons having legal authority to
determine questions affecting the rights of subjects, and having the duty to act judicially, act in excess of their legal
authority, they are subject to the controlling jurisdiction of the Kings Bench Division exercised in these writs.

A gloss was put on this by Lord Hewart CJ, in R v Legislative Committee of the Church Assembly ([1927] All ER Rep 696 at p
699; [1928] 1 KB 411 at pp 415, 416). There it was sought to prohibit the Assembly from proceeding further with the Prayer
Book Measure, 1927. That seems to me to have no resemblance to a question whether a person should be deprived of his rights
or privileges, and the case was decided on the ground that this was a deliberative or legislative body and not a judicial body.
Salter J put it in a few lines ([1927] All ER Rep at p 701; [1928] 1 KB at p 419):

The person or body to whom these writs are to go must be a judicial body in this sense, that it has power to determine
and decide, and the power carries with it, of necessity, the duty to act judicially. I think that the Church Assembly has no
such power and, therefore, no such duty.

But Lord Hewart CJ said, having quoted the passage from Lord Atkins judgment ([1928] 1 KB at p 415; [1927] All ER Rep at p
699):

The question therefore which we have to ask ourselves in this case is whether it is true to say in this matter, either of
the Church Assembly as a whole or of the Legislative Committee of the Church Assembly, that it is a body or persons
having legal authority to determine questions affecting the rights of subjects and having the duty to act judicially. It is to be
observed that in the last sentence which I have quoted from the judgment of ATKIN, L.J. [[1923] All ER Rep at p 161;
[1924] 1 KB at pp 204, 205.], the word is not or but and. In order that a body may satisfy the required test it is not
enough that it should have legal authority to determine questions affecting the rights of subjects; there must be superadded
to that characteristic the further characteristic that the body has the duty to act judicially. The duty to act judicially is an
ingredient which if the test is to be satisfied must be present. As these writs in the earlier days were issued only to bodies
which without any harshness of construction could be called and naturally would be called courts so also today these writs
do not issue except to bodies which act or are under the duty to act in a judicial capacity.

I have quoted the whole of this passage because it is typical of what has been said in several subsequent cases. If Lord Hewart
CJ, meant that it is never enough that a body simply has a duty to determine what the rights of an individual should be, but that
there must always be something more to impose on it a duty to act judicially before it can be found to observe the principles of
natural justice, then that appears to me impossible to reconcile with the earlier authorities. I could not reconcile it with what Lord
Denman CJ, said in R v Smith ((1844), 5 QB at pp 620624), or what Lord Campbell CJ, said in Ex parte Ramshay ((1852), 18
QB 173 at pp 188196), or what Lord 77 Hatherley LC said in Osgood v Nelson ((1872), LR 5 HL at pp 648650), or what was
decided in Cooper v Wandsworth Board of Works, or Hopkins v Smethwich Local Board, or what Lord Parmoor said in De
Verteuil v Knaggs ([1918] AC at pp 558563), or what Kelly CB ((1874), LR 9 Exch at pp 195199) said, with the subsequent
approval of Lord MacNaghtene, in Wood v Woad, or what Jessel MR, said in Fisher v Keane ((1878), 11 ChD at pp 357363), or
what Lord Birkenhead LC said in Weinberger v Inglis (No 2) ([1919] AC at pp 610617) and that is only a selection of the earlier
authorities. And, as I shall try to show, it cannot be what Lord Atkin ([1923] All ER Rep at pp 160, 161; [1924] 1 KB at p 204)
meant.
________________________________________
e In Lapointes case [1906] AC at p 540

In R v Electricity Comrs the commissioners had a statutory duty to make schemes with regard to electricity districts and to
hold local inquiries before making them. They made a draft scheme which in effect allocated duties to one body which the Act
required should be allocated to a different kind of body. This was held to be ultra vires and the question was whether prohibition
would lie. It was argued that the proceedings of the commissioners were purely executive and controllable by Parliament alone.
Bankes LJ said ([1923] All ER Rep at p 157; [1924] 1 KB at p 198):

On principle and on authority it is, in my opinion, open to this court to hold, and I consider that it should hold, that
powers so far-reaching, affecting as they do individuals as well as property, are powers to be exercised judicially and not
ministerially, or merely, to use the languagef of PALLES, C.B., as proceedings towards legislation.
________________________________________
f In R v Kingstown Comrs (1885), 16 LR Ir 150

So he inferred the judicial element from the nature of the power. I think that Atkin LJ ([1923] All ER Rep at pp 157 et seq.;
[1924] 1 KB at pp 198 et seq), did the same. Immediately after the passage which I said has been misunderstood, he cited a
variety of cases and in most of them I can see nothing superadded (to use Lord Hewarts word) to the duty itself. Certainly
Lord Atkin did not say that anything was superadded. A later passage in his judgment convinces me that he, like Bankes LJ (See
[1923] All ER Rep at p 157; [1924] 1 KB at p 198), inferred the judicial character of the duty from the nature of the duty itself.
Although it is long I am afraid I must quote it ([1923] All ER Rep at pp 161, 162; [1924] 1 KB at pp 206, 207):

In the present case the Electricity Commissioners have to decide whether they will constitute a joint authority in a
district in accordance with law, and with what power they will invest that body. The question necessarily involves the
withdrawal from existing bodies of undertakers of some of their existing rights, and imposing upon them of new duties
including their subjection to the control of the new body, and new financial obligations. It also provides in the new body a
person to whom may be transferred rights of purchase which at present are vested in another authority. The commissioners
are proposing to create such a new body in violation of the Act of Parliament and are proposing to hold a possibly long and
expensive inquiry into the expediency of such a scheme, in respect of which they have the power to compel representatives
of the prosecutors to attend and produce papers. I think that in deciding upon the scheme and in holding the inquiry they
are acting judicially in the sense of the authorities I have cited

There is not a word in Lord Atkins judgment to suggest disapproval of the earlier line of authority which I have cited. On the
contrary, he goes further than those authorities. I have already stated my view that it is more difficult 78 for the courts to control
an exercise of power on a large scale where the treatment to be meted out to a particular individual is only one of many matters to
be considered. This was a case of that kind, and, if Lord Atkin was prepared to infer a judicial element from the nature of the
power in this case, he could hardly disapprove such an inference when the power relates solely to the treatment of a particular
individual.
The authority chiefly relied on by the Court of Appeal in holding that the watch committee were not bound to observe the
principles of natural justice was Nakkuda Ali v M F de S Jayaratne. In that case the Controller of Textiles in Ceylon made an
order cancelling the appellants licence to act as a dealer, and the appellant sought to have that order quashed. The controller
acted under a defence regulation, which empowered him to cancel a licence where the controller has reasonable grounds to
believe that any dealer is unfit to be allowed to continue as a dealer. The Privy Council regarded that as ([1951] AC at p 77)

imposing a condition that there must in fact exist such reasonable grounds known to the controller before he can
validly exercise the power of cancellation.

But according to their judgment certiorari did not lie and no other means was suggested whereby the appellant or anyone else in
his position could obtain redress even if the controller acted without a shred of evidence. It is quite true that the judgment went
on, admittedly unnecessarily, to find that the controller had reasonable grounds and did observe the principles of natural justice,
but the result would have been just the same if he had not. This House is not bound by decisions of the Privy Council and for my
own part nothing short of a decision of this House directly in point would induce me to accept the position that, although an
enactment expressly requires an official to have reasonable grounds for his decision, our law is so defective that a subject cannot
bring up such a decision for review however seriously he may be affected and however obvious it may be that the official acted in
breach of his statutory obligation. The judgment proceeds ([1951] AC at p 77):

But it does not seem to follow necessarily from this that the controller must be acting judicially in exercising the
power. Can one not act reasonably without acting judicially? It is not difficult to think of circumstances in which the
controller might in any ordinary sense of the word have reasonable grounds of belief without having ever confronted the
licence holder with the information which is the source of his belief. It is a long step in the argument to say that because a
man is enjoined that he must not take action unless he has reasonable grounds for believing something he can only arrive at
that belief by a course of conduct analagous to the judicial process. And yet unless that proposition is valid there is really
no ground for holding that the controller is acting judicially or quasi-judicially when he acts under this regulation. If he is
not under a duty so to act then it would not be according to law that his decision should be amenable to review and if
necessary to avoidance by the procedure of certiorari.

I would agree that in this and other defence regulation cases the legislator has substituted an obligation not to act without
reasonable grounds for the ordinary obligation to afford to the person affected an opportunity to submit his defence. It is not
necessary in this case to consider whether by so doing he has deprived the courts of the power to intervene if the officer acts
contrary to his duty. The question in the present case is not whether Parliament substituted a different safeguard for that afforded
by natural justice, but whether in the Municipal Corporations Act, 1882, it excluded the safeguard of natural justice and put
nothing in its place.
So far there is nothing in the judgment of the Privy Council directly relevant 79 to the present case. It is the next paragraph
which causes the difficulty and I must quote the crucial passage ([1951] AC at p 78):

But the basis of the jurisdiction of the courts by way of certiorari has been so exhaustively analysed in recent years
that individual instances are now only of importance as illustrating a general principle that is beyond dispute. That
principle is most precisely stated in the words of ATKIN, L.J., in R. v. Electricity Comrs.

and then follows the passagepg 80 (See p 77, letter a, ante, [1923] All ER Rep at p 161; [1924] 1 KB at p 205) with which I have
already dealt at length. And then there follows the quotation from Lord Hewart CJ (See p 77, letter f, ante, [1928] 1 KB at p 415;
[1927] All ER Rep at p 699), which I have already commented on ending with the words ([1951] AC at p 78)there must be
superadded to that characteristic the further characteristic that the body has the duty to act judicially. And then it is pointed out:
It is that characteristic that the controller lacks in acting under reg 62. Of course if it were right to say that Lord Hewart CJs
gloss on Lord Atkin stated a general principle that is beyond dispute the rest would follow. But I have given my reasons for
holding that it does no such thing, and in my judgment the older cases certainly do not illustrate any such general principle
they contradict it. No case older than 1911 was cited in Nakkuda Ali v M F de S Jayaratne on this question, and this question was
only one of several difficult questions which were argued and decided. So I am forced to the conclusion that this part of the
judgment in Nakkudas case was given under a serious misapprehension of the effect of the older authorities and therefore cannot
be regarded as authoritative.
I would sum up my opinion in this way. Between 1882 and the making of police regulations in 1920, s 191(4) of the
Municipal Corporations Act, 1882, had to be applied to every kind of case. The respondents contention is that, even where there
was a doubtful question whether a constable was guilty of a particular act of misconduct, the watch committee were under no
obligation to hear his defence before dismissing him. In my judgment it is abundantly clear from the authorities that I have
quoted that at that time the courts would have rejected any such contention. In later cases dealing with different subject-matter
opinions have been expressed in wide terms so as to appear to conflict with those earlier authorities. But learned judges who
expressed those opinions generally had no power to overrule those authorities, and in any event it is a salutary rule that a judge is
not to be assumed to have intended to overrule or disapprove of an authority which has not been cited to him and which he does
not even mention. So I would hold that the power of dismissal in the Act of 1882 could not then have been exercised and cannot
now be exercised until the watch committee have informed the constable of the grounds on which they propose to proceed and
have given him a proper opportunity to present his case in defence.
Next comes the question whether the respondents failure to follow the rules of natural justice on 7 March was made good
by the meeting on 18 March. I do not doubt that if an officer or body realises that it has acted hastily and reconsiders the whole
matter afresh after affording to the person affected a proper opportunity to present his case then its later decision will be valid.
An example is De Verteuil v Knaggs. But here the appellants solicitor was not fully informed of the charges against the appellant
and the watch committee did not annul the decision which they had already published and proceed to make a new decision. In
my judgment what was done on that day was a very inadequate substitute for a full rehearing. Even so three members of the
committee changed their minds, and it is impossible to say what the 80 decision of the committee would have been if there had
been a full hearing after disclosure to the appellant of the whole case against him. I agree with those of your lordships who hold
that this meeting of 18 March cannot affect the result of this appeal.
The other ground on which some of your lordships prefer to proceed is the watch committees failure to act in accordance
with the police regulations. I have had an opportunity of reading the speech about to be delivered by my noble and learned friend
Lord Morris of Borth-y-Gest and I agree with his views about this. I will only add that the circumstances in which the Police Act,
1919, was passed, and the consequent regulations were made, show that the regulations must have been intended to have a very
wide application and I see nothing unreasonable in applying them to this case. Dismissing a chief constable who has not been
convicted of any criminal offence is not a thing to be done lightly. If the whole of the matters against him are disclosed to him
and he refuses to admit some or all of them, it seems to me perfectly proper that there should be such an inquiry as the regulations
require. In particular to exclude this case from the ambit of the regulations because the watch committee did not proceed on any
report or allegation is a very narrow interpretation of the regulations and it would lead to a strange result. Counsel for the
respondents was constrained to admithe could not reasonably have done otherwisethat, if some busybody had formally
reported to the watch committee the observations of Donovan J and required them to deal with these allegations, then the watch
committee would have been bound to apply the regulations. But it would be absurd if the substantive rights of the appellant were
to depend on whether or not someone happened to have made a formal report or allegation to the watch committee before they
proceeded to deal with the case.
Then there was considerable argument whether in the result the watch committees decision is void or merely voidable.
Time and again in the cases I have cited it has been stated that a decision given without regard to the principles of natural justice
is void and that was expressly decided in Wood v Woad. I see no reason to doubt these authorities. The body with the power to
decide cannot lawfully proceed to make a decision until it has afforded to the person affected a proper opportunity to state his
case.
Finally there is the question whether by appealing to the Secretary of State the appellant is in some way prevented from now
asserting the nullity of the respondents decision. A person may be prevented from asserting the truth by estoppel but it is not
seriously argued that that doctrine applies here. Then it is said that the appellant elected to go to the Secretary of State and
thereby waived his right to come to the court. That appears to me to be an attempt to set up what is in effect estoppel where the
essential elements for estoppel are not present. There are many cases where two remedies are open to an aggrieved person but
there is no general rule that by going to some other tribunal he puts it out of his power thereafter to assert his rights in court; and
there was no express waiver because in appealing to the Secretary of State the appellant reserved his right to maintain that the
decision was a nullity. But then it was argued that this case is special because by statute the decision of the Secretary of State is
made final and binding. I need not consider what the result would have been if the Secretary of State had heard the case for the
appellant and then had given his own independent decision that the appellant should be dismissed. But the Secretary of State did
not do that. He merely decided that there was sufficient material on which the watch committee could properly exercise their
power of dismissal under s 191(4) of the Municipal Corporations Act, 1882. So the only operative decision is that of the watch
committee, and if it was a nullity, I do not see how this statement by the Secretary of State can make it valid.
81
Accordingly in my judgment this appeal must be allowed. There appears to have been no discussion in the courts below as
to remedies which may now be open to the appellant, and I do not think that this House should do more than declare that the
dismissal of the appellant is null and void and remit the case to the Queens Bench Division for further procedure. But it is right
to put on record that the appellant does not seek to be reinstated as chief constable: his whole concern is to avoid the serious
financial consequences involved in dismissal as against being required or allowed to resign.

LORD EVERSHED. My Lords, on the difficult problem presented by this appeal I regret to find myself differing from your
lordships; but I have felt myself constrained to agree with the conclusions reached by Streatfeild J, at the trial and by all the
members of the Court of Appeal.
It will be logical for me to deal first with the question whether the watch committee of Brighton Corporation were bound to
observe the requirements of what I will compendiously call the Police (Discipline) Regulations of 1952, before purporting to
exercise, as regards the appellant, the jurisdiction now admittedly vested in them by s 191(4) of the Municipal Corporations Act,
1882; for if they were so bound then, in the absence of such observance, it may be saidand was so contended on the appellants
partthat the watch committee had in truth no jurisdiction to reach their decision for the appellants dismissal. By the Police
(Discipline) Regulations of 1952 I refer compendiously to two statutory instruments, namely, (1) the Police (Discipline)
Regulations 1952, SI 1952 No 1705, and (2) the Police (Discipline) (Deputy Chief Constables, Assistant Chief Constables and
Chief Constables) Regulations, 1952, SI 1952 No 1706; as respectively amended by the two regulations, SI 1954 Nos. 1687 and
1688, being regulations of the same respective titles as those of 1952 already mentioned. The two instruments of 1952 were both
made on 17 September 1952, and came into operation on 1 October 1952. Similarly the two amending instruments of 1954 were
both made on 17 December 1954, and came into operation on 1 January 1955. The several instruments clearly form together a
single code. I shall have to make some references to them hereafter and I shall then refer to them respectively as Statutory
Instrument 1705 and Statutory Instrument 1706, the references being in each case intended to comprehend the amendments
made in 1954.
I have been unable to accept the argument that every case of indiscipline or of incapacity of any police officer whether a
chief constable or any other member of a police force (save only cases of incapacity arising from mental or physical illness) falls
or was intended to fall within the scope of the regulations. For my part I accept the view propounded by counsel for the watch
committee which appealed to the learned judge at the trial, that discreditable conduct and neglect of duty which constitute the
first and fourth headings in the discipline code set out in Sch 1 to Statutory Instrument 1705, should be construed as limited to the
kinds of conduct specified in those headings, each of which, be it observed, begins with the words that is to say. In the present
case the substance and gravamen of the appellants incapacity as chief constable, on which the watch committee proceeded to act,
was that expressed by Donovan J after presiding at a trial, lasting nineteen days, of the appellant and others charged with
conspiring to obstruct the course of public justice, namely that the appellant had not, in that learned and experienced judges
view, the professional or moral qualities requisite for one holding the office of chief constable. Although therefore the
appellant was himself acquitted of the charge, the learned judge thought it right to treat the appellants limitations, which he had
expressed, as justifying remission of the sentences otherwise appropriate to be passed on his two subordinate officers whom the
jury had convicted. As I understand the language of Donovan J (and as, I doubt not, the watch 82 committee also understood it),
the appellant had been shown not to possess a sense of probity or of responsibility sufficient for the office which he held, and so
had been unable to provide the essential leadership and example to the police force under his control which his office properly
required.
The first of the relevant headings in the discipline code, Discreditable conduct is thus defined:

that is to say, if a member of a police force acts in a disorderly manner or any manner prejudicial to discipline or
reasonably likely to bring discredit on the reputation of the force or of the police service.

I will not take time by reciting the more detailed expansion of the second heading, Neglect of duty g but I cannot (as could not
the learned judge in the first court) hold that the shortcomings of the appellant as chief constable, described by Donovan J fall
within either of the respective headings as expanded by their definitions. I add only that the two headings in question, being part
of the code specified in Statutory Instrument 1705, should be construed with regard to the fact that they were originally intended
to apply only to members of a police force of lower rank than chief constables or deputy or assistant chief constables though it is
true to say that by Statutory Instrument 1706 they were made applicable also to chief constables and deputy and assistant chief
constables.
________________________________________
g For this heading, see 17 Halsburys Statutory Instruments 201

My lords, it follows, in my opinion, that the watch committee were entitled to exercise their residual powers under s 191(4)
of the Act of 1882 without observance of the Police (Discipline) Regulations. I do not forget the terms of para (1)(f) of the watch
committees resolution of 7 March 1958. It is said that the subject-matter of this paragraph was that mentioned in para (c) of the
watch committees answer to the appellants appeal to the Secretary of State, namely, the suggestion that the appellant had given
false evidence at the trial before Donovan J; and that such a charge was in terms within para (b) of the heading numbered 5 in the
discipline code, namely, Falsehood or prevarication, that is to say, if a member of a police force wilfully makes any false
statement. But, assuming the premise, it is nevertheless, in my opinion, still clear that the reference was but to an incident in
the trial on the conclusion of which the watch committee were manifestly founding themselves; and must have been so
understood. I cannot think that such an incidental reference can sensibly have the startling result of making the watch
committees jurisdiction dependent on a strict application of the Police (Discipline) Regulations. It follows, if I am right in
thinking that the case against the appellant did not fall under any of the provisions of the discipline code, that it was not brought
within the regulations by cl 11 of Statutory Instrument 1706.
I also find myself in agreement with all the learned judges below in thinking that in any event this was not a case of there
having been a report or allegation to the watch committee as contemplated by the regulations. I have, for my part, been unable
to accept counsel for the appellants argument that any deliberation by the watch committee necessarily supposes the presence of
a report or allegation by someone. I do not attempt any definition of the phrase; but, in my opinion, the context of the
regulations suggests necessarily something in the nature of an accusation as distinct from a conclusion reached after proper
inquiry, and cannot sensibly be said to include a judicial conclusion after the protracted investigation of a trial. If this view be
wrong I would ask your lordships to observe the consequences. If counsel for the appellants submission be accepted, it must
follow (as counsel for the appellant indeed conceded) that the regulations were or would be equally applicable to any disciplinary
action taken by the watch committee in regard to Detective Sergeant Heath and Detective Inspector Hammersley, each found
guilty at the trial and sentenced to terms of imprisonment. If the argument submitted be correct, 83it must follow that the watch
committees duty must be or have been (notwithstanding the conclusion of their trial) to refer the cases of these two officers to an
Investigating Officer under Statutory Instrument 1705 who would report to the chief constable (sic), the officers having a
right of appeal to the watch committee. And so, on this view, it was the watch committees duty, as regards the appellant, under
Statutory Instrument 1706 first to instruct a solicitor to formulate the case against him; and then to appoint a tribunal (which
might consist of five members of the watch committee itself) whose duty it would be to report to the watch committee. My
Lords, I cannot think it right to accept an argument involving results which appear to me so manifestly absurd. In the present
case the conduct of the appellant had been the subject of a public trial lasting nineteen days; and, if the observations of Donovan
Jh, can sensibly be called a report at all, they were equivalent to the report of the investigating tribunal to the watch
committee contemplated by Statutory Instrument 1706. In this respect the situation after the conclusion of the trial was to my
mind wholly different from that at the time when the appellant was first charged and when, therefore, as it seems to me, the watch
committee rightly felt itself bound to act in accordance with the regulations in ordering the appellants suspension. My lords, any
other view, in my opinion, makes the regulations gravely offend against common sense. I agree therefore with the view of the
Court of Appealand particularly with that of Holroyd Pearce LJ ([1962] 1 All ER at p 843), that this was a special and entirely
exceptional case, outside the scope of the regulations and, as a matter of public notoriety, requiring instant action by the watch
committee. The extent of the public notoriety can fairly be gauged from the letter written by the appellants solicitor explaining
the remarkable request for his clients reinstatement as chief constable by reference to the telephone calls and offers of rewards
by newspapers to which he had been incessantly subjected.
________________________________________
h These are quoted in [1962] 1 All ER at p 837, letter b

I turn accordingly to what have appeared to me to be the most difficult questions raised in this appeal; that is to say, first,
whether the exercise of the statutory jurisdiction by the watch committee, which in my opinion was vested in them without regard
to the regulations, required the observance by the watch committee of what are called the principles of natural justice; and,
second, if so, whether on the facts of this case such principles were in fact observed.
It has been said many times that the exact requirements in any case of the so-called principles of natural justice cannot be
precisely defined; that they depend in each case on the circumstances of that case. According to Sir Frederick Pollock the
meaning of the phrase natural justice is the ultimate principle of fitness with regard to the nature of man as a rational and
social being; and he went on to point out that the origin of the principles could be traced to Aristotle and the Roman jurists.
Your lordships were therefore not unnaturally referred to a great many cases, but as I believe that your lordships agree, it is by no
means easy to treat these decisions as entirely uniform and still less easy to be able to extract from them the means of
propounding a precise statement of the circumstances or of the cases in which the principles can be invoked before the courts. I
am, however, content to assume that the invocation should not be limited to cases where the body concerned, whether a domestic
committee or some body established by a statute, is one which is exercising judicial or quasi-judicial functions strictly so called;
but that such invocation may also be had in cases where the body concerned can properly be described as administrativeso long
as it can be said, in Sir Frederick Pollocks language, that the invocation is required in order to conform to the ultimate principle
84 of fitness with regard to the nature of man as a rational and social being. On the other hand, it is (as I venture to think) no less
plain now that Parliament may by appropriate language in a statute make it clear that the activity or discretion of the body
constituted by the statute is not to be subject to any control or interference by the courts.
At this stage I venture to make two points. First, since there is no question here of bias or any suggestion that the watch
committee acted otherwise than entirely in good faith, the only principle of natural justice here involved is that enshrined in the
Latin phrase Audi alteram partem. Second, I for my part conclude that if the principles of natural justice can properly be
invoked in this case and if it should be held that such principles were not observed, then the decision of the watch committee was
not void but voidable only.
On this second question (whether the decision afterwards impugned can be said to be void or voidable only) the cases
provide, as I think, no certain answer; nor have I found one in the text-books. Indeed in the vast majority of circumstances, it
does not in the end matter whether the decision challenged is void or only voidable; for, if the court does decide to quash a
decision or otherwise set it aside, then the effect is in general the same whether such decision be considered as void or only
voidable. For my part, however, I have come to the conclusion that in a case where a body is acting within its jurisdiction but of
which the courts will say that it has failed to do substantial justice in accordance with the principles of natural justice, then the
decision is only voidable and cannot properly be described as a nullity.
Though I am in this respect anticipating what later follows, I refer first to the extremely wide and general terms of the
relevant subsection of the Act of 1882:

The watch committee may at any time dismiss any borough constable whom they think negligent in the discharge of
his duty or otherwise unfit for the same.

My lords, in my opinion it is impossible to accept the suggestion put forward on the appellants part that the final words of the
subsection, otherwise unfit for the same, must be regarded as ejusdem generis with what has gone before; that is to say, that
they are intended only to refer to some kind of negligence. I know of no authority in a case of this kind, where the jurisdiction of
the body in question is expressed in two alternatives, for supposing that the second of the alternatives does not mean what it says
but is somehow limited by the terms of the first alternative. Put in other words, where, may I ask, is the genus to be found of
which the second part of the alternative is said to be but a part? As I interpret the language used, the second part of the power
conferred is, as the language inevitably imports, intended to cover the case of someone who is regarded as unfit for his position
for reasons other than negligence.
I observe again that there is, as I think, no question here of the watch committees jurisdiction. If I had taken a different
view on the first question, namely, whether the exercise of the jurisdiction had to be subject to compliance with the regulations,
then my answer might have been different. On that assumption it is unnecessary for me to express any concluded opinion and I
do not do so. But, if I am right in thinking that there was here no question of compliance with the regulations, the only question
for your lordships is whether, admitting the jurisdiction of the watch committee, it was properly exercised having regard to any
application of the principles of natural justice to which the exercise of the jurisdiction was subject. I observe further that the
appellant in his action seeks a declaration. There was some discussion before your lordships concerning the office of a
declaration, as contrasted with that of an order for certiorari. In my judgment it must be accepted as tolerably clear that (subject
to what follows) the granting of a declaration in a case of this kind must prima facie be discretionary: and if that is so it must
equally follow that the question whether the decision of the watch committee is such that the court can quash it or otherwise
interfere with it involves the conclusion that such decision was 85 voidable and not void. If the decision was a complete nullity
(for example, on the ground that the watch committee never had any jurisdiction) then no doubt it would follow that the court
would have to say so in some form or other. But this is not, in my judgment, such a case. My lords, I have in mind on this matter
decisions such as that of the Privy Council in the recent case of Annamunthodo v Oilfield Workers Trade Union, where the
plaintiff had been expelled from the union and the board thought that the expulsion was wholly invalid. In fact, however, in that
case the plaintiff had been charged under one rule but was later expelled under another. It is also to be noted that the case was
dealing not with powers conferred by Act of Parliament but with a domestic tribunal; the point submitted (but rejected) was that
the appeal which the plaintiff had taken under the rules of the union constituted an affirmance of the jurisdiction of the council,
which had dismissed him. I have also in mind the case of Wood v Woad. My noble friend Lord Reid has stated in his opinion i
that in this case the Court of Exchequer Chamber expressly decided that a decision by a body acting in a quasi-judicial capacity
which failed to have due regard to the principle of natural justice audi alteram partem, is void and not merely voidable. With all
respect to my noble friend, I am unable to agree with this conclusion.
________________________________________
i See p 81, letter e, ante

My lords, it is, I think, necessary to have in mind what was the nature of the plaintiffs claim in Wood v Woad. The plaintiff
alleged that he had been a member of a mutual marine insurance association and, as such member, having paid to the treasurer the
appropriate deposit, had therefore been entitled to recover from the association the amount of loss incurred by him in respect of a
particular ship; that in the committee of the association was vested by its rules the whole power of the management of its affairs
and also the power, if they deemed the conduct of any member to be suspicious or that he was for any other reason unworthy of
remaining in the association, to exclude such member by appropriate notice from further participation in it; but that the
committee had wrongfully, collusively and improperly expelled the plaintiff from the association without any just reason or
probable cause; so that the plaintiff had been deprived of his right to the sum of money in respect of the damage done to his ship
and that he was accordingly entitled to recover as damages from the members of the committee the amount of such loss. This
being the nature of the claim the court decided on demurrer that the plaintiff could have, on his allegations, no cause of action for
damages at law against the members of the committee. I emphasise the important fact that the claim formulated was for damages
at law against the members of the committee (not all of whom in fact were or need have been members of the association).
It is clear from the headnote to the case that the learned barons of the Court of Exchequer Chamber did not arrive for
entirely the same reasons at the conclusion that the plaintiff could not succeed in his action. It is true that certain language in the
judgment of Kelly CB appears to support the view that in his opinion the committees failure to give to the plaintiff any
opportunity of answering the charge made by the committee against him rendered the committees decision void and a nullity.
See, for example, the chief barons citation of the decision in Blisset v Daniel. But if so, it was, in my judgment, because in the
view of the chief baron there was enough to show a collusive and unlawful exercise of power on the part of the committee
((1874), LR 9 Exch at p 198)in other words not a true exercise of the power at all or, at best, an exercise of the power of the
exceptional kind to which I later refer in the case of Osgood v Nelson. It is essential to have in mind the nature of the plaintiffs
claim as formulated by him which the Court of Exchequer Chamber rejected. On the one hand (as Kelly CB 86 pointed out
((1874), LR 9 Exch at p 196) if the discretion of the committee was absolute and if the committee in fact exercised their power
under the rules, the plaintiff could not question it. On the other hand, if, as the plaintiff in his declaration alleged, the committees
act was collusive and unlawful and therefore ineffective then the plaintiff remained a member of the association and (whatever
might be his rights or remedies in a court of equity) he therefore could have no claim for damages in law against the committee
((1874), LR 9 Exch at p 198):

The claim in this action is for damages sustained by reason of the expulsion of the plaintiff from the association; but in
law the plaintiff has sustained no damage at all, for whatever rights he may have possessed before he possesses still, as if
no action had been done calculated to deprive him of them.

Cleasby B, put his conclusion on somewhat different grounds ((1874), LR 9 Exch at pp 198, 200):

Now, we may suppose either that the committee expelled the plaintiff without just cause and without giving him
notice, or that they expelled him without just cause but did give him notice; and the declaration is framed so as to
comprehend in the breach both modes of wrongful expulsion.

After pointing out that by the rules the committee had absolute discretion, the learned baron concluded his judgment by saying
that the allegation not having made fraud the basis of the claim the declaration sought could not be sustained. Pollock Bs
judgment ((1874), LR 9 Exch at pp 201203) was to the same effectparticularly in respect of the absence of any claim based in
terms of fraud. The learned baron went on to observe that the plaintiffs declaration having alleged that the committees actions
were a nullity it was not on this premise possible for him to formulate a cause of action at law against the committee members.
Finally, Amphlett B, posed the matter thus ((1874), LR 9 Exch at p 204):

Now according to the allegations in the declaration, the defendants never gave the plaintiff that opportunity, and I
cannot entertain a doubt that if this allegation were proved, the plaintiff would, by filing a bill in a court of equity, be
restored to the enjoyment of his rights. But if so, what is his damage? He has not ceased to be a member of the society; he
has not lost the rights of a member. He is to recover damages for what? For an attempt to expel.

I have attempted at some length to analyse the reasons for the judgments of the Court of Exchequer Chamber in Wood v Woad. It
is, as I have more than once observed, of the essence of the matter in that case that the plaintiff was claiming damages personally
against the members of the committee. In such circumstances it is, as I venture to think, clear that the question whether the
purported exclusion from the association by the committee was void or voidable was not essential nor indeed material to his
claim made in the action by the plaintiff for damages against the members of the committee. Certainly in my judgment it cannot
be asserted that the judgments in the case cited, or indeed any of them, support or involve the proposition that where a body, such
as the watch committee in the present case, is invested by the express terms of a statute with a power of expulsion of any member
of the police force and purport in good faith to exercise such power, a failure on their part to observe the principle of natural
justice audi alteram partem has the result that the decision is not merely voidable by the court but is wholly void and a nullity.
My lords, I have for my part on this question, derived the greatest assistance from the case of Osgood v Nelson in which
Baron Martin gave ((1872), LR 5 HL at pp 645648) to your Lordships House the opinion of the judges. The case was
concerned with the 87 removal of the chief clerk or registrar of the Sheriffs Court in the City of London. By the Act 15 and 16
Vict c lxxvii power had been given to
the mayor, aldermen and commons in common council assembled for inability or misbehaviour or for any other
cause which may appear reasonable to the mayor and council to remove

a person in the position of the chief clerk or registrar. It was the unanimous opinion of the judges expressed by Baron Martin
((1872) LR 5 HL at pp 646, 647) that there was no doubt that the courts of law in this country would take care that any
proceeding of this kind should be conducted in a proper mannerthat is by giving to the person whose removal was in question
every opportunity of defending himself. If continued the learned Baron ((1872), LR 5 HL at pp 646, 647) your lordships are
satisfied that there was a real substantial miscarriage of justice, your lordships would not permit this amotion to remain.
Again ((1872) LR 5 HL at pp 646, 647):

we also think that it is possiblethough there is no necessity for giving any judgment upon itthat if a man was
removed from an office of this kind for any frivolous or futile cause you would in all probability be inclined to think the
removal is a nullity.

Lord Hatherley C, in adopting the view so expressed of the judges said ((1872), LR 5 HL at p 649):

The Court of Queens Bench, has always considered that it has been open to that court, as in this case it appears to
have considered, to correct a court or tribunal or body of men who may have a power of this description if it should be
found that such persons have disregarded any of the essentials of justice.

From these citations I deduce the conclusion that, save in the case where the tribunal or body of men have acted on frivolous
or futile grounds (in which case the court may treat not merely the decision but the whole proceeding as a nullity) the power of
the court is to correct the decision if, in the courts view, there has been real substantial miscarriage of justice. In other words
I think that, save in the excepted cases (of which the present cannot be said to be one) the right or duty of the court is to correct,
that is to set aside or otherwise restrain, the impugned decision if satisfied that there has been a real substantial miscarriage of
justice; a view which, if well-founded, must mean that (save in the excepted cases) the decision is voidable and not void.
My lords it is perhaps useful and necessary to inquire what in truth is meant by saying that a decision such as that of the
watch committee in the present case, is void or a nullity. Is it thereby intended that, though the proceedings up to the
pronouncement of the decision were proper and effective, the decision itself was a nullity? Or is it intended that the whole
proceedings ab initio were irregular and ineffective so that the decision was similarly and of necessity also of no effect? My
lords, the latter must in my judgment be the true analysis. In the first place, it does not to my mind appear correct or indeed
sensible to say that the decision reached was a nullity although the proceedings leading up to the decision were in order.
Second, I observe as I have earlier stated, that in cases of this kind it is not the function of the court to impugn the decision as
suchstill less to substitute its ownbut to examine the steps taken in reaching the decision and to decide whether, in the course
of those steps, there was a real substantial miscarriage of justice.
In the vast majority of cases it matters not in the result whether the decision is said to be void or voidable but avoided. It is
sufficient for the court to say that the decision cannot stand. In truth, as Sir Frederick Pollock pointed out (see Pollock on
Contract, 13th Edn, p 48) the words void and voidable are imprecise and apt to mislead. And so it is, as I venture to think,
that language such as that used by Kelly CB in Wood v Woad ((1874), LR 9 Exch at p 198) ought not to be strictly 88 construed
it was, indeed, for reasons which I have attempted to give, in any case obiter having regard to the nature of the claim in that case.
I do not doubt that in some cases the proper conclusion will be that the entire proceedings of the body or tribunal in question
(including therefore its decision) will properly be found to be wholly irregular and ineffective from first to last. The obvious case
is where the body or tribunal is shown to have been acting in excess of its jurisdiction. In this category no doubt will fall the
class of case mentioned by Martin B, in Osgood v Nelson ((1872), LR 5 HL at p 647) where the body concerned has acted on a
futile or frivolous cause; for in such case it could truthfully be said that the invocation by the body of its power was a pretence
and its proceedings no more than a sham. It may indeed well be that Kelly CB so regarded the performance of the committee in
Wood v Woad. But save in those cases, as I think, on true analysis the function and duty of the court is to correct, that is to say,
to set aside or quash the decision where it is shown that there has been some real substantial miscarriage of justice in the steps
taken by the body or tribunal in question in arriving at its decision in exercise of the powers vested in it.
My lords, I do not wish unduly to prolong this opinion but on this highly important matter it seems to me that useful analogy
may be found in the practice of the criminal courts. Thus, the Court of Criminal Appeal in the exercise of its powers under the
Criminal Appeal Act, 1907, may quash a conviction and substitute a verdict of acquittal, and may do so where there has at the
trial been what is regarded in effect as a failure to observe the principles of natural justice, eg where the jury has been told that it
must return with its verdict in ten minutes or where the jury was allowed after retirement under the bailiffs control to depart from
the court for luncheon. In these cases it is essential that there should have been an effective trial at least up to the point where the
departure from the principles of natural justice occurred; for otherwise the appeal court could not have ordered the prisoners
acquittal. If in truth the prisoner had never been really tried at all, he would be liable to be tried again for the same offence and
the appeal court could have issued a venire de novo. The second of the examples above given (ie where the jury had been
allowed to leave the court for luncheon) was involved in the case before the Court of Criminal Appeal of R v Neal and the point
with which I am concerned was in terms dealt with by Lord Goddard CJ ([1949] 2 All ER at p 441; [1949] 2 KB at p 596). I
would also refer your Lordships to the judgment of the Privy Council delivered by Lord Sumner in the case of R v Nat Bell
Liquors Ltd.
I only add that, as I apprehend, the same principles apply to an order for certiorari which has been held to be available, but at
the direction of the court, by way of declaration and an injunction in cases of decisions by statutory tribunals where the persons
affected would otherwise be without remedy (see Barnard v National Dock Labour Board following Andrews v Mitchell). On the
other hand it has also been held that certiorari will not be granted where the proceedings in the inferior tribunal are not merely
voidable but altogether voideg where the person purporting to act in a judicial capacity had in truth no authority so to do (see
Re Daws).
Finally, I venture to pose to your lordships the question, what would have been the situation had the Secretary of State
allowed the appellants appeal and held that he should be reinstated as chief constable? Would it have been open to the
corporation to refuse to give effect to such decision on the ground that the proceedings or the decision before or by the watch
committee had been a nullity?
89
I return accordingly to the first of the points previously mentioned, namely, the question whether the watch committee in
exercising its powers under the relevant section of the Act of 1882 was in the present circumstances bound to give to the
appellant an opportunity of putting forward his case and arguments before the committee. I have already said that the terms of te
discretion vested in the watch committee by the Act of Parliament have seemed to me to be of the widest. They are, as I think,
much wider than the phrase appearing in the case, to which I shall later refer, of De Verteuil v Knaggs, where the relevant
language was If it appears to the governor on sufficient grounds shown to his satisfaction. I also think that the language in
the Act of 1882 was at least as wide if not wider than the relevant language in the case of Nakkuda Ali v M F de S Jayaratne on
which the Court of Appeal considerably relied, namely [where] the controller has reasonable grounds to believe that any dealer
is unfit to be allowed to continue as a dealer. I understand that some of your lordships feel disposed to say that the decision of
the Privy Council in that case ought not to be followed. I must respectfully dissent from that view. It seems to me that on the
language of the enactment there in question there was in truth conferred on the governor an unfettered discretion. I am aware that
it is sometimes said that a different result may be appropriate where there is in question the grant or withdrawal of a licence as
distinct from the taking away of some right or proprietary interest. There is no doubt force in this argument and it has been
supported by our Court of Criminal Appeal in the case of R v Metropolitan Police Commissioner, Ex p Parker. At the same time
I would observe that though the withdrawal of a licence, which can be described as the removal of a privilege, is in some respects
different in character from the taking away of vested rights or proprietary interests, nevertheless the withdrawal of a licence from
the person from whom it is withdrawn may in fact mean the destruction of his means of livelihood.
My lords, if the only question which was here involved was whether the appellant should have ceased in March, 1958, to be
chief constable and whether therefore the watch committee was entitled without more so to decide I would, for my part, say that
there was in the circumstances no call for the watch committee to observe the so-called rule audi alteram partem. As I have
already observed the appellant had been subjected to a trial lasting nineteen days and it was as a result of the evidence in that trial
that Donovan J expressed the view that the appellant was in fact no longer fitted to act as chief constable. It would seem to me
frankly somewhat absurd that the watch committee should invite the appellant to state his points again after he had put forward a
case before the trial judge for so long a period. Moreover, as Holroyd Pearce LJ ([1962] 1 All ER at p 843), pointed out (and as I
have earlier noted), there was here a case of extreme urgency. The trial had attracted the greatest possible notoriety as had also
the observations in regard to the appellant of the trial judge. In my judgment the watch committee had a dutya duty not only to
the corporation of which they were a committee but also to the citizens of Brightonto act and to act at once so as to give effect
to what the trial judge had after so long a hearing in effect determined.
But my difficulty in the present case arises over the question of the appellants pension. For assuming it to be right that the
appellant would have to cease to be chief constableand I add in regard to that matter the not unimportant fact that his learned
counsel has not before your lordships suggested that he should have been retained as chief constablethen there were two ways
in which his appointment might be determined. First, he might have been required to resign in which case, though he would have
had so to resign, he would under the terms of the Police Pension Regulations, 1955, j have been entitled to receive the pension
which by that date had accrued in his favour. The alternative was the 90 appellants summary dismissal which was the course
adopted; though I do observe that in the watch committees minute of 7 March 1958, it is recorded that they had paid regard to
the length of the appellants service.
________________________________________
j See SI 1955 No 480, reg 52

It is undoubtedly a striking fact that the appellant had at the date when he had been suspended from his office of chief
constable served some 33 1/2 years and had risen from the rank of police constable through the various intervening ranks to that
of chief constable. During this long period of service it does not appear that there had ever been any criticism of his work in the
police force. Moreover, in March, 1958, he had attained the age of fifty-eight years and ten monthsin other words he was
within fourteen months of the age on which he would have been entitled to retire voluntarily with full pension. In these
circumstances I cannot conceal from myself that (unless the words of the statute deny it) there is shown an obvious case for
giving to the appellant an opportunity to put forward his argument for the first of the two alternatives, namely, that he should be
required to resign and not be summarily dismissed.
As I have said, I feel very great difficulty on this matter. I do not wish at all to denigrate the principles of natural justice or
of their proper invocation in the courts. On the other hand, we have, as I have already many times pointed out, the very wide
terms of the Act of Parliament here in question and the body in which was invested this wide discretion was an entirely
responsible body. To insist, as I venture to think, on the invocation of these principles whenever anyone is discharged from some
office seems to me to involve a danger of usurpation of power on the part of the courts and under the pretext of having regard to
the principles of natural justice to invoke what may often be in truth little more than sentiment; and on occasions when the courts,
though having necessarily far less knowledge of all the relevant circumstances, may be inclined to think that, had the decision
rested with them, they would have decided differently from the body in question. Yet I do observe again that it is not the decision
as such which is liable to review; it is only the circumstances in which the decision was reached and particularly in such a case as
the present the need for giving to the party dismissed an opportunity for putting his case.
I should therefore, for my part have been inclined to accept the view of the learned lord justices in the Court of Appeal. But
since, as I understand, your lordships take a different view and having read the exhaustive opinion of my noble and learned friend
Lord Reid, I will on this matter, express no concluded opinion of my own. I am ready to assume that the appellant should have
been given an opportunity at any rate to put his case for being required to resign rather than being summarily dismissed. If this
assumption be made, then the second question ariseswas justice done in all the circumstances of this case? My Lords, having
considered the whole matter with the greatest care of which I am capable I conclude that justice was here doneor, at least, that
there was no real substantial miscarriage of justice. I have already observed that in their minutes of 7 March 1958, the watch
committee (against whose good faith no kind of charge is made) stated that they had paid due regard to the length of the
appellants service. But at once after the receipt of notice of that resolution the appellants solicitor, Mr Bosley, asked the watch
committee to reconsider the matter and to hear observations that might be put forward on it. In acceding to this request it is to my
mind plain that the watch committee (and the corporation) thereupon decided that, notwithstanding their resolution of 7 March
the matter of the apellants dismissal should be held in suspenseand indeed after they had been informed of the appellants
appeal to the Secretary of State they also clearly decided that the operation of their previous resolution should be suspended
pending the result of that appeal.
It is clear that on 12 March the town clerk informed the appellants solicitor that the watch committee would reconsider their
decision with regard to the appellants pension and further told him that a meeting would be held on 18 March at which the
committee would consider such representations as might then be 91 made by or on behalf of the appellant either orally or in
writing, or both as may be preferred. Thereupon Mr Bosley sent to the town clerk his written observations dated the same day in
which in fourteen numbered paragraphs he set out the heads of his clients complaints; and in para 15 he asked, first, that the
appellant should be allowed to retire on full pension forthwith. Mr Bosley also sent to the town clerk a copy of his notice of
appeal to the Secretary of State, a lengthy document in which every kind of complaint made on the appellants behalf was
enumerated, though I would observe that nowhere in that document was any specific claim made that the appellants case really
was that of his being summarily dismissed instead of being requested to resign with the consequent right to receive his pension.
As a result there was a further special meeting of the watch committee held on 18 March. An extract from the minutes of that
meeting was duly sent to Mr Bosley and from the minute it is clear that the watch committee had given to Mr Bosley the fullest
opportunity to make such representations as he should think fit; and it is also recorded that the committee having heard all that
Mr Bosley had to say and considered also his written representations and the notice of appeal to the Home Secretary had decided
to adhere to their previous decision; though it is noted that there were three dissentients on this occasion.
My lords, having regard to all the circumstances, I have formed the view that your lordships ought not now to say that a
sufficient opportunity was not given to the appellant by himself or through his adviser to put before the watch committee such
points as he had and in particular to put before the watch committee the request that he should be required to resign rather than be
summarily dismissed. I therefore respectfully agree on this matter with the conclusion of Streatfeild J ([1961] 2 All ER at p 536),
at the trial of the present proceedings, and also with what I understand and believe to have been the view of Harman LJ as
reported ([1962] 1 WLR at p 735; cf [1962] 1 All ER at p 850) (though the language as there recorded does not contain, as I think
from a reading it should have contained, a negative). In reaching this conclusion I have derived support from the case of De
Verteuil v Knaggs above referred to. In that case the governor of Trinidad had acted in emergency with promptitude but without
giving to the person concerned any opportunity for a hearing. In the circumstances it was pointed out by Lord Parmoor ([1918]
AC at pp 560, 561) that this might well be justified provided that there was opportunity given afterwards when the original
decision might be reviewed. Similarly, in my view the present case was indeed one of grave emergency calling for the greatest
promptitude of action. But for reasons which I have attempted to state I think that assuming in the first place there was any
failure to observe the principles of natural justice by giving to the appellant an opportunity of being heard, this defect was
remedied afterwards when the original decision was suspended and an opportunity given to the appellant or his adviser or both to
make to the watch committee such representations as they wished.
But if I were wrong on the point last mentioned still in my opinion the appellant fails in the end on another point, namely, by
reason of the consequences of his appeal to the Secretary of State. This matter was also dealt with by the learned judges of the
Court of Appeal who similarly concluded that in any event the appeal to the Secretary of State barred the appellant from claiming
relief now. The case appears then to have been put on the basis of estoppel or election. For my part I prefer to rest my
conclusion simply on the terms of the relevant section in the Act itself. The Act is the Police (Appeals) Act, 1927a date, be it
noted, eight years later than the date of the Police Act, 1919, under which were promulgated the SI 1952 Nos. 1705 and 1706.
By s 1(1) of the Police (Appeals) Act, 1927, k, a member of a police force who after the passing of this Act is punished by
dismissal was given the right to appeal to the 92 Secretary of State. It is also by the Act provided that the Secretary of State
is not bound to entertain the appeal by way of hearing oral evidence if it appears to him that the case is of such a nature that it can
properly be determined without such evidence. This was in fact the course adopted by the Secretary of State in the present
circumstances. By his order of 5 July 1958, after reciting that an appeal had been made against the watch committees decision
of the previous March it is recorded that the Secretary of State having decided that the case is of such a nature that it can
properly be determined without taking oral evidence hereby order as follows; and then, in para 1, occurs the language I dismiss
the appeal.
________________________________________
k The enactment is printed as amended by the Police (Appeals) Act, 1943, s 1

It was not, as I followed the argument, suggested that the Secretary of State was acting otherwise than within the jurisdiction
conferred on him by the Act of Parliament in deciding to dispose of the appeal as he did on the written material before him and
without hearing oral argument. Indeed s 2(2) of the Act of 1927 provides thus:

The Secretary of State after considering the notice of appeal and any other documents submitted to him by the
appellant and the respondent and the report (if any) of the person or persons holding the inquiry shall by order, either
(a) allow the appeal; or (b) dismiss the appeal; or (c) vary the punishment

As it seems to me the action taken by the Secretary of State was in strict compliance with his powers and duties under the Act
and, with all respect to those who may take a contrary view, I cannot see how it can be said that the order of the Secretary of State
is ex facie unsustainable. What then is the result? By s 2(3) of the Act of 1927 it is provided that the decision on such appeal by
the Secretary of State is to be final and binding upon all parties. I agree that if it had been made out that the proceedings of the
watch committee were a nullity, then the appeal and the result of the appeal might well be regarded equally as a nullity. But for
reasons which I have endeavoured to justify it is in my opinion not true to say that the decision of the watch committee was a
nullity even if there was a failure on their part to obey the rules of natural justice by their omission to give to the appellant proper
opportunity to be heard. Their decision was voidable only. This being so, then the appellant having invoked his right under the
statute to appeal to the Secretary of State must, as I conceive, be bound by the result which Parliament has enjoined: and that
result is that after such an appeal the Secretary of States decision shall be final and binding as between himself and the watch
committee. I cannot imagine any language more explicit. Nor does it seem to me that the result can be avoided because both in
his original letter to the Secretary of State and in the documents stating his grounds of appeal itself the appellants solicitor stated
that his invocation of the power to appeal was without prejudice to his right thereafter to maintain that the watch committees
decision was in some way wrong in law. In my judgment the appellant invoked his right to appeal to the Secretary of State
under the Act of 1927 and, having done so, cannot escape the consequences which, as it seems to me, Parliament has stated in the
plainest terms.
It follows therefore that, whatever might be the right answers to the difficult questions involved in regard to the application
of the rules of natural justice, the appellant by proceeding as he did under the Police (Appeals) Act, 1927, to appeal to the
Secretary of State cannot now say other than that the conclusion of the Secretary of State which was entirely in accordance with
his statutory powers was a final and binding conclusion which put an end to any right that the appellant might otherwise have had
to invite the court in the exercise of its discretion to set aside or otherwise interfere with the watch committees decision.
My conclusion, therefore, with all respect to your lordships who take a different view, is, first, that there was in the present
case no requirement that the watch committee should observe the terms of the Police (Discipline) Regulations of 931952 and
therefore that the jurisdiction lay under s 191(4) of the Municipal Corporations Act, 1882, with the watch committee; that
therefore (second) the most that could be said against the watch committees decision was that by failing to observe the rules of
natural justice it was liable to be challenged and impugned in the courts; but (third) for reasons given, that assuming that there
was a failure to comply with the rules of natural justice in the first instance by omitting to give to the appellant the right to be
heard (before the passing of the resolution of 7 March 1958), that failure was afterwards remedied; in other words having regard
to the entirely exceptional circumstances that it cannot now be said on the appellants part that there was any real or substantial
injustice in what was done by the watch committee. But, fourth, it is my opinion that if in all other respects I am wrong the result
of invoking the Act of 1927 by way of appeal to the Secretary of State involved necessarily the result that the Secretary of States
conclusion must be regarded by your lordships as having finally disposed of all questions between the appellant and the watch
committee.
There was also raised on the appeal before your lordships a question of the true interpretation of s 220 of the Act of 1882.
So far as relevant that section is as follows:

A conviction, order, warrant or other matter made or done or purporting to be made or done by virtue of this Act shall
not be quashed for want of form, and shall not, unless it is an order of the council for payment of money out of the borough
fund, be removed by certiorari or otherwise into the High Court.

It was contended on behalf of the appellant that the terms of the section were only applicable to cases in which the question was
as regards purely formal matters. For my part I am not persuaded that this is a right construction of the words which Parliament
has used. But I prefer on this matter not to express any concluded opinion. If the view which I have tried to express and justify
were right it would follow that this section would not be a relevant consideration. Further than that, since the point was never
taken by the watch committee until the course of the argument before your lordships, it would as it seems to me, in any event be
too late for the watch committee to rely on this section if in other respects they were wrong. I therefore say no more on this
matter.
If the matter rested with me, my lords, I would dismiss the appeal.
LORD MORRIS OF BORTH-Y-GEST. My Lords, the appellant who in March, 1958, was nearly fifty-nine years of age
became a constable in the Brighton Borough Police Force in 1925 after a short period of service in another police force.
Thereafter he received progressive promotions in the Brighton Police Force. In 1935 he became a detective sergeant and in 1948
a detective inspector. In 1949 he was made detective chief inspector and in 1950 detective superintendent. In 1954 he was
promoted to be deputy chief constable. In the early part of 1956 there was a vacancy in the office of chief constable. The
appellant was an applicant for the appointment. He was one of five candidates who were interviewed by the watch committee.
The committee who had the opportunities for judging of the competence of the appellant which his prior service afforded them
resolved that subject to the approval of the Secretary of State the appellant should be appointed. He was so appointed. Amongst
other terms and conditions the appointment was to be subject to the Police Acts and Regulations.
In October of the following year the appellant and two police officers and two others were arrested. The allegation was one
of conspiracy to obstruct the course of public justice. The watch committee (who are the police authority) then took action under
the provisions of the Police (Discipline) (Deputy Chief Constables, Assistant Chief Constables and Chief Constables)
Regulations, 1952 (SI 1952 No 1706), as amended by SI 1954 No 1688. They decided to suspend him from duty as notified in a
letter dated 29 October 1957; they resolved in accordance with 94 reg 15 to pay him certain suspension allowances. The opening
part of reg 15(1) as amended provides that:

Where a report or allegation is received from which it appears that a deputy chief constable or assistant chief constable
of a borough police force may have committed an offence against discipline or a criminal offence the police authority may
suspend him from duty until such time as either
(a) it is decided that he shall not be charged with an offence against discipline, or
(b) the disciplinary proceedings referred to in these regulations are concluded.

Though that regulation refers to a deputy or assistant chief constable it may under certain circumstances be invoked in the case of
a chief constable. This is as a result of reg 18 which provides:

Where a report or allegation is received from which it appears that a chief constable may have committed an offence,
these regulations shall apply with the following modifications, adaptations and exceptions:

The regulations provided that the expression offence had the same meaning as it has in the Police (Discipline) Regulations,
1952. Regulation 1(1) of those regulations provides:

A member of a police force commits an offence against discipline (hereafter in these regulations referred to as an
offence) if he commits one or more of the offences set out in Sch. 1 hereto (hereafter in these regulations referred to as the
discipline code) or such additions thereto as may be made by the police authority for the police force with the consent of
the Secretary of State.

The position in October, 1957, was therefore that the watch committee suspended the appellant under reg 15 which was
applicable on the basis that the watch committee had received a report or allegation from which it appeared that the chief
constable might have committed one or more of the offences set out in the discipline code contained in the Police (Discipline)
Regulations, 1952. It followed that the suspension would continue either until it was decided that he would not be charged with
an offence against discipline or until any disciplinary proceedings were concluded. In December, 1957, the appellant was
committed for trial. An indictment dated 7 January 1958, charged him with the offence of conspiracy to obstruct the course of
public justice. The particulars alleged that he conspired with the four other accused and with other persons unknown to obstruct
the course of public justice in that the appellant and the two police officers accused should act contrary to their public duty as
police officers in relation to the administration of the law. The conspiracy was alleged to have been between 1 January 1949, and
18 October 1957. The trial began on 3 February 1958, and after a hearing which lasted for some nineteen days the appellant was
acquitted. That was on 27 February 1958. On 28 February his solicitors by letter requested the watch committee to remove his
suspension and to reinstate him. On that day two police officers who had been convicted by the jury were sentenced, and in
passing sentence the learned judge made certain observations in regard to the appellant. A second indictment had been preferred
against the appellant. The charge was that being a person serving under the Crown he corruptly obtained a gift of 20 from a
named person as a reward for showing favour to such person in relation to the affairs of the Crown. The appellant stood his trial
on that indictment on 6 March 1958. He pleaded Not Guilty. The prosecution offered no evidence. On the direction of the
learned judge the appellant was found not guilty. After the appellant had left the dock the learned judge made certain
observations in regard to the appellant.
On the following day, 7 March 1958, there was a meeting of the watch committee. The appellant had not been invited to
attend and was not sent for. He received a letter the same afternoon telling him that he had been summarily 95 dismissed. He
was informed of certain resolutions which the watch committee had passed. Information as to those resolutions was given to the
press. The resolutions which were passed by the watch committee as recorded in their minutes were as follows:

Resolved (unanimously):
(1) The committee after carefully considering
(a) the request by [the appellants] solicitors that [the appellants] suspension be removed and [the appellant] be
reinstated in his office as chief constable,
(b) the length of [the appellants] period of service in the Brighton Police Force,
(c) the trial of [the appellant], Detective Inspector J. R. Hammersley and Detective Sergeant T.E. Heath, two senior
members of the criminal investigation department of the Brighton Police Force and others on a charge of conspiring to
obstruct the course of public justice and the conviction of Hammersley and Heath and another,
(d) the statements of DONOVAN, J., on Feb. 28, 1958 and Mar. 6, 1958,
(e) the statements made by [the appellant] in evidence at his trial, and
(f) certain statements made today by members of the committee and the town clerk,
decide that [the appellant] has in the opinion of the committee been negligent in the discharge of his duty and is unfit
for the same and the committee in exercise of the powers conferred upon them by s. 191 of the Municipal Corporations
Act, 1882, accordingly hereby dismiss him from his office as Chief Constable of Brighton forthwith.
(2) That in accordance with the provisions of reg. 7 of the Police Regulations, 1955 the amount of [the appellants]
aggregate pension contributions be paid to him.
(3) That the town clerk be requested to arrange for the foregoing resolutions to be conveyed to [the appellant] and to
Messrs. Bosley and Co.
(4) That resolutions (1) and (2) be made available to the press at a conference to be held at 2.45 p.m. this day and the
chairman and the town clerk be requested to inform the press that no other statement will be made or questions answered in
amplification thereof.
(5) That no statements or disclosures be made by members of the committee concerning the matter other than the
foregoing resolutions.
Your Lordships were informed that the transcript of the proceedings at the criminal trial were not available for the watch
committee on 7 March but that there was a transcript of the statements which had been made by the learned judge on 28 February
and 6 March.
Section 191(4) of the Municipal Corporations Act, 1882, is in the following terms:

The watch committee, or any two justices having jurisdiction in the borough, may at any time suspend, and the watch
committee may at any time dismiss, any borough constable whom they think negligent in the discharge of his duty, or
otherwise unfit for the same.

By s 4(1) of the Police Act, 1919, it was provided:

It shall be lawful for the Secretary of State to make regulations as to the government, mutual aid, pay, allowances,
pensions, clothing, expenses and conditions of service of the members of all police forces within England and Wales, and
every police authority shall comply with the regulations so made.

At the material times the following regulations made by the Secretary of State pursuant to that power were in force: (a) Police
Regulations, 1952, SI 1952 No 1704; (b) Police (Discipline) Regulations, 1952, SI 1952 No 1705, as amended by SI 1954 No
1687; and (c) Police (Discipline) (Deputy Chief Constables, 96Assistant Chief Constables and Chief Constables) Regulations,
1952, SI 1706, as amended by SI 1954 No 1688. The regulations, SI 1952 No 1705, show that a member of a police force
commits an offence against discipline if he commits any of the offences which are set out in the discipline code contained in Sch
1 to the regulations. The regulations contain detailed provisions as to the procedure which must be followed where a report or
allegation is received from which it appears that a member of a police force may have committed an offence. Chief constables,
deputy chief constables and assistant chief constables are governed by the regulations, SI 1952 No 1706, which were in fact made
on the same day as those in SI 1952 No 1705. These officers are also subject to the discipline code and the regulations, SI
1952 No 1706, contain detailed provisions as to the procedure which must be followed where a report or allegation is received
from which it appears that a chief constable may have committed an offence, ie an offence contained in the discipline code (see
reg 18).
Amongst the many offences included in the discipline code are the following:

1. Discreditable conduct, that is to say, if a member of a police force acts in a disorderly manner or any manner
prejudicial to discipline or reasonably likely to bring discredit on the reputation of the force or of the police service.
4. Neglect of duty, that is to say, if a member of a police force
(a) neglects, or without good and sufficient cause omits, promptly and diligently to attend to or carry out anything
which is his duty as a constable,
5. Falsehood or prevarication, that is to say, if a member of a police force (b) wilfully or negligently makes any
false, misleading or inaccurate statement,
17. Conviction for a criminal offence, that is to say, if a member of a police force has been found guilty by a court of
law of a criminal offence.

If there is a report or allegation from which it appears that a chief constable may have committed an offence against the
discipline code then the police authority mustl
________________________________________
l SI 1952 No 1706, reg 1, applied by reg 18

unless they are satisfied that he has not committed an offence inform him in writing of the report or allegation and ask
him whether or not he admits that he has committed an offence and give him an opportunity, if he so desires, of making to
the police authority any oral or written statement he may wish to make concerning the matter.

Regulation 2 (of SI 1952 No 1706) as applied to chief constables by reg 18 provides that if the chief constable

admits that he has committed an offence, the police authority may impose a punishment in accordance with reg. 11 of
these regulations without the case being heard in accordance with the following provisions of these regulations.

The power to punish for an offence without a hearing was therefore made conditional on there being an admission of an offence.
Thereafter if the police authority are not satisfied with the statement of a chief constable who does not admit that he has
committed an offence the police authority must instruct a solicitor to enter on a discipline form the offence with which the chief
constable is charged and must give such particulars as will leave the chief constable in no doubt as to the precise nature of the
alleged offence. That having been done a chief constable must be supplied with (a) a copy of the discipline form (b) a copy of
the report or allegation on which the charge is founded and any reports thereon notwithstanding that they may be confidential (c)
a copy of any statement relating to the charge made by any witness to be called in support of the charge together with the
witnesss name and address and (d) a copy of any statement relating to the charge made by any person other than a witness to be
called in support of the charge, to the police authority or to anybody on their behalf, 97together with the persons name and
address. Thereafter there must be a hearing by a tribunal (which could consist of a person selected from a list of nominated
persons and assisted on matters pertaining to the police by an assessor or could consist of not more than five members of the
police authority). The regulations lay down the procedure for the hearing and provide that after the hearing the tribunal must
submit a report to the police authority and send a copy of it to the accused chief constable. The report must contain (inter alia) a
statement as to the facts found or admitted, as to the charges found to be proved, and as to any recommendation as to any
punishment. Then the police authority must come to a decision. That will be only on receipt of the report of the tribunal. They
may decide to dismiss the case: alternatively they may decide to impose any one of the following punishments (i) dismissal (ii)
requirement to resign either forthwith or on such date as may be specified in the decision as an alternative to dismissal (iii)
reprimand (see reg 11).
By the Police Pensions Act, 1948, it was provided that regulations to be made by the Secretary of State were to make
provision as to the pensions to be paid whether as of right or otherwise and also as to the times at which and the circumstances
in which members of police forces are or may be required to retire otherwise than on the ground of misconduct. The Police
Pensions Regulations, 1955 (SI 1955 No 480), contain the following provisions:

1.(1) Subject to the provisions of these regulations, every man or woman who is a regular policeman, that is to say,
a member of a home police force who is not an auxiliary policeman and a member of an overseas corps who is a
reversionary member of a home police force, shall, on retiring from the force of which he is a member, be entitled to an
award under these regulations.
3.(1) Subject to the provisions of these regulations, where a regular policeman who is entitled to reckon twenty-five
years pensionable service retires from a police force, the award shall be an ordinary pension.
7.(2) Where a member of a police force is dismissed the police authority shall pay an amount equal to the amount of
his aggregate pension contributions to such one of those persons hereinafter described as, in their discretion, they may
think fit or, if in their discretion they think fit, shall distribute that amount among such of those persons in such shares and
in such manner as in their discretion they may think fit.
52. If a police authority determine that the retention in the force of a regular policeman who if required to retire is
entitled to receive a pension of an amount not less than two-thirds of his average pensionable pay would not be in the
general interests of efficiency, he may be required to retire on such date as the police authority determine.

It is to be observed that s 191(4) of the Municipal (Corporations) Act, 1882, gives to the watch committee a power of
suspension. As I have already mentioned, a power to suspend a chief constable is also given to the police authority by the
regulations, SI 1952 No 1706, as amended. The case has proceeded on the basis that the watch committee suspended the
appellant in October, 1957, under the powers given to them by those regulations. At the trial of the action the appellant gave
evidence that after his arrest on 25 October 1957, the town clerk came to see him on the same day and said that he had been
suspended from duty and the appellants recollection was that the town clerk added under police regulations or in accord with
police regulations. The appellants recollection as to this was not challenged in cross-examination. The suspension allowances
which thereafter were paid to the appellant were payable because on 28 October 1957, the watch committee resolved that in
accordance with reg 15 of the regulations such allowances should be paid. It is not suggested by the appellant that there was any
irregularity in his suspension. The power to suspend arose because the watch committee must have received a report or
allegation from which it appeared that 98 the appellant may have committed some offence against discipline (ie some offence
against the discipline code).
It seems very probable that the watch committee received an oral report that the appellant had been arrested on criminal
charges, and it is abundantly clear that, if he were guilty of criminal charges, he would have committed one or more of the
offences set out in the discipline code. Quite apart from various other offences he would have been guilty of the offence under
para 17 of the discipline code as previously set out.
After the appellant was acquitted of all criminal charges there were various courses which were open to the watch
committee. They could have decided not to charge the appellant with any offence against discipline. In that event his
suspension would have ceased. They could have decided to charge him with some offence or offences against discipline. In that
event the suspension would have continued. That would be on the basis that there was some report or allegation that the
appellant may have committed an offence against discipline. The appellant would then have had the right to be informed of, and
to make a personal explanation concerning, any such report or allegation and the procedure laid down in the regulations would
have to be followed. Had there been disciplinary proceedings, the appellant would have had all the opportunities to defend
himself which the regulations give. If any charges were found to be proved and if the case were not dismissed, then there might
have been dismissal or a requirement to resign in lieu of dismissal or a reprimand. Another course which was open to the watch
committee was to consider (pursuant to reg 52 of the Police Pensions Regulations, 1955, SI 1955 No 480) whether the appellant
was one who if required to retire was entitled to receive a pension of an amount not less than two-thirds of his average
pensionable pay, and, if so, to consider whether the retention of the appellant in the force would not be in the general interests of
efficiency, and to decide whether to require the appellant to retire.
The documents show what the watch committee did. The documents further show their reasons for doing what they did.
What they did was summarily to dismiss the appellant without any prior communication of any sort to him and without inviting
any submission from him. They purported to exercise powers given by s 191(4). It is beyond dispute that the procedure of the
regulations was in no way operated. The issue that is raised is therefore whether the powers given by s 191(4) may be invoked
without paying any regard to the provisions contained in regulations, SI 1952 No 1706. The further issue that is raised is whether
such powers may also be invoked without paying regard to those principles which are conveniently referred to as the principles of
natural justice. I propose to deal with these issues separately.
The powers given by s 191(4) are impressive. There is, first, a power to suspend. It may well be that different
considerations apply to suspensions as compared to dismissals. It may well be that a power to suspend if exercised by a watch
committee in good faith may have to be exercised without any hearing and without any procedural requirements. That does not
have to be decided in the present case. The power to dismiss (given by the section to a watch committee but not to justices)
relates to any borough constable (which term includes a chief constable) whom the watch committee think negligent in the
discharge of his duty or otherwise unfit for the same. My lords, I consider that in the context the word otherwise denotes that
there may be dismissal of a constable if the watch committee considers that he is unfit for the discharge of his duties even though
he may not have been negligent in their discharge. In the section it seems to me that the words unfit for the same were
designed to cover situations where even apart from any misconduct or lack of care and even apart from any physical or health
condition a constable was thought to be unfit for the discharge of his duty. It was not contended before your lordships that s
191(4) has been impliedly repealed. Having regard to s 1(4) of the Police Act, 1946, and para 3(2) in Sch 2 to that Act, that
would have been a difficult contention to advance. But 99 though the powers given by s 191(4) are still exercisable I consider, in
agreement with the Court of Appeal, that the effect of the Police Act, 1919, is that the powers given by s 191(4) must be exercised
in accordance with any regulations made under the Police Act, 1919, which are applicable.
Pearce LJ said ([1962] 1 All ER at p 842) that in cases coming within the regulations the statutory power of the watch
committee must be used in accordance with the regulations, and that in such cases the watch committee must act judicially or
quasi-judicially. Harman LJ ([1962] 1 All ER at p 848), said that in cases to which the regulations are applicable the power to
dismiss given by s 191(4) is controlled. Davies LJ ([1962] 1 All ER at p 851), likewise agreed that the power is controlled by the
regulations. These conclusions followed and were in accord with the judgment of Greer LJ, in Cooper v Wilson. In that case
Greer LJ while rejecting a suggestion that the power to make regulations under s 4(1) of the Police Act, 1919, had impliedly
repealed s 191(4) of the Act of 1882 said ([1937] All ER at p 730; [1937] 2 KB at p 316):

The regulations, in my judgment, must be read as applying to the way in which the watch committee are to exercise
their powers in a borough

It may well be that the various Police Regulations and Police Pensions Regulations were designed to cover all the circumstances
and situations with which police authorities are likely to be faced, and in practice I would think that police authorities would
invariably wish to follow the spirit as well as the letter of the carefully devised procedures which the regulations lay down. As a
matter of construction, however, I am not prepared to say every power is controlled by the regulations: they do not appear to
make provisions in regard to the power to dismiss a constable who is thought by the watch committee to be unfit for the discharge
of his duties. If, however, a constable is thought to have been negligent in the discharge of his duties, and so guilty of an offence
under s 4 of the discipline code, or if he is thought to have been guilty of some other offence, then the provisions of the
regulations would be applicable.
The action of the watch committee in summarily dismissing the appellant was stated to be for two reasons. In the first place
they decided that the appellant had been negligent in the discharge of his duty. In the second place they decided that he was unfit
for the discharge of his duty. Had they been merely of the opinion that the appellant had become unfit for the discharge of his
duty, but had not been in any way negligent in the discharge of his duty, it would seem to be inherently unlikely that they would
have exercised a power of summary dismissal. They had appointed the appellant in 1956, after he had been successively
promoted in the period of years after 1925, when he first joined the Brighton Borough Police Force. He was nearing the time
when he could have retired on a pension. If the watch committee thought that there were reasons why his retention in the force
would not be in the general interests of efficiency they could have required him to retire (see reg 52 of the Police Pensions
Regulations, 1955). If the watch committee had thought that there was no element of misconduct and no suggestion of
negligence in the discharge of duty, then, assuming that the power given by s 191(4) to dismiss, where there is unfitness for duty,
is a power which is not governed by and has not been affected by regulations which have been made, then on such hypothesis I
do not suppose that summary dismissal would have been contemplated. In fact there was a decision in the discharge of his duty.
There was therefore a finding of misconduct, although there had not been a charge and although there had not been a hearing. A
member of a police force is guilty of an offence if he neglects or without good and sufficient cause omits promptly and
diligently to attend to or carry out anything which is his duty as a constable. I do not think that it can be open to doubt that, if
someone is found to have been negligent in the discharge of his duty, 100he is found to have been guilty of the offence defined
by these words. The watch committee therefore found the appellant guilty of this offence ad summarily dismissed him for it.
They found him guilty without giving him particulars and without charging him and without giving him any opportunity to
defend himself. They made no attempt to pay heed to the regulations. The explanation of this that is advanced is that they were
not obliged to do so, because they had not received any report or allegation from which it appeared that the appellant might have
committed an offence. I find this a most surprising suggestion. If they had not received any report or allegation to the effect that
he might have committed an offence then why did they suddenly decide that he had committed an offence? How could they find
him guilty of being negligent in the discharge of his duty without some suggestion or some information which amounted to a
report or allegation to that effect? Indeed, it is difficult to understand how the committee could ever act in a disciplinary matter
without first having some report or some allegation that an offence may have been committed. The minutes of the meeting record
that they carefully considered the trial of the appellant and the other accused. The trial had lasted some nineteen days. The
transcript of the evidence was not before the committee on 7 March. If the committee had acquired knowledge that in the
proceedings at the trial there was some material which might lead to the view that the appellant had been negligent in the
discharge of his duty, then they were treating that material as a report or allegation. The minutes further record that the
committee considered the statements made by the learned judge on 28 February and 6 March. They had transcripts before them
of what the learned judge had said. The statements made by the learned judge manifestly called for the most careful
consideration by the watch committee who would obviously pay the greatest heed to them. One of the statements besides
commenting on the failure of the appellant to give proper leadership raised two specific matters, viz (1) that the appellant had
contrived to go to a suspected briber of the police in private (that was a reference to a man named Leach) and (2) that the
appellant had admitted a much convicted and hectoring bookmaker to his private room and had discussed with him the policy
of the police in certain matters (that was a reference to a man named Page). My lords, I find it impossible to say that in
considering those statements the committee had not received, and were not considering, a report or allegation to the effect that the
appellant might have been guilty of an offence. The learned judge on 6 March had said in terms that he realised that the matter of
the leadership of the Brighton Police Force was about to engage the attention of those persons whose responsibility it is. No
words used by the learned judge, however, could or did in any way suggest that the matter was to be dealt with in disregard of the
requirements of the law.
The minutes of the watch committee further record that the committee considered certain statements made today by
members of the committee and the town clerk. If those were statements that might lead members to conclude that the appellant
had been negligent in the discharge of his duty then they must have been reports or allegations from which it appeared that the
appellant might have committed either the offence of neglect of duty or some other offence. As no evidence was called on behalf
of the watch committee at the trial of the action, it was not possible to ask any member of the watch committee what these
statements were. There is, however, material from which their nature may be reasonably inferred. In circumstances to which
reference must later be made, the solicitors for the appellant gave notice of appeal to the Secretary of State against his dismissal,
while stating that the appeal was without prejudice to the appellants rights to contend that the purported notice of dismissal was
bad in law as being contrary to natural justice and not in accordance with the appropriate statutes and regulations. Thereafter in a
written statement submitted by the watch committee to the Secretary of State the facts and contentions on which reliance was
placed included the following:

(c) In the course of the said trial the appellant gave false evidence in 101 respect of two matters of material
importance, namely (i) that he had reported to the deputy town clerk and also to the chairman of the watch committee the
facts relating to an interview between Alderman Cullen and one Page, and (ii) that he had reported to the then chief
constable the facts relating to an interview which he (the appellant) had had with one Mrs. Cherryman.
(d) The appellant failed to investigate and to take any action whatsoever in respect of complaints of a serious nature
made by the said Mrs. Cherryman as to the manner in which the Astor Club was conducted and as to the trustworthiness of
the Brighton Criminal Investigation Department.
(e) Following a report of an attempt to bribe a police officer, the appellant went to the house of the man concerned,
namely Harry Leach, and there interviewed him privately and alone.
(f) The appellant permitted a man with a criminal record, namely the said Page, to interview him in his private room
and to discuss with him matters of police policy.
(i) The [watch committee] contend that, having regard to the facts and matters aforesaid, the appellant has both been
negligent in the discharge of his duty and is also unfit for the same and that, therefore, they were entitled to dismiss him
pursuant to the provisions of s. 191 of the Municipal Corporations Act, 1882.

It is in my judgment a reasonable inference that the statements made at the meeting of the watch committee covered the matters
set out under (c). If someone made the serious suggestion that the appellant had at his criminal trial given false evidence on
material matters, that was surely a report or allegation from which it appeared that the appellant might have committed an
offence. There would have been an offence under para 5 of the discipline code.
The watch committee were under a statutory obligation (see Police Act, 1919, s 4(1)) to comply with the regulations made
under the Act. They dismissed the appellant after finding that he had been negligent in the discharge of his duty. That was a
finding of guilt of the offence of neglecting or omitting diligently to attend to or to carry out his duty. Yet they had preferred no
charge against the appellant and gave him no notice. They gave him no opportunity to defend himself or to be heard. Though
their good faith is in no way impugned they completely disregarded the regulations and did not begin to comply with them. My
lords, I cannot think that any decision so reached can have any validity and unless later events have made it valid it ought not to
be allowed to stand. Had the regulations been applied but if there had been some minor procedural failure different
considerations might have applied. There was, however, no kind of compliance with them. In my judgment once there was a
report or allegation from which it appeared that a chief constable might have committed an offence it was a condition precedent
to any dismissal based on a finding of guilt of such offence that the regulations should in essentials have been put into operation.
They included and incorporated the principles of natural justice which, as Harman LJ ([1962] 1 All ER at p 850), said, is only fair
play in action. It is well established that the essential requirements of natural justice at least include that before someone is
condemned he is to have an opportunity of defending himself and in order that he may do so that he is to be made aware of the
charges or allegations or suggestions which he has to meet; see Kanda v Government of The Federation of Malaya. My lords,
here is something which is basic to our system: the importance of upholding it far transcends the significance of any particular
case.
The circumstances existing in March, 1958, made it, in my judgment, particularly necessary quite apart from its being a
matter of legal obligation that before considering whether the appellant had been negligent in the discharge of his duty the
procedure of the regulations should be observed. The appellant had after a 102 long trial been acquitted on criminal charges.
There had been much publicity. He had not been on trial on charges of neglect of duty or of unfitness for duty. If any facts had
emerged which, while insufficient to support the charges at the criminal trial, had seemed to the watch committee to suggest that
the appellant had been negligent in the discharge of his duty it would have been so easy to state them. If at the criminal trial any
admission had been made or evidence given which seemed to support the view that there had been negligencehow simple it
would have been to state it. If some facts were clear and plainso that denials would have been unlikely or explanations
difficultthen the opportunity to make a statement might have shown a course ahead. But if, on the other hand, facts could be
explained and if conduct could be defended and if charges of neglect or of unfitness could be repelled, was the appellant to be
denied a hearing? It is to be noted that whatever suggestion or charge might be formulated in regard to the Leach matter raised
questions as to the desirability or propriety of a visit that had taken place as far back as 1954, which was some time before the
appellant was appointed to be chief constable. The appellants case was that he had told his then chief constable in advance of his
proposed visit to Leach. His case further was that neither in respect of the Leach matter nor in respect of the Page matter was
there any impropriety in his conduct or actions. On the charges brought against him in a court of law and on which he was tried
he was found not guilty. If a new charge of neglect of duty was to be brought against him was he not even to be told about it or
asked about it? Was he to have no chance of dealing with matters which may have influenced the committee? Were the
safeguards of a criminal trial, of which the law is a jealous protector, to find no reflection in the days that followed an acquittal?
My lords, before further considering the result of disregarding the regulations it becomes necessary to mention certain
events that followed the dismissal. The solicitor for the appellant addressed a letter to the Secretary of State on 7 March 1958, in
which he contended that the dismissal was contrary to natural justice and bad in law and gave notice of appeal. The letter pointed
out that the notice of dismissal had merely recited a general finding of negligence and unfitness without specifying any details.
The Police (Appeals) Act, 1927, as amended by the Police (Appeals) Act, 1943, by s 1(1) provides:

A member of a police force who after the passing of this Act is punished by dismissal, by being required to resign as
an alternative to dismissal, by reduction in rank, or a reduction in rate of pay, may appeal to a Secretary of State in
accordance with this Act and the rules made thereunder, if he gives notice of appeal in the prescribed manner and within
the prescribed time.

The Police (Appeals) Rules, 1943 (SI 1943 No 473), which apply to all appeals by a member of a police force provide m that
notice of appeal must be sent within ten days from the date when the appellant received on the misconduct form the notification
of the decision against which he desires to appeal. The solicitor for the appellant followed his letter of 7 March with another
dated 10 March in which he stated that the appeal was without prejudice to the appellants rights to contend that the purported
dismissal was bad in law as being contrary to natural justice and not in accordance with the appropriate statutes and regulations.
In his notice of appeal which was dated 12 March he set out some thirty grounds of appeal. While denying any neglect or any
unfitness, he set out that he had been given no notice of what was alleged against him and no opportunity of being heard. He
further set out that by lodging his appeal he did not recognise the legality of the watch committees decision, and that his appeal
was without prejudice to his contentions that the watch committees decision was invalid, and he stated that his notice of appeal
was only given within the limited time in case it should be held that the watch committees procedure was valid. In due 103
course a written statement dated 18 April 1958, was submitted to the Secretary of State on behalf of the watch committee: it set
out the facts and contentions on which the watch committee relied in opposing the appeal.
________________________________________
m See reg 2(2)

Following on the dismissal of the appellant his solicitor made a request to appear before the watch committee. He wished to
be informed about the case against his client so as to be able to deal with it, and furthermore he wished to submit that the best
way of dealing with the situation would be to allow his client to resign and to have his pension. A copy of the appellants written
statement to the Secretary of State and in addition some written observations were sent to the watch committee. In those
observations it was submitted that the appellant should be allowed to retire on full pension forthwith. The committee decided to
meet on 19 March and stated that they would consider any representations which were then made by or on behalf of the appellant
either orally or in writing, and that such representations need not be limited to the matter of the pension. The appellants solicitor
attended and addressed the watch committee. In the course of his address he stated that before being dismissed the appellant had
been given no notice of what was charged against him or of being heard. The solicitor was received with courtesy, but in silence.
It seems, however, to be beyond dispute that he was given no further particulars of the case against the appellant than appeared in
the letter of 7 March. The watch committees later submissions to the Secretary of State which were dated 18 April were, of
course, not then available. The result of the meeting of 18 March was that the watch committee by a majority resolved to adhere
to their previous decision: nine members voted in favour of such resolution and three members voted against it.
The written statement (dated 18 April 1958) submitted to the Secretary of State by the watch committee set out their
contentions, and they included the paragraphs to which I have already referred. The Secretary of State decided that the case
could be determined without taking oral evidence (see Police (Appeals) Act, 1927, s 2) and on 5 July 1958, he dismissed the
appeal. He came to the conclusion that there was sufficient material on which the watch committee could properly exercise their
power of dismissal under s 191(4). He did not take into account, as no evidence in support of them was before him, certain
allegations on which the watch committee relied, viz (i) that the appellant did not report to the deputy town clerk and to the
chairman of the watch committee the facts relating to an interview between Alderman Cullen and one Page; (ii) that the appellant
did not report to the then chief constable the facts relating to an interview which the appellant had with one Mrs Cherryman; and
(iii) that the appellant in giving evidence at his trial that he had so reported those matters gave false evidence.
My lords, in my judgment, inasmuch as the decision of the watch committee was that the appellant had committed an
offence or offences against the discipline code and inasmuch as the decision was arrived at in complete disregard of the
regulations it must be regarded as void and of no effect. The power to dismiss for an offence was a power that could only be
exercised if the procedure of the regulations was set in motion. A purported dismissal in complete disregard of them cannot be
recognised as having any validity. In Andrews v Mitchell a member of a friendly society, who had been duly summoned before
the arbitration committee for a breach of the rules, was in his absence expelled from the society by a resolution of the committee
on a different charge, ie of fraud and disgraceful conduct of which no written notice had been given to him as required by the
rules. By one of the general laws of the friendly society any member proved guilty of fraud or any conduct or offence calculated
to bring disgrace on the order before any recognised arbitration committee, provided a charge had been preferred against him as
required by the general laws with regard to arbitrations, might be expelled or suffer some less penalty. It was held that the
decision of the committee 104 was null and void. In his speech ([19047] All ER Rep at p 600; [1905] AC at p 80) the Earl of
Halsbury LC said that there are some principles of justice which it is impossible to disregard. He pointed out that while there
was a rule which justified expulsion it justified expulsion on the express proviso that the charge had been made as provided by
the rules. He added ([19047] All ER Rep at p 600; [1905] AC at p 81):
In this case the charge never was made as provided by the rules; and if you have no power given under the rules to
expel a member except upon a charge made and tried according to the rules, you have no power to expel in a case like
this.

He described the summoning of a member pursuant to the rules and giving him time to consider what he had to do and giving
him the charge against him in writing as matters of substance and not mere matters of form. He concluded that the arbitration
committee had no jurisdiction to entertain the matter. Lord Davey said ([19047] All ER Rep at p 601; [1905] AC at pp 82,
83):

It is not contended that this charge was properly made according to the rules, but it is said that it may be regarded that
as a mere informality which might be set right. But it was an informality which went to the root of the jurisdiction, and the
omission to follow the directions of the rules for preferring charges has had the unfortunate effect of making the resolution
which was come to for the expulsion of the respondent, in my opinion altogether invalid, null and void.

My lords, if the regulations were applicable in this case, as in my judgment they were, reg 2 of SI 1952 No 1706, to which I have
referred above, only gives a power to impose punishment without a hearing if a condition is satisfied, viz, if there is an admission
of the commission of an offence. In the present case there was no such admission and the watch committee therefore lacked
power to impose punishment for an offence without a hearing: in purporting to dismiss the appellant they acted without
jurisdiction and their decision was a nullity.
In Lapointe v Lassociation de Bienfaisance et de Retraite de la Police de Montreal, the appellant, who was a member of the
respondent benevolent and pension society, had been obliged to resign from the police force. Under those circumstances he
became entitled according to the rules to have his case for a gratuity or pension considered by the board of directors and his right
to such gratuity or pension determined by a majority of the board. The board in fact acted in a most extraordinary manner. In
delivering the judgment of the Privy Council Lord MacNaghten said ([1906] AC at p 538):

They first appointed a committee of four from their own body to investigate the reason of Lapointes resignation.
There would have been no objection to this course if the committee had been deputed to consider and report whether or not
there was a prima facie case for inquiry. But what the committee did was to listen to all sorts of stories about Lapointes
past history, and rake up everything that was against him during his connexion with the force. Then, without telling
Lapointe what the charges against him were, or giving him any opportunity of defending himself, they advised the board
that the pension should be refused. Thereupon the board abnegated their judicial duties altogether. They summoned a
general meeting of the members and submitted a question, which they were bound to determine themselves, to a popular
vote. The meeting was held on Apr. 26, 1892, when by a large majority of the members present it was resolved that
Lapointes name should not be entered on the pension roll of the society.
The whole of these proceedings were irregular, contrary to the rules of 105 the society, and above all contrary to the
elementary principles of justice. And the position of the board was certainly not improved by a formal resolution stating
solemnly, what was contrary to the truth, that after having inquired into the facts and circumstances which brought about
Lapointes resignation, and having deliberated upon his claim, the board desires that the pension on which he claims be
refused, seeing that he was obliged to tender his resignation.

Lord MacNaghten said ([1906] AC at p 451) that it was obvious that the so-called determination of the board was void and of no
effect, and the order which they humbly advised included a declaration and determination as required by the rules and that the
proceedings were null and void.
In Annamunthodo v Oilfields Workers Trade Union, it was held in the Privy Council that a decision of the general council of
the trade union was vitiated because it convicted the appellant of an offence against the rules with which he had never been
charged and it was held that it should be declared that the purported expulsion of the appellant was invalid and that an order
should be made to set it aside. My lords, so here should it, in my judgment, be declared that the purported termination on 7
March 1958, of the appellants appointment was void unless it be that later events debar the appellant from obtaining this relief.
If they do not then the effect of such a declaration will be that the respondents will have to consider what action to take and in
any course that they follow they must act according to law.
The appellant, through his counsel, has stated that he has no intention of applying for reinstatement, but would be content to
retire (as from March, 1958), with his pension. I apprehend that in all the circumstances it would not be appropriate for your
lordships to do more than to declare that the purported termination on 7 March 1958, of the appellants appointment was void.
Included in the other claims of the appellant in his action is a claim for his salary as from 7 March 1958. It would not seem
appropriate at the present stage to deal with the appellants claim for salary, and it would not be for your lordships to decide any
question as to a pension.
The question next arises whether the events subsequent to 7 March form any bar to the appellants claim. I have already
referred to the meeting of 18 March. That occasion afforded an opportunity for the watch committee to tell the appellant and his
solicitor what were the allegations that he had to meet. The documents which the solicitor sent to the watch committee
emphasised the point that the appellant had been given no notice of them: mention was also made of the fact that the appellant
did not know what were the statements made by members of the watch committee referred to in the letter of 7 March. He most
certainly had no hint that it was being said of him that he had given some false evidence. The appellants case is that he never
had the chancewhich he would have welcomedof refuting that suggestion before the watch committee and the chance of
calling such evidence as he might desire to call to deal with the suggestion. The oral request for information made by the
solicitor met with no response. Even though the watch committee had communicated their previous decision to the press a full
inquiry might still have been possible but the watch committee neither took the opportunity then to begin compliance with the
regulations nor even, in default of that, to give information to the appellant as to the case that he had to meet. In the result, in my
judgment, nothing occurred on 18 March to give validity to what the watch committee had purported to do on 7 March. Nor in
my view did the action of the appellant in appealing to the Secretary of State have any such effect. If the decision of 7 March
was a nullity and void the fact that the appellant appealed made no difference. The decision of 7 March remained a nullity. The
appellant made it as plain as possible that he was adhering to and was in no way abandoning his submission that the decision of 7
March had no validity. In 106 these circumstances the provision in s 2(3) of the Police (Appeals) Act, 1927, that the decision of
the Secretary of State on an appeal is to be final and binding upon all parties cannot produce the result that validity is given to
that which is a nullity.
The watch committee referred to s 220 of the Municipal Corporations Act, 1882. That section was not pleaded and was not
mentioned in the watch committees case, but it was argued that it could be relied on in support of the contention that the court
could not declare against the validity of the decision of the watch committee. I deal with the point because, if it had validity, it
would go to jurisdiction. I would not regard the complaints of the appellant as covered by the words want of form, nor would I
regard the words removed by certiorari or otherwise as apt to exclude the claims made in this action. Furthermore it would
clearly be contrary to the intention of s 11(1) of the Tribunals and Inquiries Act, 1958, if its effect on s 220 could be construed as
having the result that the court could make an order of certiorari, but could not entertain an action for a declaration.
In view of the opinions which I have expressed as to the applicability of the regulations and as to the consequences of
disregarding them I propose only to deal briefly with the question whether, had there been no regulations, the police authority
would have been bound to have regard to the principles of natural justice. In my view the regulations incorporate those
principles, but had there not been any and had the police authority in the exercise of powers given them by s 191(4) contemplated
dismissing the appellant on the ground of neglect of duty, they would in my view have been under obligation to give him an
opportunity to be heard and would have had to consider anything that he might say. I cannot think that the dismissal of the
appellant should be regarded as an executive or administrative act if based on a suggestion of neglect of duty: before it could be
decided that there had been neglect of duty it would be a pre-requisite that the question should be considered in a judicial spirit.
In order to give the appellant an opportunity to defend himself against a charge of neglect of duty he would have to be told what
the alleged neglect of duty was.
In a case in which a consistory court had made an order requiring a vicar to pay certain expenses and costs, but had given
him no opportunity of being heard in his defence, a writ of prohibition directed to the chancellor was issued (see R v North, Ex p
Oakey) and Scrutton LJ said ([1927] 1 KB at p 502):

In my view an order that any one shall pay the cost of work that has been obliterated without a faculty is in the nature
of a penalty for an ecclesiastical offence, and one of the most fundamental principles of English law is that if you are going
to impose on a person a penalty for an offence you must first clearly inform him that an application to that effect is going to
be made against him, so that he may know what he is charged with and have an opportunity of attending to meet it.

The proceedings in the consistory court were therefore without jurisdiction and prohibition lay. The application of the
fundamental principle is, however, not limited to proceedings in courts or to cases where penalties for offences are being
imposed. The conduct of James Bagg (see Baggs case) was hardly commendable but it was held that it did not give good cause
for his disfranchisement. In any case he had not been heard and the court said ((1615), 11 Co Rep at pp 98b, 99a):

And although they have lawful authority either by charter or prescription to remove any one from the freedom and that
they have just cause to remove him: yet it appears by the return that they have proceeded against him without hearing him
answer to what was objected or that he was not reasonably warned, such removal is void and shall not bind the party. Such
a removal was against justice and right.
107

So also did the courts come to the aid of Dr Bentley and grant a peremptory mandamus to restore him to his degree ( R v
University of Cambridge). Though the court was roundly critical of Dr Bentleys behaviour they considered that even if he had
been guilty of a contempt to the Vice-Chancellors court that court had no power to deprive him of his degrees: but they held that
in any event he could not be deprived without notice. The words of Fortescue J ((1723), 1 Stra at p 566), were emphatic:

Besides, the objection for want of notice can never be got over. The laws of God and man both give the party an
opportunity to make his defence if he has any.

In Wood v Woad, Kelly CB in speaking of the rule expressed in the maxim audi alteram partem said ((1874), LR 9 Exch at p
196):

This rule is not confined to the conduct of strictly legal tribunals but is applicable to every tribunal or body of persons
invested with authority to adjudicate upon matters involving civil consequences to individuals.

The relationship between the watch committee and the appellant was not that of master and servant. Nor was the appellant
one who held an office at pleasure with the consequence that he could be required at pleasure to relinquish it. He was in a
different position from someone possessing a licence to do various acts. The appellant held an office from which the watch
committee could at any time dismiss him if they thought he had been negligent in the discharge of his duty. The watch committee
did not however have an unfettered or unrestricted discretion. If it be assumed that no regulation had been made, then the fact
that s 191(4) is silent as to any procedure for a hearing does not involve that there could be a dismissal without a hearing. The
justice of the common law would require it, for, as Byles J said in Cooper v Wandsworth Board of Works ((1863), 14 CBNS at
p 194):

a long course of decisions, beginning with Dr. Bentleys case and ending with some very recent cases, establish that
although there are no positive words in a statute requiring that the party shall be heard, yet the justice of the common law
will supply the omission of the legislature.

In that case it was held that, although s 76 of the Metropolis Management Act, 1855, empowered the district board to alter or
demolish a house where the builder had neglected to give notice of his intention to build seven days before proceeding to lay or
dig the foundation, yet the district board were not empowered to demolish the building without first giving the party guilty of the
omission an opportunity of being heard. Erle CJ said ((1863), 14 CBNS at p 189):

It has been said that the principle that no man shall be deprived of his property without an opportunity of being heard
is limited to a judicial proceeding and that a district board ordering a house to be pulled down cannot be said to be doing a
judicial act. I do not quite agree with that; neither do I undertake to rest my judgment solely upon the ground that the
district board is a court exercising judicial discretion upon the point: but the law, I think, has been applied to many
exercises of power which in common understanding would not be at all more a judicial proceeding than would be the act of
the district board in ordering a house to be pulled down.

So Willis J said ((1863), 14 CBNS at p 190):

I apprehend that a tribunal which is by law invested with power to affect the property of one of Her Majestys subjects
is bound to give such subject an opportunity of being heard before it proceeds: and that that rule 108 is of universal
application and founded upon the plainest principles of justice.

So also in Spackman v Plumstead Board of Works, the Earl of Selborne LC said ((1885), 10 App Cas at p 240):

No doubt in the absence of special provisions as to how the person who is to decide is to proceed, the law will imply
no more than that substantial requirements of justice shall not be violated. He is not a judge in the proper sense of the
word: but he must give the parties an opportunity of being heard before him and stating their case and their view. He must
give notice when he will proceed with the matter and he must act honestly and impartially and not under the dictation of
some other person or persons to whom the authority is not given by law. There must be no malversation of any kind.
There would be no decision within the meaning of the statute if there were anything of that sort done contrary to the
essence of justice.
Lord Selborne was there speaking of the decision of an architect (under s 75 of the Metropolis Management Act, 1862) as to the
general line of buildings in a road. If the principles, to which he adverts, apply where property rights are in issue, surely they
must at least apply with equal effect where the issue is whether there has been misconduct which merits dismissal from an office.
It is to be remembered also that in the case of the appellant his summary dismissal involved the loss of valuable pension rights.
Property rights were at stake in Local Government Board v Arlidge. Viscount Haldane LC (([191415] All ER Rep at p 7; [1915]
AC at pp 132, 133), there expressed his approval of the view indicated by Lord Loreburn in Board of Education v Rice ([1911
13] All ER Rep 36 at p 38; [1911] AC 179 at p 182) that an administrative body to which the decision of a question in dispute
between parties has been entrusted must act in good faith and listen fairly to both sides. Lord Parmoor said ([191415] All ER
Rep at p 15; [1915] AC at p 142) that, whether in that case the order of the Local Government Board was to be regarded as of an
administrative or of a quasi-judicial character, if the order affected the rights and property of the respondent he was entitled to
have the matter determined in a judicial spirit in accordance with the principles of substantial justice. A right to be heard before
property rights were affected was upheld in the circumstances applying in Cooper v Wandsworth Board of Works, in Hopkins v
Smethwick Local Board of Health, and in Urban Housing Co Ltd v Oxford Corpn. Similarly a right to be heard in regard to
removal from an office was recognised in Osgood v Nelson, in Ex p Ramshay and in R v Gaskin. So also it has been recognised
that expulsion from a club must not take place in disregard either of the rules of the club or of the rules of natural justice. (The
cases of Fisher v Keane and Dawkins v Antrobus may be mentioned as typical examples.)
Being of the view that even if there had been no applicable regulations a decision to dismiss the appellant for neglect of duty
ought only to have been taken in the exercise of a quasi-judicial function which demanded an observance of the rules of natural
justiceI entertain no doubt that such rules were not observed. Before 7 March there was neither notice of what was alleged nor
opportunity to deal with what was alleged. It was contended that the criminal trial had been the appellants opportunity. My
lords, I cannot think that such a contention is valid. The trial was concerned with specific charges. In respect of them the
appellant was found not guilty. If there were other charges or charges of a different nature which were not submitted to the jury
but which the watch committee proposed 109 to consider, then it was for the watch committee to formulate them and only to
reach decision in regard to them after hearing and considering what the appellant or any witnesses of his had to say. For the
reasons that I have already given the hearing of 18 March did not remedy the previous defects. The consequence, in my view, is
that there was an abnegation of the quasi-judicial duties involved in the function of the watch committee with the result that their
decision must be regarded as of no effect and invalid, and so can be declared by the court to be void (see Baggs case, R v
University of Cambridge, Wood v Woad, Fisher v Keane).
It was submitted that the decision of the watch committee was voidable but not void. But this involves the inquiry as to the
sense in which the word voidable, a word deriving from the law of contract, is in this connexion used. If the appellant had
bowed to the decision of the watch committee and had not asserted that it was void, then no occasion to use either word would
have arisen. When the appellant in fact at once repudiated and challenged the decision, so claiming that it was invalid, and when
in fact the watch committee adhered to their decision, so claiming that it was valid, only the court could decide who was right. If
in that situation it was said that the decision was voidable, that was only to say that the decision of the court was awaited. But if
and when the court decides that the appellant was right, the court is deciding that the decision of the watch committee was invalid
and of no effect and null and void. The word voidable is, therefore, apposite in the sense that it became necessary for the
appellant to take his stand: he was obliged to take action for unless he did the view of the watch committee, who were in
authority, would prevail. In that sense the decision of the watch committee could be said to be voidable. The appellant could, I
think, have applied for an order of certiorari: he was not saying that those who purported to dismiss him were not the watch
committee; he was recognising that they had a power and jurisdiction to dismiss but he was saying that whether the regulations
applied or whether they did not the committee could only exercise their power and jurisdiction after hearing his reply to what was
said against him. In these circumstances he could, I think, have applied for an order of certiorari (though considerations of
convenience would probably have pointed against pursuing such a course) or he could have asked for a declaration. In either
proceeding the question of acquiescence by him might be raised or the question whether by some binding election he had barred
himself from taking proceedings in court or whether in some way he was estopped. It seems to me that he made it abundantly
clear that by his appeal to the Secretary of State he was not in any way abandoning his right to contend that the decision of the
watch committee was invalid. An appeal to the Secretary of State raises a question whether a decision, which as a decision has
validity, should or should not on the facts and on the merits be upheld. The question raised and reserved by the appellant was the
fundamental point that the purported decision of the watch committee was no decision. It would not have been unreasonable if
the Secretary of State had asked that that point should first be adjudicated; but in the events which happened I cannot think that
the careful steps which were taken to protect the appellants position ought to be held to have in fact compromised it. Compare
Annamunthodo v Oilfields Workers Trade Union. The appellant never abandoned his point and in my view nothing done by him
or by the Secretary of State gave validity to a decision which is now shown to have had none.
My lords, it was submitted to your lordships that the decision of the watch committee should be upheld as having been the
only reasonable decision. I consider this to be an entirely erroneous submission. Since no charges have been formulated it is
impossible to assess their weight or the weight of the answering evidence of the appellant and others. When the appellant was in
the witness-box 110 in the present action he was questioned as to what witnesses he would have wished to call in order to deal
with the Leach and the Page matters. As charges in respect of those matters were not formulated, I cannot think that it was
appropriate to elicit the names of certain witnesses whom the appellant might have decided to call and then without hearing or
being able to hear such witnesses to seek to discount their value and effectiveness and then to seek to draw a vague and artificial
conclusion that if matters had been regularly done and if the appellant had been heard and if his witnesses had been heard a result
adverse to him would have followed. All the defects and all the unfairness of the original irregularity are inherent in any such
approach. The suggested conclusion must fail because it is based on a perpetuation of the very defects which vitiate the dismissal
of the appellant and also because the process involves endowing the court with a function that belongs elsewhere.
I do not find it necessary to express any concluded opinion whether, if there were no suggestion of having been negligent in
the discharge of duty, a decision to dismiss on the ground of being unfit for the discharge of duty could be taken without giving
an opportunity to be heard. Clearly it would be desirable and reasonable to give such an opportunity even though the alleged
unfitness did not involve misconduct.
For the reasons that I have given I would allow the appeal.

LORD HODSON. My Lords, I have reached the conclusion apart from the application of the Police Act, 1919, and the
regulations which followed, that this appeal should succeed on the ground that the appellant was entitled to and did not receive
natural justice at the hands of the watch committee of Brighton when he was dismissed on 7 March 1958. Streatfeild J ([1961] 2
All ER at p 534), who heard the appellants suit at first instance held that the power given to the watch committee by the
Municipal Corporations Act, 1882, s 191(4), at any time to dismiss any borough constable, whom they think negligent in the
discharge of his duty or otherwise unfit for the same, was a power which had to be exercised in accordance with the principles of
natural justice but that the watch committee had acted in that manner. The Court of Appeal took a different view and held that the
watch committee were not bound, in taking the executive action of dismissing their chief constable, to hold an inquiry of a
judicial or quasi-judicial nature (per Pearce LJ ([1962] 1 All ER at p 844)). Harman LJ ([1962] 1 All ER at p 849), was of
opinion that the watch committee were acting in exercise of their administrative functions just as they were when they made the
appointment under s 191(1) of the Act of 1882 and that the principles of natural justice did not come into the case. He pointed
out that the defendants were not deciding a question between two opposing parties and that there was no lis and nothing to
decide. Davies LJ ([1962] 1 All ER at p 852), said that the exercise by the watch committee of their powers under s 191(4) of the
Act of 1882 was not a quasi-judicial but an executive one, emphasising the words whom they think as being very strong indeed
and much stronger than the sort of words to be found in most of the authorities cited to the court such as on sufficient grounds
shown to his satisfaction (De Verteuil v Knaggs).
I should add that Streatfeild J although holding that the principles of natural justice should prima facie have been applied,
held ([1961] 2 All ER at p 536) that in this case the appellant had at the Old Bailey, for the purposes of his trial for all the world
as well as the watch committee to hear, convicted himself of unfitness to hold the office of chief constable. The judge concluded
that on the evidence which the appellant had himself given at the Old Bailey there was no need for the watch committee to do
other than they in fact did, whatever also they might have done to be on the safe side. It would be unrealistic to suppose that the
watch committee had not a good idea of what took place at the criminal trial, 111although they were not provided with a
transcript of the evidence until after they had reached their conclusion, but in my opinion it will not do to say that the case was so
plain there was no need for the appellant to be heard and that, therefore, the claims of natural justice were satisfied.
What the watch committee had before them was primarily the observations of Donovan J who in the course of sentencing
the two police officers who were convicted, the appellant having been acquitted, gave as a ground for the moderation of his
sentences on these two men the extenuating circumstances that in his opinion they had not had the leadership to which they were
entitled. I think that it is clear from the learned judges observations at that time, and at the conclusion of the trial, that he
intended that what he said should be brought to the notice of the watch committee to act on as they thought fit. The watch
committee no doubt felt it necessary to act promptly, but there was nothing in the learned judges observations which would
suggest that the appellant could be dealt with on the basis that any charges had been proved against him and that no further
hearing was required.
I do not find that the answer put by counsel for the watch committee to your lordships that the case was as plain as a pike-
staff is an answer to the demand for natural justice. The case on natural justice does not rest on the events of 7 March 1958, alone
for the appellant was given a further opportunity on 18 March 1958, to address the watch committee and of this he availed
himself by his solicitor who appeared and was allowed to address the committee without restriction. I agree with Pearce LJ that
at that stage the watch committee could have re-opened the matter, and indeed three out of the twelve were in favour of so doing
(cf De Verteuil v Knaggs) but the position was then that the watch committee had given their decision that the appellant be
dismissed not only on the ground of unfitness but also on other grounds which included not only negligence in discharge of his
duty but also unspecified matters, which were said to be certain statements made today by members of the committee and the
town clerk. It was not until 5 April when the watch committee communicated with the Home Secretary prior to the appellants
appeal to the minister that it emerged that these statements had reference to allegations of perjury against the appellant. On 18
March Mr Bosley was given not only a full but a courteous hearing by the watch committee, but received no indication of the
nature of the charges which his client had to answer notwithstanding his repeated statements that he did not know what they
were. It is plain therefore that, if there were a failure on 7 March to give justice to the appellant, this was not cured on 18 March
when the watch committee confirmed their previous decision. At this hearing it was made plain by Mr Bosley that his client was
not seeking reinstatement but only his pension rights of which he had been deprived by his dismissal. This position is maintained
by the appellant through his counsel before your lordships.
I should not delay further before referring to the terms of the Municipal Corporations Act, 1882, itself, for it is on the
construction of that statute that the answer to the question posed before your lordships depends. It is quite true that on its terms
there is a power to dismiss any borough constable (and this applies to the appellant), whom the watch committee think negligent
in the discharge of his duty or otherwise unfit for the same. I entirely accept the reasoning underlying the judgments of the lords
justices that, if a statute gives an unfettered power to dismiss at pleasure without more, that is an end of the matter.
The topic is, however, not as simple as would seem. A large number of authorities were cited to your lordships beginning
with Baggs case and extending to the present day. I will not travel over the field of the authorities, which I am bound to say are
not easy to reconcile with one another, for if I did I should surely omit some which might be thought to be of equal or greater
importance than those I mentioned, but certain matters seem to me clearly to 112 emerge. One is that the absence of a lis or
dispute between opposing parties is not a decisive feature, although no doubt the presence of a lis would involve the necessity for
the applications of the principles of natural justice. Secondly, the answer in a given case is not provided by the statement that the
giver of the decision is acting in an executive or administrative capacity, as if that was the antithesis of a judicial capacity. The
cases seem to me to show that persons acting in a capacity which is not on the face of it judicial, but rather executive or
administrative, have been held by the courts to be subject to the principles of natural justice. Perhaps the most striking example
is to be found in the old case of Capel v Child, which is referred to at length by North J in Fisher v Jackson ([1891] 2 Ch 84 at p
95). The facts were these. By s 50 of the Act, 57 Geo 3 c 99, it was provided that:

Whenever it shall appear to the satisfaction of any bishop, either of his own knowledge, or upon proof by affidavit laid
before him, that by reason of the number of churches or chapels belonging to any benefice locally situate within his
diocese, or the distance of such churches or chapels from each other, and the distance of the residence of the spiritual
person serving the same from such churches or chapels, or any or either of them, or the negligence of the spiritual person
holding the same, that the ecclesiastical duties of such benefice are inadequately performed,

then, to put it shortly, the bishop may appoint a curate to perform or assist in performing the duties and may throw the burden
of the stipend of that curate on the person the insufficiency of whose performance of the duties has led to the necessity of the
appointment. The Bishop of London on the 18 January 1880, served a requisition on the plaintiff by virtue of the Act of
Parliament above mentioned reciting that of his own knowledge the ecclesiastical duties of the vicarage and parish church of
Watford were inadequately performed by reason of the plaintiffs negligence and requiring him to nominate a fit person with a
stipend to assist in performing those duties. The plaintiff did not appoint a curate and the bishop did so, assigning to him a
stipend. The stipend remained unpaid and the plaintiff was accordingly summoned before the bishop. The plaintiff did not attend
and the plaintiff was monished to pay the stipend. He then appeared for the first time and alleged that he had not had a proper
opportunity of being heard on the original application. Lord Lyndhurst CB used this language ((1832), 2 C & J at p 577):

Here is a new jurisdiction givena new authority given: a power is given to the bishop to pronounce a judgment;
and, according to every principle of law and equity, such judgment could not be pronounced, or, if pronounced, could not
for a moment be sustained, unless the party in the first instance had the opportunity of being heard in his defence, which in
this case he had not; and not only no charge is made against him which he had an opportunity of meeting, but he had not
been summoned that he might meet any charge.

Baron Bayley said ((1832), 2 C & J at pp 578, 579):

Upon the general principles of law, it would have been essential, if the bishop had proceeded by way of affidavit, to
have given the opposite party an opportunity of being heard. When the bishop proceeds on his own knowledge, I am of
opinion that it cannot possibly and within the meaning of this Act, appear to the satisfaction of the bishop, and of his own
knowledge, unless he gives the party an opportunity of being heard, in answer to that which the bishop states on his own
knowledge to be the foundation on which he proceeds It would be quite sufficient if the bishop were to call the party
before him, and to state to him the grounds on which he thought the duties were inadequately performed, by reason of his
113 negligence; and he should have asked whether he had or had not any grounds on which he could answer that charge.
But, is it not a common principle in every case which has in itself the character of a judicial proceeding, that the party
against whom the judgment is to operate should have an opportunity of being heard?

It is true that emphasis is laid on the judicial character of the proceedings in the view of both learned judges but it is not clear to
me that it could not be said in that case that the bishop was acting administratively. The situation under the Act, under which the
bishop was exercising his powers, was not unlike that of the watch committee here exercising powers under another Act, and it so
happens that the charge involved, that of negligence, was the same in each case. The matter which, to my mind, is relevant in this
case is that where the power to be exercised involves a charge made against the person who is dismissed, by that I mean a charge
of misconduct, the principles of natural justice have to be observed before the power is exercised.
One of the difficulties felt in applying principles of natural justice is that there is a certain vagueness in the term and as
Tucker LJ, said in Russell v Duke of Norfolk ([1949] 1 All ER 109 at p 118):

There are no words which are of universal application to every kind of inquiry and every kind of domestic
tribunal. The requirements of natural justice must depend on the circumstances of the case, the nature of the inquiry, the
rules under which the tribunal is acting, the subject-matter that is being dealt with, and so forth.

If it be said that this makes natural justice so vague as to be inapplicable, I would not agree. No one, I think, disputes that three
features of natural justice stand out(1) the right to be heard by an unbiassed tribunal, (2) the right to have notice of charges of
misconduct, (3) the right to be heard in answer to those charges. The first does not arise in the case before your lordships, but the
two last most certainly do and the proceedings before the watch committee therefore, in my opinion, cannot be allowed to stand.
I have reached this conclusion on the construction of the statute with some hesitation, not only because of the different view
taken by the Court of Appeal but also because of that taken by my noble and learned friend Lord Evershed, who also feels that
notwithstanding their findings of fault made against the appellant the watch committee had a perfect right to act as they did. In
one respect no doubt the watch committee were given an absolute discretion to act as they might think, that is to say, I agree that
their residual power to dismiss for unfitness may well be unfettered. I do not accept the contention of the appellant that unfitness
is to be construed ejusdem generis with negligence: indeed I think that it is the antithesis of negligence, and covers cases where
there is no fault in the accepted sense of the word in the officer dismissed. A man may be unfit because he is stupid, vacillating,
unable to meet a crisis or generally to command others, but I do not see this as the subject-matter of a charge. As I have
indicated, it is not clear to me that Donovan J necessarily had anything more in mind than absence of the qualities necessary for
leadership, when he made the observations that he did, but the watch committee went outside unfitness and made findings of
negligence and inferentially of perjury without giving the appellant any notice or opportunity of being heard. Even if the residual
power to dismiss for unfitness remains unimpaired, one could not conceive any watch committee exercising this power and at the
same time leaving the dismissed officer without a pension. This would only be expected where charges as here were made
against him. I cannot see that the general words of the statute are in the light of the authorities as I understand them wide enough
to cover a case of this character where allegations of misconduct are involved invoking the loss of an office and an element of
punishment for 114 offences committed. There is imposed a clog on the discretion in that it cannot be exercised arbitrarily
without regard to natural justice. I am aware that what I have said may not be thought to be in line with those cases where wide
words have been held sufficient to cover the exercise of an arbitrary power as in the matter of issue and withdrawal of licences
where no question of punishment arises, cf Nakkuda Ali v M F de S Jayaratne and R v Metropolitan Police Commissioner. It
may be that I must retreat to the last refuge of one confronted with as difficult a problem as this, namely, that each case depends
on its own facts and that here the deprivation of a pension without a hearing is on the face of it a denial of justice which cannot be
justified on the language of the subsection under consideration
I have little to add to what has already been said about the application of the Police Regulations, 1952. It was not contended
before your lordships that the Act of 1882 had been repealed by the Police Act, 1919, or any regulations made thereunder but it
was contended, in my opinion rightly, that, where a report or allegation against a police officer has been made, the regulations
apply and govern the form of the inquiry which must follow. Here there were no formulated charges, no tribunal appointed for
the purpose of hearing the charges and reporting to the police authority a statement of the charges found to be proved. The
learned judge at the trial and all the members of the Court of Appeal were of opinion that the regulations did not apply, because
no report or allegation was received from which it appeared that the appellant had committed an offence. My noble and learned
friend Lord Evershed, indeed, is of the same opinion. With all respect, I cannot agree. It is plain that the action taken by the
watch committee followed directly from the observations of Donovan J after the trial at the Old Bailey, which were intended for
the ears of the appropriate authority and did in fact reach the watch committee before it dismissed the appellant on 7 March. The
appellant had been acquitted of the offence with which he had been charged at the criminal trial but on a fair reading of those
observations which were severely critical of the appellant it cannot, I think, be said that it did not appear from them that the
appellant had committed an offence under the regulations. I need only read two of the offences named in the discipline code set
out in Sch 1 to the regulationsn:
________________________________________
n Viz, Sch 1 to the Police (Discipline) Regulations, 1952, SI 1952 No 1705

1. Discreditable conduct, that is to say, if a member of a police force acts in any manner prejudicial to discipline
or reasonably likely to bring discredit on the reputation of the force or of the police service.
4. Neglect of duty, that is to say, if a member of a police force(a) neglects or without good and sufficient cause
omits, promptly and diligently to carry out anything which is his duty as a constable.

It is difficult to see how any action would be taken in any event in respect of breaches of the police discipline code without a
report or allegation of some kind being made and I am quite unable to accept the submission that something different perhaps of a
formal nature or some complaint from an extra judicial source is necessary before there can be said to be a report or allegation.
Streatfeild J ([1961] 2 All ER at p 533), accepted the submission of counsel for the watch committee that their action arose, not as
a result of any report or allegation, but from the knowledge which was common to them and the country as a whole that the
appellant was unfit for office.
I am unable to accept that this was the position. The watch committee did not act solely on the ground that the appellant was
unfit for office irrespective of any offence that he might have committed, as their finding shows. They found him guilty of
offences which were founded on a report or allegation which they had received from the learned judge who had presided at the
trial and certain statements made by members of the committee and the town clerk. 115I have not taken into account any other
reports or allegations, for, whatever the watch committee may have known personally about the trial, they did not have a
transcript of the evidence, it now appears, until after they had given their decision on 7 March. There is, I should add, no
substance in the point taken that the reference to a copy of the report or allegation on which the charge is based contained in reg 4
of the Police (Discipline) (Deputy Chief Constables etc) Regulations, 1952 (SI 1952 No 1706), shows that there must be a written
report or allegation ab initio. No doubt an oral allegation will have to be reduced to writing, but it may well originate as an oral
allegation will have to be reduced to writing, but it may well originate as an oral statement, as it did in this case before the
transcript of the observations of Donovan J was sent to the watch committee.
Once the position is reached that the Police Regulations apply as, in my opinion, they did, it is clear that no attempt was
made by the watch committee to follow the regulations. These have been set out in detail by my noble and learned friend Lord
Morris Of Borth-Y-Gest whose judgment I have had the opportunity of reading and with which I respectfully agree. As he says,
and the Court of Appeal would have taken the same view if they had regarded the Police Regulations as applicable, the watch
committee disregarded the regulations and did not begin to comply with them.
On both grounds therefore, failure to comply with the requirements of natural justice and failure to comply with the Police
Regulations, I would hold that the decision of the watch committee to dismiss the appellant taken on 7 March 1958, was invalid.
This is not an end of the matter for the appellant did not let matters rest but appealed to the Home Secretary as he was
entitled to do under the Police (Appeals) Act, 1927, from the dismissal under s 191 of the Municipal Corporations Act, 1882.
Thus, it is said, since the decision of the Home Secretary by virtue of the Police (Appeals) Act, 1927, was final, the appellant had
waived his right to bring an action in the courts alleging that invalidity. I doubt whether any question of waiver arises, but I
appreciate the force of the opinion expressed by my noble and learned friend Lord Evershed that if Parliament has stated that the
appeal is final, that is an end of the matter and the appellant cannot, as it were, start again and by an action for a declaration seek
to undermine the decision from which he has unsuccessfully appealed. The answer to this point is, I think, and here again I find
myself in disagreement with the Court of Appeal, as well as with my noble and learned friend Lord Evershed, that the decision of
7 March 1958, taken by the watch committee was at all times a nullity, and nothing that was done thereafter by way of appeal
could give it validity.
In all the cases where the courts have held that the principles of natural justice have been flouted I can find none where the
language does not indicate the opinion held that the decision impugned was void. It is true that the distinction between void and
voidable is not drawn explicitly in the cases, but the language used shows that where there is a want of jurisdiction, as opposed to
a failure to follow a procedural requirement, the result is a nullity. This was indeed decided by the Court of Exchequer in Wood v
Woad, where as here there was a failure to give a hearing.
In Spackman v Plumstead Board of Works, referring to another statute Lord Selborne said ((1885), 10 App Cas at p 240):

There would be no decision within the meaning of the statute if there was anything of that sort done contrary to the
essence of justice.

I would apply this language, whether the Municipal Corporations Act, 1882, or the Police Regulations are to be considered. In
either case the watch 116 committee in failing to give a hearing to the appellant acted without jurisdiction.
I would allow the appeal accordingly.

LORD DEVLIN. My Lords, I am satisfied that s 191(4) of the Municipal Corporations Act, 1882, is wide enough to permit the
dismissal of a constable on the grounds of unfitness in the sense of inadequacy as well as on the grounds of negligence or
misconduct. The way in which this power may be used has, since 1919, been controlled as to conditions of service by
regulations made by the Home Secretary under s 4(1) of the Police Act, 1919, which requires that every police authority shall
comply with the regulations so made. The Police (Discipline) Regulations, 1952, o create a number of disciplinary offences
contained in a disciplinary code, and provide in detail for the way in which a charge of such an offence is to be investigated and
determined before a decision to dismiss is taken. I do not find it necessary to determine whether, before 1919, the power to
dismiss for neglect of duty could be exercised administratively and without any sort of judicial inquiry. Nor do I need to decide
whether or not the power to dismiss for inadequacy is purely administrative. I am satisfied that in all matters to which the
regulations apply the power to dismiss must be exercised in accordance with them.
________________________________________
o 183 Viz, SI 1952 No 1705, as amended by SI 1953 No 636, and cf, the Police (Discipline) (Deputy Chief Constables, etc) Regulations, 1952
(SI 1952 No 1706)

It is argued that the regulations do not apply in the present case for two reasons. It is said in the first place that the
disciplinary code is expressed in phraseology unsuited to the activities of chief constables, and in particular does not cover the
gravamen of the charge against the appellant as indicated by Donovan J which was that he was revealed by his conduct as a bad
example to and a bad influence on the Brighton constabulary. Undoubtedly the discipline code appears to be drafted with the
lower ranks in mind. But by SI 1952 No 1706 it is expressly made applicable to chief constables p and it must be construed
accordingly. It contains a number of specific offences which a chief constable could hardly commit but also a number which he
certainly could. There are specific matters put against the appellant in this case which I think certainly fall under the head of
discreditable conduct, if not also of neglect of duty. In my judgment the disciplinary code should be regarded as a
compendium covering all misconduct and neglect of duty in the case of all ranks from chief constable downwards. I find it
impossible to believe that there was intended to be a residue of neglect to be dealt with at large and in relation to which the
offender is deprived of the protection afforded by the regulations. If a case of inefficiency or inadequacy can be made without
proof of misconduct or neglect, the regulations do not apply; but if the case involves an allegation (and I use that word as will be
seen hereafter in its widest sense) of a disciplinary offence the procedure laid down by the regulations must be followed.
________________________________________
p Police (Discipline) (Deputy Chief Constables, etc) Regulations, 1952, reg 18

This in my opinion is the result of the impact of the Act of 1919 on the earlier one of 1882. This division of the power under
the Act of 1882 is in practice less inconvenient than it might sound. In and before 1919 there was a power, such as is now
contained in the Police Pensions Act, 1948, s 1(1) (c), to provide by regulation for cases in which policemen may be required to
retire otherwise than on the ground of misconduct. It is difficult to believe that the power of summary dismissal would now be
exercised in any case in which no fault is alleged, so that in practice the power under s 191(4) of the Act of 1882 has become a
controlled one. Legally, however, the power remains and can be used. It is unnecessary to consider whether or not it could have
been used in this case because one of the grounds given for the appellants dismissal was neglect of duty. The watch
committee ought not to have reached a decision on this ground without following the regulations, unless it can be said (and this is
the second 117 point to be considered) that the regulations are by their own terms inapplicable on the facts of this case. It is
argued that reg 2 requiresq that before the procedure laid down can be instituted a report or allegation must be made; and that
where as in the present case a matter comes to the knowledge of the watch committee as one of public notoriety, the regulations
do not apply. I need not elaborate on the extraordinary resultsmy noble and learned friends Lord Reid and Lord Morris of
Borth-y-Gest have mentioned themwhich as it seems to me would follow if the protection against dismissal depends on
whether or not the supposed misdemeanours of a police officer have been reported in the press. Such a construction ought not to
be put on reg 2 unless the language compels it, and in my opinion the language of the regulation does not. I think that the word
allegation is to be given a wide meaning. The main object of reg 2, as is shown by the introductory words italicised r in SI 1952
No 1706, is not to provide for some formal initiation of proceedings, equivalent to a writ of summons or an information, but to
ensure that an officer is told of any allegation made against him so that nothing is done behind his back. I do not see how the
watch committee can deal with any disciplinary matter unless an allegation of some sort is made even if it be only by one of their
own number; and I think it fair to assume that the word is chosen as the widest one that could be thought of to comprehend every
way in which such proceedings could be started.
________________________________________
q Police (Discipline) Regulations, 1952, reg 2; cf, reg 1 of SI 1952 No 1706
r Regulation 1 of SI 1952 No 1706 is printed beneath a heading, which is in italics and reads Right of accused to be informed of, and to
make a personal explanation concerning, allegations. Regulation 2(1) of SI 1952 No 1705 and reg 1 of SI 1952 No 1706 both begin
Where a report or allegation is received from which it appears

It is not disputed that if the regulations are applicable, as I think that they are, they were not complied with. On this basis
two further questions arise. The first is whether it is open to the House to question the decision of the watch committee on this
ground, and here I agree entirely with the conclusion reached by my noble and learned friend Lord Morris of Borth-y-Gest that it
is. The second is what is the effect of non-compliance on the decision. It is argued for the appellant that the effect is to avoid ab
initio the decision of the committee. That must mean that the committee had no statutory authority to make any decision at all. If
they had, then, although the decision they made might be a bad one and one that could be quashed by the court by virtue of its
supervisory jurisdiction over the proceedings of inferior tribunals, it would not be void ab initio, but would be good until
quashed. To make it void ab initio there must be some condition precedent to the conferment of authority on the committee
which has not been fulfilled. It is argued that compliance with the regulations is a condition precedent. It is not expressly made
so and I am not prepared to make the implication. I am very reluctant to imply such a condition where none is expressed, for the
utter avoidance of a decision of this sort is a very grave matter. All that has been done on the face of it falls to the ground. Even
if the appellant were satisfied with it, it could be impugned by any third party. The court would have no discretion to quash or
not to quash. It can only declare to be a nullity that which in law has never been done at all.
I see no reason therefore why I should do more than read the regulations into the Act of 1882, not as a condition precedent to
the power to dismiss, but simply as rules that the committee is required to observe. I do not hold that compliance with all the
rules is by implication a condition precedent to the power to dismiss under s 191(4). But if one of the regulations itself imposes
expressly a condition precedent, it is another matter. I am driven to the conclusion that reg 11(1) does s. Regulation 5 providest
that the case shall be heard by a tribunal appointed by the police authority and reg 11(1) provides that u 118the decision of the
police authority on receipt of the report of the tribunal shall be either to dismiss the case or to impose various punishments,
including dismissal. I cannot regard the power of dismissal under reg 11(1) as something distinct from the power of dismissal
under s 191(4), and I think that the effect of reg 11(1) is to make the power to dismiss conditional on the receipt of the report. I
do not say that any defect in a report would invalidate a dismissal. But where, as here, there has been no report at all and no
inquiry to substantiate one, I think that the statutory authority to dismiss never was created, so that the act of dismissal was a
nullity. If it was a nullity, it is not seriously argued that any subsequent proceedings before the Secretary of State could bring it to
life. The result in my opinion is that your lordships should allow the appeal and declare the dismissal to be void.
________________________________________
s Viz, reg 5 and reg 11(1) of the Police (Discipline) (Deputy Chief Constables, etc) Regulations, 1952 (SI 1952 No 1706)
t Viz, reg 5 and reg 11(1) of the Police (Discipline) (Deputy Chief Constables, etc) Regulations, 1952 (SI 1952 No 1706)
u Viz, reg 5 and reg 11(1) of the Police (Discipline) (Deputy Chief Constables, etc) Regulations, 1952 (SI 1952 No 1706)

My lords, I cannot say that I regard this result as altogether satisfactory. It is not that I regard the watch committees
decision as inevitably right or as one that can be faulted only on the ground that justice has not appeared to be done. The
appellant has not seriously complained about being put out of office; and since he has told your lordships that he will not seek to
be reinstated, it is permissible for me to say that I think the decision on that point to have been inevitable. But he could, instead
of being dismissed, have been compelled to retire, and thus he would have been saved some or all of his pension rights. That is
an issue of substance deserving of careful consideration. What is unfortunate about the result is that it means that during the
whole time taken up in the elucidation of this difficult point of law, the appellant has legally been in office and entitled to the
appropriate emoluments. That would be so, I suppose, even if he had been in profitable employment elsewhere, for his claim
would be for salary and not for damages for wrongful dismissal. Whatever course is now taken, the appellant is likely to reap a
substantial benefit from the fact that the committee fell into the pardonable error (pardonable if only because their view of the
law was the same as that taken by all the lords justices in the Court of Appeal) that they were entitled to deal with this matter
administratively and in their unfettered discretion.
It can be said with much force that all this is the result of ousting the ordinary jurisdiction of the courts. If the statute was
drafted so as to make a dismissal, as the common law does in contracts of service, effective whether rightful or wrongful and to
give compensation for wrongful dismissal, the issue would have been tried by an ordinary court of law and the appellant would
have got no more and no less than his deserts. But the statute gives the judicial power to a committee or tribunal. If the object of
that were to make one side a judge in its own cause, I should not be sad to see it miscarry. But the object here is the creation of a
special code, stricter in some respects at least than the ordinary obligations of a contract of service, and of an independent tribunal
to aid in its administration. Such tribunals must always be subject to the supervisory jurisdiction of the High Court. But it does
not by any means follow that a defect of natural justice sufficiently grave to be a ground for quashing the resulting decision,
inevitably leads as in the present case to a declaration that the decision is void ab initio. It is necessary always to bear in mind the
distinction so clearly drawn by Lord Sumner in R v Nat Bell Liquors Ltd ([1922] All ER Rep 335 at p 348, letter h; [1922] 2 AC
128 at p 151) between a wrong exercise of a jurisdiction which a judge has and a usurpation of a jurisdiction which he has not. If
there is no jurisdiction, the decision is a nullity whether the court quashes or not. If there is jurisdiction but there has been a
miscarriage of natural justice, the decision stands good until quashed. The occurrence of a miscarriage does not require the court
to quash if it is satisfied that justice can be done in some other way. The court in a case like the present, for example, if the
decision had been voidable and not void, might have left the appellant to his remedy in damages, if any. Your lordships heard
some argument about 119 whether the court could, if it exercised its discretion to quash, do so on terms which would have put the
parties back into the position in which they would have been if the proper procedure had been followed from the outset. I need
not say more than that I should be prepared to listen to such a contention in an appropriate case and I should certainly be glad if
the court had the power to do justice in that sort of way when reviewing the decisions of inferior tribunals.
In the view that I take of this case there is not much that I can usefully say about the principles of natural justice and their
application to the procedure under s 191(4) of the Act of 1882. Whether or not they are to be applied to any statutory procedure
depends on an implication to be drawn from the statute itself; and the question whether such an implication should be drawn in
this case cannot be answered without a consideration of the Police Act, 1919, and the regulations made thereunder from which s
191(4) cannot be divorced. Since the regulations themselves prescribe the rules of justice that are to be followed, it seems to me
that there is nothing to be gained by seeking to ascertain what the position would be if the discipline code did not apply.
There are three points, however, on which I desire to comment. First, I express no dissent from the view that if s 191(4)
stood alone the decision to be made under it is not purely administrative. Secondly, I do most emphatically dissent from the view
that natural justice did not require the watch committee to hear the appellant because, as it was said, he had had a full opportunity
of putting his case before the trial judge. The appellant was not and could not have been compelled to put any case at all before
the trial judge; he was there to answer an indictment on trial by jury. It would be quite wrong if an accused was to be
embarrassed in the conduct on a criminal charge by the reflection that if he did not also satisfy the trial judge about the propriety
of his actions in other respects, it might thereafter be the worse for him. Thirdly, if there was apart from the regulations a
miscarriage of justice in this case (and I think on the whole that there was), I agree with the opinion of my noble and learned
friend Lord Evershed for the reasons which he has given that the miscarriage rendered the watch committees decision voidable
and not null and void ab initio.
I agree with the order proposed by my noble and learned friend on the woolsack.

Appeal allowed.

Solicitors: Haslewoods agents for Bosley & Co, Brighton (for the appellant); Sharpe, Pritchard & Co agents for Town clerk,
Brighton (for the defendants).

C G Leonard Esq Barrister.


120
[1963] 2 All ER 121

Gordon v Cradock
CIVIL PROCEDURE

COURT OF APPEAL
WILLMER AND UPJOHN LJJ
5 MARCH 1963

Court of Appeal Interlocutory appeal Leave to appeal Conditional leave to defend Whether leave to appeal required
Supreme Court of Judicature (Consolidation) Act, 1925 (15 & 16 Geo 5 c 49), s 31(1) (i), (2).

Practice Summary judgment Leave to defend Conditional leave to defend Appeal Grounds on which Court of Appeal
may interfere with conditions imposed by judge in exercise of his discretion under RSC, Ord 14, r 6.

No leave is required by either a plaintiff or a defendant to appeal against an interlocutory order made by a judge granting a
defendant leave to defend on conditions, since such an order is an order refusing unconditional leave to defend an action,
within those words in s 31(2) of the Supreme Court of Judicature (Consolidation) Act, 1925, and is deemed, by s 31(2), not to be
an interlocutory order, with the consequence that s 31(1)(i) (which imposes a general prohibition on appeal without leave from an
interlocutory order) does not apply (see p 122, letters b, e and h, post).
Where a judge has exercised his discretion under RSC, Ord 14, r 6, and imposed conditions as a term of giving a defendant
leave to defend, the Court of Appeal will not interfere with the exercise of his discretion unless there has been some error of
principle or misapprehension of fact on his part, or unless he has given undue weight to a particular aspect of the facts (see p 123,
letters b and d, post).

Notes
As to appeals to the Court of Appeal against interlocutory orders, see 30 Halsburys Laws (3rd Edn) 452, 459, 460, paras 855 and
868; and for cases on the subject, see 16 Digest 178, 835, and 3rd Digest Supp.
For the Supreme Court of Judicature (Consolidation) Act, 1925, s 31(1)(i), (2), see 5 Halsburys Statutes (2nd Edn) 360, 361.

Interlocutory appeal
The plaintiff appealed against an interlocutory order of Megaw J made on 7 February 1963, varying the conditions on which
leave to defend an action was given by Master Diamond from payment into court of 20,000, the whole sum claimed by the
plaintiff in the action, to payment into court of 5,000 having regard to certain special features of the case sworn by the plaintiff.
At the hearing of the appeal the plaintiff contended that leave to appeal was not required, and he gave notice in his notice of
appeal that, if leave to appeal were required, he would apply to the court for leave. He sought to have the judges order reversed
and the order of the master restored on the ground that the judge was wrong in law or in principle in varying the masters order
merely because the plaintiffs case contained sufficient peculiar features, so as not to warrant that the whole 20,000 be paid
into court; and on the ground that there was no evidence that the plaintiffs case had such peculiar features or alternatively, that
such features (if any) were not a sufficient reason for varying the masters order. Counsel for the defendant drew the attention of
the Court of Appeal to various matters which might be said to be peculiar features, eg that sums so large as 10,000 were
handed over in cash in a hotel, that the money was advanced in furtherance of a real estate scheme of which no particulars were
given, that there had been very considerable delay in instituting proceedings, and that it seemed doubtful whether the alleged
transaction could have been carried out without contravening exchange control legislation.

S Chapman QC and C W G Ross-Munro for the plaintiff.


M Littman QC and R Gavin Freeman for the defendant.
121

5 March 1963. The following judgments were delivered.

WILLMER LJ. A preliminary question has been raised on this appeal whether leave is required. Leave was sought from the
judge, but, as I understand it, was refused. There is no doubt that a plaintiff has no right of appeal against an order granting to a
defendant unconditional leave to defend; that is provided by s 31(1)(c) of the Supreme Court of Judicature (Consolidation) Act,
1925. There is equally no doubt that a defendant who is refused unconditional leave to defend has an absolute right of appeal
without the necessity of asking leave; that, I think, must be the effect of s 31(2) of the Act of 1925. Here the somewhat unusual
position arises that leave to defend on conditions has been granted, and the appeal is an appeal by the plaintiff. We have not
heard the appeal yet, but I understand that the plaintiff intends to complain of the conditions that have been imposed. In those
circumstances it is said that s 31(2) does not apply; that this is an ordinary interlocutory appeal, and that consequently it requires
the leave of the judge, or (on the judges refusal) the leave of this court in pursuance of s 31(1)(i). Curiously enough, we are told,
there is no direct authority on the point. In those circumstances it falls to us to make a decision one way or the other.
So far as I am concerned, I feel no doubt what the decision must be, viz, that this is a case in which no leave to appeal is
required. In my judgment the words of s 31(2) are much too plain to admit of any other construction. Section 31(2) is not
dealing in terms with the right of appeal; the object of the subsection is to define whether a particular order is or is not an
interlocutory one. The wording is:
An order refusing unconditional leave to defend an action shall not be deemed to be an interlocutory order within the
meaning of this section.

If one construes those words according to their ordinary meaning, it must follow that no leave is required when an appeal is put
forward by either party against an order such as that in the present case, ie, an order refusing unconditional leave.
I would only add this. It appears to me, with all respect to the argument to the contrary, that any other view would lead to
the strangest possible results. To take this case, for instance: supposing the defendant, as well as the plaintiff, had wished to
appeal against the order made by the judge (the order being one refusing him unconditional leave to defend) he would admittedly
have had an absolute right of appeal. On the defendants argument, the plaintiff would have had no right to cross-appeal unless
leave were obtained. That would seem to me to be a very strange result; and for my part I do not think that it is a result which
follows from the words of the statute if they are construed in accordance with their ordinary sense. In my judgment, therefore,
there is here an absolute right of appeal without leave, and the plaintiffs counsel may therefore proceed with the presentation of
the appeal without asking us for leave.

UPJOHN LJ. I agree entirely with the judgment which has just been delivered. The point is of the shortest. My Lord has
expressed exactly, and with his usual cogency, the reasons which impel me to the same conclusion; and I should only be repeating
in less felicitous language what he has already said if I added further words of my own. I therefore agree that no leave is required
for the plaintiff to prosecute this appeal.
[Their Lordships then heard the plaintiffs appeal on the merits.]

WILLMER LJ. This is an appeal from an order made by Megaw J, in chambers on 7 February of this year, whereby he varied a
previous order made by Master Diamond in proceedings under RSC, Ord 14. The effect of the order both of the master and of the
judge was that the defendant should have leave to 122 defend, but on conditions. It was in relation to the conditions that the
judge differed from the master. We are now invited on behalf of the plaintiff to restore the conditions imposed by the master, and
to that extent (and to that extent only) to allow the appeal from the judges order. [His Lordship referred to the facts of the case
and continued:] I find it impossible to say that the judge was wrong to take these matters into consideration. It appears to me that
the question what terms ought to be imposed on a defendant as a condition of giving him leave to defend is very much a matter of
discretion for the judge. That being so, this court is naturally reluctant to interfere with the exercise of discretion by the judge
below, particularly so experienced a judge, unless it can be shown that there has been some error of principle on his part, or that
he has in some way misapprehended the facts, or has given undue weight to this or that aspect of the facts. I do not say that if I
had been occupying his seat and the matter had come before me, I would necessarily have arrived at the same conclusion as he
did. It was clear at that stage that some condition ought to be imposed on the defendant before giving him leave to defend; and it
may very well be that, had I been sitting at first instance, I might have imposed some different condition. But, giving the matter
the best consideration that I can, I find it impossible to say that the judges exercise of his discretion was wrong, so as to justify
this court in interfering. On the whole, therefore, I come to the conclusion that this appeal should be dismissed.

UPJOHN LJ. I agree with the judgment which has just been delivered. Like my Lord, it might be, had I been sitting at first
instance, I would not have made the same order as the judge. That is not the matter that we have to consider. The judge, who has
a very wide discretion under RSC, Ord 14, r 6, decided on what conditions he would give leave to defend. That was the only
question that he had to consider; and the only matter that we have to consider is whether he has wrongly exercised his discretion.
For the reasons given fully by my Lord, I agree that we cannot disturb that conclusion, and I agree that the appeal must be
dismissed.

No leave to appeal to the Court of Appeal required. Appeal dismissed.

Solicitors: Cooper, Bake, Fettes, Roche & Wade (for the plaintiff); Basil Greenby & Co (for the defendant).

F A Amies Esq Barrister.


123
[1963] 2 All ER 124

St Pancras Borough Council v Frey


LANDLORD AND TENANT; Tenancies

QUEENS BENCH DIVISION


LORD PARKER CJ, ASHWORTH AND WINN JJ
8 MARCH 1963

Landlord and Tenant Small tenement Recovery of possession Housing purposes Evidence Whether necessary to prove
that possession was required for the purpose of exercising housing functions Small Tenements Recovery Act, 1838 (1 & 2 Vict c
74), s 1 Housing Act, 1957 (5 & 6 Eliz 2 c 56), s 158(2).

A tenancy of premises let at a rent exceeding 20 a year by the appellant local authority having been duly determined by notice to
quit, the local authority served noticea on the tenant of their intention to recover possession under the Small Tenements Recovery
Act, 1838. At the hearing of the complaint the local authority proved the tenancy, its determination and the service of the
statutory notice, but tendered no other evidence, in particular no evidence of the purpose for which possession was required.
Section 158(2)b of the Housing Act, 1957, empowered a local authority to obtain possession under the Act of 1838 of any
building, whatever its rent, where the authority required possession of it for the purpose of exercising their powers under any
enactment relating to housing. The justices dismissed the complaint.
________________________________________
a This notice is required by the Small Tenements Recovery Act, 1838, s 2; 13 Halsburys Statutes (2nd Edn) 858
b Section 158(2) is as follows: Where a local authority for the purpose of exercising their powers under any enactment relating to housing,
require possession of any building or any part of a building of which they are the owners, then, whatever may be the value or rent of the
building or part of a building, they may obtain possession thereof under the Small Tenements Recovery Act, 1838, as in the cases therein
provided for, at any time after the tenancy of the occupier has expired, or has been determined.

Held The complaint was rightly dismissed, because, in order to be entitled to take advantage of the rights conferred by s 158(2)
of the Act of 1957, a local authority must tender some evidence to show that the condition precedent to jurisdiction, viz, that
possession was required for the purpose of exercising the authoritys housing powers, was satisfied.
Shelley v London County Council ([1948] 2 All ER 898) distinguished.
Dictum of Cassels J, in R v Snell, Ex p Marylebone Borough Council ([1942] 1 All ER at pp 614, 615) and Jenkins v
Paddington Borough Council (12 May 1954, unreported) considered.
Appeal dismissed.

Notes
As to the recovery of possession of small tenements, see 23 Halsburys Laws (3rd Edn) 710, para 1454; and for cases on the
subject, see 31 Digest (Repl) 615618, 73057332.
For the Small Tenements Recovery Act, 1838, see 13 Halsburys Statutes (2nd Edn) 855; and for the Housing Act, 1957, s
158, see 37 Halsburys Statutes (2nd Edn) 441.

Cases referred to in judgments


Jenkins v Paddington Borough Council (12 May 1954), unreported.
R v Snell, Ex p Marylebone Borough Council [1942] 1 All ER 612, [1942] 2 KB 137, 111 LJKB 530, 167 LT 13, 106 JP 160, 31
Digest (Repl) 616, 7318.
Shelley v London County Council, Harcourt v London County Council [1948] 2 All ER 898, [1949] AC 56, [1949] LJR 57, 113
JP 1, 31 Digest (Repl) 616, 7317.

Case Stated
This was a Case Stated by justices for the county of London acting in and for the petty sessional division of New River, in respect
of their adjudication as a magistrates court sitting at St Pancras Town Hall, in the county of London.
124
On 8 August 1962, a complaint was preferred by the appellants, the Mayor, Aldermen and Councillors of the Metropolitan
Borough of St Pancras, against the respondent, William Edward Frey, alleging that the appellants let to the respondent a tenement
consisting of 191, Caversham Road, NW5, that the tenancy had been determined by notice to quit on 7 May 1962, and that on 1
August 1962, the appellants had served on the respondent a notice in writing of their intention to apply to recover possession of
the tenement by giving the notice, and reading over and explaining the purport and intent of it, to the respondent c. The appellants
further alleged that notwithstanding the notice, the respondent neglected to deliver up possession of the tenement and still
detained it. The complaint was preferred under the Small Tenements Recovery Act, 1838, as extended by the Housing Act, 1957,
s 158(2). The following facts were found. A verbal weekly tenancy existed between the appellants and the respondent and the
tenancy was determined by a notice to quit on 7 May 1962. The appellants served on the respondent on 1 August 1962, a notice
in writing in accordance with the requirements of the Small Tenements Recovery Act, 1838, but the respondent did not deliver up
possession of the premises.
________________________________________
c This is the procedure laid down by s 2 of the Small Tenements Recovery Act, 1838; 13 Halsburys Statutes (2nd Edn) 858

It was contended by the appellants that on the facts the justices must issue a warrant of ejectment pursuant to the Small
Tenements Recovery Act, 1838, as extended by s 158 of the Housing Act, 1957, and that the appellants need not give any reason
for their application. It was contended by the respondent that, in order to succeed on their application, the appellants must prove
that they required possession for the purpose of exercising their powers under an enactment relating to housing within the
meaning of s 158(2) of the Housing Act, 1957, and that they had failed to do this.
The justices were of the opinion that the contention of the respondent was correct and that the appellants had not sufficiently
satisfied them that they required possession for the purpose of exercising their powers under an enactment relating to housing.
Accordingly, they refused the application. The appellants now appealed.

H B Grant for the appellants.


R G Rougier for the respondent.

8 March 1963. The following judgments were delivered.

ASHWORTH J delivered the first judgment at the request of Lord Parker CJ. The respondent was a weekly tenant of the
appellants of a tenement dwelling at a rent exceeding 20 per annum and the tenancy was determined by notice to quit on 7 May
1962. On 1 August 1962, notice in writing in accordance with the Small Tenements Recovery Act, 1838, was duly served on the
respondent, who did not give up possession of the premises. The complaint was heard on 16 August when evidence was adduced
on behalf of the appellants to prove the tenancy and its determination, and also the statutory notice required under the Act of
1838. No other evidence was tendered on their behalf: in particular there was no evidence to the effect that the respondent was
an unsatisfactory tenant or had committed a breach of the tenancy agreement, nor was there any specific evidence that the
appellants required possession for the purpose of exercising their powers under any enactment relating to housing. The justices
upheld a submission put forward on behalf of the respondent that in the circumstances the appellants claim had not been made
out, and accordingly they dismissed the complaint.
The statutory basis for the appellants claim before the justices is to be found in s 158 of the Housing Act, 1957, which re-
enacted (subject to an alteration not material to the present case) the provisions of s 156 of the Housing Act, 1936. Speaking
quite generally, the effect of s 158 is to remove from a tenant in the position of the present respondent the protection of the Rent
Restriction Acts and also to widen the jurisdiction under the Small Tenements Recovery Act, 1251838, by removing the
limitationd that the rent must not exceed 20 per annum. The advantages conferred on local authorities by s 158 are however
expressly confined to cases in which possession of premises is required for the purpose of enabling a local authority to exercise
their powers under any enactment relating to housing. The issue in this appeal is whether it is incumbent on a local authority,
seeking to recover possession of premises in pursuance of the provisions of s 158, to adduce evidence that possession is required
for the purpose mentioned in the section. Reference was naturally made to the decision of the House of Lords in Shelley v
London County Council in which the corresponding provisions of the Housing Act, 1936, were under consideration. The main
point of that decision relates, in my view, to the scope of a local authoritys powers of management regulation and control of
houses provided by them under the Act, and the problem raised in the present appeal was not considered. Lord Porter said
([1948] 2 All ER at p 900; [1949] AC at p 66):
________________________________________
d See s 1 of the Act of 1838; 13 Halsburys Statutes (2nd Edn) 855

It is, to my mind, one of the important duties of management that the local body should be able to pick and choose
their tenants at their will.

In that case it was accepted that the local authority on recovering possession intended to re-let the premises to another tenant, and
the possibility that possession was or might be required for some purpose not connected with housing did not arise. For my part I
do not consider that the decision in Shelley v London County Council is conclusive of the present appeal. It is certainly no
authority for the proposition advanced by counsel on behalf of the appellants that the mere service of a notice to quit is evidence
that such notice was given for the purpose of exercising their powers relating to housing.
Although the decision in R v Snell, Ex p Marylebone Borough Council turned on a different point, there is a passage in the
judgment of Cassels J, which foreshadows the issue raised in this appeal ([1942] 1 All ER at pp 614, 615; [1942] 2 KB at pp 144,
145):

If it could have been contended in this case that this local authority did not require possession of these premises for the
purpose of letting to another member of the working classes, but for the purpose of erecting a school, or widening a road or
for some other purpose which was not included in the Act, the tenant would have been entitled to succeed, and, indeed, the
magistrate would have been entitled to say that the court of summary jurisdiction had no jurisdiction to hear that
application.

I respectfully agree with that comment but it does not, and did not purport to, decide whether a local authority is bound to tender
evidence that their purpose in seeking possession is the statutory purpose mentioned in s 158 of the Housing Act, 1957.
Apart from these two reported cases reference was also made to an unreported decision of this court: Jenkins v Paddington
Borough Councile. In that case the local authority had passed a resolution that the tenancy should be terminated as it was not in
all respects satisfactory and this resolution was in evidence before the magistrate. The local authority did not however tender
evidence of an intention to re-let. The tenants appeal to this court was dismissed on the ground that the removal of an
unsatisfactory tenant was an exercise of the statutory powers of management and control of houses vested in the local authority.
For my part I do not read the judgments in that case as indicating that no evidence need be tendered by a local authority showing
that possession is required for the purpose of exercising their powers under an enactment relating to housing. The decision was
that evidence actually tendered was enough.
126
________________________________________
e (12 May 1954), unreported; see The Times 13 May 1954

In my judgment the effect of s 158 of the Housing Act, 1957, is to confer rights on a local authority in regard to recovering
possession of premises if (but only if) possession is required for the purpose mentioned in the section. It seems to me that in
order to take advantage of those rights a local authority must tender some evidence to show that their claim falls within the
section by establishing that the condition precedent to jurisdiction is satisfied. In most cases it is probable that there will be no
dispute and it is to be emphasised that, as Shelley v London County Council shows, the local authority can pick and choose its
tenants. Its motive for getting rid of a particular tenant or putting in another tenant is quite irrelevant in proceedings such as the
present proceedings. I would dismiss the appeal.

WINN J. I agree.

LORD PARKER CJ. I also agree.

Appeal dismissed.

Solicitors: Town clerk, St Pancras (for the appellants); M S Marks & Co (for the respondent).

W Beckett Terrell Barrister.


[1963] 2 All ER 127

Harvey v Smith-Wood
CIVIL PROCEDURE

QUEENS BENCH DIVISION


LAWTON J
7 MARCH 1963

Document Admissibility in evidence Previous written statement of witness at variance with his evidence Statement adduced
in examination-in-chief as evidence of the facts Evidence Act, 1938 (1 & 2 Geo 6 c 28), s 1(1).

A witness on behalf of the plaintiff gave evidence which was apparently in conflict with the plaintiffs own evidence. Counsel
for the plaintiff then sought to put in evidence a statement in writing made by the witness, who was an elderly man, more than six
years before the hearing.

Held The written statement was admissible under the Evidence Act, 1938, s 1(1) a, although this use of the provision should be
adopted only in very special circumstances.
________________________________________
a Section 1(1) so far as relevant, provides: In any civil proceedings where direct oral evidence of a fact would be admissible, any statement
made by a person in a document and tending to establish that fact shall, on production of the original document, be admissible as evidence
of that fact if the following conditions are satisfied, that is to say(i) if the maker of the statement either(a) had personal knowledge of
the matters dealt with by the statement; and (ii) if the maker of the statement is called as a witness in the proceedings

Cartwright v W Richardson & Co Ltd ([1955] 1 All ER 742), and Bearmans, Ltd v Metropolitan Police District Receiver
([1961] 1 All ER 384) considered.
Notes
As to the admissibility of written statements made by witnesses, see 15 Halsburys Laws (3rd Edn) 315, para 573.
For the Evidence Act, 1938, s 1(1), see 9 Halsburys Statutes (2nd Edn) 626.

Cases referred to in judgment


Bearmans Ltd v Metropolitan Police District Receiver [1961] 1 All ER 384, [1961] 1 WLR 634, 125 JP 268, 3rd Digest Supp.
Cartwright v W Richardson & Co Ltd [1955] 1 All ER 742, [1955] 1 WLR 340, 3rd Digest Supp.

Action
In this action the plaintiff claimed damages for negligence and the defendant counter-claimed damages for defamation. The case
is reported only with regard 127 to the submission by counsel for the plaintiff that a written statement made more than six years
before the hearing by one of the plaintiffs witnesses was admissible in evidence. The facts and the argument are stated in the
judges ruling.

D J C Ackner QC and S Terrell for the plaintiff.


W L Mars-Jones QC and D C W Hirst for the defendant.

7 March 1963. The following judgment was delivered.

LAWTON J. Counsel for the plaintiff has asked me to admit in evidence under the Evidence Act, 1938, a statement made by a
witness, Mervyn Drummond, who is at the moment giving evidence before me. The circumstances in which counsel comes to
make his application are as follows. It has become evident in this case that an important question of fact, with which I shall have
to deal when I come to deliver judgment, is what happened at the plaintiffs house, Orchard Lea, on or about 8 January 1951,
when the defendant, as he admits, called at that house and took possession of two deed boxes. The plaintiff in her evidence,
which has already been given, asserted that she did not see the defendant on that occasion; and she asked me to infer that the
defendant had not looked into the deed boxes whilst he was at her house. It is manifest that the object of calling Mr Mervyn
Drummond was to support the plaintiffs evidence in that respect. But when Mr Drummond went into the witnessbox he seemed
to be giving evidence which was in conflict with that which the plaintiff had already given. Counsel for the plaintiff, quite
properly, made one or two attempts to call Mr Drummonds attention to the point of the questions which he was putting, because
counsel seemed to be under the impression that Mr Drummond had not got his mind on the questions. Mr Drummond, however,
stuck to the evidence which he had already given: namely, that the boxes had been opened in his presence, at least, by the
defendant, and possibly in the presence of the plaintiff, though that is not altogether clear from what Mr Drummond in fact said.
Thereupon counsel for the plaintiff asked me to look at a statement in writing which had been made by Mr Drummond in 1956.
Counsel for the plaintiff has submitted to me that that statement is admissible under s 1(1) of the Evidence Act, 1938 b.
Counsel for the defendant has resisted that application strongly, and he has called my attention to the judgment of Barry J, in
Cartwright v W Richardson & Co Ltd. An analogous situation arose in that case, but it is not on all fours with the present
situation, because in the case dealt with by Barry J counsel for the defendants sought in re-examination to get admitted into
evidence a document containing something which the witness had said in writing on an earlier occasion. It is clear, however, that
when Barry J was delivering his judgment in that case he dealt with it, not on a technical point of procedure, but on general lines
as to the policy of the Act, saying in effect that the Evidence Act, 1938, had not changed the law relating to the rules on
previously inconsistent statements.
________________________________________
b The relevant terms of s 1(1) are set out in the footnote at p 127, ante

Counsel for the plaintiff has replied to that by calling my attention to the decision of the Court of Appeal in Bearmans Ltd v
Metropolitan Police District Receiver. That case, although it was decided on its own facts, as Sellers LJ, was at pains to stress
([1961] 1 All ER at p 388), did, however, call the attention of judges to the general purposes for which the Evidence Act, 1938,
was passed and the policy of that Act; and, as Devlin LJ pointed out ([1961] 1 All ER at p 392), the Act draws a distinction
between the question of admissibility and the question of the weight which is to be attached to a document when it has been
admitted.
Counsel for the plaintiff also called my attention to a passage in the text-book 128 written by Dr Rupert Cross, who is a
Fellow of Magdalen College, Oxford. It is Cross on Evidence (1958 reprint), and the passage in question comes at p 214. Dr
Cross puts the problem which I am called on to consider in the following words:

If A. calls W. to testify on a number of points, and W.s testimony concerning one of them is less favourable to A. than
a statement contained in W.s proof, that document would appear to fall precisely within the terms of s. 1(1) of the Act of
1938, but, in Cartwright v. W. Richardson & Co., Ltd. BARRY, J., held that this was not so. He reached this conclusion
because the Act has not changed the ordinary rules of procedure applicable in the trial of civil actions, and, if such evidence
were admissible, it might have the result of shaking the faith of the court in the evidence given by the witness in the box.
With respect, the rule that the consistency of witnesses may not be proved by their previous statements and the rule that
prior contradictory statements of witnesses under cross-examination are not evidence of the facts stated related to the
procedure at trials of civil actions, but we have seen that it is difficult to escape the conclusion that they have been reversed
by the Act of 1938. Accordingly, it is submitted that this part of the decision in Cartwright v. W. Richardson & Co. Ltd.
may have to be reconsidered on some future occasion. The truth is that the full effect of the Evidence Act on the
admissibility and utilisation of the previous statements of witnesses has not yet fully been worked out in the reported
cases.

I have come to the conclusion, with some regret, that on the proper construction of s 1(1) of the Act of 1938, the document which
counsel for the plaintiff seeks to have admitted in evidence can be so admitted. I say that I have come to that conclusion with
some regret because it seems to me that it is an unfortunate situation if counsel can call a witness and, when that witness does not
come up to proof, counsel should be allowed to produce some earlier document which shows that on some other occasion the
witness made a different statement. I appreciateand I have borne it in mind in coming to my decision as to the proper
construction of the Actthat the object of the Act is to see that the relevant evidence is before the court; and it may well be in the
sort of situation which has arisen in this case, it is right, in case one side or the other seeks to rely on evidence given by a
particular witness, that the court should have full information as to what the witness has said on some previous occasion.
Nevertheless, it seems to me that the use of this provision should be one which counsel should hesitate to adopt except in very
special circumstances. Having myself observed Mr Mervyn Drummond in the witness-box, I will content myself by saying at
this stage that I can well understand counsel thinking that this is such a special case. Mr Mervyn Drummond is an elderly man,
and it may well be that he is not as fit now as he was at the time when he made the statement which is sought to be admitted in
evidence.

Ruling accordingly.
Solicitors: Victor Mishcon & Co (for the plaintiff); Gascoin & Co (for the defendant).

Mary Colton Barrister.


129
[1963] 2 All ER 130

Inland Revenue Commissioners v Olive Mill Ltd (in liquidation)


Inland Revenue Commissioners v Olive Mill Spinners Ltd (in liquidation)
TAXATION; Other Taxation

CHANCERY DIVISION
BUCKLEY J
4, 5 MARCH 1963

Profits Tax Chargeable accounting period Last period of company Investment companys voluntary winding-up No longer
investment company after winding-up resolution Finance Act, 1937 (1 Edw 8 & 1 Geo 6 c 54), s 19(4).

Profits Tax Subsidiary company Principal companys notice that subsidiarys profits be treated as principals Determination
by winding-up of principal company Finance Act, 1937 (1 Edw 8 & 1 Geo 6 c 54), s 22(1).

In 1956 an investment company which held all the shares in a cotton spinning company gave a grouping notice pursuant to s 22
of the Finance Act, 1937, as a result of which the profits of the spinning company fell to be treated as the profits of the
investment company for profits tax purposes. On 12 December 1957, both companies passed resolutions for voluntary winding-
up. The investment company was assessed to the profits tax in the sum of 2,142 18s and to a distribution charge of 30,644 for
the period 27 January 1957, to 12 December 1957, which period the Commissioners of Inland Inland Revenue determined to be
an accounting period in accordance with s 20(2) of the Act of 1937. The spinning company was assessed to the profits tax in the
sum of 193 16s and a distribution charge of 71 4s for the chargeable accounting period 13 December 1957, to 25 January 1958,
on the basis that the grouping notice ceased to be effective on the commencement of the winding-up of the companies on 12
December 1957. The Special Commissioners of Income Tax found that the functions of the investment company, viz, the holding
of investments, continued after 12 December 1957, to be what they had been before that date and that the period then ending was
therefore not the companys last chargeable accounting period, and they discharged the assessments.

Held (i) The assessment on the investment company would be upheld because
(a) the commissioners finding that the functions of the investment company continued to be the same before and after the
resolution for voluntary winding-up was one of mixed fact and law, for its propriety depended on the proper interpretation of s
19(4) of the Finance Act, 1937; and therefore the finding was open to review (see p 137, letter i, post); and
(b) on the commencement of the winding-up of the investment company, its functions, which meant its activities appropriate
to its business, viz, its objective, ceased to be the holding of investments for the purpose of obtaining a benefit by doing so and
became the realisation of its assets and distribution of the proceeds among creditors and members (see p 138, letters c and e,
post).
Dictum of Atkinson J, in Inland Revenue Comrs v Buxton Palace Hotel, Ltd ((1948), 29 Tax Cas at p 344), and dictum of
Lord Reid in Henry Briggs, Son & Co Ltd (in liquidation) v Inland Revenue Comrs ([1961] 1 All ER 223) applied.
(ii) the assessment on the spinning company would be upheld because on 12 December 1957 (the commencement of the
winding-up of the investment company) the grouping notice under s 22 of the Finance Act, 1937, ceased to be operative with the
consequence that the spinning company became assessable to the profits tax in respect of any subsequent chargeable accounting
period, for the following two reasons
(a) the spinning company ceased to be a subsidiary of the investment 130 company within the definition of subsidiary in
s 42(1) of the Finance Act, 1938, because on the commencement of the winding-up of the investment company the capital of the
spinning company then held by the investment company ceased to belong beneficially to the investment company and became
trust property to be applied in discharge of its liabilities and in due course of administration (see p 139, letters c and f, post).
Dicta of James and Mellish LJJ, in Re Oriental Inland Steam Co ((1874), 9 Ch App at pp 559, 561) applied.
(b) the last accounting period of the investment company ended on 12 December 1957, and the investment company
thereafter was not a company to which the relevant sections of the Finance Act, 1937, were applicable (see p 139, letter b, post).
Appeals allowed.

Notes
As to the meaning of chargeable accounting period for the purposes of the profits tax, see 20 Halsburys Laws (3rd Edn) 617,
618, para 1204.
As to the duration of grouping notices, see ibid, 641, para 1253.
For the Finance Act, 1937, s 19(4) and s 22(1) see 12 Halsburys Statutes (2nd Edn) 375, 377.

Cases referred to in judgment


Briggs (Henry) Son & Co Ltd (in voluntary liquidation) v Inland Revenue Comrs [1961] 1 All ER 220, 39 Tax Cas 410, [1961] 1
WLR 68, 3rd Digest Supp.
Carpet Agencies Ltd v Inland Revenue Comrs (1958), 38 Tax Cas 223, 3rd Digest Supp.
Inland Revenue Comrs v Buxton Palace Hotel Ltd (1948), 29 Tax Cas 329, 28 Digest (Repl) 375, 1638.
Oriental Inland Steam Co, Re, Ex p Scinde Ry Co (1874), LR 9 Ch App 557, 43 LJCh 699, 31 LT 5, 10 Digest (Repl) 903, 6137.

Cases stated
The first taxpayer company, the holding company, appealed to the Special Commissioners of Income Tax against an assessment
to the profits tax for the chargeable accounting period 27 January 1957, to 12 December 1957, in the sum of 2,142 18s, and a
distribution charge for the same period in the sum of 30,644. The question for determination was whether the accounting period
to 12 December 1957, was the last chargeable accounting period of the company, as the Crown alleged, in which case the
distribution charge was admitted to be justified by virtue of various statutory enactments and particularly s 35(1) of the Finance
Act, 1947, or whether, as the company contended, its functions continued after 12 December 1957, to be what they were before
that date, viz, the holding of investments, so that it continued to be chargeable to the profits tax after 12 December 1957. The
commissioners found that the functions of the company, viz, the holding of investments, continued to be the same before and
after the resolution to wind up which was passed on 12 December 1957. Accordingly, the chargeable accounting period ending
on 12 December 1957, was not the last chargeable accounting period of the company, and its appeal succeeded. The Crown
appealed by way of Case Stated to the High Court.
The second taxpayer company, the spinning company, appealed to the Special Commissioners of Income Tax against an
assessment to the profits tax for the chargeable accounting period 13 December 1957, to 25 January 1958, in the sum of 193
16s, and a distribution charge for the same period of 71 4s. It was agreed that the decision in the case depended on the decision
in that of the first taxpayer company, the holding company, which owned all the shares in the spinning company. If the
commissioners decided, as they did, that the period to 12 December 1957, was not the last chargeable accounting period of the
holding company, then by reason of the grouping notice given by the holding company under s 22 of the Finance Act, 1937, with
respect to the profits of the spinning company, 131the profits of that company continued to be treated as the profits of the
holding company and the assessment on the spinning company fell to be discharged. In view of their decision in the holding
companys case the commissioners discharged the assessment on the spinning company. The Crown appealed by way of Case
Stated to the High Court.
The cases noted belowa were cited during the argument in addition to those referred to in the judgment.
________________________________________
a Re Farrows Bank Ltd [1921] All ER Rep 511, [1921] 2 Ch 164, Re General Rolling Stock Co, Joint Stock Discount Cos Claim, (1872), 7
Ch App 646, Inland Revenue Comrs v Costa Rica Ry Co Ltd (1942), 29 Tax Cas 34

H Magnus QC, E Blanshard Stamp and A S Orr for the Crown.


R Borneman QC and P Rees for the taxpayer companies.

5 March 1963. The following judgment was delivered.

BUCKLEY J. These are two appeals from decisions of the Special Commissioners of Income Tax relating to assessments to the
profits tax made on two companies, Olive Mill Ltd, which I will call the holding company, and Olive Mill Spinners Ltd, which I
will call the spinning company. Before 1953 the holding company had carried on the trade of cotton spinning. In 1953 the
spinning company was incorporated and acquired from the holding company its business of spinning cotton in consideration of
an issue to the holding company of shares of the spinning company credited as fully paid. From that time forward the holding
company held shares in the capital of the spinning company and also owned and held certain other investments. In fact, it
became an investment company. The spinning company continued to carry on the trade of spinning cotton. That state of affairs
continued until 12 December 1957, when both companies passed appropriate resolutions to go into members voluntary winding-
up.
In the course of 1956 the holding company gave a grouping notice pursuant to s 22 of the Finance Act, 1937, as a result of
which, so long as that notice remained effective, the profits of the spinning company fell to be treated as the profits of the holding
company for profits tax purposes. It is not disputed that, until the date of the resolution to wind up, the holding company was
chargeable to the profits tax in respect both of its own profits and of the profits of the spinning company in consequence of the
grouping notice. The major question in this case is whether, as a result of the resolution to wind up the holding company, the
chargeable accounting period for profits tax purposes applicable to the holding company which was then current and which then
came to an end was the last chargeable accounting period of that company, or whether the company remained chargeable to
profits tax so that the period ended 12 December 1957, was not the last chargeable accounting period.
Section 19(1) of the Finance Act, 1937, creates the charge to the profits tax in respect of each chargeable accounting period
of the liable company. Subsection (2) provides:

(2) Subject as hereafter provided, the trades and businesses to which this section applies are all trades or businesses of
any description carried on in the United Kingdom, or carried on, whether personally or through an agent, by persons
ordinarily resident in the United Kingdom.

Subsection (4) is in the following terms:

(4) Where the functions of a company or society incorporated by or under any enactment consist wholly or mainly in
the holding of investments or other property, the holding of the investments or property shall be deemed for the purpose of
this section to be a business carried on by the company or society.

Whether in the absence of sub-s (4) the business of an investment company would be held to be a trade or business within sub-s
(2) is perhaps now not very relevant because the section does in fact include sub-s (4). Subsection (4) may perhaps have been
included in the section ex majore cautela to make 132 sure that the business of an investment company would be caught by the
section; but as the section stands it is, I think, clear that the business of an investment company is chargeable if it falls within sub-
s (4), but not otherwise; in other words, the business of an investment company is not within sub-s (2). That view was shared by
Harman J, who in Carpet Agencies Ltd v Inland Revenue Comrs ((1958), 38 Tax Cas at p 229) said this:

This clearly means that, in the view of the legislature, in the absence of the subsection [sub-s. (4)] the mere holding of
investments or property would not constitute the carrying on of a business.

Section 30 of the Finance Act, 1947, contains provisions which are relevant to this case, but which I need not read, relating to
reliefs in respect of profits which are not distributed and distribution charges which fall to be made when retained profits are
distributed in a later accounting period.
The holding company was assessed to the profits tax for the chargeable accounting period 27 January 1957, to 12 December
1957, in the sum of 2,142 18s and a distribution charge for the same period in the sum of 30,644. The company appealed to
the Special Commissioners against that assessment. The propriety of that assessment depends on the proper view of the question
whether the accounting period ended 12 December 1957, was or was not the last chargeable accounting period of the company or
whether, as the company contended before the Special Commissioners, its functions continued after 12 December 1957, to be
what they were before that date, viz, the holding of investments, so that it continued after that date to be chargeable to the profits
tax; ie, so that it continued after that date to fall within the terms of s 19(4) of the Act of 1937.
Among the facts found by the Special Commissioners in their Case stated in the holding companys appeal was that in 1957,
in consequence of the difficulties that had then arisen in the cotton spinning trade, the directors of both companies were minded
to dispose of their assets to the best advantage before losses, which they anticipated, might be incurred. Various courses of action
were considered and investigated, but eventually the two companies were put into liquidation by the resolutions of 12 December
1957. The liquidator who then came into the saddle continued the efforts, which the directors had theretofore been making, to
dispose of the assets of the spinning company to the best advantage or to dispose of the shares of members of the holding
company. The liquidator for a time continued to hold the shares of the spinning company, and also continued to hold the other
investments of the holding company. Those other investments were sold in January, 1958, and in the same month the liquidator
sold the mill and machinery which belonged to the spinning company, but by arrangement with the purchasers he continued to
process yarn which belonged to the spinning company at the mill and to have the use of the offices and warehouse house there.
The yarn so processed was sold as occasion offered. The last sale of yarn did not take place until October, 1958. The secretary of
the holding company was retained in the employment of that company until that date because, as the commissioners find, he had
a very good knowledge of the cotton industry which presumably was of value to the liquidator. During this period there were
negotiations for the sale of the shares of the holding company on terms which contemplated that the liquidation might be stayed,
but these came to nothing.
On 21 May 1958, the liquidator, as liquidator of the spinning company, made a first distribution to the holding company as a
shareholder in the spinning company of a sum of 124,000, and on 17 March 1959, in the same capacity he made a further
distribution to the holding company of 18,000. On those same dates as liquidator of the holding company he made a first
distribution of sums amounting to 180,764, and on 24 March 1959, a further distribution amounting to 18,076. The
Commissioners of Inland Revenue under their statutory 133 powers determined that the period from 27 January 1957, to 12
December 1957, should be an accounting period for the relevant purposes of the holding company.
The contention of the holding company before the Special Commissioners was that its function of holding investments
continued after 12 December 1957, in the same way as it had existed before that date, and that accordingly the period ending on
12 December 1957, was not its last chargeable accounting period, because it remained a company chargeable to profits tax under
s 19(4) of the Act of 1937. On the other hand, the Crown contended that the companys trade or business ceased for the relevant
purposes on 12 December 1957, at the latest, and that that date was the end of the last chargeable accounting period. In para 6 of
the Case Stated the Special Commissioners say this:

We the commissioners who heard the appeal, found that the functions of the company, namely, the holding of
investments, continued to be the same before and after the resolution to wind up which was passed on Dec. 12, 1957.
Accordingly, the chargeable accounting period ending on Dec. 12, 1957, was not the last chargeable accounting period of
the company and the appeal must succeed.

It is from that conclusion that the Crown now seek to appeal.


As regards the spinning company, an assessment was made for the chargeable accounting period from 13 December 1957, to
25 January 1958, in the sum of 193 16s (tax) and a distribution charge for the same period in the sum of 71 4s. That
assessment was made on the footing that on 12 December 1957, the grouping notice which had been given by the holding
company ceased to operate, so that from that date the spinning company was chargeable in its own right to the profits tax in
respect of the profits earned as the result of the carrying on of the spinning companys business by the liquidator in the manner
that I have already indicated. The spinning company appealed against that assessment, and before the Special Commissioners the
matter was dealt with on the footing that the result of that appeal depended on the decision in the appeal of the holding company;
ie, on the footing that, if it could be shown that the holding company remained chargeable to the profits tax and that the functions
of the holding company remained the same after 12 December 1957, as before, the grouping notice would continue in operation
so that the profits of the spinning company would be treated as the profits of the holding company and the spinning company
would not be separately assessable.
Before me a point of law has been taken which I will explain later, which was not taken before the Special Commissioners
or brought to their notice.
The position with regard to the holding company turns on the question whether on the true interpretation of s 19 of the Act
of 1937 the functions of the holding company continued after 12 December 1957, to be wholly or mainly the holding of
investments or other property, and in that connexion I have been referred to various authorities. There are only two to which I
shall need to refer in any detail, but there are earlier decisions which are relevant and are sufficiently cited in those authorities for
it to be unnecessary for me to refer to them separately.
In Carpet Agencies Ltd v Inland Revenue Comrs the relevant company carried on the trade of dealing in carpets until 31
March 1953. It then ceased to deal in carpets. At that time it held certain investments and other assets but had disposed of all its
trading stock. The question which arose was whether after 31 March 1953, the company was still chargeable to the profits tax. It
had ceased to carry on its trade of dealing in carpets and so had ceased to come within the terms of s 19(2). Had it brought itself,
however, within the terms of s 19(4)? Harman J referred amongst other authorities, to the decision of Atkinson J in Inland
Revenue Comrs v Buxton Palace Hotal Ltd. In that case Atkinson J considered the construction of s 19(4), in relation to a
company the business of which had been carrying on a hotel but that hotel 134 had been requisitioned so that at the relevant time
the company was merely in receipt of a compensation rent. Atkinson J had to consider whether in those circumstances the
company could be said to be a company the functions of which consisted wholly or mainly in holding investments or other
property. Atkinson J said this ((1948), 29 Tax Cas at p 334; 38 Tax Cas at p 229):

Now, what is the meaning of the word functions? The dictionary definition (I think it is a very good one for the
purposes of this case) is: the activities appropriate to any business; so that it ought to read: Where the activities
appropriate to any business consist wholly or mainly in the holding of investments or other property. So read, of
course, it is plain that the words would only apply to a company whose business could be so described. The activities
appropriate to this business of hotel proprietors certainly did not consist wholly or mainly or at all in the holding of
investments or other property, but it is in effect argued that the words should be read as equivalent to activities simpliciter
during the relevant periodwhat, in fact, it is asked, during the relevant chargeable period were the activities of the
company? I do not accept that interpretation of the word. I think that sub-s. (4) is aimed at companies of a particular
nature and quality, companies who set out to make profit in the way indicated or who have changed their business into one
of that character.

Harman J said that he entirely agreed with those observations and then he, for himself, said this ((1958), 38 Tax Cas at p 230):

It seems to me that in order to get within s. 19(4) you must prove not merely that the company is one that happens to
hold some income-bearing investments but that one of its functions, that is to say one of its purposes, has always been or
has been for a considerable time the making of money by the holding of investments.

In a later case he somewhat modified that statement because he said that he did not think he was justified in saying that one had
to find that it had always been or had been for a considerable time one of the purposes of the company. The question was
whether at the relevant time it was one of the purposes of the company b.
________________________________________
b Henry Briggs, Son & Co Ltd (in liquidation) v Inland Revenue Comrs, [1960] 1 All ER at p 804; 39 Tax Cas at p 427

After that reference to Inland Revenue Comrs v Buxton Palace Hotel Ltd. Harman J went on thus ((1958), 38 Tax Cas at p
230):

On the facts stated in this Case, which I need not repeat, the commissioners held that this was not a company within s.
19(4) of the 1937 Act. With that finding I find myself entirely in agreement. I am of the opinion that the appellants never
had but one business, the business of carpet merchants. It is true it held some investments, but those were wholly ancillary
to its merchanting business and were used in fact to finance that very business,

and accordingly he reached the conclusion that on the true interpretation of s 19(4) the company, Carpet Agencies Ltd did not
come within the terms of that subsection.
The other case to which I must refer is Henry Briggs, Son & Co Ltd (in voluntary liquidation) v Inland Revenue Comrs. In
this case the relevant company, Briggs Collieries Ltd had owned coal mines and had engaged in the business of coal mining. As a
result of the nationalisation of the coal industry those assets, ie, the companys mine and coal mining business, had been
compulsorily acquired and the company became entitled to receive compensation under the 135 Coal Industry (Nationalisation)
Act, 1946. On the nationalisation of its colliery interests the company considered various ways of employing its resources in
different fields, but after 1948 it devoted itself to securing its compensation under the nationalisation provisions and ceased to
seek for fresh business. Eventually it went into liquidation. The question there under consideration was whether, during the
period when the company was entitled to receive the compensation but not carrying on any other kind of business, it could be
properly described as a company the functions of which consisted wholly or mainly in the holding of property. Upjohn J after
referring to the passage from Atkinson J, which I have read, reached this conclusion at the end of his judgment ((1961), 39 Tax
Cas at p 423):

A company whose business is temporarily stopped by requisition does not satisfy that test [i.e., ATKINSON, J.s test];
nor does one which has given up tradingsee the cases I have mentioned. They do not bind me, because in this field every
case depends on its own facts once the true meaning of the word functions has been determined; and in this case it is true
to say that Briggs never could re-enter into its colliery activities. Nevertheless, on ceasing to trade as a colliery company, a
company might do one of several things. It might decide to engage in some other trading activity, in which case, in my
judgment, it would remain a trading company; it might decide to become an investment trust company, in which case it
seems to me clear that it would engage in a new business of holding property. If it does neither but resolves to wind up, it
does not seem to me to gain any new character in the sense of carrying on an activity of holding property as being
appropriate to its business. It really has no business. It seems to me it is carrying out a residual function as a colliery
company. It can no longer trade as such; it can only receive its compensation and die,

and on those grounds he came to the conclusion that the company was not within s 19(4).
I do not think that I need read any part of the judgment of the Court of Appeal. Once again Atkinson Js interpretation of the
word functions is referred to without disapprovalindeed, with approvaland Upjohn Js decision was upheld. The case went
to the House of Lords, and in the House of Lords Lord Reid said this ([1961] 1 All ER at p 223):

The Court of Appeal and UPJOHN, J., reached that result by giving to the word functions a meaning which is to my
mind narrower than its ordinary meaning. I do not at all dissent from their view, but I would prefer to lay more stress on
the word holding. It appears to me that, in this context, the phrase the holding of investments or other property does not
mean simply the owning of investments or other property. The word holding often involves the idea of retention
permanently or for an indefinite time, and I think that that is involved in the use of that word in the context. I have said
that the apparent purpose of sub-s. (4) is to bring within the scope of the charge companies conducting something at least
analogous to a trade or business. From that point of view, there appears to me to be an essential distinction between
companies whose function or purpose or objective is to make profit by continuing to hold their assets in the form of
investments or property and companies which are not trying to make profitable use of their assets but whose whole
activities are directed to realising their assets as soon as possible. In the former case, the companys activities are of a
business character. They will almost always involve some degree of active management. Circumstances may for a time
require the company to remain quiescent, but it still has the purpose of deriving profit from continuing to hold its assets in
the form of investments or property. But, in the latter case, the function or purpose or objective of the company 136 is not
to continue to hold investments or property but to cease holding its assets in that form as soon as they can be turned into
money.

On those grounds he agreed with the view which had been taken by the lower courts.
Lord Morris of Borth-y-Gest, at the end of his speech says this ([1961] 1 All ER at p 225):

[It is to be observed that the subsection (i.e., s. 19(4)] does not provide that the mere ownership by a company of
investments or other property brings about the result that such ownership is to be deemed to be a business carried on by the
company. There is only such a deeming if the functions of the company consist wholly or mainly in the holding of
investments or other property. The conjunction of the words functions and holding suggest, to my mind, something
more than a mere ownership of some property and certainly something more than an enforced or involuntary ownership of
rights to receive compensation money, which ownership it is hoped will be terminated at the earliest possible moment by
the receipt of the money. The company did not set out to acquire or to own investments or property, and, just as there was
nothing intentional or purposeful in its acquisition of its right to compensation, so there was no intention or wish to
continue to own such rights but, on the contrary, there was an eager wish for an early settlement followed by a dissolution.
In these circumstances I cannot think that the company was one of which it could aptly be said that its functions consisted
wholly or mainly in the holding of investments or other property.

On those authorities counsel for the Crown submits that, on the true construction of s 19(4), the word functions is
synonymous with the word purposes or objectives. Secondly, he submits that, for a company to be within s 19(4), it must be
shown not merely that it holds investments or other property but also that its purpose is to continue to hold them in order to
derive advantages therefrom. Thirdly, he says that it is impossible, as a matter of law, to attribute such a purpose to a company in
liquidation, for on liquidation it ceases to become the beneficial owner of its assets and the statutory duty of the liquidator is to
realise those assets as soon as is convenient for the benefit of creditors and members of the company. In that connexion I was
referred to the Companies Act, 1948, s 281, which provides:

In the case of a voluntary winding-up, the company shall, from the commencement of the winding-up, cease to carry
on its business, except so far as may be required for the beneficial winding-up thereof.

The commencement of the winding-up in the case of a voluntary winding-up is, of course, the date of the resolution.
On the other hand, counsel for the holding company has drawn attention to the fact that in none of the cases to which I have
referred was there any earlier history of the company being what I may call a s 19(4) company. The question in those cases was:
had the company in each case acquired the character of being a s 19(4) company? He says that it is common ground that, until
the date of the winding-up resolution, the functions of the holding company were wholly or mainly the holding of investments or
other property, and he says that there was no significant change in that state of affairs at the time of the resolution to wind up. He
has contended that the question what the functions of the company are is a question of fact, and he has submitted that the
commissioners in para 6 of the Case Stated have found the fact that the functions of the company continued to be the same before
and after the resolution to wind up. That finding, I think, must be at least a finding of mixed fact and law, for the propriety of the
finding must depend on the proper interpretation of s 19(4) and the application of that section to the facts of this particular case.
In my judgment it is not possible 137 to say that this court is precluded from considering whether the Special Commissioners
came to the right conclusion because they have framed that paragraph in the language that I have read c. Counsel for the holding
company submits that one must distinguish the functions of the liquidator relating to the realisation of the companys assets and
the application of the proceeds of such realisation from the functions of the company, and he submits, as I say, that the functions
of the company were unaffected by the resolution to wind up.
________________________________________
c See p 134, letter c, ante

Now, although the cases to which I have referred are certainly distinguishable from the present case on the facts, it appears
to me that the decision of the House of Lords in Henry Briggs, Son & Co Ltd v Inland Revenue Comrs provides the answer to the
present problem; for, from the moment of the passing of the resolution to wind up, it was no longer the objective of the company
to continue to hold investments for the purpose of obtaining a benefit by doing so. From that moment the objective of the
company became the realisation of its assets as soon as they could advantageously be realised and the distribution of the proceeds
amongst creditors and members in accordance with their rights in a winding up. To borrow the language of Lord Reid, the
function or purpose or objective of the company ceased to be to continue to hold investments or property and became the
realisation of those assets or turning them into money. No doubt, in order that the assets might be realised in an orderly and
advantageous manner it was right and proper that there should have been some period of time before the assets were in fact
realised, but the objective of the company, from 12 December 1957, onwards was not, in my judgment, that of holding
investments; its objective was to realise its investments.
In that connexion it is important to bear in mind, I think, what Atkinson J said. It is not merely the activities of the company
that are relevant but the activities appropriate to its business, ie, appropriate to its objective. As I read his judgment in the light of
the later decision of the House of Lords, that is really what he means when he uses the words activities appropriate to any
business. He emphasises that activities simpliciter are not the test; one has got to see what are the activities appropriate to a
business set on a particular course. From 12 December 1957, onwards, the course on which the company here was set was the
realisation of its assets. No doubt, it is quite true, that, if the liquidator had found that it would be advantageous for the members
of the company to sell their shares to purchasers who might have wished to have the liquidation stayed, that course might have
been adopted. That course was considered, although that idea came to no fruition. But that was not one of the liquidators duties.
His duties were those provided by the Companies Act, 1948, ie, to realise the assets and to distribute the proceeds amongst the
creditors and members in accordance with their rights. For these reasons I think that the Special Commissioners took a mistaken
view of the proper interpretation to be put on s 19(4) and misdirected themselves in that respect. I think that the conclusions at
which they arrived were wrong and that the appeal in the holding companys case succeeds.
Now, the appeal in the spinning companys case depends on the question whether or not the spinning company remained
grouped with the holding company for the purpose of the Finance Act, 1937. The relevant section for this purpose is s 22 of the
Act of 1937, which, by sub-s (1) enables a company of which another company is subsidiary to give a grouping notice in respect
of the chargeable accounting period in which the notice is given, and that notice continues during all subsequent chargeable
accounting periods throughout which the subsidiary company remains a subsidiary of the principal company. By s 42 of the
Finance Act, 1938, the term subsidiary company is defined. Subsection (1) provides:

For the purposes of s. 22 of the Finance Act, 1937 a body corporate shall be deemed to be a subsidiary of
another body corporate if and so long as 138 not less than three-quarters of its ordinary share capital is owned by that other
body corporate,

and sub-s (3) provides that references to ownership shall be construed as references to beneficial ownership. So that the test is
whether the parent company is the beneficial owner of not less than three-quarters of the ordinary share capital of the subsidiary
company. It seems to me that in fact the grouping notice ceased to be operative on 12 December 1957, on two grounds. First, on
the view that I have taken on the holding companys appeal, the last accounting period of the holding company ended on that date
and, therefore, that company was after that date no longer a company to which the relevant sections were applicable, and the
grouping notice would on that ground have then ceased to operate so that the spinning company would be assessable in its own
right in respect of profits earned after that date.
The grouping notice, I think, also falls to the ground for the reason that the holding company, on going into liquidation,
ceased to be the beneficial owner of the shares in the spinning company within the meaning of the section that I have just read.
In this connexion I refer to the decision of the Court of Appeal in Re Oriental Inland Steam Co Ex p Scinde Ry Co. In that case
James LJ said this ((1874), 9 Ch App at p 559):

The English Act of Parliament has enacted that in the case of a winding-up the assets of the company so wound up are
to be collected and applied in discharge of its liabilities. That makes the property of the company clearly trust property. It
is property affected by the Act of Parliament with an obligation to be dealt with by the proper officer in a particular way.
Then it has ceased to be beneficially the property of the company; and, being so, it has ceased to be liable to be seized by
the execution creditors of the company.

Mellish LJ expressed a similar view. He said ((1874), 9 Ch App at p 561): But, in my opinion, the beneficial interest is clearly
taken out of the company. It has not been suggested to me that there has been any subsequent decision indicating that the views
of the learned lords justices are not still good law, and it therefore appears to me to be established by the authority of that case
that, on the winding-up resolution being passed, the beneficial interest in the spinning companys shares held by the holding
company ceased to reside in the holding company. Consequently the spinning company ceased to be a subsidiary of the holding
company within the meaning of these statutory provisions. For these reasons I think that the appeal of the Crown in the spinning
companys case also succeeds and that the assessment on the spinning company should be restored.

Appeals allowed.

Solicitors: Solicitor of Inland Revenue; Field, Roscoe & Co (for the taxpayer companies).

F A Amies Esq Barrister.


139

[1963] 2 All ER 140


Hall v Hall
FAMILY; Children

COURT OF APPEAL
ORMEROD, DONOVAN AND RUSSELL LJJ
28 FEBRUARY, 1 MARCH 1963

Divorce Custody Jurisdiction Infants Wards of court Infants made wards of court in Chancery Division No order of
chancery judge as to custody or access Prior order of Divorce Division as to access Jurisdiction of Divorce Court to vary
order as to access Matrimonial Causes Act, 1950 (14 Geo 6 c 25), s 26(1) Law Reform (Miscellaneous Provisions) Act, 1949
(12, 13 & 14 Geo 6 c 100), s 9(2).

The fact that proceedings have been taken in the Chancery Division, by virtue of which infants become wards of court, does not
of itself (the Chancery Division having made no order as to custody) oust the jurisdiction of the Divorce Division under s 26(1) a
of the Matrimonial Causes Act, 1950, to make an order as to the custody of or access to the infants (see p 142, letter h, p 144,
letter g, and p 145, letter h, post); but it seems (per Russell and Ormerod LJJ), that in the event of any conflict between orders as
to custody of or access to infants made in the Chancery Division and the Divorce Division, the infants being wards of court in
proceedings in the Chancery Division, the orders of the Chancery Division would prevail (see p 146, letter f, and p 142, letter h,
to p 143, letter a, post).
________________________________________
a Section 26(1) is printed at p 141, letter h, post

Andrews v Andrews and Sullivan ([1958] 2 All ER 305) not followed.


Re Andrews (Infants) ([1958] 2 All ER 308) considered.
A husband and wife were married in 1942. There were three daughters, children of the marriage, aged sixteen, ten and eight
years. In July, 1961, the wife left the husband, and subsequently the magistrates made an order on her application giving her
custody of the children and allowing the husband access. In January, 1962, the wife petitioned for divorce on the ground of
cruelty. In May, 1962, an order was made in this suit allowing the husband access to the children. In July, 1962, the husband
issued an originating summons (under the Law Reform (Miscellaneous Provisions) Act, 1949, s 9) in the Chancery Division
making the children wards of court; the children continued to be wards of court, but no order was made in the Chancery Division
as to custody or access. In October, 1962, applications were made in the Divorce Division by both husband and wife to vary the
order as to access. On appeal from an order of the judge, the effect of which was to deny access to the husband pending the
hearing of the petition, which was expedited,

Held There was jurisdiction under s 26(1) of the Matrimonial Causes Act, 1950, to make the order and in the circumstances the
Court of Appeal would not interfere with the judges exercise of his discretion.
Appeal dismissed.

Notes
As to the power of the Divorce Court to make orders for the custody of and access to infants, see 12 Halsburys Laws (3rd Edn)
393, para 872.
For the Matrimonial Causes Act, 1950, s 26(1), see 29 Halsburys Statutes (2nd Edn) 413; and for the Law Reform
(Miscellaneous Provisions) Act, 1949, s 9(2), see 28 ibid, 777.

Cases referred to in judgments


Andrews v Andrews and Sullivan [1958] 2 All ER 305, [1958] P 217, [1958] 2 WLR 942, 28 Digest (Repl) 710, 2191.
Andrews (Infants), Re [1958] 2 All ER 308, [1958] Ch 665, [1958] 2 WLR 946, 28 Digest (Repl) 706, 2152.

Interlocutory appeal
This was an appeal by a husband, the respondent to a divorce petition by his 140 wife, from an order of Scarman J the effect of
which was to deprive him of access pending suit to the children of the marriage. The facts are stated in the judgment of Ormerod
LJ.

Peter Sheridan for the husband.


C W S Lubbock for the wife.

1 March 1963. The following judgments were delivered.

ORMEROD LJ. This appeal is from a decision of Scarman J given on 20 November 1962, on appeal by the wife from a
decision of the registrar dated 25 October 1962, on a summons for a variation of an order for custody in respect of the children of
the marriage. The parties were married in 1942. There are three children of the marriage: Ann Denise, now aged 16 and,
therefore, not the subject of any custody order; Marie Suzette, aged 10, and Yvonne Teresa, aged 8. The parties lived together
with their children until 18 July 1961, when the wife left the husband. She issued a summons against the husband on 16
November 1961, on the ground of the husbands desertion, persistent cruelty and neglect to maintain (we have not seen the
summons) and the magistrates made an order inter alia giving the custody of the three children to the wife, with access to the
husband. On 23 January 1962, the wife petitioned the High Court for a decree of dissolution on the grounds of the cruelty of her
husband towards her. On 30 May 1962, the husband applied to the Divorce Registry, when an order was made in his favour for
access. The part of the order with which we are concerned is that the husband should have access to the three children of the
marriage for

one weekend in each month on the Saturday from 2 p.m. to 7 p.m. and on the Sunday from 10 a.m. to 5.30 p.m.

For reasons into which I will go in a moment, the attempts on the part of the husband to have access to the children were not very
successful. On 25 October there was before the registrar a summons by both parties to vary the order as to access. So far as the
husband was concerned, it was an application that the children should come to him for Christmas; the wifes application was that
the husband should no longer have access, pending suit, to the children of the marriage. The learned registrar dismissed the
application and, therefore, the matter came before Scarman J who decided that he should allow the appeal. The effect of this was
that the husband should have no access to the children pending a decision in the suit. It is on appeal from that decision that the
matter comes before this court.
The first of the two points that have been argued by counsel for the husband is that, in the circumstances of the case, the
Divorce Division of the High Court has no jurisdiction to make an order. The matter was argued before the learned judge, and
the only authority on the point, Andrews v Andrews and Sullivan, was cited to him. He came to the conclusion that it was a case
in which he had jurisdiction and proceeded to hear the case on its merits. The order sought to be made in this case is an order
under s 26(1) of the Matrimonial Causes Act, 1950, which says:

In any proceedings for divorce or nullity of marriage or judicial separation, the court may from time to time, either
before or by or after the final decree, make such provision as appears just with respect to the custody, maintenance and
education of the children the marriage of whose parents is the subject of the proceedings or, if it thinks fit, direct proper
proceedings to be taken by placing the children under the protection of the court

and that would include the making of the children wards of court. On the face of it, provided that the question of access can be
properly included in the term custody, maintenance and education of the children, it would appear that the wording of the
section is sufficiently clear to include an order of this kind, and in the ordinary way I have not the slightest doubt that it would.
141
However, there is a complication which arises, because, on 3 July 1962, a date subsequent to the hearing of the summons by
the learned registrar, but prior to the hearing of the appeal by Scarman J an originating summons was taken out in the Chancery
Division the effect of which was that the children became wards of court b. So far as this court is aware, these children are still
wards of courtc, but no further steps appear to have been taken in regard to the wardship.
________________________________________
b By virtue of the Law Reform (Miscellaneous Provisions) Act, 1949, s 9(2) and RSC, Ord 54p., r 3, under which an infant becomes a ward of
court on the issuing of the originating summons but, unless within twenty-one days after its issue, an appointment is obtained for the
hearing of the summons, the infant ceases to be a ward of court at the end of the twenty-one days. On an appointment being obtained the
infant continues to be a ward of court until the determination of the application
c It is understood that an appointment was duly obtained before the Chancery Master and that the hearing of the summons was adjourned
generally

The submission made to the court today is that there can be no jurisdiction to make an order under s 26(1) of the
Matrimonial Causes Act if that order is in conflict, as it must be in this case, with the fact that the children have already become
wards of court. On the face of it, that contention would not seem to have substance, because Parliament has enacted that the
court (that is, the High Court) shall have power to make orders. It cannot be questioned, in the ordinary course of events, that the
order which is sought in this case is an order which comes within the terms of s 26(1). The simple answer to this submission
would be that, if the legislature had intended that there should be the restriction on the making of orders under s 26(1), then it
would have said so; whereas it has said in plain terms that this power would be given to the court. Clearly the effect of giving
such a power to the court (if it has in fact been given) can mean that in certain circumstances there could be a head-on clash
between, on the one hand, a judge exercising this jurisdiction in the Probate, Divorce and Admiralty Division and, on the other, a
judge of the Chancery Division exercising his jurisdiction over some infant who has properly been made a ward by originating
summons in the Chancery Division.
There has been, so far as we have been told, only one other case which might be said to deal with this point, and that is
Andrews v Andrews and Sullivan, which is a decision of Wrangham J, and a decision of Upjohn J, in Re Andrews (Infants) in the
Chancery Division of questions arising on the same matter. It is fair to say, after referring to the judgments in those two cases,
that Wrangham J was of the view that, in the circumstances of that case, as the children had been made wards of court, he had no
jurisdiction under s 26(1) to make the appropriate order or any order for custody or maintenance, and he declined to make such
an order; whereas Upjohn J in his judgment, would go no further than to say that it was probable that there was no jurisdiction
under s 26(1) to make an order of the kind referred to, but that the matter depended very largely on the comity which exists
between judges, and it would be necessary to rely on that comity to avoid a head-on clash if the two jurisdictions might be
attempted to be exercised.
How far those decisions are right is probably not a matter for the consideration of this court. I am quite satisfied that the
court in this case had jurisdiction to make the order asked for under s 26(1), because it appears to me to be clear, provided the
other conditions are fulfilled, that the words of the section put no limitation on orders which may be made by the court. I would
not, therefore, hesitate to say that the court has jurisdiction to make an order, even although in the meantime the infant has been
made a ward of court. It may be that the real position is that if, an infant having been made a ward of court, the judge of the
Chancery Division to whom that particular infant is assigned makes some order which clashes with an order which may be made
under s 26(1), the order of the chancery judge might well prevail; but that is a matter with which we are 142 not concerned. I go
no further than to say that, in the circumstances of that case, if it arose, as at present advised, such might be the position.
But that is not this case. This is a case where three infants have become wards of court in the Chancery Division and no
further step has been taken in regard to them in the Chancery Division. I see no reason why the jurisdiction given to the court
under s 26(1) should be ousted in this way. Therefore, I would say that the first ground on which counsel for the husband bases
his appeal is wrong, and I would dismiss that part of the appeal.
However, counsel for the husband goes further and says that, in the particular circumstances of the case, the decision was
wrong because the learned judge, who clearly had a wide discretion in this matter, exercised his discretion on wrong principles
and, therefore, this court has the power to interfere; and, more than that, should interfere by making a different order. The effect
of the learned judges order was this. As I have already said, proceedings for divorce have been started. The date of the petition
was 23 January 1962. In 1962 there were various attempts, set out in the affidavits, on the part of the father to achieve the access
awarded to him. The history of those attempts shows that, even on his own evidence, only on one afternoonthat is, a Saturday
afternoon in Junecould he say that there was any meeting between him and the children which could be in any way described
as successful. That is denied by the mother, presumably on information she had from the children. So far as the other attempts
were concerned, they were, unfortunately, a failure, because, according to the affidavits, of the reluctance of the children to meet
their father and spend time with him.
In those circumstances, the learned judge has come to the conclusion that it would not be wise to make an order for access
pending the hearing of the suit, and the ground on which he based his decision is this. While the suit is pending, clearly there are
bitter feelings between the husband and wife, and in the circumstances of this petition it is perhaps not unnatural that those bitter
feelings still exist. As the children live with the mother, it is almost impossible that they should not be to some extent influenced
by the feelings between their parents. In those circumstances, it would be unlikely that they would have a kindly feeling towards
their father. The learned judge has come to the conclusion that once the proceedings are over, and a decree is granted or refused,
the feeling between the parties may well be a good deal less bitter and access may be easier to achieve. It is not for me to
conclude one way or the other what is the real cause of the reluctance of the children to meet their father or to spend any time
with him. Whether this lies in the attitude of the father, or whether they have in some way or other been put against their father
by their mother, either directly or indirectly, I do not know. The learned judge has ordered that the proceedings should be
expedited in order that as short a time as possible should elapse before the matter is finally decided, and the question of custody
can then be considered afresh.
I see no ground for saying that the learned judge is wrong. Counsel for the husband has argued that on the decided cases the
real object of an order for custody pending suit is to maintain, so far as possible, the status quo which presumably existed at the
time of the separation. Of course, it is a forceful argument that if the husband cannot have access to the children now there is
little reason why he should have access when the decree nisi is made (if it is made) and the question of access is considered
again; but it appears to me that any guide which can be derived from the decided cases on this subject can only be a guide to the
exercise of discretion having regard to the general circumstances of the case. The learned judge has, I think, considered the
various matters which he should consider in deciding whether these children should be, as it were, compelled to see their father
on the occasions when access is ordered; and he has decided that for the time being, and I think for the time being only, it would
be unwise on the childrens behalf to make an order of that kind. In consequence, he has come to the conclusion that he should
allow the appeal from the learned registrar and make an 143 order which in effect denies access to the children by the father
pending the hearing of the divorce proceedings. All I can say, in conclusion, is that I am particularly anxious not to say anything
which might influence any other court when the question of custody has to be considered. The learned judge has made an order
that the proceedings should be expedited so that the matter can be dealt with as quickly as possible. I can only hope that both
parties will take every advantage of that and that at as early a date as possible the issue between the mother and father can be
decided, and an order can then be considered on the question of custody. It may be that in those circumstances some order for
access may be given to the father; but it is not for me to predict, and it is not for me to attempt to influence any court which may
at a later stage have to consider that question. All I say is that I would dismiss the appeal.

DONOVAN LJ stated the facts and continued. The proceedings in the High Court for divorce had begun, and an order for access
had been made in those proceedings, before the children became wards of court. It is in those circumstances that this question of
jurisdiction has to be considered.
Section 26(1) of the Matrimonial Causes Act, 1950, provides inter alia that once proceedings for divorce have begun the
court (which by s 33 of the Act means the High Court) may from time to time, either before or after final decree, make orders as
to the custody, maintenance and education of the children; or it may, if it thinks fit, direct that proper proceedings be taken to
place the children under the protection of the court. Once the Divorce Division of the High Court has begun to exercise
jurisdiction under the subsection, as, for example, by making interim orders for custody, which is the case here, I do not think that
that jurisdiction can as a matter of law be ousted by one or even both of the parties going to the Chancery Division and making
the children wards of court. I draw attention to the words of the subsection which give the High Court, and, therefore, the
Divorce Division of it, jurisdiction from time to time; and give it also jurisdiction, if it thinks fit, to direct other proceedings for
the protection of the children, which would include wardship proceedings. The husbands contention in the present case would
confer on a party to a matrimonial suit power to deprive the Divorce Division of the discretion which Parliament has conferred on
it; and for such a result, I think, clear words would be required. One recognises the risk that may arise in a case like the present,
of conflicting orders, and of some inconvenience if two divisions of the High Court are seized of the same matter; but the comity
between judges should be sufficient to take care of such a risk.
Upjohn J dealt with this aspect of the matter in Re Andrews in terms with which I respectfully agree. If Wrangham J in
Andrews v Andrews and Sullivan intended to say (which is not completely clear) that making a child a ward of court entirely
ousted the jurisdiction of the Divorce Division under s 26(1), I should be bound respectfully to disagree. The matter is purely one
of the construction of s 26(1) and I cannot spell such a result out of its language.
As to the merits of the case, I feel that we should not interfere with the learned judges decision, which, after all, is not a
permanent one; and, while I understand the fathers feelings, they are probably such as not to make him the wisest judge of what
to do for the best at this moment. Looking at the problem dispassionately, I should have thought that there was much to be said
for an interval for feelings on both sides to subside. I think it was short-sighted folly for the mother to insist on being present at
the first meeting; and for the future she should remember that as they grow up these children may well need the help and
affection of a father, and it would be a grievous wrong to the children deliberately to deprive them of that advantage. I agree that
the appeal should be dismissed.

RUSSELL LJ. On the question of the merits of the case, it seems to me that the learned judge considered on the evidence that
there was a considerable 144 adverse effect produced on these children by their meeting with their father. He also considered that
this adverse effect was in large measure due to the bitterness of the current dispute affecting both the parents. Against the harm
done to the children he balanced the claims of the father to access to his own children, and he decided, on balance, in effect, that
further access should be postponed until a time when the matter was decided, when it might be hoped the bitterness of the parents
would be less, and a time, therefore, when it was more likely that these two small children would benefit more from their
meetings with their father. I do not think it can be said that, in striking this balance, the learned judge, in dealing with a short-
term problem, has erred in principle.
On the question of jurisdiction, it appears to me that Wrangham J did decide as a matter of law that once a child became a
ward of court by the issue of an originating summons in the Chancery Division then the jurisdiction of the judge in the Divorce
Division over the custody of the infant under s 26(1) of the Matrimonial Causes Act, 1950, ended. I think the composer of the
headnote in Andrews v Andrews and Sullivan was right when he said:

Held, that since the powers of the Divorce Division were less extensive than those possessed by the Chancery
Division over wards of court, the former were superseded by the latter; the Chancery Division had, moreover, on the
mother making her application, become, in effect, the guardian of the wards of court, and it was that division only which
could give leave for them to go out of the jurisdiction or otherwise control their persons or property.

In Re Andrews (Infants) Upjohn J expressed the view that probably as a matter of law (though he did not put it in this way)
Wrangham J was right. I venture to differ from the Andrews v Andrews and Sullivan decision to a limited extent. A section in the
form of s 26(1), or substantially in that form, has been in the divorce legislation since the Matrimonial Causes Act, 1857. It must
be remembered that until the Law Reform (Miscellaneous Provisions) Act, 1949, which introduced the new procedure in
connexion with making infants wards of court (a procedure under which no child can become a ward of court unless an order is
made to that effect, subject only to the temporary effect of the launching of the originating summons), an infant could become a
ward of court unintentionally or accidentally, without the fact being appreciated, owing to the view that if a trust of any property
of an infant became subject to administration at the hands of the court, that infant automatically became a ward of court: indeed,
a system by which one deliberately made wards of court in former days was by producing an ad hoc 100 settlement on the infant
and asking the Chancery Division to administer the trust. It seems to follow from that that, since before 1949 there were many
instances of infants who were wards of court without anybody really appreciating it and without anybody taking any steps in the
Chancery Division in respect of the control of the infant and his custody, it would have been quite wrong to hold in those days
that automatically when an infant became a ward of court all custody jurisdiction under s 26(1), or rather its then equivalent d,
ceased. To that extent, and for that reason, I would disagree with what I take to be the decision of Wrangham J I do not think
that, either then or now, the mere fact of a child becoming a ward of court somehow ousts the custody jurisdiction of the judge in
matrimonial proceedings under s 26(1). On the other hand, to construe this section and its ancestor in 1857 e, as conferring a
jurisdiction on the judge in matrimonial proceedings to make an order setting at nought an order already made by a judge in the
Chancery Division exercising his jurisdiction over his ward would be, it seems to me, to attribute a very perverse intention to
Parliament. Before the Matrimonial 145 Causes Act, 1857,f the law undoubtedly was that in the case of a ward of court the Court
of Chancery had the absolute say in respect of matters such as custody, access, education and so on. I do not think that a
matrimonial statutory provision should be taken as authorising in effect, a conflict between an irresistible force and an immovable
object. I venture to agree with this comment by Upjohn J, in Re Andrews (Infants) ([1958] 2 All ER 308 at p 310; [1958] Ch 665
at p 668):
________________________________________
d Supreme Court of Judicature (Consolidation) Act, 1925, s 193(1), which had itself replaced the Matrimonial Causes Act, 1857, s 35, and the
like Act of 1859, s 4
e See Matrimonial Causes Act, 1857, s 35
f 20 & 21 Vict c 85; ie, before 1858

One thing is, I think, clear; an exercise by the judges of the Divorce Division of their statutory powers cannot in any
way fetter the powers of the Chancery Division exercising the jurisdiction of the Crown as parens patriae over wards of
court.

If that be right, then either s 26(1) of the Matrimonial Causes Act, 1950, must be construed in a limiting way or there is the
potential absolute conflict that I have indicated. In my judgment, as a matter of sensible approach to the statutory provision
against the background of the existing powers of the chancery judges, the appropriate approach is to construe the powers
conferred by that section on the divorce judge in matrimonial proceedings in a limited way, so as not to authorise the judge in the
Divorce Division to make an order contrary to an order of the judge in the Chancery Division.
The result of that, first of all, it seems to me, is that while the chancery guardian (if I may so describe the Chancery
Division) still slept, while he had not been stirred to come to a decision about the ward, there is no reason why the divorce judge
should not exercise his jurisdiction under the section. That was the position both when the registrar made his order and when
Scarman J, made his order; and it is the position in which we sitting in the Court of Appeal are making our decision. It may be
that in due course this question of access, amongst other questions, will be brought before a chancery judge in wardship
proceedings; it seems to me that that may well be done. He would then have to consider what course he should adopt. It might
be that the evidence before him would be different, in which case he would perhaps feel himself at liberty to say: Not only have I
as absolute guardian of this child the right to override the divorce judges view, but, having regard to various matters in this case
and this evidence, I think it right to do so. If that were to happen the decision of the judge of the Chancery Division would, in my
judgment, override the decision of the judge in the Divorce Division. If the matter comes before a judge in the Chancery
Division in this way, he will be faced with the sort of problem discussed by Upjohn J in Re Andrews and no doubt will be guided
by the considerations which are there set out whether the judge of the Chancery Division should leave it to the Divorce Division,
and probably de-ward; or should keep the child a ward, and on any particular matter content himself with the views, for example,
of Scarman J on the interim access; or should say, I will keep the child a ward, and I am going to take a different view about
that. I agree, for those reasons, perhaps differing slightly from my brothers, that both on the merits and on the question of
jurisdiction the appeal fails.

Appeal dismissed.

Solicitors: Edward F Iwi agent for Steed & Steed, Sudbury (for the wife); Dale, Parkinson & Co (for the husband).

Henry Summerfield Esq Barrister.


146
[1963] 2 All ER 147

Ministry of Agriculture, Fisheries and Food v Jenkins and Another


AGRICULTURE: LANDLORD AND TENANT; Tenancies: CONSTITUTIONAL; Crown

COURT OF APPEAL
LORD DENNING MR, DANCKWERTS AND DAVIES LJJ
12 MARCH 1963

Agriculture Agricultural holding Notice to quit Crown land Required for non-agricultural use No consent of
Agricultural Land Tribunal necessary Agricultural Holdings Act, 1948 (11 & 12 Geo 6 c 6), s 24(1), (2) (b), as amended by the
Agriculture Act, 1958 (6 & 7 Eliz 2 c 71), s 8, Sch 1, para 8 Town and Country Planning Act, 1947 (10 & 11 Geo 6 c 51), s
87(2) (b).

The Minister of Agriculture, Fisheries and Food gave notice to quit in respect of two agricultural holdings of which the minister
was the landlord. The stated purpose for which the notices to quit were given was afforestation by the Forestry Commission, a
non-agricultural use. The tenants served counter-notices. The consent of the Agricultural Land Tribunal to the operation of the
notices to quit was not obtained. Accordingly the notices would be invalidated by s 24(1) of the Agricultural Holdings Act, 1948,
unless they fell within s 24(2)(b)a of that Act, by which enactment the tribunals consent would not be needed if the notices to
quit were given for a non-agricultural use for which permission under the enactments relating to town and country planning was
not required otherwise than by virtue of any provision of those enactments.
________________________________________
a The relevant terms of s 24(2)(b) are printed at p 149, letter a, post

Held The Crown did not require planning permission in respect of its own interests in land, because the Town and Country
Planning Act, 1947, did not bind the Crown (s 87(2)(b) of that Act proceeding, on its true construction, on that assumption), and
this reason was one that was not by virtue of any provision of that Act; therefore the notices to quit fell within the exemption
enacted by s 24(2)(b) of the Agricultural Holdings Act, 1948, and the consent of the Agricultural Land Tribunal was not required.

Notes
As to the circumstances in which counter-notices do not restrict the operation of a notice to quit an agricultural holding, see 1
Halsburys Laws (3rd Edn) 284, 285, para 601; and for cases on the subject, see 2 Digest (Repl) 1618, 7176.
As to the presumption that the Crown is not bound by a statute unless the contrary is expressly stated or necessarily implied,
see 36 Halsburys Laws (3rd Edn) 430432, para 652; and for cases on the subject, see 42 Digest 689693, 10361082.
For the Agricultural Holdings Act, 1948, s 24, see 28 Halsburys Statutes (2nd Edn) 46, 47; and for the Agriculture Act,
1958, s 8 and Sch 1, para 8, see 38 ibid 76, 81.
For the Town and Country Planning Act, 1947, s 87(2), see 25 Halsburys Statutes (2nd Edn) 603.

Appeal
This was an appeal from the decision and order of His Honour Judge Trevor Morgan QC, given on 19 December 1962, on a
Special Case Stated for the opinion of the court by Frank Kynaston Ikin, the arbitrator appointed in an arbitration between the
Ministry of Agriculture, Fisheries and Food (the landlords) and Isaac R Jenkins (the first tenant) and Jenkins Hughes (the second
tenant) whereby differences were referred to arbitration in accordance with Sch 6 to the Agricultural Holdings Act, 1948. The
following were the material facts. Before 11 August 1955, the first tenant was the tenant on an annual Michaelmas tenancy of a
farm holding in the county of Cardigan, and the second tenant was the tenant of a similar holding in another parish in that county.
On 11 August 1955, the landlords, in exercise of powers conferred by the Forestry Act, 1945, purchased the reversion expectant
on both of these tenancies and became the landlords 147 of both tenants. By notice to quit dated 18 September 1961, the
landlords required the first tenant to quit his holding on 29 September 1962; the notice stated that it was given pursuant to s 24(2)
and s 31(1) and (2) of the Agricultural Holdings Act, 1948, with a view to the use of the land to which it related for the planting
of trees, and on the ground that the land was required for use other than agriculture for which (otherwise than by virtue of any
provision of the enactments relating to town and country planning) permission was not required, namely, the use of the land by
the Forestry Commission for the planting of trees, not being a use for woodlands where that use was ancillary to the farming of
land. On the same day a similar notice was similarly served by the landlords on the second tenant. On 10 October 1961, each of
the tenants served on the landlords a notice stating that he wished to contest the reason stated in the notice to quit served on him,
and that he required the question to be determined by arbitration under the Agricultural Holdings Act, 1948. The material facts
proved at the arbitration were (a) that the land to which the tenancies were subject was purchased in 1955 by the Minister of
Agriculture, Fisheries and Food under powers given by the Forestry Act, 1945; (b) that on 5 October 1955, the minister duly
made an order under the Act placing the land at the disposal of the Forestry Commission, and (c) that the commission required
the land for the purpose of afforestation, such afforestation not being ancillary to agriculture. At the hearing of the Special Case
the learned county court judge decided that the notices to quit were valid and effective. By notice of appeal dated 4 January
1963, the first tenant and the second tenant gave notice of appeal to the Court of Appeal on the grounds that the learned county
court judge was wrong in law in finding that the notices to quit were valid, and that he was wrong in law in finding that the
Crown was not bound by material provisions of the Town and Country Planning Act, 1947, namely, s 87 thereof, so far as the
interests of the first and second defendants were concerned in the said holdings.

Anthony Cripps QC and John Morris for the appellant tenants.


J C Leonard for the respondent ministry.

12 March 1963. The following judgments were delivered.

LORD DENNING MR. This raises a question of some importance under the Agricultural Holdings Act, 1948, and the Town
and Country Planning Act, 1947. The Ministry of Agriculture, Fisheries and Food are the landlords of extensive tracts of land in
Wales. On 18 September 1961, they gave notices to quit to two farmers, Mr Jenkins and Mr Hughes, in respect of considerable
areas of land there. The notices to quit were a years notice in each case to expire on 29 September 1962. No doubt the farmers
had used these tracts of land as sheep runs, but the ministry, having the interests of the Forestry Commission in their hands,
desired to use them for the planting of forests, and so they gave notice to quit for that purpose. The question which arises in this
case is whether those notices to quit have effect without more ado. The farmers say that they have no effect unless the
Agricultural Land Tribunal gives its consent to the operation of them.
Section 24(1) of the Agricultural Holdings Act, 1948, as amended b in consequence of s 3(1) of the Agriculture Act, 1958,
says that
________________________________________
b The amendment is effected by s 8 of and Sch 1, para 8, to the Agriculture Act, 1958

where notice to quit an agricultural holding is given to the tenant thereof [as was done here], and the tenant
serves on the landlord a counter-notice [as each of these tenants did here] the notice to quit shall not have effect unless
the Agricultural Land Tribunal consents to the operation thereof.

It is quite plain from s 87(2) of the Act of 1948, c that that provision applies to the areas of land here in question, notwithstanding
that they are Crown lands. So, stopping there, it would seem that the notices to quit would not have effect unless the Agricultural
Land Tribunal consented to the operation thereof 148and no such consent has been given. But sub-s (2)(b) of s 24, which is a
rather complicated section, goes on to say that sub-s (1) does not apply, that is to say, the consent of the tribunal is not required,
where
________________________________________
c 28 Halsburys Statutes (2nd Edn) 90

the notice to quit is given on the ground that the land is required for a use, other than for agriculture, for which
permission has been granted on an application made under the enactments relating to town and country planning, or for
which (otherwise than by virtue of any provision of those enactments) such permission is not required, and that fact is
stated in the notice.

That is a provision which I find difficult to follow. It means, I think, that a landlord does not have to get the consent of the
Agricultural Land Tribunal if (a) he requires the land for a non-agricultural use, and (b) in addition he shows either (i) that he has
got permission under the town and county planning Acts so to use it or (ii) that he does not require permission under those Acts.
In a case under (i) where he has already got planning permission, he must have made a specific application for non-agricultural
use and have been granted specific permission for it. The general permission under the Town and Country Planning General
Development Order, 1950,d will not do. And in a case under (ii), where he does not require permission for the non-agricultural
use, the reason must be by virtue of some extrinsic good reason and not by virtue of any provision in the town and country
planning Acts.
________________________________________
d SI 1950 No 728, art 3; see 21 Halsburys Statutory Instruments (1st Re-issue) 88

In order to illustrate the operation of s 24(2)(b) of the Agricultural Holdings Act, 1948, I take the case of a private landlord,
who desires to turn his land over to afforestation instead of agriculture. He does not have to get planning permission, for the
simple reason that the use of land for afforestation is not development of the land; see s 12(2)(e) of the Town and Country
Planning Act, 1947, which says that the use of any land for the purposes of agriculture or forestry (including afforestation) shall
not be deemed to involve development of the land. So planning permission is not required. But in that case it is by virtue of a
provision of the enactments relating to town and country planning that it is not required, namely, by virtue of s 12(2)(e) of the
Act of 1947. He is not therefore exempted under s 24(2)(b) of the Agricultural Holdings Act, 1948, from obtaining the consent of
the Agricultural Land Tribunal. It follows that a private landlord, who desires to turn his land over to afforestation, must get the
consent of the tribunal in order to make his notice to quit effective.
But what about the Crown? If the Crown desires to turn Crown lands over to afforestation, it does not have to get planning
permission. That is quite plain. But why does the Crown not have to get planning permission? Is it by virtue of any provision in
the town and country planning Acts? That is to say, is it by virtue of s 12(2)(e) of the Town and Country Planning Act, 1947, as it
would be in the case of a private landlord? Or is it by virtue of s 87(2)(b) of the Town and Country Planning Act, 1947? Or is it
by virtue of the fact that it is the Crown and, as such, it is not bound by an Act of Parliament except in so far as it is included
expressly or by reasonable implication?
Looking at the whole of the Town and Country Planning Act, 1947, I am satisfied that the Crown does not need to get
planning permission in respect of its own interest in Crown lands. The reason why it is exempt is, not by virtue of any provision
in the Act itself, but by reason of the general principle that the Crown is not bound by an Act unless it is expressly or impliedly
included. Section 87(2)(b) does not exempt the Crown. It proceeds on the assumption that the Crown is already exempt. It says
that,

Notwithstanding any interest of the Crown in land being Crown land any [planning] restrictions shall apply
and be exercisable in relation 149 to the land, to the extent of any interest therein held otherwise than by or on behalf of the
Crown.

That provision assumes that the Crown is already exempt in respect of its own interest in Crown land. All it does is to make sure
that other persons (eg its tenants) have to get planning permission in respect of their interests: and it preserves the Crown
exemption in respect of its own interests.
Coming back then to s 24(2)(b) of the Agricultural Holdings Act, 1948, I hold that planning permission is not required by
the Crown in order to use these lands for afforestation, and that the reason why it is not required is because it is the Crown, not by
virtue of any provision in the town and country planning Acts. The Crown is entitled therefore to the benefit of s 24(2)(b) and it
is unnecessary for it to get the consent of the Agricultural Land Tribunal. The notices to quit operate and are effective, even
without the consent of the Agricultural Land Tribunal.
I find myself, therefore, in agreement with the decision reached by the county court judge and I would dismiss this appeal.

DANCKWERTS LJ. I agree. At one period during the course of the arguments I was in considerable confusion, but I have
emerged from the wallow of statutory enactments into which counsel for the tenants dragged us with the clear idea that the
argument of counsel for the Ministry of Agriculture, Fisheries and Food is right. The basis of his argument is the indisputable
one that the Crown is not bound by statute, unless it is expressly mentioned or involved by necessary implication. I think that he
is right in saying that there is nothing which does involve the Crown in that way in the Town and Country Planning Act, 1947.
Accordingly, the claim of the Crown in the present case to be free from the question of consent is not based on any provision in
that Act. It does not have to rely on the exemption, or whatever it may be, of forestry from the definition of development in the
Town and Country Planning Act, 1947, and its claim could equally be put if the desired development or desired operation were
the provision of a camp for the military and not merely the planting of trees. In that case it seems to me that the Crown, in the
words of this rather peculiar subsection, has a case where such permission is not required otherwise than by virtue of any
provision of those enactmentsthose enactments for the present purpose being the Town and Country Planning Act, 1947.
Therefore, I think that the county court judge came to the right conclusion and that the appeal should be dismissed.

DAVIES LJ. I also have suffered on the same muddy journey as Danckwerts LJ but I feel that I have at last emerged on to firm
ground. I can express my conclusion in one sentence, I agree with counsel for the ministrys submission that counsel for the
tenants argument in effect comes to this, that the parenthesis in s 24(2)(b) of the Agricultural Holdings Act, 1948, otherwise
than by virtue of any provision of those enactments, really means by some Act of Parliament other than those enactments.
But, as my lord has said, the position of the ministry here does not arise out of any Act of Parliament; it arises out of the fact that
they represent for this purpose the Crown. I agree, therefore, that the appeal fails.

Appeal dismissed. Leave to appeal to the House of Lords granted.

Solicitors: Horace C Davies & Co agents for D Emrys Williams & Co, Aberystwyth (for the tenants); Solicitor, Ministry of
Agriculture, Fisheries and Food.

F Guttman Esq Barrister.


150
[1963] 2 All ER 151

Lewis and another v Daily Telegraph Ltd


Same v Associated Newspapers Ltd
TORTS; Defamation

HOUSE OF LORDS
LORD REID, LORD JENKINS, LORD MORRIS OF BORTH-Y-GEST, LORD HODSON AND LORD DEVLIN
3, 4, 5, 6, 12, 13, 17, 18, 19 DECEMBER 1962, 26 MARCH 1963

Libel Innuendo No extrinsic evidence to support innuendo Innuendo meaning allegedly implicit in ordinary meaning
Whether defendants entitled to ruling whether words capable of bearing innuendo meaning Newspaper report that police fraud
squad inquiring into companys affairs, and naming chairman Admission that words defamatory in ordinary meaning
Innuendo that chairman guilty of fraud Whether words capable of imputing guilt of fraud as distinct from suspicion Pleading
innuendoes RSC, Ord 19, r 6(2).

Libel Damages Assessment Jury Appeal Unreasonable award.

Libel Damages Income tax to be taken into account where loss of income results.

Libel Damages Two actions for similar libels in different newspapers Direction to jury to consider whether and how far
damage attributable solely to the libel in one action Defamation Act, 1952 (15 & 16 Geo 6 & 1 Eliz 2 c 66), s 12.

Where words are defamatory in their natural or ordinary meaning an innuendo does not constitute a separate cause of action
unless it is an innuendo that requires the support of extrinsic fact (see p 160, letter b, p 159, letter c, p 167, letter b, and p 170,
letter g, post; cf p 154, letter e, post).
Grubb v Bristol United Press Ltd ([1962] 2 All ER 380) approved.
Observations on the pleading of innuendoes (see p 171, letters b to d, post; cf p 166, letter h, post).
Two newspapers each published statements that officers of the City of London Fraud Squad were inquiring into the affairs
of the [R Co] and its subsidiary companies and that the chairman of the R Co was L L and the R Co brought actions for libel
against each newspaper. The two sets of actions were tried separately. L pleaded an innuendo to the effect that the statement
meant that he had been guilty of fraud or was suspected by the police of having been guilty of fraud or dishonesty in connexion
with R Cos affairs. R Co pleaded an analogous innuendo. The plaintiffs did not allege special damage. The defendants
admitted that the words were defamatory in their ordinary meaning, but pleaded justification in that the fraud squad were at the
time of publication inquiring into the affairs of R Co. The defendants did not seek to justify the extended meaning pleaded in the
innuendo. At the trial no extrinsic fact was proved in support of the innuendo, but the trial judge rejected the defendants
submission that the innuendo should not be left to the jury. The plaintiffs did not tender in chief any evidence of financial
damage. The defendants established that an inquiry by the fraud squad into R Cos affairs was in existence at the the material
time. The trial judge left to the jury two questions, namely, (1) whether they found for the plaintiffs or the defendants and, (2) if
for the plaintiffs, for what damages; but he did not leave separate questions to the jury as to the natural and ordinary meaning of
the words and as to the innuendo, and did not give any directions to the jury as to the innuendohe simply left to the jury the
innuendo meaning as a possible ordinary meaning. He did not caution the jury on the absence of any real evidence of loss of
business resulting from the libel. He referred to the fact that a similar action was about to be heard for a similar article published
at the same time, and intimated that the jury should take that into account giving to it such weight as they thought that it deserved
in assessing damages. In the first trial the jury awarded L 25,000 and the R Co 75,000 damages. Two 151 days later the jury
in the second trial awarded L 17,000 and R Co 100,000 damages against the other newspaper. On appeal,

Held (i) On the question of misdirection concerning the particular defamatory meaning of the words complained of
(a) where an extended defamatory meaning of words admittedly libellous in their ordinary meaning was put forward by a
plaintiff, the defendant was entitled to a ruling whether the words were capable of bearing that particular extended meaning; in
the present case the trial judge had not so ruled (viz had not ruled whether the words were capable of imputing guilt of fraud as
distinct from suspicion), and (Lord Morris of Borth-y-Gest dissenting) his failure to do so amounted to a misdirection, sufficient
to require a new trial, since the ordinary man would not infer guilt of fraud merely from the existence of an inquiry (see p 154,
letter h, p 155, letter f, p 160, letter g, p 168, letter a, and p 174, letter i, to p 175, letter a, post; cf p 164, letter b, post).
Capital and Counties Bank v Henty ((1880), 5 CPD 514) and Nevill v Fine Art and General Insurance Co ([1897] AC 68)
considered.
(ii) on the question of damages
(a) the damages awarded were in any event so excessive that they could not be allowed to stand (see p 155, letter i, p 164,
letter c, p 168, letter i, p 175, letter b, and p 157, letter b, post).
(b) (per Lord Reid; Lord Jenkins, Lord Morris of Borth-y-Gest and Lord Hodson, concurring) where there is more than one
action arising on similar libels, the jury in each action should be directed that, in considering the evidence submitted to them, they
should consider how far the damage suffered by the plaintiffs can reasonably be attributed solely to the libel with which they are
concerned, and how far it ought to be regarded as the joint result of the libels (see p 156, letter c, p 157, letter b, p 164, letter d,
and p 169, letter b, post).
(c) (per Lord Reid; Lord Jenkins, Lord Morris of Borth-y-Gest and Lord Hodson concurring) where a plaintiff proved loss of
income as a result of a libel, then, if the plaintiff were a company, income tax at the standard rate on the loss of profit should be
taken into account, or, if the plaintiff were an individual and proved such loss of income, income tax (including surtax, if
applicable) should be taken into account, and the jury should be directed accordingly (see p 156, letter i, to p 157, letter a, p 157,
letter b, p 164, letter d, and p 169, letter a, post).
British Transport Commission v Gourley ([1955] 3 All ER 796) applied. Decision of the Court of Appeal ([1962] 2 All ER
698) affirmed.

Notes
As to meaning and sufficiency of an innuendo, see 24 Halsburys Laws (3rd Edn) 86, 87 paras 154156; and for cases on the
subject, see 32 Digest 6468, 912963; and as to the duty of a judge in relation to an innuendo, see 24 Halsburys Laws (3rd Edn)
108, para 199.
As to justification of a libel, see 24 Halsburys Laws (3rd Edn) 4348, paras 7585; and for cases on the subject, see 32
Digest 9199, 12191305.
As to when a new trial is granted on the ground of excessive damages in a libel action, see 24 Halsburys Laws (3rd Edn)
121, para 225.

Cases referred to in opinions


Barhams Case (1590), 1 Sav 121, 23 Digest (Repl) 74, 681.
British Transport Commission v Gourley [1955] 3 All ER 796, [1956] AC 185, [1956] 2 WLR 41, 220 LT 354, 3rd Digest Supp.
Capital and Counties Bank v Henty (George) & Sons (1880), 5 CPD 514, CA, affd (1882), 7 App Cas 741, 52 LJQB 232, 47 LT
662, 47 JP 214, HL, 32 Digest 21, 121.
Cookson v Harewood [1931] All ER Rep 533, [1932] 2 KB 478, n, 101 LJKB 328, 145 LT 1, Digest Supp.
152
English and Scottish Co-operative Properties Mortgage and Investment Society Ltd v Odhams Press Ltd [1940] 1 All ER 1,
[1940] 1 KB 440, 109 LJKB 273, 162 LT 82, 2nd Digest Supp.
Grubb v Bristol United Press Ltd [1962] 2 All ER 380, [1963] 1 QB 309, [1962] 3 WLR 25.
Harvey v French (1832), 1 Cr & M 11, 2 Moo & S 591, 1 LJEx 231, 32 Digest 61, 893.
Loughans v Odhams Press Ltd [1962] 1 All ER 404, [1963] 1 QB 299, [1962] 2 WLR 692.
Nevill v Fine Art and General Insurance Co [1897] AC 68, 66 LJQB 195, 75 LT 606, 61 JP 500, 32 Digest 72, 1009.
R v Horne (1777), 2 Cowp 672, 98 ER 1300, 32 Digest 197, 2458.
Sim v Stretch [1936] 2 All ER 1237, Digest Supp.
Simmons v Mitchell (1880), 6 App Cas 156, 50 LJPC 11, 43 LT 710, 45 JP 237, 32 Digest 49, 577.
Stubbs Ltd v Russell [1913] AC 386, 82 LJPC 98, 108 LT 529, 32 Digest 34, 292.
Turner (orse Robertson) v Metro-Goldwyn-Mayer Pictures Ltd [1950] 1 All ER 449, 2nd Digest Supp.
Watkin v Hall (1868), LR 3 QB 396, 9 B & S 279, 37 LJQB 125, 18 LT 561, 32 JP 485, 32 Digest 96, 1272.
Youssoupoff v Metro-Goldwyn-Mayer Pictures Ltd (1934), 50 TLR 581, Digest Supp.

Appeals
This was an appeal from an order of the Court of Appeal (Holroyd Pearce, Davies LJJ and Havers J) dated 4 April 1962, and
reported in [1962] 2 All ER 698, by which the verdicts given and the judgments entered on the trial of two consolidated actions
brought by the appellants separately against the respondents Daily Telegraph Ltd before Salmon J and a jury on 19 July 1961,
were wholly set aside and a new trial was ordered. This appeal was heard together with another appeal by the same appellants
from a like order of the same Court of Appeal, dated 4 April 1962, reported in [1962] 2 All ER 698, by which the verdicts given
and the judgments entered on the trial of consolidated actions brought by the appellants against the respondents, Associated
Newspapers Ltd before Salmon J and a jury on 21 July 1961, were wholly set aside and a new trial was ordered.
The words of the libels and the relevant terms of the statements of claim are set out at p 164, letters g to i, and p 157, letters
f, g and i, post.

H P J Milmo QC and Colin Duncan for the appellants.


Neville Faulks QC and H M Davidson for the respondents, Daily Telegraph Ltd.
Neville Faulks QC and David Hirst for the respondents, Associated Newspapers Ltd.

Their Lordships took time for consideration

26 March 1963. The following opinions were delivered.

LORD REID. My Lords, these are appeals in two actions for libel brought by the appellants, Mr Lewis and a company of which
he is managing director, against the proprietors of the Daily Telegraph and the Daily Mail in respect of paragraphs referring
to them which appeared on the front pages of those newspapers on 23 December 1958. I have had an opportunity of reading the
speeches about to be delivered by my noble and learned friends who deal fully with the facts and I shall not set out the passages
of which complaint is made. On 18 July 1961, a jury awarded damages against the Daily Telegraph of 25,000 to Mr Lewis
and 75,000 to his company. On the next day a different jury awarded against the Daily Mail 17,000 to Mr Lewis and
100,000 to his company. The Court of Appeal ordered new trials on several grounds of which the two most 153 important are
that the trial judge misdirected the juries and that the damages are so excessive that the awards cannot be allowed to stand. On
the matter of misdirection there is no material difference between the two cases.
The essence of the controversy between the parties is that the appellants maintain that these passages are capable of meaning
that they were guilty of fraud. The respondents deny this: they admit that the paragraphs are libellous but maintain that the juries
ought to have been directed that they are not capable of the meaning which the appellants attribute to them. The learned judge
directed the juries in such a way as to leave it open to them to accept the appellants contention and it is obvious from the
amounts of damages awarded that the juries must have done this.
The gist of the two paragraphs is that the police, the City Fraud Squad, were inquiring into the appellants affairs. There is
no doubt that in actions for libel the question is what the words would convey to the ordinary man: it is not one of construction in
the legal sense. The ordinary man does not live in an ivory tower and he is not inhibited by a knowledge of the rules of
construction. So he can and does read between the lines in the light of his general knowledge and experience of worldly affairs.
I leave aside questions of innuendo where the reader has some special knowledge which might lead him to attribute to the words
a meaning not apparent to those who do not have that knowledge. That only arises indirectly in this case. There has been much
argument about innuendoes, true or false, and about proper methods of pleading. My noble and learned friends intend to deal
with those matters and I shall not add to their explanations. I shall only make some observations on the footing that in this case
there is no question of innuendo in the true sense.
What the ordinary man would infer without special knowledge has generally been called the natural and ordinary meaning
of the words. But that expression is rather misleading in that it conceals the fact that there are two elements in it. Sometimes it is
not necessary to go beyond the words themselves as where the plaintiff has been called a thief or a murderer. But more often the
sting is not so much in the words themselves as in what the ordinary man will infer from them and that is also regarded as part of
their natural and ordinary meaning. Here there would be nothing libellous in saying that an inquiry into the appellants affairs
was proceeding: the inquiry might be by a statistician or other expert. The sting is in inferences drawn from the fact that it is the
fraud squad which is making the inquiry. What those inferences should be is ultimately a question for the jury but the trial judge
has an important duty to perform.
Generally the controversy is whether the words are capable of having a libellous meaning at all and undoubtedly it is the
judges duty to rule on that. I shall have to deal later with the test which he must apply. Here the controversy is in a different
form. The respondents admit that their words were libellous, although I am still in some doubt as to what is the admitted
libellous meaning. But they sought and seek a ruling that these words are not capable of having the particular meaning which the
appellants attribute to them. I think that they are entitled to such a ruling and that the test must be the same as that applied in
deciding whether words are capable of having any libellous meaning. I say that because it appears that when a particular
meaning has been pleaded either as a true or a false innuendo it has not been doubted that the judge must rule on the
innuendo. And the case surely cannot be different where a part of the natural and ordinary meaning is, and where it is not,
expressly pleaded. The leading case is Capital and Counties Bank v George Henty & Sons ((1882), 7 App Cas 741 at p 745). In
that case Lord Selborne LC said ((1882), 7 App Cas 741 at p 745):

The test according to the authorities is whether, under the circumstances in which the writing was published,
reasonable men to whom the publication was made would be likely to understand it in a libellous sense.
154

Each of the four noble Lords who formed the majority stated the test, in a different way, and the speeches of Lord Blackburn
((1882), 7 App Cas at p 771) and Lord Watson ((1882), 7 App Cas at p 778) could be read as imposing a heavier burden on the
plaintiff. But I do not think that they should now be so read. In Nevill v Fine Art and General Insurance Co, Lord Halsbury said
([1897] AC 68 at p 72):

What is the sense in which any ordinary reasonable man would understand the words of this communication so as to
expose the plaintiff to hatred or contempt or ridicule.
It is not enough to say that by some person or another the words might be understood in a defamatory sense [[1897] AC
at p 73].
These statements of the law appear to have been generally accepted and I would not attempt to restate the general principle.
In this case it is, I think, sufficient to put the test in this way. Ordinary men and women have different temperaments and
outlooks. Some are unusually suspicious and some are unusually nave. One must try to envisage people between these two
extremes and see what is the most damaging meaning that they would put on the words in question. So let me suppose a number
of ordinary people discussing one of these paragraphs which they had read in the newspaper. No doubt one of them might say
Oh, if the fraud squad are after these people you can take it they are guilty. But I would expect the others to turn on him, if
he did say that, with such remarks asBe fair. This is not a police state. No doubt their affairs are in a mess or the police
would not be interested. But that could be because Lewis or the cashier has been very stupid or careless. We really must not
jump to conclusions. The police are fair and know their job and we shall know soon enough if there is anything in it. Wait till we
see if they charge him. I wouldnt trust him until this is cleared up, but it is another thing to condemn him unheard.
What the ordinary man, not avid for scandal, would read into the words complained of must be a matter of impression. I can
only say that I do not think that he would infer guilt of fraud merely because an inquiry is on foot. And if that is so then it is the
duty of the trial judge to direct the jury that it is for them to determine the meaning of the paragraph but that they must not hold it
to impute guilt of fraud because as a matter of law the paragraph is not capable of having that meaning. So there was here, in my
opinion, misdirection of the two juries sufficiently serious to require that there must be new trials.
Before leaving this part of the case I must notice an argument to the effect that you can only justify a libel that the plaintiffs
have so conducted their affairs as to give rise to suspicion of fraud, or as to give rise to an inquiry whether there has been fraud,
by proving that they have acted fraudulently. Then it is said that, if that is so, there can be no difference between an allegation of
suspicious conduct and an allegation of guilt. To my mind there is a great difference between saying that a man has behaved in a
suspicious manner and saying that he is guilty of an offence and I am not convinced that you can only justify the former
statement by proving guilt. I can well understand that if you say there is a rumour that X is guilty you can only justify by proving
that he is guilty because repeating someone elses libellous statement is just as bad as making the statement directly. But I do not
think that it is necessary to reach a decision on this matter of justification in order to decide that these paragraphs can mean
suspicion but cannot be held to infer guilt.
Even if the paragraphs were capable of meaning that the appellants were guilty of fraud I would think that the damages
awarded were far too high, and a fortiori the awards could not stand if the most that could be read into the words is that they had
conducted their affairs in such a way as to give rise to suspicion or to justify a police inquiry. I do not say that these amounts of
damages could never 155 be justified but at least there would have to be evidence of a very different kind from that adduced in
these cases. I do not intend to analyse the evidence already given because that might hamper the conduct of the new trials with
regard to both the plaintiffs case and the defendants pleas of justification. But two particular matters raised in argument will
probably arise at the new trials and they require some clarification.
Here there were similar libels published in two national newspapers on the same day and each has to be dealt with by a
different jury. If each jury were to award damages without regard to the fact that the plaintiffs are also entitled to damages
against the other newspaper, the aggregate of the damages in the two actions would almost certainly be too large. Section 12 of
the Defamation Act, 1952, is intended to deal with that. In effect it requires that each jury shall be told about the other action, but
the question is what each jury should be told. I do not think that it is sufficient merely to tell each jury to make such allowance as
they may think fit. They ought, in my view, to be directed that in considering the evidence submitted to them they should
consider how far the damage suffered by the plaintiffs can reasonably be attributed solely to the libel with which they are
concerned and how far it ought to be regarded as the joint result of the two libels. If they think that some part of the damage is
the joint result of the two libels they should bear in mind that the plaintiffs ought not to be compensated twice for the same loss.
They can only deal with this matter on very broad lines and they must take it that the other jury will be given a similar direction.
They must do the best they can to ensure that the sum which they award will fully compensate the plaintiffs for the damage
caused by the libel with which they are concerned but will not take into account that part of the total damage suffered by the
plaintiffs which ought to enter into the other jurys assessment.
The other question arises out of the decision of this House in British Transport Commission v Gourley which deals with
damages for loss of income caused by a tort. In that case Mr Gourley had been seriously injured in a railway accident. He had
been earning a large income and it was found that his loss of income due to his injuries was 37,720. But this was the gross
income which he would have received but for his injuries. Out of it he would have had to pay income tax and surtax. And it was
found that after paying tax he would only have retained 6,695. So his real loss was only 6,695 because he could never have
derived any advantage from the balance which he would have had to pay away in tax. As damages are not subject to tax he
would have recovered far more than his real loss if he had recovered the gross amount of 37,720 and accordingly it was held
that he was only entitled to receive 6,695 in respect of his loss of income as this was sufficient to compensate him fully for the
income which he had lost by the fault of the defendants.
There can be no difference in principle between loss of income caused by negligence and loss of income caused by a libel.
Let me take first the case of the plaintiff company. A company cannot be injured in its feelings, it can only be injured in its
pocket. Its reputation can be injured by a libel, but that injury must sound in money. The injury need not necessarily be confined
to loss of income. Its goodwill may be injured. But in so far as the company establishes that the libel has, or has probably,
diminished its profits, I think that Gourleys case is relevant. Damages for libel have to be assessed by a jury, and juries are not
expected to make mathematical calculations, so they can only deal with this matter on broad lines. I think that a jury ought to be
directed to the effect that if they think that the plaintiff company has proved that it has suffered or will suffer loss of profit as a
result of the libel they must bear in mind that the company would have had to pay income tax at the standard rate out of that
profit if it had been earned and would only have been entitled to keep the balance. So in assessing damages they ought not to
take into account the whole of that 156 profit but should make allowance for the obligation to pay income tax out of it. The
position with regard to an individual plaintiff is rather different. He may be entitled to very substantial damages although his
income has not been affected by the libel. But if he does attempt to prove loss of income as a result of the libel then I think that a
similar direction must be given to the jury and it may be necessary to mention surtax as well as income tax.
Accordingly I shall move in each case that the appeal should be dismissed, that a new trial be ordered, that costs of the
abortive trial should abide the result of the new trial, and that the appellants be ordered to pay the respondents costs in the Court
of Appeal and in this House.
My noble and learned friend LORD JENKINS is unable to be present this morning and he desires me to say that he
concurs.

LORD MORRIS OF BORTH-Y-GEST. My Lords, on 23 December 1958, on the front page of the Daily Telegraph there
appeared the following words:

INQUIRY ON FIRM BY CITY POLICE

Daily Telegraph Reporter


Officers of the City of London Fraud Squad are inquiring into the affairs of Rubber Improvement, Ltd. and its
subsidiary companies. The investigation was requested after criticisms of the Chairmans statement and the accounts by a
shareholder at the recent company meeting.
The Chairman of the Company, which has an authorised capital of 1 million, is Mr. John Lewis, a former Socialist
M.P. for Bolton.

The company and Mr Lewis issued writs on that date and claimed damages for libel. The two actions were later consolidated.
Included in the statement of claim of the company, in addition to a paragraph pleading that the words had been falsely and
maliciously printed and published, was a paragraph which I consider, having regard to the fact that particulars under RSC, Ord
19, r 6(2) were set out, must be regarded as pleading an innuendo using that word in the strict or legal sense. The paragraph was
as follows:

By the said words the defendants meant and were understood to mean that the affairs of the plaintiffs and/or its
subsidiaries were conducted fraudulently or dishonestly or in such a way that the police suspected that their affairs were so
conducted.

Particulars pursuant to R.S.C., Ord. 19, r. 6(2)


(1) The plaintiffs repeat para. 1 hereof.
(2) The plaintiffs will rely on the tone and heading of the said article.
(3) It is generally known that the City Fraud Squad investigate serious cases of company fraud.

Paragraph 1 of the statement of claim had stated that the company was a public company carrying on a large and extensive
business both on its own account and through subsidiaries mainly in plastics and rubber industries. The provisions of RSC, Ord
19, r 6(2) require a plaintiff who alleges that words have been used in a defamatory sense other than their ordinary meaning to
give particulars of the facts and matters on which he relies in support of that sense.
In the statement of claim of the personal plaintiff there was a comparable paragraph. Apart from its supporting particulars it
was as follows:

By the said words the defendants meant and were understood to mean that the plaintiff had been guilty or was
suspected by the police of having been guilty of fraud or dishonesty in connexion with the affairs of the said company
and/or its subsidiaries and/or that he had caused or permitted the affairs of the said company and/or its subsidiaries to be
conducted fraudulently or dishonestly or in such a way that the police suspected that the affairs of the said company and/or
its subsidiaries had been so conducted and/or that the plaintiff was unfit to hold either of his said offices.
157

In the defences of the Daily Telegraph there were two paragraphs (in addition to alternative paragraphs containing pleas in
mitigation of damages) of special importance. In one there was a plea that the words in their natural and ordinary meaning
were true in substance and in fact. In the other there was a plea that the words do not bear and were not understood to bear and
were incapable of bearing the meanings attributed to them in the paragraphs in the statements of claim which I have set out. It
was admitted that the personal plaintiff was the chairman and managing director of the company and that the company carried on
a large and extensive business. The plaintiffs made a request for particulars of the justification which had been pleaded. This
matter was dealt with before the master in chambers, then on appeal to the judge in chambers and then on appeal in the Court of
Appeal. The master made the limited order that particulars should be given of the request for an investigation stating by whom
such request was made and when and the nature of the investigation requested. The learned judge allowed an appeal from that
order which he varied by ordering particulars of the facts and matters relied on in support of the allegation that the words in their
natural and ordinary meaning were true in substance and in fact. It is agreed that in the Court of Appeal learned counsel for the
Daily Telegraph made it clear that justification was only being asserted in regard to the limited fact that officers of the City of
London Fraud Squad had (after being requested as stated) inquired into the affairs of the company and its subsidiary companies.
On that being made clear the Court of Appeal set aside the order of the learned judge. Accordingly the limited order made by the
master was restored.
Before your Lordships it was common ground that the fact that certain meanings were alleged by way of innuendo did not
debar the plaintiffs from contending that such meanings were in fact the direct or ordinary or primary meanings of the words.
After the evidence was concluded at the trial a submission was made in the absence of the jury that the innuendoes should in
each case be withdrawn from the jury. There is no note as to the terms of the submission that was made, but it was the
recollection of learned counsel who made it that it was not limited to a submission that the two paragraphs should be withdrawn
because the particulars did not support the innuendo or were not proved, but that it extended to a submission that apart from any
question of an innuendo the words themselves in their ordinary meaning were incapable of bearing the meanings set out by the
plaintiffs in those paragraphs. The conclusion of the matter can best be recorded by setting out what the learned judge actually
said and what learned counsel said:

SALMON, J. [See [1962] 2 All ER 698 at 700]: I am inclined to think that no innuendo here was necessary. I can
well understand, however, that where there is any doubt about the matter the learned pleader very properly puts in an
innuendo. As counsel for the defendants candidly admits, that cannot do the defendants any harm, because it forewarns
them as to what the plaintiff is going to submit the words mean. Where there is no innuendo, it may in some cases be
difficult for the defendant to know. Even although the innuendo may strictly be unnecessary, I do not think that in the
exercise of my discretion I ought to strike it out; nor need it be amended. I can only say that the practice was at one time
always to plead an innuendo. The practice has altered, fortunately, so that innuendoes now are rarely pleaded, but there
may be casesand I agree with Mr. Duncan that this is one of themwhere there may be a doubt as to whether it is
necessary to plead an innuendo. Where there is such a doubt, there can be no harm in pleading it. It certainly cannot hurt
the defendants. I do not accept the argument that by pleading an innuendo you are necessarily alleging affirmatively that
the words in their ordinary and natural meaning mean something other than that which is 158 pleaded in the innuendo. I do
not propose to make any order on the application of the defendants.

The learned judge then said, addressing counsel for the defendants and counsel for the plaintiffs:

I propose only to leave one question to the jury: do you find for the plaintiff or for the defendant, and, if for the
plaintiff, how much? I will hear either of you if you wish to urge me to leave a series of questions to the jury.
Counsel for the defendants: No, my Lord, I am quite in accord with what your Lordship says.
Counsel for the plaintiffs: My Lord, so am I.

It is clearly settled that an innuendo constitutes a cause of action separate from the libel itself and in respect of which a separate
verdict should be returned and separate damages (if to be awarded) should be assessed; (see Sim v Stretch and Watkin v Hall).
Unless the court otherwise permits, any payment into court referable to an innuendo must be a separate payment.
The words of the learned judge show that he fully appreciated the difficulty that faced the pleader and that he considered that
the pleading of innuendoes had really been unnecessary. That was because the innuendoes did not go beyond the meanings that
the plaintiffs said were conveyed by the words of the libel. The effect of what the learned judge did was that the case proceeded
on the footing that the paragraphs should be treated as being no more than paragraphs which recorded what the plaintiffs
submitted were the ordinary meanings of the words. The paragraphs were however in form and must be regarded as being in fact
paragraphs which pleaded innuendoes. That being so if the learned judge took the view that no extrinsic facts were proved which
could support an innuendo he should, I think, have said in direct terms that he was not leaving any innuendo in its true sense to
the jury and instead of refusing the application should have, at least to some extent, acceded to it. If he was prepared to allow the
paragraphs to remain in some form he should perhaps have required that they be amended so that they were no longer paragraphs
which pleaded innuendoes. But the case went on just as though he had done that. The paragraphs were treated as though they
did not contain innuendoes in a true sense. They were regarded as being of the style of paragraphs which, in pleadings before the
introduction of RSC, Ord 19, r 6(2), used the word innuendo in a more general way and not in its strict or technical sense. So
no harm to the respondents resulted from the ruling of the learned judge. The case continued in spite of the actual language of his
ruling, just as the respondents suggested that it should proceed. No innuendo (using that word in its strict sense) was in fact left
to the jury. The summing-up directed the jury to consider what the words themselves meant and conveyed. Had the learned
judge left an innuendo to the jury, he would have had to require the jury to deal with it as a separate issue. His words show that
he regarded the paragraphs as harmless paragraphs, which had not hurt the defendants, but had perhaps helped them by
forewarning them as to what the plaintiffs said that the words meant. He did, however, consider that the words complained of
were of and by themselves capable of bearing the meanings alleged by the plaintiffs, and he left it to the jury to say what they
thought that the words meant. Here I think (apart from the issues concerning the damages) is the real issue in the case. Was the
learned judge right in deciding (as he implicitly did) that the words were capable of bearing all the alleged meanings? Once the
learned judge had reached that conclusion then any question as to innuendoes seems to me to have dropped out of the case.
Where a plaintiff brings an action for libel he may sustain his case (where there is a trial with a jury) if the judge rules that
the words, in what has been called their natural and ordinary meaning (or their ordinary meaningsee RSC, 159Ord 19, r
6(2)) are capable of being defamatory and if the jury find that they are defamatory. A plaintiff may, however, sustain his case in a
different way. He may plead an innuendo. He may establish that because there were extrinsic facts which were known to readers
of the words, such readers would be reasonably induced to understand the words in a defamatory sense which went beyond or
which altered their natural and ordinary meaning and which could be regarded as a secondary or as an extended meaning. The
nature of an innuendo (using that word in its correct legal sense) has recently been reviewed in the valuable judgments delivered
in the Court of Appeal in Grubb v Bristol United Press Ltd. A defamatory meaning which derives no support from extrinsic facts,
but which is said to be implied from the words which are used, is not a true innuendo. If there are some special extrinsic facts the
result may be that to those who know them words may convey a meaning which the words taken by themselves do not convey.
In the present case I am disposed to agree with the Court of Appeal that no extrinsic facts were proved which yielded the
necessary support to sustain an innuendo. This, however, became a matter of no consequence in the case for if the meanings
alleged in the pleaded innuendo were no more than the meanings expressed or conveyed by or to be implied from the words
themselves, then there was no need to plead innuendoes. It was not really being alleged that the words were used in a defamatory
sense other than their ordinary meaning. Though the two paras 4 were undoubtedly pleaded as innuendoes, once it was clear that
the contention of the plaintiffs was that the words themselves would be understood by ordinary readers to be conveying and
expressing the meanings recorded in those paragraphs, then the case for the plaintiffs was direct and straightforward and was not
in any way advanced or assisted by any mention of an innuendo. Though the paragraphs were not struck out the only
significance of their remaining was that they usefully contained and recorded the ordinary meanings which the plaintiffs said
were conveyed by the words printed in the newspaper. It followed that it was quite unnecessary for the learned judge to tell the
jury what an innuendo was or even to use the word (and he did not) or to leave any issue or question to them concerning an
innuendo. The question left to the jurywithin the limits of the meanings which the learned judge regarded the words as being
capable of bearingwas what they thought ordinary people would consider that the words meant.
In a case where there is no innuendo pleaded, it is not essential for a plaintiff to record and define in his pleadings what he
says are the ordinary or direct or natural or implied meanings of the words. If, however, he does do so (as may often be helpful
provided it is made clear what is being done) and, if the judge considers that the words are not capable of bearing any one or
more of such meanings, he ought so to rule. If the plaintiff does not do so the various meanings suggested by the plaintiff will
almost invariably be canvassed during the trial and if the judge considers that the words are not capable of bearing any one or
more of them again he ought so to rule.
It is of some importance to consider how the issues in the case rested. Publication of the words complained of was admitted.
The separate plaintiffs (the company and Mr Lewis) claimed and the defendants denied that in their natural and ordinary meaning
the words meant what was set out in the two paragraphs. It was not denied by the defendants and it was therefore tacitly admitted
that the words in their natural and ordinary meaning were defamatory of the plaintiffs but the defendants contention was that in
their natural and ordinary meanings the words only meant that there was an inquiry by the City of London Fraud Squad. The
defendants plea of justification was accordingly and for that reason limited to that meaning which was the only defamatory
meaning that they said that the words bore. They said that there had been an inquiry. The plaintiffs said that there had not been
anything that could be called an inquiry or that the 160 defendants had not proved that there had been such an inquiry. The
defendants pleaded in mitigation of damages that on the 24 December 1958 they published a statement by Mr Lewis expressing
his view of the facts and they also pleaded that the plaintiffs had claimed damages for the same or a similar libel from Associated
Newspapers Ltd. If the learned judge was correct in holding that the words were capable of bearing the meanings that the affairs
of the company or its subsidiaries were conducted fraudulently or dishonestly or that the personal plaintiff had been guilty of
fraud or dishonesty in connexion with the affairs of the company or its subsidiaries, then I see no grounds for criticism of his
summing-up on the issues of liability. He invited the jury to decide what they thought ordinary reasonable people would consider
the words to mean. Having regard to the guidance given by Lord Selborne LC in his speech in Capital and Counties Bank v
George Henty & Sons that was, I think, an entirely correct approach. Lord Selborne said ((1882), 7 App Cas at p 745):

The test according to the authorities is whether under the circumstances in which the writing was published reasonable
men to whom the publication was made would be likely to understand it in a libellous sense.

See also the words of Lord Blackburn ((1882), 7 App Cas at p 772).
My lords, words are but the instruments which men use to express and convey their meanings. The learned judge asked the
jury to say what meanings the words in question would convey, not to people with some special or particular knowledge, but just
to ordinary men and women going about their ordinary affairs. It is in this sense that in defamation cases the phrase natural and
ordinary meaning (which may include an implied or indirect meaning) is used. Not resting on any technical process of analysis
or construction, nor on a process of critical reading, the inquiry is as to what meanings are conveyed to hearers or readers by the
medium of words. This is a matter for the jury though a jury must not be asked to consider a meaning which the words in
question are not reasonably capable of bearing.
It was said in the Court of Appeal that the learned judge had failed to remember that the defendants were admitting that the
words, in what they said was their only natural meaning, were defamatory. I do not think that there is any substance in this for
the learned judge asked the jury whether, if the words bore the very limited meaning contended for by the newspaper, they
considered that the words were justifiable as being true: the necessity to consider the defence of justification would only arise on
the basis that the words were defamatory and there is no reason to think that the judge was either under a misapprehension or that
he need have said more to the jury than he did. He put very fairly before the jury the rival contentions as to what the words
meant. We do not know exactly what the jury decided that the words meant because with the assent of both sides only the one
question set out above was left to them. As it is important to see how the matter was put to the jury, I venture to quote the words
of the learned judge (See [1962] 2 All ER 698 at p 702):

This case very largely depends on what in your view those words mean. The question is, what would they have meant
to the ordinary man and woman when he or she read them on the morning of Dec. 23? It has been said that the ordinary
man, with his cup of tea in one hand, reading this paper, does not read it with a suspicious tortuous and sinister mind. That
is very true, you may think.
On the other hand, it has been said that when the ordinary man spreads his paper out on the table and reads it with his
cup of tea in one hand, he does not necessarily hold the scales of justice in delicate equipoise in the other. You have got to
think of the ordinary man. How would the ordinary man understand this?
161
The two views which have been canvassed before you are these: [Counsel for the defendants] has said: Well, the
ordinary man is not very suspicious; he would just regard it as a piece of intelligence, the police are looking into it, and it
would not really produce any other effect on his mind. [Counsel for the plaintiffs] says: Well, the ordinary man seeing this
City Police. Officers of the City of London Fraud Squad are inquiring into the affairs of Rubber Improvement, Ltd.the
ordinary man, not being any more suspicious than his neighbour, would immediately say to himself, says [counsel for the
plaintiffs]either he would say to himself: There is a fraud here, or the police would not be looking into it; or, he would
say to himself: At any rate, there is enough in this for the police to suspect that there is fraud.
I cannot really help you. Those are the two rival contentions. It is for you to say what it means. When you read the
newspapers, what would you have thought when you read that? You see, the only way you can get at what the ordinary
man and woman think is by getting a jury of twelve people together, who are ordinary men and women, and asking them
what they would have thought. You may ask yourselves, what would people in the City think if they woke up one morning
and read that in the paper?
Members of the jury, anything is defamatory which tends to lower you in the esteem of right thinking people. But if
anyone reading this thoughtany ordinary reasonable man reading this thoughtthat it meant that Mr. Lewis had been
guilty of fraud, or that the police suspected that Mr. Lewis had been guilty of fraud; or that he had allowed the affairs of the
company to be conducted fraudulently or dishonestly; or the police suspected that he had, would that tend to lower him in
the esteem of right thinking people?
And as far as the company is concerned, it is suggested by the plaintiffs that these words mean to the ordinary man that
the affairs of the company and its subsidiaries were conducted fraudulently or dishonestly; or that the police suspected that
they were so conducted. That is what is said by the plaintiffs these words would convey to the ordinary man and woman,
and that the ordinary man and woman would not merely say to themselves: Oh well, it is a very interesting piece of
intelligence: the police are inquiring into it. There may be a routine examination. We do not draw any conclusions at all.
As I say, consider that you get up one morning in a perfectly reasonable frame of mind; you are not feeling suspicious
particularly, but you have a look at that: what would it mean to you?

The learned judge then went on to deal with the issue of justification and to direct the jury that if they thought that the meaning
conveyed was no more than that the police were making an inquiry, then they had to consider whether the defendants had proved
that an inquiry had been made. The learned judge reminded the jury of the evidence and said:Does that constitute an inquiry
into these matters by the police? You may think it does: you may think it does not.
My lords, I turn to consider the question whether the words were capable of bearing the meaning that the affairs of the
company and/or its subsidiaries were conducted fraudulently or dishonestly. I do not understand any of your lordships to be of
the view that the words were not capable of bearing the meaning that the police suspected that the affairs of the company or its
subsidiaries were conducted fraudulently or dishonestly: nor did I understand any submission to be made that the words were not
so capable. It is a grave thing to say that someone is fraudulent. It is a different thing to say that someone is suspected of being
fraudulent. How much less wounding and damaging this would be must be a matter of opinion depending on the circumstances.
Similarly in the case of the personal plaintiff the submission is made that the words while capable of bearing 162 some of the
alleged meanings were not capable of bearing the meanings that Mr Lewis had been guilty of fraud or dishonesty in connexion
with the affairs of the company or its subsidiaries or had caused or permitted the affairs to be conducted fraudulently or
dishonestly.
My lords, the only question that now arises is not whether the words did bear, but whether they were capable of bearing, the
meanings to which I have referred. What could ordinary reasonable readers think? Some I consider might reasonably take the
view that there was just an inquiry to find out whether or not there had been any fraud or dishonesty. Some I consider might
reasonably take the view that the words meant that there was an inquiry because there had been fraud or dishonesty which
occasioned or required inquiry by the police. Some I consider might reasonably take the view that the words meant that the
inquiry was either (a) because there had been fraud or dishonesty or (b) because of a suspicion that there had been.
My lords, it is not for me to say what I think was the meaning which the words conveyed to the ordinary reasonable reader
of a newspaper nor is it for me to express any opinion as to what conclusion a jury should reach as to this matter but I do not
consider that that meaning which involved that there had been fraud or dishonesty was a meaning which the jury should have
been prohibited from considering on the basis that it was a meaning of which the words were not capable. I do not think that it
can be said that twelve jurors could not reasonably have come to the conclusion that the words bore the meaning now being
considered. In using this language I am following the approach suggested by Lord Porter in his speech in Turner v Metro-
Goldwyn-Mayer Pictures Ltd ([1950] 1 All ER 449 at pp 452, 453). See also Nevill v Fine Art and General Insurance Co.
My lords, a reasonable reader will probably be a fair-minded reader. The fair-minded reader would assume that a
responsible newspaper would also be fair. If there was some private police inquiry in progress, the purpose of which was to
ascertain whether or not there had been fraud or dishonesty what possible justification could there be for proclaiming this far and
wide to all the readers of a newspaper? If confidential information was received to the effect that there was a police inquiry on
what basis could the publishing of such information be warranted? There is no suggestion that the police had asked that any
notice should be published. Under certain circumstances a newspaper may enjoy qualified privilege if it publishes a notice issued
for the information of the public by or on behalf of a chief officer of police (see Defamation Act, 1952, s 7). If there was a police
inquiry by a Fraud Squad which might result in the conclusion that any suspicion of fraud or dishonesty was wholly
unwarranted how manifestly unfair it would be to make public mention of the inquiry. What purpose could there be in doing so?
With these thoughts and questions in his mind, a reasonable reader might well consider that no responsible newspaper would dare
to publish, or would be so cruel as to publish, the words in question unless the confidential information, which in some manner
they had obtained, was not information merely to the effect that there was some kind of inquiry in progress but was information
to the effect that there was fraud or dishonesty. Some reasonable readers might therefore think that the words conveyed the
meaning that there must have been fraud or dishonesty. Furthermore, a reasonable reader might reflect that while the police may
be concerned with inquiries as to whether some crime has or has not been committed, they are probably more often only
concerned after a crime has been committed. They have to inquire whether they possess the necessary evidence for the launching
of a prosecution. Reasonable readers might also think that inquiries into the affairs of a company if such inquiries were not
concerned with fraud or dishonesty would not be conducted by the police at all. They would be conducted by persons 163 or
departments having no connexion with the City of London Police Fraud Squad. Some of such readers might therefore be led to
believe that if there was an inquiry by the City of London Fraud Squad, which a newspaper felt justified in mentioning, it must
have been an inquiry to collect and marshall evidence in order to launch a prosecution for some offences involving fraud or
dishonesty which had been committed.
My lords, it was for the jury to determine what they considered was the meaning that the words would convey to ordinary
men and women: we have only to decide as to the limits of the range of meanings of which the words were capable. For the
reasons that I have given I have the misfortune to differ from your lordships as to this very important part of the case. I consider
that the learned judge was fully entitled to leave the matter to the jury in the way in which he did and I consider that his directions
concerning liability were clear and correct and fair.
My lords, in the consolidated action against the Daily Mail a similar issue arises to that which I have been discussing.
On the difficult issue as to damages I do not differ from your lordships or from the Court of Appeal that the awards of
damages were excessive and cannot stand. This issue was fully debated and the relevant evidence was carefully examined. As
there must be new trials I do not think that there is need to say more in regard to this matter. I would wish to add that having had
the privilege of reading in advance the speech which has been delivered by my noble and learned friend, Lord Reid, I am in
agreement with his observations in regard to s 12 of the Defamation Ac,t 1952, and in regard to the extent of any relevance of the
case of British Transport Commission v Gourley.
Because I do not dissent on the issue as to damages I agree that there must be new trials and that therefore the appeals
should be dismissed.

LORD HODSON. My Lords, In these actions large damages were awarded to the plaintiff Mr Lewis and to the company
Rubber Improvement Ltd of which the first plaintiff is the managing director. In the first pair of actions, which were consolidated
with one another, Daily Telegraph Ltd was the defendant, in the second pair, likewise consolidated, Associated Newspapers Ltd
proprietors of the Daily Mail, were defendants. The pairs of actions were tried separately no step being taken to have them
consolidated although the language of the libels is similar and each defendant is a newspaper having a wide national circulation.
The words complained of in the first actions are as follows:

INQUIRY ON FIRM BY CITY POLICE

Daily Telegraph Reporter.


Officers of the City of London Fraud Squad are inquiring into the affairs of Rubber Improvement, Ltd. and its
subsidiary companies. The investigation was requested after criticisms of the chairmans statement and their accounts by a
shareholder at the recent company meeting.
The chairman of the company, which has an authorised capital of 1 million, is Mr. John Lewis, former Socialist M.P.
for Bolton.

In the second action the words were

FRAUD SQUAD PROBE FIRM


The City Fraud Squad, under Superintendent Francis Lea, are inquiring into the affairs of Rubber Improvement, Ltd.
Chairman of the 4,000,000 group, whose shares have dropped from 22s. last year to 7s. 4 1/2d. yesterday, is Mr. John
Lewis, former Socialist M.P.
The company specialises in flexible rubber conveyor belting designed for the National Coal Board.

Both articles appeared on the front page of the respective newspapers on 23 December 1958.
164
The facts leading up to and surrounding the publication differed to some extent in respect of the two publications but the
general effect was the same. In the Daily Telegraph actions the plaintiff Lewis obtained 25,000 damages and the plaintiff
company 75,000 and in the Daily Mail actions he obtained 17,000 and the company 100,000. The Court of Appeal ordered
a new trial holding that in any event the damages were so excessive that no reasonable jury could have awarded so large a figure
and that there was a misdirection on the part of the trial judge in respect of the meaning of the libels.
The defendants did not deny that the words complained of were defamatory of the plaintiffs. They justified the words as
true in their natural and ordinary meaning and denied that they bore any of the meanings which they were said to bear by
innuendo in effect that the plaintiffs were guilty or suspected by the police of fraud or dishonesty in connexion with the affairs of
the company or its subsidiaries.
The appellants recognise that in awarding such large damages on each trial the juries must have taken the view that the
words of which they complain meant that they had been actually guilty of fraud, a meaning which the defendants have
throughout disclaimed.
No one doubts that it is for the jury to decide the meaning of words, not as a question of pure construction but as a question
of fact, as Lord Tenterden CJ put it in Harvey v French ((1832), 1 Cr & M 11 at p 18),

a court must read these words in the sense in which ordinary persons or in which we ourselves out of court would
understand them.

Whether the words are capable of defamatory meaning is for the judge, and where the words, whether on the face of them they
are or are not innocent in themselves, bear a defamatory or more defamatory meaning because of extraneous facts known to those
to whom the libel has been published, it is the duty of the judge to rule whether there is evidence of such extraneous facts fit to be
left to the jury.
It is in conjunction with secondary meanings that much of the difficulty surrounding the law of libel exists. These secondary
meanings are covered by the word innuendo which signifies pointing out what and who is meant by the words complained of.
Who is meant raises no problem here but what is meant is of necessity divided into two parts much discussed in this case. Libels
are of infinite variety and the literal meaning of the words even of such simple phrases as X is a thief does not carry one very
far for they may have been spoken in play or other circumstances showing that they could not be taken by reasonable persons as
imputing an accusation of theft. Conversely to say that a man is a good advertiser only becomes capable of a defamatory
meaning if coupled with proof, for example, that he was a professional man whose reputation would suffer if such were believed
of him.
The first subdivision of the innuendo has lately been called the false innuendo as it is no more than an elaboration or
embroidering of the words used without proof of extraneous facts. The true innuendo is that which depends on extraneous facts
which the plaintiff has to prove in order to give the words the secondary meaning of which he complains.
The classic example is to be found in Barhams case referred to by De Grey LCJ, in R v Horne ((1777), 2 Cowp 672 at p
684) in the following passage relating to a charge of criminal libel:
But as an innuendo is only used as a word of explanation it cannot extend the sense of the expressions in the libel
beyond their own meaning unless something is put upon the record for it to explain. As in actions upon the case against a
man for saying of another: He has burnt my barn. [Barhams case] the plaintiff cannot there by way of innuendo say 165
meaning his barn full of corn, because that is not an explanation of what was said before but an addition to it. But if in
the introduction it had been averred that the defendant had a barn full of corn and that in a discussion about the barn the
defendant had spoken the words charged in the libel of the plaintiff; an innuendo of its being the barn full of corn would
have been good: for by coupling the innuendo in the libel with the introductory averment, his barn full of corn, would
have made it complete.

The innuendo in this case was set out in the statement of claim as a separate paragraph and was or purported to be a true
innuendo, for the plaintiff gave particulars pursuant to RSC, Ord 19, r 6(2), of the facts and matters which he relied on in support
of a sense other than the ordinary meaning. It is plain on the authorities that, since the Common Law Procedure Act, 1852 a,
which did away with the necessity for a prefatory averment showing the sense in which words were used and enacted that where
the words or matter set forth with or without the alleged meaning show a cause of action the declaration shall be sufficient the
true innuendo has been treated as a separate cause of action from that which arose from the words in their natural and ordinary
meaning (with or without inferential meanings commonly called false innuendoes): see per Blackburn J, in Watkin v Hall
((1868), LR 3 QB 396 at p 402) and per Lord Atkin in Sim v Stretch ([1936] 2 All ER 1237 at p 1239).
________________________________________
a See ibid, s 61; 18 Halsburys Statutes (2nd Edn) 388

After the passing of the Common Law Procedure Act, 1852, until the year 1949, when RSC, Ord 19, r 6(2) came into force,
there was in many cases no distinction to be found between the true and the false innuendo. No special facts had to be pleaded to
support the innuendo and the distinction became blurred between the true innuendo and that which was very often nothing but a
wordy explanation or attempted explanation of the words complained of in their natural and ordinary meaning. This blurring is
manifest in the pleadings of the plaintiff here, for it contains the plaintiffs contention as to the natural and ordinary meaning of
the words complained of, that is to say, that they mean that he and his company were fraudulent or suspected of fraud, and it is at
the same time supported by particulars given under RSC, Ord 19, r 6(2) as of a true innuendo. As the Court of Appeal found, and
I have no doubt that they were right, the particulars did not show, nor was any evidence given of, extraneous facts in support of
the innuendo, and accordingly the innuendo should not have been left to the jury. Although the innuendo was not pleaded in the
alternative, yet as the Court of Appeal held, in my opinion quite rightly, this did not prevent the plaintiffs seeking to show, if they
could, that the natural and ordinary meaning of the words complained of was to the same effect. To hold otherwise and not to
permit the jury to impute to the ordinary meaning of the words any part of the failed innuendo would, as Holroyd Pearce LJ
([1962] 2 All ER 698 at pp 709, 710; [1963] 1 QB 340 at p 368) pointed out, have the effect of removing the jurys decision on
whether the words are in their ordinary sense a libel into an unreal technical and artificial sphere.
I agree with the observations of Holroyd Pearce LJ in Grubb v Bristol United Press Ltd ([1962] 2 All ER 380 at p 389;
[1963] 1 QB 309 at p 326) to the effect that RSC, Ord 19, r 6(2) makes no alteration in the law except in cases where a true
innuendo is pleaded. A pleader is entitled to allege in his statement of claim what the words in their natural and ordinary meaning
convey, provided he makes it clear that he is not relying on a true innuendo, which gives a separate cause of action and requires a
separate verdict from the jury. It is desirable that he should do so, for, where there is no true innuendo, the judge should define
the limits of the natural and ordinary meaning of the libel and leave to the jury only those meanings which he rules are capable of
being defamatory. If the natural and ordinary meaning is pleaded the defence will know what the contentions of the plaintiff are,
and the judge will not have to analyse the submissions of counsel in his charge to the jury 166 without having the benefit of a
pleading setting out what those submissions are.
There is no conflict in my opinion between the decisions of the Court of Appeal in Grubb v Bristol United Press Ltd and in
Loughans v Odhams Press Ltd when properly understood, as indeed was pointed out by Upjohn LJ ([1962] 2 All ER at p 393;
[1963] 1 QB at p 331) in the former case. The difficulty arises from some words (perhaps unguarded) used by Diplock LJ in
Loughans case ([1962] 1 All ER at p 409; [1963] 1 QB at 306) and cited by Davies LJ in Grubbs case ([1962] 2 All ER at p 396;
[1963] 1 QB at p 338), to the effect that the plaintiff may require a verdict from a jury whether the words bear a special
defamatory meaning, even though he had led no evidence of facts and matters on which he relies other than the words themselves
as giving rise to the meaning alleged in the innuendo. This, I agree with Davies LJ, he cannot do. There is one cause of action
based on the words in their natural and ordinary meaning and another based on the words in such meaning as may be alleged in a
true innuendo but not a third cause of action based on the false innuendo.
The defendants, having admitted that the words are defamatory in their ordinary meaning, have always maintained that their
ordinary meaning does not go so far as to include actual guilt of fraud. They have sought to justify by proving that an inquiry
was in fact held, not by proving actual suspicion of fraud. This is the gist of the whole case. Salmon J, who tried both pairs of
actions, took the view that the words were capable of imputing guilt of fraud. Davies LJ ([1962] 2 All ER at p 725; [1963] 1 QB
at p 394) was inclined to the same opinion, and my noble and learned friend Lord Morris of Borth-y-Gest has expressed the same
opinion as Salmon J, Holroyd Pearce LJ and Havers J took the contrary view. In view of this difference of judicial opinion one
naturally hesitates before expressing a concluded opinion of ones own, but after listening to many days of argument I am myself
satisfied that the words cannot reasonably be understood to impute guilt. Suspicion no doubt can be inferred from the fact of the
inquiry being held, if such was the case, but to take the further step and infer guilt is in my view wholly unreasonable. This is to
draw an inference from an inference and to take two substantial steps at the same time.
The distinction between suspicion and guilt is illustrated by the case of Simmons v Mitchell which decided that spoken
words which convey a mere suspicion that the plaintiff has committed a crime punishable by imprisonment will not support an
action without proof of special damage. It has been argued before your lordships that suspicion cannot be justified without proof
of actual guilt on the analogy of the rumour cases such as Watkins v Hall. Rumour and suspicion do, however, essentially differ
from one another. To say that something is rumoured to be the fact is, if the words are defamatory, a republication of the libel.
One cannot defend an action for libel by saying that one has been told the libel by someone else, for this might be only to make
the libel worse. The principle, as stated by Blackburn J, in Watkins v Hall ((1868), LR 3 QB at p 401), is that a party is not the
less entitled to recover damages from a court of law for injurious matter published concerning him because another person
previously published it. It is wholly different with suspicion. It may be defamatory to say that someone is suspected of an
offence, but it does not carry with it that that person has committed the offence, for this must surely offend against the ideas of
justice, which reasonable persons are supposed to entertain. If one repeats a rumour one adds ones own authority to it, and
implies that it is well founded, that is to say, that it is true. It is otherwise when one says or implies that a person is under
suspicion of guilt. 167This does not imply that he is in fact guilty, but only that there are reasonable grounds for suspicion,
which is a different matter.
Having reached the conclusion that the innuendo should not have been left to the jury as a separate issue and that the natural
and ordinary meaning of the words does not convey actual guilt of fraud, I agree with the Court of Appeal that there must be a
new trial, for the learned judge left the question to the jury Did they find for plaintiffs or defendants? without a direction that
the words were incapable of the extreme meaning which I have rejected.
I would not, but for this misdirection as to the meaning of the words, have thought that a new trial should be ordered simply
because the innuendo was wrongly left to the jury, for no harm would have been done if there had been no misdirection as to the
meaning of the words. The vital misdirection was as to the meaning which the plaintiffs sought to ascribe to the words. As to
this, in a Scottish case Stubbs Ltd v Russell ([1913] AC 386 at p 393), Lord Kinnear said:

The law is perfectly well settled. Before a question of libel or slander is submitted to a jury the court must be satisfied
that the words complained of are capable of the defamatory meaning ascribed to them. That is a matter of law for the
court.

This is also the law of England; cf English and Scottish Co-operative Properties Mortgage and Investment Society Ltd v Odhams
Press Ltd, a case where there was a long paragraph of innuendoes suggesting various meanings to be attributed to the words
complained of but no true innuendo supported by extrinsic facts. All the innuendo meanings were left to the jury and both
Slesser LJ ([1940] 1 All ER at p 7; [1940] 1 KB at p 453), and Goddard LJ ([1940] 1 All ER at p 12; [1940] 1 KB at p 460),
referred to the duty of the judge to withdraw meanings from the jury if the words are incapable of bearing such meanings. I have
mentioned this last point because at one stage of the argument it seemed that it might be contended that, once the judge had ruled
the words were capable of any as opposed to the defamatory meaning ascribed to them, the jury were masters of the
situation, but the contention I have adumbrated was not advanced before your lordships and was expressly disclaimed by counsel
in the course of the hearing before the Court of Appeal.
The responsibility of the judge to exclude a particular meaning which the plaintiff seeks to ascribe to words in their natural
or ordinary meaning is, I think, clearly established by the decision of this House in Capital and Counties Bank v George Henty &
Sons. Henty & Sons had sent out a circular to a number of their customers giving notice that they would not receive in payment
cheques drawn on any of the vouchers of the bank. There was no evidence to support the innuendo that the words imputed
insolvency to the bank, and it was held that in their natural and ordinary meaning the words were not libellous. Lord Blackburn
said ((1882), 7 App Cas at p 776):

Since Foxs Act [32 Geo 3 c 60] at least, however the law may have been before, the prosecutor or plaintiff must also
satisfy a jury that the words are such and so published as to convey the libellous imputation. If the defendant can get either
the court or the jury to be in his favour he succeeds. The prosecutor, or plaintiff, cannot succeed unless he gets both the
court and the jury to decide for him.

Since in my judgment there must be a new trial in order that the jury in each pair of cases may be directed as to the natural
and ordinary meaning of the words published in the two newspapers, I need say nothing on the question of damages except that I
agree with the Court of Appeal that the damages were in each case so excessive that they cannot be allowed to stand. I also agree
that, as a result 168 of the decision of your Lordships House in British Transport Commission v Gourley, the jury in each case so
far as the plaintiff company is concerned should have been directed that, since a company can only suffer in its pocket by loss of
revenue attributable to a libel, so regard must be had to the fact that the profits of the company will in large measure be passed on
to the revenue and not retained for the benefit of the shareholders.
I am further of opinion that a direction should be given to the jury as to the effect of s 12 of the Defamation Act, 1952,
which enables other claims by the plaintiffs to be disclosed to the jury with the object of preventing compensation being given
twice over for the same libel, so that the jury should be directed to apply themselves to the injury inflicted in the particular case.
I would dismiss the appeal.

LORD DEVLIN. My Lords, the natural and ordinary meaning of words ought in theory to be the same for the lawyer as for the
layman because the lawyers first rule of construction is that words are to be given their natural and ordinary meaning as
popularly understood. The proposition that ordinary words are the same for the lawyer as for the layman is, as a matter of pure
construction, undoubtedly true. But it is very difficult to draw the line between pure construction and implication, and the
laymans capacity for implication is much greater than the lawyers. The lawyers rule is that the implication must be necessary
as well as reasonable. The layman reads in an implication much more freely; and, unfortunately, as the law of defamation has to
take into account, is especially prone to do so when it is derogatory.
In the law of defamation these wider sorts of implication are called innuendoes. The word explains itself and is very apt for
the purpose. In R v Horne ((1777) 2 Cowp at p 484) De Grey LCJ, said:

In the case of a libel which does not in itself contain the crime, without some extrinsic aid, it is necessary that it should
be put upon the record, by way of introduction, if it is new matter; or by way of innuendo, if it is only matter of
explanation. For an innuendo means nothing more than the words, id est, scilicet, or meaning, or aforesaid, as,
explanatory of a subject-matter sufficiently expressed before;

An innuendo had to be pleaded and the line between an ordinary meaning and an innuendo might not always be easy to draw. A
derogatory implication may be so near the surface that it is hardly hidden at all or it may be more difficult to detect. If it is said
of a man that he is a fornicator the statement cannot be enlarged by innuendo. If it is said of him that he was seen going into a
brothel, the same meaning would probably be conveyed to nine men out of ten. But the lawyer might say that in the latter case a
derogatory meaning was not a necessary one because a man might go to a brothel for an innocent purpose. An innuendo pleading
that the words were understood to mean that he went there for an immoral purpose would not therefore be ridiculous. To be on
the safe side, a pleader used an innuendo whenever the defamation was not absolutely explicit. That was very frequent since
scandalmongers are induced by the penalties for defamation to veil their meaning to some extent. Moreover, there were some
pleaders who got to think that a statement of claim was somehow made more forceful by an innuendo, however plain the words.
So rhetorical innuendoes were pleaded, such as to say of a man that he was a fornicator meant and was understood to mean that
he was not fit to associate with his wife and family and was a man who ought to be shunned by all decent persons and so forth.
Your lordships were told, and I have no doubt it is true, that before 1949 it was very rare indeed to find a statement of claim in
defamation without an innuendo paragraph.
I have said that a derogatory implication might be easy or difficult to detect; and of course it might not be detected at all
except by a person who was already in possession of some specific information. Thus, to say of a man that he was 169 seen to
enter a named house would contain a derogatory implication for anyone who knew that that house was a brothel but not for
anyone who did not. In the passage which I have quoted De Grey LCJ ((1777), 2 Cowp at p 484), distinguished between this sort
of implication and the implication that is to be derived from the words themselves without extrinsic aid, and he treats the term
innuendo as descriptive only of the latter. Since then the term has come to be used for both sorts of implication. Either sort
had to be put upon the record, as the chief justice said, and extrinsic facts had to be pleaded by way of introduction, as the
chief justice said, or as a prefatory averment, as it came to be called. The Common Law Procedure Act, 1852, s 61 did away
with the necessity of pleading the prefatory averment, while leaving it necessary to plead the innuendo: the section provided that

the plaintiff should be at liberty to aver that the words or matter complained of were used in a defamatory sense,
specifying such defamatory sense without any prefatory averment.

My lords, a system of pleading was built up on this basis which in 1949 was disconcerted by the introduction of a new rule
RSC, Ord 19, r 6(2). The object of the rule was to require that extrinsic facts must not only be proved but also be pleaded, thus
restoring the position before 1852. The object was simple enough. It is the language of the rule that has caused the difficulties
which have recently been brought to a head and have been the subject of three decisions, including the present one, by the Court
of Appeal. The rule reads:

(2) In an action for libel or slander if the plaintiff alleges that the words or matter complained of were used in a
defamatory sense other than their ordinary meaning, he shall give particulars of the facts and matters on which he relies in
support of such sense.

The word innuendo is not used. But the effect of the language is that any meaning that does not require the support of extrinsic
fact is assumed to be part of the ordinary meaning of the words. Accordingly, an innuendo, however well concealed, that is
capable of being detected in the language used is deemed to be part of the ordinary meaning.
This might be an academic matter if it were not for the principle that the ordinary meaning of words and the meaning
enlarged by innuendo give rise to separate causes of action. This principle, which originated out of the old forms of pleading,
seems to me in modern times to be of dubious value. But it is now firmly settled on the authority of Sim v Stretch, and the House
was not asked to qualify it. How is this principle affected by the new rule? Are there now three causes of action? If there are
only two, to which of them does the innuendo that is inherent in the words belong? In Grubb v Bristol United Press Ltd the
Court of Appeal, disagreeing with some observations made by Diplock LJ in Loughans v Odhams Press Ltd ([1962] 1 All ER at p
409; [1963] 1 QB at p 306), decided in effect that there were only two causes of action and that the innuendo cause of action
comprised only the innuendo that was supported by extrinsic facts.
My lords, I think on the whole that this is the better solution though it brings with it a consequence that I dislike, namely,
that at two points there is a divergence between the popular and the legal meaning of words. Just as the popular and legal
meanings of malice have drifted apart, so the popular and legal meanings of innuendo must now be separated. I shall in the
rest of my speech describe as a legal innuendo the innuendo that is the subject-matter of a separate cause of action. I suppose that
it does not matter what terminology is used so long as it is agreed. But I do not care for the description of the popular innuendo
as a false innuendo; it is the law and not popular usage that gives a false and restricted meaning to the word. The other respect is
that the natural and ordinary 170 meaning of words for the purposes of defamation is not their natural and ordinary meaning for
other purposes of the law. There must be added to the implications which a court is prepared to make as a matter of construction
all such insinuations and innuendoes as could reasonably be read into them by the ordinary man.
The consequence of all this is, I think, that there will have to be three paragraphs in a statement of claim where previously
two have served. In the first paragraph the defamatory words will be set out as hitherto. It may be that they will speak for
themselves. If not, a second paragraph will set out those innuendoes or indirect meanings that go beyond the literal meaning of
the words but which the pleader claims to be inherent in them. Thirdly, if the pleader has the necessary material, he can plead a
secondary meaning or legal innuendo supported by particulars under RSC, Ord 19, r 6(2). Hitherto it has been customary to put
the whole innuendo into one paragraph, but now this may easily result in the confusion of two causes of action and in consequent
embarrassment. The essential distinction between the second and third paragraph will lie in the fact that particulars under the rule
must be appended to the third. That is, so to speak, the hallmark of the legal innuendo. The pleader can if he chooses emphasise
the character of the second paragraph by including in it some such words as were used in Loughans v Odhams Press Ltd. That
case was, in my opinion, rightly decided and rightly distinguished from Grubb v Bristol United Press Ltd by Upjohn LJ ([1962] 2
All ER at p 393; [1963] 1 QB at p 331), in the latter case. Or the pleader can, as was suggested by Holroyd Pearce LJ in Grubbs
case ([1962] 2 All ER at p 391; [1963] 1 QB at p 329), plead in the second paragraph that the words in their natural and ordinary
meaning were understood to mean one thing; and then he could plead in the third paragraph that, by reason of the facts
thereinafter particularised, they were understood to mean another. The meanings alleged in the third paragraph can be the same
as those alleged in the second paragraph if the pleader is relying on the legal innuendo only as an alternative; or they can be
different. But the essential thing is that, if a paragraph is unaccompanied by particulars, it cannot be a legal innuendo, since for a
legal innuendo particulars are mandatory and the innuendo cannot be proved without them.
It was suggested in argument that the division of the innuendo into two paragraphs would be awkward for the pleader. It is
said that it may not always be easy to decide whether an extrinsic factor relied on is a matter of special knowledge, or whether it
is just general knowledge in the light of which the ordinary though indirect meaning of the words has to be ascertained. I do not
think that this should present any difficulty in practice. The pleader must ask himself whether he contemplates that evidence will
be called in support of the allegation: if he does, it is a legal innuendo and if he does not, it is not. If he is in doubt, he can plead
in two paragraphs; and then if at the trial his opponent agrees or the judge rules that it is a matter of general knowledge, the legal
innuendo can be dropped.
It was also suggested to your lordships that the pleading of a middle paragraph was unnecessary and even improper and your
lordships were told that, since 1949, some judges have discouraged the pleading of all innuendoes that are not legal innuendoes.
I should certainly like to see what I have called rhetorical innuendoes discouraged; but I am satisfied that the pleading of an
innuendo in every case where the defamatory meaning is not quite explicit is at the least highly desirable, and I am glad to
observe that in Loughans case the attempt to strike out the innuendo failed. An attempt of this sort is no doubt inspired by the
thought that it is unnecessary to plead the ordinary meaning of words and that that is all that the popular innuendo is. I think that
that thought is fallacious. It does not take into account the difference 171 which I have pointed out between the meaning of
words in the law of defamation and their meaning for the general purposes of the law. In general the meaning of words is a
matter of law and therefore need not be pleaded, though, where there is a difficult question of construction in issue, it is usual and
convenient to do so. But in defamation the meaning of words is a question of fact, that is, there is libel or no libel according to
the impression the words convey to the jury and not according to the construction put on them by the judge. I do not mean that
ingenuity should be expended in devising and setting out different shades of meaning. Distinct meanings are what should be
pleaded: and a reasonable test of distinctness would be whether the justification would be substantially different. In the present
case, for example, there could have been three distinct categories of justificationproof of the fact of an inquiry, proof of
reasonable grounds for it and proof of guilt. If no innuendo had been pleaded and there had been full proof of grounds for
inquiry, I cannot think that in a closing speech the plaintiff could without any previous notice invite the jury to say that the words
meant guilt and to reject the justification as insufficient. Moreover, where distinct meanings are possible and the judge is invited
to rule separately on one or more, it is desirable that the meanings put to the jury should be on the record. But that touches on a
point of substance which I shall consider later.
I understand your lordships all to be of the opinion that the pleading of the ordinary or popular innuendo is permissible, but
do not intend that the House should rule on whether it is necessary. I agree that the point does not arise directly in this case, and,
therefore, I too shall reserve my judgment on it. But I make the comment that, if it is not necessary, it is nevertheless a form of
pleading universally used from the earliest times until 1949, and I can see nothing in the new rule that should alter so well
established a practice.
My lords, I have made a very long preliminary to the consideration of the pleading point in this case. Your lordships were
invited from the Bar to deal in detail with all the difficulties of pleading involved in that point and that have recently come to the
fore in other cases and I have thought it right to do so. I must now state how in the light of what I have said generally I should
decide the point at issue. Paragraph 4 of the statement of claim is as follows:
4. By the said words the defendants meant and were understood to mean that the affairs of the plaintiffs and/or its
subsidiaries were conducted fraudulently or dishonestly or in such a way that the police suspected that their affairs were so
conducted.

The Court of Appeal considered this paragraph to be defective and I agree with them. This does not involve any sort of criticism
of the learned pleader who drafted his statement of claim at a time when it was possible to take almost any view of the points I
have been canvassing. It is plain now that para 4 must be treated as in form a plea of a legal innuendo. But in substance it is not
a legal innuendo because no extrinsic facts are pleaded: general knowledge is, as I have indicated already, not an extrinsic fact
for the purpose of RSC, Ord 19, r 6(2), but is matter, not requiring to be proved, in the light of which the jury can interpret the
publication. In substance the paragraph is a plea of a popular innuendo, and the confusion between substance and form makes it
embarrassing. But I cannot with respect agree with the Court of Appeal that the way in which the judge treated this point is by
itself a ground for a new trial. He went by the substance of the paragraph and left it to the jury as an ordinary innuendo, not a
legal one. Perhaps he ought to have insisted on an amendment in the form, but he stated the course that he was going to take and
neither counsel offered any objection to it. I cannot think that the jury could have been in any way misled. There has been some
discussion about whether the plaintiffs will have to amend before proceeding to a new trial. That is for them to say. In the light
of all this discussion they would perhaps be wise not to take indulgence for granted.
172
I turn now to the main ground for ordering a new trial. This was that the judge misdirected the jury by failing to tell them
that the words were not capable of bearing one or more of the defamatory meanings alleged in para 4 of the statement of claim. It
is admitted that the words are capable of some defamatory meaning, and I think that it is undoubtedly defamatory of a company
to say that its affairs are being inquired into by the police. But para 4 alleges that the words meant

that the affairs of the plaintiffs and/or its subsidiaries were conducted fraudulently or dishonestly or in such a way that
the police suspected that their affairs were so conducted.

This is saying that the words mean either that the plaintiffs were guilty of fraud or that they were suspected of fraud. If it is
permissible to distinguish between these two meanings, then for reasons which I shall give as I proceed I should hold that the
words are capable of the latter meaning but not of the former, and I should on this basis agree with the Court of Appeal that the
jury should have been so directed and that, since they were not, there should be a new trial. But counsel for the appellants has
submitted that it is not right so to distinguish.
In the first place, he relies on what are called the rumour cases. I agree, of course, that one cannot escape liability for
defamation by putting the libel behind a prefix such as I have been told that or It is rumoured that , and then asserting
that it was true that one had been told or that it was in fact being rumoured. You have, as Horridge J said, in a passage that was
quoted with approval by Greer LJ, in Cookson v Harewood ([1931] All ER Rep 533 at p 536; [1932] 2 KB 478 at p 485), to
prove that the subject-matter of the rumour was true. But this is not a case of repetition or rumour. I agree with the distinction
drawn by Horridge J on this point, though not necessarily with his limited view of the effect of the libel in that case. Anyway,
even if this is to be treated as a rumour case, it is still necessary to find out what the rumour is. A rumour that a man is suspected
of fraud is different from one that he is guilty of it. For the purpose of the law of libel a hearsay statement is the same as a direct
statement, and that is all there is to it.
The real point, I think, that counsel for the appellants makes is that whether the libel is looked at as a statement or as a
rumour, there is no difference between saying that a man is suspected of fraud and saying that he is guilty of it. It is undoubtedly
defamatory, he submits, to say of a man that he is suspected of fraud, but it is defamatory only because it suggests that he is guilty
of fraud: so there is no distinction between the two. This is to me an attractive way of putting the point. On analysis I think that
the reason for its attraction is that as a maxim for practical application, though not as a proposition of law, it is about three-
quarters true. When an imputation is made in a general way, the ordinary man is not likely to distinguish between hints and
allegations, suspicion and guilt. It is the broad effect that counts and it is no use submitting to a judge that he ought to dissect the
statement before he submits it to the jury. But if, on the other hand, the distinction clearly emerges from the words used, it cannot
be ignored. If it is said of a manI do not believe that he is guilty of fraud but I cannot deny that he has given grounds for
suspicion, it seems to me to be wrong to say that in no circumstances can they be justified except by the speaker proving the
truth of that which he has expressly said that he did not believe. It must depend on whether the impression conveyed by the
speaker is one of frankness or one of insinuation. Equally in my opinion it is wrong to say that, if in truth the person spoken of
never gave any cause for suspicion at all, he has no remedy because he was expressly exonerated of fraud. A mans reputation
can suffer if it can truly be said of him that although innocent he behaved in a suspicious way; but it will suffer much more if it is
said that he is not innocent.
It is not therefore correct to say as a matter of law that a statement of suspicion imputes guilt. It can be said as a matter of
practice that it very often does so, 173because although suspicion of guilt is something different from proof of guilt, it is the
broad impression conveyed by the libel that has to be considered and not the meaning of each word under analysis. A man who
wants to talk at large about smoke may have to pick his words very carefully, if he wants to exclude the suggestion that there is
also a fire; but it can be done. One always gets back to the fundamental question: what is the meaning that the words convey to
the ordinary man; a rule cannot be made about that. They can convey a meaning of suspicion short of guilt; but loose talk about
suspicion can very easily convey the impression that it is a suspicion that is well founded.
In the libel that the House has to consider there is, however, no mention of suspicion at all. What is said is simply that the
plaintiffs affairs are being inquired into. That is defamatory, as is admitted, because a mans reputation may in fact be injured by
such a statement even though it is quite consistent with innocence. I daresay that it would not be injured if everybody bore in
mind, as they ought to, that no man is guilty until he is proved so, but unfortunately they do not. It can be defamatory without it
being necessary to suggest that the words contained a hidden allegation that there were good grounds for inquiry. A statement
that a woman has been raped can affect her reputation, although logically it means that she is innocent of any impurity:
Youssoupoff v Metro-Goldwyn-Mayer Pictures Ltd. So a statement that a man has been acquitted of a crime with which in fact he
was never charged might lower his reputation. Logic is not the test. But a statement that an inquiry is on foot may go further and
may positively convey the impression that there are grounds for the inquiry, ie, that there is something to suspect. Just as a bare
statement of suspicion may convey the impression that there are grounds for belief in guilt, so a bare statement of the fact of an
inquiry may convey the impression that there are grounds for suspicion. I do not say that in this case it does; but I think that the
words in their context and in the circumstances of publication are capable of conveying that impression. But can they convey an
impression of guilt? Let it be supposed, first, that a statement that there is an inquiry conveys an impression of suspicion; and,
secondly, that a statement of suspicion conveys an impression of guilt. It does not follow from these two suppositions that a
statement that there is an inquiry conveys an impression of guilt. For that, two fences have to be taken instead of one. While, as
I have said, I am prepared to accept that the jury could take the first I do not think that in a case like the present, where there is
only the bare statement that a police inquiry is being made, it could take the second in the same stride. If the ordinary sensible
man was capable of thinking that wherever there was a police inquiry there was guilt, it would be almost impossible to give
accurate information about anything: but in my opinion he is not. I agree with the view of the Court of Appeal.
There is on this branch of the case a final point to be considered. It is undoubtedly the law that the judge should not leave
the question libel or no libel to the jury unless the words are reasonably capable of a defamatory meaning. But if several
defamatory meanings are pleaded or suggested, can the judge direct the jury that the words are capable of one meaning but not of
another? The point is important here, because the defendants admit that the words are defamatory in one sense but dispute that
they are defamatory in the senses pleaded in the statements of claim, and contend that the judge should have so directed the jury.
Both counsel for the appellants appear at one time to have argued in the Court of Appeal that the function of the judge was
exhausted when he ruled that the words were capable of being defamatory; and that it was not for him to inquire whether they
were or were not capable of any particular defamatory meaning. But later they abandoned the point; and, therefore, did not
initiate the discussion of it here. Nevertheless there was considerable discussion of it, because some of your lordships at one time
felt that it was a point which ought to be considered. In the result I think that all your lordships are 174 now clearly of the
opinion that the judge must rule whether the words are capable of bearing each of the defamatory meanings, if there be more than
one, put forward by the plaintiff. This supports indirectly my view on the desirability of pleading different meanings. If the
plaintiff can get before the jury only those meanings which the judge rules as capable of being defamatory, there is good reason
for having the meanings alleged set out precisely as part of the record.
For the reasons that I have given earlier, I agree that there must be a new trial on the ground of misdirection: but I should in
any event have considered that there should be a new trial on the issue of damages as they are, in my opinion, ridiculously out of
proportion to the injury suffered.

Appeals dismissed.

Solicitors: Zeffertt, Heard & Morley Lawson (for the appellants); Swepstone, Walsh & Son (for the respondents, Associated
Newspapers Ltd); Simmons & Simmons (for the respondents, Daily Telegraph Ltd).

C G Leonard Esq Barrister.


[1963] 2 All ER 175

Britt v Buckinghamshire County Council


TOWN AND COUNTRY PLANNING

COURT OF APPEAL
SELLERS, HARMAN AND PEARSON LJJ
6, 7, 8 FEBRUARY 1963

Town and Country Planning Amenity Notice to abate injury to amenity by condition of vacant site or open land Use of land
in agricultural area as car dump Established use since before 1947, but no express planning permission granted Validity of
notice Town and Country Planning Act, 1947 (10 & 11 Geo 6 c 51), s 33(1).

The power conferred by s 33a of the Town and Country Planning Act, 1947, to require the abatement (without compensation) of a
condition of a vacant site or open land b that is injurious to the amenity of an area extends to requiring the abatement by a
landowner of an active use, or a condition brought about by such use, as well as to requiring the abatement of a condition not
attributable to any active use by the landowner; and the power is exercisable, unless planning permission for the use has been
granted, notwithstanding that the use is an established use existing before 1 July 1948, and thus is outside the control of other
provisions of Part 3 of the Act of 1947, and that the exercise of that other control (if it had been applicable) would have entitled
the landowner to compensation (see p 178, letters b, g and h, p 179, letters g and h, p 180, letter g, and p 182, letter c, post).
________________________________________
a Section 33(1), (2) are printed at pp 176, 177, post
b The statutory power extends in terms to any garden, vacant site or other open land, but the word garden is not descriptive of the land
concerned in the present case

Statute Construction Statutory instrument Use of regulations for interpretation of section of statute Regulations modifying
statutory provisions rendered applicable by the section Town and Country Planning Act, 1947 (10 & 11 Geo 6 c 51), s 33(2).

Section 33(2) of the Town and Country Planning Act, 1947, conferred power to modify by regulations certain provisions of the
Act made applicable for the purposes of s 33(1). Modifications were prescribed by the Town and Country Planning (General)
Regulations, 1948, reg 7, which took effect contemporaneously with the coming into operation of s 33.

Held In construing s 33 regard might be had to reg 7 of the regulations of 1948 (see p 177, letter f, p 180, letter a, and p 182,
letter i, post).
Dictum of Upjohn LJ in Stephens v Cuckfield Rural District Council ([1960] 2 All ER at p 718) distinguished.
Observations on inaccuracy of marginal note to s 33 as rendering it of no assistance on interpretation (see p 183, letters b
and c, and p 178, letter c, post).
Appeal dismissed.
175

Notes
By the Caravan Sites and Control of Development Act, 1960, ss 48, 50(4) and Sch 4, as from 29 August 1960, the words in the
manner prescribed by regulations under this Act in s 33(1), and s 33(2), of the Town and Country Planning Act, 1947, have been
repealed. Regulation 7 of the Town and Country Planning (General) Regulations, 1948 (SI 1948 No 1380), has been revoked by
the Town and Country Planning (General) (Amendment) Regulations, 1960 (SI 1960 No 1475). Provisions as to appeal against a
notice served under s 33(1) of the Act of 1947 are now contained in Sch 3, paras 812, to the Act of 1960(40 Halsburys Statutes
(2nd Edn) 1110, 1111). The Town and Country Planning Act, 1947, s 23(3) (in part), (4) and (5), and s 24 (in part), have also
been repealed as from 29 August 1960, by the Act of 1960.
As to power to require abatement of injury to amenity by condition of garden, vacant site or other open land, see Halsburys
Laws (3rd Edn) 399, para 506.
For the Town and Country Planning Act, 1947, s 33, see 25 Halsburys Statutes (2nd Edn) 541.

Case referred to in judgments


Stephens v Cuckfield Rural District Council [1960] 2 All ER 716, [1960] 2 QB 373, 124 JP 420, [1960] 3 WLR 248, 3rd Digest
Supp, affg, [1959] 1 All ER 635, [1959] 1 QB 516, [1959] 2 WLR 480.

Appeal
This was an appeal by the plaintiff landowner and a cross-appeal by the defendant local planning authority from an order of
Widgery J, dated 4 July 1962. Since before 1947, Robert George Graham Britt, the plaintiff landowner, had used certain land
which he owned and occupied in an otherwise agricultural area at Prestwood, Buckinghamshire, as a car-dump, this making the
land an eyesore. On 1 September 1958, the defendant local planning authority, Buckinghamshire County Council, served on the
landowner, inter alia, a notice under s 33 of the Town and Country Planning Act, 1947, requiring him to cease this use of the land.
The landowner brought this action against the local planning authority for a declaration, inter alia, that the notice was invalid.
The local planning authority counter-claimed for a declaration that the notice was valid. Widgery J, declared that the notice was
not invalid, and dismissed the landowners claim.

V G Wellings for the landowner.


Desmond Ackner QC and David Trustram Eve for the local planning authority.

8 February 1963. The following judgments were delivered.

SELLERS LJ. As far back as 1958, if not earlier, the local planning authority, the defendant county council, in the performance
of their public duty and in the interests of the community, decided to exercise the powers which they thought that they had under
the Town and Country Planning Act, 1947. In 1963 the validity of the various steps which they took is still being called in
question before this court. I will allow that bare statement of fact to stand without comment. In the course of this appeal happily
some of the matters that have been in contention have been abandoned, and this court is called on to decide one short point only
under one section of the Act of 1947. It relates to a notice given by the local planning authority on 1 September 1958, in respect
of a piece of land fronting on the easterly side of Hampden Road, Prestwood, Buckinghamshire, which was, and is, owned and
occupied by the plaintiff landowner.
Section 33 of the Town and Country Planning Act, 1947, is as follows:

(1) If it appears to a local planning authority that the amenity of any part of the area of that authority, or of any
adjoining area, is seriously injured by the condition of any garden, vacant site or other open land in their area, then, subject
to any directions given by the minister, the 176 authority may serve on the owner and occupier of the land, in the manner
prescribed by regulations under this Act, a notice requiring such steps for abating the injury as may be specified in the
notice to be taken within such period as may be so specified. (2) In relation to any notice served under this section, the
provisions of sub-ss. (3) to (5) of s. 23 of this Act, and of s. 24 of this Act shall, subject to such exceptions and
modifications as may be prescribed by regulations under this Act, apply as those provisions apply in relation to an
enforcement notice served under s. 23.

No direction limiting the scope of sub-s (1) has been given by the minister as could have been done if it was desired to curtail its
effect. Under powers given by the Act of 1947, a regulation was made modifying s 33(2); namely, reg 7 of the Town and Country
Planning (General) Regulations, 1948. Regulation 7(1) provides for the manner of the service of a notice under s 33(1), and para
(2) reads as follows:

The following exceptions and modifications shall be made in sub-ss. (3) to (5) of s. 23 and s. 24 of the Act in their
application to any such notice as is referred to in para. (1) of this regulation, namely: (a) in sub-s. (3) of s. 23, para. (a) of
the proviso shall be omitted; (b) for heads (a) and (b) of sub-s. (4) of s. 23, there shall be substituted the following heads:
(a) if satisfied that the condition of the land to which the notice relates is such as results in the ordinary course of events
from operations or a use for which permission was granted under this Part of this Act, shall quash the notice to which the
appeal relates; (b) if not so satisfied, but satisfied that the requirements of the notice exceed what is necessary for abating
the injury, shall vary the notice accordingly.

I have restricted my citation to those provisions and have not read the whole regulation, because it is only that portion which
becomes relevant in the task before this court of construing the scope of s 33. The regulations came into force in 1948 at the
same time as the Town and Country Planning Act, 1947, and, having regard to the terms of s 33(2), the regulations must, I think,
be read in conjunction with that section, however unusual it may be to insert modifications in this way. It is not, I think, using the
regulations quite in the manner criticised by Upjohn LJ, in his judgment in Stephens v Cuckfield Rural District Council ([1960] 2
All ER 716 at p 718; [1960] 2 QB 373 at p 380).
Section 33 is within Part 3 of the Act of 1947, which is headed Control of Development, etc. Permission to develop land.
Section 12 commences this Part, and a number of sections sets out the circumstances in which permission can be given and the
method by which the control is to be exercised. Section 23 provides for the enforcement of planning control, and, by sub-s (4),
any person on whom an enforcement notice is served may, within a specified period, appeal against the notice to a court of
summary jurisdiction, and reg 7, to which I have referred, adapts s 23 to the circumstances of s 33. Then, prior to s 26, there is
inserted a heading Additional powers of control, under which falls s 33. On the face of it, the power to abate serious injury to
the amenity of an area would appear to be additional to rights to prevent user on some other ground.
The contention which was advanced before us was that which was argued also before Widgery J (from whom this appeal
comes), that is to say, that s 33 only applies

to a case where the injury to amenity results from the condition of the land as opposed to the use to which land is put,
and that, where the injury is a necessary result of the particular use to which the land is put, the planning authority must
take steps to determine that use under the earlier provisions of the Act (paying compensation in appropriate cases) and
cannot proceed under s. 33.
177

What was submitted was that regard could be had only to what one of the learned counsel described as the inactive condition and
not to the active condition brought about by the use. In construing this matter, I think that the regulation, modifying as it does the
section of the Act and being read with it, makes it clear that that contention cannot prevail and that the learned judge was right in
the conclusion to which he camealthough he himself did indicate that, apart from authority, he might have been inclined to take
a different view. Quite apart from that additional assistance, the words of s 33 read in their ordinary meaning do not limit the
word condition in any way. In its ordinary meaning, the condition would be the state in which it was, irrespective of how it
was brought about, the condition of any garden, vacant site or other open land in their area. If there were any doubt as to that, I
think that it is resolved in looking at the modification brought about by the regulation, which contemplates that operations or use
may seriously injure amenities. Attention was drawn to the sidenote, Power to require proper maintenance of waste land, etc.
This is a case, I think, which demonstrates how unreliable it would be, even if permissible, to pay attention to the sidenote. It
seems to me to be inconsistent with the precise wording of the section itself, to which the word maintenance is inappropriate.
It was in accordance with s 33 that the notice was given to which I have earlier referred. The circumstance which gives rise
to the argument is that there is a small part of the land occupied by the landowner which is not subject to other provisions of the
Act of 1947. The local planning authority have relied on the additional powers of control in s 33 in respect of all the land,
including the small part otherwise immune, and in other respects and in respect of the other land they have relied on notices given
under s 23, dealing with the substantive control of the development of land. But it was said that the power ought not to be
exercised because it was, in effect, using s 33 to terminate the user of part of the land which had acquired rights which could not
be interfered with, and to do so without compensation. It appears that, before the passing of the Act of 1947, the landowner had
some years before (as the judgment of Widgery J shows) used this area of otherwise peaceful and attractive agricultural land in
the heart of Buckinghamshire to carry on a business of a somewhat strange character. He leaves there, apparently for a very long
time, a collection of old motor vehicles, damaged and beyond use, taking part of one to make up another, and so forth. There is
no question, on the facts of this case, as the local planning authority thought and as the judge also upheld, indeed as counsel for
the landowner had to admit, that the whole of this area is rightly described as an eyesore. It infringes the provisions of s 33 in
that it seriously injures the amenities of that locality. In those circumstances, the order was rightly made. No objection is made
to this particular notice save that it is not within the power of the local planning authority to make it because it would interfere
with an established right which was beyond the control of the other sections of the Act of 1947. It seems to me that that is not a
tenable argument. If there had been, on the facts of this case, a use established for which permission had been granted under this
Part of the Act, then that would have been an answer to the notice which was given, but without thatand it was not contended
that such a permission had been grantedI think that this case must fail, the learned judge came to the right conclusion, the
notice was good and full effect should be given to it.
There is a cross-notice asking that the declaration which the learned judge made should be altered in a minor form from a
declaration that the notice to which I have made reference was not invalid to an affirmative declaration that it was valid. That has
not been disputed by the learned counsel on the other side and I am of the opinion, with all respect to the meticulous refinement
of the learned judges view, that the order should be varied accordingly. Subject to that small variation, I would dismiss the
appeal.
178

HARMAN LJ. Hard indeed are the paths of local authorities in striving to administer the town and country planning legislation
of recent years. It is a sorry comment on the law and those who administer it that, between the years 1947 and 1960, they had
succeeded in so bedevilling the whole administration of that legislation that Parliament was compelled to come to the rescue and
remove a great portion of it from the purview of the courts. Not for nothing was I offered a book yesterday called
Encyclopaedia of Planning. It is a subject which stinks in the noses of the public, and not without reason. Local authorities,
until they have been recently rescued have had practically to employ conveyancing counsel to settle these notices which they
serve in the interests of planning the countryside or the towns which they control. Instead of their trying to make this thing
simpler, lawyers succeeded day by day in making it more difficult and less comprehensible, until it has reached a stage where it is
very much like the state of the land which the plaintiff landowner has brought about by his operationsan eyesore, a wilderness,
and a scandal. It may be hoped that this is one of the last of the actions of this sort where declarations are asked in respect of the
multitude of notices served by the poor bewildered local planning authority striving to do their best to cure an eyesore glaring in
the face of the countryside.
By the time the argument before us came to an end, the numerous points agitated before the judge below and, indeed, to
some extent opened to us had all faded away, and we were left with this one question under s 33 of the Town and Country
Planning Act, 1947. It has never seemed to me that there was any merit in the contention that, in some way or other, a limitation
had to be put on the words of this sectionwhich exactly fit, on the face of them, the state of things which was to be found here.
Some limitation (it was said) had to be put on them because the Control of development sections, which begin with s 23,
formed a kind of code, and as the earlier ones provided a scheme for preventing various users of land and for stopping abuses
which were not allowed by the local planning authority and giving compensation to those whose rights were taken away, one
must not, by having recourse to s 33, take a short cut, get away from all those and simply say: This is an eyesore: it is doing
serious injury to the amenity of the countryside: abate it. It is said that the man who is told to abate an eyesore ought to be
compensated for it. That seems to me to be a most astonishing doctrine. I can see no reason why the landowner, who has for
years made the country hideous by his goings-on, should not be made to put his house in order, and no reason at all why he
should be paid by the public for doing it. However, that was the plea that was urged before us, and to some extent it found favour
with the learned judge below, but I cannot think that his view was right. He did, in fact, bow to authority, Stephens v Cuckfield
Rural District Council, and decide that s 33 did fit the case here, and in that, in my judgment, he was right.
If help were wanted to aid the construction of s 33 and show that it means what I think that it saysnamely, that a use, as
well as a non-use, can be prohibited under that sectionthen I think it would be legitimate to look at the Town and Country
Planning (General) Regulations, 1948, which were made in 1948 and which came into existence at the same time as the Act itself.
My Lord has read them; I shall not re-read them; but it was said in Stephens v Cuckfield RDC (where Lord Parker CJ had applied
that principle), that the court there doubted whether that was a correct thing to do. What was said was this ([1960] 2 All ER at p
718; [1960] 2 QB at p 381):

We doubt very much whether it is right to construe the words of the section by reference to regulations made under
powers therein contained

If that were the real process, I should agree with that comment, but in this case 179 there is a process which I have not seen
anywhere else. A power is given to the minister by the Act itself to modify another section of the Act so that, when the minister
does produce that modification (as he did in the 1948 regulations), that regulation becomes in fact part of the Act. It is like an
amending section of the Act; so that, in my judgment, reference can be made to that regulation because it is embodied in the Act
itself and, having a quasi-parliamentary validity, is a good indication of the wishes of the legislature, just as much as if it were
enacted in the Act itself. I do not myself think that aid is necessary. I should have arrived at the same conclusion without it; but,
if it be necessary, is not a question of construing an Act by subsequent regulations, in the ordinary sense of those words.
I agree with what my Lord has said with regard to the cross-notice.
For these reasons, I think that the judge reached the right conclusion and that this appeal fails.

PEARSON LJ. In this appeal we are in the end concerned only with the notice referred to as notice H, which was served
under s 33 of the Town and Country Planning Act, 1947. [His Lordship referred to other notices, served under s 23 of the Act of
1947, with regard to which the plaintiff landowner had not pursued his appeal, and continued:] The contention, and the only
contention, argued in this appeal appears most conveniently in the learned judges judgment, where he said this, referring to the
argument of counsel for the landowner:

Counsel for the landowner does not contend that the facts of this case do not justify the allegation contained in notice
H, nor does he dispute that the land in question is a garden, vacant site or other open land within the meaning of the
section. He attacks notice H on the broad ground that s. 33 applied only to a case where the injury to amenity results from
the condition of the land as opposed to the use to which the land is put, and that, where the injury is a necessary result of
the particular use to which the land is put, the planning authority must take steps to determine that use under the earlier
provisions of the Act (paying compensation in appropriate cases) and cannot proceed under s. 33.

That is a contention as to the power conferred by s 33 of the Act of 1947, and it involves a consideration of the construction and
effect of s 33. Section 33 has already been read and I need not repeat it. I would entirely concur with what has already been said,
that, on the wording of that section, taken by itself without any extraneous aid to construction, quite plainly the words seriously
injured by the condition of any garden, vacant site or other open land in their area must refer to the actual condition, however it
may have been caused. Whether it results from inactivity or whether it results from some activity, it is, nevertheless, the
condition of the land. Therefore, the natural and ordinary meaning of the words is in favour of the decision which the learned
judge has given.
If I go into further questions of construction and extraneous aids, it is not with a view to introducing further complications
into the construction of this Act but merely with a view to showing that the natural and ordinary meaning of the wording in s 33
ought to be adhered to. In the first place, as counsel for the local planning authority has pointed out to us, there is no reason for
not giving s 33 its natural and full extent and operation, because it is a section designed to enable a local planning authority to
protect the amenity of part of its area. Moreover, certain safeguards are provided by the section itself. The power exists subject
to any directions given by the minister, and there are safeguards imported by sub-s (2) to which I am about to refer. Secondly, I
come to s 33(2), and I will consider it at the outset by itself without bringing 180 in the ministers regulation made under it,
because, in my view, s 33(2) by itself, even without any regulation, gives support to the natural and ordinary meaning of the
words in sub-s (1). The point is a little complicated, but I think that it is worth making. I will first of all read s 33(2):

In relation to any notice served under this section, the provisions of sub-ss. (3) to (5) of s. 23 of this Act, and of s. 24
of this Act shall, subject to such exceptions and modifications as may be prescribed by regulations under this Act, and of s.
24 of this Act shall, subject to such exceptions and modifications as may be prescribed by regulations under this Act, apply
as those provisions apply in relation to an enforcement notice served under s. 23.

It is to be observed that the subsections of s 23 which are to apply subject to any exceptions and modifications which the minister
may introduce include s 23(4). It is sub-s (4) which is material for the present purpose. Before going to that, I have to look back
to s 12(2), which defines the expression development as meaning

the carrying out of building, engineering, mining or other operations in, on, over or under land, or the making of any
material change in the use of any buildings or other land.

I have only read the principal part of the definition; the other parts are not material for the present purpose. Thus, the word
development includes operations and includes any material change in the use of the land. I then go to s 23(4), which, in
relation to the enforcement of planning control with respect to developments of land, contains this provision:

If any person on whom an enforcement notice is served under this section is aggrieved by the notice, he may, at any
time within the period mentioned in the last foregoing sub-section, appeal against the notice to a court of summary
jurisdiction for the petty sessional division or place within which the land to which the notice relates is situated; and on any
such appeal the court(a) if satisfied that permission was granted under this Part of this Act for the development to which
the notice relates, or that no such permission was required in respect thereof, or, as the case may be, that the conditions
subject to which such permission was granted have been complied with, shall quash the notice to which the appeal relates.

I need not read (b) and (c) of that provision. The application of s 23(4) of the Town and Country Planning Act, 1947, for the
purposes of s 33, albeit with modifications to be made by regulations, raises an inference that a notice under s 33 may affect a
permitted development and, therefore, may relate to a condition which is caused by operations taking place on the land or to a use
being made of the land. Therefore, even if we paused there and did not proceed to consider the regulations, in my view s 33(2) in
itself, when taken in conjunction with the other provisions to which I have referred, does afford some support for the learned
judges construction of s 33; but then, thirdly, we have to pass on to the peculiar position of regulations made by the minister for
the purposes of s 33(2). I need not read that subsection again, but I should refer to s 111(1) of the Act of 1947, because that
confers the power on the minister to make regulations, and then, by sub-s (2), it provides that regulations shall be laid before
Parliament, and if either House resolves that the regulations be annulled they are to be annulled accordingly. Section 120(2)
provides that the Act of 1947 shall come into force on the appointed day, and the appointed day is defined in s 119(1) as such
day as the minister may by order appoint.
Therefore, we have the position that regulations are required to be made under this Act. The relevant regulations are the
Town and Country Planning (General) Regulations, 1948 (SI 1948 No 1380). They are expressed to be made in exercise of the
power conferred on the minister by various sections of the Act of 1947, including s 33. Regulation 1 provides These regulations
shall come into force on the appointed day. Therefore, both the Act and the 181 regulations would come into force on the
same day. Then there is reg 7, which contains provisions under s 33. Regulation 7(2) provides that:

The following exceptions and modifications shall be made in sub-ss. (3) to (5) of s. 23 and s. 24 of the Act in their
application to any such notice as is referred to in para. (1) of this regulation.

That was a notice served under s 33. Then under head (b) of that paragraph, it is provided:

For heads (a) and (b) of sub-s. (4) of s. 23, there shall be substituted the following heads:(a) if satisfied that the
condition of the land to which the notice relates is such as results in the ordinary course of events from operations or a use
for which permission was granted under this Part of this Act, shall quash the notice to which the appeal relates; (b) if not so
satisfied, but satisfied that the requirements of the notice exceed what is necessary for abating the injury, shall vary the
notice accordingly.

If it is permissible to take that provision into account, it shows quite clearly and unmistakably that the conditions referred to in s
33(1) must include a condition which results from operations on the land or of use made of the land. The only problem is
whether it is permissible to take that provision of the regulations into account. There is a quite peculiar and exceptional position
arising here. I do not say that there may not be other cases, but at any rate I think that they must be cases which arise very rarely.
That which was to come into force and did come into force on the appointed day was a composite instrument consisting of the
provisions of the Act of 1947 as modified by the provisions of the regulations. The statutory force of this composite instrument is
derived from s 33(2), rather than from the regulations themselves. The regulations make their own provisions, but, in the end, it
is a composite instrument which has to be construed as such. Then it becomes quite clear from the terms of s 23(4), as modified
by reg 7(2)(a) and (b), that a notice under s 33 may refer to a condition which results from operations taking place on the land or
from a use made of the land.
I should refer to a passage in the judgment of this court in Stephens v Cuckfield Rural District Council ([1960] 2 All ER at p
718; [1960] 2 QB 373 at pp 380, 381). Upjohn LJ delivering the judgment of the court, said:

In the court below the learned Lord Chief Justice expressed the view that the prima facie meaning of open land in s.
33 covered operations on or user of any open land in the sense of land which was not built on [See [1959] 1 All ER at p
637; [1959] 1 QB at p 524]. He was to some extent influenced by the regulations made by the minister under s. 33(2),
which showed that in the view of the minister open land might include land on which some business was carried on. We
doubt very much whether it is right to construe the words of the section by reference to regulations made under powers
therein contained, especially when such regulations are directed solely to procedural matters such as conferring rights of
appeal in certain limited circumstances from an order of the local authority.

I would fully share that doubt in so far as the regulations are considered only as an instrument made under the Act. In the present
case, however, the argument has been carried at least one stage furtherit may be several stages furtherand our attention has
been called to the peculiar position arising under s 33(2) and reg 7, para (2), which create, as I have said, a composite instrument
consisting of provisions of the Act modified by provisions of the regulations. When that composite instrument is construed as
such, the modified provisions of s 33(4), applying to notices under s 33, must have their due effect on the interpretation of s
33(1).
There are one or two other points to be mentioned briefly. First of all, reference was made to the marginal note to s 33.
There again I ought to read 182 something that was said by this court in Stephens v Cuckfield Rural District Council ([1960] 2
All ER at p 720; [1960] 2 QB at p 383):

While the marginal note to a section cannot control the language used in the section, it is at least permissible to
approach a consideration of its general purpose and the mischief at which it is aimed with the note in mind.

Assuming that one can take into account the marginal note to that limited extent, I find that the marginal note here is in these
words: Power to require proper maintenance of waste land, etc This marginal note is on its own merits of no assistance
whatever in construing or understanding or applying s 23. It is clearly inaccurate in two important respects. The expression
waste land is not appropriate to include a garden. The expression Power to require proper maintenance is far from accurate,
because the power of requiring the removal or abatement of an injury to amenity falls very far short of a power to require proper
maintenance. I, therefore, derive no assistance for the present purpose from this marginal note, although it is fair to add that it
would help anyone who was looking for the section to find it.
There was a further point made by counsel for the local planning authority which I will mention briefly. He pointed out that
whereas one might find in certain earlier provisions a set of co-ordinated and interdependent provisions, one finds s 26 and
following sections introduced by the headnote Additional powers of control; and it is, I think, well established that it is
permissible to take a headnote into consideration. That is only a small matter herea small makeweightbut it tends to show
that one can construe s 33 to some extent by itself, and that its construction ought not to be controlled by the earlier provisions of
s 23, s 24 and s 25. Then it was also mentioned that the Caravan Sites and Control of Development Act, 1960, contains certain
provisions which we did not examine in any detail but which, it appears, may relieve hardship or cure defects which existed or
were created under the Act of 1947. If there were certain hardships arising under s 33, they can simply be accepted and
recognised as hardships which could be created under that Act but not of sufficient importance to affect the proper construction.
For those reasons, I agree that this notice H should be upheld as a valid notice, properly issued in exercise of the powers
conferred on the local planning authority by s 33 of the Act of 1947.

Appeal dismissed. Declaration that the notice was valid. Leave to appeal to House of Lords refused.

Solicitors: S Sydney Silverman (for the landowner); Sharpe, Pritchard & Co agents for Clerk to Buckinghamshire County
Council (for the local planning authority).

Henry Summerfield Esq Barrister.


183
[1963] 2 All ER 184

Bland v Cowan
LEISURE AND LICENSING

QUEENS BENCH DIVISION


LORD PARKER CJ, ASHWORTH AND WINN JJ
27 MARCH 1963

Gaming Betting Street or public place Receiving bets Negotiation of bets by agent of bookmakers in factory Betting
slips thrown over factory gate by agent to manager of bookmakers in street Manager frequented street for the purpose of
receiving betting slips Whether that was for purposes of receiving bets within Street Betting Act, 1906 (6 Edw 7 c 43), s 1(1).

B, an agent of the company of which the appellant was manager, negotiated bets in a factory. The appellant was wont to frequent
the street outside the factory gates, where B would throw to him a roll of paper over the gates. This roll contained the betting
slips which evidenced the bets made by punters in the factory with B on behalf of the company. The appellant was convicted of
frequenting the street for the purpose of receiving bets contrary to s 1(1) of the Street Betting Act, 1906 a. On appeal,
________________________________________
a The relevant part of s 1 is set out at p 186, letter h, post

Held In the context or paying or receiving or settling bets in s 1(1) of the Street Betting Act, 1906, the phrase receiving
bets referred to a financial transaction resulting from a betting contract, viz, receiving the losses resulting from the bets, and did
not refer to the physical receipt of slips of paper evidencing betting contracts, viz, in the present case, the betting slips (see p 187,
letters c and i, post); therefore the appellant was not guilty of the offence charged.
Appeal allowed.
Notes
The Street Betting Act, 1906, was repealed as from 28 March 1963, by the Betting, Gaming and Lotteries Act, 1963, s 57(1) and
Sch 8; for the replacing provisions of the new Act, see s 8(1), and cf s 52(3) as regards second or subsequent offences.
As to frequenting streets for the purposes of betting, see 18 Halsburys Laws (3rd Edn) 196, 197, para 388; and for cases on
the subject, see 25 Digest (Repl) 465, 363366.
For the Street Betting Act, 1906, s 1, see 10 Halsburys Statutes (2nd Edn) 778.
For the Betting and Gaming Act, 1960, s 3, see 40 Halsburys Statutes (2nd Edn) 333.

Cases referred to in judgments


Cassidy v Langmuir 1935 SC (J) 65, 25 Digest (Repl) 470, 346.
Williamson v Wright 1924 SC (J) 57, 25 Digest (Repl) 470, 344.

Case Stated
This was a Case Stated by justices for the county borough of Wolverhampton in respect of their adjudication as a magistrates
court sitting at Wolverhampton on 3 October 1962.
On 15 October 1962, the appellant Percy Hubert Bland was arrested by the respondent, William Crosby Cowan, a police-
constable, and charged with having on 15 October 1962, frequented a certain street called Marston Road in the county borough of
Wolverhampton on behalf of Lowe Brothers (Turf Accountants) Ltd bookmakers, of 364, Dudley Road, Wolverhampton, for the
purpose of receiving bets contrary to s 1 of the Street Betting Act, 1906, and s 6(1) b of the Betting and Gaming Act, 1960.
________________________________________
b Section 6(1) of the Act of 1960 increase the penalty for an offence under s 1(1) of the Act of 1906

The following facts were found. (a) The appellant was manager of the betting office of a firm of bookmakers, Lowe
Brothers (Turf Accountants) Ltd situate at 364, Dudley Road, Wolverhampton (hereinafter called the bookmakers) operating as
such with the necessary permit and licence under the Betting and 184 Gaming Act, 1960; (b) Frank Terence Brannon (hereinafter
called the agent) was an employee of Villiers Engineering Co Ltd and produced his card showing that he was authorised in
accordance with s 3(1) of the Betting and Gaming Act, 1960, to receive or negotiate bets as servant or agent of the bookmakers
and that he was in the habit of collecting and receiving bets from fellow employees within the factory of his employers at
Marston Road, Wolverhampton; (c) On 8 October, 9 October and 10 October 1962, the appellant drove into Marston Road in a
motor car, alighted from it and stood in the street outside the gates of the factory; (d) On each occasion the agent was on the
factory side of the gates of the factory and he threw or dropped over the gates to the appellant a roll containing slips of paper,
known as betting slips, bearing the names of horses running in races on the days in question which he had received from punters
within the factory. No money changed hands on any of these occasions; (e) On 15 October 1962, after the appellant had received
twenty-four betting slips in this manner he was arrested by the respondent and admitted receiving the betting slips but stated that
he did not realise he had committed an offence; (f) The slips bore the names of horses running in races at Dunstall Park,
Wolverhampton, on 15 October together with instructions regarding the terms of the bets being placed; (g) The appellant was
charged with the said offence and under caution replied Nothing to say; (h) The appellant collected or received the slips on
behalf of the book-makers and took them to their betting office; (i) No money changed hands on any of the occasions referred to
in the evidence given before the justices, and the agent accounted for the money later each day at the bookmakers offices; (j) ;
(k) At the conclusion of the prosecutions case a submission of no case to answer was made to the justices by the appellants
solicitor who contended that the betting slips had been passed out by an accredited agent within the factory and that they were not
bets received by or effected with the appellant; that the betting slips were being transmitted by the appellant who was acting in a
similar capacity to that of a postman taking bets through the post. The justices overruled this submission and the appellant
thereupon gave evidence and called witnesses on his own behalf; (1) The appellant admitted that he had carried out the practice
referred to in sub-paras (c), (d) and (e) hereof on numerous occasions by arrangement with the agent and that the latter collected
bets within the factory on behalf of the bookmakers; (m) The appellant produced a book of registered agents which included the
name of F Brannon (the agent); (n) The appellant admitted in cross-examination that the slips of paper which he received from
the agent were bets; (o) There was a conflict of evidence between the appellant on the one hand and his witnesses, the agent and
Thomas William Lowe (a director of the bookmakers) on the other, as regards the moment of time when the bets were regarded
as on or, as the justices understood that term, accepted as binding by both parties to the transaction; (p) The appellant
maintained that the bets were on or binding when they reached the bookmakers office whereas his said witnesses stated that
this occurred when the bets were received by the agent within the factory.
It was contended by the appellant (1) that the intention of the Street Betting Act, 1906, was to prohibit betting transactions
between a punter and a book-maker or bookmakers agent in a street; (2) that the bets had been legally received from punters by
the properly authorised agent within the factory; (3) that the bets were made when so received by the agent; (4) that the appellant
was merely acting in the role of a postman or messenger when collecting the betting slips for delivery to the office and that he
was not receiving bets within the meaning of s 1 of the Street Betting Act, 1906.
It was contended by the respondent (a) that the slips of paper bearing the names of horses were bets received by the agent in
the factory as such; (b) that their character had not been changed when they were received in Marston Road, Wolverhampton, by
the appellant; (c) that the appellant had frequented Marston Road and that he had received the bets from the agent whilst he, the
appellant, 185was in such road which was a street within the meaning of the Street Betting Act, 1906.
The justices were of opinion that (i) the appellant, whatever his purpose, was frequenting a street (Marston Road) on behalf
of the bookmakers as there was evidence of four specific occasions and reference to others. This was admitted by the appellant
and they understood that this point was not in dispute; (ii) the rolls or slips of paper or betting slips which the appellant received
from the agent who was inside the factory were bets and that the appellants reception of them in such street completed the
offence with which he was charged, as this was the purpose for which he frequented the street; (iii) in view of these findings they
considered that the question whether the appellant was acting merely as a postman or messenger was not material, and in this
connexion they had in mind that the position concerning postal betting was legalised by the repeal of the Betting Act, 1853, by s
1 of the Betting and Gaming Act, 1960, which did not however alter the situation regarding the offence under review; (iv) the
bets were on or binding on both parties when they were received by the agent within the factory but this did not affect their
findings referred to in (i) and (ii) above.
The justices, therefore, convicted the appellant of the offence charged and fined him 5, allowing him fourteen days for
payment. The appellant now appealed.
The cases noted below were cited in argument in addition to those referred to in the judgment c.
________________________________________
c Stovell v Jameson [1939] 4 All ER 76, [1940] 1 KB 92, R v Way, [1949] 2 All ER 365

D A McI Kemp for the appellant.


M K Harrison-Hall for the respondent.

27 March 1963. The following judgments were delivered.


LORD PARKER CJ. The appellant was the manager of the betting office of Lowe Brothers (Turf Accountants) Ltd
Bookmakers. The bookmakers quite lawfully had in the factory of Villiers Engineering Co Ltd an agent, a man called Brannon,
authorised for that purpose in accordance with s 3 of the Betting and Gaming Act, 1960. He would receive or negotiate bets in
the factory as servant or agent of the bookmaker. In order to ascertain what the agent had done, the appellant was wont to go
outside the factory gates, and there Brannon, the agent, would throw over the gates a roll containing slips of paper, which were
called the betting slips. Those bore the names of the horses, running in races on the day, which he had received from punters
within the factory. The appellant had done this, and admitted that he had done it, on many occasions, and there was little doubt
that he did frequent the street, the sole question being whether he frequented the street for the purpose of receiving bets. The
justices found that the bets were on in the sense that they were accepted as binding between punters and bookmaker in the
factory, and that when the betting slips were thrown over the gates, concluded contracts had been made. Nevertheless, the
justices took the view that the appellant was receiving bets when he obtained the betting slips over the factory gates, which by
that time had become the mere record of contracts made on behalf of the bookmakers by this agent Brannon. The section in
question, s 1(1) of the Street Betting Act, 1906, reads as follows:

Any person frequenting or loitering in streets or public places, on behalf either of himself or of any other person, for
the purpose of bookmaking, or betting, or wagering, or agreeing to bet or wager, or paying or receiving or settling bets
shall

be guilty of an offence.
As it seems to me, the sole question here is what is meant by receiving bets in that phrase or paying or receiving or settling
bets.
There are three possible meanings; the first is the one which I understand to 186 be that which the justices accepted, namely,
that the phrase receiving bets, covered the physical receipt of the slips of paper which evidenced the contracts made by
Brannon; the second possible meaning, which is the one advocated by counsel for the appellant, is that receiving bets means
accepting bets in the sense of accepting an offer and entering into a contract. If that be the true meaning, then on the finding of
the justices that it was Brannon within the factory grounds who had received bets in that sense and this appellant was not guilty
of the offence charged. Some support for that view is certainly to be obtained from the Betting and Gaming Act, 1960, which in
many places, and I need only refer for this purpose to s 3, uses the words receive bets within the phrase receive or negotiate
bets. Thus s 3(1) it is provided that:

No person shall by way of business receive or negotiate bets as servant or agent to another bookmaker or to the Board
unless

certain conditions are fulfilled. To the same effect are the words in sub-s (2), and indeed in the proviso to sub-s (1) of s 2, and in
other places throughout the Act of 1960. The third possible meaning, and the one which I think is correct, is that receiving bets
in the context of sub-s (1) of s 1 of the Act of 1906 is referring to a financial transaction resulting from the contract which has
been entered into. It will be observed in the first place that the earlier part of that subsection is dealing with the formation of the
contract:

Any person frequenting or loitering for the purpose of bookmaking, or betting, or wagering, or agreeing to bet or
wager,

whereas the words that come after the comma and within the one phrase or paying or receiving or settling bets, are pointing to
a later period of time when not only the contract has been negotiated and formed, but also the transaction is being wound up.
It seems to me that the phrase is dealing in the first place with paying the amount of winnings of the punter; secondly, with
receiving the amount of losses incurred by the punter; and, thirdly, with settling the account between them, that latter provision
covering the case where money does not pass but the matter becomes one of account or cross account. It seems to me that in the
context of s 1(1) of the Act of 1906, which is quite a different context to that in which the phrase receive or negotiate bets
appears in the Act of 1960, the phrase is dealing purely with the financial settlement once the contract is entered into and the
event has been determined. That, as I say, seems to me the most natural meaning, but it is to be observed that in two cases in
Scotland it has been assumed, without being decided, that somebody who, like the appellant in the present case, is acting purely
as a messenger to convey a record of a betting transaction entered into, to his principal, is guilty of receiving bets. The two cases
are, first, a case in 1924, Williamson v Wright; the other is the case of Cassidy v Langmuir. In both those cases the real question
was whether the messenger, who was operating in a motor car, could be said to be frequenting or loitering in the street; it was
assumed in both cases and never argued that, if he was frequenting or loitering, it was for the purposes of receiving bets.
At the same time Lord Anderson in the former case said (1924 SC (J) at p 60):

That he was in the burgh for the purpose of receiving bets, and that he did receive bets on that day I have no doubt
whatever. And I think that the magistrate was well justified in drawing the inference that bets were received on that
occasion.

He alone of the members of the court dealt with the point at all; it was never argued; it had been assumed. For may part, for the
reasons which I have stated, I think that the natural meaning of receiving bets in this context is to receive the losses resulting
from the bets.
187
I would add only that it may be, and I am not deciding the question, that this appellant could have been properly charged
with frequenting this street on behalf of his principals, Messrs Lowe, for the purpose of bookmaking, words which are very
wide indeed and would cover, I conceive, every aspect connected with a bookmakers business, including the knowledge of
betting contracts entered into by an agent, which knowledge he might well require in order to lay off some of the bets. In these
circumstances I have come to the conclusion that the justices were wrong, and that this appeal succeeds.

ASHWORTH J. I agree.

WINN J. I agree.

Appeal allowed.

Solicitors: Gibson & Weldon agents for Challinor & Roberts, Smethwick (for the appellant); Sharpe, Pritchard & Co agents for
Town clerk, Wolverhampton (for the respondent).

N P Metcalfe Esq Barrister.


[1963] 2 All ER 188
Re Michelhams Will Trusts
TRUSTS

CHANCERY DIVISION
BUCKLEY J
21, 22 MARCH 1963

Trust and Trustee Variation of trusts by the court Restraint of marriage Arrangement involving Swiss contracts that might
operate in restraint of marriage Whether arrangement should be approved Variation of Trusts Act, 1958 (6 & 7 Eliz 2 c 53), s
1.

Property was held on trust in two equal shares, subject to an annuity to Lord M for life, as to one share for Lord M for life and
after his death for any wife of his for her life, and thereafter for any child or children of Lord M as he should appoint or, in
default of appointment, for any child or all children of his equally and, subject thereto, for Lord M absolutely. The other share
was held on similar trusts in favour of Lord Ms brother, and any wife, child or children of his. Lord M and his brother applied
for variation of the trusts. Lord M was sixty-five and his brother was fifty-nine years of age. Neither was married at the time of
the application, and each deposed that he had no wish to marry; and neither applicant had had children. Approval was sought on
behalf of any future wife or unborn child of either of the applicants for an arrangement whereby trust property should be
transferred in agreed proportions to the applicants absolutely, insurance policies being effected which would ensure that, if either
applicant married, certain sums would become available to replace the funds so transferred. The policies included a stipulation
that the insurers should be indemnified by a Swiss bank if the policy moneys became payable. Both applicants lived in
Switzerland. The Swiss bank proposed to give the indemnity on terms that it, in turn, should be indemnified by one or other of
the applicants in case the policy moneys became payable.

Held Although by virtue of the counter-indemnities given by the applicants to the Swiss bank the scheme might be regarded as
tending to discourage the applicants from marrying, yet the doctrine of public policy, whereby a contract in restraint of marriage
was unenforceable, did not affect the validity of the proposed arrangement, for, even if the counter-indemnities were not
enforceable by the Swiss bank (which was a question of Swiss law), that would not relieve the Swiss bank from its obligation to
indemnify the insurers; accordingly the court would approve the arrangement under the Variation of Trusts Act, 1958, s 1, on
behalf of any future wife or unborn child of either applicant, the arrangement being, on the facts, calculated to benefit such
possible beneficiaries (see p 191, letter i, and p 193, letter a, post).
188

Notes
As to the jurisdiction to vary trusts, see 38 Halsburys Laws (3rd Edn) 10291031, para 1772.
For the Variation of Trusts Act, 1958, s 1, see 38 Halsburys Statutes (2nd Edn) 1130.

Cases referred to in judgment


Baker v White (1690), 2 Vern 215, 23 ER 740, 12 Digest (Repl) 278, 2133.
Hartley v Rice (1808), 10 East, 22, 103 ER 683, 12 Digest (Repl) 278, 2137.
Lowe v Peers (1768), 4 Burr 2225, Wilm 364, 98 ER 160, 12 Digest (Repl) 278, 2131.

Adjourned Summons
This was an application by originating summons dated 4 May 1960, by the Right Hon Herman Alfred, second Baron Michelham
of Hellingly, and the Hon Jack Herbert Michelham who were beneficially interested under the will dated 27 September 1918, of
the first Baron Michelham, deceased, a settlement dated 22 October 1926, and the will dated 19 November 1925, of Aimee
Geraldine Almy, deceased. The application was that the court might, pursuant to the Variation of Trusts Act, 1958, approve, with
or without modification, on behalf of any future wife and unborn issue of either of the applicants who might become interested
under the trusts of the above-mentioned wills and settlement, an arrangement varying the trusts thereof. The respondent was
Coutts & Co the sole trustee of the wills and settlement. The facts appear in the judgment.
The case noted belowa was cited in argument in addition to those referred to in the judgment.
________________________________________
a Robinson v Ommanney (1883), 23 Ch D 285

R W Goff QC and G B H Dillon for the applicants.


E I Goulding QC and M J Fox for the respondents.

22 March 1963. The following judgment was delivered.

BUCKLEY J. This is an application under the Variation of Trusts Act, 1958, to vary the trusts of the will of the first Lord
Michelham and of a settlement dated 22 October 1926, which was made as a consequence of certain compromise negotiations
which took place in litigation following Lord Michelhams death, and to vary the trusts of the will of the lady who was Lord
Michelhams wife, who subsequently remarried and became Mrs Almy. The only persons who at present have any beneficial
interests under any of those trusts are the two applicants, the present Lord Michelham and his brother, Mr Michelham.
Under the will of the first Lord Michelham, his residuary estate is held, in the events which have happened, on trust to pay
an annuity to the present Lord Michelham during his life, and, subject thereto, the residue is held in equal shares for Lord
Michelham and his brother, each share being settled and held on trust either for Lord Michelham or for his brother, as the case
may be, for life with remainder in each case to any wife of the life tenant who may survive him during her life. Then there is a
power for Lord Michelham or his brother, as the case may be to appoint among his children or issue, and a trust in default of
appointment in the ordinary form for his children who attain the age of twenty-one or, being female, marry, with an ultimate trust
on failure of all the preceding trusts for Lord Michelham or his brother, as the case may be, absolutely.
Under the settlement, certain chattels were settled, which are now mainly represented by investments of the proceeds of sale
thereof, although some of the chattels remain unrealised. There are two funds of settled property under the settlement, one of
which is called the pictures fund and the other the works of art fund. The pictures fund is subject to a trust for Lord
Michelham for life, with remainder to his sons successively in tail male, with remainder to Mr Michelham for life, with
remainder to his sons successively in tail male. If those trusts fail, the fund will belong to Lord Michelham and Mr Michelham in
equal shares in consequence of the provisions of their mothers will. The works of art fund is held on trust for Mr Michelham for
life, with remainder to his sons successively in tail male, with remainder to Lord Michelham for his life, with remainder 189 to
his sons successively in tail male, and, if all those limitations fail, that fund will be divisible between Lord Michelham and his
brother equally.
Under the will of Mrs Almy, there is a fund which is held on trusts identical with the trusts of the pictures fund.
Lord Michelham is now sixty-two years of age. Mr Michelham is now fifty-nine years of age. Lord Michelham has been
married. His wife died in 1961, but in fact Lord Michelham and his wife had been living apart for many years before that. There
is evidence that Lord Michelham has no wish or desire ever to remarry, and that there is no likelihood that he will ever do so. Mr
Michelham is a bachelor, and he has deposed to the fact that he has no wish or desire ever to marry and that there is no likelihood
that he will ever do so. If those present intentions are maintained, the only persons who will ever have an interest in this fund are
the two applicants. But there is, of course, always the possibility that one or other of the applicants might change his mind, in
which case he might leave a widow surviving him who would take an interest in his settled share of his fathers residuary estate,
and he might leave issue interested under the trusts of the will of the first Lord Michelham, or a son who would take an interest
under the trusts of the settlement or of Mrs Almys will.
In these circumstances, a scheme has been propounded, the effect of which I can state quite shortly. Each of the funds with
which I am concerned is at present invested partly in investments which are exempt from tax so long as the person interested in
them is domiciled and usually resident out of this country, and partly in investments which are not so exempt. There are pictures,
which are subject to the trusts of the pictures fund, which are valued at a little over 3,000. There are other works of art, which
are subject to the trusts of the works of art fund, which are valued at rather over 17,000. Those pictures and works of art are
thought to be such as would be exempt from duty, in any event if they were not sold following the death of the beneficiary
entitled to an interest in them. The greater part of the investment representing each of the funds is in investments which are not
exempt, except in the case of the fund subject to the trusts of Mrs Almys will, where the greater part of the fund is in exempt
securities and a smaller part is in securities that are not exempt. It is proposed that all this property should be transferred, in
proportions which have been agreed between Lord Michelham and his brother, to them absolutely, but that insurance policies
should be effected which would ensure that, in certain events, certain sums would become available to replace the funds so
handed over to them. The amounts to be secured by those policies have been calculated in this way. The prospective rate of duty
payable on the death of each of the applicants has been estimated, and that part of each fund which is invested in securities that
are not exempt from tax in the way which I have mentioned has been assumed to be liable to duty at that rate, the remainder of
the fund which is in exempt securities being assumed not to be liable to duty. The amount of the fund which would remain after
payment of duty if the present tenant for life were to die forthwith has been ascertained, and that net sum has been written up by
ten per cent to arrive at the figure to be secured by the relevant insurance policies. So the policies are designed to ensure
repayment to the trustees of a fund ten per cent more than would be left if the tenant for life were to die today and if that duty
were only to be exacted on that part of the trust fund which is invested in securities that are not exempt.
The policies are to be payable in the following events. The policies taken out to secure repayment in respect of the two
settled shares of the testators residuary estate are to mature in the event of Lord Michelham, in the one case, and Mr Michelham
in the other case, marrying at any time after the scheme comes into operation. The policies that are intended to deal with the
position in respect of the pictures fund and the works of art fund, and the fund subject to the trusts of Mrs Almys will, will be
policies that will mature in any of the following events (whichever shall first happen), that is to say, the death of Lord Michelham
having had a son, the death of Mr Michelham having had a son, the birth of a 190 posthumous son to Lord Michelham, or the
birth of a posthumous son to Mr Michelham.
Now there are two points which have been ventilated on the scheme. One relates to the quantum of the sums to be secured
by the policies; the other is a question of general application whether there is some element in this scheme which, because of a
tendency in the scheme to discourage Lord Michelham or Mr Michelham from marrying, might conflict with public policy.
I will deal, first of all, with the question of quantum. The present position with regard to Lord Michelham and Mr
Michelham in relation to their domicil and residence is this. Lord Michelham has for upwards of twenty years resided in Geneva.
He has had business interests in many parts of the world. He has not had any home in this country for many years. There is
evidence before me which affords good grounds for concluding that the right view today is that his domicil is Swiss, and,
although, if the issue of domicil were ever contested in the future, other evidence might be produced which would result in a
different conclusion, I am content to proceed on the footing that Lord Michelhams domicil should, for present purposes, be
regarded as being in Switzerland. Mr Michelham has recently acquired a residence in Geneva. Previously his life has been
nomadic as regards residence. But it appears that he has now got a settled home in Geneva, and, in his case also, the evidence
satisfies me that I can proceed to consider this scheme on the footing that he is at present domiciled in Switzerland. Each of them
says that his intention is to continue to reside in Switzerland, and that he has no intention of making a home anywhere else. But,
of course, that intention might change hereafter, and, if either of these applicants were to lose his domicil of choice in Geneva, as
the law stands at present his domicil of origin would revive unless he acquired some other domicil of choice, and the domicil of
origin of each of these gentlemen is in this country. Moreover, without a change of domicil, either of them might cease to be
resident in Switzerland and might become resident in this country. And those possibilities might be greater than they are today if
the only event with which I am really concerned, that is to say, either of them marrying, were to occur, for it might be that, for
reasons connected with the marriage, there would be a change of heart about domicil or about residence.
As matters stand, if either Lord Michelham or his brother were to die, that part of each fund which is invested in exempt
securities would escape duty, and, on that footing, the amount to be provided in the events with which I am concerned by the
policies would be more than what would be left of the funds in which that applicant is interested, if he were to die forthwith and
duty were to be exacted at the rate which is expected.
It was pointed out by counsel who appeared for the trustees that, if either Lord Michelham or his brother were to marry, the
trustees might feel obliged to alter their investment policy and invest a larger proportion of the funds in exempt securities, in
which case the margin of benefit provided by the sums secured by the policies would probably be decreased and might even be
eliminated altogether.
I have got to look at this scheme as a whole, and at the probable events as a whole, and I have not got to consider whether
the scheme is bound to confer a benefit on everyone who may become interested under the trusts in every event, but whether in
the probable events it is calculated to confer a benefit. It seems to me that I have got to take into consideration the probable
investment policy of the trustees in the future, and the possibility either that the domicil of one or other of these applicants might
be established not to be Swiss, or that there might be a change of domicil or there might be a change of residence such as to put
an end to the exemption in respect of the exempt parts of the fund. Balancing all the considerations it is fair to say that the
scheme is one which is calculated to confer a benefit on possible unascertained or unborn beneficiaries and, on the quantum
aspect of the matter, I am satisfied, although this is perhaps a case which is a little nearer to the line than some.
191
I turn now to the question of public policy. A contract which is in general restraint of marriage is unenforceable on the
grounds of public policy, whether it contains an express undertaking that the subject will not marry, or whether it is merely a
contract of a kind which, on financial or other grounds, will tend to discourage the subject from marrying. Thus, in Baker v
White a widow who gave a bond of 200 payable in the event of her remarrying was held to be entitled to have such bond
delivered up and cancelled; in Lowe v Peers where the defendant had covenanted not to marry any woman other than the plaintiff
and to pay the plaintiff 1,000 in the event of his marrying any other woman, he was held not to be liable to make the payment
notwithstanding that he had married another woman on the ground that the covenant was illegal as being a restraint on
matrimony. See also Hartley v Rice, where the plaintiff had entered into a wager with the defendant that the plaintiff would not
marry within six years. The first and third of these cases were cases which did not involve a direct obligation not to marry, but
tended, for financial reasons, to discourage marriage, and they were decided on the ground that for that reason the court would
not countenance the contract because it offended against the public interest.
Before I develop this, I should mention another fact. The terms on which the insurers are prepared to grant the policies
payable in the event of Lord Michelham and Mr Michelham marrying include a stipulation that they shall be indemnified in the
event of the policy moneys becoming payable under either of these policies. It is proposed that they shall be indemnified by a
Swiss bank, and the proposed form of the deed of indemnity has been produced. It contains a recital that the Swiss bank is
prepared to grant the indemnity on terms that the Swiss bank is itself indemnified in the one case by Lord Michelham and in the
other case by Mr Michelham. It is this counter-indemnity which gives rise to this question of public policy because it is
suggested that, looking at the scheme as a whole, it is one which involves Lord Michelham or Mr Michelham undertaking a
financial liability which will mature in the event of his marrying, and therefore it is suggested that the arrangement is one which
conflicts with the principle of public policy that I have mentioned. It is said that the counter-indemnity to be given to the Swiss
bank must tend to discourage Lord Michelham and Mr Michelham from marrying.
The trustees, who are not opposing this application but thought it their duty to bring this point to the notice of the court,
posed this question: whether, having regard to the scheme as a whole, the insurers might not legitimately, and I say
legitimately for I am not concerned with ethics but with legal obligation, resist a claim under the policies which are the subject-
matter of these indemnities because of this element in the scheme? The policies will be issued in response to proposals by Lord
Michelham and Mr Michelham, and they will pay the premiums out of their own resources. There will be no consideration
moving from the trustees to the insurers, but the policies will be under seal, and no difficulty can arise on this ground in respect
of the trustees power to sue on the policies. As between the insurers and Lord Michelham and Mr Michelham, however, the
premiums will not be the only consideration, for, as I have indicated, Lord Michelham and Mr Michelham are to procure the
indemnities of the Swiss bank. What arrangements the Swiss bank may choose to make to protect itself seem to me to be of no
interest to the insurers and no concern of theirs. The nature of any such arrangements which the Swiss bank may see fit to make
could not qualify in any way the obligation of the Swiss bank to indemnify the insurers, and a fortiori I think it could not qualify
the obligation of the insurers to pay under the policies. Even if I assume that the counter-indemnities to be given by Lord
Michelham and Mr Michelham would be unenforceable by the Swiss bank, this would not, in my judgment, relieve the Swiss
bank of liability or the insurers of liability under the policies. But I have no reason to suppose that the Swiss 192 bank would be
unable to enforce the counter-indemnities. These will, no doubt, be governed by Swiss law, and I think that I am entitled to
assume that the Swiss bank will satisfy itself that whatever protection it requires will be protection of which it can avail itself.
For these reasons, I think that, although it is quite proper that the trustees should bring the matter to the attention of the court, the
case is not one in which the doctrine of public policy can in any way affect the validity or effectiveness of the scheme.
In these circumstances, I do not think that it is necessary for me to deal with the various other points that were touched on in
the argument, but I will just mention what they were. First, it was suggested that, Lord Michelham having been already married,
the doctrine of public policy was not one which would affect him, and there is some support for that view in some of the
authorities that were read to me. Secondly, it was suggested that Mr Michelham, who many years ago acquired Cuban nationality
and still retains that nationality, was not a person to whom the doctrine would apply. And, thirdly, it was suggested that, by
analogy to the reasoning which has been employed in some of the cases in which a similar doctrine has been applied to
dispositions of property either under wills or settlements where interests have been made subject to defeasance on marriage, it is
relevant to look at the intention of the parties generally and see whether the arrangement is one which is really directed to
preventing marriage or whether it is directed to some other proper and prudent object. If that were a relevant consideration in the
present case, and I do not say whether it is or is not, it seems to me that the aim of the present arrangement is one which can be
accepted as proper, for it is in essence this: the trustees are being asked to distribute to Lord Michelham and his brother a fund
which they can only distribute by committing a breach of trust, subject, of course, to the protection which the consent of the
beneficiaries and the order of this court will give them. The object of the scheme, so far as the insurance policies and the
indemnities that will support them are concerned, is that Lord Michelham and his brother will replace in the hands of the trustees,
or procure to be reimbursed to the trustees, some part of what they are getting in that way in the event of any other persons
coming into existence who might have a beneficial interest under the trusts, and, in those circumstances, it would be only right
and proper that they should make such reimbursement. So that it seems to me that the scheme is one which is a very proper one
for them to enter into, even if it does involve some obligations which may be said to tend to discourage either of them from
marrying.
In these circumstances, I am prepared to approve the arrangement.

Order accordingly.

Solicitors: Herbert Oppenheimer & Vandyk (for the applicants); Farrer & Co (for the respondents).

Jenifer Sandell Barrister.


193
[1963] 2 All ER 194

Cope v United Dairies (London) Ltd


ADMINISTRATION OF JUSTICE; Legal Aid and Advice

QUEENS BENCH DIVISION


MEGAW J
15 MARCH 1963

Legal Aid Costs Dismissal of action for want of prosecution Legal aid certificate discharged before dismissal of action
Whether there had been a trial or hearing of action Whether defendants entitled to costs incurred prior to discharge of assisted
persons certificate Legal Aid and Advice Act, 1949 (12 & 13 Geo 6 c 51), s 2(2)(e) Legal Aid (General) Regulations, 1962
(SI 1962 No 148), reg 18(1).

In April, 1961, a legal aid certificate was issued to the plaintiff to bring an action in the High Court against the defendants for
damages for personal injuries. A writ was issued and pleadings delivered. In April, 1962, his legal aid certificate was discharged,
and he thereafter took no further steps in the action. On the defendants application by summons the master made an order in
June, 1962, on the ground of want of prosecution that the action be dismissed with costs, unless the plaintiff issued a summons
for directions within twenty-one days, and that the defendants costs be taxed and paid by the plaintiff. The defendants bill of
costs, brought in for taxation, included the whole of their costs of the action. On taxation all costs referable to the period between
the granting and the discharge of the legal aid certificate were disallowed. By s 2(2)(e) a of the Legal Aid and Advice Act, 1949,
where a person receives legal aid in connexion with any proceedings, his liability by virtue of an order for costs made against him
with respect to the proceedings is not to exceed the amount, if any, which it is reasonable for him to pay; and by reg 18(1) of the
Legal Aid (General) Regulations, 1962 b, where such an order is made, the determination of the amount of liability in accordance
with s 2(2)(e) is to be made at the trial or hearing of the action. On application for an order to review taxation,
________________________________________
a Section 2(2)(e) is set out at p 196, letter e, post
b The material terms of reg 18(1) are set out at p 197, letter a, post

Held (i) The determination of costs in an action to which an assisted person was a party involved two phases, viz, (first) the
determination by the court in its discretion whether an order for costs should be made against the assisted person, and (second) a
separate determination of the amount of the liability of the assisted person for costs (see p 198, letters a and b, post); the second
phase was required by reg 18(1) of the Legal Aid (General) Regulations, 1962, to be done, and could be done only, at the trial or
hearing of the action, but, since on the true construction of reg 18 the coming into effect of the dismissal of the action under the
order of June, 1962, was not a trial or hearing of the action within reg 18(1), there was no jurisdiction, either at the time when
the order of June, 1962, was made or subsequently, to make a determination of the amount of the plaintiffs liability for costs
incurred during the currency of his legal aid certificate (see p 197, letter g, and p 198, letters d and g, post).
(ii) The order of June, 1962, however, on its true construction, neither determined the amount of the plaintiffs liability for
costs during the currency of the legal aid certificate, nor required the taxing master to assess such costs, and accordingly the
taxing master could not properly allow any costs against the plaintiff in respect of that period (see p 198, letter g, post).
Application dismissed.

Notes
As to the award of costs against an assisted person, see 30 Halsburys Laws (3rd Edn) 502, 503, para 933.
For the Legal Aid and Advice Act, 1949, s 2, see 18 Halsburys Statutes (2nd Edn) 535.
For the Legal Aid (General) Regulations, 1962, reg 18(1), see 5 Halsburys Statutory Instruments (1st Re-issue) 243.
194

Cases referred to in judgment


Blatcher v Heaysman [1960] 2 All ER 721, [1960] 1 WLR 663, 3rd Digest Supp.
Wozniak v Wozniak [1953] 1 All ER 1192, [1953] P 179, [1953] 2 WLR 1075, 3rd Digest Supp.

Review of taxation
The defendants applied under r 35 of the Supreme Courts Cost Rules, 1959, for a review of the taxation of their costs in an action
for personal injuries commenced against them by the plaintiff in the Queens Bench Division which was dismissed by Master
Ritchie for want of prosecution on 8 June 1962. The master ordered that the defendants costs be taxed and paid by the plaintiff.
Megaw J, heard the application in chambers on 14 March 1963, and delivered judgment in open court at the request of the
defendants. The facts are set out in the judgment.
The cases noted belowc were cited during the argument in addition to those referred to in the judgment and were supplied by
the courtesy of counsel.
________________________________________
c Arnott v Amber Chemicals Ltd (19 May 1953), The Times, 20 May 1953, MacCarthy v Agard, [1933] All ER Rep 991, [1933] 2 KB 417,
Wilson v Metcalfe, 1826), 1 Russ 530

J H Hames as amicus curiae.


A Lipfriend and Eleanor Platt for the defendants.

Cur adv vult

15 March 1963. The following judgment was delivered.

MEGAW J read the following judgment. On 26 April 1961, a legal aid certificate was issued to the plaintiff in respect of
proceedings by him against the defendants, claiming damages for personal injuries said to have been sustained while he was
employed by the defendants. A writ was issued and pleadings were delivered. On 27 April 1962, the plaintiffs legal aid
certificate was discharged. The plaintiff thereafter took no further steps in the action, but no application was made by him or on
his behalf for discontinuance. On 4 June 1962, a summons was issued by the defendants solicitors in the following terms. It
asked for

an order that this action be dismissed with costs for want of prosecution, the plaintiff having failed to issue a summons
for directions pursuant to R.S.C., Ord. 30, r. 1, and that the defendants costs be taxed and paid by the plaintiff.

The summons was heard by Master Ritchie on 8 June 1962. The plaintiff did not appear and was not represented. The
defendants were represented by their solicitors, who fairly and properly informed the master that the plaintiff had had a legal aid
certificate and that it had been discharged. The master made an order in the following terms:

It is ordered that, unless the plaintiff issues a summons for directions within twenty-one days from the date of this
order, this action be dismissed with costs for want of prosecution and the defendants costs be taxed and paid by the
plaintiff, and that the costs of this application be the defendants costs in any event.

The plaintiff was thereafter notified of the order and of the intended taxation of costs.
When the defendants solicitors brought in their bill of costs for taxation, they included therein the whole of the costs of the
action, including the costs incurred by them during the period when the plaintiff had his legal aid certificate. The taxing officer
refused to allow any items in the bill referable to the period between the granting and the discharge of the legal aid certificate.
The balance which he allowed, referable to the period after the discharge of the legal aid certificate, was 46 5s 8d. The
defendants raised objections to the disallowances. On review, the senior taxing master overruled the objections. The defendants
now ask for a review under r 35 of the Supreme Court Costs Rules, 1959. I have 195 been very much assisted by the careful and
able arguments presented yesterday in chambers by counsel for the defendants, and by counsel who was instructed by the Law
Society on my suggestion that it would be helpful if counsel were to be instructed as amicus curiae, in view of the nature of the
issues raised. At the request of the defendants, I adjourned the application into open court for judgment.
The defendants submit that the senior taxing master exceeded his jurisdiction. They say that the order made by Master
Ritchie was clear and unambiguous; and that, even if the taxing master thought that it was wrong or ultra vires, he had no option
but to comply. He was not entitled to disallow certain items on the ground that they were wrongly or irregularly made the subject
of taxation. If the order were wrong, the only remedy would have been by way of appeal from that order to the judge in
chambers. No such appeal had been made. The defendants further submitted that Master Ritchies order was not wrong,
irregular or ultra vires.
Counsel for the Law Society agreed that the taxing master could not properly refuse to carry out an order for taxation, in
whole or in part, because he considered it to be wrong or ultra vires, and that the same applies to this court on a review. With that
proposition, I agree. Counsel instructed by the Law Society submitted, however, that what the senior taxing master did in this
case was right; not because he had any jurisdiction to refuse to carry out a taxation as directed, but because, in the circumstances,
on the true construction of Master Ritchies order of 8 June 1962, it required taxation only of those costs which were incurred
outside the period during which the legal aid certificate was in force. In my judgment, that contention is right.
Section 2(2) of the Legal Aid and Advice Act, 1949, provides as follows:

Where a person receives legal aid in connexion with any proceedings (e) his liability by virtue of an order for costs
made against him with respect to the proceedings shall not exceed the amount (if any) which is a reasonable one for him to
pay having regard to all the circumstances, including the means of all the parties and their conduct in connexion with the
dispute.

Section 2(3) provides:

Regulations shall make provision as to the court, tribunal or person by whom the amount referred to in para. (e) of the
last foregoing subsection is to be determined and the extent to which any determination thereof is to be final.

Regulation 13(1) of the Legal Aid (General) Regulations, 1962, which deals with the position of a person who has once been
legally aided but whose certificate has, as here, been discharged, provides:

Subject to the provisions of this regulation, a person whose certificate is revoked shall be deemed never to have been
an assisted person in relation to the claim or proceedings to which the certificate related, and a person whose certificate is
discharged shall, from the date of discharge, cease to be an assisted person in the claim or proceedings.

That paragraph is, however, qualified as regards costs by reg 13(6). The relevant part of that paragraph provides:

Where a certificate has been discharged and where he [that is the person who had the certificate] continues to
assert or dispute the claim or to take, defend or be a party to the proceedings to which the certificate related (b) those
provisions of s. 2 of the Act which relate to an assisted persons liability by virtue of an order for costs made against him
shall apply in so far as the costs were incurred while he was an assisted person.

The effect is that, in relation to the plaintiffs liability for the defendants costs incurred in the period in which he was an assisted
person, he is to be treated, for 196 the purposes of s 2(2)(e) and s 2(3) of the Act, and reg 18 made thereunder, as an assisted
person. Regulation 18(1) is as follows:

Where an order for costs is made against an assisted person, the determination of the amount of his liability for costs
in accordance with s. 2(2)(e) of the Act shall be made at the trial or hearing of the action, cause or matter.

Then there is a proviso which is here irrelevant. I should also read reg 18(5):

Where an assisted person serves notice of discontinuance or where an order for costs is made against him by reason of
his default of appearance or defence or by reason of summary judgment, or upon application by him for leave to
discontinue, he shall be liable for the full amount of the costs, unless he makes an application for determination of the
amount of his liability under the provisions of s. 2(2)(e) of the Act, in which case the court may make any such order for
payment as may be made in determining an assisted persons liability by virtue of an order for costs made against him at a
trial or hearing of an action, cause or matter, and shall to the same extent be final.

That paragraph has no direct application here, first, possibly, because it applies to an assisted person and not to one who has
ceased to be an assisted person; and, secondly, because, perhaps by inadvertence, it does not refer to the termination of
proceedings by an order for dismissal for want of prosecution. There is thus no provision, in the circumstances of this case,
making the plaintiff liable for the full amount of the costs unless he makes application for the determination of the amount of his
liability under s 2(2)(e) of the Legal Aid and Advice Act, 1949. It may well be that such a provision would not be appropriate in
the case of a person whose certificate has been discharged, since he might have no legal advice and could not fairly be expected
to know of, and assert, his right to seek a reduction of his liability for the full amount of costs incurred while he was an assisted
person. Regulation 18(5) is, however, relevant in one way to the present issue. It throws light on the meaning of the trial or
hearing of the action, cause or matter in reg 18(1). Paragraph (5) uses those same words in, I think, deliberate contrast with
other methods of putting an end to proceedings, such as discontinuance, default of appearance or summary judgment. That
indicates that the dismissal of an action for want of prosecution is not comprehended within the words trial or hearing of the
action, cause or matter in reg 18(1). It follows that there never has been a trial or hearing of the action in this case. Therefore,
the mandatory provision of reg 18(1) for the determination of the amount of the plaintiffs liability for costs incurred during the
period covered by the legal aid certificate could not be complied with.
Counsel for the defendants submitted that, although there was not a trial or hearing of the action on 8 June 1962, when
Master Ritchie made the order for dismissal of the action at the end of twenty-one days, unless the plaintiff meanwhile should
issue a summons for directions, nevertheless there was a notional trial or hearing of the action when the twenty-one days
expired and the condition subsequent was fulfilled. He said that trial or hearing of the action meant no more than the final
determination of the matter. He pointed out that, if it were otherwise, there would be real difficulties in the way of a non-
assisted person, in circumstances such as the present, obtaining a determination of the amount of costs payable by the plaintiff in
respect of the legal aid certificate period. He said that the suggestion of counsel for the Law Society that the defendants could
have themselves issued a summons for directions and obtained directions and thereafter a judgment by default, was unrealistic. It
would necessarily involve the incurring of further, probably irrecoverable, costs. That may, indeed, be so on the regulations as
they stand, and it may well be unsatisfactory. But I am unable to hold that the coming into effect of the dismissal of the action
twenty-one days after the order of 8 June 1962, was a trial or hearing of the action. It is inconsistent with any ordinary, or, I
think, 197any possible, meaning of the words. It is inconsistent with the indication of the meaning of the words to be derived
from reg 18(5).
Counsel for the Law Society, rightly as I think, submits that, where there is a determination of costs in an action to which an
assisted person is a party, that determination involves two phases, though they sometimes take place at the same time. The first
phase is the determination by the court, in its discretion, whether or not it is proper that an order for costs should be made against
the assisted person. This phase, by reason of s 1(7) of the Act, is carried out without reference to any consideration as to the party
being, or having ever been, an assisted person. The second phase, which arises in relation to a person who is, or has been,
assisted, is the separate determination as to the amount, if any, of the liability for costs which the assisted person should be
ordered to pay. Frequently, in fact, these two phases fall to be dealt with at different stages. Thus, where costs are awarded
against an assisted person on an interlocutory application, the amount must not be assessed at that stage. This is clearly
established by Wozniak v Wozniak. The doctrine of the two phases is implicit in the judgment of the Court of Appeal in Blatcher
v Heaysman. It is reflected in the opening words of reg 18(1), and, I think, in s 2(2)(e) of the Act.
For the reasons which I have given, if Master Ritchie purported to make an order determining the amount of the plaintiffs
liability for costs in respect of the period when he had a legal aid certificate, he had no jurisdiction to do so, because this could
only be doneand had to be doneat the trial or hearing of the action. Further, if he purported to do so, I should not regard
an order for costs in the form of the order made by the learned master as constituting a determination of the amount of the
plaintiffs liability for costs in respect of the legal aid certificate period. There is, in the order, no determination of amount. If,
indeed, that was the meaning and effect of the order, the taxing master would have been wrong in failing to carry it out, and I
should have had no power to uphold the taxing masters decision. The plaintiffs only remedy would be to seek leave to appeal
out of time and to ask that the order as to costs be set aside or varied. However, I agree with the submission of counsel for the
Law Society that that was not the meaning or effect of Master Ritchies order. All that the master was asked to do by the
defendants summons, and all that he did and purported to do by his order was, first, to make the general order as to costs,
referable to the period subsequent to the discharge of the legal aid certificate; and, secondly, in relation to the costs of the period
covered by the legal aid certificate, to deal with the first phase, with which alone he had jurisdiction to deal. He was not invited
to make, and could not properly make, and did not purport to make, any determination of the amount of the plaintiffs liability for
costs incurred during the currency of the legal aid certificate. Such a determination has never been made, and, in view of the
course of the proceedings, could never have been made under the regulations as they now are. The senior taxing master,
therefore, has properly given effect to Master Ritchies order in accordance with its true meaning. He could not himself
determine the amount of the costs during the legal aid period, and he could not properly allow any such costs on taxation, since
they had never been assessed as required by the Legal Aid and Advice Act, 1949, and the Legal Aid (General) Regulations, 1962.
Master Ritchies order did not require him to do so.
I cannot accept counsel for the defendants argument that, prima facie, an assisted person is obliged to pay the whole costs,
and that it is up to him, if he sees fit, to invoke the special protection given by the Act of 1949 to an assisted person. Indeed,
there was no time in the course of these proceedings when he could have required such a determination. The special protection is
the responsibility of the court, whether or not the assisted person, or previously assisted person, invokes it. It is only by virtue of
the special provisions of reg 18(5) that 198 it ever falls on the assisted person to invoke that protection. Regulation 18(5) is not
applicable here. Accordingly, though perhaps for rather different reasons from those given by the senior taxing master, I hold that
his disallowance of the items which he disallowed on this bill of costs was right.
I should add that it will be clear from what I have already said that, in my view, the regulations, and in particular reg 18(1)
and (5), may merit reconsideration, especially with regard to the time and manner of the determination of a previously assisted
persons costs where, after the discharge of his legal aid certificate, the action is dismissed for want of prosecution. If I am right
in this judgment, the regulations may involve hardship on the non-assisted party. If I should be wrong in this judgment, the
formerly assisted person, with no available legal advice, may be deprived of the protection which I think that the Legal Aid and
Advice Act, 1949, intended that he should have.

Application dismissed. Leave to appeal to the Court of Appeal.

Solicitors: Blount, Petre & Co (for the defendants); Secretary of the Law Society (for the Law Society).

Mary Colton Barrister.


[1963] 2 All ER 199

N W Robbie & Co Ltd v Witney Warehouse Company Ltd


COMPANY; Charges

LIVERPOOL ASSIZES
WIDGERY J
23, 24 JANUARY, 4 FEBRUARY 1963

Company Debenture Floating charge Receiver and manager appointed Crystallisation of floating charge Goods
supplied by company on credit before and after charge became fixed Set-off Purchasers of goods obtaining, after receiver
appointed, assignment of debts due from company which arose after the debenture was issued but before receiver appointed
Whether the assigned debts could be set-off against the purchasers indebtedness to plaintiff company for goods bought.

On 26 January 1960, the plaintiff (a company incorporated in Ireland) issued to its bankers a debenture securing all moneys from
time to time due to them from the company by a floating charge on the assets of the company. Between 24 May 1961, and 6 July
1961, the plaintiff company sold on credit to the defendants goods to the value of about 95. On 6 July 1961, the bank, as
debenture-holders, appointed a receiver and manager of the property comprised in the debenture. After 6 July 1961, the plaintiff
company, with the approval of the receiver and manager, sold further goods on credit to the defendants to the value of 1,251 6s
1d. The goods so supplied formed part of the assets of the plaintiff company at the date when the receiver was appointed, so that
the floating charge had attached to them. Between the beginning of November, 1960, and the end of January, 1961, the plaintiff
company had bought goods on credit from a third company, a subsidiary of the same parent company as the defendants, in respect
of which 852 18s 4d remained owing. On 6 October 1961, the benefit of the debt of 852 18s 4d was assigned by the third
company to the defendants, and due notice of the assignment was given to the plaintiff company. The plaintiff company sued the
defendants for 1,346 6s 1d (viz the total of 95 and 1,251 6s 1d), being the price of goods sold and delivered by the plaintiff
company between 24 May and 6 July 1961, and after 6 July 1961. The defendants sought to set-off the debt of 852 18s 4d,
admitting the balance, 493 7s 9d, to be due.
Held (i) The charge created by the debenture, which became a fixed charge on the appointment of the receiver and manager on
6 July 1961, attached to the debt of 95 owing by the defendants in respect of goods sold and delivered before 6 July 1961, and
accordingly that debt was not due between the same parties in the same right at the date (6 October 1961) of the 199 assignment
(since the debenture-holders had become entitled to an interest in the debt); therefore, there could be no partial set-off of the 852
18s 4d as against the 95 (see p 202, letter f, post).
(ii) But the fixed charge created by virtue of the debenture on the appointment of the receiver and manager was not
transferred from goods of the plaintiff company, sold on credit with his approval, to the indebtedness for the purchase price;
accordingly the defendants, as assignees of the plaintiff companys debt for 852 18s 4d, were entitled to a set-off of that sum
against their indebtedness for the price of goods supplied to them by the plaintiff company on and after 6 July 1961, and,
therefore, the plaintiff company was entitled to recover only the 493 7s 9d, which sum included the 95 referred to at (i) ante
(see p 203, letters e and f, post).

Notes
As to the effect of a floating charge becoming fixed, see 6 Halsburys Laws (3rd Edn) 476, para 920; and for cases on the subject,
see 10 Digest (Repl) 781783, 50705088.
As to set-off being confined to claims between same persons in same right, see 34 Halsburys Laws (3rd Edn) 399, para 679;
and for cases on the subject, see 40 Digest (Repl) 407, 408, 1321, 409, 38 et seq, 411, 412, 6872.

Case referred to in judgment


Biggerstaff v Rowatts Wharf Ltd [1896] 2 Ch 93, 65 LJCh 356, 74 LT 473, 9 Digest (Repl) 559, 3702.

Action
The plaintiff company, NW Robbie & Co Ltd sued the defendants, Witney Warehouse Co Ltd for 1,346 6s 1d, the price of goods
sold and delivered. By its statement of claim, which was indorsed on the writ in the action dated 4 January 1962, the plaintiff
company claimed the total sum of 1,346 6s 1d and gave particulars showing sales on sixteen dates, the last date being 12
September 1961. The first four items were24 May 29 7s 7d; 28 June 24 2s 4d; 4 July 42 7s 3d; and 17 July 44 6s 8d.
By their defence the defendants admitted 493 7s 9d to be still owing to the plaintiff company and claimed set-off of the
balance, amounting to 852 18s 4d, for the price of goods sold and delivered by English Spinners Ltd to the plaintiff company,
which debt was assigned on 6 October 1961, to the defendants. Particulars of the debt were given, consisting of four items,
totalling 1,602 18s 4d, credit being given for 750 paid. The four items were11 November 1960, 538 7s 6d; 16 December
1960, 248 5s 11d; 13 January 1961, 291 3s 6d and 31 January 525 1s 5d.
By its reply the plaintiff company pleaded that it was subject to the law of Ireland, and, among other matters, that the
defendants were not entitled to the set-off that they claimed, that the Bank of Ireland were holders of a debenture dated 26
January 1960, securing all moneys due to the bank from the plaintiff company by a floating charge on the assets of the plaintiff
company and that a receiver and manager of the undertaking and assets of the plaintiff company had been appointed on 6 July
1961, by the bank as debenture-holder.
The facts were agreed in letters exchanged between the parties solicitors, and it was also agreed (i) that the law of Ireland as
to debentures, receivers and their powers is the same as the law of England, and (ii) that the law of Ireland as to assignment of
debts is the same as English law as it was before 1 January 1926.
The cases noted belowa were cited in argument in addition to the case referred to in the judgment.
________________________________________
a Parsons v Sovereign Bank of Canada [1913] AC 160, Bennett v White, [1910] 2 KB 643

J H Bamber for the plaintiff company.


H C Easton for the defendants.

Cur adv vult

4 February 1963. The following judgment was delivered.

WIDGERY J delivered the following judgment. In this action the plaintiff, a company incorporated in Ireland, sues the
defendants for the 200 sum of 1,346 6s 1d, the price of goods sold and delivered to the defendants. The defendants admit
liability in the sum of 493 7s 9d, but seek to meet the balance of the claim, namely 852 18s 4d, by pleading a set-off of equal
amount. The only issue for me to decide is the validity of this set-off, but this issue has not proved to be one entirely suitable for
trial in the Queens Bench Division on circuit, as the relevant principles are more familiar in the Chancery Division, and the
opportunity for research in the authorities is limited. No oral evidence has been given at the trial, both counsel having been
content with the following statement of facts, which I am told is agreed.
On 26 January 1960, the plaintiff company issued a debenture to the Bank of Ireland to secure the plaintiff companys
overdraft on its current account with the bank. So far as the law of Ireland is material to this case, it is the same as the law in
force in England prior to 1 January 1926. The debenture appears to be in common form, and provides in para 3:

The company hereby as beneficial owner charges with such payments its undertaking and all its property and assets
present and future including its uncalled capital for the time being and goodwill, and also as a specific charge, the
hereditaments and premises specified in the schedule hereto.

The conditions to which the debenture was expressed to be subject include the following:

(1) This debenture is one for securing the payment of all moneys for the time being due by the company to the bank.
The debenture is to rank as a first charge on the property within mentioned and such charge is to be as regards the
companys lands and premises for the time being and all its uncalled capital for the time being a specific charge and as
regards all other property and assets of the company a floating security but so that the company is not to be at liberty to
create any mortgage or charge on its property for the time being in priority to or pari passu with the said debenture.

Condition (10) reads:

At any time after the principal moneys hereby secured become payable, the registered holder of this debenture may
appoint by writing any person to be a receiver and manager of the property charged by this debenture and may from time to
time remove any such receiver and manager, and any receiver and manager so appointed shall have power:
1. To take possession of, collect and get in the property charged by the debenture, and for that purpose to take any
proceedings in the name of the company or otherwise as may seem expedient.
2. To carry on or concur in carrying on the business of the company.
3. To sell or concur in selling, let or concur in letting any of the property charged by this debenture, and to carry any
such sale into effect by deed in the name and on behalf of the company, or otherwise, to convey the same to the purchaser.
4. To make any arrangement or compromise which he or they shall think expedient in the interest of the debenture-
holders.
And all moneys received by such receiver and manager shall, after providing for the matters specified in s. 24(8)(i), (ii)
and (iii) of the Conveyancing Act, 1881, and for the purposes aforesaid, be applied in or towards satisfaction of the said
debenture, and the foregoing provisions in the condition shall take effect as and by way of variation and extension of the
provisions of s. 19 to s. 24, inclusive, of the said Act, as amended by the Conveyancing Act, 1911, which provisions so
varied and extended shall be regarded as incorporated herein. The receiver and manager so appointed shall be the agent of
the company and the company shall be solely responsible for such receiver and managers acts or defaults.

Following the issue of this debenture the plaintiff company continued to trade, and on and after 24 May 1961, the plaintiff
company sold on credit to the 201 defendants the goods which are the subject of this claim. On 6 July 1961, the Bank of Ireland
appointed a receiver and manager of the property comprised in the debenture to which I have referred.
Notwithstanding the appointment of this receiver the plaintiff company continued to supply goods on credit to the
defendants, and I am told that I may assume that this was done with the approval of the receiver, and that the goods so supplied
formed part of the assets of the plaintiff company at the date when the receiver was appointed. The value of the goods so
supplied and dates of supply are correctly set out in the statement of claim.
Between November, 1960, and January, 1961, the plaintiff company bought goods on credit from English Spinners Ltd to
the total value of 1,602 18s 4d. Payment on account reduced the amount of this debt to 852 18s 4d. English Spinners Ltd are a
company associated with the defendants in the sense that both are subsidiaries of the same parent company, and on 6 October
1961, the benefit of this debt, due from the plaintiff company to English Spinners Ltd was assigned by deed to the defendants.
Due notice of this assignment having been given to the plaintiff company, the assignment amounted to a legal assignment of the
debt to the defendants, and this is the debt pleaded as a set-off in this action.
Certain propositions of law are not in issue and can be stated as follows. First, the defendants are entitled to this set-off,
notwithstanding that their title to the debt set-off was derived from an assignment made by the original creditors, if immediately
following the assignment the debt claimed and the debt set-off existed between the same parties in the same right. Secondly, the
existence of a floating charge on the plaintiff companys assets would not in itself amount to an assignment to the debenture-
holders of the debt claimed so as to prevent the claim and set-off from existing between the same parties in the same right; for
that proposition see Biggerstaff v Rowatts Wharf Ltd. Thirdly, the appointment of the receiver and manager caused the floating
charge created by the debenture to crystallise or become fixed on the assets of the plaintiff company at the date of the
appointment. The debt represented by the first three items b in the statement of claim and amounting to some 95 existed at the
date when the receiver was appointed. It was subject to this fixed charge, and thus became assigned to the debenture-holders.
The debt subsequently assigned by English Spinners Ltd to the defendants cannot be set-off against this part of the plaintiff
companys claim, because this part of the debt claimed had become vested in the debentureholders before the right of set-off was
acquired against the plaintiff company. Fourthly, the floating charge created by the debenture became fixed on the assets of the
plaintiff company existing at the date of the appointment of the receiver, but does not of itself attach to assets of the plaintiff
company subsequently acquired.
________________________________________
b See p 200, letter e, ante, where the first four items are stated

On these agreed facts, and accepting those principles, the plaintiff company resists the defendants right to set-off the debt of
852 18s 4d against the plaintiff companys claim. Counsels contention on behalf of the plaintiff company is that where the
receiver and manager sells, on behalf of the company, assets on which a former floating charge has become fixed, the charge
attaches to the proceeds of sale of those assets, as it formerly attached to the assets themselves. He argues that if the plaintiff
company through the agency of the receiver had sold the goods to the defendants for cash, the charge which formerly attached to
those goods would automatically attach to the proceeds of sale in the receivers hands; and he submits that if the goods are sold
on credit, the charge attaches to the debt due to the plaintiff company for the price of the goods. Accordingly, he contends that
when the goods subsequent to item 3 in the statement of claim were sold to the defendants, the resultant debt was automatically
charged or assigned to the debenture-holders, and that this prohibits the defendants from 202 relying on the set-off of a debt
subsequently acquired by the defendants. Counsel for the plaintiff company has not referred me to any textbook or authority for
this contention, but says that this result must follow if the receivers power to continue the business of the plaintiff company and
realise the assets of the plaintiff company is to exist as a practical possibility.
Counsel for the defendants disputes the plaintiff companys contention that when the goods subject to a fixed charge are sold
by the receiver, the charge is transferred to the proceeds of sale. He concedes, as I understand his argument, that if the receiver
sells such goods for cash, that cash must be paid by the receiver to the debenture-holders, and will not be available for the
satisfaction of the plaintiff companys unsecured creditors; but he says that this result flows from the fact that the conditions of
the debenture required the receiver to hold moneys so received on trust for the debenture-holders, and does not depend on any
notional transfer of the original charge to the proceeds of sale. When the sale of the plaintiff companys assets is on credit, he
contends that there are no proceeds of sale to which the trust created by the debenture can attach, and he says there is nothing in
the debenture which subjects the debt due to the company to any such trust, because the debt is not moneys received within
condition 10. He contends that the debt is due to the plaintiff company and not due to the receiver, and cannot be subject to any
trust of which the receiver is trustee. In result, so the argument goes, the debt due to the plaintiff company from the defendants,
as a result of trading subsequent to the appointment of the receiver, is a debt in which the debenture-holders have no present
interest at all. Consequently, the debt claimed and the debt to be set-off exist between the same parties in the same right, and the
set-off claim should be allowed.
Having reflected on these arguments I have decided that counsel for the defendants is right. Where a receiver carries on the
companys business in the name of the company, those with whom he deals should not lightly be deprived of the rights of set-off
to which they would have been entitled if the business were being carried on by the company on its own account.
The principle for which counsel for the plaintiff company contends, namely, that the debenture-holders charge
automatically attaches to debts due to the company as a result of such trade, is not supported by authority or by any principle of
law or of equity which I recognise. Accordingly in my judgment that contention fails, the set-off is a valid one, and the amount
for which judgment should be given is 493 7s 9dc only.
________________________________________
c The sum of 95 (see p 202, letter f, ante) was included in the 493 7s 9d

As the plaintiff company is in Ireland it would seem convenient to have payment out in this country. There will be judgment
for the plaintiff company with costs up to the date of payment in. The defendants will have their costs after the date of payment
in. Accounts for costs will be set-off, and any balance due to the defendants will be deducted from the amount paid into court
before the balance of the amount is paid out to the plaintiff company.

Judgment accordingly.
Solicitors: Boote, Edgar & Co, Manchester (for the plaintiff company); Skelton & Co, Manchester (for the defendants).

K Buckley Edwards Esq Barrister.


203
[1963] 2 All ER 204

National Provincial Bank Ltd v Hastings Car Mart Ltd and others
LAND; Property Rights: EQUITY

CHANCERY DIVISION
CROSS J
4, 5, 27 MARCH 1963

Husband and Wife Deserted wifes right to remain in occupation of matrimonial home Registered land Overriding interest
Deserted wife, and children, remaining in occupation of husbands dwelling-house Transfer of house to private company in
which husband principal shareholder Mortgage by company Mortgagees unaware of husbands desertion Right of
mortgagees to vacant possession to enforce security Whether wifes equity was right in rem Whether an overriding interest
Land Registration Act, 1925 (15 & 16 Geo 5 c 21), s 70(1) (g).

A deserted wifes equity or claim to remain in occupation of the matrimonial home is simply a right to appeal to the court for
protection against unconscionable conduct on the part of the husband or the husbands successor in title, but it is not a right in
rem; the wifes equity is not, therefore, an overriding interest within s 70(1)(g) a of the Land Registration Act, 1925 (see p 208,
letter f, p 209, letter d, post).
________________________________________
a The terms of s 70(1)(g) are set out at p 207, letter h, post

Westminster Bank Ltd v Lee ([1955] 2 All ER 883) considered.


In 1957 the husband deserted the wife. The wife and the four infant children of the marriage continued to reside in the
matrimonial home, of which the husband was registered proprietor with absolute title. The husband went to live with his mother,
some half a mile away. In 1958 the husband mortgaged the property to the bank to secure a loan and further advances, and the
mortgage was duly registered in the charges register. The bank did not know that the husband had deserted the wife. In 1959 a
company was formed to take over the husbands business and properties; in the memorandum of association the address stated as
the husbands was the address where his mother lived. The company became the registered proprietor of the matrimonial home,
and charged it and other property to the bank, the husband giving a guarantee to the bank for the companys indebtedness, and the
former charge on the matrimonial home being discharged. The new charge was duly registered in the charges register. In 1961
the wife obtained a decree of judicial separation and an order for permanent alimony, which was made on the basis that the
husband was providing the wife and children with a rent-free home at the matrimonial home. At this time the bank learnt that the
husband had deserted the wife and had left her in occupation of the matrimonial home. The company having failed to comply
with a notice from the bank calling in its debt, the bank issued a summons for possession of the mortgaged property. At the date
of the hearing there was pending an application by the wife, who was in receipt of national assistance, for an order under s 2 of
the Matrimonial Causes (Property and Maintenance) Act, 1958, setting aside the conveyance of the matrimonial home by the
husband to the company on the ground that it had been made with the intention of defeating her claim against him for financial
relief.

Held The bank was entitled to an order for possession, but as it would not be right for the court to compel the wife to give up
possession before the determination of her application under s 2

Das könnte Ihnen auch gefallen